· Web viewSubject. Topic. Summary. Execution. English 1 . Chapter 1 naming words . Page 8. Write...

85
Rishi Aurobindo Memorial Academy 134 P. K. Guha Road DumDum Kol-28 Study Material of 16/4/2020 For Class Nursery to XII Class Nursery Subject Topic Execution English Practice book Pg 15 Reading book Pg 7, 8 Write in book Read and repeat Math Practice book Pg 41 (15) Reading book Pg 52 Write in book Read and repeat Hindi Letter ‘ऋ’ Write in copy Bengali ऋऋऋऋऋऋऋ ऋऋऋऋऋ Pg 10, 11 ऋऋऋऋऋ ऋऋऋ ऋऋऋऋ ऋऋऋ ऋऋऋ – read and repeat Class KG I Subject Topic Execution English Reading book Pg 16, 17 Write in book Math Counting Numbers 1-25 (Write in copy) Class KG II Subject Topic Execution English (Term I) Pg 27, 28 Pg 65, 66 Write in cursive letters in book Read & learn Math Table of 3 Write in copy Bengali ऋऋऋऋऋऋऋ ऋऋऋऋऋ Pg 16 Write in book Hindi ऋऋऋ ऋऋऋ ऋऋऋऋऋ ऋऋऋऋ read & repeat Class I Subject Topic Summary Execution Englis h 1 Chapter 1 naming words Page 8 Write the names of these pictures:- Person:- 1. father 2.Firefighter 3.doctor 4.teacher Animal:- 1.Horse 2.Penguin 3. Mouse 4.Snake Thing:- 1. balloon 2. train 3.cake 4. hat Place:- 1. farm 2. zoo 3.park 4.School Englis h 2 Orchid Chapter 3 the caring Book page 25 answer the following questions 1.What does Nancy want? Ans-Nancy wants the moon.

Transcript of  · Web viewSubject. Topic. Summary. Execution. English 1 . Chapter 1 naming words . Page 8. Write...

Page 1:  · Web viewSubject. Topic. Summary. Execution. English 1 . Chapter 1 naming words . Page 8. Write the names of these pictures:- Person:-1. father. 2.Firefighter 3.doctor 4 ...

Rishi Aurobindo Memorial Academy134 P K Guha Road DumDum Kol-28

Study Material of 1642020For Class Nursery to XII

Class NurserySubject Topic Execution

English Practice book Pg 15 Reading book Pg 7 8

Write in book Read and repeat

Math Practice book Pg 41 (15) Reading book Pg 52

Write in book Read and repeat

Hindi Letter lsquoऋrsquo Write in copyBengali সাহিতয কহি Pg 10 11 নীচের ছহি দেচে নাম চো ndash read and repeat

Class KG ISubject Topic ExecutionEnglish Reading book Pg 16 17 Write in bookMath Counting Numbers 1-25 (Write in copy)

Class KG IISubject Topic Execution

English (Term I) Pg 27 28 Pg 65 66

Write in cursive letters in book Read amp learn

Math Table of 3 Write in copyBengali সাহিতয কসম Pg 16 Write in bookHindi बाल गीत मममी पापा read amp repeat

Class ISubject Topic Summary Execution

English 1 Chapter 1 naming words

Page 8 Write the names of these pictures- Person-1 father2Firefighter 3doctor 4teacher Animal-1Horse 2Penguin3 Mouse 4Snake Thing-1 balloon 2 train 3cake 4 hat Place-1 farm 2 zoo 3park 4School

English 2 Orchid

Chapter 3 the caring granny

Book page 25 answer the following questions

1What does Nancy wantAns-Nancy wants the moon2What does Nancy see in the bowlAns-Nancy sees the moon in the bowl3What is Nancy doing in another roomAns- Nancy is looking at the sky4What does Sahil love to eatAns-Sahil loves to eat hot chapatis with vegetables

Maths Chapter 4 addition

Page 200 Find the sum (vertical addition) 6 4 5

and subtraction

+ 1 + 3 +1 77 6

8 1+ 0 +7 8 8

EVS Chapter 5 festivals and celebrations

Book Page 110 time to answer

Choose the correct option1The President of India hoists the Indian flag on RepublicDay2Gandhi Jayanti is the birthday of Gandhiji3Republic Day is a national festival

Computer CHAPTER-2PARTS OF COMPUTER

DONE IN THE PREVIOUS CLASS PAGE NO 18ATICK THE CORRECT OPTION 1HEADPHONE2KEYBOARD3PEN DRIVE4MOUSE5CPU

Hindi वयाकरण फलो क नाम 1चपा 2कमल 3बला 4पलाश 5कनर 6गलाब 7मोगरा

Bengali ইndash পাঠndashাকয রনানমনা দেওয়া চোহিনচে াকয গঠন অভযাস কর

মাndashআহিম মা rsquo দেক ভাচোাহিসনীndashনীদেত মাছ থাচেকগাছndashতা গাছ উাউndashাউ একতারা াহিচেয় গান কচেরাndashরাচেতর আকাচে( া ওচেঠ১ দেথচেক ২০ সংযা ও ানান অভযাস কচেরা

Class IISubject Topic Summary Execution

Eng literature

Ch 4 Bob and the Bathtub

Refer to pg 47 Do in the book

II Choose the correct word for each blank-Answers

1 years2 gives3 clothes4 with5 in

Eng language

Ch 6 pronouns Refer to pg 32 and 33Ex A Do in the book

EX A ] REWRITE THESE AFTER REPLACING THE HIGHLIGHTED WORDS WITH PRONOUNS- he she it they or weAnswers

1 She dances very well2 He goes to office3 They have gone to watch a film4 We went trekking5 They are looked after very well at

the zoo6 It talks a lot7 They gave some flowers to the

teacher

8 She must finish her homework9 It rings at 630 am everyday10 They sing beautifully

EVS Ch 5 Eating healthy

Refer to pg 118Do in the book

C Match the followingAnswers

1 Drinking water ndash clean and filtered2 Eggs ndash food from animals3 Carrot and radish ndash vegetables4 A healthy mix of food items ndash

balanced diet5 Apple an orange ndashfruits

Bengali ইndashাংাসাহিতযপহিরয়

পাঠndashহিসংওইচেররগলপঅন(ীনীরপরচোততর

৯ াকযরনা -পাাndashআহিমপাাভাচোাহিসগাndashপাাচেরগায়হিসংচেরাসাহিছহিসংndashহিসংদেকপশরাায়ইরndashইরমানচে4রঅহিনষটকচেরহি(কাহিরndashহি(কাহিরচেনহি(কারকচের

Hindi वयाकरण Chapter 4 सजञा

सजञा -इस दनिनया म सभी का कोई ना कोई नाम होता ह यही नाम सजञा कहलाता ह नाम कई परकार क होत हजस- पश-पकषी जगह दि-नो क नाम वयकतियो क नाम आदि-

Read the lesson

Mathematics CHAPTER-3NUMBERS UPTO 1000

PLACE VALUE PAGE NO 1882WRITE THE PLACE VALUE FOR GIVEN 3-DIGIT NUMBERSb)368Place value of 8 is ones8Place value of 6 is tens60Place value of 3 is hundred300(Practice the rest exercise)

Class IIISubject Topic Summary Execution

ENGLISH II CHAPTER-2 THE WOODEN BOWL

Refer to pg 27COMPOSITION[ DO IN THE BLANK SPACE GIVEN IN YOUR BOOK]

COMPOSITION ABOUT A THING YOU MADE READ AND THE ANSWER EACH QUESTION BRIEFLY AND THEN DRAW AND COLOUR THE THING YOU HAD MADE FOR EXAMPLE IT CAN BE A CARD A EASY CRAFT ETC

SST INDIA-THE FESTIVALS OF INDIA

DONE Encircle the odd one out and give reason for your option1RajpathRajpath is not related to the India Independence

2Santa ClausSanta Claus is related to the Christmas

3 LangarLangar is related to the Gurupurab festival

4August 15August 15 is a National festival

BENGALI ই ndash াংা সাহিতয পহিরয়

পাঠ ndash ১একতাই অন(ীনীর পরচোততর

৫ হিপরীত - চো times দোকা স times ঃ হি( times অহি( াধা times ছাা আস times নক সহিতয times হিমচেথয৭ াকয রনা -মযাহিক ndash আমার কাকা ভাচো মযাহিক দোয় া(ঝা ndash রাচেতর অনধকাচের া(ঝা দেচে আমার ভয় াচেগ অহিনষট ndash অচেনযর অহিনষট হিনতা করচেত দেনই াহি= ndash কাগচের াহি= চে পচেরাচেনা স নহিথ পাওয়া দেগচো

HINDI वयाकरण Chapter 4 सजञा

सजञा- इस दनिनया म सभी का कोई ना कोई नाम होता ह यही नाम सजञा कहलाता ह नाम कई तरह क होत ह जस -वयकतियो क नाम पराणिणयो क नाम सथानो क नाम भावो क नाम आदि-

1सजञा निकस कहत ह उ-ाहरण -ीजिजएउततर- निकसी भी वयकति वसत सथान या भाव क नाम को सजञा कहत हजस- राम सकल घडी गरमी आदि-

MATHEMATICS CHAPTER-4 SUBTRACTION

Subtraction (with decomposing)Subtraction of 3-digit numbers

Exercise 18Example 1 (p-48)

Fill in the place holders3 hundreds 2 tens 5 ones ndash 2 hundreds 5 tens 7 ones= 3 hundreds 1 tens ___ ones - 2 hundreds 5 tens 7 ones= 2 hundreds ___ tens 15 ones - 2 hundreds 5 tens 7 ones= 6 tens ___ ones

Solution3 hundreds 2 tens 5 ones ndash 2 hundreds 5 tens 7 ones= 3 hundreds 1tens 15 ones - 2 hundreds 5 tens 7 ones= 2 hundreds 11 tens 15 ones - 2 hundreds 5 tens 7 ones= 6 tens 8 ones

Example 4 (p-51)Subtract 389 from 600

Solution H T O 6 0 0 - 3 8 9

2 1 1We cannot subtract 9 ones from 0 ones so we borrows 1 tens Here there is no ten so first we borrow 1 hundreds or 10 tens having behind 5 hundreds From 10 tens we borrow 1 tens or 10 ones leaving behind 9 tensNow 10 ones ndash 9 ones = 1 onesAnd 9 tens ndash 8 tens = 1 tens5 hundreds ndash 3 hundreds = 2 hundreds 600 ndash 389 = 211

Exercise2 Fill in the place holders5 hundreds 2 tens 8 ones ndash 3 hundreds 8

tens 3 ones= 4 hundreds ___ tens 8 ones - 3 hundreds 8 tens 3 ones= 1 hundreds ___ tens ___ ones

Solution5 hundreds 2 tens 8 ones ndash 3 hundreds 8 tens 3 ones= 4 hundreds 12 tens 8 ones - 3 hundreds 8 tens 3 ones= 1 hundreds 4 tens 5 ones

Class IVSubject Topic Summary Execution

English literature

Hercules andDeianira (tales from Greece and Rome)

Hercules spent the greater part of his life in helping the poor and weak people But Juno still wanted to harm him So she sent him into all sorts of dangers But he was brave and strong and he overcame all the dangers Juno made him a slave to the king of Argos Hercules worked hard for the king The king took pity on him and told him that he would set him free if Hercules perform twelve difficult tasks Hercules accepted the challenge

1How did Hercules spent the greater part of his lifeAns Hercules spentthe greater part of his life in helping the poor and weak people

2How did Juno tried to harm HerculesAns Juno tried to harm Hercules by sending him into all sorts of dangers

3Under which king Hercules was working as a slaveAns King of Argos

4What was the challenge that king of Argosgive to HerculesAns King of Argos told Hercules that he would set him free if he perform twelve difficult tasks which Hercules accepted

Social studies Major landforms on earth

Answer the following questions5What do oceans provide usAns Oceans provide us a variety of sea food

6What is alluviumAns The rivers carry sand soil and small pieces of rocks This fine soil or silt known as alluvium is very good for growing crops

COMPUTER160420

CHAPTER 3 EDITING IN MS WORD

DRAG AND DROPTHE DRAG AND DROP METHOD OF MOVING THE TEXT ALLOWS US TO MOVE THE SELECTED TEXT USING MOUSE FROM ONE LOCATION TO ANOTHER WITHIN A DOCUMENT

Q7) HOW TO WE DRAG AND DROP TEXT IN MS WORDAns) WE CAN DRAG AND DROP TEXT IN MS WORD FOLLOWING STEPS ARE---

SELECT THE TEXT PLACE THE MOUSE POINTER

ANYWHERE ON THE SELECTED TEXT

CLICK AND HOLD THE LEFT MOUSE BUTTON UNTIL THE INSERTION POINT CHANGES INTO A WHITE ARROW

WHEN WE REACH THE PLACE IN THE DOCUMENT WHERE WE WANT TO MOVE THE TEXT RELEASE THE MOUSE BUTTON

Science Adaptations in Summary Execution

Animals Animals are divided into different groups according to their habitats Some animals live on land some animals live both on land and in water some animals adapted themselves for food and some for protection They develop special features that help them to survive in their habitats Animals that live on land are called Terrestrial animals such as lions tigers Yaks polar bears Polar Bears live cold polar regions Yaks live in mountains These animals have thick fur on their body and a thick layer of fat under their skin which keeps them warm

Solved exercise question and answersA Write the correct answer1Terrestrial animals live here - on land 2This animal lives in polar regions ndash polar bear 3 These help a fish to breathe in water - gills4 This helps carnivorous birds to tear flesh - a hooked beak

MATHEMATICS

Ch 6Multiplication

Exercise 17 Example 2

Multiply 224835 by 4 Solution

2 2 4 8 3 5 times 48 9 9 3 4 0

Exercise

18 3 1 1 4 6 7 19 2 1 2 7 8 5 times 3 times 4 9 3 4 4 0 1 8 5 1 1 4 0

20 2 0 6 1 3 times 9 1 8 5 5 1 7

Class VSubject Topic Summary Execution

Science Chapter 2 - The Skeletal System

Posture

The way in which we hold our body while sitting standing walking and lying down is known as posture

Importance of correct posture and exercise

1 Correct posture and exercise makes our bones and muscles strong 2 It provides our body more energy to do works

3 It keeps our body stressless 4 It also prevent back ache and muscle pain

Some important yoga asanas are-

1 Veerabhadrasana ( Warrior pose )

2 Dhanurasana ( Bow pose )

3 Trikonasana ( Triangle pose )

D Answer these questions

5How do our muscle work

Ans ndash Our muscles work by contracting and relaxing

6Why should we maintain a correct posture

Ans ndash We should maintain a correct posture because ndash

i Correct posture and exercise makes our bones and muscles strong

ii It provides our body more energy to do works

iii It keeps our

body stressles

s iv It also prevent

back ache and muscle pain

Social studies Conquering distances

Put a tick on the correct option1 b Ship 2b water transport 3 b trains 4 c 13 5 b Wright brothersTrue or false1 False 2 True 3 True 4 True

MATHEMATICS

Ch 3Addition and Subtraction

Exercise 126 A school needs Rs 4987653 for its building It has only Rs 3592468 in its accounts Estimate the money by rounding off to the nearest lakh it has still to raise

Solution Estimated amount need Rs 5000000 The school has only Rs 3600000 Still to raise Rs 1400000

MATHEMATICS

Ch 4

Mul

tiplic

ation

and

Div

ision

We have learnt multiplication tables up to 15 Let us extend the tables up to 20X 11 12 13 14 15 16 17 18 19 201 11 12 13 14 15 16 17 18 19 202 22 24 26 28 30 32 34 36 38 403 33 36 39 42 45 48 51 54 57 604 44 48 52 56 60 64 68 72 76 805 55 60 65 70 75 80 85 90 95 1006 66 72 78 84 90 96 102 108 114 1207 77 84 91 98 105 112 119 126 133 1408 88 96 104 112 120 128 136 144 152 1609 99 108 117 126 135 144 153 162 171 180

10 110 120 130 140 150 160 170 180 190 200Properties of Multiplication1 The product of two numbers does not change when the order of numbers is changed eg 503times23 = 23times503 [This property is called Commutative Property of multiplication]

2 The product of three numbers does not change when the grouping of numbers is change eg (15times18iquesttimes10=15 times(18times 10) = (15times10iquesttimes18 [This property is called Associative Property of multiplication]

3 The product of a number and 1 is the number itself eg 1513 times 1 = 1513 [This property is called Identity Property of multiplication and the integer 1 is called Identity Element of multiplication]

4 The product of a number and 0 is 0 eg 718205times 0 = 0 times 718205 = 05 The product of a number by the sum of two numbers is equal to the sum of the

products of that number by the two numbers separately eg123 times (105+ 48) = 123 times 105 + 123 times 48 [ This property is called Distributive Property of multiplication over

addition]English language

Transitive and intransitive verb

Pick out the verbs from the following sentences and say whether they are transitive or intransitive verb6 Was fullndash verb Intransitive verb7 Have been decorated- verb Transitive verb8 Happy cheerful ndash verb Intransitive verb9 Shall come back ndash verb Transitive verb

English 2 The fall of Lanka

This is the story of the fight between Rama and Ravana as told by Valmiki in lsquoThe Ramayana The monkeys worked all day and all night and at last built a bridge so that Rama and

Write the synonyms of

1 Stationed-

his army could cross to Lanka and rescue Sita assign2 Invade- enter

a country or a region so as to subjugate or occupy it

3 Prowess- bravery in battle

4 Haughty- arrogantly

5 Puny- small and weak

6 Spy- secret agent

7 Dreadful- causing or involving great suffering

8 Violate- disobey

9 Ghastly- causing great horror or fear

10 Deception ndash misleading

Class VISubject Topic Summary Execution

HISTORY AND CIVICS

CHAPTER 3

MAHAVIRA AND BUDDHA ndash GREAT PREACHERS

BUDDHA

Impact of Buddhism on Indian Cultures

Impact on Religion

Buddharsquos practical and simple doctrines made their impact on HinduismThe principle of ahimsaIt brought about a great change in the performance of costly yanjnas and sacrifices which previously involved immense loss of life The Mahayana Buddhists adopted the practice of worshipping Buddha and bodhisattvas making idols and erecting temples in their honour

Impact on Literature

After the death of Buddha his teachings were compiled and called TripitakasThe Jatakas contain tales dealing with the previous births of Buddha

Impact on Education

The Buddhist monasteries became great centres of learning These centres of learning developed into famous universities- Nalanda Taxila Vikramshila etc

Impact on Art and Architecture

The gateways and railing of the Sanchi Stupa were covered with sculptured figuresCave- temples were also constructed which were decorated with beautiful frescoes

1) How many parts of Tripitakas are thereAns -There are three parts of Tripitakas- Sutta Pitaka Vinaya PitakaAbhidhamma Pitaka

2)What are the subjects taught in these monasteries Ans ndash Buddhist scriptures logic Philosophy medicine astronomy etc

3) Which art was developed under Buddhist patronage Ans ndash Gandhara art

The Gandhara art was developed under Buddhist patronage

BIOLOGY The Leaf Photosynthesis The process by which green plants make their own food from carbon dioxide and water in the presence of sunlight and chlorophyll is called photosynthesis

All green plants need the following to make their food ndash

water carbon dioxide chlorophyll and energy in the form of sunlight

Carbon dioxide + water ------------- Glucose + oxygen

The end product of photosynthesis is glucose

Fill in the blanks

1 Plants make their food by the process of photosynthesis

2 The inner wall of the guard cell is thicker than the outer wall

3 The extra glucose is converted into starch and sucrose

4 The leaf is boiled in alcohol to remove chlorophyll

5 The rate of transpiration is more on the hot day then a cold day

6 Photosynthesis helps to observe water and minerals from the soil

English 1 Pronouns Kinds of pronouns 1 Personal pronouns2 Possessive pronouns3 Reflexive pronouns4 Interrogative pronouns5 Relative pronouns6 Demonstrative pronouns7 Indefinite pronouns

Personal pronouns they refer to first second and third person in sentences First person- the speakerSecond person-the listenerThird person-the objectperson being spoken aboutPersonal pronouns should have the same gender and number as the nouns they refer to

Possessive pronouns these are used to indicate the relationship between the objects and people These pronouns include mine ours yours his hersand theirs

ExerciseBFill in the blanks with suitable pronounsThere was much excitement among the childrenTheywere eagerly looking forward to the annual picniclsquoAre they going to Lodhi Gardens toorsquo wondered AneeshlsquoNo they are going to Buddh Jayanti Park with Mrs Jain said Mrs ChopraThe children looked disappointedlsquoWonrsquot you be taking us Marsquoam rsquo they askedlsquoSorry children I have to go to Mumbai for a week to look after my sick mother But you will have fun with Mrs Jain she is full of laughter and you will love being with her the whole daylsquoIt will not be the same they grumbled

English 2 The great train journey- Ruskin Bond

The great journey by Ruskin Bond is a story about Suraj who loved trains and wanted to go to places One day while wandering along the railway tracks he enters into a carriage compartment The train suddenly starts moving with him in the compartment and after a journey returns back to the same place from where it had begun The story is about his experience during that journey

State true or false1 When the train had passed leaving behind the

hot empty track Suraj was lonely2 It was winter holidays

and Suraj did not know what to do with himself

3 He plunged his hands into the straw and pulled out an apple

4 A dirty bearded face was looking out at him from behind a pile of crates

5 Suraj wanted to go to Japan

Hindi 2nd lang

गललबाजलडका खालीसथानोकोभरो-6 गो-ामसनिनकलकरहमगराजमआगए7 माबोधराजकोराकषससमझतीथी8 चीलरोशन-ानमसअ-रआकरतहसीलपरबठगई9 तीनचारतीनकऔररईकगोलउडलनिकनघोसलानहीनिगरा10 वहसवयतोघोसलातोडनककतिलएगललउठालायाथा

11 -ीवारकसाथलगतगोहपजोकसहार-ीवारपकडलतीह12 बोधराजअभीभीटकटकीबाधचीलकीओर-खरहाथा13 बोधराजअपनीजबमबहतसाचगगाभरकरलायाथा14 मरनिपताजीकीतरककीहईऔरहमलोगएकबडघरमजाकररहनलग15 बागमजातातोफलपरबठीनिततलीको-खनिततलीकोपकडकरउगकतिलयोकबीचमसल-ता

BENGALI(2ND LANGUAGE)

সহিনধসবরপওসবরসহিনধ

সবরসহিনধরহিনয়ম- ৯ই-কারহিকংাঈ-কাচেররপচেরইাঈহিভননঅনযসবররণCথাকচেইাঈসথাদেনয-ফায়এংওইয- ফাপCচেরণCযকতয়

১০উ-কারহিকংাঊ-কাচেররপচেরইাঈহিভননঅনযসবররণCথাকচেউাঊসথাদেন-ফায়এংওই- ফাপCচেরণCযকতয়

১১ঋ- কাচেররপচেরঋহিভননঅনযসবররণCথাকচেঋসথাচেনর -ফায়এংওইর পCচেরণCযকতয়

১২সবররণCপচেরথাকচেপCতMএ-সথাচেনঅয় ঐ- সথাচেনআয় ও- সথাচেনঅএংঔ- সথাচেনঅায়

৯ই+ অ= য- ফাআহি+ অনত= আযনত অহিধ+ অয়ন= অধযয়নই+ আ=য- ফা+ াইহিত+ আহি= ইতযাহি পরহিত+ আতC ন= পরতযাতC নই+ উ=য- ফা+ উঅহিত+ উহিকত= অতযহিকত হি+ উৎপহিতত= যৎপহিততই+ ঊ= য- ফা+ ঊ ই+ এ= য- ফা+ এঈ+ অ= য- ফা পরহিত+ ঊ4= পরতয4 পরহিত+ এক= পরচেতযকঈ+ অ আ= য- ফা+ অ আনী+ অমব= নযমব মসী+ আধার= মসযাধার

১০উ+ অ= অন+ অয়= অনবয় পশ+ অধম= পশবধমউ+ আ= াস+ আগত= সবাগত পশ+ আহি= পশবাহিউ+ ঈ= হিঅন+ ইত= অহিনবতউ+ এ= দেঅন+ এ4রণ= অচেনব4রণউ+ ঈ= ীসাধ+ ঈ= সবাধবী তন+ ঈ= তনবী

১১ঋ+ অ= র মাত+ অনমহিত= মাতরনমহিতঋ+ আ= রা হিপত+ আয়= হিপতরায়ঋ+ ই= হির মাত+ ইচছা= মাতচছাঋ+ ঈ= রী ধাত+ ঈ= ধাতরীঋ+ উ= র ভরাত+ উপচে(= ভরাতরপচে(

১২এ+ অ= অয় দেন+ অন= নয়নঐ+ অ= আয় গৈগ+ অক= গায়কও+ অ= অ দেপা+ অন= পনও+ ই= অ দেপা+ ইতর= পহিতরও+ এ= অ দেগা+ এ4রণা= গচে4রণাঔ+ অ= অা দেপৌ+ অক= পাকঔ+ ই= অা দেনৌ+ ইক= নাহিকঔ+ উ= অা দেভৌ+ উক= ভাক

MATHS Topic NumbersChapter Natural numbers and whole numbers

Study item properties of whole numbers for multiplication

1 Closure property If x and y are two whole numbers then xtimesy is also a whole numberExample If x = 9 and y =3 then xtimesy = 9times3 = 27 which is a whole number

2 Commutative property If x and y are two whole numbers then xtimesy = ytimesxExample If x = 5 and y = 2 then xtimesy = 5times2 = 10y times x = 2times5 = 10Therefore 5times2 = 2times5

3 Associative property If x y and z are three whole numbers then x times(ytimesz) = (xtimesy) times zExample If x =3 y = 5 and z = 7 then 3 times (5times7) = 3 times (35) = 105And (3times5) times7 = (15) times 7 = 105Therefore x times (ytimesz) = (xtimesy) timesz

4 Distributive property If x y and z are three whole numbers then xtimes (y + z) = x times y + x times z

Therefore the multiplication of whole numbers is distributive over their additionExample If x = 5 y = 3 and z= 2Therefore x times (y + z) = 5 times (3 + 2) = 5times5 =25And x times y + xtimes z = 5times3 +5times2 =15 +10 = 25Again x times (y ndash z ) = x times y ndash x timesz Therefore 5 times ( 3 - 2) = 5 times1 = 5 and 5times3 ndash 5 times2 = 15 ndash 10 = 5Therefore the multiplication of whole numbers is also distributive over their subtraction if y is greater than z

5 Existence of identity If x is a whole number then

X times1 = x 1 times x = xTherefore we can write x times1 = 1 times xTherefore the multiplication of any whole number with 1 is the number itselfTherefore we can say that 1 is multiplicative identity or identity element for multiplicationExample 5 times1 = 5 1 times 5 = 5 Therefore 5 times 1 = 5

6 Multiplicative inverse If x is any whole number ( x is not equal to zero ) then its multiplicative inverse will be 1xSo x times 1x = 1 but 1x is a whole number if x = 1For other values of whole number 1x is not a whole number therefore we can write its multiplicative inverse does not exists

7 Cancellation law of multiplication If x y and z are three non- zero whole numbers then x times y = x times z

Or y = zExample 9 times y = 9 timeszTherefore y = z

Class VIISubject Topic Summary Execution

English 2 Sentences based on meanings

Kinds of sentences

Assertive or declarative to convey information or simply make a statement

Interrogative to ask different types of questions

Imperative to command or instruct someone or make a request

Exclamatory to express strong feelings and emotions

Exercise c1 What a nice compliment that is

That is a nice compliment2 How well- behaved the children

areThe children are very well-behaved

3 What great chefs we areWe are great chefs

4 What a shame it isIt is a shame

5 What a fantastic idea you haveYou have a fantastic idea

Homework 6 -10English

LiteratureThe Listeners III) Answer the following questions-

d) Identify two words used in the poem to give the poem an eerie atmosphereAns- Two words used to give the poem an eerie atmosphere are ldquogreyrdquo and ldquophantomrdquo

e) Who do you think are the inmates of the houseAns- I think the inmates of the house are phantom who once used to dwell in it

f) Why was the poet ldquoperplexed and stillrdquoAns- He was lsquo perplexed and stillrsquo because he was expecting an answer from the inmates of the house But despite of repeated calls there was no response

CHEMISTRY Chapter 2 ndashElement and Compound

Activity Series of MetalsThe activity series is a chart of metals listed in order of declining relative reactivity The top metals are more reactive than the metals on the bottomMetal SymbolReactivity

Lithium Li displaces H2 gas from water steam and acids and forms hydroxides

Potassium K

Strontium Sr

Calcium Ca

Sodium Na

Magnesium Mg displaces H2 gas from steam and acids and forms hydroxides

Aluminum Al

Zinc Zn

Chromium Cr

Iron Fe displaces H2 gas from acids only and forms hydroxides

Cadmium Cd

Cobalt Co

Nickel Ni

Tin Sn

Lead Pb

Hydrogen gas

H2 included for comparison

Antimony Sb combines with O2 to form oxides and cannot displace H2

Arsenic As

Bismuth Bi

Copper Cu

Mercury Hg found free in nature oxides decompose with heating

Silver Ag

Palladium Pd

Platinum Pt

Gold Au

Answer the following Q)Difference Between Metals And Nonmetals

Metals Nonmetals

These are solids at room temperature except mercury

These exist in all three states

These are very hard except sodium

These are soft except diamond

These are malleable and ductile

These are brittle and can be breakdown into pieces

These are shiny These are non-lustrous except iodine

Electropositive in nature Electronegative in nature

Have high densities Have low densities

Math Number System

Chapter Fraction

Study item Some solved sums from exercise 3(B)1) For each pair given below state whether it from like fractions or unlike

fractions (i) 58 and 78

= Like Fraction because denominators same(ii) 815 and 821

= Unlike Fraction because denominators are not same

(iii) 49 and 94 = Unlike Fraction

2) Convert given fractions into fractions with equal denominators(iii) 45 1720 2340 and 1116Solution Given fraction 45 1720 2340 and 1116Therefore the LCM of 5 20 40 and 16 is 80Therefore 45 = 4times165times16 = 64801720 = 17times420times4 = 68802340 = 23times240times2 = 4680 1116 = 11times516times5 = 5580

3) Convert given fractions into fractions with equal numerators(iii) 1519 2528 911 and 4547Solution Given fractions 1519 2528 911 and 4547Therefore the LCM of 15 25 9 and 45 is 2251519 = 15times1519times15 = 225285 2528 = 25times928times9 = 225252911 = 9times2511times25 = 2252754547 = 45times547times5 = 225235

4) Put the given fractions in ascending order by making denominators equal

(iii) 57 38 914 and 2021Solution Given fraction 57 38 914 and 2021Therefore the LCM of the denominators is 16857 = 5times247times24 = 12016838 = 3times218times21 = 63168914 = 9times1214times12 = 1081682021 = 20times821times8 = 160168Therefore ascending order 63168lt108168lt120168lt160168Therefore ascending order of given fractions38lt914lt57lt2021

COMPUTER CHAPTER-1COMPUTER FUNDAMENTALS

DONE IN THE PREVIOUS CLASSES PAGE 10CWRITE TRUE AND FALSE

1 True2 False3 False4 False5 True

GEOGRAPHY CHAPTER 7EUROPE

CHAPTER COMPLETE 1)Europe is home to a famous mountain range called the Alps

2)River Rhine originates in Switzerland

3)The Eiffel Tower one of the tallest structures in Europe

4) Vatican City is one of the most densely populated European countries

5)Sognefjordin Norway is the largest fjord in Norway

Class VIIISubject Topic Summary Execution

MATHEMATICS Ch 6Sets

Exercise 6 (D)1 Given A = x x isin N and 3iquest x le 6 and B = x x isin W and xlt4 find (i) Sets A and B in roster form (ii) A cup B (iii)

A cap B(iv) A ndash B (v) B ndashA

Solution (i) A = 456 and B = 0123

(ii) A cup B = 0123456 (iii) A cap B = ϕ (iv) A ndash B = 456 (v)B ndash A = 0123

3 If A = 56789 B = x 3 lt x lt 8 and x isin W and C = x xle5 and x isin N Find (i) A cup B and (A cup B) cup C (ii) B

cup C and A cup ( B cup C)

(iii) A cap B and (A cap B) cap C (iv) B cap C and A cap (B cap C)

Is (A cup B) cup C = A cup (B cup C)

Is (A cap B) cap C = A cap (B cap C)

SolutionA = 56789 B = 4567 C = 12345

there4 (i) A cap B = 456789 and (A cup B) cup C = 123456789

(ii) B cup C = 1234567 and A cup ( B cup C) = 123456789

(iii) A cap B = 567 and (A cap B) cap C = 5

(iv) B cap C = 45 and A cap (B cap C) = 5

Now (A cup B) cup C = 123456789

And A cup ( B cup C) = 123456789 there4 (A cup B) cup C = A cup (B cup C)

Again (A cap B) cap C = 5 and A cap (B cap C) = 5

there4 (A cap B) cap C = A cap (B cap C)

4 Given A = 012345 B = 02468 and C = 0369 Show that (i) A cup (B cup C) = (A cup B) cup C ie the union

of sets is associative (ii) A cap (B cap C) = (A cap B) cap C ie the intersection of sets is associative

SolutionNow B cup C = 0234689 and A cup B = 01234568

there4 A cup (B cup C) = 012345689 and

(A cup B) cup C = 012345689

So (i) A cup (B cup C) = (A cup B) cup C ie the union of sets is associative

Again B cap C = 06 and A cap B = 024

there4 A cap (B cap C) = 0 and (A cap B) cap C = 0

So (ii) A cap (B cap C) = (A cap B) cap C ie the intersection of sets is associative

Physics Chapter 2 Physical Quatites and Measurements

Here We Will Do Some QuestionsRelated To Chapter 2

A density bottle has a marking 25 mL on it It means that

1 the mass of density bottle is 25g

2 the density bottle will store 25 ml of any liquid in it

3 the density bottle will store 25 ml of water but more volume of liquid denser than water

4 the density bottle will store 25 ml of water but more volume of a liquid lighter than water

Solution 2 the density bottle will store 25 ml of any liquid in it

COMPUTER CHAPTER-2Spreadsheet Functions and Charts

SELECTING RANGE IN ROWSCOLUMNSWHEN TWO OR MORE CELLS ARE SELECTED IT IS CALLED A RANGEA RANGE OF CELLS CAN BE FORMED IN TWO WAYS--a) SELECTING RANGE BY USING THE MOUSEb) SELECTING RANGE BY USING THE KEYBOARD

Q1)WRITE THE STEPS TO SELECT PARTIAL RANGE IN A ROW

Ans)THE STEPS ARE-6 SELECT THE ROW7 BRING THE CELL POINTER TO THE DESIRED

LOCATION FROM WHERE YOU WANT TO START YOUR SELECTION

8 CLICK THE LEFT MOUSE BUTTON AND KEEP DRAGGING TO YOUR RIGHT TILL YOU REACH THE LAST CELL TO NE SELECTED

RELEASE THE MOUSE BUTTON

GEOGRAPHY Asia

CLIMATE

Asia experiences great extremes of climate Jacobabad in the Sind province of Pakistan is one of the hottest places in the WorldVerkhoyansk in Siberia is one of the coldest places in the WorldCherrapunji and Mawsynram in India are two wettest places in WorldArabia Tibet Gobi and Mongolia are extremely dry regionsFactors Affecting Climate of Asia-The factors influencing the climate of Asia are-

Factors Affecting Climate of Asia-Thoroughly read the table in page number 60

Latitudinal extent

Continentality

Relief features

Presence of low pressure trough

Jet streams

English Language The Sentence A complex sentence contains one independent clause and at least one dependent clause The dependent clause in a complex sentence is introduced with subordinating conjunctions or relative pronouns

Commonly Used Subordinating Conjunctions-Time after before while when since untilCause And Effect because now since as in order that soOpposition although though even though whereas while in spite ofCondition if unless only if whether or not even if in case(that)

Commonly Used Relative Pronouns-Who whose whom which whoever whomever whichever that

Class IXSubject Topic Summary Execution

1-BENGALI(2ND LANGUAGE)

ldquo বঙগভমিরপরমিrdquo াইকেলধসদনদতত

আচেগর পর উততর পচো-১ ২ ৩ এং নীচের পর টি াহির কা- ৪মহিbকাও গচেনা দেগা পহিচে অমত হরচে- ক) কার দো দেকান কহিতার অং( ) কতা দেক পরসঙগ কী উহিকতটির তাৎপযC আচোনা কচেরা৫দেসই ধনয নরকচে দোচেক যাচে নাহি ভচে মচেনর মহিeচের সাচেসচে সCন ক) কহির কায C ার উচেf(য হিক হিছ কহি কন কহিতাটি দেচেন) কহি কার কাচেছ হিমনহিত কচেরচেছনগ) কহি এই পহিথীচেত কাচের ধনয মচেন কচেরনঘ) কহি হিক রকম অমর তাাভ করচেত ান

Hindi 2nd lang

काकी(कतिसयारामशरण गपत)

इस कहानी म लखक न यह बतान का परयास निकया ह निक बचच अपनी मा स निकतना परम करत ह शयाम अबोध बालक ह वह अपनी मा क मरन क बा- उसन अपनी मा क कतिलए बहत रोया बा- म उस पता चला निक उसकी मा राम क घर चली गई ह आकाश म उडती हई पतग -खकर उस हरष हआ निक पतग क दवारा वह अपनी मा को नीच उतारगा इसक कतिलए वह अपनी निपता की जब स -ो बार सवा रपया निनकालकर पतग और -ो मोटी सी मन वाली अपन भाई स काकी एक कागज पर कतिलखवा कर पतग म कतिशव का दि-यानिनकालकर पतग और -ो मोटी सी मन वाली अपन भाई स काकी एक कागज पर कतिलखवा कर पतग म कतिचपका दि-याभोला और शयाम कोठरी म रससी बाधनी रह थ तभी उसक निपता करोध म आकर उन स पछ निक कया उनकी जब स रपया निनकाला हभोला डर क मार बताया निक शयाम इस पतग क दवारा अपनी काकी को राम क यहा स उतारना चाहता हनिवशशवर(शयाम क निपता)न फटी पतग उठाकर -खी तो उस पर काकी कतिलखा थावह हत बजिa होकर वही खड रह गएउनहोन सोचा निक मन अपन पतर को मारा जोनिक अनजान और निन-dरष थावह अपनी मा कोनिकतना पयार करता ह

उस दि-न बड सवर शयाम की नी- खली तो -खा निक घर भर म कोहराम मचा हआ ह

क) घर म कोहराम कयो मचा हआ था शयाम को कया लगा

ख) काकी को ल जात समय शयाम न कया उपदरव मचाया

ग) काकी क बार म उस कया बताया गया कया सतय उस कतिछपा रहा

घ) वह बठा-बठा शनय मन स आकाश की ओर कयोकरता

उततरक) शयाम की मा का -हात हो गया था इसकतिलए

घर म कोहराम मचा हआ था शयाम की लगा निक उसकी मा सफ- कपडा ओढ हए भमिम पर सो रही ह

ख) लोग जब उमा यानी शयाम की मा को उठाकर ल जान लग तब शयाम न बडा उपदरव मचाया लोगो क हाथ स झठ करवा उमा क ऊपर जा निगरा और बोला काकी सो रही ह उस कहा ल जा रह हो

ग) काकी क बार म बजिaमान लोगो न उस निवशवास दि-लाया निक उसकी का निक उसक मामा क यहा गई ह लनिकन सतय अमिधक दि-नो तक कतिछपाना रह सका आसपास क अबोध बालको क मह स यह बात परकट हो गई निक उसकी मा का -हात हो गया ह

घ) कई दि-नकई दि-न लगातार रोत-रोत उसका रोना तो शान हो गया पर उसक ह-य म शोक भर गया था वह चपचाप बठा आकाश की और टाका करता निक शाय- उसकी काकी कही दि-ख जाए

ldquoदि-न उसन ऊपर आसमान म पतग उडती -खी न जान कया सोच कर उसका निहर-य एक-म खिखल उठाrdquo

क) निकसन पतग ऊपर उडत -खी और वह कयो खश हआ

ख) उसन अपन निपता स कया कहा उनका कया उतर थाश

ग) उसन निफर कया निकया और निकसन उसकी सहायता की

घ) उसकी योजना कया थी उततर -क) शयाम न एक दि-न आसमान म पतग उडती

-खी तो उसन सोचा निक पता आसमान म राम क यहा जाकर रकगी वही पर मरी काकी ह यह सोचकर वह बहत खश हआ

ख) उसन अपन निपता स कहा काका मझ एक पतग मगा -ो उसक निपता न भटक हए मन क भाव स कहा निक मगा -ग यह कह कर उ-ास भाव स वह कही और चल गए पतग नही आई

ग) उसन चपचाप निवशशवर क टगहए कोट स एक चवननी निनकाल ली और सखिखया -ासी क लडक भोला की सहायता स एक पतग मगवानी भोला उसकी बराबर उमर का ही था

घ) उसकी योजना यह थी निक वह अपनी पतग को आकाश म राम क यहा भजगा और उस पतग क सहार उसकी काफी नीच उतर जाएगी इस योजना पर उस परा निवशवास था इसकतिलए वह और भोला -ोनो यह काम करन म लग गए

Continue to nexthelliphellipEVS CHAPTER - 1

(UNDERSTANDING OUR ENVIRONMENT)

Sustainable development

The development that meets the needs of the present without compromising the ability of future generations to meet their own needs is called Sustainable development

Sustainable societies ndash

An environmentally sustainable community is one that meets the current and future basic resource needs of its people in a just and equitable manner without compromising the ability of future generations to meet their basic needs

Q ) What are Eco Villages

Ans - Eco village are the urban or rural communities of people who strive to integrate a supportive social environment with a low impact way of life

Q ) To ensure sustainable development the depletion of renewable resources should not take place at a rate faster than their regeneration Justify your answer

Ans ndash Renewable resources do not have a fixed quantity - more can always be

generated However if the rate of use exceeds the rate of renewal - that is the

source is used more than its being recreated - its continued use will become

used up faster than it can regenerate

To promote sustainable society the following things need to be done ndash

1 Using renewable energy sources 2 By improving the quality of human

health 3 By promoting sustainable agriculture 4 By forming ecovillage

it will eventually be entirely depleted So Toensure sustainable development the depletion of

renewable resources should nottake place at a rate faster than their regeneration

Q ) What do you mean by Sustainable societies

Ans - Sustainable societies are defined as towns and cities that have taken steps to remain healthy over the long term These communities value healthy ecosystems use resources efficiently and actively seek to retain and enhance a locally based economy Sustainable development concerns everybody in a society

Q ) What are the effects of pollution on human health

Ans ndash Some health problem occurs due to air pollution are ndash

Respiratory diseases Cardiovascular damage Fatigue headaches and anxiety Irritation of the eyes nose and throat Damage to reproductive organs Harm to the liver spleen and blood Nervous system damage

Some health problem occurs due to water pollution are ndash

Typhoid Cholera Dysentry Jaundice

Some health problem occurs due to noise pollution are ndash

Fatigue headaches and anxiety High blood pressure Hearing damage

Physics Motion in 1D First go through previous notes Now here we will solve some numerical related to that

Question 3What information about the motion of a body is obtained from the displacement-time graphSolution 3From displacement-time graph the nature of motion (or state of rest) can be understood The slope of this graph gives the value of velocity of the body at any instant of time using which the velocity-time graph can also be drawn

Question 4(a)What does the slope of a displacement-time graph represent(b)Can displacement-time sketch be parallel to the displacement axis Give a reason to your answerSolution 4(a) Slope of a displacement-time graph represents velocity(b) The displacement-time graph can never be parallel to the displacement axis because such a line would mean that the distance covered by the body in a certain direction increases without any

increase in time which is not possible

Chemistry Language of Chemistry

How to balance a chemical equationThere are two methods of balancing an equation(i)Hit and trial method(ii)Partial equation methodBalancing by hit and trial methodThis method consists of counting the number of atoms of each elements on both sides and trying to equalize themTake the following steps(i)Count the number of times (frequency) an element occurs on either side(ii)The element with the least frequency of occurrence is balanced first(iii)When two or more elements have the same frequencythe metallic element is balanced firstExample-1 On heatinglead nitrate decomposes to give lead dioxidenitrogen dioxide and oxygenPb(NO3)2rarrPbO+NO2+O2

In this equationLead occurs twiceNitrogen occurs twiceOxygen occurs four timesSince lead is a metalbalance it firstThe number of atom of lead is equal on the two sidestherefore it needs no balancingNow balance nitrogenOn the reactant sidethere are two atoms of nitrogenwhile on the product side oneSomultiply the product containing nitrogenon the product sideby two Pb (NO3)2rarrPbO+2NO2+O2Nowthe number of oxygen atoms on the reactant side 6while on the product sideit is 7Somultiply the entire equation by 2except oxygen to get balanced equation2Pb(NO3)2rarr2PbO+4NO2+O2Multiplication by 2 is done only when atoms of all the elements except one element are balanced and the unbalanced atom occurs separately at least once and also there is a difference of only one such atom

Math Topic AlgebraChapter

Factorisation

Study item Difference of two squares a2 ndash b2 = (a+b) (a-b)1) (i) 4x2ndash 25y2

= (2x) 2 ndash (5y) 2= (2x + 5y) (2x - 5y)

(ii) 9x2 ndash 1= (3x)2ndash(1)2= (3x + 1)(3x ndash 1)

2) (i) 150 ndash 6a2= 6(25 ndash a2)= 6(5)2 ndash(a)2= 6 (5 + a) (5 ndash a)

(ii) 32x2 ndash 18y2=2(16x2 ndash 9y2)=2(4x)2 ndash (3y)2= 2(4x + 3y)(4x - 3y)3)(i) (x ndashy )2 ndash 9 = (x ndash y )2 ndash (3)2= (x ndash y + 3) (x ndash y ndash 3)(ii) 9(x + y) 2ndash x 2= (3)2(x + y)2 ndash (x)2=3(x + y)2 ndash (x)2= (3x +3y ) 2ndash(x)2= (3x + 3y + x)(3x +3y ndash x)= (4x + 3y) ( 2x + 3y )

Commercial studies

Basic accounting terms

Today I will give you some questions from the previous study material

Questions1) Define accounting2) What do you mean by debit and

credit

3) Explain the types of account4) Define the following terms

a) Assetsb) Capitalc) Purchased) Debtorse) Transactions

5) Name the types of accounts given below

a) Krishnas accountb) Machinery accountc) Royalty accountd) Salary accounte) Furniture accountf) Audit fee account

Economics Revision Today I will give you some revision questions

Questions1) What do you mean by the terms

rdquowantsrdquo2) Write the difference between

consumer goods and producer goods

3) Define the term utility 4) Explain the different types of utility5) Define

a) Total utilityb) Marginal utility

Subject Eng Literature (The Merchant of Venice ndash William Shakespeare)Topic Act I Scene 3 Lines 1 to 48 (Shylock hellip Cursed be my tribe if I forgive him) Date 16th April 2020 (5th Period)

[Students should read the original play and also the paraphrase given in the school prescribed textbook]Summary Questions amp Answers

This scene takes place in Venice and we are introduced to the rich Jew Shylock Bassanio and Shylock are talking and Bassanio tells Shylock that he wants a loan of three thousand ducats for three months on the personal security of Antonio

o Shylock feels glad because he will be able to bind down Antonio by means of a bond on account of the loan but he tells Bassanio that all the fortunes of Antonio being invested in the merchant ships on the sea it is difficult to depend upon his credit Even under such circumstances Shylock is willing to advance the money on the personal security of Antonio

o Bassanio then invites Shylock to dine with him Shylock says that he is prepared to do anything with the Christians but not eat or drink or pray with them

o While Bassanio and Shylock are talking Antonio appears on the scene Shylock does not seem to take any notice of Antonio but goes on brooding within

(1) SHYLOCK Ho no no no no- my meaning in (Line 15-26)saying he is a good man is to have you understand me that he is sufficient Yet his means are in suppositionhe hath an argosy bound to Tripolis another to the Indies I understand moreover upon the Rialto he hath a third at Mexico a fourth for England and other ventures he hath squanderd abroad Butships are but boards sailors but men there be land-rats and water-rats land-thieves and water-thieves I mean pirates and then there is the peril of waters winds and rocks The man is notwithstanding sufficientmdashthree thousand ducats mdashI think I may take his bond

(a) Who is talking in the beginning of this scene What does Bassanio want from Shylock How does Shylock feel

In the beginning of the scene Bassanio and Shylock are talking to each other Bassanio wants to borrow three thousand ducats from Shylock for three months on the security of Antonio Shylock feels glad at heart that he will get the opportunity of binding Antonio with a bond(b) What risks does Shylock weigh in advancing the money

Shylock says that Antonio has invested all his capital in trading by sea-going ships But the ships are made of wood and the sailors of those ships are ordinary human beings The wood can

himself how he hates Antonio because of his being a Christian because he abuses Shylock in public places Shylock decides that if ever he can get Antonio to his advantage he will teach him a lesson

come to harm and men can commit mistakes and thus the capital invested in ships may be lost Then there are other dangers The goods loaded on the ships can be damaged by rats and thieves which are found both on land and water The ships can also be harmed through sea-storms submerged rocks etc(c) What two important functions does this scene have

The scene has two important functions First it completes the exposition of the two major plot lines of the play Antonio agrees to Shylockrsquos bond ndash three thousand ducats for a pound of flesh and second and more important dramatically this scene introduces Shylock himself In this scene Shakespeare makes it clear at once why Shylock is the most powerful dramatic figure in the play and why so many great actors have regarded this part as one of the most rewarding roles in all Shakespearean dramas(d) Where does this scene take place What kind of treatment has Antonio been giving to Shylock What does Shylock say when Bassanio invites him to dine with him

The action of this scene takes place in Venice Antonio has been in the habit of behaving harshly with Shylock ndash spitting on his beard and footing him like a stranger cur When Bassanio invites Shylock to dine Shylock says that he is prepared to do anything with the Christians but not eat and drink or pray with them

(2) SHYLOCK How like a fawning publican he looks (Line 38-48)I hate him for he is a Christian

But more for that in low simplicity

He lends out money gratis and brings downThe rate of usance here with us in VeniceIf I can catch him once upon the hipI will feed fat the ancient grudge I bear him

He hates our sacred nation and he railsEven there where merchants most do congregateOn me my bargains and my well-won thriftWhich he calls interest Cursed be my tribeIf I forgive him

(a) What is the context in which these words are spoken and what is the idea expressed in it

These remarks are made by Shylock when he sees Antonio coming along after Bassanio told him that the merchant will be his surety for the bond The above mentioned passage reveals Shylockrsquos hatred for Antonio Shylock says that he hates Antonio because he is a Christian and also because he gives loan without taking interest on them thereby bringing down the rate of interest in Venice(b) Explain the meaning of the phrase lsquoa fawning publicanrsquo

The phrase lsquoa fawning publicanrsquo refers to Roman tax collector It is a term of contempt and hatred on the lips of a Jew lsquoFawning Publicansrsquo were Roman tax-gatherers whose ordinary bearings towards the Jews was bullying but whose false pose of lsquohumility and contritionrsquo is touched upon in the parable in New Testament(c ) What light does the above passage throw on the character

of Shylock

The above mentioned speech of Shylock reveals him to be a wicked character having an extreme greed for wealth His intense hatred for Antonio is unjustified He hates Antonio only because he is a Christian and because he lends money without taking any interest on it thereby adversely affecting Shylockrsquos business of lending money on high interest(d) What information do you gather about Antonio from the above given lines

Shylockrsquos statement throws a valuable light on the character of Antonio Antonio appears to be a good Christian and a good human being He helps the people in need by lending them money without charging any interest on it He is a man of simple and good nature This very goodness makes him Shylockrsquos enemy(e) What does Shylock debate within himself and when To whom are the lines mentioned above addressed to

When Bassanio asks the Jew to lend him three thousand ducats on Antoniorsquos surety Shylock begins to debate within himself as to how he should exploit the opportunity of a business deal with his old enemy Antonio

The lines mentioned above are not addressed to anyone The lines are a soliloquy ie a speech made by a character to himself and not meant to be heard by the other characters present

Class XSubject Topic Summary ExecutionEnglish

LiteratureThe Blue Bead 2nd part

Things took a turn for the worst and all of a sudden a crocodile attacked the woman biting on the womanrsquos leg At that moment Sibia got up sprinted grabbed the hay fork and stabbed the crocodile in the eye with all her power Immediately the crocodile let go and went away Sibia saw a small blue bead lying by the river she grabbed it Since she was poor she didnrsquot have necklace Shersquod always wanted one like the other women now she could make one with the blue bead After that she went home and told her mother all about it

Hindi 2nd

Langबड घर की बटी( मशी परमच-)

lsquoबड घर की बटी कहानी का उददशय मधयम वग की घरल समसया को सलझा कर सगदिठत परिरवार म मिमल जलकर परम स रहन का स-श -ना हघर म शानित सथानिपत करन की जिजमम-ारी नारी की होती ह यदि- नारी समझ-ार ह उसम धय और परिरवार क परनित परम ह तो कोई भी घटना परिरवार को निवघदिटत नही कर सकती या कहानी परिरवार को सगदिठत करत हए परम सौहा- स एक दसर की भावनाओ को समझ करउनका सहयोग करत हए जीवन यापन करन की पररणा -ती हमशी परमचदर जी न इस कहानी म सय परिरवार का परनितनिनमिधतव निकया ह यह कहानी बनी माधव सिसह जो गौरी पर क जमी-ार क उनक -ो पतरो की हशरी कठ लाल निबहारीशरीकात का निववाह एकजमी-ार घरान की पतरी आन-ी स हआ थाआन-ी न ख- को ससराल क वातावरण म ढाकतिलया थाएक दि-न आन-ी का अपन -वर लाल निबहारी स झगडा हो जाता ह -ोनो भाई एक दसर स अलग होन की कोकतिशश करत हसभी बह आन-ी न अपन मधर वयवहार स लाल निबहारी को

ldquoइन नतर निपरय गणो को बीए-इनही -ो अकषर पर नयोछावर कर दि-या था इन -ो अकषर न उनक शरीर को निनबल और चहर को कानित ही बना दि-या थाldquo

क) परसतत पकतियो म निकस वयकति क बार म कहा गया ह

ख) इन पकतियो म कौन स नतर निपरय गणो क बार म कहा गया ह

ग) बीए की निडगरी परापत कर लन पर भी उपय वयकति क सवभाव की कया निवशरषता थी

घ) यह नतर निपरय गण निकस वयकति म निवदयमान थ उसक वयकतितव की कया निवशरषता थी

उततर ndashक) परसतत पकति म गौरी पर गाव क जमी-ार

क बड बट शरीकात क बार म कहा गया ह उसन बहत परिरशरम और उ-म क बा- ba की निडगरी परापत की थी अब वह एक -फतर

घर छोडकर जान स रोक कतिलयाइस पर बनी माधव सिसह न कहा निक बड घर की बटी ऐसी ही होती ह जो निबगडा काम बना लती ह अतः शीरषक साथक ह बड घर की बटी आन-ी ह

म कमचारी थाख) भरा हआ चहरा चौडी छाती और डटकर

खाना आदि- एक सबजी ल जवान क गण मान जात ह परत शरीकात न इनही नतर निपरय गणो को अपनी पढाई पर नयोछावर कर दि-या था

ग) बीए की निडगरी परापत कर लन पर भी उपय वयकति(शरी कठ की शारिररिरक तौर पर निनबल और चहर स कानित ही लगत थ इतना ही नही वह मानकतिसक तौर पर भी निपछड हए थ पाशचातय सामाजिजक कथा उस घणा एव पराचीन सभयता का गणगान उनकी निवचारधारा क परमख अग थ

घ) यह नतर निपरय गण गौरीपर गाव क जमी-ार क छोट बट लाल निबहारी सिसह म निवदयमान थ वह सजीलाजवान था और भस का दध शर दध वह सवर उठकर पी जाता था

ldquoयही कारण था निक गाव की लललन आए उनकी निन-क थी कोई कोई तो उह अपना शतर समझन म भी सकोच ना करती थी सवय उनकी पतनी को इस निवरषय म उनस निवरोध थाldquo

क) उपय पकति म इस वयकति क बार म कहा गया ह

ख) गाव की लललन आए उनकी निन-ा कयो निकया करती थी

ग) उनकी पतनी का कया नाम था उनह निकस निवरषय म अपन पनित क निवरa था और कयो

घ) इस कहानी का कया उददशय ह Continue to next helliphelliphellip

Bengali 2nd Language

ফ ফটক না ফটক( কহিতা )

পর) ldquo(ান াধাচেনা ফটপাচেথ পাথচের পাড হিচেয় এক কাঠচোটটা গাছ কহিকহি পাতায় পার ফাটিচেয় াসচেছldquoক) কার দো দেকান কহিতার অং( ) lsquo(ান াধাচেনা ফটপাচেথ পাথচের পাডহিচেয়lsquo চেত কী দোঝাচেনা চেয়চেছ গ) আচো য অংচে(lsquo এক কাঠচোটটা গাছ lsquoচেত কী দোঝাচেনা চেয়চেছ ঘ) ldquoকহিকহি পাতায় পার ফাটিচেয় াসচেছldquo ----- একথার পরকত অথC কী উততর ) ক) আচো য অং(টি পর যাত কহি সভা4 মচোপাধ যাচেয়র দো lsquoফ ফটক না ফটকrsquo কহিতার অং()কহি সভা4 মচোপাধ যায় হিছচেন দেপরচেমর কহি দেপরমচেক নানা ভহিঙগমায় হিতহিন ফটিচেয় তচেচেছন দেপরম মানচের স মচেতC র সঙগী কহিতার কহিতায় এক রb সb হচেয়র দেপরম াগরচেনর কথা চেচেছন (ান অথCাৎ দেযাচেন দেকান রস দেনই দেযাচেন দেকান মহিনতা দেনই অথ তার মধ দেযও দেপরম থাকচেত পাচের একথাই কহি তচে ধরচেত দেচেয়চেছন একটি মানচে4র মচেন দেযাচেন দেকামতার দেকান সথান দেনই পাথচেরর মচেতা হিনরসতার মচেনর মধ দেযও দেয দেপরম আসচেত পাচের দেস কথাই কহি চেচেছনগ)নারীচের যথC দেপরচেমর ছহি এই কহিতায় অকপচেট উচেঠ এচেসচেছ কহি এই কহিতায় কাটচোটটা গাছ কথাটি যার কচেরচেছন নারী দেয দেপরম দেথচেক হিতাহিত এং দেসই দেপরম সঠিক সমচেয় না পাওয়ার ন য দেপরম সমপচেকC হিচেr4 গৈতরী য় দেপরচেমর দেয গৈহি(ষট য মাধযC য সরসতা দেকামত এই সমসতর হিপরীত যথা রbতা শষকতা কচেঠার তা পরভহিত দোঝাচেত এক কাঠচোটটা গাছ কথাটি যার কচেরচেছনঘ) এাচেন এক নারীর যথC দেপরচেমর কথা হিনহিCপত ভাচে চেচেছন কহি অসমচেয় নারীর ীচেন দেপরম দেচেগচেছ এতহিন তার হয় রb কচেঠার হিছ দেপরচেমর অভাচে ঠাৎ দেসই শষক মরভহিমচেত সচের আভাস এচেসচেছ দেপরম দেযন 4Cার স(ীত তাই পরায় মত গাচেছ কহিকহি পাতা গহিচেয় উচেঠচেছ

Biology Chapter - 01Controlling Air Pollution

Today we will discuss how we control air pollution from domestic combustion

Q1Describe any five ways of reducing air pollution from domestic sources bull The number of pollutants in the air is verylarge and we always try to control them byfollowing ways

i) Solar cooker and solar heater It use no fuel reduce damage of environment by fuel use or reducing deforestation It maintains coolness of house It releases very less orno oil gas or grease

ii) Piped natural gas (PNG) It emits very less by products into the atmosphere As it isdistributed through pipe lines so there iscontinuous supply of fuel is possible

iii) Liquefied Petroleum Gas (LPG) It hasa higher heating value LPG doesntcontain sulphur so it burns a lot cleanerenergy sources It releases very less oralmost no fume in air

iv) Electricity based cooking Emission free cooking alternative for urban dwellers causeselimination of adverse health impactsofindoor air pollution It helps to avoid theinconveniences associated with procurement of LPG

v) Biogas It contains 75 methane whichmakes it an excellent fuel It burns without smoke and biogas plant leaves no residue like ash in wood charcoal etc Thus it isaclean fuel

Economics

Factors of Production

Today firstly we would recall the last class for 5 mins and then we would proceed with the further topics of the chapter

The concept meaning of land characteristics of land and importance of land to be repeated for the absentees as well as the students who were there in the class the previous day

Today we will start with the last portion of land before it the meaning of land to be repeated onceAs by now we all know that

Questions1What do you mean by productivity of landAnswer By productivity of land we mean the capacity of a piece of land to produce a crop

Thus it refers to the average output per unit of landSay per acre per hectare etc= (OutputArea of land)

2 What are the factors influencing the productivity of landAnswer

Natural factors Productivity of land is largely determined by the natural

Land is defined to include not only the surface of the earth but also all other free gifts of nature(for example mineral resources forest resources and indeed anything that helps us to carry out the production of goods and services but is provided by nature free of cost)

We will move on to the last portion of land by discussing Productivity of Land

By productivity of land we mean the capacity of a piece of land to produce a crop

Thus it refers to the average output per unit of land

Say per acre per hectare etc= (OutputArea of land)

With this we shall proceed further with the main factors that determine the productivity of land

Natural factors Human factors Improvements on land Location of land Organisation Ownership of land Availability of capital Proper use of land State help

Note economic development of a country depends upon the quality of its land If the land is fertile it will quicken the pace of development of the country

qualities of land such as fertility etc

Human factors Land cannot produce anything by itself Man has to apply labour on it to produce for himself So productivity of land depends on the knowledge and skills of workers

Improvements on land production of land is affected by land development measures like provision of well or tubewell irrigation proper drainage

State help The government of a country especially less developed country can play a vital role in improving the agricultural productivity by providing better irrigation facilities

Organisation Productivity of land also fdepends upon the way how the factors of production like labour and capital are organised

In order to increase productivity trained workers modern implements scientific methods good seeds are all essential

3 lsquoImproved technology affects the productivity of landrsquo Explain this statement with the help of suitable example Answer Use of improved technology raises the productivity of land Example By using HYV seeds chemical manures and modern machines per hectare output increases

Physics Force (Summary)

Question Write the expression for the moment of force about a given axisSolutionsThe expression for the moment of force is given byMoment of force about a given axis = Force times perpendicular distance of force from the axis of rotationQuestion What do you understand by the clockwise and anticlockwise moment of force When is it taken positiveSolutionsIf the effect on the body is to turn it anticlockwise moment of force is called the anticlockwise moment and it is taken as positive while if the effect on the

body is to turn it clockwise moment of force is called the clockwise moment and it is taken as negative

Math Topic Commercial Mathematics

Chapter Goods and services Tax

Study item Some solved sums from exercise ndash 1 A retailer buys a TV from a wholesaler for Rs 40000 He marks the price of the TV 15 above his cost price sells it to the consumer at 5 discount on the marked price If the sales are intra ndash state and the rate of GST is 12 find

(i) The marked price of the TV(ii) The amount which the consumer pays for the TV(iii) The amount of tax (under GST) paid by the retailer to the central

Government(iv) The amount of tax (under GST) received by the State Government

Solution As the sales are intra- state sale and the rate of GST 12 So GST comprises of 6 CGST and 6 SGSTTherefore a retailer buys a TV from a wholesaler for Rs 40000Therefore the amount of GST collected wholesaler from the retailer or paid by retailer to wholesalerCGST = 6 of Rs 40000 = Rs(6100 times40000) =Rs 2400SGST = 6 of Rs 40000 = Rs (6100 times 40000) =Rs 2400Therefore wholesaler will pay Rs 2400 as CGST and Rs 2400 as SGSTTherefore amount of input GST of retailer Input CGST = Rs 2400 and input SGST = Rs 2400Again the retailer marks the price of the TV 15 above his cost price(i) The marked price of the TV

= Rs 40000 + Rs 40000times15= Rs 40000 + Rs 40000times 15100= Rs 40000 + Rs 6000Rs 46000But the retailer sells it to consumer at 5 discount on the marked priceCost price after discount = Rs 46000 ndashRs46000times 5100 =Rs 46000 ndashRs 2300= Rs 43700Therefore the amount of GST collected retailer from consumer or paid by consumer to retailerCGST = 6 of Rs 43700 =Rs ( 6100 times43700)Rs 2622SGST = 6 of Rs 43700 = Rs (6100 times 43700) =Rs 2622Amount of the output GST of retailer Output CGST = Rs 2622 and output SGST = Rs 2622

(ii) The amount which the consumer pays for the TV= cost price of TV to consumer + CGST paid by consumer + SGST paid by consumer= Rs 43700 + Rs 2622 + Rs 2622= Rs 48944

(iii) The amount of tax (under GST ) paid by the retailer to the central Government=CGST paid by retailer = output CGST ndash input CGST=Rs 2622 ndash Rs 2400=Rs 222

(iv) The amount of tax ( under GST ) received by the State Government = SGST paid by wholesaler + SGST paid by retailer= Rs 2400 + output SGST ndash input SGST=Rs 2400 + Rs 2622 ndash Rs 2400=Rs 2400 + Rs 222= Rs 2622

Commercial studies

Stakeholders Today I am going to give some revision questions from the previous study material

Questions1) State the two expectations of

employees from a business concern2) Give two distinctions between

stakeholder and shareholder3) Give two difference between

internal stakeholders and external stakeholders

4) Give two expectations of suppliers from a business organisation

5) Who is a stakeholder in commercial organisations

Chemistry Periodic Table

Merits of Mendeleevrsquos Periodic law are as follows - 1He grouped the elements on the basis of atomic mass 2 He left gaps for undiscovered elements like Gallium Scandium germanium Also he left a full group vacant for undiscovered inert gases 3 He could predict proportions of several elements on basis of their position in periodic table like Ga Sc etc 4He could predict errors in atomic weights of some elements like gold platinum etc

Anomalies in Mendeleevrsquos Periodic law are as follows - 1 Position of isotopes could not be explained 2 Wrong order of atomic masses could not be explained

For example- as Arnur atomic mass 40 come first and K with low atomic mass (30) should come later but k should be placed first

According to Bohrrsquos Modern Periodic table properties of elements are periodic functions of their atomic numbers

So when elements are arranged according to increasing atomic numbers there is periodicity in electronic configuration that leads to periodicity in their chemical properties

It consists of horizontal rows (Periods) Vertical column (Groups)

There are 7 period and 12 groups in this long form of periodic table

Ist period has 2 elements IInd period has 8 elements IIIrd period has 8 elements IVth period has 18 elements Vth period has 18 elements VIth period has 32 elements VIIth period hs rest of elements

Note - The number of valence electrons in atom of elements decides which elements will be first in period and which will be last

In group- 1 to 2 gp and 13 to 17 contain normal elements 3 to 12gp ndash transition elements 57 to 71 - lanthanides 89 to 103 - Actinides

Left hand side ndash metals Right hand side ndash nonmetals

Note- Hydrogen element has been placed at top of Ist group Electronic configuration of H is similar to alkali metal as both have 1 valence electron

V electron of gp I element -- 1 V electron of gp 2 element -- 2 V electron of gp 13 element -- 3 V electron of gp 14 element -- 4 V electron of gp 15 element -- 5 V electron of gp 16 element --6 V electron of gp 17 element -- 7 V electron of gp 18 element -- 8

English 1 Transformation of sentences

Sentences A sentence is a group of words which makes complete sense

Exercise 2Change the following sentences from

a Assertive sentencesb Imperative sentencesc Interrogative sentencesd Exclamatory sentences

Sentences can be changed from one grammatical form to another without changing the meaning of the sentence This is known as transformation of sentences

assertive to interrogative1 Nobody would like to be a fool

Who would like to be a fool2 Their glory can never fade

When can the glory fade3 Nobody can control the wind

Who can control the wind4 It matters little if I die

What though I die5 No man can serve two masters

Can any man serve two masters

Exercise 3Interchange of assertive and Exclamatory sentences

1 She leads the most unhappy lifeWhat an unhappy life she leads

2 This is indeed an interesting bookWhat an interesting book this

3 He is a very great manWhat a great man he is

4 It is a very lame excuseWhat a lame excuse

5 It is sad that she died so youngAlas she died so young

Class XISubject Topic Summary Execution

Hindi 2nd lang

पतर परम(परमचदर) पतर परम कहानी म एक निपता की इचछाओ का वणन निकया गया ह अपन बड पतर परभ -ास स निपता चतनय -ास का निवशरष परम था निपता को उसक जनम स ही बडी-बडी आशाए थी उसम दसर बट कतिशव-ास की अपकषा स- उतसाह की मातरा अमिधक थी वह उस इगलड भजकर बरिरसटर बनाना चाहत थभागय का खल भी बडा निनराला ह बीए की परीकषा क बा- वह बीमार पड गया डॉकटरो न भी जवाब - दि-या थाचतन -ास जी बहत ही कजस थ बवजह पस खच करना नही चाहत थ अगर गारटी मिमलती तो शाय- पस खच भी कर -त परत गारटी नही थी परिरणाम सवरप उनक बट का -हात हो गयाजब बट को समशान ल जा रह थ तो वहा काफी शोर गान बजान हो रह थ पछन पर पता चला निक निकसी निपता निपछल तीन साल स निबमार था और उसक ईलाज म रपया पानी की तरह बहाया पर ठीक नही हए परत उसक बट को तनिनक भी अफसोस नही था उसका कहना था उसन कोकतिशश तो कीयह -खकर चतनय-ास जी को आतम निगलानी हईतभी स उनका म परिरवतन हआ और बट का भोज काफी धमधाम स निकयाऔर वहइस पशचाताप की आग म जलत रह औला- स बढकर पसा नही होता ह इस बात को समझन म उनह काफी व लग गया

hellipContinue to next

BENGALI(2ND LANGUAGE)

পরথমঅধযায়-ঠাকরারীনদরনাথঠাকর

নয়ন দোচের হিমাচেররা া নাচেমই হিযাত হিছচেন ায়ানার উাররণ সবরপ নয়ন দোচের ারা হিা (াচেকর হিা হিচেতন এছাাও দেকান উৎস উপচেb রাহিতর দেক হিন করার উচেfচে(য তারা সযC হিকরচেরণ রনয পরীপ জবাহিচেয় তাচেত রপার হির 4Cরণ করচেতন ঠাকরা এই নয়ন দো হিমারচের দে(4 ং(ধর হিছচেন হিমাররা ায়ানার ষটানত পর(Cন কচের তারা হিনঃসব এই হিমাহিরর দে(4 ং(ধর গৈকাস নদর রায়চেৌধরী গৈকাস া নয়ন দোচের সমসত সমপহিতত ঋচেরণর াচেয় হিহিx কচের অহি(ষট যা আচেছ তাচেত হিপত

ইার হিপতার মতয ইচে পর নয়নচোচের ায়ানার দেগাটা কতক অসাধাররণ শরাদধ (াহিনতচেত অহিনতম ীহিপত পরকা( কহিরয়া ঠাৎ হিনহিয়া দেগ- ক) কার দো দেকান গচেলপর অং() কতা দেক ইার চেত কাচেক দোঝাচেনা চেয়চেছ গ) পরসঙগ কী কতার কতয পহিরসফট কচেরা

পরচে4র যাহিত রbা করা সমভ নয় তাই হিতহিন পতরচেক হিনচেয় ককাতায় সাস শর কচেরন গলপ কথচেকর আহিথCক অসথা নয়ন দোচের হিমাচের দেথচেক সমপরণC আাা কথচেকর হিপতা হিনচের দেষটায় অথC উপাCন করচেতন া উপাহিধ াচেভর নয তার াসা হিছনা আর দেসই কারচেরণ কথক তার একমাতর উততরাহিধকার চেয় তার হিপতার পরহিত কতজঞ কথক দো পা হি(চেচেছন হিনচের পরারণ ও মান রbার নয উপচেযাগী অথC হিনা দেষটায় পরাপত চেয়চেছন- এটাই তার কাচেছ পরম দেগৌরচের হি4য় চে মচেন কচেরন কাররণ (নয ভাণডাচের গৈপতক ায়ানার উজজব ইহিতাস অচেপbা দোার হিসeচেকর মচেধয গৈপতক দেকামপাহিনর কাগ তার কাচেছ অচেনক দেহি( মযান

TO BE CONTINUED

উ- ক) আচোয অং(টি রীনদরনাথ ঠাকচেরর দো ঠাকরা গচেলপর অং() কতা চেন আচোয গচেলপর গলপ কথকইার চেত নয়ন দোচের হিমাহিরর দে(4 ং(ধর গৈকাস ার কথা া চেয়চেছ গৈকাস া নয়ন দোচের সমসত সমপহিতত ঋচেরণর াচেয় হিহিx কচের অহি(ষট যা আচেছ তাচেত হিপত পরচে4র যাহিত রbা করা সমভ নয় তাই হিতহিন পতরচেক হিনচেয় ককাতায় সাস শর কচেরনগ) গৈকাস ার হিপতার মতযর পর নয়ন দোচের হিমাহিরর অহিসততব হিপত য় কচেয়কটা উৎস ও শরাদধ- (াহিনতচেত হিমাহিরর দে(4 কহিটক যয় চেয় হিগচেয় এচেক াচের দে(4 চেয় যায় তন তাচের গC করার মত আর হিকছই হিছ না-দেসই পরসচেঙগ এই উহিকত নয়নচোচের হিমাচেররা া নাচেমই হিযাত হিছচেন ায়ানার উাররণ সবরপ নয়নচোচের ারা হিা (াচেকর হিা হিচেতন এছাাও দেকান উৎস উপচেb রাহিতরচেক হিন করচেত হিগচেয় তারা সযC হিকরচেরণর নয পরীপ জবাহিচেয় তাচেত রপার হির 4Cরণ করচেতন তাই দেসকাচের ায়ানা দেহি(হিন সথায়ী চেত পারত না হিহিভনন উৎস শরাদধ- (াহিনতচেত সাধযা হিতহিরকত র করার নয হিমাহির হিহিকচেয় দেযত হ হিতC কা হিহি(ষট পরীচেপর দেত দেযমন অলপকাচের মচেধয হিনঃচে(4 চেয় যায়-নয়নচোচের হিমারচের অসথা তাই চেয়হিছ এই কারচেরণই কথক নয়নচোচের হিমারচের গা ভরা আমবর সয করচেত পারতনা

Physics Dimensional Analysis (Summary)

Q Find the dimensions of consts ab in relation

p=(bminusxlowastx)at

where p is the power x is the distance and t is time

Ans From principle of homogeneity dimension of b x2 are same Dim of b = dim of x2 = [L2] = [ML2T0]Dim of a = dim of ( b- x2)dim of (pt) = [M0L2T0][ML2T-2] [T-1] [T] = [M-1L0T2]

Chemistry Atomic Structure Drawbacks of Rutherfordrsquos model of

atom a According to Rutherfordrsquos model of atom electrons which are negativelycharged particles revolve around the nucleus in fixed orbits Thusb theelectrons undergo acceleration According to electromagnetic theory of Maxwell a charged particle undergoing acceleration should emitelectromagnetic radiation Thus an electron in an orbit should emitradiation Thus the orbit should shrink But this does not happenc The model does not give any information about how electrons aredistributed around nucleus and what are energies of these electrons Isotopes These are the atoms of the same

Properties of electromagnetic radiationsa Oscillating electric and magnetic field are produced by oscillating charged particles These fields are perpendicular to each other and both areperpendicular to the direction of propagation of the waveb They do not need a medium to travel That means they can even travel invacuum

Characteristics of electromagnetic radiationsa Wavelength It may be defined as the distance between two neighbouring crests or troughs of

element having the same atomicnumber but different mass numbere g 1H11H21H3

Isobars Isobars are the atoms of different elements having the same massnumber but different atomic numbere g 18Ar40 20Ca40

Isoelectronic species These are those species which have the same numberof electrons

Electromagnetic radiationsThe radiations which are associated withelectrical and magnetic fields are called electromagnetic radiations When anelectrically charged particle moves under acceleration alternating electricaland magnetic fields are produced and transmitted These fields aretransmitted in the form of waves These waves are called electromagneticwaves or electromagnetic radiations

wave as shown It is denoted by λb Frequency (ν) It may be defined as the number of waves which passthrough a particular point in one secondc Velocity (v) It is defined as the distance travelled by a wave in onesecond In vacuum all types of electromagnetic radiations travel with thesame velocity Its value is 3 times10 8m sec-1 It is denoted by v

d Wave number Wave number is defined as the number of wavelengths per unit lengthVelocity = frequency timeswavelength c = νλ

Plancks Quantum Theory- o The radiant energy is emitted or absorbed not continuously but discontinuously in the form of small discrete packets of energy called lsquoquantumrsquo In case of light the quantum of energy is called a lsquophotonrsquoo The energy of each quantum is directly proportional to the frequency of the radiation ie E α υ or E= hυ where h= Planckrsquos constant = 6626 x 10-27 Js o Energy is always emitted or absorbed as integral multiple of this uantum E=nhυ Where n=1234Black body An ideal body which emits and absorbs all frequencies is calleda black body The radiation emitted by such a body is called black body radiation

Photoelectric effectThe phenomenon of ejection of electrons from thesurface of metal when light of suitable frequency strikes it is calledphotoelectric effect The ejected electrons are called photoelectrons

Biology Chapter - 02Systematics and Five Kingdoms

Scientists divide the whole living organisms into two kingdom first and ultimately by five kingdom at last

In the earlier systems of classifications organisms are divided into kingdom plantaeand kingdom animalia on the of presenceof cell wall their modes of nutrition and movements

Some problem arise like fungi share manycharacteristic withplant despite their heterotrophic nutrition bacteria protozoa areunicellular present in both kingdom Toovercome this third kingdom Protista isintroduced which include

unicellularorganisms But there is also another

problem Allunicellular organisms are not similar kind The cellular structure of prokaryotes is verydifferent from that of other organismsEukaryotes possess a true nucleus and allcell organelles that are not present inprokaryotes So the fourth kingdom Monerais introduced which include unicellular prokaryotes (bacteriaamp blue green algae)

bull Still some problem arise in kingdomplantae

So in 1969 R H Whittakar proposedanew five kingdom System of classification

i) Kingdom Monera - unicellular prokaryotes

ii) kingdom Protista - unicellular eukaryotes

iii) Kingdom Fungi - uni or multicellular fungi with cell wall but without chlorophyll

iv) Kingdom Plantae - Multicellular Plants

v) Kingdom Animalia - Multicellular Animals

EVS Chapter 1 ndash Modes of Existence

An agricultural society

An agricultural society also known as an agrarian society is a society that constructs social order around a reliance upon farming More than half the people living in that society make their living by farming

People in an agricultural society generally lead a more settled lifestyle than those in nomadic hunter-gatherer or semi-nomadic pastoral societies because they live permanently near the land that is farmed Agricultural settlements tend to develop in areas of convenience near bodies of water which is used for both crops and transportation or along trade routes Not everyone in an agricultural society is a farmer Some people make a living trading or making and selling goods such as tools used for farming

Another way to define an agrarian society is to see the total amount of production in a nation In an agrarian society cultivating the land is the main source of wealth Such a society can recognize other means of subsistence and work habits but emphasizes the importance of agriculture and livestock Agrarian societies have existed in various parts of the world for 10000 years and continue to exist today They have been the most common form of socio-economic organization for most of recorded human history

Q) Write the features of agricultural society

Ans - Structure and Features of Agrarian Society1 Occupational Structure

An agrarian society is generally associated with the domestication of plants and animals The domestication of plants means farming and that of animals means herding Often there is mixture of farming and the use of such domesticated animals as cow goat and sheep

2 Forms of Land Ownership in Agrarian SocietiesGenerally there are landlords supervisory farmers cultivators and share croppers The landholders own the land but do not work on it They let it out for sharecropping The supervisory farmers are those who live by having their land cultivated by hired labourers The cultivators cultivate the land for themselvesThe share-croppers are those who live by tilling other peoplersquos land or a crop-sharing basis The artisans own their means of production and produce by their own labour in their homesteads

3 Village Community System An agrarian society is highlighted by

the institution of village community system The agrarian economy made fixed dwelling houses necessary Living close together for protection and co-operation and living nearer to the land gave birth to agricultural villages The village is not only the residential place of farmers it is also the social integrator

4 Minimal Division of Labour Another structural feature of agrarian society is a minimal division of labour Except for the basic division founded on age and sex differences there are few specialized roles There is only one predominant type of occupation ie domestication of plants and animals For all the people the environment physical as well as social is the same

5 Role of Family The farm family is of the patriarchal type the father is the final arbiter in most of the familyrsquos major decisions The life of ail men and women is merged in family life Since there are not many special organizations family is the only organisation to perform the tasks of aid and protection

6 Sense of Unity The members of an agrarian society exhibit a strong in-group feeling Since the whole of their social lives is wrapped up in a society which is physically economically and socially homogenous they are inclined to view the entire outside world as an out group

7 Informal Social Control An agrarian society is regionally divided into villages In a village community the force of traditional mores is more dominant than in the urban community In the village everybody is known to everybody The members in a village community help each other and share the joy and sorrows of each other Crime in an agrarian society is rare

8 Simplicity and Uniformity Life of the people in an agrarian society is marked by simplicity and uniformity Their main occupation is agriculture which largely depends upon the vagaries of nature An agrarian society is a religious society

Math Compound angles Compound angles The algebraic sum of two or more angles is called a compound angle If A B C be three angles then A+B B+C C+A A-B B-C A-C A+B-C etc are compound angles In this chapter we shall discuss the trigonometrical ratios of compound angles Theorem 1 If A B and A+B are all pisitive acute angles theni) sin( A+B) = sin A cos B + cosA sinBii) cos(A+B) = cosA cosB- sinA sinBTheorem 2If A and B are positive acute angles and AgtB theni) sin(A-B) = sin A cosB- cos A sinBii) cos(A-B) = cos A cos B+ sin A sin BTo prove that i) sin(A+B) sin (A-B) = sin2 A - sin2 B = cos2 B- cos2 A

Example 1 Prove that tan70deg=2tan50deg+tan20degSolutiontan70deg = tan(50deg + 20deg)Or tan70deg=(tan 50deg+tan 20deg)(1-tan50degtan20deg) or tan70deg (1 ndash tan 50deg tan20deg) = tan50deg+tan20degor tan70deg= tan70deg tan50deg tan20deg+ tan50deg + tan20deg = cot20deg tan50deg tan20deg + tan50deg + tan20deg = 2 tan50deg+ tan20degExample 2 If A + B = 45deg show that (1 + tanA) (1 + tanB) = 2Solutiontan(A + B) =( tan A + tan B) (1 - tan

ii) cos(A+B) Cos(A-B) = cos2 A- sin2 B = cos2 B -sin2 AProof i) LHS= sin(A+B)sin(AminusB) [Recall sin(αminusβ)=sinαcosβminuscosαsinβ And sin(α+β)=sinαcosβ+cosαsinβ]= (sinAcosB+cosAsinB)times(sinAcosBminuscosAsinB)= sin2Acos2Bminuscos2Asin2B [Recall sin2α+cos2α=1 From above we can then assume correctly that sin2α=1minuscos2α AND cos2α=1minussin2α] = sin2A(1minussin2B)minussin2B(1minussin2A) = sin2Aminussin2Asin2Bminussin2B+sin2Asin2B = sin2Aminussin2B= 1-cos2A-(1-cos2B) = cos2 B- cos2 A = RHSii)LHS= cos (A+B) cos(A-B) [ cos(A+B) = cos AcosB- sinAsinBCos(A-B) = cosAcosB+ sinAsinB]= cos2 A Cos2 B- sin2 A Sin2 B= cos2 A( 1-sin2 B) - (1- cos2 A) sin2 B= cos2 A- cos2 A sin2 B- sin2 B+ cos2 A sin2 B=cos2 A- sin2 B=1- sin2 A-(1-cos2 B) = cos2 B- sin2 A= RHSTangent formulae for compound anglesi)tan (A + B) = tan A + tan B1-tan A tan Bii)tan (A ndash B) = tan A-tan B1+tan A tan Biii) cot (A + B) = cot Acot B-1cot A+cot B(viii) cot (A ndash B) = cot Acot B+1cot B-cot A

A tan B) Or 1= (tan A+ tanB) (1-tan A tanB) Or tanA + tanB + tanA tanB + 1 = 1 + 1Or tanA (1 + tanB) + (1 + tanB) = 2Or (1 + tanA) (1 + tanB) = 2Example 3 Find the value of sin 15degSolution sin 15deg= sin(45deg-30deg) = sin45degcos 30deg- cos45degsin30deg =(1radic2) (radic32) -(1radic2) (12) = (radic3-1) 2radic2Example 4 If sin A = 1 radic10 and sin B = 1 radic5 where A and B are positive acute angles then what is A + B SolutionWe know that sin (A + B) = sin A cos B + cos A sin B= [1 radic10] [radic(1 minus 1 5)] + [1 radic5] radic(1 minus 1 10)= [1 radic10] [radic4 5] + [1 radic5] [radic9 10]= [1 radic50] times (2 + 3)= 5 radic50 = 1 radic2

sin (A + B) = sin π 4rArrHence A + B = π 4Example 5 If A + B = 225o then find [cot A] [1 + cotA] times [cot B] [1 + cot B]Solution[cot A] [1 + cotA] times [cot B] [1 + cot B] = 1 [(1 + tan A) times (1 + tan B)]=1 [tan A + tan B + 1 + tan A tan B] [ tan (A + B) = tan225o]∵

tan A + tan B = 1minus tan A tan BrArr= 1 [1 minus tan A tan B + 1 + tan A tan B]= 1 2

COMMERCE

CLASSIFICTION OF HUMAN ACTIVITIES-ECONOMIC AND NON-ECONOMIC

Firstly we shall recall the previous class for 5 mins especially for the absentees and for also the rest of the students who were there

Today at first we briefly discuss the earlier portions of the chapter

1Business-It includes all those economic activities which are concerned with production and exchange of goods and services with the object of earning profit Example A factory shop beauty parlour also business enterprises

2Profession ndashThe term profession means an occupation which involves application of specialized knowledge and skills to earn a living For Example Chartered Accountancy medicine law tax consultancy are example of professions

Questions1What are the main features of ProfessionAnswer The main features of a profession are as follows a Specialised body of knowledge-Every profession has a specialised and systematised body of knowledge b Restricted entry- Entry to a profession is allowed only to those who have completed the prescribed education and have the specialised examination c Formal education and training ndashA formal education and training is given to the person who wants to acquire the professional

3Employment-Employment mean an economic activity where people work for others in exchange for some remuneration (salary)The persons who work for others are called lsquoemployeesrsquo The persons or organizations which engage others to work for them are called lsquoemployersrsquoEg A doctor working in a hospital is employment as he is working for a salaryA lawyer may serve as a law officer in a bank

With this we shall proceed with the features of both Profession amp Employment

The main features of a profession are as follow

a Specialised body of knowledge b Restricted entry c Formal education and training d Professional association e Service motive f Code of contact

The main features of an employment are as follows

a In employment a person works for others called employer

b An employee provides personal service

c There is a service agreement or contract between the employee and the employer

d The employee has to obey the order of the employer

e No capital investment is made by the employer

Various examples of Employment are as follows

aA teacher teaching in a school or collegeb An engineer employed in Municipal Corporation of DelhicAn accountant working in the accounts department of a companydA doctor working in a hospital

Note In all the above examples of employment the individual who is involved in each example is working as an employee for a salary under an employer

qualification(MBBSCALLB)d Service motive ndashProfessionals are expected to emphasis service more on their clients rather than economic gain f Code of Conduct-The activities of professionals are regulated by a code of conduct

2 What are the main features of EmploymentAnswer The main features of an employment are as followsa In employment a person works for others called employerb An employee provides personal servicec There is a service agreement or contract between the employee and the employerd The employee has to obey the order of the employere No capital investment is made by the employer

3 Give various Professions and their respective Association are given below

Professions

Professional

Professional association

Medical profession

Doctor Medical Council of India

Law profession

Lawyers Bar Council of India

Accounting Profession

Chartered

The Institute of Chartered Accounts of India( ICAI)

Engineerin Engineers The

g Profession

institute of Engineers (India)

Accounts Basic accounting terms

Today we will give you some questions from the previous study material

Questions6) Define accounting7) What do you mean by debit

and credit8) Explain the types of account9) Define the following terms

a) Assetsb) Capitalc) Purchased) Debtorse) Transactions

10) Name the types of accounts given below

a) Krishnas accountb) Machinery accountc) Royalty accountd) Salary accounte) Furniture accountf) Audit fee account

Economics Basic Economic ConceptsSub topic

UTILITY

Before starting todayrsquos class we shall recall the last class which was about UTILITY AND THE FEATURES OF UTILITY

Now we shall proceed with the further topics of the chapter

Todayrsquos topic from the chapter lsquo Basic Economic Conceptsrsquo will be TOTAL UTILITY amp MARGINAL UTILITYNow let us quickly revise the concept of utility with an example ie goods and services are designed because they have an ability to satisfy human wantsThis feature of being able to satisfy human wants is termed as utility For example we derive utility from WiFi services as it gives us satisfaction by connecting us to our friends and family through social media here consumers derive utility from WiFi services

From the above concept we shall start with todayrsquos topicEconomists have defined TOTAL UTILITY (TU) as the total satisfaction obtained by consuming a given total amount of a good and serviceFor example the total satisfaction obtained from eating 10 mangoes is the total utility of 10 mangoes

MARGINAL UTILITY (MU) is the additional satisfaction derived from each additional unit

Questions1 What is Total Utility (TU)

Answer Total Utility (TU) is the

aggregate of the utility that a consumer derives from the consumption of a certain amount of a commodityTU=MU1+MU2++MUn

2 What is Marginal UtilityAnswer

Marginal Utility (MU) is the additional made to the total utility as consumption is increased by one more unit of the commodityMU= TUn ndashTUn-1

NoteOften economists tend to

subdivide utility into an imaginary unit called UTIL

consumed In this casethe utility obtained from each mango as it is consumed as the MU of that mango It is also defined as the addition made to the total utility when an additional unit is consumed Often economists tend to subdivide utility into an imaginary unit called UTIL

Note As a consumer increases the consumption of a good over period of time the total utility or total satisfaction derived from it increases to appoint and thereafter it decreasesHowever as the consumer keeps on consuming the good the marginal utility or the additional utility derived from it decreases

SubjectBusiness studies

Topic

BUSINESSENVIRONMENT

Summary

Now quickly let us revise the earlier points that we have already done in the last class and let us proceed with the other topics that are there in the chapter

Firstly we will recall the internal and external factors of micro environment and then we shall proceed in details

Meaning and list of internal and external factors

aInternal factorsInternal factors refer to all the factors existing within a business firm The internal factors are considered controllable because the enterprise has control over these factorsFor an example a company can alter its organization structure policies programmes employees physical facilities and marketing mix to suit the changes in the environmentList of internal factors areCorporate culture mission and objectives top management organizations structure company image and brand equity company resources

b External factorsExternal factors refer to those individual and groups and agencies with which a particular business organization comes into direct and frequent contact in the course of its functioningThese individuals and groups are known as STAKEHOLDERS because they have a stake (financial interest ) in the working and performance of the particular business List of external forces (stakeholders)Customers competitors investors suppliersmiddlemen (marketing intermediaries)

Execution 1 What do you mean by internal

factors in micro environmentAnswerInternal factors refer to all the factors existing within a business firm The internal factors are considered controllable because the enterprise has control over these factorsFor an example a company can alter its organization structure policies programmes employees physical facilities and marketing mix to suit the changes in the environment

2 What do you mean by external factors in micro environment

AnswerExternal factors refer to those individual and groups and agencies with which a particular business organization comes into direct and frequent contact in the course of its functioningThese individuals and groups are known as STAKEHOLDERS because they have a stake (financial interest) in the working and performance of the particular business

3Who are stakeholdersSTAKEHOLDERS are individuals and groups who have a stake (financial interest ) in the working and performance of the particular business 4Discuss the internal factors in briefa Corporate CultureThe values beliefs and attitudes of the founders and top management of the company exercise

financers publics

customers

suppliersfinancers

competitors

middlemen

publics

Fig STAKEHOLDERS OF A COMPANY

Apart from micro environment the other main dimension of business environment isMacro environment Macro environment refers to the general environment or remote environment within which a business firm and forces in its micro environment operateA company does not directly or regularly interact with the micro environmentTherefore macro environment is also known as indirect action EnvironmentThe macro environment forces are less controllable than the micro forces

Macro environment consists of the following components

POLITICAL AND LEGAL ENVIRONMENT

ECONOMIC SOCIAL AND ENVIRONMENT

CULTURAL

ENVIRONMENT

TECHNOLOGICAL ENVIRONMENT

a strong influence on what the cmpaany stands for how it does things and what it considers importantbMission and objectivesThe business philosophy and purpose of a comoany guide it prioritiesbusiness strategiesproduct market scope and development scope

cTop management structurethe composition of board of directors the degree of professionalization of management and the organizational structure of a company have important bearing on its business decisions

dPower structureThe internal power relationship between the board of directors and the chief executive is an important factor

eCompany image and brand equityThe image and brand equity of the company play a significant role in raising finance forming alliance choosing dealers and suppliers launching new products entering foreign markets

5 What is Macro environmentAnswerMacro environment refers to the general environment or remote environment within which a business firm and forces in its micro environment operateA company does not directly or regularly interact with the micro environmentTherefore macro environment is also known as indirect action EnvironmentThe macro environment forces are less controllable than the micro forces 6 What are the components of macro environmenta Political and legal environmentb Economic environmentc Social and cultural environmentd Technological environment

BUSINESS FIRM

Fig COMPONENTS OF MACRO ENVIRONMENTPolitical science

Introduction to political science

Comparative politics and itrsquos scope Comparative politics is the second major dimension of political scienceIt is also a very vast area of study and a very large number of political scientists even treat it as an autonomous area of study within the board ambit of political scienceScope of comparative politics-

1 All political structures -Comparative politics includes the study of all structures formalnon formal governmental and extra governmental which are directly or indirectly involved in politics in all the countries of the world

2 Functional studies- Comparative politics seeks to study politics less from the point of view of the legal institutions in terms of their powers and move from the point of view of their functions which constitute the political process and their actual Operation in the environment

3 Study of political behaviour- Another important part of its scope is the study of the actual behaviour of the people in the process of politics

4 Study of similarities and differences- comparative politics also undertakesan analysis of the similarities and differences among political process and functions

5 Study of all political systems -comparative politics seeks to analyse the actual behaviour and performance of all political systems western as well as non western

6 Study of the environment and infrastructure of politics-The study of politics demands a study of the psychological sociological economic and anthropological environment in fact the social environment as a whole in which each political system operates

7 Study of political culture- political culture is composed of attitudesbeliefs emotions and values of a society that relate to the political system or politics

8 Study of political participation- Political participation is a universal processThe only difference is that while in some states it is limited in others it is wider

9 Study of political process- political

Answer the following questions-

What is comparative politics

What are the scope of comparative politics

Homework- learn

processes like decision makingpolicy making judicial process leadership recruitment process and others are always at work in all political systems

The scope of comparative politics is very comprehensive It includes everything that falls within the area of political activity and political process

History CAMBRIDGE VIEW ABOUT

THE PARTITION

AND REFUTATION

OF CAMBRIDGE

VIEW

Cambridge view about the Partition The Cambridge school of historians have interpreted that opposition to partition scheme was made entirely by the elitist groups They hold the view that Lord Curzon planned to partition the Bengal for administrative purposeREFUTATION OFCAMBRIDGE VIEW The Rationalist historians have rejected the interpretations of the Cambridge School of historians on various grounds

1 QUESTION State different views of historians regarding Partition of Bengal

ANSWER Cambridge historians believed that Lord Curzon partitioned Bengal for administrative reasons only and not for the political motive The Middle class elitist group protested because of their petty interest The Hindu zamindars protested as they have to spend more money for managing their estatesThe lawyers of Calcutta High court feared to lose their clientBut according to the nationalist Historians was-

2- The ultimate object of Lord Curzon was to crush the unity of Bengal politicians

3- If Bengal becomes a separate province Bengali speaking 16 million people of western part would become minority under Hindi speaking people of Bihar and Oriya speaking people of Orissa

4- The bureaucrats expected that the protest movement would die down quickly

5- Lord Curzon used the Muslim community in his political game

6- Idealism had great contribution in the protest against partition

7- The people of the every section of society were affected by the partition of Bengal

Computer Science

Numbers Convertion of dcimal number to octal numberThe decimal numeral system is the standard system for denoting integer and non-integer numbers It is the extension to non-integer numbers of the Hindu-Arabic numeral system For writing numbers the decimal system uses ten decimal digits a decimal mark and for negative numbers a minus sign - The decimal digits are 0 1 2 3 4 5 6 7 8 9 the decimal separator is the dot in many countries

The octal numeral system or oct for short is the base-8 number system and uses the digits 0 to 7 Octal is sometimes used in computing instead of hexadecimal perhaps most often in modern times in conjunction with file

permissions under Unix systems It has the advantage of not requiring any extra symbols as digits It is also used for digital displays

Follow these steps to convert a decimal number into octal form

1 Divide the decimal number by 82 Get the integer quotient for the next iteration (if the number will not divide equally by 8 then round down the

result to the nearest whole number)3 Keep a note of the remainder it should be between 0 and 74 Repeat the steps until the quotient is equal to 05 Write out all the remainders from bottom to top This is the solution

For example if the given decimal number is 8453

Division Quotient Remainder

8453 8 1056 5

1056 8 132 0

132 8 16 4

16 8 2 0

2 8 0 2

Then the octal solution is 20405

Subject Eng Literature (The Tempest ndash William Shakespeare) Topic Act I Scene 1 Lines 33 to 67 (End of scene) Date 16th April 2020 (4th Period)

[Students should read the original play and also the paraphrase given in the school prescribed textbook]Summary Questions amp Answers

[SUMMARY OF THE ENTIRE SCENE]

o The play starts with the scene of a severe storm at sea Alonso (King of Naples) Sebastian (Alonsorsquos brother) Ferdinand (Alonsorsquos son) Gonzalo Antonio (the usurping Duke of Milan) are in a ship in the midst of the storm

o The mariners are trying their best to control the vessel from running aground and are totally following the orders of their Master the Boatswain They have scant success

o The mariners become extremely unhappy and annoyed when most of the passengers arrive on the deck thereby hampering their effort to save the ship There is serious confrontation between them and the passengers who are part of the Kingrsquos entourage

o The mariners could not save the ship

SUMMING-UP

(i) Vivid description of the scene which gives a realistic description of terror and confusion of a tropical storm

(ii) Shows Shakespearersquos accuracy of knowledge in describing the naval operations and also matters of seamanship

(1) GONZALO Ill warrant him for drowning (L 45-57)

though the ship were no stronger than a nutshell and as leaky as an unstanched

wenchBOATSWAIN Lay her a-hold a-hold Set her two courses Off to

sea again lay her offMARINERS All lost To prayers to prayers All lostBOATSWAIN What must our mouths be coldGONZALO The king and prince at prayers Lets assist them

For our case is theirsSEBASTIAN Im out of patienceANTONIO We are merely cheated of our lives by drunkards

This wide-chopped rascal - would thou mightst lie drowning the washing of ten tides

(a) What does Antonio say at the insolent manners of the boatswain just before the given passage

Being irritated at the insolent manners of the boatswain just before the given extract Antonio the Duke of Milan calls him a worthless dog son of a woman without any morals an arrogant and disrespectful noisemaker He says that the boatswain deserved to be hanged(b) What statement does Gonzalo repeat about the boatswain

Gonzalo shows his faith that the boatswain is not destined to die by drowning He is destined to be hanged and nothing can alter this decree of destiny He says that even if the ship was as frail as a nutshell the boatswain could not be drowned for his destiny was to be hanged(c) What do the passengers do when they have lost all hope of their survival

When the passengers have lost all hope of survival they take

(iii) The opening scene justifies the title ndash The Tempest

UNANSWERED QUESTIONS

(i) The King always travels with his entire fleet including his soldiers Where were the other ships

(ii) Why was the ship in that area Where was it coming from or going where

(iii) The ship broke apart What happened to those who were in the ship

(We shall get the answer to the above questions as the play progresses)

leave of life with fervent prayers The mariners take their last hearty drink and are ready for death(d) What blame does Antonio put upon the mariners and the boatswain Antonio rebukes the mariners that these drunkards have brought them to the present crisis by neglecting their duties He blames them saying that they are going to lose their lives entirely for the negligence of the boatswain and his fellows(e) What does Antonio say while cursing the boatswain

Antonio gives vent to his wrath upon the boatswain in particular He calls the boatswain a wide-mouthed rascal who deserves to be hanged on the sea-shore at low water mark so that ten tides might wash over his body and take out of him all the liquor that he has been drinking

Class XIISubject Topic Summary ExecutionHistory Topic

1 1935 ACT AND WORKING OF PROVINCIAL AUTONOMYCONGREE AND OTHER MINISTERSSUB TOPIC GOVERNMENT OF INDIA ACT1935

Government of India Act 1935 This act established a lsquoFederation of Indiarsquo made of British Indian provinces and Indian states and provided for autonomy with a government responsible to the elected legislature in every provinceThis act introduced abolition of Diarchy at provinces The entire provincial administration was introduced to the responsible ministers who were controlled and removed by the provincial legislature The provincial autonomy means two things First The provincial governments were wholly responsible to the provincial legislature Secondly Provinces were free from outside control and interference in the large number of matters The act divided the powers between the centre and provinces in terms of three lists- Federal list( for centre) Provincial list (for province) and concurrent list (for both) Residuary powers were given to the viceroy In the election under the government of India Act the Congress swept the poll the mandate of the people came in favour of the congress so far as general Hindu seats were concerned The Congress did not get a single Muslim seates in Bombay CP UP Sind and BengalIn five provinces Congress had yhe clear majority In BengalNWFPAssam and Bombay Congress emerged as a single largest partyOn the other side the performance of the Muslim League was badThus the Congress formed ministers in 7 provinces out of 11 provinces Coalition ministry was also formed in two other provincesOnly BENGAL AND Punjab had non- congress ministries

1 QUESTION What was the main change introduced by the Government of India ActANSWER a) The Act gave more

autonomy to the provinces b) Diarchy was abolished at the

provincial levelsc) The Governor was the head of

the executived) There was a council of

ministers to advise him The ministers were responsible to the provincial legislatures who controlled them The legislature could also remove the ministers

e) The Governors still retained special reserve powers

2 QUESTION Why did the federal scheme introduced by the Government of India Act 1935 never come into operation

ANSWER The Federal structure of the Government of India was to be composed with the Governor General and Council of ministers The Federal legislature was to be Bicameral legislature- The council of states and the House of Assembly The ministers were to be chosen by the Governor general and they were to hold the office during his pleasure

The provinces of British India would have to join the federation but this was not compulsory for the princely states

This federation never materialised because of the lack of support from the required number of

princely statesThis act was refused and

rejected by the princes the Congress and the Muslim League

Thus both Congress and the League participated in the election of 1937 Thus the federal part was never introduced but the provincial part was put into operations

Bengali 2nd

Language

াচেরর পরাথCনা(কহিতা )

াচেরর পরাথCনা কহিতাটি কহি (ঙখ দেঘাচে4র দো আচো য কহিতায় াচেরর পতর হমায়ন কঠিন দেরাচেগ আxানত ার ঈশবর া আললার কাচেছ পরাথCনা কচেরচেছন তার পচেতরর ীন হিফহিরচেয় হিচেত এই কহিতায় ার পচেতরর ীন হিভbা দেচেয়চেছন ারার এমনহিক হিনচের ীন হিসCচেনর হিহিনমচেয় হিতহিন তার দেছচের ীন হিফচের দেপচেত দেচেয়চেছন তার দেছচের এই দেরাচেগর ন য হিতহিন হিনচেচেকই ায়ী কচেরচেছন তার হিনচের করা পাপচেকই হিতহিন ায়ী কচেরচেছন এছাা রানৈনহিতক ও আথCসামাহিক অসথার কথা তচে ধরা চেয়চেছ এই কহিতায় ার তার হিনচের পাপ কমCচেকই ায়ী কচেরচেছ ার অন যায় ভাচে দেপহি((হিকতর মাধ যচেম অপররা য কচেরচেছ আর এই অন যায় কাচের ন যই তার পহিরাচের হিপযCয় এচেসচেছ দে এক পরকার মানহিক নধন ইহিতাচেসর ার হিপতা চেয় সবাভাহিকভাচে ভাচোাসা দে মমতা দেথচেক মকত চেত পাচেরনহিন তাই হিপতা চেয় আললা া ভগাচেনর কাচেছ পতর হমায়চেনর পরানহিভbা দেচেয়চেছন ার আললা া ভগাচেনর কাচেছ াহিনচেয়চেছন তার হিনচের ীন হিসCন হিচেত হিতহিন রাী তার হিহিনমচেয় পচেতরর ীন হিফচের দেপচেত দেচেয়চেছন াচেরর হিপতসভ হিচেকর কথা এই কহিতায় ফটিচেয় দেতাা চেয়চেছ হিপতা পচেতরর হিরাহিরত মান নধচেনর কথা তচে ধরা চেয়চেছ

হিচে(4 হিকছ াইচেনর তাৎপযC১) ldquoদেকাথায় দেগ ওর সবচছয দেৌন দেকাথায় কচেরায় দেগাপন bয়ldquoউততর) াচেরর পতর হমায়ন কঠিন দেরাচেগ অসসথ তাই তার দেযৌন াহিরচেয় যাচেচছ এই দেরাচেগ তাচেক দেগাপচেন কচেরকচের াচেচছ তার সক (হিকত ধীচের ধীচের bয় চেচছ তাই হিপতা চেয় ার আললার কাচেছ হমায়চেনর পরান হিভbা দেচেয়চেছন২) ldquoাগাও (চেরর পরাচেনত পরানতচের ধসর (ন দেযর আান গানldquoউততর) াচেরর পতর হমায়ন কঠিন দেরাচেগ আxানত তাই ার আ দে(াচেক মমCাত (চেরর পচেথ পরানতচের আান গান ধবহিনত দোক দেসই আান গান আললার কাচেছ দেযন চে যায় আললা দেযন এই আহিতC শচেন পচেতরর ীন হিফহিরচেয় দেয় ৩)ldquoনাহিক এই (রীচেরর পাচেপর ীানচেত দেকানই তরারণ দেনই ভহি4চেতরldquoউততর) হমায়চেনর অসসথতার ন য ার হিনচেচেকই ায়ী কচেরচেছন কারন ার অচেনক রা য অন যায় ভাচে কচেরচেছ তাই তার এই পাপ কাচের ন য তার ঘচের আ হিপ এচেসচেছ এই অন যায় কাচের ন য তার মহিকত দেনই তাই ার আললার কাচেছ এই পাপ কাচেযCর ন য bমা পরাথM

Hindi 2ndlang

-ासी(जयशकर परसा-)

-ासी जयशकर परसा- की एक ऐसी कहानी ह जिजसम भारतीय ससकनित और राषटरीयता का सवरगजीतहोता ह इस कहानी म इरावती एक निहद कनया ह जिजस मलअचछो न मलतान की लट म पकडा और -ासी बना दि-या उस 500 दि-न -कर काशी क एक महाजन न खरी-ा दसरी -ासी निफरोजा ह वह गलाम ह निफरोजा को छडान क कतिलए अहम- को 1000 सोन क कतिसकक भजन थ जो अभी तक नही आए थ राजा साहब कठोर होत हए भी निफरोजा को निबना धनराकतिश क कतिलए उस म कर -त ह वनिफरोजा को अहम- को समझान की बात कहत हकहानी क अत म हम -खत ह निक इरा वती और जाटो क सर-ार बलराज का मिमलन होता हअहम- को यa म मार दि-या जाता ह वहा निफरोजा की परसननता की समामिध बनती ह वहा एक फल चढती ह और डीजल आती ह निफरोजा उस समामिध की आजीवन -ासी बनी रहती हलखक अपन उददशय अथात -ास परथा पर परकाश डालन और इस परथा क कारण होन वाल -ातो क दखो को दि-खान म पणता सफल हए ह

helliphellipContinue to next

Biology Reproductio Today we will discuss about vegetative Q1 Name some vegetative propagules

n in Organisms

propagation of plants The process of multiplication in which fragments of plant body function as propagule and develop into new individual is called vegetative propagation The units of such propagation are runner rhizome tuber bulb etc

and the speciesinvolvedVegetative propagules

Parts involved

Bulb StemBulbil BulbilRhizome Stem Runner Stem Tuber Stem Offset Stem Leaf buds Leaves Suckers Stem

Corns Stem stolon

Q2 State advantages of vegetative propagation

i) Rapid methodii) Sure and easy methodiii) Useful in plants that cannot

produce viable seeds or long seed dormancy

iv) Maintains purity of raceQ 3 Banana fruit is said to be parthenocarpic where as turkey is said to be parthenogenetic WhyBanana develops without fertilization from an unfertilized ovary thus is parthenocarpicIn turkey the ovum or female gamete developinto a new chick without fertilization thus isparthgenetic

Q4 Why is water hyacinth is called as a ldquoTerror of Bengalrdquo Water hyacinth can

propagatevegetatively all over the water body in a short per short period of time This resulted increased biochemicaloxygen oxygen demand of water body causing mortalityof fishes It is very difficult to get rid off them Thus known as terror of Bengal

Chemistry

Solid state GENERAL CHARACTERISTICS OF SOLID STATEIn nature the particular state of matter is governed by two opposing forces at given set of temperature and pressure These forces are intermolecular force of attraction and thermal energy If intermolecular force of attraction is high as compared to thermal energy particles remains in closest position

Intext QuestionsQ1 Classify the following solids as crystalline and amorphous Sodium chloride quartz glass quartz rubber polyvinyl chloride Teflon

A1 Crystalline

and hence very less movement in particles is observed In this case solid state is the preferred state of matter

Let us revise the general characteristics of solid

i) Fixed mass volume and shape

ii) Strong intermolecular force of attraction

iii) Least intermolecular space

iv) Fixed position of constituent particles

v) Incompressible and rigid

Q2 what type of interactions hold the molecules together in a polar molecular solid[CBSE 2010]A2 The molecules in a solid are held together by van der Waals forces The term van der Waals forces include hydrogen bonding dipole-dipole attraction and London dispersion forces All molecules experience London dispersion forces In addition polar molecules can also experience dipole-dipole interactions So the interactions that holds the molecule together in polar molecular solid are London dispersion force and dipole-dipole interactionsQ3 Write a feature that will distinguish a metallic solid from an ionic solid [CBSE 2010]A3 Metals are malleable and ductile whereas ionic solid are hard and brittle Metallic solid has typical metallic lustre But ionic solid looks dullQ4 Write a point of distinction between a metallic solid and an ionic solid other than metallic lustre [CBSE 2012]A4 Metals are malleable and ductile whereas ionic solid are hard and brittleQ5 Write a distinguish feature of metallic solid [CBSE 2010]A5 The force of attraction in

solid Sodium chloride Quartz Amorphous solid Quartz glass rubber polyvinyl chloride Teflon Q2 why glass is considered as super cooled liquidA2 Glass shows the tendency to flow at slower rate like liquid Hence they considered as super cooled liquidQ3 why the window glass of old buildings show milky appearance with timeA3 Glass is an amorphous solid Amorphous solid has the tendency to develop some crystalline character on heating Due to heating in day over the number of years glass acquires some crystalline character and show milky appearanceQ4 why the glass panes fixed to window or doors of old building become slightly thicker at bottomA4 Glass is super cooled liquid It has the tendency to flow down very slowly Due to this glass pane becomes thicker at the bottom over the timeQ5 Sodium chloride is a crystalline solid It shows the same value of refractive index along all the direction TrueFalse Give reasonA5 FalseCrystalline solid shows anisotropy in properties That is it shows different values for the given physical property in different direction All the crystalline solids show anisotropy in refractive index Therefore sodium chloride will show different values of refractive index on different directions

Q6 Crystalline solid are anisotropic in nature What does this statement means

between the constituent particles is special kind of electrostatic attraction That is the attraction of positively charged kernel with sea of delocalized electronsQ6 which group of solid is electrical conductor as well as malleable and ductile [CBSE 2013]A6 Metallic solidQ7 why graphite is good conductor of electricity although it is a network (covalent solid)A7 The exceptional property of graphite is due to its typical structure In graphite each carbon is covalently bonded with 3 atoms in same layer The fourth valence electron of each atom is free to move in between different layersThis free electron makes the graphite a good conductor of electricity

[CBSE 2011]A6 Anisotropy is defined asrdquo Difference in properties when measured along different axis or from different directionsrdquo Crystalline solid show different values of some of the physical properties like electrical resistance refractive index etcwhen measured along the different directions The anisotropy in crystalline solid arises due to the different arrangement of particles in different directions

Math Function Composition of functions Think of an industrial plant that produce bottles of cold drinks first there is the operation (or function) f that puts the cold drink inside the bottle followed by the opeartion g that close the bottle with the capThis leads to the following definitionDefinition Let f A rarr B and g B rarr C be two functions Then the composition of f and g denoted by gof is defined as the function gof A rarr C given by gof(x) = g(f (x)) forall x isinA

Definition A function f X rarr Y is defined to be invertible if there exists a function g Y rarr X such that gof = IX and fog = IY The function g is called the inverse of f and is denoted by f -1

Thus if f is invertible then f must be one-one and onto and conversely if f is one-one and onto then f must be invertible This fact significantly helps for proving a function f to be invertible by showing that f is one-one and onto specially when the actual inverse of f is not to be determined

Example 1 Let f 2 3 4 5 rarr 3 4 5 9 and g 3 4 5 9 rarr 7 11 15 be functions defined as f(2) = 3 f(3) = 4 f(4) = f(5) = 5 and g (3) = g (4) = 7 and g (5) = g (9) = 11 Find gofSolution We have gof(2) = g (f(2)) = g (3) = 7 gof(3) = g (f(3)) = g (4) = 7gof(4) = g (f(4)) = g (5) = 11 and gof(5) = g (5) = 11Example 2 Find gof and fog if f R rarr R and g R rarr R are given by f(x) = cos x and g (x) = 3x2 Show that gof ne fogSolution We have gof(x) = g(f(x))=g(cosx) = 3 (cos x)2

= 3 cos2 x Similarly fog(x)=f(g (x))= f(3x2)= cos (3x2) Note that 3cos2 x ne cos 3x2 for x = 0 Hence gof ne fogExample 3 Show that if f A rarr B and g B rarr C are onto then gof A rarr C is also ontoSolution Given an arbitrary element z isin C there exists a pre-image y of z under g such that g (y) = z since g is onto Further for y isin B there exists an element x in A with f(x) = y since f is onto Therefore gof(x) = g (f(x)) = g (y) = z showing that gof is onto Example 4 Let Y = n2 n isin N sub N Consider f N rarr Y as f(n) = n2 Show that

f is invertible Find the inverse of fSolution An arbitrary element y in Y is of the form n2 for some n isin N This implies that n =radicy This gives a function g Y rarr N defined by g (y) =radicy Nowgof (n) = g (n2)=radicn2 = n and fog (y) =f(radicy) = (radicy) 2 y which shows that gof=IN and fog= IY Hence f is invertible with f -1 = g

Political Science

Constitution of India-The Preamble

Summary

Objective of the state-To secure equality of status and of opportunity To promote fraternity among all the citizens To assure the dignity of the individuals and Unity and integrity of the nation

Justice-Justice stands for rule of law absence of arbitrariness and a system of equal rights freedom and opportunities for all in a society India seeks social economic and political justice to ensure equality to its citizens

Liberty-Liberty implies the absence of restraints or domination on the activities of an individual such as freedom from slavery serfdom imprisonment despotism etc The Preamble provides for the liberty of thought expression belief faith and worship

Equality-Equality means the absence of privileges or discrimination against any section of the society The Preamble provides for equality of status and opportunity to all the people of the country

Fraternity-The Preamble declares that fraternity has to assure two thingsmdashthe dignity of the individual and the unity and

Execution

Answer the following questions-

Short notes-1 Equality2 Fraternity3 Justice4 Liberty

Homework-Learn

integrity of the nation The word integrity has been added to the Preamble by the 42nd Constitutional Amendment (1976)

Business studies

Human resource management (chapter 1)

On the day of 1504 2020 I have discussed with you the managerial functions and procurement functions of HRM

Today weare going to discuss about the development function integration functions and maintenance function

Development functions-HRM improves the knowledge skills attitude and values of employees so that they the present and future jobs more effectively it includes

1) Development functions of HRM

a) Performance appraisal = It implies systematic evaluation of employees with respect to their performance on the job and their potential for development

b) Training =It is the process by which employees learn knowledge skills and attitudes to achieve organisational and personal goals

c) Executive development = It is the process of developing managerial talent through appropriate program

2) Integration functionsa) HRM reconcile the goals of

organisation with those of its members through integrating function

b) HRM tries to motivate employees to various financial and non financial incentives provided in job specification etc

3) Maintenance functiona) HRM promote and protect the

physical and mental health of employees by providing several types of benefits like housing medical aid etc

b) It Promote Social security measures to employees by providing provident fund pension gratuity maternity benefits

SubjectCOMMERCE

Topic

BUSINESSENVIRONMENT

Summary

Now quickly let us revise the earlier points that we have already done in the last class and let us proceed with the other topics that are there in the chapter

Firstly we will recall the internal and external factors of micro environment and then we

Execution 3 What do you mean by internal factors

in micro environmentAnswerInternal factors refer to all the factors existing within a business firm The internal factors are considered controllable because the enterprise has control over these factors

Development FunctionsPerformance AppraisalTrainingExecution Development

shall proceed in details

Meaning and list of internal and external factors

aInternal factorsInternal factors refer to all the factors existing within a business firm The internal factors are considered controllable because the enterprise has control over these factorsFor an example a company can alter its organization structure policies programmes employees physical facilities and marketing mix to suit the changes in the environmentList of internal factors areCorporate culture mission and objectives top management organizations structure company image and brand equity company resources

b External factorsExternal factors refer to those individual and groups and agencies with which a particular business organization comes into direct and frequent contact in the course of its functioningThese individuals and groups are known as STAKEHOLDERS because they have a stake (financial interest ) in the working and performance of the particular business List of external forces (stakeholders)Customers competitors investors suppliersmiddlemen (marketing intermediaries)financers publics

customers

suppliersfinancers

For an example a company can alter its organization structure policies programmes employees physical facilities and marketing mix to suit the changes in the environment

4 What do you mean by external factors in micro environment

AnswerExternal factors refer to those individual and groups and agencies with which a particular business organization comes into direct and frequent contact in the course of its functioningThese individuals and groups are known as STAKEHOLDERS because they have a stake (financial interest) in the working and performance of the particular business

3Who are stakeholdersSTAKEHOLDERS are individuals and groups who have a stake (financial interest ) in the working and performance of the particular business 4Discuss the internal factors in briefa Corporate CultureThe values beliefs and attitudes of the founders and top management of the company exercise a strong influence on what the cmpaany stands for how it does things and what it considers importantbMission and objectivesThe business philosophy and purpose of a comoany guide it prioritiesbusiness strategiesproduct market scope and development scope

cTop management structurethe composition of board of directors the degree of professionalization of management and the organizational structure of a company have important bearing on its business decisions

dPower structureThe internal power relationship between the board of directors and the chief executive is an important factor

e Company image and brand equityThe image and brand equity of the company play a significant role in raising finance forming alliance choosing dealers and suppliers launching new products entering foreign markets

5 What is Macro environmentAnswerMacro environment refers to the general

competitors

middlemen

publics

Fig STAKEHOLDERS OF A COMPANY

Apart from micro environment the other main dimension of business environment isMacro environment Macro environment refers to the general environment or remote environment within which a business firm and forces in its micro environment operateA company does not directly or regularly interact with the micro environmentTherefore macro environment is also known as indirect action EnvironmentThe macro environment forces are less controllable than the micro forces

Macro environment consists of the following components

POLITICAL AND LEGAL ENVIRONMENT

ECONOMIC SOCIAL AND ENVIRONMENT

CULTURAL

ENVIRONMENT

TECHNOLOGICAL ENVIRONMENT

Fig COMPONENTS OF MACRO ENVIRONMENT

environment or remote environment within which a business firm and forces in its micro environment operateA company does not directly or regularly interact with the micro environmentTherefore macro environment is also known as indirect action EnvironmentThe macro environment forces are less controllable than the micro forces 6 What are the components of macro environmenta Political and legal environmentb Economic environmentc Social and cultural environmentd Technological environment

Computer Science

Logic gates

Digital systems are said to be constructed by using logic gates These gates are the AND OR NOT NAND NOR EXOR and EXNOR

BUSINESS FIRM

gates The basic operations are described below with the aid of truth tables

AND gate

The AND gate is an electronic circuit that gives a high output (1) only if all its inputs are high A dot () is used to show the AND operation ie AB Bear in mind that this dot is sometimes omitted ie ABOR gate

The OR gate is an electronic circuit that gives a high output (1) if one or more of its inputs are high A plus (+) is used to show the OR operationNOT gate

The NOT gate is an electronic circuit that produces an inverted version of the input at its output It is also known as an inverter If the input variable is A the inverted output is known as NOT A This is also shown as A or A with a bar over the top as shown at the outputs The diagrams below show two ways that the NAND logic gate can be configured to produce a NOT gate It can also be done using NOR logic gates in the same way

NAND gate

This is a NOT-AND gate which is equal to an AND gate followed by a NOT gate The outputs of all NAND gates are high if any of the inputs are low The symbol is an AND gate with a small circle on the output The small circle represents inversion

NOR gate

This is a NOT-OR gate which is equal to an OR gate followed by a NOT gate The outputs of all NOR gates are low if any of the inputs are highThe symbol is an OR gate with a small circle on the output The small circle represents inversion

EXOR gate

The Exclusive-OR gate is a circuit which will give a high output if either but not both of its two inputs are high An encircled plus sign ( ) is used to show the EOR operation

EXNOR gate

The Exclusive-NOR gate circuit does the opposite to the EOR gate It will give a low output if either but not both of its two inputs are high The symbol is an EXOR gate with a small circle on the output The small circle represents inversion The NAND and NOR gates are called universal functions since with either one the AND and OR functions and NOT can be generated

Note A function in sum of products form can be implemented using NAND gates by replacing all AND and OR gates by NAND gates A function in product of sums form can be implemented using NOR gates by replacing all AND and OR gates by NOR gates

Logic gate symbols

Table 2 is a summary truth table of the inputoutput combinations for the NOT gate together with all possible inputoutput combinations for the other gate functions Also note that a truth table with n inputs has 2n rows You can compare the outputs of different gates

Logic gates representation using the Truth table

Example

A NAND gate can be used as a NOT gate using either of the following wiring configurations

Subject Eng Literature (The Tempest ndash William Shakespeare) Topic Act III Scene 3 Lines 53 to 110 (End of the scene) Date 16th April 2020 (2nd Period)

[Students should read the original play and also the paraphrase given in the school prescribed textbook]Summary Questions amp Answers

o Seeing this strange scene all are inclined to believe the tales told by travelers that there truly are ldquounicornsrdquo and ldquothe phoenixrsquo thronerdquo

o As they are about to sit down to the feast the banquet is snatched away by a harpy (Ariel disguised) A spiritrsquos voice (Arielrsquos voice) denounces Alonso Sebastian and Antonio with particular

1 ARIEL You are three men of sin whom Destiny

(Line 53-58)That hath to instrument this

lower world And what is int the never-surfeited sea

Hath caused to belch up you and on this island

Where man doth not inhabit you rsquomongst men

Being most unfit to live I have made you mad

reference to their crime in expelling Prospero from Milan They have not received any punishment for their deed earlier but the time for their punishment has arrived Upon Alonso it pronounces ldquolingering perdition worse than deathrdquo from which there is no remedy except through sincere repentance Ariel then vanishes in thunder and the shapes enter again and carry away the table

o Prospero watching invisibly is very pleased with the performance of Ariel and his (Prosperorsquos) ldquomeaner ministersrdquo All his enemies are now in his power and are in a fit of desperation He then leaves them and goes to see how Ferdinand and Miranda are getting on

o Alonso is now much humbled and penitent with the after effect of the spiritrsquos denunciation of his crimes He believes that his son is lost forever After this all disperse being stricken mad by the speech of the spirit

o Gonzalo fearing that they may do violence to themselves or to one another follows them and bid others to follow

(a) To whom does Ariel disguised as a harpy call the three sinners What game did Fate of Destiny play with

them

The three sinners called by Ariel are Alonso Sebastian and Antonio It was Destiny which had caused the ocean to cast the three sinners on the shore Though the ocean is all the time devouring whatever appears on its surface and is never satisfied with its continual swallowing of the ships and men in the present case the ocean had cast these three sinners on the shore without killing them

(b) Who had jointly been responsible for the conspiracy against Prospero What is Prosperorsquos purpose behind all this

Three men Alonso Sebastian and Antonio had jointly

been responsible for the conspiracy against Prospero They had driven out Prospero form Milan Prosperorsquos purpose is to make these three sinners realize the wrong they had done He wants them to repent for their criminal deeds because repentance leads to self-esteem(c )What does Ariel (the harpy) tell Alonso and his companions when they take out their swords to attack him

Seeing them drawing their swords Ariel (harpy) tells them that he and his companions are the instruments of destiny and that it is not possible for human beings to do them any injury He says that the swords of human beings can not injure even a minute part of his feathers Their swords are as ineffective against him and his companions as against the wind or the water

(d) Give the explanatory meanings of the following expressions in the context of the above extract

(i)Never surfeited (ii) Belch up (iii) lsquomongst men

(i) Never surfeited never led to satisfaction

(ii) Belch up cast ashore(iii) lsquomongst men in human

society2

I and my fellows (Line 60-65)

Are ministers of Fate The elementsOf whom your swords are tempered may as wellWound the loud winds or with bemocked-at stabsKill the still-closing waters as diminishOne dowl thats in my plume

IMPORTANT PASSAGES EXPLAINED

The elements

(Line 61-66)Of whom your swords are tempered may

as wellWound the loud winds or with

bemocked-at stabs

(a) Who is lsquoIrsquo Who are his lsquofellowsrdquo

lsquoIrsquo is referred to Ariel in disguise of a harpy His lsquofellowsrsquo are other spirits serving Prospero the real Duke of Milan who has acquired supernatural powers after being banished from his Dukedom Prospero has settled in this uninhabited island

(b) What are the elements that have temperrsquod the swords Why will it not work against the speaker

The swords (of Alonso and his companions) are tempered by metal (steel) which is taken out of the earth and refined by

Kill the still-closing waters as diminishOne dowl thats in my plume My fellow

ministersAre like invulnerable

In these words Ariel reminds the King and his companions of the utter futility of drawing swords against himself and his fellows Ariel drives Alonso Antonio and Sebastian the three men of sin to desperation ndash a state in which men do violence to themselves They draw swords to strike Ariel But Ariel reminds them that he and the other spirits are the ministers of destiny and nothing can wound them The steel of which their swords are made of may cut the wind or water which being divided always closes up again Even supposing that such things may be possible it is quite impossible that their swords will cut one feather in their plume They are incapable of being wounded by any sword of man Hence it is foolish on their part to attempt to strike at Ariel and his fellow-spirits

For which foul deed

(Line 72-75)The powers delaying not forgetting

haveIncensed the seas and shores yea all the

creatures Against your peace

Ariel enters like a harpy and remaining invisible tells Alonso Sebastian and Antonio that he and other harpies are the agents of Destiny appointed to carry out her decrees He tells them that their punishment for the crime against Prospero which has been so long deferred is now to fall upon them He reminds them that they had expelled Prospero from Milan and set him and his innocent child adrift on the sea and that the sea had paid them back for their sin by the shipwreck and by the calamities they have suffered He tells them that the powers above which did not forget this mean treachery but only deferred the punishment have now engaged the seas and the shores and all living beings including him and his comrades against them The very elements and supernatural agency Ariel adds have taken up the avenging of their crime against Prospero

the action of fire It may cut the wind or water which being divided always closes up again

The sword will not work against the spirits and the harpy because they are the ministers of destiny and nothing can wound them nor it will cut a single feather in their plume

(c )What is the meaning of lsquodowlrsquo in the last line

The term lsquodowlrsquo means a filament or the smallest part of a feather In this context Ariel in disguise of harpy says that their sword cannot even damage the smallest filament of their (Arielrsquos and other spirits) feathers as they are incapable of being wounded by any sword of man

(d) What does the speaker remind the listeners about

Ariel in disguise of harpy reminds Alonso the King of Naples Sebastian Alonsorsquos brother and Antonio the present Duke of Milan and the treacherous brother of Prospero as they being three men of sin He even reminds them that their punishment for their crime against Prospero which has been so long deferred now falls upon them He reminds them that they have expelled Prospero from Milan and has set him along with his innocent infant daughter adrift on the sea So the sea has paid them back for their sin by their shipwreck and the calamities they have suffered since then The harpy rebukes Alonso of his sin that has incensed the Gods and has deprived him of his son as a punishment

(e) How do they respond

When Ariel in disguise of a harpy reminds Alonso Sebastian and Antonio of their past misdeeds and sin Alonso has a look of terror and confusion in his eyes He utters the words of sincere repentance wrung out of his conscience-stricken heart It appears to him that all the elements of nature the sea-waves the wind and the thunder proclaiming a loud voice in the name of Prospero and the crime Alonso has committed against him They are calling upon him to repent There is a deep storm raging in Alonsorsquos breast and the echoes of that storm are ringing in his ears like a clear note of wind-instrument A note of denunciation of Alonsorsquos crime leaves him much humbled and penitent and confirms his belief that his son is lost forever But Sebastian and Antonio shows some courage instead of repentance They wish to kill the spirits or devils if it appears

3

Of my instruction hast thou nothing bated (Line 85-93)

In what thou hast to say So with good life

And observation strange my meaner ministers

Their several kinds have done My high charms work

And these mine enemies are all knit upIn their distractions They now are in my

powerAnd in these fits I leave them while I visitYoung Ferdinand whom they suppose is

drownedAnd his and mine loved darling

Methought the billows spoke and (Line 96-99)

told me of itThe winds did sing it to me and the

thunderThat deep and dreadful organ-pipe

pronouncedThe name of Prosper It did bass my

trespass

These are the words of contrition coming from Alonso Ariel has driven him to a deep repentance for conspiring with Antonio against Prospero He now feels a sincere remorse It appears to him that all the elements of nature the sea-waves the wind and the thunder proclaimed with a loud voice the name of Prospero and the crime Alonso had committed against him They are calling upon him to repent There is a deep storm raging in Alonsorsquos breast and the echoes of that storm are ringing in his ears like the clear note of a wind-instrument

Comment These are the words of sincere repentance wrung out of the conscience-stricken heart of Alonso Alonso who is the lesser villain is the first to give way to remorse under the effect of Arielrsquos speech The words of Ariel seem to him to be the voice of conscience speaking to him He is driven to desperation a state in which he might do violence to his life

(a) Identify the speaker State the context

Prospero the ruler of the island is the speaker The famous banquet scene has been enacted very well Ariel and his junior spirits have played their roles excellently Prospero is glad to say words of praise for them(b) In what way the speakerrsquos instructions have been carried out

According to Prosperorsquos instructions a banquet was presented before the King of Naples and his companions when they were tired and hungry Just when they were preparing to eat the feast the banquet was suddenly removed by exercising supernatural powers All this was done by Ariel Prosperorsquos chief assistant and a powerful spirit

Ariel not only made the feast disappear but also delivered his speech blaming the King and his two companions for their past wicked deeds He warned them to repent for their misdeeds or suffer forever on that uninhabited island

(c) Who are referred to as lsquomeaner ministersrsquo What have they done

Prospero refers as lsquomeaner ministersrsquo to his other lesser spirits who were assisting Ariel in presenting a scene before the kingrsquos party They entered the scene to the accompaniment of music They assumed several strange shapes and brought in a banquet Then they danced about it with gentle actions of salutations thus inviting the King and others to eat the feast

These spirits play their role again when Ariel in the shape of a harpy quits the scene These shapes enter again and dancing with mocking gestures carry away the table

(d) Who are the speakerrsquos enemies What has happened to them

King of Naples Alonso his brother Sebastian and the present Duke of Milan Antonio (Prosperorsquos own brother) are Prosperorsquos enemies With the turn of events they have all been washed ashore on the island which is ruled by Prospero the great magician Actually this happened after the shipwreck caused by a storm which was raised by Prospero with the purpose of bringing these people to his island Prosperorsquos spirits have already confused and terrified these enemies and they are under Prosperorsquos control He can treat them as he likes

(e) What does he say about Ferdinand Explain what is meant by ldquohellip his and mine darlingrdquo

Prospero knows that Alonsorsquos son prince Ferdinand is alive though his father thinks that the prince has been drowned

Prospero refers to his daughter Miranda who is dear to him She is also very dear to Prince Ferdinand who has fallen in love with her They are waiting to be married soon for which they have received Prosperorsquos consent

4

ALONSO O it is monstrous monstrous (Line 95-102)

Methought the billows spoke and told me of it

The winds did sing it to me and the thunderThat deep and dreadful organ-

pipe pronouncedThe name of Prosper It did bass

my trespassTherefore my son ithrsquo ooze is

bedded andIll seek him deeper than eer

plummet soundedAnd with him there lie mudded

(a) In what way does Alonso express his horror when his conscience is awakened by Arielrsquos words

When Alonsorsquos conscience is awakened by Arielrsquos words he expresses his horror at what he has heard He gets the feeling that the waves of the ocean the wind and the loud thunder have spoken to him and uttered the name of Prospero Because of being reminded of his crime in a very loud and rough voice he comes to realize that he has lost his son for his past misdeeds

(b) What does Alonso imagine about his son What does Alonso want to do in his desperate state

Alonso imagines that his son is lying in the mud at the bottom of the sea He feels desperate that he wants to drown himself in the ocean deeper than the plumb-line has ever gone He wants to lie with his son at the bottom of the sea

(c) How do Sebastian and Antonio want to face the evil spirits

Sebastian says that he is not at all afraid of what the harpy has said and that he is prepared to fight any number of such monsters if they appear before him only one at a time Antonio says that he would support Sebastian in the fight against the fiendsyyy

(d) Why does Gonzalo ask Adrian to follow the three men

Gonzalo tells Adrian that all the three men namely Alonso Sebastian and Antonio are in a wild and reckless mood The thought of the heinous crime of which they are guilty has begun to torment their minds So he asks Adrian to follow those three men without loss of time and prevent them from doing anything which the turmoil in their minds might lead them to do

(e) What opinion do you form of Alonso from the above extract

Alonso who is the lesser villain is the first to give way to remorse under the effect of Arielrsquos speech The words of Ariel seem to him to be the voice of conscience speaking to him He is driven to desperation a state in which he might do violence to his life

Subject =Accounts

Ac-12 15420 topic-pL Appropriation ac

PROFIT AND LOSS APPROPRIATION ACCOUNT

MEANING AND PREPARATIONProfit and Loss Appropriation Account is merely an extension of the Profit and Loss Account of the firm The profit of the firm has to be distributed amongst the partners in their respective profit sharing ratio But before its distribution it needs to be adjusted All Adjustments like partnerrsquos salary partnerrsquos commission interest on capital interest on drawings etc are made in this account These adjustments will reduce the amount of profit for distribution This adjusted profit will be distributed amongst the partners in their profit sharing ratio To prepare it at first the balance of Profit and Loss Account is transferred to this account The journal entries for the preparation of Profit and Loss Appropriation Account are given below

1 for transfer of the balance of Profit and Loss Account to Profit and Loss Appropriation Account

(a) In case of Net Profit

Profit and Loss Ac helliphelliphelliphelliphellipDrTo Profit and Loss Appropriation Ac(Net Profit transferred to Profit and Loss Appropriation Ac)

(b)In case of Net Loss

Profit and Loss Appropriation Achelliphelliphellip DrTo Profit and Loss Ac(Net Loss transferred to Profit and Loss Appropriation Ac)

2 for Interest on Capital

For transferring on Interest on CapitalProfit and Loss Appropriation Achelliphelliphellip DrTo Interest on Capital Ac(Interest on capital transferred to Profit amp Loss Appropriation Ac)

3 for Interest on Drawings

For transferring Interest on Drawings Interest on Drawings Achelliphelliphelliphelliphelliphellip DrTo Profit and Loss Appropriation Ac(Interest on drawing transferred to Profit amp Loss Appropriation Ac)

4 For Partnerrsquos SalaryFor transfer of partnerrsquos SalaryProfit and Loss Appropriation Achelliphellip DrTo Salary Ac(Salary transferred to profit amp Loss Appropriation Ac)

5 For Partnerrsquos CommissionFor transferring commissionProfit and Loss Appropriation Achelliphelliphellip DrTo Commission Ac(Commission transferred to Profit and Loss Appropriation Ac)

6 For Transfer of agreed amount to General ReserveProfit and Loss Appropriation Ac helliphellipDrTo General Reserve Ac(Transfer to General Reserve)

7 for share of Profit or Loss appropriation(a) If ProfitProfit and Loss Appropriation Achelliphellip DrTo Partnerrsquos CapitalCurrent Ac(Profit transferred to capitalcurrent Ac)(b) If LossPartnerrsquos Capital Current Achelliphelliphelliphellip DrTo Profit and Loss Appropriation Ac(Loss transferred to capitalcurrent Ac)

THE FORMAT OF PROFIT AND LOSS APPROPRIATION

Profit and Loss Appropriation Account for the year endedhelliphelliphelliphellip

Particulars Amount Particulars Amount

To PL Ac (loss) By pL Ac (profit)

To Interest on capital BY Interest on drawings

To partner`s commission by Partner`s capital Ac ( loss)

To Partner`s salary To Interest on partner`s loan To General Reserve To Partner`s Capital AC (Profit)

Subject= Economics

MOVEMENT ALONG THE DEMAND CURVE (CHANGE IN QUANTITY DEMANDED)In law of demand you have already studied the inverse relationship between price and quantity demanded When quantity demanded of a commodity changes due to change in its price keeping other factors constant it is called change in quantity demanded It is graphically expressed as a movement along the same demand curve There can be either a downward movement or an upward movement along the same demand curve Upward movement along the same demand curve is called contraction of demand or decrease in quantity demanded and downward movement along the same demand curve is known as expansion of demand or increase in quantity demanded

Extention of demandd

price (rs)p A

B Extentionp1 d

Q Q1

Quantity demanded ( in units)

Contraction of demandd

p2 Ccontraction

p APrice (Rs)

d

Q2 Q

Quantity demanded (in units)

Explanation of movement of demand A fall in price from OP to OP1 leads to increase in quantity demanded from OQ to OQ1 (expansion of demand) resulting in a downward movement from point A to point B along the same demand curve DD When Price rises from OP to OP2 quantity demanded falls from OQ to OQ2 (contraction of demand) leading to an upward movement from point A to point C along the same demand curve DD

  • Activity Series of Metals
    • Drawbacks of Rutherfordrsquos model of atom
      • Electromagnetic radiations
      • Properties of electromagnetic radiations
      • Characteristics of electromagnetic radiations
        • Plancks Quantum Theory-
        • Photoelectric effect
          • Intext Questions
            • Logic gates
            • Digital systems are said to be constructed by using logic gates These gates are the AND OR NOT NAND NOR EXOR and EXNOR gates The basic operations are described below with the aid of truth tables
            • AND gate
            • Example
Page 2:  · Web viewSubject. Topic. Summary. Execution. English 1 . Chapter 1 naming words . Page 8. Write the names of these pictures:- Person:-1. father. 2.Firefighter 3.doctor 4 ...

and subtraction

+ 1 + 3 +1 77 6

8 1+ 0 +7 8 8

EVS Chapter 5 festivals and celebrations

Book Page 110 time to answer

Choose the correct option1The President of India hoists the Indian flag on RepublicDay2Gandhi Jayanti is the birthday of Gandhiji3Republic Day is a national festival

Computer CHAPTER-2PARTS OF COMPUTER

DONE IN THE PREVIOUS CLASS PAGE NO 18ATICK THE CORRECT OPTION 1HEADPHONE2KEYBOARD3PEN DRIVE4MOUSE5CPU

Hindi वयाकरण फलो क नाम 1चपा 2कमल 3बला 4पलाश 5कनर 6गलाब 7मोगरा

Bengali ইndash পাঠndashাকয রনানমনা দেওয়া চোহিনচে াকয গঠন অভযাস কর

মাndashআহিম মা rsquo দেক ভাচোাহিসনীndashনীদেত মাছ থাচেকগাছndashতা গাছ উাউndashাউ একতারা াহিচেয় গান কচেরাndashরাচেতর আকাচে( া ওচেঠ১ দেথচেক ২০ সংযা ও ানান অভযাস কচেরা

Class IISubject Topic Summary Execution

Eng literature

Ch 4 Bob and the Bathtub

Refer to pg 47 Do in the book

II Choose the correct word for each blank-Answers

1 years2 gives3 clothes4 with5 in

Eng language

Ch 6 pronouns Refer to pg 32 and 33Ex A Do in the book

EX A ] REWRITE THESE AFTER REPLACING THE HIGHLIGHTED WORDS WITH PRONOUNS- he she it they or weAnswers

1 She dances very well2 He goes to office3 They have gone to watch a film4 We went trekking5 They are looked after very well at

the zoo6 It talks a lot7 They gave some flowers to the

teacher

8 She must finish her homework9 It rings at 630 am everyday10 They sing beautifully

EVS Ch 5 Eating healthy

Refer to pg 118Do in the book

C Match the followingAnswers

1 Drinking water ndash clean and filtered2 Eggs ndash food from animals3 Carrot and radish ndash vegetables4 A healthy mix of food items ndash

balanced diet5 Apple an orange ndashfruits

Bengali ইndashাংাসাহিতযপহিরয়

পাঠndashহিসংওইচেররগলপঅন(ীনীরপরচোততর

৯ াকযরনা -পাাndashআহিমপাাভাচোাহিসগাndashপাাচেরগায়হিসংচেরাসাহিছহিসংndashহিসংদেকপশরাায়ইরndashইরমানচে4রঅহিনষটকচেরহি(কাহিরndashহি(কাহিরচেনহি(কারকচের

Hindi वयाकरण Chapter 4 सजञा

सजञा -इस दनिनया म सभी का कोई ना कोई नाम होता ह यही नाम सजञा कहलाता ह नाम कई परकार क होत हजस- पश-पकषी जगह दि-नो क नाम वयकतियो क नाम आदि-

Read the lesson

Mathematics CHAPTER-3NUMBERS UPTO 1000

PLACE VALUE PAGE NO 1882WRITE THE PLACE VALUE FOR GIVEN 3-DIGIT NUMBERSb)368Place value of 8 is ones8Place value of 6 is tens60Place value of 3 is hundred300(Practice the rest exercise)

Class IIISubject Topic Summary Execution

ENGLISH II CHAPTER-2 THE WOODEN BOWL

Refer to pg 27COMPOSITION[ DO IN THE BLANK SPACE GIVEN IN YOUR BOOK]

COMPOSITION ABOUT A THING YOU MADE READ AND THE ANSWER EACH QUESTION BRIEFLY AND THEN DRAW AND COLOUR THE THING YOU HAD MADE FOR EXAMPLE IT CAN BE A CARD A EASY CRAFT ETC

SST INDIA-THE FESTIVALS OF INDIA

DONE Encircle the odd one out and give reason for your option1RajpathRajpath is not related to the India Independence

2Santa ClausSanta Claus is related to the Christmas

3 LangarLangar is related to the Gurupurab festival

4August 15August 15 is a National festival

BENGALI ই ndash াংা সাহিতয পহিরয়

পাঠ ndash ১একতাই অন(ীনীর পরচোততর

৫ হিপরীত - চো times দোকা স times ঃ হি( times অহি( াধা times ছাা আস times নক সহিতয times হিমচেথয৭ াকয রনা -মযাহিক ndash আমার কাকা ভাচো মযাহিক দোয় া(ঝা ndash রাচেতর অনধকাচের া(ঝা দেচে আমার ভয় াচেগ অহিনষট ndash অচেনযর অহিনষট হিনতা করচেত দেনই াহি= ndash কাগচের াহি= চে পচেরাচেনা স নহিথ পাওয়া দেগচো

HINDI वयाकरण Chapter 4 सजञा

सजञा- इस दनिनया म सभी का कोई ना कोई नाम होता ह यही नाम सजञा कहलाता ह नाम कई तरह क होत ह जस -वयकतियो क नाम पराणिणयो क नाम सथानो क नाम भावो क नाम आदि-

1सजञा निकस कहत ह उ-ाहरण -ीजिजएउततर- निकसी भी वयकति वसत सथान या भाव क नाम को सजञा कहत हजस- राम सकल घडी गरमी आदि-

MATHEMATICS CHAPTER-4 SUBTRACTION

Subtraction (with decomposing)Subtraction of 3-digit numbers

Exercise 18Example 1 (p-48)

Fill in the place holders3 hundreds 2 tens 5 ones ndash 2 hundreds 5 tens 7 ones= 3 hundreds 1 tens ___ ones - 2 hundreds 5 tens 7 ones= 2 hundreds ___ tens 15 ones - 2 hundreds 5 tens 7 ones= 6 tens ___ ones

Solution3 hundreds 2 tens 5 ones ndash 2 hundreds 5 tens 7 ones= 3 hundreds 1tens 15 ones - 2 hundreds 5 tens 7 ones= 2 hundreds 11 tens 15 ones - 2 hundreds 5 tens 7 ones= 6 tens 8 ones

Example 4 (p-51)Subtract 389 from 600

Solution H T O 6 0 0 - 3 8 9

2 1 1We cannot subtract 9 ones from 0 ones so we borrows 1 tens Here there is no ten so first we borrow 1 hundreds or 10 tens having behind 5 hundreds From 10 tens we borrow 1 tens or 10 ones leaving behind 9 tensNow 10 ones ndash 9 ones = 1 onesAnd 9 tens ndash 8 tens = 1 tens5 hundreds ndash 3 hundreds = 2 hundreds 600 ndash 389 = 211

Exercise2 Fill in the place holders5 hundreds 2 tens 8 ones ndash 3 hundreds 8

tens 3 ones= 4 hundreds ___ tens 8 ones - 3 hundreds 8 tens 3 ones= 1 hundreds ___ tens ___ ones

Solution5 hundreds 2 tens 8 ones ndash 3 hundreds 8 tens 3 ones= 4 hundreds 12 tens 8 ones - 3 hundreds 8 tens 3 ones= 1 hundreds 4 tens 5 ones

Class IVSubject Topic Summary Execution

English literature

Hercules andDeianira (tales from Greece and Rome)

Hercules spent the greater part of his life in helping the poor and weak people But Juno still wanted to harm him So she sent him into all sorts of dangers But he was brave and strong and he overcame all the dangers Juno made him a slave to the king of Argos Hercules worked hard for the king The king took pity on him and told him that he would set him free if Hercules perform twelve difficult tasks Hercules accepted the challenge

1How did Hercules spent the greater part of his lifeAns Hercules spentthe greater part of his life in helping the poor and weak people

2How did Juno tried to harm HerculesAns Juno tried to harm Hercules by sending him into all sorts of dangers

3Under which king Hercules was working as a slaveAns King of Argos

4What was the challenge that king of Argosgive to HerculesAns King of Argos told Hercules that he would set him free if he perform twelve difficult tasks which Hercules accepted

Social studies Major landforms on earth

Answer the following questions5What do oceans provide usAns Oceans provide us a variety of sea food

6What is alluviumAns The rivers carry sand soil and small pieces of rocks This fine soil or silt known as alluvium is very good for growing crops

COMPUTER160420

CHAPTER 3 EDITING IN MS WORD

DRAG AND DROPTHE DRAG AND DROP METHOD OF MOVING THE TEXT ALLOWS US TO MOVE THE SELECTED TEXT USING MOUSE FROM ONE LOCATION TO ANOTHER WITHIN A DOCUMENT

Q7) HOW TO WE DRAG AND DROP TEXT IN MS WORDAns) WE CAN DRAG AND DROP TEXT IN MS WORD FOLLOWING STEPS ARE---

SELECT THE TEXT PLACE THE MOUSE POINTER

ANYWHERE ON THE SELECTED TEXT

CLICK AND HOLD THE LEFT MOUSE BUTTON UNTIL THE INSERTION POINT CHANGES INTO A WHITE ARROW

WHEN WE REACH THE PLACE IN THE DOCUMENT WHERE WE WANT TO MOVE THE TEXT RELEASE THE MOUSE BUTTON

Science Adaptations in Summary Execution

Animals Animals are divided into different groups according to their habitats Some animals live on land some animals live both on land and in water some animals adapted themselves for food and some for protection They develop special features that help them to survive in their habitats Animals that live on land are called Terrestrial animals such as lions tigers Yaks polar bears Polar Bears live cold polar regions Yaks live in mountains These animals have thick fur on their body and a thick layer of fat under their skin which keeps them warm

Solved exercise question and answersA Write the correct answer1Terrestrial animals live here - on land 2This animal lives in polar regions ndash polar bear 3 These help a fish to breathe in water - gills4 This helps carnivorous birds to tear flesh - a hooked beak

MATHEMATICS

Ch 6Multiplication

Exercise 17 Example 2

Multiply 224835 by 4 Solution

2 2 4 8 3 5 times 48 9 9 3 4 0

Exercise

18 3 1 1 4 6 7 19 2 1 2 7 8 5 times 3 times 4 9 3 4 4 0 1 8 5 1 1 4 0

20 2 0 6 1 3 times 9 1 8 5 5 1 7

Class VSubject Topic Summary Execution

Science Chapter 2 - The Skeletal System

Posture

The way in which we hold our body while sitting standing walking and lying down is known as posture

Importance of correct posture and exercise

1 Correct posture and exercise makes our bones and muscles strong 2 It provides our body more energy to do works

3 It keeps our body stressless 4 It also prevent back ache and muscle pain

Some important yoga asanas are-

1 Veerabhadrasana ( Warrior pose )

2 Dhanurasana ( Bow pose )

3 Trikonasana ( Triangle pose )

D Answer these questions

5How do our muscle work

Ans ndash Our muscles work by contracting and relaxing

6Why should we maintain a correct posture

Ans ndash We should maintain a correct posture because ndash

i Correct posture and exercise makes our bones and muscles strong

ii It provides our body more energy to do works

iii It keeps our

body stressles

s iv It also prevent

back ache and muscle pain

Social studies Conquering distances

Put a tick on the correct option1 b Ship 2b water transport 3 b trains 4 c 13 5 b Wright brothersTrue or false1 False 2 True 3 True 4 True

MATHEMATICS

Ch 3Addition and Subtraction

Exercise 126 A school needs Rs 4987653 for its building It has only Rs 3592468 in its accounts Estimate the money by rounding off to the nearest lakh it has still to raise

Solution Estimated amount need Rs 5000000 The school has only Rs 3600000 Still to raise Rs 1400000

MATHEMATICS

Ch 4

Mul

tiplic

ation

and

Div

ision

We have learnt multiplication tables up to 15 Let us extend the tables up to 20X 11 12 13 14 15 16 17 18 19 201 11 12 13 14 15 16 17 18 19 202 22 24 26 28 30 32 34 36 38 403 33 36 39 42 45 48 51 54 57 604 44 48 52 56 60 64 68 72 76 805 55 60 65 70 75 80 85 90 95 1006 66 72 78 84 90 96 102 108 114 1207 77 84 91 98 105 112 119 126 133 1408 88 96 104 112 120 128 136 144 152 1609 99 108 117 126 135 144 153 162 171 180

10 110 120 130 140 150 160 170 180 190 200Properties of Multiplication1 The product of two numbers does not change when the order of numbers is changed eg 503times23 = 23times503 [This property is called Commutative Property of multiplication]

2 The product of three numbers does not change when the grouping of numbers is change eg (15times18iquesttimes10=15 times(18times 10) = (15times10iquesttimes18 [This property is called Associative Property of multiplication]

3 The product of a number and 1 is the number itself eg 1513 times 1 = 1513 [This property is called Identity Property of multiplication and the integer 1 is called Identity Element of multiplication]

4 The product of a number and 0 is 0 eg 718205times 0 = 0 times 718205 = 05 The product of a number by the sum of two numbers is equal to the sum of the

products of that number by the two numbers separately eg123 times (105+ 48) = 123 times 105 + 123 times 48 [ This property is called Distributive Property of multiplication over

addition]English language

Transitive and intransitive verb

Pick out the verbs from the following sentences and say whether they are transitive or intransitive verb6 Was fullndash verb Intransitive verb7 Have been decorated- verb Transitive verb8 Happy cheerful ndash verb Intransitive verb9 Shall come back ndash verb Transitive verb

English 2 The fall of Lanka

This is the story of the fight between Rama and Ravana as told by Valmiki in lsquoThe Ramayana The monkeys worked all day and all night and at last built a bridge so that Rama and

Write the synonyms of

1 Stationed-

his army could cross to Lanka and rescue Sita assign2 Invade- enter

a country or a region so as to subjugate or occupy it

3 Prowess- bravery in battle

4 Haughty- arrogantly

5 Puny- small and weak

6 Spy- secret agent

7 Dreadful- causing or involving great suffering

8 Violate- disobey

9 Ghastly- causing great horror or fear

10 Deception ndash misleading

Class VISubject Topic Summary Execution

HISTORY AND CIVICS

CHAPTER 3

MAHAVIRA AND BUDDHA ndash GREAT PREACHERS

BUDDHA

Impact of Buddhism on Indian Cultures

Impact on Religion

Buddharsquos practical and simple doctrines made their impact on HinduismThe principle of ahimsaIt brought about a great change in the performance of costly yanjnas and sacrifices which previously involved immense loss of life The Mahayana Buddhists adopted the practice of worshipping Buddha and bodhisattvas making idols and erecting temples in their honour

Impact on Literature

After the death of Buddha his teachings were compiled and called TripitakasThe Jatakas contain tales dealing with the previous births of Buddha

Impact on Education

The Buddhist monasteries became great centres of learning These centres of learning developed into famous universities- Nalanda Taxila Vikramshila etc

Impact on Art and Architecture

The gateways and railing of the Sanchi Stupa were covered with sculptured figuresCave- temples were also constructed which were decorated with beautiful frescoes

1) How many parts of Tripitakas are thereAns -There are three parts of Tripitakas- Sutta Pitaka Vinaya PitakaAbhidhamma Pitaka

2)What are the subjects taught in these monasteries Ans ndash Buddhist scriptures logic Philosophy medicine astronomy etc

3) Which art was developed under Buddhist patronage Ans ndash Gandhara art

The Gandhara art was developed under Buddhist patronage

BIOLOGY The Leaf Photosynthesis The process by which green plants make their own food from carbon dioxide and water in the presence of sunlight and chlorophyll is called photosynthesis

All green plants need the following to make their food ndash

water carbon dioxide chlorophyll and energy in the form of sunlight

Carbon dioxide + water ------------- Glucose + oxygen

The end product of photosynthesis is glucose

Fill in the blanks

1 Plants make their food by the process of photosynthesis

2 The inner wall of the guard cell is thicker than the outer wall

3 The extra glucose is converted into starch and sucrose

4 The leaf is boiled in alcohol to remove chlorophyll

5 The rate of transpiration is more on the hot day then a cold day

6 Photosynthesis helps to observe water and minerals from the soil

English 1 Pronouns Kinds of pronouns 1 Personal pronouns2 Possessive pronouns3 Reflexive pronouns4 Interrogative pronouns5 Relative pronouns6 Demonstrative pronouns7 Indefinite pronouns

Personal pronouns they refer to first second and third person in sentences First person- the speakerSecond person-the listenerThird person-the objectperson being spoken aboutPersonal pronouns should have the same gender and number as the nouns they refer to

Possessive pronouns these are used to indicate the relationship between the objects and people These pronouns include mine ours yours his hersand theirs

ExerciseBFill in the blanks with suitable pronounsThere was much excitement among the childrenTheywere eagerly looking forward to the annual picniclsquoAre they going to Lodhi Gardens toorsquo wondered AneeshlsquoNo they are going to Buddh Jayanti Park with Mrs Jain said Mrs ChopraThe children looked disappointedlsquoWonrsquot you be taking us Marsquoam rsquo they askedlsquoSorry children I have to go to Mumbai for a week to look after my sick mother But you will have fun with Mrs Jain she is full of laughter and you will love being with her the whole daylsquoIt will not be the same they grumbled

English 2 The great train journey- Ruskin Bond

The great journey by Ruskin Bond is a story about Suraj who loved trains and wanted to go to places One day while wandering along the railway tracks he enters into a carriage compartment The train suddenly starts moving with him in the compartment and after a journey returns back to the same place from where it had begun The story is about his experience during that journey

State true or false1 When the train had passed leaving behind the

hot empty track Suraj was lonely2 It was winter holidays

and Suraj did not know what to do with himself

3 He plunged his hands into the straw and pulled out an apple

4 A dirty bearded face was looking out at him from behind a pile of crates

5 Suraj wanted to go to Japan

Hindi 2nd lang

गललबाजलडका खालीसथानोकोभरो-6 गो-ामसनिनकलकरहमगराजमआगए7 माबोधराजकोराकषससमझतीथी8 चीलरोशन-ानमसअ-रआकरतहसीलपरबठगई9 तीनचारतीनकऔररईकगोलउडलनिकनघोसलानहीनिगरा10 वहसवयतोघोसलातोडनककतिलएगललउठालायाथा

11 -ीवारकसाथलगतगोहपजोकसहार-ीवारपकडलतीह12 बोधराजअभीभीटकटकीबाधचीलकीओर-खरहाथा13 बोधराजअपनीजबमबहतसाचगगाभरकरलायाथा14 मरनिपताजीकीतरककीहईऔरहमलोगएकबडघरमजाकररहनलग15 बागमजातातोफलपरबठीनिततलीको-खनिततलीकोपकडकरउगकतिलयोकबीचमसल-ता

BENGALI(2ND LANGUAGE)

সহিনধসবরপওসবরসহিনধ

সবরসহিনধরহিনয়ম- ৯ই-কারহিকংাঈ-কাচেররপচেরইাঈহিভননঅনযসবররণCথাকচেইাঈসথাদেনয-ফায়এংওইয- ফাপCচেরণCযকতয়

১০উ-কারহিকংাঊ-কাচেররপচেরইাঈহিভননঅনযসবররণCথাকচেউাঊসথাদেন-ফায়এংওই- ফাপCচেরণCযকতয়

১১ঋ- কাচেররপচেরঋহিভননঅনযসবররণCথাকচেঋসথাচেনর -ফায়এংওইর পCচেরণCযকতয়

১২সবররণCপচেরথাকচেপCতMএ-সথাচেনঅয় ঐ- সথাচেনআয় ও- সথাচেনঅএংঔ- সথাচেনঅায়

৯ই+ অ= য- ফাআহি+ অনত= আযনত অহিধ+ অয়ন= অধযয়নই+ আ=য- ফা+ াইহিত+ আহি= ইতযাহি পরহিত+ আতC ন= পরতযাতC নই+ উ=য- ফা+ উঅহিত+ উহিকত= অতযহিকত হি+ উৎপহিতত= যৎপহিততই+ ঊ= য- ফা+ ঊ ই+ এ= য- ফা+ এঈ+ অ= য- ফা পরহিত+ ঊ4= পরতয4 পরহিত+ এক= পরচেতযকঈ+ অ আ= য- ফা+ অ আনী+ অমব= নযমব মসী+ আধার= মসযাধার

১০উ+ অ= অন+ অয়= অনবয় পশ+ অধম= পশবধমউ+ আ= াস+ আগত= সবাগত পশ+ আহি= পশবাহিউ+ ঈ= হিঅন+ ইত= অহিনবতউ+ এ= দেঅন+ এ4রণ= অচেনব4রণউ+ ঈ= ীসাধ+ ঈ= সবাধবী তন+ ঈ= তনবী

১১ঋ+ অ= র মাত+ অনমহিত= মাতরনমহিতঋ+ আ= রা হিপত+ আয়= হিপতরায়ঋ+ ই= হির মাত+ ইচছা= মাতচছাঋ+ ঈ= রী ধাত+ ঈ= ধাতরীঋ+ উ= র ভরাত+ উপচে(= ভরাতরপচে(

১২এ+ অ= অয় দেন+ অন= নয়নঐ+ অ= আয় গৈগ+ অক= গায়কও+ অ= অ দেপা+ অন= পনও+ ই= অ দেপা+ ইতর= পহিতরও+ এ= অ দেগা+ এ4রণা= গচে4রণাঔ+ অ= অা দেপৌ+ অক= পাকঔ+ ই= অা দেনৌ+ ইক= নাহিকঔ+ উ= অা দেভৌ+ উক= ভাক

MATHS Topic NumbersChapter Natural numbers and whole numbers

Study item properties of whole numbers for multiplication

1 Closure property If x and y are two whole numbers then xtimesy is also a whole numberExample If x = 9 and y =3 then xtimesy = 9times3 = 27 which is a whole number

2 Commutative property If x and y are two whole numbers then xtimesy = ytimesxExample If x = 5 and y = 2 then xtimesy = 5times2 = 10y times x = 2times5 = 10Therefore 5times2 = 2times5

3 Associative property If x y and z are three whole numbers then x times(ytimesz) = (xtimesy) times zExample If x =3 y = 5 and z = 7 then 3 times (5times7) = 3 times (35) = 105And (3times5) times7 = (15) times 7 = 105Therefore x times (ytimesz) = (xtimesy) timesz

4 Distributive property If x y and z are three whole numbers then xtimes (y + z) = x times y + x times z

Therefore the multiplication of whole numbers is distributive over their additionExample If x = 5 y = 3 and z= 2Therefore x times (y + z) = 5 times (3 + 2) = 5times5 =25And x times y + xtimes z = 5times3 +5times2 =15 +10 = 25Again x times (y ndash z ) = x times y ndash x timesz Therefore 5 times ( 3 - 2) = 5 times1 = 5 and 5times3 ndash 5 times2 = 15 ndash 10 = 5Therefore the multiplication of whole numbers is also distributive over their subtraction if y is greater than z

5 Existence of identity If x is a whole number then

X times1 = x 1 times x = xTherefore we can write x times1 = 1 times xTherefore the multiplication of any whole number with 1 is the number itselfTherefore we can say that 1 is multiplicative identity or identity element for multiplicationExample 5 times1 = 5 1 times 5 = 5 Therefore 5 times 1 = 5

6 Multiplicative inverse If x is any whole number ( x is not equal to zero ) then its multiplicative inverse will be 1xSo x times 1x = 1 but 1x is a whole number if x = 1For other values of whole number 1x is not a whole number therefore we can write its multiplicative inverse does not exists

7 Cancellation law of multiplication If x y and z are three non- zero whole numbers then x times y = x times z

Or y = zExample 9 times y = 9 timeszTherefore y = z

Class VIISubject Topic Summary Execution

English 2 Sentences based on meanings

Kinds of sentences

Assertive or declarative to convey information or simply make a statement

Interrogative to ask different types of questions

Imperative to command or instruct someone or make a request

Exclamatory to express strong feelings and emotions

Exercise c1 What a nice compliment that is

That is a nice compliment2 How well- behaved the children

areThe children are very well-behaved

3 What great chefs we areWe are great chefs

4 What a shame it isIt is a shame

5 What a fantastic idea you haveYou have a fantastic idea

Homework 6 -10English

LiteratureThe Listeners III) Answer the following questions-

d) Identify two words used in the poem to give the poem an eerie atmosphereAns- Two words used to give the poem an eerie atmosphere are ldquogreyrdquo and ldquophantomrdquo

e) Who do you think are the inmates of the houseAns- I think the inmates of the house are phantom who once used to dwell in it

f) Why was the poet ldquoperplexed and stillrdquoAns- He was lsquo perplexed and stillrsquo because he was expecting an answer from the inmates of the house But despite of repeated calls there was no response

CHEMISTRY Chapter 2 ndashElement and Compound

Activity Series of MetalsThe activity series is a chart of metals listed in order of declining relative reactivity The top metals are more reactive than the metals on the bottomMetal SymbolReactivity

Lithium Li displaces H2 gas from water steam and acids and forms hydroxides

Potassium K

Strontium Sr

Calcium Ca

Sodium Na

Magnesium Mg displaces H2 gas from steam and acids and forms hydroxides

Aluminum Al

Zinc Zn

Chromium Cr

Iron Fe displaces H2 gas from acids only and forms hydroxides

Cadmium Cd

Cobalt Co

Nickel Ni

Tin Sn

Lead Pb

Hydrogen gas

H2 included for comparison

Antimony Sb combines with O2 to form oxides and cannot displace H2

Arsenic As

Bismuth Bi

Copper Cu

Mercury Hg found free in nature oxides decompose with heating

Silver Ag

Palladium Pd

Platinum Pt

Gold Au

Answer the following Q)Difference Between Metals And Nonmetals

Metals Nonmetals

These are solids at room temperature except mercury

These exist in all three states

These are very hard except sodium

These are soft except diamond

These are malleable and ductile

These are brittle and can be breakdown into pieces

These are shiny These are non-lustrous except iodine

Electropositive in nature Electronegative in nature

Have high densities Have low densities

Math Number System

Chapter Fraction

Study item Some solved sums from exercise 3(B)1) For each pair given below state whether it from like fractions or unlike

fractions (i) 58 and 78

= Like Fraction because denominators same(ii) 815 and 821

= Unlike Fraction because denominators are not same

(iii) 49 and 94 = Unlike Fraction

2) Convert given fractions into fractions with equal denominators(iii) 45 1720 2340 and 1116Solution Given fraction 45 1720 2340 and 1116Therefore the LCM of 5 20 40 and 16 is 80Therefore 45 = 4times165times16 = 64801720 = 17times420times4 = 68802340 = 23times240times2 = 4680 1116 = 11times516times5 = 5580

3) Convert given fractions into fractions with equal numerators(iii) 1519 2528 911 and 4547Solution Given fractions 1519 2528 911 and 4547Therefore the LCM of 15 25 9 and 45 is 2251519 = 15times1519times15 = 225285 2528 = 25times928times9 = 225252911 = 9times2511times25 = 2252754547 = 45times547times5 = 225235

4) Put the given fractions in ascending order by making denominators equal

(iii) 57 38 914 and 2021Solution Given fraction 57 38 914 and 2021Therefore the LCM of the denominators is 16857 = 5times247times24 = 12016838 = 3times218times21 = 63168914 = 9times1214times12 = 1081682021 = 20times821times8 = 160168Therefore ascending order 63168lt108168lt120168lt160168Therefore ascending order of given fractions38lt914lt57lt2021

COMPUTER CHAPTER-1COMPUTER FUNDAMENTALS

DONE IN THE PREVIOUS CLASSES PAGE 10CWRITE TRUE AND FALSE

1 True2 False3 False4 False5 True

GEOGRAPHY CHAPTER 7EUROPE

CHAPTER COMPLETE 1)Europe is home to a famous mountain range called the Alps

2)River Rhine originates in Switzerland

3)The Eiffel Tower one of the tallest structures in Europe

4) Vatican City is one of the most densely populated European countries

5)Sognefjordin Norway is the largest fjord in Norway

Class VIIISubject Topic Summary Execution

MATHEMATICS Ch 6Sets

Exercise 6 (D)1 Given A = x x isin N and 3iquest x le 6 and B = x x isin W and xlt4 find (i) Sets A and B in roster form (ii) A cup B (iii)

A cap B(iv) A ndash B (v) B ndashA

Solution (i) A = 456 and B = 0123

(ii) A cup B = 0123456 (iii) A cap B = ϕ (iv) A ndash B = 456 (v)B ndash A = 0123

3 If A = 56789 B = x 3 lt x lt 8 and x isin W and C = x xle5 and x isin N Find (i) A cup B and (A cup B) cup C (ii) B

cup C and A cup ( B cup C)

(iii) A cap B and (A cap B) cap C (iv) B cap C and A cap (B cap C)

Is (A cup B) cup C = A cup (B cup C)

Is (A cap B) cap C = A cap (B cap C)

SolutionA = 56789 B = 4567 C = 12345

there4 (i) A cap B = 456789 and (A cup B) cup C = 123456789

(ii) B cup C = 1234567 and A cup ( B cup C) = 123456789

(iii) A cap B = 567 and (A cap B) cap C = 5

(iv) B cap C = 45 and A cap (B cap C) = 5

Now (A cup B) cup C = 123456789

And A cup ( B cup C) = 123456789 there4 (A cup B) cup C = A cup (B cup C)

Again (A cap B) cap C = 5 and A cap (B cap C) = 5

there4 (A cap B) cap C = A cap (B cap C)

4 Given A = 012345 B = 02468 and C = 0369 Show that (i) A cup (B cup C) = (A cup B) cup C ie the union

of sets is associative (ii) A cap (B cap C) = (A cap B) cap C ie the intersection of sets is associative

SolutionNow B cup C = 0234689 and A cup B = 01234568

there4 A cup (B cup C) = 012345689 and

(A cup B) cup C = 012345689

So (i) A cup (B cup C) = (A cup B) cup C ie the union of sets is associative

Again B cap C = 06 and A cap B = 024

there4 A cap (B cap C) = 0 and (A cap B) cap C = 0

So (ii) A cap (B cap C) = (A cap B) cap C ie the intersection of sets is associative

Physics Chapter 2 Physical Quatites and Measurements

Here We Will Do Some QuestionsRelated To Chapter 2

A density bottle has a marking 25 mL on it It means that

1 the mass of density bottle is 25g

2 the density bottle will store 25 ml of any liquid in it

3 the density bottle will store 25 ml of water but more volume of liquid denser than water

4 the density bottle will store 25 ml of water but more volume of a liquid lighter than water

Solution 2 the density bottle will store 25 ml of any liquid in it

COMPUTER CHAPTER-2Spreadsheet Functions and Charts

SELECTING RANGE IN ROWSCOLUMNSWHEN TWO OR MORE CELLS ARE SELECTED IT IS CALLED A RANGEA RANGE OF CELLS CAN BE FORMED IN TWO WAYS--a) SELECTING RANGE BY USING THE MOUSEb) SELECTING RANGE BY USING THE KEYBOARD

Q1)WRITE THE STEPS TO SELECT PARTIAL RANGE IN A ROW

Ans)THE STEPS ARE-6 SELECT THE ROW7 BRING THE CELL POINTER TO THE DESIRED

LOCATION FROM WHERE YOU WANT TO START YOUR SELECTION

8 CLICK THE LEFT MOUSE BUTTON AND KEEP DRAGGING TO YOUR RIGHT TILL YOU REACH THE LAST CELL TO NE SELECTED

RELEASE THE MOUSE BUTTON

GEOGRAPHY Asia

CLIMATE

Asia experiences great extremes of climate Jacobabad in the Sind province of Pakistan is one of the hottest places in the WorldVerkhoyansk in Siberia is one of the coldest places in the WorldCherrapunji and Mawsynram in India are two wettest places in WorldArabia Tibet Gobi and Mongolia are extremely dry regionsFactors Affecting Climate of Asia-The factors influencing the climate of Asia are-

Factors Affecting Climate of Asia-Thoroughly read the table in page number 60

Latitudinal extent

Continentality

Relief features

Presence of low pressure trough

Jet streams

English Language The Sentence A complex sentence contains one independent clause and at least one dependent clause The dependent clause in a complex sentence is introduced with subordinating conjunctions or relative pronouns

Commonly Used Subordinating Conjunctions-Time after before while when since untilCause And Effect because now since as in order that soOpposition although though even though whereas while in spite ofCondition if unless only if whether or not even if in case(that)

Commonly Used Relative Pronouns-Who whose whom which whoever whomever whichever that

Class IXSubject Topic Summary Execution

1-BENGALI(2ND LANGUAGE)

ldquo বঙগভমিরপরমিrdquo াইকেলধসদনদতত

আচেগর পর উততর পচো-১ ২ ৩ এং নীচের পর টি াহির কা- ৪মহিbকাও গচেনা দেগা পহিচে অমত হরচে- ক) কার দো দেকান কহিতার অং( ) কতা দেক পরসঙগ কী উহিকতটির তাৎপযC আচোনা কচেরা৫দেসই ধনয নরকচে দোচেক যাচে নাহি ভচে মচেনর মহিeচের সাচেসচে সCন ক) কহির কায C ার উচেf(য হিক হিছ কহি কন কহিতাটি দেচেন) কহি কার কাচেছ হিমনহিত কচেরচেছনগ) কহি এই পহিথীচেত কাচের ধনয মচেন কচেরনঘ) কহি হিক রকম অমর তাাভ করচেত ান

Hindi 2nd lang

काकी(कतिसयारामशरण गपत)

इस कहानी म लखक न यह बतान का परयास निकया ह निक बचच अपनी मा स निकतना परम करत ह शयाम अबोध बालक ह वह अपनी मा क मरन क बा- उसन अपनी मा क कतिलए बहत रोया बा- म उस पता चला निक उसकी मा राम क घर चली गई ह आकाश म उडती हई पतग -खकर उस हरष हआ निक पतग क दवारा वह अपनी मा को नीच उतारगा इसक कतिलए वह अपनी निपता की जब स -ो बार सवा रपया निनकालकर पतग और -ो मोटी सी मन वाली अपन भाई स काकी एक कागज पर कतिलखवा कर पतग म कतिशव का दि-यानिनकालकर पतग और -ो मोटी सी मन वाली अपन भाई स काकी एक कागज पर कतिलखवा कर पतग म कतिचपका दि-याभोला और शयाम कोठरी म रससी बाधनी रह थ तभी उसक निपता करोध म आकर उन स पछ निक कया उनकी जब स रपया निनकाला हभोला डर क मार बताया निक शयाम इस पतग क दवारा अपनी काकी को राम क यहा स उतारना चाहता हनिवशशवर(शयाम क निपता)न फटी पतग उठाकर -खी तो उस पर काकी कतिलखा थावह हत बजिa होकर वही खड रह गएउनहोन सोचा निक मन अपन पतर को मारा जोनिक अनजान और निन-dरष थावह अपनी मा कोनिकतना पयार करता ह

उस दि-न बड सवर शयाम की नी- खली तो -खा निक घर भर म कोहराम मचा हआ ह

क) घर म कोहराम कयो मचा हआ था शयाम को कया लगा

ख) काकी को ल जात समय शयाम न कया उपदरव मचाया

ग) काकी क बार म उस कया बताया गया कया सतय उस कतिछपा रहा

घ) वह बठा-बठा शनय मन स आकाश की ओर कयोकरता

उततरक) शयाम की मा का -हात हो गया था इसकतिलए

घर म कोहराम मचा हआ था शयाम की लगा निक उसकी मा सफ- कपडा ओढ हए भमिम पर सो रही ह

ख) लोग जब उमा यानी शयाम की मा को उठाकर ल जान लग तब शयाम न बडा उपदरव मचाया लोगो क हाथ स झठ करवा उमा क ऊपर जा निगरा और बोला काकी सो रही ह उस कहा ल जा रह हो

ग) काकी क बार म बजिaमान लोगो न उस निवशवास दि-लाया निक उसकी का निक उसक मामा क यहा गई ह लनिकन सतय अमिधक दि-नो तक कतिछपाना रह सका आसपास क अबोध बालको क मह स यह बात परकट हो गई निक उसकी मा का -हात हो गया ह

घ) कई दि-नकई दि-न लगातार रोत-रोत उसका रोना तो शान हो गया पर उसक ह-य म शोक भर गया था वह चपचाप बठा आकाश की और टाका करता निक शाय- उसकी काकी कही दि-ख जाए

ldquoदि-न उसन ऊपर आसमान म पतग उडती -खी न जान कया सोच कर उसका निहर-य एक-म खिखल उठाrdquo

क) निकसन पतग ऊपर उडत -खी और वह कयो खश हआ

ख) उसन अपन निपता स कया कहा उनका कया उतर थाश

ग) उसन निफर कया निकया और निकसन उसकी सहायता की

घ) उसकी योजना कया थी उततर -क) शयाम न एक दि-न आसमान म पतग उडती

-खी तो उसन सोचा निक पता आसमान म राम क यहा जाकर रकगी वही पर मरी काकी ह यह सोचकर वह बहत खश हआ

ख) उसन अपन निपता स कहा काका मझ एक पतग मगा -ो उसक निपता न भटक हए मन क भाव स कहा निक मगा -ग यह कह कर उ-ास भाव स वह कही और चल गए पतग नही आई

ग) उसन चपचाप निवशशवर क टगहए कोट स एक चवननी निनकाल ली और सखिखया -ासी क लडक भोला की सहायता स एक पतग मगवानी भोला उसकी बराबर उमर का ही था

घ) उसकी योजना यह थी निक वह अपनी पतग को आकाश म राम क यहा भजगा और उस पतग क सहार उसकी काफी नीच उतर जाएगी इस योजना पर उस परा निवशवास था इसकतिलए वह और भोला -ोनो यह काम करन म लग गए

Continue to nexthelliphellipEVS CHAPTER - 1

(UNDERSTANDING OUR ENVIRONMENT)

Sustainable development

The development that meets the needs of the present without compromising the ability of future generations to meet their own needs is called Sustainable development

Sustainable societies ndash

An environmentally sustainable community is one that meets the current and future basic resource needs of its people in a just and equitable manner without compromising the ability of future generations to meet their basic needs

Q ) What are Eco Villages

Ans - Eco village are the urban or rural communities of people who strive to integrate a supportive social environment with a low impact way of life

Q ) To ensure sustainable development the depletion of renewable resources should not take place at a rate faster than their regeneration Justify your answer

Ans ndash Renewable resources do not have a fixed quantity - more can always be

generated However if the rate of use exceeds the rate of renewal - that is the

source is used more than its being recreated - its continued use will become

used up faster than it can regenerate

To promote sustainable society the following things need to be done ndash

1 Using renewable energy sources 2 By improving the quality of human

health 3 By promoting sustainable agriculture 4 By forming ecovillage

it will eventually be entirely depleted So Toensure sustainable development the depletion of

renewable resources should nottake place at a rate faster than their regeneration

Q ) What do you mean by Sustainable societies

Ans - Sustainable societies are defined as towns and cities that have taken steps to remain healthy over the long term These communities value healthy ecosystems use resources efficiently and actively seek to retain and enhance a locally based economy Sustainable development concerns everybody in a society

Q ) What are the effects of pollution on human health

Ans ndash Some health problem occurs due to air pollution are ndash

Respiratory diseases Cardiovascular damage Fatigue headaches and anxiety Irritation of the eyes nose and throat Damage to reproductive organs Harm to the liver spleen and blood Nervous system damage

Some health problem occurs due to water pollution are ndash

Typhoid Cholera Dysentry Jaundice

Some health problem occurs due to noise pollution are ndash

Fatigue headaches and anxiety High blood pressure Hearing damage

Physics Motion in 1D First go through previous notes Now here we will solve some numerical related to that

Question 3What information about the motion of a body is obtained from the displacement-time graphSolution 3From displacement-time graph the nature of motion (or state of rest) can be understood The slope of this graph gives the value of velocity of the body at any instant of time using which the velocity-time graph can also be drawn

Question 4(a)What does the slope of a displacement-time graph represent(b)Can displacement-time sketch be parallel to the displacement axis Give a reason to your answerSolution 4(a) Slope of a displacement-time graph represents velocity(b) The displacement-time graph can never be parallel to the displacement axis because such a line would mean that the distance covered by the body in a certain direction increases without any

increase in time which is not possible

Chemistry Language of Chemistry

How to balance a chemical equationThere are two methods of balancing an equation(i)Hit and trial method(ii)Partial equation methodBalancing by hit and trial methodThis method consists of counting the number of atoms of each elements on both sides and trying to equalize themTake the following steps(i)Count the number of times (frequency) an element occurs on either side(ii)The element with the least frequency of occurrence is balanced first(iii)When two or more elements have the same frequencythe metallic element is balanced firstExample-1 On heatinglead nitrate decomposes to give lead dioxidenitrogen dioxide and oxygenPb(NO3)2rarrPbO+NO2+O2

In this equationLead occurs twiceNitrogen occurs twiceOxygen occurs four timesSince lead is a metalbalance it firstThe number of atom of lead is equal on the two sidestherefore it needs no balancingNow balance nitrogenOn the reactant sidethere are two atoms of nitrogenwhile on the product side oneSomultiply the product containing nitrogenon the product sideby two Pb (NO3)2rarrPbO+2NO2+O2Nowthe number of oxygen atoms on the reactant side 6while on the product sideit is 7Somultiply the entire equation by 2except oxygen to get balanced equation2Pb(NO3)2rarr2PbO+4NO2+O2Multiplication by 2 is done only when atoms of all the elements except one element are balanced and the unbalanced atom occurs separately at least once and also there is a difference of only one such atom

Math Topic AlgebraChapter

Factorisation

Study item Difference of two squares a2 ndash b2 = (a+b) (a-b)1) (i) 4x2ndash 25y2

= (2x) 2 ndash (5y) 2= (2x + 5y) (2x - 5y)

(ii) 9x2 ndash 1= (3x)2ndash(1)2= (3x + 1)(3x ndash 1)

2) (i) 150 ndash 6a2= 6(25 ndash a2)= 6(5)2 ndash(a)2= 6 (5 + a) (5 ndash a)

(ii) 32x2 ndash 18y2=2(16x2 ndash 9y2)=2(4x)2 ndash (3y)2= 2(4x + 3y)(4x - 3y)3)(i) (x ndashy )2 ndash 9 = (x ndash y )2 ndash (3)2= (x ndash y + 3) (x ndash y ndash 3)(ii) 9(x + y) 2ndash x 2= (3)2(x + y)2 ndash (x)2=3(x + y)2 ndash (x)2= (3x +3y ) 2ndash(x)2= (3x + 3y + x)(3x +3y ndash x)= (4x + 3y) ( 2x + 3y )

Commercial studies

Basic accounting terms

Today I will give you some questions from the previous study material

Questions1) Define accounting2) What do you mean by debit and

credit

3) Explain the types of account4) Define the following terms

a) Assetsb) Capitalc) Purchased) Debtorse) Transactions

5) Name the types of accounts given below

a) Krishnas accountb) Machinery accountc) Royalty accountd) Salary accounte) Furniture accountf) Audit fee account

Economics Revision Today I will give you some revision questions

Questions1) What do you mean by the terms

rdquowantsrdquo2) Write the difference between

consumer goods and producer goods

3) Define the term utility 4) Explain the different types of utility5) Define

a) Total utilityb) Marginal utility

Subject Eng Literature (The Merchant of Venice ndash William Shakespeare)Topic Act I Scene 3 Lines 1 to 48 (Shylock hellip Cursed be my tribe if I forgive him) Date 16th April 2020 (5th Period)

[Students should read the original play and also the paraphrase given in the school prescribed textbook]Summary Questions amp Answers

This scene takes place in Venice and we are introduced to the rich Jew Shylock Bassanio and Shylock are talking and Bassanio tells Shylock that he wants a loan of three thousand ducats for three months on the personal security of Antonio

o Shylock feels glad because he will be able to bind down Antonio by means of a bond on account of the loan but he tells Bassanio that all the fortunes of Antonio being invested in the merchant ships on the sea it is difficult to depend upon his credit Even under such circumstances Shylock is willing to advance the money on the personal security of Antonio

o Bassanio then invites Shylock to dine with him Shylock says that he is prepared to do anything with the Christians but not eat or drink or pray with them

o While Bassanio and Shylock are talking Antonio appears on the scene Shylock does not seem to take any notice of Antonio but goes on brooding within

(1) SHYLOCK Ho no no no no- my meaning in (Line 15-26)saying he is a good man is to have you understand me that he is sufficient Yet his means are in suppositionhe hath an argosy bound to Tripolis another to the Indies I understand moreover upon the Rialto he hath a third at Mexico a fourth for England and other ventures he hath squanderd abroad Butships are but boards sailors but men there be land-rats and water-rats land-thieves and water-thieves I mean pirates and then there is the peril of waters winds and rocks The man is notwithstanding sufficientmdashthree thousand ducats mdashI think I may take his bond

(a) Who is talking in the beginning of this scene What does Bassanio want from Shylock How does Shylock feel

In the beginning of the scene Bassanio and Shylock are talking to each other Bassanio wants to borrow three thousand ducats from Shylock for three months on the security of Antonio Shylock feels glad at heart that he will get the opportunity of binding Antonio with a bond(b) What risks does Shylock weigh in advancing the money

Shylock says that Antonio has invested all his capital in trading by sea-going ships But the ships are made of wood and the sailors of those ships are ordinary human beings The wood can

himself how he hates Antonio because of his being a Christian because he abuses Shylock in public places Shylock decides that if ever he can get Antonio to his advantage he will teach him a lesson

come to harm and men can commit mistakes and thus the capital invested in ships may be lost Then there are other dangers The goods loaded on the ships can be damaged by rats and thieves which are found both on land and water The ships can also be harmed through sea-storms submerged rocks etc(c) What two important functions does this scene have

The scene has two important functions First it completes the exposition of the two major plot lines of the play Antonio agrees to Shylockrsquos bond ndash three thousand ducats for a pound of flesh and second and more important dramatically this scene introduces Shylock himself In this scene Shakespeare makes it clear at once why Shylock is the most powerful dramatic figure in the play and why so many great actors have regarded this part as one of the most rewarding roles in all Shakespearean dramas(d) Where does this scene take place What kind of treatment has Antonio been giving to Shylock What does Shylock say when Bassanio invites him to dine with him

The action of this scene takes place in Venice Antonio has been in the habit of behaving harshly with Shylock ndash spitting on his beard and footing him like a stranger cur When Bassanio invites Shylock to dine Shylock says that he is prepared to do anything with the Christians but not eat and drink or pray with them

(2) SHYLOCK How like a fawning publican he looks (Line 38-48)I hate him for he is a Christian

But more for that in low simplicity

He lends out money gratis and brings downThe rate of usance here with us in VeniceIf I can catch him once upon the hipI will feed fat the ancient grudge I bear him

He hates our sacred nation and he railsEven there where merchants most do congregateOn me my bargains and my well-won thriftWhich he calls interest Cursed be my tribeIf I forgive him

(a) What is the context in which these words are spoken and what is the idea expressed in it

These remarks are made by Shylock when he sees Antonio coming along after Bassanio told him that the merchant will be his surety for the bond The above mentioned passage reveals Shylockrsquos hatred for Antonio Shylock says that he hates Antonio because he is a Christian and also because he gives loan without taking interest on them thereby bringing down the rate of interest in Venice(b) Explain the meaning of the phrase lsquoa fawning publicanrsquo

The phrase lsquoa fawning publicanrsquo refers to Roman tax collector It is a term of contempt and hatred on the lips of a Jew lsquoFawning Publicansrsquo were Roman tax-gatherers whose ordinary bearings towards the Jews was bullying but whose false pose of lsquohumility and contritionrsquo is touched upon in the parable in New Testament(c ) What light does the above passage throw on the character

of Shylock

The above mentioned speech of Shylock reveals him to be a wicked character having an extreme greed for wealth His intense hatred for Antonio is unjustified He hates Antonio only because he is a Christian and because he lends money without taking any interest on it thereby adversely affecting Shylockrsquos business of lending money on high interest(d) What information do you gather about Antonio from the above given lines

Shylockrsquos statement throws a valuable light on the character of Antonio Antonio appears to be a good Christian and a good human being He helps the people in need by lending them money without charging any interest on it He is a man of simple and good nature This very goodness makes him Shylockrsquos enemy(e) What does Shylock debate within himself and when To whom are the lines mentioned above addressed to

When Bassanio asks the Jew to lend him three thousand ducats on Antoniorsquos surety Shylock begins to debate within himself as to how he should exploit the opportunity of a business deal with his old enemy Antonio

The lines mentioned above are not addressed to anyone The lines are a soliloquy ie a speech made by a character to himself and not meant to be heard by the other characters present

Class XSubject Topic Summary ExecutionEnglish

LiteratureThe Blue Bead 2nd part

Things took a turn for the worst and all of a sudden a crocodile attacked the woman biting on the womanrsquos leg At that moment Sibia got up sprinted grabbed the hay fork and stabbed the crocodile in the eye with all her power Immediately the crocodile let go and went away Sibia saw a small blue bead lying by the river she grabbed it Since she was poor she didnrsquot have necklace Shersquod always wanted one like the other women now she could make one with the blue bead After that she went home and told her mother all about it

Hindi 2nd

Langबड घर की बटी( मशी परमच-)

lsquoबड घर की बटी कहानी का उददशय मधयम वग की घरल समसया को सलझा कर सगदिठत परिरवार म मिमल जलकर परम स रहन का स-श -ना हघर म शानित सथानिपत करन की जिजमम-ारी नारी की होती ह यदि- नारी समझ-ार ह उसम धय और परिरवार क परनित परम ह तो कोई भी घटना परिरवार को निवघदिटत नही कर सकती या कहानी परिरवार को सगदिठत करत हए परम सौहा- स एक दसर की भावनाओ को समझ करउनका सहयोग करत हए जीवन यापन करन की पररणा -ती हमशी परमचदर जी न इस कहानी म सय परिरवार का परनितनिनमिधतव निकया ह यह कहानी बनी माधव सिसह जो गौरी पर क जमी-ार क उनक -ो पतरो की हशरी कठ लाल निबहारीशरीकात का निववाह एकजमी-ार घरान की पतरी आन-ी स हआ थाआन-ी न ख- को ससराल क वातावरण म ढाकतिलया थाएक दि-न आन-ी का अपन -वर लाल निबहारी स झगडा हो जाता ह -ोनो भाई एक दसर स अलग होन की कोकतिशश करत हसभी बह आन-ी न अपन मधर वयवहार स लाल निबहारी को

ldquoइन नतर निपरय गणो को बीए-इनही -ो अकषर पर नयोछावर कर दि-या था इन -ो अकषर न उनक शरीर को निनबल और चहर को कानित ही बना दि-या थाldquo

क) परसतत पकतियो म निकस वयकति क बार म कहा गया ह

ख) इन पकतियो म कौन स नतर निपरय गणो क बार म कहा गया ह

ग) बीए की निडगरी परापत कर लन पर भी उपय वयकति क सवभाव की कया निवशरषता थी

घ) यह नतर निपरय गण निकस वयकति म निवदयमान थ उसक वयकतितव की कया निवशरषता थी

उततर ndashक) परसतत पकति म गौरी पर गाव क जमी-ार

क बड बट शरीकात क बार म कहा गया ह उसन बहत परिरशरम और उ-म क बा- ba की निडगरी परापत की थी अब वह एक -फतर

घर छोडकर जान स रोक कतिलयाइस पर बनी माधव सिसह न कहा निक बड घर की बटी ऐसी ही होती ह जो निबगडा काम बना लती ह अतः शीरषक साथक ह बड घर की बटी आन-ी ह

म कमचारी थाख) भरा हआ चहरा चौडी छाती और डटकर

खाना आदि- एक सबजी ल जवान क गण मान जात ह परत शरीकात न इनही नतर निपरय गणो को अपनी पढाई पर नयोछावर कर दि-या था

ग) बीए की निडगरी परापत कर लन पर भी उपय वयकति(शरी कठ की शारिररिरक तौर पर निनबल और चहर स कानित ही लगत थ इतना ही नही वह मानकतिसक तौर पर भी निपछड हए थ पाशचातय सामाजिजक कथा उस घणा एव पराचीन सभयता का गणगान उनकी निवचारधारा क परमख अग थ

घ) यह नतर निपरय गण गौरीपर गाव क जमी-ार क छोट बट लाल निबहारी सिसह म निवदयमान थ वह सजीलाजवान था और भस का दध शर दध वह सवर उठकर पी जाता था

ldquoयही कारण था निक गाव की लललन आए उनकी निन-क थी कोई कोई तो उह अपना शतर समझन म भी सकोच ना करती थी सवय उनकी पतनी को इस निवरषय म उनस निवरोध थाldquo

क) उपय पकति म इस वयकति क बार म कहा गया ह

ख) गाव की लललन आए उनकी निन-ा कयो निकया करती थी

ग) उनकी पतनी का कया नाम था उनह निकस निवरषय म अपन पनित क निवरa था और कयो

घ) इस कहानी का कया उददशय ह Continue to next helliphelliphellip

Bengali 2nd Language

ফ ফটক না ফটক( কহিতা )

পর) ldquo(ান াধাচেনা ফটপাচেথ পাথচের পাড হিচেয় এক কাঠচোটটা গাছ কহিকহি পাতায় পার ফাটিচেয় াসচেছldquoক) কার দো দেকান কহিতার অং( ) lsquo(ান াধাচেনা ফটপাচেথ পাথচের পাডহিচেয়lsquo চেত কী দোঝাচেনা চেয়চেছ গ) আচো য অংচে(lsquo এক কাঠচোটটা গাছ lsquoচেত কী দোঝাচেনা চেয়চেছ ঘ) ldquoকহিকহি পাতায় পার ফাটিচেয় াসচেছldquo ----- একথার পরকত অথC কী উততর ) ক) আচো য অং(টি পর যাত কহি সভা4 মচোপাধ যাচেয়র দো lsquoফ ফটক না ফটকrsquo কহিতার অং()কহি সভা4 মচোপাধ যায় হিছচেন দেপরচেমর কহি দেপরমচেক নানা ভহিঙগমায় হিতহিন ফটিচেয় তচেচেছন দেপরম মানচের স মচেতC র সঙগী কহিতার কহিতায় এক রb সb হচেয়র দেপরম াগরচেনর কথা চেচেছন (ান অথCাৎ দেযাচেন দেকান রস দেনই দেযাচেন দেকান মহিনতা দেনই অথ তার মধ দেযও দেপরম থাকচেত পাচের একথাই কহি তচে ধরচেত দেচেয়চেছন একটি মানচে4র মচেন দেযাচেন দেকামতার দেকান সথান দেনই পাথচেরর মচেতা হিনরসতার মচেনর মধ দেযও দেয দেপরম আসচেত পাচের দেস কথাই কহি চেচেছনগ)নারীচের যথC দেপরচেমর ছহি এই কহিতায় অকপচেট উচেঠ এচেসচেছ কহি এই কহিতায় কাটচোটটা গাছ কথাটি যার কচেরচেছন নারী দেয দেপরম দেথচেক হিতাহিত এং দেসই দেপরম সঠিক সমচেয় না পাওয়ার ন য দেপরম সমপচেকC হিচেr4 গৈতরী য় দেপরচেমর দেয গৈহি(ষট য মাধযC য সরসতা দেকামত এই সমসতর হিপরীত যথা রbতা শষকতা কচেঠার তা পরভহিত দোঝাচেত এক কাঠচোটটা গাছ কথাটি যার কচেরচেছনঘ) এাচেন এক নারীর যথC দেপরচেমর কথা হিনহিCপত ভাচে চেচেছন কহি অসমচেয় নারীর ীচেন দেপরম দেচেগচেছ এতহিন তার হয় রb কচেঠার হিছ দেপরচেমর অভাচে ঠাৎ দেসই শষক মরভহিমচেত সচের আভাস এচেসচেছ দেপরম দেযন 4Cার স(ীত তাই পরায় মত গাচেছ কহিকহি পাতা গহিচেয় উচেঠচেছ

Biology Chapter - 01Controlling Air Pollution

Today we will discuss how we control air pollution from domestic combustion

Q1Describe any five ways of reducing air pollution from domestic sources bull The number of pollutants in the air is verylarge and we always try to control them byfollowing ways

i) Solar cooker and solar heater It use no fuel reduce damage of environment by fuel use or reducing deforestation It maintains coolness of house It releases very less orno oil gas or grease

ii) Piped natural gas (PNG) It emits very less by products into the atmosphere As it isdistributed through pipe lines so there iscontinuous supply of fuel is possible

iii) Liquefied Petroleum Gas (LPG) It hasa higher heating value LPG doesntcontain sulphur so it burns a lot cleanerenergy sources It releases very less oralmost no fume in air

iv) Electricity based cooking Emission free cooking alternative for urban dwellers causeselimination of adverse health impactsofindoor air pollution It helps to avoid theinconveniences associated with procurement of LPG

v) Biogas It contains 75 methane whichmakes it an excellent fuel It burns without smoke and biogas plant leaves no residue like ash in wood charcoal etc Thus it isaclean fuel

Economics

Factors of Production

Today firstly we would recall the last class for 5 mins and then we would proceed with the further topics of the chapter

The concept meaning of land characteristics of land and importance of land to be repeated for the absentees as well as the students who were there in the class the previous day

Today we will start with the last portion of land before it the meaning of land to be repeated onceAs by now we all know that

Questions1What do you mean by productivity of landAnswer By productivity of land we mean the capacity of a piece of land to produce a crop

Thus it refers to the average output per unit of landSay per acre per hectare etc= (OutputArea of land)

2 What are the factors influencing the productivity of landAnswer

Natural factors Productivity of land is largely determined by the natural

Land is defined to include not only the surface of the earth but also all other free gifts of nature(for example mineral resources forest resources and indeed anything that helps us to carry out the production of goods and services but is provided by nature free of cost)

We will move on to the last portion of land by discussing Productivity of Land

By productivity of land we mean the capacity of a piece of land to produce a crop

Thus it refers to the average output per unit of land

Say per acre per hectare etc= (OutputArea of land)

With this we shall proceed further with the main factors that determine the productivity of land

Natural factors Human factors Improvements on land Location of land Organisation Ownership of land Availability of capital Proper use of land State help

Note economic development of a country depends upon the quality of its land If the land is fertile it will quicken the pace of development of the country

qualities of land such as fertility etc

Human factors Land cannot produce anything by itself Man has to apply labour on it to produce for himself So productivity of land depends on the knowledge and skills of workers

Improvements on land production of land is affected by land development measures like provision of well or tubewell irrigation proper drainage

State help The government of a country especially less developed country can play a vital role in improving the agricultural productivity by providing better irrigation facilities

Organisation Productivity of land also fdepends upon the way how the factors of production like labour and capital are organised

In order to increase productivity trained workers modern implements scientific methods good seeds are all essential

3 lsquoImproved technology affects the productivity of landrsquo Explain this statement with the help of suitable example Answer Use of improved technology raises the productivity of land Example By using HYV seeds chemical manures and modern machines per hectare output increases

Physics Force (Summary)

Question Write the expression for the moment of force about a given axisSolutionsThe expression for the moment of force is given byMoment of force about a given axis = Force times perpendicular distance of force from the axis of rotationQuestion What do you understand by the clockwise and anticlockwise moment of force When is it taken positiveSolutionsIf the effect on the body is to turn it anticlockwise moment of force is called the anticlockwise moment and it is taken as positive while if the effect on the

body is to turn it clockwise moment of force is called the clockwise moment and it is taken as negative

Math Topic Commercial Mathematics

Chapter Goods and services Tax

Study item Some solved sums from exercise ndash 1 A retailer buys a TV from a wholesaler for Rs 40000 He marks the price of the TV 15 above his cost price sells it to the consumer at 5 discount on the marked price If the sales are intra ndash state and the rate of GST is 12 find

(i) The marked price of the TV(ii) The amount which the consumer pays for the TV(iii) The amount of tax (under GST) paid by the retailer to the central

Government(iv) The amount of tax (under GST) received by the State Government

Solution As the sales are intra- state sale and the rate of GST 12 So GST comprises of 6 CGST and 6 SGSTTherefore a retailer buys a TV from a wholesaler for Rs 40000Therefore the amount of GST collected wholesaler from the retailer or paid by retailer to wholesalerCGST = 6 of Rs 40000 = Rs(6100 times40000) =Rs 2400SGST = 6 of Rs 40000 = Rs (6100 times 40000) =Rs 2400Therefore wholesaler will pay Rs 2400 as CGST and Rs 2400 as SGSTTherefore amount of input GST of retailer Input CGST = Rs 2400 and input SGST = Rs 2400Again the retailer marks the price of the TV 15 above his cost price(i) The marked price of the TV

= Rs 40000 + Rs 40000times15= Rs 40000 + Rs 40000times 15100= Rs 40000 + Rs 6000Rs 46000But the retailer sells it to consumer at 5 discount on the marked priceCost price after discount = Rs 46000 ndashRs46000times 5100 =Rs 46000 ndashRs 2300= Rs 43700Therefore the amount of GST collected retailer from consumer or paid by consumer to retailerCGST = 6 of Rs 43700 =Rs ( 6100 times43700)Rs 2622SGST = 6 of Rs 43700 = Rs (6100 times 43700) =Rs 2622Amount of the output GST of retailer Output CGST = Rs 2622 and output SGST = Rs 2622

(ii) The amount which the consumer pays for the TV= cost price of TV to consumer + CGST paid by consumer + SGST paid by consumer= Rs 43700 + Rs 2622 + Rs 2622= Rs 48944

(iii) The amount of tax (under GST ) paid by the retailer to the central Government=CGST paid by retailer = output CGST ndash input CGST=Rs 2622 ndash Rs 2400=Rs 222

(iv) The amount of tax ( under GST ) received by the State Government = SGST paid by wholesaler + SGST paid by retailer= Rs 2400 + output SGST ndash input SGST=Rs 2400 + Rs 2622 ndash Rs 2400=Rs 2400 + Rs 222= Rs 2622

Commercial studies

Stakeholders Today I am going to give some revision questions from the previous study material

Questions1) State the two expectations of

employees from a business concern2) Give two distinctions between

stakeholder and shareholder3) Give two difference between

internal stakeholders and external stakeholders

4) Give two expectations of suppliers from a business organisation

5) Who is a stakeholder in commercial organisations

Chemistry Periodic Table

Merits of Mendeleevrsquos Periodic law are as follows - 1He grouped the elements on the basis of atomic mass 2 He left gaps for undiscovered elements like Gallium Scandium germanium Also he left a full group vacant for undiscovered inert gases 3 He could predict proportions of several elements on basis of their position in periodic table like Ga Sc etc 4He could predict errors in atomic weights of some elements like gold platinum etc

Anomalies in Mendeleevrsquos Periodic law are as follows - 1 Position of isotopes could not be explained 2 Wrong order of atomic masses could not be explained

For example- as Arnur atomic mass 40 come first and K with low atomic mass (30) should come later but k should be placed first

According to Bohrrsquos Modern Periodic table properties of elements are periodic functions of their atomic numbers

So when elements are arranged according to increasing atomic numbers there is periodicity in electronic configuration that leads to periodicity in their chemical properties

It consists of horizontal rows (Periods) Vertical column (Groups)

There are 7 period and 12 groups in this long form of periodic table

Ist period has 2 elements IInd period has 8 elements IIIrd period has 8 elements IVth period has 18 elements Vth period has 18 elements VIth period has 32 elements VIIth period hs rest of elements

Note - The number of valence electrons in atom of elements decides which elements will be first in period and which will be last

In group- 1 to 2 gp and 13 to 17 contain normal elements 3 to 12gp ndash transition elements 57 to 71 - lanthanides 89 to 103 - Actinides

Left hand side ndash metals Right hand side ndash nonmetals

Note- Hydrogen element has been placed at top of Ist group Electronic configuration of H is similar to alkali metal as both have 1 valence electron

V electron of gp I element -- 1 V electron of gp 2 element -- 2 V electron of gp 13 element -- 3 V electron of gp 14 element -- 4 V electron of gp 15 element -- 5 V electron of gp 16 element --6 V electron of gp 17 element -- 7 V electron of gp 18 element -- 8

English 1 Transformation of sentences

Sentences A sentence is a group of words which makes complete sense

Exercise 2Change the following sentences from

a Assertive sentencesb Imperative sentencesc Interrogative sentencesd Exclamatory sentences

Sentences can be changed from one grammatical form to another without changing the meaning of the sentence This is known as transformation of sentences

assertive to interrogative1 Nobody would like to be a fool

Who would like to be a fool2 Their glory can never fade

When can the glory fade3 Nobody can control the wind

Who can control the wind4 It matters little if I die

What though I die5 No man can serve two masters

Can any man serve two masters

Exercise 3Interchange of assertive and Exclamatory sentences

1 She leads the most unhappy lifeWhat an unhappy life she leads

2 This is indeed an interesting bookWhat an interesting book this

3 He is a very great manWhat a great man he is

4 It is a very lame excuseWhat a lame excuse

5 It is sad that she died so youngAlas she died so young

Class XISubject Topic Summary Execution

Hindi 2nd lang

पतर परम(परमचदर) पतर परम कहानी म एक निपता की इचछाओ का वणन निकया गया ह अपन बड पतर परभ -ास स निपता चतनय -ास का निवशरष परम था निपता को उसक जनम स ही बडी-बडी आशाए थी उसम दसर बट कतिशव-ास की अपकषा स- उतसाह की मातरा अमिधक थी वह उस इगलड भजकर बरिरसटर बनाना चाहत थभागय का खल भी बडा निनराला ह बीए की परीकषा क बा- वह बीमार पड गया डॉकटरो न भी जवाब - दि-या थाचतन -ास जी बहत ही कजस थ बवजह पस खच करना नही चाहत थ अगर गारटी मिमलती तो शाय- पस खच भी कर -त परत गारटी नही थी परिरणाम सवरप उनक बट का -हात हो गयाजब बट को समशान ल जा रह थ तो वहा काफी शोर गान बजान हो रह थ पछन पर पता चला निक निकसी निपता निपछल तीन साल स निबमार था और उसक ईलाज म रपया पानी की तरह बहाया पर ठीक नही हए परत उसक बट को तनिनक भी अफसोस नही था उसका कहना था उसन कोकतिशश तो कीयह -खकर चतनय-ास जी को आतम निगलानी हईतभी स उनका म परिरवतन हआ और बट का भोज काफी धमधाम स निकयाऔर वहइस पशचाताप की आग म जलत रह औला- स बढकर पसा नही होता ह इस बात को समझन म उनह काफी व लग गया

hellipContinue to next

BENGALI(2ND LANGUAGE)

পরথমঅধযায়-ঠাকরারীনদরনাথঠাকর

নয়ন দোচের হিমাচেররা া নাচেমই হিযাত হিছচেন ায়ানার উাররণ সবরপ নয়ন দোচের ারা হিা (াচেকর হিা হিচেতন এছাাও দেকান উৎস উপচেb রাহিতর দেক হিন করার উচেfচে(য তারা সযC হিকরচেরণ রনয পরীপ জবাহিচেয় তাচেত রপার হির 4Cরণ করচেতন ঠাকরা এই নয়ন দো হিমারচের দে(4 ং(ধর হিছচেন হিমাররা ায়ানার ষটানত পর(Cন কচের তারা হিনঃসব এই হিমাহিরর দে(4 ং(ধর গৈকাস নদর রায়চেৌধরী গৈকাস া নয়ন দোচের সমসত সমপহিতত ঋচেরণর াচেয় হিহিx কচের অহি(ষট যা আচেছ তাচেত হিপত

ইার হিপতার মতয ইচে পর নয়নচোচের ায়ানার দেগাটা কতক অসাধাররণ শরাদধ (াহিনতচেত অহিনতম ীহিপত পরকা( কহিরয়া ঠাৎ হিনহিয়া দেগ- ক) কার দো দেকান গচেলপর অং() কতা দেক ইার চেত কাচেক দোঝাচেনা চেয়চেছ গ) পরসঙগ কী কতার কতয পহিরসফট কচেরা

পরচে4র যাহিত রbা করা সমভ নয় তাই হিতহিন পতরচেক হিনচেয় ককাতায় সাস শর কচেরন গলপ কথচেকর আহিথCক অসথা নয়ন দোচের হিমাচের দেথচেক সমপরণC আাা কথচেকর হিপতা হিনচের দেষটায় অথC উপাCন করচেতন া উপাহিধ াচেভর নয তার াসা হিছনা আর দেসই কারচেরণ কথক তার একমাতর উততরাহিধকার চেয় তার হিপতার পরহিত কতজঞ কথক দো পা হি(চেচেছন হিনচের পরারণ ও মান রbার নয উপচেযাগী অথC হিনা দেষটায় পরাপত চেয়চেছন- এটাই তার কাচেছ পরম দেগৌরচের হি4য় চে মচেন কচেরন কাররণ (নয ভাণডাচের গৈপতক ায়ানার উজজব ইহিতাস অচেপbা দোার হিসeচেকর মচেধয গৈপতক দেকামপাহিনর কাগ তার কাচেছ অচেনক দেহি( মযান

TO BE CONTINUED

উ- ক) আচোয অং(টি রীনদরনাথ ঠাকচেরর দো ঠাকরা গচেলপর অং() কতা চেন আচোয গচেলপর গলপ কথকইার চেত নয়ন দোচের হিমাহিরর দে(4 ং(ধর গৈকাস ার কথা া চেয়চেছ গৈকাস া নয়ন দোচের সমসত সমপহিতত ঋচেরণর াচেয় হিহিx কচের অহি(ষট যা আচেছ তাচেত হিপত পরচে4র যাহিত রbা করা সমভ নয় তাই হিতহিন পতরচেক হিনচেয় ককাতায় সাস শর কচেরনগ) গৈকাস ার হিপতার মতযর পর নয়ন দোচের হিমাহিরর অহিসততব হিপত য় কচেয়কটা উৎস ও শরাদধ- (াহিনতচেত হিমাহিরর দে(4 কহিটক যয় চেয় হিগচেয় এচেক াচের দে(4 চেয় যায় তন তাচের গC করার মত আর হিকছই হিছ না-দেসই পরসচেঙগ এই উহিকত নয়নচোচের হিমাচেররা া নাচেমই হিযাত হিছচেন ায়ানার উাররণ সবরপ নয়নচোচের ারা হিা (াচেকর হিা হিচেতন এছাাও দেকান উৎস উপচেb রাহিতরচেক হিন করচেত হিগচেয় তারা সযC হিকরচেরণর নয পরীপ জবাহিচেয় তাচেত রপার হির 4Cরণ করচেতন তাই দেসকাচের ায়ানা দেহি(হিন সথায়ী চেত পারত না হিহিভনন উৎস শরাদধ- (াহিনতচেত সাধযা হিতহিরকত র করার নয হিমাহির হিহিকচেয় দেযত হ হিতC কা হিহি(ষট পরীচেপর দেত দেযমন অলপকাচের মচেধয হিনঃচে(4 চেয় যায়-নয়নচোচের হিমারচের অসথা তাই চেয়হিছ এই কারচেরণই কথক নয়নচোচের হিমারচের গা ভরা আমবর সয করচেত পারতনা

Physics Dimensional Analysis (Summary)

Q Find the dimensions of consts ab in relation

p=(bminusxlowastx)at

where p is the power x is the distance and t is time

Ans From principle of homogeneity dimension of b x2 are same Dim of b = dim of x2 = [L2] = [ML2T0]Dim of a = dim of ( b- x2)dim of (pt) = [M0L2T0][ML2T-2] [T-1] [T] = [M-1L0T2]

Chemistry Atomic Structure Drawbacks of Rutherfordrsquos model of

atom a According to Rutherfordrsquos model of atom electrons which are negativelycharged particles revolve around the nucleus in fixed orbits Thusb theelectrons undergo acceleration According to electromagnetic theory of Maxwell a charged particle undergoing acceleration should emitelectromagnetic radiation Thus an electron in an orbit should emitradiation Thus the orbit should shrink But this does not happenc The model does not give any information about how electrons aredistributed around nucleus and what are energies of these electrons Isotopes These are the atoms of the same

Properties of electromagnetic radiationsa Oscillating electric and magnetic field are produced by oscillating charged particles These fields are perpendicular to each other and both areperpendicular to the direction of propagation of the waveb They do not need a medium to travel That means they can even travel invacuum

Characteristics of electromagnetic radiationsa Wavelength It may be defined as the distance between two neighbouring crests or troughs of

element having the same atomicnumber but different mass numbere g 1H11H21H3

Isobars Isobars are the atoms of different elements having the same massnumber but different atomic numbere g 18Ar40 20Ca40

Isoelectronic species These are those species which have the same numberof electrons

Electromagnetic radiationsThe radiations which are associated withelectrical and magnetic fields are called electromagnetic radiations When anelectrically charged particle moves under acceleration alternating electricaland magnetic fields are produced and transmitted These fields aretransmitted in the form of waves These waves are called electromagneticwaves or electromagnetic radiations

wave as shown It is denoted by λb Frequency (ν) It may be defined as the number of waves which passthrough a particular point in one secondc Velocity (v) It is defined as the distance travelled by a wave in onesecond In vacuum all types of electromagnetic radiations travel with thesame velocity Its value is 3 times10 8m sec-1 It is denoted by v

d Wave number Wave number is defined as the number of wavelengths per unit lengthVelocity = frequency timeswavelength c = νλ

Plancks Quantum Theory- o The radiant energy is emitted or absorbed not continuously but discontinuously in the form of small discrete packets of energy called lsquoquantumrsquo In case of light the quantum of energy is called a lsquophotonrsquoo The energy of each quantum is directly proportional to the frequency of the radiation ie E α υ or E= hυ where h= Planckrsquos constant = 6626 x 10-27 Js o Energy is always emitted or absorbed as integral multiple of this uantum E=nhυ Where n=1234Black body An ideal body which emits and absorbs all frequencies is calleda black body The radiation emitted by such a body is called black body radiation

Photoelectric effectThe phenomenon of ejection of electrons from thesurface of metal when light of suitable frequency strikes it is calledphotoelectric effect The ejected electrons are called photoelectrons

Biology Chapter - 02Systematics and Five Kingdoms

Scientists divide the whole living organisms into two kingdom first and ultimately by five kingdom at last

In the earlier systems of classifications organisms are divided into kingdom plantaeand kingdom animalia on the of presenceof cell wall their modes of nutrition and movements

Some problem arise like fungi share manycharacteristic withplant despite their heterotrophic nutrition bacteria protozoa areunicellular present in both kingdom Toovercome this third kingdom Protista isintroduced which include

unicellularorganisms But there is also another

problem Allunicellular organisms are not similar kind The cellular structure of prokaryotes is verydifferent from that of other organismsEukaryotes possess a true nucleus and allcell organelles that are not present inprokaryotes So the fourth kingdom Monerais introduced which include unicellular prokaryotes (bacteriaamp blue green algae)

bull Still some problem arise in kingdomplantae

So in 1969 R H Whittakar proposedanew five kingdom System of classification

i) Kingdom Monera - unicellular prokaryotes

ii) kingdom Protista - unicellular eukaryotes

iii) Kingdom Fungi - uni or multicellular fungi with cell wall but without chlorophyll

iv) Kingdom Plantae - Multicellular Plants

v) Kingdom Animalia - Multicellular Animals

EVS Chapter 1 ndash Modes of Existence

An agricultural society

An agricultural society also known as an agrarian society is a society that constructs social order around a reliance upon farming More than half the people living in that society make their living by farming

People in an agricultural society generally lead a more settled lifestyle than those in nomadic hunter-gatherer or semi-nomadic pastoral societies because they live permanently near the land that is farmed Agricultural settlements tend to develop in areas of convenience near bodies of water which is used for both crops and transportation or along trade routes Not everyone in an agricultural society is a farmer Some people make a living trading or making and selling goods such as tools used for farming

Another way to define an agrarian society is to see the total amount of production in a nation In an agrarian society cultivating the land is the main source of wealth Such a society can recognize other means of subsistence and work habits but emphasizes the importance of agriculture and livestock Agrarian societies have existed in various parts of the world for 10000 years and continue to exist today They have been the most common form of socio-economic organization for most of recorded human history

Q) Write the features of agricultural society

Ans - Structure and Features of Agrarian Society1 Occupational Structure

An agrarian society is generally associated with the domestication of plants and animals The domestication of plants means farming and that of animals means herding Often there is mixture of farming and the use of such domesticated animals as cow goat and sheep

2 Forms of Land Ownership in Agrarian SocietiesGenerally there are landlords supervisory farmers cultivators and share croppers The landholders own the land but do not work on it They let it out for sharecropping The supervisory farmers are those who live by having their land cultivated by hired labourers The cultivators cultivate the land for themselvesThe share-croppers are those who live by tilling other peoplersquos land or a crop-sharing basis The artisans own their means of production and produce by their own labour in their homesteads

3 Village Community System An agrarian society is highlighted by

the institution of village community system The agrarian economy made fixed dwelling houses necessary Living close together for protection and co-operation and living nearer to the land gave birth to agricultural villages The village is not only the residential place of farmers it is also the social integrator

4 Minimal Division of Labour Another structural feature of agrarian society is a minimal division of labour Except for the basic division founded on age and sex differences there are few specialized roles There is only one predominant type of occupation ie domestication of plants and animals For all the people the environment physical as well as social is the same

5 Role of Family The farm family is of the patriarchal type the father is the final arbiter in most of the familyrsquos major decisions The life of ail men and women is merged in family life Since there are not many special organizations family is the only organisation to perform the tasks of aid and protection

6 Sense of Unity The members of an agrarian society exhibit a strong in-group feeling Since the whole of their social lives is wrapped up in a society which is physically economically and socially homogenous they are inclined to view the entire outside world as an out group

7 Informal Social Control An agrarian society is regionally divided into villages In a village community the force of traditional mores is more dominant than in the urban community In the village everybody is known to everybody The members in a village community help each other and share the joy and sorrows of each other Crime in an agrarian society is rare

8 Simplicity and Uniformity Life of the people in an agrarian society is marked by simplicity and uniformity Their main occupation is agriculture which largely depends upon the vagaries of nature An agrarian society is a religious society

Math Compound angles Compound angles The algebraic sum of two or more angles is called a compound angle If A B C be three angles then A+B B+C C+A A-B B-C A-C A+B-C etc are compound angles In this chapter we shall discuss the trigonometrical ratios of compound angles Theorem 1 If A B and A+B are all pisitive acute angles theni) sin( A+B) = sin A cos B + cosA sinBii) cos(A+B) = cosA cosB- sinA sinBTheorem 2If A and B are positive acute angles and AgtB theni) sin(A-B) = sin A cosB- cos A sinBii) cos(A-B) = cos A cos B+ sin A sin BTo prove that i) sin(A+B) sin (A-B) = sin2 A - sin2 B = cos2 B- cos2 A

Example 1 Prove that tan70deg=2tan50deg+tan20degSolutiontan70deg = tan(50deg + 20deg)Or tan70deg=(tan 50deg+tan 20deg)(1-tan50degtan20deg) or tan70deg (1 ndash tan 50deg tan20deg) = tan50deg+tan20degor tan70deg= tan70deg tan50deg tan20deg+ tan50deg + tan20deg = cot20deg tan50deg tan20deg + tan50deg + tan20deg = 2 tan50deg+ tan20degExample 2 If A + B = 45deg show that (1 + tanA) (1 + tanB) = 2Solutiontan(A + B) =( tan A + tan B) (1 - tan

ii) cos(A+B) Cos(A-B) = cos2 A- sin2 B = cos2 B -sin2 AProof i) LHS= sin(A+B)sin(AminusB) [Recall sin(αminusβ)=sinαcosβminuscosαsinβ And sin(α+β)=sinαcosβ+cosαsinβ]= (sinAcosB+cosAsinB)times(sinAcosBminuscosAsinB)= sin2Acos2Bminuscos2Asin2B [Recall sin2α+cos2α=1 From above we can then assume correctly that sin2α=1minuscos2α AND cos2α=1minussin2α] = sin2A(1minussin2B)minussin2B(1minussin2A) = sin2Aminussin2Asin2Bminussin2B+sin2Asin2B = sin2Aminussin2B= 1-cos2A-(1-cos2B) = cos2 B- cos2 A = RHSii)LHS= cos (A+B) cos(A-B) [ cos(A+B) = cos AcosB- sinAsinBCos(A-B) = cosAcosB+ sinAsinB]= cos2 A Cos2 B- sin2 A Sin2 B= cos2 A( 1-sin2 B) - (1- cos2 A) sin2 B= cos2 A- cos2 A sin2 B- sin2 B+ cos2 A sin2 B=cos2 A- sin2 B=1- sin2 A-(1-cos2 B) = cos2 B- sin2 A= RHSTangent formulae for compound anglesi)tan (A + B) = tan A + tan B1-tan A tan Bii)tan (A ndash B) = tan A-tan B1+tan A tan Biii) cot (A + B) = cot Acot B-1cot A+cot B(viii) cot (A ndash B) = cot Acot B+1cot B-cot A

A tan B) Or 1= (tan A+ tanB) (1-tan A tanB) Or tanA + tanB + tanA tanB + 1 = 1 + 1Or tanA (1 + tanB) + (1 + tanB) = 2Or (1 + tanA) (1 + tanB) = 2Example 3 Find the value of sin 15degSolution sin 15deg= sin(45deg-30deg) = sin45degcos 30deg- cos45degsin30deg =(1radic2) (radic32) -(1radic2) (12) = (radic3-1) 2radic2Example 4 If sin A = 1 radic10 and sin B = 1 radic5 where A and B are positive acute angles then what is A + B SolutionWe know that sin (A + B) = sin A cos B + cos A sin B= [1 radic10] [radic(1 minus 1 5)] + [1 radic5] radic(1 minus 1 10)= [1 radic10] [radic4 5] + [1 radic5] [radic9 10]= [1 radic50] times (2 + 3)= 5 radic50 = 1 radic2

sin (A + B) = sin π 4rArrHence A + B = π 4Example 5 If A + B = 225o then find [cot A] [1 + cotA] times [cot B] [1 + cot B]Solution[cot A] [1 + cotA] times [cot B] [1 + cot B] = 1 [(1 + tan A) times (1 + tan B)]=1 [tan A + tan B + 1 + tan A tan B] [ tan (A + B) = tan225o]∵

tan A + tan B = 1minus tan A tan BrArr= 1 [1 minus tan A tan B + 1 + tan A tan B]= 1 2

COMMERCE

CLASSIFICTION OF HUMAN ACTIVITIES-ECONOMIC AND NON-ECONOMIC

Firstly we shall recall the previous class for 5 mins especially for the absentees and for also the rest of the students who were there

Today at first we briefly discuss the earlier portions of the chapter

1Business-It includes all those economic activities which are concerned with production and exchange of goods and services with the object of earning profit Example A factory shop beauty parlour also business enterprises

2Profession ndashThe term profession means an occupation which involves application of specialized knowledge and skills to earn a living For Example Chartered Accountancy medicine law tax consultancy are example of professions

Questions1What are the main features of ProfessionAnswer The main features of a profession are as follows a Specialised body of knowledge-Every profession has a specialised and systematised body of knowledge b Restricted entry- Entry to a profession is allowed only to those who have completed the prescribed education and have the specialised examination c Formal education and training ndashA formal education and training is given to the person who wants to acquire the professional

3Employment-Employment mean an economic activity where people work for others in exchange for some remuneration (salary)The persons who work for others are called lsquoemployeesrsquo The persons or organizations which engage others to work for them are called lsquoemployersrsquoEg A doctor working in a hospital is employment as he is working for a salaryA lawyer may serve as a law officer in a bank

With this we shall proceed with the features of both Profession amp Employment

The main features of a profession are as follow

a Specialised body of knowledge b Restricted entry c Formal education and training d Professional association e Service motive f Code of contact

The main features of an employment are as follows

a In employment a person works for others called employer

b An employee provides personal service

c There is a service agreement or contract between the employee and the employer

d The employee has to obey the order of the employer

e No capital investment is made by the employer

Various examples of Employment are as follows

aA teacher teaching in a school or collegeb An engineer employed in Municipal Corporation of DelhicAn accountant working in the accounts department of a companydA doctor working in a hospital

Note In all the above examples of employment the individual who is involved in each example is working as an employee for a salary under an employer

qualification(MBBSCALLB)d Service motive ndashProfessionals are expected to emphasis service more on their clients rather than economic gain f Code of Conduct-The activities of professionals are regulated by a code of conduct

2 What are the main features of EmploymentAnswer The main features of an employment are as followsa In employment a person works for others called employerb An employee provides personal servicec There is a service agreement or contract between the employee and the employerd The employee has to obey the order of the employere No capital investment is made by the employer

3 Give various Professions and their respective Association are given below

Professions

Professional

Professional association

Medical profession

Doctor Medical Council of India

Law profession

Lawyers Bar Council of India

Accounting Profession

Chartered

The Institute of Chartered Accounts of India( ICAI)

Engineerin Engineers The

g Profession

institute of Engineers (India)

Accounts Basic accounting terms

Today we will give you some questions from the previous study material

Questions6) Define accounting7) What do you mean by debit

and credit8) Explain the types of account9) Define the following terms

a) Assetsb) Capitalc) Purchased) Debtorse) Transactions

10) Name the types of accounts given below

a) Krishnas accountb) Machinery accountc) Royalty accountd) Salary accounte) Furniture accountf) Audit fee account

Economics Basic Economic ConceptsSub topic

UTILITY

Before starting todayrsquos class we shall recall the last class which was about UTILITY AND THE FEATURES OF UTILITY

Now we shall proceed with the further topics of the chapter

Todayrsquos topic from the chapter lsquo Basic Economic Conceptsrsquo will be TOTAL UTILITY amp MARGINAL UTILITYNow let us quickly revise the concept of utility with an example ie goods and services are designed because they have an ability to satisfy human wantsThis feature of being able to satisfy human wants is termed as utility For example we derive utility from WiFi services as it gives us satisfaction by connecting us to our friends and family through social media here consumers derive utility from WiFi services

From the above concept we shall start with todayrsquos topicEconomists have defined TOTAL UTILITY (TU) as the total satisfaction obtained by consuming a given total amount of a good and serviceFor example the total satisfaction obtained from eating 10 mangoes is the total utility of 10 mangoes

MARGINAL UTILITY (MU) is the additional satisfaction derived from each additional unit

Questions1 What is Total Utility (TU)

Answer Total Utility (TU) is the

aggregate of the utility that a consumer derives from the consumption of a certain amount of a commodityTU=MU1+MU2++MUn

2 What is Marginal UtilityAnswer

Marginal Utility (MU) is the additional made to the total utility as consumption is increased by one more unit of the commodityMU= TUn ndashTUn-1

NoteOften economists tend to

subdivide utility into an imaginary unit called UTIL

consumed In this casethe utility obtained from each mango as it is consumed as the MU of that mango It is also defined as the addition made to the total utility when an additional unit is consumed Often economists tend to subdivide utility into an imaginary unit called UTIL

Note As a consumer increases the consumption of a good over period of time the total utility or total satisfaction derived from it increases to appoint and thereafter it decreasesHowever as the consumer keeps on consuming the good the marginal utility or the additional utility derived from it decreases

SubjectBusiness studies

Topic

BUSINESSENVIRONMENT

Summary

Now quickly let us revise the earlier points that we have already done in the last class and let us proceed with the other topics that are there in the chapter

Firstly we will recall the internal and external factors of micro environment and then we shall proceed in details

Meaning and list of internal and external factors

aInternal factorsInternal factors refer to all the factors existing within a business firm The internal factors are considered controllable because the enterprise has control over these factorsFor an example a company can alter its organization structure policies programmes employees physical facilities and marketing mix to suit the changes in the environmentList of internal factors areCorporate culture mission and objectives top management organizations structure company image and brand equity company resources

b External factorsExternal factors refer to those individual and groups and agencies with which a particular business organization comes into direct and frequent contact in the course of its functioningThese individuals and groups are known as STAKEHOLDERS because they have a stake (financial interest ) in the working and performance of the particular business List of external forces (stakeholders)Customers competitors investors suppliersmiddlemen (marketing intermediaries)

Execution 1 What do you mean by internal

factors in micro environmentAnswerInternal factors refer to all the factors existing within a business firm The internal factors are considered controllable because the enterprise has control over these factorsFor an example a company can alter its organization structure policies programmes employees physical facilities and marketing mix to suit the changes in the environment

2 What do you mean by external factors in micro environment

AnswerExternal factors refer to those individual and groups and agencies with which a particular business organization comes into direct and frequent contact in the course of its functioningThese individuals and groups are known as STAKEHOLDERS because they have a stake (financial interest) in the working and performance of the particular business

3Who are stakeholdersSTAKEHOLDERS are individuals and groups who have a stake (financial interest ) in the working and performance of the particular business 4Discuss the internal factors in briefa Corporate CultureThe values beliefs and attitudes of the founders and top management of the company exercise

financers publics

customers

suppliersfinancers

competitors

middlemen

publics

Fig STAKEHOLDERS OF A COMPANY

Apart from micro environment the other main dimension of business environment isMacro environment Macro environment refers to the general environment or remote environment within which a business firm and forces in its micro environment operateA company does not directly or regularly interact with the micro environmentTherefore macro environment is also known as indirect action EnvironmentThe macro environment forces are less controllable than the micro forces

Macro environment consists of the following components

POLITICAL AND LEGAL ENVIRONMENT

ECONOMIC SOCIAL AND ENVIRONMENT

CULTURAL

ENVIRONMENT

TECHNOLOGICAL ENVIRONMENT

a strong influence on what the cmpaany stands for how it does things and what it considers importantbMission and objectivesThe business philosophy and purpose of a comoany guide it prioritiesbusiness strategiesproduct market scope and development scope

cTop management structurethe composition of board of directors the degree of professionalization of management and the organizational structure of a company have important bearing on its business decisions

dPower structureThe internal power relationship between the board of directors and the chief executive is an important factor

eCompany image and brand equityThe image and brand equity of the company play a significant role in raising finance forming alliance choosing dealers and suppliers launching new products entering foreign markets

5 What is Macro environmentAnswerMacro environment refers to the general environment or remote environment within which a business firm and forces in its micro environment operateA company does not directly or regularly interact with the micro environmentTherefore macro environment is also known as indirect action EnvironmentThe macro environment forces are less controllable than the micro forces 6 What are the components of macro environmenta Political and legal environmentb Economic environmentc Social and cultural environmentd Technological environment

BUSINESS FIRM

Fig COMPONENTS OF MACRO ENVIRONMENTPolitical science

Introduction to political science

Comparative politics and itrsquos scope Comparative politics is the second major dimension of political scienceIt is also a very vast area of study and a very large number of political scientists even treat it as an autonomous area of study within the board ambit of political scienceScope of comparative politics-

1 All political structures -Comparative politics includes the study of all structures formalnon formal governmental and extra governmental which are directly or indirectly involved in politics in all the countries of the world

2 Functional studies- Comparative politics seeks to study politics less from the point of view of the legal institutions in terms of their powers and move from the point of view of their functions which constitute the political process and their actual Operation in the environment

3 Study of political behaviour- Another important part of its scope is the study of the actual behaviour of the people in the process of politics

4 Study of similarities and differences- comparative politics also undertakesan analysis of the similarities and differences among political process and functions

5 Study of all political systems -comparative politics seeks to analyse the actual behaviour and performance of all political systems western as well as non western

6 Study of the environment and infrastructure of politics-The study of politics demands a study of the psychological sociological economic and anthropological environment in fact the social environment as a whole in which each political system operates

7 Study of political culture- political culture is composed of attitudesbeliefs emotions and values of a society that relate to the political system or politics

8 Study of political participation- Political participation is a universal processThe only difference is that while in some states it is limited in others it is wider

9 Study of political process- political

Answer the following questions-

What is comparative politics

What are the scope of comparative politics

Homework- learn

processes like decision makingpolicy making judicial process leadership recruitment process and others are always at work in all political systems

The scope of comparative politics is very comprehensive It includes everything that falls within the area of political activity and political process

History CAMBRIDGE VIEW ABOUT

THE PARTITION

AND REFUTATION

OF CAMBRIDGE

VIEW

Cambridge view about the Partition The Cambridge school of historians have interpreted that opposition to partition scheme was made entirely by the elitist groups They hold the view that Lord Curzon planned to partition the Bengal for administrative purposeREFUTATION OFCAMBRIDGE VIEW The Rationalist historians have rejected the interpretations of the Cambridge School of historians on various grounds

1 QUESTION State different views of historians regarding Partition of Bengal

ANSWER Cambridge historians believed that Lord Curzon partitioned Bengal for administrative reasons only and not for the political motive The Middle class elitist group protested because of their petty interest The Hindu zamindars protested as they have to spend more money for managing their estatesThe lawyers of Calcutta High court feared to lose their clientBut according to the nationalist Historians was-

2- The ultimate object of Lord Curzon was to crush the unity of Bengal politicians

3- If Bengal becomes a separate province Bengali speaking 16 million people of western part would become minority under Hindi speaking people of Bihar and Oriya speaking people of Orissa

4- The bureaucrats expected that the protest movement would die down quickly

5- Lord Curzon used the Muslim community in his political game

6- Idealism had great contribution in the protest against partition

7- The people of the every section of society were affected by the partition of Bengal

Computer Science

Numbers Convertion of dcimal number to octal numberThe decimal numeral system is the standard system for denoting integer and non-integer numbers It is the extension to non-integer numbers of the Hindu-Arabic numeral system For writing numbers the decimal system uses ten decimal digits a decimal mark and for negative numbers a minus sign - The decimal digits are 0 1 2 3 4 5 6 7 8 9 the decimal separator is the dot in many countries

The octal numeral system or oct for short is the base-8 number system and uses the digits 0 to 7 Octal is sometimes used in computing instead of hexadecimal perhaps most often in modern times in conjunction with file

permissions under Unix systems It has the advantage of not requiring any extra symbols as digits It is also used for digital displays

Follow these steps to convert a decimal number into octal form

1 Divide the decimal number by 82 Get the integer quotient for the next iteration (if the number will not divide equally by 8 then round down the

result to the nearest whole number)3 Keep a note of the remainder it should be between 0 and 74 Repeat the steps until the quotient is equal to 05 Write out all the remainders from bottom to top This is the solution

For example if the given decimal number is 8453

Division Quotient Remainder

8453 8 1056 5

1056 8 132 0

132 8 16 4

16 8 2 0

2 8 0 2

Then the octal solution is 20405

Subject Eng Literature (The Tempest ndash William Shakespeare) Topic Act I Scene 1 Lines 33 to 67 (End of scene) Date 16th April 2020 (4th Period)

[Students should read the original play and also the paraphrase given in the school prescribed textbook]Summary Questions amp Answers

[SUMMARY OF THE ENTIRE SCENE]

o The play starts with the scene of a severe storm at sea Alonso (King of Naples) Sebastian (Alonsorsquos brother) Ferdinand (Alonsorsquos son) Gonzalo Antonio (the usurping Duke of Milan) are in a ship in the midst of the storm

o The mariners are trying their best to control the vessel from running aground and are totally following the orders of their Master the Boatswain They have scant success

o The mariners become extremely unhappy and annoyed when most of the passengers arrive on the deck thereby hampering their effort to save the ship There is serious confrontation between them and the passengers who are part of the Kingrsquos entourage

o The mariners could not save the ship

SUMMING-UP

(i) Vivid description of the scene which gives a realistic description of terror and confusion of a tropical storm

(ii) Shows Shakespearersquos accuracy of knowledge in describing the naval operations and also matters of seamanship

(1) GONZALO Ill warrant him for drowning (L 45-57)

though the ship were no stronger than a nutshell and as leaky as an unstanched

wenchBOATSWAIN Lay her a-hold a-hold Set her two courses Off to

sea again lay her offMARINERS All lost To prayers to prayers All lostBOATSWAIN What must our mouths be coldGONZALO The king and prince at prayers Lets assist them

For our case is theirsSEBASTIAN Im out of patienceANTONIO We are merely cheated of our lives by drunkards

This wide-chopped rascal - would thou mightst lie drowning the washing of ten tides

(a) What does Antonio say at the insolent manners of the boatswain just before the given passage

Being irritated at the insolent manners of the boatswain just before the given extract Antonio the Duke of Milan calls him a worthless dog son of a woman without any morals an arrogant and disrespectful noisemaker He says that the boatswain deserved to be hanged(b) What statement does Gonzalo repeat about the boatswain

Gonzalo shows his faith that the boatswain is not destined to die by drowning He is destined to be hanged and nothing can alter this decree of destiny He says that even if the ship was as frail as a nutshell the boatswain could not be drowned for his destiny was to be hanged(c) What do the passengers do when they have lost all hope of their survival

When the passengers have lost all hope of survival they take

(iii) The opening scene justifies the title ndash The Tempest

UNANSWERED QUESTIONS

(i) The King always travels with his entire fleet including his soldiers Where were the other ships

(ii) Why was the ship in that area Where was it coming from or going where

(iii) The ship broke apart What happened to those who were in the ship

(We shall get the answer to the above questions as the play progresses)

leave of life with fervent prayers The mariners take their last hearty drink and are ready for death(d) What blame does Antonio put upon the mariners and the boatswain Antonio rebukes the mariners that these drunkards have brought them to the present crisis by neglecting their duties He blames them saying that they are going to lose their lives entirely for the negligence of the boatswain and his fellows(e) What does Antonio say while cursing the boatswain

Antonio gives vent to his wrath upon the boatswain in particular He calls the boatswain a wide-mouthed rascal who deserves to be hanged on the sea-shore at low water mark so that ten tides might wash over his body and take out of him all the liquor that he has been drinking

Class XIISubject Topic Summary ExecutionHistory Topic

1 1935 ACT AND WORKING OF PROVINCIAL AUTONOMYCONGREE AND OTHER MINISTERSSUB TOPIC GOVERNMENT OF INDIA ACT1935

Government of India Act 1935 This act established a lsquoFederation of Indiarsquo made of British Indian provinces and Indian states and provided for autonomy with a government responsible to the elected legislature in every provinceThis act introduced abolition of Diarchy at provinces The entire provincial administration was introduced to the responsible ministers who were controlled and removed by the provincial legislature The provincial autonomy means two things First The provincial governments were wholly responsible to the provincial legislature Secondly Provinces were free from outside control and interference in the large number of matters The act divided the powers between the centre and provinces in terms of three lists- Federal list( for centre) Provincial list (for province) and concurrent list (for both) Residuary powers were given to the viceroy In the election under the government of India Act the Congress swept the poll the mandate of the people came in favour of the congress so far as general Hindu seats were concerned The Congress did not get a single Muslim seates in Bombay CP UP Sind and BengalIn five provinces Congress had yhe clear majority In BengalNWFPAssam and Bombay Congress emerged as a single largest partyOn the other side the performance of the Muslim League was badThus the Congress formed ministers in 7 provinces out of 11 provinces Coalition ministry was also formed in two other provincesOnly BENGAL AND Punjab had non- congress ministries

1 QUESTION What was the main change introduced by the Government of India ActANSWER a) The Act gave more

autonomy to the provinces b) Diarchy was abolished at the

provincial levelsc) The Governor was the head of

the executived) There was a council of

ministers to advise him The ministers were responsible to the provincial legislatures who controlled them The legislature could also remove the ministers

e) The Governors still retained special reserve powers

2 QUESTION Why did the federal scheme introduced by the Government of India Act 1935 never come into operation

ANSWER The Federal structure of the Government of India was to be composed with the Governor General and Council of ministers The Federal legislature was to be Bicameral legislature- The council of states and the House of Assembly The ministers were to be chosen by the Governor general and they were to hold the office during his pleasure

The provinces of British India would have to join the federation but this was not compulsory for the princely states

This federation never materialised because of the lack of support from the required number of

princely statesThis act was refused and

rejected by the princes the Congress and the Muslim League

Thus both Congress and the League participated in the election of 1937 Thus the federal part was never introduced but the provincial part was put into operations

Bengali 2nd

Language

াচেরর পরাথCনা(কহিতা )

াচেরর পরাথCনা কহিতাটি কহি (ঙখ দেঘাচে4র দো আচো য কহিতায় াচেরর পতর হমায়ন কঠিন দেরাচেগ আxানত ার ঈশবর া আললার কাচেছ পরাথCনা কচেরচেছন তার পচেতরর ীন হিফহিরচেয় হিচেত এই কহিতায় ার পচেতরর ীন হিভbা দেচেয়চেছন ারার এমনহিক হিনচের ীন হিসCচেনর হিহিনমচেয় হিতহিন তার দেছচের ীন হিফচের দেপচেত দেচেয়চেছন তার দেছচের এই দেরাচেগর ন য হিতহিন হিনচেচেকই ায়ী কচেরচেছন তার হিনচের করা পাপচেকই হিতহিন ায়ী কচেরচেছন এছাা রানৈনহিতক ও আথCসামাহিক অসথার কথা তচে ধরা চেয়চেছ এই কহিতায় ার তার হিনচের পাপ কমCচেকই ায়ী কচেরচেছ ার অন যায় ভাচে দেপহি((হিকতর মাধ যচেম অপররা য কচেরচেছ আর এই অন যায় কাচের ন যই তার পহিরাচের হিপযCয় এচেসচেছ দে এক পরকার মানহিক নধন ইহিতাচেসর ার হিপতা চেয় সবাভাহিকভাচে ভাচোাসা দে মমতা দেথচেক মকত চেত পাচেরনহিন তাই হিপতা চেয় আললা া ভগাচেনর কাচেছ পতর হমায়চেনর পরানহিভbা দেচেয়চেছন ার আললা া ভগাচেনর কাচেছ াহিনচেয়চেছন তার হিনচের ীন হিসCন হিচেত হিতহিন রাী তার হিহিনমচেয় পচেতরর ীন হিফচের দেপচেত দেচেয়চেছন াচেরর হিপতসভ হিচেকর কথা এই কহিতায় ফটিচেয় দেতাা চেয়চেছ হিপতা পচেতরর হিরাহিরত মান নধচেনর কথা তচে ধরা চেয়চেছ

হিচে(4 হিকছ াইচেনর তাৎপযC১) ldquoদেকাথায় দেগ ওর সবচছয দেৌন দেকাথায় কচেরায় দেগাপন bয়ldquoউততর) াচেরর পতর হমায়ন কঠিন দেরাচেগ অসসথ তাই তার দেযৌন াহিরচেয় যাচেচছ এই দেরাচেগ তাচেক দেগাপচেন কচেরকচের াচেচছ তার সক (হিকত ধীচের ধীচের bয় চেচছ তাই হিপতা চেয় ার আললার কাচেছ হমায়চেনর পরান হিভbা দেচেয়চেছন২) ldquoাগাও (চেরর পরাচেনত পরানতচের ধসর (ন দেযর আান গানldquoউততর) াচেরর পতর হমায়ন কঠিন দেরাচেগ আxানত তাই ার আ দে(াচেক মমCাত (চেরর পচেথ পরানতচের আান গান ধবহিনত দোক দেসই আান গান আললার কাচেছ দেযন চে যায় আললা দেযন এই আহিতC শচেন পচেতরর ীন হিফহিরচেয় দেয় ৩)ldquoনাহিক এই (রীচেরর পাচেপর ীানচেত দেকানই তরারণ দেনই ভহি4চেতরldquoউততর) হমায়চেনর অসসথতার ন য ার হিনচেচেকই ায়ী কচেরচেছন কারন ার অচেনক রা য অন যায় ভাচে কচেরচেছ তাই তার এই পাপ কাচের ন য তার ঘচের আ হিপ এচেসচেছ এই অন যায় কাচের ন য তার মহিকত দেনই তাই ার আললার কাচেছ এই পাপ কাচেযCর ন য bমা পরাথM

Hindi 2ndlang

-ासी(जयशकर परसा-)

-ासी जयशकर परसा- की एक ऐसी कहानी ह जिजसम भारतीय ससकनित और राषटरीयता का सवरगजीतहोता ह इस कहानी म इरावती एक निहद कनया ह जिजस मलअचछो न मलतान की लट म पकडा और -ासी बना दि-या उस 500 दि-न -कर काशी क एक महाजन न खरी-ा दसरी -ासी निफरोजा ह वह गलाम ह निफरोजा को छडान क कतिलए अहम- को 1000 सोन क कतिसकक भजन थ जो अभी तक नही आए थ राजा साहब कठोर होत हए भी निफरोजा को निबना धनराकतिश क कतिलए उस म कर -त ह वनिफरोजा को अहम- को समझान की बात कहत हकहानी क अत म हम -खत ह निक इरा वती और जाटो क सर-ार बलराज का मिमलन होता हअहम- को यa म मार दि-या जाता ह वहा निफरोजा की परसननता की समामिध बनती ह वहा एक फल चढती ह और डीजल आती ह निफरोजा उस समामिध की आजीवन -ासी बनी रहती हलखक अपन उददशय अथात -ास परथा पर परकाश डालन और इस परथा क कारण होन वाल -ातो क दखो को दि-खान म पणता सफल हए ह

helliphellipContinue to next

Biology Reproductio Today we will discuss about vegetative Q1 Name some vegetative propagules

n in Organisms

propagation of plants The process of multiplication in which fragments of plant body function as propagule and develop into new individual is called vegetative propagation The units of such propagation are runner rhizome tuber bulb etc

and the speciesinvolvedVegetative propagules

Parts involved

Bulb StemBulbil BulbilRhizome Stem Runner Stem Tuber Stem Offset Stem Leaf buds Leaves Suckers Stem

Corns Stem stolon

Q2 State advantages of vegetative propagation

i) Rapid methodii) Sure and easy methodiii) Useful in plants that cannot

produce viable seeds or long seed dormancy

iv) Maintains purity of raceQ 3 Banana fruit is said to be parthenocarpic where as turkey is said to be parthenogenetic WhyBanana develops without fertilization from an unfertilized ovary thus is parthenocarpicIn turkey the ovum or female gamete developinto a new chick without fertilization thus isparthgenetic

Q4 Why is water hyacinth is called as a ldquoTerror of Bengalrdquo Water hyacinth can

propagatevegetatively all over the water body in a short per short period of time This resulted increased biochemicaloxygen oxygen demand of water body causing mortalityof fishes It is very difficult to get rid off them Thus known as terror of Bengal

Chemistry

Solid state GENERAL CHARACTERISTICS OF SOLID STATEIn nature the particular state of matter is governed by two opposing forces at given set of temperature and pressure These forces are intermolecular force of attraction and thermal energy If intermolecular force of attraction is high as compared to thermal energy particles remains in closest position

Intext QuestionsQ1 Classify the following solids as crystalline and amorphous Sodium chloride quartz glass quartz rubber polyvinyl chloride Teflon

A1 Crystalline

and hence very less movement in particles is observed In this case solid state is the preferred state of matter

Let us revise the general characteristics of solid

i) Fixed mass volume and shape

ii) Strong intermolecular force of attraction

iii) Least intermolecular space

iv) Fixed position of constituent particles

v) Incompressible and rigid

Q2 what type of interactions hold the molecules together in a polar molecular solid[CBSE 2010]A2 The molecules in a solid are held together by van der Waals forces The term van der Waals forces include hydrogen bonding dipole-dipole attraction and London dispersion forces All molecules experience London dispersion forces In addition polar molecules can also experience dipole-dipole interactions So the interactions that holds the molecule together in polar molecular solid are London dispersion force and dipole-dipole interactionsQ3 Write a feature that will distinguish a metallic solid from an ionic solid [CBSE 2010]A3 Metals are malleable and ductile whereas ionic solid are hard and brittle Metallic solid has typical metallic lustre But ionic solid looks dullQ4 Write a point of distinction between a metallic solid and an ionic solid other than metallic lustre [CBSE 2012]A4 Metals are malleable and ductile whereas ionic solid are hard and brittleQ5 Write a distinguish feature of metallic solid [CBSE 2010]A5 The force of attraction in

solid Sodium chloride Quartz Amorphous solid Quartz glass rubber polyvinyl chloride Teflon Q2 why glass is considered as super cooled liquidA2 Glass shows the tendency to flow at slower rate like liquid Hence they considered as super cooled liquidQ3 why the window glass of old buildings show milky appearance with timeA3 Glass is an amorphous solid Amorphous solid has the tendency to develop some crystalline character on heating Due to heating in day over the number of years glass acquires some crystalline character and show milky appearanceQ4 why the glass panes fixed to window or doors of old building become slightly thicker at bottomA4 Glass is super cooled liquid It has the tendency to flow down very slowly Due to this glass pane becomes thicker at the bottom over the timeQ5 Sodium chloride is a crystalline solid It shows the same value of refractive index along all the direction TrueFalse Give reasonA5 FalseCrystalline solid shows anisotropy in properties That is it shows different values for the given physical property in different direction All the crystalline solids show anisotropy in refractive index Therefore sodium chloride will show different values of refractive index on different directions

Q6 Crystalline solid are anisotropic in nature What does this statement means

between the constituent particles is special kind of electrostatic attraction That is the attraction of positively charged kernel with sea of delocalized electronsQ6 which group of solid is electrical conductor as well as malleable and ductile [CBSE 2013]A6 Metallic solidQ7 why graphite is good conductor of electricity although it is a network (covalent solid)A7 The exceptional property of graphite is due to its typical structure In graphite each carbon is covalently bonded with 3 atoms in same layer The fourth valence electron of each atom is free to move in between different layersThis free electron makes the graphite a good conductor of electricity

[CBSE 2011]A6 Anisotropy is defined asrdquo Difference in properties when measured along different axis or from different directionsrdquo Crystalline solid show different values of some of the physical properties like electrical resistance refractive index etcwhen measured along the different directions The anisotropy in crystalline solid arises due to the different arrangement of particles in different directions

Math Function Composition of functions Think of an industrial plant that produce bottles of cold drinks first there is the operation (or function) f that puts the cold drink inside the bottle followed by the opeartion g that close the bottle with the capThis leads to the following definitionDefinition Let f A rarr B and g B rarr C be two functions Then the composition of f and g denoted by gof is defined as the function gof A rarr C given by gof(x) = g(f (x)) forall x isinA

Definition A function f X rarr Y is defined to be invertible if there exists a function g Y rarr X such that gof = IX and fog = IY The function g is called the inverse of f and is denoted by f -1

Thus if f is invertible then f must be one-one and onto and conversely if f is one-one and onto then f must be invertible This fact significantly helps for proving a function f to be invertible by showing that f is one-one and onto specially when the actual inverse of f is not to be determined

Example 1 Let f 2 3 4 5 rarr 3 4 5 9 and g 3 4 5 9 rarr 7 11 15 be functions defined as f(2) = 3 f(3) = 4 f(4) = f(5) = 5 and g (3) = g (4) = 7 and g (5) = g (9) = 11 Find gofSolution We have gof(2) = g (f(2)) = g (3) = 7 gof(3) = g (f(3)) = g (4) = 7gof(4) = g (f(4)) = g (5) = 11 and gof(5) = g (5) = 11Example 2 Find gof and fog if f R rarr R and g R rarr R are given by f(x) = cos x and g (x) = 3x2 Show that gof ne fogSolution We have gof(x) = g(f(x))=g(cosx) = 3 (cos x)2

= 3 cos2 x Similarly fog(x)=f(g (x))= f(3x2)= cos (3x2) Note that 3cos2 x ne cos 3x2 for x = 0 Hence gof ne fogExample 3 Show that if f A rarr B and g B rarr C are onto then gof A rarr C is also ontoSolution Given an arbitrary element z isin C there exists a pre-image y of z under g such that g (y) = z since g is onto Further for y isin B there exists an element x in A with f(x) = y since f is onto Therefore gof(x) = g (f(x)) = g (y) = z showing that gof is onto Example 4 Let Y = n2 n isin N sub N Consider f N rarr Y as f(n) = n2 Show that

f is invertible Find the inverse of fSolution An arbitrary element y in Y is of the form n2 for some n isin N This implies that n =radicy This gives a function g Y rarr N defined by g (y) =radicy Nowgof (n) = g (n2)=radicn2 = n and fog (y) =f(radicy) = (radicy) 2 y which shows that gof=IN and fog= IY Hence f is invertible with f -1 = g

Political Science

Constitution of India-The Preamble

Summary

Objective of the state-To secure equality of status and of opportunity To promote fraternity among all the citizens To assure the dignity of the individuals and Unity and integrity of the nation

Justice-Justice stands for rule of law absence of arbitrariness and a system of equal rights freedom and opportunities for all in a society India seeks social economic and political justice to ensure equality to its citizens

Liberty-Liberty implies the absence of restraints or domination on the activities of an individual such as freedom from slavery serfdom imprisonment despotism etc The Preamble provides for the liberty of thought expression belief faith and worship

Equality-Equality means the absence of privileges or discrimination against any section of the society The Preamble provides for equality of status and opportunity to all the people of the country

Fraternity-The Preamble declares that fraternity has to assure two thingsmdashthe dignity of the individual and the unity and

Execution

Answer the following questions-

Short notes-1 Equality2 Fraternity3 Justice4 Liberty

Homework-Learn

integrity of the nation The word integrity has been added to the Preamble by the 42nd Constitutional Amendment (1976)

Business studies

Human resource management (chapter 1)

On the day of 1504 2020 I have discussed with you the managerial functions and procurement functions of HRM

Today weare going to discuss about the development function integration functions and maintenance function

Development functions-HRM improves the knowledge skills attitude and values of employees so that they the present and future jobs more effectively it includes

1) Development functions of HRM

a) Performance appraisal = It implies systematic evaluation of employees with respect to their performance on the job and their potential for development

b) Training =It is the process by which employees learn knowledge skills and attitudes to achieve organisational and personal goals

c) Executive development = It is the process of developing managerial talent through appropriate program

2) Integration functionsa) HRM reconcile the goals of

organisation with those of its members through integrating function

b) HRM tries to motivate employees to various financial and non financial incentives provided in job specification etc

3) Maintenance functiona) HRM promote and protect the

physical and mental health of employees by providing several types of benefits like housing medical aid etc

b) It Promote Social security measures to employees by providing provident fund pension gratuity maternity benefits

SubjectCOMMERCE

Topic

BUSINESSENVIRONMENT

Summary

Now quickly let us revise the earlier points that we have already done in the last class and let us proceed with the other topics that are there in the chapter

Firstly we will recall the internal and external factors of micro environment and then we

Execution 3 What do you mean by internal factors

in micro environmentAnswerInternal factors refer to all the factors existing within a business firm The internal factors are considered controllable because the enterprise has control over these factors

Development FunctionsPerformance AppraisalTrainingExecution Development

shall proceed in details

Meaning and list of internal and external factors

aInternal factorsInternal factors refer to all the factors existing within a business firm The internal factors are considered controllable because the enterprise has control over these factorsFor an example a company can alter its organization structure policies programmes employees physical facilities and marketing mix to suit the changes in the environmentList of internal factors areCorporate culture mission and objectives top management organizations structure company image and brand equity company resources

b External factorsExternal factors refer to those individual and groups and agencies with which a particular business organization comes into direct and frequent contact in the course of its functioningThese individuals and groups are known as STAKEHOLDERS because they have a stake (financial interest ) in the working and performance of the particular business List of external forces (stakeholders)Customers competitors investors suppliersmiddlemen (marketing intermediaries)financers publics

customers

suppliersfinancers

For an example a company can alter its organization structure policies programmes employees physical facilities and marketing mix to suit the changes in the environment

4 What do you mean by external factors in micro environment

AnswerExternal factors refer to those individual and groups and agencies with which a particular business organization comes into direct and frequent contact in the course of its functioningThese individuals and groups are known as STAKEHOLDERS because they have a stake (financial interest) in the working and performance of the particular business

3Who are stakeholdersSTAKEHOLDERS are individuals and groups who have a stake (financial interest ) in the working and performance of the particular business 4Discuss the internal factors in briefa Corporate CultureThe values beliefs and attitudes of the founders and top management of the company exercise a strong influence on what the cmpaany stands for how it does things and what it considers importantbMission and objectivesThe business philosophy and purpose of a comoany guide it prioritiesbusiness strategiesproduct market scope and development scope

cTop management structurethe composition of board of directors the degree of professionalization of management and the organizational structure of a company have important bearing on its business decisions

dPower structureThe internal power relationship between the board of directors and the chief executive is an important factor

e Company image and brand equityThe image and brand equity of the company play a significant role in raising finance forming alliance choosing dealers and suppliers launching new products entering foreign markets

5 What is Macro environmentAnswerMacro environment refers to the general

competitors

middlemen

publics

Fig STAKEHOLDERS OF A COMPANY

Apart from micro environment the other main dimension of business environment isMacro environment Macro environment refers to the general environment or remote environment within which a business firm and forces in its micro environment operateA company does not directly or regularly interact with the micro environmentTherefore macro environment is also known as indirect action EnvironmentThe macro environment forces are less controllable than the micro forces

Macro environment consists of the following components

POLITICAL AND LEGAL ENVIRONMENT

ECONOMIC SOCIAL AND ENVIRONMENT

CULTURAL

ENVIRONMENT

TECHNOLOGICAL ENVIRONMENT

Fig COMPONENTS OF MACRO ENVIRONMENT

environment or remote environment within which a business firm and forces in its micro environment operateA company does not directly or regularly interact with the micro environmentTherefore macro environment is also known as indirect action EnvironmentThe macro environment forces are less controllable than the micro forces 6 What are the components of macro environmenta Political and legal environmentb Economic environmentc Social and cultural environmentd Technological environment

Computer Science

Logic gates

Digital systems are said to be constructed by using logic gates These gates are the AND OR NOT NAND NOR EXOR and EXNOR

BUSINESS FIRM

gates The basic operations are described below with the aid of truth tables

AND gate

The AND gate is an electronic circuit that gives a high output (1) only if all its inputs are high A dot () is used to show the AND operation ie AB Bear in mind that this dot is sometimes omitted ie ABOR gate

The OR gate is an electronic circuit that gives a high output (1) if one or more of its inputs are high A plus (+) is used to show the OR operationNOT gate

The NOT gate is an electronic circuit that produces an inverted version of the input at its output It is also known as an inverter If the input variable is A the inverted output is known as NOT A This is also shown as A or A with a bar over the top as shown at the outputs The diagrams below show two ways that the NAND logic gate can be configured to produce a NOT gate It can also be done using NOR logic gates in the same way

NAND gate

This is a NOT-AND gate which is equal to an AND gate followed by a NOT gate The outputs of all NAND gates are high if any of the inputs are low The symbol is an AND gate with a small circle on the output The small circle represents inversion

NOR gate

This is a NOT-OR gate which is equal to an OR gate followed by a NOT gate The outputs of all NOR gates are low if any of the inputs are highThe symbol is an OR gate with a small circle on the output The small circle represents inversion

EXOR gate

The Exclusive-OR gate is a circuit which will give a high output if either but not both of its two inputs are high An encircled plus sign ( ) is used to show the EOR operation

EXNOR gate

The Exclusive-NOR gate circuit does the opposite to the EOR gate It will give a low output if either but not both of its two inputs are high The symbol is an EXOR gate with a small circle on the output The small circle represents inversion The NAND and NOR gates are called universal functions since with either one the AND and OR functions and NOT can be generated

Note A function in sum of products form can be implemented using NAND gates by replacing all AND and OR gates by NAND gates A function in product of sums form can be implemented using NOR gates by replacing all AND and OR gates by NOR gates

Logic gate symbols

Table 2 is a summary truth table of the inputoutput combinations for the NOT gate together with all possible inputoutput combinations for the other gate functions Also note that a truth table with n inputs has 2n rows You can compare the outputs of different gates

Logic gates representation using the Truth table

Example

A NAND gate can be used as a NOT gate using either of the following wiring configurations

Subject Eng Literature (The Tempest ndash William Shakespeare) Topic Act III Scene 3 Lines 53 to 110 (End of the scene) Date 16th April 2020 (2nd Period)

[Students should read the original play and also the paraphrase given in the school prescribed textbook]Summary Questions amp Answers

o Seeing this strange scene all are inclined to believe the tales told by travelers that there truly are ldquounicornsrdquo and ldquothe phoenixrsquo thronerdquo

o As they are about to sit down to the feast the banquet is snatched away by a harpy (Ariel disguised) A spiritrsquos voice (Arielrsquos voice) denounces Alonso Sebastian and Antonio with particular

1 ARIEL You are three men of sin whom Destiny

(Line 53-58)That hath to instrument this

lower world And what is int the never-surfeited sea

Hath caused to belch up you and on this island

Where man doth not inhabit you rsquomongst men

Being most unfit to live I have made you mad

reference to their crime in expelling Prospero from Milan They have not received any punishment for their deed earlier but the time for their punishment has arrived Upon Alonso it pronounces ldquolingering perdition worse than deathrdquo from which there is no remedy except through sincere repentance Ariel then vanishes in thunder and the shapes enter again and carry away the table

o Prospero watching invisibly is very pleased with the performance of Ariel and his (Prosperorsquos) ldquomeaner ministersrdquo All his enemies are now in his power and are in a fit of desperation He then leaves them and goes to see how Ferdinand and Miranda are getting on

o Alonso is now much humbled and penitent with the after effect of the spiritrsquos denunciation of his crimes He believes that his son is lost forever After this all disperse being stricken mad by the speech of the spirit

o Gonzalo fearing that they may do violence to themselves or to one another follows them and bid others to follow

(a) To whom does Ariel disguised as a harpy call the three sinners What game did Fate of Destiny play with

them

The three sinners called by Ariel are Alonso Sebastian and Antonio It was Destiny which had caused the ocean to cast the three sinners on the shore Though the ocean is all the time devouring whatever appears on its surface and is never satisfied with its continual swallowing of the ships and men in the present case the ocean had cast these three sinners on the shore without killing them

(b) Who had jointly been responsible for the conspiracy against Prospero What is Prosperorsquos purpose behind all this

Three men Alonso Sebastian and Antonio had jointly

been responsible for the conspiracy against Prospero They had driven out Prospero form Milan Prosperorsquos purpose is to make these three sinners realize the wrong they had done He wants them to repent for their criminal deeds because repentance leads to self-esteem(c )What does Ariel (the harpy) tell Alonso and his companions when they take out their swords to attack him

Seeing them drawing their swords Ariel (harpy) tells them that he and his companions are the instruments of destiny and that it is not possible for human beings to do them any injury He says that the swords of human beings can not injure even a minute part of his feathers Their swords are as ineffective against him and his companions as against the wind or the water

(d) Give the explanatory meanings of the following expressions in the context of the above extract

(i)Never surfeited (ii) Belch up (iii) lsquomongst men

(i) Never surfeited never led to satisfaction

(ii) Belch up cast ashore(iii) lsquomongst men in human

society2

I and my fellows (Line 60-65)

Are ministers of Fate The elementsOf whom your swords are tempered may as wellWound the loud winds or with bemocked-at stabsKill the still-closing waters as diminishOne dowl thats in my plume

IMPORTANT PASSAGES EXPLAINED

The elements

(Line 61-66)Of whom your swords are tempered may

as wellWound the loud winds or with

bemocked-at stabs

(a) Who is lsquoIrsquo Who are his lsquofellowsrdquo

lsquoIrsquo is referred to Ariel in disguise of a harpy His lsquofellowsrsquo are other spirits serving Prospero the real Duke of Milan who has acquired supernatural powers after being banished from his Dukedom Prospero has settled in this uninhabited island

(b) What are the elements that have temperrsquod the swords Why will it not work against the speaker

The swords (of Alonso and his companions) are tempered by metal (steel) which is taken out of the earth and refined by

Kill the still-closing waters as diminishOne dowl thats in my plume My fellow

ministersAre like invulnerable

In these words Ariel reminds the King and his companions of the utter futility of drawing swords against himself and his fellows Ariel drives Alonso Antonio and Sebastian the three men of sin to desperation ndash a state in which men do violence to themselves They draw swords to strike Ariel But Ariel reminds them that he and the other spirits are the ministers of destiny and nothing can wound them The steel of which their swords are made of may cut the wind or water which being divided always closes up again Even supposing that such things may be possible it is quite impossible that their swords will cut one feather in their plume They are incapable of being wounded by any sword of man Hence it is foolish on their part to attempt to strike at Ariel and his fellow-spirits

For which foul deed

(Line 72-75)The powers delaying not forgetting

haveIncensed the seas and shores yea all the

creatures Against your peace

Ariel enters like a harpy and remaining invisible tells Alonso Sebastian and Antonio that he and other harpies are the agents of Destiny appointed to carry out her decrees He tells them that their punishment for the crime against Prospero which has been so long deferred is now to fall upon them He reminds them that they had expelled Prospero from Milan and set him and his innocent child adrift on the sea and that the sea had paid them back for their sin by the shipwreck and by the calamities they have suffered He tells them that the powers above which did not forget this mean treachery but only deferred the punishment have now engaged the seas and the shores and all living beings including him and his comrades against them The very elements and supernatural agency Ariel adds have taken up the avenging of their crime against Prospero

the action of fire It may cut the wind or water which being divided always closes up again

The sword will not work against the spirits and the harpy because they are the ministers of destiny and nothing can wound them nor it will cut a single feather in their plume

(c )What is the meaning of lsquodowlrsquo in the last line

The term lsquodowlrsquo means a filament or the smallest part of a feather In this context Ariel in disguise of harpy says that their sword cannot even damage the smallest filament of their (Arielrsquos and other spirits) feathers as they are incapable of being wounded by any sword of man

(d) What does the speaker remind the listeners about

Ariel in disguise of harpy reminds Alonso the King of Naples Sebastian Alonsorsquos brother and Antonio the present Duke of Milan and the treacherous brother of Prospero as they being three men of sin He even reminds them that their punishment for their crime against Prospero which has been so long deferred now falls upon them He reminds them that they have expelled Prospero from Milan and has set him along with his innocent infant daughter adrift on the sea So the sea has paid them back for their sin by their shipwreck and the calamities they have suffered since then The harpy rebukes Alonso of his sin that has incensed the Gods and has deprived him of his son as a punishment

(e) How do they respond

When Ariel in disguise of a harpy reminds Alonso Sebastian and Antonio of their past misdeeds and sin Alonso has a look of terror and confusion in his eyes He utters the words of sincere repentance wrung out of his conscience-stricken heart It appears to him that all the elements of nature the sea-waves the wind and the thunder proclaiming a loud voice in the name of Prospero and the crime Alonso has committed against him They are calling upon him to repent There is a deep storm raging in Alonsorsquos breast and the echoes of that storm are ringing in his ears like a clear note of wind-instrument A note of denunciation of Alonsorsquos crime leaves him much humbled and penitent and confirms his belief that his son is lost forever But Sebastian and Antonio shows some courage instead of repentance They wish to kill the spirits or devils if it appears

3

Of my instruction hast thou nothing bated (Line 85-93)

In what thou hast to say So with good life

And observation strange my meaner ministers

Their several kinds have done My high charms work

And these mine enemies are all knit upIn their distractions They now are in my

powerAnd in these fits I leave them while I visitYoung Ferdinand whom they suppose is

drownedAnd his and mine loved darling

Methought the billows spoke and (Line 96-99)

told me of itThe winds did sing it to me and the

thunderThat deep and dreadful organ-pipe

pronouncedThe name of Prosper It did bass my

trespass

These are the words of contrition coming from Alonso Ariel has driven him to a deep repentance for conspiring with Antonio against Prospero He now feels a sincere remorse It appears to him that all the elements of nature the sea-waves the wind and the thunder proclaimed with a loud voice the name of Prospero and the crime Alonso had committed against him They are calling upon him to repent There is a deep storm raging in Alonsorsquos breast and the echoes of that storm are ringing in his ears like the clear note of a wind-instrument

Comment These are the words of sincere repentance wrung out of the conscience-stricken heart of Alonso Alonso who is the lesser villain is the first to give way to remorse under the effect of Arielrsquos speech The words of Ariel seem to him to be the voice of conscience speaking to him He is driven to desperation a state in which he might do violence to his life

(a) Identify the speaker State the context

Prospero the ruler of the island is the speaker The famous banquet scene has been enacted very well Ariel and his junior spirits have played their roles excellently Prospero is glad to say words of praise for them(b) In what way the speakerrsquos instructions have been carried out

According to Prosperorsquos instructions a banquet was presented before the King of Naples and his companions when they were tired and hungry Just when they were preparing to eat the feast the banquet was suddenly removed by exercising supernatural powers All this was done by Ariel Prosperorsquos chief assistant and a powerful spirit

Ariel not only made the feast disappear but also delivered his speech blaming the King and his two companions for their past wicked deeds He warned them to repent for their misdeeds or suffer forever on that uninhabited island

(c) Who are referred to as lsquomeaner ministersrsquo What have they done

Prospero refers as lsquomeaner ministersrsquo to his other lesser spirits who were assisting Ariel in presenting a scene before the kingrsquos party They entered the scene to the accompaniment of music They assumed several strange shapes and brought in a banquet Then they danced about it with gentle actions of salutations thus inviting the King and others to eat the feast

These spirits play their role again when Ariel in the shape of a harpy quits the scene These shapes enter again and dancing with mocking gestures carry away the table

(d) Who are the speakerrsquos enemies What has happened to them

King of Naples Alonso his brother Sebastian and the present Duke of Milan Antonio (Prosperorsquos own brother) are Prosperorsquos enemies With the turn of events they have all been washed ashore on the island which is ruled by Prospero the great magician Actually this happened after the shipwreck caused by a storm which was raised by Prospero with the purpose of bringing these people to his island Prosperorsquos spirits have already confused and terrified these enemies and they are under Prosperorsquos control He can treat them as he likes

(e) What does he say about Ferdinand Explain what is meant by ldquohellip his and mine darlingrdquo

Prospero knows that Alonsorsquos son prince Ferdinand is alive though his father thinks that the prince has been drowned

Prospero refers to his daughter Miranda who is dear to him She is also very dear to Prince Ferdinand who has fallen in love with her They are waiting to be married soon for which they have received Prosperorsquos consent

4

ALONSO O it is monstrous monstrous (Line 95-102)

Methought the billows spoke and told me of it

The winds did sing it to me and the thunderThat deep and dreadful organ-

pipe pronouncedThe name of Prosper It did bass

my trespassTherefore my son ithrsquo ooze is

bedded andIll seek him deeper than eer

plummet soundedAnd with him there lie mudded

(a) In what way does Alonso express his horror when his conscience is awakened by Arielrsquos words

When Alonsorsquos conscience is awakened by Arielrsquos words he expresses his horror at what he has heard He gets the feeling that the waves of the ocean the wind and the loud thunder have spoken to him and uttered the name of Prospero Because of being reminded of his crime in a very loud and rough voice he comes to realize that he has lost his son for his past misdeeds

(b) What does Alonso imagine about his son What does Alonso want to do in his desperate state

Alonso imagines that his son is lying in the mud at the bottom of the sea He feels desperate that he wants to drown himself in the ocean deeper than the plumb-line has ever gone He wants to lie with his son at the bottom of the sea

(c) How do Sebastian and Antonio want to face the evil spirits

Sebastian says that he is not at all afraid of what the harpy has said and that he is prepared to fight any number of such monsters if they appear before him only one at a time Antonio says that he would support Sebastian in the fight against the fiendsyyy

(d) Why does Gonzalo ask Adrian to follow the three men

Gonzalo tells Adrian that all the three men namely Alonso Sebastian and Antonio are in a wild and reckless mood The thought of the heinous crime of which they are guilty has begun to torment their minds So he asks Adrian to follow those three men without loss of time and prevent them from doing anything which the turmoil in their minds might lead them to do

(e) What opinion do you form of Alonso from the above extract

Alonso who is the lesser villain is the first to give way to remorse under the effect of Arielrsquos speech The words of Ariel seem to him to be the voice of conscience speaking to him He is driven to desperation a state in which he might do violence to his life

Subject =Accounts

Ac-12 15420 topic-pL Appropriation ac

PROFIT AND LOSS APPROPRIATION ACCOUNT

MEANING AND PREPARATIONProfit and Loss Appropriation Account is merely an extension of the Profit and Loss Account of the firm The profit of the firm has to be distributed amongst the partners in their respective profit sharing ratio But before its distribution it needs to be adjusted All Adjustments like partnerrsquos salary partnerrsquos commission interest on capital interest on drawings etc are made in this account These adjustments will reduce the amount of profit for distribution This adjusted profit will be distributed amongst the partners in their profit sharing ratio To prepare it at first the balance of Profit and Loss Account is transferred to this account The journal entries for the preparation of Profit and Loss Appropriation Account are given below

1 for transfer of the balance of Profit and Loss Account to Profit and Loss Appropriation Account

(a) In case of Net Profit

Profit and Loss Ac helliphelliphelliphelliphellipDrTo Profit and Loss Appropriation Ac(Net Profit transferred to Profit and Loss Appropriation Ac)

(b)In case of Net Loss

Profit and Loss Appropriation Achelliphelliphellip DrTo Profit and Loss Ac(Net Loss transferred to Profit and Loss Appropriation Ac)

2 for Interest on Capital

For transferring on Interest on CapitalProfit and Loss Appropriation Achelliphelliphellip DrTo Interest on Capital Ac(Interest on capital transferred to Profit amp Loss Appropriation Ac)

3 for Interest on Drawings

For transferring Interest on Drawings Interest on Drawings Achelliphelliphelliphelliphelliphellip DrTo Profit and Loss Appropriation Ac(Interest on drawing transferred to Profit amp Loss Appropriation Ac)

4 For Partnerrsquos SalaryFor transfer of partnerrsquos SalaryProfit and Loss Appropriation Achelliphellip DrTo Salary Ac(Salary transferred to profit amp Loss Appropriation Ac)

5 For Partnerrsquos CommissionFor transferring commissionProfit and Loss Appropriation Achelliphelliphellip DrTo Commission Ac(Commission transferred to Profit and Loss Appropriation Ac)

6 For Transfer of agreed amount to General ReserveProfit and Loss Appropriation Ac helliphellipDrTo General Reserve Ac(Transfer to General Reserve)

7 for share of Profit or Loss appropriation(a) If ProfitProfit and Loss Appropriation Achelliphellip DrTo Partnerrsquos CapitalCurrent Ac(Profit transferred to capitalcurrent Ac)(b) If LossPartnerrsquos Capital Current Achelliphelliphelliphellip DrTo Profit and Loss Appropriation Ac(Loss transferred to capitalcurrent Ac)

THE FORMAT OF PROFIT AND LOSS APPROPRIATION

Profit and Loss Appropriation Account for the year endedhelliphelliphelliphellip

Particulars Amount Particulars Amount

To PL Ac (loss) By pL Ac (profit)

To Interest on capital BY Interest on drawings

To partner`s commission by Partner`s capital Ac ( loss)

To Partner`s salary To Interest on partner`s loan To General Reserve To Partner`s Capital AC (Profit)

Subject= Economics

MOVEMENT ALONG THE DEMAND CURVE (CHANGE IN QUANTITY DEMANDED)In law of demand you have already studied the inverse relationship between price and quantity demanded When quantity demanded of a commodity changes due to change in its price keeping other factors constant it is called change in quantity demanded It is graphically expressed as a movement along the same demand curve There can be either a downward movement or an upward movement along the same demand curve Upward movement along the same demand curve is called contraction of demand or decrease in quantity demanded and downward movement along the same demand curve is known as expansion of demand or increase in quantity demanded

Extention of demandd

price (rs)p A

B Extentionp1 d

Q Q1

Quantity demanded ( in units)

Contraction of demandd

p2 Ccontraction

p APrice (Rs)

d

Q2 Q

Quantity demanded (in units)

Explanation of movement of demand A fall in price from OP to OP1 leads to increase in quantity demanded from OQ to OQ1 (expansion of demand) resulting in a downward movement from point A to point B along the same demand curve DD When Price rises from OP to OP2 quantity demanded falls from OQ to OQ2 (contraction of demand) leading to an upward movement from point A to point C along the same demand curve DD

  • Activity Series of Metals
    • Drawbacks of Rutherfordrsquos model of atom
      • Electromagnetic radiations
      • Properties of electromagnetic radiations
      • Characteristics of electromagnetic radiations
        • Plancks Quantum Theory-
        • Photoelectric effect
          • Intext Questions
            • Logic gates
            • Digital systems are said to be constructed by using logic gates These gates are the AND OR NOT NAND NOR EXOR and EXNOR gates The basic operations are described below with the aid of truth tables
            • AND gate
            • Example
Page 3:  · Web viewSubject. Topic. Summary. Execution. English 1 . Chapter 1 naming words . Page 8. Write the names of these pictures:- Person:-1. father. 2.Firefighter 3.doctor 4 ...

8 She must finish her homework9 It rings at 630 am everyday10 They sing beautifully

EVS Ch 5 Eating healthy

Refer to pg 118Do in the book

C Match the followingAnswers

1 Drinking water ndash clean and filtered2 Eggs ndash food from animals3 Carrot and radish ndash vegetables4 A healthy mix of food items ndash

balanced diet5 Apple an orange ndashfruits

Bengali ইndashাংাসাহিতযপহিরয়

পাঠndashহিসংওইচেররগলপঅন(ীনীরপরচোততর

৯ াকযরনা -পাাndashআহিমপাাভাচোাহিসগাndashপাাচেরগায়হিসংচেরাসাহিছহিসংndashহিসংদেকপশরাায়ইরndashইরমানচে4রঅহিনষটকচেরহি(কাহিরndashহি(কাহিরচেনহি(কারকচের

Hindi वयाकरण Chapter 4 सजञा

सजञा -इस दनिनया म सभी का कोई ना कोई नाम होता ह यही नाम सजञा कहलाता ह नाम कई परकार क होत हजस- पश-पकषी जगह दि-नो क नाम वयकतियो क नाम आदि-

Read the lesson

Mathematics CHAPTER-3NUMBERS UPTO 1000

PLACE VALUE PAGE NO 1882WRITE THE PLACE VALUE FOR GIVEN 3-DIGIT NUMBERSb)368Place value of 8 is ones8Place value of 6 is tens60Place value of 3 is hundred300(Practice the rest exercise)

Class IIISubject Topic Summary Execution

ENGLISH II CHAPTER-2 THE WOODEN BOWL

Refer to pg 27COMPOSITION[ DO IN THE BLANK SPACE GIVEN IN YOUR BOOK]

COMPOSITION ABOUT A THING YOU MADE READ AND THE ANSWER EACH QUESTION BRIEFLY AND THEN DRAW AND COLOUR THE THING YOU HAD MADE FOR EXAMPLE IT CAN BE A CARD A EASY CRAFT ETC

SST INDIA-THE FESTIVALS OF INDIA

DONE Encircle the odd one out and give reason for your option1RajpathRajpath is not related to the India Independence

2Santa ClausSanta Claus is related to the Christmas

3 LangarLangar is related to the Gurupurab festival

4August 15August 15 is a National festival

BENGALI ই ndash াংা সাহিতয পহিরয়

পাঠ ndash ১একতাই অন(ীনীর পরচোততর

৫ হিপরীত - চো times দোকা স times ঃ হি( times অহি( াধা times ছাা আস times নক সহিতয times হিমচেথয৭ াকয রনা -মযাহিক ndash আমার কাকা ভাচো মযাহিক দোয় া(ঝা ndash রাচেতর অনধকাচের া(ঝা দেচে আমার ভয় াচেগ অহিনষট ndash অচেনযর অহিনষট হিনতা করচেত দেনই াহি= ndash কাগচের াহি= চে পচেরাচেনা স নহিথ পাওয়া দেগচো

HINDI वयाकरण Chapter 4 सजञा

सजञा- इस दनिनया म सभी का कोई ना कोई नाम होता ह यही नाम सजञा कहलाता ह नाम कई तरह क होत ह जस -वयकतियो क नाम पराणिणयो क नाम सथानो क नाम भावो क नाम आदि-

1सजञा निकस कहत ह उ-ाहरण -ीजिजएउततर- निकसी भी वयकति वसत सथान या भाव क नाम को सजञा कहत हजस- राम सकल घडी गरमी आदि-

MATHEMATICS CHAPTER-4 SUBTRACTION

Subtraction (with decomposing)Subtraction of 3-digit numbers

Exercise 18Example 1 (p-48)

Fill in the place holders3 hundreds 2 tens 5 ones ndash 2 hundreds 5 tens 7 ones= 3 hundreds 1 tens ___ ones - 2 hundreds 5 tens 7 ones= 2 hundreds ___ tens 15 ones - 2 hundreds 5 tens 7 ones= 6 tens ___ ones

Solution3 hundreds 2 tens 5 ones ndash 2 hundreds 5 tens 7 ones= 3 hundreds 1tens 15 ones - 2 hundreds 5 tens 7 ones= 2 hundreds 11 tens 15 ones - 2 hundreds 5 tens 7 ones= 6 tens 8 ones

Example 4 (p-51)Subtract 389 from 600

Solution H T O 6 0 0 - 3 8 9

2 1 1We cannot subtract 9 ones from 0 ones so we borrows 1 tens Here there is no ten so first we borrow 1 hundreds or 10 tens having behind 5 hundreds From 10 tens we borrow 1 tens or 10 ones leaving behind 9 tensNow 10 ones ndash 9 ones = 1 onesAnd 9 tens ndash 8 tens = 1 tens5 hundreds ndash 3 hundreds = 2 hundreds 600 ndash 389 = 211

Exercise2 Fill in the place holders5 hundreds 2 tens 8 ones ndash 3 hundreds 8

tens 3 ones= 4 hundreds ___ tens 8 ones - 3 hundreds 8 tens 3 ones= 1 hundreds ___ tens ___ ones

Solution5 hundreds 2 tens 8 ones ndash 3 hundreds 8 tens 3 ones= 4 hundreds 12 tens 8 ones - 3 hundreds 8 tens 3 ones= 1 hundreds 4 tens 5 ones

Class IVSubject Topic Summary Execution

English literature

Hercules andDeianira (tales from Greece and Rome)

Hercules spent the greater part of his life in helping the poor and weak people But Juno still wanted to harm him So she sent him into all sorts of dangers But he was brave and strong and he overcame all the dangers Juno made him a slave to the king of Argos Hercules worked hard for the king The king took pity on him and told him that he would set him free if Hercules perform twelve difficult tasks Hercules accepted the challenge

1How did Hercules spent the greater part of his lifeAns Hercules spentthe greater part of his life in helping the poor and weak people

2How did Juno tried to harm HerculesAns Juno tried to harm Hercules by sending him into all sorts of dangers

3Under which king Hercules was working as a slaveAns King of Argos

4What was the challenge that king of Argosgive to HerculesAns King of Argos told Hercules that he would set him free if he perform twelve difficult tasks which Hercules accepted

Social studies Major landforms on earth

Answer the following questions5What do oceans provide usAns Oceans provide us a variety of sea food

6What is alluviumAns The rivers carry sand soil and small pieces of rocks This fine soil or silt known as alluvium is very good for growing crops

COMPUTER160420

CHAPTER 3 EDITING IN MS WORD

DRAG AND DROPTHE DRAG AND DROP METHOD OF MOVING THE TEXT ALLOWS US TO MOVE THE SELECTED TEXT USING MOUSE FROM ONE LOCATION TO ANOTHER WITHIN A DOCUMENT

Q7) HOW TO WE DRAG AND DROP TEXT IN MS WORDAns) WE CAN DRAG AND DROP TEXT IN MS WORD FOLLOWING STEPS ARE---

SELECT THE TEXT PLACE THE MOUSE POINTER

ANYWHERE ON THE SELECTED TEXT

CLICK AND HOLD THE LEFT MOUSE BUTTON UNTIL THE INSERTION POINT CHANGES INTO A WHITE ARROW

WHEN WE REACH THE PLACE IN THE DOCUMENT WHERE WE WANT TO MOVE THE TEXT RELEASE THE MOUSE BUTTON

Science Adaptations in Summary Execution

Animals Animals are divided into different groups according to their habitats Some animals live on land some animals live both on land and in water some animals adapted themselves for food and some for protection They develop special features that help them to survive in their habitats Animals that live on land are called Terrestrial animals such as lions tigers Yaks polar bears Polar Bears live cold polar regions Yaks live in mountains These animals have thick fur on their body and a thick layer of fat under their skin which keeps them warm

Solved exercise question and answersA Write the correct answer1Terrestrial animals live here - on land 2This animal lives in polar regions ndash polar bear 3 These help a fish to breathe in water - gills4 This helps carnivorous birds to tear flesh - a hooked beak

MATHEMATICS

Ch 6Multiplication

Exercise 17 Example 2

Multiply 224835 by 4 Solution

2 2 4 8 3 5 times 48 9 9 3 4 0

Exercise

18 3 1 1 4 6 7 19 2 1 2 7 8 5 times 3 times 4 9 3 4 4 0 1 8 5 1 1 4 0

20 2 0 6 1 3 times 9 1 8 5 5 1 7

Class VSubject Topic Summary Execution

Science Chapter 2 - The Skeletal System

Posture

The way in which we hold our body while sitting standing walking and lying down is known as posture

Importance of correct posture and exercise

1 Correct posture and exercise makes our bones and muscles strong 2 It provides our body more energy to do works

3 It keeps our body stressless 4 It also prevent back ache and muscle pain

Some important yoga asanas are-

1 Veerabhadrasana ( Warrior pose )

2 Dhanurasana ( Bow pose )

3 Trikonasana ( Triangle pose )

D Answer these questions

5How do our muscle work

Ans ndash Our muscles work by contracting and relaxing

6Why should we maintain a correct posture

Ans ndash We should maintain a correct posture because ndash

i Correct posture and exercise makes our bones and muscles strong

ii It provides our body more energy to do works

iii It keeps our

body stressles

s iv It also prevent

back ache and muscle pain

Social studies Conquering distances

Put a tick on the correct option1 b Ship 2b water transport 3 b trains 4 c 13 5 b Wright brothersTrue or false1 False 2 True 3 True 4 True

MATHEMATICS

Ch 3Addition and Subtraction

Exercise 126 A school needs Rs 4987653 for its building It has only Rs 3592468 in its accounts Estimate the money by rounding off to the nearest lakh it has still to raise

Solution Estimated amount need Rs 5000000 The school has only Rs 3600000 Still to raise Rs 1400000

MATHEMATICS

Ch 4

Mul

tiplic

ation

and

Div

ision

We have learnt multiplication tables up to 15 Let us extend the tables up to 20X 11 12 13 14 15 16 17 18 19 201 11 12 13 14 15 16 17 18 19 202 22 24 26 28 30 32 34 36 38 403 33 36 39 42 45 48 51 54 57 604 44 48 52 56 60 64 68 72 76 805 55 60 65 70 75 80 85 90 95 1006 66 72 78 84 90 96 102 108 114 1207 77 84 91 98 105 112 119 126 133 1408 88 96 104 112 120 128 136 144 152 1609 99 108 117 126 135 144 153 162 171 180

10 110 120 130 140 150 160 170 180 190 200Properties of Multiplication1 The product of two numbers does not change when the order of numbers is changed eg 503times23 = 23times503 [This property is called Commutative Property of multiplication]

2 The product of three numbers does not change when the grouping of numbers is change eg (15times18iquesttimes10=15 times(18times 10) = (15times10iquesttimes18 [This property is called Associative Property of multiplication]

3 The product of a number and 1 is the number itself eg 1513 times 1 = 1513 [This property is called Identity Property of multiplication and the integer 1 is called Identity Element of multiplication]

4 The product of a number and 0 is 0 eg 718205times 0 = 0 times 718205 = 05 The product of a number by the sum of two numbers is equal to the sum of the

products of that number by the two numbers separately eg123 times (105+ 48) = 123 times 105 + 123 times 48 [ This property is called Distributive Property of multiplication over

addition]English language

Transitive and intransitive verb

Pick out the verbs from the following sentences and say whether they are transitive or intransitive verb6 Was fullndash verb Intransitive verb7 Have been decorated- verb Transitive verb8 Happy cheerful ndash verb Intransitive verb9 Shall come back ndash verb Transitive verb

English 2 The fall of Lanka

This is the story of the fight between Rama and Ravana as told by Valmiki in lsquoThe Ramayana The monkeys worked all day and all night and at last built a bridge so that Rama and

Write the synonyms of

1 Stationed-

his army could cross to Lanka and rescue Sita assign2 Invade- enter

a country or a region so as to subjugate or occupy it

3 Prowess- bravery in battle

4 Haughty- arrogantly

5 Puny- small and weak

6 Spy- secret agent

7 Dreadful- causing or involving great suffering

8 Violate- disobey

9 Ghastly- causing great horror or fear

10 Deception ndash misleading

Class VISubject Topic Summary Execution

HISTORY AND CIVICS

CHAPTER 3

MAHAVIRA AND BUDDHA ndash GREAT PREACHERS

BUDDHA

Impact of Buddhism on Indian Cultures

Impact on Religion

Buddharsquos practical and simple doctrines made their impact on HinduismThe principle of ahimsaIt brought about a great change in the performance of costly yanjnas and sacrifices which previously involved immense loss of life The Mahayana Buddhists adopted the practice of worshipping Buddha and bodhisattvas making idols and erecting temples in their honour

Impact on Literature

After the death of Buddha his teachings were compiled and called TripitakasThe Jatakas contain tales dealing with the previous births of Buddha

Impact on Education

The Buddhist monasteries became great centres of learning These centres of learning developed into famous universities- Nalanda Taxila Vikramshila etc

Impact on Art and Architecture

The gateways and railing of the Sanchi Stupa were covered with sculptured figuresCave- temples were also constructed which were decorated with beautiful frescoes

1) How many parts of Tripitakas are thereAns -There are three parts of Tripitakas- Sutta Pitaka Vinaya PitakaAbhidhamma Pitaka

2)What are the subjects taught in these monasteries Ans ndash Buddhist scriptures logic Philosophy medicine astronomy etc

3) Which art was developed under Buddhist patronage Ans ndash Gandhara art

The Gandhara art was developed under Buddhist patronage

BIOLOGY The Leaf Photosynthesis The process by which green plants make their own food from carbon dioxide and water in the presence of sunlight and chlorophyll is called photosynthesis

All green plants need the following to make their food ndash

water carbon dioxide chlorophyll and energy in the form of sunlight

Carbon dioxide + water ------------- Glucose + oxygen

The end product of photosynthesis is glucose

Fill in the blanks

1 Plants make their food by the process of photosynthesis

2 The inner wall of the guard cell is thicker than the outer wall

3 The extra glucose is converted into starch and sucrose

4 The leaf is boiled in alcohol to remove chlorophyll

5 The rate of transpiration is more on the hot day then a cold day

6 Photosynthesis helps to observe water and minerals from the soil

English 1 Pronouns Kinds of pronouns 1 Personal pronouns2 Possessive pronouns3 Reflexive pronouns4 Interrogative pronouns5 Relative pronouns6 Demonstrative pronouns7 Indefinite pronouns

Personal pronouns they refer to first second and third person in sentences First person- the speakerSecond person-the listenerThird person-the objectperson being spoken aboutPersonal pronouns should have the same gender and number as the nouns they refer to

Possessive pronouns these are used to indicate the relationship between the objects and people These pronouns include mine ours yours his hersand theirs

ExerciseBFill in the blanks with suitable pronounsThere was much excitement among the childrenTheywere eagerly looking forward to the annual picniclsquoAre they going to Lodhi Gardens toorsquo wondered AneeshlsquoNo they are going to Buddh Jayanti Park with Mrs Jain said Mrs ChopraThe children looked disappointedlsquoWonrsquot you be taking us Marsquoam rsquo they askedlsquoSorry children I have to go to Mumbai for a week to look after my sick mother But you will have fun with Mrs Jain she is full of laughter and you will love being with her the whole daylsquoIt will not be the same they grumbled

English 2 The great train journey- Ruskin Bond

The great journey by Ruskin Bond is a story about Suraj who loved trains and wanted to go to places One day while wandering along the railway tracks he enters into a carriage compartment The train suddenly starts moving with him in the compartment and after a journey returns back to the same place from where it had begun The story is about his experience during that journey

State true or false1 When the train had passed leaving behind the

hot empty track Suraj was lonely2 It was winter holidays

and Suraj did not know what to do with himself

3 He plunged his hands into the straw and pulled out an apple

4 A dirty bearded face was looking out at him from behind a pile of crates

5 Suraj wanted to go to Japan

Hindi 2nd lang

गललबाजलडका खालीसथानोकोभरो-6 गो-ामसनिनकलकरहमगराजमआगए7 माबोधराजकोराकषससमझतीथी8 चीलरोशन-ानमसअ-रआकरतहसीलपरबठगई9 तीनचारतीनकऔररईकगोलउडलनिकनघोसलानहीनिगरा10 वहसवयतोघोसलातोडनककतिलएगललउठालायाथा

11 -ीवारकसाथलगतगोहपजोकसहार-ीवारपकडलतीह12 बोधराजअभीभीटकटकीबाधचीलकीओर-खरहाथा13 बोधराजअपनीजबमबहतसाचगगाभरकरलायाथा14 मरनिपताजीकीतरककीहईऔरहमलोगएकबडघरमजाकररहनलग15 बागमजातातोफलपरबठीनिततलीको-खनिततलीकोपकडकरउगकतिलयोकबीचमसल-ता

BENGALI(2ND LANGUAGE)

সহিনধসবরপওসবরসহিনধ

সবরসহিনধরহিনয়ম- ৯ই-কারহিকংাঈ-কাচেররপচেরইাঈহিভননঅনযসবররণCথাকচেইাঈসথাদেনয-ফায়এংওইয- ফাপCচেরণCযকতয়

১০উ-কারহিকংাঊ-কাচেররপচেরইাঈহিভননঅনযসবররণCথাকচেউাঊসথাদেন-ফায়এংওই- ফাপCচেরণCযকতয়

১১ঋ- কাচেররপচেরঋহিভননঅনযসবররণCথাকচেঋসথাচেনর -ফায়এংওইর পCচেরণCযকতয়

১২সবররণCপচেরথাকচেপCতMএ-সথাচেনঅয় ঐ- সথাচেনআয় ও- সথাচেনঅএংঔ- সথাচেনঅায়

৯ই+ অ= য- ফাআহি+ অনত= আযনত অহিধ+ অয়ন= অধযয়নই+ আ=য- ফা+ াইহিত+ আহি= ইতযাহি পরহিত+ আতC ন= পরতযাতC নই+ উ=য- ফা+ উঅহিত+ উহিকত= অতযহিকত হি+ উৎপহিতত= যৎপহিততই+ ঊ= য- ফা+ ঊ ই+ এ= য- ফা+ এঈ+ অ= য- ফা পরহিত+ ঊ4= পরতয4 পরহিত+ এক= পরচেতযকঈ+ অ আ= য- ফা+ অ আনী+ অমব= নযমব মসী+ আধার= মসযাধার

১০উ+ অ= অন+ অয়= অনবয় পশ+ অধম= পশবধমউ+ আ= াস+ আগত= সবাগত পশ+ আহি= পশবাহিউ+ ঈ= হিঅন+ ইত= অহিনবতউ+ এ= দেঅন+ এ4রণ= অচেনব4রণউ+ ঈ= ীসাধ+ ঈ= সবাধবী তন+ ঈ= তনবী

১১ঋ+ অ= র মাত+ অনমহিত= মাতরনমহিতঋ+ আ= রা হিপত+ আয়= হিপতরায়ঋ+ ই= হির মাত+ ইচছা= মাতচছাঋ+ ঈ= রী ধাত+ ঈ= ধাতরীঋ+ উ= র ভরাত+ উপচে(= ভরাতরপচে(

১২এ+ অ= অয় দেন+ অন= নয়নঐ+ অ= আয় গৈগ+ অক= গায়কও+ অ= অ দেপা+ অন= পনও+ ই= অ দেপা+ ইতর= পহিতরও+ এ= অ দেগা+ এ4রণা= গচে4রণাঔ+ অ= অা দেপৌ+ অক= পাকঔ+ ই= অা দেনৌ+ ইক= নাহিকঔ+ উ= অা দেভৌ+ উক= ভাক

MATHS Topic NumbersChapter Natural numbers and whole numbers

Study item properties of whole numbers for multiplication

1 Closure property If x and y are two whole numbers then xtimesy is also a whole numberExample If x = 9 and y =3 then xtimesy = 9times3 = 27 which is a whole number

2 Commutative property If x and y are two whole numbers then xtimesy = ytimesxExample If x = 5 and y = 2 then xtimesy = 5times2 = 10y times x = 2times5 = 10Therefore 5times2 = 2times5

3 Associative property If x y and z are three whole numbers then x times(ytimesz) = (xtimesy) times zExample If x =3 y = 5 and z = 7 then 3 times (5times7) = 3 times (35) = 105And (3times5) times7 = (15) times 7 = 105Therefore x times (ytimesz) = (xtimesy) timesz

4 Distributive property If x y and z are three whole numbers then xtimes (y + z) = x times y + x times z

Therefore the multiplication of whole numbers is distributive over their additionExample If x = 5 y = 3 and z= 2Therefore x times (y + z) = 5 times (3 + 2) = 5times5 =25And x times y + xtimes z = 5times3 +5times2 =15 +10 = 25Again x times (y ndash z ) = x times y ndash x timesz Therefore 5 times ( 3 - 2) = 5 times1 = 5 and 5times3 ndash 5 times2 = 15 ndash 10 = 5Therefore the multiplication of whole numbers is also distributive over their subtraction if y is greater than z

5 Existence of identity If x is a whole number then

X times1 = x 1 times x = xTherefore we can write x times1 = 1 times xTherefore the multiplication of any whole number with 1 is the number itselfTherefore we can say that 1 is multiplicative identity or identity element for multiplicationExample 5 times1 = 5 1 times 5 = 5 Therefore 5 times 1 = 5

6 Multiplicative inverse If x is any whole number ( x is not equal to zero ) then its multiplicative inverse will be 1xSo x times 1x = 1 but 1x is a whole number if x = 1For other values of whole number 1x is not a whole number therefore we can write its multiplicative inverse does not exists

7 Cancellation law of multiplication If x y and z are three non- zero whole numbers then x times y = x times z

Or y = zExample 9 times y = 9 timeszTherefore y = z

Class VIISubject Topic Summary Execution

English 2 Sentences based on meanings

Kinds of sentences

Assertive or declarative to convey information or simply make a statement

Interrogative to ask different types of questions

Imperative to command or instruct someone or make a request

Exclamatory to express strong feelings and emotions

Exercise c1 What a nice compliment that is

That is a nice compliment2 How well- behaved the children

areThe children are very well-behaved

3 What great chefs we areWe are great chefs

4 What a shame it isIt is a shame

5 What a fantastic idea you haveYou have a fantastic idea

Homework 6 -10English

LiteratureThe Listeners III) Answer the following questions-

d) Identify two words used in the poem to give the poem an eerie atmosphereAns- Two words used to give the poem an eerie atmosphere are ldquogreyrdquo and ldquophantomrdquo

e) Who do you think are the inmates of the houseAns- I think the inmates of the house are phantom who once used to dwell in it

f) Why was the poet ldquoperplexed and stillrdquoAns- He was lsquo perplexed and stillrsquo because he was expecting an answer from the inmates of the house But despite of repeated calls there was no response

CHEMISTRY Chapter 2 ndashElement and Compound

Activity Series of MetalsThe activity series is a chart of metals listed in order of declining relative reactivity The top metals are more reactive than the metals on the bottomMetal SymbolReactivity

Lithium Li displaces H2 gas from water steam and acids and forms hydroxides

Potassium K

Strontium Sr

Calcium Ca

Sodium Na

Magnesium Mg displaces H2 gas from steam and acids and forms hydroxides

Aluminum Al

Zinc Zn

Chromium Cr

Iron Fe displaces H2 gas from acids only and forms hydroxides

Cadmium Cd

Cobalt Co

Nickel Ni

Tin Sn

Lead Pb

Hydrogen gas

H2 included for comparison

Antimony Sb combines with O2 to form oxides and cannot displace H2

Arsenic As

Bismuth Bi

Copper Cu

Mercury Hg found free in nature oxides decompose with heating

Silver Ag

Palladium Pd

Platinum Pt

Gold Au

Answer the following Q)Difference Between Metals And Nonmetals

Metals Nonmetals

These are solids at room temperature except mercury

These exist in all three states

These are very hard except sodium

These are soft except diamond

These are malleable and ductile

These are brittle and can be breakdown into pieces

These are shiny These are non-lustrous except iodine

Electropositive in nature Electronegative in nature

Have high densities Have low densities

Math Number System

Chapter Fraction

Study item Some solved sums from exercise 3(B)1) For each pair given below state whether it from like fractions or unlike

fractions (i) 58 and 78

= Like Fraction because denominators same(ii) 815 and 821

= Unlike Fraction because denominators are not same

(iii) 49 and 94 = Unlike Fraction

2) Convert given fractions into fractions with equal denominators(iii) 45 1720 2340 and 1116Solution Given fraction 45 1720 2340 and 1116Therefore the LCM of 5 20 40 and 16 is 80Therefore 45 = 4times165times16 = 64801720 = 17times420times4 = 68802340 = 23times240times2 = 4680 1116 = 11times516times5 = 5580

3) Convert given fractions into fractions with equal numerators(iii) 1519 2528 911 and 4547Solution Given fractions 1519 2528 911 and 4547Therefore the LCM of 15 25 9 and 45 is 2251519 = 15times1519times15 = 225285 2528 = 25times928times9 = 225252911 = 9times2511times25 = 2252754547 = 45times547times5 = 225235

4) Put the given fractions in ascending order by making denominators equal

(iii) 57 38 914 and 2021Solution Given fraction 57 38 914 and 2021Therefore the LCM of the denominators is 16857 = 5times247times24 = 12016838 = 3times218times21 = 63168914 = 9times1214times12 = 1081682021 = 20times821times8 = 160168Therefore ascending order 63168lt108168lt120168lt160168Therefore ascending order of given fractions38lt914lt57lt2021

COMPUTER CHAPTER-1COMPUTER FUNDAMENTALS

DONE IN THE PREVIOUS CLASSES PAGE 10CWRITE TRUE AND FALSE

1 True2 False3 False4 False5 True

GEOGRAPHY CHAPTER 7EUROPE

CHAPTER COMPLETE 1)Europe is home to a famous mountain range called the Alps

2)River Rhine originates in Switzerland

3)The Eiffel Tower one of the tallest structures in Europe

4) Vatican City is one of the most densely populated European countries

5)Sognefjordin Norway is the largest fjord in Norway

Class VIIISubject Topic Summary Execution

MATHEMATICS Ch 6Sets

Exercise 6 (D)1 Given A = x x isin N and 3iquest x le 6 and B = x x isin W and xlt4 find (i) Sets A and B in roster form (ii) A cup B (iii)

A cap B(iv) A ndash B (v) B ndashA

Solution (i) A = 456 and B = 0123

(ii) A cup B = 0123456 (iii) A cap B = ϕ (iv) A ndash B = 456 (v)B ndash A = 0123

3 If A = 56789 B = x 3 lt x lt 8 and x isin W and C = x xle5 and x isin N Find (i) A cup B and (A cup B) cup C (ii) B

cup C and A cup ( B cup C)

(iii) A cap B and (A cap B) cap C (iv) B cap C and A cap (B cap C)

Is (A cup B) cup C = A cup (B cup C)

Is (A cap B) cap C = A cap (B cap C)

SolutionA = 56789 B = 4567 C = 12345

there4 (i) A cap B = 456789 and (A cup B) cup C = 123456789

(ii) B cup C = 1234567 and A cup ( B cup C) = 123456789

(iii) A cap B = 567 and (A cap B) cap C = 5

(iv) B cap C = 45 and A cap (B cap C) = 5

Now (A cup B) cup C = 123456789

And A cup ( B cup C) = 123456789 there4 (A cup B) cup C = A cup (B cup C)

Again (A cap B) cap C = 5 and A cap (B cap C) = 5

there4 (A cap B) cap C = A cap (B cap C)

4 Given A = 012345 B = 02468 and C = 0369 Show that (i) A cup (B cup C) = (A cup B) cup C ie the union

of sets is associative (ii) A cap (B cap C) = (A cap B) cap C ie the intersection of sets is associative

SolutionNow B cup C = 0234689 and A cup B = 01234568

there4 A cup (B cup C) = 012345689 and

(A cup B) cup C = 012345689

So (i) A cup (B cup C) = (A cup B) cup C ie the union of sets is associative

Again B cap C = 06 and A cap B = 024

there4 A cap (B cap C) = 0 and (A cap B) cap C = 0

So (ii) A cap (B cap C) = (A cap B) cap C ie the intersection of sets is associative

Physics Chapter 2 Physical Quatites and Measurements

Here We Will Do Some QuestionsRelated To Chapter 2

A density bottle has a marking 25 mL on it It means that

1 the mass of density bottle is 25g

2 the density bottle will store 25 ml of any liquid in it

3 the density bottle will store 25 ml of water but more volume of liquid denser than water

4 the density bottle will store 25 ml of water but more volume of a liquid lighter than water

Solution 2 the density bottle will store 25 ml of any liquid in it

COMPUTER CHAPTER-2Spreadsheet Functions and Charts

SELECTING RANGE IN ROWSCOLUMNSWHEN TWO OR MORE CELLS ARE SELECTED IT IS CALLED A RANGEA RANGE OF CELLS CAN BE FORMED IN TWO WAYS--a) SELECTING RANGE BY USING THE MOUSEb) SELECTING RANGE BY USING THE KEYBOARD

Q1)WRITE THE STEPS TO SELECT PARTIAL RANGE IN A ROW

Ans)THE STEPS ARE-6 SELECT THE ROW7 BRING THE CELL POINTER TO THE DESIRED

LOCATION FROM WHERE YOU WANT TO START YOUR SELECTION

8 CLICK THE LEFT MOUSE BUTTON AND KEEP DRAGGING TO YOUR RIGHT TILL YOU REACH THE LAST CELL TO NE SELECTED

RELEASE THE MOUSE BUTTON

GEOGRAPHY Asia

CLIMATE

Asia experiences great extremes of climate Jacobabad in the Sind province of Pakistan is one of the hottest places in the WorldVerkhoyansk in Siberia is one of the coldest places in the WorldCherrapunji and Mawsynram in India are two wettest places in WorldArabia Tibet Gobi and Mongolia are extremely dry regionsFactors Affecting Climate of Asia-The factors influencing the climate of Asia are-

Factors Affecting Climate of Asia-Thoroughly read the table in page number 60

Latitudinal extent

Continentality

Relief features

Presence of low pressure trough

Jet streams

English Language The Sentence A complex sentence contains one independent clause and at least one dependent clause The dependent clause in a complex sentence is introduced with subordinating conjunctions or relative pronouns

Commonly Used Subordinating Conjunctions-Time after before while when since untilCause And Effect because now since as in order that soOpposition although though even though whereas while in spite ofCondition if unless only if whether or not even if in case(that)

Commonly Used Relative Pronouns-Who whose whom which whoever whomever whichever that

Class IXSubject Topic Summary Execution

1-BENGALI(2ND LANGUAGE)

ldquo বঙগভমিরপরমিrdquo াইকেলধসদনদতত

আচেগর পর উততর পচো-১ ২ ৩ এং নীচের পর টি াহির কা- ৪মহিbকাও গচেনা দেগা পহিচে অমত হরচে- ক) কার দো দেকান কহিতার অং( ) কতা দেক পরসঙগ কী উহিকতটির তাৎপযC আচোনা কচেরা৫দেসই ধনয নরকচে দোচেক যাচে নাহি ভচে মচেনর মহিeচের সাচেসচে সCন ক) কহির কায C ার উচেf(য হিক হিছ কহি কন কহিতাটি দেচেন) কহি কার কাচেছ হিমনহিত কচেরচেছনগ) কহি এই পহিথীচেত কাচের ধনয মচেন কচেরনঘ) কহি হিক রকম অমর তাাভ করচেত ান

Hindi 2nd lang

काकी(कतिसयारामशरण गपत)

इस कहानी म लखक न यह बतान का परयास निकया ह निक बचच अपनी मा स निकतना परम करत ह शयाम अबोध बालक ह वह अपनी मा क मरन क बा- उसन अपनी मा क कतिलए बहत रोया बा- म उस पता चला निक उसकी मा राम क घर चली गई ह आकाश म उडती हई पतग -खकर उस हरष हआ निक पतग क दवारा वह अपनी मा को नीच उतारगा इसक कतिलए वह अपनी निपता की जब स -ो बार सवा रपया निनकालकर पतग और -ो मोटी सी मन वाली अपन भाई स काकी एक कागज पर कतिलखवा कर पतग म कतिशव का दि-यानिनकालकर पतग और -ो मोटी सी मन वाली अपन भाई स काकी एक कागज पर कतिलखवा कर पतग म कतिचपका दि-याभोला और शयाम कोठरी म रससी बाधनी रह थ तभी उसक निपता करोध म आकर उन स पछ निक कया उनकी जब स रपया निनकाला हभोला डर क मार बताया निक शयाम इस पतग क दवारा अपनी काकी को राम क यहा स उतारना चाहता हनिवशशवर(शयाम क निपता)न फटी पतग उठाकर -खी तो उस पर काकी कतिलखा थावह हत बजिa होकर वही खड रह गएउनहोन सोचा निक मन अपन पतर को मारा जोनिक अनजान और निन-dरष थावह अपनी मा कोनिकतना पयार करता ह

उस दि-न बड सवर शयाम की नी- खली तो -खा निक घर भर म कोहराम मचा हआ ह

क) घर म कोहराम कयो मचा हआ था शयाम को कया लगा

ख) काकी को ल जात समय शयाम न कया उपदरव मचाया

ग) काकी क बार म उस कया बताया गया कया सतय उस कतिछपा रहा

घ) वह बठा-बठा शनय मन स आकाश की ओर कयोकरता

उततरक) शयाम की मा का -हात हो गया था इसकतिलए

घर म कोहराम मचा हआ था शयाम की लगा निक उसकी मा सफ- कपडा ओढ हए भमिम पर सो रही ह

ख) लोग जब उमा यानी शयाम की मा को उठाकर ल जान लग तब शयाम न बडा उपदरव मचाया लोगो क हाथ स झठ करवा उमा क ऊपर जा निगरा और बोला काकी सो रही ह उस कहा ल जा रह हो

ग) काकी क बार म बजिaमान लोगो न उस निवशवास दि-लाया निक उसकी का निक उसक मामा क यहा गई ह लनिकन सतय अमिधक दि-नो तक कतिछपाना रह सका आसपास क अबोध बालको क मह स यह बात परकट हो गई निक उसकी मा का -हात हो गया ह

घ) कई दि-नकई दि-न लगातार रोत-रोत उसका रोना तो शान हो गया पर उसक ह-य म शोक भर गया था वह चपचाप बठा आकाश की और टाका करता निक शाय- उसकी काकी कही दि-ख जाए

ldquoदि-न उसन ऊपर आसमान म पतग उडती -खी न जान कया सोच कर उसका निहर-य एक-म खिखल उठाrdquo

क) निकसन पतग ऊपर उडत -खी और वह कयो खश हआ

ख) उसन अपन निपता स कया कहा उनका कया उतर थाश

ग) उसन निफर कया निकया और निकसन उसकी सहायता की

घ) उसकी योजना कया थी उततर -क) शयाम न एक दि-न आसमान म पतग उडती

-खी तो उसन सोचा निक पता आसमान म राम क यहा जाकर रकगी वही पर मरी काकी ह यह सोचकर वह बहत खश हआ

ख) उसन अपन निपता स कहा काका मझ एक पतग मगा -ो उसक निपता न भटक हए मन क भाव स कहा निक मगा -ग यह कह कर उ-ास भाव स वह कही और चल गए पतग नही आई

ग) उसन चपचाप निवशशवर क टगहए कोट स एक चवननी निनकाल ली और सखिखया -ासी क लडक भोला की सहायता स एक पतग मगवानी भोला उसकी बराबर उमर का ही था

घ) उसकी योजना यह थी निक वह अपनी पतग को आकाश म राम क यहा भजगा और उस पतग क सहार उसकी काफी नीच उतर जाएगी इस योजना पर उस परा निवशवास था इसकतिलए वह और भोला -ोनो यह काम करन म लग गए

Continue to nexthelliphellipEVS CHAPTER - 1

(UNDERSTANDING OUR ENVIRONMENT)

Sustainable development

The development that meets the needs of the present without compromising the ability of future generations to meet their own needs is called Sustainable development

Sustainable societies ndash

An environmentally sustainable community is one that meets the current and future basic resource needs of its people in a just and equitable manner without compromising the ability of future generations to meet their basic needs

Q ) What are Eco Villages

Ans - Eco village are the urban or rural communities of people who strive to integrate a supportive social environment with a low impact way of life

Q ) To ensure sustainable development the depletion of renewable resources should not take place at a rate faster than their regeneration Justify your answer

Ans ndash Renewable resources do not have a fixed quantity - more can always be

generated However if the rate of use exceeds the rate of renewal - that is the

source is used more than its being recreated - its continued use will become

used up faster than it can regenerate

To promote sustainable society the following things need to be done ndash

1 Using renewable energy sources 2 By improving the quality of human

health 3 By promoting sustainable agriculture 4 By forming ecovillage

it will eventually be entirely depleted So Toensure sustainable development the depletion of

renewable resources should nottake place at a rate faster than their regeneration

Q ) What do you mean by Sustainable societies

Ans - Sustainable societies are defined as towns and cities that have taken steps to remain healthy over the long term These communities value healthy ecosystems use resources efficiently and actively seek to retain and enhance a locally based economy Sustainable development concerns everybody in a society

Q ) What are the effects of pollution on human health

Ans ndash Some health problem occurs due to air pollution are ndash

Respiratory diseases Cardiovascular damage Fatigue headaches and anxiety Irritation of the eyes nose and throat Damage to reproductive organs Harm to the liver spleen and blood Nervous system damage

Some health problem occurs due to water pollution are ndash

Typhoid Cholera Dysentry Jaundice

Some health problem occurs due to noise pollution are ndash

Fatigue headaches and anxiety High blood pressure Hearing damage

Physics Motion in 1D First go through previous notes Now here we will solve some numerical related to that

Question 3What information about the motion of a body is obtained from the displacement-time graphSolution 3From displacement-time graph the nature of motion (or state of rest) can be understood The slope of this graph gives the value of velocity of the body at any instant of time using which the velocity-time graph can also be drawn

Question 4(a)What does the slope of a displacement-time graph represent(b)Can displacement-time sketch be parallel to the displacement axis Give a reason to your answerSolution 4(a) Slope of a displacement-time graph represents velocity(b) The displacement-time graph can never be parallel to the displacement axis because such a line would mean that the distance covered by the body in a certain direction increases without any

increase in time which is not possible

Chemistry Language of Chemistry

How to balance a chemical equationThere are two methods of balancing an equation(i)Hit and trial method(ii)Partial equation methodBalancing by hit and trial methodThis method consists of counting the number of atoms of each elements on both sides and trying to equalize themTake the following steps(i)Count the number of times (frequency) an element occurs on either side(ii)The element with the least frequency of occurrence is balanced first(iii)When two or more elements have the same frequencythe metallic element is balanced firstExample-1 On heatinglead nitrate decomposes to give lead dioxidenitrogen dioxide and oxygenPb(NO3)2rarrPbO+NO2+O2

In this equationLead occurs twiceNitrogen occurs twiceOxygen occurs four timesSince lead is a metalbalance it firstThe number of atom of lead is equal on the two sidestherefore it needs no balancingNow balance nitrogenOn the reactant sidethere are two atoms of nitrogenwhile on the product side oneSomultiply the product containing nitrogenon the product sideby two Pb (NO3)2rarrPbO+2NO2+O2Nowthe number of oxygen atoms on the reactant side 6while on the product sideit is 7Somultiply the entire equation by 2except oxygen to get balanced equation2Pb(NO3)2rarr2PbO+4NO2+O2Multiplication by 2 is done only when atoms of all the elements except one element are balanced and the unbalanced atom occurs separately at least once and also there is a difference of only one such atom

Math Topic AlgebraChapter

Factorisation

Study item Difference of two squares a2 ndash b2 = (a+b) (a-b)1) (i) 4x2ndash 25y2

= (2x) 2 ndash (5y) 2= (2x + 5y) (2x - 5y)

(ii) 9x2 ndash 1= (3x)2ndash(1)2= (3x + 1)(3x ndash 1)

2) (i) 150 ndash 6a2= 6(25 ndash a2)= 6(5)2 ndash(a)2= 6 (5 + a) (5 ndash a)

(ii) 32x2 ndash 18y2=2(16x2 ndash 9y2)=2(4x)2 ndash (3y)2= 2(4x + 3y)(4x - 3y)3)(i) (x ndashy )2 ndash 9 = (x ndash y )2 ndash (3)2= (x ndash y + 3) (x ndash y ndash 3)(ii) 9(x + y) 2ndash x 2= (3)2(x + y)2 ndash (x)2=3(x + y)2 ndash (x)2= (3x +3y ) 2ndash(x)2= (3x + 3y + x)(3x +3y ndash x)= (4x + 3y) ( 2x + 3y )

Commercial studies

Basic accounting terms

Today I will give you some questions from the previous study material

Questions1) Define accounting2) What do you mean by debit and

credit

3) Explain the types of account4) Define the following terms

a) Assetsb) Capitalc) Purchased) Debtorse) Transactions

5) Name the types of accounts given below

a) Krishnas accountb) Machinery accountc) Royalty accountd) Salary accounte) Furniture accountf) Audit fee account

Economics Revision Today I will give you some revision questions

Questions1) What do you mean by the terms

rdquowantsrdquo2) Write the difference between

consumer goods and producer goods

3) Define the term utility 4) Explain the different types of utility5) Define

a) Total utilityb) Marginal utility

Subject Eng Literature (The Merchant of Venice ndash William Shakespeare)Topic Act I Scene 3 Lines 1 to 48 (Shylock hellip Cursed be my tribe if I forgive him) Date 16th April 2020 (5th Period)

[Students should read the original play and also the paraphrase given in the school prescribed textbook]Summary Questions amp Answers

This scene takes place in Venice and we are introduced to the rich Jew Shylock Bassanio and Shylock are talking and Bassanio tells Shylock that he wants a loan of three thousand ducats for three months on the personal security of Antonio

o Shylock feels glad because he will be able to bind down Antonio by means of a bond on account of the loan but he tells Bassanio that all the fortunes of Antonio being invested in the merchant ships on the sea it is difficult to depend upon his credit Even under such circumstances Shylock is willing to advance the money on the personal security of Antonio

o Bassanio then invites Shylock to dine with him Shylock says that he is prepared to do anything with the Christians but not eat or drink or pray with them

o While Bassanio and Shylock are talking Antonio appears on the scene Shylock does not seem to take any notice of Antonio but goes on brooding within

(1) SHYLOCK Ho no no no no- my meaning in (Line 15-26)saying he is a good man is to have you understand me that he is sufficient Yet his means are in suppositionhe hath an argosy bound to Tripolis another to the Indies I understand moreover upon the Rialto he hath a third at Mexico a fourth for England and other ventures he hath squanderd abroad Butships are but boards sailors but men there be land-rats and water-rats land-thieves and water-thieves I mean pirates and then there is the peril of waters winds and rocks The man is notwithstanding sufficientmdashthree thousand ducats mdashI think I may take his bond

(a) Who is talking in the beginning of this scene What does Bassanio want from Shylock How does Shylock feel

In the beginning of the scene Bassanio and Shylock are talking to each other Bassanio wants to borrow three thousand ducats from Shylock for three months on the security of Antonio Shylock feels glad at heart that he will get the opportunity of binding Antonio with a bond(b) What risks does Shylock weigh in advancing the money

Shylock says that Antonio has invested all his capital in trading by sea-going ships But the ships are made of wood and the sailors of those ships are ordinary human beings The wood can

himself how he hates Antonio because of his being a Christian because he abuses Shylock in public places Shylock decides that if ever he can get Antonio to his advantage he will teach him a lesson

come to harm and men can commit mistakes and thus the capital invested in ships may be lost Then there are other dangers The goods loaded on the ships can be damaged by rats and thieves which are found both on land and water The ships can also be harmed through sea-storms submerged rocks etc(c) What two important functions does this scene have

The scene has two important functions First it completes the exposition of the two major plot lines of the play Antonio agrees to Shylockrsquos bond ndash three thousand ducats for a pound of flesh and second and more important dramatically this scene introduces Shylock himself In this scene Shakespeare makes it clear at once why Shylock is the most powerful dramatic figure in the play and why so many great actors have regarded this part as one of the most rewarding roles in all Shakespearean dramas(d) Where does this scene take place What kind of treatment has Antonio been giving to Shylock What does Shylock say when Bassanio invites him to dine with him

The action of this scene takes place in Venice Antonio has been in the habit of behaving harshly with Shylock ndash spitting on his beard and footing him like a stranger cur When Bassanio invites Shylock to dine Shylock says that he is prepared to do anything with the Christians but not eat and drink or pray with them

(2) SHYLOCK How like a fawning publican he looks (Line 38-48)I hate him for he is a Christian

But more for that in low simplicity

He lends out money gratis and brings downThe rate of usance here with us in VeniceIf I can catch him once upon the hipI will feed fat the ancient grudge I bear him

He hates our sacred nation and he railsEven there where merchants most do congregateOn me my bargains and my well-won thriftWhich he calls interest Cursed be my tribeIf I forgive him

(a) What is the context in which these words are spoken and what is the idea expressed in it

These remarks are made by Shylock when he sees Antonio coming along after Bassanio told him that the merchant will be his surety for the bond The above mentioned passage reveals Shylockrsquos hatred for Antonio Shylock says that he hates Antonio because he is a Christian and also because he gives loan without taking interest on them thereby bringing down the rate of interest in Venice(b) Explain the meaning of the phrase lsquoa fawning publicanrsquo

The phrase lsquoa fawning publicanrsquo refers to Roman tax collector It is a term of contempt and hatred on the lips of a Jew lsquoFawning Publicansrsquo were Roman tax-gatherers whose ordinary bearings towards the Jews was bullying but whose false pose of lsquohumility and contritionrsquo is touched upon in the parable in New Testament(c ) What light does the above passage throw on the character

of Shylock

The above mentioned speech of Shylock reveals him to be a wicked character having an extreme greed for wealth His intense hatred for Antonio is unjustified He hates Antonio only because he is a Christian and because he lends money without taking any interest on it thereby adversely affecting Shylockrsquos business of lending money on high interest(d) What information do you gather about Antonio from the above given lines

Shylockrsquos statement throws a valuable light on the character of Antonio Antonio appears to be a good Christian and a good human being He helps the people in need by lending them money without charging any interest on it He is a man of simple and good nature This very goodness makes him Shylockrsquos enemy(e) What does Shylock debate within himself and when To whom are the lines mentioned above addressed to

When Bassanio asks the Jew to lend him three thousand ducats on Antoniorsquos surety Shylock begins to debate within himself as to how he should exploit the opportunity of a business deal with his old enemy Antonio

The lines mentioned above are not addressed to anyone The lines are a soliloquy ie a speech made by a character to himself and not meant to be heard by the other characters present

Class XSubject Topic Summary ExecutionEnglish

LiteratureThe Blue Bead 2nd part

Things took a turn for the worst and all of a sudden a crocodile attacked the woman biting on the womanrsquos leg At that moment Sibia got up sprinted grabbed the hay fork and stabbed the crocodile in the eye with all her power Immediately the crocodile let go and went away Sibia saw a small blue bead lying by the river she grabbed it Since she was poor she didnrsquot have necklace Shersquod always wanted one like the other women now she could make one with the blue bead After that she went home and told her mother all about it

Hindi 2nd

Langबड घर की बटी( मशी परमच-)

lsquoबड घर की बटी कहानी का उददशय मधयम वग की घरल समसया को सलझा कर सगदिठत परिरवार म मिमल जलकर परम स रहन का स-श -ना हघर म शानित सथानिपत करन की जिजमम-ारी नारी की होती ह यदि- नारी समझ-ार ह उसम धय और परिरवार क परनित परम ह तो कोई भी घटना परिरवार को निवघदिटत नही कर सकती या कहानी परिरवार को सगदिठत करत हए परम सौहा- स एक दसर की भावनाओ को समझ करउनका सहयोग करत हए जीवन यापन करन की पररणा -ती हमशी परमचदर जी न इस कहानी म सय परिरवार का परनितनिनमिधतव निकया ह यह कहानी बनी माधव सिसह जो गौरी पर क जमी-ार क उनक -ो पतरो की हशरी कठ लाल निबहारीशरीकात का निववाह एकजमी-ार घरान की पतरी आन-ी स हआ थाआन-ी न ख- को ससराल क वातावरण म ढाकतिलया थाएक दि-न आन-ी का अपन -वर लाल निबहारी स झगडा हो जाता ह -ोनो भाई एक दसर स अलग होन की कोकतिशश करत हसभी बह आन-ी न अपन मधर वयवहार स लाल निबहारी को

ldquoइन नतर निपरय गणो को बीए-इनही -ो अकषर पर नयोछावर कर दि-या था इन -ो अकषर न उनक शरीर को निनबल और चहर को कानित ही बना दि-या थाldquo

क) परसतत पकतियो म निकस वयकति क बार म कहा गया ह

ख) इन पकतियो म कौन स नतर निपरय गणो क बार म कहा गया ह

ग) बीए की निडगरी परापत कर लन पर भी उपय वयकति क सवभाव की कया निवशरषता थी

घ) यह नतर निपरय गण निकस वयकति म निवदयमान थ उसक वयकतितव की कया निवशरषता थी

उततर ndashक) परसतत पकति म गौरी पर गाव क जमी-ार

क बड बट शरीकात क बार म कहा गया ह उसन बहत परिरशरम और उ-म क बा- ba की निडगरी परापत की थी अब वह एक -फतर

घर छोडकर जान स रोक कतिलयाइस पर बनी माधव सिसह न कहा निक बड घर की बटी ऐसी ही होती ह जो निबगडा काम बना लती ह अतः शीरषक साथक ह बड घर की बटी आन-ी ह

म कमचारी थाख) भरा हआ चहरा चौडी छाती और डटकर

खाना आदि- एक सबजी ल जवान क गण मान जात ह परत शरीकात न इनही नतर निपरय गणो को अपनी पढाई पर नयोछावर कर दि-या था

ग) बीए की निडगरी परापत कर लन पर भी उपय वयकति(शरी कठ की शारिररिरक तौर पर निनबल और चहर स कानित ही लगत थ इतना ही नही वह मानकतिसक तौर पर भी निपछड हए थ पाशचातय सामाजिजक कथा उस घणा एव पराचीन सभयता का गणगान उनकी निवचारधारा क परमख अग थ

घ) यह नतर निपरय गण गौरीपर गाव क जमी-ार क छोट बट लाल निबहारी सिसह म निवदयमान थ वह सजीलाजवान था और भस का दध शर दध वह सवर उठकर पी जाता था

ldquoयही कारण था निक गाव की लललन आए उनकी निन-क थी कोई कोई तो उह अपना शतर समझन म भी सकोच ना करती थी सवय उनकी पतनी को इस निवरषय म उनस निवरोध थाldquo

क) उपय पकति म इस वयकति क बार म कहा गया ह

ख) गाव की लललन आए उनकी निन-ा कयो निकया करती थी

ग) उनकी पतनी का कया नाम था उनह निकस निवरषय म अपन पनित क निवरa था और कयो

घ) इस कहानी का कया उददशय ह Continue to next helliphelliphellip

Bengali 2nd Language

ফ ফটক না ফটক( কহিতা )

পর) ldquo(ান াধাচেনা ফটপাচেথ পাথচের পাড হিচেয় এক কাঠচোটটা গাছ কহিকহি পাতায় পার ফাটিচেয় াসচেছldquoক) কার দো দেকান কহিতার অং( ) lsquo(ান াধাচেনা ফটপাচেথ পাথচের পাডহিচেয়lsquo চেত কী দোঝাচেনা চেয়চেছ গ) আচো য অংচে(lsquo এক কাঠচোটটা গাছ lsquoচেত কী দোঝাচেনা চেয়চেছ ঘ) ldquoকহিকহি পাতায় পার ফাটিচেয় াসচেছldquo ----- একথার পরকত অথC কী উততর ) ক) আচো য অং(টি পর যাত কহি সভা4 মচোপাধ যাচেয়র দো lsquoফ ফটক না ফটকrsquo কহিতার অং()কহি সভা4 মচোপাধ যায় হিছচেন দেপরচেমর কহি দেপরমচেক নানা ভহিঙগমায় হিতহিন ফটিচেয় তচেচেছন দেপরম মানচের স মচেতC র সঙগী কহিতার কহিতায় এক রb সb হচেয়র দেপরম াগরচেনর কথা চেচেছন (ান অথCাৎ দেযাচেন দেকান রস দেনই দেযাচেন দেকান মহিনতা দেনই অথ তার মধ দেযও দেপরম থাকচেত পাচের একথাই কহি তচে ধরচেত দেচেয়চেছন একটি মানচে4র মচেন দেযাচেন দেকামতার দেকান সথান দেনই পাথচেরর মচেতা হিনরসতার মচেনর মধ দেযও দেয দেপরম আসচেত পাচের দেস কথাই কহি চেচেছনগ)নারীচের যথC দেপরচেমর ছহি এই কহিতায় অকপচেট উচেঠ এচেসচেছ কহি এই কহিতায় কাটচোটটা গাছ কথাটি যার কচেরচেছন নারী দেয দেপরম দেথচেক হিতাহিত এং দেসই দেপরম সঠিক সমচেয় না পাওয়ার ন য দেপরম সমপচেকC হিচেr4 গৈতরী য় দেপরচেমর দেয গৈহি(ষট য মাধযC য সরসতা দেকামত এই সমসতর হিপরীত যথা রbতা শষকতা কচেঠার তা পরভহিত দোঝাচেত এক কাঠচোটটা গাছ কথাটি যার কচেরচেছনঘ) এাচেন এক নারীর যথC দেপরচেমর কথা হিনহিCপত ভাচে চেচেছন কহি অসমচেয় নারীর ীচেন দেপরম দেচেগচেছ এতহিন তার হয় রb কচেঠার হিছ দেপরচেমর অভাচে ঠাৎ দেসই শষক মরভহিমচেত সচের আভাস এচেসচেছ দেপরম দেযন 4Cার স(ীত তাই পরায় মত গাচেছ কহিকহি পাতা গহিচেয় উচেঠচেছ

Biology Chapter - 01Controlling Air Pollution

Today we will discuss how we control air pollution from domestic combustion

Q1Describe any five ways of reducing air pollution from domestic sources bull The number of pollutants in the air is verylarge and we always try to control them byfollowing ways

i) Solar cooker and solar heater It use no fuel reduce damage of environment by fuel use or reducing deforestation It maintains coolness of house It releases very less orno oil gas or grease

ii) Piped natural gas (PNG) It emits very less by products into the atmosphere As it isdistributed through pipe lines so there iscontinuous supply of fuel is possible

iii) Liquefied Petroleum Gas (LPG) It hasa higher heating value LPG doesntcontain sulphur so it burns a lot cleanerenergy sources It releases very less oralmost no fume in air

iv) Electricity based cooking Emission free cooking alternative for urban dwellers causeselimination of adverse health impactsofindoor air pollution It helps to avoid theinconveniences associated with procurement of LPG

v) Biogas It contains 75 methane whichmakes it an excellent fuel It burns without smoke and biogas plant leaves no residue like ash in wood charcoal etc Thus it isaclean fuel

Economics

Factors of Production

Today firstly we would recall the last class for 5 mins and then we would proceed with the further topics of the chapter

The concept meaning of land characteristics of land and importance of land to be repeated for the absentees as well as the students who were there in the class the previous day

Today we will start with the last portion of land before it the meaning of land to be repeated onceAs by now we all know that

Questions1What do you mean by productivity of landAnswer By productivity of land we mean the capacity of a piece of land to produce a crop

Thus it refers to the average output per unit of landSay per acre per hectare etc= (OutputArea of land)

2 What are the factors influencing the productivity of landAnswer

Natural factors Productivity of land is largely determined by the natural

Land is defined to include not only the surface of the earth but also all other free gifts of nature(for example mineral resources forest resources and indeed anything that helps us to carry out the production of goods and services but is provided by nature free of cost)

We will move on to the last portion of land by discussing Productivity of Land

By productivity of land we mean the capacity of a piece of land to produce a crop

Thus it refers to the average output per unit of land

Say per acre per hectare etc= (OutputArea of land)

With this we shall proceed further with the main factors that determine the productivity of land

Natural factors Human factors Improvements on land Location of land Organisation Ownership of land Availability of capital Proper use of land State help

Note economic development of a country depends upon the quality of its land If the land is fertile it will quicken the pace of development of the country

qualities of land such as fertility etc

Human factors Land cannot produce anything by itself Man has to apply labour on it to produce for himself So productivity of land depends on the knowledge and skills of workers

Improvements on land production of land is affected by land development measures like provision of well or tubewell irrigation proper drainage

State help The government of a country especially less developed country can play a vital role in improving the agricultural productivity by providing better irrigation facilities

Organisation Productivity of land also fdepends upon the way how the factors of production like labour and capital are organised

In order to increase productivity trained workers modern implements scientific methods good seeds are all essential

3 lsquoImproved technology affects the productivity of landrsquo Explain this statement with the help of suitable example Answer Use of improved technology raises the productivity of land Example By using HYV seeds chemical manures and modern machines per hectare output increases

Physics Force (Summary)

Question Write the expression for the moment of force about a given axisSolutionsThe expression for the moment of force is given byMoment of force about a given axis = Force times perpendicular distance of force from the axis of rotationQuestion What do you understand by the clockwise and anticlockwise moment of force When is it taken positiveSolutionsIf the effect on the body is to turn it anticlockwise moment of force is called the anticlockwise moment and it is taken as positive while if the effect on the

body is to turn it clockwise moment of force is called the clockwise moment and it is taken as negative

Math Topic Commercial Mathematics

Chapter Goods and services Tax

Study item Some solved sums from exercise ndash 1 A retailer buys a TV from a wholesaler for Rs 40000 He marks the price of the TV 15 above his cost price sells it to the consumer at 5 discount on the marked price If the sales are intra ndash state and the rate of GST is 12 find

(i) The marked price of the TV(ii) The amount which the consumer pays for the TV(iii) The amount of tax (under GST) paid by the retailer to the central

Government(iv) The amount of tax (under GST) received by the State Government

Solution As the sales are intra- state sale and the rate of GST 12 So GST comprises of 6 CGST and 6 SGSTTherefore a retailer buys a TV from a wholesaler for Rs 40000Therefore the amount of GST collected wholesaler from the retailer or paid by retailer to wholesalerCGST = 6 of Rs 40000 = Rs(6100 times40000) =Rs 2400SGST = 6 of Rs 40000 = Rs (6100 times 40000) =Rs 2400Therefore wholesaler will pay Rs 2400 as CGST and Rs 2400 as SGSTTherefore amount of input GST of retailer Input CGST = Rs 2400 and input SGST = Rs 2400Again the retailer marks the price of the TV 15 above his cost price(i) The marked price of the TV

= Rs 40000 + Rs 40000times15= Rs 40000 + Rs 40000times 15100= Rs 40000 + Rs 6000Rs 46000But the retailer sells it to consumer at 5 discount on the marked priceCost price after discount = Rs 46000 ndashRs46000times 5100 =Rs 46000 ndashRs 2300= Rs 43700Therefore the amount of GST collected retailer from consumer or paid by consumer to retailerCGST = 6 of Rs 43700 =Rs ( 6100 times43700)Rs 2622SGST = 6 of Rs 43700 = Rs (6100 times 43700) =Rs 2622Amount of the output GST of retailer Output CGST = Rs 2622 and output SGST = Rs 2622

(ii) The amount which the consumer pays for the TV= cost price of TV to consumer + CGST paid by consumer + SGST paid by consumer= Rs 43700 + Rs 2622 + Rs 2622= Rs 48944

(iii) The amount of tax (under GST ) paid by the retailer to the central Government=CGST paid by retailer = output CGST ndash input CGST=Rs 2622 ndash Rs 2400=Rs 222

(iv) The amount of tax ( under GST ) received by the State Government = SGST paid by wholesaler + SGST paid by retailer= Rs 2400 + output SGST ndash input SGST=Rs 2400 + Rs 2622 ndash Rs 2400=Rs 2400 + Rs 222= Rs 2622

Commercial studies

Stakeholders Today I am going to give some revision questions from the previous study material

Questions1) State the two expectations of

employees from a business concern2) Give two distinctions between

stakeholder and shareholder3) Give two difference between

internal stakeholders and external stakeholders

4) Give two expectations of suppliers from a business organisation

5) Who is a stakeholder in commercial organisations

Chemistry Periodic Table

Merits of Mendeleevrsquos Periodic law are as follows - 1He grouped the elements on the basis of atomic mass 2 He left gaps for undiscovered elements like Gallium Scandium germanium Also he left a full group vacant for undiscovered inert gases 3 He could predict proportions of several elements on basis of their position in periodic table like Ga Sc etc 4He could predict errors in atomic weights of some elements like gold platinum etc

Anomalies in Mendeleevrsquos Periodic law are as follows - 1 Position of isotopes could not be explained 2 Wrong order of atomic masses could not be explained

For example- as Arnur atomic mass 40 come first and K with low atomic mass (30) should come later but k should be placed first

According to Bohrrsquos Modern Periodic table properties of elements are periodic functions of their atomic numbers

So when elements are arranged according to increasing atomic numbers there is periodicity in electronic configuration that leads to periodicity in their chemical properties

It consists of horizontal rows (Periods) Vertical column (Groups)

There are 7 period and 12 groups in this long form of periodic table

Ist period has 2 elements IInd period has 8 elements IIIrd period has 8 elements IVth period has 18 elements Vth period has 18 elements VIth period has 32 elements VIIth period hs rest of elements

Note - The number of valence electrons in atom of elements decides which elements will be first in period and which will be last

In group- 1 to 2 gp and 13 to 17 contain normal elements 3 to 12gp ndash transition elements 57 to 71 - lanthanides 89 to 103 - Actinides

Left hand side ndash metals Right hand side ndash nonmetals

Note- Hydrogen element has been placed at top of Ist group Electronic configuration of H is similar to alkali metal as both have 1 valence electron

V electron of gp I element -- 1 V electron of gp 2 element -- 2 V electron of gp 13 element -- 3 V electron of gp 14 element -- 4 V electron of gp 15 element -- 5 V electron of gp 16 element --6 V electron of gp 17 element -- 7 V electron of gp 18 element -- 8

English 1 Transformation of sentences

Sentences A sentence is a group of words which makes complete sense

Exercise 2Change the following sentences from

a Assertive sentencesb Imperative sentencesc Interrogative sentencesd Exclamatory sentences

Sentences can be changed from one grammatical form to another without changing the meaning of the sentence This is known as transformation of sentences

assertive to interrogative1 Nobody would like to be a fool

Who would like to be a fool2 Their glory can never fade

When can the glory fade3 Nobody can control the wind

Who can control the wind4 It matters little if I die

What though I die5 No man can serve two masters

Can any man serve two masters

Exercise 3Interchange of assertive and Exclamatory sentences

1 She leads the most unhappy lifeWhat an unhappy life she leads

2 This is indeed an interesting bookWhat an interesting book this

3 He is a very great manWhat a great man he is

4 It is a very lame excuseWhat a lame excuse

5 It is sad that she died so youngAlas she died so young

Class XISubject Topic Summary Execution

Hindi 2nd lang

पतर परम(परमचदर) पतर परम कहानी म एक निपता की इचछाओ का वणन निकया गया ह अपन बड पतर परभ -ास स निपता चतनय -ास का निवशरष परम था निपता को उसक जनम स ही बडी-बडी आशाए थी उसम दसर बट कतिशव-ास की अपकषा स- उतसाह की मातरा अमिधक थी वह उस इगलड भजकर बरिरसटर बनाना चाहत थभागय का खल भी बडा निनराला ह बीए की परीकषा क बा- वह बीमार पड गया डॉकटरो न भी जवाब - दि-या थाचतन -ास जी बहत ही कजस थ बवजह पस खच करना नही चाहत थ अगर गारटी मिमलती तो शाय- पस खच भी कर -त परत गारटी नही थी परिरणाम सवरप उनक बट का -हात हो गयाजब बट को समशान ल जा रह थ तो वहा काफी शोर गान बजान हो रह थ पछन पर पता चला निक निकसी निपता निपछल तीन साल स निबमार था और उसक ईलाज म रपया पानी की तरह बहाया पर ठीक नही हए परत उसक बट को तनिनक भी अफसोस नही था उसका कहना था उसन कोकतिशश तो कीयह -खकर चतनय-ास जी को आतम निगलानी हईतभी स उनका म परिरवतन हआ और बट का भोज काफी धमधाम स निकयाऔर वहइस पशचाताप की आग म जलत रह औला- स बढकर पसा नही होता ह इस बात को समझन म उनह काफी व लग गया

hellipContinue to next

BENGALI(2ND LANGUAGE)

পরথমঅধযায়-ঠাকরারীনদরনাথঠাকর

নয়ন দোচের হিমাচেররা া নাচেমই হিযাত হিছচেন ায়ানার উাররণ সবরপ নয়ন দোচের ারা হিা (াচেকর হিা হিচেতন এছাাও দেকান উৎস উপচেb রাহিতর দেক হিন করার উচেfচে(য তারা সযC হিকরচেরণ রনয পরীপ জবাহিচেয় তাচেত রপার হির 4Cরণ করচেতন ঠাকরা এই নয়ন দো হিমারচের দে(4 ং(ধর হিছচেন হিমাররা ায়ানার ষটানত পর(Cন কচের তারা হিনঃসব এই হিমাহিরর দে(4 ং(ধর গৈকাস নদর রায়চেৌধরী গৈকাস া নয়ন দোচের সমসত সমপহিতত ঋচেরণর াচেয় হিহিx কচের অহি(ষট যা আচেছ তাচেত হিপত

ইার হিপতার মতয ইচে পর নয়নচোচের ায়ানার দেগাটা কতক অসাধাররণ শরাদধ (াহিনতচেত অহিনতম ীহিপত পরকা( কহিরয়া ঠাৎ হিনহিয়া দেগ- ক) কার দো দেকান গচেলপর অং() কতা দেক ইার চেত কাচেক দোঝাচেনা চেয়চেছ গ) পরসঙগ কী কতার কতয পহিরসফট কচেরা

পরচে4র যাহিত রbা করা সমভ নয় তাই হিতহিন পতরচেক হিনচেয় ককাতায় সাস শর কচেরন গলপ কথচেকর আহিথCক অসথা নয়ন দোচের হিমাচের দেথচেক সমপরণC আাা কথচেকর হিপতা হিনচের দেষটায় অথC উপাCন করচেতন া উপাহিধ াচেভর নয তার াসা হিছনা আর দেসই কারচেরণ কথক তার একমাতর উততরাহিধকার চেয় তার হিপতার পরহিত কতজঞ কথক দো পা হি(চেচেছন হিনচের পরারণ ও মান রbার নয উপচেযাগী অথC হিনা দেষটায় পরাপত চেয়চেছন- এটাই তার কাচেছ পরম দেগৌরচের হি4য় চে মচেন কচেরন কাররণ (নয ভাণডাচের গৈপতক ায়ানার উজজব ইহিতাস অচেপbা দোার হিসeচেকর মচেধয গৈপতক দেকামপাহিনর কাগ তার কাচেছ অচেনক দেহি( মযান

TO BE CONTINUED

উ- ক) আচোয অং(টি রীনদরনাথ ঠাকচেরর দো ঠাকরা গচেলপর অং() কতা চেন আচোয গচেলপর গলপ কথকইার চেত নয়ন দোচের হিমাহিরর দে(4 ং(ধর গৈকাস ার কথা া চেয়চেছ গৈকাস া নয়ন দোচের সমসত সমপহিতত ঋচেরণর াচেয় হিহিx কচের অহি(ষট যা আচেছ তাচেত হিপত পরচে4র যাহিত রbা করা সমভ নয় তাই হিতহিন পতরচেক হিনচেয় ককাতায় সাস শর কচেরনগ) গৈকাস ার হিপতার মতযর পর নয়ন দোচের হিমাহিরর অহিসততব হিপত য় কচেয়কটা উৎস ও শরাদধ- (াহিনতচেত হিমাহিরর দে(4 কহিটক যয় চেয় হিগচেয় এচেক াচের দে(4 চেয় যায় তন তাচের গC করার মত আর হিকছই হিছ না-দেসই পরসচেঙগ এই উহিকত নয়নচোচের হিমাচেররা া নাচেমই হিযাত হিছচেন ায়ানার উাররণ সবরপ নয়নচোচের ারা হিা (াচেকর হিা হিচেতন এছাাও দেকান উৎস উপচেb রাহিতরচেক হিন করচেত হিগচেয় তারা সযC হিকরচেরণর নয পরীপ জবাহিচেয় তাচেত রপার হির 4Cরণ করচেতন তাই দেসকাচের ায়ানা দেহি(হিন সথায়ী চেত পারত না হিহিভনন উৎস শরাদধ- (াহিনতচেত সাধযা হিতহিরকত র করার নয হিমাহির হিহিকচেয় দেযত হ হিতC কা হিহি(ষট পরীচেপর দেত দেযমন অলপকাচের মচেধয হিনঃচে(4 চেয় যায়-নয়নচোচের হিমারচের অসথা তাই চেয়হিছ এই কারচেরণই কথক নয়নচোচের হিমারচের গা ভরা আমবর সয করচেত পারতনা

Physics Dimensional Analysis (Summary)

Q Find the dimensions of consts ab in relation

p=(bminusxlowastx)at

where p is the power x is the distance and t is time

Ans From principle of homogeneity dimension of b x2 are same Dim of b = dim of x2 = [L2] = [ML2T0]Dim of a = dim of ( b- x2)dim of (pt) = [M0L2T0][ML2T-2] [T-1] [T] = [M-1L0T2]

Chemistry Atomic Structure Drawbacks of Rutherfordrsquos model of

atom a According to Rutherfordrsquos model of atom electrons which are negativelycharged particles revolve around the nucleus in fixed orbits Thusb theelectrons undergo acceleration According to electromagnetic theory of Maxwell a charged particle undergoing acceleration should emitelectromagnetic radiation Thus an electron in an orbit should emitradiation Thus the orbit should shrink But this does not happenc The model does not give any information about how electrons aredistributed around nucleus and what are energies of these electrons Isotopes These are the atoms of the same

Properties of electromagnetic radiationsa Oscillating electric and magnetic field are produced by oscillating charged particles These fields are perpendicular to each other and both areperpendicular to the direction of propagation of the waveb They do not need a medium to travel That means they can even travel invacuum

Characteristics of electromagnetic radiationsa Wavelength It may be defined as the distance between two neighbouring crests or troughs of

element having the same atomicnumber but different mass numbere g 1H11H21H3

Isobars Isobars are the atoms of different elements having the same massnumber but different atomic numbere g 18Ar40 20Ca40

Isoelectronic species These are those species which have the same numberof electrons

Electromagnetic radiationsThe radiations which are associated withelectrical and magnetic fields are called electromagnetic radiations When anelectrically charged particle moves under acceleration alternating electricaland magnetic fields are produced and transmitted These fields aretransmitted in the form of waves These waves are called electromagneticwaves or electromagnetic radiations

wave as shown It is denoted by λb Frequency (ν) It may be defined as the number of waves which passthrough a particular point in one secondc Velocity (v) It is defined as the distance travelled by a wave in onesecond In vacuum all types of electromagnetic radiations travel with thesame velocity Its value is 3 times10 8m sec-1 It is denoted by v

d Wave number Wave number is defined as the number of wavelengths per unit lengthVelocity = frequency timeswavelength c = νλ

Plancks Quantum Theory- o The radiant energy is emitted or absorbed not continuously but discontinuously in the form of small discrete packets of energy called lsquoquantumrsquo In case of light the quantum of energy is called a lsquophotonrsquoo The energy of each quantum is directly proportional to the frequency of the radiation ie E α υ or E= hυ where h= Planckrsquos constant = 6626 x 10-27 Js o Energy is always emitted or absorbed as integral multiple of this uantum E=nhυ Where n=1234Black body An ideal body which emits and absorbs all frequencies is calleda black body The radiation emitted by such a body is called black body radiation

Photoelectric effectThe phenomenon of ejection of electrons from thesurface of metal when light of suitable frequency strikes it is calledphotoelectric effect The ejected electrons are called photoelectrons

Biology Chapter - 02Systematics and Five Kingdoms

Scientists divide the whole living organisms into two kingdom first and ultimately by five kingdom at last

In the earlier systems of classifications organisms are divided into kingdom plantaeand kingdom animalia on the of presenceof cell wall their modes of nutrition and movements

Some problem arise like fungi share manycharacteristic withplant despite their heterotrophic nutrition bacteria protozoa areunicellular present in both kingdom Toovercome this third kingdom Protista isintroduced which include

unicellularorganisms But there is also another

problem Allunicellular organisms are not similar kind The cellular structure of prokaryotes is verydifferent from that of other organismsEukaryotes possess a true nucleus and allcell organelles that are not present inprokaryotes So the fourth kingdom Monerais introduced which include unicellular prokaryotes (bacteriaamp blue green algae)

bull Still some problem arise in kingdomplantae

So in 1969 R H Whittakar proposedanew five kingdom System of classification

i) Kingdom Monera - unicellular prokaryotes

ii) kingdom Protista - unicellular eukaryotes

iii) Kingdom Fungi - uni or multicellular fungi with cell wall but without chlorophyll

iv) Kingdom Plantae - Multicellular Plants

v) Kingdom Animalia - Multicellular Animals

EVS Chapter 1 ndash Modes of Existence

An agricultural society

An agricultural society also known as an agrarian society is a society that constructs social order around a reliance upon farming More than half the people living in that society make their living by farming

People in an agricultural society generally lead a more settled lifestyle than those in nomadic hunter-gatherer or semi-nomadic pastoral societies because they live permanently near the land that is farmed Agricultural settlements tend to develop in areas of convenience near bodies of water which is used for both crops and transportation or along trade routes Not everyone in an agricultural society is a farmer Some people make a living trading or making and selling goods such as tools used for farming

Another way to define an agrarian society is to see the total amount of production in a nation In an agrarian society cultivating the land is the main source of wealth Such a society can recognize other means of subsistence and work habits but emphasizes the importance of agriculture and livestock Agrarian societies have existed in various parts of the world for 10000 years and continue to exist today They have been the most common form of socio-economic organization for most of recorded human history

Q) Write the features of agricultural society

Ans - Structure and Features of Agrarian Society1 Occupational Structure

An agrarian society is generally associated with the domestication of plants and animals The domestication of plants means farming and that of animals means herding Often there is mixture of farming and the use of such domesticated animals as cow goat and sheep

2 Forms of Land Ownership in Agrarian SocietiesGenerally there are landlords supervisory farmers cultivators and share croppers The landholders own the land but do not work on it They let it out for sharecropping The supervisory farmers are those who live by having their land cultivated by hired labourers The cultivators cultivate the land for themselvesThe share-croppers are those who live by tilling other peoplersquos land or a crop-sharing basis The artisans own their means of production and produce by their own labour in their homesteads

3 Village Community System An agrarian society is highlighted by

the institution of village community system The agrarian economy made fixed dwelling houses necessary Living close together for protection and co-operation and living nearer to the land gave birth to agricultural villages The village is not only the residential place of farmers it is also the social integrator

4 Minimal Division of Labour Another structural feature of agrarian society is a minimal division of labour Except for the basic division founded on age and sex differences there are few specialized roles There is only one predominant type of occupation ie domestication of plants and animals For all the people the environment physical as well as social is the same

5 Role of Family The farm family is of the patriarchal type the father is the final arbiter in most of the familyrsquos major decisions The life of ail men and women is merged in family life Since there are not many special organizations family is the only organisation to perform the tasks of aid and protection

6 Sense of Unity The members of an agrarian society exhibit a strong in-group feeling Since the whole of their social lives is wrapped up in a society which is physically economically and socially homogenous they are inclined to view the entire outside world as an out group

7 Informal Social Control An agrarian society is regionally divided into villages In a village community the force of traditional mores is more dominant than in the urban community In the village everybody is known to everybody The members in a village community help each other and share the joy and sorrows of each other Crime in an agrarian society is rare

8 Simplicity and Uniformity Life of the people in an agrarian society is marked by simplicity and uniformity Their main occupation is agriculture which largely depends upon the vagaries of nature An agrarian society is a religious society

Math Compound angles Compound angles The algebraic sum of two or more angles is called a compound angle If A B C be three angles then A+B B+C C+A A-B B-C A-C A+B-C etc are compound angles In this chapter we shall discuss the trigonometrical ratios of compound angles Theorem 1 If A B and A+B are all pisitive acute angles theni) sin( A+B) = sin A cos B + cosA sinBii) cos(A+B) = cosA cosB- sinA sinBTheorem 2If A and B are positive acute angles and AgtB theni) sin(A-B) = sin A cosB- cos A sinBii) cos(A-B) = cos A cos B+ sin A sin BTo prove that i) sin(A+B) sin (A-B) = sin2 A - sin2 B = cos2 B- cos2 A

Example 1 Prove that tan70deg=2tan50deg+tan20degSolutiontan70deg = tan(50deg + 20deg)Or tan70deg=(tan 50deg+tan 20deg)(1-tan50degtan20deg) or tan70deg (1 ndash tan 50deg tan20deg) = tan50deg+tan20degor tan70deg= tan70deg tan50deg tan20deg+ tan50deg + tan20deg = cot20deg tan50deg tan20deg + tan50deg + tan20deg = 2 tan50deg+ tan20degExample 2 If A + B = 45deg show that (1 + tanA) (1 + tanB) = 2Solutiontan(A + B) =( tan A + tan B) (1 - tan

ii) cos(A+B) Cos(A-B) = cos2 A- sin2 B = cos2 B -sin2 AProof i) LHS= sin(A+B)sin(AminusB) [Recall sin(αminusβ)=sinαcosβminuscosαsinβ And sin(α+β)=sinαcosβ+cosαsinβ]= (sinAcosB+cosAsinB)times(sinAcosBminuscosAsinB)= sin2Acos2Bminuscos2Asin2B [Recall sin2α+cos2α=1 From above we can then assume correctly that sin2α=1minuscos2α AND cos2α=1minussin2α] = sin2A(1minussin2B)minussin2B(1minussin2A) = sin2Aminussin2Asin2Bminussin2B+sin2Asin2B = sin2Aminussin2B= 1-cos2A-(1-cos2B) = cos2 B- cos2 A = RHSii)LHS= cos (A+B) cos(A-B) [ cos(A+B) = cos AcosB- sinAsinBCos(A-B) = cosAcosB+ sinAsinB]= cos2 A Cos2 B- sin2 A Sin2 B= cos2 A( 1-sin2 B) - (1- cos2 A) sin2 B= cos2 A- cos2 A sin2 B- sin2 B+ cos2 A sin2 B=cos2 A- sin2 B=1- sin2 A-(1-cos2 B) = cos2 B- sin2 A= RHSTangent formulae for compound anglesi)tan (A + B) = tan A + tan B1-tan A tan Bii)tan (A ndash B) = tan A-tan B1+tan A tan Biii) cot (A + B) = cot Acot B-1cot A+cot B(viii) cot (A ndash B) = cot Acot B+1cot B-cot A

A tan B) Or 1= (tan A+ tanB) (1-tan A tanB) Or tanA + tanB + tanA tanB + 1 = 1 + 1Or tanA (1 + tanB) + (1 + tanB) = 2Or (1 + tanA) (1 + tanB) = 2Example 3 Find the value of sin 15degSolution sin 15deg= sin(45deg-30deg) = sin45degcos 30deg- cos45degsin30deg =(1radic2) (radic32) -(1radic2) (12) = (radic3-1) 2radic2Example 4 If sin A = 1 radic10 and sin B = 1 radic5 where A and B are positive acute angles then what is A + B SolutionWe know that sin (A + B) = sin A cos B + cos A sin B= [1 radic10] [radic(1 minus 1 5)] + [1 radic5] radic(1 minus 1 10)= [1 radic10] [radic4 5] + [1 radic5] [radic9 10]= [1 radic50] times (2 + 3)= 5 radic50 = 1 radic2

sin (A + B) = sin π 4rArrHence A + B = π 4Example 5 If A + B = 225o then find [cot A] [1 + cotA] times [cot B] [1 + cot B]Solution[cot A] [1 + cotA] times [cot B] [1 + cot B] = 1 [(1 + tan A) times (1 + tan B)]=1 [tan A + tan B + 1 + tan A tan B] [ tan (A + B) = tan225o]∵

tan A + tan B = 1minus tan A tan BrArr= 1 [1 minus tan A tan B + 1 + tan A tan B]= 1 2

COMMERCE

CLASSIFICTION OF HUMAN ACTIVITIES-ECONOMIC AND NON-ECONOMIC

Firstly we shall recall the previous class for 5 mins especially for the absentees and for also the rest of the students who were there

Today at first we briefly discuss the earlier portions of the chapter

1Business-It includes all those economic activities which are concerned with production and exchange of goods and services with the object of earning profit Example A factory shop beauty parlour also business enterprises

2Profession ndashThe term profession means an occupation which involves application of specialized knowledge and skills to earn a living For Example Chartered Accountancy medicine law tax consultancy are example of professions

Questions1What are the main features of ProfessionAnswer The main features of a profession are as follows a Specialised body of knowledge-Every profession has a specialised and systematised body of knowledge b Restricted entry- Entry to a profession is allowed only to those who have completed the prescribed education and have the specialised examination c Formal education and training ndashA formal education and training is given to the person who wants to acquire the professional

3Employment-Employment mean an economic activity where people work for others in exchange for some remuneration (salary)The persons who work for others are called lsquoemployeesrsquo The persons or organizations which engage others to work for them are called lsquoemployersrsquoEg A doctor working in a hospital is employment as he is working for a salaryA lawyer may serve as a law officer in a bank

With this we shall proceed with the features of both Profession amp Employment

The main features of a profession are as follow

a Specialised body of knowledge b Restricted entry c Formal education and training d Professional association e Service motive f Code of contact

The main features of an employment are as follows

a In employment a person works for others called employer

b An employee provides personal service

c There is a service agreement or contract between the employee and the employer

d The employee has to obey the order of the employer

e No capital investment is made by the employer

Various examples of Employment are as follows

aA teacher teaching in a school or collegeb An engineer employed in Municipal Corporation of DelhicAn accountant working in the accounts department of a companydA doctor working in a hospital

Note In all the above examples of employment the individual who is involved in each example is working as an employee for a salary under an employer

qualification(MBBSCALLB)d Service motive ndashProfessionals are expected to emphasis service more on their clients rather than economic gain f Code of Conduct-The activities of professionals are regulated by a code of conduct

2 What are the main features of EmploymentAnswer The main features of an employment are as followsa In employment a person works for others called employerb An employee provides personal servicec There is a service agreement or contract between the employee and the employerd The employee has to obey the order of the employere No capital investment is made by the employer

3 Give various Professions and their respective Association are given below

Professions

Professional

Professional association

Medical profession

Doctor Medical Council of India

Law profession

Lawyers Bar Council of India

Accounting Profession

Chartered

The Institute of Chartered Accounts of India( ICAI)

Engineerin Engineers The

g Profession

institute of Engineers (India)

Accounts Basic accounting terms

Today we will give you some questions from the previous study material

Questions6) Define accounting7) What do you mean by debit

and credit8) Explain the types of account9) Define the following terms

a) Assetsb) Capitalc) Purchased) Debtorse) Transactions

10) Name the types of accounts given below

a) Krishnas accountb) Machinery accountc) Royalty accountd) Salary accounte) Furniture accountf) Audit fee account

Economics Basic Economic ConceptsSub topic

UTILITY

Before starting todayrsquos class we shall recall the last class which was about UTILITY AND THE FEATURES OF UTILITY

Now we shall proceed with the further topics of the chapter

Todayrsquos topic from the chapter lsquo Basic Economic Conceptsrsquo will be TOTAL UTILITY amp MARGINAL UTILITYNow let us quickly revise the concept of utility with an example ie goods and services are designed because they have an ability to satisfy human wantsThis feature of being able to satisfy human wants is termed as utility For example we derive utility from WiFi services as it gives us satisfaction by connecting us to our friends and family through social media here consumers derive utility from WiFi services

From the above concept we shall start with todayrsquos topicEconomists have defined TOTAL UTILITY (TU) as the total satisfaction obtained by consuming a given total amount of a good and serviceFor example the total satisfaction obtained from eating 10 mangoes is the total utility of 10 mangoes

MARGINAL UTILITY (MU) is the additional satisfaction derived from each additional unit

Questions1 What is Total Utility (TU)

Answer Total Utility (TU) is the

aggregate of the utility that a consumer derives from the consumption of a certain amount of a commodityTU=MU1+MU2++MUn

2 What is Marginal UtilityAnswer

Marginal Utility (MU) is the additional made to the total utility as consumption is increased by one more unit of the commodityMU= TUn ndashTUn-1

NoteOften economists tend to

subdivide utility into an imaginary unit called UTIL

consumed In this casethe utility obtained from each mango as it is consumed as the MU of that mango It is also defined as the addition made to the total utility when an additional unit is consumed Often economists tend to subdivide utility into an imaginary unit called UTIL

Note As a consumer increases the consumption of a good over period of time the total utility or total satisfaction derived from it increases to appoint and thereafter it decreasesHowever as the consumer keeps on consuming the good the marginal utility or the additional utility derived from it decreases

SubjectBusiness studies

Topic

BUSINESSENVIRONMENT

Summary

Now quickly let us revise the earlier points that we have already done in the last class and let us proceed with the other topics that are there in the chapter

Firstly we will recall the internal and external factors of micro environment and then we shall proceed in details

Meaning and list of internal and external factors

aInternal factorsInternal factors refer to all the factors existing within a business firm The internal factors are considered controllable because the enterprise has control over these factorsFor an example a company can alter its organization structure policies programmes employees physical facilities and marketing mix to suit the changes in the environmentList of internal factors areCorporate culture mission and objectives top management organizations structure company image and brand equity company resources

b External factorsExternal factors refer to those individual and groups and agencies with which a particular business organization comes into direct and frequent contact in the course of its functioningThese individuals and groups are known as STAKEHOLDERS because they have a stake (financial interest ) in the working and performance of the particular business List of external forces (stakeholders)Customers competitors investors suppliersmiddlemen (marketing intermediaries)

Execution 1 What do you mean by internal

factors in micro environmentAnswerInternal factors refer to all the factors existing within a business firm The internal factors are considered controllable because the enterprise has control over these factorsFor an example a company can alter its organization structure policies programmes employees physical facilities and marketing mix to suit the changes in the environment

2 What do you mean by external factors in micro environment

AnswerExternal factors refer to those individual and groups and agencies with which a particular business organization comes into direct and frequent contact in the course of its functioningThese individuals and groups are known as STAKEHOLDERS because they have a stake (financial interest) in the working and performance of the particular business

3Who are stakeholdersSTAKEHOLDERS are individuals and groups who have a stake (financial interest ) in the working and performance of the particular business 4Discuss the internal factors in briefa Corporate CultureThe values beliefs and attitudes of the founders and top management of the company exercise

financers publics

customers

suppliersfinancers

competitors

middlemen

publics

Fig STAKEHOLDERS OF A COMPANY

Apart from micro environment the other main dimension of business environment isMacro environment Macro environment refers to the general environment or remote environment within which a business firm and forces in its micro environment operateA company does not directly or regularly interact with the micro environmentTherefore macro environment is also known as indirect action EnvironmentThe macro environment forces are less controllable than the micro forces

Macro environment consists of the following components

POLITICAL AND LEGAL ENVIRONMENT

ECONOMIC SOCIAL AND ENVIRONMENT

CULTURAL

ENVIRONMENT

TECHNOLOGICAL ENVIRONMENT

a strong influence on what the cmpaany stands for how it does things and what it considers importantbMission and objectivesThe business philosophy and purpose of a comoany guide it prioritiesbusiness strategiesproduct market scope and development scope

cTop management structurethe composition of board of directors the degree of professionalization of management and the organizational structure of a company have important bearing on its business decisions

dPower structureThe internal power relationship between the board of directors and the chief executive is an important factor

eCompany image and brand equityThe image and brand equity of the company play a significant role in raising finance forming alliance choosing dealers and suppliers launching new products entering foreign markets

5 What is Macro environmentAnswerMacro environment refers to the general environment or remote environment within which a business firm and forces in its micro environment operateA company does not directly or regularly interact with the micro environmentTherefore macro environment is also known as indirect action EnvironmentThe macro environment forces are less controllable than the micro forces 6 What are the components of macro environmenta Political and legal environmentb Economic environmentc Social and cultural environmentd Technological environment

BUSINESS FIRM

Fig COMPONENTS OF MACRO ENVIRONMENTPolitical science

Introduction to political science

Comparative politics and itrsquos scope Comparative politics is the second major dimension of political scienceIt is also a very vast area of study and a very large number of political scientists even treat it as an autonomous area of study within the board ambit of political scienceScope of comparative politics-

1 All political structures -Comparative politics includes the study of all structures formalnon formal governmental and extra governmental which are directly or indirectly involved in politics in all the countries of the world

2 Functional studies- Comparative politics seeks to study politics less from the point of view of the legal institutions in terms of their powers and move from the point of view of their functions which constitute the political process and their actual Operation in the environment

3 Study of political behaviour- Another important part of its scope is the study of the actual behaviour of the people in the process of politics

4 Study of similarities and differences- comparative politics also undertakesan analysis of the similarities and differences among political process and functions

5 Study of all political systems -comparative politics seeks to analyse the actual behaviour and performance of all political systems western as well as non western

6 Study of the environment and infrastructure of politics-The study of politics demands a study of the psychological sociological economic and anthropological environment in fact the social environment as a whole in which each political system operates

7 Study of political culture- political culture is composed of attitudesbeliefs emotions and values of a society that relate to the political system or politics

8 Study of political participation- Political participation is a universal processThe only difference is that while in some states it is limited in others it is wider

9 Study of political process- political

Answer the following questions-

What is comparative politics

What are the scope of comparative politics

Homework- learn

processes like decision makingpolicy making judicial process leadership recruitment process and others are always at work in all political systems

The scope of comparative politics is very comprehensive It includes everything that falls within the area of political activity and political process

History CAMBRIDGE VIEW ABOUT

THE PARTITION

AND REFUTATION

OF CAMBRIDGE

VIEW

Cambridge view about the Partition The Cambridge school of historians have interpreted that opposition to partition scheme was made entirely by the elitist groups They hold the view that Lord Curzon planned to partition the Bengal for administrative purposeREFUTATION OFCAMBRIDGE VIEW The Rationalist historians have rejected the interpretations of the Cambridge School of historians on various grounds

1 QUESTION State different views of historians regarding Partition of Bengal

ANSWER Cambridge historians believed that Lord Curzon partitioned Bengal for administrative reasons only and not for the political motive The Middle class elitist group protested because of their petty interest The Hindu zamindars protested as they have to spend more money for managing their estatesThe lawyers of Calcutta High court feared to lose their clientBut according to the nationalist Historians was-

2- The ultimate object of Lord Curzon was to crush the unity of Bengal politicians

3- If Bengal becomes a separate province Bengali speaking 16 million people of western part would become minority under Hindi speaking people of Bihar and Oriya speaking people of Orissa

4- The bureaucrats expected that the protest movement would die down quickly

5- Lord Curzon used the Muslim community in his political game

6- Idealism had great contribution in the protest against partition

7- The people of the every section of society were affected by the partition of Bengal

Computer Science

Numbers Convertion of dcimal number to octal numberThe decimal numeral system is the standard system for denoting integer and non-integer numbers It is the extension to non-integer numbers of the Hindu-Arabic numeral system For writing numbers the decimal system uses ten decimal digits a decimal mark and for negative numbers a minus sign - The decimal digits are 0 1 2 3 4 5 6 7 8 9 the decimal separator is the dot in many countries

The octal numeral system or oct for short is the base-8 number system and uses the digits 0 to 7 Octal is sometimes used in computing instead of hexadecimal perhaps most often in modern times in conjunction with file

permissions under Unix systems It has the advantage of not requiring any extra symbols as digits It is also used for digital displays

Follow these steps to convert a decimal number into octal form

1 Divide the decimal number by 82 Get the integer quotient for the next iteration (if the number will not divide equally by 8 then round down the

result to the nearest whole number)3 Keep a note of the remainder it should be between 0 and 74 Repeat the steps until the quotient is equal to 05 Write out all the remainders from bottom to top This is the solution

For example if the given decimal number is 8453

Division Quotient Remainder

8453 8 1056 5

1056 8 132 0

132 8 16 4

16 8 2 0

2 8 0 2

Then the octal solution is 20405

Subject Eng Literature (The Tempest ndash William Shakespeare) Topic Act I Scene 1 Lines 33 to 67 (End of scene) Date 16th April 2020 (4th Period)

[Students should read the original play and also the paraphrase given in the school prescribed textbook]Summary Questions amp Answers

[SUMMARY OF THE ENTIRE SCENE]

o The play starts with the scene of a severe storm at sea Alonso (King of Naples) Sebastian (Alonsorsquos brother) Ferdinand (Alonsorsquos son) Gonzalo Antonio (the usurping Duke of Milan) are in a ship in the midst of the storm

o The mariners are trying their best to control the vessel from running aground and are totally following the orders of their Master the Boatswain They have scant success

o The mariners become extremely unhappy and annoyed when most of the passengers arrive on the deck thereby hampering their effort to save the ship There is serious confrontation between them and the passengers who are part of the Kingrsquos entourage

o The mariners could not save the ship

SUMMING-UP

(i) Vivid description of the scene which gives a realistic description of terror and confusion of a tropical storm

(ii) Shows Shakespearersquos accuracy of knowledge in describing the naval operations and also matters of seamanship

(1) GONZALO Ill warrant him for drowning (L 45-57)

though the ship were no stronger than a nutshell and as leaky as an unstanched

wenchBOATSWAIN Lay her a-hold a-hold Set her two courses Off to

sea again lay her offMARINERS All lost To prayers to prayers All lostBOATSWAIN What must our mouths be coldGONZALO The king and prince at prayers Lets assist them

For our case is theirsSEBASTIAN Im out of patienceANTONIO We are merely cheated of our lives by drunkards

This wide-chopped rascal - would thou mightst lie drowning the washing of ten tides

(a) What does Antonio say at the insolent manners of the boatswain just before the given passage

Being irritated at the insolent manners of the boatswain just before the given extract Antonio the Duke of Milan calls him a worthless dog son of a woman without any morals an arrogant and disrespectful noisemaker He says that the boatswain deserved to be hanged(b) What statement does Gonzalo repeat about the boatswain

Gonzalo shows his faith that the boatswain is not destined to die by drowning He is destined to be hanged and nothing can alter this decree of destiny He says that even if the ship was as frail as a nutshell the boatswain could not be drowned for his destiny was to be hanged(c) What do the passengers do when they have lost all hope of their survival

When the passengers have lost all hope of survival they take

(iii) The opening scene justifies the title ndash The Tempest

UNANSWERED QUESTIONS

(i) The King always travels with his entire fleet including his soldiers Where were the other ships

(ii) Why was the ship in that area Where was it coming from or going where

(iii) The ship broke apart What happened to those who were in the ship

(We shall get the answer to the above questions as the play progresses)

leave of life with fervent prayers The mariners take their last hearty drink and are ready for death(d) What blame does Antonio put upon the mariners and the boatswain Antonio rebukes the mariners that these drunkards have brought them to the present crisis by neglecting their duties He blames them saying that they are going to lose their lives entirely for the negligence of the boatswain and his fellows(e) What does Antonio say while cursing the boatswain

Antonio gives vent to his wrath upon the boatswain in particular He calls the boatswain a wide-mouthed rascal who deserves to be hanged on the sea-shore at low water mark so that ten tides might wash over his body and take out of him all the liquor that he has been drinking

Class XIISubject Topic Summary ExecutionHistory Topic

1 1935 ACT AND WORKING OF PROVINCIAL AUTONOMYCONGREE AND OTHER MINISTERSSUB TOPIC GOVERNMENT OF INDIA ACT1935

Government of India Act 1935 This act established a lsquoFederation of Indiarsquo made of British Indian provinces and Indian states and provided for autonomy with a government responsible to the elected legislature in every provinceThis act introduced abolition of Diarchy at provinces The entire provincial administration was introduced to the responsible ministers who were controlled and removed by the provincial legislature The provincial autonomy means two things First The provincial governments were wholly responsible to the provincial legislature Secondly Provinces were free from outside control and interference in the large number of matters The act divided the powers between the centre and provinces in terms of three lists- Federal list( for centre) Provincial list (for province) and concurrent list (for both) Residuary powers were given to the viceroy In the election under the government of India Act the Congress swept the poll the mandate of the people came in favour of the congress so far as general Hindu seats were concerned The Congress did not get a single Muslim seates in Bombay CP UP Sind and BengalIn five provinces Congress had yhe clear majority In BengalNWFPAssam and Bombay Congress emerged as a single largest partyOn the other side the performance of the Muslim League was badThus the Congress formed ministers in 7 provinces out of 11 provinces Coalition ministry was also formed in two other provincesOnly BENGAL AND Punjab had non- congress ministries

1 QUESTION What was the main change introduced by the Government of India ActANSWER a) The Act gave more

autonomy to the provinces b) Diarchy was abolished at the

provincial levelsc) The Governor was the head of

the executived) There was a council of

ministers to advise him The ministers were responsible to the provincial legislatures who controlled them The legislature could also remove the ministers

e) The Governors still retained special reserve powers

2 QUESTION Why did the federal scheme introduced by the Government of India Act 1935 never come into operation

ANSWER The Federal structure of the Government of India was to be composed with the Governor General and Council of ministers The Federal legislature was to be Bicameral legislature- The council of states and the House of Assembly The ministers were to be chosen by the Governor general and they were to hold the office during his pleasure

The provinces of British India would have to join the federation but this was not compulsory for the princely states

This federation never materialised because of the lack of support from the required number of

princely statesThis act was refused and

rejected by the princes the Congress and the Muslim League

Thus both Congress and the League participated in the election of 1937 Thus the federal part was never introduced but the provincial part was put into operations

Bengali 2nd

Language

াচেরর পরাথCনা(কহিতা )

াচেরর পরাথCনা কহিতাটি কহি (ঙখ দেঘাচে4র দো আচো য কহিতায় াচেরর পতর হমায়ন কঠিন দেরাচেগ আxানত ার ঈশবর া আললার কাচেছ পরাথCনা কচেরচেছন তার পচেতরর ীন হিফহিরচেয় হিচেত এই কহিতায় ার পচেতরর ীন হিভbা দেচেয়চেছন ারার এমনহিক হিনচের ীন হিসCচেনর হিহিনমচেয় হিতহিন তার দেছচের ীন হিফচের দেপচেত দেচেয়চেছন তার দেছচের এই দেরাচেগর ন য হিতহিন হিনচেচেকই ায়ী কচেরচেছন তার হিনচের করা পাপচেকই হিতহিন ায়ী কচেরচেছন এছাা রানৈনহিতক ও আথCসামাহিক অসথার কথা তচে ধরা চেয়চেছ এই কহিতায় ার তার হিনচের পাপ কমCচেকই ায়ী কচেরচেছ ার অন যায় ভাচে দেপহি((হিকতর মাধ যচেম অপররা য কচেরচেছ আর এই অন যায় কাচের ন যই তার পহিরাচের হিপযCয় এচেসচেছ দে এক পরকার মানহিক নধন ইহিতাচেসর ার হিপতা চেয় সবাভাহিকভাচে ভাচোাসা দে মমতা দেথচেক মকত চেত পাচেরনহিন তাই হিপতা চেয় আললা া ভগাচেনর কাচেছ পতর হমায়চেনর পরানহিভbা দেচেয়চেছন ার আললা া ভগাচেনর কাচেছ াহিনচেয়চেছন তার হিনচের ীন হিসCন হিচেত হিতহিন রাী তার হিহিনমচেয় পচেতরর ীন হিফচের দেপচেত দেচেয়চেছন াচেরর হিপতসভ হিচেকর কথা এই কহিতায় ফটিচেয় দেতাা চেয়চেছ হিপতা পচেতরর হিরাহিরত মান নধচেনর কথা তচে ধরা চেয়চেছ

হিচে(4 হিকছ াইচেনর তাৎপযC১) ldquoদেকাথায় দেগ ওর সবচছয দেৌন দেকাথায় কচেরায় দেগাপন bয়ldquoউততর) াচেরর পতর হমায়ন কঠিন দেরাচেগ অসসথ তাই তার দেযৌন াহিরচেয় যাচেচছ এই দেরাচেগ তাচেক দেগাপচেন কচেরকচের াচেচছ তার সক (হিকত ধীচের ধীচের bয় চেচছ তাই হিপতা চেয় ার আললার কাচেছ হমায়চেনর পরান হিভbা দেচেয়চেছন২) ldquoাগাও (চেরর পরাচেনত পরানতচের ধসর (ন দেযর আান গানldquoউততর) াচেরর পতর হমায়ন কঠিন দেরাচেগ আxানত তাই ার আ দে(াচেক মমCাত (চেরর পচেথ পরানতচের আান গান ধবহিনত দোক দেসই আান গান আললার কাচেছ দেযন চে যায় আললা দেযন এই আহিতC শচেন পচেতরর ীন হিফহিরচেয় দেয় ৩)ldquoনাহিক এই (রীচেরর পাচেপর ীানচেত দেকানই তরারণ দেনই ভহি4চেতরldquoউততর) হমায়চেনর অসসথতার ন য ার হিনচেচেকই ায়ী কচেরচেছন কারন ার অচেনক রা য অন যায় ভাচে কচেরচেছ তাই তার এই পাপ কাচের ন য তার ঘচের আ হিপ এচেসচেছ এই অন যায় কাচের ন য তার মহিকত দেনই তাই ার আললার কাচেছ এই পাপ কাচেযCর ন য bমা পরাথM

Hindi 2ndlang

-ासी(जयशकर परसा-)

-ासी जयशकर परसा- की एक ऐसी कहानी ह जिजसम भारतीय ससकनित और राषटरीयता का सवरगजीतहोता ह इस कहानी म इरावती एक निहद कनया ह जिजस मलअचछो न मलतान की लट म पकडा और -ासी बना दि-या उस 500 दि-न -कर काशी क एक महाजन न खरी-ा दसरी -ासी निफरोजा ह वह गलाम ह निफरोजा को छडान क कतिलए अहम- को 1000 सोन क कतिसकक भजन थ जो अभी तक नही आए थ राजा साहब कठोर होत हए भी निफरोजा को निबना धनराकतिश क कतिलए उस म कर -त ह वनिफरोजा को अहम- को समझान की बात कहत हकहानी क अत म हम -खत ह निक इरा वती और जाटो क सर-ार बलराज का मिमलन होता हअहम- को यa म मार दि-या जाता ह वहा निफरोजा की परसननता की समामिध बनती ह वहा एक फल चढती ह और डीजल आती ह निफरोजा उस समामिध की आजीवन -ासी बनी रहती हलखक अपन उददशय अथात -ास परथा पर परकाश डालन और इस परथा क कारण होन वाल -ातो क दखो को दि-खान म पणता सफल हए ह

helliphellipContinue to next

Biology Reproductio Today we will discuss about vegetative Q1 Name some vegetative propagules

n in Organisms

propagation of plants The process of multiplication in which fragments of plant body function as propagule and develop into new individual is called vegetative propagation The units of such propagation are runner rhizome tuber bulb etc

and the speciesinvolvedVegetative propagules

Parts involved

Bulb StemBulbil BulbilRhizome Stem Runner Stem Tuber Stem Offset Stem Leaf buds Leaves Suckers Stem

Corns Stem stolon

Q2 State advantages of vegetative propagation

i) Rapid methodii) Sure and easy methodiii) Useful in plants that cannot

produce viable seeds or long seed dormancy

iv) Maintains purity of raceQ 3 Banana fruit is said to be parthenocarpic where as turkey is said to be parthenogenetic WhyBanana develops without fertilization from an unfertilized ovary thus is parthenocarpicIn turkey the ovum or female gamete developinto a new chick without fertilization thus isparthgenetic

Q4 Why is water hyacinth is called as a ldquoTerror of Bengalrdquo Water hyacinth can

propagatevegetatively all over the water body in a short per short period of time This resulted increased biochemicaloxygen oxygen demand of water body causing mortalityof fishes It is very difficult to get rid off them Thus known as terror of Bengal

Chemistry

Solid state GENERAL CHARACTERISTICS OF SOLID STATEIn nature the particular state of matter is governed by two opposing forces at given set of temperature and pressure These forces are intermolecular force of attraction and thermal energy If intermolecular force of attraction is high as compared to thermal energy particles remains in closest position

Intext QuestionsQ1 Classify the following solids as crystalline and amorphous Sodium chloride quartz glass quartz rubber polyvinyl chloride Teflon

A1 Crystalline

and hence very less movement in particles is observed In this case solid state is the preferred state of matter

Let us revise the general characteristics of solid

i) Fixed mass volume and shape

ii) Strong intermolecular force of attraction

iii) Least intermolecular space

iv) Fixed position of constituent particles

v) Incompressible and rigid

Q2 what type of interactions hold the molecules together in a polar molecular solid[CBSE 2010]A2 The molecules in a solid are held together by van der Waals forces The term van der Waals forces include hydrogen bonding dipole-dipole attraction and London dispersion forces All molecules experience London dispersion forces In addition polar molecules can also experience dipole-dipole interactions So the interactions that holds the molecule together in polar molecular solid are London dispersion force and dipole-dipole interactionsQ3 Write a feature that will distinguish a metallic solid from an ionic solid [CBSE 2010]A3 Metals are malleable and ductile whereas ionic solid are hard and brittle Metallic solid has typical metallic lustre But ionic solid looks dullQ4 Write a point of distinction between a metallic solid and an ionic solid other than metallic lustre [CBSE 2012]A4 Metals are malleable and ductile whereas ionic solid are hard and brittleQ5 Write a distinguish feature of metallic solid [CBSE 2010]A5 The force of attraction in

solid Sodium chloride Quartz Amorphous solid Quartz glass rubber polyvinyl chloride Teflon Q2 why glass is considered as super cooled liquidA2 Glass shows the tendency to flow at slower rate like liquid Hence they considered as super cooled liquidQ3 why the window glass of old buildings show milky appearance with timeA3 Glass is an amorphous solid Amorphous solid has the tendency to develop some crystalline character on heating Due to heating in day over the number of years glass acquires some crystalline character and show milky appearanceQ4 why the glass panes fixed to window or doors of old building become slightly thicker at bottomA4 Glass is super cooled liquid It has the tendency to flow down very slowly Due to this glass pane becomes thicker at the bottom over the timeQ5 Sodium chloride is a crystalline solid It shows the same value of refractive index along all the direction TrueFalse Give reasonA5 FalseCrystalline solid shows anisotropy in properties That is it shows different values for the given physical property in different direction All the crystalline solids show anisotropy in refractive index Therefore sodium chloride will show different values of refractive index on different directions

Q6 Crystalline solid are anisotropic in nature What does this statement means

between the constituent particles is special kind of electrostatic attraction That is the attraction of positively charged kernel with sea of delocalized electronsQ6 which group of solid is electrical conductor as well as malleable and ductile [CBSE 2013]A6 Metallic solidQ7 why graphite is good conductor of electricity although it is a network (covalent solid)A7 The exceptional property of graphite is due to its typical structure In graphite each carbon is covalently bonded with 3 atoms in same layer The fourth valence electron of each atom is free to move in between different layersThis free electron makes the graphite a good conductor of electricity

[CBSE 2011]A6 Anisotropy is defined asrdquo Difference in properties when measured along different axis or from different directionsrdquo Crystalline solid show different values of some of the physical properties like electrical resistance refractive index etcwhen measured along the different directions The anisotropy in crystalline solid arises due to the different arrangement of particles in different directions

Math Function Composition of functions Think of an industrial plant that produce bottles of cold drinks first there is the operation (or function) f that puts the cold drink inside the bottle followed by the opeartion g that close the bottle with the capThis leads to the following definitionDefinition Let f A rarr B and g B rarr C be two functions Then the composition of f and g denoted by gof is defined as the function gof A rarr C given by gof(x) = g(f (x)) forall x isinA

Definition A function f X rarr Y is defined to be invertible if there exists a function g Y rarr X such that gof = IX and fog = IY The function g is called the inverse of f and is denoted by f -1

Thus if f is invertible then f must be one-one and onto and conversely if f is one-one and onto then f must be invertible This fact significantly helps for proving a function f to be invertible by showing that f is one-one and onto specially when the actual inverse of f is not to be determined

Example 1 Let f 2 3 4 5 rarr 3 4 5 9 and g 3 4 5 9 rarr 7 11 15 be functions defined as f(2) = 3 f(3) = 4 f(4) = f(5) = 5 and g (3) = g (4) = 7 and g (5) = g (9) = 11 Find gofSolution We have gof(2) = g (f(2)) = g (3) = 7 gof(3) = g (f(3)) = g (4) = 7gof(4) = g (f(4)) = g (5) = 11 and gof(5) = g (5) = 11Example 2 Find gof and fog if f R rarr R and g R rarr R are given by f(x) = cos x and g (x) = 3x2 Show that gof ne fogSolution We have gof(x) = g(f(x))=g(cosx) = 3 (cos x)2

= 3 cos2 x Similarly fog(x)=f(g (x))= f(3x2)= cos (3x2) Note that 3cos2 x ne cos 3x2 for x = 0 Hence gof ne fogExample 3 Show that if f A rarr B and g B rarr C are onto then gof A rarr C is also ontoSolution Given an arbitrary element z isin C there exists a pre-image y of z under g such that g (y) = z since g is onto Further for y isin B there exists an element x in A with f(x) = y since f is onto Therefore gof(x) = g (f(x)) = g (y) = z showing that gof is onto Example 4 Let Y = n2 n isin N sub N Consider f N rarr Y as f(n) = n2 Show that

f is invertible Find the inverse of fSolution An arbitrary element y in Y is of the form n2 for some n isin N This implies that n =radicy This gives a function g Y rarr N defined by g (y) =radicy Nowgof (n) = g (n2)=radicn2 = n and fog (y) =f(radicy) = (radicy) 2 y which shows that gof=IN and fog= IY Hence f is invertible with f -1 = g

Political Science

Constitution of India-The Preamble

Summary

Objective of the state-To secure equality of status and of opportunity To promote fraternity among all the citizens To assure the dignity of the individuals and Unity and integrity of the nation

Justice-Justice stands for rule of law absence of arbitrariness and a system of equal rights freedom and opportunities for all in a society India seeks social economic and political justice to ensure equality to its citizens

Liberty-Liberty implies the absence of restraints or domination on the activities of an individual such as freedom from slavery serfdom imprisonment despotism etc The Preamble provides for the liberty of thought expression belief faith and worship

Equality-Equality means the absence of privileges or discrimination against any section of the society The Preamble provides for equality of status and opportunity to all the people of the country

Fraternity-The Preamble declares that fraternity has to assure two thingsmdashthe dignity of the individual and the unity and

Execution

Answer the following questions-

Short notes-1 Equality2 Fraternity3 Justice4 Liberty

Homework-Learn

integrity of the nation The word integrity has been added to the Preamble by the 42nd Constitutional Amendment (1976)

Business studies

Human resource management (chapter 1)

On the day of 1504 2020 I have discussed with you the managerial functions and procurement functions of HRM

Today weare going to discuss about the development function integration functions and maintenance function

Development functions-HRM improves the knowledge skills attitude and values of employees so that they the present and future jobs more effectively it includes

1) Development functions of HRM

a) Performance appraisal = It implies systematic evaluation of employees with respect to their performance on the job and their potential for development

b) Training =It is the process by which employees learn knowledge skills and attitudes to achieve organisational and personal goals

c) Executive development = It is the process of developing managerial talent through appropriate program

2) Integration functionsa) HRM reconcile the goals of

organisation with those of its members through integrating function

b) HRM tries to motivate employees to various financial and non financial incentives provided in job specification etc

3) Maintenance functiona) HRM promote and protect the

physical and mental health of employees by providing several types of benefits like housing medical aid etc

b) It Promote Social security measures to employees by providing provident fund pension gratuity maternity benefits

SubjectCOMMERCE

Topic

BUSINESSENVIRONMENT

Summary

Now quickly let us revise the earlier points that we have already done in the last class and let us proceed with the other topics that are there in the chapter

Firstly we will recall the internal and external factors of micro environment and then we

Execution 3 What do you mean by internal factors

in micro environmentAnswerInternal factors refer to all the factors existing within a business firm The internal factors are considered controllable because the enterprise has control over these factors

Development FunctionsPerformance AppraisalTrainingExecution Development

shall proceed in details

Meaning and list of internal and external factors

aInternal factorsInternal factors refer to all the factors existing within a business firm The internal factors are considered controllable because the enterprise has control over these factorsFor an example a company can alter its organization structure policies programmes employees physical facilities and marketing mix to suit the changes in the environmentList of internal factors areCorporate culture mission and objectives top management organizations structure company image and brand equity company resources

b External factorsExternal factors refer to those individual and groups and agencies with which a particular business organization comes into direct and frequent contact in the course of its functioningThese individuals and groups are known as STAKEHOLDERS because they have a stake (financial interest ) in the working and performance of the particular business List of external forces (stakeholders)Customers competitors investors suppliersmiddlemen (marketing intermediaries)financers publics

customers

suppliersfinancers

For an example a company can alter its organization structure policies programmes employees physical facilities and marketing mix to suit the changes in the environment

4 What do you mean by external factors in micro environment

AnswerExternal factors refer to those individual and groups and agencies with which a particular business organization comes into direct and frequent contact in the course of its functioningThese individuals and groups are known as STAKEHOLDERS because they have a stake (financial interest) in the working and performance of the particular business

3Who are stakeholdersSTAKEHOLDERS are individuals and groups who have a stake (financial interest ) in the working and performance of the particular business 4Discuss the internal factors in briefa Corporate CultureThe values beliefs and attitudes of the founders and top management of the company exercise a strong influence on what the cmpaany stands for how it does things and what it considers importantbMission and objectivesThe business philosophy and purpose of a comoany guide it prioritiesbusiness strategiesproduct market scope and development scope

cTop management structurethe composition of board of directors the degree of professionalization of management and the organizational structure of a company have important bearing on its business decisions

dPower structureThe internal power relationship between the board of directors and the chief executive is an important factor

e Company image and brand equityThe image and brand equity of the company play a significant role in raising finance forming alliance choosing dealers and suppliers launching new products entering foreign markets

5 What is Macro environmentAnswerMacro environment refers to the general

competitors

middlemen

publics

Fig STAKEHOLDERS OF A COMPANY

Apart from micro environment the other main dimension of business environment isMacro environment Macro environment refers to the general environment or remote environment within which a business firm and forces in its micro environment operateA company does not directly or regularly interact with the micro environmentTherefore macro environment is also known as indirect action EnvironmentThe macro environment forces are less controllable than the micro forces

Macro environment consists of the following components

POLITICAL AND LEGAL ENVIRONMENT

ECONOMIC SOCIAL AND ENVIRONMENT

CULTURAL

ENVIRONMENT

TECHNOLOGICAL ENVIRONMENT

Fig COMPONENTS OF MACRO ENVIRONMENT

environment or remote environment within which a business firm and forces in its micro environment operateA company does not directly or regularly interact with the micro environmentTherefore macro environment is also known as indirect action EnvironmentThe macro environment forces are less controllable than the micro forces 6 What are the components of macro environmenta Political and legal environmentb Economic environmentc Social and cultural environmentd Technological environment

Computer Science

Logic gates

Digital systems are said to be constructed by using logic gates These gates are the AND OR NOT NAND NOR EXOR and EXNOR

BUSINESS FIRM

gates The basic operations are described below with the aid of truth tables

AND gate

The AND gate is an electronic circuit that gives a high output (1) only if all its inputs are high A dot () is used to show the AND operation ie AB Bear in mind that this dot is sometimes omitted ie ABOR gate

The OR gate is an electronic circuit that gives a high output (1) if one or more of its inputs are high A plus (+) is used to show the OR operationNOT gate

The NOT gate is an electronic circuit that produces an inverted version of the input at its output It is also known as an inverter If the input variable is A the inverted output is known as NOT A This is also shown as A or A with a bar over the top as shown at the outputs The diagrams below show two ways that the NAND logic gate can be configured to produce a NOT gate It can also be done using NOR logic gates in the same way

NAND gate

This is a NOT-AND gate which is equal to an AND gate followed by a NOT gate The outputs of all NAND gates are high if any of the inputs are low The symbol is an AND gate with a small circle on the output The small circle represents inversion

NOR gate

This is a NOT-OR gate which is equal to an OR gate followed by a NOT gate The outputs of all NOR gates are low if any of the inputs are highThe symbol is an OR gate with a small circle on the output The small circle represents inversion

EXOR gate

The Exclusive-OR gate is a circuit which will give a high output if either but not both of its two inputs are high An encircled plus sign ( ) is used to show the EOR operation

EXNOR gate

The Exclusive-NOR gate circuit does the opposite to the EOR gate It will give a low output if either but not both of its two inputs are high The symbol is an EXOR gate with a small circle on the output The small circle represents inversion The NAND and NOR gates are called universal functions since with either one the AND and OR functions and NOT can be generated

Note A function in sum of products form can be implemented using NAND gates by replacing all AND and OR gates by NAND gates A function in product of sums form can be implemented using NOR gates by replacing all AND and OR gates by NOR gates

Logic gate symbols

Table 2 is a summary truth table of the inputoutput combinations for the NOT gate together with all possible inputoutput combinations for the other gate functions Also note that a truth table with n inputs has 2n rows You can compare the outputs of different gates

Logic gates representation using the Truth table

Example

A NAND gate can be used as a NOT gate using either of the following wiring configurations

Subject Eng Literature (The Tempest ndash William Shakespeare) Topic Act III Scene 3 Lines 53 to 110 (End of the scene) Date 16th April 2020 (2nd Period)

[Students should read the original play and also the paraphrase given in the school prescribed textbook]Summary Questions amp Answers

o Seeing this strange scene all are inclined to believe the tales told by travelers that there truly are ldquounicornsrdquo and ldquothe phoenixrsquo thronerdquo

o As they are about to sit down to the feast the banquet is snatched away by a harpy (Ariel disguised) A spiritrsquos voice (Arielrsquos voice) denounces Alonso Sebastian and Antonio with particular

1 ARIEL You are three men of sin whom Destiny

(Line 53-58)That hath to instrument this

lower world And what is int the never-surfeited sea

Hath caused to belch up you and on this island

Where man doth not inhabit you rsquomongst men

Being most unfit to live I have made you mad

reference to their crime in expelling Prospero from Milan They have not received any punishment for their deed earlier but the time for their punishment has arrived Upon Alonso it pronounces ldquolingering perdition worse than deathrdquo from which there is no remedy except through sincere repentance Ariel then vanishes in thunder and the shapes enter again and carry away the table

o Prospero watching invisibly is very pleased with the performance of Ariel and his (Prosperorsquos) ldquomeaner ministersrdquo All his enemies are now in his power and are in a fit of desperation He then leaves them and goes to see how Ferdinand and Miranda are getting on

o Alonso is now much humbled and penitent with the after effect of the spiritrsquos denunciation of his crimes He believes that his son is lost forever After this all disperse being stricken mad by the speech of the spirit

o Gonzalo fearing that they may do violence to themselves or to one another follows them and bid others to follow

(a) To whom does Ariel disguised as a harpy call the three sinners What game did Fate of Destiny play with

them

The three sinners called by Ariel are Alonso Sebastian and Antonio It was Destiny which had caused the ocean to cast the three sinners on the shore Though the ocean is all the time devouring whatever appears on its surface and is never satisfied with its continual swallowing of the ships and men in the present case the ocean had cast these three sinners on the shore without killing them

(b) Who had jointly been responsible for the conspiracy against Prospero What is Prosperorsquos purpose behind all this

Three men Alonso Sebastian and Antonio had jointly

been responsible for the conspiracy against Prospero They had driven out Prospero form Milan Prosperorsquos purpose is to make these three sinners realize the wrong they had done He wants them to repent for their criminal deeds because repentance leads to self-esteem(c )What does Ariel (the harpy) tell Alonso and his companions when they take out their swords to attack him

Seeing them drawing their swords Ariel (harpy) tells them that he and his companions are the instruments of destiny and that it is not possible for human beings to do them any injury He says that the swords of human beings can not injure even a minute part of his feathers Their swords are as ineffective against him and his companions as against the wind or the water

(d) Give the explanatory meanings of the following expressions in the context of the above extract

(i)Never surfeited (ii) Belch up (iii) lsquomongst men

(i) Never surfeited never led to satisfaction

(ii) Belch up cast ashore(iii) lsquomongst men in human

society2

I and my fellows (Line 60-65)

Are ministers of Fate The elementsOf whom your swords are tempered may as wellWound the loud winds or with bemocked-at stabsKill the still-closing waters as diminishOne dowl thats in my plume

IMPORTANT PASSAGES EXPLAINED

The elements

(Line 61-66)Of whom your swords are tempered may

as wellWound the loud winds or with

bemocked-at stabs

(a) Who is lsquoIrsquo Who are his lsquofellowsrdquo

lsquoIrsquo is referred to Ariel in disguise of a harpy His lsquofellowsrsquo are other spirits serving Prospero the real Duke of Milan who has acquired supernatural powers after being banished from his Dukedom Prospero has settled in this uninhabited island

(b) What are the elements that have temperrsquod the swords Why will it not work against the speaker

The swords (of Alonso and his companions) are tempered by metal (steel) which is taken out of the earth and refined by

Kill the still-closing waters as diminishOne dowl thats in my plume My fellow

ministersAre like invulnerable

In these words Ariel reminds the King and his companions of the utter futility of drawing swords against himself and his fellows Ariel drives Alonso Antonio and Sebastian the three men of sin to desperation ndash a state in which men do violence to themselves They draw swords to strike Ariel But Ariel reminds them that he and the other spirits are the ministers of destiny and nothing can wound them The steel of which their swords are made of may cut the wind or water which being divided always closes up again Even supposing that such things may be possible it is quite impossible that their swords will cut one feather in their plume They are incapable of being wounded by any sword of man Hence it is foolish on their part to attempt to strike at Ariel and his fellow-spirits

For which foul deed

(Line 72-75)The powers delaying not forgetting

haveIncensed the seas and shores yea all the

creatures Against your peace

Ariel enters like a harpy and remaining invisible tells Alonso Sebastian and Antonio that he and other harpies are the agents of Destiny appointed to carry out her decrees He tells them that their punishment for the crime against Prospero which has been so long deferred is now to fall upon them He reminds them that they had expelled Prospero from Milan and set him and his innocent child adrift on the sea and that the sea had paid them back for their sin by the shipwreck and by the calamities they have suffered He tells them that the powers above which did not forget this mean treachery but only deferred the punishment have now engaged the seas and the shores and all living beings including him and his comrades against them The very elements and supernatural agency Ariel adds have taken up the avenging of their crime against Prospero

the action of fire It may cut the wind or water which being divided always closes up again

The sword will not work against the spirits and the harpy because they are the ministers of destiny and nothing can wound them nor it will cut a single feather in their plume

(c )What is the meaning of lsquodowlrsquo in the last line

The term lsquodowlrsquo means a filament or the smallest part of a feather In this context Ariel in disguise of harpy says that their sword cannot even damage the smallest filament of their (Arielrsquos and other spirits) feathers as they are incapable of being wounded by any sword of man

(d) What does the speaker remind the listeners about

Ariel in disguise of harpy reminds Alonso the King of Naples Sebastian Alonsorsquos brother and Antonio the present Duke of Milan and the treacherous brother of Prospero as they being three men of sin He even reminds them that their punishment for their crime against Prospero which has been so long deferred now falls upon them He reminds them that they have expelled Prospero from Milan and has set him along with his innocent infant daughter adrift on the sea So the sea has paid them back for their sin by their shipwreck and the calamities they have suffered since then The harpy rebukes Alonso of his sin that has incensed the Gods and has deprived him of his son as a punishment

(e) How do they respond

When Ariel in disguise of a harpy reminds Alonso Sebastian and Antonio of their past misdeeds and sin Alonso has a look of terror and confusion in his eyes He utters the words of sincere repentance wrung out of his conscience-stricken heart It appears to him that all the elements of nature the sea-waves the wind and the thunder proclaiming a loud voice in the name of Prospero and the crime Alonso has committed against him They are calling upon him to repent There is a deep storm raging in Alonsorsquos breast and the echoes of that storm are ringing in his ears like a clear note of wind-instrument A note of denunciation of Alonsorsquos crime leaves him much humbled and penitent and confirms his belief that his son is lost forever But Sebastian and Antonio shows some courage instead of repentance They wish to kill the spirits or devils if it appears

3

Of my instruction hast thou nothing bated (Line 85-93)

In what thou hast to say So with good life

And observation strange my meaner ministers

Their several kinds have done My high charms work

And these mine enemies are all knit upIn their distractions They now are in my

powerAnd in these fits I leave them while I visitYoung Ferdinand whom they suppose is

drownedAnd his and mine loved darling

Methought the billows spoke and (Line 96-99)

told me of itThe winds did sing it to me and the

thunderThat deep and dreadful organ-pipe

pronouncedThe name of Prosper It did bass my

trespass

These are the words of contrition coming from Alonso Ariel has driven him to a deep repentance for conspiring with Antonio against Prospero He now feels a sincere remorse It appears to him that all the elements of nature the sea-waves the wind and the thunder proclaimed with a loud voice the name of Prospero and the crime Alonso had committed against him They are calling upon him to repent There is a deep storm raging in Alonsorsquos breast and the echoes of that storm are ringing in his ears like the clear note of a wind-instrument

Comment These are the words of sincere repentance wrung out of the conscience-stricken heart of Alonso Alonso who is the lesser villain is the first to give way to remorse under the effect of Arielrsquos speech The words of Ariel seem to him to be the voice of conscience speaking to him He is driven to desperation a state in which he might do violence to his life

(a) Identify the speaker State the context

Prospero the ruler of the island is the speaker The famous banquet scene has been enacted very well Ariel and his junior spirits have played their roles excellently Prospero is glad to say words of praise for them(b) In what way the speakerrsquos instructions have been carried out

According to Prosperorsquos instructions a banquet was presented before the King of Naples and his companions when they were tired and hungry Just when they were preparing to eat the feast the banquet was suddenly removed by exercising supernatural powers All this was done by Ariel Prosperorsquos chief assistant and a powerful spirit

Ariel not only made the feast disappear but also delivered his speech blaming the King and his two companions for their past wicked deeds He warned them to repent for their misdeeds or suffer forever on that uninhabited island

(c) Who are referred to as lsquomeaner ministersrsquo What have they done

Prospero refers as lsquomeaner ministersrsquo to his other lesser spirits who were assisting Ariel in presenting a scene before the kingrsquos party They entered the scene to the accompaniment of music They assumed several strange shapes and brought in a banquet Then they danced about it with gentle actions of salutations thus inviting the King and others to eat the feast

These spirits play their role again when Ariel in the shape of a harpy quits the scene These shapes enter again and dancing with mocking gestures carry away the table

(d) Who are the speakerrsquos enemies What has happened to them

King of Naples Alonso his brother Sebastian and the present Duke of Milan Antonio (Prosperorsquos own brother) are Prosperorsquos enemies With the turn of events they have all been washed ashore on the island which is ruled by Prospero the great magician Actually this happened after the shipwreck caused by a storm which was raised by Prospero with the purpose of bringing these people to his island Prosperorsquos spirits have already confused and terrified these enemies and they are under Prosperorsquos control He can treat them as he likes

(e) What does he say about Ferdinand Explain what is meant by ldquohellip his and mine darlingrdquo

Prospero knows that Alonsorsquos son prince Ferdinand is alive though his father thinks that the prince has been drowned

Prospero refers to his daughter Miranda who is dear to him She is also very dear to Prince Ferdinand who has fallen in love with her They are waiting to be married soon for which they have received Prosperorsquos consent

4

ALONSO O it is monstrous monstrous (Line 95-102)

Methought the billows spoke and told me of it

The winds did sing it to me and the thunderThat deep and dreadful organ-

pipe pronouncedThe name of Prosper It did bass

my trespassTherefore my son ithrsquo ooze is

bedded andIll seek him deeper than eer

plummet soundedAnd with him there lie mudded

(a) In what way does Alonso express his horror when his conscience is awakened by Arielrsquos words

When Alonsorsquos conscience is awakened by Arielrsquos words he expresses his horror at what he has heard He gets the feeling that the waves of the ocean the wind and the loud thunder have spoken to him and uttered the name of Prospero Because of being reminded of his crime in a very loud and rough voice he comes to realize that he has lost his son for his past misdeeds

(b) What does Alonso imagine about his son What does Alonso want to do in his desperate state

Alonso imagines that his son is lying in the mud at the bottom of the sea He feels desperate that he wants to drown himself in the ocean deeper than the plumb-line has ever gone He wants to lie with his son at the bottom of the sea

(c) How do Sebastian and Antonio want to face the evil spirits

Sebastian says that he is not at all afraid of what the harpy has said and that he is prepared to fight any number of such monsters if they appear before him only one at a time Antonio says that he would support Sebastian in the fight against the fiendsyyy

(d) Why does Gonzalo ask Adrian to follow the three men

Gonzalo tells Adrian that all the three men namely Alonso Sebastian and Antonio are in a wild and reckless mood The thought of the heinous crime of which they are guilty has begun to torment their minds So he asks Adrian to follow those three men without loss of time and prevent them from doing anything which the turmoil in their minds might lead them to do

(e) What opinion do you form of Alonso from the above extract

Alonso who is the lesser villain is the first to give way to remorse under the effect of Arielrsquos speech The words of Ariel seem to him to be the voice of conscience speaking to him He is driven to desperation a state in which he might do violence to his life

Subject =Accounts

Ac-12 15420 topic-pL Appropriation ac

PROFIT AND LOSS APPROPRIATION ACCOUNT

MEANING AND PREPARATIONProfit and Loss Appropriation Account is merely an extension of the Profit and Loss Account of the firm The profit of the firm has to be distributed amongst the partners in their respective profit sharing ratio But before its distribution it needs to be adjusted All Adjustments like partnerrsquos salary partnerrsquos commission interest on capital interest on drawings etc are made in this account These adjustments will reduce the amount of profit for distribution This adjusted profit will be distributed amongst the partners in their profit sharing ratio To prepare it at first the balance of Profit and Loss Account is transferred to this account The journal entries for the preparation of Profit and Loss Appropriation Account are given below

1 for transfer of the balance of Profit and Loss Account to Profit and Loss Appropriation Account

(a) In case of Net Profit

Profit and Loss Ac helliphelliphelliphelliphellipDrTo Profit and Loss Appropriation Ac(Net Profit transferred to Profit and Loss Appropriation Ac)

(b)In case of Net Loss

Profit and Loss Appropriation Achelliphelliphellip DrTo Profit and Loss Ac(Net Loss transferred to Profit and Loss Appropriation Ac)

2 for Interest on Capital

For transferring on Interest on CapitalProfit and Loss Appropriation Achelliphelliphellip DrTo Interest on Capital Ac(Interest on capital transferred to Profit amp Loss Appropriation Ac)

3 for Interest on Drawings

For transferring Interest on Drawings Interest on Drawings Achelliphelliphelliphelliphelliphellip DrTo Profit and Loss Appropriation Ac(Interest on drawing transferred to Profit amp Loss Appropriation Ac)

4 For Partnerrsquos SalaryFor transfer of partnerrsquos SalaryProfit and Loss Appropriation Achelliphellip DrTo Salary Ac(Salary transferred to profit amp Loss Appropriation Ac)

5 For Partnerrsquos CommissionFor transferring commissionProfit and Loss Appropriation Achelliphelliphellip DrTo Commission Ac(Commission transferred to Profit and Loss Appropriation Ac)

6 For Transfer of agreed amount to General ReserveProfit and Loss Appropriation Ac helliphellipDrTo General Reserve Ac(Transfer to General Reserve)

7 for share of Profit or Loss appropriation(a) If ProfitProfit and Loss Appropriation Achelliphellip DrTo Partnerrsquos CapitalCurrent Ac(Profit transferred to capitalcurrent Ac)(b) If LossPartnerrsquos Capital Current Achelliphelliphelliphellip DrTo Profit and Loss Appropriation Ac(Loss transferred to capitalcurrent Ac)

THE FORMAT OF PROFIT AND LOSS APPROPRIATION

Profit and Loss Appropriation Account for the year endedhelliphelliphelliphellip

Particulars Amount Particulars Amount

To PL Ac (loss) By pL Ac (profit)

To Interest on capital BY Interest on drawings

To partner`s commission by Partner`s capital Ac ( loss)

To Partner`s salary To Interest on partner`s loan To General Reserve To Partner`s Capital AC (Profit)

Subject= Economics

MOVEMENT ALONG THE DEMAND CURVE (CHANGE IN QUANTITY DEMANDED)In law of demand you have already studied the inverse relationship between price and quantity demanded When quantity demanded of a commodity changes due to change in its price keeping other factors constant it is called change in quantity demanded It is graphically expressed as a movement along the same demand curve There can be either a downward movement or an upward movement along the same demand curve Upward movement along the same demand curve is called contraction of demand or decrease in quantity demanded and downward movement along the same demand curve is known as expansion of demand or increase in quantity demanded

Extention of demandd

price (rs)p A

B Extentionp1 d

Q Q1

Quantity demanded ( in units)

Contraction of demandd

p2 Ccontraction

p APrice (Rs)

d

Q2 Q

Quantity demanded (in units)

Explanation of movement of demand A fall in price from OP to OP1 leads to increase in quantity demanded from OQ to OQ1 (expansion of demand) resulting in a downward movement from point A to point B along the same demand curve DD When Price rises from OP to OP2 quantity demanded falls from OQ to OQ2 (contraction of demand) leading to an upward movement from point A to point C along the same demand curve DD

  • Activity Series of Metals
    • Drawbacks of Rutherfordrsquos model of atom
      • Electromagnetic radiations
      • Properties of electromagnetic radiations
      • Characteristics of electromagnetic radiations
        • Plancks Quantum Theory-
        • Photoelectric effect
          • Intext Questions
            • Logic gates
            • Digital systems are said to be constructed by using logic gates These gates are the AND OR NOT NAND NOR EXOR and EXNOR gates The basic operations are described below with the aid of truth tables
            • AND gate
            • Example
Page 4:  · Web viewSubject. Topic. Summary. Execution. English 1 . Chapter 1 naming words . Page 8. Write the names of these pictures:- Person:-1. father. 2.Firefighter 3.doctor 4 ...

BENGALI ই ndash াংা সাহিতয পহিরয়

পাঠ ndash ১একতাই অন(ীনীর পরচোততর

৫ হিপরীত - চো times দোকা স times ঃ হি( times অহি( াধা times ছাা আস times নক সহিতয times হিমচেথয৭ াকয রনা -মযাহিক ndash আমার কাকা ভাচো মযাহিক দোয় া(ঝা ndash রাচেতর অনধকাচের া(ঝা দেচে আমার ভয় াচেগ অহিনষট ndash অচেনযর অহিনষট হিনতা করচেত দেনই াহি= ndash কাগচের াহি= চে পচেরাচেনা স নহিথ পাওয়া দেগচো

HINDI वयाकरण Chapter 4 सजञा

सजञा- इस दनिनया म सभी का कोई ना कोई नाम होता ह यही नाम सजञा कहलाता ह नाम कई तरह क होत ह जस -वयकतियो क नाम पराणिणयो क नाम सथानो क नाम भावो क नाम आदि-

1सजञा निकस कहत ह उ-ाहरण -ीजिजएउततर- निकसी भी वयकति वसत सथान या भाव क नाम को सजञा कहत हजस- राम सकल घडी गरमी आदि-

MATHEMATICS CHAPTER-4 SUBTRACTION

Subtraction (with decomposing)Subtraction of 3-digit numbers

Exercise 18Example 1 (p-48)

Fill in the place holders3 hundreds 2 tens 5 ones ndash 2 hundreds 5 tens 7 ones= 3 hundreds 1 tens ___ ones - 2 hundreds 5 tens 7 ones= 2 hundreds ___ tens 15 ones - 2 hundreds 5 tens 7 ones= 6 tens ___ ones

Solution3 hundreds 2 tens 5 ones ndash 2 hundreds 5 tens 7 ones= 3 hundreds 1tens 15 ones - 2 hundreds 5 tens 7 ones= 2 hundreds 11 tens 15 ones - 2 hundreds 5 tens 7 ones= 6 tens 8 ones

Example 4 (p-51)Subtract 389 from 600

Solution H T O 6 0 0 - 3 8 9

2 1 1We cannot subtract 9 ones from 0 ones so we borrows 1 tens Here there is no ten so first we borrow 1 hundreds or 10 tens having behind 5 hundreds From 10 tens we borrow 1 tens or 10 ones leaving behind 9 tensNow 10 ones ndash 9 ones = 1 onesAnd 9 tens ndash 8 tens = 1 tens5 hundreds ndash 3 hundreds = 2 hundreds 600 ndash 389 = 211

Exercise2 Fill in the place holders5 hundreds 2 tens 8 ones ndash 3 hundreds 8

tens 3 ones= 4 hundreds ___ tens 8 ones - 3 hundreds 8 tens 3 ones= 1 hundreds ___ tens ___ ones

Solution5 hundreds 2 tens 8 ones ndash 3 hundreds 8 tens 3 ones= 4 hundreds 12 tens 8 ones - 3 hundreds 8 tens 3 ones= 1 hundreds 4 tens 5 ones

Class IVSubject Topic Summary Execution

English literature

Hercules andDeianira (tales from Greece and Rome)

Hercules spent the greater part of his life in helping the poor and weak people But Juno still wanted to harm him So she sent him into all sorts of dangers But he was brave and strong and he overcame all the dangers Juno made him a slave to the king of Argos Hercules worked hard for the king The king took pity on him and told him that he would set him free if Hercules perform twelve difficult tasks Hercules accepted the challenge

1How did Hercules spent the greater part of his lifeAns Hercules spentthe greater part of his life in helping the poor and weak people

2How did Juno tried to harm HerculesAns Juno tried to harm Hercules by sending him into all sorts of dangers

3Under which king Hercules was working as a slaveAns King of Argos

4What was the challenge that king of Argosgive to HerculesAns King of Argos told Hercules that he would set him free if he perform twelve difficult tasks which Hercules accepted

Social studies Major landforms on earth

Answer the following questions5What do oceans provide usAns Oceans provide us a variety of sea food

6What is alluviumAns The rivers carry sand soil and small pieces of rocks This fine soil or silt known as alluvium is very good for growing crops

COMPUTER160420

CHAPTER 3 EDITING IN MS WORD

DRAG AND DROPTHE DRAG AND DROP METHOD OF MOVING THE TEXT ALLOWS US TO MOVE THE SELECTED TEXT USING MOUSE FROM ONE LOCATION TO ANOTHER WITHIN A DOCUMENT

Q7) HOW TO WE DRAG AND DROP TEXT IN MS WORDAns) WE CAN DRAG AND DROP TEXT IN MS WORD FOLLOWING STEPS ARE---

SELECT THE TEXT PLACE THE MOUSE POINTER

ANYWHERE ON THE SELECTED TEXT

CLICK AND HOLD THE LEFT MOUSE BUTTON UNTIL THE INSERTION POINT CHANGES INTO A WHITE ARROW

WHEN WE REACH THE PLACE IN THE DOCUMENT WHERE WE WANT TO MOVE THE TEXT RELEASE THE MOUSE BUTTON

Science Adaptations in Summary Execution

Animals Animals are divided into different groups according to their habitats Some animals live on land some animals live both on land and in water some animals adapted themselves for food and some for protection They develop special features that help them to survive in their habitats Animals that live on land are called Terrestrial animals such as lions tigers Yaks polar bears Polar Bears live cold polar regions Yaks live in mountains These animals have thick fur on their body and a thick layer of fat under their skin which keeps them warm

Solved exercise question and answersA Write the correct answer1Terrestrial animals live here - on land 2This animal lives in polar regions ndash polar bear 3 These help a fish to breathe in water - gills4 This helps carnivorous birds to tear flesh - a hooked beak

MATHEMATICS

Ch 6Multiplication

Exercise 17 Example 2

Multiply 224835 by 4 Solution

2 2 4 8 3 5 times 48 9 9 3 4 0

Exercise

18 3 1 1 4 6 7 19 2 1 2 7 8 5 times 3 times 4 9 3 4 4 0 1 8 5 1 1 4 0

20 2 0 6 1 3 times 9 1 8 5 5 1 7

Class VSubject Topic Summary Execution

Science Chapter 2 - The Skeletal System

Posture

The way in which we hold our body while sitting standing walking and lying down is known as posture

Importance of correct posture and exercise

1 Correct posture and exercise makes our bones and muscles strong 2 It provides our body more energy to do works

3 It keeps our body stressless 4 It also prevent back ache and muscle pain

Some important yoga asanas are-

1 Veerabhadrasana ( Warrior pose )

2 Dhanurasana ( Bow pose )

3 Trikonasana ( Triangle pose )

D Answer these questions

5How do our muscle work

Ans ndash Our muscles work by contracting and relaxing

6Why should we maintain a correct posture

Ans ndash We should maintain a correct posture because ndash

i Correct posture and exercise makes our bones and muscles strong

ii It provides our body more energy to do works

iii It keeps our

body stressles

s iv It also prevent

back ache and muscle pain

Social studies Conquering distances

Put a tick on the correct option1 b Ship 2b water transport 3 b trains 4 c 13 5 b Wright brothersTrue or false1 False 2 True 3 True 4 True

MATHEMATICS

Ch 3Addition and Subtraction

Exercise 126 A school needs Rs 4987653 for its building It has only Rs 3592468 in its accounts Estimate the money by rounding off to the nearest lakh it has still to raise

Solution Estimated amount need Rs 5000000 The school has only Rs 3600000 Still to raise Rs 1400000

MATHEMATICS

Ch 4

Mul

tiplic

ation

and

Div

ision

We have learnt multiplication tables up to 15 Let us extend the tables up to 20X 11 12 13 14 15 16 17 18 19 201 11 12 13 14 15 16 17 18 19 202 22 24 26 28 30 32 34 36 38 403 33 36 39 42 45 48 51 54 57 604 44 48 52 56 60 64 68 72 76 805 55 60 65 70 75 80 85 90 95 1006 66 72 78 84 90 96 102 108 114 1207 77 84 91 98 105 112 119 126 133 1408 88 96 104 112 120 128 136 144 152 1609 99 108 117 126 135 144 153 162 171 180

10 110 120 130 140 150 160 170 180 190 200Properties of Multiplication1 The product of two numbers does not change when the order of numbers is changed eg 503times23 = 23times503 [This property is called Commutative Property of multiplication]

2 The product of three numbers does not change when the grouping of numbers is change eg (15times18iquesttimes10=15 times(18times 10) = (15times10iquesttimes18 [This property is called Associative Property of multiplication]

3 The product of a number and 1 is the number itself eg 1513 times 1 = 1513 [This property is called Identity Property of multiplication and the integer 1 is called Identity Element of multiplication]

4 The product of a number and 0 is 0 eg 718205times 0 = 0 times 718205 = 05 The product of a number by the sum of two numbers is equal to the sum of the

products of that number by the two numbers separately eg123 times (105+ 48) = 123 times 105 + 123 times 48 [ This property is called Distributive Property of multiplication over

addition]English language

Transitive and intransitive verb

Pick out the verbs from the following sentences and say whether they are transitive or intransitive verb6 Was fullndash verb Intransitive verb7 Have been decorated- verb Transitive verb8 Happy cheerful ndash verb Intransitive verb9 Shall come back ndash verb Transitive verb

English 2 The fall of Lanka

This is the story of the fight between Rama and Ravana as told by Valmiki in lsquoThe Ramayana The monkeys worked all day and all night and at last built a bridge so that Rama and

Write the synonyms of

1 Stationed-

his army could cross to Lanka and rescue Sita assign2 Invade- enter

a country or a region so as to subjugate or occupy it

3 Prowess- bravery in battle

4 Haughty- arrogantly

5 Puny- small and weak

6 Spy- secret agent

7 Dreadful- causing or involving great suffering

8 Violate- disobey

9 Ghastly- causing great horror or fear

10 Deception ndash misleading

Class VISubject Topic Summary Execution

HISTORY AND CIVICS

CHAPTER 3

MAHAVIRA AND BUDDHA ndash GREAT PREACHERS

BUDDHA

Impact of Buddhism on Indian Cultures

Impact on Religion

Buddharsquos practical and simple doctrines made their impact on HinduismThe principle of ahimsaIt brought about a great change in the performance of costly yanjnas and sacrifices which previously involved immense loss of life The Mahayana Buddhists adopted the practice of worshipping Buddha and bodhisattvas making idols and erecting temples in their honour

Impact on Literature

After the death of Buddha his teachings were compiled and called TripitakasThe Jatakas contain tales dealing with the previous births of Buddha

Impact on Education

The Buddhist monasteries became great centres of learning These centres of learning developed into famous universities- Nalanda Taxila Vikramshila etc

Impact on Art and Architecture

The gateways and railing of the Sanchi Stupa were covered with sculptured figuresCave- temples were also constructed which were decorated with beautiful frescoes

1) How many parts of Tripitakas are thereAns -There are three parts of Tripitakas- Sutta Pitaka Vinaya PitakaAbhidhamma Pitaka

2)What are the subjects taught in these monasteries Ans ndash Buddhist scriptures logic Philosophy medicine astronomy etc

3) Which art was developed under Buddhist patronage Ans ndash Gandhara art

The Gandhara art was developed under Buddhist patronage

BIOLOGY The Leaf Photosynthesis The process by which green plants make their own food from carbon dioxide and water in the presence of sunlight and chlorophyll is called photosynthesis

All green plants need the following to make their food ndash

water carbon dioxide chlorophyll and energy in the form of sunlight

Carbon dioxide + water ------------- Glucose + oxygen

The end product of photosynthesis is glucose

Fill in the blanks

1 Plants make their food by the process of photosynthesis

2 The inner wall of the guard cell is thicker than the outer wall

3 The extra glucose is converted into starch and sucrose

4 The leaf is boiled in alcohol to remove chlorophyll

5 The rate of transpiration is more on the hot day then a cold day

6 Photosynthesis helps to observe water and minerals from the soil

English 1 Pronouns Kinds of pronouns 1 Personal pronouns2 Possessive pronouns3 Reflexive pronouns4 Interrogative pronouns5 Relative pronouns6 Demonstrative pronouns7 Indefinite pronouns

Personal pronouns they refer to first second and third person in sentences First person- the speakerSecond person-the listenerThird person-the objectperson being spoken aboutPersonal pronouns should have the same gender and number as the nouns they refer to

Possessive pronouns these are used to indicate the relationship between the objects and people These pronouns include mine ours yours his hersand theirs

ExerciseBFill in the blanks with suitable pronounsThere was much excitement among the childrenTheywere eagerly looking forward to the annual picniclsquoAre they going to Lodhi Gardens toorsquo wondered AneeshlsquoNo they are going to Buddh Jayanti Park with Mrs Jain said Mrs ChopraThe children looked disappointedlsquoWonrsquot you be taking us Marsquoam rsquo they askedlsquoSorry children I have to go to Mumbai for a week to look after my sick mother But you will have fun with Mrs Jain she is full of laughter and you will love being with her the whole daylsquoIt will not be the same they grumbled

English 2 The great train journey- Ruskin Bond

The great journey by Ruskin Bond is a story about Suraj who loved trains and wanted to go to places One day while wandering along the railway tracks he enters into a carriage compartment The train suddenly starts moving with him in the compartment and after a journey returns back to the same place from where it had begun The story is about his experience during that journey

State true or false1 When the train had passed leaving behind the

hot empty track Suraj was lonely2 It was winter holidays

and Suraj did not know what to do with himself

3 He plunged his hands into the straw and pulled out an apple

4 A dirty bearded face was looking out at him from behind a pile of crates

5 Suraj wanted to go to Japan

Hindi 2nd lang

गललबाजलडका खालीसथानोकोभरो-6 गो-ामसनिनकलकरहमगराजमआगए7 माबोधराजकोराकषससमझतीथी8 चीलरोशन-ानमसअ-रआकरतहसीलपरबठगई9 तीनचारतीनकऔररईकगोलउडलनिकनघोसलानहीनिगरा10 वहसवयतोघोसलातोडनककतिलएगललउठालायाथा

11 -ीवारकसाथलगतगोहपजोकसहार-ीवारपकडलतीह12 बोधराजअभीभीटकटकीबाधचीलकीओर-खरहाथा13 बोधराजअपनीजबमबहतसाचगगाभरकरलायाथा14 मरनिपताजीकीतरककीहईऔरहमलोगएकबडघरमजाकररहनलग15 बागमजातातोफलपरबठीनिततलीको-खनिततलीकोपकडकरउगकतिलयोकबीचमसल-ता

BENGALI(2ND LANGUAGE)

সহিনধসবরপওসবরসহিনধ

সবরসহিনধরহিনয়ম- ৯ই-কারহিকংাঈ-কাচেররপচেরইাঈহিভননঅনযসবররণCথাকচেইাঈসথাদেনয-ফায়এংওইয- ফাপCচেরণCযকতয়

১০উ-কারহিকংাঊ-কাচেররপচেরইাঈহিভননঅনযসবররণCথাকচেউাঊসথাদেন-ফায়এংওই- ফাপCচেরণCযকতয়

১১ঋ- কাচেররপচেরঋহিভননঅনযসবররণCথাকচেঋসথাচেনর -ফায়এংওইর পCচেরণCযকতয়

১২সবররণCপচেরথাকচেপCতMএ-সথাচেনঅয় ঐ- সথাচেনআয় ও- সথাচেনঅএংঔ- সথাচেনঅায়

৯ই+ অ= য- ফাআহি+ অনত= আযনত অহিধ+ অয়ন= অধযয়নই+ আ=য- ফা+ াইহিত+ আহি= ইতযাহি পরহিত+ আতC ন= পরতযাতC নই+ উ=য- ফা+ উঅহিত+ উহিকত= অতযহিকত হি+ উৎপহিতত= যৎপহিততই+ ঊ= য- ফা+ ঊ ই+ এ= য- ফা+ এঈ+ অ= য- ফা পরহিত+ ঊ4= পরতয4 পরহিত+ এক= পরচেতযকঈ+ অ আ= য- ফা+ অ আনী+ অমব= নযমব মসী+ আধার= মসযাধার

১০উ+ অ= অন+ অয়= অনবয় পশ+ অধম= পশবধমউ+ আ= াস+ আগত= সবাগত পশ+ আহি= পশবাহিউ+ ঈ= হিঅন+ ইত= অহিনবতউ+ এ= দেঅন+ এ4রণ= অচেনব4রণউ+ ঈ= ীসাধ+ ঈ= সবাধবী তন+ ঈ= তনবী

১১ঋ+ অ= র মাত+ অনমহিত= মাতরনমহিতঋ+ আ= রা হিপত+ আয়= হিপতরায়ঋ+ ই= হির মাত+ ইচছা= মাতচছাঋ+ ঈ= রী ধাত+ ঈ= ধাতরীঋ+ উ= র ভরাত+ উপচে(= ভরাতরপচে(

১২এ+ অ= অয় দেন+ অন= নয়নঐ+ অ= আয় গৈগ+ অক= গায়কও+ অ= অ দেপা+ অন= পনও+ ই= অ দেপা+ ইতর= পহিতরও+ এ= অ দেগা+ এ4রণা= গচে4রণাঔ+ অ= অা দেপৌ+ অক= পাকঔ+ ই= অা দেনৌ+ ইক= নাহিকঔ+ উ= অা দেভৌ+ উক= ভাক

MATHS Topic NumbersChapter Natural numbers and whole numbers

Study item properties of whole numbers for multiplication

1 Closure property If x and y are two whole numbers then xtimesy is also a whole numberExample If x = 9 and y =3 then xtimesy = 9times3 = 27 which is a whole number

2 Commutative property If x and y are two whole numbers then xtimesy = ytimesxExample If x = 5 and y = 2 then xtimesy = 5times2 = 10y times x = 2times5 = 10Therefore 5times2 = 2times5

3 Associative property If x y and z are three whole numbers then x times(ytimesz) = (xtimesy) times zExample If x =3 y = 5 and z = 7 then 3 times (5times7) = 3 times (35) = 105And (3times5) times7 = (15) times 7 = 105Therefore x times (ytimesz) = (xtimesy) timesz

4 Distributive property If x y and z are three whole numbers then xtimes (y + z) = x times y + x times z

Therefore the multiplication of whole numbers is distributive over their additionExample If x = 5 y = 3 and z= 2Therefore x times (y + z) = 5 times (3 + 2) = 5times5 =25And x times y + xtimes z = 5times3 +5times2 =15 +10 = 25Again x times (y ndash z ) = x times y ndash x timesz Therefore 5 times ( 3 - 2) = 5 times1 = 5 and 5times3 ndash 5 times2 = 15 ndash 10 = 5Therefore the multiplication of whole numbers is also distributive over their subtraction if y is greater than z

5 Existence of identity If x is a whole number then

X times1 = x 1 times x = xTherefore we can write x times1 = 1 times xTherefore the multiplication of any whole number with 1 is the number itselfTherefore we can say that 1 is multiplicative identity or identity element for multiplicationExample 5 times1 = 5 1 times 5 = 5 Therefore 5 times 1 = 5

6 Multiplicative inverse If x is any whole number ( x is not equal to zero ) then its multiplicative inverse will be 1xSo x times 1x = 1 but 1x is a whole number if x = 1For other values of whole number 1x is not a whole number therefore we can write its multiplicative inverse does not exists

7 Cancellation law of multiplication If x y and z are three non- zero whole numbers then x times y = x times z

Or y = zExample 9 times y = 9 timeszTherefore y = z

Class VIISubject Topic Summary Execution

English 2 Sentences based on meanings

Kinds of sentences

Assertive or declarative to convey information or simply make a statement

Interrogative to ask different types of questions

Imperative to command or instruct someone or make a request

Exclamatory to express strong feelings and emotions

Exercise c1 What a nice compliment that is

That is a nice compliment2 How well- behaved the children

areThe children are very well-behaved

3 What great chefs we areWe are great chefs

4 What a shame it isIt is a shame

5 What a fantastic idea you haveYou have a fantastic idea

Homework 6 -10English

LiteratureThe Listeners III) Answer the following questions-

d) Identify two words used in the poem to give the poem an eerie atmosphereAns- Two words used to give the poem an eerie atmosphere are ldquogreyrdquo and ldquophantomrdquo

e) Who do you think are the inmates of the houseAns- I think the inmates of the house are phantom who once used to dwell in it

f) Why was the poet ldquoperplexed and stillrdquoAns- He was lsquo perplexed and stillrsquo because he was expecting an answer from the inmates of the house But despite of repeated calls there was no response

CHEMISTRY Chapter 2 ndashElement and Compound

Activity Series of MetalsThe activity series is a chart of metals listed in order of declining relative reactivity The top metals are more reactive than the metals on the bottomMetal SymbolReactivity

Lithium Li displaces H2 gas from water steam and acids and forms hydroxides

Potassium K

Strontium Sr

Calcium Ca

Sodium Na

Magnesium Mg displaces H2 gas from steam and acids and forms hydroxides

Aluminum Al

Zinc Zn

Chromium Cr

Iron Fe displaces H2 gas from acids only and forms hydroxides

Cadmium Cd

Cobalt Co

Nickel Ni

Tin Sn

Lead Pb

Hydrogen gas

H2 included for comparison

Antimony Sb combines with O2 to form oxides and cannot displace H2

Arsenic As

Bismuth Bi

Copper Cu

Mercury Hg found free in nature oxides decompose with heating

Silver Ag

Palladium Pd

Platinum Pt

Gold Au

Answer the following Q)Difference Between Metals And Nonmetals

Metals Nonmetals

These are solids at room temperature except mercury

These exist in all three states

These are very hard except sodium

These are soft except diamond

These are malleable and ductile

These are brittle and can be breakdown into pieces

These are shiny These are non-lustrous except iodine

Electropositive in nature Electronegative in nature

Have high densities Have low densities

Math Number System

Chapter Fraction

Study item Some solved sums from exercise 3(B)1) For each pair given below state whether it from like fractions or unlike

fractions (i) 58 and 78

= Like Fraction because denominators same(ii) 815 and 821

= Unlike Fraction because denominators are not same

(iii) 49 and 94 = Unlike Fraction

2) Convert given fractions into fractions with equal denominators(iii) 45 1720 2340 and 1116Solution Given fraction 45 1720 2340 and 1116Therefore the LCM of 5 20 40 and 16 is 80Therefore 45 = 4times165times16 = 64801720 = 17times420times4 = 68802340 = 23times240times2 = 4680 1116 = 11times516times5 = 5580

3) Convert given fractions into fractions with equal numerators(iii) 1519 2528 911 and 4547Solution Given fractions 1519 2528 911 and 4547Therefore the LCM of 15 25 9 and 45 is 2251519 = 15times1519times15 = 225285 2528 = 25times928times9 = 225252911 = 9times2511times25 = 2252754547 = 45times547times5 = 225235

4) Put the given fractions in ascending order by making denominators equal

(iii) 57 38 914 and 2021Solution Given fraction 57 38 914 and 2021Therefore the LCM of the denominators is 16857 = 5times247times24 = 12016838 = 3times218times21 = 63168914 = 9times1214times12 = 1081682021 = 20times821times8 = 160168Therefore ascending order 63168lt108168lt120168lt160168Therefore ascending order of given fractions38lt914lt57lt2021

COMPUTER CHAPTER-1COMPUTER FUNDAMENTALS

DONE IN THE PREVIOUS CLASSES PAGE 10CWRITE TRUE AND FALSE

1 True2 False3 False4 False5 True

GEOGRAPHY CHAPTER 7EUROPE

CHAPTER COMPLETE 1)Europe is home to a famous mountain range called the Alps

2)River Rhine originates in Switzerland

3)The Eiffel Tower one of the tallest structures in Europe

4) Vatican City is one of the most densely populated European countries

5)Sognefjordin Norway is the largest fjord in Norway

Class VIIISubject Topic Summary Execution

MATHEMATICS Ch 6Sets

Exercise 6 (D)1 Given A = x x isin N and 3iquest x le 6 and B = x x isin W and xlt4 find (i) Sets A and B in roster form (ii) A cup B (iii)

A cap B(iv) A ndash B (v) B ndashA

Solution (i) A = 456 and B = 0123

(ii) A cup B = 0123456 (iii) A cap B = ϕ (iv) A ndash B = 456 (v)B ndash A = 0123

3 If A = 56789 B = x 3 lt x lt 8 and x isin W and C = x xle5 and x isin N Find (i) A cup B and (A cup B) cup C (ii) B

cup C and A cup ( B cup C)

(iii) A cap B and (A cap B) cap C (iv) B cap C and A cap (B cap C)

Is (A cup B) cup C = A cup (B cup C)

Is (A cap B) cap C = A cap (B cap C)

SolutionA = 56789 B = 4567 C = 12345

there4 (i) A cap B = 456789 and (A cup B) cup C = 123456789

(ii) B cup C = 1234567 and A cup ( B cup C) = 123456789

(iii) A cap B = 567 and (A cap B) cap C = 5

(iv) B cap C = 45 and A cap (B cap C) = 5

Now (A cup B) cup C = 123456789

And A cup ( B cup C) = 123456789 there4 (A cup B) cup C = A cup (B cup C)

Again (A cap B) cap C = 5 and A cap (B cap C) = 5

there4 (A cap B) cap C = A cap (B cap C)

4 Given A = 012345 B = 02468 and C = 0369 Show that (i) A cup (B cup C) = (A cup B) cup C ie the union

of sets is associative (ii) A cap (B cap C) = (A cap B) cap C ie the intersection of sets is associative

SolutionNow B cup C = 0234689 and A cup B = 01234568

there4 A cup (B cup C) = 012345689 and

(A cup B) cup C = 012345689

So (i) A cup (B cup C) = (A cup B) cup C ie the union of sets is associative

Again B cap C = 06 and A cap B = 024

there4 A cap (B cap C) = 0 and (A cap B) cap C = 0

So (ii) A cap (B cap C) = (A cap B) cap C ie the intersection of sets is associative

Physics Chapter 2 Physical Quatites and Measurements

Here We Will Do Some QuestionsRelated To Chapter 2

A density bottle has a marking 25 mL on it It means that

1 the mass of density bottle is 25g

2 the density bottle will store 25 ml of any liquid in it

3 the density bottle will store 25 ml of water but more volume of liquid denser than water

4 the density bottle will store 25 ml of water but more volume of a liquid lighter than water

Solution 2 the density bottle will store 25 ml of any liquid in it

COMPUTER CHAPTER-2Spreadsheet Functions and Charts

SELECTING RANGE IN ROWSCOLUMNSWHEN TWO OR MORE CELLS ARE SELECTED IT IS CALLED A RANGEA RANGE OF CELLS CAN BE FORMED IN TWO WAYS--a) SELECTING RANGE BY USING THE MOUSEb) SELECTING RANGE BY USING THE KEYBOARD

Q1)WRITE THE STEPS TO SELECT PARTIAL RANGE IN A ROW

Ans)THE STEPS ARE-6 SELECT THE ROW7 BRING THE CELL POINTER TO THE DESIRED

LOCATION FROM WHERE YOU WANT TO START YOUR SELECTION

8 CLICK THE LEFT MOUSE BUTTON AND KEEP DRAGGING TO YOUR RIGHT TILL YOU REACH THE LAST CELL TO NE SELECTED

RELEASE THE MOUSE BUTTON

GEOGRAPHY Asia

CLIMATE

Asia experiences great extremes of climate Jacobabad in the Sind province of Pakistan is one of the hottest places in the WorldVerkhoyansk in Siberia is one of the coldest places in the WorldCherrapunji and Mawsynram in India are two wettest places in WorldArabia Tibet Gobi and Mongolia are extremely dry regionsFactors Affecting Climate of Asia-The factors influencing the climate of Asia are-

Factors Affecting Climate of Asia-Thoroughly read the table in page number 60

Latitudinal extent

Continentality

Relief features

Presence of low pressure trough

Jet streams

English Language The Sentence A complex sentence contains one independent clause and at least one dependent clause The dependent clause in a complex sentence is introduced with subordinating conjunctions or relative pronouns

Commonly Used Subordinating Conjunctions-Time after before while when since untilCause And Effect because now since as in order that soOpposition although though even though whereas while in spite ofCondition if unless only if whether or not even if in case(that)

Commonly Used Relative Pronouns-Who whose whom which whoever whomever whichever that

Class IXSubject Topic Summary Execution

1-BENGALI(2ND LANGUAGE)

ldquo বঙগভমিরপরমিrdquo াইকেলধসদনদতত

আচেগর পর উততর পচো-১ ২ ৩ এং নীচের পর টি াহির কা- ৪মহিbকাও গচেনা দেগা পহিচে অমত হরচে- ক) কার দো দেকান কহিতার অং( ) কতা দেক পরসঙগ কী উহিকতটির তাৎপযC আচোনা কচেরা৫দেসই ধনয নরকচে দোচেক যাচে নাহি ভচে মচেনর মহিeচের সাচেসচে সCন ক) কহির কায C ার উচেf(য হিক হিছ কহি কন কহিতাটি দেচেন) কহি কার কাচেছ হিমনহিত কচেরচেছনগ) কহি এই পহিথীচেত কাচের ধনয মচেন কচেরনঘ) কহি হিক রকম অমর তাাভ করচেত ান

Hindi 2nd lang

काकी(कतिसयारामशरण गपत)

इस कहानी म लखक न यह बतान का परयास निकया ह निक बचच अपनी मा स निकतना परम करत ह शयाम अबोध बालक ह वह अपनी मा क मरन क बा- उसन अपनी मा क कतिलए बहत रोया बा- म उस पता चला निक उसकी मा राम क घर चली गई ह आकाश म उडती हई पतग -खकर उस हरष हआ निक पतग क दवारा वह अपनी मा को नीच उतारगा इसक कतिलए वह अपनी निपता की जब स -ो बार सवा रपया निनकालकर पतग और -ो मोटी सी मन वाली अपन भाई स काकी एक कागज पर कतिलखवा कर पतग म कतिशव का दि-यानिनकालकर पतग और -ो मोटी सी मन वाली अपन भाई स काकी एक कागज पर कतिलखवा कर पतग म कतिचपका दि-याभोला और शयाम कोठरी म रससी बाधनी रह थ तभी उसक निपता करोध म आकर उन स पछ निक कया उनकी जब स रपया निनकाला हभोला डर क मार बताया निक शयाम इस पतग क दवारा अपनी काकी को राम क यहा स उतारना चाहता हनिवशशवर(शयाम क निपता)न फटी पतग उठाकर -खी तो उस पर काकी कतिलखा थावह हत बजिa होकर वही खड रह गएउनहोन सोचा निक मन अपन पतर को मारा जोनिक अनजान और निन-dरष थावह अपनी मा कोनिकतना पयार करता ह

उस दि-न बड सवर शयाम की नी- खली तो -खा निक घर भर म कोहराम मचा हआ ह

क) घर म कोहराम कयो मचा हआ था शयाम को कया लगा

ख) काकी को ल जात समय शयाम न कया उपदरव मचाया

ग) काकी क बार म उस कया बताया गया कया सतय उस कतिछपा रहा

घ) वह बठा-बठा शनय मन स आकाश की ओर कयोकरता

उततरक) शयाम की मा का -हात हो गया था इसकतिलए

घर म कोहराम मचा हआ था शयाम की लगा निक उसकी मा सफ- कपडा ओढ हए भमिम पर सो रही ह

ख) लोग जब उमा यानी शयाम की मा को उठाकर ल जान लग तब शयाम न बडा उपदरव मचाया लोगो क हाथ स झठ करवा उमा क ऊपर जा निगरा और बोला काकी सो रही ह उस कहा ल जा रह हो

ग) काकी क बार म बजिaमान लोगो न उस निवशवास दि-लाया निक उसकी का निक उसक मामा क यहा गई ह लनिकन सतय अमिधक दि-नो तक कतिछपाना रह सका आसपास क अबोध बालको क मह स यह बात परकट हो गई निक उसकी मा का -हात हो गया ह

घ) कई दि-नकई दि-न लगातार रोत-रोत उसका रोना तो शान हो गया पर उसक ह-य म शोक भर गया था वह चपचाप बठा आकाश की और टाका करता निक शाय- उसकी काकी कही दि-ख जाए

ldquoदि-न उसन ऊपर आसमान म पतग उडती -खी न जान कया सोच कर उसका निहर-य एक-म खिखल उठाrdquo

क) निकसन पतग ऊपर उडत -खी और वह कयो खश हआ

ख) उसन अपन निपता स कया कहा उनका कया उतर थाश

ग) उसन निफर कया निकया और निकसन उसकी सहायता की

घ) उसकी योजना कया थी उततर -क) शयाम न एक दि-न आसमान म पतग उडती

-खी तो उसन सोचा निक पता आसमान म राम क यहा जाकर रकगी वही पर मरी काकी ह यह सोचकर वह बहत खश हआ

ख) उसन अपन निपता स कहा काका मझ एक पतग मगा -ो उसक निपता न भटक हए मन क भाव स कहा निक मगा -ग यह कह कर उ-ास भाव स वह कही और चल गए पतग नही आई

ग) उसन चपचाप निवशशवर क टगहए कोट स एक चवननी निनकाल ली और सखिखया -ासी क लडक भोला की सहायता स एक पतग मगवानी भोला उसकी बराबर उमर का ही था

घ) उसकी योजना यह थी निक वह अपनी पतग को आकाश म राम क यहा भजगा और उस पतग क सहार उसकी काफी नीच उतर जाएगी इस योजना पर उस परा निवशवास था इसकतिलए वह और भोला -ोनो यह काम करन म लग गए

Continue to nexthelliphellipEVS CHAPTER - 1

(UNDERSTANDING OUR ENVIRONMENT)

Sustainable development

The development that meets the needs of the present without compromising the ability of future generations to meet their own needs is called Sustainable development

Sustainable societies ndash

An environmentally sustainable community is one that meets the current and future basic resource needs of its people in a just and equitable manner without compromising the ability of future generations to meet their basic needs

Q ) What are Eco Villages

Ans - Eco village are the urban or rural communities of people who strive to integrate a supportive social environment with a low impact way of life

Q ) To ensure sustainable development the depletion of renewable resources should not take place at a rate faster than their regeneration Justify your answer

Ans ndash Renewable resources do not have a fixed quantity - more can always be

generated However if the rate of use exceeds the rate of renewal - that is the

source is used more than its being recreated - its continued use will become

used up faster than it can regenerate

To promote sustainable society the following things need to be done ndash

1 Using renewable energy sources 2 By improving the quality of human

health 3 By promoting sustainable agriculture 4 By forming ecovillage

it will eventually be entirely depleted So Toensure sustainable development the depletion of

renewable resources should nottake place at a rate faster than their regeneration

Q ) What do you mean by Sustainable societies

Ans - Sustainable societies are defined as towns and cities that have taken steps to remain healthy over the long term These communities value healthy ecosystems use resources efficiently and actively seek to retain and enhance a locally based economy Sustainable development concerns everybody in a society

Q ) What are the effects of pollution on human health

Ans ndash Some health problem occurs due to air pollution are ndash

Respiratory diseases Cardiovascular damage Fatigue headaches and anxiety Irritation of the eyes nose and throat Damage to reproductive organs Harm to the liver spleen and blood Nervous system damage

Some health problem occurs due to water pollution are ndash

Typhoid Cholera Dysentry Jaundice

Some health problem occurs due to noise pollution are ndash

Fatigue headaches and anxiety High blood pressure Hearing damage

Physics Motion in 1D First go through previous notes Now here we will solve some numerical related to that

Question 3What information about the motion of a body is obtained from the displacement-time graphSolution 3From displacement-time graph the nature of motion (or state of rest) can be understood The slope of this graph gives the value of velocity of the body at any instant of time using which the velocity-time graph can also be drawn

Question 4(a)What does the slope of a displacement-time graph represent(b)Can displacement-time sketch be parallel to the displacement axis Give a reason to your answerSolution 4(a) Slope of a displacement-time graph represents velocity(b) The displacement-time graph can never be parallel to the displacement axis because such a line would mean that the distance covered by the body in a certain direction increases without any

increase in time which is not possible

Chemistry Language of Chemistry

How to balance a chemical equationThere are two methods of balancing an equation(i)Hit and trial method(ii)Partial equation methodBalancing by hit and trial methodThis method consists of counting the number of atoms of each elements on both sides and trying to equalize themTake the following steps(i)Count the number of times (frequency) an element occurs on either side(ii)The element with the least frequency of occurrence is balanced first(iii)When two or more elements have the same frequencythe metallic element is balanced firstExample-1 On heatinglead nitrate decomposes to give lead dioxidenitrogen dioxide and oxygenPb(NO3)2rarrPbO+NO2+O2

In this equationLead occurs twiceNitrogen occurs twiceOxygen occurs four timesSince lead is a metalbalance it firstThe number of atom of lead is equal on the two sidestherefore it needs no balancingNow balance nitrogenOn the reactant sidethere are two atoms of nitrogenwhile on the product side oneSomultiply the product containing nitrogenon the product sideby two Pb (NO3)2rarrPbO+2NO2+O2Nowthe number of oxygen atoms on the reactant side 6while on the product sideit is 7Somultiply the entire equation by 2except oxygen to get balanced equation2Pb(NO3)2rarr2PbO+4NO2+O2Multiplication by 2 is done only when atoms of all the elements except one element are balanced and the unbalanced atom occurs separately at least once and also there is a difference of only one such atom

Math Topic AlgebraChapter

Factorisation

Study item Difference of two squares a2 ndash b2 = (a+b) (a-b)1) (i) 4x2ndash 25y2

= (2x) 2 ndash (5y) 2= (2x + 5y) (2x - 5y)

(ii) 9x2 ndash 1= (3x)2ndash(1)2= (3x + 1)(3x ndash 1)

2) (i) 150 ndash 6a2= 6(25 ndash a2)= 6(5)2 ndash(a)2= 6 (5 + a) (5 ndash a)

(ii) 32x2 ndash 18y2=2(16x2 ndash 9y2)=2(4x)2 ndash (3y)2= 2(4x + 3y)(4x - 3y)3)(i) (x ndashy )2 ndash 9 = (x ndash y )2 ndash (3)2= (x ndash y + 3) (x ndash y ndash 3)(ii) 9(x + y) 2ndash x 2= (3)2(x + y)2 ndash (x)2=3(x + y)2 ndash (x)2= (3x +3y ) 2ndash(x)2= (3x + 3y + x)(3x +3y ndash x)= (4x + 3y) ( 2x + 3y )

Commercial studies

Basic accounting terms

Today I will give you some questions from the previous study material

Questions1) Define accounting2) What do you mean by debit and

credit

3) Explain the types of account4) Define the following terms

a) Assetsb) Capitalc) Purchased) Debtorse) Transactions

5) Name the types of accounts given below

a) Krishnas accountb) Machinery accountc) Royalty accountd) Salary accounte) Furniture accountf) Audit fee account

Economics Revision Today I will give you some revision questions

Questions1) What do you mean by the terms

rdquowantsrdquo2) Write the difference between

consumer goods and producer goods

3) Define the term utility 4) Explain the different types of utility5) Define

a) Total utilityb) Marginal utility

Subject Eng Literature (The Merchant of Venice ndash William Shakespeare)Topic Act I Scene 3 Lines 1 to 48 (Shylock hellip Cursed be my tribe if I forgive him) Date 16th April 2020 (5th Period)

[Students should read the original play and also the paraphrase given in the school prescribed textbook]Summary Questions amp Answers

This scene takes place in Venice and we are introduced to the rich Jew Shylock Bassanio and Shylock are talking and Bassanio tells Shylock that he wants a loan of three thousand ducats for three months on the personal security of Antonio

o Shylock feels glad because he will be able to bind down Antonio by means of a bond on account of the loan but he tells Bassanio that all the fortunes of Antonio being invested in the merchant ships on the sea it is difficult to depend upon his credit Even under such circumstances Shylock is willing to advance the money on the personal security of Antonio

o Bassanio then invites Shylock to dine with him Shylock says that he is prepared to do anything with the Christians but not eat or drink or pray with them

o While Bassanio and Shylock are talking Antonio appears on the scene Shylock does not seem to take any notice of Antonio but goes on brooding within

(1) SHYLOCK Ho no no no no- my meaning in (Line 15-26)saying he is a good man is to have you understand me that he is sufficient Yet his means are in suppositionhe hath an argosy bound to Tripolis another to the Indies I understand moreover upon the Rialto he hath a third at Mexico a fourth for England and other ventures he hath squanderd abroad Butships are but boards sailors but men there be land-rats and water-rats land-thieves and water-thieves I mean pirates and then there is the peril of waters winds and rocks The man is notwithstanding sufficientmdashthree thousand ducats mdashI think I may take his bond

(a) Who is talking in the beginning of this scene What does Bassanio want from Shylock How does Shylock feel

In the beginning of the scene Bassanio and Shylock are talking to each other Bassanio wants to borrow three thousand ducats from Shylock for three months on the security of Antonio Shylock feels glad at heart that he will get the opportunity of binding Antonio with a bond(b) What risks does Shylock weigh in advancing the money

Shylock says that Antonio has invested all his capital in trading by sea-going ships But the ships are made of wood and the sailors of those ships are ordinary human beings The wood can

himself how he hates Antonio because of his being a Christian because he abuses Shylock in public places Shylock decides that if ever he can get Antonio to his advantage he will teach him a lesson

come to harm and men can commit mistakes and thus the capital invested in ships may be lost Then there are other dangers The goods loaded on the ships can be damaged by rats and thieves which are found both on land and water The ships can also be harmed through sea-storms submerged rocks etc(c) What two important functions does this scene have

The scene has two important functions First it completes the exposition of the two major plot lines of the play Antonio agrees to Shylockrsquos bond ndash three thousand ducats for a pound of flesh and second and more important dramatically this scene introduces Shylock himself In this scene Shakespeare makes it clear at once why Shylock is the most powerful dramatic figure in the play and why so many great actors have regarded this part as one of the most rewarding roles in all Shakespearean dramas(d) Where does this scene take place What kind of treatment has Antonio been giving to Shylock What does Shylock say when Bassanio invites him to dine with him

The action of this scene takes place in Venice Antonio has been in the habit of behaving harshly with Shylock ndash spitting on his beard and footing him like a stranger cur When Bassanio invites Shylock to dine Shylock says that he is prepared to do anything with the Christians but not eat and drink or pray with them

(2) SHYLOCK How like a fawning publican he looks (Line 38-48)I hate him for he is a Christian

But more for that in low simplicity

He lends out money gratis and brings downThe rate of usance here with us in VeniceIf I can catch him once upon the hipI will feed fat the ancient grudge I bear him

He hates our sacred nation and he railsEven there where merchants most do congregateOn me my bargains and my well-won thriftWhich he calls interest Cursed be my tribeIf I forgive him

(a) What is the context in which these words are spoken and what is the idea expressed in it

These remarks are made by Shylock when he sees Antonio coming along after Bassanio told him that the merchant will be his surety for the bond The above mentioned passage reveals Shylockrsquos hatred for Antonio Shylock says that he hates Antonio because he is a Christian and also because he gives loan without taking interest on them thereby bringing down the rate of interest in Venice(b) Explain the meaning of the phrase lsquoa fawning publicanrsquo

The phrase lsquoa fawning publicanrsquo refers to Roman tax collector It is a term of contempt and hatred on the lips of a Jew lsquoFawning Publicansrsquo were Roman tax-gatherers whose ordinary bearings towards the Jews was bullying but whose false pose of lsquohumility and contritionrsquo is touched upon in the parable in New Testament(c ) What light does the above passage throw on the character

of Shylock

The above mentioned speech of Shylock reveals him to be a wicked character having an extreme greed for wealth His intense hatred for Antonio is unjustified He hates Antonio only because he is a Christian and because he lends money without taking any interest on it thereby adversely affecting Shylockrsquos business of lending money on high interest(d) What information do you gather about Antonio from the above given lines

Shylockrsquos statement throws a valuable light on the character of Antonio Antonio appears to be a good Christian and a good human being He helps the people in need by lending them money without charging any interest on it He is a man of simple and good nature This very goodness makes him Shylockrsquos enemy(e) What does Shylock debate within himself and when To whom are the lines mentioned above addressed to

When Bassanio asks the Jew to lend him three thousand ducats on Antoniorsquos surety Shylock begins to debate within himself as to how he should exploit the opportunity of a business deal with his old enemy Antonio

The lines mentioned above are not addressed to anyone The lines are a soliloquy ie a speech made by a character to himself and not meant to be heard by the other characters present

Class XSubject Topic Summary ExecutionEnglish

LiteratureThe Blue Bead 2nd part

Things took a turn for the worst and all of a sudden a crocodile attacked the woman biting on the womanrsquos leg At that moment Sibia got up sprinted grabbed the hay fork and stabbed the crocodile in the eye with all her power Immediately the crocodile let go and went away Sibia saw a small blue bead lying by the river she grabbed it Since she was poor she didnrsquot have necklace Shersquod always wanted one like the other women now she could make one with the blue bead After that she went home and told her mother all about it

Hindi 2nd

Langबड घर की बटी( मशी परमच-)

lsquoबड घर की बटी कहानी का उददशय मधयम वग की घरल समसया को सलझा कर सगदिठत परिरवार म मिमल जलकर परम स रहन का स-श -ना हघर म शानित सथानिपत करन की जिजमम-ारी नारी की होती ह यदि- नारी समझ-ार ह उसम धय और परिरवार क परनित परम ह तो कोई भी घटना परिरवार को निवघदिटत नही कर सकती या कहानी परिरवार को सगदिठत करत हए परम सौहा- स एक दसर की भावनाओ को समझ करउनका सहयोग करत हए जीवन यापन करन की पररणा -ती हमशी परमचदर जी न इस कहानी म सय परिरवार का परनितनिनमिधतव निकया ह यह कहानी बनी माधव सिसह जो गौरी पर क जमी-ार क उनक -ो पतरो की हशरी कठ लाल निबहारीशरीकात का निववाह एकजमी-ार घरान की पतरी आन-ी स हआ थाआन-ी न ख- को ससराल क वातावरण म ढाकतिलया थाएक दि-न आन-ी का अपन -वर लाल निबहारी स झगडा हो जाता ह -ोनो भाई एक दसर स अलग होन की कोकतिशश करत हसभी बह आन-ी न अपन मधर वयवहार स लाल निबहारी को

ldquoइन नतर निपरय गणो को बीए-इनही -ो अकषर पर नयोछावर कर दि-या था इन -ो अकषर न उनक शरीर को निनबल और चहर को कानित ही बना दि-या थाldquo

क) परसतत पकतियो म निकस वयकति क बार म कहा गया ह

ख) इन पकतियो म कौन स नतर निपरय गणो क बार म कहा गया ह

ग) बीए की निडगरी परापत कर लन पर भी उपय वयकति क सवभाव की कया निवशरषता थी

घ) यह नतर निपरय गण निकस वयकति म निवदयमान थ उसक वयकतितव की कया निवशरषता थी

उततर ndashक) परसतत पकति म गौरी पर गाव क जमी-ार

क बड बट शरीकात क बार म कहा गया ह उसन बहत परिरशरम और उ-म क बा- ba की निडगरी परापत की थी अब वह एक -फतर

घर छोडकर जान स रोक कतिलयाइस पर बनी माधव सिसह न कहा निक बड घर की बटी ऐसी ही होती ह जो निबगडा काम बना लती ह अतः शीरषक साथक ह बड घर की बटी आन-ी ह

म कमचारी थाख) भरा हआ चहरा चौडी छाती और डटकर

खाना आदि- एक सबजी ल जवान क गण मान जात ह परत शरीकात न इनही नतर निपरय गणो को अपनी पढाई पर नयोछावर कर दि-या था

ग) बीए की निडगरी परापत कर लन पर भी उपय वयकति(शरी कठ की शारिररिरक तौर पर निनबल और चहर स कानित ही लगत थ इतना ही नही वह मानकतिसक तौर पर भी निपछड हए थ पाशचातय सामाजिजक कथा उस घणा एव पराचीन सभयता का गणगान उनकी निवचारधारा क परमख अग थ

घ) यह नतर निपरय गण गौरीपर गाव क जमी-ार क छोट बट लाल निबहारी सिसह म निवदयमान थ वह सजीलाजवान था और भस का दध शर दध वह सवर उठकर पी जाता था

ldquoयही कारण था निक गाव की लललन आए उनकी निन-क थी कोई कोई तो उह अपना शतर समझन म भी सकोच ना करती थी सवय उनकी पतनी को इस निवरषय म उनस निवरोध थाldquo

क) उपय पकति म इस वयकति क बार म कहा गया ह

ख) गाव की लललन आए उनकी निन-ा कयो निकया करती थी

ग) उनकी पतनी का कया नाम था उनह निकस निवरषय म अपन पनित क निवरa था और कयो

घ) इस कहानी का कया उददशय ह Continue to next helliphelliphellip

Bengali 2nd Language

ফ ফটক না ফটক( কহিতা )

পর) ldquo(ান াধাচেনা ফটপাচেথ পাথচের পাড হিচেয় এক কাঠচোটটা গাছ কহিকহি পাতায় পার ফাটিচেয় াসচেছldquoক) কার দো দেকান কহিতার অং( ) lsquo(ান াধাচেনা ফটপাচেথ পাথচের পাডহিচেয়lsquo চেত কী দোঝাচেনা চেয়চেছ গ) আচো য অংচে(lsquo এক কাঠচোটটা গাছ lsquoচেত কী দোঝাচেনা চেয়চেছ ঘ) ldquoকহিকহি পাতায় পার ফাটিচেয় াসচেছldquo ----- একথার পরকত অথC কী উততর ) ক) আচো য অং(টি পর যাত কহি সভা4 মচোপাধ যাচেয়র দো lsquoফ ফটক না ফটকrsquo কহিতার অং()কহি সভা4 মচোপাধ যায় হিছচেন দেপরচেমর কহি দেপরমচেক নানা ভহিঙগমায় হিতহিন ফটিচেয় তচেচেছন দেপরম মানচের স মচেতC র সঙগী কহিতার কহিতায় এক রb সb হচেয়র দেপরম াগরচেনর কথা চেচেছন (ান অথCাৎ দেযাচেন দেকান রস দেনই দেযাচেন দেকান মহিনতা দেনই অথ তার মধ দেযও দেপরম থাকচেত পাচের একথাই কহি তচে ধরচেত দেচেয়চেছন একটি মানচে4র মচেন দেযাচেন দেকামতার দেকান সথান দেনই পাথচেরর মচেতা হিনরসতার মচেনর মধ দেযও দেয দেপরম আসচেত পাচের দেস কথাই কহি চেচেছনগ)নারীচের যথC দেপরচেমর ছহি এই কহিতায় অকপচেট উচেঠ এচেসচেছ কহি এই কহিতায় কাটচোটটা গাছ কথাটি যার কচেরচেছন নারী দেয দেপরম দেথচেক হিতাহিত এং দেসই দেপরম সঠিক সমচেয় না পাওয়ার ন য দেপরম সমপচেকC হিচেr4 গৈতরী য় দেপরচেমর দেয গৈহি(ষট য মাধযC য সরসতা দেকামত এই সমসতর হিপরীত যথা রbতা শষকতা কচেঠার তা পরভহিত দোঝাচেত এক কাঠচোটটা গাছ কথাটি যার কচেরচেছনঘ) এাচেন এক নারীর যথC দেপরচেমর কথা হিনহিCপত ভাচে চেচেছন কহি অসমচেয় নারীর ীচেন দেপরম দেচেগচেছ এতহিন তার হয় রb কচেঠার হিছ দেপরচেমর অভাচে ঠাৎ দেসই শষক মরভহিমচেত সচের আভাস এচেসচেছ দেপরম দেযন 4Cার স(ীত তাই পরায় মত গাচেছ কহিকহি পাতা গহিচেয় উচেঠচেছ

Biology Chapter - 01Controlling Air Pollution

Today we will discuss how we control air pollution from domestic combustion

Q1Describe any five ways of reducing air pollution from domestic sources bull The number of pollutants in the air is verylarge and we always try to control them byfollowing ways

i) Solar cooker and solar heater It use no fuel reduce damage of environment by fuel use or reducing deforestation It maintains coolness of house It releases very less orno oil gas or grease

ii) Piped natural gas (PNG) It emits very less by products into the atmosphere As it isdistributed through pipe lines so there iscontinuous supply of fuel is possible

iii) Liquefied Petroleum Gas (LPG) It hasa higher heating value LPG doesntcontain sulphur so it burns a lot cleanerenergy sources It releases very less oralmost no fume in air

iv) Electricity based cooking Emission free cooking alternative for urban dwellers causeselimination of adverse health impactsofindoor air pollution It helps to avoid theinconveniences associated with procurement of LPG

v) Biogas It contains 75 methane whichmakes it an excellent fuel It burns without smoke and biogas plant leaves no residue like ash in wood charcoal etc Thus it isaclean fuel

Economics

Factors of Production

Today firstly we would recall the last class for 5 mins and then we would proceed with the further topics of the chapter

The concept meaning of land characteristics of land and importance of land to be repeated for the absentees as well as the students who were there in the class the previous day

Today we will start with the last portion of land before it the meaning of land to be repeated onceAs by now we all know that

Questions1What do you mean by productivity of landAnswer By productivity of land we mean the capacity of a piece of land to produce a crop

Thus it refers to the average output per unit of landSay per acre per hectare etc= (OutputArea of land)

2 What are the factors influencing the productivity of landAnswer

Natural factors Productivity of land is largely determined by the natural

Land is defined to include not only the surface of the earth but also all other free gifts of nature(for example mineral resources forest resources and indeed anything that helps us to carry out the production of goods and services but is provided by nature free of cost)

We will move on to the last portion of land by discussing Productivity of Land

By productivity of land we mean the capacity of a piece of land to produce a crop

Thus it refers to the average output per unit of land

Say per acre per hectare etc= (OutputArea of land)

With this we shall proceed further with the main factors that determine the productivity of land

Natural factors Human factors Improvements on land Location of land Organisation Ownership of land Availability of capital Proper use of land State help

Note economic development of a country depends upon the quality of its land If the land is fertile it will quicken the pace of development of the country

qualities of land such as fertility etc

Human factors Land cannot produce anything by itself Man has to apply labour on it to produce for himself So productivity of land depends on the knowledge and skills of workers

Improvements on land production of land is affected by land development measures like provision of well or tubewell irrigation proper drainage

State help The government of a country especially less developed country can play a vital role in improving the agricultural productivity by providing better irrigation facilities

Organisation Productivity of land also fdepends upon the way how the factors of production like labour and capital are organised

In order to increase productivity trained workers modern implements scientific methods good seeds are all essential

3 lsquoImproved technology affects the productivity of landrsquo Explain this statement with the help of suitable example Answer Use of improved technology raises the productivity of land Example By using HYV seeds chemical manures and modern machines per hectare output increases

Physics Force (Summary)

Question Write the expression for the moment of force about a given axisSolutionsThe expression for the moment of force is given byMoment of force about a given axis = Force times perpendicular distance of force from the axis of rotationQuestion What do you understand by the clockwise and anticlockwise moment of force When is it taken positiveSolutionsIf the effect on the body is to turn it anticlockwise moment of force is called the anticlockwise moment and it is taken as positive while if the effect on the

body is to turn it clockwise moment of force is called the clockwise moment and it is taken as negative

Math Topic Commercial Mathematics

Chapter Goods and services Tax

Study item Some solved sums from exercise ndash 1 A retailer buys a TV from a wholesaler for Rs 40000 He marks the price of the TV 15 above his cost price sells it to the consumer at 5 discount on the marked price If the sales are intra ndash state and the rate of GST is 12 find

(i) The marked price of the TV(ii) The amount which the consumer pays for the TV(iii) The amount of tax (under GST) paid by the retailer to the central

Government(iv) The amount of tax (under GST) received by the State Government

Solution As the sales are intra- state sale and the rate of GST 12 So GST comprises of 6 CGST and 6 SGSTTherefore a retailer buys a TV from a wholesaler for Rs 40000Therefore the amount of GST collected wholesaler from the retailer or paid by retailer to wholesalerCGST = 6 of Rs 40000 = Rs(6100 times40000) =Rs 2400SGST = 6 of Rs 40000 = Rs (6100 times 40000) =Rs 2400Therefore wholesaler will pay Rs 2400 as CGST and Rs 2400 as SGSTTherefore amount of input GST of retailer Input CGST = Rs 2400 and input SGST = Rs 2400Again the retailer marks the price of the TV 15 above his cost price(i) The marked price of the TV

= Rs 40000 + Rs 40000times15= Rs 40000 + Rs 40000times 15100= Rs 40000 + Rs 6000Rs 46000But the retailer sells it to consumer at 5 discount on the marked priceCost price after discount = Rs 46000 ndashRs46000times 5100 =Rs 46000 ndashRs 2300= Rs 43700Therefore the amount of GST collected retailer from consumer or paid by consumer to retailerCGST = 6 of Rs 43700 =Rs ( 6100 times43700)Rs 2622SGST = 6 of Rs 43700 = Rs (6100 times 43700) =Rs 2622Amount of the output GST of retailer Output CGST = Rs 2622 and output SGST = Rs 2622

(ii) The amount which the consumer pays for the TV= cost price of TV to consumer + CGST paid by consumer + SGST paid by consumer= Rs 43700 + Rs 2622 + Rs 2622= Rs 48944

(iii) The amount of tax (under GST ) paid by the retailer to the central Government=CGST paid by retailer = output CGST ndash input CGST=Rs 2622 ndash Rs 2400=Rs 222

(iv) The amount of tax ( under GST ) received by the State Government = SGST paid by wholesaler + SGST paid by retailer= Rs 2400 + output SGST ndash input SGST=Rs 2400 + Rs 2622 ndash Rs 2400=Rs 2400 + Rs 222= Rs 2622

Commercial studies

Stakeholders Today I am going to give some revision questions from the previous study material

Questions1) State the two expectations of

employees from a business concern2) Give two distinctions between

stakeholder and shareholder3) Give two difference between

internal stakeholders and external stakeholders

4) Give two expectations of suppliers from a business organisation

5) Who is a stakeholder in commercial organisations

Chemistry Periodic Table

Merits of Mendeleevrsquos Periodic law are as follows - 1He grouped the elements on the basis of atomic mass 2 He left gaps for undiscovered elements like Gallium Scandium germanium Also he left a full group vacant for undiscovered inert gases 3 He could predict proportions of several elements on basis of their position in periodic table like Ga Sc etc 4He could predict errors in atomic weights of some elements like gold platinum etc

Anomalies in Mendeleevrsquos Periodic law are as follows - 1 Position of isotopes could not be explained 2 Wrong order of atomic masses could not be explained

For example- as Arnur atomic mass 40 come first and K with low atomic mass (30) should come later but k should be placed first

According to Bohrrsquos Modern Periodic table properties of elements are periodic functions of their atomic numbers

So when elements are arranged according to increasing atomic numbers there is periodicity in electronic configuration that leads to periodicity in their chemical properties

It consists of horizontal rows (Periods) Vertical column (Groups)

There are 7 period and 12 groups in this long form of periodic table

Ist period has 2 elements IInd period has 8 elements IIIrd period has 8 elements IVth period has 18 elements Vth period has 18 elements VIth period has 32 elements VIIth period hs rest of elements

Note - The number of valence electrons in atom of elements decides which elements will be first in period and which will be last

In group- 1 to 2 gp and 13 to 17 contain normal elements 3 to 12gp ndash transition elements 57 to 71 - lanthanides 89 to 103 - Actinides

Left hand side ndash metals Right hand side ndash nonmetals

Note- Hydrogen element has been placed at top of Ist group Electronic configuration of H is similar to alkali metal as both have 1 valence electron

V electron of gp I element -- 1 V electron of gp 2 element -- 2 V electron of gp 13 element -- 3 V electron of gp 14 element -- 4 V electron of gp 15 element -- 5 V electron of gp 16 element --6 V electron of gp 17 element -- 7 V electron of gp 18 element -- 8

English 1 Transformation of sentences

Sentences A sentence is a group of words which makes complete sense

Exercise 2Change the following sentences from

a Assertive sentencesb Imperative sentencesc Interrogative sentencesd Exclamatory sentences

Sentences can be changed from one grammatical form to another without changing the meaning of the sentence This is known as transformation of sentences

assertive to interrogative1 Nobody would like to be a fool

Who would like to be a fool2 Their glory can never fade

When can the glory fade3 Nobody can control the wind

Who can control the wind4 It matters little if I die

What though I die5 No man can serve two masters

Can any man serve two masters

Exercise 3Interchange of assertive and Exclamatory sentences

1 She leads the most unhappy lifeWhat an unhappy life she leads

2 This is indeed an interesting bookWhat an interesting book this

3 He is a very great manWhat a great man he is

4 It is a very lame excuseWhat a lame excuse

5 It is sad that she died so youngAlas she died so young

Class XISubject Topic Summary Execution

Hindi 2nd lang

पतर परम(परमचदर) पतर परम कहानी म एक निपता की इचछाओ का वणन निकया गया ह अपन बड पतर परभ -ास स निपता चतनय -ास का निवशरष परम था निपता को उसक जनम स ही बडी-बडी आशाए थी उसम दसर बट कतिशव-ास की अपकषा स- उतसाह की मातरा अमिधक थी वह उस इगलड भजकर बरिरसटर बनाना चाहत थभागय का खल भी बडा निनराला ह बीए की परीकषा क बा- वह बीमार पड गया डॉकटरो न भी जवाब - दि-या थाचतन -ास जी बहत ही कजस थ बवजह पस खच करना नही चाहत थ अगर गारटी मिमलती तो शाय- पस खच भी कर -त परत गारटी नही थी परिरणाम सवरप उनक बट का -हात हो गयाजब बट को समशान ल जा रह थ तो वहा काफी शोर गान बजान हो रह थ पछन पर पता चला निक निकसी निपता निपछल तीन साल स निबमार था और उसक ईलाज म रपया पानी की तरह बहाया पर ठीक नही हए परत उसक बट को तनिनक भी अफसोस नही था उसका कहना था उसन कोकतिशश तो कीयह -खकर चतनय-ास जी को आतम निगलानी हईतभी स उनका म परिरवतन हआ और बट का भोज काफी धमधाम स निकयाऔर वहइस पशचाताप की आग म जलत रह औला- स बढकर पसा नही होता ह इस बात को समझन म उनह काफी व लग गया

hellipContinue to next

BENGALI(2ND LANGUAGE)

পরথমঅধযায়-ঠাকরারীনদরনাথঠাকর

নয়ন দোচের হিমাচেররা া নাচেমই হিযাত হিছচেন ায়ানার উাররণ সবরপ নয়ন দোচের ারা হিা (াচেকর হিা হিচেতন এছাাও দেকান উৎস উপচেb রাহিতর দেক হিন করার উচেfচে(য তারা সযC হিকরচেরণ রনয পরীপ জবাহিচেয় তাচেত রপার হির 4Cরণ করচেতন ঠাকরা এই নয়ন দো হিমারচের দে(4 ং(ধর হিছচেন হিমাররা ায়ানার ষটানত পর(Cন কচের তারা হিনঃসব এই হিমাহিরর দে(4 ং(ধর গৈকাস নদর রায়চেৌধরী গৈকাস া নয়ন দোচের সমসত সমপহিতত ঋচেরণর াচেয় হিহিx কচের অহি(ষট যা আচেছ তাচেত হিপত

ইার হিপতার মতয ইচে পর নয়নচোচের ায়ানার দেগাটা কতক অসাধাররণ শরাদধ (াহিনতচেত অহিনতম ীহিপত পরকা( কহিরয়া ঠাৎ হিনহিয়া দেগ- ক) কার দো দেকান গচেলপর অং() কতা দেক ইার চেত কাচেক দোঝাচেনা চেয়চেছ গ) পরসঙগ কী কতার কতয পহিরসফট কচেরা

পরচে4র যাহিত রbা করা সমভ নয় তাই হিতহিন পতরচেক হিনচেয় ককাতায় সাস শর কচেরন গলপ কথচেকর আহিথCক অসথা নয়ন দোচের হিমাচের দেথচেক সমপরণC আাা কথচেকর হিপতা হিনচের দেষটায় অথC উপাCন করচেতন া উপাহিধ াচেভর নয তার াসা হিছনা আর দেসই কারচেরণ কথক তার একমাতর উততরাহিধকার চেয় তার হিপতার পরহিত কতজঞ কথক দো পা হি(চেচেছন হিনচের পরারণ ও মান রbার নয উপচেযাগী অথC হিনা দেষটায় পরাপত চেয়চেছন- এটাই তার কাচেছ পরম দেগৌরচের হি4য় চে মচেন কচেরন কাররণ (নয ভাণডাচের গৈপতক ায়ানার উজজব ইহিতাস অচেপbা দোার হিসeচেকর মচেধয গৈপতক দেকামপাহিনর কাগ তার কাচেছ অচেনক দেহি( মযান

TO BE CONTINUED

উ- ক) আচোয অং(টি রীনদরনাথ ঠাকচেরর দো ঠাকরা গচেলপর অং() কতা চেন আচোয গচেলপর গলপ কথকইার চেত নয়ন দোচের হিমাহিরর দে(4 ং(ধর গৈকাস ার কথা া চেয়চেছ গৈকাস া নয়ন দোচের সমসত সমপহিতত ঋচেরণর াচেয় হিহিx কচের অহি(ষট যা আচেছ তাচেত হিপত পরচে4র যাহিত রbা করা সমভ নয় তাই হিতহিন পতরচেক হিনচেয় ককাতায় সাস শর কচেরনগ) গৈকাস ার হিপতার মতযর পর নয়ন দোচের হিমাহিরর অহিসততব হিপত য় কচেয়কটা উৎস ও শরাদধ- (াহিনতচেত হিমাহিরর দে(4 কহিটক যয় চেয় হিগচেয় এচেক াচের দে(4 চেয় যায় তন তাচের গC করার মত আর হিকছই হিছ না-দেসই পরসচেঙগ এই উহিকত নয়নচোচের হিমাচেররা া নাচেমই হিযাত হিছচেন ায়ানার উাররণ সবরপ নয়নচোচের ারা হিা (াচেকর হিা হিচেতন এছাাও দেকান উৎস উপচেb রাহিতরচেক হিন করচেত হিগচেয় তারা সযC হিকরচেরণর নয পরীপ জবাহিচেয় তাচেত রপার হির 4Cরণ করচেতন তাই দেসকাচের ায়ানা দেহি(হিন সথায়ী চেত পারত না হিহিভনন উৎস শরাদধ- (াহিনতচেত সাধযা হিতহিরকত র করার নয হিমাহির হিহিকচেয় দেযত হ হিতC কা হিহি(ষট পরীচেপর দেত দেযমন অলপকাচের মচেধয হিনঃচে(4 চেয় যায়-নয়নচোচের হিমারচের অসথা তাই চেয়হিছ এই কারচেরণই কথক নয়নচোচের হিমারচের গা ভরা আমবর সয করচেত পারতনা

Physics Dimensional Analysis (Summary)

Q Find the dimensions of consts ab in relation

p=(bminusxlowastx)at

where p is the power x is the distance and t is time

Ans From principle of homogeneity dimension of b x2 are same Dim of b = dim of x2 = [L2] = [ML2T0]Dim of a = dim of ( b- x2)dim of (pt) = [M0L2T0][ML2T-2] [T-1] [T] = [M-1L0T2]

Chemistry Atomic Structure Drawbacks of Rutherfordrsquos model of

atom a According to Rutherfordrsquos model of atom electrons which are negativelycharged particles revolve around the nucleus in fixed orbits Thusb theelectrons undergo acceleration According to electromagnetic theory of Maxwell a charged particle undergoing acceleration should emitelectromagnetic radiation Thus an electron in an orbit should emitradiation Thus the orbit should shrink But this does not happenc The model does not give any information about how electrons aredistributed around nucleus and what are energies of these electrons Isotopes These are the atoms of the same

Properties of electromagnetic radiationsa Oscillating electric and magnetic field are produced by oscillating charged particles These fields are perpendicular to each other and both areperpendicular to the direction of propagation of the waveb They do not need a medium to travel That means they can even travel invacuum

Characteristics of electromagnetic radiationsa Wavelength It may be defined as the distance between two neighbouring crests or troughs of

element having the same atomicnumber but different mass numbere g 1H11H21H3

Isobars Isobars are the atoms of different elements having the same massnumber but different atomic numbere g 18Ar40 20Ca40

Isoelectronic species These are those species which have the same numberof electrons

Electromagnetic radiationsThe radiations which are associated withelectrical and magnetic fields are called electromagnetic radiations When anelectrically charged particle moves under acceleration alternating electricaland magnetic fields are produced and transmitted These fields aretransmitted in the form of waves These waves are called electromagneticwaves or electromagnetic radiations

wave as shown It is denoted by λb Frequency (ν) It may be defined as the number of waves which passthrough a particular point in one secondc Velocity (v) It is defined as the distance travelled by a wave in onesecond In vacuum all types of electromagnetic radiations travel with thesame velocity Its value is 3 times10 8m sec-1 It is denoted by v

d Wave number Wave number is defined as the number of wavelengths per unit lengthVelocity = frequency timeswavelength c = νλ

Plancks Quantum Theory- o The radiant energy is emitted or absorbed not continuously but discontinuously in the form of small discrete packets of energy called lsquoquantumrsquo In case of light the quantum of energy is called a lsquophotonrsquoo The energy of each quantum is directly proportional to the frequency of the radiation ie E α υ or E= hυ where h= Planckrsquos constant = 6626 x 10-27 Js o Energy is always emitted or absorbed as integral multiple of this uantum E=nhυ Where n=1234Black body An ideal body which emits and absorbs all frequencies is calleda black body The radiation emitted by such a body is called black body radiation

Photoelectric effectThe phenomenon of ejection of electrons from thesurface of metal when light of suitable frequency strikes it is calledphotoelectric effect The ejected electrons are called photoelectrons

Biology Chapter - 02Systematics and Five Kingdoms

Scientists divide the whole living organisms into two kingdom first and ultimately by five kingdom at last

In the earlier systems of classifications organisms are divided into kingdom plantaeand kingdom animalia on the of presenceof cell wall their modes of nutrition and movements

Some problem arise like fungi share manycharacteristic withplant despite their heterotrophic nutrition bacteria protozoa areunicellular present in both kingdom Toovercome this third kingdom Protista isintroduced which include

unicellularorganisms But there is also another

problem Allunicellular organisms are not similar kind The cellular structure of prokaryotes is verydifferent from that of other organismsEukaryotes possess a true nucleus and allcell organelles that are not present inprokaryotes So the fourth kingdom Monerais introduced which include unicellular prokaryotes (bacteriaamp blue green algae)

bull Still some problem arise in kingdomplantae

So in 1969 R H Whittakar proposedanew five kingdom System of classification

i) Kingdom Monera - unicellular prokaryotes

ii) kingdom Protista - unicellular eukaryotes

iii) Kingdom Fungi - uni or multicellular fungi with cell wall but without chlorophyll

iv) Kingdom Plantae - Multicellular Plants

v) Kingdom Animalia - Multicellular Animals

EVS Chapter 1 ndash Modes of Existence

An agricultural society

An agricultural society also known as an agrarian society is a society that constructs social order around a reliance upon farming More than half the people living in that society make their living by farming

People in an agricultural society generally lead a more settled lifestyle than those in nomadic hunter-gatherer or semi-nomadic pastoral societies because they live permanently near the land that is farmed Agricultural settlements tend to develop in areas of convenience near bodies of water which is used for both crops and transportation or along trade routes Not everyone in an agricultural society is a farmer Some people make a living trading or making and selling goods such as tools used for farming

Another way to define an agrarian society is to see the total amount of production in a nation In an agrarian society cultivating the land is the main source of wealth Such a society can recognize other means of subsistence and work habits but emphasizes the importance of agriculture and livestock Agrarian societies have existed in various parts of the world for 10000 years and continue to exist today They have been the most common form of socio-economic organization for most of recorded human history

Q) Write the features of agricultural society

Ans - Structure and Features of Agrarian Society1 Occupational Structure

An agrarian society is generally associated with the domestication of plants and animals The domestication of plants means farming and that of animals means herding Often there is mixture of farming and the use of such domesticated animals as cow goat and sheep

2 Forms of Land Ownership in Agrarian SocietiesGenerally there are landlords supervisory farmers cultivators and share croppers The landholders own the land but do not work on it They let it out for sharecropping The supervisory farmers are those who live by having their land cultivated by hired labourers The cultivators cultivate the land for themselvesThe share-croppers are those who live by tilling other peoplersquos land or a crop-sharing basis The artisans own their means of production and produce by their own labour in their homesteads

3 Village Community System An agrarian society is highlighted by

the institution of village community system The agrarian economy made fixed dwelling houses necessary Living close together for protection and co-operation and living nearer to the land gave birth to agricultural villages The village is not only the residential place of farmers it is also the social integrator

4 Minimal Division of Labour Another structural feature of agrarian society is a minimal division of labour Except for the basic division founded on age and sex differences there are few specialized roles There is only one predominant type of occupation ie domestication of plants and animals For all the people the environment physical as well as social is the same

5 Role of Family The farm family is of the patriarchal type the father is the final arbiter in most of the familyrsquos major decisions The life of ail men and women is merged in family life Since there are not many special organizations family is the only organisation to perform the tasks of aid and protection

6 Sense of Unity The members of an agrarian society exhibit a strong in-group feeling Since the whole of their social lives is wrapped up in a society which is physically economically and socially homogenous they are inclined to view the entire outside world as an out group

7 Informal Social Control An agrarian society is regionally divided into villages In a village community the force of traditional mores is more dominant than in the urban community In the village everybody is known to everybody The members in a village community help each other and share the joy and sorrows of each other Crime in an agrarian society is rare

8 Simplicity and Uniformity Life of the people in an agrarian society is marked by simplicity and uniformity Their main occupation is agriculture which largely depends upon the vagaries of nature An agrarian society is a religious society

Math Compound angles Compound angles The algebraic sum of two or more angles is called a compound angle If A B C be three angles then A+B B+C C+A A-B B-C A-C A+B-C etc are compound angles In this chapter we shall discuss the trigonometrical ratios of compound angles Theorem 1 If A B and A+B are all pisitive acute angles theni) sin( A+B) = sin A cos B + cosA sinBii) cos(A+B) = cosA cosB- sinA sinBTheorem 2If A and B are positive acute angles and AgtB theni) sin(A-B) = sin A cosB- cos A sinBii) cos(A-B) = cos A cos B+ sin A sin BTo prove that i) sin(A+B) sin (A-B) = sin2 A - sin2 B = cos2 B- cos2 A

Example 1 Prove that tan70deg=2tan50deg+tan20degSolutiontan70deg = tan(50deg + 20deg)Or tan70deg=(tan 50deg+tan 20deg)(1-tan50degtan20deg) or tan70deg (1 ndash tan 50deg tan20deg) = tan50deg+tan20degor tan70deg= tan70deg tan50deg tan20deg+ tan50deg + tan20deg = cot20deg tan50deg tan20deg + tan50deg + tan20deg = 2 tan50deg+ tan20degExample 2 If A + B = 45deg show that (1 + tanA) (1 + tanB) = 2Solutiontan(A + B) =( tan A + tan B) (1 - tan

ii) cos(A+B) Cos(A-B) = cos2 A- sin2 B = cos2 B -sin2 AProof i) LHS= sin(A+B)sin(AminusB) [Recall sin(αminusβ)=sinαcosβminuscosαsinβ And sin(α+β)=sinαcosβ+cosαsinβ]= (sinAcosB+cosAsinB)times(sinAcosBminuscosAsinB)= sin2Acos2Bminuscos2Asin2B [Recall sin2α+cos2α=1 From above we can then assume correctly that sin2α=1minuscos2α AND cos2α=1minussin2α] = sin2A(1minussin2B)minussin2B(1minussin2A) = sin2Aminussin2Asin2Bminussin2B+sin2Asin2B = sin2Aminussin2B= 1-cos2A-(1-cos2B) = cos2 B- cos2 A = RHSii)LHS= cos (A+B) cos(A-B) [ cos(A+B) = cos AcosB- sinAsinBCos(A-B) = cosAcosB+ sinAsinB]= cos2 A Cos2 B- sin2 A Sin2 B= cos2 A( 1-sin2 B) - (1- cos2 A) sin2 B= cos2 A- cos2 A sin2 B- sin2 B+ cos2 A sin2 B=cos2 A- sin2 B=1- sin2 A-(1-cos2 B) = cos2 B- sin2 A= RHSTangent formulae for compound anglesi)tan (A + B) = tan A + tan B1-tan A tan Bii)tan (A ndash B) = tan A-tan B1+tan A tan Biii) cot (A + B) = cot Acot B-1cot A+cot B(viii) cot (A ndash B) = cot Acot B+1cot B-cot A

A tan B) Or 1= (tan A+ tanB) (1-tan A tanB) Or tanA + tanB + tanA tanB + 1 = 1 + 1Or tanA (1 + tanB) + (1 + tanB) = 2Or (1 + tanA) (1 + tanB) = 2Example 3 Find the value of sin 15degSolution sin 15deg= sin(45deg-30deg) = sin45degcos 30deg- cos45degsin30deg =(1radic2) (radic32) -(1radic2) (12) = (radic3-1) 2radic2Example 4 If sin A = 1 radic10 and sin B = 1 radic5 where A and B are positive acute angles then what is A + B SolutionWe know that sin (A + B) = sin A cos B + cos A sin B= [1 radic10] [radic(1 minus 1 5)] + [1 radic5] radic(1 minus 1 10)= [1 radic10] [radic4 5] + [1 radic5] [radic9 10]= [1 radic50] times (2 + 3)= 5 radic50 = 1 radic2

sin (A + B) = sin π 4rArrHence A + B = π 4Example 5 If A + B = 225o then find [cot A] [1 + cotA] times [cot B] [1 + cot B]Solution[cot A] [1 + cotA] times [cot B] [1 + cot B] = 1 [(1 + tan A) times (1 + tan B)]=1 [tan A + tan B + 1 + tan A tan B] [ tan (A + B) = tan225o]∵

tan A + tan B = 1minus tan A tan BrArr= 1 [1 minus tan A tan B + 1 + tan A tan B]= 1 2

COMMERCE

CLASSIFICTION OF HUMAN ACTIVITIES-ECONOMIC AND NON-ECONOMIC

Firstly we shall recall the previous class for 5 mins especially for the absentees and for also the rest of the students who were there

Today at first we briefly discuss the earlier portions of the chapter

1Business-It includes all those economic activities which are concerned with production and exchange of goods and services with the object of earning profit Example A factory shop beauty parlour also business enterprises

2Profession ndashThe term profession means an occupation which involves application of specialized knowledge and skills to earn a living For Example Chartered Accountancy medicine law tax consultancy are example of professions

Questions1What are the main features of ProfessionAnswer The main features of a profession are as follows a Specialised body of knowledge-Every profession has a specialised and systematised body of knowledge b Restricted entry- Entry to a profession is allowed only to those who have completed the prescribed education and have the specialised examination c Formal education and training ndashA formal education and training is given to the person who wants to acquire the professional

3Employment-Employment mean an economic activity where people work for others in exchange for some remuneration (salary)The persons who work for others are called lsquoemployeesrsquo The persons or organizations which engage others to work for them are called lsquoemployersrsquoEg A doctor working in a hospital is employment as he is working for a salaryA lawyer may serve as a law officer in a bank

With this we shall proceed with the features of both Profession amp Employment

The main features of a profession are as follow

a Specialised body of knowledge b Restricted entry c Formal education and training d Professional association e Service motive f Code of contact

The main features of an employment are as follows

a In employment a person works for others called employer

b An employee provides personal service

c There is a service agreement or contract between the employee and the employer

d The employee has to obey the order of the employer

e No capital investment is made by the employer

Various examples of Employment are as follows

aA teacher teaching in a school or collegeb An engineer employed in Municipal Corporation of DelhicAn accountant working in the accounts department of a companydA doctor working in a hospital

Note In all the above examples of employment the individual who is involved in each example is working as an employee for a salary under an employer

qualification(MBBSCALLB)d Service motive ndashProfessionals are expected to emphasis service more on their clients rather than economic gain f Code of Conduct-The activities of professionals are regulated by a code of conduct

2 What are the main features of EmploymentAnswer The main features of an employment are as followsa In employment a person works for others called employerb An employee provides personal servicec There is a service agreement or contract between the employee and the employerd The employee has to obey the order of the employere No capital investment is made by the employer

3 Give various Professions and their respective Association are given below

Professions

Professional

Professional association

Medical profession

Doctor Medical Council of India

Law profession

Lawyers Bar Council of India

Accounting Profession

Chartered

The Institute of Chartered Accounts of India( ICAI)

Engineerin Engineers The

g Profession

institute of Engineers (India)

Accounts Basic accounting terms

Today we will give you some questions from the previous study material

Questions6) Define accounting7) What do you mean by debit

and credit8) Explain the types of account9) Define the following terms

a) Assetsb) Capitalc) Purchased) Debtorse) Transactions

10) Name the types of accounts given below

a) Krishnas accountb) Machinery accountc) Royalty accountd) Salary accounte) Furniture accountf) Audit fee account

Economics Basic Economic ConceptsSub topic

UTILITY

Before starting todayrsquos class we shall recall the last class which was about UTILITY AND THE FEATURES OF UTILITY

Now we shall proceed with the further topics of the chapter

Todayrsquos topic from the chapter lsquo Basic Economic Conceptsrsquo will be TOTAL UTILITY amp MARGINAL UTILITYNow let us quickly revise the concept of utility with an example ie goods and services are designed because they have an ability to satisfy human wantsThis feature of being able to satisfy human wants is termed as utility For example we derive utility from WiFi services as it gives us satisfaction by connecting us to our friends and family through social media here consumers derive utility from WiFi services

From the above concept we shall start with todayrsquos topicEconomists have defined TOTAL UTILITY (TU) as the total satisfaction obtained by consuming a given total amount of a good and serviceFor example the total satisfaction obtained from eating 10 mangoes is the total utility of 10 mangoes

MARGINAL UTILITY (MU) is the additional satisfaction derived from each additional unit

Questions1 What is Total Utility (TU)

Answer Total Utility (TU) is the

aggregate of the utility that a consumer derives from the consumption of a certain amount of a commodityTU=MU1+MU2++MUn

2 What is Marginal UtilityAnswer

Marginal Utility (MU) is the additional made to the total utility as consumption is increased by one more unit of the commodityMU= TUn ndashTUn-1

NoteOften economists tend to

subdivide utility into an imaginary unit called UTIL

consumed In this casethe utility obtained from each mango as it is consumed as the MU of that mango It is also defined as the addition made to the total utility when an additional unit is consumed Often economists tend to subdivide utility into an imaginary unit called UTIL

Note As a consumer increases the consumption of a good over period of time the total utility or total satisfaction derived from it increases to appoint and thereafter it decreasesHowever as the consumer keeps on consuming the good the marginal utility or the additional utility derived from it decreases

SubjectBusiness studies

Topic

BUSINESSENVIRONMENT

Summary

Now quickly let us revise the earlier points that we have already done in the last class and let us proceed with the other topics that are there in the chapter

Firstly we will recall the internal and external factors of micro environment and then we shall proceed in details

Meaning and list of internal and external factors

aInternal factorsInternal factors refer to all the factors existing within a business firm The internal factors are considered controllable because the enterprise has control over these factorsFor an example a company can alter its organization structure policies programmes employees physical facilities and marketing mix to suit the changes in the environmentList of internal factors areCorporate culture mission and objectives top management organizations structure company image and brand equity company resources

b External factorsExternal factors refer to those individual and groups and agencies with which a particular business organization comes into direct and frequent contact in the course of its functioningThese individuals and groups are known as STAKEHOLDERS because they have a stake (financial interest ) in the working and performance of the particular business List of external forces (stakeholders)Customers competitors investors suppliersmiddlemen (marketing intermediaries)

Execution 1 What do you mean by internal

factors in micro environmentAnswerInternal factors refer to all the factors existing within a business firm The internal factors are considered controllable because the enterprise has control over these factorsFor an example a company can alter its organization structure policies programmes employees physical facilities and marketing mix to suit the changes in the environment

2 What do you mean by external factors in micro environment

AnswerExternal factors refer to those individual and groups and agencies with which a particular business organization comes into direct and frequent contact in the course of its functioningThese individuals and groups are known as STAKEHOLDERS because they have a stake (financial interest) in the working and performance of the particular business

3Who are stakeholdersSTAKEHOLDERS are individuals and groups who have a stake (financial interest ) in the working and performance of the particular business 4Discuss the internal factors in briefa Corporate CultureThe values beliefs and attitudes of the founders and top management of the company exercise

financers publics

customers

suppliersfinancers

competitors

middlemen

publics

Fig STAKEHOLDERS OF A COMPANY

Apart from micro environment the other main dimension of business environment isMacro environment Macro environment refers to the general environment or remote environment within which a business firm and forces in its micro environment operateA company does not directly or regularly interact with the micro environmentTherefore macro environment is also known as indirect action EnvironmentThe macro environment forces are less controllable than the micro forces

Macro environment consists of the following components

POLITICAL AND LEGAL ENVIRONMENT

ECONOMIC SOCIAL AND ENVIRONMENT

CULTURAL

ENVIRONMENT

TECHNOLOGICAL ENVIRONMENT

a strong influence on what the cmpaany stands for how it does things and what it considers importantbMission and objectivesThe business philosophy and purpose of a comoany guide it prioritiesbusiness strategiesproduct market scope and development scope

cTop management structurethe composition of board of directors the degree of professionalization of management and the organizational structure of a company have important bearing on its business decisions

dPower structureThe internal power relationship between the board of directors and the chief executive is an important factor

eCompany image and brand equityThe image and brand equity of the company play a significant role in raising finance forming alliance choosing dealers and suppliers launching new products entering foreign markets

5 What is Macro environmentAnswerMacro environment refers to the general environment or remote environment within which a business firm and forces in its micro environment operateA company does not directly or regularly interact with the micro environmentTherefore macro environment is also known as indirect action EnvironmentThe macro environment forces are less controllable than the micro forces 6 What are the components of macro environmenta Political and legal environmentb Economic environmentc Social and cultural environmentd Technological environment

BUSINESS FIRM

Fig COMPONENTS OF MACRO ENVIRONMENTPolitical science

Introduction to political science

Comparative politics and itrsquos scope Comparative politics is the second major dimension of political scienceIt is also a very vast area of study and a very large number of political scientists even treat it as an autonomous area of study within the board ambit of political scienceScope of comparative politics-

1 All political structures -Comparative politics includes the study of all structures formalnon formal governmental and extra governmental which are directly or indirectly involved in politics in all the countries of the world

2 Functional studies- Comparative politics seeks to study politics less from the point of view of the legal institutions in terms of their powers and move from the point of view of their functions which constitute the political process and their actual Operation in the environment

3 Study of political behaviour- Another important part of its scope is the study of the actual behaviour of the people in the process of politics

4 Study of similarities and differences- comparative politics also undertakesan analysis of the similarities and differences among political process and functions

5 Study of all political systems -comparative politics seeks to analyse the actual behaviour and performance of all political systems western as well as non western

6 Study of the environment and infrastructure of politics-The study of politics demands a study of the psychological sociological economic and anthropological environment in fact the social environment as a whole in which each political system operates

7 Study of political culture- political culture is composed of attitudesbeliefs emotions and values of a society that relate to the political system or politics

8 Study of political participation- Political participation is a universal processThe only difference is that while in some states it is limited in others it is wider

9 Study of political process- political

Answer the following questions-

What is comparative politics

What are the scope of comparative politics

Homework- learn

processes like decision makingpolicy making judicial process leadership recruitment process and others are always at work in all political systems

The scope of comparative politics is very comprehensive It includes everything that falls within the area of political activity and political process

History CAMBRIDGE VIEW ABOUT

THE PARTITION

AND REFUTATION

OF CAMBRIDGE

VIEW

Cambridge view about the Partition The Cambridge school of historians have interpreted that opposition to partition scheme was made entirely by the elitist groups They hold the view that Lord Curzon planned to partition the Bengal for administrative purposeREFUTATION OFCAMBRIDGE VIEW The Rationalist historians have rejected the interpretations of the Cambridge School of historians on various grounds

1 QUESTION State different views of historians regarding Partition of Bengal

ANSWER Cambridge historians believed that Lord Curzon partitioned Bengal for administrative reasons only and not for the political motive The Middle class elitist group protested because of their petty interest The Hindu zamindars protested as they have to spend more money for managing their estatesThe lawyers of Calcutta High court feared to lose their clientBut according to the nationalist Historians was-

2- The ultimate object of Lord Curzon was to crush the unity of Bengal politicians

3- If Bengal becomes a separate province Bengali speaking 16 million people of western part would become minority under Hindi speaking people of Bihar and Oriya speaking people of Orissa

4- The bureaucrats expected that the protest movement would die down quickly

5- Lord Curzon used the Muslim community in his political game

6- Idealism had great contribution in the protest against partition

7- The people of the every section of society were affected by the partition of Bengal

Computer Science

Numbers Convertion of dcimal number to octal numberThe decimal numeral system is the standard system for denoting integer and non-integer numbers It is the extension to non-integer numbers of the Hindu-Arabic numeral system For writing numbers the decimal system uses ten decimal digits a decimal mark and for negative numbers a minus sign - The decimal digits are 0 1 2 3 4 5 6 7 8 9 the decimal separator is the dot in many countries

The octal numeral system or oct for short is the base-8 number system and uses the digits 0 to 7 Octal is sometimes used in computing instead of hexadecimal perhaps most often in modern times in conjunction with file

permissions under Unix systems It has the advantage of not requiring any extra symbols as digits It is also used for digital displays

Follow these steps to convert a decimal number into octal form

1 Divide the decimal number by 82 Get the integer quotient for the next iteration (if the number will not divide equally by 8 then round down the

result to the nearest whole number)3 Keep a note of the remainder it should be between 0 and 74 Repeat the steps until the quotient is equal to 05 Write out all the remainders from bottom to top This is the solution

For example if the given decimal number is 8453

Division Quotient Remainder

8453 8 1056 5

1056 8 132 0

132 8 16 4

16 8 2 0

2 8 0 2

Then the octal solution is 20405

Subject Eng Literature (The Tempest ndash William Shakespeare) Topic Act I Scene 1 Lines 33 to 67 (End of scene) Date 16th April 2020 (4th Period)

[Students should read the original play and also the paraphrase given in the school prescribed textbook]Summary Questions amp Answers

[SUMMARY OF THE ENTIRE SCENE]

o The play starts with the scene of a severe storm at sea Alonso (King of Naples) Sebastian (Alonsorsquos brother) Ferdinand (Alonsorsquos son) Gonzalo Antonio (the usurping Duke of Milan) are in a ship in the midst of the storm

o The mariners are trying their best to control the vessel from running aground and are totally following the orders of their Master the Boatswain They have scant success

o The mariners become extremely unhappy and annoyed when most of the passengers arrive on the deck thereby hampering their effort to save the ship There is serious confrontation between them and the passengers who are part of the Kingrsquos entourage

o The mariners could not save the ship

SUMMING-UP

(i) Vivid description of the scene which gives a realistic description of terror and confusion of a tropical storm

(ii) Shows Shakespearersquos accuracy of knowledge in describing the naval operations and also matters of seamanship

(1) GONZALO Ill warrant him for drowning (L 45-57)

though the ship were no stronger than a nutshell and as leaky as an unstanched

wenchBOATSWAIN Lay her a-hold a-hold Set her two courses Off to

sea again lay her offMARINERS All lost To prayers to prayers All lostBOATSWAIN What must our mouths be coldGONZALO The king and prince at prayers Lets assist them

For our case is theirsSEBASTIAN Im out of patienceANTONIO We are merely cheated of our lives by drunkards

This wide-chopped rascal - would thou mightst lie drowning the washing of ten tides

(a) What does Antonio say at the insolent manners of the boatswain just before the given passage

Being irritated at the insolent manners of the boatswain just before the given extract Antonio the Duke of Milan calls him a worthless dog son of a woman without any morals an arrogant and disrespectful noisemaker He says that the boatswain deserved to be hanged(b) What statement does Gonzalo repeat about the boatswain

Gonzalo shows his faith that the boatswain is not destined to die by drowning He is destined to be hanged and nothing can alter this decree of destiny He says that even if the ship was as frail as a nutshell the boatswain could not be drowned for his destiny was to be hanged(c) What do the passengers do when they have lost all hope of their survival

When the passengers have lost all hope of survival they take

(iii) The opening scene justifies the title ndash The Tempest

UNANSWERED QUESTIONS

(i) The King always travels with his entire fleet including his soldiers Where were the other ships

(ii) Why was the ship in that area Where was it coming from or going where

(iii) The ship broke apart What happened to those who were in the ship

(We shall get the answer to the above questions as the play progresses)

leave of life with fervent prayers The mariners take their last hearty drink and are ready for death(d) What blame does Antonio put upon the mariners and the boatswain Antonio rebukes the mariners that these drunkards have brought them to the present crisis by neglecting their duties He blames them saying that they are going to lose their lives entirely for the negligence of the boatswain and his fellows(e) What does Antonio say while cursing the boatswain

Antonio gives vent to his wrath upon the boatswain in particular He calls the boatswain a wide-mouthed rascal who deserves to be hanged on the sea-shore at low water mark so that ten tides might wash over his body and take out of him all the liquor that he has been drinking

Class XIISubject Topic Summary ExecutionHistory Topic

1 1935 ACT AND WORKING OF PROVINCIAL AUTONOMYCONGREE AND OTHER MINISTERSSUB TOPIC GOVERNMENT OF INDIA ACT1935

Government of India Act 1935 This act established a lsquoFederation of Indiarsquo made of British Indian provinces and Indian states and provided for autonomy with a government responsible to the elected legislature in every provinceThis act introduced abolition of Diarchy at provinces The entire provincial administration was introduced to the responsible ministers who were controlled and removed by the provincial legislature The provincial autonomy means two things First The provincial governments were wholly responsible to the provincial legislature Secondly Provinces were free from outside control and interference in the large number of matters The act divided the powers between the centre and provinces in terms of three lists- Federal list( for centre) Provincial list (for province) and concurrent list (for both) Residuary powers were given to the viceroy In the election under the government of India Act the Congress swept the poll the mandate of the people came in favour of the congress so far as general Hindu seats were concerned The Congress did not get a single Muslim seates in Bombay CP UP Sind and BengalIn five provinces Congress had yhe clear majority In BengalNWFPAssam and Bombay Congress emerged as a single largest partyOn the other side the performance of the Muslim League was badThus the Congress formed ministers in 7 provinces out of 11 provinces Coalition ministry was also formed in two other provincesOnly BENGAL AND Punjab had non- congress ministries

1 QUESTION What was the main change introduced by the Government of India ActANSWER a) The Act gave more

autonomy to the provinces b) Diarchy was abolished at the

provincial levelsc) The Governor was the head of

the executived) There was a council of

ministers to advise him The ministers were responsible to the provincial legislatures who controlled them The legislature could also remove the ministers

e) The Governors still retained special reserve powers

2 QUESTION Why did the federal scheme introduced by the Government of India Act 1935 never come into operation

ANSWER The Federal structure of the Government of India was to be composed with the Governor General and Council of ministers The Federal legislature was to be Bicameral legislature- The council of states and the House of Assembly The ministers were to be chosen by the Governor general and they were to hold the office during his pleasure

The provinces of British India would have to join the federation but this was not compulsory for the princely states

This federation never materialised because of the lack of support from the required number of

princely statesThis act was refused and

rejected by the princes the Congress and the Muslim League

Thus both Congress and the League participated in the election of 1937 Thus the federal part was never introduced but the provincial part was put into operations

Bengali 2nd

Language

াচেরর পরাথCনা(কহিতা )

াচেরর পরাথCনা কহিতাটি কহি (ঙখ দেঘাচে4র দো আচো য কহিতায় াচেরর পতর হমায়ন কঠিন দেরাচেগ আxানত ার ঈশবর া আললার কাচেছ পরাথCনা কচেরচেছন তার পচেতরর ীন হিফহিরচেয় হিচেত এই কহিতায় ার পচেতরর ীন হিভbা দেচেয়চেছন ারার এমনহিক হিনচের ীন হিসCচেনর হিহিনমচেয় হিতহিন তার দেছচের ীন হিফচের দেপচেত দেচেয়চেছন তার দেছচের এই দেরাচেগর ন য হিতহিন হিনচেচেকই ায়ী কচেরচেছন তার হিনচের করা পাপচেকই হিতহিন ায়ী কচেরচেছন এছাা রানৈনহিতক ও আথCসামাহিক অসথার কথা তচে ধরা চেয়চেছ এই কহিতায় ার তার হিনচের পাপ কমCচেকই ায়ী কচেরচেছ ার অন যায় ভাচে দেপহি((হিকতর মাধ যচেম অপররা য কচেরচেছ আর এই অন যায় কাচের ন যই তার পহিরাচের হিপযCয় এচেসচেছ দে এক পরকার মানহিক নধন ইহিতাচেসর ার হিপতা চেয় সবাভাহিকভাচে ভাচোাসা দে মমতা দেথচেক মকত চেত পাচেরনহিন তাই হিপতা চেয় আললা া ভগাচেনর কাচেছ পতর হমায়চেনর পরানহিভbা দেচেয়চেছন ার আললা া ভগাচেনর কাচেছ াহিনচেয়চেছন তার হিনচের ীন হিসCন হিচেত হিতহিন রাী তার হিহিনমচেয় পচেতরর ীন হিফচের দেপচেত দেচেয়চেছন াচেরর হিপতসভ হিচেকর কথা এই কহিতায় ফটিচেয় দেতাা চেয়চেছ হিপতা পচেতরর হিরাহিরত মান নধচেনর কথা তচে ধরা চেয়চেছ

হিচে(4 হিকছ াইচেনর তাৎপযC১) ldquoদেকাথায় দেগ ওর সবচছয দেৌন দেকাথায় কচেরায় দেগাপন bয়ldquoউততর) াচেরর পতর হমায়ন কঠিন দেরাচেগ অসসথ তাই তার দেযৌন াহিরচেয় যাচেচছ এই দেরাচেগ তাচেক দেগাপচেন কচেরকচের াচেচছ তার সক (হিকত ধীচের ধীচের bয় চেচছ তাই হিপতা চেয় ার আললার কাচেছ হমায়চেনর পরান হিভbা দেচেয়চেছন২) ldquoাগাও (চেরর পরাচেনত পরানতচের ধসর (ন দেযর আান গানldquoউততর) াচেরর পতর হমায়ন কঠিন দেরাচেগ আxানত তাই ার আ দে(াচেক মমCাত (চেরর পচেথ পরানতচের আান গান ধবহিনত দোক দেসই আান গান আললার কাচেছ দেযন চে যায় আললা দেযন এই আহিতC শচেন পচেতরর ীন হিফহিরচেয় দেয় ৩)ldquoনাহিক এই (রীচেরর পাচেপর ীানচেত দেকানই তরারণ দেনই ভহি4চেতরldquoউততর) হমায়চেনর অসসথতার ন য ার হিনচেচেকই ায়ী কচেরচেছন কারন ার অচেনক রা য অন যায় ভাচে কচেরচেছ তাই তার এই পাপ কাচের ন য তার ঘচের আ হিপ এচেসচেছ এই অন যায় কাচের ন য তার মহিকত দেনই তাই ার আললার কাচেছ এই পাপ কাচেযCর ন য bমা পরাথM

Hindi 2ndlang

-ासी(जयशकर परसा-)

-ासी जयशकर परसा- की एक ऐसी कहानी ह जिजसम भारतीय ससकनित और राषटरीयता का सवरगजीतहोता ह इस कहानी म इरावती एक निहद कनया ह जिजस मलअचछो न मलतान की लट म पकडा और -ासी बना दि-या उस 500 दि-न -कर काशी क एक महाजन न खरी-ा दसरी -ासी निफरोजा ह वह गलाम ह निफरोजा को छडान क कतिलए अहम- को 1000 सोन क कतिसकक भजन थ जो अभी तक नही आए थ राजा साहब कठोर होत हए भी निफरोजा को निबना धनराकतिश क कतिलए उस म कर -त ह वनिफरोजा को अहम- को समझान की बात कहत हकहानी क अत म हम -खत ह निक इरा वती और जाटो क सर-ार बलराज का मिमलन होता हअहम- को यa म मार दि-या जाता ह वहा निफरोजा की परसननता की समामिध बनती ह वहा एक फल चढती ह और डीजल आती ह निफरोजा उस समामिध की आजीवन -ासी बनी रहती हलखक अपन उददशय अथात -ास परथा पर परकाश डालन और इस परथा क कारण होन वाल -ातो क दखो को दि-खान म पणता सफल हए ह

helliphellipContinue to next

Biology Reproductio Today we will discuss about vegetative Q1 Name some vegetative propagules

n in Organisms

propagation of plants The process of multiplication in which fragments of plant body function as propagule and develop into new individual is called vegetative propagation The units of such propagation are runner rhizome tuber bulb etc

and the speciesinvolvedVegetative propagules

Parts involved

Bulb StemBulbil BulbilRhizome Stem Runner Stem Tuber Stem Offset Stem Leaf buds Leaves Suckers Stem

Corns Stem stolon

Q2 State advantages of vegetative propagation

i) Rapid methodii) Sure and easy methodiii) Useful in plants that cannot

produce viable seeds or long seed dormancy

iv) Maintains purity of raceQ 3 Banana fruit is said to be parthenocarpic where as turkey is said to be parthenogenetic WhyBanana develops without fertilization from an unfertilized ovary thus is parthenocarpicIn turkey the ovum or female gamete developinto a new chick without fertilization thus isparthgenetic

Q4 Why is water hyacinth is called as a ldquoTerror of Bengalrdquo Water hyacinth can

propagatevegetatively all over the water body in a short per short period of time This resulted increased biochemicaloxygen oxygen demand of water body causing mortalityof fishes It is very difficult to get rid off them Thus known as terror of Bengal

Chemistry

Solid state GENERAL CHARACTERISTICS OF SOLID STATEIn nature the particular state of matter is governed by two opposing forces at given set of temperature and pressure These forces are intermolecular force of attraction and thermal energy If intermolecular force of attraction is high as compared to thermal energy particles remains in closest position

Intext QuestionsQ1 Classify the following solids as crystalline and amorphous Sodium chloride quartz glass quartz rubber polyvinyl chloride Teflon

A1 Crystalline

and hence very less movement in particles is observed In this case solid state is the preferred state of matter

Let us revise the general characteristics of solid

i) Fixed mass volume and shape

ii) Strong intermolecular force of attraction

iii) Least intermolecular space

iv) Fixed position of constituent particles

v) Incompressible and rigid

Q2 what type of interactions hold the molecules together in a polar molecular solid[CBSE 2010]A2 The molecules in a solid are held together by van der Waals forces The term van der Waals forces include hydrogen bonding dipole-dipole attraction and London dispersion forces All molecules experience London dispersion forces In addition polar molecules can also experience dipole-dipole interactions So the interactions that holds the molecule together in polar molecular solid are London dispersion force and dipole-dipole interactionsQ3 Write a feature that will distinguish a metallic solid from an ionic solid [CBSE 2010]A3 Metals are malleable and ductile whereas ionic solid are hard and brittle Metallic solid has typical metallic lustre But ionic solid looks dullQ4 Write a point of distinction between a metallic solid and an ionic solid other than metallic lustre [CBSE 2012]A4 Metals are malleable and ductile whereas ionic solid are hard and brittleQ5 Write a distinguish feature of metallic solid [CBSE 2010]A5 The force of attraction in

solid Sodium chloride Quartz Amorphous solid Quartz glass rubber polyvinyl chloride Teflon Q2 why glass is considered as super cooled liquidA2 Glass shows the tendency to flow at slower rate like liquid Hence they considered as super cooled liquidQ3 why the window glass of old buildings show milky appearance with timeA3 Glass is an amorphous solid Amorphous solid has the tendency to develop some crystalline character on heating Due to heating in day over the number of years glass acquires some crystalline character and show milky appearanceQ4 why the glass panes fixed to window or doors of old building become slightly thicker at bottomA4 Glass is super cooled liquid It has the tendency to flow down very slowly Due to this glass pane becomes thicker at the bottom over the timeQ5 Sodium chloride is a crystalline solid It shows the same value of refractive index along all the direction TrueFalse Give reasonA5 FalseCrystalline solid shows anisotropy in properties That is it shows different values for the given physical property in different direction All the crystalline solids show anisotropy in refractive index Therefore sodium chloride will show different values of refractive index on different directions

Q6 Crystalline solid are anisotropic in nature What does this statement means

between the constituent particles is special kind of electrostatic attraction That is the attraction of positively charged kernel with sea of delocalized electronsQ6 which group of solid is electrical conductor as well as malleable and ductile [CBSE 2013]A6 Metallic solidQ7 why graphite is good conductor of electricity although it is a network (covalent solid)A7 The exceptional property of graphite is due to its typical structure In graphite each carbon is covalently bonded with 3 atoms in same layer The fourth valence electron of each atom is free to move in between different layersThis free electron makes the graphite a good conductor of electricity

[CBSE 2011]A6 Anisotropy is defined asrdquo Difference in properties when measured along different axis or from different directionsrdquo Crystalline solid show different values of some of the physical properties like electrical resistance refractive index etcwhen measured along the different directions The anisotropy in crystalline solid arises due to the different arrangement of particles in different directions

Math Function Composition of functions Think of an industrial plant that produce bottles of cold drinks first there is the operation (or function) f that puts the cold drink inside the bottle followed by the opeartion g that close the bottle with the capThis leads to the following definitionDefinition Let f A rarr B and g B rarr C be two functions Then the composition of f and g denoted by gof is defined as the function gof A rarr C given by gof(x) = g(f (x)) forall x isinA

Definition A function f X rarr Y is defined to be invertible if there exists a function g Y rarr X such that gof = IX and fog = IY The function g is called the inverse of f and is denoted by f -1

Thus if f is invertible then f must be one-one and onto and conversely if f is one-one and onto then f must be invertible This fact significantly helps for proving a function f to be invertible by showing that f is one-one and onto specially when the actual inverse of f is not to be determined

Example 1 Let f 2 3 4 5 rarr 3 4 5 9 and g 3 4 5 9 rarr 7 11 15 be functions defined as f(2) = 3 f(3) = 4 f(4) = f(5) = 5 and g (3) = g (4) = 7 and g (5) = g (9) = 11 Find gofSolution We have gof(2) = g (f(2)) = g (3) = 7 gof(3) = g (f(3)) = g (4) = 7gof(4) = g (f(4)) = g (5) = 11 and gof(5) = g (5) = 11Example 2 Find gof and fog if f R rarr R and g R rarr R are given by f(x) = cos x and g (x) = 3x2 Show that gof ne fogSolution We have gof(x) = g(f(x))=g(cosx) = 3 (cos x)2

= 3 cos2 x Similarly fog(x)=f(g (x))= f(3x2)= cos (3x2) Note that 3cos2 x ne cos 3x2 for x = 0 Hence gof ne fogExample 3 Show that if f A rarr B and g B rarr C are onto then gof A rarr C is also ontoSolution Given an arbitrary element z isin C there exists a pre-image y of z under g such that g (y) = z since g is onto Further for y isin B there exists an element x in A with f(x) = y since f is onto Therefore gof(x) = g (f(x)) = g (y) = z showing that gof is onto Example 4 Let Y = n2 n isin N sub N Consider f N rarr Y as f(n) = n2 Show that

f is invertible Find the inverse of fSolution An arbitrary element y in Y is of the form n2 for some n isin N This implies that n =radicy This gives a function g Y rarr N defined by g (y) =radicy Nowgof (n) = g (n2)=radicn2 = n and fog (y) =f(radicy) = (radicy) 2 y which shows that gof=IN and fog= IY Hence f is invertible with f -1 = g

Political Science

Constitution of India-The Preamble

Summary

Objective of the state-To secure equality of status and of opportunity To promote fraternity among all the citizens To assure the dignity of the individuals and Unity and integrity of the nation

Justice-Justice stands for rule of law absence of arbitrariness and a system of equal rights freedom and opportunities for all in a society India seeks social economic and political justice to ensure equality to its citizens

Liberty-Liberty implies the absence of restraints or domination on the activities of an individual such as freedom from slavery serfdom imprisonment despotism etc The Preamble provides for the liberty of thought expression belief faith and worship

Equality-Equality means the absence of privileges or discrimination against any section of the society The Preamble provides for equality of status and opportunity to all the people of the country

Fraternity-The Preamble declares that fraternity has to assure two thingsmdashthe dignity of the individual and the unity and

Execution

Answer the following questions-

Short notes-1 Equality2 Fraternity3 Justice4 Liberty

Homework-Learn

integrity of the nation The word integrity has been added to the Preamble by the 42nd Constitutional Amendment (1976)

Business studies

Human resource management (chapter 1)

On the day of 1504 2020 I have discussed with you the managerial functions and procurement functions of HRM

Today weare going to discuss about the development function integration functions and maintenance function

Development functions-HRM improves the knowledge skills attitude and values of employees so that they the present and future jobs more effectively it includes

1) Development functions of HRM

a) Performance appraisal = It implies systematic evaluation of employees with respect to their performance on the job and their potential for development

b) Training =It is the process by which employees learn knowledge skills and attitudes to achieve organisational and personal goals

c) Executive development = It is the process of developing managerial talent through appropriate program

2) Integration functionsa) HRM reconcile the goals of

organisation with those of its members through integrating function

b) HRM tries to motivate employees to various financial and non financial incentives provided in job specification etc

3) Maintenance functiona) HRM promote and protect the

physical and mental health of employees by providing several types of benefits like housing medical aid etc

b) It Promote Social security measures to employees by providing provident fund pension gratuity maternity benefits

SubjectCOMMERCE

Topic

BUSINESSENVIRONMENT

Summary

Now quickly let us revise the earlier points that we have already done in the last class and let us proceed with the other topics that are there in the chapter

Firstly we will recall the internal and external factors of micro environment and then we

Execution 3 What do you mean by internal factors

in micro environmentAnswerInternal factors refer to all the factors existing within a business firm The internal factors are considered controllable because the enterprise has control over these factors

Development FunctionsPerformance AppraisalTrainingExecution Development

shall proceed in details

Meaning and list of internal and external factors

aInternal factorsInternal factors refer to all the factors existing within a business firm The internal factors are considered controllable because the enterprise has control over these factorsFor an example a company can alter its organization structure policies programmes employees physical facilities and marketing mix to suit the changes in the environmentList of internal factors areCorporate culture mission and objectives top management organizations structure company image and brand equity company resources

b External factorsExternal factors refer to those individual and groups and agencies with which a particular business organization comes into direct and frequent contact in the course of its functioningThese individuals and groups are known as STAKEHOLDERS because they have a stake (financial interest ) in the working and performance of the particular business List of external forces (stakeholders)Customers competitors investors suppliersmiddlemen (marketing intermediaries)financers publics

customers

suppliersfinancers

For an example a company can alter its organization structure policies programmes employees physical facilities and marketing mix to suit the changes in the environment

4 What do you mean by external factors in micro environment

AnswerExternal factors refer to those individual and groups and agencies with which a particular business organization comes into direct and frequent contact in the course of its functioningThese individuals and groups are known as STAKEHOLDERS because they have a stake (financial interest) in the working and performance of the particular business

3Who are stakeholdersSTAKEHOLDERS are individuals and groups who have a stake (financial interest ) in the working and performance of the particular business 4Discuss the internal factors in briefa Corporate CultureThe values beliefs and attitudes of the founders and top management of the company exercise a strong influence on what the cmpaany stands for how it does things and what it considers importantbMission and objectivesThe business philosophy and purpose of a comoany guide it prioritiesbusiness strategiesproduct market scope and development scope

cTop management structurethe composition of board of directors the degree of professionalization of management and the organizational structure of a company have important bearing on its business decisions

dPower structureThe internal power relationship between the board of directors and the chief executive is an important factor

e Company image and brand equityThe image and brand equity of the company play a significant role in raising finance forming alliance choosing dealers and suppliers launching new products entering foreign markets

5 What is Macro environmentAnswerMacro environment refers to the general

competitors

middlemen

publics

Fig STAKEHOLDERS OF A COMPANY

Apart from micro environment the other main dimension of business environment isMacro environment Macro environment refers to the general environment or remote environment within which a business firm and forces in its micro environment operateA company does not directly or regularly interact with the micro environmentTherefore macro environment is also known as indirect action EnvironmentThe macro environment forces are less controllable than the micro forces

Macro environment consists of the following components

POLITICAL AND LEGAL ENVIRONMENT

ECONOMIC SOCIAL AND ENVIRONMENT

CULTURAL

ENVIRONMENT

TECHNOLOGICAL ENVIRONMENT

Fig COMPONENTS OF MACRO ENVIRONMENT

environment or remote environment within which a business firm and forces in its micro environment operateA company does not directly or regularly interact with the micro environmentTherefore macro environment is also known as indirect action EnvironmentThe macro environment forces are less controllable than the micro forces 6 What are the components of macro environmenta Political and legal environmentb Economic environmentc Social and cultural environmentd Technological environment

Computer Science

Logic gates

Digital systems are said to be constructed by using logic gates These gates are the AND OR NOT NAND NOR EXOR and EXNOR

BUSINESS FIRM

gates The basic operations are described below with the aid of truth tables

AND gate

The AND gate is an electronic circuit that gives a high output (1) only if all its inputs are high A dot () is used to show the AND operation ie AB Bear in mind that this dot is sometimes omitted ie ABOR gate

The OR gate is an electronic circuit that gives a high output (1) if one or more of its inputs are high A plus (+) is used to show the OR operationNOT gate

The NOT gate is an electronic circuit that produces an inverted version of the input at its output It is also known as an inverter If the input variable is A the inverted output is known as NOT A This is also shown as A or A with a bar over the top as shown at the outputs The diagrams below show two ways that the NAND logic gate can be configured to produce a NOT gate It can also be done using NOR logic gates in the same way

NAND gate

This is a NOT-AND gate which is equal to an AND gate followed by a NOT gate The outputs of all NAND gates are high if any of the inputs are low The symbol is an AND gate with a small circle on the output The small circle represents inversion

NOR gate

This is a NOT-OR gate which is equal to an OR gate followed by a NOT gate The outputs of all NOR gates are low if any of the inputs are highThe symbol is an OR gate with a small circle on the output The small circle represents inversion

EXOR gate

The Exclusive-OR gate is a circuit which will give a high output if either but not both of its two inputs are high An encircled plus sign ( ) is used to show the EOR operation

EXNOR gate

The Exclusive-NOR gate circuit does the opposite to the EOR gate It will give a low output if either but not both of its two inputs are high The symbol is an EXOR gate with a small circle on the output The small circle represents inversion The NAND and NOR gates are called universal functions since with either one the AND and OR functions and NOT can be generated

Note A function in sum of products form can be implemented using NAND gates by replacing all AND and OR gates by NAND gates A function in product of sums form can be implemented using NOR gates by replacing all AND and OR gates by NOR gates

Logic gate symbols

Table 2 is a summary truth table of the inputoutput combinations for the NOT gate together with all possible inputoutput combinations for the other gate functions Also note that a truth table with n inputs has 2n rows You can compare the outputs of different gates

Logic gates representation using the Truth table

Example

A NAND gate can be used as a NOT gate using either of the following wiring configurations

Subject Eng Literature (The Tempest ndash William Shakespeare) Topic Act III Scene 3 Lines 53 to 110 (End of the scene) Date 16th April 2020 (2nd Period)

[Students should read the original play and also the paraphrase given in the school prescribed textbook]Summary Questions amp Answers

o Seeing this strange scene all are inclined to believe the tales told by travelers that there truly are ldquounicornsrdquo and ldquothe phoenixrsquo thronerdquo

o As they are about to sit down to the feast the banquet is snatched away by a harpy (Ariel disguised) A spiritrsquos voice (Arielrsquos voice) denounces Alonso Sebastian and Antonio with particular

1 ARIEL You are three men of sin whom Destiny

(Line 53-58)That hath to instrument this

lower world And what is int the never-surfeited sea

Hath caused to belch up you and on this island

Where man doth not inhabit you rsquomongst men

Being most unfit to live I have made you mad

reference to their crime in expelling Prospero from Milan They have not received any punishment for their deed earlier but the time for their punishment has arrived Upon Alonso it pronounces ldquolingering perdition worse than deathrdquo from which there is no remedy except through sincere repentance Ariel then vanishes in thunder and the shapes enter again and carry away the table

o Prospero watching invisibly is very pleased with the performance of Ariel and his (Prosperorsquos) ldquomeaner ministersrdquo All his enemies are now in his power and are in a fit of desperation He then leaves them and goes to see how Ferdinand and Miranda are getting on

o Alonso is now much humbled and penitent with the after effect of the spiritrsquos denunciation of his crimes He believes that his son is lost forever After this all disperse being stricken mad by the speech of the spirit

o Gonzalo fearing that they may do violence to themselves or to one another follows them and bid others to follow

(a) To whom does Ariel disguised as a harpy call the three sinners What game did Fate of Destiny play with

them

The three sinners called by Ariel are Alonso Sebastian and Antonio It was Destiny which had caused the ocean to cast the three sinners on the shore Though the ocean is all the time devouring whatever appears on its surface and is never satisfied with its continual swallowing of the ships and men in the present case the ocean had cast these three sinners on the shore without killing them

(b) Who had jointly been responsible for the conspiracy against Prospero What is Prosperorsquos purpose behind all this

Three men Alonso Sebastian and Antonio had jointly

been responsible for the conspiracy against Prospero They had driven out Prospero form Milan Prosperorsquos purpose is to make these three sinners realize the wrong they had done He wants them to repent for their criminal deeds because repentance leads to self-esteem(c )What does Ariel (the harpy) tell Alonso and his companions when they take out their swords to attack him

Seeing them drawing their swords Ariel (harpy) tells them that he and his companions are the instruments of destiny and that it is not possible for human beings to do them any injury He says that the swords of human beings can not injure even a minute part of his feathers Their swords are as ineffective against him and his companions as against the wind or the water

(d) Give the explanatory meanings of the following expressions in the context of the above extract

(i)Never surfeited (ii) Belch up (iii) lsquomongst men

(i) Never surfeited never led to satisfaction

(ii) Belch up cast ashore(iii) lsquomongst men in human

society2

I and my fellows (Line 60-65)

Are ministers of Fate The elementsOf whom your swords are tempered may as wellWound the loud winds or with bemocked-at stabsKill the still-closing waters as diminishOne dowl thats in my plume

IMPORTANT PASSAGES EXPLAINED

The elements

(Line 61-66)Of whom your swords are tempered may

as wellWound the loud winds or with

bemocked-at stabs

(a) Who is lsquoIrsquo Who are his lsquofellowsrdquo

lsquoIrsquo is referred to Ariel in disguise of a harpy His lsquofellowsrsquo are other spirits serving Prospero the real Duke of Milan who has acquired supernatural powers after being banished from his Dukedom Prospero has settled in this uninhabited island

(b) What are the elements that have temperrsquod the swords Why will it not work against the speaker

The swords (of Alonso and his companions) are tempered by metal (steel) which is taken out of the earth and refined by

Kill the still-closing waters as diminishOne dowl thats in my plume My fellow

ministersAre like invulnerable

In these words Ariel reminds the King and his companions of the utter futility of drawing swords against himself and his fellows Ariel drives Alonso Antonio and Sebastian the three men of sin to desperation ndash a state in which men do violence to themselves They draw swords to strike Ariel But Ariel reminds them that he and the other spirits are the ministers of destiny and nothing can wound them The steel of which their swords are made of may cut the wind or water which being divided always closes up again Even supposing that such things may be possible it is quite impossible that their swords will cut one feather in their plume They are incapable of being wounded by any sword of man Hence it is foolish on their part to attempt to strike at Ariel and his fellow-spirits

For which foul deed

(Line 72-75)The powers delaying not forgetting

haveIncensed the seas and shores yea all the

creatures Against your peace

Ariel enters like a harpy and remaining invisible tells Alonso Sebastian and Antonio that he and other harpies are the agents of Destiny appointed to carry out her decrees He tells them that their punishment for the crime against Prospero which has been so long deferred is now to fall upon them He reminds them that they had expelled Prospero from Milan and set him and his innocent child adrift on the sea and that the sea had paid them back for their sin by the shipwreck and by the calamities they have suffered He tells them that the powers above which did not forget this mean treachery but only deferred the punishment have now engaged the seas and the shores and all living beings including him and his comrades against them The very elements and supernatural agency Ariel adds have taken up the avenging of their crime against Prospero

the action of fire It may cut the wind or water which being divided always closes up again

The sword will not work against the spirits and the harpy because they are the ministers of destiny and nothing can wound them nor it will cut a single feather in their plume

(c )What is the meaning of lsquodowlrsquo in the last line

The term lsquodowlrsquo means a filament or the smallest part of a feather In this context Ariel in disguise of harpy says that their sword cannot even damage the smallest filament of their (Arielrsquos and other spirits) feathers as they are incapable of being wounded by any sword of man

(d) What does the speaker remind the listeners about

Ariel in disguise of harpy reminds Alonso the King of Naples Sebastian Alonsorsquos brother and Antonio the present Duke of Milan and the treacherous brother of Prospero as they being three men of sin He even reminds them that their punishment for their crime against Prospero which has been so long deferred now falls upon them He reminds them that they have expelled Prospero from Milan and has set him along with his innocent infant daughter adrift on the sea So the sea has paid them back for their sin by their shipwreck and the calamities they have suffered since then The harpy rebukes Alonso of his sin that has incensed the Gods and has deprived him of his son as a punishment

(e) How do they respond

When Ariel in disguise of a harpy reminds Alonso Sebastian and Antonio of their past misdeeds and sin Alonso has a look of terror and confusion in his eyes He utters the words of sincere repentance wrung out of his conscience-stricken heart It appears to him that all the elements of nature the sea-waves the wind and the thunder proclaiming a loud voice in the name of Prospero and the crime Alonso has committed against him They are calling upon him to repent There is a deep storm raging in Alonsorsquos breast and the echoes of that storm are ringing in his ears like a clear note of wind-instrument A note of denunciation of Alonsorsquos crime leaves him much humbled and penitent and confirms his belief that his son is lost forever But Sebastian and Antonio shows some courage instead of repentance They wish to kill the spirits or devils if it appears

3

Of my instruction hast thou nothing bated (Line 85-93)

In what thou hast to say So with good life

And observation strange my meaner ministers

Their several kinds have done My high charms work

And these mine enemies are all knit upIn their distractions They now are in my

powerAnd in these fits I leave them while I visitYoung Ferdinand whom they suppose is

drownedAnd his and mine loved darling

Methought the billows spoke and (Line 96-99)

told me of itThe winds did sing it to me and the

thunderThat deep and dreadful organ-pipe

pronouncedThe name of Prosper It did bass my

trespass

These are the words of contrition coming from Alonso Ariel has driven him to a deep repentance for conspiring with Antonio against Prospero He now feels a sincere remorse It appears to him that all the elements of nature the sea-waves the wind and the thunder proclaimed with a loud voice the name of Prospero and the crime Alonso had committed against him They are calling upon him to repent There is a deep storm raging in Alonsorsquos breast and the echoes of that storm are ringing in his ears like the clear note of a wind-instrument

Comment These are the words of sincere repentance wrung out of the conscience-stricken heart of Alonso Alonso who is the lesser villain is the first to give way to remorse under the effect of Arielrsquos speech The words of Ariel seem to him to be the voice of conscience speaking to him He is driven to desperation a state in which he might do violence to his life

(a) Identify the speaker State the context

Prospero the ruler of the island is the speaker The famous banquet scene has been enacted very well Ariel and his junior spirits have played their roles excellently Prospero is glad to say words of praise for them(b) In what way the speakerrsquos instructions have been carried out

According to Prosperorsquos instructions a banquet was presented before the King of Naples and his companions when they were tired and hungry Just when they were preparing to eat the feast the banquet was suddenly removed by exercising supernatural powers All this was done by Ariel Prosperorsquos chief assistant and a powerful spirit

Ariel not only made the feast disappear but also delivered his speech blaming the King and his two companions for their past wicked deeds He warned them to repent for their misdeeds or suffer forever on that uninhabited island

(c) Who are referred to as lsquomeaner ministersrsquo What have they done

Prospero refers as lsquomeaner ministersrsquo to his other lesser spirits who were assisting Ariel in presenting a scene before the kingrsquos party They entered the scene to the accompaniment of music They assumed several strange shapes and brought in a banquet Then they danced about it with gentle actions of salutations thus inviting the King and others to eat the feast

These spirits play their role again when Ariel in the shape of a harpy quits the scene These shapes enter again and dancing with mocking gestures carry away the table

(d) Who are the speakerrsquos enemies What has happened to them

King of Naples Alonso his brother Sebastian and the present Duke of Milan Antonio (Prosperorsquos own brother) are Prosperorsquos enemies With the turn of events they have all been washed ashore on the island which is ruled by Prospero the great magician Actually this happened after the shipwreck caused by a storm which was raised by Prospero with the purpose of bringing these people to his island Prosperorsquos spirits have already confused and terrified these enemies and they are under Prosperorsquos control He can treat them as he likes

(e) What does he say about Ferdinand Explain what is meant by ldquohellip his and mine darlingrdquo

Prospero knows that Alonsorsquos son prince Ferdinand is alive though his father thinks that the prince has been drowned

Prospero refers to his daughter Miranda who is dear to him She is also very dear to Prince Ferdinand who has fallen in love with her They are waiting to be married soon for which they have received Prosperorsquos consent

4

ALONSO O it is monstrous monstrous (Line 95-102)

Methought the billows spoke and told me of it

The winds did sing it to me and the thunderThat deep and dreadful organ-

pipe pronouncedThe name of Prosper It did bass

my trespassTherefore my son ithrsquo ooze is

bedded andIll seek him deeper than eer

plummet soundedAnd with him there lie mudded

(a) In what way does Alonso express his horror when his conscience is awakened by Arielrsquos words

When Alonsorsquos conscience is awakened by Arielrsquos words he expresses his horror at what he has heard He gets the feeling that the waves of the ocean the wind and the loud thunder have spoken to him and uttered the name of Prospero Because of being reminded of his crime in a very loud and rough voice he comes to realize that he has lost his son for his past misdeeds

(b) What does Alonso imagine about his son What does Alonso want to do in his desperate state

Alonso imagines that his son is lying in the mud at the bottom of the sea He feels desperate that he wants to drown himself in the ocean deeper than the plumb-line has ever gone He wants to lie with his son at the bottom of the sea

(c) How do Sebastian and Antonio want to face the evil spirits

Sebastian says that he is not at all afraid of what the harpy has said and that he is prepared to fight any number of such monsters if they appear before him only one at a time Antonio says that he would support Sebastian in the fight against the fiendsyyy

(d) Why does Gonzalo ask Adrian to follow the three men

Gonzalo tells Adrian that all the three men namely Alonso Sebastian and Antonio are in a wild and reckless mood The thought of the heinous crime of which they are guilty has begun to torment their minds So he asks Adrian to follow those three men without loss of time and prevent them from doing anything which the turmoil in their minds might lead them to do

(e) What opinion do you form of Alonso from the above extract

Alonso who is the lesser villain is the first to give way to remorse under the effect of Arielrsquos speech The words of Ariel seem to him to be the voice of conscience speaking to him He is driven to desperation a state in which he might do violence to his life

Subject =Accounts

Ac-12 15420 topic-pL Appropriation ac

PROFIT AND LOSS APPROPRIATION ACCOUNT

MEANING AND PREPARATIONProfit and Loss Appropriation Account is merely an extension of the Profit and Loss Account of the firm The profit of the firm has to be distributed amongst the partners in their respective profit sharing ratio But before its distribution it needs to be adjusted All Adjustments like partnerrsquos salary partnerrsquos commission interest on capital interest on drawings etc are made in this account These adjustments will reduce the amount of profit for distribution This adjusted profit will be distributed amongst the partners in their profit sharing ratio To prepare it at first the balance of Profit and Loss Account is transferred to this account The journal entries for the preparation of Profit and Loss Appropriation Account are given below

1 for transfer of the balance of Profit and Loss Account to Profit and Loss Appropriation Account

(a) In case of Net Profit

Profit and Loss Ac helliphelliphelliphelliphellipDrTo Profit and Loss Appropriation Ac(Net Profit transferred to Profit and Loss Appropriation Ac)

(b)In case of Net Loss

Profit and Loss Appropriation Achelliphelliphellip DrTo Profit and Loss Ac(Net Loss transferred to Profit and Loss Appropriation Ac)

2 for Interest on Capital

For transferring on Interest on CapitalProfit and Loss Appropriation Achelliphelliphellip DrTo Interest on Capital Ac(Interest on capital transferred to Profit amp Loss Appropriation Ac)

3 for Interest on Drawings

For transferring Interest on Drawings Interest on Drawings Achelliphelliphelliphelliphelliphellip DrTo Profit and Loss Appropriation Ac(Interest on drawing transferred to Profit amp Loss Appropriation Ac)

4 For Partnerrsquos SalaryFor transfer of partnerrsquos SalaryProfit and Loss Appropriation Achelliphellip DrTo Salary Ac(Salary transferred to profit amp Loss Appropriation Ac)

5 For Partnerrsquos CommissionFor transferring commissionProfit and Loss Appropriation Achelliphelliphellip DrTo Commission Ac(Commission transferred to Profit and Loss Appropriation Ac)

6 For Transfer of agreed amount to General ReserveProfit and Loss Appropriation Ac helliphellipDrTo General Reserve Ac(Transfer to General Reserve)

7 for share of Profit or Loss appropriation(a) If ProfitProfit and Loss Appropriation Achelliphellip DrTo Partnerrsquos CapitalCurrent Ac(Profit transferred to capitalcurrent Ac)(b) If LossPartnerrsquos Capital Current Achelliphelliphelliphellip DrTo Profit and Loss Appropriation Ac(Loss transferred to capitalcurrent Ac)

THE FORMAT OF PROFIT AND LOSS APPROPRIATION

Profit and Loss Appropriation Account for the year endedhelliphelliphelliphellip

Particulars Amount Particulars Amount

To PL Ac (loss) By pL Ac (profit)

To Interest on capital BY Interest on drawings

To partner`s commission by Partner`s capital Ac ( loss)

To Partner`s salary To Interest on partner`s loan To General Reserve To Partner`s Capital AC (Profit)

Subject= Economics

MOVEMENT ALONG THE DEMAND CURVE (CHANGE IN QUANTITY DEMANDED)In law of demand you have already studied the inverse relationship between price and quantity demanded When quantity demanded of a commodity changes due to change in its price keeping other factors constant it is called change in quantity demanded It is graphically expressed as a movement along the same demand curve There can be either a downward movement or an upward movement along the same demand curve Upward movement along the same demand curve is called contraction of demand or decrease in quantity demanded and downward movement along the same demand curve is known as expansion of demand or increase in quantity demanded

Extention of demandd

price (rs)p A

B Extentionp1 d

Q Q1

Quantity demanded ( in units)

Contraction of demandd

p2 Ccontraction

p APrice (Rs)

d

Q2 Q

Quantity demanded (in units)

Explanation of movement of demand A fall in price from OP to OP1 leads to increase in quantity demanded from OQ to OQ1 (expansion of demand) resulting in a downward movement from point A to point B along the same demand curve DD When Price rises from OP to OP2 quantity demanded falls from OQ to OQ2 (contraction of demand) leading to an upward movement from point A to point C along the same demand curve DD

  • Activity Series of Metals
    • Drawbacks of Rutherfordrsquos model of atom
      • Electromagnetic radiations
      • Properties of electromagnetic radiations
      • Characteristics of electromagnetic radiations
        • Plancks Quantum Theory-
        • Photoelectric effect
          • Intext Questions
            • Logic gates
            • Digital systems are said to be constructed by using logic gates These gates are the AND OR NOT NAND NOR EXOR and EXNOR gates The basic operations are described below with the aid of truth tables
            • AND gate
            • Example
Page 5:  · Web viewSubject. Topic. Summary. Execution. English 1 . Chapter 1 naming words . Page 8. Write the names of these pictures:- Person:-1. father. 2.Firefighter 3.doctor 4 ...

tens 3 ones= 4 hundreds ___ tens 8 ones - 3 hundreds 8 tens 3 ones= 1 hundreds ___ tens ___ ones

Solution5 hundreds 2 tens 8 ones ndash 3 hundreds 8 tens 3 ones= 4 hundreds 12 tens 8 ones - 3 hundreds 8 tens 3 ones= 1 hundreds 4 tens 5 ones

Class IVSubject Topic Summary Execution

English literature

Hercules andDeianira (tales from Greece and Rome)

Hercules spent the greater part of his life in helping the poor and weak people But Juno still wanted to harm him So she sent him into all sorts of dangers But he was brave and strong and he overcame all the dangers Juno made him a slave to the king of Argos Hercules worked hard for the king The king took pity on him and told him that he would set him free if Hercules perform twelve difficult tasks Hercules accepted the challenge

1How did Hercules spent the greater part of his lifeAns Hercules spentthe greater part of his life in helping the poor and weak people

2How did Juno tried to harm HerculesAns Juno tried to harm Hercules by sending him into all sorts of dangers

3Under which king Hercules was working as a slaveAns King of Argos

4What was the challenge that king of Argosgive to HerculesAns King of Argos told Hercules that he would set him free if he perform twelve difficult tasks which Hercules accepted

Social studies Major landforms on earth

Answer the following questions5What do oceans provide usAns Oceans provide us a variety of sea food

6What is alluviumAns The rivers carry sand soil and small pieces of rocks This fine soil or silt known as alluvium is very good for growing crops

COMPUTER160420

CHAPTER 3 EDITING IN MS WORD

DRAG AND DROPTHE DRAG AND DROP METHOD OF MOVING THE TEXT ALLOWS US TO MOVE THE SELECTED TEXT USING MOUSE FROM ONE LOCATION TO ANOTHER WITHIN A DOCUMENT

Q7) HOW TO WE DRAG AND DROP TEXT IN MS WORDAns) WE CAN DRAG AND DROP TEXT IN MS WORD FOLLOWING STEPS ARE---

SELECT THE TEXT PLACE THE MOUSE POINTER

ANYWHERE ON THE SELECTED TEXT

CLICK AND HOLD THE LEFT MOUSE BUTTON UNTIL THE INSERTION POINT CHANGES INTO A WHITE ARROW

WHEN WE REACH THE PLACE IN THE DOCUMENT WHERE WE WANT TO MOVE THE TEXT RELEASE THE MOUSE BUTTON

Science Adaptations in Summary Execution

Animals Animals are divided into different groups according to their habitats Some animals live on land some animals live both on land and in water some animals adapted themselves for food and some for protection They develop special features that help them to survive in their habitats Animals that live on land are called Terrestrial animals such as lions tigers Yaks polar bears Polar Bears live cold polar regions Yaks live in mountains These animals have thick fur on their body and a thick layer of fat under their skin which keeps them warm

Solved exercise question and answersA Write the correct answer1Terrestrial animals live here - on land 2This animal lives in polar regions ndash polar bear 3 These help a fish to breathe in water - gills4 This helps carnivorous birds to tear flesh - a hooked beak

MATHEMATICS

Ch 6Multiplication

Exercise 17 Example 2

Multiply 224835 by 4 Solution

2 2 4 8 3 5 times 48 9 9 3 4 0

Exercise

18 3 1 1 4 6 7 19 2 1 2 7 8 5 times 3 times 4 9 3 4 4 0 1 8 5 1 1 4 0

20 2 0 6 1 3 times 9 1 8 5 5 1 7

Class VSubject Topic Summary Execution

Science Chapter 2 - The Skeletal System

Posture

The way in which we hold our body while sitting standing walking and lying down is known as posture

Importance of correct posture and exercise

1 Correct posture and exercise makes our bones and muscles strong 2 It provides our body more energy to do works

3 It keeps our body stressless 4 It also prevent back ache and muscle pain

Some important yoga asanas are-

1 Veerabhadrasana ( Warrior pose )

2 Dhanurasana ( Bow pose )

3 Trikonasana ( Triangle pose )

D Answer these questions

5How do our muscle work

Ans ndash Our muscles work by contracting and relaxing

6Why should we maintain a correct posture

Ans ndash We should maintain a correct posture because ndash

i Correct posture and exercise makes our bones and muscles strong

ii It provides our body more energy to do works

iii It keeps our

body stressles

s iv It also prevent

back ache and muscle pain

Social studies Conquering distances

Put a tick on the correct option1 b Ship 2b water transport 3 b trains 4 c 13 5 b Wright brothersTrue or false1 False 2 True 3 True 4 True

MATHEMATICS

Ch 3Addition and Subtraction

Exercise 126 A school needs Rs 4987653 for its building It has only Rs 3592468 in its accounts Estimate the money by rounding off to the nearest lakh it has still to raise

Solution Estimated amount need Rs 5000000 The school has only Rs 3600000 Still to raise Rs 1400000

MATHEMATICS

Ch 4

Mul

tiplic

ation

and

Div

ision

We have learnt multiplication tables up to 15 Let us extend the tables up to 20X 11 12 13 14 15 16 17 18 19 201 11 12 13 14 15 16 17 18 19 202 22 24 26 28 30 32 34 36 38 403 33 36 39 42 45 48 51 54 57 604 44 48 52 56 60 64 68 72 76 805 55 60 65 70 75 80 85 90 95 1006 66 72 78 84 90 96 102 108 114 1207 77 84 91 98 105 112 119 126 133 1408 88 96 104 112 120 128 136 144 152 1609 99 108 117 126 135 144 153 162 171 180

10 110 120 130 140 150 160 170 180 190 200Properties of Multiplication1 The product of two numbers does not change when the order of numbers is changed eg 503times23 = 23times503 [This property is called Commutative Property of multiplication]

2 The product of three numbers does not change when the grouping of numbers is change eg (15times18iquesttimes10=15 times(18times 10) = (15times10iquesttimes18 [This property is called Associative Property of multiplication]

3 The product of a number and 1 is the number itself eg 1513 times 1 = 1513 [This property is called Identity Property of multiplication and the integer 1 is called Identity Element of multiplication]

4 The product of a number and 0 is 0 eg 718205times 0 = 0 times 718205 = 05 The product of a number by the sum of two numbers is equal to the sum of the

products of that number by the two numbers separately eg123 times (105+ 48) = 123 times 105 + 123 times 48 [ This property is called Distributive Property of multiplication over

addition]English language

Transitive and intransitive verb

Pick out the verbs from the following sentences and say whether they are transitive or intransitive verb6 Was fullndash verb Intransitive verb7 Have been decorated- verb Transitive verb8 Happy cheerful ndash verb Intransitive verb9 Shall come back ndash verb Transitive verb

English 2 The fall of Lanka

This is the story of the fight between Rama and Ravana as told by Valmiki in lsquoThe Ramayana The monkeys worked all day and all night and at last built a bridge so that Rama and

Write the synonyms of

1 Stationed-

his army could cross to Lanka and rescue Sita assign2 Invade- enter

a country or a region so as to subjugate or occupy it

3 Prowess- bravery in battle

4 Haughty- arrogantly

5 Puny- small and weak

6 Spy- secret agent

7 Dreadful- causing or involving great suffering

8 Violate- disobey

9 Ghastly- causing great horror or fear

10 Deception ndash misleading

Class VISubject Topic Summary Execution

HISTORY AND CIVICS

CHAPTER 3

MAHAVIRA AND BUDDHA ndash GREAT PREACHERS

BUDDHA

Impact of Buddhism on Indian Cultures

Impact on Religion

Buddharsquos practical and simple doctrines made their impact on HinduismThe principle of ahimsaIt brought about a great change in the performance of costly yanjnas and sacrifices which previously involved immense loss of life The Mahayana Buddhists adopted the practice of worshipping Buddha and bodhisattvas making idols and erecting temples in their honour

Impact on Literature

After the death of Buddha his teachings were compiled and called TripitakasThe Jatakas contain tales dealing with the previous births of Buddha

Impact on Education

The Buddhist monasteries became great centres of learning These centres of learning developed into famous universities- Nalanda Taxila Vikramshila etc

Impact on Art and Architecture

The gateways and railing of the Sanchi Stupa were covered with sculptured figuresCave- temples were also constructed which were decorated with beautiful frescoes

1) How many parts of Tripitakas are thereAns -There are three parts of Tripitakas- Sutta Pitaka Vinaya PitakaAbhidhamma Pitaka

2)What are the subjects taught in these monasteries Ans ndash Buddhist scriptures logic Philosophy medicine astronomy etc

3) Which art was developed under Buddhist patronage Ans ndash Gandhara art

The Gandhara art was developed under Buddhist patronage

BIOLOGY The Leaf Photosynthesis The process by which green plants make their own food from carbon dioxide and water in the presence of sunlight and chlorophyll is called photosynthesis

All green plants need the following to make their food ndash

water carbon dioxide chlorophyll and energy in the form of sunlight

Carbon dioxide + water ------------- Glucose + oxygen

The end product of photosynthesis is glucose

Fill in the blanks

1 Plants make their food by the process of photosynthesis

2 The inner wall of the guard cell is thicker than the outer wall

3 The extra glucose is converted into starch and sucrose

4 The leaf is boiled in alcohol to remove chlorophyll

5 The rate of transpiration is more on the hot day then a cold day

6 Photosynthesis helps to observe water and minerals from the soil

English 1 Pronouns Kinds of pronouns 1 Personal pronouns2 Possessive pronouns3 Reflexive pronouns4 Interrogative pronouns5 Relative pronouns6 Demonstrative pronouns7 Indefinite pronouns

Personal pronouns they refer to first second and third person in sentences First person- the speakerSecond person-the listenerThird person-the objectperson being spoken aboutPersonal pronouns should have the same gender and number as the nouns they refer to

Possessive pronouns these are used to indicate the relationship between the objects and people These pronouns include mine ours yours his hersand theirs

ExerciseBFill in the blanks with suitable pronounsThere was much excitement among the childrenTheywere eagerly looking forward to the annual picniclsquoAre they going to Lodhi Gardens toorsquo wondered AneeshlsquoNo they are going to Buddh Jayanti Park with Mrs Jain said Mrs ChopraThe children looked disappointedlsquoWonrsquot you be taking us Marsquoam rsquo they askedlsquoSorry children I have to go to Mumbai for a week to look after my sick mother But you will have fun with Mrs Jain she is full of laughter and you will love being with her the whole daylsquoIt will not be the same they grumbled

English 2 The great train journey- Ruskin Bond

The great journey by Ruskin Bond is a story about Suraj who loved trains and wanted to go to places One day while wandering along the railway tracks he enters into a carriage compartment The train suddenly starts moving with him in the compartment and after a journey returns back to the same place from where it had begun The story is about his experience during that journey

State true or false1 When the train had passed leaving behind the

hot empty track Suraj was lonely2 It was winter holidays

and Suraj did not know what to do with himself

3 He plunged his hands into the straw and pulled out an apple

4 A dirty bearded face was looking out at him from behind a pile of crates

5 Suraj wanted to go to Japan

Hindi 2nd lang

गललबाजलडका खालीसथानोकोभरो-6 गो-ामसनिनकलकरहमगराजमआगए7 माबोधराजकोराकषससमझतीथी8 चीलरोशन-ानमसअ-रआकरतहसीलपरबठगई9 तीनचारतीनकऔररईकगोलउडलनिकनघोसलानहीनिगरा10 वहसवयतोघोसलातोडनककतिलएगललउठालायाथा

11 -ीवारकसाथलगतगोहपजोकसहार-ीवारपकडलतीह12 बोधराजअभीभीटकटकीबाधचीलकीओर-खरहाथा13 बोधराजअपनीजबमबहतसाचगगाभरकरलायाथा14 मरनिपताजीकीतरककीहईऔरहमलोगएकबडघरमजाकररहनलग15 बागमजातातोफलपरबठीनिततलीको-खनिततलीकोपकडकरउगकतिलयोकबीचमसल-ता

BENGALI(2ND LANGUAGE)

সহিনধসবরপওসবরসহিনধ

সবরসহিনধরহিনয়ম- ৯ই-কারহিকংাঈ-কাচেররপচেরইাঈহিভননঅনযসবররণCথাকচেইাঈসথাদেনয-ফায়এংওইয- ফাপCচেরণCযকতয়

১০উ-কারহিকংাঊ-কাচেররপচেরইাঈহিভননঅনযসবররণCথাকচেউাঊসথাদেন-ফায়এংওই- ফাপCচেরণCযকতয়

১১ঋ- কাচেররপচেরঋহিভননঅনযসবররণCথাকচেঋসথাচেনর -ফায়এংওইর পCচেরণCযকতয়

১২সবররণCপচেরথাকচেপCতMএ-সথাচেনঅয় ঐ- সথাচেনআয় ও- সথাচেনঅএংঔ- সথাচেনঅায়

৯ই+ অ= য- ফাআহি+ অনত= আযনত অহিধ+ অয়ন= অধযয়নই+ আ=য- ফা+ াইহিত+ আহি= ইতযাহি পরহিত+ আতC ন= পরতযাতC নই+ উ=য- ফা+ উঅহিত+ উহিকত= অতযহিকত হি+ উৎপহিতত= যৎপহিততই+ ঊ= য- ফা+ ঊ ই+ এ= য- ফা+ এঈ+ অ= য- ফা পরহিত+ ঊ4= পরতয4 পরহিত+ এক= পরচেতযকঈ+ অ আ= য- ফা+ অ আনী+ অমব= নযমব মসী+ আধার= মসযাধার

১০উ+ অ= অন+ অয়= অনবয় পশ+ অধম= পশবধমউ+ আ= াস+ আগত= সবাগত পশ+ আহি= পশবাহিউ+ ঈ= হিঅন+ ইত= অহিনবতউ+ এ= দেঅন+ এ4রণ= অচেনব4রণউ+ ঈ= ীসাধ+ ঈ= সবাধবী তন+ ঈ= তনবী

১১ঋ+ অ= র মাত+ অনমহিত= মাতরনমহিতঋ+ আ= রা হিপত+ আয়= হিপতরায়ঋ+ ই= হির মাত+ ইচছা= মাতচছাঋ+ ঈ= রী ধাত+ ঈ= ধাতরীঋ+ উ= র ভরাত+ উপচে(= ভরাতরপচে(

১২এ+ অ= অয় দেন+ অন= নয়নঐ+ অ= আয় গৈগ+ অক= গায়কও+ অ= অ দেপা+ অন= পনও+ ই= অ দেপা+ ইতর= পহিতরও+ এ= অ দেগা+ এ4রণা= গচে4রণাঔ+ অ= অা দেপৌ+ অক= পাকঔ+ ই= অা দেনৌ+ ইক= নাহিকঔ+ উ= অা দেভৌ+ উক= ভাক

MATHS Topic NumbersChapter Natural numbers and whole numbers

Study item properties of whole numbers for multiplication

1 Closure property If x and y are two whole numbers then xtimesy is also a whole numberExample If x = 9 and y =3 then xtimesy = 9times3 = 27 which is a whole number

2 Commutative property If x and y are two whole numbers then xtimesy = ytimesxExample If x = 5 and y = 2 then xtimesy = 5times2 = 10y times x = 2times5 = 10Therefore 5times2 = 2times5

3 Associative property If x y and z are three whole numbers then x times(ytimesz) = (xtimesy) times zExample If x =3 y = 5 and z = 7 then 3 times (5times7) = 3 times (35) = 105And (3times5) times7 = (15) times 7 = 105Therefore x times (ytimesz) = (xtimesy) timesz

4 Distributive property If x y and z are three whole numbers then xtimes (y + z) = x times y + x times z

Therefore the multiplication of whole numbers is distributive over their additionExample If x = 5 y = 3 and z= 2Therefore x times (y + z) = 5 times (3 + 2) = 5times5 =25And x times y + xtimes z = 5times3 +5times2 =15 +10 = 25Again x times (y ndash z ) = x times y ndash x timesz Therefore 5 times ( 3 - 2) = 5 times1 = 5 and 5times3 ndash 5 times2 = 15 ndash 10 = 5Therefore the multiplication of whole numbers is also distributive over their subtraction if y is greater than z

5 Existence of identity If x is a whole number then

X times1 = x 1 times x = xTherefore we can write x times1 = 1 times xTherefore the multiplication of any whole number with 1 is the number itselfTherefore we can say that 1 is multiplicative identity or identity element for multiplicationExample 5 times1 = 5 1 times 5 = 5 Therefore 5 times 1 = 5

6 Multiplicative inverse If x is any whole number ( x is not equal to zero ) then its multiplicative inverse will be 1xSo x times 1x = 1 but 1x is a whole number if x = 1For other values of whole number 1x is not a whole number therefore we can write its multiplicative inverse does not exists

7 Cancellation law of multiplication If x y and z are three non- zero whole numbers then x times y = x times z

Or y = zExample 9 times y = 9 timeszTherefore y = z

Class VIISubject Topic Summary Execution

English 2 Sentences based on meanings

Kinds of sentences

Assertive or declarative to convey information or simply make a statement

Interrogative to ask different types of questions

Imperative to command or instruct someone or make a request

Exclamatory to express strong feelings and emotions

Exercise c1 What a nice compliment that is

That is a nice compliment2 How well- behaved the children

areThe children are very well-behaved

3 What great chefs we areWe are great chefs

4 What a shame it isIt is a shame

5 What a fantastic idea you haveYou have a fantastic idea

Homework 6 -10English

LiteratureThe Listeners III) Answer the following questions-

d) Identify two words used in the poem to give the poem an eerie atmosphereAns- Two words used to give the poem an eerie atmosphere are ldquogreyrdquo and ldquophantomrdquo

e) Who do you think are the inmates of the houseAns- I think the inmates of the house are phantom who once used to dwell in it

f) Why was the poet ldquoperplexed and stillrdquoAns- He was lsquo perplexed and stillrsquo because he was expecting an answer from the inmates of the house But despite of repeated calls there was no response

CHEMISTRY Chapter 2 ndashElement and Compound

Activity Series of MetalsThe activity series is a chart of metals listed in order of declining relative reactivity The top metals are more reactive than the metals on the bottomMetal SymbolReactivity

Lithium Li displaces H2 gas from water steam and acids and forms hydroxides

Potassium K

Strontium Sr

Calcium Ca

Sodium Na

Magnesium Mg displaces H2 gas from steam and acids and forms hydroxides

Aluminum Al

Zinc Zn

Chromium Cr

Iron Fe displaces H2 gas from acids only and forms hydroxides

Cadmium Cd

Cobalt Co

Nickel Ni

Tin Sn

Lead Pb

Hydrogen gas

H2 included for comparison

Antimony Sb combines with O2 to form oxides and cannot displace H2

Arsenic As

Bismuth Bi

Copper Cu

Mercury Hg found free in nature oxides decompose with heating

Silver Ag

Palladium Pd

Platinum Pt

Gold Au

Answer the following Q)Difference Between Metals And Nonmetals

Metals Nonmetals

These are solids at room temperature except mercury

These exist in all three states

These are very hard except sodium

These are soft except diamond

These are malleable and ductile

These are brittle and can be breakdown into pieces

These are shiny These are non-lustrous except iodine

Electropositive in nature Electronegative in nature

Have high densities Have low densities

Math Number System

Chapter Fraction

Study item Some solved sums from exercise 3(B)1) For each pair given below state whether it from like fractions or unlike

fractions (i) 58 and 78

= Like Fraction because denominators same(ii) 815 and 821

= Unlike Fraction because denominators are not same

(iii) 49 and 94 = Unlike Fraction

2) Convert given fractions into fractions with equal denominators(iii) 45 1720 2340 and 1116Solution Given fraction 45 1720 2340 and 1116Therefore the LCM of 5 20 40 and 16 is 80Therefore 45 = 4times165times16 = 64801720 = 17times420times4 = 68802340 = 23times240times2 = 4680 1116 = 11times516times5 = 5580

3) Convert given fractions into fractions with equal numerators(iii) 1519 2528 911 and 4547Solution Given fractions 1519 2528 911 and 4547Therefore the LCM of 15 25 9 and 45 is 2251519 = 15times1519times15 = 225285 2528 = 25times928times9 = 225252911 = 9times2511times25 = 2252754547 = 45times547times5 = 225235

4) Put the given fractions in ascending order by making denominators equal

(iii) 57 38 914 and 2021Solution Given fraction 57 38 914 and 2021Therefore the LCM of the denominators is 16857 = 5times247times24 = 12016838 = 3times218times21 = 63168914 = 9times1214times12 = 1081682021 = 20times821times8 = 160168Therefore ascending order 63168lt108168lt120168lt160168Therefore ascending order of given fractions38lt914lt57lt2021

COMPUTER CHAPTER-1COMPUTER FUNDAMENTALS

DONE IN THE PREVIOUS CLASSES PAGE 10CWRITE TRUE AND FALSE

1 True2 False3 False4 False5 True

GEOGRAPHY CHAPTER 7EUROPE

CHAPTER COMPLETE 1)Europe is home to a famous mountain range called the Alps

2)River Rhine originates in Switzerland

3)The Eiffel Tower one of the tallest structures in Europe

4) Vatican City is one of the most densely populated European countries

5)Sognefjordin Norway is the largest fjord in Norway

Class VIIISubject Topic Summary Execution

MATHEMATICS Ch 6Sets

Exercise 6 (D)1 Given A = x x isin N and 3iquest x le 6 and B = x x isin W and xlt4 find (i) Sets A and B in roster form (ii) A cup B (iii)

A cap B(iv) A ndash B (v) B ndashA

Solution (i) A = 456 and B = 0123

(ii) A cup B = 0123456 (iii) A cap B = ϕ (iv) A ndash B = 456 (v)B ndash A = 0123

3 If A = 56789 B = x 3 lt x lt 8 and x isin W and C = x xle5 and x isin N Find (i) A cup B and (A cup B) cup C (ii) B

cup C and A cup ( B cup C)

(iii) A cap B and (A cap B) cap C (iv) B cap C and A cap (B cap C)

Is (A cup B) cup C = A cup (B cup C)

Is (A cap B) cap C = A cap (B cap C)

SolutionA = 56789 B = 4567 C = 12345

there4 (i) A cap B = 456789 and (A cup B) cup C = 123456789

(ii) B cup C = 1234567 and A cup ( B cup C) = 123456789

(iii) A cap B = 567 and (A cap B) cap C = 5

(iv) B cap C = 45 and A cap (B cap C) = 5

Now (A cup B) cup C = 123456789

And A cup ( B cup C) = 123456789 there4 (A cup B) cup C = A cup (B cup C)

Again (A cap B) cap C = 5 and A cap (B cap C) = 5

there4 (A cap B) cap C = A cap (B cap C)

4 Given A = 012345 B = 02468 and C = 0369 Show that (i) A cup (B cup C) = (A cup B) cup C ie the union

of sets is associative (ii) A cap (B cap C) = (A cap B) cap C ie the intersection of sets is associative

SolutionNow B cup C = 0234689 and A cup B = 01234568

there4 A cup (B cup C) = 012345689 and

(A cup B) cup C = 012345689

So (i) A cup (B cup C) = (A cup B) cup C ie the union of sets is associative

Again B cap C = 06 and A cap B = 024

there4 A cap (B cap C) = 0 and (A cap B) cap C = 0

So (ii) A cap (B cap C) = (A cap B) cap C ie the intersection of sets is associative

Physics Chapter 2 Physical Quatites and Measurements

Here We Will Do Some QuestionsRelated To Chapter 2

A density bottle has a marking 25 mL on it It means that

1 the mass of density bottle is 25g

2 the density bottle will store 25 ml of any liquid in it

3 the density bottle will store 25 ml of water but more volume of liquid denser than water

4 the density bottle will store 25 ml of water but more volume of a liquid lighter than water

Solution 2 the density bottle will store 25 ml of any liquid in it

COMPUTER CHAPTER-2Spreadsheet Functions and Charts

SELECTING RANGE IN ROWSCOLUMNSWHEN TWO OR MORE CELLS ARE SELECTED IT IS CALLED A RANGEA RANGE OF CELLS CAN BE FORMED IN TWO WAYS--a) SELECTING RANGE BY USING THE MOUSEb) SELECTING RANGE BY USING THE KEYBOARD

Q1)WRITE THE STEPS TO SELECT PARTIAL RANGE IN A ROW

Ans)THE STEPS ARE-6 SELECT THE ROW7 BRING THE CELL POINTER TO THE DESIRED

LOCATION FROM WHERE YOU WANT TO START YOUR SELECTION

8 CLICK THE LEFT MOUSE BUTTON AND KEEP DRAGGING TO YOUR RIGHT TILL YOU REACH THE LAST CELL TO NE SELECTED

RELEASE THE MOUSE BUTTON

GEOGRAPHY Asia

CLIMATE

Asia experiences great extremes of climate Jacobabad in the Sind province of Pakistan is one of the hottest places in the WorldVerkhoyansk in Siberia is one of the coldest places in the WorldCherrapunji and Mawsynram in India are two wettest places in WorldArabia Tibet Gobi and Mongolia are extremely dry regionsFactors Affecting Climate of Asia-The factors influencing the climate of Asia are-

Factors Affecting Climate of Asia-Thoroughly read the table in page number 60

Latitudinal extent

Continentality

Relief features

Presence of low pressure trough

Jet streams

English Language The Sentence A complex sentence contains one independent clause and at least one dependent clause The dependent clause in a complex sentence is introduced with subordinating conjunctions or relative pronouns

Commonly Used Subordinating Conjunctions-Time after before while when since untilCause And Effect because now since as in order that soOpposition although though even though whereas while in spite ofCondition if unless only if whether or not even if in case(that)

Commonly Used Relative Pronouns-Who whose whom which whoever whomever whichever that

Class IXSubject Topic Summary Execution

1-BENGALI(2ND LANGUAGE)

ldquo বঙগভমিরপরমিrdquo াইকেলধসদনদতত

আচেগর পর উততর পচো-১ ২ ৩ এং নীচের পর টি াহির কা- ৪মহিbকাও গচেনা দেগা পহিচে অমত হরচে- ক) কার দো দেকান কহিতার অং( ) কতা দেক পরসঙগ কী উহিকতটির তাৎপযC আচোনা কচেরা৫দেসই ধনয নরকচে দোচেক যাচে নাহি ভচে মচেনর মহিeচের সাচেসচে সCন ক) কহির কায C ার উচেf(য হিক হিছ কহি কন কহিতাটি দেচেন) কহি কার কাচেছ হিমনহিত কচেরচেছনগ) কহি এই পহিথীচেত কাচের ধনয মচেন কচেরনঘ) কহি হিক রকম অমর তাাভ করচেত ান

Hindi 2nd lang

काकी(कतिसयारामशरण गपत)

इस कहानी म लखक न यह बतान का परयास निकया ह निक बचच अपनी मा स निकतना परम करत ह शयाम अबोध बालक ह वह अपनी मा क मरन क बा- उसन अपनी मा क कतिलए बहत रोया बा- म उस पता चला निक उसकी मा राम क घर चली गई ह आकाश म उडती हई पतग -खकर उस हरष हआ निक पतग क दवारा वह अपनी मा को नीच उतारगा इसक कतिलए वह अपनी निपता की जब स -ो बार सवा रपया निनकालकर पतग और -ो मोटी सी मन वाली अपन भाई स काकी एक कागज पर कतिलखवा कर पतग म कतिशव का दि-यानिनकालकर पतग और -ो मोटी सी मन वाली अपन भाई स काकी एक कागज पर कतिलखवा कर पतग म कतिचपका दि-याभोला और शयाम कोठरी म रससी बाधनी रह थ तभी उसक निपता करोध म आकर उन स पछ निक कया उनकी जब स रपया निनकाला हभोला डर क मार बताया निक शयाम इस पतग क दवारा अपनी काकी को राम क यहा स उतारना चाहता हनिवशशवर(शयाम क निपता)न फटी पतग उठाकर -खी तो उस पर काकी कतिलखा थावह हत बजिa होकर वही खड रह गएउनहोन सोचा निक मन अपन पतर को मारा जोनिक अनजान और निन-dरष थावह अपनी मा कोनिकतना पयार करता ह

उस दि-न बड सवर शयाम की नी- खली तो -खा निक घर भर म कोहराम मचा हआ ह

क) घर म कोहराम कयो मचा हआ था शयाम को कया लगा

ख) काकी को ल जात समय शयाम न कया उपदरव मचाया

ग) काकी क बार म उस कया बताया गया कया सतय उस कतिछपा रहा

घ) वह बठा-बठा शनय मन स आकाश की ओर कयोकरता

उततरक) शयाम की मा का -हात हो गया था इसकतिलए

घर म कोहराम मचा हआ था शयाम की लगा निक उसकी मा सफ- कपडा ओढ हए भमिम पर सो रही ह

ख) लोग जब उमा यानी शयाम की मा को उठाकर ल जान लग तब शयाम न बडा उपदरव मचाया लोगो क हाथ स झठ करवा उमा क ऊपर जा निगरा और बोला काकी सो रही ह उस कहा ल जा रह हो

ग) काकी क बार म बजिaमान लोगो न उस निवशवास दि-लाया निक उसकी का निक उसक मामा क यहा गई ह लनिकन सतय अमिधक दि-नो तक कतिछपाना रह सका आसपास क अबोध बालको क मह स यह बात परकट हो गई निक उसकी मा का -हात हो गया ह

घ) कई दि-नकई दि-न लगातार रोत-रोत उसका रोना तो शान हो गया पर उसक ह-य म शोक भर गया था वह चपचाप बठा आकाश की और टाका करता निक शाय- उसकी काकी कही दि-ख जाए

ldquoदि-न उसन ऊपर आसमान म पतग उडती -खी न जान कया सोच कर उसका निहर-य एक-म खिखल उठाrdquo

क) निकसन पतग ऊपर उडत -खी और वह कयो खश हआ

ख) उसन अपन निपता स कया कहा उनका कया उतर थाश

ग) उसन निफर कया निकया और निकसन उसकी सहायता की

घ) उसकी योजना कया थी उततर -क) शयाम न एक दि-न आसमान म पतग उडती

-खी तो उसन सोचा निक पता आसमान म राम क यहा जाकर रकगी वही पर मरी काकी ह यह सोचकर वह बहत खश हआ

ख) उसन अपन निपता स कहा काका मझ एक पतग मगा -ो उसक निपता न भटक हए मन क भाव स कहा निक मगा -ग यह कह कर उ-ास भाव स वह कही और चल गए पतग नही आई

ग) उसन चपचाप निवशशवर क टगहए कोट स एक चवननी निनकाल ली और सखिखया -ासी क लडक भोला की सहायता स एक पतग मगवानी भोला उसकी बराबर उमर का ही था

घ) उसकी योजना यह थी निक वह अपनी पतग को आकाश म राम क यहा भजगा और उस पतग क सहार उसकी काफी नीच उतर जाएगी इस योजना पर उस परा निवशवास था इसकतिलए वह और भोला -ोनो यह काम करन म लग गए

Continue to nexthelliphellipEVS CHAPTER - 1

(UNDERSTANDING OUR ENVIRONMENT)

Sustainable development

The development that meets the needs of the present without compromising the ability of future generations to meet their own needs is called Sustainable development

Sustainable societies ndash

An environmentally sustainable community is one that meets the current and future basic resource needs of its people in a just and equitable manner without compromising the ability of future generations to meet their basic needs

Q ) What are Eco Villages

Ans - Eco village are the urban or rural communities of people who strive to integrate a supportive social environment with a low impact way of life

Q ) To ensure sustainable development the depletion of renewable resources should not take place at a rate faster than their regeneration Justify your answer

Ans ndash Renewable resources do not have a fixed quantity - more can always be

generated However if the rate of use exceeds the rate of renewal - that is the

source is used more than its being recreated - its continued use will become

used up faster than it can regenerate

To promote sustainable society the following things need to be done ndash

1 Using renewable energy sources 2 By improving the quality of human

health 3 By promoting sustainable agriculture 4 By forming ecovillage

it will eventually be entirely depleted So Toensure sustainable development the depletion of

renewable resources should nottake place at a rate faster than their regeneration

Q ) What do you mean by Sustainable societies

Ans - Sustainable societies are defined as towns and cities that have taken steps to remain healthy over the long term These communities value healthy ecosystems use resources efficiently and actively seek to retain and enhance a locally based economy Sustainable development concerns everybody in a society

Q ) What are the effects of pollution on human health

Ans ndash Some health problem occurs due to air pollution are ndash

Respiratory diseases Cardiovascular damage Fatigue headaches and anxiety Irritation of the eyes nose and throat Damage to reproductive organs Harm to the liver spleen and blood Nervous system damage

Some health problem occurs due to water pollution are ndash

Typhoid Cholera Dysentry Jaundice

Some health problem occurs due to noise pollution are ndash

Fatigue headaches and anxiety High blood pressure Hearing damage

Physics Motion in 1D First go through previous notes Now here we will solve some numerical related to that

Question 3What information about the motion of a body is obtained from the displacement-time graphSolution 3From displacement-time graph the nature of motion (or state of rest) can be understood The slope of this graph gives the value of velocity of the body at any instant of time using which the velocity-time graph can also be drawn

Question 4(a)What does the slope of a displacement-time graph represent(b)Can displacement-time sketch be parallel to the displacement axis Give a reason to your answerSolution 4(a) Slope of a displacement-time graph represents velocity(b) The displacement-time graph can never be parallel to the displacement axis because such a line would mean that the distance covered by the body in a certain direction increases without any

increase in time which is not possible

Chemistry Language of Chemistry

How to balance a chemical equationThere are two methods of balancing an equation(i)Hit and trial method(ii)Partial equation methodBalancing by hit and trial methodThis method consists of counting the number of atoms of each elements on both sides and trying to equalize themTake the following steps(i)Count the number of times (frequency) an element occurs on either side(ii)The element with the least frequency of occurrence is balanced first(iii)When two or more elements have the same frequencythe metallic element is balanced firstExample-1 On heatinglead nitrate decomposes to give lead dioxidenitrogen dioxide and oxygenPb(NO3)2rarrPbO+NO2+O2

In this equationLead occurs twiceNitrogen occurs twiceOxygen occurs four timesSince lead is a metalbalance it firstThe number of atom of lead is equal on the two sidestherefore it needs no balancingNow balance nitrogenOn the reactant sidethere are two atoms of nitrogenwhile on the product side oneSomultiply the product containing nitrogenon the product sideby two Pb (NO3)2rarrPbO+2NO2+O2Nowthe number of oxygen atoms on the reactant side 6while on the product sideit is 7Somultiply the entire equation by 2except oxygen to get balanced equation2Pb(NO3)2rarr2PbO+4NO2+O2Multiplication by 2 is done only when atoms of all the elements except one element are balanced and the unbalanced atom occurs separately at least once and also there is a difference of only one such atom

Math Topic AlgebraChapter

Factorisation

Study item Difference of two squares a2 ndash b2 = (a+b) (a-b)1) (i) 4x2ndash 25y2

= (2x) 2 ndash (5y) 2= (2x + 5y) (2x - 5y)

(ii) 9x2 ndash 1= (3x)2ndash(1)2= (3x + 1)(3x ndash 1)

2) (i) 150 ndash 6a2= 6(25 ndash a2)= 6(5)2 ndash(a)2= 6 (5 + a) (5 ndash a)

(ii) 32x2 ndash 18y2=2(16x2 ndash 9y2)=2(4x)2 ndash (3y)2= 2(4x + 3y)(4x - 3y)3)(i) (x ndashy )2 ndash 9 = (x ndash y )2 ndash (3)2= (x ndash y + 3) (x ndash y ndash 3)(ii) 9(x + y) 2ndash x 2= (3)2(x + y)2 ndash (x)2=3(x + y)2 ndash (x)2= (3x +3y ) 2ndash(x)2= (3x + 3y + x)(3x +3y ndash x)= (4x + 3y) ( 2x + 3y )

Commercial studies

Basic accounting terms

Today I will give you some questions from the previous study material

Questions1) Define accounting2) What do you mean by debit and

credit

3) Explain the types of account4) Define the following terms

a) Assetsb) Capitalc) Purchased) Debtorse) Transactions

5) Name the types of accounts given below

a) Krishnas accountb) Machinery accountc) Royalty accountd) Salary accounte) Furniture accountf) Audit fee account

Economics Revision Today I will give you some revision questions

Questions1) What do you mean by the terms

rdquowantsrdquo2) Write the difference between

consumer goods and producer goods

3) Define the term utility 4) Explain the different types of utility5) Define

a) Total utilityb) Marginal utility

Subject Eng Literature (The Merchant of Venice ndash William Shakespeare)Topic Act I Scene 3 Lines 1 to 48 (Shylock hellip Cursed be my tribe if I forgive him) Date 16th April 2020 (5th Period)

[Students should read the original play and also the paraphrase given in the school prescribed textbook]Summary Questions amp Answers

This scene takes place in Venice and we are introduced to the rich Jew Shylock Bassanio and Shylock are talking and Bassanio tells Shylock that he wants a loan of three thousand ducats for three months on the personal security of Antonio

o Shylock feels glad because he will be able to bind down Antonio by means of a bond on account of the loan but he tells Bassanio that all the fortunes of Antonio being invested in the merchant ships on the sea it is difficult to depend upon his credit Even under such circumstances Shylock is willing to advance the money on the personal security of Antonio

o Bassanio then invites Shylock to dine with him Shylock says that he is prepared to do anything with the Christians but not eat or drink or pray with them

o While Bassanio and Shylock are talking Antonio appears on the scene Shylock does not seem to take any notice of Antonio but goes on brooding within

(1) SHYLOCK Ho no no no no- my meaning in (Line 15-26)saying he is a good man is to have you understand me that he is sufficient Yet his means are in suppositionhe hath an argosy bound to Tripolis another to the Indies I understand moreover upon the Rialto he hath a third at Mexico a fourth for England and other ventures he hath squanderd abroad Butships are but boards sailors but men there be land-rats and water-rats land-thieves and water-thieves I mean pirates and then there is the peril of waters winds and rocks The man is notwithstanding sufficientmdashthree thousand ducats mdashI think I may take his bond

(a) Who is talking in the beginning of this scene What does Bassanio want from Shylock How does Shylock feel

In the beginning of the scene Bassanio and Shylock are talking to each other Bassanio wants to borrow three thousand ducats from Shylock for three months on the security of Antonio Shylock feels glad at heart that he will get the opportunity of binding Antonio with a bond(b) What risks does Shylock weigh in advancing the money

Shylock says that Antonio has invested all his capital in trading by sea-going ships But the ships are made of wood and the sailors of those ships are ordinary human beings The wood can

himself how he hates Antonio because of his being a Christian because he abuses Shylock in public places Shylock decides that if ever he can get Antonio to his advantage he will teach him a lesson

come to harm and men can commit mistakes and thus the capital invested in ships may be lost Then there are other dangers The goods loaded on the ships can be damaged by rats and thieves which are found both on land and water The ships can also be harmed through sea-storms submerged rocks etc(c) What two important functions does this scene have

The scene has two important functions First it completes the exposition of the two major plot lines of the play Antonio agrees to Shylockrsquos bond ndash three thousand ducats for a pound of flesh and second and more important dramatically this scene introduces Shylock himself In this scene Shakespeare makes it clear at once why Shylock is the most powerful dramatic figure in the play and why so many great actors have regarded this part as one of the most rewarding roles in all Shakespearean dramas(d) Where does this scene take place What kind of treatment has Antonio been giving to Shylock What does Shylock say when Bassanio invites him to dine with him

The action of this scene takes place in Venice Antonio has been in the habit of behaving harshly with Shylock ndash spitting on his beard and footing him like a stranger cur When Bassanio invites Shylock to dine Shylock says that he is prepared to do anything with the Christians but not eat and drink or pray with them

(2) SHYLOCK How like a fawning publican he looks (Line 38-48)I hate him for he is a Christian

But more for that in low simplicity

He lends out money gratis and brings downThe rate of usance here with us in VeniceIf I can catch him once upon the hipI will feed fat the ancient grudge I bear him

He hates our sacred nation and he railsEven there where merchants most do congregateOn me my bargains and my well-won thriftWhich he calls interest Cursed be my tribeIf I forgive him

(a) What is the context in which these words are spoken and what is the idea expressed in it

These remarks are made by Shylock when he sees Antonio coming along after Bassanio told him that the merchant will be his surety for the bond The above mentioned passage reveals Shylockrsquos hatred for Antonio Shylock says that he hates Antonio because he is a Christian and also because he gives loan without taking interest on them thereby bringing down the rate of interest in Venice(b) Explain the meaning of the phrase lsquoa fawning publicanrsquo

The phrase lsquoa fawning publicanrsquo refers to Roman tax collector It is a term of contempt and hatred on the lips of a Jew lsquoFawning Publicansrsquo were Roman tax-gatherers whose ordinary bearings towards the Jews was bullying but whose false pose of lsquohumility and contritionrsquo is touched upon in the parable in New Testament(c ) What light does the above passage throw on the character

of Shylock

The above mentioned speech of Shylock reveals him to be a wicked character having an extreme greed for wealth His intense hatred for Antonio is unjustified He hates Antonio only because he is a Christian and because he lends money without taking any interest on it thereby adversely affecting Shylockrsquos business of lending money on high interest(d) What information do you gather about Antonio from the above given lines

Shylockrsquos statement throws a valuable light on the character of Antonio Antonio appears to be a good Christian and a good human being He helps the people in need by lending them money without charging any interest on it He is a man of simple and good nature This very goodness makes him Shylockrsquos enemy(e) What does Shylock debate within himself and when To whom are the lines mentioned above addressed to

When Bassanio asks the Jew to lend him three thousand ducats on Antoniorsquos surety Shylock begins to debate within himself as to how he should exploit the opportunity of a business deal with his old enemy Antonio

The lines mentioned above are not addressed to anyone The lines are a soliloquy ie a speech made by a character to himself and not meant to be heard by the other characters present

Class XSubject Topic Summary ExecutionEnglish

LiteratureThe Blue Bead 2nd part

Things took a turn for the worst and all of a sudden a crocodile attacked the woman biting on the womanrsquos leg At that moment Sibia got up sprinted grabbed the hay fork and stabbed the crocodile in the eye with all her power Immediately the crocodile let go and went away Sibia saw a small blue bead lying by the river she grabbed it Since she was poor she didnrsquot have necklace Shersquod always wanted one like the other women now she could make one with the blue bead After that she went home and told her mother all about it

Hindi 2nd

Langबड घर की बटी( मशी परमच-)

lsquoबड घर की बटी कहानी का उददशय मधयम वग की घरल समसया को सलझा कर सगदिठत परिरवार म मिमल जलकर परम स रहन का स-श -ना हघर म शानित सथानिपत करन की जिजमम-ारी नारी की होती ह यदि- नारी समझ-ार ह उसम धय और परिरवार क परनित परम ह तो कोई भी घटना परिरवार को निवघदिटत नही कर सकती या कहानी परिरवार को सगदिठत करत हए परम सौहा- स एक दसर की भावनाओ को समझ करउनका सहयोग करत हए जीवन यापन करन की पररणा -ती हमशी परमचदर जी न इस कहानी म सय परिरवार का परनितनिनमिधतव निकया ह यह कहानी बनी माधव सिसह जो गौरी पर क जमी-ार क उनक -ो पतरो की हशरी कठ लाल निबहारीशरीकात का निववाह एकजमी-ार घरान की पतरी आन-ी स हआ थाआन-ी न ख- को ससराल क वातावरण म ढाकतिलया थाएक दि-न आन-ी का अपन -वर लाल निबहारी स झगडा हो जाता ह -ोनो भाई एक दसर स अलग होन की कोकतिशश करत हसभी बह आन-ी न अपन मधर वयवहार स लाल निबहारी को

ldquoइन नतर निपरय गणो को बीए-इनही -ो अकषर पर नयोछावर कर दि-या था इन -ो अकषर न उनक शरीर को निनबल और चहर को कानित ही बना दि-या थाldquo

क) परसतत पकतियो म निकस वयकति क बार म कहा गया ह

ख) इन पकतियो म कौन स नतर निपरय गणो क बार म कहा गया ह

ग) बीए की निडगरी परापत कर लन पर भी उपय वयकति क सवभाव की कया निवशरषता थी

घ) यह नतर निपरय गण निकस वयकति म निवदयमान थ उसक वयकतितव की कया निवशरषता थी

उततर ndashक) परसतत पकति म गौरी पर गाव क जमी-ार

क बड बट शरीकात क बार म कहा गया ह उसन बहत परिरशरम और उ-म क बा- ba की निडगरी परापत की थी अब वह एक -फतर

घर छोडकर जान स रोक कतिलयाइस पर बनी माधव सिसह न कहा निक बड घर की बटी ऐसी ही होती ह जो निबगडा काम बना लती ह अतः शीरषक साथक ह बड घर की बटी आन-ी ह

म कमचारी थाख) भरा हआ चहरा चौडी छाती और डटकर

खाना आदि- एक सबजी ल जवान क गण मान जात ह परत शरीकात न इनही नतर निपरय गणो को अपनी पढाई पर नयोछावर कर दि-या था

ग) बीए की निडगरी परापत कर लन पर भी उपय वयकति(शरी कठ की शारिररिरक तौर पर निनबल और चहर स कानित ही लगत थ इतना ही नही वह मानकतिसक तौर पर भी निपछड हए थ पाशचातय सामाजिजक कथा उस घणा एव पराचीन सभयता का गणगान उनकी निवचारधारा क परमख अग थ

घ) यह नतर निपरय गण गौरीपर गाव क जमी-ार क छोट बट लाल निबहारी सिसह म निवदयमान थ वह सजीलाजवान था और भस का दध शर दध वह सवर उठकर पी जाता था

ldquoयही कारण था निक गाव की लललन आए उनकी निन-क थी कोई कोई तो उह अपना शतर समझन म भी सकोच ना करती थी सवय उनकी पतनी को इस निवरषय म उनस निवरोध थाldquo

क) उपय पकति म इस वयकति क बार म कहा गया ह

ख) गाव की लललन आए उनकी निन-ा कयो निकया करती थी

ग) उनकी पतनी का कया नाम था उनह निकस निवरषय म अपन पनित क निवरa था और कयो

घ) इस कहानी का कया उददशय ह Continue to next helliphelliphellip

Bengali 2nd Language

ফ ফটক না ফটক( কহিতা )

পর) ldquo(ান াধাচেনা ফটপাচেথ পাথচের পাড হিচেয় এক কাঠচোটটা গাছ কহিকহি পাতায় পার ফাটিচেয় াসচেছldquoক) কার দো দেকান কহিতার অং( ) lsquo(ান াধাচেনা ফটপাচেথ পাথচের পাডহিচেয়lsquo চেত কী দোঝাচেনা চেয়চেছ গ) আচো য অংচে(lsquo এক কাঠচোটটা গাছ lsquoচেত কী দোঝাচেনা চেয়চেছ ঘ) ldquoকহিকহি পাতায় পার ফাটিচেয় াসচেছldquo ----- একথার পরকত অথC কী উততর ) ক) আচো য অং(টি পর যাত কহি সভা4 মচোপাধ যাচেয়র দো lsquoফ ফটক না ফটকrsquo কহিতার অং()কহি সভা4 মচোপাধ যায় হিছচেন দেপরচেমর কহি দেপরমচেক নানা ভহিঙগমায় হিতহিন ফটিচেয় তচেচেছন দেপরম মানচের স মচেতC র সঙগী কহিতার কহিতায় এক রb সb হচেয়র দেপরম াগরচেনর কথা চেচেছন (ান অথCাৎ দেযাচেন দেকান রস দেনই দেযাচেন দেকান মহিনতা দেনই অথ তার মধ দেযও দেপরম থাকচেত পাচের একথাই কহি তচে ধরচেত দেচেয়চেছন একটি মানচে4র মচেন দেযাচেন দেকামতার দেকান সথান দেনই পাথচেরর মচেতা হিনরসতার মচেনর মধ দেযও দেয দেপরম আসচেত পাচের দেস কথাই কহি চেচেছনগ)নারীচের যথC দেপরচেমর ছহি এই কহিতায় অকপচেট উচেঠ এচেসচেছ কহি এই কহিতায় কাটচোটটা গাছ কথাটি যার কচেরচেছন নারী দেয দেপরম দেথচেক হিতাহিত এং দেসই দেপরম সঠিক সমচেয় না পাওয়ার ন য দেপরম সমপচেকC হিচেr4 গৈতরী য় দেপরচেমর দেয গৈহি(ষট য মাধযC য সরসতা দেকামত এই সমসতর হিপরীত যথা রbতা শষকতা কচেঠার তা পরভহিত দোঝাচেত এক কাঠচোটটা গাছ কথাটি যার কচেরচেছনঘ) এাচেন এক নারীর যথC দেপরচেমর কথা হিনহিCপত ভাচে চেচেছন কহি অসমচেয় নারীর ীচেন দেপরম দেচেগচেছ এতহিন তার হয় রb কচেঠার হিছ দেপরচেমর অভাচে ঠাৎ দেসই শষক মরভহিমচেত সচের আভাস এচেসচেছ দেপরম দেযন 4Cার স(ীত তাই পরায় মত গাচেছ কহিকহি পাতা গহিচেয় উচেঠচেছ

Biology Chapter - 01Controlling Air Pollution

Today we will discuss how we control air pollution from domestic combustion

Q1Describe any five ways of reducing air pollution from domestic sources bull The number of pollutants in the air is verylarge and we always try to control them byfollowing ways

i) Solar cooker and solar heater It use no fuel reduce damage of environment by fuel use or reducing deforestation It maintains coolness of house It releases very less orno oil gas or grease

ii) Piped natural gas (PNG) It emits very less by products into the atmosphere As it isdistributed through pipe lines so there iscontinuous supply of fuel is possible

iii) Liquefied Petroleum Gas (LPG) It hasa higher heating value LPG doesntcontain sulphur so it burns a lot cleanerenergy sources It releases very less oralmost no fume in air

iv) Electricity based cooking Emission free cooking alternative for urban dwellers causeselimination of adverse health impactsofindoor air pollution It helps to avoid theinconveniences associated with procurement of LPG

v) Biogas It contains 75 methane whichmakes it an excellent fuel It burns without smoke and biogas plant leaves no residue like ash in wood charcoal etc Thus it isaclean fuel

Economics

Factors of Production

Today firstly we would recall the last class for 5 mins and then we would proceed with the further topics of the chapter

The concept meaning of land characteristics of land and importance of land to be repeated for the absentees as well as the students who were there in the class the previous day

Today we will start with the last portion of land before it the meaning of land to be repeated onceAs by now we all know that

Questions1What do you mean by productivity of landAnswer By productivity of land we mean the capacity of a piece of land to produce a crop

Thus it refers to the average output per unit of landSay per acre per hectare etc= (OutputArea of land)

2 What are the factors influencing the productivity of landAnswer

Natural factors Productivity of land is largely determined by the natural

Land is defined to include not only the surface of the earth but also all other free gifts of nature(for example mineral resources forest resources and indeed anything that helps us to carry out the production of goods and services but is provided by nature free of cost)

We will move on to the last portion of land by discussing Productivity of Land

By productivity of land we mean the capacity of a piece of land to produce a crop

Thus it refers to the average output per unit of land

Say per acre per hectare etc= (OutputArea of land)

With this we shall proceed further with the main factors that determine the productivity of land

Natural factors Human factors Improvements on land Location of land Organisation Ownership of land Availability of capital Proper use of land State help

Note economic development of a country depends upon the quality of its land If the land is fertile it will quicken the pace of development of the country

qualities of land such as fertility etc

Human factors Land cannot produce anything by itself Man has to apply labour on it to produce for himself So productivity of land depends on the knowledge and skills of workers

Improvements on land production of land is affected by land development measures like provision of well or tubewell irrigation proper drainage

State help The government of a country especially less developed country can play a vital role in improving the agricultural productivity by providing better irrigation facilities

Organisation Productivity of land also fdepends upon the way how the factors of production like labour and capital are organised

In order to increase productivity trained workers modern implements scientific methods good seeds are all essential

3 lsquoImproved technology affects the productivity of landrsquo Explain this statement with the help of suitable example Answer Use of improved technology raises the productivity of land Example By using HYV seeds chemical manures and modern machines per hectare output increases

Physics Force (Summary)

Question Write the expression for the moment of force about a given axisSolutionsThe expression for the moment of force is given byMoment of force about a given axis = Force times perpendicular distance of force from the axis of rotationQuestion What do you understand by the clockwise and anticlockwise moment of force When is it taken positiveSolutionsIf the effect on the body is to turn it anticlockwise moment of force is called the anticlockwise moment and it is taken as positive while if the effect on the

body is to turn it clockwise moment of force is called the clockwise moment and it is taken as negative

Math Topic Commercial Mathematics

Chapter Goods and services Tax

Study item Some solved sums from exercise ndash 1 A retailer buys a TV from a wholesaler for Rs 40000 He marks the price of the TV 15 above his cost price sells it to the consumer at 5 discount on the marked price If the sales are intra ndash state and the rate of GST is 12 find

(i) The marked price of the TV(ii) The amount which the consumer pays for the TV(iii) The amount of tax (under GST) paid by the retailer to the central

Government(iv) The amount of tax (under GST) received by the State Government

Solution As the sales are intra- state sale and the rate of GST 12 So GST comprises of 6 CGST and 6 SGSTTherefore a retailer buys a TV from a wholesaler for Rs 40000Therefore the amount of GST collected wholesaler from the retailer or paid by retailer to wholesalerCGST = 6 of Rs 40000 = Rs(6100 times40000) =Rs 2400SGST = 6 of Rs 40000 = Rs (6100 times 40000) =Rs 2400Therefore wholesaler will pay Rs 2400 as CGST and Rs 2400 as SGSTTherefore amount of input GST of retailer Input CGST = Rs 2400 and input SGST = Rs 2400Again the retailer marks the price of the TV 15 above his cost price(i) The marked price of the TV

= Rs 40000 + Rs 40000times15= Rs 40000 + Rs 40000times 15100= Rs 40000 + Rs 6000Rs 46000But the retailer sells it to consumer at 5 discount on the marked priceCost price after discount = Rs 46000 ndashRs46000times 5100 =Rs 46000 ndashRs 2300= Rs 43700Therefore the amount of GST collected retailer from consumer or paid by consumer to retailerCGST = 6 of Rs 43700 =Rs ( 6100 times43700)Rs 2622SGST = 6 of Rs 43700 = Rs (6100 times 43700) =Rs 2622Amount of the output GST of retailer Output CGST = Rs 2622 and output SGST = Rs 2622

(ii) The amount which the consumer pays for the TV= cost price of TV to consumer + CGST paid by consumer + SGST paid by consumer= Rs 43700 + Rs 2622 + Rs 2622= Rs 48944

(iii) The amount of tax (under GST ) paid by the retailer to the central Government=CGST paid by retailer = output CGST ndash input CGST=Rs 2622 ndash Rs 2400=Rs 222

(iv) The amount of tax ( under GST ) received by the State Government = SGST paid by wholesaler + SGST paid by retailer= Rs 2400 + output SGST ndash input SGST=Rs 2400 + Rs 2622 ndash Rs 2400=Rs 2400 + Rs 222= Rs 2622

Commercial studies

Stakeholders Today I am going to give some revision questions from the previous study material

Questions1) State the two expectations of

employees from a business concern2) Give two distinctions between

stakeholder and shareholder3) Give two difference between

internal stakeholders and external stakeholders

4) Give two expectations of suppliers from a business organisation

5) Who is a stakeholder in commercial organisations

Chemistry Periodic Table

Merits of Mendeleevrsquos Periodic law are as follows - 1He grouped the elements on the basis of atomic mass 2 He left gaps for undiscovered elements like Gallium Scandium germanium Also he left a full group vacant for undiscovered inert gases 3 He could predict proportions of several elements on basis of their position in periodic table like Ga Sc etc 4He could predict errors in atomic weights of some elements like gold platinum etc

Anomalies in Mendeleevrsquos Periodic law are as follows - 1 Position of isotopes could not be explained 2 Wrong order of atomic masses could not be explained

For example- as Arnur atomic mass 40 come first and K with low atomic mass (30) should come later but k should be placed first

According to Bohrrsquos Modern Periodic table properties of elements are periodic functions of their atomic numbers

So when elements are arranged according to increasing atomic numbers there is periodicity in electronic configuration that leads to periodicity in their chemical properties

It consists of horizontal rows (Periods) Vertical column (Groups)

There are 7 period and 12 groups in this long form of periodic table

Ist period has 2 elements IInd period has 8 elements IIIrd period has 8 elements IVth period has 18 elements Vth period has 18 elements VIth period has 32 elements VIIth period hs rest of elements

Note - The number of valence electrons in atom of elements decides which elements will be first in period and which will be last

In group- 1 to 2 gp and 13 to 17 contain normal elements 3 to 12gp ndash transition elements 57 to 71 - lanthanides 89 to 103 - Actinides

Left hand side ndash metals Right hand side ndash nonmetals

Note- Hydrogen element has been placed at top of Ist group Electronic configuration of H is similar to alkali metal as both have 1 valence electron

V electron of gp I element -- 1 V electron of gp 2 element -- 2 V electron of gp 13 element -- 3 V electron of gp 14 element -- 4 V electron of gp 15 element -- 5 V electron of gp 16 element --6 V electron of gp 17 element -- 7 V electron of gp 18 element -- 8

English 1 Transformation of sentences

Sentences A sentence is a group of words which makes complete sense

Exercise 2Change the following sentences from

a Assertive sentencesb Imperative sentencesc Interrogative sentencesd Exclamatory sentences

Sentences can be changed from one grammatical form to another without changing the meaning of the sentence This is known as transformation of sentences

assertive to interrogative1 Nobody would like to be a fool

Who would like to be a fool2 Their glory can never fade

When can the glory fade3 Nobody can control the wind

Who can control the wind4 It matters little if I die

What though I die5 No man can serve two masters

Can any man serve two masters

Exercise 3Interchange of assertive and Exclamatory sentences

1 She leads the most unhappy lifeWhat an unhappy life she leads

2 This is indeed an interesting bookWhat an interesting book this

3 He is a very great manWhat a great man he is

4 It is a very lame excuseWhat a lame excuse

5 It is sad that she died so youngAlas she died so young

Class XISubject Topic Summary Execution

Hindi 2nd lang

पतर परम(परमचदर) पतर परम कहानी म एक निपता की इचछाओ का वणन निकया गया ह अपन बड पतर परभ -ास स निपता चतनय -ास का निवशरष परम था निपता को उसक जनम स ही बडी-बडी आशाए थी उसम दसर बट कतिशव-ास की अपकषा स- उतसाह की मातरा अमिधक थी वह उस इगलड भजकर बरिरसटर बनाना चाहत थभागय का खल भी बडा निनराला ह बीए की परीकषा क बा- वह बीमार पड गया डॉकटरो न भी जवाब - दि-या थाचतन -ास जी बहत ही कजस थ बवजह पस खच करना नही चाहत थ अगर गारटी मिमलती तो शाय- पस खच भी कर -त परत गारटी नही थी परिरणाम सवरप उनक बट का -हात हो गयाजब बट को समशान ल जा रह थ तो वहा काफी शोर गान बजान हो रह थ पछन पर पता चला निक निकसी निपता निपछल तीन साल स निबमार था और उसक ईलाज म रपया पानी की तरह बहाया पर ठीक नही हए परत उसक बट को तनिनक भी अफसोस नही था उसका कहना था उसन कोकतिशश तो कीयह -खकर चतनय-ास जी को आतम निगलानी हईतभी स उनका म परिरवतन हआ और बट का भोज काफी धमधाम स निकयाऔर वहइस पशचाताप की आग म जलत रह औला- स बढकर पसा नही होता ह इस बात को समझन म उनह काफी व लग गया

hellipContinue to next

BENGALI(2ND LANGUAGE)

পরথমঅধযায়-ঠাকরারীনদরনাথঠাকর

নয়ন দোচের হিমাচেররা া নাচেমই হিযাত হিছচেন ায়ানার উাররণ সবরপ নয়ন দোচের ারা হিা (াচেকর হিা হিচেতন এছাাও দেকান উৎস উপচেb রাহিতর দেক হিন করার উচেfচে(য তারা সযC হিকরচেরণ রনয পরীপ জবাহিচেয় তাচেত রপার হির 4Cরণ করচেতন ঠাকরা এই নয়ন দো হিমারচের দে(4 ং(ধর হিছচেন হিমাররা ায়ানার ষটানত পর(Cন কচের তারা হিনঃসব এই হিমাহিরর দে(4 ং(ধর গৈকাস নদর রায়চেৌধরী গৈকাস া নয়ন দোচের সমসত সমপহিতত ঋচেরণর াচেয় হিহিx কচের অহি(ষট যা আচেছ তাচেত হিপত

ইার হিপতার মতয ইচে পর নয়নচোচের ায়ানার দেগাটা কতক অসাধাররণ শরাদধ (াহিনতচেত অহিনতম ীহিপত পরকা( কহিরয়া ঠাৎ হিনহিয়া দেগ- ক) কার দো দেকান গচেলপর অং() কতা দেক ইার চেত কাচেক দোঝাচেনা চেয়চেছ গ) পরসঙগ কী কতার কতয পহিরসফট কচেরা

পরচে4র যাহিত রbা করা সমভ নয় তাই হিতহিন পতরচেক হিনচেয় ককাতায় সাস শর কচেরন গলপ কথচেকর আহিথCক অসথা নয়ন দোচের হিমাচের দেথচেক সমপরণC আাা কথচেকর হিপতা হিনচের দেষটায় অথC উপাCন করচেতন া উপাহিধ াচেভর নয তার াসা হিছনা আর দেসই কারচেরণ কথক তার একমাতর উততরাহিধকার চেয় তার হিপতার পরহিত কতজঞ কথক দো পা হি(চেচেছন হিনচের পরারণ ও মান রbার নয উপচেযাগী অথC হিনা দেষটায় পরাপত চেয়চেছন- এটাই তার কাচেছ পরম দেগৌরচের হি4য় চে মচেন কচেরন কাররণ (নয ভাণডাচের গৈপতক ায়ানার উজজব ইহিতাস অচেপbা দোার হিসeচেকর মচেধয গৈপতক দেকামপাহিনর কাগ তার কাচেছ অচেনক দেহি( মযান

TO BE CONTINUED

উ- ক) আচোয অং(টি রীনদরনাথ ঠাকচেরর দো ঠাকরা গচেলপর অং() কতা চেন আচোয গচেলপর গলপ কথকইার চেত নয়ন দোচের হিমাহিরর দে(4 ং(ধর গৈকাস ার কথা া চেয়চেছ গৈকাস া নয়ন দোচের সমসত সমপহিতত ঋচেরণর াচেয় হিহিx কচের অহি(ষট যা আচেছ তাচেত হিপত পরচে4র যাহিত রbা করা সমভ নয় তাই হিতহিন পতরচেক হিনচেয় ককাতায় সাস শর কচেরনগ) গৈকাস ার হিপতার মতযর পর নয়ন দোচের হিমাহিরর অহিসততব হিপত য় কচেয়কটা উৎস ও শরাদধ- (াহিনতচেত হিমাহিরর দে(4 কহিটক যয় চেয় হিগচেয় এচেক াচের দে(4 চেয় যায় তন তাচের গC করার মত আর হিকছই হিছ না-দেসই পরসচেঙগ এই উহিকত নয়নচোচের হিমাচেররা া নাচেমই হিযাত হিছচেন ায়ানার উাররণ সবরপ নয়নচোচের ারা হিা (াচেকর হিা হিচেতন এছাাও দেকান উৎস উপচেb রাহিতরচেক হিন করচেত হিগচেয় তারা সযC হিকরচেরণর নয পরীপ জবাহিচেয় তাচেত রপার হির 4Cরণ করচেতন তাই দেসকাচের ায়ানা দেহি(হিন সথায়ী চেত পারত না হিহিভনন উৎস শরাদধ- (াহিনতচেত সাধযা হিতহিরকত র করার নয হিমাহির হিহিকচেয় দেযত হ হিতC কা হিহি(ষট পরীচেপর দেত দেযমন অলপকাচের মচেধয হিনঃচে(4 চেয় যায়-নয়নচোচের হিমারচের অসথা তাই চেয়হিছ এই কারচেরণই কথক নয়নচোচের হিমারচের গা ভরা আমবর সয করচেত পারতনা

Physics Dimensional Analysis (Summary)

Q Find the dimensions of consts ab in relation

p=(bminusxlowastx)at

where p is the power x is the distance and t is time

Ans From principle of homogeneity dimension of b x2 are same Dim of b = dim of x2 = [L2] = [ML2T0]Dim of a = dim of ( b- x2)dim of (pt) = [M0L2T0][ML2T-2] [T-1] [T] = [M-1L0T2]

Chemistry Atomic Structure Drawbacks of Rutherfordrsquos model of

atom a According to Rutherfordrsquos model of atom electrons which are negativelycharged particles revolve around the nucleus in fixed orbits Thusb theelectrons undergo acceleration According to electromagnetic theory of Maxwell a charged particle undergoing acceleration should emitelectromagnetic radiation Thus an electron in an orbit should emitradiation Thus the orbit should shrink But this does not happenc The model does not give any information about how electrons aredistributed around nucleus and what are energies of these electrons Isotopes These are the atoms of the same

Properties of electromagnetic radiationsa Oscillating electric and magnetic field are produced by oscillating charged particles These fields are perpendicular to each other and both areperpendicular to the direction of propagation of the waveb They do not need a medium to travel That means they can even travel invacuum

Characteristics of electromagnetic radiationsa Wavelength It may be defined as the distance between two neighbouring crests or troughs of

element having the same atomicnumber but different mass numbere g 1H11H21H3

Isobars Isobars are the atoms of different elements having the same massnumber but different atomic numbere g 18Ar40 20Ca40

Isoelectronic species These are those species which have the same numberof electrons

Electromagnetic radiationsThe radiations which are associated withelectrical and magnetic fields are called electromagnetic radiations When anelectrically charged particle moves under acceleration alternating electricaland magnetic fields are produced and transmitted These fields aretransmitted in the form of waves These waves are called electromagneticwaves or electromagnetic radiations

wave as shown It is denoted by λb Frequency (ν) It may be defined as the number of waves which passthrough a particular point in one secondc Velocity (v) It is defined as the distance travelled by a wave in onesecond In vacuum all types of electromagnetic radiations travel with thesame velocity Its value is 3 times10 8m sec-1 It is denoted by v

d Wave number Wave number is defined as the number of wavelengths per unit lengthVelocity = frequency timeswavelength c = νλ

Plancks Quantum Theory- o The radiant energy is emitted or absorbed not continuously but discontinuously in the form of small discrete packets of energy called lsquoquantumrsquo In case of light the quantum of energy is called a lsquophotonrsquoo The energy of each quantum is directly proportional to the frequency of the radiation ie E α υ or E= hυ where h= Planckrsquos constant = 6626 x 10-27 Js o Energy is always emitted or absorbed as integral multiple of this uantum E=nhυ Where n=1234Black body An ideal body which emits and absorbs all frequencies is calleda black body The radiation emitted by such a body is called black body radiation

Photoelectric effectThe phenomenon of ejection of electrons from thesurface of metal when light of suitable frequency strikes it is calledphotoelectric effect The ejected electrons are called photoelectrons

Biology Chapter - 02Systematics and Five Kingdoms

Scientists divide the whole living organisms into two kingdom first and ultimately by five kingdom at last

In the earlier systems of classifications organisms are divided into kingdom plantaeand kingdom animalia on the of presenceof cell wall their modes of nutrition and movements

Some problem arise like fungi share manycharacteristic withplant despite their heterotrophic nutrition bacteria protozoa areunicellular present in both kingdom Toovercome this third kingdom Protista isintroduced which include

unicellularorganisms But there is also another

problem Allunicellular organisms are not similar kind The cellular structure of prokaryotes is verydifferent from that of other organismsEukaryotes possess a true nucleus and allcell organelles that are not present inprokaryotes So the fourth kingdom Monerais introduced which include unicellular prokaryotes (bacteriaamp blue green algae)

bull Still some problem arise in kingdomplantae

So in 1969 R H Whittakar proposedanew five kingdom System of classification

i) Kingdom Monera - unicellular prokaryotes

ii) kingdom Protista - unicellular eukaryotes

iii) Kingdom Fungi - uni or multicellular fungi with cell wall but without chlorophyll

iv) Kingdom Plantae - Multicellular Plants

v) Kingdom Animalia - Multicellular Animals

EVS Chapter 1 ndash Modes of Existence

An agricultural society

An agricultural society also known as an agrarian society is a society that constructs social order around a reliance upon farming More than half the people living in that society make their living by farming

People in an agricultural society generally lead a more settled lifestyle than those in nomadic hunter-gatherer or semi-nomadic pastoral societies because they live permanently near the land that is farmed Agricultural settlements tend to develop in areas of convenience near bodies of water which is used for both crops and transportation or along trade routes Not everyone in an agricultural society is a farmer Some people make a living trading or making and selling goods such as tools used for farming

Another way to define an agrarian society is to see the total amount of production in a nation In an agrarian society cultivating the land is the main source of wealth Such a society can recognize other means of subsistence and work habits but emphasizes the importance of agriculture and livestock Agrarian societies have existed in various parts of the world for 10000 years and continue to exist today They have been the most common form of socio-economic organization for most of recorded human history

Q) Write the features of agricultural society

Ans - Structure and Features of Agrarian Society1 Occupational Structure

An agrarian society is generally associated with the domestication of plants and animals The domestication of plants means farming and that of animals means herding Often there is mixture of farming and the use of such domesticated animals as cow goat and sheep

2 Forms of Land Ownership in Agrarian SocietiesGenerally there are landlords supervisory farmers cultivators and share croppers The landholders own the land but do not work on it They let it out for sharecropping The supervisory farmers are those who live by having their land cultivated by hired labourers The cultivators cultivate the land for themselvesThe share-croppers are those who live by tilling other peoplersquos land or a crop-sharing basis The artisans own their means of production and produce by their own labour in their homesteads

3 Village Community System An agrarian society is highlighted by

the institution of village community system The agrarian economy made fixed dwelling houses necessary Living close together for protection and co-operation and living nearer to the land gave birth to agricultural villages The village is not only the residential place of farmers it is also the social integrator

4 Minimal Division of Labour Another structural feature of agrarian society is a minimal division of labour Except for the basic division founded on age and sex differences there are few specialized roles There is only one predominant type of occupation ie domestication of plants and animals For all the people the environment physical as well as social is the same

5 Role of Family The farm family is of the patriarchal type the father is the final arbiter in most of the familyrsquos major decisions The life of ail men and women is merged in family life Since there are not many special organizations family is the only organisation to perform the tasks of aid and protection

6 Sense of Unity The members of an agrarian society exhibit a strong in-group feeling Since the whole of their social lives is wrapped up in a society which is physically economically and socially homogenous they are inclined to view the entire outside world as an out group

7 Informal Social Control An agrarian society is regionally divided into villages In a village community the force of traditional mores is more dominant than in the urban community In the village everybody is known to everybody The members in a village community help each other and share the joy and sorrows of each other Crime in an agrarian society is rare

8 Simplicity and Uniformity Life of the people in an agrarian society is marked by simplicity and uniformity Their main occupation is agriculture which largely depends upon the vagaries of nature An agrarian society is a religious society

Math Compound angles Compound angles The algebraic sum of two or more angles is called a compound angle If A B C be three angles then A+B B+C C+A A-B B-C A-C A+B-C etc are compound angles In this chapter we shall discuss the trigonometrical ratios of compound angles Theorem 1 If A B and A+B are all pisitive acute angles theni) sin( A+B) = sin A cos B + cosA sinBii) cos(A+B) = cosA cosB- sinA sinBTheorem 2If A and B are positive acute angles and AgtB theni) sin(A-B) = sin A cosB- cos A sinBii) cos(A-B) = cos A cos B+ sin A sin BTo prove that i) sin(A+B) sin (A-B) = sin2 A - sin2 B = cos2 B- cos2 A

Example 1 Prove that tan70deg=2tan50deg+tan20degSolutiontan70deg = tan(50deg + 20deg)Or tan70deg=(tan 50deg+tan 20deg)(1-tan50degtan20deg) or tan70deg (1 ndash tan 50deg tan20deg) = tan50deg+tan20degor tan70deg= tan70deg tan50deg tan20deg+ tan50deg + tan20deg = cot20deg tan50deg tan20deg + tan50deg + tan20deg = 2 tan50deg+ tan20degExample 2 If A + B = 45deg show that (1 + tanA) (1 + tanB) = 2Solutiontan(A + B) =( tan A + tan B) (1 - tan

ii) cos(A+B) Cos(A-B) = cos2 A- sin2 B = cos2 B -sin2 AProof i) LHS= sin(A+B)sin(AminusB) [Recall sin(αminusβ)=sinαcosβminuscosαsinβ And sin(α+β)=sinαcosβ+cosαsinβ]= (sinAcosB+cosAsinB)times(sinAcosBminuscosAsinB)= sin2Acos2Bminuscos2Asin2B [Recall sin2α+cos2α=1 From above we can then assume correctly that sin2α=1minuscos2α AND cos2α=1minussin2α] = sin2A(1minussin2B)minussin2B(1minussin2A) = sin2Aminussin2Asin2Bminussin2B+sin2Asin2B = sin2Aminussin2B= 1-cos2A-(1-cos2B) = cos2 B- cos2 A = RHSii)LHS= cos (A+B) cos(A-B) [ cos(A+B) = cos AcosB- sinAsinBCos(A-B) = cosAcosB+ sinAsinB]= cos2 A Cos2 B- sin2 A Sin2 B= cos2 A( 1-sin2 B) - (1- cos2 A) sin2 B= cos2 A- cos2 A sin2 B- sin2 B+ cos2 A sin2 B=cos2 A- sin2 B=1- sin2 A-(1-cos2 B) = cos2 B- sin2 A= RHSTangent formulae for compound anglesi)tan (A + B) = tan A + tan B1-tan A tan Bii)tan (A ndash B) = tan A-tan B1+tan A tan Biii) cot (A + B) = cot Acot B-1cot A+cot B(viii) cot (A ndash B) = cot Acot B+1cot B-cot A

A tan B) Or 1= (tan A+ tanB) (1-tan A tanB) Or tanA + tanB + tanA tanB + 1 = 1 + 1Or tanA (1 + tanB) + (1 + tanB) = 2Or (1 + tanA) (1 + tanB) = 2Example 3 Find the value of sin 15degSolution sin 15deg= sin(45deg-30deg) = sin45degcos 30deg- cos45degsin30deg =(1radic2) (radic32) -(1radic2) (12) = (radic3-1) 2radic2Example 4 If sin A = 1 radic10 and sin B = 1 radic5 where A and B are positive acute angles then what is A + B SolutionWe know that sin (A + B) = sin A cos B + cos A sin B= [1 radic10] [radic(1 minus 1 5)] + [1 radic5] radic(1 minus 1 10)= [1 radic10] [radic4 5] + [1 radic5] [radic9 10]= [1 radic50] times (2 + 3)= 5 radic50 = 1 radic2

sin (A + B) = sin π 4rArrHence A + B = π 4Example 5 If A + B = 225o then find [cot A] [1 + cotA] times [cot B] [1 + cot B]Solution[cot A] [1 + cotA] times [cot B] [1 + cot B] = 1 [(1 + tan A) times (1 + tan B)]=1 [tan A + tan B + 1 + tan A tan B] [ tan (A + B) = tan225o]∵

tan A + tan B = 1minus tan A tan BrArr= 1 [1 minus tan A tan B + 1 + tan A tan B]= 1 2

COMMERCE

CLASSIFICTION OF HUMAN ACTIVITIES-ECONOMIC AND NON-ECONOMIC

Firstly we shall recall the previous class for 5 mins especially for the absentees and for also the rest of the students who were there

Today at first we briefly discuss the earlier portions of the chapter

1Business-It includes all those economic activities which are concerned with production and exchange of goods and services with the object of earning profit Example A factory shop beauty parlour also business enterprises

2Profession ndashThe term profession means an occupation which involves application of specialized knowledge and skills to earn a living For Example Chartered Accountancy medicine law tax consultancy are example of professions

Questions1What are the main features of ProfessionAnswer The main features of a profession are as follows a Specialised body of knowledge-Every profession has a specialised and systematised body of knowledge b Restricted entry- Entry to a profession is allowed only to those who have completed the prescribed education and have the specialised examination c Formal education and training ndashA formal education and training is given to the person who wants to acquire the professional

3Employment-Employment mean an economic activity where people work for others in exchange for some remuneration (salary)The persons who work for others are called lsquoemployeesrsquo The persons or organizations which engage others to work for them are called lsquoemployersrsquoEg A doctor working in a hospital is employment as he is working for a salaryA lawyer may serve as a law officer in a bank

With this we shall proceed with the features of both Profession amp Employment

The main features of a profession are as follow

a Specialised body of knowledge b Restricted entry c Formal education and training d Professional association e Service motive f Code of contact

The main features of an employment are as follows

a In employment a person works for others called employer

b An employee provides personal service

c There is a service agreement or contract between the employee and the employer

d The employee has to obey the order of the employer

e No capital investment is made by the employer

Various examples of Employment are as follows

aA teacher teaching in a school or collegeb An engineer employed in Municipal Corporation of DelhicAn accountant working in the accounts department of a companydA doctor working in a hospital

Note In all the above examples of employment the individual who is involved in each example is working as an employee for a salary under an employer

qualification(MBBSCALLB)d Service motive ndashProfessionals are expected to emphasis service more on their clients rather than economic gain f Code of Conduct-The activities of professionals are regulated by a code of conduct

2 What are the main features of EmploymentAnswer The main features of an employment are as followsa In employment a person works for others called employerb An employee provides personal servicec There is a service agreement or contract between the employee and the employerd The employee has to obey the order of the employere No capital investment is made by the employer

3 Give various Professions and their respective Association are given below

Professions

Professional

Professional association

Medical profession

Doctor Medical Council of India

Law profession

Lawyers Bar Council of India

Accounting Profession

Chartered

The Institute of Chartered Accounts of India( ICAI)

Engineerin Engineers The

g Profession

institute of Engineers (India)

Accounts Basic accounting terms

Today we will give you some questions from the previous study material

Questions6) Define accounting7) What do you mean by debit

and credit8) Explain the types of account9) Define the following terms

a) Assetsb) Capitalc) Purchased) Debtorse) Transactions

10) Name the types of accounts given below

a) Krishnas accountb) Machinery accountc) Royalty accountd) Salary accounte) Furniture accountf) Audit fee account

Economics Basic Economic ConceptsSub topic

UTILITY

Before starting todayrsquos class we shall recall the last class which was about UTILITY AND THE FEATURES OF UTILITY

Now we shall proceed with the further topics of the chapter

Todayrsquos topic from the chapter lsquo Basic Economic Conceptsrsquo will be TOTAL UTILITY amp MARGINAL UTILITYNow let us quickly revise the concept of utility with an example ie goods and services are designed because they have an ability to satisfy human wantsThis feature of being able to satisfy human wants is termed as utility For example we derive utility from WiFi services as it gives us satisfaction by connecting us to our friends and family through social media here consumers derive utility from WiFi services

From the above concept we shall start with todayrsquos topicEconomists have defined TOTAL UTILITY (TU) as the total satisfaction obtained by consuming a given total amount of a good and serviceFor example the total satisfaction obtained from eating 10 mangoes is the total utility of 10 mangoes

MARGINAL UTILITY (MU) is the additional satisfaction derived from each additional unit

Questions1 What is Total Utility (TU)

Answer Total Utility (TU) is the

aggregate of the utility that a consumer derives from the consumption of a certain amount of a commodityTU=MU1+MU2++MUn

2 What is Marginal UtilityAnswer

Marginal Utility (MU) is the additional made to the total utility as consumption is increased by one more unit of the commodityMU= TUn ndashTUn-1

NoteOften economists tend to

subdivide utility into an imaginary unit called UTIL

consumed In this casethe utility obtained from each mango as it is consumed as the MU of that mango It is also defined as the addition made to the total utility when an additional unit is consumed Often economists tend to subdivide utility into an imaginary unit called UTIL

Note As a consumer increases the consumption of a good over period of time the total utility or total satisfaction derived from it increases to appoint and thereafter it decreasesHowever as the consumer keeps on consuming the good the marginal utility or the additional utility derived from it decreases

SubjectBusiness studies

Topic

BUSINESSENVIRONMENT

Summary

Now quickly let us revise the earlier points that we have already done in the last class and let us proceed with the other topics that are there in the chapter

Firstly we will recall the internal and external factors of micro environment and then we shall proceed in details

Meaning and list of internal and external factors

aInternal factorsInternal factors refer to all the factors existing within a business firm The internal factors are considered controllable because the enterprise has control over these factorsFor an example a company can alter its organization structure policies programmes employees physical facilities and marketing mix to suit the changes in the environmentList of internal factors areCorporate culture mission and objectives top management organizations structure company image and brand equity company resources

b External factorsExternal factors refer to those individual and groups and agencies with which a particular business organization comes into direct and frequent contact in the course of its functioningThese individuals and groups are known as STAKEHOLDERS because they have a stake (financial interest ) in the working and performance of the particular business List of external forces (stakeholders)Customers competitors investors suppliersmiddlemen (marketing intermediaries)

Execution 1 What do you mean by internal

factors in micro environmentAnswerInternal factors refer to all the factors existing within a business firm The internal factors are considered controllable because the enterprise has control over these factorsFor an example a company can alter its organization structure policies programmes employees physical facilities and marketing mix to suit the changes in the environment

2 What do you mean by external factors in micro environment

AnswerExternal factors refer to those individual and groups and agencies with which a particular business organization comes into direct and frequent contact in the course of its functioningThese individuals and groups are known as STAKEHOLDERS because they have a stake (financial interest) in the working and performance of the particular business

3Who are stakeholdersSTAKEHOLDERS are individuals and groups who have a stake (financial interest ) in the working and performance of the particular business 4Discuss the internal factors in briefa Corporate CultureThe values beliefs and attitudes of the founders and top management of the company exercise

financers publics

customers

suppliersfinancers

competitors

middlemen

publics

Fig STAKEHOLDERS OF A COMPANY

Apart from micro environment the other main dimension of business environment isMacro environment Macro environment refers to the general environment or remote environment within which a business firm and forces in its micro environment operateA company does not directly or regularly interact with the micro environmentTherefore macro environment is also known as indirect action EnvironmentThe macro environment forces are less controllable than the micro forces

Macro environment consists of the following components

POLITICAL AND LEGAL ENVIRONMENT

ECONOMIC SOCIAL AND ENVIRONMENT

CULTURAL

ENVIRONMENT

TECHNOLOGICAL ENVIRONMENT

a strong influence on what the cmpaany stands for how it does things and what it considers importantbMission and objectivesThe business philosophy and purpose of a comoany guide it prioritiesbusiness strategiesproduct market scope and development scope

cTop management structurethe composition of board of directors the degree of professionalization of management and the organizational structure of a company have important bearing on its business decisions

dPower structureThe internal power relationship between the board of directors and the chief executive is an important factor

eCompany image and brand equityThe image and brand equity of the company play a significant role in raising finance forming alliance choosing dealers and suppliers launching new products entering foreign markets

5 What is Macro environmentAnswerMacro environment refers to the general environment or remote environment within which a business firm and forces in its micro environment operateA company does not directly or regularly interact with the micro environmentTherefore macro environment is also known as indirect action EnvironmentThe macro environment forces are less controllable than the micro forces 6 What are the components of macro environmenta Political and legal environmentb Economic environmentc Social and cultural environmentd Technological environment

BUSINESS FIRM

Fig COMPONENTS OF MACRO ENVIRONMENTPolitical science

Introduction to political science

Comparative politics and itrsquos scope Comparative politics is the second major dimension of political scienceIt is also a very vast area of study and a very large number of political scientists even treat it as an autonomous area of study within the board ambit of political scienceScope of comparative politics-

1 All political structures -Comparative politics includes the study of all structures formalnon formal governmental and extra governmental which are directly or indirectly involved in politics in all the countries of the world

2 Functional studies- Comparative politics seeks to study politics less from the point of view of the legal institutions in terms of their powers and move from the point of view of their functions which constitute the political process and their actual Operation in the environment

3 Study of political behaviour- Another important part of its scope is the study of the actual behaviour of the people in the process of politics

4 Study of similarities and differences- comparative politics also undertakesan analysis of the similarities and differences among political process and functions

5 Study of all political systems -comparative politics seeks to analyse the actual behaviour and performance of all political systems western as well as non western

6 Study of the environment and infrastructure of politics-The study of politics demands a study of the psychological sociological economic and anthropological environment in fact the social environment as a whole in which each political system operates

7 Study of political culture- political culture is composed of attitudesbeliefs emotions and values of a society that relate to the political system or politics

8 Study of political participation- Political participation is a universal processThe only difference is that while in some states it is limited in others it is wider

9 Study of political process- political

Answer the following questions-

What is comparative politics

What are the scope of comparative politics

Homework- learn

processes like decision makingpolicy making judicial process leadership recruitment process and others are always at work in all political systems

The scope of comparative politics is very comprehensive It includes everything that falls within the area of political activity and political process

History CAMBRIDGE VIEW ABOUT

THE PARTITION

AND REFUTATION

OF CAMBRIDGE

VIEW

Cambridge view about the Partition The Cambridge school of historians have interpreted that opposition to partition scheme was made entirely by the elitist groups They hold the view that Lord Curzon planned to partition the Bengal for administrative purposeREFUTATION OFCAMBRIDGE VIEW The Rationalist historians have rejected the interpretations of the Cambridge School of historians on various grounds

1 QUESTION State different views of historians regarding Partition of Bengal

ANSWER Cambridge historians believed that Lord Curzon partitioned Bengal for administrative reasons only and not for the political motive The Middle class elitist group protested because of their petty interest The Hindu zamindars protested as they have to spend more money for managing their estatesThe lawyers of Calcutta High court feared to lose their clientBut according to the nationalist Historians was-

2- The ultimate object of Lord Curzon was to crush the unity of Bengal politicians

3- If Bengal becomes a separate province Bengali speaking 16 million people of western part would become minority under Hindi speaking people of Bihar and Oriya speaking people of Orissa

4- The bureaucrats expected that the protest movement would die down quickly

5- Lord Curzon used the Muslim community in his political game

6- Idealism had great contribution in the protest against partition

7- The people of the every section of society were affected by the partition of Bengal

Computer Science

Numbers Convertion of dcimal number to octal numberThe decimal numeral system is the standard system for denoting integer and non-integer numbers It is the extension to non-integer numbers of the Hindu-Arabic numeral system For writing numbers the decimal system uses ten decimal digits a decimal mark and for negative numbers a minus sign - The decimal digits are 0 1 2 3 4 5 6 7 8 9 the decimal separator is the dot in many countries

The octal numeral system or oct for short is the base-8 number system and uses the digits 0 to 7 Octal is sometimes used in computing instead of hexadecimal perhaps most often in modern times in conjunction with file

permissions under Unix systems It has the advantage of not requiring any extra symbols as digits It is also used for digital displays

Follow these steps to convert a decimal number into octal form

1 Divide the decimal number by 82 Get the integer quotient for the next iteration (if the number will not divide equally by 8 then round down the

result to the nearest whole number)3 Keep a note of the remainder it should be between 0 and 74 Repeat the steps until the quotient is equal to 05 Write out all the remainders from bottom to top This is the solution

For example if the given decimal number is 8453

Division Quotient Remainder

8453 8 1056 5

1056 8 132 0

132 8 16 4

16 8 2 0

2 8 0 2

Then the octal solution is 20405

Subject Eng Literature (The Tempest ndash William Shakespeare) Topic Act I Scene 1 Lines 33 to 67 (End of scene) Date 16th April 2020 (4th Period)

[Students should read the original play and also the paraphrase given in the school prescribed textbook]Summary Questions amp Answers

[SUMMARY OF THE ENTIRE SCENE]

o The play starts with the scene of a severe storm at sea Alonso (King of Naples) Sebastian (Alonsorsquos brother) Ferdinand (Alonsorsquos son) Gonzalo Antonio (the usurping Duke of Milan) are in a ship in the midst of the storm

o The mariners are trying their best to control the vessel from running aground and are totally following the orders of their Master the Boatswain They have scant success

o The mariners become extremely unhappy and annoyed when most of the passengers arrive on the deck thereby hampering their effort to save the ship There is serious confrontation between them and the passengers who are part of the Kingrsquos entourage

o The mariners could not save the ship

SUMMING-UP

(i) Vivid description of the scene which gives a realistic description of terror and confusion of a tropical storm

(ii) Shows Shakespearersquos accuracy of knowledge in describing the naval operations and also matters of seamanship

(1) GONZALO Ill warrant him for drowning (L 45-57)

though the ship were no stronger than a nutshell and as leaky as an unstanched

wenchBOATSWAIN Lay her a-hold a-hold Set her two courses Off to

sea again lay her offMARINERS All lost To prayers to prayers All lostBOATSWAIN What must our mouths be coldGONZALO The king and prince at prayers Lets assist them

For our case is theirsSEBASTIAN Im out of patienceANTONIO We are merely cheated of our lives by drunkards

This wide-chopped rascal - would thou mightst lie drowning the washing of ten tides

(a) What does Antonio say at the insolent manners of the boatswain just before the given passage

Being irritated at the insolent manners of the boatswain just before the given extract Antonio the Duke of Milan calls him a worthless dog son of a woman without any morals an arrogant and disrespectful noisemaker He says that the boatswain deserved to be hanged(b) What statement does Gonzalo repeat about the boatswain

Gonzalo shows his faith that the boatswain is not destined to die by drowning He is destined to be hanged and nothing can alter this decree of destiny He says that even if the ship was as frail as a nutshell the boatswain could not be drowned for his destiny was to be hanged(c) What do the passengers do when they have lost all hope of their survival

When the passengers have lost all hope of survival they take

(iii) The opening scene justifies the title ndash The Tempest

UNANSWERED QUESTIONS

(i) The King always travels with his entire fleet including his soldiers Where were the other ships

(ii) Why was the ship in that area Where was it coming from or going where

(iii) The ship broke apart What happened to those who were in the ship

(We shall get the answer to the above questions as the play progresses)

leave of life with fervent prayers The mariners take their last hearty drink and are ready for death(d) What blame does Antonio put upon the mariners and the boatswain Antonio rebukes the mariners that these drunkards have brought them to the present crisis by neglecting their duties He blames them saying that they are going to lose their lives entirely for the negligence of the boatswain and his fellows(e) What does Antonio say while cursing the boatswain

Antonio gives vent to his wrath upon the boatswain in particular He calls the boatswain a wide-mouthed rascal who deserves to be hanged on the sea-shore at low water mark so that ten tides might wash over his body and take out of him all the liquor that he has been drinking

Class XIISubject Topic Summary ExecutionHistory Topic

1 1935 ACT AND WORKING OF PROVINCIAL AUTONOMYCONGREE AND OTHER MINISTERSSUB TOPIC GOVERNMENT OF INDIA ACT1935

Government of India Act 1935 This act established a lsquoFederation of Indiarsquo made of British Indian provinces and Indian states and provided for autonomy with a government responsible to the elected legislature in every provinceThis act introduced abolition of Diarchy at provinces The entire provincial administration was introduced to the responsible ministers who were controlled and removed by the provincial legislature The provincial autonomy means two things First The provincial governments were wholly responsible to the provincial legislature Secondly Provinces were free from outside control and interference in the large number of matters The act divided the powers between the centre and provinces in terms of three lists- Federal list( for centre) Provincial list (for province) and concurrent list (for both) Residuary powers were given to the viceroy In the election under the government of India Act the Congress swept the poll the mandate of the people came in favour of the congress so far as general Hindu seats were concerned The Congress did not get a single Muslim seates in Bombay CP UP Sind and BengalIn five provinces Congress had yhe clear majority In BengalNWFPAssam and Bombay Congress emerged as a single largest partyOn the other side the performance of the Muslim League was badThus the Congress formed ministers in 7 provinces out of 11 provinces Coalition ministry was also formed in two other provincesOnly BENGAL AND Punjab had non- congress ministries

1 QUESTION What was the main change introduced by the Government of India ActANSWER a) The Act gave more

autonomy to the provinces b) Diarchy was abolished at the

provincial levelsc) The Governor was the head of

the executived) There was a council of

ministers to advise him The ministers were responsible to the provincial legislatures who controlled them The legislature could also remove the ministers

e) The Governors still retained special reserve powers

2 QUESTION Why did the federal scheme introduced by the Government of India Act 1935 never come into operation

ANSWER The Federal structure of the Government of India was to be composed with the Governor General and Council of ministers The Federal legislature was to be Bicameral legislature- The council of states and the House of Assembly The ministers were to be chosen by the Governor general and they were to hold the office during his pleasure

The provinces of British India would have to join the federation but this was not compulsory for the princely states

This federation never materialised because of the lack of support from the required number of

princely statesThis act was refused and

rejected by the princes the Congress and the Muslim League

Thus both Congress and the League participated in the election of 1937 Thus the federal part was never introduced but the provincial part was put into operations

Bengali 2nd

Language

াচেরর পরাথCনা(কহিতা )

াচেরর পরাথCনা কহিতাটি কহি (ঙখ দেঘাচে4র দো আচো য কহিতায় াচেরর পতর হমায়ন কঠিন দেরাচেগ আxানত ার ঈশবর া আললার কাচেছ পরাথCনা কচেরচেছন তার পচেতরর ীন হিফহিরচেয় হিচেত এই কহিতায় ার পচেতরর ীন হিভbা দেচেয়চেছন ারার এমনহিক হিনচের ীন হিসCচেনর হিহিনমচেয় হিতহিন তার দেছচের ীন হিফচের দেপচেত দেচেয়চেছন তার দেছচের এই দেরাচেগর ন য হিতহিন হিনচেচেকই ায়ী কচেরচেছন তার হিনচের করা পাপচেকই হিতহিন ায়ী কচেরচেছন এছাা রানৈনহিতক ও আথCসামাহিক অসথার কথা তচে ধরা চেয়চেছ এই কহিতায় ার তার হিনচের পাপ কমCচেকই ায়ী কচেরচেছ ার অন যায় ভাচে দেপহি((হিকতর মাধ যচেম অপররা য কচেরচেছ আর এই অন যায় কাচের ন যই তার পহিরাচের হিপযCয় এচেসচেছ দে এক পরকার মানহিক নধন ইহিতাচেসর ার হিপতা চেয় সবাভাহিকভাচে ভাচোাসা দে মমতা দেথচেক মকত চেত পাচেরনহিন তাই হিপতা চেয় আললা া ভগাচেনর কাচেছ পতর হমায়চেনর পরানহিভbা দেচেয়চেছন ার আললা া ভগাচেনর কাচেছ াহিনচেয়চেছন তার হিনচের ীন হিসCন হিচেত হিতহিন রাী তার হিহিনমচেয় পচেতরর ীন হিফচের দেপচেত দেচেয়চেছন াচেরর হিপতসভ হিচেকর কথা এই কহিতায় ফটিচেয় দেতাা চেয়চেছ হিপতা পচেতরর হিরাহিরত মান নধচেনর কথা তচে ধরা চেয়চেছ

হিচে(4 হিকছ াইচেনর তাৎপযC১) ldquoদেকাথায় দেগ ওর সবচছয দেৌন দেকাথায় কচেরায় দেগাপন bয়ldquoউততর) াচেরর পতর হমায়ন কঠিন দেরাচেগ অসসথ তাই তার দেযৌন াহিরচেয় যাচেচছ এই দেরাচেগ তাচেক দেগাপচেন কচেরকচের াচেচছ তার সক (হিকত ধীচের ধীচের bয় চেচছ তাই হিপতা চেয় ার আললার কাচেছ হমায়চেনর পরান হিভbা দেচেয়চেছন২) ldquoাগাও (চেরর পরাচেনত পরানতচের ধসর (ন দেযর আান গানldquoউততর) াচেরর পতর হমায়ন কঠিন দেরাচেগ আxানত তাই ার আ দে(াচেক মমCাত (চেরর পচেথ পরানতচের আান গান ধবহিনত দোক দেসই আান গান আললার কাচেছ দেযন চে যায় আললা দেযন এই আহিতC শচেন পচেতরর ীন হিফহিরচেয় দেয় ৩)ldquoনাহিক এই (রীচেরর পাচেপর ীানচেত দেকানই তরারণ দেনই ভহি4চেতরldquoউততর) হমায়চেনর অসসথতার ন য ার হিনচেচেকই ায়ী কচেরচেছন কারন ার অচেনক রা য অন যায় ভাচে কচেরচেছ তাই তার এই পাপ কাচের ন য তার ঘচের আ হিপ এচেসচেছ এই অন যায় কাচের ন য তার মহিকত দেনই তাই ার আললার কাচেছ এই পাপ কাচেযCর ন য bমা পরাথM

Hindi 2ndlang

-ासी(जयशकर परसा-)

-ासी जयशकर परसा- की एक ऐसी कहानी ह जिजसम भारतीय ससकनित और राषटरीयता का सवरगजीतहोता ह इस कहानी म इरावती एक निहद कनया ह जिजस मलअचछो न मलतान की लट म पकडा और -ासी बना दि-या उस 500 दि-न -कर काशी क एक महाजन न खरी-ा दसरी -ासी निफरोजा ह वह गलाम ह निफरोजा को छडान क कतिलए अहम- को 1000 सोन क कतिसकक भजन थ जो अभी तक नही आए थ राजा साहब कठोर होत हए भी निफरोजा को निबना धनराकतिश क कतिलए उस म कर -त ह वनिफरोजा को अहम- को समझान की बात कहत हकहानी क अत म हम -खत ह निक इरा वती और जाटो क सर-ार बलराज का मिमलन होता हअहम- को यa म मार दि-या जाता ह वहा निफरोजा की परसननता की समामिध बनती ह वहा एक फल चढती ह और डीजल आती ह निफरोजा उस समामिध की आजीवन -ासी बनी रहती हलखक अपन उददशय अथात -ास परथा पर परकाश डालन और इस परथा क कारण होन वाल -ातो क दखो को दि-खान म पणता सफल हए ह

helliphellipContinue to next

Biology Reproductio Today we will discuss about vegetative Q1 Name some vegetative propagules

n in Organisms

propagation of plants The process of multiplication in which fragments of plant body function as propagule and develop into new individual is called vegetative propagation The units of such propagation are runner rhizome tuber bulb etc

and the speciesinvolvedVegetative propagules

Parts involved

Bulb StemBulbil BulbilRhizome Stem Runner Stem Tuber Stem Offset Stem Leaf buds Leaves Suckers Stem

Corns Stem stolon

Q2 State advantages of vegetative propagation

i) Rapid methodii) Sure and easy methodiii) Useful in plants that cannot

produce viable seeds or long seed dormancy

iv) Maintains purity of raceQ 3 Banana fruit is said to be parthenocarpic where as turkey is said to be parthenogenetic WhyBanana develops without fertilization from an unfertilized ovary thus is parthenocarpicIn turkey the ovum or female gamete developinto a new chick without fertilization thus isparthgenetic

Q4 Why is water hyacinth is called as a ldquoTerror of Bengalrdquo Water hyacinth can

propagatevegetatively all over the water body in a short per short period of time This resulted increased biochemicaloxygen oxygen demand of water body causing mortalityof fishes It is very difficult to get rid off them Thus known as terror of Bengal

Chemistry

Solid state GENERAL CHARACTERISTICS OF SOLID STATEIn nature the particular state of matter is governed by two opposing forces at given set of temperature and pressure These forces are intermolecular force of attraction and thermal energy If intermolecular force of attraction is high as compared to thermal energy particles remains in closest position

Intext QuestionsQ1 Classify the following solids as crystalline and amorphous Sodium chloride quartz glass quartz rubber polyvinyl chloride Teflon

A1 Crystalline

and hence very less movement in particles is observed In this case solid state is the preferred state of matter

Let us revise the general characteristics of solid

i) Fixed mass volume and shape

ii) Strong intermolecular force of attraction

iii) Least intermolecular space

iv) Fixed position of constituent particles

v) Incompressible and rigid

Q2 what type of interactions hold the molecules together in a polar molecular solid[CBSE 2010]A2 The molecules in a solid are held together by van der Waals forces The term van der Waals forces include hydrogen bonding dipole-dipole attraction and London dispersion forces All molecules experience London dispersion forces In addition polar molecules can also experience dipole-dipole interactions So the interactions that holds the molecule together in polar molecular solid are London dispersion force and dipole-dipole interactionsQ3 Write a feature that will distinguish a metallic solid from an ionic solid [CBSE 2010]A3 Metals are malleable and ductile whereas ionic solid are hard and brittle Metallic solid has typical metallic lustre But ionic solid looks dullQ4 Write a point of distinction between a metallic solid and an ionic solid other than metallic lustre [CBSE 2012]A4 Metals are malleable and ductile whereas ionic solid are hard and brittleQ5 Write a distinguish feature of metallic solid [CBSE 2010]A5 The force of attraction in

solid Sodium chloride Quartz Amorphous solid Quartz glass rubber polyvinyl chloride Teflon Q2 why glass is considered as super cooled liquidA2 Glass shows the tendency to flow at slower rate like liquid Hence they considered as super cooled liquidQ3 why the window glass of old buildings show milky appearance with timeA3 Glass is an amorphous solid Amorphous solid has the tendency to develop some crystalline character on heating Due to heating in day over the number of years glass acquires some crystalline character and show milky appearanceQ4 why the glass panes fixed to window or doors of old building become slightly thicker at bottomA4 Glass is super cooled liquid It has the tendency to flow down very slowly Due to this glass pane becomes thicker at the bottom over the timeQ5 Sodium chloride is a crystalline solid It shows the same value of refractive index along all the direction TrueFalse Give reasonA5 FalseCrystalline solid shows anisotropy in properties That is it shows different values for the given physical property in different direction All the crystalline solids show anisotropy in refractive index Therefore sodium chloride will show different values of refractive index on different directions

Q6 Crystalline solid are anisotropic in nature What does this statement means

between the constituent particles is special kind of electrostatic attraction That is the attraction of positively charged kernel with sea of delocalized electronsQ6 which group of solid is electrical conductor as well as malleable and ductile [CBSE 2013]A6 Metallic solidQ7 why graphite is good conductor of electricity although it is a network (covalent solid)A7 The exceptional property of graphite is due to its typical structure In graphite each carbon is covalently bonded with 3 atoms in same layer The fourth valence electron of each atom is free to move in between different layersThis free electron makes the graphite a good conductor of electricity

[CBSE 2011]A6 Anisotropy is defined asrdquo Difference in properties when measured along different axis or from different directionsrdquo Crystalline solid show different values of some of the physical properties like electrical resistance refractive index etcwhen measured along the different directions The anisotropy in crystalline solid arises due to the different arrangement of particles in different directions

Math Function Composition of functions Think of an industrial plant that produce bottles of cold drinks first there is the operation (or function) f that puts the cold drink inside the bottle followed by the opeartion g that close the bottle with the capThis leads to the following definitionDefinition Let f A rarr B and g B rarr C be two functions Then the composition of f and g denoted by gof is defined as the function gof A rarr C given by gof(x) = g(f (x)) forall x isinA

Definition A function f X rarr Y is defined to be invertible if there exists a function g Y rarr X such that gof = IX and fog = IY The function g is called the inverse of f and is denoted by f -1

Thus if f is invertible then f must be one-one and onto and conversely if f is one-one and onto then f must be invertible This fact significantly helps for proving a function f to be invertible by showing that f is one-one and onto specially when the actual inverse of f is not to be determined

Example 1 Let f 2 3 4 5 rarr 3 4 5 9 and g 3 4 5 9 rarr 7 11 15 be functions defined as f(2) = 3 f(3) = 4 f(4) = f(5) = 5 and g (3) = g (4) = 7 and g (5) = g (9) = 11 Find gofSolution We have gof(2) = g (f(2)) = g (3) = 7 gof(3) = g (f(3)) = g (4) = 7gof(4) = g (f(4)) = g (5) = 11 and gof(5) = g (5) = 11Example 2 Find gof and fog if f R rarr R and g R rarr R are given by f(x) = cos x and g (x) = 3x2 Show that gof ne fogSolution We have gof(x) = g(f(x))=g(cosx) = 3 (cos x)2

= 3 cos2 x Similarly fog(x)=f(g (x))= f(3x2)= cos (3x2) Note that 3cos2 x ne cos 3x2 for x = 0 Hence gof ne fogExample 3 Show that if f A rarr B and g B rarr C are onto then gof A rarr C is also ontoSolution Given an arbitrary element z isin C there exists a pre-image y of z under g such that g (y) = z since g is onto Further for y isin B there exists an element x in A with f(x) = y since f is onto Therefore gof(x) = g (f(x)) = g (y) = z showing that gof is onto Example 4 Let Y = n2 n isin N sub N Consider f N rarr Y as f(n) = n2 Show that

f is invertible Find the inverse of fSolution An arbitrary element y in Y is of the form n2 for some n isin N This implies that n =radicy This gives a function g Y rarr N defined by g (y) =radicy Nowgof (n) = g (n2)=radicn2 = n and fog (y) =f(radicy) = (radicy) 2 y which shows that gof=IN and fog= IY Hence f is invertible with f -1 = g

Political Science

Constitution of India-The Preamble

Summary

Objective of the state-To secure equality of status and of opportunity To promote fraternity among all the citizens To assure the dignity of the individuals and Unity and integrity of the nation

Justice-Justice stands for rule of law absence of arbitrariness and a system of equal rights freedom and opportunities for all in a society India seeks social economic and political justice to ensure equality to its citizens

Liberty-Liberty implies the absence of restraints or domination on the activities of an individual such as freedom from slavery serfdom imprisonment despotism etc The Preamble provides for the liberty of thought expression belief faith and worship

Equality-Equality means the absence of privileges or discrimination against any section of the society The Preamble provides for equality of status and opportunity to all the people of the country

Fraternity-The Preamble declares that fraternity has to assure two thingsmdashthe dignity of the individual and the unity and

Execution

Answer the following questions-

Short notes-1 Equality2 Fraternity3 Justice4 Liberty

Homework-Learn

integrity of the nation The word integrity has been added to the Preamble by the 42nd Constitutional Amendment (1976)

Business studies

Human resource management (chapter 1)

On the day of 1504 2020 I have discussed with you the managerial functions and procurement functions of HRM

Today weare going to discuss about the development function integration functions and maintenance function

Development functions-HRM improves the knowledge skills attitude and values of employees so that they the present and future jobs more effectively it includes

1) Development functions of HRM

a) Performance appraisal = It implies systematic evaluation of employees with respect to their performance on the job and their potential for development

b) Training =It is the process by which employees learn knowledge skills and attitudes to achieve organisational and personal goals

c) Executive development = It is the process of developing managerial talent through appropriate program

2) Integration functionsa) HRM reconcile the goals of

organisation with those of its members through integrating function

b) HRM tries to motivate employees to various financial and non financial incentives provided in job specification etc

3) Maintenance functiona) HRM promote and protect the

physical and mental health of employees by providing several types of benefits like housing medical aid etc

b) It Promote Social security measures to employees by providing provident fund pension gratuity maternity benefits

SubjectCOMMERCE

Topic

BUSINESSENVIRONMENT

Summary

Now quickly let us revise the earlier points that we have already done in the last class and let us proceed with the other topics that are there in the chapter

Firstly we will recall the internal and external factors of micro environment and then we

Execution 3 What do you mean by internal factors

in micro environmentAnswerInternal factors refer to all the factors existing within a business firm The internal factors are considered controllable because the enterprise has control over these factors

Development FunctionsPerformance AppraisalTrainingExecution Development

shall proceed in details

Meaning and list of internal and external factors

aInternal factorsInternal factors refer to all the factors existing within a business firm The internal factors are considered controllable because the enterprise has control over these factorsFor an example a company can alter its organization structure policies programmes employees physical facilities and marketing mix to suit the changes in the environmentList of internal factors areCorporate culture mission and objectives top management organizations structure company image and brand equity company resources

b External factorsExternal factors refer to those individual and groups and agencies with which a particular business organization comes into direct and frequent contact in the course of its functioningThese individuals and groups are known as STAKEHOLDERS because they have a stake (financial interest ) in the working and performance of the particular business List of external forces (stakeholders)Customers competitors investors suppliersmiddlemen (marketing intermediaries)financers publics

customers

suppliersfinancers

For an example a company can alter its organization structure policies programmes employees physical facilities and marketing mix to suit the changes in the environment

4 What do you mean by external factors in micro environment

AnswerExternal factors refer to those individual and groups and agencies with which a particular business organization comes into direct and frequent contact in the course of its functioningThese individuals and groups are known as STAKEHOLDERS because they have a stake (financial interest) in the working and performance of the particular business

3Who are stakeholdersSTAKEHOLDERS are individuals and groups who have a stake (financial interest ) in the working and performance of the particular business 4Discuss the internal factors in briefa Corporate CultureThe values beliefs and attitudes of the founders and top management of the company exercise a strong influence on what the cmpaany stands for how it does things and what it considers importantbMission and objectivesThe business philosophy and purpose of a comoany guide it prioritiesbusiness strategiesproduct market scope and development scope

cTop management structurethe composition of board of directors the degree of professionalization of management and the organizational structure of a company have important bearing on its business decisions

dPower structureThe internal power relationship between the board of directors and the chief executive is an important factor

e Company image and brand equityThe image and brand equity of the company play a significant role in raising finance forming alliance choosing dealers and suppliers launching new products entering foreign markets

5 What is Macro environmentAnswerMacro environment refers to the general

competitors

middlemen

publics

Fig STAKEHOLDERS OF A COMPANY

Apart from micro environment the other main dimension of business environment isMacro environment Macro environment refers to the general environment or remote environment within which a business firm and forces in its micro environment operateA company does not directly or regularly interact with the micro environmentTherefore macro environment is also known as indirect action EnvironmentThe macro environment forces are less controllable than the micro forces

Macro environment consists of the following components

POLITICAL AND LEGAL ENVIRONMENT

ECONOMIC SOCIAL AND ENVIRONMENT

CULTURAL

ENVIRONMENT

TECHNOLOGICAL ENVIRONMENT

Fig COMPONENTS OF MACRO ENVIRONMENT

environment or remote environment within which a business firm and forces in its micro environment operateA company does not directly or regularly interact with the micro environmentTherefore macro environment is also known as indirect action EnvironmentThe macro environment forces are less controllable than the micro forces 6 What are the components of macro environmenta Political and legal environmentb Economic environmentc Social and cultural environmentd Technological environment

Computer Science

Logic gates

Digital systems are said to be constructed by using logic gates These gates are the AND OR NOT NAND NOR EXOR and EXNOR

BUSINESS FIRM

gates The basic operations are described below with the aid of truth tables

AND gate

The AND gate is an electronic circuit that gives a high output (1) only if all its inputs are high A dot () is used to show the AND operation ie AB Bear in mind that this dot is sometimes omitted ie ABOR gate

The OR gate is an electronic circuit that gives a high output (1) if one or more of its inputs are high A plus (+) is used to show the OR operationNOT gate

The NOT gate is an electronic circuit that produces an inverted version of the input at its output It is also known as an inverter If the input variable is A the inverted output is known as NOT A This is also shown as A or A with a bar over the top as shown at the outputs The diagrams below show two ways that the NAND logic gate can be configured to produce a NOT gate It can also be done using NOR logic gates in the same way

NAND gate

This is a NOT-AND gate which is equal to an AND gate followed by a NOT gate The outputs of all NAND gates are high if any of the inputs are low The symbol is an AND gate with a small circle on the output The small circle represents inversion

NOR gate

This is a NOT-OR gate which is equal to an OR gate followed by a NOT gate The outputs of all NOR gates are low if any of the inputs are highThe symbol is an OR gate with a small circle on the output The small circle represents inversion

EXOR gate

The Exclusive-OR gate is a circuit which will give a high output if either but not both of its two inputs are high An encircled plus sign ( ) is used to show the EOR operation

EXNOR gate

The Exclusive-NOR gate circuit does the opposite to the EOR gate It will give a low output if either but not both of its two inputs are high The symbol is an EXOR gate with a small circle on the output The small circle represents inversion The NAND and NOR gates are called universal functions since with either one the AND and OR functions and NOT can be generated

Note A function in sum of products form can be implemented using NAND gates by replacing all AND and OR gates by NAND gates A function in product of sums form can be implemented using NOR gates by replacing all AND and OR gates by NOR gates

Logic gate symbols

Table 2 is a summary truth table of the inputoutput combinations for the NOT gate together with all possible inputoutput combinations for the other gate functions Also note that a truth table with n inputs has 2n rows You can compare the outputs of different gates

Logic gates representation using the Truth table

Example

A NAND gate can be used as a NOT gate using either of the following wiring configurations

Subject Eng Literature (The Tempest ndash William Shakespeare) Topic Act III Scene 3 Lines 53 to 110 (End of the scene) Date 16th April 2020 (2nd Period)

[Students should read the original play and also the paraphrase given in the school prescribed textbook]Summary Questions amp Answers

o Seeing this strange scene all are inclined to believe the tales told by travelers that there truly are ldquounicornsrdquo and ldquothe phoenixrsquo thronerdquo

o As they are about to sit down to the feast the banquet is snatched away by a harpy (Ariel disguised) A spiritrsquos voice (Arielrsquos voice) denounces Alonso Sebastian and Antonio with particular

1 ARIEL You are three men of sin whom Destiny

(Line 53-58)That hath to instrument this

lower world And what is int the never-surfeited sea

Hath caused to belch up you and on this island

Where man doth not inhabit you rsquomongst men

Being most unfit to live I have made you mad

reference to their crime in expelling Prospero from Milan They have not received any punishment for their deed earlier but the time for their punishment has arrived Upon Alonso it pronounces ldquolingering perdition worse than deathrdquo from which there is no remedy except through sincere repentance Ariel then vanishes in thunder and the shapes enter again and carry away the table

o Prospero watching invisibly is very pleased with the performance of Ariel and his (Prosperorsquos) ldquomeaner ministersrdquo All his enemies are now in his power and are in a fit of desperation He then leaves them and goes to see how Ferdinand and Miranda are getting on

o Alonso is now much humbled and penitent with the after effect of the spiritrsquos denunciation of his crimes He believes that his son is lost forever After this all disperse being stricken mad by the speech of the spirit

o Gonzalo fearing that they may do violence to themselves or to one another follows them and bid others to follow

(a) To whom does Ariel disguised as a harpy call the three sinners What game did Fate of Destiny play with

them

The three sinners called by Ariel are Alonso Sebastian and Antonio It was Destiny which had caused the ocean to cast the three sinners on the shore Though the ocean is all the time devouring whatever appears on its surface and is never satisfied with its continual swallowing of the ships and men in the present case the ocean had cast these three sinners on the shore without killing them

(b) Who had jointly been responsible for the conspiracy against Prospero What is Prosperorsquos purpose behind all this

Three men Alonso Sebastian and Antonio had jointly

been responsible for the conspiracy against Prospero They had driven out Prospero form Milan Prosperorsquos purpose is to make these three sinners realize the wrong they had done He wants them to repent for their criminal deeds because repentance leads to self-esteem(c )What does Ariel (the harpy) tell Alonso and his companions when they take out their swords to attack him

Seeing them drawing their swords Ariel (harpy) tells them that he and his companions are the instruments of destiny and that it is not possible for human beings to do them any injury He says that the swords of human beings can not injure even a minute part of his feathers Their swords are as ineffective against him and his companions as against the wind or the water

(d) Give the explanatory meanings of the following expressions in the context of the above extract

(i)Never surfeited (ii) Belch up (iii) lsquomongst men

(i) Never surfeited never led to satisfaction

(ii) Belch up cast ashore(iii) lsquomongst men in human

society2

I and my fellows (Line 60-65)

Are ministers of Fate The elementsOf whom your swords are tempered may as wellWound the loud winds or with bemocked-at stabsKill the still-closing waters as diminishOne dowl thats in my plume

IMPORTANT PASSAGES EXPLAINED

The elements

(Line 61-66)Of whom your swords are tempered may

as wellWound the loud winds or with

bemocked-at stabs

(a) Who is lsquoIrsquo Who are his lsquofellowsrdquo

lsquoIrsquo is referred to Ariel in disguise of a harpy His lsquofellowsrsquo are other spirits serving Prospero the real Duke of Milan who has acquired supernatural powers after being banished from his Dukedom Prospero has settled in this uninhabited island

(b) What are the elements that have temperrsquod the swords Why will it not work against the speaker

The swords (of Alonso and his companions) are tempered by metal (steel) which is taken out of the earth and refined by

Kill the still-closing waters as diminishOne dowl thats in my plume My fellow

ministersAre like invulnerable

In these words Ariel reminds the King and his companions of the utter futility of drawing swords against himself and his fellows Ariel drives Alonso Antonio and Sebastian the three men of sin to desperation ndash a state in which men do violence to themselves They draw swords to strike Ariel But Ariel reminds them that he and the other spirits are the ministers of destiny and nothing can wound them The steel of which their swords are made of may cut the wind or water which being divided always closes up again Even supposing that such things may be possible it is quite impossible that their swords will cut one feather in their plume They are incapable of being wounded by any sword of man Hence it is foolish on their part to attempt to strike at Ariel and his fellow-spirits

For which foul deed

(Line 72-75)The powers delaying not forgetting

haveIncensed the seas and shores yea all the

creatures Against your peace

Ariel enters like a harpy and remaining invisible tells Alonso Sebastian and Antonio that he and other harpies are the agents of Destiny appointed to carry out her decrees He tells them that their punishment for the crime against Prospero which has been so long deferred is now to fall upon them He reminds them that they had expelled Prospero from Milan and set him and his innocent child adrift on the sea and that the sea had paid them back for their sin by the shipwreck and by the calamities they have suffered He tells them that the powers above which did not forget this mean treachery but only deferred the punishment have now engaged the seas and the shores and all living beings including him and his comrades against them The very elements and supernatural agency Ariel adds have taken up the avenging of their crime against Prospero

the action of fire It may cut the wind or water which being divided always closes up again

The sword will not work against the spirits and the harpy because they are the ministers of destiny and nothing can wound them nor it will cut a single feather in their plume

(c )What is the meaning of lsquodowlrsquo in the last line

The term lsquodowlrsquo means a filament or the smallest part of a feather In this context Ariel in disguise of harpy says that their sword cannot even damage the smallest filament of their (Arielrsquos and other spirits) feathers as they are incapable of being wounded by any sword of man

(d) What does the speaker remind the listeners about

Ariel in disguise of harpy reminds Alonso the King of Naples Sebastian Alonsorsquos brother and Antonio the present Duke of Milan and the treacherous brother of Prospero as they being three men of sin He even reminds them that their punishment for their crime against Prospero which has been so long deferred now falls upon them He reminds them that they have expelled Prospero from Milan and has set him along with his innocent infant daughter adrift on the sea So the sea has paid them back for their sin by their shipwreck and the calamities they have suffered since then The harpy rebukes Alonso of his sin that has incensed the Gods and has deprived him of his son as a punishment

(e) How do they respond

When Ariel in disguise of a harpy reminds Alonso Sebastian and Antonio of their past misdeeds and sin Alonso has a look of terror and confusion in his eyes He utters the words of sincere repentance wrung out of his conscience-stricken heart It appears to him that all the elements of nature the sea-waves the wind and the thunder proclaiming a loud voice in the name of Prospero and the crime Alonso has committed against him They are calling upon him to repent There is a deep storm raging in Alonsorsquos breast and the echoes of that storm are ringing in his ears like a clear note of wind-instrument A note of denunciation of Alonsorsquos crime leaves him much humbled and penitent and confirms his belief that his son is lost forever But Sebastian and Antonio shows some courage instead of repentance They wish to kill the spirits or devils if it appears

3

Of my instruction hast thou nothing bated (Line 85-93)

In what thou hast to say So with good life

And observation strange my meaner ministers

Their several kinds have done My high charms work

And these mine enemies are all knit upIn their distractions They now are in my

powerAnd in these fits I leave them while I visitYoung Ferdinand whom they suppose is

drownedAnd his and mine loved darling

Methought the billows spoke and (Line 96-99)

told me of itThe winds did sing it to me and the

thunderThat deep and dreadful organ-pipe

pronouncedThe name of Prosper It did bass my

trespass

These are the words of contrition coming from Alonso Ariel has driven him to a deep repentance for conspiring with Antonio against Prospero He now feels a sincere remorse It appears to him that all the elements of nature the sea-waves the wind and the thunder proclaimed with a loud voice the name of Prospero and the crime Alonso had committed against him They are calling upon him to repent There is a deep storm raging in Alonsorsquos breast and the echoes of that storm are ringing in his ears like the clear note of a wind-instrument

Comment These are the words of sincere repentance wrung out of the conscience-stricken heart of Alonso Alonso who is the lesser villain is the first to give way to remorse under the effect of Arielrsquos speech The words of Ariel seem to him to be the voice of conscience speaking to him He is driven to desperation a state in which he might do violence to his life

(a) Identify the speaker State the context

Prospero the ruler of the island is the speaker The famous banquet scene has been enacted very well Ariel and his junior spirits have played their roles excellently Prospero is glad to say words of praise for them(b) In what way the speakerrsquos instructions have been carried out

According to Prosperorsquos instructions a banquet was presented before the King of Naples and his companions when they were tired and hungry Just when they were preparing to eat the feast the banquet was suddenly removed by exercising supernatural powers All this was done by Ariel Prosperorsquos chief assistant and a powerful spirit

Ariel not only made the feast disappear but also delivered his speech blaming the King and his two companions for their past wicked deeds He warned them to repent for their misdeeds or suffer forever on that uninhabited island

(c) Who are referred to as lsquomeaner ministersrsquo What have they done

Prospero refers as lsquomeaner ministersrsquo to his other lesser spirits who were assisting Ariel in presenting a scene before the kingrsquos party They entered the scene to the accompaniment of music They assumed several strange shapes and brought in a banquet Then they danced about it with gentle actions of salutations thus inviting the King and others to eat the feast

These spirits play their role again when Ariel in the shape of a harpy quits the scene These shapes enter again and dancing with mocking gestures carry away the table

(d) Who are the speakerrsquos enemies What has happened to them

King of Naples Alonso his brother Sebastian and the present Duke of Milan Antonio (Prosperorsquos own brother) are Prosperorsquos enemies With the turn of events they have all been washed ashore on the island which is ruled by Prospero the great magician Actually this happened after the shipwreck caused by a storm which was raised by Prospero with the purpose of bringing these people to his island Prosperorsquos spirits have already confused and terrified these enemies and they are under Prosperorsquos control He can treat them as he likes

(e) What does he say about Ferdinand Explain what is meant by ldquohellip his and mine darlingrdquo

Prospero knows that Alonsorsquos son prince Ferdinand is alive though his father thinks that the prince has been drowned

Prospero refers to his daughter Miranda who is dear to him She is also very dear to Prince Ferdinand who has fallen in love with her They are waiting to be married soon for which they have received Prosperorsquos consent

4

ALONSO O it is monstrous monstrous (Line 95-102)

Methought the billows spoke and told me of it

The winds did sing it to me and the thunderThat deep and dreadful organ-

pipe pronouncedThe name of Prosper It did bass

my trespassTherefore my son ithrsquo ooze is

bedded andIll seek him deeper than eer

plummet soundedAnd with him there lie mudded

(a) In what way does Alonso express his horror when his conscience is awakened by Arielrsquos words

When Alonsorsquos conscience is awakened by Arielrsquos words he expresses his horror at what he has heard He gets the feeling that the waves of the ocean the wind and the loud thunder have spoken to him and uttered the name of Prospero Because of being reminded of his crime in a very loud and rough voice he comes to realize that he has lost his son for his past misdeeds

(b) What does Alonso imagine about his son What does Alonso want to do in his desperate state

Alonso imagines that his son is lying in the mud at the bottom of the sea He feels desperate that he wants to drown himself in the ocean deeper than the plumb-line has ever gone He wants to lie with his son at the bottom of the sea

(c) How do Sebastian and Antonio want to face the evil spirits

Sebastian says that he is not at all afraid of what the harpy has said and that he is prepared to fight any number of such monsters if they appear before him only one at a time Antonio says that he would support Sebastian in the fight against the fiendsyyy

(d) Why does Gonzalo ask Adrian to follow the three men

Gonzalo tells Adrian that all the three men namely Alonso Sebastian and Antonio are in a wild and reckless mood The thought of the heinous crime of which they are guilty has begun to torment their minds So he asks Adrian to follow those three men without loss of time and prevent them from doing anything which the turmoil in their minds might lead them to do

(e) What opinion do you form of Alonso from the above extract

Alonso who is the lesser villain is the first to give way to remorse under the effect of Arielrsquos speech The words of Ariel seem to him to be the voice of conscience speaking to him He is driven to desperation a state in which he might do violence to his life

Subject =Accounts

Ac-12 15420 topic-pL Appropriation ac

PROFIT AND LOSS APPROPRIATION ACCOUNT

MEANING AND PREPARATIONProfit and Loss Appropriation Account is merely an extension of the Profit and Loss Account of the firm The profit of the firm has to be distributed amongst the partners in their respective profit sharing ratio But before its distribution it needs to be adjusted All Adjustments like partnerrsquos salary partnerrsquos commission interest on capital interest on drawings etc are made in this account These adjustments will reduce the amount of profit for distribution This adjusted profit will be distributed amongst the partners in their profit sharing ratio To prepare it at first the balance of Profit and Loss Account is transferred to this account The journal entries for the preparation of Profit and Loss Appropriation Account are given below

1 for transfer of the balance of Profit and Loss Account to Profit and Loss Appropriation Account

(a) In case of Net Profit

Profit and Loss Ac helliphelliphelliphelliphellipDrTo Profit and Loss Appropriation Ac(Net Profit transferred to Profit and Loss Appropriation Ac)

(b)In case of Net Loss

Profit and Loss Appropriation Achelliphelliphellip DrTo Profit and Loss Ac(Net Loss transferred to Profit and Loss Appropriation Ac)

2 for Interest on Capital

For transferring on Interest on CapitalProfit and Loss Appropriation Achelliphelliphellip DrTo Interest on Capital Ac(Interest on capital transferred to Profit amp Loss Appropriation Ac)

3 for Interest on Drawings

For transferring Interest on Drawings Interest on Drawings Achelliphelliphelliphelliphelliphellip DrTo Profit and Loss Appropriation Ac(Interest on drawing transferred to Profit amp Loss Appropriation Ac)

4 For Partnerrsquos SalaryFor transfer of partnerrsquos SalaryProfit and Loss Appropriation Achelliphellip DrTo Salary Ac(Salary transferred to profit amp Loss Appropriation Ac)

5 For Partnerrsquos CommissionFor transferring commissionProfit and Loss Appropriation Achelliphelliphellip DrTo Commission Ac(Commission transferred to Profit and Loss Appropriation Ac)

6 For Transfer of agreed amount to General ReserveProfit and Loss Appropriation Ac helliphellipDrTo General Reserve Ac(Transfer to General Reserve)

7 for share of Profit or Loss appropriation(a) If ProfitProfit and Loss Appropriation Achelliphellip DrTo Partnerrsquos CapitalCurrent Ac(Profit transferred to capitalcurrent Ac)(b) If LossPartnerrsquos Capital Current Achelliphelliphelliphellip DrTo Profit and Loss Appropriation Ac(Loss transferred to capitalcurrent Ac)

THE FORMAT OF PROFIT AND LOSS APPROPRIATION

Profit and Loss Appropriation Account for the year endedhelliphelliphelliphellip

Particulars Amount Particulars Amount

To PL Ac (loss) By pL Ac (profit)

To Interest on capital BY Interest on drawings

To partner`s commission by Partner`s capital Ac ( loss)

To Partner`s salary To Interest on partner`s loan To General Reserve To Partner`s Capital AC (Profit)

Subject= Economics

MOVEMENT ALONG THE DEMAND CURVE (CHANGE IN QUANTITY DEMANDED)In law of demand you have already studied the inverse relationship between price and quantity demanded When quantity demanded of a commodity changes due to change in its price keeping other factors constant it is called change in quantity demanded It is graphically expressed as a movement along the same demand curve There can be either a downward movement or an upward movement along the same demand curve Upward movement along the same demand curve is called contraction of demand or decrease in quantity demanded and downward movement along the same demand curve is known as expansion of demand or increase in quantity demanded

Extention of demandd

price (rs)p A

B Extentionp1 d

Q Q1

Quantity demanded ( in units)

Contraction of demandd

p2 Ccontraction

p APrice (Rs)

d

Q2 Q

Quantity demanded (in units)

Explanation of movement of demand A fall in price from OP to OP1 leads to increase in quantity demanded from OQ to OQ1 (expansion of demand) resulting in a downward movement from point A to point B along the same demand curve DD When Price rises from OP to OP2 quantity demanded falls from OQ to OQ2 (contraction of demand) leading to an upward movement from point A to point C along the same demand curve DD

  • Activity Series of Metals
    • Drawbacks of Rutherfordrsquos model of atom
      • Electromagnetic radiations
      • Properties of electromagnetic radiations
      • Characteristics of electromagnetic radiations
        • Plancks Quantum Theory-
        • Photoelectric effect
          • Intext Questions
            • Logic gates
            • Digital systems are said to be constructed by using logic gates These gates are the AND OR NOT NAND NOR EXOR and EXNOR gates The basic operations are described below with the aid of truth tables
            • AND gate
            • Example
Page 6:  · Web viewSubject. Topic. Summary. Execution. English 1 . Chapter 1 naming words . Page 8. Write the names of these pictures:- Person:-1. father. 2.Firefighter 3.doctor 4 ...

Animals Animals are divided into different groups according to their habitats Some animals live on land some animals live both on land and in water some animals adapted themselves for food and some for protection They develop special features that help them to survive in their habitats Animals that live on land are called Terrestrial animals such as lions tigers Yaks polar bears Polar Bears live cold polar regions Yaks live in mountains These animals have thick fur on their body and a thick layer of fat under their skin which keeps them warm

Solved exercise question and answersA Write the correct answer1Terrestrial animals live here - on land 2This animal lives in polar regions ndash polar bear 3 These help a fish to breathe in water - gills4 This helps carnivorous birds to tear flesh - a hooked beak

MATHEMATICS

Ch 6Multiplication

Exercise 17 Example 2

Multiply 224835 by 4 Solution

2 2 4 8 3 5 times 48 9 9 3 4 0

Exercise

18 3 1 1 4 6 7 19 2 1 2 7 8 5 times 3 times 4 9 3 4 4 0 1 8 5 1 1 4 0

20 2 0 6 1 3 times 9 1 8 5 5 1 7

Class VSubject Topic Summary Execution

Science Chapter 2 - The Skeletal System

Posture

The way in which we hold our body while sitting standing walking and lying down is known as posture

Importance of correct posture and exercise

1 Correct posture and exercise makes our bones and muscles strong 2 It provides our body more energy to do works

3 It keeps our body stressless 4 It also prevent back ache and muscle pain

Some important yoga asanas are-

1 Veerabhadrasana ( Warrior pose )

2 Dhanurasana ( Bow pose )

3 Trikonasana ( Triangle pose )

D Answer these questions

5How do our muscle work

Ans ndash Our muscles work by contracting and relaxing

6Why should we maintain a correct posture

Ans ndash We should maintain a correct posture because ndash

i Correct posture and exercise makes our bones and muscles strong

ii It provides our body more energy to do works

iii It keeps our

body stressles

s iv It also prevent

back ache and muscle pain

Social studies Conquering distances

Put a tick on the correct option1 b Ship 2b water transport 3 b trains 4 c 13 5 b Wright brothersTrue or false1 False 2 True 3 True 4 True

MATHEMATICS

Ch 3Addition and Subtraction

Exercise 126 A school needs Rs 4987653 for its building It has only Rs 3592468 in its accounts Estimate the money by rounding off to the nearest lakh it has still to raise

Solution Estimated amount need Rs 5000000 The school has only Rs 3600000 Still to raise Rs 1400000

MATHEMATICS

Ch 4

Mul

tiplic

ation

and

Div

ision

We have learnt multiplication tables up to 15 Let us extend the tables up to 20X 11 12 13 14 15 16 17 18 19 201 11 12 13 14 15 16 17 18 19 202 22 24 26 28 30 32 34 36 38 403 33 36 39 42 45 48 51 54 57 604 44 48 52 56 60 64 68 72 76 805 55 60 65 70 75 80 85 90 95 1006 66 72 78 84 90 96 102 108 114 1207 77 84 91 98 105 112 119 126 133 1408 88 96 104 112 120 128 136 144 152 1609 99 108 117 126 135 144 153 162 171 180

10 110 120 130 140 150 160 170 180 190 200Properties of Multiplication1 The product of two numbers does not change when the order of numbers is changed eg 503times23 = 23times503 [This property is called Commutative Property of multiplication]

2 The product of three numbers does not change when the grouping of numbers is change eg (15times18iquesttimes10=15 times(18times 10) = (15times10iquesttimes18 [This property is called Associative Property of multiplication]

3 The product of a number and 1 is the number itself eg 1513 times 1 = 1513 [This property is called Identity Property of multiplication and the integer 1 is called Identity Element of multiplication]

4 The product of a number and 0 is 0 eg 718205times 0 = 0 times 718205 = 05 The product of a number by the sum of two numbers is equal to the sum of the

products of that number by the two numbers separately eg123 times (105+ 48) = 123 times 105 + 123 times 48 [ This property is called Distributive Property of multiplication over

addition]English language

Transitive and intransitive verb

Pick out the verbs from the following sentences and say whether they are transitive or intransitive verb6 Was fullndash verb Intransitive verb7 Have been decorated- verb Transitive verb8 Happy cheerful ndash verb Intransitive verb9 Shall come back ndash verb Transitive verb

English 2 The fall of Lanka

This is the story of the fight between Rama and Ravana as told by Valmiki in lsquoThe Ramayana The monkeys worked all day and all night and at last built a bridge so that Rama and

Write the synonyms of

1 Stationed-

his army could cross to Lanka and rescue Sita assign2 Invade- enter

a country or a region so as to subjugate or occupy it

3 Prowess- bravery in battle

4 Haughty- arrogantly

5 Puny- small and weak

6 Spy- secret agent

7 Dreadful- causing or involving great suffering

8 Violate- disobey

9 Ghastly- causing great horror or fear

10 Deception ndash misleading

Class VISubject Topic Summary Execution

HISTORY AND CIVICS

CHAPTER 3

MAHAVIRA AND BUDDHA ndash GREAT PREACHERS

BUDDHA

Impact of Buddhism on Indian Cultures

Impact on Religion

Buddharsquos practical and simple doctrines made their impact on HinduismThe principle of ahimsaIt brought about a great change in the performance of costly yanjnas and sacrifices which previously involved immense loss of life The Mahayana Buddhists adopted the practice of worshipping Buddha and bodhisattvas making idols and erecting temples in their honour

Impact on Literature

After the death of Buddha his teachings were compiled and called TripitakasThe Jatakas contain tales dealing with the previous births of Buddha

Impact on Education

The Buddhist monasteries became great centres of learning These centres of learning developed into famous universities- Nalanda Taxila Vikramshila etc

Impact on Art and Architecture

The gateways and railing of the Sanchi Stupa were covered with sculptured figuresCave- temples were also constructed which were decorated with beautiful frescoes

1) How many parts of Tripitakas are thereAns -There are three parts of Tripitakas- Sutta Pitaka Vinaya PitakaAbhidhamma Pitaka

2)What are the subjects taught in these monasteries Ans ndash Buddhist scriptures logic Philosophy medicine astronomy etc

3) Which art was developed under Buddhist patronage Ans ndash Gandhara art

The Gandhara art was developed under Buddhist patronage

BIOLOGY The Leaf Photosynthesis The process by which green plants make their own food from carbon dioxide and water in the presence of sunlight and chlorophyll is called photosynthesis

All green plants need the following to make their food ndash

water carbon dioxide chlorophyll and energy in the form of sunlight

Carbon dioxide + water ------------- Glucose + oxygen

The end product of photosynthesis is glucose

Fill in the blanks

1 Plants make their food by the process of photosynthesis

2 The inner wall of the guard cell is thicker than the outer wall

3 The extra glucose is converted into starch and sucrose

4 The leaf is boiled in alcohol to remove chlorophyll

5 The rate of transpiration is more on the hot day then a cold day

6 Photosynthesis helps to observe water and minerals from the soil

English 1 Pronouns Kinds of pronouns 1 Personal pronouns2 Possessive pronouns3 Reflexive pronouns4 Interrogative pronouns5 Relative pronouns6 Demonstrative pronouns7 Indefinite pronouns

Personal pronouns they refer to first second and third person in sentences First person- the speakerSecond person-the listenerThird person-the objectperson being spoken aboutPersonal pronouns should have the same gender and number as the nouns they refer to

Possessive pronouns these are used to indicate the relationship between the objects and people These pronouns include mine ours yours his hersand theirs

ExerciseBFill in the blanks with suitable pronounsThere was much excitement among the childrenTheywere eagerly looking forward to the annual picniclsquoAre they going to Lodhi Gardens toorsquo wondered AneeshlsquoNo they are going to Buddh Jayanti Park with Mrs Jain said Mrs ChopraThe children looked disappointedlsquoWonrsquot you be taking us Marsquoam rsquo they askedlsquoSorry children I have to go to Mumbai for a week to look after my sick mother But you will have fun with Mrs Jain she is full of laughter and you will love being with her the whole daylsquoIt will not be the same they grumbled

English 2 The great train journey- Ruskin Bond

The great journey by Ruskin Bond is a story about Suraj who loved trains and wanted to go to places One day while wandering along the railway tracks he enters into a carriage compartment The train suddenly starts moving with him in the compartment and after a journey returns back to the same place from where it had begun The story is about his experience during that journey

State true or false1 When the train had passed leaving behind the

hot empty track Suraj was lonely2 It was winter holidays

and Suraj did not know what to do with himself

3 He plunged his hands into the straw and pulled out an apple

4 A dirty bearded face was looking out at him from behind a pile of crates

5 Suraj wanted to go to Japan

Hindi 2nd lang

गललबाजलडका खालीसथानोकोभरो-6 गो-ामसनिनकलकरहमगराजमआगए7 माबोधराजकोराकषससमझतीथी8 चीलरोशन-ानमसअ-रआकरतहसीलपरबठगई9 तीनचारतीनकऔररईकगोलउडलनिकनघोसलानहीनिगरा10 वहसवयतोघोसलातोडनककतिलएगललउठालायाथा

11 -ीवारकसाथलगतगोहपजोकसहार-ीवारपकडलतीह12 बोधराजअभीभीटकटकीबाधचीलकीओर-खरहाथा13 बोधराजअपनीजबमबहतसाचगगाभरकरलायाथा14 मरनिपताजीकीतरककीहईऔरहमलोगएकबडघरमजाकररहनलग15 बागमजातातोफलपरबठीनिततलीको-खनिततलीकोपकडकरउगकतिलयोकबीचमसल-ता

BENGALI(2ND LANGUAGE)

সহিনধসবরপওসবরসহিনধ

সবরসহিনধরহিনয়ম- ৯ই-কারহিকংাঈ-কাচেররপচেরইাঈহিভননঅনযসবররণCথাকচেইাঈসথাদেনয-ফায়এংওইয- ফাপCচেরণCযকতয়

১০উ-কারহিকংাঊ-কাচেররপচেরইাঈহিভননঅনযসবররণCথাকচেউাঊসথাদেন-ফায়এংওই- ফাপCচেরণCযকতয়

১১ঋ- কাচেররপচেরঋহিভননঅনযসবররণCথাকচেঋসথাচেনর -ফায়এংওইর পCচেরণCযকতয়

১২সবররণCপচেরথাকচেপCতMএ-সথাচেনঅয় ঐ- সথাচেনআয় ও- সথাচেনঅএংঔ- সথাচেনঅায়

৯ই+ অ= য- ফাআহি+ অনত= আযনত অহিধ+ অয়ন= অধযয়নই+ আ=য- ফা+ াইহিত+ আহি= ইতযাহি পরহিত+ আতC ন= পরতযাতC নই+ উ=য- ফা+ উঅহিত+ উহিকত= অতযহিকত হি+ উৎপহিতত= যৎপহিততই+ ঊ= য- ফা+ ঊ ই+ এ= য- ফা+ এঈ+ অ= য- ফা পরহিত+ ঊ4= পরতয4 পরহিত+ এক= পরচেতযকঈ+ অ আ= য- ফা+ অ আনী+ অমব= নযমব মসী+ আধার= মসযাধার

১০উ+ অ= অন+ অয়= অনবয় পশ+ অধম= পশবধমউ+ আ= াস+ আগত= সবাগত পশ+ আহি= পশবাহিউ+ ঈ= হিঅন+ ইত= অহিনবতউ+ এ= দেঅন+ এ4রণ= অচেনব4রণউ+ ঈ= ীসাধ+ ঈ= সবাধবী তন+ ঈ= তনবী

১১ঋ+ অ= র মাত+ অনমহিত= মাতরনমহিতঋ+ আ= রা হিপত+ আয়= হিপতরায়ঋ+ ই= হির মাত+ ইচছা= মাতচছাঋ+ ঈ= রী ধাত+ ঈ= ধাতরীঋ+ উ= র ভরাত+ উপচে(= ভরাতরপচে(

১২এ+ অ= অয় দেন+ অন= নয়নঐ+ অ= আয় গৈগ+ অক= গায়কও+ অ= অ দেপা+ অন= পনও+ ই= অ দেপা+ ইতর= পহিতরও+ এ= অ দেগা+ এ4রণা= গচে4রণাঔ+ অ= অা দেপৌ+ অক= পাকঔ+ ই= অা দেনৌ+ ইক= নাহিকঔ+ উ= অা দেভৌ+ উক= ভাক

MATHS Topic NumbersChapter Natural numbers and whole numbers

Study item properties of whole numbers for multiplication

1 Closure property If x and y are two whole numbers then xtimesy is also a whole numberExample If x = 9 and y =3 then xtimesy = 9times3 = 27 which is a whole number

2 Commutative property If x and y are two whole numbers then xtimesy = ytimesxExample If x = 5 and y = 2 then xtimesy = 5times2 = 10y times x = 2times5 = 10Therefore 5times2 = 2times5

3 Associative property If x y and z are three whole numbers then x times(ytimesz) = (xtimesy) times zExample If x =3 y = 5 and z = 7 then 3 times (5times7) = 3 times (35) = 105And (3times5) times7 = (15) times 7 = 105Therefore x times (ytimesz) = (xtimesy) timesz

4 Distributive property If x y and z are three whole numbers then xtimes (y + z) = x times y + x times z

Therefore the multiplication of whole numbers is distributive over their additionExample If x = 5 y = 3 and z= 2Therefore x times (y + z) = 5 times (3 + 2) = 5times5 =25And x times y + xtimes z = 5times3 +5times2 =15 +10 = 25Again x times (y ndash z ) = x times y ndash x timesz Therefore 5 times ( 3 - 2) = 5 times1 = 5 and 5times3 ndash 5 times2 = 15 ndash 10 = 5Therefore the multiplication of whole numbers is also distributive over their subtraction if y is greater than z

5 Existence of identity If x is a whole number then

X times1 = x 1 times x = xTherefore we can write x times1 = 1 times xTherefore the multiplication of any whole number with 1 is the number itselfTherefore we can say that 1 is multiplicative identity or identity element for multiplicationExample 5 times1 = 5 1 times 5 = 5 Therefore 5 times 1 = 5

6 Multiplicative inverse If x is any whole number ( x is not equal to zero ) then its multiplicative inverse will be 1xSo x times 1x = 1 but 1x is a whole number if x = 1For other values of whole number 1x is not a whole number therefore we can write its multiplicative inverse does not exists

7 Cancellation law of multiplication If x y and z are three non- zero whole numbers then x times y = x times z

Or y = zExample 9 times y = 9 timeszTherefore y = z

Class VIISubject Topic Summary Execution

English 2 Sentences based on meanings

Kinds of sentences

Assertive or declarative to convey information or simply make a statement

Interrogative to ask different types of questions

Imperative to command or instruct someone or make a request

Exclamatory to express strong feelings and emotions

Exercise c1 What a nice compliment that is

That is a nice compliment2 How well- behaved the children

areThe children are very well-behaved

3 What great chefs we areWe are great chefs

4 What a shame it isIt is a shame

5 What a fantastic idea you haveYou have a fantastic idea

Homework 6 -10English

LiteratureThe Listeners III) Answer the following questions-

d) Identify two words used in the poem to give the poem an eerie atmosphereAns- Two words used to give the poem an eerie atmosphere are ldquogreyrdquo and ldquophantomrdquo

e) Who do you think are the inmates of the houseAns- I think the inmates of the house are phantom who once used to dwell in it

f) Why was the poet ldquoperplexed and stillrdquoAns- He was lsquo perplexed and stillrsquo because he was expecting an answer from the inmates of the house But despite of repeated calls there was no response

CHEMISTRY Chapter 2 ndashElement and Compound

Activity Series of MetalsThe activity series is a chart of metals listed in order of declining relative reactivity The top metals are more reactive than the metals on the bottomMetal SymbolReactivity

Lithium Li displaces H2 gas from water steam and acids and forms hydroxides

Potassium K

Strontium Sr

Calcium Ca

Sodium Na

Magnesium Mg displaces H2 gas from steam and acids and forms hydroxides

Aluminum Al

Zinc Zn

Chromium Cr

Iron Fe displaces H2 gas from acids only and forms hydroxides

Cadmium Cd

Cobalt Co

Nickel Ni

Tin Sn

Lead Pb

Hydrogen gas

H2 included for comparison

Antimony Sb combines with O2 to form oxides and cannot displace H2

Arsenic As

Bismuth Bi

Copper Cu

Mercury Hg found free in nature oxides decompose with heating

Silver Ag

Palladium Pd

Platinum Pt

Gold Au

Answer the following Q)Difference Between Metals And Nonmetals

Metals Nonmetals

These are solids at room temperature except mercury

These exist in all three states

These are very hard except sodium

These are soft except diamond

These are malleable and ductile

These are brittle and can be breakdown into pieces

These are shiny These are non-lustrous except iodine

Electropositive in nature Electronegative in nature

Have high densities Have low densities

Math Number System

Chapter Fraction

Study item Some solved sums from exercise 3(B)1) For each pair given below state whether it from like fractions or unlike

fractions (i) 58 and 78

= Like Fraction because denominators same(ii) 815 and 821

= Unlike Fraction because denominators are not same

(iii) 49 and 94 = Unlike Fraction

2) Convert given fractions into fractions with equal denominators(iii) 45 1720 2340 and 1116Solution Given fraction 45 1720 2340 and 1116Therefore the LCM of 5 20 40 and 16 is 80Therefore 45 = 4times165times16 = 64801720 = 17times420times4 = 68802340 = 23times240times2 = 4680 1116 = 11times516times5 = 5580

3) Convert given fractions into fractions with equal numerators(iii) 1519 2528 911 and 4547Solution Given fractions 1519 2528 911 and 4547Therefore the LCM of 15 25 9 and 45 is 2251519 = 15times1519times15 = 225285 2528 = 25times928times9 = 225252911 = 9times2511times25 = 2252754547 = 45times547times5 = 225235

4) Put the given fractions in ascending order by making denominators equal

(iii) 57 38 914 and 2021Solution Given fraction 57 38 914 and 2021Therefore the LCM of the denominators is 16857 = 5times247times24 = 12016838 = 3times218times21 = 63168914 = 9times1214times12 = 1081682021 = 20times821times8 = 160168Therefore ascending order 63168lt108168lt120168lt160168Therefore ascending order of given fractions38lt914lt57lt2021

COMPUTER CHAPTER-1COMPUTER FUNDAMENTALS

DONE IN THE PREVIOUS CLASSES PAGE 10CWRITE TRUE AND FALSE

1 True2 False3 False4 False5 True

GEOGRAPHY CHAPTER 7EUROPE

CHAPTER COMPLETE 1)Europe is home to a famous mountain range called the Alps

2)River Rhine originates in Switzerland

3)The Eiffel Tower one of the tallest structures in Europe

4) Vatican City is one of the most densely populated European countries

5)Sognefjordin Norway is the largest fjord in Norway

Class VIIISubject Topic Summary Execution

MATHEMATICS Ch 6Sets

Exercise 6 (D)1 Given A = x x isin N and 3iquest x le 6 and B = x x isin W and xlt4 find (i) Sets A and B in roster form (ii) A cup B (iii)

A cap B(iv) A ndash B (v) B ndashA

Solution (i) A = 456 and B = 0123

(ii) A cup B = 0123456 (iii) A cap B = ϕ (iv) A ndash B = 456 (v)B ndash A = 0123

3 If A = 56789 B = x 3 lt x lt 8 and x isin W and C = x xle5 and x isin N Find (i) A cup B and (A cup B) cup C (ii) B

cup C and A cup ( B cup C)

(iii) A cap B and (A cap B) cap C (iv) B cap C and A cap (B cap C)

Is (A cup B) cup C = A cup (B cup C)

Is (A cap B) cap C = A cap (B cap C)

SolutionA = 56789 B = 4567 C = 12345

there4 (i) A cap B = 456789 and (A cup B) cup C = 123456789

(ii) B cup C = 1234567 and A cup ( B cup C) = 123456789

(iii) A cap B = 567 and (A cap B) cap C = 5

(iv) B cap C = 45 and A cap (B cap C) = 5

Now (A cup B) cup C = 123456789

And A cup ( B cup C) = 123456789 there4 (A cup B) cup C = A cup (B cup C)

Again (A cap B) cap C = 5 and A cap (B cap C) = 5

there4 (A cap B) cap C = A cap (B cap C)

4 Given A = 012345 B = 02468 and C = 0369 Show that (i) A cup (B cup C) = (A cup B) cup C ie the union

of sets is associative (ii) A cap (B cap C) = (A cap B) cap C ie the intersection of sets is associative

SolutionNow B cup C = 0234689 and A cup B = 01234568

there4 A cup (B cup C) = 012345689 and

(A cup B) cup C = 012345689

So (i) A cup (B cup C) = (A cup B) cup C ie the union of sets is associative

Again B cap C = 06 and A cap B = 024

there4 A cap (B cap C) = 0 and (A cap B) cap C = 0

So (ii) A cap (B cap C) = (A cap B) cap C ie the intersection of sets is associative

Physics Chapter 2 Physical Quatites and Measurements

Here We Will Do Some QuestionsRelated To Chapter 2

A density bottle has a marking 25 mL on it It means that

1 the mass of density bottle is 25g

2 the density bottle will store 25 ml of any liquid in it

3 the density bottle will store 25 ml of water but more volume of liquid denser than water

4 the density bottle will store 25 ml of water but more volume of a liquid lighter than water

Solution 2 the density bottle will store 25 ml of any liquid in it

COMPUTER CHAPTER-2Spreadsheet Functions and Charts

SELECTING RANGE IN ROWSCOLUMNSWHEN TWO OR MORE CELLS ARE SELECTED IT IS CALLED A RANGEA RANGE OF CELLS CAN BE FORMED IN TWO WAYS--a) SELECTING RANGE BY USING THE MOUSEb) SELECTING RANGE BY USING THE KEYBOARD

Q1)WRITE THE STEPS TO SELECT PARTIAL RANGE IN A ROW

Ans)THE STEPS ARE-6 SELECT THE ROW7 BRING THE CELL POINTER TO THE DESIRED

LOCATION FROM WHERE YOU WANT TO START YOUR SELECTION

8 CLICK THE LEFT MOUSE BUTTON AND KEEP DRAGGING TO YOUR RIGHT TILL YOU REACH THE LAST CELL TO NE SELECTED

RELEASE THE MOUSE BUTTON

GEOGRAPHY Asia

CLIMATE

Asia experiences great extremes of climate Jacobabad in the Sind province of Pakistan is one of the hottest places in the WorldVerkhoyansk in Siberia is one of the coldest places in the WorldCherrapunji and Mawsynram in India are two wettest places in WorldArabia Tibet Gobi and Mongolia are extremely dry regionsFactors Affecting Climate of Asia-The factors influencing the climate of Asia are-

Factors Affecting Climate of Asia-Thoroughly read the table in page number 60

Latitudinal extent

Continentality

Relief features

Presence of low pressure trough

Jet streams

English Language The Sentence A complex sentence contains one independent clause and at least one dependent clause The dependent clause in a complex sentence is introduced with subordinating conjunctions or relative pronouns

Commonly Used Subordinating Conjunctions-Time after before while when since untilCause And Effect because now since as in order that soOpposition although though even though whereas while in spite ofCondition if unless only if whether or not even if in case(that)

Commonly Used Relative Pronouns-Who whose whom which whoever whomever whichever that

Class IXSubject Topic Summary Execution

1-BENGALI(2ND LANGUAGE)

ldquo বঙগভমিরপরমিrdquo াইকেলধসদনদতত

আচেগর পর উততর পচো-১ ২ ৩ এং নীচের পর টি াহির কা- ৪মহিbকাও গচেনা দেগা পহিচে অমত হরচে- ক) কার দো দেকান কহিতার অং( ) কতা দেক পরসঙগ কী উহিকতটির তাৎপযC আচোনা কচেরা৫দেসই ধনয নরকচে দোচেক যাচে নাহি ভচে মচেনর মহিeচের সাচেসচে সCন ক) কহির কায C ার উচেf(য হিক হিছ কহি কন কহিতাটি দেচেন) কহি কার কাচেছ হিমনহিত কচেরচেছনগ) কহি এই পহিথীচেত কাচের ধনয মচেন কচেরনঘ) কহি হিক রকম অমর তাাভ করচেত ান

Hindi 2nd lang

काकी(कतिसयारामशरण गपत)

इस कहानी म लखक न यह बतान का परयास निकया ह निक बचच अपनी मा स निकतना परम करत ह शयाम अबोध बालक ह वह अपनी मा क मरन क बा- उसन अपनी मा क कतिलए बहत रोया बा- म उस पता चला निक उसकी मा राम क घर चली गई ह आकाश म उडती हई पतग -खकर उस हरष हआ निक पतग क दवारा वह अपनी मा को नीच उतारगा इसक कतिलए वह अपनी निपता की जब स -ो बार सवा रपया निनकालकर पतग और -ो मोटी सी मन वाली अपन भाई स काकी एक कागज पर कतिलखवा कर पतग म कतिशव का दि-यानिनकालकर पतग और -ो मोटी सी मन वाली अपन भाई स काकी एक कागज पर कतिलखवा कर पतग म कतिचपका दि-याभोला और शयाम कोठरी म रससी बाधनी रह थ तभी उसक निपता करोध म आकर उन स पछ निक कया उनकी जब स रपया निनकाला हभोला डर क मार बताया निक शयाम इस पतग क दवारा अपनी काकी को राम क यहा स उतारना चाहता हनिवशशवर(शयाम क निपता)न फटी पतग उठाकर -खी तो उस पर काकी कतिलखा थावह हत बजिa होकर वही खड रह गएउनहोन सोचा निक मन अपन पतर को मारा जोनिक अनजान और निन-dरष थावह अपनी मा कोनिकतना पयार करता ह

उस दि-न बड सवर शयाम की नी- खली तो -खा निक घर भर म कोहराम मचा हआ ह

क) घर म कोहराम कयो मचा हआ था शयाम को कया लगा

ख) काकी को ल जात समय शयाम न कया उपदरव मचाया

ग) काकी क बार म उस कया बताया गया कया सतय उस कतिछपा रहा

घ) वह बठा-बठा शनय मन स आकाश की ओर कयोकरता

उततरक) शयाम की मा का -हात हो गया था इसकतिलए

घर म कोहराम मचा हआ था शयाम की लगा निक उसकी मा सफ- कपडा ओढ हए भमिम पर सो रही ह

ख) लोग जब उमा यानी शयाम की मा को उठाकर ल जान लग तब शयाम न बडा उपदरव मचाया लोगो क हाथ स झठ करवा उमा क ऊपर जा निगरा और बोला काकी सो रही ह उस कहा ल जा रह हो

ग) काकी क बार म बजिaमान लोगो न उस निवशवास दि-लाया निक उसकी का निक उसक मामा क यहा गई ह लनिकन सतय अमिधक दि-नो तक कतिछपाना रह सका आसपास क अबोध बालको क मह स यह बात परकट हो गई निक उसकी मा का -हात हो गया ह

घ) कई दि-नकई दि-न लगातार रोत-रोत उसका रोना तो शान हो गया पर उसक ह-य म शोक भर गया था वह चपचाप बठा आकाश की और टाका करता निक शाय- उसकी काकी कही दि-ख जाए

ldquoदि-न उसन ऊपर आसमान म पतग उडती -खी न जान कया सोच कर उसका निहर-य एक-म खिखल उठाrdquo

क) निकसन पतग ऊपर उडत -खी और वह कयो खश हआ

ख) उसन अपन निपता स कया कहा उनका कया उतर थाश

ग) उसन निफर कया निकया और निकसन उसकी सहायता की

घ) उसकी योजना कया थी उततर -क) शयाम न एक दि-न आसमान म पतग उडती

-खी तो उसन सोचा निक पता आसमान म राम क यहा जाकर रकगी वही पर मरी काकी ह यह सोचकर वह बहत खश हआ

ख) उसन अपन निपता स कहा काका मझ एक पतग मगा -ो उसक निपता न भटक हए मन क भाव स कहा निक मगा -ग यह कह कर उ-ास भाव स वह कही और चल गए पतग नही आई

ग) उसन चपचाप निवशशवर क टगहए कोट स एक चवननी निनकाल ली और सखिखया -ासी क लडक भोला की सहायता स एक पतग मगवानी भोला उसकी बराबर उमर का ही था

घ) उसकी योजना यह थी निक वह अपनी पतग को आकाश म राम क यहा भजगा और उस पतग क सहार उसकी काफी नीच उतर जाएगी इस योजना पर उस परा निवशवास था इसकतिलए वह और भोला -ोनो यह काम करन म लग गए

Continue to nexthelliphellipEVS CHAPTER - 1

(UNDERSTANDING OUR ENVIRONMENT)

Sustainable development

The development that meets the needs of the present without compromising the ability of future generations to meet their own needs is called Sustainable development

Sustainable societies ndash

An environmentally sustainable community is one that meets the current and future basic resource needs of its people in a just and equitable manner without compromising the ability of future generations to meet their basic needs

Q ) What are Eco Villages

Ans - Eco village are the urban or rural communities of people who strive to integrate a supportive social environment with a low impact way of life

Q ) To ensure sustainable development the depletion of renewable resources should not take place at a rate faster than their regeneration Justify your answer

Ans ndash Renewable resources do not have a fixed quantity - more can always be

generated However if the rate of use exceeds the rate of renewal - that is the

source is used more than its being recreated - its continued use will become

used up faster than it can regenerate

To promote sustainable society the following things need to be done ndash

1 Using renewable energy sources 2 By improving the quality of human

health 3 By promoting sustainable agriculture 4 By forming ecovillage

it will eventually be entirely depleted So Toensure sustainable development the depletion of

renewable resources should nottake place at a rate faster than their regeneration

Q ) What do you mean by Sustainable societies

Ans - Sustainable societies are defined as towns and cities that have taken steps to remain healthy over the long term These communities value healthy ecosystems use resources efficiently and actively seek to retain and enhance a locally based economy Sustainable development concerns everybody in a society

Q ) What are the effects of pollution on human health

Ans ndash Some health problem occurs due to air pollution are ndash

Respiratory diseases Cardiovascular damage Fatigue headaches and anxiety Irritation of the eyes nose and throat Damage to reproductive organs Harm to the liver spleen and blood Nervous system damage

Some health problem occurs due to water pollution are ndash

Typhoid Cholera Dysentry Jaundice

Some health problem occurs due to noise pollution are ndash

Fatigue headaches and anxiety High blood pressure Hearing damage

Physics Motion in 1D First go through previous notes Now here we will solve some numerical related to that

Question 3What information about the motion of a body is obtained from the displacement-time graphSolution 3From displacement-time graph the nature of motion (or state of rest) can be understood The slope of this graph gives the value of velocity of the body at any instant of time using which the velocity-time graph can also be drawn

Question 4(a)What does the slope of a displacement-time graph represent(b)Can displacement-time sketch be parallel to the displacement axis Give a reason to your answerSolution 4(a) Slope of a displacement-time graph represents velocity(b) The displacement-time graph can never be parallel to the displacement axis because such a line would mean that the distance covered by the body in a certain direction increases without any

increase in time which is not possible

Chemistry Language of Chemistry

How to balance a chemical equationThere are two methods of balancing an equation(i)Hit and trial method(ii)Partial equation methodBalancing by hit and trial methodThis method consists of counting the number of atoms of each elements on both sides and trying to equalize themTake the following steps(i)Count the number of times (frequency) an element occurs on either side(ii)The element with the least frequency of occurrence is balanced first(iii)When two or more elements have the same frequencythe metallic element is balanced firstExample-1 On heatinglead nitrate decomposes to give lead dioxidenitrogen dioxide and oxygenPb(NO3)2rarrPbO+NO2+O2

In this equationLead occurs twiceNitrogen occurs twiceOxygen occurs four timesSince lead is a metalbalance it firstThe number of atom of lead is equal on the two sidestherefore it needs no balancingNow balance nitrogenOn the reactant sidethere are two atoms of nitrogenwhile on the product side oneSomultiply the product containing nitrogenon the product sideby two Pb (NO3)2rarrPbO+2NO2+O2Nowthe number of oxygen atoms on the reactant side 6while on the product sideit is 7Somultiply the entire equation by 2except oxygen to get balanced equation2Pb(NO3)2rarr2PbO+4NO2+O2Multiplication by 2 is done only when atoms of all the elements except one element are balanced and the unbalanced atom occurs separately at least once and also there is a difference of only one such atom

Math Topic AlgebraChapter

Factorisation

Study item Difference of two squares a2 ndash b2 = (a+b) (a-b)1) (i) 4x2ndash 25y2

= (2x) 2 ndash (5y) 2= (2x + 5y) (2x - 5y)

(ii) 9x2 ndash 1= (3x)2ndash(1)2= (3x + 1)(3x ndash 1)

2) (i) 150 ndash 6a2= 6(25 ndash a2)= 6(5)2 ndash(a)2= 6 (5 + a) (5 ndash a)

(ii) 32x2 ndash 18y2=2(16x2 ndash 9y2)=2(4x)2 ndash (3y)2= 2(4x + 3y)(4x - 3y)3)(i) (x ndashy )2 ndash 9 = (x ndash y )2 ndash (3)2= (x ndash y + 3) (x ndash y ndash 3)(ii) 9(x + y) 2ndash x 2= (3)2(x + y)2 ndash (x)2=3(x + y)2 ndash (x)2= (3x +3y ) 2ndash(x)2= (3x + 3y + x)(3x +3y ndash x)= (4x + 3y) ( 2x + 3y )

Commercial studies

Basic accounting terms

Today I will give you some questions from the previous study material

Questions1) Define accounting2) What do you mean by debit and

credit

3) Explain the types of account4) Define the following terms

a) Assetsb) Capitalc) Purchased) Debtorse) Transactions

5) Name the types of accounts given below

a) Krishnas accountb) Machinery accountc) Royalty accountd) Salary accounte) Furniture accountf) Audit fee account

Economics Revision Today I will give you some revision questions

Questions1) What do you mean by the terms

rdquowantsrdquo2) Write the difference between

consumer goods and producer goods

3) Define the term utility 4) Explain the different types of utility5) Define

a) Total utilityb) Marginal utility

Subject Eng Literature (The Merchant of Venice ndash William Shakespeare)Topic Act I Scene 3 Lines 1 to 48 (Shylock hellip Cursed be my tribe if I forgive him) Date 16th April 2020 (5th Period)

[Students should read the original play and also the paraphrase given in the school prescribed textbook]Summary Questions amp Answers

This scene takes place in Venice and we are introduced to the rich Jew Shylock Bassanio and Shylock are talking and Bassanio tells Shylock that he wants a loan of three thousand ducats for three months on the personal security of Antonio

o Shylock feels glad because he will be able to bind down Antonio by means of a bond on account of the loan but he tells Bassanio that all the fortunes of Antonio being invested in the merchant ships on the sea it is difficult to depend upon his credit Even under such circumstances Shylock is willing to advance the money on the personal security of Antonio

o Bassanio then invites Shylock to dine with him Shylock says that he is prepared to do anything with the Christians but not eat or drink or pray with them

o While Bassanio and Shylock are talking Antonio appears on the scene Shylock does not seem to take any notice of Antonio but goes on brooding within

(1) SHYLOCK Ho no no no no- my meaning in (Line 15-26)saying he is a good man is to have you understand me that he is sufficient Yet his means are in suppositionhe hath an argosy bound to Tripolis another to the Indies I understand moreover upon the Rialto he hath a third at Mexico a fourth for England and other ventures he hath squanderd abroad Butships are but boards sailors but men there be land-rats and water-rats land-thieves and water-thieves I mean pirates and then there is the peril of waters winds and rocks The man is notwithstanding sufficientmdashthree thousand ducats mdashI think I may take his bond

(a) Who is talking in the beginning of this scene What does Bassanio want from Shylock How does Shylock feel

In the beginning of the scene Bassanio and Shylock are talking to each other Bassanio wants to borrow three thousand ducats from Shylock for three months on the security of Antonio Shylock feels glad at heart that he will get the opportunity of binding Antonio with a bond(b) What risks does Shylock weigh in advancing the money

Shylock says that Antonio has invested all his capital in trading by sea-going ships But the ships are made of wood and the sailors of those ships are ordinary human beings The wood can

himself how he hates Antonio because of his being a Christian because he abuses Shylock in public places Shylock decides that if ever he can get Antonio to his advantage he will teach him a lesson

come to harm and men can commit mistakes and thus the capital invested in ships may be lost Then there are other dangers The goods loaded on the ships can be damaged by rats and thieves which are found both on land and water The ships can also be harmed through sea-storms submerged rocks etc(c) What two important functions does this scene have

The scene has two important functions First it completes the exposition of the two major plot lines of the play Antonio agrees to Shylockrsquos bond ndash three thousand ducats for a pound of flesh and second and more important dramatically this scene introduces Shylock himself In this scene Shakespeare makes it clear at once why Shylock is the most powerful dramatic figure in the play and why so many great actors have regarded this part as one of the most rewarding roles in all Shakespearean dramas(d) Where does this scene take place What kind of treatment has Antonio been giving to Shylock What does Shylock say when Bassanio invites him to dine with him

The action of this scene takes place in Venice Antonio has been in the habit of behaving harshly with Shylock ndash spitting on his beard and footing him like a stranger cur When Bassanio invites Shylock to dine Shylock says that he is prepared to do anything with the Christians but not eat and drink or pray with them

(2) SHYLOCK How like a fawning publican he looks (Line 38-48)I hate him for he is a Christian

But more for that in low simplicity

He lends out money gratis and brings downThe rate of usance here with us in VeniceIf I can catch him once upon the hipI will feed fat the ancient grudge I bear him

He hates our sacred nation and he railsEven there where merchants most do congregateOn me my bargains and my well-won thriftWhich he calls interest Cursed be my tribeIf I forgive him

(a) What is the context in which these words are spoken and what is the idea expressed in it

These remarks are made by Shylock when he sees Antonio coming along after Bassanio told him that the merchant will be his surety for the bond The above mentioned passage reveals Shylockrsquos hatred for Antonio Shylock says that he hates Antonio because he is a Christian and also because he gives loan without taking interest on them thereby bringing down the rate of interest in Venice(b) Explain the meaning of the phrase lsquoa fawning publicanrsquo

The phrase lsquoa fawning publicanrsquo refers to Roman tax collector It is a term of contempt and hatred on the lips of a Jew lsquoFawning Publicansrsquo were Roman tax-gatherers whose ordinary bearings towards the Jews was bullying but whose false pose of lsquohumility and contritionrsquo is touched upon in the parable in New Testament(c ) What light does the above passage throw on the character

of Shylock

The above mentioned speech of Shylock reveals him to be a wicked character having an extreme greed for wealth His intense hatred for Antonio is unjustified He hates Antonio only because he is a Christian and because he lends money without taking any interest on it thereby adversely affecting Shylockrsquos business of lending money on high interest(d) What information do you gather about Antonio from the above given lines

Shylockrsquos statement throws a valuable light on the character of Antonio Antonio appears to be a good Christian and a good human being He helps the people in need by lending them money without charging any interest on it He is a man of simple and good nature This very goodness makes him Shylockrsquos enemy(e) What does Shylock debate within himself and when To whom are the lines mentioned above addressed to

When Bassanio asks the Jew to lend him three thousand ducats on Antoniorsquos surety Shylock begins to debate within himself as to how he should exploit the opportunity of a business deal with his old enemy Antonio

The lines mentioned above are not addressed to anyone The lines are a soliloquy ie a speech made by a character to himself and not meant to be heard by the other characters present

Class XSubject Topic Summary ExecutionEnglish

LiteratureThe Blue Bead 2nd part

Things took a turn for the worst and all of a sudden a crocodile attacked the woman biting on the womanrsquos leg At that moment Sibia got up sprinted grabbed the hay fork and stabbed the crocodile in the eye with all her power Immediately the crocodile let go and went away Sibia saw a small blue bead lying by the river she grabbed it Since she was poor she didnrsquot have necklace Shersquod always wanted one like the other women now she could make one with the blue bead After that she went home and told her mother all about it

Hindi 2nd

Langबड घर की बटी( मशी परमच-)

lsquoबड घर की बटी कहानी का उददशय मधयम वग की घरल समसया को सलझा कर सगदिठत परिरवार म मिमल जलकर परम स रहन का स-श -ना हघर म शानित सथानिपत करन की जिजमम-ारी नारी की होती ह यदि- नारी समझ-ार ह उसम धय और परिरवार क परनित परम ह तो कोई भी घटना परिरवार को निवघदिटत नही कर सकती या कहानी परिरवार को सगदिठत करत हए परम सौहा- स एक दसर की भावनाओ को समझ करउनका सहयोग करत हए जीवन यापन करन की पररणा -ती हमशी परमचदर जी न इस कहानी म सय परिरवार का परनितनिनमिधतव निकया ह यह कहानी बनी माधव सिसह जो गौरी पर क जमी-ार क उनक -ो पतरो की हशरी कठ लाल निबहारीशरीकात का निववाह एकजमी-ार घरान की पतरी आन-ी स हआ थाआन-ी न ख- को ससराल क वातावरण म ढाकतिलया थाएक दि-न आन-ी का अपन -वर लाल निबहारी स झगडा हो जाता ह -ोनो भाई एक दसर स अलग होन की कोकतिशश करत हसभी बह आन-ी न अपन मधर वयवहार स लाल निबहारी को

ldquoइन नतर निपरय गणो को बीए-इनही -ो अकषर पर नयोछावर कर दि-या था इन -ो अकषर न उनक शरीर को निनबल और चहर को कानित ही बना दि-या थाldquo

क) परसतत पकतियो म निकस वयकति क बार म कहा गया ह

ख) इन पकतियो म कौन स नतर निपरय गणो क बार म कहा गया ह

ग) बीए की निडगरी परापत कर लन पर भी उपय वयकति क सवभाव की कया निवशरषता थी

घ) यह नतर निपरय गण निकस वयकति म निवदयमान थ उसक वयकतितव की कया निवशरषता थी

उततर ndashक) परसतत पकति म गौरी पर गाव क जमी-ार

क बड बट शरीकात क बार म कहा गया ह उसन बहत परिरशरम और उ-म क बा- ba की निडगरी परापत की थी अब वह एक -फतर

घर छोडकर जान स रोक कतिलयाइस पर बनी माधव सिसह न कहा निक बड घर की बटी ऐसी ही होती ह जो निबगडा काम बना लती ह अतः शीरषक साथक ह बड घर की बटी आन-ी ह

म कमचारी थाख) भरा हआ चहरा चौडी छाती और डटकर

खाना आदि- एक सबजी ल जवान क गण मान जात ह परत शरीकात न इनही नतर निपरय गणो को अपनी पढाई पर नयोछावर कर दि-या था

ग) बीए की निडगरी परापत कर लन पर भी उपय वयकति(शरी कठ की शारिररिरक तौर पर निनबल और चहर स कानित ही लगत थ इतना ही नही वह मानकतिसक तौर पर भी निपछड हए थ पाशचातय सामाजिजक कथा उस घणा एव पराचीन सभयता का गणगान उनकी निवचारधारा क परमख अग थ

घ) यह नतर निपरय गण गौरीपर गाव क जमी-ार क छोट बट लाल निबहारी सिसह म निवदयमान थ वह सजीलाजवान था और भस का दध शर दध वह सवर उठकर पी जाता था

ldquoयही कारण था निक गाव की लललन आए उनकी निन-क थी कोई कोई तो उह अपना शतर समझन म भी सकोच ना करती थी सवय उनकी पतनी को इस निवरषय म उनस निवरोध थाldquo

क) उपय पकति म इस वयकति क बार म कहा गया ह

ख) गाव की लललन आए उनकी निन-ा कयो निकया करती थी

ग) उनकी पतनी का कया नाम था उनह निकस निवरषय म अपन पनित क निवरa था और कयो

घ) इस कहानी का कया उददशय ह Continue to next helliphelliphellip

Bengali 2nd Language

ফ ফটক না ফটক( কহিতা )

পর) ldquo(ান াধাচেনা ফটপাচেথ পাথচের পাড হিচেয় এক কাঠচোটটা গাছ কহিকহি পাতায় পার ফাটিচেয় াসচেছldquoক) কার দো দেকান কহিতার অং( ) lsquo(ান াধাচেনা ফটপাচেথ পাথচের পাডহিচেয়lsquo চেত কী দোঝাচেনা চেয়চেছ গ) আচো য অংচে(lsquo এক কাঠচোটটা গাছ lsquoচেত কী দোঝাচেনা চেয়চেছ ঘ) ldquoকহিকহি পাতায় পার ফাটিচেয় াসচেছldquo ----- একথার পরকত অথC কী উততর ) ক) আচো য অং(টি পর যাত কহি সভা4 মচোপাধ যাচেয়র দো lsquoফ ফটক না ফটকrsquo কহিতার অং()কহি সভা4 মচোপাধ যায় হিছচেন দেপরচেমর কহি দেপরমচেক নানা ভহিঙগমায় হিতহিন ফটিচেয় তচেচেছন দেপরম মানচের স মচেতC র সঙগী কহিতার কহিতায় এক রb সb হচেয়র দেপরম াগরচেনর কথা চেচেছন (ান অথCাৎ দেযাচেন দেকান রস দেনই দেযাচেন দেকান মহিনতা দেনই অথ তার মধ দেযও দেপরম থাকচেত পাচের একথাই কহি তচে ধরচেত দেচেয়চেছন একটি মানচে4র মচেন দেযাচেন দেকামতার দেকান সথান দেনই পাথচেরর মচেতা হিনরসতার মচেনর মধ দেযও দেয দেপরম আসচেত পাচের দেস কথাই কহি চেচেছনগ)নারীচের যথC দেপরচেমর ছহি এই কহিতায় অকপচেট উচেঠ এচেসচেছ কহি এই কহিতায় কাটচোটটা গাছ কথাটি যার কচেরচেছন নারী দেয দেপরম দেথচেক হিতাহিত এং দেসই দেপরম সঠিক সমচেয় না পাওয়ার ন য দেপরম সমপচেকC হিচেr4 গৈতরী য় দেপরচেমর দেয গৈহি(ষট য মাধযC য সরসতা দেকামত এই সমসতর হিপরীত যথা রbতা শষকতা কচেঠার তা পরভহিত দোঝাচেত এক কাঠচোটটা গাছ কথাটি যার কচেরচেছনঘ) এাচেন এক নারীর যথC দেপরচেমর কথা হিনহিCপত ভাচে চেচেছন কহি অসমচেয় নারীর ীচেন দেপরম দেচেগচেছ এতহিন তার হয় রb কচেঠার হিছ দেপরচেমর অভাচে ঠাৎ দেসই শষক মরভহিমচেত সচের আভাস এচেসচেছ দেপরম দেযন 4Cার স(ীত তাই পরায় মত গাচেছ কহিকহি পাতা গহিচেয় উচেঠচেছ

Biology Chapter - 01Controlling Air Pollution

Today we will discuss how we control air pollution from domestic combustion

Q1Describe any five ways of reducing air pollution from domestic sources bull The number of pollutants in the air is verylarge and we always try to control them byfollowing ways

i) Solar cooker and solar heater It use no fuel reduce damage of environment by fuel use or reducing deforestation It maintains coolness of house It releases very less orno oil gas or grease

ii) Piped natural gas (PNG) It emits very less by products into the atmosphere As it isdistributed through pipe lines so there iscontinuous supply of fuel is possible

iii) Liquefied Petroleum Gas (LPG) It hasa higher heating value LPG doesntcontain sulphur so it burns a lot cleanerenergy sources It releases very less oralmost no fume in air

iv) Electricity based cooking Emission free cooking alternative for urban dwellers causeselimination of adverse health impactsofindoor air pollution It helps to avoid theinconveniences associated with procurement of LPG

v) Biogas It contains 75 methane whichmakes it an excellent fuel It burns without smoke and biogas plant leaves no residue like ash in wood charcoal etc Thus it isaclean fuel

Economics

Factors of Production

Today firstly we would recall the last class for 5 mins and then we would proceed with the further topics of the chapter

The concept meaning of land characteristics of land and importance of land to be repeated for the absentees as well as the students who were there in the class the previous day

Today we will start with the last portion of land before it the meaning of land to be repeated onceAs by now we all know that

Questions1What do you mean by productivity of landAnswer By productivity of land we mean the capacity of a piece of land to produce a crop

Thus it refers to the average output per unit of landSay per acre per hectare etc= (OutputArea of land)

2 What are the factors influencing the productivity of landAnswer

Natural factors Productivity of land is largely determined by the natural

Land is defined to include not only the surface of the earth but also all other free gifts of nature(for example mineral resources forest resources and indeed anything that helps us to carry out the production of goods and services but is provided by nature free of cost)

We will move on to the last portion of land by discussing Productivity of Land

By productivity of land we mean the capacity of a piece of land to produce a crop

Thus it refers to the average output per unit of land

Say per acre per hectare etc= (OutputArea of land)

With this we shall proceed further with the main factors that determine the productivity of land

Natural factors Human factors Improvements on land Location of land Organisation Ownership of land Availability of capital Proper use of land State help

Note economic development of a country depends upon the quality of its land If the land is fertile it will quicken the pace of development of the country

qualities of land such as fertility etc

Human factors Land cannot produce anything by itself Man has to apply labour on it to produce for himself So productivity of land depends on the knowledge and skills of workers

Improvements on land production of land is affected by land development measures like provision of well or tubewell irrigation proper drainage

State help The government of a country especially less developed country can play a vital role in improving the agricultural productivity by providing better irrigation facilities

Organisation Productivity of land also fdepends upon the way how the factors of production like labour and capital are organised

In order to increase productivity trained workers modern implements scientific methods good seeds are all essential

3 lsquoImproved technology affects the productivity of landrsquo Explain this statement with the help of suitable example Answer Use of improved technology raises the productivity of land Example By using HYV seeds chemical manures and modern machines per hectare output increases

Physics Force (Summary)

Question Write the expression for the moment of force about a given axisSolutionsThe expression for the moment of force is given byMoment of force about a given axis = Force times perpendicular distance of force from the axis of rotationQuestion What do you understand by the clockwise and anticlockwise moment of force When is it taken positiveSolutionsIf the effect on the body is to turn it anticlockwise moment of force is called the anticlockwise moment and it is taken as positive while if the effect on the

body is to turn it clockwise moment of force is called the clockwise moment and it is taken as negative

Math Topic Commercial Mathematics

Chapter Goods and services Tax

Study item Some solved sums from exercise ndash 1 A retailer buys a TV from a wholesaler for Rs 40000 He marks the price of the TV 15 above his cost price sells it to the consumer at 5 discount on the marked price If the sales are intra ndash state and the rate of GST is 12 find

(i) The marked price of the TV(ii) The amount which the consumer pays for the TV(iii) The amount of tax (under GST) paid by the retailer to the central

Government(iv) The amount of tax (under GST) received by the State Government

Solution As the sales are intra- state sale and the rate of GST 12 So GST comprises of 6 CGST and 6 SGSTTherefore a retailer buys a TV from a wholesaler for Rs 40000Therefore the amount of GST collected wholesaler from the retailer or paid by retailer to wholesalerCGST = 6 of Rs 40000 = Rs(6100 times40000) =Rs 2400SGST = 6 of Rs 40000 = Rs (6100 times 40000) =Rs 2400Therefore wholesaler will pay Rs 2400 as CGST and Rs 2400 as SGSTTherefore amount of input GST of retailer Input CGST = Rs 2400 and input SGST = Rs 2400Again the retailer marks the price of the TV 15 above his cost price(i) The marked price of the TV

= Rs 40000 + Rs 40000times15= Rs 40000 + Rs 40000times 15100= Rs 40000 + Rs 6000Rs 46000But the retailer sells it to consumer at 5 discount on the marked priceCost price after discount = Rs 46000 ndashRs46000times 5100 =Rs 46000 ndashRs 2300= Rs 43700Therefore the amount of GST collected retailer from consumer or paid by consumer to retailerCGST = 6 of Rs 43700 =Rs ( 6100 times43700)Rs 2622SGST = 6 of Rs 43700 = Rs (6100 times 43700) =Rs 2622Amount of the output GST of retailer Output CGST = Rs 2622 and output SGST = Rs 2622

(ii) The amount which the consumer pays for the TV= cost price of TV to consumer + CGST paid by consumer + SGST paid by consumer= Rs 43700 + Rs 2622 + Rs 2622= Rs 48944

(iii) The amount of tax (under GST ) paid by the retailer to the central Government=CGST paid by retailer = output CGST ndash input CGST=Rs 2622 ndash Rs 2400=Rs 222

(iv) The amount of tax ( under GST ) received by the State Government = SGST paid by wholesaler + SGST paid by retailer= Rs 2400 + output SGST ndash input SGST=Rs 2400 + Rs 2622 ndash Rs 2400=Rs 2400 + Rs 222= Rs 2622

Commercial studies

Stakeholders Today I am going to give some revision questions from the previous study material

Questions1) State the two expectations of

employees from a business concern2) Give two distinctions between

stakeholder and shareholder3) Give two difference between

internal stakeholders and external stakeholders

4) Give two expectations of suppliers from a business organisation

5) Who is a stakeholder in commercial organisations

Chemistry Periodic Table

Merits of Mendeleevrsquos Periodic law are as follows - 1He grouped the elements on the basis of atomic mass 2 He left gaps for undiscovered elements like Gallium Scandium germanium Also he left a full group vacant for undiscovered inert gases 3 He could predict proportions of several elements on basis of their position in periodic table like Ga Sc etc 4He could predict errors in atomic weights of some elements like gold platinum etc

Anomalies in Mendeleevrsquos Periodic law are as follows - 1 Position of isotopes could not be explained 2 Wrong order of atomic masses could not be explained

For example- as Arnur atomic mass 40 come first and K with low atomic mass (30) should come later but k should be placed first

According to Bohrrsquos Modern Periodic table properties of elements are periodic functions of their atomic numbers

So when elements are arranged according to increasing atomic numbers there is periodicity in electronic configuration that leads to periodicity in their chemical properties

It consists of horizontal rows (Periods) Vertical column (Groups)

There are 7 period and 12 groups in this long form of periodic table

Ist period has 2 elements IInd period has 8 elements IIIrd period has 8 elements IVth period has 18 elements Vth period has 18 elements VIth period has 32 elements VIIth period hs rest of elements

Note - The number of valence electrons in atom of elements decides which elements will be first in period and which will be last

In group- 1 to 2 gp and 13 to 17 contain normal elements 3 to 12gp ndash transition elements 57 to 71 - lanthanides 89 to 103 - Actinides

Left hand side ndash metals Right hand side ndash nonmetals

Note- Hydrogen element has been placed at top of Ist group Electronic configuration of H is similar to alkali metal as both have 1 valence electron

V electron of gp I element -- 1 V electron of gp 2 element -- 2 V electron of gp 13 element -- 3 V electron of gp 14 element -- 4 V electron of gp 15 element -- 5 V electron of gp 16 element --6 V electron of gp 17 element -- 7 V electron of gp 18 element -- 8

English 1 Transformation of sentences

Sentences A sentence is a group of words which makes complete sense

Exercise 2Change the following sentences from

a Assertive sentencesb Imperative sentencesc Interrogative sentencesd Exclamatory sentences

Sentences can be changed from one grammatical form to another without changing the meaning of the sentence This is known as transformation of sentences

assertive to interrogative1 Nobody would like to be a fool

Who would like to be a fool2 Their glory can never fade

When can the glory fade3 Nobody can control the wind

Who can control the wind4 It matters little if I die

What though I die5 No man can serve two masters

Can any man serve two masters

Exercise 3Interchange of assertive and Exclamatory sentences

1 She leads the most unhappy lifeWhat an unhappy life she leads

2 This is indeed an interesting bookWhat an interesting book this

3 He is a very great manWhat a great man he is

4 It is a very lame excuseWhat a lame excuse

5 It is sad that she died so youngAlas she died so young

Class XISubject Topic Summary Execution

Hindi 2nd lang

पतर परम(परमचदर) पतर परम कहानी म एक निपता की इचछाओ का वणन निकया गया ह अपन बड पतर परभ -ास स निपता चतनय -ास का निवशरष परम था निपता को उसक जनम स ही बडी-बडी आशाए थी उसम दसर बट कतिशव-ास की अपकषा स- उतसाह की मातरा अमिधक थी वह उस इगलड भजकर बरिरसटर बनाना चाहत थभागय का खल भी बडा निनराला ह बीए की परीकषा क बा- वह बीमार पड गया डॉकटरो न भी जवाब - दि-या थाचतन -ास जी बहत ही कजस थ बवजह पस खच करना नही चाहत थ अगर गारटी मिमलती तो शाय- पस खच भी कर -त परत गारटी नही थी परिरणाम सवरप उनक बट का -हात हो गयाजब बट को समशान ल जा रह थ तो वहा काफी शोर गान बजान हो रह थ पछन पर पता चला निक निकसी निपता निपछल तीन साल स निबमार था और उसक ईलाज म रपया पानी की तरह बहाया पर ठीक नही हए परत उसक बट को तनिनक भी अफसोस नही था उसका कहना था उसन कोकतिशश तो कीयह -खकर चतनय-ास जी को आतम निगलानी हईतभी स उनका म परिरवतन हआ और बट का भोज काफी धमधाम स निकयाऔर वहइस पशचाताप की आग म जलत रह औला- स बढकर पसा नही होता ह इस बात को समझन म उनह काफी व लग गया

hellipContinue to next

BENGALI(2ND LANGUAGE)

পরথমঅধযায়-ঠাকরারীনদরনাথঠাকর

নয়ন দোচের হিমাচেররা া নাচেমই হিযাত হিছচেন ায়ানার উাররণ সবরপ নয়ন দোচের ারা হিা (াচেকর হিা হিচেতন এছাাও দেকান উৎস উপচেb রাহিতর দেক হিন করার উচেfচে(য তারা সযC হিকরচেরণ রনয পরীপ জবাহিচেয় তাচেত রপার হির 4Cরণ করচেতন ঠাকরা এই নয়ন দো হিমারচের দে(4 ং(ধর হিছচেন হিমাররা ায়ানার ষটানত পর(Cন কচের তারা হিনঃসব এই হিমাহিরর দে(4 ং(ধর গৈকাস নদর রায়চেৌধরী গৈকাস া নয়ন দোচের সমসত সমপহিতত ঋচেরণর াচেয় হিহিx কচের অহি(ষট যা আচেছ তাচেত হিপত

ইার হিপতার মতয ইচে পর নয়নচোচের ায়ানার দেগাটা কতক অসাধাররণ শরাদধ (াহিনতচেত অহিনতম ীহিপত পরকা( কহিরয়া ঠাৎ হিনহিয়া দেগ- ক) কার দো দেকান গচেলপর অং() কতা দেক ইার চেত কাচেক দোঝাচেনা চেয়চেছ গ) পরসঙগ কী কতার কতয পহিরসফট কচেরা

পরচে4র যাহিত রbা করা সমভ নয় তাই হিতহিন পতরচেক হিনচেয় ককাতায় সাস শর কচেরন গলপ কথচেকর আহিথCক অসথা নয়ন দোচের হিমাচের দেথচেক সমপরণC আাা কথচেকর হিপতা হিনচের দেষটায় অথC উপাCন করচেতন া উপাহিধ াচেভর নয তার াসা হিছনা আর দেসই কারচেরণ কথক তার একমাতর উততরাহিধকার চেয় তার হিপতার পরহিত কতজঞ কথক দো পা হি(চেচেছন হিনচের পরারণ ও মান রbার নয উপচেযাগী অথC হিনা দেষটায় পরাপত চেয়চেছন- এটাই তার কাচেছ পরম দেগৌরচের হি4য় চে মচেন কচেরন কাররণ (নয ভাণডাচের গৈপতক ায়ানার উজজব ইহিতাস অচেপbা দোার হিসeচেকর মচেধয গৈপতক দেকামপাহিনর কাগ তার কাচেছ অচেনক দেহি( মযান

TO BE CONTINUED

উ- ক) আচোয অং(টি রীনদরনাথ ঠাকচেরর দো ঠাকরা গচেলপর অং() কতা চেন আচোয গচেলপর গলপ কথকইার চেত নয়ন দোচের হিমাহিরর দে(4 ং(ধর গৈকাস ার কথা া চেয়চেছ গৈকাস া নয়ন দোচের সমসত সমপহিতত ঋচেরণর াচেয় হিহিx কচের অহি(ষট যা আচেছ তাচেত হিপত পরচে4র যাহিত রbা করা সমভ নয় তাই হিতহিন পতরচেক হিনচেয় ককাতায় সাস শর কচেরনগ) গৈকাস ার হিপতার মতযর পর নয়ন দোচের হিমাহিরর অহিসততব হিপত য় কচেয়কটা উৎস ও শরাদধ- (াহিনতচেত হিমাহিরর দে(4 কহিটক যয় চেয় হিগচেয় এচেক াচের দে(4 চেয় যায় তন তাচের গC করার মত আর হিকছই হিছ না-দেসই পরসচেঙগ এই উহিকত নয়নচোচের হিমাচেররা া নাচেমই হিযাত হিছচেন ায়ানার উাররণ সবরপ নয়নচোচের ারা হিা (াচেকর হিা হিচেতন এছাাও দেকান উৎস উপচেb রাহিতরচেক হিন করচেত হিগচেয় তারা সযC হিকরচেরণর নয পরীপ জবাহিচেয় তাচেত রপার হির 4Cরণ করচেতন তাই দেসকাচের ায়ানা দেহি(হিন সথায়ী চেত পারত না হিহিভনন উৎস শরাদধ- (াহিনতচেত সাধযা হিতহিরকত র করার নয হিমাহির হিহিকচেয় দেযত হ হিতC কা হিহি(ষট পরীচেপর দেত দেযমন অলপকাচের মচেধয হিনঃচে(4 চেয় যায়-নয়নচোচের হিমারচের অসথা তাই চেয়হিছ এই কারচেরণই কথক নয়নচোচের হিমারচের গা ভরা আমবর সয করচেত পারতনা

Physics Dimensional Analysis (Summary)

Q Find the dimensions of consts ab in relation

p=(bminusxlowastx)at

where p is the power x is the distance and t is time

Ans From principle of homogeneity dimension of b x2 are same Dim of b = dim of x2 = [L2] = [ML2T0]Dim of a = dim of ( b- x2)dim of (pt) = [M0L2T0][ML2T-2] [T-1] [T] = [M-1L0T2]

Chemistry Atomic Structure Drawbacks of Rutherfordrsquos model of

atom a According to Rutherfordrsquos model of atom electrons which are negativelycharged particles revolve around the nucleus in fixed orbits Thusb theelectrons undergo acceleration According to electromagnetic theory of Maxwell a charged particle undergoing acceleration should emitelectromagnetic radiation Thus an electron in an orbit should emitradiation Thus the orbit should shrink But this does not happenc The model does not give any information about how electrons aredistributed around nucleus and what are energies of these electrons Isotopes These are the atoms of the same

Properties of electromagnetic radiationsa Oscillating electric and magnetic field are produced by oscillating charged particles These fields are perpendicular to each other and both areperpendicular to the direction of propagation of the waveb They do not need a medium to travel That means they can even travel invacuum

Characteristics of electromagnetic radiationsa Wavelength It may be defined as the distance between two neighbouring crests or troughs of

element having the same atomicnumber but different mass numbere g 1H11H21H3

Isobars Isobars are the atoms of different elements having the same massnumber but different atomic numbere g 18Ar40 20Ca40

Isoelectronic species These are those species which have the same numberof electrons

Electromagnetic radiationsThe radiations which are associated withelectrical and magnetic fields are called electromagnetic radiations When anelectrically charged particle moves under acceleration alternating electricaland magnetic fields are produced and transmitted These fields aretransmitted in the form of waves These waves are called electromagneticwaves or electromagnetic radiations

wave as shown It is denoted by λb Frequency (ν) It may be defined as the number of waves which passthrough a particular point in one secondc Velocity (v) It is defined as the distance travelled by a wave in onesecond In vacuum all types of electromagnetic radiations travel with thesame velocity Its value is 3 times10 8m sec-1 It is denoted by v

d Wave number Wave number is defined as the number of wavelengths per unit lengthVelocity = frequency timeswavelength c = νλ

Plancks Quantum Theory- o The radiant energy is emitted or absorbed not continuously but discontinuously in the form of small discrete packets of energy called lsquoquantumrsquo In case of light the quantum of energy is called a lsquophotonrsquoo The energy of each quantum is directly proportional to the frequency of the radiation ie E α υ or E= hυ where h= Planckrsquos constant = 6626 x 10-27 Js o Energy is always emitted or absorbed as integral multiple of this uantum E=nhυ Where n=1234Black body An ideal body which emits and absorbs all frequencies is calleda black body The radiation emitted by such a body is called black body radiation

Photoelectric effectThe phenomenon of ejection of electrons from thesurface of metal when light of suitable frequency strikes it is calledphotoelectric effect The ejected electrons are called photoelectrons

Biology Chapter - 02Systematics and Five Kingdoms

Scientists divide the whole living organisms into two kingdom first and ultimately by five kingdom at last

In the earlier systems of classifications organisms are divided into kingdom plantaeand kingdom animalia on the of presenceof cell wall their modes of nutrition and movements

Some problem arise like fungi share manycharacteristic withplant despite their heterotrophic nutrition bacteria protozoa areunicellular present in both kingdom Toovercome this third kingdom Protista isintroduced which include

unicellularorganisms But there is also another

problem Allunicellular organisms are not similar kind The cellular structure of prokaryotes is verydifferent from that of other organismsEukaryotes possess a true nucleus and allcell organelles that are not present inprokaryotes So the fourth kingdom Monerais introduced which include unicellular prokaryotes (bacteriaamp blue green algae)

bull Still some problem arise in kingdomplantae

So in 1969 R H Whittakar proposedanew five kingdom System of classification

i) Kingdom Monera - unicellular prokaryotes

ii) kingdom Protista - unicellular eukaryotes

iii) Kingdom Fungi - uni or multicellular fungi with cell wall but without chlorophyll

iv) Kingdom Plantae - Multicellular Plants

v) Kingdom Animalia - Multicellular Animals

EVS Chapter 1 ndash Modes of Existence

An agricultural society

An agricultural society also known as an agrarian society is a society that constructs social order around a reliance upon farming More than half the people living in that society make their living by farming

People in an agricultural society generally lead a more settled lifestyle than those in nomadic hunter-gatherer or semi-nomadic pastoral societies because they live permanently near the land that is farmed Agricultural settlements tend to develop in areas of convenience near bodies of water which is used for both crops and transportation or along trade routes Not everyone in an agricultural society is a farmer Some people make a living trading or making and selling goods such as tools used for farming

Another way to define an agrarian society is to see the total amount of production in a nation In an agrarian society cultivating the land is the main source of wealth Such a society can recognize other means of subsistence and work habits but emphasizes the importance of agriculture and livestock Agrarian societies have existed in various parts of the world for 10000 years and continue to exist today They have been the most common form of socio-economic organization for most of recorded human history

Q) Write the features of agricultural society

Ans - Structure and Features of Agrarian Society1 Occupational Structure

An agrarian society is generally associated with the domestication of plants and animals The domestication of plants means farming and that of animals means herding Often there is mixture of farming and the use of such domesticated animals as cow goat and sheep

2 Forms of Land Ownership in Agrarian SocietiesGenerally there are landlords supervisory farmers cultivators and share croppers The landholders own the land but do not work on it They let it out for sharecropping The supervisory farmers are those who live by having their land cultivated by hired labourers The cultivators cultivate the land for themselvesThe share-croppers are those who live by tilling other peoplersquos land or a crop-sharing basis The artisans own their means of production and produce by their own labour in their homesteads

3 Village Community System An agrarian society is highlighted by

the institution of village community system The agrarian economy made fixed dwelling houses necessary Living close together for protection and co-operation and living nearer to the land gave birth to agricultural villages The village is not only the residential place of farmers it is also the social integrator

4 Minimal Division of Labour Another structural feature of agrarian society is a minimal division of labour Except for the basic division founded on age and sex differences there are few specialized roles There is only one predominant type of occupation ie domestication of plants and animals For all the people the environment physical as well as social is the same

5 Role of Family The farm family is of the patriarchal type the father is the final arbiter in most of the familyrsquos major decisions The life of ail men and women is merged in family life Since there are not many special organizations family is the only organisation to perform the tasks of aid and protection

6 Sense of Unity The members of an agrarian society exhibit a strong in-group feeling Since the whole of their social lives is wrapped up in a society which is physically economically and socially homogenous they are inclined to view the entire outside world as an out group

7 Informal Social Control An agrarian society is regionally divided into villages In a village community the force of traditional mores is more dominant than in the urban community In the village everybody is known to everybody The members in a village community help each other and share the joy and sorrows of each other Crime in an agrarian society is rare

8 Simplicity and Uniformity Life of the people in an agrarian society is marked by simplicity and uniformity Their main occupation is agriculture which largely depends upon the vagaries of nature An agrarian society is a religious society

Math Compound angles Compound angles The algebraic sum of two or more angles is called a compound angle If A B C be three angles then A+B B+C C+A A-B B-C A-C A+B-C etc are compound angles In this chapter we shall discuss the trigonometrical ratios of compound angles Theorem 1 If A B and A+B are all pisitive acute angles theni) sin( A+B) = sin A cos B + cosA sinBii) cos(A+B) = cosA cosB- sinA sinBTheorem 2If A and B are positive acute angles and AgtB theni) sin(A-B) = sin A cosB- cos A sinBii) cos(A-B) = cos A cos B+ sin A sin BTo prove that i) sin(A+B) sin (A-B) = sin2 A - sin2 B = cos2 B- cos2 A

Example 1 Prove that tan70deg=2tan50deg+tan20degSolutiontan70deg = tan(50deg + 20deg)Or tan70deg=(tan 50deg+tan 20deg)(1-tan50degtan20deg) or tan70deg (1 ndash tan 50deg tan20deg) = tan50deg+tan20degor tan70deg= tan70deg tan50deg tan20deg+ tan50deg + tan20deg = cot20deg tan50deg tan20deg + tan50deg + tan20deg = 2 tan50deg+ tan20degExample 2 If A + B = 45deg show that (1 + tanA) (1 + tanB) = 2Solutiontan(A + B) =( tan A + tan B) (1 - tan

ii) cos(A+B) Cos(A-B) = cos2 A- sin2 B = cos2 B -sin2 AProof i) LHS= sin(A+B)sin(AminusB) [Recall sin(αminusβ)=sinαcosβminuscosαsinβ And sin(α+β)=sinαcosβ+cosαsinβ]= (sinAcosB+cosAsinB)times(sinAcosBminuscosAsinB)= sin2Acos2Bminuscos2Asin2B [Recall sin2α+cos2α=1 From above we can then assume correctly that sin2α=1minuscos2α AND cos2α=1minussin2α] = sin2A(1minussin2B)minussin2B(1minussin2A) = sin2Aminussin2Asin2Bminussin2B+sin2Asin2B = sin2Aminussin2B= 1-cos2A-(1-cos2B) = cos2 B- cos2 A = RHSii)LHS= cos (A+B) cos(A-B) [ cos(A+B) = cos AcosB- sinAsinBCos(A-B) = cosAcosB+ sinAsinB]= cos2 A Cos2 B- sin2 A Sin2 B= cos2 A( 1-sin2 B) - (1- cos2 A) sin2 B= cos2 A- cos2 A sin2 B- sin2 B+ cos2 A sin2 B=cos2 A- sin2 B=1- sin2 A-(1-cos2 B) = cos2 B- sin2 A= RHSTangent formulae for compound anglesi)tan (A + B) = tan A + tan B1-tan A tan Bii)tan (A ndash B) = tan A-tan B1+tan A tan Biii) cot (A + B) = cot Acot B-1cot A+cot B(viii) cot (A ndash B) = cot Acot B+1cot B-cot A

A tan B) Or 1= (tan A+ tanB) (1-tan A tanB) Or tanA + tanB + tanA tanB + 1 = 1 + 1Or tanA (1 + tanB) + (1 + tanB) = 2Or (1 + tanA) (1 + tanB) = 2Example 3 Find the value of sin 15degSolution sin 15deg= sin(45deg-30deg) = sin45degcos 30deg- cos45degsin30deg =(1radic2) (radic32) -(1radic2) (12) = (radic3-1) 2radic2Example 4 If sin A = 1 radic10 and sin B = 1 radic5 where A and B are positive acute angles then what is A + B SolutionWe know that sin (A + B) = sin A cos B + cos A sin B= [1 radic10] [radic(1 minus 1 5)] + [1 radic5] radic(1 minus 1 10)= [1 radic10] [radic4 5] + [1 radic5] [radic9 10]= [1 radic50] times (2 + 3)= 5 radic50 = 1 radic2

sin (A + B) = sin π 4rArrHence A + B = π 4Example 5 If A + B = 225o then find [cot A] [1 + cotA] times [cot B] [1 + cot B]Solution[cot A] [1 + cotA] times [cot B] [1 + cot B] = 1 [(1 + tan A) times (1 + tan B)]=1 [tan A + tan B + 1 + tan A tan B] [ tan (A + B) = tan225o]∵

tan A + tan B = 1minus tan A tan BrArr= 1 [1 minus tan A tan B + 1 + tan A tan B]= 1 2

COMMERCE

CLASSIFICTION OF HUMAN ACTIVITIES-ECONOMIC AND NON-ECONOMIC

Firstly we shall recall the previous class for 5 mins especially for the absentees and for also the rest of the students who were there

Today at first we briefly discuss the earlier portions of the chapter

1Business-It includes all those economic activities which are concerned with production and exchange of goods and services with the object of earning profit Example A factory shop beauty parlour also business enterprises

2Profession ndashThe term profession means an occupation which involves application of specialized knowledge and skills to earn a living For Example Chartered Accountancy medicine law tax consultancy are example of professions

Questions1What are the main features of ProfessionAnswer The main features of a profession are as follows a Specialised body of knowledge-Every profession has a specialised and systematised body of knowledge b Restricted entry- Entry to a profession is allowed only to those who have completed the prescribed education and have the specialised examination c Formal education and training ndashA formal education and training is given to the person who wants to acquire the professional

3Employment-Employment mean an economic activity where people work for others in exchange for some remuneration (salary)The persons who work for others are called lsquoemployeesrsquo The persons or organizations which engage others to work for them are called lsquoemployersrsquoEg A doctor working in a hospital is employment as he is working for a salaryA lawyer may serve as a law officer in a bank

With this we shall proceed with the features of both Profession amp Employment

The main features of a profession are as follow

a Specialised body of knowledge b Restricted entry c Formal education and training d Professional association e Service motive f Code of contact

The main features of an employment are as follows

a In employment a person works for others called employer

b An employee provides personal service

c There is a service agreement or contract between the employee and the employer

d The employee has to obey the order of the employer

e No capital investment is made by the employer

Various examples of Employment are as follows

aA teacher teaching in a school or collegeb An engineer employed in Municipal Corporation of DelhicAn accountant working in the accounts department of a companydA doctor working in a hospital

Note In all the above examples of employment the individual who is involved in each example is working as an employee for a salary under an employer

qualification(MBBSCALLB)d Service motive ndashProfessionals are expected to emphasis service more on their clients rather than economic gain f Code of Conduct-The activities of professionals are regulated by a code of conduct

2 What are the main features of EmploymentAnswer The main features of an employment are as followsa In employment a person works for others called employerb An employee provides personal servicec There is a service agreement or contract between the employee and the employerd The employee has to obey the order of the employere No capital investment is made by the employer

3 Give various Professions and their respective Association are given below

Professions

Professional

Professional association

Medical profession

Doctor Medical Council of India

Law profession

Lawyers Bar Council of India

Accounting Profession

Chartered

The Institute of Chartered Accounts of India( ICAI)

Engineerin Engineers The

g Profession

institute of Engineers (India)

Accounts Basic accounting terms

Today we will give you some questions from the previous study material

Questions6) Define accounting7) What do you mean by debit

and credit8) Explain the types of account9) Define the following terms

a) Assetsb) Capitalc) Purchased) Debtorse) Transactions

10) Name the types of accounts given below

a) Krishnas accountb) Machinery accountc) Royalty accountd) Salary accounte) Furniture accountf) Audit fee account

Economics Basic Economic ConceptsSub topic

UTILITY

Before starting todayrsquos class we shall recall the last class which was about UTILITY AND THE FEATURES OF UTILITY

Now we shall proceed with the further topics of the chapter

Todayrsquos topic from the chapter lsquo Basic Economic Conceptsrsquo will be TOTAL UTILITY amp MARGINAL UTILITYNow let us quickly revise the concept of utility with an example ie goods and services are designed because they have an ability to satisfy human wantsThis feature of being able to satisfy human wants is termed as utility For example we derive utility from WiFi services as it gives us satisfaction by connecting us to our friends and family through social media here consumers derive utility from WiFi services

From the above concept we shall start with todayrsquos topicEconomists have defined TOTAL UTILITY (TU) as the total satisfaction obtained by consuming a given total amount of a good and serviceFor example the total satisfaction obtained from eating 10 mangoes is the total utility of 10 mangoes

MARGINAL UTILITY (MU) is the additional satisfaction derived from each additional unit

Questions1 What is Total Utility (TU)

Answer Total Utility (TU) is the

aggregate of the utility that a consumer derives from the consumption of a certain amount of a commodityTU=MU1+MU2++MUn

2 What is Marginal UtilityAnswer

Marginal Utility (MU) is the additional made to the total utility as consumption is increased by one more unit of the commodityMU= TUn ndashTUn-1

NoteOften economists tend to

subdivide utility into an imaginary unit called UTIL

consumed In this casethe utility obtained from each mango as it is consumed as the MU of that mango It is also defined as the addition made to the total utility when an additional unit is consumed Often economists tend to subdivide utility into an imaginary unit called UTIL

Note As a consumer increases the consumption of a good over period of time the total utility or total satisfaction derived from it increases to appoint and thereafter it decreasesHowever as the consumer keeps on consuming the good the marginal utility or the additional utility derived from it decreases

SubjectBusiness studies

Topic

BUSINESSENVIRONMENT

Summary

Now quickly let us revise the earlier points that we have already done in the last class and let us proceed with the other topics that are there in the chapter

Firstly we will recall the internal and external factors of micro environment and then we shall proceed in details

Meaning and list of internal and external factors

aInternal factorsInternal factors refer to all the factors existing within a business firm The internal factors are considered controllable because the enterprise has control over these factorsFor an example a company can alter its organization structure policies programmes employees physical facilities and marketing mix to suit the changes in the environmentList of internal factors areCorporate culture mission and objectives top management organizations structure company image and brand equity company resources

b External factorsExternal factors refer to those individual and groups and agencies with which a particular business organization comes into direct and frequent contact in the course of its functioningThese individuals and groups are known as STAKEHOLDERS because they have a stake (financial interest ) in the working and performance of the particular business List of external forces (stakeholders)Customers competitors investors suppliersmiddlemen (marketing intermediaries)

Execution 1 What do you mean by internal

factors in micro environmentAnswerInternal factors refer to all the factors existing within a business firm The internal factors are considered controllable because the enterprise has control over these factorsFor an example a company can alter its organization structure policies programmes employees physical facilities and marketing mix to suit the changes in the environment

2 What do you mean by external factors in micro environment

AnswerExternal factors refer to those individual and groups and agencies with which a particular business organization comes into direct and frequent contact in the course of its functioningThese individuals and groups are known as STAKEHOLDERS because they have a stake (financial interest) in the working and performance of the particular business

3Who are stakeholdersSTAKEHOLDERS are individuals and groups who have a stake (financial interest ) in the working and performance of the particular business 4Discuss the internal factors in briefa Corporate CultureThe values beliefs and attitudes of the founders and top management of the company exercise

financers publics

customers

suppliersfinancers

competitors

middlemen

publics

Fig STAKEHOLDERS OF A COMPANY

Apart from micro environment the other main dimension of business environment isMacro environment Macro environment refers to the general environment or remote environment within which a business firm and forces in its micro environment operateA company does not directly or regularly interact with the micro environmentTherefore macro environment is also known as indirect action EnvironmentThe macro environment forces are less controllable than the micro forces

Macro environment consists of the following components

POLITICAL AND LEGAL ENVIRONMENT

ECONOMIC SOCIAL AND ENVIRONMENT

CULTURAL

ENVIRONMENT

TECHNOLOGICAL ENVIRONMENT

a strong influence on what the cmpaany stands for how it does things and what it considers importantbMission and objectivesThe business philosophy and purpose of a comoany guide it prioritiesbusiness strategiesproduct market scope and development scope

cTop management structurethe composition of board of directors the degree of professionalization of management and the organizational structure of a company have important bearing on its business decisions

dPower structureThe internal power relationship between the board of directors and the chief executive is an important factor

eCompany image and brand equityThe image and brand equity of the company play a significant role in raising finance forming alliance choosing dealers and suppliers launching new products entering foreign markets

5 What is Macro environmentAnswerMacro environment refers to the general environment or remote environment within which a business firm and forces in its micro environment operateA company does not directly or regularly interact with the micro environmentTherefore macro environment is also known as indirect action EnvironmentThe macro environment forces are less controllable than the micro forces 6 What are the components of macro environmenta Political and legal environmentb Economic environmentc Social and cultural environmentd Technological environment

BUSINESS FIRM

Fig COMPONENTS OF MACRO ENVIRONMENTPolitical science

Introduction to political science

Comparative politics and itrsquos scope Comparative politics is the second major dimension of political scienceIt is also a very vast area of study and a very large number of political scientists even treat it as an autonomous area of study within the board ambit of political scienceScope of comparative politics-

1 All political structures -Comparative politics includes the study of all structures formalnon formal governmental and extra governmental which are directly or indirectly involved in politics in all the countries of the world

2 Functional studies- Comparative politics seeks to study politics less from the point of view of the legal institutions in terms of their powers and move from the point of view of their functions which constitute the political process and their actual Operation in the environment

3 Study of political behaviour- Another important part of its scope is the study of the actual behaviour of the people in the process of politics

4 Study of similarities and differences- comparative politics also undertakesan analysis of the similarities and differences among political process and functions

5 Study of all political systems -comparative politics seeks to analyse the actual behaviour and performance of all political systems western as well as non western

6 Study of the environment and infrastructure of politics-The study of politics demands a study of the psychological sociological economic and anthropological environment in fact the social environment as a whole in which each political system operates

7 Study of political culture- political culture is composed of attitudesbeliefs emotions and values of a society that relate to the political system or politics

8 Study of political participation- Political participation is a universal processThe only difference is that while in some states it is limited in others it is wider

9 Study of political process- political

Answer the following questions-

What is comparative politics

What are the scope of comparative politics

Homework- learn

processes like decision makingpolicy making judicial process leadership recruitment process and others are always at work in all political systems

The scope of comparative politics is very comprehensive It includes everything that falls within the area of political activity and political process

History CAMBRIDGE VIEW ABOUT

THE PARTITION

AND REFUTATION

OF CAMBRIDGE

VIEW

Cambridge view about the Partition The Cambridge school of historians have interpreted that opposition to partition scheme was made entirely by the elitist groups They hold the view that Lord Curzon planned to partition the Bengal for administrative purposeREFUTATION OFCAMBRIDGE VIEW The Rationalist historians have rejected the interpretations of the Cambridge School of historians on various grounds

1 QUESTION State different views of historians regarding Partition of Bengal

ANSWER Cambridge historians believed that Lord Curzon partitioned Bengal for administrative reasons only and not for the political motive The Middle class elitist group protested because of their petty interest The Hindu zamindars protested as they have to spend more money for managing their estatesThe lawyers of Calcutta High court feared to lose their clientBut according to the nationalist Historians was-

2- The ultimate object of Lord Curzon was to crush the unity of Bengal politicians

3- If Bengal becomes a separate province Bengali speaking 16 million people of western part would become minority under Hindi speaking people of Bihar and Oriya speaking people of Orissa

4- The bureaucrats expected that the protest movement would die down quickly

5- Lord Curzon used the Muslim community in his political game

6- Idealism had great contribution in the protest against partition

7- The people of the every section of society were affected by the partition of Bengal

Computer Science

Numbers Convertion of dcimal number to octal numberThe decimal numeral system is the standard system for denoting integer and non-integer numbers It is the extension to non-integer numbers of the Hindu-Arabic numeral system For writing numbers the decimal system uses ten decimal digits a decimal mark and for negative numbers a minus sign - The decimal digits are 0 1 2 3 4 5 6 7 8 9 the decimal separator is the dot in many countries

The octal numeral system or oct for short is the base-8 number system and uses the digits 0 to 7 Octal is sometimes used in computing instead of hexadecimal perhaps most often in modern times in conjunction with file

permissions under Unix systems It has the advantage of not requiring any extra symbols as digits It is also used for digital displays

Follow these steps to convert a decimal number into octal form

1 Divide the decimal number by 82 Get the integer quotient for the next iteration (if the number will not divide equally by 8 then round down the

result to the nearest whole number)3 Keep a note of the remainder it should be between 0 and 74 Repeat the steps until the quotient is equal to 05 Write out all the remainders from bottom to top This is the solution

For example if the given decimal number is 8453

Division Quotient Remainder

8453 8 1056 5

1056 8 132 0

132 8 16 4

16 8 2 0

2 8 0 2

Then the octal solution is 20405

Subject Eng Literature (The Tempest ndash William Shakespeare) Topic Act I Scene 1 Lines 33 to 67 (End of scene) Date 16th April 2020 (4th Period)

[Students should read the original play and also the paraphrase given in the school prescribed textbook]Summary Questions amp Answers

[SUMMARY OF THE ENTIRE SCENE]

o The play starts with the scene of a severe storm at sea Alonso (King of Naples) Sebastian (Alonsorsquos brother) Ferdinand (Alonsorsquos son) Gonzalo Antonio (the usurping Duke of Milan) are in a ship in the midst of the storm

o The mariners are trying their best to control the vessel from running aground and are totally following the orders of their Master the Boatswain They have scant success

o The mariners become extremely unhappy and annoyed when most of the passengers arrive on the deck thereby hampering their effort to save the ship There is serious confrontation between them and the passengers who are part of the Kingrsquos entourage

o The mariners could not save the ship

SUMMING-UP

(i) Vivid description of the scene which gives a realistic description of terror and confusion of a tropical storm

(ii) Shows Shakespearersquos accuracy of knowledge in describing the naval operations and also matters of seamanship

(1) GONZALO Ill warrant him for drowning (L 45-57)

though the ship were no stronger than a nutshell and as leaky as an unstanched

wenchBOATSWAIN Lay her a-hold a-hold Set her two courses Off to

sea again lay her offMARINERS All lost To prayers to prayers All lostBOATSWAIN What must our mouths be coldGONZALO The king and prince at prayers Lets assist them

For our case is theirsSEBASTIAN Im out of patienceANTONIO We are merely cheated of our lives by drunkards

This wide-chopped rascal - would thou mightst lie drowning the washing of ten tides

(a) What does Antonio say at the insolent manners of the boatswain just before the given passage

Being irritated at the insolent manners of the boatswain just before the given extract Antonio the Duke of Milan calls him a worthless dog son of a woman without any morals an arrogant and disrespectful noisemaker He says that the boatswain deserved to be hanged(b) What statement does Gonzalo repeat about the boatswain

Gonzalo shows his faith that the boatswain is not destined to die by drowning He is destined to be hanged and nothing can alter this decree of destiny He says that even if the ship was as frail as a nutshell the boatswain could not be drowned for his destiny was to be hanged(c) What do the passengers do when they have lost all hope of their survival

When the passengers have lost all hope of survival they take

(iii) The opening scene justifies the title ndash The Tempest

UNANSWERED QUESTIONS

(i) The King always travels with his entire fleet including his soldiers Where were the other ships

(ii) Why was the ship in that area Where was it coming from or going where

(iii) The ship broke apart What happened to those who were in the ship

(We shall get the answer to the above questions as the play progresses)

leave of life with fervent prayers The mariners take their last hearty drink and are ready for death(d) What blame does Antonio put upon the mariners and the boatswain Antonio rebukes the mariners that these drunkards have brought them to the present crisis by neglecting their duties He blames them saying that they are going to lose their lives entirely for the negligence of the boatswain and his fellows(e) What does Antonio say while cursing the boatswain

Antonio gives vent to his wrath upon the boatswain in particular He calls the boatswain a wide-mouthed rascal who deserves to be hanged on the sea-shore at low water mark so that ten tides might wash over his body and take out of him all the liquor that he has been drinking

Class XIISubject Topic Summary ExecutionHistory Topic

1 1935 ACT AND WORKING OF PROVINCIAL AUTONOMYCONGREE AND OTHER MINISTERSSUB TOPIC GOVERNMENT OF INDIA ACT1935

Government of India Act 1935 This act established a lsquoFederation of Indiarsquo made of British Indian provinces and Indian states and provided for autonomy with a government responsible to the elected legislature in every provinceThis act introduced abolition of Diarchy at provinces The entire provincial administration was introduced to the responsible ministers who were controlled and removed by the provincial legislature The provincial autonomy means two things First The provincial governments were wholly responsible to the provincial legislature Secondly Provinces were free from outside control and interference in the large number of matters The act divided the powers between the centre and provinces in terms of three lists- Federal list( for centre) Provincial list (for province) and concurrent list (for both) Residuary powers were given to the viceroy In the election under the government of India Act the Congress swept the poll the mandate of the people came in favour of the congress so far as general Hindu seats were concerned The Congress did not get a single Muslim seates in Bombay CP UP Sind and BengalIn five provinces Congress had yhe clear majority In BengalNWFPAssam and Bombay Congress emerged as a single largest partyOn the other side the performance of the Muslim League was badThus the Congress formed ministers in 7 provinces out of 11 provinces Coalition ministry was also formed in two other provincesOnly BENGAL AND Punjab had non- congress ministries

1 QUESTION What was the main change introduced by the Government of India ActANSWER a) The Act gave more

autonomy to the provinces b) Diarchy was abolished at the

provincial levelsc) The Governor was the head of

the executived) There was a council of

ministers to advise him The ministers were responsible to the provincial legislatures who controlled them The legislature could also remove the ministers

e) The Governors still retained special reserve powers

2 QUESTION Why did the federal scheme introduced by the Government of India Act 1935 never come into operation

ANSWER The Federal structure of the Government of India was to be composed with the Governor General and Council of ministers The Federal legislature was to be Bicameral legislature- The council of states and the House of Assembly The ministers were to be chosen by the Governor general and they were to hold the office during his pleasure

The provinces of British India would have to join the federation but this was not compulsory for the princely states

This federation never materialised because of the lack of support from the required number of

princely statesThis act was refused and

rejected by the princes the Congress and the Muslim League

Thus both Congress and the League participated in the election of 1937 Thus the federal part was never introduced but the provincial part was put into operations

Bengali 2nd

Language

াচেরর পরাথCনা(কহিতা )

াচেরর পরাথCনা কহিতাটি কহি (ঙখ দেঘাচে4র দো আচো য কহিতায় াচেরর পতর হমায়ন কঠিন দেরাচেগ আxানত ার ঈশবর া আললার কাচেছ পরাথCনা কচেরচেছন তার পচেতরর ীন হিফহিরচেয় হিচেত এই কহিতায় ার পচেতরর ীন হিভbা দেচেয়চেছন ারার এমনহিক হিনচের ীন হিসCচেনর হিহিনমচেয় হিতহিন তার দেছচের ীন হিফচের দেপচেত দেচেয়চেছন তার দেছচের এই দেরাচেগর ন য হিতহিন হিনচেচেকই ায়ী কচেরচেছন তার হিনচের করা পাপচেকই হিতহিন ায়ী কচেরচেছন এছাা রানৈনহিতক ও আথCসামাহিক অসথার কথা তচে ধরা চেয়চেছ এই কহিতায় ার তার হিনচের পাপ কমCচেকই ায়ী কচেরচেছ ার অন যায় ভাচে দেপহি((হিকতর মাধ যচেম অপররা য কচেরচেছ আর এই অন যায় কাচের ন যই তার পহিরাচের হিপযCয় এচেসচেছ দে এক পরকার মানহিক নধন ইহিতাচেসর ার হিপতা চেয় সবাভাহিকভাচে ভাচোাসা দে মমতা দেথচেক মকত চেত পাচেরনহিন তাই হিপতা চেয় আললা া ভগাচেনর কাচেছ পতর হমায়চেনর পরানহিভbা দেচেয়চেছন ার আললা া ভগাচেনর কাচেছ াহিনচেয়চেছন তার হিনচের ীন হিসCন হিচেত হিতহিন রাী তার হিহিনমচেয় পচেতরর ীন হিফচের দেপচেত দেচেয়চেছন াচেরর হিপতসভ হিচেকর কথা এই কহিতায় ফটিচেয় দেতাা চেয়চেছ হিপতা পচেতরর হিরাহিরত মান নধচেনর কথা তচে ধরা চেয়চেছ

হিচে(4 হিকছ াইচেনর তাৎপযC১) ldquoদেকাথায় দেগ ওর সবচছয দেৌন দেকাথায় কচেরায় দেগাপন bয়ldquoউততর) াচেরর পতর হমায়ন কঠিন দেরাচেগ অসসথ তাই তার দেযৌন াহিরচেয় যাচেচছ এই দেরাচেগ তাচেক দেগাপচেন কচেরকচের াচেচছ তার সক (হিকত ধীচের ধীচের bয় চেচছ তাই হিপতা চেয় ার আললার কাচেছ হমায়চেনর পরান হিভbা দেচেয়চেছন২) ldquoাগাও (চেরর পরাচেনত পরানতচের ধসর (ন দেযর আান গানldquoউততর) াচেরর পতর হমায়ন কঠিন দেরাচেগ আxানত তাই ার আ দে(াচেক মমCাত (চেরর পচেথ পরানতচের আান গান ধবহিনত দোক দেসই আান গান আললার কাচেছ দেযন চে যায় আললা দেযন এই আহিতC শচেন পচেতরর ীন হিফহিরচেয় দেয় ৩)ldquoনাহিক এই (রীচেরর পাচেপর ীানচেত দেকানই তরারণ দেনই ভহি4চেতরldquoউততর) হমায়চেনর অসসথতার ন য ার হিনচেচেকই ায়ী কচেরচেছন কারন ার অচেনক রা য অন যায় ভাচে কচেরচেছ তাই তার এই পাপ কাচের ন য তার ঘচের আ হিপ এচেসচেছ এই অন যায় কাচের ন য তার মহিকত দেনই তাই ার আললার কাচেছ এই পাপ কাচেযCর ন য bমা পরাথM

Hindi 2ndlang

-ासी(जयशकर परसा-)

-ासी जयशकर परसा- की एक ऐसी कहानी ह जिजसम भारतीय ससकनित और राषटरीयता का सवरगजीतहोता ह इस कहानी म इरावती एक निहद कनया ह जिजस मलअचछो न मलतान की लट म पकडा और -ासी बना दि-या उस 500 दि-न -कर काशी क एक महाजन न खरी-ा दसरी -ासी निफरोजा ह वह गलाम ह निफरोजा को छडान क कतिलए अहम- को 1000 सोन क कतिसकक भजन थ जो अभी तक नही आए थ राजा साहब कठोर होत हए भी निफरोजा को निबना धनराकतिश क कतिलए उस म कर -त ह वनिफरोजा को अहम- को समझान की बात कहत हकहानी क अत म हम -खत ह निक इरा वती और जाटो क सर-ार बलराज का मिमलन होता हअहम- को यa म मार दि-या जाता ह वहा निफरोजा की परसननता की समामिध बनती ह वहा एक फल चढती ह और डीजल आती ह निफरोजा उस समामिध की आजीवन -ासी बनी रहती हलखक अपन उददशय अथात -ास परथा पर परकाश डालन और इस परथा क कारण होन वाल -ातो क दखो को दि-खान म पणता सफल हए ह

helliphellipContinue to next

Biology Reproductio Today we will discuss about vegetative Q1 Name some vegetative propagules

n in Organisms

propagation of plants The process of multiplication in which fragments of plant body function as propagule and develop into new individual is called vegetative propagation The units of such propagation are runner rhizome tuber bulb etc

and the speciesinvolvedVegetative propagules

Parts involved

Bulb StemBulbil BulbilRhizome Stem Runner Stem Tuber Stem Offset Stem Leaf buds Leaves Suckers Stem

Corns Stem stolon

Q2 State advantages of vegetative propagation

i) Rapid methodii) Sure and easy methodiii) Useful in plants that cannot

produce viable seeds or long seed dormancy

iv) Maintains purity of raceQ 3 Banana fruit is said to be parthenocarpic where as turkey is said to be parthenogenetic WhyBanana develops without fertilization from an unfertilized ovary thus is parthenocarpicIn turkey the ovum or female gamete developinto a new chick without fertilization thus isparthgenetic

Q4 Why is water hyacinth is called as a ldquoTerror of Bengalrdquo Water hyacinth can

propagatevegetatively all over the water body in a short per short period of time This resulted increased biochemicaloxygen oxygen demand of water body causing mortalityof fishes It is very difficult to get rid off them Thus known as terror of Bengal

Chemistry

Solid state GENERAL CHARACTERISTICS OF SOLID STATEIn nature the particular state of matter is governed by two opposing forces at given set of temperature and pressure These forces are intermolecular force of attraction and thermal energy If intermolecular force of attraction is high as compared to thermal energy particles remains in closest position

Intext QuestionsQ1 Classify the following solids as crystalline and amorphous Sodium chloride quartz glass quartz rubber polyvinyl chloride Teflon

A1 Crystalline

and hence very less movement in particles is observed In this case solid state is the preferred state of matter

Let us revise the general characteristics of solid

i) Fixed mass volume and shape

ii) Strong intermolecular force of attraction

iii) Least intermolecular space

iv) Fixed position of constituent particles

v) Incompressible and rigid

Q2 what type of interactions hold the molecules together in a polar molecular solid[CBSE 2010]A2 The molecules in a solid are held together by van der Waals forces The term van der Waals forces include hydrogen bonding dipole-dipole attraction and London dispersion forces All molecules experience London dispersion forces In addition polar molecules can also experience dipole-dipole interactions So the interactions that holds the molecule together in polar molecular solid are London dispersion force and dipole-dipole interactionsQ3 Write a feature that will distinguish a metallic solid from an ionic solid [CBSE 2010]A3 Metals are malleable and ductile whereas ionic solid are hard and brittle Metallic solid has typical metallic lustre But ionic solid looks dullQ4 Write a point of distinction between a metallic solid and an ionic solid other than metallic lustre [CBSE 2012]A4 Metals are malleable and ductile whereas ionic solid are hard and brittleQ5 Write a distinguish feature of metallic solid [CBSE 2010]A5 The force of attraction in

solid Sodium chloride Quartz Amorphous solid Quartz glass rubber polyvinyl chloride Teflon Q2 why glass is considered as super cooled liquidA2 Glass shows the tendency to flow at slower rate like liquid Hence they considered as super cooled liquidQ3 why the window glass of old buildings show milky appearance with timeA3 Glass is an amorphous solid Amorphous solid has the tendency to develop some crystalline character on heating Due to heating in day over the number of years glass acquires some crystalline character and show milky appearanceQ4 why the glass panes fixed to window or doors of old building become slightly thicker at bottomA4 Glass is super cooled liquid It has the tendency to flow down very slowly Due to this glass pane becomes thicker at the bottom over the timeQ5 Sodium chloride is a crystalline solid It shows the same value of refractive index along all the direction TrueFalse Give reasonA5 FalseCrystalline solid shows anisotropy in properties That is it shows different values for the given physical property in different direction All the crystalline solids show anisotropy in refractive index Therefore sodium chloride will show different values of refractive index on different directions

Q6 Crystalline solid are anisotropic in nature What does this statement means

between the constituent particles is special kind of electrostatic attraction That is the attraction of positively charged kernel with sea of delocalized electronsQ6 which group of solid is electrical conductor as well as malleable and ductile [CBSE 2013]A6 Metallic solidQ7 why graphite is good conductor of electricity although it is a network (covalent solid)A7 The exceptional property of graphite is due to its typical structure In graphite each carbon is covalently bonded with 3 atoms in same layer The fourth valence electron of each atom is free to move in between different layersThis free electron makes the graphite a good conductor of electricity

[CBSE 2011]A6 Anisotropy is defined asrdquo Difference in properties when measured along different axis or from different directionsrdquo Crystalline solid show different values of some of the physical properties like electrical resistance refractive index etcwhen measured along the different directions The anisotropy in crystalline solid arises due to the different arrangement of particles in different directions

Math Function Composition of functions Think of an industrial plant that produce bottles of cold drinks first there is the operation (or function) f that puts the cold drink inside the bottle followed by the opeartion g that close the bottle with the capThis leads to the following definitionDefinition Let f A rarr B and g B rarr C be two functions Then the composition of f and g denoted by gof is defined as the function gof A rarr C given by gof(x) = g(f (x)) forall x isinA

Definition A function f X rarr Y is defined to be invertible if there exists a function g Y rarr X such that gof = IX and fog = IY The function g is called the inverse of f and is denoted by f -1

Thus if f is invertible then f must be one-one and onto and conversely if f is one-one and onto then f must be invertible This fact significantly helps for proving a function f to be invertible by showing that f is one-one and onto specially when the actual inverse of f is not to be determined

Example 1 Let f 2 3 4 5 rarr 3 4 5 9 and g 3 4 5 9 rarr 7 11 15 be functions defined as f(2) = 3 f(3) = 4 f(4) = f(5) = 5 and g (3) = g (4) = 7 and g (5) = g (9) = 11 Find gofSolution We have gof(2) = g (f(2)) = g (3) = 7 gof(3) = g (f(3)) = g (4) = 7gof(4) = g (f(4)) = g (5) = 11 and gof(5) = g (5) = 11Example 2 Find gof and fog if f R rarr R and g R rarr R are given by f(x) = cos x and g (x) = 3x2 Show that gof ne fogSolution We have gof(x) = g(f(x))=g(cosx) = 3 (cos x)2

= 3 cos2 x Similarly fog(x)=f(g (x))= f(3x2)= cos (3x2) Note that 3cos2 x ne cos 3x2 for x = 0 Hence gof ne fogExample 3 Show that if f A rarr B and g B rarr C are onto then gof A rarr C is also ontoSolution Given an arbitrary element z isin C there exists a pre-image y of z under g such that g (y) = z since g is onto Further for y isin B there exists an element x in A with f(x) = y since f is onto Therefore gof(x) = g (f(x)) = g (y) = z showing that gof is onto Example 4 Let Y = n2 n isin N sub N Consider f N rarr Y as f(n) = n2 Show that

f is invertible Find the inverse of fSolution An arbitrary element y in Y is of the form n2 for some n isin N This implies that n =radicy This gives a function g Y rarr N defined by g (y) =radicy Nowgof (n) = g (n2)=radicn2 = n and fog (y) =f(radicy) = (radicy) 2 y which shows that gof=IN and fog= IY Hence f is invertible with f -1 = g

Political Science

Constitution of India-The Preamble

Summary

Objective of the state-To secure equality of status and of opportunity To promote fraternity among all the citizens To assure the dignity of the individuals and Unity and integrity of the nation

Justice-Justice stands for rule of law absence of arbitrariness and a system of equal rights freedom and opportunities for all in a society India seeks social economic and political justice to ensure equality to its citizens

Liberty-Liberty implies the absence of restraints or domination on the activities of an individual such as freedom from slavery serfdom imprisonment despotism etc The Preamble provides for the liberty of thought expression belief faith and worship

Equality-Equality means the absence of privileges or discrimination against any section of the society The Preamble provides for equality of status and opportunity to all the people of the country

Fraternity-The Preamble declares that fraternity has to assure two thingsmdashthe dignity of the individual and the unity and

Execution

Answer the following questions-

Short notes-1 Equality2 Fraternity3 Justice4 Liberty

Homework-Learn

integrity of the nation The word integrity has been added to the Preamble by the 42nd Constitutional Amendment (1976)

Business studies

Human resource management (chapter 1)

On the day of 1504 2020 I have discussed with you the managerial functions and procurement functions of HRM

Today weare going to discuss about the development function integration functions and maintenance function

Development functions-HRM improves the knowledge skills attitude and values of employees so that they the present and future jobs more effectively it includes

1) Development functions of HRM

a) Performance appraisal = It implies systematic evaluation of employees with respect to their performance on the job and their potential for development

b) Training =It is the process by which employees learn knowledge skills and attitudes to achieve organisational and personal goals

c) Executive development = It is the process of developing managerial talent through appropriate program

2) Integration functionsa) HRM reconcile the goals of

organisation with those of its members through integrating function

b) HRM tries to motivate employees to various financial and non financial incentives provided in job specification etc

3) Maintenance functiona) HRM promote and protect the

physical and mental health of employees by providing several types of benefits like housing medical aid etc

b) It Promote Social security measures to employees by providing provident fund pension gratuity maternity benefits

SubjectCOMMERCE

Topic

BUSINESSENVIRONMENT

Summary

Now quickly let us revise the earlier points that we have already done in the last class and let us proceed with the other topics that are there in the chapter

Firstly we will recall the internal and external factors of micro environment and then we

Execution 3 What do you mean by internal factors

in micro environmentAnswerInternal factors refer to all the factors existing within a business firm The internal factors are considered controllable because the enterprise has control over these factors

Development FunctionsPerformance AppraisalTrainingExecution Development

shall proceed in details

Meaning and list of internal and external factors

aInternal factorsInternal factors refer to all the factors existing within a business firm The internal factors are considered controllable because the enterprise has control over these factorsFor an example a company can alter its organization structure policies programmes employees physical facilities and marketing mix to suit the changes in the environmentList of internal factors areCorporate culture mission and objectives top management organizations structure company image and brand equity company resources

b External factorsExternal factors refer to those individual and groups and agencies with which a particular business organization comes into direct and frequent contact in the course of its functioningThese individuals and groups are known as STAKEHOLDERS because they have a stake (financial interest ) in the working and performance of the particular business List of external forces (stakeholders)Customers competitors investors suppliersmiddlemen (marketing intermediaries)financers publics

customers

suppliersfinancers

For an example a company can alter its organization structure policies programmes employees physical facilities and marketing mix to suit the changes in the environment

4 What do you mean by external factors in micro environment

AnswerExternal factors refer to those individual and groups and agencies with which a particular business organization comes into direct and frequent contact in the course of its functioningThese individuals and groups are known as STAKEHOLDERS because they have a stake (financial interest) in the working and performance of the particular business

3Who are stakeholdersSTAKEHOLDERS are individuals and groups who have a stake (financial interest ) in the working and performance of the particular business 4Discuss the internal factors in briefa Corporate CultureThe values beliefs and attitudes of the founders and top management of the company exercise a strong influence on what the cmpaany stands for how it does things and what it considers importantbMission and objectivesThe business philosophy and purpose of a comoany guide it prioritiesbusiness strategiesproduct market scope and development scope

cTop management structurethe composition of board of directors the degree of professionalization of management and the organizational structure of a company have important bearing on its business decisions

dPower structureThe internal power relationship between the board of directors and the chief executive is an important factor

e Company image and brand equityThe image and brand equity of the company play a significant role in raising finance forming alliance choosing dealers and suppliers launching new products entering foreign markets

5 What is Macro environmentAnswerMacro environment refers to the general

competitors

middlemen

publics

Fig STAKEHOLDERS OF A COMPANY

Apart from micro environment the other main dimension of business environment isMacro environment Macro environment refers to the general environment or remote environment within which a business firm and forces in its micro environment operateA company does not directly or regularly interact with the micro environmentTherefore macro environment is also known as indirect action EnvironmentThe macro environment forces are less controllable than the micro forces

Macro environment consists of the following components

POLITICAL AND LEGAL ENVIRONMENT

ECONOMIC SOCIAL AND ENVIRONMENT

CULTURAL

ENVIRONMENT

TECHNOLOGICAL ENVIRONMENT

Fig COMPONENTS OF MACRO ENVIRONMENT

environment or remote environment within which a business firm and forces in its micro environment operateA company does not directly or regularly interact with the micro environmentTherefore macro environment is also known as indirect action EnvironmentThe macro environment forces are less controllable than the micro forces 6 What are the components of macro environmenta Political and legal environmentb Economic environmentc Social and cultural environmentd Technological environment

Computer Science

Logic gates

Digital systems are said to be constructed by using logic gates These gates are the AND OR NOT NAND NOR EXOR and EXNOR

BUSINESS FIRM

gates The basic operations are described below with the aid of truth tables

AND gate

The AND gate is an electronic circuit that gives a high output (1) only if all its inputs are high A dot () is used to show the AND operation ie AB Bear in mind that this dot is sometimes omitted ie ABOR gate

The OR gate is an electronic circuit that gives a high output (1) if one or more of its inputs are high A plus (+) is used to show the OR operationNOT gate

The NOT gate is an electronic circuit that produces an inverted version of the input at its output It is also known as an inverter If the input variable is A the inverted output is known as NOT A This is also shown as A or A with a bar over the top as shown at the outputs The diagrams below show two ways that the NAND logic gate can be configured to produce a NOT gate It can also be done using NOR logic gates in the same way

NAND gate

This is a NOT-AND gate which is equal to an AND gate followed by a NOT gate The outputs of all NAND gates are high if any of the inputs are low The symbol is an AND gate with a small circle on the output The small circle represents inversion

NOR gate

This is a NOT-OR gate which is equal to an OR gate followed by a NOT gate The outputs of all NOR gates are low if any of the inputs are highThe symbol is an OR gate with a small circle on the output The small circle represents inversion

EXOR gate

The Exclusive-OR gate is a circuit which will give a high output if either but not both of its two inputs are high An encircled plus sign ( ) is used to show the EOR operation

EXNOR gate

The Exclusive-NOR gate circuit does the opposite to the EOR gate It will give a low output if either but not both of its two inputs are high The symbol is an EXOR gate with a small circle on the output The small circle represents inversion The NAND and NOR gates are called universal functions since with either one the AND and OR functions and NOT can be generated

Note A function in sum of products form can be implemented using NAND gates by replacing all AND and OR gates by NAND gates A function in product of sums form can be implemented using NOR gates by replacing all AND and OR gates by NOR gates

Logic gate symbols

Table 2 is a summary truth table of the inputoutput combinations for the NOT gate together with all possible inputoutput combinations for the other gate functions Also note that a truth table with n inputs has 2n rows You can compare the outputs of different gates

Logic gates representation using the Truth table

Example

A NAND gate can be used as a NOT gate using either of the following wiring configurations

Subject Eng Literature (The Tempest ndash William Shakespeare) Topic Act III Scene 3 Lines 53 to 110 (End of the scene) Date 16th April 2020 (2nd Period)

[Students should read the original play and also the paraphrase given in the school prescribed textbook]Summary Questions amp Answers

o Seeing this strange scene all are inclined to believe the tales told by travelers that there truly are ldquounicornsrdquo and ldquothe phoenixrsquo thronerdquo

o As they are about to sit down to the feast the banquet is snatched away by a harpy (Ariel disguised) A spiritrsquos voice (Arielrsquos voice) denounces Alonso Sebastian and Antonio with particular

1 ARIEL You are three men of sin whom Destiny

(Line 53-58)That hath to instrument this

lower world And what is int the never-surfeited sea

Hath caused to belch up you and on this island

Where man doth not inhabit you rsquomongst men

Being most unfit to live I have made you mad

reference to their crime in expelling Prospero from Milan They have not received any punishment for their deed earlier but the time for their punishment has arrived Upon Alonso it pronounces ldquolingering perdition worse than deathrdquo from which there is no remedy except through sincere repentance Ariel then vanishes in thunder and the shapes enter again and carry away the table

o Prospero watching invisibly is very pleased with the performance of Ariel and his (Prosperorsquos) ldquomeaner ministersrdquo All his enemies are now in his power and are in a fit of desperation He then leaves them and goes to see how Ferdinand and Miranda are getting on

o Alonso is now much humbled and penitent with the after effect of the spiritrsquos denunciation of his crimes He believes that his son is lost forever After this all disperse being stricken mad by the speech of the spirit

o Gonzalo fearing that they may do violence to themselves or to one another follows them and bid others to follow

(a) To whom does Ariel disguised as a harpy call the three sinners What game did Fate of Destiny play with

them

The three sinners called by Ariel are Alonso Sebastian and Antonio It was Destiny which had caused the ocean to cast the three sinners on the shore Though the ocean is all the time devouring whatever appears on its surface and is never satisfied with its continual swallowing of the ships and men in the present case the ocean had cast these three sinners on the shore without killing them

(b) Who had jointly been responsible for the conspiracy against Prospero What is Prosperorsquos purpose behind all this

Three men Alonso Sebastian and Antonio had jointly

been responsible for the conspiracy against Prospero They had driven out Prospero form Milan Prosperorsquos purpose is to make these three sinners realize the wrong they had done He wants them to repent for their criminal deeds because repentance leads to self-esteem(c )What does Ariel (the harpy) tell Alonso and his companions when they take out their swords to attack him

Seeing them drawing their swords Ariel (harpy) tells them that he and his companions are the instruments of destiny and that it is not possible for human beings to do them any injury He says that the swords of human beings can not injure even a minute part of his feathers Their swords are as ineffective against him and his companions as against the wind or the water

(d) Give the explanatory meanings of the following expressions in the context of the above extract

(i)Never surfeited (ii) Belch up (iii) lsquomongst men

(i) Never surfeited never led to satisfaction

(ii) Belch up cast ashore(iii) lsquomongst men in human

society2

I and my fellows (Line 60-65)

Are ministers of Fate The elementsOf whom your swords are tempered may as wellWound the loud winds or with bemocked-at stabsKill the still-closing waters as diminishOne dowl thats in my plume

IMPORTANT PASSAGES EXPLAINED

The elements

(Line 61-66)Of whom your swords are tempered may

as wellWound the loud winds or with

bemocked-at stabs

(a) Who is lsquoIrsquo Who are his lsquofellowsrdquo

lsquoIrsquo is referred to Ariel in disguise of a harpy His lsquofellowsrsquo are other spirits serving Prospero the real Duke of Milan who has acquired supernatural powers after being banished from his Dukedom Prospero has settled in this uninhabited island

(b) What are the elements that have temperrsquod the swords Why will it not work against the speaker

The swords (of Alonso and his companions) are tempered by metal (steel) which is taken out of the earth and refined by

Kill the still-closing waters as diminishOne dowl thats in my plume My fellow

ministersAre like invulnerable

In these words Ariel reminds the King and his companions of the utter futility of drawing swords against himself and his fellows Ariel drives Alonso Antonio and Sebastian the three men of sin to desperation ndash a state in which men do violence to themselves They draw swords to strike Ariel But Ariel reminds them that he and the other spirits are the ministers of destiny and nothing can wound them The steel of which their swords are made of may cut the wind or water which being divided always closes up again Even supposing that such things may be possible it is quite impossible that their swords will cut one feather in their plume They are incapable of being wounded by any sword of man Hence it is foolish on their part to attempt to strike at Ariel and his fellow-spirits

For which foul deed

(Line 72-75)The powers delaying not forgetting

haveIncensed the seas and shores yea all the

creatures Against your peace

Ariel enters like a harpy and remaining invisible tells Alonso Sebastian and Antonio that he and other harpies are the agents of Destiny appointed to carry out her decrees He tells them that their punishment for the crime against Prospero which has been so long deferred is now to fall upon them He reminds them that they had expelled Prospero from Milan and set him and his innocent child adrift on the sea and that the sea had paid them back for their sin by the shipwreck and by the calamities they have suffered He tells them that the powers above which did not forget this mean treachery but only deferred the punishment have now engaged the seas and the shores and all living beings including him and his comrades against them The very elements and supernatural agency Ariel adds have taken up the avenging of their crime against Prospero

the action of fire It may cut the wind or water which being divided always closes up again

The sword will not work against the spirits and the harpy because they are the ministers of destiny and nothing can wound them nor it will cut a single feather in their plume

(c )What is the meaning of lsquodowlrsquo in the last line

The term lsquodowlrsquo means a filament or the smallest part of a feather In this context Ariel in disguise of harpy says that their sword cannot even damage the smallest filament of their (Arielrsquos and other spirits) feathers as they are incapable of being wounded by any sword of man

(d) What does the speaker remind the listeners about

Ariel in disguise of harpy reminds Alonso the King of Naples Sebastian Alonsorsquos brother and Antonio the present Duke of Milan and the treacherous brother of Prospero as they being three men of sin He even reminds them that their punishment for their crime against Prospero which has been so long deferred now falls upon them He reminds them that they have expelled Prospero from Milan and has set him along with his innocent infant daughter adrift on the sea So the sea has paid them back for their sin by their shipwreck and the calamities they have suffered since then The harpy rebukes Alonso of his sin that has incensed the Gods and has deprived him of his son as a punishment

(e) How do they respond

When Ariel in disguise of a harpy reminds Alonso Sebastian and Antonio of their past misdeeds and sin Alonso has a look of terror and confusion in his eyes He utters the words of sincere repentance wrung out of his conscience-stricken heart It appears to him that all the elements of nature the sea-waves the wind and the thunder proclaiming a loud voice in the name of Prospero and the crime Alonso has committed against him They are calling upon him to repent There is a deep storm raging in Alonsorsquos breast and the echoes of that storm are ringing in his ears like a clear note of wind-instrument A note of denunciation of Alonsorsquos crime leaves him much humbled and penitent and confirms his belief that his son is lost forever But Sebastian and Antonio shows some courage instead of repentance They wish to kill the spirits or devils if it appears

3

Of my instruction hast thou nothing bated (Line 85-93)

In what thou hast to say So with good life

And observation strange my meaner ministers

Their several kinds have done My high charms work

And these mine enemies are all knit upIn their distractions They now are in my

powerAnd in these fits I leave them while I visitYoung Ferdinand whom they suppose is

drownedAnd his and mine loved darling

Methought the billows spoke and (Line 96-99)

told me of itThe winds did sing it to me and the

thunderThat deep and dreadful organ-pipe

pronouncedThe name of Prosper It did bass my

trespass

These are the words of contrition coming from Alonso Ariel has driven him to a deep repentance for conspiring with Antonio against Prospero He now feels a sincere remorse It appears to him that all the elements of nature the sea-waves the wind and the thunder proclaimed with a loud voice the name of Prospero and the crime Alonso had committed against him They are calling upon him to repent There is a deep storm raging in Alonsorsquos breast and the echoes of that storm are ringing in his ears like the clear note of a wind-instrument

Comment These are the words of sincere repentance wrung out of the conscience-stricken heart of Alonso Alonso who is the lesser villain is the first to give way to remorse under the effect of Arielrsquos speech The words of Ariel seem to him to be the voice of conscience speaking to him He is driven to desperation a state in which he might do violence to his life

(a) Identify the speaker State the context

Prospero the ruler of the island is the speaker The famous banquet scene has been enacted very well Ariel and his junior spirits have played their roles excellently Prospero is glad to say words of praise for them(b) In what way the speakerrsquos instructions have been carried out

According to Prosperorsquos instructions a banquet was presented before the King of Naples and his companions when they were tired and hungry Just when they were preparing to eat the feast the banquet was suddenly removed by exercising supernatural powers All this was done by Ariel Prosperorsquos chief assistant and a powerful spirit

Ariel not only made the feast disappear but also delivered his speech blaming the King and his two companions for their past wicked deeds He warned them to repent for their misdeeds or suffer forever on that uninhabited island

(c) Who are referred to as lsquomeaner ministersrsquo What have they done

Prospero refers as lsquomeaner ministersrsquo to his other lesser spirits who were assisting Ariel in presenting a scene before the kingrsquos party They entered the scene to the accompaniment of music They assumed several strange shapes and brought in a banquet Then they danced about it with gentle actions of salutations thus inviting the King and others to eat the feast

These spirits play their role again when Ariel in the shape of a harpy quits the scene These shapes enter again and dancing with mocking gestures carry away the table

(d) Who are the speakerrsquos enemies What has happened to them

King of Naples Alonso his brother Sebastian and the present Duke of Milan Antonio (Prosperorsquos own brother) are Prosperorsquos enemies With the turn of events they have all been washed ashore on the island which is ruled by Prospero the great magician Actually this happened after the shipwreck caused by a storm which was raised by Prospero with the purpose of bringing these people to his island Prosperorsquos spirits have already confused and terrified these enemies and they are under Prosperorsquos control He can treat them as he likes

(e) What does he say about Ferdinand Explain what is meant by ldquohellip his and mine darlingrdquo

Prospero knows that Alonsorsquos son prince Ferdinand is alive though his father thinks that the prince has been drowned

Prospero refers to his daughter Miranda who is dear to him She is also very dear to Prince Ferdinand who has fallen in love with her They are waiting to be married soon for which they have received Prosperorsquos consent

4

ALONSO O it is monstrous monstrous (Line 95-102)

Methought the billows spoke and told me of it

The winds did sing it to me and the thunderThat deep and dreadful organ-

pipe pronouncedThe name of Prosper It did bass

my trespassTherefore my son ithrsquo ooze is

bedded andIll seek him deeper than eer

plummet soundedAnd with him there lie mudded

(a) In what way does Alonso express his horror when his conscience is awakened by Arielrsquos words

When Alonsorsquos conscience is awakened by Arielrsquos words he expresses his horror at what he has heard He gets the feeling that the waves of the ocean the wind and the loud thunder have spoken to him and uttered the name of Prospero Because of being reminded of his crime in a very loud and rough voice he comes to realize that he has lost his son for his past misdeeds

(b) What does Alonso imagine about his son What does Alonso want to do in his desperate state

Alonso imagines that his son is lying in the mud at the bottom of the sea He feels desperate that he wants to drown himself in the ocean deeper than the plumb-line has ever gone He wants to lie with his son at the bottom of the sea

(c) How do Sebastian and Antonio want to face the evil spirits

Sebastian says that he is not at all afraid of what the harpy has said and that he is prepared to fight any number of such monsters if they appear before him only one at a time Antonio says that he would support Sebastian in the fight against the fiendsyyy

(d) Why does Gonzalo ask Adrian to follow the three men

Gonzalo tells Adrian that all the three men namely Alonso Sebastian and Antonio are in a wild and reckless mood The thought of the heinous crime of which they are guilty has begun to torment their minds So he asks Adrian to follow those three men without loss of time and prevent them from doing anything which the turmoil in their minds might lead them to do

(e) What opinion do you form of Alonso from the above extract

Alonso who is the lesser villain is the first to give way to remorse under the effect of Arielrsquos speech The words of Ariel seem to him to be the voice of conscience speaking to him He is driven to desperation a state in which he might do violence to his life

Subject =Accounts

Ac-12 15420 topic-pL Appropriation ac

PROFIT AND LOSS APPROPRIATION ACCOUNT

MEANING AND PREPARATIONProfit and Loss Appropriation Account is merely an extension of the Profit and Loss Account of the firm The profit of the firm has to be distributed amongst the partners in their respective profit sharing ratio But before its distribution it needs to be adjusted All Adjustments like partnerrsquos salary partnerrsquos commission interest on capital interest on drawings etc are made in this account These adjustments will reduce the amount of profit for distribution This adjusted profit will be distributed amongst the partners in their profit sharing ratio To prepare it at first the balance of Profit and Loss Account is transferred to this account The journal entries for the preparation of Profit and Loss Appropriation Account are given below

1 for transfer of the balance of Profit and Loss Account to Profit and Loss Appropriation Account

(a) In case of Net Profit

Profit and Loss Ac helliphelliphelliphelliphellipDrTo Profit and Loss Appropriation Ac(Net Profit transferred to Profit and Loss Appropriation Ac)

(b)In case of Net Loss

Profit and Loss Appropriation Achelliphelliphellip DrTo Profit and Loss Ac(Net Loss transferred to Profit and Loss Appropriation Ac)

2 for Interest on Capital

For transferring on Interest on CapitalProfit and Loss Appropriation Achelliphelliphellip DrTo Interest on Capital Ac(Interest on capital transferred to Profit amp Loss Appropriation Ac)

3 for Interest on Drawings

For transferring Interest on Drawings Interest on Drawings Achelliphelliphelliphelliphelliphellip DrTo Profit and Loss Appropriation Ac(Interest on drawing transferred to Profit amp Loss Appropriation Ac)

4 For Partnerrsquos SalaryFor transfer of partnerrsquos SalaryProfit and Loss Appropriation Achelliphellip DrTo Salary Ac(Salary transferred to profit amp Loss Appropriation Ac)

5 For Partnerrsquos CommissionFor transferring commissionProfit and Loss Appropriation Achelliphelliphellip DrTo Commission Ac(Commission transferred to Profit and Loss Appropriation Ac)

6 For Transfer of agreed amount to General ReserveProfit and Loss Appropriation Ac helliphellipDrTo General Reserve Ac(Transfer to General Reserve)

7 for share of Profit or Loss appropriation(a) If ProfitProfit and Loss Appropriation Achelliphellip DrTo Partnerrsquos CapitalCurrent Ac(Profit transferred to capitalcurrent Ac)(b) If LossPartnerrsquos Capital Current Achelliphelliphelliphellip DrTo Profit and Loss Appropriation Ac(Loss transferred to capitalcurrent Ac)

THE FORMAT OF PROFIT AND LOSS APPROPRIATION

Profit and Loss Appropriation Account for the year endedhelliphelliphelliphellip

Particulars Amount Particulars Amount

To PL Ac (loss) By pL Ac (profit)

To Interest on capital BY Interest on drawings

To partner`s commission by Partner`s capital Ac ( loss)

To Partner`s salary To Interest on partner`s loan To General Reserve To Partner`s Capital AC (Profit)

Subject= Economics

MOVEMENT ALONG THE DEMAND CURVE (CHANGE IN QUANTITY DEMANDED)In law of demand you have already studied the inverse relationship between price and quantity demanded When quantity demanded of a commodity changes due to change in its price keeping other factors constant it is called change in quantity demanded It is graphically expressed as a movement along the same demand curve There can be either a downward movement or an upward movement along the same demand curve Upward movement along the same demand curve is called contraction of demand or decrease in quantity demanded and downward movement along the same demand curve is known as expansion of demand or increase in quantity demanded

Extention of demandd

price (rs)p A

B Extentionp1 d

Q Q1

Quantity demanded ( in units)

Contraction of demandd

p2 Ccontraction

p APrice (Rs)

d

Q2 Q

Quantity demanded (in units)

Explanation of movement of demand A fall in price from OP to OP1 leads to increase in quantity demanded from OQ to OQ1 (expansion of demand) resulting in a downward movement from point A to point B along the same demand curve DD When Price rises from OP to OP2 quantity demanded falls from OQ to OQ2 (contraction of demand) leading to an upward movement from point A to point C along the same demand curve DD

  • Activity Series of Metals
    • Drawbacks of Rutherfordrsquos model of atom
      • Electromagnetic radiations
      • Properties of electromagnetic radiations
      • Characteristics of electromagnetic radiations
        • Plancks Quantum Theory-
        • Photoelectric effect
          • Intext Questions
            • Logic gates
            • Digital systems are said to be constructed by using logic gates These gates are the AND OR NOT NAND NOR EXOR and EXNOR gates The basic operations are described below with the aid of truth tables
            • AND gate
            • Example
Page 7:  · Web viewSubject. Topic. Summary. Execution. English 1 . Chapter 1 naming words . Page 8. Write the names of these pictures:- Person:-1. father. 2.Firefighter 3.doctor 4 ...

s iv It also prevent

back ache and muscle pain

Social studies Conquering distances

Put a tick on the correct option1 b Ship 2b water transport 3 b trains 4 c 13 5 b Wright brothersTrue or false1 False 2 True 3 True 4 True

MATHEMATICS

Ch 3Addition and Subtraction

Exercise 126 A school needs Rs 4987653 for its building It has only Rs 3592468 in its accounts Estimate the money by rounding off to the nearest lakh it has still to raise

Solution Estimated amount need Rs 5000000 The school has only Rs 3600000 Still to raise Rs 1400000

MATHEMATICS

Ch 4

Mul

tiplic

ation

and

Div

ision

We have learnt multiplication tables up to 15 Let us extend the tables up to 20X 11 12 13 14 15 16 17 18 19 201 11 12 13 14 15 16 17 18 19 202 22 24 26 28 30 32 34 36 38 403 33 36 39 42 45 48 51 54 57 604 44 48 52 56 60 64 68 72 76 805 55 60 65 70 75 80 85 90 95 1006 66 72 78 84 90 96 102 108 114 1207 77 84 91 98 105 112 119 126 133 1408 88 96 104 112 120 128 136 144 152 1609 99 108 117 126 135 144 153 162 171 180

10 110 120 130 140 150 160 170 180 190 200Properties of Multiplication1 The product of two numbers does not change when the order of numbers is changed eg 503times23 = 23times503 [This property is called Commutative Property of multiplication]

2 The product of three numbers does not change when the grouping of numbers is change eg (15times18iquesttimes10=15 times(18times 10) = (15times10iquesttimes18 [This property is called Associative Property of multiplication]

3 The product of a number and 1 is the number itself eg 1513 times 1 = 1513 [This property is called Identity Property of multiplication and the integer 1 is called Identity Element of multiplication]

4 The product of a number and 0 is 0 eg 718205times 0 = 0 times 718205 = 05 The product of a number by the sum of two numbers is equal to the sum of the

products of that number by the two numbers separately eg123 times (105+ 48) = 123 times 105 + 123 times 48 [ This property is called Distributive Property of multiplication over

addition]English language

Transitive and intransitive verb

Pick out the verbs from the following sentences and say whether they are transitive or intransitive verb6 Was fullndash verb Intransitive verb7 Have been decorated- verb Transitive verb8 Happy cheerful ndash verb Intransitive verb9 Shall come back ndash verb Transitive verb

English 2 The fall of Lanka

This is the story of the fight between Rama and Ravana as told by Valmiki in lsquoThe Ramayana The monkeys worked all day and all night and at last built a bridge so that Rama and

Write the synonyms of

1 Stationed-

his army could cross to Lanka and rescue Sita assign2 Invade- enter

a country or a region so as to subjugate or occupy it

3 Prowess- bravery in battle

4 Haughty- arrogantly

5 Puny- small and weak

6 Spy- secret agent

7 Dreadful- causing or involving great suffering

8 Violate- disobey

9 Ghastly- causing great horror or fear

10 Deception ndash misleading

Class VISubject Topic Summary Execution

HISTORY AND CIVICS

CHAPTER 3

MAHAVIRA AND BUDDHA ndash GREAT PREACHERS

BUDDHA

Impact of Buddhism on Indian Cultures

Impact on Religion

Buddharsquos practical and simple doctrines made their impact on HinduismThe principle of ahimsaIt brought about a great change in the performance of costly yanjnas and sacrifices which previously involved immense loss of life The Mahayana Buddhists adopted the practice of worshipping Buddha and bodhisattvas making idols and erecting temples in their honour

Impact on Literature

After the death of Buddha his teachings were compiled and called TripitakasThe Jatakas contain tales dealing with the previous births of Buddha

Impact on Education

The Buddhist monasteries became great centres of learning These centres of learning developed into famous universities- Nalanda Taxila Vikramshila etc

Impact on Art and Architecture

The gateways and railing of the Sanchi Stupa were covered with sculptured figuresCave- temples were also constructed which were decorated with beautiful frescoes

1) How many parts of Tripitakas are thereAns -There are three parts of Tripitakas- Sutta Pitaka Vinaya PitakaAbhidhamma Pitaka

2)What are the subjects taught in these monasteries Ans ndash Buddhist scriptures logic Philosophy medicine astronomy etc

3) Which art was developed under Buddhist patronage Ans ndash Gandhara art

The Gandhara art was developed under Buddhist patronage

BIOLOGY The Leaf Photosynthesis The process by which green plants make their own food from carbon dioxide and water in the presence of sunlight and chlorophyll is called photosynthesis

All green plants need the following to make their food ndash

water carbon dioxide chlorophyll and energy in the form of sunlight

Carbon dioxide + water ------------- Glucose + oxygen

The end product of photosynthesis is glucose

Fill in the blanks

1 Plants make their food by the process of photosynthesis

2 The inner wall of the guard cell is thicker than the outer wall

3 The extra glucose is converted into starch and sucrose

4 The leaf is boiled in alcohol to remove chlorophyll

5 The rate of transpiration is more on the hot day then a cold day

6 Photosynthesis helps to observe water and minerals from the soil

English 1 Pronouns Kinds of pronouns 1 Personal pronouns2 Possessive pronouns3 Reflexive pronouns4 Interrogative pronouns5 Relative pronouns6 Demonstrative pronouns7 Indefinite pronouns

Personal pronouns they refer to first second and third person in sentences First person- the speakerSecond person-the listenerThird person-the objectperson being spoken aboutPersonal pronouns should have the same gender and number as the nouns they refer to

Possessive pronouns these are used to indicate the relationship between the objects and people These pronouns include mine ours yours his hersand theirs

ExerciseBFill in the blanks with suitable pronounsThere was much excitement among the childrenTheywere eagerly looking forward to the annual picniclsquoAre they going to Lodhi Gardens toorsquo wondered AneeshlsquoNo they are going to Buddh Jayanti Park with Mrs Jain said Mrs ChopraThe children looked disappointedlsquoWonrsquot you be taking us Marsquoam rsquo they askedlsquoSorry children I have to go to Mumbai for a week to look after my sick mother But you will have fun with Mrs Jain she is full of laughter and you will love being with her the whole daylsquoIt will not be the same they grumbled

English 2 The great train journey- Ruskin Bond

The great journey by Ruskin Bond is a story about Suraj who loved trains and wanted to go to places One day while wandering along the railway tracks he enters into a carriage compartment The train suddenly starts moving with him in the compartment and after a journey returns back to the same place from where it had begun The story is about his experience during that journey

State true or false1 When the train had passed leaving behind the

hot empty track Suraj was lonely2 It was winter holidays

and Suraj did not know what to do with himself

3 He plunged his hands into the straw and pulled out an apple

4 A dirty bearded face was looking out at him from behind a pile of crates

5 Suraj wanted to go to Japan

Hindi 2nd lang

गललबाजलडका खालीसथानोकोभरो-6 गो-ामसनिनकलकरहमगराजमआगए7 माबोधराजकोराकषससमझतीथी8 चीलरोशन-ानमसअ-रआकरतहसीलपरबठगई9 तीनचारतीनकऔररईकगोलउडलनिकनघोसलानहीनिगरा10 वहसवयतोघोसलातोडनककतिलएगललउठालायाथा

11 -ीवारकसाथलगतगोहपजोकसहार-ीवारपकडलतीह12 बोधराजअभीभीटकटकीबाधचीलकीओर-खरहाथा13 बोधराजअपनीजबमबहतसाचगगाभरकरलायाथा14 मरनिपताजीकीतरककीहईऔरहमलोगएकबडघरमजाकररहनलग15 बागमजातातोफलपरबठीनिततलीको-खनिततलीकोपकडकरउगकतिलयोकबीचमसल-ता

BENGALI(2ND LANGUAGE)

সহিনধসবরপওসবরসহিনধ

সবরসহিনধরহিনয়ম- ৯ই-কারহিকংাঈ-কাচেররপচেরইাঈহিভননঅনযসবররণCথাকচেইাঈসথাদেনয-ফায়এংওইয- ফাপCচেরণCযকতয়

১০উ-কারহিকংাঊ-কাচেররপচেরইাঈহিভননঅনযসবররণCথাকচেউাঊসথাদেন-ফায়এংওই- ফাপCচেরণCযকতয়

১১ঋ- কাচেররপচেরঋহিভননঅনযসবররণCথাকচেঋসথাচেনর -ফায়এংওইর পCচেরণCযকতয়

১২সবররণCপচেরথাকচেপCতMএ-সথাচেনঅয় ঐ- সথাচেনআয় ও- সথাচেনঅএংঔ- সথাচেনঅায়

৯ই+ অ= য- ফাআহি+ অনত= আযনত অহিধ+ অয়ন= অধযয়নই+ আ=য- ফা+ াইহিত+ আহি= ইতযাহি পরহিত+ আতC ন= পরতযাতC নই+ উ=য- ফা+ উঅহিত+ উহিকত= অতযহিকত হি+ উৎপহিতত= যৎপহিততই+ ঊ= য- ফা+ ঊ ই+ এ= য- ফা+ এঈ+ অ= য- ফা পরহিত+ ঊ4= পরতয4 পরহিত+ এক= পরচেতযকঈ+ অ আ= য- ফা+ অ আনী+ অমব= নযমব মসী+ আধার= মসযাধার

১০উ+ অ= অন+ অয়= অনবয় পশ+ অধম= পশবধমউ+ আ= াস+ আগত= সবাগত পশ+ আহি= পশবাহিউ+ ঈ= হিঅন+ ইত= অহিনবতউ+ এ= দেঅন+ এ4রণ= অচেনব4রণউ+ ঈ= ীসাধ+ ঈ= সবাধবী তন+ ঈ= তনবী

১১ঋ+ অ= র মাত+ অনমহিত= মাতরনমহিতঋ+ আ= রা হিপত+ আয়= হিপতরায়ঋ+ ই= হির মাত+ ইচছা= মাতচছাঋ+ ঈ= রী ধাত+ ঈ= ধাতরীঋ+ উ= র ভরাত+ উপচে(= ভরাতরপচে(

১২এ+ অ= অয় দেন+ অন= নয়নঐ+ অ= আয় গৈগ+ অক= গায়কও+ অ= অ দেপা+ অন= পনও+ ই= অ দেপা+ ইতর= পহিতরও+ এ= অ দেগা+ এ4রণা= গচে4রণাঔ+ অ= অা দেপৌ+ অক= পাকঔ+ ই= অা দেনৌ+ ইক= নাহিকঔ+ উ= অা দেভৌ+ উক= ভাক

MATHS Topic NumbersChapter Natural numbers and whole numbers

Study item properties of whole numbers for multiplication

1 Closure property If x and y are two whole numbers then xtimesy is also a whole numberExample If x = 9 and y =3 then xtimesy = 9times3 = 27 which is a whole number

2 Commutative property If x and y are two whole numbers then xtimesy = ytimesxExample If x = 5 and y = 2 then xtimesy = 5times2 = 10y times x = 2times5 = 10Therefore 5times2 = 2times5

3 Associative property If x y and z are three whole numbers then x times(ytimesz) = (xtimesy) times zExample If x =3 y = 5 and z = 7 then 3 times (5times7) = 3 times (35) = 105And (3times5) times7 = (15) times 7 = 105Therefore x times (ytimesz) = (xtimesy) timesz

4 Distributive property If x y and z are three whole numbers then xtimes (y + z) = x times y + x times z

Therefore the multiplication of whole numbers is distributive over their additionExample If x = 5 y = 3 and z= 2Therefore x times (y + z) = 5 times (3 + 2) = 5times5 =25And x times y + xtimes z = 5times3 +5times2 =15 +10 = 25Again x times (y ndash z ) = x times y ndash x timesz Therefore 5 times ( 3 - 2) = 5 times1 = 5 and 5times3 ndash 5 times2 = 15 ndash 10 = 5Therefore the multiplication of whole numbers is also distributive over their subtraction if y is greater than z

5 Existence of identity If x is a whole number then

X times1 = x 1 times x = xTherefore we can write x times1 = 1 times xTherefore the multiplication of any whole number with 1 is the number itselfTherefore we can say that 1 is multiplicative identity or identity element for multiplicationExample 5 times1 = 5 1 times 5 = 5 Therefore 5 times 1 = 5

6 Multiplicative inverse If x is any whole number ( x is not equal to zero ) then its multiplicative inverse will be 1xSo x times 1x = 1 but 1x is a whole number if x = 1For other values of whole number 1x is not a whole number therefore we can write its multiplicative inverse does not exists

7 Cancellation law of multiplication If x y and z are three non- zero whole numbers then x times y = x times z

Or y = zExample 9 times y = 9 timeszTherefore y = z

Class VIISubject Topic Summary Execution

English 2 Sentences based on meanings

Kinds of sentences

Assertive or declarative to convey information or simply make a statement

Interrogative to ask different types of questions

Imperative to command or instruct someone or make a request

Exclamatory to express strong feelings and emotions

Exercise c1 What a nice compliment that is

That is a nice compliment2 How well- behaved the children

areThe children are very well-behaved

3 What great chefs we areWe are great chefs

4 What a shame it isIt is a shame

5 What a fantastic idea you haveYou have a fantastic idea

Homework 6 -10English

LiteratureThe Listeners III) Answer the following questions-

d) Identify two words used in the poem to give the poem an eerie atmosphereAns- Two words used to give the poem an eerie atmosphere are ldquogreyrdquo and ldquophantomrdquo

e) Who do you think are the inmates of the houseAns- I think the inmates of the house are phantom who once used to dwell in it

f) Why was the poet ldquoperplexed and stillrdquoAns- He was lsquo perplexed and stillrsquo because he was expecting an answer from the inmates of the house But despite of repeated calls there was no response

CHEMISTRY Chapter 2 ndashElement and Compound

Activity Series of MetalsThe activity series is a chart of metals listed in order of declining relative reactivity The top metals are more reactive than the metals on the bottomMetal SymbolReactivity

Lithium Li displaces H2 gas from water steam and acids and forms hydroxides

Potassium K

Strontium Sr

Calcium Ca

Sodium Na

Magnesium Mg displaces H2 gas from steam and acids and forms hydroxides

Aluminum Al

Zinc Zn

Chromium Cr

Iron Fe displaces H2 gas from acids only and forms hydroxides

Cadmium Cd

Cobalt Co

Nickel Ni

Tin Sn

Lead Pb

Hydrogen gas

H2 included for comparison

Antimony Sb combines with O2 to form oxides and cannot displace H2

Arsenic As

Bismuth Bi

Copper Cu

Mercury Hg found free in nature oxides decompose with heating

Silver Ag

Palladium Pd

Platinum Pt

Gold Au

Answer the following Q)Difference Between Metals And Nonmetals

Metals Nonmetals

These are solids at room temperature except mercury

These exist in all three states

These are very hard except sodium

These are soft except diamond

These are malleable and ductile

These are brittle and can be breakdown into pieces

These are shiny These are non-lustrous except iodine

Electropositive in nature Electronegative in nature

Have high densities Have low densities

Math Number System

Chapter Fraction

Study item Some solved sums from exercise 3(B)1) For each pair given below state whether it from like fractions or unlike

fractions (i) 58 and 78

= Like Fraction because denominators same(ii) 815 and 821

= Unlike Fraction because denominators are not same

(iii) 49 and 94 = Unlike Fraction

2) Convert given fractions into fractions with equal denominators(iii) 45 1720 2340 and 1116Solution Given fraction 45 1720 2340 and 1116Therefore the LCM of 5 20 40 and 16 is 80Therefore 45 = 4times165times16 = 64801720 = 17times420times4 = 68802340 = 23times240times2 = 4680 1116 = 11times516times5 = 5580

3) Convert given fractions into fractions with equal numerators(iii) 1519 2528 911 and 4547Solution Given fractions 1519 2528 911 and 4547Therefore the LCM of 15 25 9 and 45 is 2251519 = 15times1519times15 = 225285 2528 = 25times928times9 = 225252911 = 9times2511times25 = 2252754547 = 45times547times5 = 225235

4) Put the given fractions in ascending order by making denominators equal

(iii) 57 38 914 and 2021Solution Given fraction 57 38 914 and 2021Therefore the LCM of the denominators is 16857 = 5times247times24 = 12016838 = 3times218times21 = 63168914 = 9times1214times12 = 1081682021 = 20times821times8 = 160168Therefore ascending order 63168lt108168lt120168lt160168Therefore ascending order of given fractions38lt914lt57lt2021

COMPUTER CHAPTER-1COMPUTER FUNDAMENTALS

DONE IN THE PREVIOUS CLASSES PAGE 10CWRITE TRUE AND FALSE

1 True2 False3 False4 False5 True

GEOGRAPHY CHAPTER 7EUROPE

CHAPTER COMPLETE 1)Europe is home to a famous mountain range called the Alps

2)River Rhine originates in Switzerland

3)The Eiffel Tower one of the tallest structures in Europe

4) Vatican City is one of the most densely populated European countries

5)Sognefjordin Norway is the largest fjord in Norway

Class VIIISubject Topic Summary Execution

MATHEMATICS Ch 6Sets

Exercise 6 (D)1 Given A = x x isin N and 3iquest x le 6 and B = x x isin W and xlt4 find (i) Sets A and B in roster form (ii) A cup B (iii)

A cap B(iv) A ndash B (v) B ndashA

Solution (i) A = 456 and B = 0123

(ii) A cup B = 0123456 (iii) A cap B = ϕ (iv) A ndash B = 456 (v)B ndash A = 0123

3 If A = 56789 B = x 3 lt x lt 8 and x isin W and C = x xle5 and x isin N Find (i) A cup B and (A cup B) cup C (ii) B

cup C and A cup ( B cup C)

(iii) A cap B and (A cap B) cap C (iv) B cap C and A cap (B cap C)

Is (A cup B) cup C = A cup (B cup C)

Is (A cap B) cap C = A cap (B cap C)

SolutionA = 56789 B = 4567 C = 12345

there4 (i) A cap B = 456789 and (A cup B) cup C = 123456789

(ii) B cup C = 1234567 and A cup ( B cup C) = 123456789

(iii) A cap B = 567 and (A cap B) cap C = 5

(iv) B cap C = 45 and A cap (B cap C) = 5

Now (A cup B) cup C = 123456789

And A cup ( B cup C) = 123456789 there4 (A cup B) cup C = A cup (B cup C)

Again (A cap B) cap C = 5 and A cap (B cap C) = 5

there4 (A cap B) cap C = A cap (B cap C)

4 Given A = 012345 B = 02468 and C = 0369 Show that (i) A cup (B cup C) = (A cup B) cup C ie the union

of sets is associative (ii) A cap (B cap C) = (A cap B) cap C ie the intersection of sets is associative

SolutionNow B cup C = 0234689 and A cup B = 01234568

there4 A cup (B cup C) = 012345689 and

(A cup B) cup C = 012345689

So (i) A cup (B cup C) = (A cup B) cup C ie the union of sets is associative

Again B cap C = 06 and A cap B = 024

there4 A cap (B cap C) = 0 and (A cap B) cap C = 0

So (ii) A cap (B cap C) = (A cap B) cap C ie the intersection of sets is associative

Physics Chapter 2 Physical Quatites and Measurements

Here We Will Do Some QuestionsRelated To Chapter 2

A density bottle has a marking 25 mL on it It means that

1 the mass of density bottle is 25g

2 the density bottle will store 25 ml of any liquid in it

3 the density bottle will store 25 ml of water but more volume of liquid denser than water

4 the density bottle will store 25 ml of water but more volume of a liquid lighter than water

Solution 2 the density bottle will store 25 ml of any liquid in it

COMPUTER CHAPTER-2Spreadsheet Functions and Charts

SELECTING RANGE IN ROWSCOLUMNSWHEN TWO OR MORE CELLS ARE SELECTED IT IS CALLED A RANGEA RANGE OF CELLS CAN BE FORMED IN TWO WAYS--a) SELECTING RANGE BY USING THE MOUSEb) SELECTING RANGE BY USING THE KEYBOARD

Q1)WRITE THE STEPS TO SELECT PARTIAL RANGE IN A ROW

Ans)THE STEPS ARE-6 SELECT THE ROW7 BRING THE CELL POINTER TO THE DESIRED

LOCATION FROM WHERE YOU WANT TO START YOUR SELECTION

8 CLICK THE LEFT MOUSE BUTTON AND KEEP DRAGGING TO YOUR RIGHT TILL YOU REACH THE LAST CELL TO NE SELECTED

RELEASE THE MOUSE BUTTON

GEOGRAPHY Asia

CLIMATE

Asia experiences great extremes of climate Jacobabad in the Sind province of Pakistan is one of the hottest places in the WorldVerkhoyansk in Siberia is one of the coldest places in the WorldCherrapunji and Mawsynram in India are two wettest places in WorldArabia Tibet Gobi and Mongolia are extremely dry regionsFactors Affecting Climate of Asia-The factors influencing the climate of Asia are-

Factors Affecting Climate of Asia-Thoroughly read the table in page number 60

Latitudinal extent

Continentality

Relief features

Presence of low pressure trough

Jet streams

English Language The Sentence A complex sentence contains one independent clause and at least one dependent clause The dependent clause in a complex sentence is introduced with subordinating conjunctions or relative pronouns

Commonly Used Subordinating Conjunctions-Time after before while when since untilCause And Effect because now since as in order that soOpposition although though even though whereas while in spite ofCondition if unless only if whether or not even if in case(that)

Commonly Used Relative Pronouns-Who whose whom which whoever whomever whichever that

Class IXSubject Topic Summary Execution

1-BENGALI(2ND LANGUAGE)

ldquo বঙগভমিরপরমিrdquo াইকেলধসদনদতত

আচেগর পর উততর পচো-১ ২ ৩ এং নীচের পর টি াহির কা- ৪মহিbকাও গচেনা দেগা পহিচে অমত হরচে- ক) কার দো দেকান কহিতার অং( ) কতা দেক পরসঙগ কী উহিকতটির তাৎপযC আচোনা কচেরা৫দেসই ধনয নরকচে দোচেক যাচে নাহি ভচে মচেনর মহিeচের সাচেসচে সCন ক) কহির কায C ার উচেf(য হিক হিছ কহি কন কহিতাটি দেচেন) কহি কার কাচেছ হিমনহিত কচেরচেছনগ) কহি এই পহিথীচেত কাচের ধনয মচেন কচেরনঘ) কহি হিক রকম অমর তাাভ করচেত ান

Hindi 2nd lang

काकी(कतिसयारामशरण गपत)

इस कहानी म लखक न यह बतान का परयास निकया ह निक बचच अपनी मा स निकतना परम करत ह शयाम अबोध बालक ह वह अपनी मा क मरन क बा- उसन अपनी मा क कतिलए बहत रोया बा- म उस पता चला निक उसकी मा राम क घर चली गई ह आकाश म उडती हई पतग -खकर उस हरष हआ निक पतग क दवारा वह अपनी मा को नीच उतारगा इसक कतिलए वह अपनी निपता की जब स -ो बार सवा रपया निनकालकर पतग और -ो मोटी सी मन वाली अपन भाई स काकी एक कागज पर कतिलखवा कर पतग म कतिशव का दि-यानिनकालकर पतग और -ो मोटी सी मन वाली अपन भाई स काकी एक कागज पर कतिलखवा कर पतग म कतिचपका दि-याभोला और शयाम कोठरी म रससी बाधनी रह थ तभी उसक निपता करोध म आकर उन स पछ निक कया उनकी जब स रपया निनकाला हभोला डर क मार बताया निक शयाम इस पतग क दवारा अपनी काकी को राम क यहा स उतारना चाहता हनिवशशवर(शयाम क निपता)न फटी पतग उठाकर -खी तो उस पर काकी कतिलखा थावह हत बजिa होकर वही खड रह गएउनहोन सोचा निक मन अपन पतर को मारा जोनिक अनजान और निन-dरष थावह अपनी मा कोनिकतना पयार करता ह

उस दि-न बड सवर शयाम की नी- खली तो -खा निक घर भर म कोहराम मचा हआ ह

क) घर म कोहराम कयो मचा हआ था शयाम को कया लगा

ख) काकी को ल जात समय शयाम न कया उपदरव मचाया

ग) काकी क बार म उस कया बताया गया कया सतय उस कतिछपा रहा

घ) वह बठा-बठा शनय मन स आकाश की ओर कयोकरता

उततरक) शयाम की मा का -हात हो गया था इसकतिलए

घर म कोहराम मचा हआ था शयाम की लगा निक उसकी मा सफ- कपडा ओढ हए भमिम पर सो रही ह

ख) लोग जब उमा यानी शयाम की मा को उठाकर ल जान लग तब शयाम न बडा उपदरव मचाया लोगो क हाथ स झठ करवा उमा क ऊपर जा निगरा और बोला काकी सो रही ह उस कहा ल जा रह हो

ग) काकी क बार म बजिaमान लोगो न उस निवशवास दि-लाया निक उसकी का निक उसक मामा क यहा गई ह लनिकन सतय अमिधक दि-नो तक कतिछपाना रह सका आसपास क अबोध बालको क मह स यह बात परकट हो गई निक उसकी मा का -हात हो गया ह

घ) कई दि-नकई दि-न लगातार रोत-रोत उसका रोना तो शान हो गया पर उसक ह-य म शोक भर गया था वह चपचाप बठा आकाश की और टाका करता निक शाय- उसकी काकी कही दि-ख जाए

ldquoदि-न उसन ऊपर आसमान म पतग उडती -खी न जान कया सोच कर उसका निहर-य एक-म खिखल उठाrdquo

क) निकसन पतग ऊपर उडत -खी और वह कयो खश हआ

ख) उसन अपन निपता स कया कहा उनका कया उतर थाश

ग) उसन निफर कया निकया और निकसन उसकी सहायता की

घ) उसकी योजना कया थी उततर -क) शयाम न एक दि-न आसमान म पतग उडती

-खी तो उसन सोचा निक पता आसमान म राम क यहा जाकर रकगी वही पर मरी काकी ह यह सोचकर वह बहत खश हआ

ख) उसन अपन निपता स कहा काका मझ एक पतग मगा -ो उसक निपता न भटक हए मन क भाव स कहा निक मगा -ग यह कह कर उ-ास भाव स वह कही और चल गए पतग नही आई

ग) उसन चपचाप निवशशवर क टगहए कोट स एक चवननी निनकाल ली और सखिखया -ासी क लडक भोला की सहायता स एक पतग मगवानी भोला उसकी बराबर उमर का ही था

घ) उसकी योजना यह थी निक वह अपनी पतग को आकाश म राम क यहा भजगा और उस पतग क सहार उसकी काफी नीच उतर जाएगी इस योजना पर उस परा निवशवास था इसकतिलए वह और भोला -ोनो यह काम करन म लग गए

Continue to nexthelliphellipEVS CHAPTER - 1

(UNDERSTANDING OUR ENVIRONMENT)

Sustainable development

The development that meets the needs of the present without compromising the ability of future generations to meet their own needs is called Sustainable development

Sustainable societies ndash

An environmentally sustainable community is one that meets the current and future basic resource needs of its people in a just and equitable manner without compromising the ability of future generations to meet their basic needs

Q ) What are Eco Villages

Ans - Eco village are the urban or rural communities of people who strive to integrate a supportive social environment with a low impact way of life

Q ) To ensure sustainable development the depletion of renewable resources should not take place at a rate faster than their regeneration Justify your answer

Ans ndash Renewable resources do not have a fixed quantity - more can always be

generated However if the rate of use exceeds the rate of renewal - that is the

source is used more than its being recreated - its continued use will become

used up faster than it can regenerate

To promote sustainable society the following things need to be done ndash

1 Using renewable energy sources 2 By improving the quality of human

health 3 By promoting sustainable agriculture 4 By forming ecovillage

it will eventually be entirely depleted So Toensure sustainable development the depletion of

renewable resources should nottake place at a rate faster than their regeneration

Q ) What do you mean by Sustainable societies

Ans - Sustainable societies are defined as towns and cities that have taken steps to remain healthy over the long term These communities value healthy ecosystems use resources efficiently and actively seek to retain and enhance a locally based economy Sustainable development concerns everybody in a society

Q ) What are the effects of pollution on human health

Ans ndash Some health problem occurs due to air pollution are ndash

Respiratory diseases Cardiovascular damage Fatigue headaches and anxiety Irritation of the eyes nose and throat Damage to reproductive organs Harm to the liver spleen and blood Nervous system damage

Some health problem occurs due to water pollution are ndash

Typhoid Cholera Dysentry Jaundice

Some health problem occurs due to noise pollution are ndash

Fatigue headaches and anxiety High blood pressure Hearing damage

Physics Motion in 1D First go through previous notes Now here we will solve some numerical related to that

Question 3What information about the motion of a body is obtained from the displacement-time graphSolution 3From displacement-time graph the nature of motion (or state of rest) can be understood The slope of this graph gives the value of velocity of the body at any instant of time using which the velocity-time graph can also be drawn

Question 4(a)What does the slope of a displacement-time graph represent(b)Can displacement-time sketch be parallel to the displacement axis Give a reason to your answerSolution 4(a) Slope of a displacement-time graph represents velocity(b) The displacement-time graph can never be parallel to the displacement axis because such a line would mean that the distance covered by the body in a certain direction increases without any

increase in time which is not possible

Chemistry Language of Chemistry

How to balance a chemical equationThere are two methods of balancing an equation(i)Hit and trial method(ii)Partial equation methodBalancing by hit and trial methodThis method consists of counting the number of atoms of each elements on both sides and trying to equalize themTake the following steps(i)Count the number of times (frequency) an element occurs on either side(ii)The element with the least frequency of occurrence is balanced first(iii)When two or more elements have the same frequencythe metallic element is balanced firstExample-1 On heatinglead nitrate decomposes to give lead dioxidenitrogen dioxide and oxygenPb(NO3)2rarrPbO+NO2+O2

In this equationLead occurs twiceNitrogen occurs twiceOxygen occurs four timesSince lead is a metalbalance it firstThe number of atom of lead is equal on the two sidestherefore it needs no balancingNow balance nitrogenOn the reactant sidethere are two atoms of nitrogenwhile on the product side oneSomultiply the product containing nitrogenon the product sideby two Pb (NO3)2rarrPbO+2NO2+O2Nowthe number of oxygen atoms on the reactant side 6while on the product sideit is 7Somultiply the entire equation by 2except oxygen to get balanced equation2Pb(NO3)2rarr2PbO+4NO2+O2Multiplication by 2 is done only when atoms of all the elements except one element are balanced and the unbalanced atom occurs separately at least once and also there is a difference of only one such atom

Math Topic AlgebraChapter

Factorisation

Study item Difference of two squares a2 ndash b2 = (a+b) (a-b)1) (i) 4x2ndash 25y2

= (2x) 2 ndash (5y) 2= (2x + 5y) (2x - 5y)

(ii) 9x2 ndash 1= (3x)2ndash(1)2= (3x + 1)(3x ndash 1)

2) (i) 150 ndash 6a2= 6(25 ndash a2)= 6(5)2 ndash(a)2= 6 (5 + a) (5 ndash a)

(ii) 32x2 ndash 18y2=2(16x2 ndash 9y2)=2(4x)2 ndash (3y)2= 2(4x + 3y)(4x - 3y)3)(i) (x ndashy )2 ndash 9 = (x ndash y )2 ndash (3)2= (x ndash y + 3) (x ndash y ndash 3)(ii) 9(x + y) 2ndash x 2= (3)2(x + y)2 ndash (x)2=3(x + y)2 ndash (x)2= (3x +3y ) 2ndash(x)2= (3x + 3y + x)(3x +3y ndash x)= (4x + 3y) ( 2x + 3y )

Commercial studies

Basic accounting terms

Today I will give you some questions from the previous study material

Questions1) Define accounting2) What do you mean by debit and

credit

3) Explain the types of account4) Define the following terms

a) Assetsb) Capitalc) Purchased) Debtorse) Transactions

5) Name the types of accounts given below

a) Krishnas accountb) Machinery accountc) Royalty accountd) Salary accounte) Furniture accountf) Audit fee account

Economics Revision Today I will give you some revision questions

Questions1) What do you mean by the terms

rdquowantsrdquo2) Write the difference between

consumer goods and producer goods

3) Define the term utility 4) Explain the different types of utility5) Define

a) Total utilityb) Marginal utility

Subject Eng Literature (The Merchant of Venice ndash William Shakespeare)Topic Act I Scene 3 Lines 1 to 48 (Shylock hellip Cursed be my tribe if I forgive him) Date 16th April 2020 (5th Period)

[Students should read the original play and also the paraphrase given in the school prescribed textbook]Summary Questions amp Answers

This scene takes place in Venice and we are introduced to the rich Jew Shylock Bassanio and Shylock are talking and Bassanio tells Shylock that he wants a loan of three thousand ducats for three months on the personal security of Antonio

o Shylock feels glad because he will be able to bind down Antonio by means of a bond on account of the loan but he tells Bassanio that all the fortunes of Antonio being invested in the merchant ships on the sea it is difficult to depend upon his credit Even under such circumstances Shylock is willing to advance the money on the personal security of Antonio

o Bassanio then invites Shylock to dine with him Shylock says that he is prepared to do anything with the Christians but not eat or drink or pray with them

o While Bassanio and Shylock are talking Antonio appears on the scene Shylock does not seem to take any notice of Antonio but goes on brooding within

(1) SHYLOCK Ho no no no no- my meaning in (Line 15-26)saying he is a good man is to have you understand me that he is sufficient Yet his means are in suppositionhe hath an argosy bound to Tripolis another to the Indies I understand moreover upon the Rialto he hath a third at Mexico a fourth for England and other ventures he hath squanderd abroad Butships are but boards sailors but men there be land-rats and water-rats land-thieves and water-thieves I mean pirates and then there is the peril of waters winds and rocks The man is notwithstanding sufficientmdashthree thousand ducats mdashI think I may take his bond

(a) Who is talking in the beginning of this scene What does Bassanio want from Shylock How does Shylock feel

In the beginning of the scene Bassanio and Shylock are talking to each other Bassanio wants to borrow three thousand ducats from Shylock for three months on the security of Antonio Shylock feels glad at heart that he will get the opportunity of binding Antonio with a bond(b) What risks does Shylock weigh in advancing the money

Shylock says that Antonio has invested all his capital in trading by sea-going ships But the ships are made of wood and the sailors of those ships are ordinary human beings The wood can

himself how he hates Antonio because of his being a Christian because he abuses Shylock in public places Shylock decides that if ever he can get Antonio to his advantage he will teach him a lesson

come to harm and men can commit mistakes and thus the capital invested in ships may be lost Then there are other dangers The goods loaded on the ships can be damaged by rats and thieves which are found both on land and water The ships can also be harmed through sea-storms submerged rocks etc(c) What two important functions does this scene have

The scene has two important functions First it completes the exposition of the two major plot lines of the play Antonio agrees to Shylockrsquos bond ndash three thousand ducats for a pound of flesh and second and more important dramatically this scene introduces Shylock himself In this scene Shakespeare makes it clear at once why Shylock is the most powerful dramatic figure in the play and why so many great actors have regarded this part as one of the most rewarding roles in all Shakespearean dramas(d) Where does this scene take place What kind of treatment has Antonio been giving to Shylock What does Shylock say when Bassanio invites him to dine with him

The action of this scene takes place in Venice Antonio has been in the habit of behaving harshly with Shylock ndash spitting on his beard and footing him like a stranger cur When Bassanio invites Shylock to dine Shylock says that he is prepared to do anything with the Christians but not eat and drink or pray with them

(2) SHYLOCK How like a fawning publican he looks (Line 38-48)I hate him for he is a Christian

But more for that in low simplicity

He lends out money gratis and brings downThe rate of usance here with us in VeniceIf I can catch him once upon the hipI will feed fat the ancient grudge I bear him

He hates our sacred nation and he railsEven there where merchants most do congregateOn me my bargains and my well-won thriftWhich he calls interest Cursed be my tribeIf I forgive him

(a) What is the context in which these words are spoken and what is the idea expressed in it

These remarks are made by Shylock when he sees Antonio coming along after Bassanio told him that the merchant will be his surety for the bond The above mentioned passage reveals Shylockrsquos hatred for Antonio Shylock says that he hates Antonio because he is a Christian and also because he gives loan without taking interest on them thereby bringing down the rate of interest in Venice(b) Explain the meaning of the phrase lsquoa fawning publicanrsquo

The phrase lsquoa fawning publicanrsquo refers to Roman tax collector It is a term of contempt and hatred on the lips of a Jew lsquoFawning Publicansrsquo were Roman tax-gatherers whose ordinary bearings towards the Jews was bullying but whose false pose of lsquohumility and contritionrsquo is touched upon in the parable in New Testament(c ) What light does the above passage throw on the character

of Shylock

The above mentioned speech of Shylock reveals him to be a wicked character having an extreme greed for wealth His intense hatred for Antonio is unjustified He hates Antonio only because he is a Christian and because he lends money without taking any interest on it thereby adversely affecting Shylockrsquos business of lending money on high interest(d) What information do you gather about Antonio from the above given lines

Shylockrsquos statement throws a valuable light on the character of Antonio Antonio appears to be a good Christian and a good human being He helps the people in need by lending them money without charging any interest on it He is a man of simple and good nature This very goodness makes him Shylockrsquos enemy(e) What does Shylock debate within himself and when To whom are the lines mentioned above addressed to

When Bassanio asks the Jew to lend him three thousand ducats on Antoniorsquos surety Shylock begins to debate within himself as to how he should exploit the opportunity of a business deal with his old enemy Antonio

The lines mentioned above are not addressed to anyone The lines are a soliloquy ie a speech made by a character to himself and not meant to be heard by the other characters present

Class XSubject Topic Summary ExecutionEnglish

LiteratureThe Blue Bead 2nd part

Things took a turn for the worst and all of a sudden a crocodile attacked the woman biting on the womanrsquos leg At that moment Sibia got up sprinted grabbed the hay fork and stabbed the crocodile in the eye with all her power Immediately the crocodile let go and went away Sibia saw a small blue bead lying by the river she grabbed it Since she was poor she didnrsquot have necklace Shersquod always wanted one like the other women now she could make one with the blue bead After that she went home and told her mother all about it

Hindi 2nd

Langबड घर की बटी( मशी परमच-)

lsquoबड घर की बटी कहानी का उददशय मधयम वग की घरल समसया को सलझा कर सगदिठत परिरवार म मिमल जलकर परम स रहन का स-श -ना हघर म शानित सथानिपत करन की जिजमम-ारी नारी की होती ह यदि- नारी समझ-ार ह उसम धय और परिरवार क परनित परम ह तो कोई भी घटना परिरवार को निवघदिटत नही कर सकती या कहानी परिरवार को सगदिठत करत हए परम सौहा- स एक दसर की भावनाओ को समझ करउनका सहयोग करत हए जीवन यापन करन की पररणा -ती हमशी परमचदर जी न इस कहानी म सय परिरवार का परनितनिनमिधतव निकया ह यह कहानी बनी माधव सिसह जो गौरी पर क जमी-ार क उनक -ो पतरो की हशरी कठ लाल निबहारीशरीकात का निववाह एकजमी-ार घरान की पतरी आन-ी स हआ थाआन-ी न ख- को ससराल क वातावरण म ढाकतिलया थाएक दि-न आन-ी का अपन -वर लाल निबहारी स झगडा हो जाता ह -ोनो भाई एक दसर स अलग होन की कोकतिशश करत हसभी बह आन-ी न अपन मधर वयवहार स लाल निबहारी को

ldquoइन नतर निपरय गणो को बीए-इनही -ो अकषर पर नयोछावर कर दि-या था इन -ो अकषर न उनक शरीर को निनबल और चहर को कानित ही बना दि-या थाldquo

क) परसतत पकतियो म निकस वयकति क बार म कहा गया ह

ख) इन पकतियो म कौन स नतर निपरय गणो क बार म कहा गया ह

ग) बीए की निडगरी परापत कर लन पर भी उपय वयकति क सवभाव की कया निवशरषता थी

घ) यह नतर निपरय गण निकस वयकति म निवदयमान थ उसक वयकतितव की कया निवशरषता थी

उततर ndashक) परसतत पकति म गौरी पर गाव क जमी-ार

क बड बट शरीकात क बार म कहा गया ह उसन बहत परिरशरम और उ-म क बा- ba की निडगरी परापत की थी अब वह एक -फतर

घर छोडकर जान स रोक कतिलयाइस पर बनी माधव सिसह न कहा निक बड घर की बटी ऐसी ही होती ह जो निबगडा काम बना लती ह अतः शीरषक साथक ह बड घर की बटी आन-ी ह

म कमचारी थाख) भरा हआ चहरा चौडी छाती और डटकर

खाना आदि- एक सबजी ल जवान क गण मान जात ह परत शरीकात न इनही नतर निपरय गणो को अपनी पढाई पर नयोछावर कर दि-या था

ग) बीए की निडगरी परापत कर लन पर भी उपय वयकति(शरी कठ की शारिररिरक तौर पर निनबल और चहर स कानित ही लगत थ इतना ही नही वह मानकतिसक तौर पर भी निपछड हए थ पाशचातय सामाजिजक कथा उस घणा एव पराचीन सभयता का गणगान उनकी निवचारधारा क परमख अग थ

घ) यह नतर निपरय गण गौरीपर गाव क जमी-ार क छोट बट लाल निबहारी सिसह म निवदयमान थ वह सजीलाजवान था और भस का दध शर दध वह सवर उठकर पी जाता था

ldquoयही कारण था निक गाव की लललन आए उनकी निन-क थी कोई कोई तो उह अपना शतर समझन म भी सकोच ना करती थी सवय उनकी पतनी को इस निवरषय म उनस निवरोध थाldquo

क) उपय पकति म इस वयकति क बार म कहा गया ह

ख) गाव की लललन आए उनकी निन-ा कयो निकया करती थी

ग) उनकी पतनी का कया नाम था उनह निकस निवरषय म अपन पनित क निवरa था और कयो

घ) इस कहानी का कया उददशय ह Continue to next helliphelliphellip

Bengali 2nd Language

ফ ফটক না ফটক( কহিতা )

পর) ldquo(ান াধাচেনা ফটপাচেথ পাথচের পাড হিচেয় এক কাঠচোটটা গাছ কহিকহি পাতায় পার ফাটিচেয় াসচেছldquoক) কার দো দেকান কহিতার অং( ) lsquo(ান াধাচেনা ফটপাচেথ পাথচের পাডহিচেয়lsquo চেত কী দোঝাচেনা চেয়চেছ গ) আচো য অংচে(lsquo এক কাঠচোটটা গাছ lsquoচেত কী দোঝাচেনা চেয়চেছ ঘ) ldquoকহিকহি পাতায় পার ফাটিচেয় াসচেছldquo ----- একথার পরকত অথC কী উততর ) ক) আচো য অং(টি পর যাত কহি সভা4 মচোপাধ যাচেয়র দো lsquoফ ফটক না ফটকrsquo কহিতার অং()কহি সভা4 মচোপাধ যায় হিছচেন দেপরচেমর কহি দেপরমচেক নানা ভহিঙগমায় হিতহিন ফটিচেয় তচেচেছন দেপরম মানচের স মচেতC র সঙগী কহিতার কহিতায় এক রb সb হচেয়র দেপরম াগরচেনর কথা চেচেছন (ান অথCাৎ দেযাচেন দেকান রস দেনই দেযাচেন দেকান মহিনতা দেনই অথ তার মধ দেযও দেপরম থাকচেত পাচের একথাই কহি তচে ধরচেত দেচেয়চেছন একটি মানচে4র মচেন দেযাচেন দেকামতার দেকান সথান দেনই পাথচেরর মচেতা হিনরসতার মচেনর মধ দেযও দেয দেপরম আসচেত পাচের দেস কথাই কহি চেচেছনগ)নারীচের যথC দেপরচেমর ছহি এই কহিতায় অকপচেট উচেঠ এচেসচেছ কহি এই কহিতায় কাটচোটটা গাছ কথাটি যার কচেরচেছন নারী দেয দেপরম দেথচেক হিতাহিত এং দেসই দেপরম সঠিক সমচেয় না পাওয়ার ন য দেপরম সমপচেকC হিচেr4 গৈতরী য় দেপরচেমর দেয গৈহি(ষট য মাধযC য সরসতা দেকামত এই সমসতর হিপরীত যথা রbতা শষকতা কচেঠার তা পরভহিত দোঝাচেত এক কাঠচোটটা গাছ কথাটি যার কচেরচেছনঘ) এাচেন এক নারীর যথC দেপরচেমর কথা হিনহিCপত ভাচে চেচেছন কহি অসমচেয় নারীর ীচেন দেপরম দেচেগচেছ এতহিন তার হয় রb কচেঠার হিছ দেপরচেমর অভাচে ঠাৎ দেসই শষক মরভহিমচেত সচের আভাস এচেসচেছ দেপরম দেযন 4Cার স(ীত তাই পরায় মত গাচেছ কহিকহি পাতা গহিচেয় উচেঠচেছ

Biology Chapter - 01Controlling Air Pollution

Today we will discuss how we control air pollution from domestic combustion

Q1Describe any five ways of reducing air pollution from domestic sources bull The number of pollutants in the air is verylarge and we always try to control them byfollowing ways

i) Solar cooker and solar heater It use no fuel reduce damage of environment by fuel use or reducing deforestation It maintains coolness of house It releases very less orno oil gas or grease

ii) Piped natural gas (PNG) It emits very less by products into the atmosphere As it isdistributed through pipe lines so there iscontinuous supply of fuel is possible

iii) Liquefied Petroleum Gas (LPG) It hasa higher heating value LPG doesntcontain sulphur so it burns a lot cleanerenergy sources It releases very less oralmost no fume in air

iv) Electricity based cooking Emission free cooking alternative for urban dwellers causeselimination of adverse health impactsofindoor air pollution It helps to avoid theinconveniences associated with procurement of LPG

v) Biogas It contains 75 methane whichmakes it an excellent fuel It burns without smoke and biogas plant leaves no residue like ash in wood charcoal etc Thus it isaclean fuel

Economics

Factors of Production

Today firstly we would recall the last class for 5 mins and then we would proceed with the further topics of the chapter

The concept meaning of land characteristics of land and importance of land to be repeated for the absentees as well as the students who were there in the class the previous day

Today we will start with the last portion of land before it the meaning of land to be repeated onceAs by now we all know that

Questions1What do you mean by productivity of landAnswer By productivity of land we mean the capacity of a piece of land to produce a crop

Thus it refers to the average output per unit of landSay per acre per hectare etc= (OutputArea of land)

2 What are the factors influencing the productivity of landAnswer

Natural factors Productivity of land is largely determined by the natural

Land is defined to include not only the surface of the earth but also all other free gifts of nature(for example mineral resources forest resources and indeed anything that helps us to carry out the production of goods and services but is provided by nature free of cost)

We will move on to the last portion of land by discussing Productivity of Land

By productivity of land we mean the capacity of a piece of land to produce a crop

Thus it refers to the average output per unit of land

Say per acre per hectare etc= (OutputArea of land)

With this we shall proceed further with the main factors that determine the productivity of land

Natural factors Human factors Improvements on land Location of land Organisation Ownership of land Availability of capital Proper use of land State help

Note economic development of a country depends upon the quality of its land If the land is fertile it will quicken the pace of development of the country

qualities of land such as fertility etc

Human factors Land cannot produce anything by itself Man has to apply labour on it to produce for himself So productivity of land depends on the knowledge and skills of workers

Improvements on land production of land is affected by land development measures like provision of well or tubewell irrigation proper drainage

State help The government of a country especially less developed country can play a vital role in improving the agricultural productivity by providing better irrigation facilities

Organisation Productivity of land also fdepends upon the way how the factors of production like labour and capital are organised

In order to increase productivity trained workers modern implements scientific methods good seeds are all essential

3 lsquoImproved technology affects the productivity of landrsquo Explain this statement with the help of suitable example Answer Use of improved technology raises the productivity of land Example By using HYV seeds chemical manures and modern machines per hectare output increases

Physics Force (Summary)

Question Write the expression for the moment of force about a given axisSolutionsThe expression for the moment of force is given byMoment of force about a given axis = Force times perpendicular distance of force from the axis of rotationQuestion What do you understand by the clockwise and anticlockwise moment of force When is it taken positiveSolutionsIf the effect on the body is to turn it anticlockwise moment of force is called the anticlockwise moment and it is taken as positive while if the effect on the

body is to turn it clockwise moment of force is called the clockwise moment and it is taken as negative

Math Topic Commercial Mathematics

Chapter Goods and services Tax

Study item Some solved sums from exercise ndash 1 A retailer buys a TV from a wholesaler for Rs 40000 He marks the price of the TV 15 above his cost price sells it to the consumer at 5 discount on the marked price If the sales are intra ndash state and the rate of GST is 12 find

(i) The marked price of the TV(ii) The amount which the consumer pays for the TV(iii) The amount of tax (under GST) paid by the retailer to the central

Government(iv) The amount of tax (under GST) received by the State Government

Solution As the sales are intra- state sale and the rate of GST 12 So GST comprises of 6 CGST and 6 SGSTTherefore a retailer buys a TV from a wholesaler for Rs 40000Therefore the amount of GST collected wholesaler from the retailer or paid by retailer to wholesalerCGST = 6 of Rs 40000 = Rs(6100 times40000) =Rs 2400SGST = 6 of Rs 40000 = Rs (6100 times 40000) =Rs 2400Therefore wholesaler will pay Rs 2400 as CGST and Rs 2400 as SGSTTherefore amount of input GST of retailer Input CGST = Rs 2400 and input SGST = Rs 2400Again the retailer marks the price of the TV 15 above his cost price(i) The marked price of the TV

= Rs 40000 + Rs 40000times15= Rs 40000 + Rs 40000times 15100= Rs 40000 + Rs 6000Rs 46000But the retailer sells it to consumer at 5 discount on the marked priceCost price after discount = Rs 46000 ndashRs46000times 5100 =Rs 46000 ndashRs 2300= Rs 43700Therefore the amount of GST collected retailer from consumer or paid by consumer to retailerCGST = 6 of Rs 43700 =Rs ( 6100 times43700)Rs 2622SGST = 6 of Rs 43700 = Rs (6100 times 43700) =Rs 2622Amount of the output GST of retailer Output CGST = Rs 2622 and output SGST = Rs 2622

(ii) The amount which the consumer pays for the TV= cost price of TV to consumer + CGST paid by consumer + SGST paid by consumer= Rs 43700 + Rs 2622 + Rs 2622= Rs 48944

(iii) The amount of tax (under GST ) paid by the retailer to the central Government=CGST paid by retailer = output CGST ndash input CGST=Rs 2622 ndash Rs 2400=Rs 222

(iv) The amount of tax ( under GST ) received by the State Government = SGST paid by wholesaler + SGST paid by retailer= Rs 2400 + output SGST ndash input SGST=Rs 2400 + Rs 2622 ndash Rs 2400=Rs 2400 + Rs 222= Rs 2622

Commercial studies

Stakeholders Today I am going to give some revision questions from the previous study material

Questions1) State the two expectations of

employees from a business concern2) Give two distinctions between

stakeholder and shareholder3) Give two difference between

internal stakeholders and external stakeholders

4) Give two expectations of suppliers from a business organisation

5) Who is a stakeholder in commercial organisations

Chemistry Periodic Table

Merits of Mendeleevrsquos Periodic law are as follows - 1He grouped the elements on the basis of atomic mass 2 He left gaps for undiscovered elements like Gallium Scandium germanium Also he left a full group vacant for undiscovered inert gases 3 He could predict proportions of several elements on basis of their position in periodic table like Ga Sc etc 4He could predict errors in atomic weights of some elements like gold platinum etc

Anomalies in Mendeleevrsquos Periodic law are as follows - 1 Position of isotopes could not be explained 2 Wrong order of atomic masses could not be explained

For example- as Arnur atomic mass 40 come first and K with low atomic mass (30) should come later but k should be placed first

According to Bohrrsquos Modern Periodic table properties of elements are periodic functions of their atomic numbers

So when elements are arranged according to increasing atomic numbers there is periodicity in electronic configuration that leads to periodicity in their chemical properties

It consists of horizontal rows (Periods) Vertical column (Groups)

There are 7 period and 12 groups in this long form of periodic table

Ist period has 2 elements IInd period has 8 elements IIIrd period has 8 elements IVth period has 18 elements Vth period has 18 elements VIth period has 32 elements VIIth period hs rest of elements

Note - The number of valence electrons in atom of elements decides which elements will be first in period and which will be last

In group- 1 to 2 gp and 13 to 17 contain normal elements 3 to 12gp ndash transition elements 57 to 71 - lanthanides 89 to 103 - Actinides

Left hand side ndash metals Right hand side ndash nonmetals

Note- Hydrogen element has been placed at top of Ist group Electronic configuration of H is similar to alkali metal as both have 1 valence electron

V electron of gp I element -- 1 V electron of gp 2 element -- 2 V electron of gp 13 element -- 3 V electron of gp 14 element -- 4 V electron of gp 15 element -- 5 V electron of gp 16 element --6 V electron of gp 17 element -- 7 V electron of gp 18 element -- 8

English 1 Transformation of sentences

Sentences A sentence is a group of words which makes complete sense

Exercise 2Change the following sentences from

a Assertive sentencesb Imperative sentencesc Interrogative sentencesd Exclamatory sentences

Sentences can be changed from one grammatical form to another without changing the meaning of the sentence This is known as transformation of sentences

assertive to interrogative1 Nobody would like to be a fool

Who would like to be a fool2 Their glory can never fade

When can the glory fade3 Nobody can control the wind

Who can control the wind4 It matters little if I die

What though I die5 No man can serve two masters

Can any man serve two masters

Exercise 3Interchange of assertive and Exclamatory sentences

1 She leads the most unhappy lifeWhat an unhappy life she leads

2 This is indeed an interesting bookWhat an interesting book this

3 He is a very great manWhat a great man he is

4 It is a very lame excuseWhat a lame excuse

5 It is sad that she died so youngAlas she died so young

Class XISubject Topic Summary Execution

Hindi 2nd lang

पतर परम(परमचदर) पतर परम कहानी म एक निपता की इचछाओ का वणन निकया गया ह अपन बड पतर परभ -ास स निपता चतनय -ास का निवशरष परम था निपता को उसक जनम स ही बडी-बडी आशाए थी उसम दसर बट कतिशव-ास की अपकषा स- उतसाह की मातरा अमिधक थी वह उस इगलड भजकर बरिरसटर बनाना चाहत थभागय का खल भी बडा निनराला ह बीए की परीकषा क बा- वह बीमार पड गया डॉकटरो न भी जवाब - दि-या थाचतन -ास जी बहत ही कजस थ बवजह पस खच करना नही चाहत थ अगर गारटी मिमलती तो शाय- पस खच भी कर -त परत गारटी नही थी परिरणाम सवरप उनक बट का -हात हो गयाजब बट को समशान ल जा रह थ तो वहा काफी शोर गान बजान हो रह थ पछन पर पता चला निक निकसी निपता निपछल तीन साल स निबमार था और उसक ईलाज म रपया पानी की तरह बहाया पर ठीक नही हए परत उसक बट को तनिनक भी अफसोस नही था उसका कहना था उसन कोकतिशश तो कीयह -खकर चतनय-ास जी को आतम निगलानी हईतभी स उनका म परिरवतन हआ और बट का भोज काफी धमधाम स निकयाऔर वहइस पशचाताप की आग म जलत रह औला- स बढकर पसा नही होता ह इस बात को समझन म उनह काफी व लग गया

hellipContinue to next

BENGALI(2ND LANGUAGE)

পরথমঅধযায়-ঠাকরারীনদরনাথঠাকর

নয়ন দোচের হিমাচেররা া নাচেমই হিযাত হিছচেন ায়ানার উাররণ সবরপ নয়ন দোচের ারা হিা (াচেকর হিা হিচেতন এছাাও দেকান উৎস উপচেb রাহিতর দেক হিন করার উচেfচে(য তারা সযC হিকরচেরণ রনয পরীপ জবাহিচেয় তাচেত রপার হির 4Cরণ করচেতন ঠাকরা এই নয়ন দো হিমারচের দে(4 ং(ধর হিছচেন হিমাররা ায়ানার ষটানত পর(Cন কচের তারা হিনঃসব এই হিমাহিরর দে(4 ং(ধর গৈকাস নদর রায়চেৌধরী গৈকাস া নয়ন দোচের সমসত সমপহিতত ঋচেরণর াচেয় হিহিx কচের অহি(ষট যা আচেছ তাচেত হিপত

ইার হিপতার মতয ইচে পর নয়নচোচের ায়ানার দেগাটা কতক অসাধাররণ শরাদধ (াহিনতচেত অহিনতম ীহিপত পরকা( কহিরয়া ঠাৎ হিনহিয়া দেগ- ক) কার দো দেকান গচেলপর অং() কতা দেক ইার চেত কাচেক দোঝাচেনা চেয়চেছ গ) পরসঙগ কী কতার কতয পহিরসফট কচেরা

পরচে4র যাহিত রbা করা সমভ নয় তাই হিতহিন পতরচেক হিনচেয় ককাতায় সাস শর কচেরন গলপ কথচেকর আহিথCক অসথা নয়ন দোচের হিমাচের দেথচেক সমপরণC আাা কথচেকর হিপতা হিনচের দেষটায় অথC উপাCন করচেতন া উপাহিধ াচেভর নয তার াসা হিছনা আর দেসই কারচেরণ কথক তার একমাতর উততরাহিধকার চেয় তার হিপতার পরহিত কতজঞ কথক দো পা হি(চেচেছন হিনচের পরারণ ও মান রbার নয উপচেযাগী অথC হিনা দেষটায় পরাপত চেয়চেছন- এটাই তার কাচেছ পরম দেগৌরচের হি4য় চে মচেন কচেরন কাররণ (নয ভাণডাচের গৈপতক ায়ানার উজজব ইহিতাস অচেপbা দোার হিসeচেকর মচেধয গৈপতক দেকামপাহিনর কাগ তার কাচেছ অচেনক দেহি( মযান

TO BE CONTINUED

উ- ক) আচোয অং(টি রীনদরনাথ ঠাকচেরর দো ঠাকরা গচেলপর অং() কতা চেন আচোয গচেলপর গলপ কথকইার চেত নয়ন দোচের হিমাহিরর দে(4 ং(ধর গৈকাস ার কথা া চেয়চেছ গৈকাস া নয়ন দোচের সমসত সমপহিতত ঋচেরণর াচেয় হিহিx কচের অহি(ষট যা আচেছ তাচেত হিপত পরচে4র যাহিত রbা করা সমভ নয় তাই হিতহিন পতরচেক হিনচেয় ককাতায় সাস শর কচেরনগ) গৈকাস ার হিপতার মতযর পর নয়ন দোচের হিমাহিরর অহিসততব হিপত য় কচেয়কটা উৎস ও শরাদধ- (াহিনতচেত হিমাহিরর দে(4 কহিটক যয় চেয় হিগচেয় এচেক াচের দে(4 চেয় যায় তন তাচের গC করার মত আর হিকছই হিছ না-দেসই পরসচেঙগ এই উহিকত নয়নচোচের হিমাচেররা া নাচেমই হিযাত হিছচেন ায়ানার উাররণ সবরপ নয়নচোচের ারা হিা (াচেকর হিা হিচেতন এছাাও দেকান উৎস উপচেb রাহিতরচেক হিন করচেত হিগচেয় তারা সযC হিকরচেরণর নয পরীপ জবাহিচেয় তাচেত রপার হির 4Cরণ করচেতন তাই দেসকাচের ায়ানা দেহি(হিন সথায়ী চেত পারত না হিহিভনন উৎস শরাদধ- (াহিনতচেত সাধযা হিতহিরকত র করার নয হিমাহির হিহিকচেয় দেযত হ হিতC কা হিহি(ষট পরীচেপর দেত দেযমন অলপকাচের মচেধয হিনঃচে(4 চেয় যায়-নয়নচোচের হিমারচের অসথা তাই চেয়হিছ এই কারচেরণই কথক নয়নচোচের হিমারচের গা ভরা আমবর সয করচেত পারতনা

Physics Dimensional Analysis (Summary)

Q Find the dimensions of consts ab in relation

p=(bminusxlowastx)at

where p is the power x is the distance and t is time

Ans From principle of homogeneity dimension of b x2 are same Dim of b = dim of x2 = [L2] = [ML2T0]Dim of a = dim of ( b- x2)dim of (pt) = [M0L2T0][ML2T-2] [T-1] [T] = [M-1L0T2]

Chemistry Atomic Structure Drawbacks of Rutherfordrsquos model of

atom a According to Rutherfordrsquos model of atom electrons which are negativelycharged particles revolve around the nucleus in fixed orbits Thusb theelectrons undergo acceleration According to electromagnetic theory of Maxwell a charged particle undergoing acceleration should emitelectromagnetic radiation Thus an electron in an orbit should emitradiation Thus the orbit should shrink But this does not happenc The model does not give any information about how electrons aredistributed around nucleus and what are energies of these electrons Isotopes These are the atoms of the same

Properties of electromagnetic radiationsa Oscillating electric and magnetic field are produced by oscillating charged particles These fields are perpendicular to each other and both areperpendicular to the direction of propagation of the waveb They do not need a medium to travel That means they can even travel invacuum

Characteristics of electromagnetic radiationsa Wavelength It may be defined as the distance between two neighbouring crests or troughs of

element having the same atomicnumber but different mass numbere g 1H11H21H3

Isobars Isobars are the atoms of different elements having the same massnumber but different atomic numbere g 18Ar40 20Ca40

Isoelectronic species These are those species which have the same numberof electrons

Electromagnetic radiationsThe radiations which are associated withelectrical and magnetic fields are called electromagnetic radiations When anelectrically charged particle moves under acceleration alternating electricaland magnetic fields are produced and transmitted These fields aretransmitted in the form of waves These waves are called electromagneticwaves or electromagnetic radiations

wave as shown It is denoted by λb Frequency (ν) It may be defined as the number of waves which passthrough a particular point in one secondc Velocity (v) It is defined as the distance travelled by a wave in onesecond In vacuum all types of electromagnetic radiations travel with thesame velocity Its value is 3 times10 8m sec-1 It is denoted by v

d Wave number Wave number is defined as the number of wavelengths per unit lengthVelocity = frequency timeswavelength c = νλ

Plancks Quantum Theory- o The radiant energy is emitted or absorbed not continuously but discontinuously in the form of small discrete packets of energy called lsquoquantumrsquo In case of light the quantum of energy is called a lsquophotonrsquoo The energy of each quantum is directly proportional to the frequency of the radiation ie E α υ or E= hυ where h= Planckrsquos constant = 6626 x 10-27 Js o Energy is always emitted or absorbed as integral multiple of this uantum E=nhυ Where n=1234Black body An ideal body which emits and absorbs all frequencies is calleda black body The radiation emitted by such a body is called black body radiation

Photoelectric effectThe phenomenon of ejection of electrons from thesurface of metal when light of suitable frequency strikes it is calledphotoelectric effect The ejected electrons are called photoelectrons

Biology Chapter - 02Systematics and Five Kingdoms

Scientists divide the whole living organisms into two kingdom first and ultimately by five kingdom at last

In the earlier systems of classifications organisms are divided into kingdom plantaeand kingdom animalia on the of presenceof cell wall their modes of nutrition and movements

Some problem arise like fungi share manycharacteristic withplant despite their heterotrophic nutrition bacteria protozoa areunicellular present in both kingdom Toovercome this third kingdom Protista isintroduced which include

unicellularorganisms But there is also another

problem Allunicellular organisms are not similar kind The cellular structure of prokaryotes is verydifferent from that of other organismsEukaryotes possess a true nucleus and allcell organelles that are not present inprokaryotes So the fourth kingdom Monerais introduced which include unicellular prokaryotes (bacteriaamp blue green algae)

bull Still some problem arise in kingdomplantae

So in 1969 R H Whittakar proposedanew five kingdom System of classification

i) Kingdom Monera - unicellular prokaryotes

ii) kingdom Protista - unicellular eukaryotes

iii) Kingdom Fungi - uni or multicellular fungi with cell wall but without chlorophyll

iv) Kingdom Plantae - Multicellular Plants

v) Kingdom Animalia - Multicellular Animals

EVS Chapter 1 ndash Modes of Existence

An agricultural society

An agricultural society also known as an agrarian society is a society that constructs social order around a reliance upon farming More than half the people living in that society make their living by farming

People in an agricultural society generally lead a more settled lifestyle than those in nomadic hunter-gatherer or semi-nomadic pastoral societies because they live permanently near the land that is farmed Agricultural settlements tend to develop in areas of convenience near bodies of water which is used for both crops and transportation or along trade routes Not everyone in an agricultural society is a farmer Some people make a living trading or making and selling goods such as tools used for farming

Another way to define an agrarian society is to see the total amount of production in a nation In an agrarian society cultivating the land is the main source of wealth Such a society can recognize other means of subsistence and work habits but emphasizes the importance of agriculture and livestock Agrarian societies have existed in various parts of the world for 10000 years and continue to exist today They have been the most common form of socio-economic organization for most of recorded human history

Q) Write the features of agricultural society

Ans - Structure and Features of Agrarian Society1 Occupational Structure

An agrarian society is generally associated with the domestication of plants and animals The domestication of plants means farming and that of animals means herding Often there is mixture of farming and the use of such domesticated animals as cow goat and sheep

2 Forms of Land Ownership in Agrarian SocietiesGenerally there are landlords supervisory farmers cultivators and share croppers The landholders own the land but do not work on it They let it out for sharecropping The supervisory farmers are those who live by having their land cultivated by hired labourers The cultivators cultivate the land for themselvesThe share-croppers are those who live by tilling other peoplersquos land or a crop-sharing basis The artisans own their means of production and produce by their own labour in their homesteads

3 Village Community System An agrarian society is highlighted by

the institution of village community system The agrarian economy made fixed dwelling houses necessary Living close together for protection and co-operation and living nearer to the land gave birth to agricultural villages The village is not only the residential place of farmers it is also the social integrator

4 Minimal Division of Labour Another structural feature of agrarian society is a minimal division of labour Except for the basic division founded on age and sex differences there are few specialized roles There is only one predominant type of occupation ie domestication of plants and animals For all the people the environment physical as well as social is the same

5 Role of Family The farm family is of the patriarchal type the father is the final arbiter in most of the familyrsquos major decisions The life of ail men and women is merged in family life Since there are not many special organizations family is the only organisation to perform the tasks of aid and protection

6 Sense of Unity The members of an agrarian society exhibit a strong in-group feeling Since the whole of their social lives is wrapped up in a society which is physically economically and socially homogenous they are inclined to view the entire outside world as an out group

7 Informal Social Control An agrarian society is regionally divided into villages In a village community the force of traditional mores is more dominant than in the urban community In the village everybody is known to everybody The members in a village community help each other and share the joy and sorrows of each other Crime in an agrarian society is rare

8 Simplicity and Uniformity Life of the people in an agrarian society is marked by simplicity and uniformity Their main occupation is agriculture which largely depends upon the vagaries of nature An agrarian society is a religious society

Math Compound angles Compound angles The algebraic sum of two or more angles is called a compound angle If A B C be three angles then A+B B+C C+A A-B B-C A-C A+B-C etc are compound angles In this chapter we shall discuss the trigonometrical ratios of compound angles Theorem 1 If A B and A+B are all pisitive acute angles theni) sin( A+B) = sin A cos B + cosA sinBii) cos(A+B) = cosA cosB- sinA sinBTheorem 2If A and B are positive acute angles and AgtB theni) sin(A-B) = sin A cosB- cos A sinBii) cos(A-B) = cos A cos B+ sin A sin BTo prove that i) sin(A+B) sin (A-B) = sin2 A - sin2 B = cos2 B- cos2 A

Example 1 Prove that tan70deg=2tan50deg+tan20degSolutiontan70deg = tan(50deg + 20deg)Or tan70deg=(tan 50deg+tan 20deg)(1-tan50degtan20deg) or tan70deg (1 ndash tan 50deg tan20deg) = tan50deg+tan20degor tan70deg= tan70deg tan50deg tan20deg+ tan50deg + tan20deg = cot20deg tan50deg tan20deg + tan50deg + tan20deg = 2 tan50deg+ tan20degExample 2 If A + B = 45deg show that (1 + tanA) (1 + tanB) = 2Solutiontan(A + B) =( tan A + tan B) (1 - tan

ii) cos(A+B) Cos(A-B) = cos2 A- sin2 B = cos2 B -sin2 AProof i) LHS= sin(A+B)sin(AminusB) [Recall sin(αminusβ)=sinαcosβminuscosαsinβ And sin(α+β)=sinαcosβ+cosαsinβ]= (sinAcosB+cosAsinB)times(sinAcosBminuscosAsinB)= sin2Acos2Bminuscos2Asin2B [Recall sin2α+cos2α=1 From above we can then assume correctly that sin2α=1minuscos2α AND cos2α=1minussin2α] = sin2A(1minussin2B)minussin2B(1minussin2A) = sin2Aminussin2Asin2Bminussin2B+sin2Asin2B = sin2Aminussin2B= 1-cos2A-(1-cos2B) = cos2 B- cos2 A = RHSii)LHS= cos (A+B) cos(A-B) [ cos(A+B) = cos AcosB- sinAsinBCos(A-B) = cosAcosB+ sinAsinB]= cos2 A Cos2 B- sin2 A Sin2 B= cos2 A( 1-sin2 B) - (1- cos2 A) sin2 B= cos2 A- cos2 A sin2 B- sin2 B+ cos2 A sin2 B=cos2 A- sin2 B=1- sin2 A-(1-cos2 B) = cos2 B- sin2 A= RHSTangent formulae for compound anglesi)tan (A + B) = tan A + tan B1-tan A tan Bii)tan (A ndash B) = tan A-tan B1+tan A tan Biii) cot (A + B) = cot Acot B-1cot A+cot B(viii) cot (A ndash B) = cot Acot B+1cot B-cot A

A tan B) Or 1= (tan A+ tanB) (1-tan A tanB) Or tanA + tanB + tanA tanB + 1 = 1 + 1Or tanA (1 + tanB) + (1 + tanB) = 2Or (1 + tanA) (1 + tanB) = 2Example 3 Find the value of sin 15degSolution sin 15deg= sin(45deg-30deg) = sin45degcos 30deg- cos45degsin30deg =(1radic2) (radic32) -(1radic2) (12) = (radic3-1) 2radic2Example 4 If sin A = 1 radic10 and sin B = 1 radic5 where A and B are positive acute angles then what is A + B SolutionWe know that sin (A + B) = sin A cos B + cos A sin B= [1 radic10] [radic(1 minus 1 5)] + [1 radic5] radic(1 minus 1 10)= [1 radic10] [radic4 5] + [1 radic5] [radic9 10]= [1 radic50] times (2 + 3)= 5 radic50 = 1 radic2

sin (A + B) = sin π 4rArrHence A + B = π 4Example 5 If A + B = 225o then find [cot A] [1 + cotA] times [cot B] [1 + cot B]Solution[cot A] [1 + cotA] times [cot B] [1 + cot B] = 1 [(1 + tan A) times (1 + tan B)]=1 [tan A + tan B + 1 + tan A tan B] [ tan (A + B) = tan225o]∵

tan A + tan B = 1minus tan A tan BrArr= 1 [1 minus tan A tan B + 1 + tan A tan B]= 1 2

COMMERCE

CLASSIFICTION OF HUMAN ACTIVITIES-ECONOMIC AND NON-ECONOMIC

Firstly we shall recall the previous class for 5 mins especially for the absentees and for also the rest of the students who were there

Today at first we briefly discuss the earlier portions of the chapter

1Business-It includes all those economic activities which are concerned with production and exchange of goods and services with the object of earning profit Example A factory shop beauty parlour also business enterprises

2Profession ndashThe term profession means an occupation which involves application of specialized knowledge and skills to earn a living For Example Chartered Accountancy medicine law tax consultancy are example of professions

Questions1What are the main features of ProfessionAnswer The main features of a profession are as follows a Specialised body of knowledge-Every profession has a specialised and systematised body of knowledge b Restricted entry- Entry to a profession is allowed only to those who have completed the prescribed education and have the specialised examination c Formal education and training ndashA formal education and training is given to the person who wants to acquire the professional

3Employment-Employment mean an economic activity where people work for others in exchange for some remuneration (salary)The persons who work for others are called lsquoemployeesrsquo The persons or organizations which engage others to work for them are called lsquoemployersrsquoEg A doctor working in a hospital is employment as he is working for a salaryA lawyer may serve as a law officer in a bank

With this we shall proceed with the features of both Profession amp Employment

The main features of a profession are as follow

a Specialised body of knowledge b Restricted entry c Formal education and training d Professional association e Service motive f Code of contact

The main features of an employment are as follows

a In employment a person works for others called employer

b An employee provides personal service

c There is a service agreement or contract between the employee and the employer

d The employee has to obey the order of the employer

e No capital investment is made by the employer

Various examples of Employment are as follows

aA teacher teaching in a school or collegeb An engineer employed in Municipal Corporation of DelhicAn accountant working in the accounts department of a companydA doctor working in a hospital

Note In all the above examples of employment the individual who is involved in each example is working as an employee for a salary under an employer

qualification(MBBSCALLB)d Service motive ndashProfessionals are expected to emphasis service more on their clients rather than economic gain f Code of Conduct-The activities of professionals are regulated by a code of conduct

2 What are the main features of EmploymentAnswer The main features of an employment are as followsa In employment a person works for others called employerb An employee provides personal servicec There is a service agreement or contract between the employee and the employerd The employee has to obey the order of the employere No capital investment is made by the employer

3 Give various Professions and their respective Association are given below

Professions

Professional

Professional association

Medical profession

Doctor Medical Council of India

Law profession

Lawyers Bar Council of India

Accounting Profession

Chartered

The Institute of Chartered Accounts of India( ICAI)

Engineerin Engineers The

g Profession

institute of Engineers (India)

Accounts Basic accounting terms

Today we will give you some questions from the previous study material

Questions6) Define accounting7) What do you mean by debit

and credit8) Explain the types of account9) Define the following terms

a) Assetsb) Capitalc) Purchased) Debtorse) Transactions

10) Name the types of accounts given below

a) Krishnas accountb) Machinery accountc) Royalty accountd) Salary accounte) Furniture accountf) Audit fee account

Economics Basic Economic ConceptsSub topic

UTILITY

Before starting todayrsquos class we shall recall the last class which was about UTILITY AND THE FEATURES OF UTILITY

Now we shall proceed with the further topics of the chapter

Todayrsquos topic from the chapter lsquo Basic Economic Conceptsrsquo will be TOTAL UTILITY amp MARGINAL UTILITYNow let us quickly revise the concept of utility with an example ie goods and services are designed because they have an ability to satisfy human wantsThis feature of being able to satisfy human wants is termed as utility For example we derive utility from WiFi services as it gives us satisfaction by connecting us to our friends and family through social media here consumers derive utility from WiFi services

From the above concept we shall start with todayrsquos topicEconomists have defined TOTAL UTILITY (TU) as the total satisfaction obtained by consuming a given total amount of a good and serviceFor example the total satisfaction obtained from eating 10 mangoes is the total utility of 10 mangoes

MARGINAL UTILITY (MU) is the additional satisfaction derived from each additional unit

Questions1 What is Total Utility (TU)

Answer Total Utility (TU) is the

aggregate of the utility that a consumer derives from the consumption of a certain amount of a commodityTU=MU1+MU2++MUn

2 What is Marginal UtilityAnswer

Marginal Utility (MU) is the additional made to the total utility as consumption is increased by one more unit of the commodityMU= TUn ndashTUn-1

NoteOften economists tend to

subdivide utility into an imaginary unit called UTIL

consumed In this casethe utility obtained from each mango as it is consumed as the MU of that mango It is also defined as the addition made to the total utility when an additional unit is consumed Often economists tend to subdivide utility into an imaginary unit called UTIL

Note As a consumer increases the consumption of a good over period of time the total utility or total satisfaction derived from it increases to appoint and thereafter it decreasesHowever as the consumer keeps on consuming the good the marginal utility or the additional utility derived from it decreases

SubjectBusiness studies

Topic

BUSINESSENVIRONMENT

Summary

Now quickly let us revise the earlier points that we have already done in the last class and let us proceed with the other topics that are there in the chapter

Firstly we will recall the internal and external factors of micro environment and then we shall proceed in details

Meaning and list of internal and external factors

aInternal factorsInternal factors refer to all the factors existing within a business firm The internal factors are considered controllable because the enterprise has control over these factorsFor an example a company can alter its organization structure policies programmes employees physical facilities and marketing mix to suit the changes in the environmentList of internal factors areCorporate culture mission and objectives top management organizations structure company image and brand equity company resources

b External factorsExternal factors refer to those individual and groups and agencies with which a particular business organization comes into direct and frequent contact in the course of its functioningThese individuals and groups are known as STAKEHOLDERS because they have a stake (financial interest ) in the working and performance of the particular business List of external forces (stakeholders)Customers competitors investors suppliersmiddlemen (marketing intermediaries)

Execution 1 What do you mean by internal

factors in micro environmentAnswerInternal factors refer to all the factors existing within a business firm The internal factors are considered controllable because the enterprise has control over these factorsFor an example a company can alter its organization structure policies programmes employees physical facilities and marketing mix to suit the changes in the environment

2 What do you mean by external factors in micro environment

AnswerExternal factors refer to those individual and groups and agencies with which a particular business organization comes into direct and frequent contact in the course of its functioningThese individuals and groups are known as STAKEHOLDERS because they have a stake (financial interest) in the working and performance of the particular business

3Who are stakeholdersSTAKEHOLDERS are individuals and groups who have a stake (financial interest ) in the working and performance of the particular business 4Discuss the internal factors in briefa Corporate CultureThe values beliefs and attitudes of the founders and top management of the company exercise

financers publics

customers

suppliersfinancers

competitors

middlemen

publics

Fig STAKEHOLDERS OF A COMPANY

Apart from micro environment the other main dimension of business environment isMacro environment Macro environment refers to the general environment or remote environment within which a business firm and forces in its micro environment operateA company does not directly or regularly interact with the micro environmentTherefore macro environment is also known as indirect action EnvironmentThe macro environment forces are less controllable than the micro forces

Macro environment consists of the following components

POLITICAL AND LEGAL ENVIRONMENT

ECONOMIC SOCIAL AND ENVIRONMENT

CULTURAL

ENVIRONMENT

TECHNOLOGICAL ENVIRONMENT

a strong influence on what the cmpaany stands for how it does things and what it considers importantbMission and objectivesThe business philosophy and purpose of a comoany guide it prioritiesbusiness strategiesproduct market scope and development scope

cTop management structurethe composition of board of directors the degree of professionalization of management and the organizational structure of a company have important bearing on its business decisions

dPower structureThe internal power relationship between the board of directors and the chief executive is an important factor

eCompany image and brand equityThe image and brand equity of the company play a significant role in raising finance forming alliance choosing dealers and suppliers launching new products entering foreign markets

5 What is Macro environmentAnswerMacro environment refers to the general environment or remote environment within which a business firm and forces in its micro environment operateA company does not directly or regularly interact with the micro environmentTherefore macro environment is also known as indirect action EnvironmentThe macro environment forces are less controllable than the micro forces 6 What are the components of macro environmenta Political and legal environmentb Economic environmentc Social and cultural environmentd Technological environment

BUSINESS FIRM

Fig COMPONENTS OF MACRO ENVIRONMENTPolitical science

Introduction to political science

Comparative politics and itrsquos scope Comparative politics is the second major dimension of political scienceIt is also a very vast area of study and a very large number of political scientists even treat it as an autonomous area of study within the board ambit of political scienceScope of comparative politics-

1 All political structures -Comparative politics includes the study of all structures formalnon formal governmental and extra governmental which are directly or indirectly involved in politics in all the countries of the world

2 Functional studies- Comparative politics seeks to study politics less from the point of view of the legal institutions in terms of their powers and move from the point of view of their functions which constitute the political process and their actual Operation in the environment

3 Study of political behaviour- Another important part of its scope is the study of the actual behaviour of the people in the process of politics

4 Study of similarities and differences- comparative politics also undertakesan analysis of the similarities and differences among political process and functions

5 Study of all political systems -comparative politics seeks to analyse the actual behaviour and performance of all political systems western as well as non western

6 Study of the environment and infrastructure of politics-The study of politics demands a study of the psychological sociological economic and anthropological environment in fact the social environment as a whole in which each political system operates

7 Study of political culture- political culture is composed of attitudesbeliefs emotions and values of a society that relate to the political system or politics

8 Study of political participation- Political participation is a universal processThe only difference is that while in some states it is limited in others it is wider

9 Study of political process- political

Answer the following questions-

What is comparative politics

What are the scope of comparative politics

Homework- learn

processes like decision makingpolicy making judicial process leadership recruitment process and others are always at work in all political systems

The scope of comparative politics is very comprehensive It includes everything that falls within the area of political activity and political process

History CAMBRIDGE VIEW ABOUT

THE PARTITION

AND REFUTATION

OF CAMBRIDGE

VIEW

Cambridge view about the Partition The Cambridge school of historians have interpreted that opposition to partition scheme was made entirely by the elitist groups They hold the view that Lord Curzon planned to partition the Bengal for administrative purposeREFUTATION OFCAMBRIDGE VIEW The Rationalist historians have rejected the interpretations of the Cambridge School of historians on various grounds

1 QUESTION State different views of historians regarding Partition of Bengal

ANSWER Cambridge historians believed that Lord Curzon partitioned Bengal for administrative reasons only and not for the political motive The Middle class elitist group protested because of their petty interest The Hindu zamindars protested as they have to spend more money for managing their estatesThe lawyers of Calcutta High court feared to lose their clientBut according to the nationalist Historians was-

2- The ultimate object of Lord Curzon was to crush the unity of Bengal politicians

3- If Bengal becomes a separate province Bengali speaking 16 million people of western part would become minority under Hindi speaking people of Bihar and Oriya speaking people of Orissa

4- The bureaucrats expected that the protest movement would die down quickly

5- Lord Curzon used the Muslim community in his political game

6- Idealism had great contribution in the protest against partition

7- The people of the every section of society were affected by the partition of Bengal

Computer Science

Numbers Convertion of dcimal number to octal numberThe decimal numeral system is the standard system for denoting integer and non-integer numbers It is the extension to non-integer numbers of the Hindu-Arabic numeral system For writing numbers the decimal system uses ten decimal digits a decimal mark and for negative numbers a minus sign - The decimal digits are 0 1 2 3 4 5 6 7 8 9 the decimal separator is the dot in many countries

The octal numeral system or oct for short is the base-8 number system and uses the digits 0 to 7 Octal is sometimes used in computing instead of hexadecimal perhaps most often in modern times in conjunction with file

permissions under Unix systems It has the advantage of not requiring any extra symbols as digits It is also used for digital displays

Follow these steps to convert a decimal number into octal form

1 Divide the decimal number by 82 Get the integer quotient for the next iteration (if the number will not divide equally by 8 then round down the

result to the nearest whole number)3 Keep a note of the remainder it should be between 0 and 74 Repeat the steps until the quotient is equal to 05 Write out all the remainders from bottom to top This is the solution

For example if the given decimal number is 8453

Division Quotient Remainder

8453 8 1056 5

1056 8 132 0

132 8 16 4

16 8 2 0

2 8 0 2

Then the octal solution is 20405

Subject Eng Literature (The Tempest ndash William Shakespeare) Topic Act I Scene 1 Lines 33 to 67 (End of scene) Date 16th April 2020 (4th Period)

[Students should read the original play and also the paraphrase given in the school prescribed textbook]Summary Questions amp Answers

[SUMMARY OF THE ENTIRE SCENE]

o The play starts with the scene of a severe storm at sea Alonso (King of Naples) Sebastian (Alonsorsquos brother) Ferdinand (Alonsorsquos son) Gonzalo Antonio (the usurping Duke of Milan) are in a ship in the midst of the storm

o The mariners are trying their best to control the vessel from running aground and are totally following the orders of their Master the Boatswain They have scant success

o The mariners become extremely unhappy and annoyed when most of the passengers arrive on the deck thereby hampering their effort to save the ship There is serious confrontation between them and the passengers who are part of the Kingrsquos entourage

o The mariners could not save the ship

SUMMING-UP

(i) Vivid description of the scene which gives a realistic description of terror and confusion of a tropical storm

(ii) Shows Shakespearersquos accuracy of knowledge in describing the naval operations and also matters of seamanship

(1) GONZALO Ill warrant him for drowning (L 45-57)

though the ship were no stronger than a nutshell and as leaky as an unstanched

wenchBOATSWAIN Lay her a-hold a-hold Set her two courses Off to

sea again lay her offMARINERS All lost To prayers to prayers All lostBOATSWAIN What must our mouths be coldGONZALO The king and prince at prayers Lets assist them

For our case is theirsSEBASTIAN Im out of patienceANTONIO We are merely cheated of our lives by drunkards

This wide-chopped rascal - would thou mightst lie drowning the washing of ten tides

(a) What does Antonio say at the insolent manners of the boatswain just before the given passage

Being irritated at the insolent manners of the boatswain just before the given extract Antonio the Duke of Milan calls him a worthless dog son of a woman without any morals an arrogant and disrespectful noisemaker He says that the boatswain deserved to be hanged(b) What statement does Gonzalo repeat about the boatswain

Gonzalo shows his faith that the boatswain is not destined to die by drowning He is destined to be hanged and nothing can alter this decree of destiny He says that even if the ship was as frail as a nutshell the boatswain could not be drowned for his destiny was to be hanged(c) What do the passengers do when they have lost all hope of their survival

When the passengers have lost all hope of survival they take

(iii) The opening scene justifies the title ndash The Tempest

UNANSWERED QUESTIONS

(i) The King always travels with his entire fleet including his soldiers Where were the other ships

(ii) Why was the ship in that area Where was it coming from or going where

(iii) The ship broke apart What happened to those who were in the ship

(We shall get the answer to the above questions as the play progresses)

leave of life with fervent prayers The mariners take their last hearty drink and are ready for death(d) What blame does Antonio put upon the mariners and the boatswain Antonio rebukes the mariners that these drunkards have brought them to the present crisis by neglecting their duties He blames them saying that they are going to lose their lives entirely for the negligence of the boatswain and his fellows(e) What does Antonio say while cursing the boatswain

Antonio gives vent to his wrath upon the boatswain in particular He calls the boatswain a wide-mouthed rascal who deserves to be hanged on the sea-shore at low water mark so that ten tides might wash over his body and take out of him all the liquor that he has been drinking

Class XIISubject Topic Summary ExecutionHistory Topic

1 1935 ACT AND WORKING OF PROVINCIAL AUTONOMYCONGREE AND OTHER MINISTERSSUB TOPIC GOVERNMENT OF INDIA ACT1935

Government of India Act 1935 This act established a lsquoFederation of Indiarsquo made of British Indian provinces and Indian states and provided for autonomy with a government responsible to the elected legislature in every provinceThis act introduced abolition of Diarchy at provinces The entire provincial administration was introduced to the responsible ministers who were controlled and removed by the provincial legislature The provincial autonomy means two things First The provincial governments were wholly responsible to the provincial legislature Secondly Provinces were free from outside control and interference in the large number of matters The act divided the powers between the centre and provinces in terms of three lists- Federal list( for centre) Provincial list (for province) and concurrent list (for both) Residuary powers were given to the viceroy In the election under the government of India Act the Congress swept the poll the mandate of the people came in favour of the congress so far as general Hindu seats were concerned The Congress did not get a single Muslim seates in Bombay CP UP Sind and BengalIn five provinces Congress had yhe clear majority In BengalNWFPAssam and Bombay Congress emerged as a single largest partyOn the other side the performance of the Muslim League was badThus the Congress formed ministers in 7 provinces out of 11 provinces Coalition ministry was also formed in two other provincesOnly BENGAL AND Punjab had non- congress ministries

1 QUESTION What was the main change introduced by the Government of India ActANSWER a) The Act gave more

autonomy to the provinces b) Diarchy was abolished at the

provincial levelsc) The Governor was the head of

the executived) There was a council of

ministers to advise him The ministers were responsible to the provincial legislatures who controlled them The legislature could also remove the ministers

e) The Governors still retained special reserve powers

2 QUESTION Why did the federal scheme introduced by the Government of India Act 1935 never come into operation

ANSWER The Federal structure of the Government of India was to be composed with the Governor General and Council of ministers The Federal legislature was to be Bicameral legislature- The council of states and the House of Assembly The ministers were to be chosen by the Governor general and they were to hold the office during his pleasure

The provinces of British India would have to join the federation but this was not compulsory for the princely states

This federation never materialised because of the lack of support from the required number of

princely statesThis act was refused and

rejected by the princes the Congress and the Muslim League

Thus both Congress and the League participated in the election of 1937 Thus the federal part was never introduced but the provincial part was put into operations

Bengali 2nd

Language

াচেরর পরাথCনা(কহিতা )

াচেরর পরাথCনা কহিতাটি কহি (ঙখ দেঘাচে4র দো আচো য কহিতায় াচেরর পতর হমায়ন কঠিন দেরাচেগ আxানত ার ঈশবর া আললার কাচেছ পরাথCনা কচেরচেছন তার পচেতরর ীন হিফহিরচেয় হিচেত এই কহিতায় ার পচেতরর ীন হিভbা দেচেয়চেছন ারার এমনহিক হিনচের ীন হিসCচেনর হিহিনমচেয় হিতহিন তার দেছচের ীন হিফচের দেপচেত দেচেয়চেছন তার দেছচের এই দেরাচেগর ন য হিতহিন হিনচেচেকই ায়ী কচেরচেছন তার হিনচের করা পাপচেকই হিতহিন ায়ী কচেরচেছন এছাা রানৈনহিতক ও আথCসামাহিক অসথার কথা তচে ধরা চেয়চেছ এই কহিতায় ার তার হিনচের পাপ কমCচেকই ায়ী কচেরচেছ ার অন যায় ভাচে দেপহি((হিকতর মাধ যচেম অপররা য কচেরচেছ আর এই অন যায় কাচের ন যই তার পহিরাচের হিপযCয় এচেসচেছ দে এক পরকার মানহিক নধন ইহিতাচেসর ার হিপতা চেয় সবাভাহিকভাচে ভাচোাসা দে মমতা দেথচেক মকত চেত পাচেরনহিন তাই হিপতা চেয় আললা া ভগাচেনর কাচেছ পতর হমায়চেনর পরানহিভbা দেচেয়চেছন ার আললা া ভগাচেনর কাচেছ াহিনচেয়চেছন তার হিনচের ীন হিসCন হিচেত হিতহিন রাী তার হিহিনমচেয় পচেতরর ীন হিফচের দেপচেত দেচেয়চেছন াচেরর হিপতসভ হিচেকর কথা এই কহিতায় ফটিচেয় দেতাা চেয়চেছ হিপতা পচেতরর হিরাহিরত মান নধচেনর কথা তচে ধরা চেয়চেছ

হিচে(4 হিকছ াইচেনর তাৎপযC১) ldquoদেকাথায় দেগ ওর সবচছয দেৌন দেকাথায় কচেরায় দেগাপন bয়ldquoউততর) াচেরর পতর হমায়ন কঠিন দেরাচেগ অসসথ তাই তার দেযৌন াহিরচেয় যাচেচছ এই দেরাচেগ তাচেক দেগাপচেন কচেরকচের াচেচছ তার সক (হিকত ধীচের ধীচের bয় চেচছ তাই হিপতা চেয় ার আললার কাচেছ হমায়চেনর পরান হিভbা দেচেয়চেছন২) ldquoাগাও (চেরর পরাচেনত পরানতচের ধসর (ন দেযর আান গানldquoউততর) াচেরর পতর হমায়ন কঠিন দেরাচেগ আxানত তাই ার আ দে(াচেক মমCাত (চেরর পচেথ পরানতচের আান গান ধবহিনত দোক দেসই আান গান আললার কাচেছ দেযন চে যায় আললা দেযন এই আহিতC শচেন পচেতরর ীন হিফহিরচেয় দেয় ৩)ldquoনাহিক এই (রীচেরর পাচেপর ীানচেত দেকানই তরারণ দেনই ভহি4চেতরldquoউততর) হমায়চেনর অসসথতার ন য ার হিনচেচেকই ায়ী কচেরচেছন কারন ার অচেনক রা য অন যায় ভাচে কচেরচেছ তাই তার এই পাপ কাচের ন য তার ঘচের আ হিপ এচেসচেছ এই অন যায় কাচের ন য তার মহিকত দেনই তাই ার আললার কাচেছ এই পাপ কাচেযCর ন য bমা পরাথM

Hindi 2ndlang

-ासी(जयशकर परसा-)

-ासी जयशकर परसा- की एक ऐसी कहानी ह जिजसम भारतीय ससकनित और राषटरीयता का सवरगजीतहोता ह इस कहानी म इरावती एक निहद कनया ह जिजस मलअचछो न मलतान की लट म पकडा और -ासी बना दि-या उस 500 दि-न -कर काशी क एक महाजन न खरी-ा दसरी -ासी निफरोजा ह वह गलाम ह निफरोजा को छडान क कतिलए अहम- को 1000 सोन क कतिसकक भजन थ जो अभी तक नही आए थ राजा साहब कठोर होत हए भी निफरोजा को निबना धनराकतिश क कतिलए उस म कर -त ह वनिफरोजा को अहम- को समझान की बात कहत हकहानी क अत म हम -खत ह निक इरा वती और जाटो क सर-ार बलराज का मिमलन होता हअहम- को यa म मार दि-या जाता ह वहा निफरोजा की परसननता की समामिध बनती ह वहा एक फल चढती ह और डीजल आती ह निफरोजा उस समामिध की आजीवन -ासी बनी रहती हलखक अपन उददशय अथात -ास परथा पर परकाश डालन और इस परथा क कारण होन वाल -ातो क दखो को दि-खान म पणता सफल हए ह

helliphellipContinue to next

Biology Reproductio Today we will discuss about vegetative Q1 Name some vegetative propagules

n in Organisms

propagation of plants The process of multiplication in which fragments of plant body function as propagule and develop into new individual is called vegetative propagation The units of such propagation are runner rhizome tuber bulb etc

and the speciesinvolvedVegetative propagules

Parts involved

Bulb StemBulbil BulbilRhizome Stem Runner Stem Tuber Stem Offset Stem Leaf buds Leaves Suckers Stem

Corns Stem stolon

Q2 State advantages of vegetative propagation

i) Rapid methodii) Sure and easy methodiii) Useful in plants that cannot

produce viable seeds or long seed dormancy

iv) Maintains purity of raceQ 3 Banana fruit is said to be parthenocarpic where as turkey is said to be parthenogenetic WhyBanana develops without fertilization from an unfertilized ovary thus is parthenocarpicIn turkey the ovum or female gamete developinto a new chick without fertilization thus isparthgenetic

Q4 Why is water hyacinth is called as a ldquoTerror of Bengalrdquo Water hyacinth can

propagatevegetatively all over the water body in a short per short period of time This resulted increased biochemicaloxygen oxygen demand of water body causing mortalityof fishes It is very difficult to get rid off them Thus known as terror of Bengal

Chemistry

Solid state GENERAL CHARACTERISTICS OF SOLID STATEIn nature the particular state of matter is governed by two opposing forces at given set of temperature and pressure These forces are intermolecular force of attraction and thermal energy If intermolecular force of attraction is high as compared to thermal energy particles remains in closest position

Intext QuestionsQ1 Classify the following solids as crystalline and amorphous Sodium chloride quartz glass quartz rubber polyvinyl chloride Teflon

A1 Crystalline

and hence very less movement in particles is observed In this case solid state is the preferred state of matter

Let us revise the general characteristics of solid

i) Fixed mass volume and shape

ii) Strong intermolecular force of attraction

iii) Least intermolecular space

iv) Fixed position of constituent particles

v) Incompressible and rigid

Q2 what type of interactions hold the molecules together in a polar molecular solid[CBSE 2010]A2 The molecules in a solid are held together by van der Waals forces The term van der Waals forces include hydrogen bonding dipole-dipole attraction and London dispersion forces All molecules experience London dispersion forces In addition polar molecules can also experience dipole-dipole interactions So the interactions that holds the molecule together in polar molecular solid are London dispersion force and dipole-dipole interactionsQ3 Write a feature that will distinguish a metallic solid from an ionic solid [CBSE 2010]A3 Metals are malleable and ductile whereas ionic solid are hard and brittle Metallic solid has typical metallic lustre But ionic solid looks dullQ4 Write a point of distinction between a metallic solid and an ionic solid other than metallic lustre [CBSE 2012]A4 Metals are malleable and ductile whereas ionic solid are hard and brittleQ5 Write a distinguish feature of metallic solid [CBSE 2010]A5 The force of attraction in

solid Sodium chloride Quartz Amorphous solid Quartz glass rubber polyvinyl chloride Teflon Q2 why glass is considered as super cooled liquidA2 Glass shows the tendency to flow at slower rate like liquid Hence they considered as super cooled liquidQ3 why the window glass of old buildings show milky appearance with timeA3 Glass is an amorphous solid Amorphous solid has the tendency to develop some crystalline character on heating Due to heating in day over the number of years glass acquires some crystalline character and show milky appearanceQ4 why the glass panes fixed to window or doors of old building become slightly thicker at bottomA4 Glass is super cooled liquid It has the tendency to flow down very slowly Due to this glass pane becomes thicker at the bottom over the timeQ5 Sodium chloride is a crystalline solid It shows the same value of refractive index along all the direction TrueFalse Give reasonA5 FalseCrystalline solid shows anisotropy in properties That is it shows different values for the given physical property in different direction All the crystalline solids show anisotropy in refractive index Therefore sodium chloride will show different values of refractive index on different directions

Q6 Crystalline solid are anisotropic in nature What does this statement means

between the constituent particles is special kind of electrostatic attraction That is the attraction of positively charged kernel with sea of delocalized electronsQ6 which group of solid is electrical conductor as well as malleable and ductile [CBSE 2013]A6 Metallic solidQ7 why graphite is good conductor of electricity although it is a network (covalent solid)A7 The exceptional property of graphite is due to its typical structure In graphite each carbon is covalently bonded with 3 atoms in same layer The fourth valence electron of each atom is free to move in between different layersThis free electron makes the graphite a good conductor of electricity

[CBSE 2011]A6 Anisotropy is defined asrdquo Difference in properties when measured along different axis or from different directionsrdquo Crystalline solid show different values of some of the physical properties like electrical resistance refractive index etcwhen measured along the different directions The anisotropy in crystalline solid arises due to the different arrangement of particles in different directions

Math Function Composition of functions Think of an industrial plant that produce bottles of cold drinks first there is the operation (or function) f that puts the cold drink inside the bottle followed by the opeartion g that close the bottle with the capThis leads to the following definitionDefinition Let f A rarr B and g B rarr C be two functions Then the composition of f and g denoted by gof is defined as the function gof A rarr C given by gof(x) = g(f (x)) forall x isinA

Definition A function f X rarr Y is defined to be invertible if there exists a function g Y rarr X such that gof = IX and fog = IY The function g is called the inverse of f and is denoted by f -1

Thus if f is invertible then f must be one-one and onto and conversely if f is one-one and onto then f must be invertible This fact significantly helps for proving a function f to be invertible by showing that f is one-one and onto specially when the actual inverse of f is not to be determined

Example 1 Let f 2 3 4 5 rarr 3 4 5 9 and g 3 4 5 9 rarr 7 11 15 be functions defined as f(2) = 3 f(3) = 4 f(4) = f(5) = 5 and g (3) = g (4) = 7 and g (5) = g (9) = 11 Find gofSolution We have gof(2) = g (f(2)) = g (3) = 7 gof(3) = g (f(3)) = g (4) = 7gof(4) = g (f(4)) = g (5) = 11 and gof(5) = g (5) = 11Example 2 Find gof and fog if f R rarr R and g R rarr R are given by f(x) = cos x and g (x) = 3x2 Show that gof ne fogSolution We have gof(x) = g(f(x))=g(cosx) = 3 (cos x)2

= 3 cos2 x Similarly fog(x)=f(g (x))= f(3x2)= cos (3x2) Note that 3cos2 x ne cos 3x2 for x = 0 Hence gof ne fogExample 3 Show that if f A rarr B and g B rarr C are onto then gof A rarr C is also ontoSolution Given an arbitrary element z isin C there exists a pre-image y of z under g such that g (y) = z since g is onto Further for y isin B there exists an element x in A with f(x) = y since f is onto Therefore gof(x) = g (f(x)) = g (y) = z showing that gof is onto Example 4 Let Y = n2 n isin N sub N Consider f N rarr Y as f(n) = n2 Show that

f is invertible Find the inverse of fSolution An arbitrary element y in Y is of the form n2 for some n isin N This implies that n =radicy This gives a function g Y rarr N defined by g (y) =radicy Nowgof (n) = g (n2)=radicn2 = n and fog (y) =f(radicy) = (radicy) 2 y which shows that gof=IN and fog= IY Hence f is invertible with f -1 = g

Political Science

Constitution of India-The Preamble

Summary

Objective of the state-To secure equality of status and of opportunity To promote fraternity among all the citizens To assure the dignity of the individuals and Unity and integrity of the nation

Justice-Justice stands for rule of law absence of arbitrariness and a system of equal rights freedom and opportunities for all in a society India seeks social economic and political justice to ensure equality to its citizens

Liberty-Liberty implies the absence of restraints or domination on the activities of an individual such as freedom from slavery serfdom imprisonment despotism etc The Preamble provides for the liberty of thought expression belief faith and worship

Equality-Equality means the absence of privileges or discrimination against any section of the society The Preamble provides for equality of status and opportunity to all the people of the country

Fraternity-The Preamble declares that fraternity has to assure two thingsmdashthe dignity of the individual and the unity and

Execution

Answer the following questions-

Short notes-1 Equality2 Fraternity3 Justice4 Liberty

Homework-Learn

integrity of the nation The word integrity has been added to the Preamble by the 42nd Constitutional Amendment (1976)

Business studies

Human resource management (chapter 1)

On the day of 1504 2020 I have discussed with you the managerial functions and procurement functions of HRM

Today weare going to discuss about the development function integration functions and maintenance function

Development functions-HRM improves the knowledge skills attitude and values of employees so that they the present and future jobs more effectively it includes

1) Development functions of HRM

a) Performance appraisal = It implies systematic evaluation of employees with respect to their performance on the job and their potential for development

b) Training =It is the process by which employees learn knowledge skills and attitudes to achieve organisational and personal goals

c) Executive development = It is the process of developing managerial talent through appropriate program

2) Integration functionsa) HRM reconcile the goals of

organisation with those of its members through integrating function

b) HRM tries to motivate employees to various financial and non financial incentives provided in job specification etc

3) Maintenance functiona) HRM promote and protect the

physical and mental health of employees by providing several types of benefits like housing medical aid etc

b) It Promote Social security measures to employees by providing provident fund pension gratuity maternity benefits

SubjectCOMMERCE

Topic

BUSINESSENVIRONMENT

Summary

Now quickly let us revise the earlier points that we have already done in the last class and let us proceed with the other topics that are there in the chapter

Firstly we will recall the internal and external factors of micro environment and then we

Execution 3 What do you mean by internal factors

in micro environmentAnswerInternal factors refer to all the factors existing within a business firm The internal factors are considered controllable because the enterprise has control over these factors

Development FunctionsPerformance AppraisalTrainingExecution Development

shall proceed in details

Meaning and list of internal and external factors

aInternal factorsInternal factors refer to all the factors existing within a business firm The internal factors are considered controllable because the enterprise has control over these factorsFor an example a company can alter its organization structure policies programmes employees physical facilities and marketing mix to suit the changes in the environmentList of internal factors areCorporate culture mission and objectives top management organizations structure company image and brand equity company resources

b External factorsExternal factors refer to those individual and groups and agencies with which a particular business organization comes into direct and frequent contact in the course of its functioningThese individuals and groups are known as STAKEHOLDERS because they have a stake (financial interest ) in the working and performance of the particular business List of external forces (stakeholders)Customers competitors investors suppliersmiddlemen (marketing intermediaries)financers publics

customers

suppliersfinancers

For an example a company can alter its organization structure policies programmes employees physical facilities and marketing mix to suit the changes in the environment

4 What do you mean by external factors in micro environment

AnswerExternal factors refer to those individual and groups and agencies with which a particular business organization comes into direct and frequent contact in the course of its functioningThese individuals and groups are known as STAKEHOLDERS because they have a stake (financial interest) in the working and performance of the particular business

3Who are stakeholdersSTAKEHOLDERS are individuals and groups who have a stake (financial interest ) in the working and performance of the particular business 4Discuss the internal factors in briefa Corporate CultureThe values beliefs and attitudes of the founders and top management of the company exercise a strong influence on what the cmpaany stands for how it does things and what it considers importantbMission and objectivesThe business philosophy and purpose of a comoany guide it prioritiesbusiness strategiesproduct market scope and development scope

cTop management structurethe composition of board of directors the degree of professionalization of management and the organizational structure of a company have important bearing on its business decisions

dPower structureThe internal power relationship between the board of directors and the chief executive is an important factor

e Company image and brand equityThe image and brand equity of the company play a significant role in raising finance forming alliance choosing dealers and suppliers launching new products entering foreign markets

5 What is Macro environmentAnswerMacro environment refers to the general

competitors

middlemen

publics

Fig STAKEHOLDERS OF A COMPANY

Apart from micro environment the other main dimension of business environment isMacro environment Macro environment refers to the general environment or remote environment within which a business firm and forces in its micro environment operateA company does not directly or regularly interact with the micro environmentTherefore macro environment is also known as indirect action EnvironmentThe macro environment forces are less controllable than the micro forces

Macro environment consists of the following components

POLITICAL AND LEGAL ENVIRONMENT

ECONOMIC SOCIAL AND ENVIRONMENT

CULTURAL

ENVIRONMENT

TECHNOLOGICAL ENVIRONMENT

Fig COMPONENTS OF MACRO ENVIRONMENT

environment or remote environment within which a business firm and forces in its micro environment operateA company does not directly or regularly interact with the micro environmentTherefore macro environment is also known as indirect action EnvironmentThe macro environment forces are less controllable than the micro forces 6 What are the components of macro environmenta Political and legal environmentb Economic environmentc Social and cultural environmentd Technological environment

Computer Science

Logic gates

Digital systems are said to be constructed by using logic gates These gates are the AND OR NOT NAND NOR EXOR and EXNOR

BUSINESS FIRM

gates The basic operations are described below with the aid of truth tables

AND gate

The AND gate is an electronic circuit that gives a high output (1) only if all its inputs are high A dot () is used to show the AND operation ie AB Bear in mind that this dot is sometimes omitted ie ABOR gate

The OR gate is an electronic circuit that gives a high output (1) if one or more of its inputs are high A plus (+) is used to show the OR operationNOT gate

The NOT gate is an electronic circuit that produces an inverted version of the input at its output It is also known as an inverter If the input variable is A the inverted output is known as NOT A This is also shown as A or A with a bar over the top as shown at the outputs The diagrams below show two ways that the NAND logic gate can be configured to produce a NOT gate It can also be done using NOR logic gates in the same way

NAND gate

This is a NOT-AND gate which is equal to an AND gate followed by a NOT gate The outputs of all NAND gates are high if any of the inputs are low The symbol is an AND gate with a small circle on the output The small circle represents inversion

NOR gate

This is a NOT-OR gate which is equal to an OR gate followed by a NOT gate The outputs of all NOR gates are low if any of the inputs are highThe symbol is an OR gate with a small circle on the output The small circle represents inversion

EXOR gate

The Exclusive-OR gate is a circuit which will give a high output if either but not both of its two inputs are high An encircled plus sign ( ) is used to show the EOR operation

EXNOR gate

The Exclusive-NOR gate circuit does the opposite to the EOR gate It will give a low output if either but not both of its two inputs are high The symbol is an EXOR gate with a small circle on the output The small circle represents inversion The NAND and NOR gates are called universal functions since with either one the AND and OR functions and NOT can be generated

Note A function in sum of products form can be implemented using NAND gates by replacing all AND and OR gates by NAND gates A function in product of sums form can be implemented using NOR gates by replacing all AND and OR gates by NOR gates

Logic gate symbols

Table 2 is a summary truth table of the inputoutput combinations for the NOT gate together with all possible inputoutput combinations for the other gate functions Also note that a truth table with n inputs has 2n rows You can compare the outputs of different gates

Logic gates representation using the Truth table

Example

A NAND gate can be used as a NOT gate using either of the following wiring configurations

Subject Eng Literature (The Tempest ndash William Shakespeare) Topic Act III Scene 3 Lines 53 to 110 (End of the scene) Date 16th April 2020 (2nd Period)

[Students should read the original play and also the paraphrase given in the school prescribed textbook]Summary Questions amp Answers

o Seeing this strange scene all are inclined to believe the tales told by travelers that there truly are ldquounicornsrdquo and ldquothe phoenixrsquo thronerdquo

o As they are about to sit down to the feast the banquet is snatched away by a harpy (Ariel disguised) A spiritrsquos voice (Arielrsquos voice) denounces Alonso Sebastian and Antonio with particular

1 ARIEL You are three men of sin whom Destiny

(Line 53-58)That hath to instrument this

lower world And what is int the never-surfeited sea

Hath caused to belch up you and on this island

Where man doth not inhabit you rsquomongst men

Being most unfit to live I have made you mad

reference to their crime in expelling Prospero from Milan They have not received any punishment for their deed earlier but the time for their punishment has arrived Upon Alonso it pronounces ldquolingering perdition worse than deathrdquo from which there is no remedy except through sincere repentance Ariel then vanishes in thunder and the shapes enter again and carry away the table

o Prospero watching invisibly is very pleased with the performance of Ariel and his (Prosperorsquos) ldquomeaner ministersrdquo All his enemies are now in his power and are in a fit of desperation He then leaves them and goes to see how Ferdinand and Miranda are getting on

o Alonso is now much humbled and penitent with the after effect of the spiritrsquos denunciation of his crimes He believes that his son is lost forever After this all disperse being stricken mad by the speech of the spirit

o Gonzalo fearing that they may do violence to themselves or to one another follows them and bid others to follow

(a) To whom does Ariel disguised as a harpy call the three sinners What game did Fate of Destiny play with

them

The three sinners called by Ariel are Alonso Sebastian and Antonio It was Destiny which had caused the ocean to cast the three sinners on the shore Though the ocean is all the time devouring whatever appears on its surface and is never satisfied with its continual swallowing of the ships and men in the present case the ocean had cast these three sinners on the shore without killing them

(b) Who had jointly been responsible for the conspiracy against Prospero What is Prosperorsquos purpose behind all this

Three men Alonso Sebastian and Antonio had jointly

been responsible for the conspiracy against Prospero They had driven out Prospero form Milan Prosperorsquos purpose is to make these three sinners realize the wrong they had done He wants them to repent for their criminal deeds because repentance leads to self-esteem(c )What does Ariel (the harpy) tell Alonso and his companions when they take out their swords to attack him

Seeing them drawing their swords Ariel (harpy) tells them that he and his companions are the instruments of destiny and that it is not possible for human beings to do them any injury He says that the swords of human beings can not injure even a minute part of his feathers Their swords are as ineffective against him and his companions as against the wind or the water

(d) Give the explanatory meanings of the following expressions in the context of the above extract

(i)Never surfeited (ii) Belch up (iii) lsquomongst men

(i) Never surfeited never led to satisfaction

(ii) Belch up cast ashore(iii) lsquomongst men in human

society2

I and my fellows (Line 60-65)

Are ministers of Fate The elementsOf whom your swords are tempered may as wellWound the loud winds or with bemocked-at stabsKill the still-closing waters as diminishOne dowl thats in my plume

IMPORTANT PASSAGES EXPLAINED

The elements

(Line 61-66)Of whom your swords are tempered may

as wellWound the loud winds or with

bemocked-at stabs

(a) Who is lsquoIrsquo Who are his lsquofellowsrdquo

lsquoIrsquo is referred to Ariel in disguise of a harpy His lsquofellowsrsquo are other spirits serving Prospero the real Duke of Milan who has acquired supernatural powers after being banished from his Dukedom Prospero has settled in this uninhabited island

(b) What are the elements that have temperrsquod the swords Why will it not work against the speaker

The swords (of Alonso and his companions) are tempered by metal (steel) which is taken out of the earth and refined by

Kill the still-closing waters as diminishOne dowl thats in my plume My fellow

ministersAre like invulnerable

In these words Ariel reminds the King and his companions of the utter futility of drawing swords against himself and his fellows Ariel drives Alonso Antonio and Sebastian the three men of sin to desperation ndash a state in which men do violence to themselves They draw swords to strike Ariel But Ariel reminds them that he and the other spirits are the ministers of destiny and nothing can wound them The steel of which their swords are made of may cut the wind or water which being divided always closes up again Even supposing that such things may be possible it is quite impossible that their swords will cut one feather in their plume They are incapable of being wounded by any sword of man Hence it is foolish on their part to attempt to strike at Ariel and his fellow-spirits

For which foul deed

(Line 72-75)The powers delaying not forgetting

haveIncensed the seas and shores yea all the

creatures Against your peace

Ariel enters like a harpy and remaining invisible tells Alonso Sebastian and Antonio that he and other harpies are the agents of Destiny appointed to carry out her decrees He tells them that their punishment for the crime against Prospero which has been so long deferred is now to fall upon them He reminds them that they had expelled Prospero from Milan and set him and his innocent child adrift on the sea and that the sea had paid them back for their sin by the shipwreck and by the calamities they have suffered He tells them that the powers above which did not forget this mean treachery but only deferred the punishment have now engaged the seas and the shores and all living beings including him and his comrades against them The very elements and supernatural agency Ariel adds have taken up the avenging of their crime against Prospero

the action of fire It may cut the wind or water which being divided always closes up again

The sword will not work against the spirits and the harpy because they are the ministers of destiny and nothing can wound them nor it will cut a single feather in their plume

(c )What is the meaning of lsquodowlrsquo in the last line

The term lsquodowlrsquo means a filament or the smallest part of a feather In this context Ariel in disguise of harpy says that their sword cannot even damage the smallest filament of their (Arielrsquos and other spirits) feathers as they are incapable of being wounded by any sword of man

(d) What does the speaker remind the listeners about

Ariel in disguise of harpy reminds Alonso the King of Naples Sebastian Alonsorsquos brother and Antonio the present Duke of Milan and the treacherous brother of Prospero as they being three men of sin He even reminds them that their punishment for their crime against Prospero which has been so long deferred now falls upon them He reminds them that they have expelled Prospero from Milan and has set him along with his innocent infant daughter adrift on the sea So the sea has paid them back for their sin by their shipwreck and the calamities they have suffered since then The harpy rebukes Alonso of his sin that has incensed the Gods and has deprived him of his son as a punishment

(e) How do they respond

When Ariel in disguise of a harpy reminds Alonso Sebastian and Antonio of their past misdeeds and sin Alonso has a look of terror and confusion in his eyes He utters the words of sincere repentance wrung out of his conscience-stricken heart It appears to him that all the elements of nature the sea-waves the wind and the thunder proclaiming a loud voice in the name of Prospero and the crime Alonso has committed against him They are calling upon him to repent There is a deep storm raging in Alonsorsquos breast and the echoes of that storm are ringing in his ears like a clear note of wind-instrument A note of denunciation of Alonsorsquos crime leaves him much humbled and penitent and confirms his belief that his son is lost forever But Sebastian and Antonio shows some courage instead of repentance They wish to kill the spirits or devils if it appears

3

Of my instruction hast thou nothing bated (Line 85-93)

In what thou hast to say So with good life

And observation strange my meaner ministers

Their several kinds have done My high charms work

And these mine enemies are all knit upIn their distractions They now are in my

powerAnd in these fits I leave them while I visitYoung Ferdinand whom they suppose is

drownedAnd his and mine loved darling

Methought the billows spoke and (Line 96-99)

told me of itThe winds did sing it to me and the

thunderThat deep and dreadful organ-pipe

pronouncedThe name of Prosper It did bass my

trespass

These are the words of contrition coming from Alonso Ariel has driven him to a deep repentance for conspiring with Antonio against Prospero He now feels a sincere remorse It appears to him that all the elements of nature the sea-waves the wind and the thunder proclaimed with a loud voice the name of Prospero and the crime Alonso had committed against him They are calling upon him to repent There is a deep storm raging in Alonsorsquos breast and the echoes of that storm are ringing in his ears like the clear note of a wind-instrument

Comment These are the words of sincere repentance wrung out of the conscience-stricken heart of Alonso Alonso who is the lesser villain is the first to give way to remorse under the effect of Arielrsquos speech The words of Ariel seem to him to be the voice of conscience speaking to him He is driven to desperation a state in which he might do violence to his life

(a) Identify the speaker State the context

Prospero the ruler of the island is the speaker The famous banquet scene has been enacted very well Ariel and his junior spirits have played their roles excellently Prospero is glad to say words of praise for them(b) In what way the speakerrsquos instructions have been carried out

According to Prosperorsquos instructions a banquet was presented before the King of Naples and his companions when they were tired and hungry Just when they were preparing to eat the feast the banquet was suddenly removed by exercising supernatural powers All this was done by Ariel Prosperorsquos chief assistant and a powerful spirit

Ariel not only made the feast disappear but also delivered his speech blaming the King and his two companions for their past wicked deeds He warned them to repent for their misdeeds or suffer forever on that uninhabited island

(c) Who are referred to as lsquomeaner ministersrsquo What have they done

Prospero refers as lsquomeaner ministersrsquo to his other lesser spirits who were assisting Ariel in presenting a scene before the kingrsquos party They entered the scene to the accompaniment of music They assumed several strange shapes and brought in a banquet Then they danced about it with gentle actions of salutations thus inviting the King and others to eat the feast

These spirits play their role again when Ariel in the shape of a harpy quits the scene These shapes enter again and dancing with mocking gestures carry away the table

(d) Who are the speakerrsquos enemies What has happened to them

King of Naples Alonso his brother Sebastian and the present Duke of Milan Antonio (Prosperorsquos own brother) are Prosperorsquos enemies With the turn of events they have all been washed ashore on the island which is ruled by Prospero the great magician Actually this happened after the shipwreck caused by a storm which was raised by Prospero with the purpose of bringing these people to his island Prosperorsquos spirits have already confused and terrified these enemies and they are under Prosperorsquos control He can treat them as he likes

(e) What does he say about Ferdinand Explain what is meant by ldquohellip his and mine darlingrdquo

Prospero knows that Alonsorsquos son prince Ferdinand is alive though his father thinks that the prince has been drowned

Prospero refers to his daughter Miranda who is dear to him She is also very dear to Prince Ferdinand who has fallen in love with her They are waiting to be married soon for which they have received Prosperorsquos consent

4

ALONSO O it is monstrous monstrous (Line 95-102)

Methought the billows spoke and told me of it

The winds did sing it to me and the thunderThat deep and dreadful organ-

pipe pronouncedThe name of Prosper It did bass

my trespassTherefore my son ithrsquo ooze is

bedded andIll seek him deeper than eer

plummet soundedAnd with him there lie mudded

(a) In what way does Alonso express his horror when his conscience is awakened by Arielrsquos words

When Alonsorsquos conscience is awakened by Arielrsquos words he expresses his horror at what he has heard He gets the feeling that the waves of the ocean the wind and the loud thunder have spoken to him and uttered the name of Prospero Because of being reminded of his crime in a very loud and rough voice he comes to realize that he has lost his son for his past misdeeds

(b) What does Alonso imagine about his son What does Alonso want to do in his desperate state

Alonso imagines that his son is lying in the mud at the bottom of the sea He feels desperate that he wants to drown himself in the ocean deeper than the plumb-line has ever gone He wants to lie with his son at the bottom of the sea

(c) How do Sebastian and Antonio want to face the evil spirits

Sebastian says that he is not at all afraid of what the harpy has said and that he is prepared to fight any number of such monsters if they appear before him only one at a time Antonio says that he would support Sebastian in the fight against the fiendsyyy

(d) Why does Gonzalo ask Adrian to follow the three men

Gonzalo tells Adrian that all the three men namely Alonso Sebastian and Antonio are in a wild and reckless mood The thought of the heinous crime of which they are guilty has begun to torment their minds So he asks Adrian to follow those three men without loss of time and prevent them from doing anything which the turmoil in their minds might lead them to do

(e) What opinion do you form of Alonso from the above extract

Alonso who is the lesser villain is the first to give way to remorse under the effect of Arielrsquos speech The words of Ariel seem to him to be the voice of conscience speaking to him He is driven to desperation a state in which he might do violence to his life

Subject =Accounts

Ac-12 15420 topic-pL Appropriation ac

PROFIT AND LOSS APPROPRIATION ACCOUNT

MEANING AND PREPARATIONProfit and Loss Appropriation Account is merely an extension of the Profit and Loss Account of the firm The profit of the firm has to be distributed amongst the partners in their respective profit sharing ratio But before its distribution it needs to be adjusted All Adjustments like partnerrsquos salary partnerrsquos commission interest on capital interest on drawings etc are made in this account These adjustments will reduce the amount of profit for distribution This adjusted profit will be distributed amongst the partners in their profit sharing ratio To prepare it at first the balance of Profit and Loss Account is transferred to this account The journal entries for the preparation of Profit and Loss Appropriation Account are given below

1 for transfer of the balance of Profit and Loss Account to Profit and Loss Appropriation Account

(a) In case of Net Profit

Profit and Loss Ac helliphelliphelliphelliphellipDrTo Profit and Loss Appropriation Ac(Net Profit transferred to Profit and Loss Appropriation Ac)

(b)In case of Net Loss

Profit and Loss Appropriation Achelliphelliphellip DrTo Profit and Loss Ac(Net Loss transferred to Profit and Loss Appropriation Ac)

2 for Interest on Capital

For transferring on Interest on CapitalProfit and Loss Appropriation Achelliphelliphellip DrTo Interest on Capital Ac(Interest on capital transferred to Profit amp Loss Appropriation Ac)

3 for Interest on Drawings

For transferring Interest on Drawings Interest on Drawings Achelliphelliphelliphelliphelliphellip DrTo Profit and Loss Appropriation Ac(Interest on drawing transferred to Profit amp Loss Appropriation Ac)

4 For Partnerrsquos SalaryFor transfer of partnerrsquos SalaryProfit and Loss Appropriation Achelliphellip DrTo Salary Ac(Salary transferred to profit amp Loss Appropriation Ac)

5 For Partnerrsquos CommissionFor transferring commissionProfit and Loss Appropriation Achelliphelliphellip DrTo Commission Ac(Commission transferred to Profit and Loss Appropriation Ac)

6 For Transfer of agreed amount to General ReserveProfit and Loss Appropriation Ac helliphellipDrTo General Reserve Ac(Transfer to General Reserve)

7 for share of Profit or Loss appropriation(a) If ProfitProfit and Loss Appropriation Achelliphellip DrTo Partnerrsquos CapitalCurrent Ac(Profit transferred to capitalcurrent Ac)(b) If LossPartnerrsquos Capital Current Achelliphelliphelliphellip DrTo Profit and Loss Appropriation Ac(Loss transferred to capitalcurrent Ac)

THE FORMAT OF PROFIT AND LOSS APPROPRIATION

Profit and Loss Appropriation Account for the year endedhelliphelliphelliphellip

Particulars Amount Particulars Amount

To PL Ac (loss) By pL Ac (profit)

To Interest on capital BY Interest on drawings

To partner`s commission by Partner`s capital Ac ( loss)

To Partner`s salary To Interest on partner`s loan To General Reserve To Partner`s Capital AC (Profit)

Subject= Economics

MOVEMENT ALONG THE DEMAND CURVE (CHANGE IN QUANTITY DEMANDED)In law of demand you have already studied the inverse relationship between price and quantity demanded When quantity demanded of a commodity changes due to change in its price keeping other factors constant it is called change in quantity demanded It is graphically expressed as a movement along the same demand curve There can be either a downward movement or an upward movement along the same demand curve Upward movement along the same demand curve is called contraction of demand or decrease in quantity demanded and downward movement along the same demand curve is known as expansion of demand or increase in quantity demanded

Extention of demandd

price (rs)p A

B Extentionp1 d

Q Q1

Quantity demanded ( in units)

Contraction of demandd

p2 Ccontraction

p APrice (Rs)

d

Q2 Q

Quantity demanded (in units)

Explanation of movement of demand A fall in price from OP to OP1 leads to increase in quantity demanded from OQ to OQ1 (expansion of demand) resulting in a downward movement from point A to point B along the same demand curve DD When Price rises from OP to OP2 quantity demanded falls from OQ to OQ2 (contraction of demand) leading to an upward movement from point A to point C along the same demand curve DD

  • Activity Series of Metals
    • Drawbacks of Rutherfordrsquos model of atom
      • Electromagnetic radiations
      • Properties of electromagnetic radiations
      • Characteristics of electromagnetic radiations
        • Plancks Quantum Theory-
        • Photoelectric effect
          • Intext Questions
            • Logic gates
            • Digital systems are said to be constructed by using logic gates These gates are the AND OR NOT NAND NOR EXOR and EXNOR gates The basic operations are described below with the aid of truth tables
            • AND gate
            • Example
Page 8:  · Web viewSubject. Topic. Summary. Execution. English 1 . Chapter 1 naming words . Page 8. Write the names of these pictures:- Person:-1. father. 2.Firefighter 3.doctor 4 ...

his army could cross to Lanka and rescue Sita assign2 Invade- enter

a country or a region so as to subjugate or occupy it

3 Prowess- bravery in battle

4 Haughty- arrogantly

5 Puny- small and weak

6 Spy- secret agent

7 Dreadful- causing or involving great suffering

8 Violate- disobey

9 Ghastly- causing great horror or fear

10 Deception ndash misleading

Class VISubject Topic Summary Execution

HISTORY AND CIVICS

CHAPTER 3

MAHAVIRA AND BUDDHA ndash GREAT PREACHERS

BUDDHA

Impact of Buddhism on Indian Cultures

Impact on Religion

Buddharsquos practical and simple doctrines made their impact on HinduismThe principle of ahimsaIt brought about a great change in the performance of costly yanjnas and sacrifices which previously involved immense loss of life The Mahayana Buddhists adopted the practice of worshipping Buddha and bodhisattvas making idols and erecting temples in their honour

Impact on Literature

After the death of Buddha his teachings were compiled and called TripitakasThe Jatakas contain tales dealing with the previous births of Buddha

Impact on Education

The Buddhist monasteries became great centres of learning These centres of learning developed into famous universities- Nalanda Taxila Vikramshila etc

Impact on Art and Architecture

The gateways and railing of the Sanchi Stupa were covered with sculptured figuresCave- temples were also constructed which were decorated with beautiful frescoes

1) How many parts of Tripitakas are thereAns -There are three parts of Tripitakas- Sutta Pitaka Vinaya PitakaAbhidhamma Pitaka

2)What are the subjects taught in these monasteries Ans ndash Buddhist scriptures logic Philosophy medicine astronomy etc

3) Which art was developed under Buddhist patronage Ans ndash Gandhara art

The Gandhara art was developed under Buddhist patronage

BIOLOGY The Leaf Photosynthesis The process by which green plants make their own food from carbon dioxide and water in the presence of sunlight and chlorophyll is called photosynthesis

All green plants need the following to make their food ndash

water carbon dioxide chlorophyll and energy in the form of sunlight

Carbon dioxide + water ------------- Glucose + oxygen

The end product of photosynthesis is glucose

Fill in the blanks

1 Plants make their food by the process of photosynthesis

2 The inner wall of the guard cell is thicker than the outer wall

3 The extra glucose is converted into starch and sucrose

4 The leaf is boiled in alcohol to remove chlorophyll

5 The rate of transpiration is more on the hot day then a cold day

6 Photosynthesis helps to observe water and minerals from the soil

English 1 Pronouns Kinds of pronouns 1 Personal pronouns2 Possessive pronouns3 Reflexive pronouns4 Interrogative pronouns5 Relative pronouns6 Demonstrative pronouns7 Indefinite pronouns

Personal pronouns they refer to first second and third person in sentences First person- the speakerSecond person-the listenerThird person-the objectperson being spoken aboutPersonal pronouns should have the same gender and number as the nouns they refer to

Possessive pronouns these are used to indicate the relationship between the objects and people These pronouns include mine ours yours his hersand theirs

ExerciseBFill in the blanks with suitable pronounsThere was much excitement among the childrenTheywere eagerly looking forward to the annual picniclsquoAre they going to Lodhi Gardens toorsquo wondered AneeshlsquoNo they are going to Buddh Jayanti Park with Mrs Jain said Mrs ChopraThe children looked disappointedlsquoWonrsquot you be taking us Marsquoam rsquo they askedlsquoSorry children I have to go to Mumbai for a week to look after my sick mother But you will have fun with Mrs Jain she is full of laughter and you will love being with her the whole daylsquoIt will not be the same they grumbled

English 2 The great train journey- Ruskin Bond

The great journey by Ruskin Bond is a story about Suraj who loved trains and wanted to go to places One day while wandering along the railway tracks he enters into a carriage compartment The train suddenly starts moving with him in the compartment and after a journey returns back to the same place from where it had begun The story is about his experience during that journey

State true or false1 When the train had passed leaving behind the

hot empty track Suraj was lonely2 It was winter holidays

and Suraj did not know what to do with himself

3 He plunged his hands into the straw and pulled out an apple

4 A dirty bearded face was looking out at him from behind a pile of crates

5 Suraj wanted to go to Japan

Hindi 2nd lang

गललबाजलडका खालीसथानोकोभरो-6 गो-ामसनिनकलकरहमगराजमआगए7 माबोधराजकोराकषससमझतीथी8 चीलरोशन-ानमसअ-रआकरतहसीलपरबठगई9 तीनचारतीनकऔररईकगोलउडलनिकनघोसलानहीनिगरा10 वहसवयतोघोसलातोडनककतिलएगललउठालायाथा

11 -ीवारकसाथलगतगोहपजोकसहार-ीवारपकडलतीह12 बोधराजअभीभीटकटकीबाधचीलकीओर-खरहाथा13 बोधराजअपनीजबमबहतसाचगगाभरकरलायाथा14 मरनिपताजीकीतरककीहईऔरहमलोगएकबडघरमजाकररहनलग15 बागमजातातोफलपरबठीनिततलीको-खनिततलीकोपकडकरउगकतिलयोकबीचमसल-ता

BENGALI(2ND LANGUAGE)

সহিনধসবরপওসবরসহিনধ

সবরসহিনধরহিনয়ম- ৯ই-কারহিকংাঈ-কাচেররপচেরইাঈহিভননঅনযসবররণCথাকচেইাঈসথাদেনয-ফায়এংওইয- ফাপCচেরণCযকতয়

১০উ-কারহিকংাঊ-কাচেররপচেরইাঈহিভননঅনযসবররণCথাকচেউাঊসথাদেন-ফায়এংওই- ফাপCচেরণCযকতয়

১১ঋ- কাচেররপচেরঋহিভননঅনযসবররণCথাকচেঋসথাচেনর -ফায়এংওইর পCচেরণCযকতয়

১২সবররণCপচেরথাকচেপCতMএ-সথাচেনঅয় ঐ- সথাচেনআয় ও- সথাচেনঅএংঔ- সথাচেনঅায়

৯ই+ অ= য- ফাআহি+ অনত= আযনত অহিধ+ অয়ন= অধযয়নই+ আ=য- ফা+ াইহিত+ আহি= ইতযাহি পরহিত+ আতC ন= পরতযাতC নই+ উ=য- ফা+ উঅহিত+ উহিকত= অতযহিকত হি+ উৎপহিতত= যৎপহিততই+ ঊ= য- ফা+ ঊ ই+ এ= য- ফা+ এঈ+ অ= য- ফা পরহিত+ ঊ4= পরতয4 পরহিত+ এক= পরচেতযকঈ+ অ আ= য- ফা+ অ আনী+ অমব= নযমব মসী+ আধার= মসযাধার

১০উ+ অ= অন+ অয়= অনবয় পশ+ অধম= পশবধমউ+ আ= াস+ আগত= সবাগত পশ+ আহি= পশবাহিউ+ ঈ= হিঅন+ ইত= অহিনবতউ+ এ= দেঅন+ এ4রণ= অচেনব4রণউ+ ঈ= ীসাধ+ ঈ= সবাধবী তন+ ঈ= তনবী

১১ঋ+ অ= র মাত+ অনমহিত= মাতরনমহিতঋ+ আ= রা হিপত+ আয়= হিপতরায়ঋ+ ই= হির মাত+ ইচছা= মাতচছাঋ+ ঈ= রী ধাত+ ঈ= ধাতরীঋ+ উ= র ভরাত+ উপচে(= ভরাতরপচে(

১২এ+ অ= অয় দেন+ অন= নয়নঐ+ অ= আয় গৈগ+ অক= গায়কও+ অ= অ দেপা+ অন= পনও+ ই= অ দেপা+ ইতর= পহিতরও+ এ= অ দেগা+ এ4রণা= গচে4রণাঔ+ অ= অা দেপৌ+ অক= পাকঔ+ ই= অা দেনৌ+ ইক= নাহিকঔ+ উ= অা দেভৌ+ উক= ভাক

MATHS Topic NumbersChapter Natural numbers and whole numbers

Study item properties of whole numbers for multiplication

1 Closure property If x and y are two whole numbers then xtimesy is also a whole numberExample If x = 9 and y =3 then xtimesy = 9times3 = 27 which is a whole number

2 Commutative property If x and y are two whole numbers then xtimesy = ytimesxExample If x = 5 and y = 2 then xtimesy = 5times2 = 10y times x = 2times5 = 10Therefore 5times2 = 2times5

3 Associative property If x y and z are three whole numbers then x times(ytimesz) = (xtimesy) times zExample If x =3 y = 5 and z = 7 then 3 times (5times7) = 3 times (35) = 105And (3times5) times7 = (15) times 7 = 105Therefore x times (ytimesz) = (xtimesy) timesz

4 Distributive property If x y and z are three whole numbers then xtimes (y + z) = x times y + x times z

Therefore the multiplication of whole numbers is distributive over their additionExample If x = 5 y = 3 and z= 2Therefore x times (y + z) = 5 times (3 + 2) = 5times5 =25And x times y + xtimes z = 5times3 +5times2 =15 +10 = 25Again x times (y ndash z ) = x times y ndash x timesz Therefore 5 times ( 3 - 2) = 5 times1 = 5 and 5times3 ndash 5 times2 = 15 ndash 10 = 5Therefore the multiplication of whole numbers is also distributive over their subtraction if y is greater than z

5 Existence of identity If x is a whole number then

X times1 = x 1 times x = xTherefore we can write x times1 = 1 times xTherefore the multiplication of any whole number with 1 is the number itselfTherefore we can say that 1 is multiplicative identity or identity element for multiplicationExample 5 times1 = 5 1 times 5 = 5 Therefore 5 times 1 = 5

6 Multiplicative inverse If x is any whole number ( x is not equal to zero ) then its multiplicative inverse will be 1xSo x times 1x = 1 but 1x is a whole number if x = 1For other values of whole number 1x is not a whole number therefore we can write its multiplicative inverse does not exists

7 Cancellation law of multiplication If x y and z are three non- zero whole numbers then x times y = x times z

Or y = zExample 9 times y = 9 timeszTherefore y = z

Class VIISubject Topic Summary Execution

English 2 Sentences based on meanings

Kinds of sentences

Assertive or declarative to convey information or simply make a statement

Interrogative to ask different types of questions

Imperative to command or instruct someone or make a request

Exclamatory to express strong feelings and emotions

Exercise c1 What a nice compliment that is

That is a nice compliment2 How well- behaved the children

areThe children are very well-behaved

3 What great chefs we areWe are great chefs

4 What a shame it isIt is a shame

5 What a fantastic idea you haveYou have a fantastic idea

Homework 6 -10English

LiteratureThe Listeners III) Answer the following questions-

d) Identify two words used in the poem to give the poem an eerie atmosphereAns- Two words used to give the poem an eerie atmosphere are ldquogreyrdquo and ldquophantomrdquo

e) Who do you think are the inmates of the houseAns- I think the inmates of the house are phantom who once used to dwell in it

f) Why was the poet ldquoperplexed and stillrdquoAns- He was lsquo perplexed and stillrsquo because he was expecting an answer from the inmates of the house But despite of repeated calls there was no response

CHEMISTRY Chapter 2 ndashElement and Compound

Activity Series of MetalsThe activity series is a chart of metals listed in order of declining relative reactivity The top metals are more reactive than the metals on the bottomMetal SymbolReactivity

Lithium Li displaces H2 gas from water steam and acids and forms hydroxides

Potassium K

Strontium Sr

Calcium Ca

Sodium Na

Magnesium Mg displaces H2 gas from steam and acids and forms hydroxides

Aluminum Al

Zinc Zn

Chromium Cr

Iron Fe displaces H2 gas from acids only and forms hydroxides

Cadmium Cd

Cobalt Co

Nickel Ni

Tin Sn

Lead Pb

Hydrogen gas

H2 included for comparison

Antimony Sb combines with O2 to form oxides and cannot displace H2

Arsenic As

Bismuth Bi

Copper Cu

Mercury Hg found free in nature oxides decompose with heating

Silver Ag

Palladium Pd

Platinum Pt

Gold Au

Answer the following Q)Difference Between Metals And Nonmetals

Metals Nonmetals

These are solids at room temperature except mercury

These exist in all three states

These are very hard except sodium

These are soft except diamond

These are malleable and ductile

These are brittle and can be breakdown into pieces

These are shiny These are non-lustrous except iodine

Electropositive in nature Electronegative in nature

Have high densities Have low densities

Math Number System

Chapter Fraction

Study item Some solved sums from exercise 3(B)1) For each pair given below state whether it from like fractions or unlike

fractions (i) 58 and 78

= Like Fraction because denominators same(ii) 815 and 821

= Unlike Fraction because denominators are not same

(iii) 49 and 94 = Unlike Fraction

2) Convert given fractions into fractions with equal denominators(iii) 45 1720 2340 and 1116Solution Given fraction 45 1720 2340 and 1116Therefore the LCM of 5 20 40 and 16 is 80Therefore 45 = 4times165times16 = 64801720 = 17times420times4 = 68802340 = 23times240times2 = 4680 1116 = 11times516times5 = 5580

3) Convert given fractions into fractions with equal numerators(iii) 1519 2528 911 and 4547Solution Given fractions 1519 2528 911 and 4547Therefore the LCM of 15 25 9 and 45 is 2251519 = 15times1519times15 = 225285 2528 = 25times928times9 = 225252911 = 9times2511times25 = 2252754547 = 45times547times5 = 225235

4) Put the given fractions in ascending order by making denominators equal

(iii) 57 38 914 and 2021Solution Given fraction 57 38 914 and 2021Therefore the LCM of the denominators is 16857 = 5times247times24 = 12016838 = 3times218times21 = 63168914 = 9times1214times12 = 1081682021 = 20times821times8 = 160168Therefore ascending order 63168lt108168lt120168lt160168Therefore ascending order of given fractions38lt914lt57lt2021

COMPUTER CHAPTER-1COMPUTER FUNDAMENTALS

DONE IN THE PREVIOUS CLASSES PAGE 10CWRITE TRUE AND FALSE

1 True2 False3 False4 False5 True

GEOGRAPHY CHAPTER 7EUROPE

CHAPTER COMPLETE 1)Europe is home to a famous mountain range called the Alps

2)River Rhine originates in Switzerland

3)The Eiffel Tower one of the tallest structures in Europe

4) Vatican City is one of the most densely populated European countries

5)Sognefjordin Norway is the largest fjord in Norway

Class VIIISubject Topic Summary Execution

MATHEMATICS Ch 6Sets

Exercise 6 (D)1 Given A = x x isin N and 3iquest x le 6 and B = x x isin W and xlt4 find (i) Sets A and B in roster form (ii) A cup B (iii)

A cap B(iv) A ndash B (v) B ndashA

Solution (i) A = 456 and B = 0123

(ii) A cup B = 0123456 (iii) A cap B = ϕ (iv) A ndash B = 456 (v)B ndash A = 0123

3 If A = 56789 B = x 3 lt x lt 8 and x isin W and C = x xle5 and x isin N Find (i) A cup B and (A cup B) cup C (ii) B

cup C and A cup ( B cup C)

(iii) A cap B and (A cap B) cap C (iv) B cap C and A cap (B cap C)

Is (A cup B) cup C = A cup (B cup C)

Is (A cap B) cap C = A cap (B cap C)

SolutionA = 56789 B = 4567 C = 12345

there4 (i) A cap B = 456789 and (A cup B) cup C = 123456789

(ii) B cup C = 1234567 and A cup ( B cup C) = 123456789

(iii) A cap B = 567 and (A cap B) cap C = 5

(iv) B cap C = 45 and A cap (B cap C) = 5

Now (A cup B) cup C = 123456789

And A cup ( B cup C) = 123456789 there4 (A cup B) cup C = A cup (B cup C)

Again (A cap B) cap C = 5 and A cap (B cap C) = 5

there4 (A cap B) cap C = A cap (B cap C)

4 Given A = 012345 B = 02468 and C = 0369 Show that (i) A cup (B cup C) = (A cup B) cup C ie the union

of sets is associative (ii) A cap (B cap C) = (A cap B) cap C ie the intersection of sets is associative

SolutionNow B cup C = 0234689 and A cup B = 01234568

there4 A cup (B cup C) = 012345689 and

(A cup B) cup C = 012345689

So (i) A cup (B cup C) = (A cup B) cup C ie the union of sets is associative

Again B cap C = 06 and A cap B = 024

there4 A cap (B cap C) = 0 and (A cap B) cap C = 0

So (ii) A cap (B cap C) = (A cap B) cap C ie the intersection of sets is associative

Physics Chapter 2 Physical Quatites and Measurements

Here We Will Do Some QuestionsRelated To Chapter 2

A density bottle has a marking 25 mL on it It means that

1 the mass of density bottle is 25g

2 the density bottle will store 25 ml of any liquid in it

3 the density bottle will store 25 ml of water but more volume of liquid denser than water

4 the density bottle will store 25 ml of water but more volume of a liquid lighter than water

Solution 2 the density bottle will store 25 ml of any liquid in it

COMPUTER CHAPTER-2Spreadsheet Functions and Charts

SELECTING RANGE IN ROWSCOLUMNSWHEN TWO OR MORE CELLS ARE SELECTED IT IS CALLED A RANGEA RANGE OF CELLS CAN BE FORMED IN TWO WAYS--a) SELECTING RANGE BY USING THE MOUSEb) SELECTING RANGE BY USING THE KEYBOARD

Q1)WRITE THE STEPS TO SELECT PARTIAL RANGE IN A ROW

Ans)THE STEPS ARE-6 SELECT THE ROW7 BRING THE CELL POINTER TO THE DESIRED

LOCATION FROM WHERE YOU WANT TO START YOUR SELECTION

8 CLICK THE LEFT MOUSE BUTTON AND KEEP DRAGGING TO YOUR RIGHT TILL YOU REACH THE LAST CELL TO NE SELECTED

RELEASE THE MOUSE BUTTON

GEOGRAPHY Asia

CLIMATE

Asia experiences great extremes of climate Jacobabad in the Sind province of Pakistan is one of the hottest places in the WorldVerkhoyansk in Siberia is one of the coldest places in the WorldCherrapunji and Mawsynram in India are two wettest places in WorldArabia Tibet Gobi and Mongolia are extremely dry regionsFactors Affecting Climate of Asia-The factors influencing the climate of Asia are-

Factors Affecting Climate of Asia-Thoroughly read the table in page number 60

Latitudinal extent

Continentality

Relief features

Presence of low pressure trough

Jet streams

English Language The Sentence A complex sentence contains one independent clause and at least one dependent clause The dependent clause in a complex sentence is introduced with subordinating conjunctions or relative pronouns

Commonly Used Subordinating Conjunctions-Time after before while when since untilCause And Effect because now since as in order that soOpposition although though even though whereas while in spite ofCondition if unless only if whether or not even if in case(that)

Commonly Used Relative Pronouns-Who whose whom which whoever whomever whichever that

Class IXSubject Topic Summary Execution

1-BENGALI(2ND LANGUAGE)

ldquo বঙগভমিরপরমিrdquo াইকেলধসদনদতত

আচেগর পর উততর পচো-১ ২ ৩ এং নীচের পর টি াহির কা- ৪মহিbকাও গচেনা দেগা পহিচে অমত হরচে- ক) কার দো দেকান কহিতার অং( ) কতা দেক পরসঙগ কী উহিকতটির তাৎপযC আচোনা কচেরা৫দেসই ধনয নরকচে দোচেক যাচে নাহি ভচে মচেনর মহিeচের সাচেসচে সCন ক) কহির কায C ার উচেf(য হিক হিছ কহি কন কহিতাটি দেচেন) কহি কার কাচেছ হিমনহিত কচেরচেছনগ) কহি এই পহিথীচেত কাচের ধনয মচেন কচেরনঘ) কহি হিক রকম অমর তাাভ করচেত ান

Hindi 2nd lang

काकी(कतिसयारामशरण गपत)

इस कहानी म लखक न यह बतान का परयास निकया ह निक बचच अपनी मा स निकतना परम करत ह शयाम अबोध बालक ह वह अपनी मा क मरन क बा- उसन अपनी मा क कतिलए बहत रोया बा- म उस पता चला निक उसकी मा राम क घर चली गई ह आकाश म उडती हई पतग -खकर उस हरष हआ निक पतग क दवारा वह अपनी मा को नीच उतारगा इसक कतिलए वह अपनी निपता की जब स -ो बार सवा रपया निनकालकर पतग और -ो मोटी सी मन वाली अपन भाई स काकी एक कागज पर कतिलखवा कर पतग म कतिशव का दि-यानिनकालकर पतग और -ो मोटी सी मन वाली अपन भाई स काकी एक कागज पर कतिलखवा कर पतग म कतिचपका दि-याभोला और शयाम कोठरी म रससी बाधनी रह थ तभी उसक निपता करोध म आकर उन स पछ निक कया उनकी जब स रपया निनकाला हभोला डर क मार बताया निक शयाम इस पतग क दवारा अपनी काकी को राम क यहा स उतारना चाहता हनिवशशवर(शयाम क निपता)न फटी पतग उठाकर -खी तो उस पर काकी कतिलखा थावह हत बजिa होकर वही खड रह गएउनहोन सोचा निक मन अपन पतर को मारा जोनिक अनजान और निन-dरष थावह अपनी मा कोनिकतना पयार करता ह

उस दि-न बड सवर शयाम की नी- खली तो -खा निक घर भर म कोहराम मचा हआ ह

क) घर म कोहराम कयो मचा हआ था शयाम को कया लगा

ख) काकी को ल जात समय शयाम न कया उपदरव मचाया

ग) काकी क बार म उस कया बताया गया कया सतय उस कतिछपा रहा

घ) वह बठा-बठा शनय मन स आकाश की ओर कयोकरता

उततरक) शयाम की मा का -हात हो गया था इसकतिलए

घर म कोहराम मचा हआ था शयाम की लगा निक उसकी मा सफ- कपडा ओढ हए भमिम पर सो रही ह

ख) लोग जब उमा यानी शयाम की मा को उठाकर ल जान लग तब शयाम न बडा उपदरव मचाया लोगो क हाथ स झठ करवा उमा क ऊपर जा निगरा और बोला काकी सो रही ह उस कहा ल जा रह हो

ग) काकी क बार म बजिaमान लोगो न उस निवशवास दि-लाया निक उसकी का निक उसक मामा क यहा गई ह लनिकन सतय अमिधक दि-नो तक कतिछपाना रह सका आसपास क अबोध बालको क मह स यह बात परकट हो गई निक उसकी मा का -हात हो गया ह

घ) कई दि-नकई दि-न लगातार रोत-रोत उसका रोना तो शान हो गया पर उसक ह-य म शोक भर गया था वह चपचाप बठा आकाश की और टाका करता निक शाय- उसकी काकी कही दि-ख जाए

ldquoदि-न उसन ऊपर आसमान म पतग उडती -खी न जान कया सोच कर उसका निहर-य एक-म खिखल उठाrdquo

क) निकसन पतग ऊपर उडत -खी और वह कयो खश हआ

ख) उसन अपन निपता स कया कहा उनका कया उतर थाश

ग) उसन निफर कया निकया और निकसन उसकी सहायता की

घ) उसकी योजना कया थी उततर -क) शयाम न एक दि-न आसमान म पतग उडती

-खी तो उसन सोचा निक पता आसमान म राम क यहा जाकर रकगी वही पर मरी काकी ह यह सोचकर वह बहत खश हआ

ख) उसन अपन निपता स कहा काका मझ एक पतग मगा -ो उसक निपता न भटक हए मन क भाव स कहा निक मगा -ग यह कह कर उ-ास भाव स वह कही और चल गए पतग नही आई

ग) उसन चपचाप निवशशवर क टगहए कोट स एक चवननी निनकाल ली और सखिखया -ासी क लडक भोला की सहायता स एक पतग मगवानी भोला उसकी बराबर उमर का ही था

घ) उसकी योजना यह थी निक वह अपनी पतग को आकाश म राम क यहा भजगा और उस पतग क सहार उसकी काफी नीच उतर जाएगी इस योजना पर उस परा निवशवास था इसकतिलए वह और भोला -ोनो यह काम करन म लग गए

Continue to nexthelliphellipEVS CHAPTER - 1

(UNDERSTANDING OUR ENVIRONMENT)

Sustainable development

The development that meets the needs of the present without compromising the ability of future generations to meet their own needs is called Sustainable development

Sustainable societies ndash

An environmentally sustainable community is one that meets the current and future basic resource needs of its people in a just and equitable manner without compromising the ability of future generations to meet their basic needs

Q ) What are Eco Villages

Ans - Eco village are the urban or rural communities of people who strive to integrate a supportive social environment with a low impact way of life

Q ) To ensure sustainable development the depletion of renewable resources should not take place at a rate faster than their regeneration Justify your answer

Ans ndash Renewable resources do not have a fixed quantity - more can always be

generated However if the rate of use exceeds the rate of renewal - that is the

source is used more than its being recreated - its continued use will become

used up faster than it can regenerate

To promote sustainable society the following things need to be done ndash

1 Using renewable energy sources 2 By improving the quality of human

health 3 By promoting sustainable agriculture 4 By forming ecovillage

it will eventually be entirely depleted So Toensure sustainable development the depletion of

renewable resources should nottake place at a rate faster than their regeneration

Q ) What do you mean by Sustainable societies

Ans - Sustainable societies are defined as towns and cities that have taken steps to remain healthy over the long term These communities value healthy ecosystems use resources efficiently and actively seek to retain and enhance a locally based economy Sustainable development concerns everybody in a society

Q ) What are the effects of pollution on human health

Ans ndash Some health problem occurs due to air pollution are ndash

Respiratory diseases Cardiovascular damage Fatigue headaches and anxiety Irritation of the eyes nose and throat Damage to reproductive organs Harm to the liver spleen and blood Nervous system damage

Some health problem occurs due to water pollution are ndash

Typhoid Cholera Dysentry Jaundice

Some health problem occurs due to noise pollution are ndash

Fatigue headaches and anxiety High blood pressure Hearing damage

Physics Motion in 1D First go through previous notes Now here we will solve some numerical related to that

Question 3What information about the motion of a body is obtained from the displacement-time graphSolution 3From displacement-time graph the nature of motion (or state of rest) can be understood The slope of this graph gives the value of velocity of the body at any instant of time using which the velocity-time graph can also be drawn

Question 4(a)What does the slope of a displacement-time graph represent(b)Can displacement-time sketch be parallel to the displacement axis Give a reason to your answerSolution 4(a) Slope of a displacement-time graph represents velocity(b) The displacement-time graph can never be parallel to the displacement axis because such a line would mean that the distance covered by the body in a certain direction increases without any

increase in time which is not possible

Chemistry Language of Chemistry

How to balance a chemical equationThere are two methods of balancing an equation(i)Hit and trial method(ii)Partial equation methodBalancing by hit and trial methodThis method consists of counting the number of atoms of each elements on both sides and trying to equalize themTake the following steps(i)Count the number of times (frequency) an element occurs on either side(ii)The element with the least frequency of occurrence is balanced first(iii)When two or more elements have the same frequencythe metallic element is balanced firstExample-1 On heatinglead nitrate decomposes to give lead dioxidenitrogen dioxide and oxygenPb(NO3)2rarrPbO+NO2+O2

In this equationLead occurs twiceNitrogen occurs twiceOxygen occurs four timesSince lead is a metalbalance it firstThe number of atom of lead is equal on the two sidestherefore it needs no balancingNow balance nitrogenOn the reactant sidethere are two atoms of nitrogenwhile on the product side oneSomultiply the product containing nitrogenon the product sideby two Pb (NO3)2rarrPbO+2NO2+O2Nowthe number of oxygen atoms on the reactant side 6while on the product sideit is 7Somultiply the entire equation by 2except oxygen to get balanced equation2Pb(NO3)2rarr2PbO+4NO2+O2Multiplication by 2 is done only when atoms of all the elements except one element are balanced and the unbalanced atom occurs separately at least once and also there is a difference of only one such atom

Math Topic AlgebraChapter

Factorisation

Study item Difference of two squares a2 ndash b2 = (a+b) (a-b)1) (i) 4x2ndash 25y2

= (2x) 2 ndash (5y) 2= (2x + 5y) (2x - 5y)

(ii) 9x2 ndash 1= (3x)2ndash(1)2= (3x + 1)(3x ndash 1)

2) (i) 150 ndash 6a2= 6(25 ndash a2)= 6(5)2 ndash(a)2= 6 (5 + a) (5 ndash a)

(ii) 32x2 ndash 18y2=2(16x2 ndash 9y2)=2(4x)2 ndash (3y)2= 2(4x + 3y)(4x - 3y)3)(i) (x ndashy )2 ndash 9 = (x ndash y )2 ndash (3)2= (x ndash y + 3) (x ndash y ndash 3)(ii) 9(x + y) 2ndash x 2= (3)2(x + y)2 ndash (x)2=3(x + y)2 ndash (x)2= (3x +3y ) 2ndash(x)2= (3x + 3y + x)(3x +3y ndash x)= (4x + 3y) ( 2x + 3y )

Commercial studies

Basic accounting terms

Today I will give you some questions from the previous study material

Questions1) Define accounting2) What do you mean by debit and

credit

3) Explain the types of account4) Define the following terms

a) Assetsb) Capitalc) Purchased) Debtorse) Transactions

5) Name the types of accounts given below

a) Krishnas accountb) Machinery accountc) Royalty accountd) Salary accounte) Furniture accountf) Audit fee account

Economics Revision Today I will give you some revision questions

Questions1) What do you mean by the terms

rdquowantsrdquo2) Write the difference between

consumer goods and producer goods

3) Define the term utility 4) Explain the different types of utility5) Define

a) Total utilityb) Marginal utility

Subject Eng Literature (The Merchant of Venice ndash William Shakespeare)Topic Act I Scene 3 Lines 1 to 48 (Shylock hellip Cursed be my tribe if I forgive him) Date 16th April 2020 (5th Period)

[Students should read the original play and also the paraphrase given in the school prescribed textbook]Summary Questions amp Answers

This scene takes place in Venice and we are introduced to the rich Jew Shylock Bassanio and Shylock are talking and Bassanio tells Shylock that he wants a loan of three thousand ducats for three months on the personal security of Antonio

o Shylock feels glad because he will be able to bind down Antonio by means of a bond on account of the loan but he tells Bassanio that all the fortunes of Antonio being invested in the merchant ships on the sea it is difficult to depend upon his credit Even under such circumstances Shylock is willing to advance the money on the personal security of Antonio

o Bassanio then invites Shylock to dine with him Shylock says that he is prepared to do anything with the Christians but not eat or drink or pray with them

o While Bassanio and Shylock are talking Antonio appears on the scene Shylock does not seem to take any notice of Antonio but goes on brooding within

(1) SHYLOCK Ho no no no no- my meaning in (Line 15-26)saying he is a good man is to have you understand me that he is sufficient Yet his means are in suppositionhe hath an argosy bound to Tripolis another to the Indies I understand moreover upon the Rialto he hath a third at Mexico a fourth for England and other ventures he hath squanderd abroad Butships are but boards sailors but men there be land-rats and water-rats land-thieves and water-thieves I mean pirates and then there is the peril of waters winds and rocks The man is notwithstanding sufficientmdashthree thousand ducats mdashI think I may take his bond

(a) Who is talking in the beginning of this scene What does Bassanio want from Shylock How does Shylock feel

In the beginning of the scene Bassanio and Shylock are talking to each other Bassanio wants to borrow three thousand ducats from Shylock for three months on the security of Antonio Shylock feels glad at heart that he will get the opportunity of binding Antonio with a bond(b) What risks does Shylock weigh in advancing the money

Shylock says that Antonio has invested all his capital in trading by sea-going ships But the ships are made of wood and the sailors of those ships are ordinary human beings The wood can

himself how he hates Antonio because of his being a Christian because he abuses Shylock in public places Shylock decides that if ever he can get Antonio to his advantage he will teach him a lesson

come to harm and men can commit mistakes and thus the capital invested in ships may be lost Then there are other dangers The goods loaded on the ships can be damaged by rats and thieves which are found both on land and water The ships can also be harmed through sea-storms submerged rocks etc(c) What two important functions does this scene have

The scene has two important functions First it completes the exposition of the two major plot lines of the play Antonio agrees to Shylockrsquos bond ndash three thousand ducats for a pound of flesh and second and more important dramatically this scene introduces Shylock himself In this scene Shakespeare makes it clear at once why Shylock is the most powerful dramatic figure in the play and why so many great actors have regarded this part as one of the most rewarding roles in all Shakespearean dramas(d) Where does this scene take place What kind of treatment has Antonio been giving to Shylock What does Shylock say when Bassanio invites him to dine with him

The action of this scene takes place in Venice Antonio has been in the habit of behaving harshly with Shylock ndash spitting on his beard and footing him like a stranger cur When Bassanio invites Shylock to dine Shylock says that he is prepared to do anything with the Christians but not eat and drink or pray with them

(2) SHYLOCK How like a fawning publican he looks (Line 38-48)I hate him for he is a Christian

But more for that in low simplicity

He lends out money gratis and brings downThe rate of usance here with us in VeniceIf I can catch him once upon the hipI will feed fat the ancient grudge I bear him

He hates our sacred nation and he railsEven there where merchants most do congregateOn me my bargains and my well-won thriftWhich he calls interest Cursed be my tribeIf I forgive him

(a) What is the context in which these words are spoken and what is the idea expressed in it

These remarks are made by Shylock when he sees Antonio coming along after Bassanio told him that the merchant will be his surety for the bond The above mentioned passage reveals Shylockrsquos hatred for Antonio Shylock says that he hates Antonio because he is a Christian and also because he gives loan without taking interest on them thereby bringing down the rate of interest in Venice(b) Explain the meaning of the phrase lsquoa fawning publicanrsquo

The phrase lsquoa fawning publicanrsquo refers to Roman tax collector It is a term of contempt and hatred on the lips of a Jew lsquoFawning Publicansrsquo were Roman tax-gatherers whose ordinary bearings towards the Jews was bullying but whose false pose of lsquohumility and contritionrsquo is touched upon in the parable in New Testament(c ) What light does the above passage throw on the character

of Shylock

The above mentioned speech of Shylock reveals him to be a wicked character having an extreme greed for wealth His intense hatred for Antonio is unjustified He hates Antonio only because he is a Christian and because he lends money without taking any interest on it thereby adversely affecting Shylockrsquos business of lending money on high interest(d) What information do you gather about Antonio from the above given lines

Shylockrsquos statement throws a valuable light on the character of Antonio Antonio appears to be a good Christian and a good human being He helps the people in need by lending them money without charging any interest on it He is a man of simple and good nature This very goodness makes him Shylockrsquos enemy(e) What does Shylock debate within himself and when To whom are the lines mentioned above addressed to

When Bassanio asks the Jew to lend him three thousand ducats on Antoniorsquos surety Shylock begins to debate within himself as to how he should exploit the opportunity of a business deal with his old enemy Antonio

The lines mentioned above are not addressed to anyone The lines are a soliloquy ie a speech made by a character to himself and not meant to be heard by the other characters present

Class XSubject Topic Summary ExecutionEnglish

LiteratureThe Blue Bead 2nd part

Things took a turn for the worst and all of a sudden a crocodile attacked the woman biting on the womanrsquos leg At that moment Sibia got up sprinted grabbed the hay fork and stabbed the crocodile in the eye with all her power Immediately the crocodile let go and went away Sibia saw a small blue bead lying by the river she grabbed it Since she was poor she didnrsquot have necklace Shersquod always wanted one like the other women now she could make one with the blue bead After that she went home and told her mother all about it

Hindi 2nd

Langबड घर की बटी( मशी परमच-)

lsquoबड घर की बटी कहानी का उददशय मधयम वग की घरल समसया को सलझा कर सगदिठत परिरवार म मिमल जलकर परम स रहन का स-श -ना हघर म शानित सथानिपत करन की जिजमम-ारी नारी की होती ह यदि- नारी समझ-ार ह उसम धय और परिरवार क परनित परम ह तो कोई भी घटना परिरवार को निवघदिटत नही कर सकती या कहानी परिरवार को सगदिठत करत हए परम सौहा- स एक दसर की भावनाओ को समझ करउनका सहयोग करत हए जीवन यापन करन की पररणा -ती हमशी परमचदर जी न इस कहानी म सय परिरवार का परनितनिनमिधतव निकया ह यह कहानी बनी माधव सिसह जो गौरी पर क जमी-ार क उनक -ो पतरो की हशरी कठ लाल निबहारीशरीकात का निववाह एकजमी-ार घरान की पतरी आन-ी स हआ थाआन-ी न ख- को ससराल क वातावरण म ढाकतिलया थाएक दि-न आन-ी का अपन -वर लाल निबहारी स झगडा हो जाता ह -ोनो भाई एक दसर स अलग होन की कोकतिशश करत हसभी बह आन-ी न अपन मधर वयवहार स लाल निबहारी को

ldquoइन नतर निपरय गणो को बीए-इनही -ो अकषर पर नयोछावर कर दि-या था इन -ो अकषर न उनक शरीर को निनबल और चहर को कानित ही बना दि-या थाldquo

क) परसतत पकतियो म निकस वयकति क बार म कहा गया ह

ख) इन पकतियो म कौन स नतर निपरय गणो क बार म कहा गया ह

ग) बीए की निडगरी परापत कर लन पर भी उपय वयकति क सवभाव की कया निवशरषता थी

घ) यह नतर निपरय गण निकस वयकति म निवदयमान थ उसक वयकतितव की कया निवशरषता थी

उततर ndashक) परसतत पकति म गौरी पर गाव क जमी-ार

क बड बट शरीकात क बार म कहा गया ह उसन बहत परिरशरम और उ-म क बा- ba की निडगरी परापत की थी अब वह एक -फतर

घर छोडकर जान स रोक कतिलयाइस पर बनी माधव सिसह न कहा निक बड घर की बटी ऐसी ही होती ह जो निबगडा काम बना लती ह अतः शीरषक साथक ह बड घर की बटी आन-ी ह

म कमचारी थाख) भरा हआ चहरा चौडी छाती और डटकर

खाना आदि- एक सबजी ल जवान क गण मान जात ह परत शरीकात न इनही नतर निपरय गणो को अपनी पढाई पर नयोछावर कर दि-या था

ग) बीए की निडगरी परापत कर लन पर भी उपय वयकति(शरी कठ की शारिररिरक तौर पर निनबल और चहर स कानित ही लगत थ इतना ही नही वह मानकतिसक तौर पर भी निपछड हए थ पाशचातय सामाजिजक कथा उस घणा एव पराचीन सभयता का गणगान उनकी निवचारधारा क परमख अग थ

घ) यह नतर निपरय गण गौरीपर गाव क जमी-ार क छोट बट लाल निबहारी सिसह म निवदयमान थ वह सजीलाजवान था और भस का दध शर दध वह सवर उठकर पी जाता था

ldquoयही कारण था निक गाव की लललन आए उनकी निन-क थी कोई कोई तो उह अपना शतर समझन म भी सकोच ना करती थी सवय उनकी पतनी को इस निवरषय म उनस निवरोध थाldquo

क) उपय पकति म इस वयकति क बार म कहा गया ह

ख) गाव की लललन आए उनकी निन-ा कयो निकया करती थी

ग) उनकी पतनी का कया नाम था उनह निकस निवरषय म अपन पनित क निवरa था और कयो

घ) इस कहानी का कया उददशय ह Continue to next helliphelliphellip

Bengali 2nd Language

ফ ফটক না ফটক( কহিতা )

পর) ldquo(ান াধাচেনা ফটপাচেথ পাথচের পাড হিচেয় এক কাঠচোটটা গাছ কহিকহি পাতায় পার ফাটিচেয় াসচেছldquoক) কার দো দেকান কহিতার অং( ) lsquo(ান াধাচেনা ফটপাচেথ পাথচের পাডহিচেয়lsquo চেত কী দোঝাচেনা চেয়চেছ গ) আচো য অংচে(lsquo এক কাঠচোটটা গাছ lsquoচেত কী দোঝাচেনা চেয়চেছ ঘ) ldquoকহিকহি পাতায় পার ফাটিচেয় াসচেছldquo ----- একথার পরকত অথC কী উততর ) ক) আচো য অং(টি পর যাত কহি সভা4 মচোপাধ যাচেয়র দো lsquoফ ফটক না ফটকrsquo কহিতার অং()কহি সভা4 মচোপাধ যায় হিছচেন দেপরচেমর কহি দেপরমচেক নানা ভহিঙগমায় হিতহিন ফটিচেয় তচেচেছন দেপরম মানচের স মচেতC র সঙগী কহিতার কহিতায় এক রb সb হচেয়র দেপরম াগরচেনর কথা চেচেছন (ান অথCাৎ দেযাচেন দেকান রস দেনই দেযাচেন দেকান মহিনতা দেনই অথ তার মধ দেযও দেপরম থাকচেত পাচের একথাই কহি তচে ধরচেত দেচেয়চেছন একটি মানচে4র মচেন দেযাচেন দেকামতার দেকান সথান দেনই পাথচেরর মচেতা হিনরসতার মচেনর মধ দেযও দেয দেপরম আসচেত পাচের দেস কথাই কহি চেচেছনগ)নারীচের যথC দেপরচেমর ছহি এই কহিতায় অকপচেট উচেঠ এচেসচেছ কহি এই কহিতায় কাটচোটটা গাছ কথাটি যার কচেরচেছন নারী দেয দেপরম দেথচেক হিতাহিত এং দেসই দেপরম সঠিক সমচেয় না পাওয়ার ন য দেপরম সমপচেকC হিচেr4 গৈতরী য় দেপরচেমর দেয গৈহি(ষট য মাধযC য সরসতা দেকামত এই সমসতর হিপরীত যথা রbতা শষকতা কচেঠার তা পরভহিত দোঝাচেত এক কাঠচোটটা গাছ কথাটি যার কচেরচেছনঘ) এাচেন এক নারীর যথC দেপরচেমর কথা হিনহিCপত ভাচে চেচেছন কহি অসমচেয় নারীর ীচেন দেপরম দেচেগচেছ এতহিন তার হয় রb কচেঠার হিছ দেপরচেমর অভাচে ঠাৎ দেসই শষক মরভহিমচেত সচের আভাস এচেসচেছ দেপরম দেযন 4Cার স(ীত তাই পরায় মত গাচেছ কহিকহি পাতা গহিচেয় উচেঠচেছ

Biology Chapter - 01Controlling Air Pollution

Today we will discuss how we control air pollution from domestic combustion

Q1Describe any five ways of reducing air pollution from domestic sources bull The number of pollutants in the air is verylarge and we always try to control them byfollowing ways

i) Solar cooker and solar heater It use no fuel reduce damage of environment by fuel use or reducing deforestation It maintains coolness of house It releases very less orno oil gas or grease

ii) Piped natural gas (PNG) It emits very less by products into the atmosphere As it isdistributed through pipe lines so there iscontinuous supply of fuel is possible

iii) Liquefied Petroleum Gas (LPG) It hasa higher heating value LPG doesntcontain sulphur so it burns a lot cleanerenergy sources It releases very less oralmost no fume in air

iv) Electricity based cooking Emission free cooking alternative for urban dwellers causeselimination of adverse health impactsofindoor air pollution It helps to avoid theinconveniences associated with procurement of LPG

v) Biogas It contains 75 methane whichmakes it an excellent fuel It burns without smoke and biogas plant leaves no residue like ash in wood charcoal etc Thus it isaclean fuel

Economics

Factors of Production

Today firstly we would recall the last class for 5 mins and then we would proceed with the further topics of the chapter

The concept meaning of land characteristics of land and importance of land to be repeated for the absentees as well as the students who were there in the class the previous day

Today we will start with the last portion of land before it the meaning of land to be repeated onceAs by now we all know that

Questions1What do you mean by productivity of landAnswer By productivity of land we mean the capacity of a piece of land to produce a crop

Thus it refers to the average output per unit of landSay per acre per hectare etc= (OutputArea of land)

2 What are the factors influencing the productivity of landAnswer

Natural factors Productivity of land is largely determined by the natural

Land is defined to include not only the surface of the earth but also all other free gifts of nature(for example mineral resources forest resources and indeed anything that helps us to carry out the production of goods and services but is provided by nature free of cost)

We will move on to the last portion of land by discussing Productivity of Land

By productivity of land we mean the capacity of a piece of land to produce a crop

Thus it refers to the average output per unit of land

Say per acre per hectare etc= (OutputArea of land)

With this we shall proceed further with the main factors that determine the productivity of land

Natural factors Human factors Improvements on land Location of land Organisation Ownership of land Availability of capital Proper use of land State help

Note economic development of a country depends upon the quality of its land If the land is fertile it will quicken the pace of development of the country

qualities of land such as fertility etc

Human factors Land cannot produce anything by itself Man has to apply labour on it to produce for himself So productivity of land depends on the knowledge and skills of workers

Improvements on land production of land is affected by land development measures like provision of well or tubewell irrigation proper drainage

State help The government of a country especially less developed country can play a vital role in improving the agricultural productivity by providing better irrigation facilities

Organisation Productivity of land also fdepends upon the way how the factors of production like labour and capital are organised

In order to increase productivity trained workers modern implements scientific methods good seeds are all essential

3 lsquoImproved technology affects the productivity of landrsquo Explain this statement with the help of suitable example Answer Use of improved technology raises the productivity of land Example By using HYV seeds chemical manures and modern machines per hectare output increases

Physics Force (Summary)

Question Write the expression for the moment of force about a given axisSolutionsThe expression for the moment of force is given byMoment of force about a given axis = Force times perpendicular distance of force from the axis of rotationQuestion What do you understand by the clockwise and anticlockwise moment of force When is it taken positiveSolutionsIf the effect on the body is to turn it anticlockwise moment of force is called the anticlockwise moment and it is taken as positive while if the effect on the

body is to turn it clockwise moment of force is called the clockwise moment and it is taken as negative

Math Topic Commercial Mathematics

Chapter Goods and services Tax

Study item Some solved sums from exercise ndash 1 A retailer buys a TV from a wholesaler for Rs 40000 He marks the price of the TV 15 above his cost price sells it to the consumer at 5 discount on the marked price If the sales are intra ndash state and the rate of GST is 12 find

(i) The marked price of the TV(ii) The amount which the consumer pays for the TV(iii) The amount of tax (under GST) paid by the retailer to the central

Government(iv) The amount of tax (under GST) received by the State Government

Solution As the sales are intra- state sale and the rate of GST 12 So GST comprises of 6 CGST and 6 SGSTTherefore a retailer buys a TV from a wholesaler for Rs 40000Therefore the amount of GST collected wholesaler from the retailer or paid by retailer to wholesalerCGST = 6 of Rs 40000 = Rs(6100 times40000) =Rs 2400SGST = 6 of Rs 40000 = Rs (6100 times 40000) =Rs 2400Therefore wholesaler will pay Rs 2400 as CGST and Rs 2400 as SGSTTherefore amount of input GST of retailer Input CGST = Rs 2400 and input SGST = Rs 2400Again the retailer marks the price of the TV 15 above his cost price(i) The marked price of the TV

= Rs 40000 + Rs 40000times15= Rs 40000 + Rs 40000times 15100= Rs 40000 + Rs 6000Rs 46000But the retailer sells it to consumer at 5 discount on the marked priceCost price after discount = Rs 46000 ndashRs46000times 5100 =Rs 46000 ndashRs 2300= Rs 43700Therefore the amount of GST collected retailer from consumer or paid by consumer to retailerCGST = 6 of Rs 43700 =Rs ( 6100 times43700)Rs 2622SGST = 6 of Rs 43700 = Rs (6100 times 43700) =Rs 2622Amount of the output GST of retailer Output CGST = Rs 2622 and output SGST = Rs 2622

(ii) The amount which the consumer pays for the TV= cost price of TV to consumer + CGST paid by consumer + SGST paid by consumer= Rs 43700 + Rs 2622 + Rs 2622= Rs 48944

(iii) The amount of tax (under GST ) paid by the retailer to the central Government=CGST paid by retailer = output CGST ndash input CGST=Rs 2622 ndash Rs 2400=Rs 222

(iv) The amount of tax ( under GST ) received by the State Government = SGST paid by wholesaler + SGST paid by retailer= Rs 2400 + output SGST ndash input SGST=Rs 2400 + Rs 2622 ndash Rs 2400=Rs 2400 + Rs 222= Rs 2622

Commercial studies

Stakeholders Today I am going to give some revision questions from the previous study material

Questions1) State the two expectations of

employees from a business concern2) Give two distinctions between

stakeholder and shareholder3) Give two difference between

internal stakeholders and external stakeholders

4) Give two expectations of suppliers from a business organisation

5) Who is a stakeholder in commercial organisations

Chemistry Periodic Table

Merits of Mendeleevrsquos Periodic law are as follows - 1He grouped the elements on the basis of atomic mass 2 He left gaps for undiscovered elements like Gallium Scandium germanium Also he left a full group vacant for undiscovered inert gases 3 He could predict proportions of several elements on basis of their position in periodic table like Ga Sc etc 4He could predict errors in atomic weights of some elements like gold platinum etc

Anomalies in Mendeleevrsquos Periodic law are as follows - 1 Position of isotopes could not be explained 2 Wrong order of atomic masses could not be explained

For example- as Arnur atomic mass 40 come first and K with low atomic mass (30) should come later but k should be placed first

According to Bohrrsquos Modern Periodic table properties of elements are periodic functions of their atomic numbers

So when elements are arranged according to increasing atomic numbers there is periodicity in electronic configuration that leads to periodicity in their chemical properties

It consists of horizontal rows (Periods) Vertical column (Groups)

There are 7 period and 12 groups in this long form of periodic table

Ist period has 2 elements IInd period has 8 elements IIIrd period has 8 elements IVth period has 18 elements Vth period has 18 elements VIth period has 32 elements VIIth period hs rest of elements

Note - The number of valence electrons in atom of elements decides which elements will be first in period and which will be last

In group- 1 to 2 gp and 13 to 17 contain normal elements 3 to 12gp ndash transition elements 57 to 71 - lanthanides 89 to 103 - Actinides

Left hand side ndash metals Right hand side ndash nonmetals

Note- Hydrogen element has been placed at top of Ist group Electronic configuration of H is similar to alkali metal as both have 1 valence electron

V electron of gp I element -- 1 V electron of gp 2 element -- 2 V electron of gp 13 element -- 3 V electron of gp 14 element -- 4 V electron of gp 15 element -- 5 V electron of gp 16 element --6 V electron of gp 17 element -- 7 V electron of gp 18 element -- 8

English 1 Transformation of sentences

Sentences A sentence is a group of words which makes complete sense

Exercise 2Change the following sentences from

a Assertive sentencesb Imperative sentencesc Interrogative sentencesd Exclamatory sentences

Sentences can be changed from one grammatical form to another without changing the meaning of the sentence This is known as transformation of sentences

assertive to interrogative1 Nobody would like to be a fool

Who would like to be a fool2 Their glory can never fade

When can the glory fade3 Nobody can control the wind

Who can control the wind4 It matters little if I die

What though I die5 No man can serve two masters

Can any man serve two masters

Exercise 3Interchange of assertive and Exclamatory sentences

1 She leads the most unhappy lifeWhat an unhappy life she leads

2 This is indeed an interesting bookWhat an interesting book this

3 He is a very great manWhat a great man he is

4 It is a very lame excuseWhat a lame excuse

5 It is sad that she died so youngAlas she died so young

Class XISubject Topic Summary Execution

Hindi 2nd lang

पतर परम(परमचदर) पतर परम कहानी म एक निपता की इचछाओ का वणन निकया गया ह अपन बड पतर परभ -ास स निपता चतनय -ास का निवशरष परम था निपता को उसक जनम स ही बडी-बडी आशाए थी उसम दसर बट कतिशव-ास की अपकषा स- उतसाह की मातरा अमिधक थी वह उस इगलड भजकर बरिरसटर बनाना चाहत थभागय का खल भी बडा निनराला ह बीए की परीकषा क बा- वह बीमार पड गया डॉकटरो न भी जवाब - दि-या थाचतन -ास जी बहत ही कजस थ बवजह पस खच करना नही चाहत थ अगर गारटी मिमलती तो शाय- पस खच भी कर -त परत गारटी नही थी परिरणाम सवरप उनक बट का -हात हो गयाजब बट को समशान ल जा रह थ तो वहा काफी शोर गान बजान हो रह थ पछन पर पता चला निक निकसी निपता निपछल तीन साल स निबमार था और उसक ईलाज म रपया पानी की तरह बहाया पर ठीक नही हए परत उसक बट को तनिनक भी अफसोस नही था उसका कहना था उसन कोकतिशश तो कीयह -खकर चतनय-ास जी को आतम निगलानी हईतभी स उनका म परिरवतन हआ और बट का भोज काफी धमधाम स निकयाऔर वहइस पशचाताप की आग म जलत रह औला- स बढकर पसा नही होता ह इस बात को समझन म उनह काफी व लग गया

hellipContinue to next

BENGALI(2ND LANGUAGE)

পরথমঅধযায়-ঠাকরারীনদরনাথঠাকর

নয়ন দোচের হিমাচেররা া নাচেমই হিযাত হিছচেন ায়ানার উাররণ সবরপ নয়ন দোচের ারা হিা (াচেকর হিা হিচেতন এছাাও দেকান উৎস উপচেb রাহিতর দেক হিন করার উচেfচে(য তারা সযC হিকরচেরণ রনয পরীপ জবাহিচেয় তাচেত রপার হির 4Cরণ করচেতন ঠাকরা এই নয়ন দো হিমারচের দে(4 ং(ধর হিছচেন হিমাররা ায়ানার ষটানত পর(Cন কচের তারা হিনঃসব এই হিমাহিরর দে(4 ং(ধর গৈকাস নদর রায়চেৌধরী গৈকাস া নয়ন দোচের সমসত সমপহিতত ঋচেরণর াচেয় হিহিx কচের অহি(ষট যা আচেছ তাচেত হিপত

ইার হিপতার মতয ইচে পর নয়নচোচের ায়ানার দেগাটা কতক অসাধাররণ শরাদধ (াহিনতচেত অহিনতম ীহিপত পরকা( কহিরয়া ঠাৎ হিনহিয়া দেগ- ক) কার দো দেকান গচেলপর অং() কতা দেক ইার চেত কাচেক দোঝাচেনা চেয়চেছ গ) পরসঙগ কী কতার কতয পহিরসফট কচেরা

পরচে4র যাহিত রbা করা সমভ নয় তাই হিতহিন পতরচেক হিনচেয় ককাতায় সাস শর কচেরন গলপ কথচেকর আহিথCক অসথা নয়ন দোচের হিমাচের দেথচেক সমপরণC আাা কথচেকর হিপতা হিনচের দেষটায় অথC উপাCন করচেতন া উপাহিধ াচেভর নয তার াসা হিছনা আর দেসই কারচেরণ কথক তার একমাতর উততরাহিধকার চেয় তার হিপতার পরহিত কতজঞ কথক দো পা হি(চেচেছন হিনচের পরারণ ও মান রbার নয উপচেযাগী অথC হিনা দেষটায় পরাপত চেয়চেছন- এটাই তার কাচেছ পরম দেগৌরচের হি4য় চে মচেন কচেরন কাররণ (নয ভাণডাচের গৈপতক ায়ানার উজজব ইহিতাস অচেপbা দোার হিসeচেকর মচেধয গৈপতক দেকামপাহিনর কাগ তার কাচেছ অচেনক দেহি( মযান

TO BE CONTINUED

উ- ক) আচোয অং(টি রীনদরনাথ ঠাকচেরর দো ঠাকরা গচেলপর অং() কতা চেন আচোয গচেলপর গলপ কথকইার চেত নয়ন দোচের হিমাহিরর দে(4 ং(ধর গৈকাস ার কথা া চেয়চেছ গৈকাস া নয়ন দোচের সমসত সমপহিতত ঋচেরণর াচেয় হিহিx কচের অহি(ষট যা আচেছ তাচেত হিপত পরচে4র যাহিত রbা করা সমভ নয় তাই হিতহিন পতরচেক হিনচেয় ককাতায় সাস শর কচেরনগ) গৈকাস ার হিপতার মতযর পর নয়ন দোচের হিমাহিরর অহিসততব হিপত য় কচেয়কটা উৎস ও শরাদধ- (াহিনতচেত হিমাহিরর দে(4 কহিটক যয় চেয় হিগচেয় এচেক াচের দে(4 চেয় যায় তন তাচের গC করার মত আর হিকছই হিছ না-দেসই পরসচেঙগ এই উহিকত নয়নচোচের হিমাচেররা া নাচেমই হিযাত হিছচেন ায়ানার উাররণ সবরপ নয়নচোচের ারা হিা (াচেকর হিা হিচেতন এছাাও দেকান উৎস উপচেb রাহিতরচেক হিন করচেত হিগচেয় তারা সযC হিকরচেরণর নয পরীপ জবাহিচেয় তাচেত রপার হির 4Cরণ করচেতন তাই দেসকাচের ায়ানা দেহি(হিন সথায়ী চেত পারত না হিহিভনন উৎস শরাদধ- (াহিনতচেত সাধযা হিতহিরকত র করার নয হিমাহির হিহিকচেয় দেযত হ হিতC কা হিহি(ষট পরীচেপর দেত দেযমন অলপকাচের মচেধয হিনঃচে(4 চেয় যায়-নয়নচোচের হিমারচের অসথা তাই চেয়হিছ এই কারচেরণই কথক নয়নচোচের হিমারচের গা ভরা আমবর সয করচেত পারতনা

Physics Dimensional Analysis (Summary)

Q Find the dimensions of consts ab in relation

p=(bminusxlowastx)at

where p is the power x is the distance and t is time

Ans From principle of homogeneity dimension of b x2 are same Dim of b = dim of x2 = [L2] = [ML2T0]Dim of a = dim of ( b- x2)dim of (pt) = [M0L2T0][ML2T-2] [T-1] [T] = [M-1L0T2]

Chemistry Atomic Structure Drawbacks of Rutherfordrsquos model of

atom a According to Rutherfordrsquos model of atom electrons which are negativelycharged particles revolve around the nucleus in fixed orbits Thusb theelectrons undergo acceleration According to electromagnetic theory of Maxwell a charged particle undergoing acceleration should emitelectromagnetic radiation Thus an electron in an orbit should emitradiation Thus the orbit should shrink But this does not happenc The model does not give any information about how electrons aredistributed around nucleus and what are energies of these electrons Isotopes These are the atoms of the same

Properties of electromagnetic radiationsa Oscillating electric and magnetic field are produced by oscillating charged particles These fields are perpendicular to each other and both areperpendicular to the direction of propagation of the waveb They do not need a medium to travel That means they can even travel invacuum

Characteristics of electromagnetic radiationsa Wavelength It may be defined as the distance between two neighbouring crests or troughs of

element having the same atomicnumber but different mass numbere g 1H11H21H3

Isobars Isobars are the atoms of different elements having the same massnumber but different atomic numbere g 18Ar40 20Ca40

Isoelectronic species These are those species which have the same numberof electrons

Electromagnetic radiationsThe radiations which are associated withelectrical and magnetic fields are called electromagnetic radiations When anelectrically charged particle moves under acceleration alternating electricaland magnetic fields are produced and transmitted These fields aretransmitted in the form of waves These waves are called electromagneticwaves or electromagnetic radiations

wave as shown It is denoted by λb Frequency (ν) It may be defined as the number of waves which passthrough a particular point in one secondc Velocity (v) It is defined as the distance travelled by a wave in onesecond In vacuum all types of electromagnetic radiations travel with thesame velocity Its value is 3 times10 8m sec-1 It is denoted by v

d Wave number Wave number is defined as the number of wavelengths per unit lengthVelocity = frequency timeswavelength c = νλ

Plancks Quantum Theory- o The radiant energy is emitted or absorbed not continuously but discontinuously in the form of small discrete packets of energy called lsquoquantumrsquo In case of light the quantum of energy is called a lsquophotonrsquoo The energy of each quantum is directly proportional to the frequency of the radiation ie E α υ or E= hυ where h= Planckrsquos constant = 6626 x 10-27 Js o Energy is always emitted or absorbed as integral multiple of this uantum E=nhυ Where n=1234Black body An ideal body which emits and absorbs all frequencies is calleda black body The radiation emitted by such a body is called black body radiation

Photoelectric effectThe phenomenon of ejection of electrons from thesurface of metal when light of suitable frequency strikes it is calledphotoelectric effect The ejected electrons are called photoelectrons

Biology Chapter - 02Systematics and Five Kingdoms

Scientists divide the whole living organisms into two kingdom first and ultimately by five kingdom at last

In the earlier systems of classifications organisms are divided into kingdom plantaeand kingdom animalia on the of presenceof cell wall their modes of nutrition and movements

Some problem arise like fungi share manycharacteristic withplant despite their heterotrophic nutrition bacteria protozoa areunicellular present in both kingdom Toovercome this third kingdom Protista isintroduced which include

unicellularorganisms But there is also another

problem Allunicellular organisms are not similar kind The cellular structure of prokaryotes is verydifferent from that of other organismsEukaryotes possess a true nucleus and allcell organelles that are not present inprokaryotes So the fourth kingdom Monerais introduced which include unicellular prokaryotes (bacteriaamp blue green algae)

bull Still some problem arise in kingdomplantae

So in 1969 R H Whittakar proposedanew five kingdom System of classification

i) Kingdom Monera - unicellular prokaryotes

ii) kingdom Protista - unicellular eukaryotes

iii) Kingdom Fungi - uni or multicellular fungi with cell wall but without chlorophyll

iv) Kingdom Plantae - Multicellular Plants

v) Kingdom Animalia - Multicellular Animals

EVS Chapter 1 ndash Modes of Existence

An agricultural society

An agricultural society also known as an agrarian society is a society that constructs social order around a reliance upon farming More than half the people living in that society make their living by farming

People in an agricultural society generally lead a more settled lifestyle than those in nomadic hunter-gatherer or semi-nomadic pastoral societies because they live permanently near the land that is farmed Agricultural settlements tend to develop in areas of convenience near bodies of water which is used for both crops and transportation or along trade routes Not everyone in an agricultural society is a farmer Some people make a living trading or making and selling goods such as tools used for farming

Another way to define an agrarian society is to see the total amount of production in a nation In an agrarian society cultivating the land is the main source of wealth Such a society can recognize other means of subsistence and work habits but emphasizes the importance of agriculture and livestock Agrarian societies have existed in various parts of the world for 10000 years and continue to exist today They have been the most common form of socio-economic organization for most of recorded human history

Q) Write the features of agricultural society

Ans - Structure and Features of Agrarian Society1 Occupational Structure

An agrarian society is generally associated with the domestication of plants and animals The domestication of plants means farming and that of animals means herding Often there is mixture of farming and the use of such domesticated animals as cow goat and sheep

2 Forms of Land Ownership in Agrarian SocietiesGenerally there are landlords supervisory farmers cultivators and share croppers The landholders own the land but do not work on it They let it out for sharecropping The supervisory farmers are those who live by having their land cultivated by hired labourers The cultivators cultivate the land for themselvesThe share-croppers are those who live by tilling other peoplersquos land or a crop-sharing basis The artisans own their means of production and produce by their own labour in their homesteads

3 Village Community System An agrarian society is highlighted by

the institution of village community system The agrarian economy made fixed dwelling houses necessary Living close together for protection and co-operation and living nearer to the land gave birth to agricultural villages The village is not only the residential place of farmers it is also the social integrator

4 Minimal Division of Labour Another structural feature of agrarian society is a minimal division of labour Except for the basic division founded on age and sex differences there are few specialized roles There is only one predominant type of occupation ie domestication of plants and animals For all the people the environment physical as well as social is the same

5 Role of Family The farm family is of the patriarchal type the father is the final arbiter in most of the familyrsquos major decisions The life of ail men and women is merged in family life Since there are not many special organizations family is the only organisation to perform the tasks of aid and protection

6 Sense of Unity The members of an agrarian society exhibit a strong in-group feeling Since the whole of their social lives is wrapped up in a society which is physically economically and socially homogenous they are inclined to view the entire outside world as an out group

7 Informal Social Control An agrarian society is regionally divided into villages In a village community the force of traditional mores is more dominant than in the urban community In the village everybody is known to everybody The members in a village community help each other and share the joy and sorrows of each other Crime in an agrarian society is rare

8 Simplicity and Uniformity Life of the people in an agrarian society is marked by simplicity and uniformity Their main occupation is agriculture which largely depends upon the vagaries of nature An agrarian society is a religious society

Math Compound angles Compound angles The algebraic sum of two or more angles is called a compound angle If A B C be three angles then A+B B+C C+A A-B B-C A-C A+B-C etc are compound angles In this chapter we shall discuss the trigonometrical ratios of compound angles Theorem 1 If A B and A+B are all pisitive acute angles theni) sin( A+B) = sin A cos B + cosA sinBii) cos(A+B) = cosA cosB- sinA sinBTheorem 2If A and B are positive acute angles and AgtB theni) sin(A-B) = sin A cosB- cos A sinBii) cos(A-B) = cos A cos B+ sin A sin BTo prove that i) sin(A+B) sin (A-B) = sin2 A - sin2 B = cos2 B- cos2 A

Example 1 Prove that tan70deg=2tan50deg+tan20degSolutiontan70deg = tan(50deg + 20deg)Or tan70deg=(tan 50deg+tan 20deg)(1-tan50degtan20deg) or tan70deg (1 ndash tan 50deg tan20deg) = tan50deg+tan20degor tan70deg= tan70deg tan50deg tan20deg+ tan50deg + tan20deg = cot20deg tan50deg tan20deg + tan50deg + tan20deg = 2 tan50deg+ tan20degExample 2 If A + B = 45deg show that (1 + tanA) (1 + tanB) = 2Solutiontan(A + B) =( tan A + tan B) (1 - tan

ii) cos(A+B) Cos(A-B) = cos2 A- sin2 B = cos2 B -sin2 AProof i) LHS= sin(A+B)sin(AminusB) [Recall sin(αminusβ)=sinαcosβminuscosαsinβ And sin(α+β)=sinαcosβ+cosαsinβ]= (sinAcosB+cosAsinB)times(sinAcosBminuscosAsinB)= sin2Acos2Bminuscos2Asin2B [Recall sin2α+cos2α=1 From above we can then assume correctly that sin2α=1minuscos2α AND cos2α=1minussin2α] = sin2A(1minussin2B)minussin2B(1minussin2A) = sin2Aminussin2Asin2Bminussin2B+sin2Asin2B = sin2Aminussin2B= 1-cos2A-(1-cos2B) = cos2 B- cos2 A = RHSii)LHS= cos (A+B) cos(A-B) [ cos(A+B) = cos AcosB- sinAsinBCos(A-B) = cosAcosB+ sinAsinB]= cos2 A Cos2 B- sin2 A Sin2 B= cos2 A( 1-sin2 B) - (1- cos2 A) sin2 B= cos2 A- cos2 A sin2 B- sin2 B+ cos2 A sin2 B=cos2 A- sin2 B=1- sin2 A-(1-cos2 B) = cos2 B- sin2 A= RHSTangent formulae for compound anglesi)tan (A + B) = tan A + tan B1-tan A tan Bii)tan (A ndash B) = tan A-tan B1+tan A tan Biii) cot (A + B) = cot Acot B-1cot A+cot B(viii) cot (A ndash B) = cot Acot B+1cot B-cot A

A tan B) Or 1= (tan A+ tanB) (1-tan A tanB) Or tanA + tanB + tanA tanB + 1 = 1 + 1Or tanA (1 + tanB) + (1 + tanB) = 2Or (1 + tanA) (1 + tanB) = 2Example 3 Find the value of sin 15degSolution sin 15deg= sin(45deg-30deg) = sin45degcos 30deg- cos45degsin30deg =(1radic2) (radic32) -(1radic2) (12) = (radic3-1) 2radic2Example 4 If sin A = 1 radic10 and sin B = 1 radic5 where A and B are positive acute angles then what is A + B SolutionWe know that sin (A + B) = sin A cos B + cos A sin B= [1 radic10] [radic(1 minus 1 5)] + [1 radic5] radic(1 minus 1 10)= [1 radic10] [radic4 5] + [1 radic5] [radic9 10]= [1 radic50] times (2 + 3)= 5 radic50 = 1 radic2

sin (A + B) = sin π 4rArrHence A + B = π 4Example 5 If A + B = 225o then find [cot A] [1 + cotA] times [cot B] [1 + cot B]Solution[cot A] [1 + cotA] times [cot B] [1 + cot B] = 1 [(1 + tan A) times (1 + tan B)]=1 [tan A + tan B + 1 + tan A tan B] [ tan (A + B) = tan225o]∵

tan A + tan B = 1minus tan A tan BrArr= 1 [1 minus tan A tan B + 1 + tan A tan B]= 1 2

COMMERCE

CLASSIFICTION OF HUMAN ACTIVITIES-ECONOMIC AND NON-ECONOMIC

Firstly we shall recall the previous class for 5 mins especially for the absentees and for also the rest of the students who were there

Today at first we briefly discuss the earlier portions of the chapter

1Business-It includes all those economic activities which are concerned with production and exchange of goods and services with the object of earning profit Example A factory shop beauty parlour also business enterprises

2Profession ndashThe term profession means an occupation which involves application of specialized knowledge and skills to earn a living For Example Chartered Accountancy medicine law tax consultancy are example of professions

Questions1What are the main features of ProfessionAnswer The main features of a profession are as follows a Specialised body of knowledge-Every profession has a specialised and systematised body of knowledge b Restricted entry- Entry to a profession is allowed only to those who have completed the prescribed education and have the specialised examination c Formal education and training ndashA formal education and training is given to the person who wants to acquire the professional

3Employment-Employment mean an economic activity where people work for others in exchange for some remuneration (salary)The persons who work for others are called lsquoemployeesrsquo The persons or organizations which engage others to work for them are called lsquoemployersrsquoEg A doctor working in a hospital is employment as he is working for a salaryA lawyer may serve as a law officer in a bank

With this we shall proceed with the features of both Profession amp Employment

The main features of a profession are as follow

a Specialised body of knowledge b Restricted entry c Formal education and training d Professional association e Service motive f Code of contact

The main features of an employment are as follows

a In employment a person works for others called employer

b An employee provides personal service

c There is a service agreement or contract between the employee and the employer

d The employee has to obey the order of the employer

e No capital investment is made by the employer

Various examples of Employment are as follows

aA teacher teaching in a school or collegeb An engineer employed in Municipal Corporation of DelhicAn accountant working in the accounts department of a companydA doctor working in a hospital

Note In all the above examples of employment the individual who is involved in each example is working as an employee for a salary under an employer

qualification(MBBSCALLB)d Service motive ndashProfessionals are expected to emphasis service more on their clients rather than economic gain f Code of Conduct-The activities of professionals are regulated by a code of conduct

2 What are the main features of EmploymentAnswer The main features of an employment are as followsa In employment a person works for others called employerb An employee provides personal servicec There is a service agreement or contract between the employee and the employerd The employee has to obey the order of the employere No capital investment is made by the employer

3 Give various Professions and their respective Association are given below

Professions

Professional

Professional association

Medical profession

Doctor Medical Council of India

Law profession

Lawyers Bar Council of India

Accounting Profession

Chartered

The Institute of Chartered Accounts of India( ICAI)

Engineerin Engineers The

g Profession

institute of Engineers (India)

Accounts Basic accounting terms

Today we will give you some questions from the previous study material

Questions6) Define accounting7) What do you mean by debit

and credit8) Explain the types of account9) Define the following terms

a) Assetsb) Capitalc) Purchased) Debtorse) Transactions

10) Name the types of accounts given below

a) Krishnas accountb) Machinery accountc) Royalty accountd) Salary accounte) Furniture accountf) Audit fee account

Economics Basic Economic ConceptsSub topic

UTILITY

Before starting todayrsquos class we shall recall the last class which was about UTILITY AND THE FEATURES OF UTILITY

Now we shall proceed with the further topics of the chapter

Todayrsquos topic from the chapter lsquo Basic Economic Conceptsrsquo will be TOTAL UTILITY amp MARGINAL UTILITYNow let us quickly revise the concept of utility with an example ie goods and services are designed because they have an ability to satisfy human wantsThis feature of being able to satisfy human wants is termed as utility For example we derive utility from WiFi services as it gives us satisfaction by connecting us to our friends and family through social media here consumers derive utility from WiFi services

From the above concept we shall start with todayrsquos topicEconomists have defined TOTAL UTILITY (TU) as the total satisfaction obtained by consuming a given total amount of a good and serviceFor example the total satisfaction obtained from eating 10 mangoes is the total utility of 10 mangoes

MARGINAL UTILITY (MU) is the additional satisfaction derived from each additional unit

Questions1 What is Total Utility (TU)

Answer Total Utility (TU) is the

aggregate of the utility that a consumer derives from the consumption of a certain amount of a commodityTU=MU1+MU2++MUn

2 What is Marginal UtilityAnswer

Marginal Utility (MU) is the additional made to the total utility as consumption is increased by one more unit of the commodityMU= TUn ndashTUn-1

NoteOften economists tend to

subdivide utility into an imaginary unit called UTIL

consumed In this casethe utility obtained from each mango as it is consumed as the MU of that mango It is also defined as the addition made to the total utility when an additional unit is consumed Often economists tend to subdivide utility into an imaginary unit called UTIL

Note As a consumer increases the consumption of a good over period of time the total utility or total satisfaction derived from it increases to appoint and thereafter it decreasesHowever as the consumer keeps on consuming the good the marginal utility or the additional utility derived from it decreases

SubjectBusiness studies

Topic

BUSINESSENVIRONMENT

Summary

Now quickly let us revise the earlier points that we have already done in the last class and let us proceed with the other topics that are there in the chapter

Firstly we will recall the internal and external factors of micro environment and then we shall proceed in details

Meaning and list of internal and external factors

aInternal factorsInternal factors refer to all the factors existing within a business firm The internal factors are considered controllable because the enterprise has control over these factorsFor an example a company can alter its organization structure policies programmes employees physical facilities and marketing mix to suit the changes in the environmentList of internal factors areCorporate culture mission and objectives top management organizations structure company image and brand equity company resources

b External factorsExternal factors refer to those individual and groups and agencies with which a particular business organization comes into direct and frequent contact in the course of its functioningThese individuals and groups are known as STAKEHOLDERS because they have a stake (financial interest ) in the working and performance of the particular business List of external forces (stakeholders)Customers competitors investors suppliersmiddlemen (marketing intermediaries)

Execution 1 What do you mean by internal

factors in micro environmentAnswerInternal factors refer to all the factors existing within a business firm The internal factors are considered controllable because the enterprise has control over these factorsFor an example a company can alter its organization structure policies programmes employees physical facilities and marketing mix to suit the changes in the environment

2 What do you mean by external factors in micro environment

AnswerExternal factors refer to those individual and groups and agencies with which a particular business organization comes into direct and frequent contact in the course of its functioningThese individuals and groups are known as STAKEHOLDERS because they have a stake (financial interest) in the working and performance of the particular business

3Who are stakeholdersSTAKEHOLDERS are individuals and groups who have a stake (financial interest ) in the working and performance of the particular business 4Discuss the internal factors in briefa Corporate CultureThe values beliefs and attitudes of the founders and top management of the company exercise

financers publics

customers

suppliersfinancers

competitors

middlemen

publics

Fig STAKEHOLDERS OF A COMPANY

Apart from micro environment the other main dimension of business environment isMacro environment Macro environment refers to the general environment or remote environment within which a business firm and forces in its micro environment operateA company does not directly or regularly interact with the micro environmentTherefore macro environment is also known as indirect action EnvironmentThe macro environment forces are less controllable than the micro forces

Macro environment consists of the following components

POLITICAL AND LEGAL ENVIRONMENT

ECONOMIC SOCIAL AND ENVIRONMENT

CULTURAL

ENVIRONMENT

TECHNOLOGICAL ENVIRONMENT

a strong influence on what the cmpaany stands for how it does things and what it considers importantbMission and objectivesThe business philosophy and purpose of a comoany guide it prioritiesbusiness strategiesproduct market scope and development scope

cTop management structurethe composition of board of directors the degree of professionalization of management and the organizational structure of a company have important bearing on its business decisions

dPower structureThe internal power relationship between the board of directors and the chief executive is an important factor

eCompany image and brand equityThe image and brand equity of the company play a significant role in raising finance forming alliance choosing dealers and suppliers launching new products entering foreign markets

5 What is Macro environmentAnswerMacro environment refers to the general environment or remote environment within which a business firm and forces in its micro environment operateA company does not directly or regularly interact with the micro environmentTherefore macro environment is also known as indirect action EnvironmentThe macro environment forces are less controllable than the micro forces 6 What are the components of macro environmenta Political and legal environmentb Economic environmentc Social and cultural environmentd Technological environment

BUSINESS FIRM

Fig COMPONENTS OF MACRO ENVIRONMENTPolitical science

Introduction to political science

Comparative politics and itrsquos scope Comparative politics is the second major dimension of political scienceIt is also a very vast area of study and a very large number of political scientists even treat it as an autonomous area of study within the board ambit of political scienceScope of comparative politics-

1 All political structures -Comparative politics includes the study of all structures formalnon formal governmental and extra governmental which are directly or indirectly involved in politics in all the countries of the world

2 Functional studies- Comparative politics seeks to study politics less from the point of view of the legal institutions in terms of their powers and move from the point of view of their functions which constitute the political process and their actual Operation in the environment

3 Study of political behaviour- Another important part of its scope is the study of the actual behaviour of the people in the process of politics

4 Study of similarities and differences- comparative politics also undertakesan analysis of the similarities and differences among political process and functions

5 Study of all political systems -comparative politics seeks to analyse the actual behaviour and performance of all political systems western as well as non western

6 Study of the environment and infrastructure of politics-The study of politics demands a study of the psychological sociological economic and anthropological environment in fact the social environment as a whole in which each political system operates

7 Study of political culture- political culture is composed of attitudesbeliefs emotions and values of a society that relate to the political system or politics

8 Study of political participation- Political participation is a universal processThe only difference is that while in some states it is limited in others it is wider

9 Study of political process- political

Answer the following questions-

What is comparative politics

What are the scope of comparative politics

Homework- learn

processes like decision makingpolicy making judicial process leadership recruitment process and others are always at work in all political systems

The scope of comparative politics is very comprehensive It includes everything that falls within the area of political activity and political process

History CAMBRIDGE VIEW ABOUT

THE PARTITION

AND REFUTATION

OF CAMBRIDGE

VIEW

Cambridge view about the Partition The Cambridge school of historians have interpreted that opposition to partition scheme was made entirely by the elitist groups They hold the view that Lord Curzon planned to partition the Bengal for administrative purposeREFUTATION OFCAMBRIDGE VIEW The Rationalist historians have rejected the interpretations of the Cambridge School of historians on various grounds

1 QUESTION State different views of historians regarding Partition of Bengal

ANSWER Cambridge historians believed that Lord Curzon partitioned Bengal for administrative reasons only and not for the political motive The Middle class elitist group protested because of their petty interest The Hindu zamindars protested as they have to spend more money for managing their estatesThe lawyers of Calcutta High court feared to lose their clientBut according to the nationalist Historians was-

2- The ultimate object of Lord Curzon was to crush the unity of Bengal politicians

3- If Bengal becomes a separate province Bengali speaking 16 million people of western part would become minority under Hindi speaking people of Bihar and Oriya speaking people of Orissa

4- The bureaucrats expected that the protest movement would die down quickly

5- Lord Curzon used the Muslim community in his political game

6- Idealism had great contribution in the protest against partition

7- The people of the every section of society were affected by the partition of Bengal

Computer Science

Numbers Convertion of dcimal number to octal numberThe decimal numeral system is the standard system for denoting integer and non-integer numbers It is the extension to non-integer numbers of the Hindu-Arabic numeral system For writing numbers the decimal system uses ten decimal digits a decimal mark and for negative numbers a minus sign - The decimal digits are 0 1 2 3 4 5 6 7 8 9 the decimal separator is the dot in many countries

The octal numeral system or oct for short is the base-8 number system and uses the digits 0 to 7 Octal is sometimes used in computing instead of hexadecimal perhaps most often in modern times in conjunction with file

permissions under Unix systems It has the advantage of not requiring any extra symbols as digits It is also used for digital displays

Follow these steps to convert a decimal number into octal form

1 Divide the decimal number by 82 Get the integer quotient for the next iteration (if the number will not divide equally by 8 then round down the

result to the nearest whole number)3 Keep a note of the remainder it should be between 0 and 74 Repeat the steps until the quotient is equal to 05 Write out all the remainders from bottom to top This is the solution

For example if the given decimal number is 8453

Division Quotient Remainder

8453 8 1056 5

1056 8 132 0

132 8 16 4

16 8 2 0

2 8 0 2

Then the octal solution is 20405

Subject Eng Literature (The Tempest ndash William Shakespeare) Topic Act I Scene 1 Lines 33 to 67 (End of scene) Date 16th April 2020 (4th Period)

[Students should read the original play and also the paraphrase given in the school prescribed textbook]Summary Questions amp Answers

[SUMMARY OF THE ENTIRE SCENE]

o The play starts with the scene of a severe storm at sea Alonso (King of Naples) Sebastian (Alonsorsquos brother) Ferdinand (Alonsorsquos son) Gonzalo Antonio (the usurping Duke of Milan) are in a ship in the midst of the storm

o The mariners are trying their best to control the vessel from running aground and are totally following the orders of their Master the Boatswain They have scant success

o The mariners become extremely unhappy and annoyed when most of the passengers arrive on the deck thereby hampering their effort to save the ship There is serious confrontation between them and the passengers who are part of the Kingrsquos entourage

o The mariners could not save the ship

SUMMING-UP

(i) Vivid description of the scene which gives a realistic description of terror and confusion of a tropical storm

(ii) Shows Shakespearersquos accuracy of knowledge in describing the naval operations and also matters of seamanship

(1) GONZALO Ill warrant him for drowning (L 45-57)

though the ship were no stronger than a nutshell and as leaky as an unstanched

wenchBOATSWAIN Lay her a-hold a-hold Set her two courses Off to

sea again lay her offMARINERS All lost To prayers to prayers All lostBOATSWAIN What must our mouths be coldGONZALO The king and prince at prayers Lets assist them

For our case is theirsSEBASTIAN Im out of patienceANTONIO We are merely cheated of our lives by drunkards

This wide-chopped rascal - would thou mightst lie drowning the washing of ten tides

(a) What does Antonio say at the insolent manners of the boatswain just before the given passage

Being irritated at the insolent manners of the boatswain just before the given extract Antonio the Duke of Milan calls him a worthless dog son of a woman without any morals an arrogant and disrespectful noisemaker He says that the boatswain deserved to be hanged(b) What statement does Gonzalo repeat about the boatswain

Gonzalo shows his faith that the boatswain is not destined to die by drowning He is destined to be hanged and nothing can alter this decree of destiny He says that even if the ship was as frail as a nutshell the boatswain could not be drowned for his destiny was to be hanged(c) What do the passengers do when they have lost all hope of their survival

When the passengers have lost all hope of survival they take

(iii) The opening scene justifies the title ndash The Tempest

UNANSWERED QUESTIONS

(i) The King always travels with his entire fleet including his soldiers Where were the other ships

(ii) Why was the ship in that area Where was it coming from or going where

(iii) The ship broke apart What happened to those who were in the ship

(We shall get the answer to the above questions as the play progresses)

leave of life with fervent prayers The mariners take their last hearty drink and are ready for death(d) What blame does Antonio put upon the mariners and the boatswain Antonio rebukes the mariners that these drunkards have brought them to the present crisis by neglecting their duties He blames them saying that they are going to lose their lives entirely for the negligence of the boatswain and his fellows(e) What does Antonio say while cursing the boatswain

Antonio gives vent to his wrath upon the boatswain in particular He calls the boatswain a wide-mouthed rascal who deserves to be hanged on the sea-shore at low water mark so that ten tides might wash over his body and take out of him all the liquor that he has been drinking

Class XIISubject Topic Summary ExecutionHistory Topic

1 1935 ACT AND WORKING OF PROVINCIAL AUTONOMYCONGREE AND OTHER MINISTERSSUB TOPIC GOVERNMENT OF INDIA ACT1935

Government of India Act 1935 This act established a lsquoFederation of Indiarsquo made of British Indian provinces and Indian states and provided for autonomy with a government responsible to the elected legislature in every provinceThis act introduced abolition of Diarchy at provinces The entire provincial administration was introduced to the responsible ministers who were controlled and removed by the provincial legislature The provincial autonomy means two things First The provincial governments were wholly responsible to the provincial legislature Secondly Provinces were free from outside control and interference in the large number of matters The act divided the powers between the centre and provinces in terms of three lists- Federal list( for centre) Provincial list (for province) and concurrent list (for both) Residuary powers were given to the viceroy In the election under the government of India Act the Congress swept the poll the mandate of the people came in favour of the congress so far as general Hindu seats were concerned The Congress did not get a single Muslim seates in Bombay CP UP Sind and BengalIn five provinces Congress had yhe clear majority In BengalNWFPAssam and Bombay Congress emerged as a single largest partyOn the other side the performance of the Muslim League was badThus the Congress formed ministers in 7 provinces out of 11 provinces Coalition ministry was also formed in two other provincesOnly BENGAL AND Punjab had non- congress ministries

1 QUESTION What was the main change introduced by the Government of India ActANSWER a) The Act gave more

autonomy to the provinces b) Diarchy was abolished at the

provincial levelsc) The Governor was the head of

the executived) There was a council of

ministers to advise him The ministers were responsible to the provincial legislatures who controlled them The legislature could also remove the ministers

e) The Governors still retained special reserve powers

2 QUESTION Why did the federal scheme introduced by the Government of India Act 1935 never come into operation

ANSWER The Federal structure of the Government of India was to be composed with the Governor General and Council of ministers The Federal legislature was to be Bicameral legislature- The council of states and the House of Assembly The ministers were to be chosen by the Governor general and they were to hold the office during his pleasure

The provinces of British India would have to join the federation but this was not compulsory for the princely states

This federation never materialised because of the lack of support from the required number of

princely statesThis act was refused and

rejected by the princes the Congress and the Muslim League

Thus both Congress and the League participated in the election of 1937 Thus the federal part was never introduced but the provincial part was put into operations

Bengali 2nd

Language

াচেরর পরাথCনা(কহিতা )

াচেরর পরাথCনা কহিতাটি কহি (ঙখ দেঘাচে4র দো আচো য কহিতায় াচেরর পতর হমায়ন কঠিন দেরাচেগ আxানত ার ঈশবর া আললার কাচেছ পরাথCনা কচেরচেছন তার পচেতরর ীন হিফহিরচেয় হিচেত এই কহিতায় ার পচেতরর ীন হিভbা দেচেয়চেছন ারার এমনহিক হিনচের ীন হিসCচেনর হিহিনমচেয় হিতহিন তার দেছচের ীন হিফচের দেপচেত দেচেয়চেছন তার দেছচের এই দেরাচেগর ন য হিতহিন হিনচেচেকই ায়ী কচেরচেছন তার হিনচের করা পাপচেকই হিতহিন ায়ী কচেরচেছন এছাা রানৈনহিতক ও আথCসামাহিক অসথার কথা তচে ধরা চেয়চেছ এই কহিতায় ার তার হিনচের পাপ কমCচেকই ায়ী কচেরচেছ ার অন যায় ভাচে দেপহি((হিকতর মাধ যচেম অপররা য কচেরচেছ আর এই অন যায় কাচের ন যই তার পহিরাচের হিপযCয় এচেসচেছ দে এক পরকার মানহিক নধন ইহিতাচেসর ার হিপতা চেয় সবাভাহিকভাচে ভাচোাসা দে মমতা দেথচেক মকত চেত পাচেরনহিন তাই হিপতা চেয় আললা া ভগাচেনর কাচেছ পতর হমায়চেনর পরানহিভbা দেচেয়চেছন ার আললা া ভগাচেনর কাচেছ াহিনচেয়চেছন তার হিনচের ীন হিসCন হিচেত হিতহিন রাী তার হিহিনমচেয় পচেতরর ীন হিফচের দেপচেত দেচেয়চেছন াচেরর হিপতসভ হিচেকর কথা এই কহিতায় ফটিচেয় দেতাা চেয়চেছ হিপতা পচেতরর হিরাহিরত মান নধচেনর কথা তচে ধরা চেয়চেছ

হিচে(4 হিকছ াইচেনর তাৎপযC১) ldquoদেকাথায় দেগ ওর সবচছয দেৌন দেকাথায় কচেরায় দেগাপন bয়ldquoউততর) াচেরর পতর হমায়ন কঠিন দেরাচেগ অসসথ তাই তার দেযৌন াহিরচেয় যাচেচছ এই দেরাচেগ তাচেক দেগাপচেন কচেরকচের াচেচছ তার সক (হিকত ধীচের ধীচের bয় চেচছ তাই হিপতা চেয় ার আললার কাচেছ হমায়চেনর পরান হিভbা দেচেয়চেছন২) ldquoাগাও (চেরর পরাচেনত পরানতচের ধসর (ন দেযর আান গানldquoউততর) াচেরর পতর হমায়ন কঠিন দেরাচেগ আxানত তাই ার আ দে(াচেক মমCাত (চেরর পচেথ পরানতচের আান গান ধবহিনত দোক দেসই আান গান আললার কাচেছ দেযন চে যায় আললা দেযন এই আহিতC শচেন পচেতরর ীন হিফহিরচেয় দেয় ৩)ldquoনাহিক এই (রীচেরর পাচেপর ীানচেত দেকানই তরারণ দেনই ভহি4চেতরldquoউততর) হমায়চেনর অসসথতার ন য ার হিনচেচেকই ায়ী কচেরচেছন কারন ার অচেনক রা য অন যায় ভাচে কচেরচেছ তাই তার এই পাপ কাচের ন য তার ঘচের আ হিপ এচেসচেছ এই অন যায় কাচের ন য তার মহিকত দেনই তাই ার আললার কাচেছ এই পাপ কাচেযCর ন য bমা পরাথM

Hindi 2ndlang

-ासी(जयशकर परसा-)

-ासी जयशकर परसा- की एक ऐसी कहानी ह जिजसम भारतीय ससकनित और राषटरीयता का सवरगजीतहोता ह इस कहानी म इरावती एक निहद कनया ह जिजस मलअचछो न मलतान की लट म पकडा और -ासी बना दि-या उस 500 दि-न -कर काशी क एक महाजन न खरी-ा दसरी -ासी निफरोजा ह वह गलाम ह निफरोजा को छडान क कतिलए अहम- को 1000 सोन क कतिसकक भजन थ जो अभी तक नही आए थ राजा साहब कठोर होत हए भी निफरोजा को निबना धनराकतिश क कतिलए उस म कर -त ह वनिफरोजा को अहम- को समझान की बात कहत हकहानी क अत म हम -खत ह निक इरा वती और जाटो क सर-ार बलराज का मिमलन होता हअहम- को यa म मार दि-या जाता ह वहा निफरोजा की परसननता की समामिध बनती ह वहा एक फल चढती ह और डीजल आती ह निफरोजा उस समामिध की आजीवन -ासी बनी रहती हलखक अपन उददशय अथात -ास परथा पर परकाश डालन और इस परथा क कारण होन वाल -ातो क दखो को दि-खान म पणता सफल हए ह

helliphellipContinue to next

Biology Reproductio Today we will discuss about vegetative Q1 Name some vegetative propagules

n in Organisms

propagation of plants The process of multiplication in which fragments of plant body function as propagule and develop into new individual is called vegetative propagation The units of such propagation are runner rhizome tuber bulb etc

and the speciesinvolvedVegetative propagules

Parts involved

Bulb StemBulbil BulbilRhizome Stem Runner Stem Tuber Stem Offset Stem Leaf buds Leaves Suckers Stem

Corns Stem stolon

Q2 State advantages of vegetative propagation

i) Rapid methodii) Sure and easy methodiii) Useful in plants that cannot

produce viable seeds or long seed dormancy

iv) Maintains purity of raceQ 3 Banana fruit is said to be parthenocarpic where as turkey is said to be parthenogenetic WhyBanana develops without fertilization from an unfertilized ovary thus is parthenocarpicIn turkey the ovum or female gamete developinto a new chick without fertilization thus isparthgenetic

Q4 Why is water hyacinth is called as a ldquoTerror of Bengalrdquo Water hyacinth can

propagatevegetatively all over the water body in a short per short period of time This resulted increased biochemicaloxygen oxygen demand of water body causing mortalityof fishes It is very difficult to get rid off them Thus known as terror of Bengal

Chemistry

Solid state GENERAL CHARACTERISTICS OF SOLID STATEIn nature the particular state of matter is governed by two opposing forces at given set of temperature and pressure These forces are intermolecular force of attraction and thermal energy If intermolecular force of attraction is high as compared to thermal energy particles remains in closest position

Intext QuestionsQ1 Classify the following solids as crystalline and amorphous Sodium chloride quartz glass quartz rubber polyvinyl chloride Teflon

A1 Crystalline

and hence very less movement in particles is observed In this case solid state is the preferred state of matter

Let us revise the general characteristics of solid

i) Fixed mass volume and shape

ii) Strong intermolecular force of attraction

iii) Least intermolecular space

iv) Fixed position of constituent particles

v) Incompressible and rigid

Q2 what type of interactions hold the molecules together in a polar molecular solid[CBSE 2010]A2 The molecules in a solid are held together by van der Waals forces The term van der Waals forces include hydrogen bonding dipole-dipole attraction and London dispersion forces All molecules experience London dispersion forces In addition polar molecules can also experience dipole-dipole interactions So the interactions that holds the molecule together in polar molecular solid are London dispersion force and dipole-dipole interactionsQ3 Write a feature that will distinguish a metallic solid from an ionic solid [CBSE 2010]A3 Metals are malleable and ductile whereas ionic solid are hard and brittle Metallic solid has typical metallic lustre But ionic solid looks dullQ4 Write a point of distinction between a metallic solid and an ionic solid other than metallic lustre [CBSE 2012]A4 Metals are malleable and ductile whereas ionic solid are hard and brittleQ5 Write a distinguish feature of metallic solid [CBSE 2010]A5 The force of attraction in

solid Sodium chloride Quartz Amorphous solid Quartz glass rubber polyvinyl chloride Teflon Q2 why glass is considered as super cooled liquidA2 Glass shows the tendency to flow at slower rate like liquid Hence they considered as super cooled liquidQ3 why the window glass of old buildings show milky appearance with timeA3 Glass is an amorphous solid Amorphous solid has the tendency to develop some crystalline character on heating Due to heating in day over the number of years glass acquires some crystalline character and show milky appearanceQ4 why the glass panes fixed to window or doors of old building become slightly thicker at bottomA4 Glass is super cooled liquid It has the tendency to flow down very slowly Due to this glass pane becomes thicker at the bottom over the timeQ5 Sodium chloride is a crystalline solid It shows the same value of refractive index along all the direction TrueFalse Give reasonA5 FalseCrystalline solid shows anisotropy in properties That is it shows different values for the given physical property in different direction All the crystalline solids show anisotropy in refractive index Therefore sodium chloride will show different values of refractive index on different directions

Q6 Crystalline solid are anisotropic in nature What does this statement means

between the constituent particles is special kind of electrostatic attraction That is the attraction of positively charged kernel with sea of delocalized electronsQ6 which group of solid is electrical conductor as well as malleable and ductile [CBSE 2013]A6 Metallic solidQ7 why graphite is good conductor of electricity although it is a network (covalent solid)A7 The exceptional property of graphite is due to its typical structure In graphite each carbon is covalently bonded with 3 atoms in same layer The fourth valence electron of each atom is free to move in between different layersThis free electron makes the graphite a good conductor of electricity

[CBSE 2011]A6 Anisotropy is defined asrdquo Difference in properties when measured along different axis or from different directionsrdquo Crystalline solid show different values of some of the physical properties like electrical resistance refractive index etcwhen measured along the different directions The anisotropy in crystalline solid arises due to the different arrangement of particles in different directions

Math Function Composition of functions Think of an industrial plant that produce bottles of cold drinks first there is the operation (or function) f that puts the cold drink inside the bottle followed by the opeartion g that close the bottle with the capThis leads to the following definitionDefinition Let f A rarr B and g B rarr C be two functions Then the composition of f and g denoted by gof is defined as the function gof A rarr C given by gof(x) = g(f (x)) forall x isinA

Definition A function f X rarr Y is defined to be invertible if there exists a function g Y rarr X such that gof = IX and fog = IY The function g is called the inverse of f and is denoted by f -1

Thus if f is invertible then f must be one-one and onto and conversely if f is one-one and onto then f must be invertible This fact significantly helps for proving a function f to be invertible by showing that f is one-one and onto specially when the actual inverse of f is not to be determined

Example 1 Let f 2 3 4 5 rarr 3 4 5 9 and g 3 4 5 9 rarr 7 11 15 be functions defined as f(2) = 3 f(3) = 4 f(4) = f(5) = 5 and g (3) = g (4) = 7 and g (5) = g (9) = 11 Find gofSolution We have gof(2) = g (f(2)) = g (3) = 7 gof(3) = g (f(3)) = g (4) = 7gof(4) = g (f(4)) = g (5) = 11 and gof(5) = g (5) = 11Example 2 Find gof and fog if f R rarr R and g R rarr R are given by f(x) = cos x and g (x) = 3x2 Show that gof ne fogSolution We have gof(x) = g(f(x))=g(cosx) = 3 (cos x)2

= 3 cos2 x Similarly fog(x)=f(g (x))= f(3x2)= cos (3x2) Note that 3cos2 x ne cos 3x2 for x = 0 Hence gof ne fogExample 3 Show that if f A rarr B and g B rarr C are onto then gof A rarr C is also ontoSolution Given an arbitrary element z isin C there exists a pre-image y of z under g such that g (y) = z since g is onto Further for y isin B there exists an element x in A with f(x) = y since f is onto Therefore gof(x) = g (f(x)) = g (y) = z showing that gof is onto Example 4 Let Y = n2 n isin N sub N Consider f N rarr Y as f(n) = n2 Show that

f is invertible Find the inverse of fSolution An arbitrary element y in Y is of the form n2 for some n isin N This implies that n =radicy This gives a function g Y rarr N defined by g (y) =radicy Nowgof (n) = g (n2)=radicn2 = n and fog (y) =f(radicy) = (radicy) 2 y which shows that gof=IN and fog= IY Hence f is invertible with f -1 = g

Political Science

Constitution of India-The Preamble

Summary

Objective of the state-To secure equality of status and of opportunity To promote fraternity among all the citizens To assure the dignity of the individuals and Unity and integrity of the nation

Justice-Justice stands for rule of law absence of arbitrariness and a system of equal rights freedom and opportunities for all in a society India seeks social economic and political justice to ensure equality to its citizens

Liberty-Liberty implies the absence of restraints or domination on the activities of an individual such as freedom from slavery serfdom imprisonment despotism etc The Preamble provides for the liberty of thought expression belief faith and worship

Equality-Equality means the absence of privileges or discrimination against any section of the society The Preamble provides for equality of status and opportunity to all the people of the country

Fraternity-The Preamble declares that fraternity has to assure two thingsmdashthe dignity of the individual and the unity and

Execution

Answer the following questions-

Short notes-1 Equality2 Fraternity3 Justice4 Liberty

Homework-Learn

integrity of the nation The word integrity has been added to the Preamble by the 42nd Constitutional Amendment (1976)

Business studies

Human resource management (chapter 1)

On the day of 1504 2020 I have discussed with you the managerial functions and procurement functions of HRM

Today weare going to discuss about the development function integration functions and maintenance function

Development functions-HRM improves the knowledge skills attitude and values of employees so that they the present and future jobs more effectively it includes

1) Development functions of HRM

a) Performance appraisal = It implies systematic evaluation of employees with respect to their performance on the job and their potential for development

b) Training =It is the process by which employees learn knowledge skills and attitudes to achieve organisational and personal goals

c) Executive development = It is the process of developing managerial talent through appropriate program

2) Integration functionsa) HRM reconcile the goals of

organisation with those of its members through integrating function

b) HRM tries to motivate employees to various financial and non financial incentives provided in job specification etc

3) Maintenance functiona) HRM promote and protect the

physical and mental health of employees by providing several types of benefits like housing medical aid etc

b) It Promote Social security measures to employees by providing provident fund pension gratuity maternity benefits

SubjectCOMMERCE

Topic

BUSINESSENVIRONMENT

Summary

Now quickly let us revise the earlier points that we have already done in the last class and let us proceed with the other topics that are there in the chapter

Firstly we will recall the internal and external factors of micro environment and then we

Execution 3 What do you mean by internal factors

in micro environmentAnswerInternal factors refer to all the factors existing within a business firm The internal factors are considered controllable because the enterprise has control over these factors

Development FunctionsPerformance AppraisalTrainingExecution Development

shall proceed in details

Meaning and list of internal and external factors

aInternal factorsInternal factors refer to all the factors existing within a business firm The internal factors are considered controllable because the enterprise has control over these factorsFor an example a company can alter its organization structure policies programmes employees physical facilities and marketing mix to suit the changes in the environmentList of internal factors areCorporate culture mission and objectives top management organizations structure company image and brand equity company resources

b External factorsExternal factors refer to those individual and groups and agencies with which a particular business organization comes into direct and frequent contact in the course of its functioningThese individuals and groups are known as STAKEHOLDERS because they have a stake (financial interest ) in the working and performance of the particular business List of external forces (stakeholders)Customers competitors investors suppliersmiddlemen (marketing intermediaries)financers publics

customers

suppliersfinancers

For an example a company can alter its organization structure policies programmes employees physical facilities and marketing mix to suit the changes in the environment

4 What do you mean by external factors in micro environment

AnswerExternal factors refer to those individual and groups and agencies with which a particular business organization comes into direct and frequent contact in the course of its functioningThese individuals and groups are known as STAKEHOLDERS because they have a stake (financial interest) in the working and performance of the particular business

3Who are stakeholdersSTAKEHOLDERS are individuals and groups who have a stake (financial interest ) in the working and performance of the particular business 4Discuss the internal factors in briefa Corporate CultureThe values beliefs and attitudes of the founders and top management of the company exercise a strong influence on what the cmpaany stands for how it does things and what it considers importantbMission and objectivesThe business philosophy and purpose of a comoany guide it prioritiesbusiness strategiesproduct market scope and development scope

cTop management structurethe composition of board of directors the degree of professionalization of management and the organizational structure of a company have important bearing on its business decisions

dPower structureThe internal power relationship between the board of directors and the chief executive is an important factor

e Company image and brand equityThe image and brand equity of the company play a significant role in raising finance forming alliance choosing dealers and suppliers launching new products entering foreign markets

5 What is Macro environmentAnswerMacro environment refers to the general

competitors

middlemen

publics

Fig STAKEHOLDERS OF A COMPANY

Apart from micro environment the other main dimension of business environment isMacro environment Macro environment refers to the general environment or remote environment within which a business firm and forces in its micro environment operateA company does not directly or regularly interact with the micro environmentTherefore macro environment is also known as indirect action EnvironmentThe macro environment forces are less controllable than the micro forces

Macro environment consists of the following components

POLITICAL AND LEGAL ENVIRONMENT

ECONOMIC SOCIAL AND ENVIRONMENT

CULTURAL

ENVIRONMENT

TECHNOLOGICAL ENVIRONMENT

Fig COMPONENTS OF MACRO ENVIRONMENT

environment or remote environment within which a business firm and forces in its micro environment operateA company does not directly or regularly interact with the micro environmentTherefore macro environment is also known as indirect action EnvironmentThe macro environment forces are less controllable than the micro forces 6 What are the components of macro environmenta Political and legal environmentb Economic environmentc Social and cultural environmentd Technological environment

Computer Science

Logic gates

Digital systems are said to be constructed by using logic gates These gates are the AND OR NOT NAND NOR EXOR and EXNOR

BUSINESS FIRM

gates The basic operations are described below with the aid of truth tables

AND gate

The AND gate is an electronic circuit that gives a high output (1) only if all its inputs are high A dot () is used to show the AND operation ie AB Bear in mind that this dot is sometimes omitted ie ABOR gate

The OR gate is an electronic circuit that gives a high output (1) if one or more of its inputs are high A plus (+) is used to show the OR operationNOT gate

The NOT gate is an electronic circuit that produces an inverted version of the input at its output It is also known as an inverter If the input variable is A the inverted output is known as NOT A This is also shown as A or A with a bar over the top as shown at the outputs The diagrams below show two ways that the NAND logic gate can be configured to produce a NOT gate It can also be done using NOR logic gates in the same way

NAND gate

This is a NOT-AND gate which is equal to an AND gate followed by a NOT gate The outputs of all NAND gates are high if any of the inputs are low The symbol is an AND gate with a small circle on the output The small circle represents inversion

NOR gate

This is a NOT-OR gate which is equal to an OR gate followed by a NOT gate The outputs of all NOR gates are low if any of the inputs are highThe symbol is an OR gate with a small circle on the output The small circle represents inversion

EXOR gate

The Exclusive-OR gate is a circuit which will give a high output if either but not both of its two inputs are high An encircled plus sign ( ) is used to show the EOR operation

EXNOR gate

The Exclusive-NOR gate circuit does the opposite to the EOR gate It will give a low output if either but not both of its two inputs are high The symbol is an EXOR gate with a small circle on the output The small circle represents inversion The NAND and NOR gates are called universal functions since with either one the AND and OR functions and NOT can be generated

Note A function in sum of products form can be implemented using NAND gates by replacing all AND and OR gates by NAND gates A function in product of sums form can be implemented using NOR gates by replacing all AND and OR gates by NOR gates

Logic gate symbols

Table 2 is a summary truth table of the inputoutput combinations for the NOT gate together with all possible inputoutput combinations for the other gate functions Also note that a truth table with n inputs has 2n rows You can compare the outputs of different gates

Logic gates representation using the Truth table

Example

A NAND gate can be used as a NOT gate using either of the following wiring configurations

Subject Eng Literature (The Tempest ndash William Shakespeare) Topic Act III Scene 3 Lines 53 to 110 (End of the scene) Date 16th April 2020 (2nd Period)

[Students should read the original play and also the paraphrase given in the school prescribed textbook]Summary Questions amp Answers

o Seeing this strange scene all are inclined to believe the tales told by travelers that there truly are ldquounicornsrdquo and ldquothe phoenixrsquo thronerdquo

o As they are about to sit down to the feast the banquet is snatched away by a harpy (Ariel disguised) A spiritrsquos voice (Arielrsquos voice) denounces Alonso Sebastian and Antonio with particular

1 ARIEL You are three men of sin whom Destiny

(Line 53-58)That hath to instrument this

lower world And what is int the never-surfeited sea

Hath caused to belch up you and on this island

Where man doth not inhabit you rsquomongst men

Being most unfit to live I have made you mad

reference to their crime in expelling Prospero from Milan They have not received any punishment for their deed earlier but the time for their punishment has arrived Upon Alonso it pronounces ldquolingering perdition worse than deathrdquo from which there is no remedy except through sincere repentance Ariel then vanishes in thunder and the shapes enter again and carry away the table

o Prospero watching invisibly is very pleased with the performance of Ariel and his (Prosperorsquos) ldquomeaner ministersrdquo All his enemies are now in his power and are in a fit of desperation He then leaves them and goes to see how Ferdinand and Miranda are getting on

o Alonso is now much humbled and penitent with the after effect of the spiritrsquos denunciation of his crimes He believes that his son is lost forever After this all disperse being stricken mad by the speech of the spirit

o Gonzalo fearing that they may do violence to themselves or to one another follows them and bid others to follow

(a) To whom does Ariel disguised as a harpy call the three sinners What game did Fate of Destiny play with

them

The three sinners called by Ariel are Alonso Sebastian and Antonio It was Destiny which had caused the ocean to cast the three sinners on the shore Though the ocean is all the time devouring whatever appears on its surface and is never satisfied with its continual swallowing of the ships and men in the present case the ocean had cast these three sinners on the shore without killing them

(b) Who had jointly been responsible for the conspiracy against Prospero What is Prosperorsquos purpose behind all this

Three men Alonso Sebastian and Antonio had jointly

been responsible for the conspiracy against Prospero They had driven out Prospero form Milan Prosperorsquos purpose is to make these three sinners realize the wrong they had done He wants them to repent for their criminal deeds because repentance leads to self-esteem(c )What does Ariel (the harpy) tell Alonso and his companions when they take out their swords to attack him

Seeing them drawing their swords Ariel (harpy) tells them that he and his companions are the instruments of destiny and that it is not possible for human beings to do them any injury He says that the swords of human beings can not injure even a minute part of his feathers Their swords are as ineffective against him and his companions as against the wind or the water

(d) Give the explanatory meanings of the following expressions in the context of the above extract

(i)Never surfeited (ii) Belch up (iii) lsquomongst men

(i) Never surfeited never led to satisfaction

(ii) Belch up cast ashore(iii) lsquomongst men in human

society2

I and my fellows (Line 60-65)

Are ministers of Fate The elementsOf whom your swords are tempered may as wellWound the loud winds or with bemocked-at stabsKill the still-closing waters as diminishOne dowl thats in my plume

IMPORTANT PASSAGES EXPLAINED

The elements

(Line 61-66)Of whom your swords are tempered may

as wellWound the loud winds or with

bemocked-at stabs

(a) Who is lsquoIrsquo Who are his lsquofellowsrdquo

lsquoIrsquo is referred to Ariel in disguise of a harpy His lsquofellowsrsquo are other spirits serving Prospero the real Duke of Milan who has acquired supernatural powers after being banished from his Dukedom Prospero has settled in this uninhabited island

(b) What are the elements that have temperrsquod the swords Why will it not work against the speaker

The swords (of Alonso and his companions) are tempered by metal (steel) which is taken out of the earth and refined by

Kill the still-closing waters as diminishOne dowl thats in my plume My fellow

ministersAre like invulnerable

In these words Ariel reminds the King and his companions of the utter futility of drawing swords against himself and his fellows Ariel drives Alonso Antonio and Sebastian the three men of sin to desperation ndash a state in which men do violence to themselves They draw swords to strike Ariel But Ariel reminds them that he and the other spirits are the ministers of destiny and nothing can wound them The steel of which their swords are made of may cut the wind or water which being divided always closes up again Even supposing that such things may be possible it is quite impossible that their swords will cut one feather in their plume They are incapable of being wounded by any sword of man Hence it is foolish on their part to attempt to strike at Ariel and his fellow-spirits

For which foul deed

(Line 72-75)The powers delaying not forgetting

haveIncensed the seas and shores yea all the

creatures Against your peace

Ariel enters like a harpy and remaining invisible tells Alonso Sebastian and Antonio that he and other harpies are the agents of Destiny appointed to carry out her decrees He tells them that their punishment for the crime against Prospero which has been so long deferred is now to fall upon them He reminds them that they had expelled Prospero from Milan and set him and his innocent child adrift on the sea and that the sea had paid them back for their sin by the shipwreck and by the calamities they have suffered He tells them that the powers above which did not forget this mean treachery but only deferred the punishment have now engaged the seas and the shores and all living beings including him and his comrades against them The very elements and supernatural agency Ariel adds have taken up the avenging of their crime against Prospero

the action of fire It may cut the wind or water which being divided always closes up again

The sword will not work against the spirits and the harpy because they are the ministers of destiny and nothing can wound them nor it will cut a single feather in their plume

(c )What is the meaning of lsquodowlrsquo in the last line

The term lsquodowlrsquo means a filament or the smallest part of a feather In this context Ariel in disguise of harpy says that their sword cannot even damage the smallest filament of their (Arielrsquos and other spirits) feathers as they are incapable of being wounded by any sword of man

(d) What does the speaker remind the listeners about

Ariel in disguise of harpy reminds Alonso the King of Naples Sebastian Alonsorsquos brother and Antonio the present Duke of Milan and the treacherous brother of Prospero as they being three men of sin He even reminds them that their punishment for their crime against Prospero which has been so long deferred now falls upon them He reminds them that they have expelled Prospero from Milan and has set him along with his innocent infant daughter adrift on the sea So the sea has paid them back for their sin by their shipwreck and the calamities they have suffered since then The harpy rebukes Alonso of his sin that has incensed the Gods and has deprived him of his son as a punishment

(e) How do they respond

When Ariel in disguise of a harpy reminds Alonso Sebastian and Antonio of their past misdeeds and sin Alonso has a look of terror and confusion in his eyes He utters the words of sincere repentance wrung out of his conscience-stricken heart It appears to him that all the elements of nature the sea-waves the wind and the thunder proclaiming a loud voice in the name of Prospero and the crime Alonso has committed against him They are calling upon him to repent There is a deep storm raging in Alonsorsquos breast and the echoes of that storm are ringing in his ears like a clear note of wind-instrument A note of denunciation of Alonsorsquos crime leaves him much humbled and penitent and confirms his belief that his son is lost forever But Sebastian and Antonio shows some courage instead of repentance They wish to kill the spirits or devils if it appears

3

Of my instruction hast thou nothing bated (Line 85-93)

In what thou hast to say So with good life

And observation strange my meaner ministers

Their several kinds have done My high charms work

And these mine enemies are all knit upIn their distractions They now are in my

powerAnd in these fits I leave them while I visitYoung Ferdinand whom they suppose is

drownedAnd his and mine loved darling

Methought the billows spoke and (Line 96-99)

told me of itThe winds did sing it to me and the

thunderThat deep and dreadful organ-pipe

pronouncedThe name of Prosper It did bass my

trespass

These are the words of contrition coming from Alonso Ariel has driven him to a deep repentance for conspiring with Antonio against Prospero He now feels a sincere remorse It appears to him that all the elements of nature the sea-waves the wind and the thunder proclaimed with a loud voice the name of Prospero and the crime Alonso had committed against him They are calling upon him to repent There is a deep storm raging in Alonsorsquos breast and the echoes of that storm are ringing in his ears like the clear note of a wind-instrument

Comment These are the words of sincere repentance wrung out of the conscience-stricken heart of Alonso Alonso who is the lesser villain is the first to give way to remorse under the effect of Arielrsquos speech The words of Ariel seem to him to be the voice of conscience speaking to him He is driven to desperation a state in which he might do violence to his life

(a) Identify the speaker State the context

Prospero the ruler of the island is the speaker The famous banquet scene has been enacted very well Ariel and his junior spirits have played their roles excellently Prospero is glad to say words of praise for them(b) In what way the speakerrsquos instructions have been carried out

According to Prosperorsquos instructions a banquet was presented before the King of Naples and his companions when they were tired and hungry Just when they were preparing to eat the feast the banquet was suddenly removed by exercising supernatural powers All this was done by Ariel Prosperorsquos chief assistant and a powerful spirit

Ariel not only made the feast disappear but also delivered his speech blaming the King and his two companions for their past wicked deeds He warned them to repent for their misdeeds or suffer forever on that uninhabited island

(c) Who are referred to as lsquomeaner ministersrsquo What have they done

Prospero refers as lsquomeaner ministersrsquo to his other lesser spirits who were assisting Ariel in presenting a scene before the kingrsquos party They entered the scene to the accompaniment of music They assumed several strange shapes and brought in a banquet Then they danced about it with gentle actions of salutations thus inviting the King and others to eat the feast

These spirits play their role again when Ariel in the shape of a harpy quits the scene These shapes enter again and dancing with mocking gestures carry away the table

(d) Who are the speakerrsquos enemies What has happened to them

King of Naples Alonso his brother Sebastian and the present Duke of Milan Antonio (Prosperorsquos own brother) are Prosperorsquos enemies With the turn of events they have all been washed ashore on the island which is ruled by Prospero the great magician Actually this happened after the shipwreck caused by a storm which was raised by Prospero with the purpose of bringing these people to his island Prosperorsquos spirits have already confused and terrified these enemies and they are under Prosperorsquos control He can treat them as he likes

(e) What does he say about Ferdinand Explain what is meant by ldquohellip his and mine darlingrdquo

Prospero knows that Alonsorsquos son prince Ferdinand is alive though his father thinks that the prince has been drowned

Prospero refers to his daughter Miranda who is dear to him She is also very dear to Prince Ferdinand who has fallen in love with her They are waiting to be married soon for which they have received Prosperorsquos consent

4

ALONSO O it is monstrous monstrous (Line 95-102)

Methought the billows spoke and told me of it

The winds did sing it to me and the thunderThat deep and dreadful organ-

pipe pronouncedThe name of Prosper It did bass

my trespassTherefore my son ithrsquo ooze is

bedded andIll seek him deeper than eer

plummet soundedAnd with him there lie mudded

(a) In what way does Alonso express his horror when his conscience is awakened by Arielrsquos words

When Alonsorsquos conscience is awakened by Arielrsquos words he expresses his horror at what he has heard He gets the feeling that the waves of the ocean the wind and the loud thunder have spoken to him and uttered the name of Prospero Because of being reminded of his crime in a very loud and rough voice he comes to realize that he has lost his son for his past misdeeds

(b) What does Alonso imagine about his son What does Alonso want to do in his desperate state

Alonso imagines that his son is lying in the mud at the bottom of the sea He feels desperate that he wants to drown himself in the ocean deeper than the plumb-line has ever gone He wants to lie with his son at the bottom of the sea

(c) How do Sebastian and Antonio want to face the evil spirits

Sebastian says that he is not at all afraid of what the harpy has said and that he is prepared to fight any number of such monsters if they appear before him only one at a time Antonio says that he would support Sebastian in the fight against the fiendsyyy

(d) Why does Gonzalo ask Adrian to follow the three men

Gonzalo tells Adrian that all the three men namely Alonso Sebastian and Antonio are in a wild and reckless mood The thought of the heinous crime of which they are guilty has begun to torment their minds So he asks Adrian to follow those three men without loss of time and prevent them from doing anything which the turmoil in their minds might lead them to do

(e) What opinion do you form of Alonso from the above extract

Alonso who is the lesser villain is the first to give way to remorse under the effect of Arielrsquos speech The words of Ariel seem to him to be the voice of conscience speaking to him He is driven to desperation a state in which he might do violence to his life

Subject =Accounts

Ac-12 15420 topic-pL Appropriation ac

PROFIT AND LOSS APPROPRIATION ACCOUNT

MEANING AND PREPARATIONProfit and Loss Appropriation Account is merely an extension of the Profit and Loss Account of the firm The profit of the firm has to be distributed amongst the partners in their respective profit sharing ratio But before its distribution it needs to be adjusted All Adjustments like partnerrsquos salary partnerrsquos commission interest on capital interest on drawings etc are made in this account These adjustments will reduce the amount of profit for distribution This adjusted profit will be distributed amongst the partners in their profit sharing ratio To prepare it at first the balance of Profit and Loss Account is transferred to this account The journal entries for the preparation of Profit and Loss Appropriation Account are given below

1 for transfer of the balance of Profit and Loss Account to Profit and Loss Appropriation Account

(a) In case of Net Profit

Profit and Loss Ac helliphelliphelliphelliphellipDrTo Profit and Loss Appropriation Ac(Net Profit transferred to Profit and Loss Appropriation Ac)

(b)In case of Net Loss

Profit and Loss Appropriation Achelliphelliphellip DrTo Profit and Loss Ac(Net Loss transferred to Profit and Loss Appropriation Ac)

2 for Interest on Capital

For transferring on Interest on CapitalProfit and Loss Appropriation Achelliphelliphellip DrTo Interest on Capital Ac(Interest on capital transferred to Profit amp Loss Appropriation Ac)

3 for Interest on Drawings

For transferring Interest on Drawings Interest on Drawings Achelliphelliphelliphelliphelliphellip DrTo Profit and Loss Appropriation Ac(Interest on drawing transferred to Profit amp Loss Appropriation Ac)

4 For Partnerrsquos SalaryFor transfer of partnerrsquos SalaryProfit and Loss Appropriation Achelliphellip DrTo Salary Ac(Salary transferred to profit amp Loss Appropriation Ac)

5 For Partnerrsquos CommissionFor transferring commissionProfit and Loss Appropriation Achelliphelliphellip DrTo Commission Ac(Commission transferred to Profit and Loss Appropriation Ac)

6 For Transfer of agreed amount to General ReserveProfit and Loss Appropriation Ac helliphellipDrTo General Reserve Ac(Transfer to General Reserve)

7 for share of Profit or Loss appropriation(a) If ProfitProfit and Loss Appropriation Achelliphellip DrTo Partnerrsquos CapitalCurrent Ac(Profit transferred to capitalcurrent Ac)(b) If LossPartnerrsquos Capital Current Achelliphelliphelliphellip DrTo Profit and Loss Appropriation Ac(Loss transferred to capitalcurrent Ac)

THE FORMAT OF PROFIT AND LOSS APPROPRIATION

Profit and Loss Appropriation Account for the year endedhelliphelliphelliphellip

Particulars Amount Particulars Amount

To PL Ac (loss) By pL Ac (profit)

To Interest on capital BY Interest on drawings

To partner`s commission by Partner`s capital Ac ( loss)

To Partner`s salary To Interest on partner`s loan To General Reserve To Partner`s Capital AC (Profit)

Subject= Economics

MOVEMENT ALONG THE DEMAND CURVE (CHANGE IN QUANTITY DEMANDED)In law of demand you have already studied the inverse relationship between price and quantity demanded When quantity demanded of a commodity changes due to change in its price keeping other factors constant it is called change in quantity demanded It is graphically expressed as a movement along the same demand curve There can be either a downward movement or an upward movement along the same demand curve Upward movement along the same demand curve is called contraction of demand or decrease in quantity demanded and downward movement along the same demand curve is known as expansion of demand or increase in quantity demanded

Extention of demandd

price (rs)p A

B Extentionp1 d

Q Q1

Quantity demanded ( in units)

Contraction of demandd

p2 Ccontraction

p APrice (Rs)

d

Q2 Q

Quantity demanded (in units)

Explanation of movement of demand A fall in price from OP to OP1 leads to increase in quantity demanded from OQ to OQ1 (expansion of demand) resulting in a downward movement from point A to point B along the same demand curve DD When Price rises from OP to OP2 quantity demanded falls from OQ to OQ2 (contraction of demand) leading to an upward movement from point A to point C along the same demand curve DD

  • Activity Series of Metals
    • Drawbacks of Rutherfordrsquos model of atom
      • Electromagnetic radiations
      • Properties of electromagnetic radiations
      • Characteristics of electromagnetic radiations
        • Plancks Quantum Theory-
        • Photoelectric effect
          • Intext Questions
            • Logic gates
            • Digital systems are said to be constructed by using logic gates These gates are the AND OR NOT NAND NOR EXOR and EXNOR gates The basic operations are described below with the aid of truth tables
            • AND gate
            • Example
Page 9:  · Web viewSubject. Topic. Summary. Execution. English 1 . Chapter 1 naming words . Page 8. Write the names of these pictures:- Person:-1. father. 2.Firefighter 3.doctor 4 ...

The Gandhara art was developed under Buddhist patronage

BIOLOGY The Leaf Photosynthesis The process by which green plants make their own food from carbon dioxide and water in the presence of sunlight and chlorophyll is called photosynthesis

All green plants need the following to make their food ndash

water carbon dioxide chlorophyll and energy in the form of sunlight

Carbon dioxide + water ------------- Glucose + oxygen

The end product of photosynthesis is glucose

Fill in the blanks

1 Plants make their food by the process of photosynthesis

2 The inner wall of the guard cell is thicker than the outer wall

3 The extra glucose is converted into starch and sucrose

4 The leaf is boiled in alcohol to remove chlorophyll

5 The rate of transpiration is more on the hot day then a cold day

6 Photosynthesis helps to observe water and minerals from the soil

English 1 Pronouns Kinds of pronouns 1 Personal pronouns2 Possessive pronouns3 Reflexive pronouns4 Interrogative pronouns5 Relative pronouns6 Demonstrative pronouns7 Indefinite pronouns

Personal pronouns they refer to first second and third person in sentences First person- the speakerSecond person-the listenerThird person-the objectperson being spoken aboutPersonal pronouns should have the same gender and number as the nouns they refer to

Possessive pronouns these are used to indicate the relationship between the objects and people These pronouns include mine ours yours his hersand theirs

ExerciseBFill in the blanks with suitable pronounsThere was much excitement among the childrenTheywere eagerly looking forward to the annual picniclsquoAre they going to Lodhi Gardens toorsquo wondered AneeshlsquoNo they are going to Buddh Jayanti Park with Mrs Jain said Mrs ChopraThe children looked disappointedlsquoWonrsquot you be taking us Marsquoam rsquo they askedlsquoSorry children I have to go to Mumbai for a week to look after my sick mother But you will have fun with Mrs Jain she is full of laughter and you will love being with her the whole daylsquoIt will not be the same they grumbled

English 2 The great train journey- Ruskin Bond

The great journey by Ruskin Bond is a story about Suraj who loved trains and wanted to go to places One day while wandering along the railway tracks he enters into a carriage compartment The train suddenly starts moving with him in the compartment and after a journey returns back to the same place from where it had begun The story is about his experience during that journey

State true or false1 When the train had passed leaving behind the

hot empty track Suraj was lonely2 It was winter holidays

and Suraj did not know what to do with himself

3 He plunged his hands into the straw and pulled out an apple

4 A dirty bearded face was looking out at him from behind a pile of crates

5 Suraj wanted to go to Japan

Hindi 2nd lang

गललबाजलडका खालीसथानोकोभरो-6 गो-ामसनिनकलकरहमगराजमआगए7 माबोधराजकोराकषससमझतीथी8 चीलरोशन-ानमसअ-रआकरतहसीलपरबठगई9 तीनचारतीनकऔररईकगोलउडलनिकनघोसलानहीनिगरा10 वहसवयतोघोसलातोडनककतिलएगललउठालायाथा

11 -ीवारकसाथलगतगोहपजोकसहार-ीवारपकडलतीह12 बोधराजअभीभीटकटकीबाधचीलकीओर-खरहाथा13 बोधराजअपनीजबमबहतसाचगगाभरकरलायाथा14 मरनिपताजीकीतरककीहईऔरहमलोगएकबडघरमजाकररहनलग15 बागमजातातोफलपरबठीनिततलीको-खनिततलीकोपकडकरउगकतिलयोकबीचमसल-ता

BENGALI(2ND LANGUAGE)

সহিনধসবরপওসবরসহিনধ

সবরসহিনধরহিনয়ম- ৯ই-কারহিকংাঈ-কাচেররপচেরইাঈহিভননঅনযসবররণCথাকচেইাঈসথাদেনয-ফায়এংওইয- ফাপCচেরণCযকতয়

১০উ-কারহিকংাঊ-কাচেররপচেরইাঈহিভননঅনযসবররণCথাকচেউাঊসথাদেন-ফায়এংওই- ফাপCচেরণCযকতয়

১১ঋ- কাচেররপচেরঋহিভননঅনযসবররণCথাকচেঋসথাচেনর -ফায়এংওইর পCচেরণCযকতয়

১২সবররণCপচেরথাকচেপCতMএ-সথাচেনঅয় ঐ- সথাচেনআয় ও- সথাচেনঅএংঔ- সথাচেনঅায়

৯ই+ অ= য- ফাআহি+ অনত= আযনত অহিধ+ অয়ন= অধযয়নই+ আ=য- ফা+ াইহিত+ আহি= ইতযাহি পরহিত+ আতC ন= পরতযাতC নই+ উ=য- ফা+ উঅহিত+ উহিকত= অতযহিকত হি+ উৎপহিতত= যৎপহিততই+ ঊ= য- ফা+ ঊ ই+ এ= য- ফা+ এঈ+ অ= য- ফা পরহিত+ ঊ4= পরতয4 পরহিত+ এক= পরচেতযকঈ+ অ আ= য- ফা+ অ আনী+ অমব= নযমব মসী+ আধার= মসযাধার

১০উ+ অ= অন+ অয়= অনবয় পশ+ অধম= পশবধমউ+ আ= াস+ আগত= সবাগত পশ+ আহি= পশবাহিউ+ ঈ= হিঅন+ ইত= অহিনবতউ+ এ= দেঅন+ এ4রণ= অচেনব4রণউ+ ঈ= ীসাধ+ ঈ= সবাধবী তন+ ঈ= তনবী

১১ঋ+ অ= র মাত+ অনমহিত= মাতরনমহিতঋ+ আ= রা হিপত+ আয়= হিপতরায়ঋ+ ই= হির মাত+ ইচছা= মাতচছাঋ+ ঈ= রী ধাত+ ঈ= ধাতরীঋ+ উ= র ভরাত+ উপচে(= ভরাতরপচে(

১২এ+ অ= অয় দেন+ অন= নয়নঐ+ অ= আয় গৈগ+ অক= গায়কও+ অ= অ দেপা+ অন= পনও+ ই= অ দেপা+ ইতর= পহিতরও+ এ= অ দেগা+ এ4রণা= গচে4রণাঔ+ অ= অা দেপৌ+ অক= পাকঔ+ ই= অা দেনৌ+ ইক= নাহিকঔ+ উ= অা দেভৌ+ উক= ভাক

MATHS Topic NumbersChapter Natural numbers and whole numbers

Study item properties of whole numbers for multiplication

1 Closure property If x and y are two whole numbers then xtimesy is also a whole numberExample If x = 9 and y =3 then xtimesy = 9times3 = 27 which is a whole number

2 Commutative property If x and y are two whole numbers then xtimesy = ytimesxExample If x = 5 and y = 2 then xtimesy = 5times2 = 10y times x = 2times5 = 10Therefore 5times2 = 2times5

3 Associative property If x y and z are three whole numbers then x times(ytimesz) = (xtimesy) times zExample If x =3 y = 5 and z = 7 then 3 times (5times7) = 3 times (35) = 105And (3times5) times7 = (15) times 7 = 105Therefore x times (ytimesz) = (xtimesy) timesz

4 Distributive property If x y and z are three whole numbers then xtimes (y + z) = x times y + x times z

Therefore the multiplication of whole numbers is distributive over their additionExample If x = 5 y = 3 and z= 2Therefore x times (y + z) = 5 times (3 + 2) = 5times5 =25And x times y + xtimes z = 5times3 +5times2 =15 +10 = 25Again x times (y ndash z ) = x times y ndash x timesz Therefore 5 times ( 3 - 2) = 5 times1 = 5 and 5times3 ndash 5 times2 = 15 ndash 10 = 5Therefore the multiplication of whole numbers is also distributive over their subtraction if y is greater than z

5 Existence of identity If x is a whole number then

X times1 = x 1 times x = xTherefore we can write x times1 = 1 times xTherefore the multiplication of any whole number with 1 is the number itselfTherefore we can say that 1 is multiplicative identity or identity element for multiplicationExample 5 times1 = 5 1 times 5 = 5 Therefore 5 times 1 = 5

6 Multiplicative inverse If x is any whole number ( x is not equal to zero ) then its multiplicative inverse will be 1xSo x times 1x = 1 but 1x is a whole number if x = 1For other values of whole number 1x is not a whole number therefore we can write its multiplicative inverse does not exists

7 Cancellation law of multiplication If x y and z are three non- zero whole numbers then x times y = x times z

Or y = zExample 9 times y = 9 timeszTherefore y = z

Class VIISubject Topic Summary Execution

English 2 Sentences based on meanings

Kinds of sentences

Assertive or declarative to convey information or simply make a statement

Interrogative to ask different types of questions

Imperative to command or instruct someone or make a request

Exclamatory to express strong feelings and emotions

Exercise c1 What a nice compliment that is

That is a nice compliment2 How well- behaved the children

areThe children are very well-behaved

3 What great chefs we areWe are great chefs

4 What a shame it isIt is a shame

5 What a fantastic idea you haveYou have a fantastic idea

Homework 6 -10English

LiteratureThe Listeners III) Answer the following questions-

d) Identify two words used in the poem to give the poem an eerie atmosphereAns- Two words used to give the poem an eerie atmosphere are ldquogreyrdquo and ldquophantomrdquo

e) Who do you think are the inmates of the houseAns- I think the inmates of the house are phantom who once used to dwell in it

f) Why was the poet ldquoperplexed and stillrdquoAns- He was lsquo perplexed and stillrsquo because he was expecting an answer from the inmates of the house But despite of repeated calls there was no response

CHEMISTRY Chapter 2 ndashElement and Compound

Activity Series of MetalsThe activity series is a chart of metals listed in order of declining relative reactivity The top metals are more reactive than the metals on the bottomMetal SymbolReactivity

Lithium Li displaces H2 gas from water steam and acids and forms hydroxides

Potassium K

Strontium Sr

Calcium Ca

Sodium Na

Magnesium Mg displaces H2 gas from steam and acids and forms hydroxides

Aluminum Al

Zinc Zn

Chromium Cr

Iron Fe displaces H2 gas from acids only and forms hydroxides

Cadmium Cd

Cobalt Co

Nickel Ni

Tin Sn

Lead Pb

Hydrogen gas

H2 included for comparison

Antimony Sb combines with O2 to form oxides and cannot displace H2

Arsenic As

Bismuth Bi

Copper Cu

Mercury Hg found free in nature oxides decompose with heating

Silver Ag

Palladium Pd

Platinum Pt

Gold Au

Answer the following Q)Difference Between Metals And Nonmetals

Metals Nonmetals

These are solids at room temperature except mercury

These exist in all three states

These are very hard except sodium

These are soft except diamond

These are malleable and ductile

These are brittle and can be breakdown into pieces

These are shiny These are non-lustrous except iodine

Electropositive in nature Electronegative in nature

Have high densities Have low densities

Math Number System

Chapter Fraction

Study item Some solved sums from exercise 3(B)1) For each pair given below state whether it from like fractions or unlike

fractions (i) 58 and 78

= Like Fraction because denominators same(ii) 815 and 821

= Unlike Fraction because denominators are not same

(iii) 49 and 94 = Unlike Fraction

2) Convert given fractions into fractions with equal denominators(iii) 45 1720 2340 and 1116Solution Given fraction 45 1720 2340 and 1116Therefore the LCM of 5 20 40 and 16 is 80Therefore 45 = 4times165times16 = 64801720 = 17times420times4 = 68802340 = 23times240times2 = 4680 1116 = 11times516times5 = 5580

3) Convert given fractions into fractions with equal numerators(iii) 1519 2528 911 and 4547Solution Given fractions 1519 2528 911 and 4547Therefore the LCM of 15 25 9 and 45 is 2251519 = 15times1519times15 = 225285 2528 = 25times928times9 = 225252911 = 9times2511times25 = 2252754547 = 45times547times5 = 225235

4) Put the given fractions in ascending order by making denominators equal

(iii) 57 38 914 and 2021Solution Given fraction 57 38 914 and 2021Therefore the LCM of the denominators is 16857 = 5times247times24 = 12016838 = 3times218times21 = 63168914 = 9times1214times12 = 1081682021 = 20times821times8 = 160168Therefore ascending order 63168lt108168lt120168lt160168Therefore ascending order of given fractions38lt914lt57lt2021

COMPUTER CHAPTER-1COMPUTER FUNDAMENTALS

DONE IN THE PREVIOUS CLASSES PAGE 10CWRITE TRUE AND FALSE

1 True2 False3 False4 False5 True

GEOGRAPHY CHAPTER 7EUROPE

CHAPTER COMPLETE 1)Europe is home to a famous mountain range called the Alps

2)River Rhine originates in Switzerland

3)The Eiffel Tower one of the tallest structures in Europe

4) Vatican City is one of the most densely populated European countries

5)Sognefjordin Norway is the largest fjord in Norway

Class VIIISubject Topic Summary Execution

MATHEMATICS Ch 6Sets

Exercise 6 (D)1 Given A = x x isin N and 3iquest x le 6 and B = x x isin W and xlt4 find (i) Sets A and B in roster form (ii) A cup B (iii)

A cap B(iv) A ndash B (v) B ndashA

Solution (i) A = 456 and B = 0123

(ii) A cup B = 0123456 (iii) A cap B = ϕ (iv) A ndash B = 456 (v)B ndash A = 0123

3 If A = 56789 B = x 3 lt x lt 8 and x isin W and C = x xle5 and x isin N Find (i) A cup B and (A cup B) cup C (ii) B

cup C and A cup ( B cup C)

(iii) A cap B and (A cap B) cap C (iv) B cap C and A cap (B cap C)

Is (A cup B) cup C = A cup (B cup C)

Is (A cap B) cap C = A cap (B cap C)

SolutionA = 56789 B = 4567 C = 12345

there4 (i) A cap B = 456789 and (A cup B) cup C = 123456789

(ii) B cup C = 1234567 and A cup ( B cup C) = 123456789

(iii) A cap B = 567 and (A cap B) cap C = 5

(iv) B cap C = 45 and A cap (B cap C) = 5

Now (A cup B) cup C = 123456789

And A cup ( B cup C) = 123456789 there4 (A cup B) cup C = A cup (B cup C)

Again (A cap B) cap C = 5 and A cap (B cap C) = 5

there4 (A cap B) cap C = A cap (B cap C)

4 Given A = 012345 B = 02468 and C = 0369 Show that (i) A cup (B cup C) = (A cup B) cup C ie the union

of sets is associative (ii) A cap (B cap C) = (A cap B) cap C ie the intersection of sets is associative

SolutionNow B cup C = 0234689 and A cup B = 01234568

there4 A cup (B cup C) = 012345689 and

(A cup B) cup C = 012345689

So (i) A cup (B cup C) = (A cup B) cup C ie the union of sets is associative

Again B cap C = 06 and A cap B = 024

there4 A cap (B cap C) = 0 and (A cap B) cap C = 0

So (ii) A cap (B cap C) = (A cap B) cap C ie the intersection of sets is associative

Physics Chapter 2 Physical Quatites and Measurements

Here We Will Do Some QuestionsRelated To Chapter 2

A density bottle has a marking 25 mL on it It means that

1 the mass of density bottle is 25g

2 the density bottle will store 25 ml of any liquid in it

3 the density bottle will store 25 ml of water but more volume of liquid denser than water

4 the density bottle will store 25 ml of water but more volume of a liquid lighter than water

Solution 2 the density bottle will store 25 ml of any liquid in it

COMPUTER CHAPTER-2Spreadsheet Functions and Charts

SELECTING RANGE IN ROWSCOLUMNSWHEN TWO OR MORE CELLS ARE SELECTED IT IS CALLED A RANGEA RANGE OF CELLS CAN BE FORMED IN TWO WAYS--a) SELECTING RANGE BY USING THE MOUSEb) SELECTING RANGE BY USING THE KEYBOARD

Q1)WRITE THE STEPS TO SELECT PARTIAL RANGE IN A ROW

Ans)THE STEPS ARE-6 SELECT THE ROW7 BRING THE CELL POINTER TO THE DESIRED

LOCATION FROM WHERE YOU WANT TO START YOUR SELECTION

8 CLICK THE LEFT MOUSE BUTTON AND KEEP DRAGGING TO YOUR RIGHT TILL YOU REACH THE LAST CELL TO NE SELECTED

RELEASE THE MOUSE BUTTON

GEOGRAPHY Asia

CLIMATE

Asia experiences great extremes of climate Jacobabad in the Sind province of Pakistan is one of the hottest places in the WorldVerkhoyansk in Siberia is one of the coldest places in the WorldCherrapunji and Mawsynram in India are two wettest places in WorldArabia Tibet Gobi and Mongolia are extremely dry regionsFactors Affecting Climate of Asia-The factors influencing the climate of Asia are-

Factors Affecting Climate of Asia-Thoroughly read the table in page number 60

Latitudinal extent

Continentality

Relief features

Presence of low pressure trough

Jet streams

English Language The Sentence A complex sentence contains one independent clause and at least one dependent clause The dependent clause in a complex sentence is introduced with subordinating conjunctions or relative pronouns

Commonly Used Subordinating Conjunctions-Time after before while when since untilCause And Effect because now since as in order that soOpposition although though even though whereas while in spite ofCondition if unless only if whether or not even if in case(that)

Commonly Used Relative Pronouns-Who whose whom which whoever whomever whichever that

Class IXSubject Topic Summary Execution

1-BENGALI(2ND LANGUAGE)

ldquo বঙগভমিরপরমিrdquo াইকেলধসদনদতত

আচেগর পর উততর পচো-১ ২ ৩ এং নীচের পর টি াহির কা- ৪মহিbকাও গচেনা দেগা পহিচে অমত হরচে- ক) কার দো দেকান কহিতার অং( ) কতা দেক পরসঙগ কী উহিকতটির তাৎপযC আচোনা কচেরা৫দেসই ধনয নরকচে দোচেক যাচে নাহি ভচে মচেনর মহিeচের সাচেসচে সCন ক) কহির কায C ার উচেf(য হিক হিছ কহি কন কহিতাটি দেচেন) কহি কার কাচেছ হিমনহিত কচেরচেছনগ) কহি এই পহিথীচেত কাচের ধনয মচেন কচেরনঘ) কহি হিক রকম অমর তাাভ করচেত ান

Hindi 2nd lang

काकी(कतिसयारामशरण गपत)

इस कहानी म लखक न यह बतान का परयास निकया ह निक बचच अपनी मा स निकतना परम करत ह शयाम अबोध बालक ह वह अपनी मा क मरन क बा- उसन अपनी मा क कतिलए बहत रोया बा- म उस पता चला निक उसकी मा राम क घर चली गई ह आकाश म उडती हई पतग -खकर उस हरष हआ निक पतग क दवारा वह अपनी मा को नीच उतारगा इसक कतिलए वह अपनी निपता की जब स -ो बार सवा रपया निनकालकर पतग और -ो मोटी सी मन वाली अपन भाई स काकी एक कागज पर कतिलखवा कर पतग म कतिशव का दि-यानिनकालकर पतग और -ो मोटी सी मन वाली अपन भाई स काकी एक कागज पर कतिलखवा कर पतग म कतिचपका दि-याभोला और शयाम कोठरी म रससी बाधनी रह थ तभी उसक निपता करोध म आकर उन स पछ निक कया उनकी जब स रपया निनकाला हभोला डर क मार बताया निक शयाम इस पतग क दवारा अपनी काकी को राम क यहा स उतारना चाहता हनिवशशवर(शयाम क निपता)न फटी पतग उठाकर -खी तो उस पर काकी कतिलखा थावह हत बजिa होकर वही खड रह गएउनहोन सोचा निक मन अपन पतर को मारा जोनिक अनजान और निन-dरष थावह अपनी मा कोनिकतना पयार करता ह

उस दि-न बड सवर शयाम की नी- खली तो -खा निक घर भर म कोहराम मचा हआ ह

क) घर म कोहराम कयो मचा हआ था शयाम को कया लगा

ख) काकी को ल जात समय शयाम न कया उपदरव मचाया

ग) काकी क बार म उस कया बताया गया कया सतय उस कतिछपा रहा

घ) वह बठा-बठा शनय मन स आकाश की ओर कयोकरता

उततरक) शयाम की मा का -हात हो गया था इसकतिलए

घर म कोहराम मचा हआ था शयाम की लगा निक उसकी मा सफ- कपडा ओढ हए भमिम पर सो रही ह

ख) लोग जब उमा यानी शयाम की मा को उठाकर ल जान लग तब शयाम न बडा उपदरव मचाया लोगो क हाथ स झठ करवा उमा क ऊपर जा निगरा और बोला काकी सो रही ह उस कहा ल जा रह हो

ग) काकी क बार म बजिaमान लोगो न उस निवशवास दि-लाया निक उसकी का निक उसक मामा क यहा गई ह लनिकन सतय अमिधक दि-नो तक कतिछपाना रह सका आसपास क अबोध बालको क मह स यह बात परकट हो गई निक उसकी मा का -हात हो गया ह

घ) कई दि-नकई दि-न लगातार रोत-रोत उसका रोना तो शान हो गया पर उसक ह-य म शोक भर गया था वह चपचाप बठा आकाश की और टाका करता निक शाय- उसकी काकी कही दि-ख जाए

ldquoदि-न उसन ऊपर आसमान म पतग उडती -खी न जान कया सोच कर उसका निहर-य एक-म खिखल उठाrdquo

क) निकसन पतग ऊपर उडत -खी और वह कयो खश हआ

ख) उसन अपन निपता स कया कहा उनका कया उतर थाश

ग) उसन निफर कया निकया और निकसन उसकी सहायता की

घ) उसकी योजना कया थी उततर -क) शयाम न एक दि-न आसमान म पतग उडती

-खी तो उसन सोचा निक पता आसमान म राम क यहा जाकर रकगी वही पर मरी काकी ह यह सोचकर वह बहत खश हआ

ख) उसन अपन निपता स कहा काका मझ एक पतग मगा -ो उसक निपता न भटक हए मन क भाव स कहा निक मगा -ग यह कह कर उ-ास भाव स वह कही और चल गए पतग नही आई

ग) उसन चपचाप निवशशवर क टगहए कोट स एक चवननी निनकाल ली और सखिखया -ासी क लडक भोला की सहायता स एक पतग मगवानी भोला उसकी बराबर उमर का ही था

घ) उसकी योजना यह थी निक वह अपनी पतग को आकाश म राम क यहा भजगा और उस पतग क सहार उसकी काफी नीच उतर जाएगी इस योजना पर उस परा निवशवास था इसकतिलए वह और भोला -ोनो यह काम करन म लग गए

Continue to nexthelliphellipEVS CHAPTER - 1

(UNDERSTANDING OUR ENVIRONMENT)

Sustainable development

The development that meets the needs of the present without compromising the ability of future generations to meet their own needs is called Sustainable development

Sustainable societies ndash

An environmentally sustainable community is one that meets the current and future basic resource needs of its people in a just and equitable manner without compromising the ability of future generations to meet their basic needs

Q ) What are Eco Villages

Ans - Eco village are the urban or rural communities of people who strive to integrate a supportive social environment with a low impact way of life

Q ) To ensure sustainable development the depletion of renewable resources should not take place at a rate faster than their regeneration Justify your answer

Ans ndash Renewable resources do not have a fixed quantity - more can always be

generated However if the rate of use exceeds the rate of renewal - that is the

source is used more than its being recreated - its continued use will become

used up faster than it can regenerate

To promote sustainable society the following things need to be done ndash

1 Using renewable energy sources 2 By improving the quality of human

health 3 By promoting sustainable agriculture 4 By forming ecovillage

it will eventually be entirely depleted So Toensure sustainable development the depletion of

renewable resources should nottake place at a rate faster than their regeneration

Q ) What do you mean by Sustainable societies

Ans - Sustainable societies are defined as towns and cities that have taken steps to remain healthy over the long term These communities value healthy ecosystems use resources efficiently and actively seek to retain and enhance a locally based economy Sustainable development concerns everybody in a society

Q ) What are the effects of pollution on human health

Ans ndash Some health problem occurs due to air pollution are ndash

Respiratory diseases Cardiovascular damage Fatigue headaches and anxiety Irritation of the eyes nose and throat Damage to reproductive organs Harm to the liver spleen and blood Nervous system damage

Some health problem occurs due to water pollution are ndash

Typhoid Cholera Dysentry Jaundice

Some health problem occurs due to noise pollution are ndash

Fatigue headaches and anxiety High blood pressure Hearing damage

Physics Motion in 1D First go through previous notes Now here we will solve some numerical related to that

Question 3What information about the motion of a body is obtained from the displacement-time graphSolution 3From displacement-time graph the nature of motion (or state of rest) can be understood The slope of this graph gives the value of velocity of the body at any instant of time using which the velocity-time graph can also be drawn

Question 4(a)What does the slope of a displacement-time graph represent(b)Can displacement-time sketch be parallel to the displacement axis Give a reason to your answerSolution 4(a) Slope of a displacement-time graph represents velocity(b) The displacement-time graph can never be parallel to the displacement axis because such a line would mean that the distance covered by the body in a certain direction increases without any

increase in time which is not possible

Chemistry Language of Chemistry

How to balance a chemical equationThere are two methods of balancing an equation(i)Hit and trial method(ii)Partial equation methodBalancing by hit and trial methodThis method consists of counting the number of atoms of each elements on both sides and trying to equalize themTake the following steps(i)Count the number of times (frequency) an element occurs on either side(ii)The element with the least frequency of occurrence is balanced first(iii)When two or more elements have the same frequencythe metallic element is balanced firstExample-1 On heatinglead nitrate decomposes to give lead dioxidenitrogen dioxide and oxygenPb(NO3)2rarrPbO+NO2+O2

In this equationLead occurs twiceNitrogen occurs twiceOxygen occurs four timesSince lead is a metalbalance it firstThe number of atom of lead is equal on the two sidestherefore it needs no balancingNow balance nitrogenOn the reactant sidethere are two atoms of nitrogenwhile on the product side oneSomultiply the product containing nitrogenon the product sideby two Pb (NO3)2rarrPbO+2NO2+O2Nowthe number of oxygen atoms on the reactant side 6while on the product sideit is 7Somultiply the entire equation by 2except oxygen to get balanced equation2Pb(NO3)2rarr2PbO+4NO2+O2Multiplication by 2 is done only when atoms of all the elements except one element are balanced and the unbalanced atom occurs separately at least once and also there is a difference of only one such atom

Math Topic AlgebraChapter

Factorisation

Study item Difference of two squares a2 ndash b2 = (a+b) (a-b)1) (i) 4x2ndash 25y2

= (2x) 2 ndash (5y) 2= (2x + 5y) (2x - 5y)

(ii) 9x2 ndash 1= (3x)2ndash(1)2= (3x + 1)(3x ndash 1)

2) (i) 150 ndash 6a2= 6(25 ndash a2)= 6(5)2 ndash(a)2= 6 (5 + a) (5 ndash a)

(ii) 32x2 ndash 18y2=2(16x2 ndash 9y2)=2(4x)2 ndash (3y)2= 2(4x + 3y)(4x - 3y)3)(i) (x ndashy )2 ndash 9 = (x ndash y )2 ndash (3)2= (x ndash y + 3) (x ndash y ndash 3)(ii) 9(x + y) 2ndash x 2= (3)2(x + y)2 ndash (x)2=3(x + y)2 ndash (x)2= (3x +3y ) 2ndash(x)2= (3x + 3y + x)(3x +3y ndash x)= (4x + 3y) ( 2x + 3y )

Commercial studies

Basic accounting terms

Today I will give you some questions from the previous study material

Questions1) Define accounting2) What do you mean by debit and

credit

3) Explain the types of account4) Define the following terms

a) Assetsb) Capitalc) Purchased) Debtorse) Transactions

5) Name the types of accounts given below

a) Krishnas accountb) Machinery accountc) Royalty accountd) Salary accounte) Furniture accountf) Audit fee account

Economics Revision Today I will give you some revision questions

Questions1) What do you mean by the terms

rdquowantsrdquo2) Write the difference between

consumer goods and producer goods

3) Define the term utility 4) Explain the different types of utility5) Define

a) Total utilityb) Marginal utility

Subject Eng Literature (The Merchant of Venice ndash William Shakespeare)Topic Act I Scene 3 Lines 1 to 48 (Shylock hellip Cursed be my tribe if I forgive him) Date 16th April 2020 (5th Period)

[Students should read the original play and also the paraphrase given in the school prescribed textbook]Summary Questions amp Answers

This scene takes place in Venice and we are introduced to the rich Jew Shylock Bassanio and Shylock are talking and Bassanio tells Shylock that he wants a loan of three thousand ducats for three months on the personal security of Antonio

o Shylock feels glad because he will be able to bind down Antonio by means of a bond on account of the loan but he tells Bassanio that all the fortunes of Antonio being invested in the merchant ships on the sea it is difficult to depend upon his credit Even under such circumstances Shylock is willing to advance the money on the personal security of Antonio

o Bassanio then invites Shylock to dine with him Shylock says that he is prepared to do anything with the Christians but not eat or drink or pray with them

o While Bassanio and Shylock are talking Antonio appears on the scene Shylock does not seem to take any notice of Antonio but goes on brooding within

(1) SHYLOCK Ho no no no no- my meaning in (Line 15-26)saying he is a good man is to have you understand me that he is sufficient Yet his means are in suppositionhe hath an argosy bound to Tripolis another to the Indies I understand moreover upon the Rialto he hath a third at Mexico a fourth for England and other ventures he hath squanderd abroad Butships are but boards sailors but men there be land-rats and water-rats land-thieves and water-thieves I mean pirates and then there is the peril of waters winds and rocks The man is notwithstanding sufficientmdashthree thousand ducats mdashI think I may take his bond

(a) Who is talking in the beginning of this scene What does Bassanio want from Shylock How does Shylock feel

In the beginning of the scene Bassanio and Shylock are talking to each other Bassanio wants to borrow three thousand ducats from Shylock for three months on the security of Antonio Shylock feels glad at heart that he will get the opportunity of binding Antonio with a bond(b) What risks does Shylock weigh in advancing the money

Shylock says that Antonio has invested all his capital in trading by sea-going ships But the ships are made of wood and the sailors of those ships are ordinary human beings The wood can

himself how he hates Antonio because of his being a Christian because he abuses Shylock in public places Shylock decides that if ever he can get Antonio to his advantage he will teach him a lesson

come to harm and men can commit mistakes and thus the capital invested in ships may be lost Then there are other dangers The goods loaded on the ships can be damaged by rats and thieves which are found both on land and water The ships can also be harmed through sea-storms submerged rocks etc(c) What two important functions does this scene have

The scene has two important functions First it completes the exposition of the two major plot lines of the play Antonio agrees to Shylockrsquos bond ndash three thousand ducats for a pound of flesh and second and more important dramatically this scene introduces Shylock himself In this scene Shakespeare makes it clear at once why Shylock is the most powerful dramatic figure in the play and why so many great actors have regarded this part as one of the most rewarding roles in all Shakespearean dramas(d) Where does this scene take place What kind of treatment has Antonio been giving to Shylock What does Shylock say when Bassanio invites him to dine with him

The action of this scene takes place in Venice Antonio has been in the habit of behaving harshly with Shylock ndash spitting on his beard and footing him like a stranger cur When Bassanio invites Shylock to dine Shylock says that he is prepared to do anything with the Christians but not eat and drink or pray with them

(2) SHYLOCK How like a fawning publican he looks (Line 38-48)I hate him for he is a Christian

But more for that in low simplicity

He lends out money gratis and brings downThe rate of usance here with us in VeniceIf I can catch him once upon the hipI will feed fat the ancient grudge I bear him

He hates our sacred nation and he railsEven there where merchants most do congregateOn me my bargains and my well-won thriftWhich he calls interest Cursed be my tribeIf I forgive him

(a) What is the context in which these words are spoken and what is the idea expressed in it

These remarks are made by Shylock when he sees Antonio coming along after Bassanio told him that the merchant will be his surety for the bond The above mentioned passage reveals Shylockrsquos hatred for Antonio Shylock says that he hates Antonio because he is a Christian and also because he gives loan without taking interest on them thereby bringing down the rate of interest in Venice(b) Explain the meaning of the phrase lsquoa fawning publicanrsquo

The phrase lsquoa fawning publicanrsquo refers to Roman tax collector It is a term of contempt and hatred on the lips of a Jew lsquoFawning Publicansrsquo were Roman tax-gatherers whose ordinary bearings towards the Jews was bullying but whose false pose of lsquohumility and contritionrsquo is touched upon in the parable in New Testament(c ) What light does the above passage throw on the character

of Shylock

The above mentioned speech of Shylock reveals him to be a wicked character having an extreme greed for wealth His intense hatred for Antonio is unjustified He hates Antonio only because he is a Christian and because he lends money without taking any interest on it thereby adversely affecting Shylockrsquos business of lending money on high interest(d) What information do you gather about Antonio from the above given lines

Shylockrsquos statement throws a valuable light on the character of Antonio Antonio appears to be a good Christian and a good human being He helps the people in need by lending them money without charging any interest on it He is a man of simple and good nature This very goodness makes him Shylockrsquos enemy(e) What does Shylock debate within himself and when To whom are the lines mentioned above addressed to

When Bassanio asks the Jew to lend him three thousand ducats on Antoniorsquos surety Shylock begins to debate within himself as to how he should exploit the opportunity of a business deal with his old enemy Antonio

The lines mentioned above are not addressed to anyone The lines are a soliloquy ie a speech made by a character to himself and not meant to be heard by the other characters present

Class XSubject Topic Summary ExecutionEnglish

LiteratureThe Blue Bead 2nd part

Things took a turn for the worst and all of a sudden a crocodile attacked the woman biting on the womanrsquos leg At that moment Sibia got up sprinted grabbed the hay fork and stabbed the crocodile in the eye with all her power Immediately the crocodile let go and went away Sibia saw a small blue bead lying by the river she grabbed it Since she was poor she didnrsquot have necklace Shersquod always wanted one like the other women now she could make one with the blue bead After that she went home and told her mother all about it

Hindi 2nd

Langबड घर की बटी( मशी परमच-)

lsquoबड घर की बटी कहानी का उददशय मधयम वग की घरल समसया को सलझा कर सगदिठत परिरवार म मिमल जलकर परम स रहन का स-श -ना हघर म शानित सथानिपत करन की जिजमम-ारी नारी की होती ह यदि- नारी समझ-ार ह उसम धय और परिरवार क परनित परम ह तो कोई भी घटना परिरवार को निवघदिटत नही कर सकती या कहानी परिरवार को सगदिठत करत हए परम सौहा- स एक दसर की भावनाओ को समझ करउनका सहयोग करत हए जीवन यापन करन की पररणा -ती हमशी परमचदर जी न इस कहानी म सय परिरवार का परनितनिनमिधतव निकया ह यह कहानी बनी माधव सिसह जो गौरी पर क जमी-ार क उनक -ो पतरो की हशरी कठ लाल निबहारीशरीकात का निववाह एकजमी-ार घरान की पतरी आन-ी स हआ थाआन-ी न ख- को ससराल क वातावरण म ढाकतिलया थाएक दि-न आन-ी का अपन -वर लाल निबहारी स झगडा हो जाता ह -ोनो भाई एक दसर स अलग होन की कोकतिशश करत हसभी बह आन-ी न अपन मधर वयवहार स लाल निबहारी को

ldquoइन नतर निपरय गणो को बीए-इनही -ो अकषर पर नयोछावर कर दि-या था इन -ो अकषर न उनक शरीर को निनबल और चहर को कानित ही बना दि-या थाldquo

क) परसतत पकतियो म निकस वयकति क बार म कहा गया ह

ख) इन पकतियो म कौन स नतर निपरय गणो क बार म कहा गया ह

ग) बीए की निडगरी परापत कर लन पर भी उपय वयकति क सवभाव की कया निवशरषता थी

घ) यह नतर निपरय गण निकस वयकति म निवदयमान थ उसक वयकतितव की कया निवशरषता थी

उततर ndashक) परसतत पकति म गौरी पर गाव क जमी-ार

क बड बट शरीकात क बार म कहा गया ह उसन बहत परिरशरम और उ-म क बा- ba की निडगरी परापत की थी अब वह एक -फतर

घर छोडकर जान स रोक कतिलयाइस पर बनी माधव सिसह न कहा निक बड घर की बटी ऐसी ही होती ह जो निबगडा काम बना लती ह अतः शीरषक साथक ह बड घर की बटी आन-ी ह

म कमचारी थाख) भरा हआ चहरा चौडी छाती और डटकर

खाना आदि- एक सबजी ल जवान क गण मान जात ह परत शरीकात न इनही नतर निपरय गणो को अपनी पढाई पर नयोछावर कर दि-या था

ग) बीए की निडगरी परापत कर लन पर भी उपय वयकति(शरी कठ की शारिररिरक तौर पर निनबल और चहर स कानित ही लगत थ इतना ही नही वह मानकतिसक तौर पर भी निपछड हए थ पाशचातय सामाजिजक कथा उस घणा एव पराचीन सभयता का गणगान उनकी निवचारधारा क परमख अग थ

घ) यह नतर निपरय गण गौरीपर गाव क जमी-ार क छोट बट लाल निबहारी सिसह म निवदयमान थ वह सजीलाजवान था और भस का दध शर दध वह सवर उठकर पी जाता था

ldquoयही कारण था निक गाव की लललन आए उनकी निन-क थी कोई कोई तो उह अपना शतर समझन म भी सकोच ना करती थी सवय उनकी पतनी को इस निवरषय म उनस निवरोध थाldquo

क) उपय पकति म इस वयकति क बार म कहा गया ह

ख) गाव की लललन आए उनकी निन-ा कयो निकया करती थी

ग) उनकी पतनी का कया नाम था उनह निकस निवरषय म अपन पनित क निवरa था और कयो

घ) इस कहानी का कया उददशय ह Continue to next helliphelliphellip

Bengali 2nd Language

ফ ফটক না ফটক( কহিতা )

পর) ldquo(ান াধাচেনা ফটপাচেথ পাথচের পাড হিচেয় এক কাঠচোটটা গাছ কহিকহি পাতায় পার ফাটিচেয় াসচেছldquoক) কার দো দেকান কহিতার অং( ) lsquo(ান াধাচেনা ফটপাচেথ পাথচের পাডহিচেয়lsquo চেত কী দোঝাচেনা চেয়চেছ গ) আচো য অংচে(lsquo এক কাঠচোটটা গাছ lsquoচেত কী দোঝাচেনা চেয়চেছ ঘ) ldquoকহিকহি পাতায় পার ফাটিচেয় াসচেছldquo ----- একথার পরকত অথC কী উততর ) ক) আচো য অং(টি পর যাত কহি সভা4 মচোপাধ যাচেয়র দো lsquoফ ফটক না ফটকrsquo কহিতার অং()কহি সভা4 মচোপাধ যায় হিছচেন দেপরচেমর কহি দেপরমচেক নানা ভহিঙগমায় হিতহিন ফটিচেয় তচেচেছন দেপরম মানচের স মচেতC র সঙগী কহিতার কহিতায় এক রb সb হচেয়র দেপরম াগরচেনর কথা চেচেছন (ান অথCাৎ দেযাচেন দেকান রস দেনই দেযাচেন দেকান মহিনতা দেনই অথ তার মধ দেযও দেপরম থাকচেত পাচের একথাই কহি তচে ধরচেত দেচেয়চেছন একটি মানচে4র মচেন দেযাচেন দেকামতার দেকান সথান দেনই পাথচেরর মচেতা হিনরসতার মচেনর মধ দেযও দেয দেপরম আসচেত পাচের দেস কথাই কহি চেচেছনগ)নারীচের যথC দেপরচেমর ছহি এই কহিতায় অকপচেট উচেঠ এচেসচেছ কহি এই কহিতায় কাটচোটটা গাছ কথাটি যার কচেরচেছন নারী দেয দেপরম দেথচেক হিতাহিত এং দেসই দেপরম সঠিক সমচেয় না পাওয়ার ন য দেপরম সমপচেকC হিচেr4 গৈতরী য় দেপরচেমর দেয গৈহি(ষট য মাধযC য সরসতা দেকামত এই সমসতর হিপরীত যথা রbতা শষকতা কচেঠার তা পরভহিত দোঝাচেত এক কাঠচোটটা গাছ কথাটি যার কচেরচেছনঘ) এাচেন এক নারীর যথC দেপরচেমর কথা হিনহিCপত ভাচে চেচেছন কহি অসমচেয় নারীর ীচেন দেপরম দেচেগচেছ এতহিন তার হয় রb কচেঠার হিছ দেপরচেমর অভাচে ঠাৎ দেসই শষক মরভহিমচেত সচের আভাস এচেসচেছ দেপরম দেযন 4Cার স(ীত তাই পরায় মত গাচেছ কহিকহি পাতা গহিচেয় উচেঠচেছ

Biology Chapter - 01Controlling Air Pollution

Today we will discuss how we control air pollution from domestic combustion

Q1Describe any five ways of reducing air pollution from domestic sources bull The number of pollutants in the air is verylarge and we always try to control them byfollowing ways

i) Solar cooker and solar heater It use no fuel reduce damage of environment by fuel use or reducing deforestation It maintains coolness of house It releases very less orno oil gas or grease

ii) Piped natural gas (PNG) It emits very less by products into the atmosphere As it isdistributed through pipe lines so there iscontinuous supply of fuel is possible

iii) Liquefied Petroleum Gas (LPG) It hasa higher heating value LPG doesntcontain sulphur so it burns a lot cleanerenergy sources It releases very less oralmost no fume in air

iv) Electricity based cooking Emission free cooking alternative for urban dwellers causeselimination of adverse health impactsofindoor air pollution It helps to avoid theinconveniences associated with procurement of LPG

v) Biogas It contains 75 methane whichmakes it an excellent fuel It burns without smoke and biogas plant leaves no residue like ash in wood charcoal etc Thus it isaclean fuel

Economics

Factors of Production

Today firstly we would recall the last class for 5 mins and then we would proceed with the further topics of the chapter

The concept meaning of land characteristics of land and importance of land to be repeated for the absentees as well as the students who were there in the class the previous day

Today we will start with the last portion of land before it the meaning of land to be repeated onceAs by now we all know that

Questions1What do you mean by productivity of landAnswer By productivity of land we mean the capacity of a piece of land to produce a crop

Thus it refers to the average output per unit of landSay per acre per hectare etc= (OutputArea of land)

2 What are the factors influencing the productivity of landAnswer

Natural factors Productivity of land is largely determined by the natural

Land is defined to include not only the surface of the earth but also all other free gifts of nature(for example mineral resources forest resources and indeed anything that helps us to carry out the production of goods and services but is provided by nature free of cost)

We will move on to the last portion of land by discussing Productivity of Land

By productivity of land we mean the capacity of a piece of land to produce a crop

Thus it refers to the average output per unit of land

Say per acre per hectare etc= (OutputArea of land)

With this we shall proceed further with the main factors that determine the productivity of land

Natural factors Human factors Improvements on land Location of land Organisation Ownership of land Availability of capital Proper use of land State help

Note economic development of a country depends upon the quality of its land If the land is fertile it will quicken the pace of development of the country

qualities of land such as fertility etc

Human factors Land cannot produce anything by itself Man has to apply labour on it to produce for himself So productivity of land depends on the knowledge and skills of workers

Improvements on land production of land is affected by land development measures like provision of well or tubewell irrigation proper drainage

State help The government of a country especially less developed country can play a vital role in improving the agricultural productivity by providing better irrigation facilities

Organisation Productivity of land also fdepends upon the way how the factors of production like labour and capital are organised

In order to increase productivity trained workers modern implements scientific methods good seeds are all essential

3 lsquoImproved technology affects the productivity of landrsquo Explain this statement with the help of suitable example Answer Use of improved technology raises the productivity of land Example By using HYV seeds chemical manures and modern machines per hectare output increases

Physics Force (Summary)

Question Write the expression for the moment of force about a given axisSolutionsThe expression for the moment of force is given byMoment of force about a given axis = Force times perpendicular distance of force from the axis of rotationQuestion What do you understand by the clockwise and anticlockwise moment of force When is it taken positiveSolutionsIf the effect on the body is to turn it anticlockwise moment of force is called the anticlockwise moment and it is taken as positive while if the effect on the

body is to turn it clockwise moment of force is called the clockwise moment and it is taken as negative

Math Topic Commercial Mathematics

Chapter Goods and services Tax

Study item Some solved sums from exercise ndash 1 A retailer buys a TV from a wholesaler for Rs 40000 He marks the price of the TV 15 above his cost price sells it to the consumer at 5 discount on the marked price If the sales are intra ndash state and the rate of GST is 12 find

(i) The marked price of the TV(ii) The amount which the consumer pays for the TV(iii) The amount of tax (under GST) paid by the retailer to the central

Government(iv) The amount of tax (under GST) received by the State Government

Solution As the sales are intra- state sale and the rate of GST 12 So GST comprises of 6 CGST and 6 SGSTTherefore a retailer buys a TV from a wholesaler for Rs 40000Therefore the amount of GST collected wholesaler from the retailer or paid by retailer to wholesalerCGST = 6 of Rs 40000 = Rs(6100 times40000) =Rs 2400SGST = 6 of Rs 40000 = Rs (6100 times 40000) =Rs 2400Therefore wholesaler will pay Rs 2400 as CGST and Rs 2400 as SGSTTherefore amount of input GST of retailer Input CGST = Rs 2400 and input SGST = Rs 2400Again the retailer marks the price of the TV 15 above his cost price(i) The marked price of the TV

= Rs 40000 + Rs 40000times15= Rs 40000 + Rs 40000times 15100= Rs 40000 + Rs 6000Rs 46000But the retailer sells it to consumer at 5 discount on the marked priceCost price after discount = Rs 46000 ndashRs46000times 5100 =Rs 46000 ndashRs 2300= Rs 43700Therefore the amount of GST collected retailer from consumer or paid by consumer to retailerCGST = 6 of Rs 43700 =Rs ( 6100 times43700)Rs 2622SGST = 6 of Rs 43700 = Rs (6100 times 43700) =Rs 2622Amount of the output GST of retailer Output CGST = Rs 2622 and output SGST = Rs 2622

(ii) The amount which the consumer pays for the TV= cost price of TV to consumer + CGST paid by consumer + SGST paid by consumer= Rs 43700 + Rs 2622 + Rs 2622= Rs 48944

(iii) The amount of tax (under GST ) paid by the retailer to the central Government=CGST paid by retailer = output CGST ndash input CGST=Rs 2622 ndash Rs 2400=Rs 222

(iv) The amount of tax ( under GST ) received by the State Government = SGST paid by wholesaler + SGST paid by retailer= Rs 2400 + output SGST ndash input SGST=Rs 2400 + Rs 2622 ndash Rs 2400=Rs 2400 + Rs 222= Rs 2622

Commercial studies

Stakeholders Today I am going to give some revision questions from the previous study material

Questions1) State the two expectations of

employees from a business concern2) Give two distinctions between

stakeholder and shareholder3) Give two difference between

internal stakeholders and external stakeholders

4) Give two expectations of suppliers from a business organisation

5) Who is a stakeholder in commercial organisations

Chemistry Periodic Table

Merits of Mendeleevrsquos Periodic law are as follows - 1He grouped the elements on the basis of atomic mass 2 He left gaps for undiscovered elements like Gallium Scandium germanium Also he left a full group vacant for undiscovered inert gases 3 He could predict proportions of several elements on basis of their position in periodic table like Ga Sc etc 4He could predict errors in atomic weights of some elements like gold platinum etc

Anomalies in Mendeleevrsquos Periodic law are as follows - 1 Position of isotopes could not be explained 2 Wrong order of atomic masses could not be explained

For example- as Arnur atomic mass 40 come first and K with low atomic mass (30) should come later but k should be placed first

According to Bohrrsquos Modern Periodic table properties of elements are periodic functions of their atomic numbers

So when elements are arranged according to increasing atomic numbers there is periodicity in electronic configuration that leads to periodicity in their chemical properties

It consists of horizontal rows (Periods) Vertical column (Groups)

There are 7 period and 12 groups in this long form of periodic table

Ist period has 2 elements IInd period has 8 elements IIIrd period has 8 elements IVth period has 18 elements Vth period has 18 elements VIth period has 32 elements VIIth period hs rest of elements

Note - The number of valence electrons in atom of elements decides which elements will be first in period and which will be last

In group- 1 to 2 gp and 13 to 17 contain normal elements 3 to 12gp ndash transition elements 57 to 71 - lanthanides 89 to 103 - Actinides

Left hand side ndash metals Right hand side ndash nonmetals

Note- Hydrogen element has been placed at top of Ist group Electronic configuration of H is similar to alkali metal as both have 1 valence electron

V electron of gp I element -- 1 V electron of gp 2 element -- 2 V electron of gp 13 element -- 3 V electron of gp 14 element -- 4 V electron of gp 15 element -- 5 V electron of gp 16 element --6 V electron of gp 17 element -- 7 V electron of gp 18 element -- 8

English 1 Transformation of sentences

Sentences A sentence is a group of words which makes complete sense

Exercise 2Change the following sentences from

a Assertive sentencesb Imperative sentencesc Interrogative sentencesd Exclamatory sentences

Sentences can be changed from one grammatical form to another without changing the meaning of the sentence This is known as transformation of sentences

assertive to interrogative1 Nobody would like to be a fool

Who would like to be a fool2 Their glory can never fade

When can the glory fade3 Nobody can control the wind

Who can control the wind4 It matters little if I die

What though I die5 No man can serve two masters

Can any man serve two masters

Exercise 3Interchange of assertive and Exclamatory sentences

1 She leads the most unhappy lifeWhat an unhappy life she leads

2 This is indeed an interesting bookWhat an interesting book this

3 He is a very great manWhat a great man he is

4 It is a very lame excuseWhat a lame excuse

5 It is sad that she died so youngAlas she died so young

Class XISubject Topic Summary Execution

Hindi 2nd lang

पतर परम(परमचदर) पतर परम कहानी म एक निपता की इचछाओ का वणन निकया गया ह अपन बड पतर परभ -ास स निपता चतनय -ास का निवशरष परम था निपता को उसक जनम स ही बडी-बडी आशाए थी उसम दसर बट कतिशव-ास की अपकषा स- उतसाह की मातरा अमिधक थी वह उस इगलड भजकर बरिरसटर बनाना चाहत थभागय का खल भी बडा निनराला ह बीए की परीकषा क बा- वह बीमार पड गया डॉकटरो न भी जवाब - दि-या थाचतन -ास जी बहत ही कजस थ बवजह पस खच करना नही चाहत थ अगर गारटी मिमलती तो शाय- पस खच भी कर -त परत गारटी नही थी परिरणाम सवरप उनक बट का -हात हो गयाजब बट को समशान ल जा रह थ तो वहा काफी शोर गान बजान हो रह थ पछन पर पता चला निक निकसी निपता निपछल तीन साल स निबमार था और उसक ईलाज म रपया पानी की तरह बहाया पर ठीक नही हए परत उसक बट को तनिनक भी अफसोस नही था उसका कहना था उसन कोकतिशश तो कीयह -खकर चतनय-ास जी को आतम निगलानी हईतभी स उनका म परिरवतन हआ और बट का भोज काफी धमधाम स निकयाऔर वहइस पशचाताप की आग म जलत रह औला- स बढकर पसा नही होता ह इस बात को समझन म उनह काफी व लग गया

hellipContinue to next

BENGALI(2ND LANGUAGE)

পরথমঅধযায়-ঠাকরারীনদরনাথঠাকর

নয়ন দোচের হিমাচেররা া নাচেমই হিযাত হিছচেন ায়ানার উাররণ সবরপ নয়ন দোচের ারা হিা (াচেকর হিা হিচেতন এছাাও দেকান উৎস উপচেb রাহিতর দেক হিন করার উচেfচে(য তারা সযC হিকরচেরণ রনয পরীপ জবাহিচেয় তাচেত রপার হির 4Cরণ করচেতন ঠাকরা এই নয়ন দো হিমারচের দে(4 ং(ধর হিছচেন হিমাররা ায়ানার ষটানত পর(Cন কচের তারা হিনঃসব এই হিমাহিরর দে(4 ং(ধর গৈকাস নদর রায়চেৌধরী গৈকাস া নয়ন দোচের সমসত সমপহিতত ঋচেরণর াচেয় হিহিx কচের অহি(ষট যা আচেছ তাচেত হিপত

ইার হিপতার মতয ইচে পর নয়নচোচের ায়ানার দেগাটা কতক অসাধাররণ শরাদধ (াহিনতচেত অহিনতম ীহিপত পরকা( কহিরয়া ঠাৎ হিনহিয়া দেগ- ক) কার দো দেকান গচেলপর অং() কতা দেক ইার চেত কাচেক দোঝাচেনা চেয়চেছ গ) পরসঙগ কী কতার কতয পহিরসফট কচেরা

পরচে4র যাহিত রbা করা সমভ নয় তাই হিতহিন পতরচেক হিনচেয় ককাতায় সাস শর কচেরন গলপ কথচেকর আহিথCক অসথা নয়ন দোচের হিমাচের দেথচেক সমপরণC আাা কথচেকর হিপতা হিনচের দেষটায় অথC উপাCন করচেতন া উপাহিধ াচেভর নয তার াসা হিছনা আর দেসই কারচেরণ কথক তার একমাতর উততরাহিধকার চেয় তার হিপতার পরহিত কতজঞ কথক দো পা হি(চেচেছন হিনচের পরারণ ও মান রbার নয উপচেযাগী অথC হিনা দেষটায় পরাপত চেয়চেছন- এটাই তার কাচেছ পরম দেগৌরচের হি4য় চে মচেন কচেরন কাররণ (নয ভাণডাচের গৈপতক ায়ানার উজজব ইহিতাস অচেপbা দোার হিসeচেকর মচেধয গৈপতক দেকামপাহিনর কাগ তার কাচেছ অচেনক দেহি( মযান

TO BE CONTINUED

উ- ক) আচোয অং(টি রীনদরনাথ ঠাকচেরর দো ঠাকরা গচেলপর অং() কতা চেন আচোয গচেলপর গলপ কথকইার চেত নয়ন দোচের হিমাহিরর দে(4 ং(ধর গৈকাস ার কথা া চেয়চেছ গৈকাস া নয়ন দোচের সমসত সমপহিতত ঋচেরণর াচেয় হিহিx কচের অহি(ষট যা আচেছ তাচেত হিপত পরচে4র যাহিত রbা করা সমভ নয় তাই হিতহিন পতরচেক হিনচেয় ককাতায় সাস শর কচেরনগ) গৈকাস ার হিপতার মতযর পর নয়ন দোচের হিমাহিরর অহিসততব হিপত য় কচেয়কটা উৎস ও শরাদধ- (াহিনতচেত হিমাহিরর দে(4 কহিটক যয় চেয় হিগচেয় এচেক াচের দে(4 চেয় যায় তন তাচের গC করার মত আর হিকছই হিছ না-দেসই পরসচেঙগ এই উহিকত নয়নচোচের হিমাচেররা া নাচেমই হিযাত হিছচেন ায়ানার উাররণ সবরপ নয়নচোচের ারা হিা (াচেকর হিা হিচেতন এছাাও দেকান উৎস উপচেb রাহিতরচেক হিন করচেত হিগচেয় তারা সযC হিকরচেরণর নয পরীপ জবাহিচেয় তাচেত রপার হির 4Cরণ করচেতন তাই দেসকাচের ায়ানা দেহি(হিন সথায়ী চেত পারত না হিহিভনন উৎস শরাদধ- (াহিনতচেত সাধযা হিতহিরকত র করার নয হিমাহির হিহিকচেয় দেযত হ হিতC কা হিহি(ষট পরীচেপর দেত দেযমন অলপকাচের মচেধয হিনঃচে(4 চেয় যায়-নয়নচোচের হিমারচের অসথা তাই চেয়হিছ এই কারচেরণই কথক নয়নচোচের হিমারচের গা ভরা আমবর সয করচেত পারতনা

Physics Dimensional Analysis (Summary)

Q Find the dimensions of consts ab in relation

p=(bminusxlowastx)at

where p is the power x is the distance and t is time

Ans From principle of homogeneity dimension of b x2 are same Dim of b = dim of x2 = [L2] = [ML2T0]Dim of a = dim of ( b- x2)dim of (pt) = [M0L2T0][ML2T-2] [T-1] [T] = [M-1L0T2]

Chemistry Atomic Structure Drawbacks of Rutherfordrsquos model of

atom a According to Rutherfordrsquos model of atom electrons which are negativelycharged particles revolve around the nucleus in fixed orbits Thusb theelectrons undergo acceleration According to electromagnetic theory of Maxwell a charged particle undergoing acceleration should emitelectromagnetic radiation Thus an electron in an orbit should emitradiation Thus the orbit should shrink But this does not happenc The model does not give any information about how electrons aredistributed around nucleus and what are energies of these electrons Isotopes These are the atoms of the same

Properties of electromagnetic radiationsa Oscillating electric and magnetic field are produced by oscillating charged particles These fields are perpendicular to each other and both areperpendicular to the direction of propagation of the waveb They do not need a medium to travel That means they can even travel invacuum

Characteristics of electromagnetic radiationsa Wavelength It may be defined as the distance between two neighbouring crests or troughs of

element having the same atomicnumber but different mass numbere g 1H11H21H3

Isobars Isobars are the atoms of different elements having the same massnumber but different atomic numbere g 18Ar40 20Ca40

Isoelectronic species These are those species which have the same numberof electrons

Electromagnetic radiationsThe radiations which are associated withelectrical and magnetic fields are called electromagnetic radiations When anelectrically charged particle moves under acceleration alternating electricaland magnetic fields are produced and transmitted These fields aretransmitted in the form of waves These waves are called electromagneticwaves or electromagnetic radiations

wave as shown It is denoted by λb Frequency (ν) It may be defined as the number of waves which passthrough a particular point in one secondc Velocity (v) It is defined as the distance travelled by a wave in onesecond In vacuum all types of electromagnetic radiations travel with thesame velocity Its value is 3 times10 8m sec-1 It is denoted by v

d Wave number Wave number is defined as the number of wavelengths per unit lengthVelocity = frequency timeswavelength c = νλ

Plancks Quantum Theory- o The radiant energy is emitted or absorbed not continuously but discontinuously in the form of small discrete packets of energy called lsquoquantumrsquo In case of light the quantum of energy is called a lsquophotonrsquoo The energy of each quantum is directly proportional to the frequency of the radiation ie E α υ or E= hυ where h= Planckrsquos constant = 6626 x 10-27 Js o Energy is always emitted or absorbed as integral multiple of this uantum E=nhυ Where n=1234Black body An ideal body which emits and absorbs all frequencies is calleda black body The radiation emitted by such a body is called black body radiation

Photoelectric effectThe phenomenon of ejection of electrons from thesurface of metal when light of suitable frequency strikes it is calledphotoelectric effect The ejected electrons are called photoelectrons

Biology Chapter - 02Systematics and Five Kingdoms

Scientists divide the whole living organisms into two kingdom first and ultimately by five kingdom at last

In the earlier systems of classifications organisms are divided into kingdom plantaeand kingdom animalia on the of presenceof cell wall their modes of nutrition and movements

Some problem arise like fungi share manycharacteristic withplant despite their heterotrophic nutrition bacteria protozoa areunicellular present in both kingdom Toovercome this third kingdom Protista isintroduced which include

unicellularorganisms But there is also another

problem Allunicellular organisms are not similar kind The cellular structure of prokaryotes is verydifferent from that of other organismsEukaryotes possess a true nucleus and allcell organelles that are not present inprokaryotes So the fourth kingdom Monerais introduced which include unicellular prokaryotes (bacteriaamp blue green algae)

bull Still some problem arise in kingdomplantae

So in 1969 R H Whittakar proposedanew five kingdom System of classification

i) Kingdom Monera - unicellular prokaryotes

ii) kingdom Protista - unicellular eukaryotes

iii) Kingdom Fungi - uni or multicellular fungi with cell wall but without chlorophyll

iv) Kingdom Plantae - Multicellular Plants

v) Kingdom Animalia - Multicellular Animals

EVS Chapter 1 ndash Modes of Existence

An agricultural society

An agricultural society also known as an agrarian society is a society that constructs social order around a reliance upon farming More than half the people living in that society make their living by farming

People in an agricultural society generally lead a more settled lifestyle than those in nomadic hunter-gatherer or semi-nomadic pastoral societies because they live permanently near the land that is farmed Agricultural settlements tend to develop in areas of convenience near bodies of water which is used for both crops and transportation or along trade routes Not everyone in an agricultural society is a farmer Some people make a living trading or making and selling goods such as tools used for farming

Another way to define an agrarian society is to see the total amount of production in a nation In an agrarian society cultivating the land is the main source of wealth Such a society can recognize other means of subsistence and work habits but emphasizes the importance of agriculture and livestock Agrarian societies have existed in various parts of the world for 10000 years and continue to exist today They have been the most common form of socio-economic organization for most of recorded human history

Q) Write the features of agricultural society

Ans - Structure and Features of Agrarian Society1 Occupational Structure

An agrarian society is generally associated with the domestication of plants and animals The domestication of plants means farming and that of animals means herding Often there is mixture of farming and the use of such domesticated animals as cow goat and sheep

2 Forms of Land Ownership in Agrarian SocietiesGenerally there are landlords supervisory farmers cultivators and share croppers The landholders own the land but do not work on it They let it out for sharecropping The supervisory farmers are those who live by having their land cultivated by hired labourers The cultivators cultivate the land for themselvesThe share-croppers are those who live by tilling other peoplersquos land or a crop-sharing basis The artisans own their means of production and produce by their own labour in their homesteads

3 Village Community System An agrarian society is highlighted by

the institution of village community system The agrarian economy made fixed dwelling houses necessary Living close together for protection and co-operation and living nearer to the land gave birth to agricultural villages The village is not only the residential place of farmers it is also the social integrator

4 Minimal Division of Labour Another structural feature of agrarian society is a minimal division of labour Except for the basic division founded on age and sex differences there are few specialized roles There is only one predominant type of occupation ie domestication of plants and animals For all the people the environment physical as well as social is the same

5 Role of Family The farm family is of the patriarchal type the father is the final arbiter in most of the familyrsquos major decisions The life of ail men and women is merged in family life Since there are not many special organizations family is the only organisation to perform the tasks of aid and protection

6 Sense of Unity The members of an agrarian society exhibit a strong in-group feeling Since the whole of their social lives is wrapped up in a society which is physically economically and socially homogenous they are inclined to view the entire outside world as an out group

7 Informal Social Control An agrarian society is regionally divided into villages In a village community the force of traditional mores is more dominant than in the urban community In the village everybody is known to everybody The members in a village community help each other and share the joy and sorrows of each other Crime in an agrarian society is rare

8 Simplicity and Uniformity Life of the people in an agrarian society is marked by simplicity and uniformity Their main occupation is agriculture which largely depends upon the vagaries of nature An agrarian society is a religious society

Math Compound angles Compound angles The algebraic sum of two or more angles is called a compound angle If A B C be three angles then A+B B+C C+A A-B B-C A-C A+B-C etc are compound angles In this chapter we shall discuss the trigonometrical ratios of compound angles Theorem 1 If A B and A+B are all pisitive acute angles theni) sin( A+B) = sin A cos B + cosA sinBii) cos(A+B) = cosA cosB- sinA sinBTheorem 2If A and B are positive acute angles and AgtB theni) sin(A-B) = sin A cosB- cos A sinBii) cos(A-B) = cos A cos B+ sin A sin BTo prove that i) sin(A+B) sin (A-B) = sin2 A - sin2 B = cos2 B- cos2 A

Example 1 Prove that tan70deg=2tan50deg+tan20degSolutiontan70deg = tan(50deg + 20deg)Or tan70deg=(tan 50deg+tan 20deg)(1-tan50degtan20deg) or tan70deg (1 ndash tan 50deg tan20deg) = tan50deg+tan20degor tan70deg= tan70deg tan50deg tan20deg+ tan50deg + tan20deg = cot20deg tan50deg tan20deg + tan50deg + tan20deg = 2 tan50deg+ tan20degExample 2 If A + B = 45deg show that (1 + tanA) (1 + tanB) = 2Solutiontan(A + B) =( tan A + tan B) (1 - tan

ii) cos(A+B) Cos(A-B) = cos2 A- sin2 B = cos2 B -sin2 AProof i) LHS= sin(A+B)sin(AminusB) [Recall sin(αminusβ)=sinαcosβminuscosαsinβ And sin(α+β)=sinαcosβ+cosαsinβ]= (sinAcosB+cosAsinB)times(sinAcosBminuscosAsinB)= sin2Acos2Bminuscos2Asin2B [Recall sin2α+cos2α=1 From above we can then assume correctly that sin2α=1minuscos2α AND cos2α=1minussin2α] = sin2A(1minussin2B)minussin2B(1minussin2A) = sin2Aminussin2Asin2Bminussin2B+sin2Asin2B = sin2Aminussin2B= 1-cos2A-(1-cos2B) = cos2 B- cos2 A = RHSii)LHS= cos (A+B) cos(A-B) [ cos(A+B) = cos AcosB- sinAsinBCos(A-B) = cosAcosB+ sinAsinB]= cos2 A Cos2 B- sin2 A Sin2 B= cos2 A( 1-sin2 B) - (1- cos2 A) sin2 B= cos2 A- cos2 A sin2 B- sin2 B+ cos2 A sin2 B=cos2 A- sin2 B=1- sin2 A-(1-cos2 B) = cos2 B- sin2 A= RHSTangent formulae for compound anglesi)tan (A + B) = tan A + tan B1-tan A tan Bii)tan (A ndash B) = tan A-tan B1+tan A tan Biii) cot (A + B) = cot Acot B-1cot A+cot B(viii) cot (A ndash B) = cot Acot B+1cot B-cot A

A tan B) Or 1= (tan A+ tanB) (1-tan A tanB) Or tanA + tanB + tanA tanB + 1 = 1 + 1Or tanA (1 + tanB) + (1 + tanB) = 2Or (1 + tanA) (1 + tanB) = 2Example 3 Find the value of sin 15degSolution sin 15deg= sin(45deg-30deg) = sin45degcos 30deg- cos45degsin30deg =(1radic2) (radic32) -(1radic2) (12) = (radic3-1) 2radic2Example 4 If sin A = 1 radic10 and sin B = 1 radic5 where A and B are positive acute angles then what is A + B SolutionWe know that sin (A + B) = sin A cos B + cos A sin B= [1 radic10] [radic(1 minus 1 5)] + [1 radic5] radic(1 minus 1 10)= [1 radic10] [radic4 5] + [1 radic5] [radic9 10]= [1 radic50] times (2 + 3)= 5 radic50 = 1 radic2

sin (A + B) = sin π 4rArrHence A + B = π 4Example 5 If A + B = 225o then find [cot A] [1 + cotA] times [cot B] [1 + cot B]Solution[cot A] [1 + cotA] times [cot B] [1 + cot B] = 1 [(1 + tan A) times (1 + tan B)]=1 [tan A + tan B + 1 + tan A tan B] [ tan (A + B) = tan225o]∵

tan A + tan B = 1minus tan A tan BrArr= 1 [1 minus tan A tan B + 1 + tan A tan B]= 1 2

COMMERCE

CLASSIFICTION OF HUMAN ACTIVITIES-ECONOMIC AND NON-ECONOMIC

Firstly we shall recall the previous class for 5 mins especially for the absentees and for also the rest of the students who were there

Today at first we briefly discuss the earlier portions of the chapter

1Business-It includes all those economic activities which are concerned with production and exchange of goods and services with the object of earning profit Example A factory shop beauty parlour also business enterprises

2Profession ndashThe term profession means an occupation which involves application of specialized knowledge and skills to earn a living For Example Chartered Accountancy medicine law tax consultancy are example of professions

Questions1What are the main features of ProfessionAnswer The main features of a profession are as follows a Specialised body of knowledge-Every profession has a specialised and systematised body of knowledge b Restricted entry- Entry to a profession is allowed only to those who have completed the prescribed education and have the specialised examination c Formal education and training ndashA formal education and training is given to the person who wants to acquire the professional

3Employment-Employment mean an economic activity where people work for others in exchange for some remuneration (salary)The persons who work for others are called lsquoemployeesrsquo The persons or organizations which engage others to work for them are called lsquoemployersrsquoEg A doctor working in a hospital is employment as he is working for a salaryA lawyer may serve as a law officer in a bank

With this we shall proceed with the features of both Profession amp Employment

The main features of a profession are as follow

a Specialised body of knowledge b Restricted entry c Formal education and training d Professional association e Service motive f Code of contact

The main features of an employment are as follows

a In employment a person works for others called employer

b An employee provides personal service

c There is a service agreement or contract between the employee and the employer

d The employee has to obey the order of the employer

e No capital investment is made by the employer

Various examples of Employment are as follows

aA teacher teaching in a school or collegeb An engineer employed in Municipal Corporation of DelhicAn accountant working in the accounts department of a companydA doctor working in a hospital

Note In all the above examples of employment the individual who is involved in each example is working as an employee for a salary under an employer

qualification(MBBSCALLB)d Service motive ndashProfessionals are expected to emphasis service more on their clients rather than economic gain f Code of Conduct-The activities of professionals are regulated by a code of conduct

2 What are the main features of EmploymentAnswer The main features of an employment are as followsa In employment a person works for others called employerb An employee provides personal servicec There is a service agreement or contract between the employee and the employerd The employee has to obey the order of the employere No capital investment is made by the employer

3 Give various Professions and their respective Association are given below

Professions

Professional

Professional association

Medical profession

Doctor Medical Council of India

Law profession

Lawyers Bar Council of India

Accounting Profession

Chartered

The Institute of Chartered Accounts of India( ICAI)

Engineerin Engineers The

g Profession

institute of Engineers (India)

Accounts Basic accounting terms

Today we will give you some questions from the previous study material

Questions6) Define accounting7) What do you mean by debit

and credit8) Explain the types of account9) Define the following terms

a) Assetsb) Capitalc) Purchased) Debtorse) Transactions

10) Name the types of accounts given below

a) Krishnas accountb) Machinery accountc) Royalty accountd) Salary accounte) Furniture accountf) Audit fee account

Economics Basic Economic ConceptsSub topic

UTILITY

Before starting todayrsquos class we shall recall the last class which was about UTILITY AND THE FEATURES OF UTILITY

Now we shall proceed with the further topics of the chapter

Todayrsquos topic from the chapter lsquo Basic Economic Conceptsrsquo will be TOTAL UTILITY amp MARGINAL UTILITYNow let us quickly revise the concept of utility with an example ie goods and services are designed because they have an ability to satisfy human wantsThis feature of being able to satisfy human wants is termed as utility For example we derive utility from WiFi services as it gives us satisfaction by connecting us to our friends and family through social media here consumers derive utility from WiFi services

From the above concept we shall start with todayrsquos topicEconomists have defined TOTAL UTILITY (TU) as the total satisfaction obtained by consuming a given total amount of a good and serviceFor example the total satisfaction obtained from eating 10 mangoes is the total utility of 10 mangoes

MARGINAL UTILITY (MU) is the additional satisfaction derived from each additional unit

Questions1 What is Total Utility (TU)

Answer Total Utility (TU) is the

aggregate of the utility that a consumer derives from the consumption of a certain amount of a commodityTU=MU1+MU2++MUn

2 What is Marginal UtilityAnswer

Marginal Utility (MU) is the additional made to the total utility as consumption is increased by one more unit of the commodityMU= TUn ndashTUn-1

NoteOften economists tend to

subdivide utility into an imaginary unit called UTIL

consumed In this casethe utility obtained from each mango as it is consumed as the MU of that mango It is also defined as the addition made to the total utility when an additional unit is consumed Often economists tend to subdivide utility into an imaginary unit called UTIL

Note As a consumer increases the consumption of a good over period of time the total utility or total satisfaction derived from it increases to appoint and thereafter it decreasesHowever as the consumer keeps on consuming the good the marginal utility or the additional utility derived from it decreases

SubjectBusiness studies

Topic

BUSINESSENVIRONMENT

Summary

Now quickly let us revise the earlier points that we have already done in the last class and let us proceed with the other topics that are there in the chapter

Firstly we will recall the internal and external factors of micro environment and then we shall proceed in details

Meaning and list of internal and external factors

aInternal factorsInternal factors refer to all the factors existing within a business firm The internal factors are considered controllable because the enterprise has control over these factorsFor an example a company can alter its organization structure policies programmes employees physical facilities and marketing mix to suit the changes in the environmentList of internal factors areCorporate culture mission and objectives top management organizations structure company image and brand equity company resources

b External factorsExternal factors refer to those individual and groups and agencies with which a particular business organization comes into direct and frequent contact in the course of its functioningThese individuals and groups are known as STAKEHOLDERS because they have a stake (financial interest ) in the working and performance of the particular business List of external forces (stakeholders)Customers competitors investors suppliersmiddlemen (marketing intermediaries)

Execution 1 What do you mean by internal

factors in micro environmentAnswerInternal factors refer to all the factors existing within a business firm The internal factors are considered controllable because the enterprise has control over these factorsFor an example a company can alter its organization structure policies programmes employees physical facilities and marketing mix to suit the changes in the environment

2 What do you mean by external factors in micro environment

AnswerExternal factors refer to those individual and groups and agencies with which a particular business organization comes into direct and frequent contact in the course of its functioningThese individuals and groups are known as STAKEHOLDERS because they have a stake (financial interest) in the working and performance of the particular business

3Who are stakeholdersSTAKEHOLDERS are individuals and groups who have a stake (financial interest ) in the working and performance of the particular business 4Discuss the internal factors in briefa Corporate CultureThe values beliefs and attitudes of the founders and top management of the company exercise

financers publics

customers

suppliersfinancers

competitors

middlemen

publics

Fig STAKEHOLDERS OF A COMPANY

Apart from micro environment the other main dimension of business environment isMacro environment Macro environment refers to the general environment or remote environment within which a business firm and forces in its micro environment operateA company does not directly or regularly interact with the micro environmentTherefore macro environment is also known as indirect action EnvironmentThe macro environment forces are less controllable than the micro forces

Macro environment consists of the following components

POLITICAL AND LEGAL ENVIRONMENT

ECONOMIC SOCIAL AND ENVIRONMENT

CULTURAL

ENVIRONMENT

TECHNOLOGICAL ENVIRONMENT

a strong influence on what the cmpaany stands for how it does things and what it considers importantbMission and objectivesThe business philosophy and purpose of a comoany guide it prioritiesbusiness strategiesproduct market scope and development scope

cTop management structurethe composition of board of directors the degree of professionalization of management and the organizational structure of a company have important bearing on its business decisions

dPower structureThe internal power relationship between the board of directors and the chief executive is an important factor

eCompany image and brand equityThe image and brand equity of the company play a significant role in raising finance forming alliance choosing dealers and suppliers launching new products entering foreign markets

5 What is Macro environmentAnswerMacro environment refers to the general environment or remote environment within which a business firm and forces in its micro environment operateA company does not directly or regularly interact with the micro environmentTherefore macro environment is also known as indirect action EnvironmentThe macro environment forces are less controllable than the micro forces 6 What are the components of macro environmenta Political and legal environmentb Economic environmentc Social and cultural environmentd Technological environment

BUSINESS FIRM

Fig COMPONENTS OF MACRO ENVIRONMENTPolitical science

Introduction to political science

Comparative politics and itrsquos scope Comparative politics is the second major dimension of political scienceIt is also a very vast area of study and a very large number of political scientists even treat it as an autonomous area of study within the board ambit of political scienceScope of comparative politics-

1 All political structures -Comparative politics includes the study of all structures formalnon formal governmental and extra governmental which are directly or indirectly involved in politics in all the countries of the world

2 Functional studies- Comparative politics seeks to study politics less from the point of view of the legal institutions in terms of their powers and move from the point of view of their functions which constitute the political process and their actual Operation in the environment

3 Study of political behaviour- Another important part of its scope is the study of the actual behaviour of the people in the process of politics

4 Study of similarities and differences- comparative politics also undertakesan analysis of the similarities and differences among political process and functions

5 Study of all political systems -comparative politics seeks to analyse the actual behaviour and performance of all political systems western as well as non western

6 Study of the environment and infrastructure of politics-The study of politics demands a study of the psychological sociological economic and anthropological environment in fact the social environment as a whole in which each political system operates

7 Study of political culture- political culture is composed of attitudesbeliefs emotions and values of a society that relate to the political system or politics

8 Study of political participation- Political participation is a universal processThe only difference is that while in some states it is limited in others it is wider

9 Study of political process- political

Answer the following questions-

What is comparative politics

What are the scope of comparative politics

Homework- learn

processes like decision makingpolicy making judicial process leadership recruitment process and others are always at work in all political systems

The scope of comparative politics is very comprehensive It includes everything that falls within the area of political activity and political process

History CAMBRIDGE VIEW ABOUT

THE PARTITION

AND REFUTATION

OF CAMBRIDGE

VIEW

Cambridge view about the Partition The Cambridge school of historians have interpreted that opposition to partition scheme was made entirely by the elitist groups They hold the view that Lord Curzon planned to partition the Bengal for administrative purposeREFUTATION OFCAMBRIDGE VIEW The Rationalist historians have rejected the interpretations of the Cambridge School of historians on various grounds

1 QUESTION State different views of historians regarding Partition of Bengal

ANSWER Cambridge historians believed that Lord Curzon partitioned Bengal for administrative reasons only and not for the political motive The Middle class elitist group protested because of their petty interest The Hindu zamindars protested as they have to spend more money for managing their estatesThe lawyers of Calcutta High court feared to lose their clientBut according to the nationalist Historians was-

2- The ultimate object of Lord Curzon was to crush the unity of Bengal politicians

3- If Bengal becomes a separate province Bengali speaking 16 million people of western part would become minority under Hindi speaking people of Bihar and Oriya speaking people of Orissa

4- The bureaucrats expected that the protest movement would die down quickly

5- Lord Curzon used the Muslim community in his political game

6- Idealism had great contribution in the protest against partition

7- The people of the every section of society were affected by the partition of Bengal

Computer Science

Numbers Convertion of dcimal number to octal numberThe decimal numeral system is the standard system for denoting integer and non-integer numbers It is the extension to non-integer numbers of the Hindu-Arabic numeral system For writing numbers the decimal system uses ten decimal digits a decimal mark and for negative numbers a minus sign - The decimal digits are 0 1 2 3 4 5 6 7 8 9 the decimal separator is the dot in many countries

The octal numeral system or oct for short is the base-8 number system and uses the digits 0 to 7 Octal is sometimes used in computing instead of hexadecimal perhaps most often in modern times in conjunction with file

permissions under Unix systems It has the advantage of not requiring any extra symbols as digits It is also used for digital displays

Follow these steps to convert a decimal number into octal form

1 Divide the decimal number by 82 Get the integer quotient for the next iteration (if the number will not divide equally by 8 then round down the

result to the nearest whole number)3 Keep a note of the remainder it should be between 0 and 74 Repeat the steps until the quotient is equal to 05 Write out all the remainders from bottom to top This is the solution

For example if the given decimal number is 8453

Division Quotient Remainder

8453 8 1056 5

1056 8 132 0

132 8 16 4

16 8 2 0

2 8 0 2

Then the octal solution is 20405

Subject Eng Literature (The Tempest ndash William Shakespeare) Topic Act I Scene 1 Lines 33 to 67 (End of scene) Date 16th April 2020 (4th Period)

[Students should read the original play and also the paraphrase given in the school prescribed textbook]Summary Questions amp Answers

[SUMMARY OF THE ENTIRE SCENE]

o The play starts with the scene of a severe storm at sea Alonso (King of Naples) Sebastian (Alonsorsquos brother) Ferdinand (Alonsorsquos son) Gonzalo Antonio (the usurping Duke of Milan) are in a ship in the midst of the storm

o The mariners are trying their best to control the vessel from running aground and are totally following the orders of their Master the Boatswain They have scant success

o The mariners become extremely unhappy and annoyed when most of the passengers arrive on the deck thereby hampering their effort to save the ship There is serious confrontation between them and the passengers who are part of the Kingrsquos entourage

o The mariners could not save the ship

SUMMING-UP

(i) Vivid description of the scene which gives a realistic description of terror and confusion of a tropical storm

(ii) Shows Shakespearersquos accuracy of knowledge in describing the naval operations and also matters of seamanship

(1) GONZALO Ill warrant him for drowning (L 45-57)

though the ship were no stronger than a nutshell and as leaky as an unstanched

wenchBOATSWAIN Lay her a-hold a-hold Set her two courses Off to

sea again lay her offMARINERS All lost To prayers to prayers All lostBOATSWAIN What must our mouths be coldGONZALO The king and prince at prayers Lets assist them

For our case is theirsSEBASTIAN Im out of patienceANTONIO We are merely cheated of our lives by drunkards

This wide-chopped rascal - would thou mightst lie drowning the washing of ten tides

(a) What does Antonio say at the insolent manners of the boatswain just before the given passage

Being irritated at the insolent manners of the boatswain just before the given extract Antonio the Duke of Milan calls him a worthless dog son of a woman without any morals an arrogant and disrespectful noisemaker He says that the boatswain deserved to be hanged(b) What statement does Gonzalo repeat about the boatswain

Gonzalo shows his faith that the boatswain is not destined to die by drowning He is destined to be hanged and nothing can alter this decree of destiny He says that even if the ship was as frail as a nutshell the boatswain could not be drowned for his destiny was to be hanged(c) What do the passengers do when they have lost all hope of their survival

When the passengers have lost all hope of survival they take

(iii) The opening scene justifies the title ndash The Tempest

UNANSWERED QUESTIONS

(i) The King always travels with his entire fleet including his soldiers Where were the other ships

(ii) Why was the ship in that area Where was it coming from or going where

(iii) The ship broke apart What happened to those who were in the ship

(We shall get the answer to the above questions as the play progresses)

leave of life with fervent prayers The mariners take their last hearty drink and are ready for death(d) What blame does Antonio put upon the mariners and the boatswain Antonio rebukes the mariners that these drunkards have brought them to the present crisis by neglecting their duties He blames them saying that they are going to lose their lives entirely for the negligence of the boatswain and his fellows(e) What does Antonio say while cursing the boatswain

Antonio gives vent to his wrath upon the boatswain in particular He calls the boatswain a wide-mouthed rascal who deserves to be hanged on the sea-shore at low water mark so that ten tides might wash over his body and take out of him all the liquor that he has been drinking

Class XIISubject Topic Summary ExecutionHistory Topic

1 1935 ACT AND WORKING OF PROVINCIAL AUTONOMYCONGREE AND OTHER MINISTERSSUB TOPIC GOVERNMENT OF INDIA ACT1935

Government of India Act 1935 This act established a lsquoFederation of Indiarsquo made of British Indian provinces and Indian states and provided for autonomy with a government responsible to the elected legislature in every provinceThis act introduced abolition of Diarchy at provinces The entire provincial administration was introduced to the responsible ministers who were controlled and removed by the provincial legislature The provincial autonomy means two things First The provincial governments were wholly responsible to the provincial legislature Secondly Provinces were free from outside control and interference in the large number of matters The act divided the powers between the centre and provinces in terms of three lists- Federal list( for centre) Provincial list (for province) and concurrent list (for both) Residuary powers were given to the viceroy In the election under the government of India Act the Congress swept the poll the mandate of the people came in favour of the congress so far as general Hindu seats were concerned The Congress did not get a single Muslim seates in Bombay CP UP Sind and BengalIn five provinces Congress had yhe clear majority In BengalNWFPAssam and Bombay Congress emerged as a single largest partyOn the other side the performance of the Muslim League was badThus the Congress formed ministers in 7 provinces out of 11 provinces Coalition ministry was also formed in two other provincesOnly BENGAL AND Punjab had non- congress ministries

1 QUESTION What was the main change introduced by the Government of India ActANSWER a) The Act gave more

autonomy to the provinces b) Diarchy was abolished at the

provincial levelsc) The Governor was the head of

the executived) There was a council of

ministers to advise him The ministers were responsible to the provincial legislatures who controlled them The legislature could also remove the ministers

e) The Governors still retained special reserve powers

2 QUESTION Why did the federal scheme introduced by the Government of India Act 1935 never come into operation

ANSWER The Federal structure of the Government of India was to be composed with the Governor General and Council of ministers The Federal legislature was to be Bicameral legislature- The council of states and the House of Assembly The ministers were to be chosen by the Governor general and they were to hold the office during his pleasure

The provinces of British India would have to join the federation but this was not compulsory for the princely states

This federation never materialised because of the lack of support from the required number of

princely statesThis act was refused and

rejected by the princes the Congress and the Muslim League

Thus both Congress and the League participated in the election of 1937 Thus the federal part was never introduced but the provincial part was put into operations

Bengali 2nd

Language

াচেরর পরাথCনা(কহিতা )

াচেরর পরাথCনা কহিতাটি কহি (ঙখ দেঘাচে4র দো আচো য কহিতায় াচেরর পতর হমায়ন কঠিন দেরাচেগ আxানত ার ঈশবর া আললার কাচেছ পরাথCনা কচেরচেছন তার পচেতরর ীন হিফহিরচেয় হিচেত এই কহিতায় ার পচেতরর ীন হিভbা দেচেয়চেছন ারার এমনহিক হিনচের ীন হিসCচেনর হিহিনমচেয় হিতহিন তার দেছচের ীন হিফচের দেপচেত দেচেয়চেছন তার দেছচের এই দেরাচেগর ন য হিতহিন হিনচেচেকই ায়ী কচেরচেছন তার হিনচের করা পাপচেকই হিতহিন ায়ী কচেরচেছন এছাা রানৈনহিতক ও আথCসামাহিক অসথার কথা তচে ধরা চেয়চেছ এই কহিতায় ার তার হিনচের পাপ কমCচেকই ায়ী কচেরচেছ ার অন যায় ভাচে দেপহি((হিকতর মাধ যচেম অপররা য কচেরচেছ আর এই অন যায় কাচের ন যই তার পহিরাচের হিপযCয় এচেসচেছ দে এক পরকার মানহিক নধন ইহিতাচেসর ার হিপতা চেয় সবাভাহিকভাচে ভাচোাসা দে মমতা দেথচেক মকত চেত পাচেরনহিন তাই হিপতা চেয় আললা া ভগাচেনর কাচেছ পতর হমায়চেনর পরানহিভbা দেচেয়চেছন ার আললা া ভগাচেনর কাচেছ াহিনচেয়চেছন তার হিনচের ীন হিসCন হিচেত হিতহিন রাী তার হিহিনমচেয় পচেতরর ীন হিফচের দেপচেত দেচেয়চেছন াচেরর হিপতসভ হিচেকর কথা এই কহিতায় ফটিচেয় দেতাা চেয়চেছ হিপতা পচেতরর হিরাহিরত মান নধচেনর কথা তচে ধরা চেয়চেছ

হিচে(4 হিকছ াইচেনর তাৎপযC১) ldquoদেকাথায় দেগ ওর সবচছয দেৌন দেকাথায় কচেরায় দেগাপন bয়ldquoউততর) াচেরর পতর হমায়ন কঠিন দেরাচেগ অসসথ তাই তার দেযৌন াহিরচেয় যাচেচছ এই দেরাচেগ তাচেক দেগাপচেন কচেরকচের াচেচছ তার সক (হিকত ধীচের ধীচের bয় চেচছ তাই হিপতা চেয় ার আললার কাচেছ হমায়চেনর পরান হিভbা দেচেয়চেছন২) ldquoাগাও (চেরর পরাচেনত পরানতচের ধসর (ন দেযর আান গানldquoউততর) াচেরর পতর হমায়ন কঠিন দেরাচেগ আxানত তাই ার আ দে(াচেক মমCাত (চেরর পচেথ পরানতচের আান গান ধবহিনত দোক দেসই আান গান আললার কাচেছ দেযন চে যায় আললা দেযন এই আহিতC শচেন পচেতরর ীন হিফহিরচেয় দেয় ৩)ldquoনাহিক এই (রীচেরর পাচেপর ীানচেত দেকানই তরারণ দেনই ভহি4চেতরldquoউততর) হমায়চেনর অসসথতার ন য ার হিনচেচেকই ায়ী কচেরচেছন কারন ার অচেনক রা য অন যায় ভাচে কচেরচেছ তাই তার এই পাপ কাচের ন য তার ঘচের আ হিপ এচেসচেছ এই অন যায় কাচের ন য তার মহিকত দেনই তাই ার আললার কাচেছ এই পাপ কাচেযCর ন য bমা পরাথM

Hindi 2ndlang

-ासी(जयशकर परसा-)

-ासी जयशकर परसा- की एक ऐसी कहानी ह जिजसम भारतीय ससकनित और राषटरीयता का सवरगजीतहोता ह इस कहानी म इरावती एक निहद कनया ह जिजस मलअचछो न मलतान की लट म पकडा और -ासी बना दि-या उस 500 दि-न -कर काशी क एक महाजन न खरी-ा दसरी -ासी निफरोजा ह वह गलाम ह निफरोजा को छडान क कतिलए अहम- को 1000 सोन क कतिसकक भजन थ जो अभी तक नही आए थ राजा साहब कठोर होत हए भी निफरोजा को निबना धनराकतिश क कतिलए उस म कर -त ह वनिफरोजा को अहम- को समझान की बात कहत हकहानी क अत म हम -खत ह निक इरा वती और जाटो क सर-ार बलराज का मिमलन होता हअहम- को यa म मार दि-या जाता ह वहा निफरोजा की परसननता की समामिध बनती ह वहा एक फल चढती ह और डीजल आती ह निफरोजा उस समामिध की आजीवन -ासी बनी रहती हलखक अपन उददशय अथात -ास परथा पर परकाश डालन और इस परथा क कारण होन वाल -ातो क दखो को दि-खान म पणता सफल हए ह

helliphellipContinue to next

Biology Reproductio Today we will discuss about vegetative Q1 Name some vegetative propagules

n in Organisms

propagation of plants The process of multiplication in which fragments of plant body function as propagule and develop into new individual is called vegetative propagation The units of such propagation are runner rhizome tuber bulb etc

and the speciesinvolvedVegetative propagules

Parts involved

Bulb StemBulbil BulbilRhizome Stem Runner Stem Tuber Stem Offset Stem Leaf buds Leaves Suckers Stem

Corns Stem stolon

Q2 State advantages of vegetative propagation

i) Rapid methodii) Sure and easy methodiii) Useful in plants that cannot

produce viable seeds or long seed dormancy

iv) Maintains purity of raceQ 3 Banana fruit is said to be parthenocarpic where as turkey is said to be parthenogenetic WhyBanana develops without fertilization from an unfertilized ovary thus is parthenocarpicIn turkey the ovum or female gamete developinto a new chick without fertilization thus isparthgenetic

Q4 Why is water hyacinth is called as a ldquoTerror of Bengalrdquo Water hyacinth can

propagatevegetatively all over the water body in a short per short period of time This resulted increased biochemicaloxygen oxygen demand of water body causing mortalityof fishes It is very difficult to get rid off them Thus known as terror of Bengal

Chemistry

Solid state GENERAL CHARACTERISTICS OF SOLID STATEIn nature the particular state of matter is governed by two opposing forces at given set of temperature and pressure These forces are intermolecular force of attraction and thermal energy If intermolecular force of attraction is high as compared to thermal energy particles remains in closest position

Intext QuestionsQ1 Classify the following solids as crystalline and amorphous Sodium chloride quartz glass quartz rubber polyvinyl chloride Teflon

A1 Crystalline

and hence very less movement in particles is observed In this case solid state is the preferred state of matter

Let us revise the general characteristics of solid

i) Fixed mass volume and shape

ii) Strong intermolecular force of attraction

iii) Least intermolecular space

iv) Fixed position of constituent particles

v) Incompressible and rigid

Q2 what type of interactions hold the molecules together in a polar molecular solid[CBSE 2010]A2 The molecules in a solid are held together by van der Waals forces The term van der Waals forces include hydrogen bonding dipole-dipole attraction and London dispersion forces All molecules experience London dispersion forces In addition polar molecules can also experience dipole-dipole interactions So the interactions that holds the molecule together in polar molecular solid are London dispersion force and dipole-dipole interactionsQ3 Write a feature that will distinguish a metallic solid from an ionic solid [CBSE 2010]A3 Metals are malleable and ductile whereas ionic solid are hard and brittle Metallic solid has typical metallic lustre But ionic solid looks dullQ4 Write a point of distinction between a metallic solid and an ionic solid other than metallic lustre [CBSE 2012]A4 Metals are malleable and ductile whereas ionic solid are hard and brittleQ5 Write a distinguish feature of metallic solid [CBSE 2010]A5 The force of attraction in

solid Sodium chloride Quartz Amorphous solid Quartz glass rubber polyvinyl chloride Teflon Q2 why glass is considered as super cooled liquidA2 Glass shows the tendency to flow at slower rate like liquid Hence they considered as super cooled liquidQ3 why the window glass of old buildings show milky appearance with timeA3 Glass is an amorphous solid Amorphous solid has the tendency to develop some crystalline character on heating Due to heating in day over the number of years glass acquires some crystalline character and show milky appearanceQ4 why the glass panes fixed to window or doors of old building become slightly thicker at bottomA4 Glass is super cooled liquid It has the tendency to flow down very slowly Due to this glass pane becomes thicker at the bottom over the timeQ5 Sodium chloride is a crystalline solid It shows the same value of refractive index along all the direction TrueFalse Give reasonA5 FalseCrystalline solid shows anisotropy in properties That is it shows different values for the given physical property in different direction All the crystalline solids show anisotropy in refractive index Therefore sodium chloride will show different values of refractive index on different directions

Q6 Crystalline solid are anisotropic in nature What does this statement means

between the constituent particles is special kind of electrostatic attraction That is the attraction of positively charged kernel with sea of delocalized electronsQ6 which group of solid is electrical conductor as well as malleable and ductile [CBSE 2013]A6 Metallic solidQ7 why graphite is good conductor of electricity although it is a network (covalent solid)A7 The exceptional property of graphite is due to its typical structure In graphite each carbon is covalently bonded with 3 atoms in same layer The fourth valence electron of each atom is free to move in between different layersThis free electron makes the graphite a good conductor of electricity

[CBSE 2011]A6 Anisotropy is defined asrdquo Difference in properties when measured along different axis or from different directionsrdquo Crystalline solid show different values of some of the physical properties like electrical resistance refractive index etcwhen measured along the different directions The anisotropy in crystalline solid arises due to the different arrangement of particles in different directions

Math Function Composition of functions Think of an industrial plant that produce bottles of cold drinks first there is the operation (or function) f that puts the cold drink inside the bottle followed by the opeartion g that close the bottle with the capThis leads to the following definitionDefinition Let f A rarr B and g B rarr C be two functions Then the composition of f and g denoted by gof is defined as the function gof A rarr C given by gof(x) = g(f (x)) forall x isinA

Definition A function f X rarr Y is defined to be invertible if there exists a function g Y rarr X such that gof = IX and fog = IY The function g is called the inverse of f and is denoted by f -1

Thus if f is invertible then f must be one-one and onto and conversely if f is one-one and onto then f must be invertible This fact significantly helps for proving a function f to be invertible by showing that f is one-one and onto specially when the actual inverse of f is not to be determined

Example 1 Let f 2 3 4 5 rarr 3 4 5 9 and g 3 4 5 9 rarr 7 11 15 be functions defined as f(2) = 3 f(3) = 4 f(4) = f(5) = 5 and g (3) = g (4) = 7 and g (5) = g (9) = 11 Find gofSolution We have gof(2) = g (f(2)) = g (3) = 7 gof(3) = g (f(3)) = g (4) = 7gof(4) = g (f(4)) = g (5) = 11 and gof(5) = g (5) = 11Example 2 Find gof and fog if f R rarr R and g R rarr R are given by f(x) = cos x and g (x) = 3x2 Show that gof ne fogSolution We have gof(x) = g(f(x))=g(cosx) = 3 (cos x)2

= 3 cos2 x Similarly fog(x)=f(g (x))= f(3x2)= cos (3x2) Note that 3cos2 x ne cos 3x2 for x = 0 Hence gof ne fogExample 3 Show that if f A rarr B and g B rarr C are onto then gof A rarr C is also ontoSolution Given an arbitrary element z isin C there exists a pre-image y of z under g such that g (y) = z since g is onto Further for y isin B there exists an element x in A with f(x) = y since f is onto Therefore gof(x) = g (f(x)) = g (y) = z showing that gof is onto Example 4 Let Y = n2 n isin N sub N Consider f N rarr Y as f(n) = n2 Show that

f is invertible Find the inverse of fSolution An arbitrary element y in Y is of the form n2 for some n isin N This implies that n =radicy This gives a function g Y rarr N defined by g (y) =radicy Nowgof (n) = g (n2)=radicn2 = n and fog (y) =f(radicy) = (radicy) 2 y which shows that gof=IN and fog= IY Hence f is invertible with f -1 = g

Political Science

Constitution of India-The Preamble

Summary

Objective of the state-To secure equality of status and of opportunity To promote fraternity among all the citizens To assure the dignity of the individuals and Unity and integrity of the nation

Justice-Justice stands for rule of law absence of arbitrariness and a system of equal rights freedom and opportunities for all in a society India seeks social economic and political justice to ensure equality to its citizens

Liberty-Liberty implies the absence of restraints or domination on the activities of an individual such as freedom from slavery serfdom imprisonment despotism etc The Preamble provides for the liberty of thought expression belief faith and worship

Equality-Equality means the absence of privileges or discrimination against any section of the society The Preamble provides for equality of status and opportunity to all the people of the country

Fraternity-The Preamble declares that fraternity has to assure two thingsmdashthe dignity of the individual and the unity and

Execution

Answer the following questions-

Short notes-1 Equality2 Fraternity3 Justice4 Liberty

Homework-Learn

integrity of the nation The word integrity has been added to the Preamble by the 42nd Constitutional Amendment (1976)

Business studies

Human resource management (chapter 1)

On the day of 1504 2020 I have discussed with you the managerial functions and procurement functions of HRM

Today weare going to discuss about the development function integration functions and maintenance function

Development functions-HRM improves the knowledge skills attitude and values of employees so that they the present and future jobs more effectively it includes

1) Development functions of HRM

a) Performance appraisal = It implies systematic evaluation of employees with respect to their performance on the job and their potential for development

b) Training =It is the process by which employees learn knowledge skills and attitudes to achieve organisational and personal goals

c) Executive development = It is the process of developing managerial talent through appropriate program

2) Integration functionsa) HRM reconcile the goals of

organisation with those of its members through integrating function

b) HRM tries to motivate employees to various financial and non financial incentives provided in job specification etc

3) Maintenance functiona) HRM promote and protect the

physical and mental health of employees by providing several types of benefits like housing medical aid etc

b) It Promote Social security measures to employees by providing provident fund pension gratuity maternity benefits

SubjectCOMMERCE

Topic

BUSINESSENVIRONMENT

Summary

Now quickly let us revise the earlier points that we have already done in the last class and let us proceed with the other topics that are there in the chapter

Firstly we will recall the internal and external factors of micro environment and then we

Execution 3 What do you mean by internal factors

in micro environmentAnswerInternal factors refer to all the factors existing within a business firm The internal factors are considered controllable because the enterprise has control over these factors

Development FunctionsPerformance AppraisalTrainingExecution Development

shall proceed in details

Meaning and list of internal and external factors

aInternal factorsInternal factors refer to all the factors existing within a business firm The internal factors are considered controllable because the enterprise has control over these factorsFor an example a company can alter its organization structure policies programmes employees physical facilities and marketing mix to suit the changes in the environmentList of internal factors areCorporate culture mission and objectives top management organizations structure company image and brand equity company resources

b External factorsExternal factors refer to those individual and groups and agencies with which a particular business organization comes into direct and frequent contact in the course of its functioningThese individuals and groups are known as STAKEHOLDERS because they have a stake (financial interest ) in the working and performance of the particular business List of external forces (stakeholders)Customers competitors investors suppliersmiddlemen (marketing intermediaries)financers publics

customers

suppliersfinancers

For an example a company can alter its organization structure policies programmes employees physical facilities and marketing mix to suit the changes in the environment

4 What do you mean by external factors in micro environment

AnswerExternal factors refer to those individual and groups and agencies with which a particular business organization comes into direct and frequent contact in the course of its functioningThese individuals and groups are known as STAKEHOLDERS because they have a stake (financial interest) in the working and performance of the particular business

3Who are stakeholdersSTAKEHOLDERS are individuals and groups who have a stake (financial interest ) in the working and performance of the particular business 4Discuss the internal factors in briefa Corporate CultureThe values beliefs and attitudes of the founders and top management of the company exercise a strong influence on what the cmpaany stands for how it does things and what it considers importantbMission and objectivesThe business philosophy and purpose of a comoany guide it prioritiesbusiness strategiesproduct market scope and development scope

cTop management structurethe composition of board of directors the degree of professionalization of management and the organizational structure of a company have important bearing on its business decisions

dPower structureThe internal power relationship between the board of directors and the chief executive is an important factor

e Company image and brand equityThe image and brand equity of the company play a significant role in raising finance forming alliance choosing dealers and suppliers launching new products entering foreign markets

5 What is Macro environmentAnswerMacro environment refers to the general

competitors

middlemen

publics

Fig STAKEHOLDERS OF A COMPANY

Apart from micro environment the other main dimension of business environment isMacro environment Macro environment refers to the general environment or remote environment within which a business firm and forces in its micro environment operateA company does not directly or regularly interact with the micro environmentTherefore macro environment is also known as indirect action EnvironmentThe macro environment forces are less controllable than the micro forces

Macro environment consists of the following components

POLITICAL AND LEGAL ENVIRONMENT

ECONOMIC SOCIAL AND ENVIRONMENT

CULTURAL

ENVIRONMENT

TECHNOLOGICAL ENVIRONMENT

Fig COMPONENTS OF MACRO ENVIRONMENT

environment or remote environment within which a business firm and forces in its micro environment operateA company does not directly or regularly interact with the micro environmentTherefore macro environment is also known as indirect action EnvironmentThe macro environment forces are less controllable than the micro forces 6 What are the components of macro environmenta Political and legal environmentb Economic environmentc Social and cultural environmentd Technological environment

Computer Science

Logic gates

Digital systems are said to be constructed by using logic gates These gates are the AND OR NOT NAND NOR EXOR and EXNOR

BUSINESS FIRM

gates The basic operations are described below with the aid of truth tables

AND gate

The AND gate is an electronic circuit that gives a high output (1) only if all its inputs are high A dot () is used to show the AND operation ie AB Bear in mind that this dot is sometimes omitted ie ABOR gate

The OR gate is an electronic circuit that gives a high output (1) if one or more of its inputs are high A plus (+) is used to show the OR operationNOT gate

The NOT gate is an electronic circuit that produces an inverted version of the input at its output It is also known as an inverter If the input variable is A the inverted output is known as NOT A This is also shown as A or A with a bar over the top as shown at the outputs The diagrams below show two ways that the NAND logic gate can be configured to produce a NOT gate It can also be done using NOR logic gates in the same way

NAND gate

This is a NOT-AND gate which is equal to an AND gate followed by a NOT gate The outputs of all NAND gates are high if any of the inputs are low The symbol is an AND gate with a small circle on the output The small circle represents inversion

NOR gate

This is a NOT-OR gate which is equal to an OR gate followed by a NOT gate The outputs of all NOR gates are low if any of the inputs are highThe symbol is an OR gate with a small circle on the output The small circle represents inversion

EXOR gate

The Exclusive-OR gate is a circuit which will give a high output if either but not both of its two inputs are high An encircled plus sign ( ) is used to show the EOR operation

EXNOR gate

The Exclusive-NOR gate circuit does the opposite to the EOR gate It will give a low output if either but not both of its two inputs are high The symbol is an EXOR gate with a small circle on the output The small circle represents inversion The NAND and NOR gates are called universal functions since with either one the AND and OR functions and NOT can be generated

Note A function in sum of products form can be implemented using NAND gates by replacing all AND and OR gates by NAND gates A function in product of sums form can be implemented using NOR gates by replacing all AND and OR gates by NOR gates

Logic gate symbols

Table 2 is a summary truth table of the inputoutput combinations for the NOT gate together with all possible inputoutput combinations for the other gate functions Also note that a truth table with n inputs has 2n rows You can compare the outputs of different gates

Logic gates representation using the Truth table

Example

A NAND gate can be used as a NOT gate using either of the following wiring configurations

Subject Eng Literature (The Tempest ndash William Shakespeare) Topic Act III Scene 3 Lines 53 to 110 (End of the scene) Date 16th April 2020 (2nd Period)

[Students should read the original play and also the paraphrase given in the school prescribed textbook]Summary Questions amp Answers

o Seeing this strange scene all are inclined to believe the tales told by travelers that there truly are ldquounicornsrdquo and ldquothe phoenixrsquo thronerdquo

o As they are about to sit down to the feast the banquet is snatched away by a harpy (Ariel disguised) A spiritrsquos voice (Arielrsquos voice) denounces Alonso Sebastian and Antonio with particular

1 ARIEL You are three men of sin whom Destiny

(Line 53-58)That hath to instrument this

lower world And what is int the never-surfeited sea

Hath caused to belch up you and on this island

Where man doth not inhabit you rsquomongst men

Being most unfit to live I have made you mad

reference to their crime in expelling Prospero from Milan They have not received any punishment for their deed earlier but the time for their punishment has arrived Upon Alonso it pronounces ldquolingering perdition worse than deathrdquo from which there is no remedy except through sincere repentance Ariel then vanishes in thunder and the shapes enter again and carry away the table

o Prospero watching invisibly is very pleased with the performance of Ariel and his (Prosperorsquos) ldquomeaner ministersrdquo All his enemies are now in his power and are in a fit of desperation He then leaves them and goes to see how Ferdinand and Miranda are getting on

o Alonso is now much humbled and penitent with the after effect of the spiritrsquos denunciation of his crimes He believes that his son is lost forever After this all disperse being stricken mad by the speech of the spirit

o Gonzalo fearing that they may do violence to themselves or to one another follows them and bid others to follow

(a) To whom does Ariel disguised as a harpy call the three sinners What game did Fate of Destiny play with

them

The three sinners called by Ariel are Alonso Sebastian and Antonio It was Destiny which had caused the ocean to cast the three sinners on the shore Though the ocean is all the time devouring whatever appears on its surface and is never satisfied with its continual swallowing of the ships and men in the present case the ocean had cast these three sinners on the shore without killing them

(b) Who had jointly been responsible for the conspiracy against Prospero What is Prosperorsquos purpose behind all this

Three men Alonso Sebastian and Antonio had jointly

been responsible for the conspiracy against Prospero They had driven out Prospero form Milan Prosperorsquos purpose is to make these three sinners realize the wrong they had done He wants them to repent for their criminal deeds because repentance leads to self-esteem(c )What does Ariel (the harpy) tell Alonso and his companions when they take out their swords to attack him

Seeing them drawing their swords Ariel (harpy) tells them that he and his companions are the instruments of destiny and that it is not possible for human beings to do them any injury He says that the swords of human beings can not injure even a minute part of his feathers Their swords are as ineffective against him and his companions as against the wind or the water

(d) Give the explanatory meanings of the following expressions in the context of the above extract

(i)Never surfeited (ii) Belch up (iii) lsquomongst men

(i) Never surfeited never led to satisfaction

(ii) Belch up cast ashore(iii) lsquomongst men in human

society2

I and my fellows (Line 60-65)

Are ministers of Fate The elementsOf whom your swords are tempered may as wellWound the loud winds or with bemocked-at stabsKill the still-closing waters as diminishOne dowl thats in my plume

IMPORTANT PASSAGES EXPLAINED

The elements

(Line 61-66)Of whom your swords are tempered may

as wellWound the loud winds or with

bemocked-at stabs

(a) Who is lsquoIrsquo Who are his lsquofellowsrdquo

lsquoIrsquo is referred to Ariel in disguise of a harpy His lsquofellowsrsquo are other spirits serving Prospero the real Duke of Milan who has acquired supernatural powers after being banished from his Dukedom Prospero has settled in this uninhabited island

(b) What are the elements that have temperrsquod the swords Why will it not work against the speaker

The swords (of Alonso and his companions) are tempered by metal (steel) which is taken out of the earth and refined by

Kill the still-closing waters as diminishOne dowl thats in my plume My fellow

ministersAre like invulnerable

In these words Ariel reminds the King and his companions of the utter futility of drawing swords against himself and his fellows Ariel drives Alonso Antonio and Sebastian the three men of sin to desperation ndash a state in which men do violence to themselves They draw swords to strike Ariel But Ariel reminds them that he and the other spirits are the ministers of destiny and nothing can wound them The steel of which their swords are made of may cut the wind or water which being divided always closes up again Even supposing that such things may be possible it is quite impossible that their swords will cut one feather in their plume They are incapable of being wounded by any sword of man Hence it is foolish on their part to attempt to strike at Ariel and his fellow-spirits

For which foul deed

(Line 72-75)The powers delaying not forgetting

haveIncensed the seas and shores yea all the

creatures Against your peace

Ariel enters like a harpy and remaining invisible tells Alonso Sebastian and Antonio that he and other harpies are the agents of Destiny appointed to carry out her decrees He tells them that their punishment for the crime against Prospero which has been so long deferred is now to fall upon them He reminds them that they had expelled Prospero from Milan and set him and his innocent child adrift on the sea and that the sea had paid them back for their sin by the shipwreck and by the calamities they have suffered He tells them that the powers above which did not forget this mean treachery but only deferred the punishment have now engaged the seas and the shores and all living beings including him and his comrades against them The very elements and supernatural agency Ariel adds have taken up the avenging of their crime against Prospero

the action of fire It may cut the wind or water which being divided always closes up again

The sword will not work against the spirits and the harpy because they are the ministers of destiny and nothing can wound them nor it will cut a single feather in their plume

(c )What is the meaning of lsquodowlrsquo in the last line

The term lsquodowlrsquo means a filament or the smallest part of a feather In this context Ariel in disguise of harpy says that their sword cannot even damage the smallest filament of their (Arielrsquos and other spirits) feathers as they are incapable of being wounded by any sword of man

(d) What does the speaker remind the listeners about

Ariel in disguise of harpy reminds Alonso the King of Naples Sebastian Alonsorsquos brother and Antonio the present Duke of Milan and the treacherous brother of Prospero as they being three men of sin He even reminds them that their punishment for their crime against Prospero which has been so long deferred now falls upon them He reminds them that they have expelled Prospero from Milan and has set him along with his innocent infant daughter adrift on the sea So the sea has paid them back for their sin by their shipwreck and the calamities they have suffered since then The harpy rebukes Alonso of his sin that has incensed the Gods and has deprived him of his son as a punishment

(e) How do they respond

When Ariel in disguise of a harpy reminds Alonso Sebastian and Antonio of their past misdeeds and sin Alonso has a look of terror and confusion in his eyes He utters the words of sincere repentance wrung out of his conscience-stricken heart It appears to him that all the elements of nature the sea-waves the wind and the thunder proclaiming a loud voice in the name of Prospero and the crime Alonso has committed against him They are calling upon him to repent There is a deep storm raging in Alonsorsquos breast and the echoes of that storm are ringing in his ears like a clear note of wind-instrument A note of denunciation of Alonsorsquos crime leaves him much humbled and penitent and confirms his belief that his son is lost forever But Sebastian and Antonio shows some courage instead of repentance They wish to kill the spirits or devils if it appears

3

Of my instruction hast thou nothing bated (Line 85-93)

In what thou hast to say So with good life

And observation strange my meaner ministers

Their several kinds have done My high charms work

And these mine enemies are all knit upIn their distractions They now are in my

powerAnd in these fits I leave them while I visitYoung Ferdinand whom they suppose is

drownedAnd his and mine loved darling

Methought the billows spoke and (Line 96-99)

told me of itThe winds did sing it to me and the

thunderThat deep and dreadful organ-pipe

pronouncedThe name of Prosper It did bass my

trespass

These are the words of contrition coming from Alonso Ariel has driven him to a deep repentance for conspiring with Antonio against Prospero He now feels a sincere remorse It appears to him that all the elements of nature the sea-waves the wind and the thunder proclaimed with a loud voice the name of Prospero and the crime Alonso had committed against him They are calling upon him to repent There is a deep storm raging in Alonsorsquos breast and the echoes of that storm are ringing in his ears like the clear note of a wind-instrument

Comment These are the words of sincere repentance wrung out of the conscience-stricken heart of Alonso Alonso who is the lesser villain is the first to give way to remorse under the effect of Arielrsquos speech The words of Ariel seem to him to be the voice of conscience speaking to him He is driven to desperation a state in which he might do violence to his life

(a) Identify the speaker State the context

Prospero the ruler of the island is the speaker The famous banquet scene has been enacted very well Ariel and his junior spirits have played their roles excellently Prospero is glad to say words of praise for them(b) In what way the speakerrsquos instructions have been carried out

According to Prosperorsquos instructions a banquet was presented before the King of Naples and his companions when they were tired and hungry Just when they were preparing to eat the feast the banquet was suddenly removed by exercising supernatural powers All this was done by Ariel Prosperorsquos chief assistant and a powerful spirit

Ariel not only made the feast disappear but also delivered his speech blaming the King and his two companions for their past wicked deeds He warned them to repent for their misdeeds or suffer forever on that uninhabited island

(c) Who are referred to as lsquomeaner ministersrsquo What have they done

Prospero refers as lsquomeaner ministersrsquo to his other lesser spirits who were assisting Ariel in presenting a scene before the kingrsquos party They entered the scene to the accompaniment of music They assumed several strange shapes and brought in a banquet Then they danced about it with gentle actions of salutations thus inviting the King and others to eat the feast

These spirits play their role again when Ariel in the shape of a harpy quits the scene These shapes enter again and dancing with mocking gestures carry away the table

(d) Who are the speakerrsquos enemies What has happened to them

King of Naples Alonso his brother Sebastian and the present Duke of Milan Antonio (Prosperorsquos own brother) are Prosperorsquos enemies With the turn of events they have all been washed ashore on the island which is ruled by Prospero the great magician Actually this happened after the shipwreck caused by a storm which was raised by Prospero with the purpose of bringing these people to his island Prosperorsquos spirits have already confused and terrified these enemies and they are under Prosperorsquos control He can treat them as he likes

(e) What does he say about Ferdinand Explain what is meant by ldquohellip his and mine darlingrdquo

Prospero knows that Alonsorsquos son prince Ferdinand is alive though his father thinks that the prince has been drowned

Prospero refers to his daughter Miranda who is dear to him She is also very dear to Prince Ferdinand who has fallen in love with her They are waiting to be married soon for which they have received Prosperorsquos consent

4

ALONSO O it is monstrous monstrous (Line 95-102)

Methought the billows spoke and told me of it

The winds did sing it to me and the thunderThat deep and dreadful organ-

pipe pronouncedThe name of Prosper It did bass

my trespassTherefore my son ithrsquo ooze is

bedded andIll seek him deeper than eer

plummet soundedAnd with him there lie mudded

(a) In what way does Alonso express his horror when his conscience is awakened by Arielrsquos words

When Alonsorsquos conscience is awakened by Arielrsquos words he expresses his horror at what he has heard He gets the feeling that the waves of the ocean the wind and the loud thunder have spoken to him and uttered the name of Prospero Because of being reminded of his crime in a very loud and rough voice he comes to realize that he has lost his son for his past misdeeds

(b) What does Alonso imagine about his son What does Alonso want to do in his desperate state

Alonso imagines that his son is lying in the mud at the bottom of the sea He feels desperate that he wants to drown himself in the ocean deeper than the plumb-line has ever gone He wants to lie with his son at the bottom of the sea

(c) How do Sebastian and Antonio want to face the evil spirits

Sebastian says that he is not at all afraid of what the harpy has said and that he is prepared to fight any number of such monsters if they appear before him only one at a time Antonio says that he would support Sebastian in the fight against the fiendsyyy

(d) Why does Gonzalo ask Adrian to follow the three men

Gonzalo tells Adrian that all the three men namely Alonso Sebastian and Antonio are in a wild and reckless mood The thought of the heinous crime of which they are guilty has begun to torment their minds So he asks Adrian to follow those three men without loss of time and prevent them from doing anything which the turmoil in their minds might lead them to do

(e) What opinion do you form of Alonso from the above extract

Alonso who is the lesser villain is the first to give way to remorse under the effect of Arielrsquos speech The words of Ariel seem to him to be the voice of conscience speaking to him He is driven to desperation a state in which he might do violence to his life

Subject =Accounts

Ac-12 15420 topic-pL Appropriation ac

PROFIT AND LOSS APPROPRIATION ACCOUNT

MEANING AND PREPARATIONProfit and Loss Appropriation Account is merely an extension of the Profit and Loss Account of the firm The profit of the firm has to be distributed amongst the partners in their respective profit sharing ratio But before its distribution it needs to be adjusted All Adjustments like partnerrsquos salary partnerrsquos commission interest on capital interest on drawings etc are made in this account These adjustments will reduce the amount of profit for distribution This adjusted profit will be distributed amongst the partners in their profit sharing ratio To prepare it at first the balance of Profit and Loss Account is transferred to this account The journal entries for the preparation of Profit and Loss Appropriation Account are given below

1 for transfer of the balance of Profit and Loss Account to Profit and Loss Appropriation Account

(a) In case of Net Profit

Profit and Loss Ac helliphelliphelliphelliphellipDrTo Profit and Loss Appropriation Ac(Net Profit transferred to Profit and Loss Appropriation Ac)

(b)In case of Net Loss

Profit and Loss Appropriation Achelliphelliphellip DrTo Profit and Loss Ac(Net Loss transferred to Profit and Loss Appropriation Ac)

2 for Interest on Capital

For transferring on Interest on CapitalProfit and Loss Appropriation Achelliphelliphellip DrTo Interest on Capital Ac(Interest on capital transferred to Profit amp Loss Appropriation Ac)

3 for Interest on Drawings

For transferring Interest on Drawings Interest on Drawings Achelliphelliphelliphelliphelliphellip DrTo Profit and Loss Appropriation Ac(Interest on drawing transferred to Profit amp Loss Appropriation Ac)

4 For Partnerrsquos SalaryFor transfer of partnerrsquos SalaryProfit and Loss Appropriation Achelliphellip DrTo Salary Ac(Salary transferred to profit amp Loss Appropriation Ac)

5 For Partnerrsquos CommissionFor transferring commissionProfit and Loss Appropriation Achelliphelliphellip DrTo Commission Ac(Commission transferred to Profit and Loss Appropriation Ac)

6 For Transfer of agreed amount to General ReserveProfit and Loss Appropriation Ac helliphellipDrTo General Reserve Ac(Transfer to General Reserve)

7 for share of Profit or Loss appropriation(a) If ProfitProfit and Loss Appropriation Achelliphellip DrTo Partnerrsquos CapitalCurrent Ac(Profit transferred to capitalcurrent Ac)(b) If LossPartnerrsquos Capital Current Achelliphelliphelliphellip DrTo Profit and Loss Appropriation Ac(Loss transferred to capitalcurrent Ac)

THE FORMAT OF PROFIT AND LOSS APPROPRIATION

Profit and Loss Appropriation Account for the year endedhelliphelliphelliphellip

Particulars Amount Particulars Amount

To PL Ac (loss) By pL Ac (profit)

To Interest on capital BY Interest on drawings

To partner`s commission by Partner`s capital Ac ( loss)

To Partner`s salary To Interest on partner`s loan To General Reserve To Partner`s Capital AC (Profit)

Subject= Economics

MOVEMENT ALONG THE DEMAND CURVE (CHANGE IN QUANTITY DEMANDED)In law of demand you have already studied the inverse relationship between price and quantity demanded When quantity demanded of a commodity changes due to change in its price keeping other factors constant it is called change in quantity demanded It is graphically expressed as a movement along the same demand curve There can be either a downward movement or an upward movement along the same demand curve Upward movement along the same demand curve is called contraction of demand or decrease in quantity demanded and downward movement along the same demand curve is known as expansion of demand or increase in quantity demanded

Extention of demandd

price (rs)p A

B Extentionp1 d

Q Q1

Quantity demanded ( in units)

Contraction of demandd

p2 Ccontraction

p APrice (Rs)

d

Q2 Q

Quantity demanded (in units)

Explanation of movement of demand A fall in price from OP to OP1 leads to increase in quantity demanded from OQ to OQ1 (expansion of demand) resulting in a downward movement from point A to point B along the same demand curve DD When Price rises from OP to OP2 quantity demanded falls from OQ to OQ2 (contraction of demand) leading to an upward movement from point A to point C along the same demand curve DD

  • Activity Series of Metals
    • Drawbacks of Rutherfordrsquos model of atom
      • Electromagnetic radiations
      • Properties of electromagnetic radiations
      • Characteristics of electromagnetic radiations
        • Plancks Quantum Theory-
        • Photoelectric effect
          • Intext Questions
            • Logic gates
            • Digital systems are said to be constructed by using logic gates These gates are the AND OR NOT NAND NOR EXOR and EXNOR gates The basic operations are described below with the aid of truth tables
            • AND gate
            • Example
Page 10:  · Web viewSubject. Topic. Summary. Execution. English 1 . Chapter 1 naming words . Page 8. Write the names of these pictures:- Person:-1. father. 2.Firefighter 3.doctor 4 ...

11 -ीवारकसाथलगतगोहपजोकसहार-ीवारपकडलतीह12 बोधराजअभीभीटकटकीबाधचीलकीओर-खरहाथा13 बोधराजअपनीजबमबहतसाचगगाभरकरलायाथा14 मरनिपताजीकीतरककीहईऔरहमलोगएकबडघरमजाकररहनलग15 बागमजातातोफलपरबठीनिततलीको-खनिततलीकोपकडकरउगकतिलयोकबीचमसल-ता

BENGALI(2ND LANGUAGE)

সহিনধসবরপওসবরসহিনধ

সবরসহিনধরহিনয়ম- ৯ই-কারহিকংাঈ-কাচেররপচেরইাঈহিভননঅনযসবররণCথাকচেইাঈসথাদেনয-ফায়এংওইয- ফাপCচেরণCযকতয়

১০উ-কারহিকংাঊ-কাচেররপচেরইাঈহিভননঅনযসবররণCথাকচেউাঊসথাদেন-ফায়এংওই- ফাপCচেরণCযকতয়

১১ঋ- কাচেররপচেরঋহিভননঅনযসবররণCথাকচেঋসথাচেনর -ফায়এংওইর পCচেরণCযকতয়

১২সবররণCপচেরথাকচেপCতMএ-সথাচেনঅয় ঐ- সথাচেনআয় ও- সথাচেনঅএংঔ- সথাচেনঅায়

৯ই+ অ= য- ফাআহি+ অনত= আযনত অহিধ+ অয়ন= অধযয়নই+ আ=য- ফা+ াইহিত+ আহি= ইতযাহি পরহিত+ আতC ন= পরতযাতC নই+ উ=য- ফা+ উঅহিত+ উহিকত= অতযহিকত হি+ উৎপহিতত= যৎপহিততই+ ঊ= য- ফা+ ঊ ই+ এ= য- ফা+ এঈ+ অ= য- ফা পরহিত+ ঊ4= পরতয4 পরহিত+ এক= পরচেতযকঈ+ অ আ= য- ফা+ অ আনী+ অমব= নযমব মসী+ আধার= মসযাধার

১০উ+ অ= অন+ অয়= অনবয় পশ+ অধম= পশবধমউ+ আ= াস+ আগত= সবাগত পশ+ আহি= পশবাহিউ+ ঈ= হিঅন+ ইত= অহিনবতউ+ এ= দেঅন+ এ4রণ= অচেনব4রণউ+ ঈ= ীসাধ+ ঈ= সবাধবী তন+ ঈ= তনবী

১১ঋ+ অ= র মাত+ অনমহিত= মাতরনমহিতঋ+ আ= রা হিপত+ আয়= হিপতরায়ঋ+ ই= হির মাত+ ইচছা= মাতচছাঋ+ ঈ= রী ধাত+ ঈ= ধাতরীঋ+ উ= র ভরাত+ উপচে(= ভরাতরপচে(

১২এ+ অ= অয় দেন+ অন= নয়নঐ+ অ= আয় গৈগ+ অক= গায়কও+ অ= অ দেপা+ অন= পনও+ ই= অ দেপা+ ইতর= পহিতরও+ এ= অ দেগা+ এ4রণা= গচে4রণাঔ+ অ= অা দেপৌ+ অক= পাকঔ+ ই= অা দেনৌ+ ইক= নাহিকঔ+ উ= অা দেভৌ+ উক= ভাক

MATHS Topic NumbersChapter Natural numbers and whole numbers

Study item properties of whole numbers for multiplication

1 Closure property If x and y are two whole numbers then xtimesy is also a whole numberExample If x = 9 and y =3 then xtimesy = 9times3 = 27 which is a whole number

2 Commutative property If x and y are two whole numbers then xtimesy = ytimesxExample If x = 5 and y = 2 then xtimesy = 5times2 = 10y times x = 2times5 = 10Therefore 5times2 = 2times5

3 Associative property If x y and z are three whole numbers then x times(ytimesz) = (xtimesy) times zExample If x =3 y = 5 and z = 7 then 3 times (5times7) = 3 times (35) = 105And (3times5) times7 = (15) times 7 = 105Therefore x times (ytimesz) = (xtimesy) timesz

4 Distributive property If x y and z are three whole numbers then xtimes (y + z) = x times y + x times z

Therefore the multiplication of whole numbers is distributive over their additionExample If x = 5 y = 3 and z= 2Therefore x times (y + z) = 5 times (3 + 2) = 5times5 =25And x times y + xtimes z = 5times3 +5times2 =15 +10 = 25Again x times (y ndash z ) = x times y ndash x timesz Therefore 5 times ( 3 - 2) = 5 times1 = 5 and 5times3 ndash 5 times2 = 15 ndash 10 = 5Therefore the multiplication of whole numbers is also distributive over their subtraction if y is greater than z

5 Existence of identity If x is a whole number then

X times1 = x 1 times x = xTherefore we can write x times1 = 1 times xTherefore the multiplication of any whole number with 1 is the number itselfTherefore we can say that 1 is multiplicative identity or identity element for multiplicationExample 5 times1 = 5 1 times 5 = 5 Therefore 5 times 1 = 5

6 Multiplicative inverse If x is any whole number ( x is not equal to zero ) then its multiplicative inverse will be 1xSo x times 1x = 1 but 1x is a whole number if x = 1For other values of whole number 1x is not a whole number therefore we can write its multiplicative inverse does not exists

7 Cancellation law of multiplication If x y and z are three non- zero whole numbers then x times y = x times z

Or y = zExample 9 times y = 9 timeszTherefore y = z

Class VIISubject Topic Summary Execution

English 2 Sentences based on meanings

Kinds of sentences

Assertive or declarative to convey information or simply make a statement

Interrogative to ask different types of questions

Imperative to command or instruct someone or make a request

Exclamatory to express strong feelings and emotions

Exercise c1 What a nice compliment that is

That is a nice compliment2 How well- behaved the children

areThe children are very well-behaved

3 What great chefs we areWe are great chefs

4 What a shame it isIt is a shame

5 What a fantastic idea you haveYou have a fantastic idea

Homework 6 -10English

LiteratureThe Listeners III) Answer the following questions-

d) Identify two words used in the poem to give the poem an eerie atmosphereAns- Two words used to give the poem an eerie atmosphere are ldquogreyrdquo and ldquophantomrdquo

e) Who do you think are the inmates of the houseAns- I think the inmates of the house are phantom who once used to dwell in it

f) Why was the poet ldquoperplexed and stillrdquoAns- He was lsquo perplexed and stillrsquo because he was expecting an answer from the inmates of the house But despite of repeated calls there was no response

CHEMISTRY Chapter 2 ndashElement and Compound

Activity Series of MetalsThe activity series is a chart of metals listed in order of declining relative reactivity The top metals are more reactive than the metals on the bottomMetal SymbolReactivity

Lithium Li displaces H2 gas from water steam and acids and forms hydroxides

Potassium K

Strontium Sr

Calcium Ca

Sodium Na

Magnesium Mg displaces H2 gas from steam and acids and forms hydroxides

Aluminum Al

Zinc Zn

Chromium Cr

Iron Fe displaces H2 gas from acids only and forms hydroxides

Cadmium Cd

Cobalt Co

Nickel Ni

Tin Sn

Lead Pb

Hydrogen gas

H2 included for comparison

Antimony Sb combines with O2 to form oxides and cannot displace H2

Arsenic As

Bismuth Bi

Copper Cu

Mercury Hg found free in nature oxides decompose with heating

Silver Ag

Palladium Pd

Platinum Pt

Gold Au

Answer the following Q)Difference Between Metals And Nonmetals

Metals Nonmetals

These are solids at room temperature except mercury

These exist in all three states

These are very hard except sodium

These are soft except diamond

These are malleable and ductile

These are brittle and can be breakdown into pieces

These are shiny These are non-lustrous except iodine

Electropositive in nature Electronegative in nature

Have high densities Have low densities

Math Number System

Chapter Fraction

Study item Some solved sums from exercise 3(B)1) For each pair given below state whether it from like fractions or unlike

fractions (i) 58 and 78

= Like Fraction because denominators same(ii) 815 and 821

= Unlike Fraction because denominators are not same

(iii) 49 and 94 = Unlike Fraction

2) Convert given fractions into fractions with equal denominators(iii) 45 1720 2340 and 1116Solution Given fraction 45 1720 2340 and 1116Therefore the LCM of 5 20 40 and 16 is 80Therefore 45 = 4times165times16 = 64801720 = 17times420times4 = 68802340 = 23times240times2 = 4680 1116 = 11times516times5 = 5580

3) Convert given fractions into fractions with equal numerators(iii) 1519 2528 911 and 4547Solution Given fractions 1519 2528 911 and 4547Therefore the LCM of 15 25 9 and 45 is 2251519 = 15times1519times15 = 225285 2528 = 25times928times9 = 225252911 = 9times2511times25 = 2252754547 = 45times547times5 = 225235

4) Put the given fractions in ascending order by making denominators equal

(iii) 57 38 914 and 2021Solution Given fraction 57 38 914 and 2021Therefore the LCM of the denominators is 16857 = 5times247times24 = 12016838 = 3times218times21 = 63168914 = 9times1214times12 = 1081682021 = 20times821times8 = 160168Therefore ascending order 63168lt108168lt120168lt160168Therefore ascending order of given fractions38lt914lt57lt2021

COMPUTER CHAPTER-1COMPUTER FUNDAMENTALS

DONE IN THE PREVIOUS CLASSES PAGE 10CWRITE TRUE AND FALSE

1 True2 False3 False4 False5 True

GEOGRAPHY CHAPTER 7EUROPE

CHAPTER COMPLETE 1)Europe is home to a famous mountain range called the Alps

2)River Rhine originates in Switzerland

3)The Eiffel Tower one of the tallest structures in Europe

4) Vatican City is one of the most densely populated European countries

5)Sognefjordin Norway is the largest fjord in Norway

Class VIIISubject Topic Summary Execution

MATHEMATICS Ch 6Sets

Exercise 6 (D)1 Given A = x x isin N and 3iquest x le 6 and B = x x isin W and xlt4 find (i) Sets A and B in roster form (ii) A cup B (iii)

A cap B(iv) A ndash B (v) B ndashA

Solution (i) A = 456 and B = 0123

(ii) A cup B = 0123456 (iii) A cap B = ϕ (iv) A ndash B = 456 (v)B ndash A = 0123

3 If A = 56789 B = x 3 lt x lt 8 and x isin W and C = x xle5 and x isin N Find (i) A cup B and (A cup B) cup C (ii) B

cup C and A cup ( B cup C)

(iii) A cap B and (A cap B) cap C (iv) B cap C and A cap (B cap C)

Is (A cup B) cup C = A cup (B cup C)

Is (A cap B) cap C = A cap (B cap C)

SolutionA = 56789 B = 4567 C = 12345

there4 (i) A cap B = 456789 and (A cup B) cup C = 123456789

(ii) B cup C = 1234567 and A cup ( B cup C) = 123456789

(iii) A cap B = 567 and (A cap B) cap C = 5

(iv) B cap C = 45 and A cap (B cap C) = 5

Now (A cup B) cup C = 123456789

And A cup ( B cup C) = 123456789 there4 (A cup B) cup C = A cup (B cup C)

Again (A cap B) cap C = 5 and A cap (B cap C) = 5

there4 (A cap B) cap C = A cap (B cap C)

4 Given A = 012345 B = 02468 and C = 0369 Show that (i) A cup (B cup C) = (A cup B) cup C ie the union

of sets is associative (ii) A cap (B cap C) = (A cap B) cap C ie the intersection of sets is associative

SolutionNow B cup C = 0234689 and A cup B = 01234568

there4 A cup (B cup C) = 012345689 and

(A cup B) cup C = 012345689

So (i) A cup (B cup C) = (A cup B) cup C ie the union of sets is associative

Again B cap C = 06 and A cap B = 024

there4 A cap (B cap C) = 0 and (A cap B) cap C = 0

So (ii) A cap (B cap C) = (A cap B) cap C ie the intersection of sets is associative

Physics Chapter 2 Physical Quatites and Measurements

Here We Will Do Some QuestionsRelated To Chapter 2

A density bottle has a marking 25 mL on it It means that

1 the mass of density bottle is 25g

2 the density bottle will store 25 ml of any liquid in it

3 the density bottle will store 25 ml of water but more volume of liquid denser than water

4 the density bottle will store 25 ml of water but more volume of a liquid lighter than water

Solution 2 the density bottle will store 25 ml of any liquid in it

COMPUTER CHAPTER-2Spreadsheet Functions and Charts

SELECTING RANGE IN ROWSCOLUMNSWHEN TWO OR MORE CELLS ARE SELECTED IT IS CALLED A RANGEA RANGE OF CELLS CAN BE FORMED IN TWO WAYS--a) SELECTING RANGE BY USING THE MOUSEb) SELECTING RANGE BY USING THE KEYBOARD

Q1)WRITE THE STEPS TO SELECT PARTIAL RANGE IN A ROW

Ans)THE STEPS ARE-6 SELECT THE ROW7 BRING THE CELL POINTER TO THE DESIRED

LOCATION FROM WHERE YOU WANT TO START YOUR SELECTION

8 CLICK THE LEFT MOUSE BUTTON AND KEEP DRAGGING TO YOUR RIGHT TILL YOU REACH THE LAST CELL TO NE SELECTED

RELEASE THE MOUSE BUTTON

GEOGRAPHY Asia

CLIMATE

Asia experiences great extremes of climate Jacobabad in the Sind province of Pakistan is one of the hottest places in the WorldVerkhoyansk in Siberia is one of the coldest places in the WorldCherrapunji and Mawsynram in India are two wettest places in WorldArabia Tibet Gobi and Mongolia are extremely dry regionsFactors Affecting Climate of Asia-The factors influencing the climate of Asia are-

Factors Affecting Climate of Asia-Thoroughly read the table in page number 60

Latitudinal extent

Continentality

Relief features

Presence of low pressure trough

Jet streams

English Language The Sentence A complex sentence contains one independent clause and at least one dependent clause The dependent clause in a complex sentence is introduced with subordinating conjunctions or relative pronouns

Commonly Used Subordinating Conjunctions-Time after before while when since untilCause And Effect because now since as in order that soOpposition although though even though whereas while in spite ofCondition if unless only if whether or not even if in case(that)

Commonly Used Relative Pronouns-Who whose whom which whoever whomever whichever that

Class IXSubject Topic Summary Execution

1-BENGALI(2ND LANGUAGE)

ldquo বঙগভমিরপরমিrdquo াইকেলধসদনদতত

আচেগর পর উততর পচো-১ ২ ৩ এং নীচের পর টি াহির কা- ৪মহিbকাও গচেনা দেগা পহিচে অমত হরচে- ক) কার দো দেকান কহিতার অং( ) কতা দেক পরসঙগ কী উহিকতটির তাৎপযC আচোনা কচেরা৫দেসই ধনয নরকচে দোচেক যাচে নাহি ভচে মচেনর মহিeচের সাচেসচে সCন ক) কহির কায C ার উচেf(য হিক হিছ কহি কন কহিতাটি দেচেন) কহি কার কাচেছ হিমনহিত কচেরচেছনগ) কহি এই পহিথীচেত কাচের ধনয মচেন কচেরনঘ) কহি হিক রকম অমর তাাভ করচেত ান

Hindi 2nd lang

काकी(कतिसयारामशरण गपत)

इस कहानी म लखक न यह बतान का परयास निकया ह निक बचच अपनी मा स निकतना परम करत ह शयाम अबोध बालक ह वह अपनी मा क मरन क बा- उसन अपनी मा क कतिलए बहत रोया बा- म उस पता चला निक उसकी मा राम क घर चली गई ह आकाश म उडती हई पतग -खकर उस हरष हआ निक पतग क दवारा वह अपनी मा को नीच उतारगा इसक कतिलए वह अपनी निपता की जब स -ो बार सवा रपया निनकालकर पतग और -ो मोटी सी मन वाली अपन भाई स काकी एक कागज पर कतिलखवा कर पतग म कतिशव का दि-यानिनकालकर पतग और -ो मोटी सी मन वाली अपन भाई स काकी एक कागज पर कतिलखवा कर पतग म कतिचपका दि-याभोला और शयाम कोठरी म रससी बाधनी रह थ तभी उसक निपता करोध म आकर उन स पछ निक कया उनकी जब स रपया निनकाला हभोला डर क मार बताया निक शयाम इस पतग क दवारा अपनी काकी को राम क यहा स उतारना चाहता हनिवशशवर(शयाम क निपता)न फटी पतग उठाकर -खी तो उस पर काकी कतिलखा थावह हत बजिa होकर वही खड रह गएउनहोन सोचा निक मन अपन पतर को मारा जोनिक अनजान और निन-dरष थावह अपनी मा कोनिकतना पयार करता ह

उस दि-न बड सवर शयाम की नी- खली तो -खा निक घर भर म कोहराम मचा हआ ह

क) घर म कोहराम कयो मचा हआ था शयाम को कया लगा

ख) काकी को ल जात समय शयाम न कया उपदरव मचाया

ग) काकी क बार म उस कया बताया गया कया सतय उस कतिछपा रहा

घ) वह बठा-बठा शनय मन स आकाश की ओर कयोकरता

उततरक) शयाम की मा का -हात हो गया था इसकतिलए

घर म कोहराम मचा हआ था शयाम की लगा निक उसकी मा सफ- कपडा ओढ हए भमिम पर सो रही ह

ख) लोग जब उमा यानी शयाम की मा को उठाकर ल जान लग तब शयाम न बडा उपदरव मचाया लोगो क हाथ स झठ करवा उमा क ऊपर जा निगरा और बोला काकी सो रही ह उस कहा ल जा रह हो

ग) काकी क बार म बजिaमान लोगो न उस निवशवास दि-लाया निक उसकी का निक उसक मामा क यहा गई ह लनिकन सतय अमिधक दि-नो तक कतिछपाना रह सका आसपास क अबोध बालको क मह स यह बात परकट हो गई निक उसकी मा का -हात हो गया ह

घ) कई दि-नकई दि-न लगातार रोत-रोत उसका रोना तो शान हो गया पर उसक ह-य म शोक भर गया था वह चपचाप बठा आकाश की और टाका करता निक शाय- उसकी काकी कही दि-ख जाए

ldquoदि-न उसन ऊपर आसमान म पतग उडती -खी न जान कया सोच कर उसका निहर-य एक-म खिखल उठाrdquo

क) निकसन पतग ऊपर उडत -खी और वह कयो खश हआ

ख) उसन अपन निपता स कया कहा उनका कया उतर थाश

ग) उसन निफर कया निकया और निकसन उसकी सहायता की

घ) उसकी योजना कया थी उततर -क) शयाम न एक दि-न आसमान म पतग उडती

-खी तो उसन सोचा निक पता आसमान म राम क यहा जाकर रकगी वही पर मरी काकी ह यह सोचकर वह बहत खश हआ

ख) उसन अपन निपता स कहा काका मझ एक पतग मगा -ो उसक निपता न भटक हए मन क भाव स कहा निक मगा -ग यह कह कर उ-ास भाव स वह कही और चल गए पतग नही आई

ग) उसन चपचाप निवशशवर क टगहए कोट स एक चवननी निनकाल ली और सखिखया -ासी क लडक भोला की सहायता स एक पतग मगवानी भोला उसकी बराबर उमर का ही था

घ) उसकी योजना यह थी निक वह अपनी पतग को आकाश म राम क यहा भजगा और उस पतग क सहार उसकी काफी नीच उतर जाएगी इस योजना पर उस परा निवशवास था इसकतिलए वह और भोला -ोनो यह काम करन म लग गए

Continue to nexthelliphellipEVS CHAPTER - 1

(UNDERSTANDING OUR ENVIRONMENT)

Sustainable development

The development that meets the needs of the present without compromising the ability of future generations to meet their own needs is called Sustainable development

Sustainable societies ndash

An environmentally sustainable community is one that meets the current and future basic resource needs of its people in a just and equitable manner without compromising the ability of future generations to meet their basic needs

Q ) What are Eco Villages

Ans - Eco village are the urban or rural communities of people who strive to integrate a supportive social environment with a low impact way of life

Q ) To ensure sustainable development the depletion of renewable resources should not take place at a rate faster than their regeneration Justify your answer

Ans ndash Renewable resources do not have a fixed quantity - more can always be

generated However if the rate of use exceeds the rate of renewal - that is the

source is used more than its being recreated - its continued use will become

used up faster than it can regenerate

To promote sustainable society the following things need to be done ndash

1 Using renewable energy sources 2 By improving the quality of human

health 3 By promoting sustainable agriculture 4 By forming ecovillage

it will eventually be entirely depleted So Toensure sustainable development the depletion of

renewable resources should nottake place at a rate faster than their regeneration

Q ) What do you mean by Sustainable societies

Ans - Sustainable societies are defined as towns and cities that have taken steps to remain healthy over the long term These communities value healthy ecosystems use resources efficiently and actively seek to retain and enhance a locally based economy Sustainable development concerns everybody in a society

Q ) What are the effects of pollution on human health

Ans ndash Some health problem occurs due to air pollution are ndash

Respiratory diseases Cardiovascular damage Fatigue headaches and anxiety Irritation of the eyes nose and throat Damage to reproductive organs Harm to the liver spleen and blood Nervous system damage

Some health problem occurs due to water pollution are ndash

Typhoid Cholera Dysentry Jaundice

Some health problem occurs due to noise pollution are ndash

Fatigue headaches and anxiety High blood pressure Hearing damage

Physics Motion in 1D First go through previous notes Now here we will solve some numerical related to that

Question 3What information about the motion of a body is obtained from the displacement-time graphSolution 3From displacement-time graph the nature of motion (or state of rest) can be understood The slope of this graph gives the value of velocity of the body at any instant of time using which the velocity-time graph can also be drawn

Question 4(a)What does the slope of a displacement-time graph represent(b)Can displacement-time sketch be parallel to the displacement axis Give a reason to your answerSolution 4(a) Slope of a displacement-time graph represents velocity(b) The displacement-time graph can never be parallel to the displacement axis because such a line would mean that the distance covered by the body in a certain direction increases without any

increase in time which is not possible

Chemistry Language of Chemistry

How to balance a chemical equationThere are two methods of balancing an equation(i)Hit and trial method(ii)Partial equation methodBalancing by hit and trial methodThis method consists of counting the number of atoms of each elements on both sides and trying to equalize themTake the following steps(i)Count the number of times (frequency) an element occurs on either side(ii)The element with the least frequency of occurrence is balanced first(iii)When two or more elements have the same frequencythe metallic element is balanced firstExample-1 On heatinglead nitrate decomposes to give lead dioxidenitrogen dioxide and oxygenPb(NO3)2rarrPbO+NO2+O2

In this equationLead occurs twiceNitrogen occurs twiceOxygen occurs four timesSince lead is a metalbalance it firstThe number of atom of lead is equal on the two sidestherefore it needs no balancingNow balance nitrogenOn the reactant sidethere are two atoms of nitrogenwhile on the product side oneSomultiply the product containing nitrogenon the product sideby two Pb (NO3)2rarrPbO+2NO2+O2Nowthe number of oxygen atoms on the reactant side 6while on the product sideit is 7Somultiply the entire equation by 2except oxygen to get balanced equation2Pb(NO3)2rarr2PbO+4NO2+O2Multiplication by 2 is done only when atoms of all the elements except one element are balanced and the unbalanced atom occurs separately at least once and also there is a difference of only one such atom

Math Topic AlgebraChapter

Factorisation

Study item Difference of two squares a2 ndash b2 = (a+b) (a-b)1) (i) 4x2ndash 25y2

= (2x) 2 ndash (5y) 2= (2x + 5y) (2x - 5y)

(ii) 9x2 ndash 1= (3x)2ndash(1)2= (3x + 1)(3x ndash 1)

2) (i) 150 ndash 6a2= 6(25 ndash a2)= 6(5)2 ndash(a)2= 6 (5 + a) (5 ndash a)

(ii) 32x2 ndash 18y2=2(16x2 ndash 9y2)=2(4x)2 ndash (3y)2= 2(4x + 3y)(4x - 3y)3)(i) (x ndashy )2 ndash 9 = (x ndash y )2 ndash (3)2= (x ndash y + 3) (x ndash y ndash 3)(ii) 9(x + y) 2ndash x 2= (3)2(x + y)2 ndash (x)2=3(x + y)2 ndash (x)2= (3x +3y ) 2ndash(x)2= (3x + 3y + x)(3x +3y ndash x)= (4x + 3y) ( 2x + 3y )

Commercial studies

Basic accounting terms

Today I will give you some questions from the previous study material

Questions1) Define accounting2) What do you mean by debit and

credit

3) Explain the types of account4) Define the following terms

a) Assetsb) Capitalc) Purchased) Debtorse) Transactions

5) Name the types of accounts given below

a) Krishnas accountb) Machinery accountc) Royalty accountd) Salary accounte) Furniture accountf) Audit fee account

Economics Revision Today I will give you some revision questions

Questions1) What do you mean by the terms

rdquowantsrdquo2) Write the difference between

consumer goods and producer goods

3) Define the term utility 4) Explain the different types of utility5) Define

a) Total utilityb) Marginal utility

Subject Eng Literature (The Merchant of Venice ndash William Shakespeare)Topic Act I Scene 3 Lines 1 to 48 (Shylock hellip Cursed be my tribe if I forgive him) Date 16th April 2020 (5th Period)

[Students should read the original play and also the paraphrase given in the school prescribed textbook]Summary Questions amp Answers

This scene takes place in Venice and we are introduced to the rich Jew Shylock Bassanio and Shylock are talking and Bassanio tells Shylock that he wants a loan of three thousand ducats for three months on the personal security of Antonio

o Shylock feels glad because he will be able to bind down Antonio by means of a bond on account of the loan but he tells Bassanio that all the fortunes of Antonio being invested in the merchant ships on the sea it is difficult to depend upon his credit Even under such circumstances Shylock is willing to advance the money on the personal security of Antonio

o Bassanio then invites Shylock to dine with him Shylock says that he is prepared to do anything with the Christians but not eat or drink or pray with them

o While Bassanio and Shylock are talking Antonio appears on the scene Shylock does not seem to take any notice of Antonio but goes on brooding within

(1) SHYLOCK Ho no no no no- my meaning in (Line 15-26)saying he is a good man is to have you understand me that he is sufficient Yet his means are in suppositionhe hath an argosy bound to Tripolis another to the Indies I understand moreover upon the Rialto he hath a third at Mexico a fourth for England and other ventures he hath squanderd abroad Butships are but boards sailors but men there be land-rats and water-rats land-thieves and water-thieves I mean pirates and then there is the peril of waters winds and rocks The man is notwithstanding sufficientmdashthree thousand ducats mdashI think I may take his bond

(a) Who is talking in the beginning of this scene What does Bassanio want from Shylock How does Shylock feel

In the beginning of the scene Bassanio and Shylock are talking to each other Bassanio wants to borrow three thousand ducats from Shylock for three months on the security of Antonio Shylock feels glad at heart that he will get the opportunity of binding Antonio with a bond(b) What risks does Shylock weigh in advancing the money

Shylock says that Antonio has invested all his capital in trading by sea-going ships But the ships are made of wood and the sailors of those ships are ordinary human beings The wood can

himself how he hates Antonio because of his being a Christian because he abuses Shylock in public places Shylock decides that if ever he can get Antonio to his advantage he will teach him a lesson

come to harm and men can commit mistakes and thus the capital invested in ships may be lost Then there are other dangers The goods loaded on the ships can be damaged by rats and thieves which are found both on land and water The ships can also be harmed through sea-storms submerged rocks etc(c) What two important functions does this scene have

The scene has two important functions First it completes the exposition of the two major plot lines of the play Antonio agrees to Shylockrsquos bond ndash three thousand ducats for a pound of flesh and second and more important dramatically this scene introduces Shylock himself In this scene Shakespeare makes it clear at once why Shylock is the most powerful dramatic figure in the play and why so many great actors have regarded this part as one of the most rewarding roles in all Shakespearean dramas(d) Where does this scene take place What kind of treatment has Antonio been giving to Shylock What does Shylock say when Bassanio invites him to dine with him

The action of this scene takes place in Venice Antonio has been in the habit of behaving harshly with Shylock ndash spitting on his beard and footing him like a stranger cur When Bassanio invites Shylock to dine Shylock says that he is prepared to do anything with the Christians but not eat and drink or pray with them

(2) SHYLOCK How like a fawning publican he looks (Line 38-48)I hate him for he is a Christian

But more for that in low simplicity

He lends out money gratis and brings downThe rate of usance here with us in VeniceIf I can catch him once upon the hipI will feed fat the ancient grudge I bear him

He hates our sacred nation and he railsEven there where merchants most do congregateOn me my bargains and my well-won thriftWhich he calls interest Cursed be my tribeIf I forgive him

(a) What is the context in which these words are spoken and what is the idea expressed in it

These remarks are made by Shylock when he sees Antonio coming along after Bassanio told him that the merchant will be his surety for the bond The above mentioned passage reveals Shylockrsquos hatred for Antonio Shylock says that he hates Antonio because he is a Christian and also because he gives loan without taking interest on them thereby bringing down the rate of interest in Venice(b) Explain the meaning of the phrase lsquoa fawning publicanrsquo

The phrase lsquoa fawning publicanrsquo refers to Roman tax collector It is a term of contempt and hatred on the lips of a Jew lsquoFawning Publicansrsquo were Roman tax-gatherers whose ordinary bearings towards the Jews was bullying but whose false pose of lsquohumility and contritionrsquo is touched upon in the parable in New Testament(c ) What light does the above passage throw on the character

of Shylock

The above mentioned speech of Shylock reveals him to be a wicked character having an extreme greed for wealth His intense hatred for Antonio is unjustified He hates Antonio only because he is a Christian and because he lends money without taking any interest on it thereby adversely affecting Shylockrsquos business of lending money on high interest(d) What information do you gather about Antonio from the above given lines

Shylockrsquos statement throws a valuable light on the character of Antonio Antonio appears to be a good Christian and a good human being He helps the people in need by lending them money without charging any interest on it He is a man of simple and good nature This very goodness makes him Shylockrsquos enemy(e) What does Shylock debate within himself and when To whom are the lines mentioned above addressed to

When Bassanio asks the Jew to lend him three thousand ducats on Antoniorsquos surety Shylock begins to debate within himself as to how he should exploit the opportunity of a business deal with his old enemy Antonio

The lines mentioned above are not addressed to anyone The lines are a soliloquy ie a speech made by a character to himself and not meant to be heard by the other characters present

Class XSubject Topic Summary ExecutionEnglish

LiteratureThe Blue Bead 2nd part

Things took a turn for the worst and all of a sudden a crocodile attacked the woman biting on the womanrsquos leg At that moment Sibia got up sprinted grabbed the hay fork and stabbed the crocodile in the eye with all her power Immediately the crocodile let go and went away Sibia saw a small blue bead lying by the river she grabbed it Since she was poor she didnrsquot have necklace Shersquod always wanted one like the other women now she could make one with the blue bead After that she went home and told her mother all about it

Hindi 2nd

Langबड घर की बटी( मशी परमच-)

lsquoबड घर की बटी कहानी का उददशय मधयम वग की घरल समसया को सलझा कर सगदिठत परिरवार म मिमल जलकर परम स रहन का स-श -ना हघर म शानित सथानिपत करन की जिजमम-ारी नारी की होती ह यदि- नारी समझ-ार ह उसम धय और परिरवार क परनित परम ह तो कोई भी घटना परिरवार को निवघदिटत नही कर सकती या कहानी परिरवार को सगदिठत करत हए परम सौहा- स एक दसर की भावनाओ को समझ करउनका सहयोग करत हए जीवन यापन करन की पररणा -ती हमशी परमचदर जी न इस कहानी म सय परिरवार का परनितनिनमिधतव निकया ह यह कहानी बनी माधव सिसह जो गौरी पर क जमी-ार क उनक -ो पतरो की हशरी कठ लाल निबहारीशरीकात का निववाह एकजमी-ार घरान की पतरी आन-ी स हआ थाआन-ी न ख- को ससराल क वातावरण म ढाकतिलया थाएक दि-न आन-ी का अपन -वर लाल निबहारी स झगडा हो जाता ह -ोनो भाई एक दसर स अलग होन की कोकतिशश करत हसभी बह आन-ी न अपन मधर वयवहार स लाल निबहारी को

ldquoइन नतर निपरय गणो को बीए-इनही -ो अकषर पर नयोछावर कर दि-या था इन -ो अकषर न उनक शरीर को निनबल और चहर को कानित ही बना दि-या थाldquo

क) परसतत पकतियो म निकस वयकति क बार म कहा गया ह

ख) इन पकतियो म कौन स नतर निपरय गणो क बार म कहा गया ह

ग) बीए की निडगरी परापत कर लन पर भी उपय वयकति क सवभाव की कया निवशरषता थी

घ) यह नतर निपरय गण निकस वयकति म निवदयमान थ उसक वयकतितव की कया निवशरषता थी

उततर ndashक) परसतत पकति म गौरी पर गाव क जमी-ार

क बड बट शरीकात क बार म कहा गया ह उसन बहत परिरशरम और उ-म क बा- ba की निडगरी परापत की थी अब वह एक -फतर

घर छोडकर जान स रोक कतिलयाइस पर बनी माधव सिसह न कहा निक बड घर की बटी ऐसी ही होती ह जो निबगडा काम बना लती ह अतः शीरषक साथक ह बड घर की बटी आन-ी ह

म कमचारी थाख) भरा हआ चहरा चौडी छाती और डटकर

खाना आदि- एक सबजी ल जवान क गण मान जात ह परत शरीकात न इनही नतर निपरय गणो को अपनी पढाई पर नयोछावर कर दि-या था

ग) बीए की निडगरी परापत कर लन पर भी उपय वयकति(शरी कठ की शारिररिरक तौर पर निनबल और चहर स कानित ही लगत थ इतना ही नही वह मानकतिसक तौर पर भी निपछड हए थ पाशचातय सामाजिजक कथा उस घणा एव पराचीन सभयता का गणगान उनकी निवचारधारा क परमख अग थ

घ) यह नतर निपरय गण गौरीपर गाव क जमी-ार क छोट बट लाल निबहारी सिसह म निवदयमान थ वह सजीलाजवान था और भस का दध शर दध वह सवर उठकर पी जाता था

ldquoयही कारण था निक गाव की लललन आए उनकी निन-क थी कोई कोई तो उह अपना शतर समझन म भी सकोच ना करती थी सवय उनकी पतनी को इस निवरषय म उनस निवरोध थाldquo

क) उपय पकति म इस वयकति क बार म कहा गया ह

ख) गाव की लललन आए उनकी निन-ा कयो निकया करती थी

ग) उनकी पतनी का कया नाम था उनह निकस निवरषय म अपन पनित क निवरa था और कयो

घ) इस कहानी का कया उददशय ह Continue to next helliphelliphellip

Bengali 2nd Language

ফ ফটক না ফটক( কহিতা )

পর) ldquo(ান াধাচেনা ফটপাচেথ পাথচের পাড হিচেয় এক কাঠচোটটা গাছ কহিকহি পাতায় পার ফাটিচেয় াসচেছldquoক) কার দো দেকান কহিতার অং( ) lsquo(ান াধাচেনা ফটপাচেথ পাথচের পাডহিচেয়lsquo চেত কী দোঝাচেনা চেয়চেছ গ) আচো য অংচে(lsquo এক কাঠচোটটা গাছ lsquoচেত কী দোঝাচেনা চেয়চেছ ঘ) ldquoকহিকহি পাতায় পার ফাটিচেয় াসচেছldquo ----- একথার পরকত অথC কী উততর ) ক) আচো য অং(টি পর যাত কহি সভা4 মচোপাধ যাচেয়র দো lsquoফ ফটক না ফটকrsquo কহিতার অং()কহি সভা4 মচোপাধ যায় হিছচেন দেপরচেমর কহি দেপরমচেক নানা ভহিঙগমায় হিতহিন ফটিচেয় তচেচেছন দেপরম মানচের স মচেতC র সঙগী কহিতার কহিতায় এক রb সb হচেয়র দেপরম াগরচেনর কথা চেচেছন (ান অথCাৎ দেযাচেন দেকান রস দেনই দেযাচেন দেকান মহিনতা দেনই অথ তার মধ দেযও দেপরম থাকচেত পাচের একথাই কহি তচে ধরচেত দেচেয়চেছন একটি মানচে4র মচেন দেযাচেন দেকামতার দেকান সথান দেনই পাথচেরর মচেতা হিনরসতার মচেনর মধ দেযও দেয দেপরম আসচেত পাচের দেস কথাই কহি চেচেছনগ)নারীচের যথC দেপরচেমর ছহি এই কহিতায় অকপচেট উচেঠ এচেসচেছ কহি এই কহিতায় কাটচোটটা গাছ কথাটি যার কচেরচেছন নারী দেয দেপরম দেথচেক হিতাহিত এং দেসই দেপরম সঠিক সমচেয় না পাওয়ার ন য দেপরম সমপচেকC হিচেr4 গৈতরী য় দেপরচেমর দেয গৈহি(ষট য মাধযC য সরসতা দেকামত এই সমসতর হিপরীত যথা রbতা শষকতা কচেঠার তা পরভহিত দোঝাচেত এক কাঠচোটটা গাছ কথাটি যার কচেরচেছনঘ) এাচেন এক নারীর যথC দেপরচেমর কথা হিনহিCপত ভাচে চেচেছন কহি অসমচেয় নারীর ীচেন দেপরম দেচেগচেছ এতহিন তার হয় রb কচেঠার হিছ দেপরচেমর অভাচে ঠাৎ দেসই শষক মরভহিমচেত সচের আভাস এচেসচেছ দেপরম দেযন 4Cার স(ীত তাই পরায় মত গাচেছ কহিকহি পাতা গহিচেয় উচেঠচেছ

Biology Chapter - 01Controlling Air Pollution

Today we will discuss how we control air pollution from domestic combustion

Q1Describe any five ways of reducing air pollution from domestic sources bull The number of pollutants in the air is verylarge and we always try to control them byfollowing ways

i) Solar cooker and solar heater It use no fuel reduce damage of environment by fuel use or reducing deforestation It maintains coolness of house It releases very less orno oil gas or grease

ii) Piped natural gas (PNG) It emits very less by products into the atmosphere As it isdistributed through pipe lines so there iscontinuous supply of fuel is possible

iii) Liquefied Petroleum Gas (LPG) It hasa higher heating value LPG doesntcontain sulphur so it burns a lot cleanerenergy sources It releases very less oralmost no fume in air

iv) Electricity based cooking Emission free cooking alternative for urban dwellers causeselimination of adverse health impactsofindoor air pollution It helps to avoid theinconveniences associated with procurement of LPG

v) Biogas It contains 75 methane whichmakes it an excellent fuel It burns without smoke and biogas plant leaves no residue like ash in wood charcoal etc Thus it isaclean fuel

Economics

Factors of Production

Today firstly we would recall the last class for 5 mins and then we would proceed with the further topics of the chapter

The concept meaning of land characteristics of land and importance of land to be repeated for the absentees as well as the students who were there in the class the previous day

Today we will start with the last portion of land before it the meaning of land to be repeated onceAs by now we all know that

Questions1What do you mean by productivity of landAnswer By productivity of land we mean the capacity of a piece of land to produce a crop

Thus it refers to the average output per unit of landSay per acre per hectare etc= (OutputArea of land)

2 What are the factors influencing the productivity of landAnswer

Natural factors Productivity of land is largely determined by the natural

Land is defined to include not only the surface of the earth but also all other free gifts of nature(for example mineral resources forest resources and indeed anything that helps us to carry out the production of goods and services but is provided by nature free of cost)

We will move on to the last portion of land by discussing Productivity of Land

By productivity of land we mean the capacity of a piece of land to produce a crop

Thus it refers to the average output per unit of land

Say per acre per hectare etc= (OutputArea of land)

With this we shall proceed further with the main factors that determine the productivity of land

Natural factors Human factors Improvements on land Location of land Organisation Ownership of land Availability of capital Proper use of land State help

Note economic development of a country depends upon the quality of its land If the land is fertile it will quicken the pace of development of the country

qualities of land such as fertility etc

Human factors Land cannot produce anything by itself Man has to apply labour on it to produce for himself So productivity of land depends on the knowledge and skills of workers

Improvements on land production of land is affected by land development measures like provision of well or tubewell irrigation proper drainage

State help The government of a country especially less developed country can play a vital role in improving the agricultural productivity by providing better irrigation facilities

Organisation Productivity of land also fdepends upon the way how the factors of production like labour and capital are organised

In order to increase productivity trained workers modern implements scientific methods good seeds are all essential

3 lsquoImproved technology affects the productivity of landrsquo Explain this statement with the help of suitable example Answer Use of improved technology raises the productivity of land Example By using HYV seeds chemical manures and modern machines per hectare output increases

Physics Force (Summary)

Question Write the expression for the moment of force about a given axisSolutionsThe expression for the moment of force is given byMoment of force about a given axis = Force times perpendicular distance of force from the axis of rotationQuestion What do you understand by the clockwise and anticlockwise moment of force When is it taken positiveSolutionsIf the effect on the body is to turn it anticlockwise moment of force is called the anticlockwise moment and it is taken as positive while if the effect on the

body is to turn it clockwise moment of force is called the clockwise moment and it is taken as negative

Math Topic Commercial Mathematics

Chapter Goods and services Tax

Study item Some solved sums from exercise ndash 1 A retailer buys a TV from a wholesaler for Rs 40000 He marks the price of the TV 15 above his cost price sells it to the consumer at 5 discount on the marked price If the sales are intra ndash state and the rate of GST is 12 find

(i) The marked price of the TV(ii) The amount which the consumer pays for the TV(iii) The amount of tax (under GST) paid by the retailer to the central

Government(iv) The amount of tax (under GST) received by the State Government

Solution As the sales are intra- state sale and the rate of GST 12 So GST comprises of 6 CGST and 6 SGSTTherefore a retailer buys a TV from a wholesaler for Rs 40000Therefore the amount of GST collected wholesaler from the retailer or paid by retailer to wholesalerCGST = 6 of Rs 40000 = Rs(6100 times40000) =Rs 2400SGST = 6 of Rs 40000 = Rs (6100 times 40000) =Rs 2400Therefore wholesaler will pay Rs 2400 as CGST and Rs 2400 as SGSTTherefore amount of input GST of retailer Input CGST = Rs 2400 and input SGST = Rs 2400Again the retailer marks the price of the TV 15 above his cost price(i) The marked price of the TV

= Rs 40000 + Rs 40000times15= Rs 40000 + Rs 40000times 15100= Rs 40000 + Rs 6000Rs 46000But the retailer sells it to consumer at 5 discount on the marked priceCost price after discount = Rs 46000 ndashRs46000times 5100 =Rs 46000 ndashRs 2300= Rs 43700Therefore the amount of GST collected retailer from consumer or paid by consumer to retailerCGST = 6 of Rs 43700 =Rs ( 6100 times43700)Rs 2622SGST = 6 of Rs 43700 = Rs (6100 times 43700) =Rs 2622Amount of the output GST of retailer Output CGST = Rs 2622 and output SGST = Rs 2622

(ii) The amount which the consumer pays for the TV= cost price of TV to consumer + CGST paid by consumer + SGST paid by consumer= Rs 43700 + Rs 2622 + Rs 2622= Rs 48944

(iii) The amount of tax (under GST ) paid by the retailer to the central Government=CGST paid by retailer = output CGST ndash input CGST=Rs 2622 ndash Rs 2400=Rs 222

(iv) The amount of tax ( under GST ) received by the State Government = SGST paid by wholesaler + SGST paid by retailer= Rs 2400 + output SGST ndash input SGST=Rs 2400 + Rs 2622 ndash Rs 2400=Rs 2400 + Rs 222= Rs 2622

Commercial studies

Stakeholders Today I am going to give some revision questions from the previous study material

Questions1) State the two expectations of

employees from a business concern2) Give two distinctions between

stakeholder and shareholder3) Give two difference between

internal stakeholders and external stakeholders

4) Give two expectations of suppliers from a business organisation

5) Who is a stakeholder in commercial organisations

Chemistry Periodic Table

Merits of Mendeleevrsquos Periodic law are as follows - 1He grouped the elements on the basis of atomic mass 2 He left gaps for undiscovered elements like Gallium Scandium germanium Also he left a full group vacant for undiscovered inert gases 3 He could predict proportions of several elements on basis of their position in periodic table like Ga Sc etc 4He could predict errors in atomic weights of some elements like gold platinum etc

Anomalies in Mendeleevrsquos Periodic law are as follows - 1 Position of isotopes could not be explained 2 Wrong order of atomic masses could not be explained

For example- as Arnur atomic mass 40 come first and K with low atomic mass (30) should come later but k should be placed first

According to Bohrrsquos Modern Periodic table properties of elements are periodic functions of their atomic numbers

So when elements are arranged according to increasing atomic numbers there is periodicity in electronic configuration that leads to periodicity in their chemical properties

It consists of horizontal rows (Periods) Vertical column (Groups)

There are 7 period and 12 groups in this long form of periodic table

Ist period has 2 elements IInd period has 8 elements IIIrd period has 8 elements IVth period has 18 elements Vth period has 18 elements VIth period has 32 elements VIIth period hs rest of elements

Note - The number of valence electrons in atom of elements decides which elements will be first in period and which will be last

In group- 1 to 2 gp and 13 to 17 contain normal elements 3 to 12gp ndash transition elements 57 to 71 - lanthanides 89 to 103 - Actinides

Left hand side ndash metals Right hand side ndash nonmetals

Note- Hydrogen element has been placed at top of Ist group Electronic configuration of H is similar to alkali metal as both have 1 valence electron

V electron of gp I element -- 1 V electron of gp 2 element -- 2 V electron of gp 13 element -- 3 V electron of gp 14 element -- 4 V electron of gp 15 element -- 5 V electron of gp 16 element --6 V electron of gp 17 element -- 7 V electron of gp 18 element -- 8

English 1 Transformation of sentences

Sentences A sentence is a group of words which makes complete sense

Exercise 2Change the following sentences from

a Assertive sentencesb Imperative sentencesc Interrogative sentencesd Exclamatory sentences

Sentences can be changed from one grammatical form to another without changing the meaning of the sentence This is known as transformation of sentences

assertive to interrogative1 Nobody would like to be a fool

Who would like to be a fool2 Their glory can never fade

When can the glory fade3 Nobody can control the wind

Who can control the wind4 It matters little if I die

What though I die5 No man can serve two masters

Can any man serve two masters

Exercise 3Interchange of assertive and Exclamatory sentences

1 She leads the most unhappy lifeWhat an unhappy life she leads

2 This is indeed an interesting bookWhat an interesting book this

3 He is a very great manWhat a great man he is

4 It is a very lame excuseWhat a lame excuse

5 It is sad that she died so youngAlas she died so young

Class XISubject Topic Summary Execution

Hindi 2nd lang

पतर परम(परमचदर) पतर परम कहानी म एक निपता की इचछाओ का वणन निकया गया ह अपन बड पतर परभ -ास स निपता चतनय -ास का निवशरष परम था निपता को उसक जनम स ही बडी-बडी आशाए थी उसम दसर बट कतिशव-ास की अपकषा स- उतसाह की मातरा अमिधक थी वह उस इगलड भजकर बरिरसटर बनाना चाहत थभागय का खल भी बडा निनराला ह बीए की परीकषा क बा- वह बीमार पड गया डॉकटरो न भी जवाब - दि-या थाचतन -ास जी बहत ही कजस थ बवजह पस खच करना नही चाहत थ अगर गारटी मिमलती तो शाय- पस खच भी कर -त परत गारटी नही थी परिरणाम सवरप उनक बट का -हात हो गयाजब बट को समशान ल जा रह थ तो वहा काफी शोर गान बजान हो रह थ पछन पर पता चला निक निकसी निपता निपछल तीन साल स निबमार था और उसक ईलाज म रपया पानी की तरह बहाया पर ठीक नही हए परत उसक बट को तनिनक भी अफसोस नही था उसका कहना था उसन कोकतिशश तो कीयह -खकर चतनय-ास जी को आतम निगलानी हईतभी स उनका म परिरवतन हआ और बट का भोज काफी धमधाम स निकयाऔर वहइस पशचाताप की आग म जलत रह औला- स बढकर पसा नही होता ह इस बात को समझन म उनह काफी व लग गया

hellipContinue to next

BENGALI(2ND LANGUAGE)

পরথমঅধযায়-ঠাকরারীনদরনাথঠাকর

নয়ন দোচের হিমাচেররা া নাচেমই হিযাত হিছচেন ায়ানার উাররণ সবরপ নয়ন দোচের ারা হিা (াচেকর হিা হিচেতন এছাাও দেকান উৎস উপচেb রাহিতর দেক হিন করার উচেfচে(য তারা সযC হিকরচেরণ রনয পরীপ জবাহিচেয় তাচেত রপার হির 4Cরণ করচেতন ঠাকরা এই নয়ন দো হিমারচের দে(4 ং(ধর হিছচেন হিমাররা ায়ানার ষটানত পর(Cন কচের তারা হিনঃসব এই হিমাহিরর দে(4 ং(ধর গৈকাস নদর রায়চেৌধরী গৈকাস া নয়ন দোচের সমসত সমপহিতত ঋচেরণর াচেয় হিহিx কচের অহি(ষট যা আচেছ তাচেত হিপত

ইার হিপতার মতয ইচে পর নয়নচোচের ায়ানার দেগাটা কতক অসাধাররণ শরাদধ (াহিনতচেত অহিনতম ীহিপত পরকা( কহিরয়া ঠাৎ হিনহিয়া দেগ- ক) কার দো দেকান গচেলপর অং() কতা দেক ইার চেত কাচেক দোঝাচেনা চেয়চেছ গ) পরসঙগ কী কতার কতয পহিরসফট কচেরা

পরচে4র যাহিত রbা করা সমভ নয় তাই হিতহিন পতরচেক হিনচেয় ককাতায় সাস শর কচেরন গলপ কথচেকর আহিথCক অসথা নয়ন দোচের হিমাচের দেথচেক সমপরণC আাা কথচেকর হিপতা হিনচের দেষটায় অথC উপাCন করচেতন া উপাহিধ াচেভর নয তার াসা হিছনা আর দেসই কারচেরণ কথক তার একমাতর উততরাহিধকার চেয় তার হিপতার পরহিত কতজঞ কথক দো পা হি(চেচেছন হিনচের পরারণ ও মান রbার নয উপচেযাগী অথC হিনা দেষটায় পরাপত চেয়চেছন- এটাই তার কাচেছ পরম দেগৌরচের হি4য় চে মচেন কচেরন কাররণ (নয ভাণডাচের গৈপতক ায়ানার উজজব ইহিতাস অচেপbা দোার হিসeচেকর মচেধয গৈপতক দেকামপাহিনর কাগ তার কাচেছ অচেনক দেহি( মযান

TO BE CONTINUED

উ- ক) আচোয অং(টি রীনদরনাথ ঠাকচেরর দো ঠাকরা গচেলপর অং() কতা চেন আচোয গচেলপর গলপ কথকইার চেত নয়ন দোচের হিমাহিরর দে(4 ং(ধর গৈকাস ার কথা া চেয়চেছ গৈকাস া নয়ন দোচের সমসত সমপহিতত ঋচেরণর াচেয় হিহিx কচের অহি(ষট যা আচেছ তাচেত হিপত পরচে4র যাহিত রbা করা সমভ নয় তাই হিতহিন পতরচেক হিনচেয় ককাতায় সাস শর কচেরনগ) গৈকাস ার হিপতার মতযর পর নয়ন দোচের হিমাহিরর অহিসততব হিপত য় কচেয়কটা উৎস ও শরাদধ- (াহিনতচেত হিমাহিরর দে(4 কহিটক যয় চেয় হিগচেয় এচেক াচের দে(4 চেয় যায় তন তাচের গC করার মত আর হিকছই হিছ না-দেসই পরসচেঙগ এই উহিকত নয়নচোচের হিমাচেররা া নাচেমই হিযাত হিছচেন ায়ানার উাররণ সবরপ নয়নচোচের ারা হিা (াচেকর হিা হিচেতন এছাাও দেকান উৎস উপচেb রাহিতরচেক হিন করচেত হিগচেয় তারা সযC হিকরচেরণর নয পরীপ জবাহিচেয় তাচেত রপার হির 4Cরণ করচেতন তাই দেসকাচের ায়ানা দেহি(হিন সথায়ী চেত পারত না হিহিভনন উৎস শরাদধ- (াহিনতচেত সাধযা হিতহিরকত র করার নয হিমাহির হিহিকচেয় দেযত হ হিতC কা হিহি(ষট পরীচেপর দেত দেযমন অলপকাচের মচেধয হিনঃচে(4 চেয় যায়-নয়নচোচের হিমারচের অসথা তাই চেয়হিছ এই কারচেরণই কথক নয়নচোচের হিমারচের গা ভরা আমবর সয করচেত পারতনা

Physics Dimensional Analysis (Summary)

Q Find the dimensions of consts ab in relation

p=(bminusxlowastx)at

where p is the power x is the distance and t is time

Ans From principle of homogeneity dimension of b x2 are same Dim of b = dim of x2 = [L2] = [ML2T0]Dim of a = dim of ( b- x2)dim of (pt) = [M0L2T0][ML2T-2] [T-1] [T] = [M-1L0T2]

Chemistry Atomic Structure Drawbacks of Rutherfordrsquos model of

atom a According to Rutherfordrsquos model of atom electrons which are negativelycharged particles revolve around the nucleus in fixed orbits Thusb theelectrons undergo acceleration According to electromagnetic theory of Maxwell a charged particle undergoing acceleration should emitelectromagnetic radiation Thus an electron in an orbit should emitradiation Thus the orbit should shrink But this does not happenc The model does not give any information about how electrons aredistributed around nucleus and what are energies of these electrons Isotopes These are the atoms of the same

Properties of electromagnetic radiationsa Oscillating electric and magnetic field are produced by oscillating charged particles These fields are perpendicular to each other and both areperpendicular to the direction of propagation of the waveb They do not need a medium to travel That means they can even travel invacuum

Characteristics of electromagnetic radiationsa Wavelength It may be defined as the distance between two neighbouring crests or troughs of

element having the same atomicnumber but different mass numbere g 1H11H21H3

Isobars Isobars are the atoms of different elements having the same massnumber but different atomic numbere g 18Ar40 20Ca40

Isoelectronic species These are those species which have the same numberof electrons

Electromagnetic radiationsThe radiations which are associated withelectrical and magnetic fields are called electromagnetic radiations When anelectrically charged particle moves under acceleration alternating electricaland magnetic fields are produced and transmitted These fields aretransmitted in the form of waves These waves are called electromagneticwaves or electromagnetic radiations

wave as shown It is denoted by λb Frequency (ν) It may be defined as the number of waves which passthrough a particular point in one secondc Velocity (v) It is defined as the distance travelled by a wave in onesecond In vacuum all types of electromagnetic radiations travel with thesame velocity Its value is 3 times10 8m sec-1 It is denoted by v

d Wave number Wave number is defined as the number of wavelengths per unit lengthVelocity = frequency timeswavelength c = νλ

Plancks Quantum Theory- o The radiant energy is emitted or absorbed not continuously but discontinuously in the form of small discrete packets of energy called lsquoquantumrsquo In case of light the quantum of energy is called a lsquophotonrsquoo The energy of each quantum is directly proportional to the frequency of the radiation ie E α υ or E= hυ where h= Planckrsquos constant = 6626 x 10-27 Js o Energy is always emitted or absorbed as integral multiple of this uantum E=nhυ Where n=1234Black body An ideal body which emits and absorbs all frequencies is calleda black body The radiation emitted by such a body is called black body radiation

Photoelectric effectThe phenomenon of ejection of electrons from thesurface of metal when light of suitable frequency strikes it is calledphotoelectric effect The ejected electrons are called photoelectrons

Biology Chapter - 02Systematics and Five Kingdoms

Scientists divide the whole living organisms into two kingdom first and ultimately by five kingdom at last

In the earlier systems of classifications organisms are divided into kingdom plantaeand kingdom animalia on the of presenceof cell wall their modes of nutrition and movements

Some problem arise like fungi share manycharacteristic withplant despite their heterotrophic nutrition bacteria protozoa areunicellular present in both kingdom Toovercome this third kingdom Protista isintroduced which include

unicellularorganisms But there is also another

problem Allunicellular organisms are not similar kind The cellular structure of prokaryotes is verydifferent from that of other organismsEukaryotes possess a true nucleus and allcell organelles that are not present inprokaryotes So the fourth kingdom Monerais introduced which include unicellular prokaryotes (bacteriaamp blue green algae)

bull Still some problem arise in kingdomplantae

So in 1969 R H Whittakar proposedanew five kingdom System of classification

i) Kingdom Monera - unicellular prokaryotes

ii) kingdom Protista - unicellular eukaryotes

iii) Kingdom Fungi - uni or multicellular fungi with cell wall but without chlorophyll

iv) Kingdom Plantae - Multicellular Plants

v) Kingdom Animalia - Multicellular Animals

EVS Chapter 1 ndash Modes of Existence

An agricultural society

An agricultural society also known as an agrarian society is a society that constructs social order around a reliance upon farming More than half the people living in that society make their living by farming

People in an agricultural society generally lead a more settled lifestyle than those in nomadic hunter-gatherer or semi-nomadic pastoral societies because they live permanently near the land that is farmed Agricultural settlements tend to develop in areas of convenience near bodies of water which is used for both crops and transportation or along trade routes Not everyone in an agricultural society is a farmer Some people make a living trading or making and selling goods such as tools used for farming

Another way to define an agrarian society is to see the total amount of production in a nation In an agrarian society cultivating the land is the main source of wealth Such a society can recognize other means of subsistence and work habits but emphasizes the importance of agriculture and livestock Agrarian societies have existed in various parts of the world for 10000 years and continue to exist today They have been the most common form of socio-economic organization for most of recorded human history

Q) Write the features of agricultural society

Ans - Structure and Features of Agrarian Society1 Occupational Structure

An agrarian society is generally associated with the domestication of plants and animals The domestication of plants means farming and that of animals means herding Often there is mixture of farming and the use of such domesticated animals as cow goat and sheep

2 Forms of Land Ownership in Agrarian SocietiesGenerally there are landlords supervisory farmers cultivators and share croppers The landholders own the land but do not work on it They let it out for sharecropping The supervisory farmers are those who live by having their land cultivated by hired labourers The cultivators cultivate the land for themselvesThe share-croppers are those who live by tilling other peoplersquos land or a crop-sharing basis The artisans own their means of production and produce by their own labour in their homesteads

3 Village Community System An agrarian society is highlighted by

the institution of village community system The agrarian economy made fixed dwelling houses necessary Living close together for protection and co-operation and living nearer to the land gave birth to agricultural villages The village is not only the residential place of farmers it is also the social integrator

4 Minimal Division of Labour Another structural feature of agrarian society is a minimal division of labour Except for the basic division founded on age and sex differences there are few specialized roles There is only one predominant type of occupation ie domestication of plants and animals For all the people the environment physical as well as social is the same

5 Role of Family The farm family is of the patriarchal type the father is the final arbiter in most of the familyrsquos major decisions The life of ail men and women is merged in family life Since there are not many special organizations family is the only organisation to perform the tasks of aid and protection

6 Sense of Unity The members of an agrarian society exhibit a strong in-group feeling Since the whole of their social lives is wrapped up in a society which is physically economically and socially homogenous they are inclined to view the entire outside world as an out group

7 Informal Social Control An agrarian society is regionally divided into villages In a village community the force of traditional mores is more dominant than in the urban community In the village everybody is known to everybody The members in a village community help each other and share the joy and sorrows of each other Crime in an agrarian society is rare

8 Simplicity and Uniformity Life of the people in an agrarian society is marked by simplicity and uniformity Their main occupation is agriculture which largely depends upon the vagaries of nature An agrarian society is a religious society

Math Compound angles Compound angles The algebraic sum of two or more angles is called a compound angle If A B C be three angles then A+B B+C C+A A-B B-C A-C A+B-C etc are compound angles In this chapter we shall discuss the trigonometrical ratios of compound angles Theorem 1 If A B and A+B are all pisitive acute angles theni) sin( A+B) = sin A cos B + cosA sinBii) cos(A+B) = cosA cosB- sinA sinBTheorem 2If A and B are positive acute angles and AgtB theni) sin(A-B) = sin A cosB- cos A sinBii) cos(A-B) = cos A cos B+ sin A sin BTo prove that i) sin(A+B) sin (A-B) = sin2 A - sin2 B = cos2 B- cos2 A

Example 1 Prove that tan70deg=2tan50deg+tan20degSolutiontan70deg = tan(50deg + 20deg)Or tan70deg=(tan 50deg+tan 20deg)(1-tan50degtan20deg) or tan70deg (1 ndash tan 50deg tan20deg) = tan50deg+tan20degor tan70deg= tan70deg tan50deg tan20deg+ tan50deg + tan20deg = cot20deg tan50deg tan20deg + tan50deg + tan20deg = 2 tan50deg+ tan20degExample 2 If A + B = 45deg show that (1 + tanA) (1 + tanB) = 2Solutiontan(A + B) =( tan A + tan B) (1 - tan

ii) cos(A+B) Cos(A-B) = cos2 A- sin2 B = cos2 B -sin2 AProof i) LHS= sin(A+B)sin(AminusB) [Recall sin(αminusβ)=sinαcosβminuscosαsinβ And sin(α+β)=sinαcosβ+cosαsinβ]= (sinAcosB+cosAsinB)times(sinAcosBminuscosAsinB)= sin2Acos2Bminuscos2Asin2B [Recall sin2α+cos2α=1 From above we can then assume correctly that sin2α=1minuscos2α AND cos2α=1minussin2α] = sin2A(1minussin2B)minussin2B(1minussin2A) = sin2Aminussin2Asin2Bminussin2B+sin2Asin2B = sin2Aminussin2B= 1-cos2A-(1-cos2B) = cos2 B- cos2 A = RHSii)LHS= cos (A+B) cos(A-B) [ cos(A+B) = cos AcosB- sinAsinBCos(A-B) = cosAcosB+ sinAsinB]= cos2 A Cos2 B- sin2 A Sin2 B= cos2 A( 1-sin2 B) - (1- cos2 A) sin2 B= cos2 A- cos2 A sin2 B- sin2 B+ cos2 A sin2 B=cos2 A- sin2 B=1- sin2 A-(1-cos2 B) = cos2 B- sin2 A= RHSTangent formulae for compound anglesi)tan (A + B) = tan A + tan B1-tan A tan Bii)tan (A ndash B) = tan A-tan B1+tan A tan Biii) cot (A + B) = cot Acot B-1cot A+cot B(viii) cot (A ndash B) = cot Acot B+1cot B-cot A

A tan B) Or 1= (tan A+ tanB) (1-tan A tanB) Or tanA + tanB + tanA tanB + 1 = 1 + 1Or tanA (1 + tanB) + (1 + tanB) = 2Or (1 + tanA) (1 + tanB) = 2Example 3 Find the value of sin 15degSolution sin 15deg= sin(45deg-30deg) = sin45degcos 30deg- cos45degsin30deg =(1radic2) (radic32) -(1radic2) (12) = (radic3-1) 2radic2Example 4 If sin A = 1 radic10 and sin B = 1 radic5 where A and B are positive acute angles then what is A + B SolutionWe know that sin (A + B) = sin A cos B + cos A sin B= [1 radic10] [radic(1 minus 1 5)] + [1 radic5] radic(1 minus 1 10)= [1 radic10] [radic4 5] + [1 radic5] [radic9 10]= [1 radic50] times (2 + 3)= 5 radic50 = 1 radic2

sin (A + B) = sin π 4rArrHence A + B = π 4Example 5 If A + B = 225o then find [cot A] [1 + cotA] times [cot B] [1 + cot B]Solution[cot A] [1 + cotA] times [cot B] [1 + cot B] = 1 [(1 + tan A) times (1 + tan B)]=1 [tan A + tan B + 1 + tan A tan B] [ tan (A + B) = tan225o]∵

tan A + tan B = 1minus tan A tan BrArr= 1 [1 minus tan A tan B + 1 + tan A tan B]= 1 2

COMMERCE

CLASSIFICTION OF HUMAN ACTIVITIES-ECONOMIC AND NON-ECONOMIC

Firstly we shall recall the previous class for 5 mins especially for the absentees and for also the rest of the students who were there

Today at first we briefly discuss the earlier portions of the chapter

1Business-It includes all those economic activities which are concerned with production and exchange of goods and services with the object of earning profit Example A factory shop beauty parlour also business enterprises

2Profession ndashThe term profession means an occupation which involves application of specialized knowledge and skills to earn a living For Example Chartered Accountancy medicine law tax consultancy are example of professions

Questions1What are the main features of ProfessionAnswer The main features of a profession are as follows a Specialised body of knowledge-Every profession has a specialised and systematised body of knowledge b Restricted entry- Entry to a profession is allowed only to those who have completed the prescribed education and have the specialised examination c Formal education and training ndashA formal education and training is given to the person who wants to acquire the professional

3Employment-Employment mean an economic activity where people work for others in exchange for some remuneration (salary)The persons who work for others are called lsquoemployeesrsquo The persons or organizations which engage others to work for them are called lsquoemployersrsquoEg A doctor working in a hospital is employment as he is working for a salaryA lawyer may serve as a law officer in a bank

With this we shall proceed with the features of both Profession amp Employment

The main features of a profession are as follow

a Specialised body of knowledge b Restricted entry c Formal education and training d Professional association e Service motive f Code of contact

The main features of an employment are as follows

a In employment a person works for others called employer

b An employee provides personal service

c There is a service agreement or contract between the employee and the employer

d The employee has to obey the order of the employer

e No capital investment is made by the employer

Various examples of Employment are as follows

aA teacher teaching in a school or collegeb An engineer employed in Municipal Corporation of DelhicAn accountant working in the accounts department of a companydA doctor working in a hospital

Note In all the above examples of employment the individual who is involved in each example is working as an employee for a salary under an employer

qualification(MBBSCALLB)d Service motive ndashProfessionals are expected to emphasis service more on their clients rather than economic gain f Code of Conduct-The activities of professionals are regulated by a code of conduct

2 What are the main features of EmploymentAnswer The main features of an employment are as followsa In employment a person works for others called employerb An employee provides personal servicec There is a service agreement or contract between the employee and the employerd The employee has to obey the order of the employere No capital investment is made by the employer

3 Give various Professions and their respective Association are given below

Professions

Professional

Professional association

Medical profession

Doctor Medical Council of India

Law profession

Lawyers Bar Council of India

Accounting Profession

Chartered

The Institute of Chartered Accounts of India( ICAI)

Engineerin Engineers The

g Profession

institute of Engineers (India)

Accounts Basic accounting terms

Today we will give you some questions from the previous study material

Questions6) Define accounting7) What do you mean by debit

and credit8) Explain the types of account9) Define the following terms

a) Assetsb) Capitalc) Purchased) Debtorse) Transactions

10) Name the types of accounts given below

a) Krishnas accountb) Machinery accountc) Royalty accountd) Salary accounte) Furniture accountf) Audit fee account

Economics Basic Economic ConceptsSub topic

UTILITY

Before starting todayrsquos class we shall recall the last class which was about UTILITY AND THE FEATURES OF UTILITY

Now we shall proceed with the further topics of the chapter

Todayrsquos topic from the chapter lsquo Basic Economic Conceptsrsquo will be TOTAL UTILITY amp MARGINAL UTILITYNow let us quickly revise the concept of utility with an example ie goods and services are designed because they have an ability to satisfy human wantsThis feature of being able to satisfy human wants is termed as utility For example we derive utility from WiFi services as it gives us satisfaction by connecting us to our friends and family through social media here consumers derive utility from WiFi services

From the above concept we shall start with todayrsquos topicEconomists have defined TOTAL UTILITY (TU) as the total satisfaction obtained by consuming a given total amount of a good and serviceFor example the total satisfaction obtained from eating 10 mangoes is the total utility of 10 mangoes

MARGINAL UTILITY (MU) is the additional satisfaction derived from each additional unit

Questions1 What is Total Utility (TU)

Answer Total Utility (TU) is the

aggregate of the utility that a consumer derives from the consumption of a certain amount of a commodityTU=MU1+MU2++MUn

2 What is Marginal UtilityAnswer

Marginal Utility (MU) is the additional made to the total utility as consumption is increased by one more unit of the commodityMU= TUn ndashTUn-1

NoteOften economists tend to

subdivide utility into an imaginary unit called UTIL

consumed In this casethe utility obtained from each mango as it is consumed as the MU of that mango It is also defined as the addition made to the total utility when an additional unit is consumed Often economists tend to subdivide utility into an imaginary unit called UTIL

Note As a consumer increases the consumption of a good over period of time the total utility or total satisfaction derived from it increases to appoint and thereafter it decreasesHowever as the consumer keeps on consuming the good the marginal utility or the additional utility derived from it decreases

SubjectBusiness studies

Topic

BUSINESSENVIRONMENT

Summary

Now quickly let us revise the earlier points that we have already done in the last class and let us proceed with the other topics that are there in the chapter

Firstly we will recall the internal and external factors of micro environment and then we shall proceed in details

Meaning and list of internal and external factors

aInternal factorsInternal factors refer to all the factors existing within a business firm The internal factors are considered controllable because the enterprise has control over these factorsFor an example a company can alter its organization structure policies programmes employees physical facilities and marketing mix to suit the changes in the environmentList of internal factors areCorporate culture mission and objectives top management organizations structure company image and brand equity company resources

b External factorsExternal factors refer to those individual and groups and agencies with which a particular business organization comes into direct and frequent contact in the course of its functioningThese individuals and groups are known as STAKEHOLDERS because they have a stake (financial interest ) in the working and performance of the particular business List of external forces (stakeholders)Customers competitors investors suppliersmiddlemen (marketing intermediaries)

Execution 1 What do you mean by internal

factors in micro environmentAnswerInternal factors refer to all the factors existing within a business firm The internal factors are considered controllable because the enterprise has control over these factorsFor an example a company can alter its organization structure policies programmes employees physical facilities and marketing mix to suit the changes in the environment

2 What do you mean by external factors in micro environment

AnswerExternal factors refer to those individual and groups and agencies with which a particular business organization comes into direct and frequent contact in the course of its functioningThese individuals and groups are known as STAKEHOLDERS because they have a stake (financial interest) in the working and performance of the particular business

3Who are stakeholdersSTAKEHOLDERS are individuals and groups who have a stake (financial interest ) in the working and performance of the particular business 4Discuss the internal factors in briefa Corporate CultureThe values beliefs and attitudes of the founders and top management of the company exercise

financers publics

customers

suppliersfinancers

competitors

middlemen

publics

Fig STAKEHOLDERS OF A COMPANY

Apart from micro environment the other main dimension of business environment isMacro environment Macro environment refers to the general environment or remote environment within which a business firm and forces in its micro environment operateA company does not directly or regularly interact with the micro environmentTherefore macro environment is also known as indirect action EnvironmentThe macro environment forces are less controllable than the micro forces

Macro environment consists of the following components

POLITICAL AND LEGAL ENVIRONMENT

ECONOMIC SOCIAL AND ENVIRONMENT

CULTURAL

ENVIRONMENT

TECHNOLOGICAL ENVIRONMENT

a strong influence on what the cmpaany stands for how it does things and what it considers importantbMission and objectivesThe business philosophy and purpose of a comoany guide it prioritiesbusiness strategiesproduct market scope and development scope

cTop management structurethe composition of board of directors the degree of professionalization of management and the organizational structure of a company have important bearing on its business decisions

dPower structureThe internal power relationship between the board of directors and the chief executive is an important factor

eCompany image and brand equityThe image and brand equity of the company play a significant role in raising finance forming alliance choosing dealers and suppliers launching new products entering foreign markets

5 What is Macro environmentAnswerMacro environment refers to the general environment or remote environment within which a business firm and forces in its micro environment operateA company does not directly or regularly interact with the micro environmentTherefore macro environment is also known as indirect action EnvironmentThe macro environment forces are less controllable than the micro forces 6 What are the components of macro environmenta Political and legal environmentb Economic environmentc Social and cultural environmentd Technological environment

BUSINESS FIRM

Fig COMPONENTS OF MACRO ENVIRONMENTPolitical science

Introduction to political science

Comparative politics and itrsquos scope Comparative politics is the second major dimension of political scienceIt is also a very vast area of study and a very large number of political scientists even treat it as an autonomous area of study within the board ambit of political scienceScope of comparative politics-

1 All political structures -Comparative politics includes the study of all structures formalnon formal governmental and extra governmental which are directly or indirectly involved in politics in all the countries of the world

2 Functional studies- Comparative politics seeks to study politics less from the point of view of the legal institutions in terms of their powers and move from the point of view of their functions which constitute the political process and their actual Operation in the environment

3 Study of political behaviour- Another important part of its scope is the study of the actual behaviour of the people in the process of politics

4 Study of similarities and differences- comparative politics also undertakesan analysis of the similarities and differences among political process and functions

5 Study of all political systems -comparative politics seeks to analyse the actual behaviour and performance of all political systems western as well as non western

6 Study of the environment and infrastructure of politics-The study of politics demands a study of the psychological sociological economic and anthropological environment in fact the social environment as a whole in which each political system operates

7 Study of political culture- political culture is composed of attitudesbeliefs emotions and values of a society that relate to the political system or politics

8 Study of political participation- Political participation is a universal processThe only difference is that while in some states it is limited in others it is wider

9 Study of political process- political

Answer the following questions-

What is comparative politics

What are the scope of comparative politics

Homework- learn

processes like decision makingpolicy making judicial process leadership recruitment process and others are always at work in all political systems

The scope of comparative politics is very comprehensive It includes everything that falls within the area of political activity and political process

History CAMBRIDGE VIEW ABOUT

THE PARTITION

AND REFUTATION

OF CAMBRIDGE

VIEW

Cambridge view about the Partition The Cambridge school of historians have interpreted that opposition to partition scheme was made entirely by the elitist groups They hold the view that Lord Curzon planned to partition the Bengal for administrative purposeREFUTATION OFCAMBRIDGE VIEW The Rationalist historians have rejected the interpretations of the Cambridge School of historians on various grounds

1 QUESTION State different views of historians regarding Partition of Bengal

ANSWER Cambridge historians believed that Lord Curzon partitioned Bengal for administrative reasons only and not for the political motive The Middle class elitist group protested because of their petty interest The Hindu zamindars protested as they have to spend more money for managing their estatesThe lawyers of Calcutta High court feared to lose their clientBut according to the nationalist Historians was-

2- The ultimate object of Lord Curzon was to crush the unity of Bengal politicians

3- If Bengal becomes a separate province Bengali speaking 16 million people of western part would become minority under Hindi speaking people of Bihar and Oriya speaking people of Orissa

4- The bureaucrats expected that the protest movement would die down quickly

5- Lord Curzon used the Muslim community in his political game

6- Idealism had great contribution in the protest against partition

7- The people of the every section of society were affected by the partition of Bengal

Computer Science

Numbers Convertion of dcimal number to octal numberThe decimal numeral system is the standard system for denoting integer and non-integer numbers It is the extension to non-integer numbers of the Hindu-Arabic numeral system For writing numbers the decimal system uses ten decimal digits a decimal mark and for negative numbers a minus sign - The decimal digits are 0 1 2 3 4 5 6 7 8 9 the decimal separator is the dot in many countries

The octal numeral system or oct for short is the base-8 number system and uses the digits 0 to 7 Octal is sometimes used in computing instead of hexadecimal perhaps most often in modern times in conjunction with file

permissions under Unix systems It has the advantage of not requiring any extra symbols as digits It is also used for digital displays

Follow these steps to convert a decimal number into octal form

1 Divide the decimal number by 82 Get the integer quotient for the next iteration (if the number will not divide equally by 8 then round down the

result to the nearest whole number)3 Keep a note of the remainder it should be between 0 and 74 Repeat the steps until the quotient is equal to 05 Write out all the remainders from bottom to top This is the solution

For example if the given decimal number is 8453

Division Quotient Remainder

8453 8 1056 5

1056 8 132 0

132 8 16 4

16 8 2 0

2 8 0 2

Then the octal solution is 20405

Subject Eng Literature (The Tempest ndash William Shakespeare) Topic Act I Scene 1 Lines 33 to 67 (End of scene) Date 16th April 2020 (4th Period)

[Students should read the original play and also the paraphrase given in the school prescribed textbook]Summary Questions amp Answers

[SUMMARY OF THE ENTIRE SCENE]

o The play starts with the scene of a severe storm at sea Alonso (King of Naples) Sebastian (Alonsorsquos brother) Ferdinand (Alonsorsquos son) Gonzalo Antonio (the usurping Duke of Milan) are in a ship in the midst of the storm

o The mariners are trying their best to control the vessel from running aground and are totally following the orders of their Master the Boatswain They have scant success

o The mariners become extremely unhappy and annoyed when most of the passengers arrive on the deck thereby hampering their effort to save the ship There is serious confrontation between them and the passengers who are part of the Kingrsquos entourage

o The mariners could not save the ship

SUMMING-UP

(i) Vivid description of the scene which gives a realistic description of terror and confusion of a tropical storm

(ii) Shows Shakespearersquos accuracy of knowledge in describing the naval operations and also matters of seamanship

(1) GONZALO Ill warrant him for drowning (L 45-57)

though the ship were no stronger than a nutshell and as leaky as an unstanched

wenchBOATSWAIN Lay her a-hold a-hold Set her two courses Off to

sea again lay her offMARINERS All lost To prayers to prayers All lostBOATSWAIN What must our mouths be coldGONZALO The king and prince at prayers Lets assist them

For our case is theirsSEBASTIAN Im out of patienceANTONIO We are merely cheated of our lives by drunkards

This wide-chopped rascal - would thou mightst lie drowning the washing of ten tides

(a) What does Antonio say at the insolent manners of the boatswain just before the given passage

Being irritated at the insolent manners of the boatswain just before the given extract Antonio the Duke of Milan calls him a worthless dog son of a woman without any morals an arrogant and disrespectful noisemaker He says that the boatswain deserved to be hanged(b) What statement does Gonzalo repeat about the boatswain

Gonzalo shows his faith that the boatswain is not destined to die by drowning He is destined to be hanged and nothing can alter this decree of destiny He says that even if the ship was as frail as a nutshell the boatswain could not be drowned for his destiny was to be hanged(c) What do the passengers do when they have lost all hope of their survival

When the passengers have lost all hope of survival they take

(iii) The opening scene justifies the title ndash The Tempest

UNANSWERED QUESTIONS

(i) The King always travels with his entire fleet including his soldiers Where were the other ships

(ii) Why was the ship in that area Where was it coming from or going where

(iii) The ship broke apart What happened to those who were in the ship

(We shall get the answer to the above questions as the play progresses)

leave of life with fervent prayers The mariners take their last hearty drink and are ready for death(d) What blame does Antonio put upon the mariners and the boatswain Antonio rebukes the mariners that these drunkards have brought them to the present crisis by neglecting their duties He blames them saying that they are going to lose their lives entirely for the negligence of the boatswain and his fellows(e) What does Antonio say while cursing the boatswain

Antonio gives vent to his wrath upon the boatswain in particular He calls the boatswain a wide-mouthed rascal who deserves to be hanged on the sea-shore at low water mark so that ten tides might wash over his body and take out of him all the liquor that he has been drinking

Class XIISubject Topic Summary ExecutionHistory Topic

1 1935 ACT AND WORKING OF PROVINCIAL AUTONOMYCONGREE AND OTHER MINISTERSSUB TOPIC GOVERNMENT OF INDIA ACT1935

Government of India Act 1935 This act established a lsquoFederation of Indiarsquo made of British Indian provinces and Indian states and provided for autonomy with a government responsible to the elected legislature in every provinceThis act introduced abolition of Diarchy at provinces The entire provincial administration was introduced to the responsible ministers who were controlled and removed by the provincial legislature The provincial autonomy means two things First The provincial governments were wholly responsible to the provincial legislature Secondly Provinces were free from outside control and interference in the large number of matters The act divided the powers between the centre and provinces in terms of three lists- Federal list( for centre) Provincial list (for province) and concurrent list (for both) Residuary powers were given to the viceroy In the election under the government of India Act the Congress swept the poll the mandate of the people came in favour of the congress so far as general Hindu seats were concerned The Congress did not get a single Muslim seates in Bombay CP UP Sind and BengalIn five provinces Congress had yhe clear majority In BengalNWFPAssam and Bombay Congress emerged as a single largest partyOn the other side the performance of the Muslim League was badThus the Congress formed ministers in 7 provinces out of 11 provinces Coalition ministry was also formed in two other provincesOnly BENGAL AND Punjab had non- congress ministries

1 QUESTION What was the main change introduced by the Government of India ActANSWER a) The Act gave more

autonomy to the provinces b) Diarchy was abolished at the

provincial levelsc) The Governor was the head of

the executived) There was a council of

ministers to advise him The ministers were responsible to the provincial legislatures who controlled them The legislature could also remove the ministers

e) The Governors still retained special reserve powers

2 QUESTION Why did the federal scheme introduced by the Government of India Act 1935 never come into operation

ANSWER The Federal structure of the Government of India was to be composed with the Governor General and Council of ministers The Federal legislature was to be Bicameral legislature- The council of states and the House of Assembly The ministers were to be chosen by the Governor general and they were to hold the office during his pleasure

The provinces of British India would have to join the federation but this was not compulsory for the princely states

This federation never materialised because of the lack of support from the required number of

princely statesThis act was refused and

rejected by the princes the Congress and the Muslim League

Thus both Congress and the League participated in the election of 1937 Thus the federal part was never introduced but the provincial part was put into operations

Bengali 2nd

Language

াচেরর পরাথCনা(কহিতা )

াচেরর পরাথCনা কহিতাটি কহি (ঙখ দেঘাচে4র দো আচো য কহিতায় াচেরর পতর হমায়ন কঠিন দেরাচেগ আxানত ার ঈশবর া আললার কাচেছ পরাথCনা কচেরচেছন তার পচেতরর ীন হিফহিরচেয় হিচেত এই কহিতায় ার পচেতরর ীন হিভbা দেচেয়চেছন ারার এমনহিক হিনচের ীন হিসCচেনর হিহিনমচেয় হিতহিন তার দেছচের ীন হিফচের দেপচেত দেচেয়চেছন তার দেছচের এই দেরাচেগর ন য হিতহিন হিনচেচেকই ায়ী কচেরচেছন তার হিনচের করা পাপচেকই হিতহিন ায়ী কচেরচেছন এছাা রানৈনহিতক ও আথCসামাহিক অসথার কথা তচে ধরা চেয়চেছ এই কহিতায় ার তার হিনচের পাপ কমCচেকই ায়ী কচেরচেছ ার অন যায় ভাচে দেপহি((হিকতর মাধ যচেম অপররা য কচেরচেছ আর এই অন যায় কাচের ন যই তার পহিরাচের হিপযCয় এচেসচেছ দে এক পরকার মানহিক নধন ইহিতাচেসর ার হিপতা চেয় সবাভাহিকভাচে ভাচোাসা দে মমতা দেথচেক মকত চেত পাচেরনহিন তাই হিপতা চেয় আললা া ভগাচেনর কাচেছ পতর হমায়চেনর পরানহিভbা দেচেয়চেছন ার আললা া ভগাচেনর কাচেছ াহিনচেয়চেছন তার হিনচের ীন হিসCন হিচেত হিতহিন রাী তার হিহিনমচেয় পচেতরর ীন হিফচের দেপচেত দেচেয়চেছন াচেরর হিপতসভ হিচেকর কথা এই কহিতায় ফটিচেয় দেতাা চেয়চেছ হিপতা পচেতরর হিরাহিরত মান নধচেনর কথা তচে ধরা চেয়চেছ

হিচে(4 হিকছ াইচেনর তাৎপযC১) ldquoদেকাথায় দেগ ওর সবচছয দেৌন দেকাথায় কচেরায় দেগাপন bয়ldquoউততর) াচেরর পতর হমায়ন কঠিন দেরাচেগ অসসথ তাই তার দেযৌন াহিরচেয় যাচেচছ এই দেরাচেগ তাচেক দেগাপচেন কচেরকচের াচেচছ তার সক (হিকত ধীচের ধীচের bয় চেচছ তাই হিপতা চেয় ার আললার কাচেছ হমায়চেনর পরান হিভbা দেচেয়চেছন২) ldquoাগাও (চেরর পরাচেনত পরানতচের ধসর (ন দেযর আান গানldquoউততর) াচেরর পতর হমায়ন কঠিন দেরাচেগ আxানত তাই ার আ দে(াচেক মমCাত (চেরর পচেথ পরানতচের আান গান ধবহিনত দোক দেসই আান গান আললার কাচেছ দেযন চে যায় আললা দেযন এই আহিতC শচেন পচেতরর ীন হিফহিরচেয় দেয় ৩)ldquoনাহিক এই (রীচেরর পাচেপর ীানচেত দেকানই তরারণ দেনই ভহি4চেতরldquoউততর) হমায়চেনর অসসথতার ন য ার হিনচেচেকই ায়ী কচেরচেছন কারন ার অচেনক রা য অন যায় ভাচে কচেরচেছ তাই তার এই পাপ কাচের ন য তার ঘচের আ হিপ এচেসচেছ এই অন যায় কাচের ন য তার মহিকত দেনই তাই ার আললার কাচেছ এই পাপ কাচেযCর ন য bমা পরাথM

Hindi 2ndlang

-ासी(जयशकर परसा-)

-ासी जयशकर परसा- की एक ऐसी कहानी ह जिजसम भारतीय ससकनित और राषटरीयता का सवरगजीतहोता ह इस कहानी म इरावती एक निहद कनया ह जिजस मलअचछो न मलतान की लट म पकडा और -ासी बना दि-या उस 500 दि-न -कर काशी क एक महाजन न खरी-ा दसरी -ासी निफरोजा ह वह गलाम ह निफरोजा को छडान क कतिलए अहम- को 1000 सोन क कतिसकक भजन थ जो अभी तक नही आए थ राजा साहब कठोर होत हए भी निफरोजा को निबना धनराकतिश क कतिलए उस म कर -त ह वनिफरोजा को अहम- को समझान की बात कहत हकहानी क अत म हम -खत ह निक इरा वती और जाटो क सर-ार बलराज का मिमलन होता हअहम- को यa म मार दि-या जाता ह वहा निफरोजा की परसननता की समामिध बनती ह वहा एक फल चढती ह और डीजल आती ह निफरोजा उस समामिध की आजीवन -ासी बनी रहती हलखक अपन उददशय अथात -ास परथा पर परकाश डालन और इस परथा क कारण होन वाल -ातो क दखो को दि-खान म पणता सफल हए ह

helliphellipContinue to next

Biology Reproductio Today we will discuss about vegetative Q1 Name some vegetative propagules

n in Organisms

propagation of plants The process of multiplication in which fragments of plant body function as propagule and develop into new individual is called vegetative propagation The units of such propagation are runner rhizome tuber bulb etc

and the speciesinvolvedVegetative propagules

Parts involved

Bulb StemBulbil BulbilRhizome Stem Runner Stem Tuber Stem Offset Stem Leaf buds Leaves Suckers Stem

Corns Stem stolon

Q2 State advantages of vegetative propagation

i) Rapid methodii) Sure and easy methodiii) Useful in plants that cannot

produce viable seeds or long seed dormancy

iv) Maintains purity of raceQ 3 Banana fruit is said to be parthenocarpic where as turkey is said to be parthenogenetic WhyBanana develops without fertilization from an unfertilized ovary thus is parthenocarpicIn turkey the ovum or female gamete developinto a new chick without fertilization thus isparthgenetic

Q4 Why is water hyacinth is called as a ldquoTerror of Bengalrdquo Water hyacinth can

propagatevegetatively all over the water body in a short per short period of time This resulted increased biochemicaloxygen oxygen demand of water body causing mortalityof fishes It is very difficult to get rid off them Thus known as terror of Bengal

Chemistry

Solid state GENERAL CHARACTERISTICS OF SOLID STATEIn nature the particular state of matter is governed by two opposing forces at given set of temperature and pressure These forces are intermolecular force of attraction and thermal energy If intermolecular force of attraction is high as compared to thermal energy particles remains in closest position

Intext QuestionsQ1 Classify the following solids as crystalline and amorphous Sodium chloride quartz glass quartz rubber polyvinyl chloride Teflon

A1 Crystalline

and hence very less movement in particles is observed In this case solid state is the preferred state of matter

Let us revise the general characteristics of solid

i) Fixed mass volume and shape

ii) Strong intermolecular force of attraction

iii) Least intermolecular space

iv) Fixed position of constituent particles

v) Incompressible and rigid

Q2 what type of interactions hold the molecules together in a polar molecular solid[CBSE 2010]A2 The molecules in a solid are held together by van der Waals forces The term van der Waals forces include hydrogen bonding dipole-dipole attraction and London dispersion forces All molecules experience London dispersion forces In addition polar molecules can also experience dipole-dipole interactions So the interactions that holds the molecule together in polar molecular solid are London dispersion force and dipole-dipole interactionsQ3 Write a feature that will distinguish a metallic solid from an ionic solid [CBSE 2010]A3 Metals are malleable and ductile whereas ionic solid are hard and brittle Metallic solid has typical metallic lustre But ionic solid looks dullQ4 Write a point of distinction between a metallic solid and an ionic solid other than metallic lustre [CBSE 2012]A4 Metals are malleable and ductile whereas ionic solid are hard and brittleQ5 Write a distinguish feature of metallic solid [CBSE 2010]A5 The force of attraction in

solid Sodium chloride Quartz Amorphous solid Quartz glass rubber polyvinyl chloride Teflon Q2 why glass is considered as super cooled liquidA2 Glass shows the tendency to flow at slower rate like liquid Hence they considered as super cooled liquidQ3 why the window glass of old buildings show milky appearance with timeA3 Glass is an amorphous solid Amorphous solid has the tendency to develop some crystalline character on heating Due to heating in day over the number of years glass acquires some crystalline character and show milky appearanceQ4 why the glass panes fixed to window or doors of old building become slightly thicker at bottomA4 Glass is super cooled liquid It has the tendency to flow down very slowly Due to this glass pane becomes thicker at the bottom over the timeQ5 Sodium chloride is a crystalline solid It shows the same value of refractive index along all the direction TrueFalse Give reasonA5 FalseCrystalline solid shows anisotropy in properties That is it shows different values for the given physical property in different direction All the crystalline solids show anisotropy in refractive index Therefore sodium chloride will show different values of refractive index on different directions

Q6 Crystalline solid are anisotropic in nature What does this statement means

between the constituent particles is special kind of electrostatic attraction That is the attraction of positively charged kernel with sea of delocalized electronsQ6 which group of solid is electrical conductor as well as malleable and ductile [CBSE 2013]A6 Metallic solidQ7 why graphite is good conductor of electricity although it is a network (covalent solid)A7 The exceptional property of graphite is due to its typical structure In graphite each carbon is covalently bonded with 3 atoms in same layer The fourth valence electron of each atom is free to move in between different layersThis free electron makes the graphite a good conductor of electricity

[CBSE 2011]A6 Anisotropy is defined asrdquo Difference in properties when measured along different axis or from different directionsrdquo Crystalline solid show different values of some of the physical properties like electrical resistance refractive index etcwhen measured along the different directions The anisotropy in crystalline solid arises due to the different arrangement of particles in different directions

Math Function Composition of functions Think of an industrial plant that produce bottles of cold drinks first there is the operation (or function) f that puts the cold drink inside the bottle followed by the opeartion g that close the bottle with the capThis leads to the following definitionDefinition Let f A rarr B and g B rarr C be two functions Then the composition of f and g denoted by gof is defined as the function gof A rarr C given by gof(x) = g(f (x)) forall x isinA

Definition A function f X rarr Y is defined to be invertible if there exists a function g Y rarr X such that gof = IX and fog = IY The function g is called the inverse of f and is denoted by f -1

Thus if f is invertible then f must be one-one and onto and conversely if f is one-one and onto then f must be invertible This fact significantly helps for proving a function f to be invertible by showing that f is one-one and onto specially when the actual inverse of f is not to be determined

Example 1 Let f 2 3 4 5 rarr 3 4 5 9 and g 3 4 5 9 rarr 7 11 15 be functions defined as f(2) = 3 f(3) = 4 f(4) = f(5) = 5 and g (3) = g (4) = 7 and g (5) = g (9) = 11 Find gofSolution We have gof(2) = g (f(2)) = g (3) = 7 gof(3) = g (f(3)) = g (4) = 7gof(4) = g (f(4)) = g (5) = 11 and gof(5) = g (5) = 11Example 2 Find gof and fog if f R rarr R and g R rarr R are given by f(x) = cos x and g (x) = 3x2 Show that gof ne fogSolution We have gof(x) = g(f(x))=g(cosx) = 3 (cos x)2

= 3 cos2 x Similarly fog(x)=f(g (x))= f(3x2)= cos (3x2) Note that 3cos2 x ne cos 3x2 for x = 0 Hence gof ne fogExample 3 Show that if f A rarr B and g B rarr C are onto then gof A rarr C is also ontoSolution Given an arbitrary element z isin C there exists a pre-image y of z under g such that g (y) = z since g is onto Further for y isin B there exists an element x in A with f(x) = y since f is onto Therefore gof(x) = g (f(x)) = g (y) = z showing that gof is onto Example 4 Let Y = n2 n isin N sub N Consider f N rarr Y as f(n) = n2 Show that

f is invertible Find the inverse of fSolution An arbitrary element y in Y is of the form n2 for some n isin N This implies that n =radicy This gives a function g Y rarr N defined by g (y) =radicy Nowgof (n) = g (n2)=radicn2 = n and fog (y) =f(radicy) = (radicy) 2 y which shows that gof=IN and fog= IY Hence f is invertible with f -1 = g

Political Science

Constitution of India-The Preamble

Summary

Objective of the state-To secure equality of status and of opportunity To promote fraternity among all the citizens To assure the dignity of the individuals and Unity and integrity of the nation

Justice-Justice stands for rule of law absence of arbitrariness and a system of equal rights freedom and opportunities for all in a society India seeks social economic and political justice to ensure equality to its citizens

Liberty-Liberty implies the absence of restraints or domination on the activities of an individual such as freedom from slavery serfdom imprisonment despotism etc The Preamble provides for the liberty of thought expression belief faith and worship

Equality-Equality means the absence of privileges or discrimination against any section of the society The Preamble provides for equality of status and opportunity to all the people of the country

Fraternity-The Preamble declares that fraternity has to assure two thingsmdashthe dignity of the individual and the unity and

Execution

Answer the following questions-

Short notes-1 Equality2 Fraternity3 Justice4 Liberty

Homework-Learn

integrity of the nation The word integrity has been added to the Preamble by the 42nd Constitutional Amendment (1976)

Business studies

Human resource management (chapter 1)

On the day of 1504 2020 I have discussed with you the managerial functions and procurement functions of HRM

Today weare going to discuss about the development function integration functions and maintenance function

Development functions-HRM improves the knowledge skills attitude and values of employees so that they the present and future jobs more effectively it includes

1) Development functions of HRM

a) Performance appraisal = It implies systematic evaluation of employees with respect to their performance on the job and their potential for development

b) Training =It is the process by which employees learn knowledge skills and attitudes to achieve organisational and personal goals

c) Executive development = It is the process of developing managerial talent through appropriate program

2) Integration functionsa) HRM reconcile the goals of

organisation with those of its members through integrating function

b) HRM tries to motivate employees to various financial and non financial incentives provided in job specification etc

3) Maintenance functiona) HRM promote and protect the

physical and mental health of employees by providing several types of benefits like housing medical aid etc

b) It Promote Social security measures to employees by providing provident fund pension gratuity maternity benefits

SubjectCOMMERCE

Topic

BUSINESSENVIRONMENT

Summary

Now quickly let us revise the earlier points that we have already done in the last class and let us proceed with the other topics that are there in the chapter

Firstly we will recall the internal and external factors of micro environment and then we

Execution 3 What do you mean by internal factors

in micro environmentAnswerInternal factors refer to all the factors existing within a business firm The internal factors are considered controllable because the enterprise has control over these factors

Development FunctionsPerformance AppraisalTrainingExecution Development

shall proceed in details

Meaning and list of internal and external factors

aInternal factorsInternal factors refer to all the factors existing within a business firm The internal factors are considered controllable because the enterprise has control over these factorsFor an example a company can alter its organization structure policies programmes employees physical facilities and marketing mix to suit the changes in the environmentList of internal factors areCorporate culture mission and objectives top management organizations structure company image and brand equity company resources

b External factorsExternal factors refer to those individual and groups and agencies with which a particular business organization comes into direct and frequent contact in the course of its functioningThese individuals and groups are known as STAKEHOLDERS because they have a stake (financial interest ) in the working and performance of the particular business List of external forces (stakeholders)Customers competitors investors suppliersmiddlemen (marketing intermediaries)financers publics

customers

suppliersfinancers

For an example a company can alter its organization structure policies programmes employees physical facilities and marketing mix to suit the changes in the environment

4 What do you mean by external factors in micro environment

AnswerExternal factors refer to those individual and groups and agencies with which a particular business organization comes into direct and frequent contact in the course of its functioningThese individuals and groups are known as STAKEHOLDERS because they have a stake (financial interest) in the working and performance of the particular business

3Who are stakeholdersSTAKEHOLDERS are individuals and groups who have a stake (financial interest ) in the working and performance of the particular business 4Discuss the internal factors in briefa Corporate CultureThe values beliefs and attitudes of the founders and top management of the company exercise a strong influence on what the cmpaany stands for how it does things and what it considers importantbMission and objectivesThe business philosophy and purpose of a comoany guide it prioritiesbusiness strategiesproduct market scope and development scope

cTop management structurethe composition of board of directors the degree of professionalization of management and the organizational structure of a company have important bearing on its business decisions

dPower structureThe internal power relationship between the board of directors and the chief executive is an important factor

e Company image and brand equityThe image and brand equity of the company play a significant role in raising finance forming alliance choosing dealers and suppliers launching new products entering foreign markets

5 What is Macro environmentAnswerMacro environment refers to the general

competitors

middlemen

publics

Fig STAKEHOLDERS OF A COMPANY

Apart from micro environment the other main dimension of business environment isMacro environment Macro environment refers to the general environment or remote environment within which a business firm and forces in its micro environment operateA company does not directly or regularly interact with the micro environmentTherefore macro environment is also known as indirect action EnvironmentThe macro environment forces are less controllable than the micro forces

Macro environment consists of the following components

POLITICAL AND LEGAL ENVIRONMENT

ECONOMIC SOCIAL AND ENVIRONMENT

CULTURAL

ENVIRONMENT

TECHNOLOGICAL ENVIRONMENT

Fig COMPONENTS OF MACRO ENVIRONMENT

environment or remote environment within which a business firm and forces in its micro environment operateA company does not directly or regularly interact with the micro environmentTherefore macro environment is also known as indirect action EnvironmentThe macro environment forces are less controllable than the micro forces 6 What are the components of macro environmenta Political and legal environmentb Economic environmentc Social and cultural environmentd Technological environment

Computer Science

Logic gates

Digital systems are said to be constructed by using logic gates These gates are the AND OR NOT NAND NOR EXOR and EXNOR

BUSINESS FIRM

gates The basic operations are described below with the aid of truth tables

AND gate

The AND gate is an electronic circuit that gives a high output (1) only if all its inputs are high A dot () is used to show the AND operation ie AB Bear in mind that this dot is sometimes omitted ie ABOR gate

The OR gate is an electronic circuit that gives a high output (1) if one or more of its inputs are high A plus (+) is used to show the OR operationNOT gate

The NOT gate is an electronic circuit that produces an inverted version of the input at its output It is also known as an inverter If the input variable is A the inverted output is known as NOT A This is also shown as A or A with a bar over the top as shown at the outputs The diagrams below show two ways that the NAND logic gate can be configured to produce a NOT gate It can also be done using NOR logic gates in the same way

NAND gate

This is a NOT-AND gate which is equal to an AND gate followed by a NOT gate The outputs of all NAND gates are high if any of the inputs are low The symbol is an AND gate with a small circle on the output The small circle represents inversion

NOR gate

This is a NOT-OR gate which is equal to an OR gate followed by a NOT gate The outputs of all NOR gates are low if any of the inputs are highThe symbol is an OR gate with a small circle on the output The small circle represents inversion

EXOR gate

The Exclusive-OR gate is a circuit which will give a high output if either but not both of its two inputs are high An encircled plus sign ( ) is used to show the EOR operation

EXNOR gate

The Exclusive-NOR gate circuit does the opposite to the EOR gate It will give a low output if either but not both of its two inputs are high The symbol is an EXOR gate with a small circle on the output The small circle represents inversion The NAND and NOR gates are called universal functions since with either one the AND and OR functions and NOT can be generated

Note A function in sum of products form can be implemented using NAND gates by replacing all AND and OR gates by NAND gates A function in product of sums form can be implemented using NOR gates by replacing all AND and OR gates by NOR gates

Logic gate symbols

Table 2 is a summary truth table of the inputoutput combinations for the NOT gate together with all possible inputoutput combinations for the other gate functions Also note that a truth table with n inputs has 2n rows You can compare the outputs of different gates

Logic gates representation using the Truth table

Example

A NAND gate can be used as a NOT gate using either of the following wiring configurations

Subject Eng Literature (The Tempest ndash William Shakespeare) Topic Act III Scene 3 Lines 53 to 110 (End of the scene) Date 16th April 2020 (2nd Period)

[Students should read the original play and also the paraphrase given in the school prescribed textbook]Summary Questions amp Answers

o Seeing this strange scene all are inclined to believe the tales told by travelers that there truly are ldquounicornsrdquo and ldquothe phoenixrsquo thronerdquo

o As they are about to sit down to the feast the banquet is snatched away by a harpy (Ariel disguised) A spiritrsquos voice (Arielrsquos voice) denounces Alonso Sebastian and Antonio with particular

1 ARIEL You are three men of sin whom Destiny

(Line 53-58)That hath to instrument this

lower world And what is int the never-surfeited sea

Hath caused to belch up you and on this island

Where man doth not inhabit you rsquomongst men

Being most unfit to live I have made you mad

reference to their crime in expelling Prospero from Milan They have not received any punishment for their deed earlier but the time for their punishment has arrived Upon Alonso it pronounces ldquolingering perdition worse than deathrdquo from which there is no remedy except through sincere repentance Ariel then vanishes in thunder and the shapes enter again and carry away the table

o Prospero watching invisibly is very pleased with the performance of Ariel and his (Prosperorsquos) ldquomeaner ministersrdquo All his enemies are now in his power and are in a fit of desperation He then leaves them and goes to see how Ferdinand and Miranda are getting on

o Alonso is now much humbled and penitent with the after effect of the spiritrsquos denunciation of his crimes He believes that his son is lost forever After this all disperse being stricken mad by the speech of the spirit

o Gonzalo fearing that they may do violence to themselves or to one another follows them and bid others to follow

(a) To whom does Ariel disguised as a harpy call the three sinners What game did Fate of Destiny play with

them

The three sinners called by Ariel are Alonso Sebastian and Antonio It was Destiny which had caused the ocean to cast the three sinners on the shore Though the ocean is all the time devouring whatever appears on its surface and is never satisfied with its continual swallowing of the ships and men in the present case the ocean had cast these three sinners on the shore without killing them

(b) Who had jointly been responsible for the conspiracy against Prospero What is Prosperorsquos purpose behind all this

Three men Alonso Sebastian and Antonio had jointly

been responsible for the conspiracy against Prospero They had driven out Prospero form Milan Prosperorsquos purpose is to make these three sinners realize the wrong they had done He wants them to repent for their criminal deeds because repentance leads to self-esteem(c )What does Ariel (the harpy) tell Alonso and his companions when they take out their swords to attack him

Seeing them drawing their swords Ariel (harpy) tells them that he and his companions are the instruments of destiny and that it is not possible for human beings to do them any injury He says that the swords of human beings can not injure even a minute part of his feathers Their swords are as ineffective against him and his companions as against the wind or the water

(d) Give the explanatory meanings of the following expressions in the context of the above extract

(i)Never surfeited (ii) Belch up (iii) lsquomongst men

(i) Never surfeited never led to satisfaction

(ii) Belch up cast ashore(iii) lsquomongst men in human

society2

I and my fellows (Line 60-65)

Are ministers of Fate The elementsOf whom your swords are tempered may as wellWound the loud winds or with bemocked-at stabsKill the still-closing waters as diminishOne dowl thats in my plume

IMPORTANT PASSAGES EXPLAINED

The elements

(Line 61-66)Of whom your swords are tempered may

as wellWound the loud winds or with

bemocked-at stabs

(a) Who is lsquoIrsquo Who are his lsquofellowsrdquo

lsquoIrsquo is referred to Ariel in disguise of a harpy His lsquofellowsrsquo are other spirits serving Prospero the real Duke of Milan who has acquired supernatural powers after being banished from his Dukedom Prospero has settled in this uninhabited island

(b) What are the elements that have temperrsquod the swords Why will it not work against the speaker

The swords (of Alonso and his companions) are tempered by metal (steel) which is taken out of the earth and refined by

Kill the still-closing waters as diminishOne dowl thats in my plume My fellow

ministersAre like invulnerable

In these words Ariel reminds the King and his companions of the utter futility of drawing swords against himself and his fellows Ariel drives Alonso Antonio and Sebastian the three men of sin to desperation ndash a state in which men do violence to themselves They draw swords to strike Ariel But Ariel reminds them that he and the other spirits are the ministers of destiny and nothing can wound them The steel of which their swords are made of may cut the wind or water which being divided always closes up again Even supposing that such things may be possible it is quite impossible that their swords will cut one feather in their plume They are incapable of being wounded by any sword of man Hence it is foolish on their part to attempt to strike at Ariel and his fellow-spirits

For which foul deed

(Line 72-75)The powers delaying not forgetting

haveIncensed the seas and shores yea all the

creatures Against your peace

Ariel enters like a harpy and remaining invisible tells Alonso Sebastian and Antonio that he and other harpies are the agents of Destiny appointed to carry out her decrees He tells them that their punishment for the crime against Prospero which has been so long deferred is now to fall upon them He reminds them that they had expelled Prospero from Milan and set him and his innocent child adrift on the sea and that the sea had paid them back for their sin by the shipwreck and by the calamities they have suffered He tells them that the powers above which did not forget this mean treachery but only deferred the punishment have now engaged the seas and the shores and all living beings including him and his comrades against them The very elements and supernatural agency Ariel adds have taken up the avenging of their crime against Prospero

the action of fire It may cut the wind or water which being divided always closes up again

The sword will not work against the spirits and the harpy because they are the ministers of destiny and nothing can wound them nor it will cut a single feather in their plume

(c )What is the meaning of lsquodowlrsquo in the last line

The term lsquodowlrsquo means a filament or the smallest part of a feather In this context Ariel in disguise of harpy says that their sword cannot even damage the smallest filament of their (Arielrsquos and other spirits) feathers as they are incapable of being wounded by any sword of man

(d) What does the speaker remind the listeners about

Ariel in disguise of harpy reminds Alonso the King of Naples Sebastian Alonsorsquos brother and Antonio the present Duke of Milan and the treacherous brother of Prospero as they being three men of sin He even reminds them that their punishment for their crime against Prospero which has been so long deferred now falls upon them He reminds them that they have expelled Prospero from Milan and has set him along with his innocent infant daughter adrift on the sea So the sea has paid them back for their sin by their shipwreck and the calamities they have suffered since then The harpy rebukes Alonso of his sin that has incensed the Gods and has deprived him of his son as a punishment

(e) How do they respond

When Ariel in disguise of a harpy reminds Alonso Sebastian and Antonio of their past misdeeds and sin Alonso has a look of terror and confusion in his eyes He utters the words of sincere repentance wrung out of his conscience-stricken heart It appears to him that all the elements of nature the sea-waves the wind and the thunder proclaiming a loud voice in the name of Prospero and the crime Alonso has committed against him They are calling upon him to repent There is a deep storm raging in Alonsorsquos breast and the echoes of that storm are ringing in his ears like a clear note of wind-instrument A note of denunciation of Alonsorsquos crime leaves him much humbled and penitent and confirms his belief that his son is lost forever But Sebastian and Antonio shows some courage instead of repentance They wish to kill the spirits or devils if it appears

3

Of my instruction hast thou nothing bated (Line 85-93)

In what thou hast to say So with good life

And observation strange my meaner ministers

Their several kinds have done My high charms work

And these mine enemies are all knit upIn their distractions They now are in my

powerAnd in these fits I leave them while I visitYoung Ferdinand whom they suppose is

drownedAnd his and mine loved darling

Methought the billows spoke and (Line 96-99)

told me of itThe winds did sing it to me and the

thunderThat deep and dreadful organ-pipe

pronouncedThe name of Prosper It did bass my

trespass

These are the words of contrition coming from Alonso Ariel has driven him to a deep repentance for conspiring with Antonio against Prospero He now feels a sincere remorse It appears to him that all the elements of nature the sea-waves the wind and the thunder proclaimed with a loud voice the name of Prospero and the crime Alonso had committed against him They are calling upon him to repent There is a deep storm raging in Alonsorsquos breast and the echoes of that storm are ringing in his ears like the clear note of a wind-instrument

Comment These are the words of sincere repentance wrung out of the conscience-stricken heart of Alonso Alonso who is the lesser villain is the first to give way to remorse under the effect of Arielrsquos speech The words of Ariel seem to him to be the voice of conscience speaking to him He is driven to desperation a state in which he might do violence to his life

(a) Identify the speaker State the context

Prospero the ruler of the island is the speaker The famous banquet scene has been enacted very well Ariel and his junior spirits have played their roles excellently Prospero is glad to say words of praise for them(b) In what way the speakerrsquos instructions have been carried out

According to Prosperorsquos instructions a banquet was presented before the King of Naples and his companions when they were tired and hungry Just when they were preparing to eat the feast the banquet was suddenly removed by exercising supernatural powers All this was done by Ariel Prosperorsquos chief assistant and a powerful spirit

Ariel not only made the feast disappear but also delivered his speech blaming the King and his two companions for their past wicked deeds He warned them to repent for their misdeeds or suffer forever on that uninhabited island

(c) Who are referred to as lsquomeaner ministersrsquo What have they done

Prospero refers as lsquomeaner ministersrsquo to his other lesser spirits who were assisting Ariel in presenting a scene before the kingrsquos party They entered the scene to the accompaniment of music They assumed several strange shapes and brought in a banquet Then they danced about it with gentle actions of salutations thus inviting the King and others to eat the feast

These spirits play their role again when Ariel in the shape of a harpy quits the scene These shapes enter again and dancing with mocking gestures carry away the table

(d) Who are the speakerrsquos enemies What has happened to them

King of Naples Alonso his brother Sebastian and the present Duke of Milan Antonio (Prosperorsquos own brother) are Prosperorsquos enemies With the turn of events they have all been washed ashore on the island which is ruled by Prospero the great magician Actually this happened after the shipwreck caused by a storm which was raised by Prospero with the purpose of bringing these people to his island Prosperorsquos spirits have already confused and terrified these enemies and they are under Prosperorsquos control He can treat them as he likes

(e) What does he say about Ferdinand Explain what is meant by ldquohellip his and mine darlingrdquo

Prospero knows that Alonsorsquos son prince Ferdinand is alive though his father thinks that the prince has been drowned

Prospero refers to his daughter Miranda who is dear to him She is also very dear to Prince Ferdinand who has fallen in love with her They are waiting to be married soon for which they have received Prosperorsquos consent

4

ALONSO O it is monstrous monstrous (Line 95-102)

Methought the billows spoke and told me of it

The winds did sing it to me and the thunderThat deep and dreadful organ-

pipe pronouncedThe name of Prosper It did bass

my trespassTherefore my son ithrsquo ooze is

bedded andIll seek him deeper than eer

plummet soundedAnd with him there lie mudded

(a) In what way does Alonso express his horror when his conscience is awakened by Arielrsquos words

When Alonsorsquos conscience is awakened by Arielrsquos words he expresses his horror at what he has heard He gets the feeling that the waves of the ocean the wind and the loud thunder have spoken to him and uttered the name of Prospero Because of being reminded of his crime in a very loud and rough voice he comes to realize that he has lost his son for his past misdeeds

(b) What does Alonso imagine about his son What does Alonso want to do in his desperate state

Alonso imagines that his son is lying in the mud at the bottom of the sea He feels desperate that he wants to drown himself in the ocean deeper than the plumb-line has ever gone He wants to lie with his son at the bottom of the sea

(c) How do Sebastian and Antonio want to face the evil spirits

Sebastian says that he is not at all afraid of what the harpy has said and that he is prepared to fight any number of such monsters if they appear before him only one at a time Antonio says that he would support Sebastian in the fight against the fiendsyyy

(d) Why does Gonzalo ask Adrian to follow the three men

Gonzalo tells Adrian that all the three men namely Alonso Sebastian and Antonio are in a wild and reckless mood The thought of the heinous crime of which they are guilty has begun to torment their minds So he asks Adrian to follow those three men without loss of time and prevent them from doing anything which the turmoil in their minds might lead them to do

(e) What opinion do you form of Alonso from the above extract

Alonso who is the lesser villain is the first to give way to remorse under the effect of Arielrsquos speech The words of Ariel seem to him to be the voice of conscience speaking to him He is driven to desperation a state in which he might do violence to his life

Subject =Accounts

Ac-12 15420 topic-pL Appropriation ac

PROFIT AND LOSS APPROPRIATION ACCOUNT

MEANING AND PREPARATIONProfit and Loss Appropriation Account is merely an extension of the Profit and Loss Account of the firm The profit of the firm has to be distributed amongst the partners in their respective profit sharing ratio But before its distribution it needs to be adjusted All Adjustments like partnerrsquos salary partnerrsquos commission interest on capital interest on drawings etc are made in this account These adjustments will reduce the amount of profit for distribution This adjusted profit will be distributed amongst the partners in their profit sharing ratio To prepare it at first the balance of Profit and Loss Account is transferred to this account The journal entries for the preparation of Profit and Loss Appropriation Account are given below

1 for transfer of the balance of Profit and Loss Account to Profit and Loss Appropriation Account

(a) In case of Net Profit

Profit and Loss Ac helliphelliphelliphelliphellipDrTo Profit and Loss Appropriation Ac(Net Profit transferred to Profit and Loss Appropriation Ac)

(b)In case of Net Loss

Profit and Loss Appropriation Achelliphelliphellip DrTo Profit and Loss Ac(Net Loss transferred to Profit and Loss Appropriation Ac)

2 for Interest on Capital

For transferring on Interest on CapitalProfit and Loss Appropriation Achelliphelliphellip DrTo Interest on Capital Ac(Interest on capital transferred to Profit amp Loss Appropriation Ac)

3 for Interest on Drawings

For transferring Interest on Drawings Interest on Drawings Achelliphelliphelliphelliphelliphellip DrTo Profit and Loss Appropriation Ac(Interest on drawing transferred to Profit amp Loss Appropriation Ac)

4 For Partnerrsquos SalaryFor transfer of partnerrsquos SalaryProfit and Loss Appropriation Achelliphellip DrTo Salary Ac(Salary transferred to profit amp Loss Appropriation Ac)

5 For Partnerrsquos CommissionFor transferring commissionProfit and Loss Appropriation Achelliphelliphellip DrTo Commission Ac(Commission transferred to Profit and Loss Appropriation Ac)

6 For Transfer of agreed amount to General ReserveProfit and Loss Appropriation Ac helliphellipDrTo General Reserve Ac(Transfer to General Reserve)

7 for share of Profit or Loss appropriation(a) If ProfitProfit and Loss Appropriation Achelliphellip DrTo Partnerrsquos CapitalCurrent Ac(Profit transferred to capitalcurrent Ac)(b) If LossPartnerrsquos Capital Current Achelliphelliphelliphellip DrTo Profit and Loss Appropriation Ac(Loss transferred to capitalcurrent Ac)

THE FORMAT OF PROFIT AND LOSS APPROPRIATION

Profit and Loss Appropriation Account for the year endedhelliphelliphelliphellip

Particulars Amount Particulars Amount

To PL Ac (loss) By pL Ac (profit)

To Interest on capital BY Interest on drawings

To partner`s commission by Partner`s capital Ac ( loss)

To Partner`s salary To Interest on partner`s loan To General Reserve To Partner`s Capital AC (Profit)

Subject= Economics

MOVEMENT ALONG THE DEMAND CURVE (CHANGE IN QUANTITY DEMANDED)In law of demand you have already studied the inverse relationship between price and quantity demanded When quantity demanded of a commodity changes due to change in its price keeping other factors constant it is called change in quantity demanded It is graphically expressed as a movement along the same demand curve There can be either a downward movement or an upward movement along the same demand curve Upward movement along the same demand curve is called contraction of demand or decrease in quantity demanded and downward movement along the same demand curve is known as expansion of demand or increase in quantity demanded

Extention of demandd

price (rs)p A

B Extentionp1 d

Q Q1

Quantity demanded ( in units)

Contraction of demandd

p2 Ccontraction

p APrice (Rs)

d

Q2 Q

Quantity demanded (in units)

Explanation of movement of demand A fall in price from OP to OP1 leads to increase in quantity demanded from OQ to OQ1 (expansion of demand) resulting in a downward movement from point A to point B along the same demand curve DD When Price rises from OP to OP2 quantity demanded falls from OQ to OQ2 (contraction of demand) leading to an upward movement from point A to point C along the same demand curve DD

  • Activity Series of Metals
    • Drawbacks of Rutherfordrsquos model of atom
      • Electromagnetic radiations
      • Properties of electromagnetic radiations
      • Characteristics of electromagnetic radiations
        • Plancks Quantum Theory-
        • Photoelectric effect
          • Intext Questions
            • Logic gates
            • Digital systems are said to be constructed by using logic gates These gates are the AND OR NOT NAND NOR EXOR and EXNOR gates The basic operations are described below with the aid of truth tables
            • AND gate
            • Example
Page 11:  · Web viewSubject. Topic. Summary. Execution. English 1 . Chapter 1 naming words . Page 8. Write the names of these pictures:- Person:-1. father. 2.Firefighter 3.doctor 4 ...

Therefore the multiplication of whole numbers is distributive over their additionExample If x = 5 y = 3 and z= 2Therefore x times (y + z) = 5 times (3 + 2) = 5times5 =25And x times y + xtimes z = 5times3 +5times2 =15 +10 = 25Again x times (y ndash z ) = x times y ndash x timesz Therefore 5 times ( 3 - 2) = 5 times1 = 5 and 5times3 ndash 5 times2 = 15 ndash 10 = 5Therefore the multiplication of whole numbers is also distributive over their subtraction if y is greater than z

5 Existence of identity If x is a whole number then

X times1 = x 1 times x = xTherefore we can write x times1 = 1 times xTherefore the multiplication of any whole number with 1 is the number itselfTherefore we can say that 1 is multiplicative identity or identity element for multiplicationExample 5 times1 = 5 1 times 5 = 5 Therefore 5 times 1 = 5

6 Multiplicative inverse If x is any whole number ( x is not equal to zero ) then its multiplicative inverse will be 1xSo x times 1x = 1 but 1x is a whole number if x = 1For other values of whole number 1x is not a whole number therefore we can write its multiplicative inverse does not exists

7 Cancellation law of multiplication If x y and z are three non- zero whole numbers then x times y = x times z

Or y = zExample 9 times y = 9 timeszTherefore y = z

Class VIISubject Topic Summary Execution

English 2 Sentences based on meanings

Kinds of sentences

Assertive or declarative to convey information or simply make a statement

Interrogative to ask different types of questions

Imperative to command or instruct someone or make a request

Exclamatory to express strong feelings and emotions

Exercise c1 What a nice compliment that is

That is a nice compliment2 How well- behaved the children

areThe children are very well-behaved

3 What great chefs we areWe are great chefs

4 What a shame it isIt is a shame

5 What a fantastic idea you haveYou have a fantastic idea

Homework 6 -10English

LiteratureThe Listeners III) Answer the following questions-

d) Identify two words used in the poem to give the poem an eerie atmosphereAns- Two words used to give the poem an eerie atmosphere are ldquogreyrdquo and ldquophantomrdquo

e) Who do you think are the inmates of the houseAns- I think the inmates of the house are phantom who once used to dwell in it

f) Why was the poet ldquoperplexed and stillrdquoAns- He was lsquo perplexed and stillrsquo because he was expecting an answer from the inmates of the house But despite of repeated calls there was no response

CHEMISTRY Chapter 2 ndashElement and Compound

Activity Series of MetalsThe activity series is a chart of metals listed in order of declining relative reactivity The top metals are more reactive than the metals on the bottomMetal SymbolReactivity

Lithium Li displaces H2 gas from water steam and acids and forms hydroxides

Potassium K

Strontium Sr

Calcium Ca

Sodium Na

Magnesium Mg displaces H2 gas from steam and acids and forms hydroxides

Aluminum Al

Zinc Zn

Chromium Cr

Iron Fe displaces H2 gas from acids only and forms hydroxides

Cadmium Cd

Cobalt Co

Nickel Ni

Tin Sn

Lead Pb

Hydrogen gas

H2 included for comparison

Antimony Sb combines with O2 to form oxides and cannot displace H2

Arsenic As

Bismuth Bi

Copper Cu

Mercury Hg found free in nature oxides decompose with heating

Silver Ag

Palladium Pd

Platinum Pt

Gold Au

Answer the following Q)Difference Between Metals And Nonmetals

Metals Nonmetals

These are solids at room temperature except mercury

These exist in all three states

These are very hard except sodium

These are soft except diamond

These are malleable and ductile

These are brittle and can be breakdown into pieces

These are shiny These are non-lustrous except iodine

Electropositive in nature Electronegative in nature

Have high densities Have low densities

Math Number System

Chapter Fraction

Study item Some solved sums from exercise 3(B)1) For each pair given below state whether it from like fractions or unlike

fractions (i) 58 and 78

= Like Fraction because denominators same(ii) 815 and 821

= Unlike Fraction because denominators are not same

(iii) 49 and 94 = Unlike Fraction

2) Convert given fractions into fractions with equal denominators(iii) 45 1720 2340 and 1116Solution Given fraction 45 1720 2340 and 1116Therefore the LCM of 5 20 40 and 16 is 80Therefore 45 = 4times165times16 = 64801720 = 17times420times4 = 68802340 = 23times240times2 = 4680 1116 = 11times516times5 = 5580

3) Convert given fractions into fractions with equal numerators(iii) 1519 2528 911 and 4547Solution Given fractions 1519 2528 911 and 4547Therefore the LCM of 15 25 9 and 45 is 2251519 = 15times1519times15 = 225285 2528 = 25times928times9 = 225252911 = 9times2511times25 = 2252754547 = 45times547times5 = 225235

4) Put the given fractions in ascending order by making denominators equal

(iii) 57 38 914 and 2021Solution Given fraction 57 38 914 and 2021Therefore the LCM of the denominators is 16857 = 5times247times24 = 12016838 = 3times218times21 = 63168914 = 9times1214times12 = 1081682021 = 20times821times8 = 160168Therefore ascending order 63168lt108168lt120168lt160168Therefore ascending order of given fractions38lt914lt57lt2021

COMPUTER CHAPTER-1COMPUTER FUNDAMENTALS

DONE IN THE PREVIOUS CLASSES PAGE 10CWRITE TRUE AND FALSE

1 True2 False3 False4 False5 True

GEOGRAPHY CHAPTER 7EUROPE

CHAPTER COMPLETE 1)Europe is home to a famous mountain range called the Alps

2)River Rhine originates in Switzerland

3)The Eiffel Tower one of the tallest structures in Europe

4) Vatican City is one of the most densely populated European countries

5)Sognefjordin Norway is the largest fjord in Norway

Class VIIISubject Topic Summary Execution

MATHEMATICS Ch 6Sets

Exercise 6 (D)1 Given A = x x isin N and 3iquest x le 6 and B = x x isin W and xlt4 find (i) Sets A and B in roster form (ii) A cup B (iii)

A cap B(iv) A ndash B (v) B ndashA

Solution (i) A = 456 and B = 0123

(ii) A cup B = 0123456 (iii) A cap B = ϕ (iv) A ndash B = 456 (v)B ndash A = 0123

3 If A = 56789 B = x 3 lt x lt 8 and x isin W and C = x xle5 and x isin N Find (i) A cup B and (A cup B) cup C (ii) B

cup C and A cup ( B cup C)

(iii) A cap B and (A cap B) cap C (iv) B cap C and A cap (B cap C)

Is (A cup B) cup C = A cup (B cup C)

Is (A cap B) cap C = A cap (B cap C)

SolutionA = 56789 B = 4567 C = 12345

there4 (i) A cap B = 456789 and (A cup B) cup C = 123456789

(ii) B cup C = 1234567 and A cup ( B cup C) = 123456789

(iii) A cap B = 567 and (A cap B) cap C = 5

(iv) B cap C = 45 and A cap (B cap C) = 5

Now (A cup B) cup C = 123456789

And A cup ( B cup C) = 123456789 there4 (A cup B) cup C = A cup (B cup C)

Again (A cap B) cap C = 5 and A cap (B cap C) = 5

there4 (A cap B) cap C = A cap (B cap C)

4 Given A = 012345 B = 02468 and C = 0369 Show that (i) A cup (B cup C) = (A cup B) cup C ie the union

of sets is associative (ii) A cap (B cap C) = (A cap B) cap C ie the intersection of sets is associative

SolutionNow B cup C = 0234689 and A cup B = 01234568

there4 A cup (B cup C) = 012345689 and

(A cup B) cup C = 012345689

So (i) A cup (B cup C) = (A cup B) cup C ie the union of sets is associative

Again B cap C = 06 and A cap B = 024

there4 A cap (B cap C) = 0 and (A cap B) cap C = 0

So (ii) A cap (B cap C) = (A cap B) cap C ie the intersection of sets is associative

Physics Chapter 2 Physical Quatites and Measurements

Here We Will Do Some QuestionsRelated To Chapter 2

A density bottle has a marking 25 mL on it It means that

1 the mass of density bottle is 25g

2 the density bottle will store 25 ml of any liquid in it

3 the density bottle will store 25 ml of water but more volume of liquid denser than water

4 the density bottle will store 25 ml of water but more volume of a liquid lighter than water

Solution 2 the density bottle will store 25 ml of any liquid in it

COMPUTER CHAPTER-2Spreadsheet Functions and Charts

SELECTING RANGE IN ROWSCOLUMNSWHEN TWO OR MORE CELLS ARE SELECTED IT IS CALLED A RANGEA RANGE OF CELLS CAN BE FORMED IN TWO WAYS--a) SELECTING RANGE BY USING THE MOUSEb) SELECTING RANGE BY USING THE KEYBOARD

Q1)WRITE THE STEPS TO SELECT PARTIAL RANGE IN A ROW

Ans)THE STEPS ARE-6 SELECT THE ROW7 BRING THE CELL POINTER TO THE DESIRED

LOCATION FROM WHERE YOU WANT TO START YOUR SELECTION

8 CLICK THE LEFT MOUSE BUTTON AND KEEP DRAGGING TO YOUR RIGHT TILL YOU REACH THE LAST CELL TO NE SELECTED

RELEASE THE MOUSE BUTTON

GEOGRAPHY Asia

CLIMATE

Asia experiences great extremes of climate Jacobabad in the Sind province of Pakistan is one of the hottest places in the WorldVerkhoyansk in Siberia is one of the coldest places in the WorldCherrapunji and Mawsynram in India are two wettest places in WorldArabia Tibet Gobi and Mongolia are extremely dry regionsFactors Affecting Climate of Asia-The factors influencing the climate of Asia are-

Factors Affecting Climate of Asia-Thoroughly read the table in page number 60

Latitudinal extent

Continentality

Relief features

Presence of low pressure trough

Jet streams

English Language The Sentence A complex sentence contains one independent clause and at least one dependent clause The dependent clause in a complex sentence is introduced with subordinating conjunctions or relative pronouns

Commonly Used Subordinating Conjunctions-Time after before while when since untilCause And Effect because now since as in order that soOpposition although though even though whereas while in spite ofCondition if unless only if whether or not even if in case(that)

Commonly Used Relative Pronouns-Who whose whom which whoever whomever whichever that

Class IXSubject Topic Summary Execution

1-BENGALI(2ND LANGUAGE)

ldquo বঙগভমিরপরমিrdquo াইকেলধসদনদতত

আচেগর পর উততর পচো-১ ২ ৩ এং নীচের পর টি াহির কা- ৪মহিbকাও গচেনা দেগা পহিচে অমত হরচে- ক) কার দো দেকান কহিতার অং( ) কতা দেক পরসঙগ কী উহিকতটির তাৎপযC আচোনা কচেরা৫দেসই ধনয নরকচে দোচেক যাচে নাহি ভচে মচেনর মহিeচের সাচেসচে সCন ক) কহির কায C ার উচেf(য হিক হিছ কহি কন কহিতাটি দেচেন) কহি কার কাচেছ হিমনহিত কচেরচেছনগ) কহি এই পহিথীচেত কাচের ধনয মচেন কচেরনঘ) কহি হিক রকম অমর তাাভ করচেত ান

Hindi 2nd lang

काकी(कतिसयारामशरण गपत)

इस कहानी म लखक न यह बतान का परयास निकया ह निक बचच अपनी मा स निकतना परम करत ह शयाम अबोध बालक ह वह अपनी मा क मरन क बा- उसन अपनी मा क कतिलए बहत रोया बा- म उस पता चला निक उसकी मा राम क घर चली गई ह आकाश म उडती हई पतग -खकर उस हरष हआ निक पतग क दवारा वह अपनी मा को नीच उतारगा इसक कतिलए वह अपनी निपता की जब स -ो बार सवा रपया निनकालकर पतग और -ो मोटी सी मन वाली अपन भाई स काकी एक कागज पर कतिलखवा कर पतग म कतिशव का दि-यानिनकालकर पतग और -ो मोटी सी मन वाली अपन भाई स काकी एक कागज पर कतिलखवा कर पतग म कतिचपका दि-याभोला और शयाम कोठरी म रससी बाधनी रह थ तभी उसक निपता करोध म आकर उन स पछ निक कया उनकी जब स रपया निनकाला हभोला डर क मार बताया निक शयाम इस पतग क दवारा अपनी काकी को राम क यहा स उतारना चाहता हनिवशशवर(शयाम क निपता)न फटी पतग उठाकर -खी तो उस पर काकी कतिलखा थावह हत बजिa होकर वही खड रह गएउनहोन सोचा निक मन अपन पतर को मारा जोनिक अनजान और निन-dरष थावह अपनी मा कोनिकतना पयार करता ह

उस दि-न बड सवर शयाम की नी- खली तो -खा निक घर भर म कोहराम मचा हआ ह

क) घर म कोहराम कयो मचा हआ था शयाम को कया लगा

ख) काकी को ल जात समय शयाम न कया उपदरव मचाया

ग) काकी क बार म उस कया बताया गया कया सतय उस कतिछपा रहा

घ) वह बठा-बठा शनय मन स आकाश की ओर कयोकरता

उततरक) शयाम की मा का -हात हो गया था इसकतिलए

घर म कोहराम मचा हआ था शयाम की लगा निक उसकी मा सफ- कपडा ओढ हए भमिम पर सो रही ह

ख) लोग जब उमा यानी शयाम की मा को उठाकर ल जान लग तब शयाम न बडा उपदरव मचाया लोगो क हाथ स झठ करवा उमा क ऊपर जा निगरा और बोला काकी सो रही ह उस कहा ल जा रह हो

ग) काकी क बार म बजिaमान लोगो न उस निवशवास दि-लाया निक उसकी का निक उसक मामा क यहा गई ह लनिकन सतय अमिधक दि-नो तक कतिछपाना रह सका आसपास क अबोध बालको क मह स यह बात परकट हो गई निक उसकी मा का -हात हो गया ह

घ) कई दि-नकई दि-न लगातार रोत-रोत उसका रोना तो शान हो गया पर उसक ह-य म शोक भर गया था वह चपचाप बठा आकाश की और टाका करता निक शाय- उसकी काकी कही दि-ख जाए

ldquoदि-न उसन ऊपर आसमान म पतग उडती -खी न जान कया सोच कर उसका निहर-य एक-म खिखल उठाrdquo

क) निकसन पतग ऊपर उडत -खी और वह कयो खश हआ

ख) उसन अपन निपता स कया कहा उनका कया उतर थाश

ग) उसन निफर कया निकया और निकसन उसकी सहायता की

घ) उसकी योजना कया थी उततर -क) शयाम न एक दि-न आसमान म पतग उडती

-खी तो उसन सोचा निक पता आसमान म राम क यहा जाकर रकगी वही पर मरी काकी ह यह सोचकर वह बहत खश हआ

ख) उसन अपन निपता स कहा काका मझ एक पतग मगा -ो उसक निपता न भटक हए मन क भाव स कहा निक मगा -ग यह कह कर उ-ास भाव स वह कही और चल गए पतग नही आई

ग) उसन चपचाप निवशशवर क टगहए कोट स एक चवननी निनकाल ली और सखिखया -ासी क लडक भोला की सहायता स एक पतग मगवानी भोला उसकी बराबर उमर का ही था

घ) उसकी योजना यह थी निक वह अपनी पतग को आकाश म राम क यहा भजगा और उस पतग क सहार उसकी काफी नीच उतर जाएगी इस योजना पर उस परा निवशवास था इसकतिलए वह और भोला -ोनो यह काम करन म लग गए

Continue to nexthelliphellipEVS CHAPTER - 1

(UNDERSTANDING OUR ENVIRONMENT)

Sustainable development

The development that meets the needs of the present without compromising the ability of future generations to meet their own needs is called Sustainable development

Sustainable societies ndash

An environmentally sustainable community is one that meets the current and future basic resource needs of its people in a just and equitable manner without compromising the ability of future generations to meet their basic needs

Q ) What are Eco Villages

Ans - Eco village are the urban or rural communities of people who strive to integrate a supportive social environment with a low impact way of life

Q ) To ensure sustainable development the depletion of renewable resources should not take place at a rate faster than their regeneration Justify your answer

Ans ndash Renewable resources do not have a fixed quantity - more can always be

generated However if the rate of use exceeds the rate of renewal - that is the

source is used more than its being recreated - its continued use will become

used up faster than it can regenerate

To promote sustainable society the following things need to be done ndash

1 Using renewable energy sources 2 By improving the quality of human

health 3 By promoting sustainable agriculture 4 By forming ecovillage

it will eventually be entirely depleted So Toensure sustainable development the depletion of

renewable resources should nottake place at a rate faster than their regeneration

Q ) What do you mean by Sustainable societies

Ans - Sustainable societies are defined as towns and cities that have taken steps to remain healthy over the long term These communities value healthy ecosystems use resources efficiently and actively seek to retain and enhance a locally based economy Sustainable development concerns everybody in a society

Q ) What are the effects of pollution on human health

Ans ndash Some health problem occurs due to air pollution are ndash

Respiratory diseases Cardiovascular damage Fatigue headaches and anxiety Irritation of the eyes nose and throat Damage to reproductive organs Harm to the liver spleen and blood Nervous system damage

Some health problem occurs due to water pollution are ndash

Typhoid Cholera Dysentry Jaundice

Some health problem occurs due to noise pollution are ndash

Fatigue headaches and anxiety High blood pressure Hearing damage

Physics Motion in 1D First go through previous notes Now here we will solve some numerical related to that

Question 3What information about the motion of a body is obtained from the displacement-time graphSolution 3From displacement-time graph the nature of motion (or state of rest) can be understood The slope of this graph gives the value of velocity of the body at any instant of time using which the velocity-time graph can also be drawn

Question 4(a)What does the slope of a displacement-time graph represent(b)Can displacement-time sketch be parallel to the displacement axis Give a reason to your answerSolution 4(a) Slope of a displacement-time graph represents velocity(b) The displacement-time graph can never be parallel to the displacement axis because such a line would mean that the distance covered by the body in a certain direction increases without any

increase in time which is not possible

Chemistry Language of Chemistry

How to balance a chemical equationThere are two methods of balancing an equation(i)Hit and trial method(ii)Partial equation methodBalancing by hit and trial methodThis method consists of counting the number of atoms of each elements on both sides and trying to equalize themTake the following steps(i)Count the number of times (frequency) an element occurs on either side(ii)The element with the least frequency of occurrence is balanced first(iii)When two or more elements have the same frequencythe metallic element is balanced firstExample-1 On heatinglead nitrate decomposes to give lead dioxidenitrogen dioxide and oxygenPb(NO3)2rarrPbO+NO2+O2

In this equationLead occurs twiceNitrogen occurs twiceOxygen occurs four timesSince lead is a metalbalance it firstThe number of atom of lead is equal on the two sidestherefore it needs no balancingNow balance nitrogenOn the reactant sidethere are two atoms of nitrogenwhile on the product side oneSomultiply the product containing nitrogenon the product sideby two Pb (NO3)2rarrPbO+2NO2+O2Nowthe number of oxygen atoms on the reactant side 6while on the product sideit is 7Somultiply the entire equation by 2except oxygen to get balanced equation2Pb(NO3)2rarr2PbO+4NO2+O2Multiplication by 2 is done only when atoms of all the elements except one element are balanced and the unbalanced atom occurs separately at least once and also there is a difference of only one such atom

Math Topic AlgebraChapter

Factorisation

Study item Difference of two squares a2 ndash b2 = (a+b) (a-b)1) (i) 4x2ndash 25y2

= (2x) 2 ndash (5y) 2= (2x + 5y) (2x - 5y)

(ii) 9x2 ndash 1= (3x)2ndash(1)2= (3x + 1)(3x ndash 1)

2) (i) 150 ndash 6a2= 6(25 ndash a2)= 6(5)2 ndash(a)2= 6 (5 + a) (5 ndash a)

(ii) 32x2 ndash 18y2=2(16x2 ndash 9y2)=2(4x)2 ndash (3y)2= 2(4x + 3y)(4x - 3y)3)(i) (x ndashy )2 ndash 9 = (x ndash y )2 ndash (3)2= (x ndash y + 3) (x ndash y ndash 3)(ii) 9(x + y) 2ndash x 2= (3)2(x + y)2 ndash (x)2=3(x + y)2 ndash (x)2= (3x +3y ) 2ndash(x)2= (3x + 3y + x)(3x +3y ndash x)= (4x + 3y) ( 2x + 3y )

Commercial studies

Basic accounting terms

Today I will give you some questions from the previous study material

Questions1) Define accounting2) What do you mean by debit and

credit

3) Explain the types of account4) Define the following terms

a) Assetsb) Capitalc) Purchased) Debtorse) Transactions

5) Name the types of accounts given below

a) Krishnas accountb) Machinery accountc) Royalty accountd) Salary accounte) Furniture accountf) Audit fee account

Economics Revision Today I will give you some revision questions

Questions1) What do you mean by the terms

rdquowantsrdquo2) Write the difference between

consumer goods and producer goods

3) Define the term utility 4) Explain the different types of utility5) Define

a) Total utilityb) Marginal utility

Subject Eng Literature (The Merchant of Venice ndash William Shakespeare)Topic Act I Scene 3 Lines 1 to 48 (Shylock hellip Cursed be my tribe if I forgive him) Date 16th April 2020 (5th Period)

[Students should read the original play and also the paraphrase given in the school prescribed textbook]Summary Questions amp Answers

This scene takes place in Venice and we are introduced to the rich Jew Shylock Bassanio and Shylock are talking and Bassanio tells Shylock that he wants a loan of three thousand ducats for three months on the personal security of Antonio

o Shylock feels glad because he will be able to bind down Antonio by means of a bond on account of the loan but he tells Bassanio that all the fortunes of Antonio being invested in the merchant ships on the sea it is difficult to depend upon his credit Even under such circumstances Shylock is willing to advance the money on the personal security of Antonio

o Bassanio then invites Shylock to dine with him Shylock says that he is prepared to do anything with the Christians but not eat or drink or pray with them

o While Bassanio and Shylock are talking Antonio appears on the scene Shylock does not seem to take any notice of Antonio but goes on brooding within

(1) SHYLOCK Ho no no no no- my meaning in (Line 15-26)saying he is a good man is to have you understand me that he is sufficient Yet his means are in suppositionhe hath an argosy bound to Tripolis another to the Indies I understand moreover upon the Rialto he hath a third at Mexico a fourth for England and other ventures he hath squanderd abroad Butships are but boards sailors but men there be land-rats and water-rats land-thieves and water-thieves I mean pirates and then there is the peril of waters winds and rocks The man is notwithstanding sufficientmdashthree thousand ducats mdashI think I may take his bond

(a) Who is talking in the beginning of this scene What does Bassanio want from Shylock How does Shylock feel

In the beginning of the scene Bassanio and Shylock are talking to each other Bassanio wants to borrow three thousand ducats from Shylock for three months on the security of Antonio Shylock feels glad at heart that he will get the opportunity of binding Antonio with a bond(b) What risks does Shylock weigh in advancing the money

Shylock says that Antonio has invested all his capital in trading by sea-going ships But the ships are made of wood and the sailors of those ships are ordinary human beings The wood can

himself how he hates Antonio because of his being a Christian because he abuses Shylock in public places Shylock decides that if ever he can get Antonio to his advantage he will teach him a lesson

come to harm and men can commit mistakes and thus the capital invested in ships may be lost Then there are other dangers The goods loaded on the ships can be damaged by rats and thieves which are found both on land and water The ships can also be harmed through sea-storms submerged rocks etc(c) What two important functions does this scene have

The scene has two important functions First it completes the exposition of the two major plot lines of the play Antonio agrees to Shylockrsquos bond ndash three thousand ducats for a pound of flesh and second and more important dramatically this scene introduces Shylock himself In this scene Shakespeare makes it clear at once why Shylock is the most powerful dramatic figure in the play and why so many great actors have regarded this part as one of the most rewarding roles in all Shakespearean dramas(d) Where does this scene take place What kind of treatment has Antonio been giving to Shylock What does Shylock say when Bassanio invites him to dine with him

The action of this scene takes place in Venice Antonio has been in the habit of behaving harshly with Shylock ndash spitting on his beard and footing him like a stranger cur When Bassanio invites Shylock to dine Shylock says that he is prepared to do anything with the Christians but not eat and drink or pray with them

(2) SHYLOCK How like a fawning publican he looks (Line 38-48)I hate him for he is a Christian

But more for that in low simplicity

He lends out money gratis and brings downThe rate of usance here with us in VeniceIf I can catch him once upon the hipI will feed fat the ancient grudge I bear him

He hates our sacred nation and he railsEven there where merchants most do congregateOn me my bargains and my well-won thriftWhich he calls interest Cursed be my tribeIf I forgive him

(a) What is the context in which these words are spoken and what is the idea expressed in it

These remarks are made by Shylock when he sees Antonio coming along after Bassanio told him that the merchant will be his surety for the bond The above mentioned passage reveals Shylockrsquos hatred for Antonio Shylock says that he hates Antonio because he is a Christian and also because he gives loan without taking interest on them thereby bringing down the rate of interest in Venice(b) Explain the meaning of the phrase lsquoa fawning publicanrsquo

The phrase lsquoa fawning publicanrsquo refers to Roman tax collector It is a term of contempt and hatred on the lips of a Jew lsquoFawning Publicansrsquo were Roman tax-gatherers whose ordinary bearings towards the Jews was bullying but whose false pose of lsquohumility and contritionrsquo is touched upon in the parable in New Testament(c ) What light does the above passage throw on the character

of Shylock

The above mentioned speech of Shylock reveals him to be a wicked character having an extreme greed for wealth His intense hatred for Antonio is unjustified He hates Antonio only because he is a Christian and because he lends money without taking any interest on it thereby adversely affecting Shylockrsquos business of lending money on high interest(d) What information do you gather about Antonio from the above given lines

Shylockrsquos statement throws a valuable light on the character of Antonio Antonio appears to be a good Christian and a good human being He helps the people in need by lending them money without charging any interest on it He is a man of simple and good nature This very goodness makes him Shylockrsquos enemy(e) What does Shylock debate within himself and when To whom are the lines mentioned above addressed to

When Bassanio asks the Jew to lend him three thousand ducats on Antoniorsquos surety Shylock begins to debate within himself as to how he should exploit the opportunity of a business deal with his old enemy Antonio

The lines mentioned above are not addressed to anyone The lines are a soliloquy ie a speech made by a character to himself and not meant to be heard by the other characters present

Class XSubject Topic Summary ExecutionEnglish

LiteratureThe Blue Bead 2nd part

Things took a turn for the worst and all of a sudden a crocodile attacked the woman biting on the womanrsquos leg At that moment Sibia got up sprinted grabbed the hay fork and stabbed the crocodile in the eye with all her power Immediately the crocodile let go and went away Sibia saw a small blue bead lying by the river she grabbed it Since she was poor she didnrsquot have necklace Shersquod always wanted one like the other women now she could make one with the blue bead After that she went home and told her mother all about it

Hindi 2nd

Langबड घर की बटी( मशी परमच-)

lsquoबड घर की बटी कहानी का उददशय मधयम वग की घरल समसया को सलझा कर सगदिठत परिरवार म मिमल जलकर परम स रहन का स-श -ना हघर म शानित सथानिपत करन की जिजमम-ारी नारी की होती ह यदि- नारी समझ-ार ह उसम धय और परिरवार क परनित परम ह तो कोई भी घटना परिरवार को निवघदिटत नही कर सकती या कहानी परिरवार को सगदिठत करत हए परम सौहा- स एक दसर की भावनाओ को समझ करउनका सहयोग करत हए जीवन यापन करन की पररणा -ती हमशी परमचदर जी न इस कहानी म सय परिरवार का परनितनिनमिधतव निकया ह यह कहानी बनी माधव सिसह जो गौरी पर क जमी-ार क उनक -ो पतरो की हशरी कठ लाल निबहारीशरीकात का निववाह एकजमी-ार घरान की पतरी आन-ी स हआ थाआन-ी न ख- को ससराल क वातावरण म ढाकतिलया थाएक दि-न आन-ी का अपन -वर लाल निबहारी स झगडा हो जाता ह -ोनो भाई एक दसर स अलग होन की कोकतिशश करत हसभी बह आन-ी न अपन मधर वयवहार स लाल निबहारी को

ldquoइन नतर निपरय गणो को बीए-इनही -ो अकषर पर नयोछावर कर दि-या था इन -ो अकषर न उनक शरीर को निनबल और चहर को कानित ही बना दि-या थाldquo

क) परसतत पकतियो म निकस वयकति क बार म कहा गया ह

ख) इन पकतियो म कौन स नतर निपरय गणो क बार म कहा गया ह

ग) बीए की निडगरी परापत कर लन पर भी उपय वयकति क सवभाव की कया निवशरषता थी

घ) यह नतर निपरय गण निकस वयकति म निवदयमान थ उसक वयकतितव की कया निवशरषता थी

उततर ndashक) परसतत पकति म गौरी पर गाव क जमी-ार

क बड बट शरीकात क बार म कहा गया ह उसन बहत परिरशरम और उ-म क बा- ba की निडगरी परापत की थी अब वह एक -फतर

घर छोडकर जान स रोक कतिलयाइस पर बनी माधव सिसह न कहा निक बड घर की बटी ऐसी ही होती ह जो निबगडा काम बना लती ह अतः शीरषक साथक ह बड घर की बटी आन-ी ह

म कमचारी थाख) भरा हआ चहरा चौडी छाती और डटकर

खाना आदि- एक सबजी ल जवान क गण मान जात ह परत शरीकात न इनही नतर निपरय गणो को अपनी पढाई पर नयोछावर कर दि-या था

ग) बीए की निडगरी परापत कर लन पर भी उपय वयकति(शरी कठ की शारिररिरक तौर पर निनबल और चहर स कानित ही लगत थ इतना ही नही वह मानकतिसक तौर पर भी निपछड हए थ पाशचातय सामाजिजक कथा उस घणा एव पराचीन सभयता का गणगान उनकी निवचारधारा क परमख अग थ

घ) यह नतर निपरय गण गौरीपर गाव क जमी-ार क छोट बट लाल निबहारी सिसह म निवदयमान थ वह सजीलाजवान था और भस का दध शर दध वह सवर उठकर पी जाता था

ldquoयही कारण था निक गाव की लललन आए उनकी निन-क थी कोई कोई तो उह अपना शतर समझन म भी सकोच ना करती थी सवय उनकी पतनी को इस निवरषय म उनस निवरोध थाldquo

क) उपय पकति म इस वयकति क बार म कहा गया ह

ख) गाव की लललन आए उनकी निन-ा कयो निकया करती थी

ग) उनकी पतनी का कया नाम था उनह निकस निवरषय म अपन पनित क निवरa था और कयो

घ) इस कहानी का कया उददशय ह Continue to next helliphelliphellip

Bengali 2nd Language

ফ ফটক না ফটক( কহিতা )

পর) ldquo(ান াধাচেনা ফটপাচেথ পাথচের পাড হিচেয় এক কাঠচোটটা গাছ কহিকহি পাতায় পার ফাটিচেয় াসচেছldquoক) কার দো দেকান কহিতার অং( ) lsquo(ান াধাচেনা ফটপাচেথ পাথচের পাডহিচেয়lsquo চেত কী দোঝাচেনা চেয়চেছ গ) আচো য অংচে(lsquo এক কাঠচোটটা গাছ lsquoচেত কী দোঝাচেনা চেয়চেছ ঘ) ldquoকহিকহি পাতায় পার ফাটিচেয় াসচেছldquo ----- একথার পরকত অথC কী উততর ) ক) আচো য অং(টি পর যাত কহি সভা4 মচোপাধ যাচেয়র দো lsquoফ ফটক না ফটকrsquo কহিতার অং()কহি সভা4 মচোপাধ যায় হিছচেন দেপরচেমর কহি দেপরমচেক নানা ভহিঙগমায় হিতহিন ফটিচেয় তচেচেছন দেপরম মানচের স মচেতC র সঙগী কহিতার কহিতায় এক রb সb হচেয়র দেপরম াগরচেনর কথা চেচেছন (ান অথCাৎ দেযাচেন দেকান রস দেনই দেযাচেন দেকান মহিনতা দেনই অথ তার মধ দেযও দেপরম থাকচেত পাচের একথাই কহি তচে ধরচেত দেচেয়চেছন একটি মানচে4র মচেন দেযাচেন দেকামতার দেকান সথান দেনই পাথচেরর মচেতা হিনরসতার মচেনর মধ দেযও দেয দেপরম আসচেত পাচের দেস কথাই কহি চেচেছনগ)নারীচের যথC দেপরচেমর ছহি এই কহিতায় অকপচেট উচেঠ এচেসচেছ কহি এই কহিতায় কাটচোটটা গাছ কথাটি যার কচেরচেছন নারী দেয দেপরম দেথচেক হিতাহিত এং দেসই দেপরম সঠিক সমচেয় না পাওয়ার ন য দেপরম সমপচেকC হিচেr4 গৈতরী য় দেপরচেমর দেয গৈহি(ষট য মাধযC য সরসতা দেকামত এই সমসতর হিপরীত যথা রbতা শষকতা কচেঠার তা পরভহিত দোঝাচেত এক কাঠচোটটা গাছ কথাটি যার কচেরচেছনঘ) এাচেন এক নারীর যথC দেপরচেমর কথা হিনহিCপত ভাচে চেচেছন কহি অসমচেয় নারীর ীচেন দেপরম দেচেগচেছ এতহিন তার হয় রb কচেঠার হিছ দেপরচেমর অভাচে ঠাৎ দেসই শষক মরভহিমচেত সচের আভাস এচেসচেছ দেপরম দেযন 4Cার স(ীত তাই পরায় মত গাচেছ কহিকহি পাতা গহিচেয় উচেঠচেছ

Biology Chapter - 01Controlling Air Pollution

Today we will discuss how we control air pollution from domestic combustion

Q1Describe any five ways of reducing air pollution from domestic sources bull The number of pollutants in the air is verylarge and we always try to control them byfollowing ways

i) Solar cooker and solar heater It use no fuel reduce damage of environment by fuel use or reducing deforestation It maintains coolness of house It releases very less orno oil gas or grease

ii) Piped natural gas (PNG) It emits very less by products into the atmosphere As it isdistributed through pipe lines so there iscontinuous supply of fuel is possible

iii) Liquefied Petroleum Gas (LPG) It hasa higher heating value LPG doesntcontain sulphur so it burns a lot cleanerenergy sources It releases very less oralmost no fume in air

iv) Electricity based cooking Emission free cooking alternative for urban dwellers causeselimination of adverse health impactsofindoor air pollution It helps to avoid theinconveniences associated with procurement of LPG

v) Biogas It contains 75 methane whichmakes it an excellent fuel It burns without smoke and biogas plant leaves no residue like ash in wood charcoal etc Thus it isaclean fuel

Economics

Factors of Production

Today firstly we would recall the last class for 5 mins and then we would proceed with the further topics of the chapter

The concept meaning of land characteristics of land and importance of land to be repeated for the absentees as well as the students who were there in the class the previous day

Today we will start with the last portion of land before it the meaning of land to be repeated onceAs by now we all know that

Questions1What do you mean by productivity of landAnswer By productivity of land we mean the capacity of a piece of land to produce a crop

Thus it refers to the average output per unit of landSay per acre per hectare etc= (OutputArea of land)

2 What are the factors influencing the productivity of landAnswer

Natural factors Productivity of land is largely determined by the natural

Land is defined to include not only the surface of the earth but also all other free gifts of nature(for example mineral resources forest resources and indeed anything that helps us to carry out the production of goods and services but is provided by nature free of cost)

We will move on to the last portion of land by discussing Productivity of Land

By productivity of land we mean the capacity of a piece of land to produce a crop

Thus it refers to the average output per unit of land

Say per acre per hectare etc= (OutputArea of land)

With this we shall proceed further with the main factors that determine the productivity of land

Natural factors Human factors Improvements on land Location of land Organisation Ownership of land Availability of capital Proper use of land State help

Note economic development of a country depends upon the quality of its land If the land is fertile it will quicken the pace of development of the country

qualities of land such as fertility etc

Human factors Land cannot produce anything by itself Man has to apply labour on it to produce for himself So productivity of land depends on the knowledge and skills of workers

Improvements on land production of land is affected by land development measures like provision of well or tubewell irrigation proper drainage

State help The government of a country especially less developed country can play a vital role in improving the agricultural productivity by providing better irrigation facilities

Organisation Productivity of land also fdepends upon the way how the factors of production like labour and capital are organised

In order to increase productivity trained workers modern implements scientific methods good seeds are all essential

3 lsquoImproved technology affects the productivity of landrsquo Explain this statement with the help of suitable example Answer Use of improved technology raises the productivity of land Example By using HYV seeds chemical manures and modern machines per hectare output increases

Physics Force (Summary)

Question Write the expression for the moment of force about a given axisSolutionsThe expression for the moment of force is given byMoment of force about a given axis = Force times perpendicular distance of force from the axis of rotationQuestion What do you understand by the clockwise and anticlockwise moment of force When is it taken positiveSolutionsIf the effect on the body is to turn it anticlockwise moment of force is called the anticlockwise moment and it is taken as positive while if the effect on the

body is to turn it clockwise moment of force is called the clockwise moment and it is taken as negative

Math Topic Commercial Mathematics

Chapter Goods and services Tax

Study item Some solved sums from exercise ndash 1 A retailer buys a TV from a wholesaler for Rs 40000 He marks the price of the TV 15 above his cost price sells it to the consumer at 5 discount on the marked price If the sales are intra ndash state and the rate of GST is 12 find

(i) The marked price of the TV(ii) The amount which the consumer pays for the TV(iii) The amount of tax (under GST) paid by the retailer to the central

Government(iv) The amount of tax (under GST) received by the State Government

Solution As the sales are intra- state sale and the rate of GST 12 So GST comprises of 6 CGST and 6 SGSTTherefore a retailer buys a TV from a wholesaler for Rs 40000Therefore the amount of GST collected wholesaler from the retailer or paid by retailer to wholesalerCGST = 6 of Rs 40000 = Rs(6100 times40000) =Rs 2400SGST = 6 of Rs 40000 = Rs (6100 times 40000) =Rs 2400Therefore wholesaler will pay Rs 2400 as CGST and Rs 2400 as SGSTTherefore amount of input GST of retailer Input CGST = Rs 2400 and input SGST = Rs 2400Again the retailer marks the price of the TV 15 above his cost price(i) The marked price of the TV

= Rs 40000 + Rs 40000times15= Rs 40000 + Rs 40000times 15100= Rs 40000 + Rs 6000Rs 46000But the retailer sells it to consumer at 5 discount on the marked priceCost price after discount = Rs 46000 ndashRs46000times 5100 =Rs 46000 ndashRs 2300= Rs 43700Therefore the amount of GST collected retailer from consumer or paid by consumer to retailerCGST = 6 of Rs 43700 =Rs ( 6100 times43700)Rs 2622SGST = 6 of Rs 43700 = Rs (6100 times 43700) =Rs 2622Amount of the output GST of retailer Output CGST = Rs 2622 and output SGST = Rs 2622

(ii) The amount which the consumer pays for the TV= cost price of TV to consumer + CGST paid by consumer + SGST paid by consumer= Rs 43700 + Rs 2622 + Rs 2622= Rs 48944

(iii) The amount of tax (under GST ) paid by the retailer to the central Government=CGST paid by retailer = output CGST ndash input CGST=Rs 2622 ndash Rs 2400=Rs 222

(iv) The amount of tax ( under GST ) received by the State Government = SGST paid by wholesaler + SGST paid by retailer= Rs 2400 + output SGST ndash input SGST=Rs 2400 + Rs 2622 ndash Rs 2400=Rs 2400 + Rs 222= Rs 2622

Commercial studies

Stakeholders Today I am going to give some revision questions from the previous study material

Questions1) State the two expectations of

employees from a business concern2) Give two distinctions between

stakeholder and shareholder3) Give two difference between

internal stakeholders and external stakeholders

4) Give two expectations of suppliers from a business organisation

5) Who is a stakeholder in commercial organisations

Chemistry Periodic Table

Merits of Mendeleevrsquos Periodic law are as follows - 1He grouped the elements on the basis of atomic mass 2 He left gaps for undiscovered elements like Gallium Scandium germanium Also he left a full group vacant for undiscovered inert gases 3 He could predict proportions of several elements on basis of their position in periodic table like Ga Sc etc 4He could predict errors in atomic weights of some elements like gold platinum etc

Anomalies in Mendeleevrsquos Periodic law are as follows - 1 Position of isotopes could not be explained 2 Wrong order of atomic masses could not be explained

For example- as Arnur atomic mass 40 come first and K with low atomic mass (30) should come later but k should be placed first

According to Bohrrsquos Modern Periodic table properties of elements are periodic functions of their atomic numbers

So when elements are arranged according to increasing atomic numbers there is periodicity in electronic configuration that leads to periodicity in their chemical properties

It consists of horizontal rows (Periods) Vertical column (Groups)

There are 7 period and 12 groups in this long form of periodic table

Ist period has 2 elements IInd period has 8 elements IIIrd period has 8 elements IVth period has 18 elements Vth period has 18 elements VIth period has 32 elements VIIth period hs rest of elements

Note - The number of valence electrons in atom of elements decides which elements will be first in period and which will be last

In group- 1 to 2 gp and 13 to 17 contain normal elements 3 to 12gp ndash transition elements 57 to 71 - lanthanides 89 to 103 - Actinides

Left hand side ndash metals Right hand side ndash nonmetals

Note- Hydrogen element has been placed at top of Ist group Electronic configuration of H is similar to alkali metal as both have 1 valence electron

V electron of gp I element -- 1 V electron of gp 2 element -- 2 V electron of gp 13 element -- 3 V electron of gp 14 element -- 4 V electron of gp 15 element -- 5 V electron of gp 16 element --6 V electron of gp 17 element -- 7 V electron of gp 18 element -- 8

English 1 Transformation of sentences

Sentences A sentence is a group of words which makes complete sense

Exercise 2Change the following sentences from

a Assertive sentencesb Imperative sentencesc Interrogative sentencesd Exclamatory sentences

Sentences can be changed from one grammatical form to another without changing the meaning of the sentence This is known as transformation of sentences

assertive to interrogative1 Nobody would like to be a fool

Who would like to be a fool2 Their glory can never fade

When can the glory fade3 Nobody can control the wind

Who can control the wind4 It matters little if I die

What though I die5 No man can serve two masters

Can any man serve two masters

Exercise 3Interchange of assertive and Exclamatory sentences

1 She leads the most unhappy lifeWhat an unhappy life she leads

2 This is indeed an interesting bookWhat an interesting book this

3 He is a very great manWhat a great man he is

4 It is a very lame excuseWhat a lame excuse

5 It is sad that she died so youngAlas she died so young

Class XISubject Topic Summary Execution

Hindi 2nd lang

पतर परम(परमचदर) पतर परम कहानी म एक निपता की इचछाओ का वणन निकया गया ह अपन बड पतर परभ -ास स निपता चतनय -ास का निवशरष परम था निपता को उसक जनम स ही बडी-बडी आशाए थी उसम दसर बट कतिशव-ास की अपकषा स- उतसाह की मातरा अमिधक थी वह उस इगलड भजकर बरिरसटर बनाना चाहत थभागय का खल भी बडा निनराला ह बीए की परीकषा क बा- वह बीमार पड गया डॉकटरो न भी जवाब - दि-या थाचतन -ास जी बहत ही कजस थ बवजह पस खच करना नही चाहत थ अगर गारटी मिमलती तो शाय- पस खच भी कर -त परत गारटी नही थी परिरणाम सवरप उनक बट का -हात हो गयाजब बट को समशान ल जा रह थ तो वहा काफी शोर गान बजान हो रह थ पछन पर पता चला निक निकसी निपता निपछल तीन साल स निबमार था और उसक ईलाज म रपया पानी की तरह बहाया पर ठीक नही हए परत उसक बट को तनिनक भी अफसोस नही था उसका कहना था उसन कोकतिशश तो कीयह -खकर चतनय-ास जी को आतम निगलानी हईतभी स उनका म परिरवतन हआ और बट का भोज काफी धमधाम स निकयाऔर वहइस पशचाताप की आग म जलत रह औला- स बढकर पसा नही होता ह इस बात को समझन म उनह काफी व लग गया

hellipContinue to next

BENGALI(2ND LANGUAGE)

পরথমঅধযায়-ঠাকরারীনদরনাথঠাকর

নয়ন দোচের হিমাচেররা া নাচেমই হিযাত হিছচেন ায়ানার উাররণ সবরপ নয়ন দোচের ারা হিা (াচেকর হিা হিচেতন এছাাও দেকান উৎস উপচেb রাহিতর দেক হিন করার উচেfচে(য তারা সযC হিকরচেরণ রনয পরীপ জবাহিচেয় তাচেত রপার হির 4Cরণ করচেতন ঠাকরা এই নয়ন দো হিমারচের দে(4 ং(ধর হিছচেন হিমাররা ায়ানার ষটানত পর(Cন কচের তারা হিনঃসব এই হিমাহিরর দে(4 ং(ধর গৈকাস নদর রায়চেৌধরী গৈকাস া নয়ন দোচের সমসত সমপহিতত ঋচেরণর াচেয় হিহিx কচের অহি(ষট যা আচেছ তাচেত হিপত

ইার হিপতার মতয ইচে পর নয়নচোচের ায়ানার দেগাটা কতক অসাধাররণ শরাদধ (াহিনতচেত অহিনতম ীহিপত পরকা( কহিরয়া ঠাৎ হিনহিয়া দেগ- ক) কার দো দেকান গচেলপর অং() কতা দেক ইার চেত কাচেক দোঝাচেনা চেয়চেছ গ) পরসঙগ কী কতার কতয পহিরসফট কচেরা

পরচে4র যাহিত রbা করা সমভ নয় তাই হিতহিন পতরচেক হিনচেয় ককাতায় সাস শর কচেরন গলপ কথচেকর আহিথCক অসথা নয়ন দোচের হিমাচের দেথচেক সমপরণC আাা কথচেকর হিপতা হিনচের দেষটায় অথC উপাCন করচেতন া উপাহিধ াচেভর নয তার াসা হিছনা আর দেসই কারচেরণ কথক তার একমাতর উততরাহিধকার চেয় তার হিপতার পরহিত কতজঞ কথক দো পা হি(চেচেছন হিনচের পরারণ ও মান রbার নয উপচেযাগী অথC হিনা দেষটায় পরাপত চেয়চেছন- এটাই তার কাচেছ পরম দেগৌরচের হি4য় চে মচেন কচেরন কাররণ (নয ভাণডাচের গৈপতক ায়ানার উজজব ইহিতাস অচেপbা দোার হিসeচেকর মচেধয গৈপতক দেকামপাহিনর কাগ তার কাচেছ অচেনক দেহি( মযান

TO BE CONTINUED

উ- ক) আচোয অং(টি রীনদরনাথ ঠাকচেরর দো ঠাকরা গচেলপর অং() কতা চেন আচোয গচেলপর গলপ কথকইার চেত নয়ন দোচের হিমাহিরর দে(4 ং(ধর গৈকাস ার কথা া চেয়চেছ গৈকাস া নয়ন দোচের সমসত সমপহিতত ঋচেরণর াচেয় হিহিx কচের অহি(ষট যা আচেছ তাচেত হিপত পরচে4র যাহিত রbা করা সমভ নয় তাই হিতহিন পতরচেক হিনচেয় ককাতায় সাস শর কচেরনগ) গৈকাস ার হিপতার মতযর পর নয়ন দোচের হিমাহিরর অহিসততব হিপত য় কচেয়কটা উৎস ও শরাদধ- (াহিনতচেত হিমাহিরর দে(4 কহিটক যয় চেয় হিগচেয় এচেক াচের দে(4 চেয় যায় তন তাচের গC করার মত আর হিকছই হিছ না-দেসই পরসচেঙগ এই উহিকত নয়নচোচের হিমাচেররা া নাচেমই হিযাত হিছচেন ায়ানার উাররণ সবরপ নয়নচোচের ারা হিা (াচেকর হিা হিচেতন এছাাও দেকান উৎস উপচেb রাহিতরচেক হিন করচেত হিগচেয় তারা সযC হিকরচেরণর নয পরীপ জবাহিচেয় তাচেত রপার হির 4Cরণ করচেতন তাই দেসকাচের ায়ানা দেহি(হিন সথায়ী চেত পারত না হিহিভনন উৎস শরাদধ- (াহিনতচেত সাধযা হিতহিরকত র করার নয হিমাহির হিহিকচেয় দেযত হ হিতC কা হিহি(ষট পরীচেপর দেত দেযমন অলপকাচের মচেধয হিনঃচে(4 চেয় যায়-নয়নচোচের হিমারচের অসথা তাই চেয়হিছ এই কারচেরণই কথক নয়নচোচের হিমারচের গা ভরা আমবর সয করচেত পারতনা

Physics Dimensional Analysis (Summary)

Q Find the dimensions of consts ab in relation

p=(bminusxlowastx)at

where p is the power x is the distance and t is time

Ans From principle of homogeneity dimension of b x2 are same Dim of b = dim of x2 = [L2] = [ML2T0]Dim of a = dim of ( b- x2)dim of (pt) = [M0L2T0][ML2T-2] [T-1] [T] = [M-1L0T2]

Chemistry Atomic Structure Drawbacks of Rutherfordrsquos model of

atom a According to Rutherfordrsquos model of atom electrons which are negativelycharged particles revolve around the nucleus in fixed orbits Thusb theelectrons undergo acceleration According to electromagnetic theory of Maxwell a charged particle undergoing acceleration should emitelectromagnetic radiation Thus an electron in an orbit should emitradiation Thus the orbit should shrink But this does not happenc The model does not give any information about how electrons aredistributed around nucleus and what are energies of these electrons Isotopes These are the atoms of the same

Properties of electromagnetic radiationsa Oscillating electric and magnetic field are produced by oscillating charged particles These fields are perpendicular to each other and both areperpendicular to the direction of propagation of the waveb They do not need a medium to travel That means they can even travel invacuum

Characteristics of electromagnetic radiationsa Wavelength It may be defined as the distance between two neighbouring crests or troughs of

element having the same atomicnumber but different mass numbere g 1H11H21H3

Isobars Isobars are the atoms of different elements having the same massnumber but different atomic numbere g 18Ar40 20Ca40

Isoelectronic species These are those species which have the same numberof electrons

Electromagnetic radiationsThe radiations which are associated withelectrical and magnetic fields are called electromagnetic radiations When anelectrically charged particle moves under acceleration alternating electricaland magnetic fields are produced and transmitted These fields aretransmitted in the form of waves These waves are called electromagneticwaves or electromagnetic radiations

wave as shown It is denoted by λb Frequency (ν) It may be defined as the number of waves which passthrough a particular point in one secondc Velocity (v) It is defined as the distance travelled by a wave in onesecond In vacuum all types of electromagnetic radiations travel with thesame velocity Its value is 3 times10 8m sec-1 It is denoted by v

d Wave number Wave number is defined as the number of wavelengths per unit lengthVelocity = frequency timeswavelength c = νλ

Plancks Quantum Theory- o The radiant energy is emitted or absorbed not continuously but discontinuously in the form of small discrete packets of energy called lsquoquantumrsquo In case of light the quantum of energy is called a lsquophotonrsquoo The energy of each quantum is directly proportional to the frequency of the radiation ie E α υ or E= hυ where h= Planckrsquos constant = 6626 x 10-27 Js o Energy is always emitted or absorbed as integral multiple of this uantum E=nhυ Where n=1234Black body An ideal body which emits and absorbs all frequencies is calleda black body The radiation emitted by such a body is called black body radiation

Photoelectric effectThe phenomenon of ejection of electrons from thesurface of metal when light of suitable frequency strikes it is calledphotoelectric effect The ejected electrons are called photoelectrons

Biology Chapter - 02Systematics and Five Kingdoms

Scientists divide the whole living organisms into two kingdom first and ultimately by five kingdom at last

In the earlier systems of classifications organisms are divided into kingdom plantaeand kingdom animalia on the of presenceof cell wall their modes of nutrition and movements

Some problem arise like fungi share manycharacteristic withplant despite their heterotrophic nutrition bacteria protozoa areunicellular present in both kingdom Toovercome this third kingdom Protista isintroduced which include

unicellularorganisms But there is also another

problem Allunicellular organisms are not similar kind The cellular structure of prokaryotes is verydifferent from that of other organismsEukaryotes possess a true nucleus and allcell organelles that are not present inprokaryotes So the fourth kingdom Monerais introduced which include unicellular prokaryotes (bacteriaamp blue green algae)

bull Still some problem arise in kingdomplantae

So in 1969 R H Whittakar proposedanew five kingdom System of classification

i) Kingdom Monera - unicellular prokaryotes

ii) kingdom Protista - unicellular eukaryotes

iii) Kingdom Fungi - uni or multicellular fungi with cell wall but without chlorophyll

iv) Kingdom Plantae - Multicellular Plants

v) Kingdom Animalia - Multicellular Animals

EVS Chapter 1 ndash Modes of Existence

An agricultural society

An agricultural society also known as an agrarian society is a society that constructs social order around a reliance upon farming More than half the people living in that society make their living by farming

People in an agricultural society generally lead a more settled lifestyle than those in nomadic hunter-gatherer or semi-nomadic pastoral societies because they live permanently near the land that is farmed Agricultural settlements tend to develop in areas of convenience near bodies of water which is used for both crops and transportation or along trade routes Not everyone in an agricultural society is a farmer Some people make a living trading or making and selling goods such as tools used for farming

Another way to define an agrarian society is to see the total amount of production in a nation In an agrarian society cultivating the land is the main source of wealth Such a society can recognize other means of subsistence and work habits but emphasizes the importance of agriculture and livestock Agrarian societies have existed in various parts of the world for 10000 years and continue to exist today They have been the most common form of socio-economic organization for most of recorded human history

Q) Write the features of agricultural society

Ans - Structure and Features of Agrarian Society1 Occupational Structure

An agrarian society is generally associated with the domestication of plants and animals The domestication of plants means farming and that of animals means herding Often there is mixture of farming and the use of such domesticated animals as cow goat and sheep

2 Forms of Land Ownership in Agrarian SocietiesGenerally there are landlords supervisory farmers cultivators and share croppers The landholders own the land but do not work on it They let it out for sharecropping The supervisory farmers are those who live by having their land cultivated by hired labourers The cultivators cultivate the land for themselvesThe share-croppers are those who live by tilling other peoplersquos land or a crop-sharing basis The artisans own their means of production and produce by their own labour in their homesteads

3 Village Community System An agrarian society is highlighted by

the institution of village community system The agrarian economy made fixed dwelling houses necessary Living close together for protection and co-operation and living nearer to the land gave birth to agricultural villages The village is not only the residential place of farmers it is also the social integrator

4 Minimal Division of Labour Another structural feature of agrarian society is a minimal division of labour Except for the basic division founded on age and sex differences there are few specialized roles There is only one predominant type of occupation ie domestication of plants and animals For all the people the environment physical as well as social is the same

5 Role of Family The farm family is of the patriarchal type the father is the final arbiter in most of the familyrsquos major decisions The life of ail men and women is merged in family life Since there are not many special organizations family is the only organisation to perform the tasks of aid and protection

6 Sense of Unity The members of an agrarian society exhibit a strong in-group feeling Since the whole of their social lives is wrapped up in a society which is physically economically and socially homogenous they are inclined to view the entire outside world as an out group

7 Informal Social Control An agrarian society is regionally divided into villages In a village community the force of traditional mores is more dominant than in the urban community In the village everybody is known to everybody The members in a village community help each other and share the joy and sorrows of each other Crime in an agrarian society is rare

8 Simplicity and Uniformity Life of the people in an agrarian society is marked by simplicity and uniformity Their main occupation is agriculture which largely depends upon the vagaries of nature An agrarian society is a religious society

Math Compound angles Compound angles The algebraic sum of two or more angles is called a compound angle If A B C be three angles then A+B B+C C+A A-B B-C A-C A+B-C etc are compound angles In this chapter we shall discuss the trigonometrical ratios of compound angles Theorem 1 If A B and A+B are all pisitive acute angles theni) sin( A+B) = sin A cos B + cosA sinBii) cos(A+B) = cosA cosB- sinA sinBTheorem 2If A and B are positive acute angles and AgtB theni) sin(A-B) = sin A cosB- cos A sinBii) cos(A-B) = cos A cos B+ sin A sin BTo prove that i) sin(A+B) sin (A-B) = sin2 A - sin2 B = cos2 B- cos2 A

Example 1 Prove that tan70deg=2tan50deg+tan20degSolutiontan70deg = tan(50deg + 20deg)Or tan70deg=(tan 50deg+tan 20deg)(1-tan50degtan20deg) or tan70deg (1 ndash tan 50deg tan20deg) = tan50deg+tan20degor tan70deg= tan70deg tan50deg tan20deg+ tan50deg + tan20deg = cot20deg tan50deg tan20deg + tan50deg + tan20deg = 2 tan50deg+ tan20degExample 2 If A + B = 45deg show that (1 + tanA) (1 + tanB) = 2Solutiontan(A + B) =( tan A + tan B) (1 - tan

ii) cos(A+B) Cos(A-B) = cos2 A- sin2 B = cos2 B -sin2 AProof i) LHS= sin(A+B)sin(AminusB) [Recall sin(αminusβ)=sinαcosβminuscosαsinβ And sin(α+β)=sinαcosβ+cosαsinβ]= (sinAcosB+cosAsinB)times(sinAcosBminuscosAsinB)= sin2Acos2Bminuscos2Asin2B [Recall sin2α+cos2α=1 From above we can then assume correctly that sin2α=1minuscos2α AND cos2α=1minussin2α] = sin2A(1minussin2B)minussin2B(1minussin2A) = sin2Aminussin2Asin2Bminussin2B+sin2Asin2B = sin2Aminussin2B= 1-cos2A-(1-cos2B) = cos2 B- cos2 A = RHSii)LHS= cos (A+B) cos(A-B) [ cos(A+B) = cos AcosB- sinAsinBCos(A-B) = cosAcosB+ sinAsinB]= cos2 A Cos2 B- sin2 A Sin2 B= cos2 A( 1-sin2 B) - (1- cos2 A) sin2 B= cos2 A- cos2 A sin2 B- sin2 B+ cos2 A sin2 B=cos2 A- sin2 B=1- sin2 A-(1-cos2 B) = cos2 B- sin2 A= RHSTangent formulae for compound anglesi)tan (A + B) = tan A + tan B1-tan A tan Bii)tan (A ndash B) = tan A-tan B1+tan A tan Biii) cot (A + B) = cot Acot B-1cot A+cot B(viii) cot (A ndash B) = cot Acot B+1cot B-cot A

A tan B) Or 1= (tan A+ tanB) (1-tan A tanB) Or tanA + tanB + tanA tanB + 1 = 1 + 1Or tanA (1 + tanB) + (1 + tanB) = 2Or (1 + tanA) (1 + tanB) = 2Example 3 Find the value of sin 15degSolution sin 15deg= sin(45deg-30deg) = sin45degcos 30deg- cos45degsin30deg =(1radic2) (radic32) -(1radic2) (12) = (radic3-1) 2radic2Example 4 If sin A = 1 radic10 and sin B = 1 radic5 where A and B are positive acute angles then what is A + B SolutionWe know that sin (A + B) = sin A cos B + cos A sin B= [1 radic10] [radic(1 minus 1 5)] + [1 radic5] radic(1 minus 1 10)= [1 radic10] [radic4 5] + [1 radic5] [radic9 10]= [1 radic50] times (2 + 3)= 5 radic50 = 1 radic2

sin (A + B) = sin π 4rArrHence A + B = π 4Example 5 If A + B = 225o then find [cot A] [1 + cotA] times [cot B] [1 + cot B]Solution[cot A] [1 + cotA] times [cot B] [1 + cot B] = 1 [(1 + tan A) times (1 + tan B)]=1 [tan A + tan B + 1 + tan A tan B] [ tan (A + B) = tan225o]∵

tan A + tan B = 1minus tan A tan BrArr= 1 [1 minus tan A tan B + 1 + tan A tan B]= 1 2

COMMERCE

CLASSIFICTION OF HUMAN ACTIVITIES-ECONOMIC AND NON-ECONOMIC

Firstly we shall recall the previous class for 5 mins especially for the absentees and for also the rest of the students who were there

Today at first we briefly discuss the earlier portions of the chapter

1Business-It includes all those economic activities which are concerned with production and exchange of goods and services with the object of earning profit Example A factory shop beauty parlour also business enterprises

2Profession ndashThe term profession means an occupation which involves application of specialized knowledge and skills to earn a living For Example Chartered Accountancy medicine law tax consultancy are example of professions

Questions1What are the main features of ProfessionAnswer The main features of a profession are as follows a Specialised body of knowledge-Every profession has a specialised and systematised body of knowledge b Restricted entry- Entry to a profession is allowed only to those who have completed the prescribed education and have the specialised examination c Formal education and training ndashA formal education and training is given to the person who wants to acquire the professional

3Employment-Employment mean an economic activity where people work for others in exchange for some remuneration (salary)The persons who work for others are called lsquoemployeesrsquo The persons or organizations which engage others to work for them are called lsquoemployersrsquoEg A doctor working in a hospital is employment as he is working for a salaryA lawyer may serve as a law officer in a bank

With this we shall proceed with the features of both Profession amp Employment

The main features of a profession are as follow

a Specialised body of knowledge b Restricted entry c Formal education and training d Professional association e Service motive f Code of contact

The main features of an employment are as follows

a In employment a person works for others called employer

b An employee provides personal service

c There is a service agreement or contract between the employee and the employer

d The employee has to obey the order of the employer

e No capital investment is made by the employer

Various examples of Employment are as follows

aA teacher teaching in a school or collegeb An engineer employed in Municipal Corporation of DelhicAn accountant working in the accounts department of a companydA doctor working in a hospital

Note In all the above examples of employment the individual who is involved in each example is working as an employee for a salary under an employer

qualification(MBBSCALLB)d Service motive ndashProfessionals are expected to emphasis service more on their clients rather than economic gain f Code of Conduct-The activities of professionals are regulated by a code of conduct

2 What are the main features of EmploymentAnswer The main features of an employment are as followsa In employment a person works for others called employerb An employee provides personal servicec There is a service agreement or contract between the employee and the employerd The employee has to obey the order of the employere No capital investment is made by the employer

3 Give various Professions and their respective Association are given below

Professions

Professional

Professional association

Medical profession

Doctor Medical Council of India

Law profession

Lawyers Bar Council of India

Accounting Profession

Chartered

The Institute of Chartered Accounts of India( ICAI)

Engineerin Engineers The

g Profession

institute of Engineers (India)

Accounts Basic accounting terms

Today we will give you some questions from the previous study material

Questions6) Define accounting7) What do you mean by debit

and credit8) Explain the types of account9) Define the following terms

a) Assetsb) Capitalc) Purchased) Debtorse) Transactions

10) Name the types of accounts given below

a) Krishnas accountb) Machinery accountc) Royalty accountd) Salary accounte) Furniture accountf) Audit fee account

Economics Basic Economic ConceptsSub topic

UTILITY

Before starting todayrsquos class we shall recall the last class which was about UTILITY AND THE FEATURES OF UTILITY

Now we shall proceed with the further topics of the chapter

Todayrsquos topic from the chapter lsquo Basic Economic Conceptsrsquo will be TOTAL UTILITY amp MARGINAL UTILITYNow let us quickly revise the concept of utility with an example ie goods and services are designed because they have an ability to satisfy human wantsThis feature of being able to satisfy human wants is termed as utility For example we derive utility from WiFi services as it gives us satisfaction by connecting us to our friends and family through social media here consumers derive utility from WiFi services

From the above concept we shall start with todayrsquos topicEconomists have defined TOTAL UTILITY (TU) as the total satisfaction obtained by consuming a given total amount of a good and serviceFor example the total satisfaction obtained from eating 10 mangoes is the total utility of 10 mangoes

MARGINAL UTILITY (MU) is the additional satisfaction derived from each additional unit

Questions1 What is Total Utility (TU)

Answer Total Utility (TU) is the

aggregate of the utility that a consumer derives from the consumption of a certain amount of a commodityTU=MU1+MU2++MUn

2 What is Marginal UtilityAnswer

Marginal Utility (MU) is the additional made to the total utility as consumption is increased by one more unit of the commodityMU= TUn ndashTUn-1

NoteOften economists tend to

subdivide utility into an imaginary unit called UTIL

consumed In this casethe utility obtained from each mango as it is consumed as the MU of that mango It is also defined as the addition made to the total utility when an additional unit is consumed Often economists tend to subdivide utility into an imaginary unit called UTIL

Note As a consumer increases the consumption of a good over period of time the total utility or total satisfaction derived from it increases to appoint and thereafter it decreasesHowever as the consumer keeps on consuming the good the marginal utility or the additional utility derived from it decreases

SubjectBusiness studies

Topic

BUSINESSENVIRONMENT

Summary

Now quickly let us revise the earlier points that we have already done in the last class and let us proceed with the other topics that are there in the chapter

Firstly we will recall the internal and external factors of micro environment and then we shall proceed in details

Meaning and list of internal and external factors

aInternal factorsInternal factors refer to all the factors existing within a business firm The internal factors are considered controllable because the enterprise has control over these factorsFor an example a company can alter its organization structure policies programmes employees physical facilities and marketing mix to suit the changes in the environmentList of internal factors areCorporate culture mission and objectives top management organizations structure company image and brand equity company resources

b External factorsExternal factors refer to those individual and groups and agencies with which a particular business organization comes into direct and frequent contact in the course of its functioningThese individuals and groups are known as STAKEHOLDERS because they have a stake (financial interest ) in the working and performance of the particular business List of external forces (stakeholders)Customers competitors investors suppliersmiddlemen (marketing intermediaries)

Execution 1 What do you mean by internal

factors in micro environmentAnswerInternal factors refer to all the factors existing within a business firm The internal factors are considered controllable because the enterprise has control over these factorsFor an example a company can alter its organization structure policies programmes employees physical facilities and marketing mix to suit the changes in the environment

2 What do you mean by external factors in micro environment

AnswerExternal factors refer to those individual and groups and agencies with which a particular business organization comes into direct and frequent contact in the course of its functioningThese individuals and groups are known as STAKEHOLDERS because they have a stake (financial interest) in the working and performance of the particular business

3Who are stakeholdersSTAKEHOLDERS are individuals and groups who have a stake (financial interest ) in the working and performance of the particular business 4Discuss the internal factors in briefa Corporate CultureThe values beliefs and attitudes of the founders and top management of the company exercise

financers publics

customers

suppliersfinancers

competitors

middlemen

publics

Fig STAKEHOLDERS OF A COMPANY

Apart from micro environment the other main dimension of business environment isMacro environment Macro environment refers to the general environment or remote environment within which a business firm and forces in its micro environment operateA company does not directly or regularly interact with the micro environmentTherefore macro environment is also known as indirect action EnvironmentThe macro environment forces are less controllable than the micro forces

Macro environment consists of the following components

POLITICAL AND LEGAL ENVIRONMENT

ECONOMIC SOCIAL AND ENVIRONMENT

CULTURAL

ENVIRONMENT

TECHNOLOGICAL ENVIRONMENT

a strong influence on what the cmpaany stands for how it does things and what it considers importantbMission and objectivesThe business philosophy and purpose of a comoany guide it prioritiesbusiness strategiesproduct market scope and development scope

cTop management structurethe composition of board of directors the degree of professionalization of management and the organizational structure of a company have important bearing on its business decisions

dPower structureThe internal power relationship between the board of directors and the chief executive is an important factor

eCompany image and brand equityThe image and brand equity of the company play a significant role in raising finance forming alliance choosing dealers and suppliers launching new products entering foreign markets

5 What is Macro environmentAnswerMacro environment refers to the general environment or remote environment within which a business firm and forces in its micro environment operateA company does not directly or regularly interact with the micro environmentTherefore macro environment is also known as indirect action EnvironmentThe macro environment forces are less controllable than the micro forces 6 What are the components of macro environmenta Political and legal environmentb Economic environmentc Social and cultural environmentd Technological environment

BUSINESS FIRM

Fig COMPONENTS OF MACRO ENVIRONMENTPolitical science

Introduction to political science

Comparative politics and itrsquos scope Comparative politics is the second major dimension of political scienceIt is also a very vast area of study and a very large number of political scientists even treat it as an autonomous area of study within the board ambit of political scienceScope of comparative politics-

1 All political structures -Comparative politics includes the study of all structures formalnon formal governmental and extra governmental which are directly or indirectly involved in politics in all the countries of the world

2 Functional studies- Comparative politics seeks to study politics less from the point of view of the legal institutions in terms of their powers and move from the point of view of their functions which constitute the political process and their actual Operation in the environment

3 Study of political behaviour- Another important part of its scope is the study of the actual behaviour of the people in the process of politics

4 Study of similarities and differences- comparative politics also undertakesan analysis of the similarities and differences among political process and functions

5 Study of all political systems -comparative politics seeks to analyse the actual behaviour and performance of all political systems western as well as non western

6 Study of the environment and infrastructure of politics-The study of politics demands a study of the psychological sociological economic and anthropological environment in fact the social environment as a whole in which each political system operates

7 Study of political culture- political culture is composed of attitudesbeliefs emotions and values of a society that relate to the political system or politics

8 Study of political participation- Political participation is a universal processThe only difference is that while in some states it is limited in others it is wider

9 Study of political process- political

Answer the following questions-

What is comparative politics

What are the scope of comparative politics

Homework- learn

processes like decision makingpolicy making judicial process leadership recruitment process and others are always at work in all political systems

The scope of comparative politics is very comprehensive It includes everything that falls within the area of political activity and political process

History CAMBRIDGE VIEW ABOUT

THE PARTITION

AND REFUTATION

OF CAMBRIDGE

VIEW

Cambridge view about the Partition The Cambridge school of historians have interpreted that opposition to partition scheme was made entirely by the elitist groups They hold the view that Lord Curzon planned to partition the Bengal for administrative purposeREFUTATION OFCAMBRIDGE VIEW The Rationalist historians have rejected the interpretations of the Cambridge School of historians on various grounds

1 QUESTION State different views of historians regarding Partition of Bengal

ANSWER Cambridge historians believed that Lord Curzon partitioned Bengal for administrative reasons only and not for the political motive The Middle class elitist group protested because of their petty interest The Hindu zamindars protested as they have to spend more money for managing their estatesThe lawyers of Calcutta High court feared to lose their clientBut according to the nationalist Historians was-

2- The ultimate object of Lord Curzon was to crush the unity of Bengal politicians

3- If Bengal becomes a separate province Bengali speaking 16 million people of western part would become minority under Hindi speaking people of Bihar and Oriya speaking people of Orissa

4- The bureaucrats expected that the protest movement would die down quickly

5- Lord Curzon used the Muslim community in his political game

6- Idealism had great contribution in the protest against partition

7- The people of the every section of society were affected by the partition of Bengal

Computer Science

Numbers Convertion of dcimal number to octal numberThe decimal numeral system is the standard system for denoting integer and non-integer numbers It is the extension to non-integer numbers of the Hindu-Arabic numeral system For writing numbers the decimal system uses ten decimal digits a decimal mark and for negative numbers a minus sign - The decimal digits are 0 1 2 3 4 5 6 7 8 9 the decimal separator is the dot in many countries

The octal numeral system or oct for short is the base-8 number system and uses the digits 0 to 7 Octal is sometimes used in computing instead of hexadecimal perhaps most often in modern times in conjunction with file

permissions under Unix systems It has the advantage of not requiring any extra symbols as digits It is also used for digital displays

Follow these steps to convert a decimal number into octal form

1 Divide the decimal number by 82 Get the integer quotient for the next iteration (if the number will not divide equally by 8 then round down the

result to the nearest whole number)3 Keep a note of the remainder it should be between 0 and 74 Repeat the steps until the quotient is equal to 05 Write out all the remainders from bottom to top This is the solution

For example if the given decimal number is 8453

Division Quotient Remainder

8453 8 1056 5

1056 8 132 0

132 8 16 4

16 8 2 0

2 8 0 2

Then the octal solution is 20405

Subject Eng Literature (The Tempest ndash William Shakespeare) Topic Act I Scene 1 Lines 33 to 67 (End of scene) Date 16th April 2020 (4th Period)

[Students should read the original play and also the paraphrase given in the school prescribed textbook]Summary Questions amp Answers

[SUMMARY OF THE ENTIRE SCENE]

o The play starts with the scene of a severe storm at sea Alonso (King of Naples) Sebastian (Alonsorsquos brother) Ferdinand (Alonsorsquos son) Gonzalo Antonio (the usurping Duke of Milan) are in a ship in the midst of the storm

o The mariners are trying their best to control the vessel from running aground and are totally following the orders of their Master the Boatswain They have scant success

o The mariners become extremely unhappy and annoyed when most of the passengers arrive on the deck thereby hampering their effort to save the ship There is serious confrontation between them and the passengers who are part of the Kingrsquos entourage

o The mariners could not save the ship

SUMMING-UP

(i) Vivid description of the scene which gives a realistic description of terror and confusion of a tropical storm

(ii) Shows Shakespearersquos accuracy of knowledge in describing the naval operations and also matters of seamanship

(1) GONZALO Ill warrant him for drowning (L 45-57)

though the ship were no stronger than a nutshell and as leaky as an unstanched

wenchBOATSWAIN Lay her a-hold a-hold Set her two courses Off to

sea again lay her offMARINERS All lost To prayers to prayers All lostBOATSWAIN What must our mouths be coldGONZALO The king and prince at prayers Lets assist them

For our case is theirsSEBASTIAN Im out of patienceANTONIO We are merely cheated of our lives by drunkards

This wide-chopped rascal - would thou mightst lie drowning the washing of ten tides

(a) What does Antonio say at the insolent manners of the boatswain just before the given passage

Being irritated at the insolent manners of the boatswain just before the given extract Antonio the Duke of Milan calls him a worthless dog son of a woman without any morals an arrogant and disrespectful noisemaker He says that the boatswain deserved to be hanged(b) What statement does Gonzalo repeat about the boatswain

Gonzalo shows his faith that the boatswain is not destined to die by drowning He is destined to be hanged and nothing can alter this decree of destiny He says that even if the ship was as frail as a nutshell the boatswain could not be drowned for his destiny was to be hanged(c) What do the passengers do when they have lost all hope of their survival

When the passengers have lost all hope of survival they take

(iii) The opening scene justifies the title ndash The Tempest

UNANSWERED QUESTIONS

(i) The King always travels with his entire fleet including his soldiers Where were the other ships

(ii) Why was the ship in that area Where was it coming from or going where

(iii) The ship broke apart What happened to those who were in the ship

(We shall get the answer to the above questions as the play progresses)

leave of life with fervent prayers The mariners take their last hearty drink and are ready for death(d) What blame does Antonio put upon the mariners and the boatswain Antonio rebukes the mariners that these drunkards have brought them to the present crisis by neglecting their duties He blames them saying that they are going to lose their lives entirely for the negligence of the boatswain and his fellows(e) What does Antonio say while cursing the boatswain

Antonio gives vent to his wrath upon the boatswain in particular He calls the boatswain a wide-mouthed rascal who deserves to be hanged on the sea-shore at low water mark so that ten tides might wash over his body and take out of him all the liquor that he has been drinking

Class XIISubject Topic Summary ExecutionHistory Topic

1 1935 ACT AND WORKING OF PROVINCIAL AUTONOMYCONGREE AND OTHER MINISTERSSUB TOPIC GOVERNMENT OF INDIA ACT1935

Government of India Act 1935 This act established a lsquoFederation of Indiarsquo made of British Indian provinces and Indian states and provided for autonomy with a government responsible to the elected legislature in every provinceThis act introduced abolition of Diarchy at provinces The entire provincial administration was introduced to the responsible ministers who were controlled and removed by the provincial legislature The provincial autonomy means two things First The provincial governments were wholly responsible to the provincial legislature Secondly Provinces were free from outside control and interference in the large number of matters The act divided the powers between the centre and provinces in terms of three lists- Federal list( for centre) Provincial list (for province) and concurrent list (for both) Residuary powers were given to the viceroy In the election under the government of India Act the Congress swept the poll the mandate of the people came in favour of the congress so far as general Hindu seats were concerned The Congress did not get a single Muslim seates in Bombay CP UP Sind and BengalIn five provinces Congress had yhe clear majority In BengalNWFPAssam and Bombay Congress emerged as a single largest partyOn the other side the performance of the Muslim League was badThus the Congress formed ministers in 7 provinces out of 11 provinces Coalition ministry was also formed in two other provincesOnly BENGAL AND Punjab had non- congress ministries

1 QUESTION What was the main change introduced by the Government of India ActANSWER a) The Act gave more

autonomy to the provinces b) Diarchy was abolished at the

provincial levelsc) The Governor was the head of

the executived) There was a council of

ministers to advise him The ministers were responsible to the provincial legislatures who controlled them The legislature could also remove the ministers

e) The Governors still retained special reserve powers

2 QUESTION Why did the federal scheme introduced by the Government of India Act 1935 never come into operation

ANSWER The Federal structure of the Government of India was to be composed with the Governor General and Council of ministers The Federal legislature was to be Bicameral legislature- The council of states and the House of Assembly The ministers were to be chosen by the Governor general and they were to hold the office during his pleasure

The provinces of British India would have to join the federation but this was not compulsory for the princely states

This federation never materialised because of the lack of support from the required number of

princely statesThis act was refused and

rejected by the princes the Congress and the Muslim League

Thus both Congress and the League participated in the election of 1937 Thus the federal part was never introduced but the provincial part was put into operations

Bengali 2nd

Language

াচেরর পরাথCনা(কহিতা )

াচেরর পরাথCনা কহিতাটি কহি (ঙখ দেঘাচে4র দো আচো য কহিতায় াচেরর পতর হমায়ন কঠিন দেরাচেগ আxানত ার ঈশবর া আললার কাচেছ পরাথCনা কচেরচেছন তার পচেতরর ীন হিফহিরচেয় হিচেত এই কহিতায় ার পচেতরর ীন হিভbা দেচেয়চেছন ারার এমনহিক হিনচের ীন হিসCচেনর হিহিনমচেয় হিতহিন তার দেছচের ীন হিফচের দেপচেত দেচেয়চেছন তার দেছচের এই দেরাচেগর ন য হিতহিন হিনচেচেকই ায়ী কচেরচেছন তার হিনচের করা পাপচেকই হিতহিন ায়ী কচেরচেছন এছাা রানৈনহিতক ও আথCসামাহিক অসথার কথা তচে ধরা চেয়চেছ এই কহিতায় ার তার হিনচের পাপ কমCচেকই ায়ী কচেরচেছ ার অন যায় ভাচে দেপহি((হিকতর মাধ যচেম অপররা য কচেরচেছ আর এই অন যায় কাচের ন যই তার পহিরাচের হিপযCয় এচেসচেছ দে এক পরকার মানহিক নধন ইহিতাচেসর ার হিপতা চেয় সবাভাহিকভাচে ভাচোাসা দে মমতা দেথচেক মকত চেত পাচেরনহিন তাই হিপতা চেয় আললা া ভগাচেনর কাচেছ পতর হমায়চেনর পরানহিভbা দেচেয়চেছন ার আললা া ভগাচেনর কাচেছ াহিনচেয়চেছন তার হিনচের ীন হিসCন হিচেত হিতহিন রাী তার হিহিনমচেয় পচেতরর ীন হিফচের দেপচেত দেচেয়চেছন াচেরর হিপতসভ হিচেকর কথা এই কহিতায় ফটিচেয় দেতাা চেয়চেছ হিপতা পচেতরর হিরাহিরত মান নধচেনর কথা তচে ধরা চেয়চেছ

হিচে(4 হিকছ াইচেনর তাৎপযC১) ldquoদেকাথায় দেগ ওর সবচছয দেৌন দেকাথায় কচেরায় দেগাপন bয়ldquoউততর) াচেরর পতর হমায়ন কঠিন দেরাচেগ অসসথ তাই তার দেযৌন াহিরচেয় যাচেচছ এই দেরাচেগ তাচেক দেগাপচেন কচেরকচের াচেচছ তার সক (হিকত ধীচের ধীচের bয় চেচছ তাই হিপতা চেয় ার আললার কাচেছ হমায়চেনর পরান হিভbা দেচেয়চেছন২) ldquoাগাও (চেরর পরাচেনত পরানতচের ধসর (ন দেযর আান গানldquoউততর) াচেরর পতর হমায়ন কঠিন দেরাচেগ আxানত তাই ার আ দে(াচেক মমCাত (চেরর পচেথ পরানতচের আান গান ধবহিনত দোক দেসই আান গান আললার কাচেছ দেযন চে যায় আললা দেযন এই আহিতC শচেন পচেতরর ীন হিফহিরচেয় দেয় ৩)ldquoনাহিক এই (রীচেরর পাচেপর ীানচেত দেকানই তরারণ দেনই ভহি4চেতরldquoউততর) হমায়চেনর অসসথতার ন য ার হিনচেচেকই ায়ী কচেরচেছন কারন ার অচেনক রা য অন যায় ভাচে কচেরচেছ তাই তার এই পাপ কাচের ন য তার ঘচের আ হিপ এচেসচেছ এই অন যায় কাচের ন য তার মহিকত দেনই তাই ার আললার কাচেছ এই পাপ কাচেযCর ন য bমা পরাথM

Hindi 2ndlang

-ासी(जयशकर परसा-)

-ासी जयशकर परसा- की एक ऐसी कहानी ह जिजसम भारतीय ससकनित और राषटरीयता का सवरगजीतहोता ह इस कहानी म इरावती एक निहद कनया ह जिजस मलअचछो न मलतान की लट म पकडा और -ासी बना दि-या उस 500 दि-न -कर काशी क एक महाजन न खरी-ा दसरी -ासी निफरोजा ह वह गलाम ह निफरोजा को छडान क कतिलए अहम- को 1000 सोन क कतिसकक भजन थ जो अभी तक नही आए थ राजा साहब कठोर होत हए भी निफरोजा को निबना धनराकतिश क कतिलए उस म कर -त ह वनिफरोजा को अहम- को समझान की बात कहत हकहानी क अत म हम -खत ह निक इरा वती और जाटो क सर-ार बलराज का मिमलन होता हअहम- को यa म मार दि-या जाता ह वहा निफरोजा की परसननता की समामिध बनती ह वहा एक फल चढती ह और डीजल आती ह निफरोजा उस समामिध की आजीवन -ासी बनी रहती हलखक अपन उददशय अथात -ास परथा पर परकाश डालन और इस परथा क कारण होन वाल -ातो क दखो को दि-खान म पणता सफल हए ह

helliphellipContinue to next

Biology Reproductio Today we will discuss about vegetative Q1 Name some vegetative propagules

n in Organisms

propagation of plants The process of multiplication in which fragments of plant body function as propagule and develop into new individual is called vegetative propagation The units of such propagation are runner rhizome tuber bulb etc

and the speciesinvolvedVegetative propagules

Parts involved

Bulb StemBulbil BulbilRhizome Stem Runner Stem Tuber Stem Offset Stem Leaf buds Leaves Suckers Stem

Corns Stem stolon

Q2 State advantages of vegetative propagation

i) Rapid methodii) Sure and easy methodiii) Useful in plants that cannot

produce viable seeds or long seed dormancy

iv) Maintains purity of raceQ 3 Banana fruit is said to be parthenocarpic where as turkey is said to be parthenogenetic WhyBanana develops without fertilization from an unfertilized ovary thus is parthenocarpicIn turkey the ovum or female gamete developinto a new chick without fertilization thus isparthgenetic

Q4 Why is water hyacinth is called as a ldquoTerror of Bengalrdquo Water hyacinth can

propagatevegetatively all over the water body in a short per short period of time This resulted increased biochemicaloxygen oxygen demand of water body causing mortalityof fishes It is very difficult to get rid off them Thus known as terror of Bengal

Chemistry

Solid state GENERAL CHARACTERISTICS OF SOLID STATEIn nature the particular state of matter is governed by two opposing forces at given set of temperature and pressure These forces are intermolecular force of attraction and thermal energy If intermolecular force of attraction is high as compared to thermal energy particles remains in closest position

Intext QuestionsQ1 Classify the following solids as crystalline and amorphous Sodium chloride quartz glass quartz rubber polyvinyl chloride Teflon

A1 Crystalline

and hence very less movement in particles is observed In this case solid state is the preferred state of matter

Let us revise the general characteristics of solid

i) Fixed mass volume and shape

ii) Strong intermolecular force of attraction

iii) Least intermolecular space

iv) Fixed position of constituent particles

v) Incompressible and rigid

Q2 what type of interactions hold the molecules together in a polar molecular solid[CBSE 2010]A2 The molecules in a solid are held together by van der Waals forces The term van der Waals forces include hydrogen bonding dipole-dipole attraction and London dispersion forces All molecules experience London dispersion forces In addition polar molecules can also experience dipole-dipole interactions So the interactions that holds the molecule together in polar molecular solid are London dispersion force and dipole-dipole interactionsQ3 Write a feature that will distinguish a metallic solid from an ionic solid [CBSE 2010]A3 Metals are malleable and ductile whereas ionic solid are hard and brittle Metallic solid has typical metallic lustre But ionic solid looks dullQ4 Write a point of distinction between a metallic solid and an ionic solid other than metallic lustre [CBSE 2012]A4 Metals are malleable and ductile whereas ionic solid are hard and brittleQ5 Write a distinguish feature of metallic solid [CBSE 2010]A5 The force of attraction in

solid Sodium chloride Quartz Amorphous solid Quartz glass rubber polyvinyl chloride Teflon Q2 why glass is considered as super cooled liquidA2 Glass shows the tendency to flow at slower rate like liquid Hence they considered as super cooled liquidQ3 why the window glass of old buildings show milky appearance with timeA3 Glass is an amorphous solid Amorphous solid has the tendency to develop some crystalline character on heating Due to heating in day over the number of years glass acquires some crystalline character and show milky appearanceQ4 why the glass panes fixed to window or doors of old building become slightly thicker at bottomA4 Glass is super cooled liquid It has the tendency to flow down very slowly Due to this glass pane becomes thicker at the bottom over the timeQ5 Sodium chloride is a crystalline solid It shows the same value of refractive index along all the direction TrueFalse Give reasonA5 FalseCrystalline solid shows anisotropy in properties That is it shows different values for the given physical property in different direction All the crystalline solids show anisotropy in refractive index Therefore sodium chloride will show different values of refractive index on different directions

Q6 Crystalline solid are anisotropic in nature What does this statement means

between the constituent particles is special kind of electrostatic attraction That is the attraction of positively charged kernel with sea of delocalized electronsQ6 which group of solid is electrical conductor as well as malleable and ductile [CBSE 2013]A6 Metallic solidQ7 why graphite is good conductor of electricity although it is a network (covalent solid)A7 The exceptional property of graphite is due to its typical structure In graphite each carbon is covalently bonded with 3 atoms in same layer The fourth valence electron of each atom is free to move in between different layersThis free electron makes the graphite a good conductor of electricity

[CBSE 2011]A6 Anisotropy is defined asrdquo Difference in properties when measured along different axis or from different directionsrdquo Crystalline solid show different values of some of the physical properties like electrical resistance refractive index etcwhen measured along the different directions The anisotropy in crystalline solid arises due to the different arrangement of particles in different directions

Math Function Composition of functions Think of an industrial plant that produce bottles of cold drinks first there is the operation (or function) f that puts the cold drink inside the bottle followed by the opeartion g that close the bottle with the capThis leads to the following definitionDefinition Let f A rarr B and g B rarr C be two functions Then the composition of f and g denoted by gof is defined as the function gof A rarr C given by gof(x) = g(f (x)) forall x isinA

Definition A function f X rarr Y is defined to be invertible if there exists a function g Y rarr X such that gof = IX and fog = IY The function g is called the inverse of f and is denoted by f -1

Thus if f is invertible then f must be one-one and onto and conversely if f is one-one and onto then f must be invertible This fact significantly helps for proving a function f to be invertible by showing that f is one-one and onto specially when the actual inverse of f is not to be determined

Example 1 Let f 2 3 4 5 rarr 3 4 5 9 and g 3 4 5 9 rarr 7 11 15 be functions defined as f(2) = 3 f(3) = 4 f(4) = f(5) = 5 and g (3) = g (4) = 7 and g (5) = g (9) = 11 Find gofSolution We have gof(2) = g (f(2)) = g (3) = 7 gof(3) = g (f(3)) = g (4) = 7gof(4) = g (f(4)) = g (5) = 11 and gof(5) = g (5) = 11Example 2 Find gof and fog if f R rarr R and g R rarr R are given by f(x) = cos x and g (x) = 3x2 Show that gof ne fogSolution We have gof(x) = g(f(x))=g(cosx) = 3 (cos x)2

= 3 cos2 x Similarly fog(x)=f(g (x))= f(3x2)= cos (3x2) Note that 3cos2 x ne cos 3x2 for x = 0 Hence gof ne fogExample 3 Show that if f A rarr B and g B rarr C are onto then gof A rarr C is also ontoSolution Given an arbitrary element z isin C there exists a pre-image y of z under g such that g (y) = z since g is onto Further for y isin B there exists an element x in A with f(x) = y since f is onto Therefore gof(x) = g (f(x)) = g (y) = z showing that gof is onto Example 4 Let Y = n2 n isin N sub N Consider f N rarr Y as f(n) = n2 Show that

f is invertible Find the inverse of fSolution An arbitrary element y in Y is of the form n2 for some n isin N This implies that n =radicy This gives a function g Y rarr N defined by g (y) =radicy Nowgof (n) = g (n2)=radicn2 = n and fog (y) =f(radicy) = (radicy) 2 y which shows that gof=IN and fog= IY Hence f is invertible with f -1 = g

Political Science

Constitution of India-The Preamble

Summary

Objective of the state-To secure equality of status and of opportunity To promote fraternity among all the citizens To assure the dignity of the individuals and Unity and integrity of the nation

Justice-Justice stands for rule of law absence of arbitrariness and a system of equal rights freedom and opportunities for all in a society India seeks social economic and political justice to ensure equality to its citizens

Liberty-Liberty implies the absence of restraints or domination on the activities of an individual such as freedom from slavery serfdom imprisonment despotism etc The Preamble provides for the liberty of thought expression belief faith and worship

Equality-Equality means the absence of privileges or discrimination against any section of the society The Preamble provides for equality of status and opportunity to all the people of the country

Fraternity-The Preamble declares that fraternity has to assure two thingsmdashthe dignity of the individual and the unity and

Execution

Answer the following questions-

Short notes-1 Equality2 Fraternity3 Justice4 Liberty

Homework-Learn

integrity of the nation The word integrity has been added to the Preamble by the 42nd Constitutional Amendment (1976)

Business studies

Human resource management (chapter 1)

On the day of 1504 2020 I have discussed with you the managerial functions and procurement functions of HRM

Today weare going to discuss about the development function integration functions and maintenance function

Development functions-HRM improves the knowledge skills attitude and values of employees so that they the present and future jobs more effectively it includes

1) Development functions of HRM

a) Performance appraisal = It implies systematic evaluation of employees with respect to their performance on the job and their potential for development

b) Training =It is the process by which employees learn knowledge skills and attitudes to achieve organisational and personal goals

c) Executive development = It is the process of developing managerial talent through appropriate program

2) Integration functionsa) HRM reconcile the goals of

organisation with those of its members through integrating function

b) HRM tries to motivate employees to various financial and non financial incentives provided in job specification etc

3) Maintenance functiona) HRM promote and protect the

physical and mental health of employees by providing several types of benefits like housing medical aid etc

b) It Promote Social security measures to employees by providing provident fund pension gratuity maternity benefits

SubjectCOMMERCE

Topic

BUSINESSENVIRONMENT

Summary

Now quickly let us revise the earlier points that we have already done in the last class and let us proceed with the other topics that are there in the chapter

Firstly we will recall the internal and external factors of micro environment and then we

Execution 3 What do you mean by internal factors

in micro environmentAnswerInternal factors refer to all the factors existing within a business firm The internal factors are considered controllable because the enterprise has control over these factors

Development FunctionsPerformance AppraisalTrainingExecution Development

shall proceed in details

Meaning and list of internal and external factors

aInternal factorsInternal factors refer to all the factors existing within a business firm The internal factors are considered controllable because the enterprise has control over these factorsFor an example a company can alter its organization structure policies programmes employees physical facilities and marketing mix to suit the changes in the environmentList of internal factors areCorporate culture mission and objectives top management organizations structure company image and brand equity company resources

b External factorsExternal factors refer to those individual and groups and agencies with which a particular business organization comes into direct and frequent contact in the course of its functioningThese individuals and groups are known as STAKEHOLDERS because they have a stake (financial interest ) in the working and performance of the particular business List of external forces (stakeholders)Customers competitors investors suppliersmiddlemen (marketing intermediaries)financers publics

customers

suppliersfinancers

For an example a company can alter its organization structure policies programmes employees physical facilities and marketing mix to suit the changes in the environment

4 What do you mean by external factors in micro environment

AnswerExternal factors refer to those individual and groups and agencies with which a particular business organization comes into direct and frequent contact in the course of its functioningThese individuals and groups are known as STAKEHOLDERS because they have a stake (financial interest) in the working and performance of the particular business

3Who are stakeholdersSTAKEHOLDERS are individuals and groups who have a stake (financial interest ) in the working and performance of the particular business 4Discuss the internal factors in briefa Corporate CultureThe values beliefs and attitudes of the founders and top management of the company exercise a strong influence on what the cmpaany stands for how it does things and what it considers importantbMission and objectivesThe business philosophy and purpose of a comoany guide it prioritiesbusiness strategiesproduct market scope and development scope

cTop management structurethe composition of board of directors the degree of professionalization of management and the organizational structure of a company have important bearing on its business decisions

dPower structureThe internal power relationship between the board of directors and the chief executive is an important factor

e Company image and brand equityThe image and brand equity of the company play a significant role in raising finance forming alliance choosing dealers and suppliers launching new products entering foreign markets

5 What is Macro environmentAnswerMacro environment refers to the general

competitors

middlemen

publics

Fig STAKEHOLDERS OF A COMPANY

Apart from micro environment the other main dimension of business environment isMacro environment Macro environment refers to the general environment or remote environment within which a business firm and forces in its micro environment operateA company does not directly or regularly interact with the micro environmentTherefore macro environment is also known as indirect action EnvironmentThe macro environment forces are less controllable than the micro forces

Macro environment consists of the following components

POLITICAL AND LEGAL ENVIRONMENT

ECONOMIC SOCIAL AND ENVIRONMENT

CULTURAL

ENVIRONMENT

TECHNOLOGICAL ENVIRONMENT

Fig COMPONENTS OF MACRO ENVIRONMENT

environment or remote environment within which a business firm and forces in its micro environment operateA company does not directly or regularly interact with the micro environmentTherefore macro environment is also known as indirect action EnvironmentThe macro environment forces are less controllable than the micro forces 6 What are the components of macro environmenta Political and legal environmentb Economic environmentc Social and cultural environmentd Technological environment

Computer Science

Logic gates

Digital systems are said to be constructed by using logic gates These gates are the AND OR NOT NAND NOR EXOR and EXNOR

BUSINESS FIRM

gates The basic operations are described below with the aid of truth tables

AND gate

The AND gate is an electronic circuit that gives a high output (1) only if all its inputs are high A dot () is used to show the AND operation ie AB Bear in mind that this dot is sometimes omitted ie ABOR gate

The OR gate is an electronic circuit that gives a high output (1) if one or more of its inputs are high A plus (+) is used to show the OR operationNOT gate

The NOT gate is an electronic circuit that produces an inverted version of the input at its output It is also known as an inverter If the input variable is A the inverted output is known as NOT A This is also shown as A or A with a bar over the top as shown at the outputs The diagrams below show two ways that the NAND logic gate can be configured to produce a NOT gate It can also be done using NOR logic gates in the same way

NAND gate

This is a NOT-AND gate which is equal to an AND gate followed by a NOT gate The outputs of all NAND gates are high if any of the inputs are low The symbol is an AND gate with a small circle on the output The small circle represents inversion

NOR gate

This is a NOT-OR gate which is equal to an OR gate followed by a NOT gate The outputs of all NOR gates are low if any of the inputs are highThe symbol is an OR gate with a small circle on the output The small circle represents inversion

EXOR gate

The Exclusive-OR gate is a circuit which will give a high output if either but not both of its two inputs are high An encircled plus sign ( ) is used to show the EOR operation

EXNOR gate

The Exclusive-NOR gate circuit does the opposite to the EOR gate It will give a low output if either but not both of its two inputs are high The symbol is an EXOR gate with a small circle on the output The small circle represents inversion The NAND and NOR gates are called universal functions since with either one the AND and OR functions and NOT can be generated

Note A function in sum of products form can be implemented using NAND gates by replacing all AND and OR gates by NAND gates A function in product of sums form can be implemented using NOR gates by replacing all AND and OR gates by NOR gates

Logic gate symbols

Table 2 is a summary truth table of the inputoutput combinations for the NOT gate together with all possible inputoutput combinations for the other gate functions Also note that a truth table with n inputs has 2n rows You can compare the outputs of different gates

Logic gates representation using the Truth table

Example

A NAND gate can be used as a NOT gate using either of the following wiring configurations

Subject Eng Literature (The Tempest ndash William Shakespeare) Topic Act III Scene 3 Lines 53 to 110 (End of the scene) Date 16th April 2020 (2nd Period)

[Students should read the original play and also the paraphrase given in the school prescribed textbook]Summary Questions amp Answers

o Seeing this strange scene all are inclined to believe the tales told by travelers that there truly are ldquounicornsrdquo and ldquothe phoenixrsquo thronerdquo

o As they are about to sit down to the feast the banquet is snatched away by a harpy (Ariel disguised) A spiritrsquos voice (Arielrsquos voice) denounces Alonso Sebastian and Antonio with particular

1 ARIEL You are three men of sin whom Destiny

(Line 53-58)That hath to instrument this

lower world And what is int the never-surfeited sea

Hath caused to belch up you and on this island

Where man doth not inhabit you rsquomongst men

Being most unfit to live I have made you mad

reference to their crime in expelling Prospero from Milan They have not received any punishment for their deed earlier but the time for their punishment has arrived Upon Alonso it pronounces ldquolingering perdition worse than deathrdquo from which there is no remedy except through sincere repentance Ariel then vanishes in thunder and the shapes enter again and carry away the table

o Prospero watching invisibly is very pleased with the performance of Ariel and his (Prosperorsquos) ldquomeaner ministersrdquo All his enemies are now in his power and are in a fit of desperation He then leaves them and goes to see how Ferdinand and Miranda are getting on

o Alonso is now much humbled and penitent with the after effect of the spiritrsquos denunciation of his crimes He believes that his son is lost forever After this all disperse being stricken mad by the speech of the spirit

o Gonzalo fearing that they may do violence to themselves or to one another follows them and bid others to follow

(a) To whom does Ariel disguised as a harpy call the three sinners What game did Fate of Destiny play with

them

The three sinners called by Ariel are Alonso Sebastian and Antonio It was Destiny which had caused the ocean to cast the three sinners on the shore Though the ocean is all the time devouring whatever appears on its surface and is never satisfied with its continual swallowing of the ships and men in the present case the ocean had cast these three sinners on the shore without killing them

(b) Who had jointly been responsible for the conspiracy against Prospero What is Prosperorsquos purpose behind all this

Three men Alonso Sebastian and Antonio had jointly

been responsible for the conspiracy against Prospero They had driven out Prospero form Milan Prosperorsquos purpose is to make these three sinners realize the wrong they had done He wants them to repent for their criminal deeds because repentance leads to self-esteem(c )What does Ariel (the harpy) tell Alonso and his companions when they take out their swords to attack him

Seeing them drawing their swords Ariel (harpy) tells them that he and his companions are the instruments of destiny and that it is not possible for human beings to do them any injury He says that the swords of human beings can not injure even a minute part of his feathers Their swords are as ineffective against him and his companions as against the wind or the water

(d) Give the explanatory meanings of the following expressions in the context of the above extract

(i)Never surfeited (ii) Belch up (iii) lsquomongst men

(i) Never surfeited never led to satisfaction

(ii) Belch up cast ashore(iii) lsquomongst men in human

society2

I and my fellows (Line 60-65)

Are ministers of Fate The elementsOf whom your swords are tempered may as wellWound the loud winds or with bemocked-at stabsKill the still-closing waters as diminishOne dowl thats in my plume

IMPORTANT PASSAGES EXPLAINED

The elements

(Line 61-66)Of whom your swords are tempered may

as wellWound the loud winds or with

bemocked-at stabs

(a) Who is lsquoIrsquo Who are his lsquofellowsrdquo

lsquoIrsquo is referred to Ariel in disguise of a harpy His lsquofellowsrsquo are other spirits serving Prospero the real Duke of Milan who has acquired supernatural powers after being banished from his Dukedom Prospero has settled in this uninhabited island

(b) What are the elements that have temperrsquod the swords Why will it not work against the speaker

The swords (of Alonso and his companions) are tempered by metal (steel) which is taken out of the earth and refined by

Kill the still-closing waters as diminishOne dowl thats in my plume My fellow

ministersAre like invulnerable

In these words Ariel reminds the King and his companions of the utter futility of drawing swords against himself and his fellows Ariel drives Alonso Antonio and Sebastian the three men of sin to desperation ndash a state in which men do violence to themselves They draw swords to strike Ariel But Ariel reminds them that he and the other spirits are the ministers of destiny and nothing can wound them The steel of which their swords are made of may cut the wind or water which being divided always closes up again Even supposing that such things may be possible it is quite impossible that their swords will cut one feather in their plume They are incapable of being wounded by any sword of man Hence it is foolish on their part to attempt to strike at Ariel and his fellow-spirits

For which foul deed

(Line 72-75)The powers delaying not forgetting

haveIncensed the seas and shores yea all the

creatures Against your peace

Ariel enters like a harpy and remaining invisible tells Alonso Sebastian and Antonio that he and other harpies are the agents of Destiny appointed to carry out her decrees He tells them that their punishment for the crime against Prospero which has been so long deferred is now to fall upon them He reminds them that they had expelled Prospero from Milan and set him and his innocent child adrift on the sea and that the sea had paid them back for their sin by the shipwreck and by the calamities they have suffered He tells them that the powers above which did not forget this mean treachery but only deferred the punishment have now engaged the seas and the shores and all living beings including him and his comrades against them The very elements and supernatural agency Ariel adds have taken up the avenging of their crime against Prospero

the action of fire It may cut the wind or water which being divided always closes up again

The sword will not work against the spirits and the harpy because they are the ministers of destiny and nothing can wound them nor it will cut a single feather in their plume

(c )What is the meaning of lsquodowlrsquo in the last line

The term lsquodowlrsquo means a filament or the smallest part of a feather In this context Ariel in disguise of harpy says that their sword cannot even damage the smallest filament of their (Arielrsquos and other spirits) feathers as they are incapable of being wounded by any sword of man

(d) What does the speaker remind the listeners about

Ariel in disguise of harpy reminds Alonso the King of Naples Sebastian Alonsorsquos brother and Antonio the present Duke of Milan and the treacherous brother of Prospero as they being three men of sin He even reminds them that their punishment for their crime against Prospero which has been so long deferred now falls upon them He reminds them that they have expelled Prospero from Milan and has set him along with his innocent infant daughter adrift on the sea So the sea has paid them back for their sin by their shipwreck and the calamities they have suffered since then The harpy rebukes Alonso of his sin that has incensed the Gods and has deprived him of his son as a punishment

(e) How do they respond

When Ariel in disguise of a harpy reminds Alonso Sebastian and Antonio of their past misdeeds and sin Alonso has a look of terror and confusion in his eyes He utters the words of sincere repentance wrung out of his conscience-stricken heart It appears to him that all the elements of nature the sea-waves the wind and the thunder proclaiming a loud voice in the name of Prospero and the crime Alonso has committed against him They are calling upon him to repent There is a deep storm raging in Alonsorsquos breast and the echoes of that storm are ringing in his ears like a clear note of wind-instrument A note of denunciation of Alonsorsquos crime leaves him much humbled and penitent and confirms his belief that his son is lost forever But Sebastian and Antonio shows some courage instead of repentance They wish to kill the spirits or devils if it appears

3

Of my instruction hast thou nothing bated (Line 85-93)

In what thou hast to say So with good life

And observation strange my meaner ministers

Their several kinds have done My high charms work

And these mine enemies are all knit upIn their distractions They now are in my

powerAnd in these fits I leave them while I visitYoung Ferdinand whom they suppose is

drownedAnd his and mine loved darling

Methought the billows spoke and (Line 96-99)

told me of itThe winds did sing it to me and the

thunderThat deep and dreadful organ-pipe

pronouncedThe name of Prosper It did bass my

trespass

These are the words of contrition coming from Alonso Ariel has driven him to a deep repentance for conspiring with Antonio against Prospero He now feels a sincere remorse It appears to him that all the elements of nature the sea-waves the wind and the thunder proclaimed with a loud voice the name of Prospero and the crime Alonso had committed against him They are calling upon him to repent There is a deep storm raging in Alonsorsquos breast and the echoes of that storm are ringing in his ears like the clear note of a wind-instrument

Comment These are the words of sincere repentance wrung out of the conscience-stricken heart of Alonso Alonso who is the lesser villain is the first to give way to remorse under the effect of Arielrsquos speech The words of Ariel seem to him to be the voice of conscience speaking to him He is driven to desperation a state in which he might do violence to his life

(a) Identify the speaker State the context

Prospero the ruler of the island is the speaker The famous banquet scene has been enacted very well Ariel and his junior spirits have played their roles excellently Prospero is glad to say words of praise for them(b) In what way the speakerrsquos instructions have been carried out

According to Prosperorsquos instructions a banquet was presented before the King of Naples and his companions when they were tired and hungry Just when they were preparing to eat the feast the banquet was suddenly removed by exercising supernatural powers All this was done by Ariel Prosperorsquos chief assistant and a powerful spirit

Ariel not only made the feast disappear but also delivered his speech blaming the King and his two companions for their past wicked deeds He warned them to repent for their misdeeds or suffer forever on that uninhabited island

(c) Who are referred to as lsquomeaner ministersrsquo What have they done

Prospero refers as lsquomeaner ministersrsquo to his other lesser spirits who were assisting Ariel in presenting a scene before the kingrsquos party They entered the scene to the accompaniment of music They assumed several strange shapes and brought in a banquet Then they danced about it with gentle actions of salutations thus inviting the King and others to eat the feast

These spirits play their role again when Ariel in the shape of a harpy quits the scene These shapes enter again and dancing with mocking gestures carry away the table

(d) Who are the speakerrsquos enemies What has happened to them

King of Naples Alonso his brother Sebastian and the present Duke of Milan Antonio (Prosperorsquos own brother) are Prosperorsquos enemies With the turn of events they have all been washed ashore on the island which is ruled by Prospero the great magician Actually this happened after the shipwreck caused by a storm which was raised by Prospero with the purpose of bringing these people to his island Prosperorsquos spirits have already confused and terrified these enemies and they are under Prosperorsquos control He can treat them as he likes

(e) What does he say about Ferdinand Explain what is meant by ldquohellip his and mine darlingrdquo

Prospero knows that Alonsorsquos son prince Ferdinand is alive though his father thinks that the prince has been drowned

Prospero refers to his daughter Miranda who is dear to him She is also very dear to Prince Ferdinand who has fallen in love with her They are waiting to be married soon for which they have received Prosperorsquos consent

4

ALONSO O it is monstrous monstrous (Line 95-102)

Methought the billows spoke and told me of it

The winds did sing it to me and the thunderThat deep and dreadful organ-

pipe pronouncedThe name of Prosper It did bass

my trespassTherefore my son ithrsquo ooze is

bedded andIll seek him deeper than eer

plummet soundedAnd with him there lie mudded

(a) In what way does Alonso express his horror when his conscience is awakened by Arielrsquos words

When Alonsorsquos conscience is awakened by Arielrsquos words he expresses his horror at what he has heard He gets the feeling that the waves of the ocean the wind and the loud thunder have spoken to him and uttered the name of Prospero Because of being reminded of his crime in a very loud and rough voice he comes to realize that he has lost his son for his past misdeeds

(b) What does Alonso imagine about his son What does Alonso want to do in his desperate state

Alonso imagines that his son is lying in the mud at the bottom of the sea He feels desperate that he wants to drown himself in the ocean deeper than the plumb-line has ever gone He wants to lie with his son at the bottom of the sea

(c) How do Sebastian and Antonio want to face the evil spirits

Sebastian says that he is not at all afraid of what the harpy has said and that he is prepared to fight any number of such monsters if they appear before him only one at a time Antonio says that he would support Sebastian in the fight against the fiendsyyy

(d) Why does Gonzalo ask Adrian to follow the three men

Gonzalo tells Adrian that all the three men namely Alonso Sebastian and Antonio are in a wild and reckless mood The thought of the heinous crime of which they are guilty has begun to torment their minds So he asks Adrian to follow those three men without loss of time and prevent them from doing anything which the turmoil in their minds might lead them to do

(e) What opinion do you form of Alonso from the above extract

Alonso who is the lesser villain is the first to give way to remorse under the effect of Arielrsquos speech The words of Ariel seem to him to be the voice of conscience speaking to him He is driven to desperation a state in which he might do violence to his life

Subject =Accounts

Ac-12 15420 topic-pL Appropriation ac

PROFIT AND LOSS APPROPRIATION ACCOUNT

MEANING AND PREPARATIONProfit and Loss Appropriation Account is merely an extension of the Profit and Loss Account of the firm The profit of the firm has to be distributed amongst the partners in their respective profit sharing ratio But before its distribution it needs to be adjusted All Adjustments like partnerrsquos salary partnerrsquos commission interest on capital interest on drawings etc are made in this account These adjustments will reduce the amount of profit for distribution This adjusted profit will be distributed amongst the partners in their profit sharing ratio To prepare it at first the balance of Profit and Loss Account is transferred to this account The journal entries for the preparation of Profit and Loss Appropriation Account are given below

1 for transfer of the balance of Profit and Loss Account to Profit and Loss Appropriation Account

(a) In case of Net Profit

Profit and Loss Ac helliphelliphelliphelliphellipDrTo Profit and Loss Appropriation Ac(Net Profit transferred to Profit and Loss Appropriation Ac)

(b)In case of Net Loss

Profit and Loss Appropriation Achelliphelliphellip DrTo Profit and Loss Ac(Net Loss transferred to Profit and Loss Appropriation Ac)

2 for Interest on Capital

For transferring on Interest on CapitalProfit and Loss Appropriation Achelliphelliphellip DrTo Interest on Capital Ac(Interest on capital transferred to Profit amp Loss Appropriation Ac)

3 for Interest on Drawings

For transferring Interest on Drawings Interest on Drawings Achelliphelliphelliphelliphelliphellip DrTo Profit and Loss Appropriation Ac(Interest on drawing transferred to Profit amp Loss Appropriation Ac)

4 For Partnerrsquos SalaryFor transfer of partnerrsquos SalaryProfit and Loss Appropriation Achelliphellip DrTo Salary Ac(Salary transferred to profit amp Loss Appropriation Ac)

5 For Partnerrsquos CommissionFor transferring commissionProfit and Loss Appropriation Achelliphelliphellip DrTo Commission Ac(Commission transferred to Profit and Loss Appropriation Ac)

6 For Transfer of agreed amount to General ReserveProfit and Loss Appropriation Ac helliphellipDrTo General Reserve Ac(Transfer to General Reserve)

7 for share of Profit or Loss appropriation(a) If ProfitProfit and Loss Appropriation Achelliphellip DrTo Partnerrsquos CapitalCurrent Ac(Profit transferred to capitalcurrent Ac)(b) If LossPartnerrsquos Capital Current Achelliphelliphelliphellip DrTo Profit and Loss Appropriation Ac(Loss transferred to capitalcurrent Ac)

THE FORMAT OF PROFIT AND LOSS APPROPRIATION

Profit and Loss Appropriation Account for the year endedhelliphelliphelliphellip

Particulars Amount Particulars Amount

To PL Ac (loss) By pL Ac (profit)

To Interest on capital BY Interest on drawings

To partner`s commission by Partner`s capital Ac ( loss)

To Partner`s salary To Interest on partner`s loan To General Reserve To Partner`s Capital AC (Profit)

Subject= Economics

MOVEMENT ALONG THE DEMAND CURVE (CHANGE IN QUANTITY DEMANDED)In law of demand you have already studied the inverse relationship between price and quantity demanded When quantity demanded of a commodity changes due to change in its price keeping other factors constant it is called change in quantity demanded It is graphically expressed as a movement along the same demand curve There can be either a downward movement or an upward movement along the same demand curve Upward movement along the same demand curve is called contraction of demand or decrease in quantity demanded and downward movement along the same demand curve is known as expansion of demand or increase in quantity demanded

Extention of demandd

price (rs)p A

B Extentionp1 d

Q Q1

Quantity demanded ( in units)

Contraction of demandd

p2 Ccontraction

p APrice (Rs)

d

Q2 Q

Quantity demanded (in units)

Explanation of movement of demand A fall in price from OP to OP1 leads to increase in quantity demanded from OQ to OQ1 (expansion of demand) resulting in a downward movement from point A to point B along the same demand curve DD When Price rises from OP to OP2 quantity demanded falls from OQ to OQ2 (contraction of demand) leading to an upward movement from point A to point C along the same demand curve DD

  • Activity Series of Metals
    • Drawbacks of Rutherfordrsquos model of atom
      • Electromagnetic radiations
      • Properties of electromagnetic radiations
      • Characteristics of electromagnetic radiations
        • Plancks Quantum Theory-
        • Photoelectric effect
          • Intext Questions
            • Logic gates
            • Digital systems are said to be constructed by using logic gates These gates are the AND OR NOT NAND NOR EXOR and EXNOR gates The basic operations are described below with the aid of truth tables
            • AND gate
            • Example
Page 12:  · Web viewSubject. Topic. Summary. Execution. English 1 . Chapter 1 naming words . Page 8. Write the names of these pictures:- Person:-1. father. 2.Firefighter 3.doctor 4 ...

f) Why was the poet ldquoperplexed and stillrdquoAns- He was lsquo perplexed and stillrsquo because he was expecting an answer from the inmates of the house But despite of repeated calls there was no response

CHEMISTRY Chapter 2 ndashElement and Compound

Activity Series of MetalsThe activity series is a chart of metals listed in order of declining relative reactivity The top metals are more reactive than the metals on the bottomMetal SymbolReactivity

Lithium Li displaces H2 gas from water steam and acids and forms hydroxides

Potassium K

Strontium Sr

Calcium Ca

Sodium Na

Magnesium Mg displaces H2 gas from steam and acids and forms hydroxides

Aluminum Al

Zinc Zn

Chromium Cr

Iron Fe displaces H2 gas from acids only and forms hydroxides

Cadmium Cd

Cobalt Co

Nickel Ni

Tin Sn

Lead Pb

Hydrogen gas

H2 included for comparison

Antimony Sb combines with O2 to form oxides and cannot displace H2

Arsenic As

Bismuth Bi

Copper Cu

Mercury Hg found free in nature oxides decompose with heating

Silver Ag

Palladium Pd

Platinum Pt

Gold Au

Answer the following Q)Difference Between Metals And Nonmetals

Metals Nonmetals

These are solids at room temperature except mercury

These exist in all three states

These are very hard except sodium

These are soft except diamond

These are malleable and ductile

These are brittle and can be breakdown into pieces

These are shiny These are non-lustrous except iodine

Electropositive in nature Electronegative in nature

Have high densities Have low densities

Math Number System

Chapter Fraction

Study item Some solved sums from exercise 3(B)1) For each pair given below state whether it from like fractions or unlike

fractions (i) 58 and 78

= Like Fraction because denominators same(ii) 815 and 821

= Unlike Fraction because denominators are not same

(iii) 49 and 94 = Unlike Fraction

2) Convert given fractions into fractions with equal denominators(iii) 45 1720 2340 and 1116Solution Given fraction 45 1720 2340 and 1116Therefore the LCM of 5 20 40 and 16 is 80Therefore 45 = 4times165times16 = 64801720 = 17times420times4 = 68802340 = 23times240times2 = 4680 1116 = 11times516times5 = 5580

3) Convert given fractions into fractions with equal numerators(iii) 1519 2528 911 and 4547Solution Given fractions 1519 2528 911 and 4547Therefore the LCM of 15 25 9 and 45 is 2251519 = 15times1519times15 = 225285 2528 = 25times928times9 = 225252911 = 9times2511times25 = 2252754547 = 45times547times5 = 225235

4) Put the given fractions in ascending order by making denominators equal

(iii) 57 38 914 and 2021Solution Given fraction 57 38 914 and 2021Therefore the LCM of the denominators is 16857 = 5times247times24 = 12016838 = 3times218times21 = 63168914 = 9times1214times12 = 1081682021 = 20times821times8 = 160168Therefore ascending order 63168lt108168lt120168lt160168Therefore ascending order of given fractions38lt914lt57lt2021

COMPUTER CHAPTER-1COMPUTER FUNDAMENTALS

DONE IN THE PREVIOUS CLASSES PAGE 10CWRITE TRUE AND FALSE

1 True2 False3 False4 False5 True

GEOGRAPHY CHAPTER 7EUROPE

CHAPTER COMPLETE 1)Europe is home to a famous mountain range called the Alps

2)River Rhine originates in Switzerland

3)The Eiffel Tower one of the tallest structures in Europe

4) Vatican City is one of the most densely populated European countries

5)Sognefjordin Norway is the largest fjord in Norway

Class VIIISubject Topic Summary Execution

MATHEMATICS Ch 6Sets

Exercise 6 (D)1 Given A = x x isin N and 3iquest x le 6 and B = x x isin W and xlt4 find (i) Sets A and B in roster form (ii) A cup B (iii)

A cap B(iv) A ndash B (v) B ndashA

Solution (i) A = 456 and B = 0123

(ii) A cup B = 0123456 (iii) A cap B = ϕ (iv) A ndash B = 456 (v)B ndash A = 0123

3 If A = 56789 B = x 3 lt x lt 8 and x isin W and C = x xle5 and x isin N Find (i) A cup B and (A cup B) cup C (ii) B

cup C and A cup ( B cup C)

(iii) A cap B and (A cap B) cap C (iv) B cap C and A cap (B cap C)

Is (A cup B) cup C = A cup (B cup C)

Is (A cap B) cap C = A cap (B cap C)

SolutionA = 56789 B = 4567 C = 12345

there4 (i) A cap B = 456789 and (A cup B) cup C = 123456789

(ii) B cup C = 1234567 and A cup ( B cup C) = 123456789

(iii) A cap B = 567 and (A cap B) cap C = 5

(iv) B cap C = 45 and A cap (B cap C) = 5

Now (A cup B) cup C = 123456789

And A cup ( B cup C) = 123456789 there4 (A cup B) cup C = A cup (B cup C)

Again (A cap B) cap C = 5 and A cap (B cap C) = 5

there4 (A cap B) cap C = A cap (B cap C)

4 Given A = 012345 B = 02468 and C = 0369 Show that (i) A cup (B cup C) = (A cup B) cup C ie the union

of sets is associative (ii) A cap (B cap C) = (A cap B) cap C ie the intersection of sets is associative

SolutionNow B cup C = 0234689 and A cup B = 01234568

there4 A cup (B cup C) = 012345689 and

(A cup B) cup C = 012345689

So (i) A cup (B cup C) = (A cup B) cup C ie the union of sets is associative

Again B cap C = 06 and A cap B = 024

there4 A cap (B cap C) = 0 and (A cap B) cap C = 0

So (ii) A cap (B cap C) = (A cap B) cap C ie the intersection of sets is associative

Physics Chapter 2 Physical Quatites and Measurements

Here We Will Do Some QuestionsRelated To Chapter 2

A density bottle has a marking 25 mL on it It means that

1 the mass of density bottle is 25g

2 the density bottle will store 25 ml of any liquid in it

3 the density bottle will store 25 ml of water but more volume of liquid denser than water

4 the density bottle will store 25 ml of water but more volume of a liquid lighter than water

Solution 2 the density bottle will store 25 ml of any liquid in it

COMPUTER CHAPTER-2Spreadsheet Functions and Charts

SELECTING RANGE IN ROWSCOLUMNSWHEN TWO OR MORE CELLS ARE SELECTED IT IS CALLED A RANGEA RANGE OF CELLS CAN BE FORMED IN TWO WAYS--a) SELECTING RANGE BY USING THE MOUSEb) SELECTING RANGE BY USING THE KEYBOARD

Q1)WRITE THE STEPS TO SELECT PARTIAL RANGE IN A ROW

Ans)THE STEPS ARE-6 SELECT THE ROW7 BRING THE CELL POINTER TO THE DESIRED

LOCATION FROM WHERE YOU WANT TO START YOUR SELECTION

8 CLICK THE LEFT MOUSE BUTTON AND KEEP DRAGGING TO YOUR RIGHT TILL YOU REACH THE LAST CELL TO NE SELECTED

RELEASE THE MOUSE BUTTON

GEOGRAPHY Asia

CLIMATE

Asia experiences great extremes of climate Jacobabad in the Sind province of Pakistan is one of the hottest places in the WorldVerkhoyansk in Siberia is one of the coldest places in the WorldCherrapunji and Mawsynram in India are two wettest places in WorldArabia Tibet Gobi and Mongolia are extremely dry regionsFactors Affecting Climate of Asia-The factors influencing the climate of Asia are-

Factors Affecting Climate of Asia-Thoroughly read the table in page number 60

Latitudinal extent

Continentality

Relief features

Presence of low pressure trough

Jet streams

English Language The Sentence A complex sentence contains one independent clause and at least one dependent clause The dependent clause in a complex sentence is introduced with subordinating conjunctions or relative pronouns

Commonly Used Subordinating Conjunctions-Time after before while when since untilCause And Effect because now since as in order that soOpposition although though even though whereas while in spite ofCondition if unless only if whether or not even if in case(that)

Commonly Used Relative Pronouns-Who whose whom which whoever whomever whichever that

Class IXSubject Topic Summary Execution

1-BENGALI(2ND LANGUAGE)

ldquo বঙগভমিরপরমিrdquo াইকেলধসদনদতত

আচেগর পর উততর পচো-১ ২ ৩ এং নীচের পর টি াহির কা- ৪মহিbকাও গচেনা দেগা পহিচে অমত হরচে- ক) কার দো দেকান কহিতার অং( ) কতা দেক পরসঙগ কী উহিকতটির তাৎপযC আচোনা কচেরা৫দেসই ধনয নরকচে দোচেক যাচে নাহি ভচে মচেনর মহিeচের সাচেসচে সCন ক) কহির কায C ার উচেf(য হিক হিছ কহি কন কহিতাটি দেচেন) কহি কার কাচেছ হিমনহিত কচেরচেছনগ) কহি এই পহিথীচেত কাচের ধনয মচেন কচেরনঘ) কহি হিক রকম অমর তাাভ করচেত ান

Hindi 2nd lang

काकी(कतिसयारामशरण गपत)

इस कहानी म लखक न यह बतान का परयास निकया ह निक बचच अपनी मा स निकतना परम करत ह शयाम अबोध बालक ह वह अपनी मा क मरन क बा- उसन अपनी मा क कतिलए बहत रोया बा- म उस पता चला निक उसकी मा राम क घर चली गई ह आकाश म उडती हई पतग -खकर उस हरष हआ निक पतग क दवारा वह अपनी मा को नीच उतारगा इसक कतिलए वह अपनी निपता की जब स -ो बार सवा रपया निनकालकर पतग और -ो मोटी सी मन वाली अपन भाई स काकी एक कागज पर कतिलखवा कर पतग म कतिशव का दि-यानिनकालकर पतग और -ो मोटी सी मन वाली अपन भाई स काकी एक कागज पर कतिलखवा कर पतग म कतिचपका दि-याभोला और शयाम कोठरी म रससी बाधनी रह थ तभी उसक निपता करोध म आकर उन स पछ निक कया उनकी जब स रपया निनकाला हभोला डर क मार बताया निक शयाम इस पतग क दवारा अपनी काकी को राम क यहा स उतारना चाहता हनिवशशवर(शयाम क निपता)न फटी पतग उठाकर -खी तो उस पर काकी कतिलखा थावह हत बजिa होकर वही खड रह गएउनहोन सोचा निक मन अपन पतर को मारा जोनिक अनजान और निन-dरष थावह अपनी मा कोनिकतना पयार करता ह

उस दि-न बड सवर शयाम की नी- खली तो -खा निक घर भर म कोहराम मचा हआ ह

क) घर म कोहराम कयो मचा हआ था शयाम को कया लगा

ख) काकी को ल जात समय शयाम न कया उपदरव मचाया

ग) काकी क बार म उस कया बताया गया कया सतय उस कतिछपा रहा

घ) वह बठा-बठा शनय मन स आकाश की ओर कयोकरता

उततरक) शयाम की मा का -हात हो गया था इसकतिलए

घर म कोहराम मचा हआ था शयाम की लगा निक उसकी मा सफ- कपडा ओढ हए भमिम पर सो रही ह

ख) लोग जब उमा यानी शयाम की मा को उठाकर ल जान लग तब शयाम न बडा उपदरव मचाया लोगो क हाथ स झठ करवा उमा क ऊपर जा निगरा और बोला काकी सो रही ह उस कहा ल जा रह हो

ग) काकी क बार म बजिaमान लोगो न उस निवशवास दि-लाया निक उसकी का निक उसक मामा क यहा गई ह लनिकन सतय अमिधक दि-नो तक कतिछपाना रह सका आसपास क अबोध बालको क मह स यह बात परकट हो गई निक उसकी मा का -हात हो गया ह

घ) कई दि-नकई दि-न लगातार रोत-रोत उसका रोना तो शान हो गया पर उसक ह-य म शोक भर गया था वह चपचाप बठा आकाश की और टाका करता निक शाय- उसकी काकी कही दि-ख जाए

ldquoदि-न उसन ऊपर आसमान म पतग उडती -खी न जान कया सोच कर उसका निहर-य एक-म खिखल उठाrdquo

क) निकसन पतग ऊपर उडत -खी और वह कयो खश हआ

ख) उसन अपन निपता स कया कहा उनका कया उतर थाश

ग) उसन निफर कया निकया और निकसन उसकी सहायता की

घ) उसकी योजना कया थी उततर -क) शयाम न एक दि-न आसमान म पतग उडती

-खी तो उसन सोचा निक पता आसमान म राम क यहा जाकर रकगी वही पर मरी काकी ह यह सोचकर वह बहत खश हआ

ख) उसन अपन निपता स कहा काका मझ एक पतग मगा -ो उसक निपता न भटक हए मन क भाव स कहा निक मगा -ग यह कह कर उ-ास भाव स वह कही और चल गए पतग नही आई

ग) उसन चपचाप निवशशवर क टगहए कोट स एक चवननी निनकाल ली और सखिखया -ासी क लडक भोला की सहायता स एक पतग मगवानी भोला उसकी बराबर उमर का ही था

घ) उसकी योजना यह थी निक वह अपनी पतग को आकाश म राम क यहा भजगा और उस पतग क सहार उसकी काफी नीच उतर जाएगी इस योजना पर उस परा निवशवास था इसकतिलए वह और भोला -ोनो यह काम करन म लग गए

Continue to nexthelliphellipEVS CHAPTER - 1

(UNDERSTANDING OUR ENVIRONMENT)

Sustainable development

The development that meets the needs of the present without compromising the ability of future generations to meet their own needs is called Sustainable development

Sustainable societies ndash

An environmentally sustainable community is one that meets the current and future basic resource needs of its people in a just and equitable manner without compromising the ability of future generations to meet their basic needs

Q ) What are Eco Villages

Ans - Eco village are the urban or rural communities of people who strive to integrate a supportive social environment with a low impact way of life

Q ) To ensure sustainable development the depletion of renewable resources should not take place at a rate faster than their regeneration Justify your answer

Ans ndash Renewable resources do not have a fixed quantity - more can always be

generated However if the rate of use exceeds the rate of renewal - that is the

source is used more than its being recreated - its continued use will become

used up faster than it can regenerate

To promote sustainable society the following things need to be done ndash

1 Using renewable energy sources 2 By improving the quality of human

health 3 By promoting sustainable agriculture 4 By forming ecovillage

it will eventually be entirely depleted So Toensure sustainable development the depletion of

renewable resources should nottake place at a rate faster than their regeneration

Q ) What do you mean by Sustainable societies

Ans - Sustainable societies are defined as towns and cities that have taken steps to remain healthy over the long term These communities value healthy ecosystems use resources efficiently and actively seek to retain and enhance a locally based economy Sustainable development concerns everybody in a society

Q ) What are the effects of pollution on human health

Ans ndash Some health problem occurs due to air pollution are ndash

Respiratory diseases Cardiovascular damage Fatigue headaches and anxiety Irritation of the eyes nose and throat Damage to reproductive organs Harm to the liver spleen and blood Nervous system damage

Some health problem occurs due to water pollution are ndash

Typhoid Cholera Dysentry Jaundice

Some health problem occurs due to noise pollution are ndash

Fatigue headaches and anxiety High blood pressure Hearing damage

Physics Motion in 1D First go through previous notes Now here we will solve some numerical related to that

Question 3What information about the motion of a body is obtained from the displacement-time graphSolution 3From displacement-time graph the nature of motion (or state of rest) can be understood The slope of this graph gives the value of velocity of the body at any instant of time using which the velocity-time graph can also be drawn

Question 4(a)What does the slope of a displacement-time graph represent(b)Can displacement-time sketch be parallel to the displacement axis Give a reason to your answerSolution 4(a) Slope of a displacement-time graph represents velocity(b) The displacement-time graph can never be parallel to the displacement axis because such a line would mean that the distance covered by the body in a certain direction increases without any

increase in time which is not possible

Chemistry Language of Chemistry

How to balance a chemical equationThere are two methods of balancing an equation(i)Hit and trial method(ii)Partial equation methodBalancing by hit and trial methodThis method consists of counting the number of atoms of each elements on both sides and trying to equalize themTake the following steps(i)Count the number of times (frequency) an element occurs on either side(ii)The element with the least frequency of occurrence is balanced first(iii)When two or more elements have the same frequencythe metallic element is balanced firstExample-1 On heatinglead nitrate decomposes to give lead dioxidenitrogen dioxide and oxygenPb(NO3)2rarrPbO+NO2+O2

In this equationLead occurs twiceNitrogen occurs twiceOxygen occurs four timesSince lead is a metalbalance it firstThe number of atom of lead is equal on the two sidestherefore it needs no balancingNow balance nitrogenOn the reactant sidethere are two atoms of nitrogenwhile on the product side oneSomultiply the product containing nitrogenon the product sideby two Pb (NO3)2rarrPbO+2NO2+O2Nowthe number of oxygen atoms on the reactant side 6while on the product sideit is 7Somultiply the entire equation by 2except oxygen to get balanced equation2Pb(NO3)2rarr2PbO+4NO2+O2Multiplication by 2 is done only when atoms of all the elements except one element are balanced and the unbalanced atom occurs separately at least once and also there is a difference of only one such atom

Math Topic AlgebraChapter

Factorisation

Study item Difference of two squares a2 ndash b2 = (a+b) (a-b)1) (i) 4x2ndash 25y2

= (2x) 2 ndash (5y) 2= (2x + 5y) (2x - 5y)

(ii) 9x2 ndash 1= (3x)2ndash(1)2= (3x + 1)(3x ndash 1)

2) (i) 150 ndash 6a2= 6(25 ndash a2)= 6(5)2 ndash(a)2= 6 (5 + a) (5 ndash a)

(ii) 32x2 ndash 18y2=2(16x2 ndash 9y2)=2(4x)2 ndash (3y)2= 2(4x + 3y)(4x - 3y)3)(i) (x ndashy )2 ndash 9 = (x ndash y )2 ndash (3)2= (x ndash y + 3) (x ndash y ndash 3)(ii) 9(x + y) 2ndash x 2= (3)2(x + y)2 ndash (x)2=3(x + y)2 ndash (x)2= (3x +3y ) 2ndash(x)2= (3x + 3y + x)(3x +3y ndash x)= (4x + 3y) ( 2x + 3y )

Commercial studies

Basic accounting terms

Today I will give you some questions from the previous study material

Questions1) Define accounting2) What do you mean by debit and

credit

3) Explain the types of account4) Define the following terms

a) Assetsb) Capitalc) Purchased) Debtorse) Transactions

5) Name the types of accounts given below

a) Krishnas accountb) Machinery accountc) Royalty accountd) Salary accounte) Furniture accountf) Audit fee account

Economics Revision Today I will give you some revision questions

Questions1) What do you mean by the terms

rdquowantsrdquo2) Write the difference between

consumer goods and producer goods

3) Define the term utility 4) Explain the different types of utility5) Define

a) Total utilityb) Marginal utility

Subject Eng Literature (The Merchant of Venice ndash William Shakespeare)Topic Act I Scene 3 Lines 1 to 48 (Shylock hellip Cursed be my tribe if I forgive him) Date 16th April 2020 (5th Period)

[Students should read the original play and also the paraphrase given in the school prescribed textbook]Summary Questions amp Answers

This scene takes place in Venice and we are introduced to the rich Jew Shylock Bassanio and Shylock are talking and Bassanio tells Shylock that he wants a loan of three thousand ducats for three months on the personal security of Antonio

o Shylock feels glad because he will be able to bind down Antonio by means of a bond on account of the loan but he tells Bassanio that all the fortunes of Antonio being invested in the merchant ships on the sea it is difficult to depend upon his credit Even under such circumstances Shylock is willing to advance the money on the personal security of Antonio

o Bassanio then invites Shylock to dine with him Shylock says that he is prepared to do anything with the Christians but not eat or drink or pray with them

o While Bassanio and Shylock are talking Antonio appears on the scene Shylock does not seem to take any notice of Antonio but goes on brooding within

(1) SHYLOCK Ho no no no no- my meaning in (Line 15-26)saying he is a good man is to have you understand me that he is sufficient Yet his means are in suppositionhe hath an argosy bound to Tripolis another to the Indies I understand moreover upon the Rialto he hath a third at Mexico a fourth for England and other ventures he hath squanderd abroad Butships are but boards sailors but men there be land-rats and water-rats land-thieves and water-thieves I mean pirates and then there is the peril of waters winds and rocks The man is notwithstanding sufficientmdashthree thousand ducats mdashI think I may take his bond

(a) Who is talking in the beginning of this scene What does Bassanio want from Shylock How does Shylock feel

In the beginning of the scene Bassanio and Shylock are talking to each other Bassanio wants to borrow three thousand ducats from Shylock for three months on the security of Antonio Shylock feels glad at heart that he will get the opportunity of binding Antonio with a bond(b) What risks does Shylock weigh in advancing the money

Shylock says that Antonio has invested all his capital in trading by sea-going ships But the ships are made of wood and the sailors of those ships are ordinary human beings The wood can

himself how he hates Antonio because of his being a Christian because he abuses Shylock in public places Shylock decides that if ever he can get Antonio to his advantage he will teach him a lesson

come to harm and men can commit mistakes and thus the capital invested in ships may be lost Then there are other dangers The goods loaded on the ships can be damaged by rats and thieves which are found both on land and water The ships can also be harmed through sea-storms submerged rocks etc(c) What two important functions does this scene have

The scene has two important functions First it completes the exposition of the two major plot lines of the play Antonio agrees to Shylockrsquos bond ndash three thousand ducats for a pound of flesh and second and more important dramatically this scene introduces Shylock himself In this scene Shakespeare makes it clear at once why Shylock is the most powerful dramatic figure in the play and why so many great actors have regarded this part as one of the most rewarding roles in all Shakespearean dramas(d) Where does this scene take place What kind of treatment has Antonio been giving to Shylock What does Shylock say when Bassanio invites him to dine with him

The action of this scene takes place in Venice Antonio has been in the habit of behaving harshly with Shylock ndash spitting on his beard and footing him like a stranger cur When Bassanio invites Shylock to dine Shylock says that he is prepared to do anything with the Christians but not eat and drink or pray with them

(2) SHYLOCK How like a fawning publican he looks (Line 38-48)I hate him for he is a Christian

But more for that in low simplicity

He lends out money gratis and brings downThe rate of usance here with us in VeniceIf I can catch him once upon the hipI will feed fat the ancient grudge I bear him

He hates our sacred nation and he railsEven there where merchants most do congregateOn me my bargains and my well-won thriftWhich he calls interest Cursed be my tribeIf I forgive him

(a) What is the context in which these words are spoken and what is the idea expressed in it

These remarks are made by Shylock when he sees Antonio coming along after Bassanio told him that the merchant will be his surety for the bond The above mentioned passage reveals Shylockrsquos hatred for Antonio Shylock says that he hates Antonio because he is a Christian and also because he gives loan without taking interest on them thereby bringing down the rate of interest in Venice(b) Explain the meaning of the phrase lsquoa fawning publicanrsquo

The phrase lsquoa fawning publicanrsquo refers to Roman tax collector It is a term of contempt and hatred on the lips of a Jew lsquoFawning Publicansrsquo were Roman tax-gatherers whose ordinary bearings towards the Jews was bullying but whose false pose of lsquohumility and contritionrsquo is touched upon in the parable in New Testament(c ) What light does the above passage throw on the character

of Shylock

The above mentioned speech of Shylock reveals him to be a wicked character having an extreme greed for wealth His intense hatred for Antonio is unjustified He hates Antonio only because he is a Christian and because he lends money without taking any interest on it thereby adversely affecting Shylockrsquos business of lending money on high interest(d) What information do you gather about Antonio from the above given lines

Shylockrsquos statement throws a valuable light on the character of Antonio Antonio appears to be a good Christian and a good human being He helps the people in need by lending them money without charging any interest on it He is a man of simple and good nature This very goodness makes him Shylockrsquos enemy(e) What does Shylock debate within himself and when To whom are the lines mentioned above addressed to

When Bassanio asks the Jew to lend him three thousand ducats on Antoniorsquos surety Shylock begins to debate within himself as to how he should exploit the opportunity of a business deal with his old enemy Antonio

The lines mentioned above are not addressed to anyone The lines are a soliloquy ie a speech made by a character to himself and not meant to be heard by the other characters present

Class XSubject Topic Summary ExecutionEnglish

LiteratureThe Blue Bead 2nd part

Things took a turn for the worst and all of a sudden a crocodile attacked the woman biting on the womanrsquos leg At that moment Sibia got up sprinted grabbed the hay fork and stabbed the crocodile in the eye with all her power Immediately the crocodile let go and went away Sibia saw a small blue bead lying by the river she grabbed it Since she was poor she didnrsquot have necklace Shersquod always wanted one like the other women now she could make one with the blue bead After that she went home and told her mother all about it

Hindi 2nd

Langबड घर की बटी( मशी परमच-)

lsquoबड घर की बटी कहानी का उददशय मधयम वग की घरल समसया को सलझा कर सगदिठत परिरवार म मिमल जलकर परम स रहन का स-श -ना हघर म शानित सथानिपत करन की जिजमम-ारी नारी की होती ह यदि- नारी समझ-ार ह उसम धय और परिरवार क परनित परम ह तो कोई भी घटना परिरवार को निवघदिटत नही कर सकती या कहानी परिरवार को सगदिठत करत हए परम सौहा- स एक दसर की भावनाओ को समझ करउनका सहयोग करत हए जीवन यापन करन की पररणा -ती हमशी परमचदर जी न इस कहानी म सय परिरवार का परनितनिनमिधतव निकया ह यह कहानी बनी माधव सिसह जो गौरी पर क जमी-ार क उनक -ो पतरो की हशरी कठ लाल निबहारीशरीकात का निववाह एकजमी-ार घरान की पतरी आन-ी स हआ थाआन-ी न ख- को ससराल क वातावरण म ढाकतिलया थाएक दि-न आन-ी का अपन -वर लाल निबहारी स झगडा हो जाता ह -ोनो भाई एक दसर स अलग होन की कोकतिशश करत हसभी बह आन-ी न अपन मधर वयवहार स लाल निबहारी को

ldquoइन नतर निपरय गणो को बीए-इनही -ो अकषर पर नयोछावर कर दि-या था इन -ो अकषर न उनक शरीर को निनबल और चहर को कानित ही बना दि-या थाldquo

क) परसतत पकतियो म निकस वयकति क बार म कहा गया ह

ख) इन पकतियो म कौन स नतर निपरय गणो क बार म कहा गया ह

ग) बीए की निडगरी परापत कर लन पर भी उपय वयकति क सवभाव की कया निवशरषता थी

घ) यह नतर निपरय गण निकस वयकति म निवदयमान थ उसक वयकतितव की कया निवशरषता थी

उततर ndashक) परसतत पकति म गौरी पर गाव क जमी-ार

क बड बट शरीकात क बार म कहा गया ह उसन बहत परिरशरम और उ-म क बा- ba की निडगरी परापत की थी अब वह एक -फतर

घर छोडकर जान स रोक कतिलयाइस पर बनी माधव सिसह न कहा निक बड घर की बटी ऐसी ही होती ह जो निबगडा काम बना लती ह अतः शीरषक साथक ह बड घर की बटी आन-ी ह

म कमचारी थाख) भरा हआ चहरा चौडी छाती और डटकर

खाना आदि- एक सबजी ल जवान क गण मान जात ह परत शरीकात न इनही नतर निपरय गणो को अपनी पढाई पर नयोछावर कर दि-या था

ग) बीए की निडगरी परापत कर लन पर भी उपय वयकति(शरी कठ की शारिररिरक तौर पर निनबल और चहर स कानित ही लगत थ इतना ही नही वह मानकतिसक तौर पर भी निपछड हए थ पाशचातय सामाजिजक कथा उस घणा एव पराचीन सभयता का गणगान उनकी निवचारधारा क परमख अग थ

घ) यह नतर निपरय गण गौरीपर गाव क जमी-ार क छोट बट लाल निबहारी सिसह म निवदयमान थ वह सजीलाजवान था और भस का दध शर दध वह सवर उठकर पी जाता था

ldquoयही कारण था निक गाव की लललन आए उनकी निन-क थी कोई कोई तो उह अपना शतर समझन म भी सकोच ना करती थी सवय उनकी पतनी को इस निवरषय म उनस निवरोध थाldquo

क) उपय पकति म इस वयकति क बार म कहा गया ह

ख) गाव की लललन आए उनकी निन-ा कयो निकया करती थी

ग) उनकी पतनी का कया नाम था उनह निकस निवरषय म अपन पनित क निवरa था और कयो

घ) इस कहानी का कया उददशय ह Continue to next helliphelliphellip

Bengali 2nd Language

ফ ফটক না ফটক( কহিতা )

পর) ldquo(ান াধাচেনা ফটপাচেথ পাথচের পাড হিচেয় এক কাঠচোটটা গাছ কহিকহি পাতায় পার ফাটিচেয় াসচেছldquoক) কার দো দেকান কহিতার অং( ) lsquo(ান াধাচেনা ফটপাচেথ পাথচের পাডহিচেয়lsquo চেত কী দোঝাচেনা চেয়চেছ গ) আচো য অংচে(lsquo এক কাঠচোটটা গাছ lsquoচেত কী দোঝাচেনা চেয়চেছ ঘ) ldquoকহিকহি পাতায় পার ফাটিচেয় াসচেছldquo ----- একথার পরকত অথC কী উততর ) ক) আচো য অং(টি পর যাত কহি সভা4 মচোপাধ যাচেয়র দো lsquoফ ফটক না ফটকrsquo কহিতার অং()কহি সভা4 মচোপাধ যায় হিছচেন দেপরচেমর কহি দেপরমচেক নানা ভহিঙগমায় হিতহিন ফটিচেয় তচেচেছন দেপরম মানচের স মচেতC র সঙগী কহিতার কহিতায় এক রb সb হচেয়র দেপরম াগরচেনর কথা চেচেছন (ান অথCাৎ দেযাচেন দেকান রস দেনই দেযাচেন দেকান মহিনতা দেনই অথ তার মধ দেযও দেপরম থাকচেত পাচের একথাই কহি তচে ধরচেত দেচেয়চেছন একটি মানচে4র মচেন দেযাচেন দেকামতার দেকান সথান দেনই পাথচেরর মচেতা হিনরসতার মচেনর মধ দেযও দেয দেপরম আসচেত পাচের দেস কথাই কহি চেচেছনগ)নারীচের যথC দেপরচেমর ছহি এই কহিতায় অকপচেট উচেঠ এচেসচেছ কহি এই কহিতায় কাটচোটটা গাছ কথাটি যার কচেরচেছন নারী দেয দেপরম দেথচেক হিতাহিত এং দেসই দেপরম সঠিক সমচেয় না পাওয়ার ন য দেপরম সমপচেকC হিচেr4 গৈতরী য় দেপরচেমর দেয গৈহি(ষট য মাধযC য সরসতা দেকামত এই সমসতর হিপরীত যথা রbতা শষকতা কচেঠার তা পরভহিত দোঝাচেত এক কাঠচোটটা গাছ কথাটি যার কচেরচেছনঘ) এাচেন এক নারীর যথC দেপরচেমর কথা হিনহিCপত ভাচে চেচেছন কহি অসমচেয় নারীর ীচেন দেপরম দেচেগচেছ এতহিন তার হয় রb কচেঠার হিছ দেপরচেমর অভাচে ঠাৎ দেসই শষক মরভহিমচেত সচের আভাস এচেসচেছ দেপরম দেযন 4Cার স(ীত তাই পরায় মত গাচেছ কহিকহি পাতা গহিচেয় উচেঠচেছ

Biology Chapter - 01Controlling Air Pollution

Today we will discuss how we control air pollution from domestic combustion

Q1Describe any five ways of reducing air pollution from domestic sources bull The number of pollutants in the air is verylarge and we always try to control them byfollowing ways

i) Solar cooker and solar heater It use no fuel reduce damage of environment by fuel use or reducing deforestation It maintains coolness of house It releases very less orno oil gas or grease

ii) Piped natural gas (PNG) It emits very less by products into the atmosphere As it isdistributed through pipe lines so there iscontinuous supply of fuel is possible

iii) Liquefied Petroleum Gas (LPG) It hasa higher heating value LPG doesntcontain sulphur so it burns a lot cleanerenergy sources It releases very less oralmost no fume in air

iv) Electricity based cooking Emission free cooking alternative for urban dwellers causeselimination of adverse health impactsofindoor air pollution It helps to avoid theinconveniences associated with procurement of LPG

v) Biogas It contains 75 methane whichmakes it an excellent fuel It burns without smoke and biogas plant leaves no residue like ash in wood charcoal etc Thus it isaclean fuel

Economics

Factors of Production

Today firstly we would recall the last class for 5 mins and then we would proceed with the further topics of the chapter

The concept meaning of land characteristics of land and importance of land to be repeated for the absentees as well as the students who were there in the class the previous day

Today we will start with the last portion of land before it the meaning of land to be repeated onceAs by now we all know that

Questions1What do you mean by productivity of landAnswer By productivity of land we mean the capacity of a piece of land to produce a crop

Thus it refers to the average output per unit of landSay per acre per hectare etc= (OutputArea of land)

2 What are the factors influencing the productivity of landAnswer

Natural factors Productivity of land is largely determined by the natural

Land is defined to include not only the surface of the earth but also all other free gifts of nature(for example mineral resources forest resources and indeed anything that helps us to carry out the production of goods and services but is provided by nature free of cost)

We will move on to the last portion of land by discussing Productivity of Land

By productivity of land we mean the capacity of a piece of land to produce a crop

Thus it refers to the average output per unit of land

Say per acre per hectare etc= (OutputArea of land)

With this we shall proceed further with the main factors that determine the productivity of land

Natural factors Human factors Improvements on land Location of land Organisation Ownership of land Availability of capital Proper use of land State help

Note economic development of a country depends upon the quality of its land If the land is fertile it will quicken the pace of development of the country

qualities of land such as fertility etc

Human factors Land cannot produce anything by itself Man has to apply labour on it to produce for himself So productivity of land depends on the knowledge and skills of workers

Improvements on land production of land is affected by land development measures like provision of well or tubewell irrigation proper drainage

State help The government of a country especially less developed country can play a vital role in improving the agricultural productivity by providing better irrigation facilities

Organisation Productivity of land also fdepends upon the way how the factors of production like labour and capital are organised

In order to increase productivity trained workers modern implements scientific methods good seeds are all essential

3 lsquoImproved technology affects the productivity of landrsquo Explain this statement with the help of suitable example Answer Use of improved technology raises the productivity of land Example By using HYV seeds chemical manures and modern machines per hectare output increases

Physics Force (Summary)

Question Write the expression for the moment of force about a given axisSolutionsThe expression for the moment of force is given byMoment of force about a given axis = Force times perpendicular distance of force from the axis of rotationQuestion What do you understand by the clockwise and anticlockwise moment of force When is it taken positiveSolutionsIf the effect on the body is to turn it anticlockwise moment of force is called the anticlockwise moment and it is taken as positive while if the effect on the

body is to turn it clockwise moment of force is called the clockwise moment and it is taken as negative

Math Topic Commercial Mathematics

Chapter Goods and services Tax

Study item Some solved sums from exercise ndash 1 A retailer buys a TV from a wholesaler for Rs 40000 He marks the price of the TV 15 above his cost price sells it to the consumer at 5 discount on the marked price If the sales are intra ndash state and the rate of GST is 12 find

(i) The marked price of the TV(ii) The amount which the consumer pays for the TV(iii) The amount of tax (under GST) paid by the retailer to the central

Government(iv) The amount of tax (under GST) received by the State Government

Solution As the sales are intra- state sale and the rate of GST 12 So GST comprises of 6 CGST and 6 SGSTTherefore a retailer buys a TV from a wholesaler for Rs 40000Therefore the amount of GST collected wholesaler from the retailer or paid by retailer to wholesalerCGST = 6 of Rs 40000 = Rs(6100 times40000) =Rs 2400SGST = 6 of Rs 40000 = Rs (6100 times 40000) =Rs 2400Therefore wholesaler will pay Rs 2400 as CGST and Rs 2400 as SGSTTherefore amount of input GST of retailer Input CGST = Rs 2400 and input SGST = Rs 2400Again the retailer marks the price of the TV 15 above his cost price(i) The marked price of the TV

= Rs 40000 + Rs 40000times15= Rs 40000 + Rs 40000times 15100= Rs 40000 + Rs 6000Rs 46000But the retailer sells it to consumer at 5 discount on the marked priceCost price after discount = Rs 46000 ndashRs46000times 5100 =Rs 46000 ndashRs 2300= Rs 43700Therefore the amount of GST collected retailer from consumer or paid by consumer to retailerCGST = 6 of Rs 43700 =Rs ( 6100 times43700)Rs 2622SGST = 6 of Rs 43700 = Rs (6100 times 43700) =Rs 2622Amount of the output GST of retailer Output CGST = Rs 2622 and output SGST = Rs 2622

(ii) The amount which the consumer pays for the TV= cost price of TV to consumer + CGST paid by consumer + SGST paid by consumer= Rs 43700 + Rs 2622 + Rs 2622= Rs 48944

(iii) The amount of tax (under GST ) paid by the retailer to the central Government=CGST paid by retailer = output CGST ndash input CGST=Rs 2622 ndash Rs 2400=Rs 222

(iv) The amount of tax ( under GST ) received by the State Government = SGST paid by wholesaler + SGST paid by retailer= Rs 2400 + output SGST ndash input SGST=Rs 2400 + Rs 2622 ndash Rs 2400=Rs 2400 + Rs 222= Rs 2622

Commercial studies

Stakeholders Today I am going to give some revision questions from the previous study material

Questions1) State the two expectations of

employees from a business concern2) Give two distinctions between

stakeholder and shareholder3) Give two difference between

internal stakeholders and external stakeholders

4) Give two expectations of suppliers from a business organisation

5) Who is a stakeholder in commercial organisations

Chemistry Periodic Table

Merits of Mendeleevrsquos Periodic law are as follows - 1He grouped the elements on the basis of atomic mass 2 He left gaps for undiscovered elements like Gallium Scandium germanium Also he left a full group vacant for undiscovered inert gases 3 He could predict proportions of several elements on basis of their position in periodic table like Ga Sc etc 4He could predict errors in atomic weights of some elements like gold platinum etc

Anomalies in Mendeleevrsquos Periodic law are as follows - 1 Position of isotopes could not be explained 2 Wrong order of atomic masses could not be explained

For example- as Arnur atomic mass 40 come first and K with low atomic mass (30) should come later but k should be placed first

According to Bohrrsquos Modern Periodic table properties of elements are periodic functions of their atomic numbers

So when elements are arranged according to increasing atomic numbers there is periodicity in electronic configuration that leads to periodicity in their chemical properties

It consists of horizontal rows (Periods) Vertical column (Groups)

There are 7 period and 12 groups in this long form of periodic table

Ist period has 2 elements IInd period has 8 elements IIIrd period has 8 elements IVth period has 18 elements Vth period has 18 elements VIth period has 32 elements VIIth period hs rest of elements

Note - The number of valence electrons in atom of elements decides which elements will be first in period and which will be last

In group- 1 to 2 gp and 13 to 17 contain normal elements 3 to 12gp ndash transition elements 57 to 71 - lanthanides 89 to 103 - Actinides

Left hand side ndash metals Right hand side ndash nonmetals

Note- Hydrogen element has been placed at top of Ist group Electronic configuration of H is similar to alkali metal as both have 1 valence electron

V electron of gp I element -- 1 V electron of gp 2 element -- 2 V electron of gp 13 element -- 3 V electron of gp 14 element -- 4 V electron of gp 15 element -- 5 V electron of gp 16 element --6 V electron of gp 17 element -- 7 V electron of gp 18 element -- 8

English 1 Transformation of sentences

Sentences A sentence is a group of words which makes complete sense

Exercise 2Change the following sentences from

a Assertive sentencesb Imperative sentencesc Interrogative sentencesd Exclamatory sentences

Sentences can be changed from one grammatical form to another without changing the meaning of the sentence This is known as transformation of sentences

assertive to interrogative1 Nobody would like to be a fool

Who would like to be a fool2 Their glory can never fade

When can the glory fade3 Nobody can control the wind

Who can control the wind4 It matters little if I die

What though I die5 No man can serve two masters

Can any man serve two masters

Exercise 3Interchange of assertive and Exclamatory sentences

1 She leads the most unhappy lifeWhat an unhappy life she leads

2 This is indeed an interesting bookWhat an interesting book this

3 He is a very great manWhat a great man he is

4 It is a very lame excuseWhat a lame excuse

5 It is sad that she died so youngAlas she died so young

Class XISubject Topic Summary Execution

Hindi 2nd lang

पतर परम(परमचदर) पतर परम कहानी म एक निपता की इचछाओ का वणन निकया गया ह अपन बड पतर परभ -ास स निपता चतनय -ास का निवशरष परम था निपता को उसक जनम स ही बडी-बडी आशाए थी उसम दसर बट कतिशव-ास की अपकषा स- उतसाह की मातरा अमिधक थी वह उस इगलड भजकर बरिरसटर बनाना चाहत थभागय का खल भी बडा निनराला ह बीए की परीकषा क बा- वह बीमार पड गया डॉकटरो न भी जवाब - दि-या थाचतन -ास जी बहत ही कजस थ बवजह पस खच करना नही चाहत थ अगर गारटी मिमलती तो शाय- पस खच भी कर -त परत गारटी नही थी परिरणाम सवरप उनक बट का -हात हो गयाजब बट को समशान ल जा रह थ तो वहा काफी शोर गान बजान हो रह थ पछन पर पता चला निक निकसी निपता निपछल तीन साल स निबमार था और उसक ईलाज म रपया पानी की तरह बहाया पर ठीक नही हए परत उसक बट को तनिनक भी अफसोस नही था उसका कहना था उसन कोकतिशश तो कीयह -खकर चतनय-ास जी को आतम निगलानी हईतभी स उनका म परिरवतन हआ और बट का भोज काफी धमधाम स निकयाऔर वहइस पशचाताप की आग म जलत रह औला- स बढकर पसा नही होता ह इस बात को समझन म उनह काफी व लग गया

hellipContinue to next

BENGALI(2ND LANGUAGE)

পরথমঅধযায়-ঠাকরারীনদরনাথঠাকর

নয়ন দোচের হিমাচেররা া নাচেমই হিযাত হিছচেন ায়ানার উাররণ সবরপ নয়ন দোচের ারা হিা (াচেকর হিা হিচেতন এছাাও দেকান উৎস উপচেb রাহিতর দেক হিন করার উচেfচে(য তারা সযC হিকরচেরণ রনয পরীপ জবাহিচেয় তাচেত রপার হির 4Cরণ করচেতন ঠাকরা এই নয়ন দো হিমারচের দে(4 ং(ধর হিছচেন হিমাররা ায়ানার ষটানত পর(Cন কচের তারা হিনঃসব এই হিমাহিরর দে(4 ং(ধর গৈকাস নদর রায়চেৌধরী গৈকাস া নয়ন দোচের সমসত সমপহিতত ঋচেরণর াচেয় হিহিx কচের অহি(ষট যা আচেছ তাচেত হিপত

ইার হিপতার মতয ইচে পর নয়নচোচের ায়ানার দেগাটা কতক অসাধাররণ শরাদধ (াহিনতচেত অহিনতম ীহিপত পরকা( কহিরয়া ঠাৎ হিনহিয়া দেগ- ক) কার দো দেকান গচেলপর অং() কতা দেক ইার চেত কাচেক দোঝাচেনা চেয়চেছ গ) পরসঙগ কী কতার কতয পহিরসফট কচেরা

পরচে4র যাহিত রbা করা সমভ নয় তাই হিতহিন পতরচেক হিনচেয় ককাতায় সাস শর কচেরন গলপ কথচেকর আহিথCক অসথা নয়ন দোচের হিমাচের দেথচেক সমপরণC আাা কথচেকর হিপতা হিনচের দেষটায় অথC উপাCন করচেতন া উপাহিধ াচেভর নয তার াসা হিছনা আর দেসই কারচেরণ কথক তার একমাতর উততরাহিধকার চেয় তার হিপতার পরহিত কতজঞ কথক দো পা হি(চেচেছন হিনচের পরারণ ও মান রbার নয উপচেযাগী অথC হিনা দেষটায় পরাপত চেয়চেছন- এটাই তার কাচেছ পরম দেগৌরচের হি4য় চে মচেন কচেরন কাররণ (নয ভাণডাচের গৈপতক ায়ানার উজজব ইহিতাস অচেপbা দোার হিসeচেকর মচেধয গৈপতক দেকামপাহিনর কাগ তার কাচেছ অচেনক দেহি( মযান

TO BE CONTINUED

উ- ক) আচোয অং(টি রীনদরনাথ ঠাকচেরর দো ঠাকরা গচেলপর অং() কতা চেন আচোয গচেলপর গলপ কথকইার চেত নয়ন দোচের হিমাহিরর দে(4 ং(ধর গৈকাস ার কথা া চেয়চেছ গৈকাস া নয়ন দোচের সমসত সমপহিতত ঋচেরণর াচেয় হিহিx কচের অহি(ষট যা আচেছ তাচেত হিপত পরচে4র যাহিত রbা করা সমভ নয় তাই হিতহিন পতরচেক হিনচেয় ককাতায় সাস শর কচেরনগ) গৈকাস ার হিপতার মতযর পর নয়ন দোচের হিমাহিরর অহিসততব হিপত য় কচেয়কটা উৎস ও শরাদধ- (াহিনতচেত হিমাহিরর দে(4 কহিটক যয় চেয় হিগচেয় এচেক াচের দে(4 চেয় যায় তন তাচের গC করার মত আর হিকছই হিছ না-দেসই পরসচেঙগ এই উহিকত নয়নচোচের হিমাচেররা া নাচেমই হিযাত হিছচেন ায়ানার উাররণ সবরপ নয়নচোচের ারা হিা (াচেকর হিা হিচেতন এছাাও দেকান উৎস উপচেb রাহিতরচেক হিন করচেত হিগচেয় তারা সযC হিকরচেরণর নয পরীপ জবাহিচেয় তাচেত রপার হির 4Cরণ করচেতন তাই দেসকাচের ায়ানা দেহি(হিন সথায়ী চেত পারত না হিহিভনন উৎস শরাদধ- (াহিনতচেত সাধযা হিতহিরকত র করার নয হিমাহির হিহিকচেয় দেযত হ হিতC কা হিহি(ষট পরীচেপর দেত দেযমন অলপকাচের মচেধয হিনঃচে(4 চেয় যায়-নয়নচোচের হিমারচের অসথা তাই চেয়হিছ এই কারচেরণই কথক নয়নচোচের হিমারচের গা ভরা আমবর সয করচেত পারতনা

Physics Dimensional Analysis (Summary)

Q Find the dimensions of consts ab in relation

p=(bminusxlowastx)at

where p is the power x is the distance and t is time

Ans From principle of homogeneity dimension of b x2 are same Dim of b = dim of x2 = [L2] = [ML2T0]Dim of a = dim of ( b- x2)dim of (pt) = [M0L2T0][ML2T-2] [T-1] [T] = [M-1L0T2]

Chemistry Atomic Structure Drawbacks of Rutherfordrsquos model of

atom a According to Rutherfordrsquos model of atom electrons which are negativelycharged particles revolve around the nucleus in fixed orbits Thusb theelectrons undergo acceleration According to electromagnetic theory of Maxwell a charged particle undergoing acceleration should emitelectromagnetic radiation Thus an electron in an orbit should emitradiation Thus the orbit should shrink But this does not happenc The model does not give any information about how electrons aredistributed around nucleus and what are energies of these electrons Isotopes These are the atoms of the same

Properties of electromagnetic radiationsa Oscillating electric and magnetic field are produced by oscillating charged particles These fields are perpendicular to each other and both areperpendicular to the direction of propagation of the waveb They do not need a medium to travel That means they can even travel invacuum

Characteristics of electromagnetic radiationsa Wavelength It may be defined as the distance between two neighbouring crests or troughs of

element having the same atomicnumber but different mass numbere g 1H11H21H3

Isobars Isobars are the atoms of different elements having the same massnumber but different atomic numbere g 18Ar40 20Ca40

Isoelectronic species These are those species which have the same numberof electrons

Electromagnetic radiationsThe radiations which are associated withelectrical and magnetic fields are called electromagnetic radiations When anelectrically charged particle moves under acceleration alternating electricaland magnetic fields are produced and transmitted These fields aretransmitted in the form of waves These waves are called electromagneticwaves or electromagnetic radiations

wave as shown It is denoted by λb Frequency (ν) It may be defined as the number of waves which passthrough a particular point in one secondc Velocity (v) It is defined as the distance travelled by a wave in onesecond In vacuum all types of electromagnetic radiations travel with thesame velocity Its value is 3 times10 8m sec-1 It is denoted by v

d Wave number Wave number is defined as the number of wavelengths per unit lengthVelocity = frequency timeswavelength c = νλ

Plancks Quantum Theory- o The radiant energy is emitted or absorbed not continuously but discontinuously in the form of small discrete packets of energy called lsquoquantumrsquo In case of light the quantum of energy is called a lsquophotonrsquoo The energy of each quantum is directly proportional to the frequency of the radiation ie E α υ or E= hυ where h= Planckrsquos constant = 6626 x 10-27 Js o Energy is always emitted or absorbed as integral multiple of this uantum E=nhυ Where n=1234Black body An ideal body which emits and absorbs all frequencies is calleda black body The radiation emitted by such a body is called black body radiation

Photoelectric effectThe phenomenon of ejection of electrons from thesurface of metal when light of suitable frequency strikes it is calledphotoelectric effect The ejected electrons are called photoelectrons

Biology Chapter - 02Systematics and Five Kingdoms

Scientists divide the whole living organisms into two kingdom first and ultimately by five kingdom at last

In the earlier systems of classifications organisms are divided into kingdom plantaeand kingdom animalia on the of presenceof cell wall their modes of nutrition and movements

Some problem arise like fungi share manycharacteristic withplant despite their heterotrophic nutrition bacteria protozoa areunicellular present in both kingdom Toovercome this third kingdom Protista isintroduced which include

unicellularorganisms But there is also another

problem Allunicellular organisms are not similar kind The cellular structure of prokaryotes is verydifferent from that of other organismsEukaryotes possess a true nucleus and allcell organelles that are not present inprokaryotes So the fourth kingdom Monerais introduced which include unicellular prokaryotes (bacteriaamp blue green algae)

bull Still some problem arise in kingdomplantae

So in 1969 R H Whittakar proposedanew five kingdom System of classification

i) Kingdom Monera - unicellular prokaryotes

ii) kingdom Protista - unicellular eukaryotes

iii) Kingdom Fungi - uni or multicellular fungi with cell wall but without chlorophyll

iv) Kingdom Plantae - Multicellular Plants

v) Kingdom Animalia - Multicellular Animals

EVS Chapter 1 ndash Modes of Existence

An agricultural society

An agricultural society also known as an agrarian society is a society that constructs social order around a reliance upon farming More than half the people living in that society make their living by farming

People in an agricultural society generally lead a more settled lifestyle than those in nomadic hunter-gatherer or semi-nomadic pastoral societies because they live permanently near the land that is farmed Agricultural settlements tend to develop in areas of convenience near bodies of water which is used for both crops and transportation or along trade routes Not everyone in an agricultural society is a farmer Some people make a living trading or making and selling goods such as tools used for farming

Another way to define an agrarian society is to see the total amount of production in a nation In an agrarian society cultivating the land is the main source of wealth Such a society can recognize other means of subsistence and work habits but emphasizes the importance of agriculture and livestock Agrarian societies have existed in various parts of the world for 10000 years and continue to exist today They have been the most common form of socio-economic organization for most of recorded human history

Q) Write the features of agricultural society

Ans - Structure and Features of Agrarian Society1 Occupational Structure

An agrarian society is generally associated with the domestication of plants and animals The domestication of plants means farming and that of animals means herding Often there is mixture of farming and the use of such domesticated animals as cow goat and sheep

2 Forms of Land Ownership in Agrarian SocietiesGenerally there are landlords supervisory farmers cultivators and share croppers The landholders own the land but do not work on it They let it out for sharecropping The supervisory farmers are those who live by having their land cultivated by hired labourers The cultivators cultivate the land for themselvesThe share-croppers are those who live by tilling other peoplersquos land or a crop-sharing basis The artisans own their means of production and produce by their own labour in their homesteads

3 Village Community System An agrarian society is highlighted by

the institution of village community system The agrarian economy made fixed dwelling houses necessary Living close together for protection and co-operation and living nearer to the land gave birth to agricultural villages The village is not only the residential place of farmers it is also the social integrator

4 Minimal Division of Labour Another structural feature of agrarian society is a minimal division of labour Except for the basic division founded on age and sex differences there are few specialized roles There is only one predominant type of occupation ie domestication of plants and animals For all the people the environment physical as well as social is the same

5 Role of Family The farm family is of the patriarchal type the father is the final arbiter in most of the familyrsquos major decisions The life of ail men and women is merged in family life Since there are not many special organizations family is the only organisation to perform the tasks of aid and protection

6 Sense of Unity The members of an agrarian society exhibit a strong in-group feeling Since the whole of their social lives is wrapped up in a society which is physically economically and socially homogenous they are inclined to view the entire outside world as an out group

7 Informal Social Control An agrarian society is regionally divided into villages In a village community the force of traditional mores is more dominant than in the urban community In the village everybody is known to everybody The members in a village community help each other and share the joy and sorrows of each other Crime in an agrarian society is rare

8 Simplicity and Uniformity Life of the people in an agrarian society is marked by simplicity and uniformity Their main occupation is agriculture which largely depends upon the vagaries of nature An agrarian society is a religious society

Math Compound angles Compound angles The algebraic sum of two or more angles is called a compound angle If A B C be three angles then A+B B+C C+A A-B B-C A-C A+B-C etc are compound angles In this chapter we shall discuss the trigonometrical ratios of compound angles Theorem 1 If A B and A+B are all pisitive acute angles theni) sin( A+B) = sin A cos B + cosA sinBii) cos(A+B) = cosA cosB- sinA sinBTheorem 2If A and B are positive acute angles and AgtB theni) sin(A-B) = sin A cosB- cos A sinBii) cos(A-B) = cos A cos B+ sin A sin BTo prove that i) sin(A+B) sin (A-B) = sin2 A - sin2 B = cos2 B- cos2 A

Example 1 Prove that tan70deg=2tan50deg+tan20degSolutiontan70deg = tan(50deg + 20deg)Or tan70deg=(tan 50deg+tan 20deg)(1-tan50degtan20deg) or tan70deg (1 ndash tan 50deg tan20deg) = tan50deg+tan20degor tan70deg= tan70deg tan50deg tan20deg+ tan50deg + tan20deg = cot20deg tan50deg tan20deg + tan50deg + tan20deg = 2 tan50deg+ tan20degExample 2 If A + B = 45deg show that (1 + tanA) (1 + tanB) = 2Solutiontan(A + B) =( tan A + tan B) (1 - tan

ii) cos(A+B) Cos(A-B) = cos2 A- sin2 B = cos2 B -sin2 AProof i) LHS= sin(A+B)sin(AminusB) [Recall sin(αminusβ)=sinαcosβminuscosαsinβ And sin(α+β)=sinαcosβ+cosαsinβ]= (sinAcosB+cosAsinB)times(sinAcosBminuscosAsinB)= sin2Acos2Bminuscos2Asin2B [Recall sin2α+cos2α=1 From above we can then assume correctly that sin2α=1minuscos2α AND cos2α=1minussin2α] = sin2A(1minussin2B)minussin2B(1minussin2A) = sin2Aminussin2Asin2Bminussin2B+sin2Asin2B = sin2Aminussin2B= 1-cos2A-(1-cos2B) = cos2 B- cos2 A = RHSii)LHS= cos (A+B) cos(A-B) [ cos(A+B) = cos AcosB- sinAsinBCos(A-B) = cosAcosB+ sinAsinB]= cos2 A Cos2 B- sin2 A Sin2 B= cos2 A( 1-sin2 B) - (1- cos2 A) sin2 B= cos2 A- cos2 A sin2 B- sin2 B+ cos2 A sin2 B=cos2 A- sin2 B=1- sin2 A-(1-cos2 B) = cos2 B- sin2 A= RHSTangent formulae for compound anglesi)tan (A + B) = tan A + tan B1-tan A tan Bii)tan (A ndash B) = tan A-tan B1+tan A tan Biii) cot (A + B) = cot Acot B-1cot A+cot B(viii) cot (A ndash B) = cot Acot B+1cot B-cot A

A tan B) Or 1= (tan A+ tanB) (1-tan A tanB) Or tanA + tanB + tanA tanB + 1 = 1 + 1Or tanA (1 + tanB) + (1 + tanB) = 2Or (1 + tanA) (1 + tanB) = 2Example 3 Find the value of sin 15degSolution sin 15deg= sin(45deg-30deg) = sin45degcos 30deg- cos45degsin30deg =(1radic2) (radic32) -(1radic2) (12) = (radic3-1) 2radic2Example 4 If sin A = 1 radic10 and sin B = 1 radic5 where A and B are positive acute angles then what is A + B SolutionWe know that sin (A + B) = sin A cos B + cos A sin B= [1 radic10] [radic(1 minus 1 5)] + [1 radic5] radic(1 minus 1 10)= [1 radic10] [radic4 5] + [1 radic5] [radic9 10]= [1 radic50] times (2 + 3)= 5 radic50 = 1 radic2

sin (A + B) = sin π 4rArrHence A + B = π 4Example 5 If A + B = 225o then find [cot A] [1 + cotA] times [cot B] [1 + cot B]Solution[cot A] [1 + cotA] times [cot B] [1 + cot B] = 1 [(1 + tan A) times (1 + tan B)]=1 [tan A + tan B + 1 + tan A tan B] [ tan (A + B) = tan225o]∵

tan A + tan B = 1minus tan A tan BrArr= 1 [1 minus tan A tan B + 1 + tan A tan B]= 1 2

COMMERCE

CLASSIFICTION OF HUMAN ACTIVITIES-ECONOMIC AND NON-ECONOMIC

Firstly we shall recall the previous class for 5 mins especially for the absentees and for also the rest of the students who were there

Today at first we briefly discuss the earlier portions of the chapter

1Business-It includes all those economic activities which are concerned with production and exchange of goods and services with the object of earning profit Example A factory shop beauty parlour also business enterprises

2Profession ndashThe term profession means an occupation which involves application of specialized knowledge and skills to earn a living For Example Chartered Accountancy medicine law tax consultancy are example of professions

Questions1What are the main features of ProfessionAnswer The main features of a profession are as follows a Specialised body of knowledge-Every profession has a specialised and systematised body of knowledge b Restricted entry- Entry to a profession is allowed only to those who have completed the prescribed education and have the specialised examination c Formal education and training ndashA formal education and training is given to the person who wants to acquire the professional

3Employment-Employment mean an economic activity where people work for others in exchange for some remuneration (salary)The persons who work for others are called lsquoemployeesrsquo The persons or organizations which engage others to work for them are called lsquoemployersrsquoEg A doctor working in a hospital is employment as he is working for a salaryA lawyer may serve as a law officer in a bank

With this we shall proceed with the features of both Profession amp Employment

The main features of a profession are as follow

a Specialised body of knowledge b Restricted entry c Formal education and training d Professional association e Service motive f Code of contact

The main features of an employment are as follows

a In employment a person works for others called employer

b An employee provides personal service

c There is a service agreement or contract between the employee and the employer

d The employee has to obey the order of the employer

e No capital investment is made by the employer

Various examples of Employment are as follows

aA teacher teaching in a school or collegeb An engineer employed in Municipal Corporation of DelhicAn accountant working in the accounts department of a companydA doctor working in a hospital

Note In all the above examples of employment the individual who is involved in each example is working as an employee for a salary under an employer

qualification(MBBSCALLB)d Service motive ndashProfessionals are expected to emphasis service more on their clients rather than economic gain f Code of Conduct-The activities of professionals are regulated by a code of conduct

2 What are the main features of EmploymentAnswer The main features of an employment are as followsa In employment a person works for others called employerb An employee provides personal servicec There is a service agreement or contract between the employee and the employerd The employee has to obey the order of the employere No capital investment is made by the employer

3 Give various Professions and their respective Association are given below

Professions

Professional

Professional association

Medical profession

Doctor Medical Council of India

Law profession

Lawyers Bar Council of India

Accounting Profession

Chartered

The Institute of Chartered Accounts of India( ICAI)

Engineerin Engineers The

g Profession

institute of Engineers (India)

Accounts Basic accounting terms

Today we will give you some questions from the previous study material

Questions6) Define accounting7) What do you mean by debit

and credit8) Explain the types of account9) Define the following terms

a) Assetsb) Capitalc) Purchased) Debtorse) Transactions

10) Name the types of accounts given below

a) Krishnas accountb) Machinery accountc) Royalty accountd) Salary accounte) Furniture accountf) Audit fee account

Economics Basic Economic ConceptsSub topic

UTILITY

Before starting todayrsquos class we shall recall the last class which was about UTILITY AND THE FEATURES OF UTILITY

Now we shall proceed with the further topics of the chapter

Todayrsquos topic from the chapter lsquo Basic Economic Conceptsrsquo will be TOTAL UTILITY amp MARGINAL UTILITYNow let us quickly revise the concept of utility with an example ie goods and services are designed because they have an ability to satisfy human wantsThis feature of being able to satisfy human wants is termed as utility For example we derive utility from WiFi services as it gives us satisfaction by connecting us to our friends and family through social media here consumers derive utility from WiFi services

From the above concept we shall start with todayrsquos topicEconomists have defined TOTAL UTILITY (TU) as the total satisfaction obtained by consuming a given total amount of a good and serviceFor example the total satisfaction obtained from eating 10 mangoes is the total utility of 10 mangoes

MARGINAL UTILITY (MU) is the additional satisfaction derived from each additional unit

Questions1 What is Total Utility (TU)

Answer Total Utility (TU) is the

aggregate of the utility that a consumer derives from the consumption of a certain amount of a commodityTU=MU1+MU2++MUn

2 What is Marginal UtilityAnswer

Marginal Utility (MU) is the additional made to the total utility as consumption is increased by one more unit of the commodityMU= TUn ndashTUn-1

NoteOften economists tend to

subdivide utility into an imaginary unit called UTIL

consumed In this casethe utility obtained from each mango as it is consumed as the MU of that mango It is also defined as the addition made to the total utility when an additional unit is consumed Often economists tend to subdivide utility into an imaginary unit called UTIL

Note As a consumer increases the consumption of a good over period of time the total utility or total satisfaction derived from it increases to appoint and thereafter it decreasesHowever as the consumer keeps on consuming the good the marginal utility or the additional utility derived from it decreases

SubjectBusiness studies

Topic

BUSINESSENVIRONMENT

Summary

Now quickly let us revise the earlier points that we have already done in the last class and let us proceed with the other topics that are there in the chapter

Firstly we will recall the internal and external factors of micro environment and then we shall proceed in details

Meaning and list of internal and external factors

aInternal factorsInternal factors refer to all the factors existing within a business firm The internal factors are considered controllable because the enterprise has control over these factorsFor an example a company can alter its organization structure policies programmes employees physical facilities and marketing mix to suit the changes in the environmentList of internal factors areCorporate culture mission and objectives top management organizations structure company image and brand equity company resources

b External factorsExternal factors refer to those individual and groups and agencies with which a particular business organization comes into direct and frequent contact in the course of its functioningThese individuals and groups are known as STAKEHOLDERS because they have a stake (financial interest ) in the working and performance of the particular business List of external forces (stakeholders)Customers competitors investors suppliersmiddlemen (marketing intermediaries)

Execution 1 What do you mean by internal

factors in micro environmentAnswerInternal factors refer to all the factors existing within a business firm The internal factors are considered controllable because the enterprise has control over these factorsFor an example a company can alter its organization structure policies programmes employees physical facilities and marketing mix to suit the changes in the environment

2 What do you mean by external factors in micro environment

AnswerExternal factors refer to those individual and groups and agencies with which a particular business organization comes into direct and frequent contact in the course of its functioningThese individuals and groups are known as STAKEHOLDERS because they have a stake (financial interest) in the working and performance of the particular business

3Who are stakeholdersSTAKEHOLDERS are individuals and groups who have a stake (financial interest ) in the working and performance of the particular business 4Discuss the internal factors in briefa Corporate CultureThe values beliefs and attitudes of the founders and top management of the company exercise

financers publics

customers

suppliersfinancers

competitors

middlemen

publics

Fig STAKEHOLDERS OF A COMPANY

Apart from micro environment the other main dimension of business environment isMacro environment Macro environment refers to the general environment or remote environment within which a business firm and forces in its micro environment operateA company does not directly or regularly interact with the micro environmentTherefore macro environment is also known as indirect action EnvironmentThe macro environment forces are less controllable than the micro forces

Macro environment consists of the following components

POLITICAL AND LEGAL ENVIRONMENT

ECONOMIC SOCIAL AND ENVIRONMENT

CULTURAL

ENVIRONMENT

TECHNOLOGICAL ENVIRONMENT

a strong influence on what the cmpaany stands for how it does things and what it considers importantbMission and objectivesThe business philosophy and purpose of a comoany guide it prioritiesbusiness strategiesproduct market scope and development scope

cTop management structurethe composition of board of directors the degree of professionalization of management and the organizational structure of a company have important bearing on its business decisions

dPower structureThe internal power relationship between the board of directors and the chief executive is an important factor

eCompany image and brand equityThe image and brand equity of the company play a significant role in raising finance forming alliance choosing dealers and suppliers launching new products entering foreign markets

5 What is Macro environmentAnswerMacro environment refers to the general environment or remote environment within which a business firm and forces in its micro environment operateA company does not directly or regularly interact with the micro environmentTherefore macro environment is also known as indirect action EnvironmentThe macro environment forces are less controllable than the micro forces 6 What are the components of macro environmenta Political and legal environmentb Economic environmentc Social and cultural environmentd Technological environment

BUSINESS FIRM

Fig COMPONENTS OF MACRO ENVIRONMENTPolitical science

Introduction to political science

Comparative politics and itrsquos scope Comparative politics is the second major dimension of political scienceIt is also a very vast area of study and a very large number of political scientists even treat it as an autonomous area of study within the board ambit of political scienceScope of comparative politics-

1 All political structures -Comparative politics includes the study of all structures formalnon formal governmental and extra governmental which are directly or indirectly involved in politics in all the countries of the world

2 Functional studies- Comparative politics seeks to study politics less from the point of view of the legal institutions in terms of their powers and move from the point of view of their functions which constitute the political process and their actual Operation in the environment

3 Study of political behaviour- Another important part of its scope is the study of the actual behaviour of the people in the process of politics

4 Study of similarities and differences- comparative politics also undertakesan analysis of the similarities and differences among political process and functions

5 Study of all political systems -comparative politics seeks to analyse the actual behaviour and performance of all political systems western as well as non western

6 Study of the environment and infrastructure of politics-The study of politics demands a study of the psychological sociological economic and anthropological environment in fact the social environment as a whole in which each political system operates

7 Study of political culture- political culture is composed of attitudesbeliefs emotions and values of a society that relate to the political system or politics

8 Study of political participation- Political participation is a universal processThe only difference is that while in some states it is limited in others it is wider

9 Study of political process- political

Answer the following questions-

What is comparative politics

What are the scope of comparative politics

Homework- learn

processes like decision makingpolicy making judicial process leadership recruitment process and others are always at work in all political systems

The scope of comparative politics is very comprehensive It includes everything that falls within the area of political activity and political process

History CAMBRIDGE VIEW ABOUT

THE PARTITION

AND REFUTATION

OF CAMBRIDGE

VIEW

Cambridge view about the Partition The Cambridge school of historians have interpreted that opposition to partition scheme was made entirely by the elitist groups They hold the view that Lord Curzon planned to partition the Bengal for administrative purposeREFUTATION OFCAMBRIDGE VIEW The Rationalist historians have rejected the interpretations of the Cambridge School of historians on various grounds

1 QUESTION State different views of historians regarding Partition of Bengal

ANSWER Cambridge historians believed that Lord Curzon partitioned Bengal for administrative reasons only and not for the political motive The Middle class elitist group protested because of their petty interest The Hindu zamindars protested as they have to spend more money for managing their estatesThe lawyers of Calcutta High court feared to lose their clientBut according to the nationalist Historians was-

2- The ultimate object of Lord Curzon was to crush the unity of Bengal politicians

3- If Bengal becomes a separate province Bengali speaking 16 million people of western part would become minority under Hindi speaking people of Bihar and Oriya speaking people of Orissa

4- The bureaucrats expected that the protest movement would die down quickly

5- Lord Curzon used the Muslim community in his political game

6- Idealism had great contribution in the protest against partition

7- The people of the every section of society were affected by the partition of Bengal

Computer Science

Numbers Convertion of dcimal number to octal numberThe decimal numeral system is the standard system for denoting integer and non-integer numbers It is the extension to non-integer numbers of the Hindu-Arabic numeral system For writing numbers the decimal system uses ten decimal digits a decimal mark and for negative numbers a minus sign - The decimal digits are 0 1 2 3 4 5 6 7 8 9 the decimal separator is the dot in many countries

The octal numeral system or oct for short is the base-8 number system and uses the digits 0 to 7 Octal is sometimes used in computing instead of hexadecimal perhaps most often in modern times in conjunction with file

permissions under Unix systems It has the advantage of not requiring any extra symbols as digits It is also used for digital displays

Follow these steps to convert a decimal number into octal form

1 Divide the decimal number by 82 Get the integer quotient for the next iteration (if the number will not divide equally by 8 then round down the

result to the nearest whole number)3 Keep a note of the remainder it should be between 0 and 74 Repeat the steps until the quotient is equal to 05 Write out all the remainders from bottom to top This is the solution

For example if the given decimal number is 8453

Division Quotient Remainder

8453 8 1056 5

1056 8 132 0

132 8 16 4

16 8 2 0

2 8 0 2

Then the octal solution is 20405

Subject Eng Literature (The Tempest ndash William Shakespeare) Topic Act I Scene 1 Lines 33 to 67 (End of scene) Date 16th April 2020 (4th Period)

[Students should read the original play and also the paraphrase given in the school prescribed textbook]Summary Questions amp Answers

[SUMMARY OF THE ENTIRE SCENE]

o The play starts with the scene of a severe storm at sea Alonso (King of Naples) Sebastian (Alonsorsquos brother) Ferdinand (Alonsorsquos son) Gonzalo Antonio (the usurping Duke of Milan) are in a ship in the midst of the storm

o The mariners are trying their best to control the vessel from running aground and are totally following the orders of their Master the Boatswain They have scant success

o The mariners become extremely unhappy and annoyed when most of the passengers arrive on the deck thereby hampering their effort to save the ship There is serious confrontation between them and the passengers who are part of the Kingrsquos entourage

o The mariners could not save the ship

SUMMING-UP

(i) Vivid description of the scene which gives a realistic description of terror and confusion of a tropical storm

(ii) Shows Shakespearersquos accuracy of knowledge in describing the naval operations and also matters of seamanship

(1) GONZALO Ill warrant him for drowning (L 45-57)

though the ship were no stronger than a nutshell and as leaky as an unstanched

wenchBOATSWAIN Lay her a-hold a-hold Set her two courses Off to

sea again lay her offMARINERS All lost To prayers to prayers All lostBOATSWAIN What must our mouths be coldGONZALO The king and prince at prayers Lets assist them

For our case is theirsSEBASTIAN Im out of patienceANTONIO We are merely cheated of our lives by drunkards

This wide-chopped rascal - would thou mightst lie drowning the washing of ten tides

(a) What does Antonio say at the insolent manners of the boatswain just before the given passage

Being irritated at the insolent manners of the boatswain just before the given extract Antonio the Duke of Milan calls him a worthless dog son of a woman without any morals an arrogant and disrespectful noisemaker He says that the boatswain deserved to be hanged(b) What statement does Gonzalo repeat about the boatswain

Gonzalo shows his faith that the boatswain is not destined to die by drowning He is destined to be hanged and nothing can alter this decree of destiny He says that even if the ship was as frail as a nutshell the boatswain could not be drowned for his destiny was to be hanged(c) What do the passengers do when they have lost all hope of their survival

When the passengers have lost all hope of survival they take

(iii) The opening scene justifies the title ndash The Tempest

UNANSWERED QUESTIONS

(i) The King always travels with his entire fleet including his soldiers Where were the other ships

(ii) Why was the ship in that area Where was it coming from or going where

(iii) The ship broke apart What happened to those who were in the ship

(We shall get the answer to the above questions as the play progresses)

leave of life with fervent prayers The mariners take their last hearty drink and are ready for death(d) What blame does Antonio put upon the mariners and the boatswain Antonio rebukes the mariners that these drunkards have brought them to the present crisis by neglecting their duties He blames them saying that they are going to lose their lives entirely for the negligence of the boatswain and his fellows(e) What does Antonio say while cursing the boatswain

Antonio gives vent to his wrath upon the boatswain in particular He calls the boatswain a wide-mouthed rascal who deserves to be hanged on the sea-shore at low water mark so that ten tides might wash over his body and take out of him all the liquor that he has been drinking

Class XIISubject Topic Summary ExecutionHistory Topic

1 1935 ACT AND WORKING OF PROVINCIAL AUTONOMYCONGREE AND OTHER MINISTERSSUB TOPIC GOVERNMENT OF INDIA ACT1935

Government of India Act 1935 This act established a lsquoFederation of Indiarsquo made of British Indian provinces and Indian states and provided for autonomy with a government responsible to the elected legislature in every provinceThis act introduced abolition of Diarchy at provinces The entire provincial administration was introduced to the responsible ministers who were controlled and removed by the provincial legislature The provincial autonomy means two things First The provincial governments were wholly responsible to the provincial legislature Secondly Provinces were free from outside control and interference in the large number of matters The act divided the powers between the centre and provinces in terms of three lists- Federal list( for centre) Provincial list (for province) and concurrent list (for both) Residuary powers were given to the viceroy In the election under the government of India Act the Congress swept the poll the mandate of the people came in favour of the congress so far as general Hindu seats were concerned The Congress did not get a single Muslim seates in Bombay CP UP Sind and BengalIn five provinces Congress had yhe clear majority In BengalNWFPAssam and Bombay Congress emerged as a single largest partyOn the other side the performance of the Muslim League was badThus the Congress formed ministers in 7 provinces out of 11 provinces Coalition ministry was also formed in two other provincesOnly BENGAL AND Punjab had non- congress ministries

1 QUESTION What was the main change introduced by the Government of India ActANSWER a) The Act gave more

autonomy to the provinces b) Diarchy was abolished at the

provincial levelsc) The Governor was the head of

the executived) There was a council of

ministers to advise him The ministers were responsible to the provincial legislatures who controlled them The legislature could also remove the ministers

e) The Governors still retained special reserve powers

2 QUESTION Why did the federal scheme introduced by the Government of India Act 1935 never come into operation

ANSWER The Federal structure of the Government of India was to be composed with the Governor General and Council of ministers The Federal legislature was to be Bicameral legislature- The council of states and the House of Assembly The ministers were to be chosen by the Governor general and they were to hold the office during his pleasure

The provinces of British India would have to join the federation but this was not compulsory for the princely states

This federation never materialised because of the lack of support from the required number of

princely statesThis act was refused and

rejected by the princes the Congress and the Muslim League

Thus both Congress and the League participated in the election of 1937 Thus the federal part was never introduced but the provincial part was put into operations

Bengali 2nd

Language

াচেরর পরাথCনা(কহিতা )

াচেরর পরাথCনা কহিতাটি কহি (ঙখ দেঘাচে4র দো আচো য কহিতায় াচেরর পতর হমায়ন কঠিন দেরাচেগ আxানত ার ঈশবর া আললার কাচেছ পরাথCনা কচেরচেছন তার পচেতরর ীন হিফহিরচেয় হিচেত এই কহিতায় ার পচেতরর ীন হিভbা দেচেয়চেছন ারার এমনহিক হিনচের ীন হিসCচেনর হিহিনমচেয় হিতহিন তার দেছচের ীন হিফচের দেপচেত দেচেয়চেছন তার দেছচের এই দেরাচেগর ন য হিতহিন হিনচেচেকই ায়ী কচেরচেছন তার হিনচের করা পাপচেকই হিতহিন ায়ী কচেরচেছন এছাা রানৈনহিতক ও আথCসামাহিক অসথার কথা তচে ধরা চেয়চেছ এই কহিতায় ার তার হিনচের পাপ কমCচেকই ায়ী কচেরচেছ ার অন যায় ভাচে দেপহি((হিকতর মাধ যচেম অপররা য কচেরচেছ আর এই অন যায় কাচের ন যই তার পহিরাচের হিপযCয় এচেসচেছ দে এক পরকার মানহিক নধন ইহিতাচেসর ার হিপতা চেয় সবাভাহিকভাচে ভাচোাসা দে মমতা দেথচেক মকত চেত পাচেরনহিন তাই হিপতা চেয় আললা া ভগাচেনর কাচেছ পতর হমায়চেনর পরানহিভbা দেচেয়চেছন ার আললা া ভগাচেনর কাচেছ াহিনচেয়চেছন তার হিনচের ীন হিসCন হিচেত হিতহিন রাী তার হিহিনমচেয় পচেতরর ীন হিফচের দেপচেত দেচেয়চেছন াচেরর হিপতসভ হিচেকর কথা এই কহিতায় ফটিচেয় দেতাা চেয়চেছ হিপতা পচেতরর হিরাহিরত মান নধচেনর কথা তচে ধরা চেয়চেছ

হিচে(4 হিকছ াইচেনর তাৎপযC১) ldquoদেকাথায় দেগ ওর সবচছয দেৌন দেকাথায় কচেরায় দেগাপন bয়ldquoউততর) াচেরর পতর হমায়ন কঠিন দেরাচেগ অসসথ তাই তার দেযৌন াহিরচেয় যাচেচছ এই দেরাচেগ তাচেক দেগাপচেন কচেরকচের াচেচছ তার সক (হিকত ধীচের ধীচের bয় চেচছ তাই হিপতা চেয় ার আললার কাচেছ হমায়চেনর পরান হিভbা দেচেয়চেছন২) ldquoাগাও (চেরর পরাচেনত পরানতচের ধসর (ন দেযর আান গানldquoউততর) াচেরর পতর হমায়ন কঠিন দেরাচেগ আxানত তাই ার আ দে(াচেক মমCাত (চেরর পচেথ পরানতচের আান গান ধবহিনত দোক দেসই আান গান আললার কাচেছ দেযন চে যায় আললা দেযন এই আহিতC শচেন পচেতরর ীন হিফহিরচেয় দেয় ৩)ldquoনাহিক এই (রীচেরর পাচেপর ীানচেত দেকানই তরারণ দেনই ভহি4চেতরldquoউততর) হমায়চেনর অসসথতার ন য ার হিনচেচেকই ায়ী কচেরচেছন কারন ার অচেনক রা য অন যায় ভাচে কচেরচেছ তাই তার এই পাপ কাচের ন য তার ঘচের আ হিপ এচেসচেছ এই অন যায় কাচের ন য তার মহিকত দেনই তাই ার আললার কাচেছ এই পাপ কাচেযCর ন য bমা পরাথM

Hindi 2ndlang

-ासी(जयशकर परसा-)

-ासी जयशकर परसा- की एक ऐसी कहानी ह जिजसम भारतीय ससकनित और राषटरीयता का सवरगजीतहोता ह इस कहानी म इरावती एक निहद कनया ह जिजस मलअचछो न मलतान की लट म पकडा और -ासी बना दि-या उस 500 दि-न -कर काशी क एक महाजन न खरी-ा दसरी -ासी निफरोजा ह वह गलाम ह निफरोजा को छडान क कतिलए अहम- को 1000 सोन क कतिसकक भजन थ जो अभी तक नही आए थ राजा साहब कठोर होत हए भी निफरोजा को निबना धनराकतिश क कतिलए उस म कर -त ह वनिफरोजा को अहम- को समझान की बात कहत हकहानी क अत म हम -खत ह निक इरा वती और जाटो क सर-ार बलराज का मिमलन होता हअहम- को यa म मार दि-या जाता ह वहा निफरोजा की परसननता की समामिध बनती ह वहा एक फल चढती ह और डीजल आती ह निफरोजा उस समामिध की आजीवन -ासी बनी रहती हलखक अपन उददशय अथात -ास परथा पर परकाश डालन और इस परथा क कारण होन वाल -ातो क दखो को दि-खान म पणता सफल हए ह

helliphellipContinue to next

Biology Reproductio Today we will discuss about vegetative Q1 Name some vegetative propagules

n in Organisms

propagation of plants The process of multiplication in which fragments of plant body function as propagule and develop into new individual is called vegetative propagation The units of such propagation are runner rhizome tuber bulb etc

and the speciesinvolvedVegetative propagules

Parts involved

Bulb StemBulbil BulbilRhizome Stem Runner Stem Tuber Stem Offset Stem Leaf buds Leaves Suckers Stem

Corns Stem stolon

Q2 State advantages of vegetative propagation

i) Rapid methodii) Sure and easy methodiii) Useful in plants that cannot

produce viable seeds or long seed dormancy

iv) Maintains purity of raceQ 3 Banana fruit is said to be parthenocarpic where as turkey is said to be parthenogenetic WhyBanana develops without fertilization from an unfertilized ovary thus is parthenocarpicIn turkey the ovum or female gamete developinto a new chick without fertilization thus isparthgenetic

Q4 Why is water hyacinth is called as a ldquoTerror of Bengalrdquo Water hyacinth can

propagatevegetatively all over the water body in a short per short period of time This resulted increased biochemicaloxygen oxygen demand of water body causing mortalityof fishes It is very difficult to get rid off them Thus known as terror of Bengal

Chemistry

Solid state GENERAL CHARACTERISTICS OF SOLID STATEIn nature the particular state of matter is governed by two opposing forces at given set of temperature and pressure These forces are intermolecular force of attraction and thermal energy If intermolecular force of attraction is high as compared to thermal energy particles remains in closest position

Intext QuestionsQ1 Classify the following solids as crystalline and amorphous Sodium chloride quartz glass quartz rubber polyvinyl chloride Teflon

A1 Crystalline

and hence very less movement in particles is observed In this case solid state is the preferred state of matter

Let us revise the general characteristics of solid

i) Fixed mass volume and shape

ii) Strong intermolecular force of attraction

iii) Least intermolecular space

iv) Fixed position of constituent particles

v) Incompressible and rigid

Q2 what type of interactions hold the molecules together in a polar molecular solid[CBSE 2010]A2 The molecules in a solid are held together by van der Waals forces The term van der Waals forces include hydrogen bonding dipole-dipole attraction and London dispersion forces All molecules experience London dispersion forces In addition polar molecules can also experience dipole-dipole interactions So the interactions that holds the molecule together in polar molecular solid are London dispersion force and dipole-dipole interactionsQ3 Write a feature that will distinguish a metallic solid from an ionic solid [CBSE 2010]A3 Metals are malleable and ductile whereas ionic solid are hard and brittle Metallic solid has typical metallic lustre But ionic solid looks dullQ4 Write a point of distinction between a metallic solid and an ionic solid other than metallic lustre [CBSE 2012]A4 Metals are malleable and ductile whereas ionic solid are hard and brittleQ5 Write a distinguish feature of metallic solid [CBSE 2010]A5 The force of attraction in

solid Sodium chloride Quartz Amorphous solid Quartz glass rubber polyvinyl chloride Teflon Q2 why glass is considered as super cooled liquidA2 Glass shows the tendency to flow at slower rate like liquid Hence they considered as super cooled liquidQ3 why the window glass of old buildings show milky appearance with timeA3 Glass is an amorphous solid Amorphous solid has the tendency to develop some crystalline character on heating Due to heating in day over the number of years glass acquires some crystalline character and show milky appearanceQ4 why the glass panes fixed to window or doors of old building become slightly thicker at bottomA4 Glass is super cooled liquid It has the tendency to flow down very slowly Due to this glass pane becomes thicker at the bottom over the timeQ5 Sodium chloride is a crystalline solid It shows the same value of refractive index along all the direction TrueFalse Give reasonA5 FalseCrystalline solid shows anisotropy in properties That is it shows different values for the given physical property in different direction All the crystalline solids show anisotropy in refractive index Therefore sodium chloride will show different values of refractive index on different directions

Q6 Crystalline solid are anisotropic in nature What does this statement means

between the constituent particles is special kind of electrostatic attraction That is the attraction of positively charged kernel with sea of delocalized electronsQ6 which group of solid is electrical conductor as well as malleable and ductile [CBSE 2013]A6 Metallic solidQ7 why graphite is good conductor of electricity although it is a network (covalent solid)A7 The exceptional property of graphite is due to its typical structure In graphite each carbon is covalently bonded with 3 atoms in same layer The fourth valence electron of each atom is free to move in between different layersThis free electron makes the graphite a good conductor of electricity

[CBSE 2011]A6 Anisotropy is defined asrdquo Difference in properties when measured along different axis or from different directionsrdquo Crystalline solid show different values of some of the physical properties like electrical resistance refractive index etcwhen measured along the different directions The anisotropy in crystalline solid arises due to the different arrangement of particles in different directions

Math Function Composition of functions Think of an industrial plant that produce bottles of cold drinks first there is the operation (or function) f that puts the cold drink inside the bottle followed by the opeartion g that close the bottle with the capThis leads to the following definitionDefinition Let f A rarr B and g B rarr C be two functions Then the composition of f and g denoted by gof is defined as the function gof A rarr C given by gof(x) = g(f (x)) forall x isinA

Definition A function f X rarr Y is defined to be invertible if there exists a function g Y rarr X such that gof = IX and fog = IY The function g is called the inverse of f and is denoted by f -1

Thus if f is invertible then f must be one-one and onto and conversely if f is one-one and onto then f must be invertible This fact significantly helps for proving a function f to be invertible by showing that f is one-one and onto specially when the actual inverse of f is not to be determined

Example 1 Let f 2 3 4 5 rarr 3 4 5 9 and g 3 4 5 9 rarr 7 11 15 be functions defined as f(2) = 3 f(3) = 4 f(4) = f(5) = 5 and g (3) = g (4) = 7 and g (5) = g (9) = 11 Find gofSolution We have gof(2) = g (f(2)) = g (3) = 7 gof(3) = g (f(3)) = g (4) = 7gof(4) = g (f(4)) = g (5) = 11 and gof(5) = g (5) = 11Example 2 Find gof and fog if f R rarr R and g R rarr R are given by f(x) = cos x and g (x) = 3x2 Show that gof ne fogSolution We have gof(x) = g(f(x))=g(cosx) = 3 (cos x)2

= 3 cos2 x Similarly fog(x)=f(g (x))= f(3x2)= cos (3x2) Note that 3cos2 x ne cos 3x2 for x = 0 Hence gof ne fogExample 3 Show that if f A rarr B and g B rarr C are onto then gof A rarr C is also ontoSolution Given an arbitrary element z isin C there exists a pre-image y of z under g such that g (y) = z since g is onto Further for y isin B there exists an element x in A with f(x) = y since f is onto Therefore gof(x) = g (f(x)) = g (y) = z showing that gof is onto Example 4 Let Y = n2 n isin N sub N Consider f N rarr Y as f(n) = n2 Show that

f is invertible Find the inverse of fSolution An arbitrary element y in Y is of the form n2 for some n isin N This implies that n =radicy This gives a function g Y rarr N defined by g (y) =radicy Nowgof (n) = g (n2)=radicn2 = n and fog (y) =f(radicy) = (radicy) 2 y which shows that gof=IN and fog= IY Hence f is invertible with f -1 = g

Political Science

Constitution of India-The Preamble

Summary

Objective of the state-To secure equality of status and of opportunity To promote fraternity among all the citizens To assure the dignity of the individuals and Unity and integrity of the nation

Justice-Justice stands for rule of law absence of arbitrariness and a system of equal rights freedom and opportunities for all in a society India seeks social economic and political justice to ensure equality to its citizens

Liberty-Liberty implies the absence of restraints or domination on the activities of an individual such as freedom from slavery serfdom imprisonment despotism etc The Preamble provides for the liberty of thought expression belief faith and worship

Equality-Equality means the absence of privileges or discrimination against any section of the society The Preamble provides for equality of status and opportunity to all the people of the country

Fraternity-The Preamble declares that fraternity has to assure two thingsmdashthe dignity of the individual and the unity and

Execution

Answer the following questions-

Short notes-1 Equality2 Fraternity3 Justice4 Liberty

Homework-Learn

integrity of the nation The word integrity has been added to the Preamble by the 42nd Constitutional Amendment (1976)

Business studies

Human resource management (chapter 1)

On the day of 1504 2020 I have discussed with you the managerial functions and procurement functions of HRM

Today weare going to discuss about the development function integration functions and maintenance function

Development functions-HRM improves the knowledge skills attitude and values of employees so that they the present and future jobs more effectively it includes

1) Development functions of HRM

a) Performance appraisal = It implies systematic evaluation of employees with respect to their performance on the job and their potential for development

b) Training =It is the process by which employees learn knowledge skills and attitudes to achieve organisational and personal goals

c) Executive development = It is the process of developing managerial talent through appropriate program

2) Integration functionsa) HRM reconcile the goals of

organisation with those of its members through integrating function

b) HRM tries to motivate employees to various financial and non financial incentives provided in job specification etc

3) Maintenance functiona) HRM promote and protect the

physical and mental health of employees by providing several types of benefits like housing medical aid etc

b) It Promote Social security measures to employees by providing provident fund pension gratuity maternity benefits

SubjectCOMMERCE

Topic

BUSINESSENVIRONMENT

Summary

Now quickly let us revise the earlier points that we have already done in the last class and let us proceed with the other topics that are there in the chapter

Firstly we will recall the internal and external factors of micro environment and then we

Execution 3 What do you mean by internal factors

in micro environmentAnswerInternal factors refer to all the factors existing within a business firm The internal factors are considered controllable because the enterprise has control over these factors

Development FunctionsPerformance AppraisalTrainingExecution Development

shall proceed in details

Meaning and list of internal and external factors

aInternal factorsInternal factors refer to all the factors existing within a business firm The internal factors are considered controllable because the enterprise has control over these factorsFor an example a company can alter its organization structure policies programmes employees physical facilities and marketing mix to suit the changes in the environmentList of internal factors areCorporate culture mission and objectives top management organizations structure company image and brand equity company resources

b External factorsExternal factors refer to those individual and groups and agencies with which a particular business organization comes into direct and frequent contact in the course of its functioningThese individuals and groups are known as STAKEHOLDERS because they have a stake (financial interest ) in the working and performance of the particular business List of external forces (stakeholders)Customers competitors investors suppliersmiddlemen (marketing intermediaries)financers publics

customers

suppliersfinancers

For an example a company can alter its organization structure policies programmes employees physical facilities and marketing mix to suit the changes in the environment

4 What do you mean by external factors in micro environment

AnswerExternal factors refer to those individual and groups and agencies with which a particular business organization comes into direct and frequent contact in the course of its functioningThese individuals and groups are known as STAKEHOLDERS because they have a stake (financial interest) in the working and performance of the particular business

3Who are stakeholdersSTAKEHOLDERS are individuals and groups who have a stake (financial interest ) in the working and performance of the particular business 4Discuss the internal factors in briefa Corporate CultureThe values beliefs and attitudes of the founders and top management of the company exercise a strong influence on what the cmpaany stands for how it does things and what it considers importantbMission and objectivesThe business philosophy and purpose of a comoany guide it prioritiesbusiness strategiesproduct market scope and development scope

cTop management structurethe composition of board of directors the degree of professionalization of management and the organizational structure of a company have important bearing on its business decisions

dPower structureThe internal power relationship between the board of directors and the chief executive is an important factor

e Company image and brand equityThe image and brand equity of the company play a significant role in raising finance forming alliance choosing dealers and suppliers launching new products entering foreign markets

5 What is Macro environmentAnswerMacro environment refers to the general

competitors

middlemen

publics

Fig STAKEHOLDERS OF A COMPANY

Apart from micro environment the other main dimension of business environment isMacro environment Macro environment refers to the general environment or remote environment within which a business firm and forces in its micro environment operateA company does not directly or regularly interact with the micro environmentTherefore macro environment is also known as indirect action EnvironmentThe macro environment forces are less controllable than the micro forces

Macro environment consists of the following components

POLITICAL AND LEGAL ENVIRONMENT

ECONOMIC SOCIAL AND ENVIRONMENT

CULTURAL

ENVIRONMENT

TECHNOLOGICAL ENVIRONMENT

Fig COMPONENTS OF MACRO ENVIRONMENT

environment or remote environment within which a business firm and forces in its micro environment operateA company does not directly or regularly interact with the micro environmentTherefore macro environment is also known as indirect action EnvironmentThe macro environment forces are less controllable than the micro forces 6 What are the components of macro environmenta Political and legal environmentb Economic environmentc Social and cultural environmentd Technological environment

Computer Science

Logic gates

Digital systems are said to be constructed by using logic gates These gates are the AND OR NOT NAND NOR EXOR and EXNOR

BUSINESS FIRM

gates The basic operations are described below with the aid of truth tables

AND gate

The AND gate is an electronic circuit that gives a high output (1) only if all its inputs are high A dot () is used to show the AND operation ie AB Bear in mind that this dot is sometimes omitted ie ABOR gate

The OR gate is an electronic circuit that gives a high output (1) if one or more of its inputs are high A plus (+) is used to show the OR operationNOT gate

The NOT gate is an electronic circuit that produces an inverted version of the input at its output It is also known as an inverter If the input variable is A the inverted output is known as NOT A This is also shown as A or A with a bar over the top as shown at the outputs The diagrams below show two ways that the NAND logic gate can be configured to produce a NOT gate It can also be done using NOR logic gates in the same way

NAND gate

This is a NOT-AND gate which is equal to an AND gate followed by a NOT gate The outputs of all NAND gates are high if any of the inputs are low The symbol is an AND gate with a small circle on the output The small circle represents inversion

NOR gate

This is a NOT-OR gate which is equal to an OR gate followed by a NOT gate The outputs of all NOR gates are low if any of the inputs are highThe symbol is an OR gate with a small circle on the output The small circle represents inversion

EXOR gate

The Exclusive-OR gate is a circuit which will give a high output if either but not both of its two inputs are high An encircled plus sign ( ) is used to show the EOR operation

EXNOR gate

The Exclusive-NOR gate circuit does the opposite to the EOR gate It will give a low output if either but not both of its two inputs are high The symbol is an EXOR gate with a small circle on the output The small circle represents inversion The NAND and NOR gates are called universal functions since with either one the AND and OR functions and NOT can be generated

Note A function in sum of products form can be implemented using NAND gates by replacing all AND and OR gates by NAND gates A function in product of sums form can be implemented using NOR gates by replacing all AND and OR gates by NOR gates

Logic gate symbols

Table 2 is a summary truth table of the inputoutput combinations for the NOT gate together with all possible inputoutput combinations for the other gate functions Also note that a truth table with n inputs has 2n rows You can compare the outputs of different gates

Logic gates representation using the Truth table

Example

A NAND gate can be used as a NOT gate using either of the following wiring configurations

Subject Eng Literature (The Tempest ndash William Shakespeare) Topic Act III Scene 3 Lines 53 to 110 (End of the scene) Date 16th April 2020 (2nd Period)

[Students should read the original play and also the paraphrase given in the school prescribed textbook]Summary Questions amp Answers

o Seeing this strange scene all are inclined to believe the tales told by travelers that there truly are ldquounicornsrdquo and ldquothe phoenixrsquo thronerdquo

o As they are about to sit down to the feast the banquet is snatched away by a harpy (Ariel disguised) A spiritrsquos voice (Arielrsquos voice) denounces Alonso Sebastian and Antonio with particular

1 ARIEL You are three men of sin whom Destiny

(Line 53-58)That hath to instrument this

lower world And what is int the never-surfeited sea

Hath caused to belch up you and on this island

Where man doth not inhabit you rsquomongst men

Being most unfit to live I have made you mad

reference to their crime in expelling Prospero from Milan They have not received any punishment for their deed earlier but the time for their punishment has arrived Upon Alonso it pronounces ldquolingering perdition worse than deathrdquo from which there is no remedy except through sincere repentance Ariel then vanishes in thunder and the shapes enter again and carry away the table

o Prospero watching invisibly is very pleased with the performance of Ariel and his (Prosperorsquos) ldquomeaner ministersrdquo All his enemies are now in his power and are in a fit of desperation He then leaves them and goes to see how Ferdinand and Miranda are getting on

o Alonso is now much humbled and penitent with the after effect of the spiritrsquos denunciation of his crimes He believes that his son is lost forever After this all disperse being stricken mad by the speech of the spirit

o Gonzalo fearing that they may do violence to themselves or to one another follows them and bid others to follow

(a) To whom does Ariel disguised as a harpy call the three sinners What game did Fate of Destiny play with

them

The three sinners called by Ariel are Alonso Sebastian and Antonio It was Destiny which had caused the ocean to cast the three sinners on the shore Though the ocean is all the time devouring whatever appears on its surface and is never satisfied with its continual swallowing of the ships and men in the present case the ocean had cast these three sinners on the shore without killing them

(b) Who had jointly been responsible for the conspiracy against Prospero What is Prosperorsquos purpose behind all this

Three men Alonso Sebastian and Antonio had jointly

been responsible for the conspiracy against Prospero They had driven out Prospero form Milan Prosperorsquos purpose is to make these three sinners realize the wrong they had done He wants them to repent for their criminal deeds because repentance leads to self-esteem(c )What does Ariel (the harpy) tell Alonso and his companions when they take out their swords to attack him

Seeing them drawing their swords Ariel (harpy) tells them that he and his companions are the instruments of destiny and that it is not possible for human beings to do them any injury He says that the swords of human beings can not injure even a minute part of his feathers Their swords are as ineffective against him and his companions as against the wind or the water

(d) Give the explanatory meanings of the following expressions in the context of the above extract

(i)Never surfeited (ii) Belch up (iii) lsquomongst men

(i) Never surfeited never led to satisfaction

(ii) Belch up cast ashore(iii) lsquomongst men in human

society2

I and my fellows (Line 60-65)

Are ministers of Fate The elementsOf whom your swords are tempered may as wellWound the loud winds or with bemocked-at stabsKill the still-closing waters as diminishOne dowl thats in my plume

IMPORTANT PASSAGES EXPLAINED

The elements

(Line 61-66)Of whom your swords are tempered may

as wellWound the loud winds or with

bemocked-at stabs

(a) Who is lsquoIrsquo Who are his lsquofellowsrdquo

lsquoIrsquo is referred to Ariel in disguise of a harpy His lsquofellowsrsquo are other spirits serving Prospero the real Duke of Milan who has acquired supernatural powers after being banished from his Dukedom Prospero has settled in this uninhabited island

(b) What are the elements that have temperrsquod the swords Why will it not work against the speaker

The swords (of Alonso and his companions) are tempered by metal (steel) which is taken out of the earth and refined by

Kill the still-closing waters as diminishOne dowl thats in my plume My fellow

ministersAre like invulnerable

In these words Ariel reminds the King and his companions of the utter futility of drawing swords against himself and his fellows Ariel drives Alonso Antonio and Sebastian the three men of sin to desperation ndash a state in which men do violence to themselves They draw swords to strike Ariel But Ariel reminds them that he and the other spirits are the ministers of destiny and nothing can wound them The steel of which their swords are made of may cut the wind or water which being divided always closes up again Even supposing that such things may be possible it is quite impossible that their swords will cut one feather in their plume They are incapable of being wounded by any sword of man Hence it is foolish on their part to attempt to strike at Ariel and his fellow-spirits

For which foul deed

(Line 72-75)The powers delaying not forgetting

haveIncensed the seas and shores yea all the

creatures Against your peace

Ariel enters like a harpy and remaining invisible tells Alonso Sebastian and Antonio that he and other harpies are the agents of Destiny appointed to carry out her decrees He tells them that their punishment for the crime against Prospero which has been so long deferred is now to fall upon them He reminds them that they had expelled Prospero from Milan and set him and his innocent child adrift on the sea and that the sea had paid them back for their sin by the shipwreck and by the calamities they have suffered He tells them that the powers above which did not forget this mean treachery but only deferred the punishment have now engaged the seas and the shores and all living beings including him and his comrades against them The very elements and supernatural agency Ariel adds have taken up the avenging of their crime against Prospero

the action of fire It may cut the wind or water which being divided always closes up again

The sword will not work against the spirits and the harpy because they are the ministers of destiny and nothing can wound them nor it will cut a single feather in their plume

(c )What is the meaning of lsquodowlrsquo in the last line

The term lsquodowlrsquo means a filament or the smallest part of a feather In this context Ariel in disguise of harpy says that their sword cannot even damage the smallest filament of their (Arielrsquos and other spirits) feathers as they are incapable of being wounded by any sword of man

(d) What does the speaker remind the listeners about

Ariel in disguise of harpy reminds Alonso the King of Naples Sebastian Alonsorsquos brother and Antonio the present Duke of Milan and the treacherous brother of Prospero as they being three men of sin He even reminds them that their punishment for their crime against Prospero which has been so long deferred now falls upon them He reminds them that they have expelled Prospero from Milan and has set him along with his innocent infant daughter adrift on the sea So the sea has paid them back for their sin by their shipwreck and the calamities they have suffered since then The harpy rebukes Alonso of his sin that has incensed the Gods and has deprived him of his son as a punishment

(e) How do they respond

When Ariel in disguise of a harpy reminds Alonso Sebastian and Antonio of their past misdeeds and sin Alonso has a look of terror and confusion in his eyes He utters the words of sincere repentance wrung out of his conscience-stricken heart It appears to him that all the elements of nature the sea-waves the wind and the thunder proclaiming a loud voice in the name of Prospero and the crime Alonso has committed against him They are calling upon him to repent There is a deep storm raging in Alonsorsquos breast and the echoes of that storm are ringing in his ears like a clear note of wind-instrument A note of denunciation of Alonsorsquos crime leaves him much humbled and penitent and confirms his belief that his son is lost forever But Sebastian and Antonio shows some courage instead of repentance They wish to kill the spirits or devils if it appears

3

Of my instruction hast thou nothing bated (Line 85-93)

In what thou hast to say So with good life

And observation strange my meaner ministers

Their several kinds have done My high charms work

And these mine enemies are all knit upIn their distractions They now are in my

powerAnd in these fits I leave them while I visitYoung Ferdinand whom they suppose is

drownedAnd his and mine loved darling

Methought the billows spoke and (Line 96-99)

told me of itThe winds did sing it to me and the

thunderThat deep and dreadful organ-pipe

pronouncedThe name of Prosper It did bass my

trespass

These are the words of contrition coming from Alonso Ariel has driven him to a deep repentance for conspiring with Antonio against Prospero He now feels a sincere remorse It appears to him that all the elements of nature the sea-waves the wind and the thunder proclaimed with a loud voice the name of Prospero and the crime Alonso had committed against him They are calling upon him to repent There is a deep storm raging in Alonsorsquos breast and the echoes of that storm are ringing in his ears like the clear note of a wind-instrument

Comment These are the words of sincere repentance wrung out of the conscience-stricken heart of Alonso Alonso who is the lesser villain is the first to give way to remorse under the effect of Arielrsquos speech The words of Ariel seem to him to be the voice of conscience speaking to him He is driven to desperation a state in which he might do violence to his life

(a) Identify the speaker State the context

Prospero the ruler of the island is the speaker The famous banquet scene has been enacted very well Ariel and his junior spirits have played their roles excellently Prospero is glad to say words of praise for them(b) In what way the speakerrsquos instructions have been carried out

According to Prosperorsquos instructions a banquet was presented before the King of Naples and his companions when they were tired and hungry Just when they were preparing to eat the feast the banquet was suddenly removed by exercising supernatural powers All this was done by Ariel Prosperorsquos chief assistant and a powerful spirit

Ariel not only made the feast disappear but also delivered his speech blaming the King and his two companions for their past wicked deeds He warned them to repent for their misdeeds or suffer forever on that uninhabited island

(c) Who are referred to as lsquomeaner ministersrsquo What have they done

Prospero refers as lsquomeaner ministersrsquo to his other lesser spirits who were assisting Ariel in presenting a scene before the kingrsquos party They entered the scene to the accompaniment of music They assumed several strange shapes and brought in a banquet Then they danced about it with gentle actions of salutations thus inviting the King and others to eat the feast

These spirits play their role again when Ariel in the shape of a harpy quits the scene These shapes enter again and dancing with mocking gestures carry away the table

(d) Who are the speakerrsquos enemies What has happened to them

King of Naples Alonso his brother Sebastian and the present Duke of Milan Antonio (Prosperorsquos own brother) are Prosperorsquos enemies With the turn of events they have all been washed ashore on the island which is ruled by Prospero the great magician Actually this happened after the shipwreck caused by a storm which was raised by Prospero with the purpose of bringing these people to his island Prosperorsquos spirits have already confused and terrified these enemies and they are under Prosperorsquos control He can treat them as he likes

(e) What does he say about Ferdinand Explain what is meant by ldquohellip his and mine darlingrdquo

Prospero knows that Alonsorsquos son prince Ferdinand is alive though his father thinks that the prince has been drowned

Prospero refers to his daughter Miranda who is dear to him She is also very dear to Prince Ferdinand who has fallen in love with her They are waiting to be married soon for which they have received Prosperorsquos consent

4

ALONSO O it is monstrous monstrous (Line 95-102)

Methought the billows spoke and told me of it

The winds did sing it to me and the thunderThat deep and dreadful organ-

pipe pronouncedThe name of Prosper It did bass

my trespassTherefore my son ithrsquo ooze is

bedded andIll seek him deeper than eer

plummet soundedAnd with him there lie mudded

(a) In what way does Alonso express his horror when his conscience is awakened by Arielrsquos words

When Alonsorsquos conscience is awakened by Arielrsquos words he expresses his horror at what he has heard He gets the feeling that the waves of the ocean the wind and the loud thunder have spoken to him and uttered the name of Prospero Because of being reminded of his crime in a very loud and rough voice he comes to realize that he has lost his son for his past misdeeds

(b) What does Alonso imagine about his son What does Alonso want to do in his desperate state

Alonso imagines that his son is lying in the mud at the bottom of the sea He feels desperate that he wants to drown himself in the ocean deeper than the plumb-line has ever gone He wants to lie with his son at the bottom of the sea

(c) How do Sebastian and Antonio want to face the evil spirits

Sebastian says that he is not at all afraid of what the harpy has said and that he is prepared to fight any number of such monsters if they appear before him only one at a time Antonio says that he would support Sebastian in the fight against the fiendsyyy

(d) Why does Gonzalo ask Adrian to follow the three men

Gonzalo tells Adrian that all the three men namely Alonso Sebastian and Antonio are in a wild and reckless mood The thought of the heinous crime of which they are guilty has begun to torment their minds So he asks Adrian to follow those three men without loss of time and prevent them from doing anything which the turmoil in their minds might lead them to do

(e) What opinion do you form of Alonso from the above extract

Alonso who is the lesser villain is the first to give way to remorse under the effect of Arielrsquos speech The words of Ariel seem to him to be the voice of conscience speaking to him He is driven to desperation a state in which he might do violence to his life

Subject =Accounts

Ac-12 15420 topic-pL Appropriation ac

PROFIT AND LOSS APPROPRIATION ACCOUNT

MEANING AND PREPARATIONProfit and Loss Appropriation Account is merely an extension of the Profit and Loss Account of the firm The profit of the firm has to be distributed amongst the partners in their respective profit sharing ratio But before its distribution it needs to be adjusted All Adjustments like partnerrsquos salary partnerrsquos commission interest on capital interest on drawings etc are made in this account These adjustments will reduce the amount of profit for distribution This adjusted profit will be distributed amongst the partners in their profit sharing ratio To prepare it at first the balance of Profit and Loss Account is transferred to this account The journal entries for the preparation of Profit and Loss Appropriation Account are given below

1 for transfer of the balance of Profit and Loss Account to Profit and Loss Appropriation Account

(a) In case of Net Profit

Profit and Loss Ac helliphelliphelliphelliphellipDrTo Profit and Loss Appropriation Ac(Net Profit transferred to Profit and Loss Appropriation Ac)

(b)In case of Net Loss

Profit and Loss Appropriation Achelliphelliphellip DrTo Profit and Loss Ac(Net Loss transferred to Profit and Loss Appropriation Ac)

2 for Interest on Capital

For transferring on Interest on CapitalProfit and Loss Appropriation Achelliphelliphellip DrTo Interest on Capital Ac(Interest on capital transferred to Profit amp Loss Appropriation Ac)

3 for Interest on Drawings

For transferring Interest on Drawings Interest on Drawings Achelliphelliphelliphelliphelliphellip DrTo Profit and Loss Appropriation Ac(Interest on drawing transferred to Profit amp Loss Appropriation Ac)

4 For Partnerrsquos SalaryFor transfer of partnerrsquos SalaryProfit and Loss Appropriation Achelliphellip DrTo Salary Ac(Salary transferred to profit amp Loss Appropriation Ac)

5 For Partnerrsquos CommissionFor transferring commissionProfit and Loss Appropriation Achelliphelliphellip DrTo Commission Ac(Commission transferred to Profit and Loss Appropriation Ac)

6 For Transfer of agreed amount to General ReserveProfit and Loss Appropriation Ac helliphellipDrTo General Reserve Ac(Transfer to General Reserve)

7 for share of Profit or Loss appropriation(a) If ProfitProfit and Loss Appropriation Achelliphellip DrTo Partnerrsquos CapitalCurrent Ac(Profit transferred to capitalcurrent Ac)(b) If LossPartnerrsquos Capital Current Achelliphelliphelliphellip DrTo Profit and Loss Appropriation Ac(Loss transferred to capitalcurrent Ac)

THE FORMAT OF PROFIT AND LOSS APPROPRIATION

Profit and Loss Appropriation Account for the year endedhelliphelliphelliphellip

Particulars Amount Particulars Amount

To PL Ac (loss) By pL Ac (profit)

To Interest on capital BY Interest on drawings

To partner`s commission by Partner`s capital Ac ( loss)

To Partner`s salary To Interest on partner`s loan To General Reserve To Partner`s Capital AC (Profit)

Subject= Economics

MOVEMENT ALONG THE DEMAND CURVE (CHANGE IN QUANTITY DEMANDED)In law of demand you have already studied the inverse relationship between price and quantity demanded When quantity demanded of a commodity changes due to change in its price keeping other factors constant it is called change in quantity demanded It is graphically expressed as a movement along the same demand curve There can be either a downward movement or an upward movement along the same demand curve Upward movement along the same demand curve is called contraction of demand or decrease in quantity demanded and downward movement along the same demand curve is known as expansion of demand or increase in quantity demanded

Extention of demandd

price (rs)p A

B Extentionp1 d

Q Q1

Quantity demanded ( in units)

Contraction of demandd

p2 Ccontraction

p APrice (Rs)

d

Q2 Q

Quantity demanded (in units)

Explanation of movement of demand A fall in price from OP to OP1 leads to increase in quantity demanded from OQ to OQ1 (expansion of demand) resulting in a downward movement from point A to point B along the same demand curve DD When Price rises from OP to OP2 quantity demanded falls from OQ to OQ2 (contraction of demand) leading to an upward movement from point A to point C along the same demand curve DD

  • Activity Series of Metals
    • Drawbacks of Rutherfordrsquos model of atom
      • Electromagnetic radiations
      • Properties of electromagnetic radiations
      • Characteristics of electromagnetic radiations
        • Plancks Quantum Theory-
        • Photoelectric effect
          • Intext Questions
            • Logic gates
            • Digital systems are said to be constructed by using logic gates These gates are the AND OR NOT NAND NOR EXOR and EXNOR gates The basic operations are described below with the aid of truth tables
            • AND gate
            • Example
Page 13:  · Web viewSubject. Topic. Summary. Execution. English 1 . Chapter 1 naming words . Page 8. Write the names of these pictures:- Person:-1. father. 2.Firefighter 3.doctor 4 ...

(iii) 49 and 94 = Unlike Fraction

2) Convert given fractions into fractions with equal denominators(iii) 45 1720 2340 and 1116Solution Given fraction 45 1720 2340 and 1116Therefore the LCM of 5 20 40 and 16 is 80Therefore 45 = 4times165times16 = 64801720 = 17times420times4 = 68802340 = 23times240times2 = 4680 1116 = 11times516times5 = 5580

3) Convert given fractions into fractions with equal numerators(iii) 1519 2528 911 and 4547Solution Given fractions 1519 2528 911 and 4547Therefore the LCM of 15 25 9 and 45 is 2251519 = 15times1519times15 = 225285 2528 = 25times928times9 = 225252911 = 9times2511times25 = 2252754547 = 45times547times5 = 225235

4) Put the given fractions in ascending order by making denominators equal

(iii) 57 38 914 and 2021Solution Given fraction 57 38 914 and 2021Therefore the LCM of the denominators is 16857 = 5times247times24 = 12016838 = 3times218times21 = 63168914 = 9times1214times12 = 1081682021 = 20times821times8 = 160168Therefore ascending order 63168lt108168lt120168lt160168Therefore ascending order of given fractions38lt914lt57lt2021

COMPUTER CHAPTER-1COMPUTER FUNDAMENTALS

DONE IN THE PREVIOUS CLASSES PAGE 10CWRITE TRUE AND FALSE

1 True2 False3 False4 False5 True

GEOGRAPHY CHAPTER 7EUROPE

CHAPTER COMPLETE 1)Europe is home to a famous mountain range called the Alps

2)River Rhine originates in Switzerland

3)The Eiffel Tower one of the tallest structures in Europe

4) Vatican City is one of the most densely populated European countries

5)Sognefjordin Norway is the largest fjord in Norway

Class VIIISubject Topic Summary Execution

MATHEMATICS Ch 6Sets

Exercise 6 (D)1 Given A = x x isin N and 3iquest x le 6 and B = x x isin W and xlt4 find (i) Sets A and B in roster form (ii) A cup B (iii)

A cap B(iv) A ndash B (v) B ndashA

Solution (i) A = 456 and B = 0123

(ii) A cup B = 0123456 (iii) A cap B = ϕ (iv) A ndash B = 456 (v)B ndash A = 0123

3 If A = 56789 B = x 3 lt x lt 8 and x isin W and C = x xle5 and x isin N Find (i) A cup B and (A cup B) cup C (ii) B

cup C and A cup ( B cup C)

(iii) A cap B and (A cap B) cap C (iv) B cap C and A cap (B cap C)

Is (A cup B) cup C = A cup (B cup C)

Is (A cap B) cap C = A cap (B cap C)

SolutionA = 56789 B = 4567 C = 12345

there4 (i) A cap B = 456789 and (A cup B) cup C = 123456789

(ii) B cup C = 1234567 and A cup ( B cup C) = 123456789

(iii) A cap B = 567 and (A cap B) cap C = 5

(iv) B cap C = 45 and A cap (B cap C) = 5

Now (A cup B) cup C = 123456789

And A cup ( B cup C) = 123456789 there4 (A cup B) cup C = A cup (B cup C)

Again (A cap B) cap C = 5 and A cap (B cap C) = 5

there4 (A cap B) cap C = A cap (B cap C)

4 Given A = 012345 B = 02468 and C = 0369 Show that (i) A cup (B cup C) = (A cup B) cup C ie the union

of sets is associative (ii) A cap (B cap C) = (A cap B) cap C ie the intersection of sets is associative

SolutionNow B cup C = 0234689 and A cup B = 01234568

there4 A cup (B cup C) = 012345689 and

(A cup B) cup C = 012345689

So (i) A cup (B cup C) = (A cup B) cup C ie the union of sets is associative

Again B cap C = 06 and A cap B = 024

there4 A cap (B cap C) = 0 and (A cap B) cap C = 0

So (ii) A cap (B cap C) = (A cap B) cap C ie the intersection of sets is associative

Physics Chapter 2 Physical Quatites and Measurements

Here We Will Do Some QuestionsRelated To Chapter 2

A density bottle has a marking 25 mL on it It means that

1 the mass of density bottle is 25g

2 the density bottle will store 25 ml of any liquid in it

3 the density bottle will store 25 ml of water but more volume of liquid denser than water

4 the density bottle will store 25 ml of water but more volume of a liquid lighter than water

Solution 2 the density bottle will store 25 ml of any liquid in it

COMPUTER CHAPTER-2Spreadsheet Functions and Charts

SELECTING RANGE IN ROWSCOLUMNSWHEN TWO OR MORE CELLS ARE SELECTED IT IS CALLED A RANGEA RANGE OF CELLS CAN BE FORMED IN TWO WAYS--a) SELECTING RANGE BY USING THE MOUSEb) SELECTING RANGE BY USING THE KEYBOARD

Q1)WRITE THE STEPS TO SELECT PARTIAL RANGE IN A ROW

Ans)THE STEPS ARE-6 SELECT THE ROW7 BRING THE CELL POINTER TO THE DESIRED

LOCATION FROM WHERE YOU WANT TO START YOUR SELECTION

8 CLICK THE LEFT MOUSE BUTTON AND KEEP DRAGGING TO YOUR RIGHT TILL YOU REACH THE LAST CELL TO NE SELECTED

RELEASE THE MOUSE BUTTON

GEOGRAPHY Asia

CLIMATE

Asia experiences great extremes of climate Jacobabad in the Sind province of Pakistan is one of the hottest places in the WorldVerkhoyansk in Siberia is one of the coldest places in the WorldCherrapunji and Mawsynram in India are two wettest places in WorldArabia Tibet Gobi and Mongolia are extremely dry regionsFactors Affecting Climate of Asia-The factors influencing the climate of Asia are-

Factors Affecting Climate of Asia-Thoroughly read the table in page number 60

Latitudinal extent

Continentality

Relief features

Presence of low pressure trough

Jet streams

English Language The Sentence A complex sentence contains one independent clause and at least one dependent clause The dependent clause in a complex sentence is introduced with subordinating conjunctions or relative pronouns

Commonly Used Subordinating Conjunctions-Time after before while when since untilCause And Effect because now since as in order that soOpposition although though even though whereas while in spite ofCondition if unless only if whether or not even if in case(that)

Commonly Used Relative Pronouns-Who whose whom which whoever whomever whichever that

Class IXSubject Topic Summary Execution

1-BENGALI(2ND LANGUAGE)

ldquo বঙগভমিরপরমিrdquo াইকেলধসদনদতত

আচেগর পর উততর পচো-১ ২ ৩ এং নীচের পর টি াহির কা- ৪মহিbকাও গচেনা দেগা পহিচে অমত হরচে- ক) কার দো দেকান কহিতার অং( ) কতা দেক পরসঙগ কী উহিকতটির তাৎপযC আচোনা কচেরা৫দেসই ধনয নরকচে দোচেক যাচে নাহি ভচে মচেনর মহিeচের সাচেসচে সCন ক) কহির কায C ার উচেf(য হিক হিছ কহি কন কহিতাটি দেচেন) কহি কার কাচেছ হিমনহিত কচেরচেছনগ) কহি এই পহিথীচেত কাচের ধনয মচেন কচেরনঘ) কহি হিক রকম অমর তাাভ করচেত ান

Hindi 2nd lang

काकी(कतिसयारामशरण गपत)

इस कहानी म लखक न यह बतान का परयास निकया ह निक बचच अपनी मा स निकतना परम करत ह शयाम अबोध बालक ह वह अपनी मा क मरन क बा- उसन अपनी मा क कतिलए बहत रोया बा- म उस पता चला निक उसकी मा राम क घर चली गई ह आकाश म उडती हई पतग -खकर उस हरष हआ निक पतग क दवारा वह अपनी मा को नीच उतारगा इसक कतिलए वह अपनी निपता की जब स -ो बार सवा रपया निनकालकर पतग और -ो मोटी सी मन वाली अपन भाई स काकी एक कागज पर कतिलखवा कर पतग म कतिशव का दि-यानिनकालकर पतग और -ो मोटी सी मन वाली अपन भाई स काकी एक कागज पर कतिलखवा कर पतग म कतिचपका दि-याभोला और शयाम कोठरी म रससी बाधनी रह थ तभी उसक निपता करोध म आकर उन स पछ निक कया उनकी जब स रपया निनकाला हभोला डर क मार बताया निक शयाम इस पतग क दवारा अपनी काकी को राम क यहा स उतारना चाहता हनिवशशवर(शयाम क निपता)न फटी पतग उठाकर -खी तो उस पर काकी कतिलखा थावह हत बजिa होकर वही खड रह गएउनहोन सोचा निक मन अपन पतर को मारा जोनिक अनजान और निन-dरष थावह अपनी मा कोनिकतना पयार करता ह

उस दि-न बड सवर शयाम की नी- खली तो -खा निक घर भर म कोहराम मचा हआ ह

क) घर म कोहराम कयो मचा हआ था शयाम को कया लगा

ख) काकी को ल जात समय शयाम न कया उपदरव मचाया

ग) काकी क बार म उस कया बताया गया कया सतय उस कतिछपा रहा

घ) वह बठा-बठा शनय मन स आकाश की ओर कयोकरता

उततरक) शयाम की मा का -हात हो गया था इसकतिलए

घर म कोहराम मचा हआ था शयाम की लगा निक उसकी मा सफ- कपडा ओढ हए भमिम पर सो रही ह

ख) लोग जब उमा यानी शयाम की मा को उठाकर ल जान लग तब शयाम न बडा उपदरव मचाया लोगो क हाथ स झठ करवा उमा क ऊपर जा निगरा और बोला काकी सो रही ह उस कहा ल जा रह हो

ग) काकी क बार म बजिaमान लोगो न उस निवशवास दि-लाया निक उसकी का निक उसक मामा क यहा गई ह लनिकन सतय अमिधक दि-नो तक कतिछपाना रह सका आसपास क अबोध बालको क मह स यह बात परकट हो गई निक उसकी मा का -हात हो गया ह

घ) कई दि-नकई दि-न लगातार रोत-रोत उसका रोना तो शान हो गया पर उसक ह-य म शोक भर गया था वह चपचाप बठा आकाश की और टाका करता निक शाय- उसकी काकी कही दि-ख जाए

ldquoदि-न उसन ऊपर आसमान म पतग उडती -खी न जान कया सोच कर उसका निहर-य एक-म खिखल उठाrdquo

क) निकसन पतग ऊपर उडत -खी और वह कयो खश हआ

ख) उसन अपन निपता स कया कहा उनका कया उतर थाश

ग) उसन निफर कया निकया और निकसन उसकी सहायता की

घ) उसकी योजना कया थी उततर -क) शयाम न एक दि-न आसमान म पतग उडती

-खी तो उसन सोचा निक पता आसमान म राम क यहा जाकर रकगी वही पर मरी काकी ह यह सोचकर वह बहत खश हआ

ख) उसन अपन निपता स कहा काका मझ एक पतग मगा -ो उसक निपता न भटक हए मन क भाव स कहा निक मगा -ग यह कह कर उ-ास भाव स वह कही और चल गए पतग नही आई

ग) उसन चपचाप निवशशवर क टगहए कोट स एक चवननी निनकाल ली और सखिखया -ासी क लडक भोला की सहायता स एक पतग मगवानी भोला उसकी बराबर उमर का ही था

घ) उसकी योजना यह थी निक वह अपनी पतग को आकाश म राम क यहा भजगा और उस पतग क सहार उसकी काफी नीच उतर जाएगी इस योजना पर उस परा निवशवास था इसकतिलए वह और भोला -ोनो यह काम करन म लग गए

Continue to nexthelliphellipEVS CHAPTER - 1

(UNDERSTANDING OUR ENVIRONMENT)

Sustainable development

The development that meets the needs of the present without compromising the ability of future generations to meet their own needs is called Sustainable development

Sustainable societies ndash

An environmentally sustainable community is one that meets the current and future basic resource needs of its people in a just and equitable manner without compromising the ability of future generations to meet their basic needs

Q ) What are Eco Villages

Ans - Eco village are the urban or rural communities of people who strive to integrate a supportive social environment with a low impact way of life

Q ) To ensure sustainable development the depletion of renewable resources should not take place at a rate faster than their regeneration Justify your answer

Ans ndash Renewable resources do not have a fixed quantity - more can always be

generated However if the rate of use exceeds the rate of renewal - that is the

source is used more than its being recreated - its continued use will become

used up faster than it can regenerate

To promote sustainable society the following things need to be done ndash

1 Using renewable energy sources 2 By improving the quality of human

health 3 By promoting sustainable agriculture 4 By forming ecovillage

it will eventually be entirely depleted So Toensure sustainable development the depletion of

renewable resources should nottake place at a rate faster than their regeneration

Q ) What do you mean by Sustainable societies

Ans - Sustainable societies are defined as towns and cities that have taken steps to remain healthy over the long term These communities value healthy ecosystems use resources efficiently and actively seek to retain and enhance a locally based economy Sustainable development concerns everybody in a society

Q ) What are the effects of pollution on human health

Ans ndash Some health problem occurs due to air pollution are ndash

Respiratory diseases Cardiovascular damage Fatigue headaches and anxiety Irritation of the eyes nose and throat Damage to reproductive organs Harm to the liver spleen and blood Nervous system damage

Some health problem occurs due to water pollution are ndash

Typhoid Cholera Dysentry Jaundice

Some health problem occurs due to noise pollution are ndash

Fatigue headaches and anxiety High blood pressure Hearing damage

Physics Motion in 1D First go through previous notes Now here we will solve some numerical related to that

Question 3What information about the motion of a body is obtained from the displacement-time graphSolution 3From displacement-time graph the nature of motion (or state of rest) can be understood The slope of this graph gives the value of velocity of the body at any instant of time using which the velocity-time graph can also be drawn

Question 4(a)What does the slope of a displacement-time graph represent(b)Can displacement-time sketch be parallel to the displacement axis Give a reason to your answerSolution 4(a) Slope of a displacement-time graph represents velocity(b) The displacement-time graph can never be parallel to the displacement axis because such a line would mean that the distance covered by the body in a certain direction increases without any

increase in time which is not possible

Chemistry Language of Chemistry

How to balance a chemical equationThere are two methods of balancing an equation(i)Hit and trial method(ii)Partial equation methodBalancing by hit and trial methodThis method consists of counting the number of atoms of each elements on both sides and trying to equalize themTake the following steps(i)Count the number of times (frequency) an element occurs on either side(ii)The element with the least frequency of occurrence is balanced first(iii)When two or more elements have the same frequencythe metallic element is balanced firstExample-1 On heatinglead nitrate decomposes to give lead dioxidenitrogen dioxide and oxygenPb(NO3)2rarrPbO+NO2+O2

In this equationLead occurs twiceNitrogen occurs twiceOxygen occurs four timesSince lead is a metalbalance it firstThe number of atom of lead is equal on the two sidestherefore it needs no balancingNow balance nitrogenOn the reactant sidethere are two atoms of nitrogenwhile on the product side oneSomultiply the product containing nitrogenon the product sideby two Pb (NO3)2rarrPbO+2NO2+O2Nowthe number of oxygen atoms on the reactant side 6while on the product sideit is 7Somultiply the entire equation by 2except oxygen to get balanced equation2Pb(NO3)2rarr2PbO+4NO2+O2Multiplication by 2 is done only when atoms of all the elements except one element are balanced and the unbalanced atom occurs separately at least once and also there is a difference of only one such atom

Math Topic AlgebraChapter

Factorisation

Study item Difference of two squares a2 ndash b2 = (a+b) (a-b)1) (i) 4x2ndash 25y2

= (2x) 2 ndash (5y) 2= (2x + 5y) (2x - 5y)

(ii) 9x2 ndash 1= (3x)2ndash(1)2= (3x + 1)(3x ndash 1)

2) (i) 150 ndash 6a2= 6(25 ndash a2)= 6(5)2 ndash(a)2= 6 (5 + a) (5 ndash a)

(ii) 32x2 ndash 18y2=2(16x2 ndash 9y2)=2(4x)2 ndash (3y)2= 2(4x + 3y)(4x - 3y)3)(i) (x ndashy )2 ndash 9 = (x ndash y )2 ndash (3)2= (x ndash y + 3) (x ndash y ndash 3)(ii) 9(x + y) 2ndash x 2= (3)2(x + y)2 ndash (x)2=3(x + y)2 ndash (x)2= (3x +3y ) 2ndash(x)2= (3x + 3y + x)(3x +3y ndash x)= (4x + 3y) ( 2x + 3y )

Commercial studies

Basic accounting terms

Today I will give you some questions from the previous study material

Questions1) Define accounting2) What do you mean by debit and

credit

3) Explain the types of account4) Define the following terms

a) Assetsb) Capitalc) Purchased) Debtorse) Transactions

5) Name the types of accounts given below

a) Krishnas accountb) Machinery accountc) Royalty accountd) Salary accounte) Furniture accountf) Audit fee account

Economics Revision Today I will give you some revision questions

Questions1) What do you mean by the terms

rdquowantsrdquo2) Write the difference between

consumer goods and producer goods

3) Define the term utility 4) Explain the different types of utility5) Define

a) Total utilityb) Marginal utility

Subject Eng Literature (The Merchant of Venice ndash William Shakespeare)Topic Act I Scene 3 Lines 1 to 48 (Shylock hellip Cursed be my tribe if I forgive him) Date 16th April 2020 (5th Period)

[Students should read the original play and also the paraphrase given in the school prescribed textbook]Summary Questions amp Answers

This scene takes place in Venice and we are introduced to the rich Jew Shylock Bassanio and Shylock are talking and Bassanio tells Shylock that he wants a loan of three thousand ducats for three months on the personal security of Antonio

o Shylock feels glad because he will be able to bind down Antonio by means of a bond on account of the loan but he tells Bassanio that all the fortunes of Antonio being invested in the merchant ships on the sea it is difficult to depend upon his credit Even under such circumstances Shylock is willing to advance the money on the personal security of Antonio

o Bassanio then invites Shylock to dine with him Shylock says that he is prepared to do anything with the Christians but not eat or drink or pray with them

o While Bassanio and Shylock are talking Antonio appears on the scene Shylock does not seem to take any notice of Antonio but goes on brooding within

(1) SHYLOCK Ho no no no no- my meaning in (Line 15-26)saying he is a good man is to have you understand me that he is sufficient Yet his means are in suppositionhe hath an argosy bound to Tripolis another to the Indies I understand moreover upon the Rialto he hath a third at Mexico a fourth for England and other ventures he hath squanderd abroad Butships are but boards sailors but men there be land-rats and water-rats land-thieves and water-thieves I mean pirates and then there is the peril of waters winds and rocks The man is notwithstanding sufficientmdashthree thousand ducats mdashI think I may take his bond

(a) Who is talking in the beginning of this scene What does Bassanio want from Shylock How does Shylock feel

In the beginning of the scene Bassanio and Shylock are talking to each other Bassanio wants to borrow three thousand ducats from Shylock for three months on the security of Antonio Shylock feels glad at heart that he will get the opportunity of binding Antonio with a bond(b) What risks does Shylock weigh in advancing the money

Shylock says that Antonio has invested all his capital in trading by sea-going ships But the ships are made of wood and the sailors of those ships are ordinary human beings The wood can

himself how he hates Antonio because of his being a Christian because he abuses Shylock in public places Shylock decides that if ever he can get Antonio to his advantage he will teach him a lesson

come to harm and men can commit mistakes and thus the capital invested in ships may be lost Then there are other dangers The goods loaded on the ships can be damaged by rats and thieves which are found both on land and water The ships can also be harmed through sea-storms submerged rocks etc(c) What two important functions does this scene have

The scene has two important functions First it completes the exposition of the two major plot lines of the play Antonio agrees to Shylockrsquos bond ndash three thousand ducats for a pound of flesh and second and more important dramatically this scene introduces Shylock himself In this scene Shakespeare makes it clear at once why Shylock is the most powerful dramatic figure in the play and why so many great actors have regarded this part as one of the most rewarding roles in all Shakespearean dramas(d) Where does this scene take place What kind of treatment has Antonio been giving to Shylock What does Shylock say when Bassanio invites him to dine with him

The action of this scene takes place in Venice Antonio has been in the habit of behaving harshly with Shylock ndash spitting on his beard and footing him like a stranger cur When Bassanio invites Shylock to dine Shylock says that he is prepared to do anything with the Christians but not eat and drink or pray with them

(2) SHYLOCK How like a fawning publican he looks (Line 38-48)I hate him for he is a Christian

But more for that in low simplicity

He lends out money gratis and brings downThe rate of usance here with us in VeniceIf I can catch him once upon the hipI will feed fat the ancient grudge I bear him

He hates our sacred nation and he railsEven there where merchants most do congregateOn me my bargains and my well-won thriftWhich he calls interest Cursed be my tribeIf I forgive him

(a) What is the context in which these words are spoken and what is the idea expressed in it

These remarks are made by Shylock when he sees Antonio coming along after Bassanio told him that the merchant will be his surety for the bond The above mentioned passage reveals Shylockrsquos hatred for Antonio Shylock says that he hates Antonio because he is a Christian and also because he gives loan without taking interest on them thereby bringing down the rate of interest in Venice(b) Explain the meaning of the phrase lsquoa fawning publicanrsquo

The phrase lsquoa fawning publicanrsquo refers to Roman tax collector It is a term of contempt and hatred on the lips of a Jew lsquoFawning Publicansrsquo were Roman tax-gatherers whose ordinary bearings towards the Jews was bullying but whose false pose of lsquohumility and contritionrsquo is touched upon in the parable in New Testament(c ) What light does the above passage throw on the character

of Shylock

The above mentioned speech of Shylock reveals him to be a wicked character having an extreme greed for wealth His intense hatred for Antonio is unjustified He hates Antonio only because he is a Christian and because he lends money without taking any interest on it thereby adversely affecting Shylockrsquos business of lending money on high interest(d) What information do you gather about Antonio from the above given lines

Shylockrsquos statement throws a valuable light on the character of Antonio Antonio appears to be a good Christian and a good human being He helps the people in need by lending them money without charging any interest on it He is a man of simple and good nature This very goodness makes him Shylockrsquos enemy(e) What does Shylock debate within himself and when To whom are the lines mentioned above addressed to

When Bassanio asks the Jew to lend him three thousand ducats on Antoniorsquos surety Shylock begins to debate within himself as to how he should exploit the opportunity of a business deal with his old enemy Antonio

The lines mentioned above are not addressed to anyone The lines are a soliloquy ie a speech made by a character to himself and not meant to be heard by the other characters present

Class XSubject Topic Summary ExecutionEnglish

LiteratureThe Blue Bead 2nd part

Things took a turn for the worst and all of a sudden a crocodile attacked the woman biting on the womanrsquos leg At that moment Sibia got up sprinted grabbed the hay fork and stabbed the crocodile in the eye with all her power Immediately the crocodile let go and went away Sibia saw a small blue bead lying by the river she grabbed it Since she was poor she didnrsquot have necklace Shersquod always wanted one like the other women now she could make one with the blue bead After that she went home and told her mother all about it

Hindi 2nd

Langबड घर की बटी( मशी परमच-)

lsquoबड घर की बटी कहानी का उददशय मधयम वग की घरल समसया को सलझा कर सगदिठत परिरवार म मिमल जलकर परम स रहन का स-श -ना हघर म शानित सथानिपत करन की जिजमम-ारी नारी की होती ह यदि- नारी समझ-ार ह उसम धय और परिरवार क परनित परम ह तो कोई भी घटना परिरवार को निवघदिटत नही कर सकती या कहानी परिरवार को सगदिठत करत हए परम सौहा- स एक दसर की भावनाओ को समझ करउनका सहयोग करत हए जीवन यापन करन की पररणा -ती हमशी परमचदर जी न इस कहानी म सय परिरवार का परनितनिनमिधतव निकया ह यह कहानी बनी माधव सिसह जो गौरी पर क जमी-ार क उनक -ो पतरो की हशरी कठ लाल निबहारीशरीकात का निववाह एकजमी-ार घरान की पतरी आन-ी स हआ थाआन-ी न ख- को ससराल क वातावरण म ढाकतिलया थाएक दि-न आन-ी का अपन -वर लाल निबहारी स झगडा हो जाता ह -ोनो भाई एक दसर स अलग होन की कोकतिशश करत हसभी बह आन-ी न अपन मधर वयवहार स लाल निबहारी को

ldquoइन नतर निपरय गणो को बीए-इनही -ो अकषर पर नयोछावर कर दि-या था इन -ो अकषर न उनक शरीर को निनबल और चहर को कानित ही बना दि-या थाldquo

क) परसतत पकतियो म निकस वयकति क बार म कहा गया ह

ख) इन पकतियो म कौन स नतर निपरय गणो क बार म कहा गया ह

ग) बीए की निडगरी परापत कर लन पर भी उपय वयकति क सवभाव की कया निवशरषता थी

घ) यह नतर निपरय गण निकस वयकति म निवदयमान थ उसक वयकतितव की कया निवशरषता थी

उततर ndashक) परसतत पकति म गौरी पर गाव क जमी-ार

क बड बट शरीकात क बार म कहा गया ह उसन बहत परिरशरम और उ-म क बा- ba की निडगरी परापत की थी अब वह एक -फतर

घर छोडकर जान स रोक कतिलयाइस पर बनी माधव सिसह न कहा निक बड घर की बटी ऐसी ही होती ह जो निबगडा काम बना लती ह अतः शीरषक साथक ह बड घर की बटी आन-ी ह

म कमचारी थाख) भरा हआ चहरा चौडी छाती और डटकर

खाना आदि- एक सबजी ल जवान क गण मान जात ह परत शरीकात न इनही नतर निपरय गणो को अपनी पढाई पर नयोछावर कर दि-या था

ग) बीए की निडगरी परापत कर लन पर भी उपय वयकति(शरी कठ की शारिररिरक तौर पर निनबल और चहर स कानित ही लगत थ इतना ही नही वह मानकतिसक तौर पर भी निपछड हए थ पाशचातय सामाजिजक कथा उस घणा एव पराचीन सभयता का गणगान उनकी निवचारधारा क परमख अग थ

घ) यह नतर निपरय गण गौरीपर गाव क जमी-ार क छोट बट लाल निबहारी सिसह म निवदयमान थ वह सजीलाजवान था और भस का दध शर दध वह सवर उठकर पी जाता था

ldquoयही कारण था निक गाव की लललन आए उनकी निन-क थी कोई कोई तो उह अपना शतर समझन म भी सकोच ना करती थी सवय उनकी पतनी को इस निवरषय म उनस निवरोध थाldquo

क) उपय पकति म इस वयकति क बार म कहा गया ह

ख) गाव की लललन आए उनकी निन-ा कयो निकया करती थी

ग) उनकी पतनी का कया नाम था उनह निकस निवरषय म अपन पनित क निवरa था और कयो

घ) इस कहानी का कया उददशय ह Continue to next helliphelliphellip

Bengali 2nd Language

ফ ফটক না ফটক( কহিতা )

পর) ldquo(ান াধাচেনা ফটপাচেথ পাথচের পাড হিচেয় এক কাঠচোটটা গাছ কহিকহি পাতায় পার ফাটিচেয় াসচেছldquoক) কার দো দেকান কহিতার অং( ) lsquo(ান াধাচেনা ফটপাচেথ পাথচের পাডহিচেয়lsquo চেত কী দোঝাচেনা চেয়চেছ গ) আচো য অংচে(lsquo এক কাঠচোটটা গাছ lsquoচেত কী দোঝাচেনা চেয়চেছ ঘ) ldquoকহিকহি পাতায় পার ফাটিচেয় াসচেছldquo ----- একথার পরকত অথC কী উততর ) ক) আচো য অং(টি পর যাত কহি সভা4 মচোপাধ যাচেয়র দো lsquoফ ফটক না ফটকrsquo কহিতার অং()কহি সভা4 মচোপাধ যায় হিছচেন দেপরচেমর কহি দেপরমচেক নানা ভহিঙগমায় হিতহিন ফটিচেয় তচেচেছন দেপরম মানচের স মচেতC র সঙগী কহিতার কহিতায় এক রb সb হচেয়র দেপরম াগরচেনর কথা চেচেছন (ান অথCাৎ দেযাচেন দেকান রস দেনই দেযাচেন দেকান মহিনতা দেনই অথ তার মধ দেযও দেপরম থাকচেত পাচের একথাই কহি তচে ধরচেত দেচেয়চেছন একটি মানচে4র মচেন দেযাচেন দেকামতার দেকান সথান দেনই পাথচেরর মচেতা হিনরসতার মচেনর মধ দেযও দেয দেপরম আসচেত পাচের দেস কথাই কহি চেচেছনগ)নারীচের যথC দেপরচেমর ছহি এই কহিতায় অকপচেট উচেঠ এচেসচেছ কহি এই কহিতায় কাটচোটটা গাছ কথাটি যার কচেরচেছন নারী দেয দেপরম দেথচেক হিতাহিত এং দেসই দেপরম সঠিক সমচেয় না পাওয়ার ন য দেপরম সমপচেকC হিচেr4 গৈতরী য় দেপরচেমর দেয গৈহি(ষট য মাধযC য সরসতা দেকামত এই সমসতর হিপরীত যথা রbতা শষকতা কচেঠার তা পরভহিত দোঝাচেত এক কাঠচোটটা গাছ কথাটি যার কচেরচেছনঘ) এাচেন এক নারীর যথC দেপরচেমর কথা হিনহিCপত ভাচে চেচেছন কহি অসমচেয় নারীর ীচেন দেপরম দেচেগচেছ এতহিন তার হয় রb কচেঠার হিছ দেপরচেমর অভাচে ঠাৎ দেসই শষক মরভহিমচেত সচের আভাস এচেসচেছ দেপরম দেযন 4Cার স(ীত তাই পরায় মত গাচেছ কহিকহি পাতা গহিচেয় উচেঠচেছ

Biology Chapter - 01Controlling Air Pollution

Today we will discuss how we control air pollution from domestic combustion

Q1Describe any five ways of reducing air pollution from domestic sources bull The number of pollutants in the air is verylarge and we always try to control them byfollowing ways

i) Solar cooker and solar heater It use no fuel reduce damage of environment by fuel use or reducing deforestation It maintains coolness of house It releases very less orno oil gas or grease

ii) Piped natural gas (PNG) It emits very less by products into the atmosphere As it isdistributed through pipe lines so there iscontinuous supply of fuel is possible

iii) Liquefied Petroleum Gas (LPG) It hasa higher heating value LPG doesntcontain sulphur so it burns a lot cleanerenergy sources It releases very less oralmost no fume in air

iv) Electricity based cooking Emission free cooking alternative for urban dwellers causeselimination of adverse health impactsofindoor air pollution It helps to avoid theinconveniences associated with procurement of LPG

v) Biogas It contains 75 methane whichmakes it an excellent fuel It burns without smoke and biogas plant leaves no residue like ash in wood charcoal etc Thus it isaclean fuel

Economics

Factors of Production

Today firstly we would recall the last class for 5 mins and then we would proceed with the further topics of the chapter

The concept meaning of land characteristics of land and importance of land to be repeated for the absentees as well as the students who were there in the class the previous day

Today we will start with the last portion of land before it the meaning of land to be repeated onceAs by now we all know that

Questions1What do you mean by productivity of landAnswer By productivity of land we mean the capacity of a piece of land to produce a crop

Thus it refers to the average output per unit of landSay per acre per hectare etc= (OutputArea of land)

2 What are the factors influencing the productivity of landAnswer

Natural factors Productivity of land is largely determined by the natural

Land is defined to include not only the surface of the earth but also all other free gifts of nature(for example mineral resources forest resources and indeed anything that helps us to carry out the production of goods and services but is provided by nature free of cost)

We will move on to the last portion of land by discussing Productivity of Land

By productivity of land we mean the capacity of a piece of land to produce a crop

Thus it refers to the average output per unit of land

Say per acre per hectare etc= (OutputArea of land)

With this we shall proceed further with the main factors that determine the productivity of land

Natural factors Human factors Improvements on land Location of land Organisation Ownership of land Availability of capital Proper use of land State help

Note economic development of a country depends upon the quality of its land If the land is fertile it will quicken the pace of development of the country

qualities of land such as fertility etc

Human factors Land cannot produce anything by itself Man has to apply labour on it to produce for himself So productivity of land depends on the knowledge and skills of workers

Improvements on land production of land is affected by land development measures like provision of well or tubewell irrigation proper drainage

State help The government of a country especially less developed country can play a vital role in improving the agricultural productivity by providing better irrigation facilities

Organisation Productivity of land also fdepends upon the way how the factors of production like labour and capital are organised

In order to increase productivity trained workers modern implements scientific methods good seeds are all essential

3 lsquoImproved technology affects the productivity of landrsquo Explain this statement with the help of suitable example Answer Use of improved technology raises the productivity of land Example By using HYV seeds chemical manures and modern machines per hectare output increases

Physics Force (Summary)

Question Write the expression for the moment of force about a given axisSolutionsThe expression for the moment of force is given byMoment of force about a given axis = Force times perpendicular distance of force from the axis of rotationQuestion What do you understand by the clockwise and anticlockwise moment of force When is it taken positiveSolutionsIf the effect on the body is to turn it anticlockwise moment of force is called the anticlockwise moment and it is taken as positive while if the effect on the

body is to turn it clockwise moment of force is called the clockwise moment and it is taken as negative

Math Topic Commercial Mathematics

Chapter Goods and services Tax

Study item Some solved sums from exercise ndash 1 A retailer buys a TV from a wholesaler for Rs 40000 He marks the price of the TV 15 above his cost price sells it to the consumer at 5 discount on the marked price If the sales are intra ndash state and the rate of GST is 12 find

(i) The marked price of the TV(ii) The amount which the consumer pays for the TV(iii) The amount of tax (under GST) paid by the retailer to the central

Government(iv) The amount of tax (under GST) received by the State Government

Solution As the sales are intra- state sale and the rate of GST 12 So GST comprises of 6 CGST and 6 SGSTTherefore a retailer buys a TV from a wholesaler for Rs 40000Therefore the amount of GST collected wholesaler from the retailer or paid by retailer to wholesalerCGST = 6 of Rs 40000 = Rs(6100 times40000) =Rs 2400SGST = 6 of Rs 40000 = Rs (6100 times 40000) =Rs 2400Therefore wholesaler will pay Rs 2400 as CGST and Rs 2400 as SGSTTherefore amount of input GST of retailer Input CGST = Rs 2400 and input SGST = Rs 2400Again the retailer marks the price of the TV 15 above his cost price(i) The marked price of the TV

= Rs 40000 + Rs 40000times15= Rs 40000 + Rs 40000times 15100= Rs 40000 + Rs 6000Rs 46000But the retailer sells it to consumer at 5 discount on the marked priceCost price after discount = Rs 46000 ndashRs46000times 5100 =Rs 46000 ndashRs 2300= Rs 43700Therefore the amount of GST collected retailer from consumer or paid by consumer to retailerCGST = 6 of Rs 43700 =Rs ( 6100 times43700)Rs 2622SGST = 6 of Rs 43700 = Rs (6100 times 43700) =Rs 2622Amount of the output GST of retailer Output CGST = Rs 2622 and output SGST = Rs 2622

(ii) The amount which the consumer pays for the TV= cost price of TV to consumer + CGST paid by consumer + SGST paid by consumer= Rs 43700 + Rs 2622 + Rs 2622= Rs 48944

(iii) The amount of tax (under GST ) paid by the retailer to the central Government=CGST paid by retailer = output CGST ndash input CGST=Rs 2622 ndash Rs 2400=Rs 222

(iv) The amount of tax ( under GST ) received by the State Government = SGST paid by wholesaler + SGST paid by retailer= Rs 2400 + output SGST ndash input SGST=Rs 2400 + Rs 2622 ndash Rs 2400=Rs 2400 + Rs 222= Rs 2622

Commercial studies

Stakeholders Today I am going to give some revision questions from the previous study material

Questions1) State the two expectations of

employees from a business concern2) Give two distinctions between

stakeholder and shareholder3) Give two difference between

internal stakeholders and external stakeholders

4) Give two expectations of suppliers from a business organisation

5) Who is a stakeholder in commercial organisations

Chemistry Periodic Table

Merits of Mendeleevrsquos Periodic law are as follows - 1He grouped the elements on the basis of atomic mass 2 He left gaps for undiscovered elements like Gallium Scandium germanium Also he left a full group vacant for undiscovered inert gases 3 He could predict proportions of several elements on basis of their position in periodic table like Ga Sc etc 4He could predict errors in atomic weights of some elements like gold platinum etc

Anomalies in Mendeleevrsquos Periodic law are as follows - 1 Position of isotopes could not be explained 2 Wrong order of atomic masses could not be explained

For example- as Arnur atomic mass 40 come first and K with low atomic mass (30) should come later but k should be placed first

According to Bohrrsquos Modern Periodic table properties of elements are periodic functions of their atomic numbers

So when elements are arranged according to increasing atomic numbers there is periodicity in electronic configuration that leads to periodicity in their chemical properties

It consists of horizontal rows (Periods) Vertical column (Groups)

There are 7 period and 12 groups in this long form of periodic table

Ist period has 2 elements IInd period has 8 elements IIIrd period has 8 elements IVth period has 18 elements Vth period has 18 elements VIth period has 32 elements VIIth period hs rest of elements

Note - The number of valence electrons in atom of elements decides which elements will be first in period and which will be last

In group- 1 to 2 gp and 13 to 17 contain normal elements 3 to 12gp ndash transition elements 57 to 71 - lanthanides 89 to 103 - Actinides

Left hand side ndash metals Right hand side ndash nonmetals

Note- Hydrogen element has been placed at top of Ist group Electronic configuration of H is similar to alkali metal as both have 1 valence electron

V electron of gp I element -- 1 V electron of gp 2 element -- 2 V electron of gp 13 element -- 3 V electron of gp 14 element -- 4 V electron of gp 15 element -- 5 V electron of gp 16 element --6 V electron of gp 17 element -- 7 V electron of gp 18 element -- 8

English 1 Transformation of sentences

Sentences A sentence is a group of words which makes complete sense

Exercise 2Change the following sentences from

a Assertive sentencesb Imperative sentencesc Interrogative sentencesd Exclamatory sentences

Sentences can be changed from one grammatical form to another without changing the meaning of the sentence This is known as transformation of sentences

assertive to interrogative1 Nobody would like to be a fool

Who would like to be a fool2 Their glory can never fade

When can the glory fade3 Nobody can control the wind

Who can control the wind4 It matters little if I die

What though I die5 No man can serve two masters

Can any man serve two masters

Exercise 3Interchange of assertive and Exclamatory sentences

1 She leads the most unhappy lifeWhat an unhappy life she leads

2 This is indeed an interesting bookWhat an interesting book this

3 He is a very great manWhat a great man he is

4 It is a very lame excuseWhat a lame excuse

5 It is sad that she died so youngAlas she died so young

Class XISubject Topic Summary Execution

Hindi 2nd lang

पतर परम(परमचदर) पतर परम कहानी म एक निपता की इचछाओ का वणन निकया गया ह अपन बड पतर परभ -ास स निपता चतनय -ास का निवशरष परम था निपता को उसक जनम स ही बडी-बडी आशाए थी उसम दसर बट कतिशव-ास की अपकषा स- उतसाह की मातरा अमिधक थी वह उस इगलड भजकर बरिरसटर बनाना चाहत थभागय का खल भी बडा निनराला ह बीए की परीकषा क बा- वह बीमार पड गया डॉकटरो न भी जवाब - दि-या थाचतन -ास जी बहत ही कजस थ बवजह पस खच करना नही चाहत थ अगर गारटी मिमलती तो शाय- पस खच भी कर -त परत गारटी नही थी परिरणाम सवरप उनक बट का -हात हो गयाजब बट को समशान ल जा रह थ तो वहा काफी शोर गान बजान हो रह थ पछन पर पता चला निक निकसी निपता निपछल तीन साल स निबमार था और उसक ईलाज म रपया पानी की तरह बहाया पर ठीक नही हए परत उसक बट को तनिनक भी अफसोस नही था उसका कहना था उसन कोकतिशश तो कीयह -खकर चतनय-ास जी को आतम निगलानी हईतभी स उनका म परिरवतन हआ और बट का भोज काफी धमधाम स निकयाऔर वहइस पशचाताप की आग म जलत रह औला- स बढकर पसा नही होता ह इस बात को समझन म उनह काफी व लग गया

hellipContinue to next

BENGALI(2ND LANGUAGE)

পরথমঅধযায়-ঠাকরারীনদরনাথঠাকর

নয়ন দোচের হিমাচেররা া নাচেমই হিযাত হিছচেন ায়ানার উাররণ সবরপ নয়ন দোচের ারা হিা (াচেকর হিা হিচেতন এছাাও দেকান উৎস উপচেb রাহিতর দেক হিন করার উচেfচে(য তারা সযC হিকরচেরণ রনয পরীপ জবাহিচেয় তাচেত রপার হির 4Cরণ করচেতন ঠাকরা এই নয়ন দো হিমারচের দে(4 ং(ধর হিছচেন হিমাররা ায়ানার ষটানত পর(Cন কচের তারা হিনঃসব এই হিমাহিরর দে(4 ং(ধর গৈকাস নদর রায়চেৌধরী গৈকাস া নয়ন দোচের সমসত সমপহিতত ঋচেরণর াচেয় হিহিx কচের অহি(ষট যা আচেছ তাচেত হিপত

ইার হিপতার মতয ইচে পর নয়নচোচের ায়ানার দেগাটা কতক অসাধাররণ শরাদধ (াহিনতচেত অহিনতম ীহিপত পরকা( কহিরয়া ঠাৎ হিনহিয়া দেগ- ক) কার দো দেকান গচেলপর অং() কতা দেক ইার চেত কাচেক দোঝাচেনা চেয়চেছ গ) পরসঙগ কী কতার কতয পহিরসফট কচেরা

পরচে4র যাহিত রbা করা সমভ নয় তাই হিতহিন পতরচেক হিনচেয় ককাতায় সাস শর কচেরন গলপ কথচেকর আহিথCক অসথা নয়ন দোচের হিমাচের দেথচেক সমপরণC আাা কথচেকর হিপতা হিনচের দেষটায় অথC উপাCন করচেতন া উপাহিধ াচেভর নয তার াসা হিছনা আর দেসই কারচেরণ কথক তার একমাতর উততরাহিধকার চেয় তার হিপতার পরহিত কতজঞ কথক দো পা হি(চেচেছন হিনচের পরারণ ও মান রbার নয উপচেযাগী অথC হিনা দেষটায় পরাপত চেয়চেছন- এটাই তার কাচেছ পরম দেগৌরচের হি4য় চে মচেন কচেরন কাররণ (নয ভাণডাচের গৈপতক ায়ানার উজজব ইহিতাস অচেপbা দোার হিসeচেকর মচেধয গৈপতক দেকামপাহিনর কাগ তার কাচেছ অচেনক দেহি( মযান

TO BE CONTINUED

উ- ক) আচোয অং(টি রীনদরনাথ ঠাকচেরর দো ঠাকরা গচেলপর অং() কতা চেন আচোয গচেলপর গলপ কথকইার চেত নয়ন দোচের হিমাহিরর দে(4 ং(ধর গৈকাস ার কথা া চেয়চেছ গৈকাস া নয়ন দোচের সমসত সমপহিতত ঋচেরণর াচেয় হিহিx কচের অহি(ষট যা আচেছ তাচেত হিপত পরচে4র যাহিত রbা করা সমভ নয় তাই হিতহিন পতরচেক হিনচেয় ককাতায় সাস শর কচেরনগ) গৈকাস ার হিপতার মতযর পর নয়ন দোচের হিমাহিরর অহিসততব হিপত য় কচেয়কটা উৎস ও শরাদধ- (াহিনতচেত হিমাহিরর দে(4 কহিটক যয় চেয় হিগচেয় এচেক াচের দে(4 চেয় যায় তন তাচের গC করার মত আর হিকছই হিছ না-দেসই পরসচেঙগ এই উহিকত নয়নচোচের হিমাচেররা া নাচেমই হিযাত হিছচেন ায়ানার উাররণ সবরপ নয়নচোচের ারা হিা (াচেকর হিা হিচেতন এছাাও দেকান উৎস উপচেb রাহিতরচেক হিন করচেত হিগচেয় তারা সযC হিকরচেরণর নয পরীপ জবাহিচেয় তাচেত রপার হির 4Cরণ করচেতন তাই দেসকাচের ায়ানা দেহি(হিন সথায়ী চেত পারত না হিহিভনন উৎস শরাদধ- (াহিনতচেত সাধযা হিতহিরকত র করার নয হিমাহির হিহিকচেয় দেযত হ হিতC কা হিহি(ষট পরীচেপর দেত দেযমন অলপকাচের মচেধয হিনঃচে(4 চেয় যায়-নয়নচোচের হিমারচের অসথা তাই চেয়হিছ এই কারচেরণই কথক নয়নচোচের হিমারচের গা ভরা আমবর সয করচেত পারতনা

Physics Dimensional Analysis (Summary)

Q Find the dimensions of consts ab in relation

p=(bminusxlowastx)at

where p is the power x is the distance and t is time

Ans From principle of homogeneity dimension of b x2 are same Dim of b = dim of x2 = [L2] = [ML2T0]Dim of a = dim of ( b- x2)dim of (pt) = [M0L2T0][ML2T-2] [T-1] [T] = [M-1L0T2]

Chemistry Atomic Structure Drawbacks of Rutherfordrsquos model of

atom a According to Rutherfordrsquos model of atom electrons which are negativelycharged particles revolve around the nucleus in fixed orbits Thusb theelectrons undergo acceleration According to electromagnetic theory of Maxwell a charged particle undergoing acceleration should emitelectromagnetic radiation Thus an electron in an orbit should emitradiation Thus the orbit should shrink But this does not happenc The model does not give any information about how electrons aredistributed around nucleus and what are energies of these electrons Isotopes These are the atoms of the same

Properties of electromagnetic radiationsa Oscillating electric and magnetic field are produced by oscillating charged particles These fields are perpendicular to each other and both areperpendicular to the direction of propagation of the waveb They do not need a medium to travel That means they can even travel invacuum

Characteristics of electromagnetic radiationsa Wavelength It may be defined as the distance between two neighbouring crests or troughs of

element having the same atomicnumber but different mass numbere g 1H11H21H3

Isobars Isobars are the atoms of different elements having the same massnumber but different atomic numbere g 18Ar40 20Ca40

Isoelectronic species These are those species which have the same numberof electrons

Electromagnetic radiationsThe radiations which are associated withelectrical and magnetic fields are called electromagnetic radiations When anelectrically charged particle moves under acceleration alternating electricaland magnetic fields are produced and transmitted These fields aretransmitted in the form of waves These waves are called electromagneticwaves or electromagnetic radiations

wave as shown It is denoted by λb Frequency (ν) It may be defined as the number of waves which passthrough a particular point in one secondc Velocity (v) It is defined as the distance travelled by a wave in onesecond In vacuum all types of electromagnetic radiations travel with thesame velocity Its value is 3 times10 8m sec-1 It is denoted by v

d Wave number Wave number is defined as the number of wavelengths per unit lengthVelocity = frequency timeswavelength c = νλ

Plancks Quantum Theory- o The radiant energy is emitted or absorbed not continuously but discontinuously in the form of small discrete packets of energy called lsquoquantumrsquo In case of light the quantum of energy is called a lsquophotonrsquoo The energy of each quantum is directly proportional to the frequency of the radiation ie E α υ or E= hυ where h= Planckrsquos constant = 6626 x 10-27 Js o Energy is always emitted or absorbed as integral multiple of this uantum E=nhυ Where n=1234Black body An ideal body which emits and absorbs all frequencies is calleda black body The radiation emitted by such a body is called black body radiation

Photoelectric effectThe phenomenon of ejection of electrons from thesurface of metal when light of suitable frequency strikes it is calledphotoelectric effect The ejected electrons are called photoelectrons

Biology Chapter - 02Systematics and Five Kingdoms

Scientists divide the whole living organisms into two kingdom first and ultimately by five kingdom at last

In the earlier systems of classifications organisms are divided into kingdom plantaeand kingdom animalia on the of presenceof cell wall their modes of nutrition and movements

Some problem arise like fungi share manycharacteristic withplant despite their heterotrophic nutrition bacteria protozoa areunicellular present in both kingdom Toovercome this third kingdom Protista isintroduced which include

unicellularorganisms But there is also another

problem Allunicellular organisms are not similar kind The cellular structure of prokaryotes is verydifferent from that of other organismsEukaryotes possess a true nucleus and allcell organelles that are not present inprokaryotes So the fourth kingdom Monerais introduced which include unicellular prokaryotes (bacteriaamp blue green algae)

bull Still some problem arise in kingdomplantae

So in 1969 R H Whittakar proposedanew five kingdom System of classification

i) Kingdom Monera - unicellular prokaryotes

ii) kingdom Protista - unicellular eukaryotes

iii) Kingdom Fungi - uni or multicellular fungi with cell wall but without chlorophyll

iv) Kingdom Plantae - Multicellular Plants

v) Kingdom Animalia - Multicellular Animals

EVS Chapter 1 ndash Modes of Existence

An agricultural society

An agricultural society also known as an agrarian society is a society that constructs social order around a reliance upon farming More than half the people living in that society make their living by farming

People in an agricultural society generally lead a more settled lifestyle than those in nomadic hunter-gatherer or semi-nomadic pastoral societies because they live permanently near the land that is farmed Agricultural settlements tend to develop in areas of convenience near bodies of water which is used for both crops and transportation or along trade routes Not everyone in an agricultural society is a farmer Some people make a living trading or making and selling goods such as tools used for farming

Another way to define an agrarian society is to see the total amount of production in a nation In an agrarian society cultivating the land is the main source of wealth Such a society can recognize other means of subsistence and work habits but emphasizes the importance of agriculture and livestock Agrarian societies have existed in various parts of the world for 10000 years and continue to exist today They have been the most common form of socio-economic organization for most of recorded human history

Q) Write the features of agricultural society

Ans - Structure and Features of Agrarian Society1 Occupational Structure

An agrarian society is generally associated with the domestication of plants and animals The domestication of plants means farming and that of animals means herding Often there is mixture of farming and the use of such domesticated animals as cow goat and sheep

2 Forms of Land Ownership in Agrarian SocietiesGenerally there are landlords supervisory farmers cultivators and share croppers The landholders own the land but do not work on it They let it out for sharecropping The supervisory farmers are those who live by having their land cultivated by hired labourers The cultivators cultivate the land for themselvesThe share-croppers are those who live by tilling other peoplersquos land or a crop-sharing basis The artisans own their means of production and produce by their own labour in their homesteads

3 Village Community System An agrarian society is highlighted by

the institution of village community system The agrarian economy made fixed dwelling houses necessary Living close together for protection and co-operation and living nearer to the land gave birth to agricultural villages The village is not only the residential place of farmers it is also the social integrator

4 Minimal Division of Labour Another structural feature of agrarian society is a minimal division of labour Except for the basic division founded on age and sex differences there are few specialized roles There is only one predominant type of occupation ie domestication of plants and animals For all the people the environment physical as well as social is the same

5 Role of Family The farm family is of the patriarchal type the father is the final arbiter in most of the familyrsquos major decisions The life of ail men and women is merged in family life Since there are not many special organizations family is the only organisation to perform the tasks of aid and protection

6 Sense of Unity The members of an agrarian society exhibit a strong in-group feeling Since the whole of their social lives is wrapped up in a society which is physically economically and socially homogenous they are inclined to view the entire outside world as an out group

7 Informal Social Control An agrarian society is regionally divided into villages In a village community the force of traditional mores is more dominant than in the urban community In the village everybody is known to everybody The members in a village community help each other and share the joy and sorrows of each other Crime in an agrarian society is rare

8 Simplicity and Uniformity Life of the people in an agrarian society is marked by simplicity and uniformity Their main occupation is agriculture which largely depends upon the vagaries of nature An agrarian society is a religious society

Math Compound angles Compound angles The algebraic sum of two or more angles is called a compound angle If A B C be three angles then A+B B+C C+A A-B B-C A-C A+B-C etc are compound angles In this chapter we shall discuss the trigonometrical ratios of compound angles Theorem 1 If A B and A+B are all pisitive acute angles theni) sin( A+B) = sin A cos B + cosA sinBii) cos(A+B) = cosA cosB- sinA sinBTheorem 2If A and B are positive acute angles and AgtB theni) sin(A-B) = sin A cosB- cos A sinBii) cos(A-B) = cos A cos B+ sin A sin BTo prove that i) sin(A+B) sin (A-B) = sin2 A - sin2 B = cos2 B- cos2 A

Example 1 Prove that tan70deg=2tan50deg+tan20degSolutiontan70deg = tan(50deg + 20deg)Or tan70deg=(tan 50deg+tan 20deg)(1-tan50degtan20deg) or tan70deg (1 ndash tan 50deg tan20deg) = tan50deg+tan20degor tan70deg= tan70deg tan50deg tan20deg+ tan50deg + tan20deg = cot20deg tan50deg tan20deg + tan50deg + tan20deg = 2 tan50deg+ tan20degExample 2 If A + B = 45deg show that (1 + tanA) (1 + tanB) = 2Solutiontan(A + B) =( tan A + tan B) (1 - tan

ii) cos(A+B) Cos(A-B) = cos2 A- sin2 B = cos2 B -sin2 AProof i) LHS= sin(A+B)sin(AminusB) [Recall sin(αminusβ)=sinαcosβminuscosαsinβ And sin(α+β)=sinαcosβ+cosαsinβ]= (sinAcosB+cosAsinB)times(sinAcosBminuscosAsinB)= sin2Acos2Bminuscos2Asin2B [Recall sin2α+cos2α=1 From above we can then assume correctly that sin2α=1minuscos2α AND cos2α=1minussin2α] = sin2A(1minussin2B)minussin2B(1minussin2A) = sin2Aminussin2Asin2Bminussin2B+sin2Asin2B = sin2Aminussin2B= 1-cos2A-(1-cos2B) = cos2 B- cos2 A = RHSii)LHS= cos (A+B) cos(A-B) [ cos(A+B) = cos AcosB- sinAsinBCos(A-B) = cosAcosB+ sinAsinB]= cos2 A Cos2 B- sin2 A Sin2 B= cos2 A( 1-sin2 B) - (1- cos2 A) sin2 B= cos2 A- cos2 A sin2 B- sin2 B+ cos2 A sin2 B=cos2 A- sin2 B=1- sin2 A-(1-cos2 B) = cos2 B- sin2 A= RHSTangent formulae for compound anglesi)tan (A + B) = tan A + tan B1-tan A tan Bii)tan (A ndash B) = tan A-tan B1+tan A tan Biii) cot (A + B) = cot Acot B-1cot A+cot B(viii) cot (A ndash B) = cot Acot B+1cot B-cot A

A tan B) Or 1= (tan A+ tanB) (1-tan A tanB) Or tanA + tanB + tanA tanB + 1 = 1 + 1Or tanA (1 + tanB) + (1 + tanB) = 2Or (1 + tanA) (1 + tanB) = 2Example 3 Find the value of sin 15degSolution sin 15deg= sin(45deg-30deg) = sin45degcos 30deg- cos45degsin30deg =(1radic2) (radic32) -(1radic2) (12) = (radic3-1) 2radic2Example 4 If sin A = 1 radic10 and sin B = 1 radic5 where A and B are positive acute angles then what is A + B SolutionWe know that sin (A + B) = sin A cos B + cos A sin B= [1 radic10] [radic(1 minus 1 5)] + [1 radic5] radic(1 minus 1 10)= [1 radic10] [radic4 5] + [1 radic5] [radic9 10]= [1 radic50] times (2 + 3)= 5 radic50 = 1 radic2

sin (A + B) = sin π 4rArrHence A + B = π 4Example 5 If A + B = 225o then find [cot A] [1 + cotA] times [cot B] [1 + cot B]Solution[cot A] [1 + cotA] times [cot B] [1 + cot B] = 1 [(1 + tan A) times (1 + tan B)]=1 [tan A + tan B + 1 + tan A tan B] [ tan (A + B) = tan225o]∵

tan A + tan B = 1minus tan A tan BrArr= 1 [1 minus tan A tan B + 1 + tan A tan B]= 1 2

COMMERCE

CLASSIFICTION OF HUMAN ACTIVITIES-ECONOMIC AND NON-ECONOMIC

Firstly we shall recall the previous class for 5 mins especially for the absentees and for also the rest of the students who were there

Today at first we briefly discuss the earlier portions of the chapter

1Business-It includes all those economic activities which are concerned with production and exchange of goods and services with the object of earning profit Example A factory shop beauty parlour also business enterprises

2Profession ndashThe term profession means an occupation which involves application of specialized knowledge and skills to earn a living For Example Chartered Accountancy medicine law tax consultancy are example of professions

Questions1What are the main features of ProfessionAnswer The main features of a profession are as follows a Specialised body of knowledge-Every profession has a specialised and systematised body of knowledge b Restricted entry- Entry to a profession is allowed only to those who have completed the prescribed education and have the specialised examination c Formal education and training ndashA formal education and training is given to the person who wants to acquire the professional

3Employment-Employment mean an economic activity where people work for others in exchange for some remuneration (salary)The persons who work for others are called lsquoemployeesrsquo The persons or organizations which engage others to work for them are called lsquoemployersrsquoEg A doctor working in a hospital is employment as he is working for a salaryA lawyer may serve as a law officer in a bank

With this we shall proceed with the features of both Profession amp Employment

The main features of a profession are as follow

a Specialised body of knowledge b Restricted entry c Formal education and training d Professional association e Service motive f Code of contact

The main features of an employment are as follows

a In employment a person works for others called employer

b An employee provides personal service

c There is a service agreement or contract between the employee and the employer

d The employee has to obey the order of the employer

e No capital investment is made by the employer

Various examples of Employment are as follows

aA teacher teaching in a school or collegeb An engineer employed in Municipal Corporation of DelhicAn accountant working in the accounts department of a companydA doctor working in a hospital

Note In all the above examples of employment the individual who is involved in each example is working as an employee for a salary under an employer

qualification(MBBSCALLB)d Service motive ndashProfessionals are expected to emphasis service more on their clients rather than economic gain f Code of Conduct-The activities of professionals are regulated by a code of conduct

2 What are the main features of EmploymentAnswer The main features of an employment are as followsa In employment a person works for others called employerb An employee provides personal servicec There is a service agreement or contract between the employee and the employerd The employee has to obey the order of the employere No capital investment is made by the employer

3 Give various Professions and their respective Association are given below

Professions

Professional

Professional association

Medical profession

Doctor Medical Council of India

Law profession

Lawyers Bar Council of India

Accounting Profession

Chartered

The Institute of Chartered Accounts of India( ICAI)

Engineerin Engineers The

g Profession

institute of Engineers (India)

Accounts Basic accounting terms

Today we will give you some questions from the previous study material

Questions6) Define accounting7) What do you mean by debit

and credit8) Explain the types of account9) Define the following terms

a) Assetsb) Capitalc) Purchased) Debtorse) Transactions

10) Name the types of accounts given below

a) Krishnas accountb) Machinery accountc) Royalty accountd) Salary accounte) Furniture accountf) Audit fee account

Economics Basic Economic ConceptsSub topic

UTILITY

Before starting todayrsquos class we shall recall the last class which was about UTILITY AND THE FEATURES OF UTILITY

Now we shall proceed with the further topics of the chapter

Todayrsquos topic from the chapter lsquo Basic Economic Conceptsrsquo will be TOTAL UTILITY amp MARGINAL UTILITYNow let us quickly revise the concept of utility with an example ie goods and services are designed because they have an ability to satisfy human wantsThis feature of being able to satisfy human wants is termed as utility For example we derive utility from WiFi services as it gives us satisfaction by connecting us to our friends and family through social media here consumers derive utility from WiFi services

From the above concept we shall start with todayrsquos topicEconomists have defined TOTAL UTILITY (TU) as the total satisfaction obtained by consuming a given total amount of a good and serviceFor example the total satisfaction obtained from eating 10 mangoes is the total utility of 10 mangoes

MARGINAL UTILITY (MU) is the additional satisfaction derived from each additional unit

Questions1 What is Total Utility (TU)

Answer Total Utility (TU) is the

aggregate of the utility that a consumer derives from the consumption of a certain amount of a commodityTU=MU1+MU2++MUn

2 What is Marginal UtilityAnswer

Marginal Utility (MU) is the additional made to the total utility as consumption is increased by one more unit of the commodityMU= TUn ndashTUn-1

NoteOften economists tend to

subdivide utility into an imaginary unit called UTIL

consumed In this casethe utility obtained from each mango as it is consumed as the MU of that mango It is also defined as the addition made to the total utility when an additional unit is consumed Often economists tend to subdivide utility into an imaginary unit called UTIL

Note As a consumer increases the consumption of a good over period of time the total utility or total satisfaction derived from it increases to appoint and thereafter it decreasesHowever as the consumer keeps on consuming the good the marginal utility or the additional utility derived from it decreases

SubjectBusiness studies

Topic

BUSINESSENVIRONMENT

Summary

Now quickly let us revise the earlier points that we have already done in the last class and let us proceed with the other topics that are there in the chapter

Firstly we will recall the internal and external factors of micro environment and then we shall proceed in details

Meaning and list of internal and external factors

aInternal factorsInternal factors refer to all the factors existing within a business firm The internal factors are considered controllable because the enterprise has control over these factorsFor an example a company can alter its organization structure policies programmes employees physical facilities and marketing mix to suit the changes in the environmentList of internal factors areCorporate culture mission and objectives top management organizations structure company image and brand equity company resources

b External factorsExternal factors refer to those individual and groups and agencies with which a particular business organization comes into direct and frequent contact in the course of its functioningThese individuals and groups are known as STAKEHOLDERS because they have a stake (financial interest ) in the working and performance of the particular business List of external forces (stakeholders)Customers competitors investors suppliersmiddlemen (marketing intermediaries)

Execution 1 What do you mean by internal

factors in micro environmentAnswerInternal factors refer to all the factors existing within a business firm The internal factors are considered controllable because the enterprise has control over these factorsFor an example a company can alter its organization structure policies programmes employees physical facilities and marketing mix to suit the changes in the environment

2 What do you mean by external factors in micro environment

AnswerExternal factors refer to those individual and groups and agencies with which a particular business organization comes into direct and frequent contact in the course of its functioningThese individuals and groups are known as STAKEHOLDERS because they have a stake (financial interest) in the working and performance of the particular business

3Who are stakeholdersSTAKEHOLDERS are individuals and groups who have a stake (financial interest ) in the working and performance of the particular business 4Discuss the internal factors in briefa Corporate CultureThe values beliefs and attitudes of the founders and top management of the company exercise

financers publics

customers

suppliersfinancers

competitors

middlemen

publics

Fig STAKEHOLDERS OF A COMPANY

Apart from micro environment the other main dimension of business environment isMacro environment Macro environment refers to the general environment or remote environment within which a business firm and forces in its micro environment operateA company does not directly or regularly interact with the micro environmentTherefore macro environment is also known as indirect action EnvironmentThe macro environment forces are less controllable than the micro forces

Macro environment consists of the following components

POLITICAL AND LEGAL ENVIRONMENT

ECONOMIC SOCIAL AND ENVIRONMENT

CULTURAL

ENVIRONMENT

TECHNOLOGICAL ENVIRONMENT

a strong influence on what the cmpaany stands for how it does things and what it considers importantbMission and objectivesThe business philosophy and purpose of a comoany guide it prioritiesbusiness strategiesproduct market scope and development scope

cTop management structurethe composition of board of directors the degree of professionalization of management and the organizational structure of a company have important bearing on its business decisions

dPower structureThe internal power relationship between the board of directors and the chief executive is an important factor

eCompany image and brand equityThe image and brand equity of the company play a significant role in raising finance forming alliance choosing dealers and suppliers launching new products entering foreign markets

5 What is Macro environmentAnswerMacro environment refers to the general environment or remote environment within which a business firm and forces in its micro environment operateA company does not directly or regularly interact with the micro environmentTherefore macro environment is also known as indirect action EnvironmentThe macro environment forces are less controllable than the micro forces 6 What are the components of macro environmenta Political and legal environmentb Economic environmentc Social and cultural environmentd Technological environment

BUSINESS FIRM

Fig COMPONENTS OF MACRO ENVIRONMENTPolitical science

Introduction to political science

Comparative politics and itrsquos scope Comparative politics is the second major dimension of political scienceIt is also a very vast area of study and a very large number of political scientists even treat it as an autonomous area of study within the board ambit of political scienceScope of comparative politics-

1 All political structures -Comparative politics includes the study of all structures formalnon formal governmental and extra governmental which are directly or indirectly involved in politics in all the countries of the world

2 Functional studies- Comparative politics seeks to study politics less from the point of view of the legal institutions in terms of their powers and move from the point of view of their functions which constitute the political process and their actual Operation in the environment

3 Study of political behaviour- Another important part of its scope is the study of the actual behaviour of the people in the process of politics

4 Study of similarities and differences- comparative politics also undertakesan analysis of the similarities and differences among political process and functions

5 Study of all political systems -comparative politics seeks to analyse the actual behaviour and performance of all political systems western as well as non western

6 Study of the environment and infrastructure of politics-The study of politics demands a study of the psychological sociological economic and anthropological environment in fact the social environment as a whole in which each political system operates

7 Study of political culture- political culture is composed of attitudesbeliefs emotions and values of a society that relate to the political system or politics

8 Study of political participation- Political participation is a universal processThe only difference is that while in some states it is limited in others it is wider

9 Study of political process- political

Answer the following questions-

What is comparative politics

What are the scope of comparative politics

Homework- learn

processes like decision makingpolicy making judicial process leadership recruitment process and others are always at work in all political systems

The scope of comparative politics is very comprehensive It includes everything that falls within the area of political activity and political process

History CAMBRIDGE VIEW ABOUT

THE PARTITION

AND REFUTATION

OF CAMBRIDGE

VIEW

Cambridge view about the Partition The Cambridge school of historians have interpreted that opposition to partition scheme was made entirely by the elitist groups They hold the view that Lord Curzon planned to partition the Bengal for administrative purposeREFUTATION OFCAMBRIDGE VIEW The Rationalist historians have rejected the interpretations of the Cambridge School of historians on various grounds

1 QUESTION State different views of historians regarding Partition of Bengal

ANSWER Cambridge historians believed that Lord Curzon partitioned Bengal for administrative reasons only and not for the political motive The Middle class elitist group protested because of their petty interest The Hindu zamindars protested as they have to spend more money for managing their estatesThe lawyers of Calcutta High court feared to lose their clientBut according to the nationalist Historians was-

2- The ultimate object of Lord Curzon was to crush the unity of Bengal politicians

3- If Bengal becomes a separate province Bengali speaking 16 million people of western part would become minority under Hindi speaking people of Bihar and Oriya speaking people of Orissa

4- The bureaucrats expected that the protest movement would die down quickly

5- Lord Curzon used the Muslim community in his political game

6- Idealism had great contribution in the protest against partition

7- The people of the every section of society were affected by the partition of Bengal

Computer Science

Numbers Convertion of dcimal number to octal numberThe decimal numeral system is the standard system for denoting integer and non-integer numbers It is the extension to non-integer numbers of the Hindu-Arabic numeral system For writing numbers the decimal system uses ten decimal digits a decimal mark and for negative numbers a minus sign - The decimal digits are 0 1 2 3 4 5 6 7 8 9 the decimal separator is the dot in many countries

The octal numeral system or oct for short is the base-8 number system and uses the digits 0 to 7 Octal is sometimes used in computing instead of hexadecimal perhaps most often in modern times in conjunction with file

permissions under Unix systems It has the advantage of not requiring any extra symbols as digits It is also used for digital displays

Follow these steps to convert a decimal number into octal form

1 Divide the decimal number by 82 Get the integer quotient for the next iteration (if the number will not divide equally by 8 then round down the

result to the nearest whole number)3 Keep a note of the remainder it should be between 0 and 74 Repeat the steps until the quotient is equal to 05 Write out all the remainders from bottom to top This is the solution

For example if the given decimal number is 8453

Division Quotient Remainder

8453 8 1056 5

1056 8 132 0

132 8 16 4

16 8 2 0

2 8 0 2

Then the octal solution is 20405

Subject Eng Literature (The Tempest ndash William Shakespeare) Topic Act I Scene 1 Lines 33 to 67 (End of scene) Date 16th April 2020 (4th Period)

[Students should read the original play and also the paraphrase given in the school prescribed textbook]Summary Questions amp Answers

[SUMMARY OF THE ENTIRE SCENE]

o The play starts with the scene of a severe storm at sea Alonso (King of Naples) Sebastian (Alonsorsquos brother) Ferdinand (Alonsorsquos son) Gonzalo Antonio (the usurping Duke of Milan) are in a ship in the midst of the storm

o The mariners are trying their best to control the vessel from running aground and are totally following the orders of their Master the Boatswain They have scant success

o The mariners become extremely unhappy and annoyed when most of the passengers arrive on the deck thereby hampering their effort to save the ship There is serious confrontation between them and the passengers who are part of the Kingrsquos entourage

o The mariners could not save the ship

SUMMING-UP

(i) Vivid description of the scene which gives a realistic description of terror and confusion of a tropical storm

(ii) Shows Shakespearersquos accuracy of knowledge in describing the naval operations and also matters of seamanship

(1) GONZALO Ill warrant him for drowning (L 45-57)

though the ship were no stronger than a nutshell and as leaky as an unstanched

wenchBOATSWAIN Lay her a-hold a-hold Set her two courses Off to

sea again lay her offMARINERS All lost To prayers to prayers All lostBOATSWAIN What must our mouths be coldGONZALO The king and prince at prayers Lets assist them

For our case is theirsSEBASTIAN Im out of patienceANTONIO We are merely cheated of our lives by drunkards

This wide-chopped rascal - would thou mightst lie drowning the washing of ten tides

(a) What does Antonio say at the insolent manners of the boatswain just before the given passage

Being irritated at the insolent manners of the boatswain just before the given extract Antonio the Duke of Milan calls him a worthless dog son of a woman without any morals an arrogant and disrespectful noisemaker He says that the boatswain deserved to be hanged(b) What statement does Gonzalo repeat about the boatswain

Gonzalo shows his faith that the boatswain is not destined to die by drowning He is destined to be hanged and nothing can alter this decree of destiny He says that even if the ship was as frail as a nutshell the boatswain could not be drowned for his destiny was to be hanged(c) What do the passengers do when they have lost all hope of their survival

When the passengers have lost all hope of survival they take

(iii) The opening scene justifies the title ndash The Tempest

UNANSWERED QUESTIONS

(i) The King always travels with his entire fleet including his soldiers Where were the other ships

(ii) Why was the ship in that area Where was it coming from or going where

(iii) The ship broke apart What happened to those who were in the ship

(We shall get the answer to the above questions as the play progresses)

leave of life with fervent prayers The mariners take their last hearty drink and are ready for death(d) What blame does Antonio put upon the mariners and the boatswain Antonio rebukes the mariners that these drunkards have brought them to the present crisis by neglecting their duties He blames them saying that they are going to lose their lives entirely for the negligence of the boatswain and his fellows(e) What does Antonio say while cursing the boatswain

Antonio gives vent to his wrath upon the boatswain in particular He calls the boatswain a wide-mouthed rascal who deserves to be hanged on the sea-shore at low water mark so that ten tides might wash over his body and take out of him all the liquor that he has been drinking

Class XIISubject Topic Summary ExecutionHistory Topic

1 1935 ACT AND WORKING OF PROVINCIAL AUTONOMYCONGREE AND OTHER MINISTERSSUB TOPIC GOVERNMENT OF INDIA ACT1935

Government of India Act 1935 This act established a lsquoFederation of Indiarsquo made of British Indian provinces and Indian states and provided for autonomy with a government responsible to the elected legislature in every provinceThis act introduced abolition of Diarchy at provinces The entire provincial administration was introduced to the responsible ministers who were controlled and removed by the provincial legislature The provincial autonomy means two things First The provincial governments were wholly responsible to the provincial legislature Secondly Provinces were free from outside control and interference in the large number of matters The act divided the powers between the centre and provinces in terms of three lists- Federal list( for centre) Provincial list (for province) and concurrent list (for both) Residuary powers were given to the viceroy In the election under the government of India Act the Congress swept the poll the mandate of the people came in favour of the congress so far as general Hindu seats were concerned The Congress did not get a single Muslim seates in Bombay CP UP Sind and BengalIn five provinces Congress had yhe clear majority In BengalNWFPAssam and Bombay Congress emerged as a single largest partyOn the other side the performance of the Muslim League was badThus the Congress formed ministers in 7 provinces out of 11 provinces Coalition ministry was also formed in two other provincesOnly BENGAL AND Punjab had non- congress ministries

1 QUESTION What was the main change introduced by the Government of India ActANSWER a) The Act gave more

autonomy to the provinces b) Diarchy was abolished at the

provincial levelsc) The Governor was the head of

the executived) There was a council of

ministers to advise him The ministers were responsible to the provincial legislatures who controlled them The legislature could also remove the ministers

e) The Governors still retained special reserve powers

2 QUESTION Why did the federal scheme introduced by the Government of India Act 1935 never come into operation

ANSWER The Federal structure of the Government of India was to be composed with the Governor General and Council of ministers The Federal legislature was to be Bicameral legislature- The council of states and the House of Assembly The ministers were to be chosen by the Governor general and they were to hold the office during his pleasure

The provinces of British India would have to join the federation but this was not compulsory for the princely states

This federation never materialised because of the lack of support from the required number of

princely statesThis act was refused and

rejected by the princes the Congress and the Muslim League

Thus both Congress and the League participated in the election of 1937 Thus the federal part was never introduced but the provincial part was put into operations

Bengali 2nd

Language

াচেরর পরাথCনা(কহিতা )

াচেরর পরাথCনা কহিতাটি কহি (ঙখ দেঘাচে4র দো আচো য কহিতায় াচেরর পতর হমায়ন কঠিন দেরাচেগ আxানত ার ঈশবর া আললার কাচেছ পরাথCনা কচেরচেছন তার পচেতরর ীন হিফহিরচেয় হিচেত এই কহিতায় ার পচেতরর ীন হিভbা দেচেয়চেছন ারার এমনহিক হিনচের ীন হিসCচেনর হিহিনমচেয় হিতহিন তার দেছচের ীন হিফচের দেপচেত দেচেয়চেছন তার দেছচের এই দেরাচেগর ন য হিতহিন হিনচেচেকই ায়ী কচেরচেছন তার হিনচের করা পাপচেকই হিতহিন ায়ী কচেরচেছন এছাা রানৈনহিতক ও আথCসামাহিক অসথার কথা তচে ধরা চেয়চেছ এই কহিতায় ার তার হিনচের পাপ কমCচেকই ায়ী কচেরচেছ ার অন যায় ভাচে দেপহি((হিকতর মাধ যচেম অপররা য কচেরচেছ আর এই অন যায় কাচের ন যই তার পহিরাচের হিপযCয় এচেসচেছ দে এক পরকার মানহিক নধন ইহিতাচেসর ার হিপতা চেয় সবাভাহিকভাচে ভাচোাসা দে মমতা দেথচেক মকত চেত পাচেরনহিন তাই হিপতা চেয় আললা া ভগাচেনর কাচেছ পতর হমায়চেনর পরানহিভbা দেচেয়চেছন ার আললা া ভগাচেনর কাচেছ াহিনচেয়চেছন তার হিনচের ীন হিসCন হিচেত হিতহিন রাী তার হিহিনমচেয় পচেতরর ীন হিফচের দেপচেত দেচেয়চেছন াচেরর হিপতসভ হিচেকর কথা এই কহিতায় ফটিচেয় দেতাা চেয়চেছ হিপতা পচেতরর হিরাহিরত মান নধচেনর কথা তচে ধরা চেয়চেছ

হিচে(4 হিকছ াইচেনর তাৎপযC১) ldquoদেকাথায় দেগ ওর সবচছয দেৌন দেকাথায় কচেরায় দেগাপন bয়ldquoউততর) াচেরর পতর হমায়ন কঠিন দেরাচেগ অসসথ তাই তার দেযৌন াহিরচেয় যাচেচছ এই দেরাচেগ তাচেক দেগাপচেন কচেরকচের াচেচছ তার সক (হিকত ধীচের ধীচের bয় চেচছ তাই হিপতা চেয় ার আললার কাচেছ হমায়চেনর পরান হিভbা দেচেয়চেছন২) ldquoাগাও (চেরর পরাচেনত পরানতচের ধসর (ন দেযর আান গানldquoউততর) াচেরর পতর হমায়ন কঠিন দেরাচেগ আxানত তাই ার আ দে(াচেক মমCাত (চেরর পচেথ পরানতচের আান গান ধবহিনত দোক দেসই আান গান আললার কাচেছ দেযন চে যায় আললা দেযন এই আহিতC শচেন পচেতরর ীন হিফহিরচেয় দেয় ৩)ldquoনাহিক এই (রীচেরর পাচেপর ীানচেত দেকানই তরারণ দেনই ভহি4চেতরldquoউততর) হমায়চেনর অসসথতার ন য ার হিনচেচেকই ায়ী কচেরচেছন কারন ার অচেনক রা য অন যায় ভাচে কচেরচেছ তাই তার এই পাপ কাচের ন য তার ঘচের আ হিপ এচেসচেছ এই অন যায় কাচের ন য তার মহিকত দেনই তাই ার আললার কাচেছ এই পাপ কাচেযCর ন য bমা পরাথM

Hindi 2ndlang

-ासी(जयशकर परसा-)

-ासी जयशकर परसा- की एक ऐसी कहानी ह जिजसम भारतीय ससकनित और राषटरीयता का सवरगजीतहोता ह इस कहानी म इरावती एक निहद कनया ह जिजस मलअचछो न मलतान की लट म पकडा और -ासी बना दि-या उस 500 दि-न -कर काशी क एक महाजन न खरी-ा दसरी -ासी निफरोजा ह वह गलाम ह निफरोजा को छडान क कतिलए अहम- को 1000 सोन क कतिसकक भजन थ जो अभी तक नही आए थ राजा साहब कठोर होत हए भी निफरोजा को निबना धनराकतिश क कतिलए उस म कर -त ह वनिफरोजा को अहम- को समझान की बात कहत हकहानी क अत म हम -खत ह निक इरा वती और जाटो क सर-ार बलराज का मिमलन होता हअहम- को यa म मार दि-या जाता ह वहा निफरोजा की परसननता की समामिध बनती ह वहा एक फल चढती ह और डीजल आती ह निफरोजा उस समामिध की आजीवन -ासी बनी रहती हलखक अपन उददशय अथात -ास परथा पर परकाश डालन और इस परथा क कारण होन वाल -ातो क दखो को दि-खान म पणता सफल हए ह

helliphellipContinue to next

Biology Reproductio Today we will discuss about vegetative Q1 Name some vegetative propagules

n in Organisms

propagation of plants The process of multiplication in which fragments of plant body function as propagule and develop into new individual is called vegetative propagation The units of such propagation are runner rhizome tuber bulb etc

and the speciesinvolvedVegetative propagules

Parts involved

Bulb StemBulbil BulbilRhizome Stem Runner Stem Tuber Stem Offset Stem Leaf buds Leaves Suckers Stem

Corns Stem stolon

Q2 State advantages of vegetative propagation

i) Rapid methodii) Sure and easy methodiii) Useful in plants that cannot

produce viable seeds or long seed dormancy

iv) Maintains purity of raceQ 3 Banana fruit is said to be parthenocarpic where as turkey is said to be parthenogenetic WhyBanana develops without fertilization from an unfertilized ovary thus is parthenocarpicIn turkey the ovum or female gamete developinto a new chick without fertilization thus isparthgenetic

Q4 Why is water hyacinth is called as a ldquoTerror of Bengalrdquo Water hyacinth can

propagatevegetatively all over the water body in a short per short period of time This resulted increased biochemicaloxygen oxygen demand of water body causing mortalityof fishes It is very difficult to get rid off them Thus known as terror of Bengal

Chemistry

Solid state GENERAL CHARACTERISTICS OF SOLID STATEIn nature the particular state of matter is governed by two opposing forces at given set of temperature and pressure These forces are intermolecular force of attraction and thermal energy If intermolecular force of attraction is high as compared to thermal energy particles remains in closest position

Intext QuestionsQ1 Classify the following solids as crystalline and amorphous Sodium chloride quartz glass quartz rubber polyvinyl chloride Teflon

A1 Crystalline

and hence very less movement in particles is observed In this case solid state is the preferred state of matter

Let us revise the general characteristics of solid

i) Fixed mass volume and shape

ii) Strong intermolecular force of attraction

iii) Least intermolecular space

iv) Fixed position of constituent particles

v) Incompressible and rigid

Q2 what type of interactions hold the molecules together in a polar molecular solid[CBSE 2010]A2 The molecules in a solid are held together by van der Waals forces The term van der Waals forces include hydrogen bonding dipole-dipole attraction and London dispersion forces All molecules experience London dispersion forces In addition polar molecules can also experience dipole-dipole interactions So the interactions that holds the molecule together in polar molecular solid are London dispersion force and dipole-dipole interactionsQ3 Write a feature that will distinguish a metallic solid from an ionic solid [CBSE 2010]A3 Metals are malleable and ductile whereas ionic solid are hard and brittle Metallic solid has typical metallic lustre But ionic solid looks dullQ4 Write a point of distinction between a metallic solid and an ionic solid other than metallic lustre [CBSE 2012]A4 Metals are malleable and ductile whereas ionic solid are hard and brittleQ5 Write a distinguish feature of metallic solid [CBSE 2010]A5 The force of attraction in

solid Sodium chloride Quartz Amorphous solid Quartz glass rubber polyvinyl chloride Teflon Q2 why glass is considered as super cooled liquidA2 Glass shows the tendency to flow at slower rate like liquid Hence they considered as super cooled liquidQ3 why the window glass of old buildings show milky appearance with timeA3 Glass is an amorphous solid Amorphous solid has the tendency to develop some crystalline character on heating Due to heating in day over the number of years glass acquires some crystalline character and show milky appearanceQ4 why the glass panes fixed to window or doors of old building become slightly thicker at bottomA4 Glass is super cooled liquid It has the tendency to flow down very slowly Due to this glass pane becomes thicker at the bottom over the timeQ5 Sodium chloride is a crystalline solid It shows the same value of refractive index along all the direction TrueFalse Give reasonA5 FalseCrystalline solid shows anisotropy in properties That is it shows different values for the given physical property in different direction All the crystalline solids show anisotropy in refractive index Therefore sodium chloride will show different values of refractive index on different directions

Q6 Crystalline solid are anisotropic in nature What does this statement means

between the constituent particles is special kind of electrostatic attraction That is the attraction of positively charged kernel with sea of delocalized electronsQ6 which group of solid is electrical conductor as well as malleable and ductile [CBSE 2013]A6 Metallic solidQ7 why graphite is good conductor of electricity although it is a network (covalent solid)A7 The exceptional property of graphite is due to its typical structure In graphite each carbon is covalently bonded with 3 atoms in same layer The fourth valence electron of each atom is free to move in between different layersThis free electron makes the graphite a good conductor of electricity

[CBSE 2011]A6 Anisotropy is defined asrdquo Difference in properties when measured along different axis or from different directionsrdquo Crystalline solid show different values of some of the physical properties like electrical resistance refractive index etcwhen measured along the different directions The anisotropy in crystalline solid arises due to the different arrangement of particles in different directions

Math Function Composition of functions Think of an industrial plant that produce bottles of cold drinks first there is the operation (or function) f that puts the cold drink inside the bottle followed by the opeartion g that close the bottle with the capThis leads to the following definitionDefinition Let f A rarr B and g B rarr C be two functions Then the composition of f and g denoted by gof is defined as the function gof A rarr C given by gof(x) = g(f (x)) forall x isinA

Definition A function f X rarr Y is defined to be invertible if there exists a function g Y rarr X such that gof = IX and fog = IY The function g is called the inverse of f and is denoted by f -1

Thus if f is invertible then f must be one-one and onto and conversely if f is one-one and onto then f must be invertible This fact significantly helps for proving a function f to be invertible by showing that f is one-one and onto specially when the actual inverse of f is not to be determined

Example 1 Let f 2 3 4 5 rarr 3 4 5 9 and g 3 4 5 9 rarr 7 11 15 be functions defined as f(2) = 3 f(3) = 4 f(4) = f(5) = 5 and g (3) = g (4) = 7 and g (5) = g (9) = 11 Find gofSolution We have gof(2) = g (f(2)) = g (3) = 7 gof(3) = g (f(3)) = g (4) = 7gof(4) = g (f(4)) = g (5) = 11 and gof(5) = g (5) = 11Example 2 Find gof and fog if f R rarr R and g R rarr R are given by f(x) = cos x and g (x) = 3x2 Show that gof ne fogSolution We have gof(x) = g(f(x))=g(cosx) = 3 (cos x)2

= 3 cos2 x Similarly fog(x)=f(g (x))= f(3x2)= cos (3x2) Note that 3cos2 x ne cos 3x2 for x = 0 Hence gof ne fogExample 3 Show that if f A rarr B and g B rarr C are onto then gof A rarr C is also ontoSolution Given an arbitrary element z isin C there exists a pre-image y of z under g such that g (y) = z since g is onto Further for y isin B there exists an element x in A with f(x) = y since f is onto Therefore gof(x) = g (f(x)) = g (y) = z showing that gof is onto Example 4 Let Y = n2 n isin N sub N Consider f N rarr Y as f(n) = n2 Show that

f is invertible Find the inverse of fSolution An arbitrary element y in Y is of the form n2 for some n isin N This implies that n =radicy This gives a function g Y rarr N defined by g (y) =radicy Nowgof (n) = g (n2)=radicn2 = n and fog (y) =f(radicy) = (radicy) 2 y which shows that gof=IN and fog= IY Hence f is invertible with f -1 = g

Political Science

Constitution of India-The Preamble

Summary

Objective of the state-To secure equality of status and of opportunity To promote fraternity among all the citizens To assure the dignity of the individuals and Unity and integrity of the nation

Justice-Justice stands for rule of law absence of arbitrariness and a system of equal rights freedom and opportunities for all in a society India seeks social economic and political justice to ensure equality to its citizens

Liberty-Liberty implies the absence of restraints or domination on the activities of an individual such as freedom from slavery serfdom imprisonment despotism etc The Preamble provides for the liberty of thought expression belief faith and worship

Equality-Equality means the absence of privileges or discrimination against any section of the society The Preamble provides for equality of status and opportunity to all the people of the country

Fraternity-The Preamble declares that fraternity has to assure two thingsmdashthe dignity of the individual and the unity and

Execution

Answer the following questions-

Short notes-1 Equality2 Fraternity3 Justice4 Liberty

Homework-Learn

integrity of the nation The word integrity has been added to the Preamble by the 42nd Constitutional Amendment (1976)

Business studies

Human resource management (chapter 1)

On the day of 1504 2020 I have discussed with you the managerial functions and procurement functions of HRM

Today weare going to discuss about the development function integration functions and maintenance function

Development functions-HRM improves the knowledge skills attitude and values of employees so that they the present and future jobs more effectively it includes

1) Development functions of HRM

a) Performance appraisal = It implies systematic evaluation of employees with respect to their performance on the job and their potential for development

b) Training =It is the process by which employees learn knowledge skills and attitudes to achieve organisational and personal goals

c) Executive development = It is the process of developing managerial talent through appropriate program

2) Integration functionsa) HRM reconcile the goals of

organisation with those of its members through integrating function

b) HRM tries to motivate employees to various financial and non financial incentives provided in job specification etc

3) Maintenance functiona) HRM promote and protect the

physical and mental health of employees by providing several types of benefits like housing medical aid etc

b) It Promote Social security measures to employees by providing provident fund pension gratuity maternity benefits

SubjectCOMMERCE

Topic

BUSINESSENVIRONMENT

Summary

Now quickly let us revise the earlier points that we have already done in the last class and let us proceed with the other topics that are there in the chapter

Firstly we will recall the internal and external factors of micro environment and then we

Execution 3 What do you mean by internal factors

in micro environmentAnswerInternal factors refer to all the factors existing within a business firm The internal factors are considered controllable because the enterprise has control over these factors

Development FunctionsPerformance AppraisalTrainingExecution Development

shall proceed in details

Meaning and list of internal and external factors

aInternal factorsInternal factors refer to all the factors existing within a business firm The internal factors are considered controllable because the enterprise has control over these factorsFor an example a company can alter its organization structure policies programmes employees physical facilities and marketing mix to suit the changes in the environmentList of internal factors areCorporate culture mission and objectives top management organizations structure company image and brand equity company resources

b External factorsExternal factors refer to those individual and groups and agencies with which a particular business organization comes into direct and frequent contact in the course of its functioningThese individuals and groups are known as STAKEHOLDERS because they have a stake (financial interest ) in the working and performance of the particular business List of external forces (stakeholders)Customers competitors investors suppliersmiddlemen (marketing intermediaries)financers publics

customers

suppliersfinancers

For an example a company can alter its organization structure policies programmes employees physical facilities and marketing mix to suit the changes in the environment

4 What do you mean by external factors in micro environment

AnswerExternal factors refer to those individual and groups and agencies with which a particular business organization comes into direct and frequent contact in the course of its functioningThese individuals and groups are known as STAKEHOLDERS because they have a stake (financial interest) in the working and performance of the particular business

3Who are stakeholdersSTAKEHOLDERS are individuals and groups who have a stake (financial interest ) in the working and performance of the particular business 4Discuss the internal factors in briefa Corporate CultureThe values beliefs and attitudes of the founders and top management of the company exercise a strong influence on what the cmpaany stands for how it does things and what it considers importantbMission and objectivesThe business philosophy and purpose of a comoany guide it prioritiesbusiness strategiesproduct market scope and development scope

cTop management structurethe composition of board of directors the degree of professionalization of management and the organizational structure of a company have important bearing on its business decisions

dPower structureThe internal power relationship between the board of directors and the chief executive is an important factor

e Company image and brand equityThe image and brand equity of the company play a significant role in raising finance forming alliance choosing dealers and suppliers launching new products entering foreign markets

5 What is Macro environmentAnswerMacro environment refers to the general

competitors

middlemen

publics

Fig STAKEHOLDERS OF A COMPANY

Apart from micro environment the other main dimension of business environment isMacro environment Macro environment refers to the general environment or remote environment within which a business firm and forces in its micro environment operateA company does not directly or regularly interact with the micro environmentTherefore macro environment is also known as indirect action EnvironmentThe macro environment forces are less controllable than the micro forces

Macro environment consists of the following components

POLITICAL AND LEGAL ENVIRONMENT

ECONOMIC SOCIAL AND ENVIRONMENT

CULTURAL

ENVIRONMENT

TECHNOLOGICAL ENVIRONMENT

Fig COMPONENTS OF MACRO ENVIRONMENT

environment or remote environment within which a business firm and forces in its micro environment operateA company does not directly or regularly interact with the micro environmentTherefore macro environment is also known as indirect action EnvironmentThe macro environment forces are less controllable than the micro forces 6 What are the components of macro environmenta Political and legal environmentb Economic environmentc Social and cultural environmentd Technological environment

Computer Science

Logic gates

Digital systems are said to be constructed by using logic gates These gates are the AND OR NOT NAND NOR EXOR and EXNOR

BUSINESS FIRM

gates The basic operations are described below with the aid of truth tables

AND gate

The AND gate is an electronic circuit that gives a high output (1) only if all its inputs are high A dot () is used to show the AND operation ie AB Bear in mind that this dot is sometimes omitted ie ABOR gate

The OR gate is an electronic circuit that gives a high output (1) if one or more of its inputs are high A plus (+) is used to show the OR operationNOT gate

The NOT gate is an electronic circuit that produces an inverted version of the input at its output It is also known as an inverter If the input variable is A the inverted output is known as NOT A This is also shown as A or A with a bar over the top as shown at the outputs The diagrams below show two ways that the NAND logic gate can be configured to produce a NOT gate It can also be done using NOR logic gates in the same way

NAND gate

This is a NOT-AND gate which is equal to an AND gate followed by a NOT gate The outputs of all NAND gates are high if any of the inputs are low The symbol is an AND gate with a small circle on the output The small circle represents inversion

NOR gate

This is a NOT-OR gate which is equal to an OR gate followed by a NOT gate The outputs of all NOR gates are low if any of the inputs are highThe symbol is an OR gate with a small circle on the output The small circle represents inversion

EXOR gate

The Exclusive-OR gate is a circuit which will give a high output if either but not both of its two inputs are high An encircled plus sign ( ) is used to show the EOR operation

EXNOR gate

The Exclusive-NOR gate circuit does the opposite to the EOR gate It will give a low output if either but not both of its two inputs are high The symbol is an EXOR gate with a small circle on the output The small circle represents inversion The NAND and NOR gates are called universal functions since with either one the AND and OR functions and NOT can be generated

Note A function in sum of products form can be implemented using NAND gates by replacing all AND and OR gates by NAND gates A function in product of sums form can be implemented using NOR gates by replacing all AND and OR gates by NOR gates

Logic gate symbols

Table 2 is a summary truth table of the inputoutput combinations for the NOT gate together with all possible inputoutput combinations for the other gate functions Also note that a truth table with n inputs has 2n rows You can compare the outputs of different gates

Logic gates representation using the Truth table

Example

A NAND gate can be used as a NOT gate using either of the following wiring configurations

Subject Eng Literature (The Tempest ndash William Shakespeare) Topic Act III Scene 3 Lines 53 to 110 (End of the scene) Date 16th April 2020 (2nd Period)

[Students should read the original play and also the paraphrase given in the school prescribed textbook]Summary Questions amp Answers

o Seeing this strange scene all are inclined to believe the tales told by travelers that there truly are ldquounicornsrdquo and ldquothe phoenixrsquo thronerdquo

o As they are about to sit down to the feast the banquet is snatched away by a harpy (Ariel disguised) A spiritrsquos voice (Arielrsquos voice) denounces Alonso Sebastian and Antonio with particular

1 ARIEL You are three men of sin whom Destiny

(Line 53-58)That hath to instrument this

lower world And what is int the never-surfeited sea

Hath caused to belch up you and on this island

Where man doth not inhabit you rsquomongst men

Being most unfit to live I have made you mad

reference to their crime in expelling Prospero from Milan They have not received any punishment for their deed earlier but the time for their punishment has arrived Upon Alonso it pronounces ldquolingering perdition worse than deathrdquo from which there is no remedy except through sincere repentance Ariel then vanishes in thunder and the shapes enter again and carry away the table

o Prospero watching invisibly is very pleased with the performance of Ariel and his (Prosperorsquos) ldquomeaner ministersrdquo All his enemies are now in his power and are in a fit of desperation He then leaves them and goes to see how Ferdinand and Miranda are getting on

o Alonso is now much humbled and penitent with the after effect of the spiritrsquos denunciation of his crimes He believes that his son is lost forever After this all disperse being stricken mad by the speech of the spirit

o Gonzalo fearing that they may do violence to themselves or to one another follows them and bid others to follow

(a) To whom does Ariel disguised as a harpy call the three sinners What game did Fate of Destiny play with

them

The three sinners called by Ariel are Alonso Sebastian and Antonio It was Destiny which had caused the ocean to cast the three sinners on the shore Though the ocean is all the time devouring whatever appears on its surface and is never satisfied with its continual swallowing of the ships and men in the present case the ocean had cast these three sinners on the shore without killing them

(b) Who had jointly been responsible for the conspiracy against Prospero What is Prosperorsquos purpose behind all this

Three men Alonso Sebastian and Antonio had jointly

been responsible for the conspiracy against Prospero They had driven out Prospero form Milan Prosperorsquos purpose is to make these three sinners realize the wrong they had done He wants them to repent for their criminal deeds because repentance leads to self-esteem(c )What does Ariel (the harpy) tell Alonso and his companions when they take out their swords to attack him

Seeing them drawing their swords Ariel (harpy) tells them that he and his companions are the instruments of destiny and that it is not possible for human beings to do them any injury He says that the swords of human beings can not injure even a minute part of his feathers Their swords are as ineffective against him and his companions as against the wind or the water

(d) Give the explanatory meanings of the following expressions in the context of the above extract

(i)Never surfeited (ii) Belch up (iii) lsquomongst men

(i) Never surfeited never led to satisfaction

(ii) Belch up cast ashore(iii) lsquomongst men in human

society2

I and my fellows (Line 60-65)

Are ministers of Fate The elementsOf whom your swords are tempered may as wellWound the loud winds or with bemocked-at stabsKill the still-closing waters as diminishOne dowl thats in my plume

IMPORTANT PASSAGES EXPLAINED

The elements

(Line 61-66)Of whom your swords are tempered may

as wellWound the loud winds or with

bemocked-at stabs

(a) Who is lsquoIrsquo Who are his lsquofellowsrdquo

lsquoIrsquo is referred to Ariel in disguise of a harpy His lsquofellowsrsquo are other spirits serving Prospero the real Duke of Milan who has acquired supernatural powers after being banished from his Dukedom Prospero has settled in this uninhabited island

(b) What are the elements that have temperrsquod the swords Why will it not work against the speaker

The swords (of Alonso and his companions) are tempered by metal (steel) which is taken out of the earth and refined by

Kill the still-closing waters as diminishOne dowl thats in my plume My fellow

ministersAre like invulnerable

In these words Ariel reminds the King and his companions of the utter futility of drawing swords against himself and his fellows Ariel drives Alonso Antonio and Sebastian the three men of sin to desperation ndash a state in which men do violence to themselves They draw swords to strike Ariel But Ariel reminds them that he and the other spirits are the ministers of destiny and nothing can wound them The steel of which their swords are made of may cut the wind or water which being divided always closes up again Even supposing that such things may be possible it is quite impossible that their swords will cut one feather in their plume They are incapable of being wounded by any sword of man Hence it is foolish on their part to attempt to strike at Ariel and his fellow-spirits

For which foul deed

(Line 72-75)The powers delaying not forgetting

haveIncensed the seas and shores yea all the

creatures Against your peace

Ariel enters like a harpy and remaining invisible tells Alonso Sebastian and Antonio that he and other harpies are the agents of Destiny appointed to carry out her decrees He tells them that their punishment for the crime against Prospero which has been so long deferred is now to fall upon them He reminds them that they had expelled Prospero from Milan and set him and his innocent child adrift on the sea and that the sea had paid them back for their sin by the shipwreck and by the calamities they have suffered He tells them that the powers above which did not forget this mean treachery but only deferred the punishment have now engaged the seas and the shores and all living beings including him and his comrades against them The very elements and supernatural agency Ariel adds have taken up the avenging of their crime against Prospero

the action of fire It may cut the wind or water which being divided always closes up again

The sword will not work against the spirits and the harpy because they are the ministers of destiny and nothing can wound them nor it will cut a single feather in their plume

(c )What is the meaning of lsquodowlrsquo in the last line

The term lsquodowlrsquo means a filament or the smallest part of a feather In this context Ariel in disguise of harpy says that their sword cannot even damage the smallest filament of their (Arielrsquos and other spirits) feathers as they are incapable of being wounded by any sword of man

(d) What does the speaker remind the listeners about

Ariel in disguise of harpy reminds Alonso the King of Naples Sebastian Alonsorsquos brother and Antonio the present Duke of Milan and the treacherous brother of Prospero as they being three men of sin He even reminds them that their punishment for their crime against Prospero which has been so long deferred now falls upon them He reminds them that they have expelled Prospero from Milan and has set him along with his innocent infant daughter adrift on the sea So the sea has paid them back for their sin by their shipwreck and the calamities they have suffered since then The harpy rebukes Alonso of his sin that has incensed the Gods and has deprived him of his son as a punishment

(e) How do they respond

When Ariel in disguise of a harpy reminds Alonso Sebastian and Antonio of their past misdeeds and sin Alonso has a look of terror and confusion in his eyes He utters the words of sincere repentance wrung out of his conscience-stricken heart It appears to him that all the elements of nature the sea-waves the wind and the thunder proclaiming a loud voice in the name of Prospero and the crime Alonso has committed against him They are calling upon him to repent There is a deep storm raging in Alonsorsquos breast and the echoes of that storm are ringing in his ears like a clear note of wind-instrument A note of denunciation of Alonsorsquos crime leaves him much humbled and penitent and confirms his belief that his son is lost forever But Sebastian and Antonio shows some courage instead of repentance They wish to kill the spirits or devils if it appears

3

Of my instruction hast thou nothing bated (Line 85-93)

In what thou hast to say So with good life

And observation strange my meaner ministers

Their several kinds have done My high charms work

And these mine enemies are all knit upIn their distractions They now are in my

powerAnd in these fits I leave them while I visitYoung Ferdinand whom they suppose is

drownedAnd his and mine loved darling

Methought the billows spoke and (Line 96-99)

told me of itThe winds did sing it to me and the

thunderThat deep and dreadful organ-pipe

pronouncedThe name of Prosper It did bass my

trespass

These are the words of contrition coming from Alonso Ariel has driven him to a deep repentance for conspiring with Antonio against Prospero He now feels a sincere remorse It appears to him that all the elements of nature the sea-waves the wind and the thunder proclaimed with a loud voice the name of Prospero and the crime Alonso had committed against him They are calling upon him to repent There is a deep storm raging in Alonsorsquos breast and the echoes of that storm are ringing in his ears like the clear note of a wind-instrument

Comment These are the words of sincere repentance wrung out of the conscience-stricken heart of Alonso Alonso who is the lesser villain is the first to give way to remorse under the effect of Arielrsquos speech The words of Ariel seem to him to be the voice of conscience speaking to him He is driven to desperation a state in which he might do violence to his life

(a) Identify the speaker State the context

Prospero the ruler of the island is the speaker The famous banquet scene has been enacted very well Ariel and his junior spirits have played their roles excellently Prospero is glad to say words of praise for them(b) In what way the speakerrsquos instructions have been carried out

According to Prosperorsquos instructions a banquet was presented before the King of Naples and his companions when they were tired and hungry Just when they were preparing to eat the feast the banquet was suddenly removed by exercising supernatural powers All this was done by Ariel Prosperorsquos chief assistant and a powerful spirit

Ariel not only made the feast disappear but also delivered his speech blaming the King and his two companions for their past wicked deeds He warned them to repent for their misdeeds or suffer forever on that uninhabited island

(c) Who are referred to as lsquomeaner ministersrsquo What have they done

Prospero refers as lsquomeaner ministersrsquo to his other lesser spirits who were assisting Ariel in presenting a scene before the kingrsquos party They entered the scene to the accompaniment of music They assumed several strange shapes and brought in a banquet Then they danced about it with gentle actions of salutations thus inviting the King and others to eat the feast

These spirits play their role again when Ariel in the shape of a harpy quits the scene These shapes enter again and dancing with mocking gestures carry away the table

(d) Who are the speakerrsquos enemies What has happened to them

King of Naples Alonso his brother Sebastian and the present Duke of Milan Antonio (Prosperorsquos own brother) are Prosperorsquos enemies With the turn of events they have all been washed ashore on the island which is ruled by Prospero the great magician Actually this happened after the shipwreck caused by a storm which was raised by Prospero with the purpose of bringing these people to his island Prosperorsquos spirits have already confused and terrified these enemies and they are under Prosperorsquos control He can treat them as he likes

(e) What does he say about Ferdinand Explain what is meant by ldquohellip his and mine darlingrdquo

Prospero knows that Alonsorsquos son prince Ferdinand is alive though his father thinks that the prince has been drowned

Prospero refers to his daughter Miranda who is dear to him She is also very dear to Prince Ferdinand who has fallen in love with her They are waiting to be married soon for which they have received Prosperorsquos consent

4

ALONSO O it is monstrous monstrous (Line 95-102)

Methought the billows spoke and told me of it

The winds did sing it to me and the thunderThat deep and dreadful organ-

pipe pronouncedThe name of Prosper It did bass

my trespassTherefore my son ithrsquo ooze is

bedded andIll seek him deeper than eer

plummet soundedAnd with him there lie mudded

(a) In what way does Alonso express his horror when his conscience is awakened by Arielrsquos words

When Alonsorsquos conscience is awakened by Arielrsquos words he expresses his horror at what he has heard He gets the feeling that the waves of the ocean the wind and the loud thunder have spoken to him and uttered the name of Prospero Because of being reminded of his crime in a very loud and rough voice he comes to realize that he has lost his son for his past misdeeds

(b) What does Alonso imagine about his son What does Alonso want to do in his desperate state

Alonso imagines that his son is lying in the mud at the bottom of the sea He feels desperate that he wants to drown himself in the ocean deeper than the plumb-line has ever gone He wants to lie with his son at the bottom of the sea

(c) How do Sebastian and Antonio want to face the evil spirits

Sebastian says that he is not at all afraid of what the harpy has said and that he is prepared to fight any number of such monsters if they appear before him only one at a time Antonio says that he would support Sebastian in the fight against the fiendsyyy

(d) Why does Gonzalo ask Adrian to follow the three men

Gonzalo tells Adrian that all the three men namely Alonso Sebastian and Antonio are in a wild and reckless mood The thought of the heinous crime of which they are guilty has begun to torment their minds So he asks Adrian to follow those three men without loss of time and prevent them from doing anything which the turmoil in their minds might lead them to do

(e) What opinion do you form of Alonso from the above extract

Alonso who is the lesser villain is the first to give way to remorse under the effect of Arielrsquos speech The words of Ariel seem to him to be the voice of conscience speaking to him He is driven to desperation a state in which he might do violence to his life

Subject =Accounts

Ac-12 15420 topic-pL Appropriation ac

PROFIT AND LOSS APPROPRIATION ACCOUNT

MEANING AND PREPARATIONProfit and Loss Appropriation Account is merely an extension of the Profit and Loss Account of the firm The profit of the firm has to be distributed amongst the partners in their respective profit sharing ratio But before its distribution it needs to be adjusted All Adjustments like partnerrsquos salary partnerrsquos commission interest on capital interest on drawings etc are made in this account These adjustments will reduce the amount of profit for distribution This adjusted profit will be distributed amongst the partners in their profit sharing ratio To prepare it at first the balance of Profit and Loss Account is transferred to this account The journal entries for the preparation of Profit and Loss Appropriation Account are given below

1 for transfer of the balance of Profit and Loss Account to Profit and Loss Appropriation Account

(a) In case of Net Profit

Profit and Loss Ac helliphelliphelliphelliphellipDrTo Profit and Loss Appropriation Ac(Net Profit transferred to Profit and Loss Appropriation Ac)

(b)In case of Net Loss

Profit and Loss Appropriation Achelliphelliphellip DrTo Profit and Loss Ac(Net Loss transferred to Profit and Loss Appropriation Ac)

2 for Interest on Capital

For transferring on Interest on CapitalProfit and Loss Appropriation Achelliphelliphellip DrTo Interest on Capital Ac(Interest on capital transferred to Profit amp Loss Appropriation Ac)

3 for Interest on Drawings

For transferring Interest on Drawings Interest on Drawings Achelliphelliphelliphelliphelliphellip DrTo Profit and Loss Appropriation Ac(Interest on drawing transferred to Profit amp Loss Appropriation Ac)

4 For Partnerrsquos SalaryFor transfer of partnerrsquos SalaryProfit and Loss Appropriation Achelliphellip DrTo Salary Ac(Salary transferred to profit amp Loss Appropriation Ac)

5 For Partnerrsquos CommissionFor transferring commissionProfit and Loss Appropriation Achelliphelliphellip DrTo Commission Ac(Commission transferred to Profit and Loss Appropriation Ac)

6 For Transfer of agreed amount to General ReserveProfit and Loss Appropriation Ac helliphellipDrTo General Reserve Ac(Transfer to General Reserve)

7 for share of Profit or Loss appropriation(a) If ProfitProfit and Loss Appropriation Achelliphellip DrTo Partnerrsquos CapitalCurrent Ac(Profit transferred to capitalcurrent Ac)(b) If LossPartnerrsquos Capital Current Achelliphelliphelliphellip DrTo Profit and Loss Appropriation Ac(Loss transferred to capitalcurrent Ac)

THE FORMAT OF PROFIT AND LOSS APPROPRIATION

Profit and Loss Appropriation Account for the year endedhelliphelliphelliphellip

Particulars Amount Particulars Amount

To PL Ac (loss) By pL Ac (profit)

To Interest on capital BY Interest on drawings

To partner`s commission by Partner`s capital Ac ( loss)

To Partner`s salary To Interest on partner`s loan To General Reserve To Partner`s Capital AC (Profit)

Subject= Economics

MOVEMENT ALONG THE DEMAND CURVE (CHANGE IN QUANTITY DEMANDED)In law of demand you have already studied the inverse relationship between price and quantity demanded When quantity demanded of a commodity changes due to change in its price keeping other factors constant it is called change in quantity demanded It is graphically expressed as a movement along the same demand curve There can be either a downward movement or an upward movement along the same demand curve Upward movement along the same demand curve is called contraction of demand or decrease in quantity demanded and downward movement along the same demand curve is known as expansion of demand or increase in quantity demanded

Extention of demandd

price (rs)p A

B Extentionp1 d

Q Q1

Quantity demanded ( in units)

Contraction of demandd

p2 Ccontraction

p APrice (Rs)

d

Q2 Q

Quantity demanded (in units)

Explanation of movement of demand A fall in price from OP to OP1 leads to increase in quantity demanded from OQ to OQ1 (expansion of demand) resulting in a downward movement from point A to point B along the same demand curve DD When Price rises from OP to OP2 quantity demanded falls from OQ to OQ2 (contraction of demand) leading to an upward movement from point A to point C along the same demand curve DD

  • Activity Series of Metals
    • Drawbacks of Rutherfordrsquos model of atom
      • Electromagnetic radiations
      • Properties of electromagnetic radiations
      • Characteristics of electromagnetic radiations
        • Plancks Quantum Theory-
        • Photoelectric effect
          • Intext Questions
            • Logic gates
            • Digital systems are said to be constructed by using logic gates These gates are the AND OR NOT NAND NOR EXOR and EXNOR gates The basic operations are described below with the aid of truth tables
            • AND gate
            • Example
Page 14:  · Web viewSubject. Topic. Summary. Execution. English 1 . Chapter 1 naming words . Page 8. Write the names of these pictures:- Person:-1. father. 2.Firefighter 3.doctor 4 ...

(ii) A cup B = 0123456 (iii) A cap B = ϕ (iv) A ndash B = 456 (v)B ndash A = 0123

3 If A = 56789 B = x 3 lt x lt 8 and x isin W and C = x xle5 and x isin N Find (i) A cup B and (A cup B) cup C (ii) B

cup C and A cup ( B cup C)

(iii) A cap B and (A cap B) cap C (iv) B cap C and A cap (B cap C)

Is (A cup B) cup C = A cup (B cup C)

Is (A cap B) cap C = A cap (B cap C)

SolutionA = 56789 B = 4567 C = 12345

there4 (i) A cap B = 456789 and (A cup B) cup C = 123456789

(ii) B cup C = 1234567 and A cup ( B cup C) = 123456789

(iii) A cap B = 567 and (A cap B) cap C = 5

(iv) B cap C = 45 and A cap (B cap C) = 5

Now (A cup B) cup C = 123456789

And A cup ( B cup C) = 123456789 there4 (A cup B) cup C = A cup (B cup C)

Again (A cap B) cap C = 5 and A cap (B cap C) = 5

there4 (A cap B) cap C = A cap (B cap C)

4 Given A = 012345 B = 02468 and C = 0369 Show that (i) A cup (B cup C) = (A cup B) cup C ie the union

of sets is associative (ii) A cap (B cap C) = (A cap B) cap C ie the intersection of sets is associative

SolutionNow B cup C = 0234689 and A cup B = 01234568

there4 A cup (B cup C) = 012345689 and

(A cup B) cup C = 012345689

So (i) A cup (B cup C) = (A cup B) cup C ie the union of sets is associative

Again B cap C = 06 and A cap B = 024

there4 A cap (B cap C) = 0 and (A cap B) cap C = 0

So (ii) A cap (B cap C) = (A cap B) cap C ie the intersection of sets is associative

Physics Chapter 2 Physical Quatites and Measurements

Here We Will Do Some QuestionsRelated To Chapter 2

A density bottle has a marking 25 mL on it It means that

1 the mass of density bottle is 25g

2 the density bottle will store 25 ml of any liquid in it

3 the density bottle will store 25 ml of water but more volume of liquid denser than water

4 the density bottle will store 25 ml of water but more volume of a liquid lighter than water

Solution 2 the density bottle will store 25 ml of any liquid in it

COMPUTER CHAPTER-2Spreadsheet Functions and Charts

SELECTING RANGE IN ROWSCOLUMNSWHEN TWO OR MORE CELLS ARE SELECTED IT IS CALLED A RANGEA RANGE OF CELLS CAN BE FORMED IN TWO WAYS--a) SELECTING RANGE BY USING THE MOUSEb) SELECTING RANGE BY USING THE KEYBOARD

Q1)WRITE THE STEPS TO SELECT PARTIAL RANGE IN A ROW

Ans)THE STEPS ARE-6 SELECT THE ROW7 BRING THE CELL POINTER TO THE DESIRED

LOCATION FROM WHERE YOU WANT TO START YOUR SELECTION

8 CLICK THE LEFT MOUSE BUTTON AND KEEP DRAGGING TO YOUR RIGHT TILL YOU REACH THE LAST CELL TO NE SELECTED

RELEASE THE MOUSE BUTTON

GEOGRAPHY Asia

CLIMATE

Asia experiences great extremes of climate Jacobabad in the Sind province of Pakistan is one of the hottest places in the WorldVerkhoyansk in Siberia is one of the coldest places in the WorldCherrapunji and Mawsynram in India are two wettest places in WorldArabia Tibet Gobi and Mongolia are extremely dry regionsFactors Affecting Climate of Asia-The factors influencing the climate of Asia are-

Factors Affecting Climate of Asia-Thoroughly read the table in page number 60

Latitudinal extent

Continentality

Relief features

Presence of low pressure trough

Jet streams

English Language The Sentence A complex sentence contains one independent clause and at least one dependent clause The dependent clause in a complex sentence is introduced with subordinating conjunctions or relative pronouns

Commonly Used Subordinating Conjunctions-Time after before while when since untilCause And Effect because now since as in order that soOpposition although though even though whereas while in spite ofCondition if unless only if whether or not even if in case(that)

Commonly Used Relative Pronouns-Who whose whom which whoever whomever whichever that

Class IXSubject Topic Summary Execution

1-BENGALI(2ND LANGUAGE)

ldquo বঙগভমিরপরমিrdquo াইকেলধসদনদতত

আচেগর পর উততর পচো-১ ২ ৩ এং নীচের পর টি াহির কা- ৪মহিbকাও গচেনা দেগা পহিচে অমত হরচে- ক) কার দো দেকান কহিতার অং( ) কতা দেক পরসঙগ কী উহিকতটির তাৎপযC আচোনা কচেরা৫দেসই ধনয নরকচে দোচেক যাচে নাহি ভচে মচেনর মহিeচের সাচেসচে সCন ক) কহির কায C ার উচেf(য হিক হিছ কহি কন কহিতাটি দেচেন) কহি কার কাচেছ হিমনহিত কচেরচেছনগ) কহি এই পহিথীচেত কাচের ধনয মচেন কচেরনঘ) কহি হিক রকম অমর তাাভ করচেত ান

Hindi 2nd lang

काकी(कतिसयारामशरण गपत)

इस कहानी म लखक न यह बतान का परयास निकया ह निक बचच अपनी मा स निकतना परम करत ह शयाम अबोध बालक ह वह अपनी मा क मरन क बा- उसन अपनी मा क कतिलए बहत रोया बा- म उस पता चला निक उसकी मा राम क घर चली गई ह आकाश म उडती हई पतग -खकर उस हरष हआ निक पतग क दवारा वह अपनी मा को नीच उतारगा इसक कतिलए वह अपनी निपता की जब स -ो बार सवा रपया निनकालकर पतग और -ो मोटी सी मन वाली अपन भाई स काकी एक कागज पर कतिलखवा कर पतग म कतिशव का दि-यानिनकालकर पतग और -ो मोटी सी मन वाली अपन भाई स काकी एक कागज पर कतिलखवा कर पतग म कतिचपका दि-याभोला और शयाम कोठरी म रससी बाधनी रह थ तभी उसक निपता करोध म आकर उन स पछ निक कया उनकी जब स रपया निनकाला हभोला डर क मार बताया निक शयाम इस पतग क दवारा अपनी काकी को राम क यहा स उतारना चाहता हनिवशशवर(शयाम क निपता)न फटी पतग उठाकर -खी तो उस पर काकी कतिलखा थावह हत बजिa होकर वही खड रह गएउनहोन सोचा निक मन अपन पतर को मारा जोनिक अनजान और निन-dरष थावह अपनी मा कोनिकतना पयार करता ह

उस दि-न बड सवर शयाम की नी- खली तो -खा निक घर भर म कोहराम मचा हआ ह

क) घर म कोहराम कयो मचा हआ था शयाम को कया लगा

ख) काकी को ल जात समय शयाम न कया उपदरव मचाया

ग) काकी क बार म उस कया बताया गया कया सतय उस कतिछपा रहा

घ) वह बठा-बठा शनय मन स आकाश की ओर कयोकरता

उततरक) शयाम की मा का -हात हो गया था इसकतिलए

घर म कोहराम मचा हआ था शयाम की लगा निक उसकी मा सफ- कपडा ओढ हए भमिम पर सो रही ह

ख) लोग जब उमा यानी शयाम की मा को उठाकर ल जान लग तब शयाम न बडा उपदरव मचाया लोगो क हाथ स झठ करवा उमा क ऊपर जा निगरा और बोला काकी सो रही ह उस कहा ल जा रह हो

ग) काकी क बार म बजिaमान लोगो न उस निवशवास दि-लाया निक उसकी का निक उसक मामा क यहा गई ह लनिकन सतय अमिधक दि-नो तक कतिछपाना रह सका आसपास क अबोध बालको क मह स यह बात परकट हो गई निक उसकी मा का -हात हो गया ह

घ) कई दि-नकई दि-न लगातार रोत-रोत उसका रोना तो शान हो गया पर उसक ह-य म शोक भर गया था वह चपचाप बठा आकाश की और टाका करता निक शाय- उसकी काकी कही दि-ख जाए

ldquoदि-न उसन ऊपर आसमान म पतग उडती -खी न जान कया सोच कर उसका निहर-य एक-म खिखल उठाrdquo

क) निकसन पतग ऊपर उडत -खी और वह कयो खश हआ

ख) उसन अपन निपता स कया कहा उनका कया उतर थाश

ग) उसन निफर कया निकया और निकसन उसकी सहायता की

घ) उसकी योजना कया थी उततर -क) शयाम न एक दि-न आसमान म पतग उडती

-खी तो उसन सोचा निक पता आसमान म राम क यहा जाकर रकगी वही पर मरी काकी ह यह सोचकर वह बहत खश हआ

ख) उसन अपन निपता स कहा काका मझ एक पतग मगा -ो उसक निपता न भटक हए मन क भाव स कहा निक मगा -ग यह कह कर उ-ास भाव स वह कही और चल गए पतग नही आई

ग) उसन चपचाप निवशशवर क टगहए कोट स एक चवननी निनकाल ली और सखिखया -ासी क लडक भोला की सहायता स एक पतग मगवानी भोला उसकी बराबर उमर का ही था

घ) उसकी योजना यह थी निक वह अपनी पतग को आकाश म राम क यहा भजगा और उस पतग क सहार उसकी काफी नीच उतर जाएगी इस योजना पर उस परा निवशवास था इसकतिलए वह और भोला -ोनो यह काम करन म लग गए

Continue to nexthelliphellipEVS CHAPTER - 1

(UNDERSTANDING OUR ENVIRONMENT)

Sustainable development

The development that meets the needs of the present without compromising the ability of future generations to meet their own needs is called Sustainable development

Sustainable societies ndash

An environmentally sustainable community is one that meets the current and future basic resource needs of its people in a just and equitable manner without compromising the ability of future generations to meet their basic needs

Q ) What are Eco Villages

Ans - Eco village are the urban or rural communities of people who strive to integrate a supportive social environment with a low impact way of life

Q ) To ensure sustainable development the depletion of renewable resources should not take place at a rate faster than their regeneration Justify your answer

Ans ndash Renewable resources do not have a fixed quantity - more can always be

generated However if the rate of use exceeds the rate of renewal - that is the

source is used more than its being recreated - its continued use will become

used up faster than it can regenerate

To promote sustainable society the following things need to be done ndash

1 Using renewable energy sources 2 By improving the quality of human

health 3 By promoting sustainable agriculture 4 By forming ecovillage

it will eventually be entirely depleted So Toensure sustainable development the depletion of

renewable resources should nottake place at a rate faster than their regeneration

Q ) What do you mean by Sustainable societies

Ans - Sustainable societies are defined as towns and cities that have taken steps to remain healthy over the long term These communities value healthy ecosystems use resources efficiently and actively seek to retain and enhance a locally based economy Sustainable development concerns everybody in a society

Q ) What are the effects of pollution on human health

Ans ndash Some health problem occurs due to air pollution are ndash

Respiratory diseases Cardiovascular damage Fatigue headaches and anxiety Irritation of the eyes nose and throat Damage to reproductive organs Harm to the liver spleen and blood Nervous system damage

Some health problem occurs due to water pollution are ndash

Typhoid Cholera Dysentry Jaundice

Some health problem occurs due to noise pollution are ndash

Fatigue headaches and anxiety High blood pressure Hearing damage

Physics Motion in 1D First go through previous notes Now here we will solve some numerical related to that

Question 3What information about the motion of a body is obtained from the displacement-time graphSolution 3From displacement-time graph the nature of motion (or state of rest) can be understood The slope of this graph gives the value of velocity of the body at any instant of time using which the velocity-time graph can also be drawn

Question 4(a)What does the slope of a displacement-time graph represent(b)Can displacement-time sketch be parallel to the displacement axis Give a reason to your answerSolution 4(a) Slope of a displacement-time graph represents velocity(b) The displacement-time graph can never be parallel to the displacement axis because such a line would mean that the distance covered by the body in a certain direction increases without any

increase in time which is not possible

Chemistry Language of Chemistry

How to balance a chemical equationThere are two methods of balancing an equation(i)Hit and trial method(ii)Partial equation methodBalancing by hit and trial methodThis method consists of counting the number of atoms of each elements on both sides and trying to equalize themTake the following steps(i)Count the number of times (frequency) an element occurs on either side(ii)The element with the least frequency of occurrence is balanced first(iii)When two or more elements have the same frequencythe metallic element is balanced firstExample-1 On heatinglead nitrate decomposes to give lead dioxidenitrogen dioxide and oxygenPb(NO3)2rarrPbO+NO2+O2

In this equationLead occurs twiceNitrogen occurs twiceOxygen occurs four timesSince lead is a metalbalance it firstThe number of atom of lead is equal on the two sidestherefore it needs no balancingNow balance nitrogenOn the reactant sidethere are two atoms of nitrogenwhile on the product side oneSomultiply the product containing nitrogenon the product sideby two Pb (NO3)2rarrPbO+2NO2+O2Nowthe number of oxygen atoms on the reactant side 6while on the product sideit is 7Somultiply the entire equation by 2except oxygen to get balanced equation2Pb(NO3)2rarr2PbO+4NO2+O2Multiplication by 2 is done only when atoms of all the elements except one element are balanced and the unbalanced atom occurs separately at least once and also there is a difference of only one such atom

Math Topic AlgebraChapter

Factorisation

Study item Difference of two squares a2 ndash b2 = (a+b) (a-b)1) (i) 4x2ndash 25y2

= (2x) 2 ndash (5y) 2= (2x + 5y) (2x - 5y)

(ii) 9x2 ndash 1= (3x)2ndash(1)2= (3x + 1)(3x ndash 1)

2) (i) 150 ndash 6a2= 6(25 ndash a2)= 6(5)2 ndash(a)2= 6 (5 + a) (5 ndash a)

(ii) 32x2 ndash 18y2=2(16x2 ndash 9y2)=2(4x)2 ndash (3y)2= 2(4x + 3y)(4x - 3y)3)(i) (x ndashy )2 ndash 9 = (x ndash y )2 ndash (3)2= (x ndash y + 3) (x ndash y ndash 3)(ii) 9(x + y) 2ndash x 2= (3)2(x + y)2 ndash (x)2=3(x + y)2 ndash (x)2= (3x +3y ) 2ndash(x)2= (3x + 3y + x)(3x +3y ndash x)= (4x + 3y) ( 2x + 3y )

Commercial studies

Basic accounting terms

Today I will give you some questions from the previous study material

Questions1) Define accounting2) What do you mean by debit and

credit

3) Explain the types of account4) Define the following terms

a) Assetsb) Capitalc) Purchased) Debtorse) Transactions

5) Name the types of accounts given below

a) Krishnas accountb) Machinery accountc) Royalty accountd) Salary accounte) Furniture accountf) Audit fee account

Economics Revision Today I will give you some revision questions

Questions1) What do you mean by the terms

rdquowantsrdquo2) Write the difference between

consumer goods and producer goods

3) Define the term utility 4) Explain the different types of utility5) Define

a) Total utilityb) Marginal utility

Subject Eng Literature (The Merchant of Venice ndash William Shakespeare)Topic Act I Scene 3 Lines 1 to 48 (Shylock hellip Cursed be my tribe if I forgive him) Date 16th April 2020 (5th Period)

[Students should read the original play and also the paraphrase given in the school prescribed textbook]Summary Questions amp Answers

This scene takes place in Venice and we are introduced to the rich Jew Shylock Bassanio and Shylock are talking and Bassanio tells Shylock that he wants a loan of three thousand ducats for three months on the personal security of Antonio

o Shylock feels glad because he will be able to bind down Antonio by means of a bond on account of the loan but he tells Bassanio that all the fortunes of Antonio being invested in the merchant ships on the sea it is difficult to depend upon his credit Even under such circumstances Shylock is willing to advance the money on the personal security of Antonio

o Bassanio then invites Shylock to dine with him Shylock says that he is prepared to do anything with the Christians but not eat or drink or pray with them

o While Bassanio and Shylock are talking Antonio appears on the scene Shylock does not seem to take any notice of Antonio but goes on brooding within

(1) SHYLOCK Ho no no no no- my meaning in (Line 15-26)saying he is a good man is to have you understand me that he is sufficient Yet his means are in suppositionhe hath an argosy bound to Tripolis another to the Indies I understand moreover upon the Rialto he hath a third at Mexico a fourth for England and other ventures he hath squanderd abroad Butships are but boards sailors but men there be land-rats and water-rats land-thieves and water-thieves I mean pirates and then there is the peril of waters winds and rocks The man is notwithstanding sufficientmdashthree thousand ducats mdashI think I may take his bond

(a) Who is talking in the beginning of this scene What does Bassanio want from Shylock How does Shylock feel

In the beginning of the scene Bassanio and Shylock are talking to each other Bassanio wants to borrow three thousand ducats from Shylock for three months on the security of Antonio Shylock feels glad at heart that he will get the opportunity of binding Antonio with a bond(b) What risks does Shylock weigh in advancing the money

Shylock says that Antonio has invested all his capital in trading by sea-going ships But the ships are made of wood and the sailors of those ships are ordinary human beings The wood can

himself how he hates Antonio because of his being a Christian because he abuses Shylock in public places Shylock decides that if ever he can get Antonio to his advantage he will teach him a lesson

come to harm and men can commit mistakes and thus the capital invested in ships may be lost Then there are other dangers The goods loaded on the ships can be damaged by rats and thieves which are found both on land and water The ships can also be harmed through sea-storms submerged rocks etc(c) What two important functions does this scene have

The scene has two important functions First it completes the exposition of the two major plot lines of the play Antonio agrees to Shylockrsquos bond ndash three thousand ducats for a pound of flesh and second and more important dramatically this scene introduces Shylock himself In this scene Shakespeare makes it clear at once why Shylock is the most powerful dramatic figure in the play and why so many great actors have regarded this part as one of the most rewarding roles in all Shakespearean dramas(d) Where does this scene take place What kind of treatment has Antonio been giving to Shylock What does Shylock say when Bassanio invites him to dine with him

The action of this scene takes place in Venice Antonio has been in the habit of behaving harshly with Shylock ndash spitting on his beard and footing him like a stranger cur When Bassanio invites Shylock to dine Shylock says that he is prepared to do anything with the Christians but not eat and drink or pray with them

(2) SHYLOCK How like a fawning publican he looks (Line 38-48)I hate him for he is a Christian

But more for that in low simplicity

He lends out money gratis and brings downThe rate of usance here with us in VeniceIf I can catch him once upon the hipI will feed fat the ancient grudge I bear him

He hates our sacred nation and he railsEven there where merchants most do congregateOn me my bargains and my well-won thriftWhich he calls interest Cursed be my tribeIf I forgive him

(a) What is the context in which these words are spoken and what is the idea expressed in it

These remarks are made by Shylock when he sees Antonio coming along after Bassanio told him that the merchant will be his surety for the bond The above mentioned passage reveals Shylockrsquos hatred for Antonio Shylock says that he hates Antonio because he is a Christian and also because he gives loan without taking interest on them thereby bringing down the rate of interest in Venice(b) Explain the meaning of the phrase lsquoa fawning publicanrsquo

The phrase lsquoa fawning publicanrsquo refers to Roman tax collector It is a term of contempt and hatred on the lips of a Jew lsquoFawning Publicansrsquo were Roman tax-gatherers whose ordinary bearings towards the Jews was bullying but whose false pose of lsquohumility and contritionrsquo is touched upon in the parable in New Testament(c ) What light does the above passage throw on the character

of Shylock

The above mentioned speech of Shylock reveals him to be a wicked character having an extreme greed for wealth His intense hatred for Antonio is unjustified He hates Antonio only because he is a Christian and because he lends money without taking any interest on it thereby adversely affecting Shylockrsquos business of lending money on high interest(d) What information do you gather about Antonio from the above given lines

Shylockrsquos statement throws a valuable light on the character of Antonio Antonio appears to be a good Christian and a good human being He helps the people in need by lending them money without charging any interest on it He is a man of simple and good nature This very goodness makes him Shylockrsquos enemy(e) What does Shylock debate within himself and when To whom are the lines mentioned above addressed to

When Bassanio asks the Jew to lend him three thousand ducats on Antoniorsquos surety Shylock begins to debate within himself as to how he should exploit the opportunity of a business deal with his old enemy Antonio

The lines mentioned above are not addressed to anyone The lines are a soliloquy ie a speech made by a character to himself and not meant to be heard by the other characters present

Class XSubject Topic Summary ExecutionEnglish

LiteratureThe Blue Bead 2nd part

Things took a turn for the worst and all of a sudden a crocodile attacked the woman biting on the womanrsquos leg At that moment Sibia got up sprinted grabbed the hay fork and stabbed the crocodile in the eye with all her power Immediately the crocodile let go and went away Sibia saw a small blue bead lying by the river she grabbed it Since she was poor she didnrsquot have necklace Shersquod always wanted one like the other women now she could make one with the blue bead After that she went home and told her mother all about it

Hindi 2nd

Langबड घर की बटी( मशी परमच-)

lsquoबड घर की बटी कहानी का उददशय मधयम वग की घरल समसया को सलझा कर सगदिठत परिरवार म मिमल जलकर परम स रहन का स-श -ना हघर म शानित सथानिपत करन की जिजमम-ारी नारी की होती ह यदि- नारी समझ-ार ह उसम धय और परिरवार क परनित परम ह तो कोई भी घटना परिरवार को निवघदिटत नही कर सकती या कहानी परिरवार को सगदिठत करत हए परम सौहा- स एक दसर की भावनाओ को समझ करउनका सहयोग करत हए जीवन यापन करन की पररणा -ती हमशी परमचदर जी न इस कहानी म सय परिरवार का परनितनिनमिधतव निकया ह यह कहानी बनी माधव सिसह जो गौरी पर क जमी-ार क उनक -ो पतरो की हशरी कठ लाल निबहारीशरीकात का निववाह एकजमी-ार घरान की पतरी आन-ी स हआ थाआन-ी न ख- को ससराल क वातावरण म ढाकतिलया थाएक दि-न आन-ी का अपन -वर लाल निबहारी स झगडा हो जाता ह -ोनो भाई एक दसर स अलग होन की कोकतिशश करत हसभी बह आन-ी न अपन मधर वयवहार स लाल निबहारी को

ldquoइन नतर निपरय गणो को बीए-इनही -ो अकषर पर नयोछावर कर दि-या था इन -ो अकषर न उनक शरीर को निनबल और चहर को कानित ही बना दि-या थाldquo

क) परसतत पकतियो म निकस वयकति क बार म कहा गया ह

ख) इन पकतियो म कौन स नतर निपरय गणो क बार म कहा गया ह

ग) बीए की निडगरी परापत कर लन पर भी उपय वयकति क सवभाव की कया निवशरषता थी

घ) यह नतर निपरय गण निकस वयकति म निवदयमान थ उसक वयकतितव की कया निवशरषता थी

उततर ndashक) परसतत पकति म गौरी पर गाव क जमी-ार

क बड बट शरीकात क बार म कहा गया ह उसन बहत परिरशरम और उ-म क बा- ba की निडगरी परापत की थी अब वह एक -फतर

घर छोडकर जान स रोक कतिलयाइस पर बनी माधव सिसह न कहा निक बड घर की बटी ऐसी ही होती ह जो निबगडा काम बना लती ह अतः शीरषक साथक ह बड घर की बटी आन-ी ह

म कमचारी थाख) भरा हआ चहरा चौडी छाती और डटकर

खाना आदि- एक सबजी ल जवान क गण मान जात ह परत शरीकात न इनही नतर निपरय गणो को अपनी पढाई पर नयोछावर कर दि-या था

ग) बीए की निडगरी परापत कर लन पर भी उपय वयकति(शरी कठ की शारिररिरक तौर पर निनबल और चहर स कानित ही लगत थ इतना ही नही वह मानकतिसक तौर पर भी निपछड हए थ पाशचातय सामाजिजक कथा उस घणा एव पराचीन सभयता का गणगान उनकी निवचारधारा क परमख अग थ

घ) यह नतर निपरय गण गौरीपर गाव क जमी-ार क छोट बट लाल निबहारी सिसह म निवदयमान थ वह सजीलाजवान था और भस का दध शर दध वह सवर उठकर पी जाता था

ldquoयही कारण था निक गाव की लललन आए उनकी निन-क थी कोई कोई तो उह अपना शतर समझन म भी सकोच ना करती थी सवय उनकी पतनी को इस निवरषय म उनस निवरोध थाldquo

क) उपय पकति म इस वयकति क बार म कहा गया ह

ख) गाव की लललन आए उनकी निन-ा कयो निकया करती थी

ग) उनकी पतनी का कया नाम था उनह निकस निवरषय म अपन पनित क निवरa था और कयो

घ) इस कहानी का कया उददशय ह Continue to next helliphelliphellip

Bengali 2nd Language

ফ ফটক না ফটক( কহিতা )

পর) ldquo(ান াধাচেনা ফটপাচেথ পাথচের পাড হিচেয় এক কাঠচোটটা গাছ কহিকহি পাতায় পার ফাটিচেয় াসচেছldquoক) কার দো দেকান কহিতার অং( ) lsquo(ান াধাচেনা ফটপাচেথ পাথচের পাডহিচেয়lsquo চেত কী দোঝাচেনা চেয়চেছ গ) আচো য অংচে(lsquo এক কাঠচোটটা গাছ lsquoচেত কী দোঝাচেনা চেয়চেছ ঘ) ldquoকহিকহি পাতায় পার ফাটিচেয় াসচেছldquo ----- একথার পরকত অথC কী উততর ) ক) আচো য অং(টি পর যাত কহি সভা4 মচোপাধ যাচেয়র দো lsquoফ ফটক না ফটকrsquo কহিতার অং()কহি সভা4 মচোপাধ যায় হিছচেন দেপরচেমর কহি দেপরমচেক নানা ভহিঙগমায় হিতহিন ফটিচেয় তচেচেছন দেপরম মানচের স মচেতC র সঙগী কহিতার কহিতায় এক রb সb হচেয়র দেপরম াগরচেনর কথা চেচেছন (ান অথCাৎ দেযাচেন দেকান রস দেনই দেযাচেন দেকান মহিনতা দেনই অথ তার মধ দেযও দেপরম থাকচেত পাচের একথাই কহি তচে ধরচেত দেচেয়চেছন একটি মানচে4র মচেন দেযাচেন দেকামতার দেকান সথান দেনই পাথচেরর মচেতা হিনরসতার মচেনর মধ দেযও দেয দেপরম আসচেত পাচের দেস কথাই কহি চেচেছনগ)নারীচের যথC দেপরচেমর ছহি এই কহিতায় অকপচেট উচেঠ এচেসচেছ কহি এই কহিতায় কাটচোটটা গাছ কথাটি যার কচেরচেছন নারী দেয দেপরম দেথচেক হিতাহিত এং দেসই দেপরম সঠিক সমচেয় না পাওয়ার ন য দেপরম সমপচেকC হিচেr4 গৈতরী য় দেপরচেমর দেয গৈহি(ষট য মাধযC য সরসতা দেকামত এই সমসতর হিপরীত যথা রbতা শষকতা কচেঠার তা পরভহিত দোঝাচেত এক কাঠচোটটা গাছ কথাটি যার কচেরচেছনঘ) এাচেন এক নারীর যথC দেপরচেমর কথা হিনহিCপত ভাচে চেচেছন কহি অসমচেয় নারীর ীচেন দেপরম দেচেগচেছ এতহিন তার হয় রb কচেঠার হিছ দেপরচেমর অভাচে ঠাৎ দেসই শষক মরভহিমচেত সচের আভাস এচেসচেছ দেপরম দেযন 4Cার স(ীত তাই পরায় মত গাচেছ কহিকহি পাতা গহিচেয় উচেঠচেছ

Biology Chapter - 01Controlling Air Pollution

Today we will discuss how we control air pollution from domestic combustion

Q1Describe any five ways of reducing air pollution from domestic sources bull The number of pollutants in the air is verylarge and we always try to control them byfollowing ways

i) Solar cooker and solar heater It use no fuel reduce damage of environment by fuel use or reducing deforestation It maintains coolness of house It releases very less orno oil gas or grease

ii) Piped natural gas (PNG) It emits very less by products into the atmosphere As it isdistributed through pipe lines so there iscontinuous supply of fuel is possible

iii) Liquefied Petroleum Gas (LPG) It hasa higher heating value LPG doesntcontain sulphur so it burns a lot cleanerenergy sources It releases very less oralmost no fume in air

iv) Electricity based cooking Emission free cooking alternative for urban dwellers causeselimination of adverse health impactsofindoor air pollution It helps to avoid theinconveniences associated with procurement of LPG

v) Biogas It contains 75 methane whichmakes it an excellent fuel It burns without smoke and biogas plant leaves no residue like ash in wood charcoal etc Thus it isaclean fuel

Economics

Factors of Production

Today firstly we would recall the last class for 5 mins and then we would proceed with the further topics of the chapter

The concept meaning of land characteristics of land and importance of land to be repeated for the absentees as well as the students who were there in the class the previous day

Today we will start with the last portion of land before it the meaning of land to be repeated onceAs by now we all know that

Questions1What do you mean by productivity of landAnswer By productivity of land we mean the capacity of a piece of land to produce a crop

Thus it refers to the average output per unit of landSay per acre per hectare etc= (OutputArea of land)

2 What are the factors influencing the productivity of landAnswer

Natural factors Productivity of land is largely determined by the natural

Land is defined to include not only the surface of the earth but also all other free gifts of nature(for example mineral resources forest resources and indeed anything that helps us to carry out the production of goods and services but is provided by nature free of cost)

We will move on to the last portion of land by discussing Productivity of Land

By productivity of land we mean the capacity of a piece of land to produce a crop

Thus it refers to the average output per unit of land

Say per acre per hectare etc= (OutputArea of land)

With this we shall proceed further with the main factors that determine the productivity of land

Natural factors Human factors Improvements on land Location of land Organisation Ownership of land Availability of capital Proper use of land State help

Note economic development of a country depends upon the quality of its land If the land is fertile it will quicken the pace of development of the country

qualities of land such as fertility etc

Human factors Land cannot produce anything by itself Man has to apply labour on it to produce for himself So productivity of land depends on the knowledge and skills of workers

Improvements on land production of land is affected by land development measures like provision of well or tubewell irrigation proper drainage

State help The government of a country especially less developed country can play a vital role in improving the agricultural productivity by providing better irrigation facilities

Organisation Productivity of land also fdepends upon the way how the factors of production like labour and capital are organised

In order to increase productivity trained workers modern implements scientific methods good seeds are all essential

3 lsquoImproved technology affects the productivity of landrsquo Explain this statement with the help of suitable example Answer Use of improved technology raises the productivity of land Example By using HYV seeds chemical manures and modern machines per hectare output increases

Physics Force (Summary)

Question Write the expression for the moment of force about a given axisSolutionsThe expression for the moment of force is given byMoment of force about a given axis = Force times perpendicular distance of force from the axis of rotationQuestion What do you understand by the clockwise and anticlockwise moment of force When is it taken positiveSolutionsIf the effect on the body is to turn it anticlockwise moment of force is called the anticlockwise moment and it is taken as positive while if the effect on the

body is to turn it clockwise moment of force is called the clockwise moment and it is taken as negative

Math Topic Commercial Mathematics

Chapter Goods and services Tax

Study item Some solved sums from exercise ndash 1 A retailer buys a TV from a wholesaler for Rs 40000 He marks the price of the TV 15 above his cost price sells it to the consumer at 5 discount on the marked price If the sales are intra ndash state and the rate of GST is 12 find

(i) The marked price of the TV(ii) The amount which the consumer pays for the TV(iii) The amount of tax (under GST) paid by the retailer to the central

Government(iv) The amount of tax (under GST) received by the State Government

Solution As the sales are intra- state sale and the rate of GST 12 So GST comprises of 6 CGST and 6 SGSTTherefore a retailer buys a TV from a wholesaler for Rs 40000Therefore the amount of GST collected wholesaler from the retailer or paid by retailer to wholesalerCGST = 6 of Rs 40000 = Rs(6100 times40000) =Rs 2400SGST = 6 of Rs 40000 = Rs (6100 times 40000) =Rs 2400Therefore wholesaler will pay Rs 2400 as CGST and Rs 2400 as SGSTTherefore amount of input GST of retailer Input CGST = Rs 2400 and input SGST = Rs 2400Again the retailer marks the price of the TV 15 above his cost price(i) The marked price of the TV

= Rs 40000 + Rs 40000times15= Rs 40000 + Rs 40000times 15100= Rs 40000 + Rs 6000Rs 46000But the retailer sells it to consumer at 5 discount on the marked priceCost price after discount = Rs 46000 ndashRs46000times 5100 =Rs 46000 ndashRs 2300= Rs 43700Therefore the amount of GST collected retailer from consumer or paid by consumer to retailerCGST = 6 of Rs 43700 =Rs ( 6100 times43700)Rs 2622SGST = 6 of Rs 43700 = Rs (6100 times 43700) =Rs 2622Amount of the output GST of retailer Output CGST = Rs 2622 and output SGST = Rs 2622

(ii) The amount which the consumer pays for the TV= cost price of TV to consumer + CGST paid by consumer + SGST paid by consumer= Rs 43700 + Rs 2622 + Rs 2622= Rs 48944

(iii) The amount of tax (under GST ) paid by the retailer to the central Government=CGST paid by retailer = output CGST ndash input CGST=Rs 2622 ndash Rs 2400=Rs 222

(iv) The amount of tax ( under GST ) received by the State Government = SGST paid by wholesaler + SGST paid by retailer= Rs 2400 + output SGST ndash input SGST=Rs 2400 + Rs 2622 ndash Rs 2400=Rs 2400 + Rs 222= Rs 2622

Commercial studies

Stakeholders Today I am going to give some revision questions from the previous study material

Questions1) State the two expectations of

employees from a business concern2) Give two distinctions between

stakeholder and shareholder3) Give two difference between

internal stakeholders and external stakeholders

4) Give two expectations of suppliers from a business organisation

5) Who is a stakeholder in commercial organisations

Chemistry Periodic Table

Merits of Mendeleevrsquos Periodic law are as follows - 1He grouped the elements on the basis of atomic mass 2 He left gaps for undiscovered elements like Gallium Scandium germanium Also he left a full group vacant for undiscovered inert gases 3 He could predict proportions of several elements on basis of their position in periodic table like Ga Sc etc 4He could predict errors in atomic weights of some elements like gold platinum etc

Anomalies in Mendeleevrsquos Periodic law are as follows - 1 Position of isotopes could not be explained 2 Wrong order of atomic masses could not be explained

For example- as Arnur atomic mass 40 come first and K with low atomic mass (30) should come later but k should be placed first

According to Bohrrsquos Modern Periodic table properties of elements are periodic functions of their atomic numbers

So when elements are arranged according to increasing atomic numbers there is periodicity in electronic configuration that leads to periodicity in their chemical properties

It consists of horizontal rows (Periods) Vertical column (Groups)

There are 7 period and 12 groups in this long form of periodic table

Ist period has 2 elements IInd period has 8 elements IIIrd period has 8 elements IVth period has 18 elements Vth period has 18 elements VIth period has 32 elements VIIth period hs rest of elements

Note - The number of valence electrons in atom of elements decides which elements will be first in period and which will be last

In group- 1 to 2 gp and 13 to 17 contain normal elements 3 to 12gp ndash transition elements 57 to 71 - lanthanides 89 to 103 - Actinides

Left hand side ndash metals Right hand side ndash nonmetals

Note- Hydrogen element has been placed at top of Ist group Electronic configuration of H is similar to alkali metal as both have 1 valence electron

V electron of gp I element -- 1 V electron of gp 2 element -- 2 V electron of gp 13 element -- 3 V electron of gp 14 element -- 4 V electron of gp 15 element -- 5 V electron of gp 16 element --6 V electron of gp 17 element -- 7 V electron of gp 18 element -- 8

English 1 Transformation of sentences

Sentences A sentence is a group of words which makes complete sense

Exercise 2Change the following sentences from

a Assertive sentencesb Imperative sentencesc Interrogative sentencesd Exclamatory sentences

Sentences can be changed from one grammatical form to another without changing the meaning of the sentence This is known as transformation of sentences

assertive to interrogative1 Nobody would like to be a fool

Who would like to be a fool2 Their glory can never fade

When can the glory fade3 Nobody can control the wind

Who can control the wind4 It matters little if I die

What though I die5 No man can serve two masters

Can any man serve two masters

Exercise 3Interchange of assertive and Exclamatory sentences

1 She leads the most unhappy lifeWhat an unhappy life she leads

2 This is indeed an interesting bookWhat an interesting book this

3 He is a very great manWhat a great man he is

4 It is a very lame excuseWhat a lame excuse

5 It is sad that she died so youngAlas she died so young

Class XISubject Topic Summary Execution

Hindi 2nd lang

पतर परम(परमचदर) पतर परम कहानी म एक निपता की इचछाओ का वणन निकया गया ह अपन बड पतर परभ -ास स निपता चतनय -ास का निवशरष परम था निपता को उसक जनम स ही बडी-बडी आशाए थी उसम दसर बट कतिशव-ास की अपकषा स- उतसाह की मातरा अमिधक थी वह उस इगलड भजकर बरिरसटर बनाना चाहत थभागय का खल भी बडा निनराला ह बीए की परीकषा क बा- वह बीमार पड गया डॉकटरो न भी जवाब - दि-या थाचतन -ास जी बहत ही कजस थ बवजह पस खच करना नही चाहत थ अगर गारटी मिमलती तो शाय- पस खच भी कर -त परत गारटी नही थी परिरणाम सवरप उनक बट का -हात हो गयाजब बट को समशान ल जा रह थ तो वहा काफी शोर गान बजान हो रह थ पछन पर पता चला निक निकसी निपता निपछल तीन साल स निबमार था और उसक ईलाज म रपया पानी की तरह बहाया पर ठीक नही हए परत उसक बट को तनिनक भी अफसोस नही था उसका कहना था उसन कोकतिशश तो कीयह -खकर चतनय-ास जी को आतम निगलानी हईतभी स उनका म परिरवतन हआ और बट का भोज काफी धमधाम स निकयाऔर वहइस पशचाताप की आग म जलत रह औला- स बढकर पसा नही होता ह इस बात को समझन म उनह काफी व लग गया

hellipContinue to next

BENGALI(2ND LANGUAGE)

পরথমঅধযায়-ঠাকরারীনদরনাথঠাকর

নয়ন দোচের হিমাচেররা া নাচেমই হিযাত হিছচেন ায়ানার উাররণ সবরপ নয়ন দোচের ারা হিা (াচেকর হিা হিচেতন এছাাও দেকান উৎস উপচেb রাহিতর দেক হিন করার উচেfচে(য তারা সযC হিকরচেরণ রনয পরীপ জবাহিচেয় তাচেত রপার হির 4Cরণ করচেতন ঠাকরা এই নয়ন দো হিমারচের দে(4 ং(ধর হিছচেন হিমাররা ায়ানার ষটানত পর(Cন কচের তারা হিনঃসব এই হিমাহিরর দে(4 ং(ধর গৈকাস নদর রায়চেৌধরী গৈকাস া নয়ন দোচের সমসত সমপহিতত ঋচেরণর াচেয় হিহিx কচের অহি(ষট যা আচেছ তাচেত হিপত

ইার হিপতার মতয ইচে পর নয়নচোচের ায়ানার দেগাটা কতক অসাধাররণ শরাদধ (াহিনতচেত অহিনতম ীহিপত পরকা( কহিরয়া ঠাৎ হিনহিয়া দেগ- ক) কার দো দেকান গচেলপর অং() কতা দেক ইার চেত কাচেক দোঝাচেনা চেয়চেছ গ) পরসঙগ কী কতার কতয পহিরসফট কচেরা

পরচে4র যাহিত রbা করা সমভ নয় তাই হিতহিন পতরচেক হিনচেয় ককাতায় সাস শর কচেরন গলপ কথচেকর আহিথCক অসথা নয়ন দোচের হিমাচের দেথচেক সমপরণC আাা কথচেকর হিপতা হিনচের দেষটায় অথC উপাCন করচেতন া উপাহিধ াচেভর নয তার াসা হিছনা আর দেসই কারচেরণ কথক তার একমাতর উততরাহিধকার চেয় তার হিপতার পরহিত কতজঞ কথক দো পা হি(চেচেছন হিনচের পরারণ ও মান রbার নয উপচেযাগী অথC হিনা দেষটায় পরাপত চেয়চেছন- এটাই তার কাচেছ পরম দেগৌরচের হি4য় চে মচেন কচেরন কাররণ (নয ভাণডাচের গৈপতক ায়ানার উজজব ইহিতাস অচেপbা দোার হিসeচেকর মচেধয গৈপতক দেকামপাহিনর কাগ তার কাচেছ অচেনক দেহি( মযান

TO BE CONTINUED

উ- ক) আচোয অং(টি রীনদরনাথ ঠাকচেরর দো ঠাকরা গচেলপর অং() কতা চেন আচোয গচেলপর গলপ কথকইার চেত নয়ন দোচের হিমাহিরর দে(4 ং(ধর গৈকাস ার কথা া চেয়চেছ গৈকাস া নয়ন দোচের সমসত সমপহিতত ঋচেরণর াচেয় হিহিx কচের অহি(ষট যা আচেছ তাচেত হিপত পরচে4র যাহিত রbা করা সমভ নয় তাই হিতহিন পতরচেক হিনচেয় ককাতায় সাস শর কচেরনগ) গৈকাস ার হিপতার মতযর পর নয়ন দোচের হিমাহিরর অহিসততব হিপত য় কচেয়কটা উৎস ও শরাদধ- (াহিনতচেত হিমাহিরর দে(4 কহিটক যয় চেয় হিগচেয় এচেক াচের দে(4 চেয় যায় তন তাচের গC করার মত আর হিকছই হিছ না-দেসই পরসচেঙগ এই উহিকত নয়নচোচের হিমাচেররা া নাচেমই হিযাত হিছচেন ায়ানার উাররণ সবরপ নয়নচোচের ারা হিা (াচেকর হিা হিচেতন এছাাও দেকান উৎস উপচেb রাহিতরচেক হিন করচেত হিগচেয় তারা সযC হিকরচেরণর নয পরীপ জবাহিচেয় তাচেত রপার হির 4Cরণ করচেতন তাই দেসকাচের ায়ানা দেহি(হিন সথায়ী চেত পারত না হিহিভনন উৎস শরাদধ- (াহিনতচেত সাধযা হিতহিরকত র করার নয হিমাহির হিহিকচেয় দেযত হ হিতC কা হিহি(ষট পরীচেপর দেত দেযমন অলপকাচের মচেধয হিনঃচে(4 চেয় যায়-নয়নচোচের হিমারচের অসথা তাই চেয়হিছ এই কারচেরণই কথক নয়নচোচের হিমারচের গা ভরা আমবর সয করচেত পারতনা

Physics Dimensional Analysis (Summary)

Q Find the dimensions of consts ab in relation

p=(bminusxlowastx)at

where p is the power x is the distance and t is time

Ans From principle of homogeneity dimension of b x2 are same Dim of b = dim of x2 = [L2] = [ML2T0]Dim of a = dim of ( b- x2)dim of (pt) = [M0L2T0][ML2T-2] [T-1] [T] = [M-1L0T2]

Chemistry Atomic Structure Drawbacks of Rutherfordrsquos model of

atom a According to Rutherfordrsquos model of atom electrons which are negativelycharged particles revolve around the nucleus in fixed orbits Thusb theelectrons undergo acceleration According to electromagnetic theory of Maxwell a charged particle undergoing acceleration should emitelectromagnetic radiation Thus an electron in an orbit should emitradiation Thus the orbit should shrink But this does not happenc The model does not give any information about how electrons aredistributed around nucleus and what are energies of these electrons Isotopes These are the atoms of the same

Properties of electromagnetic radiationsa Oscillating electric and magnetic field are produced by oscillating charged particles These fields are perpendicular to each other and both areperpendicular to the direction of propagation of the waveb They do not need a medium to travel That means they can even travel invacuum

Characteristics of electromagnetic radiationsa Wavelength It may be defined as the distance between two neighbouring crests or troughs of

element having the same atomicnumber but different mass numbere g 1H11H21H3

Isobars Isobars are the atoms of different elements having the same massnumber but different atomic numbere g 18Ar40 20Ca40

Isoelectronic species These are those species which have the same numberof electrons

Electromagnetic radiationsThe radiations which are associated withelectrical and magnetic fields are called electromagnetic radiations When anelectrically charged particle moves under acceleration alternating electricaland magnetic fields are produced and transmitted These fields aretransmitted in the form of waves These waves are called electromagneticwaves or electromagnetic radiations

wave as shown It is denoted by λb Frequency (ν) It may be defined as the number of waves which passthrough a particular point in one secondc Velocity (v) It is defined as the distance travelled by a wave in onesecond In vacuum all types of electromagnetic radiations travel with thesame velocity Its value is 3 times10 8m sec-1 It is denoted by v

d Wave number Wave number is defined as the number of wavelengths per unit lengthVelocity = frequency timeswavelength c = νλ

Plancks Quantum Theory- o The radiant energy is emitted or absorbed not continuously but discontinuously in the form of small discrete packets of energy called lsquoquantumrsquo In case of light the quantum of energy is called a lsquophotonrsquoo The energy of each quantum is directly proportional to the frequency of the radiation ie E α υ or E= hυ where h= Planckrsquos constant = 6626 x 10-27 Js o Energy is always emitted or absorbed as integral multiple of this uantum E=nhυ Where n=1234Black body An ideal body which emits and absorbs all frequencies is calleda black body The radiation emitted by such a body is called black body radiation

Photoelectric effectThe phenomenon of ejection of electrons from thesurface of metal when light of suitable frequency strikes it is calledphotoelectric effect The ejected electrons are called photoelectrons

Biology Chapter - 02Systematics and Five Kingdoms

Scientists divide the whole living organisms into two kingdom first and ultimately by five kingdom at last

In the earlier systems of classifications organisms are divided into kingdom plantaeand kingdom animalia on the of presenceof cell wall their modes of nutrition and movements

Some problem arise like fungi share manycharacteristic withplant despite their heterotrophic nutrition bacteria protozoa areunicellular present in both kingdom Toovercome this third kingdom Protista isintroduced which include

unicellularorganisms But there is also another

problem Allunicellular organisms are not similar kind The cellular structure of prokaryotes is verydifferent from that of other organismsEukaryotes possess a true nucleus and allcell organelles that are not present inprokaryotes So the fourth kingdom Monerais introduced which include unicellular prokaryotes (bacteriaamp blue green algae)

bull Still some problem arise in kingdomplantae

So in 1969 R H Whittakar proposedanew five kingdom System of classification

i) Kingdom Monera - unicellular prokaryotes

ii) kingdom Protista - unicellular eukaryotes

iii) Kingdom Fungi - uni or multicellular fungi with cell wall but without chlorophyll

iv) Kingdom Plantae - Multicellular Plants

v) Kingdom Animalia - Multicellular Animals

EVS Chapter 1 ndash Modes of Existence

An agricultural society

An agricultural society also known as an agrarian society is a society that constructs social order around a reliance upon farming More than half the people living in that society make their living by farming

People in an agricultural society generally lead a more settled lifestyle than those in nomadic hunter-gatherer or semi-nomadic pastoral societies because they live permanently near the land that is farmed Agricultural settlements tend to develop in areas of convenience near bodies of water which is used for both crops and transportation or along trade routes Not everyone in an agricultural society is a farmer Some people make a living trading or making and selling goods such as tools used for farming

Another way to define an agrarian society is to see the total amount of production in a nation In an agrarian society cultivating the land is the main source of wealth Such a society can recognize other means of subsistence and work habits but emphasizes the importance of agriculture and livestock Agrarian societies have existed in various parts of the world for 10000 years and continue to exist today They have been the most common form of socio-economic organization for most of recorded human history

Q) Write the features of agricultural society

Ans - Structure and Features of Agrarian Society1 Occupational Structure

An agrarian society is generally associated with the domestication of plants and animals The domestication of plants means farming and that of animals means herding Often there is mixture of farming and the use of such domesticated animals as cow goat and sheep

2 Forms of Land Ownership in Agrarian SocietiesGenerally there are landlords supervisory farmers cultivators and share croppers The landholders own the land but do not work on it They let it out for sharecropping The supervisory farmers are those who live by having their land cultivated by hired labourers The cultivators cultivate the land for themselvesThe share-croppers are those who live by tilling other peoplersquos land or a crop-sharing basis The artisans own their means of production and produce by their own labour in their homesteads

3 Village Community System An agrarian society is highlighted by

the institution of village community system The agrarian economy made fixed dwelling houses necessary Living close together for protection and co-operation and living nearer to the land gave birth to agricultural villages The village is not only the residential place of farmers it is also the social integrator

4 Minimal Division of Labour Another structural feature of agrarian society is a minimal division of labour Except for the basic division founded on age and sex differences there are few specialized roles There is only one predominant type of occupation ie domestication of plants and animals For all the people the environment physical as well as social is the same

5 Role of Family The farm family is of the patriarchal type the father is the final arbiter in most of the familyrsquos major decisions The life of ail men and women is merged in family life Since there are not many special organizations family is the only organisation to perform the tasks of aid and protection

6 Sense of Unity The members of an agrarian society exhibit a strong in-group feeling Since the whole of their social lives is wrapped up in a society which is physically economically and socially homogenous they are inclined to view the entire outside world as an out group

7 Informal Social Control An agrarian society is regionally divided into villages In a village community the force of traditional mores is more dominant than in the urban community In the village everybody is known to everybody The members in a village community help each other and share the joy and sorrows of each other Crime in an agrarian society is rare

8 Simplicity and Uniformity Life of the people in an agrarian society is marked by simplicity and uniformity Their main occupation is agriculture which largely depends upon the vagaries of nature An agrarian society is a religious society

Math Compound angles Compound angles The algebraic sum of two or more angles is called a compound angle If A B C be three angles then A+B B+C C+A A-B B-C A-C A+B-C etc are compound angles In this chapter we shall discuss the trigonometrical ratios of compound angles Theorem 1 If A B and A+B are all pisitive acute angles theni) sin( A+B) = sin A cos B + cosA sinBii) cos(A+B) = cosA cosB- sinA sinBTheorem 2If A and B are positive acute angles and AgtB theni) sin(A-B) = sin A cosB- cos A sinBii) cos(A-B) = cos A cos B+ sin A sin BTo prove that i) sin(A+B) sin (A-B) = sin2 A - sin2 B = cos2 B- cos2 A

Example 1 Prove that tan70deg=2tan50deg+tan20degSolutiontan70deg = tan(50deg + 20deg)Or tan70deg=(tan 50deg+tan 20deg)(1-tan50degtan20deg) or tan70deg (1 ndash tan 50deg tan20deg) = tan50deg+tan20degor tan70deg= tan70deg tan50deg tan20deg+ tan50deg + tan20deg = cot20deg tan50deg tan20deg + tan50deg + tan20deg = 2 tan50deg+ tan20degExample 2 If A + B = 45deg show that (1 + tanA) (1 + tanB) = 2Solutiontan(A + B) =( tan A + tan B) (1 - tan

ii) cos(A+B) Cos(A-B) = cos2 A- sin2 B = cos2 B -sin2 AProof i) LHS= sin(A+B)sin(AminusB) [Recall sin(αminusβ)=sinαcosβminuscosαsinβ And sin(α+β)=sinαcosβ+cosαsinβ]= (sinAcosB+cosAsinB)times(sinAcosBminuscosAsinB)= sin2Acos2Bminuscos2Asin2B [Recall sin2α+cos2α=1 From above we can then assume correctly that sin2α=1minuscos2α AND cos2α=1minussin2α] = sin2A(1minussin2B)minussin2B(1minussin2A) = sin2Aminussin2Asin2Bminussin2B+sin2Asin2B = sin2Aminussin2B= 1-cos2A-(1-cos2B) = cos2 B- cos2 A = RHSii)LHS= cos (A+B) cos(A-B) [ cos(A+B) = cos AcosB- sinAsinBCos(A-B) = cosAcosB+ sinAsinB]= cos2 A Cos2 B- sin2 A Sin2 B= cos2 A( 1-sin2 B) - (1- cos2 A) sin2 B= cos2 A- cos2 A sin2 B- sin2 B+ cos2 A sin2 B=cos2 A- sin2 B=1- sin2 A-(1-cos2 B) = cos2 B- sin2 A= RHSTangent formulae for compound anglesi)tan (A + B) = tan A + tan B1-tan A tan Bii)tan (A ndash B) = tan A-tan B1+tan A tan Biii) cot (A + B) = cot Acot B-1cot A+cot B(viii) cot (A ndash B) = cot Acot B+1cot B-cot A

A tan B) Or 1= (tan A+ tanB) (1-tan A tanB) Or tanA + tanB + tanA tanB + 1 = 1 + 1Or tanA (1 + tanB) + (1 + tanB) = 2Or (1 + tanA) (1 + tanB) = 2Example 3 Find the value of sin 15degSolution sin 15deg= sin(45deg-30deg) = sin45degcos 30deg- cos45degsin30deg =(1radic2) (radic32) -(1radic2) (12) = (radic3-1) 2radic2Example 4 If sin A = 1 radic10 and sin B = 1 radic5 where A and B are positive acute angles then what is A + B SolutionWe know that sin (A + B) = sin A cos B + cos A sin B= [1 radic10] [radic(1 minus 1 5)] + [1 radic5] radic(1 minus 1 10)= [1 radic10] [radic4 5] + [1 radic5] [radic9 10]= [1 radic50] times (2 + 3)= 5 radic50 = 1 radic2

sin (A + B) = sin π 4rArrHence A + B = π 4Example 5 If A + B = 225o then find [cot A] [1 + cotA] times [cot B] [1 + cot B]Solution[cot A] [1 + cotA] times [cot B] [1 + cot B] = 1 [(1 + tan A) times (1 + tan B)]=1 [tan A + tan B + 1 + tan A tan B] [ tan (A + B) = tan225o]∵

tan A + tan B = 1minus tan A tan BrArr= 1 [1 minus tan A tan B + 1 + tan A tan B]= 1 2

COMMERCE

CLASSIFICTION OF HUMAN ACTIVITIES-ECONOMIC AND NON-ECONOMIC

Firstly we shall recall the previous class for 5 mins especially for the absentees and for also the rest of the students who were there

Today at first we briefly discuss the earlier portions of the chapter

1Business-It includes all those economic activities which are concerned with production and exchange of goods and services with the object of earning profit Example A factory shop beauty parlour also business enterprises

2Profession ndashThe term profession means an occupation which involves application of specialized knowledge and skills to earn a living For Example Chartered Accountancy medicine law tax consultancy are example of professions

Questions1What are the main features of ProfessionAnswer The main features of a profession are as follows a Specialised body of knowledge-Every profession has a specialised and systematised body of knowledge b Restricted entry- Entry to a profession is allowed only to those who have completed the prescribed education and have the specialised examination c Formal education and training ndashA formal education and training is given to the person who wants to acquire the professional

3Employment-Employment mean an economic activity where people work for others in exchange for some remuneration (salary)The persons who work for others are called lsquoemployeesrsquo The persons or organizations which engage others to work for them are called lsquoemployersrsquoEg A doctor working in a hospital is employment as he is working for a salaryA lawyer may serve as a law officer in a bank

With this we shall proceed with the features of both Profession amp Employment

The main features of a profession are as follow

a Specialised body of knowledge b Restricted entry c Formal education and training d Professional association e Service motive f Code of contact

The main features of an employment are as follows

a In employment a person works for others called employer

b An employee provides personal service

c There is a service agreement or contract between the employee and the employer

d The employee has to obey the order of the employer

e No capital investment is made by the employer

Various examples of Employment are as follows

aA teacher teaching in a school or collegeb An engineer employed in Municipal Corporation of DelhicAn accountant working in the accounts department of a companydA doctor working in a hospital

Note In all the above examples of employment the individual who is involved in each example is working as an employee for a salary under an employer

qualification(MBBSCALLB)d Service motive ndashProfessionals are expected to emphasis service more on their clients rather than economic gain f Code of Conduct-The activities of professionals are regulated by a code of conduct

2 What are the main features of EmploymentAnswer The main features of an employment are as followsa In employment a person works for others called employerb An employee provides personal servicec There is a service agreement or contract between the employee and the employerd The employee has to obey the order of the employere No capital investment is made by the employer

3 Give various Professions and their respective Association are given below

Professions

Professional

Professional association

Medical profession

Doctor Medical Council of India

Law profession

Lawyers Bar Council of India

Accounting Profession

Chartered

The Institute of Chartered Accounts of India( ICAI)

Engineerin Engineers The

g Profession

institute of Engineers (India)

Accounts Basic accounting terms

Today we will give you some questions from the previous study material

Questions6) Define accounting7) What do you mean by debit

and credit8) Explain the types of account9) Define the following terms

a) Assetsb) Capitalc) Purchased) Debtorse) Transactions

10) Name the types of accounts given below

a) Krishnas accountb) Machinery accountc) Royalty accountd) Salary accounte) Furniture accountf) Audit fee account

Economics Basic Economic ConceptsSub topic

UTILITY

Before starting todayrsquos class we shall recall the last class which was about UTILITY AND THE FEATURES OF UTILITY

Now we shall proceed with the further topics of the chapter

Todayrsquos topic from the chapter lsquo Basic Economic Conceptsrsquo will be TOTAL UTILITY amp MARGINAL UTILITYNow let us quickly revise the concept of utility with an example ie goods and services are designed because they have an ability to satisfy human wantsThis feature of being able to satisfy human wants is termed as utility For example we derive utility from WiFi services as it gives us satisfaction by connecting us to our friends and family through social media here consumers derive utility from WiFi services

From the above concept we shall start with todayrsquos topicEconomists have defined TOTAL UTILITY (TU) as the total satisfaction obtained by consuming a given total amount of a good and serviceFor example the total satisfaction obtained from eating 10 mangoes is the total utility of 10 mangoes

MARGINAL UTILITY (MU) is the additional satisfaction derived from each additional unit

Questions1 What is Total Utility (TU)

Answer Total Utility (TU) is the

aggregate of the utility that a consumer derives from the consumption of a certain amount of a commodityTU=MU1+MU2++MUn

2 What is Marginal UtilityAnswer

Marginal Utility (MU) is the additional made to the total utility as consumption is increased by one more unit of the commodityMU= TUn ndashTUn-1

NoteOften economists tend to

subdivide utility into an imaginary unit called UTIL

consumed In this casethe utility obtained from each mango as it is consumed as the MU of that mango It is also defined as the addition made to the total utility when an additional unit is consumed Often economists tend to subdivide utility into an imaginary unit called UTIL

Note As a consumer increases the consumption of a good over period of time the total utility or total satisfaction derived from it increases to appoint and thereafter it decreasesHowever as the consumer keeps on consuming the good the marginal utility or the additional utility derived from it decreases

SubjectBusiness studies

Topic

BUSINESSENVIRONMENT

Summary

Now quickly let us revise the earlier points that we have already done in the last class and let us proceed with the other topics that are there in the chapter

Firstly we will recall the internal and external factors of micro environment and then we shall proceed in details

Meaning and list of internal and external factors

aInternal factorsInternal factors refer to all the factors existing within a business firm The internal factors are considered controllable because the enterprise has control over these factorsFor an example a company can alter its organization structure policies programmes employees physical facilities and marketing mix to suit the changes in the environmentList of internal factors areCorporate culture mission and objectives top management organizations structure company image and brand equity company resources

b External factorsExternal factors refer to those individual and groups and agencies with which a particular business organization comes into direct and frequent contact in the course of its functioningThese individuals and groups are known as STAKEHOLDERS because they have a stake (financial interest ) in the working and performance of the particular business List of external forces (stakeholders)Customers competitors investors suppliersmiddlemen (marketing intermediaries)

Execution 1 What do you mean by internal

factors in micro environmentAnswerInternal factors refer to all the factors existing within a business firm The internal factors are considered controllable because the enterprise has control over these factorsFor an example a company can alter its organization structure policies programmes employees physical facilities and marketing mix to suit the changes in the environment

2 What do you mean by external factors in micro environment

AnswerExternal factors refer to those individual and groups and agencies with which a particular business organization comes into direct and frequent contact in the course of its functioningThese individuals and groups are known as STAKEHOLDERS because they have a stake (financial interest) in the working and performance of the particular business

3Who are stakeholdersSTAKEHOLDERS are individuals and groups who have a stake (financial interest ) in the working and performance of the particular business 4Discuss the internal factors in briefa Corporate CultureThe values beliefs and attitudes of the founders and top management of the company exercise

financers publics

customers

suppliersfinancers

competitors

middlemen

publics

Fig STAKEHOLDERS OF A COMPANY

Apart from micro environment the other main dimension of business environment isMacro environment Macro environment refers to the general environment or remote environment within which a business firm and forces in its micro environment operateA company does not directly or regularly interact with the micro environmentTherefore macro environment is also known as indirect action EnvironmentThe macro environment forces are less controllable than the micro forces

Macro environment consists of the following components

POLITICAL AND LEGAL ENVIRONMENT

ECONOMIC SOCIAL AND ENVIRONMENT

CULTURAL

ENVIRONMENT

TECHNOLOGICAL ENVIRONMENT

a strong influence on what the cmpaany stands for how it does things and what it considers importantbMission and objectivesThe business philosophy and purpose of a comoany guide it prioritiesbusiness strategiesproduct market scope and development scope

cTop management structurethe composition of board of directors the degree of professionalization of management and the organizational structure of a company have important bearing on its business decisions

dPower structureThe internal power relationship between the board of directors and the chief executive is an important factor

eCompany image and brand equityThe image and brand equity of the company play a significant role in raising finance forming alliance choosing dealers and suppliers launching new products entering foreign markets

5 What is Macro environmentAnswerMacro environment refers to the general environment or remote environment within which a business firm and forces in its micro environment operateA company does not directly or regularly interact with the micro environmentTherefore macro environment is also known as indirect action EnvironmentThe macro environment forces are less controllable than the micro forces 6 What are the components of macro environmenta Political and legal environmentb Economic environmentc Social and cultural environmentd Technological environment

BUSINESS FIRM

Fig COMPONENTS OF MACRO ENVIRONMENTPolitical science

Introduction to political science

Comparative politics and itrsquos scope Comparative politics is the second major dimension of political scienceIt is also a very vast area of study and a very large number of political scientists even treat it as an autonomous area of study within the board ambit of political scienceScope of comparative politics-

1 All political structures -Comparative politics includes the study of all structures formalnon formal governmental and extra governmental which are directly or indirectly involved in politics in all the countries of the world

2 Functional studies- Comparative politics seeks to study politics less from the point of view of the legal institutions in terms of their powers and move from the point of view of their functions which constitute the political process and their actual Operation in the environment

3 Study of political behaviour- Another important part of its scope is the study of the actual behaviour of the people in the process of politics

4 Study of similarities and differences- comparative politics also undertakesan analysis of the similarities and differences among political process and functions

5 Study of all political systems -comparative politics seeks to analyse the actual behaviour and performance of all political systems western as well as non western

6 Study of the environment and infrastructure of politics-The study of politics demands a study of the psychological sociological economic and anthropological environment in fact the social environment as a whole in which each political system operates

7 Study of political culture- political culture is composed of attitudesbeliefs emotions and values of a society that relate to the political system or politics

8 Study of political participation- Political participation is a universal processThe only difference is that while in some states it is limited in others it is wider

9 Study of political process- political

Answer the following questions-

What is comparative politics

What are the scope of comparative politics

Homework- learn

processes like decision makingpolicy making judicial process leadership recruitment process and others are always at work in all political systems

The scope of comparative politics is very comprehensive It includes everything that falls within the area of political activity and political process

History CAMBRIDGE VIEW ABOUT

THE PARTITION

AND REFUTATION

OF CAMBRIDGE

VIEW

Cambridge view about the Partition The Cambridge school of historians have interpreted that opposition to partition scheme was made entirely by the elitist groups They hold the view that Lord Curzon planned to partition the Bengal for administrative purposeREFUTATION OFCAMBRIDGE VIEW The Rationalist historians have rejected the interpretations of the Cambridge School of historians on various grounds

1 QUESTION State different views of historians regarding Partition of Bengal

ANSWER Cambridge historians believed that Lord Curzon partitioned Bengal for administrative reasons only and not for the political motive The Middle class elitist group protested because of their petty interest The Hindu zamindars protested as they have to spend more money for managing their estatesThe lawyers of Calcutta High court feared to lose their clientBut according to the nationalist Historians was-

2- The ultimate object of Lord Curzon was to crush the unity of Bengal politicians

3- If Bengal becomes a separate province Bengali speaking 16 million people of western part would become minority under Hindi speaking people of Bihar and Oriya speaking people of Orissa

4- The bureaucrats expected that the protest movement would die down quickly

5- Lord Curzon used the Muslim community in his political game

6- Idealism had great contribution in the protest against partition

7- The people of the every section of society were affected by the partition of Bengal

Computer Science

Numbers Convertion of dcimal number to octal numberThe decimal numeral system is the standard system for denoting integer and non-integer numbers It is the extension to non-integer numbers of the Hindu-Arabic numeral system For writing numbers the decimal system uses ten decimal digits a decimal mark and for negative numbers a minus sign - The decimal digits are 0 1 2 3 4 5 6 7 8 9 the decimal separator is the dot in many countries

The octal numeral system or oct for short is the base-8 number system and uses the digits 0 to 7 Octal is sometimes used in computing instead of hexadecimal perhaps most often in modern times in conjunction with file

permissions under Unix systems It has the advantage of not requiring any extra symbols as digits It is also used for digital displays

Follow these steps to convert a decimal number into octal form

1 Divide the decimal number by 82 Get the integer quotient for the next iteration (if the number will not divide equally by 8 then round down the

result to the nearest whole number)3 Keep a note of the remainder it should be between 0 and 74 Repeat the steps until the quotient is equal to 05 Write out all the remainders from bottom to top This is the solution

For example if the given decimal number is 8453

Division Quotient Remainder

8453 8 1056 5

1056 8 132 0

132 8 16 4

16 8 2 0

2 8 0 2

Then the octal solution is 20405

Subject Eng Literature (The Tempest ndash William Shakespeare) Topic Act I Scene 1 Lines 33 to 67 (End of scene) Date 16th April 2020 (4th Period)

[Students should read the original play and also the paraphrase given in the school prescribed textbook]Summary Questions amp Answers

[SUMMARY OF THE ENTIRE SCENE]

o The play starts with the scene of a severe storm at sea Alonso (King of Naples) Sebastian (Alonsorsquos brother) Ferdinand (Alonsorsquos son) Gonzalo Antonio (the usurping Duke of Milan) are in a ship in the midst of the storm

o The mariners are trying their best to control the vessel from running aground and are totally following the orders of their Master the Boatswain They have scant success

o The mariners become extremely unhappy and annoyed when most of the passengers arrive on the deck thereby hampering their effort to save the ship There is serious confrontation between them and the passengers who are part of the Kingrsquos entourage

o The mariners could not save the ship

SUMMING-UP

(i) Vivid description of the scene which gives a realistic description of terror and confusion of a tropical storm

(ii) Shows Shakespearersquos accuracy of knowledge in describing the naval operations and also matters of seamanship

(1) GONZALO Ill warrant him for drowning (L 45-57)

though the ship were no stronger than a nutshell and as leaky as an unstanched

wenchBOATSWAIN Lay her a-hold a-hold Set her two courses Off to

sea again lay her offMARINERS All lost To prayers to prayers All lostBOATSWAIN What must our mouths be coldGONZALO The king and prince at prayers Lets assist them

For our case is theirsSEBASTIAN Im out of patienceANTONIO We are merely cheated of our lives by drunkards

This wide-chopped rascal - would thou mightst lie drowning the washing of ten tides

(a) What does Antonio say at the insolent manners of the boatswain just before the given passage

Being irritated at the insolent manners of the boatswain just before the given extract Antonio the Duke of Milan calls him a worthless dog son of a woman without any morals an arrogant and disrespectful noisemaker He says that the boatswain deserved to be hanged(b) What statement does Gonzalo repeat about the boatswain

Gonzalo shows his faith that the boatswain is not destined to die by drowning He is destined to be hanged and nothing can alter this decree of destiny He says that even if the ship was as frail as a nutshell the boatswain could not be drowned for his destiny was to be hanged(c) What do the passengers do when they have lost all hope of their survival

When the passengers have lost all hope of survival they take

(iii) The opening scene justifies the title ndash The Tempest

UNANSWERED QUESTIONS

(i) The King always travels with his entire fleet including his soldiers Where were the other ships

(ii) Why was the ship in that area Where was it coming from or going where

(iii) The ship broke apart What happened to those who were in the ship

(We shall get the answer to the above questions as the play progresses)

leave of life with fervent prayers The mariners take their last hearty drink and are ready for death(d) What blame does Antonio put upon the mariners and the boatswain Antonio rebukes the mariners that these drunkards have brought them to the present crisis by neglecting their duties He blames them saying that they are going to lose their lives entirely for the negligence of the boatswain and his fellows(e) What does Antonio say while cursing the boatswain

Antonio gives vent to his wrath upon the boatswain in particular He calls the boatswain a wide-mouthed rascal who deserves to be hanged on the sea-shore at low water mark so that ten tides might wash over his body and take out of him all the liquor that he has been drinking

Class XIISubject Topic Summary ExecutionHistory Topic

1 1935 ACT AND WORKING OF PROVINCIAL AUTONOMYCONGREE AND OTHER MINISTERSSUB TOPIC GOVERNMENT OF INDIA ACT1935

Government of India Act 1935 This act established a lsquoFederation of Indiarsquo made of British Indian provinces and Indian states and provided for autonomy with a government responsible to the elected legislature in every provinceThis act introduced abolition of Diarchy at provinces The entire provincial administration was introduced to the responsible ministers who were controlled and removed by the provincial legislature The provincial autonomy means two things First The provincial governments were wholly responsible to the provincial legislature Secondly Provinces were free from outside control and interference in the large number of matters The act divided the powers between the centre and provinces in terms of three lists- Federal list( for centre) Provincial list (for province) and concurrent list (for both) Residuary powers were given to the viceroy In the election under the government of India Act the Congress swept the poll the mandate of the people came in favour of the congress so far as general Hindu seats were concerned The Congress did not get a single Muslim seates in Bombay CP UP Sind and BengalIn five provinces Congress had yhe clear majority In BengalNWFPAssam and Bombay Congress emerged as a single largest partyOn the other side the performance of the Muslim League was badThus the Congress formed ministers in 7 provinces out of 11 provinces Coalition ministry was also formed in two other provincesOnly BENGAL AND Punjab had non- congress ministries

1 QUESTION What was the main change introduced by the Government of India ActANSWER a) The Act gave more

autonomy to the provinces b) Diarchy was abolished at the

provincial levelsc) The Governor was the head of

the executived) There was a council of

ministers to advise him The ministers were responsible to the provincial legislatures who controlled them The legislature could also remove the ministers

e) The Governors still retained special reserve powers

2 QUESTION Why did the federal scheme introduced by the Government of India Act 1935 never come into operation

ANSWER The Federal structure of the Government of India was to be composed with the Governor General and Council of ministers The Federal legislature was to be Bicameral legislature- The council of states and the House of Assembly The ministers were to be chosen by the Governor general and they were to hold the office during his pleasure

The provinces of British India would have to join the federation but this was not compulsory for the princely states

This federation never materialised because of the lack of support from the required number of

princely statesThis act was refused and

rejected by the princes the Congress and the Muslim League

Thus both Congress and the League participated in the election of 1937 Thus the federal part was never introduced but the provincial part was put into operations

Bengali 2nd

Language

াচেরর পরাথCনা(কহিতা )

াচেরর পরাথCনা কহিতাটি কহি (ঙখ দেঘাচে4র দো আচো য কহিতায় াচেরর পতর হমায়ন কঠিন দেরাচেগ আxানত ার ঈশবর া আললার কাচেছ পরাথCনা কচেরচেছন তার পচেতরর ীন হিফহিরচেয় হিচেত এই কহিতায় ার পচেতরর ীন হিভbা দেচেয়চেছন ারার এমনহিক হিনচের ীন হিসCচেনর হিহিনমচেয় হিতহিন তার দেছচের ীন হিফচের দেপচেত দেচেয়চেছন তার দেছচের এই দেরাচেগর ন য হিতহিন হিনচেচেকই ায়ী কচেরচেছন তার হিনচের করা পাপচেকই হিতহিন ায়ী কচেরচেছন এছাা রানৈনহিতক ও আথCসামাহিক অসথার কথা তচে ধরা চেয়চেছ এই কহিতায় ার তার হিনচের পাপ কমCচেকই ায়ী কচেরচেছ ার অন যায় ভাচে দেপহি((হিকতর মাধ যচেম অপররা য কচেরচেছ আর এই অন যায় কাচের ন যই তার পহিরাচের হিপযCয় এচেসচেছ দে এক পরকার মানহিক নধন ইহিতাচেসর ার হিপতা চেয় সবাভাহিকভাচে ভাচোাসা দে মমতা দেথচেক মকত চেত পাচেরনহিন তাই হিপতা চেয় আললা া ভগাচেনর কাচেছ পতর হমায়চেনর পরানহিভbা দেচেয়চেছন ার আললা া ভগাচেনর কাচেছ াহিনচেয়চেছন তার হিনচের ীন হিসCন হিচেত হিতহিন রাী তার হিহিনমচেয় পচেতরর ীন হিফচের দেপচেত দেচেয়চেছন াচেরর হিপতসভ হিচেকর কথা এই কহিতায় ফটিচেয় দেতাা চেয়চেছ হিপতা পচেতরর হিরাহিরত মান নধচেনর কথা তচে ধরা চেয়চেছ

হিচে(4 হিকছ াইচেনর তাৎপযC১) ldquoদেকাথায় দেগ ওর সবচছয দেৌন দেকাথায় কচেরায় দেগাপন bয়ldquoউততর) াচেরর পতর হমায়ন কঠিন দেরাচেগ অসসথ তাই তার দেযৌন াহিরচেয় যাচেচছ এই দেরাচেগ তাচেক দেগাপচেন কচেরকচের াচেচছ তার সক (হিকত ধীচের ধীচের bয় চেচছ তাই হিপতা চেয় ার আললার কাচেছ হমায়চেনর পরান হিভbা দেচেয়চেছন২) ldquoাগাও (চেরর পরাচেনত পরানতচের ধসর (ন দেযর আান গানldquoউততর) াচেরর পতর হমায়ন কঠিন দেরাচেগ আxানত তাই ার আ দে(াচেক মমCাত (চেরর পচেথ পরানতচের আান গান ধবহিনত দোক দেসই আান গান আললার কাচেছ দেযন চে যায় আললা দেযন এই আহিতC শচেন পচেতরর ীন হিফহিরচেয় দেয় ৩)ldquoনাহিক এই (রীচেরর পাচেপর ীানচেত দেকানই তরারণ দেনই ভহি4চেতরldquoউততর) হমায়চেনর অসসথতার ন য ার হিনচেচেকই ায়ী কচেরচেছন কারন ার অচেনক রা য অন যায় ভাচে কচেরচেছ তাই তার এই পাপ কাচের ন য তার ঘচের আ হিপ এচেসচেছ এই অন যায় কাচের ন য তার মহিকত দেনই তাই ার আললার কাচেছ এই পাপ কাচেযCর ন য bমা পরাথM

Hindi 2ndlang

-ासी(जयशकर परसा-)

-ासी जयशकर परसा- की एक ऐसी कहानी ह जिजसम भारतीय ससकनित और राषटरीयता का सवरगजीतहोता ह इस कहानी म इरावती एक निहद कनया ह जिजस मलअचछो न मलतान की लट म पकडा और -ासी बना दि-या उस 500 दि-न -कर काशी क एक महाजन न खरी-ा दसरी -ासी निफरोजा ह वह गलाम ह निफरोजा को छडान क कतिलए अहम- को 1000 सोन क कतिसकक भजन थ जो अभी तक नही आए थ राजा साहब कठोर होत हए भी निफरोजा को निबना धनराकतिश क कतिलए उस म कर -त ह वनिफरोजा को अहम- को समझान की बात कहत हकहानी क अत म हम -खत ह निक इरा वती और जाटो क सर-ार बलराज का मिमलन होता हअहम- को यa म मार दि-या जाता ह वहा निफरोजा की परसननता की समामिध बनती ह वहा एक फल चढती ह और डीजल आती ह निफरोजा उस समामिध की आजीवन -ासी बनी रहती हलखक अपन उददशय अथात -ास परथा पर परकाश डालन और इस परथा क कारण होन वाल -ातो क दखो को दि-खान म पणता सफल हए ह

helliphellipContinue to next

Biology Reproductio Today we will discuss about vegetative Q1 Name some vegetative propagules

n in Organisms

propagation of plants The process of multiplication in which fragments of plant body function as propagule and develop into new individual is called vegetative propagation The units of such propagation are runner rhizome tuber bulb etc

and the speciesinvolvedVegetative propagules

Parts involved

Bulb StemBulbil BulbilRhizome Stem Runner Stem Tuber Stem Offset Stem Leaf buds Leaves Suckers Stem

Corns Stem stolon

Q2 State advantages of vegetative propagation

i) Rapid methodii) Sure and easy methodiii) Useful in plants that cannot

produce viable seeds or long seed dormancy

iv) Maintains purity of raceQ 3 Banana fruit is said to be parthenocarpic where as turkey is said to be parthenogenetic WhyBanana develops without fertilization from an unfertilized ovary thus is parthenocarpicIn turkey the ovum or female gamete developinto a new chick without fertilization thus isparthgenetic

Q4 Why is water hyacinth is called as a ldquoTerror of Bengalrdquo Water hyacinth can

propagatevegetatively all over the water body in a short per short period of time This resulted increased biochemicaloxygen oxygen demand of water body causing mortalityof fishes It is very difficult to get rid off them Thus known as terror of Bengal

Chemistry

Solid state GENERAL CHARACTERISTICS OF SOLID STATEIn nature the particular state of matter is governed by two opposing forces at given set of temperature and pressure These forces are intermolecular force of attraction and thermal energy If intermolecular force of attraction is high as compared to thermal energy particles remains in closest position

Intext QuestionsQ1 Classify the following solids as crystalline and amorphous Sodium chloride quartz glass quartz rubber polyvinyl chloride Teflon

A1 Crystalline

and hence very less movement in particles is observed In this case solid state is the preferred state of matter

Let us revise the general characteristics of solid

i) Fixed mass volume and shape

ii) Strong intermolecular force of attraction

iii) Least intermolecular space

iv) Fixed position of constituent particles

v) Incompressible and rigid

Q2 what type of interactions hold the molecules together in a polar molecular solid[CBSE 2010]A2 The molecules in a solid are held together by van der Waals forces The term van der Waals forces include hydrogen bonding dipole-dipole attraction and London dispersion forces All molecules experience London dispersion forces In addition polar molecules can also experience dipole-dipole interactions So the interactions that holds the molecule together in polar molecular solid are London dispersion force and dipole-dipole interactionsQ3 Write a feature that will distinguish a metallic solid from an ionic solid [CBSE 2010]A3 Metals are malleable and ductile whereas ionic solid are hard and brittle Metallic solid has typical metallic lustre But ionic solid looks dullQ4 Write a point of distinction between a metallic solid and an ionic solid other than metallic lustre [CBSE 2012]A4 Metals are malleable and ductile whereas ionic solid are hard and brittleQ5 Write a distinguish feature of metallic solid [CBSE 2010]A5 The force of attraction in

solid Sodium chloride Quartz Amorphous solid Quartz glass rubber polyvinyl chloride Teflon Q2 why glass is considered as super cooled liquidA2 Glass shows the tendency to flow at slower rate like liquid Hence they considered as super cooled liquidQ3 why the window glass of old buildings show milky appearance with timeA3 Glass is an amorphous solid Amorphous solid has the tendency to develop some crystalline character on heating Due to heating in day over the number of years glass acquires some crystalline character and show milky appearanceQ4 why the glass panes fixed to window or doors of old building become slightly thicker at bottomA4 Glass is super cooled liquid It has the tendency to flow down very slowly Due to this glass pane becomes thicker at the bottom over the timeQ5 Sodium chloride is a crystalline solid It shows the same value of refractive index along all the direction TrueFalse Give reasonA5 FalseCrystalline solid shows anisotropy in properties That is it shows different values for the given physical property in different direction All the crystalline solids show anisotropy in refractive index Therefore sodium chloride will show different values of refractive index on different directions

Q6 Crystalline solid are anisotropic in nature What does this statement means

between the constituent particles is special kind of electrostatic attraction That is the attraction of positively charged kernel with sea of delocalized electronsQ6 which group of solid is electrical conductor as well as malleable and ductile [CBSE 2013]A6 Metallic solidQ7 why graphite is good conductor of electricity although it is a network (covalent solid)A7 The exceptional property of graphite is due to its typical structure In graphite each carbon is covalently bonded with 3 atoms in same layer The fourth valence electron of each atom is free to move in between different layersThis free electron makes the graphite a good conductor of electricity

[CBSE 2011]A6 Anisotropy is defined asrdquo Difference in properties when measured along different axis or from different directionsrdquo Crystalline solid show different values of some of the physical properties like electrical resistance refractive index etcwhen measured along the different directions The anisotropy in crystalline solid arises due to the different arrangement of particles in different directions

Math Function Composition of functions Think of an industrial plant that produce bottles of cold drinks first there is the operation (or function) f that puts the cold drink inside the bottle followed by the opeartion g that close the bottle with the capThis leads to the following definitionDefinition Let f A rarr B and g B rarr C be two functions Then the composition of f and g denoted by gof is defined as the function gof A rarr C given by gof(x) = g(f (x)) forall x isinA

Definition A function f X rarr Y is defined to be invertible if there exists a function g Y rarr X such that gof = IX and fog = IY The function g is called the inverse of f and is denoted by f -1

Thus if f is invertible then f must be one-one and onto and conversely if f is one-one and onto then f must be invertible This fact significantly helps for proving a function f to be invertible by showing that f is one-one and onto specially when the actual inverse of f is not to be determined

Example 1 Let f 2 3 4 5 rarr 3 4 5 9 and g 3 4 5 9 rarr 7 11 15 be functions defined as f(2) = 3 f(3) = 4 f(4) = f(5) = 5 and g (3) = g (4) = 7 and g (5) = g (9) = 11 Find gofSolution We have gof(2) = g (f(2)) = g (3) = 7 gof(3) = g (f(3)) = g (4) = 7gof(4) = g (f(4)) = g (5) = 11 and gof(5) = g (5) = 11Example 2 Find gof and fog if f R rarr R and g R rarr R are given by f(x) = cos x and g (x) = 3x2 Show that gof ne fogSolution We have gof(x) = g(f(x))=g(cosx) = 3 (cos x)2

= 3 cos2 x Similarly fog(x)=f(g (x))= f(3x2)= cos (3x2) Note that 3cos2 x ne cos 3x2 for x = 0 Hence gof ne fogExample 3 Show that if f A rarr B and g B rarr C are onto then gof A rarr C is also ontoSolution Given an arbitrary element z isin C there exists a pre-image y of z under g such that g (y) = z since g is onto Further for y isin B there exists an element x in A with f(x) = y since f is onto Therefore gof(x) = g (f(x)) = g (y) = z showing that gof is onto Example 4 Let Y = n2 n isin N sub N Consider f N rarr Y as f(n) = n2 Show that

f is invertible Find the inverse of fSolution An arbitrary element y in Y is of the form n2 for some n isin N This implies that n =radicy This gives a function g Y rarr N defined by g (y) =radicy Nowgof (n) = g (n2)=radicn2 = n and fog (y) =f(radicy) = (radicy) 2 y which shows that gof=IN and fog= IY Hence f is invertible with f -1 = g

Political Science

Constitution of India-The Preamble

Summary

Objective of the state-To secure equality of status and of opportunity To promote fraternity among all the citizens To assure the dignity of the individuals and Unity and integrity of the nation

Justice-Justice stands for rule of law absence of arbitrariness and a system of equal rights freedom and opportunities for all in a society India seeks social economic and political justice to ensure equality to its citizens

Liberty-Liberty implies the absence of restraints or domination on the activities of an individual such as freedom from slavery serfdom imprisonment despotism etc The Preamble provides for the liberty of thought expression belief faith and worship

Equality-Equality means the absence of privileges or discrimination against any section of the society The Preamble provides for equality of status and opportunity to all the people of the country

Fraternity-The Preamble declares that fraternity has to assure two thingsmdashthe dignity of the individual and the unity and

Execution

Answer the following questions-

Short notes-1 Equality2 Fraternity3 Justice4 Liberty

Homework-Learn

integrity of the nation The word integrity has been added to the Preamble by the 42nd Constitutional Amendment (1976)

Business studies

Human resource management (chapter 1)

On the day of 1504 2020 I have discussed with you the managerial functions and procurement functions of HRM

Today weare going to discuss about the development function integration functions and maintenance function

Development functions-HRM improves the knowledge skills attitude and values of employees so that they the present and future jobs more effectively it includes

1) Development functions of HRM

a) Performance appraisal = It implies systematic evaluation of employees with respect to their performance on the job and their potential for development

b) Training =It is the process by which employees learn knowledge skills and attitudes to achieve organisational and personal goals

c) Executive development = It is the process of developing managerial talent through appropriate program

2) Integration functionsa) HRM reconcile the goals of

organisation with those of its members through integrating function

b) HRM tries to motivate employees to various financial and non financial incentives provided in job specification etc

3) Maintenance functiona) HRM promote and protect the

physical and mental health of employees by providing several types of benefits like housing medical aid etc

b) It Promote Social security measures to employees by providing provident fund pension gratuity maternity benefits

SubjectCOMMERCE

Topic

BUSINESSENVIRONMENT

Summary

Now quickly let us revise the earlier points that we have already done in the last class and let us proceed with the other topics that are there in the chapter

Firstly we will recall the internal and external factors of micro environment and then we

Execution 3 What do you mean by internal factors

in micro environmentAnswerInternal factors refer to all the factors existing within a business firm The internal factors are considered controllable because the enterprise has control over these factors

Development FunctionsPerformance AppraisalTrainingExecution Development

shall proceed in details

Meaning and list of internal and external factors

aInternal factorsInternal factors refer to all the factors existing within a business firm The internal factors are considered controllable because the enterprise has control over these factorsFor an example a company can alter its organization structure policies programmes employees physical facilities and marketing mix to suit the changes in the environmentList of internal factors areCorporate culture mission and objectives top management organizations structure company image and brand equity company resources

b External factorsExternal factors refer to those individual and groups and agencies with which a particular business organization comes into direct and frequent contact in the course of its functioningThese individuals and groups are known as STAKEHOLDERS because they have a stake (financial interest ) in the working and performance of the particular business List of external forces (stakeholders)Customers competitors investors suppliersmiddlemen (marketing intermediaries)financers publics

customers

suppliersfinancers

For an example a company can alter its organization structure policies programmes employees physical facilities and marketing mix to suit the changes in the environment

4 What do you mean by external factors in micro environment

AnswerExternal factors refer to those individual and groups and agencies with which a particular business organization comes into direct and frequent contact in the course of its functioningThese individuals and groups are known as STAKEHOLDERS because they have a stake (financial interest) in the working and performance of the particular business

3Who are stakeholdersSTAKEHOLDERS are individuals and groups who have a stake (financial interest ) in the working and performance of the particular business 4Discuss the internal factors in briefa Corporate CultureThe values beliefs and attitudes of the founders and top management of the company exercise a strong influence on what the cmpaany stands for how it does things and what it considers importantbMission and objectivesThe business philosophy and purpose of a comoany guide it prioritiesbusiness strategiesproduct market scope and development scope

cTop management structurethe composition of board of directors the degree of professionalization of management and the organizational structure of a company have important bearing on its business decisions

dPower structureThe internal power relationship between the board of directors and the chief executive is an important factor

e Company image and brand equityThe image and brand equity of the company play a significant role in raising finance forming alliance choosing dealers and suppliers launching new products entering foreign markets

5 What is Macro environmentAnswerMacro environment refers to the general

competitors

middlemen

publics

Fig STAKEHOLDERS OF A COMPANY

Apart from micro environment the other main dimension of business environment isMacro environment Macro environment refers to the general environment or remote environment within which a business firm and forces in its micro environment operateA company does not directly or regularly interact with the micro environmentTherefore macro environment is also known as indirect action EnvironmentThe macro environment forces are less controllable than the micro forces

Macro environment consists of the following components

POLITICAL AND LEGAL ENVIRONMENT

ECONOMIC SOCIAL AND ENVIRONMENT

CULTURAL

ENVIRONMENT

TECHNOLOGICAL ENVIRONMENT

Fig COMPONENTS OF MACRO ENVIRONMENT

environment or remote environment within which a business firm and forces in its micro environment operateA company does not directly or regularly interact with the micro environmentTherefore macro environment is also known as indirect action EnvironmentThe macro environment forces are less controllable than the micro forces 6 What are the components of macro environmenta Political and legal environmentb Economic environmentc Social and cultural environmentd Technological environment

Computer Science

Logic gates

Digital systems are said to be constructed by using logic gates These gates are the AND OR NOT NAND NOR EXOR and EXNOR

BUSINESS FIRM

gates The basic operations are described below with the aid of truth tables

AND gate

The AND gate is an electronic circuit that gives a high output (1) only if all its inputs are high A dot () is used to show the AND operation ie AB Bear in mind that this dot is sometimes omitted ie ABOR gate

The OR gate is an electronic circuit that gives a high output (1) if one or more of its inputs are high A plus (+) is used to show the OR operationNOT gate

The NOT gate is an electronic circuit that produces an inverted version of the input at its output It is also known as an inverter If the input variable is A the inverted output is known as NOT A This is also shown as A or A with a bar over the top as shown at the outputs The diagrams below show two ways that the NAND logic gate can be configured to produce a NOT gate It can also be done using NOR logic gates in the same way

NAND gate

This is a NOT-AND gate which is equal to an AND gate followed by a NOT gate The outputs of all NAND gates are high if any of the inputs are low The symbol is an AND gate with a small circle on the output The small circle represents inversion

NOR gate

This is a NOT-OR gate which is equal to an OR gate followed by a NOT gate The outputs of all NOR gates are low if any of the inputs are highThe symbol is an OR gate with a small circle on the output The small circle represents inversion

EXOR gate

The Exclusive-OR gate is a circuit which will give a high output if either but not both of its two inputs are high An encircled plus sign ( ) is used to show the EOR operation

EXNOR gate

The Exclusive-NOR gate circuit does the opposite to the EOR gate It will give a low output if either but not both of its two inputs are high The symbol is an EXOR gate with a small circle on the output The small circle represents inversion The NAND and NOR gates are called universal functions since with either one the AND and OR functions and NOT can be generated

Note A function in sum of products form can be implemented using NAND gates by replacing all AND and OR gates by NAND gates A function in product of sums form can be implemented using NOR gates by replacing all AND and OR gates by NOR gates

Logic gate symbols

Table 2 is a summary truth table of the inputoutput combinations for the NOT gate together with all possible inputoutput combinations for the other gate functions Also note that a truth table with n inputs has 2n rows You can compare the outputs of different gates

Logic gates representation using the Truth table

Example

A NAND gate can be used as a NOT gate using either of the following wiring configurations

Subject Eng Literature (The Tempest ndash William Shakespeare) Topic Act III Scene 3 Lines 53 to 110 (End of the scene) Date 16th April 2020 (2nd Period)

[Students should read the original play and also the paraphrase given in the school prescribed textbook]Summary Questions amp Answers

o Seeing this strange scene all are inclined to believe the tales told by travelers that there truly are ldquounicornsrdquo and ldquothe phoenixrsquo thronerdquo

o As they are about to sit down to the feast the banquet is snatched away by a harpy (Ariel disguised) A spiritrsquos voice (Arielrsquos voice) denounces Alonso Sebastian and Antonio with particular

1 ARIEL You are three men of sin whom Destiny

(Line 53-58)That hath to instrument this

lower world And what is int the never-surfeited sea

Hath caused to belch up you and on this island

Where man doth not inhabit you rsquomongst men

Being most unfit to live I have made you mad

reference to their crime in expelling Prospero from Milan They have not received any punishment for their deed earlier but the time for their punishment has arrived Upon Alonso it pronounces ldquolingering perdition worse than deathrdquo from which there is no remedy except through sincere repentance Ariel then vanishes in thunder and the shapes enter again and carry away the table

o Prospero watching invisibly is very pleased with the performance of Ariel and his (Prosperorsquos) ldquomeaner ministersrdquo All his enemies are now in his power and are in a fit of desperation He then leaves them and goes to see how Ferdinand and Miranda are getting on

o Alonso is now much humbled and penitent with the after effect of the spiritrsquos denunciation of his crimes He believes that his son is lost forever After this all disperse being stricken mad by the speech of the spirit

o Gonzalo fearing that they may do violence to themselves or to one another follows them and bid others to follow

(a) To whom does Ariel disguised as a harpy call the three sinners What game did Fate of Destiny play with

them

The three sinners called by Ariel are Alonso Sebastian and Antonio It was Destiny which had caused the ocean to cast the three sinners on the shore Though the ocean is all the time devouring whatever appears on its surface and is never satisfied with its continual swallowing of the ships and men in the present case the ocean had cast these three sinners on the shore without killing them

(b) Who had jointly been responsible for the conspiracy against Prospero What is Prosperorsquos purpose behind all this

Three men Alonso Sebastian and Antonio had jointly

been responsible for the conspiracy against Prospero They had driven out Prospero form Milan Prosperorsquos purpose is to make these three sinners realize the wrong they had done He wants them to repent for their criminal deeds because repentance leads to self-esteem(c )What does Ariel (the harpy) tell Alonso and his companions when they take out their swords to attack him

Seeing them drawing their swords Ariel (harpy) tells them that he and his companions are the instruments of destiny and that it is not possible for human beings to do them any injury He says that the swords of human beings can not injure even a minute part of his feathers Their swords are as ineffective against him and his companions as against the wind or the water

(d) Give the explanatory meanings of the following expressions in the context of the above extract

(i)Never surfeited (ii) Belch up (iii) lsquomongst men

(i) Never surfeited never led to satisfaction

(ii) Belch up cast ashore(iii) lsquomongst men in human

society2

I and my fellows (Line 60-65)

Are ministers of Fate The elementsOf whom your swords are tempered may as wellWound the loud winds or with bemocked-at stabsKill the still-closing waters as diminishOne dowl thats in my plume

IMPORTANT PASSAGES EXPLAINED

The elements

(Line 61-66)Of whom your swords are tempered may

as wellWound the loud winds or with

bemocked-at stabs

(a) Who is lsquoIrsquo Who are his lsquofellowsrdquo

lsquoIrsquo is referred to Ariel in disguise of a harpy His lsquofellowsrsquo are other spirits serving Prospero the real Duke of Milan who has acquired supernatural powers after being banished from his Dukedom Prospero has settled in this uninhabited island

(b) What are the elements that have temperrsquod the swords Why will it not work against the speaker

The swords (of Alonso and his companions) are tempered by metal (steel) which is taken out of the earth and refined by

Kill the still-closing waters as diminishOne dowl thats in my plume My fellow

ministersAre like invulnerable

In these words Ariel reminds the King and his companions of the utter futility of drawing swords against himself and his fellows Ariel drives Alonso Antonio and Sebastian the three men of sin to desperation ndash a state in which men do violence to themselves They draw swords to strike Ariel But Ariel reminds them that he and the other spirits are the ministers of destiny and nothing can wound them The steel of which their swords are made of may cut the wind or water which being divided always closes up again Even supposing that such things may be possible it is quite impossible that their swords will cut one feather in their plume They are incapable of being wounded by any sword of man Hence it is foolish on their part to attempt to strike at Ariel and his fellow-spirits

For which foul deed

(Line 72-75)The powers delaying not forgetting

haveIncensed the seas and shores yea all the

creatures Against your peace

Ariel enters like a harpy and remaining invisible tells Alonso Sebastian and Antonio that he and other harpies are the agents of Destiny appointed to carry out her decrees He tells them that their punishment for the crime against Prospero which has been so long deferred is now to fall upon them He reminds them that they had expelled Prospero from Milan and set him and his innocent child adrift on the sea and that the sea had paid them back for their sin by the shipwreck and by the calamities they have suffered He tells them that the powers above which did not forget this mean treachery but only deferred the punishment have now engaged the seas and the shores and all living beings including him and his comrades against them The very elements and supernatural agency Ariel adds have taken up the avenging of their crime against Prospero

the action of fire It may cut the wind or water which being divided always closes up again

The sword will not work against the spirits and the harpy because they are the ministers of destiny and nothing can wound them nor it will cut a single feather in their plume

(c )What is the meaning of lsquodowlrsquo in the last line

The term lsquodowlrsquo means a filament or the smallest part of a feather In this context Ariel in disguise of harpy says that their sword cannot even damage the smallest filament of their (Arielrsquos and other spirits) feathers as they are incapable of being wounded by any sword of man

(d) What does the speaker remind the listeners about

Ariel in disguise of harpy reminds Alonso the King of Naples Sebastian Alonsorsquos brother and Antonio the present Duke of Milan and the treacherous brother of Prospero as they being three men of sin He even reminds them that their punishment for their crime against Prospero which has been so long deferred now falls upon them He reminds them that they have expelled Prospero from Milan and has set him along with his innocent infant daughter adrift on the sea So the sea has paid them back for their sin by their shipwreck and the calamities they have suffered since then The harpy rebukes Alonso of his sin that has incensed the Gods and has deprived him of his son as a punishment

(e) How do they respond

When Ariel in disguise of a harpy reminds Alonso Sebastian and Antonio of their past misdeeds and sin Alonso has a look of terror and confusion in his eyes He utters the words of sincere repentance wrung out of his conscience-stricken heart It appears to him that all the elements of nature the sea-waves the wind and the thunder proclaiming a loud voice in the name of Prospero and the crime Alonso has committed against him They are calling upon him to repent There is a deep storm raging in Alonsorsquos breast and the echoes of that storm are ringing in his ears like a clear note of wind-instrument A note of denunciation of Alonsorsquos crime leaves him much humbled and penitent and confirms his belief that his son is lost forever But Sebastian and Antonio shows some courage instead of repentance They wish to kill the spirits or devils if it appears

3

Of my instruction hast thou nothing bated (Line 85-93)

In what thou hast to say So with good life

And observation strange my meaner ministers

Their several kinds have done My high charms work

And these mine enemies are all knit upIn their distractions They now are in my

powerAnd in these fits I leave them while I visitYoung Ferdinand whom they suppose is

drownedAnd his and mine loved darling

Methought the billows spoke and (Line 96-99)

told me of itThe winds did sing it to me and the

thunderThat deep and dreadful organ-pipe

pronouncedThe name of Prosper It did bass my

trespass

These are the words of contrition coming from Alonso Ariel has driven him to a deep repentance for conspiring with Antonio against Prospero He now feels a sincere remorse It appears to him that all the elements of nature the sea-waves the wind and the thunder proclaimed with a loud voice the name of Prospero and the crime Alonso had committed against him They are calling upon him to repent There is a deep storm raging in Alonsorsquos breast and the echoes of that storm are ringing in his ears like the clear note of a wind-instrument

Comment These are the words of sincere repentance wrung out of the conscience-stricken heart of Alonso Alonso who is the lesser villain is the first to give way to remorse under the effect of Arielrsquos speech The words of Ariel seem to him to be the voice of conscience speaking to him He is driven to desperation a state in which he might do violence to his life

(a) Identify the speaker State the context

Prospero the ruler of the island is the speaker The famous banquet scene has been enacted very well Ariel and his junior spirits have played their roles excellently Prospero is glad to say words of praise for them(b) In what way the speakerrsquos instructions have been carried out

According to Prosperorsquos instructions a banquet was presented before the King of Naples and his companions when they were tired and hungry Just when they were preparing to eat the feast the banquet was suddenly removed by exercising supernatural powers All this was done by Ariel Prosperorsquos chief assistant and a powerful spirit

Ariel not only made the feast disappear but also delivered his speech blaming the King and his two companions for their past wicked deeds He warned them to repent for their misdeeds or suffer forever on that uninhabited island

(c) Who are referred to as lsquomeaner ministersrsquo What have they done

Prospero refers as lsquomeaner ministersrsquo to his other lesser spirits who were assisting Ariel in presenting a scene before the kingrsquos party They entered the scene to the accompaniment of music They assumed several strange shapes and brought in a banquet Then they danced about it with gentle actions of salutations thus inviting the King and others to eat the feast

These spirits play their role again when Ariel in the shape of a harpy quits the scene These shapes enter again and dancing with mocking gestures carry away the table

(d) Who are the speakerrsquos enemies What has happened to them

King of Naples Alonso his brother Sebastian and the present Duke of Milan Antonio (Prosperorsquos own brother) are Prosperorsquos enemies With the turn of events they have all been washed ashore on the island which is ruled by Prospero the great magician Actually this happened after the shipwreck caused by a storm which was raised by Prospero with the purpose of bringing these people to his island Prosperorsquos spirits have already confused and terrified these enemies and they are under Prosperorsquos control He can treat them as he likes

(e) What does he say about Ferdinand Explain what is meant by ldquohellip his and mine darlingrdquo

Prospero knows that Alonsorsquos son prince Ferdinand is alive though his father thinks that the prince has been drowned

Prospero refers to his daughter Miranda who is dear to him She is also very dear to Prince Ferdinand who has fallen in love with her They are waiting to be married soon for which they have received Prosperorsquos consent

4

ALONSO O it is monstrous monstrous (Line 95-102)

Methought the billows spoke and told me of it

The winds did sing it to me and the thunderThat deep and dreadful organ-

pipe pronouncedThe name of Prosper It did bass

my trespassTherefore my son ithrsquo ooze is

bedded andIll seek him deeper than eer

plummet soundedAnd with him there lie mudded

(a) In what way does Alonso express his horror when his conscience is awakened by Arielrsquos words

When Alonsorsquos conscience is awakened by Arielrsquos words he expresses his horror at what he has heard He gets the feeling that the waves of the ocean the wind and the loud thunder have spoken to him and uttered the name of Prospero Because of being reminded of his crime in a very loud and rough voice he comes to realize that he has lost his son for his past misdeeds

(b) What does Alonso imagine about his son What does Alonso want to do in his desperate state

Alonso imagines that his son is lying in the mud at the bottom of the sea He feels desperate that he wants to drown himself in the ocean deeper than the plumb-line has ever gone He wants to lie with his son at the bottom of the sea

(c) How do Sebastian and Antonio want to face the evil spirits

Sebastian says that he is not at all afraid of what the harpy has said and that he is prepared to fight any number of such monsters if they appear before him only one at a time Antonio says that he would support Sebastian in the fight against the fiendsyyy

(d) Why does Gonzalo ask Adrian to follow the three men

Gonzalo tells Adrian that all the three men namely Alonso Sebastian and Antonio are in a wild and reckless mood The thought of the heinous crime of which they are guilty has begun to torment their minds So he asks Adrian to follow those three men without loss of time and prevent them from doing anything which the turmoil in their minds might lead them to do

(e) What opinion do you form of Alonso from the above extract

Alonso who is the lesser villain is the first to give way to remorse under the effect of Arielrsquos speech The words of Ariel seem to him to be the voice of conscience speaking to him He is driven to desperation a state in which he might do violence to his life

Subject =Accounts

Ac-12 15420 topic-pL Appropriation ac

PROFIT AND LOSS APPROPRIATION ACCOUNT

MEANING AND PREPARATIONProfit and Loss Appropriation Account is merely an extension of the Profit and Loss Account of the firm The profit of the firm has to be distributed amongst the partners in their respective profit sharing ratio But before its distribution it needs to be adjusted All Adjustments like partnerrsquos salary partnerrsquos commission interest on capital interest on drawings etc are made in this account These adjustments will reduce the amount of profit for distribution This adjusted profit will be distributed amongst the partners in their profit sharing ratio To prepare it at first the balance of Profit and Loss Account is transferred to this account The journal entries for the preparation of Profit and Loss Appropriation Account are given below

1 for transfer of the balance of Profit and Loss Account to Profit and Loss Appropriation Account

(a) In case of Net Profit

Profit and Loss Ac helliphelliphelliphelliphellipDrTo Profit and Loss Appropriation Ac(Net Profit transferred to Profit and Loss Appropriation Ac)

(b)In case of Net Loss

Profit and Loss Appropriation Achelliphelliphellip DrTo Profit and Loss Ac(Net Loss transferred to Profit and Loss Appropriation Ac)

2 for Interest on Capital

For transferring on Interest on CapitalProfit and Loss Appropriation Achelliphelliphellip DrTo Interest on Capital Ac(Interest on capital transferred to Profit amp Loss Appropriation Ac)

3 for Interest on Drawings

For transferring Interest on Drawings Interest on Drawings Achelliphelliphelliphelliphelliphellip DrTo Profit and Loss Appropriation Ac(Interest on drawing transferred to Profit amp Loss Appropriation Ac)

4 For Partnerrsquos SalaryFor transfer of partnerrsquos SalaryProfit and Loss Appropriation Achelliphellip DrTo Salary Ac(Salary transferred to profit amp Loss Appropriation Ac)

5 For Partnerrsquos CommissionFor transferring commissionProfit and Loss Appropriation Achelliphelliphellip DrTo Commission Ac(Commission transferred to Profit and Loss Appropriation Ac)

6 For Transfer of agreed amount to General ReserveProfit and Loss Appropriation Ac helliphellipDrTo General Reserve Ac(Transfer to General Reserve)

7 for share of Profit or Loss appropriation(a) If ProfitProfit and Loss Appropriation Achelliphellip DrTo Partnerrsquos CapitalCurrent Ac(Profit transferred to capitalcurrent Ac)(b) If LossPartnerrsquos Capital Current Achelliphelliphelliphellip DrTo Profit and Loss Appropriation Ac(Loss transferred to capitalcurrent Ac)

THE FORMAT OF PROFIT AND LOSS APPROPRIATION

Profit and Loss Appropriation Account for the year endedhelliphelliphelliphellip

Particulars Amount Particulars Amount

To PL Ac (loss) By pL Ac (profit)

To Interest on capital BY Interest on drawings

To partner`s commission by Partner`s capital Ac ( loss)

To Partner`s salary To Interest on partner`s loan To General Reserve To Partner`s Capital AC (Profit)

Subject= Economics

MOVEMENT ALONG THE DEMAND CURVE (CHANGE IN QUANTITY DEMANDED)In law of demand you have already studied the inverse relationship between price and quantity demanded When quantity demanded of a commodity changes due to change in its price keeping other factors constant it is called change in quantity demanded It is graphically expressed as a movement along the same demand curve There can be either a downward movement or an upward movement along the same demand curve Upward movement along the same demand curve is called contraction of demand or decrease in quantity demanded and downward movement along the same demand curve is known as expansion of demand or increase in quantity demanded

Extention of demandd

price (rs)p A

B Extentionp1 d

Q Q1

Quantity demanded ( in units)

Contraction of demandd

p2 Ccontraction

p APrice (Rs)

d

Q2 Q

Quantity demanded (in units)

Explanation of movement of demand A fall in price from OP to OP1 leads to increase in quantity demanded from OQ to OQ1 (expansion of demand) resulting in a downward movement from point A to point B along the same demand curve DD When Price rises from OP to OP2 quantity demanded falls from OQ to OQ2 (contraction of demand) leading to an upward movement from point A to point C along the same demand curve DD

  • Activity Series of Metals
    • Drawbacks of Rutherfordrsquos model of atom
      • Electromagnetic radiations
      • Properties of electromagnetic radiations
      • Characteristics of electromagnetic radiations
        • Plancks Quantum Theory-
        • Photoelectric effect
          • Intext Questions
            • Logic gates
            • Digital systems are said to be constructed by using logic gates These gates are the AND OR NOT NAND NOR EXOR and EXNOR gates The basic operations are described below with the aid of truth tables
            • AND gate
            • Example
Page 15:  · Web viewSubject. Topic. Summary. Execution. English 1 . Chapter 1 naming words . Page 8. Write the names of these pictures:- Person:-1. father. 2.Firefighter 3.doctor 4 ...

Latitudinal extent

Continentality

Relief features

Presence of low pressure trough

Jet streams

English Language The Sentence A complex sentence contains one independent clause and at least one dependent clause The dependent clause in a complex sentence is introduced with subordinating conjunctions or relative pronouns

Commonly Used Subordinating Conjunctions-Time after before while when since untilCause And Effect because now since as in order that soOpposition although though even though whereas while in spite ofCondition if unless only if whether or not even if in case(that)

Commonly Used Relative Pronouns-Who whose whom which whoever whomever whichever that

Class IXSubject Topic Summary Execution

1-BENGALI(2ND LANGUAGE)

ldquo বঙগভমিরপরমিrdquo াইকেলধসদনদতত

আচেগর পর উততর পচো-১ ২ ৩ এং নীচের পর টি াহির কা- ৪মহিbকাও গচেনা দেগা পহিচে অমত হরচে- ক) কার দো দেকান কহিতার অং( ) কতা দেক পরসঙগ কী উহিকতটির তাৎপযC আচোনা কচেরা৫দেসই ধনয নরকচে দোচেক যাচে নাহি ভচে মচেনর মহিeচের সাচেসচে সCন ক) কহির কায C ার উচেf(য হিক হিছ কহি কন কহিতাটি দেচেন) কহি কার কাচেছ হিমনহিত কচেরচেছনগ) কহি এই পহিথীচেত কাচের ধনয মচেন কচেরনঘ) কহি হিক রকম অমর তাাভ করচেত ান

Hindi 2nd lang

काकी(कतिसयारामशरण गपत)

इस कहानी म लखक न यह बतान का परयास निकया ह निक बचच अपनी मा स निकतना परम करत ह शयाम अबोध बालक ह वह अपनी मा क मरन क बा- उसन अपनी मा क कतिलए बहत रोया बा- म उस पता चला निक उसकी मा राम क घर चली गई ह आकाश म उडती हई पतग -खकर उस हरष हआ निक पतग क दवारा वह अपनी मा को नीच उतारगा इसक कतिलए वह अपनी निपता की जब स -ो बार सवा रपया निनकालकर पतग और -ो मोटी सी मन वाली अपन भाई स काकी एक कागज पर कतिलखवा कर पतग म कतिशव का दि-यानिनकालकर पतग और -ो मोटी सी मन वाली अपन भाई स काकी एक कागज पर कतिलखवा कर पतग म कतिचपका दि-याभोला और शयाम कोठरी म रससी बाधनी रह थ तभी उसक निपता करोध म आकर उन स पछ निक कया उनकी जब स रपया निनकाला हभोला डर क मार बताया निक शयाम इस पतग क दवारा अपनी काकी को राम क यहा स उतारना चाहता हनिवशशवर(शयाम क निपता)न फटी पतग उठाकर -खी तो उस पर काकी कतिलखा थावह हत बजिa होकर वही खड रह गएउनहोन सोचा निक मन अपन पतर को मारा जोनिक अनजान और निन-dरष थावह अपनी मा कोनिकतना पयार करता ह

उस दि-न बड सवर शयाम की नी- खली तो -खा निक घर भर म कोहराम मचा हआ ह

क) घर म कोहराम कयो मचा हआ था शयाम को कया लगा

ख) काकी को ल जात समय शयाम न कया उपदरव मचाया

ग) काकी क बार म उस कया बताया गया कया सतय उस कतिछपा रहा

घ) वह बठा-बठा शनय मन स आकाश की ओर कयोकरता

उततरक) शयाम की मा का -हात हो गया था इसकतिलए

घर म कोहराम मचा हआ था शयाम की लगा निक उसकी मा सफ- कपडा ओढ हए भमिम पर सो रही ह

ख) लोग जब उमा यानी शयाम की मा को उठाकर ल जान लग तब शयाम न बडा उपदरव मचाया लोगो क हाथ स झठ करवा उमा क ऊपर जा निगरा और बोला काकी सो रही ह उस कहा ल जा रह हो

ग) काकी क बार म बजिaमान लोगो न उस निवशवास दि-लाया निक उसकी का निक उसक मामा क यहा गई ह लनिकन सतय अमिधक दि-नो तक कतिछपाना रह सका आसपास क अबोध बालको क मह स यह बात परकट हो गई निक उसकी मा का -हात हो गया ह

घ) कई दि-नकई दि-न लगातार रोत-रोत उसका रोना तो शान हो गया पर उसक ह-य म शोक भर गया था वह चपचाप बठा आकाश की और टाका करता निक शाय- उसकी काकी कही दि-ख जाए

ldquoदि-न उसन ऊपर आसमान म पतग उडती -खी न जान कया सोच कर उसका निहर-य एक-म खिखल उठाrdquo

क) निकसन पतग ऊपर उडत -खी और वह कयो खश हआ

ख) उसन अपन निपता स कया कहा उनका कया उतर थाश

ग) उसन निफर कया निकया और निकसन उसकी सहायता की

घ) उसकी योजना कया थी उततर -क) शयाम न एक दि-न आसमान म पतग उडती

-खी तो उसन सोचा निक पता आसमान म राम क यहा जाकर रकगी वही पर मरी काकी ह यह सोचकर वह बहत खश हआ

ख) उसन अपन निपता स कहा काका मझ एक पतग मगा -ो उसक निपता न भटक हए मन क भाव स कहा निक मगा -ग यह कह कर उ-ास भाव स वह कही और चल गए पतग नही आई

ग) उसन चपचाप निवशशवर क टगहए कोट स एक चवननी निनकाल ली और सखिखया -ासी क लडक भोला की सहायता स एक पतग मगवानी भोला उसकी बराबर उमर का ही था

घ) उसकी योजना यह थी निक वह अपनी पतग को आकाश म राम क यहा भजगा और उस पतग क सहार उसकी काफी नीच उतर जाएगी इस योजना पर उस परा निवशवास था इसकतिलए वह और भोला -ोनो यह काम करन म लग गए

Continue to nexthelliphellipEVS CHAPTER - 1

(UNDERSTANDING OUR ENVIRONMENT)

Sustainable development

The development that meets the needs of the present without compromising the ability of future generations to meet their own needs is called Sustainable development

Sustainable societies ndash

An environmentally sustainable community is one that meets the current and future basic resource needs of its people in a just and equitable manner without compromising the ability of future generations to meet their basic needs

Q ) What are Eco Villages

Ans - Eco village are the urban or rural communities of people who strive to integrate a supportive social environment with a low impact way of life

Q ) To ensure sustainable development the depletion of renewable resources should not take place at a rate faster than their regeneration Justify your answer

Ans ndash Renewable resources do not have a fixed quantity - more can always be

generated However if the rate of use exceeds the rate of renewal - that is the

source is used more than its being recreated - its continued use will become

used up faster than it can regenerate

To promote sustainable society the following things need to be done ndash

1 Using renewable energy sources 2 By improving the quality of human

health 3 By promoting sustainable agriculture 4 By forming ecovillage

it will eventually be entirely depleted So Toensure sustainable development the depletion of

renewable resources should nottake place at a rate faster than their regeneration

Q ) What do you mean by Sustainable societies

Ans - Sustainable societies are defined as towns and cities that have taken steps to remain healthy over the long term These communities value healthy ecosystems use resources efficiently and actively seek to retain and enhance a locally based economy Sustainable development concerns everybody in a society

Q ) What are the effects of pollution on human health

Ans ndash Some health problem occurs due to air pollution are ndash

Respiratory diseases Cardiovascular damage Fatigue headaches and anxiety Irritation of the eyes nose and throat Damage to reproductive organs Harm to the liver spleen and blood Nervous system damage

Some health problem occurs due to water pollution are ndash

Typhoid Cholera Dysentry Jaundice

Some health problem occurs due to noise pollution are ndash

Fatigue headaches and anxiety High blood pressure Hearing damage

Physics Motion in 1D First go through previous notes Now here we will solve some numerical related to that

Question 3What information about the motion of a body is obtained from the displacement-time graphSolution 3From displacement-time graph the nature of motion (or state of rest) can be understood The slope of this graph gives the value of velocity of the body at any instant of time using which the velocity-time graph can also be drawn

Question 4(a)What does the slope of a displacement-time graph represent(b)Can displacement-time sketch be parallel to the displacement axis Give a reason to your answerSolution 4(a) Slope of a displacement-time graph represents velocity(b) The displacement-time graph can never be parallel to the displacement axis because such a line would mean that the distance covered by the body in a certain direction increases without any

increase in time which is not possible

Chemistry Language of Chemistry

How to balance a chemical equationThere are two methods of balancing an equation(i)Hit and trial method(ii)Partial equation methodBalancing by hit and trial methodThis method consists of counting the number of atoms of each elements on both sides and trying to equalize themTake the following steps(i)Count the number of times (frequency) an element occurs on either side(ii)The element with the least frequency of occurrence is balanced first(iii)When two or more elements have the same frequencythe metallic element is balanced firstExample-1 On heatinglead nitrate decomposes to give lead dioxidenitrogen dioxide and oxygenPb(NO3)2rarrPbO+NO2+O2

In this equationLead occurs twiceNitrogen occurs twiceOxygen occurs four timesSince lead is a metalbalance it firstThe number of atom of lead is equal on the two sidestherefore it needs no balancingNow balance nitrogenOn the reactant sidethere are two atoms of nitrogenwhile on the product side oneSomultiply the product containing nitrogenon the product sideby two Pb (NO3)2rarrPbO+2NO2+O2Nowthe number of oxygen atoms on the reactant side 6while on the product sideit is 7Somultiply the entire equation by 2except oxygen to get balanced equation2Pb(NO3)2rarr2PbO+4NO2+O2Multiplication by 2 is done only when atoms of all the elements except one element are balanced and the unbalanced atom occurs separately at least once and also there is a difference of only one such atom

Math Topic AlgebraChapter

Factorisation

Study item Difference of two squares a2 ndash b2 = (a+b) (a-b)1) (i) 4x2ndash 25y2

= (2x) 2 ndash (5y) 2= (2x + 5y) (2x - 5y)

(ii) 9x2 ndash 1= (3x)2ndash(1)2= (3x + 1)(3x ndash 1)

2) (i) 150 ndash 6a2= 6(25 ndash a2)= 6(5)2 ndash(a)2= 6 (5 + a) (5 ndash a)

(ii) 32x2 ndash 18y2=2(16x2 ndash 9y2)=2(4x)2 ndash (3y)2= 2(4x + 3y)(4x - 3y)3)(i) (x ndashy )2 ndash 9 = (x ndash y )2 ndash (3)2= (x ndash y + 3) (x ndash y ndash 3)(ii) 9(x + y) 2ndash x 2= (3)2(x + y)2 ndash (x)2=3(x + y)2 ndash (x)2= (3x +3y ) 2ndash(x)2= (3x + 3y + x)(3x +3y ndash x)= (4x + 3y) ( 2x + 3y )

Commercial studies

Basic accounting terms

Today I will give you some questions from the previous study material

Questions1) Define accounting2) What do you mean by debit and

credit

3) Explain the types of account4) Define the following terms

a) Assetsb) Capitalc) Purchased) Debtorse) Transactions

5) Name the types of accounts given below

a) Krishnas accountb) Machinery accountc) Royalty accountd) Salary accounte) Furniture accountf) Audit fee account

Economics Revision Today I will give you some revision questions

Questions1) What do you mean by the terms

rdquowantsrdquo2) Write the difference between

consumer goods and producer goods

3) Define the term utility 4) Explain the different types of utility5) Define

a) Total utilityb) Marginal utility

Subject Eng Literature (The Merchant of Venice ndash William Shakespeare)Topic Act I Scene 3 Lines 1 to 48 (Shylock hellip Cursed be my tribe if I forgive him) Date 16th April 2020 (5th Period)

[Students should read the original play and also the paraphrase given in the school prescribed textbook]Summary Questions amp Answers

This scene takes place in Venice and we are introduced to the rich Jew Shylock Bassanio and Shylock are talking and Bassanio tells Shylock that he wants a loan of three thousand ducats for three months on the personal security of Antonio

o Shylock feels glad because he will be able to bind down Antonio by means of a bond on account of the loan but he tells Bassanio that all the fortunes of Antonio being invested in the merchant ships on the sea it is difficult to depend upon his credit Even under such circumstances Shylock is willing to advance the money on the personal security of Antonio

o Bassanio then invites Shylock to dine with him Shylock says that he is prepared to do anything with the Christians but not eat or drink or pray with them

o While Bassanio and Shylock are talking Antonio appears on the scene Shylock does not seem to take any notice of Antonio but goes on brooding within

(1) SHYLOCK Ho no no no no- my meaning in (Line 15-26)saying he is a good man is to have you understand me that he is sufficient Yet his means are in suppositionhe hath an argosy bound to Tripolis another to the Indies I understand moreover upon the Rialto he hath a third at Mexico a fourth for England and other ventures he hath squanderd abroad Butships are but boards sailors but men there be land-rats and water-rats land-thieves and water-thieves I mean pirates and then there is the peril of waters winds and rocks The man is notwithstanding sufficientmdashthree thousand ducats mdashI think I may take his bond

(a) Who is talking in the beginning of this scene What does Bassanio want from Shylock How does Shylock feel

In the beginning of the scene Bassanio and Shylock are talking to each other Bassanio wants to borrow three thousand ducats from Shylock for three months on the security of Antonio Shylock feels glad at heart that he will get the opportunity of binding Antonio with a bond(b) What risks does Shylock weigh in advancing the money

Shylock says that Antonio has invested all his capital in trading by sea-going ships But the ships are made of wood and the sailors of those ships are ordinary human beings The wood can

himself how he hates Antonio because of his being a Christian because he abuses Shylock in public places Shylock decides that if ever he can get Antonio to his advantage he will teach him a lesson

come to harm and men can commit mistakes and thus the capital invested in ships may be lost Then there are other dangers The goods loaded on the ships can be damaged by rats and thieves which are found both on land and water The ships can also be harmed through sea-storms submerged rocks etc(c) What two important functions does this scene have

The scene has two important functions First it completes the exposition of the two major plot lines of the play Antonio agrees to Shylockrsquos bond ndash three thousand ducats for a pound of flesh and second and more important dramatically this scene introduces Shylock himself In this scene Shakespeare makes it clear at once why Shylock is the most powerful dramatic figure in the play and why so many great actors have regarded this part as one of the most rewarding roles in all Shakespearean dramas(d) Where does this scene take place What kind of treatment has Antonio been giving to Shylock What does Shylock say when Bassanio invites him to dine with him

The action of this scene takes place in Venice Antonio has been in the habit of behaving harshly with Shylock ndash spitting on his beard and footing him like a stranger cur When Bassanio invites Shylock to dine Shylock says that he is prepared to do anything with the Christians but not eat and drink or pray with them

(2) SHYLOCK How like a fawning publican he looks (Line 38-48)I hate him for he is a Christian

But more for that in low simplicity

He lends out money gratis and brings downThe rate of usance here with us in VeniceIf I can catch him once upon the hipI will feed fat the ancient grudge I bear him

He hates our sacred nation and he railsEven there where merchants most do congregateOn me my bargains and my well-won thriftWhich he calls interest Cursed be my tribeIf I forgive him

(a) What is the context in which these words are spoken and what is the idea expressed in it

These remarks are made by Shylock when he sees Antonio coming along after Bassanio told him that the merchant will be his surety for the bond The above mentioned passage reveals Shylockrsquos hatred for Antonio Shylock says that he hates Antonio because he is a Christian and also because he gives loan without taking interest on them thereby bringing down the rate of interest in Venice(b) Explain the meaning of the phrase lsquoa fawning publicanrsquo

The phrase lsquoa fawning publicanrsquo refers to Roman tax collector It is a term of contempt and hatred on the lips of a Jew lsquoFawning Publicansrsquo were Roman tax-gatherers whose ordinary bearings towards the Jews was bullying but whose false pose of lsquohumility and contritionrsquo is touched upon in the parable in New Testament(c ) What light does the above passage throw on the character

of Shylock

The above mentioned speech of Shylock reveals him to be a wicked character having an extreme greed for wealth His intense hatred for Antonio is unjustified He hates Antonio only because he is a Christian and because he lends money without taking any interest on it thereby adversely affecting Shylockrsquos business of lending money on high interest(d) What information do you gather about Antonio from the above given lines

Shylockrsquos statement throws a valuable light on the character of Antonio Antonio appears to be a good Christian and a good human being He helps the people in need by lending them money without charging any interest on it He is a man of simple and good nature This very goodness makes him Shylockrsquos enemy(e) What does Shylock debate within himself and when To whom are the lines mentioned above addressed to

When Bassanio asks the Jew to lend him three thousand ducats on Antoniorsquos surety Shylock begins to debate within himself as to how he should exploit the opportunity of a business deal with his old enemy Antonio

The lines mentioned above are not addressed to anyone The lines are a soliloquy ie a speech made by a character to himself and not meant to be heard by the other characters present

Class XSubject Topic Summary ExecutionEnglish

LiteratureThe Blue Bead 2nd part

Things took a turn for the worst and all of a sudden a crocodile attacked the woman biting on the womanrsquos leg At that moment Sibia got up sprinted grabbed the hay fork and stabbed the crocodile in the eye with all her power Immediately the crocodile let go and went away Sibia saw a small blue bead lying by the river she grabbed it Since she was poor she didnrsquot have necklace Shersquod always wanted one like the other women now she could make one with the blue bead After that she went home and told her mother all about it

Hindi 2nd

Langबड घर की बटी( मशी परमच-)

lsquoबड घर की बटी कहानी का उददशय मधयम वग की घरल समसया को सलझा कर सगदिठत परिरवार म मिमल जलकर परम स रहन का स-श -ना हघर म शानित सथानिपत करन की जिजमम-ारी नारी की होती ह यदि- नारी समझ-ार ह उसम धय और परिरवार क परनित परम ह तो कोई भी घटना परिरवार को निवघदिटत नही कर सकती या कहानी परिरवार को सगदिठत करत हए परम सौहा- स एक दसर की भावनाओ को समझ करउनका सहयोग करत हए जीवन यापन करन की पररणा -ती हमशी परमचदर जी न इस कहानी म सय परिरवार का परनितनिनमिधतव निकया ह यह कहानी बनी माधव सिसह जो गौरी पर क जमी-ार क उनक -ो पतरो की हशरी कठ लाल निबहारीशरीकात का निववाह एकजमी-ार घरान की पतरी आन-ी स हआ थाआन-ी न ख- को ससराल क वातावरण म ढाकतिलया थाएक दि-न आन-ी का अपन -वर लाल निबहारी स झगडा हो जाता ह -ोनो भाई एक दसर स अलग होन की कोकतिशश करत हसभी बह आन-ी न अपन मधर वयवहार स लाल निबहारी को

ldquoइन नतर निपरय गणो को बीए-इनही -ो अकषर पर नयोछावर कर दि-या था इन -ो अकषर न उनक शरीर को निनबल और चहर को कानित ही बना दि-या थाldquo

क) परसतत पकतियो म निकस वयकति क बार म कहा गया ह

ख) इन पकतियो म कौन स नतर निपरय गणो क बार म कहा गया ह

ग) बीए की निडगरी परापत कर लन पर भी उपय वयकति क सवभाव की कया निवशरषता थी

घ) यह नतर निपरय गण निकस वयकति म निवदयमान थ उसक वयकतितव की कया निवशरषता थी

उततर ndashक) परसतत पकति म गौरी पर गाव क जमी-ार

क बड बट शरीकात क बार म कहा गया ह उसन बहत परिरशरम और उ-म क बा- ba की निडगरी परापत की थी अब वह एक -फतर

घर छोडकर जान स रोक कतिलयाइस पर बनी माधव सिसह न कहा निक बड घर की बटी ऐसी ही होती ह जो निबगडा काम बना लती ह अतः शीरषक साथक ह बड घर की बटी आन-ी ह

म कमचारी थाख) भरा हआ चहरा चौडी छाती और डटकर

खाना आदि- एक सबजी ल जवान क गण मान जात ह परत शरीकात न इनही नतर निपरय गणो को अपनी पढाई पर नयोछावर कर दि-या था

ग) बीए की निडगरी परापत कर लन पर भी उपय वयकति(शरी कठ की शारिररिरक तौर पर निनबल और चहर स कानित ही लगत थ इतना ही नही वह मानकतिसक तौर पर भी निपछड हए थ पाशचातय सामाजिजक कथा उस घणा एव पराचीन सभयता का गणगान उनकी निवचारधारा क परमख अग थ

घ) यह नतर निपरय गण गौरीपर गाव क जमी-ार क छोट बट लाल निबहारी सिसह म निवदयमान थ वह सजीलाजवान था और भस का दध शर दध वह सवर उठकर पी जाता था

ldquoयही कारण था निक गाव की लललन आए उनकी निन-क थी कोई कोई तो उह अपना शतर समझन म भी सकोच ना करती थी सवय उनकी पतनी को इस निवरषय म उनस निवरोध थाldquo

क) उपय पकति म इस वयकति क बार म कहा गया ह

ख) गाव की लललन आए उनकी निन-ा कयो निकया करती थी

ग) उनकी पतनी का कया नाम था उनह निकस निवरषय म अपन पनित क निवरa था और कयो

घ) इस कहानी का कया उददशय ह Continue to next helliphelliphellip

Bengali 2nd Language

ফ ফটক না ফটক( কহিতা )

পর) ldquo(ান াধাচেনা ফটপাচেথ পাথচের পাড হিচেয় এক কাঠচোটটা গাছ কহিকহি পাতায় পার ফাটিচেয় াসচেছldquoক) কার দো দেকান কহিতার অং( ) lsquo(ান াধাচেনা ফটপাচেথ পাথচের পাডহিচেয়lsquo চেত কী দোঝাচেনা চেয়চেছ গ) আচো য অংচে(lsquo এক কাঠচোটটা গাছ lsquoচেত কী দোঝাচেনা চেয়চেছ ঘ) ldquoকহিকহি পাতায় পার ফাটিচেয় াসচেছldquo ----- একথার পরকত অথC কী উততর ) ক) আচো য অং(টি পর যাত কহি সভা4 মচোপাধ যাচেয়র দো lsquoফ ফটক না ফটকrsquo কহিতার অং()কহি সভা4 মচোপাধ যায় হিছচেন দেপরচেমর কহি দেপরমচেক নানা ভহিঙগমায় হিতহিন ফটিচেয় তচেচেছন দেপরম মানচের স মচেতC র সঙগী কহিতার কহিতায় এক রb সb হচেয়র দেপরম াগরচেনর কথা চেচেছন (ান অথCাৎ দেযাচেন দেকান রস দেনই দেযাচেন দেকান মহিনতা দেনই অথ তার মধ দেযও দেপরম থাকচেত পাচের একথাই কহি তচে ধরচেত দেচেয়চেছন একটি মানচে4র মচেন দেযাচেন দেকামতার দেকান সথান দেনই পাথচেরর মচেতা হিনরসতার মচেনর মধ দেযও দেয দেপরম আসচেত পাচের দেস কথাই কহি চেচেছনগ)নারীচের যথC দেপরচেমর ছহি এই কহিতায় অকপচেট উচেঠ এচেসচেছ কহি এই কহিতায় কাটচোটটা গাছ কথাটি যার কচেরচেছন নারী দেয দেপরম দেথচেক হিতাহিত এং দেসই দেপরম সঠিক সমচেয় না পাওয়ার ন য দেপরম সমপচেকC হিচেr4 গৈতরী য় দেপরচেমর দেয গৈহি(ষট য মাধযC য সরসতা দেকামত এই সমসতর হিপরীত যথা রbতা শষকতা কচেঠার তা পরভহিত দোঝাচেত এক কাঠচোটটা গাছ কথাটি যার কচেরচেছনঘ) এাচেন এক নারীর যথC দেপরচেমর কথা হিনহিCপত ভাচে চেচেছন কহি অসমচেয় নারীর ীচেন দেপরম দেচেগচেছ এতহিন তার হয় রb কচেঠার হিছ দেপরচেমর অভাচে ঠাৎ দেসই শষক মরভহিমচেত সচের আভাস এচেসচেছ দেপরম দেযন 4Cার স(ীত তাই পরায় মত গাচেছ কহিকহি পাতা গহিচেয় উচেঠচেছ

Biology Chapter - 01Controlling Air Pollution

Today we will discuss how we control air pollution from domestic combustion

Q1Describe any five ways of reducing air pollution from domestic sources bull The number of pollutants in the air is verylarge and we always try to control them byfollowing ways

i) Solar cooker and solar heater It use no fuel reduce damage of environment by fuel use or reducing deforestation It maintains coolness of house It releases very less orno oil gas or grease

ii) Piped natural gas (PNG) It emits very less by products into the atmosphere As it isdistributed through pipe lines so there iscontinuous supply of fuel is possible

iii) Liquefied Petroleum Gas (LPG) It hasa higher heating value LPG doesntcontain sulphur so it burns a lot cleanerenergy sources It releases very less oralmost no fume in air

iv) Electricity based cooking Emission free cooking alternative for urban dwellers causeselimination of adverse health impactsofindoor air pollution It helps to avoid theinconveniences associated with procurement of LPG

v) Biogas It contains 75 methane whichmakes it an excellent fuel It burns without smoke and biogas plant leaves no residue like ash in wood charcoal etc Thus it isaclean fuel

Economics

Factors of Production

Today firstly we would recall the last class for 5 mins and then we would proceed with the further topics of the chapter

The concept meaning of land characteristics of land and importance of land to be repeated for the absentees as well as the students who were there in the class the previous day

Today we will start with the last portion of land before it the meaning of land to be repeated onceAs by now we all know that

Questions1What do you mean by productivity of landAnswer By productivity of land we mean the capacity of a piece of land to produce a crop

Thus it refers to the average output per unit of landSay per acre per hectare etc= (OutputArea of land)

2 What are the factors influencing the productivity of landAnswer

Natural factors Productivity of land is largely determined by the natural

Land is defined to include not only the surface of the earth but also all other free gifts of nature(for example mineral resources forest resources and indeed anything that helps us to carry out the production of goods and services but is provided by nature free of cost)

We will move on to the last portion of land by discussing Productivity of Land

By productivity of land we mean the capacity of a piece of land to produce a crop

Thus it refers to the average output per unit of land

Say per acre per hectare etc= (OutputArea of land)

With this we shall proceed further with the main factors that determine the productivity of land

Natural factors Human factors Improvements on land Location of land Organisation Ownership of land Availability of capital Proper use of land State help

Note economic development of a country depends upon the quality of its land If the land is fertile it will quicken the pace of development of the country

qualities of land such as fertility etc

Human factors Land cannot produce anything by itself Man has to apply labour on it to produce for himself So productivity of land depends on the knowledge and skills of workers

Improvements on land production of land is affected by land development measures like provision of well or tubewell irrigation proper drainage

State help The government of a country especially less developed country can play a vital role in improving the agricultural productivity by providing better irrigation facilities

Organisation Productivity of land also fdepends upon the way how the factors of production like labour and capital are organised

In order to increase productivity trained workers modern implements scientific methods good seeds are all essential

3 lsquoImproved technology affects the productivity of landrsquo Explain this statement with the help of suitable example Answer Use of improved technology raises the productivity of land Example By using HYV seeds chemical manures and modern machines per hectare output increases

Physics Force (Summary)

Question Write the expression for the moment of force about a given axisSolutionsThe expression for the moment of force is given byMoment of force about a given axis = Force times perpendicular distance of force from the axis of rotationQuestion What do you understand by the clockwise and anticlockwise moment of force When is it taken positiveSolutionsIf the effect on the body is to turn it anticlockwise moment of force is called the anticlockwise moment and it is taken as positive while if the effect on the

body is to turn it clockwise moment of force is called the clockwise moment and it is taken as negative

Math Topic Commercial Mathematics

Chapter Goods and services Tax

Study item Some solved sums from exercise ndash 1 A retailer buys a TV from a wholesaler for Rs 40000 He marks the price of the TV 15 above his cost price sells it to the consumer at 5 discount on the marked price If the sales are intra ndash state and the rate of GST is 12 find

(i) The marked price of the TV(ii) The amount which the consumer pays for the TV(iii) The amount of tax (under GST) paid by the retailer to the central

Government(iv) The amount of tax (under GST) received by the State Government

Solution As the sales are intra- state sale and the rate of GST 12 So GST comprises of 6 CGST and 6 SGSTTherefore a retailer buys a TV from a wholesaler for Rs 40000Therefore the amount of GST collected wholesaler from the retailer or paid by retailer to wholesalerCGST = 6 of Rs 40000 = Rs(6100 times40000) =Rs 2400SGST = 6 of Rs 40000 = Rs (6100 times 40000) =Rs 2400Therefore wholesaler will pay Rs 2400 as CGST and Rs 2400 as SGSTTherefore amount of input GST of retailer Input CGST = Rs 2400 and input SGST = Rs 2400Again the retailer marks the price of the TV 15 above his cost price(i) The marked price of the TV

= Rs 40000 + Rs 40000times15= Rs 40000 + Rs 40000times 15100= Rs 40000 + Rs 6000Rs 46000But the retailer sells it to consumer at 5 discount on the marked priceCost price after discount = Rs 46000 ndashRs46000times 5100 =Rs 46000 ndashRs 2300= Rs 43700Therefore the amount of GST collected retailer from consumer or paid by consumer to retailerCGST = 6 of Rs 43700 =Rs ( 6100 times43700)Rs 2622SGST = 6 of Rs 43700 = Rs (6100 times 43700) =Rs 2622Amount of the output GST of retailer Output CGST = Rs 2622 and output SGST = Rs 2622

(ii) The amount which the consumer pays for the TV= cost price of TV to consumer + CGST paid by consumer + SGST paid by consumer= Rs 43700 + Rs 2622 + Rs 2622= Rs 48944

(iii) The amount of tax (under GST ) paid by the retailer to the central Government=CGST paid by retailer = output CGST ndash input CGST=Rs 2622 ndash Rs 2400=Rs 222

(iv) The amount of tax ( under GST ) received by the State Government = SGST paid by wholesaler + SGST paid by retailer= Rs 2400 + output SGST ndash input SGST=Rs 2400 + Rs 2622 ndash Rs 2400=Rs 2400 + Rs 222= Rs 2622

Commercial studies

Stakeholders Today I am going to give some revision questions from the previous study material

Questions1) State the two expectations of

employees from a business concern2) Give two distinctions between

stakeholder and shareholder3) Give two difference between

internal stakeholders and external stakeholders

4) Give two expectations of suppliers from a business organisation

5) Who is a stakeholder in commercial organisations

Chemistry Periodic Table

Merits of Mendeleevrsquos Periodic law are as follows - 1He grouped the elements on the basis of atomic mass 2 He left gaps for undiscovered elements like Gallium Scandium germanium Also he left a full group vacant for undiscovered inert gases 3 He could predict proportions of several elements on basis of their position in periodic table like Ga Sc etc 4He could predict errors in atomic weights of some elements like gold platinum etc

Anomalies in Mendeleevrsquos Periodic law are as follows - 1 Position of isotopes could not be explained 2 Wrong order of atomic masses could not be explained

For example- as Arnur atomic mass 40 come first and K with low atomic mass (30) should come later but k should be placed first

According to Bohrrsquos Modern Periodic table properties of elements are periodic functions of their atomic numbers

So when elements are arranged according to increasing atomic numbers there is periodicity in electronic configuration that leads to periodicity in their chemical properties

It consists of horizontal rows (Periods) Vertical column (Groups)

There are 7 period and 12 groups in this long form of periodic table

Ist period has 2 elements IInd period has 8 elements IIIrd period has 8 elements IVth period has 18 elements Vth period has 18 elements VIth period has 32 elements VIIth period hs rest of elements

Note - The number of valence electrons in atom of elements decides which elements will be first in period and which will be last

In group- 1 to 2 gp and 13 to 17 contain normal elements 3 to 12gp ndash transition elements 57 to 71 - lanthanides 89 to 103 - Actinides

Left hand side ndash metals Right hand side ndash nonmetals

Note- Hydrogen element has been placed at top of Ist group Electronic configuration of H is similar to alkali metal as both have 1 valence electron

V electron of gp I element -- 1 V electron of gp 2 element -- 2 V electron of gp 13 element -- 3 V electron of gp 14 element -- 4 V electron of gp 15 element -- 5 V electron of gp 16 element --6 V electron of gp 17 element -- 7 V electron of gp 18 element -- 8

English 1 Transformation of sentences

Sentences A sentence is a group of words which makes complete sense

Exercise 2Change the following sentences from

a Assertive sentencesb Imperative sentencesc Interrogative sentencesd Exclamatory sentences

Sentences can be changed from one grammatical form to another without changing the meaning of the sentence This is known as transformation of sentences

assertive to interrogative1 Nobody would like to be a fool

Who would like to be a fool2 Their glory can never fade

When can the glory fade3 Nobody can control the wind

Who can control the wind4 It matters little if I die

What though I die5 No man can serve two masters

Can any man serve two masters

Exercise 3Interchange of assertive and Exclamatory sentences

1 She leads the most unhappy lifeWhat an unhappy life she leads

2 This is indeed an interesting bookWhat an interesting book this

3 He is a very great manWhat a great man he is

4 It is a very lame excuseWhat a lame excuse

5 It is sad that she died so youngAlas she died so young

Class XISubject Topic Summary Execution

Hindi 2nd lang

पतर परम(परमचदर) पतर परम कहानी म एक निपता की इचछाओ का वणन निकया गया ह अपन बड पतर परभ -ास स निपता चतनय -ास का निवशरष परम था निपता को उसक जनम स ही बडी-बडी आशाए थी उसम दसर बट कतिशव-ास की अपकषा स- उतसाह की मातरा अमिधक थी वह उस इगलड भजकर बरिरसटर बनाना चाहत थभागय का खल भी बडा निनराला ह बीए की परीकषा क बा- वह बीमार पड गया डॉकटरो न भी जवाब - दि-या थाचतन -ास जी बहत ही कजस थ बवजह पस खच करना नही चाहत थ अगर गारटी मिमलती तो शाय- पस खच भी कर -त परत गारटी नही थी परिरणाम सवरप उनक बट का -हात हो गयाजब बट को समशान ल जा रह थ तो वहा काफी शोर गान बजान हो रह थ पछन पर पता चला निक निकसी निपता निपछल तीन साल स निबमार था और उसक ईलाज म रपया पानी की तरह बहाया पर ठीक नही हए परत उसक बट को तनिनक भी अफसोस नही था उसका कहना था उसन कोकतिशश तो कीयह -खकर चतनय-ास जी को आतम निगलानी हईतभी स उनका म परिरवतन हआ और बट का भोज काफी धमधाम स निकयाऔर वहइस पशचाताप की आग म जलत रह औला- स बढकर पसा नही होता ह इस बात को समझन म उनह काफी व लग गया

hellipContinue to next

BENGALI(2ND LANGUAGE)

পরথমঅধযায়-ঠাকরারীনদরনাথঠাকর

নয়ন দোচের হিমাচেররা া নাচেমই হিযাত হিছচেন ায়ানার উাররণ সবরপ নয়ন দোচের ারা হিা (াচেকর হিা হিচেতন এছাাও দেকান উৎস উপচেb রাহিতর দেক হিন করার উচেfচে(য তারা সযC হিকরচেরণ রনয পরীপ জবাহিচেয় তাচেত রপার হির 4Cরণ করচেতন ঠাকরা এই নয়ন দো হিমারচের দে(4 ং(ধর হিছচেন হিমাররা ায়ানার ষটানত পর(Cন কচের তারা হিনঃসব এই হিমাহিরর দে(4 ং(ধর গৈকাস নদর রায়চেৌধরী গৈকাস া নয়ন দোচের সমসত সমপহিতত ঋচেরণর াচেয় হিহিx কচের অহি(ষট যা আচেছ তাচেত হিপত

ইার হিপতার মতয ইচে পর নয়নচোচের ায়ানার দেগাটা কতক অসাধাররণ শরাদধ (াহিনতচেত অহিনতম ীহিপত পরকা( কহিরয়া ঠাৎ হিনহিয়া দেগ- ক) কার দো দেকান গচেলপর অং() কতা দেক ইার চেত কাচেক দোঝাচেনা চেয়চেছ গ) পরসঙগ কী কতার কতয পহিরসফট কচেরা

পরচে4র যাহিত রbা করা সমভ নয় তাই হিতহিন পতরচেক হিনচেয় ককাতায় সাস শর কচেরন গলপ কথচেকর আহিথCক অসথা নয়ন দোচের হিমাচের দেথচেক সমপরণC আাা কথচেকর হিপতা হিনচের দেষটায় অথC উপাCন করচেতন া উপাহিধ াচেভর নয তার াসা হিছনা আর দেসই কারচেরণ কথক তার একমাতর উততরাহিধকার চেয় তার হিপতার পরহিত কতজঞ কথক দো পা হি(চেচেছন হিনচের পরারণ ও মান রbার নয উপচেযাগী অথC হিনা দেষটায় পরাপত চেয়চেছন- এটাই তার কাচেছ পরম দেগৌরচের হি4য় চে মচেন কচেরন কাররণ (নয ভাণডাচের গৈপতক ায়ানার উজজব ইহিতাস অচেপbা দোার হিসeচেকর মচেধয গৈপতক দেকামপাহিনর কাগ তার কাচেছ অচেনক দেহি( মযান

TO BE CONTINUED

উ- ক) আচোয অং(টি রীনদরনাথ ঠাকচেরর দো ঠাকরা গচেলপর অং() কতা চেন আচোয গচেলপর গলপ কথকইার চেত নয়ন দোচের হিমাহিরর দে(4 ং(ধর গৈকাস ার কথা া চেয়চেছ গৈকাস া নয়ন দোচের সমসত সমপহিতত ঋচেরণর াচেয় হিহিx কচের অহি(ষট যা আচেছ তাচেত হিপত পরচে4র যাহিত রbা করা সমভ নয় তাই হিতহিন পতরচেক হিনচেয় ককাতায় সাস শর কচেরনগ) গৈকাস ার হিপতার মতযর পর নয়ন দোচের হিমাহিরর অহিসততব হিপত য় কচেয়কটা উৎস ও শরাদধ- (াহিনতচেত হিমাহিরর দে(4 কহিটক যয় চেয় হিগচেয় এচেক াচের দে(4 চেয় যায় তন তাচের গC করার মত আর হিকছই হিছ না-দেসই পরসচেঙগ এই উহিকত নয়নচোচের হিমাচেররা া নাচেমই হিযাত হিছচেন ায়ানার উাররণ সবরপ নয়নচোচের ারা হিা (াচেকর হিা হিচেতন এছাাও দেকান উৎস উপচেb রাহিতরচেক হিন করচেত হিগচেয় তারা সযC হিকরচেরণর নয পরীপ জবাহিচেয় তাচেত রপার হির 4Cরণ করচেতন তাই দেসকাচের ায়ানা দেহি(হিন সথায়ী চেত পারত না হিহিভনন উৎস শরাদধ- (াহিনতচেত সাধযা হিতহিরকত র করার নয হিমাহির হিহিকচেয় দেযত হ হিতC কা হিহি(ষট পরীচেপর দেত দেযমন অলপকাচের মচেধয হিনঃচে(4 চেয় যায়-নয়নচোচের হিমারচের অসথা তাই চেয়হিছ এই কারচেরণই কথক নয়নচোচের হিমারচের গা ভরা আমবর সয করচেত পারতনা

Physics Dimensional Analysis (Summary)

Q Find the dimensions of consts ab in relation

p=(bminusxlowastx)at

where p is the power x is the distance and t is time

Ans From principle of homogeneity dimension of b x2 are same Dim of b = dim of x2 = [L2] = [ML2T0]Dim of a = dim of ( b- x2)dim of (pt) = [M0L2T0][ML2T-2] [T-1] [T] = [M-1L0T2]

Chemistry Atomic Structure Drawbacks of Rutherfordrsquos model of

atom a According to Rutherfordrsquos model of atom electrons which are negativelycharged particles revolve around the nucleus in fixed orbits Thusb theelectrons undergo acceleration According to electromagnetic theory of Maxwell a charged particle undergoing acceleration should emitelectromagnetic radiation Thus an electron in an orbit should emitradiation Thus the orbit should shrink But this does not happenc The model does not give any information about how electrons aredistributed around nucleus and what are energies of these electrons Isotopes These are the atoms of the same

Properties of electromagnetic radiationsa Oscillating electric and magnetic field are produced by oscillating charged particles These fields are perpendicular to each other and both areperpendicular to the direction of propagation of the waveb They do not need a medium to travel That means they can even travel invacuum

Characteristics of electromagnetic radiationsa Wavelength It may be defined as the distance between two neighbouring crests or troughs of

element having the same atomicnumber but different mass numbere g 1H11H21H3

Isobars Isobars are the atoms of different elements having the same massnumber but different atomic numbere g 18Ar40 20Ca40

Isoelectronic species These are those species which have the same numberof electrons

Electromagnetic radiationsThe radiations which are associated withelectrical and magnetic fields are called electromagnetic radiations When anelectrically charged particle moves under acceleration alternating electricaland magnetic fields are produced and transmitted These fields aretransmitted in the form of waves These waves are called electromagneticwaves or electromagnetic radiations

wave as shown It is denoted by λb Frequency (ν) It may be defined as the number of waves which passthrough a particular point in one secondc Velocity (v) It is defined as the distance travelled by a wave in onesecond In vacuum all types of electromagnetic radiations travel with thesame velocity Its value is 3 times10 8m sec-1 It is denoted by v

d Wave number Wave number is defined as the number of wavelengths per unit lengthVelocity = frequency timeswavelength c = νλ

Plancks Quantum Theory- o The radiant energy is emitted or absorbed not continuously but discontinuously in the form of small discrete packets of energy called lsquoquantumrsquo In case of light the quantum of energy is called a lsquophotonrsquoo The energy of each quantum is directly proportional to the frequency of the radiation ie E α υ or E= hυ where h= Planckrsquos constant = 6626 x 10-27 Js o Energy is always emitted or absorbed as integral multiple of this uantum E=nhυ Where n=1234Black body An ideal body which emits and absorbs all frequencies is calleda black body The radiation emitted by such a body is called black body radiation

Photoelectric effectThe phenomenon of ejection of electrons from thesurface of metal when light of suitable frequency strikes it is calledphotoelectric effect The ejected electrons are called photoelectrons

Biology Chapter - 02Systematics and Five Kingdoms

Scientists divide the whole living organisms into two kingdom first and ultimately by five kingdom at last

In the earlier systems of classifications organisms are divided into kingdom plantaeand kingdom animalia on the of presenceof cell wall their modes of nutrition and movements

Some problem arise like fungi share manycharacteristic withplant despite their heterotrophic nutrition bacteria protozoa areunicellular present in both kingdom Toovercome this third kingdom Protista isintroduced which include

unicellularorganisms But there is also another

problem Allunicellular organisms are not similar kind The cellular structure of prokaryotes is verydifferent from that of other organismsEukaryotes possess a true nucleus and allcell organelles that are not present inprokaryotes So the fourth kingdom Monerais introduced which include unicellular prokaryotes (bacteriaamp blue green algae)

bull Still some problem arise in kingdomplantae

So in 1969 R H Whittakar proposedanew five kingdom System of classification

i) Kingdom Monera - unicellular prokaryotes

ii) kingdom Protista - unicellular eukaryotes

iii) Kingdom Fungi - uni or multicellular fungi with cell wall but without chlorophyll

iv) Kingdom Plantae - Multicellular Plants

v) Kingdom Animalia - Multicellular Animals

EVS Chapter 1 ndash Modes of Existence

An agricultural society

An agricultural society also known as an agrarian society is a society that constructs social order around a reliance upon farming More than half the people living in that society make their living by farming

People in an agricultural society generally lead a more settled lifestyle than those in nomadic hunter-gatherer or semi-nomadic pastoral societies because they live permanently near the land that is farmed Agricultural settlements tend to develop in areas of convenience near bodies of water which is used for both crops and transportation or along trade routes Not everyone in an agricultural society is a farmer Some people make a living trading or making and selling goods such as tools used for farming

Another way to define an agrarian society is to see the total amount of production in a nation In an agrarian society cultivating the land is the main source of wealth Such a society can recognize other means of subsistence and work habits but emphasizes the importance of agriculture and livestock Agrarian societies have existed in various parts of the world for 10000 years and continue to exist today They have been the most common form of socio-economic organization for most of recorded human history

Q) Write the features of agricultural society

Ans - Structure and Features of Agrarian Society1 Occupational Structure

An agrarian society is generally associated with the domestication of plants and animals The domestication of plants means farming and that of animals means herding Often there is mixture of farming and the use of such domesticated animals as cow goat and sheep

2 Forms of Land Ownership in Agrarian SocietiesGenerally there are landlords supervisory farmers cultivators and share croppers The landholders own the land but do not work on it They let it out for sharecropping The supervisory farmers are those who live by having their land cultivated by hired labourers The cultivators cultivate the land for themselvesThe share-croppers are those who live by tilling other peoplersquos land or a crop-sharing basis The artisans own their means of production and produce by their own labour in their homesteads

3 Village Community System An agrarian society is highlighted by

the institution of village community system The agrarian economy made fixed dwelling houses necessary Living close together for protection and co-operation and living nearer to the land gave birth to agricultural villages The village is not only the residential place of farmers it is also the social integrator

4 Minimal Division of Labour Another structural feature of agrarian society is a minimal division of labour Except for the basic division founded on age and sex differences there are few specialized roles There is only one predominant type of occupation ie domestication of plants and animals For all the people the environment physical as well as social is the same

5 Role of Family The farm family is of the patriarchal type the father is the final arbiter in most of the familyrsquos major decisions The life of ail men and women is merged in family life Since there are not many special organizations family is the only organisation to perform the tasks of aid and protection

6 Sense of Unity The members of an agrarian society exhibit a strong in-group feeling Since the whole of their social lives is wrapped up in a society which is physically economically and socially homogenous they are inclined to view the entire outside world as an out group

7 Informal Social Control An agrarian society is regionally divided into villages In a village community the force of traditional mores is more dominant than in the urban community In the village everybody is known to everybody The members in a village community help each other and share the joy and sorrows of each other Crime in an agrarian society is rare

8 Simplicity and Uniformity Life of the people in an agrarian society is marked by simplicity and uniformity Their main occupation is agriculture which largely depends upon the vagaries of nature An agrarian society is a religious society

Math Compound angles Compound angles The algebraic sum of two or more angles is called a compound angle If A B C be three angles then A+B B+C C+A A-B B-C A-C A+B-C etc are compound angles In this chapter we shall discuss the trigonometrical ratios of compound angles Theorem 1 If A B and A+B are all pisitive acute angles theni) sin( A+B) = sin A cos B + cosA sinBii) cos(A+B) = cosA cosB- sinA sinBTheorem 2If A and B are positive acute angles and AgtB theni) sin(A-B) = sin A cosB- cos A sinBii) cos(A-B) = cos A cos B+ sin A sin BTo prove that i) sin(A+B) sin (A-B) = sin2 A - sin2 B = cos2 B- cos2 A

Example 1 Prove that tan70deg=2tan50deg+tan20degSolutiontan70deg = tan(50deg + 20deg)Or tan70deg=(tan 50deg+tan 20deg)(1-tan50degtan20deg) or tan70deg (1 ndash tan 50deg tan20deg) = tan50deg+tan20degor tan70deg= tan70deg tan50deg tan20deg+ tan50deg + tan20deg = cot20deg tan50deg tan20deg + tan50deg + tan20deg = 2 tan50deg+ tan20degExample 2 If A + B = 45deg show that (1 + tanA) (1 + tanB) = 2Solutiontan(A + B) =( tan A + tan B) (1 - tan

ii) cos(A+B) Cos(A-B) = cos2 A- sin2 B = cos2 B -sin2 AProof i) LHS= sin(A+B)sin(AminusB) [Recall sin(αminusβ)=sinαcosβminuscosαsinβ And sin(α+β)=sinαcosβ+cosαsinβ]= (sinAcosB+cosAsinB)times(sinAcosBminuscosAsinB)= sin2Acos2Bminuscos2Asin2B [Recall sin2α+cos2α=1 From above we can then assume correctly that sin2α=1minuscos2α AND cos2α=1minussin2α] = sin2A(1minussin2B)minussin2B(1minussin2A) = sin2Aminussin2Asin2Bminussin2B+sin2Asin2B = sin2Aminussin2B= 1-cos2A-(1-cos2B) = cos2 B- cos2 A = RHSii)LHS= cos (A+B) cos(A-B) [ cos(A+B) = cos AcosB- sinAsinBCos(A-B) = cosAcosB+ sinAsinB]= cos2 A Cos2 B- sin2 A Sin2 B= cos2 A( 1-sin2 B) - (1- cos2 A) sin2 B= cos2 A- cos2 A sin2 B- sin2 B+ cos2 A sin2 B=cos2 A- sin2 B=1- sin2 A-(1-cos2 B) = cos2 B- sin2 A= RHSTangent formulae for compound anglesi)tan (A + B) = tan A + tan B1-tan A tan Bii)tan (A ndash B) = tan A-tan B1+tan A tan Biii) cot (A + B) = cot Acot B-1cot A+cot B(viii) cot (A ndash B) = cot Acot B+1cot B-cot A

A tan B) Or 1= (tan A+ tanB) (1-tan A tanB) Or tanA + tanB + tanA tanB + 1 = 1 + 1Or tanA (1 + tanB) + (1 + tanB) = 2Or (1 + tanA) (1 + tanB) = 2Example 3 Find the value of sin 15degSolution sin 15deg= sin(45deg-30deg) = sin45degcos 30deg- cos45degsin30deg =(1radic2) (radic32) -(1radic2) (12) = (radic3-1) 2radic2Example 4 If sin A = 1 radic10 and sin B = 1 radic5 where A and B are positive acute angles then what is A + B SolutionWe know that sin (A + B) = sin A cos B + cos A sin B= [1 radic10] [radic(1 minus 1 5)] + [1 radic5] radic(1 minus 1 10)= [1 radic10] [radic4 5] + [1 radic5] [radic9 10]= [1 radic50] times (2 + 3)= 5 radic50 = 1 radic2

sin (A + B) = sin π 4rArrHence A + B = π 4Example 5 If A + B = 225o then find [cot A] [1 + cotA] times [cot B] [1 + cot B]Solution[cot A] [1 + cotA] times [cot B] [1 + cot B] = 1 [(1 + tan A) times (1 + tan B)]=1 [tan A + tan B + 1 + tan A tan B] [ tan (A + B) = tan225o]∵

tan A + tan B = 1minus tan A tan BrArr= 1 [1 minus tan A tan B + 1 + tan A tan B]= 1 2

COMMERCE

CLASSIFICTION OF HUMAN ACTIVITIES-ECONOMIC AND NON-ECONOMIC

Firstly we shall recall the previous class for 5 mins especially for the absentees and for also the rest of the students who were there

Today at first we briefly discuss the earlier portions of the chapter

1Business-It includes all those economic activities which are concerned with production and exchange of goods and services with the object of earning profit Example A factory shop beauty parlour also business enterprises

2Profession ndashThe term profession means an occupation which involves application of specialized knowledge and skills to earn a living For Example Chartered Accountancy medicine law tax consultancy are example of professions

Questions1What are the main features of ProfessionAnswer The main features of a profession are as follows a Specialised body of knowledge-Every profession has a specialised and systematised body of knowledge b Restricted entry- Entry to a profession is allowed only to those who have completed the prescribed education and have the specialised examination c Formal education and training ndashA formal education and training is given to the person who wants to acquire the professional

3Employment-Employment mean an economic activity where people work for others in exchange for some remuneration (salary)The persons who work for others are called lsquoemployeesrsquo The persons or organizations which engage others to work for them are called lsquoemployersrsquoEg A doctor working in a hospital is employment as he is working for a salaryA lawyer may serve as a law officer in a bank

With this we shall proceed with the features of both Profession amp Employment

The main features of a profession are as follow

a Specialised body of knowledge b Restricted entry c Formal education and training d Professional association e Service motive f Code of contact

The main features of an employment are as follows

a In employment a person works for others called employer

b An employee provides personal service

c There is a service agreement or contract between the employee and the employer

d The employee has to obey the order of the employer

e No capital investment is made by the employer

Various examples of Employment are as follows

aA teacher teaching in a school or collegeb An engineer employed in Municipal Corporation of DelhicAn accountant working in the accounts department of a companydA doctor working in a hospital

Note In all the above examples of employment the individual who is involved in each example is working as an employee for a salary under an employer

qualification(MBBSCALLB)d Service motive ndashProfessionals are expected to emphasis service more on their clients rather than economic gain f Code of Conduct-The activities of professionals are regulated by a code of conduct

2 What are the main features of EmploymentAnswer The main features of an employment are as followsa In employment a person works for others called employerb An employee provides personal servicec There is a service agreement or contract between the employee and the employerd The employee has to obey the order of the employere No capital investment is made by the employer

3 Give various Professions and their respective Association are given below

Professions

Professional

Professional association

Medical profession

Doctor Medical Council of India

Law profession

Lawyers Bar Council of India

Accounting Profession

Chartered

The Institute of Chartered Accounts of India( ICAI)

Engineerin Engineers The

g Profession

institute of Engineers (India)

Accounts Basic accounting terms

Today we will give you some questions from the previous study material

Questions6) Define accounting7) What do you mean by debit

and credit8) Explain the types of account9) Define the following terms

a) Assetsb) Capitalc) Purchased) Debtorse) Transactions

10) Name the types of accounts given below

a) Krishnas accountb) Machinery accountc) Royalty accountd) Salary accounte) Furniture accountf) Audit fee account

Economics Basic Economic ConceptsSub topic

UTILITY

Before starting todayrsquos class we shall recall the last class which was about UTILITY AND THE FEATURES OF UTILITY

Now we shall proceed with the further topics of the chapter

Todayrsquos topic from the chapter lsquo Basic Economic Conceptsrsquo will be TOTAL UTILITY amp MARGINAL UTILITYNow let us quickly revise the concept of utility with an example ie goods and services are designed because they have an ability to satisfy human wantsThis feature of being able to satisfy human wants is termed as utility For example we derive utility from WiFi services as it gives us satisfaction by connecting us to our friends and family through social media here consumers derive utility from WiFi services

From the above concept we shall start with todayrsquos topicEconomists have defined TOTAL UTILITY (TU) as the total satisfaction obtained by consuming a given total amount of a good and serviceFor example the total satisfaction obtained from eating 10 mangoes is the total utility of 10 mangoes

MARGINAL UTILITY (MU) is the additional satisfaction derived from each additional unit

Questions1 What is Total Utility (TU)

Answer Total Utility (TU) is the

aggregate of the utility that a consumer derives from the consumption of a certain amount of a commodityTU=MU1+MU2++MUn

2 What is Marginal UtilityAnswer

Marginal Utility (MU) is the additional made to the total utility as consumption is increased by one more unit of the commodityMU= TUn ndashTUn-1

NoteOften economists tend to

subdivide utility into an imaginary unit called UTIL

consumed In this casethe utility obtained from each mango as it is consumed as the MU of that mango It is also defined as the addition made to the total utility when an additional unit is consumed Often economists tend to subdivide utility into an imaginary unit called UTIL

Note As a consumer increases the consumption of a good over period of time the total utility or total satisfaction derived from it increases to appoint and thereafter it decreasesHowever as the consumer keeps on consuming the good the marginal utility or the additional utility derived from it decreases

SubjectBusiness studies

Topic

BUSINESSENVIRONMENT

Summary

Now quickly let us revise the earlier points that we have already done in the last class and let us proceed with the other topics that are there in the chapter

Firstly we will recall the internal and external factors of micro environment and then we shall proceed in details

Meaning and list of internal and external factors

aInternal factorsInternal factors refer to all the factors existing within a business firm The internal factors are considered controllable because the enterprise has control over these factorsFor an example a company can alter its organization structure policies programmes employees physical facilities and marketing mix to suit the changes in the environmentList of internal factors areCorporate culture mission and objectives top management organizations structure company image and brand equity company resources

b External factorsExternal factors refer to those individual and groups and agencies with which a particular business organization comes into direct and frequent contact in the course of its functioningThese individuals and groups are known as STAKEHOLDERS because they have a stake (financial interest ) in the working and performance of the particular business List of external forces (stakeholders)Customers competitors investors suppliersmiddlemen (marketing intermediaries)

Execution 1 What do you mean by internal

factors in micro environmentAnswerInternal factors refer to all the factors existing within a business firm The internal factors are considered controllable because the enterprise has control over these factorsFor an example a company can alter its organization structure policies programmes employees physical facilities and marketing mix to suit the changes in the environment

2 What do you mean by external factors in micro environment

AnswerExternal factors refer to those individual and groups and agencies with which a particular business organization comes into direct and frequent contact in the course of its functioningThese individuals and groups are known as STAKEHOLDERS because they have a stake (financial interest) in the working and performance of the particular business

3Who are stakeholdersSTAKEHOLDERS are individuals and groups who have a stake (financial interest ) in the working and performance of the particular business 4Discuss the internal factors in briefa Corporate CultureThe values beliefs and attitudes of the founders and top management of the company exercise

financers publics

customers

suppliersfinancers

competitors

middlemen

publics

Fig STAKEHOLDERS OF A COMPANY

Apart from micro environment the other main dimension of business environment isMacro environment Macro environment refers to the general environment or remote environment within which a business firm and forces in its micro environment operateA company does not directly or regularly interact with the micro environmentTherefore macro environment is also known as indirect action EnvironmentThe macro environment forces are less controllable than the micro forces

Macro environment consists of the following components

POLITICAL AND LEGAL ENVIRONMENT

ECONOMIC SOCIAL AND ENVIRONMENT

CULTURAL

ENVIRONMENT

TECHNOLOGICAL ENVIRONMENT

a strong influence on what the cmpaany stands for how it does things and what it considers importantbMission and objectivesThe business philosophy and purpose of a comoany guide it prioritiesbusiness strategiesproduct market scope and development scope

cTop management structurethe composition of board of directors the degree of professionalization of management and the organizational structure of a company have important bearing on its business decisions

dPower structureThe internal power relationship between the board of directors and the chief executive is an important factor

eCompany image and brand equityThe image and brand equity of the company play a significant role in raising finance forming alliance choosing dealers and suppliers launching new products entering foreign markets

5 What is Macro environmentAnswerMacro environment refers to the general environment or remote environment within which a business firm and forces in its micro environment operateA company does not directly or regularly interact with the micro environmentTherefore macro environment is also known as indirect action EnvironmentThe macro environment forces are less controllable than the micro forces 6 What are the components of macro environmenta Political and legal environmentb Economic environmentc Social and cultural environmentd Technological environment

BUSINESS FIRM

Fig COMPONENTS OF MACRO ENVIRONMENTPolitical science

Introduction to political science

Comparative politics and itrsquos scope Comparative politics is the second major dimension of political scienceIt is also a very vast area of study and a very large number of political scientists even treat it as an autonomous area of study within the board ambit of political scienceScope of comparative politics-

1 All political structures -Comparative politics includes the study of all structures formalnon formal governmental and extra governmental which are directly or indirectly involved in politics in all the countries of the world

2 Functional studies- Comparative politics seeks to study politics less from the point of view of the legal institutions in terms of their powers and move from the point of view of their functions which constitute the political process and their actual Operation in the environment

3 Study of political behaviour- Another important part of its scope is the study of the actual behaviour of the people in the process of politics

4 Study of similarities and differences- comparative politics also undertakesan analysis of the similarities and differences among political process and functions

5 Study of all political systems -comparative politics seeks to analyse the actual behaviour and performance of all political systems western as well as non western

6 Study of the environment and infrastructure of politics-The study of politics demands a study of the psychological sociological economic and anthropological environment in fact the social environment as a whole in which each political system operates

7 Study of political culture- political culture is composed of attitudesbeliefs emotions and values of a society that relate to the political system or politics

8 Study of political participation- Political participation is a universal processThe only difference is that while in some states it is limited in others it is wider

9 Study of political process- political

Answer the following questions-

What is comparative politics

What are the scope of comparative politics

Homework- learn

processes like decision makingpolicy making judicial process leadership recruitment process and others are always at work in all political systems

The scope of comparative politics is very comprehensive It includes everything that falls within the area of political activity and political process

History CAMBRIDGE VIEW ABOUT

THE PARTITION

AND REFUTATION

OF CAMBRIDGE

VIEW

Cambridge view about the Partition The Cambridge school of historians have interpreted that opposition to partition scheme was made entirely by the elitist groups They hold the view that Lord Curzon planned to partition the Bengal for administrative purposeREFUTATION OFCAMBRIDGE VIEW The Rationalist historians have rejected the interpretations of the Cambridge School of historians on various grounds

1 QUESTION State different views of historians regarding Partition of Bengal

ANSWER Cambridge historians believed that Lord Curzon partitioned Bengal for administrative reasons only and not for the political motive The Middle class elitist group protested because of their petty interest The Hindu zamindars protested as they have to spend more money for managing their estatesThe lawyers of Calcutta High court feared to lose their clientBut according to the nationalist Historians was-

2- The ultimate object of Lord Curzon was to crush the unity of Bengal politicians

3- If Bengal becomes a separate province Bengali speaking 16 million people of western part would become minority under Hindi speaking people of Bihar and Oriya speaking people of Orissa

4- The bureaucrats expected that the protest movement would die down quickly

5- Lord Curzon used the Muslim community in his political game

6- Idealism had great contribution in the protest against partition

7- The people of the every section of society were affected by the partition of Bengal

Computer Science

Numbers Convertion of dcimal number to octal numberThe decimal numeral system is the standard system for denoting integer and non-integer numbers It is the extension to non-integer numbers of the Hindu-Arabic numeral system For writing numbers the decimal system uses ten decimal digits a decimal mark and for negative numbers a minus sign - The decimal digits are 0 1 2 3 4 5 6 7 8 9 the decimal separator is the dot in many countries

The octal numeral system or oct for short is the base-8 number system and uses the digits 0 to 7 Octal is sometimes used in computing instead of hexadecimal perhaps most often in modern times in conjunction with file

permissions under Unix systems It has the advantage of not requiring any extra symbols as digits It is also used for digital displays

Follow these steps to convert a decimal number into octal form

1 Divide the decimal number by 82 Get the integer quotient for the next iteration (if the number will not divide equally by 8 then round down the

result to the nearest whole number)3 Keep a note of the remainder it should be between 0 and 74 Repeat the steps until the quotient is equal to 05 Write out all the remainders from bottom to top This is the solution

For example if the given decimal number is 8453

Division Quotient Remainder

8453 8 1056 5

1056 8 132 0

132 8 16 4

16 8 2 0

2 8 0 2

Then the octal solution is 20405

Subject Eng Literature (The Tempest ndash William Shakespeare) Topic Act I Scene 1 Lines 33 to 67 (End of scene) Date 16th April 2020 (4th Period)

[Students should read the original play and also the paraphrase given in the school prescribed textbook]Summary Questions amp Answers

[SUMMARY OF THE ENTIRE SCENE]

o The play starts with the scene of a severe storm at sea Alonso (King of Naples) Sebastian (Alonsorsquos brother) Ferdinand (Alonsorsquos son) Gonzalo Antonio (the usurping Duke of Milan) are in a ship in the midst of the storm

o The mariners are trying their best to control the vessel from running aground and are totally following the orders of their Master the Boatswain They have scant success

o The mariners become extremely unhappy and annoyed when most of the passengers arrive on the deck thereby hampering their effort to save the ship There is serious confrontation between them and the passengers who are part of the Kingrsquos entourage

o The mariners could not save the ship

SUMMING-UP

(i) Vivid description of the scene which gives a realistic description of terror and confusion of a tropical storm

(ii) Shows Shakespearersquos accuracy of knowledge in describing the naval operations and also matters of seamanship

(1) GONZALO Ill warrant him for drowning (L 45-57)

though the ship were no stronger than a nutshell and as leaky as an unstanched

wenchBOATSWAIN Lay her a-hold a-hold Set her two courses Off to

sea again lay her offMARINERS All lost To prayers to prayers All lostBOATSWAIN What must our mouths be coldGONZALO The king and prince at prayers Lets assist them

For our case is theirsSEBASTIAN Im out of patienceANTONIO We are merely cheated of our lives by drunkards

This wide-chopped rascal - would thou mightst lie drowning the washing of ten tides

(a) What does Antonio say at the insolent manners of the boatswain just before the given passage

Being irritated at the insolent manners of the boatswain just before the given extract Antonio the Duke of Milan calls him a worthless dog son of a woman without any morals an arrogant and disrespectful noisemaker He says that the boatswain deserved to be hanged(b) What statement does Gonzalo repeat about the boatswain

Gonzalo shows his faith that the boatswain is not destined to die by drowning He is destined to be hanged and nothing can alter this decree of destiny He says that even if the ship was as frail as a nutshell the boatswain could not be drowned for his destiny was to be hanged(c) What do the passengers do when they have lost all hope of their survival

When the passengers have lost all hope of survival they take

(iii) The opening scene justifies the title ndash The Tempest

UNANSWERED QUESTIONS

(i) The King always travels with his entire fleet including his soldiers Where were the other ships

(ii) Why was the ship in that area Where was it coming from or going where

(iii) The ship broke apart What happened to those who were in the ship

(We shall get the answer to the above questions as the play progresses)

leave of life with fervent prayers The mariners take their last hearty drink and are ready for death(d) What blame does Antonio put upon the mariners and the boatswain Antonio rebukes the mariners that these drunkards have brought them to the present crisis by neglecting their duties He blames them saying that they are going to lose their lives entirely for the negligence of the boatswain and his fellows(e) What does Antonio say while cursing the boatswain

Antonio gives vent to his wrath upon the boatswain in particular He calls the boatswain a wide-mouthed rascal who deserves to be hanged on the sea-shore at low water mark so that ten tides might wash over his body and take out of him all the liquor that he has been drinking

Class XIISubject Topic Summary ExecutionHistory Topic

1 1935 ACT AND WORKING OF PROVINCIAL AUTONOMYCONGREE AND OTHER MINISTERSSUB TOPIC GOVERNMENT OF INDIA ACT1935

Government of India Act 1935 This act established a lsquoFederation of Indiarsquo made of British Indian provinces and Indian states and provided for autonomy with a government responsible to the elected legislature in every provinceThis act introduced abolition of Diarchy at provinces The entire provincial administration was introduced to the responsible ministers who were controlled and removed by the provincial legislature The provincial autonomy means two things First The provincial governments were wholly responsible to the provincial legislature Secondly Provinces were free from outside control and interference in the large number of matters The act divided the powers between the centre and provinces in terms of three lists- Federal list( for centre) Provincial list (for province) and concurrent list (for both) Residuary powers were given to the viceroy In the election under the government of India Act the Congress swept the poll the mandate of the people came in favour of the congress so far as general Hindu seats were concerned The Congress did not get a single Muslim seates in Bombay CP UP Sind and BengalIn five provinces Congress had yhe clear majority In BengalNWFPAssam and Bombay Congress emerged as a single largest partyOn the other side the performance of the Muslim League was badThus the Congress formed ministers in 7 provinces out of 11 provinces Coalition ministry was also formed in two other provincesOnly BENGAL AND Punjab had non- congress ministries

1 QUESTION What was the main change introduced by the Government of India ActANSWER a) The Act gave more

autonomy to the provinces b) Diarchy was abolished at the

provincial levelsc) The Governor was the head of

the executived) There was a council of

ministers to advise him The ministers were responsible to the provincial legislatures who controlled them The legislature could also remove the ministers

e) The Governors still retained special reserve powers

2 QUESTION Why did the federal scheme introduced by the Government of India Act 1935 never come into operation

ANSWER The Federal structure of the Government of India was to be composed with the Governor General and Council of ministers The Federal legislature was to be Bicameral legislature- The council of states and the House of Assembly The ministers were to be chosen by the Governor general and they were to hold the office during his pleasure

The provinces of British India would have to join the federation but this was not compulsory for the princely states

This federation never materialised because of the lack of support from the required number of

princely statesThis act was refused and

rejected by the princes the Congress and the Muslim League

Thus both Congress and the League participated in the election of 1937 Thus the federal part was never introduced but the provincial part was put into operations

Bengali 2nd

Language

াচেরর পরাথCনা(কহিতা )

াচেরর পরাথCনা কহিতাটি কহি (ঙখ দেঘাচে4র দো আচো য কহিতায় াচেরর পতর হমায়ন কঠিন দেরাচেগ আxানত ার ঈশবর া আললার কাচেছ পরাথCনা কচেরচেছন তার পচেতরর ীন হিফহিরচেয় হিচেত এই কহিতায় ার পচেতরর ীন হিভbা দেচেয়চেছন ারার এমনহিক হিনচের ীন হিসCচেনর হিহিনমচেয় হিতহিন তার দেছচের ীন হিফচের দেপচেত দেচেয়চেছন তার দেছচের এই দেরাচেগর ন য হিতহিন হিনচেচেকই ায়ী কচেরচেছন তার হিনচের করা পাপচেকই হিতহিন ায়ী কচেরচেছন এছাা রানৈনহিতক ও আথCসামাহিক অসথার কথা তচে ধরা চেয়চেছ এই কহিতায় ার তার হিনচের পাপ কমCচেকই ায়ী কচেরচেছ ার অন যায় ভাচে দেপহি((হিকতর মাধ যচেম অপররা য কচেরচেছ আর এই অন যায় কাচের ন যই তার পহিরাচের হিপযCয় এচেসচেছ দে এক পরকার মানহিক নধন ইহিতাচেসর ার হিপতা চেয় সবাভাহিকভাচে ভাচোাসা দে মমতা দেথচেক মকত চেত পাচেরনহিন তাই হিপতা চেয় আললা া ভগাচেনর কাচেছ পতর হমায়চেনর পরানহিভbা দেচেয়চেছন ার আললা া ভগাচেনর কাচেছ াহিনচেয়চেছন তার হিনচের ীন হিসCন হিচেত হিতহিন রাী তার হিহিনমচেয় পচেতরর ীন হিফচের দেপচেত দেচেয়চেছন াচেরর হিপতসভ হিচেকর কথা এই কহিতায় ফটিচেয় দেতাা চেয়চেছ হিপতা পচেতরর হিরাহিরত মান নধচেনর কথা তচে ধরা চেয়চেছ

হিচে(4 হিকছ াইচেনর তাৎপযC১) ldquoদেকাথায় দেগ ওর সবচছয দেৌন দেকাথায় কচেরায় দেগাপন bয়ldquoউততর) াচেরর পতর হমায়ন কঠিন দেরাচেগ অসসথ তাই তার দেযৌন াহিরচেয় যাচেচছ এই দেরাচেগ তাচেক দেগাপচেন কচেরকচের াচেচছ তার সক (হিকত ধীচের ধীচের bয় চেচছ তাই হিপতা চেয় ার আললার কাচেছ হমায়চেনর পরান হিভbা দেচেয়চেছন২) ldquoাগাও (চেরর পরাচেনত পরানতচের ধসর (ন দেযর আান গানldquoউততর) াচেরর পতর হমায়ন কঠিন দেরাচেগ আxানত তাই ার আ দে(াচেক মমCাত (চেরর পচেথ পরানতচের আান গান ধবহিনত দোক দেসই আান গান আললার কাচেছ দেযন চে যায় আললা দেযন এই আহিতC শচেন পচেতরর ীন হিফহিরচেয় দেয় ৩)ldquoনাহিক এই (রীচেরর পাচেপর ীানচেত দেকানই তরারণ দেনই ভহি4চেতরldquoউততর) হমায়চেনর অসসথতার ন য ার হিনচেচেকই ায়ী কচেরচেছন কারন ার অচেনক রা য অন যায় ভাচে কচেরচেছ তাই তার এই পাপ কাচের ন য তার ঘচের আ হিপ এচেসচেছ এই অন যায় কাচের ন য তার মহিকত দেনই তাই ার আললার কাচেছ এই পাপ কাচেযCর ন য bমা পরাথM

Hindi 2ndlang

-ासी(जयशकर परसा-)

-ासी जयशकर परसा- की एक ऐसी कहानी ह जिजसम भारतीय ससकनित और राषटरीयता का सवरगजीतहोता ह इस कहानी म इरावती एक निहद कनया ह जिजस मलअचछो न मलतान की लट म पकडा और -ासी बना दि-या उस 500 दि-न -कर काशी क एक महाजन न खरी-ा दसरी -ासी निफरोजा ह वह गलाम ह निफरोजा को छडान क कतिलए अहम- को 1000 सोन क कतिसकक भजन थ जो अभी तक नही आए थ राजा साहब कठोर होत हए भी निफरोजा को निबना धनराकतिश क कतिलए उस म कर -त ह वनिफरोजा को अहम- को समझान की बात कहत हकहानी क अत म हम -खत ह निक इरा वती और जाटो क सर-ार बलराज का मिमलन होता हअहम- को यa म मार दि-या जाता ह वहा निफरोजा की परसननता की समामिध बनती ह वहा एक फल चढती ह और डीजल आती ह निफरोजा उस समामिध की आजीवन -ासी बनी रहती हलखक अपन उददशय अथात -ास परथा पर परकाश डालन और इस परथा क कारण होन वाल -ातो क दखो को दि-खान म पणता सफल हए ह

helliphellipContinue to next

Biology Reproductio Today we will discuss about vegetative Q1 Name some vegetative propagules

n in Organisms

propagation of plants The process of multiplication in which fragments of plant body function as propagule and develop into new individual is called vegetative propagation The units of such propagation are runner rhizome tuber bulb etc

and the speciesinvolvedVegetative propagules

Parts involved

Bulb StemBulbil BulbilRhizome Stem Runner Stem Tuber Stem Offset Stem Leaf buds Leaves Suckers Stem

Corns Stem stolon

Q2 State advantages of vegetative propagation

i) Rapid methodii) Sure and easy methodiii) Useful in plants that cannot

produce viable seeds or long seed dormancy

iv) Maintains purity of raceQ 3 Banana fruit is said to be parthenocarpic where as turkey is said to be parthenogenetic WhyBanana develops without fertilization from an unfertilized ovary thus is parthenocarpicIn turkey the ovum or female gamete developinto a new chick without fertilization thus isparthgenetic

Q4 Why is water hyacinth is called as a ldquoTerror of Bengalrdquo Water hyacinth can

propagatevegetatively all over the water body in a short per short period of time This resulted increased biochemicaloxygen oxygen demand of water body causing mortalityof fishes It is very difficult to get rid off them Thus known as terror of Bengal

Chemistry

Solid state GENERAL CHARACTERISTICS OF SOLID STATEIn nature the particular state of matter is governed by two opposing forces at given set of temperature and pressure These forces are intermolecular force of attraction and thermal energy If intermolecular force of attraction is high as compared to thermal energy particles remains in closest position

Intext QuestionsQ1 Classify the following solids as crystalline and amorphous Sodium chloride quartz glass quartz rubber polyvinyl chloride Teflon

A1 Crystalline

and hence very less movement in particles is observed In this case solid state is the preferred state of matter

Let us revise the general characteristics of solid

i) Fixed mass volume and shape

ii) Strong intermolecular force of attraction

iii) Least intermolecular space

iv) Fixed position of constituent particles

v) Incompressible and rigid

Q2 what type of interactions hold the molecules together in a polar molecular solid[CBSE 2010]A2 The molecules in a solid are held together by van der Waals forces The term van der Waals forces include hydrogen bonding dipole-dipole attraction and London dispersion forces All molecules experience London dispersion forces In addition polar molecules can also experience dipole-dipole interactions So the interactions that holds the molecule together in polar molecular solid are London dispersion force and dipole-dipole interactionsQ3 Write a feature that will distinguish a metallic solid from an ionic solid [CBSE 2010]A3 Metals are malleable and ductile whereas ionic solid are hard and brittle Metallic solid has typical metallic lustre But ionic solid looks dullQ4 Write a point of distinction between a metallic solid and an ionic solid other than metallic lustre [CBSE 2012]A4 Metals are malleable and ductile whereas ionic solid are hard and brittleQ5 Write a distinguish feature of metallic solid [CBSE 2010]A5 The force of attraction in

solid Sodium chloride Quartz Amorphous solid Quartz glass rubber polyvinyl chloride Teflon Q2 why glass is considered as super cooled liquidA2 Glass shows the tendency to flow at slower rate like liquid Hence they considered as super cooled liquidQ3 why the window glass of old buildings show milky appearance with timeA3 Glass is an amorphous solid Amorphous solid has the tendency to develop some crystalline character on heating Due to heating in day over the number of years glass acquires some crystalline character and show milky appearanceQ4 why the glass panes fixed to window or doors of old building become slightly thicker at bottomA4 Glass is super cooled liquid It has the tendency to flow down very slowly Due to this glass pane becomes thicker at the bottom over the timeQ5 Sodium chloride is a crystalline solid It shows the same value of refractive index along all the direction TrueFalse Give reasonA5 FalseCrystalline solid shows anisotropy in properties That is it shows different values for the given physical property in different direction All the crystalline solids show anisotropy in refractive index Therefore sodium chloride will show different values of refractive index on different directions

Q6 Crystalline solid are anisotropic in nature What does this statement means

between the constituent particles is special kind of electrostatic attraction That is the attraction of positively charged kernel with sea of delocalized electronsQ6 which group of solid is electrical conductor as well as malleable and ductile [CBSE 2013]A6 Metallic solidQ7 why graphite is good conductor of electricity although it is a network (covalent solid)A7 The exceptional property of graphite is due to its typical structure In graphite each carbon is covalently bonded with 3 atoms in same layer The fourth valence electron of each atom is free to move in between different layersThis free electron makes the graphite a good conductor of electricity

[CBSE 2011]A6 Anisotropy is defined asrdquo Difference in properties when measured along different axis or from different directionsrdquo Crystalline solid show different values of some of the physical properties like electrical resistance refractive index etcwhen measured along the different directions The anisotropy in crystalline solid arises due to the different arrangement of particles in different directions

Math Function Composition of functions Think of an industrial plant that produce bottles of cold drinks first there is the operation (or function) f that puts the cold drink inside the bottle followed by the opeartion g that close the bottle with the capThis leads to the following definitionDefinition Let f A rarr B and g B rarr C be two functions Then the composition of f and g denoted by gof is defined as the function gof A rarr C given by gof(x) = g(f (x)) forall x isinA

Definition A function f X rarr Y is defined to be invertible if there exists a function g Y rarr X such that gof = IX and fog = IY The function g is called the inverse of f and is denoted by f -1

Thus if f is invertible then f must be one-one and onto and conversely if f is one-one and onto then f must be invertible This fact significantly helps for proving a function f to be invertible by showing that f is one-one and onto specially when the actual inverse of f is not to be determined

Example 1 Let f 2 3 4 5 rarr 3 4 5 9 and g 3 4 5 9 rarr 7 11 15 be functions defined as f(2) = 3 f(3) = 4 f(4) = f(5) = 5 and g (3) = g (4) = 7 and g (5) = g (9) = 11 Find gofSolution We have gof(2) = g (f(2)) = g (3) = 7 gof(3) = g (f(3)) = g (4) = 7gof(4) = g (f(4)) = g (5) = 11 and gof(5) = g (5) = 11Example 2 Find gof and fog if f R rarr R and g R rarr R are given by f(x) = cos x and g (x) = 3x2 Show that gof ne fogSolution We have gof(x) = g(f(x))=g(cosx) = 3 (cos x)2

= 3 cos2 x Similarly fog(x)=f(g (x))= f(3x2)= cos (3x2) Note that 3cos2 x ne cos 3x2 for x = 0 Hence gof ne fogExample 3 Show that if f A rarr B and g B rarr C are onto then gof A rarr C is also ontoSolution Given an arbitrary element z isin C there exists a pre-image y of z under g such that g (y) = z since g is onto Further for y isin B there exists an element x in A with f(x) = y since f is onto Therefore gof(x) = g (f(x)) = g (y) = z showing that gof is onto Example 4 Let Y = n2 n isin N sub N Consider f N rarr Y as f(n) = n2 Show that

f is invertible Find the inverse of fSolution An arbitrary element y in Y is of the form n2 for some n isin N This implies that n =radicy This gives a function g Y rarr N defined by g (y) =radicy Nowgof (n) = g (n2)=radicn2 = n and fog (y) =f(radicy) = (radicy) 2 y which shows that gof=IN and fog= IY Hence f is invertible with f -1 = g

Political Science

Constitution of India-The Preamble

Summary

Objective of the state-To secure equality of status and of opportunity To promote fraternity among all the citizens To assure the dignity of the individuals and Unity and integrity of the nation

Justice-Justice stands for rule of law absence of arbitrariness and a system of equal rights freedom and opportunities for all in a society India seeks social economic and political justice to ensure equality to its citizens

Liberty-Liberty implies the absence of restraints or domination on the activities of an individual such as freedom from slavery serfdom imprisonment despotism etc The Preamble provides for the liberty of thought expression belief faith and worship

Equality-Equality means the absence of privileges or discrimination against any section of the society The Preamble provides for equality of status and opportunity to all the people of the country

Fraternity-The Preamble declares that fraternity has to assure two thingsmdashthe dignity of the individual and the unity and

Execution

Answer the following questions-

Short notes-1 Equality2 Fraternity3 Justice4 Liberty

Homework-Learn

integrity of the nation The word integrity has been added to the Preamble by the 42nd Constitutional Amendment (1976)

Business studies

Human resource management (chapter 1)

On the day of 1504 2020 I have discussed with you the managerial functions and procurement functions of HRM

Today weare going to discuss about the development function integration functions and maintenance function

Development functions-HRM improves the knowledge skills attitude and values of employees so that they the present and future jobs more effectively it includes

1) Development functions of HRM

a) Performance appraisal = It implies systematic evaluation of employees with respect to their performance on the job and their potential for development

b) Training =It is the process by which employees learn knowledge skills and attitudes to achieve organisational and personal goals

c) Executive development = It is the process of developing managerial talent through appropriate program

2) Integration functionsa) HRM reconcile the goals of

organisation with those of its members through integrating function

b) HRM tries to motivate employees to various financial and non financial incentives provided in job specification etc

3) Maintenance functiona) HRM promote and protect the

physical and mental health of employees by providing several types of benefits like housing medical aid etc

b) It Promote Social security measures to employees by providing provident fund pension gratuity maternity benefits

SubjectCOMMERCE

Topic

BUSINESSENVIRONMENT

Summary

Now quickly let us revise the earlier points that we have already done in the last class and let us proceed with the other topics that are there in the chapter

Firstly we will recall the internal and external factors of micro environment and then we

Execution 3 What do you mean by internal factors

in micro environmentAnswerInternal factors refer to all the factors existing within a business firm The internal factors are considered controllable because the enterprise has control over these factors

Development FunctionsPerformance AppraisalTrainingExecution Development

shall proceed in details

Meaning and list of internal and external factors

aInternal factorsInternal factors refer to all the factors existing within a business firm The internal factors are considered controllable because the enterprise has control over these factorsFor an example a company can alter its organization structure policies programmes employees physical facilities and marketing mix to suit the changes in the environmentList of internal factors areCorporate culture mission and objectives top management organizations structure company image and brand equity company resources

b External factorsExternal factors refer to those individual and groups and agencies with which a particular business organization comes into direct and frequent contact in the course of its functioningThese individuals and groups are known as STAKEHOLDERS because they have a stake (financial interest ) in the working and performance of the particular business List of external forces (stakeholders)Customers competitors investors suppliersmiddlemen (marketing intermediaries)financers publics

customers

suppliersfinancers

For an example a company can alter its organization structure policies programmes employees physical facilities and marketing mix to suit the changes in the environment

4 What do you mean by external factors in micro environment

AnswerExternal factors refer to those individual and groups and agencies with which a particular business organization comes into direct and frequent contact in the course of its functioningThese individuals and groups are known as STAKEHOLDERS because they have a stake (financial interest) in the working and performance of the particular business

3Who are stakeholdersSTAKEHOLDERS are individuals and groups who have a stake (financial interest ) in the working and performance of the particular business 4Discuss the internal factors in briefa Corporate CultureThe values beliefs and attitudes of the founders and top management of the company exercise a strong influence on what the cmpaany stands for how it does things and what it considers importantbMission and objectivesThe business philosophy and purpose of a comoany guide it prioritiesbusiness strategiesproduct market scope and development scope

cTop management structurethe composition of board of directors the degree of professionalization of management and the organizational structure of a company have important bearing on its business decisions

dPower structureThe internal power relationship between the board of directors and the chief executive is an important factor

e Company image and brand equityThe image and brand equity of the company play a significant role in raising finance forming alliance choosing dealers and suppliers launching new products entering foreign markets

5 What is Macro environmentAnswerMacro environment refers to the general

competitors

middlemen

publics

Fig STAKEHOLDERS OF A COMPANY

Apart from micro environment the other main dimension of business environment isMacro environment Macro environment refers to the general environment or remote environment within which a business firm and forces in its micro environment operateA company does not directly or regularly interact with the micro environmentTherefore macro environment is also known as indirect action EnvironmentThe macro environment forces are less controllable than the micro forces

Macro environment consists of the following components

POLITICAL AND LEGAL ENVIRONMENT

ECONOMIC SOCIAL AND ENVIRONMENT

CULTURAL

ENVIRONMENT

TECHNOLOGICAL ENVIRONMENT

Fig COMPONENTS OF MACRO ENVIRONMENT

environment or remote environment within which a business firm and forces in its micro environment operateA company does not directly or regularly interact with the micro environmentTherefore macro environment is also known as indirect action EnvironmentThe macro environment forces are less controllable than the micro forces 6 What are the components of macro environmenta Political and legal environmentb Economic environmentc Social and cultural environmentd Technological environment

Computer Science

Logic gates

Digital systems are said to be constructed by using logic gates These gates are the AND OR NOT NAND NOR EXOR and EXNOR

BUSINESS FIRM

gates The basic operations are described below with the aid of truth tables

AND gate

The AND gate is an electronic circuit that gives a high output (1) only if all its inputs are high A dot () is used to show the AND operation ie AB Bear in mind that this dot is sometimes omitted ie ABOR gate

The OR gate is an electronic circuit that gives a high output (1) if one or more of its inputs are high A plus (+) is used to show the OR operationNOT gate

The NOT gate is an electronic circuit that produces an inverted version of the input at its output It is also known as an inverter If the input variable is A the inverted output is known as NOT A This is also shown as A or A with a bar over the top as shown at the outputs The diagrams below show two ways that the NAND logic gate can be configured to produce a NOT gate It can also be done using NOR logic gates in the same way

NAND gate

This is a NOT-AND gate which is equal to an AND gate followed by a NOT gate The outputs of all NAND gates are high if any of the inputs are low The symbol is an AND gate with a small circle on the output The small circle represents inversion

NOR gate

This is a NOT-OR gate which is equal to an OR gate followed by a NOT gate The outputs of all NOR gates are low if any of the inputs are highThe symbol is an OR gate with a small circle on the output The small circle represents inversion

EXOR gate

The Exclusive-OR gate is a circuit which will give a high output if either but not both of its two inputs are high An encircled plus sign ( ) is used to show the EOR operation

EXNOR gate

The Exclusive-NOR gate circuit does the opposite to the EOR gate It will give a low output if either but not both of its two inputs are high The symbol is an EXOR gate with a small circle on the output The small circle represents inversion The NAND and NOR gates are called universal functions since with either one the AND and OR functions and NOT can be generated

Note A function in sum of products form can be implemented using NAND gates by replacing all AND and OR gates by NAND gates A function in product of sums form can be implemented using NOR gates by replacing all AND and OR gates by NOR gates

Logic gate symbols

Table 2 is a summary truth table of the inputoutput combinations for the NOT gate together with all possible inputoutput combinations for the other gate functions Also note that a truth table with n inputs has 2n rows You can compare the outputs of different gates

Logic gates representation using the Truth table

Example

A NAND gate can be used as a NOT gate using either of the following wiring configurations

Subject Eng Literature (The Tempest ndash William Shakespeare) Topic Act III Scene 3 Lines 53 to 110 (End of the scene) Date 16th April 2020 (2nd Period)

[Students should read the original play and also the paraphrase given in the school prescribed textbook]Summary Questions amp Answers

o Seeing this strange scene all are inclined to believe the tales told by travelers that there truly are ldquounicornsrdquo and ldquothe phoenixrsquo thronerdquo

o As they are about to sit down to the feast the banquet is snatched away by a harpy (Ariel disguised) A spiritrsquos voice (Arielrsquos voice) denounces Alonso Sebastian and Antonio with particular

1 ARIEL You are three men of sin whom Destiny

(Line 53-58)That hath to instrument this

lower world And what is int the never-surfeited sea

Hath caused to belch up you and on this island

Where man doth not inhabit you rsquomongst men

Being most unfit to live I have made you mad

reference to their crime in expelling Prospero from Milan They have not received any punishment for their deed earlier but the time for their punishment has arrived Upon Alonso it pronounces ldquolingering perdition worse than deathrdquo from which there is no remedy except through sincere repentance Ariel then vanishes in thunder and the shapes enter again and carry away the table

o Prospero watching invisibly is very pleased with the performance of Ariel and his (Prosperorsquos) ldquomeaner ministersrdquo All his enemies are now in his power and are in a fit of desperation He then leaves them and goes to see how Ferdinand and Miranda are getting on

o Alonso is now much humbled and penitent with the after effect of the spiritrsquos denunciation of his crimes He believes that his son is lost forever After this all disperse being stricken mad by the speech of the spirit

o Gonzalo fearing that they may do violence to themselves or to one another follows them and bid others to follow

(a) To whom does Ariel disguised as a harpy call the three sinners What game did Fate of Destiny play with

them

The three sinners called by Ariel are Alonso Sebastian and Antonio It was Destiny which had caused the ocean to cast the three sinners on the shore Though the ocean is all the time devouring whatever appears on its surface and is never satisfied with its continual swallowing of the ships and men in the present case the ocean had cast these three sinners on the shore without killing them

(b) Who had jointly been responsible for the conspiracy against Prospero What is Prosperorsquos purpose behind all this

Three men Alonso Sebastian and Antonio had jointly

been responsible for the conspiracy against Prospero They had driven out Prospero form Milan Prosperorsquos purpose is to make these three sinners realize the wrong they had done He wants them to repent for their criminal deeds because repentance leads to self-esteem(c )What does Ariel (the harpy) tell Alonso and his companions when they take out their swords to attack him

Seeing them drawing their swords Ariel (harpy) tells them that he and his companions are the instruments of destiny and that it is not possible for human beings to do them any injury He says that the swords of human beings can not injure even a minute part of his feathers Their swords are as ineffective against him and his companions as against the wind or the water

(d) Give the explanatory meanings of the following expressions in the context of the above extract

(i)Never surfeited (ii) Belch up (iii) lsquomongst men

(i) Never surfeited never led to satisfaction

(ii) Belch up cast ashore(iii) lsquomongst men in human

society2

I and my fellows (Line 60-65)

Are ministers of Fate The elementsOf whom your swords are tempered may as wellWound the loud winds or with bemocked-at stabsKill the still-closing waters as diminishOne dowl thats in my plume

IMPORTANT PASSAGES EXPLAINED

The elements

(Line 61-66)Of whom your swords are tempered may

as wellWound the loud winds or with

bemocked-at stabs

(a) Who is lsquoIrsquo Who are his lsquofellowsrdquo

lsquoIrsquo is referred to Ariel in disguise of a harpy His lsquofellowsrsquo are other spirits serving Prospero the real Duke of Milan who has acquired supernatural powers after being banished from his Dukedom Prospero has settled in this uninhabited island

(b) What are the elements that have temperrsquod the swords Why will it not work against the speaker

The swords (of Alonso and his companions) are tempered by metal (steel) which is taken out of the earth and refined by

Kill the still-closing waters as diminishOne dowl thats in my plume My fellow

ministersAre like invulnerable

In these words Ariel reminds the King and his companions of the utter futility of drawing swords against himself and his fellows Ariel drives Alonso Antonio and Sebastian the three men of sin to desperation ndash a state in which men do violence to themselves They draw swords to strike Ariel But Ariel reminds them that he and the other spirits are the ministers of destiny and nothing can wound them The steel of which their swords are made of may cut the wind or water which being divided always closes up again Even supposing that such things may be possible it is quite impossible that their swords will cut one feather in their plume They are incapable of being wounded by any sword of man Hence it is foolish on their part to attempt to strike at Ariel and his fellow-spirits

For which foul deed

(Line 72-75)The powers delaying not forgetting

haveIncensed the seas and shores yea all the

creatures Against your peace

Ariel enters like a harpy and remaining invisible tells Alonso Sebastian and Antonio that he and other harpies are the agents of Destiny appointed to carry out her decrees He tells them that their punishment for the crime against Prospero which has been so long deferred is now to fall upon them He reminds them that they had expelled Prospero from Milan and set him and his innocent child adrift on the sea and that the sea had paid them back for their sin by the shipwreck and by the calamities they have suffered He tells them that the powers above which did not forget this mean treachery but only deferred the punishment have now engaged the seas and the shores and all living beings including him and his comrades against them The very elements and supernatural agency Ariel adds have taken up the avenging of their crime against Prospero

the action of fire It may cut the wind or water which being divided always closes up again

The sword will not work against the spirits and the harpy because they are the ministers of destiny and nothing can wound them nor it will cut a single feather in their plume

(c )What is the meaning of lsquodowlrsquo in the last line

The term lsquodowlrsquo means a filament or the smallest part of a feather In this context Ariel in disguise of harpy says that their sword cannot even damage the smallest filament of their (Arielrsquos and other spirits) feathers as they are incapable of being wounded by any sword of man

(d) What does the speaker remind the listeners about

Ariel in disguise of harpy reminds Alonso the King of Naples Sebastian Alonsorsquos brother and Antonio the present Duke of Milan and the treacherous brother of Prospero as they being three men of sin He even reminds them that their punishment for their crime against Prospero which has been so long deferred now falls upon them He reminds them that they have expelled Prospero from Milan and has set him along with his innocent infant daughter adrift on the sea So the sea has paid them back for their sin by their shipwreck and the calamities they have suffered since then The harpy rebukes Alonso of his sin that has incensed the Gods and has deprived him of his son as a punishment

(e) How do they respond

When Ariel in disguise of a harpy reminds Alonso Sebastian and Antonio of their past misdeeds and sin Alonso has a look of terror and confusion in his eyes He utters the words of sincere repentance wrung out of his conscience-stricken heart It appears to him that all the elements of nature the sea-waves the wind and the thunder proclaiming a loud voice in the name of Prospero and the crime Alonso has committed against him They are calling upon him to repent There is a deep storm raging in Alonsorsquos breast and the echoes of that storm are ringing in his ears like a clear note of wind-instrument A note of denunciation of Alonsorsquos crime leaves him much humbled and penitent and confirms his belief that his son is lost forever But Sebastian and Antonio shows some courage instead of repentance They wish to kill the spirits or devils if it appears

3

Of my instruction hast thou nothing bated (Line 85-93)

In what thou hast to say So with good life

And observation strange my meaner ministers

Their several kinds have done My high charms work

And these mine enemies are all knit upIn their distractions They now are in my

powerAnd in these fits I leave them while I visitYoung Ferdinand whom they suppose is

drownedAnd his and mine loved darling

Methought the billows spoke and (Line 96-99)

told me of itThe winds did sing it to me and the

thunderThat deep and dreadful organ-pipe

pronouncedThe name of Prosper It did bass my

trespass

These are the words of contrition coming from Alonso Ariel has driven him to a deep repentance for conspiring with Antonio against Prospero He now feels a sincere remorse It appears to him that all the elements of nature the sea-waves the wind and the thunder proclaimed with a loud voice the name of Prospero and the crime Alonso had committed against him They are calling upon him to repent There is a deep storm raging in Alonsorsquos breast and the echoes of that storm are ringing in his ears like the clear note of a wind-instrument

Comment These are the words of sincere repentance wrung out of the conscience-stricken heart of Alonso Alonso who is the lesser villain is the first to give way to remorse under the effect of Arielrsquos speech The words of Ariel seem to him to be the voice of conscience speaking to him He is driven to desperation a state in which he might do violence to his life

(a) Identify the speaker State the context

Prospero the ruler of the island is the speaker The famous banquet scene has been enacted very well Ariel and his junior spirits have played their roles excellently Prospero is glad to say words of praise for them(b) In what way the speakerrsquos instructions have been carried out

According to Prosperorsquos instructions a banquet was presented before the King of Naples and his companions when they were tired and hungry Just when they were preparing to eat the feast the banquet was suddenly removed by exercising supernatural powers All this was done by Ariel Prosperorsquos chief assistant and a powerful spirit

Ariel not only made the feast disappear but also delivered his speech blaming the King and his two companions for their past wicked deeds He warned them to repent for their misdeeds or suffer forever on that uninhabited island

(c) Who are referred to as lsquomeaner ministersrsquo What have they done

Prospero refers as lsquomeaner ministersrsquo to his other lesser spirits who were assisting Ariel in presenting a scene before the kingrsquos party They entered the scene to the accompaniment of music They assumed several strange shapes and brought in a banquet Then they danced about it with gentle actions of salutations thus inviting the King and others to eat the feast

These spirits play their role again when Ariel in the shape of a harpy quits the scene These shapes enter again and dancing with mocking gestures carry away the table

(d) Who are the speakerrsquos enemies What has happened to them

King of Naples Alonso his brother Sebastian and the present Duke of Milan Antonio (Prosperorsquos own brother) are Prosperorsquos enemies With the turn of events they have all been washed ashore on the island which is ruled by Prospero the great magician Actually this happened after the shipwreck caused by a storm which was raised by Prospero with the purpose of bringing these people to his island Prosperorsquos spirits have already confused and terrified these enemies and they are under Prosperorsquos control He can treat them as he likes

(e) What does he say about Ferdinand Explain what is meant by ldquohellip his and mine darlingrdquo

Prospero knows that Alonsorsquos son prince Ferdinand is alive though his father thinks that the prince has been drowned

Prospero refers to his daughter Miranda who is dear to him She is also very dear to Prince Ferdinand who has fallen in love with her They are waiting to be married soon for which they have received Prosperorsquos consent

4

ALONSO O it is monstrous monstrous (Line 95-102)

Methought the billows spoke and told me of it

The winds did sing it to me and the thunderThat deep and dreadful organ-

pipe pronouncedThe name of Prosper It did bass

my trespassTherefore my son ithrsquo ooze is

bedded andIll seek him deeper than eer

plummet soundedAnd with him there lie mudded

(a) In what way does Alonso express his horror when his conscience is awakened by Arielrsquos words

When Alonsorsquos conscience is awakened by Arielrsquos words he expresses his horror at what he has heard He gets the feeling that the waves of the ocean the wind and the loud thunder have spoken to him and uttered the name of Prospero Because of being reminded of his crime in a very loud and rough voice he comes to realize that he has lost his son for his past misdeeds

(b) What does Alonso imagine about his son What does Alonso want to do in his desperate state

Alonso imagines that his son is lying in the mud at the bottom of the sea He feels desperate that he wants to drown himself in the ocean deeper than the plumb-line has ever gone He wants to lie with his son at the bottom of the sea

(c) How do Sebastian and Antonio want to face the evil spirits

Sebastian says that he is not at all afraid of what the harpy has said and that he is prepared to fight any number of such monsters if they appear before him only one at a time Antonio says that he would support Sebastian in the fight against the fiendsyyy

(d) Why does Gonzalo ask Adrian to follow the three men

Gonzalo tells Adrian that all the three men namely Alonso Sebastian and Antonio are in a wild and reckless mood The thought of the heinous crime of which they are guilty has begun to torment their minds So he asks Adrian to follow those three men without loss of time and prevent them from doing anything which the turmoil in their minds might lead them to do

(e) What opinion do you form of Alonso from the above extract

Alonso who is the lesser villain is the first to give way to remorse under the effect of Arielrsquos speech The words of Ariel seem to him to be the voice of conscience speaking to him He is driven to desperation a state in which he might do violence to his life

Subject =Accounts

Ac-12 15420 topic-pL Appropriation ac

PROFIT AND LOSS APPROPRIATION ACCOUNT

MEANING AND PREPARATIONProfit and Loss Appropriation Account is merely an extension of the Profit and Loss Account of the firm The profit of the firm has to be distributed amongst the partners in their respective profit sharing ratio But before its distribution it needs to be adjusted All Adjustments like partnerrsquos salary partnerrsquos commission interest on capital interest on drawings etc are made in this account These adjustments will reduce the amount of profit for distribution This adjusted profit will be distributed amongst the partners in their profit sharing ratio To prepare it at first the balance of Profit and Loss Account is transferred to this account The journal entries for the preparation of Profit and Loss Appropriation Account are given below

1 for transfer of the balance of Profit and Loss Account to Profit and Loss Appropriation Account

(a) In case of Net Profit

Profit and Loss Ac helliphelliphelliphelliphellipDrTo Profit and Loss Appropriation Ac(Net Profit transferred to Profit and Loss Appropriation Ac)

(b)In case of Net Loss

Profit and Loss Appropriation Achelliphelliphellip DrTo Profit and Loss Ac(Net Loss transferred to Profit and Loss Appropriation Ac)

2 for Interest on Capital

For transferring on Interest on CapitalProfit and Loss Appropriation Achelliphelliphellip DrTo Interest on Capital Ac(Interest on capital transferred to Profit amp Loss Appropriation Ac)

3 for Interest on Drawings

For transferring Interest on Drawings Interest on Drawings Achelliphelliphelliphelliphelliphellip DrTo Profit and Loss Appropriation Ac(Interest on drawing transferred to Profit amp Loss Appropriation Ac)

4 For Partnerrsquos SalaryFor transfer of partnerrsquos SalaryProfit and Loss Appropriation Achelliphellip DrTo Salary Ac(Salary transferred to profit amp Loss Appropriation Ac)

5 For Partnerrsquos CommissionFor transferring commissionProfit and Loss Appropriation Achelliphelliphellip DrTo Commission Ac(Commission transferred to Profit and Loss Appropriation Ac)

6 For Transfer of agreed amount to General ReserveProfit and Loss Appropriation Ac helliphellipDrTo General Reserve Ac(Transfer to General Reserve)

7 for share of Profit or Loss appropriation(a) If ProfitProfit and Loss Appropriation Achelliphellip DrTo Partnerrsquos CapitalCurrent Ac(Profit transferred to capitalcurrent Ac)(b) If LossPartnerrsquos Capital Current Achelliphelliphelliphellip DrTo Profit and Loss Appropriation Ac(Loss transferred to capitalcurrent Ac)

THE FORMAT OF PROFIT AND LOSS APPROPRIATION

Profit and Loss Appropriation Account for the year endedhelliphelliphelliphellip

Particulars Amount Particulars Amount

To PL Ac (loss) By pL Ac (profit)

To Interest on capital BY Interest on drawings

To partner`s commission by Partner`s capital Ac ( loss)

To Partner`s salary To Interest on partner`s loan To General Reserve To Partner`s Capital AC (Profit)

Subject= Economics

MOVEMENT ALONG THE DEMAND CURVE (CHANGE IN QUANTITY DEMANDED)In law of demand you have already studied the inverse relationship between price and quantity demanded When quantity demanded of a commodity changes due to change in its price keeping other factors constant it is called change in quantity demanded It is graphically expressed as a movement along the same demand curve There can be either a downward movement or an upward movement along the same demand curve Upward movement along the same demand curve is called contraction of demand or decrease in quantity demanded and downward movement along the same demand curve is known as expansion of demand or increase in quantity demanded

Extention of demandd

price (rs)p A

B Extentionp1 d

Q Q1

Quantity demanded ( in units)

Contraction of demandd

p2 Ccontraction

p APrice (Rs)

d

Q2 Q

Quantity demanded (in units)

Explanation of movement of demand A fall in price from OP to OP1 leads to increase in quantity demanded from OQ to OQ1 (expansion of demand) resulting in a downward movement from point A to point B along the same demand curve DD When Price rises from OP to OP2 quantity demanded falls from OQ to OQ2 (contraction of demand) leading to an upward movement from point A to point C along the same demand curve DD

  • Activity Series of Metals
    • Drawbacks of Rutherfordrsquos model of atom
      • Electromagnetic radiations
      • Properties of electromagnetic radiations
      • Characteristics of electromagnetic radiations
        • Plancks Quantum Theory-
        • Photoelectric effect
          • Intext Questions
            • Logic gates
            • Digital systems are said to be constructed by using logic gates These gates are the AND OR NOT NAND NOR EXOR and EXNOR gates The basic operations are described below with the aid of truth tables
            • AND gate
            • Example
Page 16:  · Web viewSubject. Topic. Summary. Execution. English 1 . Chapter 1 naming words . Page 8. Write the names of these pictures:- Person:-1. father. 2.Firefighter 3.doctor 4 ...

घ) कई दि-नकई दि-न लगातार रोत-रोत उसका रोना तो शान हो गया पर उसक ह-य म शोक भर गया था वह चपचाप बठा आकाश की और टाका करता निक शाय- उसकी काकी कही दि-ख जाए

ldquoदि-न उसन ऊपर आसमान म पतग उडती -खी न जान कया सोच कर उसका निहर-य एक-म खिखल उठाrdquo

क) निकसन पतग ऊपर उडत -खी और वह कयो खश हआ

ख) उसन अपन निपता स कया कहा उनका कया उतर थाश

ग) उसन निफर कया निकया और निकसन उसकी सहायता की

घ) उसकी योजना कया थी उततर -क) शयाम न एक दि-न आसमान म पतग उडती

-खी तो उसन सोचा निक पता आसमान म राम क यहा जाकर रकगी वही पर मरी काकी ह यह सोचकर वह बहत खश हआ

ख) उसन अपन निपता स कहा काका मझ एक पतग मगा -ो उसक निपता न भटक हए मन क भाव स कहा निक मगा -ग यह कह कर उ-ास भाव स वह कही और चल गए पतग नही आई

ग) उसन चपचाप निवशशवर क टगहए कोट स एक चवननी निनकाल ली और सखिखया -ासी क लडक भोला की सहायता स एक पतग मगवानी भोला उसकी बराबर उमर का ही था

घ) उसकी योजना यह थी निक वह अपनी पतग को आकाश म राम क यहा भजगा और उस पतग क सहार उसकी काफी नीच उतर जाएगी इस योजना पर उस परा निवशवास था इसकतिलए वह और भोला -ोनो यह काम करन म लग गए

Continue to nexthelliphellipEVS CHAPTER - 1

(UNDERSTANDING OUR ENVIRONMENT)

Sustainable development

The development that meets the needs of the present without compromising the ability of future generations to meet their own needs is called Sustainable development

Sustainable societies ndash

An environmentally sustainable community is one that meets the current and future basic resource needs of its people in a just and equitable manner without compromising the ability of future generations to meet their basic needs

Q ) What are Eco Villages

Ans - Eco village are the urban or rural communities of people who strive to integrate a supportive social environment with a low impact way of life

Q ) To ensure sustainable development the depletion of renewable resources should not take place at a rate faster than their regeneration Justify your answer

Ans ndash Renewable resources do not have a fixed quantity - more can always be

generated However if the rate of use exceeds the rate of renewal - that is the

source is used more than its being recreated - its continued use will become

used up faster than it can regenerate

To promote sustainable society the following things need to be done ndash

1 Using renewable energy sources 2 By improving the quality of human

health 3 By promoting sustainable agriculture 4 By forming ecovillage

it will eventually be entirely depleted So Toensure sustainable development the depletion of

renewable resources should nottake place at a rate faster than their regeneration

Q ) What do you mean by Sustainable societies

Ans - Sustainable societies are defined as towns and cities that have taken steps to remain healthy over the long term These communities value healthy ecosystems use resources efficiently and actively seek to retain and enhance a locally based economy Sustainable development concerns everybody in a society

Q ) What are the effects of pollution on human health

Ans ndash Some health problem occurs due to air pollution are ndash

Respiratory diseases Cardiovascular damage Fatigue headaches and anxiety Irritation of the eyes nose and throat Damage to reproductive organs Harm to the liver spleen and blood Nervous system damage

Some health problem occurs due to water pollution are ndash

Typhoid Cholera Dysentry Jaundice

Some health problem occurs due to noise pollution are ndash

Fatigue headaches and anxiety High blood pressure Hearing damage

Physics Motion in 1D First go through previous notes Now here we will solve some numerical related to that

Question 3What information about the motion of a body is obtained from the displacement-time graphSolution 3From displacement-time graph the nature of motion (or state of rest) can be understood The slope of this graph gives the value of velocity of the body at any instant of time using which the velocity-time graph can also be drawn

Question 4(a)What does the slope of a displacement-time graph represent(b)Can displacement-time sketch be parallel to the displacement axis Give a reason to your answerSolution 4(a) Slope of a displacement-time graph represents velocity(b) The displacement-time graph can never be parallel to the displacement axis because such a line would mean that the distance covered by the body in a certain direction increases without any

increase in time which is not possible

Chemistry Language of Chemistry

How to balance a chemical equationThere are two methods of balancing an equation(i)Hit and trial method(ii)Partial equation methodBalancing by hit and trial methodThis method consists of counting the number of atoms of each elements on both sides and trying to equalize themTake the following steps(i)Count the number of times (frequency) an element occurs on either side(ii)The element with the least frequency of occurrence is balanced first(iii)When two or more elements have the same frequencythe metallic element is balanced firstExample-1 On heatinglead nitrate decomposes to give lead dioxidenitrogen dioxide and oxygenPb(NO3)2rarrPbO+NO2+O2

In this equationLead occurs twiceNitrogen occurs twiceOxygen occurs four timesSince lead is a metalbalance it firstThe number of atom of lead is equal on the two sidestherefore it needs no balancingNow balance nitrogenOn the reactant sidethere are two atoms of nitrogenwhile on the product side oneSomultiply the product containing nitrogenon the product sideby two Pb (NO3)2rarrPbO+2NO2+O2Nowthe number of oxygen atoms on the reactant side 6while on the product sideit is 7Somultiply the entire equation by 2except oxygen to get balanced equation2Pb(NO3)2rarr2PbO+4NO2+O2Multiplication by 2 is done only when atoms of all the elements except one element are balanced and the unbalanced atom occurs separately at least once and also there is a difference of only one such atom

Math Topic AlgebraChapter

Factorisation

Study item Difference of two squares a2 ndash b2 = (a+b) (a-b)1) (i) 4x2ndash 25y2

= (2x) 2 ndash (5y) 2= (2x + 5y) (2x - 5y)

(ii) 9x2 ndash 1= (3x)2ndash(1)2= (3x + 1)(3x ndash 1)

2) (i) 150 ndash 6a2= 6(25 ndash a2)= 6(5)2 ndash(a)2= 6 (5 + a) (5 ndash a)

(ii) 32x2 ndash 18y2=2(16x2 ndash 9y2)=2(4x)2 ndash (3y)2= 2(4x + 3y)(4x - 3y)3)(i) (x ndashy )2 ndash 9 = (x ndash y )2 ndash (3)2= (x ndash y + 3) (x ndash y ndash 3)(ii) 9(x + y) 2ndash x 2= (3)2(x + y)2 ndash (x)2=3(x + y)2 ndash (x)2= (3x +3y ) 2ndash(x)2= (3x + 3y + x)(3x +3y ndash x)= (4x + 3y) ( 2x + 3y )

Commercial studies

Basic accounting terms

Today I will give you some questions from the previous study material

Questions1) Define accounting2) What do you mean by debit and

credit

3) Explain the types of account4) Define the following terms

a) Assetsb) Capitalc) Purchased) Debtorse) Transactions

5) Name the types of accounts given below

a) Krishnas accountb) Machinery accountc) Royalty accountd) Salary accounte) Furniture accountf) Audit fee account

Economics Revision Today I will give you some revision questions

Questions1) What do you mean by the terms

rdquowantsrdquo2) Write the difference between

consumer goods and producer goods

3) Define the term utility 4) Explain the different types of utility5) Define

a) Total utilityb) Marginal utility

Subject Eng Literature (The Merchant of Venice ndash William Shakespeare)Topic Act I Scene 3 Lines 1 to 48 (Shylock hellip Cursed be my tribe if I forgive him) Date 16th April 2020 (5th Period)

[Students should read the original play and also the paraphrase given in the school prescribed textbook]Summary Questions amp Answers

This scene takes place in Venice and we are introduced to the rich Jew Shylock Bassanio and Shylock are talking and Bassanio tells Shylock that he wants a loan of three thousand ducats for three months on the personal security of Antonio

o Shylock feels glad because he will be able to bind down Antonio by means of a bond on account of the loan but he tells Bassanio that all the fortunes of Antonio being invested in the merchant ships on the sea it is difficult to depend upon his credit Even under such circumstances Shylock is willing to advance the money on the personal security of Antonio

o Bassanio then invites Shylock to dine with him Shylock says that he is prepared to do anything with the Christians but not eat or drink or pray with them

o While Bassanio and Shylock are talking Antonio appears on the scene Shylock does not seem to take any notice of Antonio but goes on brooding within

(1) SHYLOCK Ho no no no no- my meaning in (Line 15-26)saying he is a good man is to have you understand me that he is sufficient Yet his means are in suppositionhe hath an argosy bound to Tripolis another to the Indies I understand moreover upon the Rialto he hath a third at Mexico a fourth for England and other ventures he hath squanderd abroad Butships are but boards sailors but men there be land-rats and water-rats land-thieves and water-thieves I mean pirates and then there is the peril of waters winds and rocks The man is notwithstanding sufficientmdashthree thousand ducats mdashI think I may take his bond

(a) Who is talking in the beginning of this scene What does Bassanio want from Shylock How does Shylock feel

In the beginning of the scene Bassanio and Shylock are talking to each other Bassanio wants to borrow three thousand ducats from Shylock for three months on the security of Antonio Shylock feels glad at heart that he will get the opportunity of binding Antonio with a bond(b) What risks does Shylock weigh in advancing the money

Shylock says that Antonio has invested all his capital in trading by sea-going ships But the ships are made of wood and the sailors of those ships are ordinary human beings The wood can

himself how he hates Antonio because of his being a Christian because he abuses Shylock in public places Shylock decides that if ever he can get Antonio to his advantage he will teach him a lesson

come to harm and men can commit mistakes and thus the capital invested in ships may be lost Then there are other dangers The goods loaded on the ships can be damaged by rats and thieves which are found both on land and water The ships can also be harmed through sea-storms submerged rocks etc(c) What two important functions does this scene have

The scene has two important functions First it completes the exposition of the two major plot lines of the play Antonio agrees to Shylockrsquos bond ndash three thousand ducats for a pound of flesh and second and more important dramatically this scene introduces Shylock himself In this scene Shakespeare makes it clear at once why Shylock is the most powerful dramatic figure in the play and why so many great actors have regarded this part as one of the most rewarding roles in all Shakespearean dramas(d) Where does this scene take place What kind of treatment has Antonio been giving to Shylock What does Shylock say when Bassanio invites him to dine with him

The action of this scene takes place in Venice Antonio has been in the habit of behaving harshly with Shylock ndash spitting on his beard and footing him like a stranger cur When Bassanio invites Shylock to dine Shylock says that he is prepared to do anything with the Christians but not eat and drink or pray with them

(2) SHYLOCK How like a fawning publican he looks (Line 38-48)I hate him for he is a Christian

But more for that in low simplicity

He lends out money gratis and brings downThe rate of usance here with us in VeniceIf I can catch him once upon the hipI will feed fat the ancient grudge I bear him

He hates our sacred nation and he railsEven there where merchants most do congregateOn me my bargains and my well-won thriftWhich he calls interest Cursed be my tribeIf I forgive him

(a) What is the context in which these words are spoken and what is the idea expressed in it

These remarks are made by Shylock when he sees Antonio coming along after Bassanio told him that the merchant will be his surety for the bond The above mentioned passage reveals Shylockrsquos hatred for Antonio Shylock says that he hates Antonio because he is a Christian and also because he gives loan without taking interest on them thereby bringing down the rate of interest in Venice(b) Explain the meaning of the phrase lsquoa fawning publicanrsquo

The phrase lsquoa fawning publicanrsquo refers to Roman tax collector It is a term of contempt and hatred on the lips of a Jew lsquoFawning Publicansrsquo were Roman tax-gatherers whose ordinary bearings towards the Jews was bullying but whose false pose of lsquohumility and contritionrsquo is touched upon in the parable in New Testament(c ) What light does the above passage throw on the character

of Shylock

The above mentioned speech of Shylock reveals him to be a wicked character having an extreme greed for wealth His intense hatred for Antonio is unjustified He hates Antonio only because he is a Christian and because he lends money without taking any interest on it thereby adversely affecting Shylockrsquos business of lending money on high interest(d) What information do you gather about Antonio from the above given lines

Shylockrsquos statement throws a valuable light on the character of Antonio Antonio appears to be a good Christian and a good human being He helps the people in need by lending them money without charging any interest on it He is a man of simple and good nature This very goodness makes him Shylockrsquos enemy(e) What does Shylock debate within himself and when To whom are the lines mentioned above addressed to

When Bassanio asks the Jew to lend him three thousand ducats on Antoniorsquos surety Shylock begins to debate within himself as to how he should exploit the opportunity of a business deal with his old enemy Antonio

The lines mentioned above are not addressed to anyone The lines are a soliloquy ie a speech made by a character to himself and not meant to be heard by the other characters present

Class XSubject Topic Summary ExecutionEnglish

LiteratureThe Blue Bead 2nd part

Things took a turn for the worst and all of a sudden a crocodile attacked the woman biting on the womanrsquos leg At that moment Sibia got up sprinted grabbed the hay fork and stabbed the crocodile in the eye with all her power Immediately the crocodile let go and went away Sibia saw a small blue bead lying by the river she grabbed it Since she was poor she didnrsquot have necklace Shersquod always wanted one like the other women now she could make one with the blue bead After that she went home and told her mother all about it

Hindi 2nd

Langबड घर की बटी( मशी परमच-)

lsquoबड घर की बटी कहानी का उददशय मधयम वग की घरल समसया को सलझा कर सगदिठत परिरवार म मिमल जलकर परम स रहन का स-श -ना हघर म शानित सथानिपत करन की जिजमम-ारी नारी की होती ह यदि- नारी समझ-ार ह उसम धय और परिरवार क परनित परम ह तो कोई भी घटना परिरवार को निवघदिटत नही कर सकती या कहानी परिरवार को सगदिठत करत हए परम सौहा- स एक दसर की भावनाओ को समझ करउनका सहयोग करत हए जीवन यापन करन की पररणा -ती हमशी परमचदर जी न इस कहानी म सय परिरवार का परनितनिनमिधतव निकया ह यह कहानी बनी माधव सिसह जो गौरी पर क जमी-ार क उनक -ो पतरो की हशरी कठ लाल निबहारीशरीकात का निववाह एकजमी-ार घरान की पतरी आन-ी स हआ थाआन-ी न ख- को ससराल क वातावरण म ढाकतिलया थाएक दि-न आन-ी का अपन -वर लाल निबहारी स झगडा हो जाता ह -ोनो भाई एक दसर स अलग होन की कोकतिशश करत हसभी बह आन-ी न अपन मधर वयवहार स लाल निबहारी को

ldquoइन नतर निपरय गणो को बीए-इनही -ो अकषर पर नयोछावर कर दि-या था इन -ो अकषर न उनक शरीर को निनबल और चहर को कानित ही बना दि-या थाldquo

क) परसतत पकतियो म निकस वयकति क बार म कहा गया ह

ख) इन पकतियो म कौन स नतर निपरय गणो क बार म कहा गया ह

ग) बीए की निडगरी परापत कर लन पर भी उपय वयकति क सवभाव की कया निवशरषता थी

घ) यह नतर निपरय गण निकस वयकति म निवदयमान थ उसक वयकतितव की कया निवशरषता थी

उततर ndashक) परसतत पकति म गौरी पर गाव क जमी-ार

क बड बट शरीकात क बार म कहा गया ह उसन बहत परिरशरम और उ-म क बा- ba की निडगरी परापत की थी अब वह एक -फतर

घर छोडकर जान स रोक कतिलयाइस पर बनी माधव सिसह न कहा निक बड घर की बटी ऐसी ही होती ह जो निबगडा काम बना लती ह अतः शीरषक साथक ह बड घर की बटी आन-ी ह

म कमचारी थाख) भरा हआ चहरा चौडी छाती और डटकर

खाना आदि- एक सबजी ल जवान क गण मान जात ह परत शरीकात न इनही नतर निपरय गणो को अपनी पढाई पर नयोछावर कर दि-या था

ग) बीए की निडगरी परापत कर लन पर भी उपय वयकति(शरी कठ की शारिररिरक तौर पर निनबल और चहर स कानित ही लगत थ इतना ही नही वह मानकतिसक तौर पर भी निपछड हए थ पाशचातय सामाजिजक कथा उस घणा एव पराचीन सभयता का गणगान उनकी निवचारधारा क परमख अग थ

घ) यह नतर निपरय गण गौरीपर गाव क जमी-ार क छोट बट लाल निबहारी सिसह म निवदयमान थ वह सजीलाजवान था और भस का दध शर दध वह सवर उठकर पी जाता था

ldquoयही कारण था निक गाव की लललन आए उनकी निन-क थी कोई कोई तो उह अपना शतर समझन म भी सकोच ना करती थी सवय उनकी पतनी को इस निवरषय म उनस निवरोध थाldquo

क) उपय पकति म इस वयकति क बार म कहा गया ह

ख) गाव की लललन आए उनकी निन-ा कयो निकया करती थी

ग) उनकी पतनी का कया नाम था उनह निकस निवरषय म अपन पनित क निवरa था और कयो

घ) इस कहानी का कया उददशय ह Continue to next helliphelliphellip

Bengali 2nd Language

ফ ফটক না ফটক( কহিতা )

পর) ldquo(ান াধাচেনা ফটপাচেথ পাথচের পাড হিচেয় এক কাঠচোটটা গাছ কহিকহি পাতায় পার ফাটিচেয় াসচেছldquoক) কার দো দেকান কহিতার অং( ) lsquo(ান াধাচেনা ফটপাচেথ পাথচের পাডহিচেয়lsquo চেত কী দোঝাচেনা চেয়চেছ গ) আচো য অংচে(lsquo এক কাঠচোটটা গাছ lsquoচেত কী দোঝাচেনা চেয়চেছ ঘ) ldquoকহিকহি পাতায় পার ফাটিচেয় াসচেছldquo ----- একথার পরকত অথC কী উততর ) ক) আচো য অং(টি পর যাত কহি সভা4 মচোপাধ যাচেয়র দো lsquoফ ফটক না ফটকrsquo কহিতার অং()কহি সভা4 মচোপাধ যায় হিছচেন দেপরচেমর কহি দেপরমচেক নানা ভহিঙগমায় হিতহিন ফটিচেয় তচেচেছন দেপরম মানচের স মচেতC র সঙগী কহিতার কহিতায় এক রb সb হচেয়র দেপরম াগরচেনর কথা চেচেছন (ান অথCাৎ দেযাচেন দেকান রস দেনই দেযাচেন দেকান মহিনতা দেনই অথ তার মধ দেযও দেপরম থাকচেত পাচের একথাই কহি তচে ধরচেত দেচেয়চেছন একটি মানচে4র মচেন দেযাচেন দেকামতার দেকান সথান দেনই পাথচেরর মচেতা হিনরসতার মচেনর মধ দেযও দেয দেপরম আসচেত পাচের দেস কথাই কহি চেচেছনগ)নারীচের যথC দেপরচেমর ছহি এই কহিতায় অকপচেট উচেঠ এচেসচেছ কহি এই কহিতায় কাটচোটটা গাছ কথাটি যার কচেরচেছন নারী দেয দেপরম দেথচেক হিতাহিত এং দেসই দেপরম সঠিক সমচেয় না পাওয়ার ন য দেপরম সমপচেকC হিচেr4 গৈতরী য় দেপরচেমর দেয গৈহি(ষট য মাধযC য সরসতা দেকামত এই সমসতর হিপরীত যথা রbতা শষকতা কচেঠার তা পরভহিত দোঝাচেত এক কাঠচোটটা গাছ কথাটি যার কচেরচেছনঘ) এাচেন এক নারীর যথC দেপরচেমর কথা হিনহিCপত ভাচে চেচেছন কহি অসমচেয় নারীর ীচেন দেপরম দেচেগচেছ এতহিন তার হয় রb কচেঠার হিছ দেপরচেমর অভাচে ঠাৎ দেসই শষক মরভহিমচেত সচের আভাস এচেসচেছ দেপরম দেযন 4Cার স(ীত তাই পরায় মত গাচেছ কহিকহি পাতা গহিচেয় উচেঠচেছ

Biology Chapter - 01Controlling Air Pollution

Today we will discuss how we control air pollution from domestic combustion

Q1Describe any five ways of reducing air pollution from domestic sources bull The number of pollutants in the air is verylarge and we always try to control them byfollowing ways

i) Solar cooker and solar heater It use no fuel reduce damage of environment by fuel use or reducing deforestation It maintains coolness of house It releases very less orno oil gas or grease

ii) Piped natural gas (PNG) It emits very less by products into the atmosphere As it isdistributed through pipe lines so there iscontinuous supply of fuel is possible

iii) Liquefied Petroleum Gas (LPG) It hasa higher heating value LPG doesntcontain sulphur so it burns a lot cleanerenergy sources It releases very less oralmost no fume in air

iv) Electricity based cooking Emission free cooking alternative for urban dwellers causeselimination of adverse health impactsofindoor air pollution It helps to avoid theinconveniences associated with procurement of LPG

v) Biogas It contains 75 methane whichmakes it an excellent fuel It burns without smoke and biogas plant leaves no residue like ash in wood charcoal etc Thus it isaclean fuel

Economics

Factors of Production

Today firstly we would recall the last class for 5 mins and then we would proceed with the further topics of the chapter

The concept meaning of land characteristics of land and importance of land to be repeated for the absentees as well as the students who were there in the class the previous day

Today we will start with the last portion of land before it the meaning of land to be repeated onceAs by now we all know that

Questions1What do you mean by productivity of landAnswer By productivity of land we mean the capacity of a piece of land to produce a crop

Thus it refers to the average output per unit of landSay per acre per hectare etc= (OutputArea of land)

2 What are the factors influencing the productivity of landAnswer

Natural factors Productivity of land is largely determined by the natural

Land is defined to include not only the surface of the earth but also all other free gifts of nature(for example mineral resources forest resources and indeed anything that helps us to carry out the production of goods and services but is provided by nature free of cost)

We will move on to the last portion of land by discussing Productivity of Land

By productivity of land we mean the capacity of a piece of land to produce a crop

Thus it refers to the average output per unit of land

Say per acre per hectare etc= (OutputArea of land)

With this we shall proceed further with the main factors that determine the productivity of land

Natural factors Human factors Improvements on land Location of land Organisation Ownership of land Availability of capital Proper use of land State help

Note economic development of a country depends upon the quality of its land If the land is fertile it will quicken the pace of development of the country

qualities of land such as fertility etc

Human factors Land cannot produce anything by itself Man has to apply labour on it to produce for himself So productivity of land depends on the knowledge and skills of workers

Improvements on land production of land is affected by land development measures like provision of well or tubewell irrigation proper drainage

State help The government of a country especially less developed country can play a vital role in improving the agricultural productivity by providing better irrigation facilities

Organisation Productivity of land also fdepends upon the way how the factors of production like labour and capital are organised

In order to increase productivity trained workers modern implements scientific methods good seeds are all essential

3 lsquoImproved technology affects the productivity of landrsquo Explain this statement with the help of suitable example Answer Use of improved technology raises the productivity of land Example By using HYV seeds chemical manures and modern machines per hectare output increases

Physics Force (Summary)

Question Write the expression for the moment of force about a given axisSolutionsThe expression for the moment of force is given byMoment of force about a given axis = Force times perpendicular distance of force from the axis of rotationQuestion What do you understand by the clockwise and anticlockwise moment of force When is it taken positiveSolutionsIf the effect on the body is to turn it anticlockwise moment of force is called the anticlockwise moment and it is taken as positive while if the effect on the

body is to turn it clockwise moment of force is called the clockwise moment and it is taken as negative

Math Topic Commercial Mathematics

Chapter Goods and services Tax

Study item Some solved sums from exercise ndash 1 A retailer buys a TV from a wholesaler for Rs 40000 He marks the price of the TV 15 above his cost price sells it to the consumer at 5 discount on the marked price If the sales are intra ndash state and the rate of GST is 12 find

(i) The marked price of the TV(ii) The amount which the consumer pays for the TV(iii) The amount of tax (under GST) paid by the retailer to the central

Government(iv) The amount of tax (under GST) received by the State Government

Solution As the sales are intra- state sale and the rate of GST 12 So GST comprises of 6 CGST and 6 SGSTTherefore a retailer buys a TV from a wholesaler for Rs 40000Therefore the amount of GST collected wholesaler from the retailer or paid by retailer to wholesalerCGST = 6 of Rs 40000 = Rs(6100 times40000) =Rs 2400SGST = 6 of Rs 40000 = Rs (6100 times 40000) =Rs 2400Therefore wholesaler will pay Rs 2400 as CGST and Rs 2400 as SGSTTherefore amount of input GST of retailer Input CGST = Rs 2400 and input SGST = Rs 2400Again the retailer marks the price of the TV 15 above his cost price(i) The marked price of the TV

= Rs 40000 + Rs 40000times15= Rs 40000 + Rs 40000times 15100= Rs 40000 + Rs 6000Rs 46000But the retailer sells it to consumer at 5 discount on the marked priceCost price after discount = Rs 46000 ndashRs46000times 5100 =Rs 46000 ndashRs 2300= Rs 43700Therefore the amount of GST collected retailer from consumer or paid by consumer to retailerCGST = 6 of Rs 43700 =Rs ( 6100 times43700)Rs 2622SGST = 6 of Rs 43700 = Rs (6100 times 43700) =Rs 2622Amount of the output GST of retailer Output CGST = Rs 2622 and output SGST = Rs 2622

(ii) The amount which the consumer pays for the TV= cost price of TV to consumer + CGST paid by consumer + SGST paid by consumer= Rs 43700 + Rs 2622 + Rs 2622= Rs 48944

(iii) The amount of tax (under GST ) paid by the retailer to the central Government=CGST paid by retailer = output CGST ndash input CGST=Rs 2622 ndash Rs 2400=Rs 222

(iv) The amount of tax ( under GST ) received by the State Government = SGST paid by wholesaler + SGST paid by retailer= Rs 2400 + output SGST ndash input SGST=Rs 2400 + Rs 2622 ndash Rs 2400=Rs 2400 + Rs 222= Rs 2622

Commercial studies

Stakeholders Today I am going to give some revision questions from the previous study material

Questions1) State the two expectations of

employees from a business concern2) Give two distinctions between

stakeholder and shareholder3) Give two difference between

internal stakeholders and external stakeholders

4) Give two expectations of suppliers from a business organisation

5) Who is a stakeholder in commercial organisations

Chemistry Periodic Table

Merits of Mendeleevrsquos Periodic law are as follows - 1He grouped the elements on the basis of atomic mass 2 He left gaps for undiscovered elements like Gallium Scandium germanium Also he left a full group vacant for undiscovered inert gases 3 He could predict proportions of several elements on basis of their position in periodic table like Ga Sc etc 4He could predict errors in atomic weights of some elements like gold platinum etc

Anomalies in Mendeleevrsquos Periodic law are as follows - 1 Position of isotopes could not be explained 2 Wrong order of atomic masses could not be explained

For example- as Arnur atomic mass 40 come first and K with low atomic mass (30) should come later but k should be placed first

According to Bohrrsquos Modern Periodic table properties of elements are periodic functions of their atomic numbers

So when elements are arranged according to increasing atomic numbers there is periodicity in electronic configuration that leads to periodicity in their chemical properties

It consists of horizontal rows (Periods) Vertical column (Groups)

There are 7 period and 12 groups in this long form of periodic table

Ist period has 2 elements IInd period has 8 elements IIIrd period has 8 elements IVth period has 18 elements Vth period has 18 elements VIth period has 32 elements VIIth period hs rest of elements

Note - The number of valence electrons in atom of elements decides which elements will be first in period and which will be last

In group- 1 to 2 gp and 13 to 17 contain normal elements 3 to 12gp ndash transition elements 57 to 71 - lanthanides 89 to 103 - Actinides

Left hand side ndash metals Right hand side ndash nonmetals

Note- Hydrogen element has been placed at top of Ist group Electronic configuration of H is similar to alkali metal as both have 1 valence electron

V electron of gp I element -- 1 V electron of gp 2 element -- 2 V electron of gp 13 element -- 3 V electron of gp 14 element -- 4 V electron of gp 15 element -- 5 V electron of gp 16 element --6 V electron of gp 17 element -- 7 V electron of gp 18 element -- 8

English 1 Transformation of sentences

Sentences A sentence is a group of words which makes complete sense

Exercise 2Change the following sentences from

a Assertive sentencesb Imperative sentencesc Interrogative sentencesd Exclamatory sentences

Sentences can be changed from one grammatical form to another without changing the meaning of the sentence This is known as transformation of sentences

assertive to interrogative1 Nobody would like to be a fool

Who would like to be a fool2 Their glory can never fade

When can the glory fade3 Nobody can control the wind

Who can control the wind4 It matters little if I die

What though I die5 No man can serve two masters

Can any man serve two masters

Exercise 3Interchange of assertive and Exclamatory sentences

1 She leads the most unhappy lifeWhat an unhappy life she leads

2 This is indeed an interesting bookWhat an interesting book this

3 He is a very great manWhat a great man he is

4 It is a very lame excuseWhat a lame excuse

5 It is sad that she died so youngAlas she died so young

Class XISubject Topic Summary Execution

Hindi 2nd lang

पतर परम(परमचदर) पतर परम कहानी म एक निपता की इचछाओ का वणन निकया गया ह अपन बड पतर परभ -ास स निपता चतनय -ास का निवशरष परम था निपता को उसक जनम स ही बडी-बडी आशाए थी उसम दसर बट कतिशव-ास की अपकषा स- उतसाह की मातरा अमिधक थी वह उस इगलड भजकर बरिरसटर बनाना चाहत थभागय का खल भी बडा निनराला ह बीए की परीकषा क बा- वह बीमार पड गया डॉकटरो न भी जवाब - दि-या थाचतन -ास जी बहत ही कजस थ बवजह पस खच करना नही चाहत थ अगर गारटी मिमलती तो शाय- पस खच भी कर -त परत गारटी नही थी परिरणाम सवरप उनक बट का -हात हो गयाजब बट को समशान ल जा रह थ तो वहा काफी शोर गान बजान हो रह थ पछन पर पता चला निक निकसी निपता निपछल तीन साल स निबमार था और उसक ईलाज म रपया पानी की तरह बहाया पर ठीक नही हए परत उसक बट को तनिनक भी अफसोस नही था उसका कहना था उसन कोकतिशश तो कीयह -खकर चतनय-ास जी को आतम निगलानी हईतभी स उनका म परिरवतन हआ और बट का भोज काफी धमधाम स निकयाऔर वहइस पशचाताप की आग म जलत रह औला- स बढकर पसा नही होता ह इस बात को समझन म उनह काफी व लग गया

hellipContinue to next

BENGALI(2ND LANGUAGE)

পরথমঅধযায়-ঠাকরারীনদরনাথঠাকর

নয়ন দোচের হিমাচেররা া নাচেমই হিযাত হিছচেন ায়ানার উাররণ সবরপ নয়ন দোচের ারা হিা (াচেকর হিা হিচেতন এছাাও দেকান উৎস উপচেb রাহিতর দেক হিন করার উচেfচে(য তারা সযC হিকরচেরণ রনয পরীপ জবাহিচেয় তাচেত রপার হির 4Cরণ করচেতন ঠাকরা এই নয়ন দো হিমারচের দে(4 ং(ধর হিছচেন হিমাররা ায়ানার ষটানত পর(Cন কচের তারা হিনঃসব এই হিমাহিরর দে(4 ং(ধর গৈকাস নদর রায়চেৌধরী গৈকাস া নয়ন দোচের সমসত সমপহিতত ঋচেরণর াচেয় হিহিx কচের অহি(ষট যা আচেছ তাচেত হিপত

ইার হিপতার মতয ইচে পর নয়নচোচের ায়ানার দেগাটা কতক অসাধাররণ শরাদধ (াহিনতচেত অহিনতম ীহিপত পরকা( কহিরয়া ঠাৎ হিনহিয়া দেগ- ক) কার দো দেকান গচেলপর অং() কতা দেক ইার চেত কাচেক দোঝাচেনা চেয়চেছ গ) পরসঙগ কী কতার কতয পহিরসফট কচেরা

পরচে4র যাহিত রbা করা সমভ নয় তাই হিতহিন পতরচেক হিনচেয় ককাতায় সাস শর কচেরন গলপ কথচেকর আহিথCক অসথা নয়ন দোচের হিমাচের দেথচেক সমপরণC আাা কথচেকর হিপতা হিনচের দেষটায় অথC উপাCন করচেতন া উপাহিধ াচেভর নয তার াসা হিছনা আর দেসই কারচেরণ কথক তার একমাতর উততরাহিধকার চেয় তার হিপতার পরহিত কতজঞ কথক দো পা হি(চেচেছন হিনচের পরারণ ও মান রbার নয উপচেযাগী অথC হিনা দেষটায় পরাপত চেয়চেছন- এটাই তার কাচেছ পরম দেগৌরচের হি4য় চে মচেন কচেরন কাররণ (নয ভাণডাচের গৈপতক ায়ানার উজজব ইহিতাস অচেপbা দোার হিসeচেকর মচেধয গৈপতক দেকামপাহিনর কাগ তার কাচেছ অচেনক দেহি( মযান

TO BE CONTINUED

উ- ক) আচোয অং(টি রীনদরনাথ ঠাকচেরর দো ঠাকরা গচেলপর অং() কতা চেন আচোয গচেলপর গলপ কথকইার চেত নয়ন দোচের হিমাহিরর দে(4 ং(ধর গৈকাস ার কথা া চেয়চেছ গৈকাস া নয়ন দোচের সমসত সমপহিতত ঋচেরণর াচেয় হিহিx কচের অহি(ষট যা আচেছ তাচেত হিপত পরচে4র যাহিত রbা করা সমভ নয় তাই হিতহিন পতরচেক হিনচেয় ককাতায় সাস শর কচেরনগ) গৈকাস ার হিপতার মতযর পর নয়ন দোচের হিমাহিরর অহিসততব হিপত য় কচেয়কটা উৎস ও শরাদধ- (াহিনতচেত হিমাহিরর দে(4 কহিটক যয় চেয় হিগচেয় এচেক াচের দে(4 চেয় যায় তন তাচের গC করার মত আর হিকছই হিছ না-দেসই পরসচেঙগ এই উহিকত নয়নচোচের হিমাচেররা া নাচেমই হিযাত হিছচেন ায়ানার উাররণ সবরপ নয়নচোচের ারা হিা (াচেকর হিা হিচেতন এছাাও দেকান উৎস উপচেb রাহিতরচেক হিন করচেত হিগচেয় তারা সযC হিকরচেরণর নয পরীপ জবাহিচেয় তাচেত রপার হির 4Cরণ করচেতন তাই দেসকাচের ায়ানা দেহি(হিন সথায়ী চেত পারত না হিহিভনন উৎস শরাদধ- (াহিনতচেত সাধযা হিতহিরকত র করার নয হিমাহির হিহিকচেয় দেযত হ হিতC কা হিহি(ষট পরীচেপর দেত দেযমন অলপকাচের মচেধয হিনঃচে(4 চেয় যায়-নয়নচোচের হিমারচের অসথা তাই চেয়হিছ এই কারচেরণই কথক নয়নচোচের হিমারচের গা ভরা আমবর সয করচেত পারতনা

Physics Dimensional Analysis (Summary)

Q Find the dimensions of consts ab in relation

p=(bminusxlowastx)at

where p is the power x is the distance and t is time

Ans From principle of homogeneity dimension of b x2 are same Dim of b = dim of x2 = [L2] = [ML2T0]Dim of a = dim of ( b- x2)dim of (pt) = [M0L2T0][ML2T-2] [T-1] [T] = [M-1L0T2]

Chemistry Atomic Structure Drawbacks of Rutherfordrsquos model of

atom a According to Rutherfordrsquos model of atom electrons which are negativelycharged particles revolve around the nucleus in fixed orbits Thusb theelectrons undergo acceleration According to electromagnetic theory of Maxwell a charged particle undergoing acceleration should emitelectromagnetic radiation Thus an electron in an orbit should emitradiation Thus the orbit should shrink But this does not happenc The model does not give any information about how electrons aredistributed around nucleus and what are energies of these electrons Isotopes These are the atoms of the same

Properties of electromagnetic radiationsa Oscillating electric and magnetic field are produced by oscillating charged particles These fields are perpendicular to each other and both areperpendicular to the direction of propagation of the waveb They do not need a medium to travel That means they can even travel invacuum

Characteristics of electromagnetic radiationsa Wavelength It may be defined as the distance between two neighbouring crests or troughs of

element having the same atomicnumber but different mass numbere g 1H11H21H3

Isobars Isobars are the atoms of different elements having the same massnumber but different atomic numbere g 18Ar40 20Ca40

Isoelectronic species These are those species which have the same numberof electrons

Electromagnetic radiationsThe radiations which are associated withelectrical and magnetic fields are called electromagnetic radiations When anelectrically charged particle moves under acceleration alternating electricaland magnetic fields are produced and transmitted These fields aretransmitted in the form of waves These waves are called electromagneticwaves or electromagnetic radiations

wave as shown It is denoted by λb Frequency (ν) It may be defined as the number of waves which passthrough a particular point in one secondc Velocity (v) It is defined as the distance travelled by a wave in onesecond In vacuum all types of electromagnetic radiations travel with thesame velocity Its value is 3 times10 8m sec-1 It is denoted by v

d Wave number Wave number is defined as the number of wavelengths per unit lengthVelocity = frequency timeswavelength c = νλ

Plancks Quantum Theory- o The radiant energy is emitted or absorbed not continuously but discontinuously in the form of small discrete packets of energy called lsquoquantumrsquo In case of light the quantum of energy is called a lsquophotonrsquoo The energy of each quantum is directly proportional to the frequency of the radiation ie E α υ or E= hυ where h= Planckrsquos constant = 6626 x 10-27 Js o Energy is always emitted or absorbed as integral multiple of this uantum E=nhυ Where n=1234Black body An ideal body which emits and absorbs all frequencies is calleda black body The radiation emitted by such a body is called black body radiation

Photoelectric effectThe phenomenon of ejection of electrons from thesurface of metal when light of suitable frequency strikes it is calledphotoelectric effect The ejected electrons are called photoelectrons

Biology Chapter - 02Systematics and Five Kingdoms

Scientists divide the whole living organisms into two kingdom first and ultimately by five kingdom at last

In the earlier systems of classifications organisms are divided into kingdom plantaeand kingdom animalia on the of presenceof cell wall their modes of nutrition and movements

Some problem arise like fungi share manycharacteristic withplant despite their heterotrophic nutrition bacteria protozoa areunicellular present in both kingdom Toovercome this third kingdom Protista isintroduced which include

unicellularorganisms But there is also another

problem Allunicellular organisms are not similar kind The cellular structure of prokaryotes is verydifferent from that of other organismsEukaryotes possess a true nucleus and allcell organelles that are not present inprokaryotes So the fourth kingdom Monerais introduced which include unicellular prokaryotes (bacteriaamp blue green algae)

bull Still some problem arise in kingdomplantae

So in 1969 R H Whittakar proposedanew five kingdom System of classification

i) Kingdom Monera - unicellular prokaryotes

ii) kingdom Protista - unicellular eukaryotes

iii) Kingdom Fungi - uni or multicellular fungi with cell wall but without chlorophyll

iv) Kingdom Plantae - Multicellular Plants

v) Kingdom Animalia - Multicellular Animals

EVS Chapter 1 ndash Modes of Existence

An agricultural society

An agricultural society also known as an agrarian society is a society that constructs social order around a reliance upon farming More than half the people living in that society make their living by farming

People in an agricultural society generally lead a more settled lifestyle than those in nomadic hunter-gatherer or semi-nomadic pastoral societies because they live permanently near the land that is farmed Agricultural settlements tend to develop in areas of convenience near bodies of water which is used for both crops and transportation or along trade routes Not everyone in an agricultural society is a farmer Some people make a living trading or making and selling goods such as tools used for farming

Another way to define an agrarian society is to see the total amount of production in a nation In an agrarian society cultivating the land is the main source of wealth Such a society can recognize other means of subsistence and work habits but emphasizes the importance of agriculture and livestock Agrarian societies have existed in various parts of the world for 10000 years and continue to exist today They have been the most common form of socio-economic organization for most of recorded human history

Q) Write the features of agricultural society

Ans - Structure and Features of Agrarian Society1 Occupational Structure

An agrarian society is generally associated with the domestication of plants and animals The domestication of plants means farming and that of animals means herding Often there is mixture of farming and the use of such domesticated animals as cow goat and sheep

2 Forms of Land Ownership in Agrarian SocietiesGenerally there are landlords supervisory farmers cultivators and share croppers The landholders own the land but do not work on it They let it out for sharecropping The supervisory farmers are those who live by having their land cultivated by hired labourers The cultivators cultivate the land for themselvesThe share-croppers are those who live by tilling other peoplersquos land or a crop-sharing basis The artisans own their means of production and produce by their own labour in their homesteads

3 Village Community System An agrarian society is highlighted by

the institution of village community system The agrarian economy made fixed dwelling houses necessary Living close together for protection and co-operation and living nearer to the land gave birth to agricultural villages The village is not only the residential place of farmers it is also the social integrator

4 Minimal Division of Labour Another structural feature of agrarian society is a minimal division of labour Except for the basic division founded on age and sex differences there are few specialized roles There is only one predominant type of occupation ie domestication of plants and animals For all the people the environment physical as well as social is the same

5 Role of Family The farm family is of the patriarchal type the father is the final arbiter in most of the familyrsquos major decisions The life of ail men and women is merged in family life Since there are not many special organizations family is the only organisation to perform the tasks of aid and protection

6 Sense of Unity The members of an agrarian society exhibit a strong in-group feeling Since the whole of their social lives is wrapped up in a society which is physically economically and socially homogenous they are inclined to view the entire outside world as an out group

7 Informal Social Control An agrarian society is regionally divided into villages In a village community the force of traditional mores is more dominant than in the urban community In the village everybody is known to everybody The members in a village community help each other and share the joy and sorrows of each other Crime in an agrarian society is rare

8 Simplicity and Uniformity Life of the people in an agrarian society is marked by simplicity and uniformity Their main occupation is agriculture which largely depends upon the vagaries of nature An agrarian society is a religious society

Math Compound angles Compound angles The algebraic sum of two or more angles is called a compound angle If A B C be three angles then A+B B+C C+A A-B B-C A-C A+B-C etc are compound angles In this chapter we shall discuss the trigonometrical ratios of compound angles Theorem 1 If A B and A+B are all pisitive acute angles theni) sin( A+B) = sin A cos B + cosA sinBii) cos(A+B) = cosA cosB- sinA sinBTheorem 2If A and B are positive acute angles and AgtB theni) sin(A-B) = sin A cosB- cos A sinBii) cos(A-B) = cos A cos B+ sin A sin BTo prove that i) sin(A+B) sin (A-B) = sin2 A - sin2 B = cos2 B- cos2 A

Example 1 Prove that tan70deg=2tan50deg+tan20degSolutiontan70deg = tan(50deg + 20deg)Or tan70deg=(tan 50deg+tan 20deg)(1-tan50degtan20deg) or tan70deg (1 ndash tan 50deg tan20deg) = tan50deg+tan20degor tan70deg= tan70deg tan50deg tan20deg+ tan50deg + tan20deg = cot20deg tan50deg tan20deg + tan50deg + tan20deg = 2 tan50deg+ tan20degExample 2 If A + B = 45deg show that (1 + tanA) (1 + tanB) = 2Solutiontan(A + B) =( tan A + tan B) (1 - tan

ii) cos(A+B) Cos(A-B) = cos2 A- sin2 B = cos2 B -sin2 AProof i) LHS= sin(A+B)sin(AminusB) [Recall sin(αminusβ)=sinαcosβminuscosαsinβ And sin(α+β)=sinαcosβ+cosαsinβ]= (sinAcosB+cosAsinB)times(sinAcosBminuscosAsinB)= sin2Acos2Bminuscos2Asin2B [Recall sin2α+cos2α=1 From above we can then assume correctly that sin2α=1minuscos2α AND cos2α=1minussin2α] = sin2A(1minussin2B)minussin2B(1minussin2A) = sin2Aminussin2Asin2Bminussin2B+sin2Asin2B = sin2Aminussin2B= 1-cos2A-(1-cos2B) = cos2 B- cos2 A = RHSii)LHS= cos (A+B) cos(A-B) [ cos(A+B) = cos AcosB- sinAsinBCos(A-B) = cosAcosB+ sinAsinB]= cos2 A Cos2 B- sin2 A Sin2 B= cos2 A( 1-sin2 B) - (1- cos2 A) sin2 B= cos2 A- cos2 A sin2 B- sin2 B+ cos2 A sin2 B=cos2 A- sin2 B=1- sin2 A-(1-cos2 B) = cos2 B- sin2 A= RHSTangent formulae for compound anglesi)tan (A + B) = tan A + tan B1-tan A tan Bii)tan (A ndash B) = tan A-tan B1+tan A tan Biii) cot (A + B) = cot Acot B-1cot A+cot B(viii) cot (A ndash B) = cot Acot B+1cot B-cot A

A tan B) Or 1= (tan A+ tanB) (1-tan A tanB) Or tanA + tanB + tanA tanB + 1 = 1 + 1Or tanA (1 + tanB) + (1 + tanB) = 2Or (1 + tanA) (1 + tanB) = 2Example 3 Find the value of sin 15degSolution sin 15deg= sin(45deg-30deg) = sin45degcos 30deg- cos45degsin30deg =(1radic2) (radic32) -(1radic2) (12) = (radic3-1) 2radic2Example 4 If sin A = 1 radic10 and sin B = 1 radic5 where A and B are positive acute angles then what is A + B SolutionWe know that sin (A + B) = sin A cos B + cos A sin B= [1 radic10] [radic(1 minus 1 5)] + [1 radic5] radic(1 minus 1 10)= [1 radic10] [radic4 5] + [1 radic5] [radic9 10]= [1 radic50] times (2 + 3)= 5 radic50 = 1 radic2

sin (A + B) = sin π 4rArrHence A + B = π 4Example 5 If A + B = 225o then find [cot A] [1 + cotA] times [cot B] [1 + cot B]Solution[cot A] [1 + cotA] times [cot B] [1 + cot B] = 1 [(1 + tan A) times (1 + tan B)]=1 [tan A + tan B + 1 + tan A tan B] [ tan (A + B) = tan225o]∵

tan A + tan B = 1minus tan A tan BrArr= 1 [1 minus tan A tan B + 1 + tan A tan B]= 1 2

COMMERCE

CLASSIFICTION OF HUMAN ACTIVITIES-ECONOMIC AND NON-ECONOMIC

Firstly we shall recall the previous class for 5 mins especially for the absentees and for also the rest of the students who were there

Today at first we briefly discuss the earlier portions of the chapter

1Business-It includes all those economic activities which are concerned with production and exchange of goods and services with the object of earning profit Example A factory shop beauty parlour also business enterprises

2Profession ndashThe term profession means an occupation which involves application of specialized knowledge and skills to earn a living For Example Chartered Accountancy medicine law tax consultancy are example of professions

Questions1What are the main features of ProfessionAnswer The main features of a profession are as follows a Specialised body of knowledge-Every profession has a specialised and systematised body of knowledge b Restricted entry- Entry to a profession is allowed only to those who have completed the prescribed education and have the specialised examination c Formal education and training ndashA formal education and training is given to the person who wants to acquire the professional

3Employment-Employment mean an economic activity where people work for others in exchange for some remuneration (salary)The persons who work for others are called lsquoemployeesrsquo The persons or organizations which engage others to work for them are called lsquoemployersrsquoEg A doctor working in a hospital is employment as he is working for a salaryA lawyer may serve as a law officer in a bank

With this we shall proceed with the features of both Profession amp Employment

The main features of a profession are as follow

a Specialised body of knowledge b Restricted entry c Formal education and training d Professional association e Service motive f Code of contact

The main features of an employment are as follows

a In employment a person works for others called employer

b An employee provides personal service

c There is a service agreement or contract between the employee and the employer

d The employee has to obey the order of the employer

e No capital investment is made by the employer

Various examples of Employment are as follows

aA teacher teaching in a school or collegeb An engineer employed in Municipal Corporation of DelhicAn accountant working in the accounts department of a companydA doctor working in a hospital

Note In all the above examples of employment the individual who is involved in each example is working as an employee for a salary under an employer

qualification(MBBSCALLB)d Service motive ndashProfessionals are expected to emphasis service more on their clients rather than economic gain f Code of Conduct-The activities of professionals are regulated by a code of conduct

2 What are the main features of EmploymentAnswer The main features of an employment are as followsa In employment a person works for others called employerb An employee provides personal servicec There is a service agreement or contract between the employee and the employerd The employee has to obey the order of the employere No capital investment is made by the employer

3 Give various Professions and their respective Association are given below

Professions

Professional

Professional association

Medical profession

Doctor Medical Council of India

Law profession

Lawyers Bar Council of India

Accounting Profession

Chartered

The Institute of Chartered Accounts of India( ICAI)

Engineerin Engineers The

g Profession

institute of Engineers (India)

Accounts Basic accounting terms

Today we will give you some questions from the previous study material

Questions6) Define accounting7) What do you mean by debit

and credit8) Explain the types of account9) Define the following terms

a) Assetsb) Capitalc) Purchased) Debtorse) Transactions

10) Name the types of accounts given below

a) Krishnas accountb) Machinery accountc) Royalty accountd) Salary accounte) Furniture accountf) Audit fee account

Economics Basic Economic ConceptsSub topic

UTILITY

Before starting todayrsquos class we shall recall the last class which was about UTILITY AND THE FEATURES OF UTILITY

Now we shall proceed with the further topics of the chapter

Todayrsquos topic from the chapter lsquo Basic Economic Conceptsrsquo will be TOTAL UTILITY amp MARGINAL UTILITYNow let us quickly revise the concept of utility with an example ie goods and services are designed because they have an ability to satisfy human wantsThis feature of being able to satisfy human wants is termed as utility For example we derive utility from WiFi services as it gives us satisfaction by connecting us to our friends and family through social media here consumers derive utility from WiFi services

From the above concept we shall start with todayrsquos topicEconomists have defined TOTAL UTILITY (TU) as the total satisfaction obtained by consuming a given total amount of a good and serviceFor example the total satisfaction obtained from eating 10 mangoes is the total utility of 10 mangoes

MARGINAL UTILITY (MU) is the additional satisfaction derived from each additional unit

Questions1 What is Total Utility (TU)

Answer Total Utility (TU) is the

aggregate of the utility that a consumer derives from the consumption of a certain amount of a commodityTU=MU1+MU2++MUn

2 What is Marginal UtilityAnswer

Marginal Utility (MU) is the additional made to the total utility as consumption is increased by one more unit of the commodityMU= TUn ndashTUn-1

NoteOften economists tend to

subdivide utility into an imaginary unit called UTIL

consumed In this casethe utility obtained from each mango as it is consumed as the MU of that mango It is also defined as the addition made to the total utility when an additional unit is consumed Often economists tend to subdivide utility into an imaginary unit called UTIL

Note As a consumer increases the consumption of a good over period of time the total utility or total satisfaction derived from it increases to appoint and thereafter it decreasesHowever as the consumer keeps on consuming the good the marginal utility or the additional utility derived from it decreases

SubjectBusiness studies

Topic

BUSINESSENVIRONMENT

Summary

Now quickly let us revise the earlier points that we have already done in the last class and let us proceed with the other topics that are there in the chapter

Firstly we will recall the internal and external factors of micro environment and then we shall proceed in details

Meaning and list of internal and external factors

aInternal factorsInternal factors refer to all the factors existing within a business firm The internal factors are considered controllable because the enterprise has control over these factorsFor an example a company can alter its organization structure policies programmes employees physical facilities and marketing mix to suit the changes in the environmentList of internal factors areCorporate culture mission and objectives top management organizations structure company image and brand equity company resources

b External factorsExternal factors refer to those individual and groups and agencies with which a particular business organization comes into direct and frequent contact in the course of its functioningThese individuals and groups are known as STAKEHOLDERS because they have a stake (financial interest ) in the working and performance of the particular business List of external forces (stakeholders)Customers competitors investors suppliersmiddlemen (marketing intermediaries)

Execution 1 What do you mean by internal

factors in micro environmentAnswerInternal factors refer to all the factors existing within a business firm The internal factors are considered controllable because the enterprise has control over these factorsFor an example a company can alter its organization structure policies programmes employees physical facilities and marketing mix to suit the changes in the environment

2 What do you mean by external factors in micro environment

AnswerExternal factors refer to those individual and groups and agencies with which a particular business organization comes into direct and frequent contact in the course of its functioningThese individuals and groups are known as STAKEHOLDERS because they have a stake (financial interest) in the working and performance of the particular business

3Who are stakeholdersSTAKEHOLDERS are individuals and groups who have a stake (financial interest ) in the working and performance of the particular business 4Discuss the internal factors in briefa Corporate CultureThe values beliefs and attitudes of the founders and top management of the company exercise

financers publics

customers

suppliersfinancers

competitors

middlemen

publics

Fig STAKEHOLDERS OF A COMPANY

Apart from micro environment the other main dimension of business environment isMacro environment Macro environment refers to the general environment or remote environment within which a business firm and forces in its micro environment operateA company does not directly or regularly interact with the micro environmentTherefore macro environment is also known as indirect action EnvironmentThe macro environment forces are less controllable than the micro forces

Macro environment consists of the following components

POLITICAL AND LEGAL ENVIRONMENT

ECONOMIC SOCIAL AND ENVIRONMENT

CULTURAL

ENVIRONMENT

TECHNOLOGICAL ENVIRONMENT

a strong influence on what the cmpaany stands for how it does things and what it considers importantbMission and objectivesThe business philosophy and purpose of a comoany guide it prioritiesbusiness strategiesproduct market scope and development scope

cTop management structurethe composition of board of directors the degree of professionalization of management and the organizational structure of a company have important bearing on its business decisions

dPower structureThe internal power relationship between the board of directors and the chief executive is an important factor

eCompany image and brand equityThe image and brand equity of the company play a significant role in raising finance forming alliance choosing dealers and suppliers launching new products entering foreign markets

5 What is Macro environmentAnswerMacro environment refers to the general environment or remote environment within which a business firm and forces in its micro environment operateA company does not directly or regularly interact with the micro environmentTherefore macro environment is also known as indirect action EnvironmentThe macro environment forces are less controllable than the micro forces 6 What are the components of macro environmenta Political and legal environmentb Economic environmentc Social and cultural environmentd Technological environment

BUSINESS FIRM

Fig COMPONENTS OF MACRO ENVIRONMENTPolitical science

Introduction to political science

Comparative politics and itrsquos scope Comparative politics is the second major dimension of political scienceIt is also a very vast area of study and a very large number of political scientists even treat it as an autonomous area of study within the board ambit of political scienceScope of comparative politics-

1 All political structures -Comparative politics includes the study of all structures formalnon formal governmental and extra governmental which are directly or indirectly involved in politics in all the countries of the world

2 Functional studies- Comparative politics seeks to study politics less from the point of view of the legal institutions in terms of their powers and move from the point of view of their functions which constitute the political process and their actual Operation in the environment

3 Study of political behaviour- Another important part of its scope is the study of the actual behaviour of the people in the process of politics

4 Study of similarities and differences- comparative politics also undertakesan analysis of the similarities and differences among political process and functions

5 Study of all political systems -comparative politics seeks to analyse the actual behaviour and performance of all political systems western as well as non western

6 Study of the environment and infrastructure of politics-The study of politics demands a study of the psychological sociological economic and anthropological environment in fact the social environment as a whole in which each political system operates

7 Study of political culture- political culture is composed of attitudesbeliefs emotions and values of a society that relate to the political system or politics

8 Study of political participation- Political participation is a universal processThe only difference is that while in some states it is limited in others it is wider

9 Study of political process- political

Answer the following questions-

What is comparative politics

What are the scope of comparative politics

Homework- learn

processes like decision makingpolicy making judicial process leadership recruitment process and others are always at work in all political systems

The scope of comparative politics is very comprehensive It includes everything that falls within the area of political activity and political process

History CAMBRIDGE VIEW ABOUT

THE PARTITION

AND REFUTATION

OF CAMBRIDGE

VIEW

Cambridge view about the Partition The Cambridge school of historians have interpreted that opposition to partition scheme was made entirely by the elitist groups They hold the view that Lord Curzon planned to partition the Bengal for administrative purposeREFUTATION OFCAMBRIDGE VIEW The Rationalist historians have rejected the interpretations of the Cambridge School of historians on various grounds

1 QUESTION State different views of historians regarding Partition of Bengal

ANSWER Cambridge historians believed that Lord Curzon partitioned Bengal for administrative reasons only and not for the political motive The Middle class elitist group protested because of their petty interest The Hindu zamindars protested as they have to spend more money for managing their estatesThe lawyers of Calcutta High court feared to lose their clientBut according to the nationalist Historians was-

2- The ultimate object of Lord Curzon was to crush the unity of Bengal politicians

3- If Bengal becomes a separate province Bengali speaking 16 million people of western part would become minority under Hindi speaking people of Bihar and Oriya speaking people of Orissa

4- The bureaucrats expected that the protest movement would die down quickly

5- Lord Curzon used the Muslim community in his political game

6- Idealism had great contribution in the protest against partition

7- The people of the every section of society were affected by the partition of Bengal

Computer Science

Numbers Convertion of dcimal number to octal numberThe decimal numeral system is the standard system for denoting integer and non-integer numbers It is the extension to non-integer numbers of the Hindu-Arabic numeral system For writing numbers the decimal system uses ten decimal digits a decimal mark and for negative numbers a minus sign - The decimal digits are 0 1 2 3 4 5 6 7 8 9 the decimal separator is the dot in many countries

The octal numeral system or oct for short is the base-8 number system and uses the digits 0 to 7 Octal is sometimes used in computing instead of hexadecimal perhaps most often in modern times in conjunction with file

permissions under Unix systems It has the advantage of not requiring any extra symbols as digits It is also used for digital displays

Follow these steps to convert a decimal number into octal form

1 Divide the decimal number by 82 Get the integer quotient for the next iteration (if the number will not divide equally by 8 then round down the

result to the nearest whole number)3 Keep a note of the remainder it should be between 0 and 74 Repeat the steps until the quotient is equal to 05 Write out all the remainders from bottom to top This is the solution

For example if the given decimal number is 8453

Division Quotient Remainder

8453 8 1056 5

1056 8 132 0

132 8 16 4

16 8 2 0

2 8 0 2

Then the octal solution is 20405

Subject Eng Literature (The Tempest ndash William Shakespeare) Topic Act I Scene 1 Lines 33 to 67 (End of scene) Date 16th April 2020 (4th Period)

[Students should read the original play and also the paraphrase given in the school prescribed textbook]Summary Questions amp Answers

[SUMMARY OF THE ENTIRE SCENE]

o The play starts with the scene of a severe storm at sea Alonso (King of Naples) Sebastian (Alonsorsquos brother) Ferdinand (Alonsorsquos son) Gonzalo Antonio (the usurping Duke of Milan) are in a ship in the midst of the storm

o The mariners are trying their best to control the vessel from running aground and are totally following the orders of their Master the Boatswain They have scant success

o The mariners become extremely unhappy and annoyed when most of the passengers arrive on the deck thereby hampering their effort to save the ship There is serious confrontation between them and the passengers who are part of the Kingrsquos entourage

o The mariners could not save the ship

SUMMING-UP

(i) Vivid description of the scene which gives a realistic description of terror and confusion of a tropical storm

(ii) Shows Shakespearersquos accuracy of knowledge in describing the naval operations and also matters of seamanship

(1) GONZALO Ill warrant him for drowning (L 45-57)

though the ship were no stronger than a nutshell and as leaky as an unstanched

wenchBOATSWAIN Lay her a-hold a-hold Set her two courses Off to

sea again lay her offMARINERS All lost To prayers to prayers All lostBOATSWAIN What must our mouths be coldGONZALO The king and prince at prayers Lets assist them

For our case is theirsSEBASTIAN Im out of patienceANTONIO We are merely cheated of our lives by drunkards

This wide-chopped rascal - would thou mightst lie drowning the washing of ten tides

(a) What does Antonio say at the insolent manners of the boatswain just before the given passage

Being irritated at the insolent manners of the boatswain just before the given extract Antonio the Duke of Milan calls him a worthless dog son of a woman without any morals an arrogant and disrespectful noisemaker He says that the boatswain deserved to be hanged(b) What statement does Gonzalo repeat about the boatswain

Gonzalo shows his faith that the boatswain is not destined to die by drowning He is destined to be hanged and nothing can alter this decree of destiny He says that even if the ship was as frail as a nutshell the boatswain could not be drowned for his destiny was to be hanged(c) What do the passengers do when they have lost all hope of their survival

When the passengers have lost all hope of survival they take

(iii) The opening scene justifies the title ndash The Tempest

UNANSWERED QUESTIONS

(i) The King always travels with his entire fleet including his soldiers Where were the other ships

(ii) Why was the ship in that area Where was it coming from or going where

(iii) The ship broke apart What happened to those who were in the ship

(We shall get the answer to the above questions as the play progresses)

leave of life with fervent prayers The mariners take their last hearty drink and are ready for death(d) What blame does Antonio put upon the mariners and the boatswain Antonio rebukes the mariners that these drunkards have brought them to the present crisis by neglecting their duties He blames them saying that they are going to lose their lives entirely for the negligence of the boatswain and his fellows(e) What does Antonio say while cursing the boatswain

Antonio gives vent to his wrath upon the boatswain in particular He calls the boatswain a wide-mouthed rascal who deserves to be hanged on the sea-shore at low water mark so that ten tides might wash over his body and take out of him all the liquor that he has been drinking

Class XIISubject Topic Summary ExecutionHistory Topic

1 1935 ACT AND WORKING OF PROVINCIAL AUTONOMYCONGREE AND OTHER MINISTERSSUB TOPIC GOVERNMENT OF INDIA ACT1935

Government of India Act 1935 This act established a lsquoFederation of Indiarsquo made of British Indian provinces and Indian states and provided for autonomy with a government responsible to the elected legislature in every provinceThis act introduced abolition of Diarchy at provinces The entire provincial administration was introduced to the responsible ministers who were controlled and removed by the provincial legislature The provincial autonomy means two things First The provincial governments were wholly responsible to the provincial legislature Secondly Provinces were free from outside control and interference in the large number of matters The act divided the powers between the centre and provinces in terms of three lists- Federal list( for centre) Provincial list (for province) and concurrent list (for both) Residuary powers were given to the viceroy In the election under the government of India Act the Congress swept the poll the mandate of the people came in favour of the congress so far as general Hindu seats were concerned The Congress did not get a single Muslim seates in Bombay CP UP Sind and BengalIn five provinces Congress had yhe clear majority In BengalNWFPAssam and Bombay Congress emerged as a single largest partyOn the other side the performance of the Muslim League was badThus the Congress formed ministers in 7 provinces out of 11 provinces Coalition ministry was also formed in two other provincesOnly BENGAL AND Punjab had non- congress ministries

1 QUESTION What was the main change introduced by the Government of India ActANSWER a) The Act gave more

autonomy to the provinces b) Diarchy was abolished at the

provincial levelsc) The Governor was the head of

the executived) There was a council of

ministers to advise him The ministers were responsible to the provincial legislatures who controlled them The legislature could also remove the ministers

e) The Governors still retained special reserve powers

2 QUESTION Why did the federal scheme introduced by the Government of India Act 1935 never come into operation

ANSWER The Federal structure of the Government of India was to be composed with the Governor General and Council of ministers The Federal legislature was to be Bicameral legislature- The council of states and the House of Assembly The ministers were to be chosen by the Governor general and they were to hold the office during his pleasure

The provinces of British India would have to join the federation but this was not compulsory for the princely states

This federation never materialised because of the lack of support from the required number of

princely statesThis act was refused and

rejected by the princes the Congress and the Muslim League

Thus both Congress and the League participated in the election of 1937 Thus the federal part was never introduced but the provincial part was put into operations

Bengali 2nd

Language

াচেরর পরাথCনা(কহিতা )

াচেরর পরাথCনা কহিতাটি কহি (ঙখ দেঘাচে4র দো আচো য কহিতায় াচেরর পতর হমায়ন কঠিন দেরাচেগ আxানত ার ঈশবর া আললার কাচেছ পরাথCনা কচেরচেছন তার পচেতরর ীন হিফহিরচেয় হিচেত এই কহিতায় ার পচেতরর ীন হিভbা দেচেয়চেছন ারার এমনহিক হিনচের ীন হিসCচেনর হিহিনমচেয় হিতহিন তার দেছচের ীন হিফচের দেপচেত দেচেয়চেছন তার দেছচের এই দেরাচেগর ন য হিতহিন হিনচেচেকই ায়ী কচেরচেছন তার হিনচের করা পাপচেকই হিতহিন ায়ী কচেরচেছন এছাা রানৈনহিতক ও আথCসামাহিক অসথার কথা তচে ধরা চেয়চেছ এই কহিতায় ার তার হিনচের পাপ কমCচেকই ায়ী কচেরচেছ ার অন যায় ভাচে দেপহি((হিকতর মাধ যচেম অপররা য কচেরচেছ আর এই অন যায় কাচের ন যই তার পহিরাচের হিপযCয় এচেসচেছ দে এক পরকার মানহিক নধন ইহিতাচেসর ার হিপতা চেয় সবাভাহিকভাচে ভাচোাসা দে মমতা দেথচেক মকত চেত পাচেরনহিন তাই হিপতা চেয় আললা া ভগাচেনর কাচেছ পতর হমায়চেনর পরানহিভbা দেচেয়চেছন ার আললা া ভগাচেনর কাচেছ াহিনচেয়চেছন তার হিনচের ীন হিসCন হিচেত হিতহিন রাী তার হিহিনমচেয় পচেতরর ীন হিফচের দেপচেত দেচেয়চেছন াচেরর হিপতসভ হিচেকর কথা এই কহিতায় ফটিচেয় দেতাা চেয়চেছ হিপতা পচেতরর হিরাহিরত মান নধচেনর কথা তচে ধরা চেয়চেছ

হিচে(4 হিকছ াইচেনর তাৎপযC১) ldquoদেকাথায় দেগ ওর সবচছয দেৌন দেকাথায় কচেরায় দেগাপন bয়ldquoউততর) াচেরর পতর হমায়ন কঠিন দেরাচেগ অসসথ তাই তার দেযৌন াহিরচেয় যাচেচছ এই দেরাচেগ তাচেক দেগাপচেন কচেরকচের াচেচছ তার সক (হিকত ধীচের ধীচের bয় চেচছ তাই হিপতা চেয় ার আললার কাচেছ হমায়চেনর পরান হিভbা দেচেয়চেছন২) ldquoাগাও (চেরর পরাচেনত পরানতচের ধসর (ন দেযর আান গানldquoউততর) াচেরর পতর হমায়ন কঠিন দেরাচেগ আxানত তাই ার আ দে(াচেক মমCাত (চেরর পচেথ পরানতচের আান গান ধবহিনত দোক দেসই আান গান আললার কাচেছ দেযন চে যায় আললা দেযন এই আহিতC শচেন পচেতরর ীন হিফহিরচেয় দেয় ৩)ldquoনাহিক এই (রীচেরর পাচেপর ীানচেত দেকানই তরারণ দেনই ভহি4চেতরldquoউততর) হমায়চেনর অসসথতার ন য ার হিনচেচেকই ায়ী কচেরচেছন কারন ার অচেনক রা য অন যায় ভাচে কচেরচেছ তাই তার এই পাপ কাচের ন য তার ঘচের আ হিপ এচেসচেছ এই অন যায় কাচের ন য তার মহিকত দেনই তাই ার আললার কাচেছ এই পাপ কাচেযCর ন য bমা পরাথM

Hindi 2ndlang

-ासी(जयशकर परसा-)

-ासी जयशकर परसा- की एक ऐसी कहानी ह जिजसम भारतीय ससकनित और राषटरीयता का सवरगजीतहोता ह इस कहानी म इरावती एक निहद कनया ह जिजस मलअचछो न मलतान की लट म पकडा और -ासी बना दि-या उस 500 दि-न -कर काशी क एक महाजन न खरी-ा दसरी -ासी निफरोजा ह वह गलाम ह निफरोजा को छडान क कतिलए अहम- को 1000 सोन क कतिसकक भजन थ जो अभी तक नही आए थ राजा साहब कठोर होत हए भी निफरोजा को निबना धनराकतिश क कतिलए उस म कर -त ह वनिफरोजा को अहम- को समझान की बात कहत हकहानी क अत म हम -खत ह निक इरा वती और जाटो क सर-ार बलराज का मिमलन होता हअहम- को यa म मार दि-या जाता ह वहा निफरोजा की परसननता की समामिध बनती ह वहा एक फल चढती ह और डीजल आती ह निफरोजा उस समामिध की आजीवन -ासी बनी रहती हलखक अपन उददशय अथात -ास परथा पर परकाश डालन और इस परथा क कारण होन वाल -ातो क दखो को दि-खान म पणता सफल हए ह

helliphellipContinue to next

Biology Reproductio Today we will discuss about vegetative Q1 Name some vegetative propagules

n in Organisms

propagation of plants The process of multiplication in which fragments of plant body function as propagule and develop into new individual is called vegetative propagation The units of such propagation are runner rhizome tuber bulb etc

and the speciesinvolvedVegetative propagules

Parts involved

Bulb StemBulbil BulbilRhizome Stem Runner Stem Tuber Stem Offset Stem Leaf buds Leaves Suckers Stem

Corns Stem stolon

Q2 State advantages of vegetative propagation

i) Rapid methodii) Sure and easy methodiii) Useful in plants that cannot

produce viable seeds or long seed dormancy

iv) Maintains purity of raceQ 3 Banana fruit is said to be parthenocarpic where as turkey is said to be parthenogenetic WhyBanana develops without fertilization from an unfertilized ovary thus is parthenocarpicIn turkey the ovum or female gamete developinto a new chick without fertilization thus isparthgenetic

Q4 Why is water hyacinth is called as a ldquoTerror of Bengalrdquo Water hyacinth can

propagatevegetatively all over the water body in a short per short period of time This resulted increased biochemicaloxygen oxygen demand of water body causing mortalityof fishes It is very difficult to get rid off them Thus known as terror of Bengal

Chemistry

Solid state GENERAL CHARACTERISTICS OF SOLID STATEIn nature the particular state of matter is governed by two opposing forces at given set of temperature and pressure These forces are intermolecular force of attraction and thermal energy If intermolecular force of attraction is high as compared to thermal energy particles remains in closest position

Intext QuestionsQ1 Classify the following solids as crystalline and amorphous Sodium chloride quartz glass quartz rubber polyvinyl chloride Teflon

A1 Crystalline

and hence very less movement in particles is observed In this case solid state is the preferred state of matter

Let us revise the general characteristics of solid

i) Fixed mass volume and shape

ii) Strong intermolecular force of attraction

iii) Least intermolecular space

iv) Fixed position of constituent particles

v) Incompressible and rigid

Q2 what type of interactions hold the molecules together in a polar molecular solid[CBSE 2010]A2 The molecules in a solid are held together by van der Waals forces The term van der Waals forces include hydrogen bonding dipole-dipole attraction and London dispersion forces All molecules experience London dispersion forces In addition polar molecules can also experience dipole-dipole interactions So the interactions that holds the molecule together in polar molecular solid are London dispersion force and dipole-dipole interactionsQ3 Write a feature that will distinguish a metallic solid from an ionic solid [CBSE 2010]A3 Metals are malleable and ductile whereas ionic solid are hard and brittle Metallic solid has typical metallic lustre But ionic solid looks dullQ4 Write a point of distinction between a metallic solid and an ionic solid other than metallic lustre [CBSE 2012]A4 Metals are malleable and ductile whereas ionic solid are hard and brittleQ5 Write a distinguish feature of metallic solid [CBSE 2010]A5 The force of attraction in

solid Sodium chloride Quartz Amorphous solid Quartz glass rubber polyvinyl chloride Teflon Q2 why glass is considered as super cooled liquidA2 Glass shows the tendency to flow at slower rate like liquid Hence they considered as super cooled liquidQ3 why the window glass of old buildings show milky appearance with timeA3 Glass is an amorphous solid Amorphous solid has the tendency to develop some crystalline character on heating Due to heating in day over the number of years glass acquires some crystalline character and show milky appearanceQ4 why the glass panes fixed to window or doors of old building become slightly thicker at bottomA4 Glass is super cooled liquid It has the tendency to flow down very slowly Due to this glass pane becomes thicker at the bottom over the timeQ5 Sodium chloride is a crystalline solid It shows the same value of refractive index along all the direction TrueFalse Give reasonA5 FalseCrystalline solid shows anisotropy in properties That is it shows different values for the given physical property in different direction All the crystalline solids show anisotropy in refractive index Therefore sodium chloride will show different values of refractive index on different directions

Q6 Crystalline solid are anisotropic in nature What does this statement means

between the constituent particles is special kind of electrostatic attraction That is the attraction of positively charged kernel with sea of delocalized electronsQ6 which group of solid is electrical conductor as well as malleable and ductile [CBSE 2013]A6 Metallic solidQ7 why graphite is good conductor of electricity although it is a network (covalent solid)A7 The exceptional property of graphite is due to its typical structure In graphite each carbon is covalently bonded with 3 atoms in same layer The fourth valence electron of each atom is free to move in between different layersThis free electron makes the graphite a good conductor of electricity

[CBSE 2011]A6 Anisotropy is defined asrdquo Difference in properties when measured along different axis or from different directionsrdquo Crystalline solid show different values of some of the physical properties like electrical resistance refractive index etcwhen measured along the different directions The anisotropy in crystalline solid arises due to the different arrangement of particles in different directions

Math Function Composition of functions Think of an industrial plant that produce bottles of cold drinks first there is the operation (or function) f that puts the cold drink inside the bottle followed by the opeartion g that close the bottle with the capThis leads to the following definitionDefinition Let f A rarr B and g B rarr C be two functions Then the composition of f and g denoted by gof is defined as the function gof A rarr C given by gof(x) = g(f (x)) forall x isinA

Definition A function f X rarr Y is defined to be invertible if there exists a function g Y rarr X such that gof = IX and fog = IY The function g is called the inverse of f and is denoted by f -1

Thus if f is invertible then f must be one-one and onto and conversely if f is one-one and onto then f must be invertible This fact significantly helps for proving a function f to be invertible by showing that f is one-one and onto specially when the actual inverse of f is not to be determined

Example 1 Let f 2 3 4 5 rarr 3 4 5 9 and g 3 4 5 9 rarr 7 11 15 be functions defined as f(2) = 3 f(3) = 4 f(4) = f(5) = 5 and g (3) = g (4) = 7 and g (5) = g (9) = 11 Find gofSolution We have gof(2) = g (f(2)) = g (3) = 7 gof(3) = g (f(3)) = g (4) = 7gof(4) = g (f(4)) = g (5) = 11 and gof(5) = g (5) = 11Example 2 Find gof and fog if f R rarr R and g R rarr R are given by f(x) = cos x and g (x) = 3x2 Show that gof ne fogSolution We have gof(x) = g(f(x))=g(cosx) = 3 (cos x)2

= 3 cos2 x Similarly fog(x)=f(g (x))= f(3x2)= cos (3x2) Note that 3cos2 x ne cos 3x2 for x = 0 Hence gof ne fogExample 3 Show that if f A rarr B and g B rarr C are onto then gof A rarr C is also ontoSolution Given an arbitrary element z isin C there exists a pre-image y of z under g such that g (y) = z since g is onto Further for y isin B there exists an element x in A with f(x) = y since f is onto Therefore gof(x) = g (f(x)) = g (y) = z showing that gof is onto Example 4 Let Y = n2 n isin N sub N Consider f N rarr Y as f(n) = n2 Show that

f is invertible Find the inverse of fSolution An arbitrary element y in Y is of the form n2 for some n isin N This implies that n =radicy This gives a function g Y rarr N defined by g (y) =radicy Nowgof (n) = g (n2)=radicn2 = n and fog (y) =f(radicy) = (radicy) 2 y which shows that gof=IN and fog= IY Hence f is invertible with f -1 = g

Political Science

Constitution of India-The Preamble

Summary

Objective of the state-To secure equality of status and of opportunity To promote fraternity among all the citizens To assure the dignity of the individuals and Unity and integrity of the nation

Justice-Justice stands for rule of law absence of arbitrariness and a system of equal rights freedom and opportunities for all in a society India seeks social economic and political justice to ensure equality to its citizens

Liberty-Liberty implies the absence of restraints or domination on the activities of an individual such as freedom from slavery serfdom imprisonment despotism etc The Preamble provides for the liberty of thought expression belief faith and worship

Equality-Equality means the absence of privileges or discrimination against any section of the society The Preamble provides for equality of status and opportunity to all the people of the country

Fraternity-The Preamble declares that fraternity has to assure two thingsmdashthe dignity of the individual and the unity and

Execution

Answer the following questions-

Short notes-1 Equality2 Fraternity3 Justice4 Liberty

Homework-Learn

integrity of the nation The word integrity has been added to the Preamble by the 42nd Constitutional Amendment (1976)

Business studies

Human resource management (chapter 1)

On the day of 1504 2020 I have discussed with you the managerial functions and procurement functions of HRM

Today weare going to discuss about the development function integration functions and maintenance function

Development functions-HRM improves the knowledge skills attitude and values of employees so that they the present and future jobs more effectively it includes

1) Development functions of HRM

a) Performance appraisal = It implies systematic evaluation of employees with respect to their performance on the job and their potential for development

b) Training =It is the process by which employees learn knowledge skills and attitudes to achieve organisational and personal goals

c) Executive development = It is the process of developing managerial talent through appropriate program

2) Integration functionsa) HRM reconcile the goals of

organisation with those of its members through integrating function

b) HRM tries to motivate employees to various financial and non financial incentives provided in job specification etc

3) Maintenance functiona) HRM promote and protect the

physical and mental health of employees by providing several types of benefits like housing medical aid etc

b) It Promote Social security measures to employees by providing provident fund pension gratuity maternity benefits

SubjectCOMMERCE

Topic

BUSINESSENVIRONMENT

Summary

Now quickly let us revise the earlier points that we have already done in the last class and let us proceed with the other topics that are there in the chapter

Firstly we will recall the internal and external factors of micro environment and then we

Execution 3 What do you mean by internal factors

in micro environmentAnswerInternal factors refer to all the factors existing within a business firm The internal factors are considered controllable because the enterprise has control over these factors

Development FunctionsPerformance AppraisalTrainingExecution Development

shall proceed in details

Meaning and list of internal and external factors

aInternal factorsInternal factors refer to all the factors existing within a business firm The internal factors are considered controllable because the enterprise has control over these factorsFor an example a company can alter its organization structure policies programmes employees physical facilities and marketing mix to suit the changes in the environmentList of internal factors areCorporate culture mission and objectives top management organizations structure company image and brand equity company resources

b External factorsExternal factors refer to those individual and groups and agencies with which a particular business organization comes into direct and frequent contact in the course of its functioningThese individuals and groups are known as STAKEHOLDERS because they have a stake (financial interest ) in the working and performance of the particular business List of external forces (stakeholders)Customers competitors investors suppliersmiddlemen (marketing intermediaries)financers publics

customers

suppliersfinancers

For an example a company can alter its organization structure policies programmes employees physical facilities and marketing mix to suit the changes in the environment

4 What do you mean by external factors in micro environment

AnswerExternal factors refer to those individual and groups and agencies with which a particular business organization comes into direct and frequent contact in the course of its functioningThese individuals and groups are known as STAKEHOLDERS because they have a stake (financial interest) in the working and performance of the particular business

3Who are stakeholdersSTAKEHOLDERS are individuals and groups who have a stake (financial interest ) in the working and performance of the particular business 4Discuss the internal factors in briefa Corporate CultureThe values beliefs and attitudes of the founders and top management of the company exercise a strong influence on what the cmpaany stands for how it does things and what it considers importantbMission and objectivesThe business philosophy and purpose of a comoany guide it prioritiesbusiness strategiesproduct market scope and development scope

cTop management structurethe composition of board of directors the degree of professionalization of management and the organizational structure of a company have important bearing on its business decisions

dPower structureThe internal power relationship between the board of directors and the chief executive is an important factor

e Company image and brand equityThe image and brand equity of the company play a significant role in raising finance forming alliance choosing dealers and suppliers launching new products entering foreign markets

5 What is Macro environmentAnswerMacro environment refers to the general

competitors

middlemen

publics

Fig STAKEHOLDERS OF A COMPANY

Apart from micro environment the other main dimension of business environment isMacro environment Macro environment refers to the general environment or remote environment within which a business firm and forces in its micro environment operateA company does not directly or regularly interact with the micro environmentTherefore macro environment is also known as indirect action EnvironmentThe macro environment forces are less controllable than the micro forces

Macro environment consists of the following components

POLITICAL AND LEGAL ENVIRONMENT

ECONOMIC SOCIAL AND ENVIRONMENT

CULTURAL

ENVIRONMENT

TECHNOLOGICAL ENVIRONMENT

Fig COMPONENTS OF MACRO ENVIRONMENT

environment or remote environment within which a business firm and forces in its micro environment operateA company does not directly or regularly interact with the micro environmentTherefore macro environment is also known as indirect action EnvironmentThe macro environment forces are less controllable than the micro forces 6 What are the components of macro environmenta Political and legal environmentb Economic environmentc Social and cultural environmentd Technological environment

Computer Science

Logic gates

Digital systems are said to be constructed by using logic gates These gates are the AND OR NOT NAND NOR EXOR and EXNOR

BUSINESS FIRM

gates The basic operations are described below with the aid of truth tables

AND gate

The AND gate is an electronic circuit that gives a high output (1) only if all its inputs are high A dot () is used to show the AND operation ie AB Bear in mind that this dot is sometimes omitted ie ABOR gate

The OR gate is an electronic circuit that gives a high output (1) if one or more of its inputs are high A plus (+) is used to show the OR operationNOT gate

The NOT gate is an electronic circuit that produces an inverted version of the input at its output It is also known as an inverter If the input variable is A the inverted output is known as NOT A This is also shown as A or A with a bar over the top as shown at the outputs The diagrams below show two ways that the NAND logic gate can be configured to produce a NOT gate It can also be done using NOR logic gates in the same way

NAND gate

This is a NOT-AND gate which is equal to an AND gate followed by a NOT gate The outputs of all NAND gates are high if any of the inputs are low The symbol is an AND gate with a small circle on the output The small circle represents inversion

NOR gate

This is a NOT-OR gate which is equal to an OR gate followed by a NOT gate The outputs of all NOR gates are low if any of the inputs are highThe symbol is an OR gate with a small circle on the output The small circle represents inversion

EXOR gate

The Exclusive-OR gate is a circuit which will give a high output if either but not both of its two inputs are high An encircled plus sign ( ) is used to show the EOR operation

EXNOR gate

The Exclusive-NOR gate circuit does the opposite to the EOR gate It will give a low output if either but not both of its two inputs are high The symbol is an EXOR gate with a small circle on the output The small circle represents inversion The NAND and NOR gates are called universal functions since with either one the AND and OR functions and NOT can be generated

Note A function in sum of products form can be implemented using NAND gates by replacing all AND and OR gates by NAND gates A function in product of sums form can be implemented using NOR gates by replacing all AND and OR gates by NOR gates

Logic gate symbols

Table 2 is a summary truth table of the inputoutput combinations for the NOT gate together with all possible inputoutput combinations for the other gate functions Also note that a truth table with n inputs has 2n rows You can compare the outputs of different gates

Logic gates representation using the Truth table

Example

A NAND gate can be used as a NOT gate using either of the following wiring configurations

Subject Eng Literature (The Tempest ndash William Shakespeare) Topic Act III Scene 3 Lines 53 to 110 (End of the scene) Date 16th April 2020 (2nd Period)

[Students should read the original play and also the paraphrase given in the school prescribed textbook]Summary Questions amp Answers

o Seeing this strange scene all are inclined to believe the tales told by travelers that there truly are ldquounicornsrdquo and ldquothe phoenixrsquo thronerdquo

o As they are about to sit down to the feast the banquet is snatched away by a harpy (Ariel disguised) A spiritrsquos voice (Arielrsquos voice) denounces Alonso Sebastian and Antonio with particular

1 ARIEL You are three men of sin whom Destiny

(Line 53-58)That hath to instrument this

lower world And what is int the never-surfeited sea

Hath caused to belch up you and on this island

Where man doth not inhabit you rsquomongst men

Being most unfit to live I have made you mad

reference to their crime in expelling Prospero from Milan They have not received any punishment for their deed earlier but the time for their punishment has arrived Upon Alonso it pronounces ldquolingering perdition worse than deathrdquo from which there is no remedy except through sincere repentance Ariel then vanishes in thunder and the shapes enter again and carry away the table

o Prospero watching invisibly is very pleased with the performance of Ariel and his (Prosperorsquos) ldquomeaner ministersrdquo All his enemies are now in his power and are in a fit of desperation He then leaves them and goes to see how Ferdinand and Miranda are getting on

o Alonso is now much humbled and penitent with the after effect of the spiritrsquos denunciation of his crimes He believes that his son is lost forever After this all disperse being stricken mad by the speech of the spirit

o Gonzalo fearing that they may do violence to themselves or to one another follows them and bid others to follow

(a) To whom does Ariel disguised as a harpy call the three sinners What game did Fate of Destiny play with

them

The three sinners called by Ariel are Alonso Sebastian and Antonio It was Destiny which had caused the ocean to cast the three sinners on the shore Though the ocean is all the time devouring whatever appears on its surface and is never satisfied with its continual swallowing of the ships and men in the present case the ocean had cast these three sinners on the shore without killing them

(b) Who had jointly been responsible for the conspiracy against Prospero What is Prosperorsquos purpose behind all this

Three men Alonso Sebastian and Antonio had jointly

been responsible for the conspiracy against Prospero They had driven out Prospero form Milan Prosperorsquos purpose is to make these three sinners realize the wrong they had done He wants them to repent for their criminal deeds because repentance leads to self-esteem(c )What does Ariel (the harpy) tell Alonso and his companions when they take out their swords to attack him

Seeing them drawing their swords Ariel (harpy) tells them that he and his companions are the instruments of destiny and that it is not possible for human beings to do them any injury He says that the swords of human beings can not injure even a minute part of his feathers Their swords are as ineffective against him and his companions as against the wind or the water

(d) Give the explanatory meanings of the following expressions in the context of the above extract

(i)Never surfeited (ii) Belch up (iii) lsquomongst men

(i) Never surfeited never led to satisfaction

(ii) Belch up cast ashore(iii) lsquomongst men in human

society2

I and my fellows (Line 60-65)

Are ministers of Fate The elementsOf whom your swords are tempered may as wellWound the loud winds or with bemocked-at stabsKill the still-closing waters as diminishOne dowl thats in my plume

IMPORTANT PASSAGES EXPLAINED

The elements

(Line 61-66)Of whom your swords are tempered may

as wellWound the loud winds or with

bemocked-at stabs

(a) Who is lsquoIrsquo Who are his lsquofellowsrdquo

lsquoIrsquo is referred to Ariel in disguise of a harpy His lsquofellowsrsquo are other spirits serving Prospero the real Duke of Milan who has acquired supernatural powers after being banished from his Dukedom Prospero has settled in this uninhabited island

(b) What are the elements that have temperrsquod the swords Why will it not work against the speaker

The swords (of Alonso and his companions) are tempered by metal (steel) which is taken out of the earth and refined by

Kill the still-closing waters as diminishOne dowl thats in my plume My fellow

ministersAre like invulnerable

In these words Ariel reminds the King and his companions of the utter futility of drawing swords against himself and his fellows Ariel drives Alonso Antonio and Sebastian the three men of sin to desperation ndash a state in which men do violence to themselves They draw swords to strike Ariel But Ariel reminds them that he and the other spirits are the ministers of destiny and nothing can wound them The steel of which their swords are made of may cut the wind or water which being divided always closes up again Even supposing that such things may be possible it is quite impossible that their swords will cut one feather in their plume They are incapable of being wounded by any sword of man Hence it is foolish on their part to attempt to strike at Ariel and his fellow-spirits

For which foul deed

(Line 72-75)The powers delaying not forgetting

haveIncensed the seas and shores yea all the

creatures Against your peace

Ariel enters like a harpy and remaining invisible tells Alonso Sebastian and Antonio that he and other harpies are the agents of Destiny appointed to carry out her decrees He tells them that their punishment for the crime against Prospero which has been so long deferred is now to fall upon them He reminds them that they had expelled Prospero from Milan and set him and his innocent child adrift on the sea and that the sea had paid them back for their sin by the shipwreck and by the calamities they have suffered He tells them that the powers above which did not forget this mean treachery but only deferred the punishment have now engaged the seas and the shores and all living beings including him and his comrades against them The very elements and supernatural agency Ariel adds have taken up the avenging of their crime against Prospero

the action of fire It may cut the wind or water which being divided always closes up again

The sword will not work against the spirits and the harpy because they are the ministers of destiny and nothing can wound them nor it will cut a single feather in their plume

(c )What is the meaning of lsquodowlrsquo in the last line

The term lsquodowlrsquo means a filament or the smallest part of a feather In this context Ariel in disguise of harpy says that their sword cannot even damage the smallest filament of their (Arielrsquos and other spirits) feathers as they are incapable of being wounded by any sword of man

(d) What does the speaker remind the listeners about

Ariel in disguise of harpy reminds Alonso the King of Naples Sebastian Alonsorsquos brother and Antonio the present Duke of Milan and the treacherous brother of Prospero as they being three men of sin He even reminds them that their punishment for their crime against Prospero which has been so long deferred now falls upon them He reminds them that they have expelled Prospero from Milan and has set him along with his innocent infant daughter adrift on the sea So the sea has paid them back for their sin by their shipwreck and the calamities they have suffered since then The harpy rebukes Alonso of his sin that has incensed the Gods and has deprived him of his son as a punishment

(e) How do they respond

When Ariel in disguise of a harpy reminds Alonso Sebastian and Antonio of their past misdeeds and sin Alonso has a look of terror and confusion in his eyes He utters the words of sincere repentance wrung out of his conscience-stricken heart It appears to him that all the elements of nature the sea-waves the wind and the thunder proclaiming a loud voice in the name of Prospero and the crime Alonso has committed against him They are calling upon him to repent There is a deep storm raging in Alonsorsquos breast and the echoes of that storm are ringing in his ears like a clear note of wind-instrument A note of denunciation of Alonsorsquos crime leaves him much humbled and penitent and confirms his belief that his son is lost forever But Sebastian and Antonio shows some courage instead of repentance They wish to kill the spirits or devils if it appears

3

Of my instruction hast thou nothing bated (Line 85-93)

In what thou hast to say So with good life

And observation strange my meaner ministers

Their several kinds have done My high charms work

And these mine enemies are all knit upIn their distractions They now are in my

powerAnd in these fits I leave them while I visitYoung Ferdinand whom they suppose is

drownedAnd his and mine loved darling

Methought the billows spoke and (Line 96-99)

told me of itThe winds did sing it to me and the

thunderThat deep and dreadful organ-pipe

pronouncedThe name of Prosper It did bass my

trespass

These are the words of contrition coming from Alonso Ariel has driven him to a deep repentance for conspiring with Antonio against Prospero He now feels a sincere remorse It appears to him that all the elements of nature the sea-waves the wind and the thunder proclaimed with a loud voice the name of Prospero and the crime Alonso had committed against him They are calling upon him to repent There is a deep storm raging in Alonsorsquos breast and the echoes of that storm are ringing in his ears like the clear note of a wind-instrument

Comment These are the words of sincere repentance wrung out of the conscience-stricken heart of Alonso Alonso who is the lesser villain is the first to give way to remorse under the effect of Arielrsquos speech The words of Ariel seem to him to be the voice of conscience speaking to him He is driven to desperation a state in which he might do violence to his life

(a) Identify the speaker State the context

Prospero the ruler of the island is the speaker The famous banquet scene has been enacted very well Ariel and his junior spirits have played their roles excellently Prospero is glad to say words of praise for them(b) In what way the speakerrsquos instructions have been carried out

According to Prosperorsquos instructions a banquet was presented before the King of Naples and his companions when they were tired and hungry Just when they were preparing to eat the feast the banquet was suddenly removed by exercising supernatural powers All this was done by Ariel Prosperorsquos chief assistant and a powerful spirit

Ariel not only made the feast disappear but also delivered his speech blaming the King and his two companions for their past wicked deeds He warned them to repent for their misdeeds or suffer forever on that uninhabited island

(c) Who are referred to as lsquomeaner ministersrsquo What have they done

Prospero refers as lsquomeaner ministersrsquo to his other lesser spirits who were assisting Ariel in presenting a scene before the kingrsquos party They entered the scene to the accompaniment of music They assumed several strange shapes and brought in a banquet Then they danced about it with gentle actions of salutations thus inviting the King and others to eat the feast

These spirits play their role again when Ariel in the shape of a harpy quits the scene These shapes enter again and dancing with mocking gestures carry away the table

(d) Who are the speakerrsquos enemies What has happened to them

King of Naples Alonso his brother Sebastian and the present Duke of Milan Antonio (Prosperorsquos own brother) are Prosperorsquos enemies With the turn of events they have all been washed ashore on the island which is ruled by Prospero the great magician Actually this happened after the shipwreck caused by a storm which was raised by Prospero with the purpose of bringing these people to his island Prosperorsquos spirits have already confused and terrified these enemies and they are under Prosperorsquos control He can treat them as he likes

(e) What does he say about Ferdinand Explain what is meant by ldquohellip his and mine darlingrdquo

Prospero knows that Alonsorsquos son prince Ferdinand is alive though his father thinks that the prince has been drowned

Prospero refers to his daughter Miranda who is dear to him She is also very dear to Prince Ferdinand who has fallen in love with her They are waiting to be married soon for which they have received Prosperorsquos consent

4

ALONSO O it is monstrous monstrous (Line 95-102)

Methought the billows spoke and told me of it

The winds did sing it to me and the thunderThat deep and dreadful organ-

pipe pronouncedThe name of Prosper It did bass

my trespassTherefore my son ithrsquo ooze is

bedded andIll seek him deeper than eer

plummet soundedAnd with him there lie mudded

(a) In what way does Alonso express his horror when his conscience is awakened by Arielrsquos words

When Alonsorsquos conscience is awakened by Arielrsquos words he expresses his horror at what he has heard He gets the feeling that the waves of the ocean the wind and the loud thunder have spoken to him and uttered the name of Prospero Because of being reminded of his crime in a very loud and rough voice he comes to realize that he has lost his son for his past misdeeds

(b) What does Alonso imagine about his son What does Alonso want to do in his desperate state

Alonso imagines that his son is lying in the mud at the bottom of the sea He feels desperate that he wants to drown himself in the ocean deeper than the plumb-line has ever gone He wants to lie with his son at the bottom of the sea

(c) How do Sebastian and Antonio want to face the evil spirits

Sebastian says that he is not at all afraid of what the harpy has said and that he is prepared to fight any number of such monsters if they appear before him only one at a time Antonio says that he would support Sebastian in the fight against the fiendsyyy

(d) Why does Gonzalo ask Adrian to follow the three men

Gonzalo tells Adrian that all the three men namely Alonso Sebastian and Antonio are in a wild and reckless mood The thought of the heinous crime of which they are guilty has begun to torment their minds So he asks Adrian to follow those three men without loss of time and prevent them from doing anything which the turmoil in their minds might lead them to do

(e) What opinion do you form of Alonso from the above extract

Alonso who is the lesser villain is the first to give way to remorse under the effect of Arielrsquos speech The words of Ariel seem to him to be the voice of conscience speaking to him He is driven to desperation a state in which he might do violence to his life

Subject =Accounts

Ac-12 15420 topic-pL Appropriation ac

PROFIT AND LOSS APPROPRIATION ACCOUNT

MEANING AND PREPARATIONProfit and Loss Appropriation Account is merely an extension of the Profit and Loss Account of the firm The profit of the firm has to be distributed amongst the partners in their respective profit sharing ratio But before its distribution it needs to be adjusted All Adjustments like partnerrsquos salary partnerrsquos commission interest on capital interest on drawings etc are made in this account These adjustments will reduce the amount of profit for distribution This adjusted profit will be distributed amongst the partners in their profit sharing ratio To prepare it at first the balance of Profit and Loss Account is transferred to this account The journal entries for the preparation of Profit and Loss Appropriation Account are given below

1 for transfer of the balance of Profit and Loss Account to Profit and Loss Appropriation Account

(a) In case of Net Profit

Profit and Loss Ac helliphelliphelliphelliphellipDrTo Profit and Loss Appropriation Ac(Net Profit transferred to Profit and Loss Appropriation Ac)

(b)In case of Net Loss

Profit and Loss Appropriation Achelliphelliphellip DrTo Profit and Loss Ac(Net Loss transferred to Profit and Loss Appropriation Ac)

2 for Interest on Capital

For transferring on Interest on CapitalProfit and Loss Appropriation Achelliphelliphellip DrTo Interest on Capital Ac(Interest on capital transferred to Profit amp Loss Appropriation Ac)

3 for Interest on Drawings

For transferring Interest on Drawings Interest on Drawings Achelliphelliphelliphelliphelliphellip DrTo Profit and Loss Appropriation Ac(Interest on drawing transferred to Profit amp Loss Appropriation Ac)

4 For Partnerrsquos SalaryFor transfer of partnerrsquos SalaryProfit and Loss Appropriation Achelliphellip DrTo Salary Ac(Salary transferred to profit amp Loss Appropriation Ac)

5 For Partnerrsquos CommissionFor transferring commissionProfit and Loss Appropriation Achelliphelliphellip DrTo Commission Ac(Commission transferred to Profit and Loss Appropriation Ac)

6 For Transfer of agreed amount to General ReserveProfit and Loss Appropriation Ac helliphellipDrTo General Reserve Ac(Transfer to General Reserve)

7 for share of Profit or Loss appropriation(a) If ProfitProfit and Loss Appropriation Achelliphellip DrTo Partnerrsquos CapitalCurrent Ac(Profit transferred to capitalcurrent Ac)(b) If LossPartnerrsquos Capital Current Achelliphelliphelliphellip DrTo Profit and Loss Appropriation Ac(Loss transferred to capitalcurrent Ac)

THE FORMAT OF PROFIT AND LOSS APPROPRIATION

Profit and Loss Appropriation Account for the year endedhelliphelliphelliphellip

Particulars Amount Particulars Amount

To PL Ac (loss) By pL Ac (profit)

To Interest on capital BY Interest on drawings

To partner`s commission by Partner`s capital Ac ( loss)

To Partner`s salary To Interest on partner`s loan To General Reserve To Partner`s Capital AC (Profit)

Subject= Economics

MOVEMENT ALONG THE DEMAND CURVE (CHANGE IN QUANTITY DEMANDED)In law of demand you have already studied the inverse relationship between price and quantity demanded When quantity demanded of a commodity changes due to change in its price keeping other factors constant it is called change in quantity demanded It is graphically expressed as a movement along the same demand curve There can be either a downward movement or an upward movement along the same demand curve Upward movement along the same demand curve is called contraction of demand or decrease in quantity demanded and downward movement along the same demand curve is known as expansion of demand or increase in quantity demanded

Extention of demandd

price (rs)p A

B Extentionp1 d

Q Q1

Quantity demanded ( in units)

Contraction of demandd

p2 Ccontraction

p APrice (Rs)

d

Q2 Q

Quantity demanded (in units)

Explanation of movement of demand A fall in price from OP to OP1 leads to increase in quantity demanded from OQ to OQ1 (expansion of demand) resulting in a downward movement from point A to point B along the same demand curve DD When Price rises from OP to OP2 quantity demanded falls from OQ to OQ2 (contraction of demand) leading to an upward movement from point A to point C along the same demand curve DD

  • Activity Series of Metals
    • Drawbacks of Rutherfordrsquos model of atom
      • Electromagnetic radiations
      • Properties of electromagnetic radiations
      • Characteristics of electromagnetic radiations
        • Plancks Quantum Theory-
        • Photoelectric effect
          • Intext Questions
            • Logic gates
            • Digital systems are said to be constructed by using logic gates These gates are the AND OR NOT NAND NOR EXOR and EXNOR gates The basic operations are described below with the aid of truth tables
            • AND gate
            • Example
Page 17:  · Web viewSubject. Topic. Summary. Execution. English 1 . Chapter 1 naming words . Page 8. Write the names of these pictures:- Person:-1. father. 2.Firefighter 3.doctor 4 ...

To promote sustainable society the following things need to be done ndash

1 Using renewable energy sources 2 By improving the quality of human

health 3 By promoting sustainable agriculture 4 By forming ecovillage

it will eventually be entirely depleted So Toensure sustainable development the depletion of

renewable resources should nottake place at a rate faster than their regeneration

Q ) What do you mean by Sustainable societies

Ans - Sustainable societies are defined as towns and cities that have taken steps to remain healthy over the long term These communities value healthy ecosystems use resources efficiently and actively seek to retain and enhance a locally based economy Sustainable development concerns everybody in a society

Q ) What are the effects of pollution on human health

Ans ndash Some health problem occurs due to air pollution are ndash

Respiratory diseases Cardiovascular damage Fatigue headaches and anxiety Irritation of the eyes nose and throat Damage to reproductive organs Harm to the liver spleen and blood Nervous system damage

Some health problem occurs due to water pollution are ndash

Typhoid Cholera Dysentry Jaundice

Some health problem occurs due to noise pollution are ndash

Fatigue headaches and anxiety High blood pressure Hearing damage

Physics Motion in 1D First go through previous notes Now here we will solve some numerical related to that

Question 3What information about the motion of a body is obtained from the displacement-time graphSolution 3From displacement-time graph the nature of motion (or state of rest) can be understood The slope of this graph gives the value of velocity of the body at any instant of time using which the velocity-time graph can also be drawn

Question 4(a)What does the slope of a displacement-time graph represent(b)Can displacement-time sketch be parallel to the displacement axis Give a reason to your answerSolution 4(a) Slope of a displacement-time graph represents velocity(b) The displacement-time graph can never be parallel to the displacement axis because such a line would mean that the distance covered by the body in a certain direction increases without any

increase in time which is not possible

Chemistry Language of Chemistry

How to balance a chemical equationThere are two methods of balancing an equation(i)Hit and trial method(ii)Partial equation methodBalancing by hit and trial methodThis method consists of counting the number of atoms of each elements on both sides and trying to equalize themTake the following steps(i)Count the number of times (frequency) an element occurs on either side(ii)The element with the least frequency of occurrence is balanced first(iii)When two or more elements have the same frequencythe metallic element is balanced firstExample-1 On heatinglead nitrate decomposes to give lead dioxidenitrogen dioxide and oxygenPb(NO3)2rarrPbO+NO2+O2

In this equationLead occurs twiceNitrogen occurs twiceOxygen occurs four timesSince lead is a metalbalance it firstThe number of atom of lead is equal on the two sidestherefore it needs no balancingNow balance nitrogenOn the reactant sidethere are two atoms of nitrogenwhile on the product side oneSomultiply the product containing nitrogenon the product sideby two Pb (NO3)2rarrPbO+2NO2+O2Nowthe number of oxygen atoms on the reactant side 6while on the product sideit is 7Somultiply the entire equation by 2except oxygen to get balanced equation2Pb(NO3)2rarr2PbO+4NO2+O2Multiplication by 2 is done only when atoms of all the elements except one element are balanced and the unbalanced atom occurs separately at least once and also there is a difference of only one such atom

Math Topic AlgebraChapter

Factorisation

Study item Difference of two squares a2 ndash b2 = (a+b) (a-b)1) (i) 4x2ndash 25y2

= (2x) 2 ndash (5y) 2= (2x + 5y) (2x - 5y)

(ii) 9x2 ndash 1= (3x)2ndash(1)2= (3x + 1)(3x ndash 1)

2) (i) 150 ndash 6a2= 6(25 ndash a2)= 6(5)2 ndash(a)2= 6 (5 + a) (5 ndash a)

(ii) 32x2 ndash 18y2=2(16x2 ndash 9y2)=2(4x)2 ndash (3y)2= 2(4x + 3y)(4x - 3y)3)(i) (x ndashy )2 ndash 9 = (x ndash y )2 ndash (3)2= (x ndash y + 3) (x ndash y ndash 3)(ii) 9(x + y) 2ndash x 2= (3)2(x + y)2 ndash (x)2=3(x + y)2 ndash (x)2= (3x +3y ) 2ndash(x)2= (3x + 3y + x)(3x +3y ndash x)= (4x + 3y) ( 2x + 3y )

Commercial studies

Basic accounting terms

Today I will give you some questions from the previous study material

Questions1) Define accounting2) What do you mean by debit and

credit

3) Explain the types of account4) Define the following terms

a) Assetsb) Capitalc) Purchased) Debtorse) Transactions

5) Name the types of accounts given below

a) Krishnas accountb) Machinery accountc) Royalty accountd) Salary accounte) Furniture accountf) Audit fee account

Economics Revision Today I will give you some revision questions

Questions1) What do you mean by the terms

rdquowantsrdquo2) Write the difference between

consumer goods and producer goods

3) Define the term utility 4) Explain the different types of utility5) Define

a) Total utilityb) Marginal utility

Subject Eng Literature (The Merchant of Venice ndash William Shakespeare)Topic Act I Scene 3 Lines 1 to 48 (Shylock hellip Cursed be my tribe if I forgive him) Date 16th April 2020 (5th Period)

[Students should read the original play and also the paraphrase given in the school prescribed textbook]Summary Questions amp Answers

This scene takes place in Venice and we are introduced to the rich Jew Shylock Bassanio and Shylock are talking and Bassanio tells Shylock that he wants a loan of three thousand ducats for three months on the personal security of Antonio

o Shylock feels glad because he will be able to bind down Antonio by means of a bond on account of the loan but he tells Bassanio that all the fortunes of Antonio being invested in the merchant ships on the sea it is difficult to depend upon his credit Even under such circumstances Shylock is willing to advance the money on the personal security of Antonio

o Bassanio then invites Shylock to dine with him Shylock says that he is prepared to do anything with the Christians but not eat or drink or pray with them

o While Bassanio and Shylock are talking Antonio appears on the scene Shylock does not seem to take any notice of Antonio but goes on brooding within

(1) SHYLOCK Ho no no no no- my meaning in (Line 15-26)saying he is a good man is to have you understand me that he is sufficient Yet his means are in suppositionhe hath an argosy bound to Tripolis another to the Indies I understand moreover upon the Rialto he hath a third at Mexico a fourth for England and other ventures he hath squanderd abroad Butships are but boards sailors but men there be land-rats and water-rats land-thieves and water-thieves I mean pirates and then there is the peril of waters winds and rocks The man is notwithstanding sufficientmdashthree thousand ducats mdashI think I may take his bond

(a) Who is talking in the beginning of this scene What does Bassanio want from Shylock How does Shylock feel

In the beginning of the scene Bassanio and Shylock are talking to each other Bassanio wants to borrow three thousand ducats from Shylock for three months on the security of Antonio Shylock feels glad at heart that he will get the opportunity of binding Antonio with a bond(b) What risks does Shylock weigh in advancing the money

Shylock says that Antonio has invested all his capital in trading by sea-going ships But the ships are made of wood and the sailors of those ships are ordinary human beings The wood can

himself how he hates Antonio because of his being a Christian because he abuses Shylock in public places Shylock decides that if ever he can get Antonio to his advantage he will teach him a lesson

come to harm and men can commit mistakes and thus the capital invested in ships may be lost Then there are other dangers The goods loaded on the ships can be damaged by rats and thieves which are found both on land and water The ships can also be harmed through sea-storms submerged rocks etc(c) What two important functions does this scene have

The scene has two important functions First it completes the exposition of the two major plot lines of the play Antonio agrees to Shylockrsquos bond ndash three thousand ducats for a pound of flesh and second and more important dramatically this scene introduces Shylock himself In this scene Shakespeare makes it clear at once why Shylock is the most powerful dramatic figure in the play and why so many great actors have regarded this part as one of the most rewarding roles in all Shakespearean dramas(d) Where does this scene take place What kind of treatment has Antonio been giving to Shylock What does Shylock say when Bassanio invites him to dine with him

The action of this scene takes place in Venice Antonio has been in the habit of behaving harshly with Shylock ndash spitting on his beard and footing him like a stranger cur When Bassanio invites Shylock to dine Shylock says that he is prepared to do anything with the Christians but not eat and drink or pray with them

(2) SHYLOCK How like a fawning publican he looks (Line 38-48)I hate him for he is a Christian

But more for that in low simplicity

He lends out money gratis and brings downThe rate of usance here with us in VeniceIf I can catch him once upon the hipI will feed fat the ancient grudge I bear him

He hates our sacred nation and he railsEven there where merchants most do congregateOn me my bargains and my well-won thriftWhich he calls interest Cursed be my tribeIf I forgive him

(a) What is the context in which these words are spoken and what is the idea expressed in it

These remarks are made by Shylock when he sees Antonio coming along after Bassanio told him that the merchant will be his surety for the bond The above mentioned passage reveals Shylockrsquos hatred for Antonio Shylock says that he hates Antonio because he is a Christian and also because he gives loan without taking interest on them thereby bringing down the rate of interest in Venice(b) Explain the meaning of the phrase lsquoa fawning publicanrsquo

The phrase lsquoa fawning publicanrsquo refers to Roman tax collector It is a term of contempt and hatred on the lips of a Jew lsquoFawning Publicansrsquo were Roman tax-gatherers whose ordinary bearings towards the Jews was bullying but whose false pose of lsquohumility and contritionrsquo is touched upon in the parable in New Testament(c ) What light does the above passage throw on the character

of Shylock

The above mentioned speech of Shylock reveals him to be a wicked character having an extreme greed for wealth His intense hatred for Antonio is unjustified He hates Antonio only because he is a Christian and because he lends money without taking any interest on it thereby adversely affecting Shylockrsquos business of lending money on high interest(d) What information do you gather about Antonio from the above given lines

Shylockrsquos statement throws a valuable light on the character of Antonio Antonio appears to be a good Christian and a good human being He helps the people in need by lending them money without charging any interest on it He is a man of simple and good nature This very goodness makes him Shylockrsquos enemy(e) What does Shylock debate within himself and when To whom are the lines mentioned above addressed to

When Bassanio asks the Jew to lend him three thousand ducats on Antoniorsquos surety Shylock begins to debate within himself as to how he should exploit the opportunity of a business deal with his old enemy Antonio

The lines mentioned above are not addressed to anyone The lines are a soliloquy ie a speech made by a character to himself and not meant to be heard by the other characters present

Class XSubject Topic Summary ExecutionEnglish

LiteratureThe Blue Bead 2nd part

Things took a turn for the worst and all of a sudden a crocodile attacked the woman biting on the womanrsquos leg At that moment Sibia got up sprinted grabbed the hay fork and stabbed the crocodile in the eye with all her power Immediately the crocodile let go and went away Sibia saw a small blue bead lying by the river she grabbed it Since she was poor she didnrsquot have necklace Shersquod always wanted one like the other women now she could make one with the blue bead After that she went home and told her mother all about it

Hindi 2nd

Langबड घर की बटी( मशी परमच-)

lsquoबड घर की बटी कहानी का उददशय मधयम वग की घरल समसया को सलझा कर सगदिठत परिरवार म मिमल जलकर परम स रहन का स-श -ना हघर म शानित सथानिपत करन की जिजमम-ारी नारी की होती ह यदि- नारी समझ-ार ह उसम धय और परिरवार क परनित परम ह तो कोई भी घटना परिरवार को निवघदिटत नही कर सकती या कहानी परिरवार को सगदिठत करत हए परम सौहा- स एक दसर की भावनाओ को समझ करउनका सहयोग करत हए जीवन यापन करन की पररणा -ती हमशी परमचदर जी न इस कहानी म सय परिरवार का परनितनिनमिधतव निकया ह यह कहानी बनी माधव सिसह जो गौरी पर क जमी-ार क उनक -ो पतरो की हशरी कठ लाल निबहारीशरीकात का निववाह एकजमी-ार घरान की पतरी आन-ी स हआ थाआन-ी न ख- को ससराल क वातावरण म ढाकतिलया थाएक दि-न आन-ी का अपन -वर लाल निबहारी स झगडा हो जाता ह -ोनो भाई एक दसर स अलग होन की कोकतिशश करत हसभी बह आन-ी न अपन मधर वयवहार स लाल निबहारी को

ldquoइन नतर निपरय गणो को बीए-इनही -ो अकषर पर नयोछावर कर दि-या था इन -ो अकषर न उनक शरीर को निनबल और चहर को कानित ही बना दि-या थाldquo

क) परसतत पकतियो म निकस वयकति क बार म कहा गया ह

ख) इन पकतियो म कौन स नतर निपरय गणो क बार म कहा गया ह

ग) बीए की निडगरी परापत कर लन पर भी उपय वयकति क सवभाव की कया निवशरषता थी

घ) यह नतर निपरय गण निकस वयकति म निवदयमान थ उसक वयकतितव की कया निवशरषता थी

उततर ndashक) परसतत पकति म गौरी पर गाव क जमी-ार

क बड बट शरीकात क बार म कहा गया ह उसन बहत परिरशरम और उ-म क बा- ba की निडगरी परापत की थी अब वह एक -फतर

घर छोडकर जान स रोक कतिलयाइस पर बनी माधव सिसह न कहा निक बड घर की बटी ऐसी ही होती ह जो निबगडा काम बना लती ह अतः शीरषक साथक ह बड घर की बटी आन-ी ह

म कमचारी थाख) भरा हआ चहरा चौडी छाती और डटकर

खाना आदि- एक सबजी ल जवान क गण मान जात ह परत शरीकात न इनही नतर निपरय गणो को अपनी पढाई पर नयोछावर कर दि-या था

ग) बीए की निडगरी परापत कर लन पर भी उपय वयकति(शरी कठ की शारिररिरक तौर पर निनबल और चहर स कानित ही लगत थ इतना ही नही वह मानकतिसक तौर पर भी निपछड हए थ पाशचातय सामाजिजक कथा उस घणा एव पराचीन सभयता का गणगान उनकी निवचारधारा क परमख अग थ

घ) यह नतर निपरय गण गौरीपर गाव क जमी-ार क छोट बट लाल निबहारी सिसह म निवदयमान थ वह सजीलाजवान था और भस का दध शर दध वह सवर उठकर पी जाता था

ldquoयही कारण था निक गाव की लललन आए उनकी निन-क थी कोई कोई तो उह अपना शतर समझन म भी सकोच ना करती थी सवय उनकी पतनी को इस निवरषय म उनस निवरोध थाldquo

क) उपय पकति म इस वयकति क बार म कहा गया ह

ख) गाव की लललन आए उनकी निन-ा कयो निकया करती थी

ग) उनकी पतनी का कया नाम था उनह निकस निवरषय म अपन पनित क निवरa था और कयो

घ) इस कहानी का कया उददशय ह Continue to next helliphelliphellip

Bengali 2nd Language

ফ ফটক না ফটক( কহিতা )

পর) ldquo(ান াধাচেনা ফটপাচেথ পাথচের পাড হিচেয় এক কাঠচোটটা গাছ কহিকহি পাতায় পার ফাটিচেয় াসচেছldquoক) কার দো দেকান কহিতার অং( ) lsquo(ান াধাচেনা ফটপাচেথ পাথচের পাডহিচেয়lsquo চেত কী দোঝাচেনা চেয়চেছ গ) আচো য অংচে(lsquo এক কাঠচোটটা গাছ lsquoচেত কী দোঝাচেনা চেয়চেছ ঘ) ldquoকহিকহি পাতায় পার ফাটিচেয় াসচেছldquo ----- একথার পরকত অথC কী উততর ) ক) আচো য অং(টি পর যাত কহি সভা4 মচোপাধ যাচেয়র দো lsquoফ ফটক না ফটকrsquo কহিতার অং()কহি সভা4 মচোপাধ যায় হিছচেন দেপরচেমর কহি দেপরমচেক নানা ভহিঙগমায় হিতহিন ফটিচেয় তচেচেছন দেপরম মানচের স মচেতC র সঙগী কহিতার কহিতায় এক রb সb হচেয়র দেপরম াগরচেনর কথা চেচেছন (ান অথCাৎ দেযাচেন দেকান রস দেনই দেযাচেন দেকান মহিনতা দেনই অথ তার মধ দেযও দেপরম থাকচেত পাচের একথাই কহি তচে ধরচেত দেচেয়চেছন একটি মানচে4র মচেন দেযাচেন দেকামতার দেকান সথান দেনই পাথচেরর মচেতা হিনরসতার মচেনর মধ দেযও দেয দেপরম আসচেত পাচের দেস কথাই কহি চেচেছনগ)নারীচের যথC দেপরচেমর ছহি এই কহিতায় অকপচেট উচেঠ এচেসচেছ কহি এই কহিতায় কাটচোটটা গাছ কথাটি যার কচেরচেছন নারী দেয দেপরম দেথচেক হিতাহিত এং দেসই দেপরম সঠিক সমচেয় না পাওয়ার ন য দেপরম সমপচেকC হিচেr4 গৈতরী য় দেপরচেমর দেয গৈহি(ষট য মাধযC য সরসতা দেকামত এই সমসতর হিপরীত যথা রbতা শষকতা কচেঠার তা পরভহিত দোঝাচেত এক কাঠচোটটা গাছ কথাটি যার কচেরচেছনঘ) এাচেন এক নারীর যথC দেপরচেমর কথা হিনহিCপত ভাচে চেচেছন কহি অসমচেয় নারীর ীচেন দেপরম দেচেগচেছ এতহিন তার হয় রb কচেঠার হিছ দেপরচেমর অভাচে ঠাৎ দেসই শষক মরভহিমচেত সচের আভাস এচেসচেছ দেপরম দেযন 4Cার স(ীত তাই পরায় মত গাচেছ কহিকহি পাতা গহিচেয় উচেঠচেছ

Biology Chapter - 01Controlling Air Pollution

Today we will discuss how we control air pollution from domestic combustion

Q1Describe any five ways of reducing air pollution from domestic sources bull The number of pollutants in the air is verylarge and we always try to control them byfollowing ways

i) Solar cooker and solar heater It use no fuel reduce damage of environment by fuel use or reducing deforestation It maintains coolness of house It releases very less orno oil gas or grease

ii) Piped natural gas (PNG) It emits very less by products into the atmosphere As it isdistributed through pipe lines so there iscontinuous supply of fuel is possible

iii) Liquefied Petroleum Gas (LPG) It hasa higher heating value LPG doesntcontain sulphur so it burns a lot cleanerenergy sources It releases very less oralmost no fume in air

iv) Electricity based cooking Emission free cooking alternative for urban dwellers causeselimination of adverse health impactsofindoor air pollution It helps to avoid theinconveniences associated with procurement of LPG

v) Biogas It contains 75 methane whichmakes it an excellent fuel It burns without smoke and biogas plant leaves no residue like ash in wood charcoal etc Thus it isaclean fuel

Economics

Factors of Production

Today firstly we would recall the last class for 5 mins and then we would proceed with the further topics of the chapter

The concept meaning of land characteristics of land and importance of land to be repeated for the absentees as well as the students who were there in the class the previous day

Today we will start with the last portion of land before it the meaning of land to be repeated onceAs by now we all know that

Questions1What do you mean by productivity of landAnswer By productivity of land we mean the capacity of a piece of land to produce a crop

Thus it refers to the average output per unit of landSay per acre per hectare etc= (OutputArea of land)

2 What are the factors influencing the productivity of landAnswer

Natural factors Productivity of land is largely determined by the natural

Land is defined to include not only the surface of the earth but also all other free gifts of nature(for example mineral resources forest resources and indeed anything that helps us to carry out the production of goods and services but is provided by nature free of cost)

We will move on to the last portion of land by discussing Productivity of Land

By productivity of land we mean the capacity of a piece of land to produce a crop

Thus it refers to the average output per unit of land

Say per acre per hectare etc= (OutputArea of land)

With this we shall proceed further with the main factors that determine the productivity of land

Natural factors Human factors Improvements on land Location of land Organisation Ownership of land Availability of capital Proper use of land State help

Note economic development of a country depends upon the quality of its land If the land is fertile it will quicken the pace of development of the country

qualities of land such as fertility etc

Human factors Land cannot produce anything by itself Man has to apply labour on it to produce for himself So productivity of land depends on the knowledge and skills of workers

Improvements on land production of land is affected by land development measures like provision of well or tubewell irrigation proper drainage

State help The government of a country especially less developed country can play a vital role in improving the agricultural productivity by providing better irrigation facilities

Organisation Productivity of land also fdepends upon the way how the factors of production like labour and capital are organised

In order to increase productivity trained workers modern implements scientific methods good seeds are all essential

3 lsquoImproved technology affects the productivity of landrsquo Explain this statement with the help of suitable example Answer Use of improved technology raises the productivity of land Example By using HYV seeds chemical manures and modern machines per hectare output increases

Physics Force (Summary)

Question Write the expression for the moment of force about a given axisSolutionsThe expression for the moment of force is given byMoment of force about a given axis = Force times perpendicular distance of force from the axis of rotationQuestion What do you understand by the clockwise and anticlockwise moment of force When is it taken positiveSolutionsIf the effect on the body is to turn it anticlockwise moment of force is called the anticlockwise moment and it is taken as positive while if the effect on the

body is to turn it clockwise moment of force is called the clockwise moment and it is taken as negative

Math Topic Commercial Mathematics

Chapter Goods and services Tax

Study item Some solved sums from exercise ndash 1 A retailer buys a TV from a wholesaler for Rs 40000 He marks the price of the TV 15 above his cost price sells it to the consumer at 5 discount on the marked price If the sales are intra ndash state and the rate of GST is 12 find

(i) The marked price of the TV(ii) The amount which the consumer pays for the TV(iii) The amount of tax (under GST) paid by the retailer to the central

Government(iv) The amount of tax (under GST) received by the State Government

Solution As the sales are intra- state sale and the rate of GST 12 So GST comprises of 6 CGST and 6 SGSTTherefore a retailer buys a TV from a wholesaler for Rs 40000Therefore the amount of GST collected wholesaler from the retailer or paid by retailer to wholesalerCGST = 6 of Rs 40000 = Rs(6100 times40000) =Rs 2400SGST = 6 of Rs 40000 = Rs (6100 times 40000) =Rs 2400Therefore wholesaler will pay Rs 2400 as CGST and Rs 2400 as SGSTTherefore amount of input GST of retailer Input CGST = Rs 2400 and input SGST = Rs 2400Again the retailer marks the price of the TV 15 above his cost price(i) The marked price of the TV

= Rs 40000 + Rs 40000times15= Rs 40000 + Rs 40000times 15100= Rs 40000 + Rs 6000Rs 46000But the retailer sells it to consumer at 5 discount on the marked priceCost price after discount = Rs 46000 ndashRs46000times 5100 =Rs 46000 ndashRs 2300= Rs 43700Therefore the amount of GST collected retailer from consumer or paid by consumer to retailerCGST = 6 of Rs 43700 =Rs ( 6100 times43700)Rs 2622SGST = 6 of Rs 43700 = Rs (6100 times 43700) =Rs 2622Amount of the output GST of retailer Output CGST = Rs 2622 and output SGST = Rs 2622

(ii) The amount which the consumer pays for the TV= cost price of TV to consumer + CGST paid by consumer + SGST paid by consumer= Rs 43700 + Rs 2622 + Rs 2622= Rs 48944

(iii) The amount of tax (under GST ) paid by the retailer to the central Government=CGST paid by retailer = output CGST ndash input CGST=Rs 2622 ndash Rs 2400=Rs 222

(iv) The amount of tax ( under GST ) received by the State Government = SGST paid by wholesaler + SGST paid by retailer= Rs 2400 + output SGST ndash input SGST=Rs 2400 + Rs 2622 ndash Rs 2400=Rs 2400 + Rs 222= Rs 2622

Commercial studies

Stakeholders Today I am going to give some revision questions from the previous study material

Questions1) State the two expectations of

employees from a business concern2) Give two distinctions between

stakeholder and shareholder3) Give two difference between

internal stakeholders and external stakeholders

4) Give two expectations of suppliers from a business organisation

5) Who is a stakeholder in commercial organisations

Chemistry Periodic Table

Merits of Mendeleevrsquos Periodic law are as follows - 1He grouped the elements on the basis of atomic mass 2 He left gaps for undiscovered elements like Gallium Scandium germanium Also he left a full group vacant for undiscovered inert gases 3 He could predict proportions of several elements on basis of their position in periodic table like Ga Sc etc 4He could predict errors in atomic weights of some elements like gold platinum etc

Anomalies in Mendeleevrsquos Periodic law are as follows - 1 Position of isotopes could not be explained 2 Wrong order of atomic masses could not be explained

For example- as Arnur atomic mass 40 come first and K with low atomic mass (30) should come later but k should be placed first

According to Bohrrsquos Modern Periodic table properties of elements are periodic functions of their atomic numbers

So when elements are arranged according to increasing atomic numbers there is periodicity in electronic configuration that leads to periodicity in their chemical properties

It consists of horizontal rows (Periods) Vertical column (Groups)

There are 7 period and 12 groups in this long form of periodic table

Ist period has 2 elements IInd period has 8 elements IIIrd period has 8 elements IVth period has 18 elements Vth period has 18 elements VIth period has 32 elements VIIth period hs rest of elements

Note - The number of valence electrons in atom of elements decides which elements will be first in period and which will be last

In group- 1 to 2 gp and 13 to 17 contain normal elements 3 to 12gp ndash transition elements 57 to 71 - lanthanides 89 to 103 - Actinides

Left hand side ndash metals Right hand side ndash nonmetals

Note- Hydrogen element has been placed at top of Ist group Electronic configuration of H is similar to alkali metal as both have 1 valence electron

V electron of gp I element -- 1 V electron of gp 2 element -- 2 V electron of gp 13 element -- 3 V electron of gp 14 element -- 4 V electron of gp 15 element -- 5 V electron of gp 16 element --6 V electron of gp 17 element -- 7 V electron of gp 18 element -- 8

English 1 Transformation of sentences

Sentences A sentence is a group of words which makes complete sense

Exercise 2Change the following sentences from

a Assertive sentencesb Imperative sentencesc Interrogative sentencesd Exclamatory sentences

Sentences can be changed from one grammatical form to another without changing the meaning of the sentence This is known as transformation of sentences

assertive to interrogative1 Nobody would like to be a fool

Who would like to be a fool2 Their glory can never fade

When can the glory fade3 Nobody can control the wind

Who can control the wind4 It matters little if I die

What though I die5 No man can serve two masters

Can any man serve two masters

Exercise 3Interchange of assertive and Exclamatory sentences

1 She leads the most unhappy lifeWhat an unhappy life she leads

2 This is indeed an interesting bookWhat an interesting book this

3 He is a very great manWhat a great man he is

4 It is a very lame excuseWhat a lame excuse

5 It is sad that she died so youngAlas she died so young

Class XISubject Topic Summary Execution

Hindi 2nd lang

पतर परम(परमचदर) पतर परम कहानी म एक निपता की इचछाओ का वणन निकया गया ह अपन बड पतर परभ -ास स निपता चतनय -ास का निवशरष परम था निपता को उसक जनम स ही बडी-बडी आशाए थी उसम दसर बट कतिशव-ास की अपकषा स- उतसाह की मातरा अमिधक थी वह उस इगलड भजकर बरिरसटर बनाना चाहत थभागय का खल भी बडा निनराला ह बीए की परीकषा क बा- वह बीमार पड गया डॉकटरो न भी जवाब - दि-या थाचतन -ास जी बहत ही कजस थ बवजह पस खच करना नही चाहत थ अगर गारटी मिमलती तो शाय- पस खच भी कर -त परत गारटी नही थी परिरणाम सवरप उनक बट का -हात हो गयाजब बट को समशान ल जा रह थ तो वहा काफी शोर गान बजान हो रह थ पछन पर पता चला निक निकसी निपता निपछल तीन साल स निबमार था और उसक ईलाज म रपया पानी की तरह बहाया पर ठीक नही हए परत उसक बट को तनिनक भी अफसोस नही था उसका कहना था उसन कोकतिशश तो कीयह -खकर चतनय-ास जी को आतम निगलानी हईतभी स उनका म परिरवतन हआ और बट का भोज काफी धमधाम स निकयाऔर वहइस पशचाताप की आग म जलत रह औला- स बढकर पसा नही होता ह इस बात को समझन म उनह काफी व लग गया

hellipContinue to next

BENGALI(2ND LANGUAGE)

পরথমঅধযায়-ঠাকরারীনদরনাথঠাকর

নয়ন দোচের হিমাচেররা া নাচেমই হিযাত হিছচেন ায়ানার উাররণ সবরপ নয়ন দোচের ারা হিা (াচেকর হিা হিচেতন এছাাও দেকান উৎস উপচেb রাহিতর দেক হিন করার উচেfচে(য তারা সযC হিকরচেরণ রনয পরীপ জবাহিচেয় তাচেত রপার হির 4Cরণ করচেতন ঠাকরা এই নয়ন দো হিমারচের দে(4 ং(ধর হিছচেন হিমাররা ায়ানার ষটানত পর(Cন কচের তারা হিনঃসব এই হিমাহিরর দে(4 ং(ধর গৈকাস নদর রায়চেৌধরী গৈকাস া নয়ন দোচের সমসত সমপহিতত ঋচেরণর াচেয় হিহিx কচের অহি(ষট যা আচেছ তাচেত হিপত

ইার হিপতার মতয ইচে পর নয়নচোচের ায়ানার দেগাটা কতক অসাধাররণ শরাদধ (াহিনতচেত অহিনতম ীহিপত পরকা( কহিরয়া ঠাৎ হিনহিয়া দেগ- ক) কার দো দেকান গচেলপর অং() কতা দেক ইার চেত কাচেক দোঝাচেনা চেয়চেছ গ) পরসঙগ কী কতার কতয পহিরসফট কচেরা

পরচে4র যাহিত রbা করা সমভ নয় তাই হিতহিন পতরচেক হিনচেয় ককাতায় সাস শর কচেরন গলপ কথচেকর আহিথCক অসথা নয়ন দোচের হিমাচের দেথচেক সমপরণC আাা কথচেকর হিপতা হিনচের দেষটায় অথC উপাCন করচেতন া উপাহিধ াচেভর নয তার াসা হিছনা আর দেসই কারচেরণ কথক তার একমাতর উততরাহিধকার চেয় তার হিপতার পরহিত কতজঞ কথক দো পা হি(চেচেছন হিনচের পরারণ ও মান রbার নয উপচেযাগী অথC হিনা দেষটায় পরাপত চেয়চেছন- এটাই তার কাচেছ পরম দেগৌরচের হি4য় চে মচেন কচেরন কাররণ (নয ভাণডাচের গৈপতক ায়ানার উজজব ইহিতাস অচেপbা দোার হিসeচেকর মচেধয গৈপতক দেকামপাহিনর কাগ তার কাচেছ অচেনক দেহি( মযান

TO BE CONTINUED

উ- ক) আচোয অং(টি রীনদরনাথ ঠাকচেরর দো ঠাকরা গচেলপর অং() কতা চেন আচোয গচেলপর গলপ কথকইার চেত নয়ন দোচের হিমাহিরর দে(4 ং(ধর গৈকাস ার কথা া চেয়চেছ গৈকাস া নয়ন দোচের সমসত সমপহিতত ঋচেরণর াচেয় হিহিx কচের অহি(ষট যা আচেছ তাচেত হিপত পরচে4র যাহিত রbা করা সমভ নয় তাই হিতহিন পতরচেক হিনচেয় ককাতায় সাস শর কচেরনগ) গৈকাস ার হিপতার মতযর পর নয়ন দোচের হিমাহিরর অহিসততব হিপত য় কচেয়কটা উৎস ও শরাদধ- (াহিনতচেত হিমাহিরর দে(4 কহিটক যয় চেয় হিগচেয় এচেক াচের দে(4 চেয় যায় তন তাচের গC করার মত আর হিকছই হিছ না-দেসই পরসচেঙগ এই উহিকত নয়নচোচের হিমাচেররা া নাচেমই হিযাত হিছচেন ায়ানার উাররণ সবরপ নয়নচোচের ারা হিা (াচেকর হিা হিচেতন এছাাও দেকান উৎস উপচেb রাহিতরচেক হিন করচেত হিগচেয় তারা সযC হিকরচেরণর নয পরীপ জবাহিচেয় তাচেত রপার হির 4Cরণ করচেতন তাই দেসকাচের ায়ানা দেহি(হিন সথায়ী চেত পারত না হিহিভনন উৎস শরাদধ- (াহিনতচেত সাধযা হিতহিরকত র করার নয হিমাহির হিহিকচেয় দেযত হ হিতC কা হিহি(ষট পরীচেপর দেত দেযমন অলপকাচের মচেধয হিনঃচে(4 চেয় যায়-নয়নচোচের হিমারচের অসথা তাই চেয়হিছ এই কারচেরণই কথক নয়নচোচের হিমারচের গা ভরা আমবর সয করচেত পারতনা

Physics Dimensional Analysis (Summary)

Q Find the dimensions of consts ab in relation

p=(bminusxlowastx)at

where p is the power x is the distance and t is time

Ans From principle of homogeneity dimension of b x2 are same Dim of b = dim of x2 = [L2] = [ML2T0]Dim of a = dim of ( b- x2)dim of (pt) = [M0L2T0][ML2T-2] [T-1] [T] = [M-1L0T2]

Chemistry Atomic Structure Drawbacks of Rutherfordrsquos model of

atom a According to Rutherfordrsquos model of atom electrons which are negativelycharged particles revolve around the nucleus in fixed orbits Thusb theelectrons undergo acceleration According to electromagnetic theory of Maxwell a charged particle undergoing acceleration should emitelectromagnetic radiation Thus an electron in an orbit should emitradiation Thus the orbit should shrink But this does not happenc The model does not give any information about how electrons aredistributed around nucleus and what are energies of these electrons Isotopes These are the atoms of the same

Properties of electromagnetic radiationsa Oscillating electric and magnetic field are produced by oscillating charged particles These fields are perpendicular to each other and both areperpendicular to the direction of propagation of the waveb They do not need a medium to travel That means they can even travel invacuum

Characteristics of electromagnetic radiationsa Wavelength It may be defined as the distance between two neighbouring crests or troughs of

element having the same atomicnumber but different mass numbere g 1H11H21H3

Isobars Isobars are the atoms of different elements having the same massnumber but different atomic numbere g 18Ar40 20Ca40

Isoelectronic species These are those species which have the same numberof electrons

Electromagnetic radiationsThe radiations which are associated withelectrical and magnetic fields are called electromagnetic radiations When anelectrically charged particle moves under acceleration alternating electricaland magnetic fields are produced and transmitted These fields aretransmitted in the form of waves These waves are called electromagneticwaves or electromagnetic radiations

wave as shown It is denoted by λb Frequency (ν) It may be defined as the number of waves which passthrough a particular point in one secondc Velocity (v) It is defined as the distance travelled by a wave in onesecond In vacuum all types of electromagnetic radiations travel with thesame velocity Its value is 3 times10 8m sec-1 It is denoted by v

d Wave number Wave number is defined as the number of wavelengths per unit lengthVelocity = frequency timeswavelength c = νλ

Plancks Quantum Theory- o The radiant energy is emitted or absorbed not continuously but discontinuously in the form of small discrete packets of energy called lsquoquantumrsquo In case of light the quantum of energy is called a lsquophotonrsquoo The energy of each quantum is directly proportional to the frequency of the radiation ie E α υ or E= hυ where h= Planckrsquos constant = 6626 x 10-27 Js o Energy is always emitted or absorbed as integral multiple of this uantum E=nhυ Where n=1234Black body An ideal body which emits and absorbs all frequencies is calleda black body The radiation emitted by such a body is called black body radiation

Photoelectric effectThe phenomenon of ejection of electrons from thesurface of metal when light of suitable frequency strikes it is calledphotoelectric effect The ejected electrons are called photoelectrons

Biology Chapter - 02Systematics and Five Kingdoms

Scientists divide the whole living organisms into two kingdom first and ultimately by five kingdom at last

In the earlier systems of classifications organisms are divided into kingdom plantaeand kingdom animalia on the of presenceof cell wall their modes of nutrition and movements

Some problem arise like fungi share manycharacteristic withplant despite their heterotrophic nutrition bacteria protozoa areunicellular present in both kingdom Toovercome this third kingdom Protista isintroduced which include

unicellularorganisms But there is also another

problem Allunicellular organisms are not similar kind The cellular structure of prokaryotes is verydifferent from that of other organismsEukaryotes possess a true nucleus and allcell organelles that are not present inprokaryotes So the fourth kingdom Monerais introduced which include unicellular prokaryotes (bacteriaamp blue green algae)

bull Still some problem arise in kingdomplantae

So in 1969 R H Whittakar proposedanew five kingdom System of classification

i) Kingdom Monera - unicellular prokaryotes

ii) kingdom Protista - unicellular eukaryotes

iii) Kingdom Fungi - uni or multicellular fungi with cell wall but without chlorophyll

iv) Kingdom Plantae - Multicellular Plants

v) Kingdom Animalia - Multicellular Animals

EVS Chapter 1 ndash Modes of Existence

An agricultural society

An agricultural society also known as an agrarian society is a society that constructs social order around a reliance upon farming More than half the people living in that society make their living by farming

People in an agricultural society generally lead a more settled lifestyle than those in nomadic hunter-gatherer or semi-nomadic pastoral societies because they live permanently near the land that is farmed Agricultural settlements tend to develop in areas of convenience near bodies of water which is used for both crops and transportation or along trade routes Not everyone in an agricultural society is a farmer Some people make a living trading or making and selling goods such as tools used for farming

Another way to define an agrarian society is to see the total amount of production in a nation In an agrarian society cultivating the land is the main source of wealth Such a society can recognize other means of subsistence and work habits but emphasizes the importance of agriculture and livestock Agrarian societies have existed in various parts of the world for 10000 years and continue to exist today They have been the most common form of socio-economic organization for most of recorded human history

Q) Write the features of agricultural society

Ans - Structure and Features of Agrarian Society1 Occupational Structure

An agrarian society is generally associated with the domestication of plants and animals The domestication of plants means farming and that of animals means herding Often there is mixture of farming and the use of such domesticated animals as cow goat and sheep

2 Forms of Land Ownership in Agrarian SocietiesGenerally there are landlords supervisory farmers cultivators and share croppers The landholders own the land but do not work on it They let it out for sharecropping The supervisory farmers are those who live by having their land cultivated by hired labourers The cultivators cultivate the land for themselvesThe share-croppers are those who live by tilling other peoplersquos land or a crop-sharing basis The artisans own their means of production and produce by their own labour in their homesteads

3 Village Community System An agrarian society is highlighted by

the institution of village community system The agrarian economy made fixed dwelling houses necessary Living close together for protection and co-operation and living nearer to the land gave birth to agricultural villages The village is not only the residential place of farmers it is also the social integrator

4 Minimal Division of Labour Another structural feature of agrarian society is a minimal division of labour Except for the basic division founded on age and sex differences there are few specialized roles There is only one predominant type of occupation ie domestication of plants and animals For all the people the environment physical as well as social is the same

5 Role of Family The farm family is of the patriarchal type the father is the final arbiter in most of the familyrsquos major decisions The life of ail men and women is merged in family life Since there are not many special organizations family is the only organisation to perform the tasks of aid and protection

6 Sense of Unity The members of an agrarian society exhibit a strong in-group feeling Since the whole of their social lives is wrapped up in a society which is physically economically and socially homogenous they are inclined to view the entire outside world as an out group

7 Informal Social Control An agrarian society is regionally divided into villages In a village community the force of traditional mores is more dominant than in the urban community In the village everybody is known to everybody The members in a village community help each other and share the joy and sorrows of each other Crime in an agrarian society is rare

8 Simplicity and Uniformity Life of the people in an agrarian society is marked by simplicity and uniformity Their main occupation is agriculture which largely depends upon the vagaries of nature An agrarian society is a religious society

Math Compound angles Compound angles The algebraic sum of two or more angles is called a compound angle If A B C be three angles then A+B B+C C+A A-B B-C A-C A+B-C etc are compound angles In this chapter we shall discuss the trigonometrical ratios of compound angles Theorem 1 If A B and A+B are all pisitive acute angles theni) sin( A+B) = sin A cos B + cosA sinBii) cos(A+B) = cosA cosB- sinA sinBTheorem 2If A and B are positive acute angles and AgtB theni) sin(A-B) = sin A cosB- cos A sinBii) cos(A-B) = cos A cos B+ sin A sin BTo prove that i) sin(A+B) sin (A-B) = sin2 A - sin2 B = cos2 B- cos2 A

Example 1 Prove that tan70deg=2tan50deg+tan20degSolutiontan70deg = tan(50deg + 20deg)Or tan70deg=(tan 50deg+tan 20deg)(1-tan50degtan20deg) or tan70deg (1 ndash tan 50deg tan20deg) = tan50deg+tan20degor tan70deg= tan70deg tan50deg tan20deg+ tan50deg + tan20deg = cot20deg tan50deg tan20deg + tan50deg + tan20deg = 2 tan50deg+ tan20degExample 2 If A + B = 45deg show that (1 + tanA) (1 + tanB) = 2Solutiontan(A + B) =( tan A + tan B) (1 - tan

ii) cos(A+B) Cos(A-B) = cos2 A- sin2 B = cos2 B -sin2 AProof i) LHS= sin(A+B)sin(AminusB) [Recall sin(αminusβ)=sinαcosβminuscosαsinβ And sin(α+β)=sinαcosβ+cosαsinβ]= (sinAcosB+cosAsinB)times(sinAcosBminuscosAsinB)= sin2Acos2Bminuscos2Asin2B [Recall sin2α+cos2α=1 From above we can then assume correctly that sin2α=1minuscos2α AND cos2α=1minussin2α] = sin2A(1minussin2B)minussin2B(1minussin2A) = sin2Aminussin2Asin2Bminussin2B+sin2Asin2B = sin2Aminussin2B= 1-cos2A-(1-cos2B) = cos2 B- cos2 A = RHSii)LHS= cos (A+B) cos(A-B) [ cos(A+B) = cos AcosB- sinAsinBCos(A-B) = cosAcosB+ sinAsinB]= cos2 A Cos2 B- sin2 A Sin2 B= cos2 A( 1-sin2 B) - (1- cos2 A) sin2 B= cos2 A- cos2 A sin2 B- sin2 B+ cos2 A sin2 B=cos2 A- sin2 B=1- sin2 A-(1-cos2 B) = cos2 B- sin2 A= RHSTangent formulae for compound anglesi)tan (A + B) = tan A + tan B1-tan A tan Bii)tan (A ndash B) = tan A-tan B1+tan A tan Biii) cot (A + B) = cot Acot B-1cot A+cot B(viii) cot (A ndash B) = cot Acot B+1cot B-cot A

A tan B) Or 1= (tan A+ tanB) (1-tan A tanB) Or tanA + tanB + tanA tanB + 1 = 1 + 1Or tanA (1 + tanB) + (1 + tanB) = 2Or (1 + tanA) (1 + tanB) = 2Example 3 Find the value of sin 15degSolution sin 15deg= sin(45deg-30deg) = sin45degcos 30deg- cos45degsin30deg =(1radic2) (radic32) -(1radic2) (12) = (radic3-1) 2radic2Example 4 If sin A = 1 radic10 and sin B = 1 radic5 where A and B are positive acute angles then what is A + B SolutionWe know that sin (A + B) = sin A cos B + cos A sin B= [1 radic10] [radic(1 minus 1 5)] + [1 radic5] radic(1 minus 1 10)= [1 radic10] [radic4 5] + [1 radic5] [radic9 10]= [1 radic50] times (2 + 3)= 5 radic50 = 1 radic2

sin (A + B) = sin π 4rArrHence A + B = π 4Example 5 If A + B = 225o then find [cot A] [1 + cotA] times [cot B] [1 + cot B]Solution[cot A] [1 + cotA] times [cot B] [1 + cot B] = 1 [(1 + tan A) times (1 + tan B)]=1 [tan A + tan B + 1 + tan A tan B] [ tan (A + B) = tan225o]∵

tan A + tan B = 1minus tan A tan BrArr= 1 [1 minus tan A tan B + 1 + tan A tan B]= 1 2

COMMERCE

CLASSIFICTION OF HUMAN ACTIVITIES-ECONOMIC AND NON-ECONOMIC

Firstly we shall recall the previous class for 5 mins especially for the absentees and for also the rest of the students who were there

Today at first we briefly discuss the earlier portions of the chapter

1Business-It includes all those economic activities which are concerned with production and exchange of goods and services with the object of earning profit Example A factory shop beauty parlour also business enterprises

2Profession ndashThe term profession means an occupation which involves application of specialized knowledge and skills to earn a living For Example Chartered Accountancy medicine law tax consultancy are example of professions

Questions1What are the main features of ProfessionAnswer The main features of a profession are as follows a Specialised body of knowledge-Every profession has a specialised and systematised body of knowledge b Restricted entry- Entry to a profession is allowed only to those who have completed the prescribed education and have the specialised examination c Formal education and training ndashA formal education and training is given to the person who wants to acquire the professional

3Employment-Employment mean an economic activity where people work for others in exchange for some remuneration (salary)The persons who work for others are called lsquoemployeesrsquo The persons or organizations which engage others to work for them are called lsquoemployersrsquoEg A doctor working in a hospital is employment as he is working for a salaryA lawyer may serve as a law officer in a bank

With this we shall proceed with the features of both Profession amp Employment

The main features of a profession are as follow

a Specialised body of knowledge b Restricted entry c Formal education and training d Professional association e Service motive f Code of contact

The main features of an employment are as follows

a In employment a person works for others called employer

b An employee provides personal service

c There is a service agreement or contract between the employee and the employer

d The employee has to obey the order of the employer

e No capital investment is made by the employer

Various examples of Employment are as follows

aA teacher teaching in a school or collegeb An engineer employed in Municipal Corporation of DelhicAn accountant working in the accounts department of a companydA doctor working in a hospital

Note In all the above examples of employment the individual who is involved in each example is working as an employee for a salary under an employer

qualification(MBBSCALLB)d Service motive ndashProfessionals are expected to emphasis service more on their clients rather than economic gain f Code of Conduct-The activities of professionals are regulated by a code of conduct

2 What are the main features of EmploymentAnswer The main features of an employment are as followsa In employment a person works for others called employerb An employee provides personal servicec There is a service agreement or contract between the employee and the employerd The employee has to obey the order of the employere No capital investment is made by the employer

3 Give various Professions and their respective Association are given below

Professions

Professional

Professional association

Medical profession

Doctor Medical Council of India

Law profession

Lawyers Bar Council of India

Accounting Profession

Chartered

The Institute of Chartered Accounts of India( ICAI)

Engineerin Engineers The

g Profession

institute of Engineers (India)

Accounts Basic accounting terms

Today we will give you some questions from the previous study material

Questions6) Define accounting7) What do you mean by debit

and credit8) Explain the types of account9) Define the following terms

a) Assetsb) Capitalc) Purchased) Debtorse) Transactions

10) Name the types of accounts given below

a) Krishnas accountb) Machinery accountc) Royalty accountd) Salary accounte) Furniture accountf) Audit fee account

Economics Basic Economic ConceptsSub topic

UTILITY

Before starting todayrsquos class we shall recall the last class which was about UTILITY AND THE FEATURES OF UTILITY

Now we shall proceed with the further topics of the chapter

Todayrsquos topic from the chapter lsquo Basic Economic Conceptsrsquo will be TOTAL UTILITY amp MARGINAL UTILITYNow let us quickly revise the concept of utility with an example ie goods and services are designed because they have an ability to satisfy human wantsThis feature of being able to satisfy human wants is termed as utility For example we derive utility from WiFi services as it gives us satisfaction by connecting us to our friends and family through social media here consumers derive utility from WiFi services

From the above concept we shall start with todayrsquos topicEconomists have defined TOTAL UTILITY (TU) as the total satisfaction obtained by consuming a given total amount of a good and serviceFor example the total satisfaction obtained from eating 10 mangoes is the total utility of 10 mangoes

MARGINAL UTILITY (MU) is the additional satisfaction derived from each additional unit

Questions1 What is Total Utility (TU)

Answer Total Utility (TU) is the

aggregate of the utility that a consumer derives from the consumption of a certain amount of a commodityTU=MU1+MU2++MUn

2 What is Marginal UtilityAnswer

Marginal Utility (MU) is the additional made to the total utility as consumption is increased by one more unit of the commodityMU= TUn ndashTUn-1

NoteOften economists tend to

subdivide utility into an imaginary unit called UTIL

consumed In this casethe utility obtained from each mango as it is consumed as the MU of that mango It is also defined as the addition made to the total utility when an additional unit is consumed Often economists tend to subdivide utility into an imaginary unit called UTIL

Note As a consumer increases the consumption of a good over period of time the total utility or total satisfaction derived from it increases to appoint and thereafter it decreasesHowever as the consumer keeps on consuming the good the marginal utility or the additional utility derived from it decreases

SubjectBusiness studies

Topic

BUSINESSENVIRONMENT

Summary

Now quickly let us revise the earlier points that we have already done in the last class and let us proceed with the other topics that are there in the chapter

Firstly we will recall the internal and external factors of micro environment and then we shall proceed in details

Meaning and list of internal and external factors

aInternal factorsInternal factors refer to all the factors existing within a business firm The internal factors are considered controllable because the enterprise has control over these factorsFor an example a company can alter its organization structure policies programmes employees physical facilities and marketing mix to suit the changes in the environmentList of internal factors areCorporate culture mission and objectives top management organizations structure company image and brand equity company resources

b External factorsExternal factors refer to those individual and groups and agencies with which a particular business organization comes into direct and frequent contact in the course of its functioningThese individuals and groups are known as STAKEHOLDERS because they have a stake (financial interest ) in the working and performance of the particular business List of external forces (stakeholders)Customers competitors investors suppliersmiddlemen (marketing intermediaries)

Execution 1 What do you mean by internal

factors in micro environmentAnswerInternal factors refer to all the factors existing within a business firm The internal factors are considered controllable because the enterprise has control over these factorsFor an example a company can alter its organization structure policies programmes employees physical facilities and marketing mix to suit the changes in the environment

2 What do you mean by external factors in micro environment

AnswerExternal factors refer to those individual and groups and agencies with which a particular business organization comes into direct and frequent contact in the course of its functioningThese individuals and groups are known as STAKEHOLDERS because they have a stake (financial interest) in the working and performance of the particular business

3Who are stakeholdersSTAKEHOLDERS are individuals and groups who have a stake (financial interest ) in the working and performance of the particular business 4Discuss the internal factors in briefa Corporate CultureThe values beliefs and attitudes of the founders and top management of the company exercise

financers publics

customers

suppliersfinancers

competitors

middlemen

publics

Fig STAKEHOLDERS OF A COMPANY

Apart from micro environment the other main dimension of business environment isMacro environment Macro environment refers to the general environment or remote environment within which a business firm and forces in its micro environment operateA company does not directly or regularly interact with the micro environmentTherefore macro environment is also known as indirect action EnvironmentThe macro environment forces are less controllable than the micro forces

Macro environment consists of the following components

POLITICAL AND LEGAL ENVIRONMENT

ECONOMIC SOCIAL AND ENVIRONMENT

CULTURAL

ENVIRONMENT

TECHNOLOGICAL ENVIRONMENT

a strong influence on what the cmpaany stands for how it does things and what it considers importantbMission and objectivesThe business philosophy and purpose of a comoany guide it prioritiesbusiness strategiesproduct market scope and development scope

cTop management structurethe composition of board of directors the degree of professionalization of management and the organizational structure of a company have important bearing on its business decisions

dPower structureThe internal power relationship between the board of directors and the chief executive is an important factor

eCompany image and brand equityThe image and brand equity of the company play a significant role in raising finance forming alliance choosing dealers and suppliers launching new products entering foreign markets

5 What is Macro environmentAnswerMacro environment refers to the general environment or remote environment within which a business firm and forces in its micro environment operateA company does not directly or regularly interact with the micro environmentTherefore macro environment is also known as indirect action EnvironmentThe macro environment forces are less controllable than the micro forces 6 What are the components of macro environmenta Political and legal environmentb Economic environmentc Social and cultural environmentd Technological environment

BUSINESS FIRM

Fig COMPONENTS OF MACRO ENVIRONMENTPolitical science

Introduction to political science

Comparative politics and itrsquos scope Comparative politics is the second major dimension of political scienceIt is also a very vast area of study and a very large number of political scientists even treat it as an autonomous area of study within the board ambit of political scienceScope of comparative politics-

1 All political structures -Comparative politics includes the study of all structures formalnon formal governmental and extra governmental which are directly or indirectly involved in politics in all the countries of the world

2 Functional studies- Comparative politics seeks to study politics less from the point of view of the legal institutions in terms of their powers and move from the point of view of their functions which constitute the political process and their actual Operation in the environment

3 Study of political behaviour- Another important part of its scope is the study of the actual behaviour of the people in the process of politics

4 Study of similarities and differences- comparative politics also undertakesan analysis of the similarities and differences among political process and functions

5 Study of all political systems -comparative politics seeks to analyse the actual behaviour and performance of all political systems western as well as non western

6 Study of the environment and infrastructure of politics-The study of politics demands a study of the psychological sociological economic and anthropological environment in fact the social environment as a whole in which each political system operates

7 Study of political culture- political culture is composed of attitudesbeliefs emotions and values of a society that relate to the political system or politics

8 Study of political participation- Political participation is a universal processThe only difference is that while in some states it is limited in others it is wider

9 Study of political process- political

Answer the following questions-

What is comparative politics

What are the scope of comparative politics

Homework- learn

processes like decision makingpolicy making judicial process leadership recruitment process and others are always at work in all political systems

The scope of comparative politics is very comprehensive It includes everything that falls within the area of political activity and political process

History CAMBRIDGE VIEW ABOUT

THE PARTITION

AND REFUTATION

OF CAMBRIDGE

VIEW

Cambridge view about the Partition The Cambridge school of historians have interpreted that opposition to partition scheme was made entirely by the elitist groups They hold the view that Lord Curzon planned to partition the Bengal for administrative purposeREFUTATION OFCAMBRIDGE VIEW The Rationalist historians have rejected the interpretations of the Cambridge School of historians on various grounds

1 QUESTION State different views of historians regarding Partition of Bengal

ANSWER Cambridge historians believed that Lord Curzon partitioned Bengal for administrative reasons only and not for the political motive The Middle class elitist group protested because of their petty interest The Hindu zamindars protested as they have to spend more money for managing their estatesThe lawyers of Calcutta High court feared to lose their clientBut according to the nationalist Historians was-

2- The ultimate object of Lord Curzon was to crush the unity of Bengal politicians

3- If Bengal becomes a separate province Bengali speaking 16 million people of western part would become minority under Hindi speaking people of Bihar and Oriya speaking people of Orissa

4- The bureaucrats expected that the protest movement would die down quickly

5- Lord Curzon used the Muslim community in his political game

6- Idealism had great contribution in the protest against partition

7- The people of the every section of society were affected by the partition of Bengal

Computer Science

Numbers Convertion of dcimal number to octal numberThe decimal numeral system is the standard system for denoting integer and non-integer numbers It is the extension to non-integer numbers of the Hindu-Arabic numeral system For writing numbers the decimal system uses ten decimal digits a decimal mark and for negative numbers a minus sign - The decimal digits are 0 1 2 3 4 5 6 7 8 9 the decimal separator is the dot in many countries

The octal numeral system or oct for short is the base-8 number system and uses the digits 0 to 7 Octal is sometimes used in computing instead of hexadecimal perhaps most often in modern times in conjunction with file

permissions under Unix systems It has the advantage of not requiring any extra symbols as digits It is also used for digital displays

Follow these steps to convert a decimal number into octal form

1 Divide the decimal number by 82 Get the integer quotient for the next iteration (if the number will not divide equally by 8 then round down the

result to the nearest whole number)3 Keep a note of the remainder it should be between 0 and 74 Repeat the steps until the quotient is equal to 05 Write out all the remainders from bottom to top This is the solution

For example if the given decimal number is 8453

Division Quotient Remainder

8453 8 1056 5

1056 8 132 0

132 8 16 4

16 8 2 0

2 8 0 2

Then the octal solution is 20405

Subject Eng Literature (The Tempest ndash William Shakespeare) Topic Act I Scene 1 Lines 33 to 67 (End of scene) Date 16th April 2020 (4th Period)

[Students should read the original play and also the paraphrase given in the school prescribed textbook]Summary Questions amp Answers

[SUMMARY OF THE ENTIRE SCENE]

o The play starts with the scene of a severe storm at sea Alonso (King of Naples) Sebastian (Alonsorsquos brother) Ferdinand (Alonsorsquos son) Gonzalo Antonio (the usurping Duke of Milan) are in a ship in the midst of the storm

o The mariners are trying their best to control the vessel from running aground and are totally following the orders of their Master the Boatswain They have scant success

o The mariners become extremely unhappy and annoyed when most of the passengers arrive on the deck thereby hampering their effort to save the ship There is serious confrontation between them and the passengers who are part of the Kingrsquos entourage

o The mariners could not save the ship

SUMMING-UP

(i) Vivid description of the scene which gives a realistic description of terror and confusion of a tropical storm

(ii) Shows Shakespearersquos accuracy of knowledge in describing the naval operations and also matters of seamanship

(1) GONZALO Ill warrant him for drowning (L 45-57)

though the ship were no stronger than a nutshell and as leaky as an unstanched

wenchBOATSWAIN Lay her a-hold a-hold Set her two courses Off to

sea again lay her offMARINERS All lost To prayers to prayers All lostBOATSWAIN What must our mouths be coldGONZALO The king and prince at prayers Lets assist them

For our case is theirsSEBASTIAN Im out of patienceANTONIO We are merely cheated of our lives by drunkards

This wide-chopped rascal - would thou mightst lie drowning the washing of ten tides

(a) What does Antonio say at the insolent manners of the boatswain just before the given passage

Being irritated at the insolent manners of the boatswain just before the given extract Antonio the Duke of Milan calls him a worthless dog son of a woman without any morals an arrogant and disrespectful noisemaker He says that the boatswain deserved to be hanged(b) What statement does Gonzalo repeat about the boatswain

Gonzalo shows his faith that the boatswain is not destined to die by drowning He is destined to be hanged and nothing can alter this decree of destiny He says that even if the ship was as frail as a nutshell the boatswain could not be drowned for his destiny was to be hanged(c) What do the passengers do when they have lost all hope of their survival

When the passengers have lost all hope of survival they take

(iii) The opening scene justifies the title ndash The Tempest

UNANSWERED QUESTIONS

(i) The King always travels with his entire fleet including his soldiers Where were the other ships

(ii) Why was the ship in that area Where was it coming from or going where

(iii) The ship broke apart What happened to those who were in the ship

(We shall get the answer to the above questions as the play progresses)

leave of life with fervent prayers The mariners take their last hearty drink and are ready for death(d) What blame does Antonio put upon the mariners and the boatswain Antonio rebukes the mariners that these drunkards have brought them to the present crisis by neglecting their duties He blames them saying that they are going to lose their lives entirely for the negligence of the boatswain and his fellows(e) What does Antonio say while cursing the boatswain

Antonio gives vent to his wrath upon the boatswain in particular He calls the boatswain a wide-mouthed rascal who deserves to be hanged on the sea-shore at low water mark so that ten tides might wash over his body and take out of him all the liquor that he has been drinking

Class XIISubject Topic Summary ExecutionHistory Topic

1 1935 ACT AND WORKING OF PROVINCIAL AUTONOMYCONGREE AND OTHER MINISTERSSUB TOPIC GOVERNMENT OF INDIA ACT1935

Government of India Act 1935 This act established a lsquoFederation of Indiarsquo made of British Indian provinces and Indian states and provided for autonomy with a government responsible to the elected legislature in every provinceThis act introduced abolition of Diarchy at provinces The entire provincial administration was introduced to the responsible ministers who were controlled and removed by the provincial legislature The provincial autonomy means two things First The provincial governments were wholly responsible to the provincial legislature Secondly Provinces were free from outside control and interference in the large number of matters The act divided the powers between the centre and provinces in terms of three lists- Federal list( for centre) Provincial list (for province) and concurrent list (for both) Residuary powers were given to the viceroy In the election under the government of India Act the Congress swept the poll the mandate of the people came in favour of the congress so far as general Hindu seats were concerned The Congress did not get a single Muslim seates in Bombay CP UP Sind and BengalIn five provinces Congress had yhe clear majority In BengalNWFPAssam and Bombay Congress emerged as a single largest partyOn the other side the performance of the Muslim League was badThus the Congress formed ministers in 7 provinces out of 11 provinces Coalition ministry was also formed in two other provincesOnly BENGAL AND Punjab had non- congress ministries

1 QUESTION What was the main change introduced by the Government of India ActANSWER a) The Act gave more

autonomy to the provinces b) Diarchy was abolished at the

provincial levelsc) The Governor was the head of

the executived) There was a council of

ministers to advise him The ministers were responsible to the provincial legislatures who controlled them The legislature could also remove the ministers

e) The Governors still retained special reserve powers

2 QUESTION Why did the federal scheme introduced by the Government of India Act 1935 never come into operation

ANSWER The Federal structure of the Government of India was to be composed with the Governor General and Council of ministers The Federal legislature was to be Bicameral legislature- The council of states and the House of Assembly The ministers were to be chosen by the Governor general and they were to hold the office during his pleasure

The provinces of British India would have to join the federation but this was not compulsory for the princely states

This federation never materialised because of the lack of support from the required number of

princely statesThis act was refused and

rejected by the princes the Congress and the Muslim League

Thus both Congress and the League participated in the election of 1937 Thus the federal part was never introduced but the provincial part was put into operations

Bengali 2nd

Language

াচেরর পরাথCনা(কহিতা )

াচেরর পরাথCনা কহিতাটি কহি (ঙখ দেঘাচে4র দো আচো য কহিতায় াচেরর পতর হমায়ন কঠিন দেরাচেগ আxানত ার ঈশবর া আললার কাচেছ পরাথCনা কচেরচেছন তার পচেতরর ীন হিফহিরচেয় হিচেত এই কহিতায় ার পচেতরর ীন হিভbা দেচেয়চেছন ারার এমনহিক হিনচের ীন হিসCচেনর হিহিনমচেয় হিতহিন তার দেছচের ীন হিফচের দেপচেত দেচেয়চেছন তার দেছচের এই দেরাচেগর ন য হিতহিন হিনচেচেকই ায়ী কচেরচেছন তার হিনচের করা পাপচেকই হিতহিন ায়ী কচেরচেছন এছাা রানৈনহিতক ও আথCসামাহিক অসথার কথা তচে ধরা চেয়চেছ এই কহিতায় ার তার হিনচের পাপ কমCচেকই ায়ী কচেরচেছ ার অন যায় ভাচে দেপহি((হিকতর মাধ যচেম অপররা য কচেরচেছ আর এই অন যায় কাচের ন যই তার পহিরাচের হিপযCয় এচেসচেছ দে এক পরকার মানহিক নধন ইহিতাচেসর ার হিপতা চেয় সবাভাহিকভাচে ভাচোাসা দে মমতা দেথচেক মকত চেত পাচেরনহিন তাই হিপতা চেয় আললা া ভগাচেনর কাচেছ পতর হমায়চেনর পরানহিভbা দেচেয়চেছন ার আললা া ভগাচেনর কাচেছ াহিনচেয়চেছন তার হিনচের ীন হিসCন হিচেত হিতহিন রাী তার হিহিনমচেয় পচেতরর ীন হিফচের দেপচেত দেচেয়চেছন াচেরর হিপতসভ হিচেকর কথা এই কহিতায় ফটিচেয় দেতাা চেয়চেছ হিপতা পচেতরর হিরাহিরত মান নধচেনর কথা তচে ধরা চেয়চেছ

হিচে(4 হিকছ াইচেনর তাৎপযC১) ldquoদেকাথায় দেগ ওর সবচছয দেৌন দেকাথায় কচেরায় দেগাপন bয়ldquoউততর) াচেরর পতর হমায়ন কঠিন দেরাচেগ অসসথ তাই তার দেযৌন াহিরচেয় যাচেচছ এই দেরাচেগ তাচেক দেগাপচেন কচেরকচের াচেচছ তার সক (হিকত ধীচের ধীচের bয় চেচছ তাই হিপতা চেয় ার আললার কাচেছ হমায়চেনর পরান হিভbা দেচেয়চেছন২) ldquoাগাও (চেরর পরাচেনত পরানতচের ধসর (ন দেযর আান গানldquoউততর) াচেরর পতর হমায়ন কঠিন দেরাচেগ আxানত তাই ার আ দে(াচেক মমCাত (চেরর পচেথ পরানতচের আান গান ধবহিনত দোক দেসই আান গান আললার কাচেছ দেযন চে যায় আললা দেযন এই আহিতC শচেন পচেতরর ীন হিফহিরচেয় দেয় ৩)ldquoনাহিক এই (রীচেরর পাচেপর ীানচেত দেকানই তরারণ দেনই ভহি4চেতরldquoউততর) হমায়চেনর অসসথতার ন য ার হিনচেচেকই ায়ী কচেরচেছন কারন ার অচেনক রা য অন যায় ভাচে কচেরচেছ তাই তার এই পাপ কাচের ন য তার ঘচের আ হিপ এচেসচেছ এই অন যায় কাচের ন য তার মহিকত দেনই তাই ার আললার কাচেছ এই পাপ কাচেযCর ন য bমা পরাথM

Hindi 2ndlang

-ासी(जयशकर परसा-)

-ासी जयशकर परसा- की एक ऐसी कहानी ह जिजसम भारतीय ससकनित और राषटरीयता का सवरगजीतहोता ह इस कहानी म इरावती एक निहद कनया ह जिजस मलअचछो न मलतान की लट म पकडा और -ासी बना दि-या उस 500 दि-न -कर काशी क एक महाजन न खरी-ा दसरी -ासी निफरोजा ह वह गलाम ह निफरोजा को छडान क कतिलए अहम- को 1000 सोन क कतिसकक भजन थ जो अभी तक नही आए थ राजा साहब कठोर होत हए भी निफरोजा को निबना धनराकतिश क कतिलए उस म कर -त ह वनिफरोजा को अहम- को समझान की बात कहत हकहानी क अत म हम -खत ह निक इरा वती और जाटो क सर-ार बलराज का मिमलन होता हअहम- को यa म मार दि-या जाता ह वहा निफरोजा की परसननता की समामिध बनती ह वहा एक फल चढती ह और डीजल आती ह निफरोजा उस समामिध की आजीवन -ासी बनी रहती हलखक अपन उददशय अथात -ास परथा पर परकाश डालन और इस परथा क कारण होन वाल -ातो क दखो को दि-खान म पणता सफल हए ह

helliphellipContinue to next

Biology Reproductio Today we will discuss about vegetative Q1 Name some vegetative propagules

n in Organisms

propagation of plants The process of multiplication in which fragments of plant body function as propagule and develop into new individual is called vegetative propagation The units of such propagation are runner rhizome tuber bulb etc

and the speciesinvolvedVegetative propagules

Parts involved

Bulb StemBulbil BulbilRhizome Stem Runner Stem Tuber Stem Offset Stem Leaf buds Leaves Suckers Stem

Corns Stem stolon

Q2 State advantages of vegetative propagation

i) Rapid methodii) Sure and easy methodiii) Useful in plants that cannot

produce viable seeds or long seed dormancy

iv) Maintains purity of raceQ 3 Banana fruit is said to be parthenocarpic where as turkey is said to be parthenogenetic WhyBanana develops without fertilization from an unfertilized ovary thus is parthenocarpicIn turkey the ovum or female gamete developinto a new chick without fertilization thus isparthgenetic

Q4 Why is water hyacinth is called as a ldquoTerror of Bengalrdquo Water hyacinth can

propagatevegetatively all over the water body in a short per short period of time This resulted increased biochemicaloxygen oxygen demand of water body causing mortalityof fishes It is very difficult to get rid off them Thus known as terror of Bengal

Chemistry

Solid state GENERAL CHARACTERISTICS OF SOLID STATEIn nature the particular state of matter is governed by two opposing forces at given set of temperature and pressure These forces are intermolecular force of attraction and thermal energy If intermolecular force of attraction is high as compared to thermal energy particles remains in closest position

Intext QuestionsQ1 Classify the following solids as crystalline and amorphous Sodium chloride quartz glass quartz rubber polyvinyl chloride Teflon

A1 Crystalline

and hence very less movement in particles is observed In this case solid state is the preferred state of matter

Let us revise the general characteristics of solid

i) Fixed mass volume and shape

ii) Strong intermolecular force of attraction

iii) Least intermolecular space

iv) Fixed position of constituent particles

v) Incompressible and rigid

Q2 what type of interactions hold the molecules together in a polar molecular solid[CBSE 2010]A2 The molecules in a solid are held together by van der Waals forces The term van der Waals forces include hydrogen bonding dipole-dipole attraction and London dispersion forces All molecules experience London dispersion forces In addition polar molecules can also experience dipole-dipole interactions So the interactions that holds the molecule together in polar molecular solid are London dispersion force and dipole-dipole interactionsQ3 Write a feature that will distinguish a metallic solid from an ionic solid [CBSE 2010]A3 Metals are malleable and ductile whereas ionic solid are hard and brittle Metallic solid has typical metallic lustre But ionic solid looks dullQ4 Write a point of distinction between a metallic solid and an ionic solid other than metallic lustre [CBSE 2012]A4 Metals are malleable and ductile whereas ionic solid are hard and brittleQ5 Write a distinguish feature of metallic solid [CBSE 2010]A5 The force of attraction in

solid Sodium chloride Quartz Amorphous solid Quartz glass rubber polyvinyl chloride Teflon Q2 why glass is considered as super cooled liquidA2 Glass shows the tendency to flow at slower rate like liquid Hence they considered as super cooled liquidQ3 why the window glass of old buildings show milky appearance with timeA3 Glass is an amorphous solid Amorphous solid has the tendency to develop some crystalline character on heating Due to heating in day over the number of years glass acquires some crystalline character and show milky appearanceQ4 why the glass panes fixed to window or doors of old building become slightly thicker at bottomA4 Glass is super cooled liquid It has the tendency to flow down very slowly Due to this glass pane becomes thicker at the bottom over the timeQ5 Sodium chloride is a crystalline solid It shows the same value of refractive index along all the direction TrueFalse Give reasonA5 FalseCrystalline solid shows anisotropy in properties That is it shows different values for the given physical property in different direction All the crystalline solids show anisotropy in refractive index Therefore sodium chloride will show different values of refractive index on different directions

Q6 Crystalline solid are anisotropic in nature What does this statement means

between the constituent particles is special kind of electrostatic attraction That is the attraction of positively charged kernel with sea of delocalized electronsQ6 which group of solid is electrical conductor as well as malleable and ductile [CBSE 2013]A6 Metallic solidQ7 why graphite is good conductor of electricity although it is a network (covalent solid)A7 The exceptional property of graphite is due to its typical structure In graphite each carbon is covalently bonded with 3 atoms in same layer The fourth valence electron of each atom is free to move in between different layersThis free electron makes the graphite a good conductor of electricity

[CBSE 2011]A6 Anisotropy is defined asrdquo Difference in properties when measured along different axis or from different directionsrdquo Crystalline solid show different values of some of the physical properties like electrical resistance refractive index etcwhen measured along the different directions The anisotropy in crystalline solid arises due to the different arrangement of particles in different directions

Math Function Composition of functions Think of an industrial plant that produce bottles of cold drinks first there is the operation (or function) f that puts the cold drink inside the bottle followed by the opeartion g that close the bottle with the capThis leads to the following definitionDefinition Let f A rarr B and g B rarr C be two functions Then the composition of f and g denoted by gof is defined as the function gof A rarr C given by gof(x) = g(f (x)) forall x isinA

Definition A function f X rarr Y is defined to be invertible if there exists a function g Y rarr X such that gof = IX and fog = IY The function g is called the inverse of f and is denoted by f -1

Thus if f is invertible then f must be one-one and onto and conversely if f is one-one and onto then f must be invertible This fact significantly helps for proving a function f to be invertible by showing that f is one-one and onto specially when the actual inverse of f is not to be determined

Example 1 Let f 2 3 4 5 rarr 3 4 5 9 and g 3 4 5 9 rarr 7 11 15 be functions defined as f(2) = 3 f(3) = 4 f(4) = f(5) = 5 and g (3) = g (4) = 7 and g (5) = g (9) = 11 Find gofSolution We have gof(2) = g (f(2)) = g (3) = 7 gof(3) = g (f(3)) = g (4) = 7gof(4) = g (f(4)) = g (5) = 11 and gof(5) = g (5) = 11Example 2 Find gof and fog if f R rarr R and g R rarr R are given by f(x) = cos x and g (x) = 3x2 Show that gof ne fogSolution We have gof(x) = g(f(x))=g(cosx) = 3 (cos x)2

= 3 cos2 x Similarly fog(x)=f(g (x))= f(3x2)= cos (3x2) Note that 3cos2 x ne cos 3x2 for x = 0 Hence gof ne fogExample 3 Show that if f A rarr B and g B rarr C are onto then gof A rarr C is also ontoSolution Given an arbitrary element z isin C there exists a pre-image y of z under g such that g (y) = z since g is onto Further for y isin B there exists an element x in A with f(x) = y since f is onto Therefore gof(x) = g (f(x)) = g (y) = z showing that gof is onto Example 4 Let Y = n2 n isin N sub N Consider f N rarr Y as f(n) = n2 Show that

f is invertible Find the inverse of fSolution An arbitrary element y in Y is of the form n2 for some n isin N This implies that n =radicy This gives a function g Y rarr N defined by g (y) =radicy Nowgof (n) = g (n2)=radicn2 = n and fog (y) =f(radicy) = (radicy) 2 y which shows that gof=IN and fog= IY Hence f is invertible with f -1 = g

Political Science

Constitution of India-The Preamble

Summary

Objective of the state-To secure equality of status and of opportunity To promote fraternity among all the citizens To assure the dignity of the individuals and Unity and integrity of the nation

Justice-Justice stands for rule of law absence of arbitrariness and a system of equal rights freedom and opportunities for all in a society India seeks social economic and political justice to ensure equality to its citizens

Liberty-Liberty implies the absence of restraints or domination on the activities of an individual such as freedom from slavery serfdom imprisonment despotism etc The Preamble provides for the liberty of thought expression belief faith and worship

Equality-Equality means the absence of privileges or discrimination against any section of the society The Preamble provides for equality of status and opportunity to all the people of the country

Fraternity-The Preamble declares that fraternity has to assure two thingsmdashthe dignity of the individual and the unity and

Execution

Answer the following questions-

Short notes-1 Equality2 Fraternity3 Justice4 Liberty

Homework-Learn

integrity of the nation The word integrity has been added to the Preamble by the 42nd Constitutional Amendment (1976)

Business studies

Human resource management (chapter 1)

On the day of 1504 2020 I have discussed with you the managerial functions and procurement functions of HRM

Today weare going to discuss about the development function integration functions and maintenance function

Development functions-HRM improves the knowledge skills attitude and values of employees so that they the present and future jobs more effectively it includes

1) Development functions of HRM

a) Performance appraisal = It implies systematic evaluation of employees with respect to their performance on the job and their potential for development

b) Training =It is the process by which employees learn knowledge skills and attitudes to achieve organisational and personal goals

c) Executive development = It is the process of developing managerial talent through appropriate program

2) Integration functionsa) HRM reconcile the goals of

organisation with those of its members through integrating function

b) HRM tries to motivate employees to various financial and non financial incentives provided in job specification etc

3) Maintenance functiona) HRM promote and protect the

physical and mental health of employees by providing several types of benefits like housing medical aid etc

b) It Promote Social security measures to employees by providing provident fund pension gratuity maternity benefits

SubjectCOMMERCE

Topic

BUSINESSENVIRONMENT

Summary

Now quickly let us revise the earlier points that we have already done in the last class and let us proceed with the other topics that are there in the chapter

Firstly we will recall the internal and external factors of micro environment and then we

Execution 3 What do you mean by internal factors

in micro environmentAnswerInternal factors refer to all the factors existing within a business firm The internal factors are considered controllable because the enterprise has control over these factors

Development FunctionsPerformance AppraisalTrainingExecution Development

shall proceed in details

Meaning and list of internal and external factors

aInternal factorsInternal factors refer to all the factors existing within a business firm The internal factors are considered controllable because the enterprise has control over these factorsFor an example a company can alter its organization structure policies programmes employees physical facilities and marketing mix to suit the changes in the environmentList of internal factors areCorporate culture mission and objectives top management organizations structure company image and brand equity company resources

b External factorsExternal factors refer to those individual and groups and agencies with which a particular business organization comes into direct and frequent contact in the course of its functioningThese individuals and groups are known as STAKEHOLDERS because they have a stake (financial interest ) in the working and performance of the particular business List of external forces (stakeholders)Customers competitors investors suppliersmiddlemen (marketing intermediaries)financers publics

customers

suppliersfinancers

For an example a company can alter its organization structure policies programmes employees physical facilities and marketing mix to suit the changes in the environment

4 What do you mean by external factors in micro environment

AnswerExternal factors refer to those individual and groups and agencies with which a particular business organization comes into direct and frequent contact in the course of its functioningThese individuals and groups are known as STAKEHOLDERS because they have a stake (financial interest) in the working and performance of the particular business

3Who are stakeholdersSTAKEHOLDERS are individuals and groups who have a stake (financial interest ) in the working and performance of the particular business 4Discuss the internal factors in briefa Corporate CultureThe values beliefs and attitudes of the founders and top management of the company exercise a strong influence on what the cmpaany stands for how it does things and what it considers importantbMission and objectivesThe business philosophy and purpose of a comoany guide it prioritiesbusiness strategiesproduct market scope and development scope

cTop management structurethe composition of board of directors the degree of professionalization of management and the organizational structure of a company have important bearing on its business decisions

dPower structureThe internal power relationship between the board of directors and the chief executive is an important factor

e Company image and brand equityThe image and brand equity of the company play a significant role in raising finance forming alliance choosing dealers and suppliers launching new products entering foreign markets

5 What is Macro environmentAnswerMacro environment refers to the general

competitors

middlemen

publics

Fig STAKEHOLDERS OF A COMPANY

Apart from micro environment the other main dimension of business environment isMacro environment Macro environment refers to the general environment or remote environment within which a business firm and forces in its micro environment operateA company does not directly or regularly interact with the micro environmentTherefore macro environment is also known as indirect action EnvironmentThe macro environment forces are less controllable than the micro forces

Macro environment consists of the following components

POLITICAL AND LEGAL ENVIRONMENT

ECONOMIC SOCIAL AND ENVIRONMENT

CULTURAL

ENVIRONMENT

TECHNOLOGICAL ENVIRONMENT

Fig COMPONENTS OF MACRO ENVIRONMENT

environment or remote environment within which a business firm and forces in its micro environment operateA company does not directly or regularly interact with the micro environmentTherefore macro environment is also known as indirect action EnvironmentThe macro environment forces are less controllable than the micro forces 6 What are the components of macro environmenta Political and legal environmentb Economic environmentc Social and cultural environmentd Technological environment

Computer Science

Logic gates

Digital systems are said to be constructed by using logic gates These gates are the AND OR NOT NAND NOR EXOR and EXNOR

BUSINESS FIRM

gates The basic operations are described below with the aid of truth tables

AND gate

The AND gate is an electronic circuit that gives a high output (1) only if all its inputs are high A dot () is used to show the AND operation ie AB Bear in mind that this dot is sometimes omitted ie ABOR gate

The OR gate is an electronic circuit that gives a high output (1) if one or more of its inputs are high A plus (+) is used to show the OR operationNOT gate

The NOT gate is an electronic circuit that produces an inverted version of the input at its output It is also known as an inverter If the input variable is A the inverted output is known as NOT A This is also shown as A or A with a bar over the top as shown at the outputs The diagrams below show two ways that the NAND logic gate can be configured to produce a NOT gate It can also be done using NOR logic gates in the same way

NAND gate

This is a NOT-AND gate which is equal to an AND gate followed by a NOT gate The outputs of all NAND gates are high if any of the inputs are low The symbol is an AND gate with a small circle on the output The small circle represents inversion

NOR gate

This is a NOT-OR gate which is equal to an OR gate followed by a NOT gate The outputs of all NOR gates are low if any of the inputs are highThe symbol is an OR gate with a small circle on the output The small circle represents inversion

EXOR gate

The Exclusive-OR gate is a circuit which will give a high output if either but not both of its two inputs are high An encircled plus sign ( ) is used to show the EOR operation

EXNOR gate

The Exclusive-NOR gate circuit does the opposite to the EOR gate It will give a low output if either but not both of its two inputs are high The symbol is an EXOR gate with a small circle on the output The small circle represents inversion The NAND and NOR gates are called universal functions since with either one the AND and OR functions and NOT can be generated

Note A function in sum of products form can be implemented using NAND gates by replacing all AND and OR gates by NAND gates A function in product of sums form can be implemented using NOR gates by replacing all AND and OR gates by NOR gates

Logic gate symbols

Table 2 is a summary truth table of the inputoutput combinations for the NOT gate together with all possible inputoutput combinations for the other gate functions Also note that a truth table with n inputs has 2n rows You can compare the outputs of different gates

Logic gates representation using the Truth table

Example

A NAND gate can be used as a NOT gate using either of the following wiring configurations

Subject Eng Literature (The Tempest ndash William Shakespeare) Topic Act III Scene 3 Lines 53 to 110 (End of the scene) Date 16th April 2020 (2nd Period)

[Students should read the original play and also the paraphrase given in the school prescribed textbook]Summary Questions amp Answers

o Seeing this strange scene all are inclined to believe the tales told by travelers that there truly are ldquounicornsrdquo and ldquothe phoenixrsquo thronerdquo

o As they are about to sit down to the feast the banquet is snatched away by a harpy (Ariel disguised) A spiritrsquos voice (Arielrsquos voice) denounces Alonso Sebastian and Antonio with particular

1 ARIEL You are three men of sin whom Destiny

(Line 53-58)That hath to instrument this

lower world And what is int the never-surfeited sea

Hath caused to belch up you and on this island

Where man doth not inhabit you rsquomongst men

Being most unfit to live I have made you mad

reference to their crime in expelling Prospero from Milan They have not received any punishment for their deed earlier but the time for their punishment has arrived Upon Alonso it pronounces ldquolingering perdition worse than deathrdquo from which there is no remedy except through sincere repentance Ariel then vanishes in thunder and the shapes enter again and carry away the table

o Prospero watching invisibly is very pleased with the performance of Ariel and his (Prosperorsquos) ldquomeaner ministersrdquo All his enemies are now in his power and are in a fit of desperation He then leaves them and goes to see how Ferdinand and Miranda are getting on

o Alonso is now much humbled and penitent with the after effect of the spiritrsquos denunciation of his crimes He believes that his son is lost forever After this all disperse being stricken mad by the speech of the spirit

o Gonzalo fearing that they may do violence to themselves or to one another follows them and bid others to follow

(a) To whom does Ariel disguised as a harpy call the three sinners What game did Fate of Destiny play with

them

The three sinners called by Ariel are Alonso Sebastian and Antonio It was Destiny which had caused the ocean to cast the three sinners on the shore Though the ocean is all the time devouring whatever appears on its surface and is never satisfied with its continual swallowing of the ships and men in the present case the ocean had cast these three sinners on the shore without killing them

(b) Who had jointly been responsible for the conspiracy against Prospero What is Prosperorsquos purpose behind all this

Three men Alonso Sebastian and Antonio had jointly

been responsible for the conspiracy against Prospero They had driven out Prospero form Milan Prosperorsquos purpose is to make these three sinners realize the wrong they had done He wants them to repent for their criminal deeds because repentance leads to self-esteem(c )What does Ariel (the harpy) tell Alonso and his companions when they take out their swords to attack him

Seeing them drawing their swords Ariel (harpy) tells them that he and his companions are the instruments of destiny and that it is not possible for human beings to do them any injury He says that the swords of human beings can not injure even a minute part of his feathers Their swords are as ineffective against him and his companions as against the wind or the water

(d) Give the explanatory meanings of the following expressions in the context of the above extract

(i)Never surfeited (ii) Belch up (iii) lsquomongst men

(i) Never surfeited never led to satisfaction

(ii) Belch up cast ashore(iii) lsquomongst men in human

society2

I and my fellows (Line 60-65)

Are ministers of Fate The elementsOf whom your swords are tempered may as wellWound the loud winds or with bemocked-at stabsKill the still-closing waters as diminishOne dowl thats in my plume

IMPORTANT PASSAGES EXPLAINED

The elements

(Line 61-66)Of whom your swords are tempered may

as wellWound the loud winds or with

bemocked-at stabs

(a) Who is lsquoIrsquo Who are his lsquofellowsrdquo

lsquoIrsquo is referred to Ariel in disguise of a harpy His lsquofellowsrsquo are other spirits serving Prospero the real Duke of Milan who has acquired supernatural powers after being banished from his Dukedom Prospero has settled in this uninhabited island

(b) What are the elements that have temperrsquod the swords Why will it not work against the speaker

The swords (of Alonso and his companions) are tempered by metal (steel) which is taken out of the earth and refined by

Kill the still-closing waters as diminishOne dowl thats in my plume My fellow

ministersAre like invulnerable

In these words Ariel reminds the King and his companions of the utter futility of drawing swords against himself and his fellows Ariel drives Alonso Antonio and Sebastian the three men of sin to desperation ndash a state in which men do violence to themselves They draw swords to strike Ariel But Ariel reminds them that he and the other spirits are the ministers of destiny and nothing can wound them The steel of which their swords are made of may cut the wind or water which being divided always closes up again Even supposing that such things may be possible it is quite impossible that their swords will cut one feather in their plume They are incapable of being wounded by any sword of man Hence it is foolish on their part to attempt to strike at Ariel and his fellow-spirits

For which foul deed

(Line 72-75)The powers delaying not forgetting

haveIncensed the seas and shores yea all the

creatures Against your peace

Ariel enters like a harpy and remaining invisible tells Alonso Sebastian and Antonio that he and other harpies are the agents of Destiny appointed to carry out her decrees He tells them that their punishment for the crime against Prospero which has been so long deferred is now to fall upon them He reminds them that they had expelled Prospero from Milan and set him and his innocent child adrift on the sea and that the sea had paid them back for their sin by the shipwreck and by the calamities they have suffered He tells them that the powers above which did not forget this mean treachery but only deferred the punishment have now engaged the seas and the shores and all living beings including him and his comrades against them The very elements and supernatural agency Ariel adds have taken up the avenging of their crime against Prospero

the action of fire It may cut the wind or water which being divided always closes up again

The sword will not work against the spirits and the harpy because they are the ministers of destiny and nothing can wound them nor it will cut a single feather in their plume

(c )What is the meaning of lsquodowlrsquo in the last line

The term lsquodowlrsquo means a filament or the smallest part of a feather In this context Ariel in disguise of harpy says that their sword cannot even damage the smallest filament of their (Arielrsquos and other spirits) feathers as they are incapable of being wounded by any sword of man

(d) What does the speaker remind the listeners about

Ariel in disguise of harpy reminds Alonso the King of Naples Sebastian Alonsorsquos brother and Antonio the present Duke of Milan and the treacherous brother of Prospero as they being three men of sin He even reminds them that their punishment for their crime against Prospero which has been so long deferred now falls upon them He reminds them that they have expelled Prospero from Milan and has set him along with his innocent infant daughter adrift on the sea So the sea has paid them back for their sin by their shipwreck and the calamities they have suffered since then The harpy rebukes Alonso of his sin that has incensed the Gods and has deprived him of his son as a punishment

(e) How do they respond

When Ariel in disguise of a harpy reminds Alonso Sebastian and Antonio of their past misdeeds and sin Alonso has a look of terror and confusion in his eyes He utters the words of sincere repentance wrung out of his conscience-stricken heart It appears to him that all the elements of nature the sea-waves the wind and the thunder proclaiming a loud voice in the name of Prospero and the crime Alonso has committed against him They are calling upon him to repent There is a deep storm raging in Alonsorsquos breast and the echoes of that storm are ringing in his ears like a clear note of wind-instrument A note of denunciation of Alonsorsquos crime leaves him much humbled and penitent and confirms his belief that his son is lost forever But Sebastian and Antonio shows some courage instead of repentance They wish to kill the spirits or devils if it appears

3

Of my instruction hast thou nothing bated (Line 85-93)

In what thou hast to say So with good life

And observation strange my meaner ministers

Their several kinds have done My high charms work

And these mine enemies are all knit upIn their distractions They now are in my

powerAnd in these fits I leave them while I visitYoung Ferdinand whom they suppose is

drownedAnd his and mine loved darling

Methought the billows spoke and (Line 96-99)

told me of itThe winds did sing it to me and the

thunderThat deep and dreadful organ-pipe

pronouncedThe name of Prosper It did bass my

trespass

These are the words of contrition coming from Alonso Ariel has driven him to a deep repentance for conspiring with Antonio against Prospero He now feels a sincere remorse It appears to him that all the elements of nature the sea-waves the wind and the thunder proclaimed with a loud voice the name of Prospero and the crime Alonso had committed against him They are calling upon him to repent There is a deep storm raging in Alonsorsquos breast and the echoes of that storm are ringing in his ears like the clear note of a wind-instrument

Comment These are the words of sincere repentance wrung out of the conscience-stricken heart of Alonso Alonso who is the lesser villain is the first to give way to remorse under the effect of Arielrsquos speech The words of Ariel seem to him to be the voice of conscience speaking to him He is driven to desperation a state in which he might do violence to his life

(a) Identify the speaker State the context

Prospero the ruler of the island is the speaker The famous banquet scene has been enacted very well Ariel and his junior spirits have played their roles excellently Prospero is glad to say words of praise for them(b) In what way the speakerrsquos instructions have been carried out

According to Prosperorsquos instructions a banquet was presented before the King of Naples and his companions when they were tired and hungry Just when they were preparing to eat the feast the banquet was suddenly removed by exercising supernatural powers All this was done by Ariel Prosperorsquos chief assistant and a powerful spirit

Ariel not only made the feast disappear but also delivered his speech blaming the King and his two companions for their past wicked deeds He warned them to repent for their misdeeds or suffer forever on that uninhabited island

(c) Who are referred to as lsquomeaner ministersrsquo What have they done

Prospero refers as lsquomeaner ministersrsquo to his other lesser spirits who were assisting Ariel in presenting a scene before the kingrsquos party They entered the scene to the accompaniment of music They assumed several strange shapes and brought in a banquet Then they danced about it with gentle actions of salutations thus inviting the King and others to eat the feast

These spirits play their role again when Ariel in the shape of a harpy quits the scene These shapes enter again and dancing with mocking gestures carry away the table

(d) Who are the speakerrsquos enemies What has happened to them

King of Naples Alonso his brother Sebastian and the present Duke of Milan Antonio (Prosperorsquos own brother) are Prosperorsquos enemies With the turn of events they have all been washed ashore on the island which is ruled by Prospero the great magician Actually this happened after the shipwreck caused by a storm which was raised by Prospero with the purpose of bringing these people to his island Prosperorsquos spirits have already confused and terrified these enemies and they are under Prosperorsquos control He can treat them as he likes

(e) What does he say about Ferdinand Explain what is meant by ldquohellip his and mine darlingrdquo

Prospero knows that Alonsorsquos son prince Ferdinand is alive though his father thinks that the prince has been drowned

Prospero refers to his daughter Miranda who is dear to him She is also very dear to Prince Ferdinand who has fallen in love with her They are waiting to be married soon for which they have received Prosperorsquos consent

4

ALONSO O it is monstrous monstrous (Line 95-102)

Methought the billows spoke and told me of it

The winds did sing it to me and the thunderThat deep and dreadful organ-

pipe pronouncedThe name of Prosper It did bass

my trespassTherefore my son ithrsquo ooze is

bedded andIll seek him deeper than eer

plummet soundedAnd with him there lie mudded

(a) In what way does Alonso express his horror when his conscience is awakened by Arielrsquos words

When Alonsorsquos conscience is awakened by Arielrsquos words he expresses his horror at what he has heard He gets the feeling that the waves of the ocean the wind and the loud thunder have spoken to him and uttered the name of Prospero Because of being reminded of his crime in a very loud and rough voice he comes to realize that he has lost his son for his past misdeeds

(b) What does Alonso imagine about his son What does Alonso want to do in his desperate state

Alonso imagines that his son is lying in the mud at the bottom of the sea He feels desperate that he wants to drown himself in the ocean deeper than the plumb-line has ever gone He wants to lie with his son at the bottom of the sea

(c) How do Sebastian and Antonio want to face the evil spirits

Sebastian says that he is not at all afraid of what the harpy has said and that he is prepared to fight any number of such monsters if they appear before him only one at a time Antonio says that he would support Sebastian in the fight against the fiendsyyy

(d) Why does Gonzalo ask Adrian to follow the three men

Gonzalo tells Adrian that all the three men namely Alonso Sebastian and Antonio are in a wild and reckless mood The thought of the heinous crime of which they are guilty has begun to torment their minds So he asks Adrian to follow those three men without loss of time and prevent them from doing anything which the turmoil in their minds might lead them to do

(e) What opinion do you form of Alonso from the above extract

Alonso who is the lesser villain is the first to give way to remorse under the effect of Arielrsquos speech The words of Ariel seem to him to be the voice of conscience speaking to him He is driven to desperation a state in which he might do violence to his life

Subject =Accounts

Ac-12 15420 topic-pL Appropriation ac

PROFIT AND LOSS APPROPRIATION ACCOUNT

MEANING AND PREPARATIONProfit and Loss Appropriation Account is merely an extension of the Profit and Loss Account of the firm The profit of the firm has to be distributed amongst the partners in their respective profit sharing ratio But before its distribution it needs to be adjusted All Adjustments like partnerrsquos salary partnerrsquos commission interest on capital interest on drawings etc are made in this account These adjustments will reduce the amount of profit for distribution This adjusted profit will be distributed amongst the partners in their profit sharing ratio To prepare it at first the balance of Profit and Loss Account is transferred to this account The journal entries for the preparation of Profit and Loss Appropriation Account are given below

1 for transfer of the balance of Profit and Loss Account to Profit and Loss Appropriation Account

(a) In case of Net Profit

Profit and Loss Ac helliphelliphelliphelliphellipDrTo Profit and Loss Appropriation Ac(Net Profit transferred to Profit and Loss Appropriation Ac)

(b)In case of Net Loss

Profit and Loss Appropriation Achelliphelliphellip DrTo Profit and Loss Ac(Net Loss transferred to Profit and Loss Appropriation Ac)

2 for Interest on Capital

For transferring on Interest on CapitalProfit and Loss Appropriation Achelliphelliphellip DrTo Interest on Capital Ac(Interest on capital transferred to Profit amp Loss Appropriation Ac)

3 for Interest on Drawings

For transferring Interest on Drawings Interest on Drawings Achelliphelliphelliphelliphelliphellip DrTo Profit and Loss Appropriation Ac(Interest on drawing transferred to Profit amp Loss Appropriation Ac)

4 For Partnerrsquos SalaryFor transfer of partnerrsquos SalaryProfit and Loss Appropriation Achelliphellip DrTo Salary Ac(Salary transferred to profit amp Loss Appropriation Ac)

5 For Partnerrsquos CommissionFor transferring commissionProfit and Loss Appropriation Achelliphelliphellip DrTo Commission Ac(Commission transferred to Profit and Loss Appropriation Ac)

6 For Transfer of agreed amount to General ReserveProfit and Loss Appropriation Ac helliphellipDrTo General Reserve Ac(Transfer to General Reserve)

7 for share of Profit or Loss appropriation(a) If ProfitProfit and Loss Appropriation Achelliphellip DrTo Partnerrsquos CapitalCurrent Ac(Profit transferred to capitalcurrent Ac)(b) If LossPartnerrsquos Capital Current Achelliphelliphelliphellip DrTo Profit and Loss Appropriation Ac(Loss transferred to capitalcurrent Ac)

THE FORMAT OF PROFIT AND LOSS APPROPRIATION

Profit and Loss Appropriation Account for the year endedhelliphelliphelliphellip

Particulars Amount Particulars Amount

To PL Ac (loss) By pL Ac (profit)

To Interest on capital BY Interest on drawings

To partner`s commission by Partner`s capital Ac ( loss)

To Partner`s salary To Interest on partner`s loan To General Reserve To Partner`s Capital AC (Profit)

Subject= Economics

MOVEMENT ALONG THE DEMAND CURVE (CHANGE IN QUANTITY DEMANDED)In law of demand you have already studied the inverse relationship between price and quantity demanded When quantity demanded of a commodity changes due to change in its price keeping other factors constant it is called change in quantity demanded It is graphically expressed as a movement along the same demand curve There can be either a downward movement or an upward movement along the same demand curve Upward movement along the same demand curve is called contraction of demand or decrease in quantity demanded and downward movement along the same demand curve is known as expansion of demand or increase in quantity demanded

Extention of demandd

price (rs)p A

B Extentionp1 d

Q Q1

Quantity demanded ( in units)

Contraction of demandd

p2 Ccontraction

p APrice (Rs)

d

Q2 Q

Quantity demanded (in units)

Explanation of movement of demand A fall in price from OP to OP1 leads to increase in quantity demanded from OQ to OQ1 (expansion of demand) resulting in a downward movement from point A to point B along the same demand curve DD When Price rises from OP to OP2 quantity demanded falls from OQ to OQ2 (contraction of demand) leading to an upward movement from point A to point C along the same demand curve DD

  • Activity Series of Metals
    • Drawbacks of Rutherfordrsquos model of atom
      • Electromagnetic radiations
      • Properties of electromagnetic radiations
      • Characteristics of electromagnetic radiations
        • Plancks Quantum Theory-
        • Photoelectric effect
          • Intext Questions
            • Logic gates
            • Digital systems are said to be constructed by using logic gates These gates are the AND OR NOT NAND NOR EXOR and EXNOR gates The basic operations are described below with the aid of truth tables
            • AND gate
            • Example
Page 18:  · Web viewSubject. Topic. Summary. Execution. English 1 . Chapter 1 naming words . Page 8. Write the names of these pictures:- Person:-1. father. 2.Firefighter 3.doctor 4 ...

increase in time which is not possible

Chemistry Language of Chemistry

How to balance a chemical equationThere are two methods of balancing an equation(i)Hit and trial method(ii)Partial equation methodBalancing by hit and trial methodThis method consists of counting the number of atoms of each elements on both sides and trying to equalize themTake the following steps(i)Count the number of times (frequency) an element occurs on either side(ii)The element with the least frequency of occurrence is balanced first(iii)When two or more elements have the same frequencythe metallic element is balanced firstExample-1 On heatinglead nitrate decomposes to give lead dioxidenitrogen dioxide and oxygenPb(NO3)2rarrPbO+NO2+O2

In this equationLead occurs twiceNitrogen occurs twiceOxygen occurs four timesSince lead is a metalbalance it firstThe number of atom of lead is equal on the two sidestherefore it needs no balancingNow balance nitrogenOn the reactant sidethere are two atoms of nitrogenwhile on the product side oneSomultiply the product containing nitrogenon the product sideby two Pb (NO3)2rarrPbO+2NO2+O2Nowthe number of oxygen atoms on the reactant side 6while on the product sideit is 7Somultiply the entire equation by 2except oxygen to get balanced equation2Pb(NO3)2rarr2PbO+4NO2+O2Multiplication by 2 is done only when atoms of all the elements except one element are balanced and the unbalanced atom occurs separately at least once and also there is a difference of only one such atom

Math Topic AlgebraChapter

Factorisation

Study item Difference of two squares a2 ndash b2 = (a+b) (a-b)1) (i) 4x2ndash 25y2

= (2x) 2 ndash (5y) 2= (2x + 5y) (2x - 5y)

(ii) 9x2 ndash 1= (3x)2ndash(1)2= (3x + 1)(3x ndash 1)

2) (i) 150 ndash 6a2= 6(25 ndash a2)= 6(5)2 ndash(a)2= 6 (5 + a) (5 ndash a)

(ii) 32x2 ndash 18y2=2(16x2 ndash 9y2)=2(4x)2 ndash (3y)2= 2(4x + 3y)(4x - 3y)3)(i) (x ndashy )2 ndash 9 = (x ndash y )2 ndash (3)2= (x ndash y + 3) (x ndash y ndash 3)(ii) 9(x + y) 2ndash x 2= (3)2(x + y)2 ndash (x)2=3(x + y)2 ndash (x)2= (3x +3y ) 2ndash(x)2= (3x + 3y + x)(3x +3y ndash x)= (4x + 3y) ( 2x + 3y )

Commercial studies

Basic accounting terms

Today I will give you some questions from the previous study material

Questions1) Define accounting2) What do you mean by debit and

credit

3) Explain the types of account4) Define the following terms

a) Assetsb) Capitalc) Purchased) Debtorse) Transactions

5) Name the types of accounts given below

a) Krishnas accountb) Machinery accountc) Royalty accountd) Salary accounte) Furniture accountf) Audit fee account

Economics Revision Today I will give you some revision questions

Questions1) What do you mean by the terms

rdquowantsrdquo2) Write the difference between

consumer goods and producer goods

3) Define the term utility 4) Explain the different types of utility5) Define

a) Total utilityb) Marginal utility

Subject Eng Literature (The Merchant of Venice ndash William Shakespeare)Topic Act I Scene 3 Lines 1 to 48 (Shylock hellip Cursed be my tribe if I forgive him) Date 16th April 2020 (5th Period)

[Students should read the original play and also the paraphrase given in the school prescribed textbook]Summary Questions amp Answers

This scene takes place in Venice and we are introduced to the rich Jew Shylock Bassanio and Shylock are talking and Bassanio tells Shylock that he wants a loan of three thousand ducats for three months on the personal security of Antonio

o Shylock feels glad because he will be able to bind down Antonio by means of a bond on account of the loan but he tells Bassanio that all the fortunes of Antonio being invested in the merchant ships on the sea it is difficult to depend upon his credit Even under such circumstances Shylock is willing to advance the money on the personal security of Antonio

o Bassanio then invites Shylock to dine with him Shylock says that he is prepared to do anything with the Christians but not eat or drink or pray with them

o While Bassanio and Shylock are talking Antonio appears on the scene Shylock does not seem to take any notice of Antonio but goes on brooding within

(1) SHYLOCK Ho no no no no- my meaning in (Line 15-26)saying he is a good man is to have you understand me that he is sufficient Yet his means are in suppositionhe hath an argosy bound to Tripolis another to the Indies I understand moreover upon the Rialto he hath a third at Mexico a fourth for England and other ventures he hath squanderd abroad Butships are but boards sailors but men there be land-rats and water-rats land-thieves and water-thieves I mean pirates and then there is the peril of waters winds and rocks The man is notwithstanding sufficientmdashthree thousand ducats mdashI think I may take his bond

(a) Who is talking in the beginning of this scene What does Bassanio want from Shylock How does Shylock feel

In the beginning of the scene Bassanio and Shylock are talking to each other Bassanio wants to borrow three thousand ducats from Shylock for three months on the security of Antonio Shylock feels glad at heart that he will get the opportunity of binding Antonio with a bond(b) What risks does Shylock weigh in advancing the money

Shylock says that Antonio has invested all his capital in trading by sea-going ships But the ships are made of wood and the sailors of those ships are ordinary human beings The wood can

himself how he hates Antonio because of his being a Christian because he abuses Shylock in public places Shylock decides that if ever he can get Antonio to his advantage he will teach him a lesson

come to harm and men can commit mistakes and thus the capital invested in ships may be lost Then there are other dangers The goods loaded on the ships can be damaged by rats and thieves which are found both on land and water The ships can also be harmed through sea-storms submerged rocks etc(c) What two important functions does this scene have

The scene has two important functions First it completes the exposition of the two major plot lines of the play Antonio agrees to Shylockrsquos bond ndash three thousand ducats for a pound of flesh and second and more important dramatically this scene introduces Shylock himself In this scene Shakespeare makes it clear at once why Shylock is the most powerful dramatic figure in the play and why so many great actors have regarded this part as one of the most rewarding roles in all Shakespearean dramas(d) Where does this scene take place What kind of treatment has Antonio been giving to Shylock What does Shylock say when Bassanio invites him to dine with him

The action of this scene takes place in Venice Antonio has been in the habit of behaving harshly with Shylock ndash spitting on his beard and footing him like a stranger cur When Bassanio invites Shylock to dine Shylock says that he is prepared to do anything with the Christians but not eat and drink or pray with them

(2) SHYLOCK How like a fawning publican he looks (Line 38-48)I hate him for he is a Christian

But more for that in low simplicity

He lends out money gratis and brings downThe rate of usance here with us in VeniceIf I can catch him once upon the hipI will feed fat the ancient grudge I bear him

He hates our sacred nation and he railsEven there where merchants most do congregateOn me my bargains and my well-won thriftWhich he calls interest Cursed be my tribeIf I forgive him

(a) What is the context in which these words are spoken and what is the idea expressed in it

These remarks are made by Shylock when he sees Antonio coming along after Bassanio told him that the merchant will be his surety for the bond The above mentioned passage reveals Shylockrsquos hatred for Antonio Shylock says that he hates Antonio because he is a Christian and also because he gives loan without taking interest on them thereby bringing down the rate of interest in Venice(b) Explain the meaning of the phrase lsquoa fawning publicanrsquo

The phrase lsquoa fawning publicanrsquo refers to Roman tax collector It is a term of contempt and hatred on the lips of a Jew lsquoFawning Publicansrsquo were Roman tax-gatherers whose ordinary bearings towards the Jews was bullying but whose false pose of lsquohumility and contritionrsquo is touched upon in the parable in New Testament(c ) What light does the above passage throw on the character

of Shylock

The above mentioned speech of Shylock reveals him to be a wicked character having an extreme greed for wealth His intense hatred for Antonio is unjustified He hates Antonio only because he is a Christian and because he lends money without taking any interest on it thereby adversely affecting Shylockrsquos business of lending money on high interest(d) What information do you gather about Antonio from the above given lines

Shylockrsquos statement throws a valuable light on the character of Antonio Antonio appears to be a good Christian and a good human being He helps the people in need by lending them money without charging any interest on it He is a man of simple and good nature This very goodness makes him Shylockrsquos enemy(e) What does Shylock debate within himself and when To whom are the lines mentioned above addressed to

When Bassanio asks the Jew to lend him three thousand ducats on Antoniorsquos surety Shylock begins to debate within himself as to how he should exploit the opportunity of a business deal with his old enemy Antonio

The lines mentioned above are not addressed to anyone The lines are a soliloquy ie a speech made by a character to himself and not meant to be heard by the other characters present

Class XSubject Topic Summary ExecutionEnglish

LiteratureThe Blue Bead 2nd part

Things took a turn for the worst and all of a sudden a crocodile attacked the woman biting on the womanrsquos leg At that moment Sibia got up sprinted grabbed the hay fork and stabbed the crocodile in the eye with all her power Immediately the crocodile let go and went away Sibia saw a small blue bead lying by the river she grabbed it Since she was poor she didnrsquot have necklace Shersquod always wanted one like the other women now she could make one with the blue bead After that she went home and told her mother all about it

Hindi 2nd

Langबड घर की बटी( मशी परमच-)

lsquoबड घर की बटी कहानी का उददशय मधयम वग की घरल समसया को सलझा कर सगदिठत परिरवार म मिमल जलकर परम स रहन का स-श -ना हघर म शानित सथानिपत करन की जिजमम-ारी नारी की होती ह यदि- नारी समझ-ार ह उसम धय और परिरवार क परनित परम ह तो कोई भी घटना परिरवार को निवघदिटत नही कर सकती या कहानी परिरवार को सगदिठत करत हए परम सौहा- स एक दसर की भावनाओ को समझ करउनका सहयोग करत हए जीवन यापन करन की पररणा -ती हमशी परमचदर जी न इस कहानी म सय परिरवार का परनितनिनमिधतव निकया ह यह कहानी बनी माधव सिसह जो गौरी पर क जमी-ार क उनक -ो पतरो की हशरी कठ लाल निबहारीशरीकात का निववाह एकजमी-ार घरान की पतरी आन-ी स हआ थाआन-ी न ख- को ससराल क वातावरण म ढाकतिलया थाएक दि-न आन-ी का अपन -वर लाल निबहारी स झगडा हो जाता ह -ोनो भाई एक दसर स अलग होन की कोकतिशश करत हसभी बह आन-ी न अपन मधर वयवहार स लाल निबहारी को

ldquoइन नतर निपरय गणो को बीए-इनही -ो अकषर पर नयोछावर कर दि-या था इन -ो अकषर न उनक शरीर को निनबल और चहर को कानित ही बना दि-या थाldquo

क) परसतत पकतियो म निकस वयकति क बार म कहा गया ह

ख) इन पकतियो म कौन स नतर निपरय गणो क बार म कहा गया ह

ग) बीए की निडगरी परापत कर लन पर भी उपय वयकति क सवभाव की कया निवशरषता थी

घ) यह नतर निपरय गण निकस वयकति म निवदयमान थ उसक वयकतितव की कया निवशरषता थी

उततर ndashक) परसतत पकति म गौरी पर गाव क जमी-ार

क बड बट शरीकात क बार म कहा गया ह उसन बहत परिरशरम और उ-म क बा- ba की निडगरी परापत की थी अब वह एक -फतर

घर छोडकर जान स रोक कतिलयाइस पर बनी माधव सिसह न कहा निक बड घर की बटी ऐसी ही होती ह जो निबगडा काम बना लती ह अतः शीरषक साथक ह बड घर की बटी आन-ी ह

म कमचारी थाख) भरा हआ चहरा चौडी छाती और डटकर

खाना आदि- एक सबजी ल जवान क गण मान जात ह परत शरीकात न इनही नतर निपरय गणो को अपनी पढाई पर नयोछावर कर दि-या था

ग) बीए की निडगरी परापत कर लन पर भी उपय वयकति(शरी कठ की शारिररिरक तौर पर निनबल और चहर स कानित ही लगत थ इतना ही नही वह मानकतिसक तौर पर भी निपछड हए थ पाशचातय सामाजिजक कथा उस घणा एव पराचीन सभयता का गणगान उनकी निवचारधारा क परमख अग थ

घ) यह नतर निपरय गण गौरीपर गाव क जमी-ार क छोट बट लाल निबहारी सिसह म निवदयमान थ वह सजीलाजवान था और भस का दध शर दध वह सवर उठकर पी जाता था

ldquoयही कारण था निक गाव की लललन आए उनकी निन-क थी कोई कोई तो उह अपना शतर समझन म भी सकोच ना करती थी सवय उनकी पतनी को इस निवरषय म उनस निवरोध थाldquo

क) उपय पकति म इस वयकति क बार म कहा गया ह

ख) गाव की लललन आए उनकी निन-ा कयो निकया करती थी

ग) उनकी पतनी का कया नाम था उनह निकस निवरषय म अपन पनित क निवरa था और कयो

घ) इस कहानी का कया उददशय ह Continue to next helliphelliphellip

Bengali 2nd Language

ফ ফটক না ফটক( কহিতা )

পর) ldquo(ান াধাচেনা ফটপাচেথ পাথচের পাড হিচেয় এক কাঠচোটটা গাছ কহিকহি পাতায় পার ফাটিচেয় াসচেছldquoক) কার দো দেকান কহিতার অং( ) lsquo(ান াধাচেনা ফটপাচেথ পাথচের পাডহিচেয়lsquo চেত কী দোঝাচেনা চেয়চেছ গ) আচো য অংচে(lsquo এক কাঠচোটটা গাছ lsquoচেত কী দোঝাচেনা চেয়চেছ ঘ) ldquoকহিকহি পাতায় পার ফাটিচেয় াসচেছldquo ----- একথার পরকত অথC কী উততর ) ক) আচো য অং(টি পর যাত কহি সভা4 মচোপাধ যাচেয়র দো lsquoফ ফটক না ফটকrsquo কহিতার অং()কহি সভা4 মচোপাধ যায় হিছচেন দেপরচেমর কহি দেপরমচেক নানা ভহিঙগমায় হিতহিন ফটিচেয় তচেচেছন দেপরম মানচের স মচেতC র সঙগী কহিতার কহিতায় এক রb সb হচেয়র দেপরম াগরচেনর কথা চেচেছন (ান অথCাৎ দেযাচেন দেকান রস দেনই দেযাচেন দেকান মহিনতা দেনই অথ তার মধ দেযও দেপরম থাকচেত পাচের একথাই কহি তচে ধরচেত দেচেয়চেছন একটি মানচে4র মচেন দেযাচেন দেকামতার দেকান সথান দেনই পাথচেরর মচেতা হিনরসতার মচেনর মধ দেযও দেয দেপরম আসচেত পাচের দেস কথাই কহি চেচেছনগ)নারীচের যথC দেপরচেমর ছহি এই কহিতায় অকপচেট উচেঠ এচেসচেছ কহি এই কহিতায় কাটচোটটা গাছ কথাটি যার কচেরচেছন নারী দেয দেপরম দেথচেক হিতাহিত এং দেসই দেপরম সঠিক সমচেয় না পাওয়ার ন য দেপরম সমপচেকC হিচেr4 গৈতরী য় দেপরচেমর দেয গৈহি(ষট য মাধযC য সরসতা দেকামত এই সমসতর হিপরীত যথা রbতা শষকতা কচেঠার তা পরভহিত দোঝাচেত এক কাঠচোটটা গাছ কথাটি যার কচেরচেছনঘ) এাচেন এক নারীর যথC দেপরচেমর কথা হিনহিCপত ভাচে চেচেছন কহি অসমচেয় নারীর ীচেন দেপরম দেচেগচেছ এতহিন তার হয় রb কচেঠার হিছ দেপরচেমর অভাচে ঠাৎ দেসই শষক মরভহিমচেত সচের আভাস এচেসচেছ দেপরম দেযন 4Cার স(ীত তাই পরায় মত গাচেছ কহিকহি পাতা গহিচেয় উচেঠচেছ

Biology Chapter - 01Controlling Air Pollution

Today we will discuss how we control air pollution from domestic combustion

Q1Describe any five ways of reducing air pollution from domestic sources bull The number of pollutants in the air is verylarge and we always try to control them byfollowing ways

i) Solar cooker and solar heater It use no fuel reduce damage of environment by fuel use or reducing deforestation It maintains coolness of house It releases very less orno oil gas or grease

ii) Piped natural gas (PNG) It emits very less by products into the atmosphere As it isdistributed through pipe lines so there iscontinuous supply of fuel is possible

iii) Liquefied Petroleum Gas (LPG) It hasa higher heating value LPG doesntcontain sulphur so it burns a lot cleanerenergy sources It releases very less oralmost no fume in air

iv) Electricity based cooking Emission free cooking alternative for urban dwellers causeselimination of adverse health impactsofindoor air pollution It helps to avoid theinconveniences associated with procurement of LPG

v) Biogas It contains 75 methane whichmakes it an excellent fuel It burns without smoke and biogas plant leaves no residue like ash in wood charcoal etc Thus it isaclean fuel

Economics

Factors of Production

Today firstly we would recall the last class for 5 mins and then we would proceed with the further topics of the chapter

The concept meaning of land characteristics of land and importance of land to be repeated for the absentees as well as the students who were there in the class the previous day

Today we will start with the last portion of land before it the meaning of land to be repeated onceAs by now we all know that

Questions1What do you mean by productivity of landAnswer By productivity of land we mean the capacity of a piece of land to produce a crop

Thus it refers to the average output per unit of landSay per acre per hectare etc= (OutputArea of land)

2 What are the factors influencing the productivity of landAnswer

Natural factors Productivity of land is largely determined by the natural

Land is defined to include not only the surface of the earth but also all other free gifts of nature(for example mineral resources forest resources and indeed anything that helps us to carry out the production of goods and services but is provided by nature free of cost)

We will move on to the last portion of land by discussing Productivity of Land

By productivity of land we mean the capacity of a piece of land to produce a crop

Thus it refers to the average output per unit of land

Say per acre per hectare etc= (OutputArea of land)

With this we shall proceed further with the main factors that determine the productivity of land

Natural factors Human factors Improvements on land Location of land Organisation Ownership of land Availability of capital Proper use of land State help

Note economic development of a country depends upon the quality of its land If the land is fertile it will quicken the pace of development of the country

qualities of land such as fertility etc

Human factors Land cannot produce anything by itself Man has to apply labour on it to produce for himself So productivity of land depends on the knowledge and skills of workers

Improvements on land production of land is affected by land development measures like provision of well or tubewell irrigation proper drainage

State help The government of a country especially less developed country can play a vital role in improving the agricultural productivity by providing better irrigation facilities

Organisation Productivity of land also fdepends upon the way how the factors of production like labour and capital are organised

In order to increase productivity trained workers modern implements scientific methods good seeds are all essential

3 lsquoImproved technology affects the productivity of landrsquo Explain this statement with the help of suitable example Answer Use of improved technology raises the productivity of land Example By using HYV seeds chemical manures and modern machines per hectare output increases

Physics Force (Summary)

Question Write the expression for the moment of force about a given axisSolutionsThe expression for the moment of force is given byMoment of force about a given axis = Force times perpendicular distance of force from the axis of rotationQuestion What do you understand by the clockwise and anticlockwise moment of force When is it taken positiveSolutionsIf the effect on the body is to turn it anticlockwise moment of force is called the anticlockwise moment and it is taken as positive while if the effect on the

body is to turn it clockwise moment of force is called the clockwise moment and it is taken as negative

Math Topic Commercial Mathematics

Chapter Goods and services Tax

Study item Some solved sums from exercise ndash 1 A retailer buys a TV from a wholesaler for Rs 40000 He marks the price of the TV 15 above his cost price sells it to the consumer at 5 discount on the marked price If the sales are intra ndash state and the rate of GST is 12 find

(i) The marked price of the TV(ii) The amount which the consumer pays for the TV(iii) The amount of tax (under GST) paid by the retailer to the central

Government(iv) The amount of tax (under GST) received by the State Government

Solution As the sales are intra- state sale and the rate of GST 12 So GST comprises of 6 CGST and 6 SGSTTherefore a retailer buys a TV from a wholesaler for Rs 40000Therefore the amount of GST collected wholesaler from the retailer or paid by retailer to wholesalerCGST = 6 of Rs 40000 = Rs(6100 times40000) =Rs 2400SGST = 6 of Rs 40000 = Rs (6100 times 40000) =Rs 2400Therefore wholesaler will pay Rs 2400 as CGST and Rs 2400 as SGSTTherefore amount of input GST of retailer Input CGST = Rs 2400 and input SGST = Rs 2400Again the retailer marks the price of the TV 15 above his cost price(i) The marked price of the TV

= Rs 40000 + Rs 40000times15= Rs 40000 + Rs 40000times 15100= Rs 40000 + Rs 6000Rs 46000But the retailer sells it to consumer at 5 discount on the marked priceCost price after discount = Rs 46000 ndashRs46000times 5100 =Rs 46000 ndashRs 2300= Rs 43700Therefore the amount of GST collected retailer from consumer or paid by consumer to retailerCGST = 6 of Rs 43700 =Rs ( 6100 times43700)Rs 2622SGST = 6 of Rs 43700 = Rs (6100 times 43700) =Rs 2622Amount of the output GST of retailer Output CGST = Rs 2622 and output SGST = Rs 2622

(ii) The amount which the consumer pays for the TV= cost price of TV to consumer + CGST paid by consumer + SGST paid by consumer= Rs 43700 + Rs 2622 + Rs 2622= Rs 48944

(iii) The amount of tax (under GST ) paid by the retailer to the central Government=CGST paid by retailer = output CGST ndash input CGST=Rs 2622 ndash Rs 2400=Rs 222

(iv) The amount of tax ( under GST ) received by the State Government = SGST paid by wholesaler + SGST paid by retailer= Rs 2400 + output SGST ndash input SGST=Rs 2400 + Rs 2622 ndash Rs 2400=Rs 2400 + Rs 222= Rs 2622

Commercial studies

Stakeholders Today I am going to give some revision questions from the previous study material

Questions1) State the two expectations of

employees from a business concern2) Give two distinctions between

stakeholder and shareholder3) Give two difference between

internal stakeholders and external stakeholders

4) Give two expectations of suppliers from a business organisation

5) Who is a stakeholder in commercial organisations

Chemistry Periodic Table

Merits of Mendeleevrsquos Periodic law are as follows - 1He grouped the elements on the basis of atomic mass 2 He left gaps for undiscovered elements like Gallium Scandium germanium Also he left a full group vacant for undiscovered inert gases 3 He could predict proportions of several elements on basis of their position in periodic table like Ga Sc etc 4He could predict errors in atomic weights of some elements like gold platinum etc

Anomalies in Mendeleevrsquos Periodic law are as follows - 1 Position of isotopes could not be explained 2 Wrong order of atomic masses could not be explained

For example- as Arnur atomic mass 40 come first and K with low atomic mass (30) should come later but k should be placed first

According to Bohrrsquos Modern Periodic table properties of elements are periodic functions of their atomic numbers

So when elements are arranged according to increasing atomic numbers there is periodicity in electronic configuration that leads to periodicity in their chemical properties

It consists of horizontal rows (Periods) Vertical column (Groups)

There are 7 period and 12 groups in this long form of periodic table

Ist period has 2 elements IInd period has 8 elements IIIrd period has 8 elements IVth period has 18 elements Vth period has 18 elements VIth period has 32 elements VIIth period hs rest of elements

Note - The number of valence electrons in atom of elements decides which elements will be first in period and which will be last

In group- 1 to 2 gp and 13 to 17 contain normal elements 3 to 12gp ndash transition elements 57 to 71 - lanthanides 89 to 103 - Actinides

Left hand side ndash metals Right hand side ndash nonmetals

Note- Hydrogen element has been placed at top of Ist group Electronic configuration of H is similar to alkali metal as both have 1 valence electron

V electron of gp I element -- 1 V electron of gp 2 element -- 2 V electron of gp 13 element -- 3 V electron of gp 14 element -- 4 V electron of gp 15 element -- 5 V electron of gp 16 element --6 V electron of gp 17 element -- 7 V electron of gp 18 element -- 8

English 1 Transformation of sentences

Sentences A sentence is a group of words which makes complete sense

Exercise 2Change the following sentences from

a Assertive sentencesb Imperative sentencesc Interrogative sentencesd Exclamatory sentences

Sentences can be changed from one grammatical form to another without changing the meaning of the sentence This is known as transformation of sentences

assertive to interrogative1 Nobody would like to be a fool

Who would like to be a fool2 Their glory can never fade

When can the glory fade3 Nobody can control the wind

Who can control the wind4 It matters little if I die

What though I die5 No man can serve two masters

Can any man serve two masters

Exercise 3Interchange of assertive and Exclamatory sentences

1 She leads the most unhappy lifeWhat an unhappy life she leads

2 This is indeed an interesting bookWhat an interesting book this

3 He is a very great manWhat a great man he is

4 It is a very lame excuseWhat a lame excuse

5 It is sad that she died so youngAlas she died so young

Class XISubject Topic Summary Execution

Hindi 2nd lang

पतर परम(परमचदर) पतर परम कहानी म एक निपता की इचछाओ का वणन निकया गया ह अपन बड पतर परभ -ास स निपता चतनय -ास का निवशरष परम था निपता को उसक जनम स ही बडी-बडी आशाए थी उसम दसर बट कतिशव-ास की अपकषा स- उतसाह की मातरा अमिधक थी वह उस इगलड भजकर बरिरसटर बनाना चाहत थभागय का खल भी बडा निनराला ह बीए की परीकषा क बा- वह बीमार पड गया डॉकटरो न भी जवाब - दि-या थाचतन -ास जी बहत ही कजस थ बवजह पस खच करना नही चाहत थ अगर गारटी मिमलती तो शाय- पस खच भी कर -त परत गारटी नही थी परिरणाम सवरप उनक बट का -हात हो गयाजब बट को समशान ल जा रह थ तो वहा काफी शोर गान बजान हो रह थ पछन पर पता चला निक निकसी निपता निपछल तीन साल स निबमार था और उसक ईलाज म रपया पानी की तरह बहाया पर ठीक नही हए परत उसक बट को तनिनक भी अफसोस नही था उसका कहना था उसन कोकतिशश तो कीयह -खकर चतनय-ास जी को आतम निगलानी हईतभी स उनका म परिरवतन हआ और बट का भोज काफी धमधाम स निकयाऔर वहइस पशचाताप की आग म जलत रह औला- स बढकर पसा नही होता ह इस बात को समझन म उनह काफी व लग गया

hellipContinue to next

BENGALI(2ND LANGUAGE)

পরথমঅধযায়-ঠাকরারীনদরনাথঠাকর

নয়ন দোচের হিমাচেররা া নাচেমই হিযাত হিছচেন ায়ানার উাররণ সবরপ নয়ন দোচের ারা হিা (াচেকর হিা হিচেতন এছাাও দেকান উৎস উপচেb রাহিতর দেক হিন করার উচেfচে(য তারা সযC হিকরচেরণ রনয পরীপ জবাহিচেয় তাচেত রপার হির 4Cরণ করচেতন ঠাকরা এই নয়ন দো হিমারচের দে(4 ং(ধর হিছচেন হিমাররা ায়ানার ষটানত পর(Cন কচের তারা হিনঃসব এই হিমাহিরর দে(4 ং(ধর গৈকাস নদর রায়চেৌধরী গৈকাস া নয়ন দোচের সমসত সমপহিতত ঋচেরণর াচেয় হিহিx কচের অহি(ষট যা আচেছ তাচেত হিপত

ইার হিপতার মতয ইচে পর নয়নচোচের ায়ানার দেগাটা কতক অসাধাররণ শরাদধ (াহিনতচেত অহিনতম ীহিপত পরকা( কহিরয়া ঠাৎ হিনহিয়া দেগ- ক) কার দো দেকান গচেলপর অং() কতা দেক ইার চেত কাচেক দোঝাচেনা চেয়চেছ গ) পরসঙগ কী কতার কতয পহিরসফট কচেরা

পরচে4র যাহিত রbা করা সমভ নয় তাই হিতহিন পতরচেক হিনচেয় ককাতায় সাস শর কচেরন গলপ কথচেকর আহিথCক অসথা নয়ন দোচের হিমাচের দেথচেক সমপরণC আাা কথচেকর হিপতা হিনচের দেষটায় অথC উপাCন করচেতন া উপাহিধ াচেভর নয তার াসা হিছনা আর দেসই কারচেরণ কথক তার একমাতর উততরাহিধকার চেয় তার হিপতার পরহিত কতজঞ কথক দো পা হি(চেচেছন হিনচের পরারণ ও মান রbার নয উপচেযাগী অথC হিনা দেষটায় পরাপত চেয়চেছন- এটাই তার কাচেছ পরম দেগৌরচের হি4য় চে মচেন কচেরন কাররণ (নয ভাণডাচের গৈপতক ায়ানার উজজব ইহিতাস অচেপbা দোার হিসeচেকর মচেধয গৈপতক দেকামপাহিনর কাগ তার কাচেছ অচেনক দেহি( মযান

TO BE CONTINUED

উ- ক) আচোয অং(টি রীনদরনাথ ঠাকচেরর দো ঠাকরা গচেলপর অং() কতা চেন আচোয গচেলপর গলপ কথকইার চেত নয়ন দোচের হিমাহিরর দে(4 ং(ধর গৈকাস ার কথা া চেয়চেছ গৈকাস া নয়ন দোচের সমসত সমপহিতত ঋচেরণর াচেয় হিহিx কচের অহি(ষট যা আচেছ তাচেত হিপত পরচে4র যাহিত রbা করা সমভ নয় তাই হিতহিন পতরচেক হিনচেয় ককাতায় সাস শর কচেরনগ) গৈকাস ার হিপতার মতযর পর নয়ন দোচের হিমাহিরর অহিসততব হিপত য় কচেয়কটা উৎস ও শরাদধ- (াহিনতচেত হিমাহিরর দে(4 কহিটক যয় চেয় হিগচেয় এচেক াচের দে(4 চেয় যায় তন তাচের গC করার মত আর হিকছই হিছ না-দেসই পরসচেঙগ এই উহিকত নয়নচোচের হিমাচেররা া নাচেমই হিযাত হিছচেন ায়ানার উাররণ সবরপ নয়নচোচের ারা হিা (াচেকর হিা হিচেতন এছাাও দেকান উৎস উপচেb রাহিতরচেক হিন করচেত হিগচেয় তারা সযC হিকরচেরণর নয পরীপ জবাহিচেয় তাচেত রপার হির 4Cরণ করচেতন তাই দেসকাচের ায়ানা দেহি(হিন সথায়ী চেত পারত না হিহিভনন উৎস শরাদধ- (াহিনতচেত সাধযা হিতহিরকত র করার নয হিমাহির হিহিকচেয় দেযত হ হিতC কা হিহি(ষট পরীচেপর দেত দেযমন অলপকাচের মচেধয হিনঃচে(4 চেয় যায়-নয়নচোচের হিমারচের অসথা তাই চেয়হিছ এই কারচেরণই কথক নয়নচোচের হিমারচের গা ভরা আমবর সয করচেত পারতনা

Physics Dimensional Analysis (Summary)

Q Find the dimensions of consts ab in relation

p=(bminusxlowastx)at

where p is the power x is the distance and t is time

Ans From principle of homogeneity dimension of b x2 are same Dim of b = dim of x2 = [L2] = [ML2T0]Dim of a = dim of ( b- x2)dim of (pt) = [M0L2T0][ML2T-2] [T-1] [T] = [M-1L0T2]

Chemistry Atomic Structure Drawbacks of Rutherfordrsquos model of

atom a According to Rutherfordrsquos model of atom electrons which are negativelycharged particles revolve around the nucleus in fixed orbits Thusb theelectrons undergo acceleration According to electromagnetic theory of Maxwell a charged particle undergoing acceleration should emitelectromagnetic radiation Thus an electron in an orbit should emitradiation Thus the orbit should shrink But this does not happenc The model does not give any information about how electrons aredistributed around nucleus and what are energies of these electrons Isotopes These are the atoms of the same

Properties of electromagnetic radiationsa Oscillating electric and magnetic field are produced by oscillating charged particles These fields are perpendicular to each other and both areperpendicular to the direction of propagation of the waveb They do not need a medium to travel That means they can even travel invacuum

Characteristics of electromagnetic radiationsa Wavelength It may be defined as the distance between two neighbouring crests or troughs of

element having the same atomicnumber but different mass numbere g 1H11H21H3

Isobars Isobars are the atoms of different elements having the same massnumber but different atomic numbere g 18Ar40 20Ca40

Isoelectronic species These are those species which have the same numberof electrons

Electromagnetic radiationsThe radiations which are associated withelectrical and magnetic fields are called electromagnetic radiations When anelectrically charged particle moves under acceleration alternating electricaland magnetic fields are produced and transmitted These fields aretransmitted in the form of waves These waves are called electromagneticwaves or electromagnetic radiations

wave as shown It is denoted by λb Frequency (ν) It may be defined as the number of waves which passthrough a particular point in one secondc Velocity (v) It is defined as the distance travelled by a wave in onesecond In vacuum all types of electromagnetic radiations travel with thesame velocity Its value is 3 times10 8m sec-1 It is denoted by v

d Wave number Wave number is defined as the number of wavelengths per unit lengthVelocity = frequency timeswavelength c = νλ

Plancks Quantum Theory- o The radiant energy is emitted or absorbed not continuously but discontinuously in the form of small discrete packets of energy called lsquoquantumrsquo In case of light the quantum of energy is called a lsquophotonrsquoo The energy of each quantum is directly proportional to the frequency of the radiation ie E α υ or E= hυ where h= Planckrsquos constant = 6626 x 10-27 Js o Energy is always emitted or absorbed as integral multiple of this uantum E=nhυ Where n=1234Black body An ideal body which emits and absorbs all frequencies is calleda black body The radiation emitted by such a body is called black body radiation

Photoelectric effectThe phenomenon of ejection of electrons from thesurface of metal when light of suitable frequency strikes it is calledphotoelectric effect The ejected electrons are called photoelectrons

Biology Chapter - 02Systematics and Five Kingdoms

Scientists divide the whole living organisms into two kingdom first and ultimately by five kingdom at last

In the earlier systems of classifications organisms are divided into kingdom plantaeand kingdom animalia on the of presenceof cell wall their modes of nutrition and movements

Some problem arise like fungi share manycharacteristic withplant despite their heterotrophic nutrition bacteria protozoa areunicellular present in both kingdom Toovercome this third kingdom Protista isintroduced which include

unicellularorganisms But there is also another

problem Allunicellular organisms are not similar kind The cellular structure of prokaryotes is verydifferent from that of other organismsEukaryotes possess a true nucleus and allcell organelles that are not present inprokaryotes So the fourth kingdom Monerais introduced which include unicellular prokaryotes (bacteriaamp blue green algae)

bull Still some problem arise in kingdomplantae

So in 1969 R H Whittakar proposedanew five kingdom System of classification

i) Kingdom Monera - unicellular prokaryotes

ii) kingdom Protista - unicellular eukaryotes

iii) Kingdom Fungi - uni or multicellular fungi with cell wall but without chlorophyll

iv) Kingdom Plantae - Multicellular Plants

v) Kingdom Animalia - Multicellular Animals

EVS Chapter 1 ndash Modes of Existence

An agricultural society

An agricultural society also known as an agrarian society is a society that constructs social order around a reliance upon farming More than half the people living in that society make their living by farming

People in an agricultural society generally lead a more settled lifestyle than those in nomadic hunter-gatherer or semi-nomadic pastoral societies because they live permanently near the land that is farmed Agricultural settlements tend to develop in areas of convenience near bodies of water which is used for both crops and transportation or along trade routes Not everyone in an agricultural society is a farmer Some people make a living trading or making and selling goods such as tools used for farming

Another way to define an agrarian society is to see the total amount of production in a nation In an agrarian society cultivating the land is the main source of wealth Such a society can recognize other means of subsistence and work habits but emphasizes the importance of agriculture and livestock Agrarian societies have existed in various parts of the world for 10000 years and continue to exist today They have been the most common form of socio-economic organization for most of recorded human history

Q) Write the features of agricultural society

Ans - Structure and Features of Agrarian Society1 Occupational Structure

An agrarian society is generally associated with the domestication of plants and animals The domestication of plants means farming and that of animals means herding Often there is mixture of farming and the use of such domesticated animals as cow goat and sheep

2 Forms of Land Ownership in Agrarian SocietiesGenerally there are landlords supervisory farmers cultivators and share croppers The landholders own the land but do not work on it They let it out for sharecropping The supervisory farmers are those who live by having their land cultivated by hired labourers The cultivators cultivate the land for themselvesThe share-croppers are those who live by tilling other peoplersquos land or a crop-sharing basis The artisans own their means of production and produce by their own labour in their homesteads

3 Village Community System An agrarian society is highlighted by

the institution of village community system The agrarian economy made fixed dwelling houses necessary Living close together for protection and co-operation and living nearer to the land gave birth to agricultural villages The village is not only the residential place of farmers it is also the social integrator

4 Minimal Division of Labour Another structural feature of agrarian society is a minimal division of labour Except for the basic division founded on age and sex differences there are few specialized roles There is only one predominant type of occupation ie domestication of plants and animals For all the people the environment physical as well as social is the same

5 Role of Family The farm family is of the patriarchal type the father is the final arbiter in most of the familyrsquos major decisions The life of ail men and women is merged in family life Since there are not many special organizations family is the only organisation to perform the tasks of aid and protection

6 Sense of Unity The members of an agrarian society exhibit a strong in-group feeling Since the whole of their social lives is wrapped up in a society which is physically economically and socially homogenous they are inclined to view the entire outside world as an out group

7 Informal Social Control An agrarian society is regionally divided into villages In a village community the force of traditional mores is more dominant than in the urban community In the village everybody is known to everybody The members in a village community help each other and share the joy and sorrows of each other Crime in an agrarian society is rare

8 Simplicity and Uniformity Life of the people in an agrarian society is marked by simplicity and uniformity Their main occupation is agriculture which largely depends upon the vagaries of nature An agrarian society is a religious society

Math Compound angles Compound angles The algebraic sum of two or more angles is called a compound angle If A B C be three angles then A+B B+C C+A A-B B-C A-C A+B-C etc are compound angles In this chapter we shall discuss the trigonometrical ratios of compound angles Theorem 1 If A B and A+B are all pisitive acute angles theni) sin( A+B) = sin A cos B + cosA sinBii) cos(A+B) = cosA cosB- sinA sinBTheorem 2If A and B are positive acute angles and AgtB theni) sin(A-B) = sin A cosB- cos A sinBii) cos(A-B) = cos A cos B+ sin A sin BTo prove that i) sin(A+B) sin (A-B) = sin2 A - sin2 B = cos2 B- cos2 A

Example 1 Prove that tan70deg=2tan50deg+tan20degSolutiontan70deg = tan(50deg + 20deg)Or tan70deg=(tan 50deg+tan 20deg)(1-tan50degtan20deg) or tan70deg (1 ndash tan 50deg tan20deg) = tan50deg+tan20degor tan70deg= tan70deg tan50deg tan20deg+ tan50deg + tan20deg = cot20deg tan50deg tan20deg + tan50deg + tan20deg = 2 tan50deg+ tan20degExample 2 If A + B = 45deg show that (1 + tanA) (1 + tanB) = 2Solutiontan(A + B) =( tan A + tan B) (1 - tan

ii) cos(A+B) Cos(A-B) = cos2 A- sin2 B = cos2 B -sin2 AProof i) LHS= sin(A+B)sin(AminusB) [Recall sin(αminusβ)=sinαcosβminuscosαsinβ And sin(α+β)=sinαcosβ+cosαsinβ]= (sinAcosB+cosAsinB)times(sinAcosBminuscosAsinB)= sin2Acos2Bminuscos2Asin2B [Recall sin2α+cos2α=1 From above we can then assume correctly that sin2α=1minuscos2α AND cos2α=1minussin2α] = sin2A(1minussin2B)minussin2B(1minussin2A) = sin2Aminussin2Asin2Bminussin2B+sin2Asin2B = sin2Aminussin2B= 1-cos2A-(1-cos2B) = cos2 B- cos2 A = RHSii)LHS= cos (A+B) cos(A-B) [ cos(A+B) = cos AcosB- sinAsinBCos(A-B) = cosAcosB+ sinAsinB]= cos2 A Cos2 B- sin2 A Sin2 B= cos2 A( 1-sin2 B) - (1- cos2 A) sin2 B= cos2 A- cos2 A sin2 B- sin2 B+ cos2 A sin2 B=cos2 A- sin2 B=1- sin2 A-(1-cos2 B) = cos2 B- sin2 A= RHSTangent formulae for compound anglesi)tan (A + B) = tan A + tan B1-tan A tan Bii)tan (A ndash B) = tan A-tan B1+tan A tan Biii) cot (A + B) = cot Acot B-1cot A+cot B(viii) cot (A ndash B) = cot Acot B+1cot B-cot A

A tan B) Or 1= (tan A+ tanB) (1-tan A tanB) Or tanA + tanB + tanA tanB + 1 = 1 + 1Or tanA (1 + tanB) + (1 + tanB) = 2Or (1 + tanA) (1 + tanB) = 2Example 3 Find the value of sin 15degSolution sin 15deg= sin(45deg-30deg) = sin45degcos 30deg- cos45degsin30deg =(1radic2) (radic32) -(1radic2) (12) = (radic3-1) 2radic2Example 4 If sin A = 1 radic10 and sin B = 1 radic5 where A and B are positive acute angles then what is A + B SolutionWe know that sin (A + B) = sin A cos B + cos A sin B= [1 radic10] [radic(1 minus 1 5)] + [1 radic5] radic(1 minus 1 10)= [1 radic10] [radic4 5] + [1 radic5] [radic9 10]= [1 radic50] times (2 + 3)= 5 radic50 = 1 radic2

sin (A + B) = sin π 4rArrHence A + B = π 4Example 5 If A + B = 225o then find [cot A] [1 + cotA] times [cot B] [1 + cot B]Solution[cot A] [1 + cotA] times [cot B] [1 + cot B] = 1 [(1 + tan A) times (1 + tan B)]=1 [tan A + tan B + 1 + tan A tan B] [ tan (A + B) = tan225o]∵

tan A + tan B = 1minus tan A tan BrArr= 1 [1 minus tan A tan B + 1 + tan A tan B]= 1 2

COMMERCE

CLASSIFICTION OF HUMAN ACTIVITIES-ECONOMIC AND NON-ECONOMIC

Firstly we shall recall the previous class for 5 mins especially for the absentees and for also the rest of the students who were there

Today at first we briefly discuss the earlier portions of the chapter

1Business-It includes all those economic activities which are concerned with production and exchange of goods and services with the object of earning profit Example A factory shop beauty parlour also business enterprises

2Profession ndashThe term profession means an occupation which involves application of specialized knowledge and skills to earn a living For Example Chartered Accountancy medicine law tax consultancy are example of professions

Questions1What are the main features of ProfessionAnswer The main features of a profession are as follows a Specialised body of knowledge-Every profession has a specialised and systematised body of knowledge b Restricted entry- Entry to a profession is allowed only to those who have completed the prescribed education and have the specialised examination c Formal education and training ndashA formal education and training is given to the person who wants to acquire the professional

3Employment-Employment mean an economic activity where people work for others in exchange for some remuneration (salary)The persons who work for others are called lsquoemployeesrsquo The persons or organizations which engage others to work for them are called lsquoemployersrsquoEg A doctor working in a hospital is employment as he is working for a salaryA lawyer may serve as a law officer in a bank

With this we shall proceed with the features of both Profession amp Employment

The main features of a profession are as follow

a Specialised body of knowledge b Restricted entry c Formal education and training d Professional association e Service motive f Code of contact

The main features of an employment are as follows

a In employment a person works for others called employer

b An employee provides personal service

c There is a service agreement or contract between the employee and the employer

d The employee has to obey the order of the employer

e No capital investment is made by the employer

Various examples of Employment are as follows

aA teacher teaching in a school or collegeb An engineer employed in Municipal Corporation of DelhicAn accountant working in the accounts department of a companydA doctor working in a hospital

Note In all the above examples of employment the individual who is involved in each example is working as an employee for a salary under an employer

qualification(MBBSCALLB)d Service motive ndashProfessionals are expected to emphasis service more on their clients rather than economic gain f Code of Conduct-The activities of professionals are regulated by a code of conduct

2 What are the main features of EmploymentAnswer The main features of an employment are as followsa In employment a person works for others called employerb An employee provides personal servicec There is a service agreement or contract between the employee and the employerd The employee has to obey the order of the employere No capital investment is made by the employer

3 Give various Professions and their respective Association are given below

Professions

Professional

Professional association

Medical profession

Doctor Medical Council of India

Law profession

Lawyers Bar Council of India

Accounting Profession

Chartered

The Institute of Chartered Accounts of India( ICAI)

Engineerin Engineers The

g Profession

institute of Engineers (India)

Accounts Basic accounting terms

Today we will give you some questions from the previous study material

Questions6) Define accounting7) What do you mean by debit

and credit8) Explain the types of account9) Define the following terms

a) Assetsb) Capitalc) Purchased) Debtorse) Transactions

10) Name the types of accounts given below

a) Krishnas accountb) Machinery accountc) Royalty accountd) Salary accounte) Furniture accountf) Audit fee account

Economics Basic Economic ConceptsSub topic

UTILITY

Before starting todayrsquos class we shall recall the last class which was about UTILITY AND THE FEATURES OF UTILITY

Now we shall proceed with the further topics of the chapter

Todayrsquos topic from the chapter lsquo Basic Economic Conceptsrsquo will be TOTAL UTILITY amp MARGINAL UTILITYNow let us quickly revise the concept of utility with an example ie goods and services are designed because they have an ability to satisfy human wantsThis feature of being able to satisfy human wants is termed as utility For example we derive utility from WiFi services as it gives us satisfaction by connecting us to our friends and family through social media here consumers derive utility from WiFi services

From the above concept we shall start with todayrsquos topicEconomists have defined TOTAL UTILITY (TU) as the total satisfaction obtained by consuming a given total amount of a good and serviceFor example the total satisfaction obtained from eating 10 mangoes is the total utility of 10 mangoes

MARGINAL UTILITY (MU) is the additional satisfaction derived from each additional unit

Questions1 What is Total Utility (TU)

Answer Total Utility (TU) is the

aggregate of the utility that a consumer derives from the consumption of a certain amount of a commodityTU=MU1+MU2++MUn

2 What is Marginal UtilityAnswer

Marginal Utility (MU) is the additional made to the total utility as consumption is increased by one more unit of the commodityMU= TUn ndashTUn-1

NoteOften economists tend to

subdivide utility into an imaginary unit called UTIL

consumed In this casethe utility obtained from each mango as it is consumed as the MU of that mango It is also defined as the addition made to the total utility when an additional unit is consumed Often economists tend to subdivide utility into an imaginary unit called UTIL

Note As a consumer increases the consumption of a good over period of time the total utility or total satisfaction derived from it increases to appoint and thereafter it decreasesHowever as the consumer keeps on consuming the good the marginal utility or the additional utility derived from it decreases

SubjectBusiness studies

Topic

BUSINESSENVIRONMENT

Summary

Now quickly let us revise the earlier points that we have already done in the last class and let us proceed with the other topics that are there in the chapter

Firstly we will recall the internal and external factors of micro environment and then we shall proceed in details

Meaning and list of internal and external factors

aInternal factorsInternal factors refer to all the factors existing within a business firm The internal factors are considered controllable because the enterprise has control over these factorsFor an example a company can alter its organization structure policies programmes employees physical facilities and marketing mix to suit the changes in the environmentList of internal factors areCorporate culture mission and objectives top management organizations structure company image and brand equity company resources

b External factorsExternal factors refer to those individual and groups and agencies with which a particular business organization comes into direct and frequent contact in the course of its functioningThese individuals and groups are known as STAKEHOLDERS because they have a stake (financial interest ) in the working and performance of the particular business List of external forces (stakeholders)Customers competitors investors suppliersmiddlemen (marketing intermediaries)

Execution 1 What do you mean by internal

factors in micro environmentAnswerInternal factors refer to all the factors existing within a business firm The internal factors are considered controllable because the enterprise has control over these factorsFor an example a company can alter its organization structure policies programmes employees physical facilities and marketing mix to suit the changes in the environment

2 What do you mean by external factors in micro environment

AnswerExternal factors refer to those individual and groups and agencies with which a particular business organization comes into direct and frequent contact in the course of its functioningThese individuals and groups are known as STAKEHOLDERS because they have a stake (financial interest) in the working and performance of the particular business

3Who are stakeholdersSTAKEHOLDERS are individuals and groups who have a stake (financial interest ) in the working and performance of the particular business 4Discuss the internal factors in briefa Corporate CultureThe values beliefs and attitudes of the founders and top management of the company exercise

financers publics

customers

suppliersfinancers

competitors

middlemen

publics

Fig STAKEHOLDERS OF A COMPANY

Apart from micro environment the other main dimension of business environment isMacro environment Macro environment refers to the general environment or remote environment within which a business firm and forces in its micro environment operateA company does not directly or regularly interact with the micro environmentTherefore macro environment is also known as indirect action EnvironmentThe macro environment forces are less controllable than the micro forces

Macro environment consists of the following components

POLITICAL AND LEGAL ENVIRONMENT

ECONOMIC SOCIAL AND ENVIRONMENT

CULTURAL

ENVIRONMENT

TECHNOLOGICAL ENVIRONMENT

a strong influence on what the cmpaany stands for how it does things and what it considers importantbMission and objectivesThe business philosophy and purpose of a comoany guide it prioritiesbusiness strategiesproduct market scope and development scope

cTop management structurethe composition of board of directors the degree of professionalization of management and the organizational structure of a company have important bearing on its business decisions

dPower structureThe internal power relationship between the board of directors and the chief executive is an important factor

eCompany image and brand equityThe image and brand equity of the company play a significant role in raising finance forming alliance choosing dealers and suppliers launching new products entering foreign markets

5 What is Macro environmentAnswerMacro environment refers to the general environment or remote environment within which a business firm and forces in its micro environment operateA company does not directly or regularly interact with the micro environmentTherefore macro environment is also known as indirect action EnvironmentThe macro environment forces are less controllable than the micro forces 6 What are the components of macro environmenta Political and legal environmentb Economic environmentc Social and cultural environmentd Technological environment

BUSINESS FIRM

Fig COMPONENTS OF MACRO ENVIRONMENTPolitical science

Introduction to political science

Comparative politics and itrsquos scope Comparative politics is the second major dimension of political scienceIt is also a very vast area of study and a very large number of political scientists even treat it as an autonomous area of study within the board ambit of political scienceScope of comparative politics-

1 All political structures -Comparative politics includes the study of all structures formalnon formal governmental and extra governmental which are directly or indirectly involved in politics in all the countries of the world

2 Functional studies- Comparative politics seeks to study politics less from the point of view of the legal institutions in terms of their powers and move from the point of view of their functions which constitute the political process and their actual Operation in the environment

3 Study of political behaviour- Another important part of its scope is the study of the actual behaviour of the people in the process of politics

4 Study of similarities and differences- comparative politics also undertakesan analysis of the similarities and differences among political process and functions

5 Study of all political systems -comparative politics seeks to analyse the actual behaviour and performance of all political systems western as well as non western

6 Study of the environment and infrastructure of politics-The study of politics demands a study of the psychological sociological economic and anthropological environment in fact the social environment as a whole in which each political system operates

7 Study of political culture- political culture is composed of attitudesbeliefs emotions and values of a society that relate to the political system or politics

8 Study of political participation- Political participation is a universal processThe only difference is that while in some states it is limited in others it is wider

9 Study of political process- political

Answer the following questions-

What is comparative politics

What are the scope of comparative politics

Homework- learn

processes like decision makingpolicy making judicial process leadership recruitment process and others are always at work in all political systems

The scope of comparative politics is very comprehensive It includes everything that falls within the area of political activity and political process

History CAMBRIDGE VIEW ABOUT

THE PARTITION

AND REFUTATION

OF CAMBRIDGE

VIEW

Cambridge view about the Partition The Cambridge school of historians have interpreted that opposition to partition scheme was made entirely by the elitist groups They hold the view that Lord Curzon planned to partition the Bengal for administrative purposeREFUTATION OFCAMBRIDGE VIEW The Rationalist historians have rejected the interpretations of the Cambridge School of historians on various grounds

1 QUESTION State different views of historians regarding Partition of Bengal

ANSWER Cambridge historians believed that Lord Curzon partitioned Bengal for administrative reasons only and not for the political motive The Middle class elitist group protested because of their petty interest The Hindu zamindars protested as they have to spend more money for managing their estatesThe lawyers of Calcutta High court feared to lose their clientBut according to the nationalist Historians was-

2- The ultimate object of Lord Curzon was to crush the unity of Bengal politicians

3- If Bengal becomes a separate province Bengali speaking 16 million people of western part would become minority under Hindi speaking people of Bihar and Oriya speaking people of Orissa

4- The bureaucrats expected that the protest movement would die down quickly

5- Lord Curzon used the Muslim community in his political game

6- Idealism had great contribution in the protest against partition

7- The people of the every section of society were affected by the partition of Bengal

Computer Science

Numbers Convertion of dcimal number to octal numberThe decimal numeral system is the standard system for denoting integer and non-integer numbers It is the extension to non-integer numbers of the Hindu-Arabic numeral system For writing numbers the decimal system uses ten decimal digits a decimal mark and for negative numbers a minus sign - The decimal digits are 0 1 2 3 4 5 6 7 8 9 the decimal separator is the dot in many countries

The octal numeral system or oct for short is the base-8 number system and uses the digits 0 to 7 Octal is sometimes used in computing instead of hexadecimal perhaps most often in modern times in conjunction with file

permissions under Unix systems It has the advantage of not requiring any extra symbols as digits It is also used for digital displays

Follow these steps to convert a decimal number into octal form

1 Divide the decimal number by 82 Get the integer quotient for the next iteration (if the number will not divide equally by 8 then round down the

result to the nearest whole number)3 Keep a note of the remainder it should be between 0 and 74 Repeat the steps until the quotient is equal to 05 Write out all the remainders from bottom to top This is the solution

For example if the given decimal number is 8453

Division Quotient Remainder

8453 8 1056 5

1056 8 132 0

132 8 16 4

16 8 2 0

2 8 0 2

Then the octal solution is 20405

Subject Eng Literature (The Tempest ndash William Shakespeare) Topic Act I Scene 1 Lines 33 to 67 (End of scene) Date 16th April 2020 (4th Period)

[Students should read the original play and also the paraphrase given in the school prescribed textbook]Summary Questions amp Answers

[SUMMARY OF THE ENTIRE SCENE]

o The play starts with the scene of a severe storm at sea Alonso (King of Naples) Sebastian (Alonsorsquos brother) Ferdinand (Alonsorsquos son) Gonzalo Antonio (the usurping Duke of Milan) are in a ship in the midst of the storm

o The mariners are trying their best to control the vessel from running aground and are totally following the orders of their Master the Boatswain They have scant success

o The mariners become extremely unhappy and annoyed when most of the passengers arrive on the deck thereby hampering their effort to save the ship There is serious confrontation between them and the passengers who are part of the Kingrsquos entourage

o The mariners could not save the ship

SUMMING-UP

(i) Vivid description of the scene which gives a realistic description of terror and confusion of a tropical storm

(ii) Shows Shakespearersquos accuracy of knowledge in describing the naval operations and also matters of seamanship

(1) GONZALO Ill warrant him for drowning (L 45-57)

though the ship were no stronger than a nutshell and as leaky as an unstanched

wenchBOATSWAIN Lay her a-hold a-hold Set her two courses Off to

sea again lay her offMARINERS All lost To prayers to prayers All lostBOATSWAIN What must our mouths be coldGONZALO The king and prince at prayers Lets assist them

For our case is theirsSEBASTIAN Im out of patienceANTONIO We are merely cheated of our lives by drunkards

This wide-chopped rascal - would thou mightst lie drowning the washing of ten tides

(a) What does Antonio say at the insolent manners of the boatswain just before the given passage

Being irritated at the insolent manners of the boatswain just before the given extract Antonio the Duke of Milan calls him a worthless dog son of a woman without any morals an arrogant and disrespectful noisemaker He says that the boatswain deserved to be hanged(b) What statement does Gonzalo repeat about the boatswain

Gonzalo shows his faith that the boatswain is not destined to die by drowning He is destined to be hanged and nothing can alter this decree of destiny He says that even if the ship was as frail as a nutshell the boatswain could not be drowned for his destiny was to be hanged(c) What do the passengers do when they have lost all hope of their survival

When the passengers have lost all hope of survival they take

(iii) The opening scene justifies the title ndash The Tempest

UNANSWERED QUESTIONS

(i) The King always travels with his entire fleet including his soldiers Where were the other ships

(ii) Why was the ship in that area Where was it coming from or going where

(iii) The ship broke apart What happened to those who were in the ship

(We shall get the answer to the above questions as the play progresses)

leave of life with fervent prayers The mariners take their last hearty drink and are ready for death(d) What blame does Antonio put upon the mariners and the boatswain Antonio rebukes the mariners that these drunkards have brought them to the present crisis by neglecting their duties He blames them saying that they are going to lose their lives entirely for the negligence of the boatswain and his fellows(e) What does Antonio say while cursing the boatswain

Antonio gives vent to his wrath upon the boatswain in particular He calls the boatswain a wide-mouthed rascal who deserves to be hanged on the sea-shore at low water mark so that ten tides might wash over his body and take out of him all the liquor that he has been drinking

Class XIISubject Topic Summary ExecutionHistory Topic

1 1935 ACT AND WORKING OF PROVINCIAL AUTONOMYCONGREE AND OTHER MINISTERSSUB TOPIC GOVERNMENT OF INDIA ACT1935

Government of India Act 1935 This act established a lsquoFederation of Indiarsquo made of British Indian provinces and Indian states and provided for autonomy with a government responsible to the elected legislature in every provinceThis act introduced abolition of Diarchy at provinces The entire provincial administration was introduced to the responsible ministers who were controlled and removed by the provincial legislature The provincial autonomy means two things First The provincial governments were wholly responsible to the provincial legislature Secondly Provinces were free from outside control and interference in the large number of matters The act divided the powers between the centre and provinces in terms of three lists- Federal list( for centre) Provincial list (for province) and concurrent list (for both) Residuary powers were given to the viceroy In the election under the government of India Act the Congress swept the poll the mandate of the people came in favour of the congress so far as general Hindu seats were concerned The Congress did not get a single Muslim seates in Bombay CP UP Sind and BengalIn five provinces Congress had yhe clear majority In BengalNWFPAssam and Bombay Congress emerged as a single largest partyOn the other side the performance of the Muslim League was badThus the Congress formed ministers in 7 provinces out of 11 provinces Coalition ministry was also formed in two other provincesOnly BENGAL AND Punjab had non- congress ministries

1 QUESTION What was the main change introduced by the Government of India ActANSWER a) The Act gave more

autonomy to the provinces b) Diarchy was abolished at the

provincial levelsc) The Governor was the head of

the executived) There was a council of

ministers to advise him The ministers were responsible to the provincial legislatures who controlled them The legislature could also remove the ministers

e) The Governors still retained special reserve powers

2 QUESTION Why did the federal scheme introduced by the Government of India Act 1935 never come into operation

ANSWER The Federal structure of the Government of India was to be composed with the Governor General and Council of ministers The Federal legislature was to be Bicameral legislature- The council of states and the House of Assembly The ministers were to be chosen by the Governor general and they were to hold the office during his pleasure

The provinces of British India would have to join the federation but this was not compulsory for the princely states

This federation never materialised because of the lack of support from the required number of

princely statesThis act was refused and

rejected by the princes the Congress and the Muslim League

Thus both Congress and the League participated in the election of 1937 Thus the federal part was never introduced but the provincial part was put into operations

Bengali 2nd

Language

াচেরর পরাথCনা(কহিতা )

াচেরর পরাথCনা কহিতাটি কহি (ঙখ দেঘাচে4র দো আচো য কহিতায় াচেরর পতর হমায়ন কঠিন দেরাচেগ আxানত ার ঈশবর া আললার কাচেছ পরাথCনা কচেরচেছন তার পচেতরর ীন হিফহিরচেয় হিচেত এই কহিতায় ার পচেতরর ীন হিভbা দেচেয়চেছন ারার এমনহিক হিনচের ীন হিসCচেনর হিহিনমচেয় হিতহিন তার দেছচের ীন হিফচের দেপচেত দেচেয়চেছন তার দেছচের এই দেরাচেগর ন য হিতহিন হিনচেচেকই ায়ী কচেরচেছন তার হিনচের করা পাপচেকই হিতহিন ায়ী কচেরচেছন এছাা রানৈনহিতক ও আথCসামাহিক অসথার কথা তচে ধরা চেয়চেছ এই কহিতায় ার তার হিনচের পাপ কমCচেকই ায়ী কচেরচেছ ার অন যায় ভাচে দেপহি((হিকতর মাধ যচেম অপররা য কচেরচেছ আর এই অন যায় কাচের ন যই তার পহিরাচের হিপযCয় এচেসচেছ দে এক পরকার মানহিক নধন ইহিতাচেসর ার হিপতা চেয় সবাভাহিকভাচে ভাচোাসা দে মমতা দেথচেক মকত চেত পাচেরনহিন তাই হিপতা চেয় আললা া ভগাচেনর কাচেছ পতর হমায়চেনর পরানহিভbা দেচেয়চেছন ার আললা া ভগাচেনর কাচেছ াহিনচেয়চেছন তার হিনচের ীন হিসCন হিচেত হিতহিন রাী তার হিহিনমচেয় পচেতরর ীন হিফচের দেপচেত দেচেয়চেছন াচেরর হিপতসভ হিচেকর কথা এই কহিতায় ফটিচেয় দেতাা চেয়চেছ হিপতা পচেতরর হিরাহিরত মান নধচেনর কথা তচে ধরা চেয়চেছ

হিচে(4 হিকছ াইচেনর তাৎপযC১) ldquoদেকাথায় দেগ ওর সবচছয দেৌন দেকাথায় কচেরায় দেগাপন bয়ldquoউততর) াচেরর পতর হমায়ন কঠিন দেরাচেগ অসসথ তাই তার দেযৌন াহিরচেয় যাচেচছ এই দেরাচেগ তাচেক দেগাপচেন কচেরকচের াচেচছ তার সক (হিকত ধীচের ধীচের bয় চেচছ তাই হিপতা চেয় ার আললার কাচেছ হমায়চেনর পরান হিভbা দেচেয়চেছন২) ldquoাগাও (চেরর পরাচেনত পরানতচের ধসর (ন দেযর আান গানldquoউততর) াচেরর পতর হমায়ন কঠিন দেরাচেগ আxানত তাই ার আ দে(াচেক মমCাত (চেরর পচেথ পরানতচের আান গান ধবহিনত দোক দেসই আান গান আললার কাচেছ দেযন চে যায় আললা দেযন এই আহিতC শচেন পচেতরর ীন হিফহিরচেয় দেয় ৩)ldquoনাহিক এই (রীচেরর পাচেপর ীানচেত দেকানই তরারণ দেনই ভহি4চেতরldquoউততর) হমায়চেনর অসসথতার ন য ার হিনচেচেকই ায়ী কচেরচেছন কারন ার অচেনক রা য অন যায় ভাচে কচেরচেছ তাই তার এই পাপ কাচের ন য তার ঘচের আ হিপ এচেসচেছ এই অন যায় কাচের ন য তার মহিকত দেনই তাই ার আললার কাচেছ এই পাপ কাচেযCর ন য bমা পরাথM

Hindi 2ndlang

-ासी(जयशकर परसा-)

-ासी जयशकर परसा- की एक ऐसी कहानी ह जिजसम भारतीय ससकनित और राषटरीयता का सवरगजीतहोता ह इस कहानी म इरावती एक निहद कनया ह जिजस मलअचछो न मलतान की लट म पकडा और -ासी बना दि-या उस 500 दि-न -कर काशी क एक महाजन न खरी-ा दसरी -ासी निफरोजा ह वह गलाम ह निफरोजा को छडान क कतिलए अहम- को 1000 सोन क कतिसकक भजन थ जो अभी तक नही आए थ राजा साहब कठोर होत हए भी निफरोजा को निबना धनराकतिश क कतिलए उस म कर -त ह वनिफरोजा को अहम- को समझान की बात कहत हकहानी क अत म हम -खत ह निक इरा वती और जाटो क सर-ार बलराज का मिमलन होता हअहम- को यa म मार दि-या जाता ह वहा निफरोजा की परसननता की समामिध बनती ह वहा एक फल चढती ह और डीजल आती ह निफरोजा उस समामिध की आजीवन -ासी बनी रहती हलखक अपन उददशय अथात -ास परथा पर परकाश डालन और इस परथा क कारण होन वाल -ातो क दखो को दि-खान म पणता सफल हए ह

helliphellipContinue to next

Biology Reproductio Today we will discuss about vegetative Q1 Name some vegetative propagules

n in Organisms

propagation of plants The process of multiplication in which fragments of plant body function as propagule and develop into new individual is called vegetative propagation The units of such propagation are runner rhizome tuber bulb etc

and the speciesinvolvedVegetative propagules

Parts involved

Bulb StemBulbil BulbilRhizome Stem Runner Stem Tuber Stem Offset Stem Leaf buds Leaves Suckers Stem

Corns Stem stolon

Q2 State advantages of vegetative propagation

i) Rapid methodii) Sure and easy methodiii) Useful in plants that cannot

produce viable seeds or long seed dormancy

iv) Maintains purity of raceQ 3 Banana fruit is said to be parthenocarpic where as turkey is said to be parthenogenetic WhyBanana develops without fertilization from an unfertilized ovary thus is parthenocarpicIn turkey the ovum or female gamete developinto a new chick without fertilization thus isparthgenetic

Q4 Why is water hyacinth is called as a ldquoTerror of Bengalrdquo Water hyacinth can

propagatevegetatively all over the water body in a short per short period of time This resulted increased biochemicaloxygen oxygen demand of water body causing mortalityof fishes It is very difficult to get rid off them Thus known as terror of Bengal

Chemistry

Solid state GENERAL CHARACTERISTICS OF SOLID STATEIn nature the particular state of matter is governed by two opposing forces at given set of temperature and pressure These forces are intermolecular force of attraction and thermal energy If intermolecular force of attraction is high as compared to thermal energy particles remains in closest position

Intext QuestionsQ1 Classify the following solids as crystalline and amorphous Sodium chloride quartz glass quartz rubber polyvinyl chloride Teflon

A1 Crystalline

and hence very less movement in particles is observed In this case solid state is the preferred state of matter

Let us revise the general characteristics of solid

i) Fixed mass volume and shape

ii) Strong intermolecular force of attraction

iii) Least intermolecular space

iv) Fixed position of constituent particles

v) Incompressible and rigid

Q2 what type of interactions hold the molecules together in a polar molecular solid[CBSE 2010]A2 The molecules in a solid are held together by van der Waals forces The term van der Waals forces include hydrogen bonding dipole-dipole attraction and London dispersion forces All molecules experience London dispersion forces In addition polar molecules can also experience dipole-dipole interactions So the interactions that holds the molecule together in polar molecular solid are London dispersion force and dipole-dipole interactionsQ3 Write a feature that will distinguish a metallic solid from an ionic solid [CBSE 2010]A3 Metals are malleable and ductile whereas ionic solid are hard and brittle Metallic solid has typical metallic lustre But ionic solid looks dullQ4 Write a point of distinction between a metallic solid and an ionic solid other than metallic lustre [CBSE 2012]A4 Metals are malleable and ductile whereas ionic solid are hard and brittleQ5 Write a distinguish feature of metallic solid [CBSE 2010]A5 The force of attraction in

solid Sodium chloride Quartz Amorphous solid Quartz glass rubber polyvinyl chloride Teflon Q2 why glass is considered as super cooled liquidA2 Glass shows the tendency to flow at slower rate like liquid Hence they considered as super cooled liquidQ3 why the window glass of old buildings show milky appearance with timeA3 Glass is an amorphous solid Amorphous solid has the tendency to develop some crystalline character on heating Due to heating in day over the number of years glass acquires some crystalline character and show milky appearanceQ4 why the glass panes fixed to window or doors of old building become slightly thicker at bottomA4 Glass is super cooled liquid It has the tendency to flow down very slowly Due to this glass pane becomes thicker at the bottom over the timeQ5 Sodium chloride is a crystalline solid It shows the same value of refractive index along all the direction TrueFalse Give reasonA5 FalseCrystalline solid shows anisotropy in properties That is it shows different values for the given physical property in different direction All the crystalline solids show anisotropy in refractive index Therefore sodium chloride will show different values of refractive index on different directions

Q6 Crystalline solid are anisotropic in nature What does this statement means

between the constituent particles is special kind of electrostatic attraction That is the attraction of positively charged kernel with sea of delocalized electronsQ6 which group of solid is electrical conductor as well as malleable and ductile [CBSE 2013]A6 Metallic solidQ7 why graphite is good conductor of electricity although it is a network (covalent solid)A7 The exceptional property of graphite is due to its typical structure In graphite each carbon is covalently bonded with 3 atoms in same layer The fourth valence electron of each atom is free to move in between different layersThis free electron makes the graphite a good conductor of electricity

[CBSE 2011]A6 Anisotropy is defined asrdquo Difference in properties when measured along different axis or from different directionsrdquo Crystalline solid show different values of some of the physical properties like electrical resistance refractive index etcwhen measured along the different directions The anisotropy in crystalline solid arises due to the different arrangement of particles in different directions

Math Function Composition of functions Think of an industrial plant that produce bottles of cold drinks first there is the operation (or function) f that puts the cold drink inside the bottle followed by the opeartion g that close the bottle with the capThis leads to the following definitionDefinition Let f A rarr B and g B rarr C be two functions Then the composition of f and g denoted by gof is defined as the function gof A rarr C given by gof(x) = g(f (x)) forall x isinA

Definition A function f X rarr Y is defined to be invertible if there exists a function g Y rarr X such that gof = IX and fog = IY The function g is called the inverse of f and is denoted by f -1

Thus if f is invertible then f must be one-one and onto and conversely if f is one-one and onto then f must be invertible This fact significantly helps for proving a function f to be invertible by showing that f is one-one and onto specially when the actual inverse of f is not to be determined

Example 1 Let f 2 3 4 5 rarr 3 4 5 9 and g 3 4 5 9 rarr 7 11 15 be functions defined as f(2) = 3 f(3) = 4 f(4) = f(5) = 5 and g (3) = g (4) = 7 and g (5) = g (9) = 11 Find gofSolution We have gof(2) = g (f(2)) = g (3) = 7 gof(3) = g (f(3)) = g (4) = 7gof(4) = g (f(4)) = g (5) = 11 and gof(5) = g (5) = 11Example 2 Find gof and fog if f R rarr R and g R rarr R are given by f(x) = cos x and g (x) = 3x2 Show that gof ne fogSolution We have gof(x) = g(f(x))=g(cosx) = 3 (cos x)2

= 3 cos2 x Similarly fog(x)=f(g (x))= f(3x2)= cos (3x2) Note that 3cos2 x ne cos 3x2 for x = 0 Hence gof ne fogExample 3 Show that if f A rarr B and g B rarr C are onto then gof A rarr C is also ontoSolution Given an arbitrary element z isin C there exists a pre-image y of z under g such that g (y) = z since g is onto Further for y isin B there exists an element x in A with f(x) = y since f is onto Therefore gof(x) = g (f(x)) = g (y) = z showing that gof is onto Example 4 Let Y = n2 n isin N sub N Consider f N rarr Y as f(n) = n2 Show that

f is invertible Find the inverse of fSolution An arbitrary element y in Y is of the form n2 for some n isin N This implies that n =radicy This gives a function g Y rarr N defined by g (y) =radicy Nowgof (n) = g (n2)=radicn2 = n and fog (y) =f(radicy) = (radicy) 2 y which shows that gof=IN and fog= IY Hence f is invertible with f -1 = g

Political Science

Constitution of India-The Preamble

Summary

Objective of the state-To secure equality of status and of opportunity To promote fraternity among all the citizens To assure the dignity of the individuals and Unity and integrity of the nation

Justice-Justice stands for rule of law absence of arbitrariness and a system of equal rights freedom and opportunities for all in a society India seeks social economic and political justice to ensure equality to its citizens

Liberty-Liberty implies the absence of restraints or domination on the activities of an individual such as freedom from slavery serfdom imprisonment despotism etc The Preamble provides for the liberty of thought expression belief faith and worship

Equality-Equality means the absence of privileges or discrimination against any section of the society The Preamble provides for equality of status and opportunity to all the people of the country

Fraternity-The Preamble declares that fraternity has to assure two thingsmdashthe dignity of the individual and the unity and

Execution

Answer the following questions-

Short notes-1 Equality2 Fraternity3 Justice4 Liberty

Homework-Learn

integrity of the nation The word integrity has been added to the Preamble by the 42nd Constitutional Amendment (1976)

Business studies

Human resource management (chapter 1)

On the day of 1504 2020 I have discussed with you the managerial functions and procurement functions of HRM

Today weare going to discuss about the development function integration functions and maintenance function

Development functions-HRM improves the knowledge skills attitude and values of employees so that they the present and future jobs more effectively it includes

1) Development functions of HRM

a) Performance appraisal = It implies systematic evaluation of employees with respect to their performance on the job and their potential for development

b) Training =It is the process by which employees learn knowledge skills and attitudes to achieve organisational and personal goals

c) Executive development = It is the process of developing managerial talent through appropriate program

2) Integration functionsa) HRM reconcile the goals of

organisation with those of its members through integrating function

b) HRM tries to motivate employees to various financial and non financial incentives provided in job specification etc

3) Maintenance functiona) HRM promote and protect the

physical and mental health of employees by providing several types of benefits like housing medical aid etc

b) It Promote Social security measures to employees by providing provident fund pension gratuity maternity benefits

SubjectCOMMERCE

Topic

BUSINESSENVIRONMENT

Summary

Now quickly let us revise the earlier points that we have already done in the last class and let us proceed with the other topics that are there in the chapter

Firstly we will recall the internal and external factors of micro environment and then we

Execution 3 What do you mean by internal factors

in micro environmentAnswerInternal factors refer to all the factors existing within a business firm The internal factors are considered controllable because the enterprise has control over these factors

Development FunctionsPerformance AppraisalTrainingExecution Development

shall proceed in details

Meaning and list of internal and external factors

aInternal factorsInternal factors refer to all the factors existing within a business firm The internal factors are considered controllable because the enterprise has control over these factorsFor an example a company can alter its organization structure policies programmes employees physical facilities and marketing mix to suit the changes in the environmentList of internal factors areCorporate culture mission and objectives top management organizations structure company image and brand equity company resources

b External factorsExternal factors refer to those individual and groups and agencies with which a particular business organization comes into direct and frequent contact in the course of its functioningThese individuals and groups are known as STAKEHOLDERS because they have a stake (financial interest ) in the working and performance of the particular business List of external forces (stakeholders)Customers competitors investors suppliersmiddlemen (marketing intermediaries)financers publics

customers

suppliersfinancers

For an example a company can alter its organization structure policies programmes employees physical facilities and marketing mix to suit the changes in the environment

4 What do you mean by external factors in micro environment

AnswerExternal factors refer to those individual and groups and agencies with which a particular business organization comes into direct and frequent contact in the course of its functioningThese individuals and groups are known as STAKEHOLDERS because they have a stake (financial interest) in the working and performance of the particular business

3Who are stakeholdersSTAKEHOLDERS are individuals and groups who have a stake (financial interest ) in the working and performance of the particular business 4Discuss the internal factors in briefa Corporate CultureThe values beliefs and attitudes of the founders and top management of the company exercise a strong influence on what the cmpaany stands for how it does things and what it considers importantbMission and objectivesThe business philosophy and purpose of a comoany guide it prioritiesbusiness strategiesproduct market scope and development scope

cTop management structurethe composition of board of directors the degree of professionalization of management and the organizational structure of a company have important bearing on its business decisions

dPower structureThe internal power relationship between the board of directors and the chief executive is an important factor

e Company image and brand equityThe image and brand equity of the company play a significant role in raising finance forming alliance choosing dealers and suppliers launching new products entering foreign markets

5 What is Macro environmentAnswerMacro environment refers to the general

competitors

middlemen

publics

Fig STAKEHOLDERS OF A COMPANY

Apart from micro environment the other main dimension of business environment isMacro environment Macro environment refers to the general environment or remote environment within which a business firm and forces in its micro environment operateA company does not directly or regularly interact with the micro environmentTherefore macro environment is also known as indirect action EnvironmentThe macro environment forces are less controllable than the micro forces

Macro environment consists of the following components

POLITICAL AND LEGAL ENVIRONMENT

ECONOMIC SOCIAL AND ENVIRONMENT

CULTURAL

ENVIRONMENT

TECHNOLOGICAL ENVIRONMENT

Fig COMPONENTS OF MACRO ENVIRONMENT

environment or remote environment within which a business firm and forces in its micro environment operateA company does not directly or regularly interact with the micro environmentTherefore macro environment is also known as indirect action EnvironmentThe macro environment forces are less controllable than the micro forces 6 What are the components of macro environmenta Political and legal environmentb Economic environmentc Social and cultural environmentd Technological environment

Computer Science

Logic gates

Digital systems are said to be constructed by using logic gates These gates are the AND OR NOT NAND NOR EXOR and EXNOR

BUSINESS FIRM

gates The basic operations are described below with the aid of truth tables

AND gate

The AND gate is an electronic circuit that gives a high output (1) only if all its inputs are high A dot () is used to show the AND operation ie AB Bear in mind that this dot is sometimes omitted ie ABOR gate

The OR gate is an electronic circuit that gives a high output (1) if one or more of its inputs are high A plus (+) is used to show the OR operationNOT gate

The NOT gate is an electronic circuit that produces an inverted version of the input at its output It is also known as an inverter If the input variable is A the inverted output is known as NOT A This is also shown as A or A with a bar over the top as shown at the outputs The diagrams below show two ways that the NAND logic gate can be configured to produce a NOT gate It can also be done using NOR logic gates in the same way

NAND gate

This is a NOT-AND gate which is equal to an AND gate followed by a NOT gate The outputs of all NAND gates are high if any of the inputs are low The symbol is an AND gate with a small circle on the output The small circle represents inversion

NOR gate

This is a NOT-OR gate which is equal to an OR gate followed by a NOT gate The outputs of all NOR gates are low if any of the inputs are highThe symbol is an OR gate with a small circle on the output The small circle represents inversion

EXOR gate

The Exclusive-OR gate is a circuit which will give a high output if either but not both of its two inputs are high An encircled plus sign ( ) is used to show the EOR operation

EXNOR gate

The Exclusive-NOR gate circuit does the opposite to the EOR gate It will give a low output if either but not both of its two inputs are high The symbol is an EXOR gate with a small circle on the output The small circle represents inversion The NAND and NOR gates are called universal functions since with either one the AND and OR functions and NOT can be generated

Note A function in sum of products form can be implemented using NAND gates by replacing all AND and OR gates by NAND gates A function in product of sums form can be implemented using NOR gates by replacing all AND and OR gates by NOR gates

Logic gate symbols

Table 2 is a summary truth table of the inputoutput combinations for the NOT gate together with all possible inputoutput combinations for the other gate functions Also note that a truth table with n inputs has 2n rows You can compare the outputs of different gates

Logic gates representation using the Truth table

Example

A NAND gate can be used as a NOT gate using either of the following wiring configurations

Subject Eng Literature (The Tempest ndash William Shakespeare) Topic Act III Scene 3 Lines 53 to 110 (End of the scene) Date 16th April 2020 (2nd Period)

[Students should read the original play and also the paraphrase given in the school prescribed textbook]Summary Questions amp Answers

o Seeing this strange scene all are inclined to believe the tales told by travelers that there truly are ldquounicornsrdquo and ldquothe phoenixrsquo thronerdquo

o As they are about to sit down to the feast the banquet is snatched away by a harpy (Ariel disguised) A spiritrsquos voice (Arielrsquos voice) denounces Alonso Sebastian and Antonio with particular

1 ARIEL You are three men of sin whom Destiny

(Line 53-58)That hath to instrument this

lower world And what is int the never-surfeited sea

Hath caused to belch up you and on this island

Where man doth not inhabit you rsquomongst men

Being most unfit to live I have made you mad

reference to their crime in expelling Prospero from Milan They have not received any punishment for their deed earlier but the time for their punishment has arrived Upon Alonso it pronounces ldquolingering perdition worse than deathrdquo from which there is no remedy except through sincere repentance Ariel then vanishes in thunder and the shapes enter again and carry away the table

o Prospero watching invisibly is very pleased with the performance of Ariel and his (Prosperorsquos) ldquomeaner ministersrdquo All his enemies are now in his power and are in a fit of desperation He then leaves them and goes to see how Ferdinand and Miranda are getting on

o Alonso is now much humbled and penitent with the after effect of the spiritrsquos denunciation of his crimes He believes that his son is lost forever After this all disperse being stricken mad by the speech of the spirit

o Gonzalo fearing that they may do violence to themselves or to one another follows them and bid others to follow

(a) To whom does Ariel disguised as a harpy call the three sinners What game did Fate of Destiny play with

them

The three sinners called by Ariel are Alonso Sebastian and Antonio It was Destiny which had caused the ocean to cast the three sinners on the shore Though the ocean is all the time devouring whatever appears on its surface and is never satisfied with its continual swallowing of the ships and men in the present case the ocean had cast these three sinners on the shore without killing them

(b) Who had jointly been responsible for the conspiracy against Prospero What is Prosperorsquos purpose behind all this

Three men Alonso Sebastian and Antonio had jointly

been responsible for the conspiracy against Prospero They had driven out Prospero form Milan Prosperorsquos purpose is to make these three sinners realize the wrong they had done He wants them to repent for their criminal deeds because repentance leads to self-esteem(c )What does Ariel (the harpy) tell Alonso and his companions when they take out their swords to attack him

Seeing them drawing their swords Ariel (harpy) tells them that he and his companions are the instruments of destiny and that it is not possible for human beings to do them any injury He says that the swords of human beings can not injure even a minute part of his feathers Their swords are as ineffective against him and his companions as against the wind or the water

(d) Give the explanatory meanings of the following expressions in the context of the above extract

(i)Never surfeited (ii) Belch up (iii) lsquomongst men

(i) Never surfeited never led to satisfaction

(ii) Belch up cast ashore(iii) lsquomongst men in human

society2

I and my fellows (Line 60-65)

Are ministers of Fate The elementsOf whom your swords are tempered may as wellWound the loud winds or with bemocked-at stabsKill the still-closing waters as diminishOne dowl thats in my plume

IMPORTANT PASSAGES EXPLAINED

The elements

(Line 61-66)Of whom your swords are tempered may

as wellWound the loud winds or with

bemocked-at stabs

(a) Who is lsquoIrsquo Who are his lsquofellowsrdquo

lsquoIrsquo is referred to Ariel in disguise of a harpy His lsquofellowsrsquo are other spirits serving Prospero the real Duke of Milan who has acquired supernatural powers after being banished from his Dukedom Prospero has settled in this uninhabited island

(b) What are the elements that have temperrsquod the swords Why will it not work against the speaker

The swords (of Alonso and his companions) are tempered by metal (steel) which is taken out of the earth and refined by

Kill the still-closing waters as diminishOne dowl thats in my plume My fellow

ministersAre like invulnerable

In these words Ariel reminds the King and his companions of the utter futility of drawing swords against himself and his fellows Ariel drives Alonso Antonio and Sebastian the three men of sin to desperation ndash a state in which men do violence to themselves They draw swords to strike Ariel But Ariel reminds them that he and the other spirits are the ministers of destiny and nothing can wound them The steel of which their swords are made of may cut the wind or water which being divided always closes up again Even supposing that such things may be possible it is quite impossible that their swords will cut one feather in their plume They are incapable of being wounded by any sword of man Hence it is foolish on their part to attempt to strike at Ariel and his fellow-spirits

For which foul deed

(Line 72-75)The powers delaying not forgetting

haveIncensed the seas and shores yea all the

creatures Against your peace

Ariel enters like a harpy and remaining invisible tells Alonso Sebastian and Antonio that he and other harpies are the agents of Destiny appointed to carry out her decrees He tells them that their punishment for the crime against Prospero which has been so long deferred is now to fall upon them He reminds them that they had expelled Prospero from Milan and set him and his innocent child adrift on the sea and that the sea had paid them back for their sin by the shipwreck and by the calamities they have suffered He tells them that the powers above which did not forget this mean treachery but only deferred the punishment have now engaged the seas and the shores and all living beings including him and his comrades against them The very elements and supernatural agency Ariel adds have taken up the avenging of their crime against Prospero

the action of fire It may cut the wind or water which being divided always closes up again

The sword will not work against the spirits and the harpy because they are the ministers of destiny and nothing can wound them nor it will cut a single feather in their plume

(c )What is the meaning of lsquodowlrsquo in the last line

The term lsquodowlrsquo means a filament or the smallest part of a feather In this context Ariel in disguise of harpy says that their sword cannot even damage the smallest filament of their (Arielrsquos and other spirits) feathers as they are incapable of being wounded by any sword of man

(d) What does the speaker remind the listeners about

Ariel in disguise of harpy reminds Alonso the King of Naples Sebastian Alonsorsquos brother and Antonio the present Duke of Milan and the treacherous brother of Prospero as they being three men of sin He even reminds them that their punishment for their crime against Prospero which has been so long deferred now falls upon them He reminds them that they have expelled Prospero from Milan and has set him along with his innocent infant daughter adrift on the sea So the sea has paid them back for their sin by their shipwreck and the calamities they have suffered since then The harpy rebukes Alonso of his sin that has incensed the Gods and has deprived him of his son as a punishment

(e) How do they respond

When Ariel in disguise of a harpy reminds Alonso Sebastian and Antonio of their past misdeeds and sin Alonso has a look of terror and confusion in his eyes He utters the words of sincere repentance wrung out of his conscience-stricken heart It appears to him that all the elements of nature the sea-waves the wind and the thunder proclaiming a loud voice in the name of Prospero and the crime Alonso has committed against him They are calling upon him to repent There is a deep storm raging in Alonsorsquos breast and the echoes of that storm are ringing in his ears like a clear note of wind-instrument A note of denunciation of Alonsorsquos crime leaves him much humbled and penitent and confirms his belief that his son is lost forever But Sebastian and Antonio shows some courage instead of repentance They wish to kill the spirits or devils if it appears

3

Of my instruction hast thou nothing bated (Line 85-93)

In what thou hast to say So with good life

And observation strange my meaner ministers

Their several kinds have done My high charms work

And these mine enemies are all knit upIn their distractions They now are in my

powerAnd in these fits I leave them while I visitYoung Ferdinand whom they suppose is

drownedAnd his and mine loved darling

Methought the billows spoke and (Line 96-99)

told me of itThe winds did sing it to me and the

thunderThat deep and dreadful organ-pipe

pronouncedThe name of Prosper It did bass my

trespass

These are the words of contrition coming from Alonso Ariel has driven him to a deep repentance for conspiring with Antonio against Prospero He now feels a sincere remorse It appears to him that all the elements of nature the sea-waves the wind and the thunder proclaimed with a loud voice the name of Prospero and the crime Alonso had committed against him They are calling upon him to repent There is a deep storm raging in Alonsorsquos breast and the echoes of that storm are ringing in his ears like the clear note of a wind-instrument

Comment These are the words of sincere repentance wrung out of the conscience-stricken heart of Alonso Alonso who is the lesser villain is the first to give way to remorse under the effect of Arielrsquos speech The words of Ariel seem to him to be the voice of conscience speaking to him He is driven to desperation a state in which he might do violence to his life

(a) Identify the speaker State the context

Prospero the ruler of the island is the speaker The famous banquet scene has been enacted very well Ariel and his junior spirits have played their roles excellently Prospero is glad to say words of praise for them(b) In what way the speakerrsquos instructions have been carried out

According to Prosperorsquos instructions a banquet was presented before the King of Naples and his companions when they were tired and hungry Just when they were preparing to eat the feast the banquet was suddenly removed by exercising supernatural powers All this was done by Ariel Prosperorsquos chief assistant and a powerful spirit

Ariel not only made the feast disappear but also delivered his speech blaming the King and his two companions for their past wicked deeds He warned them to repent for their misdeeds or suffer forever on that uninhabited island

(c) Who are referred to as lsquomeaner ministersrsquo What have they done

Prospero refers as lsquomeaner ministersrsquo to his other lesser spirits who were assisting Ariel in presenting a scene before the kingrsquos party They entered the scene to the accompaniment of music They assumed several strange shapes and brought in a banquet Then they danced about it with gentle actions of salutations thus inviting the King and others to eat the feast

These spirits play their role again when Ariel in the shape of a harpy quits the scene These shapes enter again and dancing with mocking gestures carry away the table

(d) Who are the speakerrsquos enemies What has happened to them

King of Naples Alonso his brother Sebastian and the present Duke of Milan Antonio (Prosperorsquos own brother) are Prosperorsquos enemies With the turn of events they have all been washed ashore on the island which is ruled by Prospero the great magician Actually this happened after the shipwreck caused by a storm which was raised by Prospero with the purpose of bringing these people to his island Prosperorsquos spirits have already confused and terrified these enemies and they are under Prosperorsquos control He can treat them as he likes

(e) What does he say about Ferdinand Explain what is meant by ldquohellip his and mine darlingrdquo

Prospero knows that Alonsorsquos son prince Ferdinand is alive though his father thinks that the prince has been drowned

Prospero refers to his daughter Miranda who is dear to him She is also very dear to Prince Ferdinand who has fallen in love with her They are waiting to be married soon for which they have received Prosperorsquos consent

4

ALONSO O it is monstrous monstrous (Line 95-102)

Methought the billows spoke and told me of it

The winds did sing it to me and the thunderThat deep and dreadful organ-

pipe pronouncedThe name of Prosper It did bass

my trespassTherefore my son ithrsquo ooze is

bedded andIll seek him deeper than eer

plummet soundedAnd with him there lie mudded

(a) In what way does Alonso express his horror when his conscience is awakened by Arielrsquos words

When Alonsorsquos conscience is awakened by Arielrsquos words he expresses his horror at what he has heard He gets the feeling that the waves of the ocean the wind and the loud thunder have spoken to him and uttered the name of Prospero Because of being reminded of his crime in a very loud and rough voice he comes to realize that he has lost his son for his past misdeeds

(b) What does Alonso imagine about his son What does Alonso want to do in his desperate state

Alonso imagines that his son is lying in the mud at the bottom of the sea He feels desperate that he wants to drown himself in the ocean deeper than the plumb-line has ever gone He wants to lie with his son at the bottom of the sea

(c) How do Sebastian and Antonio want to face the evil spirits

Sebastian says that he is not at all afraid of what the harpy has said and that he is prepared to fight any number of such monsters if they appear before him only one at a time Antonio says that he would support Sebastian in the fight against the fiendsyyy

(d) Why does Gonzalo ask Adrian to follow the three men

Gonzalo tells Adrian that all the three men namely Alonso Sebastian and Antonio are in a wild and reckless mood The thought of the heinous crime of which they are guilty has begun to torment their minds So he asks Adrian to follow those three men without loss of time and prevent them from doing anything which the turmoil in their minds might lead them to do

(e) What opinion do you form of Alonso from the above extract

Alonso who is the lesser villain is the first to give way to remorse under the effect of Arielrsquos speech The words of Ariel seem to him to be the voice of conscience speaking to him He is driven to desperation a state in which he might do violence to his life

Subject =Accounts

Ac-12 15420 topic-pL Appropriation ac

PROFIT AND LOSS APPROPRIATION ACCOUNT

MEANING AND PREPARATIONProfit and Loss Appropriation Account is merely an extension of the Profit and Loss Account of the firm The profit of the firm has to be distributed amongst the partners in their respective profit sharing ratio But before its distribution it needs to be adjusted All Adjustments like partnerrsquos salary partnerrsquos commission interest on capital interest on drawings etc are made in this account These adjustments will reduce the amount of profit for distribution This adjusted profit will be distributed amongst the partners in their profit sharing ratio To prepare it at first the balance of Profit and Loss Account is transferred to this account The journal entries for the preparation of Profit and Loss Appropriation Account are given below

1 for transfer of the balance of Profit and Loss Account to Profit and Loss Appropriation Account

(a) In case of Net Profit

Profit and Loss Ac helliphelliphelliphelliphellipDrTo Profit and Loss Appropriation Ac(Net Profit transferred to Profit and Loss Appropriation Ac)

(b)In case of Net Loss

Profit and Loss Appropriation Achelliphelliphellip DrTo Profit and Loss Ac(Net Loss transferred to Profit and Loss Appropriation Ac)

2 for Interest on Capital

For transferring on Interest on CapitalProfit and Loss Appropriation Achelliphelliphellip DrTo Interest on Capital Ac(Interest on capital transferred to Profit amp Loss Appropriation Ac)

3 for Interest on Drawings

For transferring Interest on Drawings Interest on Drawings Achelliphelliphelliphelliphelliphellip DrTo Profit and Loss Appropriation Ac(Interest on drawing transferred to Profit amp Loss Appropriation Ac)

4 For Partnerrsquos SalaryFor transfer of partnerrsquos SalaryProfit and Loss Appropriation Achelliphellip DrTo Salary Ac(Salary transferred to profit amp Loss Appropriation Ac)

5 For Partnerrsquos CommissionFor transferring commissionProfit and Loss Appropriation Achelliphelliphellip DrTo Commission Ac(Commission transferred to Profit and Loss Appropriation Ac)

6 For Transfer of agreed amount to General ReserveProfit and Loss Appropriation Ac helliphellipDrTo General Reserve Ac(Transfer to General Reserve)

7 for share of Profit or Loss appropriation(a) If ProfitProfit and Loss Appropriation Achelliphellip DrTo Partnerrsquos CapitalCurrent Ac(Profit transferred to capitalcurrent Ac)(b) If LossPartnerrsquos Capital Current Achelliphelliphelliphellip DrTo Profit and Loss Appropriation Ac(Loss transferred to capitalcurrent Ac)

THE FORMAT OF PROFIT AND LOSS APPROPRIATION

Profit and Loss Appropriation Account for the year endedhelliphelliphelliphellip

Particulars Amount Particulars Amount

To PL Ac (loss) By pL Ac (profit)

To Interest on capital BY Interest on drawings

To partner`s commission by Partner`s capital Ac ( loss)

To Partner`s salary To Interest on partner`s loan To General Reserve To Partner`s Capital AC (Profit)

Subject= Economics

MOVEMENT ALONG THE DEMAND CURVE (CHANGE IN QUANTITY DEMANDED)In law of demand you have already studied the inverse relationship between price and quantity demanded When quantity demanded of a commodity changes due to change in its price keeping other factors constant it is called change in quantity demanded It is graphically expressed as a movement along the same demand curve There can be either a downward movement or an upward movement along the same demand curve Upward movement along the same demand curve is called contraction of demand or decrease in quantity demanded and downward movement along the same demand curve is known as expansion of demand or increase in quantity demanded

Extention of demandd

price (rs)p A

B Extentionp1 d

Q Q1

Quantity demanded ( in units)

Contraction of demandd

p2 Ccontraction

p APrice (Rs)

d

Q2 Q

Quantity demanded (in units)

Explanation of movement of demand A fall in price from OP to OP1 leads to increase in quantity demanded from OQ to OQ1 (expansion of demand) resulting in a downward movement from point A to point B along the same demand curve DD When Price rises from OP to OP2 quantity demanded falls from OQ to OQ2 (contraction of demand) leading to an upward movement from point A to point C along the same demand curve DD

  • Activity Series of Metals
    • Drawbacks of Rutherfordrsquos model of atom
      • Electromagnetic radiations
      • Properties of electromagnetic radiations
      • Characteristics of electromagnetic radiations
        • Plancks Quantum Theory-
        • Photoelectric effect
          • Intext Questions
            • Logic gates
            • Digital systems are said to be constructed by using logic gates These gates are the AND OR NOT NAND NOR EXOR and EXNOR gates The basic operations are described below with the aid of truth tables
            • AND gate
            • Example
Page 19:  · Web viewSubject. Topic. Summary. Execution. English 1 . Chapter 1 naming words . Page 8. Write the names of these pictures:- Person:-1. father. 2.Firefighter 3.doctor 4 ...

3) Explain the types of account4) Define the following terms

a) Assetsb) Capitalc) Purchased) Debtorse) Transactions

5) Name the types of accounts given below

a) Krishnas accountb) Machinery accountc) Royalty accountd) Salary accounte) Furniture accountf) Audit fee account

Economics Revision Today I will give you some revision questions

Questions1) What do you mean by the terms

rdquowantsrdquo2) Write the difference between

consumer goods and producer goods

3) Define the term utility 4) Explain the different types of utility5) Define

a) Total utilityb) Marginal utility

Subject Eng Literature (The Merchant of Venice ndash William Shakespeare)Topic Act I Scene 3 Lines 1 to 48 (Shylock hellip Cursed be my tribe if I forgive him) Date 16th April 2020 (5th Period)

[Students should read the original play and also the paraphrase given in the school prescribed textbook]Summary Questions amp Answers

This scene takes place in Venice and we are introduced to the rich Jew Shylock Bassanio and Shylock are talking and Bassanio tells Shylock that he wants a loan of three thousand ducats for three months on the personal security of Antonio

o Shylock feels glad because he will be able to bind down Antonio by means of a bond on account of the loan but he tells Bassanio that all the fortunes of Antonio being invested in the merchant ships on the sea it is difficult to depend upon his credit Even under such circumstances Shylock is willing to advance the money on the personal security of Antonio

o Bassanio then invites Shylock to dine with him Shylock says that he is prepared to do anything with the Christians but not eat or drink or pray with them

o While Bassanio and Shylock are talking Antonio appears on the scene Shylock does not seem to take any notice of Antonio but goes on brooding within

(1) SHYLOCK Ho no no no no- my meaning in (Line 15-26)saying he is a good man is to have you understand me that he is sufficient Yet his means are in suppositionhe hath an argosy bound to Tripolis another to the Indies I understand moreover upon the Rialto he hath a third at Mexico a fourth for England and other ventures he hath squanderd abroad Butships are but boards sailors but men there be land-rats and water-rats land-thieves and water-thieves I mean pirates and then there is the peril of waters winds and rocks The man is notwithstanding sufficientmdashthree thousand ducats mdashI think I may take his bond

(a) Who is talking in the beginning of this scene What does Bassanio want from Shylock How does Shylock feel

In the beginning of the scene Bassanio and Shylock are talking to each other Bassanio wants to borrow three thousand ducats from Shylock for three months on the security of Antonio Shylock feels glad at heart that he will get the opportunity of binding Antonio with a bond(b) What risks does Shylock weigh in advancing the money

Shylock says that Antonio has invested all his capital in trading by sea-going ships But the ships are made of wood and the sailors of those ships are ordinary human beings The wood can

himself how he hates Antonio because of his being a Christian because he abuses Shylock in public places Shylock decides that if ever he can get Antonio to his advantage he will teach him a lesson

come to harm and men can commit mistakes and thus the capital invested in ships may be lost Then there are other dangers The goods loaded on the ships can be damaged by rats and thieves which are found both on land and water The ships can also be harmed through sea-storms submerged rocks etc(c) What two important functions does this scene have

The scene has two important functions First it completes the exposition of the two major plot lines of the play Antonio agrees to Shylockrsquos bond ndash three thousand ducats for a pound of flesh and second and more important dramatically this scene introduces Shylock himself In this scene Shakespeare makes it clear at once why Shylock is the most powerful dramatic figure in the play and why so many great actors have regarded this part as one of the most rewarding roles in all Shakespearean dramas(d) Where does this scene take place What kind of treatment has Antonio been giving to Shylock What does Shylock say when Bassanio invites him to dine with him

The action of this scene takes place in Venice Antonio has been in the habit of behaving harshly with Shylock ndash spitting on his beard and footing him like a stranger cur When Bassanio invites Shylock to dine Shylock says that he is prepared to do anything with the Christians but not eat and drink or pray with them

(2) SHYLOCK How like a fawning publican he looks (Line 38-48)I hate him for he is a Christian

But more for that in low simplicity

He lends out money gratis and brings downThe rate of usance here with us in VeniceIf I can catch him once upon the hipI will feed fat the ancient grudge I bear him

He hates our sacred nation and he railsEven there where merchants most do congregateOn me my bargains and my well-won thriftWhich he calls interest Cursed be my tribeIf I forgive him

(a) What is the context in which these words are spoken and what is the idea expressed in it

These remarks are made by Shylock when he sees Antonio coming along after Bassanio told him that the merchant will be his surety for the bond The above mentioned passage reveals Shylockrsquos hatred for Antonio Shylock says that he hates Antonio because he is a Christian and also because he gives loan without taking interest on them thereby bringing down the rate of interest in Venice(b) Explain the meaning of the phrase lsquoa fawning publicanrsquo

The phrase lsquoa fawning publicanrsquo refers to Roman tax collector It is a term of contempt and hatred on the lips of a Jew lsquoFawning Publicansrsquo were Roman tax-gatherers whose ordinary bearings towards the Jews was bullying but whose false pose of lsquohumility and contritionrsquo is touched upon in the parable in New Testament(c ) What light does the above passage throw on the character

of Shylock

The above mentioned speech of Shylock reveals him to be a wicked character having an extreme greed for wealth His intense hatred for Antonio is unjustified He hates Antonio only because he is a Christian and because he lends money without taking any interest on it thereby adversely affecting Shylockrsquos business of lending money on high interest(d) What information do you gather about Antonio from the above given lines

Shylockrsquos statement throws a valuable light on the character of Antonio Antonio appears to be a good Christian and a good human being He helps the people in need by lending them money without charging any interest on it He is a man of simple and good nature This very goodness makes him Shylockrsquos enemy(e) What does Shylock debate within himself and when To whom are the lines mentioned above addressed to

When Bassanio asks the Jew to lend him three thousand ducats on Antoniorsquos surety Shylock begins to debate within himself as to how he should exploit the opportunity of a business deal with his old enemy Antonio

The lines mentioned above are not addressed to anyone The lines are a soliloquy ie a speech made by a character to himself and not meant to be heard by the other characters present

Class XSubject Topic Summary ExecutionEnglish

LiteratureThe Blue Bead 2nd part

Things took a turn for the worst and all of a sudden a crocodile attacked the woman biting on the womanrsquos leg At that moment Sibia got up sprinted grabbed the hay fork and stabbed the crocodile in the eye with all her power Immediately the crocodile let go and went away Sibia saw a small blue bead lying by the river she grabbed it Since she was poor she didnrsquot have necklace Shersquod always wanted one like the other women now she could make one with the blue bead After that she went home and told her mother all about it

Hindi 2nd

Langबड घर की बटी( मशी परमच-)

lsquoबड घर की बटी कहानी का उददशय मधयम वग की घरल समसया को सलझा कर सगदिठत परिरवार म मिमल जलकर परम स रहन का स-श -ना हघर म शानित सथानिपत करन की जिजमम-ारी नारी की होती ह यदि- नारी समझ-ार ह उसम धय और परिरवार क परनित परम ह तो कोई भी घटना परिरवार को निवघदिटत नही कर सकती या कहानी परिरवार को सगदिठत करत हए परम सौहा- स एक दसर की भावनाओ को समझ करउनका सहयोग करत हए जीवन यापन करन की पररणा -ती हमशी परमचदर जी न इस कहानी म सय परिरवार का परनितनिनमिधतव निकया ह यह कहानी बनी माधव सिसह जो गौरी पर क जमी-ार क उनक -ो पतरो की हशरी कठ लाल निबहारीशरीकात का निववाह एकजमी-ार घरान की पतरी आन-ी स हआ थाआन-ी न ख- को ससराल क वातावरण म ढाकतिलया थाएक दि-न आन-ी का अपन -वर लाल निबहारी स झगडा हो जाता ह -ोनो भाई एक दसर स अलग होन की कोकतिशश करत हसभी बह आन-ी न अपन मधर वयवहार स लाल निबहारी को

ldquoइन नतर निपरय गणो को बीए-इनही -ो अकषर पर नयोछावर कर दि-या था इन -ो अकषर न उनक शरीर को निनबल और चहर को कानित ही बना दि-या थाldquo

क) परसतत पकतियो म निकस वयकति क बार म कहा गया ह

ख) इन पकतियो म कौन स नतर निपरय गणो क बार म कहा गया ह

ग) बीए की निडगरी परापत कर लन पर भी उपय वयकति क सवभाव की कया निवशरषता थी

घ) यह नतर निपरय गण निकस वयकति म निवदयमान थ उसक वयकतितव की कया निवशरषता थी

उततर ndashक) परसतत पकति म गौरी पर गाव क जमी-ार

क बड बट शरीकात क बार म कहा गया ह उसन बहत परिरशरम और उ-म क बा- ba की निडगरी परापत की थी अब वह एक -फतर

घर छोडकर जान स रोक कतिलयाइस पर बनी माधव सिसह न कहा निक बड घर की बटी ऐसी ही होती ह जो निबगडा काम बना लती ह अतः शीरषक साथक ह बड घर की बटी आन-ी ह

म कमचारी थाख) भरा हआ चहरा चौडी छाती और डटकर

खाना आदि- एक सबजी ल जवान क गण मान जात ह परत शरीकात न इनही नतर निपरय गणो को अपनी पढाई पर नयोछावर कर दि-या था

ग) बीए की निडगरी परापत कर लन पर भी उपय वयकति(शरी कठ की शारिररिरक तौर पर निनबल और चहर स कानित ही लगत थ इतना ही नही वह मानकतिसक तौर पर भी निपछड हए थ पाशचातय सामाजिजक कथा उस घणा एव पराचीन सभयता का गणगान उनकी निवचारधारा क परमख अग थ

घ) यह नतर निपरय गण गौरीपर गाव क जमी-ार क छोट बट लाल निबहारी सिसह म निवदयमान थ वह सजीलाजवान था और भस का दध शर दध वह सवर उठकर पी जाता था

ldquoयही कारण था निक गाव की लललन आए उनकी निन-क थी कोई कोई तो उह अपना शतर समझन म भी सकोच ना करती थी सवय उनकी पतनी को इस निवरषय म उनस निवरोध थाldquo

क) उपय पकति म इस वयकति क बार म कहा गया ह

ख) गाव की लललन आए उनकी निन-ा कयो निकया करती थी

ग) उनकी पतनी का कया नाम था उनह निकस निवरषय म अपन पनित क निवरa था और कयो

घ) इस कहानी का कया उददशय ह Continue to next helliphelliphellip

Bengali 2nd Language

ফ ফটক না ফটক( কহিতা )

পর) ldquo(ান াধাচেনা ফটপাচেথ পাথচের পাড হিচেয় এক কাঠচোটটা গাছ কহিকহি পাতায় পার ফাটিচেয় াসচেছldquoক) কার দো দেকান কহিতার অং( ) lsquo(ান াধাচেনা ফটপাচেথ পাথচের পাডহিচেয়lsquo চেত কী দোঝাচেনা চেয়চেছ গ) আচো য অংচে(lsquo এক কাঠচোটটা গাছ lsquoচেত কী দোঝাচেনা চেয়চেছ ঘ) ldquoকহিকহি পাতায় পার ফাটিচেয় াসচেছldquo ----- একথার পরকত অথC কী উততর ) ক) আচো য অং(টি পর যাত কহি সভা4 মচোপাধ যাচেয়র দো lsquoফ ফটক না ফটকrsquo কহিতার অং()কহি সভা4 মচোপাধ যায় হিছচেন দেপরচেমর কহি দেপরমচেক নানা ভহিঙগমায় হিতহিন ফটিচেয় তচেচেছন দেপরম মানচের স মচেতC র সঙগী কহিতার কহিতায় এক রb সb হচেয়র দেপরম াগরচেনর কথা চেচেছন (ান অথCাৎ দেযাচেন দেকান রস দেনই দেযাচেন দেকান মহিনতা দেনই অথ তার মধ দেযও দেপরম থাকচেত পাচের একথাই কহি তচে ধরচেত দেচেয়চেছন একটি মানচে4র মচেন দেযাচেন দেকামতার দেকান সথান দেনই পাথচেরর মচেতা হিনরসতার মচেনর মধ দেযও দেয দেপরম আসচেত পাচের দেস কথাই কহি চেচেছনগ)নারীচের যথC দেপরচেমর ছহি এই কহিতায় অকপচেট উচেঠ এচেসচেছ কহি এই কহিতায় কাটচোটটা গাছ কথাটি যার কচেরচেছন নারী দেয দেপরম দেথচেক হিতাহিত এং দেসই দেপরম সঠিক সমচেয় না পাওয়ার ন য দেপরম সমপচেকC হিচেr4 গৈতরী য় দেপরচেমর দেয গৈহি(ষট য মাধযC য সরসতা দেকামত এই সমসতর হিপরীত যথা রbতা শষকতা কচেঠার তা পরভহিত দোঝাচেত এক কাঠচোটটা গাছ কথাটি যার কচেরচেছনঘ) এাচেন এক নারীর যথC দেপরচেমর কথা হিনহিCপত ভাচে চেচেছন কহি অসমচেয় নারীর ীচেন দেপরম দেচেগচেছ এতহিন তার হয় রb কচেঠার হিছ দেপরচেমর অভাচে ঠাৎ দেসই শষক মরভহিমচেত সচের আভাস এচেসচেছ দেপরম দেযন 4Cার স(ীত তাই পরায় মত গাচেছ কহিকহি পাতা গহিচেয় উচেঠচেছ

Biology Chapter - 01Controlling Air Pollution

Today we will discuss how we control air pollution from domestic combustion

Q1Describe any five ways of reducing air pollution from domestic sources bull The number of pollutants in the air is verylarge and we always try to control them byfollowing ways

i) Solar cooker and solar heater It use no fuel reduce damage of environment by fuel use or reducing deforestation It maintains coolness of house It releases very less orno oil gas or grease

ii) Piped natural gas (PNG) It emits very less by products into the atmosphere As it isdistributed through pipe lines so there iscontinuous supply of fuel is possible

iii) Liquefied Petroleum Gas (LPG) It hasa higher heating value LPG doesntcontain sulphur so it burns a lot cleanerenergy sources It releases very less oralmost no fume in air

iv) Electricity based cooking Emission free cooking alternative for urban dwellers causeselimination of adverse health impactsofindoor air pollution It helps to avoid theinconveniences associated with procurement of LPG

v) Biogas It contains 75 methane whichmakes it an excellent fuel It burns without smoke and biogas plant leaves no residue like ash in wood charcoal etc Thus it isaclean fuel

Economics

Factors of Production

Today firstly we would recall the last class for 5 mins and then we would proceed with the further topics of the chapter

The concept meaning of land characteristics of land and importance of land to be repeated for the absentees as well as the students who were there in the class the previous day

Today we will start with the last portion of land before it the meaning of land to be repeated onceAs by now we all know that

Questions1What do you mean by productivity of landAnswer By productivity of land we mean the capacity of a piece of land to produce a crop

Thus it refers to the average output per unit of landSay per acre per hectare etc= (OutputArea of land)

2 What are the factors influencing the productivity of landAnswer

Natural factors Productivity of land is largely determined by the natural

Land is defined to include not only the surface of the earth but also all other free gifts of nature(for example mineral resources forest resources and indeed anything that helps us to carry out the production of goods and services but is provided by nature free of cost)

We will move on to the last portion of land by discussing Productivity of Land

By productivity of land we mean the capacity of a piece of land to produce a crop

Thus it refers to the average output per unit of land

Say per acre per hectare etc= (OutputArea of land)

With this we shall proceed further with the main factors that determine the productivity of land

Natural factors Human factors Improvements on land Location of land Organisation Ownership of land Availability of capital Proper use of land State help

Note economic development of a country depends upon the quality of its land If the land is fertile it will quicken the pace of development of the country

qualities of land such as fertility etc

Human factors Land cannot produce anything by itself Man has to apply labour on it to produce for himself So productivity of land depends on the knowledge and skills of workers

Improvements on land production of land is affected by land development measures like provision of well or tubewell irrigation proper drainage

State help The government of a country especially less developed country can play a vital role in improving the agricultural productivity by providing better irrigation facilities

Organisation Productivity of land also fdepends upon the way how the factors of production like labour and capital are organised

In order to increase productivity trained workers modern implements scientific methods good seeds are all essential

3 lsquoImproved technology affects the productivity of landrsquo Explain this statement with the help of suitable example Answer Use of improved technology raises the productivity of land Example By using HYV seeds chemical manures and modern machines per hectare output increases

Physics Force (Summary)

Question Write the expression for the moment of force about a given axisSolutionsThe expression for the moment of force is given byMoment of force about a given axis = Force times perpendicular distance of force from the axis of rotationQuestion What do you understand by the clockwise and anticlockwise moment of force When is it taken positiveSolutionsIf the effect on the body is to turn it anticlockwise moment of force is called the anticlockwise moment and it is taken as positive while if the effect on the

body is to turn it clockwise moment of force is called the clockwise moment and it is taken as negative

Math Topic Commercial Mathematics

Chapter Goods and services Tax

Study item Some solved sums from exercise ndash 1 A retailer buys a TV from a wholesaler for Rs 40000 He marks the price of the TV 15 above his cost price sells it to the consumer at 5 discount on the marked price If the sales are intra ndash state and the rate of GST is 12 find

(i) The marked price of the TV(ii) The amount which the consumer pays for the TV(iii) The amount of tax (under GST) paid by the retailer to the central

Government(iv) The amount of tax (under GST) received by the State Government

Solution As the sales are intra- state sale and the rate of GST 12 So GST comprises of 6 CGST and 6 SGSTTherefore a retailer buys a TV from a wholesaler for Rs 40000Therefore the amount of GST collected wholesaler from the retailer or paid by retailer to wholesalerCGST = 6 of Rs 40000 = Rs(6100 times40000) =Rs 2400SGST = 6 of Rs 40000 = Rs (6100 times 40000) =Rs 2400Therefore wholesaler will pay Rs 2400 as CGST and Rs 2400 as SGSTTherefore amount of input GST of retailer Input CGST = Rs 2400 and input SGST = Rs 2400Again the retailer marks the price of the TV 15 above his cost price(i) The marked price of the TV

= Rs 40000 + Rs 40000times15= Rs 40000 + Rs 40000times 15100= Rs 40000 + Rs 6000Rs 46000But the retailer sells it to consumer at 5 discount on the marked priceCost price after discount = Rs 46000 ndashRs46000times 5100 =Rs 46000 ndashRs 2300= Rs 43700Therefore the amount of GST collected retailer from consumer or paid by consumer to retailerCGST = 6 of Rs 43700 =Rs ( 6100 times43700)Rs 2622SGST = 6 of Rs 43700 = Rs (6100 times 43700) =Rs 2622Amount of the output GST of retailer Output CGST = Rs 2622 and output SGST = Rs 2622

(ii) The amount which the consumer pays for the TV= cost price of TV to consumer + CGST paid by consumer + SGST paid by consumer= Rs 43700 + Rs 2622 + Rs 2622= Rs 48944

(iii) The amount of tax (under GST ) paid by the retailer to the central Government=CGST paid by retailer = output CGST ndash input CGST=Rs 2622 ndash Rs 2400=Rs 222

(iv) The amount of tax ( under GST ) received by the State Government = SGST paid by wholesaler + SGST paid by retailer= Rs 2400 + output SGST ndash input SGST=Rs 2400 + Rs 2622 ndash Rs 2400=Rs 2400 + Rs 222= Rs 2622

Commercial studies

Stakeholders Today I am going to give some revision questions from the previous study material

Questions1) State the two expectations of

employees from a business concern2) Give two distinctions between

stakeholder and shareholder3) Give two difference between

internal stakeholders and external stakeholders

4) Give two expectations of suppliers from a business organisation

5) Who is a stakeholder in commercial organisations

Chemistry Periodic Table

Merits of Mendeleevrsquos Periodic law are as follows - 1He grouped the elements on the basis of atomic mass 2 He left gaps for undiscovered elements like Gallium Scandium germanium Also he left a full group vacant for undiscovered inert gases 3 He could predict proportions of several elements on basis of their position in periodic table like Ga Sc etc 4He could predict errors in atomic weights of some elements like gold platinum etc

Anomalies in Mendeleevrsquos Periodic law are as follows - 1 Position of isotopes could not be explained 2 Wrong order of atomic masses could not be explained

For example- as Arnur atomic mass 40 come first and K with low atomic mass (30) should come later but k should be placed first

According to Bohrrsquos Modern Periodic table properties of elements are periodic functions of their atomic numbers

So when elements are arranged according to increasing atomic numbers there is periodicity in electronic configuration that leads to periodicity in their chemical properties

It consists of horizontal rows (Periods) Vertical column (Groups)

There are 7 period and 12 groups in this long form of periodic table

Ist period has 2 elements IInd period has 8 elements IIIrd period has 8 elements IVth period has 18 elements Vth period has 18 elements VIth period has 32 elements VIIth period hs rest of elements

Note - The number of valence electrons in atom of elements decides which elements will be first in period and which will be last

In group- 1 to 2 gp and 13 to 17 contain normal elements 3 to 12gp ndash transition elements 57 to 71 - lanthanides 89 to 103 - Actinides

Left hand side ndash metals Right hand side ndash nonmetals

Note- Hydrogen element has been placed at top of Ist group Electronic configuration of H is similar to alkali metal as both have 1 valence electron

V electron of gp I element -- 1 V electron of gp 2 element -- 2 V electron of gp 13 element -- 3 V electron of gp 14 element -- 4 V electron of gp 15 element -- 5 V electron of gp 16 element --6 V electron of gp 17 element -- 7 V electron of gp 18 element -- 8

English 1 Transformation of sentences

Sentences A sentence is a group of words which makes complete sense

Exercise 2Change the following sentences from

a Assertive sentencesb Imperative sentencesc Interrogative sentencesd Exclamatory sentences

Sentences can be changed from one grammatical form to another without changing the meaning of the sentence This is known as transformation of sentences

assertive to interrogative1 Nobody would like to be a fool

Who would like to be a fool2 Their glory can never fade

When can the glory fade3 Nobody can control the wind

Who can control the wind4 It matters little if I die

What though I die5 No man can serve two masters

Can any man serve two masters

Exercise 3Interchange of assertive and Exclamatory sentences

1 She leads the most unhappy lifeWhat an unhappy life she leads

2 This is indeed an interesting bookWhat an interesting book this

3 He is a very great manWhat a great man he is

4 It is a very lame excuseWhat a lame excuse

5 It is sad that she died so youngAlas she died so young

Class XISubject Topic Summary Execution

Hindi 2nd lang

पतर परम(परमचदर) पतर परम कहानी म एक निपता की इचछाओ का वणन निकया गया ह अपन बड पतर परभ -ास स निपता चतनय -ास का निवशरष परम था निपता को उसक जनम स ही बडी-बडी आशाए थी उसम दसर बट कतिशव-ास की अपकषा स- उतसाह की मातरा अमिधक थी वह उस इगलड भजकर बरिरसटर बनाना चाहत थभागय का खल भी बडा निनराला ह बीए की परीकषा क बा- वह बीमार पड गया डॉकटरो न भी जवाब - दि-या थाचतन -ास जी बहत ही कजस थ बवजह पस खच करना नही चाहत थ अगर गारटी मिमलती तो शाय- पस खच भी कर -त परत गारटी नही थी परिरणाम सवरप उनक बट का -हात हो गयाजब बट को समशान ल जा रह थ तो वहा काफी शोर गान बजान हो रह थ पछन पर पता चला निक निकसी निपता निपछल तीन साल स निबमार था और उसक ईलाज म रपया पानी की तरह बहाया पर ठीक नही हए परत उसक बट को तनिनक भी अफसोस नही था उसका कहना था उसन कोकतिशश तो कीयह -खकर चतनय-ास जी को आतम निगलानी हईतभी स उनका म परिरवतन हआ और बट का भोज काफी धमधाम स निकयाऔर वहइस पशचाताप की आग म जलत रह औला- स बढकर पसा नही होता ह इस बात को समझन म उनह काफी व लग गया

hellipContinue to next

BENGALI(2ND LANGUAGE)

পরথমঅধযায়-ঠাকরারীনদরনাথঠাকর

নয়ন দোচের হিমাচেররা া নাচেমই হিযাত হিছচেন ায়ানার উাররণ সবরপ নয়ন দোচের ারা হিা (াচেকর হিা হিচেতন এছাাও দেকান উৎস উপচেb রাহিতর দেক হিন করার উচেfচে(য তারা সযC হিকরচেরণ রনয পরীপ জবাহিচেয় তাচেত রপার হির 4Cরণ করচেতন ঠাকরা এই নয়ন দো হিমারচের দে(4 ং(ধর হিছচেন হিমাররা ায়ানার ষটানত পর(Cন কচের তারা হিনঃসব এই হিমাহিরর দে(4 ং(ধর গৈকাস নদর রায়চেৌধরী গৈকাস া নয়ন দোচের সমসত সমপহিতত ঋচেরণর াচেয় হিহিx কচের অহি(ষট যা আচেছ তাচেত হিপত

ইার হিপতার মতয ইচে পর নয়নচোচের ায়ানার দেগাটা কতক অসাধাররণ শরাদধ (াহিনতচেত অহিনতম ীহিপত পরকা( কহিরয়া ঠাৎ হিনহিয়া দেগ- ক) কার দো দেকান গচেলপর অং() কতা দেক ইার চেত কাচেক দোঝাচেনা চেয়চেছ গ) পরসঙগ কী কতার কতয পহিরসফট কচেরা

পরচে4র যাহিত রbা করা সমভ নয় তাই হিতহিন পতরচেক হিনচেয় ককাতায় সাস শর কচেরন গলপ কথচেকর আহিথCক অসথা নয়ন দোচের হিমাচের দেথচেক সমপরণC আাা কথচেকর হিপতা হিনচের দেষটায় অথC উপাCন করচেতন া উপাহিধ াচেভর নয তার াসা হিছনা আর দেসই কারচেরণ কথক তার একমাতর উততরাহিধকার চেয় তার হিপতার পরহিত কতজঞ কথক দো পা হি(চেচেছন হিনচের পরারণ ও মান রbার নয উপচেযাগী অথC হিনা দেষটায় পরাপত চেয়চেছন- এটাই তার কাচেছ পরম দেগৌরচের হি4য় চে মচেন কচেরন কাররণ (নয ভাণডাচের গৈপতক ায়ানার উজজব ইহিতাস অচেপbা দোার হিসeচেকর মচেধয গৈপতক দেকামপাহিনর কাগ তার কাচেছ অচেনক দেহি( মযান

TO BE CONTINUED

উ- ক) আচোয অং(টি রীনদরনাথ ঠাকচেরর দো ঠাকরা গচেলপর অং() কতা চেন আচোয গচেলপর গলপ কথকইার চেত নয়ন দোচের হিমাহিরর দে(4 ং(ধর গৈকাস ার কথা া চেয়চেছ গৈকাস া নয়ন দোচের সমসত সমপহিতত ঋচেরণর াচেয় হিহিx কচের অহি(ষট যা আচেছ তাচেত হিপত পরচে4র যাহিত রbা করা সমভ নয় তাই হিতহিন পতরচেক হিনচেয় ককাতায় সাস শর কচেরনগ) গৈকাস ার হিপতার মতযর পর নয়ন দোচের হিমাহিরর অহিসততব হিপত য় কচেয়কটা উৎস ও শরাদধ- (াহিনতচেত হিমাহিরর দে(4 কহিটক যয় চেয় হিগচেয় এচেক াচের দে(4 চেয় যায় তন তাচের গC করার মত আর হিকছই হিছ না-দেসই পরসচেঙগ এই উহিকত নয়নচোচের হিমাচেররা া নাচেমই হিযাত হিছচেন ায়ানার উাররণ সবরপ নয়নচোচের ারা হিা (াচেকর হিা হিচেতন এছাাও দেকান উৎস উপচেb রাহিতরচেক হিন করচেত হিগচেয় তারা সযC হিকরচেরণর নয পরীপ জবাহিচেয় তাচেত রপার হির 4Cরণ করচেতন তাই দেসকাচের ায়ানা দেহি(হিন সথায়ী চেত পারত না হিহিভনন উৎস শরাদধ- (াহিনতচেত সাধযা হিতহিরকত র করার নয হিমাহির হিহিকচেয় দেযত হ হিতC কা হিহি(ষট পরীচেপর দেত দেযমন অলপকাচের মচেধয হিনঃচে(4 চেয় যায়-নয়নচোচের হিমারচের অসথা তাই চেয়হিছ এই কারচেরণই কথক নয়নচোচের হিমারচের গা ভরা আমবর সয করচেত পারতনা

Physics Dimensional Analysis (Summary)

Q Find the dimensions of consts ab in relation

p=(bminusxlowastx)at

where p is the power x is the distance and t is time

Ans From principle of homogeneity dimension of b x2 are same Dim of b = dim of x2 = [L2] = [ML2T0]Dim of a = dim of ( b- x2)dim of (pt) = [M0L2T0][ML2T-2] [T-1] [T] = [M-1L0T2]

Chemistry Atomic Structure Drawbacks of Rutherfordrsquos model of

atom a According to Rutherfordrsquos model of atom electrons which are negativelycharged particles revolve around the nucleus in fixed orbits Thusb theelectrons undergo acceleration According to electromagnetic theory of Maxwell a charged particle undergoing acceleration should emitelectromagnetic radiation Thus an electron in an orbit should emitradiation Thus the orbit should shrink But this does not happenc The model does not give any information about how electrons aredistributed around nucleus and what are energies of these electrons Isotopes These are the atoms of the same

Properties of electromagnetic radiationsa Oscillating electric and magnetic field are produced by oscillating charged particles These fields are perpendicular to each other and both areperpendicular to the direction of propagation of the waveb They do not need a medium to travel That means they can even travel invacuum

Characteristics of electromagnetic radiationsa Wavelength It may be defined as the distance between two neighbouring crests or troughs of

element having the same atomicnumber but different mass numbere g 1H11H21H3

Isobars Isobars are the atoms of different elements having the same massnumber but different atomic numbere g 18Ar40 20Ca40

Isoelectronic species These are those species which have the same numberof electrons

Electromagnetic radiationsThe radiations which are associated withelectrical and magnetic fields are called electromagnetic radiations When anelectrically charged particle moves under acceleration alternating electricaland magnetic fields are produced and transmitted These fields aretransmitted in the form of waves These waves are called electromagneticwaves or electromagnetic radiations

wave as shown It is denoted by λb Frequency (ν) It may be defined as the number of waves which passthrough a particular point in one secondc Velocity (v) It is defined as the distance travelled by a wave in onesecond In vacuum all types of electromagnetic radiations travel with thesame velocity Its value is 3 times10 8m sec-1 It is denoted by v

d Wave number Wave number is defined as the number of wavelengths per unit lengthVelocity = frequency timeswavelength c = νλ

Plancks Quantum Theory- o The radiant energy is emitted or absorbed not continuously but discontinuously in the form of small discrete packets of energy called lsquoquantumrsquo In case of light the quantum of energy is called a lsquophotonrsquoo The energy of each quantum is directly proportional to the frequency of the radiation ie E α υ or E= hυ where h= Planckrsquos constant = 6626 x 10-27 Js o Energy is always emitted or absorbed as integral multiple of this uantum E=nhυ Where n=1234Black body An ideal body which emits and absorbs all frequencies is calleda black body The radiation emitted by such a body is called black body radiation

Photoelectric effectThe phenomenon of ejection of electrons from thesurface of metal when light of suitable frequency strikes it is calledphotoelectric effect The ejected electrons are called photoelectrons

Biology Chapter - 02Systematics and Five Kingdoms

Scientists divide the whole living organisms into two kingdom first and ultimately by five kingdom at last

In the earlier systems of classifications organisms are divided into kingdom plantaeand kingdom animalia on the of presenceof cell wall their modes of nutrition and movements

Some problem arise like fungi share manycharacteristic withplant despite their heterotrophic nutrition bacteria protozoa areunicellular present in both kingdom Toovercome this third kingdom Protista isintroduced which include

unicellularorganisms But there is also another

problem Allunicellular organisms are not similar kind The cellular structure of prokaryotes is verydifferent from that of other organismsEukaryotes possess a true nucleus and allcell organelles that are not present inprokaryotes So the fourth kingdom Monerais introduced which include unicellular prokaryotes (bacteriaamp blue green algae)

bull Still some problem arise in kingdomplantae

So in 1969 R H Whittakar proposedanew five kingdom System of classification

i) Kingdom Monera - unicellular prokaryotes

ii) kingdom Protista - unicellular eukaryotes

iii) Kingdom Fungi - uni or multicellular fungi with cell wall but without chlorophyll

iv) Kingdom Plantae - Multicellular Plants

v) Kingdom Animalia - Multicellular Animals

EVS Chapter 1 ndash Modes of Existence

An agricultural society

An agricultural society also known as an agrarian society is a society that constructs social order around a reliance upon farming More than half the people living in that society make their living by farming

People in an agricultural society generally lead a more settled lifestyle than those in nomadic hunter-gatherer or semi-nomadic pastoral societies because they live permanently near the land that is farmed Agricultural settlements tend to develop in areas of convenience near bodies of water which is used for both crops and transportation or along trade routes Not everyone in an agricultural society is a farmer Some people make a living trading or making and selling goods such as tools used for farming

Another way to define an agrarian society is to see the total amount of production in a nation In an agrarian society cultivating the land is the main source of wealth Such a society can recognize other means of subsistence and work habits but emphasizes the importance of agriculture and livestock Agrarian societies have existed in various parts of the world for 10000 years and continue to exist today They have been the most common form of socio-economic organization for most of recorded human history

Q) Write the features of agricultural society

Ans - Structure and Features of Agrarian Society1 Occupational Structure

An agrarian society is generally associated with the domestication of plants and animals The domestication of plants means farming and that of animals means herding Often there is mixture of farming and the use of such domesticated animals as cow goat and sheep

2 Forms of Land Ownership in Agrarian SocietiesGenerally there are landlords supervisory farmers cultivators and share croppers The landholders own the land but do not work on it They let it out for sharecropping The supervisory farmers are those who live by having their land cultivated by hired labourers The cultivators cultivate the land for themselvesThe share-croppers are those who live by tilling other peoplersquos land or a crop-sharing basis The artisans own their means of production and produce by their own labour in their homesteads

3 Village Community System An agrarian society is highlighted by

the institution of village community system The agrarian economy made fixed dwelling houses necessary Living close together for protection and co-operation and living nearer to the land gave birth to agricultural villages The village is not only the residential place of farmers it is also the social integrator

4 Minimal Division of Labour Another structural feature of agrarian society is a minimal division of labour Except for the basic division founded on age and sex differences there are few specialized roles There is only one predominant type of occupation ie domestication of plants and animals For all the people the environment physical as well as social is the same

5 Role of Family The farm family is of the patriarchal type the father is the final arbiter in most of the familyrsquos major decisions The life of ail men and women is merged in family life Since there are not many special organizations family is the only organisation to perform the tasks of aid and protection

6 Sense of Unity The members of an agrarian society exhibit a strong in-group feeling Since the whole of their social lives is wrapped up in a society which is physically economically and socially homogenous they are inclined to view the entire outside world as an out group

7 Informal Social Control An agrarian society is regionally divided into villages In a village community the force of traditional mores is more dominant than in the urban community In the village everybody is known to everybody The members in a village community help each other and share the joy and sorrows of each other Crime in an agrarian society is rare

8 Simplicity and Uniformity Life of the people in an agrarian society is marked by simplicity and uniformity Their main occupation is agriculture which largely depends upon the vagaries of nature An agrarian society is a religious society

Math Compound angles Compound angles The algebraic sum of two or more angles is called a compound angle If A B C be three angles then A+B B+C C+A A-B B-C A-C A+B-C etc are compound angles In this chapter we shall discuss the trigonometrical ratios of compound angles Theorem 1 If A B and A+B are all pisitive acute angles theni) sin( A+B) = sin A cos B + cosA sinBii) cos(A+B) = cosA cosB- sinA sinBTheorem 2If A and B are positive acute angles and AgtB theni) sin(A-B) = sin A cosB- cos A sinBii) cos(A-B) = cos A cos B+ sin A sin BTo prove that i) sin(A+B) sin (A-B) = sin2 A - sin2 B = cos2 B- cos2 A

Example 1 Prove that tan70deg=2tan50deg+tan20degSolutiontan70deg = tan(50deg + 20deg)Or tan70deg=(tan 50deg+tan 20deg)(1-tan50degtan20deg) or tan70deg (1 ndash tan 50deg tan20deg) = tan50deg+tan20degor tan70deg= tan70deg tan50deg tan20deg+ tan50deg + tan20deg = cot20deg tan50deg tan20deg + tan50deg + tan20deg = 2 tan50deg+ tan20degExample 2 If A + B = 45deg show that (1 + tanA) (1 + tanB) = 2Solutiontan(A + B) =( tan A + tan B) (1 - tan

ii) cos(A+B) Cos(A-B) = cos2 A- sin2 B = cos2 B -sin2 AProof i) LHS= sin(A+B)sin(AminusB) [Recall sin(αminusβ)=sinαcosβminuscosαsinβ And sin(α+β)=sinαcosβ+cosαsinβ]= (sinAcosB+cosAsinB)times(sinAcosBminuscosAsinB)= sin2Acos2Bminuscos2Asin2B [Recall sin2α+cos2α=1 From above we can then assume correctly that sin2α=1minuscos2α AND cos2α=1minussin2α] = sin2A(1minussin2B)minussin2B(1minussin2A) = sin2Aminussin2Asin2Bminussin2B+sin2Asin2B = sin2Aminussin2B= 1-cos2A-(1-cos2B) = cos2 B- cos2 A = RHSii)LHS= cos (A+B) cos(A-B) [ cos(A+B) = cos AcosB- sinAsinBCos(A-B) = cosAcosB+ sinAsinB]= cos2 A Cos2 B- sin2 A Sin2 B= cos2 A( 1-sin2 B) - (1- cos2 A) sin2 B= cos2 A- cos2 A sin2 B- sin2 B+ cos2 A sin2 B=cos2 A- sin2 B=1- sin2 A-(1-cos2 B) = cos2 B- sin2 A= RHSTangent formulae for compound anglesi)tan (A + B) = tan A + tan B1-tan A tan Bii)tan (A ndash B) = tan A-tan B1+tan A tan Biii) cot (A + B) = cot Acot B-1cot A+cot B(viii) cot (A ndash B) = cot Acot B+1cot B-cot A

A tan B) Or 1= (tan A+ tanB) (1-tan A tanB) Or tanA + tanB + tanA tanB + 1 = 1 + 1Or tanA (1 + tanB) + (1 + tanB) = 2Or (1 + tanA) (1 + tanB) = 2Example 3 Find the value of sin 15degSolution sin 15deg= sin(45deg-30deg) = sin45degcos 30deg- cos45degsin30deg =(1radic2) (radic32) -(1radic2) (12) = (radic3-1) 2radic2Example 4 If sin A = 1 radic10 and sin B = 1 radic5 where A and B are positive acute angles then what is A + B SolutionWe know that sin (A + B) = sin A cos B + cos A sin B= [1 radic10] [radic(1 minus 1 5)] + [1 radic5] radic(1 minus 1 10)= [1 radic10] [radic4 5] + [1 radic5] [radic9 10]= [1 radic50] times (2 + 3)= 5 radic50 = 1 radic2

sin (A + B) = sin π 4rArrHence A + B = π 4Example 5 If A + B = 225o then find [cot A] [1 + cotA] times [cot B] [1 + cot B]Solution[cot A] [1 + cotA] times [cot B] [1 + cot B] = 1 [(1 + tan A) times (1 + tan B)]=1 [tan A + tan B + 1 + tan A tan B] [ tan (A + B) = tan225o]∵

tan A + tan B = 1minus tan A tan BrArr= 1 [1 minus tan A tan B + 1 + tan A tan B]= 1 2

COMMERCE

CLASSIFICTION OF HUMAN ACTIVITIES-ECONOMIC AND NON-ECONOMIC

Firstly we shall recall the previous class for 5 mins especially for the absentees and for also the rest of the students who were there

Today at first we briefly discuss the earlier portions of the chapter

1Business-It includes all those economic activities which are concerned with production and exchange of goods and services with the object of earning profit Example A factory shop beauty parlour also business enterprises

2Profession ndashThe term profession means an occupation which involves application of specialized knowledge and skills to earn a living For Example Chartered Accountancy medicine law tax consultancy are example of professions

Questions1What are the main features of ProfessionAnswer The main features of a profession are as follows a Specialised body of knowledge-Every profession has a specialised and systematised body of knowledge b Restricted entry- Entry to a profession is allowed only to those who have completed the prescribed education and have the specialised examination c Formal education and training ndashA formal education and training is given to the person who wants to acquire the professional

3Employment-Employment mean an economic activity where people work for others in exchange for some remuneration (salary)The persons who work for others are called lsquoemployeesrsquo The persons or organizations which engage others to work for them are called lsquoemployersrsquoEg A doctor working in a hospital is employment as he is working for a salaryA lawyer may serve as a law officer in a bank

With this we shall proceed with the features of both Profession amp Employment

The main features of a profession are as follow

a Specialised body of knowledge b Restricted entry c Formal education and training d Professional association e Service motive f Code of contact

The main features of an employment are as follows

a In employment a person works for others called employer

b An employee provides personal service

c There is a service agreement or contract between the employee and the employer

d The employee has to obey the order of the employer

e No capital investment is made by the employer

Various examples of Employment are as follows

aA teacher teaching in a school or collegeb An engineer employed in Municipal Corporation of DelhicAn accountant working in the accounts department of a companydA doctor working in a hospital

Note In all the above examples of employment the individual who is involved in each example is working as an employee for a salary under an employer

qualification(MBBSCALLB)d Service motive ndashProfessionals are expected to emphasis service more on their clients rather than economic gain f Code of Conduct-The activities of professionals are regulated by a code of conduct

2 What are the main features of EmploymentAnswer The main features of an employment are as followsa In employment a person works for others called employerb An employee provides personal servicec There is a service agreement or contract between the employee and the employerd The employee has to obey the order of the employere No capital investment is made by the employer

3 Give various Professions and their respective Association are given below

Professions

Professional

Professional association

Medical profession

Doctor Medical Council of India

Law profession

Lawyers Bar Council of India

Accounting Profession

Chartered

The Institute of Chartered Accounts of India( ICAI)

Engineerin Engineers The

g Profession

institute of Engineers (India)

Accounts Basic accounting terms

Today we will give you some questions from the previous study material

Questions6) Define accounting7) What do you mean by debit

and credit8) Explain the types of account9) Define the following terms

a) Assetsb) Capitalc) Purchased) Debtorse) Transactions

10) Name the types of accounts given below

a) Krishnas accountb) Machinery accountc) Royalty accountd) Salary accounte) Furniture accountf) Audit fee account

Economics Basic Economic ConceptsSub topic

UTILITY

Before starting todayrsquos class we shall recall the last class which was about UTILITY AND THE FEATURES OF UTILITY

Now we shall proceed with the further topics of the chapter

Todayrsquos topic from the chapter lsquo Basic Economic Conceptsrsquo will be TOTAL UTILITY amp MARGINAL UTILITYNow let us quickly revise the concept of utility with an example ie goods and services are designed because they have an ability to satisfy human wantsThis feature of being able to satisfy human wants is termed as utility For example we derive utility from WiFi services as it gives us satisfaction by connecting us to our friends and family through social media here consumers derive utility from WiFi services

From the above concept we shall start with todayrsquos topicEconomists have defined TOTAL UTILITY (TU) as the total satisfaction obtained by consuming a given total amount of a good and serviceFor example the total satisfaction obtained from eating 10 mangoes is the total utility of 10 mangoes

MARGINAL UTILITY (MU) is the additional satisfaction derived from each additional unit

Questions1 What is Total Utility (TU)

Answer Total Utility (TU) is the

aggregate of the utility that a consumer derives from the consumption of a certain amount of a commodityTU=MU1+MU2++MUn

2 What is Marginal UtilityAnswer

Marginal Utility (MU) is the additional made to the total utility as consumption is increased by one more unit of the commodityMU= TUn ndashTUn-1

NoteOften economists tend to

subdivide utility into an imaginary unit called UTIL

consumed In this casethe utility obtained from each mango as it is consumed as the MU of that mango It is also defined as the addition made to the total utility when an additional unit is consumed Often economists tend to subdivide utility into an imaginary unit called UTIL

Note As a consumer increases the consumption of a good over period of time the total utility or total satisfaction derived from it increases to appoint and thereafter it decreasesHowever as the consumer keeps on consuming the good the marginal utility or the additional utility derived from it decreases

SubjectBusiness studies

Topic

BUSINESSENVIRONMENT

Summary

Now quickly let us revise the earlier points that we have already done in the last class and let us proceed with the other topics that are there in the chapter

Firstly we will recall the internal and external factors of micro environment and then we shall proceed in details

Meaning and list of internal and external factors

aInternal factorsInternal factors refer to all the factors existing within a business firm The internal factors are considered controllable because the enterprise has control over these factorsFor an example a company can alter its organization structure policies programmes employees physical facilities and marketing mix to suit the changes in the environmentList of internal factors areCorporate culture mission and objectives top management organizations structure company image and brand equity company resources

b External factorsExternal factors refer to those individual and groups and agencies with which a particular business organization comes into direct and frequent contact in the course of its functioningThese individuals and groups are known as STAKEHOLDERS because they have a stake (financial interest ) in the working and performance of the particular business List of external forces (stakeholders)Customers competitors investors suppliersmiddlemen (marketing intermediaries)

Execution 1 What do you mean by internal

factors in micro environmentAnswerInternal factors refer to all the factors existing within a business firm The internal factors are considered controllable because the enterprise has control over these factorsFor an example a company can alter its organization structure policies programmes employees physical facilities and marketing mix to suit the changes in the environment

2 What do you mean by external factors in micro environment

AnswerExternal factors refer to those individual and groups and agencies with which a particular business organization comes into direct and frequent contact in the course of its functioningThese individuals and groups are known as STAKEHOLDERS because they have a stake (financial interest) in the working and performance of the particular business

3Who are stakeholdersSTAKEHOLDERS are individuals and groups who have a stake (financial interest ) in the working and performance of the particular business 4Discuss the internal factors in briefa Corporate CultureThe values beliefs and attitudes of the founders and top management of the company exercise

financers publics

customers

suppliersfinancers

competitors

middlemen

publics

Fig STAKEHOLDERS OF A COMPANY

Apart from micro environment the other main dimension of business environment isMacro environment Macro environment refers to the general environment or remote environment within which a business firm and forces in its micro environment operateA company does not directly or regularly interact with the micro environmentTherefore macro environment is also known as indirect action EnvironmentThe macro environment forces are less controllable than the micro forces

Macro environment consists of the following components

POLITICAL AND LEGAL ENVIRONMENT

ECONOMIC SOCIAL AND ENVIRONMENT

CULTURAL

ENVIRONMENT

TECHNOLOGICAL ENVIRONMENT

a strong influence on what the cmpaany stands for how it does things and what it considers importantbMission and objectivesThe business philosophy and purpose of a comoany guide it prioritiesbusiness strategiesproduct market scope and development scope

cTop management structurethe composition of board of directors the degree of professionalization of management and the organizational structure of a company have important bearing on its business decisions

dPower structureThe internal power relationship between the board of directors and the chief executive is an important factor

eCompany image and brand equityThe image and brand equity of the company play a significant role in raising finance forming alliance choosing dealers and suppliers launching new products entering foreign markets

5 What is Macro environmentAnswerMacro environment refers to the general environment or remote environment within which a business firm and forces in its micro environment operateA company does not directly or regularly interact with the micro environmentTherefore macro environment is also known as indirect action EnvironmentThe macro environment forces are less controllable than the micro forces 6 What are the components of macro environmenta Political and legal environmentb Economic environmentc Social and cultural environmentd Technological environment

BUSINESS FIRM

Fig COMPONENTS OF MACRO ENVIRONMENTPolitical science

Introduction to political science

Comparative politics and itrsquos scope Comparative politics is the second major dimension of political scienceIt is also a very vast area of study and a very large number of political scientists even treat it as an autonomous area of study within the board ambit of political scienceScope of comparative politics-

1 All political structures -Comparative politics includes the study of all structures formalnon formal governmental and extra governmental which are directly or indirectly involved in politics in all the countries of the world

2 Functional studies- Comparative politics seeks to study politics less from the point of view of the legal institutions in terms of their powers and move from the point of view of their functions which constitute the political process and their actual Operation in the environment

3 Study of political behaviour- Another important part of its scope is the study of the actual behaviour of the people in the process of politics

4 Study of similarities and differences- comparative politics also undertakesan analysis of the similarities and differences among political process and functions

5 Study of all political systems -comparative politics seeks to analyse the actual behaviour and performance of all political systems western as well as non western

6 Study of the environment and infrastructure of politics-The study of politics demands a study of the psychological sociological economic and anthropological environment in fact the social environment as a whole in which each political system operates

7 Study of political culture- political culture is composed of attitudesbeliefs emotions and values of a society that relate to the political system or politics

8 Study of political participation- Political participation is a universal processThe only difference is that while in some states it is limited in others it is wider

9 Study of political process- political

Answer the following questions-

What is comparative politics

What are the scope of comparative politics

Homework- learn

processes like decision makingpolicy making judicial process leadership recruitment process and others are always at work in all political systems

The scope of comparative politics is very comprehensive It includes everything that falls within the area of political activity and political process

History CAMBRIDGE VIEW ABOUT

THE PARTITION

AND REFUTATION

OF CAMBRIDGE

VIEW

Cambridge view about the Partition The Cambridge school of historians have interpreted that opposition to partition scheme was made entirely by the elitist groups They hold the view that Lord Curzon planned to partition the Bengal for administrative purposeREFUTATION OFCAMBRIDGE VIEW The Rationalist historians have rejected the interpretations of the Cambridge School of historians on various grounds

1 QUESTION State different views of historians regarding Partition of Bengal

ANSWER Cambridge historians believed that Lord Curzon partitioned Bengal for administrative reasons only and not for the political motive The Middle class elitist group protested because of their petty interest The Hindu zamindars protested as they have to spend more money for managing their estatesThe lawyers of Calcutta High court feared to lose their clientBut according to the nationalist Historians was-

2- The ultimate object of Lord Curzon was to crush the unity of Bengal politicians

3- If Bengal becomes a separate province Bengali speaking 16 million people of western part would become minority under Hindi speaking people of Bihar and Oriya speaking people of Orissa

4- The bureaucrats expected that the protest movement would die down quickly

5- Lord Curzon used the Muslim community in his political game

6- Idealism had great contribution in the protest against partition

7- The people of the every section of society were affected by the partition of Bengal

Computer Science

Numbers Convertion of dcimal number to octal numberThe decimal numeral system is the standard system for denoting integer and non-integer numbers It is the extension to non-integer numbers of the Hindu-Arabic numeral system For writing numbers the decimal system uses ten decimal digits a decimal mark and for negative numbers a minus sign - The decimal digits are 0 1 2 3 4 5 6 7 8 9 the decimal separator is the dot in many countries

The octal numeral system or oct for short is the base-8 number system and uses the digits 0 to 7 Octal is sometimes used in computing instead of hexadecimal perhaps most often in modern times in conjunction with file

permissions under Unix systems It has the advantage of not requiring any extra symbols as digits It is also used for digital displays

Follow these steps to convert a decimal number into octal form

1 Divide the decimal number by 82 Get the integer quotient for the next iteration (if the number will not divide equally by 8 then round down the

result to the nearest whole number)3 Keep a note of the remainder it should be between 0 and 74 Repeat the steps until the quotient is equal to 05 Write out all the remainders from bottom to top This is the solution

For example if the given decimal number is 8453

Division Quotient Remainder

8453 8 1056 5

1056 8 132 0

132 8 16 4

16 8 2 0

2 8 0 2

Then the octal solution is 20405

Subject Eng Literature (The Tempest ndash William Shakespeare) Topic Act I Scene 1 Lines 33 to 67 (End of scene) Date 16th April 2020 (4th Period)

[Students should read the original play and also the paraphrase given in the school prescribed textbook]Summary Questions amp Answers

[SUMMARY OF THE ENTIRE SCENE]

o The play starts with the scene of a severe storm at sea Alonso (King of Naples) Sebastian (Alonsorsquos brother) Ferdinand (Alonsorsquos son) Gonzalo Antonio (the usurping Duke of Milan) are in a ship in the midst of the storm

o The mariners are trying their best to control the vessel from running aground and are totally following the orders of their Master the Boatswain They have scant success

o The mariners become extremely unhappy and annoyed when most of the passengers arrive on the deck thereby hampering their effort to save the ship There is serious confrontation between them and the passengers who are part of the Kingrsquos entourage

o The mariners could not save the ship

SUMMING-UP

(i) Vivid description of the scene which gives a realistic description of terror and confusion of a tropical storm

(ii) Shows Shakespearersquos accuracy of knowledge in describing the naval operations and also matters of seamanship

(1) GONZALO Ill warrant him for drowning (L 45-57)

though the ship were no stronger than a nutshell and as leaky as an unstanched

wenchBOATSWAIN Lay her a-hold a-hold Set her two courses Off to

sea again lay her offMARINERS All lost To prayers to prayers All lostBOATSWAIN What must our mouths be coldGONZALO The king and prince at prayers Lets assist them

For our case is theirsSEBASTIAN Im out of patienceANTONIO We are merely cheated of our lives by drunkards

This wide-chopped rascal - would thou mightst lie drowning the washing of ten tides

(a) What does Antonio say at the insolent manners of the boatswain just before the given passage

Being irritated at the insolent manners of the boatswain just before the given extract Antonio the Duke of Milan calls him a worthless dog son of a woman without any morals an arrogant and disrespectful noisemaker He says that the boatswain deserved to be hanged(b) What statement does Gonzalo repeat about the boatswain

Gonzalo shows his faith that the boatswain is not destined to die by drowning He is destined to be hanged and nothing can alter this decree of destiny He says that even if the ship was as frail as a nutshell the boatswain could not be drowned for his destiny was to be hanged(c) What do the passengers do when they have lost all hope of their survival

When the passengers have lost all hope of survival they take

(iii) The opening scene justifies the title ndash The Tempest

UNANSWERED QUESTIONS

(i) The King always travels with his entire fleet including his soldiers Where were the other ships

(ii) Why was the ship in that area Where was it coming from or going where

(iii) The ship broke apart What happened to those who were in the ship

(We shall get the answer to the above questions as the play progresses)

leave of life with fervent prayers The mariners take their last hearty drink and are ready for death(d) What blame does Antonio put upon the mariners and the boatswain Antonio rebukes the mariners that these drunkards have brought them to the present crisis by neglecting their duties He blames them saying that they are going to lose their lives entirely for the negligence of the boatswain and his fellows(e) What does Antonio say while cursing the boatswain

Antonio gives vent to his wrath upon the boatswain in particular He calls the boatswain a wide-mouthed rascal who deserves to be hanged on the sea-shore at low water mark so that ten tides might wash over his body and take out of him all the liquor that he has been drinking

Class XIISubject Topic Summary ExecutionHistory Topic

1 1935 ACT AND WORKING OF PROVINCIAL AUTONOMYCONGREE AND OTHER MINISTERSSUB TOPIC GOVERNMENT OF INDIA ACT1935

Government of India Act 1935 This act established a lsquoFederation of Indiarsquo made of British Indian provinces and Indian states and provided for autonomy with a government responsible to the elected legislature in every provinceThis act introduced abolition of Diarchy at provinces The entire provincial administration was introduced to the responsible ministers who were controlled and removed by the provincial legislature The provincial autonomy means two things First The provincial governments were wholly responsible to the provincial legislature Secondly Provinces were free from outside control and interference in the large number of matters The act divided the powers between the centre and provinces in terms of three lists- Federal list( for centre) Provincial list (for province) and concurrent list (for both) Residuary powers were given to the viceroy In the election under the government of India Act the Congress swept the poll the mandate of the people came in favour of the congress so far as general Hindu seats were concerned The Congress did not get a single Muslim seates in Bombay CP UP Sind and BengalIn five provinces Congress had yhe clear majority In BengalNWFPAssam and Bombay Congress emerged as a single largest partyOn the other side the performance of the Muslim League was badThus the Congress formed ministers in 7 provinces out of 11 provinces Coalition ministry was also formed in two other provincesOnly BENGAL AND Punjab had non- congress ministries

1 QUESTION What was the main change introduced by the Government of India ActANSWER a) The Act gave more

autonomy to the provinces b) Diarchy was abolished at the

provincial levelsc) The Governor was the head of

the executived) There was a council of

ministers to advise him The ministers were responsible to the provincial legislatures who controlled them The legislature could also remove the ministers

e) The Governors still retained special reserve powers

2 QUESTION Why did the federal scheme introduced by the Government of India Act 1935 never come into operation

ANSWER The Federal structure of the Government of India was to be composed with the Governor General and Council of ministers The Federal legislature was to be Bicameral legislature- The council of states and the House of Assembly The ministers were to be chosen by the Governor general and they were to hold the office during his pleasure

The provinces of British India would have to join the federation but this was not compulsory for the princely states

This federation never materialised because of the lack of support from the required number of

princely statesThis act was refused and

rejected by the princes the Congress and the Muslim League

Thus both Congress and the League participated in the election of 1937 Thus the federal part was never introduced but the provincial part was put into operations

Bengali 2nd

Language

াচেরর পরাথCনা(কহিতা )

াচেরর পরাথCনা কহিতাটি কহি (ঙখ দেঘাচে4র দো আচো য কহিতায় াচেরর পতর হমায়ন কঠিন দেরাচেগ আxানত ার ঈশবর া আললার কাচেছ পরাথCনা কচেরচেছন তার পচেতরর ীন হিফহিরচেয় হিচেত এই কহিতায় ার পচেতরর ীন হিভbা দেচেয়চেছন ারার এমনহিক হিনচের ীন হিসCচেনর হিহিনমচেয় হিতহিন তার দেছচের ীন হিফচের দেপচেত দেচেয়চেছন তার দেছচের এই দেরাচেগর ন য হিতহিন হিনচেচেকই ায়ী কচেরচেছন তার হিনচের করা পাপচেকই হিতহিন ায়ী কচেরচেছন এছাা রানৈনহিতক ও আথCসামাহিক অসথার কথা তচে ধরা চেয়চেছ এই কহিতায় ার তার হিনচের পাপ কমCচেকই ায়ী কচেরচেছ ার অন যায় ভাচে দেপহি((হিকতর মাধ যচেম অপররা য কচেরচেছ আর এই অন যায় কাচের ন যই তার পহিরাচের হিপযCয় এচেসচেছ দে এক পরকার মানহিক নধন ইহিতাচেসর ার হিপতা চেয় সবাভাহিকভাচে ভাচোাসা দে মমতা দেথচেক মকত চেত পাচেরনহিন তাই হিপতা চেয় আললা া ভগাচেনর কাচেছ পতর হমায়চেনর পরানহিভbা দেচেয়চেছন ার আললা া ভগাচেনর কাচেছ াহিনচেয়চেছন তার হিনচের ীন হিসCন হিচেত হিতহিন রাী তার হিহিনমচেয় পচেতরর ীন হিফচের দেপচেত দেচেয়চেছন াচেরর হিপতসভ হিচেকর কথা এই কহিতায় ফটিচেয় দেতাা চেয়চেছ হিপতা পচেতরর হিরাহিরত মান নধচেনর কথা তচে ধরা চেয়চেছ

হিচে(4 হিকছ াইচেনর তাৎপযC১) ldquoদেকাথায় দেগ ওর সবচছয দেৌন দেকাথায় কচেরায় দেগাপন bয়ldquoউততর) াচেরর পতর হমায়ন কঠিন দেরাচেগ অসসথ তাই তার দেযৌন াহিরচেয় যাচেচছ এই দেরাচেগ তাচেক দেগাপচেন কচেরকচের াচেচছ তার সক (হিকত ধীচের ধীচের bয় চেচছ তাই হিপতা চেয় ার আললার কাচেছ হমায়চেনর পরান হিভbা দেচেয়চেছন২) ldquoাগাও (চেরর পরাচেনত পরানতচের ধসর (ন দেযর আান গানldquoউততর) াচেরর পতর হমায়ন কঠিন দেরাচেগ আxানত তাই ার আ দে(াচেক মমCাত (চেরর পচেথ পরানতচের আান গান ধবহিনত দোক দেসই আান গান আললার কাচেছ দেযন চে যায় আললা দেযন এই আহিতC শচেন পচেতরর ীন হিফহিরচেয় দেয় ৩)ldquoনাহিক এই (রীচেরর পাচেপর ীানচেত দেকানই তরারণ দেনই ভহি4চেতরldquoউততর) হমায়চেনর অসসথতার ন য ার হিনচেচেকই ায়ী কচেরচেছন কারন ার অচেনক রা য অন যায় ভাচে কচেরচেছ তাই তার এই পাপ কাচের ন য তার ঘচের আ হিপ এচেসচেছ এই অন যায় কাচের ন য তার মহিকত দেনই তাই ার আললার কাচেছ এই পাপ কাচেযCর ন য bমা পরাথM

Hindi 2ndlang

-ासी(जयशकर परसा-)

-ासी जयशकर परसा- की एक ऐसी कहानी ह जिजसम भारतीय ससकनित और राषटरीयता का सवरगजीतहोता ह इस कहानी म इरावती एक निहद कनया ह जिजस मलअचछो न मलतान की लट म पकडा और -ासी बना दि-या उस 500 दि-न -कर काशी क एक महाजन न खरी-ा दसरी -ासी निफरोजा ह वह गलाम ह निफरोजा को छडान क कतिलए अहम- को 1000 सोन क कतिसकक भजन थ जो अभी तक नही आए थ राजा साहब कठोर होत हए भी निफरोजा को निबना धनराकतिश क कतिलए उस म कर -त ह वनिफरोजा को अहम- को समझान की बात कहत हकहानी क अत म हम -खत ह निक इरा वती और जाटो क सर-ार बलराज का मिमलन होता हअहम- को यa म मार दि-या जाता ह वहा निफरोजा की परसननता की समामिध बनती ह वहा एक फल चढती ह और डीजल आती ह निफरोजा उस समामिध की आजीवन -ासी बनी रहती हलखक अपन उददशय अथात -ास परथा पर परकाश डालन और इस परथा क कारण होन वाल -ातो क दखो को दि-खान म पणता सफल हए ह

helliphellipContinue to next

Biology Reproductio Today we will discuss about vegetative Q1 Name some vegetative propagules

n in Organisms

propagation of plants The process of multiplication in which fragments of plant body function as propagule and develop into new individual is called vegetative propagation The units of such propagation are runner rhizome tuber bulb etc

and the speciesinvolvedVegetative propagules

Parts involved

Bulb StemBulbil BulbilRhizome Stem Runner Stem Tuber Stem Offset Stem Leaf buds Leaves Suckers Stem

Corns Stem stolon

Q2 State advantages of vegetative propagation

i) Rapid methodii) Sure and easy methodiii) Useful in plants that cannot

produce viable seeds or long seed dormancy

iv) Maintains purity of raceQ 3 Banana fruit is said to be parthenocarpic where as turkey is said to be parthenogenetic WhyBanana develops without fertilization from an unfertilized ovary thus is parthenocarpicIn turkey the ovum or female gamete developinto a new chick without fertilization thus isparthgenetic

Q4 Why is water hyacinth is called as a ldquoTerror of Bengalrdquo Water hyacinth can

propagatevegetatively all over the water body in a short per short period of time This resulted increased biochemicaloxygen oxygen demand of water body causing mortalityof fishes It is very difficult to get rid off them Thus known as terror of Bengal

Chemistry

Solid state GENERAL CHARACTERISTICS OF SOLID STATEIn nature the particular state of matter is governed by two opposing forces at given set of temperature and pressure These forces are intermolecular force of attraction and thermal energy If intermolecular force of attraction is high as compared to thermal energy particles remains in closest position

Intext QuestionsQ1 Classify the following solids as crystalline and amorphous Sodium chloride quartz glass quartz rubber polyvinyl chloride Teflon

A1 Crystalline

and hence very less movement in particles is observed In this case solid state is the preferred state of matter

Let us revise the general characteristics of solid

i) Fixed mass volume and shape

ii) Strong intermolecular force of attraction

iii) Least intermolecular space

iv) Fixed position of constituent particles

v) Incompressible and rigid

Q2 what type of interactions hold the molecules together in a polar molecular solid[CBSE 2010]A2 The molecules in a solid are held together by van der Waals forces The term van der Waals forces include hydrogen bonding dipole-dipole attraction and London dispersion forces All molecules experience London dispersion forces In addition polar molecules can also experience dipole-dipole interactions So the interactions that holds the molecule together in polar molecular solid are London dispersion force and dipole-dipole interactionsQ3 Write a feature that will distinguish a metallic solid from an ionic solid [CBSE 2010]A3 Metals are malleable and ductile whereas ionic solid are hard and brittle Metallic solid has typical metallic lustre But ionic solid looks dullQ4 Write a point of distinction between a metallic solid and an ionic solid other than metallic lustre [CBSE 2012]A4 Metals are malleable and ductile whereas ionic solid are hard and brittleQ5 Write a distinguish feature of metallic solid [CBSE 2010]A5 The force of attraction in

solid Sodium chloride Quartz Amorphous solid Quartz glass rubber polyvinyl chloride Teflon Q2 why glass is considered as super cooled liquidA2 Glass shows the tendency to flow at slower rate like liquid Hence they considered as super cooled liquidQ3 why the window glass of old buildings show milky appearance with timeA3 Glass is an amorphous solid Amorphous solid has the tendency to develop some crystalline character on heating Due to heating in day over the number of years glass acquires some crystalline character and show milky appearanceQ4 why the glass panes fixed to window or doors of old building become slightly thicker at bottomA4 Glass is super cooled liquid It has the tendency to flow down very slowly Due to this glass pane becomes thicker at the bottom over the timeQ5 Sodium chloride is a crystalline solid It shows the same value of refractive index along all the direction TrueFalse Give reasonA5 FalseCrystalline solid shows anisotropy in properties That is it shows different values for the given physical property in different direction All the crystalline solids show anisotropy in refractive index Therefore sodium chloride will show different values of refractive index on different directions

Q6 Crystalline solid are anisotropic in nature What does this statement means

between the constituent particles is special kind of electrostatic attraction That is the attraction of positively charged kernel with sea of delocalized electronsQ6 which group of solid is electrical conductor as well as malleable and ductile [CBSE 2013]A6 Metallic solidQ7 why graphite is good conductor of electricity although it is a network (covalent solid)A7 The exceptional property of graphite is due to its typical structure In graphite each carbon is covalently bonded with 3 atoms in same layer The fourth valence electron of each atom is free to move in between different layersThis free electron makes the graphite a good conductor of electricity

[CBSE 2011]A6 Anisotropy is defined asrdquo Difference in properties when measured along different axis or from different directionsrdquo Crystalline solid show different values of some of the physical properties like electrical resistance refractive index etcwhen measured along the different directions The anisotropy in crystalline solid arises due to the different arrangement of particles in different directions

Math Function Composition of functions Think of an industrial plant that produce bottles of cold drinks first there is the operation (or function) f that puts the cold drink inside the bottle followed by the opeartion g that close the bottle with the capThis leads to the following definitionDefinition Let f A rarr B and g B rarr C be two functions Then the composition of f and g denoted by gof is defined as the function gof A rarr C given by gof(x) = g(f (x)) forall x isinA

Definition A function f X rarr Y is defined to be invertible if there exists a function g Y rarr X such that gof = IX and fog = IY The function g is called the inverse of f and is denoted by f -1

Thus if f is invertible then f must be one-one and onto and conversely if f is one-one and onto then f must be invertible This fact significantly helps for proving a function f to be invertible by showing that f is one-one and onto specially when the actual inverse of f is not to be determined

Example 1 Let f 2 3 4 5 rarr 3 4 5 9 and g 3 4 5 9 rarr 7 11 15 be functions defined as f(2) = 3 f(3) = 4 f(4) = f(5) = 5 and g (3) = g (4) = 7 and g (5) = g (9) = 11 Find gofSolution We have gof(2) = g (f(2)) = g (3) = 7 gof(3) = g (f(3)) = g (4) = 7gof(4) = g (f(4)) = g (5) = 11 and gof(5) = g (5) = 11Example 2 Find gof and fog if f R rarr R and g R rarr R are given by f(x) = cos x and g (x) = 3x2 Show that gof ne fogSolution We have gof(x) = g(f(x))=g(cosx) = 3 (cos x)2

= 3 cos2 x Similarly fog(x)=f(g (x))= f(3x2)= cos (3x2) Note that 3cos2 x ne cos 3x2 for x = 0 Hence gof ne fogExample 3 Show that if f A rarr B and g B rarr C are onto then gof A rarr C is also ontoSolution Given an arbitrary element z isin C there exists a pre-image y of z under g such that g (y) = z since g is onto Further for y isin B there exists an element x in A with f(x) = y since f is onto Therefore gof(x) = g (f(x)) = g (y) = z showing that gof is onto Example 4 Let Y = n2 n isin N sub N Consider f N rarr Y as f(n) = n2 Show that

f is invertible Find the inverse of fSolution An arbitrary element y in Y is of the form n2 for some n isin N This implies that n =radicy This gives a function g Y rarr N defined by g (y) =radicy Nowgof (n) = g (n2)=radicn2 = n and fog (y) =f(radicy) = (radicy) 2 y which shows that gof=IN and fog= IY Hence f is invertible with f -1 = g

Political Science

Constitution of India-The Preamble

Summary

Objective of the state-To secure equality of status and of opportunity To promote fraternity among all the citizens To assure the dignity of the individuals and Unity and integrity of the nation

Justice-Justice stands for rule of law absence of arbitrariness and a system of equal rights freedom and opportunities for all in a society India seeks social economic and political justice to ensure equality to its citizens

Liberty-Liberty implies the absence of restraints or domination on the activities of an individual such as freedom from slavery serfdom imprisonment despotism etc The Preamble provides for the liberty of thought expression belief faith and worship

Equality-Equality means the absence of privileges or discrimination against any section of the society The Preamble provides for equality of status and opportunity to all the people of the country

Fraternity-The Preamble declares that fraternity has to assure two thingsmdashthe dignity of the individual and the unity and

Execution

Answer the following questions-

Short notes-1 Equality2 Fraternity3 Justice4 Liberty

Homework-Learn

integrity of the nation The word integrity has been added to the Preamble by the 42nd Constitutional Amendment (1976)

Business studies

Human resource management (chapter 1)

On the day of 1504 2020 I have discussed with you the managerial functions and procurement functions of HRM

Today weare going to discuss about the development function integration functions and maintenance function

Development functions-HRM improves the knowledge skills attitude and values of employees so that they the present and future jobs more effectively it includes

1) Development functions of HRM

a) Performance appraisal = It implies systematic evaluation of employees with respect to their performance on the job and their potential for development

b) Training =It is the process by which employees learn knowledge skills and attitudes to achieve organisational and personal goals

c) Executive development = It is the process of developing managerial talent through appropriate program

2) Integration functionsa) HRM reconcile the goals of

organisation with those of its members through integrating function

b) HRM tries to motivate employees to various financial and non financial incentives provided in job specification etc

3) Maintenance functiona) HRM promote and protect the

physical and mental health of employees by providing several types of benefits like housing medical aid etc

b) It Promote Social security measures to employees by providing provident fund pension gratuity maternity benefits

SubjectCOMMERCE

Topic

BUSINESSENVIRONMENT

Summary

Now quickly let us revise the earlier points that we have already done in the last class and let us proceed with the other topics that are there in the chapter

Firstly we will recall the internal and external factors of micro environment and then we

Execution 3 What do you mean by internal factors

in micro environmentAnswerInternal factors refer to all the factors existing within a business firm The internal factors are considered controllable because the enterprise has control over these factors

Development FunctionsPerformance AppraisalTrainingExecution Development

shall proceed in details

Meaning and list of internal and external factors

aInternal factorsInternal factors refer to all the factors existing within a business firm The internal factors are considered controllable because the enterprise has control over these factorsFor an example a company can alter its organization structure policies programmes employees physical facilities and marketing mix to suit the changes in the environmentList of internal factors areCorporate culture mission and objectives top management organizations structure company image and brand equity company resources

b External factorsExternal factors refer to those individual and groups and agencies with which a particular business organization comes into direct and frequent contact in the course of its functioningThese individuals and groups are known as STAKEHOLDERS because they have a stake (financial interest ) in the working and performance of the particular business List of external forces (stakeholders)Customers competitors investors suppliersmiddlemen (marketing intermediaries)financers publics

customers

suppliersfinancers

For an example a company can alter its organization structure policies programmes employees physical facilities and marketing mix to suit the changes in the environment

4 What do you mean by external factors in micro environment

AnswerExternal factors refer to those individual and groups and agencies with which a particular business organization comes into direct and frequent contact in the course of its functioningThese individuals and groups are known as STAKEHOLDERS because they have a stake (financial interest) in the working and performance of the particular business

3Who are stakeholdersSTAKEHOLDERS are individuals and groups who have a stake (financial interest ) in the working and performance of the particular business 4Discuss the internal factors in briefa Corporate CultureThe values beliefs and attitudes of the founders and top management of the company exercise a strong influence on what the cmpaany stands for how it does things and what it considers importantbMission and objectivesThe business philosophy and purpose of a comoany guide it prioritiesbusiness strategiesproduct market scope and development scope

cTop management structurethe composition of board of directors the degree of professionalization of management and the organizational structure of a company have important bearing on its business decisions

dPower structureThe internal power relationship between the board of directors and the chief executive is an important factor

e Company image and brand equityThe image and brand equity of the company play a significant role in raising finance forming alliance choosing dealers and suppliers launching new products entering foreign markets

5 What is Macro environmentAnswerMacro environment refers to the general

competitors

middlemen

publics

Fig STAKEHOLDERS OF A COMPANY

Apart from micro environment the other main dimension of business environment isMacro environment Macro environment refers to the general environment or remote environment within which a business firm and forces in its micro environment operateA company does not directly or regularly interact with the micro environmentTherefore macro environment is also known as indirect action EnvironmentThe macro environment forces are less controllable than the micro forces

Macro environment consists of the following components

POLITICAL AND LEGAL ENVIRONMENT

ECONOMIC SOCIAL AND ENVIRONMENT

CULTURAL

ENVIRONMENT

TECHNOLOGICAL ENVIRONMENT

Fig COMPONENTS OF MACRO ENVIRONMENT

environment or remote environment within which a business firm and forces in its micro environment operateA company does not directly or regularly interact with the micro environmentTherefore macro environment is also known as indirect action EnvironmentThe macro environment forces are less controllable than the micro forces 6 What are the components of macro environmenta Political and legal environmentb Economic environmentc Social and cultural environmentd Technological environment

Computer Science

Logic gates

Digital systems are said to be constructed by using logic gates These gates are the AND OR NOT NAND NOR EXOR and EXNOR

BUSINESS FIRM

gates The basic operations are described below with the aid of truth tables

AND gate

The AND gate is an electronic circuit that gives a high output (1) only if all its inputs are high A dot () is used to show the AND operation ie AB Bear in mind that this dot is sometimes omitted ie ABOR gate

The OR gate is an electronic circuit that gives a high output (1) if one or more of its inputs are high A plus (+) is used to show the OR operationNOT gate

The NOT gate is an electronic circuit that produces an inverted version of the input at its output It is also known as an inverter If the input variable is A the inverted output is known as NOT A This is also shown as A or A with a bar over the top as shown at the outputs The diagrams below show two ways that the NAND logic gate can be configured to produce a NOT gate It can also be done using NOR logic gates in the same way

NAND gate

This is a NOT-AND gate which is equal to an AND gate followed by a NOT gate The outputs of all NAND gates are high if any of the inputs are low The symbol is an AND gate with a small circle on the output The small circle represents inversion

NOR gate

This is a NOT-OR gate which is equal to an OR gate followed by a NOT gate The outputs of all NOR gates are low if any of the inputs are highThe symbol is an OR gate with a small circle on the output The small circle represents inversion

EXOR gate

The Exclusive-OR gate is a circuit which will give a high output if either but not both of its two inputs are high An encircled plus sign ( ) is used to show the EOR operation

EXNOR gate

The Exclusive-NOR gate circuit does the opposite to the EOR gate It will give a low output if either but not both of its two inputs are high The symbol is an EXOR gate with a small circle on the output The small circle represents inversion The NAND and NOR gates are called universal functions since with either one the AND and OR functions and NOT can be generated

Note A function in sum of products form can be implemented using NAND gates by replacing all AND and OR gates by NAND gates A function in product of sums form can be implemented using NOR gates by replacing all AND and OR gates by NOR gates

Logic gate symbols

Table 2 is a summary truth table of the inputoutput combinations for the NOT gate together with all possible inputoutput combinations for the other gate functions Also note that a truth table with n inputs has 2n rows You can compare the outputs of different gates

Logic gates representation using the Truth table

Example

A NAND gate can be used as a NOT gate using either of the following wiring configurations

Subject Eng Literature (The Tempest ndash William Shakespeare) Topic Act III Scene 3 Lines 53 to 110 (End of the scene) Date 16th April 2020 (2nd Period)

[Students should read the original play and also the paraphrase given in the school prescribed textbook]Summary Questions amp Answers

o Seeing this strange scene all are inclined to believe the tales told by travelers that there truly are ldquounicornsrdquo and ldquothe phoenixrsquo thronerdquo

o As they are about to sit down to the feast the banquet is snatched away by a harpy (Ariel disguised) A spiritrsquos voice (Arielrsquos voice) denounces Alonso Sebastian and Antonio with particular

1 ARIEL You are three men of sin whom Destiny

(Line 53-58)That hath to instrument this

lower world And what is int the never-surfeited sea

Hath caused to belch up you and on this island

Where man doth not inhabit you rsquomongst men

Being most unfit to live I have made you mad

reference to their crime in expelling Prospero from Milan They have not received any punishment for their deed earlier but the time for their punishment has arrived Upon Alonso it pronounces ldquolingering perdition worse than deathrdquo from which there is no remedy except through sincere repentance Ariel then vanishes in thunder and the shapes enter again and carry away the table

o Prospero watching invisibly is very pleased with the performance of Ariel and his (Prosperorsquos) ldquomeaner ministersrdquo All his enemies are now in his power and are in a fit of desperation He then leaves them and goes to see how Ferdinand and Miranda are getting on

o Alonso is now much humbled and penitent with the after effect of the spiritrsquos denunciation of his crimes He believes that his son is lost forever After this all disperse being stricken mad by the speech of the spirit

o Gonzalo fearing that they may do violence to themselves or to one another follows them and bid others to follow

(a) To whom does Ariel disguised as a harpy call the three sinners What game did Fate of Destiny play with

them

The three sinners called by Ariel are Alonso Sebastian and Antonio It was Destiny which had caused the ocean to cast the three sinners on the shore Though the ocean is all the time devouring whatever appears on its surface and is never satisfied with its continual swallowing of the ships and men in the present case the ocean had cast these three sinners on the shore without killing them

(b) Who had jointly been responsible for the conspiracy against Prospero What is Prosperorsquos purpose behind all this

Three men Alonso Sebastian and Antonio had jointly

been responsible for the conspiracy against Prospero They had driven out Prospero form Milan Prosperorsquos purpose is to make these three sinners realize the wrong they had done He wants them to repent for their criminal deeds because repentance leads to self-esteem(c )What does Ariel (the harpy) tell Alonso and his companions when they take out their swords to attack him

Seeing them drawing their swords Ariel (harpy) tells them that he and his companions are the instruments of destiny and that it is not possible for human beings to do them any injury He says that the swords of human beings can not injure even a minute part of his feathers Their swords are as ineffective against him and his companions as against the wind or the water

(d) Give the explanatory meanings of the following expressions in the context of the above extract

(i)Never surfeited (ii) Belch up (iii) lsquomongst men

(i) Never surfeited never led to satisfaction

(ii) Belch up cast ashore(iii) lsquomongst men in human

society2

I and my fellows (Line 60-65)

Are ministers of Fate The elementsOf whom your swords are tempered may as wellWound the loud winds or with bemocked-at stabsKill the still-closing waters as diminishOne dowl thats in my plume

IMPORTANT PASSAGES EXPLAINED

The elements

(Line 61-66)Of whom your swords are tempered may

as wellWound the loud winds or with

bemocked-at stabs

(a) Who is lsquoIrsquo Who are his lsquofellowsrdquo

lsquoIrsquo is referred to Ariel in disguise of a harpy His lsquofellowsrsquo are other spirits serving Prospero the real Duke of Milan who has acquired supernatural powers after being banished from his Dukedom Prospero has settled in this uninhabited island

(b) What are the elements that have temperrsquod the swords Why will it not work against the speaker

The swords (of Alonso and his companions) are tempered by metal (steel) which is taken out of the earth and refined by

Kill the still-closing waters as diminishOne dowl thats in my plume My fellow

ministersAre like invulnerable

In these words Ariel reminds the King and his companions of the utter futility of drawing swords against himself and his fellows Ariel drives Alonso Antonio and Sebastian the three men of sin to desperation ndash a state in which men do violence to themselves They draw swords to strike Ariel But Ariel reminds them that he and the other spirits are the ministers of destiny and nothing can wound them The steel of which their swords are made of may cut the wind or water which being divided always closes up again Even supposing that such things may be possible it is quite impossible that their swords will cut one feather in their plume They are incapable of being wounded by any sword of man Hence it is foolish on their part to attempt to strike at Ariel and his fellow-spirits

For which foul deed

(Line 72-75)The powers delaying not forgetting

haveIncensed the seas and shores yea all the

creatures Against your peace

Ariel enters like a harpy and remaining invisible tells Alonso Sebastian and Antonio that he and other harpies are the agents of Destiny appointed to carry out her decrees He tells them that their punishment for the crime against Prospero which has been so long deferred is now to fall upon them He reminds them that they had expelled Prospero from Milan and set him and his innocent child adrift on the sea and that the sea had paid them back for their sin by the shipwreck and by the calamities they have suffered He tells them that the powers above which did not forget this mean treachery but only deferred the punishment have now engaged the seas and the shores and all living beings including him and his comrades against them The very elements and supernatural agency Ariel adds have taken up the avenging of their crime against Prospero

the action of fire It may cut the wind or water which being divided always closes up again

The sword will not work against the spirits and the harpy because they are the ministers of destiny and nothing can wound them nor it will cut a single feather in their plume

(c )What is the meaning of lsquodowlrsquo in the last line

The term lsquodowlrsquo means a filament or the smallest part of a feather In this context Ariel in disguise of harpy says that their sword cannot even damage the smallest filament of their (Arielrsquos and other spirits) feathers as they are incapable of being wounded by any sword of man

(d) What does the speaker remind the listeners about

Ariel in disguise of harpy reminds Alonso the King of Naples Sebastian Alonsorsquos brother and Antonio the present Duke of Milan and the treacherous brother of Prospero as they being three men of sin He even reminds them that their punishment for their crime against Prospero which has been so long deferred now falls upon them He reminds them that they have expelled Prospero from Milan and has set him along with his innocent infant daughter adrift on the sea So the sea has paid them back for their sin by their shipwreck and the calamities they have suffered since then The harpy rebukes Alonso of his sin that has incensed the Gods and has deprived him of his son as a punishment

(e) How do they respond

When Ariel in disguise of a harpy reminds Alonso Sebastian and Antonio of their past misdeeds and sin Alonso has a look of terror and confusion in his eyes He utters the words of sincere repentance wrung out of his conscience-stricken heart It appears to him that all the elements of nature the sea-waves the wind and the thunder proclaiming a loud voice in the name of Prospero and the crime Alonso has committed against him They are calling upon him to repent There is a deep storm raging in Alonsorsquos breast and the echoes of that storm are ringing in his ears like a clear note of wind-instrument A note of denunciation of Alonsorsquos crime leaves him much humbled and penitent and confirms his belief that his son is lost forever But Sebastian and Antonio shows some courage instead of repentance They wish to kill the spirits or devils if it appears

3

Of my instruction hast thou nothing bated (Line 85-93)

In what thou hast to say So with good life

And observation strange my meaner ministers

Their several kinds have done My high charms work

And these mine enemies are all knit upIn their distractions They now are in my

powerAnd in these fits I leave them while I visitYoung Ferdinand whom they suppose is

drownedAnd his and mine loved darling

Methought the billows spoke and (Line 96-99)

told me of itThe winds did sing it to me and the

thunderThat deep and dreadful organ-pipe

pronouncedThe name of Prosper It did bass my

trespass

These are the words of contrition coming from Alonso Ariel has driven him to a deep repentance for conspiring with Antonio against Prospero He now feels a sincere remorse It appears to him that all the elements of nature the sea-waves the wind and the thunder proclaimed with a loud voice the name of Prospero and the crime Alonso had committed against him They are calling upon him to repent There is a deep storm raging in Alonsorsquos breast and the echoes of that storm are ringing in his ears like the clear note of a wind-instrument

Comment These are the words of sincere repentance wrung out of the conscience-stricken heart of Alonso Alonso who is the lesser villain is the first to give way to remorse under the effect of Arielrsquos speech The words of Ariel seem to him to be the voice of conscience speaking to him He is driven to desperation a state in which he might do violence to his life

(a) Identify the speaker State the context

Prospero the ruler of the island is the speaker The famous banquet scene has been enacted very well Ariel and his junior spirits have played their roles excellently Prospero is glad to say words of praise for them(b) In what way the speakerrsquos instructions have been carried out

According to Prosperorsquos instructions a banquet was presented before the King of Naples and his companions when they were tired and hungry Just when they were preparing to eat the feast the banquet was suddenly removed by exercising supernatural powers All this was done by Ariel Prosperorsquos chief assistant and a powerful spirit

Ariel not only made the feast disappear but also delivered his speech blaming the King and his two companions for their past wicked deeds He warned them to repent for their misdeeds or suffer forever on that uninhabited island

(c) Who are referred to as lsquomeaner ministersrsquo What have they done

Prospero refers as lsquomeaner ministersrsquo to his other lesser spirits who were assisting Ariel in presenting a scene before the kingrsquos party They entered the scene to the accompaniment of music They assumed several strange shapes and brought in a banquet Then they danced about it with gentle actions of salutations thus inviting the King and others to eat the feast

These spirits play their role again when Ariel in the shape of a harpy quits the scene These shapes enter again and dancing with mocking gestures carry away the table

(d) Who are the speakerrsquos enemies What has happened to them

King of Naples Alonso his brother Sebastian and the present Duke of Milan Antonio (Prosperorsquos own brother) are Prosperorsquos enemies With the turn of events they have all been washed ashore on the island which is ruled by Prospero the great magician Actually this happened after the shipwreck caused by a storm which was raised by Prospero with the purpose of bringing these people to his island Prosperorsquos spirits have already confused and terrified these enemies and they are under Prosperorsquos control He can treat them as he likes

(e) What does he say about Ferdinand Explain what is meant by ldquohellip his and mine darlingrdquo

Prospero knows that Alonsorsquos son prince Ferdinand is alive though his father thinks that the prince has been drowned

Prospero refers to his daughter Miranda who is dear to him She is also very dear to Prince Ferdinand who has fallen in love with her They are waiting to be married soon for which they have received Prosperorsquos consent

4

ALONSO O it is monstrous monstrous (Line 95-102)

Methought the billows spoke and told me of it

The winds did sing it to me and the thunderThat deep and dreadful organ-

pipe pronouncedThe name of Prosper It did bass

my trespassTherefore my son ithrsquo ooze is

bedded andIll seek him deeper than eer

plummet soundedAnd with him there lie mudded

(a) In what way does Alonso express his horror when his conscience is awakened by Arielrsquos words

When Alonsorsquos conscience is awakened by Arielrsquos words he expresses his horror at what he has heard He gets the feeling that the waves of the ocean the wind and the loud thunder have spoken to him and uttered the name of Prospero Because of being reminded of his crime in a very loud and rough voice he comes to realize that he has lost his son for his past misdeeds

(b) What does Alonso imagine about his son What does Alonso want to do in his desperate state

Alonso imagines that his son is lying in the mud at the bottom of the sea He feels desperate that he wants to drown himself in the ocean deeper than the plumb-line has ever gone He wants to lie with his son at the bottom of the sea

(c) How do Sebastian and Antonio want to face the evil spirits

Sebastian says that he is not at all afraid of what the harpy has said and that he is prepared to fight any number of such monsters if they appear before him only one at a time Antonio says that he would support Sebastian in the fight against the fiendsyyy

(d) Why does Gonzalo ask Adrian to follow the three men

Gonzalo tells Adrian that all the three men namely Alonso Sebastian and Antonio are in a wild and reckless mood The thought of the heinous crime of which they are guilty has begun to torment their minds So he asks Adrian to follow those three men without loss of time and prevent them from doing anything which the turmoil in their minds might lead them to do

(e) What opinion do you form of Alonso from the above extract

Alonso who is the lesser villain is the first to give way to remorse under the effect of Arielrsquos speech The words of Ariel seem to him to be the voice of conscience speaking to him He is driven to desperation a state in which he might do violence to his life

Subject =Accounts

Ac-12 15420 topic-pL Appropriation ac

PROFIT AND LOSS APPROPRIATION ACCOUNT

MEANING AND PREPARATIONProfit and Loss Appropriation Account is merely an extension of the Profit and Loss Account of the firm The profit of the firm has to be distributed amongst the partners in their respective profit sharing ratio But before its distribution it needs to be adjusted All Adjustments like partnerrsquos salary partnerrsquos commission interest on capital interest on drawings etc are made in this account These adjustments will reduce the amount of profit for distribution This adjusted profit will be distributed amongst the partners in their profit sharing ratio To prepare it at first the balance of Profit and Loss Account is transferred to this account The journal entries for the preparation of Profit and Loss Appropriation Account are given below

1 for transfer of the balance of Profit and Loss Account to Profit and Loss Appropriation Account

(a) In case of Net Profit

Profit and Loss Ac helliphelliphelliphelliphellipDrTo Profit and Loss Appropriation Ac(Net Profit transferred to Profit and Loss Appropriation Ac)

(b)In case of Net Loss

Profit and Loss Appropriation Achelliphelliphellip DrTo Profit and Loss Ac(Net Loss transferred to Profit and Loss Appropriation Ac)

2 for Interest on Capital

For transferring on Interest on CapitalProfit and Loss Appropriation Achelliphelliphellip DrTo Interest on Capital Ac(Interest on capital transferred to Profit amp Loss Appropriation Ac)

3 for Interest on Drawings

For transferring Interest on Drawings Interest on Drawings Achelliphelliphelliphelliphelliphellip DrTo Profit and Loss Appropriation Ac(Interest on drawing transferred to Profit amp Loss Appropriation Ac)

4 For Partnerrsquos SalaryFor transfer of partnerrsquos SalaryProfit and Loss Appropriation Achelliphellip DrTo Salary Ac(Salary transferred to profit amp Loss Appropriation Ac)

5 For Partnerrsquos CommissionFor transferring commissionProfit and Loss Appropriation Achelliphelliphellip DrTo Commission Ac(Commission transferred to Profit and Loss Appropriation Ac)

6 For Transfer of agreed amount to General ReserveProfit and Loss Appropriation Ac helliphellipDrTo General Reserve Ac(Transfer to General Reserve)

7 for share of Profit or Loss appropriation(a) If ProfitProfit and Loss Appropriation Achelliphellip DrTo Partnerrsquos CapitalCurrent Ac(Profit transferred to capitalcurrent Ac)(b) If LossPartnerrsquos Capital Current Achelliphelliphelliphellip DrTo Profit and Loss Appropriation Ac(Loss transferred to capitalcurrent Ac)

THE FORMAT OF PROFIT AND LOSS APPROPRIATION

Profit and Loss Appropriation Account for the year endedhelliphelliphelliphellip

Particulars Amount Particulars Amount

To PL Ac (loss) By pL Ac (profit)

To Interest on capital BY Interest on drawings

To partner`s commission by Partner`s capital Ac ( loss)

To Partner`s salary To Interest on partner`s loan To General Reserve To Partner`s Capital AC (Profit)

Subject= Economics

MOVEMENT ALONG THE DEMAND CURVE (CHANGE IN QUANTITY DEMANDED)In law of demand you have already studied the inverse relationship between price and quantity demanded When quantity demanded of a commodity changes due to change in its price keeping other factors constant it is called change in quantity demanded It is graphically expressed as a movement along the same demand curve There can be either a downward movement or an upward movement along the same demand curve Upward movement along the same demand curve is called contraction of demand or decrease in quantity demanded and downward movement along the same demand curve is known as expansion of demand or increase in quantity demanded

Extention of demandd

price (rs)p A

B Extentionp1 d

Q Q1

Quantity demanded ( in units)

Contraction of demandd

p2 Ccontraction

p APrice (Rs)

d

Q2 Q

Quantity demanded (in units)

Explanation of movement of demand A fall in price from OP to OP1 leads to increase in quantity demanded from OQ to OQ1 (expansion of demand) resulting in a downward movement from point A to point B along the same demand curve DD When Price rises from OP to OP2 quantity demanded falls from OQ to OQ2 (contraction of demand) leading to an upward movement from point A to point C along the same demand curve DD

  • Activity Series of Metals
    • Drawbacks of Rutherfordrsquos model of atom
      • Electromagnetic radiations
      • Properties of electromagnetic radiations
      • Characteristics of electromagnetic radiations
        • Plancks Quantum Theory-
        • Photoelectric effect
          • Intext Questions
            • Logic gates
            • Digital systems are said to be constructed by using logic gates These gates are the AND OR NOT NAND NOR EXOR and EXNOR gates The basic operations are described below with the aid of truth tables
            • AND gate
            • Example
Page 20:  · Web viewSubject. Topic. Summary. Execution. English 1 . Chapter 1 naming words . Page 8. Write the names of these pictures:- Person:-1. father. 2.Firefighter 3.doctor 4 ...

himself how he hates Antonio because of his being a Christian because he abuses Shylock in public places Shylock decides that if ever he can get Antonio to his advantage he will teach him a lesson

come to harm and men can commit mistakes and thus the capital invested in ships may be lost Then there are other dangers The goods loaded on the ships can be damaged by rats and thieves which are found both on land and water The ships can also be harmed through sea-storms submerged rocks etc(c) What two important functions does this scene have

The scene has two important functions First it completes the exposition of the two major plot lines of the play Antonio agrees to Shylockrsquos bond ndash three thousand ducats for a pound of flesh and second and more important dramatically this scene introduces Shylock himself In this scene Shakespeare makes it clear at once why Shylock is the most powerful dramatic figure in the play and why so many great actors have regarded this part as one of the most rewarding roles in all Shakespearean dramas(d) Where does this scene take place What kind of treatment has Antonio been giving to Shylock What does Shylock say when Bassanio invites him to dine with him

The action of this scene takes place in Venice Antonio has been in the habit of behaving harshly with Shylock ndash spitting on his beard and footing him like a stranger cur When Bassanio invites Shylock to dine Shylock says that he is prepared to do anything with the Christians but not eat and drink or pray with them

(2) SHYLOCK How like a fawning publican he looks (Line 38-48)I hate him for he is a Christian

But more for that in low simplicity

He lends out money gratis and brings downThe rate of usance here with us in VeniceIf I can catch him once upon the hipI will feed fat the ancient grudge I bear him

He hates our sacred nation and he railsEven there where merchants most do congregateOn me my bargains and my well-won thriftWhich he calls interest Cursed be my tribeIf I forgive him

(a) What is the context in which these words are spoken and what is the idea expressed in it

These remarks are made by Shylock when he sees Antonio coming along after Bassanio told him that the merchant will be his surety for the bond The above mentioned passage reveals Shylockrsquos hatred for Antonio Shylock says that he hates Antonio because he is a Christian and also because he gives loan without taking interest on them thereby bringing down the rate of interest in Venice(b) Explain the meaning of the phrase lsquoa fawning publicanrsquo

The phrase lsquoa fawning publicanrsquo refers to Roman tax collector It is a term of contempt and hatred on the lips of a Jew lsquoFawning Publicansrsquo were Roman tax-gatherers whose ordinary bearings towards the Jews was bullying but whose false pose of lsquohumility and contritionrsquo is touched upon in the parable in New Testament(c ) What light does the above passage throw on the character

of Shylock

The above mentioned speech of Shylock reveals him to be a wicked character having an extreme greed for wealth His intense hatred for Antonio is unjustified He hates Antonio only because he is a Christian and because he lends money without taking any interest on it thereby adversely affecting Shylockrsquos business of lending money on high interest(d) What information do you gather about Antonio from the above given lines

Shylockrsquos statement throws a valuable light on the character of Antonio Antonio appears to be a good Christian and a good human being He helps the people in need by lending them money without charging any interest on it He is a man of simple and good nature This very goodness makes him Shylockrsquos enemy(e) What does Shylock debate within himself and when To whom are the lines mentioned above addressed to

When Bassanio asks the Jew to lend him three thousand ducats on Antoniorsquos surety Shylock begins to debate within himself as to how he should exploit the opportunity of a business deal with his old enemy Antonio

The lines mentioned above are not addressed to anyone The lines are a soliloquy ie a speech made by a character to himself and not meant to be heard by the other characters present

Class XSubject Topic Summary ExecutionEnglish

LiteratureThe Blue Bead 2nd part

Things took a turn for the worst and all of a sudden a crocodile attacked the woman biting on the womanrsquos leg At that moment Sibia got up sprinted grabbed the hay fork and stabbed the crocodile in the eye with all her power Immediately the crocodile let go and went away Sibia saw a small blue bead lying by the river she grabbed it Since she was poor she didnrsquot have necklace Shersquod always wanted one like the other women now she could make one with the blue bead After that she went home and told her mother all about it

Hindi 2nd

Langबड घर की बटी( मशी परमच-)

lsquoबड घर की बटी कहानी का उददशय मधयम वग की घरल समसया को सलझा कर सगदिठत परिरवार म मिमल जलकर परम स रहन का स-श -ना हघर म शानित सथानिपत करन की जिजमम-ारी नारी की होती ह यदि- नारी समझ-ार ह उसम धय और परिरवार क परनित परम ह तो कोई भी घटना परिरवार को निवघदिटत नही कर सकती या कहानी परिरवार को सगदिठत करत हए परम सौहा- स एक दसर की भावनाओ को समझ करउनका सहयोग करत हए जीवन यापन करन की पररणा -ती हमशी परमचदर जी न इस कहानी म सय परिरवार का परनितनिनमिधतव निकया ह यह कहानी बनी माधव सिसह जो गौरी पर क जमी-ार क उनक -ो पतरो की हशरी कठ लाल निबहारीशरीकात का निववाह एकजमी-ार घरान की पतरी आन-ी स हआ थाआन-ी न ख- को ससराल क वातावरण म ढाकतिलया थाएक दि-न आन-ी का अपन -वर लाल निबहारी स झगडा हो जाता ह -ोनो भाई एक दसर स अलग होन की कोकतिशश करत हसभी बह आन-ी न अपन मधर वयवहार स लाल निबहारी को

ldquoइन नतर निपरय गणो को बीए-इनही -ो अकषर पर नयोछावर कर दि-या था इन -ो अकषर न उनक शरीर को निनबल और चहर को कानित ही बना दि-या थाldquo

क) परसतत पकतियो म निकस वयकति क बार म कहा गया ह

ख) इन पकतियो म कौन स नतर निपरय गणो क बार म कहा गया ह

ग) बीए की निडगरी परापत कर लन पर भी उपय वयकति क सवभाव की कया निवशरषता थी

घ) यह नतर निपरय गण निकस वयकति म निवदयमान थ उसक वयकतितव की कया निवशरषता थी

उततर ndashक) परसतत पकति म गौरी पर गाव क जमी-ार

क बड बट शरीकात क बार म कहा गया ह उसन बहत परिरशरम और उ-म क बा- ba की निडगरी परापत की थी अब वह एक -फतर

घर छोडकर जान स रोक कतिलयाइस पर बनी माधव सिसह न कहा निक बड घर की बटी ऐसी ही होती ह जो निबगडा काम बना लती ह अतः शीरषक साथक ह बड घर की बटी आन-ी ह

म कमचारी थाख) भरा हआ चहरा चौडी छाती और डटकर

खाना आदि- एक सबजी ल जवान क गण मान जात ह परत शरीकात न इनही नतर निपरय गणो को अपनी पढाई पर नयोछावर कर दि-या था

ग) बीए की निडगरी परापत कर लन पर भी उपय वयकति(शरी कठ की शारिररिरक तौर पर निनबल और चहर स कानित ही लगत थ इतना ही नही वह मानकतिसक तौर पर भी निपछड हए थ पाशचातय सामाजिजक कथा उस घणा एव पराचीन सभयता का गणगान उनकी निवचारधारा क परमख अग थ

घ) यह नतर निपरय गण गौरीपर गाव क जमी-ार क छोट बट लाल निबहारी सिसह म निवदयमान थ वह सजीलाजवान था और भस का दध शर दध वह सवर उठकर पी जाता था

ldquoयही कारण था निक गाव की लललन आए उनकी निन-क थी कोई कोई तो उह अपना शतर समझन म भी सकोच ना करती थी सवय उनकी पतनी को इस निवरषय म उनस निवरोध थाldquo

क) उपय पकति म इस वयकति क बार म कहा गया ह

ख) गाव की लललन आए उनकी निन-ा कयो निकया करती थी

ग) उनकी पतनी का कया नाम था उनह निकस निवरषय म अपन पनित क निवरa था और कयो

घ) इस कहानी का कया उददशय ह Continue to next helliphelliphellip

Bengali 2nd Language

ফ ফটক না ফটক( কহিতা )

পর) ldquo(ান াধাচেনা ফটপাচেথ পাথচের পাড হিচেয় এক কাঠচোটটা গাছ কহিকহি পাতায় পার ফাটিচেয় াসচেছldquoক) কার দো দেকান কহিতার অং( ) lsquo(ান াধাচেনা ফটপাচেথ পাথচের পাডহিচেয়lsquo চেত কী দোঝাচেনা চেয়চেছ গ) আচো য অংচে(lsquo এক কাঠচোটটা গাছ lsquoচেত কী দোঝাচেনা চেয়চেছ ঘ) ldquoকহিকহি পাতায় পার ফাটিচেয় াসচেছldquo ----- একথার পরকত অথC কী উততর ) ক) আচো য অং(টি পর যাত কহি সভা4 মচোপাধ যাচেয়র দো lsquoফ ফটক না ফটকrsquo কহিতার অং()কহি সভা4 মচোপাধ যায় হিছচেন দেপরচেমর কহি দেপরমচেক নানা ভহিঙগমায় হিতহিন ফটিচেয় তচেচেছন দেপরম মানচের স মচেতC র সঙগী কহিতার কহিতায় এক রb সb হচেয়র দেপরম াগরচেনর কথা চেচেছন (ান অথCাৎ দেযাচেন দেকান রস দেনই দেযাচেন দেকান মহিনতা দেনই অথ তার মধ দেযও দেপরম থাকচেত পাচের একথাই কহি তচে ধরচেত দেচেয়চেছন একটি মানচে4র মচেন দেযাচেন দেকামতার দেকান সথান দেনই পাথচেরর মচেতা হিনরসতার মচেনর মধ দেযও দেয দেপরম আসচেত পাচের দেস কথাই কহি চেচেছনগ)নারীচের যথC দেপরচেমর ছহি এই কহিতায় অকপচেট উচেঠ এচেসচেছ কহি এই কহিতায় কাটচোটটা গাছ কথাটি যার কচেরচেছন নারী দেয দেপরম দেথচেক হিতাহিত এং দেসই দেপরম সঠিক সমচেয় না পাওয়ার ন য দেপরম সমপচেকC হিচেr4 গৈতরী য় দেপরচেমর দেয গৈহি(ষট য মাধযC য সরসতা দেকামত এই সমসতর হিপরীত যথা রbতা শষকতা কচেঠার তা পরভহিত দোঝাচেত এক কাঠচোটটা গাছ কথাটি যার কচেরচেছনঘ) এাচেন এক নারীর যথC দেপরচেমর কথা হিনহিCপত ভাচে চেচেছন কহি অসমচেয় নারীর ীচেন দেপরম দেচেগচেছ এতহিন তার হয় রb কচেঠার হিছ দেপরচেমর অভাচে ঠাৎ দেসই শষক মরভহিমচেত সচের আভাস এচেসচেছ দেপরম দেযন 4Cার স(ীত তাই পরায় মত গাচেছ কহিকহি পাতা গহিচেয় উচেঠচেছ

Biology Chapter - 01Controlling Air Pollution

Today we will discuss how we control air pollution from domestic combustion

Q1Describe any five ways of reducing air pollution from domestic sources bull The number of pollutants in the air is verylarge and we always try to control them byfollowing ways

i) Solar cooker and solar heater It use no fuel reduce damage of environment by fuel use or reducing deforestation It maintains coolness of house It releases very less orno oil gas or grease

ii) Piped natural gas (PNG) It emits very less by products into the atmosphere As it isdistributed through pipe lines so there iscontinuous supply of fuel is possible

iii) Liquefied Petroleum Gas (LPG) It hasa higher heating value LPG doesntcontain sulphur so it burns a lot cleanerenergy sources It releases very less oralmost no fume in air

iv) Electricity based cooking Emission free cooking alternative for urban dwellers causeselimination of adverse health impactsofindoor air pollution It helps to avoid theinconveniences associated with procurement of LPG

v) Biogas It contains 75 methane whichmakes it an excellent fuel It burns without smoke and biogas plant leaves no residue like ash in wood charcoal etc Thus it isaclean fuel

Economics

Factors of Production

Today firstly we would recall the last class for 5 mins and then we would proceed with the further topics of the chapter

The concept meaning of land characteristics of land and importance of land to be repeated for the absentees as well as the students who were there in the class the previous day

Today we will start with the last portion of land before it the meaning of land to be repeated onceAs by now we all know that

Questions1What do you mean by productivity of landAnswer By productivity of land we mean the capacity of a piece of land to produce a crop

Thus it refers to the average output per unit of landSay per acre per hectare etc= (OutputArea of land)

2 What are the factors influencing the productivity of landAnswer

Natural factors Productivity of land is largely determined by the natural

Land is defined to include not only the surface of the earth but also all other free gifts of nature(for example mineral resources forest resources and indeed anything that helps us to carry out the production of goods and services but is provided by nature free of cost)

We will move on to the last portion of land by discussing Productivity of Land

By productivity of land we mean the capacity of a piece of land to produce a crop

Thus it refers to the average output per unit of land

Say per acre per hectare etc= (OutputArea of land)

With this we shall proceed further with the main factors that determine the productivity of land

Natural factors Human factors Improvements on land Location of land Organisation Ownership of land Availability of capital Proper use of land State help

Note economic development of a country depends upon the quality of its land If the land is fertile it will quicken the pace of development of the country

qualities of land such as fertility etc

Human factors Land cannot produce anything by itself Man has to apply labour on it to produce for himself So productivity of land depends on the knowledge and skills of workers

Improvements on land production of land is affected by land development measures like provision of well or tubewell irrigation proper drainage

State help The government of a country especially less developed country can play a vital role in improving the agricultural productivity by providing better irrigation facilities

Organisation Productivity of land also fdepends upon the way how the factors of production like labour and capital are organised

In order to increase productivity trained workers modern implements scientific methods good seeds are all essential

3 lsquoImproved technology affects the productivity of landrsquo Explain this statement with the help of suitable example Answer Use of improved technology raises the productivity of land Example By using HYV seeds chemical manures and modern machines per hectare output increases

Physics Force (Summary)

Question Write the expression for the moment of force about a given axisSolutionsThe expression for the moment of force is given byMoment of force about a given axis = Force times perpendicular distance of force from the axis of rotationQuestion What do you understand by the clockwise and anticlockwise moment of force When is it taken positiveSolutionsIf the effect on the body is to turn it anticlockwise moment of force is called the anticlockwise moment and it is taken as positive while if the effect on the

body is to turn it clockwise moment of force is called the clockwise moment and it is taken as negative

Math Topic Commercial Mathematics

Chapter Goods and services Tax

Study item Some solved sums from exercise ndash 1 A retailer buys a TV from a wholesaler for Rs 40000 He marks the price of the TV 15 above his cost price sells it to the consumer at 5 discount on the marked price If the sales are intra ndash state and the rate of GST is 12 find

(i) The marked price of the TV(ii) The amount which the consumer pays for the TV(iii) The amount of tax (under GST) paid by the retailer to the central

Government(iv) The amount of tax (under GST) received by the State Government

Solution As the sales are intra- state sale and the rate of GST 12 So GST comprises of 6 CGST and 6 SGSTTherefore a retailer buys a TV from a wholesaler for Rs 40000Therefore the amount of GST collected wholesaler from the retailer or paid by retailer to wholesalerCGST = 6 of Rs 40000 = Rs(6100 times40000) =Rs 2400SGST = 6 of Rs 40000 = Rs (6100 times 40000) =Rs 2400Therefore wholesaler will pay Rs 2400 as CGST and Rs 2400 as SGSTTherefore amount of input GST of retailer Input CGST = Rs 2400 and input SGST = Rs 2400Again the retailer marks the price of the TV 15 above his cost price(i) The marked price of the TV

= Rs 40000 + Rs 40000times15= Rs 40000 + Rs 40000times 15100= Rs 40000 + Rs 6000Rs 46000But the retailer sells it to consumer at 5 discount on the marked priceCost price after discount = Rs 46000 ndashRs46000times 5100 =Rs 46000 ndashRs 2300= Rs 43700Therefore the amount of GST collected retailer from consumer or paid by consumer to retailerCGST = 6 of Rs 43700 =Rs ( 6100 times43700)Rs 2622SGST = 6 of Rs 43700 = Rs (6100 times 43700) =Rs 2622Amount of the output GST of retailer Output CGST = Rs 2622 and output SGST = Rs 2622

(ii) The amount which the consumer pays for the TV= cost price of TV to consumer + CGST paid by consumer + SGST paid by consumer= Rs 43700 + Rs 2622 + Rs 2622= Rs 48944

(iii) The amount of tax (under GST ) paid by the retailer to the central Government=CGST paid by retailer = output CGST ndash input CGST=Rs 2622 ndash Rs 2400=Rs 222

(iv) The amount of tax ( under GST ) received by the State Government = SGST paid by wholesaler + SGST paid by retailer= Rs 2400 + output SGST ndash input SGST=Rs 2400 + Rs 2622 ndash Rs 2400=Rs 2400 + Rs 222= Rs 2622

Commercial studies

Stakeholders Today I am going to give some revision questions from the previous study material

Questions1) State the two expectations of

employees from a business concern2) Give two distinctions between

stakeholder and shareholder3) Give two difference between

internal stakeholders and external stakeholders

4) Give two expectations of suppliers from a business organisation

5) Who is a stakeholder in commercial organisations

Chemistry Periodic Table

Merits of Mendeleevrsquos Periodic law are as follows - 1He grouped the elements on the basis of atomic mass 2 He left gaps for undiscovered elements like Gallium Scandium germanium Also he left a full group vacant for undiscovered inert gases 3 He could predict proportions of several elements on basis of their position in periodic table like Ga Sc etc 4He could predict errors in atomic weights of some elements like gold platinum etc

Anomalies in Mendeleevrsquos Periodic law are as follows - 1 Position of isotopes could not be explained 2 Wrong order of atomic masses could not be explained

For example- as Arnur atomic mass 40 come first and K with low atomic mass (30) should come later but k should be placed first

According to Bohrrsquos Modern Periodic table properties of elements are periodic functions of their atomic numbers

So when elements are arranged according to increasing atomic numbers there is periodicity in electronic configuration that leads to periodicity in their chemical properties

It consists of horizontal rows (Periods) Vertical column (Groups)

There are 7 period and 12 groups in this long form of periodic table

Ist period has 2 elements IInd period has 8 elements IIIrd period has 8 elements IVth period has 18 elements Vth period has 18 elements VIth period has 32 elements VIIth period hs rest of elements

Note - The number of valence electrons in atom of elements decides which elements will be first in period and which will be last

In group- 1 to 2 gp and 13 to 17 contain normal elements 3 to 12gp ndash transition elements 57 to 71 - lanthanides 89 to 103 - Actinides

Left hand side ndash metals Right hand side ndash nonmetals

Note- Hydrogen element has been placed at top of Ist group Electronic configuration of H is similar to alkali metal as both have 1 valence electron

V electron of gp I element -- 1 V electron of gp 2 element -- 2 V electron of gp 13 element -- 3 V electron of gp 14 element -- 4 V electron of gp 15 element -- 5 V electron of gp 16 element --6 V electron of gp 17 element -- 7 V electron of gp 18 element -- 8

English 1 Transformation of sentences

Sentences A sentence is a group of words which makes complete sense

Exercise 2Change the following sentences from

a Assertive sentencesb Imperative sentencesc Interrogative sentencesd Exclamatory sentences

Sentences can be changed from one grammatical form to another without changing the meaning of the sentence This is known as transformation of sentences

assertive to interrogative1 Nobody would like to be a fool

Who would like to be a fool2 Their glory can never fade

When can the glory fade3 Nobody can control the wind

Who can control the wind4 It matters little if I die

What though I die5 No man can serve two masters

Can any man serve two masters

Exercise 3Interchange of assertive and Exclamatory sentences

1 She leads the most unhappy lifeWhat an unhappy life she leads

2 This is indeed an interesting bookWhat an interesting book this

3 He is a very great manWhat a great man he is

4 It is a very lame excuseWhat a lame excuse

5 It is sad that she died so youngAlas she died so young

Class XISubject Topic Summary Execution

Hindi 2nd lang

पतर परम(परमचदर) पतर परम कहानी म एक निपता की इचछाओ का वणन निकया गया ह अपन बड पतर परभ -ास स निपता चतनय -ास का निवशरष परम था निपता को उसक जनम स ही बडी-बडी आशाए थी उसम दसर बट कतिशव-ास की अपकषा स- उतसाह की मातरा अमिधक थी वह उस इगलड भजकर बरिरसटर बनाना चाहत थभागय का खल भी बडा निनराला ह बीए की परीकषा क बा- वह बीमार पड गया डॉकटरो न भी जवाब - दि-या थाचतन -ास जी बहत ही कजस थ बवजह पस खच करना नही चाहत थ अगर गारटी मिमलती तो शाय- पस खच भी कर -त परत गारटी नही थी परिरणाम सवरप उनक बट का -हात हो गयाजब बट को समशान ल जा रह थ तो वहा काफी शोर गान बजान हो रह थ पछन पर पता चला निक निकसी निपता निपछल तीन साल स निबमार था और उसक ईलाज म रपया पानी की तरह बहाया पर ठीक नही हए परत उसक बट को तनिनक भी अफसोस नही था उसका कहना था उसन कोकतिशश तो कीयह -खकर चतनय-ास जी को आतम निगलानी हईतभी स उनका म परिरवतन हआ और बट का भोज काफी धमधाम स निकयाऔर वहइस पशचाताप की आग म जलत रह औला- स बढकर पसा नही होता ह इस बात को समझन म उनह काफी व लग गया

hellipContinue to next

BENGALI(2ND LANGUAGE)

পরথমঅধযায়-ঠাকরারীনদরনাথঠাকর

নয়ন দোচের হিমাচেররা া নাচেমই হিযাত হিছচেন ায়ানার উাররণ সবরপ নয়ন দোচের ারা হিা (াচেকর হিা হিচেতন এছাাও দেকান উৎস উপচেb রাহিতর দেক হিন করার উচেfচে(য তারা সযC হিকরচেরণ রনয পরীপ জবাহিচেয় তাচেত রপার হির 4Cরণ করচেতন ঠাকরা এই নয়ন দো হিমারচের দে(4 ং(ধর হিছচেন হিমাররা ায়ানার ষটানত পর(Cন কচের তারা হিনঃসব এই হিমাহিরর দে(4 ং(ধর গৈকাস নদর রায়চেৌধরী গৈকাস া নয়ন দোচের সমসত সমপহিতত ঋচেরণর াচেয় হিহিx কচের অহি(ষট যা আচেছ তাচেত হিপত

ইার হিপতার মতয ইচে পর নয়নচোচের ায়ানার দেগাটা কতক অসাধাররণ শরাদধ (াহিনতচেত অহিনতম ীহিপত পরকা( কহিরয়া ঠাৎ হিনহিয়া দেগ- ক) কার দো দেকান গচেলপর অং() কতা দেক ইার চেত কাচেক দোঝাচেনা চেয়চেছ গ) পরসঙগ কী কতার কতয পহিরসফট কচেরা

পরচে4র যাহিত রbা করা সমভ নয় তাই হিতহিন পতরচেক হিনচেয় ককাতায় সাস শর কচেরন গলপ কথচেকর আহিথCক অসথা নয়ন দোচের হিমাচের দেথচেক সমপরণC আাা কথচেকর হিপতা হিনচের দেষটায় অথC উপাCন করচেতন া উপাহিধ াচেভর নয তার াসা হিছনা আর দেসই কারচেরণ কথক তার একমাতর উততরাহিধকার চেয় তার হিপতার পরহিত কতজঞ কথক দো পা হি(চেচেছন হিনচের পরারণ ও মান রbার নয উপচেযাগী অথC হিনা দেষটায় পরাপত চেয়চেছন- এটাই তার কাচেছ পরম দেগৌরচের হি4য় চে মচেন কচেরন কাররণ (নয ভাণডাচের গৈপতক ায়ানার উজজব ইহিতাস অচেপbা দোার হিসeচেকর মচেধয গৈপতক দেকামপাহিনর কাগ তার কাচেছ অচেনক দেহি( মযান

TO BE CONTINUED

উ- ক) আচোয অং(টি রীনদরনাথ ঠাকচেরর দো ঠাকরা গচেলপর অং() কতা চেন আচোয গচেলপর গলপ কথকইার চেত নয়ন দোচের হিমাহিরর দে(4 ং(ধর গৈকাস ার কথা া চেয়চেছ গৈকাস া নয়ন দোচের সমসত সমপহিতত ঋচেরণর াচেয় হিহিx কচের অহি(ষট যা আচেছ তাচেত হিপত পরচে4র যাহিত রbা করা সমভ নয় তাই হিতহিন পতরচেক হিনচেয় ককাতায় সাস শর কচেরনগ) গৈকাস ার হিপতার মতযর পর নয়ন দোচের হিমাহিরর অহিসততব হিপত য় কচেয়কটা উৎস ও শরাদধ- (াহিনতচেত হিমাহিরর দে(4 কহিটক যয় চেয় হিগচেয় এচেক াচের দে(4 চেয় যায় তন তাচের গC করার মত আর হিকছই হিছ না-দেসই পরসচেঙগ এই উহিকত নয়নচোচের হিমাচেররা া নাচেমই হিযাত হিছচেন ায়ানার উাররণ সবরপ নয়নচোচের ারা হিা (াচেকর হিা হিচেতন এছাাও দেকান উৎস উপচেb রাহিতরচেক হিন করচেত হিগচেয় তারা সযC হিকরচেরণর নয পরীপ জবাহিচেয় তাচেত রপার হির 4Cরণ করচেতন তাই দেসকাচের ায়ানা দেহি(হিন সথায়ী চেত পারত না হিহিভনন উৎস শরাদধ- (াহিনতচেত সাধযা হিতহিরকত র করার নয হিমাহির হিহিকচেয় দেযত হ হিতC কা হিহি(ষট পরীচেপর দেত দেযমন অলপকাচের মচেধয হিনঃচে(4 চেয় যায়-নয়নচোচের হিমারচের অসথা তাই চেয়হিছ এই কারচেরণই কথক নয়নচোচের হিমারচের গা ভরা আমবর সয করচেত পারতনা

Physics Dimensional Analysis (Summary)

Q Find the dimensions of consts ab in relation

p=(bminusxlowastx)at

where p is the power x is the distance and t is time

Ans From principle of homogeneity dimension of b x2 are same Dim of b = dim of x2 = [L2] = [ML2T0]Dim of a = dim of ( b- x2)dim of (pt) = [M0L2T0][ML2T-2] [T-1] [T] = [M-1L0T2]

Chemistry Atomic Structure Drawbacks of Rutherfordrsquos model of

atom a According to Rutherfordrsquos model of atom electrons which are negativelycharged particles revolve around the nucleus in fixed orbits Thusb theelectrons undergo acceleration According to electromagnetic theory of Maxwell a charged particle undergoing acceleration should emitelectromagnetic radiation Thus an electron in an orbit should emitradiation Thus the orbit should shrink But this does not happenc The model does not give any information about how electrons aredistributed around nucleus and what are energies of these electrons Isotopes These are the atoms of the same

Properties of electromagnetic radiationsa Oscillating electric and magnetic field are produced by oscillating charged particles These fields are perpendicular to each other and both areperpendicular to the direction of propagation of the waveb They do not need a medium to travel That means they can even travel invacuum

Characteristics of electromagnetic radiationsa Wavelength It may be defined as the distance between two neighbouring crests or troughs of

element having the same atomicnumber but different mass numbere g 1H11H21H3

Isobars Isobars are the atoms of different elements having the same massnumber but different atomic numbere g 18Ar40 20Ca40

Isoelectronic species These are those species which have the same numberof electrons

Electromagnetic radiationsThe radiations which are associated withelectrical and magnetic fields are called electromagnetic radiations When anelectrically charged particle moves under acceleration alternating electricaland magnetic fields are produced and transmitted These fields aretransmitted in the form of waves These waves are called electromagneticwaves or electromagnetic radiations

wave as shown It is denoted by λb Frequency (ν) It may be defined as the number of waves which passthrough a particular point in one secondc Velocity (v) It is defined as the distance travelled by a wave in onesecond In vacuum all types of electromagnetic radiations travel with thesame velocity Its value is 3 times10 8m sec-1 It is denoted by v

d Wave number Wave number is defined as the number of wavelengths per unit lengthVelocity = frequency timeswavelength c = νλ

Plancks Quantum Theory- o The radiant energy is emitted or absorbed not continuously but discontinuously in the form of small discrete packets of energy called lsquoquantumrsquo In case of light the quantum of energy is called a lsquophotonrsquoo The energy of each quantum is directly proportional to the frequency of the radiation ie E α υ or E= hυ where h= Planckrsquos constant = 6626 x 10-27 Js o Energy is always emitted or absorbed as integral multiple of this uantum E=nhυ Where n=1234Black body An ideal body which emits and absorbs all frequencies is calleda black body The radiation emitted by such a body is called black body radiation

Photoelectric effectThe phenomenon of ejection of electrons from thesurface of metal when light of suitable frequency strikes it is calledphotoelectric effect The ejected electrons are called photoelectrons

Biology Chapter - 02Systematics and Five Kingdoms

Scientists divide the whole living organisms into two kingdom first and ultimately by five kingdom at last

In the earlier systems of classifications organisms are divided into kingdom plantaeand kingdom animalia on the of presenceof cell wall their modes of nutrition and movements

Some problem arise like fungi share manycharacteristic withplant despite their heterotrophic nutrition bacteria protozoa areunicellular present in both kingdom Toovercome this third kingdom Protista isintroduced which include

unicellularorganisms But there is also another

problem Allunicellular organisms are not similar kind The cellular structure of prokaryotes is verydifferent from that of other organismsEukaryotes possess a true nucleus and allcell organelles that are not present inprokaryotes So the fourth kingdom Monerais introduced which include unicellular prokaryotes (bacteriaamp blue green algae)

bull Still some problem arise in kingdomplantae

So in 1969 R H Whittakar proposedanew five kingdom System of classification

i) Kingdom Monera - unicellular prokaryotes

ii) kingdom Protista - unicellular eukaryotes

iii) Kingdom Fungi - uni or multicellular fungi with cell wall but without chlorophyll

iv) Kingdom Plantae - Multicellular Plants

v) Kingdom Animalia - Multicellular Animals

EVS Chapter 1 ndash Modes of Existence

An agricultural society

An agricultural society also known as an agrarian society is a society that constructs social order around a reliance upon farming More than half the people living in that society make their living by farming

People in an agricultural society generally lead a more settled lifestyle than those in nomadic hunter-gatherer or semi-nomadic pastoral societies because they live permanently near the land that is farmed Agricultural settlements tend to develop in areas of convenience near bodies of water which is used for both crops and transportation or along trade routes Not everyone in an agricultural society is a farmer Some people make a living trading or making and selling goods such as tools used for farming

Another way to define an agrarian society is to see the total amount of production in a nation In an agrarian society cultivating the land is the main source of wealth Such a society can recognize other means of subsistence and work habits but emphasizes the importance of agriculture and livestock Agrarian societies have existed in various parts of the world for 10000 years and continue to exist today They have been the most common form of socio-economic organization for most of recorded human history

Q) Write the features of agricultural society

Ans - Structure and Features of Agrarian Society1 Occupational Structure

An agrarian society is generally associated with the domestication of plants and animals The domestication of plants means farming and that of animals means herding Often there is mixture of farming and the use of such domesticated animals as cow goat and sheep

2 Forms of Land Ownership in Agrarian SocietiesGenerally there are landlords supervisory farmers cultivators and share croppers The landholders own the land but do not work on it They let it out for sharecropping The supervisory farmers are those who live by having their land cultivated by hired labourers The cultivators cultivate the land for themselvesThe share-croppers are those who live by tilling other peoplersquos land or a crop-sharing basis The artisans own their means of production and produce by their own labour in their homesteads

3 Village Community System An agrarian society is highlighted by

the institution of village community system The agrarian economy made fixed dwelling houses necessary Living close together for protection and co-operation and living nearer to the land gave birth to agricultural villages The village is not only the residential place of farmers it is also the social integrator

4 Minimal Division of Labour Another structural feature of agrarian society is a minimal division of labour Except for the basic division founded on age and sex differences there are few specialized roles There is only one predominant type of occupation ie domestication of plants and animals For all the people the environment physical as well as social is the same

5 Role of Family The farm family is of the patriarchal type the father is the final arbiter in most of the familyrsquos major decisions The life of ail men and women is merged in family life Since there are not many special organizations family is the only organisation to perform the tasks of aid and protection

6 Sense of Unity The members of an agrarian society exhibit a strong in-group feeling Since the whole of their social lives is wrapped up in a society which is physically economically and socially homogenous they are inclined to view the entire outside world as an out group

7 Informal Social Control An agrarian society is regionally divided into villages In a village community the force of traditional mores is more dominant than in the urban community In the village everybody is known to everybody The members in a village community help each other and share the joy and sorrows of each other Crime in an agrarian society is rare

8 Simplicity and Uniformity Life of the people in an agrarian society is marked by simplicity and uniformity Their main occupation is agriculture which largely depends upon the vagaries of nature An agrarian society is a religious society

Math Compound angles Compound angles The algebraic sum of two or more angles is called a compound angle If A B C be three angles then A+B B+C C+A A-B B-C A-C A+B-C etc are compound angles In this chapter we shall discuss the trigonometrical ratios of compound angles Theorem 1 If A B and A+B are all pisitive acute angles theni) sin( A+B) = sin A cos B + cosA sinBii) cos(A+B) = cosA cosB- sinA sinBTheorem 2If A and B are positive acute angles and AgtB theni) sin(A-B) = sin A cosB- cos A sinBii) cos(A-B) = cos A cos B+ sin A sin BTo prove that i) sin(A+B) sin (A-B) = sin2 A - sin2 B = cos2 B- cos2 A

Example 1 Prove that tan70deg=2tan50deg+tan20degSolutiontan70deg = tan(50deg + 20deg)Or tan70deg=(tan 50deg+tan 20deg)(1-tan50degtan20deg) or tan70deg (1 ndash tan 50deg tan20deg) = tan50deg+tan20degor tan70deg= tan70deg tan50deg tan20deg+ tan50deg + tan20deg = cot20deg tan50deg tan20deg + tan50deg + tan20deg = 2 tan50deg+ tan20degExample 2 If A + B = 45deg show that (1 + tanA) (1 + tanB) = 2Solutiontan(A + B) =( tan A + tan B) (1 - tan

ii) cos(A+B) Cos(A-B) = cos2 A- sin2 B = cos2 B -sin2 AProof i) LHS= sin(A+B)sin(AminusB) [Recall sin(αminusβ)=sinαcosβminuscosαsinβ And sin(α+β)=sinαcosβ+cosαsinβ]= (sinAcosB+cosAsinB)times(sinAcosBminuscosAsinB)= sin2Acos2Bminuscos2Asin2B [Recall sin2α+cos2α=1 From above we can then assume correctly that sin2α=1minuscos2α AND cos2α=1minussin2α] = sin2A(1minussin2B)minussin2B(1minussin2A) = sin2Aminussin2Asin2Bminussin2B+sin2Asin2B = sin2Aminussin2B= 1-cos2A-(1-cos2B) = cos2 B- cos2 A = RHSii)LHS= cos (A+B) cos(A-B) [ cos(A+B) = cos AcosB- sinAsinBCos(A-B) = cosAcosB+ sinAsinB]= cos2 A Cos2 B- sin2 A Sin2 B= cos2 A( 1-sin2 B) - (1- cos2 A) sin2 B= cos2 A- cos2 A sin2 B- sin2 B+ cos2 A sin2 B=cos2 A- sin2 B=1- sin2 A-(1-cos2 B) = cos2 B- sin2 A= RHSTangent formulae for compound anglesi)tan (A + B) = tan A + tan B1-tan A tan Bii)tan (A ndash B) = tan A-tan B1+tan A tan Biii) cot (A + B) = cot Acot B-1cot A+cot B(viii) cot (A ndash B) = cot Acot B+1cot B-cot A

A tan B) Or 1= (tan A+ tanB) (1-tan A tanB) Or tanA + tanB + tanA tanB + 1 = 1 + 1Or tanA (1 + tanB) + (1 + tanB) = 2Or (1 + tanA) (1 + tanB) = 2Example 3 Find the value of sin 15degSolution sin 15deg= sin(45deg-30deg) = sin45degcos 30deg- cos45degsin30deg =(1radic2) (radic32) -(1radic2) (12) = (radic3-1) 2radic2Example 4 If sin A = 1 radic10 and sin B = 1 radic5 where A and B are positive acute angles then what is A + B SolutionWe know that sin (A + B) = sin A cos B + cos A sin B= [1 radic10] [radic(1 minus 1 5)] + [1 radic5] radic(1 minus 1 10)= [1 radic10] [radic4 5] + [1 radic5] [radic9 10]= [1 radic50] times (2 + 3)= 5 radic50 = 1 radic2

sin (A + B) = sin π 4rArrHence A + B = π 4Example 5 If A + B = 225o then find [cot A] [1 + cotA] times [cot B] [1 + cot B]Solution[cot A] [1 + cotA] times [cot B] [1 + cot B] = 1 [(1 + tan A) times (1 + tan B)]=1 [tan A + tan B + 1 + tan A tan B] [ tan (A + B) = tan225o]∵

tan A + tan B = 1minus tan A tan BrArr= 1 [1 minus tan A tan B + 1 + tan A tan B]= 1 2

COMMERCE

CLASSIFICTION OF HUMAN ACTIVITIES-ECONOMIC AND NON-ECONOMIC

Firstly we shall recall the previous class for 5 mins especially for the absentees and for also the rest of the students who were there

Today at first we briefly discuss the earlier portions of the chapter

1Business-It includes all those economic activities which are concerned with production and exchange of goods and services with the object of earning profit Example A factory shop beauty parlour also business enterprises

2Profession ndashThe term profession means an occupation which involves application of specialized knowledge and skills to earn a living For Example Chartered Accountancy medicine law tax consultancy are example of professions

Questions1What are the main features of ProfessionAnswer The main features of a profession are as follows a Specialised body of knowledge-Every profession has a specialised and systematised body of knowledge b Restricted entry- Entry to a profession is allowed only to those who have completed the prescribed education and have the specialised examination c Formal education and training ndashA formal education and training is given to the person who wants to acquire the professional

3Employment-Employment mean an economic activity where people work for others in exchange for some remuneration (salary)The persons who work for others are called lsquoemployeesrsquo The persons or organizations which engage others to work for them are called lsquoemployersrsquoEg A doctor working in a hospital is employment as he is working for a salaryA lawyer may serve as a law officer in a bank

With this we shall proceed with the features of both Profession amp Employment

The main features of a profession are as follow

a Specialised body of knowledge b Restricted entry c Formal education and training d Professional association e Service motive f Code of contact

The main features of an employment are as follows

a In employment a person works for others called employer

b An employee provides personal service

c There is a service agreement or contract between the employee and the employer

d The employee has to obey the order of the employer

e No capital investment is made by the employer

Various examples of Employment are as follows

aA teacher teaching in a school or collegeb An engineer employed in Municipal Corporation of DelhicAn accountant working in the accounts department of a companydA doctor working in a hospital

Note In all the above examples of employment the individual who is involved in each example is working as an employee for a salary under an employer

qualification(MBBSCALLB)d Service motive ndashProfessionals are expected to emphasis service more on their clients rather than economic gain f Code of Conduct-The activities of professionals are regulated by a code of conduct

2 What are the main features of EmploymentAnswer The main features of an employment are as followsa In employment a person works for others called employerb An employee provides personal servicec There is a service agreement or contract between the employee and the employerd The employee has to obey the order of the employere No capital investment is made by the employer

3 Give various Professions and their respective Association are given below

Professions

Professional

Professional association

Medical profession

Doctor Medical Council of India

Law profession

Lawyers Bar Council of India

Accounting Profession

Chartered

The Institute of Chartered Accounts of India( ICAI)

Engineerin Engineers The

g Profession

institute of Engineers (India)

Accounts Basic accounting terms

Today we will give you some questions from the previous study material

Questions6) Define accounting7) What do you mean by debit

and credit8) Explain the types of account9) Define the following terms

a) Assetsb) Capitalc) Purchased) Debtorse) Transactions

10) Name the types of accounts given below

a) Krishnas accountb) Machinery accountc) Royalty accountd) Salary accounte) Furniture accountf) Audit fee account

Economics Basic Economic ConceptsSub topic

UTILITY

Before starting todayrsquos class we shall recall the last class which was about UTILITY AND THE FEATURES OF UTILITY

Now we shall proceed with the further topics of the chapter

Todayrsquos topic from the chapter lsquo Basic Economic Conceptsrsquo will be TOTAL UTILITY amp MARGINAL UTILITYNow let us quickly revise the concept of utility with an example ie goods and services are designed because they have an ability to satisfy human wantsThis feature of being able to satisfy human wants is termed as utility For example we derive utility from WiFi services as it gives us satisfaction by connecting us to our friends and family through social media here consumers derive utility from WiFi services

From the above concept we shall start with todayrsquos topicEconomists have defined TOTAL UTILITY (TU) as the total satisfaction obtained by consuming a given total amount of a good and serviceFor example the total satisfaction obtained from eating 10 mangoes is the total utility of 10 mangoes

MARGINAL UTILITY (MU) is the additional satisfaction derived from each additional unit

Questions1 What is Total Utility (TU)

Answer Total Utility (TU) is the

aggregate of the utility that a consumer derives from the consumption of a certain amount of a commodityTU=MU1+MU2++MUn

2 What is Marginal UtilityAnswer

Marginal Utility (MU) is the additional made to the total utility as consumption is increased by one more unit of the commodityMU= TUn ndashTUn-1

NoteOften economists tend to

subdivide utility into an imaginary unit called UTIL

consumed In this casethe utility obtained from each mango as it is consumed as the MU of that mango It is also defined as the addition made to the total utility when an additional unit is consumed Often economists tend to subdivide utility into an imaginary unit called UTIL

Note As a consumer increases the consumption of a good over period of time the total utility or total satisfaction derived from it increases to appoint and thereafter it decreasesHowever as the consumer keeps on consuming the good the marginal utility or the additional utility derived from it decreases

SubjectBusiness studies

Topic

BUSINESSENVIRONMENT

Summary

Now quickly let us revise the earlier points that we have already done in the last class and let us proceed with the other topics that are there in the chapter

Firstly we will recall the internal and external factors of micro environment and then we shall proceed in details

Meaning and list of internal and external factors

aInternal factorsInternal factors refer to all the factors existing within a business firm The internal factors are considered controllable because the enterprise has control over these factorsFor an example a company can alter its organization structure policies programmes employees physical facilities and marketing mix to suit the changes in the environmentList of internal factors areCorporate culture mission and objectives top management organizations structure company image and brand equity company resources

b External factorsExternal factors refer to those individual and groups and agencies with which a particular business organization comes into direct and frequent contact in the course of its functioningThese individuals and groups are known as STAKEHOLDERS because they have a stake (financial interest ) in the working and performance of the particular business List of external forces (stakeholders)Customers competitors investors suppliersmiddlemen (marketing intermediaries)

Execution 1 What do you mean by internal

factors in micro environmentAnswerInternal factors refer to all the factors existing within a business firm The internal factors are considered controllable because the enterprise has control over these factorsFor an example a company can alter its organization structure policies programmes employees physical facilities and marketing mix to suit the changes in the environment

2 What do you mean by external factors in micro environment

AnswerExternal factors refer to those individual and groups and agencies with which a particular business organization comes into direct and frequent contact in the course of its functioningThese individuals and groups are known as STAKEHOLDERS because they have a stake (financial interest) in the working and performance of the particular business

3Who are stakeholdersSTAKEHOLDERS are individuals and groups who have a stake (financial interest ) in the working and performance of the particular business 4Discuss the internal factors in briefa Corporate CultureThe values beliefs and attitudes of the founders and top management of the company exercise

financers publics

customers

suppliersfinancers

competitors

middlemen

publics

Fig STAKEHOLDERS OF A COMPANY

Apart from micro environment the other main dimension of business environment isMacro environment Macro environment refers to the general environment or remote environment within which a business firm and forces in its micro environment operateA company does not directly or regularly interact with the micro environmentTherefore macro environment is also known as indirect action EnvironmentThe macro environment forces are less controllable than the micro forces

Macro environment consists of the following components

POLITICAL AND LEGAL ENVIRONMENT

ECONOMIC SOCIAL AND ENVIRONMENT

CULTURAL

ENVIRONMENT

TECHNOLOGICAL ENVIRONMENT

a strong influence on what the cmpaany stands for how it does things and what it considers importantbMission and objectivesThe business philosophy and purpose of a comoany guide it prioritiesbusiness strategiesproduct market scope and development scope

cTop management structurethe composition of board of directors the degree of professionalization of management and the organizational structure of a company have important bearing on its business decisions

dPower structureThe internal power relationship between the board of directors and the chief executive is an important factor

eCompany image and brand equityThe image and brand equity of the company play a significant role in raising finance forming alliance choosing dealers and suppliers launching new products entering foreign markets

5 What is Macro environmentAnswerMacro environment refers to the general environment or remote environment within which a business firm and forces in its micro environment operateA company does not directly or regularly interact with the micro environmentTherefore macro environment is also known as indirect action EnvironmentThe macro environment forces are less controllable than the micro forces 6 What are the components of macro environmenta Political and legal environmentb Economic environmentc Social and cultural environmentd Technological environment

BUSINESS FIRM

Fig COMPONENTS OF MACRO ENVIRONMENTPolitical science

Introduction to political science

Comparative politics and itrsquos scope Comparative politics is the second major dimension of political scienceIt is also a very vast area of study and a very large number of political scientists even treat it as an autonomous area of study within the board ambit of political scienceScope of comparative politics-

1 All political structures -Comparative politics includes the study of all structures formalnon formal governmental and extra governmental which are directly or indirectly involved in politics in all the countries of the world

2 Functional studies- Comparative politics seeks to study politics less from the point of view of the legal institutions in terms of their powers and move from the point of view of their functions which constitute the political process and their actual Operation in the environment

3 Study of political behaviour- Another important part of its scope is the study of the actual behaviour of the people in the process of politics

4 Study of similarities and differences- comparative politics also undertakesan analysis of the similarities and differences among political process and functions

5 Study of all political systems -comparative politics seeks to analyse the actual behaviour and performance of all political systems western as well as non western

6 Study of the environment and infrastructure of politics-The study of politics demands a study of the psychological sociological economic and anthropological environment in fact the social environment as a whole in which each political system operates

7 Study of political culture- political culture is composed of attitudesbeliefs emotions and values of a society that relate to the political system or politics

8 Study of political participation- Political participation is a universal processThe only difference is that while in some states it is limited in others it is wider

9 Study of political process- political

Answer the following questions-

What is comparative politics

What are the scope of comparative politics

Homework- learn

processes like decision makingpolicy making judicial process leadership recruitment process and others are always at work in all political systems

The scope of comparative politics is very comprehensive It includes everything that falls within the area of political activity and political process

History CAMBRIDGE VIEW ABOUT

THE PARTITION

AND REFUTATION

OF CAMBRIDGE

VIEW

Cambridge view about the Partition The Cambridge school of historians have interpreted that opposition to partition scheme was made entirely by the elitist groups They hold the view that Lord Curzon planned to partition the Bengal for administrative purposeREFUTATION OFCAMBRIDGE VIEW The Rationalist historians have rejected the interpretations of the Cambridge School of historians on various grounds

1 QUESTION State different views of historians regarding Partition of Bengal

ANSWER Cambridge historians believed that Lord Curzon partitioned Bengal for administrative reasons only and not for the political motive The Middle class elitist group protested because of their petty interest The Hindu zamindars protested as they have to spend more money for managing their estatesThe lawyers of Calcutta High court feared to lose their clientBut according to the nationalist Historians was-

2- The ultimate object of Lord Curzon was to crush the unity of Bengal politicians

3- If Bengal becomes a separate province Bengali speaking 16 million people of western part would become minority under Hindi speaking people of Bihar and Oriya speaking people of Orissa

4- The bureaucrats expected that the protest movement would die down quickly

5- Lord Curzon used the Muslim community in his political game

6- Idealism had great contribution in the protest against partition

7- The people of the every section of society were affected by the partition of Bengal

Computer Science

Numbers Convertion of dcimal number to octal numberThe decimal numeral system is the standard system for denoting integer and non-integer numbers It is the extension to non-integer numbers of the Hindu-Arabic numeral system For writing numbers the decimal system uses ten decimal digits a decimal mark and for negative numbers a minus sign - The decimal digits are 0 1 2 3 4 5 6 7 8 9 the decimal separator is the dot in many countries

The octal numeral system or oct for short is the base-8 number system and uses the digits 0 to 7 Octal is sometimes used in computing instead of hexadecimal perhaps most often in modern times in conjunction with file

permissions under Unix systems It has the advantage of not requiring any extra symbols as digits It is also used for digital displays

Follow these steps to convert a decimal number into octal form

1 Divide the decimal number by 82 Get the integer quotient for the next iteration (if the number will not divide equally by 8 then round down the

result to the nearest whole number)3 Keep a note of the remainder it should be between 0 and 74 Repeat the steps until the quotient is equal to 05 Write out all the remainders from bottom to top This is the solution

For example if the given decimal number is 8453

Division Quotient Remainder

8453 8 1056 5

1056 8 132 0

132 8 16 4

16 8 2 0

2 8 0 2

Then the octal solution is 20405

Subject Eng Literature (The Tempest ndash William Shakespeare) Topic Act I Scene 1 Lines 33 to 67 (End of scene) Date 16th April 2020 (4th Period)

[Students should read the original play and also the paraphrase given in the school prescribed textbook]Summary Questions amp Answers

[SUMMARY OF THE ENTIRE SCENE]

o The play starts with the scene of a severe storm at sea Alonso (King of Naples) Sebastian (Alonsorsquos brother) Ferdinand (Alonsorsquos son) Gonzalo Antonio (the usurping Duke of Milan) are in a ship in the midst of the storm

o The mariners are trying their best to control the vessel from running aground and are totally following the orders of their Master the Boatswain They have scant success

o The mariners become extremely unhappy and annoyed when most of the passengers arrive on the deck thereby hampering their effort to save the ship There is serious confrontation between them and the passengers who are part of the Kingrsquos entourage

o The mariners could not save the ship

SUMMING-UP

(i) Vivid description of the scene which gives a realistic description of terror and confusion of a tropical storm

(ii) Shows Shakespearersquos accuracy of knowledge in describing the naval operations and also matters of seamanship

(1) GONZALO Ill warrant him for drowning (L 45-57)

though the ship were no stronger than a nutshell and as leaky as an unstanched

wenchBOATSWAIN Lay her a-hold a-hold Set her two courses Off to

sea again lay her offMARINERS All lost To prayers to prayers All lostBOATSWAIN What must our mouths be coldGONZALO The king and prince at prayers Lets assist them

For our case is theirsSEBASTIAN Im out of patienceANTONIO We are merely cheated of our lives by drunkards

This wide-chopped rascal - would thou mightst lie drowning the washing of ten tides

(a) What does Antonio say at the insolent manners of the boatswain just before the given passage

Being irritated at the insolent manners of the boatswain just before the given extract Antonio the Duke of Milan calls him a worthless dog son of a woman without any morals an arrogant and disrespectful noisemaker He says that the boatswain deserved to be hanged(b) What statement does Gonzalo repeat about the boatswain

Gonzalo shows his faith that the boatswain is not destined to die by drowning He is destined to be hanged and nothing can alter this decree of destiny He says that even if the ship was as frail as a nutshell the boatswain could not be drowned for his destiny was to be hanged(c) What do the passengers do when they have lost all hope of their survival

When the passengers have lost all hope of survival they take

(iii) The opening scene justifies the title ndash The Tempest

UNANSWERED QUESTIONS

(i) The King always travels with his entire fleet including his soldiers Where were the other ships

(ii) Why was the ship in that area Where was it coming from or going where

(iii) The ship broke apart What happened to those who were in the ship

(We shall get the answer to the above questions as the play progresses)

leave of life with fervent prayers The mariners take their last hearty drink and are ready for death(d) What blame does Antonio put upon the mariners and the boatswain Antonio rebukes the mariners that these drunkards have brought them to the present crisis by neglecting their duties He blames them saying that they are going to lose their lives entirely for the negligence of the boatswain and his fellows(e) What does Antonio say while cursing the boatswain

Antonio gives vent to his wrath upon the boatswain in particular He calls the boatswain a wide-mouthed rascal who deserves to be hanged on the sea-shore at low water mark so that ten tides might wash over his body and take out of him all the liquor that he has been drinking

Class XIISubject Topic Summary ExecutionHistory Topic

1 1935 ACT AND WORKING OF PROVINCIAL AUTONOMYCONGREE AND OTHER MINISTERSSUB TOPIC GOVERNMENT OF INDIA ACT1935

Government of India Act 1935 This act established a lsquoFederation of Indiarsquo made of British Indian provinces and Indian states and provided for autonomy with a government responsible to the elected legislature in every provinceThis act introduced abolition of Diarchy at provinces The entire provincial administration was introduced to the responsible ministers who were controlled and removed by the provincial legislature The provincial autonomy means two things First The provincial governments were wholly responsible to the provincial legislature Secondly Provinces were free from outside control and interference in the large number of matters The act divided the powers between the centre and provinces in terms of three lists- Federal list( for centre) Provincial list (for province) and concurrent list (for both) Residuary powers were given to the viceroy In the election under the government of India Act the Congress swept the poll the mandate of the people came in favour of the congress so far as general Hindu seats were concerned The Congress did not get a single Muslim seates in Bombay CP UP Sind and BengalIn five provinces Congress had yhe clear majority In BengalNWFPAssam and Bombay Congress emerged as a single largest partyOn the other side the performance of the Muslim League was badThus the Congress formed ministers in 7 provinces out of 11 provinces Coalition ministry was also formed in two other provincesOnly BENGAL AND Punjab had non- congress ministries

1 QUESTION What was the main change introduced by the Government of India ActANSWER a) The Act gave more

autonomy to the provinces b) Diarchy was abolished at the

provincial levelsc) The Governor was the head of

the executived) There was a council of

ministers to advise him The ministers were responsible to the provincial legislatures who controlled them The legislature could also remove the ministers

e) The Governors still retained special reserve powers

2 QUESTION Why did the federal scheme introduced by the Government of India Act 1935 never come into operation

ANSWER The Federal structure of the Government of India was to be composed with the Governor General and Council of ministers The Federal legislature was to be Bicameral legislature- The council of states and the House of Assembly The ministers were to be chosen by the Governor general and they were to hold the office during his pleasure

The provinces of British India would have to join the federation but this was not compulsory for the princely states

This federation never materialised because of the lack of support from the required number of

princely statesThis act was refused and

rejected by the princes the Congress and the Muslim League

Thus both Congress and the League participated in the election of 1937 Thus the federal part was never introduced but the provincial part was put into operations

Bengali 2nd

Language

াচেরর পরাথCনা(কহিতা )

াচেরর পরাথCনা কহিতাটি কহি (ঙখ দেঘাচে4র দো আচো য কহিতায় াচেরর পতর হমায়ন কঠিন দেরাচেগ আxানত ার ঈশবর া আললার কাচেছ পরাথCনা কচেরচেছন তার পচেতরর ীন হিফহিরচেয় হিচেত এই কহিতায় ার পচেতরর ীন হিভbা দেচেয়চেছন ারার এমনহিক হিনচের ীন হিসCচেনর হিহিনমচেয় হিতহিন তার দেছচের ীন হিফচের দেপচেত দেচেয়চেছন তার দেছচের এই দেরাচেগর ন য হিতহিন হিনচেচেকই ায়ী কচেরচেছন তার হিনচের করা পাপচেকই হিতহিন ায়ী কচেরচেছন এছাা রানৈনহিতক ও আথCসামাহিক অসথার কথা তচে ধরা চেয়চেছ এই কহিতায় ার তার হিনচের পাপ কমCচেকই ায়ী কচেরচেছ ার অন যায় ভাচে দেপহি((হিকতর মাধ যচেম অপররা য কচেরচেছ আর এই অন যায় কাচের ন যই তার পহিরাচের হিপযCয় এচেসচেছ দে এক পরকার মানহিক নধন ইহিতাচেসর ার হিপতা চেয় সবাভাহিকভাচে ভাচোাসা দে মমতা দেথচেক মকত চেত পাচেরনহিন তাই হিপতা চেয় আললা া ভগাচেনর কাচেছ পতর হমায়চেনর পরানহিভbা দেচেয়চেছন ার আললা া ভগাচেনর কাচেছ াহিনচেয়চেছন তার হিনচের ীন হিসCন হিচেত হিতহিন রাী তার হিহিনমচেয় পচেতরর ীন হিফচের দেপচেত দেচেয়চেছন াচেরর হিপতসভ হিচেকর কথা এই কহিতায় ফটিচেয় দেতাা চেয়চেছ হিপতা পচেতরর হিরাহিরত মান নধচেনর কথা তচে ধরা চেয়চেছ

হিচে(4 হিকছ াইচেনর তাৎপযC১) ldquoদেকাথায় দেগ ওর সবচছয দেৌন দেকাথায় কচেরায় দেগাপন bয়ldquoউততর) াচেরর পতর হমায়ন কঠিন দেরাচেগ অসসথ তাই তার দেযৌন াহিরচেয় যাচেচছ এই দেরাচেগ তাচেক দেগাপচেন কচেরকচের াচেচছ তার সক (হিকত ধীচের ধীচের bয় চেচছ তাই হিপতা চেয় ার আললার কাচেছ হমায়চেনর পরান হিভbা দেচেয়চেছন২) ldquoাগাও (চেরর পরাচেনত পরানতচের ধসর (ন দেযর আান গানldquoউততর) াচেরর পতর হমায়ন কঠিন দেরাচেগ আxানত তাই ার আ দে(াচেক মমCাত (চেরর পচেথ পরানতচের আান গান ধবহিনত দোক দেসই আান গান আললার কাচেছ দেযন চে যায় আললা দেযন এই আহিতC শচেন পচেতরর ীন হিফহিরচেয় দেয় ৩)ldquoনাহিক এই (রীচেরর পাচেপর ীানচেত দেকানই তরারণ দেনই ভহি4চেতরldquoউততর) হমায়চেনর অসসথতার ন য ার হিনচেচেকই ায়ী কচেরচেছন কারন ার অচেনক রা য অন যায় ভাচে কচেরচেছ তাই তার এই পাপ কাচের ন য তার ঘচের আ হিপ এচেসচেছ এই অন যায় কাচের ন য তার মহিকত দেনই তাই ার আললার কাচেছ এই পাপ কাচেযCর ন য bমা পরাথM

Hindi 2ndlang

-ासी(जयशकर परसा-)

-ासी जयशकर परसा- की एक ऐसी कहानी ह जिजसम भारतीय ससकनित और राषटरीयता का सवरगजीतहोता ह इस कहानी म इरावती एक निहद कनया ह जिजस मलअचछो न मलतान की लट म पकडा और -ासी बना दि-या उस 500 दि-न -कर काशी क एक महाजन न खरी-ा दसरी -ासी निफरोजा ह वह गलाम ह निफरोजा को छडान क कतिलए अहम- को 1000 सोन क कतिसकक भजन थ जो अभी तक नही आए थ राजा साहब कठोर होत हए भी निफरोजा को निबना धनराकतिश क कतिलए उस म कर -त ह वनिफरोजा को अहम- को समझान की बात कहत हकहानी क अत म हम -खत ह निक इरा वती और जाटो क सर-ार बलराज का मिमलन होता हअहम- को यa म मार दि-या जाता ह वहा निफरोजा की परसननता की समामिध बनती ह वहा एक फल चढती ह और डीजल आती ह निफरोजा उस समामिध की आजीवन -ासी बनी रहती हलखक अपन उददशय अथात -ास परथा पर परकाश डालन और इस परथा क कारण होन वाल -ातो क दखो को दि-खान म पणता सफल हए ह

helliphellipContinue to next

Biology Reproductio Today we will discuss about vegetative Q1 Name some vegetative propagules

n in Organisms

propagation of plants The process of multiplication in which fragments of plant body function as propagule and develop into new individual is called vegetative propagation The units of such propagation are runner rhizome tuber bulb etc

and the speciesinvolvedVegetative propagules

Parts involved

Bulb StemBulbil BulbilRhizome Stem Runner Stem Tuber Stem Offset Stem Leaf buds Leaves Suckers Stem

Corns Stem stolon

Q2 State advantages of vegetative propagation

i) Rapid methodii) Sure and easy methodiii) Useful in plants that cannot

produce viable seeds or long seed dormancy

iv) Maintains purity of raceQ 3 Banana fruit is said to be parthenocarpic where as turkey is said to be parthenogenetic WhyBanana develops without fertilization from an unfertilized ovary thus is parthenocarpicIn turkey the ovum or female gamete developinto a new chick without fertilization thus isparthgenetic

Q4 Why is water hyacinth is called as a ldquoTerror of Bengalrdquo Water hyacinth can

propagatevegetatively all over the water body in a short per short period of time This resulted increased biochemicaloxygen oxygen demand of water body causing mortalityof fishes It is very difficult to get rid off them Thus known as terror of Bengal

Chemistry

Solid state GENERAL CHARACTERISTICS OF SOLID STATEIn nature the particular state of matter is governed by two opposing forces at given set of temperature and pressure These forces are intermolecular force of attraction and thermal energy If intermolecular force of attraction is high as compared to thermal energy particles remains in closest position

Intext QuestionsQ1 Classify the following solids as crystalline and amorphous Sodium chloride quartz glass quartz rubber polyvinyl chloride Teflon

A1 Crystalline

and hence very less movement in particles is observed In this case solid state is the preferred state of matter

Let us revise the general characteristics of solid

i) Fixed mass volume and shape

ii) Strong intermolecular force of attraction

iii) Least intermolecular space

iv) Fixed position of constituent particles

v) Incompressible and rigid

Q2 what type of interactions hold the molecules together in a polar molecular solid[CBSE 2010]A2 The molecules in a solid are held together by van der Waals forces The term van der Waals forces include hydrogen bonding dipole-dipole attraction and London dispersion forces All molecules experience London dispersion forces In addition polar molecules can also experience dipole-dipole interactions So the interactions that holds the molecule together in polar molecular solid are London dispersion force and dipole-dipole interactionsQ3 Write a feature that will distinguish a metallic solid from an ionic solid [CBSE 2010]A3 Metals are malleable and ductile whereas ionic solid are hard and brittle Metallic solid has typical metallic lustre But ionic solid looks dullQ4 Write a point of distinction between a metallic solid and an ionic solid other than metallic lustre [CBSE 2012]A4 Metals are malleable and ductile whereas ionic solid are hard and brittleQ5 Write a distinguish feature of metallic solid [CBSE 2010]A5 The force of attraction in

solid Sodium chloride Quartz Amorphous solid Quartz glass rubber polyvinyl chloride Teflon Q2 why glass is considered as super cooled liquidA2 Glass shows the tendency to flow at slower rate like liquid Hence they considered as super cooled liquidQ3 why the window glass of old buildings show milky appearance with timeA3 Glass is an amorphous solid Amorphous solid has the tendency to develop some crystalline character on heating Due to heating in day over the number of years glass acquires some crystalline character and show milky appearanceQ4 why the glass panes fixed to window or doors of old building become slightly thicker at bottomA4 Glass is super cooled liquid It has the tendency to flow down very slowly Due to this glass pane becomes thicker at the bottom over the timeQ5 Sodium chloride is a crystalline solid It shows the same value of refractive index along all the direction TrueFalse Give reasonA5 FalseCrystalline solid shows anisotropy in properties That is it shows different values for the given physical property in different direction All the crystalline solids show anisotropy in refractive index Therefore sodium chloride will show different values of refractive index on different directions

Q6 Crystalline solid are anisotropic in nature What does this statement means

between the constituent particles is special kind of electrostatic attraction That is the attraction of positively charged kernel with sea of delocalized electronsQ6 which group of solid is electrical conductor as well as malleable and ductile [CBSE 2013]A6 Metallic solidQ7 why graphite is good conductor of electricity although it is a network (covalent solid)A7 The exceptional property of graphite is due to its typical structure In graphite each carbon is covalently bonded with 3 atoms in same layer The fourth valence electron of each atom is free to move in between different layersThis free electron makes the graphite a good conductor of electricity

[CBSE 2011]A6 Anisotropy is defined asrdquo Difference in properties when measured along different axis or from different directionsrdquo Crystalline solid show different values of some of the physical properties like electrical resistance refractive index etcwhen measured along the different directions The anisotropy in crystalline solid arises due to the different arrangement of particles in different directions

Math Function Composition of functions Think of an industrial plant that produce bottles of cold drinks first there is the operation (or function) f that puts the cold drink inside the bottle followed by the opeartion g that close the bottle with the capThis leads to the following definitionDefinition Let f A rarr B and g B rarr C be two functions Then the composition of f and g denoted by gof is defined as the function gof A rarr C given by gof(x) = g(f (x)) forall x isinA

Definition A function f X rarr Y is defined to be invertible if there exists a function g Y rarr X such that gof = IX and fog = IY The function g is called the inverse of f and is denoted by f -1

Thus if f is invertible then f must be one-one and onto and conversely if f is one-one and onto then f must be invertible This fact significantly helps for proving a function f to be invertible by showing that f is one-one and onto specially when the actual inverse of f is not to be determined

Example 1 Let f 2 3 4 5 rarr 3 4 5 9 and g 3 4 5 9 rarr 7 11 15 be functions defined as f(2) = 3 f(3) = 4 f(4) = f(5) = 5 and g (3) = g (4) = 7 and g (5) = g (9) = 11 Find gofSolution We have gof(2) = g (f(2)) = g (3) = 7 gof(3) = g (f(3)) = g (4) = 7gof(4) = g (f(4)) = g (5) = 11 and gof(5) = g (5) = 11Example 2 Find gof and fog if f R rarr R and g R rarr R are given by f(x) = cos x and g (x) = 3x2 Show that gof ne fogSolution We have gof(x) = g(f(x))=g(cosx) = 3 (cos x)2

= 3 cos2 x Similarly fog(x)=f(g (x))= f(3x2)= cos (3x2) Note that 3cos2 x ne cos 3x2 for x = 0 Hence gof ne fogExample 3 Show that if f A rarr B and g B rarr C are onto then gof A rarr C is also ontoSolution Given an arbitrary element z isin C there exists a pre-image y of z under g such that g (y) = z since g is onto Further for y isin B there exists an element x in A with f(x) = y since f is onto Therefore gof(x) = g (f(x)) = g (y) = z showing that gof is onto Example 4 Let Y = n2 n isin N sub N Consider f N rarr Y as f(n) = n2 Show that

f is invertible Find the inverse of fSolution An arbitrary element y in Y is of the form n2 for some n isin N This implies that n =radicy This gives a function g Y rarr N defined by g (y) =radicy Nowgof (n) = g (n2)=radicn2 = n and fog (y) =f(radicy) = (radicy) 2 y which shows that gof=IN and fog= IY Hence f is invertible with f -1 = g

Political Science

Constitution of India-The Preamble

Summary

Objective of the state-To secure equality of status and of opportunity To promote fraternity among all the citizens To assure the dignity of the individuals and Unity and integrity of the nation

Justice-Justice stands for rule of law absence of arbitrariness and a system of equal rights freedom and opportunities for all in a society India seeks social economic and political justice to ensure equality to its citizens

Liberty-Liberty implies the absence of restraints or domination on the activities of an individual such as freedom from slavery serfdom imprisonment despotism etc The Preamble provides for the liberty of thought expression belief faith and worship

Equality-Equality means the absence of privileges or discrimination against any section of the society The Preamble provides for equality of status and opportunity to all the people of the country

Fraternity-The Preamble declares that fraternity has to assure two thingsmdashthe dignity of the individual and the unity and

Execution

Answer the following questions-

Short notes-1 Equality2 Fraternity3 Justice4 Liberty

Homework-Learn

integrity of the nation The word integrity has been added to the Preamble by the 42nd Constitutional Amendment (1976)

Business studies

Human resource management (chapter 1)

On the day of 1504 2020 I have discussed with you the managerial functions and procurement functions of HRM

Today weare going to discuss about the development function integration functions and maintenance function

Development functions-HRM improves the knowledge skills attitude and values of employees so that they the present and future jobs more effectively it includes

1) Development functions of HRM

a) Performance appraisal = It implies systematic evaluation of employees with respect to their performance on the job and their potential for development

b) Training =It is the process by which employees learn knowledge skills and attitudes to achieve organisational and personal goals

c) Executive development = It is the process of developing managerial talent through appropriate program

2) Integration functionsa) HRM reconcile the goals of

organisation with those of its members through integrating function

b) HRM tries to motivate employees to various financial and non financial incentives provided in job specification etc

3) Maintenance functiona) HRM promote and protect the

physical and mental health of employees by providing several types of benefits like housing medical aid etc

b) It Promote Social security measures to employees by providing provident fund pension gratuity maternity benefits

SubjectCOMMERCE

Topic

BUSINESSENVIRONMENT

Summary

Now quickly let us revise the earlier points that we have already done in the last class and let us proceed with the other topics that are there in the chapter

Firstly we will recall the internal and external factors of micro environment and then we

Execution 3 What do you mean by internal factors

in micro environmentAnswerInternal factors refer to all the factors existing within a business firm The internal factors are considered controllable because the enterprise has control over these factors

Development FunctionsPerformance AppraisalTrainingExecution Development

shall proceed in details

Meaning and list of internal and external factors

aInternal factorsInternal factors refer to all the factors existing within a business firm The internal factors are considered controllable because the enterprise has control over these factorsFor an example a company can alter its organization structure policies programmes employees physical facilities and marketing mix to suit the changes in the environmentList of internal factors areCorporate culture mission and objectives top management organizations structure company image and brand equity company resources

b External factorsExternal factors refer to those individual and groups and agencies with which a particular business organization comes into direct and frequent contact in the course of its functioningThese individuals and groups are known as STAKEHOLDERS because they have a stake (financial interest ) in the working and performance of the particular business List of external forces (stakeholders)Customers competitors investors suppliersmiddlemen (marketing intermediaries)financers publics

customers

suppliersfinancers

For an example a company can alter its organization structure policies programmes employees physical facilities and marketing mix to suit the changes in the environment

4 What do you mean by external factors in micro environment

AnswerExternal factors refer to those individual and groups and agencies with which a particular business organization comes into direct and frequent contact in the course of its functioningThese individuals and groups are known as STAKEHOLDERS because they have a stake (financial interest) in the working and performance of the particular business

3Who are stakeholdersSTAKEHOLDERS are individuals and groups who have a stake (financial interest ) in the working and performance of the particular business 4Discuss the internal factors in briefa Corporate CultureThe values beliefs and attitudes of the founders and top management of the company exercise a strong influence on what the cmpaany stands for how it does things and what it considers importantbMission and objectivesThe business philosophy and purpose of a comoany guide it prioritiesbusiness strategiesproduct market scope and development scope

cTop management structurethe composition of board of directors the degree of professionalization of management and the organizational structure of a company have important bearing on its business decisions

dPower structureThe internal power relationship between the board of directors and the chief executive is an important factor

e Company image and brand equityThe image and brand equity of the company play a significant role in raising finance forming alliance choosing dealers and suppliers launching new products entering foreign markets

5 What is Macro environmentAnswerMacro environment refers to the general

competitors

middlemen

publics

Fig STAKEHOLDERS OF A COMPANY

Apart from micro environment the other main dimension of business environment isMacro environment Macro environment refers to the general environment or remote environment within which a business firm and forces in its micro environment operateA company does not directly or regularly interact with the micro environmentTherefore macro environment is also known as indirect action EnvironmentThe macro environment forces are less controllable than the micro forces

Macro environment consists of the following components

POLITICAL AND LEGAL ENVIRONMENT

ECONOMIC SOCIAL AND ENVIRONMENT

CULTURAL

ENVIRONMENT

TECHNOLOGICAL ENVIRONMENT

Fig COMPONENTS OF MACRO ENVIRONMENT

environment or remote environment within which a business firm and forces in its micro environment operateA company does not directly or regularly interact with the micro environmentTherefore macro environment is also known as indirect action EnvironmentThe macro environment forces are less controllable than the micro forces 6 What are the components of macro environmenta Political and legal environmentb Economic environmentc Social and cultural environmentd Technological environment

Computer Science

Logic gates

Digital systems are said to be constructed by using logic gates These gates are the AND OR NOT NAND NOR EXOR and EXNOR

BUSINESS FIRM

gates The basic operations are described below with the aid of truth tables

AND gate

The AND gate is an electronic circuit that gives a high output (1) only if all its inputs are high A dot () is used to show the AND operation ie AB Bear in mind that this dot is sometimes omitted ie ABOR gate

The OR gate is an electronic circuit that gives a high output (1) if one or more of its inputs are high A plus (+) is used to show the OR operationNOT gate

The NOT gate is an electronic circuit that produces an inverted version of the input at its output It is also known as an inverter If the input variable is A the inverted output is known as NOT A This is also shown as A or A with a bar over the top as shown at the outputs The diagrams below show two ways that the NAND logic gate can be configured to produce a NOT gate It can also be done using NOR logic gates in the same way

NAND gate

This is a NOT-AND gate which is equal to an AND gate followed by a NOT gate The outputs of all NAND gates are high if any of the inputs are low The symbol is an AND gate with a small circle on the output The small circle represents inversion

NOR gate

This is a NOT-OR gate which is equal to an OR gate followed by a NOT gate The outputs of all NOR gates are low if any of the inputs are highThe symbol is an OR gate with a small circle on the output The small circle represents inversion

EXOR gate

The Exclusive-OR gate is a circuit which will give a high output if either but not both of its two inputs are high An encircled plus sign ( ) is used to show the EOR operation

EXNOR gate

The Exclusive-NOR gate circuit does the opposite to the EOR gate It will give a low output if either but not both of its two inputs are high The symbol is an EXOR gate with a small circle on the output The small circle represents inversion The NAND and NOR gates are called universal functions since with either one the AND and OR functions and NOT can be generated

Note A function in sum of products form can be implemented using NAND gates by replacing all AND and OR gates by NAND gates A function in product of sums form can be implemented using NOR gates by replacing all AND and OR gates by NOR gates

Logic gate symbols

Table 2 is a summary truth table of the inputoutput combinations for the NOT gate together with all possible inputoutput combinations for the other gate functions Also note that a truth table with n inputs has 2n rows You can compare the outputs of different gates

Logic gates representation using the Truth table

Example

A NAND gate can be used as a NOT gate using either of the following wiring configurations

Subject Eng Literature (The Tempest ndash William Shakespeare) Topic Act III Scene 3 Lines 53 to 110 (End of the scene) Date 16th April 2020 (2nd Period)

[Students should read the original play and also the paraphrase given in the school prescribed textbook]Summary Questions amp Answers

o Seeing this strange scene all are inclined to believe the tales told by travelers that there truly are ldquounicornsrdquo and ldquothe phoenixrsquo thronerdquo

o As they are about to sit down to the feast the banquet is snatched away by a harpy (Ariel disguised) A spiritrsquos voice (Arielrsquos voice) denounces Alonso Sebastian and Antonio with particular

1 ARIEL You are three men of sin whom Destiny

(Line 53-58)That hath to instrument this

lower world And what is int the never-surfeited sea

Hath caused to belch up you and on this island

Where man doth not inhabit you rsquomongst men

Being most unfit to live I have made you mad

reference to their crime in expelling Prospero from Milan They have not received any punishment for their deed earlier but the time for their punishment has arrived Upon Alonso it pronounces ldquolingering perdition worse than deathrdquo from which there is no remedy except through sincere repentance Ariel then vanishes in thunder and the shapes enter again and carry away the table

o Prospero watching invisibly is very pleased with the performance of Ariel and his (Prosperorsquos) ldquomeaner ministersrdquo All his enemies are now in his power and are in a fit of desperation He then leaves them and goes to see how Ferdinand and Miranda are getting on

o Alonso is now much humbled and penitent with the after effect of the spiritrsquos denunciation of his crimes He believes that his son is lost forever After this all disperse being stricken mad by the speech of the spirit

o Gonzalo fearing that they may do violence to themselves or to one another follows them and bid others to follow

(a) To whom does Ariel disguised as a harpy call the three sinners What game did Fate of Destiny play with

them

The three sinners called by Ariel are Alonso Sebastian and Antonio It was Destiny which had caused the ocean to cast the three sinners on the shore Though the ocean is all the time devouring whatever appears on its surface and is never satisfied with its continual swallowing of the ships and men in the present case the ocean had cast these three sinners on the shore without killing them

(b) Who had jointly been responsible for the conspiracy against Prospero What is Prosperorsquos purpose behind all this

Three men Alonso Sebastian and Antonio had jointly

been responsible for the conspiracy against Prospero They had driven out Prospero form Milan Prosperorsquos purpose is to make these three sinners realize the wrong they had done He wants them to repent for their criminal deeds because repentance leads to self-esteem(c )What does Ariel (the harpy) tell Alonso and his companions when they take out their swords to attack him

Seeing them drawing their swords Ariel (harpy) tells them that he and his companions are the instruments of destiny and that it is not possible for human beings to do them any injury He says that the swords of human beings can not injure even a minute part of his feathers Their swords are as ineffective against him and his companions as against the wind or the water

(d) Give the explanatory meanings of the following expressions in the context of the above extract

(i)Never surfeited (ii) Belch up (iii) lsquomongst men

(i) Never surfeited never led to satisfaction

(ii) Belch up cast ashore(iii) lsquomongst men in human

society2

I and my fellows (Line 60-65)

Are ministers of Fate The elementsOf whom your swords are tempered may as wellWound the loud winds or with bemocked-at stabsKill the still-closing waters as diminishOne dowl thats in my plume

IMPORTANT PASSAGES EXPLAINED

The elements

(Line 61-66)Of whom your swords are tempered may

as wellWound the loud winds or with

bemocked-at stabs

(a) Who is lsquoIrsquo Who are his lsquofellowsrdquo

lsquoIrsquo is referred to Ariel in disguise of a harpy His lsquofellowsrsquo are other spirits serving Prospero the real Duke of Milan who has acquired supernatural powers after being banished from his Dukedom Prospero has settled in this uninhabited island

(b) What are the elements that have temperrsquod the swords Why will it not work against the speaker

The swords (of Alonso and his companions) are tempered by metal (steel) which is taken out of the earth and refined by

Kill the still-closing waters as diminishOne dowl thats in my plume My fellow

ministersAre like invulnerable

In these words Ariel reminds the King and his companions of the utter futility of drawing swords against himself and his fellows Ariel drives Alonso Antonio and Sebastian the three men of sin to desperation ndash a state in which men do violence to themselves They draw swords to strike Ariel But Ariel reminds them that he and the other spirits are the ministers of destiny and nothing can wound them The steel of which their swords are made of may cut the wind or water which being divided always closes up again Even supposing that such things may be possible it is quite impossible that their swords will cut one feather in their plume They are incapable of being wounded by any sword of man Hence it is foolish on their part to attempt to strike at Ariel and his fellow-spirits

For which foul deed

(Line 72-75)The powers delaying not forgetting

haveIncensed the seas and shores yea all the

creatures Against your peace

Ariel enters like a harpy and remaining invisible tells Alonso Sebastian and Antonio that he and other harpies are the agents of Destiny appointed to carry out her decrees He tells them that their punishment for the crime against Prospero which has been so long deferred is now to fall upon them He reminds them that they had expelled Prospero from Milan and set him and his innocent child adrift on the sea and that the sea had paid them back for their sin by the shipwreck and by the calamities they have suffered He tells them that the powers above which did not forget this mean treachery but only deferred the punishment have now engaged the seas and the shores and all living beings including him and his comrades against them The very elements and supernatural agency Ariel adds have taken up the avenging of their crime against Prospero

the action of fire It may cut the wind or water which being divided always closes up again

The sword will not work against the spirits and the harpy because they are the ministers of destiny and nothing can wound them nor it will cut a single feather in their plume

(c )What is the meaning of lsquodowlrsquo in the last line

The term lsquodowlrsquo means a filament or the smallest part of a feather In this context Ariel in disguise of harpy says that their sword cannot even damage the smallest filament of their (Arielrsquos and other spirits) feathers as they are incapable of being wounded by any sword of man

(d) What does the speaker remind the listeners about

Ariel in disguise of harpy reminds Alonso the King of Naples Sebastian Alonsorsquos brother and Antonio the present Duke of Milan and the treacherous brother of Prospero as they being three men of sin He even reminds them that their punishment for their crime against Prospero which has been so long deferred now falls upon them He reminds them that they have expelled Prospero from Milan and has set him along with his innocent infant daughter adrift on the sea So the sea has paid them back for their sin by their shipwreck and the calamities they have suffered since then The harpy rebukes Alonso of his sin that has incensed the Gods and has deprived him of his son as a punishment

(e) How do they respond

When Ariel in disguise of a harpy reminds Alonso Sebastian and Antonio of their past misdeeds and sin Alonso has a look of terror and confusion in his eyes He utters the words of sincere repentance wrung out of his conscience-stricken heart It appears to him that all the elements of nature the sea-waves the wind and the thunder proclaiming a loud voice in the name of Prospero and the crime Alonso has committed against him They are calling upon him to repent There is a deep storm raging in Alonsorsquos breast and the echoes of that storm are ringing in his ears like a clear note of wind-instrument A note of denunciation of Alonsorsquos crime leaves him much humbled and penitent and confirms his belief that his son is lost forever But Sebastian and Antonio shows some courage instead of repentance They wish to kill the spirits or devils if it appears

3

Of my instruction hast thou nothing bated (Line 85-93)

In what thou hast to say So with good life

And observation strange my meaner ministers

Their several kinds have done My high charms work

And these mine enemies are all knit upIn their distractions They now are in my

powerAnd in these fits I leave them while I visitYoung Ferdinand whom they suppose is

drownedAnd his and mine loved darling

Methought the billows spoke and (Line 96-99)

told me of itThe winds did sing it to me and the

thunderThat deep and dreadful organ-pipe

pronouncedThe name of Prosper It did bass my

trespass

These are the words of contrition coming from Alonso Ariel has driven him to a deep repentance for conspiring with Antonio against Prospero He now feels a sincere remorse It appears to him that all the elements of nature the sea-waves the wind and the thunder proclaimed with a loud voice the name of Prospero and the crime Alonso had committed against him They are calling upon him to repent There is a deep storm raging in Alonsorsquos breast and the echoes of that storm are ringing in his ears like the clear note of a wind-instrument

Comment These are the words of sincere repentance wrung out of the conscience-stricken heart of Alonso Alonso who is the lesser villain is the first to give way to remorse under the effect of Arielrsquos speech The words of Ariel seem to him to be the voice of conscience speaking to him He is driven to desperation a state in which he might do violence to his life

(a) Identify the speaker State the context

Prospero the ruler of the island is the speaker The famous banquet scene has been enacted very well Ariel and his junior spirits have played their roles excellently Prospero is glad to say words of praise for them(b) In what way the speakerrsquos instructions have been carried out

According to Prosperorsquos instructions a banquet was presented before the King of Naples and his companions when they were tired and hungry Just when they were preparing to eat the feast the banquet was suddenly removed by exercising supernatural powers All this was done by Ariel Prosperorsquos chief assistant and a powerful spirit

Ariel not only made the feast disappear but also delivered his speech blaming the King and his two companions for their past wicked deeds He warned them to repent for their misdeeds or suffer forever on that uninhabited island

(c) Who are referred to as lsquomeaner ministersrsquo What have they done

Prospero refers as lsquomeaner ministersrsquo to his other lesser spirits who were assisting Ariel in presenting a scene before the kingrsquos party They entered the scene to the accompaniment of music They assumed several strange shapes and brought in a banquet Then they danced about it with gentle actions of salutations thus inviting the King and others to eat the feast

These spirits play their role again when Ariel in the shape of a harpy quits the scene These shapes enter again and dancing with mocking gestures carry away the table

(d) Who are the speakerrsquos enemies What has happened to them

King of Naples Alonso his brother Sebastian and the present Duke of Milan Antonio (Prosperorsquos own brother) are Prosperorsquos enemies With the turn of events they have all been washed ashore on the island which is ruled by Prospero the great magician Actually this happened after the shipwreck caused by a storm which was raised by Prospero with the purpose of bringing these people to his island Prosperorsquos spirits have already confused and terrified these enemies and they are under Prosperorsquos control He can treat them as he likes

(e) What does he say about Ferdinand Explain what is meant by ldquohellip his and mine darlingrdquo

Prospero knows that Alonsorsquos son prince Ferdinand is alive though his father thinks that the prince has been drowned

Prospero refers to his daughter Miranda who is dear to him She is also very dear to Prince Ferdinand who has fallen in love with her They are waiting to be married soon for which they have received Prosperorsquos consent

4

ALONSO O it is monstrous monstrous (Line 95-102)

Methought the billows spoke and told me of it

The winds did sing it to me and the thunderThat deep and dreadful organ-

pipe pronouncedThe name of Prosper It did bass

my trespassTherefore my son ithrsquo ooze is

bedded andIll seek him deeper than eer

plummet soundedAnd with him there lie mudded

(a) In what way does Alonso express his horror when his conscience is awakened by Arielrsquos words

When Alonsorsquos conscience is awakened by Arielrsquos words he expresses his horror at what he has heard He gets the feeling that the waves of the ocean the wind and the loud thunder have spoken to him and uttered the name of Prospero Because of being reminded of his crime in a very loud and rough voice he comes to realize that he has lost his son for his past misdeeds

(b) What does Alonso imagine about his son What does Alonso want to do in his desperate state

Alonso imagines that his son is lying in the mud at the bottom of the sea He feels desperate that he wants to drown himself in the ocean deeper than the plumb-line has ever gone He wants to lie with his son at the bottom of the sea

(c) How do Sebastian and Antonio want to face the evil spirits

Sebastian says that he is not at all afraid of what the harpy has said and that he is prepared to fight any number of such monsters if they appear before him only one at a time Antonio says that he would support Sebastian in the fight against the fiendsyyy

(d) Why does Gonzalo ask Adrian to follow the three men

Gonzalo tells Adrian that all the three men namely Alonso Sebastian and Antonio are in a wild and reckless mood The thought of the heinous crime of which they are guilty has begun to torment their minds So he asks Adrian to follow those three men without loss of time and prevent them from doing anything which the turmoil in their minds might lead them to do

(e) What opinion do you form of Alonso from the above extract

Alonso who is the lesser villain is the first to give way to remorse under the effect of Arielrsquos speech The words of Ariel seem to him to be the voice of conscience speaking to him He is driven to desperation a state in which he might do violence to his life

Subject =Accounts

Ac-12 15420 topic-pL Appropriation ac

PROFIT AND LOSS APPROPRIATION ACCOUNT

MEANING AND PREPARATIONProfit and Loss Appropriation Account is merely an extension of the Profit and Loss Account of the firm The profit of the firm has to be distributed amongst the partners in their respective profit sharing ratio But before its distribution it needs to be adjusted All Adjustments like partnerrsquos salary partnerrsquos commission interest on capital interest on drawings etc are made in this account These adjustments will reduce the amount of profit for distribution This adjusted profit will be distributed amongst the partners in their profit sharing ratio To prepare it at first the balance of Profit and Loss Account is transferred to this account The journal entries for the preparation of Profit and Loss Appropriation Account are given below

1 for transfer of the balance of Profit and Loss Account to Profit and Loss Appropriation Account

(a) In case of Net Profit

Profit and Loss Ac helliphelliphelliphelliphellipDrTo Profit and Loss Appropriation Ac(Net Profit transferred to Profit and Loss Appropriation Ac)

(b)In case of Net Loss

Profit and Loss Appropriation Achelliphelliphellip DrTo Profit and Loss Ac(Net Loss transferred to Profit and Loss Appropriation Ac)

2 for Interest on Capital

For transferring on Interest on CapitalProfit and Loss Appropriation Achelliphelliphellip DrTo Interest on Capital Ac(Interest on capital transferred to Profit amp Loss Appropriation Ac)

3 for Interest on Drawings

For transferring Interest on Drawings Interest on Drawings Achelliphelliphelliphelliphelliphellip DrTo Profit and Loss Appropriation Ac(Interest on drawing transferred to Profit amp Loss Appropriation Ac)

4 For Partnerrsquos SalaryFor transfer of partnerrsquos SalaryProfit and Loss Appropriation Achelliphellip DrTo Salary Ac(Salary transferred to profit amp Loss Appropriation Ac)

5 For Partnerrsquos CommissionFor transferring commissionProfit and Loss Appropriation Achelliphelliphellip DrTo Commission Ac(Commission transferred to Profit and Loss Appropriation Ac)

6 For Transfer of agreed amount to General ReserveProfit and Loss Appropriation Ac helliphellipDrTo General Reserve Ac(Transfer to General Reserve)

7 for share of Profit or Loss appropriation(a) If ProfitProfit and Loss Appropriation Achelliphellip DrTo Partnerrsquos CapitalCurrent Ac(Profit transferred to capitalcurrent Ac)(b) If LossPartnerrsquos Capital Current Achelliphelliphelliphellip DrTo Profit and Loss Appropriation Ac(Loss transferred to capitalcurrent Ac)

THE FORMAT OF PROFIT AND LOSS APPROPRIATION

Profit and Loss Appropriation Account for the year endedhelliphelliphelliphellip

Particulars Amount Particulars Amount

To PL Ac (loss) By pL Ac (profit)

To Interest on capital BY Interest on drawings

To partner`s commission by Partner`s capital Ac ( loss)

To Partner`s salary To Interest on partner`s loan To General Reserve To Partner`s Capital AC (Profit)

Subject= Economics

MOVEMENT ALONG THE DEMAND CURVE (CHANGE IN QUANTITY DEMANDED)In law of demand you have already studied the inverse relationship between price and quantity demanded When quantity demanded of a commodity changes due to change in its price keeping other factors constant it is called change in quantity demanded It is graphically expressed as a movement along the same demand curve There can be either a downward movement or an upward movement along the same demand curve Upward movement along the same demand curve is called contraction of demand or decrease in quantity demanded and downward movement along the same demand curve is known as expansion of demand or increase in quantity demanded

Extention of demandd

price (rs)p A

B Extentionp1 d

Q Q1

Quantity demanded ( in units)

Contraction of demandd

p2 Ccontraction

p APrice (Rs)

d

Q2 Q

Quantity demanded (in units)

Explanation of movement of demand A fall in price from OP to OP1 leads to increase in quantity demanded from OQ to OQ1 (expansion of demand) resulting in a downward movement from point A to point B along the same demand curve DD When Price rises from OP to OP2 quantity demanded falls from OQ to OQ2 (contraction of demand) leading to an upward movement from point A to point C along the same demand curve DD

  • Activity Series of Metals
    • Drawbacks of Rutherfordrsquos model of atom
      • Electromagnetic radiations
      • Properties of electromagnetic radiations
      • Characteristics of electromagnetic radiations
        • Plancks Quantum Theory-
        • Photoelectric effect
          • Intext Questions
            • Logic gates
            • Digital systems are said to be constructed by using logic gates These gates are the AND OR NOT NAND NOR EXOR and EXNOR gates The basic operations are described below with the aid of truth tables
            • AND gate
            • Example
Page 21:  · Web viewSubject. Topic. Summary. Execution. English 1 . Chapter 1 naming words . Page 8. Write the names of these pictures:- Person:-1. father. 2.Firefighter 3.doctor 4 ...

of Shylock

The above mentioned speech of Shylock reveals him to be a wicked character having an extreme greed for wealth His intense hatred for Antonio is unjustified He hates Antonio only because he is a Christian and because he lends money without taking any interest on it thereby adversely affecting Shylockrsquos business of lending money on high interest(d) What information do you gather about Antonio from the above given lines

Shylockrsquos statement throws a valuable light on the character of Antonio Antonio appears to be a good Christian and a good human being He helps the people in need by lending them money without charging any interest on it He is a man of simple and good nature This very goodness makes him Shylockrsquos enemy(e) What does Shylock debate within himself and when To whom are the lines mentioned above addressed to

When Bassanio asks the Jew to lend him three thousand ducats on Antoniorsquos surety Shylock begins to debate within himself as to how he should exploit the opportunity of a business deal with his old enemy Antonio

The lines mentioned above are not addressed to anyone The lines are a soliloquy ie a speech made by a character to himself and not meant to be heard by the other characters present

Class XSubject Topic Summary ExecutionEnglish

LiteratureThe Blue Bead 2nd part

Things took a turn for the worst and all of a sudden a crocodile attacked the woman biting on the womanrsquos leg At that moment Sibia got up sprinted grabbed the hay fork and stabbed the crocodile in the eye with all her power Immediately the crocodile let go and went away Sibia saw a small blue bead lying by the river she grabbed it Since she was poor she didnrsquot have necklace Shersquod always wanted one like the other women now she could make one with the blue bead After that she went home and told her mother all about it

Hindi 2nd

Langबड घर की बटी( मशी परमच-)

lsquoबड घर की बटी कहानी का उददशय मधयम वग की घरल समसया को सलझा कर सगदिठत परिरवार म मिमल जलकर परम स रहन का स-श -ना हघर म शानित सथानिपत करन की जिजमम-ारी नारी की होती ह यदि- नारी समझ-ार ह उसम धय और परिरवार क परनित परम ह तो कोई भी घटना परिरवार को निवघदिटत नही कर सकती या कहानी परिरवार को सगदिठत करत हए परम सौहा- स एक दसर की भावनाओ को समझ करउनका सहयोग करत हए जीवन यापन करन की पररणा -ती हमशी परमचदर जी न इस कहानी म सय परिरवार का परनितनिनमिधतव निकया ह यह कहानी बनी माधव सिसह जो गौरी पर क जमी-ार क उनक -ो पतरो की हशरी कठ लाल निबहारीशरीकात का निववाह एकजमी-ार घरान की पतरी आन-ी स हआ थाआन-ी न ख- को ससराल क वातावरण म ढाकतिलया थाएक दि-न आन-ी का अपन -वर लाल निबहारी स झगडा हो जाता ह -ोनो भाई एक दसर स अलग होन की कोकतिशश करत हसभी बह आन-ी न अपन मधर वयवहार स लाल निबहारी को

ldquoइन नतर निपरय गणो को बीए-इनही -ो अकषर पर नयोछावर कर दि-या था इन -ो अकषर न उनक शरीर को निनबल और चहर को कानित ही बना दि-या थाldquo

क) परसतत पकतियो म निकस वयकति क बार म कहा गया ह

ख) इन पकतियो म कौन स नतर निपरय गणो क बार म कहा गया ह

ग) बीए की निडगरी परापत कर लन पर भी उपय वयकति क सवभाव की कया निवशरषता थी

घ) यह नतर निपरय गण निकस वयकति म निवदयमान थ उसक वयकतितव की कया निवशरषता थी

उततर ndashक) परसतत पकति म गौरी पर गाव क जमी-ार

क बड बट शरीकात क बार म कहा गया ह उसन बहत परिरशरम और उ-म क बा- ba की निडगरी परापत की थी अब वह एक -फतर

घर छोडकर जान स रोक कतिलयाइस पर बनी माधव सिसह न कहा निक बड घर की बटी ऐसी ही होती ह जो निबगडा काम बना लती ह अतः शीरषक साथक ह बड घर की बटी आन-ी ह

म कमचारी थाख) भरा हआ चहरा चौडी छाती और डटकर

खाना आदि- एक सबजी ल जवान क गण मान जात ह परत शरीकात न इनही नतर निपरय गणो को अपनी पढाई पर नयोछावर कर दि-या था

ग) बीए की निडगरी परापत कर लन पर भी उपय वयकति(शरी कठ की शारिररिरक तौर पर निनबल और चहर स कानित ही लगत थ इतना ही नही वह मानकतिसक तौर पर भी निपछड हए थ पाशचातय सामाजिजक कथा उस घणा एव पराचीन सभयता का गणगान उनकी निवचारधारा क परमख अग थ

घ) यह नतर निपरय गण गौरीपर गाव क जमी-ार क छोट बट लाल निबहारी सिसह म निवदयमान थ वह सजीलाजवान था और भस का दध शर दध वह सवर उठकर पी जाता था

ldquoयही कारण था निक गाव की लललन आए उनकी निन-क थी कोई कोई तो उह अपना शतर समझन म भी सकोच ना करती थी सवय उनकी पतनी को इस निवरषय म उनस निवरोध थाldquo

क) उपय पकति म इस वयकति क बार म कहा गया ह

ख) गाव की लललन आए उनकी निन-ा कयो निकया करती थी

ग) उनकी पतनी का कया नाम था उनह निकस निवरषय म अपन पनित क निवरa था और कयो

घ) इस कहानी का कया उददशय ह Continue to next helliphelliphellip

Bengali 2nd Language

ফ ফটক না ফটক( কহিতা )

পর) ldquo(ান াধাচেনা ফটপাচেথ পাথচের পাড হিচেয় এক কাঠচোটটা গাছ কহিকহি পাতায় পার ফাটিচেয় াসচেছldquoক) কার দো দেকান কহিতার অং( ) lsquo(ান াধাচেনা ফটপাচেথ পাথচের পাডহিচেয়lsquo চেত কী দোঝাচেনা চেয়চেছ গ) আচো য অংচে(lsquo এক কাঠচোটটা গাছ lsquoচেত কী দোঝাচেনা চেয়চেছ ঘ) ldquoকহিকহি পাতায় পার ফাটিচেয় াসচেছldquo ----- একথার পরকত অথC কী উততর ) ক) আচো য অং(টি পর যাত কহি সভা4 মচোপাধ যাচেয়র দো lsquoফ ফটক না ফটকrsquo কহিতার অং()কহি সভা4 মচোপাধ যায় হিছচেন দেপরচেমর কহি দেপরমচেক নানা ভহিঙগমায় হিতহিন ফটিচেয় তচেচেছন দেপরম মানচের স মচেতC র সঙগী কহিতার কহিতায় এক রb সb হচেয়র দেপরম াগরচেনর কথা চেচেছন (ান অথCাৎ দেযাচেন দেকান রস দেনই দেযাচেন দেকান মহিনতা দেনই অথ তার মধ দেযও দেপরম থাকচেত পাচের একথাই কহি তচে ধরচেত দেচেয়চেছন একটি মানচে4র মচেন দেযাচেন দেকামতার দেকান সথান দেনই পাথচেরর মচেতা হিনরসতার মচেনর মধ দেযও দেয দেপরম আসচেত পাচের দেস কথাই কহি চেচেছনগ)নারীচের যথC দেপরচেমর ছহি এই কহিতায় অকপচেট উচেঠ এচেসচেছ কহি এই কহিতায় কাটচোটটা গাছ কথাটি যার কচেরচেছন নারী দেয দেপরম দেথচেক হিতাহিত এং দেসই দেপরম সঠিক সমচেয় না পাওয়ার ন য দেপরম সমপচেকC হিচেr4 গৈতরী য় দেপরচেমর দেয গৈহি(ষট য মাধযC য সরসতা দেকামত এই সমসতর হিপরীত যথা রbতা শষকতা কচেঠার তা পরভহিত দোঝাচেত এক কাঠচোটটা গাছ কথাটি যার কচেরচেছনঘ) এাচেন এক নারীর যথC দেপরচেমর কথা হিনহিCপত ভাচে চেচেছন কহি অসমচেয় নারীর ীচেন দেপরম দেচেগচেছ এতহিন তার হয় রb কচেঠার হিছ দেপরচেমর অভাচে ঠাৎ দেসই শষক মরভহিমচেত সচের আভাস এচেসচেছ দেপরম দেযন 4Cার স(ীত তাই পরায় মত গাচেছ কহিকহি পাতা গহিচেয় উচেঠচেছ

Biology Chapter - 01Controlling Air Pollution

Today we will discuss how we control air pollution from domestic combustion

Q1Describe any five ways of reducing air pollution from domestic sources bull The number of pollutants in the air is verylarge and we always try to control them byfollowing ways

i) Solar cooker and solar heater It use no fuel reduce damage of environment by fuel use or reducing deforestation It maintains coolness of house It releases very less orno oil gas or grease

ii) Piped natural gas (PNG) It emits very less by products into the atmosphere As it isdistributed through pipe lines so there iscontinuous supply of fuel is possible

iii) Liquefied Petroleum Gas (LPG) It hasa higher heating value LPG doesntcontain sulphur so it burns a lot cleanerenergy sources It releases very less oralmost no fume in air

iv) Electricity based cooking Emission free cooking alternative for urban dwellers causeselimination of adverse health impactsofindoor air pollution It helps to avoid theinconveniences associated with procurement of LPG

v) Biogas It contains 75 methane whichmakes it an excellent fuel It burns without smoke and biogas plant leaves no residue like ash in wood charcoal etc Thus it isaclean fuel

Economics

Factors of Production

Today firstly we would recall the last class for 5 mins and then we would proceed with the further topics of the chapter

The concept meaning of land characteristics of land and importance of land to be repeated for the absentees as well as the students who were there in the class the previous day

Today we will start with the last portion of land before it the meaning of land to be repeated onceAs by now we all know that

Questions1What do you mean by productivity of landAnswer By productivity of land we mean the capacity of a piece of land to produce a crop

Thus it refers to the average output per unit of landSay per acre per hectare etc= (OutputArea of land)

2 What are the factors influencing the productivity of landAnswer

Natural factors Productivity of land is largely determined by the natural

Land is defined to include not only the surface of the earth but also all other free gifts of nature(for example mineral resources forest resources and indeed anything that helps us to carry out the production of goods and services but is provided by nature free of cost)

We will move on to the last portion of land by discussing Productivity of Land

By productivity of land we mean the capacity of a piece of land to produce a crop

Thus it refers to the average output per unit of land

Say per acre per hectare etc= (OutputArea of land)

With this we shall proceed further with the main factors that determine the productivity of land

Natural factors Human factors Improvements on land Location of land Organisation Ownership of land Availability of capital Proper use of land State help

Note economic development of a country depends upon the quality of its land If the land is fertile it will quicken the pace of development of the country

qualities of land such as fertility etc

Human factors Land cannot produce anything by itself Man has to apply labour on it to produce for himself So productivity of land depends on the knowledge and skills of workers

Improvements on land production of land is affected by land development measures like provision of well or tubewell irrigation proper drainage

State help The government of a country especially less developed country can play a vital role in improving the agricultural productivity by providing better irrigation facilities

Organisation Productivity of land also fdepends upon the way how the factors of production like labour and capital are organised

In order to increase productivity trained workers modern implements scientific methods good seeds are all essential

3 lsquoImproved technology affects the productivity of landrsquo Explain this statement with the help of suitable example Answer Use of improved technology raises the productivity of land Example By using HYV seeds chemical manures and modern machines per hectare output increases

Physics Force (Summary)

Question Write the expression for the moment of force about a given axisSolutionsThe expression for the moment of force is given byMoment of force about a given axis = Force times perpendicular distance of force from the axis of rotationQuestion What do you understand by the clockwise and anticlockwise moment of force When is it taken positiveSolutionsIf the effect on the body is to turn it anticlockwise moment of force is called the anticlockwise moment and it is taken as positive while if the effect on the

body is to turn it clockwise moment of force is called the clockwise moment and it is taken as negative

Math Topic Commercial Mathematics

Chapter Goods and services Tax

Study item Some solved sums from exercise ndash 1 A retailer buys a TV from a wholesaler for Rs 40000 He marks the price of the TV 15 above his cost price sells it to the consumer at 5 discount on the marked price If the sales are intra ndash state and the rate of GST is 12 find

(i) The marked price of the TV(ii) The amount which the consumer pays for the TV(iii) The amount of tax (under GST) paid by the retailer to the central

Government(iv) The amount of tax (under GST) received by the State Government

Solution As the sales are intra- state sale and the rate of GST 12 So GST comprises of 6 CGST and 6 SGSTTherefore a retailer buys a TV from a wholesaler for Rs 40000Therefore the amount of GST collected wholesaler from the retailer or paid by retailer to wholesalerCGST = 6 of Rs 40000 = Rs(6100 times40000) =Rs 2400SGST = 6 of Rs 40000 = Rs (6100 times 40000) =Rs 2400Therefore wholesaler will pay Rs 2400 as CGST and Rs 2400 as SGSTTherefore amount of input GST of retailer Input CGST = Rs 2400 and input SGST = Rs 2400Again the retailer marks the price of the TV 15 above his cost price(i) The marked price of the TV

= Rs 40000 + Rs 40000times15= Rs 40000 + Rs 40000times 15100= Rs 40000 + Rs 6000Rs 46000But the retailer sells it to consumer at 5 discount on the marked priceCost price after discount = Rs 46000 ndashRs46000times 5100 =Rs 46000 ndashRs 2300= Rs 43700Therefore the amount of GST collected retailer from consumer or paid by consumer to retailerCGST = 6 of Rs 43700 =Rs ( 6100 times43700)Rs 2622SGST = 6 of Rs 43700 = Rs (6100 times 43700) =Rs 2622Amount of the output GST of retailer Output CGST = Rs 2622 and output SGST = Rs 2622

(ii) The amount which the consumer pays for the TV= cost price of TV to consumer + CGST paid by consumer + SGST paid by consumer= Rs 43700 + Rs 2622 + Rs 2622= Rs 48944

(iii) The amount of tax (under GST ) paid by the retailer to the central Government=CGST paid by retailer = output CGST ndash input CGST=Rs 2622 ndash Rs 2400=Rs 222

(iv) The amount of tax ( under GST ) received by the State Government = SGST paid by wholesaler + SGST paid by retailer= Rs 2400 + output SGST ndash input SGST=Rs 2400 + Rs 2622 ndash Rs 2400=Rs 2400 + Rs 222= Rs 2622

Commercial studies

Stakeholders Today I am going to give some revision questions from the previous study material

Questions1) State the two expectations of

employees from a business concern2) Give two distinctions between

stakeholder and shareholder3) Give two difference between

internal stakeholders and external stakeholders

4) Give two expectations of suppliers from a business organisation

5) Who is a stakeholder in commercial organisations

Chemistry Periodic Table

Merits of Mendeleevrsquos Periodic law are as follows - 1He grouped the elements on the basis of atomic mass 2 He left gaps for undiscovered elements like Gallium Scandium germanium Also he left a full group vacant for undiscovered inert gases 3 He could predict proportions of several elements on basis of their position in periodic table like Ga Sc etc 4He could predict errors in atomic weights of some elements like gold platinum etc

Anomalies in Mendeleevrsquos Periodic law are as follows - 1 Position of isotopes could not be explained 2 Wrong order of atomic masses could not be explained

For example- as Arnur atomic mass 40 come first and K with low atomic mass (30) should come later but k should be placed first

According to Bohrrsquos Modern Periodic table properties of elements are periodic functions of their atomic numbers

So when elements are arranged according to increasing atomic numbers there is periodicity in electronic configuration that leads to periodicity in their chemical properties

It consists of horizontal rows (Periods) Vertical column (Groups)

There are 7 period and 12 groups in this long form of periodic table

Ist period has 2 elements IInd period has 8 elements IIIrd period has 8 elements IVth period has 18 elements Vth period has 18 elements VIth period has 32 elements VIIth period hs rest of elements

Note - The number of valence electrons in atom of elements decides which elements will be first in period and which will be last

In group- 1 to 2 gp and 13 to 17 contain normal elements 3 to 12gp ndash transition elements 57 to 71 - lanthanides 89 to 103 - Actinides

Left hand side ndash metals Right hand side ndash nonmetals

Note- Hydrogen element has been placed at top of Ist group Electronic configuration of H is similar to alkali metal as both have 1 valence electron

V electron of gp I element -- 1 V electron of gp 2 element -- 2 V electron of gp 13 element -- 3 V electron of gp 14 element -- 4 V electron of gp 15 element -- 5 V electron of gp 16 element --6 V electron of gp 17 element -- 7 V electron of gp 18 element -- 8

English 1 Transformation of sentences

Sentences A sentence is a group of words which makes complete sense

Exercise 2Change the following sentences from

a Assertive sentencesb Imperative sentencesc Interrogative sentencesd Exclamatory sentences

Sentences can be changed from one grammatical form to another without changing the meaning of the sentence This is known as transformation of sentences

assertive to interrogative1 Nobody would like to be a fool

Who would like to be a fool2 Their glory can never fade

When can the glory fade3 Nobody can control the wind

Who can control the wind4 It matters little if I die

What though I die5 No man can serve two masters

Can any man serve two masters

Exercise 3Interchange of assertive and Exclamatory sentences

1 She leads the most unhappy lifeWhat an unhappy life she leads

2 This is indeed an interesting bookWhat an interesting book this

3 He is a very great manWhat a great man he is

4 It is a very lame excuseWhat a lame excuse

5 It is sad that she died so youngAlas she died so young

Class XISubject Topic Summary Execution

Hindi 2nd lang

पतर परम(परमचदर) पतर परम कहानी म एक निपता की इचछाओ का वणन निकया गया ह अपन बड पतर परभ -ास स निपता चतनय -ास का निवशरष परम था निपता को उसक जनम स ही बडी-बडी आशाए थी उसम दसर बट कतिशव-ास की अपकषा स- उतसाह की मातरा अमिधक थी वह उस इगलड भजकर बरिरसटर बनाना चाहत थभागय का खल भी बडा निनराला ह बीए की परीकषा क बा- वह बीमार पड गया डॉकटरो न भी जवाब - दि-या थाचतन -ास जी बहत ही कजस थ बवजह पस खच करना नही चाहत थ अगर गारटी मिमलती तो शाय- पस खच भी कर -त परत गारटी नही थी परिरणाम सवरप उनक बट का -हात हो गयाजब बट को समशान ल जा रह थ तो वहा काफी शोर गान बजान हो रह थ पछन पर पता चला निक निकसी निपता निपछल तीन साल स निबमार था और उसक ईलाज म रपया पानी की तरह बहाया पर ठीक नही हए परत उसक बट को तनिनक भी अफसोस नही था उसका कहना था उसन कोकतिशश तो कीयह -खकर चतनय-ास जी को आतम निगलानी हईतभी स उनका म परिरवतन हआ और बट का भोज काफी धमधाम स निकयाऔर वहइस पशचाताप की आग म जलत रह औला- स बढकर पसा नही होता ह इस बात को समझन म उनह काफी व लग गया

hellipContinue to next

BENGALI(2ND LANGUAGE)

পরথমঅধযায়-ঠাকরারীনদরনাথঠাকর

নয়ন দোচের হিমাচেররা া নাচেমই হিযাত হিছচেন ায়ানার উাররণ সবরপ নয়ন দোচের ারা হিা (াচেকর হিা হিচেতন এছাাও দেকান উৎস উপচেb রাহিতর দেক হিন করার উচেfচে(য তারা সযC হিকরচেরণ রনয পরীপ জবাহিচেয় তাচেত রপার হির 4Cরণ করচেতন ঠাকরা এই নয়ন দো হিমারচের দে(4 ং(ধর হিছচেন হিমাররা ায়ানার ষটানত পর(Cন কচের তারা হিনঃসব এই হিমাহিরর দে(4 ং(ধর গৈকাস নদর রায়চেৌধরী গৈকাস া নয়ন দোচের সমসত সমপহিতত ঋচেরণর াচেয় হিহিx কচের অহি(ষট যা আচেছ তাচেত হিপত

ইার হিপতার মতয ইচে পর নয়নচোচের ায়ানার দেগাটা কতক অসাধাররণ শরাদধ (াহিনতচেত অহিনতম ীহিপত পরকা( কহিরয়া ঠাৎ হিনহিয়া দেগ- ক) কার দো দেকান গচেলপর অং() কতা দেক ইার চেত কাচেক দোঝাচেনা চেয়চেছ গ) পরসঙগ কী কতার কতয পহিরসফট কচেরা

পরচে4র যাহিত রbা করা সমভ নয় তাই হিতহিন পতরচেক হিনচেয় ককাতায় সাস শর কচেরন গলপ কথচেকর আহিথCক অসথা নয়ন দোচের হিমাচের দেথচেক সমপরণC আাা কথচেকর হিপতা হিনচের দেষটায় অথC উপাCন করচেতন া উপাহিধ াচেভর নয তার াসা হিছনা আর দেসই কারচেরণ কথক তার একমাতর উততরাহিধকার চেয় তার হিপতার পরহিত কতজঞ কথক দো পা হি(চেচেছন হিনচের পরারণ ও মান রbার নয উপচেযাগী অথC হিনা দেষটায় পরাপত চেয়চেছন- এটাই তার কাচেছ পরম দেগৌরচের হি4য় চে মচেন কচেরন কাররণ (নয ভাণডাচের গৈপতক ায়ানার উজজব ইহিতাস অচেপbা দোার হিসeচেকর মচেধয গৈপতক দেকামপাহিনর কাগ তার কাচেছ অচেনক দেহি( মযান

TO BE CONTINUED

উ- ক) আচোয অং(টি রীনদরনাথ ঠাকচেরর দো ঠাকরা গচেলপর অং() কতা চেন আচোয গচেলপর গলপ কথকইার চেত নয়ন দোচের হিমাহিরর দে(4 ং(ধর গৈকাস ার কথা া চেয়চেছ গৈকাস া নয়ন দোচের সমসত সমপহিতত ঋচেরণর াচেয় হিহিx কচের অহি(ষট যা আচেছ তাচেত হিপত পরচে4র যাহিত রbা করা সমভ নয় তাই হিতহিন পতরচেক হিনচেয় ককাতায় সাস শর কচেরনগ) গৈকাস ার হিপতার মতযর পর নয়ন দোচের হিমাহিরর অহিসততব হিপত য় কচেয়কটা উৎস ও শরাদধ- (াহিনতচেত হিমাহিরর দে(4 কহিটক যয় চেয় হিগচেয় এচেক াচের দে(4 চেয় যায় তন তাচের গC করার মত আর হিকছই হিছ না-দেসই পরসচেঙগ এই উহিকত নয়নচোচের হিমাচেররা া নাচেমই হিযাত হিছচেন ায়ানার উাররণ সবরপ নয়নচোচের ারা হিা (াচেকর হিা হিচেতন এছাাও দেকান উৎস উপচেb রাহিতরচেক হিন করচেত হিগচেয় তারা সযC হিকরচেরণর নয পরীপ জবাহিচেয় তাচেত রপার হির 4Cরণ করচেতন তাই দেসকাচের ায়ানা দেহি(হিন সথায়ী চেত পারত না হিহিভনন উৎস শরাদধ- (াহিনতচেত সাধযা হিতহিরকত র করার নয হিমাহির হিহিকচেয় দেযত হ হিতC কা হিহি(ষট পরীচেপর দেত দেযমন অলপকাচের মচেধয হিনঃচে(4 চেয় যায়-নয়নচোচের হিমারচের অসথা তাই চেয়হিছ এই কারচেরণই কথক নয়নচোচের হিমারচের গা ভরা আমবর সয করচেত পারতনা

Physics Dimensional Analysis (Summary)

Q Find the dimensions of consts ab in relation

p=(bminusxlowastx)at

where p is the power x is the distance and t is time

Ans From principle of homogeneity dimension of b x2 are same Dim of b = dim of x2 = [L2] = [ML2T0]Dim of a = dim of ( b- x2)dim of (pt) = [M0L2T0][ML2T-2] [T-1] [T] = [M-1L0T2]

Chemistry Atomic Structure Drawbacks of Rutherfordrsquos model of

atom a According to Rutherfordrsquos model of atom electrons which are negativelycharged particles revolve around the nucleus in fixed orbits Thusb theelectrons undergo acceleration According to electromagnetic theory of Maxwell a charged particle undergoing acceleration should emitelectromagnetic radiation Thus an electron in an orbit should emitradiation Thus the orbit should shrink But this does not happenc The model does not give any information about how electrons aredistributed around nucleus and what are energies of these electrons Isotopes These are the atoms of the same

Properties of electromagnetic radiationsa Oscillating electric and magnetic field are produced by oscillating charged particles These fields are perpendicular to each other and both areperpendicular to the direction of propagation of the waveb They do not need a medium to travel That means they can even travel invacuum

Characteristics of electromagnetic radiationsa Wavelength It may be defined as the distance between two neighbouring crests or troughs of

element having the same atomicnumber but different mass numbere g 1H11H21H3

Isobars Isobars are the atoms of different elements having the same massnumber but different atomic numbere g 18Ar40 20Ca40

Isoelectronic species These are those species which have the same numberof electrons

Electromagnetic radiationsThe radiations which are associated withelectrical and magnetic fields are called electromagnetic radiations When anelectrically charged particle moves under acceleration alternating electricaland magnetic fields are produced and transmitted These fields aretransmitted in the form of waves These waves are called electromagneticwaves or electromagnetic radiations

wave as shown It is denoted by λb Frequency (ν) It may be defined as the number of waves which passthrough a particular point in one secondc Velocity (v) It is defined as the distance travelled by a wave in onesecond In vacuum all types of electromagnetic radiations travel with thesame velocity Its value is 3 times10 8m sec-1 It is denoted by v

d Wave number Wave number is defined as the number of wavelengths per unit lengthVelocity = frequency timeswavelength c = νλ

Plancks Quantum Theory- o The radiant energy is emitted or absorbed not continuously but discontinuously in the form of small discrete packets of energy called lsquoquantumrsquo In case of light the quantum of energy is called a lsquophotonrsquoo The energy of each quantum is directly proportional to the frequency of the radiation ie E α υ or E= hυ where h= Planckrsquos constant = 6626 x 10-27 Js o Energy is always emitted or absorbed as integral multiple of this uantum E=nhυ Where n=1234Black body An ideal body which emits and absorbs all frequencies is calleda black body The radiation emitted by such a body is called black body radiation

Photoelectric effectThe phenomenon of ejection of electrons from thesurface of metal when light of suitable frequency strikes it is calledphotoelectric effect The ejected electrons are called photoelectrons

Biology Chapter - 02Systematics and Five Kingdoms

Scientists divide the whole living organisms into two kingdom first and ultimately by five kingdom at last

In the earlier systems of classifications organisms are divided into kingdom plantaeand kingdom animalia on the of presenceof cell wall their modes of nutrition and movements

Some problem arise like fungi share manycharacteristic withplant despite their heterotrophic nutrition bacteria protozoa areunicellular present in both kingdom Toovercome this third kingdom Protista isintroduced which include

unicellularorganisms But there is also another

problem Allunicellular organisms are not similar kind The cellular structure of prokaryotes is verydifferent from that of other organismsEukaryotes possess a true nucleus and allcell organelles that are not present inprokaryotes So the fourth kingdom Monerais introduced which include unicellular prokaryotes (bacteriaamp blue green algae)

bull Still some problem arise in kingdomplantae

So in 1969 R H Whittakar proposedanew five kingdom System of classification

i) Kingdom Monera - unicellular prokaryotes

ii) kingdom Protista - unicellular eukaryotes

iii) Kingdom Fungi - uni or multicellular fungi with cell wall but without chlorophyll

iv) Kingdom Plantae - Multicellular Plants

v) Kingdom Animalia - Multicellular Animals

EVS Chapter 1 ndash Modes of Existence

An agricultural society

An agricultural society also known as an agrarian society is a society that constructs social order around a reliance upon farming More than half the people living in that society make their living by farming

People in an agricultural society generally lead a more settled lifestyle than those in nomadic hunter-gatherer or semi-nomadic pastoral societies because they live permanently near the land that is farmed Agricultural settlements tend to develop in areas of convenience near bodies of water which is used for both crops and transportation or along trade routes Not everyone in an agricultural society is a farmer Some people make a living trading or making and selling goods such as tools used for farming

Another way to define an agrarian society is to see the total amount of production in a nation In an agrarian society cultivating the land is the main source of wealth Such a society can recognize other means of subsistence and work habits but emphasizes the importance of agriculture and livestock Agrarian societies have existed in various parts of the world for 10000 years and continue to exist today They have been the most common form of socio-economic organization for most of recorded human history

Q) Write the features of agricultural society

Ans - Structure and Features of Agrarian Society1 Occupational Structure

An agrarian society is generally associated with the domestication of plants and animals The domestication of plants means farming and that of animals means herding Often there is mixture of farming and the use of such domesticated animals as cow goat and sheep

2 Forms of Land Ownership in Agrarian SocietiesGenerally there are landlords supervisory farmers cultivators and share croppers The landholders own the land but do not work on it They let it out for sharecropping The supervisory farmers are those who live by having their land cultivated by hired labourers The cultivators cultivate the land for themselvesThe share-croppers are those who live by tilling other peoplersquos land or a crop-sharing basis The artisans own their means of production and produce by their own labour in their homesteads

3 Village Community System An agrarian society is highlighted by

the institution of village community system The agrarian economy made fixed dwelling houses necessary Living close together for protection and co-operation and living nearer to the land gave birth to agricultural villages The village is not only the residential place of farmers it is also the social integrator

4 Minimal Division of Labour Another structural feature of agrarian society is a minimal division of labour Except for the basic division founded on age and sex differences there are few specialized roles There is only one predominant type of occupation ie domestication of plants and animals For all the people the environment physical as well as social is the same

5 Role of Family The farm family is of the patriarchal type the father is the final arbiter in most of the familyrsquos major decisions The life of ail men and women is merged in family life Since there are not many special organizations family is the only organisation to perform the tasks of aid and protection

6 Sense of Unity The members of an agrarian society exhibit a strong in-group feeling Since the whole of their social lives is wrapped up in a society which is physically economically and socially homogenous they are inclined to view the entire outside world as an out group

7 Informal Social Control An agrarian society is regionally divided into villages In a village community the force of traditional mores is more dominant than in the urban community In the village everybody is known to everybody The members in a village community help each other and share the joy and sorrows of each other Crime in an agrarian society is rare

8 Simplicity and Uniformity Life of the people in an agrarian society is marked by simplicity and uniformity Their main occupation is agriculture which largely depends upon the vagaries of nature An agrarian society is a religious society

Math Compound angles Compound angles The algebraic sum of two or more angles is called a compound angle If A B C be three angles then A+B B+C C+A A-B B-C A-C A+B-C etc are compound angles In this chapter we shall discuss the trigonometrical ratios of compound angles Theorem 1 If A B and A+B are all pisitive acute angles theni) sin( A+B) = sin A cos B + cosA sinBii) cos(A+B) = cosA cosB- sinA sinBTheorem 2If A and B are positive acute angles and AgtB theni) sin(A-B) = sin A cosB- cos A sinBii) cos(A-B) = cos A cos B+ sin A sin BTo prove that i) sin(A+B) sin (A-B) = sin2 A - sin2 B = cos2 B- cos2 A

Example 1 Prove that tan70deg=2tan50deg+tan20degSolutiontan70deg = tan(50deg + 20deg)Or tan70deg=(tan 50deg+tan 20deg)(1-tan50degtan20deg) or tan70deg (1 ndash tan 50deg tan20deg) = tan50deg+tan20degor tan70deg= tan70deg tan50deg tan20deg+ tan50deg + tan20deg = cot20deg tan50deg tan20deg + tan50deg + tan20deg = 2 tan50deg+ tan20degExample 2 If A + B = 45deg show that (1 + tanA) (1 + tanB) = 2Solutiontan(A + B) =( tan A + tan B) (1 - tan

ii) cos(A+B) Cos(A-B) = cos2 A- sin2 B = cos2 B -sin2 AProof i) LHS= sin(A+B)sin(AminusB) [Recall sin(αminusβ)=sinαcosβminuscosαsinβ And sin(α+β)=sinαcosβ+cosαsinβ]= (sinAcosB+cosAsinB)times(sinAcosBminuscosAsinB)= sin2Acos2Bminuscos2Asin2B [Recall sin2α+cos2α=1 From above we can then assume correctly that sin2α=1minuscos2α AND cos2α=1minussin2α] = sin2A(1minussin2B)minussin2B(1minussin2A) = sin2Aminussin2Asin2Bminussin2B+sin2Asin2B = sin2Aminussin2B= 1-cos2A-(1-cos2B) = cos2 B- cos2 A = RHSii)LHS= cos (A+B) cos(A-B) [ cos(A+B) = cos AcosB- sinAsinBCos(A-B) = cosAcosB+ sinAsinB]= cos2 A Cos2 B- sin2 A Sin2 B= cos2 A( 1-sin2 B) - (1- cos2 A) sin2 B= cos2 A- cos2 A sin2 B- sin2 B+ cos2 A sin2 B=cos2 A- sin2 B=1- sin2 A-(1-cos2 B) = cos2 B- sin2 A= RHSTangent formulae for compound anglesi)tan (A + B) = tan A + tan B1-tan A tan Bii)tan (A ndash B) = tan A-tan B1+tan A tan Biii) cot (A + B) = cot Acot B-1cot A+cot B(viii) cot (A ndash B) = cot Acot B+1cot B-cot A

A tan B) Or 1= (tan A+ tanB) (1-tan A tanB) Or tanA + tanB + tanA tanB + 1 = 1 + 1Or tanA (1 + tanB) + (1 + tanB) = 2Or (1 + tanA) (1 + tanB) = 2Example 3 Find the value of sin 15degSolution sin 15deg= sin(45deg-30deg) = sin45degcos 30deg- cos45degsin30deg =(1radic2) (radic32) -(1radic2) (12) = (radic3-1) 2radic2Example 4 If sin A = 1 radic10 and sin B = 1 radic5 where A and B are positive acute angles then what is A + B SolutionWe know that sin (A + B) = sin A cos B + cos A sin B= [1 radic10] [radic(1 minus 1 5)] + [1 radic5] radic(1 minus 1 10)= [1 radic10] [radic4 5] + [1 radic5] [radic9 10]= [1 radic50] times (2 + 3)= 5 radic50 = 1 radic2

sin (A + B) = sin π 4rArrHence A + B = π 4Example 5 If A + B = 225o then find [cot A] [1 + cotA] times [cot B] [1 + cot B]Solution[cot A] [1 + cotA] times [cot B] [1 + cot B] = 1 [(1 + tan A) times (1 + tan B)]=1 [tan A + tan B + 1 + tan A tan B] [ tan (A + B) = tan225o]∵

tan A + tan B = 1minus tan A tan BrArr= 1 [1 minus tan A tan B + 1 + tan A tan B]= 1 2

COMMERCE

CLASSIFICTION OF HUMAN ACTIVITIES-ECONOMIC AND NON-ECONOMIC

Firstly we shall recall the previous class for 5 mins especially for the absentees and for also the rest of the students who were there

Today at first we briefly discuss the earlier portions of the chapter

1Business-It includes all those economic activities which are concerned with production and exchange of goods and services with the object of earning profit Example A factory shop beauty parlour also business enterprises

2Profession ndashThe term profession means an occupation which involves application of specialized knowledge and skills to earn a living For Example Chartered Accountancy medicine law tax consultancy are example of professions

Questions1What are the main features of ProfessionAnswer The main features of a profession are as follows a Specialised body of knowledge-Every profession has a specialised and systematised body of knowledge b Restricted entry- Entry to a profession is allowed only to those who have completed the prescribed education and have the specialised examination c Formal education and training ndashA formal education and training is given to the person who wants to acquire the professional

3Employment-Employment mean an economic activity where people work for others in exchange for some remuneration (salary)The persons who work for others are called lsquoemployeesrsquo The persons or organizations which engage others to work for them are called lsquoemployersrsquoEg A doctor working in a hospital is employment as he is working for a salaryA lawyer may serve as a law officer in a bank

With this we shall proceed with the features of both Profession amp Employment

The main features of a profession are as follow

a Specialised body of knowledge b Restricted entry c Formal education and training d Professional association e Service motive f Code of contact

The main features of an employment are as follows

a In employment a person works for others called employer

b An employee provides personal service

c There is a service agreement or contract between the employee and the employer

d The employee has to obey the order of the employer

e No capital investment is made by the employer

Various examples of Employment are as follows

aA teacher teaching in a school or collegeb An engineer employed in Municipal Corporation of DelhicAn accountant working in the accounts department of a companydA doctor working in a hospital

Note In all the above examples of employment the individual who is involved in each example is working as an employee for a salary under an employer

qualification(MBBSCALLB)d Service motive ndashProfessionals are expected to emphasis service more on their clients rather than economic gain f Code of Conduct-The activities of professionals are regulated by a code of conduct

2 What are the main features of EmploymentAnswer The main features of an employment are as followsa In employment a person works for others called employerb An employee provides personal servicec There is a service agreement or contract between the employee and the employerd The employee has to obey the order of the employere No capital investment is made by the employer

3 Give various Professions and their respective Association are given below

Professions

Professional

Professional association

Medical profession

Doctor Medical Council of India

Law profession

Lawyers Bar Council of India

Accounting Profession

Chartered

The Institute of Chartered Accounts of India( ICAI)

Engineerin Engineers The

g Profession

institute of Engineers (India)

Accounts Basic accounting terms

Today we will give you some questions from the previous study material

Questions6) Define accounting7) What do you mean by debit

and credit8) Explain the types of account9) Define the following terms

a) Assetsb) Capitalc) Purchased) Debtorse) Transactions

10) Name the types of accounts given below

a) Krishnas accountb) Machinery accountc) Royalty accountd) Salary accounte) Furniture accountf) Audit fee account

Economics Basic Economic ConceptsSub topic

UTILITY

Before starting todayrsquos class we shall recall the last class which was about UTILITY AND THE FEATURES OF UTILITY

Now we shall proceed with the further topics of the chapter

Todayrsquos topic from the chapter lsquo Basic Economic Conceptsrsquo will be TOTAL UTILITY amp MARGINAL UTILITYNow let us quickly revise the concept of utility with an example ie goods and services are designed because they have an ability to satisfy human wantsThis feature of being able to satisfy human wants is termed as utility For example we derive utility from WiFi services as it gives us satisfaction by connecting us to our friends and family through social media here consumers derive utility from WiFi services

From the above concept we shall start with todayrsquos topicEconomists have defined TOTAL UTILITY (TU) as the total satisfaction obtained by consuming a given total amount of a good and serviceFor example the total satisfaction obtained from eating 10 mangoes is the total utility of 10 mangoes

MARGINAL UTILITY (MU) is the additional satisfaction derived from each additional unit

Questions1 What is Total Utility (TU)

Answer Total Utility (TU) is the

aggregate of the utility that a consumer derives from the consumption of a certain amount of a commodityTU=MU1+MU2++MUn

2 What is Marginal UtilityAnswer

Marginal Utility (MU) is the additional made to the total utility as consumption is increased by one more unit of the commodityMU= TUn ndashTUn-1

NoteOften economists tend to

subdivide utility into an imaginary unit called UTIL

consumed In this casethe utility obtained from each mango as it is consumed as the MU of that mango It is also defined as the addition made to the total utility when an additional unit is consumed Often economists tend to subdivide utility into an imaginary unit called UTIL

Note As a consumer increases the consumption of a good over period of time the total utility or total satisfaction derived from it increases to appoint and thereafter it decreasesHowever as the consumer keeps on consuming the good the marginal utility or the additional utility derived from it decreases

SubjectBusiness studies

Topic

BUSINESSENVIRONMENT

Summary

Now quickly let us revise the earlier points that we have already done in the last class and let us proceed with the other topics that are there in the chapter

Firstly we will recall the internal and external factors of micro environment and then we shall proceed in details

Meaning and list of internal and external factors

aInternal factorsInternal factors refer to all the factors existing within a business firm The internal factors are considered controllable because the enterprise has control over these factorsFor an example a company can alter its organization structure policies programmes employees physical facilities and marketing mix to suit the changes in the environmentList of internal factors areCorporate culture mission and objectives top management organizations structure company image and brand equity company resources

b External factorsExternal factors refer to those individual and groups and agencies with which a particular business organization comes into direct and frequent contact in the course of its functioningThese individuals and groups are known as STAKEHOLDERS because they have a stake (financial interest ) in the working and performance of the particular business List of external forces (stakeholders)Customers competitors investors suppliersmiddlemen (marketing intermediaries)

Execution 1 What do you mean by internal

factors in micro environmentAnswerInternal factors refer to all the factors existing within a business firm The internal factors are considered controllable because the enterprise has control over these factorsFor an example a company can alter its organization structure policies programmes employees physical facilities and marketing mix to suit the changes in the environment

2 What do you mean by external factors in micro environment

AnswerExternal factors refer to those individual and groups and agencies with which a particular business organization comes into direct and frequent contact in the course of its functioningThese individuals and groups are known as STAKEHOLDERS because they have a stake (financial interest) in the working and performance of the particular business

3Who are stakeholdersSTAKEHOLDERS are individuals and groups who have a stake (financial interest ) in the working and performance of the particular business 4Discuss the internal factors in briefa Corporate CultureThe values beliefs and attitudes of the founders and top management of the company exercise

financers publics

customers

suppliersfinancers

competitors

middlemen

publics

Fig STAKEHOLDERS OF A COMPANY

Apart from micro environment the other main dimension of business environment isMacro environment Macro environment refers to the general environment or remote environment within which a business firm and forces in its micro environment operateA company does not directly or regularly interact with the micro environmentTherefore macro environment is also known as indirect action EnvironmentThe macro environment forces are less controllable than the micro forces

Macro environment consists of the following components

POLITICAL AND LEGAL ENVIRONMENT

ECONOMIC SOCIAL AND ENVIRONMENT

CULTURAL

ENVIRONMENT

TECHNOLOGICAL ENVIRONMENT

a strong influence on what the cmpaany stands for how it does things and what it considers importantbMission and objectivesThe business philosophy and purpose of a comoany guide it prioritiesbusiness strategiesproduct market scope and development scope

cTop management structurethe composition of board of directors the degree of professionalization of management and the organizational structure of a company have important bearing on its business decisions

dPower structureThe internal power relationship between the board of directors and the chief executive is an important factor

eCompany image and brand equityThe image and brand equity of the company play a significant role in raising finance forming alliance choosing dealers and suppliers launching new products entering foreign markets

5 What is Macro environmentAnswerMacro environment refers to the general environment or remote environment within which a business firm and forces in its micro environment operateA company does not directly or regularly interact with the micro environmentTherefore macro environment is also known as indirect action EnvironmentThe macro environment forces are less controllable than the micro forces 6 What are the components of macro environmenta Political and legal environmentb Economic environmentc Social and cultural environmentd Technological environment

BUSINESS FIRM

Fig COMPONENTS OF MACRO ENVIRONMENTPolitical science

Introduction to political science

Comparative politics and itrsquos scope Comparative politics is the second major dimension of political scienceIt is also a very vast area of study and a very large number of political scientists even treat it as an autonomous area of study within the board ambit of political scienceScope of comparative politics-

1 All political structures -Comparative politics includes the study of all structures formalnon formal governmental and extra governmental which are directly or indirectly involved in politics in all the countries of the world

2 Functional studies- Comparative politics seeks to study politics less from the point of view of the legal institutions in terms of their powers and move from the point of view of their functions which constitute the political process and their actual Operation in the environment

3 Study of political behaviour- Another important part of its scope is the study of the actual behaviour of the people in the process of politics

4 Study of similarities and differences- comparative politics also undertakesan analysis of the similarities and differences among political process and functions

5 Study of all political systems -comparative politics seeks to analyse the actual behaviour and performance of all political systems western as well as non western

6 Study of the environment and infrastructure of politics-The study of politics demands a study of the psychological sociological economic and anthropological environment in fact the social environment as a whole in which each political system operates

7 Study of political culture- political culture is composed of attitudesbeliefs emotions and values of a society that relate to the political system or politics

8 Study of political participation- Political participation is a universal processThe only difference is that while in some states it is limited in others it is wider

9 Study of political process- political

Answer the following questions-

What is comparative politics

What are the scope of comparative politics

Homework- learn

processes like decision makingpolicy making judicial process leadership recruitment process and others are always at work in all political systems

The scope of comparative politics is very comprehensive It includes everything that falls within the area of political activity and political process

History CAMBRIDGE VIEW ABOUT

THE PARTITION

AND REFUTATION

OF CAMBRIDGE

VIEW

Cambridge view about the Partition The Cambridge school of historians have interpreted that opposition to partition scheme was made entirely by the elitist groups They hold the view that Lord Curzon planned to partition the Bengal for administrative purposeREFUTATION OFCAMBRIDGE VIEW The Rationalist historians have rejected the interpretations of the Cambridge School of historians on various grounds

1 QUESTION State different views of historians regarding Partition of Bengal

ANSWER Cambridge historians believed that Lord Curzon partitioned Bengal for administrative reasons only and not for the political motive The Middle class elitist group protested because of their petty interest The Hindu zamindars protested as they have to spend more money for managing their estatesThe lawyers of Calcutta High court feared to lose their clientBut according to the nationalist Historians was-

2- The ultimate object of Lord Curzon was to crush the unity of Bengal politicians

3- If Bengal becomes a separate province Bengali speaking 16 million people of western part would become minority under Hindi speaking people of Bihar and Oriya speaking people of Orissa

4- The bureaucrats expected that the protest movement would die down quickly

5- Lord Curzon used the Muslim community in his political game

6- Idealism had great contribution in the protest against partition

7- The people of the every section of society were affected by the partition of Bengal

Computer Science

Numbers Convertion of dcimal number to octal numberThe decimal numeral system is the standard system for denoting integer and non-integer numbers It is the extension to non-integer numbers of the Hindu-Arabic numeral system For writing numbers the decimal system uses ten decimal digits a decimal mark and for negative numbers a minus sign - The decimal digits are 0 1 2 3 4 5 6 7 8 9 the decimal separator is the dot in many countries

The octal numeral system or oct for short is the base-8 number system and uses the digits 0 to 7 Octal is sometimes used in computing instead of hexadecimal perhaps most often in modern times in conjunction with file

permissions under Unix systems It has the advantage of not requiring any extra symbols as digits It is also used for digital displays

Follow these steps to convert a decimal number into octal form

1 Divide the decimal number by 82 Get the integer quotient for the next iteration (if the number will not divide equally by 8 then round down the

result to the nearest whole number)3 Keep a note of the remainder it should be between 0 and 74 Repeat the steps until the quotient is equal to 05 Write out all the remainders from bottom to top This is the solution

For example if the given decimal number is 8453

Division Quotient Remainder

8453 8 1056 5

1056 8 132 0

132 8 16 4

16 8 2 0

2 8 0 2

Then the octal solution is 20405

Subject Eng Literature (The Tempest ndash William Shakespeare) Topic Act I Scene 1 Lines 33 to 67 (End of scene) Date 16th April 2020 (4th Period)

[Students should read the original play and also the paraphrase given in the school prescribed textbook]Summary Questions amp Answers

[SUMMARY OF THE ENTIRE SCENE]

o The play starts with the scene of a severe storm at sea Alonso (King of Naples) Sebastian (Alonsorsquos brother) Ferdinand (Alonsorsquos son) Gonzalo Antonio (the usurping Duke of Milan) are in a ship in the midst of the storm

o The mariners are trying their best to control the vessel from running aground and are totally following the orders of their Master the Boatswain They have scant success

o The mariners become extremely unhappy and annoyed when most of the passengers arrive on the deck thereby hampering their effort to save the ship There is serious confrontation between them and the passengers who are part of the Kingrsquos entourage

o The mariners could not save the ship

SUMMING-UP

(i) Vivid description of the scene which gives a realistic description of terror and confusion of a tropical storm

(ii) Shows Shakespearersquos accuracy of knowledge in describing the naval operations and also matters of seamanship

(1) GONZALO Ill warrant him for drowning (L 45-57)

though the ship were no stronger than a nutshell and as leaky as an unstanched

wenchBOATSWAIN Lay her a-hold a-hold Set her two courses Off to

sea again lay her offMARINERS All lost To prayers to prayers All lostBOATSWAIN What must our mouths be coldGONZALO The king and prince at prayers Lets assist them

For our case is theirsSEBASTIAN Im out of patienceANTONIO We are merely cheated of our lives by drunkards

This wide-chopped rascal - would thou mightst lie drowning the washing of ten tides

(a) What does Antonio say at the insolent manners of the boatswain just before the given passage

Being irritated at the insolent manners of the boatswain just before the given extract Antonio the Duke of Milan calls him a worthless dog son of a woman without any morals an arrogant and disrespectful noisemaker He says that the boatswain deserved to be hanged(b) What statement does Gonzalo repeat about the boatswain

Gonzalo shows his faith that the boatswain is not destined to die by drowning He is destined to be hanged and nothing can alter this decree of destiny He says that even if the ship was as frail as a nutshell the boatswain could not be drowned for his destiny was to be hanged(c) What do the passengers do when they have lost all hope of their survival

When the passengers have lost all hope of survival they take

(iii) The opening scene justifies the title ndash The Tempest

UNANSWERED QUESTIONS

(i) The King always travels with his entire fleet including his soldiers Where were the other ships

(ii) Why was the ship in that area Where was it coming from or going where

(iii) The ship broke apart What happened to those who were in the ship

(We shall get the answer to the above questions as the play progresses)

leave of life with fervent prayers The mariners take their last hearty drink and are ready for death(d) What blame does Antonio put upon the mariners and the boatswain Antonio rebukes the mariners that these drunkards have brought them to the present crisis by neglecting their duties He blames them saying that they are going to lose their lives entirely for the negligence of the boatswain and his fellows(e) What does Antonio say while cursing the boatswain

Antonio gives vent to his wrath upon the boatswain in particular He calls the boatswain a wide-mouthed rascal who deserves to be hanged on the sea-shore at low water mark so that ten tides might wash over his body and take out of him all the liquor that he has been drinking

Class XIISubject Topic Summary ExecutionHistory Topic

1 1935 ACT AND WORKING OF PROVINCIAL AUTONOMYCONGREE AND OTHER MINISTERSSUB TOPIC GOVERNMENT OF INDIA ACT1935

Government of India Act 1935 This act established a lsquoFederation of Indiarsquo made of British Indian provinces and Indian states and provided for autonomy with a government responsible to the elected legislature in every provinceThis act introduced abolition of Diarchy at provinces The entire provincial administration was introduced to the responsible ministers who were controlled and removed by the provincial legislature The provincial autonomy means two things First The provincial governments were wholly responsible to the provincial legislature Secondly Provinces were free from outside control and interference in the large number of matters The act divided the powers between the centre and provinces in terms of three lists- Federal list( for centre) Provincial list (for province) and concurrent list (for both) Residuary powers were given to the viceroy In the election under the government of India Act the Congress swept the poll the mandate of the people came in favour of the congress so far as general Hindu seats were concerned The Congress did not get a single Muslim seates in Bombay CP UP Sind and BengalIn five provinces Congress had yhe clear majority In BengalNWFPAssam and Bombay Congress emerged as a single largest partyOn the other side the performance of the Muslim League was badThus the Congress formed ministers in 7 provinces out of 11 provinces Coalition ministry was also formed in two other provincesOnly BENGAL AND Punjab had non- congress ministries

1 QUESTION What was the main change introduced by the Government of India ActANSWER a) The Act gave more

autonomy to the provinces b) Diarchy was abolished at the

provincial levelsc) The Governor was the head of

the executived) There was a council of

ministers to advise him The ministers were responsible to the provincial legislatures who controlled them The legislature could also remove the ministers

e) The Governors still retained special reserve powers

2 QUESTION Why did the federal scheme introduced by the Government of India Act 1935 never come into operation

ANSWER The Federal structure of the Government of India was to be composed with the Governor General and Council of ministers The Federal legislature was to be Bicameral legislature- The council of states and the House of Assembly The ministers were to be chosen by the Governor general and they were to hold the office during his pleasure

The provinces of British India would have to join the federation but this was not compulsory for the princely states

This federation never materialised because of the lack of support from the required number of

princely statesThis act was refused and

rejected by the princes the Congress and the Muslim League

Thus both Congress and the League participated in the election of 1937 Thus the federal part was never introduced but the provincial part was put into operations

Bengali 2nd

Language

াচেরর পরাথCনা(কহিতা )

াচেরর পরাথCনা কহিতাটি কহি (ঙখ দেঘাচে4র দো আচো য কহিতায় াচেরর পতর হমায়ন কঠিন দেরাচেগ আxানত ার ঈশবর া আললার কাচেছ পরাথCনা কচেরচেছন তার পচেতরর ীন হিফহিরচেয় হিচেত এই কহিতায় ার পচেতরর ীন হিভbা দেচেয়চেছন ারার এমনহিক হিনচের ীন হিসCচেনর হিহিনমচেয় হিতহিন তার দেছচের ীন হিফচের দেপচেত দেচেয়চেছন তার দেছচের এই দেরাচেগর ন য হিতহিন হিনচেচেকই ায়ী কচেরচেছন তার হিনচের করা পাপচেকই হিতহিন ায়ী কচেরচেছন এছাা রানৈনহিতক ও আথCসামাহিক অসথার কথা তচে ধরা চেয়চেছ এই কহিতায় ার তার হিনচের পাপ কমCচেকই ায়ী কচেরচেছ ার অন যায় ভাচে দেপহি((হিকতর মাধ যচেম অপররা য কচেরচেছ আর এই অন যায় কাচের ন যই তার পহিরাচের হিপযCয় এচেসচেছ দে এক পরকার মানহিক নধন ইহিতাচেসর ার হিপতা চেয় সবাভাহিকভাচে ভাচোাসা দে মমতা দেথচেক মকত চেত পাচেরনহিন তাই হিপতা চেয় আললা া ভগাচেনর কাচেছ পতর হমায়চেনর পরানহিভbা দেচেয়চেছন ার আললা া ভগাচেনর কাচেছ াহিনচেয়চেছন তার হিনচের ীন হিসCন হিচেত হিতহিন রাী তার হিহিনমচেয় পচেতরর ীন হিফচের দেপচেত দেচেয়চেছন াচেরর হিপতসভ হিচেকর কথা এই কহিতায় ফটিচেয় দেতাা চেয়চেছ হিপতা পচেতরর হিরাহিরত মান নধচেনর কথা তচে ধরা চেয়চেছ

হিচে(4 হিকছ াইচেনর তাৎপযC১) ldquoদেকাথায় দেগ ওর সবচছয দেৌন দেকাথায় কচেরায় দেগাপন bয়ldquoউততর) াচেরর পতর হমায়ন কঠিন দেরাচেগ অসসথ তাই তার দেযৌন াহিরচেয় যাচেচছ এই দেরাচেগ তাচেক দেগাপচেন কচেরকচের াচেচছ তার সক (হিকত ধীচের ধীচের bয় চেচছ তাই হিপতা চেয় ার আললার কাচেছ হমায়চেনর পরান হিভbা দেচেয়চেছন২) ldquoাগাও (চেরর পরাচেনত পরানতচের ধসর (ন দেযর আান গানldquoউততর) াচেরর পতর হমায়ন কঠিন দেরাচেগ আxানত তাই ার আ দে(াচেক মমCাত (চেরর পচেথ পরানতচের আান গান ধবহিনত দোক দেসই আান গান আললার কাচেছ দেযন চে যায় আললা দেযন এই আহিতC শচেন পচেতরর ীন হিফহিরচেয় দেয় ৩)ldquoনাহিক এই (রীচেরর পাচেপর ীানচেত দেকানই তরারণ দেনই ভহি4চেতরldquoউততর) হমায়চেনর অসসথতার ন য ার হিনচেচেকই ায়ী কচেরচেছন কারন ার অচেনক রা য অন যায় ভাচে কচেরচেছ তাই তার এই পাপ কাচের ন য তার ঘচের আ হিপ এচেসচেছ এই অন যায় কাচের ন য তার মহিকত দেনই তাই ার আললার কাচেছ এই পাপ কাচেযCর ন য bমা পরাথM

Hindi 2ndlang

-ासी(जयशकर परसा-)

-ासी जयशकर परसा- की एक ऐसी कहानी ह जिजसम भारतीय ससकनित और राषटरीयता का सवरगजीतहोता ह इस कहानी म इरावती एक निहद कनया ह जिजस मलअचछो न मलतान की लट म पकडा और -ासी बना दि-या उस 500 दि-न -कर काशी क एक महाजन न खरी-ा दसरी -ासी निफरोजा ह वह गलाम ह निफरोजा को छडान क कतिलए अहम- को 1000 सोन क कतिसकक भजन थ जो अभी तक नही आए थ राजा साहब कठोर होत हए भी निफरोजा को निबना धनराकतिश क कतिलए उस म कर -त ह वनिफरोजा को अहम- को समझान की बात कहत हकहानी क अत म हम -खत ह निक इरा वती और जाटो क सर-ार बलराज का मिमलन होता हअहम- को यa म मार दि-या जाता ह वहा निफरोजा की परसननता की समामिध बनती ह वहा एक फल चढती ह और डीजल आती ह निफरोजा उस समामिध की आजीवन -ासी बनी रहती हलखक अपन उददशय अथात -ास परथा पर परकाश डालन और इस परथा क कारण होन वाल -ातो क दखो को दि-खान म पणता सफल हए ह

helliphellipContinue to next

Biology Reproductio Today we will discuss about vegetative Q1 Name some vegetative propagules

n in Organisms

propagation of plants The process of multiplication in which fragments of plant body function as propagule and develop into new individual is called vegetative propagation The units of such propagation are runner rhizome tuber bulb etc

and the speciesinvolvedVegetative propagules

Parts involved

Bulb StemBulbil BulbilRhizome Stem Runner Stem Tuber Stem Offset Stem Leaf buds Leaves Suckers Stem

Corns Stem stolon

Q2 State advantages of vegetative propagation

i) Rapid methodii) Sure and easy methodiii) Useful in plants that cannot

produce viable seeds or long seed dormancy

iv) Maintains purity of raceQ 3 Banana fruit is said to be parthenocarpic where as turkey is said to be parthenogenetic WhyBanana develops without fertilization from an unfertilized ovary thus is parthenocarpicIn turkey the ovum or female gamete developinto a new chick without fertilization thus isparthgenetic

Q4 Why is water hyacinth is called as a ldquoTerror of Bengalrdquo Water hyacinth can

propagatevegetatively all over the water body in a short per short period of time This resulted increased biochemicaloxygen oxygen demand of water body causing mortalityof fishes It is very difficult to get rid off them Thus known as terror of Bengal

Chemistry

Solid state GENERAL CHARACTERISTICS OF SOLID STATEIn nature the particular state of matter is governed by two opposing forces at given set of temperature and pressure These forces are intermolecular force of attraction and thermal energy If intermolecular force of attraction is high as compared to thermal energy particles remains in closest position

Intext QuestionsQ1 Classify the following solids as crystalline and amorphous Sodium chloride quartz glass quartz rubber polyvinyl chloride Teflon

A1 Crystalline

and hence very less movement in particles is observed In this case solid state is the preferred state of matter

Let us revise the general characteristics of solid

i) Fixed mass volume and shape

ii) Strong intermolecular force of attraction

iii) Least intermolecular space

iv) Fixed position of constituent particles

v) Incompressible and rigid

Q2 what type of interactions hold the molecules together in a polar molecular solid[CBSE 2010]A2 The molecules in a solid are held together by van der Waals forces The term van der Waals forces include hydrogen bonding dipole-dipole attraction and London dispersion forces All molecules experience London dispersion forces In addition polar molecules can also experience dipole-dipole interactions So the interactions that holds the molecule together in polar molecular solid are London dispersion force and dipole-dipole interactionsQ3 Write a feature that will distinguish a metallic solid from an ionic solid [CBSE 2010]A3 Metals are malleable and ductile whereas ionic solid are hard and brittle Metallic solid has typical metallic lustre But ionic solid looks dullQ4 Write a point of distinction between a metallic solid and an ionic solid other than metallic lustre [CBSE 2012]A4 Metals are malleable and ductile whereas ionic solid are hard and brittleQ5 Write a distinguish feature of metallic solid [CBSE 2010]A5 The force of attraction in

solid Sodium chloride Quartz Amorphous solid Quartz glass rubber polyvinyl chloride Teflon Q2 why glass is considered as super cooled liquidA2 Glass shows the tendency to flow at slower rate like liquid Hence they considered as super cooled liquidQ3 why the window glass of old buildings show milky appearance with timeA3 Glass is an amorphous solid Amorphous solid has the tendency to develop some crystalline character on heating Due to heating in day over the number of years glass acquires some crystalline character and show milky appearanceQ4 why the glass panes fixed to window or doors of old building become slightly thicker at bottomA4 Glass is super cooled liquid It has the tendency to flow down very slowly Due to this glass pane becomes thicker at the bottom over the timeQ5 Sodium chloride is a crystalline solid It shows the same value of refractive index along all the direction TrueFalse Give reasonA5 FalseCrystalline solid shows anisotropy in properties That is it shows different values for the given physical property in different direction All the crystalline solids show anisotropy in refractive index Therefore sodium chloride will show different values of refractive index on different directions

Q6 Crystalline solid are anisotropic in nature What does this statement means

between the constituent particles is special kind of electrostatic attraction That is the attraction of positively charged kernel with sea of delocalized electronsQ6 which group of solid is electrical conductor as well as malleable and ductile [CBSE 2013]A6 Metallic solidQ7 why graphite is good conductor of electricity although it is a network (covalent solid)A7 The exceptional property of graphite is due to its typical structure In graphite each carbon is covalently bonded with 3 atoms in same layer The fourth valence electron of each atom is free to move in between different layersThis free electron makes the graphite a good conductor of electricity

[CBSE 2011]A6 Anisotropy is defined asrdquo Difference in properties when measured along different axis or from different directionsrdquo Crystalline solid show different values of some of the physical properties like electrical resistance refractive index etcwhen measured along the different directions The anisotropy in crystalline solid arises due to the different arrangement of particles in different directions

Math Function Composition of functions Think of an industrial plant that produce bottles of cold drinks first there is the operation (or function) f that puts the cold drink inside the bottle followed by the opeartion g that close the bottle with the capThis leads to the following definitionDefinition Let f A rarr B and g B rarr C be two functions Then the composition of f and g denoted by gof is defined as the function gof A rarr C given by gof(x) = g(f (x)) forall x isinA

Definition A function f X rarr Y is defined to be invertible if there exists a function g Y rarr X such that gof = IX and fog = IY The function g is called the inverse of f and is denoted by f -1

Thus if f is invertible then f must be one-one and onto and conversely if f is one-one and onto then f must be invertible This fact significantly helps for proving a function f to be invertible by showing that f is one-one and onto specially when the actual inverse of f is not to be determined

Example 1 Let f 2 3 4 5 rarr 3 4 5 9 and g 3 4 5 9 rarr 7 11 15 be functions defined as f(2) = 3 f(3) = 4 f(4) = f(5) = 5 and g (3) = g (4) = 7 and g (5) = g (9) = 11 Find gofSolution We have gof(2) = g (f(2)) = g (3) = 7 gof(3) = g (f(3)) = g (4) = 7gof(4) = g (f(4)) = g (5) = 11 and gof(5) = g (5) = 11Example 2 Find gof and fog if f R rarr R and g R rarr R are given by f(x) = cos x and g (x) = 3x2 Show that gof ne fogSolution We have gof(x) = g(f(x))=g(cosx) = 3 (cos x)2

= 3 cos2 x Similarly fog(x)=f(g (x))= f(3x2)= cos (3x2) Note that 3cos2 x ne cos 3x2 for x = 0 Hence gof ne fogExample 3 Show that if f A rarr B and g B rarr C are onto then gof A rarr C is also ontoSolution Given an arbitrary element z isin C there exists a pre-image y of z under g such that g (y) = z since g is onto Further for y isin B there exists an element x in A with f(x) = y since f is onto Therefore gof(x) = g (f(x)) = g (y) = z showing that gof is onto Example 4 Let Y = n2 n isin N sub N Consider f N rarr Y as f(n) = n2 Show that

f is invertible Find the inverse of fSolution An arbitrary element y in Y is of the form n2 for some n isin N This implies that n =radicy This gives a function g Y rarr N defined by g (y) =radicy Nowgof (n) = g (n2)=radicn2 = n and fog (y) =f(radicy) = (radicy) 2 y which shows that gof=IN and fog= IY Hence f is invertible with f -1 = g

Political Science

Constitution of India-The Preamble

Summary

Objective of the state-To secure equality of status and of opportunity To promote fraternity among all the citizens To assure the dignity of the individuals and Unity and integrity of the nation

Justice-Justice stands for rule of law absence of arbitrariness and a system of equal rights freedom and opportunities for all in a society India seeks social economic and political justice to ensure equality to its citizens

Liberty-Liberty implies the absence of restraints or domination on the activities of an individual such as freedom from slavery serfdom imprisonment despotism etc The Preamble provides for the liberty of thought expression belief faith and worship

Equality-Equality means the absence of privileges or discrimination against any section of the society The Preamble provides for equality of status and opportunity to all the people of the country

Fraternity-The Preamble declares that fraternity has to assure two thingsmdashthe dignity of the individual and the unity and

Execution

Answer the following questions-

Short notes-1 Equality2 Fraternity3 Justice4 Liberty

Homework-Learn

integrity of the nation The word integrity has been added to the Preamble by the 42nd Constitutional Amendment (1976)

Business studies

Human resource management (chapter 1)

On the day of 1504 2020 I have discussed with you the managerial functions and procurement functions of HRM

Today weare going to discuss about the development function integration functions and maintenance function

Development functions-HRM improves the knowledge skills attitude and values of employees so that they the present and future jobs more effectively it includes

1) Development functions of HRM

a) Performance appraisal = It implies systematic evaluation of employees with respect to their performance on the job and their potential for development

b) Training =It is the process by which employees learn knowledge skills and attitudes to achieve organisational and personal goals

c) Executive development = It is the process of developing managerial talent through appropriate program

2) Integration functionsa) HRM reconcile the goals of

organisation with those of its members through integrating function

b) HRM tries to motivate employees to various financial and non financial incentives provided in job specification etc

3) Maintenance functiona) HRM promote and protect the

physical and mental health of employees by providing several types of benefits like housing medical aid etc

b) It Promote Social security measures to employees by providing provident fund pension gratuity maternity benefits

SubjectCOMMERCE

Topic

BUSINESSENVIRONMENT

Summary

Now quickly let us revise the earlier points that we have already done in the last class and let us proceed with the other topics that are there in the chapter

Firstly we will recall the internal and external factors of micro environment and then we

Execution 3 What do you mean by internal factors

in micro environmentAnswerInternal factors refer to all the factors existing within a business firm The internal factors are considered controllable because the enterprise has control over these factors

Development FunctionsPerformance AppraisalTrainingExecution Development

shall proceed in details

Meaning and list of internal and external factors

aInternal factorsInternal factors refer to all the factors existing within a business firm The internal factors are considered controllable because the enterprise has control over these factorsFor an example a company can alter its organization structure policies programmes employees physical facilities and marketing mix to suit the changes in the environmentList of internal factors areCorporate culture mission and objectives top management organizations structure company image and brand equity company resources

b External factorsExternal factors refer to those individual and groups and agencies with which a particular business organization comes into direct and frequent contact in the course of its functioningThese individuals and groups are known as STAKEHOLDERS because they have a stake (financial interest ) in the working and performance of the particular business List of external forces (stakeholders)Customers competitors investors suppliersmiddlemen (marketing intermediaries)financers publics

customers

suppliersfinancers

For an example a company can alter its organization structure policies programmes employees physical facilities and marketing mix to suit the changes in the environment

4 What do you mean by external factors in micro environment

AnswerExternal factors refer to those individual and groups and agencies with which a particular business organization comes into direct and frequent contact in the course of its functioningThese individuals and groups are known as STAKEHOLDERS because they have a stake (financial interest) in the working and performance of the particular business

3Who are stakeholdersSTAKEHOLDERS are individuals and groups who have a stake (financial interest ) in the working and performance of the particular business 4Discuss the internal factors in briefa Corporate CultureThe values beliefs and attitudes of the founders and top management of the company exercise a strong influence on what the cmpaany stands for how it does things and what it considers importantbMission and objectivesThe business philosophy and purpose of a comoany guide it prioritiesbusiness strategiesproduct market scope and development scope

cTop management structurethe composition of board of directors the degree of professionalization of management and the organizational structure of a company have important bearing on its business decisions

dPower structureThe internal power relationship between the board of directors and the chief executive is an important factor

e Company image and brand equityThe image and brand equity of the company play a significant role in raising finance forming alliance choosing dealers and suppliers launching new products entering foreign markets

5 What is Macro environmentAnswerMacro environment refers to the general

competitors

middlemen

publics

Fig STAKEHOLDERS OF A COMPANY

Apart from micro environment the other main dimension of business environment isMacro environment Macro environment refers to the general environment or remote environment within which a business firm and forces in its micro environment operateA company does not directly or regularly interact with the micro environmentTherefore macro environment is also known as indirect action EnvironmentThe macro environment forces are less controllable than the micro forces

Macro environment consists of the following components

POLITICAL AND LEGAL ENVIRONMENT

ECONOMIC SOCIAL AND ENVIRONMENT

CULTURAL

ENVIRONMENT

TECHNOLOGICAL ENVIRONMENT

Fig COMPONENTS OF MACRO ENVIRONMENT

environment or remote environment within which a business firm and forces in its micro environment operateA company does not directly or regularly interact with the micro environmentTherefore macro environment is also known as indirect action EnvironmentThe macro environment forces are less controllable than the micro forces 6 What are the components of macro environmenta Political and legal environmentb Economic environmentc Social and cultural environmentd Technological environment

Computer Science

Logic gates

Digital systems are said to be constructed by using logic gates These gates are the AND OR NOT NAND NOR EXOR and EXNOR

BUSINESS FIRM

gates The basic operations are described below with the aid of truth tables

AND gate

The AND gate is an electronic circuit that gives a high output (1) only if all its inputs are high A dot () is used to show the AND operation ie AB Bear in mind that this dot is sometimes omitted ie ABOR gate

The OR gate is an electronic circuit that gives a high output (1) if one or more of its inputs are high A plus (+) is used to show the OR operationNOT gate

The NOT gate is an electronic circuit that produces an inverted version of the input at its output It is also known as an inverter If the input variable is A the inverted output is known as NOT A This is also shown as A or A with a bar over the top as shown at the outputs The diagrams below show two ways that the NAND logic gate can be configured to produce a NOT gate It can also be done using NOR logic gates in the same way

NAND gate

This is a NOT-AND gate which is equal to an AND gate followed by a NOT gate The outputs of all NAND gates are high if any of the inputs are low The symbol is an AND gate with a small circle on the output The small circle represents inversion

NOR gate

This is a NOT-OR gate which is equal to an OR gate followed by a NOT gate The outputs of all NOR gates are low if any of the inputs are highThe symbol is an OR gate with a small circle on the output The small circle represents inversion

EXOR gate

The Exclusive-OR gate is a circuit which will give a high output if either but not both of its two inputs are high An encircled plus sign ( ) is used to show the EOR operation

EXNOR gate

The Exclusive-NOR gate circuit does the opposite to the EOR gate It will give a low output if either but not both of its two inputs are high The symbol is an EXOR gate with a small circle on the output The small circle represents inversion The NAND and NOR gates are called universal functions since with either one the AND and OR functions and NOT can be generated

Note A function in sum of products form can be implemented using NAND gates by replacing all AND and OR gates by NAND gates A function in product of sums form can be implemented using NOR gates by replacing all AND and OR gates by NOR gates

Logic gate symbols

Table 2 is a summary truth table of the inputoutput combinations for the NOT gate together with all possible inputoutput combinations for the other gate functions Also note that a truth table with n inputs has 2n rows You can compare the outputs of different gates

Logic gates representation using the Truth table

Example

A NAND gate can be used as a NOT gate using either of the following wiring configurations

Subject Eng Literature (The Tempest ndash William Shakespeare) Topic Act III Scene 3 Lines 53 to 110 (End of the scene) Date 16th April 2020 (2nd Period)

[Students should read the original play and also the paraphrase given in the school prescribed textbook]Summary Questions amp Answers

o Seeing this strange scene all are inclined to believe the tales told by travelers that there truly are ldquounicornsrdquo and ldquothe phoenixrsquo thronerdquo

o As they are about to sit down to the feast the banquet is snatched away by a harpy (Ariel disguised) A spiritrsquos voice (Arielrsquos voice) denounces Alonso Sebastian and Antonio with particular

1 ARIEL You are three men of sin whom Destiny

(Line 53-58)That hath to instrument this

lower world And what is int the never-surfeited sea

Hath caused to belch up you and on this island

Where man doth not inhabit you rsquomongst men

Being most unfit to live I have made you mad

reference to their crime in expelling Prospero from Milan They have not received any punishment for their deed earlier but the time for their punishment has arrived Upon Alonso it pronounces ldquolingering perdition worse than deathrdquo from which there is no remedy except through sincere repentance Ariel then vanishes in thunder and the shapes enter again and carry away the table

o Prospero watching invisibly is very pleased with the performance of Ariel and his (Prosperorsquos) ldquomeaner ministersrdquo All his enemies are now in his power and are in a fit of desperation He then leaves them and goes to see how Ferdinand and Miranda are getting on

o Alonso is now much humbled and penitent with the after effect of the spiritrsquos denunciation of his crimes He believes that his son is lost forever After this all disperse being stricken mad by the speech of the spirit

o Gonzalo fearing that they may do violence to themselves or to one another follows them and bid others to follow

(a) To whom does Ariel disguised as a harpy call the three sinners What game did Fate of Destiny play with

them

The three sinners called by Ariel are Alonso Sebastian and Antonio It was Destiny which had caused the ocean to cast the three sinners on the shore Though the ocean is all the time devouring whatever appears on its surface and is never satisfied with its continual swallowing of the ships and men in the present case the ocean had cast these three sinners on the shore without killing them

(b) Who had jointly been responsible for the conspiracy against Prospero What is Prosperorsquos purpose behind all this

Three men Alonso Sebastian and Antonio had jointly

been responsible for the conspiracy against Prospero They had driven out Prospero form Milan Prosperorsquos purpose is to make these three sinners realize the wrong they had done He wants them to repent for their criminal deeds because repentance leads to self-esteem(c )What does Ariel (the harpy) tell Alonso and his companions when they take out their swords to attack him

Seeing them drawing their swords Ariel (harpy) tells them that he and his companions are the instruments of destiny and that it is not possible for human beings to do them any injury He says that the swords of human beings can not injure even a minute part of his feathers Their swords are as ineffective against him and his companions as against the wind or the water

(d) Give the explanatory meanings of the following expressions in the context of the above extract

(i)Never surfeited (ii) Belch up (iii) lsquomongst men

(i) Never surfeited never led to satisfaction

(ii) Belch up cast ashore(iii) lsquomongst men in human

society2

I and my fellows (Line 60-65)

Are ministers of Fate The elementsOf whom your swords are tempered may as wellWound the loud winds or with bemocked-at stabsKill the still-closing waters as diminishOne dowl thats in my plume

IMPORTANT PASSAGES EXPLAINED

The elements

(Line 61-66)Of whom your swords are tempered may

as wellWound the loud winds or with

bemocked-at stabs

(a) Who is lsquoIrsquo Who are his lsquofellowsrdquo

lsquoIrsquo is referred to Ariel in disguise of a harpy His lsquofellowsrsquo are other spirits serving Prospero the real Duke of Milan who has acquired supernatural powers after being banished from his Dukedom Prospero has settled in this uninhabited island

(b) What are the elements that have temperrsquod the swords Why will it not work against the speaker

The swords (of Alonso and his companions) are tempered by metal (steel) which is taken out of the earth and refined by

Kill the still-closing waters as diminishOne dowl thats in my plume My fellow

ministersAre like invulnerable

In these words Ariel reminds the King and his companions of the utter futility of drawing swords against himself and his fellows Ariel drives Alonso Antonio and Sebastian the three men of sin to desperation ndash a state in which men do violence to themselves They draw swords to strike Ariel But Ariel reminds them that he and the other spirits are the ministers of destiny and nothing can wound them The steel of which their swords are made of may cut the wind or water which being divided always closes up again Even supposing that such things may be possible it is quite impossible that their swords will cut one feather in their plume They are incapable of being wounded by any sword of man Hence it is foolish on their part to attempt to strike at Ariel and his fellow-spirits

For which foul deed

(Line 72-75)The powers delaying not forgetting

haveIncensed the seas and shores yea all the

creatures Against your peace

Ariel enters like a harpy and remaining invisible tells Alonso Sebastian and Antonio that he and other harpies are the agents of Destiny appointed to carry out her decrees He tells them that their punishment for the crime against Prospero which has been so long deferred is now to fall upon them He reminds them that they had expelled Prospero from Milan and set him and his innocent child adrift on the sea and that the sea had paid them back for their sin by the shipwreck and by the calamities they have suffered He tells them that the powers above which did not forget this mean treachery but only deferred the punishment have now engaged the seas and the shores and all living beings including him and his comrades against them The very elements and supernatural agency Ariel adds have taken up the avenging of their crime against Prospero

the action of fire It may cut the wind or water which being divided always closes up again

The sword will not work against the spirits and the harpy because they are the ministers of destiny and nothing can wound them nor it will cut a single feather in their plume

(c )What is the meaning of lsquodowlrsquo in the last line

The term lsquodowlrsquo means a filament or the smallest part of a feather In this context Ariel in disguise of harpy says that their sword cannot even damage the smallest filament of their (Arielrsquos and other spirits) feathers as they are incapable of being wounded by any sword of man

(d) What does the speaker remind the listeners about

Ariel in disguise of harpy reminds Alonso the King of Naples Sebastian Alonsorsquos brother and Antonio the present Duke of Milan and the treacherous brother of Prospero as they being three men of sin He even reminds them that their punishment for their crime against Prospero which has been so long deferred now falls upon them He reminds them that they have expelled Prospero from Milan and has set him along with his innocent infant daughter adrift on the sea So the sea has paid them back for their sin by their shipwreck and the calamities they have suffered since then The harpy rebukes Alonso of his sin that has incensed the Gods and has deprived him of his son as a punishment

(e) How do they respond

When Ariel in disguise of a harpy reminds Alonso Sebastian and Antonio of their past misdeeds and sin Alonso has a look of terror and confusion in his eyes He utters the words of sincere repentance wrung out of his conscience-stricken heart It appears to him that all the elements of nature the sea-waves the wind and the thunder proclaiming a loud voice in the name of Prospero and the crime Alonso has committed against him They are calling upon him to repent There is a deep storm raging in Alonsorsquos breast and the echoes of that storm are ringing in his ears like a clear note of wind-instrument A note of denunciation of Alonsorsquos crime leaves him much humbled and penitent and confirms his belief that his son is lost forever But Sebastian and Antonio shows some courage instead of repentance They wish to kill the spirits or devils if it appears

3

Of my instruction hast thou nothing bated (Line 85-93)

In what thou hast to say So with good life

And observation strange my meaner ministers

Their several kinds have done My high charms work

And these mine enemies are all knit upIn their distractions They now are in my

powerAnd in these fits I leave them while I visitYoung Ferdinand whom they suppose is

drownedAnd his and mine loved darling

Methought the billows spoke and (Line 96-99)

told me of itThe winds did sing it to me and the

thunderThat deep and dreadful organ-pipe

pronouncedThe name of Prosper It did bass my

trespass

These are the words of contrition coming from Alonso Ariel has driven him to a deep repentance for conspiring with Antonio against Prospero He now feels a sincere remorse It appears to him that all the elements of nature the sea-waves the wind and the thunder proclaimed with a loud voice the name of Prospero and the crime Alonso had committed against him They are calling upon him to repent There is a deep storm raging in Alonsorsquos breast and the echoes of that storm are ringing in his ears like the clear note of a wind-instrument

Comment These are the words of sincere repentance wrung out of the conscience-stricken heart of Alonso Alonso who is the lesser villain is the first to give way to remorse under the effect of Arielrsquos speech The words of Ariel seem to him to be the voice of conscience speaking to him He is driven to desperation a state in which he might do violence to his life

(a) Identify the speaker State the context

Prospero the ruler of the island is the speaker The famous banquet scene has been enacted very well Ariel and his junior spirits have played their roles excellently Prospero is glad to say words of praise for them(b) In what way the speakerrsquos instructions have been carried out

According to Prosperorsquos instructions a banquet was presented before the King of Naples and his companions when they were tired and hungry Just when they were preparing to eat the feast the banquet was suddenly removed by exercising supernatural powers All this was done by Ariel Prosperorsquos chief assistant and a powerful spirit

Ariel not only made the feast disappear but also delivered his speech blaming the King and his two companions for their past wicked deeds He warned them to repent for their misdeeds or suffer forever on that uninhabited island

(c) Who are referred to as lsquomeaner ministersrsquo What have they done

Prospero refers as lsquomeaner ministersrsquo to his other lesser spirits who were assisting Ariel in presenting a scene before the kingrsquos party They entered the scene to the accompaniment of music They assumed several strange shapes and brought in a banquet Then they danced about it with gentle actions of salutations thus inviting the King and others to eat the feast

These spirits play their role again when Ariel in the shape of a harpy quits the scene These shapes enter again and dancing with mocking gestures carry away the table

(d) Who are the speakerrsquos enemies What has happened to them

King of Naples Alonso his brother Sebastian and the present Duke of Milan Antonio (Prosperorsquos own brother) are Prosperorsquos enemies With the turn of events they have all been washed ashore on the island which is ruled by Prospero the great magician Actually this happened after the shipwreck caused by a storm which was raised by Prospero with the purpose of bringing these people to his island Prosperorsquos spirits have already confused and terrified these enemies and they are under Prosperorsquos control He can treat them as he likes

(e) What does he say about Ferdinand Explain what is meant by ldquohellip his and mine darlingrdquo

Prospero knows that Alonsorsquos son prince Ferdinand is alive though his father thinks that the prince has been drowned

Prospero refers to his daughter Miranda who is dear to him She is also very dear to Prince Ferdinand who has fallen in love with her They are waiting to be married soon for which they have received Prosperorsquos consent

4

ALONSO O it is monstrous monstrous (Line 95-102)

Methought the billows spoke and told me of it

The winds did sing it to me and the thunderThat deep and dreadful organ-

pipe pronouncedThe name of Prosper It did bass

my trespassTherefore my son ithrsquo ooze is

bedded andIll seek him deeper than eer

plummet soundedAnd with him there lie mudded

(a) In what way does Alonso express his horror when his conscience is awakened by Arielrsquos words

When Alonsorsquos conscience is awakened by Arielrsquos words he expresses his horror at what he has heard He gets the feeling that the waves of the ocean the wind and the loud thunder have spoken to him and uttered the name of Prospero Because of being reminded of his crime in a very loud and rough voice he comes to realize that he has lost his son for his past misdeeds

(b) What does Alonso imagine about his son What does Alonso want to do in his desperate state

Alonso imagines that his son is lying in the mud at the bottom of the sea He feels desperate that he wants to drown himself in the ocean deeper than the plumb-line has ever gone He wants to lie with his son at the bottom of the sea

(c) How do Sebastian and Antonio want to face the evil spirits

Sebastian says that he is not at all afraid of what the harpy has said and that he is prepared to fight any number of such monsters if they appear before him only one at a time Antonio says that he would support Sebastian in the fight against the fiendsyyy

(d) Why does Gonzalo ask Adrian to follow the three men

Gonzalo tells Adrian that all the three men namely Alonso Sebastian and Antonio are in a wild and reckless mood The thought of the heinous crime of which they are guilty has begun to torment their minds So he asks Adrian to follow those three men without loss of time and prevent them from doing anything which the turmoil in their minds might lead them to do

(e) What opinion do you form of Alonso from the above extract

Alonso who is the lesser villain is the first to give way to remorse under the effect of Arielrsquos speech The words of Ariel seem to him to be the voice of conscience speaking to him He is driven to desperation a state in which he might do violence to his life

Subject =Accounts

Ac-12 15420 topic-pL Appropriation ac

PROFIT AND LOSS APPROPRIATION ACCOUNT

MEANING AND PREPARATIONProfit and Loss Appropriation Account is merely an extension of the Profit and Loss Account of the firm The profit of the firm has to be distributed amongst the partners in their respective profit sharing ratio But before its distribution it needs to be adjusted All Adjustments like partnerrsquos salary partnerrsquos commission interest on capital interest on drawings etc are made in this account These adjustments will reduce the amount of profit for distribution This adjusted profit will be distributed amongst the partners in their profit sharing ratio To prepare it at first the balance of Profit and Loss Account is transferred to this account The journal entries for the preparation of Profit and Loss Appropriation Account are given below

1 for transfer of the balance of Profit and Loss Account to Profit and Loss Appropriation Account

(a) In case of Net Profit

Profit and Loss Ac helliphelliphelliphelliphellipDrTo Profit and Loss Appropriation Ac(Net Profit transferred to Profit and Loss Appropriation Ac)

(b)In case of Net Loss

Profit and Loss Appropriation Achelliphelliphellip DrTo Profit and Loss Ac(Net Loss transferred to Profit and Loss Appropriation Ac)

2 for Interest on Capital

For transferring on Interest on CapitalProfit and Loss Appropriation Achelliphelliphellip DrTo Interest on Capital Ac(Interest on capital transferred to Profit amp Loss Appropriation Ac)

3 for Interest on Drawings

For transferring Interest on Drawings Interest on Drawings Achelliphelliphelliphelliphelliphellip DrTo Profit and Loss Appropriation Ac(Interest on drawing transferred to Profit amp Loss Appropriation Ac)

4 For Partnerrsquos SalaryFor transfer of partnerrsquos SalaryProfit and Loss Appropriation Achelliphellip DrTo Salary Ac(Salary transferred to profit amp Loss Appropriation Ac)

5 For Partnerrsquos CommissionFor transferring commissionProfit and Loss Appropriation Achelliphelliphellip DrTo Commission Ac(Commission transferred to Profit and Loss Appropriation Ac)

6 For Transfer of agreed amount to General ReserveProfit and Loss Appropriation Ac helliphellipDrTo General Reserve Ac(Transfer to General Reserve)

7 for share of Profit or Loss appropriation(a) If ProfitProfit and Loss Appropriation Achelliphellip DrTo Partnerrsquos CapitalCurrent Ac(Profit transferred to capitalcurrent Ac)(b) If LossPartnerrsquos Capital Current Achelliphelliphelliphellip DrTo Profit and Loss Appropriation Ac(Loss transferred to capitalcurrent Ac)

THE FORMAT OF PROFIT AND LOSS APPROPRIATION

Profit and Loss Appropriation Account for the year endedhelliphelliphelliphellip

Particulars Amount Particulars Amount

To PL Ac (loss) By pL Ac (profit)

To Interest on capital BY Interest on drawings

To partner`s commission by Partner`s capital Ac ( loss)

To Partner`s salary To Interest on partner`s loan To General Reserve To Partner`s Capital AC (Profit)

Subject= Economics

MOVEMENT ALONG THE DEMAND CURVE (CHANGE IN QUANTITY DEMANDED)In law of demand you have already studied the inverse relationship between price and quantity demanded When quantity demanded of a commodity changes due to change in its price keeping other factors constant it is called change in quantity demanded It is graphically expressed as a movement along the same demand curve There can be either a downward movement or an upward movement along the same demand curve Upward movement along the same demand curve is called contraction of demand or decrease in quantity demanded and downward movement along the same demand curve is known as expansion of demand or increase in quantity demanded

Extention of demandd

price (rs)p A

B Extentionp1 d

Q Q1

Quantity demanded ( in units)

Contraction of demandd

p2 Ccontraction

p APrice (Rs)

d

Q2 Q

Quantity demanded (in units)

Explanation of movement of demand A fall in price from OP to OP1 leads to increase in quantity demanded from OQ to OQ1 (expansion of demand) resulting in a downward movement from point A to point B along the same demand curve DD When Price rises from OP to OP2 quantity demanded falls from OQ to OQ2 (contraction of demand) leading to an upward movement from point A to point C along the same demand curve DD

  • Activity Series of Metals
    • Drawbacks of Rutherfordrsquos model of atom
      • Electromagnetic radiations
      • Properties of electromagnetic radiations
      • Characteristics of electromagnetic radiations
        • Plancks Quantum Theory-
        • Photoelectric effect
          • Intext Questions
            • Logic gates
            • Digital systems are said to be constructed by using logic gates These gates are the AND OR NOT NAND NOR EXOR and EXNOR gates The basic operations are described below with the aid of truth tables
            • AND gate
            • Example
Page 22:  · Web viewSubject. Topic. Summary. Execution. English 1 . Chapter 1 naming words . Page 8. Write the names of these pictures:- Person:-1. father. 2.Firefighter 3.doctor 4 ...

घर छोडकर जान स रोक कतिलयाइस पर बनी माधव सिसह न कहा निक बड घर की बटी ऐसी ही होती ह जो निबगडा काम बना लती ह अतः शीरषक साथक ह बड घर की बटी आन-ी ह

म कमचारी थाख) भरा हआ चहरा चौडी छाती और डटकर

खाना आदि- एक सबजी ल जवान क गण मान जात ह परत शरीकात न इनही नतर निपरय गणो को अपनी पढाई पर नयोछावर कर दि-या था

ग) बीए की निडगरी परापत कर लन पर भी उपय वयकति(शरी कठ की शारिररिरक तौर पर निनबल और चहर स कानित ही लगत थ इतना ही नही वह मानकतिसक तौर पर भी निपछड हए थ पाशचातय सामाजिजक कथा उस घणा एव पराचीन सभयता का गणगान उनकी निवचारधारा क परमख अग थ

घ) यह नतर निपरय गण गौरीपर गाव क जमी-ार क छोट बट लाल निबहारी सिसह म निवदयमान थ वह सजीलाजवान था और भस का दध शर दध वह सवर उठकर पी जाता था

ldquoयही कारण था निक गाव की लललन आए उनकी निन-क थी कोई कोई तो उह अपना शतर समझन म भी सकोच ना करती थी सवय उनकी पतनी को इस निवरषय म उनस निवरोध थाldquo

क) उपय पकति म इस वयकति क बार म कहा गया ह

ख) गाव की लललन आए उनकी निन-ा कयो निकया करती थी

ग) उनकी पतनी का कया नाम था उनह निकस निवरषय म अपन पनित क निवरa था और कयो

घ) इस कहानी का कया उददशय ह Continue to next helliphelliphellip

Bengali 2nd Language

ফ ফটক না ফটক( কহিতা )

পর) ldquo(ান াধাচেনা ফটপাচেথ পাথচের পাড হিচেয় এক কাঠচোটটা গাছ কহিকহি পাতায় পার ফাটিচেয় াসচেছldquoক) কার দো দেকান কহিতার অং( ) lsquo(ান াধাচেনা ফটপাচেথ পাথচের পাডহিচেয়lsquo চেত কী দোঝাচেনা চেয়চেছ গ) আচো য অংচে(lsquo এক কাঠচোটটা গাছ lsquoচেত কী দোঝাচেনা চেয়চেছ ঘ) ldquoকহিকহি পাতায় পার ফাটিচেয় াসচেছldquo ----- একথার পরকত অথC কী উততর ) ক) আচো য অং(টি পর যাত কহি সভা4 মচোপাধ যাচেয়র দো lsquoফ ফটক না ফটকrsquo কহিতার অং()কহি সভা4 মচোপাধ যায় হিছচেন দেপরচেমর কহি দেপরমচেক নানা ভহিঙগমায় হিতহিন ফটিচেয় তচেচেছন দেপরম মানচের স মচেতC র সঙগী কহিতার কহিতায় এক রb সb হচেয়র দেপরম াগরচেনর কথা চেচেছন (ান অথCাৎ দেযাচেন দেকান রস দেনই দেযাচেন দেকান মহিনতা দেনই অথ তার মধ দেযও দেপরম থাকচেত পাচের একথাই কহি তচে ধরচেত দেচেয়চেছন একটি মানচে4র মচেন দেযাচেন দেকামতার দেকান সথান দেনই পাথচেরর মচেতা হিনরসতার মচেনর মধ দেযও দেয দেপরম আসচেত পাচের দেস কথাই কহি চেচেছনগ)নারীচের যথC দেপরচেমর ছহি এই কহিতায় অকপচেট উচেঠ এচেসচেছ কহি এই কহিতায় কাটচোটটা গাছ কথাটি যার কচেরচেছন নারী দেয দেপরম দেথচেক হিতাহিত এং দেসই দেপরম সঠিক সমচেয় না পাওয়ার ন য দেপরম সমপচেকC হিচেr4 গৈতরী য় দেপরচেমর দেয গৈহি(ষট য মাধযC য সরসতা দেকামত এই সমসতর হিপরীত যথা রbতা শষকতা কচেঠার তা পরভহিত দোঝাচেত এক কাঠচোটটা গাছ কথাটি যার কচেরচেছনঘ) এাচেন এক নারীর যথC দেপরচেমর কথা হিনহিCপত ভাচে চেচেছন কহি অসমচেয় নারীর ীচেন দেপরম দেচেগচেছ এতহিন তার হয় রb কচেঠার হিছ দেপরচেমর অভাচে ঠাৎ দেসই শষক মরভহিমচেত সচের আভাস এচেসচেছ দেপরম দেযন 4Cার স(ীত তাই পরায় মত গাচেছ কহিকহি পাতা গহিচেয় উচেঠচেছ

Biology Chapter - 01Controlling Air Pollution

Today we will discuss how we control air pollution from domestic combustion

Q1Describe any five ways of reducing air pollution from domestic sources bull The number of pollutants in the air is verylarge and we always try to control them byfollowing ways

i) Solar cooker and solar heater It use no fuel reduce damage of environment by fuel use or reducing deforestation It maintains coolness of house It releases very less orno oil gas or grease

ii) Piped natural gas (PNG) It emits very less by products into the atmosphere As it isdistributed through pipe lines so there iscontinuous supply of fuel is possible

iii) Liquefied Petroleum Gas (LPG) It hasa higher heating value LPG doesntcontain sulphur so it burns a lot cleanerenergy sources It releases very less oralmost no fume in air

iv) Electricity based cooking Emission free cooking alternative for urban dwellers causeselimination of adverse health impactsofindoor air pollution It helps to avoid theinconveniences associated with procurement of LPG

v) Biogas It contains 75 methane whichmakes it an excellent fuel It burns without smoke and biogas plant leaves no residue like ash in wood charcoal etc Thus it isaclean fuel

Economics

Factors of Production

Today firstly we would recall the last class for 5 mins and then we would proceed with the further topics of the chapter

The concept meaning of land characteristics of land and importance of land to be repeated for the absentees as well as the students who were there in the class the previous day

Today we will start with the last portion of land before it the meaning of land to be repeated onceAs by now we all know that

Questions1What do you mean by productivity of landAnswer By productivity of land we mean the capacity of a piece of land to produce a crop

Thus it refers to the average output per unit of landSay per acre per hectare etc= (OutputArea of land)

2 What are the factors influencing the productivity of landAnswer

Natural factors Productivity of land is largely determined by the natural

Land is defined to include not only the surface of the earth but also all other free gifts of nature(for example mineral resources forest resources and indeed anything that helps us to carry out the production of goods and services but is provided by nature free of cost)

We will move on to the last portion of land by discussing Productivity of Land

By productivity of land we mean the capacity of a piece of land to produce a crop

Thus it refers to the average output per unit of land

Say per acre per hectare etc= (OutputArea of land)

With this we shall proceed further with the main factors that determine the productivity of land

Natural factors Human factors Improvements on land Location of land Organisation Ownership of land Availability of capital Proper use of land State help

Note economic development of a country depends upon the quality of its land If the land is fertile it will quicken the pace of development of the country

qualities of land such as fertility etc

Human factors Land cannot produce anything by itself Man has to apply labour on it to produce for himself So productivity of land depends on the knowledge and skills of workers

Improvements on land production of land is affected by land development measures like provision of well or tubewell irrigation proper drainage

State help The government of a country especially less developed country can play a vital role in improving the agricultural productivity by providing better irrigation facilities

Organisation Productivity of land also fdepends upon the way how the factors of production like labour and capital are organised

In order to increase productivity trained workers modern implements scientific methods good seeds are all essential

3 lsquoImproved technology affects the productivity of landrsquo Explain this statement with the help of suitable example Answer Use of improved technology raises the productivity of land Example By using HYV seeds chemical manures and modern machines per hectare output increases

Physics Force (Summary)

Question Write the expression for the moment of force about a given axisSolutionsThe expression for the moment of force is given byMoment of force about a given axis = Force times perpendicular distance of force from the axis of rotationQuestion What do you understand by the clockwise and anticlockwise moment of force When is it taken positiveSolutionsIf the effect on the body is to turn it anticlockwise moment of force is called the anticlockwise moment and it is taken as positive while if the effect on the

body is to turn it clockwise moment of force is called the clockwise moment and it is taken as negative

Math Topic Commercial Mathematics

Chapter Goods and services Tax

Study item Some solved sums from exercise ndash 1 A retailer buys a TV from a wholesaler for Rs 40000 He marks the price of the TV 15 above his cost price sells it to the consumer at 5 discount on the marked price If the sales are intra ndash state and the rate of GST is 12 find

(i) The marked price of the TV(ii) The amount which the consumer pays for the TV(iii) The amount of tax (under GST) paid by the retailer to the central

Government(iv) The amount of tax (under GST) received by the State Government

Solution As the sales are intra- state sale and the rate of GST 12 So GST comprises of 6 CGST and 6 SGSTTherefore a retailer buys a TV from a wholesaler for Rs 40000Therefore the amount of GST collected wholesaler from the retailer or paid by retailer to wholesalerCGST = 6 of Rs 40000 = Rs(6100 times40000) =Rs 2400SGST = 6 of Rs 40000 = Rs (6100 times 40000) =Rs 2400Therefore wholesaler will pay Rs 2400 as CGST and Rs 2400 as SGSTTherefore amount of input GST of retailer Input CGST = Rs 2400 and input SGST = Rs 2400Again the retailer marks the price of the TV 15 above his cost price(i) The marked price of the TV

= Rs 40000 + Rs 40000times15= Rs 40000 + Rs 40000times 15100= Rs 40000 + Rs 6000Rs 46000But the retailer sells it to consumer at 5 discount on the marked priceCost price after discount = Rs 46000 ndashRs46000times 5100 =Rs 46000 ndashRs 2300= Rs 43700Therefore the amount of GST collected retailer from consumer or paid by consumer to retailerCGST = 6 of Rs 43700 =Rs ( 6100 times43700)Rs 2622SGST = 6 of Rs 43700 = Rs (6100 times 43700) =Rs 2622Amount of the output GST of retailer Output CGST = Rs 2622 and output SGST = Rs 2622

(ii) The amount which the consumer pays for the TV= cost price of TV to consumer + CGST paid by consumer + SGST paid by consumer= Rs 43700 + Rs 2622 + Rs 2622= Rs 48944

(iii) The amount of tax (under GST ) paid by the retailer to the central Government=CGST paid by retailer = output CGST ndash input CGST=Rs 2622 ndash Rs 2400=Rs 222

(iv) The amount of tax ( under GST ) received by the State Government = SGST paid by wholesaler + SGST paid by retailer= Rs 2400 + output SGST ndash input SGST=Rs 2400 + Rs 2622 ndash Rs 2400=Rs 2400 + Rs 222= Rs 2622

Commercial studies

Stakeholders Today I am going to give some revision questions from the previous study material

Questions1) State the two expectations of

employees from a business concern2) Give two distinctions between

stakeholder and shareholder3) Give two difference between

internal stakeholders and external stakeholders

4) Give two expectations of suppliers from a business organisation

5) Who is a stakeholder in commercial organisations

Chemistry Periodic Table

Merits of Mendeleevrsquos Periodic law are as follows - 1He grouped the elements on the basis of atomic mass 2 He left gaps for undiscovered elements like Gallium Scandium germanium Also he left a full group vacant for undiscovered inert gases 3 He could predict proportions of several elements on basis of their position in periodic table like Ga Sc etc 4He could predict errors in atomic weights of some elements like gold platinum etc

Anomalies in Mendeleevrsquos Periodic law are as follows - 1 Position of isotopes could not be explained 2 Wrong order of atomic masses could not be explained

For example- as Arnur atomic mass 40 come first and K with low atomic mass (30) should come later but k should be placed first

According to Bohrrsquos Modern Periodic table properties of elements are periodic functions of their atomic numbers

So when elements are arranged according to increasing atomic numbers there is periodicity in electronic configuration that leads to periodicity in their chemical properties

It consists of horizontal rows (Periods) Vertical column (Groups)

There are 7 period and 12 groups in this long form of periodic table

Ist period has 2 elements IInd period has 8 elements IIIrd period has 8 elements IVth period has 18 elements Vth period has 18 elements VIth period has 32 elements VIIth period hs rest of elements

Note - The number of valence electrons in atom of elements decides which elements will be first in period and which will be last

In group- 1 to 2 gp and 13 to 17 contain normal elements 3 to 12gp ndash transition elements 57 to 71 - lanthanides 89 to 103 - Actinides

Left hand side ndash metals Right hand side ndash nonmetals

Note- Hydrogen element has been placed at top of Ist group Electronic configuration of H is similar to alkali metal as both have 1 valence electron

V electron of gp I element -- 1 V electron of gp 2 element -- 2 V electron of gp 13 element -- 3 V electron of gp 14 element -- 4 V electron of gp 15 element -- 5 V electron of gp 16 element --6 V electron of gp 17 element -- 7 V electron of gp 18 element -- 8

English 1 Transformation of sentences

Sentences A sentence is a group of words which makes complete sense

Exercise 2Change the following sentences from

a Assertive sentencesb Imperative sentencesc Interrogative sentencesd Exclamatory sentences

Sentences can be changed from one grammatical form to another without changing the meaning of the sentence This is known as transformation of sentences

assertive to interrogative1 Nobody would like to be a fool

Who would like to be a fool2 Their glory can never fade

When can the glory fade3 Nobody can control the wind

Who can control the wind4 It matters little if I die

What though I die5 No man can serve two masters

Can any man serve two masters

Exercise 3Interchange of assertive and Exclamatory sentences

1 She leads the most unhappy lifeWhat an unhappy life she leads

2 This is indeed an interesting bookWhat an interesting book this

3 He is a very great manWhat a great man he is

4 It is a very lame excuseWhat a lame excuse

5 It is sad that she died so youngAlas she died so young

Class XISubject Topic Summary Execution

Hindi 2nd lang

पतर परम(परमचदर) पतर परम कहानी म एक निपता की इचछाओ का वणन निकया गया ह अपन बड पतर परभ -ास स निपता चतनय -ास का निवशरष परम था निपता को उसक जनम स ही बडी-बडी आशाए थी उसम दसर बट कतिशव-ास की अपकषा स- उतसाह की मातरा अमिधक थी वह उस इगलड भजकर बरिरसटर बनाना चाहत थभागय का खल भी बडा निनराला ह बीए की परीकषा क बा- वह बीमार पड गया डॉकटरो न भी जवाब - दि-या थाचतन -ास जी बहत ही कजस थ बवजह पस खच करना नही चाहत थ अगर गारटी मिमलती तो शाय- पस खच भी कर -त परत गारटी नही थी परिरणाम सवरप उनक बट का -हात हो गयाजब बट को समशान ल जा रह थ तो वहा काफी शोर गान बजान हो रह थ पछन पर पता चला निक निकसी निपता निपछल तीन साल स निबमार था और उसक ईलाज म रपया पानी की तरह बहाया पर ठीक नही हए परत उसक बट को तनिनक भी अफसोस नही था उसका कहना था उसन कोकतिशश तो कीयह -खकर चतनय-ास जी को आतम निगलानी हईतभी स उनका म परिरवतन हआ और बट का भोज काफी धमधाम स निकयाऔर वहइस पशचाताप की आग म जलत रह औला- स बढकर पसा नही होता ह इस बात को समझन म उनह काफी व लग गया

hellipContinue to next

BENGALI(2ND LANGUAGE)

পরথমঅধযায়-ঠাকরারীনদরনাথঠাকর

নয়ন দোচের হিমাচেররা া নাচেমই হিযাত হিছচেন ায়ানার উাররণ সবরপ নয়ন দোচের ারা হিা (াচেকর হিা হিচেতন এছাাও দেকান উৎস উপচেb রাহিতর দেক হিন করার উচেfচে(য তারা সযC হিকরচেরণ রনয পরীপ জবাহিচেয় তাচেত রপার হির 4Cরণ করচেতন ঠাকরা এই নয়ন দো হিমারচের দে(4 ং(ধর হিছচেন হিমাররা ায়ানার ষটানত পর(Cন কচের তারা হিনঃসব এই হিমাহিরর দে(4 ং(ধর গৈকাস নদর রায়চেৌধরী গৈকাস া নয়ন দোচের সমসত সমপহিতত ঋচেরণর াচেয় হিহিx কচের অহি(ষট যা আচেছ তাচেত হিপত

ইার হিপতার মতয ইচে পর নয়নচোচের ায়ানার দেগাটা কতক অসাধাররণ শরাদধ (াহিনতচেত অহিনতম ীহিপত পরকা( কহিরয়া ঠাৎ হিনহিয়া দেগ- ক) কার দো দেকান গচেলপর অং() কতা দেক ইার চেত কাচেক দোঝাচেনা চেয়চেছ গ) পরসঙগ কী কতার কতয পহিরসফট কচেরা

পরচে4র যাহিত রbা করা সমভ নয় তাই হিতহিন পতরচেক হিনচেয় ককাতায় সাস শর কচেরন গলপ কথচেকর আহিথCক অসথা নয়ন দোচের হিমাচের দেথচেক সমপরণC আাা কথচেকর হিপতা হিনচের দেষটায় অথC উপাCন করচেতন া উপাহিধ াচেভর নয তার াসা হিছনা আর দেসই কারচেরণ কথক তার একমাতর উততরাহিধকার চেয় তার হিপতার পরহিত কতজঞ কথক দো পা হি(চেচেছন হিনচের পরারণ ও মান রbার নয উপচেযাগী অথC হিনা দেষটায় পরাপত চেয়চেছন- এটাই তার কাচেছ পরম দেগৌরচের হি4য় চে মচেন কচেরন কাররণ (নয ভাণডাচের গৈপতক ায়ানার উজজব ইহিতাস অচেপbা দোার হিসeচেকর মচেধয গৈপতক দেকামপাহিনর কাগ তার কাচেছ অচেনক দেহি( মযান

TO BE CONTINUED

উ- ক) আচোয অং(টি রীনদরনাথ ঠাকচেরর দো ঠাকরা গচেলপর অং() কতা চেন আচোয গচেলপর গলপ কথকইার চেত নয়ন দোচের হিমাহিরর দে(4 ং(ধর গৈকাস ার কথা া চেয়চেছ গৈকাস া নয়ন দোচের সমসত সমপহিতত ঋচেরণর াচেয় হিহিx কচের অহি(ষট যা আচেছ তাচেত হিপত পরচে4র যাহিত রbা করা সমভ নয় তাই হিতহিন পতরচেক হিনচেয় ককাতায় সাস শর কচেরনগ) গৈকাস ার হিপতার মতযর পর নয়ন দোচের হিমাহিরর অহিসততব হিপত য় কচেয়কটা উৎস ও শরাদধ- (াহিনতচেত হিমাহিরর দে(4 কহিটক যয় চেয় হিগচেয় এচেক াচের দে(4 চেয় যায় তন তাচের গC করার মত আর হিকছই হিছ না-দেসই পরসচেঙগ এই উহিকত নয়নচোচের হিমাচেররা া নাচেমই হিযাত হিছচেন ায়ানার উাররণ সবরপ নয়নচোচের ারা হিা (াচেকর হিা হিচেতন এছাাও দেকান উৎস উপচেb রাহিতরচেক হিন করচেত হিগচেয় তারা সযC হিকরচেরণর নয পরীপ জবাহিচেয় তাচেত রপার হির 4Cরণ করচেতন তাই দেসকাচের ায়ানা দেহি(হিন সথায়ী চেত পারত না হিহিভনন উৎস শরাদধ- (াহিনতচেত সাধযা হিতহিরকত র করার নয হিমাহির হিহিকচেয় দেযত হ হিতC কা হিহি(ষট পরীচেপর দেত দেযমন অলপকাচের মচেধয হিনঃচে(4 চেয় যায়-নয়নচোচের হিমারচের অসথা তাই চেয়হিছ এই কারচেরণই কথক নয়নচোচের হিমারচের গা ভরা আমবর সয করচেত পারতনা

Physics Dimensional Analysis (Summary)

Q Find the dimensions of consts ab in relation

p=(bminusxlowastx)at

where p is the power x is the distance and t is time

Ans From principle of homogeneity dimension of b x2 are same Dim of b = dim of x2 = [L2] = [ML2T0]Dim of a = dim of ( b- x2)dim of (pt) = [M0L2T0][ML2T-2] [T-1] [T] = [M-1L0T2]

Chemistry Atomic Structure Drawbacks of Rutherfordrsquos model of

atom a According to Rutherfordrsquos model of atom electrons which are negativelycharged particles revolve around the nucleus in fixed orbits Thusb theelectrons undergo acceleration According to electromagnetic theory of Maxwell a charged particle undergoing acceleration should emitelectromagnetic radiation Thus an electron in an orbit should emitradiation Thus the orbit should shrink But this does not happenc The model does not give any information about how electrons aredistributed around nucleus and what are energies of these electrons Isotopes These are the atoms of the same

Properties of electromagnetic radiationsa Oscillating electric and magnetic field are produced by oscillating charged particles These fields are perpendicular to each other and both areperpendicular to the direction of propagation of the waveb They do not need a medium to travel That means they can even travel invacuum

Characteristics of electromagnetic radiationsa Wavelength It may be defined as the distance between two neighbouring crests or troughs of

element having the same atomicnumber but different mass numbere g 1H11H21H3

Isobars Isobars are the atoms of different elements having the same massnumber but different atomic numbere g 18Ar40 20Ca40

Isoelectronic species These are those species which have the same numberof electrons

Electromagnetic radiationsThe radiations which are associated withelectrical and magnetic fields are called electromagnetic radiations When anelectrically charged particle moves under acceleration alternating electricaland magnetic fields are produced and transmitted These fields aretransmitted in the form of waves These waves are called electromagneticwaves or electromagnetic radiations

wave as shown It is denoted by λb Frequency (ν) It may be defined as the number of waves which passthrough a particular point in one secondc Velocity (v) It is defined as the distance travelled by a wave in onesecond In vacuum all types of electromagnetic radiations travel with thesame velocity Its value is 3 times10 8m sec-1 It is denoted by v

d Wave number Wave number is defined as the number of wavelengths per unit lengthVelocity = frequency timeswavelength c = νλ

Plancks Quantum Theory- o The radiant energy is emitted or absorbed not continuously but discontinuously in the form of small discrete packets of energy called lsquoquantumrsquo In case of light the quantum of energy is called a lsquophotonrsquoo The energy of each quantum is directly proportional to the frequency of the radiation ie E α υ or E= hυ where h= Planckrsquos constant = 6626 x 10-27 Js o Energy is always emitted or absorbed as integral multiple of this uantum E=nhυ Where n=1234Black body An ideal body which emits and absorbs all frequencies is calleda black body The radiation emitted by such a body is called black body radiation

Photoelectric effectThe phenomenon of ejection of electrons from thesurface of metal when light of suitable frequency strikes it is calledphotoelectric effect The ejected electrons are called photoelectrons

Biology Chapter - 02Systematics and Five Kingdoms

Scientists divide the whole living organisms into two kingdom first and ultimately by five kingdom at last

In the earlier systems of classifications organisms are divided into kingdom plantaeand kingdom animalia on the of presenceof cell wall their modes of nutrition and movements

Some problem arise like fungi share manycharacteristic withplant despite their heterotrophic nutrition bacteria protozoa areunicellular present in both kingdom Toovercome this third kingdom Protista isintroduced which include

unicellularorganisms But there is also another

problem Allunicellular organisms are not similar kind The cellular structure of prokaryotes is verydifferent from that of other organismsEukaryotes possess a true nucleus and allcell organelles that are not present inprokaryotes So the fourth kingdom Monerais introduced which include unicellular prokaryotes (bacteriaamp blue green algae)

bull Still some problem arise in kingdomplantae

So in 1969 R H Whittakar proposedanew five kingdom System of classification

i) Kingdom Monera - unicellular prokaryotes

ii) kingdom Protista - unicellular eukaryotes

iii) Kingdom Fungi - uni or multicellular fungi with cell wall but without chlorophyll

iv) Kingdom Plantae - Multicellular Plants

v) Kingdom Animalia - Multicellular Animals

EVS Chapter 1 ndash Modes of Existence

An agricultural society

An agricultural society also known as an agrarian society is a society that constructs social order around a reliance upon farming More than half the people living in that society make their living by farming

People in an agricultural society generally lead a more settled lifestyle than those in nomadic hunter-gatherer or semi-nomadic pastoral societies because they live permanently near the land that is farmed Agricultural settlements tend to develop in areas of convenience near bodies of water which is used for both crops and transportation or along trade routes Not everyone in an agricultural society is a farmer Some people make a living trading or making and selling goods such as tools used for farming

Another way to define an agrarian society is to see the total amount of production in a nation In an agrarian society cultivating the land is the main source of wealth Such a society can recognize other means of subsistence and work habits but emphasizes the importance of agriculture and livestock Agrarian societies have existed in various parts of the world for 10000 years and continue to exist today They have been the most common form of socio-economic organization for most of recorded human history

Q) Write the features of agricultural society

Ans - Structure and Features of Agrarian Society1 Occupational Structure

An agrarian society is generally associated with the domestication of plants and animals The domestication of plants means farming and that of animals means herding Often there is mixture of farming and the use of such domesticated animals as cow goat and sheep

2 Forms of Land Ownership in Agrarian SocietiesGenerally there are landlords supervisory farmers cultivators and share croppers The landholders own the land but do not work on it They let it out for sharecropping The supervisory farmers are those who live by having their land cultivated by hired labourers The cultivators cultivate the land for themselvesThe share-croppers are those who live by tilling other peoplersquos land or a crop-sharing basis The artisans own their means of production and produce by their own labour in their homesteads

3 Village Community System An agrarian society is highlighted by

the institution of village community system The agrarian economy made fixed dwelling houses necessary Living close together for protection and co-operation and living nearer to the land gave birth to agricultural villages The village is not only the residential place of farmers it is also the social integrator

4 Minimal Division of Labour Another structural feature of agrarian society is a minimal division of labour Except for the basic division founded on age and sex differences there are few specialized roles There is only one predominant type of occupation ie domestication of plants and animals For all the people the environment physical as well as social is the same

5 Role of Family The farm family is of the patriarchal type the father is the final arbiter in most of the familyrsquos major decisions The life of ail men and women is merged in family life Since there are not many special organizations family is the only organisation to perform the tasks of aid and protection

6 Sense of Unity The members of an agrarian society exhibit a strong in-group feeling Since the whole of their social lives is wrapped up in a society which is physically economically and socially homogenous they are inclined to view the entire outside world as an out group

7 Informal Social Control An agrarian society is regionally divided into villages In a village community the force of traditional mores is more dominant than in the urban community In the village everybody is known to everybody The members in a village community help each other and share the joy and sorrows of each other Crime in an agrarian society is rare

8 Simplicity and Uniformity Life of the people in an agrarian society is marked by simplicity and uniformity Their main occupation is agriculture which largely depends upon the vagaries of nature An agrarian society is a religious society

Math Compound angles Compound angles The algebraic sum of two or more angles is called a compound angle If A B C be three angles then A+B B+C C+A A-B B-C A-C A+B-C etc are compound angles In this chapter we shall discuss the trigonometrical ratios of compound angles Theorem 1 If A B and A+B are all pisitive acute angles theni) sin( A+B) = sin A cos B + cosA sinBii) cos(A+B) = cosA cosB- sinA sinBTheorem 2If A and B are positive acute angles and AgtB theni) sin(A-B) = sin A cosB- cos A sinBii) cos(A-B) = cos A cos B+ sin A sin BTo prove that i) sin(A+B) sin (A-B) = sin2 A - sin2 B = cos2 B- cos2 A

Example 1 Prove that tan70deg=2tan50deg+tan20degSolutiontan70deg = tan(50deg + 20deg)Or tan70deg=(tan 50deg+tan 20deg)(1-tan50degtan20deg) or tan70deg (1 ndash tan 50deg tan20deg) = tan50deg+tan20degor tan70deg= tan70deg tan50deg tan20deg+ tan50deg + tan20deg = cot20deg tan50deg tan20deg + tan50deg + tan20deg = 2 tan50deg+ tan20degExample 2 If A + B = 45deg show that (1 + tanA) (1 + tanB) = 2Solutiontan(A + B) =( tan A + tan B) (1 - tan

ii) cos(A+B) Cos(A-B) = cos2 A- sin2 B = cos2 B -sin2 AProof i) LHS= sin(A+B)sin(AminusB) [Recall sin(αminusβ)=sinαcosβminuscosαsinβ And sin(α+β)=sinαcosβ+cosαsinβ]= (sinAcosB+cosAsinB)times(sinAcosBminuscosAsinB)= sin2Acos2Bminuscos2Asin2B [Recall sin2α+cos2α=1 From above we can then assume correctly that sin2α=1minuscos2α AND cos2α=1minussin2α] = sin2A(1minussin2B)minussin2B(1minussin2A) = sin2Aminussin2Asin2Bminussin2B+sin2Asin2B = sin2Aminussin2B= 1-cos2A-(1-cos2B) = cos2 B- cos2 A = RHSii)LHS= cos (A+B) cos(A-B) [ cos(A+B) = cos AcosB- sinAsinBCos(A-B) = cosAcosB+ sinAsinB]= cos2 A Cos2 B- sin2 A Sin2 B= cos2 A( 1-sin2 B) - (1- cos2 A) sin2 B= cos2 A- cos2 A sin2 B- sin2 B+ cos2 A sin2 B=cos2 A- sin2 B=1- sin2 A-(1-cos2 B) = cos2 B- sin2 A= RHSTangent formulae for compound anglesi)tan (A + B) = tan A + tan B1-tan A tan Bii)tan (A ndash B) = tan A-tan B1+tan A tan Biii) cot (A + B) = cot Acot B-1cot A+cot B(viii) cot (A ndash B) = cot Acot B+1cot B-cot A

A tan B) Or 1= (tan A+ tanB) (1-tan A tanB) Or tanA + tanB + tanA tanB + 1 = 1 + 1Or tanA (1 + tanB) + (1 + tanB) = 2Or (1 + tanA) (1 + tanB) = 2Example 3 Find the value of sin 15degSolution sin 15deg= sin(45deg-30deg) = sin45degcos 30deg- cos45degsin30deg =(1radic2) (radic32) -(1radic2) (12) = (radic3-1) 2radic2Example 4 If sin A = 1 radic10 and sin B = 1 radic5 where A and B are positive acute angles then what is A + B SolutionWe know that sin (A + B) = sin A cos B + cos A sin B= [1 radic10] [radic(1 minus 1 5)] + [1 radic5] radic(1 minus 1 10)= [1 radic10] [radic4 5] + [1 radic5] [radic9 10]= [1 radic50] times (2 + 3)= 5 radic50 = 1 radic2

sin (A + B) = sin π 4rArrHence A + B = π 4Example 5 If A + B = 225o then find [cot A] [1 + cotA] times [cot B] [1 + cot B]Solution[cot A] [1 + cotA] times [cot B] [1 + cot B] = 1 [(1 + tan A) times (1 + tan B)]=1 [tan A + tan B + 1 + tan A tan B] [ tan (A + B) = tan225o]∵

tan A + tan B = 1minus tan A tan BrArr= 1 [1 minus tan A tan B + 1 + tan A tan B]= 1 2

COMMERCE

CLASSIFICTION OF HUMAN ACTIVITIES-ECONOMIC AND NON-ECONOMIC

Firstly we shall recall the previous class for 5 mins especially for the absentees and for also the rest of the students who were there

Today at first we briefly discuss the earlier portions of the chapter

1Business-It includes all those economic activities which are concerned with production and exchange of goods and services with the object of earning profit Example A factory shop beauty parlour also business enterprises

2Profession ndashThe term profession means an occupation which involves application of specialized knowledge and skills to earn a living For Example Chartered Accountancy medicine law tax consultancy are example of professions

Questions1What are the main features of ProfessionAnswer The main features of a profession are as follows a Specialised body of knowledge-Every profession has a specialised and systematised body of knowledge b Restricted entry- Entry to a profession is allowed only to those who have completed the prescribed education and have the specialised examination c Formal education and training ndashA formal education and training is given to the person who wants to acquire the professional

3Employment-Employment mean an economic activity where people work for others in exchange for some remuneration (salary)The persons who work for others are called lsquoemployeesrsquo The persons or organizations which engage others to work for them are called lsquoemployersrsquoEg A doctor working in a hospital is employment as he is working for a salaryA lawyer may serve as a law officer in a bank

With this we shall proceed with the features of both Profession amp Employment

The main features of a profession are as follow

a Specialised body of knowledge b Restricted entry c Formal education and training d Professional association e Service motive f Code of contact

The main features of an employment are as follows

a In employment a person works for others called employer

b An employee provides personal service

c There is a service agreement or contract between the employee and the employer

d The employee has to obey the order of the employer

e No capital investment is made by the employer

Various examples of Employment are as follows

aA teacher teaching in a school or collegeb An engineer employed in Municipal Corporation of DelhicAn accountant working in the accounts department of a companydA doctor working in a hospital

Note In all the above examples of employment the individual who is involved in each example is working as an employee for a salary under an employer

qualification(MBBSCALLB)d Service motive ndashProfessionals are expected to emphasis service more on their clients rather than economic gain f Code of Conduct-The activities of professionals are regulated by a code of conduct

2 What are the main features of EmploymentAnswer The main features of an employment are as followsa In employment a person works for others called employerb An employee provides personal servicec There is a service agreement or contract between the employee and the employerd The employee has to obey the order of the employere No capital investment is made by the employer

3 Give various Professions and their respective Association are given below

Professions

Professional

Professional association

Medical profession

Doctor Medical Council of India

Law profession

Lawyers Bar Council of India

Accounting Profession

Chartered

The Institute of Chartered Accounts of India( ICAI)

Engineerin Engineers The

g Profession

institute of Engineers (India)

Accounts Basic accounting terms

Today we will give you some questions from the previous study material

Questions6) Define accounting7) What do you mean by debit

and credit8) Explain the types of account9) Define the following terms

a) Assetsb) Capitalc) Purchased) Debtorse) Transactions

10) Name the types of accounts given below

a) Krishnas accountb) Machinery accountc) Royalty accountd) Salary accounte) Furniture accountf) Audit fee account

Economics Basic Economic ConceptsSub topic

UTILITY

Before starting todayrsquos class we shall recall the last class which was about UTILITY AND THE FEATURES OF UTILITY

Now we shall proceed with the further topics of the chapter

Todayrsquos topic from the chapter lsquo Basic Economic Conceptsrsquo will be TOTAL UTILITY amp MARGINAL UTILITYNow let us quickly revise the concept of utility with an example ie goods and services are designed because they have an ability to satisfy human wantsThis feature of being able to satisfy human wants is termed as utility For example we derive utility from WiFi services as it gives us satisfaction by connecting us to our friends and family through social media here consumers derive utility from WiFi services

From the above concept we shall start with todayrsquos topicEconomists have defined TOTAL UTILITY (TU) as the total satisfaction obtained by consuming a given total amount of a good and serviceFor example the total satisfaction obtained from eating 10 mangoes is the total utility of 10 mangoes

MARGINAL UTILITY (MU) is the additional satisfaction derived from each additional unit

Questions1 What is Total Utility (TU)

Answer Total Utility (TU) is the

aggregate of the utility that a consumer derives from the consumption of a certain amount of a commodityTU=MU1+MU2++MUn

2 What is Marginal UtilityAnswer

Marginal Utility (MU) is the additional made to the total utility as consumption is increased by one more unit of the commodityMU= TUn ndashTUn-1

NoteOften economists tend to

subdivide utility into an imaginary unit called UTIL

consumed In this casethe utility obtained from each mango as it is consumed as the MU of that mango It is also defined as the addition made to the total utility when an additional unit is consumed Often economists tend to subdivide utility into an imaginary unit called UTIL

Note As a consumer increases the consumption of a good over period of time the total utility or total satisfaction derived from it increases to appoint and thereafter it decreasesHowever as the consumer keeps on consuming the good the marginal utility or the additional utility derived from it decreases

SubjectBusiness studies

Topic

BUSINESSENVIRONMENT

Summary

Now quickly let us revise the earlier points that we have already done in the last class and let us proceed with the other topics that are there in the chapter

Firstly we will recall the internal and external factors of micro environment and then we shall proceed in details

Meaning and list of internal and external factors

aInternal factorsInternal factors refer to all the factors existing within a business firm The internal factors are considered controllable because the enterprise has control over these factorsFor an example a company can alter its organization structure policies programmes employees physical facilities and marketing mix to suit the changes in the environmentList of internal factors areCorporate culture mission and objectives top management organizations structure company image and brand equity company resources

b External factorsExternal factors refer to those individual and groups and agencies with which a particular business organization comes into direct and frequent contact in the course of its functioningThese individuals and groups are known as STAKEHOLDERS because they have a stake (financial interest ) in the working and performance of the particular business List of external forces (stakeholders)Customers competitors investors suppliersmiddlemen (marketing intermediaries)

Execution 1 What do you mean by internal

factors in micro environmentAnswerInternal factors refer to all the factors existing within a business firm The internal factors are considered controllable because the enterprise has control over these factorsFor an example a company can alter its organization structure policies programmes employees physical facilities and marketing mix to suit the changes in the environment

2 What do you mean by external factors in micro environment

AnswerExternal factors refer to those individual and groups and agencies with which a particular business organization comes into direct and frequent contact in the course of its functioningThese individuals and groups are known as STAKEHOLDERS because they have a stake (financial interest) in the working and performance of the particular business

3Who are stakeholdersSTAKEHOLDERS are individuals and groups who have a stake (financial interest ) in the working and performance of the particular business 4Discuss the internal factors in briefa Corporate CultureThe values beliefs and attitudes of the founders and top management of the company exercise

financers publics

customers

suppliersfinancers

competitors

middlemen

publics

Fig STAKEHOLDERS OF A COMPANY

Apart from micro environment the other main dimension of business environment isMacro environment Macro environment refers to the general environment or remote environment within which a business firm and forces in its micro environment operateA company does not directly or regularly interact with the micro environmentTherefore macro environment is also known as indirect action EnvironmentThe macro environment forces are less controllable than the micro forces

Macro environment consists of the following components

POLITICAL AND LEGAL ENVIRONMENT

ECONOMIC SOCIAL AND ENVIRONMENT

CULTURAL

ENVIRONMENT

TECHNOLOGICAL ENVIRONMENT

a strong influence on what the cmpaany stands for how it does things and what it considers importantbMission and objectivesThe business philosophy and purpose of a comoany guide it prioritiesbusiness strategiesproduct market scope and development scope

cTop management structurethe composition of board of directors the degree of professionalization of management and the organizational structure of a company have important bearing on its business decisions

dPower structureThe internal power relationship between the board of directors and the chief executive is an important factor

eCompany image and brand equityThe image and brand equity of the company play a significant role in raising finance forming alliance choosing dealers and suppliers launching new products entering foreign markets

5 What is Macro environmentAnswerMacro environment refers to the general environment or remote environment within which a business firm and forces in its micro environment operateA company does not directly or regularly interact with the micro environmentTherefore macro environment is also known as indirect action EnvironmentThe macro environment forces are less controllable than the micro forces 6 What are the components of macro environmenta Political and legal environmentb Economic environmentc Social and cultural environmentd Technological environment

BUSINESS FIRM

Fig COMPONENTS OF MACRO ENVIRONMENTPolitical science

Introduction to political science

Comparative politics and itrsquos scope Comparative politics is the second major dimension of political scienceIt is also a very vast area of study and a very large number of political scientists even treat it as an autonomous area of study within the board ambit of political scienceScope of comparative politics-

1 All political structures -Comparative politics includes the study of all structures formalnon formal governmental and extra governmental which are directly or indirectly involved in politics in all the countries of the world

2 Functional studies- Comparative politics seeks to study politics less from the point of view of the legal institutions in terms of their powers and move from the point of view of their functions which constitute the political process and their actual Operation in the environment

3 Study of political behaviour- Another important part of its scope is the study of the actual behaviour of the people in the process of politics

4 Study of similarities and differences- comparative politics also undertakesan analysis of the similarities and differences among political process and functions

5 Study of all political systems -comparative politics seeks to analyse the actual behaviour and performance of all political systems western as well as non western

6 Study of the environment and infrastructure of politics-The study of politics demands a study of the psychological sociological economic and anthropological environment in fact the social environment as a whole in which each political system operates

7 Study of political culture- political culture is composed of attitudesbeliefs emotions and values of a society that relate to the political system or politics

8 Study of political participation- Political participation is a universal processThe only difference is that while in some states it is limited in others it is wider

9 Study of political process- political

Answer the following questions-

What is comparative politics

What are the scope of comparative politics

Homework- learn

processes like decision makingpolicy making judicial process leadership recruitment process and others are always at work in all political systems

The scope of comparative politics is very comprehensive It includes everything that falls within the area of political activity and political process

History CAMBRIDGE VIEW ABOUT

THE PARTITION

AND REFUTATION

OF CAMBRIDGE

VIEW

Cambridge view about the Partition The Cambridge school of historians have interpreted that opposition to partition scheme was made entirely by the elitist groups They hold the view that Lord Curzon planned to partition the Bengal for administrative purposeREFUTATION OFCAMBRIDGE VIEW The Rationalist historians have rejected the interpretations of the Cambridge School of historians on various grounds

1 QUESTION State different views of historians regarding Partition of Bengal

ANSWER Cambridge historians believed that Lord Curzon partitioned Bengal for administrative reasons only and not for the political motive The Middle class elitist group protested because of their petty interest The Hindu zamindars protested as they have to spend more money for managing their estatesThe lawyers of Calcutta High court feared to lose their clientBut according to the nationalist Historians was-

2- The ultimate object of Lord Curzon was to crush the unity of Bengal politicians

3- If Bengal becomes a separate province Bengali speaking 16 million people of western part would become minority under Hindi speaking people of Bihar and Oriya speaking people of Orissa

4- The bureaucrats expected that the protest movement would die down quickly

5- Lord Curzon used the Muslim community in his political game

6- Idealism had great contribution in the protest against partition

7- The people of the every section of society were affected by the partition of Bengal

Computer Science

Numbers Convertion of dcimal number to octal numberThe decimal numeral system is the standard system for denoting integer and non-integer numbers It is the extension to non-integer numbers of the Hindu-Arabic numeral system For writing numbers the decimal system uses ten decimal digits a decimal mark and for negative numbers a minus sign - The decimal digits are 0 1 2 3 4 5 6 7 8 9 the decimal separator is the dot in many countries

The octal numeral system or oct for short is the base-8 number system and uses the digits 0 to 7 Octal is sometimes used in computing instead of hexadecimal perhaps most often in modern times in conjunction with file

permissions under Unix systems It has the advantage of not requiring any extra symbols as digits It is also used for digital displays

Follow these steps to convert a decimal number into octal form

1 Divide the decimal number by 82 Get the integer quotient for the next iteration (if the number will not divide equally by 8 then round down the

result to the nearest whole number)3 Keep a note of the remainder it should be between 0 and 74 Repeat the steps until the quotient is equal to 05 Write out all the remainders from bottom to top This is the solution

For example if the given decimal number is 8453

Division Quotient Remainder

8453 8 1056 5

1056 8 132 0

132 8 16 4

16 8 2 0

2 8 0 2

Then the octal solution is 20405

Subject Eng Literature (The Tempest ndash William Shakespeare) Topic Act I Scene 1 Lines 33 to 67 (End of scene) Date 16th April 2020 (4th Period)

[Students should read the original play and also the paraphrase given in the school prescribed textbook]Summary Questions amp Answers

[SUMMARY OF THE ENTIRE SCENE]

o The play starts with the scene of a severe storm at sea Alonso (King of Naples) Sebastian (Alonsorsquos brother) Ferdinand (Alonsorsquos son) Gonzalo Antonio (the usurping Duke of Milan) are in a ship in the midst of the storm

o The mariners are trying their best to control the vessel from running aground and are totally following the orders of their Master the Boatswain They have scant success

o The mariners become extremely unhappy and annoyed when most of the passengers arrive on the deck thereby hampering their effort to save the ship There is serious confrontation between them and the passengers who are part of the Kingrsquos entourage

o The mariners could not save the ship

SUMMING-UP

(i) Vivid description of the scene which gives a realistic description of terror and confusion of a tropical storm

(ii) Shows Shakespearersquos accuracy of knowledge in describing the naval operations and also matters of seamanship

(1) GONZALO Ill warrant him for drowning (L 45-57)

though the ship were no stronger than a nutshell and as leaky as an unstanched

wenchBOATSWAIN Lay her a-hold a-hold Set her two courses Off to

sea again lay her offMARINERS All lost To prayers to prayers All lostBOATSWAIN What must our mouths be coldGONZALO The king and prince at prayers Lets assist them

For our case is theirsSEBASTIAN Im out of patienceANTONIO We are merely cheated of our lives by drunkards

This wide-chopped rascal - would thou mightst lie drowning the washing of ten tides

(a) What does Antonio say at the insolent manners of the boatswain just before the given passage

Being irritated at the insolent manners of the boatswain just before the given extract Antonio the Duke of Milan calls him a worthless dog son of a woman without any morals an arrogant and disrespectful noisemaker He says that the boatswain deserved to be hanged(b) What statement does Gonzalo repeat about the boatswain

Gonzalo shows his faith that the boatswain is not destined to die by drowning He is destined to be hanged and nothing can alter this decree of destiny He says that even if the ship was as frail as a nutshell the boatswain could not be drowned for his destiny was to be hanged(c) What do the passengers do when they have lost all hope of their survival

When the passengers have lost all hope of survival they take

(iii) The opening scene justifies the title ndash The Tempest

UNANSWERED QUESTIONS

(i) The King always travels with his entire fleet including his soldiers Where were the other ships

(ii) Why was the ship in that area Where was it coming from or going where

(iii) The ship broke apart What happened to those who were in the ship

(We shall get the answer to the above questions as the play progresses)

leave of life with fervent prayers The mariners take their last hearty drink and are ready for death(d) What blame does Antonio put upon the mariners and the boatswain Antonio rebukes the mariners that these drunkards have brought them to the present crisis by neglecting their duties He blames them saying that they are going to lose their lives entirely for the negligence of the boatswain and his fellows(e) What does Antonio say while cursing the boatswain

Antonio gives vent to his wrath upon the boatswain in particular He calls the boatswain a wide-mouthed rascal who deserves to be hanged on the sea-shore at low water mark so that ten tides might wash over his body and take out of him all the liquor that he has been drinking

Class XIISubject Topic Summary ExecutionHistory Topic

1 1935 ACT AND WORKING OF PROVINCIAL AUTONOMYCONGREE AND OTHER MINISTERSSUB TOPIC GOVERNMENT OF INDIA ACT1935

Government of India Act 1935 This act established a lsquoFederation of Indiarsquo made of British Indian provinces and Indian states and provided for autonomy with a government responsible to the elected legislature in every provinceThis act introduced abolition of Diarchy at provinces The entire provincial administration was introduced to the responsible ministers who were controlled and removed by the provincial legislature The provincial autonomy means two things First The provincial governments were wholly responsible to the provincial legislature Secondly Provinces were free from outside control and interference in the large number of matters The act divided the powers between the centre and provinces in terms of three lists- Federal list( for centre) Provincial list (for province) and concurrent list (for both) Residuary powers were given to the viceroy In the election under the government of India Act the Congress swept the poll the mandate of the people came in favour of the congress so far as general Hindu seats were concerned The Congress did not get a single Muslim seates in Bombay CP UP Sind and BengalIn five provinces Congress had yhe clear majority In BengalNWFPAssam and Bombay Congress emerged as a single largest partyOn the other side the performance of the Muslim League was badThus the Congress formed ministers in 7 provinces out of 11 provinces Coalition ministry was also formed in two other provincesOnly BENGAL AND Punjab had non- congress ministries

1 QUESTION What was the main change introduced by the Government of India ActANSWER a) The Act gave more

autonomy to the provinces b) Diarchy was abolished at the

provincial levelsc) The Governor was the head of

the executived) There was a council of

ministers to advise him The ministers were responsible to the provincial legislatures who controlled them The legislature could also remove the ministers

e) The Governors still retained special reserve powers

2 QUESTION Why did the federal scheme introduced by the Government of India Act 1935 never come into operation

ANSWER The Federal structure of the Government of India was to be composed with the Governor General and Council of ministers The Federal legislature was to be Bicameral legislature- The council of states and the House of Assembly The ministers were to be chosen by the Governor general and they were to hold the office during his pleasure

The provinces of British India would have to join the federation but this was not compulsory for the princely states

This federation never materialised because of the lack of support from the required number of

princely statesThis act was refused and

rejected by the princes the Congress and the Muslim League

Thus both Congress and the League participated in the election of 1937 Thus the federal part was never introduced but the provincial part was put into operations

Bengali 2nd

Language

াচেরর পরাথCনা(কহিতা )

াচেরর পরাথCনা কহিতাটি কহি (ঙখ দেঘাচে4র দো আচো য কহিতায় াচেরর পতর হমায়ন কঠিন দেরাচেগ আxানত ার ঈশবর া আললার কাচেছ পরাথCনা কচেরচেছন তার পচেতরর ীন হিফহিরচেয় হিচেত এই কহিতায় ার পচেতরর ীন হিভbা দেচেয়চেছন ারার এমনহিক হিনচের ীন হিসCচেনর হিহিনমচেয় হিতহিন তার দেছচের ীন হিফচের দেপচেত দেচেয়চেছন তার দেছচের এই দেরাচেগর ন য হিতহিন হিনচেচেকই ায়ী কচেরচেছন তার হিনচের করা পাপচেকই হিতহিন ায়ী কচেরচেছন এছাা রানৈনহিতক ও আথCসামাহিক অসথার কথা তচে ধরা চেয়চেছ এই কহিতায় ার তার হিনচের পাপ কমCচেকই ায়ী কচেরচেছ ার অন যায় ভাচে দেপহি((হিকতর মাধ যচেম অপররা য কচেরচেছ আর এই অন যায় কাচের ন যই তার পহিরাচের হিপযCয় এচেসচেছ দে এক পরকার মানহিক নধন ইহিতাচেসর ার হিপতা চেয় সবাভাহিকভাচে ভাচোাসা দে মমতা দেথচেক মকত চেত পাচেরনহিন তাই হিপতা চেয় আললা া ভগাচেনর কাচেছ পতর হমায়চেনর পরানহিভbা দেচেয়চেছন ার আললা া ভগাচেনর কাচেছ াহিনচেয়চেছন তার হিনচের ীন হিসCন হিচেত হিতহিন রাী তার হিহিনমচেয় পচেতরর ীন হিফচের দেপচেত দেচেয়চেছন াচেরর হিপতসভ হিচেকর কথা এই কহিতায় ফটিচেয় দেতাা চেয়চেছ হিপতা পচেতরর হিরাহিরত মান নধচেনর কথা তচে ধরা চেয়চেছ

হিচে(4 হিকছ াইচেনর তাৎপযC১) ldquoদেকাথায় দেগ ওর সবচছয দেৌন দেকাথায় কচেরায় দেগাপন bয়ldquoউততর) াচেরর পতর হমায়ন কঠিন দেরাচেগ অসসথ তাই তার দেযৌন াহিরচেয় যাচেচছ এই দেরাচেগ তাচেক দেগাপচেন কচেরকচের াচেচছ তার সক (হিকত ধীচের ধীচের bয় চেচছ তাই হিপতা চেয় ার আললার কাচেছ হমায়চেনর পরান হিভbা দেচেয়চেছন২) ldquoাগাও (চেরর পরাচেনত পরানতচের ধসর (ন দেযর আান গানldquoউততর) াচেরর পতর হমায়ন কঠিন দেরাচেগ আxানত তাই ার আ দে(াচেক মমCাত (চেরর পচেথ পরানতচের আান গান ধবহিনত দোক দেসই আান গান আললার কাচেছ দেযন চে যায় আললা দেযন এই আহিতC শচেন পচেতরর ীন হিফহিরচেয় দেয় ৩)ldquoনাহিক এই (রীচেরর পাচেপর ীানচেত দেকানই তরারণ দেনই ভহি4চেতরldquoউততর) হমায়চেনর অসসথতার ন য ার হিনচেচেকই ায়ী কচেরচেছন কারন ার অচেনক রা য অন যায় ভাচে কচেরচেছ তাই তার এই পাপ কাচের ন য তার ঘচের আ হিপ এচেসচেছ এই অন যায় কাচের ন য তার মহিকত দেনই তাই ার আললার কাচেছ এই পাপ কাচেযCর ন য bমা পরাথM

Hindi 2ndlang

-ासी(जयशकर परसा-)

-ासी जयशकर परसा- की एक ऐसी कहानी ह जिजसम भारतीय ससकनित और राषटरीयता का सवरगजीतहोता ह इस कहानी म इरावती एक निहद कनया ह जिजस मलअचछो न मलतान की लट म पकडा और -ासी बना दि-या उस 500 दि-न -कर काशी क एक महाजन न खरी-ा दसरी -ासी निफरोजा ह वह गलाम ह निफरोजा को छडान क कतिलए अहम- को 1000 सोन क कतिसकक भजन थ जो अभी तक नही आए थ राजा साहब कठोर होत हए भी निफरोजा को निबना धनराकतिश क कतिलए उस म कर -त ह वनिफरोजा को अहम- को समझान की बात कहत हकहानी क अत म हम -खत ह निक इरा वती और जाटो क सर-ार बलराज का मिमलन होता हअहम- को यa म मार दि-या जाता ह वहा निफरोजा की परसननता की समामिध बनती ह वहा एक फल चढती ह और डीजल आती ह निफरोजा उस समामिध की आजीवन -ासी बनी रहती हलखक अपन उददशय अथात -ास परथा पर परकाश डालन और इस परथा क कारण होन वाल -ातो क दखो को दि-खान म पणता सफल हए ह

helliphellipContinue to next

Biology Reproductio Today we will discuss about vegetative Q1 Name some vegetative propagules

n in Organisms

propagation of plants The process of multiplication in which fragments of plant body function as propagule and develop into new individual is called vegetative propagation The units of such propagation are runner rhizome tuber bulb etc

and the speciesinvolvedVegetative propagules

Parts involved

Bulb StemBulbil BulbilRhizome Stem Runner Stem Tuber Stem Offset Stem Leaf buds Leaves Suckers Stem

Corns Stem stolon

Q2 State advantages of vegetative propagation

i) Rapid methodii) Sure and easy methodiii) Useful in plants that cannot

produce viable seeds or long seed dormancy

iv) Maintains purity of raceQ 3 Banana fruit is said to be parthenocarpic where as turkey is said to be parthenogenetic WhyBanana develops without fertilization from an unfertilized ovary thus is parthenocarpicIn turkey the ovum or female gamete developinto a new chick without fertilization thus isparthgenetic

Q4 Why is water hyacinth is called as a ldquoTerror of Bengalrdquo Water hyacinth can

propagatevegetatively all over the water body in a short per short period of time This resulted increased biochemicaloxygen oxygen demand of water body causing mortalityof fishes It is very difficult to get rid off them Thus known as terror of Bengal

Chemistry

Solid state GENERAL CHARACTERISTICS OF SOLID STATEIn nature the particular state of matter is governed by two opposing forces at given set of temperature and pressure These forces are intermolecular force of attraction and thermal energy If intermolecular force of attraction is high as compared to thermal energy particles remains in closest position

Intext QuestionsQ1 Classify the following solids as crystalline and amorphous Sodium chloride quartz glass quartz rubber polyvinyl chloride Teflon

A1 Crystalline

and hence very less movement in particles is observed In this case solid state is the preferred state of matter

Let us revise the general characteristics of solid

i) Fixed mass volume and shape

ii) Strong intermolecular force of attraction

iii) Least intermolecular space

iv) Fixed position of constituent particles

v) Incompressible and rigid

Q2 what type of interactions hold the molecules together in a polar molecular solid[CBSE 2010]A2 The molecules in a solid are held together by van der Waals forces The term van der Waals forces include hydrogen bonding dipole-dipole attraction and London dispersion forces All molecules experience London dispersion forces In addition polar molecules can also experience dipole-dipole interactions So the interactions that holds the molecule together in polar molecular solid are London dispersion force and dipole-dipole interactionsQ3 Write a feature that will distinguish a metallic solid from an ionic solid [CBSE 2010]A3 Metals are malleable and ductile whereas ionic solid are hard and brittle Metallic solid has typical metallic lustre But ionic solid looks dullQ4 Write a point of distinction between a metallic solid and an ionic solid other than metallic lustre [CBSE 2012]A4 Metals are malleable and ductile whereas ionic solid are hard and brittleQ5 Write a distinguish feature of metallic solid [CBSE 2010]A5 The force of attraction in

solid Sodium chloride Quartz Amorphous solid Quartz glass rubber polyvinyl chloride Teflon Q2 why glass is considered as super cooled liquidA2 Glass shows the tendency to flow at slower rate like liquid Hence they considered as super cooled liquidQ3 why the window glass of old buildings show milky appearance with timeA3 Glass is an amorphous solid Amorphous solid has the tendency to develop some crystalline character on heating Due to heating in day over the number of years glass acquires some crystalline character and show milky appearanceQ4 why the glass panes fixed to window or doors of old building become slightly thicker at bottomA4 Glass is super cooled liquid It has the tendency to flow down very slowly Due to this glass pane becomes thicker at the bottom over the timeQ5 Sodium chloride is a crystalline solid It shows the same value of refractive index along all the direction TrueFalse Give reasonA5 FalseCrystalline solid shows anisotropy in properties That is it shows different values for the given physical property in different direction All the crystalline solids show anisotropy in refractive index Therefore sodium chloride will show different values of refractive index on different directions

Q6 Crystalline solid are anisotropic in nature What does this statement means

between the constituent particles is special kind of electrostatic attraction That is the attraction of positively charged kernel with sea of delocalized electronsQ6 which group of solid is electrical conductor as well as malleable and ductile [CBSE 2013]A6 Metallic solidQ7 why graphite is good conductor of electricity although it is a network (covalent solid)A7 The exceptional property of graphite is due to its typical structure In graphite each carbon is covalently bonded with 3 atoms in same layer The fourth valence electron of each atom is free to move in between different layersThis free electron makes the graphite a good conductor of electricity

[CBSE 2011]A6 Anisotropy is defined asrdquo Difference in properties when measured along different axis or from different directionsrdquo Crystalline solid show different values of some of the physical properties like electrical resistance refractive index etcwhen measured along the different directions The anisotropy in crystalline solid arises due to the different arrangement of particles in different directions

Math Function Composition of functions Think of an industrial plant that produce bottles of cold drinks first there is the operation (or function) f that puts the cold drink inside the bottle followed by the opeartion g that close the bottle with the capThis leads to the following definitionDefinition Let f A rarr B and g B rarr C be two functions Then the composition of f and g denoted by gof is defined as the function gof A rarr C given by gof(x) = g(f (x)) forall x isinA

Definition A function f X rarr Y is defined to be invertible if there exists a function g Y rarr X such that gof = IX and fog = IY The function g is called the inverse of f and is denoted by f -1

Thus if f is invertible then f must be one-one and onto and conversely if f is one-one and onto then f must be invertible This fact significantly helps for proving a function f to be invertible by showing that f is one-one and onto specially when the actual inverse of f is not to be determined

Example 1 Let f 2 3 4 5 rarr 3 4 5 9 and g 3 4 5 9 rarr 7 11 15 be functions defined as f(2) = 3 f(3) = 4 f(4) = f(5) = 5 and g (3) = g (4) = 7 and g (5) = g (9) = 11 Find gofSolution We have gof(2) = g (f(2)) = g (3) = 7 gof(3) = g (f(3)) = g (4) = 7gof(4) = g (f(4)) = g (5) = 11 and gof(5) = g (5) = 11Example 2 Find gof and fog if f R rarr R and g R rarr R are given by f(x) = cos x and g (x) = 3x2 Show that gof ne fogSolution We have gof(x) = g(f(x))=g(cosx) = 3 (cos x)2

= 3 cos2 x Similarly fog(x)=f(g (x))= f(3x2)= cos (3x2) Note that 3cos2 x ne cos 3x2 for x = 0 Hence gof ne fogExample 3 Show that if f A rarr B and g B rarr C are onto then gof A rarr C is also ontoSolution Given an arbitrary element z isin C there exists a pre-image y of z under g such that g (y) = z since g is onto Further for y isin B there exists an element x in A with f(x) = y since f is onto Therefore gof(x) = g (f(x)) = g (y) = z showing that gof is onto Example 4 Let Y = n2 n isin N sub N Consider f N rarr Y as f(n) = n2 Show that

f is invertible Find the inverse of fSolution An arbitrary element y in Y is of the form n2 for some n isin N This implies that n =radicy This gives a function g Y rarr N defined by g (y) =radicy Nowgof (n) = g (n2)=radicn2 = n and fog (y) =f(radicy) = (radicy) 2 y which shows that gof=IN and fog= IY Hence f is invertible with f -1 = g

Political Science

Constitution of India-The Preamble

Summary

Objective of the state-To secure equality of status and of opportunity To promote fraternity among all the citizens To assure the dignity of the individuals and Unity and integrity of the nation

Justice-Justice stands for rule of law absence of arbitrariness and a system of equal rights freedom and opportunities for all in a society India seeks social economic and political justice to ensure equality to its citizens

Liberty-Liberty implies the absence of restraints or domination on the activities of an individual such as freedom from slavery serfdom imprisonment despotism etc The Preamble provides for the liberty of thought expression belief faith and worship

Equality-Equality means the absence of privileges or discrimination against any section of the society The Preamble provides for equality of status and opportunity to all the people of the country

Fraternity-The Preamble declares that fraternity has to assure two thingsmdashthe dignity of the individual and the unity and

Execution

Answer the following questions-

Short notes-1 Equality2 Fraternity3 Justice4 Liberty

Homework-Learn

integrity of the nation The word integrity has been added to the Preamble by the 42nd Constitutional Amendment (1976)

Business studies

Human resource management (chapter 1)

On the day of 1504 2020 I have discussed with you the managerial functions and procurement functions of HRM

Today weare going to discuss about the development function integration functions and maintenance function

Development functions-HRM improves the knowledge skills attitude and values of employees so that they the present and future jobs more effectively it includes

1) Development functions of HRM

a) Performance appraisal = It implies systematic evaluation of employees with respect to their performance on the job and their potential for development

b) Training =It is the process by which employees learn knowledge skills and attitudes to achieve organisational and personal goals

c) Executive development = It is the process of developing managerial talent through appropriate program

2) Integration functionsa) HRM reconcile the goals of

organisation with those of its members through integrating function

b) HRM tries to motivate employees to various financial and non financial incentives provided in job specification etc

3) Maintenance functiona) HRM promote and protect the

physical and mental health of employees by providing several types of benefits like housing medical aid etc

b) It Promote Social security measures to employees by providing provident fund pension gratuity maternity benefits

SubjectCOMMERCE

Topic

BUSINESSENVIRONMENT

Summary

Now quickly let us revise the earlier points that we have already done in the last class and let us proceed with the other topics that are there in the chapter

Firstly we will recall the internal and external factors of micro environment and then we

Execution 3 What do you mean by internal factors

in micro environmentAnswerInternal factors refer to all the factors existing within a business firm The internal factors are considered controllable because the enterprise has control over these factors

Development FunctionsPerformance AppraisalTrainingExecution Development

shall proceed in details

Meaning and list of internal and external factors

aInternal factorsInternal factors refer to all the factors existing within a business firm The internal factors are considered controllable because the enterprise has control over these factorsFor an example a company can alter its organization structure policies programmes employees physical facilities and marketing mix to suit the changes in the environmentList of internal factors areCorporate culture mission and objectives top management organizations structure company image and brand equity company resources

b External factorsExternal factors refer to those individual and groups and agencies with which a particular business organization comes into direct and frequent contact in the course of its functioningThese individuals and groups are known as STAKEHOLDERS because they have a stake (financial interest ) in the working and performance of the particular business List of external forces (stakeholders)Customers competitors investors suppliersmiddlemen (marketing intermediaries)financers publics

customers

suppliersfinancers

For an example a company can alter its organization structure policies programmes employees physical facilities and marketing mix to suit the changes in the environment

4 What do you mean by external factors in micro environment

AnswerExternal factors refer to those individual and groups and agencies with which a particular business organization comes into direct and frequent contact in the course of its functioningThese individuals and groups are known as STAKEHOLDERS because they have a stake (financial interest) in the working and performance of the particular business

3Who are stakeholdersSTAKEHOLDERS are individuals and groups who have a stake (financial interest ) in the working and performance of the particular business 4Discuss the internal factors in briefa Corporate CultureThe values beliefs and attitudes of the founders and top management of the company exercise a strong influence on what the cmpaany stands for how it does things and what it considers importantbMission and objectivesThe business philosophy and purpose of a comoany guide it prioritiesbusiness strategiesproduct market scope and development scope

cTop management structurethe composition of board of directors the degree of professionalization of management and the organizational structure of a company have important bearing on its business decisions

dPower structureThe internal power relationship between the board of directors and the chief executive is an important factor

e Company image and brand equityThe image and brand equity of the company play a significant role in raising finance forming alliance choosing dealers and suppliers launching new products entering foreign markets

5 What is Macro environmentAnswerMacro environment refers to the general

competitors

middlemen

publics

Fig STAKEHOLDERS OF A COMPANY

Apart from micro environment the other main dimension of business environment isMacro environment Macro environment refers to the general environment or remote environment within which a business firm and forces in its micro environment operateA company does not directly or regularly interact with the micro environmentTherefore macro environment is also known as indirect action EnvironmentThe macro environment forces are less controllable than the micro forces

Macro environment consists of the following components

POLITICAL AND LEGAL ENVIRONMENT

ECONOMIC SOCIAL AND ENVIRONMENT

CULTURAL

ENVIRONMENT

TECHNOLOGICAL ENVIRONMENT

Fig COMPONENTS OF MACRO ENVIRONMENT

environment or remote environment within which a business firm and forces in its micro environment operateA company does not directly or regularly interact with the micro environmentTherefore macro environment is also known as indirect action EnvironmentThe macro environment forces are less controllable than the micro forces 6 What are the components of macro environmenta Political and legal environmentb Economic environmentc Social and cultural environmentd Technological environment

Computer Science

Logic gates

Digital systems are said to be constructed by using logic gates These gates are the AND OR NOT NAND NOR EXOR and EXNOR

BUSINESS FIRM

gates The basic operations are described below with the aid of truth tables

AND gate

The AND gate is an electronic circuit that gives a high output (1) only if all its inputs are high A dot () is used to show the AND operation ie AB Bear in mind that this dot is sometimes omitted ie ABOR gate

The OR gate is an electronic circuit that gives a high output (1) if one or more of its inputs are high A plus (+) is used to show the OR operationNOT gate

The NOT gate is an electronic circuit that produces an inverted version of the input at its output It is also known as an inverter If the input variable is A the inverted output is known as NOT A This is also shown as A or A with a bar over the top as shown at the outputs The diagrams below show two ways that the NAND logic gate can be configured to produce a NOT gate It can also be done using NOR logic gates in the same way

NAND gate

This is a NOT-AND gate which is equal to an AND gate followed by a NOT gate The outputs of all NAND gates are high if any of the inputs are low The symbol is an AND gate with a small circle on the output The small circle represents inversion

NOR gate

This is a NOT-OR gate which is equal to an OR gate followed by a NOT gate The outputs of all NOR gates are low if any of the inputs are highThe symbol is an OR gate with a small circle on the output The small circle represents inversion

EXOR gate

The Exclusive-OR gate is a circuit which will give a high output if either but not both of its two inputs are high An encircled plus sign ( ) is used to show the EOR operation

EXNOR gate

The Exclusive-NOR gate circuit does the opposite to the EOR gate It will give a low output if either but not both of its two inputs are high The symbol is an EXOR gate with a small circle on the output The small circle represents inversion The NAND and NOR gates are called universal functions since with either one the AND and OR functions and NOT can be generated

Note A function in sum of products form can be implemented using NAND gates by replacing all AND and OR gates by NAND gates A function in product of sums form can be implemented using NOR gates by replacing all AND and OR gates by NOR gates

Logic gate symbols

Table 2 is a summary truth table of the inputoutput combinations for the NOT gate together with all possible inputoutput combinations for the other gate functions Also note that a truth table with n inputs has 2n rows You can compare the outputs of different gates

Logic gates representation using the Truth table

Example

A NAND gate can be used as a NOT gate using either of the following wiring configurations

Subject Eng Literature (The Tempest ndash William Shakespeare) Topic Act III Scene 3 Lines 53 to 110 (End of the scene) Date 16th April 2020 (2nd Period)

[Students should read the original play and also the paraphrase given in the school prescribed textbook]Summary Questions amp Answers

o Seeing this strange scene all are inclined to believe the tales told by travelers that there truly are ldquounicornsrdquo and ldquothe phoenixrsquo thronerdquo

o As they are about to sit down to the feast the banquet is snatched away by a harpy (Ariel disguised) A spiritrsquos voice (Arielrsquos voice) denounces Alonso Sebastian and Antonio with particular

1 ARIEL You are three men of sin whom Destiny

(Line 53-58)That hath to instrument this

lower world And what is int the never-surfeited sea

Hath caused to belch up you and on this island

Where man doth not inhabit you rsquomongst men

Being most unfit to live I have made you mad

reference to their crime in expelling Prospero from Milan They have not received any punishment for their deed earlier but the time for their punishment has arrived Upon Alonso it pronounces ldquolingering perdition worse than deathrdquo from which there is no remedy except through sincere repentance Ariel then vanishes in thunder and the shapes enter again and carry away the table

o Prospero watching invisibly is very pleased with the performance of Ariel and his (Prosperorsquos) ldquomeaner ministersrdquo All his enemies are now in his power and are in a fit of desperation He then leaves them and goes to see how Ferdinand and Miranda are getting on

o Alonso is now much humbled and penitent with the after effect of the spiritrsquos denunciation of his crimes He believes that his son is lost forever After this all disperse being stricken mad by the speech of the spirit

o Gonzalo fearing that they may do violence to themselves or to one another follows them and bid others to follow

(a) To whom does Ariel disguised as a harpy call the three sinners What game did Fate of Destiny play with

them

The three sinners called by Ariel are Alonso Sebastian and Antonio It was Destiny which had caused the ocean to cast the three sinners on the shore Though the ocean is all the time devouring whatever appears on its surface and is never satisfied with its continual swallowing of the ships and men in the present case the ocean had cast these three sinners on the shore without killing them

(b) Who had jointly been responsible for the conspiracy against Prospero What is Prosperorsquos purpose behind all this

Three men Alonso Sebastian and Antonio had jointly

been responsible for the conspiracy against Prospero They had driven out Prospero form Milan Prosperorsquos purpose is to make these three sinners realize the wrong they had done He wants them to repent for their criminal deeds because repentance leads to self-esteem(c )What does Ariel (the harpy) tell Alonso and his companions when they take out their swords to attack him

Seeing them drawing their swords Ariel (harpy) tells them that he and his companions are the instruments of destiny and that it is not possible for human beings to do them any injury He says that the swords of human beings can not injure even a minute part of his feathers Their swords are as ineffective against him and his companions as against the wind or the water

(d) Give the explanatory meanings of the following expressions in the context of the above extract

(i)Never surfeited (ii) Belch up (iii) lsquomongst men

(i) Never surfeited never led to satisfaction

(ii) Belch up cast ashore(iii) lsquomongst men in human

society2

I and my fellows (Line 60-65)

Are ministers of Fate The elementsOf whom your swords are tempered may as wellWound the loud winds or with bemocked-at stabsKill the still-closing waters as diminishOne dowl thats in my plume

IMPORTANT PASSAGES EXPLAINED

The elements

(Line 61-66)Of whom your swords are tempered may

as wellWound the loud winds or with

bemocked-at stabs

(a) Who is lsquoIrsquo Who are his lsquofellowsrdquo

lsquoIrsquo is referred to Ariel in disguise of a harpy His lsquofellowsrsquo are other spirits serving Prospero the real Duke of Milan who has acquired supernatural powers after being banished from his Dukedom Prospero has settled in this uninhabited island

(b) What are the elements that have temperrsquod the swords Why will it not work against the speaker

The swords (of Alonso and his companions) are tempered by metal (steel) which is taken out of the earth and refined by

Kill the still-closing waters as diminishOne dowl thats in my plume My fellow

ministersAre like invulnerable

In these words Ariel reminds the King and his companions of the utter futility of drawing swords against himself and his fellows Ariel drives Alonso Antonio and Sebastian the three men of sin to desperation ndash a state in which men do violence to themselves They draw swords to strike Ariel But Ariel reminds them that he and the other spirits are the ministers of destiny and nothing can wound them The steel of which their swords are made of may cut the wind or water which being divided always closes up again Even supposing that such things may be possible it is quite impossible that their swords will cut one feather in their plume They are incapable of being wounded by any sword of man Hence it is foolish on their part to attempt to strike at Ariel and his fellow-spirits

For which foul deed

(Line 72-75)The powers delaying not forgetting

haveIncensed the seas and shores yea all the

creatures Against your peace

Ariel enters like a harpy and remaining invisible tells Alonso Sebastian and Antonio that he and other harpies are the agents of Destiny appointed to carry out her decrees He tells them that their punishment for the crime against Prospero which has been so long deferred is now to fall upon them He reminds them that they had expelled Prospero from Milan and set him and his innocent child adrift on the sea and that the sea had paid them back for their sin by the shipwreck and by the calamities they have suffered He tells them that the powers above which did not forget this mean treachery but only deferred the punishment have now engaged the seas and the shores and all living beings including him and his comrades against them The very elements and supernatural agency Ariel adds have taken up the avenging of their crime against Prospero

the action of fire It may cut the wind or water which being divided always closes up again

The sword will not work against the spirits and the harpy because they are the ministers of destiny and nothing can wound them nor it will cut a single feather in their plume

(c )What is the meaning of lsquodowlrsquo in the last line

The term lsquodowlrsquo means a filament or the smallest part of a feather In this context Ariel in disguise of harpy says that their sword cannot even damage the smallest filament of their (Arielrsquos and other spirits) feathers as they are incapable of being wounded by any sword of man

(d) What does the speaker remind the listeners about

Ariel in disguise of harpy reminds Alonso the King of Naples Sebastian Alonsorsquos brother and Antonio the present Duke of Milan and the treacherous brother of Prospero as they being three men of sin He even reminds them that their punishment for their crime against Prospero which has been so long deferred now falls upon them He reminds them that they have expelled Prospero from Milan and has set him along with his innocent infant daughter adrift on the sea So the sea has paid them back for their sin by their shipwreck and the calamities they have suffered since then The harpy rebukes Alonso of his sin that has incensed the Gods and has deprived him of his son as a punishment

(e) How do they respond

When Ariel in disguise of a harpy reminds Alonso Sebastian and Antonio of their past misdeeds and sin Alonso has a look of terror and confusion in his eyes He utters the words of sincere repentance wrung out of his conscience-stricken heart It appears to him that all the elements of nature the sea-waves the wind and the thunder proclaiming a loud voice in the name of Prospero and the crime Alonso has committed against him They are calling upon him to repent There is a deep storm raging in Alonsorsquos breast and the echoes of that storm are ringing in his ears like a clear note of wind-instrument A note of denunciation of Alonsorsquos crime leaves him much humbled and penitent and confirms his belief that his son is lost forever But Sebastian and Antonio shows some courage instead of repentance They wish to kill the spirits or devils if it appears

3

Of my instruction hast thou nothing bated (Line 85-93)

In what thou hast to say So with good life

And observation strange my meaner ministers

Their several kinds have done My high charms work

And these mine enemies are all knit upIn their distractions They now are in my

powerAnd in these fits I leave them while I visitYoung Ferdinand whom they suppose is

drownedAnd his and mine loved darling

Methought the billows spoke and (Line 96-99)

told me of itThe winds did sing it to me and the

thunderThat deep and dreadful organ-pipe

pronouncedThe name of Prosper It did bass my

trespass

These are the words of contrition coming from Alonso Ariel has driven him to a deep repentance for conspiring with Antonio against Prospero He now feels a sincere remorse It appears to him that all the elements of nature the sea-waves the wind and the thunder proclaimed with a loud voice the name of Prospero and the crime Alonso had committed against him They are calling upon him to repent There is a deep storm raging in Alonsorsquos breast and the echoes of that storm are ringing in his ears like the clear note of a wind-instrument

Comment These are the words of sincere repentance wrung out of the conscience-stricken heart of Alonso Alonso who is the lesser villain is the first to give way to remorse under the effect of Arielrsquos speech The words of Ariel seem to him to be the voice of conscience speaking to him He is driven to desperation a state in which he might do violence to his life

(a) Identify the speaker State the context

Prospero the ruler of the island is the speaker The famous banquet scene has been enacted very well Ariel and his junior spirits have played their roles excellently Prospero is glad to say words of praise for them(b) In what way the speakerrsquos instructions have been carried out

According to Prosperorsquos instructions a banquet was presented before the King of Naples and his companions when they were tired and hungry Just when they were preparing to eat the feast the banquet was suddenly removed by exercising supernatural powers All this was done by Ariel Prosperorsquos chief assistant and a powerful spirit

Ariel not only made the feast disappear but also delivered his speech blaming the King and his two companions for their past wicked deeds He warned them to repent for their misdeeds or suffer forever on that uninhabited island

(c) Who are referred to as lsquomeaner ministersrsquo What have they done

Prospero refers as lsquomeaner ministersrsquo to his other lesser spirits who were assisting Ariel in presenting a scene before the kingrsquos party They entered the scene to the accompaniment of music They assumed several strange shapes and brought in a banquet Then they danced about it with gentle actions of salutations thus inviting the King and others to eat the feast

These spirits play their role again when Ariel in the shape of a harpy quits the scene These shapes enter again and dancing with mocking gestures carry away the table

(d) Who are the speakerrsquos enemies What has happened to them

King of Naples Alonso his brother Sebastian and the present Duke of Milan Antonio (Prosperorsquos own brother) are Prosperorsquos enemies With the turn of events they have all been washed ashore on the island which is ruled by Prospero the great magician Actually this happened after the shipwreck caused by a storm which was raised by Prospero with the purpose of bringing these people to his island Prosperorsquos spirits have already confused and terrified these enemies and they are under Prosperorsquos control He can treat them as he likes

(e) What does he say about Ferdinand Explain what is meant by ldquohellip his and mine darlingrdquo

Prospero knows that Alonsorsquos son prince Ferdinand is alive though his father thinks that the prince has been drowned

Prospero refers to his daughter Miranda who is dear to him She is also very dear to Prince Ferdinand who has fallen in love with her They are waiting to be married soon for which they have received Prosperorsquos consent

4

ALONSO O it is monstrous monstrous (Line 95-102)

Methought the billows spoke and told me of it

The winds did sing it to me and the thunderThat deep and dreadful organ-

pipe pronouncedThe name of Prosper It did bass

my trespassTherefore my son ithrsquo ooze is

bedded andIll seek him deeper than eer

plummet soundedAnd with him there lie mudded

(a) In what way does Alonso express his horror when his conscience is awakened by Arielrsquos words

When Alonsorsquos conscience is awakened by Arielrsquos words he expresses his horror at what he has heard He gets the feeling that the waves of the ocean the wind and the loud thunder have spoken to him and uttered the name of Prospero Because of being reminded of his crime in a very loud and rough voice he comes to realize that he has lost his son for his past misdeeds

(b) What does Alonso imagine about his son What does Alonso want to do in his desperate state

Alonso imagines that his son is lying in the mud at the bottom of the sea He feels desperate that he wants to drown himself in the ocean deeper than the plumb-line has ever gone He wants to lie with his son at the bottom of the sea

(c) How do Sebastian and Antonio want to face the evil spirits

Sebastian says that he is not at all afraid of what the harpy has said and that he is prepared to fight any number of such monsters if they appear before him only one at a time Antonio says that he would support Sebastian in the fight against the fiendsyyy

(d) Why does Gonzalo ask Adrian to follow the three men

Gonzalo tells Adrian that all the three men namely Alonso Sebastian and Antonio are in a wild and reckless mood The thought of the heinous crime of which they are guilty has begun to torment their minds So he asks Adrian to follow those three men without loss of time and prevent them from doing anything which the turmoil in their minds might lead them to do

(e) What opinion do you form of Alonso from the above extract

Alonso who is the lesser villain is the first to give way to remorse under the effect of Arielrsquos speech The words of Ariel seem to him to be the voice of conscience speaking to him He is driven to desperation a state in which he might do violence to his life

Subject =Accounts

Ac-12 15420 topic-pL Appropriation ac

PROFIT AND LOSS APPROPRIATION ACCOUNT

MEANING AND PREPARATIONProfit and Loss Appropriation Account is merely an extension of the Profit and Loss Account of the firm The profit of the firm has to be distributed amongst the partners in their respective profit sharing ratio But before its distribution it needs to be adjusted All Adjustments like partnerrsquos salary partnerrsquos commission interest on capital interest on drawings etc are made in this account These adjustments will reduce the amount of profit for distribution This adjusted profit will be distributed amongst the partners in their profit sharing ratio To prepare it at first the balance of Profit and Loss Account is transferred to this account The journal entries for the preparation of Profit and Loss Appropriation Account are given below

1 for transfer of the balance of Profit and Loss Account to Profit and Loss Appropriation Account

(a) In case of Net Profit

Profit and Loss Ac helliphelliphelliphelliphellipDrTo Profit and Loss Appropriation Ac(Net Profit transferred to Profit and Loss Appropriation Ac)

(b)In case of Net Loss

Profit and Loss Appropriation Achelliphelliphellip DrTo Profit and Loss Ac(Net Loss transferred to Profit and Loss Appropriation Ac)

2 for Interest on Capital

For transferring on Interest on CapitalProfit and Loss Appropriation Achelliphelliphellip DrTo Interest on Capital Ac(Interest on capital transferred to Profit amp Loss Appropriation Ac)

3 for Interest on Drawings

For transferring Interest on Drawings Interest on Drawings Achelliphelliphelliphelliphelliphellip DrTo Profit and Loss Appropriation Ac(Interest on drawing transferred to Profit amp Loss Appropriation Ac)

4 For Partnerrsquos SalaryFor transfer of partnerrsquos SalaryProfit and Loss Appropriation Achelliphellip DrTo Salary Ac(Salary transferred to profit amp Loss Appropriation Ac)

5 For Partnerrsquos CommissionFor transferring commissionProfit and Loss Appropriation Achelliphelliphellip DrTo Commission Ac(Commission transferred to Profit and Loss Appropriation Ac)

6 For Transfer of agreed amount to General ReserveProfit and Loss Appropriation Ac helliphellipDrTo General Reserve Ac(Transfer to General Reserve)

7 for share of Profit or Loss appropriation(a) If ProfitProfit and Loss Appropriation Achelliphellip DrTo Partnerrsquos CapitalCurrent Ac(Profit transferred to capitalcurrent Ac)(b) If LossPartnerrsquos Capital Current Achelliphelliphelliphellip DrTo Profit and Loss Appropriation Ac(Loss transferred to capitalcurrent Ac)

THE FORMAT OF PROFIT AND LOSS APPROPRIATION

Profit and Loss Appropriation Account for the year endedhelliphelliphelliphellip

Particulars Amount Particulars Amount

To PL Ac (loss) By pL Ac (profit)

To Interest on capital BY Interest on drawings

To partner`s commission by Partner`s capital Ac ( loss)

To Partner`s salary To Interest on partner`s loan To General Reserve To Partner`s Capital AC (Profit)

Subject= Economics

MOVEMENT ALONG THE DEMAND CURVE (CHANGE IN QUANTITY DEMANDED)In law of demand you have already studied the inverse relationship between price and quantity demanded When quantity demanded of a commodity changes due to change in its price keeping other factors constant it is called change in quantity demanded It is graphically expressed as a movement along the same demand curve There can be either a downward movement or an upward movement along the same demand curve Upward movement along the same demand curve is called contraction of demand or decrease in quantity demanded and downward movement along the same demand curve is known as expansion of demand or increase in quantity demanded

Extention of demandd

price (rs)p A

B Extentionp1 d

Q Q1

Quantity demanded ( in units)

Contraction of demandd

p2 Ccontraction

p APrice (Rs)

d

Q2 Q

Quantity demanded (in units)

Explanation of movement of demand A fall in price from OP to OP1 leads to increase in quantity demanded from OQ to OQ1 (expansion of demand) resulting in a downward movement from point A to point B along the same demand curve DD When Price rises from OP to OP2 quantity demanded falls from OQ to OQ2 (contraction of demand) leading to an upward movement from point A to point C along the same demand curve DD

  • Activity Series of Metals
    • Drawbacks of Rutherfordrsquos model of atom
      • Electromagnetic radiations
      • Properties of electromagnetic radiations
      • Characteristics of electromagnetic radiations
        • Plancks Quantum Theory-
        • Photoelectric effect
          • Intext Questions
            • Logic gates
            • Digital systems are said to be constructed by using logic gates These gates are the AND OR NOT NAND NOR EXOR and EXNOR gates The basic operations are described below with the aid of truth tables
            • AND gate
            • Example
Page 23:  · Web viewSubject. Topic. Summary. Execution. English 1 . Chapter 1 naming words . Page 8. Write the names of these pictures:- Person:-1. father. 2.Firefighter 3.doctor 4 ...

Biology Chapter - 01Controlling Air Pollution

Today we will discuss how we control air pollution from domestic combustion

Q1Describe any five ways of reducing air pollution from domestic sources bull The number of pollutants in the air is verylarge and we always try to control them byfollowing ways

i) Solar cooker and solar heater It use no fuel reduce damage of environment by fuel use or reducing deforestation It maintains coolness of house It releases very less orno oil gas or grease

ii) Piped natural gas (PNG) It emits very less by products into the atmosphere As it isdistributed through pipe lines so there iscontinuous supply of fuel is possible

iii) Liquefied Petroleum Gas (LPG) It hasa higher heating value LPG doesntcontain sulphur so it burns a lot cleanerenergy sources It releases very less oralmost no fume in air

iv) Electricity based cooking Emission free cooking alternative for urban dwellers causeselimination of adverse health impactsofindoor air pollution It helps to avoid theinconveniences associated with procurement of LPG

v) Biogas It contains 75 methane whichmakes it an excellent fuel It burns without smoke and biogas plant leaves no residue like ash in wood charcoal etc Thus it isaclean fuel

Economics

Factors of Production

Today firstly we would recall the last class for 5 mins and then we would proceed with the further topics of the chapter

The concept meaning of land characteristics of land and importance of land to be repeated for the absentees as well as the students who were there in the class the previous day

Today we will start with the last portion of land before it the meaning of land to be repeated onceAs by now we all know that

Questions1What do you mean by productivity of landAnswer By productivity of land we mean the capacity of a piece of land to produce a crop

Thus it refers to the average output per unit of landSay per acre per hectare etc= (OutputArea of land)

2 What are the factors influencing the productivity of landAnswer

Natural factors Productivity of land is largely determined by the natural

Land is defined to include not only the surface of the earth but also all other free gifts of nature(for example mineral resources forest resources and indeed anything that helps us to carry out the production of goods and services but is provided by nature free of cost)

We will move on to the last portion of land by discussing Productivity of Land

By productivity of land we mean the capacity of a piece of land to produce a crop

Thus it refers to the average output per unit of land

Say per acre per hectare etc= (OutputArea of land)

With this we shall proceed further with the main factors that determine the productivity of land

Natural factors Human factors Improvements on land Location of land Organisation Ownership of land Availability of capital Proper use of land State help

Note economic development of a country depends upon the quality of its land If the land is fertile it will quicken the pace of development of the country

qualities of land such as fertility etc

Human factors Land cannot produce anything by itself Man has to apply labour on it to produce for himself So productivity of land depends on the knowledge and skills of workers

Improvements on land production of land is affected by land development measures like provision of well or tubewell irrigation proper drainage

State help The government of a country especially less developed country can play a vital role in improving the agricultural productivity by providing better irrigation facilities

Organisation Productivity of land also fdepends upon the way how the factors of production like labour and capital are organised

In order to increase productivity trained workers modern implements scientific methods good seeds are all essential

3 lsquoImproved technology affects the productivity of landrsquo Explain this statement with the help of suitable example Answer Use of improved technology raises the productivity of land Example By using HYV seeds chemical manures and modern machines per hectare output increases

Physics Force (Summary)

Question Write the expression for the moment of force about a given axisSolutionsThe expression for the moment of force is given byMoment of force about a given axis = Force times perpendicular distance of force from the axis of rotationQuestion What do you understand by the clockwise and anticlockwise moment of force When is it taken positiveSolutionsIf the effect on the body is to turn it anticlockwise moment of force is called the anticlockwise moment and it is taken as positive while if the effect on the

body is to turn it clockwise moment of force is called the clockwise moment and it is taken as negative

Math Topic Commercial Mathematics

Chapter Goods and services Tax

Study item Some solved sums from exercise ndash 1 A retailer buys a TV from a wholesaler for Rs 40000 He marks the price of the TV 15 above his cost price sells it to the consumer at 5 discount on the marked price If the sales are intra ndash state and the rate of GST is 12 find

(i) The marked price of the TV(ii) The amount which the consumer pays for the TV(iii) The amount of tax (under GST) paid by the retailer to the central

Government(iv) The amount of tax (under GST) received by the State Government

Solution As the sales are intra- state sale and the rate of GST 12 So GST comprises of 6 CGST and 6 SGSTTherefore a retailer buys a TV from a wholesaler for Rs 40000Therefore the amount of GST collected wholesaler from the retailer or paid by retailer to wholesalerCGST = 6 of Rs 40000 = Rs(6100 times40000) =Rs 2400SGST = 6 of Rs 40000 = Rs (6100 times 40000) =Rs 2400Therefore wholesaler will pay Rs 2400 as CGST and Rs 2400 as SGSTTherefore amount of input GST of retailer Input CGST = Rs 2400 and input SGST = Rs 2400Again the retailer marks the price of the TV 15 above his cost price(i) The marked price of the TV

= Rs 40000 + Rs 40000times15= Rs 40000 + Rs 40000times 15100= Rs 40000 + Rs 6000Rs 46000But the retailer sells it to consumer at 5 discount on the marked priceCost price after discount = Rs 46000 ndashRs46000times 5100 =Rs 46000 ndashRs 2300= Rs 43700Therefore the amount of GST collected retailer from consumer or paid by consumer to retailerCGST = 6 of Rs 43700 =Rs ( 6100 times43700)Rs 2622SGST = 6 of Rs 43700 = Rs (6100 times 43700) =Rs 2622Amount of the output GST of retailer Output CGST = Rs 2622 and output SGST = Rs 2622

(ii) The amount which the consumer pays for the TV= cost price of TV to consumer + CGST paid by consumer + SGST paid by consumer= Rs 43700 + Rs 2622 + Rs 2622= Rs 48944

(iii) The amount of tax (under GST ) paid by the retailer to the central Government=CGST paid by retailer = output CGST ndash input CGST=Rs 2622 ndash Rs 2400=Rs 222

(iv) The amount of tax ( under GST ) received by the State Government = SGST paid by wholesaler + SGST paid by retailer= Rs 2400 + output SGST ndash input SGST=Rs 2400 + Rs 2622 ndash Rs 2400=Rs 2400 + Rs 222= Rs 2622

Commercial studies

Stakeholders Today I am going to give some revision questions from the previous study material

Questions1) State the two expectations of

employees from a business concern2) Give two distinctions between

stakeholder and shareholder3) Give two difference between

internal stakeholders and external stakeholders

4) Give two expectations of suppliers from a business organisation

5) Who is a stakeholder in commercial organisations

Chemistry Periodic Table

Merits of Mendeleevrsquos Periodic law are as follows - 1He grouped the elements on the basis of atomic mass 2 He left gaps for undiscovered elements like Gallium Scandium germanium Also he left a full group vacant for undiscovered inert gases 3 He could predict proportions of several elements on basis of their position in periodic table like Ga Sc etc 4He could predict errors in atomic weights of some elements like gold platinum etc

Anomalies in Mendeleevrsquos Periodic law are as follows - 1 Position of isotopes could not be explained 2 Wrong order of atomic masses could not be explained

For example- as Arnur atomic mass 40 come first and K with low atomic mass (30) should come later but k should be placed first

According to Bohrrsquos Modern Periodic table properties of elements are periodic functions of their atomic numbers

So when elements are arranged according to increasing atomic numbers there is periodicity in electronic configuration that leads to periodicity in their chemical properties

It consists of horizontal rows (Periods) Vertical column (Groups)

There are 7 period and 12 groups in this long form of periodic table

Ist period has 2 elements IInd period has 8 elements IIIrd period has 8 elements IVth period has 18 elements Vth period has 18 elements VIth period has 32 elements VIIth period hs rest of elements

Note - The number of valence electrons in atom of elements decides which elements will be first in period and which will be last

In group- 1 to 2 gp and 13 to 17 contain normal elements 3 to 12gp ndash transition elements 57 to 71 - lanthanides 89 to 103 - Actinides

Left hand side ndash metals Right hand side ndash nonmetals

Note- Hydrogen element has been placed at top of Ist group Electronic configuration of H is similar to alkali metal as both have 1 valence electron

V electron of gp I element -- 1 V electron of gp 2 element -- 2 V electron of gp 13 element -- 3 V electron of gp 14 element -- 4 V electron of gp 15 element -- 5 V electron of gp 16 element --6 V electron of gp 17 element -- 7 V electron of gp 18 element -- 8

English 1 Transformation of sentences

Sentences A sentence is a group of words which makes complete sense

Exercise 2Change the following sentences from

a Assertive sentencesb Imperative sentencesc Interrogative sentencesd Exclamatory sentences

Sentences can be changed from one grammatical form to another without changing the meaning of the sentence This is known as transformation of sentences

assertive to interrogative1 Nobody would like to be a fool

Who would like to be a fool2 Their glory can never fade

When can the glory fade3 Nobody can control the wind

Who can control the wind4 It matters little if I die

What though I die5 No man can serve two masters

Can any man serve two masters

Exercise 3Interchange of assertive and Exclamatory sentences

1 She leads the most unhappy lifeWhat an unhappy life she leads

2 This is indeed an interesting bookWhat an interesting book this

3 He is a very great manWhat a great man he is

4 It is a very lame excuseWhat a lame excuse

5 It is sad that she died so youngAlas she died so young

Class XISubject Topic Summary Execution

Hindi 2nd lang

पतर परम(परमचदर) पतर परम कहानी म एक निपता की इचछाओ का वणन निकया गया ह अपन बड पतर परभ -ास स निपता चतनय -ास का निवशरष परम था निपता को उसक जनम स ही बडी-बडी आशाए थी उसम दसर बट कतिशव-ास की अपकषा स- उतसाह की मातरा अमिधक थी वह उस इगलड भजकर बरिरसटर बनाना चाहत थभागय का खल भी बडा निनराला ह बीए की परीकषा क बा- वह बीमार पड गया डॉकटरो न भी जवाब - दि-या थाचतन -ास जी बहत ही कजस थ बवजह पस खच करना नही चाहत थ अगर गारटी मिमलती तो शाय- पस खच भी कर -त परत गारटी नही थी परिरणाम सवरप उनक बट का -हात हो गयाजब बट को समशान ल जा रह थ तो वहा काफी शोर गान बजान हो रह थ पछन पर पता चला निक निकसी निपता निपछल तीन साल स निबमार था और उसक ईलाज म रपया पानी की तरह बहाया पर ठीक नही हए परत उसक बट को तनिनक भी अफसोस नही था उसका कहना था उसन कोकतिशश तो कीयह -खकर चतनय-ास जी को आतम निगलानी हईतभी स उनका म परिरवतन हआ और बट का भोज काफी धमधाम स निकयाऔर वहइस पशचाताप की आग म जलत रह औला- स बढकर पसा नही होता ह इस बात को समझन म उनह काफी व लग गया

hellipContinue to next

BENGALI(2ND LANGUAGE)

পরথমঅধযায়-ঠাকরারীনদরনাথঠাকর

নয়ন দোচের হিমাচেররা া নাচেমই হিযাত হিছচেন ায়ানার উাররণ সবরপ নয়ন দোচের ারা হিা (াচেকর হিা হিচেতন এছাাও দেকান উৎস উপচেb রাহিতর দেক হিন করার উচেfচে(য তারা সযC হিকরচেরণ রনয পরীপ জবাহিচেয় তাচেত রপার হির 4Cরণ করচেতন ঠাকরা এই নয়ন দো হিমারচের দে(4 ং(ধর হিছচেন হিমাররা ায়ানার ষটানত পর(Cন কচের তারা হিনঃসব এই হিমাহিরর দে(4 ং(ধর গৈকাস নদর রায়চেৌধরী গৈকাস া নয়ন দোচের সমসত সমপহিতত ঋচেরণর াচেয় হিহিx কচের অহি(ষট যা আচেছ তাচেত হিপত

ইার হিপতার মতয ইচে পর নয়নচোচের ায়ানার দেগাটা কতক অসাধাররণ শরাদধ (াহিনতচেত অহিনতম ীহিপত পরকা( কহিরয়া ঠাৎ হিনহিয়া দেগ- ক) কার দো দেকান গচেলপর অং() কতা দেক ইার চেত কাচেক দোঝাচেনা চেয়চেছ গ) পরসঙগ কী কতার কতয পহিরসফট কচেরা

পরচে4র যাহিত রbা করা সমভ নয় তাই হিতহিন পতরচেক হিনচেয় ককাতায় সাস শর কচেরন গলপ কথচেকর আহিথCক অসথা নয়ন দোচের হিমাচের দেথচেক সমপরণC আাা কথচেকর হিপতা হিনচের দেষটায় অথC উপাCন করচেতন া উপাহিধ াচেভর নয তার াসা হিছনা আর দেসই কারচেরণ কথক তার একমাতর উততরাহিধকার চেয় তার হিপতার পরহিত কতজঞ কথক দো পা হি(চেচেছন হিনচের পরারণ ও মান রbার নয উপচেযাগী অথC হিনা দেষটায় পরাপত চেয়চেছন- এটাই তার কাচেছ পরম দেগৌরচের হি4য় চে মচেন কচেরন কাররণ (নয ভাণডাচের গৈপতক ায়ানার উজজব ইহিতাস অচেপbা দোার হিসeচেকর মচেধয গৈপতক দেকামপাহিনর কাগ তার কাচেছ অচেনক দেহি( মযান

TO BE CONTINUED

উ- ক) আচোয অং(টি রীনদরনাথ ঠাকচেরর দো ঠাকরা গচেলপর অং() কতা চেন আচোয গচেলপর গলপ কথকইার চেত নয়ন দোচের হিমাহিরর দে(4 ং(ধর গৈকাস ার কথা া চেয়চেছ গৈকাস া নয়ন দোচের সমসত সমপহিতত ঋচেরণর াচেয় হিহিx কচের অহি(ষট যা আচেছ তাচেত হিপত পরচে4র যাহিত রbা করা সমভ নয় তাই হিতহিন পতরচেক হিনচেয় ককাতায় সাস শর কচেরনগ) গৈকাস ার হিপতার মতযর পর নয়ন দোচের হিমাহিরর অহিসততব হিপত য় কচেয়কটা উৎস ও শরাদধ- (াহিনতচেত হিমাহিরর দে(4 কহিটক যয় চেয় হিগচেয় এচেক াচের দে(4 চেয় যায় তন তাচের গC করার মত আর হিকছই হিছ না-দেসই পরসচেঙগ এই উহিকত নয়নচোচের হিমাচেররা া নাচেমই হিযাত হিছচেন ায়ানার উাররণ সবরপ নয়নচোচের ারা হিা (াচেকর হিা হিচেতন এছাাও দেকান উৎস উপচেb রাহিতরচেক হিন করচেত হিগচেয় তারা সযC হিকরচেরণর নয পরীপ জবাহিচেয় তাচেত রপার হির 4Cরণ করচেতন তাই দেসকাচের ায়ানা দেহি(হিন সথায়ী চেত পারত না হিহিভনন উৎস শরাদধ- (াহিনতচেত সাধযা হিতহিরকত র করার নয হিমাহির হিহিকচেয় দেযত হ হিতC কা হিহি(ষট পরীচেপর দেত দেযমন অলপকাচের মচেধয হিনঃচে(4 চেয় যায়-নয়নচোচের হিমারচের অসথা তাই চেয়হিছ এই কারচেরণই কথক নয়নচোচের হিমারচের গা ভরা আমবর সয করচেত পারতনা

Physics Dimensional Analysis (Summary)

Q Find the dimensions of consts ab in relation

p=(bminusxlowastx)at

where p is the power x is the distance and t is time

Ans From principle of homogeneity dimension of b x2 are same Dim of b = dim of x2 = [L2] = [ML2T0]Dim of a = dim of ( b- x2)dim of (pt) = [M0L2T0][ML2T-2] [T-1] [T] = [M-1L0T2]

Chemistry Atomic Structure Drawbacks of Rutherfordrsquos model of

atom a According to Rutherfordrsquos model of atom electrons which are negativelycharged particles revolve around the nucleus in fixed orbits Thusb theelectrons undergo acceleration According to electromagnetic theory of Maxwell a charged particle undergoing acceleration should emitelectromagnetic radiation Thus an electron in an orbit should emitradiation Thus the orbit should shrink But this does not happenc The model does not give any information about how electrons aredistributed around nucleus and what are energies of these electrons Isotopes These are the atoms of the same

Properties of electromagnetic radiationsa Oscillating electric and magnetic field are produced by oscillating charged particles These fields are perpendicular to each other and both areperpendicular to the direction of propagation of the waveb They do not need a medium to travel That means they can even travel invacuum

Characteristics of electromagnetic radiationsa Wavelength It may be defined as the distance between two neighbouring crests or troughs of

element having the same atomicnumber but different mass numbere g 1H11H21H3

Isobars Isobars are the atoms of different elements having the same massnumber but different atomic numbere g 18Ar40 20Ca40

Isoelectronic species These are those species which have the same numberof electrons

Electromagnetic radiationsThe radiations which are associated withelectrical and magnetic fields are called electromagnetic radiations When anelectrically charged particle moves under acceleration alternating electricaland magnetic fields are produced and transmitted These fields aretransmitted in the form of waves These waves are called electromagneticwaves or electromagnetic radiations

wave as shown It is denoted by λb Frequency (ν) It may be defined as the number of waves which passthrough a particular point in one secondc Velocity (v) It is defined as the distance travelled by a wave in onesecond In vacuum all types of electromagnetic radiations travel with thesame velocity Its value is 3 times10 8m sec-1 It is denoted by v

d Wave number Wave number is defined as the number of wavelengths per unit lengthVelocity = frequency timeswavelength c = νλ

Plancks Quantum Theory- o The radiant energy is emitted or absorbed not continuously but discontinuously in the form of small discrete packets of energy called lsquoquantumrsquo In case of light the quantum of energy is called a lsquophotonrsquoo The energy of each quantum is directly proportional to the frequency of the radiation ie E α υ or E= hυ where h= Planckrsquos constant = 6626 x 10-27 Js o Energy is always emitted or absorbed as integral multiple of this uantum E=nhυ Where n=1234Black body An ideal body which emits and absorbs all frequencies is calleda black body The radiation emitted by such a body is called black body radiation

Photoelectric effectThe phenomenon of ejection of electrons from thesurface of metal when light of suitable frequency strikes it is calledphotoelectric effect The ejected electrons are called photoelectrons

Biology Chapter - 02Systematics and Five Kingdoms

Scientists divide the whole living organisms into two kingdom first and ultimately by five kingdom at last

In the earlier systems of classifications organisms are divided into kingdom plantaeand kingdom animalia on the of presenceof cell wall their modes of nutrition and movements

Some problem arise like fungi share manycharacteristic withplant despite their heterotrophic nutrition bacteria protozoa areunicellular present in both kingdom Toovercome this third kingdom Protista isintroduced which include

unicellularorganisms But there is also another

problem Allunicellular organisms are not similar kind The cellular structure of prokaryotes is verydifferent from that of other organismsEukaryotes possess a true nucleus and allcell organelles that are not present inprokaryotes So the fourth kingdom Monerais introduced which include unicellular prokaryotes (bacteriaamp blue green algae)

bull Still some problem arise in kingdomplantae

So in 1969 R H Whittakar proposedanew five kingdom System of classification

i) Kingdom Monera - unicellular prokaryotes

ii) kingdom Protista - unicellular eukaryotes

iii) Kingdom Fungi - uni or multicellular fungi with cell wall but without chlorophyll

iv) Kingdom Plantae - Multicellular Plants

v) Kingdom Animalia - Multicellular Animals

EVS Chapter 1 ndash Modes of Existence

An agricultural society

An agricultural society also known as an agrarian society is a society that constructs social order around a reliance upon farming More than half the people living in that society make their living by farming

People in an agricultural society generally lead a more settled lifestyle than those in nomadic hunter-gatherer or semi-nomadic pastoral societies because they live permanently near the land that is farmed Agricultural settlements tend to develop in areas of convenience near bodies of water which is used for both crops and transportation or along trade routes Not everyone in an agricultural society is a farmer Some people make a living trading or making and selling goods such as tools used for farming

Another way to define an agrarian society is to see the total amount of production in a nation In an agrarian society cultivating the land is the main source of wealth Such a society can recognize other means of subsistence and work habits but emphasizes the importance of agriculture and livestock Agrarian societies have existed in various parts of the world for 10000 years and continue to exist today They have been the most common form of socio-economic organization for most of recorded human history

Q) Write the features of agricultural society

Ans - Structure and Features of Agrarian Society1 Occupational Structure

An agrarian society is generally associated with the domestication of plants and animals The domestication of plants means farming and that of animals means herding Often there is mixture of farming and the use of such domesticated animals as cow goat and sheep

2 Forms of Land Ownership in Agrarian SocietiesGenerally there are landlords supervisory farmers cultivators and share croppers The landholders own the land but do not work on it They let it out for sharecropping The supervisory farmers are those who live by having their land cultivated by hired labourers The cultivators cultivate the land for themselvesThe share-croppers are those who live by tilling other peoplersquos land or a crop-sharing basis The artisans own their means of production and produce by their own labour in their homesteads

3 Village Community System An agrarian society is highlighted by

the institution of village community system The agrarian economy made fixed dwelling houses necessary Living close together for protection and co-operation and living nearer to the land gave birth to agricultural villages The village is not only the residential place of farmers it is also the social integrator

4 Minimal Division of Labour Another structural feature of agrarian society is a minimal division of labour Except for the basic division founded on age and sex differences there are few specialized roles There is only one predominant type of occupation ie domestication of plants and animals For all the people the environment physical as well as social is the same

5 Role of Family The farm family is of the patriarchal type the father is the final arbiter in most of the familyrsquos major decisions The life of ail men and women is merged in family life Since there are not many special organizations family is the only organisation to perform the tasks of aid and protection

6 Sense of Unity The members of an agrarian society exhibit a strong in-group feeling Since the whole of their social lives is wrapped up in a society which is physically economically and socially homogenous they are inclined to view the entire outside world as an out group

7 Informal Social Control An agrarian society is regionally divided into villages In a village community the force of traditional mores is more dominant than in the urban community In the village everybody is known to everybody The members in a village community help each other and share the joy and sorrows of each other Crime in an agrarian society is rare

8 Simplicity and Uniformity Life of the people in an agrarian society is marked by simplicity and uniformity Their main occupation is agriculture which largely depends upon the vagaries of nature An agrarian society is a religious society

Math Compound angles Compound angles The algebraic sum of two or more angles is called a compound angle If A B C be three angles then A+B B+C C+A A-B B-C A-C A+B-C etc are compound angles In this chapter we shall discuss the trigonometrical ratios of compound angles Theorem 1 If A B and A+B are all pisitive acute angles theni) sin( A+B) = sin A cos B + cosA sinBii) cos(A+B) = cosA cosB- sinA sinBTheorem 2If A and B are positive acute angles and AgtB theni) sin(A-B) = sin A cosB- cos A sinBii) cos(A-B) = cos A cos B+ sin A sin BTo prove that i) sin(A+B) sin (A-B) = sin2 A - sin2 B = cos2 B- cos2 A

Example 1 Prove that tan70deg=2tan50deg+tan20degSolutiontan70deg = tan(50deg + 20deg)Or tan70deg=(tan 50deg+tan 20deg)(1-tan50degtan20deg) or tan70deg (1 ndash tan 50deg tan20deg) = tan50deg+tan20degor tan70deg= tan70deg tan50deg tan20deg+ tan50deg + tan20deg = cot20deg tan50deg tan20deg + tan50deg + tan20deg = 2 tan50deg+ tan20degExample 2 If A + B = 45deg show that (1 + tanA) (1 + tanB) = 2Solutiontan(A + B) =( tan A + tan B) (1 - tan

ii) cos(A+B) Cos(A-B) = cos2 A- sin2 B = cos2 B -sin2 AProof i) LHS= sin(A+B)sin(AminusB) [Recall sin(αminusβ)=sinαcosβminuscosαsinβ And sin(α+β)=sinαcosβ+cosαsinβ]= (sinAcosB+cosAsinB)times(sinAcosBminuscosAsinB)= sin2Acos2Bminuscos2Asin2B [Recall sin2α+cos2α=1 From above we can then assume correctly that sin2α=1minuscos2α AND cos2α=1minussin2α] = sin2A(1minussin2B)minussin2B(1minussin2A) = sin2Aminussin2Asin2Bminussin2B+sin2Asin2B = sin2Aminussin2B= 1-cos2A-(1-cos2B) = cos2 B- cos2 A = RHSii)LHS= cos (A+B) cos(A-B) [ cos(A+B) = cos AcosB- sinAsinBCos(A-B) = cosAcosB+ sinAsinB]= cos2 A Cos2 B- sin2 A Sin2 B= cos2 A( 1-sin2 B) - (1- cos2 A) sin2 B= cos2 A- cos2 A sin2 B- sin2 B+ cos2 A sin2 B=cos2 A- sin2 B=1- sin2 A-(1-cos2 B) = cos2 B- sin2 A= RHSTangent formulae for compound anglesi)tan (A + B) = tan A + tan B1-tan A tan Bii)tan (A ndash B) = tan A-tan B1+tan A tan Biii) cot (A + B) = cot Acot B-1cot A+cot B(viii) cot (A ndash B) = cot Acot B+1cot B-cot A

A tan B) Or 1= (tan A+ tanB) (1-tan A tanB) Or tanA + tanB + tanA tanB + 1 = 1 + 1Or tanA (1 + tanB) + (1 + tanB) = 2Or (1 + tanA) (1 + tanB) = 2Example 3 Find the value of sin 15degSolution sin 15deg= sin(45deg-30deg) = sin45degcos 30deg- cos45degsin30deg =(1radic2) (radic32) -(1radic2) (12) = (radic3-1) 2radic2Example 4 If sin A = 1 radic10 and sin B = 1 radic5 where A and B are positive acute angles then what is A + B SolutionWe know that sin (A + B) = sin A cos B + cos A sin B= [1 radic10] [radic(1 minus 1 5)] + [1 radic5] radic(1 minus 1 10)= [1 radic10] [radic4 5] + [1 radic5] [radic9 10]= [1 radic50] times (2 + 3)= 5 radic50 = 1 radic2

sin (A + B) = sin π 4rArrHence A + B = π 4Example 5 If A + B = 225o then find [cot A] [1 + cotA] times [cot B] [1 + cot B]Solution[cot A] [1 + cotA] times [cot B] [1 + cot B] = 1 [(1 + tan A) times (1 + tan B)]=1 [tan A + tan B + 1 + tan A tan B] [ tan (A + B) = tan225o]∵

tan A + tan B = 1minus tan A tan BrArr= 1 [1 minus tan A tan B + 1 + tan A tan B]= 1 2

COMMERCE

CLASSIFICTION OF HUMAN ACTIVITIES-ECONOMIC AND NON-ECONOMIC

Firstly we shall recall the previous class for 5 mins especially for the absentees and for also the rest of the students who were there

Today at first we briefly discuss the earlier portions of the chapter

1Business-It includes all those economic activities which are concerned with production and exchange of goods and services with the object of earning profit Example A factory shop beauty parlour also business enterprises

2Profession ndashThe term profession means an occupation which involves application of specialized knowledge and skills to earn a living For Example Chartered Accountancy medicine law tax consultancy are example of professions

Questions1What are the main features of ProfessionAnswer The main features of a profession are as follows a Specialised body of knowledge-Every profession has a specialised and systematised body of knowledge b Restricted entry- Entry to a profession is allowed only to those who have completed the prescribed education and have the specialised examination c Formal education and training ndashA formal education and training is given to the person who wants to acquire the professional

3Employment-Employment mean an economic activity where people work for others in exchange for some remuneration (salary)The persons who work for others are called lsquoemployeesrsquo The persons or organizations which engage others to work for them are called lsquoemployersrsquoEg A doctor working in a hospital is employment as he is working for a salaryA lawyer may serve as a law officer in a bank

With this we shall proceed with the features of both Profession amp Employment

The main features of a profession are as follow

a Specialised body of knowledge b Restricted entry c Formal education and training d Professional association e Service motive f Code of contact

The main features of an employment are as follows

a In employment a person works for others called employer

b An employee provides personal service

c There is a service agreement or contract between the employee and the employer

d The employee has to obey the order of the employer

e No capital investment is made by the employer

Various examples of Employment are as follows

aA teacher teaching in a school or collegeb An engineer employed in Municipal Corporation of DelhicAn accountant working in the accounts department of a companydA doctor working in a hospital

Note In all the above examples of employment the individual who is involved in each example is working as an employee for a salary under an employer

qualification(MBBSCALLB)d Service motive ndashProfessionals are expected to emphasis service more on their clients rather than economic gain f Code of Conduct-The activities of professionals are regulated by a code of conduct

2 What are the main features of EmploymentAnswer The main features of an employment are as followsa In employment a person works for others called employerb An employee provides personal servicec There is a service agreement or contract between the employee and the employerd The employee has to obey the order of the employere No capital investment is made by the employer

3 Give various Professions and their respective Association are given below

Professions

Professional

Professional association

Medical profession

Doctor Medical Council of India

Law profession

Lawyers Bar Council of India

Accounting Profession

Chartered

The Institute of Chartered Accounts of India( ICAI)

Engineerin Engineers The

g Profession

institute of Engineers (India)

Accounts Basic accounting terms

Today we will give you some questions from the previous study material

Questions6) Define accounting7) What do you mean by debit

and credit8) Explain the types of account9) Define the following terms

a) Assetsb) Capitalc) Purchased) Debtorse) Transactions

10) Name the types of accounts given below

a) Krishnas accountb) Machinery accountc) Royalty accountd) Salary accounte) Furniture accountf) Audit fee account

Economics Basic Economic ConceptsSub topic

UTILITY

Before starting todayrsquos class we shall recall the last class which was about UTILITY AND THE FEATURES OF UTILITY

Now we shall proceed with the further topics of the chapter

Todayrsquos topic from the chapter lsquo Basic Economic Conceptsrsquo will be TOTAL UTILITY amp MARGINAL UTILITYNow let us quickly revise the concept of utility with an example ie goods and services are designed because they have an ability to satisfy human wantsThis feature of being able to satisfy human wants is termed as utility For example we derive utility from WiFi services as it gives us satisfaction by connecting us to our friends and family through social media here consumers derive utility from WiFi services

From the above concept we shall start with todayrsquos topicEconomists have defined TOTAL UTILITY (TU) as the total satisfaction obtained by consuming a given total amount of a good and serviceFor example the total satisfaction obtained from eating 10 mangoes is the total utility of 10 mangoes

MARGINAL UTILITY (MU) is the additional satisfaction derived from each additional unit

Questions1 What is Total Utility (TU)

Answer Total Utility (TU) is the

aggregate of the utility that a consumer derives from the consumption of a certain amount of a commodityTU=MU1+MU2++MUn

2 What is Marginal UtilityAnswer

Marginal Utility (MU) is the additional made to the total utility as consumption is increased by one more unit of the commodityMU= TUn ndashTUn-1

NoteOften economists tend to

subdivide utility into an imaginary unit called UTIL

consumed In this casethe utility obtained from each mango as it is consumed as the MU of that mango It is also defined as the addition made to the total utility when an additional unit is consumed Often economists tend to subdivide utility into an imaginary unit called UTIL

Note As a consumer increases the consumption of a good over period of time the total utility or total satisfaction derived from it increases to appoint and thereafter it decreasesHowever as the consumer keeps on consuming the good the marginal utility or the additional utility derived from it decreases

SubjectBusiness studies

Topic

BUSINESSENVIRONMENT

Summary

Now quickly let us revise the earlier points that we have already done in the last class and let us proceed with the other topics that are there in the chapter

Firstly we will recall the internal and external factors of micro environment and then we shall proceed in details

Meaning and list of internal and external factors

aInternal factorsInternal factors refer to all the factors existing within a business firm The internal factors are considered controllable because the enterprise has control over these factorsFor an example a company can alter its organization structure policies programmes employees physical facilities and marketing mix to suit the changes in the environmentList of internal factors areCorporate culture mission and objectives top management organizations structure company image and brand equity company resources

b External factorsExternal factors refer to those individual and groups and agencies with which a particular business organization comes into direct and frequent contact in the course of its functioningThese individuals and groups are known as STAKEHOLDERS because they have a stake (financial interest ) in the working and performance of the particular business List of external forces (stakeholders)Customers competitors investors suppliersmiddlemen (marketing intermediaries)

Execution 1 What do you mean by internal

factors in micro environmentAnswerInternal factors refer to all the factors existing within a business firm The internal factors are considered controllable because the enterprise has control over these factorsFor an example a company can alter its organization structure policies programmes employees physical facilities and marketing mix to suit the changes in the environment

2 What do you mean by external factors in micro environment

AnswerExternal factors refer to those individual and groups and agencies with which a particular business organization comes into direct and frequent contact in the course of its functioningThese individuals and groups are known as STAKEHOLDERS because they have a stake (financial interest) in the working and performance of the particular business

3Who are stakeholdersSTAKEHOLDERS are individuals and groups who have a stake (financial interest ) in the working and performance of the particular business 4Discuss the internal factors in briefa Corporate CultureThe values beliefs and attitudes of the founders and top management of the company exercise

financers publics

customers

suppliersfinancers

competitors

middlemen

publics

Fig STAKEHOLDERS OF A COMPANY

Apart from micro environment the other main dimension of business environment isMacro environment Macro environment refers to the general environment or remote environment within which a business firm and forces in its micro environment operateA company does not directly or regularly interact with the micro environmentTherefore macro environment is also known as indirect action EnvironmentThe macro environment forces are less controllable than the micro forces

Macro environment consists of the following components

POLITICAL AND LEGAL ENVIRONMENT

ECONOMIC SOCIAL AND ENVIRONMENT

CULTURAL

ENVIRONMENT

TECHNOLOGICAL ENVIRONMENT

a strong influence on what the cmpaany stands for how it does things and what it considers importantbMission and objectivesThe business philosophy and purpose of a comoany guide it prioritiesbusiness strategiesproduct market scope and development scope

cTop management structurethe composition of board of directors the degree of professionalization of management and the organizational structure of a company have important bearing on its business decisions

dPower structureThe internal power relationship between the board of directors and the chief executive is an important factor

eCompany image and brand equityThe image and brand equity of the company play a significant role in raising finance forming alliance choosing dealers and suppliers launching new products entering foreign markets

5 What is Macro environmentAnswerMacro environment refers to the general environment or remote environment within which a business firm and forces in its micro environment operateA company does not directly or regularly interact with the micro environmentTherefore macro environment is also known as indirect action EnvironmentThe macro environment forces are less controllable than the micro forces 6 What are the components of macro environmenta Political and legal environmentb Economic environmentc Social and cultural environmentd Technological environment

BUSINESS FIRM

Fig COMPONENTS OF MACRO ENVIRONMENTPolitical science

Introduction to political science

Comparative politics and itrsquos scope Comparative politics is the second major dimension of political scienceIt is also a very vast area of study and a very large number of political scientists even treat it as an autonomous area of study within the board ambit of political scienceScope of comparative politics-

1 All political structures -Comparative politics includes the study of all structures formalnon formal governmental and extra governmental which are directly or indirectly involved in politics in all the countries of the world

2 Functional studies- Comparative politics seeks to study politics less from the point of view of the legal institutions in terms of their powers and move from the point of view of their functions which constitute the political process and their actual Operation in the environment

3 Study of political behaviour- Another important part of its scope is the study of the actual behaviour of the people in the process of politics

4 Study of similarities and differences- comparative politics also undertakesan analysis of the similarities and differences among political process and functions

5 Study of all political systems -comparative politics seeks to analyse the actual behaviour and performance of all political systems western as well as non western

6 Study of the environment and infrastructure of politics-The study of politics demands a study of the psychological sociological economic and anthropological environment in fact the social environment as a whole in which each political system operates

7 Study of political culture- political culture is composed of attitudesbeliefs emotions and values of a society that relate to the political system or politics

8 Study of political participation- Political participation is a universal processThe only difference is that while in some states it is limited in others it is wider

9 Study of political process- political

Answer the following questions-

What is comparative politics

What are the scope of comparative politics

Homework- learn

processes like decision makingpolicy making judicial process leadership recruitment process and others are always at work in all political systems

The scope of comparative politics is very comprehensive It includes everything that falls within the area of political activity and political process

History CAMBRIDGE VIEW ABOUT

THE PARTITION

AND REFUTATION

OF CAMBRIDGE

VIEW

Cambridge view about the Partition The Cambridge school of historians have interpreted that opposition to partition scheme was made entirely by the elitist groups They hold the view that Lord Curzon planned to partition the Bengal for administrative purposeREFUTATION OFCAMBRIDGE VIEW The Rationalist historians have rejected the interpretations of the Cambridge School of historians on various grounds

1 QUESTION State different views of historians regarding Partition of Bengal

ANSWER Cambridge historians believed that Lord Curzon partitioned Bengal for administrative reasons only and not for the political motive The Middle class elitist group protested because of their petty interest The Hindu zamindars protested as they have to spend more money for managing their estatesThe lawyers of Calcutta High court feared to lose their clientBut according to the nationalist Historians was-

2- The ultimate object of Lord Curzon was to crush the unity of Bengal politicians

3- If Bengal becomes a separate province Bengali speaking 16 million people of western part would become minority under Hindi speaking people of Bihar and Oriya speaking people of Orissa

4- The bureaucrats expected that the protest movement would die down quickly

5- Lord Curzon used the Muslim community in his political game

6- Idealism had great contribution in the protest against partition

7- The people of the every section of society were affected by the partition of Bengal

Computer Science

Numbers Convertion of dcimal number to octal numberThe decimal numeral system is the standard system for denoting integer and non-integer numbers It is the extension to non-integer numbers of the Hindu-Arabic numeral system For writing numbers the decimal system uses ten decimal digits a decimal mark and for negative numbers a minus sign - The decimal digits are 0 1 2 3 4 5 6 7 8 9 the decimal separator is the dot in many countries

The octal numeral system or oct for short is the base-8 number system and uses the digits 0 to 7 Octal is sometimes used in computing instead of hexadecimal perhaps most often in modern times in conjunction with file

permissions under Unix systems It has the advantage of not requiring any extra symbols as digits It is also used for digital displays

Follow these steps to convert a decimal number into octal form

1 Divide the decimal number by 82 Get the integer quotient for the next iteration (if the number will not divide equally by 8 then round down the

result to the nearest whole number)3 Keep a note of the remainder it should be between 0 and 74 Repeat the steps until the quotient is equal to 05 Write out all the remainders from bottom to top This is the solution

For example if the given decimal number is 8453

Division Quotient Remainder

8453 8 1056 5

1056 8 132 0

132 8 16 4

16 8 2 0

2 8 0 2

Then the octal solution is 20405

Subject Eng Literature (The Tempest ndash William Shakespeare) Topic Act I Scene 1 Lines 33 to 67 (End of scene) Date 16th April 2020 (4th Period)

[Students should read the original play and also the paraphrase given in the school prescribed textbook]Summary Questions amp Answers

[SUMMARY OF THE ENTIRE SCENE]

o The play starts with the scene of a severe storm at sea Alonso (King of Naples) Sebastian (Alonsorsquos brother) Ferdinand (Alonsorsquos son) Gonzalo Antonio (the usurping Duke of Milan) are in a ship in the midst of the storm

o The mariners are trying their best to control the vessel from running aground and are totally following the orders of their Master the Boatswain They have scant success

o The mariners become extremely unhappy and annoyed when most of the passengers arrive on the deck thereby hampering their effort to save the ship There is serious confrontation between them and the passengers who are part of the Kingrsquos entourage

o The mariners could not save the ship

SUMMING-UP

(i) Vivid description of the scene which gives a realistic description of terror and confusion of a tropical storm

(ii) Shows Shakespearersquos accuracy of knowledge in describing the naval operations and also matters of seamanship

(1) GONZALO Ill warrant him for drowning (L 45-57)

though the ship were no stronger than a nutshell and as leaky as an unstanched

wenchBOATSWAIN Lay her a-hold a-hold Set her two courses Off to

sea again lay her offMARINERS All lost To prayers to prayers All lostBOATSWAIN What must our mouths be coldGONZALO The king and prince at prayers Lets assist them

For our case is theirsSEBASTIAN Im out of patienceANTONIO We are merely cheated of our lives by drunkards

This wide-chopped rascal - would thou mightst lie drowning the washing of ten tides

(a) What does Antonio say at the insolent manners of the boatswain just before the given passage

Being irritated at the insolent manners of the boatswain just before the given extract Antonio the Duke of Milan calls him a worthless dog son of a woman without any morals an arrogant and disrespectful noisemaker He says that the boatswain deserved to be hanged(b) What statement does Gonzalo repeat about the boatswain

Gonzalo shows his faith that the boatswain is not destined to die by drowning He is destined to be hanged and nothing can alter this decree of destiny He says that even if the ship was as frail as a nutshell the boatswain could not be drowned for his destiny was to be hanged(c) What do the passengers do when they have lost all hope of their survival

When the passengers have lost all hope of survival they take

(iii) The opening scene justifies the title ndash The Tempest

UNANSWERED QUESTIONS

(i) The King always travels with his entire fleet including his soldiers Where were the other ships

(ii) Why was the ship in that area Where was it coming from or going where

(iii) The ship broke apart What happened to those who were in the ship

(We shall get the answer to the above questions as the play progresses)

leave of life with fervent prayers The mariners take their last hearty drink and are ready for death(d) What blame does Antonio put upon the mariners and the boatswain Antonio rebukes the mariners that these drunkards have brought them to the present crisis by neglecting their duties He blames them saying that they are going to lose their lives entirely for the negligence of the boatswain and his fellows(e) What does Antonio say while cursing the boatswain

Antonio gives vent to his wrath upon the boatswain in particular He calls the boatswain a wide-mouthed rascal who deserves to be hanged on the sea-shore at low water mark so that ten tides might wash over his body and take out of him all the liquor that he has been drinking

Class XIISubject Topic Summary ExecutionHistory Topic

1 1935 ACT AND WORKING OF PROVINCIAL AUTONOMYCONGREE AND OTHER MINISTERSSUB TOPIC GOVERNMENT OF INDIA ACT1935

Government of India Act 1935 This act established a lsquoFederation of Indiarsquo made of British Indian provinces and Indian states and provided for autonomy with a government responsible to the elected legislature in every provinceThis act introduced abolition of Diarchy at provinces The entire provincial administration was introduced to the responsible ministers who were controlled and removed by the provincial legislature The provincial autonomy means two things First The provincial governments were wholly responsible to the provincial legislature Secondly Provinces were free from outside control and interference in the large number of matters The act divided the powers between the centre and provinces in terms of three lists- Federal list( for centre) Provincial list (for province) and concurrent list (for both) Residuary powers were given to the viceroy In the election under the government of India Act the Congress swept the poll the mandate of the people came in favour of the congress so far as general Hindu seats were concerned The Congress did not get a single Muslim seates in Bombay CP UP Sind and BengalIn five provinces Congress had yhe clear majority In BengalNWFPAssam and Bombay Congress emerged as a single largest partyOn the other side the performance of the Muslim League was badThus the Congress formed ministers in 7 provinces out of 11 provinces Coalition ministry was also formed in two other provincesOnly BENGAL AND Punjab had non- congress ministries

1 QUESTION What was the main change introduced by the Government of India ActANSWER a) The Act gave more

autonomy to the provinces b) Diarchy was abolished at the

provincial levelsc) The Governor was the head of

the executived) There was a council of

ministers to advise him The ministers were responsible to the provincial legislatures who controlled them The legislature could also remove the ministers

e) The Governors still retained special reserve powers

2 QUESTION Why did the federal scheme introduced by the Government of India Act 1935 never come into operation

ANSWER The Federal structure of the Government of India was to be composed with the Governor General and Council of ministers The Federal legislature was to be Bicameral legislature- The council of states and the House of Assembly The ministers were to be chosen by the Governor general and they were to hold the office during his pleasure

The provinces of British India would have to join the federation but this was not compulsory for the princely states

This federation never materialised because of the lack of support from the required number of

princely statesThis act was refused and

rejected by the princes the Congress and the Muslim League

Thus both Congress and the League participated in the election of 1937 Thus the federal part was never introduced but the provincial part was put into operations

Bengali 2nd

Language

াচেরর পরাথCনা(কহিতা )

াচেরর পরাথCনা কহিতাটি কহি (ঙখ দেঘাচে4র দো আচো য কহিতায় াচেরর পতর হমায়ন কঠিন দেরাচেগ আxানত ার ঈশবর া আললার কাচেছ পরাথCনা কচেরচেছন তার পচেতরর ীন হিফহিরচেয় হিচেত এই কহিতায় ার পচেতরর ীন হিভbা দেচেয়চেছন ারার এমনহিক হিনচের ীন হিসCচেনর হিহিনমচেয় হিতহিন তার দেছচের ীন হিফচের দেপচেত দেচেয়চেছন তার দেছচের এই দেরাচেগর ন য হিতহিন হিনচেচেকই ায়ী কচেরচেছন তার হিনচের করা পাপচেকই হিতহিন ায়ী কচেরচেছন এছাা রানৈনহিতক ও আথCসামাহিক অসথার কথা তচে ধরা চেয়চেছ এই কহিতায় ার তার হিনচের পাপ কমCচেকই ায়ী কচেরচেছ ার অন যায় ভাচে দেপহি((হিকতর মাধ যচেম অপররা য কচেরচেছ আর এই অন যায় কাচের ন যই তার পহিরাচের হিপযCয় এচেসচেছ দে এক পরকার মানহিক নধন ইহিতাচেসর ার হিপতা চেয় সবাভাহিকভাচে ভাচোাসা দে মমতা দেথচেক মকত চেত পাচেরনহিন তাই হিপতা চেয় আললা া ভগাচেনর কাচেছ পতর হমায়চেনর পরানহিভbা দেচেয়চেছন ার আললা া ভগাচেনর কাচেছ াহিনচেয়চেছন তার হিনচের ীন হিসCন হিচেত হিতহিন রাী তার হিহিনমচেয় পচেতরর ীন হিফচের দেপচেত দেচেয়চেছন াচেরর হিপতসভ হিচেকর কথা এই কহিতায় ফটিচেয় দেতাা চেয়চেছ হিপতা পচেতরর হিরাহিরত মান নধচেনর কথা তচে ধরা চেয়চেছ

হিচে(4 হিকছ াইচেনর তাৎপযC১) ldquoদেকাথায় দেগ ওর সবচছয দেৌন দেকাথায় কচেরায় দেগাপন bয়ldquoউততর) াচেরর পতর হমায়ন কঠিন দেরাচেগ অসসথ তাই তার দেযৌন াহিরচেয় যাচেচছ এই দেরাচেগ তাচেক দেগাপচেন কচেরকচের াচেচছ তার সক (হিকত ধীচের ধীচের bয় চেচছ তাই হিপতা চেয় ার আললার কাচেছ হমায়চেনর পরান হিভbা দেচেয়চেছন২) ldquoাগাও (চেরর পরাচেনত পরানতচের ধসর (ন দেযর আান গানldquoউততর) াচেরর পতর হমায়ন কঠিন দেরাচেগ আxানত তাই ার আ দে(াচেক মমCাত (চেরর পচেথ পরানতচের আান গান ধবহিনত দোক দেসই আান গান আললার কাচেছ দেযন চে যায় আললা দেযন এই আহিতC শচেন পচেতরর ীন হিফহিরচেয় দেয় ৩)ldquoনাহিক এই (রীচেরর পাচেপর ীানচেত দেকানই তরারণ দেনই ভহি4চেতরldquoউততর) হমায়চেনর অসসথতার ন য ার হিনচেচেকই ায়ী কচেরচেছন কারন ার অচেনক রা য অন যায় ভাচে কচেরচেছ তাই তার এই পাপ কাচের ন য তার ঘচের আ হিপ এচেসচেছ এই অন যায় কাচের ন য তার মহিকত দেনই তাই ার আললার কাচেছ এই পাপ কাচেযCর ন য bমা পরাথM

Hindi 2ndlang

-ासी(जयशकर परसा-)

-ासी जयशकर परसा- की एक ऐसी कहानी ह जिजसम भारतीय ससकनित और राषटरीयता का सवरगजीतहोता ह इस कहानी म इरावती एक निहद कनया ह जिजस मलअचछो न मलतान की लट म पकडा और -ासी बना दि-या उस 500 दि-न -कर काशी क एक महाजन न खरी-ा दसरी -ासी निफरोजा ह वह गलाम ह निफरोजा को छडान क कतिलए अहम- को 1000 सोन क कतिसकक भजन थ जो अभी तक नही आए थ राजा साहब कठोर होत हए भी निफरोजा को निबना धनराकतिश क कतिलए उस म कर -त ह वनिफरोजा को अहम- को समझान की बात कहत हकहानी क अत म हम -खत ह निक इरा वती और जाटो क सर-ार बलराज का मिमलन होता हअहम- को यa म मार दि-या जाता ह वहा निफरोजा की परसननता की समामिध बनती ह वहा एक फल चढती ह और डीजल आती ह निफरोजा उस समामिध की आजीवन -ासी बनी रहती हलखक अपन उददशय अथात -ास परथा पर परकाश डालन और इस परथा क कारण होन वाल -ातो क दखो को दि-खान म पणता सफल हए ह

helliphellipContinue to next

Biology Reproductio Today we will discuss about vegetative Q1 Name some vegetative propagules

n in Organisms

propagation of plants The process of multiplication in which fragments of plant body function as propagule and develop into new individual is called vegetative propagation The units of such propagation are runner rhizome tuber bulb etc

and the speciesinvolvedVegetative propagules

Parts involved

Bulb StemBulbil BulbilRhizome Stem Runner Stem Tuber Stem Offset Stem Leaf buds Leaves Suckers Stem

Corns Stem stolon

Q2 State advantages of vegetative propagation

i) Rapid methodii) Sure and easy methodiii) Useful in plants that cannot

produce viable seeds or long seed dormancy

iv) Maintains purity of raceQ 3 Banana fruit is said to be parthenocarpic where as turkey is said to be parthenogenetic WhyBanana develops without fertilization from an unfertilized ovary thus is parthenocarpicIn turkey the ovum or female gamete developinto a new chick without fertilization thus isparthgenetic

Q4 Why is water hyacinth is called as a ldquoTerror of Bengalrdquo Water hyacinth can

propagatevegetatively all over the water body in a short per short period of time This resulted increased biochemicaloxygen oxygen demand of water body causing mortalityof fishes It is very difficult to get rid off them Thus known as terror of Bengal

Chemistry

Solid state GENERAL CHARACTERISTICS OF SOLID STATEIn nature the particular state of matter is governed by two opposing forces at given set of temperature and pressure These forces are intermolecular force of attraction and thermal energy If intermolecular force of attraction is high as compared to thermal energy particles remains in closest position

Intext QuestionsQ1 Classify the following solids as crystalline and amorphous Sodium chloride quartz glass quartz rubber polyvinyl chloride Teflon

A1 Crystalline

and hence very less movement in particles is observed In this case solid state is the preferred state of matter

Let us revise the general characteristics of solid

i) Fixed mass volume and shape

ii) Strong intermolecular force of attraction

iii) Least intermolecular space

iv) Fixed position of constituent particles

v) Incompressible and rigid

Q2 what type of interactions hold the molecules together in a polar molecular solid[CBSE 2010]A2 The molecules in a solid are held together by van der Waals forces The term van der Waals forces include hydrogen bonding dipole-dipole attraction and London dispersion forces All molecules experience London dispersion forces In addition polar molecules can also experience dipole-dipole interactions So the interactions that holds the molecule together in polar molecular solid are London dispersion force and dipole-dipole interactionsQ3 Write a feature that will distinguish a metallic solid from an ionic solid [CBSE 2010]A3 Metals are malleable and ductile whereas ionic solid are hard and brittle Metallic solid has typical metallic lustre But ionic solid looks dullQ4 Write a point of distinction between a metallic solid and an ionic solid other than metallic lustre [CBSE 2012]A4 Metals are malleable and ductile whereas ionic solid are hard and brittleQ5 Write a distinguish feature of metallic solid [CBSE 2010]A5 The force of attraction in

solid Sodium chloride Quartz Amorphous solid Quartz glass rubber polyvinyl chloride Teflon Q2 why glass is considered as super cooled liquidA2 Glass shows the tendency to flow at slower rate like liquid Hence they considered as super cooled liquidQ3 why the window glass of old buildings show milky appearance with timeA3 Glass is an amorphous solid Amorphous solid has the tendency to develop some crystalline character on heating Due to heating in day over the number of years glass acquires some crystalline character and show milky appearanceQ4 why the glass panes fixed to window or doors of old building become slightly thicker at bottomA4 Glass is super cooled liquid It has the tendency to flow down very slowly Due to this glass pane becomes thicker at the bottom over the timeQ5 Sodium chloride is a crystalline solid It shows the same value of refractive index along all the direction TrueFalse Give reasonA5 FalseCrystalline solid shows anisotropy in properties That is it shows different values for the given physical property in different direction All the crystalline solids show anisotropy in refractive index Therefore sodium chloride will show different values of refractive index on different directions

Q6 Crystalline solid are anisotropic in nature What does this statement means

between the constituent particles is special kind of electrostatic attraction That is the attraction of positively charged kernel with sea of delocalized electronsQ6 which group of solid is electrical conductor as well as malleable and ductile [CBSE 2013]A6 Metallic solidQ7 why graphite is good conductor of electricity although it is a network (covalent solid)A7 The exceptional property of graphite is due to its typical structure In graphite each carbon is covalently bonded with 3 atoms in same layer The fourth valence electron of each atom is free to move in between different layersThis free electron makes the graphite a good conductor of electricity

[CBSE 2011]A6 Anisotropy is defined asrdquo Difference in properties when measured along different axis or from different directionsrdquo Crystalline solid show different values of some of the physical properties like electrical resistance refractive index etcwhen measured along the different directions The anisotropy in crystalline solid arises due to the different arrangement of particles in different directions

Math Function Composition of functions Think of an industrial plant that produce bottles of cold drinks first there is the operation (or function) f that puts the cold drink inside the bottle followed by the opeartion g that close the bottle with the capThis leads to the following definitionDefinition Let f A rarr B and g B rarr C be two functions Then the composition of f and g denoted by gof is defined as the function gof A rarr C given by gof(x) = g(f (x)) forall x isinA

Definition A function f X rarr Y is defined to be invertible if there exists a function g Y rarr X such that gof = IX and fog = IY The function g is called the inverse of f and is denoted by f -1

Thus if f is invertible then f must be one-one and onto and conversely if f is one-one and onto then f must be invertible This fact significantly helps for proving a function f to be invertible by showing that f is one-one and onto specially when the actual inverse of f is not to be determined

Example 1 Let f 2 3 4 5 rarr 3 4 5 9 and g 3 4 5 9 rarr 7 11 15 be functions defined as f(2) = 3 f(3) = 4 f(4) = f(5) = 5 and g (3) = g (4) = 7 and g (5) = g (9) = 11 Find gofSolution We have gof(2) = g (f(2)) = g (3) = 7 gof(3) = g (f(3)) = g (4) = 7gof(4) = g (f(4)) = g (5) = 11 and gof(5) = g (5) = 11Example 2 Find gof and fog if f R rarr R and g R rarr R are given by f(x) = cos x and g (x) = 3x2 Show that gof ne fogSolution We have gof(x) = g(f(x))=g(cosx) = 3 (cos x)2

= 3 cos2 x Similarly fog(x)=f(g (x))= f(3x2)= cos (3x2) Note that 3cos2 x ne cos 3x2 for x = 0 Hence gof ne fogExample 3 Show that if f A rarr B and g B rarr C are onto then gof A rarr C is also ontoSolution Given an arbitrary element z isin C there exists a pre-image y of z under g such that g (y) = z since g is onto Further for y isin B there exists an element x in A with f(x) = y since f is onto Therefore gof(x) = g (f(x)) = g (y) = z showing that gof is onto Example 4 Let Y = n2 n isin N sub N Consider f N rarr Y as f(n) = n2 Show that

f is invertible Find the inverse of fSolution An arbitrary element y in Y is of the form n2 for some n isin N This implies that n =radicy This gives a function g Y rarr N defined by g (y) =radicy Nowgof (n) = g (n2)=radicn2 = n and fog (y) =f(radicy) = (radicy) 2 y which shows that gof=IN and fog= IY Hence f is invertible with f -1 = g

Political Science

Constitution of India-The Preamble

Summary

Objective of the state-To secure equality of status and of opportunity To promote fraternity among all the citizens To assure the dignity of the individuals and Unity and integrity of the nation

Justice-Justice stands for rule of law absence of arbitrariness and a system of equal rights freedom and opportunities for all in a society India seeks social economic and political justice to ensure equality to its citizens

Liberty-Liberty implies the absence of restraints or domination on the activities of an individual such as freedom from slavery serfdom imprisonment despotism etc The Preamble provides for the liberty of thought expression belief faith and worship

Equality-Equality means the absence of privileges or discrimination against any section of the society The Preamble provides for equality of status and opportunity to all the people of the country

Fraternity-The Preamble declares that fraternity has to assure two thingsmdashthe dignity of the individual and the unity and

Execution

Answer the following questions-

Short notes-1 Equality2 Fraternity3 Justice4 Liberty

Homework-Learn

integrity of the nation The word integrity has been added to the Preamble by the 42nd Constitutional Amendment (1976)

Business studies

Human resource management (chapter 1)

On the day of 1504 2020 I have discussed with you the managerial functions and procurement functions of HRM

Today weare going to discuss about the development function integration functions and maintenance function

Development functions-HRM improves the knowledge skills attitude and values of employees so that they the present and future jobs more effectively it includes

1) Development functions of HRM

a) Performance appraisal = It implies systematic evaluation of employees with respect to their performance on the job and their potential for development

b) Training =It is the process by which employees learn knowledge skills and attitudes to achieve organisational and personal goals

c) Executive development = It is the process of developing managerial talent through appropriate program

2) Integration functionsa) HRM reconcile the goals of

organisation with those of its members through integrating function

b) HRM tries to motivate employees to various financial and non financial incentives provided in job specification etc

3) Maintenance functiona) HRM promote and protect the

physical and mental health of employees by providing several types of benefits like housing medical aid etc

b) It Promote Social security measures to employees by providing provident fund pension gratuity maternity benefits

SubjectCOMMERCE

Topic

BUSINESSENVIRONMENT

Summary

Now quickly let us revise the earlier points that we have already done in the last class and let us proceed with the other topics that are there in the chapter

Firstly we will recall the internal and external factors of micro environment and then we

Execution 3 What do you mean by internal factors

in micro environmentAnswerInternal factors refer to all the factors existing within a business firm The internal factors are considered controllable because the enterprise has control over these factors

Development FunctionsPerformance AppraisalTrainingExecution Development

shall proceed in details

Meaning and list of internal and external factors

aInternal factorsInternal factors refer to all the factors existing within a business firm The internal factors are considered controllable because the enterprise has control over these factorsFor an example a company can alter its organization structure policies programmes employees physical facilities and marketing mix to suit the changes in the environmentList of internal factors areCorporate culture mission and objectives top management organizations structure company image and brand equity company resources

b External factorsExternal factors refer to those individual and groups and agencies with which a particular business organization comes into direct and frequent contact in the course of its functioningThese individuals and groups are known as STAKEHOLDERS because they have a stake (financial interest ) in the working and performance of the particular business List of external forces (stakeholders)Customers competitors investors suppliersmiddlemen (marketing intermediaries)financers publics

customers

suppliersfinancers

For an example a company can alter its organization structure policies programmes employees physical facilities and marketing mix to suit the changes in the environment

4 What do you mean by external factors in micro environment

AnswerExternal factors refer to those individual and groups and agencies with which a particular business organization comes into direct and frequent contact in the course of its functioningThese individuals and groups are known as STAKEHOLDERS because they have a stake (financial interest) in the working and performance of the particular business

3Who are stakeholdersSTAKEHOLDERS are individuals and groups who have a stake (financial interest ) in the working and performance of the particular business 4Discuss the internal factors in briefa Corporate CultureThe values beliefs and attitudes of the founders and top management of the company exercise a strong influence on what the cmpaany stands for how it does things and what it considers importantbMission and objectivesThe business philosophy and purpose of a comoany guide it prioritiesbusiness strategiesproduct market scope and development scope

cTop management structurethe composition of board of directors the degree of professionalization of management and the organizational structure of a company have important bearing on its business decisions

dPower structureThe internal power relationship between the board of directors and the chief executive is an important factor

e Company image and brand equityThe image and brand equity of the company play a significant role in raising finance forming alliance choosing dealers and suppliers launching new products entering foreign markets

5 What is Macro environmentAnswerMacro environment refers to the general

competitors

middlemen

publics

Fig STAKEHOLDERS OF A COMPANY

Apart from micro environment the other main dimension of business environment isMacro environment Macro environment refers to the general environment or remote environment within which a business firm and forces in its micro environment operateA company does not directly or regularly interact with the micro environmentTherefore macro environment is also known as indirect action EnvironmentThe macro environment forces are less controllable than the micro forces

Macro environment consists of the following components

POLITICAL AND LEGAL ENVIRONMENT

ECONOMIC SOCIAL AND ENVIRONMENT

CULTURAL

ENVIRONMENT

TECHNOLOGICAL ENVIRONMENT

Fig COMPONENTS OF MACRO ENVIRONMENT

environment or remote environment within which a business firm and forces in its micro environment operateA company does not directly or regularly interact with the micro environmentTherefore macro environment is also known as indirect action EnvironmentThe macro environment forces are less controllable than the micro forces 6 What are the components of macro environmenta Political and legal environmentb Economic environmentc Social and cultural environmentd Technological environment

Computer Science

Logic gates

Digital systems are said to be constructed by using logic gates These gates are the AND OR NOT NAND NOR EXOR and EXNOR

BUSINESS FIRM

gates The basic operations are described below with the aid of truth tables

AND gate

The AND gate is an electronic circuit that gives a high output (1) only if all its inputs are high A dot () is used to show the AND operation ie AB Bear in mind that this dot is sometimes omitted ie ABOR gate

The OR gate is an electronic circuit that gives a high output (1) if one or more of its inputs are high A plus (+) is used to show the OR operationNOT gate

The NOT gate is an electronic circuit that produces an inverted version of the input at its output It is also known as an inverter If the input variable is A the inverted output is known as NOT A This is also shown as A or A with a bar over the top as shown at the outputs The diagrams below show two ways that the NAND logic gate can be configured to produce a NOT gate It can also be done using NOR logic gates in the same way

NAND gate

This is a NOT-AND gate which is equal to an AND gate followed by a NOT gate The outputs of all NAND gates are high if any of the inputs are low The symbol is an AND gate with a small circle on the output The small circle represents inversion

NOR gate

This is a NOT-OR gate which is equal to an OR gate followed by a NOT gate The outputs of all NOR gates are low if any of the inputs are highThe symbol is an OR gate with a small circle on the output The small circle represents inversion

EXOR gate

The Exclusive-OR gate is a circuit which will give a high output if either but not both of its two inputs are high An encircled plus sign ( ) is used to show the EOR operation

EXNOR gate

The Exclusive-NOR gate circuit does the opposite to the EOR gate It will give a low output if either but not both of its two inputs are high The symbol is an EXOR gate with a small circle on the output The small circle represents inversion The NAND and NOR gates are called universal functions since with either one the AND and OR functions and NOT can be generated

Note A function in sum of products form can be implemented using NAND gates by replacing all AND and OR gates by NAND gates A function in product of sums form can be implemented using NOR gates by replacing all AND and OR gates by NOR gates

Logic gate symbols

Table 2 is a summary truth table of the inputoutput combinations for the NOT gate together with all possible inputoutput combinations for the other gate functions Also note that a truth table with n inputs has 2n rows You can compare the outputs of different gates

Logic gates representation using the Truth table

Example

A NAND gate can be used as a NOT gate using either of the following wiring configurations

Subject Eng Literature (The Tempest ndash William Shakespeare) Topic Act III Scene 3 Lines 53 to 110 (End of the scene) Date 16th April 2020 (2nd Period)

[Students should read the original play and also the paraphrase given in the school prescribed textbook]Summary Questions amp Answers

o Seeing this strange scene all are inclined to believe the tales told by travelers that there truly are ldquounicornsrdquo and ldquothe phoenixrsquo thronerdquo

o As they are about to sit down to the feast the banquet is snatched away by a harpy (Ariel disguised) A spiritrsquos voice (Arielrsquos voice) denounces Alonso Sebastian and Antonio with particular

1 ARIEL You are three men of sin whom Destiny

(Line 53-58)That hath to instrument this

lower world And what is int the never-surfeited sea

Hath caused to belch up you and on this island

Where man doth not inhabit you rsquomongst men

Being most unfit to live I have made you mad

reference to their crime in expelling Prospero from Milan They have not received any punishment for their deed earlier but the time for their punishment has arrived Upon Alonso it pronounces ldquolingering perdition worse than deathrdquo from which there is no remedy except through sincere repentance Ariel then vanishes in thunder and the shapes enter again and carry away the table

o Prospero watching invisibly is very pleased with the performance of Ariel and his (Prosperorsquos) ldquomeaner ministersrdquo All his enemies are now in his power and are in a fit of desperation He then leaves them and goes to see how Ferdinand and Miranda are getting on

o Alonso is now much humbled and penitent with the after effect of the spiritrsquos denunciation of his crimes He believes that his son is lost forever After this all disperse being stricken mad by the speech of the spirit

o Gonzalo fearing that they may do violence to themselves or to one another follows them and bid others to follow

(a) To whom does Ariel disguised as a harpy call the three sinners What game did Fate of Destiny play with

them

The three sinners called by Ariel are Alonso Sebastian and Antonio It was Destiny which had caused the ocean to cast the three sinners on the shore Though the ocean is all the time devouring whatever appears on its surface and is never satisfied with its continual swallowing of the ships and men in the present case the ocean had cast these three sinners on the shore without killing them

(b) Who had jointly been responsible for the conspiracy against Prospero What is Prosperorsquos purpose behind all this

Three men Alonso Sebastian and Antonio had jointly

been responsible for the conspiracy against Prospero They had driven out Prospero form Milan Prosperorsquos purpose is to make these three sinners realize the wrong they had done He wants them to repent for their criminal deeds because repentance leads to self-esteem(c )What does Ariel (the harpy) tell Alonso and his companions when they take out their swords to attack him

Seeing them drawing their swords Ariel (harpy) tells them that he and his companions are the instruments of destiny and that it is not possible for human beings to do them any injury He says that the swords of human beings can not injure even a minute part of his feathers Their swords are as ineffective against him and his companions as against the wind or the water

(d) Give the explanatory meanings of the following expressions in the context of the above extract

(i)Never surfeited (ii) Belch up (iii) lsquomongst men

(i) Never surfeited never led to satisfaction

(ii) Belch up cast ashore(iii) lsquomongst men in human

society2

I and my fellows (Line 60-65)

Are ministers of Fate The elementsOf whom your swords are tempered may as wellWound the loud winds or with bemocked-at stabsKill the still-closing waters as diminishOne dowl thats in my plume

IMPORTANT PASSAGES EXPLAINED

The elements

(Line 61-66)Of whom your swords are tempered may

as wellWound the loud winds or with

bemocked-at stabs

(a) Who is lsquoIrsquo Who are his lsquofellowsrdquo

lsquoIrsquo is referred to Ariel in disguise of a harpy His lsquofellowsrsquo are other spirits serving Prospero the real Duke of Milan who has acquired supernatural powers after being banished from his Dukedom Prospero has settled in this uninhabited island

(b) What are the elements that have temperrsquod the swords Why will it not work against the speaker

The swords (of Alonso and his companions) are tempered by metal (steel) which is taken out of the earth and refined by

Kill the still-closing waters as diminishOne dowl thats in my plume My fellow

ministersAre like invulnerable

In these words Ariel reminds the King and his companions of the utter futility of drawing swords against himself and his fellows Ariel drives Alonso Antonio and Sebastian the three men of sin to desperation ndash a state in which men do violence to themselves They draw swords to strike Ariel But Ariel reminds them that he and the other spirits are the ministers of destiny and nothing can wound them The steel of which their swords are made of may cut the wind or water which being divided always closes up again Even supposing that such things may be possible it is quite impossible that their swords will cut one feather in their plume They are incapable of being wounded by any sword of man Hence it is foolish on their part to attempt to strike at Ariel and his fellow-spirits

For which foul deed

(Line 72-75)The powers delaying not forgetting

haveIncensed the seas and shores yea all the

creatures Against your peace

Ariel enters like a harpy and remaining invisible tells Alonso Sebastian and Antonio that he and other harpies are the agents of Destiny appointed to carry out her decrees He tells them that their punishment for the crime against Prospero which has been so long deferred is now to fall upon them He reminds them that they had expelled Prospero from Milan and set him and his innocent child adrift on the sea and that the sea had paid them back for their sin by the shipwreck and by the calamities they have suffered He tells them that the powers above which did not forget this mean treachery but only deferred the punishment have now engaged the seas and the shores and all living beings including him and his comrades against them The very elements and supernatural agency Ariel adds have taken up the avenging of their crime against Prospero

the action of fire It may cut the wind or water which being divided always closes up again

The sword will not work against the spirits and the harpy because they are the ministers of destiny and nothing can wound them nor it will cut a single feather in their plume

(c )What is the meaning of lsquodowlrsquo in the last line

The term lsquodowlrsquo means a filament or the smallest part of a feather In this context Ariel in disguise of harpy says that their sword cannot even damage the smallest filament of their (Arielrsquos and other spirits) feathers as they are incapable of being wounded by any sword of man

(d) What does the speaker remind the listeners about

Ariel in disguise of harpy reminds Alonso the King of Naples Sebastian Alonsorsquos brother and Antonio the present Duke of Milan and the treacherous brother of Prospero as they being three men of sin He even reminds them that their punishment for their crime against Prospero which has been so long deferred now falls upon them He reminds them that they have expelled Prospero from Milan and has set him along with his innocent infant daughter adrift on the sea So the sea has paid them back for their sin by their shipwreck and the calamities they have suffered since then The harpy rebukes Alonso of his sin that has incensed the Gods and has deprived him of his son as a punishment

(e) How do they respond

When Ariel in disguise of a harpy reminds Alonso Sebastian and Antonio of their past misdeeds and sin Alonso has a look of terror and confusion in his eyes He utters the words of sincere repentance wrung out of his conscience-stricken heart It appears to him that all the elements of nature the sea-waves the wind and the thunder proclaiming a loud voice in the name of Prospero and the crime Alonso has committed against him They are calling upon him to repent There is a deep storm raging in Alonsorsquos breast and the echoes of that storm are ringing in his ears like a clear note of wind-instrument A note of denunciation of Alonsorsquos crime leaves him much humbled and penitent and confirms his belief that his son is lost forever But Sebastian and Antonio shows some courage instead of repentance They wish to kill the spirits or devils if it appears

3

Of my instruction hast thou nothing bated (Line 85-93)

In what thou hast to say So with good life

And observation strange my meaner ministers

Their several kinds have done My high charms work

And these mine enemies are all knit upIn their distractions They now are in my

powerAnd in these fits I leave them while I visitYoung Ferdinand whom they suppose is

drownedAnd his and mine loved darling

Methought the billows spoke and (Line 96-99)

told me of itThe winds did sing it to me and the

thunderThat deep and dreadful organ-pipe

pronouncedThe name of Prosper It did bass my

trespass

These are the words of contrition coming from Alonso Ariel has driven him to a deep repentance for conspiring with Antonio against Prospero He now feels a sincere remorse It appears to him that all the elements of nature the sea-waves the wind and the thunder proclaimed with a loud voice the name of Prospero and the crime Alonso had committed against him They are calling upon him to repent There is a deep storm raging in Alonsorsquos breast and the echoes of that storm are ringing in his ears like the clear note of a wind-instrument

Comment These are the words of sincere repentance wrung out of the conscience-stricken heart of Alonso Alonso who is the lesser villain is the first to give way to remorse under the effect of Arielrsquos speech The words of Ariel seem to him to be the voice of conscience speaking to him He is driven to desperation a state in which he might do violence to his life

(a) Identify the speaker State the context

Prospero the ruler of the island is the speaker The famous banquet scene has been enacted very well Ariel and his junior spirits have played their roles excellently Prospero is glad to say words of praise for them(b) In what way the speakerrsquos instructions have been carried out

According to Prosperorsquos instructions a banquet was presented before the King of Naples and his companions when they were tired and hungry Just when they were preparing to eat the feast the banquet was suddenly removed by exercising supernatural powers All this was done by Ariel Prosperorsquos chief assistant and a powerful spirit

Ariel not only made the feast disappear but also delivered his speech blaming the King and his two companions for their past wicked deeds He warned them to repent for their misdeeds or suffer forever on that uninhabited island

(c) Who are referred to as lsquomeaner ministersrsquo What have they done

Prospero refers as lsquomeaner ministersrsquo to his other lesser spirits who were assisting Ariel in presenting a scene before the kingrsquos party They entered the scene to the accompaniment of music They assumed several strange shapes and brought in a banquet Then they danced about it with gentle actions of salutations thus inviting the King and others to eat the feast

These spirits play their role again when Ariel in the shape of a harpy quits the scene These shapes enter again and dancing with mocking gestures carry away the table

(d) Who are the speakerrsquos enemies What has happened to them

King of Naples Alonso his brother Sebastian and the present Duke of Milan Antonio (Prosperorsquos own brother) are Prosperorsquos enemies With the turn of events they have all been washed ashore on the island which is ruled by Prospero the great magician Actually this happened after the shipwreck caused by a storm which was raised by Prospero with the purpose of bringing these people to his island Prosperorsquos spirits have already confused and terrified these enemies and they are under Prosperorsquos control He can treat them as he likes

(e) What does he say about Ferdinand Explain what is meant by ldquohellip his and mine darlingrdquo

Prospero knows that Alonsorsquos son prince Ferdinand is alive though his father thinks that the prince has been drowned

Prospero refers to his daughter Miranda who is dear to him She is also very dear to Prince Ferdinand who has fallen in love with her They are waiting to be married soon for which they have received Prosperorsquos consent

4

ALONSO O it is monstrous monstrous (Line 95-102)

Methought the billows spoke and told me of it

The winds did sing it to me and the thunderThat deep and dreadful organ-

pipe pronouncedThe name of Prosper It did bass

my trespassTherefore my son ithrsquo ooze is

bedded andIll seek him deeper than eer

plummet soundedAnd with him there lie mudded

(a) In what way does Alonso express his horror when his conscience is awakened by Arielrsquos words

When Alonsorsquos conscience is awakened by Arielrsquos words he expresses his horror at what he has heard He gets the feeling that the waves of the ocean the wind and the loud thunder have spoken to him and uttered the name of Prospero Because of being reminded of his crime in a very loud and rough voice he comes to realize that he has lost his son for his past misdeeds

(b) What does Alonso imagine about his son What does Alonso want to do in his desperate state

Alonso imagines that his son is lying in the mud at the bottom of the sea He feels desperate that he wants to drown himself in the ocean deeper than the plumb-line has ever gone He wants to lie with his son at the bottom of the sea

(c) How do Sebastian and Antonio want to face the evil spirits

Sebastian says that he is not at all afraid of what the harpy has said and that he is prepared to fight any number of such monsters if they appear before him only one at a time Antonio says that he would support Sebastian in the fight against the fiendsyyy

(d) Why does Gonzalo ask Adrian to follow the three men

Gonzalo tells Adrian that all the three men namely Alonso Sebastian and Antonio are in a wild and reckless mood The thought of the heinous crime of which they are guilty has begun to torment their minds So he asks Adrian to follow those three men without loss of time and prevent them from doing anything which the turmoil in their minds might lead them to do

(e) What opinion do you form of Alonso from the above extract

Alonso who is the lesser villain is the first to give way to remorse under the effect of Arielrsquos speech The words of Ariel seem to him to be the voice of conscience speaking to him He is driven to desperation a state in which he might do violence to his life

Subject =Accounts

Ac-12 15420 topic-pL Appropriation ac

PROFIT AND LOSS APPROPRIATION ACCOUNT

MEANING AND PREPARATIONProfit and Loss Appropriation Account is merely an extension of the Profit and Loss Account of the firm The profit of the firm has to be distributed amongst the partners in their respective profit sharing ratio But before its distribution it needs to be adjusted All Adjustments like partnerrsquos salary partnerrsquos commission interest on capital interest on drawings etc are made in this account These adjustments will reduce the amount of profit for distribution This adjusted profit will be distributed amongst the partners in their profit sharing ratio To prepare it at first the balance of Profit and Loss Account is transferred to this account The journal entries for the preparation of Profit and Loss Appropriation Account are given below

1 for transfer of the balance of Profit and Loss Account to Profit and Loss Appropriation Account

(a) In case of Net Profit

Profit and Loss Ac helliphelliphelliphelliphellipDrTo Profit and Loss Appropriation Ac(Net Profit transferred to Profit and Loss Appropriation Ac)

(b)In case of Net Loss

Profit and Loss Appropriation Achelliphelliphellip DrTo Profit and Loss Ac(Net Loss transferred to Profit and Loss Appropriation Ac)

2 for Interest on Capital

For transferring on Interest on CapitalProfit and Loss Appropriation Achelliphelliphellip DrTo Interest on Capital Ac(Interest on capital transferred to Profit amp Loss Appropriation Ac)

3 for Interest on Drawings

For transferring Interest on Drawings Interest on Drawings Achelliphelliphelliphelliphelliphellip DrTo Profit and Loss Appropriation Ac(Interest on drawing transferred to Profit amp Loss Appropriation Ac)

4 For Partnerrsquos SalaryFor transfer of partnerrsquos SalaryProfit and Loss Appropriation Achelliphellip DrTo Salary Ac(Salary transferred to profit amp Loss Appropriation Ac)

5 For Partnerrsquos CommissionFor transferring commissionProfit and Loss Appropriation Achelliphelliphellip DrTo Commission Ac(Commission transferred to Profit and Loss Appropriation Ac)

6 For Transfer of agreed amount to General ReserveProfit and Loss Appropriation Ac helliphellipDrTo General Reserve Ac(Transfer to General Reserve)

7 for share of Profit or Loss appropriation(a) If ProfitProfit and Loss Appropriation Achelliphellip DrTo Partnerrsquos CapitalCurrent Ac(Profit transferred to capitalcurrent Ac)(b) If LossPartnerrsquos Capital Current Achelliphelliphelliphellip DrTo Profit and Loss Appropriation Ac(Loss transferred to capitalcurrent Ac)

THE FORMAT OF PROFIT AND LOSS APPROPRIATION

Profit and Loss Appropriation Account for the year endedhelliphelliphelliphellip

Particulars Amount Particulars Amount

To PL Ac (loss) By pL Ac (profit)

To Interest on capital BY Interest on drawings

To partner`s commission by Partner`s capital Ac ( loss)

To Partner`s salary To Interest on partner`s loan To General Reserve To Partner`s Capital AC (Profit)

Subject= Economics

MOVEMENT ALONG THE DEMAND CURVE (CHANGE IN QUANTITY DEMANDED)In law of demand you have already studied the inverse relationship between price and quantity demanded When quantity demanded of a commodity changes due to change in its price keeping other factors constant it is called change in quantity demanded It is graphically expressed as a movement along the same demand curve There can be either a downward movement or an upward movement along the same demand curve Upward movement along the same demand curve is called contraction of demand or decrease in quantity demanded and downward movement along the same demand curve is known as expansion of demand or increase in quantity demanded

Extention of demandd

price (rs)p A

B Extentionp1 d

Q Q1

Quantity demanded ( in units)

Contraction of demandd

p2 Ccontraction

p APrice (Rs)

d

Q2 Q

Quantity demanded (in units)

Explanation of movement of demand A fall in price from OP to OP1 leads to increase in quantity demanded from OQ to OQ1 (expansion of demand) resulting in a downward movement from point A to point B along the same demand curve DD When Price rises from OP to OP2 quantity demanded falls from OQ to OQ2 (contraction of demand) leading to an upward movement from point A to point C along the same demand curve DD

  • Activity Series of Metals
    • Drawbacks of Rutherfordrsquos model of atom
      • Electromagnetic radiations
      • Properties of electromagnetic radiations
      • Characteristics of electromagnetic radiations
        • Plancks Quantum Theory-
        • Photoelectric effect
          • Intext Questions
            • Logic gates
            • Digital systems are said to be constructed by using logic gates These gates are the AND OR NOT NAND NOR EXOR and EXNOR gates The basic operations are described below with the aid of truth tables
            • AND gate
            • Example
Page 24:  · Web viewSubject. Topic. Summary. Execution. English 1 . Chapter 1 naming words . Page 8. Write the names of these pictures:- Person:-1. father. 2.Firefighter 3.doctor 4 ...

Land is defined to include not only the surface of the earth but also all other free gifts of nature(for example mineral resources forest resources and indeed anything that helps us to carry out the production of goods and services but is provided by nature free of cost)

We will move on to the last portion of land by discussing Productivity of Land

By productivity of land we mean the capacity of a piece of land to produce a crop

Thus it refers to the average output per unit of land

Say per acre per hectare etc= (OutputArea of land)

With this we shall proceed further with the main factors that determine the productivity of land

Natural factors Human factors Improvements on land Location of land Organisation Ownership of land Availability of capital Proper use of land State help

Note economic development of a country depends upon the quality of its land If the land is fertile it will quicken the pace of development of the country

qualities of land such as fertility etc

Human factors Land cannot produce anything by itself Man has to apply labour on it to produce for himself So productivity of land depends on the knowledge and skills of workers

Improvements on land production of land is affected by land development measures like provision of well or tubewell irrigation proper drainage

State help The government of a country especially less developed country can play a vital role in improving the agricultural productivity by providing better irrigation facilities

Organisation Productivity of land also fdepends upon the way how the factors of production like labour and capital are organised

In order to increase productivity trained workers modern implements scientific methods good seeds are all essential

3 lsquoImproved technology affects the productivity of landrsquo Explain this statement with the help of suitable example Answer Use of improved technology raises the productivity of land Example By using HYV seeds chemical manures and modern machines per hectare output increases

Physics Force (Summary)

Question Write the expression for the moment of force about a given axisSolutionsThe expression for the moment of force is given byMoment of force about a given axis = Force times perpendicular distance of force from the axis of rotationQuestion What do you understand by the clockwise and anticlockwise moment of force When is it taken positiveSolutionsIf the effect on the body is to turn it anticlockwise moment of force is called the anticlockwise moment and it is taken as positive while if the effect on the

body is to turn it clockwise moment of force is called the clockwise moment and it is taken as negative

Math Topic Commercial Mathematics

Chapter Goods and services Tax

Study item Some solved sums from exercise ndash 1 A retailer buys a TV from a wholesaler for Rs 40000 He marks the price of the TV 15 above his cost price sells it to the consumer at 5 discount on the marked price If the sales are intra ndash state and the rate of GST is 12 find

(i) The marked price of the TV(ii) The amount which the consumer pays for the TV(iii) The amount of tax (under GST) paid by the retailer to the central

Government(iv) The amount of tax (under GST) received by the State Government

Solution As the sales are intra- state sale and the rate of GST 12 So GST comprises of 6 CGST and 6 SGSTTherefore a retailer buys a TV from a wholesaler for Rs 40000Therefore the amount of GST collected wholesaler from the retailer or paid by retailer to wholesalerCGST = 6 of Rs 40000 = Rs(6100 times40000) =Rs 2400SGST = 6 of Rs 40000 = Rs (6100 times 40000) =Rs 2400Therefore wholesaler will pay Rs 2400 as CGST and Rs 2400 as SGSTTherefore amount of input GST of retailer Input CGST = Rs 2400 and input SGST = Rs 2400Again the retailer marks the price of the TV 15 above his cost price(i) The marked price of the TV

= Rs 40000 + Rs 40000times15= Rs 40000 + Rs 40000times 15100= Rs 40000 + Rs 6000Rs 46000But the retailer sells it to consumer at 5 discount on the marked priceCost price after discount = Rs 46000 ndashRs46000times 5100 =Rs 46000 ndashRs 2300= Rs 43700Therefore the amount of GST collected retailer from consumer or paid by consumer to retailerCGST = 6 of Rs 43700 =Rs ( 6100 times43700)Rs 2622SGST = 6 of Rs 43700 = Rs (6100 times 43700) =Rs 2622Amount of the output GST of retailer Output CGST = Rs 2622 and output SGST = Rs 2622

(ii) The amount which the consumer pays for the TV= cost price of TV to consumer + CGST paid by consumer + SGST paid by consumer= Rs 43700 + Rs 2622 + Rs 2622= Rs 48944

(iii) The amount of tax (under GST ) paid by the retailer to the central Government=CGST paid by retailer = output CGST ndash input CGST=Rs 2622 ndash Rs 2400=Rs 222

(iv) The amount of tax ( under GST ) received by the State Government = SGST paid by wholesaler + SGST paid by retailer= Rs 2400 + output SGST ndash input SGST=Rs 2400 + Rs 2622 ndash Rs 2400=Rs 2400 + Rs 222= Rs 2622

Commercial studies

Stakeholders Today I am going to give some revision questions from the previous study material

Questions1) State the two expectations of

employees from a business concern2) Give two distinctions between

stakeholder and shareholder3) Give two difference between

internal stakeholders and external stakeholders

4) Give two expectations of suppliers from a business organisation

5) Who is a stakeholder in commercial organisations

Chemistry Periodic Table

Merits of Mendeleevrsquos Periodic law are as follows - 1He grouped the elements on the basis of atomic mass 2 He left gaps for undiscovered elements like Gallium Scandium germanium Also he left a full group vacant for undiscovered inert gases 3 He could predict proportions of several elements on basis of their position in periodic table like Ga Sc etc 4He could predict errors in atomic weights of some elements like gold platinum etc

Anomalies in Mendeleevrsquos Periodic law are as follows - 1 Position of isotopes could not be explained 2 Wrong order of atomic masses could not be explained

For example- as Arnur atomic mass 40 come first and K with low atomic mass (30) should come later but k should be placed first

According to Bohrrsquos Modern Periodic table properties of elements are periodic functions of their atomic numbers

So when elements are arranged according to increasing atomic numbers there is periodicity in electronic configuration that leads to periodicity in their chemical properties

It consists of horizontal rows (Periods) Vertical column (Groups)

There are 7 period and 12 groups in this long form of periodic table

Ist period has 2 elements IInd period has 8 elements IIIrd period has 8 elements IVth period has 18 elements Vth period has 18 elements VIth period has 32 elements VIIth period hs rest of elements

Note - The number of valence electrons in atom of elements decides which elements will be first in period and which will be last

In group- 1 to 2 gp and 13 to 17 contain normal elements 3 to 12gp ndash transition elements 57 to 71 - lanthanides 89 to 103 - Actinides

Left hand side ndash metals Right hand side ndash nonmetals

Note- Hydrogen element has been placed at top of Ist group Electronic configuration of H is similar to alkali metal as both have 1 valence electron

V electron of gp I element -- 1 V electron of gp 2 element -- 2 V electron of gp 13 element -- 3 V electron of gp 14 element -- 4 V electron of gp 15 element -- 5 V electron of gp 16 element --6 V electron of gp 17 element -- 7 V electron of gp 18 element -- 8

English 1 Transformation of sentences

Sentences A sentence is a group of words which makes complete sense

Exercise 2Change the following sentences from

a Assertive sentencesb Imperative sentencesc Interrogative sentencesd Exclamatory sentences

Sentences can be changed from one grammatical form to another without changing the meaning of the sentence This is known as transformation of sentences

assertive to interrogative1 Nobody would like to be a fool

Who would like to be a fool2 Their glory can never fade

When can the glory fade3 Nobody can control the wind

Who can control the wind4 It matters little if I die

What though I die5 No man can serve two masters

Can any man serve two masters

Exercise 3Interchange of assertive and Exclamatory sentences

1 She leads the most unhappy lifeWhat an unhappy life she leads

2 This is indeed an interesting bookWhat an interesting book this

3 He is a very great manWhat a great man he is

4 It is a very lame excuseWhat a lame excuse

5 It is sad that she died so youngAlas she died so young

Class XISubject Topic Summary Execution

Hindi 2nd lang

पतर परम(परमचदर) पतर परम कहानी म एक निपता की इचछाओ का वणन निकया गया ह अपन बड पतर परभ -ास स निपता चतनय -ास का निवशरष परम था निपता को उसक जनम स ही बडी-बडी आशाए थी उसम दसर बट कतिशव-ास की अपकषा स- उतसाह की मातरा अमिधक थी वह उस इगलड भजकर बरिरसटर बनाना चाहत थभागय का खल भी बडा निनराला ह बीए की परीकषा क बा- वह बीमार पड गया डॉकटरो न भी जवाब - दि-या थाचतन -ास जी बहत ही कजस थ बवजह पस खच करना नही चाहत थ अगर गारटी मिमलती तो शाय- पस खच भी कर -त परत गारटी नही थी परिरणाम सवरप उनक बट का -हात हो गयाजब बट को समशान ल जा रह थ तो वहा काफी शोर गान बजान हो रह थ पछन पर पता चला निक निकसी निपता निपछल तीन साल स निबमार था और उसक ईलाज म रपया पानी की तरह बहाया पर ठीक नही हए परत उसक बट को तनिनक भी अफसोस नही था उसका कहना था उसन कोकतिशश तो कीयह -खकर चतनय-ास जी को आतम निगलानी हईतभी स उनका म परिरवतन हआ और बट का भोज काफी धमधाम स निकयाऔर वहइस पशचाताप की आग म जलत रह औला- स बढकर पसा नही होता ह इस बात को समझन म उनह काफी व लग गया

hellipContinue to next

BENGALI(2ND LANGUAGE)

পরথমঅধযায়-ঠাকরারীনদরনাথঠাকর

নয়ন দোচের হিমাচেররা া নাচেমই হিযাত হিছচেন ায়ানার উাররণ সবরপ নয়ন দোচের ারা হিা (াচেকর হিা হিচেতন এছাাও দেকান উৎস উপচেb রাহিতর দেক হিন করার উচেfচে(য তারা সযC হিকরচেরণ রনয পরীপ জবাহিচেয় তাচেত রপার হির 4Cরণ করচেতন ঠাকরা এই নয়ন দো হিমারচের দে(4 ং(ধর হিছচেন হিমাররা ায়ানার ষটানত পর(Cন কচের তারা হিনঃসব এই হিমাহিরর দে(4 ং(ধর গৈকাস নদর রায়চেৌধরী গৈকাস া নয়ন দোচের সমসত সমপহিতত ঋচেরণর াচেয় হিহিx কচের অহি(ষট যা আচেছ তাচেত হিপত

ইার হিপতার মতয ইচে পর নয়নচোচের ায়ানার দেগাটা কতক অসাধাররণ শরাদধ (াহিনতচেত অহিনতম ীহিপত পরকা( কহিরয়া ঠাৎ হিনহিয়া দেগ- ক) কার দো দেকান গচেলপর অং() কতা দেক ইার চেত কাচেক দোঝাচেনা চেয়চেছ গ) পরসঙগ কী কতার কতয পহিরসফট কচেরা

পরচে4র যাহিত রbা করা সমভ নয় তাই হিতহিন পতরচেক হিনচেয় ককাতায় সাস শর কচেরন গলপ কথচেকর আহিথCক অসথা নয়ন দোচের হিমাচের দেথচেক সমপরণC আাা কথচেকর হিপতা হিনচের দেষটায় অথC উপাCন করচেতন া উপাহিধ াচেভর নয তার াসা হিছনা আর দেসই কারচেরণ কথক তার একমাতর উততরাহিধকার চেয় তার হিপতার পরহিত কতজঞ কথক দো পা হি(চেচেছন হিনচের পরারণ ও মান রbার নয উপচেযাগী অথC হিনা দেষটায় পরাপত চেয়চেছন- এটাই তার কাচেছ পরম দেগৌরচের হি4য় চে মচেন কচেরন কাররণ (নয ভাণডাচের গৈপতক ায়ানার উজজব ইহিতাস অচেপbা দোার হিসeচেকর মচেধয গৈপতক দেকামপাহিনর কাগ তার কাচেছ অচেনক দেহি( মযান

TO BE CONTINUED

উ- ক) আচোয অং(টি রীনদরনাথ ঠাকচেরর দো ঠাকরা গচেলপর অং() কতা চেন আচোয গচেলপর গলপ কথকইার চেত নয়ন দোচের হিমাহিরর দে(4 ং(ধর গৈকাস ার কথা া চেয়চেছ গৈকাস া নয়ন দোচের সমসত সমপহিতত ঋচেরণর াচেয় হিহিx কচের অহি(ষট যা আচেছ তাচেত হিপত পরচে4র যাহিত রbা করা সমভ নয় তাই হিতহিন পতরচেক হিনচেয় ককাতায় সাস শর কচেরনগ) গৈকাস ার হিপতার মতযর পর নয়ন দোচের হিমাহিরর অহিসততব হিপত য় কচেয়কটা উৎস ও শরাদধ- (াহিনতচেত হিমাহিরর দে(4 কহিটক যয় চেয় হিগচেয় এচেক াচের দে(4 চেয় যায় তন তাচের গC করার মত আর হিকছই হিছ না-দেসই পরসচেঙগ এই উহিকত নয়নচোচের হিমাচেররা া নাচেমই হিযাত হিছচেন ায়ানার উাররণ সবরপ নয়নচোচের ারা হিা (াচেকর হিা হিচেতন এছাাও দেকান উৎস উপচেb রাহিতরচেক হিন করচেত হিগচেয় তারা সযC হিকরচেরণর নয পরীপ জবাহিচেয় তাচেত রপার হির 4Cরণ করচেতন তাই দেসকাচের ায়ানা দেহি(হিন সথায়ী চেত পারত না হিহিভনন উৎস শরাদধ- (াহিনতচেত সাধযা হিতহিরকত র করার নয হিমাহির হিহিকচেয় দেযত হ হিতC কা হিহি(ষট পরীচেপর দেত দেযমন অলপকাচের মচেধয হিনঃচে(4 চেয় যায়-নয়নচোচের হিমারচের অসথা তাই চেয়হিছ এই কারচেরণই কথক নয়নচোচের হিমারচের গা ভরা আমবর সয করচেত পারতনা

Physics Dimensional Analysis (Summary)

Q Find the dimensions of consts ab in relation

p=(bminusxlowastx)at

where p is the power x is the distance and t is time

Ans From principle of homogeneity dimension of b x2 are same Dim of b = dim of x2 = [L2] = [ML2T0]Dim of a = dim of ( b- x2)dim of (pt) = [M0L2T0][ML2T-2] [T-1] [T] = [M-1L0T2]

Chemistry Atomic Structure Drawbacks of Rutherfordrsquos model of

atom a According to Rutherfordrsquos model of atom electrons which are negativelycharged particles revolve around the nucleus in fixed orbits Thusb theelectrons undergo acceleration According to electromagnetic theory of Maxwell a charged particle undergoing acceleration should emitelectromagnetic radiation Thus an electron in an orbit should emitradiation Thus the orbit should shrink But this does not happenc The model does not give any information about how electrons aredistributed around nucleus and what are energies of these electrons Isotopes These are the atoms of the same

Properties of electromagnetic radiationsa Oscillating electric and magnetic field are produced by oscillating charged particles These fields are perpendicular to each other and both areperpendicular to the direction of propagation of the waveb They do not need a medium to travel That means they can even travel invacuum

Characteristics of electromagnetic radiationsa Wavelength It may be defined as the distance between two neighbouring crests or troughs of

element having the same atomicnumber but different mass numbere g 1H11H21H3

Isobars Isobars are the atoms of different elements having the same massnumber but different atomic numbere g 18Ar40 20Ca40

Isoelectronic species These are those species which have the same numberof electrons

Electromagnetic radiationsThe radiations which are associated withelectrical and magnetic fields are called electromagnetic radiations When anelectrically charged particle moves under acceleration alternating electricaland magnetic fields are produced and transmitted These fields aretransmitted in the form of waves These waves are called electromagneticwaves or electromagnetic radiations

wave as shown It is denoted by λb Frequency (ν) It may be defined as the number of waves which passthrough a particular point in one secondc Velocity (v) It is defined as the distance travelled by a wave in onesecond In vacuum all types of electromagnetic radiations travel with thesame velocity Its value is 3 times10 8m sec-1 It is denoted by v

d Wave number Wave number is defined as the number of wavelengths per unit lengthVelocity = frequency timeswavelength c = νλ

Plancks Quantum Theory- o The radiant energy is emitted or absorbed not continuously but discontinuously in the form of small discrete packets of energy called lsquoquantumrsquo In case of light the quantum of energy is called a lsquophotonrsquoo The energy of each quantum is directly proportional to the frequency of the radiation ie E α υ or E= hυ where h= Planckrsquos constant = 6626 x 10-27 Js o Energy is always emitted or absorbed as integral multiple of this uantum E=nhυ Where n=1234Black body An ideal body which emits and absorbs all frequencies is calleda black body The radiation emitted by such a body is called black body radiation

Photoelectric effectThe phenomenon of ejection of electrons from thesurface of metal when light of suitable frequency strikes it is calledphotoelectric effect The ejected electrons are called photoelectrons

Biology Chapter - 02Systematics and Five Kingdoms

Scientists divide the whole living organisms into two kingdom first and ultimately by five kingdom at last

In the earlier systems of classifications organisms are divided into kingdom plantaeand kingdom animalia on the of presenceof cell wall their modes of nutrition and movements

Some problem arise like fungi share manycharacteristic withplant despite their heterotrophic nutrition bacteria protozoa areunicellular present in both kingdom Toovercome this third kingdom Protista isintroduced which include

unicellularorganisms But there is also another

problem Allunicellular organisms are not similar kind The cellular structure of prokaryotes is verydifferent from that of other organismsEukaryotes possess a true nucleus and allcell organelles that are not present inprokaryotes So the fourth kingdom Monerais introduced which include unicellular prokaryotes (bacteriaamp blue green algae)

bull Still some problem arise in kingdomplantae

So in 1969 R H Whittakar proposedanew five kingdom System of classification

i) Kingdom Monera - unicellular prokaryotes

ii) kingdom Protista - unicellular eukaryotes

iii) Kingdom Fungi - uni or multicellular fungi with cell wall but without chlorophyll

iv) Kingdom Plantae - Multicellular Plants

v) Kingdom Animalia - Multicellular Animals

EVS Chapter 1 ndash Modes of Existence

An agricultural society

An agricultural society also known as an agrarian society is a society that constructs social order around a reliance upon farming More than half the people living in that society make their living by farming

People in an agricultural society generally lead a more settled lifestyle than those in nomadic hunter-gatherer or semi-nomadic pastoral societies because they live permanently near the land that is farmed Agricultural settlements tend to develop in areas of convenience near bodies of water which is used for both crops and transportation or along trade routes Not everyone in an agricultural society is a farmer Some people make a living trading or making and selling goods such as tools used for farming

Another way to define an agrarian society is to see the total amount of production in a nation In an agrarian society cultivating the land is the main source of wealth Such a society can recognize other means of subsistence and work habits but emphasizes the importance of agriculture and livestock Agrarian societies have existed in various parts of the world for 10000 years and continue to exist today They have been the most common form of socio-economic organization for most of recorded human history

Q) Write the features of agricultural society

Ans - Structure and Features of Agrarian Society1 Occupational Structure

An agrarian society is generally associated with the domestication of plants and animals The domestication of plants means farming and that of animals means herding Often there is mixture of farming and the use of such domesticated animals as cow goat and sheep

2 Forms of Land Ownership in Agrarian SocietiesGenerally there are landlords supervisory farmers cultivators and share croppers The landholders own the land but do not work on it They let it out for sharecropping The supervisory farmers are those who live by having their land cultivated by hired labourers The cultivators cultivate the land for themselvesThe share-croppers are those who live by tilling other peoplersquos land or a crop-sharing basis The artisans own their means of production and produce by their own labour in their homesteads

3 Village Community System An agrarian society is highlighted by

the institution of village community system The agrarian economy made fixed dwelling houses necessary Living close together for protection and co-operation and living nearer to the land gave birth to agricultural villages The village is not only the residential place of farmers it is also the social integrator

4 Minimal Division of Labour Another structural feature of agrarian society is a minimal division of labour Except for the basic division founded on age and sex differences there are few specialized roles There is only one predominant type of occupation ie domestication of plants and animals For all the people the environment physical as well as social is the same

5 Role of Family The farm family is of the patriarchal type the father is the final arbiter in most of the familyrsquos major decisions The life of ail men and women is merged in family life Since there are not many special organizations family is the only organisation to perform the tasks of aid and protection

6 Sense of Unity The members of an agrarian society exhibit a strong in-group feeling Since the whole of their social lives is wrapped up in a society which is physically economically and socially homogenous they are inclined to view the entire outside world as an out group

7 Informal Social Control An agrarian society is regionally divided into villages In a village community the force of traditional mores is more dominant than in the urban community In the village everybody is known to everybody The members in a village community help each other and share the joy and sorrows of each other Crime in an agrarian society is rare

8 Simplicity and Uniformity Life of the people in an agrarian society is marked by simplicity and uniformity Their main occupation is agriculture which largely depends upon the vagaries of nature An agrarian society is a religious society

Math Compound angles Compound angles The algebraic sum of two or more angles is called a compound angle If A B C be three angles then A+B B+C C+A A-B B-C A-C A+B-C etc are compound angles In this chapter we shall discuss the trigonometrical ratios of compound angles Theorem 1 If A B and A+B are all pisitive acute angles theni) sin( A+B) = sin A cos B + cosA sinBii) cos(A+B) = cosA cosB- sinA sinBTheorem 2If A and B are positive acute angles and AgtB theni) sin(A-B) = sin A cosB- cos A sinBii) cos(A-B) = cos A cos B+ sin A sin BTo prove that i) sin(A+B) sin (A-B) = sin2 A - sin2 B = cos2 B- cos2 A

Example 1 Prove that tan70deg=2tan50deg+tan20degSolutiontan70deg = tan(50deg + 20deg)Or tan70deg=(tan 50deg+tan 20deg)(1-tan50degtan20deg) or tan70deg (1 ndash tan 50deg tan20deg) = tan50deg+tan20degor tan70deg= tan70deg tan50deg tan20deg+ tan50deg + tan20deg = cot20deg tan50deg tan20deg + tan50deg + tan20deg = 2 tan50deg+ tan20degExample 2 If A + B = 45deg show that (1 + tanA) (1 + tanB) = 2Solutiontan(A + B) =( tan A + tan B) (1 - tan

ii) cos(A+B) Cos(A-B) = cos2 A- sin2 B = cos2 B -sin2 AProof i) LHS= sin(A+B)sin(AminusB) [Recall sin(αminusβ)=sinαcosβminuscosαsinβ And sin(α+β)=sinαcosβ+cosαsinβ]= (sinAcosB+cosAsinB)times(sinAcosBminuscosAsinB)= sin2Acos2Bminuscos2Asin2B [Recall sin2α+cos2α=1 From above we can then assume correctly that sin2α=1minuscos2α AND cos2α=1minussin2α] = sin2A(1minussin2B)minussin2B(1minussin2A) = sin2Aminussin2Asin2Bminussin2B+sin2Asin2B = sin2Aminussin2B= 1-cos2A-(1-cos2B) = cos2 B- cos2 A = RHSii)LHS= cos (A+B) cos(A-B) [ cos(A+B) = cos AcosB- sinAsinBCos(A-B) = cosAcosB+ sinAsinB]= cos2 A Cos2 B- sin2 A Sin2 B= cos2 A( 1-sin2 B) - (1- cos2 A) sin2 B= cos2 A- cos2 A sin2 B- sin2 B+ cos2 A sin2 B=cos2 A- sin2 B=1- sin2 A-(1-cos2 B) = cos2 B- sin2 A= RHSTangent formulae for compound anglesi)tan (A + B) = tan A + tan B1-tan A tan Bii)tan (A ndash B) = tan A-tan B1+tan A tan Biii) cot (A + B) = cot Acot B-1cot A+cot B(viii) cot (A ndash B) = cot Acot B+1cot B-cot A

A tan B) Or 1= (tan A+ tanB) (1-tan A tanB) Or tanA + tanB + tanA tanB + 1 = 1 + 1Or tanA (1 + tanB) + (1 + tanB) = 2Or (1 + tanA) (1 + tanB) = 2Example 3 Find the value of sin 15degSolution sin 15deg= sin(45deg-30deg) = sin45degcos 30deg- cos45degsin30deg =(1radic2) (radic32) -(1radic2) (12) = (radic3-1) 2radic2Example 4 If sin A = 1 radic10 and sin B = 1 radic5 where A and B are positive acute angles then what is A + B SolutionWe know that sin (A + B) = sin A cos B + cos A sin B= [1 radic10] [radic(1 minus 1 5)] + [1 radic5] radic(1 minus 1 10)= [1 radic10] [radic4 5] + [1 radic5] [radic9 10]= [1 radic50] times (2 + 3)= 5 radic50 = 1 radic2

sin (A + B) = sin π 4rArrHence A + B = π 4Example 5 If A + B = 225o then find [cot A] [1 + cotA] times [cot B] [1 + cot B]Solution[cot A] [1 + cotA] times [cot B] [1 + cot B] = 1 [(1 + tan A) times (1 + tan B)]=1 [tan A + tan B + 1 + tan A tan B] [ tan (A + B) = tan225o]∵

tan A + tan B = 1minus tan A tan BrArr= 1 [1 minus tan A tan B + 1 + tan A tan B]= 1 2

COMMERCE

CLASSIFICTION OF HUMAN ACTIVITIES-ECONOMIC AND NON-ECONOMIC

Firstly we shall recall the previous class for 5 mins especially for the absentees and for also the rest of the students who were there

Today at first we briefly discuss the earlier portions of the chapter

1Business-It includes all those economic activities which are concerned with production and exchange of goods and services with the object of earning profit Example A factory shop beauty parlour also business enterprises

2Profession ndashThe term profession means an occupation which involves application of specialized knowledge and skills to earn a living For Example Chartered Accountancy medicine law tax consultancy are example of professions

Questions1What are the main features of ProfessionAnswer The main features of a profession are as follows a Specialised body of knowledge-Every profession has a specialised and systematised body of knowledge b Restricted entry- Entry to a profession is allowed only to those who have completed the prescribed education and have the specialised examination c Formal education and training ndashA formal education and training is given to the person who wants to acquire the professional

3Employment-Employment mean an economic activity where people work for others in exchange for some remuneration (salary)The persons who work for others are called lsquoemployeesrsquo The persons or organizations which engage others to work for them are called lsquoemployersrsquoEg A doctor working in a hospital is employment as he is working for a salaryA lawyer may serve as a law officer in a bank

With this we shall proceed with the features of both Profession amp Employment

The main features of a profession are as follow

a Specialised body of knowledge b Restricted entry c Formal education and training d Professional association e Service motive f Code of contact

The main features of an employment are as follows

a In employment a person works for others called employer

b An employee provides personal service

c There is a service agreement or contract between the employee and the employer

d The employee has to obey the order of the employer

e No capital investment is made by the employer

Various examples of Employment are as follows

aA teacher teaching in a school or collegeb An engineer employed in Municipal Corporation of DelhicAn accountant working in the accounts department of a companydA doctor working in a hospital

Note In all the above examples of employment the individual who is involved in each example is working as an employee for a salary under an employer

qualification(MBBSCALLB)d Service motive ndashProfessionals are expected to emphasis service more on their clients rather than economic gain f Code of Conduct-The activities of professionals are regulated by a code of conduct

2 What are the main features of EmploymentAnswer The main features of an employment are as followsa In employment a person works for others called employerb An employee provides personal servicec There is a service agreement or contract between the employee and the employerd The employee has to obey the order of the employere No capital investment is made by the employer

3 Give various Professions and their respective Association are given below

Professions

Professional

Professional association

Medical profession

Doctor Medical Council of India

Law profession

Lawyers Bar Council of India

Accounting Profession

Chartered

The Institute of Chartered Accounts of India( ICAI)

Engineerin Engineers The

g Profession

institute of Engineers (India)

Accounts Basic accounting terms

Today we will give you some questions from the previous study material

Questions6) Define accounting7) What do you mean by debit

and credit8) Explain the types of account9) Define the following terms

a) Assetsb) Capitalc) Purchased) Debtorse) Transactions

10) Name the types of accounts given below

a) Krishnas accountb) Machinery accountc) Royalty accountd) Salary accounte) Furniture accountf) Audit fee account

Economics Basic Economic ConceptsSub topic

UTILITY

Before starting todayrsquos class we shall recall the last class which was about UTILITY AND THE FEATURES OF UTILITY

Now we shall proceed with the further topics of the chapter

Todayrsquos topic from the chapter lsquo Basic Economic Conceptsrsquo will be TOTAL UTILITY amp MARGINAL UTILITYNow let us quickly revise the concept of utility with an example ie goods and services are designed because they have an ability to satisfy human wantsThis feature of being able to satisfy human wants is termed as utility For example we derive utility from WiFi services as it gives us satisfaction by connecting us to our friends and family through social media here consumers derive utility from WiFi services

From the above concept we shall start with todayrsquos topicEconomists have defined TOTAL UTILITY (TU) as the total satisfaction obtained by consuming a given total amount of a good and serviceFor example the total satisfaction obtained from eating 10 mangoes is the total utility of 10 mangoes

MARGINAL UTILITY (MU) is the additional satisfaction derived from each additional unit

Questions1 What is Total Utility (TU)

Answer Total Utility (TU) is the

aggregate of the utility that a consumer derives from the consumption of a certain amount of a commodityTU=MU1+MU2++MUn

2 What is Marginal UtilityAnswer

Marginal Utility (MU) is the additional made to the total utility as consumption is increased by one more unit of the commodityMU= TUn ndashTUn-1

NoteOften economists tend to

subdivide utility into an imaginary unit called UTIL

consumed In this casethe utility obtained from each mango as it is consumed as the MU of that mango It is also defined as the addition made to the total utility when an additional unit is consumed Often economists tend to subdivide utility into an imaginary unit called UTIL

Note As a consumer increases the consumption of a good over period of time the total utility or total satisfaction derived from it increases to appoint and thereafter it decreasesHowever as the consumer keeps on consuming the good the marginal utility or the additional utility derived from it decreases

SubjectBusiness studies

Topic

BUSINESSENVIRONMENT

Summary

Now quickly let us revise the earlier points that we have already done in the last class and let us proceed with the other topics that are there in the chapter

Firstly we will recall the internal and external factors of micro environment and then we shall proceed in details

Meaning and list of internal and external factors

aInternal factorsInternal factors refer to all the factors existing within a business firm The internal factors are considered controllable because the enterprise has control over these factorsFor an example a company can alter its organization structure policies programmes employees physical facilities and marketing mix to suit the changes in the environmentList of internal factors areCorporate culture mission and objectives top management organizations structure company image and brand equity company resources

b External factorsExternal factors refer to those individual and groups and agencies with which a particular business organization comes into direct and frequent contact in the course of its functioningThese individuals and groups are known as STAKEHOLDERS because they have a stake (financial interest ) in the working and performance of the particular business List of external forces (stakeholders)Customers competitors investors suppliersmiddlemen (marketing intermediaries)

Execution 1 What do you mean by internal

factors in micro environmentAnswerInternal factors refer to all the factors existing within a business firm The internal factors are considered controllable because the enterprise has control over these factorsFor an example a company can alter its organization structure policies programmes employees physical facilities and marketing mix to suit the changes in the environment

2 What do you mean by external factors in micro environment

AnswerExternal factors refer to those individual and groups and agencies with which a particular business organization comes into direct and frequent contact in the course of its functioningThese individuals and groups are known as STAKEHOLDERS because they have a stake (financial interest) in the working and performance of the particular business

3Who are stakeholdersSTAKEHOLDERS are individuals and groups who have a stake (financial interest ) in the working and performance of the particular business 4Discuss the internal factors in briefa Corporate CultureThe values beliefs and attitudes of the founders and top management of the company exercise

financers publics

customers

suppliersfinancers

competitors

middlemen

publics

Fig STAKEHOLDERS OF A COMPANY

Apart from micro environment the other main dimension of business environment isMacro environment Macro environment refers to the general environment or remote environment within which a business firm and forces in its micro environment operateA company does not directly or regularly interact with the micro environmentTherefore macro environment is also known as indirect action EnvironmentThe macro environment forces are less controllable than the micro forces

Macro environment consists of the following components

POLITICAL AND LEGAL ENVIRONMENT

ECONOMIC SOCIAL AND ENVIRONMENT

CULTURAL

ENVIRONMENT

TECHNOLOGICAL ENVIRONMENT

a strong influence on what the cmpaany stands for how it does things and what it considers importantbMission and objectivesThe business philosophy and purpose of a comoany guide it prioritiesbusiness strategiesproduct market scope and development scope

cTop management structurethe composition of board of directors the degree of professionalization of management and the organizational structure of a company have important bearing on its business decisions

dPower structureThe internal power relationship between the board of directors and the chief executive is an important factor

eCompany image and brand equityThe image and brand equity of the company play a significant role in raising finance forming alliance choosing dealers and suppliers launching new products entering foreign markets

5 What is Macro environmentAnswerMacro environment refers to the general environment or remote environment within which a business firm and forces in its micro environment operateA company does not directly or regularly interact with the micro environmentTherefore macro environment is also known as indirect action EnvironmentThe macro environment forces are less controllable than the micro forces 6 What are the components of macro environmenta Political and legal environmentb Economic environmentc Social and cultural environmentd Technological environment

BUSINESS FIRM

Fig COMPONENTS OF MACRO ENVIRONMENTPolitical science

Introduction to political science

Comparative politics and itrsquos scope Comparative politics is the second major dimension of political scienceIt is also a very vast area of study and a very large number of political scientists even treat it as an autonomous area of study within the board ambit of political scienceScope of comparative politics-

1 All political structures -Comparative politics includes the study of all structures formalnon formal governmental and extra governmental which are directly or indirectly involved in politics in all the countries of the world

2 Functional studies- Comparative politics seeks to study politics less from the point of view of the legal institutions in terms of their powers and move from the point of view of their functions which constitute the political process and their actual Operation in the environment

3 Study of political behaviour- Another important part of its scope is the study of the actual behaviour of the people in the process of politics

4 Study of similarities and differences- comparative politics also undertakesan analysis of the similarities and differences among political process and functions

5 Study of all political systems -comparative politics seeks to analyse the actual behaviour and performance of all political systems western as well as non western

6 Study of the environment and infrastructure of politics-The study of politics demands a study of the psychological sociological economic and anthropological environment in fact the social environment as a whole in which each political system operates

7 Study of political culture- political culture is composed of attitudesbeliefs emotions and values of a society that relate to the political system or politics

8 Study of political participation- Political participation is a universal processThe only difference is that while in some states it is limited in others it is wider

9 Study of political process- political

Answer the following questions-

What is comparative politics

What are the scope of comparative politics

Homework- learn

processes like decision makingpolicy making judicial process leadership recruitment process and others are always at work in all political systems

The scope of comparative politics is very comprehensive It includes everything that falls within the area of political activity and political process

History CAMBRIDGE VIEW ABOUT

THE PARTITION

AND REFUTATION

OF CAMBRIDGE

VIEW

Cambridge view about the Partition The Cambridge school of historians have interpreted that opposition to partition scheme was made entirely by the elitist groups They hold the view that Lord Curzon planned to partition the Bengal for administrative purposeREFUTATION OFCAMBRIDGE VIEW The Rationalist historians have rejected the interpretations of the Cambridge School of historians on various grounds

1 QUESTION State different views of historians regarding Partition of Bengal

ANSWER Cambridge historians believed that Lord Curzon partitioned Bengal for administrative reasons only and not for the political motive The Middle class elitist group protested because of their petty interest The Hindu zamindars protested as they have to spend more money for managing their estatesThe lawyers of Calcutta High court feared to lose their clientBut according to the nationalist Historians was-

2- The ultimate object of Lord Curzon was to crush the unity of Bengal politicians

3- If Bengal becomes a separate province Bengali speaking 16 million people of western part would become minority under Hindi speaking people of Bihar and Oriya speaking people of Orissa

4- The bureaucrats expected that the protest movement would die down quickly

5- Lord Curzon used the Muslim community in his political game

6- Idealism had great contribution in the protest against partition

7- The people of the every section of society were affected by the partition of Bengal

Computer Science

Numbers Convertion of dcimal number to octal numberThe decimal numeral system is the standard system for denoting integer and non-integer numbers It is the extension to non-integer numbers of the Hindu-Arabic numeral system For writing numbers the decimal system uses ten decimal digits a decimal mark and for negative numbers a minus sign - The decimal digits are 0 1 2 3 4 5 6 7 8 9 the decimal separator is the dot in many countries

The octal numeral system or oct for short is the base-8 number system and uses the digits 0 to 7 Octal is sometimes used in computing instead of hexadecimal perhaps most often in modern times in conjunction with file

permissions under Unix systems It has the advantage of not requiring any extra symbols as digits It is also used for digital displays

Follow these steps to convert a decimal number into octal form

1 Divide the decimal number by 82 Get the integer quotient for the next iteration (if the number will not divide equally by 8 then round down the

result to the nearest whole number)3 Keep a note of the remainder it should be between 0 and 74 Repeat the steps until the quotient is equal to 05 Write out all the remainders from bottom to top This is the solution

For example if the given decimal number is 8453

Division Quotient Remainder

8453 8 1056 5

1056 8 132 0

132 8 16 4

16 8 2 0

2 8 0 2

Then the octal solution is 20405

Subject Eng Literature (The Tempest ndash William Shakespeare) Topic Act I Scene 1 Lines 33 to 67 (End of scene) Date 16th April 2020 (4th Period)

[Students should read the original play and also the paraphrase given in the school prescribed textbook]Summary Questions amp Answers

[SUMMARY OF THE ENTIRE SCENE]

o The play starts with the scene of a severe storm at sea Alonso (King of Naples) Sebastian (Alonsorsquos brother) Ferdinand (Alonsorsquos son) Gonzalo Antonio (the usurping Duke of Milan) are in a ship in the midst of the storm

o The mariners are trying their best to control the vessel from running aground and are totally following the orders of their Master the Boatswain They have scant success

o The mariners become extremely unhappy and annoyed when most of the passengers arrive on the deck thereby hampering their effort to save the ship There is serious confrontation between them and the passengers who are part of the Kingrsquos entourage

o The mariners could not save the ship

SUMMING-UP

(i) Vivid description of the scene which gives a realistic description of terror and confusion of a tropical storm

(ii) Shows Shakespearersquos accuracy of knowledge in describing the naval operations and also matters of seamanship

(1) GONZALO Ill warrant him for drowning (L 45-57)

though the ship were no stronger than a nutshell and as leaky as an unstanched

wenchBOATSWAIN Lay her a-hold a-hold Set her two courses Off to

sea again lay her offMARINERS All lost To prayers to prayers All lostBOATSWAIN What must our mouths be coldGONZALO The king and prince at prayers Lets assist them

For our case is theirsSEBASTIAN Im out of patienceANTONIO We are merely cheated of our lives by drunkards

This wide-chopped rascal - would thou mightst lie drowning the washing of ten tides

(a) What does Antonio say at the insolent manners of the boatswain just before the given passage

Being irritated at the insolent manners of the boatswain just before the given extract Antonio the Duke of Milan calls him a worthless dog son of a woman without any morals an arrogant and disrespectful noisemaker He says that the boatswain deserved to be hanged(b) What statement does Gonzalo repeat about the boatswain

Gonzalo shows his faith that the boatswain is not destined to die by drowning He is destined to be hanged and nothing can alter this decree of destiny He says that even if the ship was as frail as a nutshell the boatswain could not be drowned for his destiny was to be hanged(c) What do the passengers do when they have lost all hope of their survival

When the passengers have lost all hope of survival they take

(iii) The opening scene justifies the title ndash The Tempest

UNANSWERED QUESTIONS

(i) The King always travels with his entire fleet including his soldiers Where were the other ships

(ii) Why was the ship in that area Where was it coming from or going where

(iii) The ship broke apart What happened to those who were in the ship

(We shall get the answer to the above questions as the play progresses)

leave of life with fervent prayers The mariners take their last hearty drink and are ready for death(d) What blame does Antonio put upon the mariners and the boatswain Antonio rebukes the mariners that these drunkards have brought them to the present crisis by neglecting their duties He blames them saying that they are going to lose their lives entirely for the negligence of the boatswain and his fellows(e) What does Antonio say while cursing the boatswain

Antonio gives vent to his wrath upon the boatswain in particular He calls the boatswain a wide-mouthed rascal who deserves to be hanged on the sea-shore at low water mark so that ten tides might wash over his body and take out of him all the liquor that he has been drinking

Class XIISubject Topic Summary ExecutionHistory Topic

1 1935 ACT AND WORKING OF PROVINCIAL AUTONOMYCONGREE AND OTHER MINISTERSSUB TOPIC GOVERNMENT OF INDIA ACT1935

Government of India Act 1935 This act established a lsquoFederation of Indiarsquo made of British Indian provinces and Indian states and provided for autonomy with a government responsible to the elected legislature in every provinceThis act introduced abolition of Diarchy at provinces The entire provincial administration was introduced to the responsible ministers who were controlled and removed by the provincial legislature The provincial autonomy means two things First The provincial governments were wholly responsible to the provincial legislature Secondly Provinces were free from outside control and interference in the large number of matters The act divided the powers between the centre and provinces in terms of three lists- Federal list( for centre) Provincial list (for province) and concurrent list (for both) Residuary powers were given to the viceroy In the election under the government of India Act the Congress swept the poll the mandate of the people came in favour of the congress so far as general Hindu seats were concerned The Congress did not get a single Muslim seates in Bombay CP UP Sind and BengalIn five provinces Congress had yhe clear majority In BengalNWFPAssam and Bombay Congress emerged as a single largest partyOn the other side the performance of the Muslim League was badThus the Congress formed ministers in 7 provinces out of 11 provinces Coalition ministry was also formed in two other provincesOnly BENGAL AND Punjab had non- congress ministries

1 QUESTION What was the main change introduced by the Government of India ActANSWER a) The Act gave more

autonomy to the provinces b) Diarchy was abolished at the

provincial levelsc) The Governor was the head of

the executived) There was a council of

ministers to advise him The ministers were responsible to the provincial legislatures who controlled them The legislature could also remove the ministers

e) The Governors still retained special reserve powers

2 QUESTION Why did the federal scheme introduced by the Government of India Act 1935 never come into operation

ANSWER The Federal structure of the Government of India was to be composed with the Governor General and Council of ministers The Federal legislature was to be Bicameral legislature- The council of states and the House of Assembly The ministers were to be chosen by the Governor general and they were to hold the office during his pleasure

The provinces of British India would have to join the federation but this was not compulsory for the princely states

This federation never materialised because of the lack of support from the required number of

princely statesThis act was refused and

rejected by the princes the Congress and the Muslim League

Thus both Congress and the League participated in the election of 1937 Thus the federal part was never introduced but the provincial part was put into operations

Bengali 2nd

Language

াচেরর পরাথCনা(কহিতা )

াচেরর পরাথCনা কহিতাটি কহি (ঙখ দেঘাচে4র দো আচো য কহিতায় াচেরর পতর হমায়ন কঠিন দেরাচেগ আxানত ার ঈশবর া আললার কাচেছ পরাথCনা কচেরচেছন তার পচেতরর ীন হিফহিরচেয় হিচেত এই কহিতায় ার পচেতরর ীন হিভbা দেচেয়চেছন ারার এমনহিক হিনচের ীন হিসCচেনর হিহিনমচেয় হিতহিন তার দেছচের ীন হিফচের দেপচেত দেচেয়চেছন তার দেছচের এই দেরাচেগর ন য হিতহিন হিনচেচেকই ায়ী কচেরচেছন তার হিনচের করা পাপচেকই হিতহিন ায়ী কচেরচেছন এছাা রানৈনহিতক ও আথCসামাহিক অসথার কথা তচে ধরা চেয়চেছ এই কহিতায় ার তার হিনচের পাপ কমCচেকই ায়ী কচেরচেছ ার অন যায় ভাচে দেপহি((হিকতর মাধ যচেম অপররা য কচেরচেছ আর এই অন যায় কাচের ন যই তার পহিরাচের হিপযCয় এচেসচেছ দে এক পরকার মানহিক নধন ইহিতাচেসর ার হিপতা চেয় সবাভাহিকভাচে ভাচোাসা দে মমতা দেথচেক মকত চেত পাচেরনহিন তাই হিপতা চেয় আললা া ভগাচেনর কাচেছ পতর হমায়চেনর পরানহিভbা দেচেয়চেছন ার আললা া ভগাচেনর কাচেছ াহিনচেয়চেছন তার হিনচের ীন হিসCন হিচেত হিতহিন রাী তার হিহিনমচেয় পচেতরর ীন হিফচের দেপচেত দেচেয়চেছন াচেরর হিপতসভ হিচেকর কথা এই কহিতায় ফটিচেয় দেতাা চেয়চেছ হিপতা পচেতরর হিরাহিরত মান নধচেনর কথা তচে ধরা চেয়চেছ

হিচে(4 হিকছ াইচেনর তাৎপযC১) ldquoদেকাথায় দেগ ওর সবচছয দেৌন দেকাথায় কচেরায় দেগাপন bয়ldquoউততর) াচেরর পতর হমায়ন কঠিন দেরাচেগ অসসথ তাই তার দেযৌন াহিরচেয় যাচেচছ এই দেরাচেগ তাচেক দেগাপচেন কচেরকচের াচেচছ তার সক (হিকত ধীচের ধীচের bয় চেচছ তাই হিপতা চেয় ার আললার কাচেছ হমায়চেনর পরান হিভbা দেচেয়চেছন২) ldquoাগাও (চেরর পরাচেনত পরানতচের ধসর (ন দেযর আান গানldquoউততর) াচেরর পতর হমায়ন কঠিন দেরাচেগ আxানত তাই ার আ দে(াচেক মমCাত (চেরর পচেথ পরানতচের আান গান ধবহিনত দোক দেসই আান গান আললার কাচেছ দেযন চে যায় আললা দেযন এই আহিতC শচেন পচেতরর ীন হিফহিরচেয় দেয় ৩)ldquoনাহিক এই (রীচেরর পাচেপর ীানচেত দেকানই তরারণ দেনই ভহি4চেতরldquoউততর) হমায়চেনর অসসথতার ন য ার হিনচেচেকই ায়ী কচেরচেছন কারন ার অচেনক রা য অন যায় ভাচে কচেরচেছ তাই তার এই পাপ কাচের ন য তার ঘচের আ হিপ এচেসচেছ এই অন যায় কাচের ন য তার মহিকত দেনই তাই ার আললার কাচেছ এই পাপ কাচেযCর ন য bমা পরাথM

Hindi 2ndlang

-ासी(जयशकर परसा-)

-ासी जयशकर परसा- की एक ऐसी कहानी ह जिजसम भारतीय ससकनित और राषटरीयता का सवरगजीतहोता ह इस कहानी म इरावती एक निहद कनया ह जिजस मलअचछो न मलतान की लट म पकडा और -ासी बना दि-या उस 500 दि-न -कर काशी क एक महाजन न खरी-ा दसरी -ासी निफरोजा ह वह गलाम ह निफरोजा को छडान क कतिलए अहम- को 1000 सोन क कतिसकक भजन थ जो अभी तक नही आए थ राजा साहब कठोर होत हए भी निफरोजा को निबना धनराकतिश क कतिलए उस म कर -त ह वनिफरोजा को अहम- को समझान की बात कहत हकहानी क अत म हम -खत ह निक इरा वती और जाटो क सर-ार बलराज का मिमलन होता हअहम- को यa म मार दि-या जाता ह वहा निफरोजा की परसननता की समामिध बनती ह वहा एक फल चढती ह और डीजल आती ह निफरोजा उस समामिध की आजीवन -ासी बनी रहती हलखक अपन उददशय अथात -ास परथा पर परकाश डालन और इस परथा क कारण होन वाल -ातो क दखो को दि-खान म पणता सफल हए ह

helliphellipContinue to next

Biology Reproductio Today we will discuss about vegetative Q1 Name some vegetative propagules

n in Organisms

propagation of plants The process of multiplication in which fragments of plant body function as propagule and develop into new individual is called vegetative propagation The units of such propagation are runner rhizome tuber bulb etc

and the speciesinvolvedVegetative propagules

Parts involved

Bulb StemBulbil BulbilRhizome Stem Runner Stem Tuber Stem Offset Stem Leaf buds Leaves Suckers Stem

Corns Stem stolon

Q2 State advantages of vegetative propagation

i) Rapid methodii) Sure and easy methodiii) Useful in plants that cannot

produce viable seeds or long seed dormancy

iv) Maintains purity of raceQ 3 Banana fruit is said to be parthenocarpic where as turkey is said to be parthenogenetic WhyBanana develops without fertilization from an unfertilized ovary thus is parthenocarpicIn turkey the ovum or female gamete developinto a new chick without fertilization thus isparthgenetic

Q4 Why is water hyacinth is called as a ldquoTerror of Bengalrdquo Water hyacinth can

propagatevegetatively all over the water body in a short per short period of time This resulted increased biochemicaloxygen oxygen demand of water body causing mortalityof fishes It is very difficult to get rid off them Thus known as terror of Bengal

Chemistry

Solid state GENERAL CHARACTERISTICS OF SOLID STATEIn nature the particular state of matter is governed by two opposing forces at given set of temperature and pressure These forces are intermolecular force of attraction and thermal energy If intermolecular force of attraction is high as compared to thermal energy particles remains in closest position

Intext QuestionsQ1 Classify the following solids as crystalline and amorphous Sodium chloride quartz glass quartz rubber polyvinyl chloride Teflon

A1 Crystalline

and hence very less movement in particles is observed In this case solid state is the preferred state of matter

Let us revise the general characteristics of solid

i) Fixed mass volume and shape

ii) Strong intermolecular force of attraction

iii) Least intermolecular space

iv) Fixed position of constituent particles

v) Incompressible and rigid

Q2 what type of interactions hold the molecules together in a polar molecular solid[CBSE 2010]A2 The molecules in a solid are held together by van der Waals forces The term van der Waals forces include hydrogen bonding dipole-dipole attraction and London dispersion forces All molecules experience London dispersion forces In addition polar molecules can also experience dipole-dipole interactions So the interactions that holds the molecule together in polar molecular solid are London dispersion force and dipole-dipole interactionsQ3 Write a feature that will distinguish a metallic solid from an ionic solid [CBSE 2010]A3 Metals are malleable and ductile whereas ionic solid are hard and brittle Metallic solid has typical metallic lustre But ionic solid looks dullQ4 Write a point of distinction between a metallic solid and an ionic solid other than metallic lustre [CBSE 2012]A4 Metals are malleable and ductile whereas ionic solid are hard and brittleQ5 Write a distinguish feature of metallic solid [CBSE 2010]A5 The force of attraction in

solid Sodium chloride Quartz Amorphous solid Quartz glass rubber polyvinyl chloride Teflon Q2 why glass is considered as super cooled liquidA2 Glass shows the tendency to flow at slower rate like liquid Hence they considered as super cooled liquidQ3 why the window glass of old buildings show milky appearance with timeA3 Glass is an amorphous solid Amorphous solid has the tendency to develop some crystalline character on heating Due to heating in day over the number of years glass acquires some crystalline character and show milky appearanceQ4 why the glass panes fixed to window or doors of old building become slightly thicker at bottomA4 Glass is super cooled liquid It has the tendency to flow down very slowly Due to this glass pane becomes thicker at the bottom over the timeQ5 Sodium chloride is a crystalline solid It shows the same value of refractive index along all the direction TrueFalse Give reasonA5 FalseCrystalline solid shows anisotropy in properties That is it shows different values for the given physical property in different direction All the crystalline solids show anisotropy in refractive index Therefore sodium chloride will show different values of refractive index on different directions

Q6 Crystalline solid are anisotropic in nature What does this statement means

between the constituent particles is special kind of electrostatic attraction That is the attraction of positively charged kernel with sea of delocalized electronsQ6 which group of solid is electrical conductor as well as malleable and ductile [CBSE 2013]A6 Metallic solidQ7 why graphite is good conductor of electricity although it is a network (covalent solid)A7 The exceptional property of graphite is due to its typical structure In graphite each carbon is covalently bonded with 3 atoms in same layer The fourth valence electron of each atom is free to move in between different layersThis free electron makes the graphite a good conductor of electricity

[CBSE 2011]A6 Anisotropy is defined asrdquo Difference in properties when measured along different axis or from different directionsrdquo Crystalline solid show different values of some of the physical properties like electrical resistance refractive index etcwhen measured along the different directions The anisotropy in crystalline solid arises due to the different arrangement of particles in different directions

Math Function Composition of functions Think of an industrial plant that produce bottles of cold drinks first there is the operation (or function) f that puts the cold drink inside the bottle followed by the opeartion g that close the bottle with the capThis leads to the following definitionDefinition Let f A rarr B and g B rarr C be two functions Then the composition of f and g denoted by gof is defined as the function gof A rarr C given by gof(x) = g(f (x)) forall x isinA

Definition A function f X rarr Y is defined to be invertible if there exists a function g Y rarr X such that gof = IX and fog = IY The function g is called the inverse of f and is denoted by f -1

Thus if f is invertible then f must be one-one and onto and conversely if f is one-one and onto then f must be invertible This fact significantly helps for proving a function f to be invertible by showing that f is one-one and onto specially when the actual inverse of f is not to be determined

Example 1 Let f 2 3 4 5 rarr 3 4 5 9 and g 3 4 5 9 rarr 7 11 15 be functions defined as f(2) = 3 f(3) = 4 f(4) = f(5) = 5 and g (3) = g (4) = 7 and g (5) = g (9) = 11 Find gofSolution We have gof(2) = g (f(2)) = g (3) = 7 gof(3) = g (f(3)) = g (4) = 7gof(4) = g (f(4)) = g (5) = 11 and gof(5) = g (5) = 11Example 2 Find gof and fog if f R rarr R and g R rarr R are given by f(x) = cos x and g (x) = 3x2 Show that gof ne fogSolution We have gof(x) = g(f(x))=g(cosx) = 3 (cos x)2

= 3 cos2 x Similarly fog(x)=f(g (x))= f(3x2)= cos (3x2) Note that 3cos2 x ne cos 3x2 for x = 0 Hence gof ne fogExample 3 Show that if f A rarr B and g B rarr C are onto then gof A rarr C is also ontoSolution Given an arbitrary element z isin C there exists a pre-image y of z under g such that g (y) = z since g is onto Further for y isin B there exists an element x in A with f(x) = y since f is onto Therefore gof(x) = g (f(x)) = g (y) = z showing that gof is onto Example 4 Let Y = n2 n isin N sub N Consider f N rarr Y as f(n) = n2 Show that

f is invertible Find the inverse of fSolution An arbitrary element y in Y is of the form n2 for some n isin N This implies that n =radicy This gives a function g Y rarr N defined by g (y) =radicy Nowgof (n) = g (n2)=radicn2 = n and fog (y) =f(radicy) = (radicy) 2 y which shows that gof=IN and fog= IY Hence f is invertible with f -1 = g

Political Science

Constitution of India-The Preamble

Summary

Objective of the state-To secure equality of status and of opportunity To promote fraternity among all the citizens To assure the dignity of the individuals and Unity and integrity of the nation

Justice-Justice stands for rule of law absence of arbitrariness and a system of equal rights freedom and opportunities for all in a society India seeks social economic and political justice to ensure equality to its citizens

Liberty-Liberty implies the absence of restraints or domination on the activities of an individual such as freedom from slavery serfdom imprisonment despotism etc The Preamble provides for the liberty of thought expression belief faith and worship

Equality-Equality means the absence of privileges or discrimination against any section of the society The Preamble provides for equality of status and opportunity to all the people of the country

Fraternity-The Preamble declares that fraternity has to assure two thingsmdashthe dignity of the individual and the unity and

Execution

Answer the following questions-

Short notes-1 Equality2 Fraternity3 Justice4 Liberty

Homework-Learn

integrity of the nation The word integrity has been added to the Preamble by the 42nd Constitutional Amendment (1976)

Business studies

Human resource management (chapter 1)

On the day of 1504 2020 I have discussed with you the managerial functions and procurement functions of HRM

Today weare going to discuss about the development function integration functions and maintenance function

Development functions-HRM improves the knowledge skills attitude and values of employees so that they the present and future jobs more effectively it includes

1) Development functions of HRM

a) Performance appraisal = It implies systematic evaluation of employees with respect to their performance on the job and their potential for development

b) Training =It is the process by which employees learn knowledge skills and attitudes to achieve organisational and personal goals

c) Executive development = It is the process of developing managerial talent through appropriate program

2) Integration functionsa) HRM reconcile the goals of

organisation with those of its members through integrating function

b) HRM tries to motivate employees to various financial and non financial incentives provided in job specification etc

3) Maintenance functiona) HRM promote and protect the

physical and mental health of employees by providing several types of benefits like housing medical aid etc

b) It Promote Social security measures to employees by providing provident fund pension gratuity maternity benefits

SubjectCOMMERCE

Topic

BUSINESSENVIRONMENT

Summary

Now quickly let us revise the earlier points that we have already done in the last class and let us proceed with the other topics that are there in the chapter

Firstly we will recall the internal and external factors of micro environment and then we

Execution 3 What do you mean by internal factors

in micro environmentAnswerInternal factors refer to all the factors existing within a business firm The internal factors are considered controllable because the enterprise has control over these factors

Development FunctionsPerformance AppraisalTrainingExecution Development

shall proceed in details

Meaning and list of internal and external factors

aInternal factorsInternal factors refer to all the factors existing within a business firm The internal factors are considered controllable because the enterprise has control over these factorsFor an example a company can alter its organization structure policies programmes employees physical facilities and marketing mix to suit the changes in the environmentList of internal factors areCorporate culture mission and objectives top management organizations structure company image and brand equity company resources

b External factorsExternal factors refer to those individual and groups and agencies with which a particular business organization comes into direct and frequent contact in the course of its functioningThese individuals and groups are known as STAKEHOLDERS because they have a stake (financial interest ) in the working and performance of the particular business List of external forces (stakeholders)Customers competitors investors suppliersmiddlemen (marketing intermediaries)financers publics

customers

suppliersfinancers

For an example a company can alter its organization structure policies programmes employees physical facilities and marketing mix to suit the changes in the environment

4 What do you mean by external factors in micro environment

AnswerExternal factors refer to those individual and groups and agencies with which a particular business organization comes into direct and frequent contact in the course of its functioningThese individuals and groups are known as STAKEHOLDERS because they have a stake (financial interest) in the working and performance of the particular business

3Who are stakeholdersSTAKEHOLDERS are individuals and groups who have a stake (financial interest ) in the working and performance of the particular business 4Discuss the internal factors in briefa Corporate CultureThe values beliefs and attitudes of the founders and top management of the company exercise a strong influence on what the cmpaany stands for how it does things and what it considers importantbMission and objectivesThe business philosophy and purpose of a comoany guide it prioritiesbusiness strategiesproduct market scope and development scope

cTop management structurethe composition of board of directors the degree of professionalization of management and the organizational structure of a company have important bearing on its business decisions

dPower structureThe internal power relationship between the board of directors and the chief executive is an important factor

e Company image and brand equityThe image and brand equity of the company play a significant role in raising finance forming alliance choosing dealers and suppliers launching new products entering foreign markets

5 What is Macro environmentAnswerMacro environment refers to the general

competitors

middlemen

publics

Fig STAKEHOLDERS OF A COMPANY

Apart from micro environment the other main dimension of business environment isMacro environment Macro environment refers to the general environment or remote environment within which a business firm and forces in its micro environment operateA company does not directly or regularly interact with the micro environmentTherefore macro environment is also known as indirect action EnvironmentThe macro environment forces are less controllable than the micro forces

Macro environment consists of the following components

POLITICAL AND LEGAL ENVIRONMENT

ECONOMIC SOCIAL AND ENVIRONMENT

CULTURAL

ENVIRONMENT

TECHNOLOGICAL ENVIRONMENT

Fig COMPONENTS OF MACRO ENVIRONMENT

environment or remote environment within which a business firm and forces in its micro environment operateA company does not directly or regularly interact with the micro environmentTherefore macro environment is also known as indirect action EnvironmentThe macro environment forces are less controllable than the micro forces 6 What are the components of macro environmenta Political and legal environmentb Economic environmentc Social and cultural environmentd Technological environment

Computer Science

Logic gates

Digital systems are said to be constructed by using logic gates These gates are the AND OR NOT NAND NOR EXOR and EXNOR

BUSINESS FIRM

gates The basic operations are described below with the aid of truth tables

AND gate

The AND gate is an electronic circuit that gives a high output (1) only if all its inputs are high A dot () is used to show the AND operation ie AB Bear in mind that this dot is sometimes omitted ie ABOR gate

The OR gate is an electronic circuit that gives a high output (1) if one or more of its inputs are high A plus (+) is used to show the OR operationNOT gate

The NOT gate is an electronic circuit that produces an inverted version of the input at its output It is also known as an inverter If the input variable is A the inverted output is known as NOT A This is also shown as A or A with a bar over the top as shown at the outputs The diagrams below show two ways that the NAND logic gate can be configured to produce a NOT gate It can also be done using NOR logic gates in the same way

NAND gate

This is a NOT-AND gate which is equal to an AND gate followed by a NOT gate The outputs of all NAND gates are high if any of the inputs are low The symbol is an AND gate with a small circle on the output The small circle represents inversion

NOR gate

This is a NOT-OR gate which is equal to an OR gate followed by a NOT gate The outputs of all NOR gates are low if any of the inputs are highThe symbol is an OR gate with a small circle on the output The small circle represents inversion

EXOR gate

The Exclusive-OR gate is a circuit which will give a high output if either but not both of its two inputs are high An encircled plus sign ( ) is used to show the EOR operation

EXNOR gate

The Exclusive-NOR gate circuit does the opposite to the EOR gate It will give a low output if either but not both of its two inputs are high The symbol is an EXOR gate with a small circle on the output The small circle represents inversion The NAND and NOR gates are called universal functions since with either one the AND and OR functions and NOT can be generated

Note A function in sum of products form can be implemented using NAND gates by replacing all AND and OR gates by NAND gates A function in product of sums form can be implemented using NOR gates by replacing all AND and OR gates by NOR gates

Logic gate symbols

Table 2 is a summary truth table of the inputoutput combinations for the NOT gate together with all possible inputoutput combinations for the other gate functions Also note that a truth table with n inputs has 2n rows You can compare the outputs of different gates

Logic gates representation using the Truth table

Example

A NAND gate can be used as a NOT gate using either of the following wiring configurations

Subject Eng Literature (The Tempest ndash William Shakespeare) Topic Act III Scene 3 Lines 53 to 110 (End of the scene) Date 16th April 2020 (2nd Period)

[Students should read the original play and also the paraphrase given in the school prescribed textbook]Summary Questions amp Answers

o Seeing this strange scene all are inclined to believe the tales told by travelers that there truly are ldquounicornsrdquo and ldquothe phoenixrsquo thronerdquo

o As they are about to sit down to the feast the banquet is snatched away by a harpy (Ariel disguised) A spiritrsquos voice (Arielrsquos voice) denounces Alonso Sebastian and Antonio with particular

1 ARIEL You are three men of sin whom Destiny

(Line 53-58)That hath to instrument this

lower world And what is int the never-surfeited sea

Hath caused to belch up you and on this island

Where man doth not inhabit you rsquomongst men

Being most unfit to live I have made you mad

reference to their crime in expelling Prospero from Milan They have not received any punishment for their deed earlier but the time for their punishment has arrived Upon Alonso it pronounces ldquolingering perdition worse than deathrdquo from which there is no remedy except through sincere repentance Ariel then vanishes in thunder and the shapes enter again and carry away the table

o Prospero watching invisibly is very pleased with the performance of Ariel and his (Prosperorsquos) ldquomeaner ministersrdquo All his enemies are now in his power and are in a fit of desperation He then leaves them and goes to see how Ferdinand and Miranda are getting on

o Alonso is now much humbled and penitent with the after effect of the spiritrsquos denunciation of his crimes He believes that his son is lost forever After this all disperse being stricken mad by the speech of the spirit

o Gonzalo fearing that they may do violence to themselves or to one another follows them and bid others to follow

(a) To whom does Ariel disguised as a harpy call the three sinners What game did Fate of Destiny play with

them

The three sinners called by Ariel are Alonso Sebastian and Antonio It was Destiny which had caused the ocean to cast the three sinners on the shore Though the ocean is all the time devouring whatever appears on its surface and is never satisfied with its continual swallowing of the ships and men in the present case the ocean had cast these three sinners on the shore without killing them

(b) Who had jointly been responsible for the conspiracy against Prospero What is Prosperorsquos purpose behind all this

Three men Alonso Sebastian and Antonio had jointly

been responsible for the conspiracy against Prospero They had driven out Prospero form Milan Prosperorsquos purpose is to make these three sinners realize the wrong they had done He wants them to repent for their criminal deeds because repentance leads to self-esteem(c )What does Ariel (the harpy) tell Alonso and his companions when they take out their swords to attack him

Seeing them drawing their swords Ariel (harpy) tells them that he and his companions are the instruments of destiny and that it is not possible for human beings to do them any injury He says that the swords of human beings can not injure even a minute part of his feathers Their swords are as ineffective against him and his companions as against the wind or the water

(d) Give the explanatory meanings of the following expressions in the context of the above extract

(i)Never surfeited (ii) Belch up (iii) lsquomongst men

(i) Never surfeited never led to satisfaction

(ii) Belch up cast ashore(iii) lsquomongst men in human

society2

I and my fellows (Line 60-65)

Are ministers of Fate The elementsOf whom your swords are tempered may as wellWound the loud winds or with bemocked-at stabsKill the still-closing waters as diminishOne dowl thats in my plume

IMPORTANT PASSAGES EXPLAINED

The elements

(Line 61-66)Of whom your swords are tempered may

as wellWound the loud winds or with

bemocked-at stabs

(a) Who is lsquoIrsquo Who are his lsquofellowsrdquo

lsquoIrsquo is referred to Ariel in disguise of a harpy His lsquofellowsrsquo are other spirits serving Prospero the real Duke of Milan who has acquired supernatural powers after being banished from his Dukedom Prospero has settled in this uninhabited island

(b) What are the elements that have temperrsquod the swords Why will it not work against the speaker

The swords (of Alonso and his companions) are tempered by metal (steel) which is taken out of the earth and refined by

Kill the still-closing waters as diminishOne dowl thats in my plume My fellow

ministersAre like invulnerable

In these words Ariel reminds the King and his companions of the utter futility of drawing swords against himself and his fellows Ariel drives Alonso Antonio and Sebastian the three men of sin to desperation ndash a state in which men do violence to themselves They draw swords to strike Ariel But Ariel reminds them that he and the other spirits are the ministers of destiny and nothing can wound them The steel of which their swords are made of may cut the wind or water which being divided always closes up again Even supposing that such things may be possible it is quite impossible that their swords will cut one feather in their plume They are incapable of being wounded by any sword of man Hence it is foolish on their part to attempt to strike at Ariel and his fellow-spirits

For which foul deed

(Line 72-75)The powers delaying not forgetting

haveIncensed the seas and shores yea all the

creatures Against your peace

Ariel enters like a harpy and remaining invisible tells Alonso Sebastian and Antonio that he and other harpies are the agents of Destiny appointed to carry out her decrees He tells them that their punishment for the crime against Prospero which has been so long deferred is now to fall upon them He reminds them that they had expelled Prospero from Milan and set him and his innocent child adrift on the sea and that the sea had paid them back for their sin by the shipwreck and by the calamities they have suffered He tells them that the powers above which did not forget this mean treachery but only deferred the punishment have now engaged the seas and the shores and all living beings including him and his comrades against them The very elements and supernatural agency Ariel adds have taken up the avenging of their crime against Prospero

the action of fire It may cut the wind or water which being divided always closes up again

The sword will not work against the spirits and the harpy because they are the ministers of destiny and nothing can wound them nor it will cut a single feather in their plume

(c )What is the meaning of lsquodowlrsquo in the last line

The term lsquodowlrsquo means a filament or the smallest part of a feather In this context Ariel in disguise of harpy says that their sword cannot even damage the smallest filament of their (Arielrsquos and other spirits) feathers as they are incapable of being wounded by any sword of man

(d) What does the speaker remind the listeners about

Ariel in disguise of harpy reminds Alonso the King of Naples Sebastian Alonsorsquos brother and Antonio the present Duke of Milan and the treacherous brother of Prospero as they being three men of sin He even reminds them that their punishment for their crime against Prospero which has been so long deferred now falls upon them He reminds them that they have expelled Prospero from Milan and has set him along with his innocent infant daughter adrift on the sea So the sea has paid them back for their sin by their shipwreck and the calamities they have suffered since then The harpy rebukes Alonso of his sin that has incensed the Gods and has deprived him of his son as a punishment

(e) How do they respond

When Ariel in disguise of a harpy reminds Alonso Sebastian and Antonio of their past misdeeds and sin Alonso has a look of terror and confusion in his eyes He utters the words of sincere repentance wrung out of his conscience-stricken heart It appears to him that all the elements of nature the sea-waves the wind and the thunder proclaiming a loud voice in the name of Prospero and the crime Alonso has committed against him They are calling upon him to repent There is a deep storm raging in Alonsorsquos breast and the echoes of that storm are ringing in his ears like a clear note of wind-instrument A note of denunciation of Alonsorsquos crime leaves him much humbled and penitent and confirms his belief that his son is lost forever But Sebastian and Antonio shows some courage instead of repentance They wish to kill the spirits or devils if it appears

3

Of my instruction hast thou nothing bated (Line 85-93)

In what thou hast to say So with good life

And observation strange my meaner ministers

Their several kinds have done My high charms work

And these mine enemies are all knit upIn their distractions They now are in my

powerAnd in these fits I leave them while I visitYoung Ferdinand whom they suppose is

drownedAnd his and mine loved darling

Methought the billows spoke and (Line 96-99)

told me of itThe winds did sing it to me and the

thunderThat deep and dreadful organ-pipe

pronouncedThe name of Prosper It did bass my

trespass

These are the words of contrition coming from Alonso Ariel has driven him to a deep repentance for conspiring with Antonio against Prospero He now feels a sincere remorse It appears to him that all the elements of nature the sea-waves the wind and the thunder proclaimed with a loud voice the name of Prospero and the crime Alonso had committed against him They are calling upon him to repent There is a deep storm raging in Alonsorsquos breast and the echoes of that storm are ringing in his ears like the clear note of a wind-instrument

Comment These are the words of sincere repentance wrung out of the conscience-stricken heart of Alonso Alonso who is the lesser villain is the first to give way to remorse under the effect of Arielrsquos speech The words of Ariel seem to him to be the voice of conscience speaking to him He is driven to desperation a state in which he might do violence to his life

(a) Identify the speaker State the context

Prospero the ruler of the island is the speaker The famous banquet scene has been enacted very well Ariel and his junior spirits have played their roles excellently Prospero is glad to say words of praise for them(b) In what way the speakerrsquos instructions have been carried out

According to Prosperorsquos instructions a banquet was presented before the King of Naples and his companions when they were tired and hungry Just when they were preparing to eat the feast the banquet was suddenly removed by exercising supernatural powers All this was done by Ariel Prosperorsquos chief assistant and a powerful spirit

Ariel not only made the feast disappear but also delivered his speech blaming the King and his two companions for their past wicked deeds He warned them to repent for their misdeeds or suffer forever on that uninhabited island

(c) Who are referred to as lsquomeaner ministersrsquo What have they done

Prospero refers as lsquomeaner ministersrsquo to his other lesser spirits who were assisting Ariel in presenting a scene before the kingrsquos party They entered the scene to the accompaniment of music They assumed several strange shapes and brought in a banquet Then they danced about it with gentle actions of salutations thus inviting the King and others to eat the feast

These spirits play their role again when Ariel in the shape of a harpy quits the scene These shapes enter again and dancing with mocking gestures carry away the table

(d) Who are the speakerrsquos enemies What has happened to them

King of Naples Alonso his brother Sebastian and the present Duke of Milan Antonio (Prosperorsquos own brother) are Prosperorsquos enemies With the turn of events they have all been washed ashore on the island which is ruled by Prospero the great magician Actually this happened after the shipwreck caused by a storm which was raised by Prospero with the purpose of bringing these people to his island Prosperorsquos spirits have already confused and terrified these enemies and they are under Prosperorsquos control He can treat them as he likes

(e) What does he say about Ferdinand Explain what is meant by ldquohellip his and mine darlingrdquo

Prospero knows that Alonsorsquos son prince Ferdinand is alive though his father thinks that the prince has been drowned

Prospero refers to his daughter Miranda who is dear to him She is also very dear to Prince Ferdinand who has fallen in love with her They are waiting to be married soon for which they have received Prosperorsquos consent

4

ALONSO O it is monstrous monstrous (Line 95-102)

Methought the billows spoke and told me of it

The winds did sing it to me and the thunderThat deep and dreadful organ-

pipe pronouncedThe name of Prosper It did bass

my trespassTherefore my son ithrsquo ooze is

bedded andIll seek him deeper than eer

plummet soundedAnd with him there lie mudded

(a) In what way does Alonso express his horror when his conscience is awakened by Arielrsquos words

When Alonsorsquos conscience is awakened by Arielrsquos words he expresses his horror at what he has heard He gets the feeling that the waves of the ocean the wind and the loud thunder have spoken to him and uttered the name of Prospero Because of being reminded of his crime in a very loud and rough voice he comes to realize that he has lost his son for his past misdeeds

(b) What does Alonso imagine about his son What does Alonso want to do in his desperate state

Alonso imagines that his son is lying in the mud at the bottom of the sea He feels desperate that he wants to drown himself in the ocean deeper than the plumb-line has ever gone He wants to lie with his son at the bottom of the sea

(c) How do Sebastian and Antonio want to face the evil spirits

Sebastian says that he is not at all afraid of what the harpy has said and that he is prepared to fight any number of such monsters if they appear before him only one at a time Antonio says that he would support Sebastian in the fight against the fiendsyyy

(d) Why does Gonzalo ask Adrian to follow the three men

Gonzalo tells Adrian that all the three men namely Alonso Sebastian and Antonio are in a wild and reckless mood The thought of the heinous crime of which they are guilty has begun to torment their minds So he asks Adrian to follow those three men without loss of time and prevent them from doing anything which the turmoil in their minds might lead them to do

(e) What opinion do you form of Alonso from the above extract

Alonso who is the lesser villain is the first to give way to remorse under the effect of Arielrsquos speech The words of Ariel seem to him to be the voice of conscience speaking to him He is driven to desperation a state in which he might do violence to his life

Subject =Accounts

Ac-12 15420 topic-pL Appropriation ac

PROFIT AND LOSS APPROPRIATION ACCOUNT

MEANING AND PREPARATIONProfit and Loss Appropriation Account is merely an extension of the Profit and Loss Account of the firm The profit of the firm has to be distributed amongst the partners in their respective profit sharing ratio But before its distribution it needs to be adjusted All Adjustments like partnerrsquos salary partnerrsquos commission interest on capital interest on drawings etc are made in this account These adjustments will reduce the amount of profit for distribution This adjusted profit will be distributed amongst the partners in their profit sharing ratio To prepare it at first the balance of Profit and Loss Account is transferred to this account The journal entries for the preparation of Profit and Loss Appropriation Account are given below

1 for transfer of the balance of Profit and Loss Account to Profit and Loss Appropriation Account

(a) In case of Net Profit

Profit and Loss Ac helliphelliphelliphelliphellipDrTo Profit and Loss Appropriation Ac(Net Profit transferred to Profit and Loss Appropriation Ac)

(b)In case of Net Loss

Profit and Loss Appropriation Achelliphelliphellip DrTo Profit and Loss Ac(Net Loss transferred to Profit and Loss Appropriation Ac)

2 for Interest on Capital

For transferring on Interest on CapitalProfit and Loss Appropriation Achelliphelliphellip DrTo Interest on Capital Ac(Interest on capital transferred to Profit amp Loss Appropriation Ac)

3 for Interest on Drawings

For transferring Interest on Drawings Interest on Drawings Achelliphelliphelliphelliphelliphellip DrTo Profit and Loss Appropriation Ac(Interest on drawing transferred to Profit amp Loss Appropriation Ac)

4 For Partnerrsquos SalaryFor transfer of partnerrsquos SalaryProfit and Loss Appropriation Achelliphellip DrTo Salary Ac(Salary transferred to profit amp Loss Appropriation Ac)

5 For Partnerrsquos CommissionFor transferring commissionProfit and Loss Appropriation Achelliphelliphellip DrTo Commission Ac(Commission transferred to Profit and Loss Appropriation Ac)

6 For Transfer of agreed amount to General ReserveProfit and Loss Appropriation Ac helliphellipDrTo General Reserve Ac(Transfer to General Reserve)

7 for share of Profit or Loss appropriation(a) If ProfitProfit and Loss Appropriation Achelliphellip DrTo Partnerrsquos CapitalCurrent Ac(Profit transferred to capitalcurrent Ac)(b) If LossPartnerrsquos Capital Current Achelliphelliphelliphellip DrTo Profit and Loss Appropriation Ac(Loss transferred to capitalcurrent Ac)

THE FORMAT OF PROFIT AND LOSS APPROPRIATION

Profit and Loss Appropriation Account for the year endedhelliphelliphelliphellip

Particulars Amount Particulars Amount

To PL Ac (loss) By pL Ac (profit)

To Interest on capital BY Interest on drawings

To partner`s commission by Partner`s capital Ac ( loss)

To Partner`s salary To Interest on partner`s loan To General Reserve To Partner`s Capital AC (Profit)

Subject= Economics

MOVEMENT ALONG THE DEMAND CURVE (CHANGE IN QUANTITY DEMANDED)In law of demand you have already studied the inverse relationship between price and quantity demanded When quantity demanded of a commodity changes due to change in its price keeping other factors constant it is called change in quantity demanded It is graphically expressed as a movement along the same demand curve There can be either a downward movement or an upward movement along the same demand curve Upward movement along the same demand curve is called contraction of demand or decrease in quantity demanded and downward movement along the same demand curve is known as expansion of demand or increase in quantity demanded

Extention of demandd

price (rs)p A

B Extentionp1 d

Q Q1

Quantity demanded ( in units)

Contraction of demandd

p2 Ccontraction

p APrice (Rs)

d

Q2 Q

Quantity demanded (in units)

Explanation of movement of demand A fall in price from OP to OP1 leads to increase in quantity demanded from OQ to OQ1 (expansion of demand) resulting in a downward movement from point A to point B along the same demand curve DD When Price rises from OP to OP2 quantity demanded falls from OQ to OQ2 (contraction of demand) leading to an upward movement from point A to point C along the same demand curve DD

  • Activity Series of Metals
    • Drawbacks of Rutherfordrsquos model of atom
      • Electromagnetic radiations
      • Properties of electromagnetic radiations
      • Characteristics of electromagnetic radiations
        • Plancks Quantum Theory-
        • Photoelectric effect
          • Intext Questions
            • Logic gates
            • Digital systems are said to be constructed by using logic gates These gates are the AND OR NOT NAND NOR EXOR and EXNOR gates The basic operations are described below with the aid of truth tables
            • AND gate
            • Example
Page 25:  · Web viewSubject. Topic. Summary. Execution. English 1 . Chapter 1 naming words . Page 8. Write the names of these pictures:- Person:-1. father. 2.Firefighter 3.doctor 4 ...

body is to turn it clockwise moment of force is called the clockwise moment and it is taken as negative

Math Topic Commercial Mathematics

Chapter Goods and services Tax

Study item Some solved sums from exercise ndash 1 A retailer buys a TV from a wholesaler for Rs 40000 He marks the price of the TV 15 above his cost price sells it to the consumer at 5 discount on the marked price If the sales are intra ndash state and the rate of GST is 12 find

(i) The marked price of the TV(ii) The amount which the consumer pays for the TV(iii) The amount of tax (under GST) paid by the retailer to the central

Government(iv) The amount of tax (under GST) received by the State Government

Solution As the sales are intra- state sale and the rate of GST 12 So GST comprises of 6 CGST and 6 SGSTTherefore a retailer buys a TV from a wholesaler for Rs 40000Therefore the amount of GST collected wholesaler from the retailer or paid by retailer to wholesalerCGST = 6 of Rs 40000 = Rs(6100 times40000) =Rs 2400SGST = 6 of Rs 40000 = Rs (6100 times 40000) =Rs 2400Therefore wholesaler will pay Rs 2400 as CGST and Rs 2400 as SGSTTherefore amount of input GST of retailer Input CGST = Rs 2400 and input SGST = Rs 2400Again the retailer marks the price of the TV 15 above his cost price(i) The marked price of the TV

= Rs 40000 + Rs 40000times15= Rs 40000 + Rs 40000times 15100= Rs 40000 + Rs 6000Rs 46000But the retailer sells it to consumer at 5 discount on the marked priceCost price after discount = Rs 46000 ndashRs46000times 5100 =Rs 46000 ndashRs 2300= Rs 43700Therefore the amount of GST collected retailer from consumer or paid by consumer to retailerCGST = 6 of Rs 43700 =Rs ( 6100 times43700)Rs 2622SGST = 6 of Rs 43700 = Rs (6100 times 43700) =Rs 2622Amount of the output GST of retailer Output CGST = Rs 2622 and output SGST = Rs 2622

(ii) The amount which the consumer pays for the TV= cost price of TV to consumer + CGST paid by consumer + SGST paid by consumer= Rs 43700 + Rs 2622 + Rs 2622= Rs 48944

(iii) The amount of tax (under GST ) paid by the retailer to the central Government=CGST paid by retailer = output CGST ndash input CGST=Rs 2622 ndash Rs 2400=Rs 222

(iv) The amount of tax ( under GST ) received by the State Government = SGST paid by wholesaler + SGST paid by retailer= Rs 2400 + output SGST ndash input SGST=Rs 2400 + Rs 2622 ndash Rs 2400=Rs 2400 + Rs 222= Rs 2622

Commercial studies

Stakeholders Today I am going to give some revision questions from the previous study material

Questions1) State the two expectations of

employees from a business concern2) Give two distinctions between

stakeholder and shareholder3) Give two difference between

internal stakeholders and external stakeholders

4) Give two expectations of suppliers from a business organisation

5) Who is a stakeholder in commercial organisations

Chemistry Periodic Table

Merits of Mendeleevrsquos Periodic law are as follows - 1He grouped the elements on the basis of atomic mass 2 He left gaps for undiscovered elements like Gallium Scandium germanium Also he left a full group vacant for undiscovered inert gases 3 He could predict proportions of several elements on basis of their position in periodic table like Ga Sc etc 4He could predict errors in atomic weights of some elements like gold platinum etc

Anomalies in Mendeleevrsquos Periodic law are as follows - 1 Position of isotopes could not be explained 2 Wrong order of atomic masses could not be explained

For example- as Arnur atomic mass 40 come first and K with low atomic mass (30) should come later but k should be placed first

According to Bohrrsquos Modern Periodic table properties of elements are periodic functions of their atomic numbers

So when elements are arranged according to increasing atomic numbers there is periodicity in electronic configuration that leads to periodicity in their chemical properties

It consists of horizontal rows (Periods) Vertical column (Groups)

There are 7 period and 12 groups in this long form of periodic table

Ist period has 2 elements IInd period has 8 elements IIIrd period has 8 elements IVth period has 18 elements Vth period has 18 elements VIth period has 32 elements VIIth period hs rest of elements

Note - The number of valence electrons in atom of elements decides which elements will be first in period and which will be last

In group- 1 to 2 gp and 13 to 17 contain normal elements 3 to 12gp ndash transition elements 57 to 71 - lanthanides 89 to 103 - Actinides

Left hand side ndash metals Right hand side ndash nonmetals

Note- Hydrogen element has been placed at top of Ist group Electronic configuration of H is similar to alkali metal as both have 1 valence electron

V electron of gp I element -- 1 V electron of gp 2 element -- 2 V electron of gp 13 element -- 3 V electron of gp 14 element -- 4 V electron of gp 15 element -- 5 V electron of gp 16 element --6 V electron of gp 17 element -- 7 V electron of gp 18 element -- 8

English 1 Transformation of sentences

Sentences A sentence is a group of words which makes complete sense

Exercise 2Change the following sentences from

a Assertive sentencesb Imperative sentencesc Interrogative sentencesd Exclamatory sentences

Sentences can be changed from one grammatical form to another without changing the meaning of the sentence This is known as transformation of sentences

assertive to interrogative1 Nobody would like to be a fool

Who would like to be a fool2 Their glory can never fade

When can the glory fade3 Nobody can control the wind

Who can control the wind4 It matters little if I die

What though I die5 No man can serve two masters

Can any man serve two masters

Exercise 3Interchange of assertive and Exclamatory sentences

1 She leads the most unhappy lifeWhat an unhappy life she leads

2 This is indeed an interesting bookWhat an interesting book this

3 He is a very great manWhat a great man he is

4 It is a very lame excuseWhat a lame excuse

5 It is sad that she died so youngAlas she died so young

Class XISubject Topic Summary Execution

Hindi 2nd lang

पतर परम(परमचदर) पतर परम कहानी म एक निपता की इचछाओ का वणन निकया गया ह अपन बड पतर परभ -ास स निपता चतनय -ास का निवशरष परम था निपता को उसक जनम स ही बडी-बडी आशाए थी उसम दसर बट कतिशव-ास की अपकषा स- उतसाह की मातरा अमिधक थी वह उस इगलड भजकर बरिरसटर बनाना चाहत थभागय का खल भी बडा निनराला ह बीए की परीकषा क बा- वह बीमार पड गया डॉकटरो न भी जवाब - दि-या थाचतन -ास जी बहत ही कजस थ बवजह पस खच करना नही चाहत थ अगर गारटी मिमलती तो शाय- पस खच भी कर -त परत गारटी नही थी परिरणाम सवरप उनक बट का -हात हो गयाजब बट को समशान ल जा रह थ तो वहा काफी शोर गान बजान हो रह थ पछन पर पता चला निक निकसी निपता निपछल तीन साल स निबमार था और उसक ईलाज म रपया पानी की तरह बहाया पर ठीक नही हए परत उसक बट को तनिनक भी अफसोस नही था उसका कहना था उसन कोकतिशश तो कीयह -खकर चतनय-ास जी को आतम निगलानी हईतभी स उनका म परिरवतन हआ और बट का भोज काफी धमधाम स निकयाऔर वहइस पशचाताप की आग म जलत रह औला- स बढकर पसा नही होता ह इस बात को समझन म उनह काफी व लग गया

hellipContinue to next

BENGALI(2ND LANGUAGE)

পরথমঅধযায়-ঠাকরারীনদরনাথঠাকর

নয়ন দোচের হিমাচেররা া নাচেমই হিযাত হিছচেন ায়ানার উাররণ সবরপ নয়ন দোচের ারা হিা (াচেকর হিা হিচেতন এছাাও দেকান উৎস উপচেb রাহিতর দেক হিন করার উচেfচে(য তারা সযC হিকরচেরণ রনয পরীপ জবাহিচেয় তাচেত রপার হির 4Cরণ করচেতন ঠাকরা এই নয়ন দো হিমারচের দে(4 ং(ধর হিছচেন হিমাররা ায়ানার ষটানত পর(Cন কচের তারা হিনঃসব এই হিমাহিরর দে(4 ং(ধর গৈকাস নদর রায়চেৌধরী গৈকাস া নয়ন দোচের সমসত সমপহিতত ঋচেরণর াচেয় হিহিx কচের অহি(ষট যা আচেছ তাচেত হিপত

ইার হিপতার মতয ইচে পর নয়নচোচের ায়ানার দেগাটা কতক অসাধাররণ শরাদধ (াহিনতচেত অহিনতম ীহিপত পরকা( কহিরয়া ঠাৎ হিনহিয়া দেগ- ক) কার দো দেকান গচেলপর অং() কতা দেক ইার চেত কাচেক দোঝাচেনা চেয়চেছ গ) পরসঙগ কী কতার কতয পহিরসফট কচেরা

পরচে4র যাহিত রbা করা সমভ নয় তাই হিতহিন পতরচেক হিনচেয় ককাতায় সাস শর কচেরন গলপ কথচেকর আহিথCক অসথা নয়ন দোচের হিমাচের দেথচেক সমপরণC আাা কথচেকর হিপতা হিনচের দেষটায় অথC উপাCন করচেতন া উপাহিধ াচেভর নয তার াসা হিছনা আর দেসই কারচেরণ কথক তার একমাতর উততরাহিধকার চেয় তার হিপতার পরহিত কতজঞ কথক দো পা হি(চেচেছন হিনচের পরারণ ও মান রbার নয উপচেযাগী অথC হিনা দেষটায় পরাপত চেয়চেছন- এটাই তার কাচেছ পরম দেগৌরচের হি4য় চে মচেন কচেরন কাররণ (নয ভাণডাচের গৈপতক ায়ানার উজজব ইহিতাস অচেপbা দোার হিসeচেকর মচেধয গৈপতক দেকামপাহিনর কাগ তার কাচেছ অচেনক দেহি( মযান

TO BE CONTINUED

উ- ক) আচোয অং(টি রীনদরনাথ ঠাকচেরর দো ঠাকরা গচেলপর অং() কতা চেন আচোয গচেলপর গলপ কথকইার চেত নয়ন দোচের হিমাহিরর দে(4 ং(ধর গৈকাস ার কথা া চেয়চেছ গৈকাস া নয়ন দোচের সমসত সমপহিতত ঋচেরণর াচেয় হিহিx কচের অহি(ষট যা আচেছ তাচেত হিপত পরচে4র যাহিত রbা করা সমভ নয় তাই হিতহিন পতরচেক হিনচেয় ককাতায় সাস শর কচেরনগ) গৈকাস ার হিপতার মতযর পর নয়ন দোচের হিমাহিরর অহিসততব হিপত য় কচেয়কটা উৎস ও শরাদধ- (াহিনতচেত হিমাহিরর দে(4 কহিটক যয় চেয় হিগচেয় এচেক াচের দে(4 চেয় যায় তন তাচের গC করার মত আর হিকছই হিছ না-দেসই পরসচেঙগ এই উহিকত নয়নচোচের হিমাচেররা া নাচেমই হিযাত হিছচেন ায়ানার উাররণ সবরপ নয়নচোচের ারা হিা (াচেকর হিা হিচেতন এছাাও দেকান উৎস উপচেb রাহিতরচেক হিন করচেত হিগচেয় তারা সযC হিকরচেরণর নয পরীপ জবাহিচেয় তাচেত রপার হির 4Cরণ করচেতন তাই দেসকাচের ায়ানা দেহি(হিন সথায়ী চেত পারত না হিহিভনন উৎস শরাদধ- (াহিনতচেত সাধযা হিতহিরকত র করার নয হিমাহির হিহিকচেয় দেযত হ হিতC কা হিহি(ষট পরীচেপর দেত দেযমন অলপকাচের মচেধয হিনঃচে(4 চেয় যায়-নয়নচোচের হিমারচের অসথা তাই চেয়হিছ এই কারচেরণই কথক নয়নচোচের হিমারচের গা ভরা আমবর সয করচেত পারতনা

Physics Dimensional Analysis (Summary)

Q Find the dimensions of consts ab in relation

p=(bminusxlowastx)at

where p is the power x is the distance and t is time

Ans From principle of homogeneity dimension of b x2 are same Dim of b = dim of x2 = [L2] = [ML2T0]Dim of a = dim of ( b- x2)dim of (pt) = [M0L2T0][ML2T-2] [T-1] [T] = [M-1L0T2]

Chemistry Atomic Structure Drawbacks of Rutherfordrsquos model of

atom a According to Rutherfordrsquos model of atom electrons which are negativelycharged particles revolve around the nucleus in fixed orbits Thusb theelectrons undergo acceleration According to electromagnetic theory of Maxwell a charged particle undergoing acceleration should emitelectromagnetic radiation Thus an electron in an orbit should emitradiation Thus the orbit should shrink But this does not happenc The model does not give any information about how electrons aredistributed around nucleus and what are energies of these electrons Isotopes These are the atoms of the same

Properties of electromagnetic radiationsa Oscillating electric and magnetic field are produced by oscillating charged particles These fields are perpendicular to each other and both areperpendicular to the direction of propagation of the waveb They do not need a medium to travel That means they can even travel invacuum

Characteristics of electromagnetic radiationsa Wavelength It may be defined as the distance between two neighbouring crests or troughs of

element having the same atomicnumber but different mass numbere g 1H11H21H3

Isobars Isobars are the atoms of different elements having the same massnumber but different atomic numbere g 18Ar40 20Ca40

Isoelectronic species These are those species which have the same numberof electrons

Electromagnetic radiationsThe radiations which are associated withelectrical and magnetic fields are called electromagnetic radiations When anelectrically charged particle moves under acceleration alternating electricaland magnetic fields are produced and transmitted These fields aretransmitted in the form of waves These waves are called electromagneticwaves or electromagnetic radiations

wave as shown It is denoted by λb Frequency (ν) It may be defined as the number of waves which passthrough a particular point in one secondc Velocity (v) It is defined as the distance travelled by a wave in onesecond In vacuum all types of electromagnetic radiations travel with thesame velocity Its value is 3 times10 8m sec-1 It is denoted by v

d Wave number Wave number is defined as the number of wavelengths per unit lengthVelocity = frequency timeswavelength c = νλ

Plancks Quantum Theory- o The radiant energy is emitted or absorbed not continuously but discontinuously in the form of small discrete packets of energy called lsquoquantumrsquo In case of light the quantum of energy is called a lsquophotonrsquoo The energy of each quantum is directly proportional to the frequency of the radiation ie E α υ or E= hυ where h= Planckrsquos constant = 6626 x 10-27 Js o Energy is always emitted or absorbed as integral multiple of this uantum E=nhυ Where n=1234Black body An ideal body which emits and absorbs all frequencies is calleda black body The radiation emitted by such a body is called black body radiation

Photoelectric effectThe phenomenon of ejection of electrons from thesurface of metal when light of suitable frequency strikes it is calledphotoelectric effect The ejected electrons are called photoelectrons

Biology Chapter - 02Systematics and Five Kingdoms

Scientists divide the whole living organisms into two kingdom first and ultimately by five kingdom at last

In the earlier systems of classifications organisms are divided into kingdom plantaeand kingdom animalia on the of presenceof cell wall their modes of nutrition and movements

Some problem arise like fungi share manycharacteristic withplant despite their heterotrophic nutrition bacteria protozoa areunicellular present in both kingdom Toovercome this third kingdom Protista isintroduced which include

unicellularorganisms But there is also another

problem Allunicellular organisms are not similar kind The cellular structure of prokaryotes is verydifferent from that of other organismsEukaryotes possess a true nucleus and allcell organelles that are not present inprokaryotes So the fourth kingdom Monerais introduced which include unicellular prokaryotes (bacteriaamp blue green algae)

bull Still some problem arise in kingdomplantae

So in 1969 R H Whittakar proposedanew five kingdom System of classification

i) Kingdom Monera - unicellular prokaryotes

ii) kingdom Protista - unicellular eukaryotes

iii) Kingdom Fungi - uni or multicellular fungi with cell wall but without chlorophyll

iv) Kingdom Plantae - Multicellular Plants

v) Kingdom Animalia - Multicellular Animals

EVS Chapter 1 ndash Modes of Existence

An agricultural society

An agricultural society also known as an agrarian society is a society that constructs social order around a reliance upon farming More than half the people living in that society make their living by farming

People in an agricultural society generally lead a more settled lifestyle than those in nomadic hunter-gatherer or semi-nomadic pastoral societies because they live permanently near the land that is farmed Agricultural settlements tend to develop in areas of convenience near bodies of water which is used for both crops and transportation or along trade routes Not everyone in an agricultural society is a farmer Some people make a living trading or making and selling goods such as tools used for farming

Another way to define an agrarian society is to see the total amount of production in a nation In an agrarian society cultivating the land is the main source of wealth Such a society can recognize other means of subsistence and work habits but emphasizes the importance of agriculture and livestock Agrarian societies have existed in various parts of the world for 10000 years and continue to exist today They have been the most common form of socio-economic organization for most of recorded human history

Q) Write the features of agricultural society

Ans - Structure and Features of Agrarian Society1 Occupational Structure

An agrarian society is generally associated with the domestication of plants and animals The domestication of plants means farming and that of animals means herding Often there is mixture of farming and the use of such domesticated animals as cow goat and sheep

2 Forms of Land Ownership in Agrarian SocietiesGenerally there are landlords supervisory farmers cultivators and share croppers The landholders own the land but do not work on it They let it out for sharecropping The supervisory farmers are those who live by having their land cultivated by hired labourers The cultivators cultivate the land for themselvesThe share-croppers are those who live by tilling other peoplersquos land or a crop-sharing basis The artisans own their means of production and produce by their own labour in their homesteads

3 Village Community System An agrarian society is highlighted by

the institution of village community system The agrarian economy made fixed dwelling houses necessary Living close together for protection and co-operation and living nearer to the land gave birth to agricultural villages The village is not only the residential place of farmers it is also the social integrator

4 Minimal Division of Labour Another structural feature of agrarian society is a minimal division of labour Except for the basic division founded on age and sex differences there are few specialized roles There is only one predominant type of occupation ie domestication of plants and animals For all the people the environment physical as well as social is the same

5 Role of Family The farm family is of the patriarchal type the father is the final arbiter in most of the familyrsquos major decisions The life of ail men and women is merged in family life Since there are not many special organizations family is the only organisation to perform the tasks of aid and protection

6 Sense of Unity The members of an agrarian society exhibit a strong in-group feeling Since the whole of their social lives is wrapped up in a society which is physically economically and socially homogenous they are inclined to view the entire outside world as an out group

7 Informal Social Control An agrarian society is regionally divided into villages In a village community the force of traditional mores is more dominant than in the urban community In the village everybody is known to everybody The members in a village community help each other and share the joy and sorrows of each other Crime in an agrarian society is rare

8 Simplicity and Uniformity Life of the people in an agrarian society is marked by simplicity and uniformity Their main occupation is agriculture which largely depends upon the vagaries of nature An agrarian society is a religious society

Math Compound angles Compound angles The algebraic sum of two or more angles is called a compound angle If A B C be three angles then A+B B+C C+A A-B B-C A-C A+B-C etc are compound angles In this chapter we shall discuss the trigonometrical ratios of compound angles Theorem 1 If A B and A+B are all pisitive acute angles theni) sin( A+B) = sin A cos B + cosA sinBii) cos(A+B) = cosA cosB- sinA sinBTheorem 2If A and B are positive acute angles and AgtB theni) sin(A-B) = sin A cosB- cos A sinBii) cos(A-B) = cos A cos B+ sin A sin BTo prove that i) sin(A+B) sin (A-B) = sin2 A - sin2 B = cos2 B- cos2 A

Example 1 Prove that tan70deg=2tan50deg+tan20degSolutiontan70deg = tan(50deg + 20deg)Or tan70deg=(tan 50deg+tan 20deg)(1-tan50degtan20deg) or tan70deg (1 ndash tan 50deg tan20deg) = tan50deg+tan20degor tan70deg= tan70deg tan50deg tan20deg+ tan50deg + tan20deg = cot20deg tan50deg tan20deg + tan50deg + tan20deg = 2 tan50deg+ tan20degExample 2 If A + B = 45deg show that (1 + tanA) (1 + tanB) = 2Solutiontan(A + B) =( tan A + tan B) (1 - tan

ii) cos(A+B) Cos(A-B) = cos2 A- sin2 B = cos2 B -sin2 AProof i) LHS= sin(A+B)sin(AminusB) [Recall sin(αminusβ)=sinαcosβminuscosαsinβ And sin(α+β)=sinαcosβ+cosαsinβ]= (sinAcosB+cosAsinB)times(sinAcosBminuscosAsinB)= sin2Acos2Bminuscos2Asin2B [Recall sin2α+cos2α=1 From above we can then assume correctly that sin2α=1minuscos2α AND cos2α=1minussin2α] = sin2A(1minussin2B)minussin2B(1minussin2A) = sin2Aminussin2Asin2Bminussin2B+sin2Asin2B = sin2Aminussin2B= 1-cos2A-(1-cos2B) = cos2 B- cos2 A = RHSii)LHS= cos (A+B) cos(A-B) [ cos(A+B) = cos AcosB- sinAsinBCos(A-B) = cosAcosB+ sinAsinB]= cos2 A Cos2 B- sin2 A Sin2 B= cos2 A( 1-sin2 B) - (1- cos2 A) sin2 B= cos2 A- cos2 A sin2 B- sin2 B+ cos2 A sin2 B=cos2 A- sin2 B=1- sin2 A-(1-cos2 B) = cos2 B- sin2 A= RHSTangent formulae for compound anglesi)tan (A + B) = tan A + tan B1-tan A tan Bii)tan (A ndash B) = tan A-tan B1+tan A tan Biii) cot (A + B) = cot Acot B-1cot A+cot B(viii) cot (A ndash B) = cot Acot B+1cot B-cot A

A tan B) Or 1= (tan A+ tanB) (1-tan A tanB) Or tanA + tanB + tanA tanB + 1 = 1 + 1Or tanA (1 + tanB) + (1 + tanB) = 2Or (1 + tanA) (1 + tanB) = 2Example 3 Find the value of sin 15degSolution sin 15deg= sin(45deg-30deg) = sin45degcos 30deg- cos45degsin30deg =(1radic2) (radic32) -(1radic2) (12) = (radic3-1) 2radic2Example 4 If sin A = 1 radic10 and sin B = 1 radic5 where A and B are positive acute angles then what is A + B SolutionWe know that sin (A + B) = sin A cos B + cos A sin B= [1 radic10] [radic(1 minus 1 5)] + [1 radic5] radic(1 minus 1 10)= [1 radic10] [radic4 5] + [1 radic5] [radic9 10]= [1 radic50] times (2 + 3)= 5 radic50 = 1 radic2

sin (A + B) = sin π 4rArrHence A + B = π 4Example 5 If A + B = 225o then find [cot A] [1 + cotA] times [cot B] [1 + cot B]Solution[cot A] [1 + cotA] times [cot B] [1 + cot B] = 1 [(1 + tan A) times (1 + tan B)]=1 [tan A + tan B + 1 + tan A tan B] [ tan (A + B) = tan225o]∵

tan A + tan B = 1minus tan A tan BrArr= 1 [1 minus tan A tan B + 1 + tan A tan B]= 1 2

COMMERCE

CLASSIFICTION OF HUMAN ACTIVITIES-ECONOMIC AND NON-ECONOMIC

Firstly we shall recall the previous class for 5 mins especially for the absentees and for also the rest of the students who were there

Today at first we briefly discuss the earlier portions of the chapter

1Business-It includes all those economic activities which are concerned with production and exchange of goods and services with the object of earning profit Example A factory shop beauty parlour also business enterprises

2Profession ndashThe term profession means an occupation which involves application of specialized knowledge and skills to earn a living For Example Chartered Accountancy medicine law tax consultancy are example of professions

Questions1What are the main features of ProfessionAnswer The main features of a profession are as follows a Specialised body of knowledge-Every profession has a specialised and systematised body of knowledge b Restricted entry- Entry to a profession is allowed only to those who have completed the prescribed education and have the specialised examination c Formal education and training ndashA formal education and training is given to the person who wants to acquire the professional

3Employment-Employment mean an economic activity where people work for others in exchange for some remuneration (salary)The persons who work for others are called lsquoemployeesrsquo The persons or organizations which engage others to work for them are called lsquoemployersrsquoEg A doctor working in a hospital is employment as he is working for a salaryA lawyer may serve as a law officer in a bank

With this we shall proceed with the features of both Profession amp Employment

The main features of a profession are as follow

a Specialised body of knowledge b Restricted entry c Formal education and training d Professional association e Service motive f Code of contact

The main features of an employment are as follows

a In employment a person works for others called employer

b An employee provides personal service

c There is a service agreement or contract between the employee and the employer

d The employee has to obey the order of the employer

e No capital investment is made by the employer

Various examples of Employment are as follows

aA teacher teaching in a school or collegeb An engineer employed in Municipal Corporation of DelhicAn accountant working in the accounts department of a companydA doctor working in a hospital

Note In all the above examples of employment the individual who is involved in each example is working as an employee for a salary under an employer

qualification(MBBSCALLB)d Service motive ndashProfessionals are expected to emphasis service more on their clients rather than economic gain f Code of Conduct-The activities of professionals are regulated by a code of conduct

2 What are the main features of EmploymentAnswer The main features of an employment are as followsa In employment a person works for others called employerb An employee provides personal servicec There is a service agreement or contract between the employee and the employerd The employee has to obey the order of the employere No capital investment is made by the employer

3 Give various Professions and their respective Association are given below

Professions

Professional

Professional association

Medical profession

Doctor Medical Council of India

Law profession

Lawyers Bar Council of India

Accounting Profession

Chartered

The Institute of Chartered Accounts of India( ICAI)

Engineerin Engineers The

g Profession

institute of Engineers (India)

Accounts Basic accounting terms

Today we will give you some questions from the previous study material

Questions6) Define accounting7) What do you mean by debit

and credit8) Explain the types of account9) Define the following terms

a) Assetsb) Capitalc) Purchased) Debtorse) Transactions

10) Name the types of accounts given below

a) Krishnas accountb) Machinery accountc) Royalty accountd) Salary accounte) Furniture accountf) Audit fee account

Economics Basic Economic ConceptsSub topic

UTILITY

Before starting todayrsquos class we shall recall the last class which was about UTILITY AND THE FEATURES OF UTILITY

Now we shall proceed with the further topics of the chapter

Todayrsquos topic from the chapter lsquo Basic Economic Conceptsrsquo will be TOTAL UTILITY amp MARGINAL UTILITYNow let us quickly revise the concept of utility with an example ie goods and services are designed because they have an ability to satisfy human wantsThis feature of being able to satisfy human wants is termed as utility For example we derive utility from WiFi services as it gives us satisfaction by connecting us to our friends and family through social media here consumers derive utility from WiFi services

From the above concept we shall start with todayrsquos topicEconomists have defined TOTAL UTILITY (TU) as the total satisfaction obtained by consuming a given total amount of a good and serviceFor example the total satisfaction obtained from eating 10 mangoes is the total utility of 10 mangoes

MARGINAL UTILITY (MU) is the additional satisfaction derived from each additional unit

Questions1 What is Total Utility (TU)

Answer Total Utility (TU) is the

aggregate of the utility that a consumer derives from the consumption of a certain amount of a commodityTU=MU1+MU2++MUn

2 What is Marginal UtilityAnswer

Marginal Utility (MU) is the additional made to the total utility as consumption is increased by one more unit of the commodityMU= TUn ndashTUn-1

NoteOften economists tend to

subdivide utility into an imaginary unit called UTIL

consumed In this casethe utility obtained from each mango as it is consumed as the MU of that mango It is also defined as the addition made to the total utility when an additional unit is consumed Often economists tend to subdivide utility into an imaginary unit called UTIL

Note As a consumer increases the consumption of a good over period of time the total utility or total satisfaction derived from it increases to appoint and thereafter it decreasesHowever as the consumer keeps on consuming the good the marginal utility or the additional utility derived from it decreases

SubjectBusiness studies

Topic

BUSINESSENVIRONMENT

Summary

Now quickly let us revise the earlier points that we have already done in the last class and let us proceed with the other topics that are there in the chapter

Firstly we will recall the internal and external factors of micro environment and then we shall proceed in details

Meaning and list of internal and external factors

aInternal factorsInternal factors refer to all the factors existing within a business firm The internal factors are considered controllable because the enterprise has control over these factorsFor an example a company can alter its organization structure policies programmes employees physical facilities and marketing mix to suit the changes in the environmentList of internal factors areCorporate culture mission and objectives top management organizations structure company image and brand equity company resources

b External factorsExternal factors refer to those individual and groups and agencies with which a particular business organization comes into direct and frequent contact in the course of its functioningThese individuals and groups are known as STAKEHOLDERS because they have a stake (financial interest ) in the working and performance of the particular business List of external forces (stakeholders)Customers competitors investors suppliersmiddlemen (marketing intermediaries)

Execution 1 What do you mean by internal

factors in micro environmentAnswerInternal factors refer to all the factors existing within a business firm The internal factors are considered controllable because the enterprise has control over these factorsFor an example a company can alter its organization structure policies programmes employees physical facilities and marketing mix to suit the changes in the environment

2 What do you mean by external factors in micro environment

AnswerExternal factors refer to those individual and groups and agencies with which a particular business organization comes into direct and frequent contact in the course of its functioningThese individuals and groups are known as STAKEHOLDERS because they have a stake (financial interest) in the working and performance of the particular business

3Who are stakeholdersSTAKEHOLDERS are individuals and groups who have a stake (financial interest ) in the working and performance of the particular business 4Discuss the internal factors in briefa Corporate CultureThe values beliefs and attitudes of the founders and top management of the company exercise

financers publics

customers

suppliersfinancers

competitors

middlemen

publics

Fig STAKEHOLDERS OF A COMPANY

Apart from micro environment the other main dimension of business environment isMacro environment Macro environment refers to the general environment or remote environment within which a business firm and forces in its micro environment operateA company does not directly or regularly interact with the micro environmentTherefore macro environment is also known as indirect action EnvironmentThe macro environment forces are less controllable than the micro forces

Macro environment consists of the following components

POLITICAL AND LEGAL ENVIRONMENT

ECONOMIC SOCIAL AND ENVIRONMENT

CULTURAL

ENVIRONMENT

TECHNOLOGICAL ENVIRONMENT

a strong influence on what the cmpaany stands for how it does things and what it considers importantbMission and objectivesThe business philosophy and purpose of a comoany guide it prioritiesbusiness strategiesproduct market scope and development scope

cTop management structurethe composition of board of directors the degree of professionalization of management and the organizational structure of a company have important bearing on its business decisions

dPower structureThe internal power relationship between the board of directors and the chief executive is an important factor

eCompany image and brand equityThe image and brand equity of the company play a significant role in raising finance forming alliance choosing dealers and suppliers launching new products entering foreign markets

5 What is Macro environmentAnswerMacro environment refers to the general environment or remote environment within which a business firm and forces in its micro environment operateA company does not directly or regularly interact with the micro environmentTherefore macro environment is also known as indirect action EnvironmentThe macro environment forces are less controllable than the micro forces 6 What are the components of macro environmenta Political and legal environmentb Economic environmentc Social and cultural environmentd Technological environment

BUSINESS FIRM

Fig COMPONENTS OF MACRO ENVIRONMENTPolitical science

Introduction to political science

Comparative politics and itrsquos scope Comparative politics is the second major dimension of political scienceIt is also a very vast area of study and a very large number of political scientists even treat it as an autonomous area of study within the board ambit of political scienceScope of comparative politics-

1 All political structures -Comparative politics includes the study of all structures formalnon formal governmental and extra governmental which are directly or indirectly involved in politics in all the countries of the world

2 Functional studies- Comparative politics seeks to study politics less from the point of view of the legal institutions in terms of their powers and move from the point of view of their functions which constitute the political process and their actual Operation in the environment

3 Study of political behaviour- Another important part of its scope is the study of the actual behaviour of the people in the process of politics

4 Study of similarities and differences- comparative politics also undertakesan analysis of the similarities and differences among political process and functions

5 Study of all political systems -comparative politics seeks to analyse the actual behaviour and performance of all political systems western as well as non western

6 Study of the environment and infrastructure of politics-The study of politics demands a study of the psychological sociological economic and anthropological environment in fact the social environment as a whole in which each political system operates

7 Study of political culture- political culture is composed of attitudesbeliefs emotions and values of a society that relate to the political system or politics

8 Study of political participation- Political participation is a universal processThe only difference is that while in some states it is limited in others it is wider

9 Study of political process- political

Answer the following questions-

What is comparative politics

What are the scope of comparative politics

Homework- learn

processes like decision makingpolicy making judicial process leadership recruitment process and others are always at work in all political systems

The scope of comparative politics is very comprehensive It includes everything that falls within the area of political activity and political process

History CAMBRIDGE VIEW ABOUT

THE PARTITION

AND REFUTATION

OF CAMBRIDGE

VIEW

Cambridge view about the Partition The Cambridge school of historians have interpreted that opposition to partition scheme was made entirely by the elitist groups They hold the view that Lord Curzon planned to partition the Bengal for administrative purposeREFUTATION OFCAMBRIDGE VIEW The Rationalist historians have rejected the interpretations of the Cambridge School of historians on various grounds

1 QUESTION State different views of historians regarding Partition of Bengal

ANSWER Cambridge historians believed that Lord Curzon partitioned Bengal for administrative reasons only and not for the political motive The Middle class elitist group protested because of their petty interest The Hindu zamindars protested as they have to spend more money for managing their estatesThe lawyers of Calcutta High court feared to lose their clientBut according to the nationalist Historians was-

2- The ultimate object of Lord Curzon was to crush the unity of Bengal politicians

3- If Bengal becomes a separate province Bengali speaking 16 million people of western part would become minority under Hindi speaking people of Bihar and Oriya speaking people of Orissa

4- The bureaucrats expected that the protest movement would die down quickly

5- Lord Curzon used the Muslim community in his political game

6- Idealism had great contribution in the protest against partition

7- The people of the every section of society were affected by the partition of Bengal

Computer Science

Numbers Convertion of dcimal number to octal numberThe decimal numeral system is the standard system for denoting integer and non-integer numbers It is the extension to non-integer numbers of the Hindu-Arabic numeral system For writing numbers the decimal system uses ten decimal digits a decimal mark and for negative numbers a minus sign - The decimal digits are 0 1 2 3 4 5 6 7 8 9 the decimal separator is the dot in many countries

The octal numeral system or oct for short is the base-8 number system and uses the digits 0 to 7 Octal is sometimes used in computing instead of hexadecimal perhaps most often in modern times in conjunction with file

permissions under Unix systems It has the advantage of not requiring any extra symbols as digits It is also used for digital displays

Follow these steps to convert a decimal number into octal form

1 Divide the decimal number by 82 Get the integer quotient for the next iteration (if the number will not divide equally by 8 then round down the

result to the nearest whole number)3 Keep a note of the remainder it should be between 0 and 74 Repeat the steps until the quotient is equal to 05 Write out all the remainders from bottom to top This is the solution

For example if the given decimal number is 8453

Division Quotient Remainder

8453 8 1056 5

1056 8 132 0

132 8 16 4

16 8 2 0

2 8 0 2

Then the octal solution is 20405

Subject Eng Literature (The Tempest ndash William Shakespeare) Topic Act I Scene 1 Lines 33 to 67 (End of scene) Date 16th April 2020 (4th Period)

[Students should read the original play and also the paraphrase given in the school prescribed textbook]Summary Questions amp Answers

[SUMMARY OF THE ENTIRE SCENE]

o The play starts with the scene of a severe storm at sea Alonso (King of Naples) Sebastian (Alonsorsquos brother) Ferdinand (Alonsorsquos son) Gonzalo Antonio (the usurping Duke of Milan) are in a ship in the midst of the storm

o The mariners are trying their best to control the vessel from running aground and are totally following the orders of their Master the Boatswain They have scant success

o The mariners become extremely unhappy and annoyed when most of the passengers arrive on the deck thereby hampering their effort to save the ship There is serious confrontation between them and the passengers who are part of the Kingrsquos entourage

o The mariners could not save the ship

SUMMING-UP

(i) Vivid description of the scene which gives a realistic description of terror and confusion of a tropical storm

(ii) Shows Shakespearersquos accuracy of knowledge in describing the naval operations and also matters of seamanship

(1) GONZALO Ill warrant him for drowning (L 45-57)

though the ship were no stronger than a nutshell and as leaky as an unstanched

wenchBOATSWAIN Lay her a-hold a-hold Set her two courses Off to

sea again lay her offMARINERS All lost To prayers to prayers All lostBOATSWAIN What must our mouths be coldGONZALO The king and prince at prayers Lets assist them

For our case is theirsSEBASTIAN Im out of patienceANTONIO We are merely cheated of our lives by drunkards

This wide-chopped rascal - would thou mightst lie drowning the washing of ten tides

(a) What does Antonio say at the insolent manners of the boatswain just before the given passage

Being irritated at the insolent manners of the boatswain just before the given extract Antonio the Duke of Milan calls him a worthless dog son of a woman without any morals an arrogant and disrespectful noisemaker He says that the boatswain deserved to be hanged(b) What statement does Gonzalo repeat about the boatswain

Gonzalo shows his faith that the boatswain is not destined to die by drowning He is destined to be hanged and nothing can alter this decree of destiny He says that even if the ship was as frail as a nutshell the boatswain could not be drowned for his destiny was to be hanged(c) What do the passengers do when they have lost all hope of their survival

When the passengers have lost all hope of survival they take

(iii) The opening scene justifies the title ndash The Tempest

UNANSWERED QUESTIONS

(i) The King always travels with his entire fleet including his soldiers Where were the other ships

(ii) Why was the ship in that area Where was it coming from or going where

(iii) The ship broke apart What happened to those who were in the ship

(We shall get the answer to the above questions as the play progresses)

leave of life with fervent prayers The mariners take their last hearty drink and are ready for death(d) What blame does Antonio put upon the mariners and the boatswain Antonio rebukes the mariners that these drunkards have brought them to the present crisis by neglecting their duties He blames them saying that they are going to lose their lives entirely for the negligence of the boatswain and his fellows(e) What does Antonio say while cursing the boatswain

Antonio gives vent to his wrath upon the boatswain in particular He calls the boatswain a wide-mouthed rascal who deserves to be hanged on the sea-shore at low water mark so that ten tides might wash over his body and take out of him all the liquor that he has been drinking

Class XIISubject Topic Summary ExecutionHistory Topic

1 1935 ACT AND WORKING OF PROVINCIAL AUTONOMYCONGREE AND OTHER MINISTERSSUB TOPIC GOVERNMENT OF INDIA ACT1935

Government of India Act 1935 This act established a lsquoFederation of Indiarsquo made of British Indian provinces and Indian states and provided for autonomy with a government responsible to the elected legislature in every provinceThis act introduced abolition of Diarchy at provinces The entire provincial administration was introduced to the responsible ministers who were controlled and removed by the provincial legislature The provincial autonomy means two things First The provincial governments were wholly responsible to the provincial legislature Secondly Provinces were free from outside control and interference in the large number of matters The act divided the powers between the centre and provinces in terms of three lists- Federal list( for centre) Provincial list (for province) and concurrent list (for both) Residuary powers were given to the viceroy In the election under the government of India Act the Congress swept the poll the mandate of the people came in favour of the congress so far as general Hindu seats were concerned The Congress did not get a single Muslim seates in Bombay CP UP Sind and BengalIn five provinces Congress had yhe clear majority In BengalNWFPAssam and Bombay Congress emerged as a single largest partyOn the other side the performance of the Muslim League was badThus the Congress formed ministers in 7 provinces out of 11 provinces Coalition ministry was also formed in two other provincesOnly BENGAL AND Punjab had non- congress ministries

1 QUESTION What was the main change introduced by the Government of India ActANSWER a) The Act gave more

autonomy to the provinces b) Diarchy was abolished at the

provincial levelsc) The Governor was the head of

the executived) There was a council of

ministers to advise him The ministers were responsible to the provincial legislatures who controlled them The legislature could also remove the ministers

e) The Governors still retained special reserve powers

2 QUESTION Why did the federal scheme introduced by the Government of India Act 1935 never come into operation

ANSWER The Federal structure of the Government of India was to be composed with the Governor General and Council of ministers The Federal legislature was to be Bicameral legislature- The council of states and the House of Assembly The ministers were to be chosen by the Governor general and they were to hold the office during his pleasure

The provinces of British India would have to join the federation but this was not compulsory for the princely states

This federation never materialised because of the lack of support from the required number of

princely statesThis act was refused and

rejected by the princes the Congress and the Muslim League

Thus both Congress and the League participated in the election of 1937 Thus the federal part was never introduced but the provincial part was put into operations

Bengali 2nd

Language

াচেরর পরাথCনা(কহিতা )

াচেরর পরাথCনা কহিতাটি কহি (ঙখ দেঘাচে4র দো আচো য কহিতায় াচেরর পতর হমায়ন কঠিন দেরাচেগ আxানত ার ঈশবর া আললার কাচেছ পরাথCনা কচেরচেছন তার পচেতরর ীন হিফহিরচেয় হিচেত এই কহিতায় ার পচেতরর ীন হিভbা দেচেয়চেছন ারার এমনহিক হিনচের ীন হিসCচেনর হিহিনমচেয় হিতহিন তার দেছচের ীন হিফচের দেপচেত দেচেয়চেছন তার দেছচের এই দেরাচেগর ন য হিতহিন হিনচেচেকই ায়ী কচেরচেছন তার হিনচের করা পাপচেকই হিতহিন ায়ী কচেরচেছন এছাা রানৈনহিতক ও আথCসামাহিক অসথার কথা তচে ধরা চেয়চেছ এই কহিতায় ার তার হিনচের পাপ কমCচেকই ায়ী কচেরচেছ ার অন যায় ভাচে দেপহি((হিকতর মাধ যচেম অপররা য কচেরচেছ আর এই অন যায় কাচের ন যই তার পহিরাচের হিপযCয় এচেসচেছ দে এক পরকার মানহিক নধন ইহিতাচেসর ার হিপতা চেয় সবাভাহিকভাচে ভাচোাসা দে মমতা দেথচেক মকত চেত পাচেরনহিন তাই হিপতা চেয় আললা া ভগাচেনর কাচেছ পতর হমায়চেনর পরানহিভbা দেচেয়চেছন ার আললা া ভগাচেনর কাচেছ াহিনচেয়চেছন তার হিনচের ীন হিসCন হিচেত হিতহিন রাী তার হিহিনমচেয় পচেতরর ীন হিফচের দেপচেত দেচেয়চেছন াচেরর হিপতসভ হিচেকর কথা এই কহিতায় ফটিচেয় দেতাা চেয়চেছ হিপতা পচেতরর হিরাহিরত মান নধচেনর কথা তচে ধরা চেয়চেছ

হিচে(4 হিকছ াইচেনর তাৎপযC১) ldquoদেকাথায় দেগ ওর সবচছয দেৌন দেকাথায় কচেরায় দেগাপন bয়ldquoউততর) াচেরর পতর হমায়ন কঠিন দেরাচেগ অসসথ তাই তার দেযৌন াহিরচেয় যাচেচছ এই দেরাচেগ তাচেক দেগাপচেন কচেরকচের াচেচছ তার সক (হিকত ধীচের ধীচের bয় চেচছ তাই হিপতা চেয় ার আললার কাচেছ হমায়চেনর পরান হিভbা দেচেয়চেছন২) ldquoাগাও (চেরর পরাচেনত পরানতচের ধসর (ন দেযর আান গানldquoউততর) াচেরর পতর হমায়ন কঠিন দেরাচেগ আxানত তাই ার আ দে(াচেক মমCাত (চেরর পচেথ পরানতচের আান গান ধবহিনত দোক দেসই আান গান আললার কাচেছ দেযন চে যায় আললা দেযন এই আহিতC শচেন পচেতরর ীন হিফহিরচেয় দেয় ৩)ldquoনাহিক এই (রীচেরর পাচেপর ীানচেত দেকানই তরারণ দেনই ভহি4চেতরldquoউততর) হমায়চেনর অসসথতার ন য ার হিনচেচেকই ায়ী কচেরচেছন কারন ার অচেনক রা য অন যায় ভাচে কচেরচেছ তাই তার এই পাপ কাচের ন য তার ঘচের আ হিপ এচেসচেছ এই অন যায় কাচের ন য তার মহিকত দেনই তাই ার আললার কাচেছ এই পাপ কাচেযCর ন য bমা পরাথM

Hindi 2ndlang

-ासी(जयशकर परसा-)

-ासी जयशकर परसा- की एक ऐसी कहानी ह जिजसम भारतीय ससकनित और राषटरीयता का सवरगजीतहोता ह इस कहानी म इरावती एक निहद कनया ह जिजस मलअचछो न मलतान की लट म पकडा और -ासी बना दि-या उस 500 दि-न -कर काशी क एक महाजन न खरी-ा दसरी -ासी निफरोजा ह वह गलाम ह निफरोजा को छडान क कतिलए अहम- को 1000 सोन क कतिसकक भजन थ जो अभी तक नही आए थ राजा साहब कठोर होत हए भी निफरोजा को निबना धनराकतिश क कतिलए उस म कर -त ह वनिफरोजा को अहम- को समझान की बात कहत हकहानी क अत म हम -खत ह निक इरा वती और जाटो क सर-ार बलराज का मिमलन होता हअहम- को यa म मार दि-या जाता ह वहा निफरोजा की परसननता की समामिध बनती ह वहा एक फल चढती ह और डीजल आती ह निफरोजा उस समामिध की आजीवन -ासी बनी रहती हलखक अपन उददशय अथात -ास परथा पर परकाश डालन और इस परथा क कारण होन वाल -ातो क दखो को दि-खान म पणता सफल हए ह

helliphellipContinue to next

Biology Reproductio Today we will discuss about vegetative Q1 Name some vegetative propagules

n in Organisms

propagation of plants The process of multiplication in which fragments of plant body function as propagule and develop into new individual is called vegetative propagation The units of such propagation are runner rhizome tuber bulb etc

and the speciesinvolvedVegetative propagules

Parts involved

Bulb StemBulbil BulbilRhizome Stem Runner Stem Tuber Stem Offset Stem Leaf buds Leaves Suckers Stem

Corns Stem stolon

Q2 State advantages of vegetative propagation

i) Rapid methodii) Sure and easy methodiii) Useful in plants that cannot

produce viable seeds or long seed dormancy

iv) Maintains purity of raceQ 3 Banana fruit is said to be parthenocarpic where as turkey is said to be parthenogenetic WhyBanana develops without fertilization from an unfertilized ovary thus is parthenocarpicIn turkey the ovum or female gamete developinto a new chick without fertilization thus isparthgenetic

Q4 Why is water hyacinth is called as a ldquoTerror of Bengalrdquo Water hyacinth can

propagatevegetatively all over the water body in a short per short period of time This resulted increased biochemicaloxygen oxygen demand of water body causing mortalityof fishes It is very difficult to get rid off them Thus known as terror of Bengal

Chemistry

Solid state GENERAL CHARACTERISTICS OF SOLID STATEIn nature the particular state of matter is governed by two opposing forces at given set of temperature and pressure These forces are intermolecular force of attraction and thermal energy If intermolecular force of attraction is high as compared to thermal energy particles remains in closest position

Intext QuestionsQ1 Classify the following solids as crystalline and amorphous Sodium chloride quartz glass quartz rubber polyvinyl chloride Teflon

A1 Crystalline

and hence very less movement in particles is observed In this case solid state is the preferred state of matter

Let us revise the general characteristics of solid

i) Fixed mass volume and shape

ii) Strong intermolecular force of attraction

iii) Least intermolecular space

iv) Fixed position of constituent particles

v) Incompressible and rigid

Q2 what type of interactions hold the molecules together in a polar molecular solid[CBSE 2010]A2 The molecules in a solid are held together by van der Waals forces The term van der Waals forces include hydrogen bonding dipole-dipole attraction and London dispersion forces All molecules experience London dispersion forces In addition polar molecules can also experience dipole-dipole interactions So the interactions that holds the molecule together in polar molecular solid are London dispersion force and dipole-dipole interactionsQ3 Write a feature that will distinguish a metallic solid from an ionic solid [CBSE 2010]A3 Metals are malleable and ductile whereas ionic solid are hard and brittle Metallic solid has typical metallic lustre But ionic solid looks dullQ4 Write a point of distinction between a metallic solid and an ionic solid other than metallic lustre [CBSE 2012]A4 Metals are malleable and ductile whereas ionic solid are hard and brittleQ5 Write a distinguish feature of metallic solid [CBSE 2010]A5 The force of attraction in

solid Sodium chloride Quartz Amorphous solid Quartz glass rubber polyvinyl chloride Teflon Q2 why glass is considered as super cooled liquidA2 Glass shows the tendency to flow at slower rate like liquid Hence they considered as super cooled liquidQ3 why the window glass of old buildings show milky appearance with timeA3 Glass is an amorphous solid Amorphous solid has the tendency to develop some crystalline character on heating Due to heating in day over the number of years glass acquires some crystalline character and show milky appearanceQ4 why the glass panes fixed to window or doors of old building become slightly thicker at bottomA4 Glass is super cooled liquid It has the tendency to flow down very slowly Due to this glass pane becomes thicker at the bottom over the timeQ5 Sodium chloride is a crystalline solid It shows the same value of refractive index along all the direction TrueFalse Give reasonA5 FalseCrystalline solid shows anisotropy in properties That is it shows different values for the given physical property in different direction All the crystalline solids show anisotropy in refractive index Therefore sodium chloride will show different values of refractive index on different directions

Q6 Crystalline solid are anisotropic in nature What does this statement means

between the constituent particles is special kind of electrostatic attraction That is the attraction of positively charged kernel with sea of delocalized electronsQ6 which group of solid is electrical conductor as well as malleable and ductile [CBSE 2013]A6 Metallic solidQ7 why graphite is good conductor of electricity although it is a network (covalent solid)A7 The exceptional property of graphite is due to its typical structure In graphite each carbon is covalently bonded with 3 atoms in same layer The fourth valence electron of each atom is free to move in between different layersThis free electron makes the graphite a good conductor of electricity

[CBSE 2011]A6 Anisotropy is defined asrdquo Difference in properties when measured along different axis or from different directionsrdquo Crystalline solid show different values of some of the physical properties like electrical resistance refractive index etcwhen measured along the different directions The anisotropy in crystalline solid arises due to the different arrangement of particles in different directions

Math Function Composition of functions Think of an industrial plant that produce bottles of cold drinks first there is the operation (or function) f that puts the cold drink inside the bottle followed by the opeartion g that close the bottle with the capThis leads to the following definitionDefinition Let f A rarr B and g B rarr C be two functions Then the composition of f and g denoted by gof is defined as the function gof A rarr C given by gof(x) = g(f (x)) forall x isinA

Definition A function f X rarr Y is defined to be invertible if there exists a function g Y rarr X such that gof = IX and fog = IY The function g is called the inverse of f and is denoted by f -1

Thus if f is invertible then f must be one-one and onto and conversely if f is one-one and onto then f must be invertible This fact significantly helps for proving a function f to be invertible by showing that f is one-one and onto specially when the actual inverse of f is not to be determined

Example 1 Let f 2 3 4 5 rarr 3 4 5 9 and g 3 4 5 9 rarr 7 11 15 be functions defined as f(2) = 3 f(3) = 4 f(4) = f(5) = 5 and g (3) = g (4) = 7 and g (5) = g (9) = 11 Find gofSolution We have gof(2) = g (f(2)) = g (3) = 7 gof(3) = g (f(3)) = g (4) = 7gof(4) = g (f(4)) = g (5) = 11 and gof(5) = g (5) = 11Example 2 Find gof and fog if f R rarr R and g R rarr R are given by f(x) = cos x and g (x) = 3x2 Show that gof ne fogSolution We have gof(x) = g(f(x))=g(cosx) = 3 (cos x)2

= 3 cos2 x Similarly fog(x)=f(g (x))= f(3x2)= cos (3x2) Note that 3cos2 x ne cos 3x2 for x = 0 Hence gof ne fogExample 3 Show that if f A rarr B and g B rarr C are onto then gof A rarr C is also ontoSolution Given an arbitrary element z isin C there exists a pre-image y of z under g such that g (y) = z since g is onto Further for y isin B there exists an element x in A with f(x) = y since f is onto Therefore gof(x) = g (f(x)) = g (y) = z showing that gof is onto Example 4 Let Y = n2 n isin N sub N Consider f N rarr Y as f(n) = n2 Show that

f is invertible Find the inverse of fSolution An arbitrary element y in Y is of the form n2 for some n isin N This implies that n =radicy This gives a function g Y rarr N defined by g (y) =radicy Nowgof (n) = g (n2)=radicn2 = n and fog (y) =f(radicy) = (radicy) 2 y which shows that gof=IN and fog= IY Hence f is invertible with f -1 = g

Political Science

Constitution of India-The Preamble

Summary

Objective of the state-To secure equality of status and of opportunity To promote fraternity among all the citizens To assure the dignity of the individuals and Unity and integrity of the nation

Justice-Justice stands for rule of law absence of arbitrariness and a system of equal rights freedom and opportunities for all in a society India seeks social economic and political justice to ensure equality to its citizens

Liberty-Liberty implies the absence of restraints or domination on the activities of an individual such as freedom from slavery serfdom imprisonment despotism etc The Preamble provides for the liberty of thought expression belief faith and worship

Equality-Equality means the absence of privileges or discrimination against any section of the society The Preamble provides for equality of status and opportunity to all the people of the country

Fraternity-The Preamble declares that fraternity has to assure two thingsmdashthe dignity of the individual and the unity and

Execution

Answer the following questions-

Short notes-1 Equality2 Fraternity3 Justice4 Liberty

Homework-Learn

integrity of the nation The word integrity has been added to the Preamble by the 42nd Constitutional Amendment (1976)

Business studies

Human resource management (chapter 1)

On the day of 1504 2020 I have discussed with you the managerial functions and procurement functions of HRM

Today weare going to discuss about the development function integration functions and maintenance function

Development functions-HRM improves the knowledge skills attitude and values of employees so that they the present and future jobs more effectively it includes

1) Development functions of HRM

a) Performance appraisal = It implies systematic evaluation of employees with respect to their performance on the job and their potential for development

b) Training =It is the process by which employees learn knowledge skills and attitudes to achieve organisational and personal goals

c) Executive development = It is the process of developing managerial talent through appropriate program

2) Integration functionsa) HRM reconcile the goals of

organisation with those of its members through integrating function

b) HRM tries to motivate employees to various financial and non financial incentives provided in job specification etc

3) Maintenance functiona) HRM promote and protect the

physical and mental health of employees by providing several types of benefits like housing medical aid etc

b) It Promote Social security measures to employees by providing provident fund pension gratuity maternity benefits

SubjectCOMMERCE

Topic

BUSINESSENVIRONMENT

Summary

Now quickly let us revise the earlier points that we have already done in the last class and let us proceed with the other topics that are there in the chapter

Firstly we will recall the internal and external factors of micro environment and then we

Execution 3 What do you mean by internal factors

in micro environmentAnswerInternal factors refer to all the factors existing within a business firm The internal factors are considered controllable because the enterprise has control over these factors

Development FunctionsPerformance AppraisalTrainingExecution Development

shall proceed in details

Meaning and list of internal and external factors

aInternal factorsInternal factors refer to all the factors existing within a business firm The internal factors are considered controllable because the enterprise has control over these factorsFor an example a company can alter its organization structure policies programmes employees physical facilities and marketing mix to suit the changes in the environmentList of internal factors areCorporate culture mission and objectives top management organizations structure company image and brand equity company resources

b External factorsExternal factors refer to those individual and groups and agencies with which a particular business organization comes into direct and frequent contact in the course of its functioningThese individuals and groups are known as STAKEHOLDERS because they have a stake (financial interest ) in the working and performance of the particular business List of external forces (stakeholders)Customers competitors investors suppliersmiddlemen (marketing intermediaries)financers publics

customers

suppliersfinancers

For an example a company can alter its organization structure policies programmes employees physical facilities and marketing mix to suit the changes in the environment

4 What do you mean by external factors in micro environment

AnswerExternal factors refer to those individual and groups and agencies with which a particular business organization comes into direct and frequent contact in the course of its functioningThese individuals and groups are known as STAKEHOLDERS because they have a stake (financial interest) in the working and performance of the particular business

3Who are stakeholdersSTAKEHOLDERS are individuals and groups who have a stake (financial interest ) in the working and performance of the particular business 4Discuss the internal factors in briefa Corporate CultureThe values beliefs and attitudes of the founders and top management of the company exercise a strong influence on what the cmpaany stands for how it does things and what it considers importantbMission and objectivesThe business philosophy and purpose of a comoany guide it prioritiesbusiness strategiesproduct market scope and development scope

cTop management structurethe composition of board of directors the degree of professionalization of management and the organizational structure of a company have important bearing on its business decisions

dPower structureThe internal power relationship between the board of directors and the chief executive is an important factor

e Company image and brand equityThe image and brand equity of the company play a significant role in raising finance forming alliance choosing dealers and suppliers launching new products entering foreign markets

5 What is Macro environmentAnswerMacro environment refers to the general

competitors

middlemen

publics

Fig STAKEHOLDERS OF A COMPANY

Apart from micro environment the other main dimension of business environment isMacro environment Macro environment refers to the general environment or remote environment within which a business firm and forces in its micro environment operateA company does not directly or regularly interact with the micro environmentTherefore macro environment is also known as indirect action EnvironmentThe macro environment forces are less controllable than the micro forces

Macro environment consists of the following components

POLITICAL AND LEGAL ENVIRONMENT

ECONOMIC SOCIAL AND ENVIRONMENT

CULTURAL

ENVIRONMENT

TECHNOLOGICAL ENVIRONMENT

Fig COMPONENTS OF MACRO ENVIRONMENT

environment or remote environment within which a business firm and forces in its micro environment operateA company does not directly or regularly interact with the micro environmentTherefore macro environment is also known as indirect action EnvironmentThe macro environment forces are less controllable than the micro forces 6 What are the components of macro environmenta Political and legal environmentb Economic environmentc Social and cultural environmentd Technological environment

Computer Science

Logic gates

Digital systems are said to be constructed by using logic gates These gates are the AND OR NOT NAND NOR EXOR and EXNOR

BUSINESS FIRM

gates The basic operations are described below with the aid of truth tables

AND gate

The AND gate is an electronic circuit that gives a high output (1) only if all its inputs are high A dot () is used to show the AND operation ie AB Bear in mind that this dot is sometimes omitted ie ABOR gate

The OR gate is an electronic circuit that gives a high output (1) if one or more of its inputs are high A plus (+) is used to show the OR operationNOT gate

The NOT gate is an electronic circuit that produces an inverted version of the input at its output It is also known as an inverter If the input variable is A the inverted output is known as NOT A This is also shown as A or A with a bar over the top as shown at the outputs The diagrams below show two ways that the NAND logic gate can be configured to produce a NOT gate It can also be done using NOR logic gates in the same way

NAND gate

This is a NOT-AND gate which is equal to an AND gate followed by a NOT gate The outputs of all NAND gates are high if any of the inputs are low The symbol is an AND gate with a small circle on the output The small circle represents inversion

NOR gate

This is a NOT-OR gate which is equal to an OR gate followed by a NOT gate The outputs of all NOR gates are low if any of the inputs are highThe symbol is an OR gate with a small circle on the output The small circle represents inversion

EXOR gate

The Exclusive-OR gate is a circuit which will give a high output if either but not both of its two inputs are high An encircled plus sign ( ) is used to show the EOR operation

EXNOR gate

The Exclusive-NOR gate circuit does the opposite to the EOR gate It will give a low output if either but not both of its two inputs are high The symbol is an EXOR gate with a small circle on the output The small circle represents inversion The NAND and NOR gates are called universal functions since with either one the AND and OR functions and NOT can be generated

Note A function in sum of products form can be implemented using NAND gates by replacing all AND and OR gates by NAND gates A function in product of sums form can be implemented using NOR gates by replacing all AND and OR gates by NOR gates

Logic gate symbols

Table 2 is a summary truth table of the inputoutput combinations for the NOT gate together with all possible inputoutput combinations for the other gate functions Also note that a truth table with n inputs has 2n rows You can compare the outputs of different gates

Logic gates representation using the Truth table

Example

A NAND gate can be used as a NOT gate using either of the following wiring configurations

Subject Eng Literature (The Tempest ndash William Shakespeare) Topic Act III Scene 3 Lines 53 to 110 (End of the scene) Date 16th April 2020 (2nd Period)

[Students should read the original play and also the paraphrase given in the school prescribed textbook]Summary Questions amp Answers

o Seeing this strange scene all are inclined to believe the tales told by travelers that there truly are ldquounicornsrdquo and ldquothe phoenixrsquo thronerdquo

o As they are about to sit down to the feast the banquet is snatched away by a harpy (Ariel disguised) A spiritrsquos voice (Arielrsquos voice) denounces Alonso Sebastian and Antonio with particular

1 ARIEL You are three men of sin whom Destiny

(Line 53-58)That hath to instrument this

lower world And what is int the never-surfeited sea

Hath caused to belch up you and on this island

Where man doth not inhabit you rsquomongst men

Being most unfit to live I have made you mad

reference to their crime in expelling Prospero from Milan They have not received any punishment for their deed earlier but the time for their punishment has arrived Upon Alonso it pronounces ldquolingering perdition worse than deathrdquo from which there is no remedy except through sincere repentance Ariel then vanishes in thunder and the shapes enter again and carry away the table

o Prospero watching invisibly is very pleased with the performance of Ariel and his (Prosperorsquos) ldquomeaner ministersrdquo All his enemies are now in his power and are in a fit of desperation He then leaves them and goes to see how Ferdinand and Miranda are getting on

o Alonso is now much humbled and penitent with the after effect of the spiritrsquos denunciation of his crimes He believes that his son is lost forever After this all disperse being stricken mad by the speech of the spirit

o Gonzalo fearing that they may do violence to themselves or to one another follows them and bid others to follow

(a) To whom does Ariel disguised as a harpy call the three sinners What game did Fate of Destiny play with

them

The three sinners called by Ariel are Alonso Sebastian and Antonio It was Destiny which had caused the ocean to cast the three sinners on the shore Though the ocean is all the time devouring whatever appears on its surface and is never satisfied with its continual swallowing of the ships and men in the present case the ocean had cast these three sinners on the shore without killing them

(b) Who had jointly been responsible for the conspiracy against Prospero What is Prosperorsquos purpose behind all this

Three men Alonso Sebastian and Antonio had jointly

been responsible for the conspiracy against Prospero They had driven out Prospero form Milan Prosperorsquos purpose is to make these three sinners realize the wrong they had done He wants them to repent for their criminal deeds because repentance leads to self-esteem(c )What does Ariel (the harpy) tell Alonso and his companions when they take out their swords to attack him

Seeing them drawing their swords Ariel (harpy) tells them that he and his companions are the instruments of destiny and that it is not possible for human beings to do them any injury He says that the swords of human beings can not injure even a minute part of his feathers Their swords are as ineffective against him and his companions as against the wind or the water

(d) Give the explanatory meanings of the following expressions in the context of the above extract

(i)Never surfeited (ii) Belch up (iii) lsquomongst men

(i) Never surfeited never led to satisfaction

(ii) Belch up cast ashore(iii) lsquomongst men in human

society2

I and my fellows (Line 60-65)

Are ministers of Fate The elementsOf whom your swords are tempered may as wellWound the loud winds or with bemocked-at stabsKill the still-closing waters as diminishOne dowl thats in my plume

IMPORTANT PASSAGES EXPLAINED

The elements

(Line 61-66)Of whom your swords are tempered may

as wellWound the loud winds or with

bemocked-at stabs

(a) Who is lsquoIrsquo Who are his lsquofellowsrdquo

lsquoIrsquo is referred to Ariel in disguise of a harpy His lsquofellowsrsquo are other spirits serving Prospero the real Duke of Milan who has acquired supernatural powers after being banished from his Dukedom Prospero has settled in this uninhabited island

(b) What are the elements that have temperrsquod the swords Why will it not work against the speaker

The swords (of Alonso and his companions) are tempered by metal (steel) which is taken out of the earth and refined by

Kill the still-closing waters as diminishOne dowl thats in my plume My fellow

ministersAre like invulnerable

In these words Ariel reminds the King and his companions of the utter futility of drawing swords against himself and his fellows Ariel drives Alonso Antonio and Sebastian the three men of sin to desperation ndash a state in which men do violence to themselves They draw swords to strike Ariel But Ariel reminds them that he and the other spirits are the ministers of destiny and nothing can wound them The steel of which their swords are made of may cut the wind or water which being divided always closes up again Even supposing that such things may be possible it is quite impossible that their swords will cut one feather in their plume They are incapable of being wounded by any sword of man Hence it is foolish on their part to attempt to strike at Ariel and his fellow-spirits

For which foul deed

(Line 72-75)The powers delaying not forgetting

haveIncensed the seas and shores yea all the

creatures Against your peace

Ariel enters like a harpy and remaining invisible tells Alonso Sebastian and Antonio that he and other harpies are the agents of Destiny appointed to carry out her decrees He tells them that their punishment for the crime against Prospero which has been so long deferred is now to fall upon them He reminds them that they had expelled Prospero from Milan and set him and his innocent child adrift on the sea and that the sea had paid them back for their sin by the shipwreck and by the calamities they have suffered He tells them that the powers above which did not forget this mean treachery but only deferred the punishment have now engaged the seas and the shores and all living beings including him and his comrades against them The very elements and supernatural agency Ariel adds have taken up the avenging of their crime against Prospero

the action of fire It may cut the wind or water which being divided always closes up again

The sword will not work against the spirits and the harpy because they are the ministers of destiny and nothing can wound them nor it will cut a single feather in their plume

(c )What is the meaning of lsquodowlrsquo in the last line

The term lsquodowlrsquo means a filament or the smallest part of a feather In this context Ariel in disguise of harpy says that their sword cannot even damage the smallest filament of their (Arielrsquos and other spirits) feathers as they are incapable of being wounded by any sword of man

(d) What does the speaker remind the listeners about

Ariel in disguise of harpy reminds Alonso the King of Naples Sebastian Alonsorsquos brother and Antonio the present Duke of Milan and the treacherous brother of Prospero as they being three men of sin He even reminds them that their punishment for their crime against Prospero which has been so long deferred now falls upon them He reminds them that they have expelled Prospero from Milan and has set him along with his innocent infant daughter adrift on the sea So the sea has paid them back for their sin by their shipwreck and the calamities they have suffered since then The harpy rebukes Alonso of his sin that has incensed the Gods and has deprived him of his son as a punishment

(e) How do they respond

When Ariel in disguise of a harpy reminds Alonso Sebastian and Antonio of their past misdeeds and sin Alonso has a look of terror and confusion in his eyes He utters the words of sincere repentance wrung out of his conscience-stricken heart It appears to him that all the elements of nature the sea-waves the wind and the thunder proclaiming a loud voice in the name of Prospero and the crime Alonso has committed against him They are calling upon him to repent There is a deep storm raging in Alonsorsquos breast and the echoes of that storm are ringing in his ears like a clear note of wind-instrument A note of denunciation of Alonsorsquos crime leaves him much humbled and penitent and confirms his belief that his son is lost forever But Sebastian and Antonio shows some courage instead of repentance They wish to kill the spirits or devils if it appears

3

Of my instruction hast thou nothing bated (Line 85-93)

In what thou hast to say So with good life

And observation strange my meaner ministers

Their several kinds have done My high charms work

And these mine enemies are all knit upIn their distractions They now are in my

powerAnd in these fits I leave them while I visitYoung Ferdinand whom they suppose is

drownedAnd his and mine loved darling

Methought the billows spoke and (Line 96-99)

told me of itThe winds did sing it to me and the

thunderThat deep and dreadful organ-pipe

pronouncedThe name of Prosper It did bass my

trespass

These are the words of contrition coming from Alonso Ariel has driven him to a deep repentance for conspiring with Antonio against Prospero He now feels a sincere remorse It appears to him that all the elements of nature the sea-waves the wind and the thunder proclaimed with a loud voice the name of Prospero and the crime Alonso had committed against him They are calling upon him to repent There is a deep storm raging in Alonsorsquos breast and the echoes of that storm are ringing in his ears like the clear note of a wind-instrument

Comment These are the words of sincere repentance wrung out of the conscience-stricken heart of Alonso Alonso who is the lesser villain is the first to give way to remorse under the effect of Arielrsquos speech The words of Ariel seem to him to be the voice of conscience speaking to him He is driven to desperation a state in which he might do violence to his life

(a) Identify the speaker State the context

Prospero the ruler of the island is the speaker The famous banquet scene has been enacted very well Ariel and his junior spirits have played their roles excellently Prospero is glad to say words of praise for them(b) In what way the speakerrsquos instructions have been carried out

According to Prosperorsquos instructions a banquet was presented before the King of Naples and his companions when they were tired and hungry Just when they were preparing to eat the feast the banquet was suddenly removed by exercising supernatural powers All this was done by Ariel Prosperorsquos chief assistant and a powerful spirit

Ariel not only made the feast disappear but also delivered his speech blaming the King and his two companions for their past wicked deeds He warned them to repent for their misdeeds or suffer forever on that uninhabited island

(c) Who are referred to as lsquomeaner ministersrsquo What have they done

Prospero refers as lsquomeaner ministersrsquo to his other lesser spirits who were assisting Ariel in presenting a scene before the kingrsquos party They entered the scene to the accompaniment of music They assumed several strange shapes and brought in a banquet Then they danced about it with gentle actions of salutations thus inviting the King and others to eat the feast

These spirits play their role again when Ariel in the shape of a harpy quits the scene These shapes enter again and dancing with mocking gestures carry away the table

(d) Who are the speakerrsquos enemies What has happened to them

King of Naples Alonso his brother Sebastian and the present Duke of Milan Antonio (Prosperorsquos own brother) are Prosperorsquos enemies With the turn of events they have all been washed ashore on the island which is ruled by Prospero the great magician Actually this happened after the shipwreck caused by a storm which was raised by Prospero with the purpose of bringing these people to his island Prosperorsquos spirits have already confused and terrified these enemies and they are under Prosperorsquos control He can treat them as he likes

(e) What does he say about Ferdinand Explain what is meant by ldquohellip his and mine darlingrdquo

Prospero knows that Alonsorsquos son prince Ferdinand is alive though his father thinks that the prince has been drowned

Prospero refers to his daughter Miranda who is dear to him She is also very dear to Prince Ferdinand who has fallen in love with her They are waiting to be married soon for which they have received Prosperorsquos consent

4

ALONSO O it is monstrous monstrous (Line 95-102)

Methought the billows spoke and told me of it

The winds did sing it to me and the thunderThat deep and dreadful organ-

pipe pronouncedThe name of Prosper It did bass

my trespassTherefore my son ithrsquo ooze is

bedded andIll seek him deeper than eer

plummet soundedAnd with him there lie mudded

(a) In what way does Alonso express his horror when his conscience is awakened by Arielrsquos words

When Alonsorsquos conscience is awakened by Arielrsquos words he expresses his horror at what he has heard He gets the feeling that the waves of the ocean the wind and the loud thunder have spoken to him and uttered the name of Prospero Because of being reminded of his crime in a very loud and rough voice he comes to realize that he has lost his son for his past misdeeds

(b) What does Alonso imagine about his son What does Alonso want to do in his desperate state

Alonso imagines that his son is lying in the mud at the bottom of the sea He feels desperate that he wants to drown himself in the ocean deeper than the plumb-line has ever gone He wants to lie with his son at the bottom of the sea

(c) How do Sebastian and Antonio want to face the evil spirits

Sebastian says that he is not at all afraid of what the harpy has said and that he is prepared to fight any number of such monsters if they appear before him only one at a time Antonio says that he would support Sebastian in the fight against the fiendsyyy

(d) Why does Gonzalo ask Adrian to follow the three men

Gonzalo tells Adrian that all the three men namely Alonso Sebastian and Antonio are in a wild and reckless mood The thought of the heinous crime of which they are guilty has begun to torment their minds So he asks Adrian to follow those three men without loss of time and prevent them from doing anything which the turmoil in their minds might lead them to do

(e) What opinion do you form of Alonso from the above extract

Alonso who is the lesser villain is the first to give way to remorse under the effect of Arielrsquos speech The words of Ariel seem to him to be the voice of conscience speaking to him He is driven to desperation a state in which he might do violence to his life

Subject =Accounts

Ac-12 15420 topic-pL Appropriation ac

PROFIT AND LOSS APPROPRIATION ACCOUNT

MEANING AND PREPARATIONProfit and Loss Appropriation Account is merely an extension of the Profit and Loss Account of the firm The profit of the firm has to be distributed amongst the partners in their respective profit sharing ratio But before its distribution it needs to be adjusted All Adjustments like partnerrsquos salary partnerrsquos commission interest on capital interest on drawings etc are made in this account These adjustments will reduce the amount of profit for distribution This adjusted profit will be distributed amongst the partners in their profit sharing ratio To prepare it at first the balance of Profit and Loss Account is transferred to this account The journal entries for the preparation of Profit and Loss Appropriation Account are given below

1 for transfer of the balance of Profit and Loss Account to Profit and Loss Appropriation Account

(a) In case of Net Profit

Profit and Loss Ac helliphelliphelliphelliphellipDrTo Profit and Loss Appropriation Ac(Net Profit transferred to Profit and Loss Appropriation Ac)

(b)In case of Net Loss

Profit and Loss Appropriation Achelliphelliphellip DrTo Profit and Loss Ac(Net Loss transferred to Profit and Loss Appropriation Ac)

2 for Interest on Capital

For transferring on Interest on CapitalProfit and Loss Appropriation Achelliphelliphellip DrTo Interest on Capital Ac(Interest on capital transferred to Profit amp Loss Appropriation Ac)

3 for Interest on Drawings

For transferring Interest on Drawings Interest on Drawings Achelliphelliphelliphelliphelliphellip DrTo Profit and Loss Appropriation Ac(Interest on drawing transferred to Profit amp Loss Appropriation Ac)

4 For Partnerrsquos SalaryFor transfer of partnerrsquos SalaryProfit and Loss Appropriation Achelliphellip DrTo Salary Ac(Salary transferred to profit amp Loss Appropriation Ac)

5 For Partnerrsquos CommissionFor transferring commissionProfit and Loss Appropriation Achelliphelliphellip DrTo Commission Ac(Commission transferred to Profit and Loss Appropriation Ac)

6 For Transfer of agreed amount to General ReserveProfit and Loss Appropriation Ac helliphellipDrTo General Reserve Ac(Transfer to General Reserve)

7 for share of Profit or Loss appropriation(a) If ProfitProfit and Loss Appropriation Achelliphellip DrTo Partnerrsquos CapitalCurrent Ac(Profit transferred to capitalcurrent Ac)(b) If LossPartnerrsquos Capital Current Achelliphelliphelliphellip DrTo Profit and Loss Appropriation Ac(Loss transferred to capitalcurrent Ac)

THE FORMAT OF PROFIT AND LOSS APPROPRIATION

Profit and Loss Appropriation Account for the year endedhelliphelliphelliphellip

Particulars Amount Particulars Amount

To PL Ac (loss) By pL Ac (profit)

To Interest on capital BY Interest on drawings

To partner`s commission by Partner`s capital Ac ( loss)

To Partner`s salary To Interest on partner`s loan To General Reserve To Partner`s Capital AC (Profit)

Subject= Economics

MOVEMENT ALONG THE DEMAND CURVE (CHANGE IN QUANTITY DEMANDED)In law of demand you have already studied the inverse relationship between price and quantity demanded When quantity demanded of a commodity changes due to change in its price keeping other factors constant it is called change in quantity demanded It is graphically expressed as a movement along the same demand curve There can be either a downward movement or an upward movement along the same demand curve Upward movement along the same demand curve is called contraction of demand or decrease in quantity demanded and downward movement along the same demand curve is known as expansion of demand or increase in quantity demanded

Extention of demandd

price (rs)p A

B Extentionp1 d

Q Q1

Quantity demanded ( in units)

Contraction of demandd

p2 Ccontraction

p APrice (Rs)

d

Q2 Q

Quantity demanded (in units)

Explanation of movement of demand A fall in price from OP to OP1 leads to increase in quantity demanded from OQ to OQ1 (expansion of demand) resulting in a downward movement from point A to point B along the same demand curve DD When Price rises from OP to OP2 quantity demanded falls from OQ to OQ2 (contraction of demand) leading to an upward movement from point A to point C along the same demand curve DD

  • Activity Series of Metals
    • Drawbacks of Rutherfordrsquos model of atom
      • Electromagnetic radiations
      • Properties of electromagnetic radiations
      • Characteristics of electromagnetic radiations
        • Plancks Quantum Theory-
        • Photoelectric effect
          • Intext Questions
            • Logic gates
            • Digital systems are said to be constructed by using logic gates These gates are the AND OR NOT NAND NOR EXOR and EXNOR gates The basic operations are described below with the aid of truth tables
            • AND gate
            • Example
Page 26:  · Web viewSubject. Topic. Summary. Execution. English 1 . Chapter 1 naming words . Page 8. Write the names of these pictures:- Person:-1. father. 2.Firefighter 3.doctor 4 ...

stakeholder and shareholder3) Give two difference between

internal stakeholders and external stakeholders

4) Give two expectations of suppliers from a business organisation

5) Who is a stakeholder in commercial organisations

Chemistry Periodic Table

Merits of Mendeleevrsquos Periodic law are as follows - 1He grouped the elements on the basis of atomic mass 2 He left gaps for undiscovered elements like Gallium Scandium germanium Also he left a full group vacant for undiscovered inert gases 3 He could predict proportions of several elements on basis of their position in periodic table like Ga Sc etc 4He could predict errors in atomic weights of some elements like gold platinum etc

Anomalies in Mendeleevrsquos Periodic law are as follows - 1 Position of isotopes could not be explained 2 Wrong order of atomic masses could not be explained

For example- as Arnur atomic mass 40 come first and K with low atomic mass (30) should come later but k should be placed first

According to Bohrrsquos Modern Periodic table properties of elements are periodic functions of their atomic numbers

So when elements are arranged according to increasing atomic numbers there is periodicity in electronic configuration that leads to periodicity in their chemical properties

It consists of horizontal rows (Periods) Vertical column (Groups)

There are 7 period and 12 groups in this long form of periodic table

Ist period has 2 elements IInd period has 8 elements IIIrd period has 8 elements IVth period has 18 elements Vth period has 18 elements VIth period has 32 elements VIIth period hs rest of elements

Note - The number of valence electrons in atom of elements decides which elements will be first in period and which will be last

In group- 1 to 2 gp and 13 to 17 contain normal elements 3 to 12gp ndash transition elements 57 to 71 - lanthanides 89 to 103 - Actinides

Left hand side ndash metals Right hand side ndash nonmetals

Note- Hydrogen element has been placed at top of Ist group Electronic configuration of H is similar to alkali metal as both have 1 valence electron

V electron of gp I element -- 1 V electron of gp 2 element -- 2 V electron of gp 13 element -- 3 V electron of gp 14 element -- 4 V electron of gp 15 element -- 5 V electron of gp 16 element --6 V electron of gp 17 element -- 7 V electron of gp 18 element -- 8

English 1 Transformation of sentences

Sentences A sentence is a group of words which makes complete sense

Exercise 2Change the following sentences from

a Assertive sentencesb Imperative sentencesc Interrogative sentencesd Exclamatory sentences

Sentences can be changed from one grammatical form to another without changing the meaning of the sentence This is known as transformation of sentences

assertive to interrogative1 Nobody would like to be a fool

Who would like to be a fool2 Their glory can never fade

When can the glory fade3 Nobody can control the wind

Who can control the wind4 It matters little if I die

What though I die5 No man can serve two masters

Can any man serve two masters

Exercise 3Interchange of assertive and Exclamatory sentences

1 She leads the most unhappy lifeWhat an unhappy life she leads

2 This is indeed an interesting bookWhat an interesting book this

3 He is a very great manWhat a great man he is

4 It is a very lame excuseWhat a lame excuse

5 It is sad that she died so youngAlas she died so young

Class XISubject Topic Summary Execution

Hindi 2nd lang

पतर परम(परमचदर) पतर परम कहानी म एक निपता की इचछाओ का वणन निकया गया ह अपन बड पतर परभ -ास स निपता चतनय -ास का निवशरष परम था निपता को उसक जनम स ही बडी-बडी आशाए थी उसम दसर बट कतिशव-ास की अपकषा स- उतसाह की मातरा अमिधक थी वह उस इगलड भजकर बरिरसटर बनाना चाहत थभागय का खल भी बडा निनराला ह बीए की परीकषा क बा- वह बीमार पड गया डॉकटरो न भी जवाब - दि-या थाचतन -ास जी बहत ही कजस थ बवजह पस खच करना नही चाहत थ अगर गारटी मिमलती तो शाय- पस खच भी कर -त परत गारटी नही थी परिरणाम सवरप उनक बट का -हात हो गयाजब बट को समशान ल जा रह थ तो वहा काफी शोर गान बजान हो रह थ पछन पर पता चला निक निकसी निपता निपछल तीन साल स निबमार था और उसक ईलाज म रपया पानी की तरह बहाया पर ठीक नही हए परत उसक बट को तनिनक भी अफसोस नही था उसका कहना था उसन कोकतिशश तो कीयह -खकर चतनय-ास जी को आतम निगलानी हईतभी स उनका म परिरवतन हआ और बट का भोज काफी धमधाम स निकयाऔर वहइस पशचाताप की आग म जलत रह औला- स बढकर पसा नही होता ह इस बात को समझन म उनह काफी व लग गया

hellipContinue to next

BENGALI(2ND LANGUAGE)

পরথমঅধযায়-ঠাকরারীনদরনাথঠাকর

নয়ন দোচের হিমাচেররা া নাচেমই হিযাত হিছচেন ায়ানার উাররণ সবরপ নয়ন দোচের ারা হিা (াচেকর হিা হিচেতন এছাাও দেকান উৎস উপচেb রাহিতর দেক হিন করার উচেfচে(য তারা সযC হিকরচেরণ রনয পরীপ জবাহিচেয় তাচেত রপার হির 4Cরণ করচেতন ঠাকরা এই নয়ন দো হিমারচের দে(4 ং(ধর হিছচেন হিমাররা ায়ানার ষটানত পর(Cন কচের তারা হিনঃসব এই হিমাহিরর দে(4 ং(ধর গৈকাস নদর রায়চেৌধরী গৈকাস া নয়ন দোচের সমসত সমপহিতত ঋচেরণর াচেয় হিহিx কচের অহি(ষট যা আচেছ তাচেত হিপত

ইার হিপতার মতয ইচে পর নয়নচোচের ায়ানার দেগাটা কতক অসাধাররণ শরাদধ (াহিনতচেত অহিনতম ীহিপত পরকা( কহিরয়া ঠাৎ হিনহিয়া দেগ- ক) কার দো দেকান গচেলপর অং() কতা দেক ইার চেত কাচেক দোঝাচেনা চেয়চেছ গ) পরসঙগ কী কতার কতয পহিরসফট কচেরা

পরচে4র যাহিত রbা করা সমভ নয় তাই হিতহিন পতরচেক হিনচেয় ককাতায় সাস শর কচেরন গলপ কথচেকর আহিথCক অসথা নয়ন দোচের হিমাচের দেথচেক সমপরণC আাা কথচেকর হিপতা হিনচের দেষটায় অথC উপাCন করচেতন া উপাহিধ াচেভর নয তার াসা হিছনা আর দেসই কারচেরণ কথক তার একমাতর উততরাহিধকার চেয় তার হিপতার পরহিত কতজঞ কথক দো পা হি(চেচেছন হিনচের পরারণ ও মান রbার নয উপচেযাগী অথC হিনা দেষটায় পরাপত চেয়চেছন- এটাই তার কাচেছ পরম দেগৌরচের হি4য় চে মচেন কচেরন কাররণ (নয ভাণডাচের গৈপতক ায়ানার উজজব ইহিতাস অচেপbা দোার হিসeচেকর মচেধয গৈপতক দেকামপাহিনর কাগ তার কাচেছ অচেনক দেহি( মযান

TO BE CONTINUED

উ- ক) আচোয অং(টি রীনদরনাথ ঠাকচেরর দো ঠাকরা গচেলপর অং() কতা চেন আচোয গচেলপর গলপ কথকইার চেত নয়ন দোচের হিমাহিরর দে(4 ং(ধর গৈকাস ার কথা া চেয়চেছ গৈকাস া নয়ন দোচের সমসত সমপহিতত ঋচেরণর াচেয় হিহিx কচের অহি(ষট যা আচেছ তাচেত হিপত পরচে4র যাহিত রbা করা সমভ নয় তাই হিতহিন পতরচেক হিনচেয় ককাতায় সাস শর কচেরনগ) গৈকাস ার হিপতার মতযর পর নয়ন দোচের হিমাহিরর অহিসততব হিপত য় কচেয়কটা উৎস ও শরাদধ- (াহিনতচেত হিমাহিরর দে(4 কহিটক যয় চেয় হিগচেয় এচেক াচের দে(4 চেয় যায় তন তাচের গC করার মত আর হিকছই হিছ না-দেসই পরসচেঙগ এই উহিকত নয়নচোচের হিমাচেররা া নাচেমই হিযাত হিছচেন ায়ানার উাররণ সবরপ নয়নচোচের ারা হিা (াচেকর হিা হিচেতন এছাাও দেকান উৎস উপচেb রাহিতরচেক হিন করচেত হিগচেয় তারা সযC হিকরচেরণর নয পরীপ জবাহিচেয় তাচেত রপার হির 4Cরণ করচেতন তাই দেসকাচের ায়ানা দেহি(হিন সথায়ী চেত পারত না হিহিভনন উৎস শরাদধ- (াহিনতচেত সাধযা হিতহিরকত র করার নয হিমাহির হিহিকচেয় দেযত হ হিতC কা হিহি(ষট পরীচেপর দেত দেযমন অলপকাচের মচেধয হিনঃচে(4 চেয় যায়-নয়নচোচের হিমারচের অসথা তাই চেয়হিছ এই কারচেরণই কথক নয়নচোচের হিমারচের গা ভরা আমবর সয করচেত পারতনা

Physics Dimensional Analysis (Summary)

Q Find the dimensions of consts ab in relation

p=(bminusxlowastx)at

where p is the power x is the distance and t is time

Ans From principle of homogeneity dimension of b x2 are same Dim of b = dim of x2 = [L2] = [ML2T0]Dim of a = dim of ( b- x2)dim of (pt) = [M0L2T0][ML2T-2] [T-1] [T] = [M-1L0T2]

Chemistry Atomic Structure Drawbacks of Rutherfordrsquos model of

atom a According to Rutherfordrsquos model of atom electrons which are negativelycharged particles revolve around the nucleus in fixed orbits Thusb theelectrons undergo acceleration According to electromagnetic theory of Maxwell a charged particle undergoing acceleration should emitelectromagnetic radiation Thus an electron in an orbit should emitradiation Thus the orbit should shrink But this does not happenc The model does not give any information about how electrons aredistributed around nucleus and what are energies of these electrons Isotopes These are the atoms of the same

Properties of electromagnetic radiationsa Oscillating electric and magnetic field are produced by oscillating charged particles These fields are perpendicular to each other and both areperpendicular to the direction of propagation of the waveb They do not need a medium to travel That means they can even travel invacuum

Characteristics of electromagnetic radiationsa Wavelength It may be defined as the distance between two neighbouring crests or troughs of

element having the same atomicnumber but different mass numbere g 1H11H21H3

Isobars Isobars are the atoms of different elements having the same massnumber but different atomic numbere g 18Ar40 20Ca40

Isoelectronic species These are those species which have the same numberof electrons

Electromagnetic radiationsThe radiations which are associated withelectrical and magnetic fields are called electromagnetic radiations When anelectrically charged particle moves under acceleration alternating electricaland magnetic fields are produced and transmitted These fields aretransmitted in the form of waves These waves are called electromagneticwaves or electromagnetic radiations

wave as shown It is denoted by λb Frequency (ν) It may be defined as the number of waves which passthrough a particular point in one secondc Velocity (v) It is defined as the distance travelled by a wave in onesecond In vacuum all types of electromagnetic radiations travel with thesame velocity Its value is 3 times10 8m sec-1 It is denoted by v

d Wave number Wave number is defined as the number of wavelengths per unit lengthVelocity = frequency timeswavelength c = νλ

Plancks Quantum Theory- o The radiant energy is emitted or absorbed not continuously but discontinuously in the form of small discrete packets of energy called lsquoquantumrsquo In case of light the quantum of energy is called a lsquophotonrsquoo The energy of each quantum is directly proportional to the frequency of the radiation ie E α υ or E= hυ where h= Planckrsquos constant = 6626 x 10-27 Js o Energy is always emitted or absorbed as integral multiple of this uantum E=nhυ Where n=1234Black body An ideal body which emits and absorbs all frequencies is calleda black body The radiation emitted by such a body is called black body radiation

Photoelectric effectThe phenomenon of ejection of electrons from thesurface of metal when light of suitable frequency strikes it is calledphotoelectric effect The ejected electrons are called photoelectrons

Biology Chapter - 02Systematics and Five Kingdoms

Scientists divide the whole living organisms into two kingdom first and ultimately by five kingdom at last

In the earlier systems of classifications organisms are divided into kingdom plantaeand kingdom animalia on the of presenceof cell wall their modes of nutrition and movements

Some problem arise like fungi share manycharacteristic withplant despite their heterotrophic nutrition bacteria protozoa areunicellular present in both kingdom Toovercome this third kingdom Protista isintroduced which include

unicellularorganisms But there is also another

problem Allunicellular organisms are not similar kind The cellular structure of prokaryotes is verydifferent from that of other organismsEukaryotes possess a true nucleus and allcell organelles that are not present inprokaryotes So the fourth kingdom Monerais introduced which include unicellular prokaryotes (bacteriaamp blue green algae)

bull Still some problem arise in kingdomplantae

So in 1969 R H Whittakar proposedanew five kingdom System of classification

i) Kingdom Monera - unicellular prokaryotes

ii) kingdom Protista - unicellular eukaryotes

iii) Kingdom Fungi - uni or multicellular fungi with cell wall but without chlorophyll

iv) Kingdom Plantae - Multicellular Plants

v) Kingdom Animalia - Multicellular Animals

EVS Chapter 1 ndash Modes of Existence

An agricultural society

An agricultural society also known as an agrarian society is a society that constructs social order around a reliance upon farming More than half the people living in that society make their living by farming

People in an agricultural society generally lead a more settled lifestyle than those in nomadic hunter-gatherer or semi-nomadic pastoral societies because they live permanently near the land that is farmed Agricultural settlements tend to develop in areas of convenience near bodies of water which is used for both crops and transportation or along trade routes Not everyone in an agricultural society is a farmer Some people make a living trading or making and selling goods such as tools used for farming

Another way to define an agrarian society is to see the total amount of production in a nation In an agrarian society cultivating the land is the main source of wealth Such a society can recognize other means of subsistence and work habits but emphasizes the importance of agriculture and livestock Agrarian societies have existed in various parts of the world for 10000 years and continue to exist today They have been the most common form of socio-economic organization for most of recorded human history

Q) Write the features of agricultural society

Ans - Structure and Features of Agrarian Society1 Occupational Structure

An agrarian society is generally associated with the domestication of plants and animals The domestication of plants means farming and that of animals means herding Often there is mixture of farming and the use of such domesticated animals as cow goat and sheep

2 Forms of Land Ownership in Agrarian SocietiesGenerally there are landlords supervisory farmers cultivators and share croppers The landholders own the land but do not work on it They let it out for sharecropping The supervisory farmers are those who live by having their land cultivated by hired labourers The cultivators cultivate the land for themselvesThe share-croppers are those who live by tilling other peoplersquos land or a crop-sharing basis The artisans own their means of production and produce by their own labour in their homesteads

3 Village Community System An agrarian society is highlighted by

the institution of village community system The agrarian economy made fixed dwelling houses necessary Living close together for protection and co-operation and living nearer to the land gave birth to agricultural villages The village is not only the residential place of farmers it is also the social integrator

4 Minimal Division of Labour Another structural feature of agrarian society is a minimal division of labour Except for the basic division founded on age and sex differences there are few specialized roles There is only one predominant type of occupation ie domestication of plants and animals For all the people the environment physical as well as social is the same

5 Role of Family The farm family is of the patriarchal type the father is the final arbiter in most of the familyrsquos major decisions The life of ail men and women is merged in family life Since there are not many special organizations family is the only organisation to perform the tasks of aid and protection

6 Sense of Unity The members of an agrarian society exhibit a strong in-group feeling Since the whole of their social lives is wrapped up in a society which is physically economically and socially homogenous they are inclined to view the entire outside world as an out group

7 Informal Social Control An agrarian society is regionally divided into villages In a village community the force of traditional mores is more dominant than in the urban community In the village everybody is known to everybody The members in a village community help each other and share the joy and sorrows of each other Crime in an agrarian society is rare

8 Simplicity and Uniformity Life of the people in an agrarian society is marked by simplicity and uniformity Their main occupation is agriculture which largely depends upon the vagaries of nature An agrarian society is a religious society

Math Compound angles Compound angles The algebraic sum of two or more angles is called a compound angle If A B C be three angles then A+B B+C C+A A-B B-C A-C A+B-C etc are compound angles In this chapter we shall discuss the trigonometrical ratios of compound angles Theorem 1 If A B and A+B are all pisitive acute angles theni) sin( A+B) = sin A cos B + cosA sinBii) cos(A+B) = cosA cosB- sinA sinBTheorem 2If A and B are positive acute angles and AgtB theni) sin(A-B) = sin A cosB- cos A sinBii) cos(A-B) = cos A cos B+ sin A sin BTo prove that i) sin(A+B) sin (A-B) = sin2 A - sin2 B = cos2 B- cos2 A

Example 1 Prove that tan70deg=2tan50deg+tan20degSolutiontan70deg = tan(50deg + 20deg)Or tan70deg=(tan 50deg+tan 20deg)(1-tan50degtan20deg) or tan70deg (1 ndash tan 50deg tan20deg) = tan50deg+tan20degor tan70deg= tan70deg tan50deg tan20deg+ tan50deg + tan20deg = cot20deg tan50deg tan20deg + tan50deg + tan20deg = 2 tan50deg+ tan20degExample 2 If A + B = 45deg show that (1 + tanA) (1 + tanB) = 2Solutiontan(A + B) =( tan A + tan B) (1 - tan

ii) cos(A+B) Cos(A-B) = cos2 A- sin2 B = cos2 B -sin2 AProof i) LHS= sin(A+B)sin(AminusB) [Recall sin(αminusβ)=sinαcosβminuscosαsinβ And sin(α+β)=sinαcosβ+cosαsinβ]= (sinAcosB+cosAsinB)times(sinAcosBminuscosAsinB)= sin2Acos2Bminuscos2Asin2B [Recall sin2α+cos2α=1 From above we can then assume correctly that sin2α=1minuscos2α AND cos2α=1minussin2α] = sin2A(1minussin2B)minussin2B(1minussin2A) = sin2Aminussin2Asin2Bminussin2B+sin2Asin2B = sin2Aminussin2B= 1-cos2A-(1-cos2B) = cos2 B- cos2 A = RHSii)LHS= cos (A+B) cos(A-B) [ cos(A+B) = cos AcosB- sinAsinBCos(A-B) = cosAcosB+ sinAsinB]= cos2 A Cos2 B- sin2 A Sin2 B= cos2 A( 1-sin2 B) - (1- cos2 A) sin2 B= cos2 A- cos2 A sin2 B- sin2 B+ cos2 A sin2 B=cos2 A- sin2 B=1- sin2 A-(1-cos2 B) = cos2 B- sin2 A= RHSTangent formulae for compound anglesi)tan (A + B) = tan A + tan B1-tan A tan Bii)tan (A ndash B) = tan A-tan B1+tan A tan Biii) cot (A + B) = cot Acot B-1cot A+cot B(viii) cot (A ndash B) = cot Acot B+1cot B-cot A

A tan B) Or 1= (tan A+ tanB) (1-tan A tanB) Or tanA + tanB + tanA tanB + 1 = 1 + 1Or tanA (1 + tanB) + (1 + tanB) = 2Or (1 + tanA) (1 + tanB) = 2Example 3 Find the value of sin 15degSolution sin 15deg= sin(45deg-30deg) = sin45degcos 30deg- cos45degsin30deg =(1radic2) (radic32) -(1radic2) (12) = (radic3-1) 2radic2Example 4 If sin A = 1 radic10 and sin B = 1 radic5 where A and B are positive acute angles then what is A + B SolutionWe know that sin (A + B) = sin A cos B + cos A sin B= [1 radic10] [radic(1 minus 1 5)] + [1 radic5] radic(1 minus 1 10)= [1 radic10] [radic4 5] + [1 radic5] [radic9 10]= [1 radic50] times (2 + 3)= 5 radic50 = 1 radic2

sin (A + B) = sin π 4rArrHence A + B = π 4Example 5 If A + B = 225o then find [cot A] [1 + cotA] times [cot B] [1 + cot B]Solution[cot A] [1 + cotA] times [cot B] [1 + cot B] = 1 [(1 + tan A) times (1 + tan B)]=1 [tan A + tan B + 1 + tan A tan B] [ tan (A + B) = tan225o]∵

tan A + tan B = 1minus tan A tan BrArr= 1 [1 minus tan A tan B + 1 + tan A tan B]= 1 2

COMMERCE

CLASSIFICTION OF HUMAN ACTIVITIES-ECONOMIC AND NON-ECONOMIC

Firstly we shall recall the previous class for 5 mins especially for the absentees and for also the rest of the students who were there

Today at first we briefly discuss the earlier portions of the chapter

1Business-It includes all those economic activities which are concerned with production and exchange of goods and services with the object of earning profit Example A factory shop beauty parlour also business enterprises

2Profession ndashThe term profession means an occupation which involves application of specialized knowledge and skills to earn a living For Example Chartered Accountancy medicine law tax consultancy are example of professions

Questions1What are the main features of ProfessionAnswer The main features of a profession are as follows a Specialised body of knowledge-Every profession has a specialised and systematised body of knowledge b Restricted entry- Entry to a profession is allowed only to those who have completed the prescribed education and have the specialised examination c Formal education and training ndashA formal education and training is given to the person who wants to acquire the professional

3Employment-Employment mean an economic activity where people work for others in exchange for some remuneration (salary)The persons who work for others are called lsquoemployeesrsquo The persons or organizations which engage others to work for them are called lsquoemployersrsquoEg A doctor working in a hospital is employment as he is working for a salaryA lawyer may serve as a law officer in a bank

With this we shall proceed with the features of both Profession amp Employment

The main features of a profession are as follow

a Specialised body of knowledge b Restricted entry c Formal education and training d Professional association e Service motive f Code of contact

The main features of an employment are as follows

a In employment a person works for others called employer

b An employee provides personal service

c There is a service agreement or contract between the employee and the employer

d The employee has to obey the order of the employer

e No capital investment is made by the employer

Various examples of Employment are as follows

aA teacher teaching in a school or collegeb An engineer employed in Municipal Corporation of DelhicAn accountant working in the accounts department of a companydA doctor working in a hospital

Note In all the above examples of employment the individual who is involved in each example is working as an employee for a salary under an employer

qualification(MBBSCALLB)d Service motive ndashProfessionals are expected to emphasis service more on their clients rather than economic gain f Code of Conduct-The activities of professionals are regulated by a code of conduct

2 What are the main features of EmploymentAnswer The main features of an employment are as followsa In employment a person works for others called employerb An employee provides personal servicec There is a service agreement or contract between the employee and the employerd The employee has to obey the order of the employere No capital investment is made by the employer

3 Give various Professions and their respective Association are given below

Professions

Professional

Professional association

Medical profession

Doctor Medical Council of India

Law profession

Lawyers Bar Council of India

Accounting Profession

Chartered

The Institute of Chartered Accounts of India( ICAI)

Engineerin Engineers The

g Profession

institute of Engineers (India)

Accounts Basic accounting terms

Today we will give you some questions from the previous study material

Questions6) Define accounting7) What do you mean by debit

and credit8) Explain the types of account9) Define the following terms

a) Assetsb) Capitalc) Purchased) Debtorse) Transactions

10) Name the types of accounts given below

a) Krishnas accountb) Machinery accountc) Royalty accountd) Salary accounte) Furniture accountf) Audit fee account

Economics Basic Economic ConceptsSub topic

UTILITY

Before starting todayrsquos class we shall recall the last class which was about UTILITY AND THE FEATURES OF UTILITY

Now we shall proceed with the further topics of the chapter

Todayrsquos topic from the chapter lsquo Basic Economic Conceptsrsquo will be TOTAL UTILITY amp MARGINAL UTILITYNow let us quickly revise the concept of utility with an example ie goods and services are designed because they have an ability to satisfy human wantsThis feature of being able to satisfy human wants is termed as utility For example we derive utility from WiFi services as it gives us satisfaction by connecting us to our friends and family through social media here consumers derive utility from WiFi services

From the above concept we shall start with todayrsquos topicEconomists have defined TOTAL UTILITY (TU) as the total satisfaction obtained by consuming a given total amount of a good and serviceFor example the total satisfaction obtained from eating 10 mangoes is the total utility of 10 mangoes

MARGINAL UTILITY (MU) is the additional satisfaction derived from each additional unit

Questions1 What is Total Utility (TU)

Answer Total Utility (TU) is the

aggregate of the utility that a consumer derives from the consumption of a certain amount of a commodityTU=MU1+MU2++MUn

2 What is Marginal UtilityAnswer

Marginal Utility (MU) is the additional made to the total utility as consumption is increased by one more unit of the commodityMU= TUn ndashTUn-1

NoteOften economists tend to

subdivide utility into an imaginary unit called UTIL

consumed In this casethe utility obtained from each mango as it is consumed as the MU of that mango It is also defined as the addition made to the total utility when an additional unit is consumed Often economists tend to subdivide utility into an imaginary unit called UTIL

Note As a consumer increases the consumption of a good over period of time the total utility or total satisfaction derived from it increases to appoint and thereafter it decreasesHowever as the consumer keeps on consuming the good the marginal utility or the additional utility derived from it decreases

SubjectBusiness studies

Topic

BUSINESSENVIRONMENT

Summary

Now quickly let us revise the earlier points that we have already done in the last class and let us proceed with the other topics that are there in the chapter

Firstly we will recall the internal and external factors of micro environment and then we shall proceed in details

Meaning and list of internal and external factors

aInternal factorsInternal factors refer to all the factors existing within a business firm The internal factors are considered controllable because the enterprise has control over these factorsFor an example a company can alter its organization structure policies programmes employees physical facilities and marketing mix to suit the changes in the environmentList of internal factors areCorporate culture mission and objectives top management organizations structure company image and brand equity company resources

b External factorsExternal factors refer to those individual and groups and agencies with which a particular business organization comes into direct and frequent contact in the course of its functioningThese individuals and groups are known as STAKEHOLDERS because they have a stake (financial interest ) in the working and performance of the particular business List of external forces (stakeholders)Customers competitors investors suppliersmiddlemen (marketing intermediaries)

Execution 1 What do you mean by internal

factors in micro environmentAnswerInternal factors refer to all the factors existing within a business firm The internal factors are considered controllable because the enterprise has control over these factorsFor an example a company can alter its organization structure policies programmes employees physical facilities and marketing mix to suit the changes in the environment

2 What do you mean by external factors in micro environment

AnswerExternal factors refer to those individual and groups and agencies with which a particular business organization comes into direct and frequent contact in the course of its functioningThese individuals and groups are known as STAKEHOLDERS because they have a stake (financial interest) in the working and performance of the particular business

3Who are stakeholdersSTAKEHOLDERS are individuals and groups who have a stake (financial interest ) in the working and performance of the particular business 4Discuss the internal factors in briefa Corporate CultureThe values beliefs and attitudes of the founders and top management of the company exercise

financers publics

customers

suppliersfinancers

competitors

middlemen

publics

Fig STAKEHOLDERS OF A COMPANY

Apart from micro environment the other main dimension of business environment isMacro environment Macro environment refers to the general environment or remote environment within which a business firm and forces in its micro environment operateA company does not directly or regularly interact with the micro environmentTherefore macro environment is also known as indirect action EnvironmentThe macro environment forces are less controllable than the micro forces

Macro environment consists of the following components

POLITICAL AND LEGAL ENVIRONMENT

ECONOMIC SOCIAL AND ENVIRONMENT

CULTURAL

ENVIRONMENT

TECHNOLOGICAL ENVIRONMENT

a strong influence on what the cmpaany stands for how it does things and what it considers importantbMission and objectivesThe business philosophy and purpose of a comoany guide it prioritiesbusiness strategiesproduct market scope and development scope

cTop management structurethe composition of board of directors the degree of professionalization of management and the organizational structure of a company have important bearing on its business decisions

dPower structureThe internal power relationship between the board of directors and the chief executive is an important factor

eCompany image and brand equityThe image and brand equity of the company play a significant role in raising finance forming alliance choosing dealers and suppliers launching new products entering foreign markets

5 What is Macro environmentAnswerMacro environment refers to the general environment or remote environment within which a business firm and forces in its micro environment operateA company does not directly or regularly interact with the micro environmentTherefore macro environment is also known as indirect action EnvironmentThe macro environment forces are less controllable than the micro forces 6 What are the components of macro environmenta Political and legal environmentb Economic environmentc Social and cultural environmentd Technological environment

BUSINESS FIRM

Fig COMPONENTS OF MACRO ENVIRONMENTPolitical science

Introduction to political science

Comparative politics and itrsquos scope Comparative politics is the second major dimension of political scienceIt is also a very vast area of study and a very large number of political scientists even treat it as an autonomous area of study within the board ambit of political scienceScope of comparative politics-

1 All political structures -Comparative politics includes the study of all structures formalnon formal governmental and extra governmental which are directly or indirectly involved in politics in all the countries of the world

2 Functional studies- Comparative politics seeks to study politics less from the point of view of the legal institutions in terms of their powers and move from the point of view of their functions which constitute the political process and their actual Operation in the environment

3 Study of political behaviour- Another important part of its scope is the study of the actual behaviour of the people in the process of politics

4 Study of similarities and differences- comparative politics also undertakesan analysis of the similarities and differences among political process and functions

5 Study of all political systems -comparative politics seeks to analyse the actual behaviour and performance of all political systems western as well as non western

6 Study of the environment and infrastructure of politics-The study of politics demands a study of the psychological sociological economic and anthropological environment in fact the social environment as a whole in which each political system operates

7 Study of political culture- political culture is composed of attitudesbeliefs emotions and values of a society that relate to the political system or politics

8 Study of political participation- Political participation is a universal processThe only difference is that while in some states it is limited in others it is wider

9 Study of political process- political

Answer the following questions-

What is comparative politics

What are the scope of comparative politics

Homework- learn

processes like decision makingpolicy making judicial process leadership recruitment process and others are always at work in all political systems

The scope of comparative politics is very comprehensive It includes everything that falls within the area of political activity and political process

History CAMBRIDGE VIEW ABOUT

THE PARTITION

AND REFUTATION

OF CAMBRIDGE

VIEW

Cambridge view about the Partition The Cambridge school of historians have interpreted that opposition to partition scheme was made entirely by the elitist groups They hold the view that Lord Curzon planned to partition the Bengal for administrative purposeREFUTATION OFCAMBRIDGE VIEW The Rationalist historians have rejected the interpretations of the Cambridge School of historians on various grounds

1 QUESTION State different views of historians regarding Partition of Bengal

ANSWER Cambridge historians believed that Lord Curzon partitioned Bengal for administrative reasons only and not for the political motive The Middle class elitist group protested because of their petty interest The Hindu zamindars protested as they have to spend more money for managing their estatesThe lawyers of Calcutta High court feared to lose their clientBut according to the nationalist Historians was-

2- The ultimate object of Lord Curzon was to crush the unity of Bengal politicians

3- If Bengal becomes a separate province Bengali speaking 16 million people of western part would become minority under Hindi speaking people of Bihar and Oriya speaking people of Orissa

4- The bureaucrats expected that the protest movement would die down quickly

5- Lord Curzon used the Muslim community in his political game

6- Idealism had great contribution in the protest against partition

7- The people of the every section of society were affected by the partition of Bengal

Computer Science

Numbers Convertion of dcimal number to octal numberThe decimal numeral system is the standard system for denoting integer and non-integer numbers It is the extension to non-integer numbers of the Hindu-Arabic numeral system For writing numbers the decimal system uses ten decimal digits a decimal mark and for negative numbers a minus sign - The decimal digits are 0 1 2 3 4 5 6 7 8 9 the decimal separator is the dot in many countries

The octal numeral system or oct for short is the base-8 number system and uses the digits 0 to 7 Octal is sometimes used in computing instead of hexadecimal perhaps most often in modern times in conjunction with file

permissions under Unix systems It has the advantage of not requiring any extra symbols as digits It is also used for digital displays

Follow these steps to convert a decimal number into octal form

1 Divide the decimal number by 82 Get the integer quotient for the next iteration (if the number will not divide equally by 8 then round down the

result to the nearest whole number)3 Keep a note of the remainder it should be between 0 and 74 Repeat the steps until the quotient is equal to 05 Write out all the remainders from bottom to top This is the solution

For example if the given decimal number is 8453

Division Quotient Remainder

8453 8 1056 5

1056 8 132 0

132 8 16 4

16 8 2 0

2 8 0 2

Then the octal solution is 20405

Subject Eng Literature (The Tempest ndash William Shakespeare) Topic Act I Scene 1 Lines 33 to 67 (End of scene) Date 16th April 2020 (4th Period)

[Students should read the original play and also the paraphrase given in the school prescribed textbook]Summary Questions amp Answers

[SUMMARY OF THE ENTIRE SCENE]

o The play starts with the scene of a severe storm at sea Alonso (King of Naples) Sebastian (Alonsorsquos brother) Ferdinand (Alonsorsquos son) Gonzalo Antonio (the usurping Duke of Milan) are in a ship in the midst of the storm

o The mariners are trying their best to control the vessel from running aground and are totally following the orders of their Master the Boatswain They have scant success

o The mariners become extremely unhappy and annoyed when most of the passengers arrive on the deck thereby hampering their effort to save the ship There is serious confrontation between them and the passengers who are part of the Kingrsquos entourage

o The mariners could not save the ship

SUMMING-UP

(i) Vivid description of the scene which gives a realistic description of terror and confusion of a tropical storm

(ii) Shows Shakespearersquos accuracy of knowledge in describing the naval operations and also matters of seamanship

(1) GONZALO Ill warrant him for drowning (L 45-57)

though the ship were no stronger than a nutshell and as leaky as an unstanched

wenchBOATSWAIN Lay her a-hold a-hold Set her two courses Off to

sea again lay her offMARINERS All lost To prayers to prayers All lostBOATSWAIN What must our mouths be coldGONZALO The king and prince at prayers Lets assist them

For our case is theirsSEBASTIAN Im out of patienceANTONIO We are merely cheated of our lives by drunkards

This wide-chopped rascal - would thou mightst lie drowning the washing of ten tides

(a) What does Antonio say at the insolent manners of the boatswain just before the given passage

Being irritated at the insolent manners of the boatswain just before the given extract Antonio the Duke of Milan calls him a worthless dog son of a woman without any morals an arrogant and disrespectful noisemaker He says that the boatswain deserved to be hanged(b) What statement does Gonzalo repeat about the boatswain

Gonzalo shows his faith that the boatswain is not destined to die by drowning He is destined to be hanged and nothing can alter this decree of destiny He says that even if the ship was as frail as a nutshell the boatswain could not be drowned for his destiny was to be hanged(c) What do the passengers do when they have lost all hope of their survival

When the passengers have lost all hope of survival they take

(iii) The opening scene justifies the title ndash The Tempest

UNANSWERED QUESTIONS

(i) The King always travels with his entire fleet including his soldiers Where were the other ships

(ii) Why was the ship in that area Where was it coming from or going where

(iii) The ship broke apart What happened to those who were in the ship

(We shall get the answer to the above questions as the play progresses)

leave of life with fervent prayers The mariners take their last hearty drink and are ready for death(d) What blame does Antonio put upon the mariners and the boatswain Antonio rebukes the mariners that these drunkards have brought them to the present crisis by neglecting their duties He blames them saying that they are going to lose their lives entirely for the negligence of the boatswain and his fellows(e) What does Antonio say while cursing the boatswain

Antonio gives vent to his wrath upon the boatswain in particular He calls the boatswain a wide-mouthed rascal who deserves to be hanged on the sea-shore at low water mark so that ten tides might wash over his body and take out of him all the liquor that he has been drinking

Class XIISubject Topic Summary ExecutionHistory Topic

1 1935 ACT AND WORKING OF PROVINCIAL AUTONOMYCONGREE AND OTHER MINISTERSSUB TOPIC GOVERNMENT OF INDIA ACT1935

Government of India Act 1935 This act established a lsquoFederation of Indiarsquo made of British Indian provinces and Indian states and provided for autonomy with a government responsible to the elected legislature in every provinceThis act introduced abolition of Diarchy at provinces The entire provincial administration was introduced to the responsible ministers who were controlled and removed by the provincial legislature The provincial autonomy means two things First The provincial governments were wholly responsible to the provincial legislature Secondly Provinces were free from outside control and interference in the large number of matters The act divided the powers between the centre and provinces in terms of three lists- Federal list( for centre) Provincial list (for province) and concurrent list (for both) Residuary powers were given to the viceroy In the election under the government of India Act the Congress swept the poll the mandate of the people came in favour of the congress so far as general Hindu seats were concerned The Congress did not get a single Muslim seates in Bombay CP UP Sind and BengalIn five provinces Congress had yhe clear majority In BengalNWFPAssam and Bombay Congress emerged as a single largest partyOn the other side the performance of the Muslim League was badThus the Congress formed ministers in 7 provinces out of 11 provinces Coalition ministry was also formed in two other provincesOnly BENGAL AND Punjab had non- congress ministries

1 QUESTION What was the main change introduced by the Government of India ActANSWER a) The Act gave more

autonomy to the provinces b) Diarchy was abolished at the

provincial levelsc) The Governor was the head of

the executived) There was a council of

ministers to advise him The ministers were responsible to the provincial legislatures who controlled them The legislature could also remove the ministers

e) The Governors still retained special reserve powers

2 QUESTION Why did the federal scheme introduced by the Government of India Act 1935 never come into operation

ANSWER The Federal structure of the Government of India was to be composed with the Governor General and Council of ministers The Federal legislature was to be Bicameral legislature- The council of states and the House of Assembly The ministers were to be chosen by the Governor general and they were to hold the office during his pleasure

The provinces of British India would have to join the federation but this was not compulsory for the princely states

This federation never materialised because of the lack of support from the required number of

princely statesThis act was refused and

rejected by the princes the Congress and the Muslim League

Thus both Congress and the League participated in the election of 1937 Thus the federal part was never introduced but the provincial part was put into operations

Bengali 2nd

Language

াচেরর পরাথCনা(কহিতা )

াচেরর পরাথCনা কহিতাটি কহি (ঙখ দেঘাচে4র দো আচো য কহিতায় াচেরর পতর হমায়ন কঠিন দেরাচেগ আxানত ার ঈশবর া আললার কাচেছ পরাথCনা কচেরচেছন তার পচেতরর ীন হিফহিরচেয় হিচেত এই কহিতায় ার পচেতরর ীন হিভbা দেচেয়চেছন ারার এমনহিক হিনচের ীন হিসCচেনর হিহিনমচেয় হিতহিন তার দেছচের ীন হিফচের দেপচেত দেচেয়চেছন তার দেছচের এই দেরাচেগর ন য হিতহিন হিনচেচেকই ায়ী কচেরচেছন তার হিনচের করা পাপচেকই হিতহিন ায়ী কচেরচেছন এছাা রানৈনহিতক ও আথCসামাহিক অসথার কথা তচে ধরা চেয়চেছ এই কহিতায় ার তার হিনচের পাপ কমCচেকই ায়ী কচেরচেছ ার অন যায় ভাচে দেপহি((হিকতর মাধ যচেম অপররা য কচেরচেছ আর এই অন যায় কাচের ন যই তার পহিরাচের হিপযCয় এচেসচেছ দে এক পরকার মানহিক নধন ইহিতাচেসর ার হিপতা চেয় সবাভাহিকভাচে ভাচোাসা দে মমতা দেথচেক মকত চেত পাচেরনহিন তাই হিপতা চেয় আললা া ভগাচেনর কাচেছ পতর হমায়চেনর পরানহিভbা দেচেয়চেছন ার আললা া ভগাচেনর কাচেছ াহিনচেয়চেছন তার হিনচের ীন হিসCন হিচেত হিতহিন রাী তার হিহিনমচেয় পচেতরর ীন হিফচের দেপচেত দেচেয়চেছন াচেরর হিপতসভ হিচেকর কথা এই কহিতায় ফটিচেয় দেতাা চেয়চেছ হিপতা পচেতরর হিরাহিরত মান নধচেনর কথা তচে ধরা চেয়চেছ

হিচে(4 হিকছ াইচেনর তাৎপযC১) ldquoদেকাথায় দেগ ওর সবচছয দেৌন দেকাথায় কচেরায় দেগাপন bয়ldquoউততর) াচেরর পতর হমায়ন কঠিন দেরাচেগ অসসথ তাই তার দেযৌন াহিরচেয় যাচেচছ এই দেরাচেগ তাচেক দেগাপচেন কচেরকচের াচেচছ তার সক (হিকত ধীচের ধীচের bয় চেচছ তাই হিপতা চেয় ার আললার কাচেছ হমায়চেনর পরান হিভbা দেচেয়চেছন২) ldquoাগাও (চেরর পরাচেনত পরানতচের ধসর (ন দেযর আান গানldquoউততর) াচেরর পতর হমায়ন কঠিন দেরাচেগ আxানত তাই ার আ দে(াচেক মমCাত (চেরর পচেথ পরানতচের আান গান ধবহিনত দোক দেসই আান গান আললার কাচেছ দেযন চে যায় আললা দেযন এই আহিতC শচেন পচেতরর ীন হিফহিরচেয় দেয় ৩)ldquoনাহিক এই (রীচেরর পাচেপর ীানচেত দেকানই তরারণ দেনই ভহি4চেতরldquoউততর) হমায়চেনর অসসথতার ন য ার হিনচেচেকই ায়ী কচেরচেছন কারন ার অচেনক রা য অন যায় ভাচে কচেরচেছ তাই তার এই পাপ কাচের ন য তার ঘচের আ হিপ এচেসচেছ এই অন যায় কাচের ন য তার মহিকত দেনই তাই ার আললার কাচেছ এই পাপ কাচেযCর ন য bমা পরাথM

Hindi 2ndlang

-ासी(जयशकर परसा-)

-ासी जयशकर परसा- की एक ऐसी कहानी ह जिजसम भारतीय ससकनित और राषटरीयता का सवरगजीतहोता ह इस कहानी म इरावती एक निहद कनया ह जिजस मलअचछो न मलतान की लट म पकडा और -ासी बना दि-या उस 500 दि-न -कर काशी क एक महाजन न खरी-ा दसरी -ासी निफरोजा ह वह गलाम ह निफरोजा को छडान क कतिलए अहम- को 1000 सोन क कतिसकक भजन थ जो अभी तक नही आए थ राजा साहब कठोर होत हए भी निफरोजा को निबना धनराकतिश क कतिलए उस म कर -त ह वनिफरोजा को अहम- को समझान की बात कहत हकहानी क अत म हम -खत ह निक इरा वती और जाटो क सर-ार बलराज का मिमलन होता हअहम- को यa म मार दि-या जाता ह वहा निफरोजा की परसननता की समामिध बनती ह वहा एक फल चढती ह और डीजल आती ह निफरोजा उस समामिध की आजीवन -ासी बनी रहती हलखक अपन उददशय अथात -ास परथा पर परकाश डालन और इस परथा क कारण होन वाल -ातो क दखो को दि-खान म पणता सफल हए ह

helliphellipContinue to next

Biology Reproductio Today we will discuss about vegetative Q1 Name some vegetative propagules

n in Organisms

propagation of plants The process of multiplication in which fragments of plant body function as propagule and develop into new individual is called vegetative propagation The units of such propagation are runner rhizome tuber bulb etc

and the speciesinvolvedVegetative propagules

Parts involved

Bulb StemBulbil BulbilRhizome Stem Runner Stem Tuber Stem Offset Stem Leaf buds Leaves Suckers Stem

Corns Stem stolon

Q2 State advantages of vegetative propagation

i) Rapid methodii) Sure and easy methodiii) Useful in plants that cannot

produce viable seeds or long seed dormancy

iv) Maintains purity of raceQ 3 Banana fruit is said to be parthenocarpic where as turkey is said to be parthenogenetic WhyBanana develops without fertilization from an unfertilized ovary thus is parthenocarpicIn turkey the ovum or female gamete developinto a new chick without fertilization thus isparthgenetic

Q4 Why is water hyacinth is called as a ldquoTerror of Bengalrdquo Water hyacinth can

propagatevegetatively all over the water body in a short per short period of time This resulted increased biochemicaloxygen oxygen demand of water body causing mortalityof fishes It is very difficult to get rid off them Thus known as terror of Bengal

Chemistry

Solid state GENERAL CHARACTERISTICS OF SOLID STATEIn nature the particular state of matter is governed by two opposing forces at given set of temperature and pressure These forces are intermolecular force of attraction and thermal energy If intermolecular force of attraction is high as compared to thermal energy particles remains in closest position

Intext QuestionsQ1 Classify the following solids as crystalline and amorphous Sodium chloride quartz glass quartz rubber polyvinyl chloride Teflon

A1 Crystalline

and hence very less movement in particles is observed In this case solid state is the preferred state of matter

Let us revise the general characteristics of solid

i) Fixed mass volume and shape

ii) Strong intermolecular force of attraction

iii) Least intermolecular space

iv) Fixed position of constituent particles

v) Incompressible and rigid

Q2 what type of interactions hold the molecules together in a polar molecular solid[CBSE 2010]A2 The molecules in a solid are held together by van der Waals forces The term van der Waals forces include hydrogen bonding dipole-dipole attraction and London dispersion forces All molecules experience London dispersion forces In addition polar molecules can also experience dipole-dipole interactions So the interactions that holds the molecule together in polar molecular solid are London dispersion force and dipole-dipole interactionsQ3 Write a feature that will distinguish a metallic solid from an ionic solid [CBSE 2010]A3 Metals are malleable and ductile whereas ionic solid are hard and brittle Metallic solid has typical metallic lustre But ionic solid looks dullQ4 Write a point of distinction between a metallic solid and an ionic solid other than metallic lustre [CBSE 2012]A4 Metals are malleable and ductile whereas ionic solid are hard and brittleQ5 Write a distinguish feature of metallic solid [CBSE 2010]A5 The force of attraction in

solid Sodium chloride Quartz Amorphous solid Quartz glass rubber polyvinyl chloride Teflon Q2 why glass is considered as super cooled liquidA2 Glass shows the tendency to flow at slower rate like liquid Hence they considered as super cooled liquidQ3 why the window glass of old buildings show milky appearance with timeA3 Glass is an amorphous solid Amorphous solid has the tendency to develop some crystalline character on heating Due to heating in day over the number of years glass acquires some crystalline character and show milky appearanceQ4 why the glass panes fixed to window or doors of old building become slightly thicker at bottomA4 Glass is super cooled liquid It has the tendency to flow down very slowly Due to this glass pane becomes thicker at the bottom over the timeQ5 Sodium chloride is a crystalline solid It shows the same value of refractive index along all the direction TrueFalse Give reasonA5 FalseCrystalline solid shows anisotropy in properties That is it shows different values for the given physical property in different direction All the crystalline solids show anisotropy in refractive index Therefore sodium chloride will show different values of refractive index on different directions

Q6 Crystalline solid are anisotropic in nature What does this statement means

between the constituent particles is special kind of electrostatic attraction That is the attraction of positively charged kernel with sea of delocalized electronsQ6 which group of solid is electrical conductor as well as malleable and ductile [CBSE 2013]A6 Metallic solidQ7 why graphite is good conductor of electricity although it is a network (covalent solid)A7 The exceptional property of graphite is due to its typical structure In graphite each carbon is covalently bonded with 3 atoms in same layer The fourth valence electron of each atom is free to move in between different layersThis free electron makes the graphite a good conductor of electricity

[CBSE 2011]A6 Anisotropy is defined asrdquo Difference in properties when measured along different axis or from different directionsrdquo Crystalline solid show different values of some of the physical properties like electrical resistance refractive index etcwhen measured along the different directions The anisotropy in crystalline solid arises due to the different arrangement of particles in different directions

Math Function Composition of functions Think of an industrial plant that produce bottles of cold drinks first there is the operation (or function) f that puts the cold drink inside the bottle followed by the opeartion g that close the bottle with the capThis leads to the following definitionDefinition Let f A rarr B and g B rarr C be two functions Then the composition of f and g denoted by gof is defined as the function gof A rarr C given by gof(x) = g(f (x)) forall x isinA

Definition A function f X rarr Y is defined to be invertible if there exists a function g Y rarr X such that gof = IX and fog = IY The function g is called the inverse of f and is denoted by f -1

Thus if f is invertible then f must be one-one and onto and conversely if f is one-one and onto then f must be invertible This fact significantly helps for proving a function f to be invertible by showing that f is one-one and onto specially when the actual inverse of f is not to be determined

Example 1 Let f 2 3 4 5 rarr 3 4 5 9 and g 3 4 5 9 rarr 7 11 15 be functions defined as f(2) = 3 f(3) = 4 f(4) = f(5) = 5 and g (3) = g (4) = 7 and g (5) = g (9) = 11 Find gofSolution We have gof(2) = g (f(2)) = g (3) = 7 gof(3) = g (f(3)) = g (4) = 7gof(4) = g (f(4)) = g (5) = 11 and gof(5) = g (5) = 11Example 2 Find gof and fog if f R rarr R and g R rarr R are given by f(x) = cos x and g (x) = 3x2 Show that gof ne fogSolution We have gof(x) = g(f(x))=g(cosx) = 3 (cos x)2

= 3 cos2 x Similarly fog(x)=f(g (x))= f(3x2)= cos (3x2) Note that 3cos2 x ne cos 3x2 for x = 0 Hence gof ne fogExample 3 Show that if f A rarr B and g B rarr C are onto then gof A rarr C is also ontoSolution Given an arbitrary element z isin C there exists a pre-image y of z under g such that g (y) = z since g is onto Further for y isin B there exists an element x in A with f(x) = y since f is onto Therefore gof(x) = g (f(x)) = g (y) = z showing that gof is onto Example 4 Let Y = n2 n isin N sub N Consider f N rarr Y as f(n) = n2 Show that

f is invertible Find the inverse of fSolution An arbitrary element y in Y is of the form n2 for some n isin N This implies that n =radicy This gives a function g Y rarr N defined by g (y) =radicy Nowgof (n) = g (n2)=radicn2 = n and fog (y) =f(radicy) = (radicy) 2 y which shows that gof=IN and fog= IY Hence f is invertible with f -1 = g

Political Science

Constitution of India-The Preamble

Summary

Objective of the state-To secure equality of status and of opportunity To promote fraternity among all the citizens To assure the dignity of the individuals and Unity and integrity of the nation

Justice-Justice stands for rule of law absence of arbitrariness and a system of equal rights freedom and opportunities for all in a society India seeks social economic and political justice to ensure equality to its citizens

Liberty-Liberty implies the absence of restraints or domination on the activities of an individual such as freedom from slavery serfdom imprisonment despotism etc The Preamble provides for the liberty of thought expression belief faith and worship

Equality-Equality means the absence of privileges or discrimination against any section of the society The Preamble provides for equality of status and opportunity to all the people of the country

Fraternity-The Preamble declares that fraternity has to assure two thingsmdashthe dignity of the individual and the unity and

Execution

Answer the following questions-

Short notes-1 Equality2 Fraternity3 Justice4 Liberty

Homework-Learn

integrity of the nation The word integrity has been added to the Preamble by the 42nd Constitutional Amendment (1976)

Business studies

Human resource management (chapter 1)

On the day of 1504 2020 I have discussed with you the managerial functions and procurement functions of HRM

Today weare going to discuss about the development function integration functions and maintenance function

Development functions-HRM improves the knowledge skills attitude and values of employees so that they the present and future jobs more effectively it includes

1) Development functions of HRM

a) Performance appraisal = It implies systematic evaluation of employees with respect to their performance on the job and their potential for development

b) Training =It is the process by which employees learn knowledge skills and attitudes to achieve organisational and personal goals

c) Executive development = It is the process of developing managerial talent through appropriate program

2) Integration functionsa) HRM reconcile the goals of

organisation with those of its members through integrating function

b) HRM tries to motivate employees to various financial and non financial incentives provided in job specification etc

3) Maintenance functiona) HRM promote and protect the

physical and mental health of employees by providing several types of benefits like housing medical aid etc

b) It Promote Social security measures to employees by providing provident fund pension gratuity maternity benefits

SubjectCOMMERCE

Topic

BUSINESSENVIRONMENT

Summary

Now quickly let us revise the earlier points that we have already done in the last class and let us proceed with the other topics that are there in the chapter

Firstly we will recall the internal and external factors of micro environment and then we

Execution 3 What do you mean by internal factors

in micro environmentAnswerInternal factors refer to all the factors existing within a business firm The internal factors are considered controllable because the enterprise has control over these factors

Development FunctionsPerformance AppraisalTrainingExecution Development

shall proceed in details

Meaning and list of internal and external factors

aInternal factorsInternal factors refer to all the factors existing within a business firm The internal factors are considered controllable because the enterprise has control over these factorsFor an example a company can alter its organization structure policies programmes employees physical facilities and marketing mix to suit the changes in the environmentList of internal factors areCorporate culture mission and objectives top management organizations structure company image and brand equity company resources

b External factorsExternal factors refer to those individual and groups and agencies with which a particular business organization comes into direct and frequent contact in the course of its functioningThese individuals and groups are known as STAKEHOLDERS because they have a stake (financial interest ) in the working and performance of the particular business List of external forces (stakeholders)Customers competitors investors suppliersmiddlemen (marketing intermediaries)financers publics

customers

suppliersfinancers

For an example a company can alter its organization structure policies programmes employees physical facilities and marketing mix to suit the changes in the environment

4 What do you mean by external factors in micro environment

AnswerExternal factors refer to those individual and groups and agencies with which a particular business organization comes into direct and frequent contact in the course of its functioningThese individuals and groups are known as STAKEHOLDERS because they have a stake (financial interest) in the working and performance of the particular business

3Who are stakeholdersSTAKEHOLDERS are individuals and groups who have a stake (financial interest ) in the working and performance of the particular business 4Discuss the internal factors in briefa Corporate CultureThe values beliefs and attitudes of the founders and top management of the company exercise a strong influence on what the cmpaany stands for how it does things and what it considers importantbMission and objectivesThe business philosophy and purpose of a comoany guide it prioritiesbusiness strategiesproduct market scope and development scope

cTop management structurethe composition of board of directors the degree of professionalization of management and the organizational structure of a company have important bearing on its business decisions

dPower structureThe internal power relationship between the board of directors and the chief executive is an important factor

e Company image and brand equityThe image and brand equity of the company play a significant role in raising finance forming alliance choosing dealers and suppliers launching new products entering foreign markets

5 What is Macro environmentAnswerMacro environment refers to the general

competitors

middlemen

publics

Fig STAKEHOLDERS OF A COMPANY

Apart from micro environment the other main dimension of business environment isMacro environment Macro environment refers to the general environment or remote environment within which a business firm and forces in its micro environment operateA company does not directly or regularly interact with the micro environmentTherefore macro environment is also known as indirect action EnvironmentThe macro environment forces are less controllable than the micro forces

Macro environment consists of the following components

POLITICAL AND LEGAL ENVIRONMENT

ECONOMIC SOCIAL AND ENVIRONMENT

CULTURAL

ENVIRONMENT

TECHNOLOGICAL ENVIRONMENT

Fig COMPONENTS OF MACRO ENVIRONMENT

environment or remote environment within which a business firm and forces in its micro environment operateA company does not directly or regularly interact with the micro environmentTherefore macro environment is also known as indirect action EnvironmentThe macro environment forces are less controllable than the micro forces 6 What are the components of macro environmenta Political and legal environmentb Economic environmentc Social and cultural environmentd Technological environment

Computer Science

Logic gates

Digital systems are said to be constructed by using logic gates These gates are the AND OR NOT NAND NOR EXOR and EXNOR

BUSINESS FIRM

gates The basic operations are described below with the aid of truth tables

AND gate

The AND gate is an electronic circuit that gives a high output (1) only if all its inputs are high A dot () is used to show the AND operation ie AB Bear in mind that this dot is sometimes omitted ie ABOR gate

The OR gate is an electronic circuit that gives a high output (1) if one or more of its inputs are high A plus (+) is used to show the OR operationNOT gate

The NOT gate is an electronic circuit that produces an inverted version of the input at its output It is also known as an inverter If the input variable is A the inverted output is known as NOT A This is also shown as A or A with a bar over the top as shown at the outputs The diagrams below show two ways that the NAND logic gate can be configured to produce a NOT gate It can also be done using NOR logic gates in the same way

NAND gate

This is a NOT-AND gate which is equal to an AND gate followed by a NOT gate The outputs of all NAND gates are high if any of the inputs are low The symbol is an AND gate with a small circle on the output The small circle represents inversion

NOR gate

This is a NOT-OR gate which is equal to an OR gate followed by a NOT gate The outputs of all NOR gates are low if any of the inputs are highThe symbol is an OR gate with a small circle on the output The small circle represents inversion

EXOR gate

The Exclusive-OR gate is a circuit which will give a high output if either but not both of its two inputs are high An encircled plus sign ( ) is used to show the EOR operation

EXNOR gate

The Exclusive-NOR gate circuit does the opposite to the EOR gate It will give a low output if either but not both of its two inputs are high The symbol is an EXOR gate with a small circle on the output The small circle represents inversion The NAND and NOR gates are called universal functions since with either one the AND and OR functions and NOT can be generated

Note A function in sum of products form can be implemented using NAND gates by replacing all AND and OR gates by NAND gates A function in product of sums form can be implemented using NOR gates by replacing all AND and OR gates by NOR gates

Logic gate symbols

Table 2 is a summary truth table of the inputoutput combinations for the NOT gate together with all possible inputoutput combinations for the other gate functions Also note that a truth table with n inputs has 2n rows You can compare the outputs of different gates

Logic gates representation using the Truth table

Example

A NAND gate can be used as a NOT gate using either of the following wiring configurations

Subject Eng Literature (The Tempest ndash William Shakespeare) Topic Act III Scene 3 Lines 53 to 110 (End of the scene) Date 16th April 2020 (2nd Period)

[Students should read the original play and also the paraphrase given in the school prescribed textbook]Summary Questions amp Answers

o Seeing this strange scene all are inclined to believe the tales told by travelers that there truly are ldquounicornsrdquo and ldquothe phoenixrsquo thronerdquo

o As they are about to sit down to the feast the banquet is snatched away by a harpy (Ariel disguised) A spiritrsquos voice (Arielrsquos voice) denounces Alonso Sebastian and Antonio with particular

1 ARIEL You are three men of sin whom Destiny

(Line 53-58)That hath to instrument this

lower world And what is int the never-surfeited sea

Hath caused to belch up you and on this island

Where man doth not inhabit you rsquomongst men

Being most unfit to live I have made you mad

reference to their crime in expelling Prospero from Milan They have not received any punishment for their deed earlier but the time for their punishment has arrived Upon Alonso it pronounces ldquolingering perdition worse than deathrdquo from which there is no remedy except through sincere repentance Ariel then vanishes in thunder and the shapes enter again and carry away the table

o Prospero watching invisibly is very pleased with the performance of Ariel and his (Prosperorsquos) ldquomeaner ministersrdquo All his enemies are now in his power and are in a fit of desperation He then leaves them and goes to see how Ferdinand and Miranda are getting on

o Alonso is now much humbled and penitent with the after effect of the spiritrsquos denunciation of his crimes He believes that his son is lost forever After this all disperse being stricken mad by the speech of the spirit

o Gonzalo fearing that they may do violence to themselves or to one another follows them and bid others to follow

(a) To whom does Ariel disguised as a harpy call the three sinners What game did Fate of Destiny play with

them

The three sinners called by Ariel are Alonso Sebastian and Antonio It was Destiny which had caused the ocean to cast the three sinners on the shore Though the ocean is all the time devouring whatever appears on its surface and is never satisfied with its continual swallowing of the ships and men in the present case the ocean had cast these three sinners on the shore without killing them

(b) Who had jointly been responsible for the conspiracy against Prospero What is Prosperorsquos purpose behind all this

Three men Alonso Sebastian and Antonio had jointly

been responsible for the conspiracy against Prospero They had driven out Prospero form Milan Prosperorsquos purpose is to make these three sinners realize the wrong they had done He wants them to repent for their criminal deeds because repentance leads to self-esteem(c )What does Ariel (the harpy) tell Alonso and his companions when they take out their swords to attack him

Seeing them drawing their swords Ariel (harpy) tells them that he and his companions are the instruments of destiny and that it is not possible for human beings to do them any injury He says that the swords of human beings can not injure even a minute part of his feathers Their swords are as ineffective against him and his companions as against the wind or the water

(d) Give the explanatory meanings of the following expressions in the context of the above extract

(i)Never surfeited (ii) Belch up (iii) lsquomongst men

(i) Never surfeited never led to satisfaction

(ii) Belch up cast ashore(iii) lsquomongst men in human

society2

I and my fellows (Line 60-65)

Are ministers of Fate The elementsOf whom your swords are tempered may as wellWound the loud winds or with bemocked-at stabsKill the still-closing waters as diminishOne dowl thats in my plume

IMPORTANT PASSAGES EXPLAINED

The elements

(Line 61-66)Of whom your swords are tempered may

as wellWound the loud winds or with

bemocked-at stabs

(a) Who is lsquoIrsquo Who are his lsquofellowsrdquo

lsquoIrsquo is referred to Ariel in disguise of a harpy His lsquofellowsrsquo are other spirits serving Prospero the real Duke of Milan who has acquired supernatural powers after being banished from his Dukedom Prospero has settled in this uninhabited island

(b) What are the elements that have temperrsquod the swords Why will it not work against the speaker

The swords (of Alonso and his companions) are tempered by metal (steel) which is taken out of the earth and refined by

Kill the still-closing waters as diminishOne dowl thats in my plume My fellow

ministersAre like invulnerable

In these words Ariel reminds the King and his companions of the utter futility of drawing swords against himself and his fellows Ariel drives Alonso Antonio and Sebastian the three men of sin to desperation ndash a state in which men do violence to themselves They draw swords to strike Ariel But Ariel reminds them that he and the other spirits are the ministers of destiny and nothing can wound them The steel of which their swords are made of may cut the wind or water which being divided always closes up again Even supposing that such things may be possible it is quite impossible that their swords will cut one feather in their plume They are incapable of being wounded by any sword of man Hence it is foolish on their part to attempt to strike at Ariel and his fellow-spirits

For which foul deed

(Line 72-75)The powers delaying not forgetting

haveIncensed the seas and shores yea all the

creatures Against your peace

Ariel enters like a harpy and remaining invisible tells Alonso Sebastian and Antonio that he and other harpies are the agents of Destiny appointed to carry out her decrees He tells them that their punishment for the crime against Prospero which has been so long deferred is now to fall upon them He reminds them that they had expelled Prospero from Milan and set him and his innocent child adrift on the sea and that the sea had paid them back for their sin by the shipwreck and by the calamities they have suffered He tells them that the powers above which did not forget this mean treachery but only deferred the punishment have now engaged the seas and the shores and all living beings including him and his comrades against them The very elements and supernatural agency Ariel adds have taken up the avenging of their crime against Prospero

the action of fire It may cut the wind or water which being divided always closes up again

The sword will not work against the spirits and the harpy because they are the ministers of destiny and nothing can wound them nor it will cut a single feather in their plume

(c )What is the meaning of lsquodowlrsquo in the last line

The term lsquodowlrsquo means a filament or the smallest part of a feather In this context Ariel in disguise of harpy says that their sword cannot even damage the smallest filament of their (Arielrsquos and other spirits) feathers as they are incapable of being wounded by any sword of man

(d) What does the speaker remind the listeners about

Ariel in disguise of harpy reminds Alonso the King of Naples Sebastian Alonsorsquos brother and Antonio the present Duke of Milan and the treacherous brother of Prospero as they being three men of sin He even reminds them that their punishment for their crime against Prospero which has been so long deferred now falls upon them He reminds them that they have expelled Prospero from Milan and has set him along with his innocent infant daughter adrift on the sea So the sea has paid them back for their sin by their shipwreck and the calamities they have suffered since then The harpy rebukes Alonso of his sin that has incensed the Gods and has deprived him of his son as a punishment

(e) How do they respond

When Ariel in disguise of a harpy reminds Alonso Sebastian and Antonio of their past misdeeds and sin Alonso has a look of terror and confusion in his eyes He utters the words of sincere repentance wrung out of his conscience-stricken heart It appears to him that all the elements of nature the sea-waves the wind and the thunder proclaiming a loud voice in the name of Prospero and the crime Alonso has committed against him They are calling upon him to repent There is a deep storm raging in Alonsorsquos breast and the echoes of that storm are ringing in his ears like a clear note of wind-instrument A note of denunciation of Alonsorsquos crime leaves him much humbled and penitent and confirms his belief that his son is lost forever But Sebastian and Antonio shows some courage instead of repentance They wish to kill the spirits or devils if it appears

3

Of my instruction hast thou nothing bated (Line 85-93)

In what thou hast to say So with good life

And observation strange my meaner ministers

Their several kinds have done My high charms work

And these mine enemies are all knit upIn their distractions They now are in my

powerAnd in these fits I leave them while I visitYoung Ferdinand whom they suppose is

drownedAnd his and mine loved darling

Methought the billows spoke and (Line 96-99)

told me of itThe winds did sing it to me and the

thunderThat deep and dreadful organ-pipe

pronouncedThe name of Prosper It did bass my

trespass

These are the words of contrition coming from Alonso Ariel has driven him to a deep repentance for conspiring with Antonio against Prospero He now feels a sincere remorse It appears to him that all the elements of nature the sea-waves the wind and the thunder proclaimed with a loud voice the name of Prospero and the crime Alonso had committed against him They are calling upon him to repent There is a deep storm raging in Alonsorsquos breast and the echoes of that storm are ringing in his ears like the clear note of a wind-instrument

Comment These are the words of sincere repentance wrung out of the conscience-stricken heart of Alonso Alonso who is the lesser villain is the first to give way to remorse under the effect of Arielrsquos speech The words of Ariel seem to him to be the voice of conscience speaking to him He is driven to desperation a state in which he might do violence to his life

(a) Identify the speaker State the context

Prospero the ruler of the island is the speaker The famous banquet scene has been enacted very well Ariel and his junior spirits have played their roles excellently Prospero is glad to say words of praise for them(b) In what way the speakerrsquos instructions have been carried out

According to Prosperorsquos instructions a banquet was presented before the King of Naples and his companions when they were tired and hungry Just when they were preparing to eat the feast the banquet was suddenly removed by exercising supernatural powers All this was done by Ariel Prosperorsquos chief assistant and a powerful spirit

Ariel not only made the feast disappear but also delivered his speech blaming the King and his two companions for their past wicked deeds He warned them to repent for their misdeeds or suffer forever on that uninhabited island

(c) Who are referred to as lsquomeaner ministersrsquo What have they done

Prospero refers as lsquomeaner ministersrsquo to his other lesser spirits who were assisting Ariel in presenting a scene before the kingrsquos party They entered the scene to the accompaniment of music They assumed several strange shapes and brought in a banquet Then they danced about it with gentle actions of salutations thus inviting the King and others to eat the feast

These spirits play their role again when Ariel in the shape of a harpy quits the scene These shapes enter again and dancing with mocking gestures carry away the table

(d) Who are the speakerrsquos enemies What has happened to them

King of Naples Alonso his brother Sebastian and the present Duke of Milan Antonio (Prosperorsquos own brother) are Prosperorsquos enemies With the turn of events they have all been washed ashore on the island which is ruled by Prospero the great magician Actually this happened after the shipwreck caused by a storm which was raised by Prospero with the purpose of bringing these people to his island Prosperorsquos spirits have already confused and terrified these enemies and they are under Prosperorsquos control He can treat them as he likes

(e) What does he say about Ferdinand Explain what is meant by ldquohellip his and mine darlingrdquo

Prospero knows that Alonsorsquos son prince Ferdinand is alive though his father thinks that the prince has been drowned

Prospero refers to his daughter Miranda who is dear to him She is also very dear to Prince Ferdinand who has fallen in love with her They are waiting to be married soon for which they have received Prosperorsquos consent

4

ALONSO O it is monstrous monstrous (Line 95-102)

Methought the billows spoke and told me of it

The winds did sing it to me and the thunderThat deep and dreadful organ-

pipe pronouncedThe name of Prosper It did bass

my trespassTherefore my son ithrsquo ooze is

bedded andIll seek him deeper than eer

plummet soundedAnd with him there lie mudded

(a) In what way does Alonso express his horror when his conscience is awakened by Arielrsquos words

When Alonsorsquos conscience is awakened by Arielrsquos words he expresses his horror at what he has heard He gets the feeling that the waves of the ocean the wind and the loud thunder have spoken to him and uttered the name of Prospero Because of being reminded of his crime in a very loud and rough voice he comes to realize that he has lost his son for his past misdeeds

(b) What does Alonso imagine about his son What does Alonso want to do in his desperate state

Alonso imagines that his son is lying in the mud at the bottom of the sea He feels desperate that he wants to drown himself in the ocean deeper than the plumb-line has ever gone He wants to lie with his son at the bottom of the sea

(c) How do Sebastian and Antonio want to face the evil spirits

Sebastian says that he is not at all afraid of what the harpy has said and that he is prepared to fight any number of such monsters if they appear before him only one at a time Antonio says that he would support Sebastian in the fight against the fiendsyyy

(d) Why does Gonzalo ask Adrian to follow the three men

Gonzalo tells Adrian that all the three men namely Alonso Sebastian and Antonio are in a wild and reckless mood The thought of the heinous crime of which they are guilty has begun to torment their minds So he asks Adrian to follow those three men without loss of time and prevent them from doing anything which the turmoil in their minds might lead them to do

(e) What opinion do you form of Alonso from the above extract

Alonso who is the lesser villain is the first to give way to remorse under the effect of Arielrsquos speech The words of Ariel seem to him to be the voice of conscience speaking to him He is driven to desperation a state in which he might do violence to his life

Subject =Accounts

Ac-12 15420 topic-pL Appropriation ac

PROFIT AND LOSS APPROPRIATION ACCOUNT

MEANING AND PREPARATIONProfit and Loss Appropriation Account is merely an extension of the Profit and Loss Account of the firm The profit of the firm has to be distributed amongst the partners in their respective profit sharing ratio But before its distribution it needs to be adjusted All Adjustments like partnerrsquos salary partnerrsquos commission interest on capital interest on drawings etc are made in this account These adjustments will reduce the amount of profit for distribution This adjusted profit will be distributed amongst the partners in their profit sharing ratio To prepare it at first the balance of Profit and Loss Account is transferred to this account The journal entries for the preparation of Profit and Loss Appropriation Account are given below

1 for transfer of the balance of Profit and Loss Account to Profit and Loss Appropriation Account

(a) In case of Net Profit

Profit and Loss Ac helliphelliphelliphelliphellipDrTo Profit and Loss Appropriation Ac(Net Profit transferred to Profit and Loss Appropriation Ac)

(b)In case of Net Loss

Profit and Loss Appropriation Achelliphelliphellip DrTo Profit and Loss Ac(Net Loss transferred to Profit and Loss Appropriation Ac)

2 for Interest on Capital

For transferring on Interest on CapitalProfit and Loss Appropriation Achelliphelliphellip DrTo Interest on Capital Ac(Interest on capital transferred to Profit amp Loss Appropriation Ac)

3 for Interest on Drawings

For transferring Interest on Drawings Interest on Drawings Achelliphelliphelliphelliphelliphellip DrTo Profit and Loss Appropriation Ac(Interest on drawing transferred to Profit amp Loss Appropriation Ac)

4 For Partnerrsquos SalaryFor transfer of partnerrsquos SalaryProfit and Loss Appropriation Achelliphellip DrTo Salary Ac(Salary transferred to profit amp Loss Appropriation Ac)

5 For Partnerrsquos CommissionFor transferring commissionProfit and Loss Appropriation Achelliphelliphellip DrTo Commission Ac(Commission transferred to Profit and Loss Appropriation Ac)

6 For Transfer of agreed amount to General ReserveProfit and Loss Appropriation Ac helliphellipDrTo General Reserve Ac(Transfer to General Reserve)

7 for share of Profit or Loss appropriation(a) If ProfitProfit and Loss Appropriation Achelliphellip DrTo Partnerrsquos CapitalCurrent Ac(Profit transferred to capitalcurrent Ac)(b) If LossPartnerrsquos Capital Current Achelliphelliphelliphellip DrTo Profit and Loss Appropriation Ac(Loss transferred to capitalcurrent Ac)

THE FORMAT OF PROFIT AND LOSS APPROPRIATION

Profit and Loss Appropriation Account for the year endedhelliphelliphelliphellip

Particulars Amount Particulars Amount

To PL Ac (loss) By pL Ac (profit)

To Interest on capital BY Interest on drawings

To partner`s commission by Partner`s capital Ac ( loss)

To Partner`s salary To Interest on partner`s loan To General Reserve To Partner`s Capital AC (Profit)

Subject= Economics

MOVEMENT ALONG THE DEMAND CURVE (CHANGE IN QUANTITY DEMANDED)In law of demand you have already studied the inverse relationship between price and quantity demanded When quantity demanded of a commodity changes due to change in its price keeping other factors constant it is called change in quantity demanded It is graphically expressed as a movement along the same demand curve There can be either a downward movement or an upward movement along the same demand curve Upward movement along the same demand curve is called contraction of demand or decrease in quantity demanded and downward movement along the same demand curve is known as expansion of demand or increase in quantity demanded

Extention of demandd

price (rs)p A

B Extentionp1 d

Q Q1

Quantity demanded ( in units)

Contraction of demandd

p2 Ccontraction

p APrice (Rs)

d

Q2 Q

Quantity demanded (in units)

Explanation of movement of demand A fall in price from OP to OP1 leads to increase in quantity demanded from OQ to OQ1 (expansion of demand) resulting in a downward movement from point A to point B along the same demand curve DD When Price rises from OP to OP2 quantity demanded falls from OQ to OQ2 (contraction of demand) leading to an upward movement from point A to point C along the same demand curve DD

  • Activity Series of Metals
    • Drawbacks of Rutherfordrsquos model of atom
      • Electromagnetic radiations
      • Properties of electromagnetic radiations
      • Characteristics of electromagnetic radiations
        • Plancks Quantum Theory-
        • Photoelectric effect
          • Intext Questions
            • Logic gates
            • Digital systems are said to be constructed by using logic gates These gates are the AND OR NOT NAND NOR EXOR and EXNOR gates The basic operations are described below with the aid of truth tables
            • AND gate
            • Example
Page 27:  · Web viewSubject. Topic. Summary. Execution. English 1 . Chapter 1 naming words . Page 8. Write the names of these pictures:- Person:-1. father. 2.Firefighter 3.doctor 4 ...

a Assertive sentencesb Imperative sentencesc Interrogative sentencesd Exclamatory sentences

Sentences can be changed from one grammatical form to another without changing the meaning of the sentence This is known as transformation of sentences

assertive to interrogative1 Nobody would like to be a fool

Who would like to be a fool2 Their glory can never fade

When can the glory fade3 Nobody can control the wind

Who can control the wind4 It matters little if I die

What though I die5 No man can serve two masters

Can any man serve two masters

Exercise 3Interchange of assertive and Exclamatory sentences

1 She leads the most unhappy lifeWhat an unhappy life she leads

2 This is indeed an interesting bookWhat an interesting book this

3 He is a very great manWhat a great man he is

4 It is a very lame excuseWhat a lame excuse

5 It is sad that she died so youngAlas she died so young

Class XISubject Topic Summary Execution

Hindi 2nd lang

पतर परम(परमचदर) पतर परम कहानी म एक निपता की इचछाओ का वणन निकया गया ह अपन बड पतर परभ -ास स निपता चतनय -ास का निवशरष परम था निपता को उसक जनम स ही बडी-बडी आशाए थी उसम दसर बट कतिशव-ास की अपकषा स- उतसाह की मातरा अमिधक थी वह उस इगलड भजकर बरिरसटर बनाना चाहत थभागय का खल भी बडा निनराला ह बीए की परीकषा क बा- वह बीमार पड गया डॉकटरो न भी जवाब - दि-या थाचतन -ास जी बहत ही कजस थ बवजह पस खच करना नही चाहत थ अगर गारटी मिमलती तो शाय- पस खच भी कर -त परत गारटी नही थी परिरणाम सवरप उनक बट का -हात हो गयाजब बट को समशान ल जा रह थ तो वहा काफी शोर गान बजान हो रह थ पछन पर पता चला निक निकसी निपता निपछल तीन साल स निबमार था और उसक ईलाज म रपया पानी की तरह बहाया पर ठीक नही हए परत उसक बट को तनिनक भी अफसोस नही था उसका कहना था उसन कोकतिशश तो कीयह -खकर चतनय-ास जी को आतम निगलानी हईतभी स उनका म परिरवतन हआ और बट का भोज काफी धमधाम स निकयाऔर वहइस पशचाताप की आग म जलत रह औला- स बढकर पसा नही होता ह इस बात को समझन म उनह काफी व लग गया

hellipContinue to next

BENGALI(2ND LANGUAGE)

পরথমঅধযায়-ঠাকরারীনদরনাথঠাকর

নয়ন দোচের হিমাচেররা া নাচেমই হিযাত হিছচেন ায়ানার উাররণ সবরপ নয়ন দোচের ারা হিা (াচেকর হিা হিচেতন এছাাও দেকান উৎস উপচেb রাহিতর দেক হিন করার উচেfচে(য তারা সযC হিকরচেরণ রনয পরীপ জবাহিচেয় তাচেত রপার হির 4Cরণ করচেতন ঠাকরা এই নয়ন দো হিমারচের দে(4 ং(ধর হিছচেন হিমাররা ায়ানার ষটানত পর(Cন কচের তারা হিনঃসব এই হিমাহিরর দে(4 ং(ধর গৈকাস নদর রায়চেৌধরী গৈকাস া নয়ন দোচের সমসত সমপহিতত ঋচেরণর াচেয় হিহিx কচের অহি(ষট যা আচেছ তাচেত হিপত

ইার হিপতার মতয ইচে পর নয়নচোচের ায়ানার দেগাটা কতক অসাধাররণ শরাদধ (াহিনতচেত অহিনতম ীহিপত পরকা( কহিরয়া ঠাৎ হিনহিয়া দেগ- ক) কার দো দেকান গচেলপর অং() কতা দেক ইার চেত কাচেক দোঝাচেনা চেয়চেছ গ) পরসঙগ কী কতার কতয পহিরসফট কচেরা

পরচে4র যাহিত রbা করা সমভ নয় তাই হিতহিন পতরচেক হিনচেয় ককাতায় সাস শর কচেরন গলপ কথচেকর আহিথCক অসথা নয়ন দোচের হিমাচের দেথচেক সমপরণC আাা কথচেকর হিপতা হিনচের দেষটায় অথC উপাCন করচেতন া উপাহিধ াচেভর নয তার াসা হিছনা আর দেসই কারচেরণ কথক তার একমাতর উততরাহিধকার চেয় তার হিপতার পরহিত কতজঞ কথক দো পা হি(চেচেছন হিনচের পরারণ ও মান রbার নয উপচেযাগী অথC হিনা দেষটায় পরাপত চেয়চেছন- এটাই তার কাচেছ পরম দেগৌরচের হি4য় চে মচেন কচেরন কাররণ (নয ভাণডাচের গৈপতক ায়ানার উজজব ইহিতাস অচেপbা দোার হিসeচেকর মচেধয গৈপতক দেকামপাহিনর কাগ তার কাচেছ অচেনক দেহি( মযান

TO BE CONTINUED

উ- ক) আচোয অং(টি রীনদরনাথ ঠাকচেরর দো ঠাকরা গচেলপর অং() কতা চেন আচোয গচেলপর গলপ কথকইার চেত নয়ন দোচের হিমাহিরর দে(4 ং(ধর গৈকাস ার কথা া চেয়চেছ গৈকাস া নয়ন দোচের সমসত সমপহিতত ঋচেরণর াচেয় হিহিx কচের অহি(ষট যা আচেছ তাচেত হিপত পরচে4র যাহিত রbা করা সমভ নয় তাই হিতহিন পতরচেক হিনচেয় ককাতায় সাস শর কচেরনগ) গৈকাস ার হিপতার মতযর পর নয়ন দোচের হিমাহিরর অহিসততব হিপত য় কচেয়কটা উৎস ও শরাদধ- (াহিনতচেত হিমাহিরর দে(4 কহিটক যয় চেয় হিগচেয় এচেক াচের দে(4 চেয় যায় তন তাচের গC করার মত আর হিকছই হিছ না-দেসই পরসচেঙগ এই উহিকত নয়নচোচের হিমাচেররা া নাচেমই হিযাত হিছচেন ায়ানার উাররণ সবরপ নয়নচোচের ারা হিা (াচেকর হিা হিচেতন এছাাও দেকান উৎস উপচেb রাহিতরচেক হিন করচেত হিগচেয় তারা সযC হিকরচেরণর নয পরীপ জবাহিচেয় তাচেত রপার হির 4Cরণ করচেতন তাই দেসকাচের ায়ানা দেহি(হিন সথায়ী চেত পারত না হিহিভনন উৎস শরাদধ- (াহিনতচেত সাধযা হিতহিরকত র করার নয হিমাহির হিহিকচেয় দেযত হ হিতC কা হিহি(ষট পরীচেপর দেত দেযমন অলপকাচের মচেধয হিনঃচে(4 চেয় যায়-নয়নচোচের হিমারচের অসথা তাই চেয়হিছ এই কারচেরণই কথক নয়নচোচের হিমারচের গা ভরা আমবর সয করচেত পারতনা

Physics Dimensional Analysis (Summary)

Q Find the dimensions of consts ab in relation

p=(bminusxlowastx)at

where p is the power x is the distance and t is time

Ans From principle of homogeneity dimension of b x2 are same Dim of b = dim of x2 = [L2] = [ML2T0]Dim of a = dim of ( b- x2)dim of (pt) = [M0L2T0][ML2T-2] [T-1] [T] = [M-1L0T2]

Chemistry Atomic Structure Drawbacks of Rutherfordrsquos model of

atom a According to Rutherfordrsquos model of atom electrons which are negativelycharged particles revolve around the nucleus in fixed orbits Thusb theelectrons undergo acceleration According to electromagnetic theory of Maxwell a charged particle undergoing acceleration should emitelectromagnetic radiation Thus an electron in an orbit should emitradiation Thus the orbit should shrink But this does not happenc The model does not give any information about how electrons aredistributed around nucleus and what are energies of these electrons Isotopes These are the atoms of the same

Properties of electromagnetic radiationsa Oscillating electric and magnetic field are produced by oscillating charged particles These fields are perpendicular to each other and both areperpendicular to the direction of propagation of the waveb They do not need a medium to travel That means they can even travel invacuum

Characteristics of electromagnetic radiationsa Wavelength It may be defined as the distance between two neighbouring crests or troughs of

element having the same atomicnumber but different mass numbere g 1H11H21H3

Isobars Isobars are the atoms of different elements having the same massnumber but different atomic numbere g 18Ar40 20Ca40

Isoelectronic species These are those species which have the same numberof electrons

Electromagnetic radiationsThe radiations which are associated withelectrical and magnetic fields are called electromagnetic radiations When anelectrically charged particle moves under acceleration alternating electricaland magnetic fields are produced and transmitted These fields aretransmitted in the form of waves These waves are called electromagneticwaves or electromagnetic radiations

wave as shown It is denoted by λb Frequency (ν) It may be defined as the number of waves which passthrough a particular point in one secondc Velocity (v) It is defined as the distance travelled by a wave in onesecond In vacuum all types of electromagnetic radiations travel with thesame velocity Its value is 3 times10 8m sec-1 It is denoted by v

d Wave number Wave number is defined as the number of wavelengths per unit lengthVelocity = frequency timeswavelength c = νλ

Plancks Quantum Theory- o The radiant energy is emitted or absorbed not continuously but discontinuously in the form of small discrete packets of energy called lsquoquantumrsquo In case of light the quantum of energy is called a lsquophotonrsquoo The energy of each quantum is directly proportional to the frequency of the radiation ie E α υ or E= hυ where h= Planckrsquos constant = 6626 x 10-27 Js o Energy is always emitted or absorbed as integral multiple of this uantum E=nhυ Where n=1234Black body An ideal body which emits and absorbs all frequencies is calleda black body The radiation emitted by such a body is called black body radiation

Photoelectric effectThe phenomenon of ejection of electrons from thesurface of metal when light of suitable frequency strikes it is calledphotoelectric effect The ejected electrons are called photoelectrons

Biology Chapter - 02Systematics and Five Kingdoms

Scientists divide the whole living organisms into two kingdom first and ultimately by five kingdom at last

In the earlier systems of classifications organisms are divided into kingdom plantaeand kingdom animalia on the of presenceof cell wall their modes of nutrition and movements

Some problem arise like fungi share manycharacteristic withplant despite their heterotrophic nutrition bacteria protozoa areunicellular present in both kingdom Toovercome this third kingdom Protista isintroduced which include

unicellularorganisms But there is also another

problem Allunicellular organisms are not similar kind The cellular structure of prokaryotes is verydifferent from that of other organismsEukaryotes possess a true nucleus and allcell organelles that are not present inprokaryotes So the fourth kingdom Monerais introduced which include unicellular prokaryotes (bacteriaamp blue green algae)

bull Still some problem arise in kingdomplantae

So in 1969 R H Whittakar proposedanew five kingdom System of classification

i) Kingdom Monera - unicellular prokaryotes

ii) kingdom Protista - unicellular eukaryotes

iii) Kingdom Fungi - uni or multicellular fungi with cell wall but without chlorophyll

iv) Kingdom Plantae - Multicellular Plants

v) Kingdom Animalia - Multicellular Animals

EVS Chapter 1 ndash Modes of Existence

An agricultural society

An agricultural society also known as an agrarian society is a society that constructs social order around a reliance upon farming More than half the people living in that society make their living by farming

People in an agricultural society generally lead a more settled lifestyle than those in nomadic hunter-gatherer or semi-nomadic pastoral societies because they live permanently near the land that is farmed Agricultural settlements tend to develop in areas of convenience near bodies of water which is used for both crops and transportation or along trade routes Not everyone in an agricultural society is a farmer Some people make a living trading or making and selling goods such as tools used for farming

Another way to define an agrarian society is to see the total amount of production in a nation In an agrarian society cultivating the land is the main source of wealth Such a society can recognize other means of subsistence and work habits but emphasizes the importance of agriculture and livestock Agrarian societies have existed in various parts of the world for 10000 years and continue to exist today They have been the most common form of socio-economic organization for most of recorded human history

Q) Write the features of agricultural society

Ans - Structure and Features of Agrarian Society1 Occupational Structure

An agrarian society is generally associated with the domestication of plants and animals The domestication of plants means farming and that of animals means herding Often there is mixture of farming and the use of such domesticated animals as cow goat and sheep

2 Forms of Land Ownership in Agrarian SocietiesGenerally there are landlords supervisory farmers cultivators and share croppers The landholders own the land but do not work on it They let it out for sharecropping The supervisory farmers are those who live by having their land cultivated by hired labourers The cultivators cultivate the land for themselvesThe share-croppers are those who live by tilling other peoplersquos land or a crop-sharing basis The artisans own their means of production and produce by their own labour in their homesteads

3 Village Community System An agrarian society is highlighted by

the institution of village community system The agrarian economy made fixed dwelling houses necessary Living close together for protection and co-operation and living nearer to the land gave birth to agricultural villages The village is not only the residential place of farmers it is also the social integrator

4 Minimal Division of Labour Another structural feature of agrarian society is a minimal division of labour Except for the basic division founded on age and sex differences there are few specialized roles There is only one predominant type of occupation ie domestication of plants and animals For all the people the environment physical as well as social is the same

5 Role of Family The farm family is of the patriarchal type the father is the final arbiter in most of the familyrsquos major decisions The life of ail men and women is merged in family life Since there are not many special organizations family is the only organisation to perform the tasks of aid and protection

6 Sense of Unity The members of an agrarian society exhibit a strong in-group feeling Since the whole of their social lives is wrapped up in a society which is physically economically and socially homogenous they are inclined to view the entire outside world as an out group

7 Informal Social Control An agrarian society is regionally divided into villages In a village community the force of traditional mores is more dominant than in the urban community In the village everybody is known to everybody The members in a village community help each other and share the joy and sorrows of each other Crime in an agrarian society is rare

8 Simplicity and Uniformity Life of the people in an agrarian society is marked by simplicity and uniformity Their main occupation is agriculture which largely depends upon the vagaries of nature An agrarian society is a religious society

Math Compound angles Compound angles The algebraic sum of two or more angles is called a compound angle If A B C be three angles then A+B B+C C+A A-B B-C A-C A+B-C etc are compound angles In this chapter we shall discuss the trigonometrical ratios of compound angles Theorem 1 If A B and A+B are all pisitive acute angles theni) sin( A+B) = sin A cos B + cosA sinBii) cos(A+B) = cosA cosB- sinA sinBTheorem 2If A and B are positive acute angles and AgtB theni) sin(A-B) = sin A cosB- cos A sinBii) cos(A-B) = cos A cos B+ sin A sin BTo prove that i) sin(A+B) sin (A-B) = sin2 A - sin2 B = cos2 B- cos2 A

Example 1 Prove that tan70deg=2tan50deg+tan20degSolutiontan70deg = tan(50deg + 20deg)Or tan70deg=(tan 50deg+tan 20deg)(1-tan50degtan20deg) or tan70deg (1 ndash tan 50deg tan20deg) = tan50deg+tan20degor tan70deg= tan70deg tan50deg tan20deg+ tan50deg + tan20deg = cot20deg tan50deg tan20deg + tan50deg + tan20deg = 2 tan50deg+ tan20degExample 2 If A + B = 45deg show that (1 + tanA) (1 + tanB) = 2Solutiontan(A + B) =( tan A + tan B) (1 - tan

ii) cos(A+B) Cos(A-B) = cos2 A- sin2 B = cos2 B -sin2 AProof i) LHS= sin(A+B)sin(AminusB) [Recall sin(αminusβ)=sinαcosβminuscosαsinβ And sin(α+β)=sinαcosβ+cosαsinβ]= (sinAcosB+cosAsinB)times(sinAcosBminuscosAsinB)= sin2Acos2Bminuscos2Asin2B [Recall sin2α+cos2α=1 From above we can then assume correctly that sin2α=1minuscos2α AND cos2α=1minussin2α] = sin2A(1minussin2B)minussin2B(1minussin2A) = sin2Aminussin2Asin2Bminussin2B+sin2Asin2B = sin2Aminussin2B= 1-cos2A-(1-cos2B) = cos2 B- cos2 A = RHSii)LHS= cos (A+B) cos(A-B) [ cos(A+B) = cos AcosB- sinAsinBCos(A-B) = cosAcosB+ sinAsinB]= cos2 A Cos2 B- sin2 A Sin2 B= cos2 A( 1-sin2 B) - (1- cos2 A) sin2 B= cos2 A- cos2 A sin2 B- sin2 B+ cos2 A sin2 B=cos2 A- sin2 B=1- sin2 A-(1-cos2 B) = cos2 B- sin2 A= RHSTangent formulae for compound anglesi)tan (A + B) = tan A + tan B1-tan A tan Bii)tan (A ndash B) = tan A-tan B1+tan A tan Biii) cot (A + B) = cot Acot B-1cot A+cot B(viii) cot (A ndash B) = cot Acot B+1cot B-cot A

A tan B) Or 1= (tan A+ tanB) (1-tan A tanB) Or tanA + tanB + tanA tanB + 1 = 1 + 1Or tanA (1 + tanB) + (1 + tanB) = 2Or (1 + tanA) (1 + tanB) = 2Example 3 Find the value of sin 15degSolution sin 15deg= sin(45deg-30deg) = sin45degcos 30deg- cos45degsin30deg =(1radic2) (radic32) -(1radic2) (12) = (radic3-1) 2radic2Example 4 If sin A = 1 radic10 and sin B = 1 radic5 where A and B are positive acute angles then what is A + B SolutionWe know that sin (A + B) = sin A cos B + cos A sin B= [1 radic10] [radic(1 minus 1 5)] + [1 radic5] radic(1 minus 1 10)= [1 radic10] [radic4 5] + [1 radic5] [radic9 10]= [1 radic50] times (2 + 3)= 5 radic50 = 1 radic2

sin (A + B) = sin π 4rArrHence A + B = π 4Example 5 If A + B = 225o then find [cot A] [1 + cotA] times [cot B] [1 + cot B]Solution[cot A] [1 + cotA] times [cot B] [1 + cot B] = 1 [(1 + tan A) times (1 + tan B)]=1 [tan A + tan B + 1 + tan A tan B] [ tan (A + B) = tan225o]∵

tan A + tan B = 1minus tan A tan BrArr= 1 [1 minus tan A tan B + 1 + tan A tan B]= 1 2

COMMERCE

CLASSIFICTION OF HUMAN ACTIVITIES-ECONOMIC AND NON-ECONOMIC

Firstly we shall recall the previous class for 5 mins especially for the absentees and for also the rest of the students who were there

Today at first we briefly discuss the earlier portions of the chapter

1Business-It includes all those economic activities which are concerned with production and exchange of goods and services with the object of earning profit Example A factory shop beauty parlour also business enterprises

2Profession ndashThe term profession means an occupation which involves application of specialized knowledge and skills to earn a living For Example Chartered Accountancy medicine law tax consultancy are example of professions

Questions1What are the main features of ProfessionAnswer The main features of a profession are as follows a Specialised body of knowledge-Every profession has a specialised and systematised body of knowledge b Restricted entry- Entry to a profession is allowed only to those who have completed the prescribed education and have the specialised examination c Formal education and training ndashA formal education and training is given to the person who wants to acquire the professional

3Employment-Employment mean an economic activity where people work for others in exchange for some remuneration (salary)The persons who work for others are called lsquoemployeesrsquo The persons or organizations which engage others to work for them are called lsquoemployersrsquoEg A doctor working in a hospital is employment as he is working for a salaryA lawyer may serve as a law officer in a bank

With this we shall proceed with the features of both Profession amp Employment

The main features of a profession are as follow

a Specialised body of knowledge b Restricted entry c Formal education and training d Professional association e Service motive f Code of contact

The main features of an employment are as follows

a In employment a person works for others called employer

b An employee provides personal service

c There is a service agreement or contract between the employee and the employer

d The employee has to obey the order of the employer

e No capital investment is made by the employer

Various examples of Employment are as follows

aA teacher teaching in a school or collegeb An engineer employed in Municipal Corporation of DelhicAn accountant working in the accounts department of a companydA doctor working in a hospital

Note In all the above examples of employment the individual who is involved in each example is working as an employee for a salary under an employer

qualification(MBBSCALLB)d Service motive ndashProfessionals are expected to emphasis service more on their clients rather than economic gain f Code of Conduct-The activities of professionals are regulated by a code of conduct

2 What are the main features of EmploymentAnswer The main features of an employment are as followsa In employment a person works for others called employerb An employee provides personal servicec There is a service agreement or contract between the employee and the employerd The employee has to obey the order of the employere No capital investment is made by the employer

3 Give various Professions and their respective Association are given below

Professions

Professional

Professional association

Medical profession

Doctor Medical Council of India

Law profession

Lawyers Bar Council of India

Accounting Profession

Chartered

The Institute of Chartered Accounts of India( ICAI)

Engineerin Engineers The

g Profession

institute of Engineers (India)

Accounts Basic accounting terms

Today we will give you some questions from the previous study material

Questions6) Define accounting7) What do you mean by debit

and credit8) Explain the types of account9) Define the following terms

a) Assetsb) Capitalc) Purchased) Debtorse) Transactions

10) Name the types of accounts given below

a) Krishnas accountb) Machinery accountc) Royalty accountd) Salary accounte) Furniture accountf) Audit fee account

Economics Basic Economic ConceptsSub topic

UTILITY

Before starting todayrsquos class we shall recall the last class which was about UTILITY AND THE FEATURES OF UTILITY

Now we shall proceed with the further topics of the chapter

Todayrsquos topic from the chapter lsquo Basic Economic Conceptsrsquo will be TOTAL UTILITY amp MARGINAL UTILITYNow let us quickly revise the concept of utility with an example ie goods and services are designed because they have an ability to satisfy human wantsThis feature of being able to satisfy human wants is termed as utility For example we derive utility from WiFi services as it gives us satisfaction by connecting us to our friends and family through social media here consumers derive utility from WiFi services

From the above concept we shall start with todayrsquos topicEconomists have defined TOTAL UTILITY (TU) as the total satisfaction obtained by consuming a given total amount of a good and serviceFor example the total satisfaction obtained from eating 10 mangoes is the total utility of 10 mangoes

MARGINAL UTILITY (MU) is the additional satisfaction derived from each additional unit

Questions1 What is Total Utility (TU)

Answer Total Utility (TU) is the

aggregate of the utility that a consumer derives from the consumption of a certain amount of a commodityTU=MU1+MU2++MUn

2 What is Marginal UtilityAnswer

Marginal Utility (MU) is the additional made to the total utility as consumption is increased by one more unit of the commodityMU= TUn ndashTUn-1

NoteOften economists tend to

subdivide utility into an imaginary unit called UTIL

consumed In this casethe utility obtained from each mango as it is consumed as the MU of that mango It is also defined as the addition made to the total utility when an additional unit is consumed Often economists tend to subdivide utility into an imaginary unit called UTIL

Note As a consumer increases the consumption of a good over period of time the total utility or total satisfaction derived from it increases to appoint and thereafter it decreasesHowever as the consumer keeps on consuming the good the marginal utility or the additional utility derived from it decreases

SubjectBusiness studies

Topic

BUSINESSENVIRONMENT

Summary

Now quickly let us revise the earlier points that we have already done in the last class and let us proceed with the other topics that are there in the chapter

Firstly we will recall the internal and external factors of micro environment and then we shall proceed in details

Meaning and list of internal and external factors

aInternal factorsInternal factors refer to all the factors existing within a business firm The internal factors are considered controllable because the enterprise has control over these factorsFor an example a company can alter its organization structure policies programmes employees physical facilities and marketing mix to suit the changes in the environmentList of internal factors areCorporate culture mission and objectives top management organizations structure company image and brand equity company resources

b External factorsExternal factors refer to those individual and groups and agencies with which a particular business organization comes into direct and frequent contact in the course of its functioningThese individuals and groups are known as STAKEHOLDERS because they have a stake (financial interest ) in the working and performance of the particular business List of external forces (stakeholders)Customers competitors investors suppliersmiddlemen (marketing intermediaries)

Execution 1 What do you mean by internal

factors in micro environmentAnswerInternal factors refer to all the factors existing within a business firm The internal factors are considered controllable because the enterprise has control over these factorsFor an example a company can alter its organization structure policies programmes employees physical facilities and marketing mix to suit the changes in the environment

2 What do you mean by external factors in micro environment

AnswerExternal factors refer to those individual and groups and agencies with which a particular business organization comes into direct and frequent contact in the course of its functioningThese individuals and groups are known as STAKEHOLDERS because they have a stake (financial interest) in the working and performance of the particular business

3Who are stakeholdersSTAKEHOLDERS are individuals and groups who have a stake (financial interest ) in the working and performance of the particular business 4Discuss the internal factors in briefa Corporate CultureThe values beliefs and attitudes of the founders and top management of the company exercise

financers publics

customers

suppliersfinancers

competitors

middlemen

publics

Fig STAKEHOLDERS OF A COMPANY

Apart from micro environment the other main dimension of business environment isMacro environment Macro environment refers to the general environment or remote environment within which a business firm and forces in its micro environment operateA company does not directly or regularly interact with the micro environmentTherefore macro environment is also known as indirect action EnvironmentThe macro environment forces are less controllable than the micro forces

Macro environment consists of the following components

POLITICAL AND LEGAL ENVIRONMENT

ECONOMIC SOCIAL AND ENVIRONMENT

CULTURAL

ENVIRONMENT

TECHNOLOGICAL ENVIRONMENT

a strong influence on what the cmpaany stands for how it does things and what it considers importantbMission and objectivesThe business philosophy and purpose of a comoany guide it prioritiesbusiness strategiesproduct market scope and development scope

cTop management structurethe composition of board of directors the degree of professionalization of management and the organizational structure of a company have important bearing on its business decisions

dPower structureThe internal power relationship between the board of directors and the chief executive is an important factor

eCompany image and brand equityThe image and brand equity of the company play a significant role in raising finance forming alliance choosing dealers and suppliers launching new products entering foreign markets

5 What is Macro environmentAnswerMacro environment refers to the general environment or remote environment within which a business firm and forces in its micro environment operateA company does not directly or regularly interact with the micro environmentTherefore macro environment is also known as indirect action EnvironmentThe macro environment forces are less controllable than the micro forces 6 What are the components of macro environmenta Political and legal environmentb Economic environmentc Social and cultural environmentd Technological environment

BUSINESS FIRM

Fig COMPONENTS OF MACRO ENVIRONMENTPolitical science

Introduction to political science

Comparative politics and itrsquos scope Comparative politics is the second major dimension of political scienceIt is also a very vast area of study and a very large number of political scientists even treat it as an autonomous area of study within the board ambit of political scienceScope of comparative politics-

1 All political structures -Comparative politics includes the study of all structures formalnon formal governmental and extra governmental which are directly or indirectly involved in politics in all the countries of the world

2 Functional studies- Comparative politics seeks to study politics less from the point of view of the legal institutions in terms of their powers and move from the point of view of their functions which constitute the political process and their actual Operation in the environment

3 Study of political behaviour- Another important part of its scope is the study of the actual behaviour of the people in the process of politics

4 Study of similarities and differences- comparative politics also undertakesan analysis of the similarities and differences among political process and functions

5 Study of all political systems -comparative politics seeks to analyse the actual behaviour and performance of all political systems western as well as non western

6 Study of the environment and infrastructure of politics-The study of politics demands a study of the psychological sociological economic and anthropological environment in fact the social environment as a whole in which each political system operates

7 Study of political culture- political culture is composed of attitudesbeliefs emotions and values of a society that relate to the political system or politics

8 Study of political participation- Political participation is a universal processThe only difference is that while in some states it is limited in others it is wider

9 Study of political process- political

Answer the following questions-

What is comparative politics

What are the scope of comparative politics

Homework- learn

processes like decision makingpolicy making judicial process leadership recruitment process and others are always at work in all political systems

The scope of comparative politics is very comprehensive It includes everything that falls within the area of political activity and political process

History CAMBRIDGE VIEW ABOUT

THE PARTITION

AND REFUTATION

OF CAMBRIDGE

VIEW

Cambridge view about the Partition The Cambridge school of historians have interpreted that opposition to partition scheme was made entirely by the elitist groups They hold the view that Lord Curzon planned to partition the Bengal for administrative purposeREFUTATION OFCAMBRIDGE VIEW The Rationalist historians have rejected the interpretations of the Cambridge School of historians on various grounds

1 QUESTION State different views of historians regarding Partition of Bengal

ANSWER Cambridge historians believed that Lord Curzon partitioned Bengal for administrative reasons only and not for the political motive The Middle class elitist group protested because of their petty interest The Hindu zamindars protested as they have to spend more money for managing their estatesThe lawyers of Calcutta High court feared to lose their clientBut according to the nationalist Historians was-

2- The ultimate object of Lord Curzon was to crush the unity of Bengal politicians

3- If Bengal becomes a separate province Bengali speaking 16 million people of western part would become minority under Hindi speaking people of Bihar and Oriya speaking people of Orissa

4- The bureaucrats expected that the protest movement would die down quickly

5- Lord Curzon used the Muslim community in his political game

6- Idealism had great contribution in the protest against partition

7- The people of the every section of society were affected by the partition of Bengal

Computer Science

Numbers Convertion of dcimal number to octal numberThe decimal numeral system is the standard system for denoting integer and non-integer numbers It is the extension to non-integer numbers of the Hindu-Arabic numeral system For writing numbers the decimal system uses ten decimal digits a decimal mark and for negative numbers a minus sign - The decimal digits are 0 1 2 3 4 5 6 7 8 9 the decimal separator is the dot in many countries

The octal numeral system or oct for short is the base-8 number system and uses the digits 0 to 7 Octal is sometimes used in computing instead of hexadecimal perhaps most often in modern times in conjunction with file

permissions under Unix systems It has the advantage of not requiring any extra symbols as digits It is also used for digital displays

Follow these steps to convert a decimal number into octal form

1 Divide the decimal number by 82 Get the integer quotient for the next iteration (if the number will not divide equally by 8 then round down the

result to the nearest whole number)3 Keep a note of the remainder it should be between 0 and 74 Repeat the steps until the quotient is equal to 05 Write out all the remainders from bottom to top This is the solution

For example if the given decimal number is 8453

Division Quotient Remainder

8453 8 1056 5

1056 8 132 0

132 8 16 4

16 8 2 0

2 8 0 2

Then the octal solution is 20405

Subject Eng Literature (The Tempest ndash William Shakespeare) Topic Act I Scene 1 Lines 33 to 67 (End of scene) Date 16th April 2020 (4th Period)

[Students should read the original play and also the paraphrase given in the school prescribed textbook]Summary Questions amp Answers

[SUMMARY OF THE ENTIRE SCENE]

o The play starts with the scene of a severe storm at sea Alonso (King of Naples) Sebastian (Alonsorsquos brother) Ferdinand (Alonsorsquos son) Gonzalo Antonio (the usurping Duke of Milan) are in a ship in the midst of the storm

o The mariners are trying their best to control the vessel from running aground and are totally following the orders of their Master the Boatswain They have scant success

o The mariners become extremely unhappy and annoyed when most of the passengers arrive on the deck thereby hampering their effort to save the ship There is serious confrontation between them and the passengers who are part of the Kingrsquos entourage

o The mariners could not save the ship

SUMMING-UP

(i) Vivid description of the scene which gives a realistic description of terror and confusion of a tropical storm

(ii) Shows Shakespearersquos accuracy of knowledge in describing the naval operations and also matters of seamanship

(1) GONZALO Ill warrant him for drowning (L 45-57)

though the ship were no stronger than a nutshell and as leaky as an unstanched

wenchBOATSWAIN Lay her a-hold a-hold Set her two courses Off to

sea again lay her offMARINERS All lost To prayers to prayers All lostBOATSWAIN What must our mouths be coldGONZALO The king and prince at prayers Lets assist them

For our case is theirsSEBASTIAN Im out of patienceANTONIO We are merely cheated of our lives by drunkards

This wide-chopped rascal - would thou mightst lie drowning the washing of ten tides

(a) What does Antonio say at the insolent manners of the boatswain just before the given passage

Being irritated at the insolent manners of the boatswain just before the given extract Antonio the Duke of Milan calls him a worthless dog son of a woman without any morals an arrogant and disrespectful noisemaker He says that the boatswain deserved to be hanged(b) What statement does Gonzalo repeat about the boatswain

Gonzalo shows his faith that the boatswain is not destined to die by drowning He is destined to be hanged and nothing can alter this decree of destiny He says that even if the ship was as frail as a nutshell the boatswain could not be drowned for his destiny was to be hanged(c) What do the passengers do when they have lost all hope of their survival

When the passengers have lost all hope of survival they take

(iii) The opening scene justifies the title ndash The Tempest

UNANSWERED QUESTIONS

(i) The King always travels with his entire fleet including his soldiers Where were the other ships

(ii) Why was the ship in that area Where was it coming from or going where

(iii) The ship broke apart What happened to those who were in the ship

(We shall get the answer to the above questions as the play progresses)

leave of life with fervent prayers The mariners take their last hearty drink and are ready for death(d) What blame does Antonio put upon the mariners and the boatswain Antonio rebukes the mariners that these drunkards have brought them to the present crisis by neglecting their duties He blames them saying that they are going to lose their lives entirely for the negligence of the boatswain and his fellows(e) What does Antonio say while cursing the boatswain

Antonio gives vent to his wrath upon the boatswain in particular He calls the boatswain a wide-mouthed rascal who deserves to be hanged on the sea-shore at low water mark so that ten tides might wash over his body and take out of him all the liquor that he has been drinking

Class XIISubject Topic Summary ExecutionHistory Topic

1 1935 ACT AND WORKING OF PROVINCIAL AUTONOMYCONGREE AND OTHER MINISTERSSUB TOPIC GOVERNMENT OF INDIA ACT1935

Government of India Act 1935 This act established a lsquoFederation of Indiarsquo made of British Indian provinces and Indian states and provided for autonomy with a government responsible to the elected legislature in every provinceThis act introduced abolition of Diarchy at provinces The entire provincial administration was introduced to the responsible ministers who were controlled and removed by the provincial legislature The provincial autonomy means two things First The provincial governments were wholly responsible to the provincial legislature Secondly Provinces were free from outside control and interference in the large number of matters The act divided the powers between the centre and provinces in terms of three lists- Federal list( for centre) Provincial list (for province) and concurrent list (for both) Residuary powers were given to the viceroy In the election under the government of India Act the Congress swept the poll the mandate of the people came in favour of the congress so far as general Hindu seats were concerned The Congress did not get a single Muslim seates in Bombay CP UP Sind and BengalIn five provinces Congress had yhe clear majority In BengalNWFPAssam and Bombay Congress emerged as a single largest partyOn the other side the performance of the Muslim League was badThus the Congress formed ministers in 7 provinces out of 11 provinces Coalition ministry was also formed in two other provincesOnly BENGAL AND Punjab had non- congress ministries

1 QUESTION What was the main change introduced by the Government of India ActANSWER a) The Act gave more

autonomy to the provinces b) Diarchy was abolished at the

provincial levelsc) The Governor was the head of

the executived) There was a council of

ministers to advise him The ministers were responsible to the provincial legislatures who controlled them The legislature could also remove the ministers

e) The Governors still retained special reserve powers

2 QUESTION Why did the federal scheme introduced by the Government of India Act 1935 never come into operation

ANSWER The Federal structure of the Government of India was to be composed with the Governor General and Council of ministers The Federal legislature was to be Bicameral legislature- The council of states and the House of Assembly The ministers were to be chosen by the Governor general and they were to hold the office during his pleasure

The provinces of British India would have to join the federation but this was not compulsory for the princely states

This federation never materialised because of the lack of support from the required number of

princely statesThis act was refused and

rejected by the princes the Congress and the Muslim League

Thus both Congress and the League participated in the election of 1937 Thus the federal part was never introduced but the provincial part was put into operations

Bengali 2nd

Language

াচেরর পরাথCনা(কহিতা )

াচেরর পরাথCনা কহিতাটি কহি (ঙখ দেঘাচে4র দো আচো য কহিতায় াচেরর পতর হমায়ন কঠিন দেরাচেগ আxানত ার ঈশবর া আললার কাচেছ পরাথCনা কচেরচেছন তার পচেতরর ীন হিফহিরচেয় হিচেত এই কহিতায় ার পচেতরর ীন হিভbা দেচেয়চেছন ারার এমনহিক হিনচের ীন হিসCচেনর হিহিনমচেয় হিতহিন তার দেছচের ীন হিফচের দেপচেত দেচেয়চেছন তার দেছচের এই দেরাচেগর ন য হিতহিন হিনচেচেকই ায়ী কচেরচেছন তার হিনচের করা পাপচেকই হিতহিন ায়ী কচেরচেছন এছাা রানৈনহিতক ও আথCসামাহিক অসথার কথা তচে ধরা চেয়চেছ এই কহিতায় ার তার হিনচের পাপ কমCচেকই ায়ী কচেরচেছ ার অন যায় ভাচে দেপহি((হিকতর মাধ যচেম অপররা য কচেরচেছ আর এই অন যায় কাচের ন যই তার পহিরাচের হিপযCয় এচেসচেছ দে এক পরকার মানহিক নধন ইহিতাচেসর ার হিপতা চেয় সবাভাহিকভাচে ভাচোাসা দে মমতা দেথচেক মকত চেত পাচেরনহিন তাই হিপতা চেয় আললা া ভগাচেনর কাচেছ পতর হমায়চেনর পরানহিভbা দেচেয়চেছন ার আললা া ভগাচেনর কাচেছ াহিনচেয়চেছন তার হিনচের ীন হিসCন হিচেত হিতহিন রাী তার হিহিনমচেয় পচেতরর ীন হিফচের দেপচেত দেচেয়চেছন াচেরর হিপতসভ হিচেকর কথা এই কহিতায় ফটিচেয় দেতাা চেয়চেছ হিপতা পচেতরর হিরাহিরত মান নধচেনর কথা তচে ধরা চেয়চেছ

হিচে(4 হিকছ াইচেনর তাৎপযC১) ldquoদেকাথায় দেগ ওর সবচছয দেৌন দেকাথায় কচেরায় দেগাপন bয়ldquoউততর) াচেরর পতর হমায়ন কঠিন দেরাচেগ অসসথ তাই তার দেযৌন াহিরচেয় যাচেচছ এই দেরাচেগ তাচেক দেগাপচেন কচেরকচের াচেচছ তার সক (হিকত ধীচের ধীচের bয় চেচছ তাই হিপতা চেয় ার আললার কাচেছ হমায়চেনর পরান হিভbা দেচেয়চেছন২) ldquoাগাও (চেরর পরাচেনত পরানতচের ধসর (ন দেযর আান গানldquoউততর) াচেরর পতর হমায়ন কঠিন দেরাচেগ আxানত তাই ার আ দে(াচেক মমCাত (চেরর পচেথ পরানতচের আান গান ধবহিনত দোক দেসই আান গান আললার কাচেছ দেযন চে যায় আললা দেযন এই আহিতC শচেন পচেতরর ীন হিফহিরচেয় দেয় ৩)ldquoনাহিক এই (রীচেরর পাচেপর ীানচেত দেকানই তরারণ দেনই ভহি4চেতরldquoউততর) হমায়চেনর অসসথতার ন য ার হিনচেচেকই ায়ী কচেরচেছন কারন ার অচেনক রা য অন যায় ভাচে কচেরচেছ তাই তার এই পাপ কাচের ন য তার ঘচের আ হিপ এচেসচেছ এই অন যায় কাচের ন য তার মহিকত দেনই তাই ার আললার কাচেছ এই পাপ কাচেযCর ন য bমা পরাথM

Hindi 2ndlang

-ासी(जयशकर परसा-)

-ासी जयशकर परसा- की एक ऐसी कहानी ह जिजसम भारतीय ससकनित और राषटरीयता का सवरगजीतहोता ह इस कहानी म इरावती एक निहद कनया ह जिजस मलअचछो न मलतान की लट म पकडा और -ासी बना दि-या उस 500 दि-न -कर काशी क एक महाजन न खरी-ा दसरी -ासी निफरोजा ह वह गलाम ह निफरोजा को छडान क कतिलए अहम- को 1000 सोन क कतिसकक भजन थ जो अभी तक नही आए थ राजा साहब कठोर होत हए भी निफरोजा को निबना धनराकतिश क कतिलए उस म कर -त ह वनिफरोजा को अहम- को समझान की बात कहत हकहानी क अत म हम -खत ह निक इरा वती और जाटो क सर-ार बलराज का मिमलन होता हअहम- को यa म मार दि-या जाता ह वहा निफरोजा की परसननता की समामिध बनती ह वहा एक फल चढती ह और डीजल आती ह निफरोजा उस समामिध की आजीवन -ासी बनी रहती हलखक अपन उददशय अथात -ास परथा पर परकाश डालन और इस परथा क कारण होन वाल -ातो क दखो को दि-खान म पणता सफल हए ह

helliphellipContinue to next

Biology Reproductio Today we will discuss about vegetative Q1 Name some vegetative propagules

n in Organisms

propagation of plants The process of multiplication in which fragments of plant body function as propagule and develop into new individual is called vegetative propagation The units of such propagation are runner rhizome tuber bulb etc

and the speciesinvolvedVegetative propagules

Parts involved

Bulb StemBulbil BulbilRhizome Stem Runner Stem Tuber Stem Offset Stem Leaf buds Leaves Suckers Stem

Corns Stem stolon

Q2 State advantages of vegetative propagation

i) Rapid methodii) Sure and easy methodiii) Useful in plants that cannot

produce viable seeds or long seed dormancy

iv) Maintains purity of raceQ 3 Banana fruit is said to be parthenocarpic where as turkey is said to be parthenogenetic WhyBanana develops without fertilization from an unfertilized ovary thus is parthenocarpicIn turkey the ovum or female gamete developinto a new chick without fertilization thus isparthgenetic

Q4 Why is water hyacinth is called as a ldquoTerror of Bengalrdquo Water hyacinth can

propagatevegetatively all over the water body in a short per short period of time This resulted increased biochemicaloxygen oxygen demand of water body causing mortalityof fishes It is very difficult to get rid off them Thus known as terror of Bengal

Chemistry

Solid state GENERAL CHARACTERISTICS OF SOLID STATEIn nature the particular state of matter is governed by two opposing forces at given set of temperature and pressure These forces are intermolecular force of attraction and thermal energy If intermolecular force of attraction is high as compared to thermal energy particles remains in closest position

Intext QuestionsQ1 Classify the following solids as crystalline and amorphous Sodium chloride quartz glass quartz rubber polyvinyl chloride Teflon

A1 Crystalline

and hence very less movement in particles is observed In this case solid state is the preferred state of matter

Let us revise the general characteristics of solid

i) Fixed mass volume and shape

ii) Strong intermolecular force of attraction

iii) Least intermolecular space

iv) Fixed position of constituent particles

v) Incompressible and rigid

Q2 what type of interactions hold the molecules together in a polar molecular solid[CBSE 2010]A2 The molecules in a solid are held together by van der Waals forces The term van der Waals forces include hydrogen bonding dipole-dipole attraction and London dispersion forces All molecules experience London dispersion forces In addition polar molecules can also experience dipole-dipole interactions So the interactions that holds the molecule together in polar molecular solid are London dispersion force and dipole-dipole interactionsQ3 Write a feature that will distinguish a metallic solid from an ionic solid [CBSE 2010]A3 Metals are malleable and ductile whereas ionic solid are hard and brittle Metallic solid has typical metallic lustre But ionic solid looks dullQ4 Write a point of distinction between a metallic solid and an ionic solid other than metallic lustre [CBSE 2012]A4 Metals are malleable and ductile whereas ionic solid are hard and brittleQ5 Write a distinguish feature of metallic solid [CBSE 2010]A5 The force of attraction in

solid Sodium chloride Quartz Amorphous solid Quartz glass rubber polyvinyl chloride Teflon Q2 why glass is considered as super cooled liquidA2 Glass shows the tendency to flow at slower rate like liquid Hence they considered as super cooled liquidQ3 why the window glass of old buildings show milky appearance with timeA3 Glass is an amorphous solid Amorphous solid has the tendency to develop some crystalline character on heating Due to heating in day over the number of years glass acquires some crystalline character and show milky appearanceQ4 why the glass panes fixed to window or doors of old building become slightly thicker at bottomA4 Glass is super cooled liquid It has the tendency to flow down very slowly Due to this glass pane becomes thicker at the bottom over the timeQ5 Sodium chloride is a crystalline solid It shows the same value of refractive index along all the direction TrueFalse Give reasonA5 FalseCrystalline solid shows anisotropy in properties That is it shows different values for the given physical property in different direction All the crystalline solids show anisotropy in refractive index Therefore sodium chloride will show different values of refractive index on different directions

Q6 Crystalline solid are anisotropic in nature What does this statement means

between the constituent particles is special kind of electrostatic attraction That is the attraction of positively charged kernel with sea of delocalized electronsQ6 which group of solid is electrical conductor as well as malleable and ductile [CBSE 2013]A6 Metallic solidQ7 why graphite is good conductor of electricity although it is a network (covalent solid)A7 The exceptional property of graphite is due to its typical structure In graphite each carbon is covalently bonded with 3 atoms in same layer The fourth valence electron of each atom is free to move in between different layersThis free electron makes the graphite a good conductor of electricity

[CBSE 2011]A6 Anisotropy is defined asrdquo Difference in properties when measured along different axis or from different directionsrdquo Crystalline solid show different values of some of the physical properties like electrical resistance refractive index etcwhen measured along the different directions The anisotropy in crystalline solid arises due to the different arrangement of particles in different directions

Math Function Composition of functions Think of an industrial plant that produce bottles of cold drinks first there is the operation (or function) f that puts the cold drink inside the bottle followed by the opeartion g that close the bottle with the capThis leads to the following definitionDefinition Let f A rarr B and g B rarr C be two functions Then the composition of f and g denoted by gof is defined as the function gof A rarr C given by gof(x) = g(f (x)) forall x isinA

Definition A function f X rarr Y is defined to be invertible if there exists a function g Y rarr X such that gof = IX and fog = IY The function g is called the inverse of f and is denoted by f -1

Thus if f is invertible then f must be one-one and onto and conversely if f is one-one and onto then f must be invertible This fact significantly helps for proving a function f to be invertible by showing that f is one-one and onto specially when the actual inverse of f is not to be determined

Example 1 Let f 2 3 4 5 rarr 3 4 5 9 and g 3 4 5 9 rarr 7 11 15 be functions defined as f(2) = 3 f(3) = 4 f(4) = f(5) = 5 and g (3) = g (4) = 7 and g (5) = g (9) = 11 Find gofSolution We have gof(2) = g (f(2)) = g (3) = 7 gof(3) = g (f(3)) = g (4) = 7gof(4) = g (f(4)) = g (5) = 11 and gof(5) = g (5) = 11Example 2 Find gof and fog if f R rarr R and g R rarr R are given by f(x) = cos x and g (x) = 3x2 Show that gof ne fogSolution We have gof(x) = g(f(x))=g(cosx) = 3 (cos x)2

= 3 cos2 x Similarly fog(x)=f(g (x))= f(3x2)= cos (3x2) Note that 3cos2 x ne cos 3x2 for x = 0 Hence gof ne fogExample 3 Show that if f A rarr B and g B rarr C are onto then gof A rarr C is also ontoSolution Given an arbitrary element z isin C there exists a pre-image y of z under g such that g (y) = z since g is onto Further for y isin B there exists an element x in A with f(x) = y since f is onto Therefore gof(x) = g (f(x)) = g (y) = z showing that gof is onto Example 4 Let Y = n2 n isin N sub N Consider f N rarr Y as f(n) = n2 Show that

f is invertible Find the inverse of fSolution An arbitrary element y in Y is of the form n2 for some n isin N This implies that n =radicy This gives a function g Y rarr N defined by g (y) =radicy Nowgof (n) = g (n2)=radicn2 = n and fog (y) =f(radicy) = (radicy) 2 y which shows that gof=IN and fog= IY Hence f is invertible with f -1 = g

Political Science

Constitution of India-The Preamble

Summary

Objective of the state-To secure equality of status and of opportunity To promote fraternity among all the citizens To assure the dignity of the individuals and Unity and integrity of the nation

Justice-Justice stands for rule of law absence of arbitrariness and a system of equal rights freedom and opportunities for all in a society India seeks social economic and political justice to ensure equality to its citizens

Liberty-Liberty implies the absence of restraints or domination on the activities of an individual such as freedom from slavery serfdom imprisonment despotism etc The Preamble provides for the liberty of thought expression belief faith and worship

Equality-Equality means the absence of privileges or discrimination against any section of the society The Preamble provides for equality of status and opportunity to all the people of the country

Fraternity-The Preamble declares that fraternity has to assure two thingsmdashthe dignity of the individual and the unity and

Execution

Answer the following questions-

Short notes-1 Equality2 Fraternity3 Justice4 Liberty

Homework-Learn

integrity of the nation The word integrity has been added to the Preamble by the 42nd Constitutional Amendment (1976)

Business studies

Human resource management (chapter 1)

On the day of 1504 2020 I have discussed with you the managerial functions and procurement functions of HRM

Today weare going to discuss about the development function integration functions and maintenance function

Development functions-HRM improves the knowledge skills attitude and values of employees so that they the present and future jobs more effectively it includes

1) Development functions of HRM

a) Performance appraisal = It implies systematic evaluation of employees with respect to their performance on the job and their potential for development

b) Training =It is the process by which employees learn knowledge skills and attitudes to achieve organisational and personal goals

c) Executive development = It is the process of developing managerial talent through appropriate program

2) Integration functionsa) HRM reconcile the goals of

organisation with those of its members through integrating function

b) HRM tries to motivate employees to various financial and non financial incentives provided in job specification etc

3) Maintenance functiona) HRM promote and protect the

physical and mental health of employees by providing several types of benefits like housing medical aid etc

b) It Promote Social security measures to employees by providing provident fund pension gratuity maternity benefits

SubjectCOMMERCE

Topic

BUSINESSENVIRONMENT

Summary

Now quickly let us revise the earlier points that we have already done in the last class and let us proceed with the other topics that are there in the chapter

Firstly we will recall the internal and external factors of micro environment and then we

Execution 3 What do you mean by internal factors

in micro environmentAnswerInternal factors refer to all the factors existing within a business firm The internal factors are considered controllable because the enterprise has control over these factors

Development FunctionsPerformance AppraisalTrainingExecution Development

shall proceed in details

Meaning and list of internal and external factors

aInternal factorsInternal factors refer to all the factors existing within a business firm The internal factors are considered controllable because the enterprise has control over these factorsFor an example a company can alter its organization structure policies programmes employees physical facilities and marketing mix to suit the changes in the environmentList of internal factors areCorporate culture mission and objectives top management organizations structure company image and brand equity company resources

b External factorsExternal factors refer to those individual and groups and agencies with which a particular business organization comes into direct and frequent contact in the course of its functioningThese individuals and groups are known as STAKEHOLDERS because they have a stake (financial interest ) in the working and performance of the particular business List of external forces (stakeholders)Customers competitors investors suppliersmiddlemen (marketing intermediaries)financers publics

customers

suppliersfinancers

For an example a company can alter its organization structure policies programmes employees physical facilities and marketing mix to suit the changes in the environment

4 What do you mean by external factors in micro environment

AnswerExternal factors refer to those individual and groups and agencies with which a particular business organization comes into direct and frequent contact in the course of its functioningThese individuals and groups are known as STAKEHOLDERS because they have a stake (financial interest) in the working and performance of the particular business

3Who are stakeholdersSTAKEHOLDERS are individuals and groups who have a stake (financial interest ) in the working and performance of the particular business 4Discuss the internal factors in briefa Corporate CultureThe values beliefs and attitudes of the founders and top management of the company exercise a strong influence on what the cmpaany stands for how it does things and what it considers importantbMission and objectivesThe business philosophy and purpose of a comoany guide it prioritiesbusiness strategiesproduct market scope and development scope

cTop management structurethe composition of board of directors the degree of professionalization of management and the organizational structure of a company have important bearing on its business decisions

dPower structureThe internal power relationship between the board of directors and the chief executive is an important factor

e Company image and brand equityThe image and brand equity of the company play a significant role in raising finance forming alliance choosing dealers and suppliers launching new products entering foreign markets

5 What is Macro environmentAnswerMacro environment refers to the general

competitors

middlemen

publics

Fig STAKEHOLDERS OF A COMPANY

Apart from micro environment the other main dimension of business environment isMacro environment Macro environment refers to the general environment or remote environment within which a business firm and forces in its micro environment operateA company does not directly or regularly interact with the micro environmentTherefore macro environment is also known as indirect action EnvironmentThe macro environment forces are less controllable than the micro forces

Macro environment consists of the following components

POLITICAL AND LEGAL ENVIRONMENT

ECONOMIC SOCIAL AND ENVIRONMENT

CULTURAL

ENVIRONMENT

TECHNOLOGICAL ENVIRONMENT

Fig COMPONENTS OF MACRO ENVIRONMENT

environment or remote environment within which a business firm and forces in its micro environment operateA company does not directly or regularly interact with the micro environmentTherefore macro environment is also known as indirect action EnvironmentThe macro environment forces are less controllable than the micro forces 6 What are the components of macro environmenta Political and legal environmentb Economic environmentc Social and cultural environmentd Technological environment

Computer Science

Logic gates

Digital systems are said to be constructed by using logic gates These gates are the AND OR NOT NAND NOR EXOR and EXNOR

BUSINESS FIRM

gates The basic operations are described below with the aid of truth tables

AND gate

The AND gate is an electronic circuit that gives a high output (1) only if all its inputs are high A dot () is used to show the AND operation ie AB Bear in mind that this dot is sometimes omitted ie ABOR gate

The OR gate is an electronic circuit that gives a high output (1) if one or more of its inputs are high A plus (+) is used to show the OR operationNOT gate

The NOT gate is an electronic circuit that produces an inverted version of the input at its output It is also known as an inverter If the input variable is A the inverted output is known as NOT A This is also shown as A or A with a bar over the top as shown at the outputs The diagrams below show two ways that the NAND logic gate can be configured to produce a NOT gate It can also be done using NOR logic gates in the same way

NAND gate

This is a NOT-AND gate which is equal to an AND gate followed by a NOT gate The outputs of all NAND gates are high if any of the inputs are low The symbol is an AND gate with a small circle on the output The small circle represents inversion

NOR gate

This is a NOT-OR gate which is equal to an OR gate followed by a NOT gate The outputs of all NOR gates are low if any of the inputs are highThe symbol is an OR gate with a small circle on the output The small circle represents inversion

EXOR gate

The Exclusive-OR gate is a circuit which will give a high output if either but not both of its two inputs are high An encircled plus sign ( ) is used to show the EOR operation

EXNOR gate

The Exclusive-NOR gate circuit does the opposite to the EOR gate It will give a low output if either but not both of its two inputs are high The symbol is an EXOR gate with a small circle on the output The small circle represents inversion The NAND and NOR gates are called universal functions since with either one the AND and OR functions and NOT can be generated

Note A function in sum of products form can be implemented using NAND gates by replacing all AND and OR gates by NAND gates A function in product of sums form can be implemented using NOR gates by replacing all AND and OR gates by NOR gates

Logic gate symbols

Table 2 is a summary truth table of the inputoutput combinations for the NOT gate together with all possible inputoutput combinations for the other gate functions Also note that a truth table with n inputs has 2n rows You can compare the outputs of different gates

Logic gates representation using the Truth table

Example

A NAND gate can be used as a NOT gate using either of the following wiring configurations

Subject Eng Literature (The Tempest ndash William Shakespeare) Topic Act III Scene 3 Lines 53 to 110 (End of the scene) Date 16th April 2020 (2nd Period)

[Students should read the original play and also the paraphrase given in the school prescribed textbook]Summary Questions amp Answers

o Seeing this strange scene all are inclined to believe the tales told by travelers that there truly are ldquounicornsrdquo and ldquothe phoenixrsquo thronerdquo

o As they are about to sit down to the feast the banquet is snatched away by a harpy (Ariel disguised) A spiritrsquos voice (Arielrsquos voice) denounces Alonso Sebastian and Antonio with particular

1 ARIEL You are three men of sin whom Destiny

(Line 53-58)That hath to instrument this

lower world And what is int the never-surfeited sea

Hath caused to belch up you and on this island

Where man doth not inhabit you rsquomongst men

Being most unfit to live I have made you mad

reference to their crime in expelling Prospero from Milan They have not received any punishment for their deed earlier but the time for their punishment has arrived Upon Alonso it pronounces ldquolingering perdition worse than deathrdquo from which there is no remedy except through sincere repentance Ariel then vanishes in thunder and the shapes enter again and carry away the table

o Prospero watching invisibly is very pleased with the performance of Ariel and his (Prosperorsquos) ldquomeaner ministersrdquo All his enemies are now in his power and are in a fit of desperation He then leaves them and goes to see how Ferdinand and Miranda are getting on

o Alonso is now much humbled and penitent with the after effect of the spiritrsquos denunciation of his crimes He believes that his son is lost forever After this all disperse being stricken mad by the speech of the spirit

o Gonzalo fearing that they may do violence to themselves or to one another follows them and bid others to follow

(a) To whom does Ariel disguised as a harpy call the three sinners What game did Fate of Destiny play with

them

The three sinners called by Ariel are Alonso Sebastian and Antonio It was Destiny which had caused the ocean to cast the three sinners on the shore Though the ocean is all the time devouring whatever appears on its surface and is never satisfied with its continual swallowing of the ships and men in the present case the ocean had cast these three sinners on the shore without killing them

(b) Who had jointly been responsible for the conspiracy against Prospero What is Prosperorsquos purpose behind all this

Three men Alonso Sebastian and Antonio had jointly

been responsible for the conspiracy against Prospero They had driven out Prospero form Milan Prosperorsquos purpose is to make these three sinners realize the wrong they had done He wants them to repent for their criminal deeds because repentance leads to self-esteem(c )What does Ariel (the harpy) tell Alonso and his companions when they take out their swords to attack him

Seeing them drawing their swords Ariel (harpy) tells them that he and his companions are the instruments of destiny and that it is not possible for human beings to do them any injury He says that the swords of human beings can not injure even a minute part of his feathers Their swords are as ineffective against him and his companions as against the wind or the water

(d) Give the explanatory meanings of the following expressions in the context of the above extract

(i)Never surfeited (ii) Belch up (iii) lsquomongst men

(i) Never surfeited never led to satisfaction

(ii) Belch up cast ashore(iii) lsquomongst men in human

society2

I and my fellows (Line 60-65)

Are ministers of Fate The elementsOf whom your swords are tempered may as wellWound the loud winds or with bemocked-at stabsKill the still-closing waters as diminishOne dowl thats in my plume

IMPORTANT PASSAGES EXPLAINED

The elements

(Line 61-66)Of whom your swords are tempered may

as wellWound the loud winds or with

bemocked-at stabs

(a) Who is lsquoIrsquo Who are his lsquofellowsrdquo

lsquoIrsquo is referred to Ariel in disguise of a harpy His lsquofellowsrsquo are other spirits serving Prospero the real Duke of Milan who has acquired supernatural powers after being banished from his Dukedom Prospero has settled in this uninhabited island

(b) What are the elements that have temperrsquod the swords Why will it not work against the speaker

The swords (of Alonso and his companions) are tempered by metal (steel) which is taken out of the earth and refined by

Kill the still-closing waters as diminishOne dowl thats in my plume My fellow

ministersAre like invulnerable

In these words Ariel reminds the King and his companions of the utter futility of drawing swords against himself and his fellows Ariel drives Alonso Antonio and Sebastian the three men of sin to desperation ndash a state in which men do violence to themselves They draw swords to strike Ariel But Ariel reminds them that he and the other spirits are the ministers of destiny and nothing can wound them The steel of which their swords are made of may cut the wind or water which being divided always closes up again Even supposing that such things may be possible it is quite impossible that their swords will cut one feather in their plume They are incapable of being wounded by any sword of man Hence it is foolish on their part to attempt to strike at Ariel and his fellow-spirits

For which foul deed

(Line 72-75)The powers delaying not forgetting

haveIncensed the seas and shores yea all the

creatures Against your peace

Ariel enters like a harpy and remaining invisible tells Alonso Sebastian and Antonio that he and other harpies are the agents of Destiny appointed to carry out her decrees He tells them that their punishment for the crime against Prospero which has been so long deferred is now to fall upon them He reminds them that they had expelled Prospero from Milan and set him and his innocent child adrift on the sea and that the sea had paid them back for their sin by the shipwreck and by the calamities they have suffered He tells them that the powers above which did not forget this mean treachery but only deferred the punishment have now engaged the seas and the shores and all living beings including him and his comrades against them The very elements and supernatural agency Ariel adds have taken up the avenging of their crime against Prospero

the action of fire It may cut the wind or water which being divided always closes up again

The sword will not work against the spirits and the harpy because they are the ministers of destiny and nothing can wound them nor it will cut a single feather in their plume

(c )What is the meaning of lsquodowlrsquo in the last line

The term lsquodowlrsquo means a filament or the smallest part of a feather In this context Ariel in disguise of harpy says that their sword cannot even damage the smallest filament of their (Arielrsquos and other spirits) feathers as they are incapable of being wounded by any sword of man

(d) What does the speaker remind the listeners about

Ariel in disguise of harpy reminds Alonso the King of Naples Sebastian Alonsorsquos brother and Antonio the present Duke of Milan and the treacherous brother of Prospero as they being three men of sin He even reminds them that their punishment for their crime against Prospero which has been so long deferred now falls upon them He reminds them that they have expelled Prospero from Milan and has set him along with his innocent infant daughter adrift on the sea So the sea has paid them back for their sin by their shipwreck and the calamities they have suffered since then The harpy rebukes Alonso of his sin that has incensed the Gods and has deprived him of his son as a punishment

(e) How do they respond

When Ariel in disguise of a harpy reminds Alonso Sebastian and Antonio of their past misdeeds and sin Alonso has a look of terror and confusion in his eyes He utters the words of sincere repentance wrung out of his conscience-stricken heart It appears to him that all the elements of nature the sea-waves the wind and the thunder proclaiming a loud voice in the name of Prospero and the crime Alonso has committed against him They are calling upon him to repent There is a deep storm raging in Alonsorsquos breast and the echoes of that storm are ringing in his ears like a clear note of wind-instrument A note of denunciation of Alonsorsquos crime leaves him much humbled and penitent and confirms his belief that his son is lost forever But Sebastian and Antonio shows some courage instead of repentance They wish to kill the spirits or devils if it appears

3

Of my instruction hast thou nothing bated (Line 85-93)

In what thou hast to say So with good life

And observation strange my meaner ministers

Their several kinds have done My high charms work

And these mine enemies are all knit upIn their distractions They now are in my

powerAnd in these fits I leave them while I visitYoung Ferdinand whom they suppose is

drownedAnd his and mine loved darling

Methought the billows spoke and (Line 96-99)

told me of itThe winds did sing it to me and the

thunderThat deep and dreadful organ-pipe

pronouncedThe name of Prosper It did bass my

trespass

These are the words of contrition coming from Alonso Ariel has driven him to a deep repentance for conspiring with Antonio against Prospero He now feels a sincere remorse It appears to him that all the elements of nature the sea-waves the wind and the thunder proclaimed with a loud voice the name of Prospero and the crime Alonso had committed against him They are calling upon him to repent There is a deep storm raging in Alonsorsquos breast and the echoes of that storm are ringing in his ears like the clear note of a wind-instrument

Comment These are the words of sincere repentance wrung out of the conscience-stricken heart of Alonso Alonso who is the lesser villain is the first to give way to remorse under the effect of Arielrsquos speech The words of Ariel seem to him to be the voice of conscience speaking to him He is driven to desperation a state in which he might do violence to his life

(a) Identify the speaker State the context

Prospero the ruler of the island is the speaker The famous banquet scene has been enacted very well Ariel and his junior spirits have played their roles excellently Prospero is glad to say words of praise for them(b) In what way the speakerrsquos instructions have been carried out

According to Prosperorsquos instructions a banquet was presented before the King of Naples and his companions when they were tired and hungry Just when they were preparing to eat the feast the banquet was suddenly removed by exercising supernatural powers All this was done by Ariel Prosperorsquos chief assistant and a powerful spirit

Ariel not only made the feast disappear but also delivered his speech blaming the King and his two companions for their past wicked deeds He warned them to repent for their misdeeds or suffer forever on that uninhabited island

(c) Who are referred to as lsquomeaner ministersrsquo What have they done

Prospero refers as lsquomeaner ministersrsquo to his other lesser spirits who were assisting Ariel in presenting a scene before the kingrsquos party They entered the scene to the accompaniment of music They assumed several strange shapes and brought in a banquet Then they danced about it with gentle actions of salutations thus inviting the King and others to eat the feast

These spirits play their role again when Ariel in the shape of a harpy quits the scene These shapes enter again and dancing with mocking gestures carry away the table

(d) Who are the speakerrsquos enemies What has happened to them

King of Naples Alonso his brother Sebastian and the present Duke of Milan Antonio (Prosperorsquos own brother) are Prosperorsquos enemies With the turn of events they have all been washed ashore on the island which is ruled by Prospero the great magician Actually this happened after the shipwreck caused by a storm which was raised by Prospero with the purpose of bringing these people to his island Prosperorsquos spirits have already confused and terrified these enemies and they are under Prosperorsquos control He can treat them as he likes

(e) What does he say about Ferdinand Explain what is meant by ldquohellip his and mine darlingrdquo

Prospero knows that Alonsorsquos son prince Ferdinand is alive though his father thinks that the prince has been drowned

Prospero refers to his daughter Miranda who is dear to him She is also very dear to Prince Ferdinand who has fallen in love with her They are waiting to be married soon for which they have received Prosperorsquos consent

4

ALONSO O it is monstrous monstrous (Line 95-102)

Methought the billows spoke and told me of it

The winds did sing it to me and the thunderThat deep and dreadful organ-

pipe pronouncedThe name of Prosper It did bass

my trespassTherefore my son ithrsquo ooze is

bedded andIll seek him deeper than eer

plummet soundedAnd with him there lie mudded

(a) In what way does Alonso express his horror when his conscience is awakened by Arielrsquos words

When Alonsorsquos conscience is awakened by Arielrsquos words he expresses his horror at what he has heard He gets the feeling that the waves of the ocean the wind and the loud thunder have spoken to him and uttered the name of Prospero Because of being reminded of his crime in a very loud and rough voice he comes to realize that he has lost his son for his past misdeeds

(b) What does Alonso imagine about his son What does Alonso want to do in his desperate state

Alonso imagines that his son is lying in the mud at the bottom of the sea He feels desperate that he wants to drown himself in the ocean deeper than the plumb-line has ever gone He wants to lie with his son at the bottom of the sea

(c) How do Sebastian and Antonio want to face the evil spirits

Sebastian says that he is not at all afraid of what the harpy has said and that he is prepared to fight any number of such monsters if they appear before him only one at a time Antonio says that he would support Sebastian in the fight against the fiendsyyy

(d) Why does Gonzalo ask Adrian to follow the three men

Gonzalo tells Adrian that all the three men namely Alonso Sebastian and Antonio are in a wild and reckless mood The thought of the heinous crime of which they are guilty has begun to torment their minds So he asks Adrian to follow those three men without loss of time and prevent them from doing anything which the turmoil in their minds might lead them to do

(e) What opinion do you form of Alonso from the above extract

Alonso who is the lesser villain is the first to give way to remorse under the effect of Arielrsquos speech The words of Ariel seem to him to be the voice of conscience speaking to him He is driven to desperation a state in which he might do violence to his life

Subject =Accounts

Ac-12 15420 topic-pL Appropriation ac

PROFIT AND LOSS APPROPRIATION ACCOUNT

MEANING AND PREPARATIONProfit and Loss Appropriation Account is merely an extension of the Profit and Loss Account of the firm The profit of the firm has to be distributed amongst the partners in their respective profit sharing ratio But before its distribution it needs to be adjusted All Adjustments like partnerrsquos salary partnerrsquos commission interest on capital interest on drawings etc are made in this account These adjustments will reduce the amount of profit for distribution This adjusted profit will be distributed amongst the partners in their profit sharing ratio To prepare it at first the balance of Profit and Loss Account is transferred to this account The journal entries for the preparation of Profit and Loss Appropriation Account are given below

1 for transfer of the balance of Profit and Loss Account to Profit and Loss Appropriation Account

(a) In case of Net Profit

Profit and Loss Ac helliphelliphelliphelliphellipDrTo Profit and Loss Appropriation Ac(Net Profit transferred to Profit and Loss Appropriation Ac)

(b)In case of Net Loss

Profit and Loss Appropriation Achelliphelliphellip DrTo Profit and Loss Ac(Net Loss transferred to Profit and Loss Appropriation Ac)

2 for Interest on Capital

For transferring on Interest on CapitalProfit and Loss Appropriation Achelliphelliphellip DrTo Interest on Capital Ac(Interest on capital transferred to Profit amp Loss Appropriation Ac)

3 for Interest on Drawings

For transferring Interest on Drawings Interest on Drawings Achelliphelliphelliphelliphelliphellip DrTo Profit and Loss Appropriation Ac(Interest on drawing transferred to Profit amp Loss Appropriation Ac)

4 For Partnerrsquos SalaryFor transfer of partnerrsquos SalaryProfit and Loss Appropriation Achelliphellip DrTo Salary Ac(Salary transferred to profit amp Loss Appropriation Ac)

5 For Partnerrsquos CommissionFor transferring commissionProfit and Loss Appropriation Achelliphelliphellip DrTo Commission Ac(Commission transferred to Profit and Loss Appropriation Ac)

6 For Transfer of agreed amount to General ReserveProfit and Loss Appropriation Ac helliphellipDrTo General Reserve Ac(Transfer to General Reserve)

7 for share of Profit or Loss appropriation(a) If ProfitProfit and Loss Appropriation Achelliphellip DrTo Partnerrsquos CapitalCurrent Ac(Profit transferred to capitalcurrent Ac)(b) If LossPartnerrsquos Capital Current Achelliphelliphelliphellip DrTo Profit and Loss Appropriation Ac(Loss transferred to capitalcurrent Ac)

THE FORMAT OF PROFIT AND LOSS APPROPRIATION

Profit and Loss Appropriation Account for the year endedhelliphelliphelliphellip

Particulars Amount Particulars Amount

To PL Ac (loss) By pL Ac (profit)

To Interest on capital BY Interest on drawings

To partner`s commission by Partner`s capital Ac ( loss)

To Partner`s salary To Interest on partner`s loan To General Reserve To Partner`s Capital AC (Profit)

Subject= Economics

MOVEMENT ALONG THE DEMAND CURVE (CHANGE IN QUANTITY DEMANDED)In law of demand you have already studied the inverse relationship between price and quantity demanded When quantity demanded of a commodity changes due to change in its price keeping other factors constant it is called change in quantity demanded It is graphically expressed as a movement along the same demand curve There can be either a downward movement or an upward movement along the same demand curve Upward movement along the same demand curve is called contraction of demand or decrease in quantity demanded and downward movement along the same demand curve is known as expansion of demand or increase in quantity demanded

Extention of demandd

price (rs)p A

B Extentionp1 d

Q Q1

Quantity demanded ( in units)

Contraction of demandd

p2 Ccontraction

p APrice (Rs)

d

Q2 Q

Quantity demanded (in units)

Explanation of movement of demand A fall in price from OP to OP1 leads to increase in quantity demanded from OQ to OQ1 (expansion of demand) resulting in a downward movement from point A to point B along the same demand curve DD When Price rises from OP to OP2 quantity demanded falls from OQ to OQ2 (contraction of demand) leading to an upward movement from point A to point C along the same demand curve DD

  • Activity Series of Metals
    • Drawbacks of Rutherfordrsquos model of atom
      • Electromagnetic radiations
      • Properties of electromagnetic radiations
      • Characteristics of electromagnetic radiations
        • Plancks Quantum Theory-
        • Photoelectric effect
          • Intext Questions
            • Logic gates
            • Digital systems are said to be constructed by using logic gates These gates are the AND OR NOT NAND NOR EXOR and EXNOR gates The basic operations are described below with the aid of truth tables
            • AND gate
            • Example
Page 28:  · Web viewSubject. Topic. Summary. Execution. English 1 . Chapter 1 naming words . Page 8. Write the names of these pictures:- Person:-1. father. 2.Firefighter 3.doctor 4 ...

পরচে4র যাহিত রbা করা সমভ নয় তাই হিতহিন পতরচেক হিনচেয় ককাতায় সাস শর কচেরন গলপ কথচেকর আহিথCক অসথা নয়ন দোচের হিমাচের দেথচেক সমপরণC আাা কথচেকর হিপতা হিনচের দেষটায় অথC উপাCন করচেতন া উপাহিধ াচেভর নয তার াসা হিছনা আর দেসই কারচেরণ কথক তার একমাতর উততরাহিধকার চেয় তার হিপতার পরহিত কতজঞ কথক দো পা হি(চেচেছন হিনচের পরারণ ও মান রbার নয উপচেযাগী অথC হিনা দেষটায় পরাপত চেয়চেছন- এটাই তার কাচেছ পরম দেগৌরচের হি4য় চে মচেন কচেরন কাররণ (নয ভাণডাচের গৈপতক ায়ানার উজজব ইহিতাস অচেপbা দোার হিসeচেকর মচেধয গৈপতক দেকামপাহিনর কাগ তার কাচেছ অচেনক দেহি( মযান

TO BE CONTINUED

উ- ক) আচোয অং(টি রীনদরনাথ ঠাকচেরর দো ঠাকরা গচেলপর অং() কতা চেন আচোয গচেলপর গলপ কথকইার চেত নয়ন দোচের হিমাহিরর দে(4 ং(ধর গৈকাস ার কথা া চেয়চেছ গৈকাস া নয়ন দোচের সমসত সমপহিতত ঋচেরণর াচেয় হিহিx কচের অহি(ষট যা আচেছ তাচেত হিপত পরচে4র যাহিত রbা করা সমভ নয় তাই হিতহিন পতরচেক হিনচেয় ককাতায় সাস শর কচেরনগ) গৈকাস ার হিপতার মতযর পর নয়ন দোচের হিমাহিরর অহিসততব হিপত য় কচেয়কটা উৎস ও শরাদধ- (াহিনতচেত হিমাহিরর দে(4 কহিটক যয় চেয় হিগচেয় এচেক াচের দে(4 চেয় যায় তন তাচের গC করার মত আর হিকছই হিছ না-দেসই পরসচেঙগ এই উহিকত নয়নচোচের হিমাচেররা া নাচেমই হিযাত হিছচেন ায়ানার উাররণ সবরপ নয়নচোচের ারা হিা (াচেকর হিা হিচেতন এছাাও দেকান উৎস উপচেb রাহিতরচেক হিন করচেত হিগচেয় তারা সযC হিকরচেরণর নয পরীপ জবাহিচেয় তাচেত রপার হির 4Cরণ করচেতন তাই দেসকাচের ায়ানা দেহি(হিন সথায়ী চেত পারত না হিহিভনন উৎস শরাদধ- (াহিনতচেত সাধযা হিতহিরকত র করার নয হিমাহির হিহিকচেয় দেযত হ হিতC কা হিহি(ষট পরীচেপর দেত দেযমন অলপকাচের মচেধয হিনঃচে(4 চেয় যায়-নয়নচোচের হিমারচের অসথা তাই চেয়হিছ এই কারচেরণই কথক নয়নচোচের হিমারচের গা ভরা আমবর সয করচেত পারতনা

Physics Dimensional Analysis (Summary)

Q Find the dimensions of consts ab in relation

p=(bminusxlowastx)at

where p is the power x is the distance and t is time

Ans From principle of homogeneity dimension of b x2 are same Dim of b = dim of x2 = [L2] = [ML2T0]Dim of a = dim of ( b- x2)dim of (pt) = [M0L2T0][ML2T-2] [T-1] [T] = [M-1L0T2]

Chemistry Atomic Structure Drawbacks of Rutherfordrsquos model of

atom a According to Rutherfordrsquos model of atom electrons which are negativelycharged particles revolve around the nucleus in fixed orbits Thusb theelectrons undergo acceleration According to electromagnetic theory of Maxwell a charged particle undergoing acceleration should emitelectromagnetic radiation Thus an electron in an orbit should emitradiation Thus the orbit should shrink But this does not happenc The model does not give any information about how electrons aredistributed around nucleus and what are energies of these electrons Isotopes These are the atoms of the same

Properties of electromagnetic radiationsa Oscillating electric and magnetic field are produced by oscillating charged particles These fields are perpendicular to each other and both areperpendicular to the direction of propagation of the waveb They do not need a medium to travel That means they can even travel invacuum

Characteristics of electromagnetic radiationsa Wavelength It may be defined as the distance between two neighbouring crests or troughs of

element having the same atomicnumber but different mass numbere g 1H11H21H3

Isobars Isobars are the atoms of different elements having the same massnumber but different atomic numbere g 18Ar40 20Ca40

Isoelectronic species These are those species which have the same numberof electrons

Electromagnetic radiationsThe radiations which are associated withelectrical and magnetic fields are called electromagnetic radiations When anelectrically charged particle moves under acceleration alternating electricaland magnetic fields are produced and transmitted These fields aretransmitted in the form of waves These waves are called electromagneticwaves or electromagnetic radiations

wave as shown It is denoted by λb Frequency (ν) It may be defined as the number of waves which passthrough a particular point in one secondc Velocity (v) It is defined as the distance travelled by a wave in onesecond In vacuum all types of electromagnetic radiations travel with thesame velocity Its value is 3 times10 8m sec-1 It is denoted by v

d Wave number Wave number is defined as the number of wavelengths per unit lengthVelocity = frequency timeswavelength c = νλ

Plancks Quantum Theory- o The radiant energy is emitted or absorbed not continuously but discontinuously in the form of small discrete packets of energy called lsquoquantumrsquo In case of light the quantum of energy is called a lsquophotonrsquoo The energy of each quantum is directly proportional to the frequency of the radiation ie E α υ or E= hυ where h= Planckrsquos constant = 6626 x 10-27 Js o Energy is always emitted or absorbed as integral multiple of this uantum E=nhυ Where n=1234Black body An ideal body which emits and absorbs all frequencies is calleda black body The radiation emitted by such a body is called black body radiation

Photoelectric effectThe phenomenon of ejection of electrons from thesurface of metal when light of suitable frequency strikes it is calledphotoelectric effect The ejected electrons are called photoelectrons

Biology Chapter - 02Systematics and Five Kingdoms

Scientists divide the whole living organisms into two kingdom first and ultimately by five kingdom at last

In the earlier systems of classifications organisms are divided into kingdom plantaeand kingdom animalia on the of presenceof cell wall their modes of nutrition and movements

Some problem arise like fungi share manycharacteristic withplant despite their heterotrophic nutrition bacteria protozoa areunicellular present in both kingdom Toovercome this third kingdom Protista isintroduced which include

unicellularorganisms But there is also another

problem Allunicellular organisms are not similar kind The cellular structure of prokaryotes is verydifferent from that of other organismsEukaryotes possess a true nucleus and allcell organelles that are not present inprokaryotes So the fourth kingdom Monerais introduced which include unicellular prokaryotes (bacteriaamp blue green algae)

bull Still some problem arise in kingdomplantae

So in 1969 R H Whittakar proposedanew five kingdom System of classification

i) Kingdom Monera - unicellular prokaryotes

ii) kingdom Protista - unicellular eukaryotes

iii) Kingdom Fungi - uni or multicellular fungi with cell wall but without chlorophyll

iv) Kingdom Plantae - Multicellular Plants

v) Kingdom Animalia - Multicellular Animals

EVS Chapter 1 ndash Modes of Existence

An agricultural society

An agricultural society also known as an agrarian society is a society that constructs social order around a reliance upon farming More than half the people living in that society make their living by farming

People in an agricultural society generally lead a more settled lifestyle than those in nomadic hunter-gatherer or semi-nomadic pastoral societies because they live permanently near the land that is farmed Agricultural settlements tend to develop in areas of convenience near bodies of water which is used for both crops and transportation or along trade routes Not everyone in an agricultural society is a farmer Some people make a living trading or making and selling goods such as tools used for farming

Another way to define an agrarian society is to see the total amount of production in a nation In an agrarian society cultivating the land is the main source of wealth Such a society can recognize other means of subsistence and work habits but emphasizes the importance of agriculture and livestock Agrarian societies have existed in various parts of the world for 10000 years and continue to exist today They have been the most common form of socio-economic organization for most of recorded human history

Q) Write the features of agricultural society

Ans - Structure and Features of Agrarian Society1 Occupational Structure

An agrarian society is generally associated with the domestication of plants and animals The domestication of plants means farming and that of animals means herding Often there is mixture of farming and the use of such domesticated animals as cow goat and sheep

2 Forms of Land Ownership in Agrarian SocietiesGenerally there are landlords supervisory farmers cultivators and share croppers The landholders own the land but do not work on it They let it out for sharecropping The supervisory farmers are those who live by having their land cultivated by hired labourers The cultivators cultivate the land for themselvesThe share-croppers are those who live by tilling other peoplersquos land or a crop-sharing basis The artisans own their means of production and produce by their own labour in their homesteads

3 Village Community System An agrarian society is highlighted by

the institution of village community system The agrarian economy made fixed dwelling houses necessary Living close together for protection and co-operation and living nearer to the land gave birth to agricultural villages The village is not only the residential place of farmers it is also the social integrator

4 Minimal Division of Labour Another structural feature of agrarian society is a minimal division of labour Except for the basic division founded on age and sex differences there are few specialized roles There is only one predominant type of occupation ie domestication of plants and animals For all the people the environment physical as well as social is the same

5 Role of Family The farm family is of the patriarchal type the father is the final arbiter in most of the familyrsquos major decisions The life of ail men and women is merged in family life Since there are not many special organizations family is the only organisation to perform the tasks of aid and protection

6 Sense of Unity The members of an agrarian society exhibit a strong in-group feeling Since the whole of their social lives is wrapped up in a society which is physically economically and socially homogenous they are inclined to view the entire outside world as an out group

7 Informal Social Control An agrarian society is regionally divided into villages In a village community the force of traditional mores is more dominant than in the urban community In the village everybody is known to everybody The members in a village community help each other and share the joy and sorrows of each other Crime in an agrarian society is rare

8 Simplicity and Uniformity Life of the people in an agrarian society is marked by simplicity and uniformity Their main occupation is agriculture which largely depends upon the vagaries of nature An agrarian society is a religious society

Math Compound angles Compound angles The algebraic sum of two or more angles is called a compound angle If A B C be three angles then A+B B+C C+A A-B B-C A-C A+B-C etc are compound angles In this chapter we shall discuss the trigonometrical ratios of compound angles Theorem 1 If A B and A+B are all pisitive acute angles theni) sin( A+B) = sin A cos B + cosA sinBii) cos(A+B) = cosA cosB- sinA sinBTheorem 2If A and B are positive acute angles and AgtB theni) sin(A-B) = sin A cosB- cos A sinBii) cos(A-B) = cos A cos B+ sin A sin BTo prove that i) sin(A+B) sin (A-B) = sin2 A - sin2 B = cos2 B- cos2 A

Example 1 Prove that tan70deg=2tan50deg+tan20degSolutiontan70deg = tan(50deg + 20deg)Or tan70deg=(tan 50deg+tan 20deg)(1-tan50degtan20deg) or tan70deg (1 ndash tan 50deg tan20deg) = tan50deg+tan20degor tan70deg= tan70deg tan50deg tan20deg+ tan50deg + tan20deg = cot20deg tan50deg tan20deg + tan50deg + tan20deg = 2 tan50deg+ tan20degExample 2 If A + B = 45deg show that (1 + tanA) (1 + tanB) = 2Solutiontan(A + B) =( tan A + tan B) (1 - tan

ii) cos(A+B) Cos(A-B) = cos2 A- sin2 B = cos2 B -sin2 AProof i) LHS= sin(A+B)sin(AminusB) [Recall sin(αminusβ)=sinαcosβminuscosαsinβ And sin(α+β)=sinαcosβ+cosαsinβ]= (sinAcosB+cosAsinB)times(sinAcosBminuscosAsinB)= sin2Acos2Bminuscos2Asin2B [Recall sin2α+cos2α=1 From above we can then assume correctly that sin2α=1minuscos2α AND cos2α=1minussin2α] = sin2A(1minussin2B)minussin2B(1minussin2A) = sin2Aminussin2Asin2Bminussin2B+sin2Asin2B = sin2Aminussin2B= 1-cos2A-(1-cos2B) = cos2 B- cos2 A = RHSii)LHS= cos (A+B) cos(A-B) [ cos(A+B) = cos AcosB- sinAsinBCos(A-B) = cosAcosB+ sinAsinB]= cos2 A Cos2 B- sin2 A Sin2 B= cos2 A( 1-sin2 B) - (1- cos2 A) sin2 B= cos2 A- cos2 A sin2 B- sin2 B+ cos2 A sin2 B=cos2 A- sin2 B=1- sin2 A-(1-cos2 B) = cos2 B- sin2 A= RHSTangent formulae for compound anglesi)tan (A + B) = tan A + tan B1-tan A tan Bii)tan (A ndash B) = tan A-tan B1+tan A tan Biii) cot (A + B) = cot Acot B-1cot A+cot B(viii) cot (A ndash B) = cot Acot B+1cot B-cot A

A tan B) Or 1= (tan A+ tanB) (1-tan A tanB) Or tanA + tanB + tanA tanB + 1 = 1 + 1Or tanA (1 + tanB) + (1 + tanB) = 2Or (1 + tanA) (1 + tanB) = 2Example 3 Find the value of sin 15degSolution sin 15deg= sin(45deg-30deg) = sin45degcos 30deg- cos45degsin30deg =(1radic2) (radic32) -(1radic2) (12) = (radic3-1) 2radic2Example 4 If sin A = 1 radic10 and sin B = 1 radic5 where A and B are positive acute angles then what is A + B SolutionWe know that sin (A + B) = sin A cos B + cos A sin B= [1 radic10] [radic(1 minus 1 5)] + [1 radic5] radic(1 minus 1 10)= [1 radic10] [radic4 5] + [1 radic5] [radic9 10]= [1 radic50] times (2 + 3)= 5 radic50 = 1 radic2

sin (A + B) = sin π 4rArrHence A + B = π 4Example 5 If A + B = 225o then find [cot A] [1 + cotA] times [cot B] [1 + cot B]Solution[cot A] [1 + cotA] times [cot B] [1 + cot B] = 1 [(1 + tan A) times (1 + tan B)]=1 [tan A + tan B + 1 + tan A tan B] [ tan (A + B) = tan225o]∵

tan A + tan B = 1minus tan A tan BrArr= 1 [1 minus tan A tan B + 1 + tan A tan B]= 1 2

COMMERCE

CLASSIFICTION OF HUMAN ACTIVITIES-ECONOMIC AND NON-ECONOMIC

Firstly we shall recall the previous class for 5 mins especially for the absentees and for also the rest of the students who were there

Today at first we briefly discuss the earlier portions of the chapter

1Business-It includes all those economic activities which are concerned with production and exchange of goods and services with the object of earning profit Example A factory shop beauty parlour also business enterprises

2Profession ndashThe term profession means an occupation which involves application of specialized knowledge and skills to earn a living For Example Chartered Accountancy medicine law tax consultancy are example of professions

Questions1What are the main features of ProfessionAnswer The main features of a profession are as follows a Specialised body of knowledge-Every profession has a specialised and systematised body of knowledge b Restricted entry- Entry to a profession is allowed only to those who have completed the prescribed education and have the specialised examination c Formal education and training ndashA formal education and training is given to the person who wants to acquire the professional

3Employment-Employment mean an economic activity where people work for others in exchange for some remuneration (salary)The persons who work for others are called lsquoemployeesrsquo The persons or organizations which engage others to work for them are called lsquoemployersrsquoEg A doctor working in a hospital is employment as he is working for a salaryA lawyer may serve as a law officer in a bank

With this we shall proceed with the features of both Profession amp Employment

The main features of a profession are as follow

a Specialised body of knowledge b Restricted entry c Formal education and training d Professional association e Service motive f Code of contact

The main features of an employment are as follows

a In employment a person works for others called employer

b An employee provides personal service

c There is a service agreement or contract between the employee and the employer

d The employee has to obey the order of the employer

e No capital investment is made by the employer

Various examples of Employment are as follows

aA teacher teaching in a school or collegeb An engineer employed in Municipal Corporation of DelhicAn accountant working in the accounts department of a companydA doctor working in a hospital

Note In all the above examples of employment the individual who is involved in each example is working as an employee for a salary under an employer

qualification(MBBSCALLB)d Service motive ndashProfessionals are expected to emphasis service more on their clients rather than economic gain f Code of Conduct-The activities of professionals are regulated by a code of conduct

2 What are the main features of EmploymentAnswer The main features of an employment are as followsa In employment a person works for others called employerb An employee provides personal servicec There is a service agreement or contract between the employee and the employerd The employee has to obey the order of the employere No capital investment is made by the employer

3 Give various Professions and their respective Association are given below

Professions

Professional

Professional association

Medical profession

Doctor Medical Council of India

Law profession

Lawyers Bar Council of India

Accounting Profession

Chartered

The Institute of Chartered Accounts of India( ICAI)

Engineerin Engineers The

g Profession

institute of Engineers (India)

Accounts Basic accounting terms

Today we will give you some questions from the previous study material

Questions6) Define accounting7) What do you mean by debit

and credit8) Explain the types of account9) Define the following terms

a) Assetsb) Capitalc) Purchased) Debtorse) Transactions

10) Name the types of accounts given below

a) Krishnas accountb) Machinery accountc) Royalty accountd) Salary accounte) Furniture accountf) Audit fee account

Economics Basic Economic ConceptsSub topic

UTILITY

Before starting todayrsquos class we shall recall the last class which was about UTILITY AND THE FEATURES OF UTILITY

Now we shall proceed with the further topics of the chapter

Todayrsquos topic from the chapter lsquo Basic Economic Conceptsrsquo will be TOTAL UTILITY amp MARGINAL UTILITYNow let us quickly revise the concept of utility with an example ie goods and services are designed because they have an ability to satisfy human wantsThis feature of being able to satisfy human wants is termed as utility For example we derive utility from WiFi services as it gives us satisfaction by connecting us to our friends and family through social media here consumers derive utility from WiFi services

From the above concept we shall start with todayrsquos topicEconomists have defined TOTAL UTILITY (TU) as the total satisfaction obtained by consuming a given total amount of a good and serviceFor example the total satisfaction obtained from eating 10 mangoes is the total utility of 10 mangoes

MARGINAL UTILITY (MU) is the additional satisfaction derived from each additional unit

Questions1 What is Total Utility (TU)

Answer Total Utility (TU) is the

aggregate of the utility that a consumer derives from the consumption of a certain amount of a commodityTU=MU1+MU2++MUn

2 What is Marginal UtilityAnswer

Marginal Utility (MU) is the additional made to the total utility as consumption is increased by one more unit of the commodityMU= TUn ndashTUn-1

NoteOften economists tend to

subdivide utility into an imaginary unit called UTIL

consumed In this casethe utility obtained from each mango as it is consumed as the MU of that mango It is also defined as the addition made to the total utility when an additional unit is consumed Often economists tend to subdivide utility into an imaginary unit called UTIL

Note As a consumer increases the consumption of a good over period of time the total utility or total satisfaction derived from it increases to appoint and thereafter it decreasesHowever as the consumer keeps on consuming the good the marginal utility or the additional utility derived from it decreases

SubjectBusiness studies

Topic

BUSINESSENVIRONMENT

Summary

Now quickly let us revise the earlier points that we have already done in the last class and let us proceed with the other topics that are there in the chapter

Firstly we will recall the internal and external factors of micro environment and then we shall proceed in details

Meaning and list of internal and external factors

aInternal factorsInternal factors refer to all the factors existing within a business firm The internal factors are considered controllable because the enterprise has control over these factorsFor an example a company can alter its organization structure policies programmes employees physical facilities and marketing mix to suit the changes in the environmentList of internal factors areCorporate culture mission and objectives top management organizations structure company image and brand equity company resources

b External factorsExternal factors refer to those individual and groups and agencies with which a particular business organization comes into direct and frequent contact in the course of its functioningThese individuals and groups are known as STAKEHOLDERS because they have a stake (financial interest ) in the working and performance of the particular business List of external forces (stakeholders)Customers competitors investors suppliersmiddlemen (marketing intermediaries)

Execution 1 What do you mean by internal

factors in micro environmentAnswerInternal factors refer to all the factors existing within a business firm The internal factors are considered controllable because the enterprise has control over these factorsFor an example a company can alter its organization structure policies programmes employees physical facilities and marketing mix to suit the changes in the environment

2 What do you mean by external factors in micro environment

AnswerExternal factors refer to those individual and groups and agencies with which a particular business organization comes into direct and frequent contact in the course of its functioningThese individuals and groups are known as STAKEHOLDERS because they have a stake (financial interest) in the working and performance of the particular business

3Who are stakeholdersSTAKEHOLDERS are individuals and groups who have a stake (financial interest ) in the working and performance of the particular business 4Discuss the internal factors in briefa Corporate CultureThe values beliefs and attitudes of the founders and top management of the company exercise

financers publics

customers

suppliersfinancers

competitors

middlemen

publics

Fig STAKEHOLDERS OF A COMPANY

Apart from micro environment the other main dimension of business environment isMacro environment Macro environment refers to the general environment or remote environment within which a business firm and forces in its micro environment operateA company does not directly or regularly interact with the micro environmentTherefore macro environment is also known as indirect action EnvironmentThe macro environment forces are less controllable than the micro forces

Macro environment consists of the following components

POLITICAL AND LEGAL ENVIRONMENT

ECONOMIC SOCIAL AND ENVIRONMENT

CULTURAL

ENVIRONMENT

TECHNOLOGICAL ENVIRONMENT

a strong influence on what the cmpaany stands for how it does things and what it considers importantbMission and objectivesThe business philosophy and purpose of a comoany guide it prioritiesbusiness strategiesproduct market scope and development scope

cTop management structurethe composition of board of directors the degree of professionalization of management and the organizational structure of a company have important bearing on its business decisions

dPower structureThe internal power relationship between the board of directors and the chief executive is an important factor

eCompany image and brand equityThe image and brand equity of the company play a significant role in raising finance forming alliance choosing dealers and suppliers launching new products entering foreign markets

5 What is Macro environmentAnswerMacro environment refers to the general environment or remote environment within which a business firm and forces in its micro environment operateA company does not directly or regularly interact with the micro environmentTherefore macro environment is also known as indirect action EnvironmentThe macro environment forces are less controllable than the micro forces 6 What are the components of macro environmenta Political and legal environmentb Economic environmentc Social and cultural environmentd Technological environment

BUSINESS FIRM

Fig COMPONENTS OF MACRO ENVIRONMENTPolitical science

Introduction to political science

Comparative politics and itrsquos scope Comparative politics is the second major dimension of political scienceIt is also a very vast area of study and a very large number of political scientists even treat it as an autonomous area of study within the board ambit of political scienceScope of comparative politics-

1 All political structures -Comparative politics includes the study of all structures formalnon formal governmental and extra governmental which are directly or indirectly involved in politics in all the countries of the world

2 Functional studies- Comparative politics seeks to study politics less from the point of view of the legal institutions in terms of their powers and move from the point of view of their functions which constitute the political process and their actual Operation in the environment

3 Study of political behaviour- Another important part of its scope is the study of the actual behaviour of the people in the process of politics

4 Study of similarities and differences- comparative politics also undertakesan analysis of the similarities and differences among political process and functions

5 Study of all political systems -comparative politics seeks to analyse the actual behaviour and performance of all political systems western as well as non western

6 Study of the environment and infrastructure of politics-The study of politics demands a study of the psychological sociological economic and anthropological environment in fact the social environment as a whole in which each political system operates

7 Study of political culture- political culture is composed of attitudesbeliefs emotions and values of a society that relate to the political system or politics

8 Study of political participation- Political participation is a universal processThe only difference is that while in some states it is limited in others it is wider

9 Study of political process- political

Answer the following questions-

What is comparative politics

What are the scope of comparative politics

Homework- learn

processes like decision makingpolicy making judicial process leadership recruitment process and others are always at work in all political systems

The scope of comparative politics is very comprehensive It includes everything that falls within the area of political activity and political process

History CAMBRIDGE VIEW ABOUT

THE PARTITION

AND REFUTATION

OF CAMBRIDGE

VIEW

Cambridge view about the Partition The Cambridge school of historians have interpreted that opposition to partition scheme was made entirely by the elitist groups They hold the view that Lord Curzon planned to partition the Bengal for administrative purposeREFUTATION OFCAMBRIDGE VIEW The Rationalist historians have rejected the interpretations of the Cambridge School of historians on various grounds

1 QUESTION State different views of historians regarding Partition of Bengal

ANSWER Cambridge historians believed that Lord Curzon partitioned Bengal for administrative reasons only and not for the political motive The Middle class elitist group protested because of their petty interest The Hindu zamindars protested as they have to spend more money for managing their estatesThe lawyers of Calcutta High court feared to lose their clientBut according to the nationalist Historians was-

2- The ultimate object of Lord Curzon was to crush the unity of Bengal politicians

3- If Bengal becomes a separate province Bengali speaking 16 million people of western part would become minority under Hindi speaking people of Bihar and Oriya speaking people of Orissa

4- The bureaucrats expected that the protest movement would die down quickly

5- Lord Curzon used the Muslim community in his political game

6- Idealism had great contribution in the protest against partition

7- The people of the every section of society were affected by the partition of Bengal

Computer Science

Numbers Convertion of dcimal number to octal numberThe decimal numeral system is the standard system for denoting integer and non-integer numbers It is the extension to non-integer numbers of the Hindu-Arabic numeral system For writing numbers the decimal system uses ten decimal digits a decimal mark and for negative numbers a minus sign - The decimal digits are 0 1 2 3 4 5 6 7 8 9 the decimal separator is the dot in many countries

The octal numeral system or oct for short is the base-8 number system and uses the digits 0 to 7 Octal is sometimes used in computing instead of hexadecimal perhaps most often in modern times in conjunction with file

permissions under Unix systems It has the advantage of not requiring any extra symbols as digits It is also used for digital displays

Follow these steps to convert a decimal number into octal form

1 Divide the decimal number by 82 Get the integer quotient for the next iteration (if the number will not divide equally by 8 then round down the

result to the nearest whole number)3 Keep a note of the remainder it should be between 0 and 74 Repeat the steps until the quotient is equal to 05 Write out all the remainders from bottom to top This is the solution

For example if the given decimal number is 8453

Division Quotient Remainder

8453 8 1056 5

1056 8 132 0

132 8 16 4

16 8 2 0

2 8 0 2

Then the octal solution is 20405

Subject Eng Literature (The Tempest ndash William Shakespeare) Topic Act I Scene 1 Lines 33 to 67 (End of scene) Date 16th April 2020 (4th Period)

[Students should read the original play and also the paraphrase given in the school prescribed textbook]Summary Questions amp Answers

[SUMMARY OF THE ENTIRE SCENE]

o The play starts with the scene of a severe storm at sea Alonso (King of Naples) Sebastian (Alonsorsquos brother) Ferdinand (Alonsorsquos son) Gonzalo Antonio (the usurping Duke of Milan) are in a ship in the midst of the storm

o The mariners are trying their best to control the vessel from running aground and are totally following the orders of their Master the Boatswain They have scant success

o The mariners become extremely unhappy and annoyed when most of the passengers arrive on the deck thereby hampering their effort to save the ship There is serious confrontation between them and the passengers who are part of the Kingrsquos entourage

o The mariners could not save the ship

SUMMING-UP

(i) Vivid description of the scene which gives a realistic description of terror and confusion of a tropical storm

(ii) Shows Shakespearersquos accuracy of knowledge in describing the naval operations and also matters of seamanship

(1) GONZALO Ill warrant him for drowning (L 45-57)

though the ship were no stronger than a nutshell and as leaky as an unstanched

wenchBOATSWAIN Lay her a-hold a-hold Set her two courses Off to

sea again lay her offMARINERS All lost To prayers to prayers All lostBOATSWAIN What must our mouths be coldGONZALO The king and prince at prayers Lets assist them

For our case is theirsSEBASTIAN Im out of patienceANTONIO We are merely cheated of our lives by drunkards

This wide-chopped rascal - would thou mightst lie drowning the washing of ten tides

(a) What does Antonio say at the insolent manners of the boatswain just before the given passage

Being irritated at the insolent manners of the boatswain just before the given extract Antonio the Duke of Milan calls him a worthless dog son of a woman without any morals an arrogant and disrespectful noisemaker He says that the boatswain deserved to be hanged(b) What statement does Gonzalo repeat about the boatswain

Gonzalo shows his faith that the boatswain is not destined to die by drowning He is destined to be hanged and nothing can alter this decree of destiny He says that even if the ship was as frail as a nutshell the boatswain could not be drowned for his destiny was to be hanged(c) What do the passengers do when they have lost all hope of their survival

When the passengers have lost all hope of survival they take

(iii) The opening scene justifies the title ndash The Tempest

UNANSWERED QUESTIONS

(i) The King always travels with his entire fleet including his soldiers Where were the other ships

(ii) Why was the ship in that area Where was it coming from or going where

(iii) The ship broke apart What happened to those who were in the ship

(We shall get the answer to the above questions as the play progresses)

leave of life with fervent prayers The mariners take their last hearty drink and are ready for death(d) What blame does Antonio put upon the mariners and the boatswain Antonio rebukes the mariners that these drunkards have brought them to the present crisis by neglecting their duties He blames them saying that they are going to lose their lives entirely for the negligence of the boatswain and his fellows(e) What does Antonio say while cursing the boatswain

Antonio gives vent to his wrath upon the boatswain in particular He calls the boatswain a wide-mouthed rascal who deserves to be hanged on the sea-shore at low water mark so that ten tides might wash over his body and take out of him all the liquor that he has been drinking

Class XIISubject Topic Summary ExecutionHistory Topic

1 1935 ACT AND WORKING OF PROVINCIAL AUTONOMYCONGREE AND OTHER MINISTERSSUB TOPIC GOVERNMENT OF INDIA ACT1935

Government of India Act 1935 This act established a lsquoFederation of Indiarsquo made of British Indian provinces and Indian states and provided for autonomy with a government responsible to the elected legislature in every provinceThis act introduced abolition of Diarchy at provinces The entire provincial administration was introduced to the responsible ministers who were controlled and removed by the provincial legislature The provincial autonomy means two things First The provincial governments were wholly responsible to the provincial legislature Secondly Provinces were free from outside control and interference in the large number of matters The act divided the powers between the centre and provinces in terms of three lists- Federal list( for centre) Provincial list (for province) and concurrent list (for both) Residuary powers were given to the viceroy In the election under the government of India Act the Congress swept the poll the mandate of the people came in favour of the congress so far as general Hindu seats were concerned The Congress did not get a single Muslim seates in Bombay CP UP Sind and BengalIn five provinces Congress had yhe clear majority In BengalNWFPAssam and Bombay Congress emerged as a single largest partyOn the other side the performance of the Muslim League was badThus the Congress formed ministers in 7 provinces out of 11 provinces Coalition ministry was also formed in two other provincesOnly BENGAL AND Punjab had non- congress ministries

1 QUESTION What was the main change introduced by the Government of India ActANSWER a) The Act gave more

autonomy to the provinces b) Diarchy was abolished at the

provincial levelsc) The Governor was the head of

the executived) There was a council of

ministers to advise him The ministers were responsible to the provincial legislatures who controlled them The legislature could also remove the ministers

e) The Governors still retained special reserve powers

2 QUESTION Why did the federal scheme introduced by the Government of India Act 1935 never come into operation

ANSWER The Federal structure of the Government of India was to be composed with the Governor General and Council of ministers The Federal legislature was to be Bicameral legislature- The council of states and the House of Assembly The ministers were to be chosen by the Governor general and they were to hold the office during his pleasure

The provinces of British India would have to join the federation but this was not compulsory for the princely states

This federation never materialised because of the lack of support from the required number of

princely statesThis act was refused and

rejected by the princes the Congress and the Muslim League

Thus both Congress and the League participated in the election of 1937 Thus the federal part was never introduced but the provincial part was put into operations

Bengali 2nd

Language

াচেরর পরাথCনা(কহিতা )

াচেরর পরাথCনা কহিতাটি কহি (ঙখ দেঘাচে4র দো আচো য কহিতায় াচেরর পতর হমায়ন কঠিন দেরাচেগ আxানত ার ঈশবর া আললার কাচেছ পরাথCনা কচেরচেছন তার পচেতরর ীন হিফহিরচেয় হিচেত এই কহিতায় ার পচেতরর ীন হিভbা দেচেয়চেছন ারার এমনহিক হিনচের ীন হিসCচেনর হিহিনমচেয় হিতহিন তার দেছচের ীন হিফচের দেপচেত দেচেয়চেছন তার দেছচের এই দেরাচেগর ন য হিতহিন হিনচেচেকই ায়ী কচেরচেছন তার হিনচের করা পাপচেকই হিতহিন ায়ী কচেরচেছন এছাা রানৈনহিতক ও আথCসামাহিক অসথার কথা তচে ধরা চেয়চেছ এই কহিতায় ার তার হিনচের পাপ কমCচেকই ায়ী কচেরচেছ ার অন যায় ভাচে দেপহি((হিকতর মাধ যচেম অপররা য কচেরচেছ আর এই অন যায় কাচের ন যই তার পহিরাচের হিপযCয় এচেসচেছ দে এক পরকার মানহিক নধন ইহিতাচেসর ার হিপতা চেয় সবাভাহিকভাচে ভাচোাসা দে মমতা দেথচেক মকত চেত পাচেরনহিন তাই হিপতা চেয় আললা া ভগাচেনর কাচেছ পতর হমায়চেনর পরানহিভbা দেচেয়চেছন ার আললা া ভগাচেনর কাচেছ াহিনচেয়চেছন তার হিনচের ীন হিসCন হিচেত হিতহিন রাী তার হিহিনমচেয় পচেতরর ীন হিফচের দেপচেত দেচেয়চেছন াচেরর হিপতসভ হিচেকর কথা এই কহিতায় ফটিচেয় দেতাা চেয়চেছ হিপতা পচেতরর হিরাহিরত মান নধচেনর কথা তচে ধরা চেয়চেছ

হিচে(4 হিকছ াইচেনর তাৎপযC১) ldquoদেকাথায় দেগ ওর সবচছয দেৌন দেকাথায় কচেরায় দেগাপন bয়ldquoউততর) াচেরর পতর হমায়ন কঠিন দেরাচেগ অসসথ তাই তার দেযৌন াহিরচেয় যাচেচছ এই দেরাচেগ তাচেক দেগাপচেন কচেরকচের াচেচছ তার সক (হিকত ধীচের ধীচের bয় চেচছ তাই হিপতা চেয় ার আললার কাচেছ হমায়চেনর পরান হিভbা দেচেয়চেছন২) ldquoাগাও (চেরর পরাচেনত পরানতচের ধসর (ন দেযর আান গানldquoউততর) াচেরর পতর হমায়ন কঠিন দেরাচেগ আxানত তাই ার আ দে(াচেক মমCাত (চেরর পচেথ পরানতচের আান গান ধবহিনত দোক দেসই আান গান আললার কাচেছ দেযন চে যায় আললা দেযন এই আহিতC শচেন পচেতরর ীন হিফহিরচেয় দেয় ৩)ldquoনাহিক এই (রীচেরর পাচেপর ীানচেত দেকানই তরারণ দেনই ভহি4চেতরldquoউততর) হমায়চেনর অসসথতার ন য ার হিনচেচেকই ায়ী কচেরচেছন কারন ার অচেনক রা য অন যায় ভাচে কচেরচেছ তাই তার এই পাপ কাচের ন য তার ঘচের আ হিপ এচেসচেছ এই অন যায় কাচের ন য তার মহিকত দেনই তাই ার আললার কাচেছ এই পাপ কাচেযCর ন য bমা পরাথM

Hindi 2ndlang

-ासी(जयशकर परसा-)

-ासी जयशकर परसा- की एक ऐसी कहानी ह जिजसम भारतीय ससकनित और राषटरीयता का सवरगजीतहोता ह इस कहानी म इरावती एक निहद कनया ह जिजस मलअचछो न मलतान की लट म पकडा और -ासी बना दि-या उस 500 दि-न -कर काशी क एक महाजन न खरी-ा दसरी -ासी निफरोजा ह वह गलाम ह निफरोजा को छडान क कतिलए अहम- को 1000 सोन क कतिसकक भजन थ जो अभी तक नही आए थ राजा साहब कठोर होत हए भी निफरोजा को निबना धनराकतिश क कतिलए उस म कर -त ह वनिफरोजा को अहम- को समझान की बात कहत हकहानी क अत म हम -खत ह निक इरा वती और जाटो क सर-ार बलराज का मिमलन होता हअहम- को यa म मार दि-या जाता ह वहा निफरोजा की परसननता की समामिध बनती ह वहा एक फल चढती ह और डीजल आती ह निफरोजा उस समामिध की आजीवन -ासी बनी रहती हलखक अपन उददशय अथात -ास परथा पर परकाश डालन और इस परथा क कारण होन वाल -ातो क दखो को दि-खान म पणता सफल हए ह

helliphellipContinue to next

Biology Reproductio Today we will discuss about vegetative Q1 Name some vegetative propagules

n in Organisms

propagation of plants The process of multiplication in which fragments of plant body function as propagule and develop into new individual is called vegetative propagation The units of such propagation are runner rhizome tuber bulb etc

and the speciesinvolvedVegetative propagules

Parts involved

Bulb StemBulbil BulbilRhizome Stem Runner Stem Tuber Stem Offset Stem Leaf buds Leaves Suckers Stem

Corns Stem stolon

Q2 State advantages of vegetative propagation

i) Rapid methodii) Sure and easy methodiii) Useful in plants that cannot

produce viable seeds or long seed dormancy

iv) Maintains purity of raceQ 3 Banana fruit is said to be parthenocarpic where as turkey is said to be parthenogenetic WhyBanana develops without fertilization from an unfertilized ovary thus is parthenocarpicIn turkey the ovum or female gamete developinto a new chick without fertilization thus isparthgenetic

Q4 Why is water hyacinth is called as a ldquoTerror of Bengalrdquo Water hyacinth can

propagatevegetatively all over the water body in a short per short period of time This resulted increased biochemicaloxygen oxygen demand of water body causing mortalityof fishes It is very difficult to get rid off them Thus known as terror of Bengal

Chemistry

Solid state GENERAL CHARACTERISTICS OF SOLID STATEIn nature the particular state of matter is governed by two opposing forces at given set of temperature and pressure These forces are intermolecular force of attraction and thermal energy If intermolecular force of attraction is high as compared to thermal energy particles remains in closest position

Intext QuestionsQ1 Classify the following solids as crystalline and amorphous Sodium chloride quartz glass quartz rubber polyvinyl chloride Teflon

A1 Crystalline

and hence very less movement in particles is observed In this case solid state is the preferred state of matter

Let us revise the general characteristics of solid

i) Fixed mass volume and shape

ii) Strong intermolecular force of attraction

iii) Least intermolecular space

iv) Fixed position of constituent particles

v) Incompressible and rigid

Q2 what type of interactions hold the molecules together in a polar molecular solid[CBSE 2010]A2 The molecules in a solid are held together by van der Waals forces The term van der Waals forces include hydrogen bonding dipole-dipole attraction and London dispersion forces All molecules experience London dispersion forces In addition polar molecules can also experience dipole-dipole interactions So the interactions that holds the molecule together in polar molecular solid are London dispersion force and dipole-dipole interactionsQ3 Write a feature that will distinguish a metallic solid from an ionic solid [CBSE 2010]A3 Metals are malleable and ductile whereas ionic solid are hard and brittle Metallic solid has typical metallic lustre But ionic solid looks dullQ4 Write a point of distinction between a metallic solid and an ionic solid other than metallic lustre [CBSE 2012]A4 Metals are malleable and ductile whereas ionic solid are hard and brittleQ5 Write a distinguish feature of metallic solid [CBSE 2010]A5 The force of attraction in

solid Sodium chloride Quartz Amorphous solid Quartz glass rubber polyvinyl chloride Teflon Q2 why glass is considered as super cooled liquidA2 Glass shows the tendency to flow at slower rate like liquid Hence they considered as super cooled liquidQ3 why the window glass of old buildings show milky appearance with timeA3 Glass is an amorphous solid Amorphous solid has the tendency to develop some crystalline character on heating Due to heating in day over the number of years glass acquires some crystalline character and show milky appearanceQ4 why the glass panes fixed to window or doors of old building become slightly thicker at bottomA4 Glass is super cooled liquid It has the tendency to flow down very slowly Due to this glass pane becomes thicker at the bottom over the timeQ5 Sodium chloride is a crystalline solid It shows the same value of refractive index along all the direction TrueFalse Give reasonA5 FalseCrystalline solid shows anisotropy in properties That is it shows different values for the given physical property in different direction All the crystalline solids show anisotropy in refractive index Therefore sodium chloride will show different values of refractive index on different directions

Q6 Crystalline solid are anisotropic in nature What does this statement means

between the constituent particles is special kind of electrostatic attraction That is the attraction of positively charged kernel with sea of delocalized electronsQ6 which group of solid is electrical conductor as well as malleable and ductile [CBSE 2013]A6 Metallic solidQ7 why graphite is good conductor of electricity although it is a network (covalent solid)A7 The exceptional property of graphite is due to its typical structure In graphite each carbon is covalently bonded with 3 atoms in same layer The fourth valence electron of each atom is free to move in between different layersThis free electron makes the graphite a good conductor of electricity

[CBSE 2011]A6 Anisotropy is defined asrdquo Difference in properties when measured along different axis or from different directionsrdquo Crystalline solid show different values of some of the physical properties like electrical resistance refractive index etcwhen measured along the different directions The anisotropy in crystalline solid arises due to the different arrangement of particles in different directions

Math Function Composition of functions Think of an industrial plant that produce bottles of cold drinks first there is the operation (or function) f that puts the cold drink inside the bottle followed by the opeartion g that close the bottle with the capThis leads to the following definitionDefinition Let f A rarr B and g B rarr C be two functions Then the composition of f and g denoted by gof is defined as the function gof A rarr C given by gof(x) = g(f (x)) forall x isinA

Definition A function f X rarr Y is defined to be invertible if there exists a function g Y rarr X such that gof = IX and fog = IY The function g is called the inverse of f and is denoted by f -1

Thus if f is invertible then f must be one-one and onto and conversely if f is one-one and onto then f must be invertible This fact significantly helps for proving a function f to be invertible by showing that f is one-one and onto specially when the actual inverse of f is not to be determined

Example 1 Let f 2 3 4 5 rarr 3 4 5 9 and g 3 4 5 9 rarr 7 11 15 be functions defined as f(2) = 3 f(3) = 4 f(4) = f(5) = 5 and g (3) = g (4) = 7 and g (5) = g (9) = 11 Find gofSolution We have gof(2) = g (f(2)) = g (3) = 7 gof(3) = g (f(3)) = g (4) = 7gof(4) = g (f(4)) = g (5) = 11 and gof(5) = g (5) = 11Example 2 Find gof and fog if f R rarr R and g R rarr R are given by f(x) = cos x and g (x) = 3x2 Show that gof ne fogSolution We have gof(x) = g(f(x))=g(cosx) = 3 (cos x)2

= 3 cos2 x Similarly fog(x)=f(g (x))= f(3x2)= cos (3x2) Note that 3cos2 x ne cos 3x2 for x = 0 Hence gof ne fogExample 3 Show that if f A rarr B and g B rarr C are onto then gof A rarr C is also ontoSolution Given an arbitrary element z isin C there exists a pre-image y of z under g such that g (y) = z since g is onto Further for y isin B there exists an element x in A with f(x) = y since f is onto Therefore gof(x) = g (f(x)) = g (y) = z showing that gof is onto Example 4 Let Y = n2 n isin N sub N Consider f N rarr Y as f(n) = n2 Show that

f is invertible Find the inverse of fSolution An arbitrary element y in Y is of the form n2 for some n isin N This implies that n =radicy This gives a function g Y rarr N defined by g (y) =radicy Nowgof (n) = g (n2)=radicn2 = n and fog (y) =f(radicy) = (radicy) 2 y which shows that gof=IN and fog= IY Hence f is invertible with f -1 = g

Political Science

Constitution of India-The Preamble

Summary

Objective of the state-To secure equality of status and of opportunity To promote fraternity among all the citizens To assure the dignity of the individuals and Unity and integrity of the nation

Justice-Justice stands for rule of law absence of arbitrariness and a system of equal rights freedom and opportunities for all in a society India seeks social economic and political justice to ensure equality to its citizens

Liberty-Liberty implies the absence of restraints or domination on the activities of an individual such as freedom from slavery serfdom imprisonment despotism etc The Preamble provides for the liberty of thought expression belief faith and worship

Equality-Equality means the absence of privileges or discrimination against any section of the society The Preamble provides for equality of status and opportunity to all the people of the country

Fraternity-The Preamble declares that fraternity has to assure two thingsmdashthe dignity of the individual and the unity and

Execution

Answer the following questions-

Short notes-1 Equality2 Fraternity3 Justice4 Liberty

Homework-Learn

integrity of the nation The word integrity has been added to the Preamble by the 42nd Constitutional Amendment (1976)

Business studies

Human resource management (chapter 1)

On the day of 1504 2020 I have discussed with you the managerial functions and procurement functions of HRM

Today weare going to discuss about the development function integration functions and maintenance function

Development functions-HRM improves the knowledge skills attitude and values of employees so that they the present and future jobs more effectively it includes

1) Development functions of HRM

a) Performance appraisal = It implies systematic evaluation of employees with respect to their performance on the job and their potential for development

b) Training =It is the process by which employees learn knowledge skills and attitudes to achieve organisational and personal goals

c) Executive development = It is the process of developing managerial talent through appropriate program

2) Integration functionsa) HRM reconcile the goals of

organisation with those of its members through integrating function

b) HRM tries to motivate employees to various financial and non financial incentives provided in job specification etc

3) Maintenance functiona) HRM promote and protect the

physical and mental health of employees by providing several types of benefits like housing medical aid etc

b) It Promote Social security measures to employees by providing provident fund pension gratuity maternity benefits

SubjectCOMMERCE

Topic

BUSINESSENVIRONMENT

Summary

Now quickly let us revise the earlier points that we have already done in the last class and let us proceed with the other topics that are there in the chapter

Firstly we will recall the internal and external factors of micro environment and then we

Execution 3 What do you mean by internal factors

in micro environmentAnswerInternal factors refer to all the factors existing within a business firm The internal factors are considered controllable because the enterprise has control over these factors

Development FunctionsPerformance AppraisalTrainingExecution Development

shall proceed in details

Meaning and list of internal and external factors

aInternal factorsInternal factors refer to all the factors existing within a business firm The internal factors are considered controllable because the enterprise has control over these factorsFor an example a company can alter its organization structure policies programmes employees physical facilities and marketing mix to suit the changes in the environmentList of internal factors areCorporate culture mission and objectives top management organizations structure company image and brand equity company resources

b External factorsExternal factors refer to those individual and groups and agencies with which a particular business organization comes into direct and frequent contact in the course of its functioningThese individuals and groups are known as STAKEHOLDERS because they have a stake (financial interest ) in the working and performance of the particular business List of external forces (stakeholders)Customers competitors investors suppliersmiddlemen (marketing intermediaries)financers publics

customers

suppliersfinancers

For an example a company can alter its organization structure policies programmes employees physical facilities and marketing mix to suit the changes in the environment

4 What do you mean by external factors in micro environment

AnswerExternal factors refer to those individual and groups and agencies with which a particular business organization comes into direct and frequent contact in the course of its functioningThese individuals and groups are known as STAKEHOLDERS because they have a stake (financial interest) in the working and performance of the particular business

3Who are stakeholdersSTAKEHOLDERS are individuals and groups who have a stake (financial interest ) in the working and performance of the particular business 4Discuss the internal factors in briefa Corporate CultureThe values beliefs and attitudes of the founders and top management of the company exercise a strong influence on what the cmpaany stands for how it does things and what it considers importantbMission and objectivesThe business philosophy and purpose of a comoany guide it prioritiesbusiness strategiesproduct market scope and development scope

cTop management structurethe composition of board of directors the degree of professionalization of management and the organizational structure of a company have important bearing on its business decisions

dPower structureThe internal power relationship between the board of directors and the chief executive is an important factor

e Company image and brand equityThe image and brand equity of the company play a significant role in raising finance forming alliance choosing dealers and suppliers launching new products entering foreign markets

5 What is Macro environmentAnswerMacro environment refers to the general

competitors

middlemen

publics

Fig STAKEHOLDERS OF A COMPANY

Apart from micro environment the other main dimension of business environment isMacro environment Macro environment refers to the general environment or remote environment within which a business firm and forces in its micro environment operateA company does not directly or regularly interact with the micro environmentTherefore macro environment is also known as indirect action EnvironmentThe macro environment forces are less controllable than the micro forces

Macro environment consists of the following components

POLITICAL AND LEGAL ENVIRONMENT

ECONOMIC SOCIAL AND ENVIRONMENT

CULTURAL

ENVIRONMENT

TECHNOLOGICAL ENVIRONMENT

Fig COMPONENTS OF MACRO ENVIRONMENT

environment or remote environment within which a business firm and forces in its micro environment operateA company does not directly or regularly interact with the micro environmentTherefore macro environment is also known as indirect action EnvironmentThe macro environment forces are less controllable than the micro forces 6 What are the components of macro environmenta Political and legal environmentb Economic environmentc Social and cultural environmentd Technological environment

Computer Science

Logic gates

Digital systems are said to be constructed by using logic gates These gates are the AND OR NOT NAND NOR EXOR and EXNOR

BUSINESS FIRM

gates The basic operations are described below with the aid of truth tables

AND gate

The AND gate is an electronic circuit that gives a high output (1) only if all its inputs are high A dot () is used to show the AND operation ie AB Bear in mind that this dot is sometimes omitted ie ABOR gate

The OR gate is an electronic circuit that gives a high output (1) if one or more of its inputs are high A plus (+) is used to show the OR operationNOT gate

The NOT gate is an electronic circuit that produces an inverted version of the input at its output It is also known as an inverter If the input variable is A the inverted output is known as NOT A This is also shown as A or A with a bar over the top as shown at the outputs The diagrams below show two ways that the NAND logic gate can be configured to produce a NOT gate It can also be done using NOR logic gates in the same way

NAND gate

This is a NOT-AND gate which is equal to an AND gate followed by a NOT gate The outputs of all NAND gates are high if any of the inputs are low The symbol is an AND gate with a small circle on the output The small circle represents inversion

NOR gate

This is a NOT-OR gate which is equal to an OR gate followed by a NOT gate The outputs of all NOR gates are low if any of the inputs are highThe symbol is an OR gate with a small circle on the output The small circle represents inversion

EXOR gate

The Exclusive-OR gate is a circuit which will give a high output if either but not both of its two inputs are high An encircled plus sign ( ) is used to show the EOR operation

EXNOR gate

The Exclusive-NOR gate circuit does the opposite to the EOR gate It will give a low output if either but not both of its two inputs are high The symbol is an EXOR gate with a small circle on the output The small circle represents inversion The NAND and NOR gates are called universal functions since with either one the AND and OR functions and NOT can be generated

Note A function in sum of products form can be implemented using NAND gates by replacing all AND and OR gates by NAND gates A function in product of sums form can be implemented using NOR gates by replacing all AND and OR gates by NOR gates

Logic gate symbols

Table 2 is a summary truth table of the inputoutput combinations for the NOT gate together with all possible inputoutput combinations for the other gate functions Also note that a truth table with n inputs has 2n rows You can compare the outputs of different gates

Logic gates representation using the Truth table

Example

A NAND gate can be used as a NOT gate using either of the following wiring configurations

Subject Eng Literature (The Tempest ndash William Shakespeare) Topic Act III Scene 3 Lines 53 to 110 (End of the scene) Date 16th April 2020 (2nd Period)

[Students should read the original play and also the paraphrase given in the school prescribed textbook]Summary Questions amp Answers

o Seeing this strange scene all are inclined to believe the tales told by travelers that there truly are ldquounicornsrdquo and ldquothe phoenixrsquo thronerdquo

o As they are about to sit down to the feast the banquet is snatched away by a harpy (Ariel disguised) A spiritrsquos voice (Arielrsquos voice) denounces Alonso Sebastian and Antonio with particular

1 ARIEL You are three men of sin whom Destiny

(Line 53-58)That hath to instrument this

lower world And what is int the never-surfeited sea

Hath caused to belch up you and on this island

Where man doth not inhabit you rsquomongst men

Being most unfit to live I have made you mad

reference to their crime in expelling Prospero from Milan They have not received any punishment for their deed earlier but the time for their punishment has arrived Upon Alonso it pronounces ldquolingering perdition worse than deathrdquo from which there is no remedy except through sincere repentance Ariel then vanishes in thunder and the shapes enter again and carry away the table

o Prospero watching invisibly is very pleased with the performance of Ariel and his (Prosperorsquos) ldquomeaner ministersrdquo All his enemies are now in his power and are in a fit of desperation He then leaves them and goes to see how Ferdinand and Miranda are getting on

o Alonso is now much humbled and penitent with the after effect of the spiritrsquos denunciation of his crimes He believes that his son is lost forever After this all disperse being stricken mad by the speech of the spirit

o Gonzalo fearing that they may do violence to themselves or to one another follows them and bid others to follow

(a) To whom does Ariel disguised as a harpy call the three sinners What game did Fate of Destiny play with

them

The three sinners called by Ariel are Alonso Sebastian and Antonio It was Destiny which had caused the ocean to cast the three sinners on the shore Though the ocean is all the time devouring whatever appears on its surface and is never satisfied with its continual swallowing of the ships and men in the present case the ocean had cast these three sinners on the shore without killing them

(b) Who had jointly been responsible for the conspiracy against Prospero What is Prosperorsquos purpose behind all this

Three men Alonso Sebastian and Antonio had jointly

been responsible for the conspiracy against Prospero They had driven out Prospero form Milan Prosperorsquos purpose is to make these three sinners realize the wrong they had done He wants them to repent for their criminal deeds because repentance leads to self-esteem(c )What does Ariel (the harpy) tell Alonso and his companions when they take out their swords to attack him

Seeing them drawing their swords Ariel (harpy) tells them that he and his companions are the instruments of destiny and that it is not possible for human beings to do them any injury He says that the swords of human beings can not injure even a minute part of his feathers Their swords are as ineffective against him and his companions as against the wind or the water

(d) Give the explanatory meanings of the following expressions in the context of the above extract

(i)Never surfeited (ii) Belch up (iii) lsquomongst men

(i) Never surfeited never led to satisfaction

(ii) Belch up cast ashore(iii) lsquomongst men in human

society2

I and my fellows (Line 60-65)

Are ministers of Fate The elementsOf whom your swords are tempered may as wellWound the loud winds or with bemocked-at stabsKill the still-closing waters as diminishOne dowl thats in my plume

IMPORTANT PASSAGES EXPLAINED

The elements

(Line 61-66)Of whom your swords are tempered may

as wellWound the loud winds or with

bemocked-at stabs

(a) Who is lsquoIrsquo Who are his lsquofellowsrdquo

lsquoIrsquo is referred to Ariel in disguise of a harpy His lsquofellowsrsquo are other spirits serving Prospero the real Duke of Milan who has acquired supernatural powers after being banished from his Dukedom Prospero has settled in this uninhabited island

(b) What are the elements that have temperrsquod the swords Why will it not work against the speaker

The swords (of Alonso and his companions) are tempered by metal (steel) which is taken out of the earth and refined by

Kill the still-closing waters as diminishOne dowl thats in my plume My fellow

ministersAre like invulnerable

In these words Ariel reminds the King and his companions of the utter futility of drawing swords against himself and his fellows Ariel drives Alonso Antonio and Sebastian the three men of sin to desperation ndash a state in which men do violence to themselves They draw swords to strike Ariel But Ariel reminds them that he and the other spirits are the ministers of destiny and nothing can wound them The steel of which their swords are made of may cut the wind or water which being divided always closes up again Even supposing that such things may be possible it is quite impossible that their swords will cut one feather in their plume They are incapable of being wounded by any sword of man Hence it is foolish on their part to attempt to strike at Ariel and his fellow-spirits

For which foul deed

(Line 72-75)The powers delaying not forgetting

haveIncensed the seas and shores yea all the

creatures Against your peace

Ariel enters like a harpy and remaining invisible tells Alonso Sebastian and Antonio that he and other harpies are the agents of Destiny appointed to carry out her decrees He tells them that their punishment for the crime against Prospero which has been so long deferred is now to fall upon them He reminds them that they had expelled Prospero from Milan and set him and his innocent child adrift on the sea and that the sea had paid them back for their sin by the shipwreck and by the calamities they have suffered He tells them that the powers above which did not forget this mean treachery but only deferred the punishment have now engaged the seas and the shores and all living beings including him and his comrades against them The very elements and supernatural agency Ariel adds have taken up the avenging of their crime against Prospero

the action of fire It may cut the wind or water which being divided always closes up again

The sword will not work against the spirits and the harpy because they are the ministers of destiny and nothing can wound them nor it will cut a single feather in their plume

(c )What is the meaning of lsquodowlrsquo in the last line

The term lsquodowlrsquo means a filament or the smallest part of a feather In this context Ariel in disguise of harpy says that their sword cannot even damage the smallest filament of their (Arielrsquos and other spirits) feathers as they are incapable of being wounded by any sword of man

(d) What does the speaker remind the listeners about

Ariel in disguise of harpy reminds Alonso the King of Naples Sebastian Alonsorsquos brother and Antonio the present Duke of Milan and the treacherous brother of Prospero as they being three men of sin He even reminds them that their punishment for their crime against Prospero which has been so long deferred now falls upon them He reminds them that they have expelled Prospero from Milan and has set him along with his innocent infant daughter adrift on the sea So the sea has paid them back for their sin by their shipwreck and the calamities they have suffered since then The harpy rebukes Alonso of his sin that has incensed the Gods and has deprived him of his son as a punishment

(e) How do they respond

When Ariel in disguise of a harpy reminds Alonso Sebastian and Antonio of their past misdeeds and sin Alonso has a look of terror and confusion in his eyes He utters the words of sincere repentance wrung out of his conscience-stricken heart It appears to him that all the elements of nature the sea-waves the wind and the thunder proclaiming a loud voice in the name of Prospero and the crime Alonso has committed against him They are calling upon him to repent There is a deep storm raging in Alonsorsquos breast and the echoes of that storm are ringing in his ears like a clear note of wind-instrument A note of denunciation of Alonsorsquos crime leaves him much humbled and penitent and confirms his belief that his son is lost forever But Sebastian and Antonio shows some courage instead of repentance They wish to kill the spirits or devils if it appears

3

Of my instruction hast thou nothing bated (Line 85-93)

In what thou hast to say So with good life

And observation strange my meaner ministers

Their several kinds have done My high charms work

And these mine enemies are all knit upIn their distractions They now are in my

powerAnd in these fits I leave them while I visitYoung Ferdinand whom they suppose is

drownedAnd his and mine loved darling

Methought the billows spoke and (Line 96-99)

told me of itThe winds did sing it to me and the

thunderThat deep and dreadful organ-pipe

pronouncedThe name of Prosper It did bass my

trespass

These are the words of contrition coming from Alonso Ariel has driven him to a deep repentance for conspiring with Antonio against Prospero He now feels a sincere remorse It appears to him that all the elements of nature the sea-waves the wind and the thunder proclaimed with a loud voice the name of Prospero and the crime Alonso had committed against him They are calling upon him to repent There is a deep storm raging in Alonsorsquos breast and the echoes of that storm are ringing in his ears like the clear note of a wind-instrument

Comment These are the words of sincere repentance wrung out of the conscience-stricken heart of Alonso Alonso who is the lesser villain is the first to give way to remorse under the effect of Arielrsquos speech The words of Ariel seem to him to be the voice of conscience speaking to him He is driven to desperation a state in which he might do violence to his life

(a) Identify the speaker State the context

Prospero the ruler of the island is the speaker The famous banquet scene has been enacted very well Ariel and his junior spirits have played their roles excellently Prospero is glad to say words of praise for them(b) In what way the speakerrsquos instructions have been carried out

According to Prosperorsquos instructions a banquet was presented before the King of Naples and his companions when they were tired and hungry Just when they were preparing to eat the feast the banquet was suddenly removed by exercising supernatural powers All this was done by Ariel Prosperorsquos chief assistant and a powerful spirit

Ariel not only made the feast disappear but also delivered his speech blaming the King and his two companions for their past wicked deeds He warned them to repent for their misdeeds or suffer forever on that uninhabited island

(c) Who are referred to as lsquomeaner ministersrsquo What have they done

Prospero refers as lsquomeaner ministersrsquo to his other lesser spirits who were assisting Ariel in presenting a scene before the kingrsquos party They entered the scene to the accompaniment of music They assumed several strange shapes and brought in a banquet Then they danced about it with gentle actions of salutations thus inviting the King and others to eat the feast

These spirits play their role again when Ariel in the shape of a harpy quits the scene These shapes enter again and dancing with mocking gestures carry away the table

(d) Who are the speakerrsquos enemies What has happened to them

King of Naples Alonso his brother Sebastian and the present Duke of Milan Antonio (Prosperorsquos own brother) are Prosperorsquos enemies With the turn of events they have all been washed ashore on the island which is ruled by Prospero the great magician Actually this happened after the shipwreck caused by a storm which was raised by Prospero with the purpose of bringing these people to his island Prosperorsquos spirits have already confused and terrified these enemies and they are under Prosperorsquos control He can treat them as he likes

(e) What does he say about Ferdinand Explain what is meant by ldquohellip his and mine darlingrdquo

Prospero knows that Alonsorsquos son prince Ferdinand is alive though his father thinks that the prince has been drowned

Prospero refers to his daughter Miranda who is dear to him She is also very dear to Prince Ferdinand who has fallen in love with her They are waiting to be married soon for which they have received Prosperorsquos consent

4

ALONSO O it is monstrous monstrous (Line 95-102)

Methought the billows spoke and told me of it

The winds did sing it to me and the thunderThat deep and dreadful organ-

pipe pronouncedThe name of Prosper It did bass

my trespassTherefore my son ithrsquo ooze is

bedded andIll seek him deeper than eer

plummet soundedAnd with him there lie mudded

(a) In what way does Alonso express his horror when his conscience is awakened by Arielrsquos words

When Alonsorsquos conscience is awakened by Arielrsquos words he expresses his horror at what he has heard He gets the feeling that the waves of the ocean the wind and the loud thunder have spoken to him and uttered the name of Prospero Because of being reminded of his crime in a very loud and rough voice he comes to realize that he has lost his son for his past misdeeds

(b) What does Alonso imagine about his son What does Alonso want to do in his desperate state

Alonso imagines that his son is lying in the mud at the bottom of the sea He feels desperate that he wants to drown himself in the ocean deeper than the plumb-line has ever gone He wants to lie with his son at the bottom of the sea

(c) How do Sebastian and Antonio want to face the evil spirits

Sebastian says that he is not at all afraid of what the harpy has said and that he is prepared to fight any number of such monsters if they appear before him only one at a time Antonio says that he would support Sebastian in the fight against the fiendsyyy

(d) Why does Gonzalo ask Adrian to follow the three men

Gonzalo tells Adrian that all the three men namely Alonso Sebastian and Antonio are in a wild and reckless mood The thought of the heinous crime of which they are guilty has begun to torment their minds So he asks Adrian to follow those three men without loss of time and prevent them from doing anything which the turmoil in their minds might lead them to do

(e) What opinion do you form of Alonso from the above extract

Alonso who is the lesser villain is the first to give way to remorse under the effect of Arielrsquos speech The words of Ariel seem to him to be the voice of conscience speaking to him He is driven to desperation a state in which he might do violence to his life

Subject =Accounts

Ac-12 15420 topic-pL Appropriation ac

PROFIT AND LOSS APPROPRIATION ACCOUNT

MEANING AND PREPARATIONProfit and Loss Appropriation Account is merely an extension of the Profit and Loss Account of the firm The profit of the firm has to be distributed amongst the partners in their respective profit sharing ratio But before its distribution it needs to be adjusted All Adjustments like partnerrsquos salary partnerrsquos commission interest on capital interest on drawings etc are made in this account These adjustments will reduce the amount of profit for distribution This adjusted profit will be distributed amongst the partners in their profit sharing ratio To prepare it at first the balance of Profit and Loss Account is transferred to this account The journal entries for the preparation of Profit and Loss Appropriation Account are given below

1 for transfer of the balance of Profit and Loss Account to Profit and Loss Appropriation Account

(a) In case of Net Profit

Profit and Loss Ac helliphelliphelliphelliphellipDrTo Profit and Loss Appropriation Ac(Net Profit transferred to Profit and Loss Appropriation Ac)

(b)In case of Net Loss

Profit and Loss Appropriation Achelliphelliphellip DrTo Profit and Loss Ac(Net Loss transferred to Profit and Loss Appropriation Ac)

2 for Interest on Capital

For transferring on Interest on CapitalProfit and Loss Appropriation Achelliphelliphellip DrTo Interest on Capital Ac(Interest on capital transferred to Profit amp Loss Appropriation Ac)

3 for Interest on Drawings

For transferring Interest on Drawings Interest on Drawings Achelliphelliphelliphelliphelliphellip DrTo Profit and Loss Appropriation Ac(Interest on drawing transferred to Profit amp Loss Appropriation Ac)

4 For Partnerrsquos SalaryFor transfer of partnerrsquos SalaryProfit and Loss Appropriation Achelliphellip DrTo Salary Ac(Salary transferred to profit amp Loss Appropriation Ac)

5 For Partnerrsquos CommissionFor transferring commissionProfit and Loss Appropriation Achelliphelliphellip DrTo Commission Ac(Commission transferred to Profit and Loss Appropriation Ac)

6 For Transfer of agreed amount to General ReserveProfit and Loss Appropriation Ac helliphellipDrTo General Reserve Ac(Transfer to General Reserve)

7 for share of Profit or Loss appropriation(a) If ProfitProfit and Loss Appropriation Achelliphellip DrTo Partnerrsquos CapitalCurrent Ac(Profit transferred to capitalcurrent Ac)(b) If LossPartnerrsquos Capital Current Achelliphelliphelliphellip DrTo Profit and Loss Appropriation Ac(Loss transferred to capitalcurrent Ac)

THE FORMAT OF PROFIT AND LOSS APPROPRIATION

Profit and Loss Appropriation Account for the year endedhelliphelliphelliphellip

Particulars Amount Particulars Amount

To PL Ac (loss) By pL Ac (profit)

To Interest on capital BY Interest on drawings

To partner`s commission by Partner`s capital Ac ( loss)

To Partner`s salary To Interest on partner`s loan To General Reserve To Partner`s Capital AC (Profit)

Subject= Economics

MOVEMENT ALONG THE DEMAND CURVE (CHANGE IN QUANTITY DEMANDED)In law of demand you have already studied the inverse relationship between price and quantity demanded When quantity demanded of a commodity changes due to change in its price keeping other factors constant it is called change in quantity demanded It is graphically expressed as a movement along the same demand curve There can be either a downward movement or an upward movement along the same demand curve Upward movement along the same demand curve is called contraction of demand or decrease in quantity demanded and downward movement along the same demand curve is known as expansion of demand or increase in quantity demanded

Extention of demandd

price (rs)p A

B Extentionp1 d

Q Q1

Quantity demanded ( in units)

Contraction of demandd

p2 Ccontraction

p APrice (Rs)

d

Q2 Q

Quantity demanded (in units)

Explanation of movement of demand A fall in price from OP to OP1 leads to increase in quantity demanded from OQ to OQ1 (expansion of demand) resulting in a downward movement from point A to point B along the same demand curve DD When Price rises from OP to OP2 quantity demanded falls from OQ to OQ2 (contraction of demand) leading to an upward movement from point A to point C along the same demand curve DD

  • Activity Series of Metals
    • Drawbacks of Rutherfordrsquos model of atom
      • Electromagnetic radiations
      • Properties of electromagnetic radiations
      • Characteristics of electromagnetic radiations
        • Plancks Quantum Theory-
        • Photoelectric effect
          • Intext Questions
            • Logic gates
            • Digital systems are said to be constructed by using logic gates These gates are the AND OR NOT NAND NOR EXOR and EXNOR gates The basic operations are described below with the aid of truth tables
            • AND gate
            • Example
Page 29:  · Web viewSubject. Topic. Summary. Execution. English 1 . Chapter 1 naming words . Page 8. Write the names of these pictures:- Person:-1. father. 2.Firefighter 3.doctor 4 ...

element having the same atomicnumber but different mass numbere g 1H11H21H3

Isobars Isobars are the atoms of different elements having the same massnumber but different atomic numbere g 18Ar40 20Ca40

Isoelectronic species These are those species which have the same numberof electrons

Electromagnetic radiationsThe radiations which are associated withelectrical and magnetic fields are called electromagnetic radiations When anelectrically charged particle moves under acceleration alternating electricaland magnetic fields are produced and transmitted These fields aretransmitted in the form of waves These waves are called electromagneticwaves or electromagnetic radiations

wave as shown It is denoted by λb Frequency (ν) It may be defined as the number of waves which passthrough a particular point in one secondc Velocity (v) It is defined as the distance travelled by a wave in onesecond In vacuum all types of electromagnetic radiations travel with thesame velocity Its value is 3 times10 8m sec-1 It is denoted by v

d Wave number Wave number is defined as the number of wavelengths per unit lengthVelocity = frequency timeswavelength c = νλ

Plancks Quantum Theory- o The radiant energy is emitted or absorbed not continuously but discontinuously in the form of small discrete packets of energy called lsquoquantumrsquo In case of light the quantum of energy is called a lsquophotonrsquoo The energy of each quantum is directly proportional to the frequency of the radiation ie E α υ or E= hυ where h= Planckrsquos constant = 6626 x 10-27 Js o Energy is always emitted or absorbed as integral multiple of this uantum E=nhυ Where n=1234Black body An ideal body which emits and absorbs all frequencies is calleda black body The radiation emitted by such a body is called black body radiation

Photoelectric effectThe phenomenon of ejection of electrons from thesurface of metal when light of suitable frequency strikes it is calledphotoelectric effect The ejected electrons are called photoelectrons

Biology Chapter - 02Systematics and Five Kingdoms

Scientists divide the whole living organisms into two kingdom first and ultimately by five kingdom at last

In the earlier systems of classifications organisms are divided into kingdom plantaeand kingdom animalia on the of presenceof cell wall their modes of nutrition and movements

Some problem arise like fungi share manycharacteristic withplant despite their heterotrophic nutrition bacteria protozoa areunicellular present in both kingdom Toovercome this third kingdom Protista isintroduced which include

unicellularorganisms But there is also another

problem Allunicellular organisms are not similar kind The cellular structure of prokaryotes is verydifferent from that of other organismsEukaryotes possess a true nucleus and allcell organelles that are not present inprokaryotes So the fourth kingdom Monerais introduced which include unicellular prokaryotes (bacteriaamp blue green algae)

bull Still some problem arise in kingdomplantae

So in 1969 R H Whittakar proposedanew five kingdom System of classification

i) Kingdom Monera - unicellular prokaryotes

ii) kingdom Protista - unicellular eukaryotes

iii) Kingdom Fungi - uni or multicellular fungi with cell wall but without chlorophyll

iv) Kingdom Plantae - Multicellular Plants

v) Kingdom Animalia - Multicellular Animals

EVS Chapter 1 ndash Modes of Existence

An agricultural society

An agricultural society also known as an agrarian society is a society that constructs social order around a reliance upon farming More than half the people living in that society make their living by farming

People in an agricultural society generally lead a more settled lifestyle than those in nomadic hunter-gatherer or semi-nomadic pastoral societies because they live permanently near the land that is farmed Agricultural settlements tend to develop in areas of convenience near bodies of water which is used for both crops and transportation or along trade routes Not everyone in an agricultural society is a farmer Some people make a living trading or making and selling goods such as tools used for farming

Another way to define an agrarian society is to see the total amount of production in a nation In an agrarian society cultivating the land is the main source of wealth Such a society can recognize other means of subsistence and work habits but emphasizes the importance of agriculture and livestock Agrarian societies have existed in various parts of the world for 10000 years and continue to exist today They have been the most common form of socio-economic organization for most of recorded human history

Q) Write the features of agricultural society

Ans - Structure and Features of Agrarian Society1 Occupational Structure

An agrarian society is generally associated with the domestication of plants and animals The domestication of plants means farming and that of animals means herding Often there is mixture of farming and the use of such domesticated animals as cow goat and sheep

2 Forms of Land Ownership in Agrarian SocietiesGenerally there are landlords supervisory farmers cultivators and share croppers The landholders own the land but do not work on it They let it out for sharecropping The supervisory farmers are those who live by having their land cultivated by hired labourers The cultivators cultivate the land for themselvesThe share-croppers are those who live by tilling other peoplersquos land or a crop-sharing basis The artisans own their means of production and produce by their own labour in their homesteads

3 Village Community System An agrarian society is highlighted by

the institution of village community system The agrarian economy made fixed dwelling houses necessary Living close together for protection and co-operation and living nearer to the land gave birth to agricultural villages The village is not only the residential place of farmers it is also the social integrator

4 Minimal Division of Labour Another structural feature of agrarian society is a minimal division of labour Except for the basic division founded on age and sex differences there are few specialized roles There is only one predominant type of occupation ie domestication of plants and animals For all the people the environment physical as well as social is the same

5 Role of Family The farm family is of the patriarchal type the father is the final arbiter in most of the familyrsquos major decisions The life of ail men and women is merged in family life Since there are not many special organizations family is the only organisation to perform the tasks of aid and protection

6 Sense of Unity The members of an agrarian society exhibit a strong in-group feeling Since the whole of their social lives is wrapped up in a society which is physically economically and socially homogenous they are inclined to view the entire outside world as an out group

7 Informal Social Control An agrarian society is regionally divided into villages In a village community the force of traditional mores is more dominant than in the urban community In the village everybody is known to everybody The members in a village community help each other and share the joy and sorrows of each other Crime in an agrarian society is rare

8 Simplicity and Uniformity Life of the people in an agrarian society is marked by simplicity and uniformity Their main occupation is agriculture which largely depends upon the vagaries of nature An agrarian society is a religious society

Math Compound angles Compound angles The algebraic sum of two or more angles is called a compound angle If A B C be three angles then A+B B+C C+A A-B B-C A-C A+B-C etc are compound angles In this chapter we shall discuss the trigonometrical ratios of compound angles Theorem 1 If A B and A+B are all pisitive acute angles theni) sin( A+B) = sin A cos B + cosA sinBii) cos(A+B) = cosA cosB- sinA sinBTheorem 2If A and B are positive acute angles and AgtB theni) sin(A-B) = sin A cosB- cos A sinBii) cos(A-B) = cos A cos B+ sin A sin BTo prove that i) sin(A+B) sin (A-B) = sin2 A - sin2 B = cos2 B- cos2 A

Example 1 Prove that tan70deg=2tan50deg+tan20degSolutiontan70deg = tan(50deg + 20deg)Or tan70deg=(tan 50deg+tan 20deg)(1-tan50degtan20deg) or tan70deg (1 ndash tan 50deg tan20deg) = tan50deg+tan20degor tan70deg= tan70deg tan50deg tan20deg+ tan50deg + tan20deg = cot20deg tan50deg tan20deg + tan50deg + tan20deg = 2 tan50deg+ tan20degExample 2 If A + B = 45deg show that (1 + tanA) (1 + tanB) = 2Solutiontan(A + B) =( tan A + tan B) (1 - tan

ii) cos(A+B) Cos(A-B) = cos2 A- sin2 B = cos2 B -sin2 AProof i) LHS= sin(A+B)sin(AminusB) [Recall sin(αminusβ)=sinαcosβminuscosαsinβ And sin(α+β)=sinαcosβ+cosαsinβ]= (sinAcosB+cosAsinB)times(sinAcosBminuscosAsinB)= sin2Acos2Bminuscos2Asin2B [Recall sin2α+cos2α=1 From above we can then assume correctly that sin2α=1minuscos2α AND cos2α=1minussin2α] = sin2A(1minussin2B)minussin2B(1minussin2A) = sin2Aminussin2Asin2Bminussin2B+sin2Asin2B = sin2Aminussin2B= 1-cos2A-(1-cos2B) = cos2 B- cos2 A = RHSii)LHS= cos (A+B) cos(A-B) [ cos(A+B) = cos AcosB- sinAsinBCos(A-B) = cosAcosB+ sinAsinB]= cos2 A Cos2 B- sin2 A Sin2 B= cos2 A( 1-sin2 B) - (1- cos2 A) sin2 B= cos2 A- cos2 A sin2 B- sin2 B+ cos2 A sin2 B=cos2 A- sin2 B=1- sin2 A-(1-cos2 B) = cos2 B- sin2 A= RHSTangent formulae for compound anglesi)tan (A + B) = tan A + tan B1-tan A tan Bii)tan (A ndash B) = tan A-tan B1+tan A tan Biii) cot (A + B) = cot Acot B-1cot A+cot B(viii) cot (A ndash B) = cot Acot B+1cot B-cot A

A tan B) Or 1= (tan A+ tanB) (1-tan A tanB) Or tanA + tanB + tanA tanB + 1 = 1 + 1Or tanA (1 + tanB) + (1 + tanB) = 2Or (1 + tanA) (1 + tanB) = 2Example 3 Find the value of sin 15degSolution sin 15deg= sin(45deg-30deg) = sin45degcos 30deg- cos45degsin30deg =(1radic2) (radic32) -(1radic2) (12) = (radic3-1) 2radic2Example 4 If sin A = 1 radic10 and sin B = 1 radic5 where A and B are positive acute angles then what is A + B SolutionWe know that sin (A + B) = sin A cos B + cos A sin B= [1 radic10] [radic(1 minus 1 5)] + [1 radic5] radic(1 minus 1 10)= [1 radic10] [radic4 5] + [1 radic5] [radic9 10]= [1 radic50] times (2 + 3)= 5 radic50 = 1 radic2

sin (A + B) = sin π 4rArrHence A + B = π 4Example 5 If A + B = 225o then find [cot A] [1 + cotA] times [cot B] [1 + cot B]Solution[cot A] [1 + cotA] times [cot B] [1 + cot B] = 1 [(1 + tan A) times (1 + tan B)]=1 [tan A + tan B + 1 + tan A tan B] [ tan (A + B) = tan225o]∵

tan A + tan B = 1minus tan A tan BrArr= 1 [1 minus tan A tan B + 1 + tan A tan B]= 1 2

COMMERCE

CLASSIFICTION OF HUMAN ACTIVITIES-ECONOMIC AND NON-ECONOMIC

Firstly we shall recall the previous class for 5 mins especially for the absentees and for also the rest of the students who were there

Today at first we briefly discuss the earlier portions of the chapter

1Business-It includes all those economic activities which are concerned with production and exchange of goods and services with the object of earning profit Example A factory shop beauty parlour also business enterprises

2Profession ndashThe term profession means an occupation which involves application of specialized knowledge and skills to earn a living For Example Chartered Accountancy medicine law tax consultancy are example of professions

Questions1What are the main features of ProfessionAnswer The main features of a profession are as follows a Specialised body of knowledge-Every profession has a specialised and systematised body of knowledge b Restricted entry- Entry to a profession is allowed only to those who have completed the prescribed education and have the specialised examination c Formal education and training ndashA formal education and training is given to the person who wants to acquire the professional

3Employment-Employment mean an economic activity where people work for others in exchange for some remuneration (salary)The persons who work for others are called lsquoemployeesrsquo The persons or organizations which engage others to work for them are called lsquoemployersrsquoEg A doctor working in a hospital is employment as he is working for a salaryA lawyer may serve as a law officer in a bank

With this we shall proceed with the features of both Profession amp Employment

The main features of a profession are as follow

a Specialised body of knowledge b Restricted entry c Formal education and training d Professional association e Service motive f Code of contact

The main features of an employment are as follows

a In employment a person works for others called employer

b An employee provides personal service

c There is a service agreement or contract between the employee and the employer

d The employee has to obey the order of the employer

e No capital investment is made by the employer

Various examples of Employment are as follows

aA teacher teaching in a school or collegeb An engineer employed in Municipal Corporation of DelhicAn accountant working in the accounts department of a companydA doctor working in a hospital

Note In all the above examples of employment the individual who is involved in each example is working as an employee for a salary under an employer

qualification(MBBSCALLB)d Service motive ndashProfessionals are expected to emphasis service more on their clients rather than economic gain f Code of Conduct-The activities of professionals are regulated by a code of conduct

2 What are the main features of EmploymentAnswer The main features of an employment are as followsa In employment a person works for others called employerb An employee provides personal servicec There is a service agreement or contract between the employee and the employerd The employee has to obey the order of the employere No capital investment is made by the employer

3 Give various Professions and their respective Association are given below

Professions

Professional

Professional association

Medical profession

Doctor Medical Council of India

Law profession

Lawyers Bar Council of India

Accounting Profession

Chartered

The Institute of Chartered Accounts of India( ICAI)

Engineerin Engineers The

g Profession

institute of Engineers (India)

Accounts Basic accounting terms

Today we will give you some questions from the previous study material

Questions6) Define accounting7) What do you mean by debit

and credit8) Explain the types of account9) Define the following terms

a) Assetsb) Capitalc) Purchased) Debtorse) Transactions

10) Name the types of accounts given below

a) Krishnas accountb) Machinery accountc) Royalty accountd) Salary accounte) Furniture accountf) Audit fee account

Economics Basic Economic ConceptsSub topic

UTILITY

Before starting todayrsquos class we shall recall the last class which was about UTILITY AND THE FEATURES OF UTILITY

Now we shall proceed with the further topics of the chapter

Todayrsquos topic from the chapter lsquo Basic Economic Conceptsrsquo will be TOTAL UTILITY amp MARGINAL UTILITYNow let us quickly revise the concept of utility with an example ie goods and services are designed because they have an ability to satisfy human wantsThis feature of being able to satisfy human wants is termed as utility For example we derive utility from WiFi services as it gives us satisfaction by connecting us to our friends and family through social media here consumers derive utility from WiFi services

From the above concept we shall start with todayrsquos topicEconomists have defined TOTAL UTILITY (TU) as the total satisfaction obtained by consuming a given total amount of a good and serviceFor example the total satisfaction obtained from eating 10 mangoes is the total utility of 10 mangoes

MARGINAL UTILITY (MU) is the additional satisfaction derived from each additional unit

Questions1 What is Total Utility (TU)

Answer Total Utility (TU) is the

aggregate of the utility that a consumer derives from the consumption of a certain amount of a commodityTU=MU1+MU2++MUn

2 What is Marginal UtilityAnswer

Marginal Utility (MU) is the additional made to the total utility as consumption is increased by one more unit of the commodityMU= TUn ndashTUn-1

NoteOften economists tend to

subdivide utility into an imaginary unit called UTIL

consumed In this casethe utility obtained from each mango as it is consumed as the MU of that mango It is also defined as the addition made to the total utility when an additional unit is consumed Often economists tend to subdivide utility into an imaginary unit called UTIL

Note As a consumer increases the consumption of a good over period of time the total utility or total satisfaction derived from it increases to appoint and thereafter it decreasesHowever as the consumer keeps on consuming the good the marginal utility or the additional utility derived from it decreases

SubjectBusiness studies

Topic

BUSINESSENVIRONMENT

Summary

Now quickly let us revise the earlier points that we have already done in the last class and let us proceed with the other topics that are there in the chapter

Firstly we will recall the internal and external factors of micro environment and then we shall proceed in details

Meaning and list of internal and external factors

aInternal factorsInternal factors refer to all the factors existing within a business firm The internal factors are considered controllable because the enterprise has control over these factorsFor an example a company can alter its organization structure policies programmes employees physical facilities and marketing mix to suit the changes in the environmentList of internal factors areCorporate culture mission and objectives top management organizations structure company image and brand equity company resources

b External factorsExternal factors refer to those individual and groups and agencies with which a particular business organization comes into direct and frequent contact in the course of its functioningThese individuals and groups are known as STAKEHOLDERS because they have a stake (financial interest ) in the working and performance of the particular business List of external forces (stakeholders)Customers competitors investors suppliersmiddlemen (marketing intermediaries)

Execution 1 What do you mean by internal

factors in micro environmentAnswerInternal factors refer to all the factors existing within a business firm The internal factors are considered controllable because the enterprise has control over these factorsFor an example a company can alter its organization structure policies programmes employees physical facilities and marketing mix to suit the changes in the environment

2 What do you mean by external factors in micro environment

AnswerExternal factors refer to those individual and groups and agencies with which a particular business organization comes into direct and frequent contact in the course of its functioningThese individuals and groups are known as STAKEHOLDERS because they have a stake (financial interest) in the working and performance of the particular business

3Who are stakeholdersSTAKEHOLDERS are individuals and groups who have a stake (financial interest ) in the working and performance of the particular business 4Discuss the internal factors in briefa Corporate CultureThe values beliefs and attitudes of the founders and top management of the company exercise

financers publics

customers

suppliersfinancers

competitors

middlemen

publics

Fig STAKEHOLDERS OF A COMPANY

Apart from micro environment the other main dimension of business environment isMacro environment Macro environment refers to the general environment or remote environment within which a business firm and forces in its micro environment operateA company does not directly or regularly interact with the micro environmentTherefore macro environment is also known as indirect action EnvironmentThe macro environment forces are less controllable than the micro forces

Macro environment consists of the following components

POLITICAL AND LEGAL ENVIRONMENT

ECONOMIC SOCIAL AND ENVIRONMENT

CULTURAL

ENVIRONMENT

TECHNOLOGICAL ENVIRONMENT

a strong influence on what the cmpaany stands for how it does things and what it considers importantbMission and objectivesThe business philosophy and purpose of a comoany guide it prioritiesbusiness strategiesproduct market scope and development scope

cTop management structurethe composition of board of directors the degree of professionalization of management and the organizational structure of a company have important bearing on its business decisions

dPower structureThe internal power relationship between the board of directors and the chief executive is an important factor

eCompany image and brand equityThe image and brand equity of the company play a significant role in raising finance forming alliance choosing dealers and suppliers launching new products entering foreign markets

5 What is Macro environmentAnswerMacro environment refers to the general environment or remote environment within which a business firm and forces in its micro environment operateA company does not directly or regularly interact with the micro environmentTherefore macro environment is also known as indirect action EnvironmentThe macro environment forces are less controllable than the micro forces 6 What are the components of macro environmenta Political and legal environmentb Economic environmentc Social and cultural environmentd Technological environment

BUSINESS FIRM

Fig COMPONENTS OF MACRO ENVIRONMENTPolitical science

Introduction to political science

Comparative politics and itrsquos scope Comparative politics is the second major dimension of political scienceIt is also a very vast area of study and a very large number of political scientists even treat it as an autonomous area of study within the board ambit of political scienceScope of comparative politics-

1 All political structures -Comparative politics includes the study of all structures formalnon formal governmental and extra governmental which are directly or indirectly involved in politics in all the countries of the world

2 Functional studies- Comparative politics seeks to study politics less from the point of view of the legal institutions in terms of their powers and move from the point of view of their functions which constitute the political process and their actual Operation in the environment

3 Study of political behaviour- Another important part of its scope is the study of the actual behaviour of the people in the process of politics

4 Study of similarities and differences- comparative politics also undertakesan analysis of the similarities and differences among political process and functions

5 Study of all political systems -comparative politics seeks to analyse the actual behaviour and performance of all political systems western as well as non western

6 Study of the environment and infrastructure of politics-The study of politics demands a study of the psychological sociological economic and anthropological environment in fact the social environment as a whole in which each political system operates

7 Study of political culture- political culture is composed of attitudesbeliefs emotions and values of a society that relate to the political system or politics

8 Study of political participation- Political participation is a universal processThe only difference is that while in some states it is limited in others it is wider

9 Study of political process- political

Answer the following questions-

What is comparative politics

What are the scope of comparative politics

Homework- learn

processes like decision makingpolicy making judicial process leadership recruitment process and others are always at work in all political systems

The scope of comparative politics is very comprehensive It includes everything that falls within the area of political activity and political process

History CAMBRIDGE VIEW ABOUT

THE PARTITION

AND REFUTATION

OF CAMBRIDGE

VIEW

Cambridge view about the Partition The Cambridge school of historians have interpreted that opposition to partition scheme was made entirely by the elitist groups They hold the view that Lord Curzon planned to partition the Bengal for administrative purposeREFUTATION OFCAMBRIDGE VIEW The Rationalist historians have rejected the interpretations of the Cambridge School of historians on various grounds

1 QUESTION State different views of historians regarding Partition of Bengal

ANSWER Cambridge historians believed that Lord Curzon partitioned Bengal for administrative reasons only and not for the political motive The Middle class elitist group protested because of their petty interest The Hindu zamindars protested as they have to spend more money for managing their estatesThe lawyers of Calcutta High court feared to lose their clientBut according to the nationalist Historians was-

2- The ultimate object of Lord Curzon was to crush the unity of Bengal politicians

3- If Bengal becomes a separate province Bengali speaking 16 million people of western part would become minority under Hindi speaking people of Bihar and Oriya speaking people of Orissa

4- The bureaucrats expected that the protest movement would die down quickly

5- Lord Curzon used the Muslim community in his political game

6- Idealism had great contribution in the protest against partition

7- The people of the every section of society were affected by the partition of Bengal

Computer Science

Numbers Convertion of dcimal number to octal numberThe decimal numeral system is the standard system for denoting integer and non-integer numbers It is the extension to non-integer numbers of the Hindu-Arabic numeral system For writing numbers the decimal system uses ten decimal digits a decimal mark and for negative numbers a minus sign - The decimal digits are 0 1 2 3 4 5 6 7 8 9 the decimal separator is the dot in many countries

The octal numeral system or oct for short is the base-8 number system and uses the digits 0 to 7 Octal is sometimes used in computing instead of hexadecimal perhaps most often in modern times in conjunction with file

permissions under Unix systems It has the advantage of not requiring any extra symbols as digits It is also used for digital displays

Follow these steps to convert a decimal number into octal form

1 Divide the decimal number by 82 Get the integer quotient for the next iteration (if the number will not divide equally by 8 then round down the

result to the nearest whole number)3 Keep a note of the remainder it should be between 0 and 74 Repeat the steps until the quotient is equal to 05 Write out all the remainders from bottom to top This is the solution

For example if the given decimal number is 8453

Division Quotient Remainder

8453 8 1056 5

1056 8 132 0

132 8 16 4

16 8 2 0

2 8 0 2

Then the octal solution is 20405

Subject Eng Literature (The Tempest ndash William Shakespeare) Topic Act I Scene 1 Lines 33 to 67 (End of scene) Date 16th April 2020 (4th Period)

[Students should read the original play and also the paraphrase given in the school prescribed textbook]Summary Questions amp Answers

[SUMMARY OF THE ENTIRE SCENE]

o The play starts with the scene of a severe storm at sea Alonso (King of Naples) Sebastian (Alonsorsquos brother) Ferdinand (Alonsorsquos son) Gonzalo Antonio (the usurping Duke of Milan) are in a ship in the midst of the storm

o The mariners are trying their best to control the vessel from running aground and are totally following the orders of their Master the Boatswain They have scant success

o The mariners become extremely unhappy and annoyed when most of the passengers arrive on the deck thereby hampering their effort to save the ship There is serious confrontation between them and the passengers who are part of the Kingrsquos entourage

o The mariners could not save the ship

SUMMING-UP

(i) Vivid description of the scene which gives a realistic description of terror and confusion of a tropical storm

(ii) Shows Shakespearersquos accuracy of knowledge in describing the naval operations and also matters of seamanship

(1) GONZALO Ill warrant him for drowning (L 45-57)

though the ship were no stronger than a nutshell and as leaky as an unstanched

wenchBOATSWAIN Lay her a-hold a-hold Set her two courses Off to

sea again lay her offMARINERS All lost To prayers to prayers All lostBOATSWAIN What must our mouths be coldGONZALO The king and prince at prayers Lets assist them

For our case is theirsSEBASTIAN Im out of patienceANTONIO We are merely cheated of our lives by drunkards

This wide-chopped rascal - would thou mightst lie drowning the washing of ten tides

(a) What does Antonio say at the insolent manners of the boatswain just before the given passage

Being irritated at the insolent manners of the boatswain just before the given extract Antonio the Duke of Milan calls him a worthless dog son of a woman without any morals an arrogant and disrespectful noisemaker He says that the boatswain deserved to be hanged(b) What statement does Gonzalo repeat about the boatswain

Gonzalo shows his faith that the boatswain is not destined to die by drowning He is destined to be hanged and nothing can alter this decree of destiny He says that even if the ship was as frail as a nutshell the boatswain could not be drowned for his destiny was to be hanged(c) What do the passengers do when they have lost all hope of their survival

When the passengers have lost all hope of survival they take

(iii) The opening scene justifies the title ndash The Tempest

UNANSWERED QUESTIONS

(i) The King always travels with his entire fleet including his soldiers Where were the other ships

(ii) Why was the ship in that area Where was it coming from or going where

(iii) The ship broke apart What happened to those who were in the ship

(We shall get the answer to the above questions as the play progresses)

leave of life with fervent prayers The mariners take their last hearty drink and are ready for death(d) What blame does Antonio put upon the mariners and the boatswain Antonio rebukes the mariners that these drunkards have brought them to the present crisis by neglecting their duties He blames them saying that they are going to lose their lives entirely for the negligence of the boatswain and his fellows(e) What does Antonio say while cursing the boatswain

Antonio gives vent to his wrath upon the boatswain in particular He calls the boatswain a wide-mouthed rascal who deserves to be hanged on the sea-shore at low water mark so that ten tides might wash over his body and take out of him all the liquor that he has been drinking

Class XIISubject Topic Summary ExecutionHistory Topic

1 1935 ACT AND WORKING OF PROVINCIAL AUTONOMYCONGREE AND OTHER MINISTERSSUB TOPIC GOVERNMENT OF INDIA ACT1935

Government of India Act 1935 This act established a lsquoFederation of Indiarsquo made of British Indian provinces and Indian states and provided for autonomy with a government responsible to the elected legislature in every provinceThis act introduced abolition of Diarchy at provinces The entire provincial administration was introduced to the responsible ministers who were controlled and removed by the provincial legislature The provincial autonomy means two things First The provincial governments were wholly responsible to the provincial legislature Secondly Provinces were free from outside control and interference in the large number of matters The act divided the powers between the centre and provinces in terms of three lists- Federal list( for centre) Provincial list (for province) and concurrent list (for both) Residuary powers were given to the viceroy In the election under the government of India Act the Congress swept the poll the mandate of the people came in favour of the congress so far as general Hindu seats were concerned The Congress did not get a single Muslim seates in Bombay CP UP Sind and BengalIn five provinces Congress had yhe clear majority In BengalNWFPAssam and Bombay Congress emerged as a single largest partyOn the other side the performance of the Muslim League was badThus the Congress formed ministers in 7 provinces out of 11 provinces Coalition ministry was also formed in two other provincesOnly BENGAL AND Punjab had non- congress ministries

1 QUESTION What was the main change introduced by the Government of India ActANSWER a) The Act gave more

autonomy to the provinces b) Diarchy was abolished at the

provincial levelsc) The Governor was the head of

the executived) There was a council of

ministers to advise him The ministers were responsible to the provincial legislatures who controlled them The legislature could also remove the ministers

e) The Governors still retained special reserve powers

2 QUESTION Why did the federal scheme introduced by the Government of India Act 1935 never come into operation

ANSWER The Federal structure of the Government of India was to be composed with the Governor General and Council of ministers The Federal legislature was to be Bicameral legislature- The council of states and the House of Assembly The ministers were to be chosen by the Governor general and they were to hold the office during his pleasure

The provinces of British India would have to join the federation but this was not compulsory for the princely states

This federation never materialised because of the lack of support from the required number of

princely statesThis act was refused and

rejected by the princes the Congress and the Muslim League

Thus both Congress and the League participated in the election of 1937 Thus the federal part was never introduced but the provincial part was put into operations

Bengali 2nd

Language

াচেরর পরাথCনা(কহিতা )

াচেরর পরাথCনা কহিতাটি কহি (ঙখ দেঘাচে4র দো আচো য কহিতায় াচেরর পতর হমায়ন কঠিন দেরাচেগ আxানত ার ঈশবর া আললার কাচেছ পরাথCনা কচেরচেছন তার পচেতরর ীন হিফহিরচেয় হিচেত এই কহিতায় ার পচেতরর ীন হিভbা দেচেয়চেছন ারার এমনহিক হিনচের ীন হিসCচেনর হিহিনমচেয় হিতহিন তার দেছচের ীন হিফচের দেপচেত দেচেয়চেছন তার দেছচের এই দেরাচেগর ন য হিতহিন হিনচেচেকই ায়ী কচেরচেছন তার হিনচের করা পাপচেকই হিতহিন ায়ী কচেরচেছন এছাা রানৈনহিতক ও আথCসামাহিক অসথার কথা তচে ধরা চেয়চেছ এই কহিতায় ার তার হিনচের পাপ কমCচেকই ায়ী কচেরচেছ ার অন যায় ভাচে দেপহি((হিকতর মাধ যচেম অপররা য কচেরচেছ আর এই অন যায় কাচের ন যই তার পহিরাচের হিপযCয় এচেসচেছ দে এক পরকার মানহিক নধন ইহিতাচেসর ার হিপতা চেয় সবাভাহিকভাচে ভাচোাসা দে মমতা দেথচেক মকত চেত পাচেরনহিন তাই হিপতা চেয় আললা া ভগাচেনর কাচেছ পতর হমায়চেনর পরানহিভbা দেচেয়চেছন ার আললা া ভগাচেনর কাচেছ াহিনচেয়চেছন তার হিনচের ীন হিসCন হিচেত হিতহিন রাী তার হিহিনমচেয় পচেতরর ীন হিফচের দেপচেত দেচেয়চেছন াচেরর হিপতসভ হিচেকর কথা এই কহিতায় ফটিচেয় দেতাা চেয়চেছ হিপতা পচেতরর হিরাহিরত মান নধচেনর কথা তচে ধরা চেয়চেছ

হিচে(4 হিকছ াইচেনর তাৎপযC১) ldquoদেকাথায় দেগ ওর সবচছয দেৌন দেকাথায় কচেরায় দেগাপন bয়ldquoউততর) াচেরর পতর হমায়ন কঠিন দেরাচেগ অসসথ তাই তার দেযৌন াহিরচেয় যাচেচছ এই দেরাচেগ তাচেক দেগাপচেন কচেরকচের াচেচছ তার সক (হিকত ধীচের ধীচের bয় চেচছ তাই হিপতা চেয় ার আললার কাচেছ হমায়চেনর পরান হিভbা দেচেয়চেছন২) ldquoাগাও (চেরর পরাচেনত পরানতচের ধসর (ন দেযর আান গানldquoউততর) াচেরর পতর হমায়ন কঠিন দেরাচেগ আxানত তাই ার আ দে(াচেক মমCাত (চেরর পচেথ পরানতচের আান গান ধবহিনত দোক দেসই আান গান আললার কাচেছ দেযন চে যায় আললা দেযন এই আহিতC শচেন পচেতরর ীন হিফহিরচেয় দেয় ৩)ldquoনাহিক এই (রীচেরর পাচেপর ীানচেত দেকানই তরারণ দেনই ভহি4চেতরldquoউততর) হমায়চেনর অসসথতার ন য ার হিনচেচেকই ায়ী কচেরচেছন কারন ার অচেনক রা য অন যায় ভাচে কচেরচেছ তাই তার এই পাপ কাচের ন য তার ঘচের আ হিপ এচেসচেছ এই অন যায় কাচের ন য তার মহিকত দেনই তাই ার আললার কাচেছ এই পাপ কাচেযCর ন য bমা পরাথM

Hindi 2ndlang

-ासी(जयशकर परसा-)

-ासी जयशकर परसा- की एक ऐसी कहानी ह जिजसम भारतीय ससकनित और राषटरीयता का सवरगजीतहोता ह इस कहानी म इरावती एक निहद कनया ह जिजस मलअचछो न मलतान की लट म पकडा और -ासी बना दि-या उस 500 दि-न -कर काशी क एक महाजन न खरी-ा दसरी -ासी निफरोजा ह वह गलाम ह निफरोजा को छडान क कतिलए अहम- को 1000 सोन क कतिसकक भजन थ जो अभी तक नही आए थ राजा साहब कठोर होत हए भी निफरोजा को निबना धनराकतिश क कतिलए उस म कर -त ह वनिफरोजा को अहम- को समझान की बात कहत हकहानी क अत म हम -खत ह निक इरा वती और जाटो क सर-ार बलराज का मिमलन होता हअहम- को यa म मार दि-या जाता ह वहा निफरोजा की परसननता की समामिध बनती ह वहा एक फल चढती ह और डीजल आती ह निफरोजा उस समामिध की आजीवन -ासी बनी रहती हलखक अपन उददशय अथात -ास परथा पर परकाश डालन और इस परथा क कारण होन वाल -ातो क दखो को दि-खान म पणता सफल हए ह

helliphellipContinue to next

Biology Reproductio Today we will discuss about vegetative Q1 Name some vegetative propagules

n in Organisms

propagation of plants The process of multiplication in which fragments of plant body function as propagule and develop into new individual is called vegetative propagation The units of such propagation are runner rhizome tuber bulb etc

and the speciesinvolvedVegetative propagules

Parts involved

Bulb StemBulbil BulbilRhizome Stem Runner Stem Tuber Stem Offset Stem Leaf buds Leaves Suckers Stem

Corns Stem stolon

Q2 State advantages of vegetative propagation

i) Rapid methodii) Sure and easy methodiii) Useful in plants that cannot

produce viable seeds or long seed dormancy

iv) Maintains purity of raceQ 3 Banana fruit is said to be parthenocarpic where as turkey is said to be parthenogenetic WhyBanana develops without fertilization from an unfertilized ovary thus is parthenocarpicIn turkey the ovum or female gamete developinto a new chick without fertilization thus isparthgenetic

Q4 Why is water hyacinth is called as a ldquoTerror of Bengalrdquo Water hyacinth can

propagatevegetatively all over the water body in a short per short period of time This resulted increased biochemicaloxygen oxygen demand of water body causing mortalityof fishes It is very difficult to get rid off them Thus known as terror of Bengal

Chemistry

Solid state GENERAL CHARACTERISTICS OF SOLID STATEIn nature the particular state of matter is governed by two opposing forces at given set of temperature and pressure These forces are intermolecular force of attraction and thermal energy If intermolecular force of attraction is high as compared to thermal energy particles remains in closest position

Intext QuestionsQ1 Classify the following solids as crystalline and amorphous Sodium chloride quartz glass quartz rubber polyvinyl chloride Teflon

A1 Crystalline

and hence very less movement in particles is observed In this case solid state is the preferred state of matter

Let us revise the general characteristics of solid

i) Fixed mass volume and shape

ii) Strong intermolecular force of attraction

iii) Least intermolecular space

iv) Fixed position of constituent particles

v) Incompressible and rigid

Q2 what type of interactions hold the molecules together in a polar molecular solid[CBSE 2010]A2 The molecules in a solid are held together by van der Waals forces The term van der Waals forces include hydrogen bonding dipole-dipole attraction and London dispersion forces All molecules experience London dispersion forces In addition polar molecules can also experience dipole-dipole interactions So the interactions that holds the molecule together in polar molecular solid are London dispersion force and dipole-dipole interactionsQ3 Write a feature that will distinguish a metallic solid from an ionic solid [CBSE 2010]A3 Metals are malleable and ductile whereas ionic solid are hard and brittle Metallic solid has typical metallic lustre But ionic solid looks dullQ4 Write a point of distinction between a metallic solid and an ionic solid other than metallic lustre [CBSE 2012]A4 Metals are malleable and ductile whereas ionic solid are hard and brittleQ5 Write a distinguish feature of metallic solid [CBSE 2010]A5 The force of attraction in

solid Sodium chloride Quartz Amorphous solid Quartz glass rubber polyvinyl chloride Teflon Q2 why glass is considered as super cooled liquidA2 Glass shows the tendency to flow at slower rate like liquid Hence they considered as super cooled liquidQ3 why the window glass of old buildings show milky appearance with timeA3 Glass is an amorphous solid Amorphous solid has the tendency to develop some crystalline character on heating Due to heating in day over the number of years glass acquires some crystalline character and show milky appearanceQ4 why the glass panes fixed to window or doors of old building become slightly thicker at bottomA4 Glass is super cooled liquid It has the tendency to flow down very slowly Due to this glass pane becomes thicker at the bottom over the timeQ5 Sodium chloride is a crystalline solid It shows the same value of refractive index along all the direction TrueFalse Give reasonA5 FalseCrystalline solid shows anisotropy in properties That is it shows different values for the given physical property in different direction All the crystalline solids show anisotropy in refractive index Therefore sodium chloride will show different values of refractive index on different directions

Q6 Crystalline solid are anisotropic in nature What does this statement means

between the constituent particles is special kind of electrostatic attraction That is the attraction of positively charged kernel with sea of delocalized electronsQ6 which group of solid is electrical conductor as well as malleable and ductile [CBSE 2013]A6 Metallic solidQ7 why graphite is good conductor of electricity although it is a network (covalent solid)A7 The exceptional property of graphite is due to its typical structure In graphite each carbon is covalently bonded with 3 atoms in same layer The fourth valence electron of each atom is free to move in between different layersThis free electron makes the graphite a good conductor of electricity

[CBSE 2011]A6 Anisotropy is defined asrdquo Difference in properties when measured along different axis or from different directionsrdquo Crystalline solid show different values of some of the physical properties like electrical resistance refractive index etcwhen measured along the different directions The anisotropy in crystalline solid arises due to the different arrangement of particles in different directions

Math Function Composition of functions Think of an industrial plant that produce bottles of cold drinks first there is the operation (or function) f that puts the cold drink inside the bottle followed by the opeartion g that close the bottle with the capThis leads to the following definitionDefinition Let f A rarr B and g B rarr C be two functions Then the composition of f and g denoted by gof is defined as the function gof A rarr C given by gof(x) = g(f (x)) forall x isinA

Definition A function f X rarr Y is defined to be invertible if there exists a function g Y rarr X such that gof = IX and fog = IY The function g is called the inverse of f and is denoted by f -1

Thus if f is invertible then f must be one-one and onto and conversely if f is one-one and onto then f must be invertible This fact significantly helps for proving a function f to be invertible by showing that f is one-one and onto specially when the actual inverse of f is not to be determined

Example 1 Let f 2 3 4 5 rarr 3 4 5 9 and g 3 4 5 9 rarr 7 11 15 be functions defined as f(2) = 3 f(3) = 4 f(4) = f(5) = 5 and g (3) = g (4) = 7 and g (5) = g (9) = 11 Find gofSolution We have gof(2) = g (f(2)) = g (3) = 7 gof(3) = g (f(3)) = g (4) = 7gof(4) = g (f(4)) = g (5) = 11 and gof(5) = g (5) = 11Example 2 Find gof and fog if f R rarr R and g R rarr R are given by f(x) = cos x and g (x) = 3x2 Show that gof ne fogSolution We have gof(x) = g(f(x))=g(cosx) = 3 (cos x)2

= 3 cos2 x Similarly fog(x)=f(g (x))= f(3x2)= cos (3x2) Note that 3cos2 x ne cos 3x2 for x = 0 Hence gof ne fogExample 3 Show that if f A rarr B and g B rarr C are onto then gof A rarr C is also ontoSolution Given an arbitrary element z isin C there exists a pre-image y of z under g such that g (y) = z since g is onto Further for y isin B there exists an element x in A with f(x) = y since f is onto Therefore gof(x) = g (f(x)) = g (y) = z showing that gof is onto Example 4 Let Y = n2 n isin N sub N Consider f N rarr Y as f(n) = n2 Show that

f is invertible Find the inverse of fSolution An arbitrary element y in Y is of the form n2 for some n isin N This implies that n =radicy This gives a function g Y rarr N defined by g (y) =radicy Nowgof (n) = g (n2)=radicn2 = n and fog (y) =f(radicy) = (radicy) 2 y which shows that gof=IN and fog= IY Hence f is invertible with f -1 = g

Political Science

Constitution of India-The Preamble

Summary

Objective of the state-To secure equality of status and of opportunity To promote fraternity among all the citizens To assure the dignity of the individuals and Unity and integrity of the nation

Justice-Justice stands for rule of law absence of arbitrariness and a system of equal rights freedom and opportunities for all in a society India seeks social economic and political justice to ensure equality to its citizens

Liberty-Liberty implies the absence of restraints or domination on the activities of an individual such as freedom from slavery serfdom imprisonment despotism etc The Preamble provides for the liberty of thought expression belief faith and worship

Equality-Equality means the absence of privileges or discrimination against any section of the society The Preamble provides for equality of status and opportunity to all the people of the country

Fraternity-The Preamble declares that fraternity has to assure two thingsmdashthe dignity of the individual and the unity and

Execution

Answer the following questions-

Short notes-1 Equality2 Fraternity3 Justice4 Liberty

Homework-Learn

integrity of the nation The word integrity has been added to the Preamble by the 42nd Constitutional Amendment (1976)

Business studies

Human resource management (chapter 1)

On the day of 1504 2020 I have discussed with you the managerial functions and procurement functions of HRM

Today weare going to discuss about the development function integration functions and maintenance function

Development functions-HRM improves the knowledge skills attitude and values of employees so that they the present and future jobs more effectively it includes

1) Development functions of HRM

a) Performance appraisal = It implies systematic evaluation of employees with respect to their performance on the job and their potential for development

b) Training =It is the process by which employees learn knowledge skills and attitudes to achieve organisational and personal goals

c) Executive development = It is the process of developing managerial talent through appropriate program

2) Integration functionsa) HRM reconcile the goals of

organisation with those of its members through integrating function

b) HRM tries to motivate employees to various financial and non financial incentives provided in job specification etc

3) Maintenance functiona) HRM promote and protect the

physical and mental health of employees by providing several types of benefits like housing medical aid etc

b) It Promote Social security measures to employees by providing provident fund pension gratuity maternity benefits

SubjectCOMMERCE

Topic

BUSINESSENVIRONMENT

Summary

Now quickly let us revise the earlier points that we have already done in the last class and let us proceed with the other topics that are there in the chapter

Firstly we will recall the internal and external factors of micro environment and then we

Execution 3 What do you mean by internal factors

in micro environmentAnswerInternal factors refer to all the factors existing within a business firm The internal factors are considered controllable because the enterprise has control over these factors

Development FunctionsPerformance AppraisalTrainingExecution Development

shall proceed in details

Meaning and list of internal and external factors

aInternal factorsInternal factors refer to all the factors existing within a business firm The internal factors are considered controllable because the enterprise has control over these factorsFor an example a company can alter its organization structure policies programmes employees physical facilities and marketing mix to suit the changes in the environmentList of internal factors areCorporate culture mission and objectives top management organizations structure company image and brand equity company resources

b External factorsExternal factors refer to those individual and groups and agencies with which a particular business organization comes into direct and frequent contact in the course of its functioningThese individuals and groups are known as STAKEHOLDERS because they have a stake (financial interest ) in the working and performance of the particular business List of external forces (stakeholders)Customers competitors investors suppliersmiddlemen (marketing intermediaries)financers publics

customers

suppliersfinancers

For an example a company can alter its organization structure policies programmes employees physical facilities and marketing mix to suit the changes in the environment

4 What do you mean by external factors in micro environment

AnswerExternal factors refer to those individual and groups and agencies with which a particular business organization comes into direct and frequent contact in the course of its functioningThese individuals and groups are known as STAKEHOLDERS because they have a stake (financial interest) in the working and performance of the particular business

3Who are stakeholdersSTAKEHOLDERS are individuals and groups who have a stake (financial interest ) in the working and performance of the particular business 4Discuss the internal factors in briefa Corporate CultureThe values beliefs and attitudes of the founders and top management of the company exercise a strong influence on what the cmpaany stands for how it does things and what it considers importantbMission and objectivesThe business philosophy and purpose of a comoany guide it prioritiesbusiness strategiesproduct market scope and development scope

cTop management structurethe composition of board of directors the degree of professionalization of management and the organizational structure of a company have important bearing on its business decisions

dPower structureThe internal power relationship between the board of directors and the chief executive is an important factor

e Company image and brand equityThe image and brand equity of the company play a significant role in raising finance forming alliance choosing dealers and suppliers launching new products entering foreign markets

5 What is Macro environmentAnswerMacro environment refers to the general

competitors

middlemen

publics

Fig STAKEHOLDERS OF A COMPANY

Apart from micro environment the other main dimension of business environment isMacro environment Macro environment refers to the general environment or remote environment within which a business firm and forces in its micro environment operateA company does not directly or regularly interact with the micro environmentTherefore macro environment is also known as indirect action EnvironmentThe macro environment forces are less controllable than the micro forces

Macro environment consists of the following components

POLITICAL AND LEGAL ENVIRONMENT

ECONOMIC SOCIAL AND ENVIRONMENT

CULTURAL

ENVIRONMENT

TECHNOLOGICAL ENVIRONMENT

Fig COMPONENTS OF MACRO ENVIRONMENT

environment or remote environment within which a business firm and forces in its micro environment operateA company does not directly or regularly interact with the micro environmentTherefore macro environment is also known as indirect action EnvironmentThe macro environment forces are less controllable than the micro forces 6 What are the components of macro environmenta Political and legal environmentb Economic environmentc Social and cultural environmentd Technological environment

Computer Science

Logic gates

Digital systems are said to be constructed by using logic gates These gates are the AND OR NOT NAND NOR EXOR and EXNOR

BUSINESS FIRM

gates The basic operations are described below with the aid of truth tables

AND gate

The AND gate is an electronic circuit that gives a high output (1) only if all its inputs are high A dot () is used to show the AND operation ie AB Bear in mind that this dot is sometimes omitted ie ABOR gate

The OR gate is an electronic circuit that gives a high output (1) if one or more of its inputs are high A plus (+) is used to show the OR operationNOT gate

The NOT gate is an electronic circuit that produces an inverted version of the input at its output It is also known as an inverter If the input variable is A the inverted output is known as NOT A This is also shown as A or A with a bar over the top as shown at the outputs The diagrams below show two ways that the NAND logic gate can be configured to produce a NOT gate It can also be done using NOR logic gates in the same way

NAND gate

This is a NOT-AND gate which is equal to an AND gate followed by a NOT gate The outputs of all NAND gates are high if any of the inputs are low The symbol is an AND gate with a small circle on the output The small circle represents inversion

NOR gate

This is a NOT-OR gate which is equal to an OR gate followed by a NOT gate The outputs of all NOR gates are low if any of the inputs are highThe symbol is an OR gate with a small circle on the output The small circle represents inversion

EXOR gate

The Exclusive-OR gate is a circuit which will give a high output if either but not both of its two inputs are high An encircled plus sign ( ) is used to show the EOR operation

EXNOR gate

The Exclusive-NOR gate circuit does the opposite to the EOR gate It will give a low output if either but not both of its two inputs are high The symbol is an EXOR gate with a small circle on the output The small circle represents inversion The NAND and NOR gates are called universal functions since with either one the AND and OR functions and NOT can be generated

Note A function in sum of products form can be implemented using NAND gates by replacing all AND and OR gates by NAND gates A function in product of sums form can be implemented using NOR gates by replacing all AND and OR gates by NOR gates

Logic gate symbols

Table 2 is a summary truth table of the inputoutput combinations for the NOT gate together with all possible inputoutput combinations for the other gate functions Also note that a truth table with n inputs has 2n rows You can compare the outputs of different gates

Logic gates representation using the Truth table

Example

A NAND gate can be used as a NOT gate using either of the following wiring configurations

Subject Eng Literature (The Tempest ndash William Shakespeare) Topic Act III Scene 3 Lines 53 to 110 (End of the scene) Date 16th April 2020 (2nd Period)

[Students should read the original play and also the paraphrase given in the school prescribed textbook]Summary Questions amp Answers

o Seeing this strange scene all are inclined to believe the tales told by travelers that there truly are ldquounicornsrdquo and ldquothe phoenixrsquo thronerdquo

o As they are about to sit down to the feast the banquet is snatched away by a harpy (Ariel disguised) A spiritrsquos voice (Arielrsquos voice) denounces Alonso Sebastian and Antonio with particular

1 ARIEL You are three men of sin whom Destiny

(Line 53-58)That hath to instrument this

lower world And what is int the never-surfeited sea

Hath caused to belch up you and on this island

Where man doth not inhabit you rsquomongst men

Being most unfit to live I have made you mad

reference to their crime in expelling Prospero from Milan They have not received any punishment for their deed earlier but the time for their punishment has arrived Upon Alonso it pronounces ldquolingering perdition worse than deathrdquo from which there is no remedy except through sincere repentance Ariel then vanishes in thunder and the shapes enter again and carry away the table

o Prospero watching invisibly is very pleased with the performance of Ariel and his (Prosperorsquos) ldquomeaner ministersrdquo All his enemies are now in his power and are in a fit of desperation He then leaves them and goes to see how Ferdinand and Miranda are getting on

o Alonso is now much humbled and penitent with the after effect of the spiritrsquos denunciation of his crimes He believes that his son is lost forever After this all disperse being stricken mad by the speech of the spirit

o Gonzalo fearing that they may do violence to themselves or to one another follows them and bid others to follow

(a) To whom does Ariel disguised as a harpy call the three sinners What game did Fate of Destiny play with

them

The three sinners called by Ariel are Alonso Sebastian and Antonio It was Destiny which had caused the ocean to cast the three sinners on the shore Though the ocean is all the time devouring whatever appears on its surface and is never satisfied with its continual swallowing of the ships and men in the present case the ocean had cast these three sinners on the shore without killing them

(b) Who had jointly been responsible for the conspiracy against Prospero What is Prosperorsquos purpose behind all this

Three men Alonso Sebastian and Antonio had jointly

been responsible for the conspiracy against Prospero They had driven out Prospero form Milan Prosperorsquos purpose is to make these three sinners realize the wrong they had done He wants them to repent for their criminal deeds because repentance leads to self-esteem(c )What does Ariel (the harpy) tell Alonso and his companions when they take out their swords to attack him

Seeing them drawing their swords Ariel (harpy) tells them that he and his companions are the instruments of destiny and that it is not possible for human beings to do them any injury He says that the swords of human beings can not injure even a minute part of his feathers Their swords are as ineffective against him and his companions as against the wind or the water

(d) Give the explanatory meanings of the following expressions in the context of the above extract

(i)Never surfeited (ii) Belch up (iii) lsquomongst men

(i) Never surfeited never led to satisfaction

(ii) Belch up cast ashore(iii) lsquomongst men in human

society2

I and my fellows (Line 60-65)

Are ministers of Fate The elementsOf whom your swords are tempered may as wellWound the loud winds or with bemocked-at stabsKill the still-closing waters as diminishOne dowl thats in my plume

IMPORTANT PASSAGES EXPLAINED

The elements

(Line 61-66)Of whom your swords are tempered may

as wellWound the loud winds or with

bemocked-at stabs

(a) Who is lsquoIrsquo Who are his lsquofellowsrdquo

lsquoIrsquo is referred to Ariel in disguise of a harpy His lsquofellowsrsquo are other spirits serving Prospero the real Duke of Milan who has acquired supernatural powers after being banished from his Dukedom Prospero has settled in this uninhabited island

(b) What are the elements that have temperrsquod the swords Why will it not work against the speaker

The swords (of Alonso and his companions) are tempered by metal (steel) which is taken out of the earth and refined by

Kill the still-closing waters as diminishOne dowl thats in my plume My fellow

ministersAre like invulnerable

In these words Ariel reminds the King and his companions of the utter futility of drawing swords against himself and his fellows Ariel drives Alonso Antonio and Sebastian the three men of sin to desperation ndash a state in which men do violence to themselves They draw swords to strike Ariel But Ariel reminds them that he and the other spirits are the ministers of destiny and nothing can wound them The steel of which their swords are made of may cut the wind or water which being divided always closes up again Even supposing that such things may be possible it is quite impossible that their swords will cut one feather in their plume They are incapable of being wounded by any sword of man Hence it is foolish on their part to attempt to strike at Ariel and his fellow-spirits

For which foul deed

(Line 72-75)The powers delaying not forgetting

haveIncensed the seas and shores yea all the

creatures Against your peace

Ariel enters like a harpy and remaining invisible tells Alonso Sebastian and Antonio that he and other harpies are the agents of Destiny appointed to carry out her decrees He tells them that their punishment for the crime against Prospero which has been so long deferred is now to fall upon them He reminds them that they had expelled Prospero from Milan and set him and his innocent child adrift on the sea and that the sea had paid them back for their sin by the shipwreck and by the calamities they have suffered He tells them that the powers above which did not forget this mean treachery but only deferred the punishment have now engaged the seas and the shores and all living beings including him and his comrades against them The very elements and supernatural agency Ariel adds have taken up the avenging of their crime against Prospero

the action of fire It may cut the wind or water which being divided always closes up again

The sword will not work against the spirits and the harpy because they are the ministers of destiny and nothing can wound them nor it will cut a single feather in their plume

(c )What is the meaning of lsquodowlrsquo in the last line

The term lsquodowlrsquo means a filament or the smallest part of a feather In this context Ariel in disguise of harpy says that their sword cannot even damage the smallest filament of their (Arielrsquos and other spirits) feathers as they are incapable of being wounded by any sword of man

(d) What does the speaker remind the listeners about

Ariel in disguise of harpy reminds Alonso the King of Naples Sebastian Alonsorsquos brother and Antonio the present Duke of Milan and the treacherous brother of Prospero as they being three men of sin He even reminds them that their punishment for their crime against Prospero which has been so long deferred now falls upon them He reminds them that they have expelled Prospero from Milan and has set him along with his innocent infant daughter adrift on the sea So the sea has paid them back for their sin by their shipwreck and the calamities they have suffered since then The harpy rebukes Alonso of his sin that has incensed the Gods and has deprived him of his son as a punishment

(e) How do they respond

When Ariel in disguise of a harpy reminds Alonso Sebastian and Antonio of their past misdeeds and sin Alonso has a look of terror and confusion in his eyes He utters the words of sincere repentance wrung out of his conscience-stricken heart It appears to him that all the elements of nature the sea-waves the wind and the thunder proclaiming a loud voice in the name of Prospero and the crime Alonso has committed against him They are calling upon him to repent There is a deep storm raging in Alonsorsquos breast and the echoes of that storm are ringing in his ears like a clear note of wind-instrument A note of denunciation of Alonsorsquos crime leaves him much humbled and penitent and confirms his belief that his son is lost forever But Sebastian and Antonio shows some courage instead of repentance They wish to kill the spirits or devils if it appears

3

Of my instruction hast thou nothing bated (Line 85-93)

In what thou hast to say So with good life

And observation strange my meaner ministers

Their several kinds have done My high charms work

And these mine enemies are all knit upIn their distractions They now are in my

powerAnd in these fits I leave them while I visitYoung Ferdinand whom they suppose is

drownedAnd his and mine loved darling

Methought the billows spoke and (Line 96-99)

told me of itThe winds did sing it to me and the

thunderThat deep and dreadful organ-pipe

pronouncedThe name of Prosper It did bass my

trespass

These are the words of contrition coming from Alonso Ariel has driven him to a deep repentance for conspiring with Antonio against Prospero He now feels a sincere remorse It appears to him that all the elements of nature the sea-waves the wind and the thunder proclaimed with a loud voice the name of Prospero and the crime Alonso had committed against him They are calling upon him to repent There is a deep storm raging in Alonsorsquos breast and the echoes of that storm are ringing in his ears like the clear note of a wind-instrument

Comment These are the words of sincere repentance wrung out of the conscience-stricken heart of Alonso Alonso who is the lesser villain is the first to give way to remorse under the effect of Arielrsquos speech The words of Ariel seem to him to be the voice of conscience speaking to him He is driven to desperation a state in which he might do violence to his life

(a) Identify the speaker State the context

Prospero the ruler of the island is the speaker The famous banquet scene has been enacted very well Ariel and his junior spirits have played their roles excellently Prospero is glad to say words of praise for them(b) In what way the speakerrsquos instructions have been carried out

According to Prosperorsquos instructions a banquet was presented before the King of Naples and his companions when they were tired and hungry Just when they were preparing to eat the feast the banquet was suddenly removed by exercising supernatural powers All this was done by Ariel Prosperorsquos chief assistant and a powerful spirit

Ariel not only made the feast disappear but also delivered his speech blaming the King and his two companions for their past wicked deeds He warned them to repent for their misdeeds or suffer forever on that uninhabited island

(c) Who are referred to as lsquomeaner ministersrsquo What have they done

Prospero refers as lsquomeaner ministersrsquo to his other lesser spirits who were assisting Ariel in presenting a scene before the kingrsquos party They entered the scene to the accompaniment of music They assumed several strange shapes and brought in a banquet Then they danced about it with gentle actions of salutations thus inviting the King and others to eat the feast

These spirits play their role again when Ariel in the shape of a harpy quits the scene These shapes enter again and dancing with mocking gestures carry away the table

(d) Who are the speakerrsquos enemies What has happened to them

King of Naples Alonso his brother Sebastian and the present Duke of Milan Antonio (Prosperorsquos own brother) are Prosperorsquos enemies With the turn of events they have all been washed ashore on the island which is ruled by Prospero the great magician Actually this happened after the shipwreck caused by a storm which was raised by Prospero with the purpose of bringing these people to his island Prosperorsquos spirits have already confused and terrified these enemies and they are under Prosperorsquos control He can treat them as he likes

(e) What does he say about Ferdinand Explain what is meant by ldquohellip his and mine darlingrdquo

Prospero knows that Alonsorsquos son prince Ferdinand is alive though his father thinks that the prince has been drowned

Prospero refers to his daughter Miranda who is dear to him She is also very dear to Prince Ferdinand who has fallen in love with her They are waiting to be married soon for which they have received Prosperorsquos consent

4

ALONSO O it is monstrous monstrous (Line 95-102)

Methought the billows spoke and told me of it

The winds did sing it to me and the thunderThat deep and dreadful organ-

pipe pronouncedThe name of Prosper It did bass

my trespassTherefore my son ithrsquo ooze is

bedded andIll seek him deeper than eer

plummet soundedAnd with him there lie mudded

(a) In what way does Alonso express his horror when his conscience is awakened by Arielrsquos words

When Alonsorsquos conscience is awakened by Arielrsquos words he expresses his horror at what he has heard He gets the feeling that the waves of the ocean the wind and the loud thunder have spoken to him and uttered the name of Prospero Because of being reminded of his crime in a very loud and rough voice he comes to realize that he has lost his son for his past misdeeds

(b) What does Alonso imagine about his son What does Alonso want to do in his desperate state

Alonso imagines that his son is lying in the mud at the bottom of the sea He feels desperate that he wants to drown himself in the ocean deeper than the plumb-line has ever gone He wants to lie with his son at the bottom of the sea

(c) How do Sebastian and Antonio want to face the evil spirits

Sebastian says that he is not at all afraid of what the harpy has said and that he is prepared to fight any number of such monsters if they appear before him only one at a time Antonio says that he would support Sebastian in the fight against the fiendsyyy

(d) Why does Gonzalo ask Adrian to follow the three men

Gonzalo tells Adrian that all the three men namely Alonso Sebastian and Antonio are in a wild and reckless mood The thought of the heinous crime of which they are guilty has begun to torment their minds So he asks Adrian to follow those three men without loss of time and prevent them from doing anything which the turmoil in their minds might lead them to do

(e) What opinion do you form of Alonso from the above extract

Alonso who is the lesser villain is the first to give way to remorse under the effect of Arielrsquos speech The words of Ariel seem to him to be the voice of conscience speaking to him He is driven to desperation a state in which he might do violence to his life

Subject =Accounts

Ac-12 15420 topic-pL Appropriation ac

PROFIT AND LOSS APPROPRIATION ACCOUNT

MEANING AND PREPARATIONProfit and Loss Appropriation Account is merely an extension of the Profit and Loss Account of the firm The profit of the firm has to be distributed amongst the partners in their respective profit sharing ratio But before its distribution it needs to be adjusted All Adjustments like partnerrsquos salary partnerrsquos commission interest on capital interest on drawings etc are made in this account These adjustments will reduce the amount of profit for distribution This adjusted profit will be distributed amongst the partners in their profit sharing ratio To prepare it at first the balance of Profit and Loss Account is transferred to this account The journal entries for the preparation of Profit and Loss Appropriation Account are given below

1 for transfer of the balance of Profit and Loss Account to Profit and Loss Appropriation Account

(a) In case of Net Profit

Profit and Loss Ac helliphelliphelliphelliphellipDrTo Profit and Loss Appropriation Ac(Net Profit transferred to Profit and Loss Appropriation Ac)

(b)In case of Net Loss

Profit and Loss Appropriation Achelliphelliphellip DrTo Profit and Loss Ac(Net Loss transferred to Profit and Loss Appropriation Ac)

2 for Interest on Capital

For transferring on Interest on CapitalProfit and Loss Appropriation Achelliphelliphellip DrTo Interest on Capital Ac(Interest on capital transferred to Profit amp Loss Appropriation Ac)

3 for Interest on Drawings

For transferring Interest on Drawings Interest on Drawings Achelliphelliphelliphelliphelliphellip DrTo Profit and Loss Appropriation Ac(Interest on drawing transferred to Profit amp Loss Appropriation Ac)

4 For Partnerrsquos SalaryFor transfer of partnerrsquos SalaryProfit and Loss Appropriation Achelliphellip DrTo Salary Ac(Salary transferred to profit amp Loss Appropriation Ac)

5 For Partnerrsquos CommissionFor transferring commissionProfit and Loss Appropriation Achelliphelliphellip DrTo Commission Ac(Commission transferred to Profit and Loss Appropriation Ac)

6 For Transfer of agreed amount to General ReserveProfit and Loss Appropriation Ac helliphellipDrTo General Reserve Ac(Transfer to General Reserve)

7 for share of Profit or Loss appropriation(a) If ProfitProfit and Loss Appropriation Achelliphellip DrTo Partnerrsquos CapitalCurrent Ac(Profit transferred to capitalcurrent Ac)(b) If LossPartnerrsquos Capital Current Achelliphelliphelliphellip DrTo Profit and Loss Appropriation Ac(Loss transferred to capitalcurrent Ac)

THE FORMAT OF PROFIT AND LOSS APPROPRIATION

Profit and Loss Appropriation Account for the year endedhelliphelliphelliphellip

Particulars Amount Particulars Amount

To PL Ac (loss) By pL Ac (profit)

To Interest on capital BY Interest on drawings

To partner`s commission by Partner`s capital Ac ( loss)

To Partner`s salary To Interest on partner`s loan To General Reserve To Partner`s Capital AC (Profit)

Subject= Economics

MOVEMENT ALONG THE DEMAND CURVE (CHANGE IN QUANTITY DEMANDED)In law of demand you have already studied the inverse relationship between price and quantity demanded When quantity demanded of a commodity changes due to change in its price keeping other factors constant it is called change in quantity demanded It is graphically expressed as a movement along the same demand curve There can be either a downward movement or an upward movement along the same demand curve Upward movement along the same demand curve is called contraction of demand or decrease in quantity demanded and downward movement along the same demand curve is known as expansion of demand or increase in quantity demanded

Extention of demandd

price (rs)p A

B Extentionp1 d

Q Q1

Quantity demanded ( in units)

Contraction of demandd

p2 Ccontraction

p APrice (Rs)

d

Q2 Q

Quantity demanded (in units)

Explanation of movement of demand A fall in price from OP to OP1 leads to increase in quantity demanded from OQ to OQ1 (expansion of demand) resulting in a downward movement from point A to point B along the same demand curve DD When Price rises from OP to OP2 quantity demanded falls from OQ to OQ2 (contraction of demand) leading to an upward movement from point A to point C along the same demand curve DD

  • Activity Series of Metals
    • Drawbacks of Rutherfordrsquos model of atom
      • Electromagnetic radiations
      • Properties of electromagnetic radiations
      • Characteristics of electromagnetic radiations
        • Plancks Quantum Theory-
        • Photoelectric effect
          • Intext Questions
            • Logic gates
            • Digital systems are said to be constructed by using logic gates These gates are the AND OR NOT NAND NOR EXOR and EXNOR gates The basic operations are described below with the aid of truth tables
            • AND gate
            • Example
Page 30:  · Web viewSubject. Topic. Summary. Execution. English 1 . Chapter 1 naming words . Page 8. Write the names of these pictures:- Person:-1. father. 2.Firefighter 3.doctor 4 ...

unicellularorganisms But there is also another

problem Allunicellular organisms are not similar kind The cellular structure of prokaryotes is verydifferent from that of other organismsEukaryotes possess a true nucleus and allcell organelles that are not present inprokaryotes So the fourth kingdom Monerais introduced which include unicellular prokaryotes (bacteriaamp blue green algae)

bull Still some problem arise in kingdomplantae

So in 1969 R H Whittakar proposedanew five kingdom System of classification

i) Kingdom Monera - unicellular prokaryotes

ii) kingdom Protista - unicellular eukaryotes

iii) Kingdom Fungi - uni or multicellular fungi with cell wall but without chlorophyll

iv) Kingdom Plantae - Multicellular Plants

v) Kingdom Animalia - Multicellular Animals

EVS Chapter 1 ndash Modes of Existence

An agricultural society

An agricultural society also known as an agrarian society is a society that constructs social order around a reliance upon farming More than half the people living in that society make their living by farming

People in an agricultural society generally lead a more settled lifestyle than those in nomadic hunter-gatherer or semi-nomadic pastoral societies because they live permanently near the land that is farmed Agricultural settlements tend to develop in areas of convenience near bodies of water which is used for both crops and transportation or along trade routes Not everyone in an agricultural society is a farmer Some people make a living trading or making and selling goods such as tools used for farming

Another way to define an agrarian society is to see the total amount of production in a nation In an agrarian society cultivating the land is the main source of wealth Such a society can recognize other means of subsistence and work habits but emphasizes the importance of agriculture and livestock Agrarian societies have existed in various parts of the world for 10000 years and continue to exist today They have been the most common form of socio-economic organization for most of recorded human history

Q) Write the features of agricultural society

Ans - Structure and Features of Agrarian Society1 Occupational Structure

An agrarian society is generally associated with the domestication of plants and animals The domestication of plants means farming and that of animals means herding Often there is mixture of farming and the use of such domesticated animals as cow goat and sheep

2 Forms of Land Ownership in Agrarian SocietiesGenerally there are landlords supervisory farmers cultivators and share croppers The landholders own the land but do not work on it They let it out for sharecropping The supervisory farmers are those who live by having their land cultivated by hired labourers The cultivators cultivate the land for themselvesThe share-croppers are those who live by tilling other peoplersquos land or a crop-sharing basis The artisans own their means of production and produce by their own labour in their homesteads

3 Village Community System An agrarian society is highlighted by

the institution of village community system The agrarian economy made fixed dwelling houses necessary Living close together for protection and co-operation and living nearer to the land gave birth to agricultural villages The village is not only the residential place of farmers it is also the social integrator

4 Minimal Division of Labour Another structural feature of agrarian society is a minimal division of labour Except for the basic division founded on age and sex differences there are few specialized roles There is only one predominant type of occupation ie domestication of plants and animals For all the people the environment physical as well as social is the same

5 Role of Family The farm family is of the patriarchal type the father is the final arbiter in most of the familyrsquos major decisions The life of ail men and women is merged in family life Since there are not many special organizations family is the only organisation to perform the tasks of aid and protection

6 Sense of Unity The members of an agrarian society exhibit a strong in-group feeling Since the whole of their social lives is wrapped up in a society which is physically economically and socially homogenous they are inclined to view the entire outside world as an out group

7 Informal Social Control An agrarian society is regionally divided into villages In a village community the force of traditional mores is more dominant than in the urban community In the village everybody is known to everybody The members in a village community help each other and share the joy and sorrows of each other Crime in an agrarian society is rare

8 Simplicity and Uniformity Life of the people in an agrarian society is marked by simplicity and uniformity Their main occupation is agriculture which largely depends upon the vagaries of nature An agrarian society is a religious society

Math Compound angles Compound angles The algebraic sum of two or more angles is called a compound angle If A B C be three angles then A+B B+C C+A A-B B-C A-C A+B-C etc are compound angles In this chapter we shall discuss the trigonometrical ratios of compound angles Theorem 1 If A B and A+B are all pisitive acute angles theni) sin( A+B) = sin A cos B + cosA sinBii) cos(A+B) = cosA cosB- sinA sinBTheorem 2If A and B are positive acute angles and AgtB theni) sin(A-B) = sin A cosB- cos A sinBii) cos(A-B) = cos A cos B+ sin A sin BTo prove that i) sin(A+B) sin (A-B) = sin2 A - sin2 B = cos2 B- cos2 A

Example 1 Prove that tan70deg=2tan50deg+tan20degSolutiontan70deg = tan(50deg + 20deg)Or tan70deg=(tan 50deg+tan 20deg)(1-tan50degtan20deg) or tan70deg (1 ndash tan 50deg tan20deg) = tan50deg+tan20degor tan70deg= tan70deg tan50deg tan20deg+ tan50deg + tan20deg = cot20deg tan50deg tan20deg + tan50deg + tan20deg = 2 tan50deg+ tan20degExample 2 If A + B = 45deg show that (1 + tanA) (1 + tanB) = 2Solutiontan(A + B) =( tan A + tan B) (1 - tan

ii) cos(A+B) Cos(A-B) = cos2 A- sin2 B = cos2 B -sin2 AProof i) LHS= sin(A+B)sin(AminusB) [Recall sin(αminusβ)=sinαcosβminuscosαsinβ And sin(α+β)=sinαcosβ+cosαsinβ]= (sinAcosB+cosAsinB)times(sinAcosBminuscosAsinB)= sin2Acos2Bminuscos2Asin2B [Recall sin2α+cos2α=1 From above we can then assume correctly that sin2α=1minuscos2α AND cos2α=1minussin2α] = sin2A(1minussin2B)minussin2B(1minussin2A) = sin2Aminussin2Asin2Bminussin2B+sin2Asin2B = sin2Aminussin2B= 1-cos2A-(1-cos2B) = cos2 B- cos2 A = RHSii)LHS= cos (A+B) cos(A-B) [ cos(A+B) = cos AcosB- sinAsinBCos(A-B) = cosAcosB+ sinAsinB]= cos2 A Cos2 B- sin2 A Sin2 B= cos2 A( 1-sin2 B) - (1- cos2 A) sin2 B= cos2 A- cos2 A sin2 B- sin2 B+ cos2 A sin2 B=cos2 A- sin2 B=1- sin2 A-(1-cos2 B) = cos2 B- sin2 A= RHSTangent formulae for compound anglesi)tan (A + B) = tan A + tan B1-tan A tan Bii)tan (A ndash B) = tan A-tan B1+tan A tan Biii) cot (A + B) = cot Acot B-1cot A+cot B(viii) cot (A ndash B) = cot Acot B+1cot B-cot A

A tan B) Or 1= (tan A+ tanB) (1-tan A tanB) Or tanA + tanB + tanA tanB + 1 = 1 + 1Or tanA (1 + tanB) + (1 + tanB) = 2Or (1 + tanA) (1 + tanB) = 2Example 3 Find the value of sin 15degSolution sin 15deg= sin(45deg-30deg) = sin45degcos 30deg- cos45degsin30deg =(1radic2) (radic32) -(1radic2) (12) = (radic3-1) 2radic2Example 4 If sin A = 1 radic10 and sin B = 1 radic5 where A and B are positive acute angles then what is A + B SolutionWe know that sin (A + B) = sin A cos B + cos A sin B= [1 radic10] [radic(1 minus 1 5)] + [1 radic5] radic(1 minus 1 10)= [1 radic10] [radic4 5] + [1 radic5] [radic9 10]= [1 radic50] times (2 + 3)= 5 radic50 = 1 radic2

sin (A + B) = sin π 4rArrHence A + B = π 4Example 5 If A + B = 225o then find [cot A] [1 + cotA] times [cot B] [1 + cot B]Solution[cot A] [1 + cotA] times [cot B] [1 + cot B] = 1 [(1 + tan A) times (1 + tan B)]=1 [tan A + tan B + 1 + tan A tan B] [ tan (A + B) = tan225o]∵

tan A + tan B = 1minus tan A tan BrArr= 1 [1 minus tan A tan B + 1 + tan A tan B]= 1 2

COMMERCE

CLASSIFICTION OF HUMAN ACTIVITIES-ECONOMIC AND NON-ECONOMIC

Firstly we shall recall the previous class for 5 mins especially for the absentees and for also the rest of the students who were there

Today at first we briefly discuss the earlier portions of the chapter

1Business-It includes all those economic activities which are concerned with production and exchange of goods and services with the object of earning profit Example A factory shop beauty parlour also business enterprises

2Profession ndashThe term profession means an occupation which involves application of specialized knowledge and skills to earn a living For Example Chartered Accountancy medicine law tax consultancy are example of professions

Questions1What are the main features of ProfessionAnswer The main features of a profession are as follows a Specialised body of knowledge-Every profession has a specialised and systematised body of knowledge b Restricted entry- Entry to a profession is allowed only to those who have completed the prescribed education and have the specialised examination c Formal education and training ndashA formal education and training is given to the person who wants to acquire the professional

3Employment-Employment mean an economic activity where people work for others in exchange for some remuneration (salary)The persons who work for others are called lsquoemployeesrsquo The persons or organizations which engage others to work for them are called lsquoemployersrsquoEg A doctor working in a hospital is employment as he is working for a salaryA lawyer may serve as a law officer in a bank

With this we shall proceed with the features of both Profession amp Employment

The main features of a profession are as follow

a Specialised body of knowledge b Restricted entry c Formal education and training d Professional association e Service motive f Code of contact

The main features of an employment are as follows

a In employment a person works for others called employer

b An employee provides personal service

c There is a service agreement or contract between the employee and the employer

d The employee has to obey the order of the employer

e No capital investment is made by the employer

Various examples of Employment are as follows

aA teacher teaching in a school or collegeb An engineer employed in Municipal Corporation of DelhicAn accountant working in the accounts department of a companydA doctor working in a hospital

Note In all the above examples of employment the individual who is involved in each example is working as an employee for a salary under an employer

qualification(MBBSCALLB)d Service motive ndashProfessionals are expected to emphasis service more on their clients rather than economic gain f Code of Conduct-The activities of professionals are regulated by a code of conduct

2 What are the main features of EmploymentAnswer The main features of an employment are as followsa In employment a person works for others called employerb An employee provides personal servicec There is a service agreement or contract between the employee and the employerd The employee has to obey the order of the employere No capital investment is made by the employer

3 Give various Professions and their respective Association are given below

Professions

Professional

Professional association

Medical profession

Doctor Medical Council of India

Law profession

Lawyers Bar Council of India

Accounting Profession

Chartered

The Institute of Chartered Accounts of India( ICAI)

Engineerin Engineers The

g Profession

institute of Engineers (India)

Accounts Basic accounting terms

Today we will give you some questions from the previous study material

Questions6) Define accounting7) What do you mean by debit

and credit8) Explain the types of account9) Define the following terms

a) Assetsb) Capitalc) Purchased) Debtorse) Transactions

10) Name the types of accounts given below

a) Krishnas accountb) Machinery accountc) Royalty accountd) Salary accounte) Furniture accountf) Audit fee account

Economics Basic Economic ConceptsSub topic

UTILITY

Before starting todayrsquos class we shall recall the last class which was about UTILITY AND THE FEATURES OF UTILITY

Now we shall proceed with the further topics of the chapter

Todayrsquos topic from the chapter lsquo Basic Economic Conceptsrsquo will be TOTAL UTILITY amp MARGINAL UTILITYNow let us quickly revise the concept of utility with an example ie goods and services are designed because they have an ability to satisfy human wantsThis feature of being able to satisfy human wants is termed as utility For example we derive utility from WiFi services as it gives us satisfaction by connecting us to our friends and family through social media here consumers derive utility from WiFi services

From the above concept we shall start with todayrsquos topicEconomists have defined TOTAL UTILITY (TU) as the total satisfaction obtained by consuming a given total amount of a good and serviceFor example the total satisfaction obtained from eating 10 mangoes is the total utility of 10 mangoes

MARGINAL UTILITY (MU) is the additional satisfaction derived from each additional unit

Questions1 What is Total Utility (TU)

Answer Total Utility (TU) is the

aggregate of the utility that a consumer derives from the consumption of a certain amount of a commodityTU=MU1+MU2++MUn

2 What is Marginal UtilityAnswer

Marginal Utility (MU) is the additional made to the total utility as consumption is increased by one more unit of the commodityMU= TUn ndashTUn-1

NoteOften economists tend to

subdivide utility into an imaginary unit called UTIL

consumed In this casethe utility obtained from each mango as it is consumed as the MU of that mango It is also defined as the addition made to the total utility when an additional unit is consumed Often economists tend to subdivide utility into an imaginary unit called UTIL

Note As a consumer increases the consumption of a good over period of time the total utility or total satisfaction derived from it increases to appoint and thereafter it decreasesHowever as the consumer keeps on consuming the good the marginal utility or the additional utility derived from it decreases

SubjectBusiness studies

Topic

BUSINESSENVIRONMENT

Summary

Now quickly let us revise the earlier points that we have already done in the last class and let us proceed with the other topics that are there in the chapter

Firstly we will recall the internal and external factors of micro environment and then we shall proceed in details

Meaning and list of internal and external factors

aInternal factorsInternal factors refer to all the factors existing within a business firm The internal factors are considered controllable because the enterprise has control over these factorsFor an example a company can alter its organization structure policies programmes employees physical facilities and marketing mix to suit the changes in the environmentList of internal factors areCorporate culture mission and objectives top management organizations structure company image and brand equity company resources

b External factorsExternal factors refer to those individual and groups and agencies with which a particular business organization comes into direct and frequent contact in the course of its functioningThese individuals and groups are known as STAKEHOLDERS because they have a stake (financial interest ) in the working and performance of the particular business List of external forces (stakeholders)Customers competitors investors suppliersmiddlemen (marketing intermediaries)

Execution 1 What do you mean by internal

factors in micro environmentAnswerInternal factors refer to all the factors existing within a business firm The internal factors are considered controllable because the enterprise has control over these factorsFor an example a company can alter its organization structure policies programmes employees physical facilities and marketing mix to suit the changes in the environment

2 What do you mean by external factors in micro environment

AnswerExternal factors refer to those individual and groups and agencies with which a particular business organization comes into direct and frequent contact in the course of its functioningThese individuals and groups are known as STAKEHOLDERS because they have a stake (financial interest) in the working and performance of the particular business

3Who are stakeholdersSTAKEHOLDERS are individuals and groups who have a stake (financial interest ) in the working and performance of the particular business 4Discuss the internal factors in briefa Corporate CultureThe values beliefs and attitudes of the founders and top management of the company exercise

financers publics

customers

suppliersfinancers

competitors

middlemen

publics

Fig STAKEHOLDERS OF A COMPANY

Apart from micro environment the other main dimension of business environment isMacro environment Macro environment refers to the general environment or remote environment within which a business firm and forces in its micro environment operateA company does not directly or regularly interact with the micro environmentTherefore macro environment is also known as indirect action EnvironmentThe macro environment forces are less controllable than the micro forces

Macro environment consists of the following components

POLITICAL AND LEGAL ENVIRONMENT

ECONOMIC SOCIAL AND ENVIRONMENT

CULTURAL

ENVIRONMENT

TECHNOLOGICAL ENVIRONMENT

a strong influence on what the cmpaany stands for how it does things and what it considers importantbMission and objectivesThe business philosophy and purpose of a comoany guide it prioritiesbusiness strategiesproduct market scope and development scope

cTop management structurethe composition of board of directors the degree of professionalization of management and the organizational structure of a company have important bearing on its business decisions

dPower structureThe internal power relationship between the board of directors and the chief executive is an important factor

eCompany image and brand equityThe image and brand equity of the company play a significant role in raising finance forming alliance choosing dealers and suppliers launching new products entering foreign markets

5 What is Macro environmentAnswerMacro environment refers to the general environment or remote environment within which a business firm and forces in its micro environment operateA company does not directly or regularly interact with the micro environmentTherefore macro environment is also known as indirect action EnvironmentThe macro environment forces are less controllable than the micro forces 6 What are the components of macro environmenta Political and legal environmentb Economic environmentc Social and cultural environmentd Technological environment

BUSINESS FIRM

Fig COMPONENTS OF MACRO ENVIRONMENTPolitical science

Introduction to political science

Comparative politics and itrsquos scope Comparative politics is the second major dimension of political scienceIt is also a very vast area of study and a very large number of political scientists even treat it as an autonomous area of study within the board ambit of political scienceScope of comparative politics-

1 All political structures -Comparative politics includes the study of all structures formalnon formal governmental and extra governmental which are directly or indirectly involved in politics in all the countries of the world

2 Functional studies- Comparative politics seeks to study politics less from the point of view of the legal institutions in terms of their powers and move from the point of view of their functions which constitute the political process and their actual Operation in the environment

3 Study of political behaviour- Another important part of its scope is the study of the actual behaviour of the people in the process of politics

4 Study of similarities and differences- comparative politics also undertakesan analysis of the similarities and differences among political process and functions

5 Study of all political systems -comparative politics seeks to analyse the actual behaviour and performance of all political systems western as well as non western

6 Study of the environment and infrastructure of politics-The study of politics demands a study of the psychological sociological economic and anthropological environment in fact the social environment as a whole in which each political system operates

7 Study of political culture- political culture is composed of attitudesbeliefs emotions and values of a society that relate to the political system or politics

8 Study of political participation- Political participation is a universal processThe only difference is that while in some states it is limited in others it is wider

9 Study of political process- political

Answer the following questions-

What is comparative politics

What are the scope of comparative politics

Homework- learn

processes like decision makingpolicy making judicial process leadership recruitment process and others are always at work in all political systems

The scope of comparative politics is very comprehensive It includes everything that falls within the area of political activity and political process

History CAMBRIDGE VIEW ABOUT

THE PARTITION

AND REFUTATION

OF CAMBRIDGE

VIEW

Cambridge view about the Partition The Cambridge school of historians have interpreted that opposition to partition scheme was made entirely by the elitist groups They hold the view that Lord Curzon planned to partition the Bengal for administrative purposeREFUTATION OFCAMBRIDGE VIEW The Rationalist historians have rejected the interpretations of the Cambridge School of historians on various grounds

1 QUESTION State different views of historians regarding Partition of Bengal

ANSWER Cambridge historians believed that Lord Curzon partitioned Bengal for administrative reasons only and not for the political motive The Middle class elitist group protested because of their petty interest The Hindu zamindars protested as they have to spend more money for managing their estatesThe lawyers of Calcutta High court feared to lose their clientBut according to the nationalist Historians was-

2- The ultimate object of Lord Curzon was to crush the unity of Bengal politicians

3- If Bengal becomes a separate province Bengali speaking 16 million people of western part would become minority under Hindi speaking people of Bihar and Oriya speaking people of Orissa

4- The bureaucrats expected that the protest movement would die down quickly

5- Lord Curzon used the Muslim community in his political game

6- Idealism had great contribution in the protest against partition

7- The people of the every section of society were affected by the partition of Bengal

Computer Science

Numbers Convertion of dcimal number to octal numberThe decimal numeral system is the standard system for denoting integer and non-integer numbers It is the extension to non-integer numbers of the Hindu-Arabic numeral system For writing numbers the decimal system uses ten decimal digits a decimal mark and for negative numbers a minus sign - The decimal digits are 0 1 2 3 4 5 6 7 8 9 the decimal separator is the dot in many countries

The octal numeral system or oct for short is the base-8 number system and uses the digits 0 to 7 Octal is sometimes used in computing instead of hexadecimal perhaps most often in modern times in conjunction with file

permissions under Unix systems It has the advantage of not requiring any extra symbols as digits It is also used for digital displays

Follow these steps to convert a decimal number into octal form

1 Divide the decimal number by 82 Get the integer quotient for the next iteration (if the number will not divide equally by 8 then round down the

result to the nearest whole number)3 Keep a note of the remainder it should be between 0 and 74 Repeat the steps until the quotient is equal to 05 Write out all the remainders from bottom to top This is the solution

For example if the given decimal number is 8453

Division Quotient Remainder

8453 8 1056 5

1056 8 132 0

132 8 16 4

16 8 2 0

2 8 0 2

Then the octal solution is 20405

Subject Eng Literature (The Tempest ndash William Shakespeare) Topic Act I Scene 1 Lines 33 to 67 (End of scene) Date 16th April 2020 (4th Period)

[Students should read the original play and also the paraphrase given in the school prescribed textbook]Summary Questions amp Answers

[SUMMARY OF THE ENTIRE SCENE]

o The play starts with the scene of a severe storm at sea Alonso (King of Naples) Sebastian (Alonsorsquos brother) Ferdinand (Alonsorsquos son) Gonzalo Antonio (the usurping Duke of Milan) are in a ship in the midst of the storm

o The mariners are trying their best to control the vessel from running aground and are totally following the orders of their Master the Boatswain They have scant success

o The mariners become extremely unhappy and annoyed when most of the passengers arrive on the deck thereby hampering their effort to save the ship There is serious confrontation between them and the passengers who are part of the Kingrsquos entourage

o The mariners could not save the ship

SUMMING-UP

(i) Vivid description of the scene which gives a realistic description of terror and confusion of a tropical storm

(ii) Shows Shakespearersquos accuracy of knowledge in describing the naval operations and also matters of seamanship

(1) GONZALO Ill warrant him for drowning (L 45-57)

though the ship were no stronger than a nutshell and as leaky as an unstanched

wenchBOATSWAIN Lay her a-hold a-hold Set her two courses Off to

sea again lay her offMARINERS All lost To prayers to prayers All lostBOATSWAIN What must our mouths be coldGONZALO The king and prince at prayers Lets assist them

For our case is theirsSEBASTIAN Im out of patienceANTONIO We are merely cheated of our lives by drunkards

This wide-chopped rascal - would thou mightst lie drowning the washing of ten tides

(a) What does Antonio say at the insolent manners of the boatswain just before the given passage

Being irritated at the insolent manners of the boatswain just before the given extract Antonio the Duke of Milan calls him a worthless dog son of a woman without any morals an arrogant and disrespectful noisemaker He says that the boatswain deserved to be hanged(b) What statement does Gonzalo repeat about the boatswain

Gonzalo shows his faith that the boatswain is not destined to die by drowning He is destined to be hanged and nothing can alter this decree of destiny He says that even if the ship was as frail as a nutshell the boatswain could not be drowned for his destiny was to be hanged(c) What do the passengers do when they have lost all hope of their survival

When the passengers have lost all hope of survival they take

(iii) The opening scene justifies the title ndash The Tempest

UNANSWERED QUESTIONS

(i) The King always travels with his entire fleet including his soldiers Where were the other ships

(ii) Why was the ship in that area Where was it coming from or going where

(iii) The ship broke apart What happened to those who were in the ship

(We shall get the answer to the above questions as the play progresses)

leave of life with fervent prayers The mariners take their last hearty drink and are ready for death(d) What blame does Antonio put upon the mariners and the boatswain Antonio rebukes the mariners that these drunkards have brought them to the present crisis by neglecting their duties He blames them saying that they are going to lose their lives entirely for the negligence of the boatswain and his fellows(e) What does Antonio say while cursing the boatswain

Antonio gives vent to his wrath upon the boatswain in particular He calls the boatswain a wide-mouthed rascal who deserves to be hanged on the sea-shore at low water mark so that ten tides might wash over his body and take out of him all the liquor that he has been drinking

Class XIISubject Topic Summary ExecutionHistory Topic

1 1935 ACT AND WORKING OF PROVINCIAL AUTONOMYCONGREE AND OTHER MINISTERSSUB TOPIC GOVERNMENT OF INDIA ACT1935

Government of India Act 1935 This act established a lsquoFederation of Indiarsquo made of British Indian provinces and Indian states and provided for autonomy with a government responsible to the elected legislature in every provinceThis act introduced abolition of Diarchy at provinces The entire provincial administration was introduced to the responsible ministers who were controlled and removed by the provincial legislature The provincial autonomy means two things First The provincial governments were wholly responsible to the provincial legislature Secondly Provinces were free from outside control and interference in the large number of matters The act divided the powers between the centre and provinces in terms of three lists- Federal list( for centre) Provincial list (for province) and concurrent list (for both) Residuary powers were given to the viceroy In the election under the government of India Act the Congress swept the poll the mandate of the people came in favour of the congress so far as general Hindu seats were concerned The Congress did not get a single Muslim seates in Bombay CP UP Sind and BengalIn five provinces Congress had yhe clear majority In BengalNWFPAssam and Bombay Congress emerged as a single largest partyOn the other side the performance of the Muslim League was badThus the Congress formed ministers in 7 provinces out of 11 provinces Coalition ministry was also formed in two other provincesOnly BENGAL AND Punjab had non- congress ministries

1 QUESTION What was the main change introduced by the Government of India ActANSWER a) The Act gave more

autonomy to the provinces b) Diarchy was abolished at the

provincial levelsc) The Governor was the head of

the executived) There was a council of

ministers to advise him The ministers were responsible to the provincial legislatures who controlled them The legislature could also remove the ministers

e) The Governors still retained special reserve powers

2 QUESTION Why did the federal scheme introduced by the Government of India Act 1935 never come into operation

ANSWER The Federal structure of the Government of India was to be composed with the Governor General and Council of ministers The Federal legislature was to be Bicameral legislature- The council of states and the House of Assembly The ministers were to be chosen by the Governor general and they were to hold the office during his pleasure

The provinces of British India would have to join the federation but this was not compulsory for the princely states

This federation never materialised because of the lack of support from the required number of

princely statesThis act was refused and

rejected by the princes the Congress and the Muslim League

Thus both Congress and the League participated in the election of 1937 Thus the federal part was never introduced but the provincial part was put into operations

Bengali 2nd

Language

াচেরর পরাথCনা(কহিতা )

াচেরর পরাথCনা কহিতাটি কহি (ঙখ দেঘাচে4র দো আচো য কহিতায় াচেরর পতর হমায়ন কঠিন দেরাচেগ আxানত ার ঈশবর া আললার কাচেছ পরাথCনা কচেরচেছন তার পচেতরর ীন হিফহিরচেয় হিচেত এই কহিতায় ার পচেতরর ীন হিভbা দেচেয়চেছন ারার এমনহিক হিনচের ীন হিসCচেনর হিহিনমচেয় হিতহিন তার দেছচের ীন হিফচের দেপচেত দেচেয়চেছন তার দেছচের এই দেরাচেগর ন য হিতহিন হিনচেচেকই ায়ী কচেরচেছন তার হিনচের করা পাপচেকই হিতহিন ায়ী কচেরচেছন এছাা রানৈনহিতক ও আথCসামাহিক অসথার কথা তচে ধরা চেয়চেছ এই কহিতায় ার তার হিনচের পাপ কমCচেকই ায়ী কচেরচেছ ার অন যায় ভাচে দেপহি((হিকতর মাধ যচেম অপররা য কচেরচেছ আর এই অন যায় কাচের ন যই তার পহিরাচের হিপযCয় এচেসচেছ দে এক পরকার মানহিক নধন ইহিতাচেসর ার হিপতা চেয় সবাভাহিকভাচে ভাচোাসা দে মমতা দেথচেক মকত চেত পাচেরনহিন তাই হিপতা চেয় আললা া ভগাচেনর কাচেছ পতর হমায়চেনর পরানহিভbা দেচেয়চেছন ার আললা া ভগাচেনর কাচেছ াহিনচেয়চেছন তার হিনচের ীন হিসCন হিচেত হিতহিন রাী তার হিহিনমচেয় পচেতরর ীন হিফচের দেপচেত দেচেয়চেছন াচেরর হিপতসভ হিচেকর কথা এই কহিতায় ফটিচেয় দেতাা চেয়চেছ হিপতা পচেতরর হিরাহিরত মান নধচেনর কথা তচে ধরা চেয়চেছ

হিচে(4 হিকছ াইচেনর তাৎপযC১) ldquoদেকাথায় দেগ ওর সবচছয দেৌন দেকাথায় কচেরায় দেগাপন bয়ldquoউততর) াচেরর পতর হমায়ন কঠিন দেরাচেগ অসসথ তাই তার দেযৌন াহিরচেয় যাচেচছ এই দেরাচেগ তাচেক দেগাপচেন কচেরকচের াচেচছ তার সক (হিকত ধীচের ধীচের bয় চেচছ তাই হিপতা চেয় ার আললার কাচেছ হমায়চেনর পরান হিভbা দেচেয়চেছন২) ldquoাগাও (চেরর পরাচেনত পরানতচের ধসর (ন দেযর আান গানldquoউততর) াচেরর পতর হমায়ন কঠিন দেরাচেগ আxানত তাই ার আ দে(াচেক মমCাত (চেরর পচেথ পরানতচের আান গান ধবহিনত দোক দেসই আান গান আললার কাচেছ দেযন চে যায় আললা দেযন এই আহিতC শচেন পচেতরর ীন হিফহিরচেয় দেয় ৩)ldquoনাহিক এই (রীচেরর পাচেপর ীানচেত দেকানই তরারণ দেনই ভহি4চেতরldquoউততর) হমায়চেনর অসসথতার ন য ার হিনচেচেকই ায়ী কচেরচেছন কারন ার অচেনক রা য অন যায় ভাচে কচেরচেছ তাই তার এই পাপ কাচের ন য তার ঘচের আ হিপ এচেসচেছ এই অন যায় কাচের ন য তার মহিকত দেনই তাই ার আললার কাচেছ এই পাপ কাচেযCর ন য bমা পরাথM

Hindi 2ndlang

-ासी(जयशकर परसा-)

-ासी जयशकर परसा- की एक ऐसी कहानी ह जिजसम भारतीय ससकनित और राषटरीयता का सवरगजीतहोता ह इस कहानी म इरावती एक निहद कनया ह जिजस मलअचछो न मलतान की लट म पकडा और -ासी बना दि-या उस 500 दि-न -कर काशी क एक महाजन न खरी-ा दसरी -ासी निफरोजा ह वह गलाम ह निफरोजा को छडान क कतिलए अहम- को 1000 सोन क कतिसकक भजन थ जो अभी तक नही आए थ राजा साहब कठोर होत हए भी निफरोजा को निबना धनराकतिश क कतिलए उस म कर -त ह वनिफरोजा को अहम- को समझान की बात कहत हकहानी क अत म हम -खत ह निक इरा वती और जाटो क सर-ार बलराज का मिमलन होता हअहम- को यa म मार दि-या जाता ह वहा निफरोजा की परसननता की समामिध बनती ह वहा एक फल चढती ह और डीजल आती ह निफरोजा उस समामिध की आजीवन -ासी बनी रहती हलखक अपन उददशय अथात -ास परथा पर परकाश डालन और इस परथा क कारण होन वाल -ातो क दखो को दि-खान म पणता सफल हए ह

helliphellipContinue to next

Biology Reproductio Today we will discuss about vegetative Q1 Name some vegetative propagules

n in Organisms

propagation of plants The process of multiplication in which fragments of plant body function as propagule and develop into new individual is called vegetative propagation The units of such propagation are runner rhizome tuber bulb etc

and the speciesinvolvedVegetative propagules

Parts involved

Bulb StemBulbil BulbilRhizome Stem Runner Stem Tuber Stem Offset Stem Leaf buds Leaves Suckers Stem

Corns Stem stolon

Q2 State advantages of vegetative propagation

i) Rapid methodii) Sure and easy methodiii) Useful in plants that cannot

produce viable seeds or long seed dormancy

iv) Maintains purity of raceQ 3 Banana fruit is said to be parthenocarpic where as turkey is said to be parthenogenetic WhyBanana develops without fertilization from an unfertilized ovary thus is parthenocarpicIn turkey the ovum or female gamete developinto a new chick without fertilization thus isparthgenetic

Q4 Why is water hyacinth is called as a ldquoTerror of Bengalrdquo Water hyacinth can

propagatevegetatively all over the water body in a short per short period of time This resulted increased biochemicaloxygen oxygen demand of water body causing mortalityof fishes It is very difficult to get rid off them Thus known as terror of Bengal

Chemistry

Solid state GENERAL CHARACTERISTICS OF SOLID STATEIn nature the particular state of matter is governed by two opposing forces at given set of temperature and pressure These forces are intermolecular force of attraction and thermal energy If intermolecular force of attraction is high as compared to thermal energy particles remains in closest position

Intext QuestionsQ1 Classify the following solids as crystalline and amorphous Sodium chloride quartz glass quartz rubber polyvinyl chloride Teflon

A1 Crystalline

and hence very less movement in particles is observed In this case solid state is the preferred state of matter

Let us revise the general characteristics of solid

i) Fixed mass volume and shape

ii) Strong intermolecular force of attraction

iii) Least intermolecular space

iv) Fixed position of constituent particles

v) Incompressible and rigid

Q2 what type of interactions hold the molecules together in a polar molecular solid[CBSE 2010]A2 The molecules in a solid are held together by van der Waals forces The term van der Waals forces include hydrogen bonding dipole-dipole attraction and London dispersion forces All molecules experience London dispersion forces In addition polar molecules can also experience dipole-dipole interactions So the interactions that holds the molecule together in polar molecular solid are London dispersion force and dipole-dipole interactionsQ3 Write a feature that will distinguish a metallic solid from an ionic solid [CBSE 2010]A3 Metals are malleable and ductile whereas ionic solid are hard and brittle Metallic solid has typical metallic lustre But ionic solid looks dullQ4 Write a point of distinction between a metallic solid and an ionic solid other than metallic lustre [CBSE 2012]A4 Metals are malleable and ductile whereas ionic solid are hard and brittleQ5 Write a distinguish feature of metallic solid [CBSE 2010]A5 The force of attraction in

solid Sodium chloride Quartz Amorphous solid Quartz glass rubber polyvinyl chloride Teflon Q2 why glass is considered as super cooled liquidA2 Glass shows the tendency to flow at slower rate like liquid Hence they considered as super cooled liquidQ3 why the window glass of old buildings show milky appearance with timeA3 Glass is an amorphous solid Amorphous solid has the tendency to develop some crystalline character on heating Due to heating in day over the number of years glass acquires some crystalline character and show milky appearanceQ4 why the glass panes fixed to window or doors of old building become slightly thicker at bottomA4 Glass is super cooled liquid It has the tendency to flow down very slowly Due to this glass pane becomes thicker at the bottom over the timeQ5 Sodium chloride is a crystalline solid It shows the same value of refractive index along all the direction TrueFalse Give reasonA5 FalseCrystalline solid shows anisotropy in properties That is it shows different values for the given physical property in different direction All the crystalline solids show anisotropy in refractive index Therefore sodium chloride will show different values of refractive index on different directions

Q6 Crystalline solid are anisotropic in nature What does this statement means

between the constituent particles is special kind of electrostatic attraction That is the attraction of positively charged kernel with sea of delocalized electronsQ6 which group of solid is electrical conductor as well as malleable and ductile [CBSE 2013]A6 Metallic solidQ7 why graphite is good conductor of electricity although it is a network (covalent solid)A7 The exceptional property of graphite is due to its typical structure In graphite each carbon is covalently bonded with 3 atoms in same layer The fourth valence electron of each atom is free to move in between different layersThis free electron makes the graphite a good conductor of electricity

[CBSE 2011]A6 Anisotropy is defined asrdquo Difference in properties when measured along different axis or from different directionsrdquo Crystalline solid show different values of some of the physical properties like electrical resistance refractive index etcwhen measured along the different directions The anisotropy in crystalline solid arises due to the different arrangement of particles in different directions

Math Function Composition of functions Think of an industrial plant that produce bottles of cold drinks first there is the operation (or function) f that puts the cold drink inside the bottle followed by the opeartion g that close the bottle with the capThis leads to the following definitionDefinition Let f A rarr B and g B rarr C be two functions Then the composition of f and g denoted by gof is defined as the function gof A rarr C given by gof(x) = g(f (x)) forall x isinA

Definition A function f X rarr Y is defined to be invertible if there exists a function g Y rarr X such that gof = IX and fog = IY The function g is called the inverse of f and is denoted by f -1

Thus if f is invertible then f must be one-one and onto and conversely if f is one-one and onto then f must be invertible This fact significantly helps for proving a function f to be invertible by showing that f is one-one and onto specially when the actual inverse of f is not to be determined

Example 1 Let f 2 3 4 5 rarr 3 4 5 9 and g 3 4 5 9 rarr 7 11 15 be functions defined as f(2) = 3 f(3) = 4 f(4) = f(5) = 5 and g (3) = g (4) = 7 and g (5) = g (9) = 11 Find gofSolution We have gof(2) = g (f(2)) = g (3) = 7 gof(3) = g (f(3)) = g (4) = 7gof(4) = g (f(4)) = g (5) = 11 and gof(5) = g (5) = 11Example 2 Find gof and fog if f R rarr R and g R rarr R are given by f(x) = cos x and g (x) = 3x2 Show that gof ne fogSolution We have gof(x) = g(f(x))=g(cosx) = 3 (cos x)2

= 3 cos2 x Similarly fog(x)=f(g (x))= f(3x2)= cos (3x2) Note that 3cos2 x ne cos 3x2 for x = 0 Hence gof ne fogExample 3 Show that if f A rarr B and g B rarr C are onto then gof A rarr C is also ontoSolution Given an arbitrary element z isin C there exists a pre-image y of z under g such that g (y) = z since g is onto Further for y isin B there exists an element x in A with f(x) = y since f is onto Therefore gof(x) = g (f(x)) = g (y) = z showing that gof is onto Example 4 Let Y = n2 n isin N sub N Consider f N rarr Y as f(n) = n2 Show that

f is invertible Find the inverse of fSolution An arbitrary element y in Y is of the form n2 for some n isin N This implies that n =radicy This gives a function g Y rarr N defined by g (y) =radicy Nowgof (n) = g (n2)=radicn2 = n and fog (y) =f(radicy) = (radicy) 2 y which shows that gof=IN and fog= IY Hence f is invertible with f -1 = g

Political Science

Constitution of India-The Preamble

Summary

Objective of the state-To secure equality of status and of opportunity To promote fraternity among all the citizens To assure the dignity of the individuals and Unity and integrity of the nation

Justice-Justice stands for rule of law absence of arbitrariness and a system of equal rights freedom and opportunities for all in a society India seeks social economic and political justice to ensure equality to its citizens

Liberty-Liberty implies the absence of restraints or domination on the activities of an individual such as freedom from slavery serfdom imprisonment despotism etc The Preamble provides for the liberty of thought expression belief faith and worship

Equality-Equality means the absence of privileges or discrimination against any section of the society The Preamble provides for equality of status and opportunity to all the people of the country

Fraternity-The Preamble declares that fraternity has to assure two thingsmdashthe dignity of the individual and the unity and

Execution

Answer the following questions-

Short notes-1 Equality2 Fraternity3 Justice4 Liberty

Homework-Learn

integrity of the nation The word integrity has been added to the Preamble by the 42nd Constitutional Amendment (1976)

Business studies

Human resource management (chapter 1)

On the day of 1504 2020 I have discussed with you the managerial functions and procurement functions of HRM

Today weare going to discuss about the development function integration functions and maintenance function

Development functions-HRM improves the knowledge skills attitude and values of employees so that they the present and future jobs more effectively it includes

1) Development functions of HRM

a) Performance appraisal = It implies systematic evaluation of employees with respect to their performance on the job and their potential for development

b) Training =It is the process by which employees learn knowledge skills and attitudes to achieve organisational and personal goals

c) Executive development = It is the process of developing managerial talent through appropriate program

2) Integration functionsa) HRM reconcile the goals of

organisation with those of its members through integrating function

b) HRM tries to motivate employees to various financial and non financial incentives provided in job specification etc

3) Maintenance functiona) HRM promote and protect the

physical and mental health of employees by providing several types of benefits like housing medical aid etc

b) It Promote Social security measures to employees by providing provident fund pension gratuity maternity benefits

SubjectCOMMERCE

Topic

BUSINESSENVIRONMENT

Summary

Now quickly let us revise the earlier points that we have already done in the last class and let us proceed with the other topics that are there in the chapter

Firstly we will recall the internal and external factors of micro environment and then we

Execution 3 What do you mean by internal factors

in micro environmentAnswerInternal factors refer to all the factors existing within a business firm The internal factors are considered controllable because the enterprise has control over these factors

Development FunctionsPerformance AppraisalTrainingExecution Development

shall proceed in details

Meaning and list of internal and external factors

aInternal factorsInternal factors refer to all the factors existing within a business firm The internal factors are considered controllable because the enterprise has control over these factorsFor an example a company can alter its organization structure policies programmes employees physical facilities and marketing mix to suit the changes in the environmentList of internal factors areCorporate culture mission and objectives top management organizations structure company image and brand equity company resources

b External factorsExternal factors refer to those individual and groups and agencies with which a particular business organization comes into direct and frequent contact in the course of its functioningThese individuals and groups are known as STAKEHOLDERS because they have a stake (financial interest ) in the working and performance of the particular business List of external forces (stakeholders)Customers competitors investors suppliersmiddlemen (marketing intermediaries)financers publics

customers

suppliersfinancers

For an example a company can alter its organization structure policies programmes employees physical facilities and marketing mix to suit the changes in the environment

4 What do you mean by external factors in micro environment

AnswerExternal factors refer to those individual and groups and agencies with which a particular business organization comes into direct and frequent contact in the course of its functioningThese individuals and groups are known as STAKEHOLDERS because they have a stake (financial interest) in the working and performance of the particular business

3Who are stakeholdersSTAKEHOLDERS are individuals and groups who have a stake (financial interest ) in the working and performance of the particular business 4Discuss the internal factors in briefa Corporate CultureThe values beliefs and attitudes of the founders and top management of the company exercise a strong influence on what the cmpaany stands for how it does things and what it considers importantbMission and objectivesThe business philosophy and purpose of a comoany guide it prioritiesbusiness strategiesproduct market scope and development scope

cTop management structurethe composition of board of directors the degree of professionalization of management and the organizational structure of a company have important bearing on its business decisions

dPower structureThe internal power relationship between the board of directors and the chief executive is an important factor

e Company image and brand equityThe image and brand equity of the company play a significant role in raising finance forming alliance choosing dealers and suppliers launching new products entering foreign markets

5 What is Macro environmentAnswerMacro environment refers to the general

competitors

middlemen

publics

Fig STAKEHOLDERS OF A COMPANY

Apart from micro environment the other main dimension of business environment isMacro environment Macro environment refers to the general environment or remote environment within which a business firm and forces in its micro environment operateA company does not directly or regularly interact with the micro environmentTherefore macro environment is also known as indirect action EnvironmentThe macro environment forces are less controllable than the micro forces

Macro environment consists of the following components

POLITICAL AND LEGAL ENVIRONMENT

ECONOMIC SOCIAL AND ENVIRONMENT

CULTURAL

ENVIRONMENT

TECHNOLOGICAL ENVIRONMENT

Fig COMPONENTS OF MACRO ENVIRONMENT

environment or remote environment within which a business firm and forces in its micro environment operateA company does not directly or regularly interact with the micro environmentTherefore macro environment is also known as indirect action EnvironmentThe macro environment forces are less controllable than the micro forces 6 What are the components of macro environmenta Political and legal environmentb Economic environmentc Social and cultural environmentd Technological environment

Computer Science

Logic gates

Digital systems are said to be constructed by using logic gates These gates are the AND OR NOT NAND NOR EXOR and EXNOR

BUSINESS FIRM

gates The basic operations are described below with the aid of truth tables

AND gate

The AND gate is an electronic circuit that gives a high output (1) only if all its inputs are high A dot () is used to show the AND operation ie AB Bear in mind that this dot is sometimes omitted ie ABOR gate

The OR gate is an electronic circuit that gives a high output (1) if one or more of its inputs are high A plus (+) is used to show the OR operationNOT gate

The NOT gate is an electronic circuit that produces an inverted version of the input at its output It is also known as an inverter If the input variable is A the inverted output is known as NOT A This is also shown as A or A with a bar over the top as shown at the outputs The diagrams below show two ways that the NAND logic gate can be configured to produce a NOT gate It can also be done using NOR logic gates in the same way

NAND gate

This is a NOT-AND gate which is equal to an AND gate followed by a NOT gate The outputs of all NAND gates are high if any of the inputs are low The symbol is an AND gate with a small circle on the output The small circle represents inversion

NOR gate

This is a NOT-OR gate which is equal to an OR gate followed by a NOT gate The outputs of all NOR gates are low if any of the inputs are highThe symbol is an OR gate with a small circle on the output The small circle represents inversion

EXOR gate

The Exclusive-OR gate is a circuit which will give a high output if either but not both of its two inputs are high An encircled plus sign ( ) is used to show the EOR operation

EXNOR gate

The Exclusive-NOR gate circuit does the opposite to the EOR gate It will give a low output if either but not both of its two inputs are high The symbol is an EXOR gate with a small circle on the output The small circle represents inversion The NAND and NOR gates are called universal functions since with either one the AND and OR functions and NOT can be generated

Note A function in sum of products form can be implemented using NAND gates by replacing all AND and OR gates by NAND gates A function in product of sums form can be implemented using NOR gates by replacing all AND and OR gates by NOR gates

Logic gate symbols

Table 2 is a summary truth table of the inputoutput combinations for the NOT gate together with all possible inputoutput combinations for the other gate functions Also note that a truth table with n inputs has 2n rows You can compare the outputs of different gates

Logic gates representation using the Truth table

Example

A NAND gate can be used as a NOT gate using either of the following wiring configurations

Subject Eng Literature (The Tempest ndash William Shakespeare) Topic Act III Scene 3 Lines 53 to 110 (End of the scene) Date 16th April 2020 (2nd Period)

[Students should read the original play and also the paraphrase given in the school prescribed textbook]Summary Questions amp Answers

o Seeing this strange scene all are inclined to believe the tales told by travelers that there truly are ldquounicornsrdquo and ldquothe phoenixrsquo thronerdquo

o As they are about to sit down to the feast the banquet is snatched away by a harpy (Ariel disguised) A spiritrsquos voice (Arielrsquos voice) denounces Alonso Sebastian and Antonio with particular

1 ARIEL You are three men of sin whom Destiny

(Line 53-58)That hath to instrument this

lower world And what is int the never-surfeited sea

Hath caused to belch up you and on this island

Where man doth not inhabit you rsquomongst men

Being most unfit to live I have made you mad

reference to their crime in expelling Prospero from Milan They have not received any punishment for their deed earlier but the time for their punishment has arrived Upon Alonso it pronounces ldquolingering perdition worse than deathrdquo from which there is no remedy except through sincere repentance Ariel then vanishes in thunder and the shapes enter again and carry away the table

o Prospero watching invisibly is very pleased with the performance of Ariel and his (Prosperorsquos) ldquomeaner ministersrdquo All his enemies are now in his power and are in a fit of desperation He then leaves them and goes to see how Ferdinand and Miranda are getting on

o Alonso is now much humbled and penitent with the after effect of the spiritrsquos denunciation of his crimes He believes that his son is lost forever After this all disperse being stricken mad by the speech of the spirit

o Gonzalo fearing that they may do violence to themselves or to one another follows them and bid others to follow

(a) To whom does Ariel disguised as a harpy call the three sinners What game did Fate of Destiny play with

them

The three sinners called by Ariel are Alonso Sebastian and Antonio It was Destiny which had caused the ocean to cast the three sinners on the shore Though the ocean is all the time devouring whatever appears on its surface and is never satisfied with its continual swallowing of the ships and men in the present case the ocean had cast these three sinners on the shore without killing them

(b) Who had jointly been responsible for the conspiracy against Prospero What is Prosperorsquos purpose behind all this

Three men Alonso Sebastian and Antonio had jointly

been responsible for the conspiracy against Prospero They had driven out Prospero form Milan Prosperorsquos purpose is to make these three sinners realize the wrong they had done He wants them to repent for their criminal deeds because repentance leads to self-esteem(c )What does Ariel (the harpy) tell Alonso and his companions when they take out their swords to attack him

Seeing them drawing their swords Ariel (harpy) tells them that he and his companions are the instruments of destiny and that it is not possible for human beings to do them any injury He says that the swords of human beings can not injure even a minute part of his feathers Their swords are as ineffective against him and his companions as against the wind or the water

(d) Give the explanatory meanings of the following expressions in the context of the above extract

(i)Never surfeited (ii) Belch up (iii) lsquomongst men

(i) Never surfeited never led to satisfaction

(ii) Belch up cast ashore(iii) lsquomongst men in human

society2

I and my fellows (Line 60-65)

Are ministers of Fate The elementsOf whom your swords are tempered may as wellWound the loud winds or with bemocked-at stabsKill the still-closing waters as diminishOne dowl thats in my plume

IMPORTANT PASSAGES EXPLAINED

The elements

(Line 61-66)Of whom your swords are tempered may

as wellWound the loud winds or with

bemocked-at stabs

(a) Who is lsquoIrsquo Who are his lsquofellowsrdquo

lsquoIrsquo is referred to Ariel in disguise of a harpy His lsquofellowsrsquo are other spirits serving Prospero the real Duke of Milan who has acquired supernatural powers after being banished from his Dukedom Prospero has settled in this uninhabited island

(b) What are the elements that have temperrsquod the swords Why will it not work against the speaker

The swords (of Alonso and his companions) are tempered by metal (steel) which is taken out of the earth and refined by

Kill the still-closing waters as diminishOne dowl thats in my plume My fellow

ministersAre like invulnerable

In these words Ariel reminds the King and his companions of the utter futility of drawing swords against himself and his fellows Ariel drives Alonso Antonio and Sebastian the three men of sin to desperation ndash a state in which men do violence to themselves They draw swords to strike Ariel But Ariel reminds them that he and the other spirits are the ministers of destiny and nothing can wound them The steel of which their swords are made of may cut the wind or water which being divided always closes up again Even supposing that such things may be possible it is quite impossible that their swords will cut one feather in their plume They are incapable of being wounded by any sword of man Hence it is foolish on their part to attempt to strike at Ariel and his fellow-spirits

For which foul deed

(Line 72-75)The powers delaying not forgetting

haveIncensed the seas and shores yea all the

creatures Against your peace

Ariel enters like a harpy and remaining invisible tells Alonso Sebastian and Antonio that he and other harpies are the agents of Destiny appointed to carry out her decrees He tells them that their punishment for the crime against Prospero which has been so long deferred is now to fall upon them He reminds them that they had expelled Prospero from Milan and set him and his innocent child adrift on the sea and that the sea had paid them back for their sin by the shipwreck and by the calamities they have suffered He tells them that the powers above which did not forget this mean treachery but only deferred the punishment have now engaged the seas and the shores and all living beings including him and his comrades against them The very elements and supernatural agency Ariel adds have taken up the avenging of their crime against Prospero

the action of fire It may cut the wind or water which being divided always closes up again

The sword will not work against the spirits and the harpy because they are the ministers of destiny and nothing can wound them nor it will cut a single feather in their plume

(c )What is the meaning of lsquodowlrsquo in the last line

The term lsquodowlrsquo means a filament or the smallest part of a feather In this context Ariel in disguise of harpy says that their sword cannot even damage the smallest filament of their (Arielrsquos and other spirits) feathers as they are incapable of being wounded by any sword of man

(d) What does the speaker remind the listeners about

Ariel in disguise of harpy reminds Alonso the King of Naples Sebastian Alonsorsquos brother and Antonio the present Duke of Milan and the treacherous brother of Prospero as they being three men of sin He even reminds them that their punishment for their crime against Prospero which has been so long deferred now falls upon them He reminds them that they have expelled Prospero from Milan and has set him along with his innocent infant daughter adrift on the sea So the sea has paid them back for their sin by their shipwreck and the calamities they have suffered since then The harpy rebukes Alonso of his sin that has incensed the Gods and has deprived him of his son as a punishment

(e) How do they respond

When Ariel in disguise of a harpy reminds Alonso Sebastian and Antonio of their past misdeeds and sin Alonso has a look of terror and confusion in his eyes He utters the words of sincere repentance wrung out of his conscience-stricken heart It appears to him that all the elements of nature the sea-waves the wind and the thunder proclaiming a loud voice in the name of Prospero and the crime Alonso has committed against him They are calling upon him to repent There is a deep storm raging in Alonsorsquos breast and the echoes of that storm are ringing in his ears like a clear note of wind-instrument A note of denunciation of Alonsorsquos crime leaves him much humbled and penitent and confirms his belief that his son is lost forever But Sebastian and Antonio shows some courage instead of repentance They wish to kill the spirits or devils if it appears

3

Of my instruction hast thou nothing bated (Line 85-93)

In what thou hast to say So with good life

And observation strange my meaner ministers

Their several kinds have done My high charms work

And these mine enemies are all knit upIn their distractions They now are in my

powerAnd in these fits I leave them while I visitYoung Ferdinand whom they suppose is

drownedAnd his and mine loved darling

Methought the billows spoke and (Line 96-99)

told me of itThe winds did sing it to me and the

thunderThat deep and dreadful organ-pipe

pronouncedThe name of Prosper It did bass my

trespass

These are the words of contrition coming from Alonso Ariel has driven him to a deep repentance for conspiring with Antonio against Prospero He now feels a sincere remorse It appears to him that all the elements of nature the sea-waves the wind and the thunder proclaimed with a loud voice the name of Prospero and the crime Alonso had committed against him They are calling upon him to repent There is a deep storm raging in Alonsorsquos breast and the echoes of that storm are ringing in his ears like the clear note of a wind-instrument

Comment These are the words of sincere repentance wrung out of the conscience-stricken heart of Alonso Alonso who is the lesser villain is the first to give way to remorse under the effect of Arielrsquos speech The words of Ariel seem to him to be the voice of conscience speaking to him He is driven to desperation a state in which he might do violence to his life

(a) Identify the speaker State the context

Prospero the ruler of the island is the speaker The famous banquet scene has been enacted very well Ariel and his junior spirits have played their roles excellently Prospero is glad to say words of praise for them(b) In what way the speakerrsquos instructions have been carried out

According to Prosperorsquos instructions a banquet was presented before the King of Naples and his companions when they were tired and hungry Just when they were preparing to eat the feast the banquet was suddenly removed by exercising supernatural powers All this was done by Ariel Prosperorsquos chief assistant and a powerful spirit

Ariel not only made the feast disappear but also delivered his speech blaming the King and his two companions for their past wicked deeds He warned them to repent for their misdeeds or suffer forever on that uninhabited island

(c) Who are referred to as lsquomeaner ministersrsquo What have they done

Prospero refers as lsquomeaner ministersrsquo to his other lesser spirits who were assisting Ariel in presenting a scene before the kingrsquos party They entered the scene to the accompaniment of music They assumed several strange shapes and brought in a banquet Then they danced about it with gentle actions of salutations thus inviting the King and others to eat the feast

These spirits play their role again when Ariel in the shape of a harpy quits the scene These shapes enter again and dancing with mocking gestures carry away the table

(d) Who are the speakerrsquos enemies What has happened to them

King of Naples Alonso his brother Sebastian and the present Duke of Milan Antonio (Prosperorsquos own brother) are Prosperorsquos enemies With the turn of events they have all been washed ashore on the island which is ruled by Prospero the great magician Actually this happened after the shipwreck caused by a storm which was raised by Prospero with the purpose of bringing these people to his island Prosperorsquos spirits have already confused and terrified these enemies and they are under Prosperorsquos control He can treat them as he likes

(e) What does he say about Ferdinand Explain what is meant by ldquohellip his and mine darlingrdquo

Prospero knows that Alonsorsquos son prince Ferdinand is alive though his father thinks that the prince has been drowned

Prospero refers to his daughter Miranda who is dear to him She is also very dear to Prince Ferdinand who has fallen in love with her They are waiting to be married soon for which they have received Prosperorsquos consent

4

ALONSO O it is monstrous monstrous (Line 95-102)

Methought the billows spoke and told me of it

The winds did sing it to me and the thunderThat deep and dreadful organ-

pipe pronouncedThe name of Prosper It did bass

my trespassTherefore my son ithrsquo ooze is

bedded andIll seek him deeper than eer

plummet soundedAnd with him there lie mudded

(a) In what way does Alonso express his horror when his conscience is awakened by Arielrsquos words

When Alonsorsquos conscience is awakened by Arielrsquos words he expresses his horror at what he has heard He gets the feeling that the waves of the ocean the wind and the loud thunder have spoken to him and uttered the name of Prospero Because of being reminded of his crime in a very loud and rough voice he comes to realize that he has lost his son for his past misdeeds

(b) What does Alonso imagine about his son What does Alonso want to do in his desperate state

Alonso imagines that his son is lying in the mud at the bottom of the sea He feels desperate that he wants to drown himself in the ocean deeper than the plumb-line has ever gone He wants to lie with his son at the bottom of the sea

(c) How do Sebastian and Antonio want to face the evil spirits

Sebastian says that he is not at all afraid of what the harpy has said and that he is prepared to fight any number of such monsters if they appear before him only one at a time Antonio says that he would support Sebastian in the fight against the fiendsyyy

(d) Why does Gonzalo ask Adrian to follow the three men

Gonzalo tells Adrian that all the three men namely Alonso Sebastian and Antonio are in a wild and reckless mood The thought of the heinous crime of which they are guilty has begun to torment their minds So he asks Adrian to follow those three men without loss of time and prevent them from doing anything which the turmoil in their minds might lead them to do

(e) What opinion do you form of Alonso from the above extract

Alonso who is the lesser villain is the first to give way to remorse under the effect of Arielrsquos speech The words of Ariel seem to him to be the voice of conscience speaking to him He is driven to desperation a state in which he might do violence to his life

Subject =Accounts

Ac-12 15420 topic-pL Appropriation ac

PROFIT AND LOSS APPROPRIATION ACCOUNT

MEANING AND PREPARATIONProfit and Loss Appropriation Account is merely an extension of the Profit and Loss Account of the firm The profit of the firm has to be distributed amongst the partners in their respective profit sharing ratio But before its distribution it needs to be adjusted All Adjustments like partnerrsquos salary partnerrsquos commission interest on capital interest on drawings etc are made in this account These adjustments will reduce the amount of profit for distribution This adjusted profit will be distributed amongst the partners in their profit sharing ratio To prepare it at first the balance of Profit and Loss Account is transferred to this account The journal entries for the preparation of Profit and Loss Appropriation Account are given below

1 for transfer of the balance of Profit and Loss Account to Profit and Loss Appropriation Account

(a) In case of Net Profit

Profit and Loss Ac helliphelliphelliphelliphellipDrTo Profit and Loss Appropriation Ac(Net Profit transferred to Profit and Loss Appropriation Ac)

(b)In case of Net Loss

Profit and Loss Appropriation Achelliphelliphellip DrTo Profit and Loss Ac(Net Loss transferred to Profit and Loss Appropriation Ac)

2 for Interest on Capital

For transferring on Interest on CapitalProfit and Loss Appropriation Achelliphelliphellip DrTo Interest on Capital Ac(Interest on capital transferred to Profit amp Loss Appropriation Ac)

3 for Interest on Drawings

For transferring Interest on Drawings Interest on Drawings Achelliphelliphelliphelliphelliphellip DrTo Profit and Loss Appropriation Ac(Interest on drawing transferred to Profit amp Loss Appropriation Ac)

4 For Partnerrsquos SalaryFor transfer of partnerrsquos SalaryProfit and Loss Appropriation Achelliphellip DrTo Salary Ac(Salary transferred to profit amp Loss Appropriation Ac)

5 For Partnerrsquos CommissionFor transferring commissionProfit and Loss Appropriation Achelliphelliphellip DrTo Commission Ac(Commission transferred to Profit and Loss Appropriation Ac)

6 For Transfer of agreed amount to General ReserveProfit and Loss Appropriation Ac helliphellipDrTo General Reserve Ac(Transfer to General Reserve)

7 for share of Profit or Loss appropriation(a) If ProfitProfit and Loss Appropriation Achelliphellip DrTo Partnerrsquos CapitalCurrent Ac(Profit transferred to capitalcurrent Ac)(b) If LossPartnerrsquos Capital Current Achelliphelliphelliphellip DrTo Profit and Loss Appropriation Ac(Loss transferred to capitalcurrent Ac)

THE FORMAT OF PROFIT AND LOSS APPROPRIATION

Profit and Loss Appropriation Account for the year endedhelliphelliphelliphellip

Particulars Amount Particulars Amount

To PL Ac (loss) By pL Ac (profit)

To Interest on capital BY Interest on drawings

To partner`s commission by Partner`s capital Ac ( loss)

To Partner`s salary To Interest on partner`s loan To General Reserve To Partner`s Capital AC (Profit)

Subject= Economics

MOVEMENT ALONG THE DEMAND CURVE (CHANGE IN QUANTITY DEMANDED)In law of demand you have already studied the inverse relationship between price and quantity demanded When quantity demanded of a commodity changes due to change in its price keeping other factors constant it is called change in quantity demanded It is graphically expressed as a movement along the same demand curve There can be either a downward movement or an upward movement along the same demand curve Upward movement along the same demand curve is called contraction of demand or decrease in quantity demanded and downward movement along the same demand curve is known as expansion of demand or increase in quantity demanded

Extention of demandd

price (rs)p A

B Extentionp1 d

Q Q1

Quantity demanded ( in units)

Contraction of demandd

p2 Ccontraction

p APrice (Rs)

d

Q2 Q

Quantity demanded (in units)

Explanation of movement of demand A fall in price from OP to OP1 leads to increase in quantity demanded from OQ to OQ1 (expansion of demand) resulting in a downward movement from point A to point B along the same demand curve DD When Price rises from OP to OP2 quantity demanded falls from OQ to OQ2 (contraction of demand) leading to an upward movement from point A to point C along the same demand curve DD

  • Activity Series of Metals
    • Drawbacks of Rutherfordrsquos model of atom
      • Electromagnetic radiations
      • Properties of electromagnetic radiations
      • Characteristics of electromagnetic radiations
        • Plancks Quantum Theory-
        • Photoelectric effect
          • Intext Questions
            • Logic gates
            • Digital systems are said to be constructed by using logic gates These gates are the AND OR NOT NAND NOR EXOR and EXNOR gates The basic operations are described below with the aid of truth tables
            • AND gate
            • Example
Page 31:  · Web viewSubject. Topic. Summary. Execution. English 1 . Chapter 1 naming words . Page 8. Write the names of these pictures:- Person:-1. father. 2.Firefighter 3.doctor 4 ...

the institution of village community system The agrarian economy made fixed dwelling houses necessary Living close together for protection and co-operation and living nearer to the land gave birth to agricultural villages The village is not only the residential place of farmers it is also the social integrator

4 Minimal Division of Labour Another structural feature of agrarian society is a minimal division of labour Except for the basic division founded on age and sex differences there are few specialized roles There is only one predominant type of occupation ie domestication of plants and animals For all the people the environment physical as well as social is the same

5 Role of Family The farm family is of the patriarchal type the father is the final arbiter in most of the familyrsquos major decisions The life of ail men and women is merged in family life Since there are not many special organizations family is the only organisation to perform the tasks of aid and protection

6 Sense of Unity The members of an agrarian society exhibit a strong in-group feeling Since the whole of their social lives is wrapped up in a society which is physically economically and socially homogenous they are inclined to view the entire outside world as an out group

7 Informal Social Control An agrarian society is regionally divided into villages In a village community the force of traditional mores is more dominant than in the urban community In the village everybody is known to everybody The members in a village community help each other and share the joy and sorrows of each other Crime in an agrarian society is rare

8 Simplicity and Uniformity Life of the people in an agrarian society is marked by simplicity and uniformity Their main occupation is agriculture which largely depends upon the vagaries of nature An agrarian society is a religious society

Math Compound angles Compound angles The algebraic sum of two or more angles is called a compound angle If A B C be three angles then A+B B+C C+A A-B B-C A-C A+B-C etc are compound angles In this chapter we shall discuss the trigonometrical ratios of compound angles Theorem 1 If A B and A+B are all pisitive acute angles theni) sin( A+B) = sin A cos B + cosA sinBii) cos(A+B) = cosA cosB- sinA sinBTheorem 2If A and B are positive acute angles and AgtB theni) sin(A-B) = sin A cosB- cos A sinBii) cos(A-B) = cos A cos B+ sin A sin BTo prove that i) sin(A+B) sin (A-B) = sin2 A - sin2 B = cos2 B- cos2 A

Example 1 Prove that tan70deg=2tan50deg+tan20degSolutiontan70deg = tan(50deg + 20deg)Or tan70deg=(tan 50deg+tan 20deg)(1-tan50degtan20deg) or tan70deg (1 ndash tan 50deg tan20deg) = tan50deg+tan20degor tan70deg= tan70deg tan50deg tan20deg+ tan50deg + tan20deg = cot20deg tan50deg tan20deg + tan50deg + tan20deg = 2 tan50deg+ tan20degExample 2 If A + B = 45deg show that (1 + tanA) (1 + tanB) = 2Solutiontan(A + B) =( tan A + tan B) (1 - tan

ii) cos(A+B) Cos(A-B) = cos2 A- sin2 B = cos2 B -sin2 AProof i) LHS= sin(A+B)sin(AminusB) [Recall sin(αminusβ)=sinαcosβminuscosαsinβ And sin(α+β)=sinαcosβ+cosαsinβ]= (sinAcosB+cosAsinB)times(sinAcosBminuscosAsinB)= sin2Acos2Bminuscos2Asin2B [Recall sin2α+cos2α=1 From above we can then assume correctly that sin2α=1minuscos2α AND cos2α=1minussin2α] = sin2A(1minussin2B)minussin2B(1minussin2A) = sin2Aminussin2Asin2Bminussin2B+sin2Asin2B = sin2Aminussin2B= 1-cos2A-(1-cos2B) = cos2 B- cos2 A = RHSii)LHS= cos (A+B) cos(A-B) [ cos(A+B) = cos AcosB- sinAsinBCos(A-B) = cosAcosB+ sinAsinB]= cos2 A Cos2 B- sin2 A Sin2 B= cos2 A( 1-sin2 B) - (1- cos2 A) sin2 B= cos2 A- cos2 A sin2 B- sin2 B+ cos2 A sin2 B=cos2 A- sin2 B=1- sin2 A-(1-cos2 B) = cos2 B- sin2 A= RHSTangent formulae for compound anglesi)tan (A + B) = tan A + tan B1-tan A tan Bii)tan (A ndash B) = tan A-tan B1+tan A tan Biii) cot (A + B) = cot Acot B-1cot A+cot B(viii) cot (A ndash B) = cot Acot B+1cot B-cot A

A tan B) Or 1= (tan A+ tanB) (1-tan A tanB) Or tanA + tanB + tanA tanB + 1 = 1 + 1Or tanA (1 + tanB) + (1 + tanB) = 2Or (1 + tanA) (1 + tanB) = 2Example 3 Find the value of sin 15degSolution sin 15deg= sin(45deg-30deg) = sin45degcos 30deg- cos45degsin30deg =(1radic2) (radic32) -(1radic2) (12) = (radic3-1) 2radic2Example 4 If sin A = 1 radic10 and sin B = 1 radic5 where A and B are positive acute angles then what is A + B SolutionWe know that sin (A + B) = sin A cos B + cos A sin B= [1 radic10] [radic(1 minus 1 5)] + [1 radic5] radic(1 minus 1 10)= [1 radic10] [radic4 5] + [1 radic5] [radic9 10]= [1 radic50] times (2 + 3)= 5 radic50 = 1 radic2

sin (A + B) = sin π 4rArrHence A + B = π 4Example 5 If A + B = 225o then find [cot A] [1 + cotA] times [cot B] [1 + cot B]Solution[cot A] [1 + cotA] times [cot B] [1 + cot B] = 1 [(1 + tan A) times (1 + tan B)]=1 [tan A + tan B + 1 + tan A tan B] [ tan (A + B) = tan225o]∵

tan A + tan B = 1minus tan A tan BrArr= 1 [1 minus tan A tan B + 1 + tan A tan B]= 1 2

COMMERCE

CLASSIFICTION OF HUMAN ACTIVITIES-ECONOMIC AND NON-ECONOMIC

Firstly we shall recall the previous class for 5 mins especially for the absentees and for also the rest of the students who were there

Today at first we briefly discuss the earlier portions of the chapter

1Business-It includes all those economic activities which are concerned with production and exchange of goods and services with the object of earning profit Example A factory shop beauty parlour also business enterprises

2Profession ndashThe term profession means an occupation which involves application of specialized knowledge and skills to earn a living For Example Chartered Accountancy medicine law tax consultancy are example of professions

Questions1What are the main features of ProfessionAnswer The main features of a profession are as follows a Specialised body of knowledge-Every profession has a specialised and systematised body of knowledge b Restricted entry- Entry to a profession is allowed only to those who have completed the prescribed education and have the specialised examination c Formal education and training ndashA formal education and training is given to the person who wants to acquire the professional

3Employment-Employment mean an economic activity where people work for others in exchange for some remuneration (salary)The persons who work for others are called lsquoemployeesrsquo The persons or organizations which engage others to work for them are called lsquoemployersrsquoEg A doctor working in a hospital is employment as he is working for a salaryA lawyer may serve as a law officer in a bank

With this we shall proceed with the features of both Profession amp Employment

The main features of a profession are as follow

a Specialised body of knowledge b Restricted entry c Formal education and training d Professional association e Service motive f Code of contact

The main features of an employment are as follows

a In employment a person works for others called employer

b An employee provides personal service

c There is a service agreement or contract between the employee and the employer

d The employee has to obey the order of the employer

e No capital investment is made by the employer

Various examples of Employment are as follows

aA teacher teaching in a school or collegeb An engineer employed in Municipal Corporation of DelhicAn accountant working in the accounts department of a companydA doctor working in a hospital

Note In all the above examples of employment the individual who is involved in each example is working as an employee for a salary under an employer

qualification(MBBSCALLB)d Service motive ndashProfessionals are expected to emphasis service more on their clients rather than economic gain f Code of Conduct-The activities of professionals are regulated by a code of conduct

2 What are the main features of EmploymentAnswer The main features of an employment are as followsa In employment a person works for others called employerb An employee provides personal servicec There is a service agreement or contract between the employee and the employerd The employee has to obey the order of the employere No capital investment is made by the employer

3 Give various Professions and their respective Association are given below

Professions

Professional

Professional association

Medical profession

Doctor Medical Council of India

Law profession

Lawyers Bar Council of India

Accounting Profession

Chartered

The Institute of Chartered Accounts of India( ICAI)

Engineerin Engineers The

g Profession

institute of Engineers (India)

Accounts Basic accounting terms

Today we will give you some questions from the previous study material

Questions6) Define accounting7) What do you mean by debit

and credit8) Explain the types of account9) Define the following terms

a) Assetsb) Capitalc) Purchased) Debtorse) Transactions

10) Name the types of accounts given below

a) Krishnas accountb) Machinery accountc) Royalty accountd) Salary accounte) Furniture accountf) Audit fee account

Economics Basic Economic ConceptsSub topic

UTILITY

Before starting todayrsquos class we shall recall the last class which was about UTILITY AND THE FEATURES OF UTILITY

Now we shall proceed with the further topics of the chapter

Todayrsquos topic from the chapter lsquo Basic Economic Conceptsrsquo will be TOTAL UTILITY amp MARGINAL UTILITYNow let us quickly revise the concept of utility with an example ie goods and services are designed because they have an ability to satisfy human wantsThis feature of being able to satisfy human wants is termed as utility For example we derive utility from WiFi services as it gives us satisfaction by connecting us to our friends and family through social media here consumers derive utility from WiFi services

From the above concept we shall start with todayrsquos topicEconomists have defined TOTAL UTILITY (TU) as the total satisfaction obtained by consuming a given total amount of a good and serviceFor example the total satisfaction obtained from eating 10 mangoes is the total utility of 10 mangoes

MARGINAL UTILITY (MU) is the additional satisfaction derived from each additional unit

Questions1 What is Total Utility (TU)

Answer Total Utility (TU) is the

aggregate of the utility that a consumer derives from the consumption of a certain amount of a commodityTU=MU1+MU2++MUn

2 What is Marginal UtilityAnswer

Marginal Utility (MU) is the additional made to the total utility as consumption is increased by one more unit of the commodityMU= TUn ndashTUn-1

NoteOften economists tend to

subdivide utility into an imaginary unit called UTIL

consumed In this casethe utility obtained from each mango as it is consumed as the MU of that mango It is also defined as the addition made to the total utility when an additional unit is consumed Often economists tend to subdivide utility into an imaginary unit called UTIL

Note As a consumer increases the consumption of a good over period of time the total utility or total satisfaction derived from it increases to appoint and thereafter it decreasesHowever as the consumer keeps on consuming the good the marginal utility or the additional utility derived from it decreases

SubjectBusiness studies

Topic

BUSINESSENVIRONMENT

Summary

Now quickly let us revise the earlier points that we have already done in the last class and let us proceed with the other topics that are there in the chapter

Firstly we will recall the internal and external factors of micro environment and then we shall proceed in details

Meaning and list of internal and external factors

aInternal factorsInternal factors refer to all the factors existing within a business firm The internal factors are considered controllable because the enterprise has control over these factorsFor an example a company can alter its organization structure policies programmes employees physical facilities and marketing mix to suit the changes in the environmentList of internal factors areCorporate culture mission and objectives top management organizations structure company image and brand equity company resources

b External factorsExternal factors refer to those individual and groups and agencies with which a particular business organization comes into direct and frequent contact in the course of its functioningThese individuals and groups are known as STAKEHOLDERS because they have a stake (financial interest ) in the working and performance of the particular business List of external forces (stakeholders)Customers competitors investors suppliersmiddlemen (marketing intermediaries)

Execution 1 What do you mean by internal

factors in micro environmentAnswerInternal factors refer to all the factors existing within a business firm The internal factors are considered controllable because the enterprise has control over these factorsFor an example a company can alter its organization structure policies programmes employees physical facilities and marketing mix to suit the changes in the environment

2 What do you mean by external factors in micro environment

AnswerExternal factors refer to those individual and groups and agencies with which a particular business organization comes into direct and frequent contact in the course of its functioningThese individuals and groups are known as STAKEHOLDERS because they have a stake (financial interest) in the working and performance of the particular business

3Who are stakeholdersSTAKEHOLDERS are individuals and groups who have a stake (financial interest ) in the working and performance of the particular business 4Discuss the internal factors in briefa Corporate CultureThe values beliefs and attitudes of the founders and top management of the company exercise

financers publics

customers

suppliersfinancers

competitors

middlemen

publics

Fig STAKEHOLDERS OF A COMPANY

Apart from micro environment the other main dimension of business environment isMacro environment Macro environment refers to the general environment or remote environment within which a business firm and forces in its micro environment operateA company does not directly or regularly interact with the micro environmentTherefore macro environment is also known as indirect action EnvironmentThe macro environment forces are less controllable than the micro forces

Macro environment consists of the following components

POLITICAL AND LEGAL ENVIRONMENT

ECONOMIC SOCIAL AND ENVIRONMENT

CULTURAL

ENVIRONMENT

TECHNOLOGICAL ENVIRONMENT

a strong influence on what the cmpaany stands for how it does things and what it considers importantbMission and objectivesThe business philosophy and purpose of a comoany guide it prioritiesbusiness strategiesproduct market scope and development scope

cTop management structurethe composition of board of directors the degree of professionalization of management and the organizational structure of a company have important bearing on its business decisions

dPower structureThe internal power relationship between the board of directors and the chief executive is an important factor

eCompany image and brand equityThe image and brand equity of the company play a significant role in raising finance forming alliance choosing dealers and suppliers launching new products entering foreign markets

5 What is Macro environmentAnswerMacro environment refers to the general environment or remote environment within which a business firm and forces in its micro environment operateA company does not directly or regularly interact with the micro environmentTherefore macro environment is also known as indirect action EnvironmentThe macro environment forces are less controllable than the micro forces 6 What are the components of macro environmenta Political and legal environmentb Economic environmentc Social and cultural environmentd Technological environment

BUSINESS FIRM

Fig COMPONENTS OF MACRO ENVIRONMENTPolitical science

Introduction to political science

Comparative politics and itrsquos scope Comparative politics is the second major dimension of political scienceIt is also a very vast area of study and a very large number of political scientists even treat it as an autonomous area of study within the board ambit of political scienceScope of comparative politics-

1 All political structures -Comparative politics includes the study of all structures formalnon formal governmental and extra governmental which are directly or indirectly involved in politics in all the countries of the world

2 Functional studies- Comparative politics seeks to study politics less from the point of view of the legal institutions in terms of their powers and move from the point of view of their functions which constitute the political process and their actual Operation in the environment

3 Study of political behaviour- Another important part of its scope is the study of the actual behaviour of the people in the process of politics

4 Study of similarities and differences- comparative politics also undertakesan analysis of the similarities and differences among political process and functions

5 Study of all political systems -comparative politics seeks to analyse the actual behaviour and performance of all political systems western as well as non western

6 Study of the environment and infrastructure of politics-The study of politics demands a study of the psychological sociological economic and anthropological environment in fact the social environment as a whole in which each political system operates

7 Study of political culture- political culture is composed of attitudesbeliefs emotions and values of a society that relate to the political system or politics

8 Study of political participation- Political participation is a universal processThe only difference is that while in some states it is limited in others it is wider

9 Study of political process- political

Answer the following questions-

What is comparative politics

What are the scope of comparative politics

Homework- learn

processes like decision makingpolicy making judicial process leadership recruitment process and others are always at work in all political systems

The scope of comparative politics is very comprehensive It includes everything that falls within the area of political activity and political process

History CAMBRIDGE VIEW ABOUT

THE PARTITION

AND REFUTATION

OF CAMBRIDGE

VIEW

Cambridge view about the Partition The Cambridge school of historians have interpreted that opposition to partition scheme was made entirely by the elitist groups They hold the view that Lord Curzon planned to partition the Bengal for administrative purposeREFUTATION OFCAMBRIDGE VIEW The Rationalist historians have rejected the interpretations of the Cambridge School of historians on various grounds

1 QUESTION State different views of historians regarding Partition of Bengal

ANSWER Cambridge historians believed that Lord Curzon partitioned Bengal for administrative reasons only and not for the political motive The Middle class elitist group protested because of their petty interest The Hindu zamindars protested as they have to spend more money for managing their estatesThe lawyers of Calcutta High court feared to lose their clientBut according to the nationalist Historians was-

2- The ultimate object of Lord Curzon was to crush the unity of Bengal politicians

3- If Bengal becomes a separate province Bengali speaking 16 million people of western part would become minority under Hindi speaking people of Bihar and Oriya speaking people of Orissa

4- The bureaucrats expected that the protest movement would die down quickly

5- Lord Curzon used the Muslim community in his political game

6- Idealism had great contribution in the protest against partition

7- The people of the every section of society were affected by the partition of Bengal

Computer Science

Numbers Convertion of dcimal number to octal numberThe decimal numeral system is the standard system for denoting integer and non-integer numbers It is the extension to non-integer numbers of the Hindu-Arabic numeral system For writing numbers the decimal system uses ten decimal digits a decimal mark and for negative numbers a minus sign - The decimal digits are 0 1 2 3 4 5 6 7 8 9 the decimal separator is the dot in many countries

The octal numeral system or oct for short is the base-8 number system and uses the digits 0 to 7 Octal is sometimes used in computing instead of hexadecimal perhaps most often in modern times in conjunction with file

permissions under Unix systems It has the advantage of not requiring any extra symbols as digits It is also used for digital displays

Follow these steps to convert a decimal number into octal form

1 Divide the decimal number by 82 Get the integer quotient for the next iteration (if the number will not divide equally by 8 then round down the

result to the nearest whole number)3 Keep a note of the remainder it should be between 0 and 74 Repeat the steps until the quotient is equal to 05 Write out all the remainders from bottom to top This is the solution

For example if the given decimal number is 8453

Division Quotient Remainder

8453 8 1056 5

1056 8 132 0

132 8 16 4

16 8 2 0

2 8 0 2

Then the octal solution is 20405

Subject Eng Literature (The Tempest ndash William Shakespeare) Topic Act I Scene 1 Lines 33 to 67 (End of scene) Date 16th April 2020 (4th Period)

[Students should read the original play and also the paraphrase given in the school prescribed textbook]Summary Questions amp Answers

[SUMMARY OF THE ENTIRE SCENE]

o The play starts with the scene of a severe storm at sea Alonso (King of Naples) Sebastian (Alonsorsquos brother) Ferdinand (Alonsorsquos son) Gonzalo Antonio (the usurping Duke of Milan) are in a ship in the midst of the storm

o The mariners are trying their best to control the vessel from running aground and are totally following the orders of their Master the Boatswain They have scant success

o The mariners become extremely unhappy and annoyed when most of the passengers arrive on the deck thereby hampering their effort to save the ship There is serious confrontation between them and the passengers who are part of the Kingrsquos entourage

o The mariners could not save the ship

SUMMING-UP

(i) Vivid description of the scene which gives a realistic description of terror and confusion of a tropical storm

(ii) Shows Shakespearersquos accuracy of knowledge in describing the naval operations and also matters of seamanship

(1) GONZALO Ill warrant him for drowning (L 45-57)

though the ship were no stronger than a nutshell and as leaky as an unstanched

wenchBOATSWAIN Lay her a-hold a-hold Set her two courses Off to

sea again lay her offMARINERS All lost To prayers to prayers All lostBOATSWAIN What must our mouths be coldGONZALO The king and prince at prayers Lets assist them

For our case is theirsSEBASTIAN Im out of patienceANTONIO We are merely cheated of our lives by drunkards

This wide-chopped rascal - would thou mightst lie drowning the washing of ten tides

(a) What does Antonio say at the insolent manners of the boatswain just before the given passage

Being irritated at the insolent manners of the boatswain just before the given extract Antonio the Duke of Milan calls him a worthless dog son of a woman without any morals an arrogant and disrespectful noisemaker He says that the boatswain deserved to be hanged(b) What statement does Gonzalo repeat about the boatswain

Gonzalo shows his faith that the boatswain is not destined to die by drowning He is destined to be hanged and nothing can alter this decree of destiny He says that even if the ship was as frail as a nutshell the boatswain could not be drowned for his destiny was to be hanged(c) What do the passengers do when they have lost all hope of their survival

When the passengers have lost all hope of survival they take

(iii) The opening scene justifies the title ndash The Tempest

UNANSWERED QUESTIONS

(i) The King always travels with his entire fleet including his soldiers Where were the other ships

(ii) Why was the ship in that area Where was it coming from or going where

(iii) The ship broke apart What happened to those who were in the ship

(We shall get the answer to the above questions as the play progresses)

leave of life with fervent prayers The mariners take their last hearty drink and are ready for death(d) What blame does Antonio put upon the mariners and the boatswain Antonio rebukes the mariners that these drunkards have brought them to the present crisis by neglecting their duties He blames them saying that they are going to lose their lives entirely for the negligence of the boatswain and his fellows(e) What does Antonio say while cursing the boatswain

Antonio gives vent to his wrath upon the boatswain in particular He calls the boatswain a wide-mouthed rascal who deserves to be hanged on the sea-shore at low water mark so that ten tides might wash over his body and take out of him all the liquor that he has been drinking

Class XIISubject Topic Summary ExecutionHistory Topic

1 1935 ACT AND WORKING OF PROVINCIAL AUTONOMYCONGREE AND OTHER MINISTERSSUB TOPIC GOVERNMENT OF INDIA ACT1935

Government of India Act 1935 This act established a lsquoFederation of Indiarsquo made of British Indian provinces and Indian states and provided for autonomy with a government responsible to the elected legislature in every provinceThis act introduced abolition of Diarchy at provinces The entire provincial administration was introduced to the responsible ministers who were controlled and removed by the provincial legislature The provincial autonomy means two things First The provincial governments were wholly responsible to the provincial legislature Secondly Provinces were free from outside control and interference in the large number of matters The act divided the powers between the centre and provinces in terms of three lists- Federal list( for centre) Provincial list (for province) and concurrent list (for both) Residuary powers were given to the viceroy In the election under the government of India Act the Congress swept the poll the mandate of the people came in favour of the congress so far as general Hindu seats were concerned The Congress did not get a single Muslim seates in Bombay CP UP Sind and BengalIn five provinces Congress had yhe clear majority In BengalNWFPAssam and Bombay Congress emerged as a single largest partyOn the other side the performance of the Muslim League was badThus the Congress formed ministers in 7 provinces out of 11 provinces Coalition ministry was also formed in two other provincesOnly BENGAL AND Punjab had non- congress ministries

1 QUESTION What was the main change introduced by the Government of India ActANSWER a) The Act gave more

autonomy to the provinces b) Diarchy was abolished at the

provincial levelsc) The Governor was the head of

the executived) There was a council of

ministers to advise him The ministers were responsible to the provincial legislatures who controlled them The legislature could also remove the ministers

e) The Governors still retained special reserve powers

2 QUESTION Why did the federal scheme introduced by the Government of India Act 1935 never come into operation

ANSWER The Federal structure of the Government of India was to be composed with the Governor General and Council of ministers The Federal legislature was to be Bicameral legislature- The council of states and the House of Assembly The ministers were to be chosen by the Governor general and they were to hold the office during his pleasure

The provinces of British India would have to join the federation but this was not compulsory for the princely states

This federation never materialised because of the lack of support from the required number of

princely statesThis act was refused and

rejected by the princes the Congress and the Muslim League

Thus both Congress and the League participated in the election of 1937 Thus the federal part was never introduced but the provincial part was put into operations

Bengali 2nd

Language

াচেরর পরাথCনা(কহিতা )

াচেরর পরাথCনা কহিতাটি কহি (ঙখ দেঘাচে4র দো আচো য কহিতায় াচেরর পতর হমায়ন কঠিন দেরাচেগ আxানত ার ঈশবর া আললার কাচেছ পরাথCনা কচেরচেছন তার পচেতরর ীন হিফহিরচেয় হিচেত এই কহিতায় ার পচেতরর ীন হিভbা দেচেয়চেছন ারার এমনহিক হিনচের ীন হিসCচেনর হিহিনমচেয় হিতহিন তার দেছচের ীন হিফচের দেপচেত দেচেয়চেছন তার দেছচের এই দেরাচেগর ন য হিতহিন হিনচেচেকই ায়ী কচেরচেছন তার হিনচের করা পাপচেকই হিতহিন ায়ী কচেরচেছন এছাা রানৈনহিতক ও আথCসামাহিক অসথার কথা তচে ধরা চেয়চেছ এই কহিতায় ার তার হিনচের পাপ কমCচেকই ায়ী কচেরচেছ ার অন যায় ভাচে দেপহি((হিকতর মাধ যচেম অপররা য কচেরচেছ আর এই অন যায় কাচের ন যই তার পহিরাচের হিপযCয় এচেসচেছ দে এক পরকার মানহিক নধন ইহিতাচেসর ার হিপতা চেয় সবাভাহিকভাচে ভাচোাসা দে মমতা দেথচেক মকত চেত পাচেরনহিন তাই হিপতা চেয় আললা া ভগাচেনর কাচেছ পতর হমায়চেনর পরানহিভbা দেচেয়চেছন ার আললা া ভগাচেনর কাচেছ াহিনচেয়চেছন তার হিনচের ীন হিসCন হিচেত হিতহিন রাী তার হিহিনমচেয় পচেতরর ীন হিফচের দেপচেত দেচেয়চেছন াচেরর হিপতসভ হিচেকর কথা এই কহিতায় ফটিচেয় দেতাা চেয়চেছ হিপতা পচেতরর হিরাহিরত মান নধচেনর কথা তচে ধরা চেয়চেছ

হিচে(4 হিকছ াইচেনর তাৎপযC১) ldquoদেকাথায় দেগ ওর সবচছয দেৌন দেকাথায় কচেরায় দেগাপন bয়ldquoউততর) াচেরর পতর হমায়ন কঠিন দেরাচেগ অসসথ তাই তার দেযৌন াহিরচেয় যাচেচছ এই দেরাচেগ তাচেক দেগাপচেন কচেরকচের াচেচছ তার সক (হিকত ধীচের ধীচের bয় চেচছ তাই হিপতা চেয় ার আললার কাচেছ হমায়চেনর পরান হিভbা দেচেয়চেছন২) ldquoাগাও (চেরর পরাচেনত পরানতচের ধসর (ন দেযর আান গানldquoউততর) াচেরর পতর হমায়ন কঠিন দেরাচেগ আxানত তাই ার আ দে(াচেক মমCাত (চেরর পচেথ পরানতচের আান গান ধবহিনত দোক দেসই আান গান আললার কাচেছ দেযন চে যায় আললা দেযন এই আহিতC শচেন পচেতরর ীন হিফহিরচেয় দেয় ৩)ldquoনাহিক এই (রীচেরর পাচেপর ীানচেত দেকানই তরারণ দেনই ভহি4চেতরldquoউততর) হমায়চেনর অসসথতার ন য ার হিনচেচেকই ায়ী কচেরচেছন কারন ার অচেনক রা য অন যায় ভাচে কচেরচেছ তাই তার এই পাপ কাচের ন য তার ঘচের আ হিপ এচেসচেছ এই অন যায় কাচের ন য তার মহিকত দেনই তাই ার আললার কাচেছ এই পাপ কাচেযCর ন য bমা পরাথM

Hindi 2ndlang

-ासी(जयशकर परसा-)

-ासी जयशकर परसा- की एक ऐसी कहानी ह जिजसम भारतीय ससकनित और राषटरीयता का सवरगजीतहोता ह इस कहानी म इरावती एक निहद कनया ह जिजस मलअचछो न मलतान की लट म पकडा और -ासी बना दि-या उस 500 दि-न -कर काशी क एक महाजन न खरी-ा दसरी -ासी निफरोजा ह वह गलाम ह निफरोजा को छडान क कतिलए अहम- को 1000 सोन क कतिसकक भजन थ जो अभी तक नही आए थ राजा साहब कठोर होत हए भी निफरोजा को निबना धनराकतिश क कतिलए उस म कर -त ह वनिफरोजा को अहम- को समझान की बात कहत हकहानी क अत म हम -खत ह निक इरा वती और जाटो क सर-ार बलराज का मिमलन होता हअहम- को यa म मार दि-या जाता ह वहा निफरोजा की परसननता की समामिध बनती ह वहा एक फल चढती ह और डीजल आती ह निफरोजा उस समामिध की आजीवन -ासी बनी रहती हलखक अपन उददशय अथात -ास परथा पर परकाश डालन और इस परथा क कारण होन वाल -ातो क दखो को दि-खान म पणता सफल हए ह

helliphellipContinue to next

Biology Reproductio Today we will discuss about vegetative Q1 Name some vegetative propagules

n in Organisms

propagation of plants The process of multiplication in which fragments of plant body function as propagule and develop into new individual is called vegetative propagation The units of such propagation are runner rhizome tuber bulb etc

and the speciesinvolvedVegetative propagules

Parts involved

Bulb StemBulbil BulbilRhizome Stem Runner Stem Tuber Stem Offset Stem Leaf buds Leaves Suckers Stem

Corns Stem stolon

Q2 State advantages of vegetative propagation

i) Rapid methodii) Sure and easy methodiii) Useful in plants that cannot

produce viable seeds or long seed dormancy

iv) Maintains purity of raceQ 3 Banana fruit is said to be parthenocarpic where as turkey is said to be parthenogenetic WhyBanana develops without fertilization from an unfertilized ovary thus is parthenocarpicIn turkey the ovum or female gamete developinto a new chick without fertilization thus isparthgenetic

Q4 Why is water hyacinth is called as a ldquoTerror of Bengalrdquo Water hyacinth can

propagatevegetatively all over the water body in a short per short period of time This resulted increased biochemicaloxygen oxygen demand of water body causing mortalityof fishes It is very difficult to get rid off them Thus known as terror of Bengal

Chemistry

Solid state GENERAL CHARACTERISTICS OF SOLID STATEIn nature the particular state of matter is governed by two opposing forces at given set of temperature and pressure These forces are intermolecular force of attraction and thermal energy If intermolecular force of attraction is high as compared to thermal energy particles remains in closest position

Intext QuestionsQ1 Classify the following solids as crystalline and amorphous Sodium chloride quartz glass quartz rubber polyvinyl chloride Teflon

A1 Crystalline

and hence very less movement in particles is observed In this case solid state is the preferred state of matter

Let us revise the general characteristics of solid

i) Fixed mass volume and shape

ii) Strong intermolecular force of attraction

iii) Least intermolecular space

iv) Fixed position of constituent particles

v) Incompressible and rigid

Q2 what type of interactions hold the molecules together in a polar molecular solid[CBSE 2010]A2 The molecules in a solid are held together by van der Waals forces The term van der Waals forces include hydrogen bonding dipole-dipole attraction and London dispersion forces All molecules experience London dispersion forces In addition polar molecules can also experience dipole-dipole interactions So the interactions that holds the molecule together in polar molecular solid are London dispersion force and dipole-dipole interactionsQ3 Write a feature that will distinguish a metallic solid from an ionic solid [CBSE 2010]A3 Metals are malleable and ductile whereas ionic solid are hard and brittle Metallic solid has typical metallic lustre But ionic solid looks dullQ4 Write a point of distinction between a metallic solid and an ionic solid other than metallic lustre [CBSE 2012]A4 Metals are malleable and ductile whereas ionic solid are hard and brittleQ5 Write a distinguish feature of metallic solid [CBSE 2010]A5 The force of attraction in

solid Sodium chloride Quartz Amorphous solid Quartz glass rubber polyvinyl chloride Teflon Q2 why glass is considered as super cooled liquidA2 Glass shows the tendency to flow at slower rate like liquid Hence they considered as super cooled liquidQ3 why the window glass of old buildings show milky appearance with timeA3 Glass is an amorphous solid Amorphous solid has the tendency to develop some crystalline character on heating Due to heating in day over the number of years glass acquires some crystalline character and show milky appearanceQ4 why the glass panes fixed to window or doors of old building become slightly thicker at bottomA4 Glass is super cooled liquid It has the tendency to flow down very slowly Due to this glass pane becomes thicker at the bottom over the timeQ5 Sodium chloride is a crystalline solid It shows the same value of refractive index along all the direction TrueFalse Give reasonA5 FalseCrystalline solid shows anisotropy in properties That is it shows different values for the given physical property in different direction All the crystalline solids show anisotropy in refractive index Therefore sodium chloride will show different values of refractive index on different directions

Q6 Crystalline solid are anisotropic in nature What does this statement means

between the constituent particles is special kind of electrostatic attraction That is the attraction of positively charged kernel with sea of delocalized electronsQ6 which group of solid is electrical conductor as well as malleable and ductile [CBSE 2013]A6 Metallic solidQ7 why graphite is good conductor of electricity although it is a network (covalent solid)A7 The exceptional property of graphite is due to its typical structure In graphite each carbon is covalently bonded with 3 atoms in same layer The fourth valence electron of each atom is free to move in between different layersThis free electron makes the graphite a good conductor of electricity

[CBSE 2011]A6 Anisotropy is defined asrdquo Difference in properties when measured along different axis or from different directionsrdquo Crystalline solid show different values of some of the physical properties like electrical resistance refractive index etcwhen measured along the different directions The anisotropy in crystalline solid arises due to the different arrangement of particles in different directions

Math Function Composition of functions Think of an industrial plant that produce bottles of cold drinks first there is the operation (or function) f that puts the cold drink inside the bottle followed by the opeartion g that close the bottle with the capThis leads to the following definitionDefinition Let f A rarr B and g B rarr C be two functions Then the composition of f and g denoted by gof is defined as the function gof A rarr C given by gof(x) = g(f (x)) forall x isinA

Definition A function f X rarr Y is defined to be invertible if there exists a function g Y rarr X such that gof = IX and fog = IY The function g is called the inverse of f and is denoted by f -1

Thus if f is invertible then f must be one-one and onto and conversely if f is one-one and onto then f must be invertible This fact significantly helps for proving a function f to be invertible by showing that f is one-one and onto specially when the actual inverse of f is not to be determined

Example 1 Let f 2 3 4 5 rarr 3 4 5 9 and g 3 4 5 9 rarr 7 11 15 be functions defined as f(2) = 3 f(3) = 4 f(4) = f(5) = 5 and g (3) = g (4) = 7 and g (5) = g (9) = 11 Find gofSolution We have gof(2) = g (f(2)) = g (3) = 7 gof(3) = g (f(3)) = g (4) = 7gof(4) = g (f(4)) = g (5) = 11 and gof(5) = g (5) = 11Example 2 Find gof and fog if f R rarr R and g R rarr R are given by f(x) = cos x and g (x) = 3x2 Show that gof ne fogSolution We have gof(x) = g(f(x))=g(cosx) = 3 (cos x)2

= 3 cos2 x Similarly fog(x)=f(g (x))= f(3x2)= cos (3x2) Note that 3cos2 x ne cos 3x2 for x = 0 Hence gof ne fogExample 3 Show that if f A rarr B and g B rarr C are onto then gof A rarr C is also ontoSolution Given an arbitrary element z isin C there exists a pre-image y of z under g such that g (y) = z since g is onto Further for y isin B there exists an element x in A with f(x) = y since f is onto Therefore gof(x) = g (f(x)) = g (y) = z showing that gof is onto Example 4 Let Y = n2 n isin N sub N Consider f N rarr Y as f(n) = n2 Show that

f is invertible Find the inverse of fSolution An arbitrary element y in Y is of the form n2 for some n isin N This implies that n =radicy This gives a function g Y rarr N defined by g (y) =radicy Nowgof (n) = g (n2)=radicn2 = n and fog (y) =f(radicy) = (radicy) 2 y which shows that gof=IN and fog= IY Hence f is invertible with f -1 = g

Political Science

Constitution of India-The Preamble

Summary

Objective of the state-To secure equality of status and of opportunity To promote fraternity among all the citizens To assure the dignity of the individuals and Unity and integrity of the nation

Justice-Justice stands for rule of law absence of arbitrariness and a system of equal rights freedom and opportunities for all in a society India seeks social economic and political justice to ensure equality to its citizens

Liberty-Liberty implies the absence of restraints or domination on the activities of an individual such as freedom from slavery serfdom imprisonment despotism etc The Preamble provides for the liberty of thought expression belief faith and worship

Equality-Equality means the absence of privileges or discrimination against any section of the society The Preamble provides for equality of status and opportunity to all the people of the country

Fraternity-The Preamble declares that fraternity has to assure two thingsmdashthe dignity of the individual and the unity and

Execution

Answer the following questions-

Short notes-1 Equality2 Fraternity3 Justice4 Liberty

Homework-Learn

integrity of the nation The word integrity has been added to the Preamble by the 42nd Constitutional Amendment (1976)

Business studies

Human resource management (chapter 1)

On the day of 1504 2020 I have discussed with you the managerial functions and procurement functions of HRM

Today weare going to discuss about the development function integration functions and maintenance function

Development functions-HRM improves the knowledge skills attitude and values of employees so that they the present and future jobs more effectively it includes

1) Development functions of HRM

a) Performance appraisal = It implies systematic evaluation of employees with respect to their performance on the job and their potential for development

b) Training =It is the process by which employees learn knowledge skills and attitudes to achieve organisational and personal goals

c) Executive development = It is the process of developing managerial talent through appropriate program

2) Integration functionsa) HRM reconcile the goals of

organisation with those of its members through integrating function

b) HRM tries to motivate employees to various financial and non financial incentives provided in job specification etc

3) Maintenance functiona) HRM promote and protect the

physical and mental health of employees by providing several types of benefits like housing medical aid etc

b) It Promote Social security measures to employees by providing provident fund pension gratuity maternity benefits

SubjectCOMMERCE

Topic

BUSINESSENVIRONMENT

Summary

Now quickly let us revise the earlier points that we have already done in the last class and let us proceed with the other topics that are there in the chapter

Firstly we will recall the internal and external factors of micro environment and then we

Execution 3 What do you mean by internal factors

in micro environmentAnswerInternal factors refer to all the factors existing within a business firm The internal factors are considered controllable because the enterprise has control over these factors

Development FunctionsPerformance AppraisalTrainingExecution Development

shall proceed in details

Meaning and list of internal and external factors

aInternal factorsInternal factors refer to all the factors existing within a business firm The internal factors are considered controllable because the enterprise has control over these factorsFor an example a company can alter its organization structure policies programmes employees physical facilities and marketing mix to suit the changes in the environmentList of internal factors areCorporate culture mission and objectives top management organizations structure company image and brand equity company resources

b External factorsExternal factors refer to those individual and groups and agencies with which a particular business organization comes into direct and frequent contact in the course of its functioningThese individuals and groups are known as STAKEHOLDERS because they have a stake (financial interest ) in the working and performance of the particular business List of external forces (stakeholders)Customers competitors investors suppliersmiddlemen (marketing intermediaries)financers publics

customers

suppliersfinancers

For an example a company can alter its organization structure policies programmes employees physical facilities and marketing mix to suit the changes in the environment

4 What do you mean by external factors in micro environment

AnswerExternal factors refer to those individual and groups and agencies with which a particular business organization comes into direct and frequent contact in the course of its functioningThese individuals and groups are known as STAKEHOLDERS because they have a stake (financial interest) in the working and performance of the particular business

3Who are stakeholdersSTAKEHOLDERS are individuals and groups who have a stake (financial interest ) in the working and performance of the particular business 4Discuss the internal factors in briefa Corporate CultureThe values beliefs and attitudes of the founders and top management of the company exercise a strong influence on what the cmpaany stands for how it does things and what it considers importantbMission and objectivesThe business philosophy and purpose of a comoany guide it prioritiesbusiness strategiesproduct market scope and development scope

cTop management structurethe composition of board of directors the degree of professionalization of management and the organizational structure of a company have important bearing on its business decisions

dPower structureThe internal power relationship between the board of directors and the chief executive is an important factor

e Company image and brand equityThe image and brand equity of the company play a significant role in raising finance forming alliance choosing dealers and suppliers launching new products entering foreign markets

5 What is Macro environmentAnswerMacro environment refers to the general

competitors

middlemen

publics

Fig STAKEHOLDERS OF A COMPANY

Apart from micro environment the other main dimension of business environment isMacro environment Macro environment refers to the general environment or remote environment within which a business firm and forces in its micro environment operateA company does not directly or regularly interact with the micro environmentTherefore macro environment is also known as indirect action EnvironmentThe macro environment forces are less controllable than the micro forces

Macro environment consists of the following components

POLITICAL AND LEGAL ENVIRONMENT

ECONOMIC SOCIAL AND ENVIRONMENT

CULTURAL

ENVIRONMENT

TECHNOLOGICAL ENVIRONMENT

Fig COMPONENTS OF MACRO ENVIRONMENT

environment or remote environment within which a business firm and forces in its micro environment operateA company does not directly or regularly interact with the micro environmentTherefore macro environment is also known as indirect action EnvironmentThe macro environment forces are less controllable than the micro forces 6 What are the components of macro environmenta Political and legal environmentb Economic environmentc Social and cultural environmentd Technological environment

Computer Science

Logic gates

Digital systems are said to be constructed by using logic gates These gates are the AND OR NOT NAND NOR EXOR and EXNOR

BUSINESS FIRM

gates The basic operations are described below with the aid of truth tables

AND gate

The AND gate is an electronic circuit that gives a high output (1) only if all its inputs are high A dot () is used to show the AND operation ie AB Bear in mind that this dot is sometimes omitted ie ABOR gate

The OR gate is an electronic circuit that gives a high output (1) if one or more of its inputs are high A plus (+) is used to show the OR operationNOT gate

The NOT gate is an electronic circuit that produces an inverted version of the input at its output It is also known as an inverter If the input variable is A the inverted output is known as NOT A This is also shown as A or A with a bar over the top as shown at the outputs The diagrams below show two ways that the NAND logic gate can be configured to produce a NOT gate It can also be done using NOR logic gates in the same way

NAND gate

This is a NOT-AND gate which is equal to an AND gate followed by a NOT gate The outputs of all NAND gates are high if any of the inputs are low The symbol is an AND gate with a small circle on the output The small circle represents inversion

NOR gate

This is a NOT-OR gate which is equal to an OR gate followed by a NOT gate The outputs of all NOR gates are low if any of the inputs are highThe symbol is an OR gate with a small circle on the output The small circle represents inversion

EXOR gate

The Exclusive-OR gate is a circuit which will give a high output if either but not both of its two inputs are high An encircled plus sign ( ) is used to show the EOR operation

EXNOR gate

The Exclusive-NOR gate circuit does the opposite to the EOR gate It will give a low output if either but not both of its two inputs are high The symbol is an EXOR gate with a small circle on the output The small circle represents inversion The NAND and NOR gates are called universal functions since with either one the AND and OR functions and NOT can be generated

Note A function in sum of products form can be implemented using NAND gates by replacing all AND and OR gates by NAND gates A function in product of sums form can be implemented using NOR gates by replacing all AND and OR gates by NOR gates

Logic gate symbols

Table 2 is a summary truth table of the inputoutput combinations for the NOT gate together with all possible inputoutput combinations for the other gate functions Also note that a truth table with n inputs has 2n rows You can compare the outputs of different gates

Logic gates representation using the Truth table

Example

A NAND gate can be used as a NOT gate using either of the following wiring configurations

Subject Eng Literature (The Tempest ndash William Shakespeare) Topic Act III Scene 3 Lines 53 to 110 (End of the scene) Date 16th April 2020 (2nd Period)

[Students should read the original play and also the paraphrase given in the school prescribed textbook]Summary Questions amp Answers

o Seeing this strange scene all are inclined to believe the tales told by travelers that there truly are ldquounicornsrdquo and ldquothe phoenixrsquo thronerdquo

o As they are about to sit down to the feast the banquet is snatched away by a harpy (Ariel disguised) A spiritrsquos voice (Arielrsquos voice) denounces Alonso Sebastian and Antonio with particular

1 ARIEL You are three men of sin whom Destiny

(Line 53-58)That hath to instrument this

lower world And what is int the never-surfeited sea

Hath caused to belch up you and on this island

Where man doth not inhabit you rsquomongst men

Being most unfit to live I have made you mad

reference to their crime in expelling Prospero from Milan They have not received any punishment for their deed earlier but the time for their punishment has arrived Upon Alonso it pronounces ldquolingering perdition worse than deathrdquo from which there is no remedy except through sincere repentance Ariel then vanishes in thunder and the shapes enter again and carry away the table

o Prospero watching invisibly is very pleased with the performance of Ariel and his (Prosperorsquos) ldquomeaner ministersrdquo All his enemies are now in his power and are in a fit of desperation He then leaves them and goes to see how Ferdinand and Miranda are getting on

o Alonso is now much humbled and penitent with the after effect of the spiritrsquos denunciation of his crimes He believes that his son is lost forever After this all disperse being stricken mad by the speech of the spirit

o Gonzalo fearing that they may do violence to themselves or to one another follows them and bid others to follow

(a) To whom does Ariel disguised as a harpy call the three sinners What game did Fate of Destiny play with

them

The three sinners called by Ariel are Alonso Sebastian and Antonio It was Destiny which had caused the ocean to cast the three sinners on the shore Though the ocean is all the time devouring whatever appears on its surface and is never satisfied with its continual swallowing of the ships and men in the present case the ocean had cast these three sinners on the shore without killing them

(b) Who had jointly been responsible for the conspiracy against Prospero What is Prosperorsquos purpose behind all this

Three men Alonso Sebastian and Antonio had jointly

been responsible for the conspiracy against Prospero They had driven out Prospero form Milan Prosperorsquos purpose is to make these three sinners realize the wrong they had done He wants them to repent for their criminal deeds because repentance leads to self-esteem(c )What does Ariel (the harpy) tell Alonso and his companions when they take out their swords to attack him

Seeing them drawing their swords Ariel (harpy) tells them that he and his companions are the instruments of destiny and that it is not possible for human beings to do them any injury He says that the swords of human beings can not injure even a minute part of his feathers Their swords are as ineffective against him and his companions as against the wind or the water

(d) Give the explanatory meanings of the following expressions in the context of the above extract

(i)Never surfeited (ii) Belch up (iii) lsquomongst men

(i) Never surfeited never led to satisfaction

(ii) Belch up cast ashore(iii) lsquomongst men in human

society2

I and my fellows (Line 60-65)

Are ministers of Fate The elementsOf whom your swords are tempered may as wellWound the loud winds or with bemocked-at stabsKill the still-closing waters as diminishOne dowl thats in my plume

IMPORTANT PASSAGES EXPLAINED

The elements

(Line 61-66)Of whom your swords are tempered may

as wellWound the loud winds or with

bemocked-at stabs

(a) Who is lsquoIrsquo Who are his lsquofellowsrdquo

lsquoIrsquo is referred to Ariel in disguise of a harpy His lsquofellowsrsquo are other spirits serving Prospero the real Duke of Milan who has acquired supernatural powers after being banished from his Dukedom Prospero has settled in this uninhabited island

(b) What are the elements that have temperrsquod the swords Why will it not work against the speaker

The swords (of Alonso and his companions) are tempered by metal (steel) which is taken out of the earth and refined by

Kill the still-closing waters as diminishOne dowl thats in my plume My fellow

ministersAre like invulnerable

In these words Ariel reminds the King and his companions of the utter futility of drawing swords against himself and his fellows Ariel drives Alonso Antonio and Sebastian the three men of sin to desperation ndash a state in which men do violence to themselves They draw swords to strike Ariel But Ariel reminds them that he and the other spirits are the ministers of destiny and nothing can wound them The steel of which their swords are made of may cut the wind or water which being divided always closes up again Even supposing that such things may be possible it is quite impossible that their swords will cut one feather in their plume They are incapable of being wounded by any sword of man Hence it is foolish on their part to attempt to strike at Ariel and his fellow-spirits

For which foul deed

(Line 72-75)The powers delaying not forgetting

haveIncensed the seas and shores yea all the

creatures Against your peace

Ariel enters like a harpy and remaining invisible tells Alonso Sebastian and Antonio that he and other harpies are the agents of Destiny appointed to carry out her decrees He tells them that their punishment for the crime against Prospero which has been so long deferred is now to fall upon them He reminds them that they had expelled Prospero from Milan and set him and his innocent child adrift on the sea and that the sea had paid them back for their sin by the shipwreck and by the calamities they have suffered He tells them that the powers above which did not forget this mean treachery but only deferred the punishment have now engaged the seas and the shores and all living beings including him and his comrades against them The very elements and supernatural agency Ariel adds have taken up the avenging of their crime against Prospero

the action of fire It may cut the wind or water which being divided always closes up again

The sword will not work against the spirits and the harpy because they are the ministers of destiny and nothing can wound them nor it will cut a single feather in their plume

(c )What is the meaning of lsquodowlrsquo in the last line

The term lsquodowlrsquo means a filament or the smallest part of a feather In this context Ariel in disguise of harpy says that their sword cannot even damage the smallest filament of their (Arielrsquos and other spirits) feathers as they are incapable of being wounded by any sword of man

(d) What does the speaker remind the listeners about

Ariel in disguise of harpy reminds Alonso the King of Naples Sebastian Alonsorsquos brother and Antonio the present Duke of Milan and the treacherous brother of Prospero as they being three men of sin He even reminds them that their punishment for their crime against Prospero which has been so long deferred now falls upon them He reminds them that they have expelled Prospero from Milan and has set him along with his innocent infant daughter adrift on the sea So the sea has paid them back for their sin by their shipwreck and the calamities they have suffered since then The harpy rebukes Alonso of his sin that has incensed the Gods and has deprived him of his son as a punishment

(e) How do they respond

When Ariel in disguise of a harpy reminds Alonso Sebastian and Antonio of their past misdeeds and sin Alonso has a look of terror and confusion in his eyes He utters the words of sincere repentance wrung out of his conscience-stricken heart It appears to him that all the elements of nature the sea-waves the wind and the thunder proclaiming a loud voice in the name of Prospero and the crime Alonso has committed against him They are calling upon him to repent There is a deep storm raging in Alonsorsquos breast and the echoes of that storm are ringing in his ears like a clear note of wind-instrument A note of denunciation of Alonsorsquos crime leaves him much humbled and penitent and confirms his belief that his son is lost forever But Sebastian and Antonio shows some courage instead of repentance They wish to kill the spirits or devils if it appears

3

Of my instruction hast thou nothing bated (Line 85-93)

In what thou hast to say So with good life

And observation strange my meaner ministers

Their several kinds have done My high charms work

And these mine enemies are all knit upIn their distractions They now are in my

powerAnd in these fits I leave them while I visitYoung Ferdinand whom they suppose is

drownedAnd his and mine loved darling

Methought the billows spoke and (Line 96-99)

told me of itThe winds did sing it to me and the

thunderThat deep and dreadful organ-pipe

pronouncedThe name of Prosper It did bass my

trespass

These are the words of contrition coming from Alonso Ariel has driven him to a deep repentance for conspiring with Antonio against Prospero He now feels a sincere remorse It appears to him that all the elements of nature the sea-waves the wind and the thunder proclaimed with a loud voice the name of Prospero and the crime Alonso had committed against him They are calling upon him to repent There is a deep storm raging in Alonsorsquos breast and the echoes of that storm are ringing in his ears like the clear note of a wind-instrument

Comment These are the words of sincere repentance wrung out of the conscience-stricken heart of Alonso Alonso who is the lesser villain is the first to give way to remorse under the effect of Arielrsquos speech The words of Ariel seem to him to be the voice of conscience speaking to him He is driven to desperation a state in which he might do violence to his life

(a) Identify the speaker State the context

Prospero the ruler of the island is the speaker The famous banquet scene has been enacted very well Ariel and his junior spirits have played their roles excellently Prospero is glad to say words of praise for them(b) In what way the speakerrsquos instructions have been carried out

According to Prosperorsquos instructions a banquet was presented before the King of Naples and his companions when they were tired and hungry Just when they were preparing to eat the feast the banquet was suddenly removed by exercising supernatural powers All this was done by Ariel Prosperorsquos chief assistant and a powerful spirit

Ariel not only made the feast disappear but also delivered his speech blaming the King and his two companions for their past wicked deeds He warned them to repent for their misdeeds or suffer forever on that uninhabited island

(c) Who are referred to as lsquomeaner ministersrsquo What have they done

Prospero refers as lsquomeaner ministersrsquo to his other lesser spirits who were assisting Ariel in presenting a scene before the kingrsquos party They entered the scene to the accompaniment of music They assumed several strange shapes and brought in a banquet Then they danced about it with gentle actions of salutations thus inviting the King and others to eat the feast

These spirits play their role again when Ariel in the shape of a harpy quits the scene These shapes enter again and dancing with mocking gestures carry away the table

(d) Who are the speakerrsquos enemies What has happened to them

King of Naples Alonso his brother Sebastian and the present Duke of Milan Antonio (Prosperorsquos own brother) are Prosperorsquos enemies With the turn of events they have all been washed ashore on the island which is ruled by Prospero the great magician Actually this happened after the shipwreck caused by a storm which was raised by Prospero with the purpose of bringing these people to his island Prosperorsquos spirits have already confused and terrified these enemies and they are under Prosperorsquos control He can treat them as he likes

(e) What does he say about Ferdinand Explain what is meant by ldquohellip his and mine darlingrdquo

Prospero knows that Alonsorsquos son prince Ferdinand is alive though his father thinks that the prince has been drowned

Prospero refers to his daughter Miranda who is dear to him She is also very dear to Prince Ferdinand who has fallen in love with her They are waiting to be married soon for which they have received Prosperorsquos consent

4

ALONSO O it is monstrous monstrous (Line 95-102)

Methought the billows spoke and told me of it

The winds did sing it to me and the thunderThat deep and dreadful organ-

pipe pronouncedThe name of Prosper It did bass

my trespassTherefore my son ithrsquo ooze is

bedded andIll seek him deeper than eer

plummet soundedAnd with him there lie mudded

(a) In what way does Alonso express his horror when his conscience is awakened by Arielrsquos words

When Alonsorsquos conscience is awakened by Arielrsquos words he expresses his horror at what he has heard He gets the feeling that the waves of the ocean the wind and the loud thunder have spoken to him and uttered the name of Prospero Because of being reminded of his crime in a very loud and rough voice he comes to realize that he has lost his son for his past misdeeds

(b) What does Alonso imagine about his son What does Alonso want to do in his desperate state

Alonso imagines that his son is lying in the mud at the bottom of the sea He feels desperate that he wants to drown himself in the ocean deeper than the plumb-line has ever gone He wants to lie with his son at the bottom of the sea

(c) How do Sebastian and Antonio want to face the evil spirits

Sebastian says that he is not at all afraid of what the harpy has said and that he is prepared to fight any number of such monsters if they appear before him only one at a time Antonio says that he would support Sebastian in the fight against the fiendsyyy

(d) Why does Gonzalo ask Adrian to follow the three men

Gonzalo tells Adrian that all the three men namely Alonso Sebastian and Antonio are in a wild and reckless mood The thought of the heinous crime of which they are guilty has begun to torment their minds So he asks Adrian to follow those three men without loss of time and prevent them from doing anything which the turmoil in their minds might lead them to do

(e) What opinion do you form of Alonso from the above extract

Alonso who is the lesser villain is the first to give way to remorse under the effect of Arielrsquos speech The words of Ariel seem to him to be the voice of conscience speaking to him He is driven to desperation a state in which he might do violence to his life

Subject =Accounts

Ac-12 15420 topic-pL Appropriation ac

PROFIT AND LOSS APPROPRIATION ACCOUNT

MEANING AND PREPARATIONProfit and Loss Appropriation Account is merely an extension of the Profit and Loss Account of the firm The profit of the firm has to be distributed amongst the partners in their respective profit sharing ratio But before its distribution it needs to be adjusted All Adjustments like partnerrsquos salary partnerrsquos commission interest on capital interest on drawings etc are made in this account These adjustments will reduce the amount of profit for distribution This adjusted profit will be distributed amongst the partners in their profit sharing ratio To prepare it at first the balance of Profit and Loss Account is transferred to this account The journal entries for the preparation of Profit and Loss Appropriation Account are given below

1 for transfer of the balance of Profit and Loss Account to Profit and Loss Appropriation Account

(a) In case of Net Profit

Profit and Loss Ac helliphelliphelliphelliphellipDrTo Profit and Loss Appropriation Ac(Net Profit transferred to Profit and Loss Appropriation Ac)

(b)In case of Net Loss

Profit and Loss Appropriation Achelliphelliphellip DrTo Profit and Loss Ac(Net Loss transferred to Profit and Loss Appropriation Ac)

2 for Interest on Capital

For transferring on Interest on CapitalProfit and Loss Appropriation Achelliphelliphellip DrTo Interest on Capital Ac(Interest on capital transferred to Profit amp Loss Appropriation Ac)

3 for Interest on Drawings

For transferring Interest on Drawings Interest on Drawings Achelliphelliphelliphelliphelliphellip DrTo Profit and Loss Appropriation Ac(Interest on drawing transferred to Profit amp Loss Appropriation Ac)

4 For Partnerrsquos SalaryFor transfer of partnerrsquos SalaryProfit and Loss Appropriation Achelliphellip DrTo Salary Ac(Salary transferred to profit amp Loss Appropriation Ac)

5 For Partnerrsquos CommissionFor transferring commissionProfit and Loss Appropriation Achelliphelliphellip DrTo Commission Ac(Commission transferred to Profit and Loss Appropriation Ac)

6 For Transfer of agreed amount to General ReserveProfit and Loss Appropriation Ac helliphellipDrTo General Reserve Ac(Transfer to General Reserve)

7 for share of Profit or Loss appropriation(a) If ProfitProfit and Loss Appropriation Achelliphellip DrTo Partnerrsquos CapitalCurrent Ac(Profit transferred to capitalcurrent Ac)(b) If LossPartnerrsquos Capital Current Achelliphelliphelliphellip DrTo Profit and Loss Appropriation Ac(Loss transferred to capitalcurrent Ac)

THE FORMAT OF PROFIT AND LOSS APPROPRIATION

Profit and Loss Appropriation Account for the year endedhelliphelliphelliphellip

Particulars Amount Particulars Amount

To PL Ac (loss) By pL Ac (profit)

To Interest on capital BY Interest on drawings

To partner`s commission by Partner`s capital Ac ( loss)

To Partner`s salary To Interest on partner`s loan To General Reserve To Partner`s Capital AC (Profit)

Subject= Economics

MOVEMENT ALONG THE DEMAND CURVE (CHANGE IN QUANTITY DEMANDED)In law of demand you have already studied the inverse relationship between price and quantity demanded When quantity demanded of a commodity changes due to change in its price keeping other factors constant it is called change in quantity demanded It is graphically expressed as a movement along the same demand curve There can be either a downward movement or an upward movement along the same demand curve Upward movement along the same demand curve is called contraction of demand or decrease in quantity demanded and downward movement along the same demand curve is known as expansion of demand or increase in quantity demanded

Extention of demandd

price (rs)p A

B Extentionp1 d

Q Q1

Quantity demanded ( in units)

Contraction of demandd

p2 Ccontraction

p APrice (Rs)

d

Q2 Q

Quantity demanded (in units)

Explanation of movement of demand A fall in price from OP to OP1 leads to increase in quantity demanded from OQ to OQ1 (expansion of demand) resulting in a downward movement from point A to point B along the same demand curve DD When Price rises from OP to OP2 quantity demanded falls from OQ to OQ2 (contraction of demand) leading to an upward movement from point A to point C along the same demand curve DD

  • Activity Series of Metals
    • Drawbacks of Rutherfordrsquos model of atom
      • Electromagnetic radiations
      • Properties of electromagnetic radiations
      • Characteristics of electromagnetic radiations
        • Plancks Quantum Theory-
        • Photoelectric effect
          • Intext Questions
            • Logic gates
            • Digital systems are said to be constructed by using logic gates These gates are the AND OR NOT NAND NOR EXOR and EXNOR gates The basic operations are described below with the aid of truth tables
            • AND gate
            • Example
Page 32:  · Web viewSubject. Topic. Summary. Execution. English 1 . Chapter 1 naming words . Page 8. Write the names of these pictures:- Person:-1. father. 2.Firefighter 3.doctor 4 ...

ii) cos(A+B) Cos(A-B) = cos2 A- sin2 B = cos2 B -sin2 AProof i) LHS= sin(A+B)sin(AminusB) [Recall sin(αminusβ)=sinαcosβminuscosαsinβ And sin(α+β)=sinαcosβ+cosαsinβ]= (sinAcosB+cosAsinB)times(sinAcosBminuscosAsinB)= sin2Acos2Bminuscos2Asin2B [Recall sin2α+cos2α=1 From above we can then assume correctly that sin2α=1minuscos2α AND cos2α=1minussin2α] = sin2A(1minussin2B)minussin2B(1minussin2A) = sin2Aminussin2Asin2Bminussin2B+sin2Asin2B = sin2Aminussin2B= 1-cos2A-(1-cos2B) = cos2 B- cos2 A = RHSii)LHS= cos (A+B) cos(A-B) [ cos(A+B) = cos AcosB- sinAsinBCos(A-B) = cosAcosB+ sinAsinB]= cos2 A Cos2 B- sin2 A Sin2 B= cos2 A( 1-sin2 B) - (1- cos2 A) sin2 B= cos2 A- cos2 A sin2 B- sin2 B+ cos2 A sin2 B=cos2 A- sin2 B=1- sin2 A-(1-cos2 B) = cos2 B- sin2 A= RHSTangent formulae for compound anglesi)tan (A + B) = tan A + tan B1-tan A tan Bii)tan (A ndash B) = tan A-tan B1+tan A tan Biii) cot (A + B) = cot Acot B-1cot A+cot B(viii) cot (A ndash B) = cot Acot B+1cot B-cot A

A tan B) Or 1= (tan A+ tanB) (1-tan A tanB) Or tanA + tanB + tanA tanB + 1 = 1 + 1Or tanA (1 + tanB) + (1 + tanB) = 2Or (1 + tanA) (1 + tanB) = 2Example 3 Find the value of sin 15degSolution sin 15deg= sin(45deg-30deg) = sin45degcos 30deg- cos45degsin30deg =(1radic2) (radic32) -(1radic2) (12) = (radic3-1) 2radic2Example 4 If sin A = 1 radic10 and sin B = 1 radic5 where A and B are positive acute angles then what is A + B SolutionWe know that sin (A + B) = sin A cos B + cos A sin B= [1 radic10] [radic(1 minus 1 5)] + [1 radic5] radic(1 minus 1 10)= [1 radic10] [radic4 5] + [1 radic5] [radic9 10]= [1 radic50] times (2 + 3)= 5 radic50 = 1 radic2

sin (A + B) = sin π 4rArrHence A + B = π 4Example 5 If A + B = 225o then find [cot A] [1 + cotA] times [cot B] [1 + cot B]Solution[cot A] [1 + cotA] times [cot B] [1 + cot B] = 1 [(1 + tan A) times (1 + tan B)]=1 [tan A + tan B + 1 + tan A tan B] [ tan (A + B) = tan225o]∵

tan A + tan B = 1minus tan A tan BrArr= 1 [1 minus tan A tan B + 1 + tan A tan B]= 1 2

COMMERCE

CLASSIFICTION OF HUMAN ACTIVITIES-ECONOMIC AND NON-ECONOMIC

Firstly we shall recall the previous class for 5 mins especially for the absentees and for also the rest of the students who were there

Today at first we briefly discuss the earlier portions of the chapter

1Business-It includes all those economic activities which are concerned with production and exchange of goods and services with the object of earning profit Example A factory shop beauty parlour also business enterprises

2Profession ndashThe term profession means an occupation which involves application of specialized knowledge and skills to earn a living For Example Chartered Accountancy medicine law tax consultancy are example of professions

Questions1What are the main features of ProfessionAnswer The main features of a profession are as follows a Specialised body of knowledge-Every profession has a specialised and systematised body of knowledge b Restricted entry- Entry to a profession is allowed only to those who have completed the prescribed education and have the specialised examination c Formal education and training ndashA formal education and training is given to the person who wants to acquire the professional

3Employment-Employment mean an economic activity where people work for others in exchange for some remuneration (salary)The persons who work for others are called lsquoemployeesrsquo The persons or organizations which engage others to work for them are called lsquoemployersrsquoEg A doctor working in a hospital is employment as he is working for a salaryA lawyer may serve as a law officer in a bank

With this we shall proceed with the features of both Profession amp Employment

The main features of a profession are as follow

a Specialised body of knowledge b Restricted entry c Formal education and training d Professional association e Service motive f Code of contact

The main features of an employment are as follows

a In employment a person works for others called employer

b An employee provides personal service

c There is a service agreement or contract between the employee and the employer

d The employee has to obey the order of the employer

e No capital investment is made by the employer

Various examples of Employment are as follows

aA teacher teaching in a school or collegeb An engineer employed in Municipal Corporation of DelhicAn accountant working in the accounts department of a companydA doctor working in a hospital

Note In all the above examples of employment the individual who is involved in each example is working as an employee for a salary under an employer

qualification(MBBSCALLB)d Service motive ndashProfessionals are expected to emphasis service more on their clients rather than economic gain f Code of Conduct-The activities of professionals are regulated by a code of conduct

2 What are the main features of EmploymentAnswer The main features of an employment are as followsa In employment a person works for others called employerb An employee provides personal servicec There is a service agreement or contract between the employee and the employerd The employee has to obey the order of the employere No capital investment is made by the employer

3 Give various Professions and their respective Association are given below

Professions

Professional

Professional association

Medical profession

Doctor Medical Council of India

Law profession

Lawyers Bar Council of India

Accounting Profession

Chartered

The Institute of Chartered Accounts of India( ICAI)

Engineerin Engineers The

g Profession

institute of Engineers (India)

Accounts Basic accounting terms

Today we will give you some questions from the previous study material

Questions6) Define accounting7) What do you mean by debit

and credit8) Explain the types of account9) Define the following terms

a) Assetsb) Capitalc) Purchased) Debtorse) Transactions

10) Name the types of accounts given below

a) Krishnas accountb) Machinery accountc) Royalty accountd) Salary accounte) Furniture accountf) Audit fee account

Economics Basic Economic ConceptsSub topic

UTILITY

Before starting todayrsquos class we shall recall the last class which was about UTILITY AND THE FEATURES OF UTILITY

Now we shall proceed with the further topics of the chapter

Todayrsquos topic from the chapter lsquo Basic Economic Conceptsrsquo will be TOTAL UTILITY amp MARGINAL UTILITYNow let us quickly revise the concept of utility with an example ie goods and services are designed because they have an ability to satisfy human wantsThis feature of being able to satisfy human wants is termed as utility For example we derive utility from WiFi services as it gives us satisfaction by connecting us to our friends and family through social media here consumers derive utility from WiFi services

From the above concept we shall start with todayrsquos topicEconomists have defined TOTAL UTILITY (TU) as the total satisfaction obtained by consuming a given total amount of a good and serviceFor example the total satisfaction obtained from eating 10 mangoes is the total utility of 10 mangoes

MARGINAL UTILITY (MU) is the additional satisfaction derived from each additional unit

Questions1 What is Total Utility (TU)

Answer Total Utility (TU) is the

aggregate of the utility that a consumer derives from the consumption of a certain amount of a commodityTU=MU1+MU2++MUn

2 What is Marginal UtilityAnswer

Marginal Utility (MU) is the additional made to the total utility as consumption is increased by one more unit of the commodityMU= TUn ndashTUn-1

NoteOften economists tend to

subdivide utility into an imaginary unit called UTIL

consumed In this casethe utility obtained from each mango as it is consumed as the MU of that mango It is also defined as the addition made to the total utility when an additional unit is consumed Often economists tend to subdivide utility into an imaginary unit called UTIL

Note As a consumer increases the consumption of a good over period of time the total utility or total satisfaction derived from it increases to appoint and thereafter it decreasesHowever as the consumer keeps on consuming the good the marginal utility or the additional utility derived from it decreases

SubjectBusiness studies

Topic

BUSINESSENVIRONMENT

Summary

Now quickly let us revise the earlier points that we have already done in the last class and let us proceed with the other topics that are there in the chapter

Firstly we will recall the internal and external factors of micro environment and then we shall proceed in details

Meaning and list of internal and external factors

aInternal factorsInternal factors refer to all the factors existing within a business firm The internal factors are considered controllable because the enterprise has control over these factorsFor an example a company can alter its organization structure policies programmes employees physical facilities and marketing mix to suit the changes in the environmentList of internal factors areCorporate culture mission and objectives top management organizations structure company image and brand equity company resources

b External factorsExternal factors refer to those individual and groups and agencies with which a particular business organization comes into direct and frequent contact in the course of its functioningThese individuals and groups are known as STAKEHOLDERS because they have a stake (financial interest ) in the working and performance of the particular business List of external forces (stakeholders)Customers competitors investors suppliersmiddlemen (marketing intermediaries)

Execution 1 What do you mean by internal

factors in micro environmentAnswerInternal factors refer to all the factors existing within a business firm The internal factors are considered controllable because the enterprise has control over these factorsFor an example a company can alter its organization structure policies programmes employees physical facilities and marketing mix to suit the changes in the environment

2 What do you mean by external factors in micro environment

AnswerExternal factors refer to those individual and groups and agencies with which a particular business organization comes into direct and frequent contact in the course of its functioningThese individuals and groups are known as STAKEHOLDERS because they have a stake (financial interest) in the working and performance of the particular business

3Who are stakeholdersSTAKEHOLDERS are individuals and groups who have a stake (financial interest ) in the working and performance of the particular business 4Discuss the internal factors in briefa Corporate CultureThe values beliefs and attitudes of the founders and top management of the company exercise

financers publics

customers

suppliersfinancers

competitors

middlemen

publics

Fig STAKEHOLDERS OF A COMPANY

Apart from micro environment the other main dimension of business environment isMacro environment Macro environment refers to the general environment or remote environment within which a business firm and forces in its micro environment operateA company does not directly or regularly interact with the micro environmentTherefore macro environment is also known as indirect action EnvironmentThe macro environment forces are less controllable than the micro forces

Macro environment consists of the following components

POLITICAL AND LEGAL ENVIRONMENT

ECONOMIC SOCIAL AND ENVIRONMENT

CULTURAL

ENVIRONMENT

TECHNOLOGICAL ENVIRONMENT

a strong influence on what the cmpaany stands for how it does things and what it considers importantbMission and objectivesThe business philosophy and purpose of a comoany guide it prioritiesbusiness strategiesproduct market scope and development scope

cTop management structurethe composition of board of directors the degree of professionalization of management and the organizational structure of a company have important bearing on its business decisions

dPower structureThe internal power relationship between the board of directors and the chief executive is an important factor

eCompany image and brand equityThe image and brand equity of the company play a significant role in raising finance forming alliance choosing dealers and suppliers launching new products entering foreign markets

5 What is Macro environmentAnswerMacro environment refers to the general environment or remote environment within which a business firm and forces in its micro environment operateA company does not directly or regularly interact with the micro environmentTherefore macro environment is also known as indirect action EnvironmentThe macro environment forces are less controllable than the micro forces 6 What are the components of macro environmenta Political and legal environmentb Economic environmentc Social and cultural environmentd Technological environment

BUSINESS FIRM

Fig COMPONENTS OF MACRO ENVIRONMENTPolitical science

Introduction to political science

Comparative politics and itrsquos scope Comparative politics is the second major dimension of political scienceIt is also a very vast area of study and a very large number of political scientists even treat it as an autonomous area of study within the board ambit of political scienceScope of comparative politics-

1 All political structures -Comparative politics includes the study of all structures formalnon formal governmental and extra governmental which are directly or indirectly involved in politics in all the countries of the world

2 Functional studies- Comparative politics seeks to study politics less from the point of view of the legal institutions in terms of their powers and move from the point of view of their functions which constitute the political process and their actual Operation in the environment

3 Study of political behaviour- Another important part of its scope is the study of the actual behaviour of the people in the process of politics

4 Study of similarities and differences- comparative politics also undertakesan analysis of the similarities and differences among political process and functions

5 Study of all political systems -comparative politics seeks to analyse the actual behaviour and performance of all political systems western as well as non western

6 Study of the environment and infrastructure of politics-The study of politics demands a study of the psychological sociological economic and anthropological environment in fact the social environment as a whole in which each political system operates

7 Study of political culture- political culture is composed of attitudesbeliefs emotions and values of a society that relate to the political system or politics

8 Study of political participation- Political participation is a universal processThe only difference is that while in some states it is limited in others it is wider

9 Study of political process- political

Answer the following questions-

What is comparative politics

What are the scope of comparative politics

Homework- learn

processes like decision makingpolicy making judicial process leadership recruitment process and others are always at work in all political systems

The scope of comparative politics is very comprehensive It includes everything that falls within the area of political activity and political process

History CAMBRIDGE VIEW ABOUT

THE PARTITION

AND REFUTATION

OF CAMBRIDGE

VIEW

Cambridge view about the Partition The Cambridge school of historians have interpreted that opposition to partition scheme was made entirely by the elitist groups They hold the view that Lord Curzon planned to partition the Bengal for administrative purposeREFUTATION OFCAMBRIDGE VIEW The Rationalist historians have rejected the interpretations of the Cambridge School of historians on various grounds

1 QUESTION State different views of historians regarding Partition of Bengal

ANSWER Cambridge historians believed that Lord Curzon partitioned Bengal for administrative reasons only and not for the political motive The Middle class elitist group protested because of their petty interest The Hindu zamindars protested as they have to spend more money for managing their estatesThe lawyers of Calcutta High court feared to lose their clientBut according to the nationalist Historians was-

2- The ultimate object of Lord Curzon was to crush the unity of Bengal politicians

3- If Bengal becomes a separate province Bengali speaking 16 million people of western part would become minority under Hindi speaking people of Bihar and Oriya speaking people of Orissa

4- The bureaucrats expected that the protest movement would die down quickly

5- Lord Curzon used the Muslim community in his political game

6- Idealism had great contribution in the protest against partition

7- The people of the every section of society were affected by the partition of Bengal

Computer Science

Numbers Convertion of dcimal number to octal numberThe decimal numeral system is the standard system for denoting integer and non-integer numbers It is the extension to non-integer numbers of the Hindu-Arabic numeral system For writing numbers the decimal system uses ten decimal digits a decimal mark and for negative numbers a minus sign - The decimal digits are 0 1 2 3 4 5 6 7 8 9 the decimal separator is the dot in many countries

The octal numeral system or oct for short is the base-8 number system and uses the digits 0 to 7 Octal is sometimes used in computing instead of hexadecimal perhaps most often in modern times in conjunction with file

permissions under Unix systems It has the advantage of not requiring any extra symbols as digits It is also used for digital displays

Follow these steps to convert a decimal number into octal form

1 Divide the decimal number by 82 Get the integer quotient for the next iteration (if the number will not divide equally by 8 then round down the

result to the nearest whole number)3 Keep a note of the remainder it should be between 0 and 74 Repeat the steps until the quotient is equal to 05 Write out all the remainders from bottom to top This is the solution

For example if the given decimal number is 8453

Division Quotient Remainder

8453 8 1056 5

1056 8 132 0

132 8 16 4

16 8 2 0

2 8 0 2

Then the octal solution is 20405

Subject Eng Literature (The Tempest ndash William Shakespeare) Topic Act I Scene 1 Lines 33 to 67 (End of scene) Date 16th April 2020 (4th Period)

[Students should read the original play and also the paraphrase given in the school prescribed textbook]Summary Questions amp Answers

[SUMMARY OF THE ENTIRE SCENE]

o The play starts with the scene of a severe storm at sea Alonso (King of Naples) Sebastian (Alonsorsquos brother) Ferdinand (Alonsorsquos son) Gonzalo Antonio (the usurping Duke of Milan) are in a ship in the midst of the storm

o The mariners are trying their best to control the vessel from running aground and are totally following the orders of their Master the Boatswain They have scant success

o The mariners become extremely unhappy and annoyed when most of the passengers arrive on the deck thereby hampering their effort to save the ship There is serious confrontation between them and the passengers who are part of the Kingrsquos entourage

o The mariners could not save the ship

SUMMING-UP

(i) Vivid description of the scene which gives a realistic description of terror and confusion of a tropical storm

(ii) Shows Shakespearersquos accuracy of knowledge in describing the naval operations and also matters of seamanship

(1) GONZALO Ill warrant him for drowning (L 45-57)

though the ship were no stronger than a nutshell and as leaky as an unstanched

wenchBOATSWAIN Lay her a-hold a-hold Set her two courses Off to

sea again lay her offMARINERS All lost To prayers to prayers All lostBOATSWAIN What must our mouths be coldGONZALO The king and prince at prayers Lets assist them

For our case is theirsSEBASTIAN Im out of patienceANTONIO We are merely cheated of our lives by drunkards

This wide-chopped rascal - would thou mightst lie drowning the washing of ten tides

(a) What does Antonio say at the insolent manners of the boatswain just before the given passage

Being irritated at the insolent manners of the boatswain just before the given extract Antonio the Duke of Milan calls him a worthless dog son of a woman without any morals an arrogant and disrespectful noisemaker He says that the boatswain deserved to be hanged(b) What statement does Gonzalo repeat about the boatswain

Gonzalo shows his faith that the boatswain is not destined to die by drowning He is destined to be hanged and nothing can alter this decree of destiny He says that even if the ship was as frail as a nutshell the boatswain could not be drowned for his destiny was to be hanged(c) What do the passengers do when they have lost all hope of their survival

When the passengers have lost all hope of survival they take

(iii) The opening scene justifies the title ndash The Tempest

UNANSWERED QUESTIONS

(i) The King always travels with his entire fleet including his soldiers Where were the other ships

(ii) Why was the ship in that area Where was it coming from or going where

(iii) The ship broke apart What happened to those who were in the ship

(We shall get the answer to the above questions as the play progresses)

leave of life with fervent prayers The mariners take their last hearty drink and are ready for death(d) What blame does Antonio put upon the mariners and the boatswain Antonio rebukes the mariners that these drunkards have brought them to the present crisis by neglecting their duties He blames them saying that they are going to lose their lives entirely for the negligence of the boatswain and his fellows(e) What does Antonio say while cursing the boatswain

Antonio gives vent to his wrath upon the boatswain in particular He calls the boatswain a wide-mouthed rascal who deserves to be hanged on the sea-shore at low water mark so that ten tides might wash over his body and take out of him all the liquor that he has been drinking

Class XIISubject Topic Summary ExecutionHistory Topic

1 1935 ACT AND WORKING OF PROVINCIAL AUTONOMYCONGREE AND OTHER MINISTERSSUB TOPIC GOVERNMENT OF INDIA ACT1935

Government of India Act 1935 This act established a lsquoFederation of Indiarsquo made of British Indian provinces and Indian states and provided for autonomy with a government responsible to the elected legislature in every provinceThis act introduced abolition of Diarchy at provinces The entire provincial administration was introduced to the responsible ministers who were controlled and removed by the provincial legislature The provincial autonomy means two things First The provincial governments were wholly responsible to the provincial legislature Secondly Provinces were free from outside control and interference in the large number of matters The act divided the powers between the centre and provinces in terms of three lists- Federal list( for centre) Provincial list (for province) and concurrent list (for both) Residuary powers were given to the viceroy In the election under the government of India Act the Congress swept the poll the mandate of the people came in favour of the congress so far as general Hindu seats were concerned The Congress did not get a single Muslim seates in Bombay CP UP Sind and BengalIn five provinces Congress had yhe clear majority In BengalNWFPAssam and Bombay Congress emerged as a single largest partyOn the other side the performance of the Muslim League was badThus the Congress formed ministers in 7 provinces out of 11 provinces Coalition ministry was also formed in two other provincesOnly BENGAL AND Punjab had non- congress ministries

1 QUESTION What was the main change introduced by the Government of India ActANSWER a) The Act gave more

autonomy to the provinces b) Diarchy was abolished at the

provincial levelsc) The Governor was the head of

the executived) There was a council of

ministers to advise him The ministers were responsible to the provincial legislatures who controlled them The legislature could also remove the ministers

e) The Governors still retained special reserve powers

2 QUESTION Why did the federal scheme introduced by the Government of India Act 1935 never come into operation

ANSWER The Federal structure of the Government of India was to be composed with the Governor General and Council of ministers The Federal legislature was to be Bicameral legislature- The council of states and the House of Assembly The ministers were to be chosen by the Governor general and they were to hold the office during his pleasure

The provinces of British India would have to join the federation but this was not compulsory for the princely states

This federation never materialised because of the lack of support from the required number of

princely statesThis act was refused and

rejected by the princes the Congress and the Muslim League

Thus both Congress and the League participated in the election of 1937 Thus the federal part was never introduced but the provincial part was put into operations

Bengali 2nd

Language

াচেরর পরাথCনা(কহিতা )

াচেরর পরাথCনা কহিতাটি কহি (ঙখ দেঘাচে4র দো আচো য কহিতায় াচেরর পতর হমায়ন কঠিন দেরাচেগ আxানত ার ঈশবর া আললার কাচেছ পরাথCনা কচেরচেছন তার পচেতরর ীন হিফহিরচেয় হিচেত এই কহিতায় ার পচেতরর ীন হিভbা দেচেয়চেছন ারার এমনহিক হিনচের ীন হিসCচেনর হিহিনমচেয় হিতহিন তার দেছচের ীন হিফচের দেপচেত দেচেয়চেছন তার দেছচের এই দেরাচেগর ন য হিতহিন হিনচেচেকই ায়ী কচেরচেছন তার হিনচের করা পাপচেকই হিতহিন ায়ী কচেরচেছন এছাা রানৈনহিতক ও আথCসামাহিক অসথার কথা তচে ধরা চেয়চেছ এই কহিতায় ার তার হিনচের পাপ কমCচেকই ায়ী কচেরচেছ ার অন যায় ভাচে দেপহি((হিকতর মাধ যচেম অপররা য কচেরচেছ আর এই অন যায় কাচের ন যই তার পহিরাচের হিপযCয় এচেসচেছ দে এক পরকার মানহিক নধন ইহিতাচেসর ার হিপতা চেয় সবাভাহিকভাচে ভাচোাসা দে মমতা দেথচেক মকত চেত পাচেরনহিন তাই হিপতা চেয় আললা া ভগাচেনর কাচেছ পতর হমায়চেনর পরানহিভbা দেচেয়চেছন ার আললা া ভগাচেনর কাচেছ াহিনচেয়চেছন তার হিনচের ীন হিসCন হিচেত হিতহিন রাী তার হিহিনমচেয় পচেতরর ীন হিফচের দেপচেত দেচেয়চেছন াচেরর হিপতসভ হিচেকর কথা এই কহিতায় ফটিচেয় দেতাা চেয়চেছ হিপতা পচেতরর হিরাহিরত মান নধচেনর কথা তচে ধরা চেয়চেছ

হিচে(4 হিকছ াইচেনর তাৎপযC১) ldquoদেকাথায় দেগ ওর সবচছয দেৌন দেকাথায় কচেরায় দেগাপন bয়ldquoউততর) াচেরর পতর হমায়ন কঠিন দেরাচেগ অসসথ তাই তার দেযৌন াহিরচেয় যাচেচছ এই দেরাচেগ তাচেক দেগাপচেন কচেরকচের াচেচছ তার সক (হিকত ধীচের ধীচের bয় চেচছ তাই হিপতা চেয় ার আললার কাচেছ হমায়চেনর পরান হিভbা দেচেয়চেছন২) ldquoাগাও (চেরর পরাচেনত পরানতচের ধসর (ন দেযর আান গানldquoউততর) াচেরর পতর হমায়ন কঠিন দেরাচেগ আxানত তাই ার আ দে(াচেক মমCাত (চেরর পচেথ পরানতচের আান গান ধবহিনত দোক দেসই আান গান আললার কাচেছ দেযন চে যায় আললা দেযন এই আহিতC শচেন পচেতরর ীন হিফহিরচেয় দেয় ৩)ldquoনাহিক এই (রীচেরর পাচেপর ীানচেত দেকানই তরারণ দেনই ভহি4চেতরldquoউততর) হমায়চেনর অসসথতার ন য ার হিনচেচেকই ায়ী কচেরচেছন কারন ার অচেনক রা য অন যায় ভাচে কচেরচেছ তাই তার এই পাপ কাচের ন য তার ঘচের আ হিপ এচেসচেছ এই অন যায় কাচের ন য তার মহিকত দেনই তাই ার আললার কাচেছ এই পাপ কাচেযCর ন য bমা পরাথM

Hindi 2ndlang

-ासी(जयशकर परसा-)

-ासी जयशकर परसा- की एक ऐसी कहानी ह जिजसम भारतीय ससकनित और राषटरीयता का सवरगजीतहोता ह इस कहानी म इरावती एक निहद कनया ह जिजस मलअचछो न मलतान की लट म पकडा और -ासी बना दि-या उस 500 दि-न -कर काशी क एक महाजन न खरी-ा दसरी -ासी निफरोजा ह वह गलाम ह निफरोजा को छडान क कतिलए अहम- को 1000 सोन क कतिसकक भजन थ जो अभी तक नही आए थ राजा साहब कठोर होत हए भी निफरोजा को निबना धनराकतिश क कतिलए उस म कर -त ह वनिफरोजा को अहम- को समझान की बात कहत हकहानी क अत म हम -खत ह निक इरा वती और जाटो क सर-ार बलराज का मिमलन होता हअहम- को यa म मार दि-या जाता ह वहा निफरोजा की परसननता की समामिध बनती ह वहा एक फल चढती ह और डीजल आती ह निफरोजा उस समामिध की आजीवन -ासी बनी रहती हलखक अपन उददशय अथात -ास परथा पर परकाश डालन और इस परथा क कारण होन वाल -ातो क दखो को दि-खान म पणता सफल हए ह

helliphellipContinue to next

Biology Reproductio Today we will discuss about vegetative Q1 Name some vegetative propagules

n in Organisms

propagation of plants The process of multiplication in which fragments of plant body function as propagule and develop into new individual is called vegetative propagation The units of such propagation are runner rhizome tuber bulb etc

and the speciesinvolvedVegetative propagules

Parts involved

Bulb StemBulbil BulbilRhizome Stem Runner Stem Tuber Stem Offset Stem Leaf buds Leaves Suckers Stem

Corns Stem stolon

Q2 State advantages of vegetative propagation

i) Rapid methodii) Sure and easy methodiii) Useful in plants that cannot

produce viable seeds or long seed dormancy

iv) Maintains purity of raceQ 3 Banana fruit is said to be parthenocarpic where as turkey is said to be parthenogenetic WhyBanana develops without fertilization from an unfertilized ovary thus is parthenocarpicIn turkey the ovum or female gamete developinto a new chick without fertilization thus isparthgenetic

Q4 Why is water hyacinth is called as a ldquoTerror of Bengalrdquo Water hyacinth can

propagatevegetatively all over the water body in a short per short period of time This resulted increased biochemicaloxygen oxygen demand of water body causing mortalityof fishes It is very difficult to get rid off them Thus known as terror of Bengal

Chemistry

Solid state GENERAL CHARACTERISTICS OF SOLID STATEIn nature the particular state of matter is governed by two opposing forces at given set of temperature and pressure These forces are intermolecular force of attraction and thermal energy If intermolecular force of attraction is high as compared to thermal energy particles remains in closest position

Intext QuestionsQ1 Classify the following solids as crystalline and amorphous Sodium chloride quartz glass quartz rubber polyvinyl chloride Teflon

A1 Crystalline

and hence very less movement in particles is observed In this case solid state is the preferred state of matter

Let us revise the general characteristics of solid

i) Fixed mass volume and shape

ii) Strong intermolecular force of attraction

iii) Least intermolecular space

iv) Fixed position of constituent particles

v) Incompressible and rigid

Q2 what type of interactions hold the molecules together in a polar molecular solid[CBSE 2010]A2 The molecules in a solid are held together by van der Waals forces The term van der Waals forces include hydrogen bonding dipole-dipole attraction and London dispersion forces All molecules experience London dispersion forces In addition polar molecules can also experience dipole-dipole interactions So the interactions that holds the molecule together in polar molecular solid are London dispersion force and dipole-dipole interactionsQ3 Write a feature that will distinguish a metallic solid from an ionic solid [CBSE 2010]A3 Metals are malleable and ductile whereas ionic solid are hard and brittle Metallic solid has typical metallic lustre But ionic solid looks dullQ4 Write a point of distinction between a metallic solid and an ionic solid other than metallic lustre [CBSE 2012]A4 Metals are malleable and ductile whereas ionic solid are hard and brittleQ5 Write a distinguish feature of metallic solid [CBSE 2010]A5 The force of attraction in

solid Sodium chloride Quartz Amorphous solid Quartz glass rubber polyvinyl chloride Teflon Q2 why glass is considered as super cooled liquidA2 Glass shows the tendency to flow at slower rate like liquid Hence they considered as super cooled liquidQ3 why the window glass of old buildings show milky appearance with timeA3 Glass is an amorphous solid Amorphous solid has the tendency to develop some crystalline character on heating Due to heating in day over the number of years glass acquires some crystalline character and show milky appearanceQ4 why the glass panes fixed to window or doors of old building become slightly thicker at bottomA4 Glass is super cooled liquid It has the tendency to flow down very slowly Due to this glass pane becomes thicker at the bottom over the timeQ5 Sodium chloride is a crystalline solid It shows the same value of refractive index along all the direction TrueFalse Give reasonA5 FalseCrystalline solid shows anisotropy in properties That is it shows different values for the given physical property in different direction All the crystalline solids show anisotropy in refractive index Therefore sodium chloride will show different values of refractive index on different directions

Q6 Crystalline solid are anisotropic in nature What does this statement means

between the constituent particles is special kind of electrostatic attraction That is the attraction of positively charged kernel with sea of delocalized electronsQ6 which group of solid is electrical conductor as well as malleable and ductile [CBSE 2013]A6 Metallic solidQ7 why graphite is good conductor of electricity although it is a network (covalent solid)A7 The exceptional property of graphite is due to its typical structure In graphite each carbon is covalently bonded with 3 atoms in same layer The fourth valence electron of each atom is free to move in between different layersThis free electron makes the graphite a good conductor of electricity

[CBSE 2011]A6 Anisotropy is defined asrdquo Difference in properties when measured along different axis or from different directionsrdquo Crystalline solid show different values of some of the physical properties like electrical resistance refractive index etcwhen measured along the different directions The anisotropy in crystalline solid arises due to the different arrangement of particles in different directions

Math Function Composition of functions Think of an industrial plant that produce bottles of cold drinks first there is the operation (or function) f that puts the cold drink inside the bottle followed by the opeartion g that close the bottle with the capThis leads to the following definitionDefinition Let f A rarr B and g B rarr C be two functions Then the composition of f and g denoted by gof is defined as the function gof A rarr C given by gof(x) = g(f (x)) forall x isinA

Definition A function f X rarr Y is defined to be invertible if there exists a function g Y rarr X such that gof = IX and fog = IY The function g is called the inverse of f and is denoted by f -1

Thus if f is invertible then f must be one-one and onto and conversely if f is one-one and onto then f must be invertible This fact significantly helps for proving a function f to be invertible by showing that f is one-one and onto specially when the actual inverse of f is not to be determined

Example 1 Let f 2 3 4 5 rarr 3 4 5 9 and g 3 4 5 9 rarr 7 11 15 be functions defined as f(2) = 3 f(3) = 4 f(4) = f(5) = 5 and g (3) = g (4) = 7 and g (5) = g (9) = 11 Find gofSolution We have gof(2) = g (f(2)) = g (3) = 7 gof(3) = g (f(3)) = g (4) = 7gof(4) = g (f(4)) = g (5) = 11 and gof(5) = g (5) = 11Example 2 Find gof and fog if f R rarr R and g R rarr R are given by f(x) = cos x and g (x) = 3x2 Show that gof ne fogSolution We have gof(x) = g(f(x))=g(cosx) = 3 (cos x)2

= 3 cos2 x Similarly fog(x)=f(g (x))= f(3x2)= cos (3x2) Note that 3cos2 x ne cos 3x2 for x = 0 Hence gof ne fogExample 3 Show that if f A rarr B and g B rarr C are onto then gof A rarr C is also ontoSolution Given an arbitrary element z isin C there exists a pre-image y of z under g such that g (y) = z since g is onto Further for y isin B there exists an element x in A with f(x) = y since f is onto Therefore gof(x) = g (f(x)) = g (y) = z showing that gof is onto Example 4 Let Y = n2 n isin N sub N Consider f N rarr Y as f(n) = n2 Show that

f is invertible Find the inverse of fSolution An arbitrary element y in Y is of the form n2 for some n isin N This implies that n =radicy This gives a function g Y rarr N defined by g (y) =radicy Nowgof (n) = g (n2)=radicn2 = n and fog (y) =f(radicy) = (radicy) 2 y which shows that gof=IN and fog= IY Hence f is invertible with f -1 = g

Political Science

Constitution of India-The Preamble

Summary

Objective of the state-To secure equality of status and of opportunity To promote fraternity among all the citizens To assure the dignity of the individuals and Unity and integrity of the nation

Justice-Justice stands for rule of law absence of arbitrariness and a system of equal rights freedom and opportunities for all in a society India seeks social economic and political justice to ensure equality to its citizens

Liberty-Liberty implies the absence of restraints or domination on the activities of an individual such as freedom from slavery serfdom imprisonment despotism etc The Preamble provides for the liberty of thought expression belief faith and worship

Equality-Equality means the absence of privileges or discrimination against any section of the society The Preamble provides for equality of status and opportunity to all the people of the country

Fraternity-The Preamble declares that fraternity has to assure two thingsmdashthe dignity of the individual and the unity and

Execution

Answer the following questions-

Short notes-1 Equality2 Fraternity3 Justice4 Liberty

Homework-Learn

integrity of the nation The word integrity has been added to the Preamble by the 42nd Constitutional Amendment (1976)

Business studies

Human resource management (chapter 1)

On the day of 1504 2020 I have discussed with you the managerial functions and procurement functions of HRM

Today weare going to discuss about the development function integration functions and maintenance function

Development functions-HRM improves the knowledge skills attitude and values of employees so that they the present and future jobs more effectively it includes

1) Development functions of HRM

a) Performance appraisal = It implies systematic evaluation of employees with respect to their performance on the job and their potential for development

b) Training =It is the process by which employees learn knowledge skills and attitudes to achieve organisational and personal goals

c) Executive development = It is the process of developing managerial talent through appropriate program

2) Integration functionsa) HRM reconcile the goals of

organisation with those of its members through integrating function

b) HRM tries to motivate employees to various financial and non financial incentives provided in job specification etc

3) Maintenance functiona) HRM promote and protect the

physical and mental health of employees by providing several types of benefits like housing medical aid etc

b) It Promote Social security measures to employees by providing provident fund pension gratuity maternity benefits

SubjectCOMMERCE

Topic

BUSINESSENVIRONMENT

Summary

Now quickly let us revise the earlier points that we have already done in the last class and let us proceed with the other topics that are there in the chapter

Firstly we will recall the internal and external factors of micro environment and then we

Execution 3 What do you mean by internal factors

in micro environmentAnswerInternal factors refer to all the factors existing within a business firm The internal factors are considered controllable because the enterprise has control over these factors

Development FunctionsPerformance AppraisalTrainingExecution Development

shall proceed in details

Meaning and list of internal and external factors

aInternal factorsInternal factors refer to all the factors existing within a business firm The internal factors are considered controllable because the enterprise has control over these factorsFor an example a company can alter its organization structure policies programmes employees physical facilities and marketing mix to suit the changes in the environmentList of internal factors areCorporate culture mission and objectives top management organizations structure company image and brand equity company resources

b External factorsExternal factors refer to those individual and groups and agencies with which a particular business organization comes into direct and frequent contact in the course of its functioningThese individuals and groups are known as STAKEHOLDERS because they have a stake (financial interest ) in the working and performance of the particular business List of external forces (stakeholders)Customers competitors investors suppliersmiddlemen (marketing intermediaries)financers publics

customers

suppliersfinancers

For an example a company can alter its organization structure policies programmes employees physical facilities and marketing mix to suit the changes in the environment

4 What do you mean by external factors in micro environment

AnswerExternal factors refer to those individual and groups and agencies with which a particular business organization comes into direct and frequent contact in the course of its functioningThese individuals and groups are known as STAKEHOLDERS because they have a stake (financial interest) in the working and performance of the particular business

3Who are stakeholdersSTAKEHOLDERS are individuals and groups who have a stake (financial interest ) in the working and performance of the particular business 4Discuss the internal factors in briefa Corporate CultureThe values beliefs and attitudes of the founders and top management of the company exercise a strong influence on what the cmpaany stands for how it does things and what it considers importantbMission and objectivesThe business philosophy and purpose of a comoany guide it prioritiesbusiness strategiesproduct market scope and development scope

cTop management structurethe composition of board of directors the degree of professionalization of management and the organizational structure of a company have important bearing on its business decisions

dPower structureThe internal power relationship between the board of directors and the chief executive is an important factor

e Company image and brand equityThe image and brand equity of the company play a significant role in raising finance forming alliance choosing dealers and suppliers launching new products entering foreign markets

5 What is Macro environmentAnswerMacro environment refers to the general

competitors

middlemen

publics

Fig STAKEHOLDERS OF A COMPANY

Apart from micro environment the other main dimension of business environment isMacro environment Macro environment refers to the general environment or remote environment within which a business firm and forces in its micro environment operateA company does not directly or regularly interact with the micro environmentTherefore macro environment is also known as indirect action EnvironmentThe macro environment forces are less controllable than the micro forces

Macro environment consists of the following components

POLITICAL AND LEGAL ENVIRONMENT

ECONOMIC SOCIAL AND ENVIRONMENT

CULTURAL

ENVIRONMENT

TECHNOLOGICAL ENVIRONMENT

Fig COMPONENTS OF MACRO ENVIRONMENT

environment or remote environment within which a business firm and forces in its micro environment operateA company does not directly or regularly interact with the micro environmentTherefore macro environment is also known as indirect action EnvironmentThe macro environment forces are less controllable than the micro forces 6 What are the components of macro environmenta Political and legal environmentb Economic environmentc Social and cultural environmentd Technological environment

Computer Science

Logic gates

Digital systems are said to be constructed by using logic gates These gates are the AND OR NOT NAND NOR EXOR and EXNOR

BUSINESS FIRM

gates The basic operations are described below with the aid of truth tables

AND gate

The AND gate is an electronic circuit that gives a high output (1) only if all its inputs are high A dot () is used to show the AND operation ie AB Bear in mind that this dot is sometimes omitted ie ABOR gate

The OR gate is an electronic circuit that gives a high output (1) if one or more of its inputs are high A plus (+) is used to show the OR operationNOT gate

The NOT gate is an electronic circuit that produces an inverted version of the input at its output It is also known as an inverter If the input variable is A the inverted output is known as NOT A This is also shown as A or A with a bar over the top as shown at the outputs The diagrams below show two ways that the NAND logic gate can be configured to produce a NOT gate It can also be done using NOR logic gates in the same way

NAND gate

This is a NOT-AND gate which is equal to an AND gate followed by a NOT gate The outputs of all NAND gates are high if any of the inputs are low The symbol is an AND gate with a small circle on the output The small circle represents inversion

NOR gate

This is a NOT-OR gate which is equal to an OR gate followed by a NOT gate The outputs of all NOR gates are low if any of the inputs are highThe symbol is an OR gate with a small circle on the output The small circle represents inversion

EXOR gate

The Exclusive-OR gate is a circuit which will give a high output if either but not both of its two inputs are high An encircled plus sign ( ) is used to show the EOR operation

EXNOR gate

The Exclusive-NOR gate circuit does the opposite to the EOR gate It will give a low output if either but not both of its two inputs are high The symbol is an EXOR gate with a small circle on the output The small circle represents inversion The NAND and NOR gates are called universal functions since with either one the AND and OR functions and NOT can be generated

Note A function in sum of products form can be implemented using NAND gates by replacing all AND and OR gates by NAND gates A function in product of sums form can be implemented using NOR gates by replacing all AND and OR gates by NOR gates

Logic gate symbols

Table 2 is a summary truth table of the inputoutput combinations for the NOT gate together with all possible inputoutput combinations for the other gate functions Also note that a truth table with n inputs has 2n rows You can compare the outputs of different gates

Logic gates representation using the Truth table

Example

A NAND gate can be used as a NOT gate using either of the following wiring configurations

Subject Eng Literature (The Tempest ndash William Shakespeare) Topic Act III Scene 3 Lines 53 to 110 (End of the scene) Date 16th April 2020 (2nd Period)

[Students should read the original play and also the paraphrase given in the school prescribed textbook]Summary Questions amp Answers

o Seeing this strange scene all are inclined to believe the tales told by travelers that there truly are ldquounicornsrdquo and ldquothe phoenixrsquo thronerdquo

o As they are about to sit down to the feast the banquet is snatched away by a harpy (Ariel disguised) A spiritrsquos voice (Arielrsquos voice) denounces Alonso Sebastian and Antonio with particular

1 ARIEL You are three men of sin whom Destiny

(Line 53-58)That hath to instrument this

lower world And what is int the never-surfeited sea

Hath caused to belch up you and on this island

Where man doth not inhabit you rsquomongst men

Being most unfit to live I have made you mad

reference to their crime in expelling Prospero from Milan They have not received any punishment for their deed earlier but the time for their punishment has arrived Upon Alonso it pronounces ldquolingering perdition worse than deathrdquo from which there is no remedy except through sincere repentance Ariel then vanishes in thunder and the shapes enter again and carry away the table

o Prospero watching invisibly is very pleased with the performance of Ariel and his (Prosperorsquos) ldquomeaner ministersrdquo All his enemies are now in his power and are in a fit of desperation He then leaves them and goes to see how Ferdinand and Miranda are getting on

o Alonso is now much humbled and penitent with the after effect of the spiritrsquos denunciation of his crimes He believes that his son is lost forever After this all disperse being stricken mad by the speech of the spirit

o Gonzalo fearing that they may do violence to themselves or to one another follows them and bid others to follow

(a) To whom does Ariel disguised as a harpy call the three sinners What game did Fate of Destiny play with

them

The three sinners called by Ariel are Alonso Sebastian and Antonio It was Destiny which had caused the ocean to cast the three sinners on the shore Though the ocean is all the time devouring whatever appears on its surface and is never satisfied with its continual swallowing of the ships and men in the present case the ocean had cast these three sinners on the shore without killing them

(b) Who had jointly been responsible for the conspiracy against Prospero What is Prosperorsquos purpose behind all this

Three men Alonso Sebastian and Antonio had jointly

been responsible for the conspiracy against Prospero They had driven out Prospero form Milan Prosperorsquos purpose is to make these three sinners realize the wrong they had done He wants them to repent for their criminal deeds because repentance leads to self-esteem(c )What does Ariel (the harpy) tell Alonso and his companions when they take out their swords to attack him

Seeing them drawing their swords Ariel (harpy) tells them that he and his companions are the instruments of destiny and that it is not possible for human beings to do them any injury He says that the swords of human beings can not injure even a minute part of his feathers Their swords are as ineffective against him and his companions as against the wind or the water

(d) Give the explanatory meanings of the following expressions in the context of the above extract

(i)Never surfeited (ii) Belch up (iii) lsquomongst men

(i) Never surfeited never led to satisfaction

(ii) Belch up cast ashore(iii) lsquomongst men in human

society2

I and my fellows (Line 60-65)

Are ministers of Fate The elementsOf whom your swords are tempered may as wellWound the loud winds or with bemocked-at stabsKill the still-closing waters as diminishOne dowl thats in my plume

IMPORTANT PASSAGES EXPLAINED

The elements

(Line 61-66)Of whom your swords are tempered may

as wellWound the loud winds or with

bemocked-at stabs

(a) Who is lsquoIrsquo Who are his lsquofellowsrdquo

lsquoIrsquo is referred to Ariel in disguise of a harpy His lsquofellowsrsquo are other spirits serving Prospero the real Duke of Milan who has acquired supernatural powers after being banished from his Dukedom Prospero has settled in this uninhabited island

(b) What are the elements that have temperrsquod the swords Why will it not work against the speaker

The swords (of Alonso and his companions) are tempered by metal (steel) which is taken out of the earth and refined by

Kill the still-closing waters as diminishOne dowl thats in my plume My fellow

ministersAre like invulnerable

In these words Ariel reminds the King and his companions of the utter futility of drawing swords against himself and his fellows Ariel drives Alonso Antonio and Sebastian the three men of sin to desperation ndash a state in which men do violence to themselves They draw swords to strike Ariel But Ariel reminds them that he and the other spirits are the ministers of destiny and nothing can wound them The steel of which their swords are made of may cut the wind or water which being divided always closes up again Even supposing that such things may be possible it is quite impossible that their swords will cut one feather in their plume They are incapable of being wounded by any sword of man Hence it is foolish on their part to attempt to strike at Ariel and his fellow-spirits

For which foul deed

(Line 72-75)The powers delaying not forgetting

haveIncensed the seas and shores yea all the

creatures Against your peace

Ariel enters like a harpy and remaining invisible tells Alonso Sebastian and Antonio that he and other harpies are the agents of Destiny appointed to carry out her decrees He tells them that their punishment for the crime against Prospero which has been so long deferred is now to fall upon them He reminds them that they had expelled Prospero from Milan and set him and his innocent child adrift on the sea and that the sea had paid them back for their sin by the shipwreck and by the calamities they have suffered He tells them that the powers above which did not forget this mean treachery but only deferred the punishment have now engaged the seas and the shores and all living beings including him and his comrades against them The very elements and supernatural agency Ariel adds have taken up the avenging of their crime against Prospero

the action of fire It may cut the wind or water which being divided always closes up again

The sword will not work against the spirits and the harpy because they are the ministers of destiny and nothing can wound them nor it will cut a single feather in their plume

(c )What is the meaning of lsquodowlrsquo in the last line

The term lsquodowlrsquo means a filament or the smallest part of a feather In this context Ariel in disguise of harpy says that their sword cannot even damage the smallest filament of their (Arielrsquos and other spirits) feathers as they are incapable of being wounded by any sword of man

(d) What does the speaker remind the listeners about

Ariel in disguise of harpy reminds Alonso the King of Naples Sebastian Alonsorsquos brother and Antonio the present Duke of Milan and the treacherous brother of Prospero as they being three men of sin He even reminds them that their punishment for their crime against Prospero which has been so long deferred now falls upon them He reminds them that they have expelled Prospero from Milan and has set him along with his innocent infant daughter adrift on the sea So the sea has paid them back for their sin by their shipwreck and the calamities they have suffered since then The harpy rebukes Alonso of his sin that has incensed the Gods and has deprived him of his son as a punishment

(e) How do they respond

When Ariel in disguise of a harpy reminds Alonso Sebastian and Antonio of their past misdeeds and sin Alonso has a look of terror and confusion in his eyes He utters the words of sincere repentance wrung out of his conscience-stricken heart It appears to him that all the elements of nature the sea-waves the wind and the thunder proclaiming a loud voice in the name of Prospero and the crime Alonso has committed against him They are calling upon him to repent There is a deep storm raging in Alonsorsquos breast and the echoes of that storm are ringing in his ears like a clear note of wind-instrument A note of denunciation of Alonsorsquos crime leaves him much humbled and penitent and confirms his belief that his son is lost forever But Sebastian and Antonio shows some courage instead of repentance They wish to kill the spirits or devils if it appears

3

Of my instruction hast thou nothing bated (Line 85-93)

In what thou hast to say So with good life

And observation strange my meaner ministers

Their several kinds have done My high charms work

And these mine enemies are all knit upIn their distractions They now are in my

powerAnd in these fits I leave them while I visitYoung Ferdinand whom they suppose is

drownedAnd his and mine loved darling

Methought the billows spoke and (Line 96-99)

told me of itThe winds did sing it to me and the

thunderThat deep and dreadful organ-pipe

pronouncedThe name of Prosper It did bass my

trespass

These are the words of contrition coming from Alonso Ariel has driven him to a deep repentance for conspiring with Antonio against Prospero He now feels a sincere remorse It appears to him that all the elements of nature the sea-waves the wind and the thunder proclaimed with a loud voice the name of Prospero and the crime Alonso had committed against him They are calling upon him to repent There is a deep storm raging in Alonsorsquos breast and the echoes of that storm are ringing in his ears like the clear note of a wind-instrument

Comment These are the words of sincere repentance wrung out of the conscience-stricken heart of Alonso Alonso who is the lesser villain is the first to give way to remorse under the effect of Arielrsquos speech The words of Ariel seem to him to be the voice of conscience speaking to him He is driven to desperation a state in which he might do violence to his life

(a) Identify the speaker State the context

Prospero the ruler of the island is the speaker The famous banquet scene has been enacted very well Ariel and his junior spirits have played their roles excellently Prospero is glad to say words of praise for them(b) In what way the speakerrsquos instructions have been carried out

According to Prosperorsquos instructions a banquet was presented before the King of Naples and his companions when they were tired and hungry Just when they were preparing to eat the feast the banquet was suddenly removed by exercising supernatural powers All this was done by Ariel Prosperorsquos chief assistant and a powerful spirit

Ariel not only made the feast disappear but also delivered his speech blaming the King and his two companions for their past wicked deeds He warned them to repent for their misdeeds or suffer forever on that uninhabited island

(c) Who are referred to as lsquomeaner ministersrsquo What have they done

Prospero refers as lsquomeaner ministersrsquo to his other lesser spirits who were assisting Ariel in presenting a scene before the kingrsquos party They entered the scene to the accompaniment of music They assumed several strange shapes and brought in a banquet Then they danced about it with gentle actions of salutations thus inviting the King and others to eat the feast

These spirits play their role again when Ariel in the shape of a harpy quits the scene These shapes enter again and dancing with mocking gestures carry away the table

(d) Who are the speakerrsquos enemies What has happened to them

King of Naples Alonso his brother Sebastian and the present Duke of Milan Antonio (Prosperorsquos own brother) are Prosperorsquos enemies With the turn of events they have all been washed ashore on the island which is ruled by Prospero the great magician Actually this happened after the shipwreck caused by a storm which was raised by Prospero with the purpose of bringing these people to his island Prosperorsquos spirits have already confused and terrified these enemies and they are under Prosperorsquos control He can treat them as he likes

(e) What does he say about Ferdinand Explain what is meant by ldquohellip his and mine darlingrdquo

Prospero knows that Alonsorsquos son prince Ferdinand is alive though his father thinks that the prince has been drowned

Prospero refers to his daughter Miranda who is dear to him She is also very dear to Prince Ferdinand who has fallen in love with her They are waiting to be married soon for which they have received Prosperorsquos consent

4

ALONSO O it is monstrous monstrous (Line 95-102)

Methought the billows spoke and told me of it

The winds did sing it to me and the thunderThat deep and dreadful organ-

pipe pronouncedThe name of Prosper It did bass

my trespassTherefore my son ithrsquo ooze is

bedded andIll seek him deeper than eer

plummet soundedAnd with him there lie mudded

(a) In what way does Alonso express his horror when his conscience is awakened by Arielrsquos words

When Alonsorsquos conscience is awakened by Arielrsquos words he expresses his horror at what he has heard He gets the feeling that the waves of the ocean the wind and the loud thunder have spoken to him and uttered the name of Prospero Because of being reminded of his crime in a very loud and rough voice he comes to realize that he has lost his son for his past misdeeds

(b) What does Alonso imagine about his son What does Alonso want to do in his desperate state

Alonso imagines that his son is lying in the mud at the bottom of the sea He feels desperate that he wants to drown himself in the ocean deeper than the plumb-line has ever gone He wants to lie with his son at the bottom of the sea

(c) How do Sebastian and Antonio want to face the evil spirits

Sebastian says that he is not at all afraid of what the harpy has said and that he is prepared to fight any number of such monsters if they appear before him only one at a time Antonio says that he would support Sebastian in the fight against the fiendsyyy

(d) Why does Gonzalo ask Adrian to follow the three men

Gonzalo tells Adrian that all the three men namely Alonso Sebastian and Antonio are in a wild and reckless mood The thought of the heinous crime of which they are guilty has begun to torment their minds So he asks Adrian to follow those three men without loss of time and prevent them from doing anything which the turmoil in their minds might lead them to do

(e) What opinion do you form of Alonso from the above extract

Alonso who is the lesser villain is the first to give way to remorse under the effect of Arielrsquos speech The words of Ariel seem to him to be the voice of conscience speaking to him He is driven to desperation a state in which he might do violence to his life

Subject =Accounts

Ac-12 15420 topic-pL Appropriation ac

PROFIT AND LOSS APPROPRIATION ACCOUNT

MEANING AND PREPARATIONProfit and Loss Appropriation Account is merely an extension of the Profit and Loss Account of the firm The profit of the firm has to be distributed amongst the partners in their respective profit sharing ratio But before its distribution it needs to be adjusted All Adjustments like partnerrsquos salary partnerrsquos commission interest on capital interest on drawings etc are made in this account These adjustments will reduce the amount of profit for distribution This adjusted profit will be distributed amongst the partners in their profit sharing ratio To prepare it at first the balance of Profit and Loss Account is transferred to this account The journal entries for the preparation of Profit and Loss Appropriation Account are given below

1 for transfer of the balance of Profit and Loss Account to Profit and Loss Appropriation Account

(a) In case of Net Profit

Profit and Loss Ac helliphelliphelliphelliphellipDrTo Profit and Loss Appropriation Ac(Net Profit transferred to Profit and Loss Appropriation Ac)

(b)In case of Net Loss

Profit and Loss Appropriation Achelliphelliphellip DrTo Profit and Loss Ac(Net Loss transferred to Profit and Loss Appropriation Ac)

2 for Interest on Capital

For transferring on Interest on CapitalProfit and Loss Appropriation Achelliphelliphellip DrTo Interest on Capital Ac(Interest on capital transferred to Profit amp Loss Appropriation Ac)

3 for Interest on Drawings

For transferring Interest on Drawings Interest on Drawings Achelliphelliphelliphelliphelliphellip DrTo Profit and Loss Appropriation Ac(Interest on drawing transferred to Profit amp Loss Appropriation Ac)

4 For Partnerrsquos SalaryFor transfer of partnerrsquos SalaryProfit and Loss Appropriation Achelliphellip DrTo Salary Ac(Salary transferred to profit amp Loss Appropriation Ac)

5 For Partnerrsquos CommissionFor transferring commissionProfit and Loss Appropriation Achelliphelliphellip DrTo Commission Ac(Commission transferred to Profit and Loss Appropriation Ac)

6 For Transfer of agreed amount to General ReserveProfit and Loss Appropriation Ac helliphellipDrTo General Reserve Ac(Transfer to General Reserve)

7 for share of Profit or Loss appropriation(a) If ProfitProfit and Loss Appropriation Achelliphellip DrTo Partnerrsquos CapitalCurrent Ac(Profit transferred to capitalcurrent Ac)(b) If LossPartnerrsquos Capital Current Achelliphelliphelliphellip DrTo Profit and Loss Appropriation Ac(Loss transferred to capitalcurrent Ac)

THE FORMAT OF PROFIT AND LOSS APPROPRIATION

Profit and Loss Appropriation Account for the year endedhelliphelliphelliphellip

Particulars Amount Particulars Amount

To PL Ac (loss) By pL Ac (profit)

To Interest on capital BY Interest on drawings

To partner`s commission by Partner`s capital Ac ( loss)

To Partner`s salary To Interest on partner`s loan To General Reserve To Partner`s Capital AC (Profit)

Subject= Economics

MOVEMENT ALONG THE DEMAND CURVE (CHANGE IN QUANTITY DEMANDED)In law of demand you have already studied the inverse relationship between price and quantity demanded When quantity demanded of a commodity changes due to change in its price keeping other factors constant it is called change in quantity demanded It is graphically expressed as a movement along the same demand curve There can be either a downward movement or an upward movement along the same demand curve Upward movement along the same demand curve is called contraction of demand or decrease in quantity demanded and downward movement along the same demand curve is known as expansion of demand or increase in quantity demanded

Extention of demandd

price (rs)p A

B Extentionp1 d

Q Q1

Quantity demanded ( in units)

Contraction of demandd

p2 Ccontraction

p APrice (Rs)

d

Q2 Q

Quantity demanded (in units)

Explanation of movement of demand A fall in price from OP to OP1 leads to increase in quantity demanded from OQ to OQ1 (expansion of demand) resulting in a downward movement from point A to point B along the same demand curve DD When Price rises from OP to OP2 quantity demanded falls from OQ to OQ2 (contraction of demand) leading to an upward movement from point A to point C along the same demand curve DD

  • Activity Series of Metals
    • Drawbacks of Rutherfordrsquos model of atom
      • Electromagnetic radiations
      • Properties of electromagnetic radiations
      • Characteristics of electromagnetic radiations
        • Plancks Quantum Theory-
        • Photoelectric effect
          • Intext Questions
            • Logic gates
            • Digital systems are said to be constructed by using logic gates These gates are the AND OR NOT NAND NOR EXOR and EXNOR gates The basic operations are described below with the aid of truth tables
            • AND gate
            • Example
Page 33:  · Web viewSubject. Topic. Summary. Execution. English 1 . Chapter 1 naming words . Page 8. Write the names of these pictures:- Person:-1. father. 2.Firefighter 3.doctor 4 ...

3Employment-Employment mean an economic activity where people work for others in exchange for some remuneration (salary)The persons who work for others are called lsquoemployeesrsquo The persons or organizations which engage others to work for them are called lsquoemployersrsquoEg A doctor working in a hospital is employment as he is working for a salaryA lawyer may serve as a law officer in a bank

With this we shall proceed with the features of both Profession amp Employment

The main features of a profession are as follow

a Specialised body of knowledge b Restricted entry c Formal education and training d Professional association e Service motive f Code of contact

The main features of an employment are as follows

a In employment a person works for others called employer

b An employee provides personal service

c There is a service agreement or contract between the employee and the employer

d The employee has to obey the order of the employer

e No capital investment is made by the employer

Various examples of Employment are as follows

aA teacher teaching in a school or collegeb An engineer employed in Municipal Corporation of DelhicAn accountant working in the accounts department of a companydA doctor working in a hospital

Note In all the above examples of employment the individual who is involved in each example is working as an employee for a salary under an employer

qualification(MBBSCALLB)d Service motive ndashProfessionals are expected to emphasis service more on their clients rather than economic gain f Code of Conduct-The activities of professionals are regulated by a code of conduct

2 What are the main features of EmploymentAnswer The main features of an employment are as followsa In employment a person works for others called employerb An employee provides personal servicec There is a service agreement or contract between the employee and the employerd The employee has to obey the order of the employere No capital investment is made by the employer

3 Give various Professions and their respective Association are given below

Professions

Professional

Professional association

Medical profession

Doctor Medical Council of India

Law profession

Lawyers Bar Council of India

Accounting Profession

Chartered

The Institute of Chartered Accounts of India( ICAI)

Engineerin Engineers The

g Profession

institute of Engineers (India)

Accounts Basic accounting terms

Today we will give you some questions from the previous study material

Questions6) Define accounting7) What do you mean by debit

and credit8) Explain the types of account9) Define the following terms

a) Assetsb) Capitalc) Purchased) Debtorse) Transactions

10) Name the types of accounts given below

a) Krishnas accountb) Machinery accountc) Royalty accountd) Salary accounte) Furniture accountf) Audit fee account

Economics Basic Economic ConceptsSub topic

UTILITY

Before starting todayrsquos class we shall recall the last class which was about UTILITY AND THE FEATURES OF UTILITY

Now we shall proceed with the further topics of the chapter

Todayrsquos topic from the chapter lsquo Basic Economic Conceptsrsquo will be TOTAL UTILITY amp MARGINAL UTILITYNow let us quickly revise the concept of utility with an example ie goods and services are designed because they have an ability to satisfy human wantsThis feature of being able to satisfy human wants is termed as utility For example we derive utility from WiFi services as it gives us satisfaction by connecting us to our friends and family through social media here consumers derive utility from WiFi services

From the above concept we shall start with todayrsquos topicEconomists have defined TOTAL UTILITY (TU) as the total satisfaction obtained by consuming a given total amount of a good and serviceFor example the total satisfaction obtained from eating 10 mangoes is the total utility of 10 mangoes

MARGINAL UTILITY (MU) is the additional satisfaction derived from each additional unit

Questions1 What is Total Utility (TU)

Answer Total Utility (TU) is the

aggregate of the utility that a consumer derives from the consumption of a certain amount of a commodityTU=MU1+MU2++MUn

2 What is Marginal UtilityAnswer

Marginal Utility (MU) is the additional made to the total utility as consumption is increased by one more unit of the commodityMU= TUn ndashTUn-1

NoteOften economists tend to

subdivide utility into an imaginary unit called UTIL

consumed In this casethe utility obtained from each mango as it is consumed as the MU of that mango It is also defined as the addition made to the total utility when an additional unit is consumed Often economists tend to subdivide utility into an imaginary unit called UTIL

Note As a consumer increases the consumption of a good over period of time the total utility or total satisfaction derived from it increases to appoint and thereafter it decreasesHowever as the consumer keeps on consuming the good the marginal utility or the additional utility derived from it decreases

SubjectBusiness studies

Topic

BUSINESSENVIRONMENT

Summary

Now quickly let us revise the earlier points that we have already done in the last class and let us proceed with the other topics that are there in the chapter

Firstly we will recall the internal and external factors of micro environment and then we shall proceed in details

Meaning and list of internal and external factors

aInternal factorsInternal factors refer to all the factors existing within a business firm The internal factors are considered controllable because the enterprise has control over these factorsFor an example a company can alter its organization structure policies programmes employees physical facilities and marketing mix to suit the changes in the environmentList of internal factors areCorporate culture mission and objectives top management organizations structure company image and brand equity company resources

b External factorsExternal factors refer to those individual and groups and agencies with which a particular business organization comes into direct and frequent contact in the course of its functioningThese individuals and groups are known as STAKEHOLDERS because they have a stake (financial interest ) in the working and performance of the particular business List of external forces (stakeholders)Customers competitors investors suppliersmiddlemen (marketing intermediaries)

Execution 1 What do you mean by internal

factors in micro environmentAnswerInternal factors refer to all the factors existing within a business firm The internal factors are considered controllable because the enterprise has control over these factorsFor an example a company can alter its organization structure policies programmes employees physical facilities and marketing mix to suit the changes in the environment

2 What do you mean by external factors in micro environment

AnswerExternal factors refer to those individual and groups and agencies with which a particular business organization comes into direct and frequent contact in the course of its functioningThese individuals and groups are known as STAKEHOLDERS because they have a stake (financial interest) in the working and performance of the particular business

3Who are stakeholdersSTAKEHOLDERS are individuals and groups who have a stake (financial interest ) in the working and performance of the particular business 4Discuss the internal factors in briefa Corporate CultureThe values beliefs and attitudes of the founders and top management of the company exercise

financers publics

customers

suppliersfinancers

competitors

middlemen

publics

Fig STAKEHOLDERS OF A COMPANY

Apart from micro environment the other main dimension of business environment isMacro environment Macro environment refers to the general environment or remote environment within which a business firm and forces in its micro environment operateA company does not directly or regularly interact with the micro environmentTherefore macro environment is also known as indirect action EnvironmentThe macro environment forces are less controllable than the micro forces

Macro environment consists of the following components

POLITICAL AND LEGAL ENVIRONMENT

ECONOMIC SOCIAL AND ENVIRONMENT

CULTURAL

ENVIRONMENT

TECHNOLOGICAL ENVIRONMENT

a strong influence on what the cmpaany stands for how it does things and what it considers importantbMission and objectivesThe business philosophy and purpose of a comoany guide it prioritiesbusiness strategiesproduct market scope and development scope

cTop management structurethe composition of board of directors the degree of professionalization of management and the organizational structure of a company have important bearing on its business decisions

dPower structureThe internal power relationship between the board of directors and the chief executive is an important factor

eCompany image and brand equityThe image and brand equity of the company play a significant role in raising finance forming alliance choosing dealers and suppliers launching new products entering foreign markets

5 What is Macro environmentAnswerMacro environment refers to the general environment or remote environment within which a business firm and forces in its micro environment operateA company does not directly or regularly interact with the micro environmentTherefore macro environment is also known as indirect action EnvironmentThe macro environment forces are less controllable than the micro forces 6 What are the components of macro environmenta Political and legal environmentb Economic environmentc Social and cultural environmentd Technological environment

BUSINESS FIRM

Fig COMPONENTS OF MACRO ENVIRONMENTPolitical science

Introduction to political science

Comparative politics and itrsquos scope Comparative politics is the second major dimension of political scienceIt is also a very vast area of study and a very large number of political scientists even treat it as an autonomous area of study within the board ambit of political scienceScope of comparative politics-

1 All political structures -Comparative politics includes the study of all structures formalnon formal governmental and extra governmental which are directly or indirectly involved in politics in all the countries of the world

2 Functional studies- Comparative politics seeks to study politics less from the point of view of the legal institutions in terms of their powers and move from the point of view of their functions which constitute the political process and their actual Operation in the environment

3 Study of political behaviour- Another important part of its scope is the study of the actual behaviour of the people in the process of politics

4 Study of similarities and differences- comparative politics also undertakesan analysis of the similarities and differences among political process and functions

5 Study of all political systems -comparative politics seeks to analyse the actual behaviour and performance of all political systems western as well as non western

6 Study of the environment and infrastructure of politics-The study of politics demands a study of the psychological sociological economic and anthropological environment in fact the social environment as a whole in which each political system operates

7 Study of political culture- political culture is composed of attitudesbeliefs emotions and values of a society that relate to the political system or politics

8 Study of political participation- Political participation is a universal processThe only difference is that while in some states it is limited in others it is wider

9 Study of political process- political

Answer the following questions-

What is comparative politics

What are the scope of comparative politics

Homework- learn

processes like decision makingpolicy making judicial process leadership recruitment process and others are always at work in all political systems

The scope of comparative politics is very comprehensive It includes everything that falls within the area of political activity and political process

History CAMBRIDGE VIEW ABOUT

THE PARTITION

AND REFUTATION

OF CAMBRIDGE

VIEW

Cambridge view about the Partition The Cambridge school of historians have interpreted that opposition to partition scheme was made entirely by the elitist groups They hold the view that Lord Curzon planned to partition the Bengal for administrative purposeREFUTATION OFCAMBRIDGE VIEW The Rationalist historians have rejected the interpretations of the Cambridge School of historians on various grounds

1 QUESTION State different views of historians regarding Partition of Bengal

ANSWER Cambridge historians believed that Lord Curzon partitioned Bengal for administrative reasons only and not for the political motive The Middle class elitist group protested because of their petty interest The Hindu zamindars protested as they have to spend more money for managing their estatesThe lawyers of Calcutta High court feared to lose their clientBut according to the nationalist Historians was-

2- The ultimate object of Lord Curzon was to crush the unity of Bengal politicians

3- If Bengal becomes a separate province Bengali speaking 16 million people of western part would become minority under Hindi speaking people of Bihar and Oriya speaking people of Orissa

4- The bureaucrats expected that the protest movement would die down quickly

5- Lord Curzon used the Muslim community in his political game

6- Idealism had great contribution in the protest against partition

7- The people of the every section of society were affected by the partition of Bengal

Computer Science

Numbers Convertion of dcimal number to octal numberThe decimal numeral system is the standard system for denoting integer and non-integer numbers It is the extension to non-integer numbers of the Hindu-Arabic numeral system For writing numbers the decimal system uses ten decimal digits a decimal mark and for negative numbers a minus sign - The decimal digits are 0 1 2 3 4 5 6 7 8 9 the decimal separator is the dot in many countries

The octal numeral system or oct for short is the base-8 number system and uses the digits 0 to 7 Octal is sometimes used in computing instead of hexadecimal perhaps most often in modern times in conjunction with file

permissions under Unix systems It has the advantage of not requiring any extra symbols as digits It is also used for digital displays

Follow these steps to convert a decimal number into octal form

1 Divide the decimal number by 82 Get the integer quotient for the next iteration (if the number will not divide equally by 8 then round down the

result to the nearest whole number)3 Keep a note of the remainder it should be between 0 and 74 Repeat the steps until the quotient is equal to 05 Write out all the remainders from bottom to top This is the solution

For example if the given decimal number is 8453

Division Quotient Remainder

8453 8 1056 5

1056 8 132 0

132 8 16 4

16 8 2 0

2 8 0 2

Then the octal solution is 20405

Subject Eng Literature (The Tempest ndash William Shakespeare) Topic Act I Scene 1 Lines 33 to 67 (End of scene) Date 16th April 2020 (4th Period)

[Students should read the original play and also the paraphrase given in the school prescribed textbook]Summary Questions amp Answers

[SUMMARY OF THE ENTIRE SCENE]

o The play starts with the scene of a severe storm at sea Alonso (King of Naples) Sebastian (Alonsorsquos brother) Ferdinand (Alonsorsquos son) Gonzalo Antonio (the usurping Duke of Milan) are in a ship in the midst of the storm

o The mariners are trying their best to control the vessel from running aground and are totally following the orders of their Master the Boatswain They have scant success

o The mariners become extremely unhappy and annoyed when most of the passengers arrive on the deck thereby hampering their effort to save the ship There is serious confrontation between them and the passengers who are part of the Kingrsquos entourage

o The mariners could not save the ship

SUMMING-UP

(i) Vivid description of the scene which gives a realistic description of terror and confusion of a tropical storm

(ii) Shows Shakespearersquos accuracy of knowledge in describing the naval operations and also matters of seamanship

(1) GONZALO Ill warrant him for drowning (L 45-57)

though the ship were no stronger than a nutshell and as leaky as an unstanched

wenchBOATSWAIN Lay her a-hold a-hold Set her two courses Off to

sea again lay her offMARINERS All lost To prayers to prayers All lostBOATSWAIN What must our mouths be coldGONZALO The king and prince at prayers Lets assist them

For our case is theirsSEBASTIAN Im out of patienceANTONIO We are merely cheated of our lives by drunkards

This wide-chopped rascal - would thou mightst lie drowning the washing of ten tides

(a) What does Antonio say at the insolent manners of the boatswain just before the given passage

Being irritated at the insolent manners of the boatswain just before the given extract Antonio the Duke of Milan calls him a worthless dog son of a woman without any morals an arrogant and disrespectful noisemaker He says that the boatswain deserved to be hanged(b) What statement does Gonzalo repeat about the boatswain

Gonzalo shows his faith that the boatswain is not destined to die by drowning He is destined to be hanged and nothing can alter this decree of destiny He says that even if the ship was as frail as a nutshell the boatswain could not be drowned for his destiny was to be hanged(c) What do the passengers do when they have lost all hope of their survival

When the passengers have lost all hope of survival they take

(iii) The opening scene justifies the title ndash The Tempest

UNANSWERED QUESTIONS

(i) The King always travels with his entire fleet including his soldiers Where were the other ships

(ii) Why was the ship in that area Where was it coming from or going where

(iii) The ship broke apart What happened to those who were in the ship

(We shall get the answer to the above questions as the play progresses)

leave of life with fervent prayers The mariners take their last hearty drink and are ready for death(d) What blame does Antonio put upon the mariners and the boatswain Antonio rebukes the mariners that these drunkards have brought them to the present crisis by neglecting their duties He blames them saying that they are going to lose their lives entirely for the negligence of the boatswain and his fellows(e) What does Antonio say while cursing the boatswain

Antonio gives vent to his wrath upon the boatswain in particular He calls the boatswain a wide-mouthed rascal who deserves to be hanged on the sea-shore at low water mark so that ten tides might wash over his body and take out of him all the liquor that he has been drinking

Class XIISubject Topic Summary ExecutionHistory Topic

1 1935 ACT AND WORKING OF PROVINCIAL AUTONOMYCONGREE AND OTHER MINISTERSSUB TOPIC GOVERNMENT OF INDIA ACT1935

Government of India Act 1935 This act established a lsquoFederation of Indiarsquo made of British Indian provinces and Indian states and provided for autonomy with a government responsible to the elected legislature in every provinceThis act introduced abolition of Diarchy at provinces The entire provincial administration was introduced to the responsible ministers who were controlled and removed by the provincial legislature The provincial autonomy means two things First The provincial governments were wholly responsible to the provincial legislature Secondly Provinces were free from outside control and interference in the large number of matters The act divided the powers between the centre and provinces in terms of three lists- Federal list( for centre) Provincial list (for province) and concurrent list (for both) Residuary powers were given to the viceroy In the election under the government of India Act the Congress swept the poll the mandate of the people came in favour of the congress so far as general Hindu seats were concerned The Congress did not get a single Muslim seates in Bombay CP UP Sind and BengalIn five provinces Congress had yhe clear majority In BengalNWFPAssam and Bombay Congress emerged as a single largest partyOn the other side the performance of the Muslim League was badThus the Congress formed ministers in 7 provinces out of 11 provinces Coalition ministry was also formed in two other provincesOnly BENGAL AND Punjab had non- congress ministries

1 QUESTION What was the main change introduced by the Government of India ActANSWER a) The Act gave more

autonomy to the provinces b) Diarchy was abolished at the

provincial levelsc) The Governor was the head of

the executived) There was a council of

ministers to advise him The ministers were responsible to the provincial legislatures who controlled them The legislature could also remove the ministers

e) The Governors still retained special reserve powers

2 QUESTION Why did the federal scheme introduced by the Government of India Act 1935 never come into operation

ANSWER The Federal structure of the Government of India was to be composed with the Governor General and Council of ministers The Federal legislature was to be Bicameral legislature- The council of states and the House of Assembly The ministers were to be chosen by the Governor general and they were to hold the office during his pleasure

The provinces of British India would have to join the federation but this was not compulsory for the princely states

This federation never materialised because of the lack of support from the required number of

princely statesThis act was refused and

rejected by the princes the Congress and the Muslim League

Thus both Congress and the League participated in the election of 1937 Thus the federal part was never introduced but the provincial part was put into operations

Bengali 2nd

Language

াচেরর পরাথCনা(কহিতা )

াচেরর পরাথCনা কহিতাটি কহি (ঙখ দেঘাচে4র দো আচো য কহিতায় াচেরর পতর হমায়ন কঠিন দেরাচেগ আxানত ার ঈশবর া আললার কাচেছ পরাথCনা কচেরচেছন তার পচেতরর ীন হিফহিরচেয় হিচেত এই কহিতায় ার পচেতরর ীন হিভbা দেচেয়চেছন ারার এমনহিক হিনচের ীন হিসCচেনর হিহিনমচেয় হিতহিন তার দেছচের ীন হিফচের দেপচেত দেচেয়চেছন তার দেছচের এই দেরাচেগর ন য হিতহিন হিনচেচেকই ায়ী কচেরচেছন তার হিনচের করা পাপচেকই হিতহিন ায়ী কচেরচেছন এছাা রানৈনহিতক ও আথCসামাহিক অসথার কথা তচে ধরা চেয়চেছ এই কহিতায় ার তার হিনচের পাপ কমCচেকই ায়ী কচেরচেছ ার অন যায় ভাচে দেপহি((হিকতর মাধ যচেম অপররা য কচেরচেছ আর এই অন যায় কাচের ন যই তার পহিরাচের হিপযCয় এচেসচেছ দে এক পরকার মানহিক নধন ইহিতাচেসর ার হিপতা চেয় সবাভাহিকভাচে ভাচোাসা দে মমতা দেথচেক মকত চেত পাচেরনহিন তাই হিপতা চেয় আললা া ভগাচেনর কাচেছ পতর হমায়চেনর পরানহিভbা দেচেয়চেছন ার আললা া ভগাচেনর কাচেছ াহিনচেয়চেছন তার হিনচের ীন হিসCন হিচেত হিতহিন রাী তার হিহিনমচেয় পচেতরর ীন হিফচের দেপচেত দেচেয়চেছন াচেরর হিপতসভ হিচেকর কথা এই কহিতায় ফটিচেয় দেতাা চেয়চেছ হিপতা পচেতরর হিরাহিরত মান নধচেনর কথা তচে ধরা চেয়চেছ

হিচে(4 হিকছ াইচেনর তাৎপযC১) ldquoদেকাথায় দেগ ওর সবচছয দেৌন দেকাথায় কচেরায় দেগাপন bয়ldquoউততর) াচেরর পতর হমায়ন কঠিন দেরাচেগ অসসথ তাই তার দেযৌন াহিরচেয় যাচেচছ এই দেরাচেগ তাচেক দেগাপচেন কচেরকচের াচেচছ তার সক (হিকত ধীচের ধীচের bয় চেচছ তাই হিপতা চেয় ার আললার কাচেছ হমায়চেনর পরান হিভbা দেচেয়চেছন২) ldquoাগাও (চেরর পরাচেনত পরানতচের ধসর (ন দেযর আান গানldquoউততর) াচেরর পতর হমায়ন কঠিন দেরাচেগ আxানত তাই ার আ দে(াচেক মমCাত (চেরর পচেথ পরানতচের আান গান ধবহিনত দোক দেসই আান গান আললার কাচেছ দেযন চে যায় আললা দেযন এই আহিতC শচেন পচেতরর ীন হিফহিরচেয় দেয় ৩)ldquoনাহিক এই (রীচেরর পাচেপর ীানচেত দেকানই তরারণ দেনই ভহি4চেতরldquoউততর) হমায়চেনর অসসথতার ন য ার হিনচেচেকই ায়ী কচেরচেছন কারন ার অচেনক রা য অন যায় ভাচে কচেরচেছ তাই তার এই পাপ কাচের ন য তার ঘচের আ হিপ এচেসচেছ এই অন যায় কাচের ন য তার মহিকত দেনই তাই ার আললার কাচেছ এই পাপ কাচেযCর ন য bমা পরাথM

Hindi 2ndlang

-ासी(जयशकर परसा-)

-ासी जयशकर परसा- की एक ऐसी कहानी ह जिजसम भारतीय ससकनित और राषटरीयता का सवरगजीतहोता ह इस कहानी म इरावती एक निहद कनया ह जिजस मलअचछो न मलतान की लट म पकडा और -ासी बना दि-या उस 500 दि-न -कर काशी क एक महाजन न खरी-ा दसरी -ासी निफरोजा ह वह गलाम ह निफरोजा को छडान क कतिलए अहम- को 1000 सोन क कतिसकक भजन थ जो अभी तक नही आए थ राजा साहब कठोर होत हए भी निफरोजा को निबना धनराकतिश क कतिलए उस म कर -त ह वनिफरोजा को अहम- को समझान की बात कहत हकहानी क अत म हम -खत ह निक इरा वती और जाटो क सर-ार बलराज का मिमलन होता हअहम- को यa म मार दि-या जाता ह वहा निफरोजा की परसननता की समामिध बनती ह वहा एक फल चढती ह और डीजल आती ह निफरोजा उस समामिध की आजीवन -ासी बनी रहती हलखक अपन उददशय अथात -ास परथा पर परकाश डालन और इस परथा क कारण होन वाल -ातो क दखो को दि-खान म पणता सफल हए ह

helliphellipContinue to next

Biology Reproductio Today we will discuss about vegetative Q1 Name some vegetative propagules

n in Organisms

propagation of plants The process of multiplication in which fragments of plant body function as propagule and develop into new individual is called vegetative propagation The units of such propagation are runner rhizome tuber bulb etc

and the speciesinvolvedVegetative propagules

Parts involved

Bulb StemBulbil BulbilRhizome Stem Runner Stem Tuber Stem Offset Stem Leaf buds Leaves Suckers Stem

Corns Stem stolon

Q2 State advantages of vegetative propagation

i) Rapid methodii) Sure and easy methodiii) Useful in plants that cannot

produce viable seeds or long seed dormancy

iv) Maintains purity of raceQ 3 Banana fruit is said to be parthenocarpic where as turkey is said to be parthenogenetic WhyBanana develops without fertilization from an unfertilized ovary thus is parthenocarpicIn turkey the ovum or female gamete developinto a new chick without fertilization thus isparthgenetic

Q4 Why is water hyacinth is called as a ldquoTerror of Bengalrdquo Water hyacinth can

propagatevegetatively all over the water body in a short per short period of time This resulted increased biochemicaloxygen oxygen demand of water body causing mortalityof fishes It is very difficult to get rid off them Thus known as terror of Bengal

Chemistry

Solid state GENERAL CHARACTERISTICS OF SOLID STATEIn nature the particular state of matter is governed by two opposing forces at given set of temperature and pressure These forces are intermolecular force of attraction and thermal energy If intermolecular force of attraction is high as compared to thermal energy particles remains in closest position

Intext QuestionsQ1 Classify the following solids as crystalline and amorphous Sodium chloride quartz glass quartz rubber polyvinyl chloride Teflon

A1 Crystalline

and hence very less movement in particles is observed In this case solid state is the preferred state of matter

Let us revise the general characteristics of solid

i) Fixed mass volume and shape

ii) Strong intermolecular force of attraction

iii) Least intermolecular space

iv) Fixed position of constituent particles

v) Incompressible and rigid

Q2 what type of interactions hold the molecules together in a polar molecular solid[CBSE 2010]A2 The molecules in a solid are held together by van der Waals forces The term van der Waals forces include hydrogen bonding dipole-dipole attraction and London dispersion forces All molecules experience London dispersion forces In addition polar molecules can also experience dipole-dipole interactions So the interactions that holds the molecule together in polar molecular solid are London dispersion force and dipole-dipole interactionsQ3 Write a feature that will distinguish a metallic solid from an ionic solid [CBSE 2010]A3 Metals are malleable and ductile whereas ionic solid are hard and brittle Metallic solid has typical metallic lustre But ionic solid looks dullQ4 Write a point of distinction between a metallic solid and an ionic solid other than metallic lustre [CBSE 2012]A4 Metals are malleable and ductile whereas ionic solid are hard and brittleQ5 Write a distinguish feature of metallic solid [CBSE 2010]A5 The force of attraction in

solid Sodium chloride Quartz Amorphous solid Quartz glass rubber polyvinyl chloride Teflon Q2 why glass is considered as super cooled liquidA2 Glass shows the tendency to flow at slower rate like liquid Hence they considered as super cooled liquidQ3 why the window glass of old buildings show milky appearance with timeA3 Glass is an amorphous solid Amorphous solid has the tendency to develop some crystalline character on heating Due to heating in day over the number of years glass acquires some crystalline character and show milky appearanceQ4 why the glass panes fixed to window or doors of old building become slightly thicker at bottomA4 Glass is super cooled liquid It has the tendency to flow down very slowly Due to this glass pane becomes thicker at the bottom over the timeQ5 Sodium chloride is a crystalline solid It shows the same value of refractive index along all the direction TrueFalse Give reasonA5 FalseCrystalline solid shows anisotropy in properties That is it shows different values for the given physical property in different direction All the crystalline solids show anisotropy in refractive index Therefore sodium chloride will show different values of refractive index on different directions

Q6 Crystalline solid are anisotropic in nature What does this statement means

between the constituent particles is special kind of electrostatic attraction That is the attraction of positively charged kernel with sea of delocalized electronsQ6 which group of solid is electrical conductor as well as malleable and ductile [CBSE 2013]A6 Metallic solidQ7 why graphite is good conductor of electricity although it is a network (covalent solid)A7 The exceptional property of graphite is due to its typical structure In graphite each carbon is covalently bonded with 3 atoms in same layer The fourth valence electron of each atom is free to move in between different layersThis free electron makes the graphite a good conductor of electricity

[CBSE 2011]A6 Anisotropy is defined asrdquo Difference in properties when measured along different axis or from different directionsrdquo Crystalline solid show different values of some of the physical properties like electrical resistance refractive index etcwhen measured along the different directions The anisotropy in crystalline solid arises due to the different arrangement of particles in different directions

Math Function Composition of functions Think of an industrial plant that produce bottles of cold drinks first there is the operation (or function) f that puts the cold drink inside the bottle followed by the opeartion g that close the bottle with the capThis leads to the following definitionDefinition Let f A rarr B and g B rarr C be two functions Then the composition of f and g denoted by gof is defined as the function gof A rarr C given by gof(x) = g(f (x)) forall x isinA

Definition A function f X rarr Y is defined to be invertible if there exists a function g Y rarr X such that gof = IX and fog = IY The function g is called the inverse of f and is denoted by f -1

Thus if f is invertible then f must be one-one and onto and conversely if f is one-one and onto then f must be invertible This fact significantly helps for proving a function f to be invertible by showing that f is one-one and onto specially when the actual inverse of f is not to be determined

Example 1 Let f 2 3 4 5 rarr 3 4 5 9 and g 3 4 5 9 rarr 7 11 15 be functions defined as f(2) = 3 f(3) = 4 f(4) = f(5) = 5 and g (3) = g (4) = 7 and g (5) = g (9) = 11 Find gofSolution We have gof(2) = g (f(2)) = g (3) = 7 gof(3) = g (f(3)) = g (4) = 7gof(4) = g (f(4)) = g (5) = 11 and gof(5) = g (5) = 11Example 2 Find gof and fog if f R rarr R and g R rarr R are given by f(x) = cos x and g (x) = 3x2 Show that gof ne fogSolution We have gof(x) = g(f(x))=g(cosx) = 3 (cos x)2

= 3 cos2 x Similarly fog(x)=f(g (x))= f(3x2)= cos (3x2) Note that 3cos2 x ne cos 3x2 for x = 0 Hence gof ne fogExample 3 Show that if f A rarr B and g B rarr C are onto then gof A rarr C is also ontoSolution Given an arbitrary element z isin C there exists a pre-image y of z under g such that g (y) = z since g is onto Further for y isin B there exists an element x in A with f(x) = y since f is onto Therefore gof(x) = g (f(x)) = g (y) = z showing that gof is onto Example 4 Let Y = n2 n isin N sub N Consider f N rarr Y as f(n) = n2 Show that

f is invertible Find the inverse of fSolution An arbitrary element y in Y is of the form n2 for some n isin N This implies that n =radicy This gives a function g Y rarr N defined by g (y) =radicy Nowgof (n) = g (n2)=radicn2 = n and fog (y) =f(radicy) = (radicy) 2 y which shows that gof=IN and fog= IY Hence f is invertible with f -1 = g

Political Science

Constitution of India-The Preamble

Summary

Objective of the state-To secure equality of status and of opportunity To promote fraternity among all the citizens To assure the dignity of the individuals and Unity and integrity of the nation

Justice-Justice stands for rule of law absence of arbitrariness and a system of equal rights freedom and opportunities for all in a society India seeks social economic and political justice to ensure equality to its citizens

Liberty-Liberty implies the absence of restraints or domination on the activities of an individual such as freedom from slavery serfdom imprisonment despotism etc The Preamble provides for the liberty of thought expression belief faith and worship

Equality-Equality means the absence of privileges or discrimination against any section of the society The Preamble provides for equality of status and opportunity to all the people of the country

Fraternity-The Preamble declares that fraternity has to assure two thingsmdashthe dignity of the individual and the unity and

Execution

Answer the following questions-

Short notes-1 Equality2 Fraternity3 Justice4 Liberty

Homework-Learn

integrity of the nation The word integrity has been added to the Preamble by the 42nd Constitutional Amendment (1976)

Business studies

Human resource management (chapter 1)

On the day of 1504 2020 I have discussed with you the managerial functions and procurement functions of HRM

Today weare going to discuss about the development function integration functions and maintenance function

Development functions-HRM improves the knowledge skills attitude and values of employees so that they the present and future jobs more effectively it includes

1) Development functions of HRM

a) Performance appraisal = It implies systematic evaluation of employees with respect to their performance on the job and their potential for development

b) Training =It is the process by which employees learn knowledge skills and attitudes to achieve organisational and personal goals

c) Executive development = It is the process of developing managerial talent through appropriate program

2) Integration functionsa) HRM reconcile the goals of

organisation with those of its members through integrating function

b) HRM tries to motivate employees to various financial and non financial incentives provided in job specification etc

3) Maintenance functiona) HRM promote and protect the

physical and mental health of employees by providing several types of benefits like housing medical aid etc

b) It Promote Social security measures to employees by providing provident fund pension gratuity maternity benefits

SubjectCOMMERCE

Topic

BUSINESSENVIRONMENT

Summary

Now quickly let us revise the earlier points that we have already done in the last class and let us proceed with the other topics that are there in the chapter

Firstly we will recall the internal and external factors of micro environment and then we

Execution 3 What do you mean by internal factors

in micro environmentAnswerInternal factors refer to all the factors existing within a business firm The internal factors are considered controllable because the enterprise has control over these factors

Development FunctionsPerformance AppraisalTrainingExecution Development

shall proceed in details

Meaning and list of internal and external factors

aInternal factorsInternal factors refer to all the factors existing within a business firm The internal factors are considered controllable because the enterprise has control over these factorsFor an example a company can alter its organization structure policies programmes employees physical facilities and marketing mix to suit the changes in the environmentList of internal factors areCorporate culture mission and objectives top management organizations structure company image and brand equity company resources

b External factorsExternal factors refer to those individual and groups and agencies with which a particular business organization comes into direct and frequent contact in the course of its functioningThese individuals and groups are known as STAKEHOLDERS because they have a stake (financial interest ) in the working and performance of the particular business List of external forces (stakeholders)Customers competitors investors suppliersmiddlemen (marketing intermediaries)financers publics

customers

suppliersfinancers

For an example a company can alter its organization structure policies programmes employees physical facilities and marketing mix to suit the changes in the environment

4 What do you mean by external factors in micro environment

AnswerExternal factors refer to those individual and groups and agencies with which a particular business organization comes into direct and frequent contact in the course of its functioningThese individuals and groups are known as STAKEHOLDERS because they have a stake (financial interest) in the working and performance of the particular business

3Who are stakeholdersSTAKEHOLDERS are individuals and groups who have a stake (financial interest ) in the working and performance of the particular business 4Discuss the internal factors in briefa Corporate CultureThe values beliefs and attitudes of the founders and top management of the company exercise a strong influence on what the cmpaany stands for how it does things and what it considers importantbMission and objectivesThe business philosophy and purpose of a comoany guide it prioritiesbusiness strategiesproduct market scope and development scope

cTop management structurethe composition of board of directors the degree of professionalization of management and the organizational structure of a company have important bearing on its business decisions

dPower structureThe internal power relationship between the board of directors and the chief executive is an important factor

e Company image and brand equityThe image and brand equity of the company play a significant role in raising finance forming alliance choosing dealers and suppliers launching new products entering foreign markets

5 What is Macro environmentAnswerMacro environment refers to the general

competitors

middlemen

publics

Fig STAKEHOLDERS OF A COMPANY

Apart from micro environment the other main dimension of business environment isMacro environment Macro environment refers to the general environment or remote environment within which a business firm and forces in its micro environment operateA company does not directly or regularly interact with the micro environmentTherefore macro environment is also known as indirect action EnvironmentThe macro environment forces are less controllable than the micro forces

Macro environment consists of the following components

POLITICAL AND LEGAL ENVIRONMENT

ECONOMIC SOCIAL AND ENVIRONMENT

CULTURAL

ENVIRONMENT

TECHNOLOGICAL ENVIRONMENT

Fig COMPONENTS OF MACRO ENVIRONMENT

environment or remote environment within which a business firm and forces in its micro environment operateA company does not directly or regularly interact with the micro environmentTherefore macro environment is also known as indirect action EnvironmentThe macro environment forces are less controllable than the micro forces 6 What are the components of macro environmenta Political and legal environmentb Economic environmentc Social and cultural environmentd Technological environment

Computer Science

Logic gates

Digital systems are said to be constructed by using logic gates These gates are the AND OR NOT NAND NOR EXOR and EXNOR

BUSINESS FIRM

gates The basic operations are described below with the aid of truth tables

AND gate

The AND gate is an electronic circuit that gives a high output (1) only if all its inputs are high A dot () is used to show the AND operation ie AB Bear in mind that this dot is sometimes omitted ie ABOR gate

The OR gate is an electronic circuit that gives a high output (1) if one or more of its inputs are high A plus (+) is used to show the OR operationNOT gate

The NOT gate is an electronic circuit that produces an inverted version of the input at its output It is also known as an inverter If the input variable is A the inverted output is known as NOT A This is also shown as A or A with a bar over the top as shown at the outputs The diagrams below show two ways that the NAND logic gate can be configured to produce a NOT gate It can also be done using NOR logic gates in the same way

NAND gate

This is a NOT-AND gate which is equal to an AND gate followed by a NOT gate The outputs of all NAND gates are high if any of the inputs are low The symbol is an AND gate with a small circle on the output The small circle represents inversion

NOR gate

This is a NOT-OR gate which is equal to an OR gate followed by a NOT gate The outputs of all NOR gates are low if any of the inputs are highThe symbol is an OR gate with a small circle on the output The small circle represents inversion

EXOR gate

The Exclusive-OR gate is a circuit which will give a high output if either but not both of its two inputs are high An encircled plus sign ( ) is used to show the EOR operation

EXNOR gate

The Exclusive-NOR gate circuit does the opposite to the EOR gate It will give a low output if either but not both of its two inputs are high The symbol is an EXOR gate with a small circle on the output The small circle represents inversion The NAND and NOR gates are called universal functions since with either one the AND and OR functions and NOT can be generated

Note A function in sum of products form can be implemented using NAND gates by replacing all AND and OR gates by NAND gates A function in product of sums form can be implemented using NOR gates by replacing all AND and OR gates by NOR gates

Logic gate symbols

Table 2 is a summary truth table of the inputoutput combinations for the NOT gate together with all possible inputoutput combinations for the other gate functions Also note that a truth table with n inputs has 2n rows You can compare the outputs of different gates

Logic gates representation using the Truth table

Example

A NAND gate can be used as a NOT gate using either of the following wiring configurations

Subject Eng Literature (The Tempest ndash William Shakespeare) Topic Act III Scene 3 Lines 53 to 110 (End of the scene) Date 16th April 2020 (2nd Period)

[Students should read the original play and also the paraphrase given in the school prescribed textbook]Summary Questions amp Answers

o Seeing this strange scene all are inclined to believe the tales told by travelers that there truly are ldquounicornsrdquo and ldquothe phoenixrsquo thronerdquo

o As they are about to sit down to the feast the banquet is snatched away by a harpy (Ariel disguised) A spiritrsquos voice (Arielrsquos voice) denounces Alonso Sebastian and Antonio with particular

1 ARIEL You are three men of sin whom Destiny

(Line 53-58)That hath to instrument this

lower world And what is int the never-surfeited sea

Hath caused to belch up you and on this island

Where man doth not inhabit you rsquomongst men

Being most unfit to live I have made you mad

reference to their crime in expelling Prospero from Milan They have not received any punishment for their deed earlier but the time for their punishment has arrived Upon Alonso it pronounces ldquolingering perdition worse than deathrdquo from which there is no remedy except through sincere repentance Ariel then vanishes in thunder and the shapes enter again and carry away the table

o Prospero watching invisibly is very pleased with the performance of Ariel and his (Prosperorsquos) ldquomeaner ministersrdquo All his enemies are now in his power and are in a fit of desperation He then leaves them and goes to see how Ferdinand and Miranda are getting on

o Alonso is now much humbled and penitent with the after effect of the spiritrsquos denunciation of his crimes He believes that his son is lost forever After this all disperse being stricken mad by the speech of the spirit

o Gonzalo fearing that they may do violence to themselves or to one another follows them and bid others to follow

(a) To whom does Ariel disguised as a harpy call the three sinners What game did Fate of Destiny play with

them

The three sinners called by Ariel are Alonso Sebastian and Antonio It was Destiny which had caused the ocean to cast the three sinners on the shore Though the ocean is all the time devouring whatever appears on its surface and is never satisfied with its continual swallowing of the ships and men in the present case the ocean had cast these three sinners on the shore without killing them

(b) Who had jointly been responsible for the conspiracy against Prospero What is Prosperorsquos purpose behind all this

Three men Alonso Sebastian and Antonio had jointly

been responsible for the conspiracy against Prospero They had driven out Prospero form Milan Prosperorsquos purpose is to make these three sinners realize the wrong they had done He wants them to repent for their criminal deeds because repentance leads to self-esteem(c )What does Ariel (the harpy) tell Alonso and his companions when they take out their swords to attack him

Seeing them drawing their swords Ariel (harpy) tells them that he and his companions are the instruments of destiny and that it is not possible for human beings to do them any injury He says that the swords of human beings can not injure even a minute part of his feathers Their swords are as ineffective against him and his companions as against the wind or the water

(d) Give the explanatory meanings of the following expressions in the context of the above extract

(i)Never surfeited (ii) Belch up (iii) lsquomongst men

(i) Never surfeited never led to satisfaction

(ii) Belch up cast ashore(iii) lsquomongst men in human

society2

I and my fellows (Line 60-65)

Are ministers of Fate The elementsOf whom your swords are tempered may as wellWound the loud winds or with bemocked-at stabsKill the still-closing waters as diminishOne dowl thats in my plume

IMPORTANT PASSAGES EXPLAINED

The elements

(Line 61-66)Of whom your swords are tempered may

as wellWound the loud winds or with

bemocked-at stabs

(a) Who is lsquoIrsquo Who are his lsquofellowsrdquo

lsquoIrsquo is referred to Ariel in disguise of a harpy His lsquofellowsrsquo are other spirits serving Prospero the real Duke of Milan who has acquired supernatural powers after being banished from his Dukedom Prospero has settled in this uninhabited island

(b) What are the elements that have temperrsquod the swords Why will it not work against the speaker

The swords (of Alonso and his companions) are tempered by metal (steel) which is taken out of the earth and refined by

Kill the still-closing waters as diminishOne dowl thats in my plume My fellow

ministersAre like invulnerable

In these words Ariel reminds the King and his companions of the utter futility of drawing swords against himself and his fellows Ariel drives Alonso Antonio and Sebastian the three men of sin to desperation ndash a state in which men do violence to themselves They draw swords to strike Ariel But Ariel reminds them that he and the other spirits are the ministers of destiny and nothing can wound them The steel of which their swords are made of may cut the wind or water which being divided always closes up again Even supposing that such things may be possible it is quite impossible that their swords will cut one feather in their plume They are incapable of being wounded by any sword of man Hence it is foolish on their part to attempt to strike at Ariel and his fellow-spirits

For which foul deed

(Line 72-75)The powers delaying not forgetting

haveIncensed the seas and shores yea all the

creatures Against your peace

Ariel enters like a harpy and remaining invisible tells Alonso Sebastian and Antonio that he and other harpies are the agents of Destiny appointed to carry out her decrees He tells them that their punishment for the crime against Prospero which has been so long deferred is now to fall upon them He reminds them that they had expelled Prospero from Milan and set him and his innocent child adrift on the sea and that the sea had paid them back for their sin by the shipwreck and by the calamities they have suffered He tells them that the powers above which did not forget this mean treachery but only deferred the punishment have now engaged the seas and the shores and all living beings including him and his comrades against them The very elements and supernatural agency Ariel adds have taken up the avenging of their crime against Prospero

the action of fire It may cut the wind or water which being divided always closes up again

The sword will not work against the spirits and the harpy because they are the ministers of destiny and nothing can wound them nor it will cut a single feather in their plume

(c )What is the meaning of lsquodowlrsquo in the last line

The term lsquodowlrsquo means a filament or the smallest part of a feather In this context Ariel in disguise of harpy says that their sword cannot even damage the smallest filament of their (Arielrsquos and other spirits) feathers as they are incapable of being wounded by any sword of man

(d) What does the speaker remind the listeners about

Ariel in disguise of harpy reminds Alonso the King of Naples Sebastian Alonsorsquos brother and Antonio the present Duke of Milan and the treacherous brother of Prospero as they being three men of sin He even reminds them that their punishment for their crime against Prospero which has been so long deferred now falls upon them He reminds them that they have expelled Prospero from Milan and has set him along with his innocent infant daughter adrift on the sea So the sea has paid them back for their sin by their shipwreck and the calamities they have suffered since then The harpy rebukes Alonso of his sin that has incensed the Gods and has deprived him of his son as a punishment

(e) How do they respond

When Ariel in disguise of a harpy reminds Alonso Sebastian and Antonio of their past misdeeds and sin Alonso has a look of terror and confusion in his eyes He utters the words of sincere repentance wrung out of his conscience-stricken heart It appears to him that all the elements of nature the sea-waves the wind and the thunder proclaiming a loud voice in the name of Prospero and the crime Alonso has committed against him They are calling upon him to repent There is a deep storm raging in Alonsorsquos breast and the echoes of that storm are ringing in his ears like a clear note of wind-instrument A note of denunciation of Alonsorsquos crime leaves him much humbled and penitent and confirms his belief that his son is lost forever But Sebastian and Antonio shows some courage instead of repentance They wish to kill the spirits or devils if it appears

3

Of my instruction hast thou nothing bated (Line 85-93)

In what thou hast to say So with good life

And observation strange my meaner ministers

Their several kinds have done My high charms work

And these mine enemies are all knit upIn their distractions They now are in my

powerAnd in these fits I leave them while I visitYoung Ferdinand whom they suppose is

drownedAnd his and mine loved darling

Methought the billows spoke and (Line 96-99)

told me of itThe winds did sing it to me and the

thunderThat deep and dreadful organ-pipe

pronouncedThe name of Prosper It did bass my

trespass

These are the words of contrition coming from Alonso Ariel has driven him to a deep repentance for conspiring with Antonio against Prospero He now feels a sincere remorse It appears to him that all the elements of nature the sea-waves the wind and the thunder proclaimed with a loud voice the name of Prospero and the crime Alonso had committed against him They are calling upon him to repent There is a deep storm raging in Alonsorsquos breast and the echoes of that storm are ringing in his ears like the clear note of a wind-instrument

Comment These are the words of sincere repentance wrung out of the conscience-stricken heart of Alonso Alonso who is the lesser villain is the first to give way to remorse under the effect of Arielrsquos speech The words of Ariel seem to him to be the voice of conscience speaking to him He is driven to desperation a state in which he might do violence to his life

(a) Identify the speaker State the context

Prospero the ruler of the island is the speaker The famous banquet scene has been enacted very well Ariel and his junior spirits have played their roles excellently Prospero is glad to say words of praise for them(b) In what way the speakerrsquos instructions have been carried out

According to Prosperorsquos instructions a banquet was presented before the King of Naples and his companions when they were tired and hungry Just when they were preparing to eat the feast the banquet was suddenly removed by exercising supernatural powers All this was done by Ariel Prosperorsquos chief assistant and a powerful spirit

Ariel not only made the feast disappear but also delivered his speech blaming the King and his two companions for their past wicked deeds He warned them to repent for their misdeeds or suffer forever on that uninhabited island

(c) Who are referred to as lsquomeaner ministersrsquo What have they done

Prospero refers as lsquomeaner ministersrsquo to his other lesser spirits who were assisting Ariel in presenting a scene before the kingrsquos party They entered the scene to the accompaniment of music They assumed several strange shapes and brought in a banquet Then they danced about it with gentle actions of salutations thus inviting the King and others to eat the feast

These spirits play their role again when Ariel in the shape of a harpy quits the scene These shapes enter again and dancing with mocking gestures carry away the table

(d) Who are the speakerrsquos enemies What has happened to them

King of Naples Alonso his brother Sebastian and the present Duke of Milan Antonio (Prosperorsquos own brother) are Prosperorsquos enemies With the turn of events they have all been washed ashore on the island which is ruled by Prospero the great magician Actually this happened after the shipwreck caused by a storm which was raised by Prospero with the purpose of bringing these people to his island Prosperorsquos spirits have already confused and terrified these enemies and they are under Prosperorsquos control He can treat them as he likes

(e) What does he say about Ferdinand Explain what is meant by ldquohellip his and mine darlingrdquo

Prospero knows that Alonsorsquos son prince Ferdinand is alive though his father thinks that the prince has been drowned

Prospero refers to his daughter Miranda who is dear to him She is also very dear to Prince Ferdinand who has fallen in love with her They are waiting to be married soon for which they have received Prosperorsquos consent

4

ALONSO O it is monstrous monstrous (Line 95-102)

Methought the billows spoke and told me of it

The winds did sing it to me and the thunderThat deep and dreadful organ-

pipe pronouncedThe name of Prosper It did bass

my trespassTherefore my son ithrsquo ooze is

bedded andIll seek him deeper than eer

plummet soundedAnd with him there lie mudded

(a) In what way does Alonso express his horror when his conscience is awakened by Arielrsquos words

When Alonsorsquos conscience is awakened by Arielrsquos words he expresses his horror at what he has heard He gets the feeling that the waves of the ocean the wind and the loud thunder have spoken to him and uttered the name of Prospero Because of being reminded of his crime in a very loud and rough voice he comes to realize that he has lost his son for his past misdeeds

(b) What does Alonso imagine about his son What does Alonso want to do in his desperate state

Alonso imagines that his son is lying in the mud at the bottom of the sea He feels desperate that he wants to drown himself in the ocean deeper than the plumb-line has ever gone He wants to lie with his son at the bottom of the sea

(c) How do Sebastian and Antonio want to face the evil spirits

Sebastian says that he is not at all afraid of what the harpy has said and that he is prepared to fight any number of such monsters if they appear before him only one at a time Antonio says that he would support Sebastian in the fight against the fiendsyyy

(d) Why does Gonzalo ask Adrian to follow the three men

Gonzalo tells Adrian that all the three men namely Alonso Sebastian and Antonio are in a wild and reckless mood The thought of the heinous crime of which they are guilty has begun to torment their minds So he asks Adrian to follow those three men without loss of time and prevent them from doing anything which the turmoil in their minds might lead them to do

(e) What opinion do you form of Alonso from the above extract

Alonso who is the lesser villain is the first to give way to remorse under the effect of Arielrsquos speech The words of Ariel seem to him to be the voice of conscience speaking to him He is driven to desperation a state in which he might do violence to his life

Subject =Accounts

Ac-12 15420 topic-pL Appropriation ac

PROFIT AND LOSS APPROPRIATION ACCOUNT

MEANING AND PREPARATIONProfit and Loss Appropriation Account is merely an extension of the Profit and Loss Account of the firm The profit of the firm has to be distributed amongst the partners in their respective profit sharing ratio But before its distribution it needs to be adjusted All Adjustments like partnerrsquos salary partnerrsquos commission interest on capital interest on drawings etc are made in this account These adjustments will reduce the amount of profit for distribution This adjusted profit will be distributed amongst the partners in their profit sharing ratio To prepare it at first the balance of Profit and Loss Account is transferred to this account The journal entries for the preparation of Profit and Loss Appropriation Account are given below

1 for transfer of the balance of Profit and Loss Account to Profit and Loss Appropriation Account

(a) In case of Net Profit

Profit and Loss Ac helliphelliphelliphelliphellipDrTo Profit and Loss Appropriation Ac(Net Profit transferred to Profit and Loss Appropriation Ac)

(b)In case of Net Loss

Profit and Loss Appropriation Achelliphelliphellip DrTo Profit and Loss Ac(Net Loss transferred to Profit and Loss Appropriation Ac)

2 for Interest on Capital

For transferring on Interest on CapitalProfit and Loss Appropriation Achelliphelliphellip DrTo Interest on Capital Ac(Interest on capital transferred to Profit amp Loss Appropriation Ac)

3 for Interest on Drawings

For transferring Interest on Drawings Interest on Drawings Achelliphelliphelliphelliphelliphellip DrTo Profit and Loss Appropriation Ac(Interest on drawing transferred to Profit amp Loss Appropriation Ac)

4 For Partnerrsquos SalaryFor transfer of partnerrsquos SalaryProfit and Loss Appropriation Achelliphellip DrTo Salary Ac(Salary transferred to profit amp Loss Appropriation Ac)

5 For Partnerrsquos CommissionFor transferring commissionProfit and Loss Appropriation Achelliphelliphellip DrTo Commission Ac(Commission transferred to Profit and Loss Appropriation Ac)

6 For Transfer of agreed amount to General ReserveProfit and Loss Appropriation Ac helliphellipDrTo General Reserve Ac(Transfer to General Reserve)

7 for share of Profit or Loss appropriation(a) If ProfitProfit and Loss Appropriation Achelliphellip DrTo Partnerrsquos CapitalCurrent Ac(Profit transferred to capitalcurrent Ac)(b) If LossPartnerrsquos Capital Current Achelliphelliphelliphellip DrTo Profit and Loss Appropriation Ac(Loss transferred to capitalcurrent Ac)

THE FORMAT OF PROFIT AND LOSS APPROPRIATION

Profit and Loss Appropriation Account for the year endedhelliphelliphelliphellip

Particulars Amount Particulars Amount

To PL Ac (loss) By pL Ac (profit)

To Interest on capital BY Interest on drawings

To partner`s commission by Partner`s capital Ac ( loss)

To Partner`s salary To Interest on partner`s loan To General Reserve To Partner`s Capital AC (Profit)

Subject= Economics

MOVEMENT ALONG THE DEMAND CURVE (CHANGE IN QUANTITY DEMANDED)In law of demand you have already studied the inverse relationship between price and quantity demanded When quantity demanded of a commodity changes due to change in its price keeping other factors constant it is called change in quantity demanded It is graphically expressed as a movement along the same demand curve There can be either a downward movement or an upward movement along the same demand curve Upward movement along the same demand curve is called contraction of demand or decrease in quantity demanded and downward movement along the same demand curve is known as expansion of demand or increase in quantity demanded

Extention of demandd

price (rs)p A

B Extentionp1 d

Q Q1

Quantity demanded ( in units)

Contraction of demandd

p2 Ccontraction

p APrice (Rs)

d

Q2 Q

Quantity demanded (in units)

Explanation of movement of demand A fall in price from OP to OP1 leads to increase in quantity demanded from OQ to OQ1 (expansion of demand) resulting in a downward movement from point A to point B along the same demand curve DD When Price rises from OP to OP2 quantity demanded falls from OQ to OQ2 (contraction of demand) leading to an upward movement from point A to point C along the same demand curve DD

  • Activity Series of Metals
    • Drawbacks of Rutherfordrsquos model of atom
      • Electromagnetic radiations
      • Properties of electromagnetic radiations
      • Characteristics of electromagnetic radiations
        • Plancks Quantum Theory-
        • Photoelectric effect
          • Intext Questions
            • Logic gates
            • Digital systems are said to be constructed by using logic gates These gates are the AND OR NOT NAND NOR EXOR and EXNOR gates The basic operations are described below with the aid of truth tables
            • AND gate
            • Example
Page 34:  · Web viewSubject. Topic. Summary. Execution. English 1 . Chapter 1 naming words . Page 8. Write the names of these pictures:- Person:-1. father. 2.Firefighter 3.doctor 4 ...

g Profession

institute of Engineers (India)

Accounts Basic accounting terms

Today we will give you some questions from the previous study material

Questions6) Define accounting7) What do you mean by debit

and credit8) Explain the types of account9) Define the following terms

a) Assetsb) Capitalc) Purchased) Debtorse) Transactions

10) Name the types of accounts given below

a) Krishnas accountb) Machinery accountc) Royalty accountd) Salary accounte) Furniture accountf) Audit fee account

Economics Basic Economic ConceptsSub topic

UTILITY

Before starting todayrsquos class we shall recall the last class which was about UTILITY AND THE FEATURES OF UTILITY

Now we shall proceed with the further topics of the chapter

Todayrsquos topic from the chapter lsquo Basic Economic Conceptsrsquo will be TOTAL UTILITY amp MARGINAL UTILITYNow let us quickly revise the concept of utility with an example ie goods and services are designed because they have an ability to satisfy human wantsThis feature of being able to satisfy human wants is termed as utility For example we derive utility from WiFi services as it gives us satisfaction by connecting us to our friends and family through social media here consumers derive utility from WiFi services

From the above concept we shall start with todayrsquos topicEconomists have defined TOTAL UTILITY (TU) as the total satisfaction obtained by consuming a given total amount of a good and serviceFor example the total satisfaction obtained from eating 10 mangoes is the total utility of 10 mangoes

MARGINAL UTILITY (MU) is the additional satisfaction derived from each additional unit

Questions1 What is Total Utility (TU)

Answer Total Utility (TU) is the

aggregate of the utility that a consumer derives from the consumption of a certain amount of a commodityTU=MU1+MU2++MUn

2 What is Marginal UtilityAnswer

Marginal Utility (MU) is the additional made to the total utility as consumption is increased by one more unit of the commodityMU= TUn ndashTUn-1

NoteOften economists tend to

subdivide utility into an imaginary unit called UTIL

consumed In this casethe utility obtained from each mango as it is consumed as the MU of that mango It is also defined as the addition made to the total utility when an additional unit is consumed Often economists tend to subdivide utility into an imaginary unit called UTIL

Note As a consumer increases the consumption of a good over period of time the total utility or total satisfaction derived from it increases to appoint and thereafter it decreasesHowever as the consumer keeps on consuming the good the marginal utility or the additional utility derived from it decreases

SubjectBusiness studies

Topic

BUSINESSENVIRONMENT

Summary

Now quickly let us revise the earlier points that we have already done in the last class and let us proceed with the other topics that are there in the chapter

Firstly we will recall the internal and external factors of micro environment and then we shall proceed in details

Meaning and list of internal and external factors

aInternal factorsInternal factors refer to all the factors existing within a business firm The internal factors are considered controllable because the enterprise has control over these factorsFor an example a company can alter its organization structure policies programmes employees physical facilities and marketing mix to suit the changes in the environmentList of internal factors areCorporate culture mission and objectives top management organizations structure company image and brand equity company resources

b External factorsExternal factors refer to those individual and groups and agencies with which a particular business organization comes into direct and frequent contact in the course of its functioningThese individuals and groups are known as STAKEHOLDERS because they have a stake (financial interest ) in the working and performance of the particular business List of external forces (stakeholders)Customers competitors investors suppliersmiddlemen (marketing intermediaries)

Execution 1 What do you mean by internal

factors in micro environmentAnswerInternal factors refer to all the factors existing within a business firm The internal factors are considered controllable because the enterprise has control over these factorsFor an example a company can alter its organization structure policies programmes employees physical facilities and marketing mix to suit the changes in the environment

2 What do you mean by external factors in micro environment

AnswerExternal factors refer to those individual and groups and agencies with which a particular business organization comes into direct and frequent contact in the course of its functioningThese individuals and groups are known as STAKEHOLDERS because they have a stake (financial interest) in the working and performance of the particular business

3Who are stakeholdersSTAKEHOLDERS are individuals and groups who have a stake (financial interest ) in the working and performance of the particular business 4Discuss the internal factors in briefa Corporate CultureThe values beliefs and attitudes of the founders and top management of the company exercise

financers publics

customers

suppliersfinancers

competitors

middlemen

publics

Fig STAKEHOLDERS OF A COMPANY

Apart from micro environment the other main dimension of business environment isMacro environment Macro environment refers to the general environment or remote environment within which a business firm and forces in its micro environment operateA company does not directly or regularly interact with the micro environmentTherefore macro environment is also known as indirect action EnvironmentThe macro environment forces are less controllable than the micro forces

Macro environment consists of the following components

POLITICAL AND LEGAL ENVIRONMENT

ECONOMIC SOCIAL AND ENVIRONMENT

CULTURAL

ENVIRONMENT

TECHNOLOGICAL ENVIRONMENT

a strong influence on what the cmpaany stands for how it does things and what it considers importantbMission and objectivesThe business philosophy and purpose of a comoany guide it prioritiesbusiness strategiesproduct market scope and development scope

cTop management structurethe composition of board of directors the degree of professionalization of management and the organizational structure of a company have important bearing on its business decisions

dPower structureThe internal power relationship between the board of directors and the chief executive is an important factor

eCompany image and brand equityThe image and brand equity of the company play a significant role in raising finance forming alliance choosing dealers and suppliers launching new products entering foreign markets

5 What is Macro environmentAnswerMacro environment refers to the general environment or remote environment within which a business firm and forces in its micro environment operateA company does not directly or regularly interact with the micro environmentTherefore macro environment is also known as indirect action EnvironmentThe macro environment forces are less controllable than the micro forces 6 What are the components of macro environmenta Political and legal environmentb Economic environmentc Social and cultural environmentd Technological environment

BUSINESS FIRM

Fig COMPONENTS OF MACRO ENVIRONMENTPolitical science

Introduction to political science

Comparative politics and itrsquos scope Comparative politics is the second major dimension of political scienceIt is also a very vast area of study and a very large number of political scientists even treat it as an autonomous area of study within the board ambit of political scienceScope of comparative politics-

1 All political structures -Comparative politics includes the study of all structures formalnon formal governmental and extra governmental which are directly or indirectly involved in politics in all the countries of the world

2 Functional studies- Comparative politics seeks to study politics less from the point of view of the legal institutions in terms of their powers and move from the point of view of their functions which constitute the political process and their actual Operation in the environment

3 Study of political behaviour- Another important part of its scope is the study of the actual behaviour of the people in the process of politics

4 Study of similarities and differences- comparative politics also undertakesan analysis of the similarities and differences among political process and functions

5 Study of all political systems -comparative politics seeks to analyse the actual behaviour and performance of all political systems western as well as non western

6 Study of the environment and infrastructure of politics-The study of politics demands a study of the psychological sociological economic and anthropological environment in fact the social environment as a whole in which each political system operates

7 Study of political culture- political culture is composed of attitudesbeliefs emotions and values of a society that relate to the political system or politics

8 Study of political participation- Political participation is a universal processThe only difference is that while in some states it is limited in others it is wider

9 Study of political process- political

Answer the following questions-

What is comparative politics

What are the scope of comparative politics

Homework- learn

processes like decision makingpolicy making judicial process leadership recruitment process and others are always at work in all political systems

The scope of comparative politics is very comprehensive It includes everything that falls within the area of political activity and political process

History CAMBRIDGE VIEW ABOUT

THE PARTITION

AND REFUTATION

OF CAMBRIDGE

VIEW

Cambridge view about the Partition The Cambridge school of historians have interpreted that opposition to partition scheme was made entirely by the elitist groups They hold the view that Lord Curzon planned to partition the Bengal for administrative purposeREFUTATION OFCAMBRIDGE VIEW The Rationalist historians have rejected the interpretations of the Cambridge School of historians on various grounds

1 QUESTION State different views of historians regarding Partition of Bengal

ANSWER Cambridge historians believed that Lord Curzon partitioned Bengal for administrative reasons only and not for the political motive The Middle class elitist group protested because of their petty interest The Hindu zamindars protested as they have to spend more money for managing their estatesThe lawyers of Calcutta High court feared to lose their clientBut according to the nationalist Historians was-

2- The ultimate object of Lord Curzon was to crush the unity of Bengal politicians

3- If Bengal becomes a separate province Bengali speaking 16 million people of western part would become minority under Hindi speaking people of Bihar and Oriya speaking people of Orissa

4- The bureaucrats expected that the protest movement would die down quickly

5- Lord Curzon used the Muslim community in his political game

6- Idealism had great contribution in the protest against partition

7- The people of the every section of society were affected by the partition of Bengal

Computer Science

Numbers Convertion of dcimal number to octal numberThe decimal numeral system is the standard system for denoting integer and non-integer numbers It is the extension to non-integer numbers of the Hindu-Arabic numeral system For writing numbers the decimal system uses ten decimal digits a decimal mark and for negative numbers a minus sign - The decimal digits are 0 1 2 3 4 5 6 7 8 9 the decimal separator is the dot in many countries

The octal numeral system or oct for short is the base-8 number system and uses the digits 0 to 7 Octal is sometimes used in computing instead of hexadecimal perhaps most often in modern times in conjunction with file

permissions under Unix systems It has the advantage of not requiring any extra symbols as digits It is also used for digital displays

Follow these steps to convert a decimal number into octal form

1 Divide the decimal number by 82 Get the integer quotient for the next iteration (if the number will not divide equally by 8 then round down the

result to the nearest whole number)3 Keep a note of the remainder it should be between 0 and 74 Repeat the steps until the quotient is equal to 05 Write out all the remainders from bottom to top This is the solution

For example if the given decimal number is 8453

Division Quotient Remainder

8453 8 1056 5

1056 8 132 0

132 8 16 4

16 8 2 0

2 8 0 2

Then the octal solution is 20405

Subject Eng Literature (The Tempest ndash William Shakespeare) Topic Act I Scene 1 Lines 33 to 67 (End of scene) Date 16th April 2020 (4th Period)

[Students should read the original play and also the paraphrase given in the school prescribed textbook]Summary Questions amp Answers

[SUMMARY OF THE ENTIRE SCENE]

o The play starts with the scene of a severe storm at sea Alonso (King of Naples) Sebastian (Alonsorsquos brother) Ferdinand (Alonsorsquos son) Gonzalo Antonio (the usurping Duke of Milan) are in a ship in the midst of the storm

o The mariners are trying their best to control the vessel from running aground and are totally following the orders of their Master the Boatswain They have scant success

o The mariners become extremely unhappy and annoyed when most of the passengers arrive on the deck thereby hampering their effort to save the ship There is serious confrontation between them and the passengers who are part of the Kingrsquos entourage

o The mariners could not save the ship

SUMMING-UP

(i) Vivid description of the scene which gives a realistic description of terror and confusion of a tropical storm

(ii) Shows Shakespearersquos accuracy of knowledge in describing the naval operations and also matters of seamanship

(1) GONZALO Ill warrant him for drowning (L 45-57)

though the ship were no stronger than a nutshell and as leaky as an unstanched

wenchBOATSWAIN Lay her a-hold a-hold Set her two courses Off to

sea again lay her offMARINERS All lost To prayers to prayers All lostBOATSWAIN What must our mouths be coldGONZALO The king and prince at prayers Lets assist them

For our case is theirsSEBASTIAN Im out of patienceANTONIO We are merely cheated of our lives by drunkards

This wide-chopped rascal - would thou mightst lie drowning the washing of ten tides

(a) What does Antonio say at the insolent manners of the boatswain just before the given passage

Being irritated at the insolent manners of the boatswain just before the given extract Antonio the Duke of Milan calls him a worthless dog son of a woman without any morals an arrogant and disrespectful noisemaker He says that the boatswain deserved to be hanged(b) What statement does Gonzalo repeat about the boatswain

Gonzalo shows his faith that the boatswain is not destined to die by drowning He is destined to be hanged and nothing can alter this decree of destiny He says that even if the ship was as frail as a nutshell the boatswain could not be drowned for his destiny was to be hanged(c) What do the passengers do when they have lost all hope of their survival

When the passengers have lost all hope of survival they take

(iii) The opening scene justifies the title ndash The Tempest

UNANSWERED QUESTIONS

(i) The King always travels with his entire fleet including his soldiers Where were the other ships

(ii) Why was the ship in that area Where was it coming from or going where

(iii) The ship broke apart What happened to those who were in the ship

(We shall get the answer to the above questions as the play progresses)

leave of life with fervent prayers The mariners take their last hearty drink and are ready for death(d) What blame does Antonio put upon the mariners and the boatswain Antonio rebukes the mariners that these drunkards have brought them to the present crisis by neglecting their duties He blames them saying that they are going to lose their lives entirely for the negligence of the boatswain and his fellows(e) What does Antonio say while cursing the boatswain

Antonio gives vent to his wrath upon the boatswain in particular He calls the boatswain a wide-mouthed rascal who deserves to be hanged on the sea-shore at low water mark so that ten tides might wash over his body and take out of him all the liquor that he has been drinking

Class XIISubject Topic Summary ExecutionHistory Topic

1 1935 ACT AND WORKING OF PROVINCIAL AUTONOMYCONGREE AND OTHER MINISTERSSUB TOPIC GOVERNMENT OF INDIA ACT1935

Government of India Act 1935 This act established a lsquoFederation of Indiarsquo made of British Indian provinces and Indian states and provided for autonomy with a government responsible to the elected legislature in every provinceThis act introduced abolition of Diarchy at provinces The entire provincial administration was introduced to the responsible ministers who were controlled and removed by the provincial legislature The provincial autonomy means two things First The provincial governments were wholly responsible to the provincial legislature Secondly Provinces were free from outside control and interference in the large number of matters The act divided the powers between the centre and provinces in terms of three lists- Federal list( for centre) Provincial list (for province) and concurrent list (for both) Residuary powers were given to the viceroy In the election under the government of India Act the Congress swept the poll the mandate of the people came in favour of the congress so far as general Hindu seats were concerned The Congress did not get a single Muslim seates in Bombay CP UP Sind and BengalIn five provinces Congress had yhe clear majority In BengalNWFPAssam and Bombay Congress emerged as a single largest partyOn the other side the performance of the Muslim League was badThus the Congress formed ministers in 7 provinces out of 11 provinces Coalition ministry was also formed in two other provincesOnly BENGAL AND Punjab had non- congress ministries

1 QUESTION What was the main change introduced by the Government of India ActANSWER a) The Act gave more

autonomy to the provinces b) Diarchy was abolished at the

provincial levelsc) The Governor was the head of

the executived) There was a council of

ministers to advise him The ministers were responsible to the provincial legislatures who controlled them The legislature could also remove the ministers

e) The Governors still retained special reserve powers

2 QUESTION Why did the federal scheme introduced by the Government of India Act 1935 never come into operation

ANSWER The Federal structure of the Government of India was to be composed with the Governor General and Council of ministers The Federal legislature was to be Bicameral legislature- The council of states and the House of Assembly The ministers were to be chosen by the Governor general and they were to hold the office during his pleasure

The provinces of British India would have to join the federation but this was not compulsory for the princely states

This federation never materialised because of the lack of support from the required number of

princely statesThis act was refused and

rejected by the princes the Congress and the Muslim League

Thus both Congress and the League participated in the election of 1937 Thus the federal part was never introduced but the provincial part was put into operations

Bengali 2nd

Language

াচেরর পরাথCনা(কহিতা )

াচেরর পরাথCনা কহিতাটি কহি (ঙখ দেঘাচে4র দো আচো য কহিতায় াচেরর পতর হমায়ন কঠিন দেরাচেগ আxানত ার ঈশবর া আললার কাচেছ পরাথCনা কচেরচেছন তার পচেতরর ীন হিফহিরচেয় হিচেত এই কহিতায় ার পচেতরর ীন হিভbা দেচেয়চেছন ারার এমনহিক হিনচের ীন হিসCচেনর হিহিনমচেয় হিতহিন তার দেছচের ীন হিফচের দেপচেত দেচেয়চেছন তার দেছচের এই দেরাচেগর ন য হিতহিন হিনচেচেকই ায়ী কচেরচেছন তার হিনচের করা পাপচেকই হিতহিন ায়ী কচেরচেছন এছাা রানৈনহিতক ও আথCসামাহিক অসথার কথা তচে ধরা চেয়চেছ এই কহিতায় ার তার হিনচের পাপ কমCচেকই ায়ী কচেরচেছ ার অন যায় ভাচে দেপহি((হিকতর মাধ যচেম অপররা য কচেরচেছ আর এই অন যায় কাচের ন যই তার পহিরাচের হিপযCয় এচেসচেছ দে এক পরকার মানহিক নধন ইহিতাচেসর ার হিপতা চেয় সবাভাহিকভাচে ভাচোাসা দে মমতা দেথচেক মকত চেত পাচেরনহিন তাই হিপতা চেয় আললা া ভগাচেনর কাচেছ পতর হমায়চেনর পরানহিভbা দেচেয়চেছন ার আললা া ভগাচেনর কাচেছ াহিনচেয়চেছন তার হিনচের ীন হিসCন হিচেত হিতহিন রাী তার হিহিনমচেয় পচেতরর ীন হিফচের দেপচেত দেচেয়চেছন াচেরর হিপতসভ হিচেকর কথা এই কহিতায় ফটিচেয় দেতাা চেয়চেছ হিপতা পচেতরর হিরাহিরত মান নধচেনর কথা তচে ধরা চেয়চেছ

হিচে(4 হিকছ াইচেনর তাৎপযC১) ldquoদেকাথায় দেগ ওর সবচছয দেৌন দেকাথায় কচেরায় দেগাপন bয়ldquoউততর) াচেরর পতর হমায়ন কঠিন দেরাচেগ অসসথ তাই তার দেযৌন াহিরচেয় যাচেচছ এই দেরাচেগ তাচেক দেগাপচেন কচেরকচের াচেচছ তার সক (হিকত ধীচের ধীচের bয় চেচছ তাই হিপতা চেয় ার আললার কাচেছ হমায়চেনর পরান হিভbা দেচেয়চেছন২) ldquoাগাও (চেরর পরাচেনত পরানতচের ধসর (ন দেযর আান গানldquoউততর) াচেরর পতর হমায়ন কঠিন দেরাচেগ আxানত তাই ার আ দে(াচেক মমCাত (চেরর পচেথ পরানতচের আান গান ধবহিনত দোক দেসই আান গান আললার কাচেছ দেযন চে যায় আললা দেযন এই আহিতC শচেন পচেতরর ীন হিফহিরচেয় দেয় ৩)ldquoনাহিক এই (রীচেরর পাচেপর ীানচেত দেকানই তরারণ দেনই ভহি4চেতরldquoউততর) হমায়চেনর অসসথতার ন য ার হিনচেচেকই ায়ী কচেরচেছন কারন ার অচেনক রা য অন যায় ভাচে কচেরচেছ তাই তার এই পাপ কাচের ন য তার ঘচের আ হিপ এচেসচেছ এই অন যায় কাচের ন য তার মহিকত দেনই তাই ার আললার কাচেছ এই পাপ কাচেযCর ন য bমা পরাথM

Hindi 2ndlang

-ासी(जयशकर परसा-)

-ासी जयशकर परसा- की एक ऐसी कहानी ह जिजसम भारतीय ससकनित और राषटरीयता का सवरगजीतहोता ह इस कहानी म इरावती एक निहद कनया ह जिजस मलअचछो न मलतान की लट म पकडा और -ासी बना दि-या उस 500 दि-न -कर काशी क एक महाजन न खरी-ा दसरी -ासी निफरोजा ह वह गलाम ह निफरोजा को छडान क कतिलए अहम- को 1000 सोन क कतिसकक भजन थ जो अभी तक नही आए थ राजा साहब कठोर होत हए भी निफरोजा को निबना धनराकतिश क कतिलए उस म कर -त ह वनिफरोजा को अहम- को समझान की बात कहत हकहानी क अत म हम -खत ह निक इरा वती और जाटो क सर-ार बलराज का मिमलन होता हअहम- को यa म मार दि-या जाता ह वहा निफरोजा की परसननता की समामिध बनती ह वहा एक फल चढती ह और डीजल आती ह निफरोजा उस समामिध की आजीवन -ासी बनी रहती हलखक अपन उददशय अथात -ास परथा पर परकाश डालन और इस परथा क कारण होन वाल -ातो क दखो को दि-खान म पणता सफल हए ह

helliphellipContinue to next

Biology Reproductio Today we will discuss about vegetative Q1 Name some vegetative propagules

n in Organisms

propagation of plants The process of multiplication in which fragments of plant body function as propagule and develop into new individual is called vegetative propagation The units of such propagation are runner rhizome tuber bulb etc

and the speciesinvolvedVegetative propagules

Parts involved

Bulb StemBulbil BulbilRhizome Stem Runner Stem Tuber Stem Offset Stem Leaf buds Leaves Suckers Stem

Corns Stem stolon

Q2 State advantages of vegetative propagation

i) Rapid methodii) Sure and easy methodiii) Useful in plants that cannot

produce viable seeds or long seed dormancy

iv) Maintains purity of raceQ 3 Banana fruit is said to be parthenocarpic where as turkey is said to be parthenogenetic WhyBanana develops without fertilization from an unfertilized ovary thus is parthenocarpicIn turkey the ovum or female gamete developinto a new chick without fertilization thus isparthgenetic

Q4 Why is water hyacinth is called as a ldquoTerror of Bengalrdquo Water hyacinth can

propagatevegetatively all over the water body in a short per short period of time This resulted increased biochemicaloxygen oxygen demand of water body causing mortalityof fishes It is very difficult to get rid off them Thus known as terror of Bengal

Chemistry

Solid state GENERAL CHARACTERISTICS OF SOLID STATEIn nature the particular state of matter is governed by two opposing forces at given set of temperature and pressure These forces are intermolecular force of attraction and thermal energy If intermolecular force of attraction is high as compared to thermal energy particles remains in closest position

Intext QuestionsQ1 Classify the following solids as crystalline and amorphous Sodium chloride quartz glass quartz rubber polyvinyl chloride Teflon

A1 Crystalline

and hence very less movement in particles is observed In this case solid state is the preferred state of matter

Let us revise the general characteristics of solid

i) Fixed mass volume and shape

ii) Strong intermolecular force of attraction

iii) Least intermolecular space

iv) Fixed position of constituent particles

v) Incompressible and rigid

Q2 what type of interactions hold the molecules together in a polar molecular solid[CBSE 2010]A2 The molecules in a solid are held together by van der Waals forces The term van der Waals forces include hydrogen bonding dipole-dipole attraction and London dispersion forces All molecules experience London dispersion forces In addition polar molecules can also experience dipole-dipole interactions So the interactions that holds the molecule together in polar molecular solid are London dispersion force and dipole-dipole interactionsQ3 Write a feature that will distinguish a metallic solid from an ionic solid [CBSE 2010]A3 Metals are malleable and ductile whereas ionic solid are hard and brittle Metallic solid has typical metallic lustre But ionic solid looks dullQ4 Write a point of distinction between a metallic solid and an ionic solid other than metallic lustre [CBSE 2012]A4 Metals are malleable and ductile whereas ionic solid are hard and brittleQ5 Write a distinguish feature of metallic solid [CBSE 2010]A5 The force of attraction in

solid Sodium chloride Quartz Amorphous solid Quartz glass rubber polyvinyl chloride Teflon Q2 why glass is considered as super cooled liquidA2 Glass shows the tendency to flow at slower rate like liquid Hence they considered as super cooled liquidQ3 why the window glass of old buildings show milky appearance with timeA3 Glass is an amorphous solid Amorphous solid has the tendency to develop some crystalline character on heating Due to heating in day over the number of years glass acquires some crystalline character and show milky appearanceQ4 why the glass panes fixed to window or doors of old building become slightly thicker at bottomA4 Glass is super cooled liquid It has the tendency to flow down very slowly Due to this glass pane becomes thicker at the bottom over the timeQ5 Sodium chloride is a crystalline solid It shows the same value of refractive index along all the direction TrueFalse Give reasonA5 FalseCrystalline solid shows anisotropy in properties That is it shows different values for the given physical property in different direction All the crystalline solids show anisotropy in refractive index Therefore sodium chloride will show different values of refractive index on different directions

Q6 Crystalline solid are anisotropic in nature What does this statement means

between the constituent particles is special kind of electrostatic attraction That is the attraction of positively charged kernel with sea of delocalized electronsQ6 which group of solid is electrical conductor as well as malleable and ductile [CBSE 2013]A6 Metallic solidQ7 why graphite is good conductor of electricity although it is a network (covalent solid)A7 The exceptional property of graphite is due to its typical structure In graphite each carbon is covalently bonded with 3 atoms in same layer The fourth valence electron of each atom is free to move in between different layersThis free electron makes the graphite a good conductor of electricity

[CBSE 2011]A6 Anisotropy is defined asrdquo Difference in properties when measured along different axis or from different directionsrdquo Crystalline solid show different values of some of the physical properties like electrical resistance refractive index etcwhen measured along the different directions The anisotropy in crystalline solid arises due to the different arrangement of particles in different directions

Math Function Composition of functions Think of an industrial plant that produce bottles of cold drinks first there is the operation (or function) f that puts the cold drink inside the bottle followed by the opeartion g that close the bottle with the capThis leads to the following definitionDefinition Let f A rarr B and g B rarr C be two functions Then the composition of f and g denoted by gof is defined as the function gof A rarr C given by gof(x) = g(f (x)) forall x isinA

Definition A function f X rarr Y is defined to be invertible if there exists a function g Y rarr X such that gof = IX and fog = IY The function g is called the inverse of f and is denoted by f -1

Thus if f is invertible then f must be one-one and onto and conversely if f is one-one and onto then f must be invertible This fact significantly helps for proving a function f to be invertible by showing that f is one-one and onto specially when the actual inverse of f is not to be determined

Example 1 Let f 2 3 4 5 rarr 3 4 5 9 and g 3 4 5 9 rarr 7 11 15 be functions defined as f(2) = 3 f(3) = 4 f(4) = f(5) = 5 and g (3) = g (4) = 7 and g (5) = g (9) = 11 Find gofSolution We have gof(2) = g (f(2)) = g (3) = 7 gof(3) = g (f(3)) = g (4) = 7gof(4) = g (f(4)) = g (5) = 11 and gof(5) = g (5) = 11Example 2 Find gof and fog if f R rarr R and g R rarr R are given by f(x) = cos x and g (x) = 3x2 Show that gof ne fogSolution We have gof(x) = g(f(x))=g(cosx) = 3 (cos x)2

= 3 cos2 x Similarly fog(x)=f(g (x))= f(3x2)= cos (3x2) Note that 3cos2 x ne cos 3x2 for x = 0 Hence gof ne fogExample 3 Show that if f A rarr B and g B rarr C are onto then gof A rarr C is also ontoSolution Given an arbitrary element z isin C there exists a pre-image y of z under g such that g (y) = z since g is onto Further for y isin B there exists an element x in A with f(x) = y since f is onto Therefore gof(x) = g (f(x)) = g (y) = z showing that gof is onto Example 4 Let Y = n2 n isin N sub N Consider f N rarr Y as f(n) = n2 Show that

f is invertible Find the inverse of fSolution An arbitrary element y in Y is of the form n2 for some n isin N This implies that n =radicy This gives a function g Y rarr N defined by g (y) =radicy Nowgof (n) = g (n2)=radicn2 = n and fog (y) =f(radicy) = (radicy) 2 y which shows that gof=IN and fog= IY Hence f is invertible with f -1 = g

Political Science

Constitution of India-The Preamble

Summary

Objective of the state-To secure equality of status and of opportunity To promote fraternity among all the citizens To assure the dignity of the individuals and Unity and integrity of the nation

Justice-Justice stands for rule of law absence of arbitrariness and a system of equal rights freedom and opportunities for all in a society India seeks social economic and political justice to ensure equality to its citizens

Liberty-Liberty implies the absence of restraints or domination on the activities of an individual such as freedom from slavery serfdom imprisonment despotism etc The Preamble provides for the liberty of thought expression belief faith and worship

Equality-Equality means the absence of privileges or discrimination against any section of the society The Preamble provides for equality of status and opportunity to all the people of the country

Fraternity-The Preamble declares that fraternity has to assure two thingsmdashthe dignity of the individual and the unity and

Execution

Answer the following questions-

Short notes-1 Equality2 Fraternity3 Justice4 Liberty

Homework-Learn

integrity of the nation The word integrity has been added to the Preamble by the 42nd Constitutional Amendment (1976)

Business studies

Human resource management (chapter 1)

On the day of 1504 2020 I have discussed with you the managerial functions and procurement functions of HRM

Today weare going to discuss about the development function integration functions and maintenance function

Development functions-HRM improves the knowledge skills attitude and values of employees so that they the present and future jobs more effectively it includes

1) Development functions of HRM

a) Performance appraisal = It implies systematic evaluation of employees with respect to their performance on the job and their potential for development

b) Training =It is the process by which employees learn knowledge skills and attitudes to achieve organisational and personal goals

c) Executive development = It is the process of developing managerial talent through appropriate program

2) Integration functionsa) HRM reconcile the goals of

organisation with those of its members through integrating function

b) HRM tries to motivate employees to various financial and non financial incentives provided in job specification etc

3) Maintenance functiona) HRM promote and protect the

physical and mental health of employees by providing several types of benefits like housing medical aid etc

b) It Promote Social security measures to employees by providing provident fund pension gratuity maternity benefits

SubjectCOMMERCE

Topic

BUSINESSENVIRONMENT

Summary

Now quickly let us revise the earlier points that we have already done in the last class and let us proceed with the other topics that are there in the chapter

Firstly we will recall the internal and external factors of micro environment and then we

Execution 3 What do you mean by internal factors

in micro environmentAnswerInternal factors refer to all the factors existing within a business firm The internal factors are considered controllable because the enterprise has control over these factors

Development FunctionsPerformance AppraisalTrainingExecution Development

shall proceed in details

Meaning and list of internal and external factors

aInternal factorsInternal factors refer to all the factors existing within a business firm The internal factors are considered controllable because the enterprise has control over these factorsFor an example a company can alter its organization structure policies programmes employees physical facilities and marketing mix to suit the changes in the environmentList of internal factors areCorporate culture mission and objectives top management organizations structure company image and brand equity company resources

b External factorsExternal factors refer to those individual and groups and agencies with which a particular business organization comes into direct and frequent contact in the course of its functioningThese individuals and groups are known as STAKEHOLDERS because they have a stake (financial interest ) in the working and performance of the particular business List of external forces (stakeholders)Customers competitors investors suppliersmiddlemen (marketing intermediaries)financers publics

customers

suppliersfinancers

For an example a company can alter its organization structure policies programmes employees physical facilities and marketing mix to suit the changes in the environment

4 What do you mean by external factors in micro environment

AnswerExternal factors refer to those individual and groups and agencies with which a particular business organization comes into direct and frequent contact in the course of its functioningThese individuals and groups are known as STAKEHOLDERS because they have a stake (financial interest) in the working and performance of the particular business

3Who are stakeholdersSTAKEHOLDERS are individuals and groups who have a stake (financial interest ) in the working and performance of the particular business 4Discuss the internal factors in briefa Corporate CultureThe values beliefs and attitudes of the founders and top management of the company exercise a strong influence on what the cmpaany stands for how it does things and what it considers importantbMission and objectivesThe business philosophy and purpose of a comoany guide it prioritiesbusiness strategiesproduct market scope and development scope

cTop management structurethe composition of board of directors the degree of professionalization of management and the organizational structure of a company have important bearing on its business decisions

dPower structureThe internal power relationship between the board of directors and the chief executive is an important factor

e Company image and brand equityThe image and brand equity of the company play a significant role in raising finance forming alliance choosing dealers and suppliers launching new products entering foreign markets

5 What is Macro environmentAnswerMacro environment refers to the general

competitors

middlemen

publics

Fig STAKEHOLDERS OF A COMPANY

Apart from micro environment the other main dimension of business environment isMacro environment Macro environment refers to the general environment or remote environment within which a business firm and forces in its micro environment operateA company does not directly or regularly interact with the micro environmentTherefore macro environment is also known as indirect action EnvironmentThe macro environment forces are less controllable than the micro forces

Macro environment consists of the following components

POLITICAL AND LEGAL ENVIRONMENT

ECONOMIC SOCIAL AND ENVIRONMENT

CULTURAL

ENVIRONMENT

TECHNOLOGICAL ENVIRONMENT

Fig COMPONENTS OF MACRO ENVIRONMENT

environment or remote environment within which a business firm and forces in its micro environment operateA company does not directly or regularly interact with the micro environmentTherefore macro environment is also known as indirect action EnvironmentThe macro environment forces are less controllable than the micro forces 6 What are the components of macro environmenta Political and legal environmentb Economic environmentc Social and cultural environmentd Technological environment

Computer Science

Logic gates

Digital systems are said to be constructed by using logic gates These gates are the AND OR NOT NAND NOR EXOR and EXNOR

BUSINESS FIRM

gates The basic operations are described below with the aid of truth tables

AND gate

The AND gate is an electronic circuit that gives a high output (1) only if all its inputs are high A dot () is used to show the AND operation ie AB Bear in mind that this dot is sometimes omitted ie ABOR gate

The OR gate is an electronic circuit that gives a high output (1) if one or more of its inputs are high A plus (+) is used to show the OR operationNOT gate

The NOT gate is an electronic circuit that produces an inverted version of the input at its output It is also known as an inverter If the input variable is A the inverted output is known as NOT A This is also shown as A or A with a bar over the top as shown at the outputs The diagrams below show two ways that the NAND logic gate can be configured to produce a NOT gate It can also be done using NOR logic gates in the same way

NAND gate

This is a NOT-AND gate which is equal to an AND gate followed by a NOT gate The outputs of all NAND gates are high if any of the inputs are low The symbol is an AND gate with a small circle on the output The small circle represents inversion

NOR gate

This is a NOT-OR gate which is equal to an OR gate followed by a NOT gate The outputs of all NOR gates are low if any of the inputs are highThe symbol is an OR gate with a small circle on the output The small circle represents inversion

EXOR gate

The Exclusive-OR gate is a circuit which will give a high output if either but not both of its two inputs are high An encircled plus sign ( ) is used to show the EOR operation

EXNOR gate

The Exclusive-NOR gate circuit does the opposite to the EOR gate It will give a low output if either but not both of its two inputs are high The symbol is an EXOR gate with a small circle on the output The small circle represents inversion The NAND and NOR gates are called universal functions since with either one the AND and OR functions and NOT can be generated

Note A function in sum of products form can be implemented using NAND gates by replacing all AND and OR gates by NAND gates A function in product of sums form can be implemented using NOR gates by replacing all AND and OR gates by NOR gates

Logic gate symbols

Table 2 is a summary truth table of the inputoutput combinations for the NOT gate together with all possible inputoutput combinations for the other gate functions Also note that a truth table with n inputs has 2n rows You can compare the outputs of different gates

Logic gates representation using the Truth table

Example

A NAND gate can be used as a NOT gate using either of the following wiring configurations

Subject Eng Literature (The Tempest ndash William Shakespeare) Topic Act III Scene 3 Lines 53 to 110 (End of the scene) Date 16th April 2020 (2nd Period)

[Students should read the original play and also the paraphrase given in the school prescribed textbook]Summary Questions amp Answers

o Seeing this strange scene all are inclined to believe the tales told by travelers that there truly are ldquounicornsrdquo and ldquothe phoenixrsquo thronerdquo

o As they are about to sit down to the feast the banquet is snatched away by a harpy (Ariel disguised) A spiritrsquos voice (Arielrsquos voice) denounces Alonso Sebastian and Antonio with particular

1 ARIEL You are three men of sin whom Destiny

(Line 53-58)That hath to instrument this

lower world And what is int the never-surfeited sea

Hath caused to belch up you and on this island

Where man doth not inhabit you rsquomongst men

Being most unfit to live I have made you mad

reference to their crime in expelling Prospero from Milan They have not received any punishment for their deed earlier but the time for their punishment has arrived Upon Alonso it pronounces ldquolingering perdition worse than deathrdquo from which there is no remedy except through sincere repentance Ariel then vanishes in thunder and the shapes enter again and carry away the table

o Prospero watching invisibly is very pleased with the performance of Ariel and his (Prosperorsquos) ldquomeaner ministersrdquo All his enemies are now in his power and are in a fit of desperation He then leaves them and goes to see how Ferdinand and Miranda are getting on

o Alonso is now much humbled and penitent with the after effect of the spiritrsquos denunciation of his crimes He believes that his son is lost forever After this all disperse being stricken mad by the speech of the spirit

o Gonzalo fearing that they may do violence to themselves or to one another follows them and bid others to follow

(a) To whom does Ariel disguised as a harpy call the three sinners What game did Fate of Destiny play with

them

The three sinners called by Ariel are Alonso Sebastian and Antonio It was Destiny which had caused the ocean to cast the three sinners on the shore Though the ocean is all the time devouring whatever appears on its surface and is never satisfied with its continual swallowing of the ships and men in the present case the ocean had cast these three sinners on the shore without killing them

(b) Who had jointly been responsible for the conspiracy against Prospero What is Prosperorsquos purpose behind all this

Three men Alonso Sebastian and Antonio had jointly

been responsible for the conspiracy against Prospero They had driven out Prospero form Milan Prosperorsquos purpose is to make these three sinners realize the wrong they had done He wants them to repent for their criminal deeds because repentance leads to self-esteem(c )What does Ariel (the harpy) tell Alonso and his companions when they take out their swords to attack him

Seeing them drawing their swords Ariel (harpy) tells them that he and his companions are the instruments of destiny and that it is not possible for human beings to do them any injury He says that the swords of human beings can not injure even a minute part of his feathers Their swords are as ineffective against him and his companions as against the wind or the water

(d) Give the explanatory meanings of the following expressions in the context of the above extract

(i)Never surfeited (ii) Belch up (iii) lsquomongst men

(i) Never surfeited never led to satisfaction

(ii) Belch up cast ashore(iii) lsquomongst men in human

society2

I and my fellows (Line 60-65)

Are ministers of Fate The elementsOf whom your swords are tempered may as wellWound the loud winds or with bemocked-at stabsKill the still-closing waters as diminishOne dowl thats in my plume

IMPORTANT PASSAGES EXPLAINED

The elements

(Line 61-66)Of whom your swords are tempered may

as wellWound the loud winds or with

bemocked-at stabs

(a) Who is lsquoIrsquo Who are his lsquofellowsrdquo

lsquoIrsquo is referred to Ariel in disguise of a harpy His lsquofellowsrsquo are other spirits serving Prospero the real Duke of Milan who has acquired supernatural powers after being banished from his Dukedom Prospero has settled in this uninhabited island

(b) What are the elements that have temperrsquod the swords Why will it not work against the speaker

The swords (of Alonso and his companions) are tempered by metal (steel) which is taken out of the earth and refined by

Kill the still-closing waters as diminishOne dowl thats in my plume My fellow

ministersAre like invulnerable

In these words Ariel reminds the King and his companions of the utter futility of drawing swords against himself and his fellows Ariel drives Alonso Antonio and Sebastian the three men of sin to desperation ndash a state in which men do violence to themselves They draw swords to strike Ariel But Ariel reminds them that he and the other spirits are the ministers of destiny and nothing can wound them The steel of which their swords are made of may cut the wind or water which being divided always closes up again Even supposing that such things may be possible it is quite impossible that their swords will cut one feather in their plume They are incapable of being wounded by any sword of man Hence it is foolish on their part to attempt to strike at Ariel and his fellow-spirits

For which foul deed

(Line 72-75)The powers delaying not forgetting

haveIncensed the seas and shores yea all the

creatures Against your peace

Ariel enters like a harpy and remaining invisible tells Alonso Sebastian and Antonio that he and other harpies are the agents of Destiny appointed to carry out her decrees He tells them that their punishment for the crime against Prospero which has been so long deferred is now to fall upon them He reminds them that they had expelled Prospero from Milan and set him and his innocent child adrift on the sea and that the sea had paid them back for their sin by the shipwreck and by the calamities they have suffered He tells them that the powers above which did not forget this mean treachery but only deferred the punishment have now engaged the seas and the shores and all living beings including him and his comrades against them The very elements and supernatural agency Ariel adds have taken up the avenging of their crime against Prospero

the action of fire It may cut the wind or water which being divided always closes up again

The sword will not work against the spirits and the harpy because they are the ministers of destiny and nothing can wound them nor it will cut a single feather in their plume

(c )What is the meaning of lsquodowlrsquo in the last line

The term lsquodowlrsquo means a filament or the smallest part of a feather In this context Ariel in disguise of harpy says that their sword cannot even damage the smallest filament of their (Arielrsquos and other spirits) feathers as they are incapable of being wounded by any sword of man

(d) What does the speaker remind the listeners about

Ariel in disguise of harpy reminds Alonso the King of Naples Sebastian Alonsorsquos brother and Antonio the present Duke of Milan and the treacherous brother of Prospero as they being three men of sin He even reminds them that their punishment for their crime against Prospero which has been so long deferred now falls upon them He reminds them that they have expelled Prospero from Milan and has set him along with his innocent infant daughter adrift on the sea So the sea has paid them back for their sin by their shipwreck and the calamities they have suffered since then The harpy rebukes Alonso of his sin that has incensed the Gods and has deprived him of his son as a punishment

(e) How do they respond

When Ariel in disguise of a harpy reminds Alonso Sebastian and Antonio of their past misdeeds and sin Alonso has a look of terror and confusion in his eyes He utters the words of sincere repentance wrung out of his conscience-stricken heart It appears to him that all the elements of nature the sea-waves the wind and the thunder proclaiming a loud voice in the name of Prospero and the crime Alonso has committed against him They are calling upon him to repent There is a deep storm raging in Alonsorsquos breast and the echoes of that storm are ringing in his ears like a clear note of wind-instrument A note of denunciation of Alonsorsquos crime leaves him much humbled and penitent and confirms his belief that his son is lost forever But Sebastian and Antonio shows some courage instead of repentance They wish to kill the spirits or devils if it appears

3

Of my instruction hast thou nothing bated (Line 85-93)

In what thou hast to say So with good life

And observation strange my meaner ministers

Their several kinds have done My high charms work

And these mine enemies are all knit upIn their distractions They now are in my

powerAnd in these fits I leave them while I visitYoung Ferdinand whom they suppose is

drownedAnd his and mine loved darling

Methought the billows spoke and (Line 96-99)

told me of itThe winds did sing it to me and the

thunderThat deep and dreadful organ-pipe

pronouncedThe name of Prosper It did bass my

trespass

These are the words of contrition coming from Alonso Ariel has driven him to a deep repentance for conspiring with Antonio against Prospero He now feels a sincere remorse It appears to him that all the elements of nature the sea-waves the wind and the thunder proclaimed with a loud voice the name of Prospero and the crime Alonso had committed against him They are calling upon him to repent There is a deep storm raging in Alonsorsquos breast and the echoes of that storm are ringing in his ears like the clear note of a wind-instrument

Comment These are the words of sincere repentance wrung out of the conscience-stricken heart of Alonso Alonso who is the lesser villain is the first to give way to remorse under the effect of Arielrsquos speech The words of Ariel seem to him to be the voice of conscience speaking to him He is driven to desperation a state in which he might do violence to his life

(a) Identify the speaker State the context

Prospero the ruler of the island is the speaker The famous banquet scene has been enacted very well Ariel and his junior spirits have played their roles excellently Prospero is glad to say words of praise for them(b) In what way the speakerrsquos instructions have been carried out

According to Prosperorsquos instructions a banquet was presented before the King of Naples and his companions when they were tired and hungry Just when they were preparing to eat the feast the banquet was suddenly removed by exercising supernatural powers All this was done by Ariel Prosperorsquos chief assistant and a powerful spirit

Ariel not only made the feast disappear but also delivered his speech blaming the King and his two companions for their past wicked deeds He warned them to repent for their misdeeds or suffer forever on that uninhabited island

(c) Who are referred to as lsquomeaner ministersrsquo What have they done

Prospero refers as lsquomeaner ministersrsquo to his other lesser spirits who were assisting Ariel in presenting a scene before the kingrsquos party They entered the scene to the accompaniment of music They assumed several strange shapes and brought in a banquet Then they danced about it with gentle actions of salutations thus inviting the King and others to eat the feast

These spirits play their role again when Ariel in the shape of a harpy quits the scene These shapes enter again and dancing with mocking gestures carry away the table

(d) Who are the speakerrsquos enemies What has happened to them

King of Naples Alonso his brother Sebastian and the present Duke of Milan Antonio (Prosperorsquos own brother) are Prosperorsquos enemies With the turn of events they have all been washed ashore on the island which is ruled by Prospero the great magician Actually this happened after the shipwreck caused by a storm which was raised by Prospero with the purpose of bringing these people to his island Prosperorsquos spirits have already confused and terrified these enemies and they are under Prosperorsquos control He can treat them as he likes

(e) What does he say about Ferdinand Explain what is meant by ldquohellip his and mine darlingrdquo

Prospero knows that Alonsorsquos son prince Ferdinand is alive though his father thinks that the prince has been drowned

Prospero refers to his daughter Miranda who is dear to him She is also very dear to Prince Ferdinand who has fallen in love with her They are waiting to be married soon for which they have received Prosperorsquos consent

4

ALONSO O it is monstrous monstrous (Line 95-102)

Methought the billows spoke and told me of it

The winds did sing it to me and the thunderThat deep and dreadful organ-

pipe pronouncedThe name of Prosper It did bass

my trespassTherefore my son ithrsquo ooze is

bedded andIll seek him deeper than eer

plummet soundedAnd with him there lie mudded

(a) In what way does Alonso express his horror when his conscience is awakened by Arielrsquos words

When Alonsorsquos conscience is awakened by Arielrsquos words he expresses his horror at what he has heard He gets the feeling that the waves of the ocean the wind and the loud thunder have spoken to him and uttered the name of Prospero Because of being reminded of his crime in a very loud and rough voice he comes to realize that he has lost his son for his past misdeeds

(b) What does Alonso imagine about his son What does Alonso want to do in his desperate state

Alonso imagines that his son is lying in the mud at the bottom of the sea He feels desperate that he wants to drown himself in the ocean deeper than the plumb-line has ever gone He wants to lie with his son at the bottom of the sea

(c) How do Sebastian and Antonio want to face the evil spirits

Sebastian says that he is not at all afraid of what the harpy has said and that he is prepared to fight any number of such monsters if they appear before him only one at a time Antonio says that he would support Sebastian in the fight against the fiendsyyy

(d) Why does Gonzalo ask Adrian to follow the three men

Gonzalo tells Adrian that all the three men namely Alonso Sebastian and Antonio are in a wild and reckless mood The thought of the heinous crime of which they are guilty has begun to torment their minds So he asks Adrian to follow those three men without loss of time and prevent them from doing anything which the turmoil in their minds might lead them to do

(e) What opinion do you form of Alonso from the above extract

Alonso who is the lesser villain is the first to give way to remorse under the effect of Arielrsquos speech The words of Ariel seem to him to be the voice of conscience speaking to him He is driven to desperation a state in which he might do violence to his life

Subject =Accounts

Ac-12 15420 topic-pL Appropriation ac

PROFIT AND LOSS APPROPRIATION ACCOUNT

MEANING AND PREPARATIONProfit and Loss Appropriation Account is merely an extension of the Profit and Loss Account of the firm The profit of the firm has to be distributed amongst the partners in their respective profit sharing ratio But before its distribution it needs to be adjusted All Adjustments like partnerrsquos salary partnerrsquos commission interest on capital interest on drawings etc are made in this account These adjustments will reduce the amount of profit for distribution This adjusted profit will be distributed amongst the partners in their profit sharing ratio To prepare it at first the balance of Profit and Loss Account is transferred to this account The journal entries for the preparation of Profit and Loss Appropriation Account are given below

1 for transfer of the balance of Profit and Loss Account to Profit and Loss Appropriation Account

(a) In case of Net Profit

Profit and Loss Ac helliphelliphelliphelliphellipDrTo Profit and Loss Appropriation Ac(Net Profit transferred to Profit and Loss Appropriation Ac)

(b)In case of Net Loss

Profit and Loss Appropriation Achelliphelliphellip DrTo Profit and Loss Ac(Net Loss transferred to Profit and Loss Appropriation Ac)

2 for Interest on Capital

For transferring on Interest on CapitalProfit and Loss Appropriation Achelliphelliphellip DrTo Interest on Capital Ac(Interest on capital transferred to Profit amp Loss Appropriation Ac)

3 for Interest on Drawings

For transferring Interest on Drawings Interest on Drawings Achelliphelliphelliphelliphelliphellip DrTo Profit and Loss Appropriation Ac(Interest on drawing transferred to Profit amp Loss Appropriation Ac)

4 For Partnerrsquos SalaryFor transfer of partnerrsquos SalaryProfit and Loss Appropriation Achelliphellip DrTo Salary Ac(Salary transferred to profit amp Loss Appropriation Ac)

5 For Partnerrsquos CommissionFor transferring commissionProfit and Loss Appropriation Achelliphelliphellip DrTo Commission Ac(Commission transferred to Profit and Loss Appropriation Ac)

6 For Transfer of agreed amount to General ReserveProfit and Loss Appropriation Ac helliphellipDrTo General Reserve Ac(Transfer to General Reserve)

7 for share of Profit or Loss appropriation(a) If ProfitProfit and Loss Appropriation Achelliphellip DrTo Partnerrsquos CapitalCurrent Ac(Profit transferred to capitalcurrent Ac)(b) If LossPartnerrsquos Capital Current Achelliphelliphelliphellip DrTo Profit and Loss Appropriation Ac(Loss transferred to capitalcurrent Ac)

THE FORMAT OF PROFIT AND LOSS APPROPRIATION

Profit and Loss Appropriation Account for the year endedhelliphelliphelliphellip

Particulars Amount Particulars Amount

To PL Ac (loss) By pL Ac (profit)

To Interest on capital BY Interest on drawings

To partner`s commission by Partner`s capital Ac ( loss)

To Partner`s salary To Interest on partner`s loan To General Reserve To Partner`s Capital AC (Profit)

Subject= Economics

MOVEMENT ALONG THE DEMAND CURVE (CHANGE IN QUANTITY DEMANDED)In law of demand you have already studied the inverse relationship between price and quantity demanded When quantity demanded of a commodity changes due to change in its price keeping other factors constant it is called change in quantity demanded It is graphically expressed as a movement along the same demand curve There can be either a downward movement or an upward movement along the same demand curve Upward movement along the same demand curve is called contraction of demand or decrease in quantity demanded and downward movement along the same demand curve is known as expansion of demand or increase in quantity demanded

Extention of demandd

price (rs)p A

B Extentionp1 d

Q Q1

Quantity demanded ( in units)

Contraction of demandd

p2 Ccontraction

p APrice (Rs)

d

Q2 Q

Quantity demanded (in units)

Explanation of movement of demand A fall in price from OP to OP1 leads to increase in quantity demanded from OQ to OQ1 (expansion of demand) resulting in a downward movement from point A to point B along the same demand curve DD When Price rises from OP to OP2 quantity demanded falls from OQ to OQ2 (contraction of demand) leading to an upward movement from point A to point C along the same demand curve DD

  • Activity Series of Metals
    • Drawbacks of Rutherfordrsquos model of atom
      • Electromagnetic radiations
      • Properties of electromagnetic radiations
      • Characteristics of electromagnetic radiations
        • Plancks Quantum Theory-
        • Photoelectric effect
          • Intext Questions
            • Logic gates
            • Digital systems are said to be constructed by using logic gates These gates are the AND OR NOT NAND NOR EXOR and EXNOR gates The basic operations are described below with the aid of truth tables
            • AND gate
            • Example
Page 35:  · Web viewSubject. Topic. Summary. Execution. English 1 . Chapter 1 naming words . Page 8. Write the names of these pictures:- Person:-1. father. 2.Firefighter 3.doctor 4 ...

consumed In this casethe utility obtained from each mango as it is consumed as the MU of that mango It is also defined as the addition made to the total utility when an additional unit is consumed Often economists tend to subdivide utility into an imaginary unit called UTIL

Note As a consumer increases the consumption of a good over period of time the total utility or total satisfaction derived from it increases to appoint and thereafter it decreasesHowever as the consumer keeps on consuming the good the marginal utility or the additional utility derived from it decreases

SubjectBusiness studies

Topic

BUSINESSENVIRONMENT

Summary

Now quickly let us revise the earlier points that we have already done in the last class and let us proceed with the other topics that are there in the chapter

Firstly we will recall the internal and external factors of micro environment and then we shall proceed in details

Meaning and list of internal and external factors

aInternal factorsInternal factors refer to all the factors existing within a business firm The internal factors are considered controllable because the enterprise has control over these factorsFor an example a company can alter its organization structure policies programmes employees physical facilities and marketing mix to suit the changes in the environmentList of internal factors areCorporate culture mission and objectives top management organizations structure company image and brand equity company resources

b External factorsExternal factors refer to those individual and groups and agencies with which a particular business organization comes into direct and frequent contact in the course of its functioningThese individuals and groups are known as STAKEHOLDERS because they have a stake (financial interest ) in the working and performance of the particular business List of external forces (stakeholders)Customers competitors investors suppliersmiddlemen (marketing intermediaries)

Execution 1 What do you mean by internal

factors in micro environmentAnswerInternal factors refer to all the factors existing within a business firm The internal factors are considered controllable because the enterprise has control over these factorsFor an example a company can alter its organization structure policies programmes employees physical facilities and marketing mix to suit the changes in the environment

2 What do you mean by external factors in micro environment

AnswerExternal factors refer to those individual and groups and agencies with which a particular business organization comes into direct and frequent contact in the course of its functioningThese individuals and groups are known as STAKEHOLDERS because they have a stake (financial interest) in the working and performance of the particular business

3Who are stakeholdersSTAKEHOLDERS are individuals and groups who have a stake (financial interest ) in the working and performance of the particular business 4Discuss the internal factors in briefa Corporate CultureThe values beliefs and attitudes of the founders and top management of the company exercise

financers publics

customers

suppliersfinancers

competitors

middlemen

publics

Fig STAKEHOLDERS OF A COMPANY

Apart from micro environment the other main dimension of business environment isMacro environment Macro environment refers to the general environment or remote environment within which a business firm and forces in its micro environment operateA company does not directly or regularly interact with the micro environmentTherefore macro environment is also known as indirect action EnvironmentThe macro environment forces are less controllable than the micro forces

Macro environment consists of the following components

POLITICAL AND LEGAL ENVIRONMENT

ECONOMIC SOCIAL AND ENVIRONMENT

CULTURAL

ENVIRONMENT

TECHNOLOGICAL ENVIRONMENT

a strong influence on what the cmpaany stands for how it does things and what it considers importantbMission and objectivesThe business philosophy and purpose of a comoany guide it prioritiesbusiness strategiesproduct market scope and development scope

cTop management structurethe composition of board of directors the degree of professionalization of management and the organizational structure of a company have important bearing on its business decisions

dPower structureThe internal power relationship between the board of directors and the chief executive is an important factor

eCompany image and brand equityThe image and brand equity of the company play a significant role in raising finance forming alliance choosing dealers and suppliers launching new products entering foreign markets

5 What is Macro environmentAnswerMacro environment refers to the general environment or remote environment within which a business firm and forces in its micro environment operateA company does not directly or regularly interact with the micro environmentTherefore macro environment is also known as indirect action EnvironmentThe macro environment forces are less controllable than the micro forces 6 What are the components of macro environmenta Political and legal environmentb Economic environmentc Social and cultural environmentd Technological environment

BUSINESS FIRM

Fig COMPONENTS OF MACRO ENVIRONMENTPolitical science

Introduction to political science

Comparative politics and itrsquos scope Comparative politics is the second major dimension of political scienceIt is also a very vast area of study and a very large number of political scientists even treat it as an autonomous area of study within the board ambit of political scienceScope of comparative politics-

1 All political structures -Comparative politics includes the study of all structures formalnon formal governmental and extra governmental which are directly or indirectly involved in politics in all the countries of the world

2 Functional studies- Comparative politics seeks to study politics less from the point of view of the legal institutions in terms of their powers and move from the point of view of their functions which constitute the political process and their actual Operation in the environment

3 Study of political behaviour- Another important part of its scope is the study of the actual behaviour of the people in the process of politics

4 Study of similarities and differences- comparative politics also undertakesan analysis of the similarities and differences among political process and functions

5 Study of all political systems -comparative politics seeks to analyse the actual behaviour and performance of all political systems western as well as non western

6 Study of the environment and infrastructure of politics-The study of politics demands a study of the psychological sociological economic and anthropological environment in fact the social environment as a whole in which each political system operates

7 Study of political culture- political culture is composed of attitudesbeliefs emotions and values of a society that relate to the political system or politics

8 Study of political participation- Political participation is a universal processThe only difference is that while in some states it is limited in others it is wider

9 Study of political process- political

Answer the following questions-

What is comparative politics

What are the scope of comparative politics

Homework- learn

processes like decision makingpolicy making judicial process leadership recruitment process and others are always at work in all political systems

The scope of comparative politics is very comprehensive It includes everything that falls within the area of political activity and political process

History CAMBRIDGE VIEW ABOUT

THE PARTITION

AND REFUTATION

OF CAMBRIDGE

VIEW

Cambridge view about the Partition The Cambridge school of historians have interpreted that opposition to partition scheme was made entirely by the elitist groups They hold the view that Lord Curzon planned to partition the Bengal for administrative purposeREFUTATION OFCAMBRIDGE VIEW The Rationalist historians have rejected the interpretations of the Cambridge School of historians on various grounds

1 QUESTION State different views of historians regarding Partition of Bengal

ANSWER Cambridge historians believed that Lord Curzon partitioned Bengal for administrative reasons only and not for the political motive The Middle class elitist group protested because of their petty interest The Hindu zamindars protested as they have to spend more money for managing their estatesThe lawyers of Calcutta High court feared to lose their clientBut according to the nationalist Historians was-

2- The ultimate object of Lord Curzon was to crush the unity of Bengal politicians

3- If Bengal becomes a separate province Bengali speaking 16 million people of western part would become minority under Hindi speaking people of Bihar and Oriya speaking people of Orissa

4- The bureaucrats expected that the protest movement would die down quickly

5- Lord Curzon used the Muslim community in his political game

6- Idealism had great contribution in the protest against partition

7- The people of the every section of society were affected by the partition of Bengal

Computer Science

Numbers Convertion of dcimal number to octal numberThe decimal numeral system is the standard system for denoting integer and non-integer numbers It is the extension to non-integer numbers of the Hindu-Arabic numeral system For writing numbers the decimal system uses ten decimal digits a decimal mark and for negative numbers a minus sign - The decimal digits are 0 1 2 3 4 5 6 7 8 9 the decimal separator is the dot in many countries

The octal numeral system or oct for short is the base-8 number system and uses the digits 0 to 7 Octal is sometimes used in computing instead of hexadecimal perhaps most often in modern times in conjunction with file

permissions under Unix systems It has the advantage of not requiring any extra symbols as digits It is also used for digital displays

Follow these steps to convert a decimal number into octal form

1 Divide the decimal number by 82 Get the integer quotient for the next iteration (if the number will not divide equally by 8 then round down the

result to the nearest whole number)3 Keep a note of the remainder it should be between 0 and 74 Repeat the steps until the quotient is equal to 05 Write out all the remainders from bottom to top This is the solution

For example if the given decimal number is 8453

Division Quotient Remainder

8453 8 1056 5

1056 8 132 0

132 8 16 4

16 8 2 0

2 8 0 2

Then the octal solution is 20405

Subject Eng Literature (The Tempest ndash William Shakespeare) Topic Act I Scene 1 Lines 33 to 67 (End of scene) Date 16th April 2020 (4th Period)

[Students should read the original play and also the paraphrase given in the school prescribed textbook]Summary Questions amp Answers

[SUMMARY OF THE ENTIRE SCENE]

o The play starts with the scene of a severe storm at sea Alonso (King of Naples) Sebastian (Alonsorsquos brother) Ferdinand (Alonsorsquos son) Gonzalo Antonio (the usurping Duke of Milan) are in a ship in the midst of the storm

o The mariners are trying their best to control the vessel from running aground and are totally following the orders of their Master the Boatswain They have scant success

o The mariners become extremely unhappy and annoyed when most of the passengers arrive on the deck thereby hampering their effort to save the ship There is serious confrontation between them and the passengers who are part of the Kingrsquos entourage

o The mariners could not save the ship

SUMMING-UP

(i) Vivid description of the scene which gives a realistic description of terror and confusion of a tropical storm

(ii) Shows Shakespearersquos accuracy of knowledge in describing the naval operations and also matters of seamanship

(1) GONZALO Ill warrant him for drowning (L 45-57)

though the ship were no stronger than a nutshell and as leaky as an unstanched

wenchBOATSWAIN Lay her a-hold a-hold Set her two courses Off to

sea again lay her offMARINERS All lost To prayers to prayers All lostBOATSWAIN What must our mouths be coldGONZALO The king and prince at prayers Lets assist them

For our case is theirsSEBASTIAN Im out of patienceANTONIO We are merely cheated of our lives by drunkards

This wide-chopped rascal - would thou mightst lie drowning the washing of ten tides

(a) What does Antonio say at the insolent manners of the boatswain just before the given passage

Being irritated at the insolent manners of the boatswain just before the given extract Antonio the Duke of Milan calls him a worthless dog son of a woman without any morals an arrogant and disrespectful noisemaker He says that the boatswain deserved to be hanged(b) What statement does Gonzalo repeat about the boatswain

Gonzalo shows his faith that the boatswain is not destined to die by drowning He is destined to be hanged and nothing can alter this decree of destiny He says that even if the ship was as frail as a nutshell the boatswain could not be drowned for his destiny was to be hanged(c) What do the passengers do when they have lost all hope of their survival

When the passengers have lost all hope of survival they take

(iii) The opening scene justifies the title ndash The Tempest

UNANSWERED QUESTIONS

(i) The King always travels with his entire fleet including his soldiers Where were the other ships

(ii) Why was the ship in that area Where was it coming from or going where

(iii) The ship broke apart What happened to those who were in the ship

(We shall get the answer to the above questions as the play progresses)

leave of life with fervent prayers The mariners take their last hearty drink and are ready for death(d) What blame does Antonio put upon the mariners and the boatswain Antonio rebukes the mariners that these drunkards have brought them to the present crisis by neglecting their duties He blames them saying that they are going to lose their lives entirely for the negligence of the boatswain and his fellows(e) What does Antonio say while cursing the boatswain

Antonio gives vent to his wrath upon the boatswain in particular He calls the boatswain a wide-mouthed rascal who deserves to be hanged on the sea-shore at low water mark so that ten tides might wash over his body and take out of him all the liquor that he has been drinking

Class XIISubject Topic Summary ExecutionHistory Topic

1 1935 ACT AND WORKING OF PROVINCIAL AUTONOMYCONGREE AND OTHER MINISTERSSUB TOPIC GOVERNMENT OF INDIA ACT1935

Government of India Act 1935 This act established a lsquoFederation of Indiarsquo made of British Indian provinces and Indian states and provided for autonomy with a government responsible to the elected legislature in every provinceThis act introduced abolition of Diarchy at provinces The entire provincial administration was introduced to the responsible ministers who were controlled and removed by the provincial legislature The provincial autonomy means two things First The provincial governments were wholly responsible to the provincial legislature Secondly Provinces were free from outside control and interference in the large number of matters The act divided the powers between the centre and provinces in terms of three lists- Federal list( for centre) Provincial list (for province) and concurrent list (for both) Residuary powers were given to the viceroy In the election under the government of India Act the Congress swept the poll the mandate of the people came in favour of the congress so far as general Hindu seats were concerned The Congress did not get a single Muslim seates in Bombay CP UP Sind and BengalIn five provinces Congress had yhe clear majority In BengalNWFPAssam and Bombay Congress emerged as a single largest partyOn the other side the performance of the Muslim League was badThus the Congress formed ministers in 7 provinces out of 11 provinces Coalition ministry was also formed in two other provincesOnly BENGAL AND Punjab had non- congress ministries

1 QUESTION What was the main change introduced by the Government of India ActANSWER a) The Act gave more

autonomy to the provinces b) Diarchy was abolished at the

provincial levelsc) The Governor was the head of

the executived) There was a council of

ministers to advise him The ministers were responsible to the provincial legislatures who controlled them The legislature could also remove the ministers

e) The Governors still retained special reserve powers

2 QUESTION Why did the federal scheme introduced by the Government of India Act 1935 never come into operation

ANSWER The Federal structure of the Government of India was to be composed with the Governor General and Council of ministers The Federal legislature was to be Bicameral legislature- The council of states and the House of Assembly The ministers were to be chosen by the Governor general and they were to hold the office during his pleasure

The provinces of British India would have to join the federation but this was not compulsory for the princely states

This federation never materialised because of the lack of support from the required number of

princely statesThis act was refused and

rejected by the princes the Congress and the Muslim League

Thus both Congress and the League participated in the election of 1937 Thus the federal part was never introduced but the provincial part was put into operations

Bengali 2nd

Language

াচেরর পরাথCনা(কহিতা )

াচেরর পরাথCনা কহিতাটি কহি (ঙখ দেঘাচে4র দো আচো য কহিতায় াচেরর পতর হমায়ন কঠিন দেরাচেগ আxানত ার ঈশবর া আললার কাচেছ পরাথCনা কচেরচেছন তার পচেতরর ীন হিফহিরচেয় হিচেত এই কহিতায় ার পচেতরর ীন হিভbা দেচেয়চেছন ারার এমনহিক হিনচের ীন হিসCচেনর হিহিনমচেয় হিতহিন তার দেছচের ীন হিফচের দেপচেত দেচেয়চেছন তার দেছচের এই দেরাচেগর ন য হিতহিন হিনচেচেকই ায়ী কচেরচেছন তার হিনচের করা পাপচেকই হিতহিন ায়ী কচেরচেছন এছাা রানৈনহিতক ও আথCসামাহিক অসথার কথা তচে ধরা চেয়চেছ এই কহিতায় ার তার হিনচের পাপ কমCচেকই ায়ী কচেরচেছ ার অন যায় ভাচে দেপহি((হিকতর মাধ যচেম অপররা য কচেরচেছ আর এই অন যায় কাচের ন যই তার পহিরাচের হিপযCয় এচেসচেছ দে এক পরকার মানহিক নধন ইহিতাচেসর ার হিপতা চেয় সবাভাহিকভাচে ভাচোাসা দে মমতা দেথচেক মকত চেত পাচেরনহিন তাই হিপতা চেয় আললা া ভগাচেনর কাচেছ পতর হমায়চেনর পরানহিভbা দেচেয়চেছন ার আললা া ভগাচেনর কাচেছ াহিনচেয়চেছন তার হিনচের ীন হিসCন হিচেত হিতহিন রাী তার হিহিনমচেয় পচেতরর ীন হিফচের দেপচেত দেচেয়চেছন াচেরর হিপতসভ হিচেকর কথা এই কহিতায় ফটিচেয় দেতাা চেয়চেছ হিপতা পচেতরর হিরাহিরত মান নধচেনর কথা তচে ধরা চেয়চেছ

হিচে(4 হিকছ াইচেনর তাৎপযC১) ldquoদেকাথায় দেগ ওর সবচছয দেৌন দেকাথায় কচেরায় দেগাপন bয়ldquoউততর) াচেরর পতর হমায়ন কঠিন দেরাচেগ অসসথ তাই তার দেযৌন াহিরচেয় যাচেচছ এই দেরাচেগ তাচেক দেগাপচেন কচেরকচের াচেচছ তার সক (হিকত ধীচের ধীচের bয় চেচছ তাই হিপতা চেয় ার আললার কাচেছ হমায়চেনর পরান হিভbা দেচেয়চেছন২) ldquoাগাও (চেরর পরাচেনত পরানতচের ধসর (ন দেযর আান গানldquoউততর) াচেরর পতর হমায়ন কঠিন দেরাচেগ আxানত তাই ার আ দে(াচেক মমCাত (চেরর পচেথ পরানতচের আান গান ধবহিনত দোক দেসই আান গান আললার কাচেছ দেযন চে যায় আললা দেযন এই আহিতC শচেন পচেতরর ীন হিফহিরচেয় দেয় ৩)ldquoনাহিক এই (রীচেরর পাচেপর ীানচেত দেকানই তরারণ দেনই ভহি4চেতরldquoউততর) হমায়চেনর অসসথতার ন য ার হিনচেচেকই ায়ী কচেরচেছন কারন ার অচেনক রা য অন যায় ভাচে কচেরচেছ তাই তার এই পাপ কাচের ন য তার ঘচের আ হিপ এচেসচেছ এই অন যায় কাচের ন য তার মহিকত দেনই তাই ার আললার কাচেছ এই পাপ কাচেযCর ন য bমা পরাথM

Hindi 2ndlang

-ासी(जयशकर परसा-)

-ासी जयशकर परसा- की एक ऐसी कहानी ह जिजसम भारतीय ससकनित और राषटरीयता का सवरगजीतहोता ह इस कहानी म इरावती एक निहद कनया ह जिजस मलअचछो न मलतान की लट म पकडा और -ासी बना दि-या उस 500 दि-न -कर काशी क एक महाजन न खरी-ा दसरी -ासी निफरोजा ह वह गलाम ह निफरोजा को छडान क कतिलए अहम- को 1000 सोन क कतिसकक भजन थ जो अभी तक नही आए थ राजा साहब कठोर होत हए भी निफरोजा को निबना धनराकतिश क कतिलए उस म कर -त ह वनिफरोजा को अहम- को समझान की बात कहत हकहानी क अत म हम -खत ह निक इरा वती और जाटो क सर-ार बलराज का मिमलन होता हअहम- को यa म मार दि-या जाता ह वहा निफरोजा की परसननता की समामिध बनती ह वहा एक फल चढती ह और डीजल आती ह निफरोजा उस समामिध की आजीवन -ासी बनी रहती हलखक अपन उददशय अथात -ास परथा पर परकाश डालन और इस परथा क कारण होन वाल -ातो क दखो को दि-खान म पणता सफल हए ह

helliphellipContinue to next

Biology Reproductio Today we will discuss about vegetative Q1 Name some vegetative propagules

n in Organisms

propagation of plants The process of multiplication in which fragments of plant body function as propagule and develop into new individual is called vegetative propagation The units of such propagation are runner rhizome tuber bulb etc

and the speciesinvolvedVegetative propagules

Parts involved

Bulb StemBulbil BulbilRhizome Stem Runner Stem Tuber Stem Offset Stem Leaf buds Leaves Suckers Stem

Corns Stem stolon

Q2 State advantages of vegetative propagation

i) Rapid methodii) Sure and easy methodiii) Useful in plants that cannot

produce viable seeds or long seed dormancy

iv) Maintains purity of raceQ 3 Banana fruit is said to be parthenocarpic where as turkey is said to be parthenogenetic WhyBanana develops without fertilization from an unfertilized ovary thus is parthenocarpicIn turkey the ovum or female gamete developinto a new chick without fertilization thus isparthgenetic

Q4 Why is water hyacinth is called as a ldquoTerror of Bengalrdquo Water hyacinth can

propagatevegetatively all over the water body in a short per short period of time This resulted increased biochemicaloxygen oxygen demand of water body causing mortalityof fishes It is very difficult to get rid off them Thus known as terror of Bengal

Chemistry

Solid state GENERAL CHARACTERISTICS OF SOLID STATEIn nature the particular state of matter is governed by two opposing forces at given set of temperature and pressure These forces are intermolecular force of attraction and thermal energy If intermolecular force of attraction is high as compared to thermal energy particles remains in closest position

Intext QuestionsQ1 Classify the following solids as crystalline and amorphous Sodium chloride quartz glass quartz rubber polyvinyl chloride Teflon

A1 Crystalline

and hence very less movement in particles is observed In this case solid state is the preferred state of matter

Let us revise the general characteristics of solid

i) Fixed mass volume and shape

ii) Strong intermolecular force of attraction

iii) Least intermolecular space

iv) Fixed position of constituent particles

v) Incompressible and rigid

Q2 what type of interactions hold the molecules together in a polar molecular solid[CBSE 2010]A2 The molecules in a solid are held together by van der Waals forces The term van der Waals forces include hydrogen bonding dipole-dipole attraction and London dispersion forces All molecules experience London dispersion forces In addition polar molecules can also experience dipole-dipole interactions So the interactions that holds the molecule together in polar molecular solid are London dispersion force and dipole-dipole interactionsQ3 Write a feature that will distinguish a metallic solid from an ionic solid [CBSE 2010]A3 Metals are malleable and ductile whereas ionic solid are hard and brittle Metallic solid has typical metallic lustre But ionic solid looks dullQ4 Write a point of distinction between a metallic solid and an ionic solid other than metallic lustre [CBSE 2012]A4 Metals are malleable and ductile whereas ionic solid are hard and brittleQ5 Write a distinguish feature of metallic solid [CBSE 2010]A5 The force of attraction in

solid Sodium chloride Quartz Amorphous solid Quartz glass rubber polyvinyl chloride Teflon Q2 why glass is considered as super cooled liquidA2 Glass shows the tendency to flow at slower rate like liquid Hence they considered as super cooled liquidQ3 why the window glass of old buildings show milky appearance with timeA3 Glass is an amorphous solid Amorphous solid has the tendency to develop some crystalline character on heating Due to heating in day over the number of years glass acquires some crystalline character and show milky appearanceQ4 why the glass panes fixed to window or doors of old building become slightly thicker at bottomA4 Glass is super cooled liquid It has the tendency to flow down very slowly Due to this glass pane becomes thicker at the bottom over the timeQ5 Sodium chloride is a crystalline solid It shows the same value of refractive index along all the direction TrueFalse Give reasonA5 FalseCrystalline solid shows anisotropy in properties That is it shows different values for the given physical property in different direction All the crystalline solids show anisotropy in refractive index Therefore sodium chloride will show different values of refractive index on different directions

Q6 Crystalline solid are anisotropic in nature What does this statement means

between the constituent particles is special kind of electrostatic attraction That is the attraction of positively charged kernel with sea of delocalized electronsQ6 which group of solid is electrical conductor as well as malleable and ductile [CBSE 2013]A6 Metallic solidQ7 why graphite is good conductor of electricity although it is a network (covalent solid)A7 The exceptional property of graphite is due to its typical structure In graphite each carbon is covalently bonded with 3 atoms in same layer The fourth valence electron of each atom is free to move in between different layersThis free electron makes the graphite a good conductor of electricity

[CBSE 2011]A6 Anisotropy is defined asrdquo Difference in properties when measured along different axis or from different directionsrdquo Crystalline solid show different values of some of the physical properties like electrical resistance refractive index etcwhen measured along the different directions The anisotropy in crystalline solid arises due to the different arrangement of particles in different directions

Math Function Composition of functions Think of an industrial plant that produce bottles of cold drinks first there is the operation (or function) f that puts the cold drink inside the bottle followed by the opeartion g that close the bottle with the capThis leads to the following definitionDefinition Let f A rarr B and g B rarr C be two functions Then the composition of f and g denoted by gof is defined as the function gof A rarr C given by gof(x) = g(f (x)) forall x isinA

Definition A function f X rarr Y is defined to be invertible if there exists a function g Y rarr X such that gof = IX and fog = IY The function g is called the inverse of f and is denoted by f -1

Thus if f is invertible then f must be one-one and onto and conversely if f is one-one and onto then f must be invertible This fact significantly helps for proving a function f to be invertible by showing that f is one-one and onto specially when the actual inverse of f is not to be determined

Example 1 Let f 2 3 4 5 rarr 3 4 5 9 and g 3 4 5 9 rarr 7 11 15 be functions defined as f(2) = 3 f(3) = 4 f(4) = f(5) = 5 and g (3) = g (4) = 7 and g (5) = g (9) = 11 Find gofSolution We have gof(2) = g (f(2)) = g (3) = 7 gof(3) = g (f(3)) = g (4) = 7gof(4) = g (f(4)) = g (5) = 11 and gof(5) = g (5) = 11Example 2 Find gof and fog if f R rarr R and g R rarr R are given by f(x) = cos x and g (x) = 3x2 Show that gof ne fogSolution We have gof(x) = g(f(x))=g(cosx) = 3 (cos x)2

= 3 cos2 x Similarly fog(x)=f(g (x))= f(3x2)= cos (3x2) Note that 3cos2 x ne cos 3x2 for x = 0 Hence gof ne fogExample 3 Show that if f A rarr B and g B rarr C are onto then gof A rarr C is also ontoSolution Given an arbitrary element z isin C there exists a pre-image y of z under g such that g (y) = z since g is onto Further for y isin B there exists an element x in A with f(x) = y since f is onto Therefore gof(x) = g (f(x)) = g (y) = z showing that gof is onto Example 4 Let Y = n2 n isin N sub N Consider f N rarr Y as f(n) = n2 Show that

f is invertible Find the inverse of fSolution An arbitrary element y in Y is of the form n2 for some n isin N This implies that n =radicy This gives a function g Y rarr N defined by g (y) =radicy Nowgof (n) = g (n2)=radicn2 = n and fog (y) =f(radicy) = (radicy) 2 y which shows that gof=IN and fog= IY Hence f is invertible with f -1 = g

Political Science

Constitution of India-The Preamble

Summary

Objective of the state-To secure equality of status and of opportunity To promote fraternity among all the citizens To assure the dignity of the individuals and Unity and integrity of the nation

Justice-Justice stands for rule of law absence of arbitrariness and a system of equal rights freedom and opportunities for all in a society India seeks social economic and political justice to ensure equality to its citizens

Liberty-Liberty implies the absence of restraints or domination on the activities of an individual such as freedom from slavery serfdom imprisonment despotism etc The Preamble provides for the liberty of thought expression belief faith and worship

Equality-Equality means the absence of privileges or discrimination against any section of the society The Preamble provides for equality of status and opportunity to all the people of the country

Fraternity-The Preamble declares that fraternity has to assure two thingsmdashthe dignity of the individual and the unity and

Execution

Answer the following questions-

Short notes-1 Equality2 Fraternity3 Justice4 Liberty

Homework-Learn

integrity of the nation The word integrity has been added to the Preamble by the 42nd Constitutional Amendment (1976)

Business studies

Human resource management (chapter 1)

On the day of 1504 2020 I have discussed with you the managerial functions and procurement functions of HRM

Today weare going to discuss about the development function integration functions and maintenance function

Development functions-HRM improves the knowledge skills attitude and values of employees so that they the present and future jobs more effectively it includes

1) Development functions of HRM

a) Performance appraisal = It implies systematic evaluation of employees with respect to their performance on the job and their potential for development

b) Training =It is the process by which employees learn knowledge skills and attitudes to achieve organisational and personal goals

c) Executive development = It is the process of developing managerial talent through appropriate program

2) Integration functionsa) HRM reconcile the goals of

organisation with those of its members through integrating function

b) HRM tries to motivate employees to various financial and non financial incentives provided in job specification etc

3) Maintenance functiona) HRM promote and protect the

physical and mental health of employees by providing several types of benefits like housing medical aid etc

b) It Promote Social security measures to employees by providing provident fund pension gratuity maternity benefits

SubjectCOMMERCE

Topic

BUSINESSENVIRONMENT

Summary

Now quickly let us revise the earlier points that we have already done in the last class and let us proceed with the other topics that are there in the chapter

Firstly we will recall the internal and external factors of micro environment and then we

Execution 3 What do you mean by internal factors

in micro environmentAnswerInternal factors refer to all the factors existing within a business firm The internal factors are considered controllable because the enterprise has control over these factors

Development FunctionsPerformance AppraisalTrainingExecution Development

shall proceed in details

Meaning and list of internal and external factors

aInternal factorsInternal factors refer to all the factors existing within a business firm The internal factors are considered controllable because the enterprise has control over these factorsFor an example a company can alter its organization structure policies programmes employees physical facilities and marketing mix to suit the changes in the environmentList of internal factors areCorporate culture mission and objectives top management organizations structure company image and brand equity company resources

b External factorsExternal factors refer to those individual and groups and agencies with which a particular business organization comes into direct and frequent contact in the course of its functioningThese individuals and groups are known as STAKEHOLDERS because they have a stake (financial interest ) in the working and performance of the particular business List of external forces (stakeholders)Customers competitors investors suppliersmiddlemen (marketing intermediaries)financers publics

customers

suppliersfinancers

For an example a company can alter its organization structure policies programmes employees physical facilities and marketing mix to suit the changes in the environment

4 What do you mean by external factors in micro environment

AnswerExternal factors refer to those individual and groups and agencies with which a particular business organization comes into direct and frequent contact in the course of its functioningThese individuals and groups are known as STAKEHOLDERS because they have a stake (financial interest) in the working and performance of the particular business

3Who are stakeholdersSTAKEHOLDERS are individuals and groups who have a stake (financial interest ) in the working and performance of the particular business 4Discuss the internal factors in briefa Corporate CultureThe values beliefs and attitudes of the founders and top management of the company exercise a strong influence on what the cmpaany stands for how it does things and what it considers importantbMission and objectivesThe business philosophy and purpose of a comoany guide it prioritiesbusiness strategiesproduct market scope and development scope

cTop management structurethe composition of board of directors the degree of professionalization of management and the organizational structure of a company have important bearing on its business decisions

dPower structureThe internal power relationship between the board of directors and the chief executive is an important factor

e Company image and brand equityThe image and brand equity of the company play a significant role in raising finance forming alliance choosing dealers and suppliers launching new products entering foreign markets

5 What is Macro environmentAnswerMacro environment refers to the general

competitors

middlemen

publics

Fig STAKEHOLDERS OF A COMPANY

Apart from micro environment the other main dimension of business environment isMacro environment Macro environment refers to the general environment or remote environment within which a business firm and forces in its micro environment operateA company does not directly or regularly interact with the micro environmentTherefore macro environment is also known as indirect action EnvironmentThe macro environment forces are less controllable than the micro forces

Macro environment consists of the following components

POLITICAL AND LEGAL ENVIRONMENT

ECONOMIC SOCIAL AND ENVIRONMENT

CULTURAL

ENVIRONMENT

TECHNOLOGICAL ENVIRONMENT

Fig COMPONENTS OF MACRO ENVIRONMENT

environment or remote environment within which a business firm and forces in its micro environment operateA company does not directly or regularly interact with the micro environmentTherefore macro environment is also known as indirect action EnvironmentThe macro environment forces are less controllable than the micro forces 6 What are the components of macro environmenta Political and legal environmentb Economic environmentc Social and cultural environmentd Technological environment

Computer Science

Logic gates

Digital systems are said to be constructed by using logic gates These gates are the AND OR NOT NAND NOR EXOR and EXNOR

BUSINESS FIRM

gates The basic operations are described below with the aid of truth tables

AND gate

The AND gate is an electronic circuit that gives a high output (1) only if all its inputs are high A dot () is used to show the AND operation ie AB Bear in mind that this dot is sometimes omitted ie ABOR gate

The OR gate is an electronic circuit that gives a high output (1) if one or more of its inputs are high A plus (+) is used to show the OR operationNOT gate

The NOT gate is an electronic circuit that produces an inverted version of the input at its output It is also known as an inverter If the input variable is A the inverted output is known as NOT A This is also shown as A or A with a bar over the top as shown at the outputs The diagrams below show two ways that the NAND logic gate can be configured to produce a NOT gate It can also be done using NOR logic gates in the same way

NAND gate

This is a NOT-AND gate which is equal to an AND gate followed by a NOT gate The outputs of all NAND gates are high if any of the inputs are low The symbol is an AND gate with a small circle on the output The small circle represents inversion

NOR gate

This is a NOT-OR gate which is equal to an OR gate followed by a NOT gate The outputs of all NOR gates are low if any of the inputs are highThe symbol is an OR gate with a small circle on the output The small circle represents inversion

EXOR gate

The Exclusive-OR gate is a circuit which will give a high output if either but not both of its two inputs are high An encircled plus sign ( ) is used to show the EOR operation

EXNOR gate

The Exclusive-NOR gate circuit does the opposite to the EOR gate It will give a low output if either but not both of its two inputs are high The symbol is an EXOR gate with a small circle on the output The small circle represents inversion The NAND and NOR gates are called universal functions since with either one the AND and OR functions and NOT can be generated

Note A function in sum of products form can be implemented using NAND gates by replacing all AND and OR gates by NAND gates A function in product of sums form can be implemented using NOR gates by replacing all AND and OR gates by NOR gates

Logic gate symbols

Table 2 is a summary truth table of the inputoutput combinations for the NOT gate together with all possible inputoutput combinations for the other gate functions Also note that a truth table with n inputs has 2n rows You can compare the outputs of different gates

Logic gates representation using the Truth table

Example

A NAND gate can be used as a NOT gate using either of the following wiring configurations

Subject Eng Literature (The Tempest ndash William Shakespeare) Topic Act III Scene 3 Lines 53 to 110 (End of the scene) Date 16th April 2020 (2nd Period)

[Students should read the original play and also the paraphrase given in the school prescribed textbook]Summary Questions amp Answers

o Seeing this strange scene all are inclined to believe the tales told by travelers that there truly are ldquounicornsrdquo and ldquothe phoenixrsquo thronerdquo

o As they are about to sit down to the feast the banquet is snatched away by a harpy (Ariel disguised) A spiritrsquos voice (Arielrsquos voice) denounces Alonso Sebastian and Antonio with particular

1 ARIEL You are three men of sin whom Destiny

(Line 53-58)That hath to instrument this

lower world And what is int the never-surfeited sea

Hath caused to belch up you and on this island

Where man doth not inhabit you rsquomongst men

Being most unfit to live I have made you mad

reference to their crime in expelling Prospero from Milan They have not received any punishment for their deed earlier but the time for their punishment has arrived Upon Alonso it pronounces ldquolingering perdition worse than deathrdquo from which there is no remedy except through sincere repentance Ariel then vanishes in thunder and the shapes enter again and carry away the table

o Prospero watching invisibly is very pleased with the performance of Ariel and his (Prosperorsquos) ldquomeaner ministersrdquo All his enemies are now in his power and are in a fit of desperation He then leaves them and goes to see how Ferdinand and Miranda are getting on

o Alonso is now much humbled and penitent with the after effect of the spiritrsquos denunciation of his crimes He believes that his son is lost forever After this all disperse being stricken mad by the speech of the spirit

o Gonzalo fearing that they may do violence to themselves or to one another follows them and bid others to follow

(a) To whom does Ariel disguised as a harpy call the three sinners What game did Fate of Destiny play with

them

The three sinners called by Ariel are Alonso Sebastian and Antonio It was Destiny which had caused the ocean to cast the three sinners on the shore Though the ocean is all the time devouring whatever appears on its surface and is never satisfied with its continual swallowing of the ships and men in the present case the ocean had cast these three sinners on the shore without killing them

(b) Who had jointly been responsible for the conspiracy against Prospero What is Prosperorsquos purpose behind all this

Three men Alonso Sebastian and Antonio had jointly

been responsible for the conspiracy against Prospero They had driven out Prospero form Milan Prosperorsquos purpose is to make these three sinners realize the wrong they had done He wants them to repent for their criminal deeds because repentance leads to self-esteem(c )What does Ariel (the harpy) tell Alonso and his companions when they take out their swords to attack him

Seeing them drawing their swords Ariel (harpy) tells them that he and his companions are the instruments of destiny and that it is not possible for human beings to do them any injury He says that the swords of human beings can not injure even a minute part of his feathers Their swords are as ineffective against him and his companions as against the wind or the water

(d) Give the explanatory meanings of the following expressions in the context of the above extract

(i)Never surfeited (ii) Belch up (iii) lsquomongst men

(i) Never surfeited never led to satisfaction

(ii) Belch up cast ashore(iii) lsquomongst men in human

society2

I and my fellows (Line 60-65)

Are ministers of Fate The elementsOf whom your swords are tempered may as wellWound the loud winds or with bemocked-at stabsKill the still-closing waters as diminishOne dowl thats in my plume

IMPORTANT PASSAGES EXPLAINED

The elements

(Line 61-66)Of whom your swords are tempered may

as wellWound the loud winds or with

bemocked-at stabs

(a) Who is lsquoIrsquo Who are his lsquofellowsrdquo

lsquoIrsquo is referred to Ariel in disguise of a harpy His lsquofellowsrsquo are other spirits serving Prospero the real Duke of Milan who has acquired supernatural powers after being banished from his Dukedom Prospero has settled in this uninhabited island

(b) What are the elements that have temperrsquod the swords Why will it not work against the speaker

The swords (of Alonso and his companions) are tempered by metal (steel) which is taken out of the earth and refined by

Kill the still-closing waters as diminishOne dowl thats in my plume My fellow

ministersAre like invulnerable

In these words Ariel reminds the King and his companions of the utter futility of drawing swords against himself and his fellows Ariel drives Alonso Antonio and Sebastian the three men of sin to desperation ndash a state in which men do violence to themselves They draw swords to strike Ariel But Ariel reminds them that he and the other spirits are the ministers of destiny and nothing can wound them The steel of which their swords are made of may cut the wind or water which being divided always closes up again Even supposing that such things may be possible it is quite impossible that their swords will cut one feather in their plume They are incapable of being wounded by any sword of man Hence it is foolish on their part to attempt to strike at Ariel and his fellow-spirits

For which foul deed

(Line 72-75)The powers delaying not forgetting

haveIncensed the seas and shores yea all the

creatures Against your peace

Ariel enters like a harpy and remaining invisible tells Alonso Sebastian and Antonio that he and other harpies are the agents of Destiny appointed to carry out her decrees He tells them that their punishment for the crime against Prospero which has been so long deferred is now to fall upon them He reminds them that they had expelled Prospero from Milan and set him and his innocent child adrift on the sea and that the sea had paid them back for their sin by the shipwreck and by the calamities they have suffered He tells them that the powers above which did not forget this mean treachery but only deferred the punishment have now engaged the seas and the shores and all living beings including him and his comrades against them The very elements and supernatural agency Ariel adds have taken up the avenging of their crime against Prospero

the action of fire It may cut the wind or water which being divided always closes up again

The sword will not work against the spirits and the harpy because they are the ministers of destiny and nothing can wound them nor it will cut a single feather in their plume

(c )What is the meaning of lsquodowlrsquo in the last line

The term lsquodowlrsquo means a filament or the smallest part of a feather In this context Ariel in disguise of harpy says that their sword cannot even damage the smallest filament of their (Arielrsquos and other spirits) feathers as they are incapable of being wounded by any sword of man

(d) What does the speaker remind the listeners about

Ariel in disguise of harpy reminds Alonso the King of Naples Sebastian Alonsorsquos brother and Antonio the present Duke of Milan and the treacherous brother of Prospero as they being three men of sin He even reminds them that their punishment for their crime against Prospero which has been so long deferred now falls upon them He reminds them that they have expelled Prospero from Milan and has set him along with his innocent infant daughter adrift on the sea So the sea has paid them back for their sin by their shipwreck and the calamities they have suffered since then The harpy rebukes Alonso of his sin that has incensed the Gods and has deprived him of his son as a punishment

(e) How do they respond

When Ariel in disguise of a harpy reminds Alonso Sebastian and Antonio of their past misdeeds and sin Alonso has a look of terror and confusion in his eyes He utters the words of sincere repentance wrung out of his conscience-stricken heart It appears to him that all the elements of nature the sea-waves the wind and the thunder proclaiming a loud voice in the name of Prospero and the crime Alonso has committed against him They are calling upon him to repent There is a deep storm raging in Alonsorsquos breast and the echoes of that storm are ringing in his ears like a clear note of wind-instrument A note of denunciation of Alonsorsquos crime leaves him much humbled and penitent and confirms his belief that his son is lost forever But Sebastian and Antonio shows some courage instead of repentance They wish to kill the spirits or devils if it appears

3

Of my instruction hast thou nothing bated (Line 85-93)

In what thou hast to say So with good life

And observation strange my meaner ministers

Their several kinds have done My high charms work

And these mine enemies are all knit upIn their distractions They now are in my

powerAnd in these fits I leave them while I visitYoung Ferdinand whom they suppose is

drownedAnd his and mine loved darling

Methought the billows spoke and (Line 96-99)

told me of itThe winds did sing it to me and the

thunderThat deep and dreadful organ-pipe

pronouncedThe name of Prosper It did bass my

trespass

These are the words of contrition coming from Alonso Ariel has driven him to a deep repentance for conspiring with Antonio against Prospero He now feels a sincere remorse It appears to him that all the elements of nature the sea-waves the wind and the thunder proclaimed with a loud voice the name of Prospero and the crime Alonso had committed against him They are calling upon him to repent There is a deep storm raging in Alonsorsquos breast and the echoes of that storm are ringing in his ears like the clear note of a wind-instrument

Comment These are the words of sincere repentance wrung out of the conscience-stricken heart of Alonso Alonso who is the lesser villain is the first to give way to remorse under the effect of Arielrsquos speech The words of Ariel seem to him to be the voice of conscience speaking to him He is driven to desperation a state in which he might do violence to his life

(a) Identify the speaker State the context

Prospero the ruler of the island is the speaker The famous banquet scene has been enacted very well Ariel and his junior spirits have played their roles excellently Prospero is glad to say words of praise for them(b) In what way the speakerrsquos instructions have been carried out

According to Prosperorsquos instructions a banquet was presented before the King of Naples and his companions when they were tired and hungry Just when they were preparing to eat the feast the banquet was suddenly removed by exercising supernatural powers All this was done by Ariel Prosperorsquos chief assistant and a powerful spirit

Ariel not only made the feast disappear but also delivered his speech blaming the King and his two companions for their past wicked deeds He warned them to repent for their misdeeds or suffer forever on that uninhabited island

(c) Who are referred to as lsquomeaner ministersrsquo What have they done

Prospero refers as lsquomeaner ministersrsquo to his other lesser spirits who were assisting Ariel in presenting a scene before the kingrsquos party They entered the scene to the accompaniment of music They assumed several strange shapes and brought in a banquet Then they danced about it with gentle actions of salutations thus inviting the King and others to eat the feast

These spirits play their role again when Ariel in the shape of a harpy quits the scene These shapes enter again and dancing with mocking gestures carry away the table

(d) Who are the speakerrsquos enemies What has happened to them

King of Naples Alonso his brother Sebastian and the present Duke of Milan Antonio (Prosperorsquos own brother) are Prosperorsquos enemies With the turn of events they have all been washed ashore on the island which is ruled by Prospero the great magician Actually this happened after the shipwreck caused by a storm which was raised by Prospero with the purpose of bringing these people to his island Prosperorsquos spirits have already confused and terrified these enemies and they are under Prosperorsquos control He can treat them as he likes

(e) What does he say about Ferdinand Explain what is meant by ldquohellip his and mine darlingrdquo

Prospero knows that Alonsorsquos son prince Ferdinand is alive though his father thinks that the prince has been drowned

Prospero refers to his daughter Miranda who is dear to him She is also very dear to Prince Ferdinand who has fallen in love with her They are waiting to be married soon for which they have received Prosperorsquos consent

4

ALONSO O it is monstrous monstrous (Line 95-102)

Methought the billows spoke and told me of it

The winds did sing it to me and the thunderThat deep and dreadful organ-

pipe pronouncedThe name of Prosper It did bass

my trespassTherefore my son ithrsquo ooze is

bedded andIll seek him deeper than eer

plummet soundedAnd with him there lie mudded

(a) In what way does Alonso express his horror when his conscience is awakened by Arielrsquos words

When Alonsorsquos conscience is awakened by Arielrsquos words he expresses his horror at what he has heard He gets the feeling that the waves of the ocean the wind and the loud thunder have spoken to him and uttered the name of Prospero Because of being reminded of his crime in a very loud and rough voice he comes to realize that he has lost his son for his past misdeeds

(b) What does Alonso imagine about his son What does Alonso want to do in his desperate state

Alonso imagines that his son is lying in the mud at the bottom of the sea He feels desperate that he wants to drown himself in the ocean deeper than the plumb-line has ever gone He wants to lie with his son at the bottom of the sea

(c) How do Sebastian and Antonio want to face the evil spirits

Sebastian says that he is not at all afraid of what the harpy has said and that he is prepared to fight any number of such monsters if they appear before him only one at a time Antonio says that he would support Sebastian in the fight against the fiendsyyy

(d) Why does Gonzalo ask Adrian to follow the three men

Gonzalo tells Adrian that all the three men namely Alonso Sebastian and Antonio are in a wild and reckless mood The thought of the heinous crime of which they are guilty has begun to torment their minds So he asks Adrian to follow those three men without loss of time and prevent them from doing anything which the turmoil in their minds might lead them to do

(e) What opinion do you form of Alonso from the above extract

Alonso who is the lesser villain is the first to give way to remorse under the effect of Arielrsquos speech The words of Ariel seem to him to be the voice of conscience speaking to him He is driven to desperation a state in which he might do violence to his life

Subject =Accounts

Ac-12 15420 topic-pL Appropriation ac

PROFIT AND LOSS APPROPRIATION ACCOUNT

MEANING AND PREPARATIONProfit and Loss Appropriation Account is merely an extension of the Profit and Loss Account of the firm The profit of the firm has to be distributed amongst the partners in their respective profit sharing ratio But before its distribution it needs to be adjusted All Adjustments like partnerrsquos salary partnerrsquos commission interest on capital interest on drawings etc are made in this account These adjustments will reduce the amount of profit for distribution This adjusted profit will be distributed amongst the partners in their profit sharing ratio To prepare it at first the balance of Profit and Loss Account is transferred to this account The journal entries for the preparation of Profit and Loss Appropriation Account are given below

1 for transfer of the balance of Profit and Loss Account to Profit and Loss Appropriation Account

(a) In case of Net Profit

Profit and Loss Ac helliphelliphelliphelliphellipDrTo Profit and Loss Appropriation Ac(Net Profit transferred to Profit and Loss Appropriation Ac)

(b)In case of Net Loss

Profit and Loss Appropriation Achelliphelliphellip DrTo Profit and Loss Ac(Net Loss transferred to Profit and Loss Appropriation Ac)

2 for Interest on Capital

For transferring on Interest on CapitalProfit and Loss Appropriation Achelliphelliphellip DrTo Interest on Capital Ac(Interest on capital transferred to Profit amp Loss Appropriation Ac)

3 for Interest on Drawings

For transferring Interest on Drawings Interest on Drawings Achelliphelliphelliphelliphelliphellip DrTo Profit and Loss Appropriation Ac(Interest on drawing transferred to Profit amp Loss Appropriation Ac)

4 For Partnerrsquos SalaryFor transfer of partnerrsquos SalaryProfit and Loss Appropriation Achelliphellip DrTo Salary Ac(Salary transferred to profit amp Loss Appropriation Ac)

5 For Partnerrsquos CommissionFor transferring commissionProfit and Loss Appropriation Achelliphelliphellip DrTo Commission Ac(Commission transferred to Profit and Loss Appropriation Ac)

6 For Transfer of agreed amount to General ReserveProfit and Loss Appropriation Ac helliphellipDrTo General Reserve Ac(Transfer to General Reserve)

7 for share of Profit or Loss appropriation(a) If ProfitProfit and Loss Appropriation Achelliphellip DrTo Partnerrsquos CapitalCurrent Ac(Profit transferred to capitalcurrent Ac)(b) If LossPartnerrsquos Capital Current Achelliphelliphelliphellip DrTo Profit and Loss Appropriation Ac(Loss transferred to capitalcurrent Ac)

THE FORMAT OF PROFIT AND LOSS APPROPRIATION

Profit and Loss Appropriation Account for the year endedhelliphelliphelliphellip

Particulars Amount Particulars Amount

To PL Ac (loss) By pL Ac (profit)

To Interest on capital BY Interest on drawings

To partner`s commission by Partner`s capital Ac ( loss)

To Partner`s salary To Interest on partner`s loan To General Reserve To Partner`s Capital AC (Profit)

Subject= Economics

MOVEMENT ALONG THE DEMAND CURVE (CHANGE IN QUANTITY DEMANDED)In law of demand you have already studied the inverse relationship between price and quantity demanded When quantity demanded of a commodity changes due to change in its price keeping other factors constant it is called change in quantity demanded It is graphically expressed as a movement along the same demand curve There can be either a downward movement or an upward movement along the same demand curve Upward movement along the same demand curve is called contraction of demand or decrease in quantity demanded and downward movement along the same demand curve is known as expansion of demand or increase in quantity demanded

Extention of demandd

price (rs)p A

B Extentionp1 d

Q Q1

Quantity demanded ( in units)

Contraction of demandd

p2 Ccontraction

p APrice (Rs)

d

Q2 Q

Quantity demanded (in units)

Explanation of movement of demand A fall in price from OP to OP1 leads to increase in quantity demanded from OQ to OQ1 (expansion of demand) resulting in a downward movement from point A to point B along the same demand curve DD When Price rises from OP to OP2 quantity demanded falls from OQ to OQ2 (contraction of demand) leading to an upward movement from point A to point C along the same demand curve DD

  • Activity Series of Metals
    • Drawbacks of Rutherfordrsquos model of atom
      • Electromagnetic radiations
      • Properties of electromagnetic radiations
      • Characteristics of electromagnetic radiations
        • Plancks Quantum Theory-
        • Photoelectric effect
          • Intext Questions
            • Logic gates
            • Digital systems are said to be constructed by using logic gates These gates are the AND OR NOT NAND NOR EXOR and EXNOR gates The basic operations are described below with the aid of truth tables
            • AND gate
            • Example
Page 36:  · Web viewSubject. Topic. Summary. Execution. English 1 . Chapter 1 naming words . Page 8. Write the names of these pictures:- Person:-1. father. 2.Firefighter 3.doctor 4 ...

financers publics

customers

suppliersfinancers

competitors

middlemen

publics

Fig STAKEHOLDERS OF A COMPANY

Apart from micro environment the other main dimension of business environment isMacro environment Macro environment refers to the general environment or remote environment within which a business firm and forces in its micro environment operateA company does not directly or regularly interact with the micro environmentTherefore macro environment is also known as indirect action EnvironmentThe macro environment forces are less controllable than the micro forces

Macro environment consists of the following components

POLITICAL AND LEGAL ENVIRONMENT

ECONOMIC SOCIAL AND ENVIRONMENT

CULTURAL

ENVIRONMENT

TECHNOLOGICAL ENVIRONMENT

a strong influence on what the cmpaany stands for how it does things and what it considers importantbMission and objectivesThe business philosophy and purpose of a comoany guide it prioritiesbusiness strategiesproduct market scope and development scope

cTop management structurethe composition of board of directors the degree of professionalization of management and the organizational structure of a company have important bearing on its business decisions

dPower structureThe internal power relationship between the board of directors and the chief executive is an important factor

eCompany image and brand equityThe image and brand equity of the company play a significant role in raising finance forming alliance choosing dealers and suppliers launching new products entering foreign markets

5 What is Macro environmentAnswerMacro environment refers to the general environment or remote environment within which a business firm and forces in its micro environment operateA company does not directly or regularly interact with the micro environmentTherefore macro environment is also known as indirect action EnvironmentThe macro environment forces are less controllable than the micro forces 6 What are the components of macro environmenta Political and legal environmentb Economic environmentc Social and cultural environmentd Technological environment

BUSINESS FIRM

Fig COMPONENTS OF MACRO ENVIRONMENTPolitical science

Introduction to political science

Comparative politics and itrsquos scope Comparative politics is the second major dimension of political scienceIt is also a very vast area of study and a very large number of political scientists even treat it as an autonomous area of study within the board ambit of political scienceScope of comparative politics-

1 All political structures -Comparative politics includes the study of all structures formalnon formal governmental and extra governmental which are directly or indirectly involved in politics in all the countries of the world

2 Functional studies- Comparative politics seeks to study politics less from the point of view of the legal institutions in terms of their powers and move from the point of view of their functions which constitute the political process and their actual Operation in the environment

3 Study of political behaviour- Another important part of its scope is the study of the actual behaviour of the people in the process of politics

4 Study of similarities and differences- comparative politics also undertakesan analysis of the similarities and differences among political process and functions

5 Study of all political systems -comparative politics seeks to analyse the actual behaviour and performance of all political systems western as well as non western

6 Study of the environment and infrastructure of politics-The study of politics demands a study of the psychological sociological economic and anthropological environment in fact the social environment as a whole in which each political system operates

7 Study of political culture- political culture is composed of attitudesbeliefs emotions and values of a society that relate to the political system or politics

8 Study of political participation- Political participation is a universal processThe only difference is that while in some states it is limited in others it is wider

9 Study of political process- political

Answer the following questions-

What is comparative politics

What are the scope of comparative politics

Homework- learn

processes like decision makingpolicy making judicial process leadership recruitment process and others are always at work in all political systems

The scope of comparative politics is very comprehensive It includes everything that falls within the area of political activity and political process

History CAMBRIDGE VIEW ABOUT

THE PARTITION

AND REFUTATION

OF CAMBRIDGE

VIEW

Cambridge view about the Partition The Cambridge school of historians have interpreted that opposition to partition scheme was made entirely by the elitist groups They hold the view that Lord Curzon planned to partition the Bengal for administrative purposeREFUTATION OFCAMBRIDGE VIEW The Rationalist historians have rejected the interpretations of the Cambridge School of historians on various grounds

1 QUESTION State different views of historians regarding Partition of Bengal

ANSWER Cambridge historians believed that Lord Curzon partitioned Bengal for administrative reasons only and not for the political motive The Middle class elitist group protested because of their petty interest The Hindu zamindars protested as they have to spend more money for managing their estatesThe lawyers of Calcutta High court feared to lose their clientBut according to the nationalist Historians was-

2- The ultimate object of Lord Curzon was to crush the unity of Bengal politicians

3- If Bengal becomes a separate province Bengali speaking 16 million people of western part would become minority under Hindi speaking people of Bihar and Oriya speaking people of Orissa

4- The bureaucrats expected that the protest movement would die down quickly

5- Lord Curzon used the Muslim community in his political game

6- Idealism had great contribution in the protest against partition

7- The people of the every section of society were affected by the partition of Bengal

Computer Science

Numbers Convertion of dcimal number to octal numberThe decimal numeral system is the standard system for denoting integer and non-integer numbers It is the extension to non-integer numbers of the Hindu-Arabic numeral system For writing numbers the decimal system uses ten decimal digits a decimal mark and for negative numbers a minus sign - The decimal digits are 0 1 2 3 4 5 6 7 8 9 the decimal separator is the dot in many countries

The octal numeral system or oct for short is the base-8 number system and uses the digits 0 to 7 Octal is sometimes used in computing instead of hexadecimal perhaps most often in modern times in conjunction with file

permissions under Unix systems It has the advantage of not requiring any extra symbols as digits It is also used for digital displays

Follow these steps to convert a decimal number into octal form

1 Divide the decimal number by 82 Get the integer quotient for the next iteration (if the number will not divide equally by 8 then round down the

result to the nearest whole number)3 Keep a note of the remainder it should be between 0 and 74 Repeat the steps until the quotient is equal to 05 Write out all the remainders from bottom to top This is the solution

For example if the given decimal number is 8453

Division Quotient Remainder

8453 8 1056 5

1056 8 132 0

132 8 16 4

16 8 2 0

2 8 0 2

Then the octal solution is 20405

Subject Eng Literature (The Tempest ndash William Shakespeare) Topic Act I Scene 1 Lines 33 to 67 (End of scene) Date 16th April 2020 (4th Period)

[Students should read the original play and also the paraphrase given in the school prescribed textbook]Summary Questions amp Answers

[SUMMARY OF THE ENTIRE SCENE]

o The play starts with the scene of a severe storm at sea Alonso (King of Naples) Sebastian (Alonsorsquos brother) Ferdinand (Alonsorsquos son) Gonzalo Antonio (the usurping Duke of Milan) are in a ship in the midst of the storm

o The mariners are trying their best to control the vessel from running aground and are totally following the orders of their Master the Boatswain They have scant success

o The mariners become extremely unhappy and annoyed when most of the passengers arrive on the deck thereby hampering their effort to save the ship There is serious confrontation between them and the passengers who are part of the Kingrsquos entourage

o The mariners could not save the ship

SUMMING-UP

(i) Vivid description of the scene which gives a realistic description of terror and confusion of a tropical storm

(ii) Shows Shakespearersquos accuracy of knowledge in describing the naval operations and also matters of seamanship

(1) GONZALO Ill warrant him for drowning (L 45-57)

though the ship were no stronger than a nutshell and as leaky as an unstanched

wenchBOATSWAIN Lay her a-hold a-hold Set her two courses Off to

sea again lay her offMARINERS All lost To prayers to prayers All lostBOATSWAIN What must our mouths be coldGONZALO The king and prince at prayers Lets assist them

For our case is theirsSEBASTIAN Im out of patienceANTONIO We are merely cheated of our lives by drunkards

This wide-chopped rascal - would thou mightst lie drowning the washing of ten tides

(a) What does Antonio say at the insolent manners of the boatswain just before the given passage

Being irritated at the insolent manners of the boatswain just before the given extract Antonio the Duke of Milan calls him a worthless dog son of a woman without any morals an arrogant and disrespectful noisemaker He says that the boatswain deserved to be hanged(b) What statement does Gonzalo repeat about the boatswain

Gonzalo shows his faith that the boatswain is not destined to die by drowning He is destined to be hanged and nothing can alter this decree of destiny He says that even if the ship was as frail as a nutshell the boatswain could not be drowned for his destiny was to be hanged(c) What do the passengers do when they have lost all hope of their survival

When the passengers have lost all hope of survival they take

(iii) The opening scene justifies the title ndash The Tempest

UNANSWERED QUESTIONS

(i) The King always travels with his entire fleet including his soldiers Where were the other ships

(ii) Why was the ship in that area Where was it coming from or going where

(iii) The ship broke apart What happened to those who were in the ship

(We shall get the answer to the above questions as the play progresses)

leave of life with fervent prayers The mariners take their last hearty drink and are ready for death(d) What blame does Antonio put upon the mariners and the boatswain Antonio rebukes the mariners that these drunkards have brought them to the present crisis by neglecting their duties He blames them saying that they are going to lose their lives entirely for the negligence of the boatswain and his fellows(e) What does Antonio say while cursing the boatswain

Antonio gives vent to his wrath upon the boatswain in particular He calls the boatswain a wide-mouthed rascal who deserves to be hanged on the sea-shore at low water mark so that ten tides might wash over his body and take out of him all the liquor that he has been drinking

Class XIISubject Topic Summary ExecutionHistory Topic

1 1935 ACT AND WORKING OF PROVINCIAL AUTONOMYCONGREE AND OTHER MINISTERSSUB TOPIC GOVERNMENT OF INDIA ACT1935

Government of India Act 1935 This act established a lsquoFederation of Indiarsquo made of British Indian provinces and Indian states and provided for autonomy with a government responsible to the elected legislature in every provinceThis act introduced abolition of Diarchy at provinces The entire provincial administration was introduced to the responsible ministers who were controlled and removed by the provincial legislature The provincial autonomy means two things First The provincial governments were wholly responsible to the provincial legislature Secondly Provinces were free from outside control and interference in the large number of matters The act divided the powers between the centre and provinces in terms of three lists- Federal list( for centre) Provincial list (for province) and concurrent list (for both) Residuary powers were given to the viceroy In the election under the government of India Act the Congress swept the poll the mandate of the people came in favour of the congress so far as general Hindu seats were concerned The Congress did not get a single Muslim seates in Bombay CP UP Sind and BengalIn five provinces Congress had yhe clear majority In BengalNWFPAssam and Bombay Congress emerged as a single largest partyOn the other side the performance of the Muslim League was badThus the Congress formed ministers in 7 provinces out of 11 provinces Coalition ministry was also formed in two other provincesOnly BENGAL AND Punjab had non- congress ministries

1 QUESTION What was the main change introduced by the Government of India ActANSWER a) The Act gave more

autonomy to the provinces b) Diarchy was abolished at the

provincial levelsc) The Governor was the head of

the executived) There was a council of

ministers to advise him The ministers were responsible to the provincial legislatures who controlled them The legislature could also remove the ministers

e) The Governors still retained special reserve powers

2 QUESTION Why did the federal scheme introduced by the Government of India Act 1935 never come into operation

ANSWER The Federal structure of the Government of India was to be composed with the Governor General and Council of ministers The Federal legislature was to be Bicameral legislature- The council of states and the House of Assembly The ministers were to be chosen by the Governor general and they were to hold the office during his pleasure

The provinces of British India would have to join the federation but this was not compulsory for the princely states

This federation never materialised because of the lack of support from the required number of

princely statesThis act was refused and

rejected by the princes the Congress and the Muslim League

Thus both Congress and the League participated in the election of 1937 Thus the federal part was never introduced but the provincial part was put into operations

Bengali 2nd

Language

াচেরর পরাথCনা(কহিতা )

াচেরর পরাথCনা কহিতাটি কহি (ঙখ দেঘাচে4র দো আচো য কহিতায় াচেরর পতর হমায়ন কঠিন দেরাচেগ আxানত ার ঈশবর া আললার কাচেছ পরাথCনা কচেরচেছন তার পচেতরর ীন হিফহিরচেয় হিচেত এই কহিতায় ার পচেতরর ীন হিভbা দেচেয়চেছন ারার এমনহিক হিনচের ীন হিসCচেনর হিহিনমচেয় হিতহিন তার দেছচের ীন হিফচের দেপচেত দেচেয়চেছন তার দেছচের এই দেরাচেগর ন য হিতহিন হিনচেচেকই ায়ী কচেরচেছন তার হিনচের করা পাপচেকই হিতহিন ায়ী কচেরচেছন এছাা রানৈনহিতক ও আথCসামাহিক অসথার কথা তচে ধরা চেয়চেছ এই কহিতায় ার তার হিনচের পাপ কমCচেকই ায়ী কচেরচেছ ার অন যায় ভাচে দেপহি((হিকতর মাধ যচেম অপররা য কচেরচেছ আর এই অন যায় কাচের ন যই তার পহিরাচের হিপযCয় এচেসচেছ দে এক পরকার মানহিক নধন ইহিতাচেসর ার হিপতা চেয় সবাভাহিকভাচে ভাচোাসা দে মমতা দেথচেক মকত চেত পাচেরনহিন তাই হিপতা চেয় আললা া ভগাচেনর কাচেছ পতর হমায়চেনর পরানহিভbা দেচেয়চেছন ার আললা া ভগাচেনর কাচেছ াহিনচেয়চেছন তার হিনচের ীন হিসCন হিচেত হিতহিন রাী তার হিহিনমচেয় পচেতরর ীন হিফচের দেপচেত দেচেয়চেছন াচেরর হিপতসভ হিচেকর কথা এই কহিতায় ফটিচেয় দেতাা চেয়চেছ হিপতা পচেতরর হিরাহিরত মান নধচেনর কথা তচে ধরা চেয়চেছ

হিচে(4 হিকছ াইচেনর তাৎপযC১) ldquoদেকাথায় দেগ ওর সবচছয দেৌন দেকাথায় কচেরায় দেগাপন bয়ldquoউততর) াচেরর পতর হমায়ন কঠিন দেরাচেগ অসসথ তাই তার দেযৌন াহিরচেয় যাচেচছ এই দেরাচেগ তাচেক দেগাপচেন কচেরকচের াচেচছ তার সক (হিকত ধীচের ধীচের bয় চেচছ তাই হিপতা চেয় ার আললার কাচেছ হমায়চেনর পরান হিভbা দেচেয়চেছন২) ldquoাগাও (চেরর পরাচেনত পরানতচের ধসর (ন দেযর আান গানldquoউততর) াচেরর পতর হমায়ন কঠিন দেরাচেগ আxানত তাই ার আ দে(াচেক মমCাত (চেরর পচেথ পরানতচের আান গান ধবহিনত দোক দেসই আান গান আললার কাচেছ দেযন চে যায় আললা দেযন এই আহিতC শচেন পচেতরর ীন হিফহিরচেয় দেয় ৩)ldquoনাহিক এই (রীচেরর পাচেপর ীানচেত দেকানই তরারণ দেনই ভহি4চেতরldquoউততর) হমায়চেনর অসসথতার ন য ার হিনচেচেকই ায়ী কচেরচেছন কারন ার অচেনক রা য অন যায় ভাচে কচেরচেছ তাই তার এই পাপ কাচের ন য তার ঘচের আ হিপ এচেসচেছ এই অন যায় কাচের ন য তার মহিকত দেনই তাই ার আললার কাচেছ এই পাপ কাচেযCর ন য bমা পরাথM

Hindi 2ndlang

-ासी(जयशकर परसा-)

-ासी जयशकर परसा- की एक ऐसी कहानी ह जिजसम भारतीय ससकनित और राषटरीयता का सवरगजीतहोता ह इस कहानी म इरावती एक निहद कनया ह जिजस मलअचछो न मलतान की लट म पकडा और -ासी बना दि-या उस 500 दि-न -कर काशी क एक महाजन न खरी-ा दसरी -ासी निफरोजा ह वह गलाम ह निफरोजा को छडान क कतिलए अहम- को 1000 सोन क कतिसकक भजन थ जो अभी तक नही आए थ राजा साहब कठोर होत हए भी निफरोजा को निबना धनराकतिश क कतिलए उस म कर -त ह वनिफरोजा को अहम- को समझान की बात कहत हकहानी क अत म हम -खत ह निक इरा वती और जाटो क सर-ार बलराज का मिमलन होता हअहम- को यa म मार दि-या जाता ह वहा निफरोजा की परसननता की समामिध बनती ह वहा एक फल चढती ह और डीजल आती ह निफरोजा उस समामिध की आजीवन -ासी बनी रहती हलखक अपन उददशय अथात -ास परथा पर परकाश डालन और इस परथा क कारण होन वाल -ातो क दखो को दि-खान म पणता सफल हए ह

helliphellipContinue to next

Biology Reproductio Today we will discuss about vegetative Q1 Name some vegetative propagules

n in Organisms

propagation of plants The process of multiplication in which fragments of plant body function as propagule and develop into new individual is called vegetative propagation The units of such propagation are runner rhizome tuber bulb etc

and the speciesinvolvedVegetative propagules

Parts involved

Bulb StemBulbil BulbilRhizome Stem Runner Stem Tuber Stem Offset Stem Leaf buds Leaves Suckers Stem

Corns Stem stolon

Q2 State advantages of vegetative propagation

i) Rapid methodii) Sure and easy methodiii) Useful in plants that cannot

produce viable seeds or long seed dormancy

iv) Maintains purity of raceQ 3 Banana fruit is said to be parthenocarpic where as turkey is said to be parthenogenetic WhyBanana develops without fertilization from an unfertilized ovary thus is parthenocarpicIn turkey the ovum or female gamete developinto a new chick without fertilization thus isparthgenetic

Q4 Why is water hyacinth is called as a ldquoTerror of Bengalrdquo Water hyacinth can

propagatevegetatively all over the water body in a short per short period of time This resulted increased biochemicaloxygen oxygen demand of water body causing mortalityof fishes It is very difficult to get rid off them Thus known as terror of Bengal

Chemistry

Solid state GENERAL CHARACTERISTICS OF SOLID STATEIn nature the particular state of matter is governed by two opposing forces at given set of temperature and pressure These forces are intermolecular force of attraction and thermal energy If intermolecular force of attraction is high as compared to thermal energy particles remains in closest position

Intext QuestionsQ1 Classify the following solids as crystalline and amorphous Sodium chloride quartz glass quartz rubber polyvinyl chloride Teflon

A1 Crystalline

and hence very less movement in particles is observed In this case solid state is the preferred state of matter

Let us revise the general characteristics of solid

i) Fixed mass volume and shape

ii) Strong intermolecular force of attraction

iii) Least intermolecular space

iv) Fixed position of constituent particles

v) Incompressible and rigid

Q2 what type of interactions hold the molecules together in a polar molecular solid[CBSE 2010]A2 The molecules in a solid are held together by van der Waals forces The term van der Waals forces include hydrogen bonding dipole-dipole attraction and London dispersion forces All molecules experience London dispersion forces In addition polar molecules can also experience dipole-dipole interactions So the interactions that holds the molecule together in polar molecular solid are London dispersion force and dipole-dipole interactionsQ3 Write a feature that will distinguish a metallic solid from an ionic solid [CBSE 2010]A3 Metals are malleable and ductile whereas ionic solid are hard and brittle Metallic solid has typical metallic lustre But ionic solid looks dullQ4 Write a point of distinction between a metallic solid and an ionic solid other than metallic lustre [CBSE 2012]A4 Metals are malleable and ductile whereas ionic solid are hard and brittleQ5 Write a distinguish feature of metallic solid [CBSE 2010]A5 The force of attraction in

solid Sodium chloride Quartz Amorphous solid Quartz glass rubber polyvinyl chloride Teflon Q2 why glass is considered as super cooled liquidA2 Glass shows the tendency to flow at slower rate like liquid Hence they considered as super cooled liquidQ3 why the window glass of old buildings show milky appearance with timeA3 Glass is an amorphous solid Amorphous solid has the tendency to develop some crystalline character on heating Due to heating in day over the number of years glass acquires some crystalline character and show milky appearanceQ4 why the glass panes fixed to window or doors of old building become slightly thicker at bottomA4 Glass is super cooled liquid It has the tendency to flow down very slowly Due to this glass pane becomes thicker at the bottom over the timeQ5 Sodium chloride is a crystalline solid It shows the same value of refractive index along all the direction TrueFalse Give reasonA5 FalseCrystalline solid shows anisotropy in properties That is it shows different values for the given physical property in different direction All the crystalline solids show anisotropy in refractive index Therefore sodium chloride will show different values of refractive index on different directions

Q6 Crystalline solid are anisotropic in nature What does this statement means

between the constituent particles is special kind of electrostatic attraction That is the attraction of positively charged kernel with sea of delocalized electronsQ6 which group of solid is electrical conductor as well as malleable and ductile [CBSE 2013]A6 Metallic solidQ7 why graphite is good conductor of electricity although it is a network (covalent solid)A7 The exceptional property of graphite is due to its typical structure In graphite each carbon is covalently bonded with 3 atoms in same layer The fourth valence electron of each atom is free to move in between different layersThis free electron makes the graphite a good conductor of electricity

[CBSE 2011]A6 Anisotropy is defined asrdquo Difference in properties when measured along different axis or from different directionsrdquo Crystalline solid show different values of some of the physical properties like electrical resistance refractive index etcwhen measured along the different directions The anisotropy in crystalline solid arises due to the different arrangement of particles in different directions

Math Function Composition of functions Think of an industrial plant that produce bottles of cold drinks first there is the operation (or function) f that puts the cold drink inside the bottle followed by the opeartion g that close the bottle with the capThis leads to the following definitionDefinition Let f A rarr B and g B rarr C be two functions Then the composition of f and g denoted by gof is defined as the function gof A rarr C given by gof(x) = g(f (x)) forall x isinA

Definition A function f X rarr Y is defined to be invertible if there exists a function g Y rarr X such that gof = IX and fog = IY The function g is called the inverse of f and is denoted by f -1

Thus if f is invertible then f must be one-one and onto and conversely if f is one-one and onto then f must be invertible This fact significantly helps for proving a function f to be invertible by showing that f is one-one and onto specially when the actual inverse of f is not to be determined

Example 1 Let f 2 3 4 5 rarr 3 4 5 9 and g 3 4 5 9 rarr 7 11 15 be functions defined as f(2) = 3 f(3) = 4 f(4) = f(5) = 5 and g (3) = g (4) = 7 and g (5) = g (9) = 11 Find gofSolution We have gof(2) = g (f(2)) = g (3) = 7 gof(3) = g (f(3)) = g (4) = 7gof(4) = g (f(4)) = g (5) = 11 and gof(5) = g (5) = 11Example 2 Find gof and fog if f R rarr R and g R rarr R are given by f(x) = cos x and g (x) = 3x2 Show that gof ne fogSolution We have gof(x) = g(f(x))=g(cosx) = 3 (cos x)2

= 3 cos2 x Similarly fog(x)=f(g (x))= f(3x2)= cos (3x2) Note that 3cos2 x ne cos 3x2 for x = 0 Hence gof ne fogExample 3 Show that if f A rarr B and g B rarr C are onto then gof A rarr C is also ontoSolution Given an arbitrary element z isin C there exists a pre-image y of z under g such that g (y) = z since g is onto Further for y isin B there exists an element x in A with f(x) = y since f is onto Therefore gof(x) = g (f(x)) = g (y) = z showing that gof is onto Example 4 Let Y = n2 n isin N sub N Consider f N rarr Y as f(n) = n2 Show that

f is invertible Find the inverse of fSolution An arbitrary element y in Y is of the form n2 for some n isin N This implies that n =radicy This gives a function g Y rarr N defined by g (y) =radicy Nowgof (n) = g (n2)=radicn2 = n and fog (y) =f(radicy) = (radicy) 2 y which shows that gof=IN and fog= IY Hence f is invertible with f -1 = g

Political Science

Constitution of India-The Preamble

Summary

Objective of the state-To secure equality of status and of opportunity To promote fraternity among all the citizens To assure the dignity of the individuals and Unity and integrity of the nation

Justice-Justice stands for rule of law absence of arbitrariness and a system of equal rights freedom and opportunities for all in a society India seeks social economic and political justice to ensure equality to its citizens

Liberty-Liberty implies the absence of restraints or domination on the activities of an individual such as freedom from slavery serfdom imprisonment despotism etc The Preamble provides for the liberty of thought expression belief faith and worship

Equality-Equality means the absence of privileges or discrimination against any section of the society The Preamble provides for equality of status and opportunity to all the people of the country

Fraternity-The Preamble declares that fraternity has to assure two thingsmdashthe dignity of the individual and the unity and

Execution

Answer the following questions-

Short notes-1 Equality2 Fraternity3 Justice4 Liberty

Homework-Learn

integrity of the nation The word integrity has been added to the Preamble by the 42nd Constitutional Amendment (1976)

Business studies

Human resource management (chapter 1)

On the day of 1504 2020 I have discussed with you the managerial functions and procurement functions of HRM

Today weare going to discuss about the development function integration functions and maintenance function

Development functions-HRM improves the knowledge skills attitude and values of employees so that they the present and future jobs more effectively it includes

1) Development functions of HRM

a) Performance appraisal = It implies systematic evaluation of employees with respect to their performance on the job and their potential for development

b) Training =It is the process by which employees learn knowledge skills and attitudes to achieve organisational and personal goals

c) Executive development = It is the process of developing managerial talent through appropriate program

2) Integration functionsa) HRM reconcile the goals of

organisation with those of its members through integrating function

b) HRM tries to motivate employees to various financial and non financial incentives provided in job specification etc

3) Maintenance functiona) HRM promote and protect the

physical and mental health of employees by providing several types of benefits like housing medical aid etc

b) It Promote Social security measures to employees by providing provident fund pension gratuity maternity benefits

SubjectCOMMERCE

Topic

BUSINESSENVIRONMENT

Summary

Now quickly let us revise the earlier points that we have already done in the last class and let us proceed with the other topics that are there in the chapter

Firstly we will recall the internal and external factors of micro environment and then we

Execution 3 What do you mean by internal factors

in micro environmentAnswerInternal factors refer to all the factors existing within a business firm The internal factors are considered controllable because the enterprise has control over these factors

Development FunctionsPerformance AppraisalTrainingExecution Development

shall proceed in details

Meaning and list of internal and external factors

aInternal factorsInternal factors refer to all the factors existing within a business firm The internal factors are considered controllable because the enterprise has control over these factorsFor an example a company can alter its organization structure policies programmes employees physical facilities and marketing mix to suit the changes in the environmentList of internal factors areCorporate culture mission and objectives top management organizations structure company image and brand equity company resources

b External factorsExternal factors refer to those individual and groups and agencies with which a particular business organization comes into direct and frequent contact in the course of its functioningThese individuals and groups are known as STAKEHOLDERS because they have a stake (financial interest ) in the working and performance of the particular business List of external forces (stakeholders)Customers competitors investors suppliersmiddlemen (marketing intermediaries)financers publics

customers

suppliersfinancers

For an example a company can alter its organization structure policies programmes employees physical facilities and marketing mix to suit the changes in the environment

4 What do you mean by external factors in micro environment

AnswerExternal factors refer to those individual and groups and agencies with which a particular business organization comes into direct and frequent contact in the course of its functioningThese individuals and groups are known as STAKEHOLDERS because they have a stake (financial interest) in the working and performance of the particular business

3Who are stakeholdersSTAKEHOLDERS are individuals and groups who have a stake (financial interest ) in the working and performance of the particular business 4Discuss the internal factors in briefa Corporate CultureThe values beliefs and attitudes of the founders and top management of the company exercise a strong influence on what the cmpaany stands for how it does things and what it considers importantbMission and objectivesThe business philosophy and purpose of a comoany guide it prioritiesbusiness strategiesproduct market scope and development scope

cTop management structurethe composition of board of directors the degree of professionalization of management and the organizational structure of a company have important bearing on its business decisions

dPower structureThe internal power relationship between the board of directors and the chief executive is an important factor

e Company image and brand equityThe image and brand equity of the company play a significant role in raising finance forming alliance choosing dealers and suppliers launching new products entering foreign markets

5 What is Macro environmentAnswerMacro environment refers to the general

competitors

middlemen

publics

Fig STAKEHOLDERS OF A COMPANY

Apart from micro environment the other main dimension of business environment isMacro environment Macro environment refers to the general environment or remote environment within which a business firm and forces in its micro environment operateA company does not directly or regularly interact with the micro environmentTherefore macro environment is also known as indirect action EnvironmentThe macro environment forces are less controllable than the micro forces

Macro environment consists of the following components

POLITICAL AND LEGAL ENVIRONMENT

ECONOMIC SOCIAL AND ENVIRONMENT

CULTURAL

ENVIRONMENT

TECHNOLOGICAL ENVIRONMENT

Fig COMPONENTS OF MACRO ENVIRONMENT

environment or remote environment within which a business firm and forces in its micro environment operateA company does not directly or regularly interact with the micro environmentTherefore macro environment is also known as indirect action EnvironmentThe macro environment forces are less controllable than the micro forces 6 What are the components of macro environmenta Political and legal environmentb Economic environmentc Social and cultural environmentd Technological environment

Computer Science

Logic gates

Digital systems are said to be constructed by using logic gates These gates are the AND OR NOT NAND NOR EXOR and EXNOR

BUSINESS FIRM

gates The basic operations are described below with the aid of truth tables

AND gate

The AND gate is an electronic circuit that gives a high output (1) only if all its inputs are high A dot () is used to show the AND operation ie AB Bear in mind that this dot is sometimes omitted ie ABOR gate

The OR gate is an electronic circuit that gives a high output (1) if one or more of its inputs are high A plus (+) is used to show the OR operationNOT gate

The NOT gate is an electronic circuit that produces an inverted version of the input at its output It is also known as an inverter If the input variable is A the inverted output is known as NOT A This is also shown as A or A with a bar over the top as shown at the outputs The diagrams below show two ways that the NAND logic gate can be configured to produce a NOT gate It can also be done using NOR logic gates in the same way

NAND gate

This is a NOT-AND gate which is equal to an AND gate followed by a NOT gate The outputs of all NAND gates are high if any of the inputs are low The symbol is an AND gate with a small circle on the output The small circle represents inversion

NOR gate

This is a NOT-OR gate which is equal to an OR gate followed by a NOT gate The outputs of all NOR gates are low if any of the inputs are highThe symbol is an OR gate with a small circle on the output The small circle represents inversion

EXOR gate

The Exclusive-OR gate is a circuit which will give a high output if either but not both of its two inputs are high An encircled plus sign ( ) is used to show the EOR operation

EXNOR gate

The Exclusive-NOR gate circuit does the opposite to the EOR gate It will give a low output if either but not both of its two inputs are high The symbol is an EXOR gate with a small circle on the output The small circle represents inversion The NAND and NOR gates are called universal functions since with either one the AND and OR functions and NOT can be generated

Note A function in sum of products form can be implemented using NAND gates by replacing all AND and OR gates by NAND gates A function in product of sums form can be implemented using NOR gates by replacing all AND and OR gates by NOR gates

Logic gate symbols

Table 2 is a summary truth table of the inputoutput combinations for the NOT gate together with all possible inputoutput combinations for the other gate functions Also note that a truth table with n inputs has 2n rows You can compare the outputs of different gates

Logic gates representation using the Truth table

Example

A NAND gate can be used as a NOT gate using either of the following wiring configurations

Subject Eng Literature (The Tempest ndash William Shakespeare) Topic Act III Scene 3 Lines 53 to 110 (End of the scene) Date 16th April 2020 (2nd Period)

[Students should read the original play and also the paraphrase given in the school prescribed textbook]Summary Questions amp Answers

o Seeing this strange scene all are inclined to believe the tales told by travelers that there truly are ldquounicornsrdquo and ldquothe phoenixrsquo thronerdquo

o As they are about to sit down to the feast the banquet is snatched away by a harpy (Ariel disguised) A spiritrsquos voice (Arielrsquos voice) denounces Alonso Sebastian and Antonio with particular

1 ARIEL You are three men of sin whom Destiny

(Line 53-58)That hath to instrument this

lower world And what is int the never-surfeited sea

Hath caused to belch up you and on this island

Where man doth not inhabit you rsquomongst men

Being most unfit to live I have made you mad

reference to their crime in expelling Prospero from Milan They have not received any punishment for their deed earlier but the time for their punishment has arrived Upon Alonso it pronounces ldquolingering perdition worse than deathrdquo from which there is no remedy except through sincere repentance Ariel then vanishes in thunder and the shapes enter again and carry away the table

o Prospero watching invisibly is very pleased with the performance of Ariel and his (Prosperorsquos) ldquomeaner ministersrdquo All his enemies are now in his power and are in a fit of desperation He then leaves them and goes to see how Ferdinand and Miranda are getting on

o Alonso is now much humbled and penitent with the after effect of the spiritrsquos denunciation of his crimes He believes that his son is lost forever After this all disperse being stricken mad by the speech of the spirit

o Gonzalo fearing that they may do violence to themselves or to one another follows them and bid others to follow

(a) To whom does Ariel disguised as a harpy call the three sinners What game did Fate of Destiny play with

them

The three sinners called by Ariel are Alonso Sebastian and Antonio It was Destiny which had caused the ocean to cast the three sinners on the shore Though the ocean is all the time devouring whatever appears on its surface and is never satisfied with its continual swallowing of the ships and men in the present case the ocean had cast these three sinners on the shore without killing them

(b) Who had jointly been responsible for the conspiracy against Prospero What is Prosperorsquos purpose behind all this

Three men Alonso Sebastian and Antonio had jointly

been responsible for the conspiracy against Prospero They had driven out Prospero form Milan Prosperorsquos purpose is to make these three sinners realize the wrong they had done He wants them to repent for their criminal deeds because repentance leads to self-esteem(c )What does Ariel (the harpy) tell Alonso and his companions when they take out their swords to attack him

Seeing them drawing their swords Ariel (harpy) tells them that he and his companions are the instruments of destiny and that it is not possible for human beings to do them any injury He says that the swords of human beings can not injure even a minute part of his feathers Their swords are as ineffective against him and his companions as against the wind or the water

(d) Give the explanatory meanings of the following expressions in the context of the above extract

(i)Never surfeited (ii) Belch up (iii) lsquomongst men

(i) Never surfeited never led to satisfaction

(ii) Belch up cast ashore(iii) lsquomongst men in human

society2

I and my fellows (Line 60-65)

Are ministers of Fate The elementsOf whom your swords are tempered may as wellWound the loud winds or with bemocked-at stabsKill the still-closing waters as diminishOne dowl thats in my plume

IMPORTANT PASSAGES EXPLAINED

The elements

(Line 61-66)Of whom your swords are tempered may

as wellWound the loud winds or with

bemocked-at stabs

(a) Who is lsquoIrsquo Who are his lsquofellowsrdquo

lsquoIrsquo is referred to Ariel in disguise of a harpy His lsquofellowsrsquo are other spirits serving Prospero the real Duke of Milan who has acquired supernatural powers after being banished from his Dukedom Prospero has settled in this uninhabited island

(b) What are the elements that have temperrsquod the swords Why will it not work against the speaker

The swords (of Alonso and his companions) are tempered by metal (steel) which is taken out of the earth and refined by

Kill the still-closing waters as diminishOne dowl thats in my plume My fellow

ministersAre like invulnerable

In these words Ariel reminds the King and his companions of the utter futility of drawing swords against himself and his fellows Ariel drives Alonso Antonio and Sebastian the three men of sin to desperation ndash a state in which men do violence to themselves They draw swords to strike Ariel But Ariel reminds them that he and the other spirits are the ministers of destiny and nothing can wound them The steel of which their swords are made of may cut the wind or water which being divided always closes up again Even supposing that such things may be possible it is quite impossible that their swords will cut one feather in their plume They are incapable of being wounded by any sword of man Hence it is foolish on their part to attempt to strike at Ariel and his fellow-spirits

For which foul deed

(Line 72-75)The powers delaying not forgetting

haveIncensed the seas and shores yea all the

creatures Against your peace

Ariel enters like a harpy and remaining invisible tells Alonso Sebastian and Antonio that he and other harpies are the agents of Destiny appointed to carry out her decrees He tells them that their punishment for the crime against Prospero which has been so long deferred is now to fall upon them He reminds them that they had expelled Prospero from Milan and set him and his innocent child adrift on the sea and that the sea had paid them back for their sin by the shipwreck and by the calamities they have suffered He tells them that the powers above which did not forget this mean treachery but only deferred the punishment have now engaged the seas and the shores and all living beings including him and his comrades against them The very elements and supernatural agency Ariel adds have taken up the avenging of their crime against Prospero

the action of fire It may cut the wind or water which being divided always closes up again

The sword will not work against the spirits and the harpy because they are the ministers of destiny and nothing can wound them nor it will cut a single feather in their plume

(c )What is the meaning of lsquodowlrsquo in the last line

The term lsquodowlrsquo means a filament or the smallest part of a feather In this context Ariel in disguise of harpy says that their sword cannot even damage the smallest filament of their (Arielrsquos and other spirits) feathers as they are incapable of being wounded by any sword of man

(d) What does the speaker remind the listeners about

Ariel in disguise of harpy reminds Alonso the King of Naples Sebastian Alonsorsquos brother and Antonio the present Duke of Milan and the treacherous brother of Prospero as they being three men of sin He even reminds them that their punishment for their crime against Prospero which has been so long deferred now falls upon them He reminds them that they have expelled Prospero from Milan and has set him along with his innocent infant daughter adrift on the sea So the sea has paid them back for their sin by their shipwreck and the calamities they have suffered since then The harpy rebukes Alonso of his sin that has incensed the Gods and has deprived him of his son as a punishment

(e) How do they respond

When Ariel in disguise of a harpy reminds Alonso Sebastian and Antonio of their past misdeeds and sin Alonso has a look of terror and confusion in his eyes He utters the words of sincere repentance wrung out of his conscience-stricken heart It appears to him that all the elements of nature the sea-waves the wind and the thunder proclaiming a loud voice in the name of Prospero and the crime Alonso has committed against him They are calling upon him to repent There is a deep storm raging in Alonsorsquos breast and the echoes of that storm are ringing in his ears like a clear note of wind-instrument A note of denunciation of Alonsorsquos crime leaves him much humbled and penitent and confirms his belief that his son is lost forever But Sebastian and Antonio shows some courage instead of repentance They wish to kill the spirits or devils if it appears

3

Of my instruction hast thou nothing bated (Line 85-93)

In what thou hast to say So with good life

And observation strange my meaner ministers

Their several kinds have done My high charms work

And these mine enemies are all knit upIn their distractions They now are in my

powerAnd in these fits I leave them while I visitYoung Ferdinand whom they suppose is

drownedAnd his and mine loved darling

Methought the billows spoke and (Line 96-99)

told me of itThe winds did sing it to me and the

thunderThat deep and dreadful organ-pipe

pronouncedThe name of Prosper It did bass my

trespass

These are the words of contrition coming from Alonso Ariel has driven him to a deep repentance for conspiring with Antonio against Prospero He now feels a sincere remorse It appears to him that all the elements of nature the sea-waves the wind and the thunder proclaimed with a loud voice the name of Prospero and the crime Alonso had committed against him They are calling upon him to repent There is a deep storm raging in Alonsorsquos breast and the echoes of that storm are ringing in his ears like the clear note of a wind-instrument

Comment These are the words of sincere repentance wrung out of the conscience-stricken heart of Alonso Alonso who is the lesser villain is the first to give way to remorse under the effect of Arielrsquos speech The words of Ariel seem to him to be the voice of conscience speaking to him He is driven to desperation a state in which he might do violence to his life

(a) Identify the speaker State the context

Prospero the ruler of the island is the speaker The famous banquet scene has been enacted very well Ariel and his junior spirits have played their roles excellently Prospero is glad to say words of praise for them(b) In what way the speakerrsquos instructions have been carried out

According to Prosperorsquos instructions a banquet was presented before the King of Naples and his companions when they were tired and hungry Just when they were preparing to eat the feast the banquet was suddenly removed by exercising supernatural powers All this was done by Ariel Prosperorsquos chief assistant and a powerful spirit

Ariel not only made the feast disappear but also delivered his speech blaming the King and his two companions for their past wicked deeds He warned them to repent for their misdeeds or suffer forever on that uninhabited island

(c) Who are referred to as lsquomeaner ministersrsquo What have they done

Prospero refers as lsquomeaner ministersrsquo to his other lesser spirits who were assisting Ariel in presenting a scene before the kingrsquos party They entered the scene to the accompaniment of music They assumed several strange shapes and brought in a banquet Then they danced about it with gentle actions of salutations thus inviting the King and others to eat the feast

These spirits play their role again when Ariel in the shape of a harpy quits the scene These shapes enter again and dancing with mocking gestures carry away the table

(d) Who are the speakerrsquos enemies What has happened to them

King of Naples Alonso his brother Sebastian and the present Duke of Milan Antonio (Prosperorsquos own brother) are Prosperorsquos enemies With the turn of events they have all been washed ashore on the island which is ruled by Prospero the great magician Actually this happened after the shipwreck caused by a storm which was raised by Prospero with the purpose of bringing these people to his island Prosperorsquos spirits have already confused and terrified these enemies and they are under Prosperorsquos control He can treat them as he likes

(e) What does he say about Ferdinand Explain what is meant by ldquohellip his and mine darlingrdquo

Prospero knows that Alonsorsquos son prince Ferdinand is alive though his father thinks that the prince has been drowned

Prospero refers to his daughter Miranda who is dear to him She is also very dear to Prince Ferdinand who has fallen in love with her They are waiting to be married soon for which they have received Prosperorsquos consent

4

ALONSO O it is monstrous monstrous (Line 95-102)

Methought the billows spoke and told me of it

The winds did sing it to me and the thunderThat deep and dreadful organ-

pipe pronouncedThe name of Prosper It did bass

my trespassTherefore my son ithrsquo ooze is

bedded andIll seek him deeper than eer

plummet soundedAnd with him there lie mudded

(a) In what way does Alonso express his horror when his conscience is awakened by Arielrsquos words

When Alonsorsquos conscience is awakened by Arielrsquos words he expresses his horror at what he has heard He gets the feeling that the waves of the ocean the wind and the loud thunder have spoken to him and uttered the name of Prospero Because of being reminded of his crime in a very loud and rough voice he comes to realize that he has lost his son for his past misdeeds

(b) What does Alonso imagine about his son What does Alonso want to do in his desperate state

Alonso imagines that his son is lying in the mud at the bottom of the sea He feels desperate that he wants to drown himself in the ocean deeper than the plumb-line has ever gone He wants to lie with his son at the bottom of the sea

(c) How do Sebastian and Antonio want to face the evil spirits

Sebastian says that he is not at all afraid of what the harpy has said and that he is prepared to fight any number of such monsters if they appear before him only one at a time Antonio says that he would support Sebastian in the fight against the fiendsyyy

(d) Why does Gonzalo ask Adrian to follow the three men

Gonzalo tells Adrian that all the three men namely Alonso Sebastian and Antonio are in a wild and reckless mood The thought of the heinous crime of which they are guilty has begun to torment their minds So he asks Adrian to follow those three men without loss of time and prevent them from doing anything which the turmoil in their minds might lead them to do

(e) What opinion do you form of Alonso from the above extract

Alonso who is the lesser villain is the first to give way to remorse under the effect of Arielrsquos speech The words of Ariel seem to him to be the voice of conscience speaking to him He is driven to desperation a state in which he might do violence to his life

Subject =Accounts

Ac-12 15420 topic-pL Appropriation ac

PROFIT AND LOSS APPROPRIATION ACCOUNT

MEANING AND PREPARATIONProfit and Loss Appropriation Account is merely an extension of the Profit and Loss Account of the firm The profit of the firm has to be distributed amongst the partners in their respective profit sharing ratio But before its distribution it needs to be adjusted All Adjustments like partnerrsquos salary partnerrsquos commission interest on capital interest on drawings etc are made in this account These adjustments will reduce the amount of profit for distribution This adjusted profit will be distributed amongst the partners in their profit sharing ratio To prepare it at first the balance of Profit and Loss Account is transferred to this account The journal entries for the preparation of Profit and Loss Appropriation Account are given below

1 for transfer of the balance of Profit and Loss Account to Profit and Loss Appropriation Account

(a) In case of Net Profit

Profit and Loss Ac helliphelliphelliphelliphellipDrTo Profit and Loss Appropriation Ac(Net Profit transferred to Profit and Loss Appropriation Ac)

(b)In case of Net Loss

Profit and Loss Appropriation Achelliphelliphellip DrTo Profit and Loss Ac(Net Loss transferred to Profit and Loss Appropriation Ac)

2 for Interest on Capital

For transferring on Interest on CapitalProfit and Loss Appropriation Achelliphelliphellip DrTo Interest on Capital Ac(Interest on capital transferred to Profit amp Loss Appropriation Ac)

3 for Interest on Drawings

For transferring Interest on Drawings Interest on Drawings Achelliphelliphelliphelliphelliphellip DrTo Profit and Loss Appropriation Ac(Interest on drawing transferred to Profit amp Loss Appropriation Ac)

4 For Partnerrsquos SalaryFor transfer of partnerrsquos SalaryProfit and Loss Appropriation Achelliphellip DrTo Salary Ac(Salary transferred to profit amp Loss Appropriation Ac)

5 For Partnerrsquos CommissionFor transferring commissionProfit and Loss Appropriation Achelliphelliphellip DrTo Commission Ac(Commission transferred to Profit and Loss Appropriation Ac)

6 For Transfer of agreed amount to General ReserveProfit and Loss Appropriation Ac helliphellipDrTo General Reserve Ac(Transfer to General Reserve)

7 for share of Profit or Loss appropriation(a) If ProfitProfit and Loss Appropriation Achelliphellip DrTo Partnerrsquos CapitalCurrent Ac(Profit transferred to capitalcurrent Ac)(b) If LossPartnerrsquos Capital Current Achelliphelliphelliphellip DrTo Profit and Loss Appropriation Ac(Loss transferred to capitalcurrent Ac)

THE FORMAT OF PROFIT AND LOSS APPROPRIATION

Profit and Loss Appropriation Account for the year endedhelliphelliphelliphellip

Particulars Amount Particulars Amount

To PL Ac (loss) By pL Ac (profit)

To Interest on capital BY Interest on drawings

To partner`s commission by Partner`s capital Ac ( loss)

To Partner`s salary To Interest on partner`s loan To General Reserve To Partner`s Capital AC (Profit)

Subject= Economics

MOVEMENT ALONG THE DEMAND CURVE (CHANGE IN QUANTITY DEMANDED)In law of demand you have already studied the inverse relationship between price and quantity demanded When quantity demanded of a commodity changes due to change in its price keeping other factors constant it is called change in quantity demanded It is graphically expressed as a movement along the same demand curve There can be either a downward movement or an upward movement along the same demand curve Upward movement along the same demand curve is called contraction of demand or decrease in quantity demanded and downward movement along the same demand curve is known as expansion of demand or increase in quantity demanded

Extention of demandd

price (rs)p A

B Extentionp1 d

Q Q1

Quantity demanded ( in units)

Contraction of demandd

p2 Ccontraction

p APrice (Rs)

d

Q2 Q

Quantity demanded (in units)

Explanation of movement of demand A fall in price from OP to OP1 leads to increase in quantity demanded from OQ to OQ1 (expansion of demand) resulting in a downward movement from point A to point B along the same demand curve DD When Price rises from OP to OP2 quantity demanded falls from OQ to OQ2 (contraction of demand) leading to an upward movement from point A to point C along the same demand curve DD

  • Activity Series of Metals
    • Drawbacks of Rutherfordrsquos model of atom
      • Electromagnetic radiations
      • Properties of electromagnetic radiations
      • Characteristics of electromagnetic radiations
        • Plancks Quantum Theory-
        • Photoelectric effect
          • Intext Questions
            • Logic gates
            • Digital systems are said to be constructed by using logic gates These gates are the AND OR NOT NAND NOR EXOR and EXNOR gates The basic operations are described below with the aid of truth tables
            • AND gate
            • Example
Page 37:  · Web viewSubject. Topic. Summary. Execution. English 1 . Chapter 1 naming words . Page 8. Write the names of these pictures:- Person:-1. father. 2.Firefighter 3.doctor 4 ...

Fig COMPONENTS OF MACRO ENVIRONMENTPolitical science

Introduction to political science

Comparative politics and itrsquos scope Comparative politics is the second major dimension of political scienceIt is also a very vast area of study and a very large number of political scientists even treat it as an autonomous area of study within the board ambit of political scienceScope of comparative politics-

1 All political structures -Comparative politics includes the study of all structures formalnon formal governmental and extra governmental which are directly or indirectly involved in politics in all the countries of the world

2 Functional studies- Comparative politics seeks to study politics less from the point of view of the legal institutions in terms of their powers and move from the point of view of their functions which constitute the political process and their actual Operation in the environment

3 Study of political behaviour- Another important part of its scope is the study of the actual behaviour of the people in the process of politics

4 Study of similarities and differences- comparative politics also undertakesan analysis of the similarities and differences among political process and functions

5 Study of all political systems -comparative politics seeks to analyse the actual behaviour and performance of all political systems western as well as non western

6 Study of the environment and infrastructure of politics-The study of politics demands a study of the psychological sociological economic and anthropological environment in fact the social environment as a whole in which each political system operates

7 Study of political culture- political culture is composed of attitudesbeliefs emotions and values of a society that relate to the political system or politics

8 Study of political participation- Political participation is a universal processThe only difference is that while in some states it is limited in others it is wider

9 Study of political process- political

Answer the following questions-

What is comparative politics

What are the scope of comparative politics

Homework- learn

processes like decision makingpolicy making judicial process leadership recruitment process and others are always at work in all political systems

The scope of comparative politics is very comprehensive It includes everything that falls within the area of political activity and political process

History CAMBRIDGE VIEW ABOUT

THE PARTITION

AND REFUTATION

OF CAMBRIDGE

VIEW

Cambridge view about the Partition The Cambridge school of historians have interpreted that opposition to partition scheme was made entirely by the elitist groups They hold the view that Lord Curzon planned to partition the Bengal for administrative purposeREFUTATION OFCAMBRIDGE VIEW The Rationalist historians have rejected the interpretations of the Cambridge School of historians on various grounds

1 QUESTION State different views of historians regarding Partition of Bengal

ANSWER Cambridge historians believed that Lord Curzon partitioned Bengal for administrative reasons only and not for the political motive The Middle class elitist group protested because of their petty interest The Hindu zamindars protested as they have to spend more money for managing their estatesThe lawyers of Calcutta High court feared to lose their clientBut according to the nationalist Historians was-

2- The ultimate object of Lord Curzon was to crush the unity of Bengal politicians

3- If Bengal becomes a separate province Bengali speaking 16 million people of western part would become minority under Hindi speaking people of Bihar and Oriya speaking people of Orissa

4- The bureaucrats expected that the protest movement would die down quickly

5- Lord Curzon used the Muslim community in his political game

6- Idealism had great contribution in the protest against partition

7- The people of the every section of society were affected by the partition of Bengal

Computer Science

Numbers Convertion of dcimal number to octal numberThe decimal numeral system is the standard system for denoting integer and non-integer numbers It is the extension to non-integer numbers of the Hindu-Arabic numeral system For writing numbers the decimal system uses ten decimal digits a decimal mark and for negative numbers a minus sign - The decimal digits are 0 1 2 3 4 5 6 7 8 9 the decimal separator is the dot in many countries

The octal numeral system or oct for short is the base-8 number system and uses the digits 0 to 7 Octal is sometimes used in computing instead of hexadecimal perhaps most often in modern times in conjunction with file

permissions under Unix systems It has the advantage of not requiring any extra symbols as digits It is also used for digital displays

Follow these steps to convert a decimal number into octal form

1 Divide the decimal number by 82 Get the integer quotient for the next iteration (if the number will not divide equally by 8 then round down the

result to the nearest whole number)3 Keep a note of the remainder it should be between 0 and 74 Repeat the steps until the quotient is equal to 05 Write out all the remainders from bottom to top This is the solution

For example if the given decimal number is 8453

Division Quotient Remainder

8453 8 1056 5

1056 8 132 0

132 8 16 4

16 8 2 0

2 8 0 2

Then the octal solution is 20405

Subject Eng Literature (The Tempest ndash William Shakespeare) Topic Act I Scene 1 Lines 33 to 67 (End of scene) Date 16th April 2020 (4th Period)

[Students should read the original play and also the paraphrase given in the school prescribed textbook]Summary Questions amp Answers

[SUMMARY OF THE ENTIRE SCENE]

o The play starts with the scene of a severe storm at sea Alonso (King of Naples) Sebastian (Alonsorsquos brother) Ferdinand (Alonsorsquos son) Gonzalo Antonio (the usurping Duke of Milan) are in a ship in the midst of the storm

o The mariners are trying their best to control the vessel from running aground and are totally following the orders of their Master the Boatswain They have scant success

o The mariners become extremely unhappy and annoyed when most of the passengers arrive on the deck thereby hampering their effort to save the ship There is serious confrontation between them and the passengers who are part of the Kingrsquos entourage

o The mariners could not save the ship

SUMMING-UP

(i) Vivid description of the scene which gives a realistic description of terror and confusion of a tropical storm

(ii) Shows Shakespearersquos accuracy of knowledge in describing the naval operations and also matters of seamanship

(1) GONZALO Ill warrant him for drowning (L 45-57)

though the ship were no stronger than a nutshell and as leaky as an unstanched

wenchBOATSWAIN Lay her a-hold a-hold Set her two courses Off to

sea again lay her offMARINERS All lost To prayers to prayers All lostBOATSWAIN What must our mouths be coldGONZALO The king and prince at prayers Lets assist them

For our case is theirsSEBASTIAN Im out of patienceANTONIO We are merely cheated of our lives by drunkards

This wide-chopped rascal - would thou mightst lie drowning the washing of ten tides

(a) What does Antonio say at the insolent manners of the boatswain just before the given passage

Being irritated at the insolent manners of the boatswain just before the given extract Antonio the Duke of Milan calls him a worthless dog son of a woman without any morals an arrogant and disrespectful noisemaker He says that the boatswain deserved to be hanged(b) What statement does Gonzalo repeat about the boatswain

Gonzalo shows his faith that the boatswain is not destined to die by drowning He is destined to be hanged and nothing can alter this decree of destiny He says that even if the ship was as frail as a nutshell the boatswain could not be drowned for his destiny was to be hanged(c) What do the passengers do when they have lost all hope of their survival

When the passengers have lost all hope of survival they take

(iii) The opening scene justifies the title ndash The Tempest

UNANSWERED QUESTIONS

(i) The King always travels with his entire fleet including his soldiers Where were the other ships

(ii) Why was the ship in that area Where was it coming from or going where

(iii) The ship broke apart What happened to those who were in the ship

(We shall get the answer to the above questions as the play progresses)

leave of life with fervent prayers The mariners take their last hearty drink and are ready for death(d) What blame does Antonio put upon the mariners and the boatswain Antonio rebukes the mariners that these drunkards have brought them to the present crisis by neglecting their duties He blames them saying that they are going to lose their lives entirely for the negligence of the boatswain and his fellows(e) What does Antonio say while cursing the boatswain

Antonio gives vent to his wrath upon the boatswain in particular He calls the boatswain a wide-mouthed rascal who deserves to be hanged on the sea-shore at low water mark so that ten tides might wash over his body and take out of him all the liquor that he has been drinking

Class XIISubject Topic Summary ExecutionHistory Topic

1 1935 ACT AND WORKING OF PROVINCIAL AUTONOMYCONGREE AND OTHER MINISTERSSUB TOPIC GOVERNMENT OF INDIA ACT1935

Government of India Act 1935 This act established a lsquoFederation of Indiarsquo made of British Indian provinces and Indian states and provided for autonomy with a government responsible to the elected legislature in every provinceThis act introduced abolition of Diarchy at provinces The entire provincial administration was introduced to the responsible ministers who were controlled and removed by the provincial legislature The provincial autonomy means two things First The provincial governments were wholly responsible to the provincial legislature Secondly Provinces were free from outside control and interference in the large number of matters The act divided the powers between the centre and provinces in terms of three lists- Federal list( for centre) Provincial list (for province) and concurrent list (for both) Residuary powers were given to the viceroy In the election under the government of India Act the Congress swept the poll the mandate of the people came in favour of the congress so far as general Hindu seats were concerned The Congress did not get a single Muslim seates in Bombay CP UP Sind and BengalIn five provinces Congress had yhe clear majority In BengalNWFPAssam and Bombay Congress emerged as a single largest partyOn the other side the performance of the Muslim League was badThus the Congress formed ministers in 7 provinces out of 11 provinces Coalition ministry was also formed in two other provincesOnly BENGAL AND Punjab had non- congress ministries

1 QUESTION What was the main change introduced by the Government of India ActANSWER a) The Act gave more

autonomy to the provinces b) Diarchy was abolished at the

provincial levelsc) The Governor was the head of

the executived) There was a council of

ministers to advise him The ministers were responsible to the provincial legislatures who controlled them The legislature could also remove the ministers

e) The Governors still retained special reserve powers

2 QUESTION Why did the federal scheme introduced by the Government of India Act 1935 never come into operation

ANSWER The Federal structure of the Government of India was to be composed with the Governor General and Council of ministers The Federal legislature was to be Bicameral legislature- The council of states and the House of Assembly The ministers were to be chosen by the Governor general and they were to hold the office during his pleasure

The provinces of British India would have to join the federation but this was not compulsory for the princely states

This federation never materialised because of the lack of support from the required number of

princely statesThis act was refused and

rejected by the princes the Congress and the Muslim League

Thus both Congress and the League participated in the election of 1937 Thus the federal part was never introduced but the provincial part was put into operations

Bengali 2nd

Language

াচেরর পরাথCনা(কহিতা )

াচেরর পরাথCনা কহিতাটি কহি (ঙখ দেঘাচে4র দো আচো য কহিতায় াচেরর পতর হমায়ন কঠিন দেরাচেগ আxানত ার ঈশবর া আললার কাচেছ পরাথCনা কচেরচেছন তার পচেতরর ীন হিফহিরচেয় হিচেত এই কহিতায় ার পচেতরর ীন হিভbা দেচেয়চেছন ারার এমনহিক হিনচের ীন হিসCচেনর হিহিনমচেয় হিতহিন তার দেছচের ীন হিফচের দেপচেত দেচেয়চেছন তার দেছচের এই দেরাচেগর ন য হিতহিন হিনচেচেকই ায়ী কচেরচেছন তার হিনচের করা পাপচেকই হিতহিন ায়ী কচেরচেছন এছাা রানৈনহিতক ও আথCসামাহিক অসথার কথা তচে ধরা চেয়চেছ এই কহিতায় ার তার হিনচের পাপ কমCচেকই ায়ী কচেরচেছ ার অন যায় ভাচে দেপহি((হিকতর মাধ যচেম অপররা য কচেরচেছ আর এই অন যায় কাচের ন যই তার পহিরাচের হিপযCয় এচেসচেছ দে এক পরকার মানহিক নধন ইহিতাচেসর ার হিপতা চেয় সবাভাহিকভাচে ভাচোাসা দে মমতা দেথচেক মকত চেত পাচেরনহিন তাই হিপতা চেয় আললা া ভগাচেনর কাচেছ পতর হমায়চেনর পরানহিভbা দেচেয়চেছন ার আললা া ভগাচেনর কাচেছ াহিনচেয়চেছন তার হিনচের ীন হিসCন হিচেত হিতহিন রাী তার হিহিনমচেয় পচেতরর ীন হিফচের দেপচেত দেচেয়চেছন াচেরর হিপতসভ হিচেকর কথা এই কহিতায় ফটিচেয় দেতাা চেয়চেছ হিপতা পচেতরর হিরাহিরত মান নধচেনর কথা তচে ধরা চেয়চেছ

হিচে(4 হিকছ াইচেনর তাৎপযC১) ldquoদেকাথায় দেগ ওর সবচছয দেৌন দেকাথায় কচেরায় দেগাপন bয়ldquoউততর) াচেরর পতর হমায়ন কঠিন দেরাচেগ অসসথ তাই তার দেযৌন াহিরচেয় যাচেচছ এই দেরাচেগ তাচেক দেগাপচেন কচেরকচের াচেচছ তার সক (হিকত ধীচের ধীচের bয় চেচছ তাই হিপতা চেয় ার আললার কাচেছ হমায়চেনর পরান হিভbা দেচেয়চেছন২) ldquoাগাও (চেরর পরাচেনত পরানতচের ধসর (ন দেযর আান গানldquoউততর) াচেরর পতর হমায়ন কঠিন দেরাচেগ আxানত তাই ার আ দে(াচেক মমCাত (চেরর পচেথ পরানতচের আান গান ধবহিনত দোক দেসই আান গান আললার কাচেছ দেযন চে যায় আললা দেযন এই আহিতC শচেন পচেতরর ীন হিফহিরচেয় দেয় ৩)ldquoনাহিক এই (রীচেরর পাচেপর ীানচেত দেকানই তরারণ দেনই ভহি4চেতরldquoউততর) হমায়চেনর অসসথতার ন য ার হিনচেচেকই ায়ী কচেরচেছন কারন ার অচেনক রা য অন যায় ভাচে কচেরচেছ তাই তার এই পাপ কাচের ন য তার ঘচের আ হিপ এচেসচেছ এই অন যায় কাচের ন য তার মহিকত দেনই তাই ার আললার কাচেছ এই পাপ কাচেযCর ন য bমা পরাথM

Hindi 2ndlang

-ासी(जयशकर परसा-)

-ासी जयशकर परसा- की एक ऐसी कहानी ह जिजसम भारतीय ससकनित और राषटरीयता का सवरगजीतहोता ह इस कहानी म इरावती एक निहद कनया ह जिजस मलअचछो न मलतान की लट म पकडा और -ासी बना दि-या उस 500 दि-न -कर काशी क एक महाजन न खरी-ा दसरी -ासी निफरोजा ह वह गलाम ह निफरोजा को छडान क कतिलए अहम- को 1000 सोन क कतिसकक भजन थ जो अभी तक नही आए थ राजा साहब कठोर होत हए भी निफरोजा को निबना धनराकतिश क कतिलए उस म कर -त ह वनिफरोजा को अहम- को समझान की बात कहत हकहानी क अत म हम -खत ह निक इरा वती और जाटो क सर-ार बलराज का मिमलन होता हअहम- को यa म मार दि-या जाता ह वहा निफरोजा की परसननता की समामिध बनती ह वहा एक फल चढती ह और डीजल आती ह निफरोजा उस समामिध की आजीवन -ासी बनी रहती हलखक अपन उददशय अथात -ास परथा पर परकाश डालन और इस परथा क कारण होन वाल -ातो क दखो को दि-खान म पणता सफल हए ह

helliphellipContinue to next

Biology Reproductio Today we will discuss about vegetative Q1 Name some vegetative propagules

n in Organisms

propagation of plants The process of multiplication in which fragments of plant body function as propagule and develop into new individual is called vegetative propagation The units of such propagation are runner rhizome tuber bulb etc

and the speciesinvolvedVegetative propagules

Parts involved

Bulb StemBulbil BulbilRhizome Stem Runner Stem Tuber Stem Offset Stem Leaf buds Leaves Suckers Stem

Corns Stem stolon

Q2 State advantages of vegetative propagation

i) Rapid methodii) Sure and easy methodiii) Useful in plants that cannot

produce viable seeds or long seed dormancy

iv) Maintains purity of raceQ 3 Banana fruit is said to be parthenocarpic where as turkey is said to be parthenogenetic WhyBanana develops without fertilization from an unfertilized ovary thus is parthenocarpicIn turkey the ovum or female gamete developinto a new chick without fertilization thus isparthgenetic

Q4 Why is water hyacinth is called as a ldquoTerror of Bengalrdquo Water hyacinth can

propagatevegetatively all over the water body in a short per short period of time This resulted increased biochemicaloxygen oxygen demand of water body causing mortalityof fishes It is very difficult to get rid off them Thus known as terror of Bengal

Chemistry

Solid state GENERAL CHARACTERISTICS OF SOLID STATEIn nature the particular state of matter is governed by two opposing forces at given set of temperature and pressure These forces are intermolecular force of attraction and thermal energy If intermolecular force of attraction is high as compared to thermal energy particles remains in closest position

Intext QuestionsQ1 Classify the following solids as crystalline and amorphous Sodium chloride quartz glass quartz rubber polyvinyl chloride Teflon

A1 Crystalline

and hence very less movement in particles is observed In this case solid state is the preferred state of matter

Let us revise the general characteristics of solid

i) Fixed mass volume and shape

ii) Strong intermolecular force of attraction

iii) Least intermolecular space

iv) Fixed position of constituent particles

v) Incompressible and rigid

Q2 what type of interactions hold the molecules together in a polar molecular solid[CBSE 2010]A2 The molecules in a solid are held together by van der Waals forces The term van der Waals forces include hydrogen bonding dipole-dipole attraction and London dispersion forces All molecules experience London dispersion forces In addition polar molecules can also experience dipole-dipole interactions So the interactions that holds the molecule together in polar molecular solid are London dispersion force and dipole-dipole interactionsQ3 Write a feature that will distinguish a metallic solid from an ionic solid [CBSE 2010]A3 Metals are malleable and ductile whereas ionic solid are hard and brittle Metallic solid has typical metallic lustre But ionic solid looks dullQ4 Write a point of distinction between a metallic solid and an ionic solid other than metallic lustre [CBSE 2012]A4 Metals are malleable and ductile whereas ionic solid are hard and brittleQ5 Write a distinguish feature of metallic solid [CBSE 2010]A5 The force of attraction in

solid Sodium chloride Quartz Amorphous solid Quartz glass rubber polyvinyl chloride Teflon Q2 why glass is considered as super cooled liquidA2 Glass shows the tendency to flow at slower rate like liquid Hence they considered as super cooled liquidQ3 why the window glass of old buildings show milky appearance with timeA3 Glass is an amorphous solid Amorphous solid has the tendency to develop some crystalline character on heating Due to heating in day over the number of years glass acquires some crystalline character and show milky appearanceQ4 why the glass panes fixed to window or doors of old building become slightly thicker at bottomA4 Glass is super cooled liquid It has the tendency to flow down very slowly Due to this glass pane becomes thicker at the bottom over the timeQ5 Sodium chloride is a crystalline solid It shows the same value of refractive index along all the direction TrueFalse Give reasonA5 FalseCrystalline solid shows anisotropy in properties That is it shows different values for the given physical property in different direction All the crystalline solids show anisotropy in refractive index Therefore sodium chloride will show different values of refractive index on different directions

Q6 Crystalline solid are anisotropic in nature What does this statement means

between the constituent particles is special kind of electrostatic attraction That is the attraction of positively charged kernel with sea of delocalized electronsQ6 which group of solid is electrical conductor as well as malleable and ductile [CBSE 2013]A6 Metallic solidQ7 why graphite is good conductor of electricity although it is a network (covalent solid)A7 The exceptional property of graphite is due to its typical structure In graphite each carbon is covalently bonded with 3 atoms in same layer The fourth valence electron of each atom is free to move in between different layersThis free electron makes the graphite a good conductor of electricity

[CBSE 2011]A6 Anisotropy is defined asrdquo Difference in properties when measured along different axis or from different directionsrdquo Crystalline solid show different values of some of the physical properties like electrical resistance refractive index etcwhen measured along the different directions The anisotropy in crystalline solid arises due to the different arrangement of particles in different directions

Math Function Composition of functions Think of an industrial plant that produce bottles of cold drinks first there is the operation (or function) f that puts the cold drink inside the bottle followed by the opeartion g that close the bottle with the capThis leads to the following definitionDefinition Let f A rarr B and g B rarr C be two functions Then the composition of f and g denoted by gof is defined as the function gof A rarr C given by gof(x) = g(f (x)) forall x isinA

Definition A function f X rarr Y is defined to be invertible if there exists a function g Y rarr X such that gof = IX and fog = IY The function g is called the inverse of f and is denoted by f -1

Thus if f is invertible then f must be one-one and onto and conversely if f is one-one and onto then f must be invertible This fact significantly helps for proving a function f to be invertible by showing that f is one-one and onto specially when the actual inverse of f is not to be determined

Example 1 Let f 2 3 4 5 rarr 3 4 5 9 and g 3 4 5 9 rarr 7 11 15 be functions defined as f(2) = 3 f(3) = 4 f(4) = f(5) = 5 and g (3) = g (4) = 7 and g (5) = g (9) = 11 Find gofSolution We have gof(2) = g (f(2)) = g (3) = 7 gof(3) = g (f(3)) = g (4) = 7gof(4) = g (f(4)) = g (5) = 11 and gof(5) = g (5) = 11Example 2 Find gof and fog if f R rarr R and g R rarr R are given by f(x) = cos x and g (x) = 3x2 Show that gof ne fogSolution We have gof(x) = g(f(x))=g(cosx) = 3 (cos x)2

= 3 cos2 x Similarly fog(x)=f(g (x))= f(3x2)= cos (3x2) Note that 3cos2 x ne cos 3x2 for x = 0 Hence gof ne fogExample 3 Show that if f A rarr B and g B rarr C are onto then gof A rarr C is also ontoSolution Given an arbitrary element z isin C there exists a pre-image y of z under g such that g (y) = z since g is onto Further for y isin B there exists an element x in A with f(x) = y since f is onto Therefore gof(x) = g (f(x)) = g (y) = z showing that gof is onto Example 4 Let Y = n2 n isin N sub N Consider f N rarr Y as f(n) = n2 Show that

f is invertible Find the inverse of fSolution An arbitrary element y in Y is of the form n2 for some n isin N This implies that n =radicy This gives a function g Y rarr N defined by g (y) =radicy Nowgof (n) = g (n2)=radicn2 = n and fog (y) =f(radicy) = (radicy) 2 y which shows that gof=IN and fog= IY Hence f is invertible with f -1 = g

Political Science

Constitution of India-The Preamble

Summary

Objective of the state-To secure equality of status and of opportunity To promote fraternity among all the citizens To assure the dignity of the individuals and Unity and integrity of the nation

Justice-Justice stands for rule of law absence of arbitrariness and a system of equal rights freedom and opportunities for all in a society India seeks social economic and political justice to ensure equality to its citizens

Liberty-Liberty implies the absence of restraints or domination on the activities of an individual such as freedom from slavery serfdom imprisonment despotism etc The Preamble provides for the liberty of thought expression belief faith and worship

Equality-Equality means the absence of privileges or discrimination against any section of the society The Preamble provides for equality of status and opportunity to all the people of the country

Fraternity-The Preamble declares that fraternity has to assure two thingsmdashthe dignity of the individual and the unity and

Execution

Answer the following questions-

Short notes-1 Equality2 Fraternity3 Justice4 Liberty

Homework-Learn

integrity of the nation The word integrity has been added to the Preamble by the 42nd Constitutional Amendment (1976)

Business studies

Human resource management (chapter 1)

On the day of 1504 2020 I have discussed with you the managerial functions and procurement functions of HRM

Today weare going to discuss about the development function integration functions and maintenance function

Development functions-HRM improves the knowledge skills attitude and values of employees so that they the present and future jobs more effectively it includes

1) Development functions of HRM

a) Performance appraisal = It implies systematic evaluation of employees with respect to their performance on the job and their potential for development

b) Training =It is the process by which employees learn knowledge skills and attitudes to achieve organisational and personal goals

c) Executive development = It is the process of developing managerial talent through appropriate program

2) Integration functionsa) HRM reconcile the goals of

organisation with those of its members through integrating function

b) HRM tries to motivate employees to various financial and non financial incentives provided in job specification etc

3) Maintenance functiona) HRM promote and protect the

physical and mental health of employees by providing several types of benefits like housing medical aid etc

b) It Promote Social security measures to employees by providing provident fund pension gratuity maternity benefits

SubjectCOMMERCE

Topic

BUSINESSENVIRONMENT

Summary

Now quickly let us revise the earlier points that we have already done in the last class and let us proceed with the other topics that are there in the chapter

Firstly we will recall the internal and external factors of micro environment and then we

Execution 3 What do you mean by internal factors

in micro environmentAnswerInternal factors refer to all the factors existing within a business firm The internal factors are considered controllable because the enterprise has control over these factors

Development FunctionsPerformance AppraisalTrainingExecution Development

shall proceed in details

Meaning and list of internal and external factors

aInternal factorsInternal factors refer to all the factors existing within a business firm The internal factors are considered controllable because the enterprise has control over these factorsFor an example a company can alter its organization structure policies programmes employees physical facilities and marketing mix to suit the changes in the environmentList of internal factors areCorporate culture mission and objectives top management organizations structure company image and brand equity company resources

b External factorsExternal factors refer to those individual and groups and agencies with which a particular business organization comes into direct and frequent contact in the course of its functioningThese individuals and groups are known as STAKEHOLDERS because they have a stake (financial interest ) in the working and performance of the particular business List of external forces (stakeholders)Customers competitors investors suppliersmiddlemen (marketing intermediaries)financers publics

customers

suppliersfinancers

For an example a company can alter its organization structure policies programmes employees physical facilities and marketing mix to suit the changes in the environment

4 What do you mean by external factors in micro environment

AnswerExternal factors refer to those individual and groups and agencies with which a particular business organization comes into direct and frequent contact in the course of its functioningThese individuals and groups are known as STAKEHOLDERS because they have a stake (financial interest) in the working and performance of the particular business

3Who are stakeholdersSTAKEHOLDERS are individuals and groups who have a stake (financial interest ) in the working and performance of the particular business 4Discuss the internal factors in briefa Corporate CultureThe values beliefs and attitudes of the founders and top management of the company exercise a strong influence on what the cmpaany stands for how it does things and what it considers importantbMission and objectivesThe business philosophy and purpose of a comoany guide it prioritiesbusiness strategiesproduct market scope and development scope

cTop management structurethe composition of board of directors the degree of professionalization of management and the organizational structure of a company have important bearing on its business decisions

dPower structureThe internal power relationship between the board of directors and the chief executive is an important factor

e Company image and brand equityThe image and brand equity of the company play a significant role in raising finance forming alliance choosing dealers and suppliers launching new products entering foreign markets

5 What is Macro environmentAnswerMacro environment refers to the general

competitors

middlemen

publics

Fig STAKEHOLDERS OF A COMPANY

Apart from micro environment the other main dimension of business environment isMacro environment Macro environment refers to the general environment or remote environment within which a business firm and forces in its micro environment operateA company does not directly or regularly interact with the micro environmentTherefore macro environment is also known as indirect action EnvironmentThe macro environment forces are less controllable than the micro forces

Macro environment consists of the following components

POLITICAL AND LEGAL ENVIRONMENT

ECONOMIC SOCIAL AND ENVIRONMENT

CULTURAL

ENVIRONMENT

TECHNOLOGICAL ENVIRONMENT

Fig COMPONENTS OF MACRO ENVIRONMENT

environment or remote environment within which a business firm and forces in its micro environment operateA company does not directly or regularly interact with the micro environmentTherefore macro environment is also known as indirect action EnvironmentThe macro environment forces are less controllable than the micro forces 6 What are the components of macro environmenta Political and legal environmentb Economic environmentc Social and cultural environmentd Technological environment

Computer Science

Logic gates

Digital systems are said to be constructed by using logic gates These gates are the AND OR NOT NAND NOR EXOR and EXNOR

BUSINESS FIRM

gates The basic operations are described below with the aid of truth tables

AND gate

The AND gate is an electronic circuit that gives a high output (1) only if all its inputs are high A dot () is used to show the AND operation ie AB Bear in mind that this dot is sometimes omitted ie ABOR gate

The OR gate is an electronic circuit that gives a high output (1) if one or more of its inputs are high A plus (+) is used to show the OR operationNOT gate

The NOT gate is an electronic circuit that produces an inverted version of the input at its output It is also known as an inverter If the input variable is A the inverted output is known as NOT A This is also shown as A or A with a bar over the top as shown at the outputs The diagrams below show two ways that the NAND logic gate can be configured to produce a NOT gate It can also be done using NOR logic gates in the same way

NAND gate

This is a NOT-AND gate which is equal to an AND gate followed by a NOT gate The outputs of all NAND gates are high if any of the inputs are low The symbol is an AND gate with a small circle on the output The small circle represents inversion

NOR gate

This is a NOT-OR gate which is equal to an OR gate followed by a NOT gate The outputs of all NOR gates are low if any of the inputs are highThe symbol is an OR gate with a small circle on the output The small circle represents inversion

EXOR gate

The Exclusive-OR gate is a circuit which will give a high output if either but not both of its two inputs are high An encircled plus sign ( ) is used to show the EOR operation

EXNOR gate

The Exclusive-NOR gate circuit does the opposite to the EOR gate It will give a low output if either but not both of its two inputs are high The symbol is an EXOR gate with a small circle on the output The small circle represents inversion The NAND and NOR gates are called universal functions since with either one the AND and OR functions and NOT can be generated

Note A function in sum of products form can be implemented using NAND gates by replacing all AND and OR gates by NAND gates A function in product of sums form can be implemented using NOR gates by replacing all AND and OR gates by NOR gates

Logic gate symbols

Table 2 is a summary truth table of the inputoutput combinations for the NOT gate together with all possible inputoutput combinations for the other gate functions Also note that a truth table with n inputs has 2n rows You can compare the outputs of different gates

Logic gates representation using the Truth table

Example

A NAND gate can be used as a NOT gate using either of the following wiring configurations

Subject Eng Literature (The Tempest ndash William Shakespeare) Topic Act III Scene 3 Lines 53 to 110 (End of the scene) Date 16th April 2020 (2nd Period)

[Students should read the original play and also the paraphrase given in the school prescribed textbook]Summary Questions amp Answers

o Seeing this strange scene all are inclined to believe the tales told by travelers that there truly are ldquounicornsrdquo and ldquothe phoenixrsquo thronerdquo

o As they are about to sit down to the feast the banquet is snatched away by a harpy (Ariel disguised) A spiritrsquos voice (Arielrsquos voice) denounces Alonso Sebastian and Antonio with particular

1 ARIEL You are three men of sin whom Destiny

(Line 53-58)That hath to instrument this

lower world And what is int the never-surfeited sea

Hath caused to belch up you and on this island

Where man doth not inhabit you rsquomongst men

Being most unfit to live I have made you mad

reference to their crime in expelling Prospero from Milan They have not received any punishment for their deed earlier but the time for their punishment has arrived Upon Alonso it pronounces ldquolingering perdition worse than deathrdquo from which there is no remedy except through sincere repentance Ariel then vanishes in thunder and the shapes enter again and carry away the table

o Prospero watching invisibly is very pleased with the performance of Ariel and his (Prosperorsquos) ldquomeaner ministersrdquo All his enemies are now in his power and are in a fit of desperation He then leaves them and goes to see how Ferdinand and Miranda are getting on

o Alonso is now much humbled and penitent with the after effect of the spiritrsquos denunciation of his crimes He believes that his son is lost forever After this all disperse being stricken mad by the speech of the spirit

o Gonzalo fearing that they may do violence to themselves or to one another follows them and bid others to follow

(a) To whom does Ariel disguised as a harpy call the three sinners What game did Fate of Destiny play with

them

The three sinners called by Ariel are Alonso Sebastian and Antonio It was Destiny which had caused the ocean to cast the three sinners on the shore Though the ocean is all the time devouring whatever appears on its surface and is never satisfied with its continual swallowing of the ships and men in the present case the ocean had cast these three sinners on the shore without killing them

(b) Who had jointly been responsible for the conspiracy against Prospero What is Prosperorsquos purpose behind all this

Three men Alonso Sebastian and Antonio had jointly

been responsible for the conspiracy against Prospero They had driven out Prospero form Milan Prosperorsquos purpose is to make these three sinners realize the wrong they had done He wants them to repent for their criminal deeds because repentance leads to self-esteem(c )What does Ariel (the harpy) tell Alonso and his companions when they take out their swords to attack him

Seeing them drawing their swords Ariel (harpy) tells them that he and his companions are the instruments of destiny and that it is not possible for human beings to do them any injury He says that the swords of human beings can not injure even a minute part of his feathers Their swords are as ineffective against him and his companions as against the wind or the water

(d) Give the explanatory meanings of the following expressions in the context of the above extract

(i)Never surfeited (ii) Belch up (iii) lsquomongst men

(i) Never surfeited never led to satisfaction

(ii) Belch up cast ashore(iii) lsquomongst men in human

society2

I and my fellows (Line 60-65)

Are ministers of Fate The elementsOf whom your swords are tempered may as wellWound the loud winds or with bemocked-at stabsKill the still-closing waters as diminishOne dowl thats in my plume

IMPORTANT PASSAGES EXPLAINED

The elements

(Line 61-66)Of whom your swords are tempered may

as wellWound the loud winds or with

bemocked-at stabs

(a) Who is lsquoIrsquo Who are his lsquofellowsrdquo

lsquoIrsquo is referred to Ariel in disguise of a harpy His lsquofellowsrsquo are other spirits serving Prospero the real Duke of Milan who has acquired supernatural powers after being banished from his Dukedom Prospero has settled in this uninhabited island

(b) What are the elements that have temperrsquod the swords Why will it not work against the speaker

The swords (of Alonso and his companions) are tempered by metal (steel) which is taken out of the earth and refined by

Kill the still-closing waters as diminishOne dowl thats in my plume My fellow

ministersAre like invulnerable

In these words Ariel reminds the King and his companions of the utter futility of drawing swords against himself and his fellows Ariel drives Alonso Antonio and Sebastian the three men of sin to desperation ndash a state in which men do violence to themselves They draw swords to strike Ariel But Ariel reminds them that he and the other spirits are the ministers of destiny and nothing can wound them The steel of which their swords are made of may cut the wind or water which being divided always closes up again Even supposing that such things may be possible it is quite impossible that their swords will cut one feather in their plume They are incapable of being wounded by any sword of man Hence it is foolish on their part to attempt to strike at Ariel and his fellow-spirits

For which foul deed

(Line 72-75)The powers delaying not forgetting

haveIncensed the seas and shores yea all the

creatures Against your peace

Ariel enters like a harpy and remaining invisible tells Alonso Sebastian and Antonio that he and other harpies are the agents of Destiny appointed to carry out her decrees He tells them that their punishment for the crime against Prospero which has been so long deferred is now to fall upon them He reminds them that they had expelled Prospero from Milan and set him and his innocent child adrift on the sea and that the sea had paid them back for their sin by the shipwreck and by the calamities they have suffered He tells them that the powers above which did not forget this mean treachery but only deferred the punishment have now engaged the seas and the shores and all living beings including him and his comrades against them The very elements and supernatural agency Ariel adds have taken up the avenging of their crime against Prospero

the action of fire It may cut the wind or water which being divided always closes up again

The sword will not work against the spirits and the harpy because they are the ministers of destiny and nothing can wound them nor it will cut a single feather in their plume

(c )What is the meaning of lsquodowlrsquo in the last line

The term lsquodowlrsquo means a filament or the smallest part of a feather In this context Ariel in disguise of harpy says that their sword cannot even damage the smallest filament of their (Arielrsquos and other spirits) feathers as they are incapable of being wounded by any sword of man

(d) What does the speaker remind the listeners about

Ariel in disguise of harpy reminds Alonso the King of Naples Sebastian Alonsorsquos brother and Antonio the present Duke of Milan and the treacherous brother of Prospero as they being three men of sin He even reminds them that their punishment for their crime against Prospero which has been so long deferred now falls upon them He reminds them that they have expelled Prospero from Milan and has set him along with his innocent infant daughter adrift on the sea So the sea has paid them back for their sin by their shipwreck and the calamities they have suffered since then The harpy rebukes Alonso of his sin that has incensed the Gods and has deprived him of his son as a punishment

(e) How do they respond

When Ariel in disguise of a harpy reminds Alonso Sebastian and Antonio of their past misdeeds and sin Alonso has a look of terror and confusion in his eyes He utters the words of sincere repentance wrung out of his conscience-stricken heart It appears to him that all the elements of nature the sea-waves the wind and the thunder proclaiming a loud voice in the name of Prospero and the crime Alonso has committed against him They are calling upon him to repent There is a deep storm raging in Alonsorsquos breast and the echoes of that storm are ringing in his ears like a clear note of wind-instrument A note of denunciation of Alonsorsquos crime leaves him much humbled and penitent and confirms his belief that his son is lost forever But Sebastian and Antonio shows some courage instead of repentance They wish to kill the spirits or devils if it appears

3

Of my instruction hast thou nothing bated (Line 85-93)

In what thou hast to say So with good life

And observation strange my meaner ministers

Their several kinds have done My high charms work

And these mine enemies are all knit upIn their distractions They now are in my

powerAnd in these fits I leave them while I visitYoung Ferdinand whom they suppose is

drownedAnd his and mine loved darling

Methought the billows spoke and (Line 96-99)

told me of itThe winds did sing it to me and the

thunderThat deep and dreadful organ-pipe

pronouncedThe name of Prosper It did bass my

trespass

These are the words of contrition coming from Alonso Ariel has driven him to a deep repentance for conspiring with Antonio against Prospero He now feels a sincere remorse It appears to him that all the elements of nature the sea-waves the wind and the thunder proclaimed with a loud voice the name of Prospero and the crime Alonso had committed against him They are calling upon him to repent There is a deep storm raging in Alonsorsquos breast and the echoes of that storm are ringing in his ears like the clear note of a wind-instrument

Comment These are the words of sincere repentance wrung out of the conscience-stricken heart of Alonso Alonso who is the lesser villain is the first to give way to remorse under the effect of Arielrsquos speech The words of Ariel seem to him to be the voice of conscience speaking to him He is driven to desperation a state in which he might do violence to his life

(a) Identify the speaker State the context

Prospero the ruler of the island is the speaker The famous banquet scene has been enacted very well Ariel and his junior spirits have played their roles excellently Prospero is glad to say words of praise for them(b) In what way the speakerrsquos instructions have been carried out

According to Prosperorsquos instructions a banquet was presented before the King of Naples and his companions when they were tired and hungry Just when they were preparing to eat the feast the banquet was suddenly removed by exercising supernatural powers All this was done by Ariel Prosperorsquos chief assistant and a powerful spirit

Ariel not only made the feast disappear but also delivered his speech blaming the King and his two companions for their past wicked deeds He warned them to repent for their misdeeds or suffer forever on that uninhabited island

(c) Who are referred to as lsquomeaner ministersrsquo What have they done

Prospero refers as lsquomeaner ministersrsquo to his other lesser spirits who were assisting Ariel in presenting a scene before the kingrsquos party They entered the scene to the accompaniment of music They assumed several strange shapes and brought in a banquet Then they danced about it with gentle actions of salutations thus inviting the King and others to eat the feast

These spirits play their role again when Ariel in the shape of a harpy quits the scene These shapes enter again and dancing with mocking gestures carry away the table

(d) Who are the speakerrsquos enemies What has happened to them

King of Naples Alonso his brother Sebastian and the present Duke of Milan Antonio (Prosperorsquos own brother) are Prosperorsquos enemies With the turn of events they have all been washed ashore on the island which is ruled by Prospero the great magician Actually this happened after the shipwreck caused by a storm which was raised by Prospero with the purpose of bringing these people to his island Prosperorsquos spirits have already confused and terrified these enemies and they are under Prosperorsquos control He can treat them as he likes

(e) What does he say about Ferdinand Explain what is meant by ldquohellip his and mine darlingrdquo

Prospero knows that Alonsorsquos son prince Ferdinand is alive though his father thinks that the prince has been drowned

Prospero refers to his daughter Miranda who is dear to him She is also very dear to Prince Ferdinand who has fallen in love with her They are waiting to be married soon for which they have received Prosperorsquos consent

4

ALONSO O it is monstrous monstrous (Line 95-102)

Methought the billows spoke and told me of it

The winds did sing it to me and the thunderThat deep and dreadful organ-

pipe pronouncedThe name of Prosper It did bass

my trespassTherefore my son ithrsquo ooze is

bedded andIll seek him deeper than eer

plummet soundedAnd with him there lie mudded

(a) In what way does Alonso express his horror when his conscience is awakened by Arielrsquos words

When Alonsorsquos conscience is awakened by Arielrsquos words he expresses his horror at what he has heard He gets the feeling that the waves of the ocean the wind and the loud thunder have spoken to him and uttered the name of Prospero Because of being reminded of his crime in a very loud and rough voice he comes to realize that he has lost his son for his past misdeeds

(b) What does Alonso imagine about his son What does Alonso want to do in his desperate state

Alonso imagines that his son is lying in the mud at the bottom of the sea He feels desperate that he wants to drown himself in the ocean deeper than the plumb-line has ever gone He wants to lie with his son at the bottom of the sea

(c) How do Sebastian and Antonio want to face the evil spirits

Sebastian says that he is not at all afraid of what the harpy has said and that he is prepared to fight any number of such monsters if they appear before him only one at a time Antonio says that he would support Sebastian in the fight against the fiendsyyy

(d) Why does Gonzalo ask Adrian to follow the three men

Gonzalo tells Adrian that all the three men namely Alonso Sebastian and Antonio are in a wild and reckless mood The thought of the heinous crime of which they are guilty has begun to torment their minds So he asks Adrian to follow those three men without loss of time and prevent them from doing anything which the turmoil in their minds might lead them to do

(e) What opinion do you form of Alonso from the above extract

Alonso who is the lesser villain is the first to give way to remorse under the effect of Arielrsquos speech The words of Ariel seem to him to be the voice of conscience speaking to him He is driven to desperation a state in which he might do violence to his life

Subject =Accounts

Ac-12 15420 topic-pL Appropriation ac

PROFIT AND LOSS APPROPRIATION ACCOUNT

MEANING AND PREPARATIONProfit and Loss Appropriation Account is merely an extension of the Profit and Loss Account of the firm The profit of the firm has to be distributed amongst the partners in their respective profit sharing ratio But before its distribution it needs to be adjusted All Adjustments like partnerrsquos salary partnerrsquos commission interest on capital interest on drawings etc are made in this account These adjustments will reduce the amount of profit for distribution This adjusted profit will be distributed amongst the partners in their profit sharing ratio To prepare it at first the balance of Profit and Loss Account is transferred to this account The journal entries for the preparation of Profit and Loss Appropriation Account are given below

1 for transfer of the balance of Profit and Loss Account to Profit and Loss Appropriation Account

(a) In case of Net Profit

Profit and Loss Ac helliphelliphelliphelliphellipDrTo Profit and Loss Appropriation Ac(Net Profit transferred to Profit and Loss Appropriation Ac)

(b)In case of Net Loss

Profit and Loss Appropriation Achelliphelliphellip DrTo Profit and Loss Ac(Net Loss transferred to Profit and Loss Appropriation Ac)

2 for Interest on Capital

For transferring on Interest on CapitalProfit and Loss Appropriation Achelliphelliphellip DrTo Interest on Capital Ac(Interest on capital transferred to Profit amp Loss Appropriation Ac)

3 for Interest on Drawings

For transferring Interest on Drawings Interest on Drawings Achelliphelliphelliphelliphelliphellip DrTo Profit and Loss Appropriation Ac(Interest on drawing transferred to Profit amp Loss Appropriation Ac)

4 For Partnerrsquos SalaryFor transfer of partnerrsquos SalaryProfit and Loss Appropriation Achelliphellip DrTo Salary Ac(Salary transferred to profit amp Loss Appropriation Ac)

5 For Partnerrsquos CommissionFor transferring commissionProfit and Loss Appropriation Achelliphelliphellip DrTo Commission Ac(Commission transferred to Profit and Loss Appropriation Ac)

6 For Transfer of agreed amount to General ReserveProfit and Loss Appropriation Ac helliphellipDrTo General Reserve Ac(Transfer to General Reserve)

7 for share of Profit or Loss appropriation(a) If ProfitProfit and Loss Appropriation Achelliphellip DrTo Partnerrsquos CapitalCurrent Ac(Profit transferred to capitalcurrent Ac)(b) If LossPartnerrsquos Capital Current Achelliphelliphelliphellip DrTo Profit and Loss Appropriation Ac(Loss transferred to capitalcurrent Ac)

THE FORMAT OF PROFIT AND LOSS APPROPRIATION

Profit and Loss Appropriation Account for the year endedhelliphelliphelliphellip

Particulars Amount Particulars Amount

To PL Ac (loss) By pL Ac (profit)

To Interest on capital BY Interest on drawings

To partner`s commission by Partner`s capital Ac ( loss)

To Partner`s salary To Interest on partner`s loan To General Reserve To Partner`s Capital AC (Profit)

Subject= Economics

MOVEMENT ALONG THE DEMAND CURVE (CHANGE IN QUANTITY DEMANDED)In law of demand you have already studied the inverse relationship between price and quantity demanded When quantity demanded of a commodity changes due to change in its price keeping other factors constant it is called change in quantity demanded It is graphically expressed as a movement along the same demand curve There can be either a downward movement or an upward movement along the same demand curve Upward movement along the same demand curve is called contraction of demand or decrease in quantity demanded and downward movement along the same demand curve is known as expansion of demand or increase in quantity demanded

Extention of demandd

price (rs)p A

B Extentionp1 d

Q Q1

Quantity demanded ( in units)

Contraction of demandd

p2 Ccontraction

p APrice (Rs)

d

Q2 Q

Quantity demanded (in units)

Explanation of movement of demand A fall in price from OP to OP1 leads to increase in quantity demanded from OQ to OQ1 (expansion of demand) resulting in a downward movement from point A to point B along the same demand curve DD When Price rises from OP to OP2 quantity demanded falls from OQ to OQ2 (contraction of demand) leading to an upward movement from point A to point C along the same demand curve DD

  • Activity Series of Metals
    • Drawbacks of Rutherfordrsquos model of atom
      • Electromagnetic radiations
      • Properties of electromagnetic radiations
      • Characteristics of electromagnetic radiations
        • Plancks Quantum Theory-
        • Photoelectric effect
          • Intext Questions
            • Logic gates
            • Digital systems are said to be constructed by using logic gates These gates are the AND OR NOT NAND NOR EXOR and EXNOR gates The basic operations are described below with the aid of truth tables
            • AND gate
            • Example
Page 38:  · Web viewSubject. Topic. Summary. Execution. English 1 . Chapter 1 naming words . Page 8. Write the names of these pictures:- Person:-1. father. 2.Firefighter 3.doctor 4 ...

processes like decision makingpolicy making judicial process leadership recruitment process and others are always at work in all political systems

The scope of comparative politics is very comprehensive It includes everything that falls within the area of political activity and political process

History CAMBRIDGE VIEW ABOUT

THE PARTITION

AND REFUTATION

OF CAMBRIDGE

VIEW

Cambridge view about the Partition The Cambridge school of historians have interpreted that opposition to partition scheme was made entirely by the elitist groups They hold the view that Lord Curzon planned to partition the Bengal for administrative purposeREFUTATION OFCAMBRIDGE VIEW The Rationalist historians have rejected the interpretations of the Cambridge School of historians on various grounds

1 QUESTION State different views of historians regarding Partition of Bengal

ANSWER Cambridge historians believed that Lord Curzon partitioned Bengal for administrative reasons only and not for the political motive The Middle class elitist group protested because of their petty interest The Hindu zamindars protested as they have to spend more money for managing their estatesThe lawyers of Calcutta High court feared to lose their clientBut according to the nationalist Historians was-

2- The ultimate object of Lord Curzon was to crush the unity of Bengal politicians

3- If Bengal becomes a separate province Bengali speaking 16 million people of western part would become minority under Hindi speaking people of Bihar and Oriya speaking people of Orissa

4- The bureaucrats expected that the protest movement would die down quickly

5- Lord Curzon used the Muslim community in his political game

6- Idealism had great contribution in the protest against partition

7- The people of the every section of society were affected by the partition of Bengal

Computer Science

Numbers Convertion of dcimal number to octal numberThe decimal numeral system is the standard system for denoting integer and non-integer numbers It is the extension to non-integer numbers of the Hindu-Arabic numeral system For writing numbers the decimal system uses ten decimal digits a decimal mark and for negative numbers a minus sign - The decimal digits are 0 1 2 3 4 5 6 7 8 9 the decimal separator is the dot in many countries

The octal numeral system or oct for short is the base-8 number system and uses the digits 0 to 7 Octal is sometimes used in computing instead of hexadecimal perhaps most often in modern times in conjunction with file

permissions under Unix systems It has the advantage of not requiring any extra symbols as digits It is also used for digital displays

Follow these steps to convert a decimal number into octal form

1 Divide the decimal number by 82 Get the integer quotient for the next iteration (if the number will not divide equally by 8 then round down the

result to the nearest whole number)3 Keep a note of the remainder it should be between 0 and 74 Repeat the steps until the quotient is equal to 05 Write out all the remainders from bottom to top This is the solution

For example if the given decimal number is 8453

Division Quotient Remainder

8453 8 1056 5

1056 8 132 0

132 8 16 4

16 8 2 0

2 8 0 2

Then the octal solution is 20405

Subject Eng Literature (The Tempest ndash William Shakespeare) Topic Act I Scene 1 Lines 33 to 67 (End of scene) Date 16th April 2020 (4th Period)

[Students should read the original play and also the paraphrase given in the school prescribed textbook]Summary Questions amp Answers

[SUMMARY OF THE ENTIRE SCENE]

o The play starts with the scene of a severe storm at sea Alonso (King of Naples) Sebastian (Alonsorsquos brother) Ferdinand (Alonsorsquos son) Gonzalo Antonio (the usurping Duke of Milan) are in a ship in the midst of the storm

o The mariners are trying their best to control the vessel from running aground and are totally following the orders of their Master the Boatswain They have scant success

o The mariners become extremely unhappy and annoyed when most of the passengers arrive on the deck thereby hampering their effort to save the ship There is serious confrontation between them and the passengers who are part of the Kingrsquos entourage

o The mariners could not save the ship

SUMMING-UP

(i) Vivid description of the scene which gives a realistic description of terror and confusion of a tropical storm

(ii) Shows Shakespearersquos accuracy of knowledge in describing the naval operations and also matters of seamanship

(1) GONZALO Ill warrant him for drowning (L 45-57)

though the ship were no stronger than a nutshell and as leaky as an unstanched

wenchBOATSWAIN Lay her a-hold a-hold Set her two courses Off to

sea again lay her offMARINERS All lost To prayers to prayers All lostBOATSWAIN What must our mouths be coldGONZALO The king and prince at prayers Lets assist them

For our case is theirsSEBASTIAN Im out of patienceANTONIO We are merely cheated of our lives by drunkards

This wide-chopped rascal - would thou mightst lie drowning the washing of ten tides

(a) What does Antonio say at the insolent manners of the boatswain just before the given passage

Being irritated at the insolent manners of the boatswain just before the given extract Antonio the Duke of Milan calls him a worthless dog son of a woman without any morals an arrogant and disrespectful noisemaker He says that the boatswain deserved to be hanged(b) What statement does Gonzalo repeat about the boatswain

Gonzalo shows his faith that the boatswain is not destined to die by drowning He is destined to be hanged and nothing can alter this decree of destiny He says that even if the ship was as frail as a nutshell the boatswain could not be drowned for his destiny was to be hanged(c) What do the passengers do when they have lost all hope of their survival

When the passengers have lost all hope of survival they take

(iii) The opening scene justifies the title ndash The Tempest

UNANSWERED QUESTIONS

(i) The King always travels with his entire fleet including his soldiers Where were the other ships

(ii) Why was the ship in that area Where was it coming from or going where

(iii) The ship broke apart What happened to those who were in the ship

(We shall get the answer to the above questions as the play progresses)

leave of life with fervent prayers The mariners take their last hearty drink and are ready for death(d) What blame does Antonio put upon the mariners and the boatswain Antonio rebukes the mariners that these drunkards have brought them to the present crisis by neglecting their duties He blames them saying that they are going to lose their lives entirely for the negligence of the boatswain and his fellows(e) What does Antonio say while cursing the boatswain

Antonio gives vent to his wrath upon the boatswain in particular He calls the boatswain a wide-mouthed rascal who deserves to be hanged on the sea-shore at low water mark so that ten tides might wash over his body and take out of him all the liquor that he has been drinking

Class XIISubject Topic Summary ExecutionHistory Topic

1 1935 ACT AND WORKING OF PROVINCIAL AUTONOMYCONGREE AND OTHER MINISTERSSUB TOPIC GOVERNMENT OF INDIA ACT1935

Government of India Act 1935 This act established a lsquoFederation of Indiarsquo made of British Indian provinces and Indian states and provided for autonomy with a government responsible to the elected legislature in every provinceThis act introduced abolition of Diarchy at provinces The entire provincial administration was introduced to the responsible ministers who were controlled and removed by the provincial legislature The provincial autonomy means two things First The provincial governments were wholly responsible to the provincial legislature Secondly Provinces were free from outside control and interference in the large number of matters The act divided the powers between the centre and provinces in terms of three lists- Federal list( for centre) Provincial list (for province) and concurrent list (for both) Residuary powers were given to the viceroy In the election under the government of India Act the Congress swept the poll the mandate of the people came in favour of the congress so far as general Hindu seats were concerned The Congress did not get a single Muslim seates in Bombay CP UP Sind and BengalIn five provinces Congress had yhe clear majority In BengalNWFPAssam and Bombay Congress emerged as a single largest partyOn the other side the performance of the Muslim League was badThus the Congress formed ministers in 7 provinces out of 11 provinces Coalition ministry was also formed in two other provincesOnly BENGAL AND Punjab had non- congress ministries

1 QUESTION What was the main change introduced by the Government of India ActANSWER a) The Act gave more

autonomy to the provinces b) Diarchy was abolished at the

provincial levelsc) The Governor was the head of

the executived) There was a council of

ministers to advise him The ministers were responsible to the provincial legislatures who controlled them The legislature could also remove the ministers

e) The Governors still retained special reserve powers

2 QUESTION Why did the federal scheme introduced by the Government of India Act 1935 never come into operation

ANSWER The Federal structure of the Government of India was to be composed with the Governor General and Council of ministers The Federal legislature was to be Bicameral legislature- The council of states and the House of Assembly The ministers were to be chosen by the Governor general and they were to hold the office during his pleasure

The provinces of British India would have to join the federation but this was not compulsory for the princely states

This federation never materialised because of the lack of support from the required number of

princely statesThis act was refused and

rejected by the princes the Congress and the Muslim League

Thus both Congress and the League participated in the election of 1937 Thus the federal part was never introduced but the provincial part was put into operations

Bengali 2nd

Language

াচেরর পরাথCনা(কহিতা )

াচেরর পরাথCনা কহিতাটি কহি (ঙখ দেঘাচে4র দো আচো য কহিতায় াচেরর পতর হমায়ন কঠিন দেরাচেগ আxানত ার ঈশবর া আললার কাচেছ পরাথCনা কচেরচেছন তার পচেতরর ীন হিফহিরচেয় হিচেত এই কহিতায় ার পচেতরর ীন হিভbা দেচেয়চেছন ারার এমনহিক হিনচের ীন হিসCচেনর হিহিনমচেয় হিতহিন তার দেছচের ীন হিফচের দেপচেত দেচেয়চেছন তার দেছচের এই দেরাচেগর ন য হিতহিন হিনচেচেকই ায়ী কচেরচেছন তার হিনচের করা পাপচেকই হিতহিন ায়ী কচেরচেছন এছাা রানৈনহিতক ও আথCসামাহিক অসথার কথা তচে ধরা চেয়চেছ এই কহিতায় ার তার হিনচের পাপ কমCচেকই ায়ী কচেরচেছ ার অন যায় ভাচে দেপহি((হিকতর মাধ যচেম অপররা য কচেরচেছ আর এই অন যায় কাচের ন যই তার পহিরাচের হিপযCয় এচেসচেছ দে এক পরকার মানহিক নধন ইহিতাচেসর ার হিপতা চেয় সবাভাহিকভাচে ভাচোাসা দে মমতা দেথচেক মকত চেত পাচেরনহিন তাই হিপতা চেয় আললা া ভগাচেনর কাচেছ পতর হমায়চেনর পরানহিভbা দেচেয়চেছন ার আললা া ভগাচেনর কাচেছ াহিনচেয়চেছন তার হিনচের ীন হিসCন হিচেত হিতহিন রাী তার হিহিনমচেয় পচেতরর ীন হিফচের দেপচেত দেচেয়চেছন াচেরর হিপতসভ হিচেকর কথা এই কহিতায় ফটিচেয় দেতাা চেয়চেছ হিপতা পচেতরর হিরাহিরত মান নধচেনর কথা তচে ধরা চেয়চেছ

হিচে(4 হিকছ াইচেনর তাৎপযC১) ldquoদেকাথায় দেগ ওর সবচছয দেৌন দেকাথায় কচেরায় দেগাপন bয়ldquoউততর) াচেরর পতর হমায়ন কঠিন দেরাচেগ অসসথ তাই তার দেযৌন াহিরচেয় যাচেচছ এই দেরাচেগ তাচেক দেগাপচেন কচেরকচের াচেচছ তার সক (হিকত ধীচের ধীচের bয় চেচছ তাই হিপতা চেয় ার আললার কাচেছ হমায়চেনর পরান হিভbা দেচেয়চেছন২) ldquoাগাও (চেরর পরাচেনত পরানতচের ধসর (ন দেযর আান গানldquoউততর) াচেরর পতর হমায়ন কঠিন দেরাচেগ আxানত তাই ার আ দে(াচেক মমCাত (চেরর পচেথ পরানতচের আান গান ধবহিনত দোক দেসই আান গান আললার কাচেছ দেযন চে যায় আললা দেযন এই আহিতC শচেন পচেতরর ীন হিফহিরচেয় দেয় ৩)ldquoনাহিক এই (রীচেরর পাচেপর ীানচেত দেকানই তরারণ দেনই ভহি4চেতরldquoউততর) হমায়চেনর অসসথতার ন য ার হিনচেচেকই ায়ী কচেরচেছন কারন ার অচেনক রা য অন যায় ভাচে কচেরচেছ তাই তার এই পাপ কাচের ন য তার ঘচের আ হিপ এচেসচেছ এই অন যায় কাচের ন য তার মহিকত দেনই তাই ার আললার কাচেছ এই পাপ কাচেযCর ন য bমা পরাথM

Hindi 2ndlang

-ासी(जयशकर परसा-)

-ासी जयशकर परसा- की एक ऐसी कहानी ह जिजसम भारतीय ससकनित और राषटरीयता का सवरगजीतहोता ह इस कहानी म इरावती एक निहद कनया ह जिजस मलअचछो न मलतान की लट म पकडा और -ासी बना दि-या उस 500 दि-न -कर काशी क एक महाजन न खरी-ा दसरी -ासी निफरोजा ह वह गलाम ह निफरोजा को छडान क कतिलए अहम- को 1000 सोन क कतिसकक भजन थ जो अभी तक नही आए थ राजा साहब कठोर होत हए भी निफरोजा को निबना धनराकतिश क कतिलए उस म कर -त ह वनिफरोजा को अहम- को समझान की बात कहत हकहानी क अत म हम -खत ह निक इरा वती और जाटो क सर-ार बलराज का मिमलन होता हअहम- को यa म मार दि-या जाता ह वहा निफरोजा की परसननता की समामिध बनती ह वहा एक फल चढती ह और डीजल आती ह निफरोजा उस समामिध की आजीवन -ासी बनी रहती हलखक अपन उददशय अथात -ास परथा पर परकाश डालन और इस परथा क कारण होन वाल -ातो क दखो को दि-खान म पणता सफल हए ह

helliphellipContinue to next

Biology Reproductio Today we will discuss about vegetative Q1 Name some vegetative propagules

n in Organisms

propagation of plants The process of multiplication in which fragments of plant body function as propagule and develop into new individual is called vegetative propagation The units of such propagation are runner rhizome tuber bulb etc

and the speciesinvolvedVegetative propagules

Parts involved

Bulb StemBulbil BulbilRhizome Stem Runner Stem Tuber Stem Offset Stem Leaf buds Leaves Suckers Stem

Corns Stem stolon

Q2 State advantages of vegetative propagation

i) Rapid methodii) Sure and easy methodiii) Useful in plants that cannot

produce viable seeds or long seed dormancy

iv) Maintains purity of raceQ 3 Banana fruit is said to be parthenocarpic where as turkey is said to be parthenogenetic WhyBanana develops without fertilization from an unfertilized ovary thus is parthenocarpicIn turkey the ovum or female gamete developinto a new chick without fertilization thus isparthgenetic

Q4 Why is water hyacinth is called as a ldquoTerror of Bengalrdquo Water hyacinth can

propagatevegetatively all over the water body in a short per short period of time This resulted increased biochemicaloxygen oxygen demand of water body causing mortalityof fishes It is very difficult to get rid off them Thus known as terror of Bengal

Chemistry

Solid state GENERAL CHARACTERISTICS OF SOLID STATEIn nature the particular state of matter is governed by two opposing forces at given set of temperature and pressure These forces are intermolecular force of attraction and thermal energy If intermolecular force of attraction is high as compared to thermal energy particles remains in closest position

Intext QuestionsQ1 Classify the following solids as crystalline and amorphous Sodium chloride quartz glass quartz rubber polyvinyl chloride Teflon

A1 Crystalline

and hence very less movement in particles is observed In this case solid state is the preferred state of matter

Let us revise the general characteristics of solid

i) Fixed mass volume and shape

ii) Strong intermolecular force of attraction

iii) Least intermolecular space

iv) Fixed position of constituent particles

v) Incompressible and rigid

Q2 what type of interactions hold the molecules together in a polar molecular solid[CBSE 2010]A2 The molecules in a solid are held together by van der Waals forces The term van der Waals forces include hydrogen bonding dipole-dipole attraction and London dispersion forces All molecules experience London dispersion forces In addition polar molecules can also experience dipole-dipole interactions So the interactions that holds the molecule together in polar molecular solid are London dispersion force and dipole-dipole interactionsQ3 Write a feature that will distinguish a metallic solid from an ionic solid [CBSE 2010]A3 Metals are malleable and ductile whereas ionic solid are hard and brittle Metallic solid has typical metallic lustre But ionic solid looks dullQ4 Write a point of distinction between a metallic solid and an ionic solid other than metallic lustre [CBSE 2012]A4 Metals are malleable and ductile whereas ionic solid are hard and brittleQ5 Write a distinguish feature of metallic solid [CBSE 2010]A5 The force of attraction in

solid Sodium chloride Quartz Amorphous solid Quartz glass rubber polyvinyl chloride Teflon Q2 why glass is considered as super cooled liquidA2 Glass shows the tendency to flow at slower rate like liquid Hence they considered as super cooled liquidQ3 why the window glass of old buildings show milky appearance with timeA3 Glass is an amorphous solid Amorphous solid has the tendency to develop some crystalline character on heating Due to heating in day over the number of years glass acquires some crystalline character and show milky appearanceQ4 why the glass panes fixed to window or doors of old building become slightly thicker at bottomA4 Glass is super cooled liquid It has the tendency to flow down very slowly Due to this glass pane becomes thicker at the bottom over the timeQ5 Sodium chloride is a crystalline solid It shows the same value of refractive index along all the direction TrueFalse Give reasonA5 FalseCrystalline solid shows anisotropy in properties That is it shows different values for the given physical property in different direction All the crystalline solids show anisotropy in refractive index Therefore sodium chloride will show different values of refractive index on different directions

Q6 Crystalline solid are anisotropic in nature What does this statement means

between the constituent particles is special kind of electrostatic attraction That is the attraction of positively charged kernel with sea of delocalized electronsQ6 which group of solid is electrical conductor as well as malleable and ductile [CBSE 2013]A6 Metallic solidQ7 why graphite is good conductor of electricity although it is a network (covalent solid)A7 The exceptional property of graphite is due to its typical structure In graphite each carbon is covalently bonded with 3 atoms in same layer The fourth valence electron of each atom is free to move in between different layersThis free electron makes the graphite a good conductor of electricity

[CBSE 2011]A6 Anisotropy is defined asrdquo Difference in properties when measured along different axis or from different directionsrdquo Crystalline solid show different values of some of the physical properties like electrical resistance refractive index etcwhen measured along the different directions The anisotropy in crystalline solid arises due to the different arrangement of particles in different directions

Math Function Composition of functions Think of an industrial plant that produce bottles of cold drinks first there is the operation (or function) f that puts the cold drink inside the bottle followed by the opeartion g that close the bottle with the capThis leads to the following definitionDefinition Let f A rarr B and g B rarr C be two functions Then the composition of f and g denoted by gof is defined as the function gof A rarr C given by gof(x) = g(f (x)) forall x isinA

Definition A function f X rarr Y is defined to be invertible if there exists a function g Y rarr X such that gof = IX and fog = IY The function g is called the inverse of f and is denoted by f -1

Thus if f is invertible then f must be one-one and onto and conversely if f is one-one and onto then f must be invertible This fact significantly helps for proving a function f to be invertible by showing that f is one-one and onto specially when the actual inverse of f is not to be determined

Example 1 Let f 2 3 4 5 rarr 3 4 5 9 and g 3 4 5 9 rarr 7 11 15 be functions defined as f(2) = 3 f(3) = 4 f(4) = f(5) = 5 and g (3) = g (4) = 7 and g (5) = g (9) = 11 Find gofSolution We have gof(2) = g (f(2)) = g (3) = 7 gof(3) = g (f(3)) = g (4) = 7gof(4) = g (f(4)) = g (5) = 11 and gof(5) = g (5) = 11Example 2 Find gof and fog if f R rarr R and g R rarr R are given by f(x) = cos x and g (x) = 3x2 Show that gof ne fogSolution We have gof(x) = g(f(x))=g(cosx) = 3 (cos x)2

= 3 cos2 x Similarly fog(x)=f(g (x))= f(3x2)= cos (3x2) Note that 3cos2 x ne cos 3x2 for x = 0 Hence gof ne fogExample 3 Show that if f A rarr B and g B rarr C are onto then gof A rarr C is also ontoSolution Given an arbitrary element z isin C there exists a pre-image y of z under g such that g (y) = z since g is onto Further for y isin B there exists an element x in A with f(x) = y since f is onto Therefore gof(x) = g (f(x)) = g (y) = z showing that gof is onto Example 4 Let Y = n2 n isin N sub N Consider f N rarr Y as f(n) = n2 Show that

f is invertible Find the inverse of fSolution An arbitrary element y in Y is of the form n2 for some n isin N This implies that n =radicy This gives a function g Y rarr N defined by g (y) =radicy Nowgof (n) = g (n2)=radicn2 = n and fog (y) =f(radicy) = (radicy) 2 y which shows that gof=IN and fog= IY Hence f is invertible with f -1 = g

Political Science

Constitution of India-The Preamble

Summary

Objective of the state-To secure equality of status and of opportunity To promote fraternity among all the citizens To assure the dignity of the individuals and Unity and integrity of the nation

Justice-Justice stands for rule of law absence of arbitrariness and a system of equal rights freedom and opportunities for all in a society India seeks social economic and political justice to ensure equality to its citizens

Liberty-Liberty implies the absence of restraints or domination on the activities of an individual such as freedom from slavery serfdom imprisonment despotism etc The Preamble provides for the liberty of thought expression belief faith and worship

Equality-Equality means the absence of privileges or discrimination against any section of the society The Preamble provides for equality of status and opportunity to all the people of the country

Fraternity-The Preamble declares that fraternity has to assure two thingsmdashthe dignity of the individual and the unity and

Execution

Answer the following questions-

Short notes-1 Equality2 Fraternity3 Justice4 Liberty

Homework-Learn

integrity of the nation The word integrity has been added to the Preamble by the 42nd Constitutional Amendment (1976)

Business studies

Human resource management (chapter 1)

On the day of 1504 2020 I have discussed with you the managerial functions and procurement functions of HRM

Today weare going to discuss about the development function integration functions and maintenance function

Development functions-HRM improves the knowledge skills attitude and values of employees so that they the present and future jobs more effectively it includes

1) Development functions of HRM

a) Performance appraisal = It implies systematic evaluation of employees with respect to their performance on the job and their potential for development

b) Training =It is the process by which employees learn knowledge skills and attitudes to achieve organisational and personal goals

c) Executive development = It is the process of developing managerial talent through appropriate program

2) Integration functionsa) HRM reconcile the goals of

organisation with those of its members through integrating function

b) HRM tries to motivate employees to various financial and non financial incentives provided in job specification etc

3) Maintenance functiona) HRM promote and protect the

physical and mental health of employees by providing several types of benefits like housing medical aid etc

b) It Promote Social security measures to employees by providing provident fund pension gratuity maternity benefits

SubjectCOMMERCE

Topic

BUSINESSENVIRONMENT

Summary

Now quickly let us revise the earlier points that we have already done in the last class and let us proceed with the other topics that are there in the chapter

Firstly we will recall the internal and external factors of micro environment and then we

Execution 3 What do you mean by internal factors

in micro environmentAnswerInternal factors refer to all the factors existing within a business firm The internal factors are considered controllable because the enterprise has control over these factors

Development FunctionsPerformance AppraisalTrainingExecution Development

shall proceed in details

Meaning and list of internal and external factors

aInternal factorsInternal factors refer to all the factors existing within a business firm The internal factors are considered controllable because the enterprise has control over these factorsFor an example a company can alter its organization structure policies programmes employees physical facilities and marketing mix to suit the changes in the environmentList of internal factors areCorporate culture mission and objectives top management organizations structure company image and brand equity company resources

b External factorsExternal factors refer to those individual and groups and agencies with which a particular business organization comes into direct and frequent contact in the course of its functioningThese individuals and groups are known as STAKEHOLDERS because they have a stake (financial interest ) in the working and performance of the particular business List of external forces (stakeholders)Customers competitors investors suppliersmiddlemen (marketing intermediaries)financers publics

customers

suppliersfinancers

For an example a company can alter its organization structure policies programmes employees physical facilities and marketing mix to suit the changes in the environment

4 What do you mean by external factors in micro environment

AnswerExternal factors refer to those individual and groups and agencies with which a particular business organization comes into direct and frequent contact in the course of its functioningThese individuals and groups are known as STAKEHOLDERS because they have a stake (financial interest) in the working and performance of the particular business

3Who are stakeholdersSTAKEHOLDERS are individuals and groups who have a stake (financial interest ) in the working and performance of the particular business 4Discuss the internal factors in briefa Corporate CultureThe values beliefs and attitudes of the founders and top management of the company exercise a strong influence on what the cmpaany stands for how it does things and what it considers importantbMission and objectivesThe business philosophy and purpose of a comoany guide it prioritiesbusiness strategiesproduct market scope and development scope

cTop management structurethe composition of board of directors the degree of professionalization of management and the organizational structure of a company have important bearing on its business decisions

dPower structureThe internal power relationship between the board of directors and the chief executive is an important factor

e Company image and brand equityThe image and brand equity of the company play a significant role in raising finance forming alliance choosing dealers and suppliers launching new products entering foreign markets

5 What is Macro environmentAnswerMacro environment refers to the general

competitors

middlemen

publics

Fig STAKEHOLDERS OF A COMPANY

Apart from micro environment the other main dimension of business environment isMacro environment Macro environment refers to the general environment or remote environment within which a business firm and forces in its micro environment operateA company does not directly or regularly interact with the micro environmentTherefore macro environment is also known as indirect action EnvironmentThe macro environment forces are less controllable than the micro forces

Macro environment consists of the following components

POLITICAL AND LEGAL ENVIRONMENT

ECONOMIC SOCIAL AND ENVIRONMENT

CULTURAL

ENVIRONMENT

TECHNOLOGICAL ENVIRONMENT

Fig COMPONENTS OF MACRO ENVIRONMENT

environment or remote environment within which a business firm and forces in its micro environment operateA company does not directly or regularly interact with the micro environmentTherefore macro environment is also known as indirect action EnvironmentThe macro environment forces are less controllable than the micro forces 6 What are the components of macro environmenta Political and legal environmentb Economic environmentc Social and cultural environmentd Technological environment

Computer Science

Logic gates

Digital systems are said to be constructed by using logic gates These gates are the AND OR NOT NAND NOR EXOR and EXNOR

BUSINESS FIRM

gates The basic operations are described below with the aid of truth tables

AND gate

The AND gate is an electronic circuit that gives a high output (1) only if all its inputs are high A dot () is used to show the AND operation ie AB Bear in mind that this dot is sometimes omitted ie ABOR gate

The OR gate is an electronic circuit that gives a high output (1) if one or more of its inputs are high A plus (+) is used to show the OR operationNOT gate

The NOT gate is an electronic circuit that produces an inverted version of the input at its output It is also known as an inverter If the input variable is A the inverted output is known as NOT A This is also shown as A or A with a bar over the top as shown at the outputs The diagrams below show two ways that the NAND logic gate can be configured to produce a NOT gate It can also be done using NOR logic gates in the same way

NAND gate

This is a NOT-AND gate which is equal to an AND gate followed by a NOT gate The outputs of all NAND gates are high if any of the inputs are low The symbol is an AND gate with a small circle on the output The small circle represents inversion

NOR gate

This is a NOT-OR gate which is equal to an OR gate followed by a NOT gate The outputs of all NOR gates are low if any of the inputs are highThe symbol is an OR gate with a small circle on the output The small circle represents inversion

EXOR gate

The Exclusive-OR gate is a circuit which will give a high output if either but not both of its two inputs are high An encircled plus sign ( ) is used to show the EOR operation

EXNOR gate

The Exclusive-NOR gate circuit does the opposite to the EOR gate It will give a low output if either but not both of its two inputs are high The symbol is an EXOR gate with a small circle on the output The small circle represents inversion The NAND and NOR gates are called universal functions since with either one the AND and OR functions and NOT can be generated

Note A function in sum of products form can be implemented using NAND gates by replacing all AND and OR gates by NAND gates A function in product of sums form can be implemented using NOR gates by replacing all AND and OR gates by NOR gates

Logic gate symbols

Table 2 is a summary truth table of the inputoutput combinations for the NOT gate together with all possible inputoutput combinations for the other gate functions Also note that a truth table with n inputs has 2n rows You can compare the outputs of different gates

Logic gates representation using the Truth table

Example

A NAND gate can be used as a NOT gate using either of the following wiring configurations

Subject Eng Literature (The Tempest ndash William Shakespeare) Topic Act III Scene 3 Lines 53 to 110 (End of the scene) Date 16th April 2020 (2nd Period)

[Students should read the original play and also the paraphrase given in the school prescribed textbook]Summary Questions amp Answers

o Seeing this strange scene all are inclined to believe the tales told by travelers that there truly are ldquounicornsrdquo and ldquothe phoenixrsquo thronerdquo

o As they are about to sit down to the feast the banquet is snatched away by a harpy (Ariel disguised) A spiritrsquos voice (Arielrsquos voice) denounces Alonso Sebastian and Antonio with particular

1 ARIEL You are three men of sin whom Destiny

(Line 53-58)That hath to instrument this

lower world And what is int the never-surfeited sea

Hath caused to belch up you and on this island

Where man doth not inhabit you rsquomongst men

Being most unfit to live I have made you mad

reference to their crime in expelling Prospero from Milan They have not received any punishment for their deed earlier but the time for their punishment has arrived Upon Alonso it pronounces ldquolingering perdition worse than deathrdquo from which there is no remedy except through sincere repentance Ariel then vanishes in thunder and the shapes enter again and carry away the table

o Prospero watching invisibly is very pleased with the performance of Ariel and his (Prosperorsquos) ldquomeaner ministersrdquo All his enemies are now in his power and are in a fit of desperation He then leaves them and goes to see how Ferdinand and Miranda are getting on

o Alonso is now much humbled and penitent with the after effect of the spiritrsquos denunciation of his crimes He believes that his son is lost forever After this all disperse being stricken mad by the speech of the spirit

o Gonzalo fearing that they may do violence to themselves or to one another follows them and bid others to follow

(a) To whom does Ariel disguised as a harpy call the three sinners What game did Fate of Destiny play with

them

The three sinners called by Ariel are Alonso Sebastian and Antonio It was Destiny which had caused the ocean to cast the three sinners on the shore Though the ocean is all the time devouring whatever appears on its surface and is never satisfied with its continual swallowing of the ships and men in the present case the ocean had cast these three sinners on the shore without killing them

(b) Who had jointly been responsible for the conspiracy against Prospero What is Prosperorsquos purpose behind all this

Three men Alonso Sebastian and Antonio had jointly

been responsible for the conspiracy against Prospero They had driven out Prospero form Milan Prosperorsquos purpose is to make these three sinners realize the wrong they had done He wants them to repent for their criminal deeds because repentance leads to self-esteem(c )What does Ariel (the harpy) tell Alonso and his companions when they take out their swords to attack him

Seeing them drawing their swords Ariel (harpy) tells them that he and his companions are the instruments of destiny and that it is not possible for human beings to do them any injury He says that the swords of human beings can not injure even a minute part of his feathers Their swords are as ineffective against him and his companions as against the wind or the water

(d) Give the explanatory meanings of the following expressions in the context of the above extract

(i)Never surfeited (ii) Belch up (iii) lsquomongst men

(i) Never surfeited never led to satisfaction

(ii) Belch up cast ashore(iii) lsquomongst men in human

society2

I and my fellows (Line 60-65)

Are ministers of Fate The elementsOf whom your swords are tempered may as wellWound the loud winds or with bemocked-at stabsKill the still-closing waters as diminishOne dowl thats in my plume

IMPORTANT PASSAGES EXPLAINED

The elements

(Line 61-66)Of whom your swords are tempered may

as wellWound the loud winds or with

bemocked-at stabs

(a) Who is lsquoIrsquo Who are his lsquofellowsrdquo

lsquoIrsquo is referred to Ariel in disguise of a harpy His lsquofellowsrsquo are other spirits serving Prospero the real Duke of Milan who has acquired supernatural powers after being banished from his Dukedom Prospero has settled in this uninhabited island

(b) What are the elements that have temperrsquod the swords Why will it not work against the speaker

The swords (of Alonso and his companions) are tempered by metal (steel) which is taken out of the earth and refined by

Kill the still-closing waters as diminishOne dowl thats in my plume My fellow

ministersAre like invulnerable

In these words Ariel reminds the King and his companions of the utter futility of drawing swords against himself and his fellows Ariel drives Alonso Antonio and Sebastian the three men of sin to desperation ndash a state in which men do violence to themselves They draw swords to strike Ariel But Ariel reminds them that he and the other spirits are the ministers of destiny and nothing can wound them The steel of which their swords are made of may cut the wind or water which being divided always closes up again Even supposing that such things may be possible it is quite impossible that their swords will cut one feather in their plume They are incapable of being wounded by any sword of man Hence it is foolish on their part to attempt to strike at Ariel and his fellow-spirits

For which foul deed

(Line 72-75)The powers delaying not forgetting

haveIncensed the seas and shores yea all the

creatures Against your peace

Ariel enters like a harpy and remaining invisible tells Alonso Sebastian and Antonio that he and other harpies are the agents of Destiny appointed to carry out her decrees He tells them that their punishment for the crime against Prospero which has been so long deferred is now to fall upon them He reminds them that they had expelled Prospero from Milan and set him and his innocent child adrift on the sea and that the sea had paid them back for their sin by the shipwreck and by the calamities they have suffered He tells them that the powers above which did not forget this mean treachery but only deferred the punishment have now engaged the seas and the shores and all living beings including him and his comrades against them The very elements and supernatural agency Ariel adds have taken up the avenging of their crime against Prospero

the action of fire It may cut the wind or water which being divided always closes up again

The sword will not work against the spirits and the harpy because they are the ministers of destiny and nothing can wound them nor it will cut a single feather in their plume

(c )What is the meaning of lsquodowlrsquo in the last line

The term lsquodowlrsquo means a filament or the smallest part of a feather In this context Ariel in disguise of harpy says that their sword cannot even damage the smallest filament of their (Arielrsquos and other spirits) feathers as they are incapable of being wounded by any sword of man

(d) What does the speaker remind the listeners about

Ariel in disguise of harpy reminds Alonso the King of Naples Sebastian Alonsorsquos brother and Antonio the present Duke of Milan and the treacherous brother of Prospero as they being three men of sin He even reminds them that their punishment for their crime against Prospero which has been so long deferred now falls upon them He reminds them that they have expelled Prospero from Milan and has set him along with his innocent infant daughter adrift on the sea So the sea has paid them back for their sin by their shipwreck and the calamities they have suffered since then The harpy rebukes Alonso of his sin that has incensed the Gods and has deprived him of his son as a punishment

(e) How do they respond

When Ariel in disguise of a harpy reminds Alonso Sebastian and Antonio of their past misdeeds and sin Alonso has a look of terror and confusion in his eyes He utters the words of sincere repentance wrung out of his conscience-stricken heart It appears to him that all the elements of nature the sea-waves the wind and the thunder proclaiming a loud voice in the name of Prospero and the crime Alonso has committed against him They are calling upon him to repent There is a deep storm raging in Alonsorsquos breast and the echoes of that storm are ringing in his ears like a clear note of wind-instrument A note of denunciation of Alonsorsquos crime leaves him much humbled and penitent and confirms his belief that his son is lost forever But Sebastian and Antonio shows some courage instead of repentance They wish to kill the spirits or devils if it appears

3

Of my instruction hast thou nothing bated (Line 85-93)

In what thou hast to say So with good life

And observation strange my meaner ministers

Their several kinds have done My high charms work

And these mine enemies are all knit upIn their distractions They now are in my

powerAnd in these fits I leave them while I visitYoung Ferdinand whom they suppose is

drownedAnd his and mine loved darling

Methought the billows spoke and (Line 96-99)

told me of itThe winds did sing it to me and the

thunderThat deep and dreadful organ-pipe

pronouncedThe name of Prosper It did bass my

trespass

These are the words of contrition coming from Alonso Ariel has driven him to a deep repentance for conspiring with Antonio against Prospero He now feels a sincere remorse It appears to him that all the elements of nature the sea-waves the wind and the thunder proclaimed with a loud voice the name of Prospero and the crime Alonso had committed against him They are calling upon him to repent There is a deep storm raging in Alonsorsquos breast and the echoes of that storm are ringing in his ears like the clear note of a wind-instrument

Comment These are the words of sincere repentance wrung out of the conscience-stricken heart of Alonso Alonso who is the lesser villain is the first to give way to remorse under the effect of Arielrsquos speech The words of Ariel seem to him to be the voice of conscience speaking to him He is driven to desperation a state in which he might do violence to his life

(a) Identify the speaker State the context

Prospero the ruler of the island is the speaker The famous banquet scene has been enacted very well Ariel and his junior spirits have played their roles excellently Prospero is glad to say words of praise for them(b) In what way the speakerrsquos instructions have been carried out

According to Prosperorsquos instructions a banquet was presented before the King of Naples and his companions when they were tired and hungry Just when they were preparing to eat the feast the banquet was suddenly removed by exercising supernatural powers All this was done by Ariel Prosperorsquos chief assistant and a powerful spirit

Ariel not only made the feast disappear but also delivered his speech blaming the King and his two companions for their past wicked deeds He warned them to repent for their misdeeds or suffer forever on that uninhabited island

(c) Who are referred to as lsquomeaner ministersrsquo What have they done

Prospero refers as lsquomeaner ministersrsquo to his other lesser spirits who were assisting Ariel in presenting a scene before the kingrsquos party They entered the scene to the accompaniment of music They assumed several strange shapes and brought in a banquet Then they danced about it with gentle actions of salutations thus inviting the King and others to eat the feast

These spirits play their role again when Ariel in the shape of a harpy quits the scene These shapes enter again and dancing with mocking gestures carry away the table

(d) Who are the speakerrsquos enemies What has happened to them

King of Naples Alonso his brother Sebastian and the present Duke of Milan Antonio (Prosperorsquos own brother) are Prosperorsquos enemies With the turn of events they have all been washed ashore on the island which is ruled by Prospero the great magician Actually this happened after the shipwreck caused by a storm which was raised by Prospero with the purpose of bringing these people to his island Prosperorsquos spirits have already confused and terrified these enemies and they are under Prosperorsquos control He can treat them as he likes

(e) What does he say about Ferdinand Explain what is meant by ldquohellip his and mine darlingrdquo

Prospero knows that Alonsorsquos son prince Ferdinand is alive though his father thinks that the prince has been drowned

Prospero refers to his daughter Miranda who is dear to him She is also very dear to Prince Ferdinand who has fallen in love with her They are waiting to be married soon for which they have received Prosperorsquos consent

4

ALONSO O it is monstrous monstrous (Line 95-102)

Methought the billows spoke and told me of it

The winds did sing it to me and the thunderThat deep and dreadful organ-

pipe pronouncedThe name of Prosper It did bass

my trespassTherefore my son ithrsquo ooze is

bedded andIll seek him deeper than eer

plummet soundedAnd with him there lie mudded

(a) In what way does Alonso express his horror when his conscience is awakened by Arielrsquos words

When Alonsorsquos conscience is awakened by Arielrsquos words he expresses his horror at what he has heard He gets the feeling that the waves of the ocean the wind and the loud thunder have spoken to him and uttered the name of Prospero Because of being reminded of his crime in a very loud and rough voice he comes to realize that he has lost his son for his past misdeeds

(b) What does Alonso imagine about his son What does Alonso want to do in his desperate state

Alonso imagines that his son is lying in the mud at the bottom of the sea He feels desperate that he wants to drown himself in the ocean deeper than the plumb-line has ever gone He wants to lie with his son at the bottom of the sea

(c) How do Sebastian and Antonio want to face the evil spirits

Sebastian says that he is not at all afraid of what the harpy has said and that he is prepared to fight any number of such monsters if they appear before him only one at a time Antonio says that he would support Sebastian in the fight against the fiendsyyy

(d) Why does Gonzalo ask Adrian to follow the three men

Gonzalo tells Adrian that all the three men namely Alonso Sebastian and Antonio are in a wild and reckless mood The thought of the heinous crime of which they are guilty has begun to torment their minds So he asks Adrian to follow those three men without loss of time and prevent them from doing anything which the turmoil in their minds might lead them to do

(e) What opinion do you form of Alonso from the above extract

Alonso who is the lesser villain is the first to give way to remorse under the effect of Arielrsquos speech The words of Ariel seem to him to be the voice of conscience speaking to him He is driven to desperation a state in which he might do violence to his life

Subject =Accounts

Ac-12 15420 topic-pL Appropriation ac

PROFIT AND LOSS APPROPRIATION ACCOUNT

MEANING AND PREPARATIONProfit and Loss Appropriation Account is merely an extension of the Profit and Loss Account of the firm The profit of the firm has to be distributed amongst the partners in their respective profit sharing ratio But before its distribution it needs to be adjusted All Adjustments like partnerrsquos salary partnerrsquos commission interest on capital interest on drawings etc are made in this account These adjustments will reduce the amount of profit for distribution This adjusted profit will be distributed amongst the partners in their profit sharing ratio To prepare it at first the balance of Profit and Loss Account is transferred to this account The journal entries for the preparation of Profit and Loss Appropriation Account are given below

1 for transfer of the balance of Profit and Loss Account to Profit and Loss Appropriation Account

(a) In case of Net Profit

Profit and Loss Ac helliphelliphelliphelliphellipDrTo Profit and Loss Appropriation Ac(Net Profit transferred to Profit and Loss Appropriation Ac)

(b)In case of Net Loss

Profit and Loss Appropriation Achelliphelliphellip DrTo Profit and Loss Ac(Net Loss transferred to Profit and Loss Appropriation Ac)

2 for Interest on Capital

For transferring on Interest on CapitalProfit and Loss Appropriation Achelliphelliphellip DrTo Interest on Capital Ac(Interest on capital transferred to Profit amp Loss Appropriation Ac)

3 for Interest on Drawings

For transferring Interest on Drawings Interest on Drawings Achelliphelliphelliphelliphelliphellip DrTo Profit and Loss Appropriation Ac(Interest on drawing transferred to Profit amp Loss Appropriation Ac)

4 For Partnerrsquos SalaryFor transfer of partnerrsquos SalaryProfit and Loss Appropriation Achelliphellip DrTo Salary Ac(Salary transferred to profit amp Loss Appropriation Ac)

5 For Partnerrsquos CommissionFor transferring commissionProfit and Loss Appropriation Achelliphelliphellip DrTo Commission Ac(Commission transferred to Profit and Loss Appropriation Ac)

6 For Transfer of agreed amount to General ReserveProfit and Loss Appropriation Ac helliphellipDrTo General Reserve Ac(Transfer to General Reserve)

7 for share of Profit or Loss appropriation(a) If ProfitProfit and Loss Appropriation Achelliphellip DrTo Partnerrsquos CapitalCurrent Ac(Profit transferred to capitalcurrent Ac)(b) If LossPartnerrsquos Capital Current Achelliphelliphelliphellip DrTo Profit and Loss Appropriation Ac(Loss transferred to capitalcurrent Ac)

THE FORMAT OF PROFIT AND LOSS APPROPRIATION

Profit and Loss Appropriation Account for the year endedhelliphelliphelliphellip

Particulars Amount Particulars Amount

To PL Ac (loss) By pL Ac (profit)

To Interest on capital BY Interest on drawings

To partner`s commission by Partner`s capital Ac ( loss)

To Partner`s salary To Interest on partner`s loan To General Reserve To Partner`s Capital AC (Profit)

Subject= Economics

MOVEMENT ALONG THE DEMAND CURVE (CHANGE IN QUANTITY DEMANDED)In law of demand you have already studied the inverse relationship between price and quantity demanded When quantity demanded of a commodity changes due to change in its price keeping other factors constant it is called change in quantity demanded It is graphically expressed as a movement along the same demand curve There can be either a downward movement or an upward movement along the same demand curve Upward movement along the same demand curve is called contraction of demand or decrease in quantity demanded and downward movement along the same demand curve is known as expansion of demand or increase in quantity demanded

Extention of demandd

price (rs)p A

B Extentionp1 d

Q Q1

Quantity demanded ( in units)

Contraction of demandd

p2 Ccontraction

p APrice (Rs)

d

Q2 Q

Quantity demanded (in units)

Explanation of movement of demand A fall in price from OP to OP1 leads to increase in quantity demanded from OQ to OQ1 (expansion of demand) resulting in a downward movement from point A to point B along the same demand curve DD When Price rises from OP to OP2 quantity demanded falls from OQ to OQ2 (contraction of demand) leading to an upward movement from point A to point C along the same demand curve DD

  • Activity Series of Metals
    • Drawbacks of Rutherfordrsquos model of atom
      • Electromagnetic radiations
      • Properties of electromagnetic radiations
      • Characteristics of electromagnetic radiations
        • Plancks Quantum Theory-
        • Photoelectric effect
          • Intext Questions
            • Logic gates
            • Digital systems are said to be constructed by using logic gates These gates are the AND OR NOT NAND NOR EXOR and EXNOR gates The basic operations are described below with the aid of truth tables
            • AND gate
            • Example
Page 39:  · Web viewSubject. Topic. Summary. Execution. English 1 . Chapter 1 naming words . Page 8. Write the names of these pictures:- Person:-1. father. 2.Firefighter 3.doctor 4 ...

permissions under Unix systems It has the advantage of not requiring any extra symbols as digits It is also used for digital displays

Follow these steps to convert a decimal number into octal form

1 Divide the decimal number by 82 Get the integer quotient for the next iteration (if the number will not divide equally by 8 then round down the

result to the nearest whole number)3 Keep a note of the remainder it should be between 0 and 74 Repeat the steps until the quotient is equal to 05 Write out all the remainders from bottom to top This is the solution

For example if the given decimal number is 8453

Division Quotient Remainder

8453 8 1056 5

1056 8 132 0

132 8 16 4

16 8 2 0

2 8 0 2

Then the octal solution is 20405

Subject Eng Literature (The Tempest ndash William Shakespeare) Topic Act I Scene 1 Lines 33 to 67 (End of scene) Date 16th April 2020 (4th Period)

[Students should read the original play and also the paraphrase given in the school prescribed textbook]Summary Questions amp Answers

[SUMMARY OF THE ENTIRE SCENE]

o The play starts with the scene of a severe storm at sea Alonso (King of Naples) Sebastian (Alonsorsquos brother) Ferdinand (Alonsorsquos son) Gonzalo Antonio (the usurping Duke of Milan) are in a ship in the midst of the storm

o The mariners are trying their best to control the vessel from running aground and are totally following the orders of their Master the Boatswain They have scant success

o The mariners become extremely unhappy and annoyed when most of the passengers arrive on the deck thereby hampering their effort to save the ship There is serious confrontation between them and the passengers who are part of the Kingrsquos entourage

o The mariners could not save the ship

SUMMING-UP

(i) Vivid description of the scene which gives a realistic description of terror and confusion of a tropical storm

(ii) Shows Shakespearersquos accuracy of knowledge in describing the naval operations and also matters of seamanship

(1) GONZALO Ill warrant him for drowning (L 45-57)

though the ship were no stronger than a nutshell and as leaky as an unstanched

wenchBOATSWAIN Lay her a-hold a-hold Set her two courses Off to

sea again lay her offMARINERS All lost To prayers to prayers All lostBOATSWAIN What must our mouths be coldGONZALO The king and prince at prayers Lets assist them

For our case is theirsSEBASTIAN Im out of patienceANTONIO We are merely cheated of our lives by drunkards

This wide-chopped rascal - would thou mightst lie drowning the washing of ten tides

(a) What does Antonio say at the insolent manners of the boatswain just before the given passage

Being irritated at the insolent manners of the boatswain just before the given extract Antonio the Duke of Milan calls him a worthless dog son of a woman without any morals an arrogant and disrespectful noisemaker He says that the boatswain deserved to be hanged(b) What statement does Gonzalo repeat about the boatswain

Gonzalo shows his faith that the boatswain is not destined to die by drowning He is destined to be hanged and nothing can alter this decree of destiny He says that even if the ship was as frail as a nutshell the boatswain could not be drowned for his destiny was to be hanged(c) What do the passengers do when they have lost all hope of their survival

When the passengers have lost all hope of survival they take

(iii) The opening scene justifies the title ndash The Tempest

UNANSWERED QUESTIONS

(i) The King always travels with his entire fleet including his soldiers Where were the other ships

(ii) Why was the ship in that area Where was it coming from or going where

(iii) The ship broke apart What happened to those who were in the ship

(We shall get the answer to the above questions as the play progresses)

leave of life with fervent prayers The mariners take their last hearty drink and are ready for death(d) What blame does Antonio put upon the mariners and the boatswain Antonio rebukes the mariners that these drunkards have brought them to the present crisis by neglecting their duties He blames them saying that they are going to lose their lives entirely for the negligence of the boatswain and his fellows(e) What does Antonio say while cursing the boatswain

Antonio gives vent to his wrath upon the boatswain in particular He calls the boatswain a wide-mouthed rascal who deserves to be hanged on the sea-shore at low water mark so that ten tides might wash over his body and take out of him all the liquor that he has been drinking

Class XIISubject Topic Summary ExecutionHistory Topic

1 1935 ACT AND WORKING OF PROVINCIAL AUTONOMYCONGREE AND OTHER MINISTERSSUB TOPIC GOVERNMENT OF INDIA ACT1935

Government of India Act 1935 This act established a lsquoFederation of Indiarsquo made of British Indian provinces and Indian states and provided for autonomy with a government responsible to the elected legislature in every provinceThis act introduced abolition of Diarchy at provinces The entire provincial administration was introduced to the responsible ministers who were controlled and removed by the provincial legislature The provincial autonomy means two things First The provincial governments were wholly responsible to the provincial legislature Secondly Provinces were free from outside control and interference in the large number of matters The act divided the powers between the centre and provinces in terms of three lists- Federal list( for centre) Provincial list (for province) and concurrent list (for both) Residuary powers were given to the viceroy In the election under the government of India Act the Congress swept the poll the mandate of the people came in favour of the congress so far as general Hindu seats were concerned The Congress did not get a single Muslim seates in Bombay CP UP Sind and BengalIn five provinces Congress had yhe clear majority In BengalNWFPAssam and Bombay Congress emerged as a single largest partyOn the other side the performance of the Muslim League was badThus the Congress formed ministers in 7 provinces out of 11 provinces Coalition ministry was also formed in two other provincesOnly BENGAL AND Punjab had non- congress ministries

1 QUESTION What was the main change introduced by the Government of India ActANSWER a) The Act gave more

autonomy to the provinces b) Diarchy was abolished at the

provincial levelsc) The Governor was the head of

the executived) There was a council of

ministers to advise him The ministers were responsible to the provincial legislatures who controlled them The legislature could also remove the ministers

e) The Governors still retained special reserve powers

2 QUESTION Why did the federal scheme introduced by the Government of India Act 1935 never come into operation

ANSWER The Federal structure of the Government of India was to be composed with the Governor General and Council of ministers The Federal legislature was to be Bicameral legislature- The council of states and the House of Assembly The ministers were to be chosen by the Governor general and they were to hold the office during his pleasure

The provinces of British India would have to join the federation but this was not compulsory for the princely states

This federation never materialised because of the lack of support from the required number of

princely statesThis act was refused and

rejected by the princes the Congress and the Muslim League

Thus both Congress and the League participated in the election of 1937 Thus the federal part was never introduced but the provincial part was put into operations

Bengali 2nd

Language

াচেরর পরাথCনা(কহিতা )

াচেরর পরাথCনা কহিতাটি কহি (ঙখ দেঘাচে4র দো আচো য কহিতায় াচেরর পতর হমায়ন কঠিন দেরাচেগ আxানত ার ঈশবর া আললার কাচেছ পরাথCনা কচেরচেছন তার পচেতরর ীন হিফহিরচেয় হিচেত এই কহিতায় ার পচেতরর ীন হিভbা দেচেয়চেছন ারার এমনহিক হিনচের ীন হিসCচেনর হিহিনমচেয় হিতহিন তার দেছচের ীন হিফচের দেপচেত দেচেয়চেছন তার দেছচের এই দেরাচেগর ন য হিতহিন হিনচেচেকই ায়ী কচেরচেছন তার হিনচের করা পাপচেকই হিতহিন ায়ী কচেরচেছন এছাা রানৈনহিতক ও আথCসামাহিক অসথার কথা তচে ধরা চেয়চেছ এই কহিতায় ার তার হিনচের পাপ কমCচেকই ায়ী কচেরচেছ ার অন যায় ভাচে দেপহি((হিকতর মাধ যচেম অপররা য কচেরচেছ আর এই অন যায় কাচের ন যই তার পহিরাচের হিপযCয় এচেসচেছ দে এক পরকার মানহিক নধন ইহিতাচেসর ার হিপতা চেয় সবাভাহিকভাচে ভাচোাসা দে মমতা দেথচেক মকত চেত পাচেরনহিন তাই হিপতা চেয় আললা া ভগাচেনর কাচেছ পতর হমায়চেনর পরানহিভbা দেচেয়চেছন ার আললা া ভগাচেনর কাচেছ াহিনচেয়চেছন তার হিনচের ীন হিসCন হিচেত হিতহিন রাী তার হিহিনমচেয় পচেতরর ীন হিফচের দেপচেত দেচেয়চেছন াচেরর হিপতসভ হিচেকর কথা এই কহিতায় ফটিচেয় দেতাা চেয়চেছ হিপতা পচেতরর হিরাহিরত মান নধচেনর কথা তচে ধরা চেয়চেছ

হিচে(4 হিকছ াইচেনর তাৎপযC১) ldquoদেকাথায় দেগ ওর সবচছয দেৌন দেকাথায় কচেরায় দেগাপন bয়ldquoউততর) াচেরর পতর হমায়ন কঠিন দেরাচেগ অসসথ তাই তার দেযৌন াহিরচেয় যাচেচছ এই দেরাচেগ তাচেক দেগাপচেন কচেরকচের াচেচছ তার সক (হিকত ধীচের ধীচের bয় চেচছ তাই হিপতা চেয় ার আললার কাচেছ হমায়চেনর পরান হিভbা দেচেয়চেছন২) ldquoাগাও (চেরর পরাচেনত পরানতচের ধসর (ন দেযর আান গানldquoউততর) াচেরর পতর হমায়ন কঠিন দেরাচেগ আxানত তাই ার আ দে(াচেক মমCাত (চেরর পচেথ পরানতচের আান গান ধবহিনত দোক দেসই আান গান আললার কাচেছ দেযন চে যায় আললা দেযন এই আহিতC শচেন পচেতরর ীন হিফহিরচেয় দেয় ৩)ldquoনাহিক এই (রীচেরর পাচেপর ীানচেত দেকানই তরারণ দেনই ভহি4চেতরldquoউততর) হমায়চেনর অসসথতার ন য ার হিনচেচেকই ায়ী কচেরচেছন কারন ার অচেনক রা য অন যায় ভাচে কচেরচেছ তাই তার এই পাপ কাচের ন য তার ঘচের আ হিপ এচেসচেছ এই অন যায় কাচের ন য তার মহিকত দেনই তাই ার আললার কাচেছ এই পাপ কাচেযCর ন য bমা পরাথM

Hindi 2ndlang

-ासी(जयशकर परसा-)

-ासी जयशकर परसा- की एक ऐसी कहानी ह जिजसम भारतीय ससकनित और राषटरीयता का सवरगजीतहोता ह इस कहानी म इरावती एक निहद कनया ह जिजस मलअचछो न मलतान की लट म पकडा और -ासी बना दि-या उस 500 दि-न -कर काशी क एक महाजन न खरी-ा दसरी -ासी निफरोजा ह वह गलाम ह निफरोजा को छडान क कतिलए अहम- को 1000 सोन क कतिसकक भजन थ जो अभी तक नही आए थ राजा साहब कठोर होत हए भी निफरोजा को निबना धनराकतिश क कतिलए उस म कर -त ह वनिफरोजा को अहम- को समझान की बात कहत हकहानी क अत म हम -खत ह निक इरा वती और जाटो क सर-ार बलराज का मिमलन होता हअहम- को यa म मार दि-या जाता ह वहा निफरोजा की परसननता की समामिध बनती ह वहा एक फल चढती ह और डीजल आती ह निफरोजा उस समामिध की आजीवन -ासी बनी रहती हलखक अपन उददशय अथात -ास परथा पर परकाश डालन और इस परथा क कारण होन वाल -ातो क दखो को दि-खान म पणता सफल हए ह

helliphellipContinue to next

Biology Reproductio Today we will discuss about vegetative Q1 Name some vegetative propagules

n in Organisms

propagation of plants The process of multiplication in which fragments of plant body function as propagule and develop into new individual is called vegetative propagation The units of such propagation are runner rhizome tuber bulb etc

and the speciesinvolvedVegetative propagules

Parts involved

Bulb StemBulbil BulbilRhizome Stem Runner Stem Tuber Stem Offset Stem Leaf buds Leaves Suckers Stem

Corns Stem stolon

Q2 State advantages of vegetative propagation

i) Rapid methodii) Sure and easy methodiii) Useful in plants that cannot

produce viable seeds or long seed dormancy

iv) Maintains purity of raceQ 3 Banana fruit is said to be parthenocarpic where as turkey is said to be parthenogenetic WhyBanana develops without fertilization from an unfertilized ovary thus is parthenocarpicIn turkey the ovum or female gamete developinto a new chick without fertilization thus isparthgenetic

Q4 Why is water hyacinth is called as a ldquoTerror of Bengalrdquo Water hyacinth can

propagatevegetatively all over the water body in a short per short period of time This resulted increased biochemicaloxygen oxygen demand of water body causing mortalityof fishes It is very difficult to get rid off them Thus known as terror of Bengal

Chemistry

Solid state GENERAL CHARACTERISTICS OF SOLID STATEIn nature the particular state of matter is governed by two opposing forces at given set of temperature and pressure These forces are intermolecular force of attraction and thermal energy If intermolecular force of attraction is high as compared to thermal energy particles remains in closest position

Intext QuestionsQ1 Classify the following solids as crystalline and amorphous Sodium chloride quartz glass quartz rubber polyvinyl chloride Teflon

A1 Crystalline

and hence very less movement in particles is observed In this case solid state is the preferred state of matter

Let us revise the general characteristics of solid

i) Fixed mass volume and shape

ii) Strong intermolecular force of attraction

iii) Least intermolecular space

iv) Fixed position of constituent particles

v) Incompressible and rigid

Q2 what type of interactions hold the molecules together in a polar molecular solid[CBSE 2010]A2 The molecules in a solid are held together by van der Waals forces The term van der Waals forces include hydrogen bonding dipole-dipole attraction and London dispersion forces All molecules experience London dispersion forces In addition polar molecules can also experience dipole-dipole interactions So the interactions that holds the molecule together in polar molecular solid are London dispersion force and dipole-dipole interactionsQ3 Write a feature that will distinguish a metallic solid from an ionic solid [CBSE 2010]A3 Metals are malleable and ductile whereas ionic solid are hard and brittle Metallic solid has typical metallic lustre But ionic solid looks dullQ4 Write a point of distinction between a metallic solid and an ionic solid other than metallic lustre [CBSE 2012]A4 Metals are malleable and ductile whereas ionic solid are hard and brittleQ5 Write a distinguish feature of metallic solid [CBSE 2010]A5 The force of attraction in

solid Sodium chloride Quartz Amorphous solid Quartz glass rubber polyvinyl chloride Teflon Q2 why glass is considered as super cooled liquidA2 Glass shows the tendency to flow at slower rate like liquid Hence they considered as super cooled liquidQ3 why the window glass of old buildings show milky appearance with timeA3 Glass is an amorphous solid Amorphous solid has the tendency to develop some crystalline character on heating Due to heating in day over the number of years glass acquires some crystalline character and show milky appearanceQ4 why the glass panes fixed to window or doors of old building become slightly thicker at bottomA4 Glass is super cooled liquid It has the tendency to flow down very slowly Due to this glass pane becomes thicker at the bottom over the timeQ5 Sodium chloride is a crystalline solid It shows the same value of refractive index along all the direction TrueFalse Give reasonA5 FalseCrystalline solid shows anisotropy in properties That is it shows different values for the given physical property in different direction All the crystalline solids show anisotropy in refractive index Therefore sodium chloride will show different values of refractive index on different directions

Q6 Crystalline solid are anisotropic in nature What does this statement means

between the constituent particles is special kind of electrostatic attraction That is the attraction of positively charged kernel with sea of delocalized electronsQ6 which group of solid is electrical conductor as well as malleable and ductile [CBSE 2013]A6 Metallic solidQ7 why graphite is good conductor of electricity although it is a network (covalent solid)A7 The exceptional property of graphite is due to its typical structure In graphite each carbon is covalently bonded with 3 atoms in same layer The fourth valence electron of each atom is free to move in between different layersThis free electron makes the graphite a good conductor of electricity

[CBSE 2011]A6 Anisotropy is defined asrdquo Difference in properties when measured along different axis or from different directionsrdquo Crystalline solid show different values of some of the physical properties like electrical resistance refractive index etcwhen measured along the different directions The anisotropy in crystalline solid arises due to the different arrangement of particles in different directions

Math Function Composition of functions Think of an industrial plant that produce bottles of cold drinks first there is the operation (or function) f that puts the cold drink inside the bottle followed by the opeartion g that close the bottle with the capThis leads to the following definitionDefinition Let f A rarr B and g B rarr C be two functions Then the composition of f and g denoted by gof is defined as the function gof A rarr C given by gof(x) = g(f (x)) forall x isinA

Definition A function f X rarr Y is defined to be invertible if there exists a function g Y rarr X such that gof = IX and fog = IY The function g is called the inverse of f and is denoted by f -1

Thus if f is invertible then f must be one-one and onto and conversely if f is one-one and onto then f must be invertible This fact significantly helps for proving a function f to be invertible by showing that f is one-one and onto specially when the actual inverse of f is not to be determined

Example 1 Let f 2 3 4 5 rarr 3 4 5 9 and g 3 4 5 9 rarr 7 11 15 be functions defined as f(2) = 3 f(3) = 4 f(4) = f(5) = 5 and g (3) = g (4) = 7 and g (5) = g (9) = 11 Find gofSolution We have gof(2) = g (f(2)) = g (3) = 7 gof(3) = g (f(3)) = g (4) = 7gof(4) = g (f(4)) = g (5) = 11 and gof(5) = g (5) = 11Example 2 Find gof and fog if f R rarr R and g R rarr R are given by f(x) = cos x and g (x) = 3x2 Show that gof ne fogSolution We have gof(x) = g(f(x))=g(cosx) = 3 (cos x)2

= 3 cos2 x Similarly fog(x)=f(g (x))= f(3x2)= cos (3x2) Note that 3cos2 x ne cos 3x2 for x = 0 Hence gof ne fogExample 3 Show that if f A rarr B and g B rarr C are onto then gof A rarr C is also ontoSolution Given an arbitrary element z isin C there exists a pre-image y of z under g such that g (y) = z since g is onto Further for y isin B there exists an element x in A with f(x) = y since f is onto Therefore gof(x) = g (f(x)) = g (y) = z showing that gof is onto Example 4 Let Y = n2 n isin N sub N Consider f N rarr Y as f(n) = n2 Show that

f is invertible Find the inverse of fSolution An arbitrary element y in Y is of the form n2 for some n isin N This implies that n =radicy This gives a function g Y rarr N defined by g (y) =radicy Nowgof (n) = g (n2)=radicn2 = n and fog (y) =f(radicy) = (radicy) 2 y which shows that gof=IN and fog= IY Hence f is invertible with f -1 = g

Political Science

Constitution of India-The Preamble

Summary

Objective of the state-To secure equality of status and of opportunity To promote fraternity among all the citizens To assure the dignity of the individuals and Unity and integrity of the nation

Justice-Justice stands for rule of law absence of arbitrariness and a system of equal rights freedom and opportunities for all in a society India seeks social economic and political justice to ensure equality to its citizens

Liberty-Liberty implies the absence of restraints or domination on the activities of an individual such as freedom from slavery serfdom imprisonment despotism etc The Preamble provides for the liberty of thought expression belief faith and worship

Equality-Equality means the absence of privileges or discrimination against any section of the society The Preamble provides for equality of status and opportunity to all the people of the country

Fraternity-The Preamble declares that fraternity has to assure two thingsmdashthe dignity of the individual and the unity and

Execution

Answer the following questions-

Short notes-1 Equality2 Fraternity3 Justice4 Liberty

Homework-Learn

integrity of the nation The word integrity has been added to the Preamble by the 42nd Constitutional Amendment (1976)

Business studies

Human resource management (chapter 1)

On the day of 1504 2020 I have discussed with you the managerial functions and procurement functions of HRM

Today weare going to discuss about the development function integration functions and maintenance function

Development functions-HRM improves the knowledge skills attitude and values of employees so that they the present and future jobs more effectively it includes

1) Development functions of HRM

a) Performance appraisal = It implies systematic evaluation of employees with respect to their performance on the job and their potential for development

b) Training =It is the process by which employees learn knowledge skills and attitudes to achieve organisational and personal goals

c) Executive development = It is the process of developing managerial talent through appropriate program

2) Integration functionsa) HRM reconcile the goals of

organisation with those of its members through integrating function

b) HRM tries to motivate employees to various financial and non financial incentives provided in job specification etc

3) Maintenance functiona) HRM promote and protect the

physical and mental health of employees by providing several types of benefits like housing medical aid etc

b) It Promote Social security measures to employees by providing provident fund pension gratuity maternity benefits

SubjectCOMMERCE

Topic

BUSINESSENVIRONMENT

Summary

Now quickly let us revise the earlier points that we have already done in the last class and let us proceed with the other topics that are there in the chapter

Firstly we will recall the internal and external factors of micro environment and then we

Execution 3 What do you mean by internal factors

in micro environmentAnswerInternal factors refer to all the factors existing within a business firm The internal factors are considered controllable because the enterprise has control over these factors

Development FunctionsPerformance AppraisalTrainingExecution Development

shall proceed in details

Meaning and list of internal and external factors

aInternal factorsInternal factors refer to all the factors existing within a business firm The internal factors are considered controllable because the enterprise has control over these factorsFor an example a company can alter its organization structure policies programmes employees physical facilities and marketing mix to suit the changes in the environmentList of internal factors areCorporate culture mission and objectives top management organizations structure company image and brand equity company resources

b External factorsExternal factors refer to those individual and groups and agencies with which a particular business organization comes into direct and frequent contact in the course of its functioningThese individuals and groups are known as STAKEHOLDERS because they have a stake (financial interest ) in the working and performance of the particular business List of external forces (stakeholders)Customers competitors investors suppliersmiddlemen (marketing intermediaries)financers publics

customers

suppliersfinancers

For an example a company can alter its organization structure policies programmes employees physical facilities and marketing mix to suit the changes in the environment

4 What do you mean by external factors in micro environment

AnswerExternal factors refer to those individual and groups and agencies with which a particular business organization comes into direct and frequent contact in the course of its functioningThese individuals and groups are known as STAKEHOLDERS because they have a stake (financial interest) in the working and performance of the particular business

3Who are stakeholdersSTAKEHOLDERS are individuals and groups who have a stake (financial interest ) in the working and performance of the particular business 4Discuss the internal factors in briefa Corporate CultureThe values beliefs and attitudes of the founders and top management of the company exercise a strong influence on what the cmpaany stands for how it does things and what it considers importantbMission and objectivesThe business philosophy and purpose of a comoany guide it prioritiesbusiness strategiesproduct market scope and development scope

cTop management structurethe composition of board of directors the degree of professionalization of management and the organizational structure of a company have important bearing on its business decisions

dPower structureThe internal power relationship between the board of directors and the chief executive is an important factor

e Company image and brand equityThe image and brand equity of the company play a significant role in raising finance forming alliance choosing dealers and suppliers launching new products entering foreign markets

5 What is Macro environmentAnswerMacro environment refers to the general

competitors

middlemen

publics

Fig STAKEHOLDERS OF A COMPANY

Apart from micro environment the other main dimension of business environment isMacro environment Macro environment refers to the general environment or remote environment within which a business firm and forces in its micro environment operateA company does not directly or regularly interact with the micro environmentTherefore macro environment is also known as indirect action EnvironmentThe macro environment forces are less controllable than the micro forces

Macro environment consists of the following components

POLITICAL AND LEGAL ENVIRONMENT

ECONOMIC SOCIAL AND ENVIRONMENT

CULTURAL

ENVIRONMENT

TECHNOLOGICAL ENVIRONMENT

Fig COMPONENTS OF MACRO ENVIRONMENT

environment or remote environment within which a business firm and forces in its micro environment operateA company does not directly or regularly interact with the micro environmentTherefore macro environment is also known as indirect action EnvironmentThe macro environment forces are less controllable than the micro forces 6 What are the components of macro environmenta Political and legal environmentb Economic environmentc Social and cultural environmentd Technological environment

Computer Science

Logic gates

Digital systems are said to be constructed by using logic gates These gates are the AND OR NOT NAND NOR EXOR and EXNOR

BUSINESS FIRM

gates The basic operations are described below with the aid of truth tables

AND gate

The AND gate is an electronic circuit that gives a high output (1) only if all its inputs are high A dot () is used to show the AND operation ie AB Bear in mind that this dot is sometimes omitted ie ABOR gate

The OR gate is an electronic circuit that gives a high output (1) if one or more of its inputs are high A plus (+) is used to show the OR operationNOT gate

The NOT gate is an electronic circuit that produces an inverted version of the input at its output It is also known as an inverter If the input variable is A the inverted output is known as NOT A This is also shown as A or A with a bar over the top as shown at the outputs The diagrams below show two ways that the NAND logic gate can be configured to produce a NOT gate It can also be done using NOR logic gates in the same way

NAND gate

This is a NOT-AND gate which is equal to an AND gate followed by a NOT gate The outputs of all NAND gates are high if any of the inputs are low The symbol is an AND gate with a small circle on the output The small circle represents inversion

NOR gate

This is a NOT-OR gate which is equal to an OR gate followed by a NOT gate The outputs of all NOR gates are low if any of the inputs are highThe symbol is an OR gate with a small circle on the output The small circle represents inversion

EXOR gate

The Exclusive-OR gate is a circuit which will give a high output if either but not both of its two inputs are high An encircled plus sign ( ) is used to show the EOR operation

EXNOR gate

The Exclusive-NOR gate circuit does the opposite to the EOR gate It will give a low output if either but not both of its two inputs are high The symbol is an EXOR gate with a small circle on the output The small circle represents inversion The NAND and NOR gates are called universal functions since with either one the AND and OR functions and NOT can be generated

Note A function in sum of products form can be implemented using NAND gates by replacing all AND and OR gates by NAND gates A function in product of sums form can be implemented using NOR gates by replacing all AND and OR gates by NOR gates

Logic gate symbols

Table 2 is a summary truth table of the inputoutput combinations for the NOT gate together with all possible inputoutput combinations for the other gate functions Also note that a truth table with n inputs has 2n rows You can compare the outputs of different gates

Logic gates representation using the Truth table

Example

A NAND gate can be used as a NOT gate using either of the following wiring configurations

Subject Eng Literature (The Tempest ndash William Shakespeare) Topic Act III Scene 3 Lines 53 to 110 (End of the scene) Date 16th April 2020 (2nd Period)

[Students should read the original play and also the paraphrase given in the school prescribed textbook]Summary Questions amp Answers

o Seeing this strange scene all are inclined to believe the tales told by travelers that there truly are ldquounicornsrdquo and ldquothe phoenixrsquo thronerdquo

o As they are about to sit down to the feast the banquet is snatched away by a harpy (Ariel disguised) A spiritrsquos voice (Arielrsquos voice) denounces Alonso Sebastian and Antonio with particular

1 ARIEL You are three men of sin whom Destiny

(Line 53-58)That hath to instrument this

lower world And what is int the never-surfeited sea

Hath caused to belch up you and on this island

Where man doth not inhabit you rsquomongst men

Being most unfit to live I have made you mad

reference to their crime in expelling Prospero from Milan They have not received any punishment for their deed earlier but the time for their punishment has arrived Upon Alonso it pronounces ldquolingering perdition worse than deathrdquo from which there is no remedy except through sincere repentance Ariel then vanishes in thunder and the shapes enter again and carry away the table

o Prospero watching invisibly is very pleased with the performance of Ariel and his (Prosperorsquos) ldquomeaner ministersrdquo All his enemies are now in his power and are in a fit of desperation He then leaves them and goes to see how Ferdinand and Miranda are getting on

o Alonso is now much humbled and penitent with the after effect of the spiritrsquos denunciation of his crimes He believes that his son is lost forever After this all disperse being stricken mad by the speech of the spirit

o Gonzalo fearing that they may do violence to themselves or to one another follows them and bid others to follow

(a) To whom does Ariel disguised as a harpy call the three sinners What game did Fate of Destiny play with

them

The three sinners called by Ariel are Alonso Sebastian and Antonio It was Destiny which had caused the ocean to cast the three sinners on the shore Though the ocean is all the time devouring whatever appears on its surface and is never satisfied with its continual swallowing of the ships and men in the present case the ocean had cast these three sinners on the shore without killing them

(b) Who had jointly been responsible for the conspiracy against Prospero What is Prosperorsquos purpose behind all this

Three men Alonso Sebastian and Antonio had jointly

been responsible for the conspiracy against Prospero They had driven out Prospero form Milan Prosperorsquos purpose is to make these three sinners realize the wrong they had done He wants them to repent for their criminal deeds because repentance leads to self-esteem(c )What does Ariel (the harpy) tell Alonso and his companions when they take out their swords to attack him

Seeing them drawing their swords Ariel (harpy) tells them that he and his companions are the instruments of destiny and that it is not possible for human beings to do them any injury He says that the swords of human beings can not injure even a minute part of his feathers Their swords are as ineffective against him and his companions as against the wind or the water

(d) Give the explanatory meanings of the following expressions in the context of the above extract

(i)Never surfeited (ii) Belch up (iii) lsquomongst men

(i) Never surfeited never led to satisfaction

(ii) Belch up cast ashore(iii) lsquomongst men in human

society2

I and my fellows (Line 60-65)

Are ministers of Fate The elementsOf whom your swords are tempered may as wellWound the loud winds or with bemocked-at stabsKill the still-closing waters as diminishOne dowl thats in my plume

IMPORTANT PASSAGES EXPLAINED

The elements

(Line 61-66)Of whom your swords are tempered may

as wellWound the loud winds or with

bemocked-at stabs

(a) Who is lsquoIrsquo Who are his lsquofellowsrdquo

lsquoIrsquo is referred to Ariel in disguise of a harpy His lsquofellowsrsquo are other spirits serving Prospero the real Duke of Milan who has acquired supernatural powers after being banished from his Dukedom Prospero has settled in this uninhabited island

(b) What are the elements that have temperrsquod the swords Why will it not work against the speaker

The swords (of Alonso and his companions) are tempered by metal (steel) which is taken out of the earth and refined by

Kill the still-closing waters as diminishOne dowl thats in my plume My fellow

ministersAre like invulnerable

In these words Ariel reminds the King and his companions of the utter futility of drawing swords against himself and his fellows Ariel drives Alonso Antonio and Sebastian the three men of sin to desperation ndash a state in which men do violence to themselves They draw swords to strike Ariel But Ariel reminds them that he and the other spirits are the ministers of destiny and nothing can wound them The steel of which their swords are made of may cut the wind or water which being divided always closes up again Even supposing that such things may be possible it is quite impossible that their swords will cut one feather in their plume They are incapable of being wounded by any sword of man Hence it is foolish on their part to attempt to strike at Ariel and his fellow-spirits

For which foul deed

(Line 72-75)The powers delaying not forgetting

haveIncensed the seas and shores yea all the

creatures Against your peace

Ariel enters like a harpy and remaining invisible tells Alonso Sebastian and Antonio that he and other harpies are the agents of Destiny appointed to carry out her decrees He tells them that their punishment for the crime against Prospero which has been so long deferred is now to fall upon them He reminds them that they had expelled Prospero from Milan and set him and his innocent child adrift on the sea and that the sea had paid them back for their sin by the shipwreck and by the calamities they have suffered He tells them that the powers above which did not forget this mean treachery but only deferred the punishment have now engaged the seas and the shores and all living beings including him and his comrades against them The very elements and supernatural agency Ariel adds have taken up the avenging of their crime against Prospero

the action of fire It may cut the wind or water which being divided always closes up again

The sword will not work against the spirits and the harpy because they are the ministers of destiny and nothing can wound them nor it will cut a single feather in their plume

(c )What is the meaning of lsquodowlrsquo in the last line

The term lsquodowlrsquo means a filament or the smallest part of a feather In this context Ariel in disguise of harpy says that their sword cannot even damage the smallest filament of their (Arielrsquos and other spirits) feathers as they are incapable of being wounded by any sword of man

(d) What does the speaker remind the listeners about

Ariel in disguise of harpy reminds Alonso the King of Naples Sebastian Alonsorsquos brother and Antonio the present Duke of Milan and the treacherous brother of Prospero as they being three men of sin He even reminds them that their punishment for their crime against Prospero which has been so long deferred now falls upon them He reminds them that they have expelled Prospero from Milan and has set him along with his innocent infant daughter adrift on the sea So the sea has paid them back for their sin by their shipwreck and the calamities they have suffered since then The harpy rebukes Alonso of his sin that has incensed the Gods and has deprived him of his son as a punishment

(e) How do they respond

When Ariel in disguise of a harpy reminds Alonso Sebastian and Antonio of their past misdeeds and sin Alonso has a look of terror and confusion in his eyes He utters the words of sincere repentance wrung out of his conscience-stricken heart It appears to him that all the elements of nature the sea-waves the wind and the thunder proclaiming a loud voice in the name of Prospero and the crime Alonso has committed against him They are calling upon him to repent There is a deep storm raging in Alonsorsquos breast and the echoes of that storm are ringing in his ears like a clear note of wind-instrument A note of denunciation of Alonsorsquos crime leaves him much humbled and penitent and confirms his belief that his son is lost forever But Sebastian and Antonio shows some courage instead of repentance They wish to kill the spirits or devils if it appears

3

Of my instruction hast thou nothing bated (Line 85-93)

In what thou hast to say So with good life

And observation strange my meaner ministers

Their several kinds have done My high charms work

And these mine enemies are all knit upIn their distractions They now are in my

powerAnd in these fits I leave them while I visitYoung Ferdinand whom they suppose is

drownedAnd his and mine loved darling

Methought the billows spoke and (Line 96-99)

told me of itThe winds did sing it to me and the

thunderThat deep and dreadful organ-pipe

pronouncedThe name of Prosper It did bass my

trespass

These are the words of contrition coming from Alonso Ariel has driven him to a deep repentance for conspiring with Antonio against Prospero He now feels a sincere remorse It appears to him that all the elements of nature the sea-waves the wind and the thunder proclaimed with a loud voice the name of Prospero and the crime Alonso had committed against him They are calling upon him to repent There is a deep storm raging in Alonsorsquos breast and the echoes of that storm are ringing in his ears like the clear note of a wind-instrument

Comment These are the words of sincere repentance wrung out of the conscience-stricken heart of Alonso Alonso who is the lesser villain is the first to give way to remorse under the effect of Arielrsquos speech The words of Ariel seem to him to be the voice of conscience speaking to him He is driven to desperation a state in which he might do violence to his life

(a) Identify the speaker State the context

Prospero the ruler of the island is the speaker The famous banquet scene has been enacted very well Ariel and his junior spirits have played their roles excellently Prospero is glad to say words of praise for them(b) In what way the speakerrsquos instructions have been carried out

According to Prosperorsquos instructions a banquet was presented before the King of Naples and his companions when they were tired and hungry Just when they were preparing to eat the feast the banquet was suddenly removed by exercising supernatural powers All this was done by Ariel Prosperorsquos chief assistant and a powerful spirit

Ariel not only made the feast disappear but also delivered his speech blaming the King and his two companions for their past wicked deeds He warned them to repent for their misdeeds or suffer forever on that uninhabited island

(c) Who are referred to as lsquomeaner ministersrsquo What have they done

Prospero refers as lsquomeaner ministersrsquo to his other lesser spirits who were assisting Ariel in presenting a scene before the kingrsquos party They entered the scene to the accompaniment of music They assumed several strange shapes and brought in a banquet Then they danced about it with gentle actions of salutations thus inviting the King and others to eat the feast

These spirits play their role again when Ariel in the shape of a harpy quits the scene These shapes enter again and dancing with mocking gestures carry away the table

(d) Who are the speakerrsquos enemies What has happened to them

King of Naples Alonso his brother Sebastian and the present Duke of Milan Antonio (Prosperorsquos own brother) are Prosperorsquos enemies With the turn of events they have all been washed ashore on the island which is ruled by Prospero the great magician Actually this happened after the shipwreck caused by a storm which was raised by Prospero with the purpose of bringing these people to his island Prosperorsquos spirits have already confused and terrified these enemies and they are under Prosperorsquos control He can treat them as he likes

(e) What does he say about Ferdinand Explain what is meant by ldquohellip his and mine darlingrdquo

Prospero knows that Alonsorsquos son prince Ferdinand is alive though his father thinks that the prince has been drowned

Prospero refers to his daughter Miranda who is dear to him She is also very dear to Prince Ferdinand who has fallen in love with her They are waiting to be married soon for which they have received Prosperorsquos consent

4

ALONSO O it is monstrous monstrous (Line 95-102)

Methought the billows spoke and told me of it

The winds did sing it to me and the thunderThat deep and dreadful organ-

pipe pronouncedThe name of Prosper It did bass

my trespassTherefore my son ithrsquo ooze is

bedded andIll seek him deeper than eer

plummet soundedAnd with him there lie mudded

(a) In what way does Alonso express his horror when his conscience is awakened by Arielrsquos words

When Alonsorsquos conscience is awakened by Arielrsquos words he expresses his horror at what he has heard He gets the feeling that the waves of the ocean the wind and the loud thunder have spoken to him and uttered the name of Prospero Because of being reminded of his crime in a very loud and rough voice he comes to realize that he has lost his son for his past misdeeds

(b) What does Alonso imagine about his son What does Alonso want to do in his desperate state

Alonso imagines that his son is lying in the mud at the bottom of the sea He feels desperate that he wants to drown himself in the ocean deeper than the plumb-line has ever gone He wants to lie with his son at the bottom of the sea

(c) How do Sebastian and Antonio want to face the evil spirits

Sebastian says that he is not at all afraid of what the harpy has said and that he is prepared to fight any number of such monsters if they appear before him only one at a time Antonio says that he would support Sebastian in the fight against the fiendsyyy

(d) Why does Gonzalo ask Adrian to follow the three men

Gonzalo tells Adrian that all the three men namely Alonso Sebastian and Antonio are in a wild and reckless mood The thought of the heinous crime of which they are guilty has begun to torment their minds So he asks Adrian to follow those three men without loss of time and prevent them from doing anything which the turmoil in their minds might lead them to do

(e) What opinion do you form of Alonso from the above extract

Alonso who is the lesser villain is the first to give way to remorse under the effect of Arielrsquos speech The words of Ariel seem to him to be the voice of conscience speaking to him He is driven to desperation a state in which he might do violence to his life

Subject =Accounts

Ac-12 15420 topic-pL Appropriation ac

PROFIT AND LOSS APPROPRIATION ACCOUNT

MEANING AND PREPARATIONProfit and Loss Appropriation Account is merely an extension of the Profit and Loss Account of the firm The profit of the firm has to be distributed amongst the partners in their respective profit sharing ratio But before its distribution it needs to be adjusted All Adjustments like partnerrsquos salary partnerrsquos commission interest on capital interest on drawings etc are made in this account These adjustments will reduce the amount of profit for distribution This adjusted profit will be distributed amongst the partners in their profit sharing ratio To prepare it at first the balance of Profit and Loss Account is transferred to this account The journal entries for the preparation of Profit and Loss Appropriation Account are given below

1 for transfer of the balance of Profit and Loss Account to Profit and Loss Appropriation Account

(a) In case of Net Profit

Profit and Loss Ac helliphelliphelliphelliphellipDrTo Profit and Loss Appropriation Ac(Net Profit transferred to Profit and Loss Appropriation Ac)

(b)In case of Net Loss

Profit and Loss Appropriation Achelliphelliphellip DrTo Profit and Loss Ac(Net Loss transferred to Profit and Loss Appropriation Ac)

2 for Interest on Capital

For transferring on Interest on CapitalProfit and Loss Appropriation Achelliphelliphellip DrTo Interest on Capital Ac(Interest on capital transferred to Profit amp Loss Appropriation Ac)

3 for Interest on Drawings

For transferring Interest on Drawings Interest on Drawings Achelliphelliphelliphelliphelliphellip DrTo Profit and Loss Appropriation Ac(Interest on drawing transferred to Profit amp Loss Appropriation Ac)

4 For Partnerrsquos SalaryFor transfer of partnerrsquos SalaryProfit and Loss Appropriation Achelliphellip DrTo Salary Ac(Salary transferred to profit amp Loss Appropriation Ac)

5 For Partnerrsquos CommissionFor transferring commissionProfit and Loss Appropriation Achelliphelliphellip DrTo Commission Ac(Commission transferred to Profit and Loss Appropriation Ac)

6 For Transfer of agreed amount to General ReserveProfit and Loss Appropriation Ac helliphellipDrTo General Reserve Ac(Transfer to General Reserve)

7 for share of Profit or Loss appropriation(a) If ProfitProfit and Loss Appropriation Achelliphellip DrTo Partnerrsquos CapitalCurrent Ac(Profit transferred to capitalcurrent Ac)(b) If LossPartnerrsquos Capital Current Achelliphelliphelliphellip DrTo Profit and Loss Appropriation Ac(Loss transferred to capitalcurrent Ac)

THE FORMAT OF PROFIT AND LOSS APPROPRIATION

Profit and Loss Appropriation Account for the year endedhelliphelliphelliphellip

Particulars Amount Particulars Amount

To PL Ac (loss) By pL Ac (profit)

To Interest on capital BY Interest on drawings

To partner`s commission by Partner`s capital Ac ( loss)

To Partner`s salary To Interest on partner`s loan To General Reserve To Partner`s Capital AC (Profit)

Subject= Economics

MOVEMENT ALONG THE DEMAND CURVE (CHANGE IN QUANTITY DEMANDED)In law of demand you have already studied the inverse relationship between price and quantity demanded When quantity demanded of a commodity changes due to change in its price keeping other factors constant it is called change in quantity demanded It is graphically expressed as a movement along the same demand curve There can be either a downward movement or an upward movement along the same demand curve Upward movement along the same demand curve is called contraction of demand or decrease in quantity demanded and downward movement along the same demand curve is known as expansion of demand or increase in quantity demanded

Extention of demandd

price (rs)p A

B Extentionp1 d

Q Q1

Quantity demanded ( in units)

Contraction of demandd

p2 Ccontraction

p APrice (Rs)

d

Q2 Q

Quantity demanded (in units)

Explanation of movement of demand A fall in price from OP to OP1 leads to increase in quantity demanded from OQ to OQ1 (expansion of demand) resulting in a downward movement from point A to point B along the same demand curve DD When Price rises from OP to OP2 quantity demanded falls from OQ to OQ2 (contraction of demand) leading to an upward movement from point A to point C along the same demand curve DD

  • Activity Series of Metals
    • Drawbacks of Rutherfordrsquos model of atom
      • Electromagnetic radiations
      • Properties of electromagnetic radiations
      • Characteristics of electromagnetic radiations
        • Plancks Quantum Theory-
        • Photoelectric effect
          • Intext Questions
            • Logic gates
            • Digital systems are said to be constructed by using logic gates These gates are the AND OR NOT NAND NOR EXOR and EXNOR gates The basic operations are described below with the aid of truth tables
            • AND gate
            • Example
Page 40:  · Web viewSubject. Topic. Summary. Execution. English 1 . Chapter 1 naming words . Page 8. Write the names of these pictures:- Person:-1. father. 2.Firefighter 3.doctor 4 ...

(iii) The opening scene justifies the title ndash The Tempest

UNANSWERED QUESTIONS

(i) The King always travels with his entire fleet including his soldiers Where were the other ships

(ii) Why was the ship in that area Where was it coming from or going where

(iii) The ship broke apart What happened to those who were in the ship

(We shall get the answer to the above questions as the play progresses)

leave of life with fervent prayers The mariners take their last hearty drink and are ready for death(d) What blame does Antonio put upon the mariners and the boatswain Antonio rebukes the mariners that these drunkards have brought them to the present crisis by neglecting their duties He blames them saying that they are going to lose their lives entirely for the negligence of the boatswain and his fellows(e) What does Antonio say while cursing the boatswain

Antonio gives vent to his wrath upon the boatswain in particular He calls the boatswain a wide-mouthed rascal who deserves to be hanged on the sea-shore at low water mark so that ten tides might wash over his body and take out of him all the liquor that he has been drinking

Class XIISubject Topic Summary ExecutionHistory Topic

1 1935 ACT AND WORKING OF PROVINCIAL AUTONOMYCONGREE AND OTHER MINISTERSSUB TOPIC GOVERNMENT OF INDIA ACT1935

Government of India Act 1935 This act established a lsquoFederation of Indiarsquo made of British Indian provinces and Indian states and provided for autonomy with a government responsible to the elected legislature in every provinceThis act introduced abolition of Diarchy at provinces The entire provincial administration was introduced to the responsible ministers who were controlled and removed by the provincial legislature The provincial autonomy means two things First The provincial governments were wholly responsible to the provincial legislature Secondly Provinces were free from outside control and interference in the large number of matters The act divided the powers between the centre and provinces in terms of three lists- Federal list( for centre) Provincial list (for province) and concurrent list (for both) Residuary powers were given to the viceroy In the election under the government of India Act the Congress swept the poll the mandate of the people came in favour of the congress so far as general Hindu seats were concerned The Congress did not get a single Muslim seates in Bombay CP UP Sind and BengalIn five provinces Congress had yhe clear majority In BengalNWFPAssam and Bombay Congress emerged as a single largest partyOn the other side the performance of the Muslim League was badThus the Congress formed ministers in 7 provinces out of 11 provinces Coalition ministry was also formed in two other provincesOnly BENGAL AND Punjab had non- congress ministries

1 QUESTION What was the main change introduced by the Government of India ActANSWER a) The Act gave more

autonomy to the provinces b) Diarchy was abolished at the

provincial levelsc) The Governor was the head of

the executived) There was a council of

ministers to advise him The ministers were responsible to the provincial legislatures who controlled them The legislature could also remove the ministers

e) The Governors still retained special reserve powers

2 QUESTION Why did the federal scheme introduced by the Government of India Act 1935 never come into operation

ANSWER The Federal structure of the Government of India was to be composed with the Governor General and Council of ministers The Federal legislature was to be Bicameral legislature- The council of states and the House of Assembly The ministers were to be chosen by the Governor general and they were to hold the office during his pleasure

The provinces of British India would have to join the federation but this was not compulsory for the princely states

This federation never materialised because of the lack of support from the required number of

princely statesThis act was refused and

rejected by the princes the Congress and the Muslim League

Thus both Congress and the League participated in the election of 1937 Thus the federal part was never introduced but the provincial part was put into operations

Bengali 2nd

Language

াচেরর পরাথCনা(কহিতা )

াচেরর পরাথCনা কহিতাটি কহি (ঙখ দেঘাচে4র দো আচো য কহিতায় াচেরর পতর হমায়ন কঠিন দেরাচেগ আxানত ার ঈশবর া আললার কাচেছ পরাথCনা কচেরচেছন তার পচেতরর ীন হিফহিরচেয় হিচেত এই কহিতায় ার পচেতরর ীন হিভbা দেচেয়চেছন ারার এমনহিক হিনচের ীন হিসCচেনর হিহিনমচেয় হিতহিন তার দেছচের ীন হিফচের দেপচেত দেচেয়চেছন তার দেছচের এই দেরাচেগর ন য হিতহিন হিনচেচেকই ায়ী কচেরচেছন তার হিনচের করা পাপচেকই হিতহিন ায়ী কচেরচেছন এছাা রানৈনহিতক ও আথCসামাহিক অসথার কথা তচে ধরা চেয়চেছ এই কহিতায় ার তার হিনচের পাপ কমCচেকই ায়ী কচেরচেছ ার অন যায় ভাচে দেপহি((হিকতর মাধ যচেম অপররা য কচেরচেছ আর এই অন যায় কাচের ন যই তার পহিরাচের হিপযCয় এচেসচেছ দে এক পরকার মানহিক নধন ইহিতাচেসর ার হিপতা চেয় সবাভাহিকভাচে ভাচোাসা দে মমতা দেথচেক মকত চেত পাচেরনহিন তাই হিপতা চেয় আললা া ভগাচেনর কাচেছ পতর হমায়চেনর পরানহিভbা দেচেয়চেছন ার আললা া ভগাচেনর কাচেছ াহিনচেয়চেছন তার হিনচের ীন হিসCন হিচেত হিতহিন রাী তার হিহিনমচেয় পচেতরর ীন হিফচের দেপচেত দেচেয়চেছন াচেরর হিপতসভ হিচেকর কথা এই কহিতায় ফটিচেয় দেতাা চেয়চেছ হিপতা পচেতরর হিরাহিরত মান নধচেনর কথা তচে ধরা চেয়চেছ

হিচে(4 হিকছ াইচেনর তাৎপযC১) ldquoদেকাথায় দেগ ওর সবচছয দেৌন দেকাথায় কচেরায় দেগাপন bয়ldquoউততর) াচেরর পতর হমায়ন কঠিন দেরাচেগ অসসথ তাই তার দেযৌন াহিরচেয় যাচেচছ এই দেরাচেগ তাচেক দেগাপচেন কচেরকচের াচেচছ তার সক (হিকত ধীচের ধীচের bয় চেচছ তাই হিপতা চেয় ার আললার কাচেছ হমায়চেনর পরান হিভbা দেচেয়চেছন২) ldquoাগাও (চেরর পরাচেনত পরানতচের ধসর (ন দেযর আান গানldquoউততর) াচেরর পতর হমায়ন কঠিন দেরাচেগ আxানত তাই ার আ দে(াচেক মমCাত (চেরর পচেথ পরানতচের আান গান ধবহিনত দোক দেসই আান গান আললার কাচেছ দেযন চে যায় আললা দেযন এই আহিতC শচেন পচেতরর ীন হিফহিরচেয় দেয় ৩)ldquoনাহিক এই (রীচেরর পাচেপর ীানচেত দেকানই তরারণ দেনই ভহি4চেতরldquoউততর) হমায়চেনর অসসথতার ন য ার হিনচেচেকই ায়ী কচেরচেছন কারন ার অচেনক রা য অন যায় ভাচে কচেরচেছ তাই তার এই পাপ কাচের ন য তার ঘচের আ হিপ এচেসচেছ এই অন যায় কাচের ন য তার মহিকত দেনই তাই ার আললার কাচেছ এই পাপ কাচেযCর ন য bমা পরাথM

Hindi 2ndlang

-ासी(जयशकर परसा-)

-ासी जयशकर परसा- की एक ऐसी कहानी ह जिजसम भारतीय ससकनित और राषटरीयता का सवरगजीतहोता ह इस कहानी म इरावती एक निहद कनया ह जिजस मलअचछो न मलतान की लट म पकडा और -ासी बना दि-या उस 500 दि-न -कर काशी क एक महाजन न खरी-ा दसरी -ासी निफरोजा ह वह गलाम ह निफरोजा को छडान क कतिलए अहम- को 1000 सोन क कतिसकक भजन थ जो अभी तक नही आए थ राजा साहब कठोर होत हए भी निफरोजा को निबना धनराकतिश क कतिलए उस म कर -त ह वनिफरोजा को अहम- को समझान की बात कहत हकहानी क अत म हम -खत ह निक इरा वती और जाटो क सर-ार बलराज का मिमलन होता हअहम- को यa म मार दि-या जाता ह वहा निफरोजा की परसननता की समामिध बनती ह वहा एक फल चढती ह और डीजल आती ह निफरोजा उस समामिध की आजीवन -ासी बनी रहती हलखक अपन उददशय अथात -ास परथा पर परकाश डालन और इस परथा क कारण होन वाल -ातो क दखो को दि-खान म पणता सफल हए ह

helliphellipContinue to next

Biology Reproductio Today we will discuss about vegetative Q1 Name some vegetative propagules

n in Organisms

propagation of plants The process of multiplication in which fragments of plant body function as propagule and develop into new individual is called vegetative propagation The units of such propagation are runner rhizome tuber bulb etc

and the speciesinvolvedVegetative propagules

Parts involved

Bulb StemBulbil BulbilRhizome Stem Runner Stem Tuber Stem Offset Stem Leaf buds Leaves Suckers Stem

Corns Stem stolon

Q2 State advantages of vegetative propagation

i) Rapid methodii) Sure and easy methodiii) Useful in plants that cannot

produce viable seeds or long seed dormancy

iv) Maintains purity of raceQ 3 Banana fruit is said to be parthenocarpic where as turkey is said to be parthenogenetic WhyBanana develops without fertilization from an unfertilized ovary thus is parthenocarpicIn turkey the ovum or female gamete developinto a new chick without fertilization thus isparthgenetic

Q4 Why is water hyacinth is called as a ldquoTerror of Bengalrdquo Water hyacinth can

propagatevegetatively all over the water body in a short per short period of time This resulted increased biochemicaloxygen oxygen demand of water body causing mortalityof fishes It is very difficult to get rid off them Thus known as terror of Bengal

Chemistry

Solid state GENERAL CHARACTERISTICS OF SOLID STATEIn nature the particular state of matter is governed by two opposing forces at given set of temperature and pressure These forces are intermolecular force of attraction and thermal energy If intermolecular force of attraction is high as compared to thermal energy particles remains in closest position

Intext QuestionsQ1 Classify the following solids as crystalline and amorphous Sodium chloride quartz glass quartz rubber polyvinyl chloride Teflon

A1 Crystalline

and hence very less movement in particles is observed In this case solid state is the preferred state of matter

Let us revise the general characteristics of solid

i) Fixed mass volume and shape

ii) Strong intermolecular force of attraction

iii) Least intermolecular space

iv) Fixed position of constituent particles

v) Incompressible and rigid

Q2 what type of interactions hold the molecules together in a polar molecular solid[CBSE 2010]A2 The molecules in a solid are held together by van der Waals forces The term van der Waals forces include hydrogen bonding dipole-dipole attraction and London dispersion forces All molecules experience London dispersion forces In addition polar molecules can also experience dipole-dipole interactions So the interactions that holds the molecule together in polar molecular solid are London dispersion force and dipole-dipole interactionsQ3 Write a feature that will distinguish a metallic solid from an ionic solid [CBSE 2010]A3 Metals are malleable and ductile whereas ionic solid are hard and brittle Metallic solid has typical metallic lustre But ionic solid looks dullQ4 Write a point of distinction between a metallic solid and an ionic solid other than metallic lustre [CBSE 2012]A4 Metals are malleable and ductile whereas ionic solid are hard and brittleQ5 Write a distinguish feature of metallic solid [CBSE 2010]A5 The force of attraction in

solid Sodium chloride Quartz Amorphous solid Quartz glass rubber polyvinyl chloride Teflon Q2 why glass is considered as super cooled liquidA2 Glass shows the tendency to flow at slower rate like liquid Hence they considered as super cooled liquidQ3 why the window glass of old buildings show milky appearance with timeA3 Glass is an amorphous solid Amorphous solid has the tendency to develop some crystalline character on heating Due to heating in day over the number of years glass acquires some crystalline character and show milky appearanceQ4 why the glass panes fixed to window or doors of old building become slightly thicker at bottomA4 Glass is super cooled liquid It has the tendency to flow down very slowly Due to this glass pane becomes thicker at the bottom over the timeQ5 Sodium chloride is a crystalline solid It shows the same value of refractive index along all the direction TrueFalse Give reasonA5 FalseCrystalline solid shows anisotropy in properties That is it shows different values for the given physical property in different direction All the crystalline solids show anisotropy in refractive index Therefore sodium chloride will show different values of refractive index on different directions

Q6 Crystalline solid are anisotropic in nature What does this statement means

between the constituent particles is special kind of electrostatic attraction That is the attraction of positively charged kernel with sea of delocalized electronsQ6 which group of solid is electrical conductor as well as malleable and ductile [CBSE 2013]A6 Metallic solidQ7 why graphite is good conductor of electricity although it is a network (covalent solid)A7 The exceptional property of graphite is due to its typical structure In graphite each carbon is covalently bonded with 3 atoms in same layer The fourth valence electron of each atom is free to move in between different layersThis free electron makes the graphite a good conductor of electricity

[CBSE 2011]A6 Anisotropy is defined asrdquo Difference in properties when measured along different axis or from different directionsrdquo Crystalline solid show different values of some of the physical properties like electrical resistance refractive index etcwhen measured along the different directions The anisotropy in crystalline solid arises due to the different arrangement of particles in different directions

Math Function Composition of functions Think of an industrial plant that produce bottles of cold drinks first there is the operation (or function) f that puts the cold drink inside the bottle followed by the opeartion g that close the bottle with the capThis leads to the following definitionDefinition Let f A rarr B and g B rarr C be two functions Then the composition of f and g denoted by gof is defined as the function gof A rarr C given by gof(x) = g(f (x)) forall x isinA

Definition A function f X rarr Y is defined to be invertible if there exists a function g Y rarr X such that gof = IX and fog = IY The function g is called the inverse of f and is denoted by f -1

Thus if f is invertible then f must be one-one and onto and conversely if f is one-one and onto then f must be invertible This fact significantly helps for proving a function f to be invertible by showing that f is one-one and onto specially when the actual inverse of f is not to be determined

Example 1 Let f 2 3 4 5 rarr 3 4 5 9 and g 3 4 5 9 rarr 7 11 15 be functions defined as f(2) = 3 f(3) = 4 f(4) = f(5) = 5 and g (3) = g (4) = 7 and g (5) = g (9) = 11 Find gofSolution We have gof(2) = g (f(2)) = g (3) = 7 gof(3) = g (f(3)) = g (4) = 7gof(4) = g (f(4)) = g (5) = 11 and gof(5) = g (5) = 11Example 2 Find gof and fog if f R rarr R and g R rarr R are given by f(x) = cos x and g (x) = 3x2 Show that gof ne fogSolution We have gof(x) = g(f(x))=g(cosx) = 3 (cos x)2

= 3 cos2 x Similarly fog(x)=f(g (x))= f(3x2)= cos (3x2) Note that 3cos2 x ne cos 3x2 for x = 0 Hence gof ne fogExample 3 Show that if f A rarr B and g B rarr C are onto then gof A rarr C is also ontoSolution Given an arbitrary element z isin C there exists a pre-image y of z under g such that g (y) = z since g is onto Further for y isin B there exists an element x in A with f(x) = y since f is onto Therefore gof(x) = g (f(x)) = g (y) = z showing that gof is onto Example 4 Let Y = n2 n isin N sub N Consider f N rarr Y as f(n) = n2 Show that

f is invertible Find the inverse of fSolution An arbitrary element y in Y is of the form n2 for some n isin N This implies that n =radicy This gives a function g Y rarr N defined by g (y) =radicy Nowgof (n) = g (n2)=radicn2 = n and fog (y) =f(radicy) = (radicy) 2 y which shows that gof=IN and fog= IY Hence f is invertible with f -1 = g

Political Science

Constitution of India-The Preamble

Summary

Objective of the state-To secure equality of status and of opportunity To promote fraternity among all the citizens To assure the dignity of the individuals and Unity and integrity of the nation

Justice-Justice stands for rule of law absence of arbitrariness and a system of equal rights freedom and opportunities for all in a society India seeks social economic and political justice to ensure equality to its citizens

Liberty-Liberty implies the absence of restraints or domination on the activities of an individual such as freedom from slavery serfdom imprisonment despotism etc The Preamble provides for the liberty of thought expression belief faith and worship

Equality-Equality means the absence of privileges or discrimination against any section of the society The Preamble provides for equality of status and opportunity to all the people of the country

Fraternity-The Preamble declares that fraternity has to assure two thingsmdashthe dignity of the individual and the unity and

Execution

Answer the following questions-

Short notes-1 Equality2 Fraternity3 Justice4 Liberty

Homework-Learn

integrity of the nation The word integrity has been added to the Preamble by the 42nd Constitutional Amendment (1976)

Business studies

Human resource management (chapter 1)

On the day of 1504 2020 I have discussed with you the managerial functions and procurement functions of HRM

Today weare going to discuss about the development function integration functions and maintenance function

Development functions-HRM improves the knowledge skills attitude and values of employees so that they the present and future jobs more effectively it includes

1) Development functions of HRM

a) Performance appraisal = It implies systematic evaluation of employees with respect to their performance on the job and their potential for development

b) Training =It is the process by which employees learn knowledge skills and attitudes to achieve organisational and personal goals

c) Executive development = It is the process of developing managerial talent through appropriate program

2) Integration functionsa) HRM reconcile the goals of

organisation with those of its members through integrating function

b) HRM tries to motivate employees to various financial and non financial incentives provided in job specification etc

3) Maintenance functiona) HRM promote and protect the

physical and mental health of employees by providing several types of benefits like housing medical aid etc

b) It Promote Social security measures to employees by providing provident fund pension gratuity maternity benefits

SubjectCOMMERCE

Topic

BUSINESSENVIRONMENT

Summary

Now quickly let us revise the earlier points that we have already done in the last class and let us proceed with the other topics that are there in the chapter

Firstly we will recall the internal and external factors of micro environment and then we

Execution 3 What do you mean by internal factors

in micro environmentAnswerInternal factors refer to all the factors existing within a business firm The internal factors are considered controllable because the enterprise has control over these factors

Development FunctionsPerformance AppraisalTrainingExecution Development

shall proceed in details

Meaning and list of internal and external factors

aInternal factorsInternal factors refer to all the factors existing within a business firm The internal factors are considered controllable because the enterprise has control over these factorsFor an example a company can alter its organization structure policies programmes employees physical facilities and marketing mix to suit the changes in the environmentList of internal factors areCorporate culture mission and objectives top management organizations structure company image and brand equity company resources

b External factorsExternal factors refer to those individual and groups and agencies with which a particular business organization comes into direct and frequent contact in the course of its functioningThese individuals and groups are known as STAKEHOLDERS because they have a stake (financial interest ) in the working and performance of the particular business List of external forces (stakeholders)Customers competitors investors suppliersmiddlemen (marketing intermediaries)financers publics

customers

suppliersfinancers

For an example a company can alter its organization structure policies programmes employees physical facilities and marketing mix to suit the changes in the environment

4 What do you mean by external factors in micro environment

AnswerExternal factors refer to those individual and groups and agencies with which a particular business organization comes into direct and frequent contact in the course of its functioningThese individuals and groups are known as STAKEHOLDERS because they have a stake (financial interest) in the working and performance of the particular business

3Who are stakeholdersSTAKEHOLDERS are individuals and groups who have a stake (financial interest ) in the working and performance of the particular business 4Discuss the internal factors in briefa Corporate CultureThe values beliefs and attitudes of the founders and top management of the company exercise a strong influence on what the cmpaany stands for how it does things and what it considers importantbMission and objectivesThe business philosophy and purpose of a comoany guide it prioritiesbusiness strategiesproduct market scope and development scope

cTop management structurethe composition of board of directors the degree of professionalization of management and the organizational structure of a company have important bearing on its business decisions

dPower structureThe internal power relationship between the board of directors and the chief executive is an important factor

e Company image and brand equityThe image and brand equity of the company play a significant role in raising finance forming alliance choosing dealers and suppliers launching new products entering foreign markets

5 What is Macro environmentAnswerMacro environment refers to the general

competitors

middlemen

publics

Fig STAKEHOLDERS OF A COMPANY

Apart from micro environment the other main dimension of business environment isMacro environment Macro environment refers to the general environment or remote environment within which a business firm and forces in its micro environment operateA company does not directly or regularly interact with the micro environmentTherefore macro environment is also known as indirect action EnvironmentThe macro environment forces are less controllable than the micro forces

Macro environment consists of the following components

POLITICAL AND LEGAL ENVIRONMENT

ECONOMIC SOCIAL AND ENVIRONMENT

CULTURAL

ENVIRONMENT

TECHNOLOGICAL ENVIRONMENT

Fig COMPONENTS OF MACRO ENVIRONMENT

environment or remote environment within which a business firm and forces in its micro environment operateA company does not directly or regularly interact with the micro environmentTherefore macro environment is also known as indirect action EnvironmentThe macro environment forces are less controllable than the micro forces 6 What are the components of macro environmenta Political and legal environmentb Economic environmentc Social and cultural environmentd Technological environment

Computer Science

Logic gates

Digital systems are said to be constructed by using logic gates These gates are the AND OR NOT NAND NOR EXOR and EXNOR

BUSINESS FIRM

gates The basic operations are described below with the aid of truth tables

AND gate

The AND gate is an electronic circuit that gives a high output (1) only if all its inputs are high A dot () is used to show the AND operation ie AB Bear in mind that this dot is sometimes omitted ie ABOR gate

The OR gate is an electronic circuit that gives a high output (1) if one or more of its inputs are high A plus (+) is used to show the OR operationNOT gate

The NOT gate is an electronic circuit that produces an inverted version of the input at its output It is also known as an inverter If the input variable is A the inverted output is known as NOT A This is also shown as A or A with a bar over the top as shown at the outputs The diagrams below show two ways that the NAND logic gate can be configured to produce a NOT gate It can also be done using NOR logic gates in the same way

NAND gate

This is a NOT-AND gate which is equal to an AND gate followed by a NOT gate The outputs of all NAND gates are high if any of the inputs are low The symbol is an AND gate with a small circle on the output The small circle represents inversion

NOR gate

This is a NOT-OR gate which is equal to an OR gate followed by a NOT gate The outputs of all NOR gates are low if any of the inputs are highThe symbol is an OR gate with a small circle on the output The small circle represents inversion

EXOR gate

The Exclusive-OR gate is a circuit which will give a high output if either but not both of its two inputs are high An encircled plus sign ( ) is used to show the EOR operation

EXNOR gate

The Exclusive-NOR gate circuit does the opposite to the EOR gate It will give a low output if either but not both of its two inputs are high The symbol is an EXOR gate with a small circle on the output The small circle represents inversion The NAND and NOR gates are called universal functions since with either one the AND and OR functions and NOT can be generated

Note A function in sum of products form can be implemented using NAND gates by replacing all AND and OR gates by NAND gates A function in product of sums form can be implemented using NOR gates by replacing all AND and OR gates by NOR gates

Logic gate symbols

Table 2 is a summary truth table of the inputoutput combinations for the NOT gate together with all possible inputoutput combinations for the other gate functions Also note that a truth table with n inputs has 2n rows You can compare the outputs of different gates

Logic gates representation using the Truth table

Example

A NAND gate can be used as a NOT gate using either of the following wiring configurations

Subject Eng Literature (The Tempest ndash William Shakespeare) Topic Act III Scene 3 Lines 53 to 110 (End of the scene) Date 16th April 2020 (2nd Period)

[Students should read the original play and also the paraphrase given in the school prescribed textbook]Summary Questions amp Answers

o Seeing this strange scene all are inclined to believe the tales told by travelers that there truly are ldquounicornsrdquo and ldquothe phoenixrsquo thronerdquo

o As they are about to sit down to the feast the banquet is snatched away by a harpy (Ariel disguised) A spiritrsquos voice (Arielrsquos voice) denounces Alonso Sebastian and Antonio with particular

1 ARIEL You are three men of sin whom Destiny

(Line 53-58)That hath to instrument this

lower world And what is int the never-surfeited sea

Hath caused to belch up you and on this island

Where man doth not inhabit you rsquomongst men

Being most unfit to live I have made you mad

reference to their crime in expelling Prospero from Milan They have not received any punishment for their deed earlier but the time for their punishment has arrived Upon Alonso it pronounces ldquolingering perdition worse than deathrdquo from which there is no remedy except through sincere repentance Ariel then vanishes in thunder and the shapes enter again and carry away the table

o Prospero watching invisibly is very pleased with the performance of Ariel and his (Prosperorsquos) ldquomeaner ministersrdquo All his enemies are now in his power and are in a fit of desperation He then leaves them and goes to see how Ferdinand and Miranda are getting on

o Alonso is now much humbled and penitent with the after effect of the spiritrsquos denunciation of his crimes He believes that his son is lost forever After this all disperse being stricken mad by the speech of the spirit

o Gonzalo fearing that they may do violence to themselves or to one another follows them and bid others to follow

(a) To whom does Ariel disguised as a harpy call the three sinners What game did Fate of Destiny play with

them

The three sinners called by Ariel are Alonso Sebastian and Antonio It was Destiny which had caused the ocean to cast the three sinners on the shore Though the ocean is all the time devouring whatever appears on its surface and is never satisfied with its continual swallowing of the ships and men in the present case the ocean had cast these three sinners on the shore without killing them

(b) Who had jointly been responsible for the conspiracy against Prospero What is Prosperorsquos purpose behind all this

Three men Alonso Sebastian and Antonio had jointly

been responsible for the conspiracy against Prospero They had driven out Prospero form Milan Prosperorsquos purpose is to make these three sinners realize the wrong they had done He wants them to repent for their criminal deeds because repentance leads to self-esteem(c )What does Ariel (the harpy) tell Alonso and his companions when they take out their swords to attack him

Seeing them drawing their swords Ariel (harpy) tells them that he and his companions are the instruments of destiny and that it is not possible for human beings to do them any injury He says that the swords of human beings can not injure even a minute part of his feathers Their swords are as ineffective against him and his companions as against the wind or the water

(d) Give the explanatory meanings of the following expressions in the context of the above extract

(i)Never surfeited (ii) Belch up (iii) lsquomongst men

(i) Never surfeited never led to satisfaction

(ii) Belch up cast ashore(iii) lsquomongst men in human

society2

I and my fellows (Line 60-65)

Are ministers of Fate The elementsOf whom your swords are tempered may as wellWound the loud winds or with bemocked-at stabsKill the still-closing waters as diminishOne dowl thats in my plume

IMPORTANT PASSAGES EXPLAINED

The elements

(Line 61-66)Of whom your swords are tempered may

as wellWound the loud winds or with

bemocked-at stabs

(a) Who is lsquoIrsquo Who are his lsquofellowsrdquo

lsquoIrsquo is referred to Ariel in disguise of a harpy His lsquofellowsrsquo are other spirits serving Prospero the real Duke of Milan who has acquired supernatural powers after being banished from his Dukedom Prospero has settled in this uninhabited island

(b) What are the elements that have temperrsquod the swords Why will it not work against the speaker

The swords (of Alonso and his companions) are tempered by metal (steel) which is taken out of the earth and refined by

Kill the still-closing waters as diminishOne dowl thats in my plume My fellow

ministersAre like invulnerable

In these words Ariel reminds the King and his companions of the utter futility of drawing swords against himself and his fellows Ariel drives Alonso Antonio and Sebastian the three men of sin to desperation ndash a state in which men do violence to themselves They draw swords to strike Ariel But Ariel reminds them that he and the other spirits are the ministers of destiny and nothing can wound them The steel of which their swords are made of may cut the wind or water which being divided always closes up again Even supposing that such things may be possible it is quite impossible that their swords will cut one feather in their plume They are incapable of being wounded by any sword of man Hence it is foolish on their part to attempt to strike at Ariel and his fellow-spirits

For which foul deed

(Line 72-75)The powers delaying not forgetting

haveIncensed the seas and shores yea all the

creatures Against your peace

Ariel enters like a harpy and remaining invisible tells Alonso Sebastian and Antonio that he and other harpies are the agents of Destiny appointed to carry out her decrees He tells them that their punishment for the crime against Prospero which has been so long deferred is now to fall upon them He reminds them that they had expelled Prospero from Milan and set him and his innocent child adrift on the sea and that the sea had paid them back for their sin by the shipwreck and by the calamities they have suffered He tells them that the powers above which did not forget this mean treachery but only deferred the punishment have now engaged the seas and the shores and all living beings including him and his comrades against them The very elements and supernatural agency Ariel adds have taken up the avenging of their crime against Prospero

the action of fire It may cut the wind or water which being divided always closes up again

The sword will not work against the spirits and the harpy because they are the ministers of destiny and nothing can wound them nor it will cut a single feather in their plume

(c )What is the meaning of lsquodowlrsquo in the last line

The term lsquodowlrsquo means a filament or the smallest part of a feather In this context Ariel in disguise of harpy says that their sword cannot even damage the smallest filament of their (Arielrsquos and other spirits) feathers as they are incapable of being wounded by any sword of man

(d) What does the speaker remind the listeners about

Ariel in disguise of harpy reminds Alonso the King of Naples Sebastian Alonsorsquos brother and Antonio the present Duke of Milan and the treacherous brother of Prospero as they being three men of sin He even reminds them that their punishment for their crime against Prospero which has been so long deferred now falls upon them He reminds them that they have expelled Prospero from Milan and has set him along with his innocent infant daughter adrift on the sea So the sea has paid them back for their sin by their shipwreck and the calamities they have suffered since then The harpy rebukes Alonso of his sin that has incensed the Gods and has deprived him of his son as a punishment

(e) How do they respond

When Ariel in disguise of a harpy reminds Alonso Sebastian and Antonio of their past misdeeds and sin Alonso has a look of terror and confusion in his eyes He utters the words of sincere repentance wrung out of his conscience-stricken heart It appears to him that all the elements of nature the sea-waves the wind and the thunder proclaiming a loud voice in the name of Prospero and the crime Alonso has committed against him They are calling upon him to repent There is a deep storm raging in Alonsorsquos breast and the echoes of that storm are ringing in his ears like a clear note of wind-instrument A note of denunciation of Alonsorsquos crime leaves him much humbled and penitent and confirms his belief that his son is lost forever But Sebastian and Antonio shows some courage instead of repentance They wish to kill the spirits or devils if it appears

3

Of my instruction hast thou nothing bated (Line 85-93)

In what thou hast to say So with good life

And observation strange my meaner ministers

Their several kinds have done My high charms work

And these mine enemies are all knit upIn their distractions They now are in my

powerAnd in these fits I leave them while I visitYoung Ferdinand whom they suppose is

drownedAnd his and mine loved darling

Methought the billows spoke and (Line 96-99)

told me of itThe winds did sing it to me and the

thunderThat deep and dreadful organ-pipe

pronouncedThe name of Prosper It did bass my

trespass

These are the words of contrition coming from Alonso Ariel has driven him to a deep repentance for conspiring with Antonio against Prospero He now feels a sincere remorse It appears to him that all the elements of nature the sea-waves the wind and the thunder proclaimed with a loud voice the name of Prospero and the crime Alonso had committed against him They are calling upon him to repent There is a deep storm raging in Alonsorsquos breast and the echoes of that storm are ringing in his ears like the clear note of a wind-instrument

Comment These are the words of sincere repentance wrung out of the conscience-stricken heart of Alonso Alonso who is the lesser villain is the first to give way to remorse under the effect of Arielrsquos speech The words of Ariel seem to him to be the voice of conscience speaking to him He is driven to desperation a state in which he might do violence to his life

(a) Identify the speaker State the context

Prospero the ruler of the island is the speaker The famous banquet scene has been enacted very well Ariel and his junior spirits have played their roles excellently Prospero is glad to say words of praise for them(b) In what way the speakerrsquos instructions have been carried out

According to Prosperorsquos instructions a banquet was presented before the King of Naples and his companions when they were tired and hungry Just when they were preparing to eat the feast the banquet was suddenly removed by exercising supernatural powers All this was done by Ariel Prosperorsquos chief assistant and a powerful spirit

Ariel not only made the feast disappear but also delivered his speech blaming the King and his two companions for their past wicked deeds He warned them to repent for their misdeeds or suffer forever on that uninhabited island

(c) Who are referred to as lsquomeaner ministersrsquo What have they done

Prospero refers as lsquomeaner ministersrsquo to his other lesser spirits who were assisting Ariel in presenting a scene before the kingrsquos party They entered the scene to the accompaniment of music They assumed several strange shapes and brought in a banquet Then they danced about it with gentle actions of salutations thus inviting the King and others to eat the feast

These spirits play their role again when Ariel in the shape of a harpy quits the scene These shapes enter again and dancing with mocking gestures carry away the table

(d) Who are the speakerrsquos enemies What has happened to them

King of Naples Alonso his brother Sebastian and the present Duke of Milan Antonio (Prosperorsquos own brother) are Prosperorsquos enemies With the turn of events they have all been washed ashore on the island which is ruled by Prospero the great magician Actually this happened after the shipwreck caused by a storm which was raised by Prospero with the purpose of bringing these people to his island Prosperorsquos spirits have already confused and terrified these enemies and they are under Prosperorsquos control He can treat them as he likes

(e) What does he say about Ferdinand Explain what is meant by ldquohellip his and mine darlingrdquo

Prospero knows that Alonsorsquos son prince Ferdinand is alive though his father thinks that the prince has been drowned

Prospero refers to his daughter Miranda who is dear to him She is also very dear to Prince Ferdinand who has fallen in love with her They are waiting to be married soon for which they have received Prosperorsquos consent

4

ALONSO O it is monstrous monstrous (Line 95-102)

Methought the billows spoke and told me of it

The winds did sing it to me and the thunderThat deep and dreadful organ-

pipe pronouncedThe name of Prosper It did bass

my trespassTherefore my son ithrsquo ooze is

bedded andIll seek him deeper than eer

plummet soundedAnd with him there lie mudded

(a) In what way does Alonso express his horror when his conscience is awakened by Arielrsquos words

When Alonsorsquos conscience is awakened by Arielrsquos words he expresses his horror at what he has heard He gets the feeling that the waves of the ocean the wind and the loud thunder have spoken to him and uttered the name of Prospero Because of being reminded of his crime in a very loud and rough voice he comes to realize that he has lost his son for his past misdeeds

(b) What does Alonso imagine about his son What does Alonso want to do in his desperate state

Alonso imagines that his son is lying in the mud at the bottom of the sea He feels desperate that he wants to drown himself in the ocean deeper than the plumb-line has ever gone He wants to lie with his son at the bottom of the sea

(c) How do Sebastian and Antonio want to face the evil spirits

Sebastian says that he is not at all afraid of what the harpy has said and that he is prepared to fight any number of such monsters if they appear before him only one at a time Antonio says that he would support Sebastian in the fight against the fiendsyyy

(d) Why does Gonzalo ask Adrian to follow the three men

Gonzalo tells Adrian that all the three men namely Alonso Sebastian and Antonio are in a wild and reckless mood The thought of the heinous crime of which they are guilty has begun to torment their minds So he asks Adrian to follow those three men without loss of time and prevent them from doing anything which the turmoil in their minds might lead them to do

(e) What opinion do you form of Alonso from the above extract

Alonso who is the lesser villain is the first to give way to remorse under the effect of Arielrsquos speech The words of Ariel seem to him to be the voice of conscience speaking to him He is driven to desperation a state in which he might do violence to his life

Subject =Accounts

Ac-12 15420 topic-pL Appropriation ac

PROFIT AND LOSS APPROPRIATION ACCOUNT

MEANING AND PREPARATIONProfit and Loss Appropriation Account is merely an extension of the Profit and Loss Account of the firm The profit of the firm has to be distributed amongst the partners in their respective profit sharing ratio But before its distribution it needs to be adjusted All Adjustments like partnerrsquos salary partnerrsquos commission interest on capital interest on drawings etc are made in this account These adjustments will reduce the amount of profit for distribution This adjusted profit will be distributed amongst the partners in their profit sharing ratio To prepare it at first the balance of Profit and Loss Account is transferred to this account The journal entries for the preparation of Profit and Loss Appropriation Account are given below

1 for transfer of the balance of Profit and Loss Account to Profit and Loss Appropriation Account

(a) In case of Net Profit

Profit and Loss Ac helliphelliphelliphelliphellipDrTo Profit and Loss Appropriation Ac(Net Profit transferred to Profit and Loss Appropriation Ac)

(b)In case of Net Loss

Profit and Loss Appropriation Achelliphelliphellip DrTo Profit and Loss Ac(Net Loss transferred to Profit and Loss Appropriation Ac)

2 for Interest on Capital

For transferring on Interest on CapitalProfit and Loss Appropriation Achelliphelliphellip DrTo Interest on Capital Ac(Interest on capital transferred to Profit amp Loss Appropriation Ac)

3 for Interest on Drawings

For transferring Interest on Drawings Interest on Drawings Achelliphelliphelliphelliphelliphellip DrTo Profit and Loss Appropriation Ac(Interest on drawing transferred to Profit amp Loss Appropriation Ac)

4 For Partnerrsquos SalaryFor transfer of partnerrsquos SalaryProfit and Loss Appropriation Achelliphellip DrTo Salary Ac(Salary transferred to profit amp Loss Appropriation Ac)

5 For Partnerrsquos CommissionFor transferring commissionProfit and Loss Appropriation Achelliphelliphellip DrTo Commission Ac(Commission transferred to Profit and Loss Appropriation Ac)

6 For Transfer of agreed amount to General ReserveProfit and Loss Appropriation Ac helliphellipDrTo General Reserve Ac(Transfer to General Reserve)

7 for share of Profit or Loss appropriation(a) If ProfitProfit and Loss Appropriation Achelliphellip DrTo Partnerrsquos CapitalCurrent Ac(Profit transferred to capitalcurrent Ac)(b) If LossPartnerrsquos Capital Current Achelliphelliphelliphellip DrTo Profit and Loss Appropriation Ac(Loss transferred to capitalcurrent Ac)

THE FORMAT OF PROFIT AND LOSS APPROPRIATION

Profit and Loss Appropriation Account for the year endedhelliphelliphelliphellip

Particulars Amount Particulars Amount

To PL Ac (loss) By pL Ac (profit)

To Interest on capital BY Interest on drawings

To partner`s commission by Partner`s capital Ac ( loss)

To Partner`s salary To Interest on partner`s loan To General Reserve To Partner`s Capital AC (Profit)

Subject= Economics

MOVEMENT ALONG THE DEMAND CURVE (CHANGE IN QUANTITY DEMANDED)In law of demand you have already studied the inverse relationship between price and quantity demanded When quantity demanded of a commodity changes due to change in its price keeping other factors constant it is called change in quantity demanded It is graphically expressed as a movement along the same demand curve There can be either a downward movement or an upward movement along the same demand curve Upward movement along the same demand curve is called contraction of demand or decrease in quantity demanded and downward movement along the same demand curve is known as expansion of demand or increase in quantity demanded

Extention of demandd

price (rs)p A

B Extentionp1 d

Q Q1

Quantity demanded ( in units)

Contraction of demandd

p2 Ccontraction

p APrice (Rs)

d

Q2 Q

Quantity demanded (in units)

Explanation of movement of demand A fall in price from OP to OP1 leads to increase in quantity demanded from OQ to OQ1 (expansion of demand) resulting in a downward movement from point A to point B along the same demand curve DD When Price rises from OP to OP2 quantity demanded falls from OQ to OQ2 (contraction of demand) leading to an upward movement from point A to point C along the same demand curve DD

  • Activity Series of Metals
    • Drawbacks of Rutherfordrsquos model of atom
      • Electromagnetic radiations
      • Properties of electromagnetic radiations
      • Characteristics of electromagnetic radiations
        • Plancks Quantum Theory-
        • Photoelectric effect
          • Intext Questions
            • Logic gates
            • Digital systems are said to be constructed by using logic gates These gates are the AND OR NOT NAND NOR EXOR and EXNOR gates The basic operations are described below with the aid of truth tables
            • AND gate
            • Example
Page 41:  · Web viewSubject. Topic. Summary. Execution. English 1 . Chapter 1 naming words . Page 8. Write the names of these pictures:- Person:-1. father. 2.Firefighter 3.doctor 4 ...

princely statesThis act was refused and

rejected by the princes the Congress and the Muslim League

Thus both Congress and the League participated in the election of 1937 Thus the federal part was never introduced but the provincial part was put into operations

Bengali 2nd

Language

াচেরর পরাথCনা(কহিতা )

াচেরর পরাথCনা কহিতাটি কহি (ঙখ দেঘাচে4র দো আচো য কহিতায় াচেরর পতর হমায়ন কঠিন দেরাচেগ আxানত ার ঈশবর া আললার কাচেছ পরাথCনা কচেরচেছন তার পচেতরর ীন হিফহিরচেয় হিচেত এই কহিতায় ার পচেতরর ীন হিভbা দেচেয়চেছন ারার এমনহিক হিনচের ীন হিসCচেনর হিহিনমচেয় হিতহিন তার দেছচের ীন হিফচের দেপচেত দেচেয়চেছন তার দেছচের এই দেরাচেগর ন য হিতহিন হিনচেচেকই ায়ী কচেরচেছন তার হিনচের করা পাপচেকই হিতহিন ায়ী কচেরচেছন এছাা রানৈনহিতক ও আথCসামাহিক অসথার কথা তচে ধরা চেয়চেছ এই কহিতায় ার তার হিনচের পাপ কমCচেকই ায়ী কচেরচেছ ার অন যায় ভাচে দেপহি((হিকতর মাধ যচেম অপররা য কচেরচেছ আর এই অন যায় কাচের ন যই তার পহিরাচের হিপযCয় এচেসচেছ দে এক পরকার মানহিক নধন ইহিতাচেসর ার হিপতা চেয় সবাভাহিকভাচে ভাচোাসা দে মমতা দেথচেক মকত চেত পাচেরনহিন তাই হিপতা চেয় আললা া ভগাচেনর কাচেছ পতর হমায়চেনর পরানহিভbা দেচেয়চেছন ার আললা া ভগাচেনর কাচেছ াহিনচেয়চেছন তার হিনচের ীন হিসCন হিচেত হিতহিন রাী তার হিহিনমচেয় পচেতরর ীন হিফচের দেপচেত দেচেয়চেছন াচেরর হিপতসভ হিচেকর কথা এই কহিতায় ফটিচেয় দেতাা চেয়চেছ হিপতা পচেতরর হিরাহিরত মান নধচেনর কথা তচে ধরা চেয়চেছ

হিচে(4 হিকছ াইচেনর তাৎপযC১) ldquoদেকাথায় দেগ ওর সবচছয দেৌন দেকাথায় কচেরায় দেগাপন bয়ldquoউততর) াচেরর পতর হমায়ন কঠিন দেরাচেগ অসসথ তাই তার দেযৌন াহিরচেয় যাচেচছ এই দেরাচেগ তাচেক দেগাপচেন কচেরকচের াচেচছ তার সক (হিকত ধীচের ধীচের bয় চেচছ তাই হিপতা চেয় ার আললার কাচেছ হমায়চেনর পরান হিভbা দেচেয়চেছন২) ldquoাগাও (চেরর পরাচেনত পরানতচের ধসর (ন দেযর আান গানldquoউততর) াচেরর পতর হমায়ন কঠিন দেরাচেগ আxানত তাই ার আ দে(াচেক মমCাত (চেরর পচেথ পরানতচের আান গান ধবহিনত দোক দেসই আান গান আললার কাচেছ দেযন চে যায় আললা দেযন এই আহিতC শচেন পচেতরর ীন হিফহিরচেয় দেয় ৩)ldquoনাহিক এই (রীচেরর পাচেপর ীানচেত দেকানই তরারণ দেনই ভহি4চেতরldquoউততর) হমায়চেনর অসসথতার ন য ার হিনচেচেকই ায়ী কচেরচেছন কারন ার অচেনক রা য অন যায় ভাচে কচেরচেছ তাই তার এই পাপ কাচের ন য তার ঘচের আ হিপ এচেসচেছ এই অন যায় কাচের ন য তার মহিকত দেনই তাই ার আললার কাচেছ এই পাপ কাচেযCর ন য bমা পরাথM

Hindi 2ndlang

-ासी(जयशकर परसा-)

-ासी जयशकर परसा- की एक ऐसी कहानी ह जिजसम भारतीय ससकनित और राषटरीयता का सवरगजीतहोता ह इस कहानी म इरावती एक निहद कनया ह जिजस मलअचछो न मलतान की लट म पकडा और -ासी बना दि-या उस 500 दि-न -कर काशी क एक महाजन न खरी-ा दसरी -ासी निफरोजा ह वह गलाम ह निफरोजा को छडान क कतिलए अहम- को 1000 सोन क कतिसकक भजन थ जो अभी तक नही आए थ राजा साहब कठोर होत हए भी निफरोजा को निबना धनराकतिश क कतिलए उस म कर -त ह वनिफरोजा को अहम- को समझान की बात कहत हकहानी क अत म हम -खत ह निक इरा वती और जाटो क सर-ार बलराज का मिमलन होता हअहम- को यa म मार दि-या जाता ह वहा निफरोजा की परसननता की समामिध बनती ह वहा एक फल चढती ह और डीजल आती ह निफरोजा उस समामिध की आजीवन -ासी बनी रहती हलखक अपन उददशय अथात -ास परथा पर परकाश डालन और इस परथा क कारण होन वाल -ातो क दखो को दि-खान म पणता सफल हए ह

helliphellipContinue to next

Biology Reproductio Today we will discuss about vegetative Q1 Name some vegetative propagules

n in Organisms

propagation of plants The process of multiplication in which fragments of plant body function as propagule and develop into new individual is called vegetative propagation The units of such propagation are runner rhizome tuber bulb etc

and the speciesinvolvedVegetative propagules

Parts involved

Bulb StemBulbil BulbilRhizome Stem Runner Stem Tuber Stem Offset Stem Leaf buds Leaves Suckers Stem

Corns Stem stolon

Q2 State advantages of vegetative propagation

i) Rapid methodii) Sure and easy methodiii) Useful in plants that cannot

produce viable seeds or long seed dormancy

iv) Maintains purity of raceQ 3 Banana fruit is said to be parthenocarpic where as turkey is said to be parthenogenetic WhyBanana develops without fertilization from an unfertilized ovary thus is parthenocarpicIn turkey the ovum or female gamete developinto a new chick without fertilization thus isparthgenetic

Q4 Why is water hyacinth is called as a ldquoTerror of Bengalrdquo Water hyacinth can

propagatevegetatively all over the water body in a short per short period of time This resulted increased biochemicaloxygen oxygen demand of water body causing mortalityof fishes It is very difficult to get rid off them Thus known as terror of Bengal

Chemistry

Solid state GENERAL CHARACTERISTICS OF SOLID STATEIn nature the particular state of matter is governed by two opposing forces at given set of temperature and pressure These forces are intermolecular force of attraction and thermal energy If intermolecular force of attraction is high as compared to thermal energy particles remains in closest position

Intext QuestionsQ1 Classify the following solids as crystalline and amorphous Sodium chloride quartz glass quartz rubber polyvinyl chloride Teflon

A1 Crystalline

and hence very less movement in particles is observed In this case solid state is the preferred state of matter

Let us revise the general characteristics of solid

i) Fixed mass volume and shape

ii) Strong intermolecular force of attraction

iii) Least intermolecular space

iv) Fixed position of constituent particles

v) Incompressible and rigid

Q2 what type of interactions hold the molecules together in a polar molecular solid[CBSE 2010]A2 The molecules in a solid are held together by van der Waals forces The term van der Waals forces include hydrogen bonding dipole-dipole attraction and London dispersion forces All molecules experience London dispersion forces In addition polar molecules can also experience dipole-dipole interactions So the interactions that holds the molecule together in polar molecular solid are London dispersion force and dipole-dipole interactionsQ3 Write a feature that will distinguish a metallic solid from an ionic solid [CBSE 2010]A3 Metals are malleable and ductile whereas ionic solid are hard and brittle Metallic solid has typical metallic lustre But ionic solid looks dullQ4 Write a point of distinction between a metallic solid and an ionic solid other than metallic lustre [CBSE 2012]A4 Metals are malleable and ductile whereas ionic solid are hard and brittleQ5 Write a distinguish feature of metallic solid [CBSE 2010]A5 The force of attraction in

solid Sodium chloride Quartz Amorphous solid Quartz glass rubber polyvinyl chloride Teflon Q2 why glass is considered as super cooled liquidA2 Glass shows the tendency to flow at slower rate like liquid Hence they considered as super cooled liquidQ3 why the window glass of old buildings show milky appearance with timeA3 Glass is an amorphous solid Amorphous solid has the tendency to develop some crystalline character on heating Due to heating in day over the number of years glass acquires some crystalline character and show milky appearanceQ4 why the glass panes fixed to window or doors of old building become slightly thicker at bottomA4 Glass is super cooled liquid It has the tendency to flow down very slowly Due to this glass pane becomes thicker at the bottom over the timeQ5 Sodium chloride is a crystalline solid It shows the same value of refractive index along all the direction TrueFalse Give reasonA5 FalseCrystalline solid shows anisotropy in properties That is it shows different values for the given physical property in different direction All the crystalline solids show anisotropy in refractive index Therefore sodium chloride will show different values of refractive index on different directions

Q6 Crystalline solid are anisotropic in nature What does this statement means

between the constituent particles is special kind of electrostatic attraction That is the attraction of positively charged kernel with sea of delocalized electronsQ6 which group of solid is electrical conductor as well as malleable and ductile [CBSE 2013]A6 Metallic solidQ7 why graphite is good conductor of electricity although it is a network (covalent solid)A7 The exceptional property of graphite is due to its typical structure In graphite each carbon is covalently bonded with 3 atoms in same layer The fourth valence electron of each atom is free to move in between different layersThis free electron makes the graphite a good conductor of electricity

[CBSE 2011]A6 Anisotropy is defined asrdquo Difference in properties when measured along different axis or from different directionsrdquo Crystalline solid show different values of some of the physical properties like electrical resistance refractive index etcwhen measured along the different directions The anisotropy in crystalline solid arises due to the different arrangement of particles in different directions

Math Function Composition of functions Think of an industrial plant that produce bottles of cold drinks first there is the operation (or function) f that puts the cold drink inside the bottle followed by the opeartion g that close the bottle with the capThis leads to the following definitionDefinition Let f A rarr B and g B rarr C be two functions Then the composition of f and g denoted by gof is defined as the function gof A rarr C given by gof(x) = g(f (x)) forall x isinA

Definition A function f X rarr Y is defined to be invertible if there exists a function g Y rarr X such that gof = IX and fog = IY The function g is called the inverse of f and is denoted by f -1

Thus if f is invertible then f must be one-one and onto and conversely if f is one-one and onto then f must be invertible This fact significantly helps for proving a function f to be invertible by showing that f is one-one and onto specially when the actual inverse of f is not to be determined

Example 1 Let f 2 3 4 5 rarr 3 4 5 9 and g 3 4 5 9 rarr 7 11 15 be functions defined as f(2) = 3 f(3) = 4 f(4) = f(5) = 5 and g (3) = g (4) = 7 and g (5) = g (9) = 11 Find gofSolution We have gof(2) = g (f(2)) = g (3) = 7 gof(3) = g (f(3)) = g (4) = 7gof(4) = g (f(4)) = g (5) = 11 and gof(5) = g (5) = 11Example 2 Find gof and fog if f R rarr R and g R rarr R are given by f(x) = cos x and g (x) = 3x2 Show that gof ne fogSolution We have gof(x) = g(f(x))=g(cosx) = 3 (cos x)2

= 3 cos2 x Similarly fog(x)=f(g (x))= f(3x2)= cos (3x2) Note that 3cos2 x ne cos 3x2 for x = 0 Hence gof ne fogExample 3 Show that if f A rarr B and g B rarr C are onto then gof A rarr C is also ontoSolution Given an arbitrary element z isin C there exists a pre-image y of z under g such that g (y) = z since g is onto Further for y isin B there exists an element x in A with f(x) = y since f is onto Therefore gof(x) = g (f(x)) = g (y) = z showing that gof is onto Example 4 Let Y = n2 n isin N sub N Consider f N rarr Y as f(n) = n2 Show that

f is invertible Find the inverse of fSolution An arbitrary element y in Y is of the form n2 for some n isin N This implies that n =radicy This gives a function g Y rarr N defined by g (y) =radicy Nowgof (n) = g (n2)=radicn2 = n and fog (y) =f(radicy) = (radicy) 2 y which shows that gof=IN and fog= IY Hence f is invertible with f -1 = g

Political Science

Constitution of India-The Preamble

Summary

Objective of the state-To secure equality of status and of opportunity To promote fraternity among all the citizens To assure the dignity of the individuals and Unity and integrity of the nation

Justice-Justice stands for rule of law absence of arbitrariness and a system of equal rights freedom and opportunities for all in a society India seeks social economic and political justice to ensure equality to its citizens

Liberty-Liberty implies the absence of restraints or domination on the activities of an individual such as freedom from slavery serfdom imprisonment despotism etc The Preamble provides for the liberty of thought expression belief faith and worship

Equality-Equality means the absence of privileges or discrimination against any section of the society The Preamble provides for equality of status and opportunity to all the people of the country

Fraternity-The Preamble declares that fraternity has to assure two thingsmdashthe dignity of the individual and the unity and

Execution

Answer the following questions-

Short notes-1 Equality2 Fraternity3 Justice4 Liberty

Homework-Learn

integrity of the nation The word integrity has been added to the Preamble by the 42nd Constitutional Amendment (1976)

Business studies

Human resource management (chapter 1)

On the day of 1504 2020 I have discussed with you the managerial functions and procurement functions of HRM

Today weare going to discuss about the development function integration functions and maintenance function

Development functions-HRM improves the knowledge skills attitude and values of employees so that they the present and future jobs more effectively it includes

1) Development functions of HRM

a) Performance appraisal = It implies systematic evaluation of employees with respect to their performance on the job and their potential for development

b) Training =It is the process by which employees learn knowledge skills and attitudes to achieve organisational and personal goals

c) Executive development = It is the process of developing managerial talent through appropriate program

2) Integration functionsa) HRM reconcile the goals of

organisation with those of its members through integrating function

b) HRM tries to motivate employees to various financial and non financial incentives provided in job specification etc

3) Maintenance functiona) HRM promote and protect the

physical and mental health of employees by providing several types of benefits like housing medical aid etc

b) It Promote Social security measures to employees by providing provident fund pension gratuity maternity benefits

SubjectCOMMERCE

Topic

BUSINESSENVIRONMENT

Summary

Now quickly let us revise the earlier points that we have already done in the last class and let us proceed with the other topics that are there in the chapter

Firstly we will recall the internal and external factors of micro environment and then we

Execution 3 What do you mean by internal factors

in micro environmentAnswerInternal factors refer to all the factors existing within a business firm The internal factors are considered controllable because the enterprise has control over these factors

Development FunctionsPerformance AppraisalTrainingExecution Development

shall proceed in details

Meaning and list of internal and external factors

aInternal factorsInternal factors refer to all the factors existing within a business firm The internal factors are considered controllable because the enterprise has control over these factorsFor an example a company can alter its organization structure policies programmes employees physical facilities and marketing mix to suit the changes in the environmentList of internal factors areCorporate culture mission and objectives top management organizations structure company image and brand equity company resources

b External factorsExternal factors refer to those individual and groups and agencies with which a particular business organization comes into direct and frequent contact in the course of its functioningThese individuals and groups are known as STAKEHOLDERS because they have a stake (financial interest ) in the working and performance of the particular business List of external forces (stakeholders)Customers competitors investors suppliersmiddlemen (marketing intermediaries)financers publics

customers

suppliersfinancers

For an example a company can alter its organization structure policies programmes employees physical facilities and marketing mix to suit the changes in the environment

4 What do you mean by external factors in micro environment

AnswerExternal factors refer to those individual and groups and agencies with which a particular business organization comes into direct and frequent contact in the course of its functioningThese individuals and groups are known as STAKEHOLDERS because they have a stake (financial interest) in the working and performance of the particular business

3Who are stakeholdersSTAKEHOLDERS are individuals and groups who have a stake (financial interest ) in the working and performance of the particular business 4Discuss the internal factors in briefa Corporate CultureThe values beliefs and attitudes of the founders and top management of the company exercise a strong influence on what the cmpaany stands for how it does things and what it considers importantbMission and objectivesThe business philosophy and purpose of a comoany guide it prioritiesbusiness strategiesproduct market scope and development scope

cTop management structurethe composition of board of directors the degree of professionalization of management and the organizational structure of a company have important bearing on its business decisions

dPower structureThe internal power relationship between the board of directors and the chief executive is an important factor

e Company image and brand equityThe image and brand equity of the company play a significant role in raising finance forming alliance choosing dealers and suppliers launching new products entering foreign markets

5 What is Macro environmentAnswerMacro environment refers to the general

competitors

middlemen

publics

Fig STAKEHOLDERS OF A COMPANY

Apart from micro environment the other main dimension of business environment isMacro environment Macro environment refers to the general environment or remote environment within which a business firm and forces in its micro environment operateA company does not directly or regularly interact with the micro environmentTherefore macro environment is also known as indirect action EnvironmentThe macro environment forces are less controllable than the micro forces

Macro environment consists of the following components

POLITICAL AND LEGAL ENVIRONMENT

ECONOMIC SOCIAL AND ENVIRONMENT

CULTURAL

ENVIRONMENT

TECHNOLOGICAL ENVIRONMENT

Fig COMPONENTS OF MACRO ENVIRONMENT

environment or remote environment within which a business firm and forces in its micro environment operateA company does not directly or regularly interact with the micro environmentTherefore macro environment is also known as indirect action EnvironmentThe macro environment forces are less controllable than the micro forces 6 What are the components of macro environmenta Political and legal environmentb Economic environmentc Social and cultural environmentd Technological environment

Computer Science

Logic gates

Digital systems are said to be constructed by using logic gates These gates are the AND OR NOT NAND NOR EXOR and EXNOR

BUSINESS FIRM

gates The basic operations are described below with the aid of truth tables

AND gate

The AND gate is an electronic circuit that gives a high output (1) only if all its inputs are high A dot () is used to show the AND operation ie AB Bear in mind that this dot is sometimes omitted ie ABOR gate

The OR gate is an electronic circuit that gives a high output (1) if one or more of its inputs are high A plus (+) is used to show the OR operationNOT gate

The NOT gate is an electronic circuit that produces an inverted version of the input at its output It is also known as an inverter If the input variable is A the inverted output is known as NOT A This is also shown as A or A with a bar over the top as shown at the outputs The diagrams below show two ways that the NAND logic gate can be configured to produce a NOT gate It can also be done using NOR logic gates in the same way

NAND gate

This is a NOT-AND gate which is equal to an AND gate followed by a NOT gate The outputs of all NAND gates are high if any of the inputs are low The symbol is an AND gate with a small circle on the output The small circle represents inversion

NOR gate

This is a NOT-OR gate which is equal to an OR gate followed by a NOT gate The outputs of all NOR gates are low if any of the inputs are highThe symbol is an OR gate with a small circle on the output The small circle represents inversion

EXOR gate

The Exclusive-OR gate is a circuit which will give a high output if either but not both of its two inputs are high An encircled plus sign ( ) is used to show the EOR operation

EXNOR gate

The Exclusive-NOR gate circuit does the opposite to the EOR gate It will give a low output if either but not both of its two inputs are high The symbol is an EXOR gate with a small circle on the output The small circle represents inversion The NAND and NOR gates are called universal functions since with either one the AND and OR functions and NOT can be generated

Note A function in sum of products form can be implemented using NAND gates by replacing all AND and OR gates by NAND gates A function in product of sums form can be implemented using NOR gates by replacing all AND and OR gates by NOR gates

Logic gate symbols

Table 2 is a summary truth table of the inputoutput combinations for the NOT gate together with all possible inputoutput combinations for the other gate functions Also note that a truth table with n inputs has 2n rows You can compare the outputs of different gates

Logic gates representation using the Truth table

Example

A NAND gate can be used as a NOT gate using either of the following wiring configurations

Subject Eng Literature (The Tempest ndash William Shakespeare) Topic Act III Scene 3 Lines 53 to 110 (End of the scene) Date 16th April 2020 (2nd Period)

[Students should read the original play and also the paraphrase given in the school prescribed textbook]Summary Questions amp Answers

o Seeing this strange scene all are inclined to believe the tales told by travelers that there truly are ldquounicornsrdquo and ldquothe phoenixrsquo thronerdquo

o As they are about to sit down to the feast the banquet is snatched away by a harpy (Ariel disguised) A spiritrsquos voice (Arielrsquos voice) denounces Alonso Sebastian and Antonio with particular

1 ARIEL You are three men of sin whom Destiny

(Line 53-58)That hath to instrument this

lower world And what is int the never-surfeited sea

Hath caused to belch up you and on this island

Where man doth not inhabit you rsquomongst men

Being most unfit to live I have made you mad

reference to their crime in expelling Prospero from Milan They have not received any punishment for their deed earlier but the time for their punishment has arrived Upon Alonso it pronounces ldquolingering perdition worse than deathrdquo from which there is no remedy except through sincere repentance Ariel then vanishes in thunder and the shapes enter again and carry away the table

o Prospero watching invisibly is very pleased with the performance of Ariel and his (Prosperorsquos) ldquomeaner ministersrdquo All his enemies are now in his power and are in a fit of desperation He then leaves them and goes to see how Ferdinand and Miranda are getting on

o Alonso is now much humbled and penitent with the after effect of the spiritrsquos denunciation of his crimes He believes that his son is lost forever After this all disperse being stricken mad by the speech of the spirit

o Gonzalo fearing that they may do violence to themselves or to one another follows them and bid others to follow

(a) To whom does Ariel disguised as a harpy call the three sinners What game did Fate of Destiny play with

them

The three sinners called by Ariel are Alonso Sebastian and Antonio It was Destiny which had caused the ocean to cast the three sinners on the shore Though the ocean is all the time devouring whatever appears on its surface and is never satisfied with its continual swallowing of the ships and men in the present case the ocean had cast these three sinners on the shore without killing them

(b) Who had jointly been responsible for the conspiracy against Prospero What is Prosperorsquos purpose behind all this

Three men Alonso Sebastian and Antonio had jointly

been responsible for the conspiracy against Prospero They had driven out Prospero form Milan Prosperorsquos purpose is to make these three sinners realize the wrong they had done He wants them to repent for their criminal deeds because repentance leads to self-esteem(c )What does Ariel (the harpy) tell Alonso and his companions when they take out their swords to attack him

Seeing them drawing their swords Ariel (harpy) tells them that he and his companions are the instruments of destiny and that it is not possible for human beings to do them any injury He says that the swords of human beings can not injure even a minute part of his feathers Their swords are as ineffective against him and his companions as against the wind or the water

(d) Give the explanatory meanings of the following expressions in the context of the above extract

(i)Never surfeited (ii) Belch up (iii) lsquomongst men

(i) Never surfeited never led to satisfaction

(ii) Belch up cast ashore(iii) lsquomongst men in human

society2

I and my fellows (Line 60-65)

Are ministers of Fate The elementsOf whom your swords are tempered may as wellWound the loud winds or with bemocked-at stabsKill the still-closing waters as diminishOne dowl thats in my plume

IMPORTANT PASSAGES EXPLAINED

The elements

(Line 61-66)Of whom your swords are tempered may

as wellWound the loud winds or with

bemocked-at stabs

(a) Who is lsquoIrsquo Who are his lsquofellowsrdquo

lsquoIrsquo is referred to Ariel in disguise of a harpy His lsquofellowsrsquo are other spirits serving Prospero the real Duke of Milan who has acquired supernatural powers after being banished from his Dukedom Prospero has settled in this uninhabited island

(b) What are the elements that have temperrsquod the swords Why will it not work against the speaker

The swords (of Alonso and his companions) are tempered by metal (steel) which is taken out of the earth and refined by

Kill the still-closing waters as diminishOne dowl thats in my plume My fellow

ministersAre like invulnerable

In these words Ariel reminds the King and his companions of the utter futility of drawing swords against himself and his fellows Ariel drives Alonso Antonio and Sebastian the three men of sin to desperation ndash a state in which men do violence to themselves They draw swords to strike Ariel But Ariel reminds them that he and the other spirits are the ministers of destiny and nothing can wound them The steel of which their swords are made of may cut the wind or water which being divided always closes up again Even supposing that such things may be possible it is quite impossible that their swords will cut one feather in their plume They are incapable of being wounded by any sword of man Hence it is foolish on their part to attempt to strike at Ariel and his fellow-spirits

For which foul deed

(Line 72-75)The powers delaying not forgetting

haveIncensed the seas and shores yea all the

creatures Against your peace

Ariel enters like a harpy and remaining invisible tells Alonso Sebastian and Antonio that he and other harpies are the agents of Destiny appointed to carry out her decrees He tells them that their punishment for the crime against Prospero which has been so long deferred is now to fall upon them He reminds them that they had expelled Prospero from Milan and set him and his innocent child adrift on the sea and that the sea had paid them back for their sin by the shipwreck and by the calamities they have suffered He tells them that the powers above which did not forget this mean treachery but only deferred the punishment have now engaged the seas and the shores and all living beings including him and his comrades against them The very elements and supernatural agency Ariel adds have taken up the avenging of their crime against Prospero

the action of fire It may cut the wind or water which being divided always closes up again

The sword will not work against the spirits and the harpy because they are the ministers of destiny and nothing can wound them nor it will cut a single feather in their plume

(c )What is the meaning of lsquodowlrsquo in the last line

The term lsquodowlrsquo means a filament or the smallest part of a feather In this context Ariel in disguise of harpy says that their sword cannot even damage the smallest filament of their (Arielrsquos and other spirits) feathers as they are incapable of being wounded by any sword of man

(d) What does the speaker remind the listeners about

Ariel in disguise of harpy reminds Alonso the King of Naples Sebastian Alonsorsquos brother and Antonio the present Duke of Milan and the treacherous brother of Prospero as they being three men of sin He even reminds them that their punishment for their crime against Prospero which has been so long deferred now falls upon them He reminds them that they have expelled Prospero from Milan and has set him along with his innocent infant daughter adrift on the sea So the sea has paid them back for their sin by their shipwreck and the calamities they have suffered since then The harpy rebukes Alonso of his sin that has incensed the Gods and has deprived him of his son as a punishment

(e) How do they respond

When Ariel in disguise of a harpy reminds Alonso Sebastian and Antonio of their past misdeeds and sin Alonso has a look of terror and confusion in his eyes He utters the words of sincere repentance wrung out of his conscience-stricken heart It appears to him that all the elements of nature the sea-waves the wind and the thunder proclaiming a loud voice in the name of Prospero and the crime Alonso has committed against him They are calling upon him to repent There is a deep storm raging in Alonsorsquos breast and the echoes of that storm are ringing in his ears like a clear note of wind-instrument A note of denunciation of Alonsorsquos crime leaves him much humbled and penitent and confirms his belief that his son is lost forever But Sebastian and Antonio shows some courage instead of repentance They wish to kill the spirits or devils if it appears

3

Of my instruction hast thou nothing bated (Line 85-93)

In what thou hast to say So with good life

And observation strange my meaner ministers

Their several kinds have done My high charms work

And these mine enemies are all knit upIn their distractions They now are in my

powerAnd in these fits I leave them while I visitYoung Ferdinand whom they suppose is

drownedAnd his and mine loved darling

Methought the billows spoke and (Line 96-99)

told me of itThe winds did sing it to me and the

thunderThat deep and dreadful organ-pipe

pronouncedThe name of Prosper It did bass my

trespass

These are the words of contrition coming from Alonso Ariel has driven him to a deep repentance for conspiring with Antonio against Prospero He now feels a sincere remorse It appears to him that all the elements of nature the sea-waves the wind and the thunder proclaimed with a loud voice the name of Prospero and the crime Alonso had committed against him They are calling upon him to repent There is a deep storm raging in Alonsorsquos breast and the echoes of that storm are ringing in his ears like the clear note of a wind-instrument

Comment These are the words of sincere repentance wrung out of the conscience-stricken heart of Alonso Alonso who is the lesser villain is the first to give way to remorse under the effect of Arielrsquos speech The words of Ariel seem to him to be the voice of conscience speaking to him He is driven to desperation a state in which he might do violence to his life

(a) Identify the speaker State the context

Prospero the ruler of the island is the speaker The famous banquet scene has been enacted very well Ariel and his junior spirits have played their roles excellently Prospero is glad to say words of praise for them(b) In what way the speakerrsquos instructions have been carried out

According to Prosperorsquos instructions a banquet was presented before the King of Naples and his companions when they were tired and hungry Just when they were preparing to eat the feast the banquet was suddenly removed by exercising supernatural powers All this was done by Ariel Prosperorsquos chief assistant and a powerful spirit

Ariel not only made the feast disappear but also delivered his speech blaming the King and his two companions for their past wicked deeds He warned them to repent for their misdeeds or suffer forever on that uninhabited island

(c) Who are referred to as lsquomeaner ministersrsquo What have they done

Prospero refers as lsquomeaner ministersrsquo to his other lesser spirits who were assisting Ariel in presenting a scene before the kingrsquos party They entered the scene to the accompaniment of music They assumed several strange shapes and brought in a banquet Then they danced about it with gentle actions of salutations thus inviting the King and others to eat the feast

These spirits play their role again when Ariel in the shape of a harpy quits the scene These shapes enter again and dancing with mocking gestures carry away the table

(d) Who are the speakerrsquos enemies What has happened to them

King of Naples Alonso his brother Sebastian and the present Duke of Milan Antonio (Prosperorsquos own brother) are Prosperorsquos enemies With the turn of events they have all been washed ashore on the island which is ruled by Prospero the great magician Actually this happened after the shipwreck caused by a storm which was raised by Prospero with the purpose of bringing these people to his island Prosperorsquos spirits have already confused and terrified these enemies and they are under Prosperorsquos control He can treat them as he likes

(e) What does he say about Ferdinand Explain what is meant by ldquohellip his and mine darlingrdquo

Prospero knows that Alonsorsquos son prince Ferdinand is alive though his father thinks that the prince has been drowned

Prospero refers to his daughter Miranda who is dear to him She is also very dear to Prince Ferdinand who has fallen in love with her They are waiting to be married soon for which they have received Prosperorsquos consent

4

ALONSO O it is monstrous monstrous (Line 95-102)

Methought the billows spoke and told me of it

The winds did sing it to me and the thunderThat deep and dreadful organ-

pipe pronouncedThe name of Prosper It did bass

my trespassTherefore my son ithrsquo ooze is

bedded andIll seek him deeper than eer

plummet soundedAnd with him there lie mudded

(a) In what way does Alonso express his horror when his conscience is awakened by Arielrsquos words

When Alonsorsquos conscience is awakened by Arielrsquos words he expresses his horror at what he has heard He gets the feeling that the waves of the ocean the wind and the loud thunder have spoken to him and uttered the name of Prospero Because of being reminded of his crime in a very loud and rough voice he comes to realize that he has lost his son for his past misdeeds

(b) What does Alonso imagine about his son What does Alonso want to do in his desperate state

Alonso imagines that his son is lying in the mud at the bottom of the sea He feels desperate that he wants to drown himself in the ocean deeper than the plumb-line has ever gone He wants to lie with his son at the bottom of the sea

(c) How do Sebastian and Antonio want to face the evil spirits

Sebastian says that he is not at all afraid of what the harpy has said and that he is prepared to fight any number of such monsters if they appear before him only one at a time Antonio says that he would support Sebastian in the fight against the fiendsyyy

(d) Why does Gonzalo ask Adrian to follow the three men

Gonzalo tells Adrian that all the three men namely Alonso Sebastian and Antonio are in a wild and reckless mood The thought of the heinous crime of which they are guilty has begun to torment their minds So he asks Adrian to follow those three men without loss of time and prevent them from doing anything which the turmoil in their minds might lead them to do

(e) What opinion do you form of Alonso from the above extract

Alonso who is the lesser villain is the first to give way to remorse under the effect of Arielrsquos speech The words of Ariel seem to him to be the voice of conscience speaking to him He is driven to desperation a state in which he might do violence to his life

Subject =Accounts

Ac-12 15420 topic-pL Appropriation ac

PROFIT AND LOSS APPROPRIATION ACCOUNT

MEANING AND PREPARATIONProfit and Loss Appropriation Account is merely an extension of the Profit and Loss Account of the firm The profit of the firm has to be distributed amongst the partners in their respective profit sharing ratio But before its distribution it needs to be adjusted All Adjustments like partnerrsquos salary partnerrsquos commission interest on capital interest on drawings etc are made in this account These adjustments will reduce the amount of profit for distribution This adjusted profit will be distributed amongst the partners in their profit sharing ratio To prepare it at first the balance of Profit and Loss Account is transferred to this account The journal entries for the preparation of Profit and Loss Appropriation Account are given below

1 for transfer of the balance of Profit and Loss Account to Profit and Loss Appropriation Account

(a) In case of Net Profit

Profit and Loss Ac helliphelliphelliphelliphellipDrTo Profit and Loss Appropriation Ac(Net Profit transferred to Profit and Loss Appropriation Ac)

(b)In case of Net Loss

Profit and Loss Appropriation Achelliphelliphellip DrTo Profit and Loss Ac(Net Loss transferred to Profit and Loss Appropriation Ac)

2 for Interest on Capital

For transferring on Interest on CapitalProfit and Loss Appropriation Achelliphelliphellip DrTo Interest on Capital Ac(Interest on capital transferred to Profit amp Loss Appropriation Ac)

3 for Interest on Drawings

For transferring Interest on Drawings Interest on Drawings Achelliphelliphelliphelliphelliphellip DrTo Profit and Loss Appropriation Ac(Interest on drawing transferred to Profit amp Loss Appropriation Ac)

4 For Partnerrsquos SalaryFor transfer of partnerrsquos SalaryProfit and Loss Appropriation Achelliphellip DrTo Salary Ac(Salary transferred to profit amp Loss Appropriation Ac)

5 For Partnerrsquos CommissionFor transferring commissionProfit and Loss Appropriation Achelliphelliphellip DrTo Commission Ac(Commission transferred to Profit and Loss Appropriation Ac)

6 For Transfer of agreed amount to General ReserveProfit and Loss Appropriation Ac helliphellipDrTo General Reserve Ac(Transfer to General Reserve)

7 for share of Profit or Loss appropriation(a) If ProfitProfit and Loss Appropriation Achelliphellip DrTo Partnerrsquos CapitalCurrent Ac(Profit transferred to capitalcurrent Ac)(b) If LossPartnerrsquos Capital Current Achelliphelliphelliphellip DrTo Profit and Loss Appropriation Ac(Loss transferred to capitalcurrent Ac)

THE FORMAT OF PROFIT AND LOSS APPROPRIATION

Profit and Loss Appropriation Account for the year endedhelliphelliphelliphellip

Particulars Amount Particulars Amount

To PL Ac (loss) By pL Ac (profit)

To Interest on capital BY Interest on drawings

To partner`s commission by Partner`s capital Ac ( loss)

To Partner`s salary To Interest on partner`s loan To General Reserve To Partner`s Capital AC (Profit)

Subject= Economics

MOVEMENT ALONG THE DEMAND CURVE (CHANGE IN QUANTITY DEMANDED)In law of demand you have already studied the inverse relationship between price and quantity demanded When quantity demanded of a commodity changes due to change in its price keeping other factors constant it is called change in quantity demanded It is graphically expressed as a movement along the same demand curve There can be either a downward movement or an upward movement along the same demand curve Upward movement along the same demand curve is called contraction of demand or decrease in quantity demanded and downward movement along the same demand curve is known as expansion of demand or increase in quantity demanded

Extention of demandd

price (rs)p A

B Extentionp1 d

Q Q1

Quantity demanded ( in units)

Contraction of demandd

p2 Ccontraction

p APrice (Rs)

d

Q2 Q

Quantity demanded (in units)

Explanation of movement of demand A fall in price from OP to OP1 leads to increase in quantity demanded from OQ to OQ1 (expansion of demand) resulting in a downward movement from point A to point B along the same demand curve DD When Price rises from OP to OP2 quantity demanded falls from OQ to OQ2 (contraction of demand) leading to an upward movement from point A to point C along the same demand curve DD

  • Activity Series of Metals
    • Drawbacks of Rutherfordrsquos model of atom
      • Electromagnetic radiations
      • Properties of electromagnetic radiations
      • Characteristics of electromagnetic radiations
        • Plancks Quantum Theory-
        • Photoelectric effect
          • Intext Questions
            • Logic gates
            • Digital systems are said to be constructed by using logic gates These gates are the AND OR NOT NAND NOR EXOR and EXNOR gates The basic operations are described below with the aid of truth tables
            • AND gate
            • Example
Page 42:  · Web viewSubject. Topic. Summary. Execution. English 1 . Chapter 1 naming words . Page 8. Write the names of these pictures:- Person:-1. father. 2.Firefighter 3.doctor 4 ...

n in Organisms

propagation of plants The process of multiplication in which fragments of plant body function as propagule and develop into new individual is called vegetative propagation The units of such propagation are runner rhizome tuber bulb etc

and the speciesinvolvedVegetative propagules

Parts involved

Bulb StemBulbil BulbilRhizome Stem Runner Stem Tuber Stem Offset Stem Leaf buds Leaves Suckers Stem

Corns Stem stolon

Q2 State advantages of vegetative propagation

i) Rapid methodii) Sure and easy methodiii) Useful in plants that cannot

produce viable seeds or long seed dormancy

iv) Maintains purity of raceQ 3 Banana fruit is said to be parthenocarpic where as turkey is said to be parthenogenetic WhyBanana develops without fertilization from an unfertilized ovary thus is parthenocarpicIn turkey the ovum or female gamete developinto a new chick without fertilization thus isparthgenetic

Q4 Why is water hyacinth is called as a ldquoTerror of Bengalrdquo Water hyacinth can

propagatevegetatively all over the water body in a short per short period of time This resulted increased biochemicaloxygen oxygen demand of water body causing mortalityof fishes It is very difficult to get rid off them Thus known as terror of Bengal

Chemistry

Solid state GENERAL CHARACTERISTICS OF SOLID STATEIn nature the particular state of matter is governed by two opposing forces at given set of temperature and pressure These forces are intermolecular force of attraction and thermal energy If intermolecular force of attraction is high as compared to thermal energy particles remains in closest position

Intext QuestionsQ1 Classify the following solids as crystalline and amorphous Sodium chloride quartz glass quartz rubber polyvinyl chloride Teflon

A1 Crystalline

and hence very less movement in particles is observed In this case solid state is the preferred state of matter

Let us revise the general characteristics of solid

i) Fixed mass volume and shape

ii) Strong intermolecular force of attraction

iii) Least intermolecular space

iv) Fixed position of constituent particles

v) Incompressible and rigid

Q2 what type of interactions hold the molecules together in a polar molecular solid[CBSE 2010]A2 The molecules in a solid are held together by van der Waals forces The term van der Waals forces include hydrogen bonding dipole-dipole attraction and London dispersion forces All molecules experience London dispersion forces In addition polar molecules can also experience dipole-dipole interactions So the interactions that holds the molecule together in polar molecular solid are London dispersion force and dipole-dipole interactionsQ3 Write a feature that will distinguish a metallic solid from an ionic solid [CBSE 2010]A3 Metals are malleable and ductile whereas ionic solid are hard and brittle Metallic solid has typical metallic lustre But ionic solid looks dullQ4 Write a point of distinction between a metallic solid and an ionic solid other than metallic lustre [CBSE 2012]A4 Metals are malleable and ductile whereas ionic solid are hard and brittleQ5 Write a distinguish feature of metallic solid [CBSE 2010]A5 The force of attraction in

solid Sodium chloride Quartz Amorphous solid Quartz glass rubber polyvinyl chloride Teflon Q2 why glass is considered as super cooled liquidA2 Glass shows the tendency to flow at slower rate like liquid Hence they considered as super cooled liquidQ3 why the window glass of old buildings show milky appearance with timeA3 Glass is an amorphous solid Amorphous solid has the tendency to develop some crystalline character on heating Due to heating in day over the number of years glass acquires some crystalline character and show milky appearanceQ4 why the glass panes fixed to window or doors of old building become slightly thicker at bottomA4 Glass is super cooled liquid It has the tendency to flow down very slowly Due to this glass pane becomes thicker at the bottom over the timeQ5 Sodium chloride is a crystalline solid It shows the same value of refractive index along all the direction TrueFalse Give reasonA5 FalseCrystalline solid shows anisotropy in properties That is it shows different values for the given physical property in different direction All the crystalline solids show anisotropy in refractive index Therefore sodium chloride will show different values of refractive index on different directions

Q6 Crystalline solid are anisotropic in nature What does this statement means

between the constituent particles is special kind of electrostatic attraction That is the attraction of positively charged kernel with sea of delocalized electronsQ6 which group of solid is electrical conductor as well as malleable and ductile [CBSE 2013]A6 Metallic solidQ7 why graphite is good conductor of electricity although it is a network (covalent solid)A7 The exceptional property of graphite is due to its typical structure In graphite each carbon is covalently bonded with 3 atoms in same layer The fourth valence electron of each atom is free to move in between different layersThis free electron makes the graphite a good conductor of electricity

[CBSE 2011]A6 Anisotropy is defined asrdquo Difference in properties when measured along different axis or from different directionsrdquo Crystalline solid show different values of some of the physical properties like electrical resistance refractive index etcwhen measured along the different directions The anisotropy in crystalline solid arises due to the different arrangement of particles in different directions

Math Function Composition of functions Think of an industrial plant that produce bottles of cold drinks first there is the operation (or function) f that puts the cold drink inside the bottle followed by the opeartion g that close the bottle with the capThis leads to the following definitionDefinition Let f A rarr B and g B rarr C be two functions Then the composition of f and g denoted by gof is defined as the function gof A rarr C given by gof(x) = g(f (x)) forall x isinA

Definition A function f X rarr Y is defined to be invertible if there exists a function g Y rarr X such that gof = IX and fog = IY The function g is called the inverse of f and is denoted by f -1

Thus if f is invertible then f must be one-one and onto and conversely if f is one-one and onto then f must be invertible This fact significantly helps for proving a function f to be invertible by showing that f is one-one and onto specially when the actual inverse of f is not to be determined

Example 1 Let f 2 3 4 5 rarr 3 4 5 9 and g 3 4 5 9 rarr 7 11 15 be functions defined as f(2) = 3 f(3) = 4 f(4) = f(5) = 5 and g (3) = g (4) = 7 and g (5) = g (9) = 11 Find gofSolution We have gof(2) = g (f(2)) = g (3) = 7 gof(3) = g (f(3)) = g (4) = 7gof(4) = g (f(4)) = g (5) = 11 and gof(5) = g (5) = 11Example 2 Find gof and fog if f R rarr R and g R rarr R are given by f(x) = cos x and g (x) = 3x2 Show that gof ne fogSolution We have gof(x) = g(f(x))=g(cosx) = 3 (cos x)2

= 3 cos2 x Similarly fog(x)=f(g (x))= f(3x2)= cos (3x2) Note that 3cos2 x ne cos 3x2 for x = 0 Hence gof ne fogExample 3 Show that if f A rarr B and g B rarr C are onto then gof A rarr C is also ontoSolution Given an arbitrary element z isin C there exists a pre-image y of z under g such that g (y) = z since g is onto Further for y isin B there exists an element x in A with f(x) = y since f is onto Therefore gof(x) = g (f(x)) = g (y) = z showing that gof is onto Example 4 Let Y = n2 n isin N sub N Consider f N rarr Y as f(n) = n2 Show that

f is invertible Find the inverse of fSolution An arbitrary element y in Y is of the form n2 for some n isin N This implies that n =radicy This gives a function g Y rarr N defined by g (y) =radicy Nowgof (n) = g (n2)=radicn2 = n and fog (y) =f(radicy) = (radicy) 2 y which shows that gof=IN and fog= IY Hence f is invertible with f -1 = g

Political Science

Constitution of India-The Preamble

Summary

Objective of the state-To secure equality of status and of opportunity To promote fraternity among all the citizens To assure the dignity of the individuals and Unity and integrity of the nation

Justice-Justice stands for rule of law absence of arbitrariness and a system of equal rights freedom and opportunities for all in a society India seeks social economic and political justice to ensure equality to its citizens

Liberty-Liberty implies the absence of restraints or domination on the activities of an individual such as freedom from slavery serfdom imprisonment despotism etc The Preamble provides for the liberty of thought expression belief faith and worship

Equality-Equality means the absence of privileges or discrimination against any section of the society The Preamble provides for equality of status and opportunity to all the people of the country

Fraternity-The Preamble declares that fraternity has to assure two thingsmdashthe dignity of the individual and the unity and

Execution

Answer the following questions-

Short notes-1 Equality2 Fraternity3 Justice4 Liberty

Homework-Learn

integrity of the nation The word integrity has been added to the Preamble by the 42nd Constitutional Amendment (1976)

Business studies

Human resource management (chapter 1)

On the day of 1504 2020 I have discussed with you the managerial functions and procurement functions of HRM

Today weare going to discuss about the development function integration functions and maintenance function

Development functions-HRM improves the knowledge skills attitude and values of employees so that they the present and future jobs more effectively it includes

1) Development functions of HRM

a) Performance appraisal = It implies systematic evaluation of employees with respect to their performance on the job and their potential for development

b) Training =It is the process by which employees learn knowledge skills and attitudes to achieve organisational and personal goals

c) Executive development = It is the process of developing managerial talent through appropriate program

2) Integration functionsa) HRM reconcile the goals of

organisation with those of its members through integrating function

b) HRM tries to motivate employees to various financial and non financial incentives provided in job specification etc

3) Maintenance functiona) HRM promote and protect the

physical and mental health of employees by providing several types of benefits like housing medical aid etc

b) It Promote Social security measures to employees by providing provident fund pension gratuity maternity benefits

SubjectCOMMERCE

Topic

BUSINESSENVIRONMENT

Summary

Now quickly let us revise the earlier points that we have already done in the last class and let us proceed with the other topics that are there in the chapter

Firstly we will recall the internal and external factors of micro environment and then we

Execution 3 What do you mean by internal factors

in micro environmentAnswerInternal factors refer to all the factors existing within a business firm The internal factors are considered controllable because the enterprise has control over these factors

Development FunctionsPerformance AppraisalTrainingExecution Development

shall proceed in details

Meaning and list of internal and external factors

aInternal factorsInternal factors refer to all the factors existing within a business firm The internal factors are considered controllable because the enterprise has control over these factorsFor an example a company can alter its organization structure policies programmes employees physical facilities and marketing mix to suit the changes in the environmentList of internal factors areCorporate culture mission and objectives top management organizations structure company image and brand equity company resources

b External factorsExternal factors refer to those individual and groups and agencies with which a particular business organization comes into direct and frequent contact in the course of its functioningThese individuals and groups are known as STAKEHOLDERS because they have a stake (financial interest ) in the working and performance of the particular business List of external forces (stakeholders)Customers competitors investors suppliersmiddlemen (marketing intermediaries)financers publics

customers

suppliersfinancers

For an example a company can alter its organization structure policies programmes employees physical facilities and marketing mix to suit the changes in the environment

4 What do you mean by external factors in micro environment

AnswerExternal factors refer to those individual and groups and agencies with which a particular business organization comes into direct and frequent contact in the course of its functioningThese individuals and groups are known as STAKEHOLDERS because they have a stake (financial interest) in the working and performance of the particular business

3Who are stakeholdersSTAKEHOLDERS are individuals and groups who have a stake (financial interest ) in the working and performance of the particular business 4Discuss the internal factors in briefa Corporate CultureThe values beliefs and attitudes of the founders and top management of the company exercise a strong influence on what the cmpaany stands for how it does things and what it considers importantbMission and objectivesThe business philosophy and purpose of a comoany guide it prioritiesbusiness strategiesproduct market scope and development scope

cTop management structurethe composition of board of directors the degree of professionalization of management and the organizational structure of a company have important bearing on its business decisions

dPower structureThe internal power relationship between the board of directors and the chief executive is an important factor

e Company image and brand equityThe image and brand equity of the company play a significant role in raising finance forming alliance choosing dealers and suppliers launching new products entering foreign markets

5 What is Macro environmentAnswerMacro environment refers to the general

competitors

middlemen

publics

Fig STAKEHOLDERS OF A COMPANY

Apart from micro environment the other main dimension of business environment isMacro environment Macro environment refers to the general environment or remote environment within which a business firm and forces in its micro environment operateA company does not directly or regularly interact with the micro environmentTherefore macro environment is also known as indirect action EnvironmentThe macro environment forces are less controllable than the micro forces

Macro environment consists of the following components

POLITICAL AND LEGAL ENVIRONMENT

ECONOMIC SOCIAL AND ENVIRONMENT

CULTURAL

ENVIRONMENT

TECHNOLOGICAL ENVIRONMENT

Fig COMPONENTS OF MACRO ENVIRONMENT

environment or remote environment within which a business firm and forces in its micro environment operateA company does not directly or regularly interact with the micro environmentTherefore macro environment is also known as indirect action EnvironmentThe macro environment forces are less controllable than the micro forces 6 What are the components of macro environmenta Political and legal environmentb Economic environmentc Social and cultural environmentd Technological environment

Computer Science

Logic gates

Digital systems are said to be constructed by using logic gates These gates are the AND OR NOT NAND NOR EXOR and EXNOR

BUSINESS FIRM

gates The basic operations are described below with the aid of truth tables

AND gate

The AND gate is an electronic circuit that gives a high output (1) only if all its inputs are high A dot () is used to show the AND operation ie AB Bear in mind that this dot is sometimes omitted ie ABOR gate

The OR gate is an electronic circuit that gives a high output (1) if one or more of its inputs are high A plus (+) is used to show the OR operationNOT gate

The NOT gate is an electronic circuit that produces an inverted version of the input at its output It is also known as an inverter If the input variable is A the inverted output is known as NOT A This is also shown as A or A with a bar over the top as shown at the outputs The diagrams below show two ways that the NAND logic gate can be configured to produce a NOT gate It can also be done using NOR logic gates in the same way

NAND gate

This is a NOT-AND gate which is equal to an AND gate followed by a NOT gate The outputs of all NAND gates are high if any of the inputs are low The symbol is an AND gate with a small circle on the output The small circle represents inversion

NOR gate

This is a NOT-OR gate which is equal to an OR gate followed by a NOT gate The outputs of all NOR gates are low if any of the inputs are highThe symbol is an OR gate with a small circle on the output The small circle represents inversion

EXOR gate

The Exclusive-OR gate is a circuit which will give a high output if either but not both of its two inputs are high An encircled plus sign ( ) is used to show the EOR operation

EXNOR gate

The Exclusive-NOR gate circuit does the opposite to the EOR gate It will give a low output if either but not both of its two inputs are high The symbol is an EXOR gate with a small circle on the output The small circle represents inversion The NAND and NOR gates are called universal functions since with either one the AND and OR functions and NOT can be generated

Note A function in sum of products form can be implemented using NAND gates by replacing all AND and OR gates by NAND gates A function in product of sums form can be implemented using NOR gates by replacing all AND and OR gates by NOR gates

Logic gate symbols

Table 2 is a summary truth table of the inputoutput combinations for the NOT gate together with all possible inputoutput combinations for the other gate functions Also note that a truth table with n inputs has 2n rows You can compare the outputs of different gates

Logic gates representation using the Truth table

Example

A NAND gate can be used as a NOT gate using either of the following wiring configurations

Subject Eng Literature (The Tempest ndash William Shakespeare) Topic Act III Scene 3 Lines 53 to 110 (End of the scene) Date 16th April 2020 (2nd Period)

[Students should read the original play and also the paraphrase given in the school prescribed textbook]Summary Questions amp Answers

o Seeing this strange scene all are inclined to believe the tales told by travelers that there truly are ldquounicornsrdquo and ldquothe phoenixrsquo thronerdquo

o As they are about to sit down to the feast the banquet is snatched away by a harpy (Ariel disguised) A spiritrsquos voice (Arielrsquos voice) denounces Alonso Sebastian and Antonio with particular

1 ARIEL You are three men of sin whom Destiny

(Line 53-58)That hath to instrument this

lower world And what is int the never-surfeited sea

Hath caused to belch up you and on this island

Where man doth not inhabit you rsquomongst men

Being most unfit to live I have made you mad

reference to their crime in expelling Prospero from Milan They have not received any punishment for their deed earlier but the time for their punishment has arrived Upon Alonso it pronounces ldquolingering perdition worse than deathrdquo from which there is no remedy except through sincere repentance Ariel then vanishes in thunder and the shapes enter again and carry away the table

o Prospero watching invisibly is very pleased with the performance of Ariel and his (Prosperorsquos) ldquomeaner ministersrdquo All his enemies are now in his power and are in a fit of desperation He then leaves them and goes to see how Ferdinand and Miranda are getting on

o Alonso is now much humbled and penitent with the after effect of the spiritrsquos denunciation of his crimes He believes that his son is lost forever After this all disperse being stricken mad by the speech of the spirit

o Gonzalo fearing that they may do violence to themselves or to one another follows them and bid others to follow

(a) To whom does Ariel disguised as a harpy call the three sinners What game did Fate of Destiny play with

them

The three sinners called by Ariel are Alonso Sebastian and Antonio It was Destiny which had caused the ocean to cast the three sinners on the shore Though the ocean is all the time devouring whatever appears on its surface and is never satisfied with its continual swallowing of the ships and men in the present case the ocean had cast these three sinners on the shore without killing them

(b) Who had jointly been responsible for the conspiracy against Prospero What is Prosperorsquos purpose behind all this

Three men Alonso Sebastian and Antonio had jointly

been responsible for the conspiracy against Prospero They had driven out Prospero form Milan Prosperorsquos purpose is to make these three sinners realize the wrong they had done He wants them to repent for their criminal deeds because repentance leads to self-esteem(c )What does Ariel (the harpy) tell Alonso and his companions when they take out their swords to attack him

Seeing them drawing their swords Ariel (harpy) tells them that he and his companions are the instruments of destiny and that it is not possible for human beings to do them any injury He says that the swords of human beings can not injure even a minute part of his feathers Their swords are as ineffective against him and his companions as against the wind or the water

(d) Give the explanatory meanings of the following expressions in the context of the above extract

(i)Never surfeited (ii) Belch up (iii) lsquomongst men

(i) Never surfeited never led to satisfaction

(ii) Belch up cast ashore(iii) lsquomongst men in human

society2

I and my fellows (Line 60-65)

Are ministers of Fate The elementsOf whom your swords are tempered may as wellWound the loud winds or with bemocked-at stabsKill the still-closing waters as diminishOne dowl thats in my plume

IMPORTANT PASSAGES EXPLAINED

The elements

(Line 61-66)Of whom your swords are tempered may

as wellWound the loud winds or with

bemocked-at stabs

(a) Who is lsquoIrsquo Who are his lsquofellowsrdquo

lsquoIrsquo is referred to Ariel in disguise of a harpy His lsquofellowsrsquo are other spirits serving Prospero the real Duke of Milan who has acquired supernatural powers after being banished from his Dukedom Prospero has settled in this uninhabited island

(b) What are the elements that have temperrsquod the swords Why will it not work against the speaker

The swords (of Alonso and his companions) are tempered by metal (steel) which is taken out of the earth and refined by

Kill the still-closing waters as diminishOne dowl thats in my plume My fellow

ministersAre like invulnerable

In these words Ariel reminds the King and his companions of the utter futility of drawing swords against himself and his fellows Ariel drives Alonso Antonio and Sebastian the three men of sin to desperation ndash a state in which men do violence to themselves They draw swords to strike Ariel But Ariel reminds them that he and the other spirits are the ministers of destiny and nothing can wound them The steel of which their swords are made of may cut the wind or water which being divided always closes up again Even supposing that such things may be possible it is quite impossible that their swords will cut one feather in their plume They are incapable of being wounded by any sword of man Hence it is foolish on their part to attempt to strike at Ariel and his fellow-spirits

For which foul deed

(Line 72-75)The powers delaying not forgetting

haveIncensed the seas and shores yea all the

creatures Against your peace

Ariel enters like a harpy and remaining invisible tells Alonso Sebastian and Antonio that he and other harpies are the agents of Destiny appointed to carry out her decrees He tells them that their punishment for the crime against Prospero which has been so long deferred is now to fall upon them He reminds them that they had expelled Prospero from Milan and set him and his innocent child adrift on the sea and that the sea had paid them back for their sin by the shipwreck and by the calamities they have suffered He tells them that the powers above which did not forget this mean treachery but only deferred the punishment have now engaged the seas and the shores and all living beings including him and his comrades against them The very elements and supernatural agency Ariel adds have taken up the avenging of their crime against Prospero

the action of fire It may cut the wind or water which being divided always closes up again

The sword will not work against the spirits and the harpy because they are the ministers of destiny and nothing can wound them nor it will cut a single feather in their plume

(c )What is the meaning of lsquodowlrsquo in the last line

The term lsquodowlrsquo means a filament or the smallest part of a feather In this context Ariel in disguise of harpy says that their sword cannot even damage the smallest filament of their (Arielrsquos and other spirits) feathers as they are incapable of being wounded by any sword of man

(d) What does the speaker remind the listeners about

Ariel in disguise of harpy reminds Alonso the King of Naples Sebastian Alonsorsquos brother and Antonio the present Duke of Milan and the treacherous brother of Prospero as they being three men of sin He even reminds them that their punishment for their crime against Prospero which has been so long deferred now falls upon them He reminds them that they have expelled Prospero from Milan and has set him along with his innocent infant daughter adrift on the sea So the sea has paid them back for their sin by their shipwreck and the calamities they have suffered since then The harpy rebukes Alonso of his sin that has incensed the Gods and has deprived him of his son as a punishment

(e) How do they respond

When Ariel in disguise of a harpy reminds Alonso Sebastian and Antonio of their past misdeeds and sin Alonso has a look of terror and confusion in his eyes He utters the words of sincere repentance wrung out of his conscience-stricken heart It appears to him that all the elements of nature the sea-waves the wind and the thunder proclaiming a loud voice in the name of Prospero and the crime Alonso has committed against him They are calling upon him to repent There is a deep storm raging in Alonsorsquos breast and the echoes of that storm are ringing in his ears like a clear note of wind-instrument A note of denunciation of Alonsorsquos crime leaves him much humbled and penitent and confirms his belief that his son is lost forever But Sebastian and Antonio shows some courage instead of repentance They wish to kill the spirits or devils if it appears

3

Of my instruction hast thou nothing bated (Line 85-93)

In what thou hast to say So with good life

And observation strange my meaner ministers

Their several kinds have done My high charms work

And these mine enemies are all knit upIn their distractions They now are in my

powerAnd in these fits I leave them while I visitYoung Ferdinand whom they suppose is

drownedAnd his and mine loved darling

Methought the billows spoke and (Line 96-99)

told me of itThe winds did sing it to me and the

thunderThat deep and dreadful organ-pipe

pronouncedThe name of Prosper It did bass my

trespass

These are the words of contrition coming from Alonso Ariel has driven him to a deep repentance for conspiring with Antonio against Prospero He now feels a sincere remorse It appears to him that all the elements of nature the sea-waves the wind and the thunder proclaimed with a loud voice the name of Prospero and the crime Alonso had committed against him They are calling upon him to repent There is a deep storm raging in Alonsorsquos breast and the echoes of that storm are ringing in his ears like the clear note of a wind-instrument

Comment These are the words of sincere repentance wrung out of the conscience-stricken heart of Alonso Alonso who is the lesser villain is the first to give way to remorse under the effect of Arielrsquos speech The words of Ariel seem to him to be the voice of conscience speaking to him He is driven to desperation a state in which he might do violence to his life

(a) Identify the speaker State the context

Prospero the ruler of the island is the speaker The famous banquet scene has been enacted very well Ariel and his junior spirits have played their roles excellently Prospero is glad to say words of praise for them(b) In what way the speakerrsquos instructions have been carried out

According to Prosperorsquos instructions a banquet was presented before the King of Naples and his companions when they were tired and hungry Just when they were preparing to eat the feast the banquet was suddenly removed by exercising supernatural powers All this was done by Ariel Prosperorsquos chief assistant and a powerful spirit

Ariel not only made the feast disappear but also delivered his speech blaming the King and his two companions for their past wicked deeds He warned them to repent for their misdeeds or suffer forever on that uninhabited island

(c) Who are referred to as lsquomeaner ministersrsquo What have they done

Prospero refers as lsquomeaner ministersrsquo to his other lesser spirits who were assisting Ariel in presenting a scene before the kingrsquos party They entered the scene to the accompaniment of music They assumed several strange shapes and brought in a banquet Then they danced about it with gentle actions of salutations thus inviting the King and others to eat the feast

These spirits play their role again when Ariel in the shape of a harpy quits the scene These shapes enter again and dancing with mocking gestures carry away the table

(d) Who are the speakerrsquos enemies What has happened to them

King of Naples Alonso his brother Sebastian and the present Duke of Milan Antonio (Prosperorsquos own brother) are Prosperorsquos enemies With the turn of events they have all been washed ashore on the island which is ruled by Prospero the great magician Actually this happened after the shipwreck caused by a storm which was raised by Prospero with the purpose of bringing these people to his island Prosperorsquos spirits have already confused and terrified these enemies and they are under Prosperorsquos control He can treat them as he likes

(e) What does he say about Ferdinand Explain what is meant by ldquohellip his and mine darlingrdquo

Prospero knows that Alonsorsquos son prince Ferdinand is alive though his father thinks that the prince has been drowned

Prospero refers to his daughter Miranda who is dear to him She is also very dear to Prince Ferdinand who has fallen in love with her They are waiting to be married soon for which they have received Prosperorsquos consent

4

ALONSO O it is monstrous monstrous (Line 95-102)

Methought the billows spoke and told me of it

The winds did sing it to me and the thunderThat deep and dreadful organ-

pipe pronouncedThe name of Prosper It did bass

my trespassTherefore my son ithrsquo ooze is

bedded andIll seek him deeper than eer

plummet soundedAnd with him there lie mudded

(a) In what way does Alonso express his horror when his conscience is awakened by Arielrsquos words

When Alonsorsquos conscience is awakened by Arielrsquos words he expresses his horror at what he has heard He gets the feeling that the waves of the ocean the wind and the loud thunder have spoken to him and uttered the name of Prospero Because of being reminded of his crime in a very loud and rough voice he comes to realize that he has lost his son for his past misdeeds

(b) What does Alonso imagine about his son What does Alonso want to do in his desperate state

Alonso imagines that his son is lying in the mud at the bottom of the sea He feels desperate that he wants to drown himself in the ocean deeper than the plumb-line has ever gone He wants to lie with his son at the bottom of the sea

(c) How do Sebastian and Antonio want to face the evil spirits

Sebastian says that he is not at all afraid of what the harpy has said and that he is prepared to fight any number of such monsters if they appear before him only one at a time Antonio says that he would support Sebastian in the fight against the fiendsyyy

(d) Why does Gonzalo ask Adrian to follow the three men

Gonzalo tells Adrian that all the three men namely Alonso Sebastian and Antonio are in a wild and reckless mood The thought of the heinous crime of which they are guilty has begun to torment their minds So he asks Adrian to follow those three men without loss of time and prevent them from doing anything which the turmoil in their minds might lead them to do

(e) What opinion do you form of Alonso from the above extract

Alonso who is the lesser villain is the first to give way to remorse under the effect of Arielrsquos speech The words of Ariel seem to him to be the voice of conscience speaking to him He is driven to desperation a state in which he might do violence to his life

Subject =Accounts

Ac-12 15420 topic-pL Appropriation ac

PROFIT AND LOSS APPROPRIATION ACCOUNT

MEANING AND PREPARATIONProfit and Loss Appropriation Account is merely an extension of the Profit and Loss Account of the firm The profit of the firm has to be distributed amongst the partners in their respective profit sharing ratio But before its distribution it needs to be adjusted All Adjustments like partnerrsquos salary partnerrsquos commission interest on capital interest on drawings etc are made in this account These adjustments will reduce the amount of profit for distribution This adjusted profit will be distributed amongst the partners in their profit sharing ratio To prepare it at first the balance of Profit and Loss Account is transferred to this account The journal entries for the preparation of Profit and Loss Appropriation Account are given below

1 for transfer of the balance of Profit and Loss Account to Profit and Loss Appropriation Account

(a) In case of Net Profit

Profit and Loss Ac helliphelliphelliphelliphellipDrTo Profit and Loss Appropriation Ac(Net Profit transferred to Profit and Loss Appropriation Ac)

(b)In case of Net Loss

Profit and Loss Appropriation Achelliphelliphellip DrTo Profit and Loss Ac(Net Loss transferred to Profit and Loss Appropriation Ac)

2 for Interest on Capital

For transferring on Interest on CapitalProfit and Loss Appropriation Achelliphelliphellip DrTo Interest on Capital Ac(Interest on capital transferred to Profit amp Loss Appropriation Ac)

3 for Interest on Drawings

For transferring Interest on Drawings Interest on Drawings Achelliphelliphelliphelliphelliphellip DrTo Profit and Loss Appropriation Ac(Interest on drawing transferred to Profit amp Loss Appropriation Ac)

4 For Partnerrsquos SalaryFor transfer of partnerrsquos SalaryProfit and Loss Appropriation Achelliphellip DrTo Salary Ac(Salary transferred to profit amp Loss Appropriation Ac)

5 For Partnerrsquos CommissionFor transferring commissionProfit and Loss Appropriation Achelliphelliphellip DrTo Commission Ac(Commission transferred to Profit and Loss Appropriation Ac)

6 For Transfer of agreed amount to General ReserveProfit and Loss Appropriation Ac helliphellipDrTo General Reserve Ac(Transfer to General Reserve)

7 for share of Profit or Loss appropriation(a) If ProfitProfit and Loss Appropriation Achelliphellip DrTo Partnerrsquos CapitalCurrent Ac(Profit transferred to capitalcurrent Ac)(b) If LossPartnerrsquos Capital Current Achelliphelliphelliphellip DrTo Profit and Loss Appropriation Ac(Loss transferred to capitalcurrent Ac)

THE FORMAT OF PROFIT AND LOSS APPROPRIATION

Profit and Loss Appropriation Account for the year endedhelliphelliphelliphellip

Particulars Amount Particulars Amount

To PL Ac (loss) By pL Ac (profit)

To Interest on capital BY Interest on drawings

To partner`s commission by Partner`s capital Ac ( loss)

To Partner`s salary To Interest on partner`s loan To General Reserve To Partner`s Capital AC (Profit)

Subject= Economics

MOVEMENT ALONG THE DEMAND CURVE (CHANGE IN QUANTITY DEMANDED)In law of demand you have already studied the inverse relationship between price and quantity demanded When quantity demanded of a commodity changes due to change in its price keeping other factors constant it is called change in quantity demanded It is graphically expressed as a movement along the same demand curve There can be either a downward movement or an upward movement along the same demand curve Upward movement along the same demand curve is called contraction of demand or decrease in quantity demanded and downward movement along the same demand curve is known as expansion of demand or increase in quantity demanded

Extention of demandd

price (rs)p A

B Extentionp1 d

Q Q1

Quantity demanded ( in units)

Contraction of demandd

p2 Ccontraction

p APrice (Rs)

d

Q2 Q

Quantity demanded (in units)

Explanation of movement of demand A fall in price from OP to OP1 leads to increase in quantity demanded from OQ to OQ1 (expansion of demand) resulting in a downward movement from point A to point B along the same demand curve DD When Price rises from OP to OP2 quantity demanded falls from OQ to OQ2 (contraction of demand) leading to an upward movement from point A to point C along the same demand curve DD

  • Activity Series of Metals
    • Drawbacks of Rutherfordrsquos model of atom
      • Electromagnetic radiations
      • Properties of electromagnetic radiations
      • Characteristics of electromagnetic radiations
        • Plancks Quantum Theory-
        • Photoelectric effect
          • Intext Questions
            • Logic gates
            • Digital systems are said to be constructed by using logic gates These gates are the AND OR NOT NAND NOR EXOR and EXNOR gates The basic operations are described below with the aid of truth tables
            • AND gate
            • Example
Page 43:  · Web viewSubject. Topic. Summary. Execution. English 1 . Chapter 1 naming words . Page 8. Write the names of these pictures:- Person:-1. father. 2.Firefighter 3.doctor 4 ...

and hence very less movement in particles is observed In this case solid state is the preferred state of matter

Let us revise the general characteristics of solid

i) Fixed mass volume and shape

ii) Strong intermolecular force of attraction

iii) Least intermolecular space

iv) Fixed position of constituent particles

v) Incompressible and rigid

Q2 what type of interactions hold the molecules together in a polar molecular solid[CBSE 2010]A2 The molecules in a solid are held together by van der Waals forces The term van der Waals forces include hydrogen bonding dipole-dipole attraction and London dispersion forces All molecules experience London dispersion forces In addition polar molecules can also experience dipole-dipole interactions So the interactions that holds the molecule together in polar molecular solid are London dispersion force and dipole-dipole interactionsQ3 Write a feature that will distinguish a metallic solid from an ionic solid [CBSE 2010]A3 Metals are malleable and ductile whereas ionic solid are hard and brittle Metallic solid has typical metallic lustre But ionic solid looks dullQ4 Write a point of distinction between a metallic solid and an ionic solid other than metallic lustre [CBSE 2012]A4 Metals are malleable and ductile whereas ionic solid are hard and brittleQ5 Write a distinguish feature of metallic solid [CBSE 2010]A5 The force of attraction in

solid Sodium chloride Quartz Amorphous solid Quartz glass rubber polyvinyl chloride Teflon Q2 why glass is considered as super cooled liquidA2 Glass shows the tendency to flow at slower rate like liquid Hence they considered as super cooled liquidQ3 why the window glass of old buildings show milky appearance with timeA3 Glass is an amorphous solid Amorphous solid has the tendency to develop some crystalline character on heating Due to heating in day over the number of years glass acquires some crystalline character and show milky appearanceQ4 why the glass panes fixed to window or doors of old building become slightly thicker at bottomA4 Glass is super cooled liquid It has the tendency to flow down very slowly Due to this glass pane becomes thicker at the bottom over the timeQ5 Sodium chloride is a crystalline solid It shows the same value of refractive index along all the direction TrueFalse Give reasonA5 FalseCrystalline solid shows anisotropy in properties That is it shows different values for the given physical property in different direction All the crystalline solids show anisotropy in refractive index Therefore sodium chloride will show different values of refractive index on different directions

Q6 Crystalline solid are anisotropic in nature What does this statement means

between the constituent particles is special kind of electrostatic attraction That is the attraction of positively charged kernel with sea of delocalized electronsQ6 which group of solid is electrical conductor as well as malleable and ductile [CBSE 2013]A6 Metallic solidQ7 why graphite is good conductor of electricity although it is a network (covalent solid)A7 The exceptional property of graphite is due to its typical structure In graphite each carbon is covalently bonded with 3 atoms in same layer The fourth valence electron of each atom is free to move in between different layersThis free electron makes the graphite a good conductor of electricity

[CBSE 2011]A6 Anisotropy is defined asrdquo Difference in properties when measured along different axis or from different directionsrdquo Crystalline solid show different values of some of the physical properties like electrical resistance refractive index etcwhen measured along the different directions The anisotropy in crystalline solid arises due to the different arrangement of particles in different directions

Math Function Composition of functions Think of an industrial plant that produce bottles of cold drinks first there is the operation (or function) f that puts the cold drink inside the bottle followed by the opeartion g that close the bottle with the capThis leads to the following definitionDefinition Let f A rarr B and g B rarr C be two functions Then the composition of f and g denoted by gof is defined as the function gof A rarr C given by gof(x) = g(f (x)) forall x isinA

Definition A function f X rarr Y is defined to be invertible if there exists a function g Y rarr X such that gof = IX and fog = IY The function g is called the inverse of f and is denoted by f -1

Thus if f is invertible then f must be one-one and onto and conversely if f is one-one and onto then f must be invertible This fact significantly helps for proving a function f to be invertible by showing that f is one-one and onto specially when the actual inverse of f is not to be determined

Example 1 Let f 2 3 4 5 rarr 3 4 5 9 and g 3 4 5 9 rarr 7 11 15 be functions defined as f(2) = 3 f(3) = 4 f(4) = f(5) = 5 and g (3) = g (4) = 7 and g (5) = g (9) = 11 Find gofSolution We have gof(2) = g (f(2)) = g (3) = 7 gof(3) = g (f(3)) = g (4) = 7gof(4) = g (f(4)) = g (5) = 11 and gof(5) = g (5) = 11Example 2 Find gof and fog if f R rarr R and g R rarr R are given by f(x) = cos x and g (x) = 3x2 Show that gof ne fogSolution We have gof(x) = g(f(x))=g(cosx) = 3 (cos x)2

= 3 cos2 x Similarly fog(x)=f(g (x))= f(3x2)= cos (3x2) Note that 3cos2 x ne cos 3x2 for x = 0 Hence gof ne fogExample 3 Show that if f A rarr B and g B rarr C are onto then gof A rarr C is also ontoSolution Given an arbitrary element z isin C there exists a pre-image y of z under g such that g (y) = z since g is onto Further for y isin B there exists an element x in A with f(x) = y since f is onto Therefore gof(x) = g (f(x)) = g (y) = z showing that gof is onto Example 4 Let Y = n2 n isin N sub N Consider f N rarr Y as f(n) = n2 Show that

f is invertible Find the inverse of fSolution An arbitrary element y in Y is of the form n2 for some n isin N This implies that n =radicy This gives a function g Y rarr N defined by g (y) =radicy Nowgof (n) = g (n2)=radicn2 = n and fog (y) =f(radicy) = (radicy) 2 y which shows that gof=IN and fog= IY Hence f is invertible with f -1 = g

Political Science

Constitution of India-The Preamble

Summary

Objective of the state-To secure equality of status and of opportunity To promote fraternity among all the citizens To assure the dignity of the individuals and Unity and integrity of the nation

Justice-Justice stands for rule of law absence of arbitrariness and a system of equal rights freedom and opportunities for all in a society India seeks social economic and political justice to ensure equality to its citizens

Liberty-Liberty implies the absence of restraints or domination on the activities of an individual such as freedom from slavery serfdom imprisonment despotism etc The Preamble provides for the liberty of thought expression belief faith and worship

Equality-Equality means the absence of privileges or discrimination against any section of the society The Preamble provides for equality of status and opportunity to all the people of the country

Fraternity-The Preamble declares that fraternity has to assure two thingsmdashthe dignity of the individual and the unity and

Execution

Answer the following questions-

Short notes-1 Equality2 Fraternity3 Justice4 Liberty

Homework-Learn

integrity of the nation The word integrity has been added to the Preamble by the 42nd Constitutional Amendment (1976)

Business studies

Human resource management (chapter 1)

On the day of 1504 2020 I have discussed with you the managerial functions and procurement functions of HRM

Today weare going to discuss about the development function integration functions and maintenance function

Development functions-HRM improves the knowledge skills attitude and values of employees so that they the present and future jobs more effectively it includes

1) Development functions of HRM

a) Performance appraisal = It implies systematic evaluation of employees with respect to their performance on the job and their potential for development

b) Training =It is the process by which employees learn knowledge skills and attitudes to achieve organisational and personal goals

c) Executive development = It is the process of developing managerial talent through appropriate program

2) Integration functionsa) HRM reconcile the goals of

organisation with those of its members through integrating function

b) HRM tries to motivate employees to various financial and non financial incentives provided in job specification etc

3) Maintenance functiona) HRM promote and protect the

physical and mental health of employees by providing several types of benefits like housing medical aid etc

b) It Promote Social security measures to employees by providing provident fund pension gratuity maternity benefits

SubjectCOMMERCE

Topic

BUSINESSENVIRONMENT

Summary

Now quickly let us revise the earlier points that we have already done in the last class and let us proceed with the other topics that are there in the chapter

Firstly we will recall the internal and external factors of micro environment and then we

Execution 3 What do you mean by internal factors

in micro environmentAnswerInternal factors refer to all the factors existing within a business firm The internal factors are considered controllable because the enterprise has control over these factors

Development FunctionsPerformance AppraisalTrainingExecution Development

shall proceed in details

Meaning and list of internal and external factors

aInternal factorsInternal factors refer to all the factors existing within a business firm The internal factors are considered controllable because the enterprise has control over these factorsFor an example a company can alter its organization structure policies programmes employees physical facilities and marketing mix to suit the changes in the environmentList of internal factors areCorporate culture mission and objectives top management organizations structure company image and brand equity company resources

b External factorsExternal factors refer to those individual and groups and agencies with which a particular business organization comes into direct and frequent contact in the course of its functioningThese individuals and groups are known as STAKEHOLDERS because they have a stake (financial interest ) in the working and performance of the particular business List of external forces (stakeholders)Customers competitors investors suppliersmiddlemen (marketing intermediaries)financers publics

customers

suppliersfinancers

For an example a company can alter its organization structure policies programmes employees physical facilities and marketing mix to suit the changes in the environment

4 What do you mean by external factors in micro environment

AnswerExternal factors refer to those individual and groups and agencies with which a particular business organization comes into direct and frequent contact in the course of its functioningThese individuals and groups are known as STAKEHOLDERS because they have a stake (financial interest) in the working and performance of the particular business

3Who are stakeholdersSTAKEHOLDERS are individuals and groups who have a stake (financial interest ) in the working and performance of the particular business 4Discuss the internal factors in briefa Corporate CultureThe values beliefs and attitudes of the founders and top management of the company exercise a strong influence on what the cmpaany stands for how it does things and what it considers importantbMission and objectivesThe business philosophy and purpose of a comoany guide it prioritiesbusiness strategiesproduct market scope and development scope

cTop management structurethe composition of board of directors the degree of professionalization of management and the organizational structure of a company have important bearing on its business decisions

dPower structureThe internal power relationship between the board of directors and the chief executive is an important factor

e Company image and brand equityThe image and brand equity of the company play a significant role in raising finance forming alliance choosing dealers and suppliers launching new products entering foreign markets

5 What is Macro environmentAnswerMacro environment refers to the general

competitors

middlemen

publics

Fig STAKEHOLDERS OF A COMPANY

Apart from micro environment the other main dimension of business environment isMacro environment Macro environment refers to the general environment or remote environment within which a business firm and forces in its micro environment operateA company does not directly or regularly interact with the micro environmentTherefore macro environment is also known as indirect action EnvironmentThe macro environment forces are less controllable than the micro forces

Macro environment consists of the following components

POLITICAL AND LEGAL ENVIRONMENT

ECONOMIC SOCIAL AND ENVIRONMENT

CULTURAL

ENVIRONMENT

TECHNOLOGICAL ENVIRONMENT

Fig COMPONENTS OF MACRO ENVIRONMENT

environment or remote environment within which a business firm and forces in its micro environment operateA company does not directly or regularly interact with the micro environmentTherefore macro environment is also known as indirect action EnvironmentThe macro environment forces are less controllable than the micro forces 6 What are the components of macro environmenta Political and legal environmentb Economic environmentc Social and cultural environmentd Technological environment

Computer Science

Logic gates

Digital systems are said to be constructed by using logic gates These gates are the AND OR NOT NAND NOR EXOR and EXNOR

BUSINESS FIRM

gates The basic operations are described below with the aid of truth tables

AND gate

The AND gate is an electronic circuit that gives a high output (1) only if all its inputs are high A dot () is used to show the AND operation ie AB Bear in mind that this dot is sometimes omitted ie ABOR gate

The OR gate is an electronic circuit that gives a high output (1) if one or more of its inputs are high A plus (+) is used to show the OR operationNOT gate

The NOT gate is an electronic circuit that produces an inverted version of the input at its output It is also known as an inverter If the input variable is A the inverted output is known as NOT A This is also shown as A or A with a bar over the top as shown at the outputs The diagrams below show two ways that the NAND logic gate can be configured to produce a NOT gate It can also be done using NOR logic gates in the same way

NAND gate

This is a NOT-AND gate which is equal to an AND gate followed by a NOT gate The outputs of all NAND gates are high if any of the inputs are low The symbol is an AND gate with a small circle on the output The small circle represents inversion

NOR gate

This is a NOT-OR gate which is equal to an OR gate followed by a NOT gate The outputs of all NOR gates are low if any of the inputs are highThe symbol is an OR gate with a small circle on the output The small circle represents inversion

EXOR gate

The Exclusive-OR gate is a circuit which will give a high output if either but not both of its two inputs are high An encircled plus sign ( ) is used to show the EOR operation

EXNOR gate

The Exclusive-NOR gate circuit does the opposite to the EOR gate It will give a low output if either but not both of its two inputs are high The symbol is an EXOR gate with a small circle on the output The small circle represents inversion The NAND and NOR gates are called universal functions since with either one the AND and OR functions and NOT can be generated

Note A function in sum of products form can be implemented using NAND gates by replacing all AND and OR gates by NAND gates A function in product of sums form can be implemented using NOR gates by replacing all AND and OR gates by NOR gates

Logic gate symbols

Table 2 is a summary truth table of the inputoutput combinations for the NOT gate together with all possible inputoutput combinations for the other gate functions Also note that a truth table with n inputs has 2n rows You can compare the outputs of different gates

Logic gates representation using the Truth table

Example

A NAND gate can be used as a NOT gate using either of the following wiring configurations

Subject Eng Literature (The Tempest ndash William Shakespeare) Topic Act III Scene 3 Lines 53 to 110 (End of the scene) Date 16th April 2020 (2nd Period)

[Students should read the original play and also the paraphrase given in the school prescribed textbook]Summary Questions amp Answers

o Seeing this strange scene all are inclined to believe the tales told by travelers that there truly are ldquounicornsrdquo and ldquothe phoenixrsquo thronerdquo

o As they are about to sit down to the feast the banquet is snatched away by a harpy (Ariel disguised) A spiritrsquos voice (Arielrsquos voice) denounces Alonso Sebastian and Antonio with particular

1 ARIEL You are three men of sin whom Destiny

(Line 53-58)That hath to instrument this

lower world And what is int the never-surfeited sea

Hath caused to belch up you and on this island

Where man doth not inhabit you rsquomongst men

Being most unfit to live I have made you mad

reference to their crime in expelling Prospero from Milan They have not received any punishment for their deed earlier but the time for their punishment has arrived Upon Alonso it pronounces ldquolingering perdition worse than deathrdquo from which there is no remedy except through sincere repentance Ariel then vanishes in thunder and the shapes enter again and carry away the table

o Prospero watching invisibly is very pleased with the performance of Ariel and his (Prosperorsquos) ldquomeaner ministersrdquo All his enemies are now in his power and are in a fit of desperation He then leaves them and goes to see how Ferdinand and Miranda are getting on

o Alonso is now much humbled and penitent with the after effect of the spiritrsquos denunciation of his crimes He believes that his son is lost forever After this all disperse being stricken mad by the speech of the spirit

o Gonzalo fearing that they may do violence to themselves or to one another follows them and bid others to follow

(a) To whom does Ariel disguised as a harpy call the three sinners What game did Fate of Destiny play with

them

The three sinners called by Ariel are Alonso Sebastian and Antonio It was Destiny which had caused the ocean to cast the three sinners on the shore Though the ocean is all the time devouring whatever appears on its surface and is never satisfied with its continual swallowing of the ships and men in the present case the ocean had cast these three sinners on the shore without killing them

(b) Who had jointly been responsible for the conspiracy against Prospero What is Prosperorsquos purpose behind all this

Three men Alonso Sebastian and Antonio had jointly

been responsible for the conspiracy against Prospero They had driven out Prospero form Milan Prosperorsquos purpose is to make these three sinners realize the wrong they had done He wants them to repent for their criminal deeds because repentance leads to self-esteem(c )What does Ariel (the harpy) tell Alonso and his companions when they take out their swords to attack him

Seeing them drawing their swords Ariel (harpy) tells them that he and his companions are the instruments of destiny and that it is not possible for human beings to do them any injury He says that the swords of human beings can not injure even a minute part of his feathers Their swords are as ineffective against him and his companions as against the wind or the water

(d) Give the explanatory meanings of the following expressions in the context of the above extract

(i)Never surfeited (ii) Belch up (iii) lsquomongst men

(i) Never surfeited never led to satisfaction

(ii) Belch up cast ashore(iii) lsquomongst men in human

society2

I and my fellows (Line 60-65)

Are ministers of Fate The elementsOf whom your swords are tempered may as wellWound the loud winds or with bemocked-at stabsKill the still-closing waters as diminishOne dowl thats in my plume

IMPORTANT PASSAGES EXPLAINED

The elements

(Line 61-66)Of whom your swords are tempered may

as wellWound the loud winds or with

bemocked-at stabs

(a) Who is lsquoIrsquo Who are his lsquofellowsrdquo

lsquoIrsquo is referred to Ariel in disguise of a harpy His lsquofellowsrsquo are other spirits serving Prospero the real Duke of Milan who has acquired supernatural powers after being banished from his Dukedom Prospero has settled in this uninhabited island

(b) What are the elements that have temperrsquod the swords Why will it not work against the speaker

The swords (of Alonso and his companions) are tempered by metal (steel) which is taken out of the earth and refined by

Kill the still-closing waters as diminishOne dowl thats in my plume My fellow

ministersAre like invulnerable

In these words Ariel reminds the King and his companions of the utter futility of drawing swords against himself and his fellows Ariel drives Alonso Antonio and Sebastian the three men of sin to desperation ndash a state in which men do violence to themselves They draw swords to strike Ariel But Ariel reminds them that he and the other spirits are the ministers of destiny and nothing can wound them The steel of which their swords are made of may cut the wind or water which being divided always closes up again Even supposing that such things may be possible it is quite impossible that their swords will cut one feather in their plume They are incapable of being wounded by any sword of man Hence it is foolish on their part to attempt to strike at Ariel and his fellow-spirits

For which foul deed

(Line 72-75)The powers delaying not forgetting

haveIncensed the seas and shores yea all the

creatures Against your peace

Ariel enters like a harpy and remaining invisible tells Alonso Sebastian and Antonio that he and other harpies are the agents of Destiny appointed to carry out her decrees He tells them that their punishment for the crime against Prospero which has been so long deferred is now to fall upon them He reminds them that they had expelled Prospero from Milan and set him and his innocent child adrift on the sea and that the sea had paid them back for their sin by the shipwreck and by the calamities they have suffered He tells them that the powers above which did not forget this mean treachery but only deferred the punishment have now engaged the seas and the shores and all living beings including him and his comrades against them The very elements and supernatural agency Ariel adds have taken up the avenging of their crime against Prospero

the action of fire It may cut the wind or water which being divided always closes up again

The sword will not work against the spirits and the harpy because they are the ministers of destiny and nothing can wound them nor it will cut a single feather in their plume

(c )What is the meaning of lsquodowlrsquo in the last line

The term lsquodowlrsquo means a filament or the smallest part of a feather In this context Ariel in disguise of harpy says that their sword cannot even damage the smallest filament of their (Arielrsquos and other spirits) feathers as they are incapable of being wounded by any sword of man

(d) What does the speaker remind the listeners about

Ariel in disguise of harpy reminds Alonso the King of Naples Sebastian Alonsorsquos brother and Antonio the present Duke of Milan and the treacherous brother of Prospero as they being three men of sin He even reminds them that their punishment for their crime against Prospero which has been so long deferred now falls upon them He reminds them that they have expelled Prospero from Milan and has set him along with his innocent infant daughter adrift on the sea So the sea has paid them back for their sin by their shipwreck and the calamities they have suffered since then The harpy rebukes Alonso of his sin that has incensed the Gods and has deprived him of his son as a punishment

(e) How do they respond

When Ariel in disguise of a harpy reminds Alonso Sebastian and Antonio of their past misdeeds and sin Alonso has a look of terror and confusion in his eyes He utters the words of sincere repentance wrung out of his conscience-stricken heart It appears to him that all the elements of nature the sea-waves the wind and the thunder proclaiming a loud voice in the name of Prospero and the crime Alonso has committed against him They are calling upon him to repent There is a deep storm raging in Alonsorsquos breast and the echoes of that storm are ringing in his ears like a clear note of wind-instrument A note of denunciation of Alonsorsquos crime leaves him much humbled and penitent and confirms his belief that his son is lost forever But Sebastian and Antonio shows some courage instead of repentance They wish to kill the spirits or devils if it appears

3

Of my instruction hast thou nothing bated (Line 85-93)

In what thou hast to say So with good life

And observation strange my meaner ministers

Their several kinds have done My high charms work

And these mine enemies are all knit upIn their distractions They now are in my

powerAnd in these fits I leave them while I visitYoung Ferdinand whom they suppose is

drownedAnd his and mine loved darling

Methought the billows spoke and (Line 96-99)

told me of itThe winds did sing it to me and the

thunderThat deep and dreadful organ-pipe

pronouncedThe name of Prosper It did bass my

trespass

These are the words of contrition coming from Alonso Ariel has driven him to a deep repentance for conspiring with Antonio against Prospero He now feels a sincere remorse It appears to him that all the elements of nature the sea-waves the wind and the thunder proclaimed with a loud voice the name of Prospero and the crime Alonso had committed against him They are calling upon him to repent There is a deep storm raging in Alonsorsquos breast and the echoes of that storm are ringing in his ears like the clear note of a wind-instrument

Comment These are the words of sincere repentance wrung out of the conscience-stricken heart of Alonso Alonso who is the lesser villain is the first to give way to remorse under the effect of Arielrsquos speech The words of Ariel seem to him to be the voice of conscience speaking to him He is driven to desperation a state in which he might do violence to his life

(a) Identify the speaker State the context

Prospero the ruler of the island is the speaker The famous banquet scene has been enacted very well Ariel and his junior spirits have played their roles excellently Prospero is glad to say words of praise for them(b) In what way the speakerrsquos instructions have been carried out

According to Prosperorsquos instructions a banquet was presented before the King of Naples and his companions when they were tired and hungry Just when they were preparing to eat the feast the banquet was suddenly removed by exercising supernatural powers All this was done by Ariel Prosperorsquos chief assistant and a powerful spirit

Ariel not only made the feast disappear but also delivered his speech blaming the King and his two companions for their past wicked deeds He warned them to repent for their misdeeds or suffer forever on that uninhabited island

(c) Who are referred to as lsquomeaner ministersrsquo What have they done

Prospero refers as lsquomeaner ministersrsquo to his other lesser spirits who were assisting Ariel in presenting a scene before the kingrsquos party They entered the scene to the accompaniment of music They assumed several strange shapes and brought in a banquet Then they danced about it with gentle actions of salutations thus inviting the King and others to eat the feast

These spirits play their role again when Ariel in the shape of a harpy quits the scene These shapes enter again and dancing with mocking gestures carry away the table

(d) Who are the speakerrsquos enemies What has happened to them

King of Naples Alonso his brother Sebastian and the present Duke of Milan Antonio (Prosperorsquos own brother) are Prosperorsquos enemies With the turn of events they have all been washed ashore on the island which is ruled by Prospero the great magician Actually this happened after the shipwreck caused by a storm which was raised by Prospero with the purpose of bringing these people to his island Prosperorsquos spirits have already confused and terrified these enemies and they are under Prosperorsquos control He can treat them as he likes

(e) What does he say about Ferdinand Explain what is meant by ldquohellip his and mine darlingrdquo

Prospero knows that Alonsorsquos son prince Ferdinand is alive though his father thinks that the prince has been drowned

Prospero refers to his daughter Miranda who is dear to him She is also very dear to Prince Ferdinand who has fallen in love with her They are waiting to be married soon for which they have received Prosperorsquos consent

4

ALONSO O it is monstrous monstrous (Line 95-102)

Methought the billows spoke and told me of it

The winds did sing it to me and the thunderThat deep and dreadful organ-

pipe pronouncedThe name of Prosper It did bass

my trespassTherefore my son ithrsquo ooze is

bedded andIll seek him deeper than eer

plummet soundedAnd with him there lie mudded

(a) In what way does Alonso express his horror when his conscience is awakened by Arielrsquos words

When Alonsorsquos conscience is awakened by Arielrsquos words he expresses his horror at what he has heard He gets the feeling that the waves of the ocean the wind and the loud thunder have spoken to him and uttered the name of Prospero Because of being reminded of his crime in a very loud and rough voice he comes to realize that he has lost his son for his past misdeeds

(b) What does Alonso imagine about his son What does Alonso want to do in his desperate state

Alonso imagines that his son is lying in the mud at the bottom of the sea He feels desperate that he wants to drown himself in the ocean deeper than the plumb-line has ever gone He wants to lie with his son at the bottom of the sea

(c) How do Sebastian and Antonio want to face the evil spirits

Sebastian says that he is not at all afraid of what the harpy has said and that he is prepared to fight any number of such monsters if they appear before him only one at a time Antonio says that he would support Sebastian in the fight against the fiendsyyy

(d) Why does Gonzalo ask Adrian to follow the three men

Gonzalo tells Adrian that all the three men namely Alonso Sebastian and Antonio are in a wild and reckless mood The thought of the heinous crime of which they are guilty has begun to torment their minds So he asks Adrian to follow those three men without loss of time and prevent them from doing anything which the turmoil in their minds might lead them to do

(e) What opinion do you form of Alonso from the above extract

Alonso who is the lesser villain is the first to give way to remorse under the effect of Arielrsquos speech The words of Ariel seem to him to be the voice of conscience speaking to him He is driven to desperation a state in which he might do violence to his life

Subject =Accounts

Ac-12 15420 topic-pL Appropriation ac

PROFIT AND LOSS APPROPRIATION ACCOUNT

MEANING AND PREPARATIONProfit and Loss Appropriation Account is merely an extension of the Profit and Loss Account of the firm The profit of the firm has to be distributed amongst the partners in their respective profit sharing ratio But before its distribution it needs to be adjusted All Adjustments like partnerrsquos salary partnerrsquos commission interest on capital interest on drawings etc are made in this account These adjustments will reduce the amount of profit for distribution This adjusted profit will be distributed amongst the partners in their profit sharing ratio To prepare it at first the balance of Profit and Loss Account is transferred to this account The journal entries for the preparation of Profit and Loss Appropriation Account are given below

1 for transfer of the balance of Profit and Loss Account to Profit and Loss Appropriation Account

(a) In case of Net Profit

Profit and Loss Ac helliphelliphelliphelliphellipDrTo Profit and Loss Appropriation Ac(Net Profit transferred to Profit and Loss Appropriation Ac)

(b)In case of Net Loss

Profit and Loss Appropriation Achelliphelliphellip DrTo Profit and Loss Ac(Net Loss transferred to Profit and Loss Appropriation Ac)

2 for Interest on Capital

For transferring on Interest on CapitalProfit and Loss Appropriation Achelliphelliphellip DrTo Interest on Capital Ac(Interest on capital transferred to Profit amp Loss Appropriation Ac)

3 for Interest on Drawings

For transferring Interest on Drawings Interest on Drawings Achelliphelliphelliphelliphelliphellip DrTo Profit and Loss Appropriation Ac(Interest on drawing transferred to Profit amp Loss Appropriation Ac)

4 For Partnerrsquos SalaryFor transfer of partnerrsquos SalaryProfit and Loss Appropriation Achelliphellip DrTo Salary Ac(Salary transferred to profit amp Loss Appropriation Ac)

5 For Partnerrsquos CommissionFor transferring commissionProfit and Loss Appropriation Achelliphelliphellip DrTo Commission Ac(Commission transferred to Profit and Loss Appropriation Ac)

6 For Transfer of agreed amount to General ReserveProfit and Loss Appropriation Ac helliphellipDrTo General Reserve Ac(Transfer to General Reserve)

7 for share of Profit or Loss appropriation(a) If ProfitProfit and Loss Appropriation Achelliphellip DrTo Partnerrsquos CapitalCurrent Ac(Profit transferred to capitalcurrent Ac)(b) If LossPartnerrsquos Capital Current Achelliphelliphelliphellip DrTo Profit and Loss Appropriation Ac(Loss transferred to capitalcurrent Ac)

THE FORMAT OF PROFIT AND LOSS APPROPRIATION

Profit and Loss Appropriation Account for the year endedhelliphelliphelliphellip

Particulars Amount Particulars Amount

To PL Ac (loss) By pL Ac (profit)

To Interest on capital BY Interest on drawings

To partner`s commission by Partner`s capital Ac ( loss)

To Partner`s salary To Interest on partner`s loan To General Reserve To Partner`s Capital AC (Profit)

Subject= Economics

MOVEMENT ALONG THE DEMAND CURVE (CHANGE IN QUANTITY DEMANDED)In law of demand you have already studied the inverse relationship between price and quantity demanded When quantity demanded of a commodity changes due to change in its price keeping other factors constant it is called change in quantity demanded It is graphically expressed as a movement along the same demand curve There can be either a downward movement or an upward movement along the same demand curve Upward movement along the same demand curve is called contraction of demand or decrease in quantity demanded and downward movement along the same demand curve is known as expansion of demand or increase in quantity demanded

Extention of demandd

price (rs)p A

B Extentionp1 d

Q Q1

Quantity demanded ( in units)

Contraction of demandd

p2 Ccontraction

p APrice (Rs)

d

Q2 Q

Quantity demanded (in units)

Explanation of movement of demand A fall in price from OP to OP1 leads to increase in quantity demanded from OQ to OQ1 (expansion of demand) resulting in a downward movement from point A to point B along the same demand curve DD When Price rises from OP to OP2 quantity demanded falls from OQ to OQ2 (contraction of demand) leading to an upward movement from point A to point C along the same demand curve DD

  • Activity Series of Metals
    • Drawbacks of Rutherfordrsquos model of atom
      • Electromagnetic radiations
      • Properties of electromagnetic radiations
      • Characteristics of electromagnetic radiations
        • Plancks Quantum Theory-
        • Photoelectric effect
          • Intext Questions
            • Logic gates
            • Digital systems are said to be constructed by using logic gates These gates are the AND OR NOT NAND NOR EXOR and EXNOR gates The basic operations are described below with the aid of truth tables
            • AND gate
            • Example
Page 44:  · Web viewSubject. Topic. Summary. Execution. English 1 . Chapter 1 naming words . Page 8. Write the names of these pictures:- Person:-1. father. 2.Firefighter 3.doctor 4 ...

between the constituent particles is special kind of electrostatic attraction That is the attraction of positively charged kernel with sea of delocalized electronsQ6 which group of solid is electrical conductor as well as malleable and ductile [CBSE 2013]A6 Metallic solidQ7 why graphite is good conductor of electricity although it is a network (covalent solid)A7 The exceptional property of graphite is due to its typical structure In graphite each carbon is covalently bonded with 3 atoms in same layer The fourth valence electron of each atom is free to move in between different layersThis free electron makes the graphite a good conductor of electricity

[CBSE 2011]A6 Anisotropy is defined asrdquo Difference in properties when measured along different axis or from different directionsrdquo Crystalline solid show different values of some of the physical properties like electrical resistance refractive index etcwhen measured along the different directions The anisotropy in crystalline solid arises due to the different arrangement of particles in different directions

Math Function Composition of functions Think of an industrial plant that produce bottles of cold drinks first there is the operation (or function) f that puts the cold drink inside the bottle followed by the opeartion g that close the bottle with the capThis leads to the following definitionDefinition Let f A rarr B and g B rarr C be two functions Then the composition of f and g denoted by gof is defined as the function gof A rarr C given by gof(x) = g(f (x)) forall x isinA

Definition A function f X rarr Y is defined to be invertible if there exists a function g Y rarr X such that gof = IX and fog = IY The function g is called the inverse of f and is denoted by f -1

Thus if f is invertible then f must be one-one and onto and conversely if f is one-one and onto then f must be invertible This fact significantly helps for proving a function f to be invertible by showing that f is one-one and onto specially when the actual inverse of f is not to be determined

Example 1 Let f 2 3 4 5 rarr 3 4 5 9 and g 3 4 5 9 rarr 7 11 15 be functions defined as f(2) = 3 f(3) = 4 f(4) = f(5) = 5 and g (3) = g (4) = 7 and g (5) = g (9) = 11 Find gofSolution We have gof(2) = g (f(2)) = g (3) = 7 gof(3) = g (f(3)) = g (4) = 7gof(4) = g (f(4)) = g (5) = 11 and gof(5) = g (5) = 11Example 2 Find gof and fog if f R rarr R and g R rarr R are given by f(x) = cos x and g (x) = 3x2 Show that gof ne fogSolution We have gof(x) = g(f(x))=g(cosx) = 3 (cos x)2

= 3 cos2 x Similarly fog(x)=f(g (x))= f(3x2)= cos (3x2) Note that 3cos2 x ne cos 3x2 for x = 0 Hence gof ne fogExample 3 Show that if f A rarr B and g B rarr C are onto then gof A rarr C is also ontoSolution Given an arbitrary element z isin C there exists a pre-image y of z under g such that g (y) = z since g is onto Further for y isin B there exists an element x in A with f(x) = y since f is onto Therefore gof(x) = g (f(x)) = g (y) = z showing that gof is onto Example 4 Let Y = n2 n isin N sub N Consider f N rarr Y as f(n) = n2 Show that

f is invertible Find the inverse of fSolution An arbitrary element y in Y is of the form n2 for some n isin N This implies that n =radicy This gives a function g Y rarr N defined by g (y) =radicy Nowgof (n) = g (n2)=radicn2 = n and fog (y) =f(radicy) = (radicy) 2 y which shows that gof=IN and fog= IY Hence f is invertible with f -1 = g

Political Science

Constitution of India-The Preamble

Summary

Objective of the state-To secure equality of status and of opportunity To promote fraternity among all the citizens To assure the dignity of the individuals and Unity and integrity of the nation

Justice-Justice stands for rule of law absence of arbitrariness and a system of equal rights freedom and opportunities for all in a society India seeks social economic and political justice to ensure equality to its citizens

Liberty-Liberty implies the absence of restraints or domination on the activities of an individual such as freedom from slavery serfdom imprisonment despotism etc The Preamble provides for the liberty of thought expression belief faith and worship

Equality-Equality means the absence of privileges or discrimination against any section of the society The Preamble provides for equality of status and opportunity to all the people of the country

Fraternity-The Preamble declares that fraternity has to assure two thingsmdashthe dignity of the individual and the unity and

Execution

Answer the following questions-

Short notes-1 Equality2 Fraternity3 Justice4 Liberty

Homework-Learn

integrity of the nation The word integrity has been added to the Preamble by the 42nd Constitutional Amendment (1976)

Business studies

Human resource management (chapter 1)

On the day of 1504 2020 I have discussed with you the managerial functions and procurement functions of HRM

Today weare going to discuss about the development function integration functions and maintenance function

Development functions-HRM improves the knowledge skills attitude and values of employees so that they the present and future jobs more effectively it includes

1) Development functions of HRM

a) Performance appraisal = It implies systematic evaluation of employees with respect to their performance on the job and their potential for development

b) Training =It is the process by which employees learn knowledge skills and attitudes to achieve organisational and personal goals

c) Executive development = It is the process of developing managerial talent through appropriate program

2) Integration functionsa) HRM reconcile the goals of

organisation with those of its members through integrating function

b) HRM tries to motivate employees to various financial and non financial incentives provided in job specification etc

3) Maintenance functiona) HRM promote and protect the

physical and mental health of employees by providing several types of benefits like housing medical aid etc

b) It Promote Social security measures to employees by providing provident fund pension gratuity maternity benefits

SubjectCOMMERCE

Topic

BUSINESSENVIRONMENT

Summary

Now quickly let us revise the earlier points that we have already done in the last class and let us proceed with the other topics that are there in the chapter

Firstly we will recall the internal and external factors of micro environment and then we

Execution 3 What do you mean by internal factors

in micro environmentAnswerInternal factors refer to all the factors existing within a business firm The internal factors are considered controllable because the enterprise has control over these factors

Development FunctionsPerformance AppraisalTrainingExecution Development

shall proceed in details

Meaning and list of internal and external factors

aInternal factorsInternal factors refer to all the factors existing within a business firm The internal factors are considered controllable because the enterprise has control over these factorsFor an example a company can alter its organization structure policies programmes employees physical facilities and marketing mix to suit the changes in the environmentList of internal factors areCorporate culture mission and objectives top management organizations structure company image and brand equity company resources

b External factorsExternal factors refer to those individual and groups and agencies with which a particular business organization comes into direct and frequent contact in the course of its functioningThese individuals and groups are known as STAKEHOLDERS because they have a stake (financial interest ) in the working and performance of the particular business List of external forces (stakeholders)Customers competitors investors suppliersmiddlemen (marketing intermediaries)financers publics

customers

suppliersfinancers

For an example a company can alter its organization structure policies programmes employees physical facilities and marketing mix to suit the changes in the environment

4 What do you mean by external factors in micro environment

AnswerExternal factors refer to those individual and groups and agencies with which a particular business organization comes into direct and frequent contact in the course of its functioningThese individuals and groups are known as STAKEHOLDERS because they have a stake (financial interest) in the working and performance of the particular business

3Who are stakeholdersSTAKEHOLDERS are individuals and groups who have a stake (financial interest ) in the working and performance of the particular business 4Discuss the internal factors in briefa Corporate CultureThe values beliefs and attitudes of the founders and top management of the company exercise a strong influence on what the cmpaany stands for how it does things and what it considers importantbMission and objectivesThe business philosophy and purpose of a comoany guide it prioritiesbusiness strategiesproduct market scope and development scope

cTop management structurethe composition of board of directors the degree of professionalization of management and the organizational structure of a company have important bearing on its business decisions

dPower structureThe internal power relationship between the board of directors and the chief executive is an important factor

e Company image and brand equityThe image and brand equity of the company play a significant role in raising finance forming alliance choosing dealers and suppliers launching new products entering foreign markets

5 What is Macro environmentAnswerMacro environment refers to the general

competitors

middlemen

publics

Fig STAKEHOLDERS OF A COMPANY

Apart from micro environment the other main dimension of business environment isMacro environment Macro environment refers to the general environment or remote environment within which a business firm and forces in its micro environment operateA company does not directly or regularly interact with the micro environmentTherefore macro environment is also known as indirect action EnvironmentThe macro environment forces are less controllable than the micro forces

Macro environment consists of the following components

POLITICAL AND LEGAL ENVIRONMENT

ECONOMIC SOCIAL AND ENVIRONMENT

CULTURAL

ENVIRONMENT

TECHNOLOGICAL ENVIRONMENT

Fig COMPONENTS OF MACRO ENVIRONMENT

environment or remote environment within which a business firm and forces in its micro environment operateA company does not directly or regularly interact with the micro environmentTherefore macro environment is also known as indirect action EnvironmentThe macro environment forces are less controllable than the micro forces 6 What are the components of macro environmenta Political and legal environmentb Economic environmentc Social and cultural environmentd Technological environment

Computer Science

Logic gates

Digital systems are said to be constructed by using logic gates These gates are the AND OR NOT NAND NOR EXOR and EXNOR

BUSINESS FIRM

gates The basic operations are described below with the aid of truth tables

AND gate

The AND gate is an electronic circuit that gives a high output (1) only if all its inputs are high A dot () is used to show the AND operation ie AB Bear in mind that this dot is sometimes omitted ie ABOR gate

The OR gate is an electronic circuit that gives a high output (1) if one or more of its inputs are high A plus (+) is used to show the OR operationNOT gate

The NOT gate is an electronic circuit that produces an inverted version of the input at its output It is also known as an inverter If the input variable is A the inverted output is known as NOT A This is also shown as A or A with a bar over the top as shown at the outputs The diagrams below show two ways that the NAND logic gate can be configured to produce a NOT gate It can also be done using NOR logic gates in the same way

NAND gate

This is a NOT-AND gate which is equal to an AND gate followed by a NOT gate The outputs of all NAND gates are high if any of the inputs are low The symbol is an AND gate with a small circle on the output The small circle represents inversion

NOR gate

This is a NOT-OR gate which is equal to an OR gate followed by a NOT gate The outputs of all NOR gates are low if any of the inputs are highThe symbol is an OR gate with a small circle on the output The small circle represents inversion

EXOR gate

The Exclusive-OR gate is a circuit which will give a high output if either but not both of its two inputs are high An encircled plus sign ( ) is used to show the EOR operation

EXNOR gate

The Exclusive-NOR gate circuit does the opposite to the EOR gate It will give a low output if either but not both of its two inputs are high The symbol is an EXOR gate with a small circle on the output The small circle represents inversion The NAND and NOR gates are called universal functions since with either one the AND and OR functions and NOT can be generated

Note A function in sum of products form can be implemented using NAND gates by replacing all AND and OR gates by NAND gates A function in product of sums form can be implemented using NOR gates by replacing all AND and OR gates by NOR gates

Logic gate symbols

Table 2 is a summary truth table of the inputoutput combinations for the NOT gate together with all possible inputoutput combinations for the other gate functions Also note that a truth table with n inputs has 2n rows You can compare the outputs of different gates

Logic gates representation using the Truth table

Example

A NAND gate can be used as a NOT gate using either of the following wiring configurations

Subject Eng Literature (The Tempest ndash William Shakespeare) Topic Act III Scene 3 Lines 53 to 110 (End of the scene) Date 16th April 2020 (2nd Period)

[Students should read the original play and also the paraphrase given in the school prescribed textbook]Summary Questions amp Answers

o Seeing this strange scene all are inclined to believe the tales told by travelers that there truly are ldquounicornsrdquo and ldquothe phoenixrsquo thronerdquo

o As they are about to sit down to the feast the banquet is snatched away by a harpy (Ariel disguised) A spiritrsquos voice (Arielrsquos voice) denounces Alonso Sebastian and Antonio with particular

1 ARIEL You are three men of sin whom Destiny

(Line 53-58)That hath to instrument this

lower world And what is int the never-surfeited sea

Hath caused to belch up you and on this island

Where man doth not inhabit you rsquomongst men

Being most unfit to live I have made you mad

reference to their crime in expelling Prospero from Milan They have not received any punishment for their deed earlier but the time for their punishment has arrived Upon Alonso it pronounces ldquolingering perdition worse than deathrdquo from which there is no remedy except through sincere repentance Ariel then vanishes in thunder and the shapes enter again and carry away the table

o Prospero watching invisibly is very pleased with the performance of Ariel and his (Prosperorsquos) ldquomeaner ministersrdquo All his enemies are now in his power and are in a fit of desperation He then leaves them and goes to see how Ferdinand and Miranda are getting on

o Alonso is now much humbled and penitent with the after effect of the spiritrsquos denunciation of his crimes He believes that his son is lost forever After this all disperse being stricken mad by the speech of the spirit

o Gonzalo fearing that they may do violence to themselves or to one another follows them and bid others to follow

(a) To whom does Ariel disguised as a harpy call the three sinners What game did Fate of Destiny play with

them

The three sinners called by Ariel are Alonso Sebastian and Antonio It was Destiny which had caused the ocean to cast the three sinners on the shore Though the ocean is all the time devouring whatever appears on its surface and is never satisfied with its continual swallowing of the ships and men in the present case the ocean had cast these three sinners on the shore without killing them

(b) Who had jointly been responsible for the conspiracy against Prospero What is Prosperorsquos purpose behind all this

Three men Alonso Sebastian and Antonio had jointly

been responsible for the conspiracy against Prospero They had driven out Prospero form Milan Prosperorsquos purpose is to make these three sinners realize the wrong they had done He wants them to repent for their criminal deeds because repentance leads to self-esteem(c )What does Ariel (the harpy) tell Alonso and his companions when they take out their swords to attack him

Seeing them drawing their swords Ariel (harpy) tells them that he and his companions are the instruments of destiny and that it is not possible for human beings to do them any injury He says that the swords of human beings can not injure even a minute part of his feathers Their swords are as ineffective against him and his companions as against the wind or the water

(d) Give the explanatory meanings of the following expressions in the context of the above extract

(i)Never surfeited (ii) Belch up (iii) lsquomongst men

(i) Never surfeited never led to satisfaction

(ii) Belch up cast ashore(iii) lsquomongst men in human

society2

I and my fellows (Line 60-65)

Are ministers of Fate The elementsOf whom your swords are tempered may as wellWound the loud winds or with bemocked-at stabsKill the still-closing waters as diminishOne dowl thats in my plume

IMPORTANT PASSAGES EXPLAINED

The elements

(Line 61-66)Of whom your swords are tempered may

as wellWound the loud winds or with

bemocked-at stabs

(a) Who is lsquoIrsquo Who are his lsquofellowsrdquo

lsquoIrsquo is referred to Ariel in disguise of a harpy His lsquofellowsrsquo are other spirits serving Prospero the real Duke of Milan who has acquired supernatural powers after being banished from his Dukedom Prospero has settled in this uninhabited island

(b) What are the elements that have temperrsquod the swords Why will it not work against the speaker

The swords (of Alonso and his companions) are tempered by metal (steel) which is taken out of the earth and refined by

Kill the still-closing waters as diminishOne dowl thats in my plume My fellow

ministersAre like invulnerable

In these words Ariel reminds the King and his companions of the utter futility of drawing swords against himself and his fellows Ariel drives Alonso Antonio and Sebastian the three men of sin to desperation ndash a state in which men do violence to themselves They draw swords to strike Ariel But Ariel reminds them that he and the other spirits are the ministers of destiny and nothing can wound them The steel of which their swords are made of may cut the wind or water which being divided always closes up again Even supposing that such things may be possible it is quite impossible that their swords will cut one feather in their plume They are incapable of being wounded by any sword of man Hence it is foolish on their part to attempt to strike at Ariel and his fellow-spirits

For which foul deed

(Line 72-75)The powers delaying not forgetting

haveIncensed the seas and shores yea all the

creatures Against your peace

Ariel enters like a harpy and remaining invisible tells Alonso Sebastian and Antonio that he and other harpies are the agents of Destiny appointed to carry out her decrees He tells them that their punishment for the crime against Prospero which has been so long deferred is now to fall upon them He reminds them that they had expelled Prospero from Milan and set him and his innocent child adrift on the sea and that the sea had paid them back for their sin by the shipwreck and by the calamities they have suffered He tells them that the powers above which did not forget this mean treachery but only deferred the punishment have now engaged the seas and the shores and all living beings including him and his comrades against them The very elements and supernatural agency Ariel adds have taken up the avenging of their crime against Prospero

the action of fire It may cut the wind or water which being divided always closes up again

The sword will not work against the spirits and the harpy because they are the ministers of destiny and nothing can wound them nor it will cut a single feather in their plume

(c )What is the meaning of lsquodowlrsquo in the last line

The term lsquodowlrsquo means a filament or the smallest part of a feather In this context Ariel in disguise of harpy says that their sword cannot even damage the smallest filament of their (Arielrsquos and other spirits) feathers as they are incapable of being wounded by any sword of man

(d) What does the speaker remind the listeners about

Ariel in disguise of harpy reminds Alonso the King of Naples Sebastian Alonsorsquos brother and Antonio the present Duke of Milan and the treacherous brother of Prospero as they being three men of sin He even reminds them that their punishment for their crime against Prospero which has been so long deferred now falls upon them He reminds them that they have expelled Prospero from Milan and has set him along with his innocent infant daughter adrift on the sea So the sea has paid them back for their sin by their shipwreck and the calamities they have suffered since then The harpy rebukes Alonso of his sin that has incensed the Gods and has deprived him of his son as a punishment

(e) How do they respond

When Ariel in disguise of a harpy reminds Alonso Sebastian and Antonio of their past misdeeds and sin Alonso has a look of terror and confusion in his eyes He utters the words of sincere repentance wrung out of his conscience-stricken heart It appears to him that all the elements of nature the sea-waves the wind and the thunder proclaiming a loud voice in the name of Prospero and the crime Alonso has committed against him They are calling upon him to repent There is a deep storm raging in Alonsorsquos breast and the echoes of that storm are ringing in his ears like a clear note of wind-instrument A note of denunciation of Alonsorsquos crime leaves him much humbled and penitent and confirms his belief that his son is lost forever But Sebastian and Antonio shows some courage instead of repentance They wish to kill the spirits or devils if it appears

3

Of my instruction hast thou nothing bated (Line 85-93)

In what thou hast to say So with good life

And observation strange my meaner ministers

Their several kinds have done My high charms work

And these mine enemies are all knit upIn their distractions They now are in my

powerAnd in these fits I leave them while I visitYoung Ferdinand whom they suppose is

drownedAnd his and mine loved darling

Methought the billows spoke and (Line 96-99)

told me of itThe winds did sing it to me and the

thunderThat deep and dreadful organ-pipe

pronouncedThe name of Prosper It did bass my

trespass

These are the words of contrition coming from Alonso Ariel has driven him to a deep repentance for conspiring with Antonio against Prospero He now feels a sincere remorse It appears to him that all the elements of nature the sea-waves the wind and the thunder proclaimed with a loud voice the name of Prospero and the crime Alonso had committed against him They are calling upon him to repent There is a deep storm raging in Alonsorsquos breast and the echoes of that storm are ringing in his ears like the clear note of a wind-instrument

Comment These are the words of sincere repentance wrung out of the conscience-stricken heart of Alonso Alonso who is the lesser villain is the first to give way to remorse under the effect of Arielrsquos speech The words of Ariel seem to him to be the voice of conscience speaking to him He is driven to desperation a state in which he might do violence to his life

(a) Identify the speaker State the context

Prospero the ruler of the island is the speaker The famous banquet scene has been enacted very well Ariel and his junior spirits have played their roles excellently Prospero is glad to say words of praise for them(b) In what way the speakerrsquos instructions have been carried out

According to Prosperorsquos instructions a banquet was presented before the King of Naples and his companions when they were tired and hungry Just when they were preparing to eat the feast the banquet was suddenly removed by exercising supernatural powers All this was done by Ariel Prosperorsquos chief assistant and a powerful spirit

Ariel not only made the feast disappear but also delivered his speech blaming the King and his two companions for their past wicked deeds He warned them to repent for their misdeeds or suffer forever on that uninhabited island

(c) Who are referred to as lsquomeaner ministersrsquo What have they done

Prospero refers as lsquomeaner ministersrsquo to his other lesser spirits who were assisting Ariel in presenting a scene before the kingrsquos party They entered the scene to the accompaniment of music They assumed several strange shapes and brought in a banquet Then they danced about it with gentle actions of salutations thus inviting the King and others to eat the feast

These spirits play their role again when Ariel in the shape of a harpy quits the scene These shapes enter again and dancing with mocking gestures carry away the table

(d) Who are the speakerrsquos enemies What has happened to them

King of Naples Alonso his brother Sebastian and the present Duke of Milan Antonio (Prosperorsquos own brother) are Prosperorsquos enemies With the turn of events they have all been washed ashore on the island which is ruled by Prospero the great magician Actually this happened after the shipwreck caused by a storm which was raised by Prospero with the purpose of bringing these people to his island Prosperorsquos spirits have already confused and terrified these enemies and they are under Prosperorsquos control He can treat them as he likes

(e) What does he say about Ferdinand Explain what is meant by ldquohellip his and mine darlingrdquo

Prospero knows that Alonsorsquos son prince Ferdinand is alive though his father thinks that the prince has been drowned

Prospero refers to his daughter Miranda who is dear to him She is also very dear to Prince Ferdinand who has fallen in love with her They are waiting to be married soon for which they have received Prosperorsquos consent

4

ALONSO O it is monstrous monstrous (Line 95-102)

Methought the billows spoke and told me of it

The winds did sing it to me and the thunderThat deep and dreadful organ-

pipe pronouncedThe name of Prosper It did bass

my trespassTherefore my son ithrsquo ooze is

bedded andIll seek him deeper than eer

plummet soundedAnd with him there lie mudded

(a) In what way does Alonso express his horror when his conscience is awakened by Arielrsquos words

When Alonsorsquos conscience is awakened by Arielrsquos words he expresses his horror at what he has heard He gets the feeling that the waves of the ocean the wind and the loud thunder have spoken to him and uttered the name of Prospero Because of being reminded of his crime in a very loud and rough voice he comes to realize that he has lost his son for his past misdeeds

(b) What does Alonso imagine about his son What does Alonso want to do in his desperate state

Alonso imagines that his son is lying in the mud at the bottom of the sea He feels desperate that he wants to drown himself in the ocean deeper than the plumb-line has ever gone He wants to lie with his son at the bottom of the sea

(c) How do Sebastian and Antonio want to face the evil spirits

Sebastian says that he is not at all afraid of what the harpy has said and that he is prepared to fight any number of such monsters if they appear before him only one at a time Antonio says that he would support Sebastian in the fight against the fiendsyyy

(d) Why does Gonzalo ask Adrian to follow the three men

Gonzalo tells Adrian that all the three men namely Alonso Sebastian and Antonio are in a wild and reckless mood The thought of the heinous crime of which they are guilty has begun to torment their minds So he asks Adrian to follow those three men without loss of time and prevent them from doing anything which the turmoil in their minds might lead them to do

(e) What opinion do you form of Alonso from the above extract

Alonso who is the lesser villain is the first to give way to remorse under the effect of Arielrsquos speech The words of Ariel seem to him to be the voice of conscience speaking to him He is driven to desperation a state in which he might do violence to his life

Subject =Accounts

Ac-12 15420 topic-pL Appropriation ac

PROFIT AND LOSS APPROPRIATION ACCOUNT

MEANING AND PREPARATIONProfit and Loss Appropriation Account is merely an extension of the Profit and Loss Account of the firm The profit of the firm has to be distributed amongst the partners in their respective profit sharing ratio But before its distribution it needs to be adjusted All Adjustments like partnerrsquos salary partnerrsquos commission interest on capital interest on drawings etc are made in this account These adjustments will reduce the amount of profit for distribution This adjusted profit will be distributed amongst the partners in their profit sharing ratio To prepare it at first the balance of Profit and Loss Account is transferred to this account The journal entries for the preparation of Profit and Loss Appropriation Account are given below

1 for transfer of the balance of Profit and Loss Account to Profit and Loss Appropriation Account

(a) In case of Net Profit

Profit and Loss Ac helliphelliphelliphelliphellipDrTo Profit and Loss Appropriation Ac(Net Profit transferred to Profit and Loss Appropriation Ac)

(b)In case of Net Loss

Profit and Loss Appropriation Achelliphelliphellip DrTo Profit and Loss Ac(Net Loss transferred to Profit and Loss Appropriation Ac)

2 for Interest on Capital

For transferring on Interest on CapitalProfit and Loss Appropriation Achelliphelliphellip DrTo Interest on Capital Ac(Interest on capital transferred to Profit amp Loss Appropriation Ac)

3 for Interest on Drawings

For transferring Interest on Drawings Interest on Drawings Achelliphelliphelliphelliphelliphellip DrTo Profit and Loss Appropriation Ac(Interest on drawing transferred to Profit amp Loss Appropriation Ac)

4 For Partnerrsquos SalaryFor transfer of partnerrsquos SalaryProfit and Loss Appropriation Achelliphellip DrTo Salary Ac(Salary transferred to profit amp Loss Appropriation Ac)

5 For Partnerrsquos CommissionFor transferring commissionProfit and Loss Appropriation Achelliphelliphellip DrTo Commission Ac(Commission transferred to Profit and Loss Appropriation Ac)

6 For Transfer of agreed amount to General ReserveProfit and Loss Appropriation Ac helliphellipDrTo General Reserve Ac(Transfer to General Reserve)

7 for share of Profit or Loss appropriation(a) If ProfitProfit and Loss Appropriation Achelliphellip DrTo Partnerrsquos CapitalCurrent Ac(Profit transferred to capitalcurrent Ac)(b) If LossPartnerrsquos Capital Current Achelliphelliphelliphellip DrTo Profit and Loss Appropriation Ac(Loss transferred to capitalcurrent Ac)

THE FORMAT OF PROFIT AND LOSS APPROPRIATION

Profit and Loss Appropriation Account for the year endedhelliphelliphelliphellip

Particulars Amount Particulars Amount

To PL Ac (loss) By pL Ac (profit)

To Interest on capital BY Interest on drawings

To partner`s commission by Partner`s capital Ac ( loss)

To Partner`s salary To Interest on partner`s loan To General Reserve To Partner`s Capital AC (Profit)

Subject= Economics

MOVEMENT ALONG THE DEMAND CURVE (CHANGE IN QUANTITY DEMANDED)In law of demand you have already studied the inverse relationship between price and quantity demanded When quantity demanded of a commodity changes due to change in its price keeping other factors constant it is called change in quantity demanded It is graphically expressed as a movement along the same demand curve There can be either a downward movement or an upward movement along the same demand curve Upward movement along the same demand curve is called contraction of demand or decrease in quantity demanded and downward movement along the same demand curve is known as expansion of demand or increase in quantity demanded

Extention of demandd

price (rs)p A

B Extentionp1 d

Q Q1

Quantity demanded ( in units)

Contraction of demandd

p2 Ccontraction

p APrice (Rs)

d

Q2 Q

Quantity demanded (in units)

Explanation of movement of demand A fall in price from OP to OP1 leads to increase in quantity demanded from OQ to OQ1 (expansion of demand) resulting in a downward movement from point A to point B along the same demand curve DD When Price rises from OP to OP2 quantity demanded falls from OQ to OQ2 (contraction of demand) leading to an upward movement from point A to point C along the same demand curve DD

  • Activity Series of Metals
    • Drawbacks of Rutherfordrsquos model of atom
      • Electromagnetic radiations
      • Properties of electromagnetic radiations
      • Characteristics of electromagnetic radiations
        • Plancks Quantum Theory-
        • Photoelectric effect
          • Intext Questions
            • Logic gates
            • Digital systems are said to be constructed by using logic gates These gates are the AND OR NOT NAND NOR EXOR and EXNOR gates The basic operations are described below with the aid of truth tables
            • AND gate
            • Example
Page 45:  · Web viewSubject. Topic. Summary. Execution. English 1 . Chapter 1 naming words . Page 8. Write the names of these pictures:- Person:-1. father. 2.Firefighter 3.doctor 4 ...

f is invertible Find the inverse of fSolution An arbitrary element y in Y is of the form n2 for some n isin N This implies that n =radicy This gives a function g Y rarr N defined by g (y) =radicy Nowgof (n) = g (n2)=radicn2 = n and fog (y) =f(radicy) = (radicy) 2 y which shows that gof=IN and fog= IY Hence f is invertible with f -1 = g

Political Science

Constitution of India-The Preamble

Summary

Objective of the state-To secure equality of status and of opportunity To promote fraternity among all the citizens To assure the dignity of the individuals and Unity and integrity of the nation

Justice-Justice stands for rule of law absence of arbitrariness and a system of equal rights freedom and opportunities for all in a society India seeks social economic and political justice to ensure equality to its citizens

Liberty-Liberty implies the absence of restraints or domination on the activities of an individual such as freedom from slavery serfdom imprisonment despotism etc The Preamble provides for the liberty of thought expression belief faith and worship

Equality-Equality means the absence of privileges or discrimination against any section of the society The Preamble provides for equality of status and opportunity to all the people of the country

Fraternity-The Preamble declares that fraternity has to assure two thingsmdashthe dignity of the individual and the unity and

Execution

Answer the following questions-

Short notes-1 Equality2 Fraternity3 Justice4 Liberty

Homework-Learn

integrity of the nation The word integrity has been added to the Preamble by the 42nd Constitutional Amendment (1976)

Business studies

Human resource management (chapter 1)

On the day of 1504 2020 I have discussed with you the managerial functions and procurement functions of HRM

Today weare going to discuss about the development function integration functions and maintenance function

Development functions-HRM improves the knowledge skills attitude and values of employees so that they the present and future jobs more effectively it includes

1) Development functions of HRM

a) Performance appraisal = It implies systematic evaluation of employees with respect to their performance on the job and their potential for development

b) Training =It is the process by which employees learn knowledge skills and attitudes to achieve organisational and personal goals

c) Executive development = It is the process of developing managerial talent through appropriate program

2) Integration functionsa) HRM reconcile the goals of

organisation with those of its members through integrating function

b) HRM tries to motivate employees to various financial and non financial incentives provided in job specification etc

3) Maintenance functiona) HRM promote and protect the

physical and mental health of employees by providing several types of benefits like housing medical aid etc

b) It Promote Social security measures to employees by providing provident fund pension gratuity maternity benefits

SubjectCOMMERCE

Topic

BUSINESSENVIRONMENT

Summary

Now quickly let us revise the earlier points that we have already done in the last class and let us proceed with the other topics that are there in the chapter

Firstly we will recall the internal and external factors of micro environment and then we

Execution 3 What do you mean by internal factors

in micro environmentAnswerInternal factors refer to all the factors existing within a business firm The internal factors are considered controllable because the enterprise has control over these factors

Development FunctionsPerformance AppraisalTrainingExecution Development

shall proceed in details

Meaning and list of internal and external factors

aInternal factorsInternal factors refer to all the factors existing within a business firm The internal factors are considered controllable because the enterprise has control over these factorsFor an example a company can alter its organization structure policies programmes employees physical facilities and marketing mix to suit the changes in the environmentList of internal factors areCorporate culture mission and objectives top management organizations structure company image and brand equity company resources

b External factorsExternal factors refer to those individual and groups and agencies with which a particular business organization comes into direct and frequent contact in the course of its functioningThese individuals and groups are known as STAKEHOLDERS because they have a stake (financial interest ) in the working and performance of the particular business List of external forces (stakeholders)Customers competitors investors suppliersmiddlemen (marketing intermediaries)financers publics

customers

suppliersfinancers

For an example a company can alter its organization structure policies programmes employees physical facilities and marketing mix to suit the changes in the environment

4 What do you mean by external factors in micro environment

AnswerExternal factors refer to those individual and groups and agencies with which a particular business organization comes into direct and frequent contact in the course of its functioningThese individuals and groups are known as STAKEHOLDERS because they have a stake (financial interest) in the working and performance of the particular business

3Who are stakeholdersSTAKEHOLDERS are individuals and groups who have a stake (financial interest ) in the working and performance of the particular business 4Discuss the internal factors in briefa Corporate CultureThe values beliefs and attitudes of the founders and top management of the company exercise a strong influence on what the cmpaany stands for how it does things and what it considers importantbMission and objectivesThe business philosophy and purpose of a comoany guide it prioritiesbusiness strategiesproduct market scope and development scope

cTop management structurethe composition of board of directors the degree of professionalization of management and the organizational structure of a company have important bearing on its business decisions

dPower structureThe internal power relationship between the board of directors and the chief executive is an important factor

e Company image and brand equityThe image and brand equity of the company play a significant role in raising finance forming alliance choosing dealers and suppliers launching new products entering foreign markets

5 What is Macro environmentAnswerMacro environment refers to the general

competitors

middlemen

publics

Fig STAKEHOLDERS OF A COMPANY

Apart from micro environment the other main dimension of business environment isMacro environment Macro environment refers to the general environment or remote environment within which a business firm and forces in its micro environment operateA company does not directly or regularly interact with the micro environmentTherefore macro environment is also known as indirect action EnvironmentThe macro environment forces are less controllable than the micro forces

Macro environment consists of the following components

POLITICAL AND LEGAL ENVIRONMENT

ECONOMIC SOCIAL AND ENVIRONMENT

CULTURAL

ENVIRONMENT

TECHNOLOGICAL ENVIRONMENT

Fig COMPONENTS OF MACRO ENVIRONMENT

environment or remote environment within which a business firm and forces in its micro environment operateA company does not directly or regularly interact with the micro environmentTherefore macro environment is also known as indirect action EnvironmentThe macro environment forces are less controllable than the micro forces 6 What are the components of macro environmenta Political and legal environmentb Economic environmentc Social and cultural environmentd Technological environment

Computer Science

Logic gates

Digital systems are said to be constructed by using logic gates These gates are the AND OR NOT NAND NOR EXOR and EXNOR

BUSINESS FIRM

gates The basic operations are described below with the aid of truth tables

AND gate

The AND gate is an electronic circuit that gives a high output (1) only if all its inputs are high A dot () is used to show the AND operation ie AB Bear in mind that this dot is sometimes omitted ie ABOR gate

The OR gate is an electronic circuit that gives a high output (1) if one or more of its inputs are high A plus (+) is used to show the OR operationNOT gate

The NOT gate is an electronic circuit that produces an inverted version of the input at its output It is also known as an inverter If the input variable is A the inverted output is known as NOT A This is also shown as A or A with a bar over the top as shown at the outputs The diagrams below show two ways that the NAND logic gate can be configured to produce a NOT gate It can also be done using NOR logic gates in the same way

NAND gate

This is a NOT-AND gate which is equal to an AND gate followed by a NOT gate The outputs of all NAND gates are high if any of the inputs are low The symbol is an AND gate with a small circle on the output The small circle represents inversion

NOR gate

This is a NOT-OR gate which is equal to an OR gate followed by a NOT gate The outputs of all NOR gates are low if any of the inputs are highThe symbol is an OR gate with a small circle on the output The small circle represents inversion

EXOR gate

The Exclusive-OR gate is a circuit which will give a high output if either but not both of its two inputs are high An encircled plus sign ( ) is used to show the EOR operation

EXNOR gate

The Exclusive-NOR gate circuit does the opposite to the EOR gate It will give a low output if either but not both of its two inputs are high The symbol is an EXOR gate with a small circle on the output The small circle represents inversion The NAND and NOR gates are called universal functions since with either one the AND and OR functions and NOT can be generated

Note A function in sum of products form can be implemented using NAND gates by replacing all AND and OR gates by NAND gates A function in product of sums form can be implemented using NOR gates by replacing all AND and OR gates by NOR gates

Logic gate symbols

Table 2 is a summary truth table of the inputoutput combinations for the NOT gate together with all possible inputoutput combinations for the other gate functions Also note that a truth table with n inputs has 2n rows You can compare the outputs of different gates

Logic gates representation using the Truth table

Example

A NAND gate can be used as a NOT gate using either of the following wiring configurations

Subject Eng Literature (The Tempest ndash William Shakespeare) Topic Act III Scene 3 Lines 53 to 110 (End of the scene) Date 16th April 2020 (2nd Period)

[Students should read the original play and also the paraphrase given in the school prescribed textbook]Summary Questions amp Answers

o Seeing this strange scene all are inclined to believe the tales told by travelers that there truly are ldquounicornsrdquo and ldquothe phoenixrsquo thronerdquo

o As they are about to sit down to the feast the banquet is snatched away by a harpy (Ariel disguised) A spiritrsquos voice (Arielrsquos voice) denounces Alonso Sebastian and Antonio with particular

1 ARIEL You are three men of sin whom Destiny

(Line 53-58)That hath to instrument this

lower world And what is int the never-surfeited sea

Hath caused to belch up you and on this island

Where man doth not inhabit you rsquomongst men

Being most unfit to live I have made you mad

reference to their crime in expelling Prospero from Milan They have not received any punishment for their deed earlier but the time for their punishment has arrived Upon Alonso it pronounces ldquolingering perdition worse than deathrdquo from which there is no remedy except through sincere repentance Ariel then vanishes in thunder and the shapes enter again and carry away the table

o Prospero watching invisibly is very pleased with the performance of Ariel and his (Prosperorsquos) ldquomeaner ministersrdquo All his enemies are now in his power and are in a fit of desperation He then leaves them and goes to see how Ferdinand and Miranda are getting on

o Alonso is now much humbled and penitent with the after effect of the spiritrsquos denunciation of his crimes He believes that his son is lost forever After this all disperse being stricken mad by the speech of the spirit

o Gonzalo fearing that they may do violence to themselves or to one another follows them and bid others to follow

(a) To whom does Ariel disguised as a harpy call the three sinners What game did Fate of Destiny play with

them

The three sinners called by Ariel are Alonso Sebastian and Antonio It was Destiny which had caused the ocean to cast the three sinners on the shore Though the ocean is all the time devouring whatever appears on its surface and is never satisfied with its continual swallowing of the ships and men in the present case the ocean had cast these three sinners on the shore without killing them

(b) Who had jointly been responsible for the conspiracy against Prospero What is Prosperorsquos purpose behind all this

Three men Alonso Sebastian and Antonio had jointly

been responsible for the conspiracy against Prospero They had driven out Prospero form Milan Prosperorsquos purpose is to make these three sinners realize the wrong they had done He wants them to repent for their criminal deeds because repentance leads to self-esteem(c )What does Ariel (the harpy) tell Alonso and his companions when they take out their swords to attack him

Seeing them drawing their swords Ariel (harpy) tells them that he and his companions are the instruments of destiny and that it is not possible for human beings to do them any injury He says that the swords of human beings can not injure even a minute part of his feathers Their swords are as ineffective against him and his companions as against the wind or the water

(d) Give the explanatory meanings of the following expressions in the context of the above extract

(i)Never surfeited (ii) Belch up (iii) lsquomongst men

(i) Never surfeited never led to satisfaction

(ii) Belch up cast ashore(iii) lsquomongst men in human

society2

I and my fellows (Line 60-65)

Are ministers of Fate The elementsOf whom your swords are tempered may as wellWound the loud winds or with bemocked-at stabsKill the still-closing waters as diminishOne dowl thats in my plume

IMPORTANT PASSAGES EXPLAINED

The elements

(Line 61-66)Of whom your swords are tempered may

as wellWound the loud winds or with

bemocked-at stabs

(a) Who is lsquoIrsquo Who are his lsquofellowsrdquo

lsquoIrsquo is referred to Ariel in disguise of a harpy His lsquofellowsrsquo are other spirits serving Prospero the real Duke of Milan who has acquired supernatural powers after being banished from his Dukedom Prospero has settled in this uninhabited island

(b) What are the elements that have temperrsquod the swords Why will it not work against the speaker

The swords (of Alonso and his companions) are tempered by metal (steel) which is taken out of the earth and refined by

Kill the still-closing waters as diminishOne dowl thats in my plume My fellow

ministersAre like invulnerable

In these words Ariel reminds the King and his companions of the utter futility of drawing swords against himself and his fellows Ariel drives Alonso Antonio and Sebastian the three men of sin to desperation ndash a state in which men do violence to themselves They draw swords to strike Ariel But Ariel reminds them that he and the other spirits are the ministers of destiny and nothing can wound them The steel of which their swords are made of may cut the wind or water which being divided always closes up again Even supposing that such things may be possible it is quite impossible that their swords will cut one feather in their plume They are incapable of being wounded by any sword of man Hence it is foolish on their part to attempt to strike at Ariel and his fellow-spirits

For which foul deed

(Line 72-75)The powers delaying not forgetting

haveIncensed the seas and shores yea all the

creatures Against your peace

Ariel enters like a harpy and remaining invisible tells Alonso Sebastian and Antonio that he and other harpies are the agents of Destiny appointed to carry out her decrees He tells them that their punishment for the crime against Prospero which has been so long deferred is now to fall upon them He reminds them that they had expelled Prospero from Milan and set him and his innocent child adrift on the sea and that the sea had paid them back for their sin by the shipwreck and by the calamities they have suffered He tells them that the powers above which did not forget this mean treachery but only deferred the punishment have now engaged the seas and the shores and all living beings including him and his comrades against them The very elements and supernatural agency Ariel adds have taken up the avenging of their crime against Prospero

the action of fire It may cut the wind or water which being divided always closes up again

The sword will not work against the spirits and the harpy because they are the ministers of destiny and nothing can wound them nor it will cut a single feather in their plume

(c )What is the meaning of lsquodowlrsquo in the last line

The term lsquodowlrsquo means a filament or the smallest part of a feather In this context Ariel in disguise of harpy says that their sword cannot even damage the smallest filament of their (Arielrsquos and other spirits) feathers as they are incapable of being wounded by any sword of man

(d) What does the speaker remind the listeners about

Ariel in disguise of harpy reminds Alonso the King of Naples Sebastian Alonsorsquos brother and Antonio the present Duke of Milan and the treacherous brother of Prospero as they being three men of sin He even reminds them that their punishment for their crime against Prospero which has been so long deferred now falls upon them He reminds them that they have expelled Prospero from Milan and has set him along with his innocent infant daughter adrift on the sea So the sea has paid them back for their sin by their shipwreck and the calamities they have suffered since then The harpy rebukes Alonso of his sin that has incensed the Gods and has deprived him of his son as a punishment

(e) How do they respond

When Ariel in disguise of a harpy reminds Alonso Sebastian and Antonio of their past misdeeds and sin Alonso has a look of terror and confusion in his eyes He utters the words of sincere repentance wrung out of his conscience-stricken heart It appears to him that all the elements of nature the sea-waves the wind and the thunder proclaiming a loud voice in the name of Prospero and the crime Alonso has committed against him They are calling upon him to repent There is a deep storm raging in Alonsorsquos breast and the echoes of that storm are ringing in his ears like a clear note of wind-instrument A note of denunciation of Alonsorsquos crime leaves him much humbled and penitent and confirms his belief that his son is lost forever But Sebastian and Antonio shows some courage instead of repentance They wish to kill the spirits or devils if it appears

3

Of my instruction hast thou nothing bated (Line 85-93)

In what thou hast to say So with good life

And observation strange my meaner ministers

Their several kinds have done My high charms work

And these mine enemies are all knit upIn their distractions They now are in my

powerAnd in these fits I leave them while I visitYoung Ferdinand whom they suppose is

drownedAnd his and mine loved darling

Methought the billows spoke and (Line 96-99)

told me of itThe winds did sing it to me and the

thunderThat deep and dreadful organ-pipe

pronouncedThe name of Prosper It did bass my

trespass

These are the words of contrition coming from Alonso Ariel has driven him to a deep repentance for conspiring with Antonio against Prospero He now feels a sincere remorse It appears to him that all the elements of nature the sea-waves the wind and the thunder proclaimed with a loud voice the name of Prospero and the crime Alonso had committed against him They are calling upon him to repent There is a deep storm raging in Alonsorsquos breast and the echoes of that storm are ringing in his ears like the clear note of a wind-instrument

Comment These are the words of sincere repentance wrung out of the conscience-stricken heart of Alonso Alonso who is the lesser villain is the first to give way to remorse under the effect of Arielrsquos speech The words of Ariel seem to him to be the voice of conscience speaking to him He is driven to desperation a state in which he might do violence to his life

(a) Identify the speaker State the context

Prospero the ruler of the island is the speaker The famous banquet scene has been enacted very well Ariel and his junior spirits have played their roles excellently Prospero is glad to say words of praise for them(b) In what way the speakerrsquos instructions have been carried out

According to Prosperorsquos instructions a banquet was presented before the King of Naples and his companions when they were tired and hungry Just when they were preparing to eat the feast the banquet was suddenly removed by exercising supernatural powers All this was done by Ariel Prosperorsquos chief assistant and a powerful spirit

Ariel not only made the feast disappear but also delivered his speech blaming the King and his two companions for their past wicked deeds He warned them to repent for their misdeeds or suffer forever on that uninhabited island

(c) Who are referred to as lsquomeaner ministersrsquo What have they done

Prospero refers as lsquomeaner ministersrsquo to his other lesser spirits who were assisting Ariel in presenting a scene before the kingrsquos party They entered the scene to the accompaniment of music They assumed several strange shapes and brought in a banquet Then they danced about it with gentle actions of salutations thus inviting the King and others to eat the feast

These spirits play their role again when Ariel in the shape of a harpy quits the scene These shapes enter again and dancing with mocking gestures carry away the table

(d) Who are the speakerrsquos enemies What has happened to them

King of Naples Alonso his brother Sebastian and the present Duke of Milan Antonio (Prosperorsquos own brother) are Prosperorsquos enemies With the turn of events they have all been washed ashore on the island which is ruled by Prospero the great magician Actually this happened after the shipwreck caused by a storm which was raised by Prospero with the purpose of bringing these people to his island Prosperorsquos spirits have already confused and terrified these enemies and they are under Prosperorsquos control He can treat them as he likes

(e) What does he say about Ferdinand Explain what is meant by ldquohellip his and mine darlingrdquo

Prospero knows that Alonsorsquos son prince Ferdinand is alive though his father thinks that the prince has been drowned

Prospero refers to his daughter Miranda who is dear to him She is also very dear to Prince Ferdinand who has fallen in love with her They are waiting to be married soon for which they have received Prosperorsquos consent

4

ALONSO O it is monstrous monstrous (Line 95-102)

Methought the billows spoke and told me of it

The winds did sing it to me and the thunderThat deep and dreadful organ-

pipe pronouncedThe name of Prosper It did bass

my trespassTherefore my son ithrsquo ooze is

bedded andIll seek him deeper than eer

plummet soundedAnd with him there lie mudded

(a) In what way does Alonso express his horror when his conscience is awakened by Arielrsquos words

When Alonsorsquos conscience is awakened by Arielrsquos words he expresses his horror at what he has heard He gets the feeling that the waves of the ocean the wind and the loud thunder have spoken to him and uttered the name of Prospero Because of being reminded of his crime in a very loud and rough voice he comes to realize that he has lost his son for his past misdeeds

(b) What does Alonso imagine about his son What does Alonso want to do in his desperate state

Alonso imagines that his son is lying in the mud at the bottom of the sea He feels desperate that he wants to drown himself in the ocean deeper than the plumb-line has ever gone He wants to lie with his son at the bottom of the sea

(c) How do Sebastian and Antonio want to face the evil spirits

Sebastian says that he is not at all afraid of what the harpy has said and that he is prepared to fight any number of such monsters if they appear before him only one at a time Antonio says that he would support Sebastian in the fight against the fiendsyyy

(d) Why does Gonzalo ask Adrian to follow the three men

Gonzalo tells Adrian that all the three men namely Alonso Sebastian and Antonio are in a wild and reckless mood The thought of the heinous crime of which they are guilty has begun to torment their minds So he asks Adrian to follow those three men without loss of time and prevent them from doing anything which the turmoil in their minds might lead them to do

(e) What opinion do you form of Alonso from the above extract

Alonso who is the lesser villain is the first to give way to remorse under the effect of Arielrsquos speech The words of Ariel seem to him to be the voice of conscience speaking to him He is driven to desperation a state in which he might do violence to his life

Subject =Accounts

Ac-12 15420 topic-pL Appropriation ac

PROFIT AND LOSS APPROPRIATION ACCOUNT

MEANING AND PREPARATIONProfit and Loss Appropriation Account is merely an extension of the Profit and Loss Account of the firm The profit of the firm has to be distributed amongst the partners in their respective profit sharing ratio But before its distribution it needs to be adjusted All Adjustments like partnerrsquos salary partnerrsquos commission interest on capital interest on drawings etc are made in this account These adjustments will reduce the amount of profit for distribution This adjusted profit will be distributed amongst the partners in their profit sharing ratio To prepare it at first the balance of Profit and Loss Account is transferred to this account The journal entries for the preparation of Profit and Loss Appropriation Account are given below

1 for transfer of the balance of Profit and Loss Account to Profit and Loss Appropriation Account

(a) In case of Net Profit

Profit and Loss Ac helliphelliphelliphelliphellipDrTo Profit and Loss Appropriation Ac(Net Profit transferred to Profit and Loss Appropriation Ac)

(b)In case of Net Loss

Profit and Loss Appropriation Achelliphelliphellip DrTo Profit and Loss Ac(Net Loss transferred to Profit and Loss Appropriation Ac)

2 for Interest on Capital

For transferring on Interest on CapitalProfit and Loss Appropriation Achelliphelliphellip DrTo Interest on Capital Ac(Interest on capital transferred to Profit amp Loss Appropriation Ac)

3 for Interest on Drawings

For transferring Interest on Drawings Interest on Drawings Achelliphelliphelliphelliphelliphellip DrTo Profit and Loss Appropriation Ac(Interest on drawing transferred to Profit amp Loss Appropriation Ac)

4 For Partnerrsquos SalaryFor transfer of partnerrsquos SalaryProfit and Loss Appropriation Achelliphellip DrTo Salary Ac(Salary transferred to profit amp Loss Appropriation Ac)

5 For Partnerrsquos CommissionFor transferring commissionProfit and Loss Appropriation Achelliphelliphellip DrTo Commission Ac(Commission transferred to Profit and Loss Appropriation Ac)

6 For Transfer of agreed amount to General ReserveProfit and Loss Appropriation Ac helliphellipDrTo General Reserve Ac(Transfer to General Reserve)

7 for share of Profit or Loss appropriation(a) If ProfitProfit and Loss Appropriation Achelliphellip DrTo Partnerrsquos CapitalCurrent Ac(Profit transferred to capitalcurrent Ac)(b) If LossPartnerrsquos Capital Current Achelliphelliphelliphellip DrTo Profit and Loss Appropriation Ac(Loss transferred to capitalcurrent Ac)

THE FORMAT OF PROFIT AND LOSS APPROPRIATION

Profit and Loss Appropriation Account for the year endedhelliphelliphelliphellip

Particulars Amount Particulars Amount

To PL Ac (loss) By pL Ac (profit)

To Interest on capital BY Interest on drawings

To partner`s commission by Partner`s capital Ac ( loss)

To Partner`s salary To Interest on partner`s loan To General Reserve To Partner`s Capital AC (Profit)

Subject= Economics

MOVEMENT ALONG THE DEMAND CURVE (CHANGE IN QUANTITY DEMANDED)In law of demand you have already studied the inverse relationship between price and quantity demanded When quantity demanded of a commodity changes due to change in its price keeping other factors constant it is called change in quantity demanded It is graphically expressed as a movement along the same demand curve There can be either a downward movement or an upward movement along the same demand curve Upward movement along the same demand curve is called contraction of demand or decrease in quantity demanded and downward movement along the same demand curve is known as expansion of demand or increase in quantity demanded

Extention of demandd

price (rs)p A

B Extentionp1 d

Q Q1

Quantity demanded ( in units)

Contraction of demandd

p2 Ccontraction

p APrice (Rs)

d

Q2 Q

Quantity demanded (in units)

Explanation of movement of demand A fall in price from OP to OP1 leads to increase in quantity demanded from OQ to OQ1 (expansion of demand) resulting in a downward movement from point A to point B along the same demand curve DD When Price rises from OP to OP2 quantity demanded falls from OQ to OQ2 (contraction of demand) leading to an upward movement from point A to point C along the same demand curve DD

  • Activity Series of Metals
    • Drawbacks of Rutherfordrsquos model of atom
      • Electromagnetic radiations
      • Properties of electromagnetic radiations
      • Characteristics of electromagnetic radiations
        • Plancks Quantum Theory-
        • Photoelectric effect
          • Intext Questions
            • Logic gates
            • Digital systems are said to be constructed by using logic gates These gates are the AND OR NOT NAND NOR EXOR and EXNOR gates The basic operations are described below with the aid of truth tables
            • AND gate
            • Example
Page 46:  · Web viewSubject. Topic. Summary. Execution. English 1 . Chapter 1 naming words . Page 8. Write the names of these pictures:- Person:-1. father. 2.Firefighter 3.doctor 4 ...

integrity of the nation The word integrity has been added to the Preamble by the 42nd Constitutional Amendment (1976)

Business studies

Human resource management (chapter 1)

On the day of 1504 2020 I have discussed with you the managerial functions and procurement functions of HRM

Today weare going to discuss about the development function integration functions and maintenance function

Development functions-HRM improves the knowledge skills attitude and values of employees so that they the present and future jobs more effectively it includes

1) Development functions of HRM

a) Performance appraisal = It implies systematic evaluation of employees with respect to their performance on the job and their potential for development

b) Training =It is the process by which employees learn knowledge skills and attitudes to achieve organisational and personal goals

c) Executive development = It is the process of developing managerial talent through appropriate program

2) Integration functionsa) HRM reconcile the goals of

organisation with those of its members through integrating function

b) HRM tries to motivate employees to various financial and non financial incentives provided in job specification etc

3) Maintenance functiona) HRM promote and protect the

physical and mental health of employees by providing several types of benefits like housing medical aid etc

b) It Promote Social security measures to employees by providing provident fund pension gratuity maternity benefits

SubjectCOMMERCE

Topic

BUSINESSENVIRONMENT

Summary

Now quickly let us revise the earlier points that we have already done in the last class and let us proceed with the other topics that are there in the chapter

Firstly we will recall the internal and external factors of micro environment and then we

Execution 3 What do you mean by internal factors

in micro environmentAnswerInternal factors refer to all the factors existing within a business firm The internal factors are considered controllable because the enterprise has control over these factors

Development FunctionsPerformance AppraisalTrainingExecution Development

shall proceed in details

Meaning and list of internal and external factors

aInternal factorsInternal factors refer to all the factors existing within a business firm The internal factors are considered controllable because the enterprise has control over these factorsFor an example a company can alter its organization structure policies programmes employees physical facilities and marketing mix to suit the changes in the environmentList of internal factors areCorporate culture mission and objectives top management organizations structure company image and brand equity company resources

b External factorsExternal factors refer to those individual and groups and agencies with which a particular business organization comes into direct and frequent contact in the course of its functioningThese individuals and groups are known as STAKEHOLDERS because they have a stake (financial interest ) in the working and performance of the particular business List of external forces (stakeholders)Customers competitors investors suppliersmiddlemen (marketing intermediaries)financers publics

customers

suppliersfinancers

For an example a company can alter its organization structure policies programmes employees physical facilities and marketing mix to suit the changes in the environment

4 What do you mean by external factors in micro environment

AnswerExternal factors refer to those individual and groups and agencies with which a particular business organization comes into direct and frequent contact in the course of its functioningThese individuals and groups are known as STAKEHOLDERS because they have a stake (financial interest) in the working and performance of the particular business

3Who are stakeholdersSTAKEHOLDERS are individuals and groups who have a stake (financial interest ) in the working and performance of the particular business 4Discuss the internal factors in briefa Corporate CultureThe values beliefs and attitudes of the founders and top management of the company exercise a strong influence on what the cmpaany stands for how it does things and what it considers importantbMission and objectivesThe business philosophy and purpose of a comoany guide it prioritiesbusiness strategiesproduct market scope and development scope

cTop management structurethe composition of board of directors the degree of professionalization of management and the organizational structure of a company have important bearing on its business decisions

dPower structureThe internal power relationship between the board of directors and the chief executive is an important factor

e Company image and brand equityThe image and brand equity of the company play a significant role in raising finance forming alliance choosing dealers and suppliers launching new products entering foreign markets

5 What is Macro environmentAnswerMacro environment refers to the general

competitors

middlemen

publics

Fig STAKEHOLDERS OF A COMPANY

Apart from micro environment the other main dimension of business environment isMacro environment Macro environment refers to the general environment or remote environment within which a business firm and forces in its micro environment operateA company does not directly or regularly interact with the micro environmentTherefore macro environment is also known as indirect action EnvironmentThe macro environment forces are less controllable than the micro forces

Macro environment consists of the following components

POLITICAL AND LEGAL ENVIRONMENT

ECONOMIC SOCIAL AND ENVIRONMENT

CULTURAL

ENVIRONMENT

TECHNOLOGICAL ENVIRONMENT

Fig COMPONENTS OF MACRO ENVIRONMENT

environment or remote environment within which a business firm and forces in its micro environment operateA company does not directly or regularly interact with the micro environmentTherefore macro environment is also known as indirect action EnvironmentThe macro environment forces are less controllable than the micro forces 6 What are the components of macro environmenta Political and legal environmentb Economic environmentc Social and cultural environmentd Technological environment

Computer Science

Logic gates

Digital systems are said to be constructed by using logic gates These gates are the AND OR NOT NAND NOR EXOR and EXNOR

BUSINESS FIRM

gates The basic operations are described below with the aid of truth tables

AND gate

The AND gate is an electronic circuit that gives a high output (1) only if all its inputs are high A dot () is used to show the AND operation ie AB Bear in mind that this dot is sometimes omitted ie ABOR gate

The OR gate is an electronic circuit that gives a high output (1) if one or more of its inputs are high A plus (+) is used to show the OR operationNOT gate

The NOT gate is an electronic circuit that produces an inverted version of the input at its output It is also known as an inverter If the input variable is A the inverted output is known as NOT A This is also shown as A or A with a bar over the top as shown at the outputs The diagrams below show two ways that the NAND logic gate can be configured to produce a NOT gate It can also be done using NOR logic gates in the same way

NAND gate

This is a NOT-AND gate which is equal to an AND gate followed by a NOT gate The outputs of all NAND gates are high if any of the inputs are low The symbol is an AND gate with a small circle on the output The small circle represents inversion

NOR gate

This is a NOT-OR gate which is equal to an OR gate followed by a NOT gate The outputs of all NOR gates are low if any of the inputs are highThe symbol is an OR gate with a small circle on the output The small circle represents inversion

EXOR gate

The Exclusive-OR gate is a circuit which will give a high output if either but not both of its two inputs are high An encircled plus sign ( ) is used to show the EOR operation

EXNOR gate

The Exclusive-NOR gate circuit does the opposite to the EOR gate It will give a low output if either but not both of its two inputs are high The symbol is an EXOR gate with a small circle on the output The small circle represents inversion The NAND and NOR gates are called universal functions since with either one the AND and OR functions and NOT can be generated

Note A function in sum of products form can be implemented using NAND gates by replacing all AND and OR gates by NAND gates A function in product of sums form can be implemented using NOR gates by replacing all AND and OR gates by NOR gates

Logic gate symbols

Table 2 is a summary truth table of the inputoutput combinations for the NOT gate together with all possible inputoutput combinations for the other gate functions Also note that a truth table with n inputs has 2n rows You can compare the outputs of different gates

Logic gates representation using the Truth table

Example

A NAND gate can be used as a NOT gate using either of the following wiring configurations

Subject Eng Literature (The Tempest ndash William Shakespeare) Topic Act III Scene 3 Lines 53 to 110 (End of the scene) Date 16th April 2020 (2nd Period)

[Students should read the original play and also the paraphrase given in the school prescribed textbook]Summary Questions amp Answers

o Seeing this strange scene all are inclined to believe the tales told by travelers that there truly are ldquounicornsrdquo and ldquothe phoenixrsquo thronerdquo

o As they are about to sit down to the feast the banquet is snatched away by a harpy (Ariel disguised) A spiritrsquos voice (Arielrsquos voice) denounces Alonso Sebastian and Antonio with particular

1 ARIEL You are three men of sin whom Destiny

(Line 53-58)That hath to instrument this

lower world And what is int the never-surfeited sea

Hath caused to belch up you and on this island

Where man doth not inhabit you rsquomongst men

Being most unfit to live I have made you mad

reference to their crime in expelling Prospero from Milan They have not received any punishment for their deed earlier but the time for their punishment has arrived Upon Alonso it pronounces ldquolingering perdition worse than deathrdquo from which there is no remedy except through sincere repentance Ariel then vanishes in thunder and the shapes enter again and carry away the table

o Prospero watching invisibly is very pleased with the performance of Ariel and his (Prosperorsquos) ldquomeaner ministersrdquo All his enemies are now in his power and are in a fit of desperation He then leaves them and goes to see how Ferdinand and Miranda are getting on

o Alonso is now much humbled and penitent with the after effect of the spiritrsquos denunciation of his crimes He believes that his son is lost forever After this all disperse being stricken mad by the speech of the spirit

o Gonzalo fearing that they may do violence to themselves or to one another follows them and bid others to follow

(a) To whom does Ariel disguised as a harpy call the three sinners What game did Fate of Destiny play with

them

The three sinners called by Ariel are Alonso Sebastian and Antonio It was Destiny which had caused the ocean to cast the three sinners on the shore Though the ocean is all the time devouring whatever appears on its surface and is never satisfied with its continual swallowing of the ships and men in the present case the ocean had cast these three sinners on the shore without killing them

(b) Who had jointly been responsible for the conspiracy against Prospero What is Prosperorsquos purpose behind all this

Three men Alonso Sebastian and Antonio had jointly

been responsible for the conspiracy against Prospero They had driven out Prospero form Milan Prosperorsquos purpose is to make these three sinners realize the wrong they had done He wants them to repent for their criminal deeds because repentance leads to self-esteem(c )What does Ariel (the harpy) tell Alonso and his companions when they take out their swords to attack him

Seeing them drawing their swords Ariel (harpy) tells them that he and his companions are the instruments of destiny and that it is not possible for human beings to do them any injury He says that the swords of human beings can not injure even a minute part of his feathers Their swords are as ineffective against him and his companions as against the wind or the water

(d) Give the explanatory meanings of the following expressions in the context of the above extract

(i)Never surfeited (ii) Belch up (iii) lsquomongst men

(i) Never surfeited never led to satisfaction

(ii) Belch up cast ashore(iii) lsquomongst men in human

society2

I and my fellows (Line 60-65)

Are ministers of Fate The elementsOf whom your swords are tempered may as wellWound the loud winds or with bemocked-at stabsKill the still-closing waters as diminishOne dowl thats in my plume

IMPORTANT PASSAGES EXPLAINED

The elements

(Line 61-66)Of whom your swords are tempered may

as wellWound the loud winds or with

bemocked-at stabs

(a) Who is lsquoIrsquo Who are his lsquofellowsrdquo

lsquoIrsquo is referred to Ariel in disguise of a harpy His lsquofellowsrsquo are other spirits serving Prospero the real Duke of Milan who has acquired supernatural powers after being banished from his Dukedom Prospero has settled in this uninhabited island

(b) What are the elements that have temperrsquod the swords Why will it not work against the speaker

The swords (of Alonso and his companions) are tempered by metal (steel) which is taken out of the earth and refined by

Kill the still-closing waters as diminishOne dowl thats in my plume My fellow

ministersAre like invulnerable

In these words Ariel reminds the King and his companions of the utter futility of drawing swords against himself and his fellows Ariel drives Alonso Antonio and Sebastian the three men of sin to desperation ndash a state in which men do violence to themselves They draw swords to strike Ariel But Ariel reminds them that he and the other spirits are the ministers of destiny and nothing can wound them The steel of which their swords are made of may cut the wind or water which being divided always closes up again Even supposing that such things may be possible it is quite impossible that their swords will cut one feather in their plume They are incapable of being wounded by any sword of man Hence it is foolish on their part to attempt to strike at Ariel and his fellow-spirits

For which foul deed

(Line 72-75)The powers delaying not forgetting

haveIncensed the seas and shores yea all the

creatures Against your peace

Ariel enters like a harpy and remaining invisible tells Alonso Sebastian and Antonio that he and other harpies are the agents of Destiny appointed to carry out her decrees He tells them that their punishment for the crime against Prospero which has been so long deferred is now to fall upon them He reminds them that they had expelled Prospero from Milan and set him and his innocent child adrift on the sea and that the sea had paid them back for their sin by the shipwreck and by the calamities they have suffered He tells them that the powers above which did not forget this mean treachery but only deferred the punishment have now engaged the seas and the shores and all living beings including him and his comrades against them The very elements and supernatural agency Ariel adds have taken up the avenging of their crime against Prospero

the action of fire It may cut the wind or water which being divided always closes up again

The sword will not work against the spirits and the harpy because they are the ministers of destiny and nothing can wound them nor it will cut a single feather in their plume

(c )What is the meaning of lsquodowlrsquo in the last line

The term lsquodowlrsquo means a filament or the smallest part of a feather In this context Ariel in disguise of harpy says that their sword cannot even damage the smallest filament of their (Arielrsquos and other spirits) feathers as they are incapable of being wounded by any sword of man

(d) What does the speaker remind the listeners about

Ariel in disguise of harpy reminds Alonso the King of Naples Sebastian Alonsorsquos brother and Antonio the present Duke of Milan and the treacherous brother of Prospero as they being three men of sin He even reminds them that their punishment for their crime against Prospero which has been so long deferred now falls upon them He reminds them that they have expelled Prospero from Milan and has set him along with his innocent infant daughter adrift on the sea So the sea has paid them back for their sin by their shipwreck and the calamities they have suffered since then The harpy rebukes Alonso of his sin that has incensed the Gods and has deprived him of his son as a punishment

(e) How do they respond

When Ariel in disguise of a harpy reminds Alonso Sebastian and Antonio of their past misdeeds and sin Alonso has a look of terror and confusion in his eyes He utters the words of sincere repentance wrung out of his conscience-stricken heart It appears to him that all the elements of nature the sea-waves the wind and the thunder proclaiming a loud voice in the name of Prospero and the crime Alonso has committed against him They are calling upon him to repent There is a deep storm raging in Alonsorsquos breast and the echoes of that storm are ringing in his ears like a clear note of wind-instrument A note of denunciation of Alonsorsquos crime leaves him much humbled and penitent and confirms his belief that his son is lost forever But Sebastian and Antonio shows some courage instead of repentance They wish to kill the spirits or devils if it appears

3

Of my instruction hast thou nothing bated (Line 85-93)

In what thou hast to say So with good life

And observation strange my meaner ministers

Their several kinds have done My high charms work

And these mine enemies are all knit upIn their distractions They now are in my

powerAnd in these fits I leave them while I visitYoung Ferdinand whom they suppose is

drownedAnd his and mine loved darling

Methought the billows spoke and (Line 96-99)

told me of itThe winds did sing it to me and the

thunderThat deep and dreadful organ-pipe

pronouncedThe name of Prosper It did bass my

trespass

These are the words of contrition coming from Alonso Ariel has driven him to a deep repentance for conspiring with Antonio against Prospero He now feels a sincere remorse It appears to him that all the elements of nature the sea-waves the wind and the thunder proclaimed with a loud voice the name of Prospero and the crime Alonso had committed against him They are calling upon him to repent There is a deep storm raging in Alonsorsquos breast and the echoes of that storm are ringing in his ears like the clear note of a wind-instrument

Comment These are the words of sincere repentance wrung out of the conscience-stricken heart of Alonso Alonso who is the lesser villain is the first to give way to remorse under the effect of Arielrsquos speech The words of Ariel seem to him to be the voice of conscience speaking to him He is driven to desperation a state in which he might do violence to his life

(a) Identify the speaker State the context

Prospero the ruler of the island is the speaker The famous banquet scene has been enacted very well Ariel and his junior spirits have played their roles excellently Prospero is glad to say words of praise for them(b) In what way the speakerrsquos instructions have been carried out

According to Prosperorsquos instructions a banquet was presented before the King of Naples and his companions when they were tired and hungry Just when they were preparing to eat the feast the banquet was suddenly removed by exercising supernatural powers All this was done by Ariel Prosperorsquos chief assistant and a powerful spirit

Ariel not only made the feast disappear but also delivered his speech blaming the King and his two companions for their past wicked deeds He warned them to repent for their misdeeds or suffer forever on that uninhabited island

(c) Who are referred to as lsquomeaner ministersrsquo What have they done

Prospero refers as lsquomeaner ministersrsquo to his other lesser spirits who were assisting Ariel in presenting a scene before the kingrsquos party They entered the scene to the accompaniment of music They assumed several strange shapes and brought in a banquet Then they danced about it with gentle actions of salutations thus inviting the King and others to eat the feast

These spirits play their role again when Ariel in the shape of a harpy quits the scene These shapes enter again and dancing with mocking gestures carry away the table

(d) Who are the speakerrsquos enemies What has happened to them

King of Naples Alonso his brother Sebastian and the present Duke of Milan Antonio (Prosperorsquos own brother) are Prosperorsquos enemies With the turn of events they have all been washed ashore on the island which is ruled by Prospero the great magician Actually this happened after the shipwreck caused by a storm which was raised by Prospero with the purpose of bringing these people to his island Prosperorsquos spirits have already confused and terrified these enemies and they are under Prosperorsquos control He can treat them as he likes

(e) What does he say about Ferdinand Explain what is meant by ldquohellip his and mine darlingrdquo

Prospero knows that Alonsorsquos son prince Ferdinand is alive though his father thinks that the prince has been drowned

Prospero refers to his daughter Miranda who is dear to him She is also very dear to Prince Ferdinand who has fallen in love with her They are waiting to be married soon for which they have received Prosperorsquos consent

4

ALONSO O it is monstrous monstrous (Line 95-102)

Methought the billows spoke and told me of it

The winds did sing it to me and the thunderThat deep and dreadful organ-

pipe pronouncedThe name of Prosper It did bass

my trespassTherefore my son ithrsquo ooze is

bedded andIll seek him deeper than eer

plummet soundedAnd with him there lie mudded

(a) In what way does Alonso express his horror when his conscience is awakened by Arielrsquos words

When Alonsorsquos conscience is awakened by Arielrsquos words he expresses his horror at what he has heard He gets the feeling that the waves of the ocean the wind and the loud thunder have spoken to him and uttered the name of Prospero Because of being reminded of his crime in a very loud and rough voice he comes to realize that he has lost his son for his past misdeeds

(b) What does Alonso imagine about his son What does Alonso want to do in his desperate state

Alonso imagines that his son is lying in the mud at the bottom of the sea He feels desperate that he wants to drown himself in the ocean deeper than the plumb-line has ever gone He wants to lie with his son at the bottom of the sea

(c) How do Sebastian and Antonio want to face the evil spirits

Sebastian says that he is not at all afraid of what the harpy has said and that he is prepared to fight any number of such monsters if they appear before him only one at a time Antonio says that he would support Sebastian in the fight against the fiendsyyy

(d) Why does Gonzalo ask Adrian to follow the three men

Gonzalo tells Adrian that all the three men namely Alonso Sebastian and Antonio are in a wild and reckless mood The thought of the heinous crime of which they are guilty has begun to torment their minds So he asks Adrian to follow those three men without loss of time and prevent them from doing anything which the turmoil in their minds might lead them to do

(e) What opinion do you form of Alonso from the above extract

Alonso who is the lesser villain is the first to give way to remorse under the effect of Arielrsquos speech The words of Ariel seem to him to be the voice of conscience speaking to him He is driven to desperation a state in which he might do violence to his life

Subject =Accounts

Ac-12 15420 topic-pL Appropriation ac

PROFIT AND LOSS APPROPRIATION ACCOUNT

MEANING AND PREPARATIONProfit and Loss Appropriation Account is merely an extension of the Profit and Loss Account of the firm The profit of the firm has to be distributed amongst the partners in their respective profit sharing ratio But before its distribution it needs to be adjusted All Adjustments like partnerrsquos salary partnerrsquos commission interest on capital interest on drawings etc are made in this account These adjustments will reduce the amount of profit for distribution This adjusted profit will be distributed amongst the partners in their profit sharing ratio To prepare it at first the balance of Profit and Loss Account is transferred to this account The journal entries for the preparation of Profit and Loss Appropriation Account are given below

1 for transfer of the balance of Profit and Loss Account to Profit and Loss Appropriation Account

(a) In case of Net Profit

Profit and Loss Ac helliphelliphelliphelliphellipDrTo Profit and Loss Appropriation Ac(Net Profit transferred to Profit and Loss Appropriation Ac)

(b)In case of Net Loss

Profit and Loss Appropriation Achelliphelliphellip DrTo Profit and Loss Ac(Net Loss transferred to Profit and Loss Appropriation Ac)

2 for Interest on Capital

For transferring on Interest on CapitalProfit and Loss Appropriation Achelliphelliphellip DrTo Interest on Capital Ac(Interest on capital transferred to Profit amp Loss Appropriation Ac)

3 for Interest on Drawings

For transferring Interest on Drawings Interest on Drawings Achelliphelliphelliphelliphelliphellip DrTo Profit and Loss Appropriation Ac(Interest on drawing transferred to Profit amp Loss Appropriation Ac)

4 For Partnerrsquos SalaryFor transfer of partnerrsquos SalaryProfit and Loss Appropriation Achelliphellip DrTo Salary Ac(Salary transferred to profit amp Loss Appropriation Ac)

5 For Partnerrsquos CommissionFor transferring commissionProfit and Loss Appropriation Achelliphelliphellip DrTo Commission Ac(Commission transferred to Profit and Loss Appropriation Ac)

6 For Transfer of agreed amount to General ReserveProfit and Loss Appropriation Ac helliphellipDrTo General Reserve Ac(Transfer to General Reserve)

7 for share of Profit or Loss appropriation(a) If ProfitProfit and Loss Appropriation Achelliphellip DrTo Partnerrsquos CapitalCurrent Ac(Profit transferred to capitalcurrent Ac)(b) If LossPartnerrsquos Capital Current Achelliphelliphelliphellip DrTo Profit and Loss Appropriation Ac(Loss transferred to capitalcurrent Ac)

THE FORMAT OF PROFIT AND LOSS APPROPRIATION

Profit and Loss Appropriation Account for the year endedhelliphelliphelliphellip

Particulars Amount Particulars Amount

To PL Ac (loss) By pL Ac (profit)

To Interest on capital BY Interest on drawings

To partner`s commission by Partner`s capital Ac ( loss)

To Partner`s salary To Interest on partner`s loan To General Reserve To Partner`s Capital AC (Profit)

Subject= Economics

MOVEMENT ALONG THE DEMAND CURVE (CHANGE IN QUANTITY DEMANDED)In law of demand you have already studied the inverse relationship between price and quantity demanded When quantity demanded of a commodity changes due to change in its price keeping other factors constant it is called change in quantity demanded It is graphically expressed as a movement along the same demand curve There can be either a downward movement or an upward movement along the same demand curve Upward movement along the same demand curve is called contraction of demand or decrease in quantity demanded and downward movement along the same demand curve is known as expansion of demand or increase in quantity demanded

Extention of demandd

price (rs)p A

B Extentionp1 d

Q Q1

Quantity demanded ( in units)

Contraction of demandd

p2 Ccontraction

p APrice (Rs)

d

Q2 Q

Quantity demanded (in units)

Explanation of movement of demand A fall in price from OP to OP1 leads to increase in quantity demanded from OQ to OQ1 (expansion of demand) resulting in a downward movement from point A to point B along the same demand curve DD When Price rises from OP to OP2 quantity demanded falls from OQ to OQ2 (contraction of demand) leading to an upward movement from point A to point C along the same demand curve DD

  • Activity Series of Metals
    • Drawbacks of Rutherfordrsquos model of atom
      • Electromagnetic radiations
      • Properties of electromagnetic radiations
      • Characteristics of electromagnetic radiations
        • Plancks Quantum Theory-
        • Photoelectric effect
          • Intext Questions
            • Logic gates
            • Digital systems are said to be constructed by using logic gates These gates are the AND OR NOT NAND NOR EXOR and EXNOR gates The basic operations are described below with the aid of truth tables
            • AND gate
            • Example
Page 47:  · Web viewSubject. Topic. Summary. Execution. English 1 . Chapter 1 naming words . Page 8. Write the names of these pictures:- Person:-1. father. 2.Firefighter 3.doctor 4 ...

shall proceed in details

Meaning and list of internal and external factors

aInternal factorsInternal factors refer to all the factors existing within a business firm The internal factors are considered controllable because the enterprise has control over these factorsFor an example a company can alter its organization structure policies programmes employees physical facilities and marketing mix to suit the changes in the environmentList of internal factors areCorporate culture mission and objectives top management organizations structure company image and brand equity company resources

b External factorsExternal factors refer to those individual and groups and agencies with which a particular business organization comes into direct and frequent contact in the course of its functioningThese individuals and groups are known as STAKEHOLDERS because they have a stake (financial interest ) in the working and performance of the particular business List of external forces (stakeholders)Customers competitors investors suppliersmiddlemen (marketing intermediaries)financers publics

customers

suppliersfinancers

For an example a company can alter its organization structure policies programmes employees physical facilities and marketing mix to suit the changes in the environment

4 What do you mean by external factors in micro environment

AnswerExternal factors refer to those individual and groups and agencies with which a particular business organization comes into direct and frequent contact in the course of its functioningThese individuals and groups are known as STAKEHOLDERS because they have a stake (financial interest) in the working and performance of the particular business

3Who are stakeholdersSTAKEHOLDERS are individuals and groups who have a stake (financial interest ) in the working and performance of the particular business 4Discuss the internal factors in briefa Corporate CultureThe values beliefs and attitudes of the founders and top management of the company exercise a strong influence on what the cmpaany stands for how it does things and what it considers importantbMission and objectivesThe business philosophy and purpose of a comoany guide it prioritiesbusiness strategiesproduct market scope and development scope

cTop management structurethe composition of board of directors the degree of professionalization of management and the organizational structure of a company have important bearing on its business decisions

dPower structureThe internal power relationship between the board of directors and the chief executive is an important factor

e Company image and brand equityThe image and brand equity of the company play a significant role in raising finance forming alliance choosing dealers and suppliers launching new products entering foreign markets

5 What is Macro environmentAnswerMacro environment refers to the general

competitors

middlemen

publics

Fig STAKEHOLDERS OF A COMPANY

Apart from micro environment the other main dimension of business environment isMacro environment Macro environment refers to the general environment or remote environment within which a business firm and forces in its micro environment operateA company does not directly or regularly interact with the micro environmentTherefore macro environment is also known as indirect action EnvironmentThe macro environment forces are less controllable than the micro forces

Macro environment consists of the following components

POLITICAL AND LEGAL ENVIRONMENT

ECONOMIC SOCIAL AND ENVIRONMENT

CULTURAL

ENVIRONMENT

TECHNOLOGICAL ENVIRONMENT

Fig COMPONENTS OF MACRO ENVIRONMENT

environment or remote environment within which a business firm and forces in its micro environment operateA company does not directly or regularly interact with the micro environmentTherefore macro environment is also known as indirect action EnvironmentThe macro environment forces are less controllable than the micro forces 6 What are the components of macro environmenta Political and legal environmentb Economic environmentc Social and cultural environmentd Technological environment

Computer Science

Logic gates

Digital systems are said to be constructed by using logic gates These gates are the AND OR NOT NAND NOR EXOR and EXNOR

BUSINESS FIRM

gates The basic operations are described below with the aid of truth tables

AND gate

The AND gate is an electronic circuit that gives a high output (1) only if all its inputs are high A dot () is used to show the AND operation ie AB Bear in mind that this dot is sometimes omitted ie ABOR gate

The OR gate is an electronic circuit that gives a high output (1) if one or more of its inputs are high A plus (+) is used to show the OR operationNOT gate

The NOT gate is an electronic circuit that produces an inverted version of the input at its output It is also known as an inverter If the input variable is A the inverted output is known as NOT A This is also shown as A or A with a bar over the top as shown at the outputs The diagrams below show two ways that the NAND logic gate can be configured to produce a NOT gate It can also be done using NOR logic gates in the same way

NAND gate

This is a NOT-AND gate which is equal to an AND gate followed by a NOT gate The outputs of all NAND gates are high if any of the inputs are low The symbol is an AND gate with a small circle on the output The small circle represents inversion

NOR gate

This is a NOT-OR gate which is equal to an OR gate followed by a NOT gate The outputs of all NOR gates are low if any of the inputs are highThe symbol is an OR gate with a small circle on the output The small circle represents inversion

EXOR gate

The Exclusive-OR gate is a circuit which will give a high output if either but not both of its two inputs are high An encircled plus sign ( ) is used to show the EOR operation

EXNOR gate

The Exclusive-NOR gate circuit does the opposite to the EOR gate It will give a low output if either but not both of its two inputs are high The symbol is an EXOR gate with a small circle on the output The small circle represents inversion The NAND and NOR gates are called universal functions since with either one the AND and OR functions and NOT can be generated

Note A function in sum of products form can be implemented using NAND gates by replacing all AND and OR gates by NAND gates A function in product of sums form can be implemented using NOR gates by replacing all AND and OR gates by NOR gates

Logic gate symbols

Table 2 is a summary truth table of the inputoutput combinations for the NOT gate together with all possible inputoutput combinations for the other gate functions Also note that a truth table with n inputs has 2n rows You can compare the outputs of different gates

Logic gates representation using the Truth table

Example

A NAND gate can be used as a NOT gate using either of the following wiring configurations

Subject Eng Literature (The Tempest ndash William Shakespeare) Topic Act III Scene 3 Lines 53 to 110 (End of the scene) Date 16th April 2020 (2nd Period)

[Students should read the original play and also the paraphrase given in the school prescribed textbook]Summary Questions amp Answers

o Seeing this strange scene all are inclined to believe the tales told by travelers that there truly are ldquounicornsrdquo and ldquothe phoenixrsquo thronerdquo

o As they are about to sit down to the feast the banquet is snatched away by a harpy (Ariel disguised) A spiritrsquos voice (Arielrsquos voice) denounces Alonso Sebastian and Antonio with particular

1 ARIEL You are three men of sin whom Destiny

(Line 53-58)That hath to instrument this

lower world And what is int the never-surfeited sea

Hath caused to belch up you and on this island

Where man doth not inhabit you rsquomongst men

Being most unfit to live I have made you mad

reference to their crime in expelling Prospero from Milan They have not received any punishment for their deed earlier but the time for their punishment has arrived Upon Alonso it pronounces ldquolingering perdition worse than deathrdquo from which there is no remedy except through sincere repentance Ariel then vanishes in thunder and the shapes enter again and carry away the table

o Prospero watching invisibly is very pleased with the performance of Ariel and his (Prosperorsquos) ldquomeaner ministersrdquo All his enemies are now in his power and are in a fit of desperation He then leaves them and goes to see how Ferdinand and Miranda are getting on

o Alonso is now much humbled and penitent with the after effect of the spiritrsquos denunciation of his crimes He believes that his son is lost forever After this all disperse being stricken mad by the speech of the spirit

o Gonzalo fearing that they may do violence to themselves or to one another follows them and bid others to follow

(a) To whom does Ariel disguised as a harpy call the three sinners What game did Fate of Destiny play with

them

The three sinners called by Ariel are Alonso Sebastian and Antonio It was Destiny which had caused the ocean to cast the three sinners on the shore Though the ocean is all the time devouring whatever appears on its surface and is never satisfied with its continual swallowing of the ships and men in the present case the ocean had cast these three sinners on the shore without killing them

(b) Who had jointly been responsible for the conspiracy against Prospero What is Prosperorsquos purpose behind all this

Three men Alonso Sebastian and Antonio had jointly

been responsible for the conspiracy against Prospero They had driven out Prospero form Milan Prosperorsquos purpose is to make these three sinners realize the wrong they had done He wants them to repent for their criminal deeds because repentance leads to self-esteem(c )What does Ariel (the harpy) tell Alonso and his companions when they take out their swords to attack him

Seeing them drawing their swords Ariel (harpy) tells them that he and his companions are the instruments of destiny and that it is not possible for human beings to do them any injury He says that the swords of human beings can not injure even a minute part of his feathers Their swords are as ineffective against him and his companions as against the wind or the water

(d) Give the explanatory meanings of the following expressions in the context of the above extract

(i)Never surfeited (ii) Belch up (iii) lsquomongst men

(i) Never surfeited never led to satisfaction

(ii) Belch up cast ashore(iii) lsquomongst men in human

society2

I and my fellows (Line 60-65)

Are ministers of Fate The elementsOf whom your swords are tempered may as wellWound the loud winds or with bemocked-at stabsKill the still-closing waters as diminishOne dowl thats in my plume

IMPORTANT PASSAGES EXPLAINED

The elements

(Line 61-66)Of whom your swords are tempered may

as wellWound the loud winds or with

bemocked-at stabs

(a) Who is lsquoIrsquo Who are his lsquofellowsrdquo

lsquoIrsquo is referred to Ariel in disguise of a harpy His lsquofellowsrsquo are other spirits serving Prospero the real Duke of Milan who has acquired supernatural powers after being banished from his Dukedom Prospero has settled in this uninhabited island

(b) What are the elements that have temperrsquod the swords Why will it not work against the speaker

The swords (of Alonso and his companions) are tempered by metal (steel) which is taken out of the earth and refined by

Kill the still-closing waters as diminishOne dowl thats in my plume My fellow

ministersAre like invulnerable

In these words Ariel reminds the King and his companions of the utter futility of drawing swords against himself and his fellows Ariel drives Alonso Antonio and Sebastian the three men of sin to desperation ndash a state in which men do violence to themselves They draw swords to strike Ariel But Ariel reminds them that he and the other spirits are the ministers of destiny and nothing can wound them The steel of which their swords are made of may cut the wind or water which being divided always closes up again Even supposing that such things may be possible it is quite impossible that their swords will cut one feather in their plume They are incapable of being wounded by any sword of man Hence it is foolish on their part to attempt to strike at Ariel and his fellow-spirits

For which foul deed

(Line 72-75)The powers delaying not forgetting

haveIncensed the seas and shores yea all the

creatures Against your peace

Ariel enters like a harpy and remaining invisible tells Alonso Sebastian and Antonio that he and other harpies are the agents of Destiny appointed to carry out her decrees He tells them that their punishment for the crime against Prospero which has been so long deferred is now to fall upon them He reminds them that they had expelled Prospero from Milan and set him and his innocent child adrift on the sea and that the sea had paid them back for their sin by the shipwreck and by the calamities they have suffered He tells them that the powers above which did not forget this mean treachery but only deferred the punishment have now engaged the seas and the shores and all living beings including him and his comrades against them The very elements and supernatural agency Ariel adds have taken up the avenging of their crime against Prospero

the action of fire It may cut the wind or water which being divided always closes up again

The sword will not work against the spirits and the harpy because they are the ministers of destiny and nothing can wound them nor it will cut a single feather in their plume

(c )What is the meaning of lsquodowlrsquo in the last line

The term lsquodowlrsquo means a filament or the smallest part of a feather In this context Ariel in disguise of harpy says that their sword cannot even damage the smallest filament of their (Arielrsquos and other spirits) feathers as they are incapable of being wounded by any sword of man

(d) What does the speaker remind the listeners about

Ariel in disguise of harpy reminds Alonso the King of Naples Sebastian Alonsorsquos brother and Antonio the present Duke of Milan and the treacherous brother of Prospero as they being three men of sin He even reminds them that their punishment for their crime against Prospero which has been so long deferred now falls upon them He reminds them that they have expelled Prospero from Milan and has set him along with his innocent infant daughter adrift on the sea So the sea has paid them back for their sin by their shipwreck and the calamities they have suffered since then The harpy rebukes Alonso of his sin that has incensed the Gods and has deprived him of his son as a punishment

(e) How do they respond

When Ariel in disguise of a harpy reminds Alonso Sebastian and Antonio of their past misdeeds and sin Alonso has a look of terror and confusion in his eyes He utters the words of sincere repentance wrung out of his conscience-stricken heart It appears to him that all the elements of nature the sea-waves the wind and the thunder proclaiming a loud voice in the name of Prospero and the crime Alonso has committed against him They are calling upon him to repent There is a deep storm raging in Alonsorsquos breast and the echoes of that storm are ringing in his ears like a clear note of wind-instrument A note of denunciation of Alonsorsquos crime leaves him much humbled and penitent and confirms his belief that his son is lost forever But Sebastian and Antonio shows some courage instead of repentance They wish to kill the spirits or devils if it appears

3

Of my instruction hast thou nothing bated (Line 85-93)

In what thou hast to say So with good life

And observation strange my meaner ministers

Their several kinds have done My high charms work

And these mine enemies are all knit upIn their distractions They now are in my

powerAnd in these fits I leave them while I visitYoung Ferdinand whom they suppose is

drownedAnd his and mine loved darling

Methought the billows spoke and (Line 96-99)

told me of itThe winds did sing it to me and the

thunderThat deep and dreadful organ-pipe

pronouncedThe name of Prosper It did bass my

trespass

These are the words of contrition coming from Alonso Ariel has driven him to a deep repentance for conspiring with Antonio against Prospero He now feels a sincere remorse It appears to him that all the elements of nature the sea-waves the wind and the thunder proclaimed with a loud voice the name of Prospero and the crime Alonso had committed against him They are calling upon him to repent There is a deep storm raging in Alonsorsquos breast and the echoes of that storm are ringing in his ears like the clear note of a wind-instrument

Comment These are the words of sincere repentance wrung out of the conscience-stricken heart of Alonso Alonso who is the lesser villain is the first to give way to remorse under the effect of Arielrsquos speech The words of Ariel seem to him to be the voice of conscience speaking to him He is driven to desperation a state in which he might do violence to his life

(a) Identify the speaker State the context

Prospero the ruler of the island is the speaker The famous banquet scene has been enacted very well Ariel and his junior spirits have played their roles excellently Prospero is glad to say words of praise for them(b) In what way the speakerrsquos instructions have been carried out

According to Prosperorsquos instructions a banquet was presented before the King of Naples and his companions when they were tired and hungry Just when they were preparing to eat the feast the banquet was suddenly removed by exercising supernatural powers All this was done by Ariel Prosperorsquos chief assistant and a powerful spirit

Ariel not only made the feast disappear but also delivered his speech blaming the King and his two companions for their past wicked deeds He warned them to repent for their misdeeds or suffer forever on that uninhabited island

(c) Who are referred to as lsquomeaner ministersrsquo What have they done

Prospero refers as lsquomeaner ministersrsquo to his other lesser spirits who were assisting Ariel in presenting a scene before the kingrsquos party They entered the scene to the accompaniment of music They assumed several strange shapes and brought in a banquet Then they danced about it with gentle actions of salutations thus inviting the King and others to eat the feast

These spirits play their role again when Ariel in the shape of a harpy quits the scene These shapes enter again and dancing with mocking gestures carry away the table

(d) Who are the speakerrsquos enemies What has happened to them

King of Naples Alonso his brother Sebastian and the present Duke of Milan Antonio (Prosperorsquos own brother) are Prosperorsquos enemies With the turn of events they have all been washed ashore on the island which is ruled by Prospero the great magician Actually this happened after the shipwreck caused by a storm which was raised by Prospero with the purpose of bringing these people to his island Prosperorsquos spirits have already confused and terrified these enemies and they are under Prosperorsquos control He can treat them as he likes

(e) What does he say about Ferdinand Explain what is meant by ldquohellip his and mine darlingrdquo

Prospero knows that Alonsorsquos son prince Ferdinand is alive though his father thinks that the prince has been drowned

Prospero refers to his daughter Miranda who is dear to him She is also very dear to Prince Ferdinand who has fallen in love with her They are waiting to be married soon for which they have received Prosperorsquos consent

4

ALONSO O it is monstrous monstrous (Line 95-102)

Methought the billows spoke and told me of it

The winds did sing it to me and the thunderThat deep and dreadful organ-

pipe pronouncedThe name of Prosper It did bass

my trespassTherefore my son ithrsquo ooze is

bedded andIll seek him deeper than eer

plummet soundedAnd with him there lie mudded

(a) In what way does Alonso express his horror when his conscience is awakened by Arielrsquos words

When Alonsorsquos conscience is awakened by Arielrsquos words he expresses his horror at what he has heard He gets the feeling that the waves of the ocean the wind and the loud thunder have spoken to him and uttered the name of Prospero Because of being reminded of his crime in a very loud and rough voice he comes to realize that he has lost his son for his past misdeeds

(b) What does Alonso imagine about his son What does Alonso want to do in his desperate state

Alonso imagines that his son is lying in the mud at the bottom of the sea He feels desperate that he wants to drown himself in the ocean deeper than the plumb-line has ever gone He wants to lie with his son at the bottom of the sea

(c) How do Sebastian and Antonio want to face the evil spirits

Sebastian says that he is not at all afraid of what the harpy has said and that he is prepared to fight any number of such monsters if they appear before him only one at a time Antonio says that he would support Sebastian in the fight against the fiendsyyy

(d) Why does Gonzalo ask Adrian to follow the three men

Gonzalo tells Adrian that all the three men namely Alonso Sebastian and Antonio are in a wild and reckless mood The thought of the heinous crime of which they are guilty has begun to torment their minds So he asks Adrian to follow those three men without loss of time and prevent them from doing anything which the turmoil in their minds might lead them to do

(e) What opinion do you form of Alonso from the above extract

Alonso who is the lesser villain is the first to give way to remorse under the effect of Arielrsquos speech The words of Ariel seem to him to be the voice of conscience speaking to him He is driven to desperation a state in which he might do violence to his life

Subject =Accounts

Ac-12 15420 topic-pL Appropriation ac

PROFIT AND LOSS APPROPRIATION ACCOUNT

MEANING AND PREPARATIONProfit and Loss Appropriation Account is merely an extension of the Profit and Loss Account of the firm The profit of the firm has to be distributed amongst the partners in their respective profit sharing ratio But before its distribution it needs to be adjusted All Adjustments like partnerrsquos salary partnerrsquos commission interest on capital interest on drawings etc are made in this account These adjustments will reduce the amount of profit for distribution This adjusted profit will be distributed amongst the partners in their profit sharing ratio To prepare it at first the balance of Profit and Loss Account is transferred to this account The journal entries for the preparation of Profit and Loss Appropriation Account are given below

1 for transfer of the balance of Profit and Loss Account to Profit and Loss Appropriation Account

(a) In case of Net Profit

Profit and Loss Ac helliphelliphelliphelliphellipDrTo Profit and Loss Appropriation Ac(Net Profit transferred to Profit and Loss Appropriation Ac)

(b)In case of Net Loss

Profit and Loss Appropriation Achelliphelliphellip DrTo Profit and Loss Ac(Net Loss transferred to Profit and Loss Appropriation Ac)

2 for Interest on Capital

For transferring on Interest on CapitalProfit and Loss Appropriation Achelliphelliphellip DrTo Interest on Capital Ac(Interest on capital transferred to Profit amp Loss Appropriation Ac)

3 for Interest on Drawings

For transferring Interest on Drawings Interest on Drawings Achelliphelliphelliphelliphelliphellip DrTo Profit and Loss Appropriation Ac(Interest on drawing transferred to Profit amp Loss Appropriation Ac)

4 For Partnerrsquos SalaryFor transfer of partnerrsquos SalaryProfit and Loss Appropriation Achelliphellip DrTo Salary Ac(Salary transferred to profit amp Loss Appropriation Ac)

5 For Partnerrsquos CommissionFor transferring commissionProfit and Loss Appropriation Achelliphelliphellip DrTo Commission Ac(Commission transferred to Profit and Loss Appropriation Ac)

6 For Transfer of agreed amount to General ReserveProfit and Loss Appropriation Ac helliphellipDrTo General Reserve Ac(Transfer to General Reserve)

7 for share of Profit or Loss appropriation(a) If ProfitProfit and Loss Appropriation Achelliphellip DrTo Partnerrsquos CapitalCurrent Ac(Profit transferred to capitalcurrent Ac)(b) If LossPartnerrsquos Capital Current Achelliphelliphelliphellip DrTo Profit and Loss Appropriation Ac(Loss transferred to capitalcurrent Ac)

THE FORMAT OF PROFIT AND LOSS APPROPRIATION

Profit and Loss Appropriation Account for the year endedhelliphelliphelliphellip

Particulars Amount Particulars Amount

To PL Ac (loss) By pL Ac (profit)

To Interest on capital BY Interest on drawings

To partner`s commission by Partner`s capital Ac ( loss)

To Partner`s salary To Interest on partner`s loan To General Reserve To Partner`s Capital AC (Profit)

Subject= Economics

MOVEMENT ALONG THE DEMAND CURVE (CHANGE IN QUANTITY DEMANDED)In law of demand you have already studied the inverse relationship between price and quantity demanded When quantity demanded of a commodity changes due to change in its price keeping other factors constant it is called change in quantity demanded It is graphically expressed as a movement along the same demand curve There can be either a downward movement or an upward movement along the same demand curve Upward movement along the same demand curve is called contraction of demand or decrease in quantity demanded and downward movement along the same demand curve is known as expansion of demand or increase in quantity demanded

Extention of demandd

price (rs)p A

B Extentionp1 d

Q Q1

Quantity demanded ( in units)

Contraction of demandd

p2 Ccontraction

p APrice (Rs)

d

Q2 Q

Quantity demanded (in units)

Explanation of movement of demand A fall in price from OP to OP1 leads to increase in quantity demanded from OQ to OQ1 (expansion of demand) resulting in a downward movement from point A to point B along the same demand curve DD When Price rises from OP to OP2 quantity demanded falls from OQ to OQ2 (contraction of demand) leading to an upward movement from point A to point C along the same demand curve DD

  • Activity Series of Metals
    • Drawbacks of Rutherfordrsquos model of atom
      • Electromagnetic radiations
      • Properties of electromagnetic radiations
      • Characteristics of electromagnetic radiations
        • Plancks Quantum Theory-
        • Photoelectric effect
          • Intext Questions
            • Logic gates
            • Digital systems are said to be constructed by using logic gates These gates are the AND OR NOT NAND NOR EXOR and EXNOR gates The basic operations are described below with the aid of truth tables
            • AND gate
            • Example
Page 48:  · Web viewSubject. Topic. Summary. Execution. English 1 . Chapter 1 naming words . Page 8. Write the names of these pictures:- Person:-1. father. 2.Firefighter 3.doctor 4 ...

competitors

middlemen

publics

Fig STAKEHOLDERS OF A COMPANY

Apart from micro environment the other main dimension of business environment isMacro environment Macro environment refers to the general environment or remote environment within which a business firm and forces in its micro environment operateA company does not directly or regularly interact with the micro environmentTherefore macro environment is also known as indirect action EnvironmentThe macro environment forces are less controllable than the micro forces

Macro environment consists of the following components

POLITICAL AND LEGAL ENVIRONMENT

ECONOMIC SOCIAL AND ENVIRONMENT

CULTURAL

ENVIRONMENT

TECHNOLOGICAL ENVIRONMENT

Fig COMPONENTS OF MACRO ENVIRONMENT

environment or remote environment within which a business firm and forces in its micro environment operateA company does not directly or regularly interact with the micro environmentTherefore macro environment is also known as indirect action EnvironmentThe macro environment forces are less controllable than the micro forces 6 What are the components of macro environmenta Political and legal environmentb Economic environmentc Social and cultural environmentd Technological environment

Computer Science

Logic gates

Digital systems are said to be constructed by using logic gates These gates are the AND OR NOT NAND NOR EXOR and EXNOR

BUSINESS FIRM

gates The basic operations are described below with the aid of truth tables

AND gate

The AND gate is an electronic circuit that gives a high output (1) only if all its inputs are high A dot () is used to show the AND operation ie AB Bear in mind that this dot is sometimes omitted ie ABOR gate

The OR gate is an electronic circuit that gives a high output (1) if one or more of its inputs are high A plus (+) is used to show the OR operationNOT gate

The NOT gate is an electronic circuit that produces an inverted version of the input at its output It is also known as an inverter If the input variable is A the inverted output is known as NOT A This is also shown as A or A with a bar over the top as shown at the outputs The diagrams below show two ways that the NAND logic gate can be configured to produce a NOT gate It can also be done using NOR logic gates in the same way

NAND gate

This is a NOT-AND gate which is equal to an AND gate followed by a NOT gate The outputs of all NAND gates are high if any of the inputs are low The symbol is an AND gate with a small circle on the output The small circle represents inversion

NOR gate

This is a NOT-OR gate which is equal to an OR gate followed by a NOT gate The outputs of all NOR gates are low if any of the inputs are highThe symbol is an OR gate with a small circle on the output The small circle represents inversion

EXOR gate

The Exclusive-OR gate is a circuit which will give a high output if either but not both of its two inputs are high An encircled plus sign ( ) is used to show the EOR operation

EXNOR gate

The Exclusive-NOR gate circuit does the opposite to the EOR gate It will give a low output if either but not both of its two inputs are high The symbol is an EXOR gate with a small circle on the output The small circle represents inversion The NAND and NOR gates are called universal functions since with either one the AND and OR functions and NOT can be generated

Note A function in sum of products form can be implemented using NAND gates by replacing all AND and OR gates by NAND gates A function in product of sums form can be implemented using NOR gates by replacing all AND and OR gates by NOR gates

Logic gate symbols

Table 2 is a summary truth table of the inputoutput combinations for the NOT gate together with all possible inputoutput combinations for the other gate functions Also note that a truth table with n inputs has 2n rows You can compare the outputs of different gates

Logic gates representation using the Truth table

Example

A NAND gate can be used as a NOT gate using either of the following wiring configurations

Subject Eng Literature (The Tempest ndash William Shakespeare) Topic Act III Scene 3 Lines 53 to 110 (End of the scene) Date 16th April 2020 (2nd Period)

[Students should read the original play and also the paraphrase given in the school prescribed textbook]Summary Questions amp Answers

o Seeing this strange scene all are inclined to believe the tales told by travelers that there truly are ldquounicornsrdquo and ldquothe phoenixrsquo thronerdquo

o As they are about to sit down to the feast the banquet is snatched away by a harpy (Ariel disguised) A spiritrsquos voice (Arielrsquos voice) denounces Alonso Sebastian and Antonio with particular

1 ARIEL You are three men of sin whom Destiny

(Line 53-58)That hath to instrument this

lower world And what is int the never-surfeited sea

Hath caused to belch up you and on this island

Where man doth not inhabit you rsquomongst men

Being most unfit to live I have made you mad

reference to their crime in expelling Prospero from Milan They have not received any punishment for their deed earlier but the time for their punishment has arrived Upon Alonso it pronounces ldquolingering perdition worse than deathrdquo from which there is no remedy except through sincere repentance Ariel then vanishes in thunder and the shapes enter again and carry away the table

o Prospero watching invisibly is very pleased with the performance of Ariel and his (Prosperorsquos) ldquomeaner ministersrdquo All his enemies are now in his power and are in a fit of desperation He then leaves them and goes to see how Ferdinand and Miranda are getting on

o Alonso is now much humbled and penitent with the after effect of the spiritrsquos denunciation of his crimes He believes that his son is lost forever After this all disperse being stricken mad by the speech of the spirit

o Gonzalo fearing that they may do violence to themselves or to one another follows them and bid others to follow

(a) To whom does Ariel disguised as a harpy call the three sinners What game did Fate of Destiny play with

them

The three sinners called by Ariel are Alonso Sebastian and Antonio It was Destiny which had caused the ocean to cast the three sinners on the shore Though the ocean is all the time devouring whatever appears on its surface and is never satisfied with its continual swallowing of the ships and men in the present case the ocean had cast these three sinners on the shore without killing them

(b) Who had jointly been responsible for the conspiracy against Prospero What is Prosperorsquos purpose behind all this

Three men Alonso Sebastian and Antonio had jointly

been responsible for the conspiracy against Prospero They had driven out Prospero form Milan Prosperorsquos purpose is to make these three sinners realize the wrong they had done He wants them to repent for their criminal deeds because repentance leads to self-esteem(c )What does Ariel (the harpy) tell Alonso and his companions when they take out their swords to attack him

Seeing them drawing their swords Ariel (harpy) tells them that he and his companions are the instruments of destiny and that it is not possible for human beings to do them any injury He says that the swords of human beings can not injure even a minute part of his feathers Their swords are as ineffective against him and his companions as against the wind or the water

(d) Give the explanatory meanings of the following expressions in the context of the above extract

(i)Never surfeited (ii) Belch up (iii) lsquomongst men

(i) Never surfeited never led to satisfaction

(ii) Belch up cast ashore(iii) lsquomongst men in human

society2

I and my fellows (Line 60-65)

Are ministers of Fate The elementsOf whom your swords are tempered may as wellWound the loud winds or with bemocked-at stabsKill the still-closing waters as diminishOne dowl thats in my plume

IMPORTANT PASSAGES EXPLAINED

The elements

(Line 61-66)Of whom your swords are tempered may

as wellWound the loud winds or with

bemocked-at stabs

(a) Who is lsquoIrsquo Who are his lsquofellowsrdquo

lsquoIrsquo is referred to Ariel in disguise of a harpy His lsquofellowsrsquo are other spirits serving Prospero the real Duke of Milan who has acquired supernatural powers after being banished from his Dukedom Prospero has settled in this uninhabited island

(b) What are the elements that have temperrsquod the swords Why will it not work against the speaker

The swords (of Alonso and his companions) are tempered by metal (steel) which is taken out of the earth and refined by

Kill the still-closing waters as diminishOne dowl thats in my plume My fellow

ministersAre like invulnerable

In these words Ariel reminds the King and his companions of the utter futility of drawing swords against himself and his fellows Ariel drives Alonso Antonio and Sebastian the three men of sin to desperation ndash a state in which men do violence to themselves They draw swords to strike Ariel But Ariel reminds them that he and the other spirits are the ministers of destiny and nothing can wound them The steel of which their swords are made of may cut the wind or water which being divided always closes up again Even supposing that such things may be possible it is quite impossible that their swords will cut one feather in their plume They are incapable of being wounded by any sword of man Hence it is foolish on their part to attempt to strike at Ariel and his fellow-spirits

For which foul deed

(Line 72-75)The powers delaying not forgetting

haveIncensed the seas and shores yea all the

creatures Against your peace

Ariel enters like a harpy and remaining invisible tells Alonso Sebastian and Antonio that he and other harpies are the agents of Destiny appointed to carry out her decrees He tells them that their punishment for the crime against Prospero which has been so long deferred is now to fall upon them He reminds them that they had expelled Prospero from Milan and set him and his innocent child adrift on the sea and that the sea had paid them back for their sin by the shipwreck and by the calamities they have suffered He tells them that the powers above which did not forget this mean treachery but only deferred the punishment have now engaged the seas and the shores and all living beings including him and his comrades against them The very elements and supernatural agency Ariel adds have taken up the avenging of their crime against Prospero

the action of fire It may cut the wind or water which being divided always closes up again

The sword will not work against the spirits and the harpy because they are the ministers of destiny and nothing can wound them nor it will cut a single feather in their plume

(c )What is the meaning of lsquodowlrsquo in the last line

The term lsquodowlrsquo means a filament or the smallest part of a feather In this context Ariel in disguise of harpy says that their sword cannot even damage the smallest filament of their (Arielrsquos and other spirits) feathers as they are incapable of being wounded by any sword of man

(d) What does the speaker remind the listeners about

Ariel in disguise of harpy reminds Alonso the King of Naples Sebastian Alonsorsquos brother and Antonio the present Duke of Milan and the treacherous brother of Prospero as they being three men of sin He even reminds them that their punishment for their crime against Prospero which has been so long deferred now falls upon them He reminds them that they have expelled Prospero from Milan and has set him along with his innocent infant daughter adrift on the sea So the sea has paid them back for their sin by their shipwreck and the calamities they have suffered since then The harpy rebukes Alonso of his sin that has incensed the Gods and has deprived him of his son as a punishment

(e) How do they respond

When Ariel in disguise of a harpy reminds Alonso Sebastian and Antonio of their past misdeeds and sin Alonso has a look of terror and confusion in his eyes He utters the words of sincere repentance wrung out of his conscience-stricken heart It appears to him that all the elements of nature the sea-waves the wind and the thunder proclaiming a loud voice in the name of Prospero and the crime Alonso has committed against him They are calling upon him to repent There is a deep storm raging in Alonsorsquos breast and the echoes of that storm are ringing in his ears like a clear note of wind-instrument A note of denunciation of Alonsorsquos crime leaves him much humbled and penitent and confirms his belief that his son is lost forever But Sebastian and Antonio shows some courage instead of repentance They wish to kill the spirits or devils if it appears

3

Of my instruction hast thou nothing bated (Line 85-93)

In what thou hast to say So with good life

And observation strange my meaner ministers

Their several kinds have done My high charms work

And these mine enemies are all knit upIn their distractions They now are in my

powerAnd in these fits I leave them while I visitYoung Ferdinand whom they suppose is

drownedAnd his and mine loved darling

Methought the billows spoke and (Line 96-99)

told me of itThe winds did sing it to me and the

thunderThat deep and dreadful organ-pipe

pronouncedThe name of Prosper It did bass my

trespass

These are the words of contrition coming from Alonso Ariel has driven him to a deep repentance for conspiring with Antonio against Prospero He now feels a sincere remorse It appears to him that all the elements of nature the sea-waves the wind and the thunder proclaimed with a loud voice the name of Prospero and the crime Alonso had committed against him They are calling upon him to repent There is a deep storm raging in Alonsorsquos breast and the echoes of that storm are ringing in his ears like the clear note of a wind-instrument

Comment These are the words of sincere repentance wrung out of the conscience-stricken heart of Alonso Alonso who is the lesser villain is the first to give way to remorse under the effect of Arielrsquos speech The words of Ariel seem to him to be the voice of conscience speaking to him He is driven to desperation a state in which he might do violence to his life

(a) Identify the speaker State the context

Prospero the ruler of the island is the speaker The famous banquet scene has been enacted very well Ariel and his junior spirits have played their roles excellently Prospero is glad to say words of praise for them(b) In what way the speakerrsquos instructions have been carried out

According to Prosperorsquos instructions a banquet was presented before the King of Naples and his companions when they were tired and hungry Just when they were preparing to eat the feast the banquet was suddenly removed by exercising supernatural powers All this was done by Ariel Prosperorsquos chief assistant and a powerful spirit

Ariel not only made the feast disappear but also delivered his speech blaming the King and his two companions for their past wicked deeds He warned them to repent for their misdeeds or suffer forever on that uninhabited island

(c) Who are referred to as lsquomeaner ministersrsquo What have they done

Prospero refers as lsquomeaner ministersrsquo to his other lesser spirits who were assisting Ariel in presenting a scene before the kingrsquos party They entered the scene to the accompaniment of music They assumed several strange shapes and brought in a banquet Then they danced about it with gentle actions of salutations thus inviting the King and others to eat the feast

These spirits play their role again when Ariel in the shape of a harpy quits the scene These shapes enter again and dancing with mocking gestures carry away the table

(d) Who are the speakerrsquos enemies What has happened to them

King of Naples Alonso his brother Sebastian and the present Duke of Milan Antonio (Prosperorsquos own brother) are Prosperorsquos enemies With the turn of events they have all been washed ashore on the island which is ruled by Prospero the great magician Actually this happened after the shipwreck caused by a storm which was raised by Prospero with the purpose of bringing these people to his island Prosperorsquos spirits have already confused and terrified these enemies and they are under Prosperorsquos control He can treat them as he likes

(e) What does he say about Ferdinand Explain what is meant by ldquohellip his and mine darlingrdquo

Prospero knows that Alonsorsquos son prince Ferdinand is alive though his father thinks that the prince has been drowned

Prospero refers to his daughter Miranda who is dear to him She is also very dear to Prince Ferdinand who has fallen in love with her They are waiting to be married soon for which they have received Prosperorsquos consent

4

ALONSO O it is monstrous monstrous (Line 95-102)

Methought the billows spoke and told me of it

The winds did sing it to me and the thunderThat deep and dreadful organ-

pipe pronouncedThe name of Prosper It did bass

my trespassTherefore my son ithrsquo ooze is

bedded andIll seek him deeper than eer

plummet soundedAnd with him there lie mudded

(a) In what way does Alonso express his horror when his conscience is awakened by Arielrsquos words

When Alonsorsquos conscience is awakened by Arielrsquos words he expresses his horror at what he has heard He gets the feeling that the waves of the ocean the wind and the loud thunder have spoken to him and uttered the name of Prospero Because of being reminded of his crime in a very loud and rough voice he comes to realize that he has lost his son for his past misdeeds

(b) What does Alonso imagine about his son What does Alonso want to do in his desperate state

Alonso imagines that his son is lying in the mud at the bottom of the sea He feels desperate that he wants to drown himself in the ocean deeper than the plumb-line has ever gone He wants to lie with his son at the bottom of the sea

(c) How do Sebastian and Antonio want to face the evil spirits

Sebastian says that he is not at all afraid of what the harpy has said and that he is prepared to fight any number of such monsters if they appear before him only one at a time Antonio says that he would support Sebastian in the fight against the fiendsyyy

(d) Why does Gonzalo ask Adrian to follow the three men

Gonzalo tells Adrian that all the three men namely Alonso Sebastian and Antonio are in a wild and reckless mood The thought of the heinous crime of which they are guilty has begun to torment their minds So he asks Adrian to follow those three men without loss of time and prevent them from doing anything which the turmoil in their minds might lead them to do

(e) What opinion do you form of Alonso from the above extract

Alonso who is the lesser villain is the first to give way to remorse under the effect of Arielrsquos speech The words of Ariel seem to him to be the voice of conscience speaking to him He is driven to desperation a state in which he might do violence to his life

Subject =Accounts

Ac-12 15420 topic-pL Appropriation ac

PROFIT AND LOSS APPROPRIATION ACCOUNT

MEANING AND PREPARATIONProfit and Loss Appropriation Account is merely an extension of the Profit and Loss Account of the firm The profit of the firm has to be distributed amongst the partners in their respective profit sharing ratio But before its distribution it needs to be adjusted All Adjustments like partnerrsquos salary partnerrsquos commission interest on capital interest on drawings etc are made in this account These adjustments will reduce the amount of profit for distribution This adjusted profit will be distributed amongst the partners in their profit sharing ratio To prepare it at first the balance of Profit and Loss Account is transferred to this account The journal entries for the preparation of Profit and Loss Appropriation Account are given below

1 for transfer of the balance of Profit and Loss Account to Profit and Loss Appropriation Account

(a) In case of Net Profit

Profit and Loss Ac helliphelliphelliphelliphellipDrTo Profit and Loss Appropriation Ac(Net Profit transferred to Profit and Loss Appropriation Ac)

(b)In case of Net Loss

Profit and Loss Appropriation Achelliphelliphellip DrTo Profit and Loss Ac(Net Loss transferred to Profit and Loss Appropriation Ac)

2 for Interest on Capital

For transferring on Interest on CapitalProfit and Loss Appropriation Achelliphelliphellip DrTo Interest on Capital Ac(Interest on capital transferred to Profit amp Loss Appropriation Ac)

3 for Interest on Drawings

For transferring Interest on Drawings Interest on Drawings Achelliphelliphelliphelliphelliphellip DrTo Profit and Loss Appropriation Ac(Interest on drawing transferred to Profit amp Loss Appropriation Ac)

4 For Partnerrsquos SalaryFor transfer of partnerrsquos SalaryProfit and Loss Appropriation Achelliphellip DrTo Salary Ac(Salary transferred to profit amp Loss Appropriation Ac)

5 For Partnerrsquos CommissionFor transferring commissionProfit and Loss Appropriation Achelliphelliphellip DrTo Commission Ac(Commission transferred to Profit and Loss Appropriation Ac)

6 For Transfer of agreed amount to General ReserveProfit and Loss Appropriation Ac helliphellipDrTo General Reserve Ac(Transfer to General Reserve)

7 for share of Profit or Loss appropriation(a) If ProfitProfit and Loss Appropriation Achelliphellip DrTo Partnerrsquos CapitalCurrent Ac(Profit transferred to capitalcurrent Ac)(b) If LossPartnerrsquos Capital Current Achelliphelliphelliphellip DrTo Profit and Loss Appropriation Ac(Loss transferred to capitalcurrent Ac)

THE FORMAT OF PROFIT AND LOSS APPROPRIATION

Profit and Loss Appropriation Account for the year endedhelliphelliphelliphellip

Particulars Amount Particulars Amount

To PL Ac (loss) By pL Ac (profit)

To Interest on capital BY Interest on drawings

To partner`s commission by Partner`s capital Ac ( loss)

To Partner`s salary To Interest on partner`s loan To General Reserve To Partner`s Capital AC (Profit)

Subject= Economics

MOVEMENT ALONG THE DEMAND CURVE (CHANGE IN QUANTITY DEMANDED)In law of demand you have already studied the inverse relationship between price and quantity demanded When quantity demanded of a commodity changes due to change in its price keeping other factors constant it is called change in quantity demanded It is graphically expressed as a movement along the same demand curve There can be either a downward movement or an upward movement along the same demand curve Upward movement along the same demand curve is called contraction of demand or decrease in quantity demanded and downward movement along the same demand curve is known as expansion of demand or increase in quantity demanded

Extention of demandd

price (rs)p A

B Extentionp1 d

Q Q1

Quantity demanded ( in units)

Contraction of demandd

p2 Ccontraction

p APrice (Rs)

d

Q2 Q

Quantity demanded (in units)

Explanation of movement of demand A fall in price from OP to OP1 leads to increase in quantity demanded from OQ to OQ1 (expansion of demand) resulting in a downward movement from point A to point B along the same demand curve DD When Price rises from OP to OP2 quantity demanded falls from OQ to OQ2 (contraction of demand) leading to an upward movement from point A to point C along the same demand curve DD

  • Activity Series of Metals
    • Drawbacks of Rutherfordrsquos model of atom
      • Electromagnetic radiations
      • Properties of electromagnetic radiations
      • Characteristics of electromagnetic radiations
        • Plancks Quantum Theory-
        • Photoelectric effect
          • Intext Questions
            • Logic gates
            • Digital systems are said to be constructed by using logic gates These gates are the AND OR NOT NAND NOR EXOR and EXNOR gates The basic operations are described below with the aid of truth tables
            • AND gate
            • Example
Page 49:  · Web viewSubject. Topic. Summary. Execution. English 1 . Chapter 1 naming words . Page 8. Write the names of these pictures:- Person:-1. father. 2.Firefighter 3.doctor 4 ...

gates The basic operations are described below with the aid of truth tables

AND gate

The AND gate is an electronic circuit that gives a high output (1) only if all its inputs are high A dot () is used to show the AND operation ie AB Bear in mind that this dot is sometimes omitted ie ABOR gate

The OR gate is an electronic circuit that gives a high output (1) if one or more of its inputs are high A plus (+) is used to show the OR operationNOT gate

The NOT gate is an electronic circuit that produces an inverted version of the input at its output It is also known as an inverter If the input variable is A the inverted output is known as NOT A This is also shown as A or A with a bar over the top as shown at the outputs The diagrams below show two ways that the NAND logic gate can be configured to produce a NOT gate It can also be done using NOR logic gates in the same way

NAND gate

This is a NOT-AND gate which is equal to an AND gate followed by a NOT gate The outputs of all NAND gates are high if any of the inputs are low The symbol is an AND gate with a small circle on the output The small circle represents inversion

NOR gate

This is a NOT-OR gate which is equal to an OR gate followed by a NOT gate The outputs of all NOR gates are low if any of the inputs are highThe symbol is an OR gate with a small circle on the output The small circle represents inversion

EXOR gate

The Exclusive-OR gate is a circuit which will give a high output if either but not both of its two inputs are high An encircled plus sign ( ) is used to show the EOR operation

EXNOR gate

The Exclusive-NOR gate circuit does the opposite to the EOR gate It will give a low output if either but not both of its two inputs are high The symbol is an EXOR gate with a small circle on the output The small circle represents inversion The NAND and NOR gates are called universal functions since with either one the AND and OR functions and NOT can be generated

Note A function in sum of products form can be implemented using NAND gates by replacing all AND and OR gates by NAND gates A function in product of sums form can be implemented using NOR gates by replacing all AND and OR gates by NOR gates

Logic gate symbols

Table 2 is a summary truth table of the inputoutput combinations for the NOT gate together with all possible inputoutput combinations for the other gate functions Also note that a truth table with n inputs has 2n rows You can compare the outputs of different gates

Logic gates representation using the Truth table

Example

A NAND gate can be used as a NOT gate using either of the following wiring configurations

Subject Eng Literature (The Tempest ndash William Shakespeare) Topic Act III Scene 3 Lines 53 to 110 (End of the scene) Date 16th April 2020 (2nd Period)

[Students should read the original play and also the paraphrase given in the school prescribed textbook]Summary Questions amp Answers

o Seeing this strange scene all are inclined to believe the tales told by travelers that there truly are ldquounicornsrdquo and ldquothe phoenixrsquo thronerdquo

o As they are about to sit down to the feast the banquet is snatched away by a harpy (Ariel disguised) A spiritrsquos voice (Arielrsquos voice) denounces Alonso Sebastian and Antonio with particular

1 ARIEL You are three men of sin whom Destiny

(Line 53-58)That hath to instrument this

lower world And what is int the never-surfeited sea

Hath caused to belch up you and on this island

Where man doth not inhabit you rsquomongst men

Being most unfit to live I have made you mad

reference to their crime in expelling Prospero from Milan They have not received any punishment for their deed earlier but the time for their punishment has arrived Upon Alonso it pronounces ldquolingering perdition worse than deathrdquo from which there is no remedy except through sincere repentance Ariel then vanishes in thunder and the shapes enter again and carry away the table

o Prospero watching invisibly is very pleased with the performance of Ariel and his (Prosperorsquos) ldquomeaner ministersrdquo All his enemies are now in his power and are in a fit of desperation He then leaves them and goes to see how Ferdinand and Miranda are getting on

o Alonso is now much humbled and penitent with the after effect of the spiritrsquos denunciation of his crimes He believes that his son is lost forever After this all disperse being stricken mad by the speech of the spirit

o Gonzalo fearing that they may do violence to themselves or to one another follows them and bid others to follow

(a) To whom does Ariel disguised as a harpy call the three sinners What game did Fate of Destiny play with

them

The three sinners called by Ariel are Alonso Sebastian and Antonio It was Destiny which had caused the ocean to cast the three sinners on the shore Though the ocean is all the time devouring whatever appears on its surface and is never satisfied with its continual swallowing of the ships and men in the present case the ocean had cast these three sinners on the shore without killing them

(b) Who had jointly been responsible for the conspiracy against Prospero What is Prosperorsquos purpose behind all this

Three men Alonso Sebastian and Antonio had jointly

been responsible for the conspiracy against Prospero They had driven out Prospero form Milan Prosperorsquos purpose is to make these three sinners realize the wrong they had done He wants them to repent for their criminal deeds because repentance leads to self-esteem(c )What does Ariel (the harpy) tell Alonso and his companions when they take out their swords to attack him

Seeing them drawing their swords Ariel (harpy) tells them that he and his companions are the instruments of destiny and that it is not possible for human beings to do them any injury He says that the swords of human beings can not injure even a minute part of his feathers Their swords are as ineffective against him and his companions as against the wind or the water

(d) Give the explanatory meanings of the following expressions in the context of the above extract

(i)Never surfeited (ii) Belch up (iii) lsquomongst men

(i) Never surfeited never led to satisfaction

(ii) Belch up cast ashore(iii) lsquomongst men in human

society2

I and my fellows (Line 60-65)

Are ministers of Fate The elementsOf whom your swords are tempered may as wellWound the loud winds or with bemocked-at stabsKill the still-closing waters as diminishOne dowl thats in my plume

IMPORTANT PASSAGES EXPLAINED

The elements

(Line 61-66)Of whom your swords are tempered may

as wellWound the loud winds or with

bemocked-at stabs

(a) Who is lsquoIrsquo Who are his lsquofellowsrdquo

lsquoIrsquo is referred to Ariel in disguise of a harpy His lsquofellowsrsquo are other spirits serving Prospero the real Duke of Milan who has acquired supernatural powers after being banished from his Dukedom Prospero has settled in this uninhabited island

(b) What are the elements that have temperrsquod the swords Why will it not work against the speaker

The swords (of Alonso and his companions) are tempered by metal (steel) which is taken out of the earth and refined by

Kill the still-closing waters as diminishOne dowl thats in my plume My fellow

ministersAre like invulnerable

In these words Ariel reminds the King and his companions of the utter futility of drawing swords against himself and his fellows Ariel drives Alonso Antonio and Sebastian the three men of sin to desperation ndash a state in which men do violence to themselves They draw swords to strike Ariel But Ariel reminds them that he and the other spirits are the ministers of destiny and nothing can wound them The steel of which their swords are made of may cut the wind or water which being divided always closes up again Even supposing that such things may be possible it is quite impossible that their swords will cut one feather in their plume They are incapable of being wounded by any sword of man Hence it is foolish on their part to attempt to strike at Ariel and his fellow-spirits

For which foul deed

(Line 72-75)The powers delaying not forgetting

haveIncensed the seas and shores yea all the

creatures Against your peace

Ariel enters like a harpy and remaining invisible tells Alonso Sebastian and Antonio that he and other harpies are the agents of Destiny appointed to carry out her decrees He tells them that their punishment for the crime against Prospero which has been so long deferred is now to fall upon them He reminds them that they had expelled Prospero from Milan and set him and his innocent child adrift on the sea and that the sea had paid them back for their sin by the shipwreck and by the calamities they have suffered He tells them that the powers above which did not forget this mean treachery but only deferred the punishment have now engaged the seas and the shores and all living beings including him and his comrades against them The very elements and supernatural agency Ariel adds have taken up the avenging of their crime against Prospero

the action of fire It may cut the wind or water which being divided always closes up again

The sword will not work against the spirits and the harpy because they are the ministers of destiny and nothing can wound them nor it will cut a single feather in their plume

(c )What is the meaning of lsquodowlrsquo in the last line

The term lsquodowlrsquo means a filament or the smallest part of a feather In this context Ariel in disguise of harpy says that their sword cannot even damage the smallest filament of their (Arielrsquos and other spirits) feathers as they are incapable of being wounded by any sword of man

(d) What does the speaker remind the listeners about

Ariel in disguise of harpy reminds Alonso the King of Naples Sebastian Alonsorsquos brother and Antonio the present Duke of Milan and the treacherous brother of Prospero as they being three men of sin He even reminds them that their punishment for their crime against Prospero which has been so long deferred now falls upon them He reminds them that they have expelled Prospero from Milan and has set him along with his innocent infant daughter adrift on the sea So the sea has paid them back for their sin by their shipwreck and the calamities they have suffered since then The harpy rebukes Alonso of his sin that has incensed the Gods and has deprived him of his son as a punishment

(e) How do they respond

When Ariel in disguise of a harpy reminds Alonso Sebastian and Antonio of their past misdeeds and sin Alonso has a look of terror and confusion in his eyes He utters the words of sincere repentance wrung out of his conscience-stricken heart It appears to him that all the elements of nature the sea-waves the wind and the thunder proclaiming a loud voice in the name of Prospero and the crime Alonso has committed against him They are calling upon him to repent There is a deep storm raging in Alonsorsquos breast and the echoes of that storm are ringing in his ears like a clear note of wind-instrument A note of denunciation of Alonsorsquos crime leaves him much humbled and penitent and confirms his belief that his son is lost forever But Sebastian and Antonio shows some courage instead of repentance They wish to kill the spirits or devils if it appears

3

Of my instruction hast thou nothing bated (Line 85-93)

In what thou hast to say So with good life

And observation strange my meaner ministers

Their several kinds have done My high charms work

And these mine enemies are all knit upIn their distractions They now are in my

powerAnd in these fits I leave them while I visitYoung Ferdinand whom they suppose is

drownedAnd his and mine loved darling

Methought the billows spoke and (Line 96-99)

told me of itThe winds did sing it to me and the

thunderThat deep and dreadful organ-pipe

pronouncedThe name of Prosper It did bass my

trespass

These are the words of contrition coming from Alonso Ariel has driven him to a deep repentance for conspiring with Antonio against Prospero He now feels a sincere remorse It appears to him that all the elements of nature the sea-waves the wind and the thunder proclaimed with a loud voice the name of Prospero and the crime Alonso had committed against him They are calling upon him to repent There is a deep storm raging in Alonsorsquos breast and the echoes of that storm are ringing in his ears like the clear note of a wind-instrument

Comment These are the words of sincere repentance wrung out of the conscience-stricken heart of Alonso Alonso who is the lesser villain is the first to give way to remorse under the effect of Arielrsquos speech The words of Ariel seem to him to be the voice of conscience speaking to him He is driven to desperation a state in which he might do violence to his life

(a) Identify the speaker State the context

Prospero the ruler of the island is the speaker The famous banquet scene has been enacted very well Ariel and his junior spirits have played their roles excellently Prospero is glad to say words of praise for them(b) In what way the speakerrsquos instructions have been carried out

According to Prosperorsquos instructions a banquet was presented before the King of Naples and his companions when they were tired and hungry Just when they were preparing to eat the feast the banquet was suddenly removed by exercising supernatural powers All this was done by Ariel Prosperorsquos chief assistant and a powerful spirit

Ariel not only made the feast disappear but also delivered his speech blaming the King and his two companions for their past wicked deeds He warned them to repent for their misdeeds or suffer forever on that uninhabited island

(c) Who are referred to as lsquomeaner ministersrsquo What have they done

Prospero refers as lsquomeaner ministersrsquo to his other lesser spirits who were assisting Ariel in presenting a scene before the kingrsquos party They entered the scene to the accompaniment of music They assumed several strange shapes and brought in a banquet Then they danced about it with gentle actions of salutations thus inviting the King and others to eat the feast

These spirits play their role again when Ariel in the shape of a harpy quits the scene These shapes enter again and dancing with mocking gestures carry away the table

(d) Who are the speakerrsquos enemies What has happened to them

King of Naples Alonso his brother Sebastian and the present Duke of Milan Antonio (Prosperorsquos own brother) are Prosperorsquos enemies With the turn of events they have all been washed ashore on the island which is ruled by Prospero the great magician Actually this happened after the shipwreck caused by a storm which was raised by Prospero with the purpose of bringing these people to his island Prosperorsquos spirits have already confused and terrified these enemies and they are under Prosperorsquos control He can treat them as he likes

(e) What does he say about Ferdinand Explain what is meant by ldquohellip his and mine darlingrdquo

Prospero knows that Alonsorsquos son prince Ferdinand is alive though his father thinks that the prince has been drowned

Prospero refers to his daughter Miranda who is dear to him She is also very dear to Prince Ferdinand who has fallen in love with her They are waiting to be married soon for which they have received Prosperorsquos consent

4

ALONSO O it is monstrous monstrous (Line 95-102)

Methought the billows spoke and told me of it

The winds did sing it to me and the thunderThat deep and dreadful organ-

pipe pronouncedThe name of Prosper It did bass

my trespassTherefore my son ithrsquo ooze is

bedded andIll seek him deeper than eer

plummet soundedAnd with him there lie mudded

(a) In what way does Alonso express his horror when his conscience is awakened by Arielrsquos words

When Alonsorsquos conscience is awakened by Arielrsquos words he expresses his horror at what he has heard He gets the feeling that the waves of the ocean the wind and the loud thunder have spoken to him and uttered the name of Prospero Because of being reminded of his crime in a very loud and rough voice he comes to realize that he has lost his son for his past misdeeds

(b) What does Alonso imagine about his son What does Alonso want to do in his desperate state

Alonso imagines that his son is lying in the mud at the bottom of the sea He feels desperate that he wants to drown himself in the ocean deeper than the plumb-line has ever gone He wants to lie with his son at the bottom of the sea

(c) How do Sebastian and Antonio want to face the evil spirits

Sebastian says that he is not at all afraid of what the harpy has said and that he is prepared to fight any number of such monsters if they appear before him only one at a time Antonio says that he would support Sebastian in the fight against the fiendsyyy

(d) Why does Gonzalo ask Adrian to follow the three men

Gonzalo tells Adrian that all the three men namely Alonso Sebastian and Antonio are in a wild and reckless mood The thought of the heinous crime of which they are guilty has begun to torment their minds So he asks Adrian to follow those three men without loss of time and prevent them from doing anything which the turmoil in their minds might lead them to do

(e) What opinion do you form of Alonso from the above extract

Alonso who is the lesser villain is the first to give way to remorse under the effect of Arielrsquos speech The words of Ariel seem to him to be the voice of conscience speaking to him He is driven to desperation a state in which he might do violence to his life

Subject =Accounts

Ac-12 15420 topic-pL Appropriation ac

PROFIT AND LOSS APPROPRIATION ACCOUNT

MEANING AND PREPARATIONProfit and Loss Appropriation Account is merely an extension of the Profit and Loss Account of the firm The profit of the firm has to be distributed amongst the partners in their respective profit sharing ratio But before its distribution it needs to be adjusted All Adjustments like partnerrsquos salary partnerrsquos commission interest on capital interest on drawings etc are made in this account These adjustments will reduce the amount of profit for distribution This adjusted profit will be distributed amongst the partners in their profit sharing ratio To prepare it at first the balance of Profit and Loss Account is transferred to this account The journal entries for the preparation of Profit and Loss Appropriation Account are given below

1 for transfer of the balance of Profit and Loss Account to Profit and Loss Appropriation Account

(a) In case of Net Profit

Profit and Loss Ac helliphelliphelliphelliphellipDrTo Profit and Loss Appropriation Ac(Net Profit transferred to Profit and Loss Appropriation Ac)

(b)In case of Net Loss

Profit and Loss Appropriation Achelliphelliphellip DrTo Profit and Loss Ac(Net Loss transferred to Profit and Loss Appropriation Ac)

2 for Interest on Capital

For transferring on Interest on CapitalProfit and Loss Appropriation Achelliphelliphellip DrTo Interest on Capital Ac(Interest on capital transferred to Profit amp Loss Appropriation Ac)

3 for Interest on Drawings

For transferring Interest on Drawings Interest on Drawings Achelliphelliphelliphelliphelliphellip DrTo Profit and Loss Appropriation Ac(Interest on drawing transferred to Profit amp Loss Appropriation Ac)

4 For Partnerrsquos SalaryFor transfer of partnerrsquos SalaryProfit and Loss Appropriation Achelliphellip DrTo Salary Ac(Salary transferred to profit amp Loss Appropriation Ac)

5 For Partnerrsquos CommissionFor transferring commissionProfit and Loss Appropriation Achelliphelliphellip DrTo Commission Ac(Commission transferred to Profit and Loss Appropriation Ac)

6 For Transfer of agreed amount to General ReserveProfit and Loss Appropriation Ac helliphellipDrTo General Reserve Ac(Transfer to General Reserve)

7 for share of Profit or Loss appropriation(a) If ProfitProfit and Loss Appropriation Achelliphellip DrTo Partnerrsquos CapitalCurrent Ac(Profit transferred to capitalcurrent Ac)(b) If LossPartnerrsquos Capital Current Achelliphelliphelliphellip DrTo Profit and Loss Appropriation Ac(Loss transferred to capitalcurrent Ac)

THE FORMAT OF PROFIT AND LOSS APPROPRIATION

Profit and Loss Appropriation Account for the year endedhelliphelliphelliphellip

Particulars Amount Particulars Amount

To PL Ac (loss) By pL Ac (profit)

To Interest on capital BY Interest on drawings

To partner`s commission by Partner`s capital Ac ( loss)

To Partner`s salary To Interest on partner`s loan To General Reserve To Partner`s Capital AC (Profit)

Subject= Economics

MOVEMENT ALONG THE DEMAND CURVE (CHANGE IN QUANTITY DEMANDED)In law of demand you have already studied the inverse relationship between price and quantity demanded When quantity demanded of a commodity changes due to change in its price keeping other factors constant it is called change in quantity demanded It is graphically expressed as a movement along the same demand curve There can be either a downward movement or an upward movement along the same demand curve Upward movement along the same demand curve is called contraction of demand or decrease in quantity demanded and downward movement along the same demand curve is known as expansion of demand or increase in quantity demanded

Extention of demandd

price (rs)p A

B Extentionp1 d

Q Q1

Quantity demanded ( in units)

Contraction of demandd

p2 Ccontraction

p APrice (Rs)

d

Q2 Q

Quantity demanded (in units)

Explanation of movement of demand A fall in price from OP to OP1 leads to increase in quantity demanded from OQ to OQ1 (expansion of demand) resulting in a downward movement from point A to point B along the same demand curve DD When Price rises from OP to OP2 quantity demanded falls from OQ to OQ2 (contraction of demand) leading to an upward movement from point A to point C along the same demand curve DD

  • Activity Series of Metals
    • Drawbacks of Rutherfordrsquos model of atom
      • Electromagnetic radiations
      • Properties of electromagnetic radiations
      • Characteristics of electromagnetic radiations
        • Plancks Quantum Theory-
        • Photoelectric effect
          • Intext Questions
            • Logic gates
            • Digital systems are said to be constructed by using logic gates These gates are the AND OR NOT NAND NOR EXOR and EXNOR gates The basic operations are described below with the aid of truth tables
            • AND gate
            • Example
Page 50:  · Web viewSubject. Topic. Summary. Execution. English 1 . Chapter 1 naming words . Page 8. Write the names of these pictures:- Person:-1. father. 2.Firefighter 3.doctor 4 ...

This is a NOT-OR gate which is equal to an OR gate followed by a NOT gate The outputs of all NOR gates are low if any of the inputs are highThe symbol is an OR gate with a small circle on the output The small circle represents inversion

EXOR gate

The Exclusive-OR gate is a circuit which will give a high output if either but not both of its two inputs are high An encircled plus sign ( ) is used to show the EOR operation

EXNOR gate

The Exclusive-NOR gate circuit does the opposite to the EOR gate It will give a low output if either but not both of its two inputs are high The symbol is an EXOR gate with a small circle on the output The small circle represents inversion The NAND and NOR gates are called universal functions since with either one the AND and OR functions and NOT can be generated

Note A function in sum of products form can be implemented using NAND gates by replacing all AND and OR gates by NAND gates A function in product of sums form can be implemented using NOR gates by replacing all AND and OR gates by NOR gates

Logic gate symbols

Table 2 is a summary truth table of the inputoutput combinations for the NOT gate together with all possible inputoutput combinations for the other gate functions Also note that a truth table with n inputs has 2n rows You can compare the outputs of different gates

Logic gates representation using the Truth table

Example

A NAND gate can be used as a NOT gate using either of the following wiring configurations

Subject Eng Literature (The Tempest ndash William Shakespeare) Topic Act III Scene 3 Lines 53 to 110 (End of the scene) Date 16th April 2020 (2nd Period)

[Students should read the original play and also the paraphrase given in the school prescribed textbook]Summary Questions amp Answers

o Seeing this strange scene all are inclined to believe the tales told by travelers that there truly are ldquounicornsrdquo and ldquothe phoenixrsquo thronerdquo

o As they are about to sit down to the feast the banquet is snatched away by a harpy (Ariel disguised) A spiritrsquos voice (Arielrsquos voice) denounces Alonso Sebastian and Antonio with particular

1 ARIEL You are three men of sin whom Destiny

(Line 53-58)That hath to instrument this

lower world And what is int the never-surfeited sea

Hath caused to belch up you and on this island

Where man doth not inhabit you rsquomongst men

Being most unfit to live I have made you mad

reference to their crime in expelling Prospero from Milan They have not received any punishment for their deed earlier but the time for their punishment has arrived Upon Alonso it pronounces ldquolingering perdition worse than deathrdquo from which there is no remedy except through sincere repentance Ariel then vanishes in thunder and the shapes enter again and carry away the table

o Prospero watching invisibly is very pleased with the performance of Ariel and his (Prosperorsquos) ldquomeaner ministersrdquo All his enemies are now in his power and are in a fit of desperation He then leaves them and goes to see how Ferdinand and Miranda are getting on

o Alonso is now much humbled and penitent with the after effect of the spiritrsquos denunciation of his crimes He believes that his son is lost forever After this all disperse being stricken mad by the speech of the spirit

o Gonzalo fearing that they may do violence to themselves or to one another follows them and bid others to follow

(a) To whom does Ariel disguised as a harpy call the three sinners What game did Fate of Destiny play with

them

The three sinners called by Ariel are Alonso Sebastian and Antonio It was Destiny which had caused the ocean to cast the three sinners on the shore Though the ocean is all the time devouring whatever appears on its surface and is never satisfied with its continual swallowing of the ships and men in the present case the ocean had cast these three sinners on the shore without killing them

(b) Who had jointly been responsible for the conspiracy against Prospero What is Prosperorsquos purpose behind all this

Three men Alonso Sebastian and Antonio had jointly

been responsible for the conspiracy against Prospero They had driven out Prospero form Milan Prosperorsquos purpose is to make these three sinners realize the wrong they had done He wants them to repent for their criminal deeds because repentance leads to self-esteem(c )What does Ariel (the harpy) tell Alonso and his companions when they take out their swords to attack him

Seeing them drawing their swords Ariel (harpy) tells them that he and his companions are the instruments of destiny and that it is not possible for human beings to do them any injury He says that the swords of human beings can not injure even a minute part of his feathers Their swords are as ineffective against him and his companions as against the wind or the water

(d) Give the explanatory meanings of the following expressions in the context of the above extract

(i)Never surfeited (ii) Belch up (iii) lsquomongst men

(i) Never surfeited never led to satisfaction

(ii) Belch up cast ashore(iii) lsquomongst men in human

society2

I and my fellows (Line 60-65)

Are ministers of Fate The elementsOf whom your swords are tempered may as wellWound the loud winds or with bemocked-at stabsKill the still-closing waters as diminishOne dowl thats in my plume

IMPORTANT PASSAGES EXPLAINED

The elements

(Line 61-66)Of whom your swords are tempered may

as wellWound the loud winds or with

bemocked-at stabs

(a) Who is lsquoIrsquo Who are his lsquofellowsrdquo

lsquoIrsquo is referred to Ariel in disguise of a harpy His lsquofellowsrsquo are other spirits serving Prospero the real Duke of Milan who has acquired supernatural powers after being banished from his Dukedom Prospero has settled in this uninhabited island

(b) What are the elements that have temperrsquod the swords Why will it not work against the speaker

The swords (of Alonso and his companions) are tempered by metal (steel) which is taken out of the earth and refined by

Kill the still-closing waters as diminishOne dowl thats in my plume My fellow

ministersAre like invulnerable

In these words Ariel reminds the King and his companions of the utter futility of drawing swords against himself and his fellows Ariel drives Alonso Antonio and Sebastian the three men of sin to desperation ndash a state in which men do violence to themselves They draw swords to strike Ariel But Ariel reminds them that he and the other spirits are the ministers of destiny and nothing can wound them The steel of which their swords are made of may cut the wind or water which being divided always closes up again Even supposing that such things may be possible it is quite impossible that their swords will cut one feather in their plume They are incapable of being wounded by any sword of man Hence it is foolish on their part to attempt to strike at Ariel and his fellow-spirits

For which foul deed

(Line 72-75)The powers delaying not forgetting

haveIncensed the seas and shores yea all the

creatures Against your peace

Ariel enters like a harpy and remaining invisible tells Alonso Sebastian and Antonio that he and other harpies are the agents of Destiny appointed to carry out her decrees He tells them that their punishment for the crime against Prospero which has been so long deferred is now to fall upon them He reminds them that they had expelled Prospero from Milan and set him and his innocent child adrift on the sea and that the sea had paid them back for their sin by the shipwreck and by the calamities they have suffered He tells them that the powers above which did not forget this mean treachery but only deferred the punishment have now engaged the seas and the shores and all living beings including him and his comrades against them The very elements and supernatural agency Ariel adds have taken up the avenging of their crime against Prospero

the action of fire It may cut the wind or water which being divided always closes up again

The sword will not work against the spirits and the harpy because they are the ministers of destiny and nothing can wound them nor it will cut a single feather in their plume

(c )What is the meaning of lsquodowlrsquo in the last line

The term lsquodowlrsquo means a filament or the smallest part of a feather In this context Ariel in disguise of harpy says that their sword cannot even damage the smallest filament of their (Arielrsquos and other spirits) feathers as they are incapable of being wounded by any sword of man

(d) What does the speaker remind the listeners about

Ariel in disguise of harpy reminds Alonso the King of Naples Sebastian Alonsorsquos brother and Antonio the present Duke of Milan and the treacherous brother of Prospero as they being three men of sin He even reminds them that their punishment for their crime against Prospero which has been so long deferred now falls upon them He reminds them that they have expelled Prospero from Milan and has set him along with his innocent infant daughter adrift on the sea So the sea has paid them back for their sin by their shipwreck and the calamities they have suffered since then The harpy rebukes Alonso of his sin that has incensed the Gods and has deprived him of his son as a punishment

(e) How do they respond

When Ariel in disguise of a harpy reminds Alonso Sebastian and Antonio of their past misdeeds and sin Alonso has a look of terror and confusion in his eyes He utters the words of sincere repentance wrung out of his conscience-stricken heart It appears to him that all the elements of nature the sea-waves the wind and the thunder proclaiming a loud voice in the name of Prospero and the crime Alonso has committed against him They are calling upon him to repent There is a deep storm raging in Alonsorsquos breast and the echoes of that storm are ringing in his ears like a clear note of wind-instrument A note of denunciation of Alonsorsquos crime leaves him much humbled and penitent and confirms his belief that his son is lost forever But Sebastian and Antonio shows some courage instead of repentance They wish to kill the spirits or devils if it appears

3

Of my instruction hast thou nothing bated (Line 85-93)

In what thou hast to say So with good life

And observation strange my meaner ministers

Their several kinds have done My high charms work

And these mine enemies are all knit upIn their distractions They now are in my

powerAnd in these fits I leave them while I visitYoung Ferdinand whom they suppose is

drownedAnd his and mine loved darling

Methought the billows spoke and (Line 96-99)

told me of itThe winds did sing it to me and the

thunderThat deep and dreadful organ-pipe

pronouncedThe name of Prosper It did bass my

trespass

These are the words of contrition coming from Alonso Ariel has driven him to a deep repentance for conspiring with Antonio against Prospero He now feels a sincere remorse It appears to him that all the elements of nature the sea-waves the wind and the thunder proclaimed with a loud voice the name of Prospero and the crime Alonso had committed against him They are calling upon him to repent There is a deep storm raging in Alonsorsquos breast and the echoes of that storm are ringing in his ears like the clear note of a wind-instrument

Comment These are the words of sincere repentance wrung out of the conscience-stricken heart of Alonso Alonso who is the lesser villain is the first to give way to remorse under the effect of Arielrsquos speech The words of Ariel seem to him to be the voice of conscience speaking to him He is driven to desperation a state in which he might do violence to his life

(a) Identify the speaker State the context

Prospero the ruler of the island is the speaker The famous banquet scene has been enacted very well Ariel and his junior spirits have played their roles excellently Prospero is glad to say words of praise for them(b) In what way the speakerrsquos instructions have been carried out

According to Prosperorsquos instructions a banquet was presented before the King of Naples and his companions when they were tired and hungry Just when they were preparing to eat the feast the banquet was suddenly removed by exercising supernatural powers All this was done by Ariel Prosperorsquos chief assistant and a powerful spirit

Ariel not only made the feast disappear but also delivered his speech blaming the King and his two companions for their past wicked deeds He warned them to repent for their misdeeds or suffer forever on that uninhabited island

(c) Who are referred to as lsquomeaner ministersrsquo What have they done

Prospero refers as lsquomeaner ministersrsquo to his other lesser spirits who were assisting Ariel in presenting a scene before the kingrsquos party They entered the scene to the accompaniment of music They assumed several strange shapes and brought in a banquet Then they danced about it with gentle actions of salutations thus inviting the King and others to eat the feast

These spirits play their role again when Ariel in the shape of a harpy quits the scene These shapes enter again and dancing with mocking gestures carry away the table

(d) Who are the speakerrsquos enemies What has happened to them

King of Naples Alonso his brother Sebastian and the present Duke of Milan Antonio (Prosperorsquos own brother) are Prosperorsquos enemies With the turn of events they have all been washed ashore on the island which is ruled by Prospero the great magician Actually this happened after the shipwreck caused by a storm which was raised by Prospero with the purpose of bringing these people to his island Prosperorsquos spirits have already confused and terrified these enemies and they are under Prosperorsquos control He can treat them as he likes

(e) What does he say about Ferdinand Explain what is meant by ldquohellip his and mine darlingrdquo

Prospero knows that Alonsorsquos son prince Ferdinand is alive though his father thinks that the prince has been drowned

Prospero refers to his daughter Miranda who is dear to him She is also very dear to Prince Ferdinand who has fallen in love with her They are waiting to be married soon for which they have received Prosperorsquos consent

4

ALONSO O it is monstrous monstrous (Line 95-102)

Methought the billows spoke and told me of it

The winds did sing it to me and the thunderThat deep and dreadful organ-

pipe pronouncedThe name of Prosper It did bass

my trespassTherefore my son ithrsquo ooze is

bedded andIll seek him deeper than eer

plummet soundedAnd with him there lie mudded

(a) In what way does Alonso express his horror when his conscience is awakened by Arielrsquos words

When Alonsorsquos conscience is awakened by Arielrsquos words he expresses his horror at what he has heard He gets the feeling that the waves of the ocean the wind and the loud thunder have spoken to him and uttered the name of Prospero Because of being reminded of his crime in a very loud and rough voice he comes to realize that he has lost his son for his past misdeeds

(b) What does Alonso imagine about his son What does Alonso want to do in his desperate state

Alonso imagines that his son is lying in the mud at the bottom of the sea He feels desperate that he wants to drown himself in the ocean deeper than the plumb-line has ever gone He wants to lie with his son at the bottom of the sea

(c) How do Sebastian and Antonio want to face the evil spirits

Sebastian says that he is not at all afraid of what the harpy has said and that he is prepared to fight any number of such monsters if they appear before him only one at a time Antonio says that he would support Sebastian in the fight against the fiendsyyy

(d) Why does Gonzalo ask Adrian to follow the three men

Gonzalo tells Adrian that all the three men namely Alonso Sebastian and Antonio are in a wild and reckless mood The thought of the heinous crime of which they are guilty has begun to torment their minds So he asks Adrian to follow those three men without loss of time and prevent them from doing anything which the turmoil in their minds might lead them to do

(e) What opinion do you form of Alonso from the above extract

Alonso who is the lesser villain is the first to give way to remorse under the effect of Arielrsquos speech The words of Ariel seem to him to be the voice of conscience speaking to him He is driven to desperation a state in which he might do violence to his life

Subject =Accounts

Ac-12 15420 topic-pL Appropriation ac

PROFIT AND LOSS APPROPRIATION ACCOUNT

MEANING AND PREPARATIONProfit and Loss Appropriation Account is merely an extension of the Profit and Loss Account of the firm The profit of the firm has to be distributed amongst the partners in their respective profit sharing ratio But before its distribution it needs to be adjusted All Adjustments like partnerrsquos salary partnerrsquos commission interest on capital interest on drawings etc are made in this account These adjustments will reduce the amount of profit for distribution This adjusted profit will be distributed amongst the partners in their profit sharing ratio To prepare it at first the balance of Profit and Loss Account is transferred to this account The journal entries for the preparation of Profit and Loss Appropriation Account are given below

1 for transfer of the balance of Profit and Loss Account to Profit and Loss Appropriation Account

(a) In case of Net Profit

Profit and Loss Ac helliphelliphelliphelliphellipDrTo Profit and Loss Appropriation Ac(Net Profit transferred to Profit and Loss Appropriation Ac)

(b)In case of Net Loss

Profit and Loss Appropriation Achelliphelliphellip DrTo Profit and Loss Ac(Net Loss transferred to Profit and Loss Appropriation Ac)

2 for Interest on Capital

For transferring on Interest on CapitalProfit and Loss Appropriation Achelliphelliphellip DrTo Interest on Capital Ac(Interest on capital transferred to Profit amp Loss Appropriation Ac)

3 for Interest on Drawings

For transferring Interest on Drawings Interest on Drawings Achelliphelliphelliphelliphelliphellip DrTo Profit and Loss Appropriation Ac(Interest on drawing transferred to Profit amp Loss Appropriation Ac)

4 For Partnerrsquos SalaryFor transfer of partnerrsquos SalaryProfit and Loss Appropriation Achelliphellip DrTo Salary Ac(Salary transferred to profit amp Loss Appropriation Ac)

5 For Partnerrsquos CommissionFor transferring commissionProfit and Loss Appropriation Achelliphelliphellip DrTo Commission Ac(Commission transferred to Profit and Loss Appropriation Ac)

6 For Transfer of agreed amount to General ReserveProfit and Loss Appropriation Ac helliphellipDrTo General Reserve Ac(Transfer to General Reserve)

7 for share of Profit or Loss appropriation(a) If ProfitProfit and Loss Appropriation Achelliphellip DrTo Partnerrsquos CapitalCurrent Ac(Profit transferred to capitalcurrent Ac)(b) If LossPartnerrsquos Capital Current Achelliphelliphelliphellip DrTo Profit and Loss Appropriation Ac(Loss transferred to capitalcurrent Ac)

THE FORMAT OF PROFIT AND LOSS APPROPRIATION

Profit and Loss Appropriation Account for the year endedhelliphelliphelliphellip

Particulars Amount Particulars Amount

To PL Ac (loss) By pL Ac (profit)

To Interest on capital BY Interest on drawings

To partner`s commission by Partner`s capital Ac ( loss)

To Partner`s salary To Interest on partner`s loan To General Reserve To Partner`s Capital AC (Profit)

Subject= Economics

MOVEMENT ALONG THE DEMAND CURVE (CHANGE IN QUANTITY DEMANDED)In law of demand you have already studied the inverse relationship between price and quantity demanded When quantity demanded of a commodity changes due to change in its price keeping other factors constant it is called change in quantity demanded It is graphically expressed as a movement along the same demand curve There can be either a downward movement or an upward movement along the same demand curve Upward movement along the same demand curve is called contraction of demand or decrease in quantity demanded and downward movement along the same demand curve is known as expansion of demand or increase in quantity demanded

Extention of demandd

price (rs)p A

B Extentionp1 d

Q Q1

Quantity demanded ( in units)

Contraction of demandd

p2 Ccontraction

p APrice (Rs)

d

Q2 Q

Quantity demanded (in units)

Explanation of movement of demand A fall in price from OP to OP1 leads to increase in quantity demanded from OQ to OQ1 (expansion of demand) resulting in a downward movement from point A to point B along the same demand curve DD When Price rises from OP to OP2 quantity demanded falls from OQ to OQ2 (contraction of demand) leading to an upward movement from point A to point C along the same demand curve DD

  • Activity Series of Metals
    • Drawbacks of Rutherfordrsquos model of atom
      • Electromagnetic radiations
      • Properties of electromagnetic radiations
      • Characteristics of electromagnetic radiations
        • Plancks Quantum Theory-
        • Photoelectric effect
          • Intext Questions
            • Logic gates
            • Digital systems are said to be constructed by using logic gates These gates are the AND OR NOT NAND NOR EXOR and EXNOR gates The basic operations are described below with the aid of truth tables
            • AND gate
            • Example
Page 51:  · Web viewSubject. Topic. Summary. Execution. English 1 . Chapter 1 naming words . Page 8. Write the names of these pictures:- Person:-1. father. 2.Firefighter 3.doctor 4 ...

Table 2 is a summary truth table of the inputoutput combinations for the NOT gate together with all possible inputoutput combinations for the other gate functions Also note that a truth table with n inputs has 2n rows You can compare the outputs of different gates

Logic gates representation using the Truth table

Example

A NAND gate can be used as a NOT gate using either of the following wiring configurations

Subject Eng Literature (The Tempest ndash William Shakespeare) Topic Act III Scene 3 Lines 53 to 110 (End of the scene) Date 16th April 2020 (2nd Period)

[Students should read the original play and also the paraphrase given in the school prescribed textbook]Summary Questions amp Answers

o Seeing this strange scene all are inclined to believe the tales told by travelers that there truly are ldquounicornsrdquo and ldquothe phoenixrsquo thronerdquo

o As they are about to sit down to the feast the banquet is snatched away by a harpy (Ariel disguised) A spiritrsquos voice (Arielrsquos voice) denounces Alonso Sebastian and Antonio with particular

1 ARIEL You are three men of sin whom Destiny

(Line 53-58)That hath to instrument this

lower world And what is int the never-surfeited sea

Hath caused to belch up you and on this island

Where man doth not inhabit you rsquomongst men

Being most unfit to live I have made you mad

reference to their crime in expelling Prospero from Milan They have not received any punishment for their deed earlier but the time for their punishment has arrived Upon Alonso it pronounces ldquolingering perdition worse than deathrdquo from which there is no remedy except through sincere repentance Ariel then vanishes in thunder and the shapes enter again and carry away the table

o Prospero watching invisibly is very pleased with the performance of Ariel and his (Prosperorsquos) ldquomeaner ministersrdquo All his enemies are now in his power and are in a fit of desperation He then leaves them and goes to see how Ferdinand and Miranda are getting on

o Alonso is now much humbled and penitent with the after effect of the spiritrsquos denunciation of his crimes He believes that his son is lost forever After this all disperse being stricken mad by the speech of the spirit

o Gonzalo fearing that they may do violence to themselves or to one another follows them and bid others to follow

(a) To whom does Ariel disguised as a harpy call the three sinners What game did Fate of Destiny play with

them

The three sinners called by Ariel are Alonso Sebastian and Antonio It was Destiny which had caused the ocean to cast the three sinners on the shore Though the ocean is all the time devouring whatever appears on its surface and is never satisfied with its continual swallowing of the ships and men in the present case the ocean had cast these three sinners on the shore without killing them

(b) Who had jointly been responsible for the conspiracy against Prospero What is Prosperorsquos purpose behind all this

Three men Alonso Sebastian and Antonio had jointly

been responsible for the conspiracy against Prospero They had driven out Prospero form Milan Prosperorsquos purpose is to make these three sinners realize the wrong they had done He wants them to repent for their criminal deeds because repentance leads to self-esteem(c )What does Ariel (the harpy) tell Alonso and his companions when they take out their swords to attack him

Seeing them drawing their swords Ariel (harpy) tells them that he and his companions are the instruments of destiny and that it is not possible for human beings to do them any injury He says that the swords of human beings can not injure even a minute part of his feathers Their swords are as ineffective against him and his companions as against the wind or the water

(d) Give the explanatory meanings of the following expressions in the context of the above extract

(i)Never surfeited (ii) Belch up (iii) lsquomongst men

(i) Never surfeited never led to satisfaction

(ii) Belch up cast ashore(iii) lsquomongst men in human

society2

I and my fellows (Line 60-65)

Are ministers of Fate The elementsOf whom your swords are tempered may as wellWound the loud winds or with bemocked-at stabsKill the still-closing waters as diminishOne dowl thats in my plume

IMPORTANT PASSAGES EXPLAINED

The elements

(Line 61-66)Of whom your swords are tempered may

as wellWound the loud winds or with

bemocked-at stabs

(a) Who is lsquoIrsquo Who are his lsquofellowsrdquo

lsquoIrsquo is referred to Ariel in disguise of a harpy His lsquofellowsrsquo are other spirits serving Prospero the real Duke of Milan who has acquired supernatural powers after being banished from his Dukedom Prospero has settled in this uninhabited island

(b) What are the elements that have temperrsquod the swords Why will it not work against the speaker

The swords (of Alonso and his companions) are tempered by metal (steel) which is taken out of the earth and refined by

Kill the still-closing waters as diminishOne dowl thats in my plume My fellow

ministersAre like invulnerable

In these words Ariel reminds the King and his companions of the utter futility of drawing swords against himself and his fellows Ariel drives Alonso Antonio and Sebastian the three men of sin to desperation ndash a state in which men do violence to themselves They draw swords to strike Ariel But Ariel reminds them that he and the other spirits are the ministers of destiny and nothing can wound them The steel of which their swords are made of may cut the wind or water which being divided always closes up again Even supposing that such things may be possible it is quite impossible that their swords will cut one feather in their plume They are incapable of being wounded by any sword of man Hence it is foolish on their part to attempt to strike at Ariel and his fellow-spirits

For which foul deed

(Line 72-75)The powers delaying not forgetting

haveIncensed the seas and shores yea all the

creatures Against your peace

Ariel enters like a harpy and remaining invisible tells Alonso Sebastian and Antonio that he and other harpies are the agents of Destiny appointed to carry out her decrees He tells them that their punishment for the crime against Prospero which has been so long deferred is now to fall upon them He reminds them that they had expelled Prospero from Milan and set him and his innocent child adrift on the sea and that the sea had paid them back for their sin by the shipwreck and by the calamities they have suffered He tells them that the powers above which did not forget this mean treachery but only deferred the punishment have now engaged the seas and the shores and all living beings including him and his comrades against them The very elements and supernatural agency Ariel adds have taken up the avenging of their crime against Prospero

the action of fire It may cut the wind or water which being divided always closes up again

The sword will not work against the spirits and the harpy because they are the ministers of destiny and nothing can wound them nor it will cut a single feather in their plume

(c )What is the meaning of lsquodowlrsquo in the last line

The term lsquodowlrsquo means a filament or the smallest part of a feather In this context Ariel in disguise of harpy says that their sword cannot even damage the smallest filament of their (Arielrsquos and other spirits) feathers as they are incapable of being wounded by any sword of man

(d) What does the speaker remind the listeners about

Ariel in disguise of harpy reminds Alonso the King of Naples Sebastian Alonsorsquos brother and Antonio the present Duke of Milan and the treacherous brother of Prospero as they being three men of sin He even reminds them that their punishment for their crime against Prospero which has been so long deferred now falls upon them He reminds them that they have expelled Prospero from Milan and has set him along with his innocent infant daughter adrift on the sea So the sea has paid them back for their sin by their shipwreck and the calamities they have suffered since then The harpy rebukes Alonso of his sin that has incensed the Gods and has deprived him of his son as a punishment

(e) How do they respond

When Ariel in disguise of a harpy reminds Alonso Sebastian and Antonio of their past misdeeds and sin Alonso has a look of terror and confusion in his eyes He utters the words of sincere repentance wrung out of his conscience-stricken heart It appears to him that all the elements of nature the sea-waves the wind and the thunder proclaiming a loud voice in the name of Prospero and the crime Alonso has committed against him They are calling upon him to repent There is a deep storm raging in Alonsorsquos breast and the echoes of that storm are ringing in his ears like a clear note of wind-instrument A note of denunciation of Alonsorsquos crime leaves him much humbled and penitent and confirms his belief that his son is lost forever But Sebastian and Antonio shows some courage instead of repentance They wish to kill the spirits or devils if it appears

3

Of my instruction hast thou nothing bated (Line 85-93)

In what thou hast to say So with good life

And observation strange my meaner ministers

Their several kinds have done My high charms work

And these mine enemies are all knit upIn their distractions They now are in my

powerAnd in these fits I leave them while I visitYoung Ferdinand whom they suppose is

drownedAnd his and mine loved darling

Methought the billows spoke and (Line 96-99)

told me of itThe winds did sing it to me and the

thunderThat deep and dreadful organ-pipe

pronouncedThe name of Prosper It did bass my

trespass

These are the words of contrition coming from Alonso Ariel has driven him to a deep repentance for conspiring with Antonio against Prospero He now feels a sincere remorse It appears to him that all the elements of nature the sea-waves the wind and the thunder proclaimed with a loud voice the name of Prospero and the crime Alonso had committed against him They are calling upon him to repent There is a deep storm raging in Alonsorsquos breast and the echoes of that storm are ringing in his ears like the clear note of a wind-instrument

Comment These are the words of sincere repentance wrung out of the conscience-stricken heart of Alonso Alonso who is the lesser villain is the first to give way to remorse under the effect of Arielrsquos speech The words of Ariel seem to him to be the voice of conscience speaking to him He is driven to desperation a state in which he might do violence to his life

(a) Identify the speaker State the context

Prospero the ruler of the island is the speaker The famous banquet scene has been enacted very well Ariel and his junior spirits have played their roles excellently Prospero is glad to say words of praise for them(b) In what way the speakerrsquos instructions have been carried out

According to Prosperorsquos instructions a banquet was presented before the King of Naples and his companions when they were tired and hungry Just when they were preparing to eat the feast the banquet was suddenly removed by exercising supernatural powers All this was done by Ariel Prosperorsquos chief assistant and a powerful spirit

Ariel not only made the feast disappear but also delivered his speech blaming the King and his two companions for their past wicked deeds He warned them to repent for their misdeeds or suffer forever on that uninhabited island

(c) Who are referred to as lsquomeaner ministersrsquo What have they done

Prospero refers as lsquomeaner ministersrsquo to his other lesser spirits who were assisting Ariel in presenting a scene before the kingrsquos party They entered the scene to the accompaniment of music They assumed several strange shapes and brought in a banquet Then they danced about it with gentle actions of salutations thus inviting the King and others to eat the feast

These spirits play their role again when Ariel in the shape of a harpy quits the scene These shapes enter again and dancing with mocking gestures carry away the table

(d) Who are the speakerrsquos enemies What has happened to them

King of Naples Alonso his brother Sebastian and the present Duke of Milan Antonio (Prosperorsquos own brother) are Prosperorsquos enemies With the turn of events they have all been washed ashore on the island which is ruled by Prospero the great magician Actually this happened after the shipwreck caused by a storm which was raised by Prospero with the purpose of bringing these people to his island Prosperorsquos spirits have already confused and terrified these enemies and they are under Prosperorsquos control He can treat them as he likes

(e) What does he say about Ferdinand Explain what is meant by ldquohellip his and mine darlingrdquo

Prospero knows that Alonsorsquos son prince Ferdinand is alive though his father thinks that the prince has been drowned

Prospero refers to his daughter Miranda who is dear to him She is also very dear to Prince Ferdinand who has fallen in love with her They are waiting to be married soon for which they have received Prosperorsquos consent

4

ALONSO O it is monstrous monstrous (Line 95-102)

Methought the billows spoke and told me of it

The winds did sing it to me and the thunderThat deep and dreadful organ-

pipe pronouncedThe name of Prosper It did bass

my trespassTherefore my son ithrsquo ooze is

bedded andIll seek him deeper than eer

plummet soundedAnd with him there lie mudded

(a) In what way does Alonso express his horror when his conscience is awakened by Arielrsquos words

When Alonsorsquos conscience is awakened by Arielrsquos words he expresses his horror at what he has heard He gets the feeling that the waves of the ocean the wind and the loud thunder have spoken to him and uttered the name of Prospero Because of being reminded of his crime in a very loud and rough voice he comes to realize that he has lost his son for his past misdeeds

(b) What does Alonso imagine about his son What does Alonso want to do in his desperate state

Alonso imagines that his son is lying in the mud at the bottom of the sea He feels desperate that he wants to drown himself in the ocean deeper than the plumb-line has ever gone He wants to lie with his son at the bottom of the sea

(c) How do Sebastian and Antonio want to face the evil spirits

Sebastian says that he is not at all afraid of what the harpy has said and that he is prepared to fight any number of such monsters if they appear before him only one at a time Antonio says that he would support Sebastian in the fight against the fiendsyyy

(d) Why does Gonzalo ask Adrian to follow the three men

Gonzalo tells Adrian that all the three men namely Alonso Sebastian and Antonio are in a wild and reckless mood The thought of the heinous crime of which they are guilty has begun to torment their minds So he asks Adrian to follow those three men without loss of time and prevent them from doing anything which the turmoil in their minds might lead them to do

(e) What opinion do you form of Alonso from the above extract

Alonso who is the lesser villain is the first to give way to remorse under the effect of Arielrsquos speech The words of Ariel seem to him to be the voice of conscience speaking to him He is driven to desperation a state in which he might do violence to his life

Subject =Accounts

Ac-12 15420 topic-pL Appropriation ac

PROFIT AND LOSS APPROPRIATION ACCOUNT

MEANING AND PREPARATIONProfit and Loss Appropriation Account is merely an extension of the Profit and Loss Account of the firm The profit of the firm has to be distributed amongst the partners in their respective profit sharing ratio But before its distribution it needs to be adjusted All Adjustments like partnerrsquos salary partnerrsquos commission interest on capital interest on drawings etc are made in this account These adjustments will reduce the amount of profit for distribution This adjusted profit will be distributed amongst the partners in their profit sharing ratio To prepare it at first the balance of Profit and Loss Account is transferred to this account The journal entries for the preparation of Profit and Loss Appropriation Account are given below

1 for transfer of the balance of Profit and Loss Account to Profit and Loss Appropriation Account

(a) In case of Net Profit

Profit and Loss Ac helliphelliphelliphelliphellipDrTo Profit and Loss Appropriation Ac(Net Profit transferred to Profit and Loss Appropriation Ac)

(b)In case of Net Loss

Profit and Loss Appropriation Achelliphelliphellip DrTo Profit and Loss Ac(Net Loss transferred to Profit and Loss Appropriation Ac)

2 for Interest on Capital

For transferring on Interest on CapitalProfit and Loss Appropriation Achelliphelliphellip DrTo Interest on Capital Ac(Interest on capital transferred to Profit amp Loss Appropriation Ac)

3 for Interest on Drawings

For transferring Interest on Drawings Interest on Drawings Achelliphelliphelliphelliphelliphellip DrTo Profit and Loss Appropriation Ac(Interest on drawing transferred to Profit amp Loss Appropriation Ac)

4 For Partnerrsquos SalaryFor transfer of partnerrsquos SalaryProfit and Loss Appropriation Achelliphellip DrTo Salary Ac(Salary transferred to profit amp Loss Appropriation Ac)

5 For Partnerrsquos CommissionFor transferring commissionProfit and Loss Appropriation Achelliphelliphellip DrTo Commission Ac(Commission transferred to Profit and Loss Appropriation Ac)

6 For Transfer of agreed amount to General ReserveProfit and Loss Appropriation Ac helliphellipDrTo General Reserve Ac(Transfer to General Reserve)

7 for share of Profit or Loss appropriation(a) If ProfitProfit and Loss Appropriation Achelliphellip DrTo Partnerrsquos CapitalCurrent Ac(Profit transferred to capitalcurrent Ac)(b) If LossPartnerrsquos Capital Current Achelliphelliphelliphellip DrTo Profit and Loss Appropriation Ac(Loss transferred to capitalcurrent Ac)

THE FORMAT OF PROFIT AND LOSS APPROPRIATION

Profit and Loss Appropriation Account for the year endedhelliphelliphelliphellip

Particulars Amount Particulars Amount

To PL Ac (loss) By pL Ac (profit)

To Interest on capital BY Interest on drawings

To partner`s commission by Partner`s capital Ac ( loss)

To Partner`s salary To Interest on partner`s loan To General Reserve To Partner`s Capital AC (Profit)

Subject= Economics

MOVEMENT ALONG THE DEMAND CURVE (CHANGE IN QUANTITY DEMANDED)In law of demand you have already studied the inverse relationship between price and quantity demanded When quantity demanded of a commodity changes due to change in its price keeping other factors constant it is called change in quantity demanded It is graphically expressed as a movement along the same demand curve There can be either a downward movement or an upward movement along the same demand curve Upward movement along the same demand curve is called contraction of demand or decrease in quantity demanded and downward movement along the same demand curve is known as expansion of demand or increase in quantity demanded

Extention of demandd

price (rs)p A

B Extentionp1 d

Q Q1

Quantity demanded ( in units)

Contraction of demandd

p2 Ccontraction

p APrice (Rs)

d

Q2 Q

Quantity demanded (in units)

Explanation of movement of demand A fall in price from OP to OP1 leads to increase in quantity demanded from OQ to OQ1 (expansion of demand) resulting in a downward movement from point A to point B along the same demand curve DD When Price rises from OP to OP2 quantity demanded falls from OQ to OQ2 (contraction of demand) leading to an upward movement from point A to point C along the same demand curve DD

  • Activity Series of Metals
    • Drawbacks of Rutherfordrsquos model of atom
      • Electromagnetic radiations
      • Properties of electromagnetic radiations
      • Characteristics of electromagnetic radiations
        • Plancks Quantum Theory-
        • Photoelectric effect
          • Intext Questions
            • Logic gates
            • Digital systems are said to be constructed by using logic gates These gates are the AND OR NOT NAND NOR EXOR and EXNOR gates The basic operations are described below with the aid of truth tables
            • AND gate
            • Example
Page 52:  · Web viewSubject. Topic. Summary. Execution. English 1 . Chapter 1 naming words . Page 8. Write the names of these pictures:- Person:-1. father. 2.Firefighter 3.doctor 4 ...

reference to their crime in expelling Prospero from Milan They have not received any punishment for their deed earlier but the time for their punishment has arrived Upon Alonso it pronounces ldquolingering perdition worse than deathrdquo from which there is no remedy except through sincere repentance Ariel then vanishes in thunder and the shapes enter again and carry away the table

o Prospero watching invisibly is very pleased with the performance of Ariel and his (Prosperorsquos) ldquomeaner ministersrdquo All his enemies are now in his power and are in a fit of desperation He then leaves them and goes to see how Ferdinand and Miranda are getting on

o Alonso is now much humbled and penitent with the after effect of the spiritrsquos denunciation of his crimes He believes that his son is lost forever After this all disperse being stricken mad by the speech of the spirit

o Gonzalo fearing that they may do violence to themselves or to one another follows them and bid others to follow

(a) To whom does Ariel disguised as a harpy call the three sinners What game did Fate of Destiny play with

them

The three sinners called by Ariel are Alonso Sebastian and Antonio It was Destiny which had caused the ocean to cast the three sinners on the shore Though the ocean is all the time devouring whatever appears on its surface and is never satisfied with its continual swallowing of the ships and men in the present case the ocean had cast these three sinners on the shore without killing them

(b) Who had jointly been responsible for the conspiracy against Prospero What is Prosperorsquos purpose behind all this

Three men Alonso Sebastian and Antonio had jointly

been responsible for the conspiracy against Prospero They had driven out Prospero form Milan Prosperorsquos purpose is to make these three sinners realize the wrong they had done He wants them to repent for their criminal deeds because repentance leads to self-esteem(c )What does Ariel (the harpy) tell Alonso and his companions when they take out their swords to attack him

Seeing them drawing their swords Ariel (harpy) tells them that he and his companions are the instruments of destiny and that it is not possible for human beings to do them any injury He says that the swords of human beings can not injure even a minute part of his feathers Their swords are as ineffective against him and his companions as against the wind or the water

(d) Give the explanatory meanings of the following expressions in the context of the above extract

(i)Never surfeited (ii) Belch up (iii) lsquomongst men

(i) Never surfeited never led to satisfaction

(ii) Belch up cast ashore(iii) lsquomongst men in human

society2

I and my fellows (Line 60-65)

Are ministers of Fate The elementsOf whom your swords are tempered may as wellWound the loud winds or with bemocked-at stabsKill the still-closing waters as diminishOne dowl thats in my plume

IMPORTANT PASSAGES EXPLAINED

The elements

(Line 61-66)Of whom your swords are tempered may

as wellWound the loud winds or with

bemocked-at stabs

(a) Who is lsquoIrsquo Who are his lsquofellowsrdquo

lsquoIrsquo is referred to Ariel in disguise of a harpy His lsquofellowsrsquo are other spirits serving Prospero the real Duke of Milan who has acquired supernatural powers after being banished from his Dukedom Prospero has settled in this uninhabited island

(b) What are the elements that have temperrsquod the swords Why will it not work against the speaker

The swords (of Alonso and his companions) are tempered by metal (steel) which is taken out of the earth and refined by

Kill the still-closing waters as diminishOne dowl thats in my plume My fellow

ministersAre like invulnerable

In these words Ariel reminds the King and his companions of the utter futility of drawing swords against himself and his fellows Ariel drives Alonso Antonio and Sebastian the three men of sin to desperation ndash a state in which men do violence to themselves They draw swords to strike Ariel But Ariel reminds them that he and the other spirits are the ministers of destiny and nothing can wound them The steel of which their swords are made of may cut the wind or water which being divided always closes up again Even supposing that such things may be possible it is quite impossible that their swords will cut one feather in their plume They are incapable of being wounded by any sword of man Hence it is foolish on their part to attempt to strike at Ariel and his fellow-spirits

For which foul deed

(Line 72-75)The powers delaying not forgetting

haveIncensed the seas and shores yea all the

creatures Against your peace

Ariel enters like a harpy and remaining invisible tells Alonso Sebastian and Antonio that he and other harpies are the agents of Destiny appointed to carry out her decrees He tells them that their punishment for the crime against Prospero which has been so long deferred is now to fall upon them He reminds them that they had expelled Prospero from Milan and set him and his innocent child adrift on the sea and that the sea had paid them back for their sin by the shipwreck and by the calamities they have suffered He tells them that the powers above which did not forget this mean treachery but only deferred the punishment have now engaged the seas and the shores and all living beings including him and his comrades against them The very elements and supernatural agency Ariel adds have taken up the avenging of their crime against Prospero

the action of fire It may cut the wind or water which being divided always closes up again

The sword will not work against the spirits and the harpy because they are the ministers of destiny and nothing can wound them nor it will cut a single feather in their plume

(c )What is the meaning of lsquodowlrsquo in the last line

The term lsquodowlrsquo means a filament or the smallest part of a feather In this context Ariel in disguise of harpy says that their sword cannot even damage the smallest filament of their (Arielrsquos and other spirits) feathers as they are incapable of being wounded by any sword of man

(d) What does the speaker remind the listeners about

Ariel in disguise of harpy reminds Alonso the King of Naples Sebastian Alonsorsquos brother and Antonio the present Duke of Milan and the treacherous brother of Prospero as they being three men of sin He even reminds them that their punishment for their crime against Prospero which has been so long deferred now falls upon them He reminds them that they have expelled Prospero from Milan and has set him along with his innocent infant daughter adrift on the sea So the sea has paid them back for their sin by their shipwreck and the calamities they have suffered since then The harpy rebukes Alonso of his sin that has incensed the Gods and has deprived him of his son as a punishment

(e) How do they respond

When Ariel in disguise of a harpy reminds Alonso Sebastian and Antonio of their past misdeeds and sin Alonso has a look of terror and confusion in his eyes He utters the words of sincere repentance wrung out of his conscience-stricken heart It appears to him that all the elements of nature the sea-waves the wind and the thunder proclaiming a loud voice in the name of Prospero and the crime Alonso has committed against him They are calling upon him to repent There is a deep storm raging in Alonsorsquos breast and the echoes of that storm are ringing in his ears like a clear note of wind-instrument A note of denunciation of Alonsorsquos crime leaves him much humbled and penitent and confirms his belief that his son is lost forever But Sebastian and Antonio shows some courage instead of repentance They wish to kill the spirits or devils if it appears

3

Of my instruction hast thou nothing bated (Line 85-93)

In what thou hast to say So with good life

And observation strange my meaner ministers

Their several kinds have done My high charms work

And these mine enemies are all knit upIn their distractions They now are in my

powerAnd in these fits I leave them while I visitYoung Ferdinand whom they suppose is

drownedAnd his and mine loved darling

Methought the billows spoke and (Line 96-99)

told me of itThe winds did sing it to me and the

thunderThat deep and dreadful organ-pipe

pronouncedThe name of Prosper It did bass my

trespass

These are the words of contrition coming from Alonso Ariel has driven him to a deep repentance for conspiring with Antonio against Prospero He now feels a sincere remorse It appears to him that all the elements of nature the sea-waves the wind and the thunder proclaimed with a loud voice the name of Prospero and the crime Alonso had committed against him They are calling upon him to repent There is a deep storm raging in Alonsorsquos breast and the echoes of that storm are ringing in his ears like the clear note of a wind-instrument

Comment These are the words of sincere repentance wrung out of the conscience-stricken heart of Alonso Alonso who is the lesser villain is the first to give way to remorse under the effect of Arielrsquos speech The words of Ariel seem to him to be the voice of conscience speaking to him He is driven to desperation a state in which he might do violence to his life

(a) Identify the speaker State the context

Prospero the ruler of the island is the speaker The famous banquet scene has been enacted very well Ariel and his junior spirits have played their roles excellently Prospero is glad to say words of praise for them(b) In what way the speakerrsquos instructions have been carried out

According to Prosperorsquos instructions a banquet was presented before the King of Naples and his companions when they were tired and hungry Just when they were preparing to eat the feast the banquet was suddenly removed by exercising supernatural powers All this was done by Ariel Prosperorsquos chief assistant and a powerful spirit

Ariel not only made the feast disappear but also delivered his speech blaming the King and his two companions for their past wicked deeds He warned them to repent for their misdeeds or suffer forever on that uninhabited island

(c) Who are referred to as lsquomeaner ministersrsquo What have they done

Prospero refers as lsquomeaner ministersrsquo to his other lesser spirits who were assisting Ariel in presenting a scene before the kingrsquos party They entered the scene to the accompaniment of music They assumed several strange shapes and brought in a banquet Then they danced about it with gentle actions of salutations thus inviting the King and others to eat the feast

These spirits play their role again when Ariel in the shape of a harpy quits the scene These shapes enter again and dancing with mocking gestures carry away the table

(d) Who are the speakerrsquos enemies What has happened to them

King of Naples Alonso his brother Sebastian and the present Duke of Milan Antonio (Prosperorsquos own brother) are Prosperorsquos enemies With the turn of events they have all been washed ashore on the island which is ruled by Prospero the great magician Actually this happened after the shipwreck caused by a storm which was raised by Prospero with the purpose of bringing these people to his island Prosperorsquos spirits have already confused and terrified these enemies and they are under Prosperorsquos control He can treat them as he likes

(e) What does he say about Ferdinand Explain what is meant by ldquohellip his and mine darlingrdquo

Prospero knows that Alonsorsquos son prince Ferdinand is alive though his father thinks that the prince has been drowned

Prospero refers to his daughter Miranda who is dear to him She is also very dear to Prince Ferdinand who has fallen in love with her They are waiting to be married soon for which they have received Prosperorsquos consent

4

ALONSO O it is monstrous monstrous (Line 95-102)

Methought the billows spoke and told me of it

The winds did sing it to me and the thunderThat deep and dreadful organ-

pipe pronouncedThe name of Prosper It did bass

my trespassTherefore my son ithrsquo ooze is

bedded andIll seek him deeper than eer

plummet soundedAnd with him there lie mudded

(a) In what way does Alonso express his horror when his conscience is awakened by Arielrsquos words

When Alonsorsquos conscience is awakened by Arielrsquos words he expresses his horror at what he has heard He gets the feeling that the waves of the ocean the wind and the loud thunder have spoken to him and uttered the name of Prospero Because of being reminded of his crime in a very loud and rough voice he comes to realize that he has lost his son for his past misdeeds

(b) What does Alonso imagine about his son What does Alonso want to do in his desperate state

Alonso imagines that his son is lying in the mud at the bottom of the sea He feels desperate that he wants to drown himself in the ocean deeper than the plumb-line has ever gone He wants to lie with his son at the bottom of the sea

(c) How do Sebastian and Antonio want to face the evil spirits

Sebastian says that he is not at all afraid of what the harpy has said and that he is prepared to fight any number of such monsters if they appear before him only one at a time Antonio says that he would support Sebastian in the fight against the fiendsyyy

(d) Why does Gonzalo ask Adrian to follow the three men

Gonzalo tells Adrian that all the three men namely Alonso Sebastian and Antonio are in a wild and reckless mood The thought of the heinous crime of which they are guilty has begun to torment their minds So he asks Adrian to follow those three men without loss of time and prevent them from doing anything which the turmoil in their minds might lead them to do

(e) What opinion do you form of Alonso from the above extract

Alonso who is the lesser villain is the first to give way to remorse under the effect of Arielrsquos speech The words of Ariel seem to him to be the voice of conscience speaking to him He is driven to desperation a state in which he might do violence to his life

Subject =Accounts

Ac-12 15420 topic-pL Appropriation ac

PROFIT AND LOSS APPROPRIATION ACCOUNT

MEANING AND PREPARATIONProfit and Loss Appropriation Account is merely an extension of the Profit and Loss Account of the firm The profit of the firm has to be distributed amongst the partners in their respective profit sharing ratio But before its distribution it needs to be adjusted All Adjustments like partnerrsquos salary partnerrsquos commission interest on capital interest on drawings etc are made in this account These adjustments will reduce the amount of profit for distribution This adjusted profit will be distributed amongst the partners in their profit sharing ratio To prepare it at first the balance of Profit and Loss Account is transferred to this account The journal entries for the preparation of Profit and Loss Appropriation Account are given below

1 for transfer of the balance of Profit and Loss Account to Profit and Loss Appropriation Account

(a) In case of Net Profit

Profit and Loss Ac helliphelliphelliphelliphellipDrTo Profit and Loss Appropriation Ac(Net Profit transferred to Profit and Loss Appropriation Ac)

(b)In case of Net Loss

Profit and Loss Appropriation Achelliphelliphellip DrTo Profit and Loss Ac(Net Loss transferred to Profit and Loss Appropriation Ac)

2 for Interest on Capital

For transferring on Interest on CapitalProfit and Loss Appropriation Achelliphelliphellip DrTo Interest on Capital Ac(Interest on capital transferred to Profit amp Loss Appropriation Ac)

3 for Interest on Drawings

For transferring Interest on Drawings Interest on Drawings Achelliphelliphelliphelliphelliphellip DrTo Profit and Loss Appropriation Ac(Interest on drawing transferred to Profit amp Loss Appropriation Ac)

4 For Partnerrsquos SalaryFor transfer of partnerrsquos SalaryProfit and Loss Appropriation Achelliphellip DrTo Salary Ac(Salary transferred to profit amp Loss Appropriation Ac)

5 For Partnerrsquos CommissionFor transferring commissionProfit and Loss Appropriation Achelliphelliphellip DrTo Commission Ac(Commission transferred to Profit and Loss Appropriation Ac)

6 For Transfer of agreed amount to General ReserveProfit and Loss Appropriation Ac helliphellipDrTo General Reserve Ac(Transfer to General Reserve)

7 for share of Profit or Loss appropriation(a) If ProfitProfit and Loss Appropriation Achelliphellip DrTo Partnerrsquos CapitalCurrent Ac(Profit transferred to capitalcurrent Ac)(b) If LossPartnerrsquos Capital Current Achelliphelliphelliphellip DrTo Profit and Loss Appropriation Ac(Loss transferred to capitalcurrent Ac)

THE FORMAT OF PROFIT AND LOSS APPROPRIATION

Profit and Loss Appropriation Account for the year endedhelliphelliphelliphellip

Particulars Amount Particulars Amount

To PL Ac (loss) By pL Ac (profit)

To Interest on capital BY Interest on drawings

To partner`s commission by Partner`s capital Ac ( loss)

To Partner`s salary To Interest on partner`s loan To General Reserve To Partner`s Capital AC (Profit)

Subject= Economics

MOVEMENT ALONG THE DEMAND CURVE (CHANGE IN QUANTITY DEMANDED)In law of demand you have already studied the inverse relationship between price and quantity demanded When quantity demanded of a commodity changes due to change in its price keeping other factors constant it is called change in quantity demanded It is graphically expressed as a movement along the same demand curve There can be either a downward movement or an upward movement along the same demand curve Upward movement along the same demand curve is called contraction of demand or decrease in quantity demanded and downward movement along the same demand curve is known as expansion of demand or increase in quantity demanded

Extention of demandd

price (rs)p A

B Extentionp1 d

Q Q1

Quantity demanded ( in units)

Contraction of demandd

p2 Ccontraction

p APrice (Rs)

d

Q2 Q

Quantity demanded (in units)

Explanation of movement of demand A fall in price from OP to OP1 leads to increase in quantity demanded from OQ to OQ1 (expansion of demand) resulting in a downward movement from point A to point B along the same demand curve DD When Price rises from OP to OP2 quantity demanded falls from OQ to OQ2 (contraction of demand) leading to an upward movement from point A to point C along the same demand curve DD

  • Activity Series of Metals
    • Drawbacks of Rutherfordrsquos model of atom
      • Electromagnetic radiations
      • Properties of electromagnetic radiations
      • Characteristics of electromagnetic radiations
        • Plancks Quantum Theory-
        • Photoelectric effect
          • Intext Questions
            • Logic gates
            • Digital systems are said to be constructed by using logic gates These gates are the AND OR NOT NAND NOR EXOR and EXNOR gates The basic operations are described below with the aid of truth tables
            • AND gate
            • Example
Page 53:  · Web viewSubject. Topic. Summary. Execution. English 1 . Chapter 1 naming words . Page 8. Write the names of these pictures:- Person:-1. father. 2.Firefighter 3.doctor 4 ...

Kill the still-closing waters as diminishOne dowl thats in my plume My fellow

ministersAre like invulnerable

In these words Ariel reminds the King and his companions of the utter futility of drawing swords against himself and his fellows Ariel drives Alonso Antonio and Sebastian the three men of sin to desperation ndash a state in which men do violence to themselves They draw swords to strike Ariel But Ariel reminds them that he and the other spirits are the ministers of destiny and nothing can wound them The steel of which their swords are made of may cut the wind or water which being divided always closes up again Even supposing that such things may be possible it is quite impossible that their swords will cut one feather in their plume They are incapable of being wounded by any sword of man Hence it is foolish on their part to attempt to strike at Ariel and his fellow-spirits

For which foul deed

(Line 72-75)The powers delaying not forgetting

haveIncensed the seas and shores yea all the

creatures Against your peace

Ariel enters like a harpy and remaining invisible tells Alonso Sebastian and Antonio that he and other harpies are the agents of Destiny appointed to carry out her decrees He tells them that their punishment for the crime against Prospero which has been so long deferred is now to fall upon them He reminds them that they had expelled Prospero from Milan and set him and his innocent child adrift on the sea and that the sea had paid them back for their sin by the shipwreck and by the calamities they have suffered He tells them that the powers above which did not forget this mean treachery but only deferred the punishment have now engaged the seas and the shores and all living beings including him and his comrades against them The very elements and supernatural agency Ariel adds have taken up the avenging of their crime against Prospero

the action of fire It may cut the wind or water which being divided always closes up again

The sword will not work against the spirits and the harpy because they are the ministers of destiny and nothing can wound them nor it will cut a single feather in their plume

(c )What is the meaning of lsquodowlrsquo in the last line

The term lsquodowlrsquo means a filament or the smallest part of a feather In this context Ariel in disguise of harpy says that their sword cannot even damage the smallest filament of their (Arielrsquos and other spirits) feathers as they are incapable of being wounded by any sword of man

(d) What does the speaker remind the listeners about

Ariel in disguise of harpy reminds Alonso the King of Naples Sebastian Alonsorsquos brother and Antonio the present Duke of Milan and the treacherous brother of Prospero as they being three men of sin He even reminds them that their punishment for their crime against Prospero which has been so long deferred now falls upon them He reminds them that they have expelled Prospero from Milan and has set him along with his innocent infant daughter adrift on the sea So the sea has paid them back for their sin by their shipwreck and the calamities they have suffered since then The harpy rebukes Alonso of his sin that has incensed the Gods and has deprived him of his son as a punishment

(e) How do they respond

When Ariel in disguise of a harpy reminds Alonso Sebastian and Antonio of their past misdeeds and sin Alonso has a look of terror and confusion in his eyes He utters the words of sincere repentance wrung out of his conscience-stricken heart It appears to him that all the elements of nature the sea-waves the wind and the thunder proclaiming a loud voice in the name of Prospero and the crime Alonso has committed against him They are calling upon him to repent There is a deep storm raging in Alonsorsquos breast and the echoes of that storm are ringing in his ears like a clear note of wind-instrument A note of denunciation of Alonsorsquos crime leaves him much humbled and penitent and confirms his belief that his son is lost forever But Sebastian and Antonio shows some courage instead of repentance They wish to kill the spirits or devils if it appears

3

Of my instruction hast thou nothing bated (Line 85-93)

In what thou hast to say So with good life

And observation strange my meaner ministers

Their several kinds have done My high charms work

And these mine enemies are all knit upIn their distractions They now are in my

powerAnd in these fits I leave them while I visitYoung Ferdinand whom they suppose is

drownedAnd his and mine loved darling

Methought the billows spoke and (Line 96-99)

told me of itThe winds did sing it to me and the

thunderThat deep and dreadful organ-pipe

pronouncedThe name of Prosper It did bass my

trespass

These are the words of contrition coming from Alonso Ariel has driven him to a deep repentance for conspiring with Antonio against Prospero He now feels a sincere remorse It appears to him that all the elements of nature the sea-waves the wind and the thunder proclaimed with a loud voice the name of Prospero and the crime Alonso had committed against him They are calling upon him to repent There is a deep storm raging in Alonsorsquos breast and the echoes of that storm are ringing in his ears like the clear note of a wind-instrument

Comment These are the words of sincere repentance wrung out of the conscience-stricken heart of Alonso Alonso who is the lesser villain is the first to give way to remorse under the effect of Arielrsquos speech The words of Ariel seem to him to be the voice of conscience speaking to him He is driven to desperation a state in which he might do violence to his life

(a) Identify the speaker State the context

Prospero the ruler of the island is the speaker The famous banquet scene has been enacted very well Ariel and his junior spirits have played their roles excellently Prospero is glad to say words of praise for them(b) In what way the speakerrsquos instructions have been carried out

According to Prosperorsquos instructions a banquet was presented before the King of Naples and his companions when they were tired and hungry Just when they were preparing to eat the feast the banquet was suddenly removed by exercising supernatural powers All this was done by Ariel Prosperorsquos chief assistant and a powerful spirit

Ariel not only made the feast disappear but also delivered his speech blaming the King and his two companions for their past wicked deeds He warned them to repent for their misdeeds or suffer forever on that uninhabited island

(c) Who are referred to as lsquomeaner ministersrsquo What have they done

Prospero refers as lsquomeaner ministersrsquo to his other lesser spirits who were assisting Ariel in presenting a scene before the kingrsquos party They entered the scene to the accompaniment of music They assumed several strange shapes and brought in a banquet Then they danced about it with gentle actions of salutations thus inviting the King and others to eat the feast

These spirits play their role again when Ariel in the shape of a harpy quits the scene These shapes enter again and dancing with mocking gestures carry away the table

(d) Who are the speakerrsquos enemies What has happened to them

King of Naples Alonso his brother Sebastian and the present Duke of Milan Antonio (Prosperorsquos own brother) are Prosperorsquos enemies With the turn of events they have all been washed ashore on the island which is ruled by Prospero the great magician Actually this happened after the shipwreck caused by a storm which was raised by Prospero with the purpose of bringing these people to his island Prosperorsquos spirits have already confused and terrified these enemies and they are under Prosperorsquos control He can treat them as he likes

(e) What does he say about Ferdinand Explain what is meant by ldquohellip his and mine darlingrdquo

Prospero knows that Alonsorsquos son prince Ferdinand is alive though his father thinks that the prince has been drowned

Prospero refers to his daughter Miranda who is dear to him She is also very dear to Prince Ferdinand who has fallen in love with her They are waiting to be married soon for which they have received Prosperorsquos consent

4

ALONSO O it is monstrous monstrous (Line 95-102)

Methought the billows spoke and told me of it

The winds did sing it to me and the thunderThat deep and dreadful organ-

pipe pronouncedThe name of Prosper It did bass

my trespassTherefore my son ithrsquo ooze is

bedded andIll seek him deeper than eer

plummet soundedAnd with him there lie mudded

(a) In what way does Alonso express his horror when his conscience is awakened by Arielrsquos words

When Alonsorsquos conscience is awakened by Arielrsquos words he expresses his horror at what he has heard He gets the feeling that the waves of the ocean the wind and the loud thunder have spoken to him and uttered the name of Prospero Because of being reminded of his crime in a very loud and rough voice he comes to realize that he has lost his son for his past misdeeds

(b) What does Alonso imagine about his son What does Alonso want to do in his desperate state

Alonso imagines that his son is lying in the mud at the bottom of the sea He feels desperate that he wants to drown himself in the ocean deeper than the plumb-line has ever gone He wants to lie with his son at the bottom of the sea

(c) How do Sebastian and Antonio want to face the evil spirits

Sebastian says that he is not at all afraid of what the harpy has said and that he is prepared to fight any number of such monsters if they appear before him only one at a time Antonio says that he would support Sebastian in the fight against the fiendsyyy

(d) Why does Gonzalo ask Adrian to follow the three men

Gonzalo tells Adrian that all the three men namely Alonso Sebastian and Antonio are in a wild and reckless mood The thought of the heinous crime of which they are guilty has begun to torment their minds So he asks Adrian to follow those three men without loss of time and prevent them from doing anything which the turmoil in their minds might lead them to do

(e) What opinion do you form of Alonso from the above extract

Alonso who is the lesser villain is the first to give way to remorse under the effect of Arielrsquos speech The words of Ariel seem to him to be the voice of conscience speaking to him He is driven to desperation a state in which he might do violence to his life

Subject =Accounts

Ac-12 15420 topic-pL Appropriation ac

PROFIT AND LOSS APPROPRIATION ACCOUNT

MEANING AND PREPARATIONProfit and Loss Appropriation Account is merely an extension of the Profit and Loss Account of the firm The profit of the firm has to be distributed amongst the partners in their respective profit sharing ratio But before its distribution it needs to be adjusted All Adjustments like partnerrsquos salary partnerrsquos commission interest on capital interest on drawings etc are made in this account These adjustments will reduce the amount of profit for distribution This adjusted profit will be distributed amongst the partners in their profit sharing ratio To prepare it at first the balance of Profit and Loss Account is transferred to this account The journal entries for the preparation of Profit and Loss Appropriation Account are given below

1 for transfer of the balance of Profit and Loss Account to Profit and Loss Appropriation Account

(a) In case of Net Profit

Profit and Loss Ac helliphelliphelliphelliphellipDrTo Profit and Loss Appropriation Ac(Net Profit transferred to Profit and Loss Appropriation Ac)

(b)In case of Net Loss

Profit and Loss Appropriation Achelliphelliphellip DrTo Profit and Loss Ac(Net Loss transferred to Profit and Loss Appropriation Ac)

2 for Interest on Capital

For transferring on Interest on CapitalProfit and Loss Appropriation Achelliphelliphellip DrTo Interest on Capital Ac(Interest on capital transferred to Profit amp Loss Appropriation Ac)

3 for Interest on Drawings

For transferring Interest on Drawings Interest on Drawings Achelliphelliphelliphelliphelliphellip DrTo Profit and Loss Appropriation Ac(Interest on drawing transferred to Profit amp Loss Appropriation Ac)

4 For Partnerrsquos SalaryFor transfer of partnerrsquos SalaryProfit and Loss Appropriation Achelliphellip DrTo Salary Ac(Salary transferred to profit amp Loss Appropriation Ac)

5 For Partnerrsquos CommissionFor transferring commissionProfit and Loss Appropriation Achelliphelliphellip DrTo Commission Ac(Commission transferred to Profit and Loss Appropriation Ac)

6 For Transfer of agreed amount to General ReserveProfit and Loss Appropriation Ac helliphellipDrTo General Reserve Ac(Transfer to General Reserve)

7 for share of Profit or Loss appropriation(a) If ProfitProfit and Loss Appropriation Achelliphellip DrTo Partnerrsquos CapitalCurrent Ac(Profit transferred to capitalcurrent Ac)(b) If LossPartnerrsquos Capital Current Achelliphelliphelliphellip DrTo Profit and Loss Appropriation Ac(Loss transferred to capitalcurrent Ac)

THE FORMAT OF PROFIT AND LOSS APPROPRIATION

Profit and Loss Appropriation Account for the year endedhelliphelliphelliphellip

Particulars Amount Particulars Amount

To PL Ac (loss) By pL Ac (profit)

To Interest on capital BY Interest on drawings

To partner`s commission by Partner`s capital Ac ( loss)

To Partner`s salary To Interest on partner`s loan To General Reserve To Partner`s Capital AC (Profit)

Subject= Economics

MOVEMENT ALONG THE DEMAND CURVE (CHANGE IN QUANTITY DEMANDED)In law of demand you have already studied the inverse relationship between price and quantity demanded When quantity demanded of a commodity changes due to change in its price keeping other factors constant it is called change in quantity demanded It is graphically expressed as a movement along the same demand curve There can be either a downward movement or an upward movement along the same demand curve Upward movement along the same demand curve is called contraction of demand or decrease in quantity demanded and downward movement along the same demand curve is known as expansion of demand or increase in quantity demanded

Extention of demandd

price (rs)p A

B Extentionp1 d

Q Q1

Quantity demanded ( in units)

Contraction of demandd

p2 Ccontraction

p APrice (Rs)

d

Q2 Q

Quantity demanded (in units)

Explanation of movement of demand A fall in price from OP to OP1 leads to increase in quantity demanded from OQ to OQ1 (expansion of demand) resulting in a downward movement from point A to point B along the same demand curve DD When Price rises from OP to OP2 quantity demanded falls from OQ to OQ2 (contraction of demand) leading to an upward movement from point A to point C along the same demand curve DD

  • Activity Series of Metals
    • Drawbacks of Rutherfordrsquos model of atom
      • Electromagnetic radiations
      • Properties of electromagnetic radiations
      • Characteristics of electromagnetic radiations
        • Plancks Quantum Theory-
        • Photoelectric effect
          • Intext Questions
            • Logic gates
            • Digital systems are said to be constructed by using logic gates These gates are the AND OR NOT NAND NOR EXOR and EXNOR gates The basic operations are described below with the aid of truth tables
            • AND gate
            • Example
Page 54:  · Web viewSubject. Topic. Summary. Execution. English 1 . Chapter 1 naming words . Page 8. Write the names of these pictures:- Person:-1. father. 2.Firefighter 3.doctor 4 ...

Methought the billows spoke and (Line 96-99)

told me of itThe winds did sing it to me and the

thunderThat deep and dreadful organ-pipe

pronouncedThe name of Prosper It did bass my

trespass

These are the words of contrition coming from Alonso Ariel has driven him to a deep repentance for conspiring with Antonio against Prospero He now feels a sincere remorse It appears to him that all the elements of nature the sea-waves the wind and the thunder proclaimed with a loud voice the name of Prospero and the crime Alonso had committed against him They are calling upon him to repent There is a deep storm raging in Alonsorsquos breast and the echoes of that storm are ringing in his ears like the clear note of a wind-instrument

Comment These are the words of sincere repentance wrung out of the conscience-stricken heart of Alonso Alonso who is the lesser villain is the first to give way to remorse under the effect of Arielrsquos speech The words of Ariel seem to him to be the voice of conscience speaking to him He is driven to desperation a state in which he might do violence to his life

(a) Identify the speaker State the context

Prospero the ruler of the island is the speaker The famous banquet scene has been enacted very well Ariel and his junior spirits have played their roles excellently Prospero is glad to say words of praise for them(b) In what way the speakerrsquos instructions have been carried out

According to Prosperorsquos instructions a banquet was presented before the King of Naples and his companions when they were tired and hungry Just when they were preparing to eat the feast the banquet was suddenly removed by exercising supernatural powers All this was done by Ariel Prosperorsquos chief assistant and a powerful spirit

Ariel not only made the feast disappear but also delivered his speech blaming the King and his two companions for their past wicked deeds He warned them to repent for their misdeeds or suffer forever on that uninhabited island

(c) Who are referred to as lsquomeaner ministersrsquo What have they done

Prospero refers as lsquomeaner ministersrsquo to his other lesser spirits who were assisting Ariel in presenting a scene before the kingrsquos party They entered the scene to the accompaniment of music They assumed several strange shapes and brought in a banquet Then they danced about it with gentle actions of salutations thus inviting the King and others to eat the feast

These spirits play their role again when Ariel in the shape of a harpy quits the scene These shapes enter again and dancing with mocking gestures carry away the table

(d) Who are the speakerrsquos enemies What has happened to them

King of Naples Alonso his brother Sebastian and the present Duke of Milan Antonio (Prosperorsquos own brother) are Prosperorsquos enemies With the turn of events they have all been washed ashore on the island which is ruled by Prospero the great magician Actually this happened after the shipwreck caused by a storm which was raised by Prospero with the purpose of bringing these people to his island Prosperorsquos spirits have already confused and terrified these enemies and they are under Prosperorsquos control He can treat them as he likes

(e) What does he say about Ferdinand Explain what is meant by ldquohellip his and mine darlingrdquo

Prospero knows that Alonsorsquos son prince Ferdinand is alive though his father thinks that the prince has been drowned

Prospero refers to his daughter Miranda who is dear to him She is also very dear to Prince Ferdinand who has fallen in love with her They are waiting to be married soon for which they have received Prosperorsquos consent

4

ALONSO O it is monstrous monstrous (Line 95-102)

Methought the billows spoke and told me of it

The winds did sing it to me and the thunderThat deep and dreadful organ-

pipe pronouncedThe name of Prosper It did bass

my trespassTherefore my son ithrsquo ooze is

bedded andIll seek him deeper than eer

plummet soundedAnd with him there lie mudded

(a) In what way does Alonso express his horror when his conscience is awakened by Arielrsquos words

When Alonsorsquos conscience is awakened by Arielrsquos words he expresses his horror at what he has heard He gets the feeling that the waves of the ocean the wind and the loud thunder have spoken to him and uttered the name of Prospero Because of being reminded of his crime in a very loud and rough voice he comes to realize that he has lost his son for his past misdeeds

(b) What does Alonso imagine about his son What does Alonso want to do in his desperate state

Alonso imagines that his son is lying in the mud at the bottom of the sea He feels desperate that he wants to drown himself in the ocean deeper than the plumb-line has ever gone He wants to lie with his son at the bottom of the sea

(c) How do Sebastian and Antonio want to face the evil spirits

Sebastian says that he is not at all afraid of what the harpy has said and that he is prepared to fight any number of such monsters if they appear before him only one at a time Antonio says that he would support Sebastian in the fight against the fiendsyyy

(d) Why does Gonzalo ask Adrian to follow the three men

Gonzalo tells Adrian that all the three men namely Alonso Sebastian and Antonio are in a wild and reckless mood The thought of the heinous crime of which they are guilty has begun to torment their minds So he asks Adrian to follow those three men without loss of time and prevent them from doing anything which the turmoil in their minds might lead them to do

(e) What opinion do you form of Alonso from the above extract

Alonso who is the lesser villain is the first to give way to remorse under the effect of Arielrsquos speech The words of Ariel seem to him to be the voice of conscience speaking to him He is driven to desperation a state in which he might do violence to his life

Subject =Accounts

Ac-12 15420 topic-pL Appropriation ac

PROFIT AND LOSS APPROPRIATION ACCOUNT

MEANING AND PREPARATIONProfit and Loss Appropriation Account is merely an extension of the Profit and Loss Account of the firm The profit of the firm has to be distributed amongst the partners in their respective profit sharing ratio But before its distribution it needs to be adjusted All Adjustments like partnerrsquos salary partnerrsquos commission interest on capital interest on drawings etc are made in this account These adjustments will reduce the amount of profit for distribution This adjusted profit will be distributed amongst the partners in their profit sharing ratio To prepare it at first the balance of Profit and Loss Account is transferred to this account The journal entries for the preparation of Profit and Loss Appropriation Account are given below

1 for transfer of the balance of Profit and Loss Account to Profit and Loss Appropriation Account

(a) In case of Net Profit

Profit and Loss Ac helliphelliphelliphelliphellipDrTo Profit and Loss Appropriation Ac(Net Profit transferred to Profit and Loss Appropriation Ac)

(b)In case of Net Loss

Profit and Loss Appropriation Achelliphelliphellip DrTo Profit and Loss Ac(Net Loss transferred to Profit and Loss Appropriation Ac)

2 for Interest on Capital

For transferring on Interest on CapitalProfit and Loss Appropriation Achelliphelliphellip DrTo Interest on Capital Ac(Interest on capital transferred to Profit amp Loss Appropriation Ac)

3 for Interest on Drawings

For transferring Interest on Drawings Interest on Drawings Achelliphelliphelliphelliphelliphellip DrTo Profit and Loss Appropriation Ac(Interest on drawing transferred to Profit amp Loss Appropriation Ac)

4 For Partnerrsquos SalaryFor transfer of partnerrsquos SalaryProfit and Loss Appropriation Achelliphellip DrTo Salary Ac(Salary transferred to profit amp Loss Appropriation Ac)

5 For Partnerrsquos CommissionFor transferring commissionProfit and Loss Appropriation Achelliphelliphellip DrTo Commission Ac(Commission transferred to Profit and Loss Appropriation Ac)

6 For Transfer of agreed amount to General ReserveProfit and Loss Appropriation Ac helliphellipDrTo General Reserve Ac(Transfer to General Reserve)

7 for share of Profit or Loss appropriation(a) If ProfitProfit and Loss Appropriation Achelliphellip DrTo Partnerrsquos CapitalCurrent Ac(Profit transferred to capitalcurrent Ac)(b) If LossPartnerrsquos Capital Current Achelliphelliphelliphellip DrTo Profit and Loss Appropriation Ac(Loss transferred to capitalcurrent Ac)

THE FORMAT OF PROFIT AND LOSS APPROPRIATION

Profit and Loss Appropriation Account for the year endedhelliphelliphelliphellip

Particulars Amount Particulars Amount

To PL Ac (loss) By pL Ac (profit)

To Interest on capital BY Interest on drawings

To partner`s commission by Partner`s capital Ac ( loss)

To Partner`s salary To Interest on partner`s loan To General Reserve To Partner`s Capital AC (Profit)

Subject= Economics

MOVEMENT ALONG THE DEMAND CURVE (CHANGE IN QUANTITY DEMANDED)In law of demand you have already studied the inverse relationship between price and quantity demanded When quantity demanded of a commodity changes due to change in its price keeping other factors constant it is called change in quantity demanded It is graphically expressed as a movement along the same demand curve There can be either a downward movement or an upward movement along the same demand curve Upward movement along the same demand curve is called contraction of demand or decrease in quantity demanded and downward movement along the same demand curve is known as expansion of demand or increase in quantity demanded

Extention of demandd

price (rs)p A

B Extentionp1 d

Q Q1

Quantity demanded ( in units)

Contraction of demandd

p2 Ccontraction

p APrice (Rs)

d

Q2 Q

Quantity demanded (in units)

Explanation of movement of demand A fall in price from OP to OP1 leads to increase in quantity demanded from OQ to OQ1 (expansion of demand) resulting in a downward movement from point A to point B along the same demand curve DD When Price rises from OP to OP2 quantity demanded falls from OQ to OQ2 (contraction of demand) leading to an upward movement from point A to point C along the same demand curve DD

  • Activity Series of Metals
    • Drawbacks of Rutherfordrsquos model of atom
      • Electromagnetic radiations
      • Properties of electromagnetic radiations
      • Characteristics of electromagnetic radiations
        • Plancks Quantum Theory-
        • Photoelectric effect
          • Intext Questions
            • Logic gates
            • Digital systems are said to be constructed by using logic gates These gates are the AND OR NOT NAND NOR EXOR and EXNOR gates The basic operations are described below with the aid of truth tables
            • AND gate
            • Example
Page 55:  · Web viewSubject. Topic. Summary. Execution. English 1 . Chapter 1 naming words . Page 8. Write the names of these pictures:- Person:-1. father. 2.Firefighter 3.doctor 4 ...

Methought the billows spoke and told me of it

The winds did sing it to me and the thunderThat deep and dreadful organ-

pipe pronouncedThe name of Prosper It did bass

my trespassTherefore my son ithrsquo ooze is

bedded andIll seek him deeper than eer

plummet soundedAnd with him there lie mudded

(a) In what way does Alonso express his horror when his conscience is awakened by Arielrsquos words

When Alonsorsquos conscience is awakened by Arielrsquos words he expresses his horror at what he has heard He gets the feeling that the waves of the ocean the wind and the loud thunder have spoken to him and uttered the name of Prospero Because of being reminded of his crime in a very loud and rough voice he comes to realize that he has lost his son for his past misdeeds

(b) What does Alonso imagine about his son What does Alonso want to do in his desperate state

Alonso imagines that his son is lying in the mud at the bottom of the sea He feels desperate that he wants to drown himself in the ocean deeper than the plumb-line has ever gone He wants to lie with his son at the bottom of the sea

(c) How do Sebastian and Antonio want to face the evil spirits

Sebastian says that he is not at all afraid of what the harpy has said and that he is prepared to fight any number of such monsters if they appear before him only one at a time Antonio says that he would support Sebastian in the fight against the fiendsyyy

(d) Why does Gonzalo ask Adrian to follow the three men

Gonzalo tells Adrian that all the three men namely Alonso Sebastian and Antonio are in a wild and reckless mood The thought of the heinous crime of which they are guilty has begun to torment their minds So he asks Adrian to follow those three men without loss of time and prevent them from doing anything which the turmoil in their minds might lead them to do

(e) What opinion do you form of Alonso from the above extract

Alonso who is the lesser villain is the first to give way to remorse under the effect of Arielrsquos speech The words of Ariel seem to him to be the voice of conscience speaking to him He is driven to desperation a state in which he might do violence to his life

Subject =Accounts

Ac-12 15420 topic-pL Appropriation ac

PROFIT AND LOSS APPROPRIATION ACCOUNT

MEANING AND PREPARATIONProfit and Loss Appropriation Account is merely an extension of the Profit and Loss Account of the firm The profit of the firm has to be distributed amongst the partners in their respective profit sharing ratio But before its distribution it needs to be adjusted All Adjustments like partnerrsquos salary partnerrsquos commission interest on capital interest on drawings etc are made in this account These adjustments will reduce the amount of profit for distribution This adjusted profit will be distributed amongst the partners in their profit sharing ratio To prepare it at first the balance of Profit and Loss Account is transferred to this account The journal entries for the preparation of Profit and Loss Appropriation Account are given below

1 for transfer of the balance of Profit and Loss Account to Profit and Loss Appropriation Account

(a) In case of Net Profit

Profit and Loss Ac helliphelliphelliphelliphellipDrTo Profit and Loss Appropriation Ac(Net Profit transferred to Profit and Loss Appropriation Ac)

(b)In case of Net Loss

Profit and Loss Appropriation Achelliphelliphellip DrTo Profit and Loss Ac(Net Loss transferred to Profit and Loss Appropriation Ac)

2 for Interest on Capital

For transferring on Interest on CapitalProfit and Loss Appropriation Achelliphelliphellip DrTo Interest on Capital Ac(Interest on capital transferred to Profit amp Loss Appropriation Ac)

3 for Interest on Drawings

For transferring Interest on Drawings Interest on Drawings Achelliphelliphelliphelliphelliphellip DrTo Profit and Loss Appropriation Ac(Interest on drawing transferred to Profit amp Loss Appropriation Ac)

4 For Partnerrsquos SalaryFor transfer of partnerrsquos SalaryProfit and Loss Appropriation Achelliphellip DrTo Salary Ac(Salary transferred to profit amp Loss Appropriation Ac)

5 For Partnerrsquos CommissionFor transferring commissionProfit and Loss Appropriation Achelliphelliphellip DrTo Commission Ac(Commission transferred to Profit and Loss Appropriation Ac)

6 For Transfer of agreed amount to General ReserveProfit and Loss Appropriation Ac helliphellipDrTo General Reserve Ac(Transfer to General Reserve)

7 for share of Profit or Loss appropriation(a) If ProfitProfit and Loss Appropriation Achelliphellip DrTo Partnerrsquos CapitalCurrent Ac(Profit transferred to capitalcurrent Ac)(b) If LossPartnerrsquos Capital Current Achelliphelliphelliphellip DrTo Profit and Loss Appropriation Ac(Loss transferred to capitalcurrent Ac)

THE FORMAT OF PROFIT AND LOSS APPROPRIATION

Profit and Loss Appropriation Account for the year endedhelliphelliphelliphellip

Particulars Amount Particulars Amount

To PL Ac (loss) By pL Ac (profit)

To Interest on capital BY Interest on drawings

To partner`s commission by Partner`s capital Ac ( loss)

To Partner`s salary To Interest on partner`s loan To General Reserve To Partner`s Capital AC (Profit)

Subject= Economics

MOVEMENT ALONG THE DEMAND CURVE (CHANGE IN QUANTITY DEMANDED)In law of demand you have already studied the inverse relationship between price and quantity demanded When quantity demanded of a commodity changes due to change in its price keeping other factors constant it is called change in quantity demanded It is graphically expressed as a movement along the same demand curve There can be either a downward movement or an upward movement along the same demand curve Upward movement along the same demand curve is called contraction of demand or decrease in quantity demanded and downward movement along the same demand curve is known as expansion of demand or increase in quantity demanded

Extention of demandd

price (rs)p A

B Extentionp1 d

Q Q1

Quantity demanded ( in units)

Contraction of demandd

p2 Ccontraction

p APrice (Rs)

d

Q2 Q

Quantity demanded (in units)

Explanation of movement of demand A fall in price from OP to OP1 leads to increase in quantity demanded from OQ to OQ1 (expansion of demand) resulting in a downward movement from point A to point B along the same demand curve DD When Price rises from OP to OP2 quantity demanded falls from OQ to OQ2 (contraction of demand) leading to an upward movement from point A to point C along the same demand curve DD

  • Activity Series of Metals
    • Drawbacks of Rutherfordrsquos model of atom
      • Electromagnetic radiations
      • Properties of electromagnetic radiations
      • Characteristics of electromagnetic radiations
        • Plancks Quantum Theory-
        • Photoelectric effect
          • Intext Questions
            • Logic gates
            • Digital systems are said to be constructed by using logic gates These gates are the AND OR NOT NAND NOR EXOR and EXNOR gates The basic operations are described below with the aid of truth tables
            • AND gate
            • Example
Page 56:  · Web viewSubject. Topic. Summary. Execution. English 1 . Chapter 1 naming words . Page 8. Write the names of these pictures:- Person:-1. father. 2.Firefighter 3.doctor 4 ...

Ac-12 15420 topic-pL Appropriation ac

PROFIT AND LOSS APPROPRIATION ACCOUNT

MEANING AND PREPARATIONProfit and Loss Appropriation Account is merely an extension of the Profit and Loss Account of the firm The profit of the firm has to be distributed amongst the partners in their respective profit sharing ratio But before its distribution it needs to be adjusted All Adjustments like partnerrsquos salary partnerrsquos commission interest on capital interest on drawings etc are made in this account These adjustments will reduce the amount of profit for distribution This adjusted profit will be distributed amongst the partners in their profit sharing ratio To prepare it at first the balance of Profit and Loss Account is transferred to this account The journal entries for the preparation of Profit and Loss Appropriation Account are given below

1 for transfer of the balance of Profit and Loss Account to Profit and Loss Appropriation Account

(a) In case of Net Profit

Profit and Loss Ac helliphelliphelliphelliphellipDrTo Profit and Loss Appropriation Ac(Net Profit transferred to Profit and Loss Appropriation Ac)

(b)In case of Net Loss

Profit and Loss Appropriation Achelliphelliphellip DrTo Profit and Loss Ac(Net Loss transferred to Profit and Loss Appropriation Ac)

2 for Interest on Capital

For transferring on Interest on CapitalProfit and Loss Appropriation Achelliphelliphellip DrTo Interest on Capital Ac(Interest on capital transferred to Profit amp Loss Appropriation Ac)

3 for Interest on Drawings

For transferring Interest on Drawings Interest on Drawings Achelliphelliphelliphelliphelliphellip DrTo Profit and Loss Appropriation Ac(Interest on drawing transferred to Profit amp Loss Appropriation Ac)

4 For Partnerrsquos SalaryFor transfer of partnerrsquos SalaryProfit and Loss Appropriation Achelliphellip DrTo Salary Ac(Salary transferred to profit amp Loss Appropriation Ac)

5 For Partnerrsquos CommissionFor transferring commissionProfit and Loss Appropriation Achelliphelliphellip DrTo Commission Ac(Commission transferred to Profit and Loss Appropriation Ac)

6 For Transfer of agreed amount to General ReserveProfit and Loss Appropriation Ac helliphellipDrTo General Reserve Ac(Transfer to General Reserve)

7 for share of Profit or Loss appropriation(a) If ProfitProfit and Loss Appropriation Achelliphellip DrTo Partnerrsquos CapitalCurrent Ac(Profit transferred to capitalcurrent Ac)(b) If LossPartnerrsquos Capital Current Achelliphelliphelliphellip DrTo Profit and Loss Appropriation Ac(Loss transferred to capitalcurrent Ac)

THE FORMAT OF PROFIT AND LOSS APPROPRIATION

Profit and Loss Appropriation Account for the year endedhelliphelliphelliphellip

Particulars Amount Particulars Amount

To PL Ac (loss) By pL Ac (profit)

To Interest on capital BY Interest on drawings

To partner`s commission by Partner`s capital Ac ( loss)

To Partner`s salary To Interest on partner`s loan To General Reserve To Partner`s Capital AC (Profit)

Subject= Economics

MOVEMENT ALONG THE DEMAND CURVE (CHANGE IN QUANTITY DEMANDED)In law of demand you have already studied the inverse relationship between price and quantity demanded When quantity demanded of a commodity changes due to change in its price keeping other factors constant it is called change in quantity demanded It is graphically expressed as a movement along the same demand curve There can be either a downward movement or an upward movement along the same demand curve Upward movement along the same demand curve is called contraction of demand or decrease in quantity demanded and downward movement along the same demand curve is known as expansion of demand or increase in quantity demanded

Extention of demandd

price (rs)p A

B Extentionp1 d

Q Q1

Quantity demanded ( in units)

Contraction of demandd

p2 Ccontraction

p APrice (Rs)

d

Q2 Q

Quantity demanded (in units)

Explanation of movement of demand A fall in price from OP to OP1 leads to increase in quantity demanded from OQ to OQ1 (expansion of demand) resulting in a downward movement from point A to point B along the same demand curve DD When Price rises from OP to OP2 quantity demanded falls from OQ to OQ2 (contraction of demand) leading to an upward movement from point A to point C along the same demand curve DD

  • Activity Series of Metals
    • Drawbacks of Rutherfordrsquos model of atom
      • Electromagnetic radiations
      • Properties of electromagnetic radiations
      • Characteristics of electromagnetic radiations
        • Plancks Quantum Theory-
        • Photoelectric effect
          • Intext Questions
            • Logic gates
            • Digital systems are said to be constructed by using logic gates These gates are the AND OR NOT NAND NOR EXOR and EXNOR gates The basic operations are described below with the aid of truth tables
            • AND gate
            • Example
Page 57:  · Web viewSubject. Topic. Summary. Execution. English 1 . Chapter 1 naming words . Page 8. Write the names of these pictures:- Person:-1. father. 2.Firefighter 3.doctor 4 ...

6 For Transfer of agreed amount to General ReserveProfit and Loss Appropriation Ac helliphellipDrTo General Reserve Ac(Transfer to General Reserve)

7 for share of Profit or Loss appropriation(a) If ProfitProfit and Loss Appropriation Achelliphellip DrTo Partnerrsquos CapitalCurrent Ac(Profit transferred to capitalcurrent Ac)(b) If LossPartnerrsquos Capital Current Achelliphelliphelliphellip DrTo Profit and Loss Appropriation Ac(Loss transferred to capitalcurrent Ac)

THE FORMAT OF PROFIT AND LOSS APPROPRIATION

Profit and Loss Appropriation Account for the year endedhelliphelliphelliphellip

Particulars Amount Particulars Amount

To PL Ac (loss) By pL Ac (profit)

To Interest on capital BY Interest on drawings

To partner`s commission by Partner`s capital Ac ( loss)

To Partner`s salary To Interest on partner`s loan To General Reserve To Partner`s Capital AC (Profit)

Subject= Economics

MOVEMENT ALONG THE DEMAND CURVE (CHANGE IN QUANTITY DEMANDED)In law of demand you have already studied the inverse relationship between price and quantity demanded When quantity demanded of a commodity changes due to change in its price keeping other factors constant it is called change in quantity demanded It is graphically expressed as a movement along the same demand curve There can be either a downward movement or an upward movement along the same demand curve Upward movement along the same demand curve is called contraction of demand or decrease in quantity demanded and downward movement along the same demand curve is known as expansion of demand or increase in quantity demanded

Extention of demandd

price (rs)p A

B Extentionp1 d

Q Q1

Quantity demanded ( in units)

Contraction of demandd

p2 Ccontraction

p APrice (Rs)

d

Q2 Q

Quantity demanded (in units)

Explanation of movement of demand A fall in price from OP to OP1 leads to increase in quantity demanded from OQ to OQ1 (expansion of demand) resulting in a downward movement from point A to point B along the same demand curve DD When Price rises from OP to OP2 quantity demanded falls from OQ to OQ2 (contraction of demand) leading to an upward movement from point A to point C along the same demand curve DD

  • Activity Series of Metals
    • Drawbacks of Rutherfordrsquos model of atom
      • Electromagnetic radiations
      • Properties of electromagnetic radiations
      • Characteristics of electromagnetic radiations
        • Plancks Quantum Theory-
        • Photoelectric effect
          • Intext Questions
            • Logic gates
            • Digital systems are said to be constructed by using logic gates These gates are the AND OR NOT NAND NOR EXOR and EXNOR gates The basic operations are described below with the aid of truth tables
            • AND gate
            • Example
Page 58:  · Web viewSubject. Topic. Summary. Execution. English 1 . Chapter 1 naming words . Page 8. Write the names of these pictures:- Person:-1. father. 2.Firefighter 3.doctor 4 ...

Extention of demandd

price (rs)p A

B Extentionp1 d

Q Q1

Quantity demanded ( in units)

Contraction of demandd

p2 Ccontraction

p APrice (Rs)

d

Q2 Q

Quantity demanded (in units)

Explanation of movement of demand A fall in price from OP to OP1 leads to increase in quantity demanded from OQ to OQ1 (expansion of demand) resulting in a downward movement from point A to point B along the same demand curve DD When Price rises from OP to OP2 quantity demanded falls from OQ to OQ2 (contraction of demand) leading to an upward movement from point A to point C along the same demand curve DD

  • Activity Series of Metals
    • Drawbacks of Rutherfordrsquos model of atom
      • Electromagnetic radiations
      • Properties of electromagnetic radiations
      • Characteristics of electromagnetic radiations
        • Plancks Quantum Theory-
        • Photoelectric effect
          • Intext Questions
            • Logic gates
            • Digital systems are said to be constructed by using logic gates These gates are the AND OR NOT NAND NOR EXOR and EXNOR gates The basic operations are described below with the aid of truth tables
            • AND gate
            • Example
Page 59:  · Web viewSubject. Topic. Summary. Execution. English 1 . Chapter 1 naming words . Page 8. Write the names of these pictures:- Person:-1. father. 2.Firefighter 3.doctor 4 ...

Quantity demanded (in units)

Explanation of movement of demand A fall in price from OP to OP1 leads to increase in quantity demanded from OQ to OQ1 (expansion of demand) resulting in a downward movement from point A to point B along the same demand curve DD When Price rises from OP to OP2 quantity demanded falls from OQ to OQ2 (contraction of demand) leading to an upward movement from point A to point C along the same demand curve DD

  • Activity Series of Metals
    • Drawbacks of Rutherfordrsquos model of atom
      • Electromagnetic radiations
      • Properties of electromagnetic radiations
      • Characteristics of electromagnetic radiations
        • Plancks Quantum Theory-
        • Photoelectric effect
          • Intext Questions
            • Logic gates
            • Digital systems are said to be constructed by using logic gates These gates are the AND OR NOT NAND NOR EXOR and EXNOR gates The basic operations are described below with the aid of truth tables
            • AND gate
            • Example